MEGA EMRCS Flashcards

1
Q

Which of the following drugs increases the rate of gastric emptying in the vagotomised stomach?

A Ondansetron
B Metoclopramide
C Cyclizine
D Erythromycin
E Chloramphenicol

A

D Erythromycin

Vagotomy seriously compromises gastric emptying which is why either a pyloroplasty or gastro- enterostomy is routinely performed at the same time.
Chloramphenicol has no effect on gastric emptying. Ondansetron slows gastric emptying slightly. Metoclopramide increases the rate of gastric emptying but its effects are mediated via the vagus nerve. Erythromycin enhances gastric emptying by acting via the motilin receptor in the gut.

How well did you know this?
1
Not at all
2
3
4
5
Perfectly
2
Q

A 36-year-old man with end-stage renal disease who is undergoing haemodialysis has normocytic normochromic anaemia. Which of the following is the most appropriate therapy?
A. Erythropoietin
B. Ferrous sulphate
C. Folate
D. Vitamin B6
E. Vitamin B12

A

ANSWER IS A
Erythropoietin, or EPO, is a glycoprotein hormone that is a cytokine for erythrocyte
(red blood cells) precursors in the bone marrow. Also called haematopoietin or haemopoietin, it is produced by the kidney and is the hormone regulating red blood cell production.
Erythropoietin is available as a therapeutic agent produced by recombinant DNA technology in mammalian cell culture. It is used in treating anaemia resulting from chronic renal failure or from cancer chemotherapy. Its use is also believed to be common as a doping agent in endurance sports such as bicycle racing, triathlons and marathon running.

How well did you know this?
1
Not at all
2
3
4
5
Perfectly
3
Q

A blood sample taken from the umbilical artery of a newborn was subjected to electrophoresis to detect antibodies (immunoglobulins). Which of the following antibodies will have the highest percentage in a newborn?
A. IgA
B. IgD
C. IgE
D. IgG
E. IgM

A

ANSWER IS D
IgG is a monomeric immunoglobulin, built of two heavy chains γ and two light chains. Each molecule has two antigen-binding sites.
This is the most abundant immunoglobulin and is approximately equally distributed in blood and in tissue liquids, constituting 75% of serum immunoglobulins in humans. This is the only isotype that can pass through the placenta, thereby providing protection to the newborn in its first weeks of life before its own immune system has developed.
It can bind to many kinds of pathogens, for example viruses, bacteria and fungi, and protects the body against them by complement activation (classic pathway),opsonisation or phagocytosis and neutralization of their toxins. There are four subclasses:
→IgG1 (66%), IgG2 (23%), IgG3 (7%) and IgG4 (4%):
- IgG1, IgG3 and IgG4 cross the placenta easily
- IgG3 is the most effective complement activator, followed by IgG1 and then IgG2
- IgG4 does not activate complement
- IgG1 and IgG3 bind with high affinity to Fc receptors on phagocytic cells
- IgG4 has intermediate affinity and IgG2 affinity is extremely

How well did you know this?
1
Not at all
2
3
4
5
Perfectly
4
Q

A 58-year-old male patient needed a blood transfusion after repair of an abdominal aortic aneurysm. His blood was sent to the laboratory. The technician, while checking for this patient’s blood group, said that the patient’s blood agglutinates with antisera anti-A and anti-D, while the patient’s serum agglutinates cells of blood group B. What is the blood group of this patient?
A. A positive
B. B positive
C. A negative
D. B negative
E. O positive

A

ANSWER IS A
According to the ABO blood typing system there are four different kinds of blood types: A, B, AB or O.
❖Blood groups A - If you belong to the blood group A, you have A antigens on the surface of your red blood cells and B antibodies in your blood plasma.
❖Blood group B - If you belong to the blood group B, you have B antigens on the surface of your red blood cells and A antibodies in your blood plasma.
❖Blood group AB - If you belong to the blood group AB, you have both A and B antigens on the surface of your red blood cells and no A or B antibodies at all in your blood plasma.
❖Blood group O - If you belong to the blood group O, you have neither A nor B antigens on the surface of your red blood cells but you have both A and B antibodies in your blood plasma.
Many people also have a so-called Rh factor on the red blood cell’s surface. This is also an antigen and those who have it are called Rh+. Those who have not are called Rh–. A person with Rh– blood does not have Rh antibodies naturally in the blood plasma (as one can have A or B antibodies, for instance) but they can develop Rh antibodies in the blood plasma if they receive blood from a person with Rh+ blood, whose Rh antigens can trigger the production of Rh antibodies.
A person with Rh+ blood can receive blood from a person with Rh– blood without any problems. So, in this vignette the patient’s blood group is A positive as he has antigen A, antibody B and Rh antigens.

How well did you know this?
1
Not at all
2
3
4
5
Perfectly
5
Q

In humans there are five types of antibody: IgA, IgD, IgE, IgG and IgM. Which of the following statements regarding IgM is CORRECT?
A. It binds to allergens
B. It functions mainly as an antigen receptor on B cells
C. It is the largest immunoglobulin molecule
D. It is the most abundant immunoglobulin
E. It is a tetramer of four

A

ANSWER IS C
IgM forms polymers where multiple immunoglobulins are covalently linked together
with disulphide bonds, normally as a pentamer or occasionally as a hexamer. It has a large molecular mass of approximately 900 kDa (in its pentamer form).
The J chain is attached to most pentamers, while hexamers do not possess the J chain due to space constraints in the complex. Because each monomer has two antigen binding sites, an IgM has 10 of them; however, it cannot bind 10 antigens at the same time because they hinder each other.
Because it is a large molecule, it cannot diffuse well and is found in the interstitium only in very low quantities. IgM is primarily found in serum; however, because of the J chain, it is also important as a secretory immunoglobulin.
Due to its polymeric nature, IgM possesses high avidity and is particularly effective at complement activation. It is sometimes called a ‘natural antibody’, but it is likely that the antibodies arise due to sensitization in the very young to antigens that are naturally occurring in nature. For example anti-A and anti-B IgM antibodies can be formed in early life as a result of exposure to anti-A- and anti-B-like substances that are present on bacteria or perhaps also on plant materials.
In germ-line cells, the gene segment encoding the μ constant region of the heavy chain is positioned first among other constant-region gene segments. For this reason, IgM is the first immunoglobulin expressed by mature B cells.
IgM is also by far the physically largest antibody in the circulation. IgM antibodies are mainly responsible for the clumping (agglutination) of red blood cells if the recipient of a blood transfusion receives blood that is not compatible with his/her blood type. IgM antibodies appear early in the course of an infection and usually do not reappear after further exposure. IgM antibodies do not pass across the human placenta. These two biological properties of IgM make it useful in the diagnosis of infectious diseases. Demonstrating IgM antibodies in a patient’s serum indicates recent infection or, in serum from a neonate, indicates intrauterine infection such as congenital rubella.

How well did you know this?
1
Not at all
2
3
4
5
Perfectly
6
Q

The lack of normal factor VIII causes haemophilia A, an inherited bleeding disorder. Factor VIII is synthesized predominantly in:
A. Hepatocytes
B. Histiocytes
C. Kupffer cells
D. Platelets
E. Vascular endothelium

A

ANSWER IS E
Factor VIII (FVIII) is an essential clotting factor. The lack of normal FVIII causes haemophilia A, an inherited bleeding disorder. The gene for Factor VIII is located on the X chromosome (Xq28). FVIII is a glycoprotein pro-cofactor. Factor VIII is synthesized predominantly in the vascular endothelium and is not affected by liver disease. In fact, levels usually are elevated in such instances. It is also synthesized and released into the bloodstream by the liver.
In the circulating blood, it is mainly bound to von Willebrand factor (vWF, also known as factor VIII-related antigen) to form a stable complex. Upon activation by thrombin or factor Xa, it dissociates from the complex to interact with factor IXa in the coagulation cascade. It is a co-factor to factor IXa in the activation of factor X, which, in turn, with its co-factor factor Va, activates more thrombin. Thrombin cleaves fibrinogen into fibrin, which polymerises and cross links (using factor XIII) into a blood clot. No longer protected by vWF, activated FVIII is proteolytically inactivated in the process (most prominently by activated protein C and factor IXa) and quickly cleared from the bloodstream.
FVIII concentrated from donated blood plasma or alternatively recombinant FVIII can be given to haemophiliacs to restore haemostasis. So, FVIII is also known as antihaemophilic factor. The transfer of a plasma by-product into the bloodstream of a patient with haemophilia often led to the transmission of diseases such as HIV and hepatitis before purification methods were improved. In the early 1990s, pharmaceutical companies began to produce recombinant synthesized factor products, which now prevent nearly all forms of disease transmission during replacement

How well did you know this?
1
Not at all
2
3
4
5
Perfectly
7
Q

A 45-year-old woman, with a past history of easy bruising and heavy menstrual periods, was admitted for elective cholecystectomy and was diagnosed with von Willebrand’s disease on routine preoperative investigations. von Willebrand’s disease is:
A. Autosomal dominant
B. Characterized by decreased bleeding time
C. Characterized by decreased factor VII
D. Characterized by decreased platelets
E. X-linked

A

ANSWER IS A
Von Willebrand’s disease (vWD) is the most common hereditary coagulation abnormality described in humans, although it can also be acquired as a result of other medical conditions. It arises from a qualitative or quantitative deficiency of von Willebrand factor (vWF), a multimeric protein that is required for platelet adhesion. It is known to affect humans and, in veterinary medicine, dogs.
There are three types of hereditary vWD, but other factors such as ABO blood group may also play a part in the cause of the condition. The various types of vWD present with varying degrees of bleeding tendency. Severe internal or joint bleeding is rare (only in type 3 vWD); bruising, nosebleeds, heavy menstrual periods (in women) and blood loss during childbirth (rare) may occur. Death may occur The vWF gene is located on chromosome 12 (12p13.2). It has 52 exons spanning 178 kbp. Types 1 and 2 are inherited as autosomal dominant traits and type 3 is inherited as autosomal recessive.
Occasionally type 2 also inherits recessively. In humans, the incidence of vWD is roughly about 1 in 100 individuals. Because most forms are rather mild, they are detected more often in women, whose bleeding tendency shows during menstruation. The actual abnormality (which does not necessarily lead to disease) occurs in 0.9–3% of the population. It may be more severe or apparent in people with blood group O. Acquired vWD can occur in patients with autoantibodies. In this case the function of vWF is not inhibited but the vWF– antibody complex is rapidly cleared from the circulation. A form of vWD occurs in patients with aortic valve stenosis, leading to gastrointestinal bleeding (Heyde’s syndrome). This form of acquired vWD may be more prevalent than is presently thought.
Acquired vWF has also been described in the following disorders: Wilms’ tumour, hypothyroidism and mesenchymal dysplasias. Patients with vWD normally require no regular treatment, although they are always at increased risk for bleeding. Prophylactic treatment is sometimes given for patients with vWD who are scheduled for surgery. They can be treated with human-derived medium purity factor VIII concentrates. Mild cases of vWD can be trialed on desmopressin (1-desamino-8-D- arginine vasopressin, DDAVP) (antihaemophilic factor, more commonly known as humate-P), which works by raising the patient’s own plasma levels of vWF by inducing release of vWF stored in the Weibel–Palade bodies in the endothelial cells

How well did you know this?
1
Not at all
2
3
4
5
Perfectly
8
Q

A 68-year-old woman complaining of easy fatigability and shortness of breath after abdominal aortic aneurysm repair was diagnosed with iron-deficiency anaemia and prescribed an oral iron preparation. Which of the following statements about iron metabolism is CORRECT?
A. Ferritin is a plasma protein that transports iron in the blood
B. Haemosiderin is a product of haemoglobin degradation
C. Iron is more efficiently absorbed in the ferrous state (Fe2+) than in the ferric
state (Fe3+)
D. Most iron in the body is stored as haemosiderin
E. The gastrointestinal rate of iron absorption is extremely high

A

ANSWER IS C
The absorption of non-haem iron in any food is strongly affected by the composition of the meals. Iron is more efficiently absorbed in the ferrous state (Fe2+) than in the ferric state (Fe3+) and commercial iron preparations often contain vitamin C to prevent oxidation of Fe2+ to Fe3+. Still, only 3–6% of the ingested daily iron is actually absorbed in the upper gastrointestinal tract.
Seventy per cent of the total body iron is used for haemoglobin and myoglobin; the remainder is stored as readily exchangeable ferritin and some is stored in less easily mobilized haemosiderin. When old red blood cells are destroyed by the tissue macrophage system, haem is separated from globin and degraded to biliverdin. Iron in the plasma is bound to the iron transporting protein transferrin. Transferrin level (total iron-binding capacity) and saturation are clinically important indicators of iron deficiency anaemia.

How well did you know this?
1
Not at all
2
3
4
5
Perfectly
9
Q

A 45-year-old man on warfarin for a mechanical mitral valve was admitted in the Accident and Emergency Department with persistent bleeding following dental extraction. He was told that his coagulation was deranged. Which of the following statements about blood coagulation is CORRECT?
A. Absence of Ca2+ promotes blood coagulation
B. Disseminated intravascular coagulation (DIC) results in depletion of fibrin
split products
C. Patients with haemophilia A usually have a normal bleeding time
D. von Willebrand factor suppresses platelet adhesion
E. von Willebrand factor suppresses blood coagulation

A

ANSWER IS C
Prolonged bleeding time is characteristic of platelet disorders, e.g., thrombocytopaenia. Patients with haemophilia A or B (i.e. absence of factor VIII or IX, respectively) have a prolonged partial thromboplastin time (PTT), but do not have a prolonged bleeding time. Ca2+ is a necessary co-factor for blood coagulation, and chelation of Ca2+ ions by citrate inhibits coagulation.
Von Willebrand factor is part of the factor VIII complex and also promotes platelet adherence to the vascular subendothelium. Patients who lack this factor (von Willebrand’s disease) have both a prolonged PTT and a prolonged bleeding time. Disseminated intravascular coagulation results in depletion of coagulation factors and accumulation of fibrin split products.

How well did you know this?
1
Not at all
2
3
4
5
Perfectly
10
Q

Nerve gas (organophosphate) is a weapon of chemical warfare that kills by causing respiratory and cardiovascular failure. The expected effect of organophosphate poisoning on the heart would be:
A. Decrease the force of myocardial contractions by potentiating the vagal tone to the ventricular muscle
B. Decrease the rate of rhythmicity of the sinoatrial (SA) node by inducing hyperpolarisation
C. Depolarize cells of the SA node by closing potassium channels under the control of the muscarinic acetylcholine receptor
D. Increase the rate of rhythmicity of the SA node by increasing the upward drift in membrane potential caused by sodium leakage
E. Increase conductivity at the atrioventricular (AV) junction by inducing depolarization

A

ANNSWER IS B
The toxic effects of nerve gas derive from its ability to inhibit the enzyme cholinesterase. The inhibition of this naturally occurring degradative enzyme engenders a massive accumulation of acetylcholine evoking an overstimulation of the acetylcholine receptors throughout the body. In the heart, specifically, acetylcholine released by the vagal nerve stimulates muscarinic receptors in the cells of the sinoatrial (SA) node.
This results in the opening of potassium channels and hyperpolarisation of the SA node. It therefore takes longer for sodium leakage to cause the membrane potentials of these cells to reach the threshold required for an action potential. The rate of rhythmicity is so decreased. A similar hyperpolarisation of the fibers at the atrioventricular (AV) junction decreases conduction velocity of atria impulses to the ventricle. The force of ventricular contractions is not affected by the vagus nerve.

How well did you know this?
1
Not at all
2
3
4
5
Perfectly
11
Q

The resting membrane potential of a neuronal cell body is –60 mV. Opening chloride channels in the neuronal membrane will most likely cause:
A. Depolarization to about –30 mV
B. Depolarization to about +30 mV
C. Hyperpolarisation to about –70 mV
D. Initiation of an action potential
E. No change in membrane potential

A

ANSWER IS C
Increasing the membrane’s conductance to chloride will result in chloride influx and the membrane potential approaching the value dictated by the chloride equilibrium potential (calculated from the Nernst equation), which is about –70 mV for neurons.
A value of –30 mV is near the Nernst potential for Cl− ions in smooth muscle cells, but not in neurons; +30 mV is near the Nernst potential for Na+ ions. The membrane potential would remain unchanged only if the cell resting membrane potential is already at the Nearest potential of the ion channels that were opened. Action potentials occur if the cell membrane is depolarised above threshold.

How well did you know this?
1
Not at all
2
3
4
5
Perfectly
12
Q

Chloride ions are associated with changes in neuronal membrane potential. Which of the following statements most accurately describes the response of a neuron to a decrease in the conductance of the cell membrane to chloride ions?
A. The cell will depolarize if its membrane potential is positive with respect to the equilibrium potential for chloride ions
B. The cell will hyperpolarize if its membrane potential is positive with respect to the equilibrium potential for chloride ions
C. The cell will hyperpolarize if the external chloride concentration is greater than the internal chloride concentration
D. The cell will hyperpolarize if the external chloride concentration is less than the internal chloride concentration
E. No change in membrane potential will occur if the external and internal chloride ion concentrations are equal

A

ANSWER IS A
Although electrogenic pumps may contribute to the membrane potential of certain cells, the major determinants of membrane potential are the external and internal concentrations of permanent ions and their relative permeabilities in the membrane. Decreasing the conductance causes the membrane potential to move away from the equilibrium potential for that ion. So, a decrease in the conductance of a membrane to chloride ions causes the cells to depolarize – that is, become more positive – if the membrane potential is positive with respect to the chloride equilibrium potential.
Conversely, increasing the conductance for an ion causes the membrane potential to approach the equilibrium potential for that ion. External and internal ion chloride concentrations are needed to calculate the Nernst potential for this ion, but a simple comparison of these two values does not allow predictions about the change in membrane potential.

How well did you know this?
1
Not at all
2
3
4
5
Perfectly
13
Q

Which of the following is not secreted by the parietal cells?
A-Hydrochloric acid
B-Mucus
C-Magnesium
D-Intrinsic factor
E-Calcium

A

Answer is B. Parietal cells: secrete HCl, Ca, Na, Mg and intrinsic factor Chief cells: secrete pepsinogen
Surface mucosal cells: secrete mucus and bicarbonate.
Gastric secretions
A working knowledge of gastric secretions is important for surgery because peptic ulcers are common, surgeons frequently prescribe anti secretory drugs and because there are still patients around who will have undergone acid lowering procedures (Vagotomy) in the past.
Gastric acid
 Is produced by the parietal cells in the stomach
 pH of gastric acid is around 2 with acidity being maintained by the H /K ATP ase pump. As part of the process bicarbonate ions will be secreted into the surrounding vessels.
 Sodium and chloride ions are actively secreted from the parietal cell into the canaliculus. This sets up a negative potential across the membrane and as a result sodium and potassium ions diffuse across into the canaliculus.
 Carbonic anhydrase forms carbonic acid which dissociates and the hydrogen ions formed by dissociation leave the cell via the H /K antiporter pump. At the same time sodium ions are actively absorbed. This leaves hydrogen and chloride ions in the canaliculus these mix and are secreted into the lumen of the oxyntic gland.

How well did you know this?
1
Not at all
2
3
4
5
Perfectly
14
Q

A 65 year old man is admitted for a below knee amputation. He is taking digoxin. Clinically the patient has an irregularly irregular pulse. What would you expect to see when you examine the jugular venous pressure?
A-Absent y waves
B-Slow y descent
C-Cannon waves
D-Steep y descent
E-Absent a waves

A

Jugular venous pressure
Absent a waves = Atrial fibrillation
Large a waves = Any cause of right ventricular hypertrophy, tricuspid stenosis Cannon waves (extra large a waves) = Complete heart block
Prominent v waves = Tricuspid regurgitation
Slow y descent = Tricuspid stenosis, right atrial myxoma
Steep y descent = Right ventricular failure, constrictive pericarditis, tricuspid regurgitation

How well did you know this?
1
Not at all
2
3
4
5
Perfectly
15
Q

Which part of the ECG represents atrial depolarization?
A-P wave
B-Q wave
C-T wave
D-QRS complex
E-P-R interval

A

Theme from April 2013 exam
Theme from April 2014 exam
The P wave represents atrial depolarization. Note that atrial repolarization is obscured within the QRS complex.

P wave
 Represents the wave of depolarization that spreads from the SA node throughout the atria
 Lasts 0.08 to 0.1 seconds (80-100 ms)
 The isoelectric period after the P wave represents the time in which the impulse is traveling within the AV node
P-R interval
 Time from the onset of the P wave to the beginning of the QRS complex
 Ranges from 0.12 to 0.20 seconds in duration
 Represents the time between the onset of atrial depolarization and the onset of ventricular depolarization
QRS complex
 Represents ventricular depolarization
 Duration of the QRS complex is normally 0.06 to 0.1 seconds

ST segment
 Isoelectric period following the QRS
 Represents period which the entire ventricle is depolarized and roughly corresponds to the plateau phase of the ventricular action potential
T wave
 Represents ventricular repolarization and is longer in duration than depolarization
 A small positive U wave may follow the T wave which represents the last remnants of ventricular repolarization.
Q-T interval
 Represents the time for both ventricular depolarization and repolarization to occur, and therefore roughly estimates the duration of an average ventricular action potential.
 Interval ranges from 0.2 to 0.4 seconds depending upon heart rate.
 At high heart rates, ventricular action potentials shorten in duration, which decreases the Q-T interval. Therefore the Q-T interval is expressed as a “corrected Q-T (QTc)” by taking the Q- T interval and dividing it by the square root of the R-R interval (interval between ventricular depolarizations). This allows an assessment of the Q-T interval that is independent of heart rate.
 Normal corrected Q-Tc interval is less than 0.44 seconds.

How well did you know this?
1
Not at all
2
3
4
5
Perfectly
16
Q

A 53 year old man undergoes a reversal of a loop colostomy. He recovers well and is discharged home. He is readmitted 10 days later with symptoms of vomiting and colicky abdominal pain. On examination he has a swelling of the loop colostomy site and it is tender. What is the most likely underlying diagnosis?
A-Haematoma
B-Intra abdominal adhesions
C-Anastomotic leak
D-Anastomotic stricture
E-Obstructed incisional hernia

A

In this scenario the most likely diagnosis would be obstructed incisional hernia. The tender swelling coupled with symptoms of obstruction point to this diagnosis. Prompt surgical exploration is warranted. Loop colostomy reversals are at high risk of this complication as the operative site is at increased risk of the development of post operative wound infections.
Acute incisional hernia
Any surgical procedure involving entry into a cavity containing viscera may be complicated by post operative hernia
The abdomen is the commonest site
The deep layer of the wound has usually broken down, allowing internal viscera to protrude through
Management is dictated by the patients clinical status and the timing of the hernia in relation to recent surgery
Bowel obstruction or tenderness at the hernia site both mandate early surgical intervention to reduce the risk of bowel necrosis Mature incisional hernias with a wide neck, and no symptoms, may be either left or listed for elective repair
Risk factors for the development of post operative incisional hernias include; post operative wound infections, long term steroid use, obesity and chronic cough

How well did you know this?
1
Not at all
2
3
4
5
Perfectly
17
Q

Theme: Abdominal stomas
A. End ileostomy
B. End colostomy
C. Loop ileostomy
D. Loop colostomy E. End jejunostomy F. Loop jejunostomy G. Caecostomy
For each of the following scenarios, please select the most appropriate type of stoma to be constructed. Each option may be selected once, more than once or not at all.
A 56 year old man is undergoing a low anterior resection for carcinoma of the rectum. A primary anastomosis is planned.

A

Loop ileostomy
Theme from April 2014 Exam
Colonic resections with an anastomosis below the peritoneal reflection may have an anastomotic leak rate (both clinical and radiological) of up to 15%. Therefore most surgeons will defunction such an anastomosis to reduce the clinical severity of an anastomotic leak. A loop ileostomy will achieve this end point and is relatively easy to reverse.

How well did you know this?
1
Not at all
2
3
4
5
Perfectly
18
Q

Theme: Abdominal stomas
A. End ileostomy
B. End colostomy
C. Loop ileostomy
D. Loop colostomy E. End jejunostomy F. Loop jejunostomy G. Caecostomy
For each of the following scenarios, please select the most appropriate type of stoma to be constructed. Each option may be selected once, more than once or not at all.

A 23 year old man with uncontrolled ulcerative colitis is undergoing an emergency sub total colectomy.

A

The correct answer is End ileostomy
Following a sub total colectomy the immediate surgical options include an end ileostomy or ileorectal anastomosis. In the emergency setting an ileorectal anastomosis would be unsafe.

How well did you know this?
1
Not at all
2
3
4
5
Perfectly
19
Q

Theme: Abdominal stomas
A. End ileostomy
B. End colostomy
C. Loop ileostomy
D. Loop colostomy E. End jejunostomy F. Loop jejunostomy G. Caecostomy
For each of the following scenarios, please select the most appropriate type of stoma to be constructed. Each option may be selected once, more than once or not at all.

A 63 year old women presents with large bowel obstruction. On examination she has a carcinoma 10cm from the anal verge.

A

The correct answer is Loop colostomy
Large bowel obstruction resulting from carcinoma should be resected, stented or defunctioned. The first two options typically apply to tumours above the peritoneal reflection. Lower tumours should be defunctioned with a loop colostomy and then formal staging undertaken prior to definitive surgery. An emergency attempted rectal resection carries a high risk of involvement of the circumferential resection margin and is not recommended.

Abdominal stomas
Stomas may be sited during a range of abdominal procedures and involve bringing the lumen or visceral contents onto the skin. In most cases this applies to the bowel. However, other organs or their contents may be diverted in case of need.
With bowel stomas the type method of construction and to a lesser extent the site will be determined by the contents of the bowel. In practice, small bowel stomas should be spouted so that their irritant contents are not in contact with the skin. Colonic stomas do not need to be spouted as their contents are less irritant.

In the ideal situation the site of the stoma should be marked with the patient prior to surgery. Stoma siting is important as it will ultimately influence the ability of the patient to manage their stoma and also reduce the risk of leakage. Leakage of stoma contents and subsequent maceration of the surrounding skin can rapidly progress into a spiraling loss of control of stoma contents.
Types of stomas
Name of stoma Use Common sites
Gastrostomy Gastric decompression or fixation Epigastrium Feeding
Loop jejunostomy
Seldom used as very high output
May be used following emergency laparotomy with planned early closure
Any location according to need
Percutaneous Usually performed for feeding purposes and site in the Usually left upper quadrant jejunostomy proximal bowel
Loop ileostomy
Defunctioning of colon e.g. following rectal cancer surgery
Does not decompress colon (if ileocaecal valve competent)
Usually right iliac fossa
End ilestomy
Usually following complete excision of colon or where ileo- colic anastomosis is not planned
May be used to defunction colon, but reversal is more difficult
Usually right iliac fossa

How well did you know this?
1
Not at all
2
3
4
5
Perfectly
20
Q

A 48 year old woman presents to the A&E department with a sudden onset, severe headache peaking within seconds. On examination, she appears anxious but has no focal neurological deficits. A non-contrast CT scan of the head is performed urgently and is found to be normal. There is no family history of aneurysms or other vascular abnormalities. What is the most appropriate next step in the management of this patient?

A. Prescribe subcutaneous sumatriptan
B. Perform lumbar puncture
C. Initiate antiviral therapy
D. Arrange outpatient neurology follow-up without further investigation
E. Request a magnetic resonance imaging (MRI) of the brain

A

B: In a patient with a sudden onset, severe headache, and normal CT scan, SAH must still be ruled out, as the CT scan may not detect biood if performed very early in the course of SAH. Lumbar puncture is the next step to look for xanthochromia and red blood cells in the cerebrospinal fluid. This will confirm or exclude the diagnosis of SAH Even if you were thinking migraine, you still need to do an LP first because a CT scan was arranged (which means that the clinicians originally thought it was a SAH) So commit to your suspected diagnosis.

How well did you know this?
1
Not at all
2
3
4
5
Perfectly
21
Q

A 32 year old woman attends her General Practice due to frequent migraines. She has 1-2 migraines a month which prevent her from going to work. She has a background of asthma controlled with salbutamol and inhaled steroids. She is in a long term relationship and uses condoms for contraception. What is the SINGLE most appropriate medication to offer for migraine prophylaxis?

A. Propranolol
B. Amitriptyline
C. Topiramate
D. Aspirin
E. Sumatriptan

A

Given the patient’s background and the need for migraine prophylaxis in a woman of childbearing age with a history of asthma, the most appropriate medication to offer would be Amitriptyline.

Explanation:

•	Amitriptyline: This tricyclic antidepressant is effective for migraine prophylaxis and is generally safe for women of childbearing age. It is not contraindicated in asthma and does not have the teratogenic risks associated with Topiramate.
•	Propranolol: Should be avoided due to the patient’s asthma.
•	Topiramate: Should be avoided in women of childbearing age due to potential teratogenic effects.
•	Aspirin: Used for acute treatment, not for prophylaxis.
•	Sumatriptan: Used for acute treatment of migraines, not for prophylaxis.

Correct Answer:

B. Amitriptyline

How well did you know this?
1
Not at all
2
3
4
5
Perfectly
22
Q

A 30 year old man presents to the clinic with a 6 day history of nasal congestion, headache, a feeling of pressure behind the eyes, and a fever. The symptoms have been progressively worsening, and the patient reports difficulty sleeping due to the congestion. There is no history of allergies or any other significant past medical history. On examination, there is tenderness over the frontal and maxillary sinuses. What is the most appropriate initial management for this patient?

A. Oral paracetamol
B. Oral amoxicilin
C. Nasal pseudoephedrine
D. Oral cetirizine
E. Nasal fluticasone

A

A: The patient presents with symptoms suggestive of acute sinusitis and has a fever, which suggests an inflammatory process. According to NICE CKS guidelines, the management of acute sinusitis is primarily focused on symptomatic relief Oral paracetamol can be used to relieve pain, reduce inflammation, and lower the fever. Nasal pseudoephedrine is a decongestant, but there is insufficient evidence to recommend its use, and the same applies to oral cetirizine, an antihistamine Nasal corticosteroids can reduce inlammation, but are not recommended as firstline treatment by NICE CKS Nasal steroids should be considered if the patient has been having symptoms for 10 days or more.

How well did you know this?
1
Not at all
2
3
4
5
Perfectly
23
Q

A 77 year old man presents to A&E with chest pain, An ECG demonstrates ST-depression and T-wave inversion in the lateral leads. Serum troponin is 88 mmoll (+5). He is offered Aspirin and Fondaparinux. A decision is made for an urgent angiogram with coronary intervention, where the left circumflex artery requires insertion of a drug-eluting stent. His background includes atrial fibrillation for which takes apixaban. He is told he will require lifelong aspirin treatment together with 12 months of another antiplatelet agent. What is the SINGLE most appropriate antithrombotic medication?

A. Clopidogrel
B. Prasugrel
C. Ticagrelor
D. Bisoprolol
E. Ramipril

A

A: Clopidogrel should be used as a second antiplatelet medication in the context of anticoagulated patients (warfanin/DOAC). Usually patients will be started on aspinin and ticagrelor post-MI.

How well did you know this?
1
Not at all
2
3
4
5
Perfectly
24
Q

A 35 year old man presents to the Emergency Department with a history of ingesting a large number of aspirin tablets about 4 hours ago. He appears restless, and on examination, his respiratory rate is 30 breaths per minute. His arterial blood gas (ABG) shows: pH 7.50 (7.35-7.45) PaO2: 12.7 kPa (10-14) PaCO2: 4 kPa (4.7-6.0) Bicarbonate 22 mmol/L (22-26) The salicylate level is found to be elevated. What is the most likely acid-base disturbance?

A. Metabolic Alkalosis
B. Metabolic Acidosis
C. Respiratory Acidosis
D. Respiratory Alkalosis
E. Mixed metabolic acidosis and respiratory alkalosis

A

D: The patient has salicylate poisoning, as indicated by the history of aspirin ingestion and the elevated salicylate level Salicylate poisoning initially causes a respiratory alkalosis due to direct stimulation of the respiratory centre in the medulla, leading to hyperventilation. This is reflected in the ABG with a high pH and low PaCO2 Though salicylate poisoning can also lead to a metabolic acidosis, in this case, the bicarbonate level is 22 mmolit, which does not support a concurrent metabolic acidosis Therefore, the primary acid-base disturbance is respiratory alkalosis

How well did you know this?
1
Not at all
2
3
4
5
Perfectly
25
Q

A 56 year old man with chronic obstructive pulmonary disease (COPD) is brought into A&E following sudden onset shortness of breath. A chest x-ray confirms a right sided pneumothorax. The chest wall-hilar measurement is 2.4cm. What is the SINGLE most appropriate management?

A. Needle aspiration
B. Cannula decompression
C. Chest drain insertion
D. Supplemental oxygen
E. Pleurodesis

A

C: Large secondary pneumothorax requires chest drain

How well did you know this?
1
Not at all
2
3
4
5
Perfectly
26
Q

A 35 year old man presents to the clinic with concerns about his weight. He mentions he has a sedentary job, working at a desk for the majority of the day. He admits to unhealthy eating habits, often consuming fast foods and sugary drinks. On examination, his BMI is calculated to be 28 kg/m2. Which is the most appropriate initial management step for this patient?

A. Commence anti-obesity medication
B. Refer to a bariatric surgeon
C. Refer to a dietician
D. Order a full blood count
E. Provide lifestyle advice

A

E- Initial management for someone who is overweight (BMI 25-29.9) is to provide detailed
advice on diet, physical activity, and behavioural changes. Referral to a dietician or
consideration of ather interventions might be appropriate if initial advice does not result in
weight loss or if there are other health issues involved.

How well did you know this?
1
Not at all
2
3
4
5
Perfectly
27
Q

A 43 year old man presents to his General Practitioner with episodes of haematuria. He describes the bleeding with associated pain in his left side, but denies any frequency or urgency. He is concerned because his father had kidney problems and died at the age of 55. His blood pressure is 156/94 and urine dip demonstrates blood and protein. Which of the following represents the inheritance pattern of this condition?

A. X-linked dominant
B. Autosomal recessive
C. X-linked recessive
D. Autosomal dominant
E. No genetic link

A

E: Adult polycystic kidney disease is an autosomal dominant condition.

How well did you know this?
1
Not at all
2
3
4
5
Perfectly
28
Q

A 26 year old woman presents to her GP with a 6-month history of effortless regurgitation of recently ingested food within minutes of eating. The regurgitation is not preceded by retching. She has maintained her weight and does not report engaging in any compensatory behaviors such as vomiting, over-exercising, or periods of restrictive eating. The patient denies feelings of guilt after eating or a fear of weight gain. Physical examination is unremarkable, and she has no electrolyte abnormalities. Her symptoms are causing her distress and impairment in social functioning. What is the most likely diagnosis?

A. Anorexia nervosa
B. Bulimia nervosa
C. Atypical eating disorder
D. Rumination syndrome
E. Binge eating disorder

A

D: This syndrome is characterized by the repeated, effortless regurgitation of recently
ingested food. it is not preceded by retching, which differentiates it from conditions like
bulimia nervosa where vomiting is induced, The food can be re-chewed, re swallowed, or
spit out. Compensatory behaviours are only seen in anorexia and bulimia.

How well did you know this?
1
Not at all
2
3
4
5
Perfectly
29
Q

A 24 year old homosexual man presents to the sexual health clinic with penile discharge. He describes the discharge as green and sticky. He also complains of pain in his penis and groin. On examination there is reddening of the penile opening. Microscopy of the penile swab demonstrates the presence of a Gram-negative diplococci. What is the SINGLE most likely causative organism?

A. Trichomoniasis vaginalis
B. Chlamydia trachomatis
C. Bacterial vaginosis
D. Neisseria gonorrhoeae
E. Mycoplasma genitalium

A

The description of the patient’s symptoms and the findings from the microscopy point towards a specific causative organism.

Key Points:

1.	Penile Discharge: Described as green and sticky.
2.	Pain: In the penis and groin.
3.	Reddening of the Penile Opening: Indicative of inflammation or infection.
4.	Microscopy Findings: Presence of Gram-negative diplococci.

Interpretation:

•	Gram-negative diplococci: This is a characteristic finding for Neisseria gonorrhoeae.

Diagnosis:

•	Neisseria gonorrhoeae is the most likely causative organism given the presentation and microscopy findings.

Correct Answer:

D. Neisseria gonorrhoeae

Explanation:

•	Trichomoniasis vaginalis: Typically causes frothy, yellow-green discharge, but is a protozoan and not a Gram-negative diplococci.
•	Chlamydia trachomatis: Often causes clear or white discharge and is an intracellular bacterium, not seen as Gram-negative diplococci.
•	Bacterial vaginosis: Primarily affects women and is characterized by a fishy odor and clue cells, not Gram-negative diplococci.
•	Mycoplasma genitalium: Causes non-gonococcal urethritis, typically clear discharge, and does not appear as Gram-negative diplococci on microscopy.

Therefore, Neisseria gonorrhoeae is the most consistent with the clinical and microscopic findings in this case.

How well did you know this?
1
Not at all
2
3
4
5
Perfectly
30
Q

60-year-old woman was found to have one focal area of microcalcification
(approximately 20 mm in diameter) in the left breast. A stereotactic core biopsy
of this area was taken for histological assessment, which revealed low-grade ductal carcinoma in situ. In light of this, what would be the most appropriate treatment modality for this patient?

A Wide local excision + postoperative radiotherapy
B Mastectomy + axillary clearance + postoperative radiotherapy
C Wide local excision + axillary clearance + postoperative radiotherapy
D Mastectomy + postoperative radiotherapy
E Mastectomy

A

Wide local excision + postoperative radiotherapy
Although mastectomy and mastectomy + postoperative radiotherapy are accepted treatment options for DCIS, wide local excision + postoperative radiotherapy is now frequently offered to women with tumours less than 40 mm in diameter of not an aggressive grade. Wide local excision attempts to conserve the unaffected breast tissue, thus avoiding the need for complete mastectomy, which carries greater psychological trauma to the patient. Mastectomy + postoperative radiotherapy is offered when DCIS is found in multiple areas of the breast, the tumour grade is aggressive, the size of the tumour is greater than 40 mm and if the ostrogen receptor status is negative. Ductal carcinoma in situ implies that the tumour is still confined to the basement membrane and has not broken through the latter. Thus axillary lymph nodes are at a greater chance of being affected once the tumour has broken through the basement membrane. With some exceptions, axillary staging and clearance is performed when multiple foci of extensive and aggressive DCIS are found.

How well did you know this?
1
Not at all
2
3
4
5
Perfectly
31
Q

A 47-year-old woman is diagnosed with an unofficial 2.5 cm Grade 3 invasive
ductal carcinoma of the right breast. Following MDT discussion the consultant
sees the patient in clinic to convey management options. Which of the following
would be the most appropriate management plan for this patient?
A Mastectomy and sentinel node biopsy
B Wide local excision
C Mastectomy and axillary clearance
D Wide local excision and sentinel node biopsy
E Wide local excision and axillary clearance

A

Wide local excision and sentinel node biopsy
The patient has a unifocal 2.5 cm malignant tumour in the right breast. The most appropriate management would be to perform a wide local excision of the breast tumour and it is essential to establish whether axillary disease is present or not. To assess disease positivity/negativity in the axilla, sentinel node biopsy should be performed. If the axillary sentinel lymph node is involved with metastatic tumour deposits originating from the primary breast tumour (e.g. macrometastases; tumour deposits. 2 mm), depending on the level of involvement of the axilla, the patient will undergo axillary clearance. This involves removing level 1-3 axillary lymph nodes to remove residual malignancy.
It is not common practice for a patient diagnosed with breast cancer to undergo axillary clearance if the axilla has not been assessed. In some cases, an axillary lymph node FNA cytology assessment may be performed if there is palpable axillary disease. If the result confirms malignant disease, then the patient will undergo axillary clearance. Otherwise, sentinel node biopsy is usually performed to plan, whether the patient requires axillary clearance.
The main indications for mastectomy are patients who have large tumours (>5 cm) that are unresponsive to chemotherapy, multifocal breast tumours, diffuse DCIS, and a large tumour relative to the patient’s breast tissue

How well did you know this?
1
Not at all
2
3
4
5
Perfectly
32
Q

65-year-old nulliparous woman presents to your clinic with a lump in her left breast, which was discovered 7 months ago. On examination you find a hard, ill-defined, non-tender, 3.5 cm lump behind the left nipple. The patient has also had bloody, non-purulent discharge from a single duct on the left nipple for over 3 months. The most likely diagnosis here is
A Periductal mastitis
B Breast carcinoma
C Mammary duct ectasia
D Duct papilloma
E Acute mastitis

A

Breast carcinoma
The most likely diagnosis here is breast carcinoma. The patient is in her sixth decade with a nulliparous history.
Furthermore, blood-stained nipple discharge coupled with the non-tender breast lump place her at high risk of breast cancer. Milky to dirty-green nipple discharge is usually seen with mammary duct ectasia (common in postmenopausal women) and often, but not always, occurs bilaterally. Acute and periductal mastitis are unlikely here due to the absence of mastalgia. Although duct papillomas are relatively common causes of bloodstained discharge, they do not classically present with a breast lump and other symptoms are rarely present.
Owing to the similar presentation of breast carcinoma, it is difficult to distinguish it from a duct papilloma by clinical examination alone.

How well did you know this?
1
Not at all
2
3
4
5
Perfectly
33
Q

A 58-year-old postmenopausal woman has been seen in clinic following discovery of a 3 cm, non-tender, irregular, firm lump in the upper outer quadrant of the left breast. Mammography and ultrasound imaging respectively reveal that the lump has areas of calcification and is a solid mass. The most appropriate course of action is

A Repeat mammography and ultrasound scans in 6 months
B Core biopsy
C Fine needle aspiration to ensure that the lump is not really fluid filled
D Repeat mammography and ultrasound scans in 3 months
E Reassure and discharge

A

Given the clinical presentation of a 58-year-old postmenopausal woman with a suspicious breast lump, the most appropriate course of action is to perform a biopsy to obtain a tissue diagnosis. The characteristics described (non-tender, irregular, firm lump with calcification on mammography and a solid mass on ultrasound) are concerning for malignancy.

Most appropriate course of action:

B. Core biopsy

Explanation:

•	Repeat mammography and ultrasound scans in 6 months (A): Delaying further investigation in a postmenopausal woman with a suspicious breast lump is not appropriate. Prompt evaluation is necessary to rule out malignancy.
•	Core biopsy (B): A core biopsy provides a tissue sample for histopathological examination, which is crucial for diagnosing the nature of the lump, especially given the concerning features.
•	Fine needle aspiration (FNA) to ensure that the lump is not really fluid filled (C): FNA can help distinguish between cystic and solid masses, but it does not provide as much tissue as a core biopsy. In this case, core biopsy is preferred for a more definitive diagnosis.
•	Repeat mammography and ultrasound scans in 3 months (D): Similar to option A, this would delay necessary diagnosis and treatment. Immediate biopsy is more appropriate.
•	Reassure and discharge (E): Given the suspicious features of the lump, reassurance and discharge without further investigation is not appropriate.

Conclusion:

A core biopsy is the most appropriate next step to definitively diagnose the nature of the breast lump and guide further management.

How well did you know this?
1
Not at all
2
3
4
5
Perfectly
34
Q

A 48-year-old perimenopausal woman presents with a 2-month history of a painful lump in her right breast. On examination you find a 2.5 cm tense, fluctuant, mobile lump in the outer lower quadrant of the right breast. The most
appropriate next course of action is

A Mammography and core biopsy
B Core biopsy
C Fine needle aspiration
D Computed tomography scan
E Mammography and ultrasound

A

Considering the patient’s age and the need for a thorough evaluation to rule out malignancy, the combination of imaging techniques is indeed crucial. In women over 40, breast lumps should be thoroughly evaluated to exclude breast cancer, even if the lump appears cystic.

Most appropriate next course of action:

E. Mammography and ultrasound

Explanation:

•	Mammography and core biopsy (A): Core biopsy is invasive and typically follows imaging if needed.
•	Core biopsy (B): Not the initial step without prior imaging.
•	Fine needle aspiration (C): While FNA can confirm a cyst and provide symptom relief, it does not provide the comprehensive assessment needed to exclude malignancy.
•	Computed tomography scan (D): Not typically used for initial evaluation of breast lumps.
•	Mammography and ultrasound (E): This combination is the best initial approach, especially in women over 40, as it provides detailed information about the nature of the lump. Mammography can identify suspicious calcifications, and ultrasound can differentiate between solid and cystic lesions.

Conclusion:

For a 48-year-old perimenopausal woman with a breast lump, mammography and ultrasound is the most appropriate initial course of action. This approach ensures a comprehensive assessment to rule out malignancy while providing detailed information about the lump’s nature.

How well did you know this?
1
Not at all
2
3
4
5
Perfectly
35
Q

During a ward round, you are asked by your surgical registrar about the
management of a phaeochromocytoma. Select from the list below the most
appropriate management plan for a phaeochromocytoma.
A Lifelong Beta and alfa blockade
B Alfa blockade, followed by ß blockade followed by surgical resection
C Surgical resection, followed by B blockade, followed by alfa blockade
D B blockade, followed by alfa blockade, followed by surgical resection
E Surgical resection

A

Alfa-blockade, followed by B-blockade followed by surgical resection Definitive treatment of a phaeochromocytoma is surgical resection, but before this is performed, pharmacological therapy is required to antagonize the effects of serum circulating catecholamines. a blockade with phenoxybenzamine is usually commenced 7-10 days before surgery, which allows blood volume expansion. Once this is achieved, ß blockade therapy is started. ß blockade cannot be initiated too early as unopposed a receptor stimulation may precipitate a hypertensive crisis. Once the patient has been established on adequate a and ß blockade, surgical resection of the tumour is carried out.

How well did you know this?
1
Not at all
2
3
4
5
Perfectly
36
Q

A 36-year-old nulliparous woman attends your clinic with a 7-day history of left breast pain after being involved in a car accident. On examining her breast, you notice a hard, irregular 3 cm, immobile, tender lump. You also notice some skin tethering and overlying bruising in the region of the lump. Ultrasound features suggest a benign pathology. The most likely diagnosis at this point is

A Breast abscess
B Breast cyst
C Mammary duct ectasia
D Fat necrosis
E Fibroadenosis

A

Fat necrosis
The most likely answer here is fat necrosis. Although the irregularity and hardness of the breast lump with overlying skin tethering suggests breast carcinoma, the history of recent trauma coupled with overlying breast lump tenderness and bruising indicates fat necrosis rather than breast cyst, fibroadenosis and breast abscess. Breast cyst, fibroadenosis and breast abscess are all unlikely here because of the history.

How well did you know this?
1
Not at all
2
3
4
5
Perfectly
37
Q

A 38-year-old woman, and mother of two healthy children, is diagnosed with a fluid-filled simple cyst after triple assessment. On ultrasound the inner surface
of the cyst looks entirely smooth. The woman does not have any significant
family history of carcinoma and the cyst is located in the outer-lower quadrant
of the right breast. What would be the most appropriate course of action?

A Follow-up appointment in 3 months
B Wide local excision
C Core biopsy
D Annual follow-up
E Reassure and discharge

A

Annual follow-up
Once a breast cyst is diagnosed and imaging features are benign, there is usually a follow-up with the patient in 1 year. Wide local excision, follow-up appointment in 3 months, reassure and discharge and core biopsy are incorrect. Benign breast cysts usually produce a strawcoloured aspirate. Once drained, fluid-filled benign cysts usually collapse and the lump disappears. If the aspirate is blood stained or the lump fails to resolve, reevaluation with triple assessment is done within a few weeks.

How well did you know this?
1
Not at all
2
3
4
5
Perfectly
38
Q

A 67-year-old woman, with a 25-year smoking history, on hormone replacement therapy, presents to clinic expressing concerns regarding an increase in the size of her right breast over the last 4 months. On examination, you find a non-tender, mobile, lobulated 10 cm mass with relatively smooth edges in the right breast. The right breast is significantly larger than the left and has a ‘teardrop” appearance and the skin looks normal. The most likely diagnosis here is
A Inflammatory breast carcinoma
B Malignant phylloides tumour
C Paget’s disease of the nipple
D Breast abscess
E Fibroadenoma

A

Phyllodes tumour
The most likely answer here is a malignant phyllodes tumour which accounts for 0.5% of all breast tumours.
The recent increase in size of the right breast coupled with the discovery of a non-tender, mobile lump, giving the right breast a ‘tear drop’ appearance, is suggestive of a malignant phyllodes tumour. Phyllodes tumours have many of the clinical and histological features of fibroadenomas (which are common in women in their early to late twenties). Inflammatory breast carcinoma, as its name suggests, is associated with pain (unlike other breast cancers), breast erythema, peau d’orange (the skin overlying the breast resembles that of red-coloured orange peel) and skin ridging with or without a palpable mass. Paget’s disease of the nipple presents as a unilateral, non-itchy, irregular eczematoid eruption of the nipple. It can be easily confused with eczema of the nipple, which usually occurs bilaterally, with pruritus, and can be nipple sparing. Breast abscess is unlikely here, and for more information, please refer to the answer to Question 2.

How well did you know this?
1
Not at all
2
3
4
5
Perfectly
39
Q

A worried 59-year-old city worker arrives at your clinic with a 1-month history of having noticed a non-itchy, persistent, burning rash in the region of her right breast. On examination, you find the right nipple and the skin overlying the
areola to be red and eczematous. The most likely cause is
A Malignant phyllodes tumour
B Basal cell carcinomal
C Breast abscess
D Mastitis
E Paget’s disease of the nipple
next

A

Paget’s disease of the nipple
The most likely answer here is Paget’s disease of the nipple. The nonitchy, eczematoid changes in the overlying skin of the right nipple and accompanying ymphadenopathy suggests underlying breast malignancy rather than eczema. Paget’s disease of the nipple is almost always associated with an underlying intraductal or invasive carcinoma. Early BCC lesions consist of raised pearly pink papules with fine overlying areas of telangiectasia. Late BCC lesions gradually ulcerate and are often referred to as ‘rodent ulcers’. A malignant phyllodes tumour is unlikely here due to the absence of a lump and increase in size of the breast. Mastitis and breast abscess are also unlikely due to the presenting history and clinical findings (please refer to the answer to Question 2).

How well did you know this?
1
Not at all
2
3
4
5
Perfectly
40
Q

A 47-year-old perimenopausal woman presents with a 3-week history of green discharge from the right nipple. On examination, the right nipple is non-tender, has a ‘slit-like’ appearance and is retracted. The most likely diagnosis is
A Breast carcinoma
B Duct papilloma
C Galactorrhoeal
D Fibroadenoma
E Mammary duct ectasia

A

Mammary duct ectasia This presentation is typical of mammary duct ectasia, which is common in the decade around the menopause. The nipple discharge can vary, ranging from milky, brown to a dirty green colour. The nipple discharge can be bilateral and occasionally it is associated with cyclical mastalgia. The history of nipple retraction can cause confusion, leading the clinician to suspect breast carcinoma. However, the presence of the slit in the nipple, coupled with the colour of the nipple discharge and the perimenopausal age of the patient, make it more likely to be duct ectasia. Galactorrhoea is unlikely here as nipple discharge is usually bilateral and milky in appearance and this usually follows lactation.
Duct papillomas usually present with blood-stained nipple discharge. Fibroadenoma does not usually present with nipple discharge and is common in women under the age of 35.

How well did you know this?
1
Not at all
2
3
4
5
Perfectly
41
Q

A 25-year-old woman is diagnosed with a 1.5 cm palpable area of fat necrosis of
the left breast by core biopsy following a traumatic injury 14 days earlier. She
has slight bruising of the lower outer quadrant of the left breast with moderate
tenderness. What would be the most appropriate course of management?

A Left mastectomy
B Wide local excision
C Reassurance and discharge
D Follow-up appointment for ultrasound in 3 months
E
Follow-up appointment for ultrasound in 6 months

A

A Reassurance and discharge Traumatic fat necrosis of the breast, in most cases, does not warrant any treatment or follow-up and will usually resolve with time. In some cases, fat necrosis can leave a hard, irregular lump (with some overlying skin tethering) which can be mistaken for carcinoma. History of recent trauma to the breasts) makes carcinoma unlikely. Core biopsy can be performed to confirm the diagnosis.
Therefore, follow-up appointment in 3 months, wide local excision, left mastectomy, and follow-up appointment in 6 months are all incorrect.

How well did you know this?
1
Not at all
2
3
4
5
Perfectly
42
Q

After a triple assessment, including core biopsy, a 28-year-old woman is diagnosed with a fibroadenoma of the left breast. The patient has a significant
family history of breast carcinoma. The non-tender lump is situated in the inner
lower quadrant of the left breast. The lump is approximately 1.5 cm x 1.5 cm.
What is the most appropriate course of management?

A The patient should be given the choice of excision or not and if not she could be discharged
B Perform triple assessment again in 6 weeks
C Reassure and follow-up after 3 months
D Fine needle aspiration
Excision of the lump

A

Excision of the lump
After a diagnosis of fibroadenoma is made, small lesions (i.e. <2.5 cm) do not warrant excision. In most cases, patients are reassured and discharged from clinic, but due to the patient’s significant family history of breast carcinoma, it is essential, and reassuring to her, for the lump to be excised. Therefore, excision of the lump is the most appropriate answer in this clinical vignette.

How well did you know this?
1
Not at all
2
3
4
5
Perfectly
43
Q

A 45-year-old perimenopausal woman is diagnosed with mammary duct ectasia of the right breast after having had small and infrequent amounts of milky green discharge from multiple ducts of the right nipple for over 2 months. The patient
has no significant family history and mammography findings are normal. What is
the most appropriate course of management?

A Mastectomy of the right breast
B Reassure and discharge
C Perform mammography of the right breast in 3 months
D Commence antibiotic therapy
E Surgical resection of the duct system of the right breast (Hadfield’s operation)

A

Reassure and discharge Mammary duct ectasia does not warrant treatment providing investigations are normal.
Duct excision is only performed if the discharge is frequent and excessive. Antibiotic therapy is not usually a treatment option for this condition and will not provide a cure. Performing a mammography in 3 months is unlikely to be beneficial. Mastectomy of the right breast is not a recommended treatment modality.

How well did you know this?
1
Not at all
2
3
4
5
Perfectly
44
Q

A 30-year-old woman who is 12 days postpartum and breastfeeding is diagnosed with acute mastitis of the left breast. Four days earlier, she discovered a painful crack in the region of the left nipple and noticed that the surrounding skin was tender, warm and red in colour. The patient is not allergic to penicillin and you decide to prescribe a course of antibiotics. What would be the most appropriate antibiotic for treating this condition?
A Flucloxacillin
B Cephalexin
C Amoxicillin
D Erythromycin
E Ciprofloxacin

A

Flucloxacillin
Flucloxacillin is the antibiotic of choice for treating lactational acute mastitis. The crack in the left nipple allows invasion by skin commensal Staphylococcus aureus, the most common cause of breast infection.
Flucloxacillin is a penicillinase-resistant antibiotic that is very effective against S. aureus infections.
Breastfeeding or milk expression is still encouraged during treatment and has shown to quicken recovery.
Amoxicillin (from the penicillin family), and ciprofloxacin (from the quinolone family) are ineffective against S. aureus. Erythromycin (from the macrolide family) is usually administered to patients who are allergic to penicillin formulas. It is not as effective as penicillinase-resistant antibiotics against S. aureus. Cephalexin (from the cephalosporin family), a moderate-spectrum antibiotic, can be used for S. aureus infections.
However, cephalosporins are not considered a first line antibiotic therapy for the treatment of breast abscesses due to moderate-spectrum activity. In this case, the patient is not allergic to penicillin and thus flucloxacillin is the more appropriate choice of treatment.

How well did you know this?
1
Not at all
2
3
4
5
Perfectly
45
Q

In which one of the following scenarios is the complication of lymphoedema of
the arm more likely to occur after resection of a breast tumour and axillary clearance?
A Mastectomy + axillary clearance + systemic chemotherapy
B Mastectomy + axillary clearance + postoperative radiotherapy to the chest wall
C Mastectomy + axillary clearance
D Mastectomy + axillary clearance + postoperative radiotherapy to the axilla
E Mastectomy + postoperative radiotherapy

A

Mastectomy + axillary clearance + postoperative radiotherapy to the axilla
Lymphoedema of the arm usually occurs following the combination of surgical axillary clearance and postoperative radiotherapy to the axilla. This therapeutic combination is therefore not offered to patients as it is associated with an unacceptably high incidence of lymphoedema. Mastectomy + axillary clearance + postoperative radiotherapy to the chest wall, mastectomy + axillary clearance + systemic chemotherapy, mastectomy + axillary clearance and mastectomy + postoperative radiotherapy are incorrect options where the incidence of lymphoedema is relatively lower compared with mastectomy + axillary clearance + postoperative radiotherapy to the axilla.

How well did you know this?
1
Not at all
2
3
4
5
Perfectly
46
Q

A 34-year-old premenopausal woman presents to your clinic with a lump in her
right breast. On examination you find a 2.5 cm fluctuant, mobile, tender lump in
the inner lower quadrant of the right breast. The ultrasound report suggests a
benign fluid-filled cyst. The most appropriate course of action is
A Ultrasound guided core biopsy
B Ultrasound guided fine needle aspiration
C Mammography
D Reassure and discharge
E Breast magnetic resonance imaging

A

Ultrasound guided fine needle aspiration
Ultrasound assessment of this patient’s breast lump confirms that it is a fluid-filled cyst with benign features.
The next best course of action is therefore to offer the patient fine needle aspiration of the cyst. The extracted fluid is usually discarded. In some cases, if the Radiologist suspects that the cyst has indeterminate features, the aspirated cystic fluid may be sent off for cytological analysis (if but in some cases the fluid may be blood). Breast MRI is used in the assessment of multifocal/ bilateral disease and also in patients with cosmetic implants who are at a predisposed risk of breast cancer. Core biopsy, reassure and discharge and mammography are thus incorrect answers. For further clarification, please refer to the answer to Question 7.

How well did you know this?
1
Not at all
2
3
4
5
Perfectly
47
Q

A 25-year-old woman presents to your clinic after discovering, for the first time, two lumps in the inner lower quadrant of her left breast. On examination you find these lumps to be 2 cm in size, solid, discrete, mobile and non-tender. The right breast is normal and there is no lymphadenopathy. The most appropriate
course of management is
A Request a core biopsy
B Request a mammogram
C Reassure the patient and discharge her
D Request fine needle aspiration
E Request an ultrasound of the left breast

A

Request an ultrasound of the left breast All breast lumps are investigated using ‘triple assessment’ which involves clinical examination, ultrasound breast scans for women younger than 35 (due to relatively denser breast tissue) or mammograms for women older than 35, followed by fine needle aspiration cytology and/or core biopsy for cytological and histological assessment, respectively. Women over the age of 35 tend to have less dense breast tissue, which increases the sensitivity and specificity of mammography.
This patient is 25, which coupled with the discovery of two new lumps, indicates that the next line investigation, following clinical examination, would be an ultrasound of the left breast. Mammography would not be suitable as this patient is under the age of 35.
Reassurance and discharge is clearly wrong as all breast lumps should undergo triple assessment before the patient is discharged. Fine needle aspiration and core biopsy, although they will be carried out, are not considered until radiological assessment has been performed.

How well did you know this?
1
Not at all
2
3
4
5
Perfectly
48
Q

You are attending a breast multidisciplinary team (MDT) meeting where the core biopsy histology results of a suspicious breast lesion are being discussed in a 55-year-old woman presenting with a right sided breast lump. The histopathologist states that the breast lesion possesses ‘B5b’ histology features. What is the most likely diagnosis?
A No breast abnormality
B Fibroadenoma
C Invasive breast carcinoma
D Benign breast cyst
E Ductal carcinoma in-situ

A

Invasive breast carcinoma
Triple assessment is adopted when investigating all breast lumps. Clinical history and examination is followed by imaging and then by cytological or histological analysis.
On palpation of a breast lump, a grade of 1-5 is given based on its clinical characteristics. These are:
• P1 - Normal breast tissue
• P2 - Benign breast tissue
• P3 - Suspicious but probably benign
• P4 - Suspicious but probably malignant
• P5 - Malignant
The same 1-5 grading is used for imaging, but an ‘M’ is used for mammography and ‘U’ for ultrasound.
Cytological grades are given as follows:
• C1 - Normal cell architecture or inadequate sample
• C2 - Benign cytology
• C3 - Suspcious but probably benign
• C4 - Suspicious but probably malignant
• C5 - Malignant
Histology grades for needle core biopsy are
• B1 - Normal breast tissue or inadequate sample
• B2 - Benign breast tissue
• B3 - Suspious but probably benign
• B4 - Suspicious but probably malignant
• B5a - Carcinoma in-situ (e.g. ductal/lobular carcinoma in situ)
• B5b - Invasive carcinoma (e.g. invasive ductal/lobular carcinoma)
With regard to the question, the breast lesion was given a ‘B5b’ histology grade from the needle core biopsy sample. This histology result implies invasive malignancy

How well did you know this?
1
Not at all
2
3
4
5
Perfectly
49
Q

A 28-year-old woman, who was hospitalized 2 months ago following a head injury, attends the outpatient clinic with a 6-week history of polyuria and
polydipsia and no other symptoms. Her blood pressure is 117/83 mmHg and her heart rate is 68 beats/min. From the list below, select the most appropriate
management option.
A Desmopressin
B Spironolactone
C Thyroxine
D Octreotide
E Carbimazole

A

Desmopressin
Symptoms of polyuria and polydipsia are common in many conditions. From the patient’s history, she had suffered a head injury (the triggering factor for her symptoms) and after 2 weeks her symptoms started.
The fact that she has no other symptoms makes it likely that this patient is suffering from diabetes insipidus, where vast amounts of dilute urine are secreted. Two forms exist: cranial diabetes insipidus (causes include head injury, cranial surgery, sarcoidosis), where there is a lack of ADH secretion from the posterior pituitary gland; and nephrogenic diabetes insipidus (causes include hypokalaemia, hypercalcaemia, drugs such as lithium and demecycline, genetic defects and heavy metal poisoning), where there is a lack of response of the kidneys to ADH. Diagnosis can be confirmed using the water deprivation test.
This patient has cranial diabetes insipidus. From the list of options, the most likely answer is treatment with desmopressin (a synthetic analogue of ADH).
Carbimazole is given to patients with hyperthyroidism.
Spironolactone, in the context of endocrine medicine, is given to patients with Conn’s syndrome. Thyroxine is given to hypothyroid patients and octreotide (a somatostatin analogue) can be used in patients with excessive growth hormone production (e.g. acromegaly) and patients with carcinoid syndrome.

How well did you know this?
1
Not at all
2
3
4
5
Perfectly
50
Q

A 33-year-old, non-smoking, breastfeeding woman is 10 days postpartum. She has a 4-day history of a slight crack on the surface of her left nipple. She presents with a 2-day history of severe continuous pain in the left breast, spiking pyrexia up to 38.8 with rigours which has prevented her from sleeping.
On examination, you find the outer quadrants of the left breast to be red, warm and tender with a hard 3 cm lump at the edge of the left nipple. The most likely diagnosis is
A Fat necrosis
B Breast cyst
C Breast abscess
D Acute mastitis
E Periductal mastitis

A

Breast abscess
Lactational breast abscesses are caused by the skin’s commensal microorganisms, nearly always staphylococci, infiltrating cracks in the nipple during breastfeeding. Segmental breast inflammation occurs, leading to cellulitis which, if not treated promptly, results in breast tissue necrosis, pus build up and abscess formation within the breast segment.
Continuous pain and sleepless nights coupled with the hard 3 cm painful lump suggest an abscess rather than acute mastitis. Fat necrosis is unlikely here with no history of previous minor local trauma. Periductal mastitis is possible, but classically occurs in non-lactating women of reproductive age and is associated with smoking. From the nature of the history and physical examination findings, it is unlikely to be a breast cyst.

How well did you know this?
1
Not at all
2
3
4
5
Perfectly
51
Q

A 21-year-old nulliparous woman presents to your clinic with a 1-month history of bilateral breast pain. The pain, which is dull and achy in nature, is poorly localized and widespread across both breasts. The pain gradually increases in
severity and is worse just before her menses. The pain usually starts to get
better once her menses start. On examination, both breasts are tender. There are no lumps, skin changes or obvious swellings. The most likely diagnosis here is
A Tietze’s syndrome
B Non-cyclical mastalgia
C Traumatic fat necrosis
D Acute bacterial mastitis
E Cyclical mastalgia

A

Cyclical mastalgia
The history of bilateral cyclical breast pain, which gradually intensifies before the start of the menses, and which is relieved once the menses start, is highly suggestive of cyclical mastalgia. The absolute cause of this condition is not yet clear, although it is thought to arise due to the sensitivity and responsiveness of breast tissue to hormones. Noncyclical mastalgia does not fit the history as it is not usually associated with the menstrual cycle. Tietze’s syndrome is characterized by tenderness usually over the second, third or fourth costochondral junctions because of chondritis of the costal cartilages. Acute bacterial mastitis is possible, but usually does not present bilaterally and is not associated with the menstrual cycle. The affected breast is usually erythematous, swollen and extremely tender.
Traumatic fat necrosis is clearly wrong as there is no history of recent trauma to the breasts.

How well did you know this?
1
Not at all
2
3
4
5
Perfectly
52
Q

A 43-year-old woman presents to your clinic with a 2-month history of localized
dull pain in the right breast. The pain intensifies just before her period. On
examination, you find a discrete 2.5 cm mobile, tense, tender, fluctuant lump in the lower inner quadrant of the right breast. The most likely diagnosis here is
A Breast cyst
B Fibroadenosis
C Fibroadenoma
D Fat necrosis
E Periductal mastitis

A

Breast cyst
Breast cysts usually occur in women over the age of 40 years through to the menopause. The aetiology in unclear, but it is thought that they occur due to hormonal imbalances around the menopause. Not all breast cysts manifest as pre-menstrual tenderness. The lump is mobile and fluctuant, suggesting that it is a fluid-filled cyst. Fibroadenomas classically present in females below the age of 35 and are infrequent after the age of 35-40. Fibroadenosis presents at a similar age, however the lump is fluctuant, making it more likely to be a cyst. Fat necrosis is unlikely here in the absence of previous minor local trauma (which is not stated in the history). Periductal mastitis is commonly seen in non-lactating women in their thirties (with a history of smoking) with pain usually developing in the areolar area.

How well did you know this?
1
Not at all
2
3
4
5
Perfectly
53
Q

A 46-year-old man is diagnosed with an ostrogen receptor positive invasive
ductal carcinoma of the right breast after having discovered a lump 3 months
before. The patient is found to have multiple involved axillary lymph nodes and
the tumour is of an aggressive phenotype. The most appropriate treatment
option for this patient is
A Cytotoxic chemotherapy and Tamoxifen but no surgery
B Palliative care programme
C Mastectomy + postoperative radiotherapy only
D Wide local excision and Tamoxifen only
E Mastectomy + axillary clearance + systemic chemotherapy + radiotherapy and tamoxifen

A

Mastectomy + axillary clearance + systemic chemotherapy + radiotherapy and Tamoxifen The treatment options for male breast cancer are similar to those for female breast cancer. The most common form of breast cancer in males is invasive ductal carcinoma, which is also the most common type in women. There are four clinical stages of ductal carcinoma and determining the clinical stage allows for the selection of initial treatment. Patients who have stage 1 or 2 (early breast cancer) are more suitable for surgery whereas surgery should be avoided in patients who have stage 3 and 4 breast cancer, that is locally advanced cancer or metastatic spread at presentation, respectively, and they should be treated with chemotherapy. In this case, the patient has stage 2 breast cancer, implying that he is suitable for surgery. In addition, this tumour is ostrogen receptor positive; 90% of male breast cancers are oestrogen receptor positive.
Mastectomy + axillary clearance, followed by systemic chemotherapy, radiotherapy and Tamoxifen, is the most suitable answer out of the five options. Chest wall radiotherapy coupled with systemic chemotherapy carries a better prognosis for the patient. Mastectomy alone would ensure tumour removal, but would not ensure the clearance of micrometastases, and the risk of recurrence/metastases would be higher without chemo/radiotherapy. Wide local excision would be difficult due to the invasive nature of the tumour and ensuring tumour clear margins of tissue. Mastectomy + radiotherapy is possible, but the addition of chemotherapy and hormonal therapy with Tamoxifen carries a better prognosis because of the higher Likelihood of removal of micrometastases. Patients who have stage 4 breast cancer (metastases at presentation) are usually offered palliative care

How well did you know this?
1
Not at all
2
3
4
5
Perfectly
54
Q

A worried 23-year-old woman, who started taking the combined contraceptive pill 3 months ago, presents with a 1-day history of discovering a painless lump in the right breast. The patient states that the lump was not there a month ago.
On examination, a slightly mobile, discrete, well-defined, non-tender, firm 1 cm diameter lump is found. There is no lymphadenopathy. The most likely diagnosis here is
A Lipoma
B Breast cyst
C Carcinoma of the breast
D Fibroadenoma
E Sebaceous cyst

A

Fibroadenoma
Fibroadenomas classically present in females below the age of 35 and are infrequent after the age of 35-40.
They are described as painless, rubbery to firm, non-fluctuant, discrete mobile breast lumps commonly referred to as ‘breast mice’. Breast cysts occur more frequently after the age of 35 and may present with breast pain. They can be fluctuant if fluid filled and characteristically are tense, discrete, mobile lumps.
Carcinoma of the breast is rare under the age of 35, and usually presents as a solitary, painless, ill-defined lump of varying size which may show signs of skin tethering.
Lipomas, although very common, are usually soft, fluctuant, irregularly defined lumps. Sebaceous cysts are typically round, soft lumps attached to the skin and have a central punctum.

How well did you know this?
1
Not at all
2
3
4
5
Perfectly
55
Q

Which statement considering the relations of nerves to the humerus is the most accurate?
A Mid-shaft humeral fractures will usually result in complete paralysis of triceps.
B The ulnar nerve is related to the lateral epicondyle.
C Deltoid may atrophy following shoulder dislocation.
D The axillary nerve runs around the anatomical neck.
E The median nerve runs in the spiral groove.

A

C-Deltoid will atrophy as a result of severe damage to the axillary nerve, which may occur following dislocation of the shoulder. The axillary nerve also innervates teres minor and skin over the lateral side of the proximal part of the arm. Sensory loss in this area suggests nerve damage.
The axillary nerve (C5, C6) winds around the surgical neck of the humerus, accompanying the posterior circumflex humeral vessels. It is prone to damage in humeral neck fractures or dislocation of the shoulder.
The radial nerve runs in the spiral groove, where it is accompanied by the profunda brachii artery.
Mid-shaft humeral fractures may damage the radial nerve as it runs in the spiral groove. The branches to the lateral and long head of triceps arise before the nerve enters the groove, and thus only the medial head is affected. The characteristic sign of this injury is wrist drop due to paralysis of the posterior compartment of the forearm.
The ulnar nerve passes posterior to the medial epicondyle where it is superficial, easily palpable and prone to injury.

How well did you know this?
1
Not at all
2
3
4
5
Perfectly
56
Q

Which of the following is not a risk factor for gallstone formation?
A Smoking
B Pregnancy
C Diet high in fats
D Contraceptive pill
E Crohn’s disease

A

Smoking.
Factors predisposing to gallstone formation include sepsis within the biliary tree, anatomical variants which predispose to stasis and changes in the composition of bile which make it lithogenic. Amirand’s triangle is a diagrammatic representation of how changes in bile composition can predispose to stone formation. Put simply, if the percentage composition of cholesterol increases, or the percentage composition of bile salts or phospholipids (lecithin) decreases, then the bile is more likely to form gallstones.
When taking a focused history and examination, candidates should aim to elicit/exclude the following risk factors for gallstone formation:
• Race (higher incidence in black and Asian populations)
• High-fat diet/history of hypercholesterolaemia
• Obesity
• High ostrogen states (CCP, pregnancy, PCOS; hence the slight female predominance)
• Haemolytic states
The loss of the terminal ileum following surgery or as a consequence of Crohn’s disease also predisposes to gallstone formation as the terminal ileum is the site of re-absorption of bile salts. Smoking is not known to directly influence the risk of gallstone formation.

How well did you know this?
1
Not at all
2
3
4
5
Perfectly
57
Q

A 60-year-oid man is found to have a 2 cm diameter mass in the upper lobe of his left lung. The lesion is excised and it is found that the lesion includes connective tissue, mature cartilage and ciliated epithelium. What is the most likely diagnosis?

A Sarcoma
B Hamartoma
C Adenomal
D Chondroma
E Carcinoma

A

Hamartomas are fundamentally comprised of disordered replications of normal tissue cells. The underlying mechanisms of anomalous replications are not fully recognized. The particular property is a clearly demarcated mass mainly containing fat and cartilage, but other tissue cells may also be present depending on the anatomic location. The size of most hamartomas is between 1 to 3 cm. References
https://www.ncbi.nlm.nih.gov/books/NBK562298/

How well did you know this?
1
Not at all
2
3
4
5
Perfectly
58
Q

A 35-year-old woman undergoes gastric bypass surgery for morbid obesity. At a subsequent surgical clinic review she complains of dizziness, sweating, palpitation and collapsing episodes after big meals without any vomiting or pain.
She is otherwise well and has no medical problems. What is the most likely explanation for her symptoms?

A Vasovagal syncope
B Operative denervation of stomach
C Release of insulin
D Release of gastrin
E Release of glucagon

A

The patient’s symptoms of dizziness, sweating, palpitations, and collapsing episodes after meals, particularly large ones, suggest dumping syndrome, a common complication after gastric bypass surgery. This condition is typically caused by the rapid transit of hyperosmolar food from the stomach into the small intestine, leading to a rapid fluid shift and subsequent release of hormones.

Key Points:

•	Symptoms: Dizziness, sweating, palpitations, collapsing episodes after meals.
•	Context: Post-gastric bypass surgery.

Likely Mechanism:

•	Early Dumping Syndrome: Occurs 15-30 minutes after eating, caused by rapid gastric emptying.
•	Pathophysiology: Rapid entry of hyperosmolar contents into the small intestine leads to fluid shift, triggering autonomic responses and release of gastrointestinal hormones.
•	Release of Insulin: The rapid absorption of carbohydrates can lead to a quick spike in blood glucose followed by a compensatory release of insulin, which can cause hypoglycemia and associated symptoms like dizziness, sweating, and palpitations.

Explanation:

•	C Release of insulin: The rapid transit of food into the small intestine and subsequent hyperglycemia results in a quick release of insulin, which then causes hypoglycemia and the described symptoms.

Answer:

C Release of insulin

How well did you know this?
1
Not at all
2
3
4
5
Perfectly
59
Q

A 22-year-old man arrives to the Emergency Department with sudden breathlessness due to a large pneumothorax. A chest drain is inserted into the fifth left intercostal space in 5th mid-axillary line. There is haemorrhage into the drainage bottle. Which of the following structures is the most likely cause of this acute haemorrhage?

A Intercostal artery
B Right ventricle of the heart
C Lingula of the lung
D Spleen
E Left pericardiophrenic artery

A

A chest drain can cause an intercostal artery bleed during insertion or movement, either by direct trauma or tissue injury to the artery. Improper technique or anatomical variations can increase this risk. Prompt recognition and management are crucial. References
https://www.ncbi.nlm.nih.gov/pmc/articles/PMC861090
4/#:-:text=Complications%20such%20as%20intercost| al% 20artery,procedure%2C%20unlike%20in%20cathet|
er%20removal.

How well did you know this?
1
Not at all
2
3
4
5
Perfectly
60
Q

A 2-year-old child is referred because of an impalpable testis in the left
scrotum. Which of the following positions describes an ectopic testis?

A in the upper part of the scrotum
B In the inguinal canal.
C At the superficial inguinal ring
D At the root of the penis
E At the deep inguinal ring

A

The most common region in descending order: superficial inguinal pouch which lies anterior to the external oblique aponeurosis, perineum / anterior to root of penis, femoral triangle / upper thigh,
contralateral scrotum. References
https://radiopaedia.org/articles/ectopic-testis

How well did you know this?
1
Not at all
2
3
4
5
Perfectly
61
Q

An 82-year-old man has complete occlusion of his inferior mesenteric artery on angiography but no symptoms or signs of colonic ischemia. Which of the following arteries is the most likely additional source of blood supply to the
territory of the inferior mesenteric artery?

A Left colic
B Left gastroepiploic
C Superior rectal
D Splenic
E Middle colic

A

The middle colic artery passes in the layers of the transverse mesocolon to the transverse colon and divides into right and left branches: Right branch supplies the right portion of the transverse colon and anastomoses at the hepatic flexure with the ascending branch of the right colic artery. Left branch supplies the left portion of the transverse colon and anastomoses at the splenic flexure with the left colic artery of the inferior mesenteric artery.

References
https://radiopaedia.org/articles/middle-colic-artery

How well did you know this?
1
Not at all
2
3
4
5
Perfectly
62
Q

A 52-year-old man is found to have multiple myeloma. What skull X-ray
appearances would be characteristic?

A Multiple osteolytic lesions
B Hair on end appearance.
C Diffuse thickening of the calvarium
D Multiple fractures
E Multiple osteosclerotic lesions

A

The raindrop skull appearance of calvarial multiple myeloma is the presence of well defined lytic lesions (punched out lesions) of various sizes scattered throughout the skull.

References
https://radiopaedia.org/cases/multiple-myeloma-skull-

How well did you know this?
1
Not at all
2
3
4
5
Perfectly
63
Q

A 30-years-old alcoholic man presents with acute severe upper abdominal pain and vomiting. He is admitted to the intensive therapy unit with a diagnosis of severe acute pancreatitis Forty-eight hours later he develops peripheral paraesthesiae and carpopedal spasms. The most likely underlying metabolic abnormality is:-

A hypercalcaemia
B hypernatraemia
C hyponatraemia
D hypocalcaemia
E hypokalaemia

A

Pancreatitis can be associated with tetany and hypocalcemia. Ranson Criteria:

Criteria and Prognosis
At Admission
• Age > 55 years
• Leukocyte count > 16 x 103/mcL
• Blood glucose > 200 mg/dL
• Serum LDH > 350 IU/L
• Serum AST > 250 IU/L
At 48 hours
• Decrease in hematocrit > 10%
• Increase in BUN of > 8 mg/dL
• Serum calcium less than 8 mg/dL
• Pa02 < 60 mm Hg
• Base deficit > 4 mEq/L
• Estimated fluid sequestration > 6,000 mL
Score < 3 = Mortality 0-3% • Score 2 3 = Mortality 11-15% • Score ≥ 6 = Mortality 40%

References
https://emedicine.medscape.com/article/241893-
overview#:-:text=Pancreatitis%20can%20be%20assoc iated%20with,secretion%20may%20play%20a%20role

How well did you know this?
1
Not at all
2
3
4
5
Perfectly
64
Q

A 26-year-old man presents to the Emergency Department with extensive bleeding from his arm after sustaining a glass injury. On examination there is a 7 cm transverse laceration across the anterior aspect of his elbow. On exploring the cubital fossa you would expect the brachial artery to be:

A medial to the median nerve
B anterior to the median nerve
C lateral to the median nerve
D superficial to the bicipital aponeurosis
E lateral to the biceps tendon

A

In the cubital fossa, the anatomical relationships of the structures are crucial for surgical exploration and management. The brachial artery has a specific relationship to the median nerve and other structures in this region.

The correct answer is C lateral to the median nerve.

Explanation:

The anatomical arrangement of structures in the cubital fossa from medial to lateral is often remembered by the mnemonic “My Brother Throws Rad Parties,” which stands for:

•	Medial nerve
•	Brachial artery
•	Tendon of the biceps brachii
•	Radial nerve (superficial and deep branches)

So, in detail:

•	The median nerve lies most medially.
•	The brachial artery lies just lateral to the median nerve.
•	The biceps tendon lies lateral to the brachial artery.
•	The radial nerve is the most lateral structure in the cubital fossa.

Given these anatomical relationships:

•	A: Medial to the median nerve – Incorrect. The brachial artery is lateral to the median nerve.
•	B: Anterior to the median nerve – Incorrect. The brachial artery is not anterior to the median nerve.
•	C: Lateral to the median nerve – Correct. This matches the anatomical relationship in the cubital fossa.
•	D: Superficial to the bicipital aponeurosis – Incorrect. The brachial artery is deep to the bicipital aponeurosis.
•	E: Lateral to the biceps tendon – Incorrect. The brachial artery is medial to the biceps tendon.
How well did you know this?
1
Not at all
2
3
4
5
Perfectly
65
Q

A 12-year-old girl has a diastolic murmur. It is maximally audible in the second
left intercostal space. Pathology of which structure is the most likely cause?
A Ductus arteriosus
B Pulmonary valve
C Aortic valve
D Mitral valve
E Tricuspid valve

A

Second left intercostal space is where pulmonary value is located.
EXPLANATION OF THE AUSCULTATION AREAS

A(Aortic area): The second intercostal space to the right of the sternum.
B(Pulmonic area: The second intercostal space to the left of the sternum.
C(Tricuspid area: The fourth intercostal space to the left of the sternum.
D(Mitral area) : The fifth intercostal space in the left midclavicular line.

How well did you know this?
1
Not at all
2
3
4
5
Perfectly
66
Q

A nulliparous 30-year-old Woman presents with a recurrent painful red area in
the areola. She gives a history of smoking cigarettes. The lesion is excised and
the histology shows squamous metaplasia of lactiferous ducts. Microbiological
culture does not grow any organism. Which of the following is the most likely diagnosis?

A Carcinoma of the breast
B Galactorrhea
C Breast abscess
D Fibroadenoma
E Subareolar abscess

A

A subareolar abscess is a painful collection of pus that forms beneath the areola, usually as a result of an infected milk duct. The presentation often includes localized pain, redness, swelling, and warmth in the affected breast area. Squamous metaplasia of lactiferous ducts can occur in cases of subareolar abscess. Squamous metaplasia is a process where the normal glandular epithelium lining the lactiferous ducts undergoes transformation into squamous epithelium.
This transformation is often associated with chronic inflammation and infection in the breast, such as in the case of a subareolar abscess.

References
https://www.ncbi.nlm.nih.gov/pmc/articles/PMC773308
8/#:-:text=Squamous%20metaplasia%20of%20lactifer
ous%20ducts%20(SMOLD)%20was%20previously%20 known%20as,changes%20to%20the%20breast%20duc
ts.

How well did you know this?
1
Not at all
2
3
4
5
Perfectly
67
Q

A 55-year-old man presents with a deep venous thrombosis. His full blood count shows a haemoglobin of 18.3 g/dL (normal 13.5-17.5), a white cell count of 8 x 10^9/L (normal 3.9- 10.0 x 10^9) and a platelet count of 200 x 10^9/L (normal
150-400 x 10^9) This hematological picture is most likely to be associated with:

A bronchial carcinoma
B transitional cell carcinoma of the kidney
C renal cell carcinoma
D prostatic carcinoma
E pancreatic carcinoma

A

The hematological picture presented, which includes an elevated hemoglobin level (polycythemia), is most likely associated with renal cell carcinoma.

Correct Answer:

C renal cell carcinoma

Explanation:

Renal cell carcinoma (RCC) is known for its potential to produce erythropoietin (EPO), a hormone that stimulates red blood cell production. This can lead to secondary polycythemia, which is characterized by elevated hemoglobin levels as seen in this patient. The other blood parameters (white cell count and platelet count) are within normal ranges, which is consistent with the isolated effect on red blood cell production typically seen in RCC-induced polycythemia.

•	A bronchial carcinoma: While bronchial carcinoma can present with various paraneoplastic syndromes, polycythemia is not commonly associated with it.
•	B transitional cell carcinoma of the kidney: This type of carcinoma does not typically produce EPO or lead to polycythemia.
•	D prostatic carcinoma: This carcinoma does not commonly cause polycythemia.
•	E pancreatic carcinoma: Pancreatic carcinoma can be associated with various paraneoplastic syndromes, but polycythemia is not typically one of them.

Given the patient’s elevated hemoglobin level and the known association of RCC with EPO production, renal cell carcinoma is the most likely diagnosis.

How well did you know this?
1
Not at all
2
3
4
5
Perfectly
68
Q

A 72-year-old man with a body mass index of 18.4 has not eaten for four days following the removal of an adenocarcinoma from his descending colon. His urea
is found to be 12 mmol/L (normal 3.2-7.5) and creatinine 346 umol/L (normal 35-110). A blood gas profile is ordered. The most likely set of results would be:

A pH 7.36, p02 11.5kPa, pCo2 5.3kPa, Bicarbonate 30.5mmol/L
В рН 7.23, p02 13.6kPa, pCo2 3.8kPa, Bicarbonate 13.5mmol/L
С рН 7.43, p02 16.4kPa, pCo2 3.7kPa, Bicarbonate 21.0mmol/L
D pH 7.49, p02 12.6kPa, pCo2 3.9KPa, Bicarbonate 24.0mmol/L
Е рН 7.30, p02 8.8kPa, pCo2 8.3kPa, Bicarbonate 30.5mmol/L

A

Starvation ketoacidosis (SKA) represents one of three metabolic acidoses caused by the accumulation of ketone bodies within the bloodstream. While easily treated, it is a diagnosis that can be easily missed in patients with an unexplained metabolic acidosis.

The clinical scenario suggests that the patient is in acute kidney injury (AKI), likely due to dehydration and reduced intake following surgery, leading to elevated urea and creatinine levels. The patient has also not eaten for four days, which can contribute to metabolic derangements.

Given the elevated creatinine, the patient likely has reduced renal function, which often leads to a metabolic acidosis, characterized by a low pH, low bicarbonate, and a compensatory low pCO2 due to hyperventilation.

Correct Answer:

B рН 7.23, pO2 13.6kPa, pCO2 3.8kPa, Bicarbonate 13.5mmol/L

Explanation:

•	pH 7.23: Indicates acidosis (pH < 7.35).
•	pO2 13.6kPa: Oxygen levels can vary but are within a reasonable range.
•	pCO2 3.8kPa: Low pCO2 indicates respiratory compensation for metabolic acidosis (hyperventilation).
•	Bicarbonate 13.5mmol/L: Low bicarbonate is consistent with metabolic acidosis.

Other options are less likely because they do not fit the expected acid-base disorder in the context of acute kidney injury and metabolic stress from not eating:

•	A: Normal pH with high bicarbonate, suggesting metabolic alkalosis, which does not fit the clinical picture.
•	C: Normal to slightly elevated pH with low bicarbonate and low pCO2, suggesting respiratory alkalosis, which does not fit the clinical picture.
•	D: Elevated pH indicating alkalosis, which does not fit with the expected acidosis.
•	E: Low pH and high bicarbonate with high pCO2, suggesting a mixed disorder but not fitting with the typical response in acute kidney injury and fasting.
How well did you know this?
1
Not at all
2
3
4
5
Perfectly
69
Q

An intravenous drug abuser is having an echocardiogram for suspected endocarditis. Closure of the tricuspid valve occurs at the onset of which phase of the cardiac cycle?

A Isovolumetric contraction
B Isovolumetric relaxation
C Rapid ejection
D Atrial systole
E Rapid ventricular filling

A

The closure of the tricuspid valve occurs at the onset of the isovolumetric contraction phase of the cardiac cycle.
References
https://www.sciencedirect.com/topics/agricultural-and-
biological-sciences/isovolumetric-contraction

How well did you know this?
1
Not at all
2
3
4
5
Perfectly
70
Q

The pressure within the pleural space is negative with respect to atmospheric pressure, except for which of the following?

A At end inspiration
B If the patient has a tracheostomy
C During a Valsalva maneuver
D When taking a deep breath
E At end expiration

A

Initially, during a Valsalva, intrathoracic (intrapleural) pressure becomes very positive because of compression of the thoracic organs by the contracting rib cage.

Manoeuvre:
• Valsalva manoeuvre refers to a forced expiration against a closed glottis.
• This causes marked decrease in the thoracic volume causing deflation of lungs. Under such circumstances the intrapleural pressure can become positive by 60-70 mmHg.
• The common everyday activities in which Valsalva manoeuvre effect
straining
defecation,
phase
coughing,
during during
parturition.

In some of the pathological conditions such as
Pneumothorax
Hydrothorax
Hemothorde
Dothorax
Tends to be more positive.
-In Valsalva Maneuver pleural pressure becomes more positive.
-In reverse Valsalva/Mueller’s Maneuver pleural pressure becomes less positive.

References
https://cvphysiology.com/hemodynamics/h014

How well did you know this?
1
Not at all
2
3
4
5
Perfectly
71
Q

A 70-year-old woman from a nursing home presents to the Emergency
Department with abdominal pain and vomiting. On examination, she is
dehydrated and her abdomen is distended. There is a 3 cm - 4 cm swelling in her right groin which is non-tender, and there is no cough impulse. At operation, a femoral Hernia is found. Which of the following lies medial to the neck of the hernia?

A Femoral artery
B Femoral vein
C Femoral nerve
D Pectineal ligament
E Lacunar ligament

A

The correct answer to the question is:

E. Lacunar ligament

Explanation: In the anatomy of the femoral region, the femoral canal, through which a femoral hernia protrudes, is bounded by the lacunar ligament medially. The lacunar ligament is a crescent-shaped fibrous band that forms the medial boundary of the femoral ring. It is located medial to the femoral vein and, therefore, medial to the neck of a femoral hernia. This anatomical relationship is crucial for understanding the path of femoral hernias and their surgical treatment.

The femoral canal is an anatomical structure located in the femoral region. It is a small compartment within the femoral sheath, and its boundaries are important to understand, especially in the context of femoral hernias. The boundaries of the femoral canal are as follows:

1.	Anterior Boundary: Inguinal ligament.
2.	Posterior Boundary: Pectineal ligament (also known as the ligament of Cooper), which lies over the pectineus muscle and fascia.
3.	Medial Boundary: Lacunar ligament.
4.	Lateral Boundary: Femoral vein.

These boundaries form a conical space that allows for the passage of lymphatics and a small amount of fat. The femoral ring, the entrance to the femoral canal, is a potential weak spot where femoral hernias can occur.

How well did you know this?
1
Not at all
2
3
4
5
Perfectly
72
Q

48-year-old woman presents with abdominal pain and five to six episodes of
bloody diarrhoea each day. She also complains of pain- in the knees and elbows and recent visual problems. What is the most likely diagnosis?

A intestinal tuberculosis
Ulcerative colitis
C Infective colitis
D Collagenous colitis
E Diverticulitis

A

The symptoms described — abdominal pain, multiple episodes of bloody diarrhea, joint pain (knees and elbows), and visual problems — are highly suggestive of an inflammatory bowel disease (IBD). The most likely diagnosis among the options provided is:

B. Ulcerative colitis

Explanation:

•	Ulcerative colitis is a chronic inflammatory bowel disease that typically presents with bloody diarrhea, abdominal pain, and can also have extra-intestinal manifestations such as arthritis (pain in joints) and uveitis or episcleritis (visual problems).
•	Intestinal tuberculosis usually presents with chronic abdominal pain, weight loss, and sometimes diarrhea, but bloody diarrhea is less typical and extra-intestinal symptoms such as joint pain and visual problems are less common.
•	Infective colitis can cause bloody diarrhea, but the chronic joint and visual symptoms are less characteristic.
•	Collagenous colitis typically presents with chronic watery diarrhea and does not usually cause bloody diarrhea or extra-intestinal symptoms like joint pain and visual issues.
•	Diverticulitis typically presents with localized abdominal pain, often in the left lower quadrant, and does not usually cause bloody diarrhea or extra-intestinal symptoms.

Therefore, the combination of gastrointestinal and extra-intestinal symptoms in this patient most strongly suggests ulcerative colitis.

How well did you know this?
1
Not at all
2
3
4
5
Perfectly
73
Q

A 65-year old woman collapses after a total hip replacement. A pulmonary embolism is suspected. Which of the following electrocardiogram changes would support this diagnosis?
A Dominant R wave in V6
B T wave inversion in V1-V3
C Left axis deviation
D ST elevation in V1-V3
E Left bundle branch block

A

The electrocardiogram (ECG) change that would support a diagnosis of pulmonary embolism is:

B. T wave inversion in V1-V3

Explanation:

•	Pulmonary embolism often causes right ventricular strain, which can be reflected on an ECG. One of the common ECG changes seen in pulmonary embolism is T wave inversion in the right precordial leads (V1-V3).
•	Dominant R wave in V6 is not typically associated with pulmonary embolism.
•	Left axis deviation is more commonly associated with left-sided heart conditions and is not a typical finding in pulmonary embolism.
•	ST elevation in V1-V3 could indicate an acute myocardial infarction (MI) but is not typical for pulmonary embolism.
•	Left bundle branch block (LBBB) indicates a delay in electrical conduction in the left bundle branch and is not specific to pulmonary embolism.

Therefore, T wave inversion in V1-V3 is the most relevant ECG finding that would support the suspicion of a pulmonary embolism.

How well did you know this?
1
Not at all
2
3
4
5
Perfectly
74
Q

An 80-year-old man dislocates his shoulder it is reduced in the Emergency Department. At review three weeks later, he is unable to actively initiate abduction of his arm. Which muscle most likely damaged?

A Teres major
B Clavicular head oi pectoralis major
C Infraspinatus
D Trapezius
E Supraspinatus

A

The muscle most likely damaged in this scenario is:

E. Supraspinatus

Explanation:

•	The supraspinatus muscle is part of the rotator cuff and is primarily responsible for initiating the abduction of the arm. If the patient is unable to actively initiate abduction, it suggests damage to this muscle.
•	Teres major is involved in the internal rotation and adduction of the arm, not abduction.
•	The clavicular head of pectoralis major assists in flexion, adduction, and internal rotation of the arm.
•	Infraspinatus is another rotator cuff muscle, but it is primarily responsible for external rotation of the arm, not abduction.
•	The trapezius muscle is involved in moving, rotating, and stabilizing the scapula, and it assists in shoulder elevation but is not responsible for initiating abduction.

Given the specific inability to initiate abduction, damage to the supraspinatus muscle is the most likely cause.

How well did you know this?
1
Not at all
2
3
4
5
Perfectly
75
Q

Consider the dermatomes of the upper limb. Which of the following statements best completes this sentence? The skin over the:
A Posterior surface of the forearm derives its cutaneous innervation from T1.
B Clavicle derives its cutaneous innervation from C4.
C Thumb derives its cutaneous innervation from T1.
D Little finger derives its cutaneous innervation from C7.
E Lateral aspect of the upper limb derives its cutaneous innervation from C8 and T1.

A

The correct statement is:

B. Clavicle derives its cutaneous innervation from C4.

Explanation:

•	A. Posterior surface of the forearm derives its cutaneous innervation from T1: This is incorrect. The posterior surface of the forearm is typically innervated by C6 and C7 dermatomes.
•	B. Clavicle derives its cutaneous innervation from C4: This is correct. The skin over the clavicle is typically innervated by the C4 dermatome.
•	C. Thumb derives its cutaneous innervation from T1: This is incorrect. The thumb is innervated by the C6 dermatome.
•	D. Little finger derives its cutaneous innervation from C7: This is incorrect. The little finger is innervated by the C8 dermatome.
•	E. Lateral aspect of the upper limb derives its cutaneous innervation from C8 and T1: This is incorrect. The lateral aspect of the upper limb is primarily innervated by C5 and C6 dermatomes.

Therefore, the correct answer is that the skin over the clavicle derives its cutaneous innervation from C4.

How well did you know this?
1
Not at all
2
3
4
5
Perfectly
76
Q

Select the most appropriate ending for the following sentence. The median nerve:
A Supplies all of flexor digitorum profundus.
B Arises from both the posterior and medial cords of the brachial plexus.
C Gives no muscular branches in the arm.
D Enters the forearm by passing between the two heads of flexor carpi ulnaris.
E Passes from the medial to the lateral side of the brachial artery in the arm.

A

The most appropriate ending for the sentence is:

C. Gives no muscular branches in the arm.

Explanation:

•	A. Supplies all of flexor digitorum profundus: This is incorrect. The median nerve supplies only the lateral half (index and middle fingers) of the flexor digitorum profundus, while the ulnar nerve supplies the medial half (ring and little fingers).
•	B. Arises from both the posterior and medial cords of the brachial plexus: This is incorrect. The median nerve arises from both the lateral and medial cords of the brachial plexus.
•	C. Gives no muscular branches in the arm: This is correct. The median nerve does not supply any muscles in the arm; its motor branches are all in the forearm and hand.
•	D. Enters the forearm by passing between the two heads of flexor carpi ulnaris: This is incorrect. The median nerve enters the forearm by passing between the two heads of the pronator teres, not the flexor carpi ulnaris.
•	E. Passes from the medial to the lateral side of the brachial artery in the arm: This is incorrect. The median nerve passes from the lateral to the medial side of the brachial artery as it descends in the arm.

Thus, the most accurate statement is that the median nerve gives no muscular branches in the arm.

How well did you know this?
1
Not at all
2
3
4
5
Perfectly
77
Q

Which of the following statements about the axillary artery is accurate?
A The axillary vein lies medial throughout its course.
B It is divided into three parts by pectoralis major.
C It becomes the brachial artery at the inferior border of teres minor.
D The cords of the brachial plexus are named according to their positions relative to the first part.
E It gives the suprascapular artery as one of its branches.

A

The accurate statement about the axillary artery is:

A. The axillary vein lies medial throughout its course.

Explanation:

•	A. The axillary vein lies medial throughout its course: This is correct. The axillary vein lies medial to the axillary artery throughout its course in the axilla.
•	B. It is divided into three parts by pectoralis major: This is incorrect. The axillary artery is divided into three parts by the pectoralis minor muscle, not pectoralis major.
•	C. It becomes the brachial artery at the inferior border of teres minor: This is incorrect. The axillary artery becomes the brachial artery at the inferior border of the teres major muscle, not teres minor.
•	D. The cords of the brachial plexus are named according to their positions relative to the first part: This is incorrect. The cords of the brachial plexus are named according to their positions relative to the second part of the axillary artery.
•	E. It gives the suprascapular artery as one of its branches: This is incorrect. The suprascapular artery is a branch of the thyrocervical trunk, which is a branch of the subclavian artery, not the axillary artery.

Therefore, the most accurate statement is that the axillary vein lies medial throughout its course.

How well did you know this?
1
Not at all
2
3
4
5
Perfectly
78
Q

Which of the following statements regarding the radius and ulna is correct?
A Both articulate with the humerus at the elbow joint.
B Both have a styloid process at the proximal end.
C Fracture is most commonly of the Smith’s type.
D Direct injury usually produces transverse fractures of both bones in the distal third.
E Both articulate with the carpal bones at the wrist joint.

A

To determine which statement regarding the radius and ulna is correct, we will analyze each option step by step.

A. Both articulate with the humerus at the elbow joint.

The radius and ulna are both forearm bones that do indeed articulate with the humerus at the elbow joint. The ulna primarily forms the hinge joint with the humerus, while the radius also plays a role in this articulation, especially in terms of rotation during pronation and supination of the forearm. Therefore, this statement is correct.
B. Both have a styloid process at the proximal end.

The radius has a styloid process at its distal end, while the ulna has a styloid process at its distal end as well but not at its proximal end. This means that this statement is incorrect because only one of them has a styloid process at the proximal end.
C. Fracture is most commonly of the Smith’s type.

A Smith’s fracture refers to a fracture of the distal radius with volar angulation (the palm side). While fractures of the radius are common, particularly in older adults due to falls, they are more often Colles’ fractures rather than Smith’s fractures. Therefore, this statement is incorrect.
D. Direct injury usually produces transverse fractures of both bones in the distal third.

Direct injuries can lead to various types of fractures depending on the mechanism involved; however, transverse fractures specifically affecting both bones in their distal third are not typically described as common outcomes from direct trauma. This statement is therefore considered incorrect.
E. Both articulate with the carpal bones at the wrist joint.

The radius articulates directly with several carpal bones (specifically scaphoid and lunate) at the wrist joint, while the ulna does not have a direct articulation with any carpal bones; instead, it is separated from them by an articular disc (the triangular fibrocartilage complex). Thus, this statement is incorrect.
After analyzing all statements:

The only correct statement regarding both radius and ulna is:

A Both articulate with the humerus at the elbow joint.

How well did you know this?
1
Not at all
2
3
4
5
Perfectly
79
Q

Which of the following best describes the forearm?
A Pronator quadratus lies deep to the long flexor tendons just proximal to the wrist.
B The median nerve is related anteriorly to flexor digitorum superficialis.
C Flexor digitorum profundus is partially supplied by the posterior interosseous nerve.
D The ulnar nerve lies superficial to flexor carpi ulnaris.
E The radial nerve carries motor fibres in the anterior compartment of the forearm.

A

Let’s analyze each option to determine which best describes the forearm:

A. Pronator quadratus lies deep to the long flexor tendons just proximal to the wrist.

•	True. Pronator quadratus is located deep to the flexor digitorum superficialis and profundus tendons near the wrist.

B. The median nerve is related anteriorly to flexor digitorum superficialis.

•	False. The median nerve runs between the flexor digitorum superficialis and the flexor digitorum profundus, not anteriorly to flexor digitorum superficialis.

C. Flexor digitorum profundus is partially supplied by the posterior interosseous nerve.

•	False. Flexor digitorum profundus is innervated by the median nerve (anterior interosseous branch) for the lateral part and the ulnar nerve for the medial part.

D. The ulnar nerve lies superficial to flexor carpi ulnaris.

•	False. The ulnar nerve actually lies deep to flexor carpi ulnaris.

E. The radial nerve carries motor fibres in the anterior compartment of the forearm.

•	False. The radial nerve supplies the muscles in the posterior compartment of the forearm.

Based on the analysis, the correct answer is:

A. Pronator quadratus lies deep to the long flexor tendons just proximal to the wrist.

Explanation: Pronator quadratus is a deep muscle in the forearm located just proximal to the wrist, lying beneath the tendons of the flexor digitorum profundus and superficialis, aligning with the description in the correct option.

How well did you know this?
1
Not at all
2
3
4
5
Perfectly
80
Q

Which of the following statements ends this sentence correctly? The spinal nerve root mediating the:
A Pectoral reflex is C5.
B Deltoid reflex is C4.
C Biceps jerk is C5.
D Triceps jerk is C6.
E Supinator jerk is C7.

A

C
The biceps jerk principally tests the C5 reflex arc, with a contribution from C6.
The supinator jerk corresponds primarily to the C6 reflex arc, with a contribution from C5.
The triceps jerk is principally mediated by C7, with a contribution from C8.
This reflex may be of occasional use, and is elicited by placing the examiner’s index finger across the tip of the shoulder on the deltoid muscle belly, and tapping the finger. It solely tests the reflex arc of spinal cord segment C5.
The pectoral reflex is elicited by placing the examiner’s index and middle fingers on the lateral border of pectoralis major and tapping them with a tendon hammer. It tests the reflex arc of C7.

How well did you know this?
1
Not at all
2
3
4
5
Perfectly
81
Q

Which of the following accurately describes the anterior compartment of the forearm?
A Palmaris longus is present in approximately 15% of arms.
B The median nerve enters the forearm between the two heads of flexor carpi ulnaris.
C Flexor digitorum profundus originates from the common flexor origin.
D The superficial flexor muscles are all innervated by the median nerve
E Flexor pollicis longus is characteristically unipennate.

A

E
Flexor pollicis longus is one of the deep group of flexor muscles. Its principle origin is from the radius. It is characteristically unipennate, which aids its identification during forearm surgery.
The median nerve enters the forearm between the two heads of pronator teres. The ulnar nerve enters the forearm by passing between the two heads of flexor carpi ulnaris.
Palmaris longus is a vestigial structure, absent in 13% of arms.
Flexor digitorum profundus is one of the deep group of flexor muscles and principally takes origin from the ulna.
The superficial group of flexor muscles are pronator teres, flexor carpi radialis, palmaris longus, flexor digitorum superficialis and flexor carpi ulnaris. The median nerve innervates all these muscles except flexor carpi ulnaris, which is innervated by the ulnar nerve.

How well did you know this?
1
Not at all
2
3
4
5
Perfectly
82
Q

Which of these statements best describes the carpus?
A The pisiform is usually the first bone to begin ossification.
B The most commonly fractured bone is the lunate.
C Fracture of the hamate may result in damage to the median nerve.
D It is markedly concave from side to side anteriorly.
E The scaphoid articulates with the 1st metacarpal.

A

D
The 8 carpal bones, arranged in 2 rows of 4, form the carpus. The carpus is convex posteriorly and concave anteriorly from side to side. This shape is maintained by the shape of the bones and by the pull of the flexor retinaculum.
The most commonly fractured bone is the scaphoid.
This occurs most commonly as a result of a fall onto the palm, with the hand extended and abducted. The principal blood supply of the scaphoid enters from its distal end. A fracture may therefore interrupt the blood supply to the proximal fragment, leading to avascular necrosis and consequent degenerative change.
The scaphoid is one of the four proximal bones of the wrist and articulates with the radius proximally. The trapezium is the carpal bone which articulates with the 1st metacarpal.
The deep branch of the ulnar nerve is closely related to the hook of the hamate, and is threatened by a hamate fracture. Decreased grip strength usually ensues.
The pisiform bone is a small, pea-shaped, sesamoid bone in the tendon of the flexor carpi ulnaris. Its ossification centre usually does not appear until the age of 9-12 years. It is the last carpal bone to begin ossification.

How well did you know this?
1
Not at all
2
3
4
5
Perfectly
83
Q

The flexor tendons of the forearm run within sheaths in the hand. Which of the following statements best describes these structures?
A The flexor digitorum profundus tendon pierces the flexor digitorum superficialis tendon.
B The cruciform pullies overlie the phalanges.
C The fibrous flexor sheaths run from the metacarpal heads to the proximal phalanges.
D The annular pullies overlie the interphalangeal joints.
E The synovial sheath of the middle finger tendon is continuous with its sheath in the carpal tunnel.

A

A
The flexor digitorum superficialis tendon splits to enclose the profundus tendon as it inserts into the middle phalanx of the finger. The profundus tendon continues to insert into the base of the distal phalanx.
The fibrous flexor sheaths run from the metacarpal heads to the distal phalanges. They are occupied by the tendons of flexors digitorum superficialis and profundus in the fingers and the tendon of flexor pollicis longus in the thumb.
The annular pullies overlie the phalanges and consist of strong transverse sheath fibres.
In the index, middle and ring fingers there is a gap between the common flexor sheath in the carpal tunnel and the synovial sheaths of the fingers. The sheaths of the thumb and little finger are continuous with those of the carpal tunnel.
The cruciform pullies overlie the interphalangeal joints and consist of loose fibres which are arranged obliquely.

How well did you know this?
1
Not at all
2
3
4
5
Perfectly
84
Q

Which of the following types of lymphedema is congenital?

A-Meiges’ disease
B-Milroy’s disease
C-Lymphoedema tarda
D-Lymphoedema following surgery
E-Lymphoedema due to venous insufficiency

A

Milroy’s disease is congenital. Meiges’ develops after birth.

Lymphoedema

  • Due to impaired lymphatic drainage in the presence of normal capillary function.
    Lymphoedema causes the accumulation of protein rich fluid, subdermal fibrosis and dermal thickening.
    Characteristically fluid is confined to the epifascial space (skin and subcutaneous tissues); muscle compartments are free of oedema. It involves the foot, unlike other forms of oedema. There may be a ‘buffalo hump’ on the dorsum of the foot and the skin cannot be pinched due to subcutaneous fibrosis.

Causes of lymphoedema

Primary
Congenital < 1 year: sporadic, Milroy’s disease
Onset 1-35 years: sporadic, Meige’s disease
> 35 years: Tarda
Secondary
Bacterial/fungal/parasitic infection (filariasis)
Lymphatic malignancy
Radiotherapy to lymph nodes
Surgical resection of lymph nodes
DVT
Thrombophlebitis

Indications for surgery
Marked disability or deformity from limb swelling
Lymphoedema caused by proximal lymphatic obstruction with patent distal lymphatics suitable for a lymphatic drainage procedure
Lymphocutaneous fistulae and megalymphatics

Procedures
Homans operation Reduction procedure with preservation of overlying skin (which must be in good condition). Skin flaps are raised and the underlying tissue excised. Limb circumference typically reduced by a third.
Charles operation All skin and subcutaneous tissue around the calf are excised down to the deep fascia. Split skin grafts are placed over the site. May be performed if overlying skin is not in good condition. Larger reduction in size than with Homans procedure.
Lymphovenous anastamosis Identifiable lymphatics are anastomosed to sub dermal venules. Usually indicated in 2% of patients with proximal lymphatic obstruction and normal distal lymphatics.

How well did you know this?
1
Not at all
2
3
4
5
Perfectly
85
Q

Which of the following statements relating to pre-operative fluid management is false?

A-5% dextrose should be given cautiously in the elderly
B-Patients undergoing elective colonic resections may continue to drink water up to 2 hours prior to surgery
C-Normal saline increases the risk of hyperchloraemic acidosis
D-A 70kg man will need approximately 100mmol of sodium daily
E-Carbohydrate rich beverages and loading drinks can cause ileus therefore should be avoided

A

Carbohydrate loading is one of the enhanced recovery principles.

Pre operative fluid management

Fluid management has been described in the British Consensus guidelines on IV fluid therapy for Adult Surgical patients (GIFTASUP) and by NICE (CG174 December 2013 updated May 2017)

The Recommendations include:
Use Ringer’s lactate or Hartmann’s when a crystalloid is needed for resuscitation or replacement of fluids. Avoid 0.9% N. Saline (due to risk of hyperchloraemic acidosis) unless patient vomiting or has gastric drainage.
Use 4%/0.18% dextrose saline or 5% dextrose in maintenance fluids. It should not be used in resuscitation or as replacement fluids.
Adult maintenance fluid requirements are: Na 50-100 mmol/day and K 40-80 mmol/day in 1.5-2.5L fluid per day.
Patients for elective surgery should NOT be nil by mouth for >2 hours (unless has disorder of gastric emptying).
Patients for elective surgery should be given carbohydrate rich drinks 2-3h before. Ideally this should form part of a normal pre op plan to facilitate recovery.
Avoid mechanical bowel preparation.
If bowel prep is used, simultaneous administration of Hartmann’s or Ringer’s lactate should be considered.
Excessive fluid losses from vomiting should be treated with a crystalloid with potassium replacement. 0.9% N. Saline should be given if there is hypochloraemia. Otherwise Hartmann’s or Ringer lactate should be given for diarrhoea/ileostomy/ileus/obstruction. Hartmann’s should also be given in sodium losses secondary to diuretics.
High risk patients should receive fluids and inotropes.
An attempt should be made to detect pre or operative hypovolaemia using flow based measurements. If this is not available, then clinical evaluation is needed i.e. JVP, pulse volume etc.
In Blood loss or infection causing hypovolaemia should be treated with a balanced crystalloid or colloid (or until blood available in blood loss). A critically ill patient is unable to excrete Na or H20 leading to a 5% risk of interstitial oedema. Therefore 5% dextrose as well as colloid should be given.
If patients need IV fluid resuscitation, use crystalloids that contain sodium in the range 130-154 mmol/l, with a bolus of 500 ml over less than 15 minutes (NICE Guidance CG 174).

How well did you know this?
1
Not at all
2
3
4
5
Perfectly
86
Q

Where does the spinal cord terminate in neonates?

A-L1
B-L2
C-L3
D-L4
E-L5

A

At the 3rd month the foetus’s spinal cord occupies the entire length of the vertebral canal. The vertebral column then grows longer exceeding the growth rate of the spinal cord. This results with the cord being at L3 at birth and L1-2 by adulthood.

Spinal cord

  • Located in a canal within the vertebral column that affords it structural support.
    Rostrally it continues to the medulla oblongata of the brain and caudally it tapers at a level corresponding to the L1-2 interspace (in the adult), a central structure, the filum terminale anchors the cord to the first coccygeal vertebra.
    The spinal cord is characterised by cervico-lumbar enlargements and these, broadly speaking, are the sites which correspond to the brachial and lumbar plexuses respectively.

There are some key points to note when considering the surgical anatomy of the spinal cord:

  • During foetal growth the spinal cord becomes shorter than the spinal canal, hence the adult site of cord termination at the L1-2 level.
  • Due to growth of the vertebral column the spine segmental levels may not always correspond to bony landmarks as they do in the cervical spine.
  • The spinal cord is incompletely divided into two symmetrical halves by a dorsal median sulcus and ventral median fissure. Grey matter surrounds a central canal that is continuous rostrally with the ventricular system of the CNS.
  • The grey matter is sub divided cytoarchitecturally into Rexeds laminae.
  • Afferent fibres entering through the dorsal roots usually terminate near their point of entry but may travel for varying distances in Lissauers tract. In this way they may establish synaptic connections over several levels
  • At the tip of the dorsal horn are afferents associated with nociceptive stimuli. The ventral horn contains neurones that innervate skeletal muscle.

The key point to remember when revising CNS anatomy is to keep a clinical perspective in mind. So it is worth classifying the ways in which the spinal cord may become injured. These include:

Trauma either direct or as a result of disc protrusion
Neoplasia either by direct invasion (rare) or as a result of pathological vertebral fracture
Inflammatory diseases such as Rheumatoid disease, or OA (formation of osteophytes compressing nerve roots etc.
Vascular either as a result of stroke (rare in cord) or as complication of aortic dissection
Infection historically diseases such as TB, epidural abscesses.

The anatomy of the cord will, to an extent dictate the clinical presentation. Some points/ conditions to remember:

Brown- Sequard syndrome-Hemisection of the cord producing ipsilateral loss of proprioception and upper motor neurone signs, plus contralateral loss of pain and temperature sensation. The explanation of this is that the fibres decussate at different levels.
Lesions below L1 will tend to present with lower motor neurone signs

How well did you know this?
1
Not at all
2
3
4
5
Perfectly
87
Q

A 25 year old junior doctor has a chest x-ray performed as part of a routine insurance medical examination. The x-ray shows evidence of rib notching. Auscultation of his chest reveals a systolic murmur which is loudest at the posterior aspect of the fourth intercostal space. What is the most likely diagnosis?

A-Patent ductus arteriosus
B-Aortic coarctation
C-Aortic dissection
D-Cervical rib
E-Subclavian steal syndrome

A

Coarctation of the aorta may occur due to the remnant of the ductus arteriosus acting as a fibrous constrictive band of the aorta. Weak arm pulses may be seen, radiofemoral delay is the classical physical finding. Collateral flow through the intercostal vessels may produce notching of the ribs, if the disease is long standing.

Vascular disease

Aortic dissection
Chest pain (anterior chest pain- ascending aorta, back pain - descending aorta)
Widening of aorta on chest x-ray
Diagnosis made by CT scanning
Treatment is either medical (Type B disease) or surgical (Type A disease)
Cervical rib
Supernumery fibrous band arising from seventh cervical vertebra
Incidence of 1 in 500
May cause thoracic outlet syndrome
Treatment involves surgical division of rib
Subclavian steal syndrome
Due to proximal stenotic lesion of the subclavian artery
Results in retrograte flow through vertebral or internal thoracic arteries
The result is that decrease in cerebral blood flow may occur and produce syncopal symptoms
A duplex scan and/ or angiogram will delineate the lesion and allow treatment to be planned
Takayasu’s arteritis
Large vessel granulomatous vasculitis
Results in intimal narrowing
Most commonly affects young asian females
Patients present with features of mild systemic illness, followed by pulseless phase with symptoms of vascular insufficiency
Treatment is with systemic steroids
Patent ductus arteriosus
Ductus arteriosus is a normal foetal vessel that closes spontaneously after birth
Results in high pressure, oxygenated blood entering the pulmonary circuit
Untreated patients develop symptoms of congestive cardiac failure
Coarctation of the aorta
Aortic stenosis at the site of the ductus arteriosus insertion
More prevalent in boys or females with Turners syndrome
Patients may present with symptoms of arterial insufficiency, such as syncope and claudication
Blood pressure mismatch may be seen, as may mismatch of pulse pressure in the upper and lower limbs
Treatment is either with angioplasty or surgical resection (the former is the most common)

How well did you know this?
1
Not at all
2
3
4
5
Perfectly
88
Q

Which of the following structures does not pass anterior to the lateral malleolus?

A-Anterior tibial artery
B-Extensor digitorum longus
C-Lateral branch of the superficial peroneal nerve
D-Peroneus brevis
E-Peroneus tertius

A

Peroneus brevis passes posterior to the lateral malleolus. Peroneus tertius is closely related to extensor digitorum and like that muscle, its tendon passes anterior to the lateral malleolus

Lateral malleolus

Structures posterior to the lateral malleolus and superficial to superior peroneal retinaculum
Sural nerve
Short saphenous vein

Structures posterior to the lateral malleolus and deep to superior peroneal retinaculum
Peroneus longus tendon
Peroneus brevis tendon

The calcaneofibular ligament is attached at the lateral malleolus

How well did you know this?
1
Not at all
2
3
4
5
Perfectly
89
Q

Which of the following drugs is not positively inotropic?

A-Dopamine
B-Glucagon
C-Theophylline
D-Sodium thiopentone
E-Dobutamine

A

Correct Answer:

D. Sodium thiopentone

Explanation:

Positive inotropic drugs increase the strength of cardiac muscle contraction, which improves cardiac output. Here’s a brief explanation of each drug:

•	Dopamine: A catecholamine that increases heart contractility and cardiac output by stimulating beta-1 adrenergic receptors.
•	Glucagon: Increases heart contractility by stimulating adenylate cyclase and increasing cyclic AMP, independent of the adrenergic receptors.
•	Theophylline: A methylxanthine that can increase heart contractility through its action as a phosphodiesterase inhibitor, leading to increased cyclic AMP levels.
•	Sodium thiopentone (Thiopental): A barbiturate used for induction of anesthesia. It has a depressive effect on the central nervous system and does not have positive inotropic effects. In fact, it can cause myocardial depression and reduce cardiac output.
•	Dobutamine: A synthetic catecholamine that acts on beta-1 adrenergic receptors, increasing heart contractility and cardiac output.

Therefore, Sodium thiopentone is the drug that is not positively inotropic.

Inotropes are a class of drugs that increase the force or cardiac contractility. This may improve cardiac output. Increased blood pressure may have direct beneficial effects for the heart in that it improves myocardial perfusion pressure. Dopamine and dobutamine are both commonly used inotropes, they should be administered via a central line and in a monitored setting. Glucagon and theophylline are also positive inotropes (although not commonly used for this purpose). In contrast sodium thiopentone causes marked myocardial depression.

Inotropes and cardiovascular receptors

Inotropes are a class of drugs which work primarily by increasing cardiac output. They should be distinguished from vasoconstrictor drugs which are used specifically when the primary problem is peripheral vasodilatation.

Catecholamine type agents are commonly used and work by increasing cAMP levels by adenylate cyclase stimulation. This in turn increases intracellular calcium ion mobilisation and thus the force of contraction. Adrenaline works as a beta adrenergic receptor agonist at lower doses and an alpha receptor agonist at higher doses. Dopamine causes dopamine receptor mediated renal and mesenteric vascular dilatation and beta 1 receptor agonism at higher doses. This results in increased cardiac output. Since both heart rate and blood pressure are raised, there is less overall myocardial ischaemia. Dobutamine is a predominantly beta 1 receptor agonist with weak beta 2 and alpha receptor agonist properties. Noradrenaline is a catecholamine type agent and predominantly acts as an alpha receptor agonist and serves as a peripheral vasoconstrictor.

Phosphodiesterase inhibitors such as milrinone act specifically on the cardiac phosphodiesterase and increase cardiac output.

Inotrope Cardiovascular receptor action
Adrenaline α-1, α-2, β-1, β-2
Noradrenaline α-1,( α-2), (β-1), (β-2)
Dobutamine β-1, (β 2)
Dopamine (α-1), (α-2), (β-1), D-1,D-2
Minor receptor effects in brackets

Effects of receptor binding
α-1, α-2 vasoconstriction
β-1 increased cardiac contractility and HR
β-2 vasodilatation
D-1 renal and spleen vasodilatation
D-2 inhibits release of noradrenaline

How well did you know this?
1
Not at all
2
3
4
5
Perfectly
90
Q

A 28 year old man falls onto an outstretched hand. On examination, there is tenderness of the anatomical snuffbox. However, forearm and hand x-rays are normal. What is the most appropriate course of action?

Discharge with reassurance

Place in arm sling and discharge

Place in futura splint and review in fracture clinic

Admit for surgical exploration

Apply an external fixation device

A

Correct Answer:

3.	Place in futura splint and review in fracture clinic

Explanation:

Tenderness in the anatomical snuffbox following a fall onto an outstretched hand is highly suggestive of a scaphoid fracture, even if initial X-rays are normal. Scaphoid fractures may not always be visible on initial X-rays and can lead to complications such as nonunion or avascular necrosis if not properly managed.

Appropriate management includes:

•	Immobilization: Placing the wrist in a splint, such as a futura splint, to immobilize the area and prevent further injury.
•	Follow-up: Arranging for a review in the fracture clinic where further imaging (e.g., repeat X-rays, MRI, or CT) can be performed if needed to confirm the diagnosis.

Options not chosen:

•	Discharge with reassurance: Inappropriate due to the high suspicion of a scaphoid fracture.
•	Place in arm sling and discharge: Insufficient as it does not adequately immobilize the wrist.
•	Admit for surgical exploration: Not indicated at this stage without a confirmed fracture.
•	Apply an external fixation device: Unnecessary for a suspected scaphoid fracture without a confirmed diagnosis.

Therefore, the best course of action is to place the wrist in a futura splint and arrange for a review in the fracture clinic.

This could well be a scaphoid fracture and should be temporarily immobilised pending further review. A futura splint will immobilise better than an arm sling for this problem. Fracture management

Bony injury resulting in a fracture may arise from trauma (excessive forces applied to bone), stress related (repetitive low velocity injury) or pathological (abnormal bone which fractures during normal use of following minimal trauma)
Diagnosis involves not just evaluating the fracture ; such as site and type of injury but also other associated injuries and distal neurovascular deficits. This may entail not just clinical examination but radiographs of proximal and distal joints.
When assessing x-rays it is important to assess for changes in length of the bone, the angulation of the distal bone, rotational effects, presence of material such as glass.

Fracture types
Fracture type Description
Oblique fracture Fracture lies obliquely to long axis of bone
Comminuted fracture >2 fragments
Segmental fracture More than one fracture along a bone
Transverse fracture Perpendicular to long axis of bone
Spiral fracture Severe oblique fracture with rotation along long axis of bone

Open Vs Closed
It is also important to distinguish open from closed injuries. The most common classification system for open fractures is the Gustilo and Anderson classification system (given below):

Grade Injury
1 Low energy wound <1cm
2 Greater than 1cm wound with moderate soft tissue damage
3 High energy wound > 10cm with extensive soft tissue damage
3 A (sub group of 3) Adequate soft tissue coverage
3 B (sub group of 3) Inadequate soft tissue coverage
3 C (sub group of 3) Associated arterial injury

Key points in management of fractures
Immobilise the fracture including the proximal and distal joints
Carefully monitor and document neurovascular status, particularly following reduction and immobilisation
Manage infection including tetanus prophylaxis
IV broad spectrum antibiotics for open injuries
As a general principle all open fractures should be thoroughly debrided ( and internal fixation devices avoided or used with extreme caution)
Open fractures constitute an emergency and should be debrided and lavaged within 6 hours of injury

How well did you know this?
1
Not at all
2
3
4
5
Perfectly
91
Q

A 56 year old man presents with episodic facial pain and discomfort whilst eating. He has suffered from halitosis recently and he frequently complains of a dry mouth. He has a smooth swelling underneath his right mandible. What is the most likely underlying diagnosis?

Stone impacted in Whartons duct

Stone impacted in Stensens duct

Benign adenoma of the submandibular gland

Adenocarcinoma of the submandibular gland

Squamous cell carcinoma of the submandibular gland

A

The symptoms are typical for sialolithiasis. The stones most commonly form in the submandibular gland and therefore may occlude Whartons duct. Stensens duct drains the parotid gland. Submandibular glands- disease

Physiology
The submandibular glands secrete approximately 800- 1000ml saliva per day. They typically produce mixed seromucinous secretions. When parasympathetic activity is dominant; the secretions will be more serous. The parasympathetic fibres are derived from the chorda tympani nerves and the submandibular ganglion. Sensory fibres are conveyed by the lingual branch of the mandibular nerve.

Sialolithiasis
80% of all salivary gland calculi occur in the submandibular gland
70% of the these calculi are radio-opaque
Stones are usually composed of calcium phosphate or calcium carbonate
Patients typically develop colicky pain and post prandial swelling of the gland
Investigation involves sialography to demonstrate the site of obstruction and associated other stones
Stones impacted in the distal aspect of Whartons duct may be removed orally, other stones and chronic inflammation will usually require gland excision

Sialadenitis
Usually occurs as a result of Staphylococcus aureus infection
Pus may be seen leaking from the duct, erythema may also be noted
Development of a sub mandibular abscess is a serious complication as it may spread through the other deep fascial spaces and occlude the airway

Submandibular tumours
Only 8% of salivary gland tumours affect the sub mandibular gland
Of these 50% are malignant (usually adenoid cystic carcinoma)
Diagnosis usually involves fine needle aspiration cytology
Imaging is with CT and MRI
In view of the high prevalence of malignancy, all masses of the submandibular glands should generally be excised.

How well did you know this?
1
Not at all
2
3
4
5
Perfectly
92
Q

A 20 year old man is admitted with bloody diarrhoea. He has been passing 10 stools per day, Hb-8.0, albumin-20. Stool culture negative. Evidence of colitis on endoscopy. He has been on intravenous steroids for 5 days and has now developed megacolon. His haemoglobin is falling and inflammatory markers are static. What is the most appropriate course of action?

Double the steroid dose

Undertake a loop ileostomy

Undertake a sub total colectomy and end ileostomy

Undertake a sub total colectomy and ileo-rectal anastomosis

Undertake a pan proctocolectomy

A

Given the clinical scenario of a 20-year-old man with bloody diarrhea, a failing response to intravenous steroids, and development of megacolon, the most appropriate course of action is a surgical intervention. The options of further medical management, such as doubling the steroid dose, are unlikely to be effective at this stage due to the risk of perforation and other complications associated with toxic megacolon.

The most appropriate course of action is:

Undertake a sub total colectomy and end ileostomy

Explanation:

1.	Development of Megacolon: The patient has developed a toxic megacolon, which is a severe complication of colitis characterized by extreme dilatation of the colon and potential for perforation. This condition requires urgent surgical intervention.
2.	Failure of Medical Management: The patient has been on intravenous steroids for 5 days without improvement, indicating that medical management has failed.
3.	Rising Complications: With falling hemoglobin levels and static inflammatory markers, the patient’s condition is worsening, necessitating immediate surgery.

A subtotal colectomy with an end ileostomy is typically recommended as it removes the diseased colon and diverts fecal stream, reducing the risk of perforation and sepsis, and stabilizing the patient.

Pan proctocolectomy should not be performed in acute unwell patients
Loop ileostomy is not conventional management of UC
This man requires a sub total colectomy. Conservative management has failed. Patients with ulcerative colitis should undergo colectomy if there is no significant improvement in 5-7 days after initiating medical therapy if they have a severe attack of the disease. To undertake a pan proctocolectomy in an acutely unwell patient is very high risk and most unwise. Management of the rectum is generally left until the patient is more stable.

IBD

Ulcerative colitis Vs Crohns

Crohn’s disease Ulcerative colitis
Distribution Mouth to anus Rectum and colon
Macroscopic changes Cobblestone appearance, apthoid ulceration Contact bleeding
Depth of disease Transmural inflammation Superficial inflammation
Distribution pattern Patchy Continuous
Histological features Granulomas (non caseating epithelioid cell aggregates with Langerhans’ giant cells) Crypt abscesses, Inflammatory cells in the lamina propria

Surgical treatment

Ulcerative colitis
In UC the main place for surgery is when medical treatment has failed, in the emergency setting this will be a sub total colectomy, end ileostomy and a mucous fistula. Electively it will be a pan proctocolectomy, an ileoanal pouch may be a selected option for some. Remember that longstanding UC increases colorectal cancer risk.

Crohn’s disease
Unlike UC Crohn’s patients need to avoid surgeons, minimal resections are the rule. They should not have ileoanal pouches as they will do poorly with them. Management of Crohn’s ano rectal sepsis is with a minimal approach, simply drain sepsis and use setons to facilitate drainage. Definitive fistula surgery should be avoided.

How well did you know this?
1
Not at all
2
3
4
5
Perfectly
93
Q

A 16 year old boy is hit by a car and sustains a blow to the right side of his head. He is initially conscious but on arrival in the emergency department is comatose. On examination his right pupil is fixed and dilated. The neurosurgeons plan immediate surgery. What type of initial approach should be made?

Left parieto-temporal craniotomy

Right parieto-temporal craniotomy

Posterior fossa craniotomy

Left parieto-temporal burr holes

None of the above

A

A unilateral dilated pupil is a classic sign of transtentorial herniation. The medial aspect of the temporal lobe (uncus) herniates across the tentorium and causes pressure on the ipsilateral oculomotor nerve, interrupting parasympathetic input to the eye and resulting in a dilated pupil. In addition the brainstem is compressed. As the ipsilateral oculomotor nerve is being compressed, craniotomy (rather than Burr Holes) should be made on the ipsilateral side.

Head injury

Patients who suffer head injuries should be managed according to ATLS principles and extra cranial injuries should be managed alongside cranial trauma. Inadequate cardiac output will compromise CNS perfusion irrespective of the nature of the cranial injury.

Types of traumatic brain injury
Extradural haematoma Bleeding into the space between the dura mater and the skull. Often results from acceleration-deceleration trauma or a blow to the side of the head. The majority of extradural haematomas occur in the temporal region where skull fractures cause a rupture of the middle meningeal artery.

Features
Raised intracranial pressure
Some patients may exhibit a lucid interval
Subdural haematoma Bleeding into the outermost meningeal layer. Most commonly occur around the frontal and parietal lobes. May be either acute or chronic.

Risk factors include old age and alcoholism.

Slower onset of symptoms than a extradural haematoma.
Subarachnoid haemorrhage Usually occurs spontaneously in the context of a ruptured cerebral aneurysm, but may be seen in association with other injuries when a patient has sustained a traumatic brain injury.

Pathophysiology
Primary brain injury may be focal (contusion/ haematoma) or diffuse (diffuse axonal injury)
Diffuse axonal injury occurs as a result of mechanical shearing following deceleration, causing disruption and tearing of axons
Intra-cranial haematomas can be extradural, subdural or intracerebral, while contusions may occur adjacent to (coup) or contralateral (contre-coup) to the side of impact
Secondary brain injury occurs when cerebral oedema, ischaemia, infection, tonsillar or tentorial herniation exacerbates the original injury. The normal cerebral auto regulatory processes are disrupted following trauma rendering the brain more susceptible to blood flow changes and hypoxia
The Cushings reflex (hypertension and bradycardia) often occurs late and is usually a pre terminal event

Management
Where there is life threatening rising ICP such as in extra dural haematoma and whilst theatre is prepared or transfer arranged use of IV mannitol/ frusemide may be required.
Diffuse cerebral oedema may require decompressive craniotomy
Exploratory Burr Holes have little management in modern practice except where scanning may be unavailable and to thus facilitate creation of formal craniotomy flap
Depressed skull fractures that are open require formal surgical reduction and debridement, closed injuries may be managed non operatively if there is minimal displacement.
ICP monitoring is appropriate in those who have GCS 3-8 and normal CT scan.
ICP monitoring is mandatory in those who have GCS 3-8 and abnormal CT scan.
Hyponatraemia is most likely to be due to syndrome of inappropriate ADH secretion.
Minimum of cerebral perfusion pressure of 70mmHg in adults.
Minimum cerebral perfusion pressure of between 40 and 70 mmHg in children.

Interpretation of pupillary findings in head injuries
Pupil size Light response Interpretation
Unilaterally dilated Sluggish or fixed 3rd nerve compression secondary to tentorial herniation
Bilaterally dilated Sluggish or fixed
Poor CNS perfusion
Bilateral 3rd nerve palsy
Unilaterally dilated or equal Cross reactive (Marcus - Gunn) Optic nerve injury
Bilaterally constricted May be difficult to assess
Opiates
Pontine lesions
Metabolic encephalopathy
Unilaterally constricted Preserved Sympathetic pathway disruption

How well did you know this?
1
Not at all
2
3
4
5
Perfectly
94
Q

Which of the following pathological explanations best describes the initial pathological processes occurring in an abdominal aortic aneurysm in an otherwise well 65 year old, hypertensive male?

Loss of elastic fibres from the adventitia

Loss of collagen from the adventitia

Loss of collagen from the media

Loss of elastic fibres from the media

Decreased matrix metalloproteinases in the adventitia

A

D In established aneurysmal disease there is dilation of all layers of the arterial wall and loss of both elastin and collagen. The primary event is loss of elastic fibres with subsequent degradation of collagen fibres. Pathology of abdominal aortic aneurysm

Abdominal aortic aneurysms occur primarily as a result of the failure of elastic proteins within the extracellular matrix. Aneurysms typically represent dilation of all layers of the arterial wall. Most aneurysms are caused by degenerative disease. After the age of 50 years the normal diameter of the infrarenal aorta is 1.5cm in females and 1.7cm in males. Diameters of 3cm and greater, are considered aneurysmal. The pathophysiology involved in the development of aneurysms is complex and the primary event is loss of the intima with loss of elastic fibres from the media. This process is associated with, and potentiated by, increased proteolytic activity and lymphocytic infiltration.

Major risk factors for the development of aneurysms include smoking and hypertension. Rare but important causes include syphilis and connective tissues diseases such as Ehlers Danlos type 1 and Marfans syndrome.

How well did you know this?
1
Not at all
2
3
4
5
Perfectly
95
Q

A 45 year old woman with breast cancer is started on a chemotherapy regime containing epirubicin. What is the primary mode of action of this drug?

Intercalation of DNA

Antimetabolite

Monoclonal antibody to epidermal growth factor

Inhibition of DNA gyrase

Inhibition of topoisomerase 1

A

Epirubicin is an anthracycline chemotherapeutic agent primarily used in the treatment of various cancers, including breast cancer.

The primary mode of action of epirubicin is:

Intercalation of DNA

Explanation:

1.	Intercalation of DNA: Epirubicin works by intercalating into DNA, which means it inserts itself between DNA base pairs. This intercalation disrupts the structure of the DNA helix, inhibiting DNA replication and transcription. Additionally, it stabilizes the topoisomerase II-DNA complex, preventing the religation of the DNA strands and leading to apoptosis of the cancer cells.
2.	Antimetabolite: Antimetabolites are drugs that interfere with the normal metabolism of cells, usually by mimicking the normal substrates of key enzymes, but this is not the primary action of epirubicin.
3.	Monoclonal antibody to epidermal growth factor: Monoclonal antibodies target specific proteins, such as the epidermal growth factor receptor (EGFR), but epirubicin is not a monoclonal antibody.
4.	Inhibition of DNA gyrase: DNA gyrase is an enzyme targeted by some antibiotics, particularly in bacteria, but it is not the target of epirubicin.
5.	Inhibition of topoisomerase 1: Epirubicin primarily inhibits topoisomerase II, not topoisomerase I.

Therefore, epirubicin’s primary mode of action is intercalation of DNA.

How well did you know this?
1
Not at all
2
3
4
5
Perfectly
96
Q

A 53 year old man is undergoing a radical gastrectomy for carcinoma of the stomach. Which of these structures will need to be divided to gain access to the coeliac axis?

Lesser omentum

Greater omentum

Falciform ligament

Median arcuate ligament

Gastrosplenic ligament

A

The lesser omentum will need to be divided. During a radical gastrectomy this forms one of the nodal stations that will need to be taken. Coeliac axis

The coeliac axis has three main branches.
Left gastric
Hepatic: branches-Right Gastric, Gastroduodenal, Superior Pancreaticoduodenal, Cystic (occasionally).
Splenic: branches- Pancreatic, Short Gastric, Left Gastroepiploic

It occasionally gives off one of the inferior phrenic arteries.

Relations
Anteriorly Lesser omentum
Right Right coeliac ganglion and caudate process of liver
Left Left coeliac ganglion and gastric cardia
Inferiorly Upper border of pancreas and renal vein

How well did you know this?
1
Not at all
2
3
4
5
Perfectly
97
Q

A 22 year old man is involved in a fight and sustains a skull fracture with an injury to the middle meningeal artery. A craniotomy is performed, and with considerable difficulty the haemorrhage from the middle meningeal artery is controlled by ligating it close to its origin. What is the most likely sensory impairment that the patient may notice post operatively?

Parasthesia of the ipsilateral external ear

Loss of taste sensation from the anterior two thirds of the tongue

Parasthesia overlying the angle of the jaw

Loss of sensation from the ipsilateral side of the tongue

Loss of taste from the posterior two thirds of the tongue

A

The most likely sensory impairment that the patient may notice post-operatively is:

Parasthesia overlying the angle of the jaw

Explanation:
The middle meningeal artery is a branch of the maxillary artery and supplies the meninges of the brain. During a craniotomy to control hemorrhage from the middle meningeal artery, nearby structures, including the auriculotemporal nerve, can be affected. The auriculotemporal nerve is a branch of the mandibular nerve (V3) and provides sensory innervation to the skin over the temple and the anterior portion of the ear, as well as the temporomandibular joint (TMJ).

Given this anatomical relationship, damage or manipulation of the auriculotemporal nerve during the surgery could lead to parasthesia (abnormal sensation) overlying the angle of the jaw. This is because the auriculotemporal nerve innervates that region.

Other options do not align as closely with the anatomical course and innervation area of the middle meningeal artery and its surrounding structures:

•	Parasthesia of the ipsilateral external ear: This would more likely involve the great auricular nerve or lesser occipital nerve.
•	Loss of taste sensation from the anterior two-thirds of the tongue: This involves the chorda tympani branch of the facial nerve (VII).
•	Loss of sensation from the ipsilateral side of the tongue: This involves the lingual nerve, a branch of the mandibular nerve (V3).
•	Loss of taste from the posterior two-thirds of the tongue: This involves the glossopharyngeal nerve (IX).
How well did you know this?
1
Not at all
2
3
4
5
Perfectly
98
Q

As regards the internal jugular vein, which of the following statements is untrue?

It lies within the carotid sheath

It is the continuation of the sigmoid sinus

The terminal part of the thoracic duct crosses anterior to it to insert into the right subclavian vein

The hypoglossal nerve is closely related to it as it passes near the atlas

The vagus nerve is closely related to it within the carotid sheath

A

The terminal part of the thoracic duct inserts into the left subclavian vein and it passes posterior to the jugular vein.

Internal jugular vein

Each jugular vein begins in the jugular foramen, where they are the continuation of the sigmoid sinus. They terminate at the medial end of the clavicle where they unite with the subclavian vein.

The vein lies within the carotid sheath throughout its course. Below the skull the internal carotid artery and last four cranial nerves are anteromedial to the vein. Thereafter it is in contact medially with the internal (then common) carotid artery. The vagus lies posteromedially.

At its superior aspect, the vein is overlapped by sternocleidomastoid and covered by it at the inferior aspect of the vein.

Below the transverse process of the atlas it is crossed on its lateral side by the accessory nerve. At its mid point it is crossed by the inferior root of the ansa cervicalis.
Posterior to the vein are the transverse processes of the cervical vertebrae, the phenic nerve as it descends on the scalenus anterior, and the first part of the subclavian artery.

On the left side its also related to the thoracic duct.

How well did you know this?
1
Not at all
2
3
4
5
Perfectly
99
Q

A 43 year old lady undergoes a live donor related renal transplant. Over the next few years it is noted that her renal function progressively deteriorates. What is the most likely underlying explanation?

Type I hypersensitivity reaction

Type III hypersensitivity reaction

Type II hypersensitivity reaction

Type IV hypersensitivity reaction

None of the above

A

Chronic rejection of renal transplants is mediated via T lymphocytes and is therefore a type IV hypersensitivity reaction. This process can be mitigated by immunosupression. Organ Transplant

A number of different organ and tissue transplants are now available. In many cases an allograft is performed, where an organ is transplanted from one individual to another. Allografts will elicit an immune response and this is one of the main reasons for organ rejection.

Graft rejection occurs because allografts have allelic differences at genes that code immunohistocompatability complex genes. The main antigens that give rise to rejection are:
ABO blood group
Human leucocyte antigens (HLA)
Minor histocompatability antigens

ABO Matching
ABO incompatibility will result in early organ rejection (hyperacute) because of pre existing antibodies to other groups. Group O donors can give organs to any type of ABO recipient whereas group AB donor can only donate to AB recipient.

HLA System
The four most important HLA alleles are:

HLA A
HLA B
HLA C
HLA DR

An ideal organ match would be one in which all 8 alleles are matched (remember 2 from each parent, four each = 8 alleles). Modern immunosuppressive regimes help to manage the potential rejection due to HLA mismatching. However, the greater the number of mismatches the worse the long term outcome will be. T lymphocytes will recognise antigens bound to HLA molecules and will then become activated. Clonal expansion then occurs with a response directed against that antigen.

Types of organ rejection
Hyperacute. This occurs immediately through presence of pre formed antibodies (such as ABO incompatibility).
Acute. Occurs during the first 6 months and is usually T cell mediated. Usually tissue infiltrates and vascular lesions.
Chronic. Occurs after the first 6 months. Vascular changes predominate.

Hyperacute
Renal transplants at greatest risk and liver transplants at least risk. Although ABO incompatibility and HLA Class I incompatible transplants will all fare worse in long term.

Acute
All organs may undergo acute rejection. Mononuclear cell infiltrates predominate. All types of transplanted organ are susceptible and it may occur in up to 50% cases.

Chronic
Again all transplants with HLA mismatch may suffer this fate. Previous acute rejections and other immunosensitising events all increase the risk. Vascular changes are most prominent with myointimal proliferation leading to organ ischaemia. Organ specific changes are also seen such as loss of acinar cells in pancreas transplants and rapidly progressive coronary artery disease in cardiac transplants.

Surgical overview-Renal transplantation
A brief overview of the steps involved in renal transplantation is given.
Patients with end stage renal failure who are dialysis dependent or likely to become so in the immediate future are considered for transplant. Exclusion criteria include; active malignancy, old age (due to limited organ availability). Patients are medically optimised.
Donor kidneys, these may be taken from live related donors and close family, members may have less HLA mismatch than members of the general population. Laparoscopic donor nephrectomy further minimises the operative morbidity for the donor. Other organs are typically taken from brain dead or dying patients who have a cardiac arrest and in whom resuscitation is futile. The key event is to minimise the warm ischaemic time in the donor phase.

The kidney once removed is usually prepared on the bench in theatre by the transplant surgeon immediately prior to implantation and factors such as accessory renal arteries and vessel length are assessed and managed.

For first time recipients the operation is performed under general anaesthesia. A Rutherford-Morison incision is made on the preferred side. This provides excellent extraperitoneal access to the iliac vessels. The external iliac artery and vein are dissected out and following systemic heparinisation are cross clamped. The vein and artery are anastamosed to the iliacs and the clamps removed. The ureter is then implanted into the bladder and a stent is usually placed to maintain patency. The wounds are then closed and the patient recovered from surgery.

In the immediate phase a common problem encountered in cadaveric kidneys is acute tubular necrosis and this tends to resolve.

Graft survival times from cadaveric donors are typically of the order of 9 years and monozygotic twin transplant (live donor) may survive as long as 25 years.

How well did you know this?
1
Not at all
2
3
4
5
Perfectly
100
Q

A 10 year old child is admitted to the emergency department after a fall. On examination, the blood pressure is 100/55mmHg, pulse rate 90, abdomen soft but tender on the left. Abdominal imaging demonstrates a grade III splenic laceration. What is the most appropriate course of action?

Undertake an immediate laparotomy and splenectomy

Undertake a laparoscopy and laparoscopic splenectomy

Admit the child to the high dependency unit for close monitoring

Arrange splenic artery embolisation

Undertake a laparotomy and splenic repair

A

Splenic trauma is nearly always managed conservatively. Hilar injuries (grade IV) are less amenable to this and will tend to come to surgery.

The most appropriate course of action for a 10-year-old child with a grade III splenic laceration and stable vital signs is:

Admit the child to the high dependency unit for close monitoring

Explanation:
In pediatric patients, the management of splenic injuries often favors non-operative treatment, especially in the context of stable vital signs. Grade III splenic lacerations, while significant, can often be managed conservatively if the patient remains hemodynamically stable. Close monitoring in a high dependency unit (HDU) is appropriate to ensure any changes in the child’s condition are promptly addressed. This approach minimizes the risks associated with surgery and preserves splenic function, which is important for immunological reasons.

•	Immediate laparotomy and splenectomy: This is typically reserved for patients who are hemodynamically unstable or have ongoing significant bleeding that cannot be controlled non-operatively.
•	Laparoscopy and laparoscopic splenectomy: Similar to open splenectomy, this is more invasive and generally not the first choice in hemodynamically stable pediatric patients.
•	Splenic artery embolisation: This is usually considered for patients with ongoing bleeding but can often be avoided in stable children.
•	Laparotomy and splenic repair: This is an option if non-operative management fails or if there is significant concern for complications that cannot be managed non-operatively. However, in a stable child, initial non-operative management is preferred.

Therefore, admitting the child to the HDU for close monitoring is the most appropriate and conservative approach, given the current stability of the patient.

Trauma management

The cornerstone of trauma management is embodied in the principles of ATLS.

Following trauma there is a trimodal death distribution:
Immediately following injury. Typically as result of brain or high spinal injuries, cardiac or great vessel damage. Salvage rate is low.
In early hours following injury. In this group deaths are due to phenomena such as splenic rupture, sub dural haematomas and haemopneumothoraces
In the days following injury. Usually due to sepsis or multi organ failure.

Aspects of trauma management
ABCDE approach.
Tension pneumothoraces will deteriorate with vigorous ventilation attempts.
External haemorrhage is managed as part of the primary survey. As a rule tourniquets should not be used. Blind application of clamps will tend to damage surrounding structures and packing is the preferred method of haemorrhage control.
Urinary catheters and naso gastric tubes may need inserting. Be wary of basal skull fractures and urethral injuries.
Patients with head and neck trauma should be assumed to have a cervical spine injury until proven otherwise.

Thoracic injuries
Simple pneumothorax
Mediastinal traversing wounds
Tracheobronchial tree injury
Haemothorax
Blunt cardiac injury
Diaphragmatic injury
Aortic disruption
Pulmonary contusion

Management of thoracic trauma
Simple pneumothorax insert chest drain. Aspiration is risky in trauma as pneumothorax may be from lung laceration and convert to tension pneumothorax.
Mediastinal traversing wounds These result from situations like stabbings. Exit and entry wounds in separate hemithoraces. The presence of a mediastinal haematoma indicates the likelihood of a great vessel injury. All patients should undergo CT angiogram and oesophageal contrast swallow. Indications for thoracotomy are largely related to blood loss and will be addressed below.
Tracheobronchial tree injury Unusual injuries. In blunt trauma most injuries occur within 4cm of the carina. Features suggesting this injury include haemoptysis and surgical emphysema. These injuries have a very large air leak and may have tension pneumothorax.
Haemothorax Usually caused by laceration of lung vessel or internal mammary artery by rib fracture. Patients should all have a wide bore 36F chest drain. Indications for thoracotomy include loss of more than 1.5L blood initially or ongoing losses of >200ml per hour for >2 hours.
Cardiac contusions Usually cardiac arrhythmias, often overlying sternal fracture. Perform echocardiography to exclude pericardial effusions and tamponade. Risk of arrhythmias falls after 24 hours.
Diaphragmatic injury Usually left sided. Direct surgical repair is performed.
Traumatic aortic disruption Commonest cause of death after RTA or falls. Usually incomplete laceration near ligamentum arteriosum. All survivors will have contained haematoma. Only 1-2% of patients with this injury will have a normal chest x-ray.
Pulmonary contusion Common and lethal. Insidious onset. Early intubation and ventilation.

Abdominal trauma
Deceleration injuries are common.
In blunt trauma requiring laparotomy the spleen is most commonly injured (40%)
Stab wounds traverse structures most commonly liver (40%)
Gunshot wounds have variable effects depending upon bullet type. Small bowel is most commonly injured (50%)
Patients with stab wounds and no peritoneal signs up to 25% will not enter the peritoneal cavity
Blood at urethral meatus suggests a urethral tear
High riding prostate on PR = urethral disruption
Mechanical testing for pelvic stability should only be performed once

Investigations in abdominal trauma

Diagnostic Peritoneal Lavage Abdominal CT scan USS
Indication Document bleeding if hypotensive Document organ injury if normotensive Document fluid if hypotensive
Advantages Early diagnosis and sensitive; 98% accurate Most specific for localising injury; 92 to 98% accurate Early diagnosis, non invasive and repeatable; 86 to 95% accurate
Disadvantages Invasive and may miss retroperitoneal and diaphragmatic injury Location of scanner away from facilities, time taken for reporting, need for contrast Operator dependent and may miss retroperitoneal injury
Amylase may be normal following pancreatic trauma
Urethrography if suspected urethral injury

How well did you know this?
1
Not at all
2
3
4
5
Perfectly
101
Q

A 10 year old boy is shot in the abdomen with an airgun pellet. He is concerned that he will get into trouble and the injury remains concealed for 10 days. Imaging using CT scanning shows it to be lodged in the left lobe of the liver. On examination, his abdomen is soft and non tender and he seems well. What is the best course of action?

Operate and remove the pellet on the next emergency list

Operate and remove the pellet on the next elective operating list

Do not operate and review the patient several weeks later

Extract the pellet using interventional radiology techniques

Perform an MRI scan

A

Given that the child is well, the risks of removing the pellet are not outweighed by the benefits and it should be left in situ at this stage. Management of acute cases- Paediatric

Children will often insert objects into orifices such as the nose and external auditory meatus
Assessment includes assessment of airway and haemodynamic status
Where the airway is not immediately threatened decisions can be made as to whether to manage in the emergency department or transfer to theatre
In general children do not tolerate procedures well and it is usually safer to remove objects in theatre and under general anaesthesia with a secure airway
A chest x-ray is required to ensure that no object is present in the chest, not all objects are radioopaque. However, signs such as focal consolidation may indicate small airway obstruction
In the case of small bore missile injuries the decision relating to surgery depends on the size of the missile and its location. Airgun pellets are a common culprit, if there is a long time interval between the incident and presentation and the object has not caused any significant problems then it may be best left alone
Airgun pellets (and glass) lodged in the soft tissues are usually notoriously difficult to localise and extract, no matter how superficial. Removal in theatre is usually the best option. If the object is radioopaque then an image intensifier should be used

How well did you know this?
1
Not at all
2
3
4
5
Perfectly
102
Q

A 54-year-old man presents to the Emergency Department with a 2 day history of a swollen, painful left knee. You aspirate the joint to avoid admission to the orthopaedic wards. Aspirated joint fluid shows calcium pyrophosphate crystals. Which of the following blood tests is most useful in revealing an underlying cause?

Transferrin saturation

ACTH

ANA

Serum ferritin

LDH

A

This is a typical presentation of pseudogout. An elevated transferrin saturation may indicate haemochromatosis, a recognised cause of pseudogout.

A high ferritin level is also seen in haemochromatosis but can be raised in a variety of infective and inflammatory processes, including pseudogout, as part of an acute phase response.

Pseudogout

Pseudogout is a form of microcrystal synovitis caused by the deposition of calcium pyrophosphate dihydrate in the synovium

Risk factors
hyperparathyroidism
hypothyroidism
haemochromatosis
acromegaly
low magnesium, low phosphate
Wilson’s disease

Features
knee, wrist and shoulders most commonly affected
joint aspiration: weakly-positively birefringent rhomboid shaped crystals
x-ray: chondrocalcinosis

Management
aspiration of joint fluid, to exclude septic arthritis
NSAIDs or intra-articular, intra-muscular or oral steroids as for gout

How well did you know this?
1
Not at all
2
3
4
5
Perfectly
103
Q

A 73 year old lady sustains a distal radius fracture and this is manipulated using a Biers block with prilocaine as the local anaesthetic agent. During the procedure the occlusion cuff deflates and the patient becomes progressively cyanosed. What is the treatment of choice?

Intravenous calcium gluconate

Exchange transfusion

Intravenous methylene blue

Intravenous sodium thiosulphate

Intravenous gelofusine

A

Prilocaine is a recognised cause of methaemoglobinaemia, this is characterised by the development of cyanosis and dyspnoea. This disorder occurs because of the change haemoglobin to a ferric subtype rather than ferrous (Fe2+). This type of change shifts the oxygen dissociation curve to the left and tissue hypoxia occurs. Methylene blue will revert the haemoglobin to the ferrous type and reverse this effect.

Local anaesthetic toxicity

Toxicity results from either accidental intravascular injection (rapid onset of symptoms-usually correct dose), or from excessive dosage (slower onset). Local anaesthetic agents not only exert a membrane stabilising effect on peripheral nerves but will also act on excitable membranes within the CNS and Heart. The sensory neurones in the CNS are suppressed before the motor ones. As a result the early symptoms will typically be those of circumoral paraesthesia and tinnitus, followed by falling GCS and eventually coma.

Management of toxicity
Stop injecting the anaesthetic agent
High flow 100% oxygen via face mask
Cardiovascular monitoring
Administer lipid emulsion (Intralipid 20%) at 1.5ml/Kg over 1 minute as a bolus
Consider lipid emulsion infusion, at 0.25ml/ Kg/ minute
If toxicity due to prilocaine then administer methylene blue

Safe doses
10ml of lignocaine 1% contains 100mg of drug, this would constitute 70% of the maximum safe dose in a 50 kg patient. Up to 7mg / kg can be administered if adrenaline is added to the solution.

Doses of local anaesthetics
Agent Dose plain Dose with adrenaline
Lignocaine 3mg/Kg 7mg/Kg
Bupivicane 2mg/Kg 2mg/Kg
Prilocaine 6mg/Kg 9mg/Kg
These are a guide only as actual doses depend on site of administration, tissue vascularity and co-morbidities.

How well did you know this?
1
Not at all
2
3
4
5
Perfectly
104
Q

A 28 year old man undergoes a completion right hemicolectomy for treatment of a 5cm appendiceal carcinoid. As part of his follow up he is due to undergo 24 hour urine collection for 5-HIAA. Which of the following causes an elevated 5-HIAA in a 24-hour urine collection?

Naproxen

Oranges

Flucloxacillin

Amiodarone

Beef

A

t is important to be aware of what can falsely elevate 5-HIAA to avoid diagnosing carcinoid syndrome incorrectly. These include:

Food: spinach, cheese, wine, caffeine, tomatoes
Drugs: Naproxen, Monoamine oxidase inhibitors
Recent surgery

Carcinoid syndrome

Carcinoid tumours secrete serotonin
Originate in neuroendocrine cells mainly in the intestine (midgut-distal ileum/appendix)
Can occur in the rectum, bronchi
Hormonal symptoms mainly occur when disease spreads outside the bowel

Clinical features
Onset: insidious over many years
Flushing face
Palpitations
Pulmonary valve stenosis and tricuspid regurgitation causing dyspnoea
Asthma
Severe diarrhoea (secretory, persists despite fasting)

Investigation
5-HIAA in a 24-hour urine collection
Somatostatin receptor scintigraphy
CT scan
Blood testing for chromogranin A

Treatment
Octreotide
Surgical removal

How well did you know this?
1
Not at all
2
3
4
5
Perfectly
105
Q

A 5 year old boy presents with recurrent headaches. As part of his assessment he undergoes an MRI scan of his brain. This demonstrates enlargement of the lateral and third ventricles. Where is the most likely site of obstruction?

Foramen of Luschka

Foramen of Magendie

Foramen of Munro

Aqueduct of Sylvius

None of the above

A

The CSF flows from the 3rd to the 4th ventricle via the Aqueduct of Sylvius.

Cerebrospinal fluid

The CSF fills the space between the arachnoid mater and pia mater (covering surface of the brain). The total volume of CSF in the brain is approximately 150ml. Approximately 500 ml is produced by the ependymal cells in the choroid plexus (70%), or blood vessels (30%). It is reabsorbed via the arachnoid granulations which project into the venous sinuses.

Circulation
1. Lateral ventricles (via foramen of Munro)
2. 3rd ventricle
3. Cerebral aqueduct (aqueduct of Sylvius)
4. 4th ventricle
5. Subarachnoid space (via foramina of Magendie and Luschka)
6. Reabsorbed into the venous system via arachnoid granulations into superior sagittal sinus

Composition
Glucose: 50-80mg/dl
Protein: 15-40 mg/dl
Red blood cells: Nil
White blood cells: 0-3 cells/ mm3

How well did you know this?
1
Not at all
2
3
4
5
Perfectly
106
Q

A 70 year old lady who has never smoked is identified as having a peripherally sited mass in her left lung. What is the most likely underlying diagnosis?

Small cell lung cancer

Gastric cancer metastasis

Adenocarcinoma

Squamous cell carcinoma

Lymphoma

A

Adenocarcinomas are the most common tumour type present in never smokers. They are usually located at the periphery.

Lung cancer

Lung cancers may be classified according to histological subtypes. The main distinction is between small cell and non small cell lung cancer. Non small cell lung cancer is the most common variant and accounts for 80% of all lung cancers.

Non small cell lung cancer
These share common features of prognosis and management. They comprise the following tumours:
Squamous cell carcinoma (25% cases)
Adenocarcinoma (40% cases)
Large cell carcinoma (10% cases)
Paraneoplastic features and early disease dissemination are less likely than with small cell lung carcinoma. Adenocarcinoma is the most common lung cancer type encountered in never smokers.

Small cell lung carcinoma
Small cell lung carcinomas are comprised of cells with a neuro endocrine differentiation. The neuroendocrine hormones may be released from these cells with a wide range of paraneoplastic associations. These tumours are strongly associated with smoking and will typically arise in the larger airways. They disseminate early in the course of the disease and although they are usually chemosensitive this seldom results in long lasting remissions.

How well did you know this?
1
Not at all
2
3
4
5
Perfectly
107
Q

A 45 year old man presents with bilateral inguinal hernias. The surgical team decide to repair these laparoscopically through an extraperitoneal approach. Through an infraumbilical incision the surgeons displace the inferior aspect of the rectus abdominis muscle anteriorly and place a prosthetic mesh into the area to repair the hernias. Which structure will lie posterior to the mesh?

Peritoneum
Internal oblique aponeurosis

External oblique aponeurosis

Posterior aspect of the rectus sheath

Bucks fascia

A

During a TEP repair of inguinal hernia the only structure to lie posterior to the mesh is peritoneum. The question is really only asking which structure lies posterior to the rectus abdominis muscle. Since this region is below the arcuate line, the transversalis fascia and peritoneum lie posterior to it. Bucks fascia lies in the penis.

Rectus abdominis muscle

The rectus sheath is formed by the aponeuroses of the lateral abdominal wall muscles. The rectus sheath has a composition that varies according to anatomical level.

  1. Above the costal margin the anterior sheath is composed of external oblique aponeurosis, the costal cartilages are posterior to it.
  2. From the costal margin to the arcuate line, the anterior rectus sheath is composed of external oblique aponeurosis and the anterior part of the internal oblique aponeurosis. The posterior part of the internal oblique aponeurosis and transversus abdominis form the posterior rectus sheath.
  3. Below the arcuate line the aponeuroses of all the abdominal muscles lie in anterior aspect of the rectus sheath. Posteriorly lies the transversalis fascia and peritoneum.

The arcuate line is the point at which the inferior epigastric vessels enter the rectus sheath.

How well did you know this?
1
Not at all
2
3
4
5
Perfectly
108
Q

A 3-month-old boy presents to your outpatient clinic as the parents describe seeing clear fluid discharge from the umbilicus. This has been noted since soon after the umbilical cord separated. Which one of the following is the most likely underlying (embryological) explanation?

A patent urachus

A patent vitello-intestinal duct

A urachal cyst

An exomphalos

An umbilical granuloma

A

After separation of the cord there should not be any discharge from the umbilicus. The presence of clear fluid points towards this being urine rather than bowel content coming from the umbilicus. A patent urachus implies a persistent collection to the bladder, allowing small amounts of urine to leak but the patient often remains completely well.
A patent urachus - correct
A patent vitello-intestinal duct - you would expect bowel content, not clear fluid with a connection to the bowel
A urachal cyst - a urachal cyst typically will not drain as the urachal tract is obliterated on either side of the cyst
An exomphalos - this is an abdominal wall defect with the bowel covered by the cord structures and would be identified soon after birth
An umbilical granuloma - a granuloma may leak small amounts of fluid but typically is more serosanguinous
The correct answer is:
A patent urachus

How well did you know this?
1
Not at all
2
3
4
5
Perfectly
109
Q

A 40-year-old man presents with clinical and endocrinological features of acromegaly. A microadenoma of the pituitary gland is identified. Which one of the following is the most compelling reason for treating this tumour?

Avoidance of cardiovascular complications

Avoidance of colonic cancer

Avoidance of diabetes

Avoidance of optic pathway compression

Avoidance of radiological progression

A

The most compelling reason for treating a pituitary microadenoma causing acromegaly is:

Avoidance of cardiovascular complications

Explanation:
Acromegaly is associated with significant morbidity and mortality due to its systemic effects. Cardiovascular complications are the leading cause of death in patients with acromegaly, primarily due to hypertension, cardiomyopathy, and other related issues. Effective treatment of the pituitary adenoma can reduce the excess production of growth hormone and mitigate these severe cardiovascular risks.

•	Avoidance of colonic cancer: While there is an increased risk of colonic polyps and cancer in acromegaly patients, it is not the most immediate or compelling reason for treatment compared to cardiovascular risks.
•	Avoidance of diabetes: Diabetes is a common complication of acromegaly due to growth hormone-induced insulin resistance, but cardiovascular complications are more critical.
•	Avoidance of optic pathway compression: This is a concern for larger pituitary adenomas (macroadenomas) that can compress adjacent structures. Microadenomas (less than 10 mm in size) typically do not cause optic pathway compression.
•	Avoidance of radiological progression: Preventing tumor growth is important, but the primary focus is on mitigating the clinical consequences of hormone excess, particularly cardiovascular risks.

Therefore, the most compelling reason for treating a pituitary microadenoma in a patient with acromegaly is to avoid cardiovascular complications, which have the most significant impact on morbidity and mortality.

How well did you know this?
1
Not at all
2
3
4
5
Perfectly
110
Q

Whilst on call for orthopaedics you diagnose a child with septic arthritis and you are to assist your consultant in theatre performing a washout of the hip joint.

The approach used is the anterior approach between the rectus femoris and gluteus medius muscles. Which one of the following inter-nervous planes is used in this approach?

Direct rami of the lumbar plexus and the inferior gluteal nerve

Femoral nerve and superior gluteal nerve

Lateral cutaneous nerve and inferior gluteal nerve

Lateral cutaneous nerve and super gluteal nerve

Super gluteal nerve and inferior gluteal nerve

A

The approach described for the washout of the hip joint in septic arthritis involves the anterior approach between the rectus femoris and the gluteus medius muscles. This approach utilizes an internervous plane to minimize the risk of nerve damage.

The correct internervous plane for this approach is:

Femoral nerve and superior gluteal nerve

Explanation:

•	The femoral nerve innervates the rectus femoris muscle.
•	The superior gluteal nerve innervates the gluteus medius muscle.

Thus, the correct answer is:
Femoral nerve and superior gluteal nerve

How well did you know this?
1
Not at all
2
3
4
5
Perfectly
111
Q

Which of the following visceral anastomoses has the lowest risk of anastomotic leak? You may assume that all are constructed in ideal circumstances.

Stapled ileocolic anastomosis

Hand sewn anastomosis of the proximal ileum

Stapled colorectal anastomosis defunctioned with loop ileostomy

Stapled colorectal anastomosis defunctioned with loop colostomy

Hand sewn oesophagojejunal anastomosis

A

Among the visceral anastomoses listed, the one with the lowest risk of anastomotic leak, assuming all are constructed in ideal circumstances, is generally considered to be:

Stapled ileocolic anastomosis

Explanation:

•	Stapled ileocolic anastomosis typically has a lower risk of anastomotic leak compared to other types of anastomoses because the ileum and colon are relatively well-vascularized and less prone to tension and technical difficulties.
•	Hand-sewn anastomoses, particularly in complex areas like the oesophagojejunal junction, carry higher risks due to the technical challenges and the poor vascularity of the tissues involved.
•	Stapled colorectal anastomoses, even when defunctioned with loop ileostomy or loop colostomy, have higher risks of leaks compared to ileocolic anastomoses because the rectal area is more prone to complications due to factors like tension, poor blood supply, and potential for contamination.

Therefore, the correct answer is:
Stapled ileocolic anastomosis

For a visceral anastomosis to heal, three factors need to be satisfied:
1. Mucosal- mucosal apposition
2. A good blood supply
3. No tension
The more these are compromised, the higher the leak rate.
Rectal and oesophageal surgery have some of the highest rates of anastomotic leakage. Following anterior resection leaks are quoted to occur in up to 10% of cases. Small bowel anastomoses are the most technically forgiving. Factors increasing the risk of anastamotic leakage include previous irradiation, sepsis, malnutrition, poor blood supply and poor technique.
The defunctioning of rectal anastomoses may reduce the clinical impact of anastomotic leak and make it amenable to percutaneous drainage, but does not necessarily reduce the incidence of leaks themselves.

How well did you know this?
1
Not at all
2
3
4
5
Perfectly
112
Q

An 18 year old man is undergoing an orchidectomy via a scrotal approach. The surgeons mobilise the spermatic cord. From which of the following is the outermost layer of this structure derived?

Internal oblique aponeurosis

External oblique aponeurosis

Transversalis fascia

Rectus sheath

Campers fascia

A

The outermost covering of the spermatic cord is derived from the external oblique aponeurosis.This layer is added as the cord passes through the superficial inguinal ring. Scrotal and testicular anatomy

Spermatic cord
Formed by the vas deferens and is covered by the following structures:
Layer Origin
Internal spermatic fascia Transversalis fascia
Cremasteric fascia From the fascial coverings of internal oblique
External spermatic fascia External oblique aponeurosis

Contents of the cord
Vas deferens Transmits sperm and accessory gland secretions
Testicular artery Branch of abdominal aorta supplies testis and epididymis
Artery of vas deferens Arises from inferior vesical artery
Cremasteric artery Arises from inferior epigastric artery
Pampiniform plexus Venous plexus, drains into right or left testicular vein
Sympathetic nerve fibres Lie on arteries, the parasympathetic fibres lie on the vas
Genital branch of the genitofemoral nerve Supplies cremaster
Lymphatic vessels Drain to lumbar and para-aortic nodes

Scrotum
Composed of skin and closely attached dartos fascia.
Arterial supply from the anterior and posterior scrotal arteries
Lymphatic drainage to the inguinal lymph nodes
Parietal layer of the tunica vaginalis is the innermost layer

Testes
The testes are surrounded by the tunica vaginalis (closed peritoneal sac). The parietal layer of the tunica vaginalis adjacent to the internal spermatic fascia.
The testicular arteries arise from the aorta immediately inferiorly to the renal arteries.
The pampiniform plexus drains into the testicular veins, the left drains into the left renal vein and the right into the inferior vena cava.
Lymphatic drainage is to the para-aortic nodes.

How well did you know this?
1
Not at all
2
3
4
5
Perfectly
113
Q

A term neonate is born by emergency cesarean section the final stages of pregnancy having been complicated by polyhydramnios. Feeding results in vomiting and a imaging is performed which demonstrates extrinsic compression of the second part of the duodenum. What is the most likely embryological origin of the structure that is causing this compression?

The dorsal pancreas

The hepatic diverticulum

The vitellointestinal duct

Ladds bands

The ventral pancreas

A

The obstruction is most likely to be the result of an annular pancreas. The dorsal and ventral pancreas structures initially lie opposite each other. However, the ventral pancreas rotates to eventually fuse with the dorsal. Sometimes this can result in an annular pancreas and some patients can develop obstruction as a result.

Pancreatic embryology

The pancreas develops from a ventral and dorsal entodermal outgrowth from the duodenum. The ventral arises close to, or in common with, the hepatic diverticulum, and the larger, dorsal outgrowth arises slightly cranial to the ventral and extends into the mesoduodenum and dorsal mesogastrium. As a result of the differential growth in the duodenal wall, the point of attachment of the ventral outgrowth with the undivided stem of the hepatic diverticulum and the ventral mesentery are displaced around the right hand side of the duodenum to a dorsal position. As a result, the two pancreatic remnants eventually lie together and then become fused. In this process the ventral rudiment forms the uncinate process and the adjacent region of the head, while the dorsal forms the anterior part of the head, the body and the tail of the pancreas.
At the time of fusion, the ducts become joined. This occurs in a manner that results in the dorsal remnant being drained by the duct of the ventral remnant, this composite duct being named the pancreatic duct.

How well did you know this?
1
Not at all
2
3
4
5
Perfectly
114
Q

A 54-year-old man who has end stage diabetic nephropathy is being assessed for a renal transplant. When assessing the HLA matching between donor and recipient what is the most important HLA antigen to match?

DP

B

DR

C

A

A

Renal transplant HLA matching - DR is the most important. Renal transplant:HLA typing and graft failure

The human leucocyte antigen (HLA) system is the name given to the major histocompatibility complex (MHC) in humans. It is coded for on chromosome 6.

Some basic points on the HLA system
Class 1 antigens include A, B and C. Class 2 antigens include DP,DQ and DR
When HLA matching for a renal transplant the relative importance of the HLA antigens are as follows DR > B > A

Graft survival
1 year = 90%, 10 years = 60% for cadaveric transplants
1 year = 95%, 10 years = 70% for living-donor transplants

Post-op problems
ATN of graft
Vascular thrombosis
Urine leakage
UTI

Hyperacute acute rejection
Due to antibodies against donor HLA type 1 antigens
Rarely seen due to HLA matching

Acute graft failure (< 6 months)
Usually due to mismatched HLA
Other causes include cytomegalovirus infection
Management: give steroids, if resistant use monoclonal antibodies

Causes of chronic graft failure (> 6 months)
Chronic allograft nephropathy
Ureteric obstruction
Recurrence of original renal disease (MCGN > IgA > FSGS)

How well did you know this?
1
Not at all
2
3
4
5
Perfectly
115
Q

A 45 year old man is referred to the breast clinic with gynaecomastia. He takes the drugs listed below. Which is least likely to be the cause of his symptoms?

Spironolactone

Carbimazole

Chlorpromazine

Cimetidine

Methyldopa

A

Mnemonic for drugs causing gynaecomastia: DISCO

D igitalis
I soniazid
S pironolactone
C imetidine
O estrogen

Mnemonic for causes of gynaecomastia: METOCLOPRAMIDE

M etoclopramide
E ctopic oestrogen
T rauma skull/tumour breast, testes
O rchitis
C imetidine, Cushings
L iver cirrhosis
O besity
P araplegia
R A
A cromegaly
M ethyldopa
I soniazid
D igoxin
E thionamide
Carbimazole is not associated with gynaecomastia. Note the question asks for the least likely cause.

Gynaecomastia

Gynaecomastia describes an abnormal amount of breast tissue in males and is usually caused by an increased oestrogen:androgen ratio. It is important to differentiate the causes of galactorrhoea (due to the actions of prolactin on breast tissue) from those of gynaecomastia

Causes of gynaecomastia

physiological: normal in puberty
syndromes with androgen deficiency: Kallman’s, Klinefelter’s
testicular failure: e.g. Mumps
liver disease
testicular cancer e.g. Seminoma secreting HCG
ectopic tumour secretion
hyperthyroidism
haemodialysis
drugs: see below

Drug causes of gynaecomastia

spironolactone (most common drug cause)
cimetidine
digoxin
cannabis
finasteride
oestrogens, anabolic steroids

Very rare drug causes of gynaecomastia
tricyclics
isoniazid
calcium channel blockers
heroin
busulfan
methyldopa

Treatment options
identify and manage any overt underlying cause
liposuction provides the best cosmetic outcome

How well did you know this?
1
Not at all
2
3
4
5
Perfectly
116
Q

A 40 year old man undergoes a complex appendicectomy and the wound fails to heal satisfactorily. The wound site itself is associated with multiple sinuses and fistulas. Pus is sent for microbiology and shows gram positive organisms and sulphur granules. What is the most likely underlying diagnosis?

Infection with Staphylococcus aureus

Infection with Bacteroides fragilis

Actinomycosis

Crohns disease

Ulcerative colitis

A

The presence of chronic sinuses together with gram positive organisms and sulphur granules is highly suggestive of Actinomycosis. Crohns disease is associated with multiple fistulae, but not gram positive organisms with sulphur granules. Actinomycosis

Chronic, progressive granulomatous disease caused by filamentous gram positive anaerobic bacteria from the Actinomycetaceae family.

Actinomyces are commensal bacteria that become pathogenic when a mucosal barrier is breached.

The disease most commonly occurs in the head and neck, although it may also occur in the abdominal cavity and in the thorax.

The mass will often enlarge across tissue planes with the formation of multiple sinus tracts.

Abdominopelvic actinomycosis occurs most frequently in individuals that have had appendicitis (65%) cases.

Pathology
On histological examination gram positive organisms and evidence of sulphur granules.
Sulphur granules are colonies of organisms that appear as round or oval basophilic masses.
They are also seen in other conditions such as nocardiosis.

Treatment
Long term antibiotic therapy usually with penicillin.
Surgical resection is indicated for extensive necrotic tissue, non healing sinus tracts, abscesses or where biopsy is needed to exclude malignancy.

References
Wong V, Turmezei T and Weston V. Actinomycosis. BMJ 2011;343d6099.

How well did you know this?
1
Not at all
2
3
4
5
Perfectly
117
Q

A 48 year old lady is undergoing a left sided adrenalectomy for an adrenal adenoma. The superior adrenal artery is injured and starts to bleed, from which of the following does this vessel arise?

Left renal artery

Inferior phrenic artery

Aorta

Splenic

None of the above

A

The superior adrenal artery is a branch of the inferior phrenic artery.

Adrenal gland anatomy

Location Superomedially to the upper pole of each kidney
Relationships of the right adrenal Diaphragm-Posteriorly, Kidney-Inferiorly, Vena Cava-Medially, Hepato-renal pouch and bare area of the liver-Anteriorly
Relationships of the left adrenal Crus of the diaphragm-Postero- medially, Pancreas and splenic vessels-Inferiorly, Lesser sac and stomach-Anteriorly
Arterial supply Superior adrenal arteries- from inferior phrenic artery, Middle adrenal arteries - from aorta, Inferior adrenal arteries -from renal arteries
Venous drainage of the right adrenal Via one central vein directly into the IVC
Venous drainage of the left adrenal Via one central vein into the left renal vein

How well did you know this?
1
Not at all
2
3
4
5
Perfectly
118
Q

A 73 year old lady is admitted with small bowel obstruction and following surgery is diagnosed as having diaphragm disease. Which of the agents listed below is the most likely precipitant?

Alcohol abuse

Oral non steroidal anti inflammatory drugs

Abdominal radiotherapy

Oral steroids

Chronic laxative misuse

A

Diaphragm disease, which is characterized by multiple thin, diaphragm-like strictures in the small intestine, is most commonly associated with:

Oral non-steroidal anti-inflammatory drugs (NSAIDs)

Explanation:

•	Oral NSAIDs are well-documented causes of diaphragm disease in the small intestine. NSAIDs can lead to mucosal injury, resulting in the formation of strictures and characteristic diaphragm-like lesions.
•	Alcohol abuse, abdominal radiotherapy, oral steroids, and chronic laxative misuse are not typically associated with diaphragm disease.

Therefore, the correct answer is:
Oral non-steroidal anti-inflammatory drugs

How well did you know this?
1
Not at all
2
3
4
5
Perfectly
119
Q

A 65 year old man presents with a new onset left sided hemiparesis. A CT scan of the brain is performed and this demonstrates a 4cm lesion within the right frontal lobe, it traverses the midline and displays marked central necrosis. There is extensive oedema surrounding the lesion. What is the most likely diagnosis?

Meningioma

Central neurocytoma

Ependymoma

Oligodendroglioma

Glioblastoma

A

Glioblastomas typically display such marked appearances and an important differential on imaging is one of cerebral abscess. A meningioma would not display such infiltrative behavior. Significant necrosis is more commonly seen with glioblastomas than with other CNS tumours. CNS tumours

60% = Glioma and metastatic disease
20% = Meningioma
10% = Pituitary lesions
In paediatric practice medulloblastomas (neuroectodermal tumours) were the commonest lesions, astrocytomas now account for the majority.
Tumours arising in right temporal and frontal lobe may reach considerable size before becoming symptomatic. Whereas tumours in the speech and visual areas will typically produce early symptoms.

Most paediatric CNS tumours are infratentorial
Most adult CNS tumours are supratentorial

Diagnosis
MRI Scanning provides the best resolution.

Treatment
Usually surgery, even if tumour cannot be completely resected conditions such as rising ICP can be addressed with tumour debulking and survival and quality of life prolonged.
Curative surgery can usually be undertaken with lesions such as meningiomas. Gliomas have a marked propensity to invade normal brain and resection of these lesions is nearly always incomplete.

How well did you know this?
1
Not at all
2
3
4
5
Perfectly
120
Q

A patient is seen in clinic complaining of abdominal pain. Routine bloods show:

Na+ 142 mmol/l
K+ 4.0 mmol/l
Chloride 104 mmol/l
Bicarbonate 19 mmol/l
Urea 7.0 mmol/l
Creatinine 112 µmol/l

What is the anion gap?

4 mmol/L

14 mmol/L

20 mmol/L

21 mmol/L

23 mmol/L

A

The anion gap may be calculated by using (sodium + potassium) - (bicarbonate + chloride)

= (142 + 4.0) - (104 + 19) = 23 mmol/L

Anion gap

The anion gap is calculated by:

(sodium + potassium) - (bicarbonate + chloride)

A normal anion gap is 8-14 mmol/L

It is useful to consider in patients with a metabolic acidosis:

Causes of a normal anion gap or hyperchloraemic metabolic acidosis
gastrointestinal bicarbonate loss: diarrhoea, ureterosigmoidostomy, fistula
renal tubular acidosis
drugs: e.g. acetazolamide
ammonium chloride injection
Addison’s disease

Causes of a raised anion gap metabolic acidosis
lactate: shock, hypoxia
ketones: diabetic ketoacidosis, alcohol
urate: renal failure
acid poisoning: salicylates, methanol

How well did you know this?
1
Not at all
2
3
4
5
Perfectly
121
Q

A 52 year old lady presents to the surgical clinic with a goitre. She is taking medication for an underlying psychiatric disorder. Which of the drugs listed below is most likely to be responsible?

Haloperidol

Imipramine

Amytryptiline

Lithium

Venlafaxine

A

A significant proportion of patients who take lithium treatment will develop a goitre and a number will become clinically hypothyroid. For this reason, monitoring of thyroid function tests is necessary during lithium therapy. Thyroid disease

Patients may present with a number of different manifestations of thyroid disease. They can be broadly sub classified according to whether they are euthyroid or have clinical signs of thyroid dysfunction. In addition it needs to be established whether they have a mass or not.

Assessment
History
Examination including USS
If a nodule is identified then it should be sampled ideally via an image guided fine needle aspiration
Radionucleotide scanning is of limited use

Thyroid Tumours
Papillary carcinoma
Follicular carcinoma
Anaplastic carcinoma
Medullary carcinoma
Lymphoma’s

Multinodular goitre
One of the most common reasons for presentation
Provided the patient is euthyroid and asymptomatic and no discrete nodules are seen, they can be reassured.
In those with compressive symptoms surgery is required and the best operation is a total thyroidectomy.
Sub total resections were practised in the past and simply result in recurrent disease that requires a difficult revisional resection.

Endocrine dysfunction
In general these patients are managed by physicians initially.
Surgery may be offered alongside radio iodine for patients with Graves disease that fails with medical management or in patients who would prefer not to be irradiated (e.g. pregnant women).
Patients with hypothyroidism do not generally get offered a thyroidectomy. Sometimes people inadvertently get offered resections during the early phase of Hashimotos thyroiditis, however, with time the toxic phase passes and patients can simply be managed with thyroxine.

Complications following surgery
Anatomical such as recurrent laryngeal nerve damage.
Bleeding. Owing to the confined space haematoma’s may rapidly lead to respiratory compromise owing to laryngeal oedema.
Damage to the parathyroid glands resulting in hypocalcaemia.

Further sources of information
1. http://www.acb.org.uk/docs/TFTguidelinefinal.pdf- Association of Clinical Biochemistry guidelines for thyroid function tests.

  1. British association of endocrine surgeons website- http://www.baets.org.uk
How well did you know this?
1
Not at all
2
3
4
5
Perfectly
122
Q

Which of the following muscles inserts onto the lesser tuberosity of the humerus?

Subscapularis

Deltoid

Supraspinatus

Teres minor

Infraspinatus

A

With the exception of subscapularis which inserts into the lesser tuberosity, the muscles of the rotator cuff insert into the greater tuberosity.

Shoulder joint

Shallow synovial ball and socket type of joint.
It is an inherently unstable joint, but is capable to a wide range of movement.
Stability is provided by muscles of the rotator cuff that pass from the scapula to insert in the greater tuberosity (all except sub scapularis-lesser tuberosity).

Glenoid labrum
Fibrocartilaginous rim attached to the free edge of the glenoid cavity
Tendon of the long head of biceps arises from within the joint from the supraglenoid tubercle, and is fused at this point to the labrum.
The long head of triceps attaches to the infraglenoid tubercle

Fibrous capsule
Attaches to the scapula external to the glenoid labrum and to the labrum itself (postero-superiorly)
Attaches to the humerus at the level of the anatomical neck superiorly and the surgical neck inferiorly
Anteriorly the capsule is in contact with the tendon of subscapularis, superiorly with the supraspinatus tendon, and posteriorly with the tendons of infraspinatus and teres minor. All these blend with the capsule towards their insertion.
Two defects in the fibrous capsule; superiorly for the tendon of biceps. Anteriorly there is a defect beneath the subscapularis tendon.
The inferior extension of the capsule is closely related to the axillary nerve at the surgical neck and this nerve is at risk in anteroinferior dislocations. It also means that proximally sited osteomyelitis may progress to septic arthritis.

Movements and muscles
Flexion Anterior part of deltoid
Pectoralis major
Biceps
Coracobrachialis
Extension Posterior deltoid
Teres major
Latissimus dorsi
Adduction Pectoralis major
Latissimus dorsi
Teres major
Coracobrachialis
Abduction Mid deltoid
Supraspinatus
Medial rotation Subscapularis
Anterior deltoid
Teres major
Latissimus dorsi
Lateral rotation Posterior deltoid
Infraspinatus
Teres minor

Important anatomical relations
Anteriorly Brachial plexus
Axillary artery and vein
Posterior Suprascapular nerve
Suprascapular vessels
Inferior Axillary nerve
Circumflex humeral vessels

How well did you know this?
1
Not at all
2
3
4
5
Perfectly
123
Q

A 22 year old man is referred to the surgical clinic. He has been complaining of varicose veins for many years. On examination he has extensive varicosities of the right leg, there are areas of marked port wine staining. The saphenofemoral junction is competent on doppler assessment. The most likely underlying diagnosis is:

Deep vein thrombosis

Klippel-Trenaunay syndrome

Varicose veins due to sapheno-popliteal junction incompetence

Sturge - Weber syndrome

Angiosarcoma

A

Sturge - Weber syndrome is a an arteriovenous malformation affecting the face and CNS, the peripheral vessels are not affected. Simple varicose veins should not typically be associated with port wine staining, nor should a DVT or angiosarcoma.

Klippel-Trenaunay syndrome

Klippel-Trenaunay-Weber syndrome generally affects a single extremity, although cases of multiple affected limbs have been reported. The leg is the most common site followed by the arms, the trunk, and rarely the head and the neck

Signs and symptoms
The birth defect is diagnosed by the presence of a combination of these symptoms:
One or more distinctive port-wine stains with sharp borders
Varicose veins
Hypertrophy of bony and soft tissues, that may lead to local gigantism or shrinking.
An improperly developed lymphatic system
In some cases, port-wine stains (capillary port wine type) may be absent. Such cases are very rare and may be classified as ‘atypical Klippel-Trenaunay syndrome’.

KTS can either affect blood vessels, lymph vessels, or both. The condition most commonly presents with a mixture of the two. Those with venous involvement experience increased pain and complications.

How well did you know this?
1
Not at all
2
3
4
5
Perfectly
124
Q

What is the mechanism of action of macrolides?

Causes misreading of mRNA

Interferes with cell wall formation

Inhibits DNA synthesis

Inhibits RNA synthesis

Inhibits protein synthesis

A

Macrolides act by inhibiting bacterial protein synthesis. If pushed to give an answer they are bacteriostatic in nature, but in reality this depends on the dose and type of organism being treated. Erythromycin was the first macrolide used clinically. Newer examples include clarithromycin and azithromycin.

Adverse effects of erythromycin
gastrointestinal side-effects are common
cholestatic jaundice: risk may be reduced if erythromycin stearate is used
P450 inhibitor

Antibiotics: mechanism of action

The lists below summarise the site of action of the commonly used antibiotics

Inhibit cell wall formation
penicillins
cephalosporins

Inhibit protein synthesis
aminoglycosides (cause misreading of mRNA)
chloramphenicol
macrolides (e.g. erythromycin)
tetracyclines
fusidic acid

Inhibit DNA synthesis
quinolones (e.g. ciprofloxacin)
metronidazole
sulphonamides
trimethoprim

Inhibit RNA synthesis
rifampicin

How well did you know this?
1
Not at all
2
3
4
5
Perfectly
125
Q

You are performing a study of weight in patients attending pre operative assessment clinic for elective knee replacement. Assuming that the results are normally distributed, what percentage of values lie within two standard deviations of the mean weight?

95.4%

5.3%

98.3%

10%

97.5%

A

In statistics, the 68-95-99.7 rule or three-sigma rule, or empirical rule states that for a normal distribution, nearly all values lie within 3 standard deviations of the mean.
About 68.27% of the values lie within 1 standard deviation of the mean. Similarly, about 95.45% of the values lie within 2 standard deviations of the mean. Nearly all (99.73%) of the values lie within 3 standard deviations of the mean
95.4% of values lie within 2 SD of the mean.

How well did you know this?
1
Not at all
2
3
4
5
Perfectly
126
Q

What is the longest part of the male urethra?

Membranous urethra

Spongy urethra

Prostatic urethra

Urethra within the internal urethra orifice

Urethra within the urethral crest

A

The spongy urethra is around 15cm long and is the longest part of the male urethra. Urethral anatomy

Female urethra
The female urethra is shorter and more acutely angulated than the male urethra. It is an extra-peritoneal structure and embedded in the endopelvic fascia. The neck of the bladder is subjected to transmitted intra-abdominal pressure and therefore deficiency in this area may result in stress urinary incontinence. Between the layers of the urogenital diaphragm the female urethra is surrounded by the external urethral sphincter, this is innervated by the pudendal nerve. It ultimately lies anterior to the vaginal orifice.

Male urethra
In males the urethra is much longer and is divided into four parts.

Pre-prostatic urethra Extremely short and lies between the bladder and prostate gland.It has a stellate lumen and is between 1 and 1.5cm long.Innervated by sympathetic noradrenergic fibres, as this region is composed of striated muscles bundles they may contract and prevent retrograde ejaculation.
Prostatic urethra This segment is wider than the membranous urethra and contains several openings for the transmission of semen (at the midpoint of the urethral crest).
Membranous urethra Narrowest part of the urethra and surrounded by external sphincter. It traverses the perineal membrane 2.5cm postero-inferior to the symphysis pubis.
Penile urethra Travels through the corpus spongiosum on the underside of the penis. It is the longest urethral segment.It is dilated at its origin as the infrabulbar fossa and again in the glans penis as the navicular fossa. The bulbo-urethral glands open into the spongiose section of the urethra 2.5cm below the perineal membrane.

The urothelium is transitional in nature near to the bladder and becomes squamous more distally.

How well did you know this?
1
Not at all
2
3
4
5
Perfectly
127
Q

Which one of the following serum proteins is most likely to increase in a patient with severe sepsis?

Transferrin

Transthyretin

Ferritin

Albumin

Cortisol binding protein

A

Ferritin can be markedly increased during acute illness. The other parameters tend to decrease during an acute phase response. Acute phase proteins

Acute phase proteins
CRP
procalcitonin
ferritin
fibrinogen
alpha-1 antitrypsin
caeruloplasmin
serum amyloid A
haptoglobin
complement

During the acute phase response the liver decreases the production of other proteins (sometimes referred to as negative acute phase proteins). Examples include:
albumin
transthyretin (formerly known as prealbumin)
transferrin
retinol binding protein
cortisol binding protein

Levels of CRP are commonly measured in acutely unwell patients. CRP is a protein synthesised in the liver and binds to phosphocholine in bacterial cells and on those cells undergoing apoptosis. In binding to these cells it is then able to activate the complement system. CRP levels are known to rise in patients following surgery. However, levels of greater than 150 at 48 hours post operatively are suggestive of evolving complications.

How well did you know this?
1
Not at all
2
3
4
5
Perfectly
128
Q

A 73 year old man undergoes a right below knee amputation for end stage peripheral vascular disease. He is reviewed in the clinic 8 weeks post operatively and complains of a persistent, burning discomfort over his amputation site stump. On examination, his wound has healed and proximal pulses have a biphasic signal on doppler ultrasound. What is the most appropriate management?

Commence amitryptyline

Commence fentanyl patch

Arrange duplex scan

Arrange MRI scan of the stump

Commence carbamazepine

A

This patient has neuropathic pain. Amitryptyline is the treatment of choice. Carbamazepine is mainly used for trigeminal neuralgia.

Neuropathic pain

Neuropathic pain may be defined as pain which arises following damage or disruption of the nervous system. It is often difficult to treat and responds poorly to standard analgesia.

Examples include:
diabetic neuropathy
post-herpetic neuralgia
trigeminal neuralgia
prolapsed intervertebral disc

NICE issued guidance in 2010 on the management of neuropathic pain:
first-line treatment*: oral amitriptyline or pregabalin
if satisfactory pain reduction is obtained with amitriptyline but the person cannot tolerate the adverse effects, consider oral imipramine or nortriptyline as an alternative
second-line treatment: if first-line treatment was with amitriptyline, switch to or combine with pregabalin. If first-line treatment was with pregabalin, switch to or
combine with amitriptyline
other options: pain management clinic, tramadol (not other strong opioids), topical lidocaine for localised pain if patients unable to take oral medication

*please note that for some specific conditions the guidance may vary. For example carbamazepine is used first-line for trigeminal neuralgia, duloxetine for diabetic neuropathy

How well did you know this?
1
Not at all
2
3
4
5
Perfectly
129
Q

A 20 year old man is involved in a road traffic accident. Following the incident he is unable to extend his wrist. However, this improves over the following weeks. Which type of injury is he most likely to have sustained?

Radial nerve neurotmesis

Radial nerve neuropraxia

Axillary nerve axonotmesis

Ulnar nerve neuropraxia

Ulnar nerve axonotmesis

A

The injury described—initial inability to extend the wrist following a road traffic accident, which then improves over the following weeks—is most consistent with a radial nerve neuropraxia.

Neuropraxia is a type of nerve injury characterized by a temporary loss of motor and sensory function due to blockage of nerve conduction. This is often caused by compression or blunt trauma, and typically, there is no physical damage to the nerve itself. Recovery is usually complete and occurs within days to weeks.

Here’s why the other options are less likely:

•	Radial nerve neurotmesis: This is a more severe form of nerve injury where the nerve is completely severed. Recovery typically requires surgical intervention and takes much longer, with less chance of full recovery.
•	Axillary nerve axonotmesis: This involves damage to the axillary nerve and the nerve fibers (axons), but the surrounding connective tissue remains intact. It would not typically result in wrist drop, as the axillary nerve primarily innervates the deltoid muscle and the teres minor muscle.
•	Ulnar nerve neuropraxia: This would affect the ulnar nerve, which primarily controls muscles in the forearm and hand. It would not cause wrist drop, which is specifically related to the radial nerve.
•	Ulnar nerve axonotmesis: Similar to the above, this involves damage to the ulnar nerve and would not result in wrist drop.

Therefore, the most likely injury is radial nerve neuropraxia.

Neuropraxia

Nerve intact but electrical conduction is affected
Myelin sheath integrity is preserved
Full recovery
Autonomic function preserved
Wallerian degeneration does not occur

How well did you know this?
1
Not at all
2
3
4
5
Perfectly
130
Q

A 50 year old male who weighs 70Kg is severely unwell with intestinal failure and cannot receive enteral nutrition. He has become malnourished and the decision is made to commence total parenteral nutrition. What is the most appropriate initial dose of calories which should be administered via this route over 24 hours?

700 Kcal

2000 Kcal

3000 Kcal

500 Kcal

2500 Kcal

A

The appropriate initial dose of calories to be administered via total parenteral nutrition (TPN) for a severely unwell, malnourished 50-year-old male weighing 70 kg would be:

2000 Kcal

This recommendation aligns with standard clinical practice for initial TPN administration, where the caloric intake typically ranges from 25 to 35 kcal/kg/day, starting at the lower end for critically ill or malnourished patients and gradually increasing as tolerated.

Explanation:

TPN is used to provide nutrition intravenously when enteral feeding is not possible. The initial caloric needs are calculated based on the patient’s weight and clinical condition. For a 70 kg male:

•	Minimum requirement:  70 \text{ kg} \times 25 \text{ kcal/kg/day} = 1750 \text{ kcal/day} 
•	Optimal requirement:  70 \text{ kg} \times 30 \text{ kcal/kg/day} = 2100 \text{ kcal/day} 

Thus, starting at 2000 kcal/day is a reasonable and appropriate initial dose.

As this person is severely ill and malnourished, the initial calories administered is reduced by 50% in the first few days.

Given that the initial caloric intake should be reduced by 50% in the first few days for a patient starting on total parenteral nutrition (TPN), the most appropriate initial dose of calories for this patient would be:

1000 Kcal

Explanation:

If we consider that the optimal caloric requirement for a 70 kg male is approximately 2000 kcal/day (as calculated previously), then reducing this by 50% for the initial period would result in an initial dose of:

2000 \text{ kcal/day} \times 0.5 = 1000 \text{ kcal/day}

Therefore, 1000 kcal/day is a safer starting dose to prevent complications such as refeeding syndrome. However, since 1000 kcal/day was not one of the options provided, the closest and most appropriate option would be:

700 Kcal

This represents a conservative approach to initiate TPN and gradually increase the caloric intake as the patient stabilizes.

Nutrition prescriptions

National institute of clinical excellence (NICE) guidelines

For people not severely ill and not at risk of refeeding syndrome aim to give
25-35 kcal/kg/day (lower if BMI > 25)
0.8-1.5g protein /kg/day
30-35 ml fluid/kg/day
Adequate electrolytes, minerals, vitamins
Severely ill patients aim to give < 50% of the energy and protein levels over the first 24-48h.

For people at high risk of refeeding syndrome:
Start at up to 10 kcal/kg/day increasing to full needs over 4-7 days
Start immediately before and during feeding: oral thiamine 200-300mg/day, vitamin B co strong 1 tds and supplements
Give K+ (2-4 mmol/kg/day), phosphate (0.3-0.6 mmol/kg/day), magnesium (0.2-0.4 mmol/kg/day)

How well did you know this?
1
Not at all
2
3
4
5
Perfectly
131
Q

A 24 year old man presents with symptoms of malaise, weight loss and lymphadenopathy. A lymph node biopsy is performed and the subsequent histology report states that there is evidence of granuloma formation and central necrosis. What is the most likely underlying cause?

Non Hodgkins lymphoma

Churg Strauss syndrome

Epstein Barr Virus infection

Rheumatoid nodule

Infection with Mycobacterium tuberculosis

A

These histological features are typically seen in TB. Necrosis occurring in granulomas is usually indicative of an underlying infective cause. Churg Strauss syndrome is a form of vasculitis, which is the usual histological finding. Granulomas are reported in the condition, but it is rare for them to demonstrate necrosis. Tuberculosis pathology

Is a form of primary chronic inflammation, caused by the inability of macrophages to kill the Mycobacterium tuberculosis.
The macrophages often migrate to regional lymph nodes, the lung lesion plus affected lymph nodes is referred to as a Ghon complex.
This leads to the formation of a granuloma which is a collection of epithelioid histiocytes.
There is the presence of caseous necrosis in the centre.
The inflammatory response is mediated by a type 4 hypersensitivity reaction.
In healthy individuals the disease may be contained, in the immunocompromised disseminated (miliary TB) may occur.

Diagnosis
Waxy membrane of mycobacteria prevents binding with normal stains. Ziehl - Neelsen staining is typically used.
Culture based methods take far longer.

Image showing acid- alcohol fast mycobacteria stained using the Ziehl- Neelsen method

How well did you know this?
1
Not at all
2
3
4
5
Perfectly
132
Q

A 7 year old boy falls off a wall the distance is 7 feet. He lands on his left side and there is left flank bruising. There is no haematuria. He is otherwise stable and haemoglobin is within normal limits. What is the most appropriate initial course of action?

Undertake a CT scan of the abdomen

Undertake an abdominal USS

Undertake diagnostic peritoneal lavage

Undertake a splenectomy

Arrange an angiogram and possibly proceed to embolisation

A

This will demonstrate any overt splenic injury. A CT scan carries a significant dose of radiation. In the absence of haemodynamic instability or other major associated injuries the use of USS to exclude intraabdominal free fluid (blood) would seem safe when coupled with active observation. An USS will also show splenic haematomas. Splenic trauma

  • The spleen is one of the more commonly injured intra abdominal organs
    In most cases the spleen can be conserved. The management is dictated by the associated injuries, haemodynamic status and extent of direct splenic injury.

Management of splenic trauma
Conservative Small subcapsular haematoma
Minimal intra abdominal blood
No hilar disruption
Laparotomy with conservation Increased amounts of intraabdominal blood
Moderate haemodynamic compromise
Tears or lacerations affecting <50%
Resection Hilar injuries
Major haemorrhage
Major associated injuries

Splenectomy
Technique
Trauma
GA
Long midline incision
If time permits insert a self retaining retractor (e.g. Balfour/ omnitract)
Large amount of free blood is usually present. Pack all 4 quadrants of the abdomen. Allow the anaesthetist to ‘catch up’
Remove the packs and assess the viability of the spleen. Hilar injuries and extensive parenchymal lacerations will usually require splenectomy.
Divide the short gastric vessels and ligate them.
Clamp the splenic artery and vein. Two clamps on the patient side are better and allow for double ligation and serve as a safety net if your assistant does not release the clamp smoothly.
Be careful not to damage the tail of the pancreas, if you do then this will need to be formally removed and the pancreatic duct closed.
Wash out the abdomen and place a tube drain to the splenic bed.
Some surgeons implant a portion of spleen into the omentum, whether you decide to do this is a matter of personal choice.
Post operatively the patient will require prophylactic penicillin V and pneumococcal vaccine.

Elective
Elective splenectomy is a very different operation from that performed in the emergency setting. The spleen is often large (sometimes massive). Most cases can be performed laparoscopically. The spleen will often be macerated inside a specimen bag to facilitate extraction.

Complications
Haemorrhage (may be early and either from short gastrics or splenic hilar vessels
Pancreatic fistula (from iatrogenic damage to pancreatic tail)
Thrombocytosis: prophylactic aspirin
Encapsulated bacteria infection e.g. Strep. pneumoniae, Haemophilus influenzae and Neisseria meningitidis

How well did you know this?
1
Not at all
2
3
4
5
Perfectly
133
Q

A 28 year old man is shot in the right chest and develops a right haemothorax necessitating a thoracotomy. The surgeons decide to place a vascular clamp across the hilum of the right lung. Which of the following structures will lie most anteriorly at this point?

Thoracic duct

Phrenic nerve

Vagus nerve

Pulmonary artery

Pulmonary vein

A

The phrenic nerve lies anteriorly at the root of the right lung.

Lung anatomy

The right lung is composed of 3 lobes divided by the oblique and transverse fissures. The left lung has two lobes divided by the oblique fissure.The apex of both lungs is approximately 4cm superior to the sterno-costal joint of the first rib. Immediately below this is a sulcus created by the subclavian artery.

Peripheral contact points of the lung
Base: diaphragm
Costal surface: corresponds to the cavity of the chest
Mediastinal surface: Contacts the mediastinal pleura. Has the cardiac impression. Above and behind this concavity is a triangular depression named the hilum, where the structures which form the root of the lung enter and leave the viscus. These structures are invested by pleura, which, below the hilum and behind the pericardial impression, forms the pulmonary ligament

Right lung
Above the hilum is the azygos vein; Superior to this is the groove for the superior vena cava and right innominate vein; behind this, and nearer the apex, is a furrow for the innominate artery. Behind the hilum and the attachment of the pulmonary ligament is a vertical groove for the oesophagus; In front and to the right of the lower part of the oesophageal groove is a deep concavity for the extrapericardiac portion of the inferior vena cava.

The root of the right lung lies behind the superior vena cava and the right atrium, and below the azygos vein.

The right main bronchus is shorter, wider and more vertical than the left main bronchus and therefore the route taken by most foreign bodies.

Left lung
Above the hilum is the furrow produced by the aortic arch, and then superiorly the groove accommodating the left subclavian artery; Behind the hilum and pulmonary ligament is a vertical groove produced by the descending aorta, and in front of this, near the base of the lung, is the lower part of the oesophagus.

The root of the left lung passes under the aortic arch and in front of the descending aorta.

Inferior borders of both lungs
6th rib in mid clavicular line
8th rib in mid axillary line
10th rib posteriorly
The pleura runs two ribs lower than the corresponding lung level.

Bronchopulmonary segments
Segment number Right lung Left lung
1 Apical Apical
2 Posterior Posterior
3 Anterior Anterior
4 Lateral Superior lingular
5 Medial Inferior lingular
6 Superior (apical) Superior (apical)
7 Medial basal Medial basal
8 Anterior basal Anterior basal
9 Lateral basal Lateral basal
10 Posterior basal Posterior basal

How well did you know this?
1
Not at all
2
3
4
5
Perfectly
134
Q

What is the lymphatic drainage of the female urethra?

Superficial inguinal nodes

Deep inguinal nodes

Internal iliac nodes

External iliac nodes

Para-aortic nodes

A

The entire female urethra drains to the internal iliac nodes. Urethral anatomy

Female urethra
The female urethra is shorter and more acutely angulated than the male urethra. It is an extra-peritoneal structure and embedded in the endopelvic fascia. The neck of the bladder is subjected to transmitted intra-abdominal pressure and therefore deficiency in this area may result in stress urinary incontinence. Between the layers of the urogenital diaphragm the female urethra is surrounded by the external urethral sphincter, this is innervated by the pudendal nerve. It ultimately lies anterior to the vaginal orifice.

Male urethra
In males the urethra is much longer and is divided into four parts.

Pre-prostatic urethra Extremely short and lies between the bladder and prostate gland.It has a stellate lumen and is between 1 and 1.5cm long.Innervated by sympathetic noradrenergic fibres, as this region is composed of striated muscles bundles they may contract and prevent retrograde ejaculation.
Prostatic urethra This segment is wider than the membranous urethra and contains several openings for the transmission of semen (at the midpoint of the urethral crest).
Membranous urethra Narrowest part of the urethra and surrounded by external sphincter. It traverses the perineal membrane 2.5cm postero-inferior to the symphysis pubis.
Penile urethra Travels through the corpus spongiosum on the underside of the penis. It is the longest urethral segment.It is dilated at its origin as the infrabulbar fossa and again in the glans penis as the navicular fossa. The bulbo-urethral glands open into the spongiose section of the urethra 2.5cm below the perineal membrane.

The urothelium is transitional in nature near to the bladder and becomes squamous more distally.

How well did you know this?
1
Not at all
2
3
4
5
Perfectly
135
Q

Which of the following statements relating to omphalocele is false?

The herniated organs lie outside the peritoneal sac

Cardiac abnormalities co-exist in 25%

The caecum is usually right sided

The defect occurs through the umbilicus

Mortality may be as high as 15%

A

The false statement regarding omphalocele is:

The herniated organs lie outside the peritoneal sac.

Explanation:

Omphalocele is a congenital defect in the abdominal wall where the intestines, liver, and occasionally other organs remain outside the abdomen in a sac because of a failure of the intestines to return to the abdominal cavity during fetal development. The sac is covered by a membrane consisting of peritoneum and amnion.

1.	The herniated organs lie outside the peritoneal sac: This is false. In omphalocele, the herniated organs are contained within a protective sac formed by the peritoneum and amnion.
2.	Cardiac abnormalities co-exist in 25%: This is true. Omphalocele is often associated with other congenital anomalies, including cardiac defects.
3.	The caecum is usually right sided: This is true. The caecum and other abdominal organs are typically in their normal positions within the sac.
4.	The defect occurs through the umbilicus: This is true. Omphalocele occurs through the umbilical ring.
5.	Mortality may be as high as 15%: This is true. Mortality rates can be high, especially if there are associated congenital anomalies and complications.

Therefore, the correct answer is that the statement “The herniated organs lie outside the peritoneal sac” is false.

Gastroschisis: Isolated abnormality, bowel lies outside abdominal wall through defect located to right of umbilicus.
Exomphalos: Liver and gut remain covered with membranous sac connected to umbilical cord. It is associated with other developmental defects.
They are contained within the peritoneal sac and therefore do not have the fluid losses seen in gastroschisis. True malrotation is unusual and minor variants may not result in a requirement for surgery.

Paediatric Gastrointestinal disorders

Pyloric stenosis
M>F
5-10% Family history in parents
Projectile non bile stained vomiting at 4-6 weeks of life
Diagnosis is made by test feed or USS
Treatment: Ramstedt pyloromyotomy (open or laparoscopic)
Acute appendicitis
Uncommon under 3 years
When occurs may present atypically
Mesenteric adenitis
Central abdominal pain and URTI
Conservative management
Intussusception
Telescoping bowel
Proximal to or at the level of, ileocaecal valve
6-9 months age
Colicky pain, diarrhoea and vomiting, sausage shaped mass, red jelly stool.
Treatment: reduction with air insufflation
Malrotation
High caecum at the midline
Feature in exomphalos, congenital diaphragmatic hernia, intrinsic duodenal atresia
May be complicated by development of volvulus, infant with volvulus may have bile stained vomiting
Diagnosis is made by upper GI contrast study and USS
Treatment is by laparotomy, if volvulus is present (or at high risk of occurring then a ladds procedure is performed
Hirschsprung’s disease
Absence of ganglion cells from myenteric and submucosal plexuses
Occurs in 1/5000 births
Full thickness rectal biopsy for diagnosis
Delayed passage of meconium and abdominal distension
Treatment is with rectal washouts initially, thereafter an anorectal pull through procedure
Oesophageal atresia
Associated with tracheo-oesophageal fistula and polyhydramnios
May present with choking and cyanotic spells following aspiration
VACTERL associations
Meconium ileus
Usually delayed passage of meconium and abdominal distension
Majority have cystic fibrosis
X-Rays may not show a fluid level as the meconium is viscid (depends upon feeding), PR contrast studies may dislodge meconium plugs and be therapeutic
Infants who do not respond to PR contrast and NG N-acetyl cysteine will require surgery to remove the plugs
Biliary atresia
Jaundice > 14 days
Increased conjugated bilirubin
Urgent Kasai procedure
Necrotising enterocolitis
Prematurity is the main risk factor
Early features include abdominal distension and passage of bloody stools
X-Rays may show pneumatosis intestinalis and evidence of free air
Increased risk when empirical antibiotics are given to infants beyond 5 days
Treatment is with total gut rest and TPN, babies with perforations will require laparotomy

How well did you know this?
1
Not at all
2
3
4
5
Perfectly
136
Q

A 28 year old man is involved in a road traffic accident and sustains a flail chest injury. On arrival in the emergency department he is hypotensive. On examination; he has an elevated jugular venous pulse and auscultation of the heart reveals quiet heart sounds. What is the most likely diagnosis?

Pneumothorax

Myocardial contusion

Cardiac tamponade

Haemothorax

Ventricular septal defect

A

The presence of a cardiac tamponade is suggested by Becks Triad:
Hypotension
Muffled heart sounds
Raised JVP

Thoracic trauma

Key points related to thoracic trauma
Less than 10% of blunt chest trauma and 15-30% of penetrating chest trauma requires operative intervention.
The physiologic consequences of thoracic trauma are hypoxia, hypercarbia, and acidosis. Contusion, hematoma, and alveolar collapse, or changes in intrathoracic pressure relationships (e.g., tension pneumothorax and open pneumothorax) cause hypoxia and lead to metabolic acidosis. Hypercarbia causes respiratory acidosis and most often follows inadequate ventilation caused by changes in intrathoracic pressure relationships and depressed level of consciousness.

Types of thoracic trauma

Tension pneumothorax
Often laceration to lung parenchyma with flap
Pressure develops in thorax
Most common cause is mechanical ventilation in patient with pleural injury
Symptoms overlap with cardiac tamponade, hyper-resonant percussion note is more likely in tension pnemothorax
Flail chest
Chest wall disconnects from thoracic cage
Multiple rib fractures (at least two fractures per rib in at least two ribs)
Associated with pulmonary contusion
Abnormal chest motion
Avoid over hydration and fluid overload
Pneumothorax
Most common cause is lung laceration with air leakage
Most traumatic pneumothoraces should have a chest drain
Patients with traumatic pneumothorax should never be mechanically ventilated until a chest drain is inserted
Haemothorax
Most commonly due to laceration of lung, intercostal vessel or internal mammary artery
Haemothoraces large enough to appear on CXR are treated with large bore chest drain
Surgical exploration is warranted if >1500ml blood drained immediately
Cardiac tamponade
Beck’s triad: elevated venous pressure, reduced arterial pressure, reduced heart sounds
Pulsus paradoxus
May occur with as little as 100ml blood
Pulmonary contusion
Most common potentially lethal chest injury
Arterial blood gases and pulse oximetry important
Early intubation within an hour if significant hypoxia
Blunt cardiac injury
Usually occurs secondary to chest wall injury
ECG may show features of myocardial infarction
Sequelae: hypotension, arrhythmias, cardiac wall motion abnormalities
Aorta disruption
Deceleration injuries
Contained haematoma
Widened mediastinum
Diaphragm disruption
Most due to motor vehicle accidents and blunt trauma causing large radial tears (laceration injuries result in small tears)
More common on left side
Insert gastric tube, may pass into intrathoracic stomach
Mediastinal traversing wounds
Entrance wound in one hemithorax and exit wound/foreign body in opposite hemithorax
Mediastinal haematoma or pleural cap suggests great vessel injury
Mortality is 20%

How well did you know this?
1
Not at all
2
3
4
5
Perfectly
137
Q

As a busy surgical trainee on the colorectal unit you are given the unenviable task of reviewing the unit’s histopathology results for colonic polyps. Which of the polyp types described below has the greatest risk of malignant transformation?

Hyperplastic polyp

Tubular adenoma

Villous adenoma

Hamartomatous polyp

Pseudopolyp

A

Villous adenomas carry the highest risk of malignant transformation. Hyperplastic polyps carry little in the way of increased risk. Although, patients with hamartomatous polyp syndromes may have a high risk of malignancy, the polyps themselves have little malignant potential.

Colonic polyps

Colonic Polyps
May occur in isolation, or greater numbers as part of the polyposis syndromes. In FAP greater than 100 polyps are typically present. The risk of malignancy in association with adenomas is related to size, and is the order of 10% in a 1cm adenoma. Isolated adenomas seldom give risk of symptoms (unless large and distal). Distally sited villous lesions may produce mucous and if very large, electrolyte disturbances may occur.

Follow up of colonic polyps
Group Action
Colorectal cancer Colonoscopy 1 year post resection
Large non pedunculated colorectal polyps (LNPCP), R0 resection One off scope at 3 years
Large non pedunculated colorectal polyps (LNPCP) R1 or non en bloc resection Site check at 2-6 months and then a further scope at 12 months
High risk findings at baseline colonoscopy One off surveillance at 3 years
No high risk findings at baseline colonoscopy No colonoscopic surveillance and invite participation in NHSBCSP programme when due

High risk findings
More than 2 premalignant polyps including 1 or more advanced colorectal polyps
OR
More than 5 pre malignant polyps

Exceptions to guidelines
If patient more than 10 years younger than lower screening age and has polyps but no high risk findings, consider colonoscopy at 5 or 10 years.

Segmental resection or complete colectomy should be considered when:

  1. Incomplete excision of malignant polyp
  2. Malignant sessile polyp
  3. Malignant pedunculated polyp with submucosal invasion
  4. Polyps with poorly differentiated carcinoma
  5. Familial polyposis coli
    -Screening from teenager up to 40 years by 2 yearly sigmoidoscopy/colonoscopy
    -Panproctocolectomy and Ileostomy or Restorative Panproctocolectomy.

Rectal polypoidal lesions may be amenable to trans anal endoscopic microsurgery.

How well did you know this?
1
Not at all
2
3
4
5
Perfectly
138
Q

A 28 year old man has sustained a non salvageable testicular injury to his left testicle. The surgeon decides to perform an orchidectomy and divides the left testicular artery. From which of the following does this vessel originate?

Abdominal aorta

Internal iliac artery

Inferior epigastric artery

Inferior vesical artery

External iliac artery

A

The testicular artery is a branch of the abdominal aorta. Scrotal and testicular anatomy

Spermatic cord
Formed by the vas deferens and is covered by the following structures:
Layer Origin
Internal spermatic fascia Transversalis fascia
Cremasteric fascia From the fascial coverings of internal oblique
External spermatic fascia External oblique aponeurosis

Contents of the cord
Vas deferens Transmits sperm and accessory gland secretions
Testicular artery Branch of abdominal aorta supplies testis and epididymis
Artery of vas deferens Arises from inferior vesical artery
Cremasteric artery Arises from inferior epigastric artery
Pampiniform plexus Venous plexus, drains into right or left testicular vein
Sympathetic nerve fibres Lie on arteries, the parasympathetic fibres lie on the vas
Genital branch of the genitofemoral nerve Supplies cremaster
Lymphatic vessels Drain to lumbar and para-aortic nodes

Scrotum
Composed of skin and closely attached dartos fascia.
Arterial supply from the anterior and posterior scrotal arteries
Lymphatic drainage to the inguinal lymph nodes
Parietal layer of the tunica vaginalis is the innermost layer

Testes
The testes are surrounded by the tunica vaginalis (closed peritoneal sac). The parietal layer of the tunica vaginalis adjacent to the internal spermatic fascia.
The testicular arteries arise from the aorta immediately inferiorly to the renal arteries.
The pampiniform plexus drains into the testicular veins, the left drains into the left renal vein and the right into the inferior vena cava.
Lymphatic drainage is to the para-aortic nodes.

How well did you know this?
1
Not at all
2
3
4
5
Perfectly
139
Q

A 62 year old lady presents with a non tender lump overlying the distal interphalangeal joint of the index finger. On examination, she has a hard, non tender lump overlying the joint and deviation of the tip of the finger. What is the nature of the lesion?

Oslers nodes

Bouchards nodes

Heberdens nodes

Osteosarcoma

Infective collection

A

Heberdens nodes may produce swelling of the distal interphalangeal joint with deviation of the finger tip.

Hand diseases

Dupuytrens contracture
Fixed flexion contracture of the hand where the fingers bend towards the palm and cannot be fully extended.
Caused by underlying contractures of the palmar aponeurosis . The ring finger and little finger are the fingers most commonly affected. The middle finger may be affected in advanced cases, but the index finger and the thumb are nearly always spared.
Progresses slowly and is usually painless. In patients with this condition, the tissues under the skin on the palm of the hand thicken and shorten so that the tendons connected to the fingers cannot move freely. The palmar aponeurosis becomes hyperplastic and undergoes contracture.
Commonest in males over 40 years of age.
Association with liver cirrhosis and alcoholism. However, many cases are idiopathic.
Treatment is surgical and involves fasciectomy. However, the condition may recur and many surgical therapies are associated with risk of neurovascular damage to the digital nerves and arteries.

Carpal tunnel syndrome
Idiopathic median neuropathy at the carpal tunnel.
Characterised by altered sensation of the lateral 3 fingers.
The condition is commoner in females and is associated with other connective tissue disorders such as rheumatoid disease. It may also occur following trauma to the distal radius.
Symptoms occur mainly at night in early stages of the condition.
Examination may demonstrate wasting of the muscles of the thenar eminence and symptoms may be reproduced by Tinels test (compression of the contents of the carpal tunnel).
Formal diagnosis is usually made by electrophysiological studies.
Treatment is by surgical decompression of the carpal tunnel, a procedure achieved by division of the flexor retinaculum. Non - surgical options include splinting and bracing.

Miscellaneous hand lumps
Osler’s nodes Osler’s nodes are painful, red, raised lesions found on the hands and feet. They are the result of the deposition of immune complexes.
Bouchard’s nodes Hard, bony outgrowths or gelatinous cysts on the proximal interphalangeal joints (the middle joints of fingers or toes.) They are a sign of osteoarthritis, and are caused by formation of calcific spurs of the articular cartilage.
Heberden’s nodes Typically develop in middle age, beginning either with a chronic swelling of the affected joints or the sudden painful onset of redness, numbness, and loss of manual dexterity. This initial inflammation and pain eventually subsides, and the patient is left with a permanent bony outgrowth that often skews the fingertip sideways. It typically affects the DIP joint.
Ganglion Swelling in association with a tendon sheath commonly near a joint. They are common lesions in the wrist and hand. Usually they are asymptomatic and cause little in the way of functional compromise. They are fluid filled although the fluid is similar to synovial fluid it is slightly more viscous. When the cysts are troublesome they may be excised.

How well did you know this?
1
Not at all
2
3
4
5
Perfectly
140
Q

A surgeon is considering using lignocaine as local anasthesia for a minor procedure. Which of the following best accounts for its actions?

Blockade of neuronal acetylcholine receptors

Blockade of neuronal nicotinic receptors

Blockade of neuronal sodium channels

Blockade of neuronal potassium channels

Blockade of neuronal calcium channels

A

Lignocaine blocks sodium channels. They will typically be activated first, hence the pain some patients experience on administration.

Local anaesthetic agents

Lidocaine
An amide
Local anaesthetic and a less commonly used antiarrhythmic (affects Na channels in the axon)
Hepatic metabolism, protein bound, renally excreted
Toxicity: due to IV or excess administration. Increased risk if liver dysfunction or low protein states. Note acidosis causes lidocaine to detach from protein binding.
Drug interactions: Beta blockers, ciprofloxacin, phenytoin
Features of toxicity: Initial CNS over activity then depression as lidocaine initially blocks inhibitory pathways then blocks both inhibitory and activating pathways. Cardiac arrhythmias.
Increased doses may be used when combined with adrenaline to limit systemic absorption.

Cocaine
Pure cocaine is a salt, usually cocaine hydrochloride. It is supplied for local anaesthetic purposes as a paste.
It is supplied for clinical use in concentrations of 4 and 10%. It may be applied topically to the nasal mucosa. It has a rapid onset of action and has the additional advantage of causing marked vasoconstriction.
It is lipophillic and will readily cross the blood brain barrier. Its systemic effects also include cardiac arrhythmias and tachycardia.
Apart from its limited use in ENT surgery it is otherwise used rarely in mainstream surgical practice.

Bupivacaine
Bupivacaine binds to the intracellular portion of sodium channels and blocks sodium influx into nerve cells, which prevents depolarization.
It has a much longer duration of action than lignocaine and this is of use in that it may be used for topical wound infiltration at the conclusion of surgical procedures with long duration analgesic effect.
It is cardiotoxic and is therefore contra indicated in regional blockage in case the tourniquet fails.
Levobupivacaine (Chirocaine) is less cardiotoxic and causes less vasodilation.

Prilocaine
Similar mechanism of action to other local anaesthetic agents. However, it is far less cardiotoxic and is therefore the agent of choice for intravenous regional anaesthesia e.g. Biers Block.

All local anaesthetic agents dissociate in tissues and this contributes to their therapeutic effect. The dissociation constant shifts in tissues that are acidic e.g. where an abscess is present, and this reduces the efficacy.

Doses of local anaesthetics
Agent Dose plain Dose with adrenaline
Lignocaine 3mg/Kg 7mg/Kg
Bupivacaine 2mg/Kg 2mg/Kg
Prilocaine 6mg/Kg 9mg/Kg
These are a guide only as actual doses depend on site of administration, tissue vascularity and co-morbidities.

Maximum total local anaesthetic doses
Lignocaine 1% plain - 3mg/ Kg - 200mg (20ml)
Lignocaine 1% with 1 in 200,000 adrenaline - 7mg/Kg - 500mg (50ml)
Bupivacaine 0.5% - 2mg/kg- 150mg (30ml)
Maximum doses are based on ideal body weight

Effects of adrenaline
Adrenaline may be added to local anaesthetic drugs. It prolongs the duration of action at the site of injection and permits usage of higher doses (see above). It is contra indicated in patients taking MAOI’s or tricyclic antidepressants. The toxicity of bupivacaine is related to protein binding and addition of adrenaline to this drug does not permit increases in the total dose of bupivacaine, in contrast to the situation with lignocaine.

How well did you know this?
1
Not at all
2
3
4
5
Perfectly
141
Q

Which of the following statements is false in relation to consent?

All adults by law are assumed to be competent

The Bolam test defines if a patient has capacity or not

Consent 2 is the form signed by parents on behalf of their children

Implied consent is a form of consent

Consent 4 is the form signed when a patient is unable to consent for a treatment or investigation

A

The Bolam test defines if a decision made by a doctor is in agreement with the professional standard of medical practise.

Consent

There are 3 types of consent:

  1. Informed
  2. Expressed
  3. Implied

Consent forms used in UK NHS
Consent Form 1 For competent adults who are able to consent for themselves where consciousness may be impaired (e.g. GA)
Consent Form 2 For an adult consenting on behalf of a child where consciousness is impaired
Consent Form 3 For an adult or child where consciousness is not impaired
Consent Form 4 For adults who lack capacity to provide informed consent

Capacity
Key points include:
1. Understand and retain information
2. Patient believes the information to be true
3. Patient is able to weigh the information to make a decision
All patients must be assumed to have capacity

Consent in minors
Young children and older children who are not Gillick competent cannot consent for themselves. In British law the patients biological mother can always provide consent. The child’s father can consent if the parents are married (and the father is the biological father), or if the father is named on the birth certificate (irrespective of marital status). If parents are not married and the father is not named on the birth certificate then the father cannot consent.

How well did you know this?
1
Not at all
2
3
4
5
Perfectly
142
Q

A well toned weight lifter attends clinic reporting weakness of his left arm. There is weakness of flexion and supination of the forearm. Which of the following nerves is likely to have been affected?

Axillary

Suprascapular

Median

Radial

Musculocutaneous

A

Musculocutaneous nerve compression due to entrapment of the nerve between biceps and brachialis. Elbow flexion and supination of the arm are affected. This is a rare isolated injury.

Musculocutaneous nerve

Branch of lateral cord of brachial plexus

Path
It penetrates the coracobrachialis muscle
Passes obliquely between the biceps brachii and the brachialis to the lateral side of the arm
Above the elbow it pierces the deep fascia lateral to the tendon of the biceps brachii
Continues into the forearm as the lateral cutaneous nerve of the forearm

Innervates
Coracobrachialis
Biceps brachii
Brachialis

How well did you know this?
1
Not at all
2
3
4
5
Perfectly
143
Q

A 32 year old rugby player is crushed in a scrum. He is briefly concussed but then regains consciousness. He then collapses and is brought to ED. His GCS on arrival is 6/15 and his left pupil is dilated. What is the best course of definitive management?

Burr Hole decompression

Parietotemporal craniotomy

Intravenous mannitol

Posterior fossa craniotomy

Insertion of intra cranial pressure bolt monitor

A

This man needs urgent decompression and extradural haematoma is the most likely event, from a lacerated middle meningeal artery. Most neurosurgeons would perform a craniotomy. However, rural units and those units without neurosurgical kit facing this emergency may resort to Burr Holes.

Head injury management- NICE Guidelines

Summary of guidelines
All patients should be assessed within 15 minutes on arrival to A&E
Document all 3 components of the GCS
If GCS <8 or = to 8, consider stabilising the airway
Full spine immobilisation until assessment if:

  • GCS < 15
  • neck pain/tenderness
  • paraesthesia extremities
  • focal neurological deficit
  • suspected c-spine injury

If a c-spine injury is suspected a 3 view c-spine x-ray is indicated. CT c-spine is preferred if:
- Intubated
- GCS <13
- Normal x-ray but continued concerns regarding c-spine injury
- Any focal neurology
- A CT head scan is being performed
- Initial plain films are abnormal

Immediate CT head (within 1 hour) if:
GCS < 12 on admission
GCS < 15 2 hours after admission
Suspected open or depressed skull fracture
Suspected skull base fracture (panda eyes, Battle’s sign, CSF from nose/ear, bleeding ear)
Focal neurology
Vomiting > 1 episode
Post traumatic seizure

Contact neurosurgeon if:
Persistent GCS < 8 or = 8
Unexplained confusion > 4h
Reduced GCS after admission
Progressive neurological signs
Incomplete recovery post seizure
Penetrating injury
Cerebrospinal fluid leak

Observations
1/2 hourly GCS until 15

In the latest guidance, there is a move to considering CT in those on anticoagulants rather than mandating it.

How well did you know this?
1
Not at all
2
3
4
5
Perfectly
144
Q

An orthopaedic surgeon makes a modification to an operative approach for total knee arthroplasty. After he has completed 25 cases, he stops and reviews his patient outcomes. He publishes the data. What level of evidence is supplied by this type of data?

II

IV

III

V

I

A

Case series that are non randomised and lack concurrent controls at best supply level IV evidence only. To qualify for level I and II evidence a prospective randomised controlled trial with appropriate blinding, control matching and power calculations is needed.
Levels of evidence

The level of evidence refers to the study design used by investigators to minimise bias.

Level of evidence Source
I Evidence obtained from systematic review of all relevant randomised controlled trials
II Evidence derived from at least one properly designed randomised controlled trial
III Evidence derived from well designed pseudo-randomised controlled trials (e.g. alternate allocation) or historical controls
IV Evidence derived from case series or case reports
V Panel or expert opinion

Many of the categories contain sub groups, detailed knowledge of these are not required for MRCS Part A.

How well did you know this?
1
Not at all
2
3
4
5
Perfectly
145
Q

A 30 year old man is admitted overnight, following a road traffic accident. He has an open tibial fracture with a 20 cm wound and extensive periosteal stripping. He is neurovascularly intact; IV antibiotics and wound dressing have been administered in the emergency department. What is the most appropriate course of action?

Immediate skeletal stabilisation and application of negative pressure dressing

Skeletal fixation followed by vascular reconstruction

Immediate vascuIar shunting, followed by temporary skeletal stabilisation and vascular reconstruction

Combined skeletal and soft tissue reconstruction on a scheduled operating list

Fasciotomy with four compartment decompression

A

This patient has a Gustillo-Anderson Grade 3B open fracture. He will require definitive skeletal and soft tissue reconstruction, which should be performed on a combined ortho-plastic scheduled operating list, as per the BOA/BAPRAS guidelines. The surgery does not have to be performed out of scheduled hours unless there is marine/ sewage contamination, vascular compromise or it is a polytrauma.
Whilst it is reasonable to apply an external fixator prior to definitive skeletal and soft tissue reconstruction, this should be converted to internal fixation within 72 hours.

Open fractures

The term open fracture refers to a disruption of the bony cortex associated with a breach in the overlying skin. Any wound that is present in the same limb as a fracture should be suspected as being representative of an open fracture. One of the main problems with open fractures is the associated injuries to the surrounding soft tissues. Whilst the skin is usually relatively resistant to trauma, underlying muscle can be damaged or devitalised, nerves, blood vessels and periosteum may all be disrupted the degree to which this occurs correlates with the severity of the injury and the outcome. These can be graded using the Gustilo and Anderson system (see below).

Grade Injury
1 Low energy wound <1cm
2 Greater than 1cm wound with moderate soft tissue damage
3 High energy wound > 10cm with extensive soft tissue damage
3 A (sub group of 3) Adequate soft tissue coverage
3 B (sub group of 3) Inadequate soft tissue coverage
3 C (sub group of 3) Associated arterial injury

In Type IIIc injuries, the mangled extremity scoring system (MESS) can help to predict the need for primary amputation.

Key points for initial management
Remove obvious contaminants from very contaminated wounds in the ED
Wound lavage should usually be undertaken in the OR
Early wound photography should be performed
Consider transfer of complex cases to centres that provide orthoplastic care
All wounds should be managed within 24 hours and high velocity ones within 12 hours, those with vascular compromise should be managed immediately
CT angiography is useful in delineating the extent of concommitant vascular injury

How well did you know this?
1
Not at all
2
3
4
5
Perfectly
146
Q

A man undergoes a high anterior resection for carcinoma of the upper rectum. Which of the following vessels will require ligation?

Superior mesenteric artery

Inferior mesenteric artery

Coeliac axis

Perineal artery

Middle colic artery

A

The IMA is usually divided during anterior resection. Not only is this borne out of oncological necessity but it also permits sufficient colonic mobilisation for anastomosis.

Colon anatomy

The colon commences with the caecum. This represents the most dilated segment of the human colon and its base (which is intraperitoneal) is marked by the convergence of teniae coli. At this point is located the vermiform appendix. The colon continues as the ascending colon, the posterior aspect of which is retroperitoneal. The line of demarcation between the intra and retro peritoneal right colon is visible as a white line, in the living, and forms the line of incision for colonic resections.

The ascending colon becomes the transverse colon after passing the hepatic flexure. At this location the colon becomes wholly intra peritoneal once again. The superior aspect of the transverse colon is the point of attachment of the transverse colon to the greater omentum. This is an important anatomical site since division of these attachments permits entry into the lesser sac. Separation of the greater omentum from the transverse colon is a routine operative step in both gastric and colonic resections.

At the left side of the abdomen the transverse colon passes to the left upper quadrant and makes an oblique inferior turn at the splenic flexure. Following this, the posterior aspect becomes retroperitoneal once again.

At the level of approximately L4 the descending colon becomes wholly intraperitoneal and becomes the sigmoid colon. Whilst the sigmoid is wholly intraperitoneal there are usually attachments laterally between the sigmoid and the lateral pelvic sidewall. These small congenital adhesions are not formal anatomical attachments but frequently require division during surgical resections.

At its distal end the sigmoid passes to the midline and at the region around the sacral promontary it becomes the upper rectum. This transition is visible macroscopically as the point where the teniae fuse. More distally the rectum passes through the peritoneum at the region of the peritoneal reflection and becomes extraperitoneal.

Arterial supply
Superior mesenteric artery and inferior mesenteric artery: linked by the marginal artery.
Ascending colon: ileocolic and right colic arteries
Transverse colon: middle colic artery
Descending and sigmoid colon: inferior mesenteric artery

Venous drainage
From regional veins (that accompany arteries) to superior and inferior mesenteric vein

Lymphatic drainage
Initially along nodal chains that accompany supplying arteries, then para-aortic nodes.

Embryology
Midgut- Second part of duodenum to 2/3 transverse colon
Hindgut- Distal 1/3 transverse colon to anus

Peritoneal location
The right and left colon are part intraperitoneal and part extraperitoneal. The sigmoid and transverse colon are generally wholly intraperitoneal. This has implications for the sequelae of perforations, which will tend to result in generalised peritonitis in the wholly intra peritoneal segments.

Colonic relations
Region of colon Relation
Caecum/ right colon Right ureter, gonadal vessels
Hepatic flexure Gallbladder (medially)
Splenic flexure Spleen and tail of pancreas
Distal sigmoid/ upper rectum Left ureter
Rectum Ureters, autonomic nerves, seminal vesicles, prostate, urethra (distally)

How well did you know this?
1
Not at all
2
3
4
5
Perfectly
147
Q

A 78 year old lady is admitted with a 3 hour history of passage of dark red blood per rectum. Prior to this event, she was otherwise well with no major medical co-morbidities. On examination, she has a mild tachycardia but other vital signs are normal, abdomen is soft and non tender. Digital rectal exam reveals dark blood but no other findings. What is the most likely underlying cause?

Diverticular disease

Meckels diverticulum

Jejunal diverticulosis

Angiodysplasia of the colon

Colonic cancer

A

Diverticular disease is the commonest cause of lower GI bleeding in adults.
Diverticular associated bleeding is the commonest cause of lower GI bleeding in older adults and it is important to note that it is not associated with diverticulitis which seldom bleeds. Most cases will stop bleeding spontaneously.
Lower Gastrointestinal bleeding

Colonic bleeding
This typically presents as bright red or dark red blood per rectum. Colonic bleeding rarely presents as malaena type stool, this is because blood in the colon has a powerful laxative effect and is rarely retained long enough for transformation to occur and because the digestive enzymes present in the small bowel are not present in the colon. Up to 15% of patients presenting with haemochezia will have an upper gastrointestinal source of haemorrhage.

As a general rule right sided bleeds tend to present with darker coloured blood than left sided bleeds. Haemorrhoidal bleeding typically presents as bright red rectal bleeding that occurs post defecation either onto toilet paper or into the toilet pan. It is very unusual for haemorrhoids alone to cause any degree of haemodynamic compromise.

Causes
Cause Presenting features
Colitis Bleeding may be brisk in advanced cases, diarrhoea is commonly present. Abdominal x-ray may show featureless colon.
Diverticular disease Acute diverticulitis often is not complicated by major bleeding and diverticular bleeds often occur sporadically. 75% all will cease spontaneously within 24-48 hours. Bleeding is often dark and of large volume.
Cancer Colonic cancers often bleed and for many patients this may be the first sign of the disease. Major bleeding from early lesions is uncommon
Haemorrhoidal bleeding Typically bright red bleeding occurring post defecation. Although patients may give graphic descriptions bleeding of sufficient volume to cause haemodynamic compromise is rare.
Angiodysplasia Apart from bleeding, which may be massive, these arteriovenous lesions cause little in the way of symptoms. The right side of the colon is more commonly affected.

Management
Prompt correction of any haemodynamic compromise is required. Unlike upper gastrointestinal bleeding the first line management is usually supportive. This is because in the acute setting endoscopy is rarely helpful.
When haemorrhoidal bleeding is suspected a proctosigmoidoscopy is reasonable as attempts at full colonoscopy are usually time consuming and often futile.
In the unstable patient the usual procedure would be an angiogram (either CT or percutaneous), when these are performed during a period of haemodynamic instability they may show a bleeding point and may be the only way of identifying a patch of angiodysplasia.
In others who are more stable the standard procedure would be a colonoscopy in the elective setting. In patients undergoing angiography attempts can be made to address the lesion in question such as coiling. Otherwise surgery will be necessary.
In patients with ulcerative colitis who have significant haemorrhage the standard approach would be a sub total colectomy, particularly if medical management has already been tried and is not effective.

Indications for surgery
Patients > 60 years
Continued bleeding despite endoscopic intervention
Recurrent bleeding
Known cardiovascular disease with poor response to hypotension

Surgery
Selective mesenteric embolisation if life threatening bleeding. This is most helpful if conducted during a period of relative haemodynamic instability. If all haemodynamic parameters are normal then the bleeding is most likely to have stopped and any angiography normal in appearance. In many units a CT angiogram will replace selective angiography but the same caveats will apply.

If the source of colonic bleeding is unclear; perform a laparotomy, on table colonic lavage and following this attempt a resection. A blind sub total colectomy is most unwise, for example bleeding from an small bowel arterio-venous malformation will not be treated by this manoeuvre.

Summary of Acute Lower GI bleeding recommendations
Consider admission if:
* Over 60 years
* Haemodynamically unstable/profuse PR bleeding
* On aspirin or NSAID
* Significant co morbidity

Management
All patients should have a history and examination, PR and proctoscopy
Colonoscopic haemostasis aimed for in post polypectomy or diverticular bleeding

How well did you know this?
1
Not at all
2
3
4
5
Perfectly
148
Q

A 25-year-old female with a history of bilateral vitreous haemorrhage is referred with bilateral lesions in the cerebellar region. What is the likely diagnosis?

Neurofibromatosis type I

Neurofibromatosis type II

Tuberous sclerosis

Von Hippel-Lindau syndrome

Sarcoidosis

A

Retinal and cerebellar haemangiomas are key features of Von Hippel-Lindau syndrome. Retinal haemangiomas are bilateral in 25% of patients and may lead to vitreous haemorrhage

Von Hippel-Lindau syndrome

Von Hippel-Lindau (VHL) syndrome is an autosomal dominant condition predisposing to neoplasia. It is due to an abnormality in the VHL gene located on short arm of chromosome 3

Features
cerebellar haemangiomas
retinal haemangiomas: vitreous haemorrhage
renal cysts (premalignant)
phaeochromocytoma
extra-renal cysts: epididymal, pancreatic, hepatic
endolymphatic sac tumours

How well did you know this?
1
Not at all
2
3
4
5
Perfectly
149
Q

A 45 year old man undergoes a surgical excision of a carotid body tumour. Histological analysis is most likely to demonstrate which of the following?

Paraganglionoma

Fibromatosis colli

Schwannoma

Lipoma

Neuroma

A

Carotid body tumours are the commonest type of head and neck paraganglionoma.

Carotid body tumour

These are rare tumours. However, they typically account for around 60% of head and neck paraganglionomas. They are usually tumours of middle age. Around 5% are bilateral and around 5% are malignant. They are rarely found as part of the MEN II or neurofibromatosis type I disease complexes.

They typically present as an asymptomatic neck mass in the anterior triangle of the neck. They are typically slow growing lesions.

Types of carotid body tumour
Sporadic - Accounts for 85% of cases
Familial - Seen in around 10% of cases and usually in younger patients
Hyperplastic - Seen in those at high altitude or in those with COPD

Imaging
They are readily imaged using duplex ultrasonography. CT angiography is sometimes helpful.

Treatment
Typically this comprises surgical resection. This is preceded by embolization in selected cases.

How well did you know this?
1
Not at all
2
3
4
5
Perfectly
150
Q

A 43 year old man has a nasogastric tube inserted. The nurse takes a small aspirate of the fluid from the stomach and tests the pH of the aspirate. What is the normal intragastric pH?

0.5

2

4

5

6

A

The intragastric pH is usually 2. Administration of proton pump inhibitors can result in almost complete abolition of acidity

Gastric secretions

A working knowledge of gastric secretions is important for surgery because peptic ulcers are common, surgeons frequently prescribe anti secretory drugs and because there are still patients around who will have undergone acid lowering procedures (Vagotomy) in the past.

Gastric acid
Is produced by the parietal cells in the stomach
pH of gastric acid is around 2 with acidity being maintained by the H+/K+ ATP ase pump. As part of the process bicarbonate ions will be secreted into the surrounding vessels.
Sodium and chloride ions are actively secreted from the parietal cell into the canaliculus. This sets up a negative potential across the membrane and as a result sodium and potassium ions diffuse across into the canaliculus.
Carbonic anhydrase forms carbonic acid which dissociates and the hydrogen ions formed by dissociation leave the cell via the H+/K+ antiporter pump. At the same time sodium ions are actively absorbed. This leaves hydrogen and chloride ions in the canaliculus these mix and are secreted into the lumen of the oxyntic gland.

Phases of gastric acid secretion
There are 3 phases of gastric secretion:

  1. Cephalic phase (smell / taste of food)
    30% acid produced
    Vagal cholinergic stimulation causing secretion of HCL and gastrin release from G cells
  2. Gastric phase (distension of stomach )
    60% acid produced
    Stomach distension/low H+/peptides causes Gastrin release
  3. Intestinal phase (food in duodenum)
    10% acid produced
    High acidity/distension/hypertonic solutions in the duodenum inhibits gastric acid secretion via enterogastrones (CCK, secretin) and neural reflexes.

Regulation of gastric acid production
Factors increasing production include:
Vagal nerve stimulation
Gastrin release
Histamine release (indirectly following gastrin release) from enterchromaffin like cells

Factors decreasing production include:
Somatostatin (inhibits histamine release)
Cholecystokinin
Secretin

The diagram below illustrates some of the factors involved in regulating gastric acid secretion and the relevant associated pharmacology

Below is a brief summary of the major hormones involved in food digestion:

Source Stimulus Actions
Gastrin
G cells in antrum of the stomach
Distension of stomach, extrinsic nerves
Inhibited by: low antral pH, somatostatin
Increase HCL, pepsinogen and IF secretion, increases gastric motility, trophic effect on gastric mucosa
CCK
I cells in upper small intestine
Partially digested proteins and triglycerides
Increases secretion of enzyme-rich fluid from pancreas, contraction of gallbladder and relaxation of sphincter of Oddi, decreases gastric emptying, trophic effect on pancreatic acinar cells, induces satiety
Secretin
S cells in upper small intestine
Acidic chyme, fatty acids
Increases secretion of bicarbonate-rich fluid from pancreas and hepatic duct cells, decreases gastric acid secretion, trophic effect on pancreatic acinar cells
VIP
Small intestine, pancreas
Neural
Stimulates secretion by pancreas and intestines, inhibits acid and pepsinogen secretion
Somatostatin
D cells in the pancreas and stomach
Fat, bile salts and glucose in the intestinal lumen
Decreases acid and pepsin secretion, decreases gastrin secretion, decreases pancreatic enzyme secretion, decreases insulin and glucagon secretion
inhibits trophic effects of gastrin, stimulates gastric mucous production

How well did you know this?
1
Not at all
2
3
4
5
Perfectly
151
Q

Which of the following structures lie between the lateral and medial heads of the triceps muscle?

Radial nerve

Median nerve

Ulnar nerve

Axillary nerve

Medial cutaneous nerve of the forearm

A

The radial nerve runs in its groove on between the two heads. The ulnar nerve lies anterior to the medial head. The axillary nerve passes through the quadrangular space. This lies superior to lateral head of the triceps muscle and thus the lateral border of the quadrangular space is the humerus. Therefore the correct answer is the radial nerve.

Triceps

Origin
Long head- infraglenoid tubercle of the scapula.
Lateral head- dorsal surface of the humerus, lateral and proximal to the groove of the radial nerve
Medial head- posterior surface of the humerus on the inferomedial side of the radial groove and both of the intermuscular septae
Insertion
Olecranon process of the ulna. Here the olecranon bursa is between the triceps tendon and olecranon.
Some fibres insert to the deep fascia of the forearm, posterior capsule of the elbow (preventing the capsule from being trapped between olecranon and olecranon fossa during extension)
Innervation Radial nerve
Blood supply Profunda brachii artery
Action Elbow extension. The long head can adduct the humerus and extend it from a flexed position
Relations The radial nerve and profunda brachii vessels lie between the lateral and medial heads

How well did you know this?
1
Not at all
2
3
4
5
Perfectly
152
Q

Which of the following is not typically seen in patients with a femoral neck fracture?

Malunion

Non union

Avascular necrosis

Shortening

External rotation

A

Malunion would be unusual with a femoral neck fracture. Because it is a weight bearing joint, if the fracture is not united then it does not heal at all. It is for this reason that most femoral neck fractures are fixed. Avascular necrosis is a well recognised complication and a total hip replacement or hemiarthroplasty is usually considered in the elderly.
Hip fractures

Background
Neck of femur (NOF) fracture is a common orthopaedic presentation, with over 65000 fractures in the UK per year. Like many orthopaedic injuries, there is a bimodal age distribution. It is imperative to distinguish between the high energy injury in a young patient, and the low energy osteoporotic fracture in the elderly, as their management aims are very different:

Young patient - Usually high energy trauma (e.g road traffic accident, horse riding) and needs treating in accordance with Advanced Trauma Life Support (ATLS) principles. Will often have associated injuries. Aim is to retain the patients own anatomy, and optimise their function.

Elderly patient - Predominantly female, fall from standing height (fragility fracture). Often patients have multiple comorbidities that will ultimately dictate their prognosis. Aim of orthopaedic treatment is to immediately regain patient mobility so that morbidity (infection, thromboembolic events, pressure sores etc) and mortality associated with prolonged bed rest is avoided. Left untreated, a neck of femur fracture can be considered a terminal event. Historically, mortality associated with elderly hip fracture is 10% at one month, and 30% at one year. However, this has been improved in the UK with the introduction of multidisciplinary, orthogeriatric lead care and the National Hip Fracture Database and Best Practice Tariff.

Pertinent anatomy
Osteology - normal neck-shaft angle is 130 +/- 7 degrees, and 10 +/- 7 degrees of neck anteversion.
Vascular supply - The predominant blood supply to the femoral head and neck is from the medial and lateral femoral circumflex arteries (branches of profunda femoris). These anastomose and pierce the joint capsule at the base of the neck, mainly posteriorly. There is a small vascular contribution from the artery of the ligament teres. Understanding the blood supply is fundamental to the decision making process in treating NOF fractures.

Presentation and initial management
Typically, patients present with pain in the hip/groin, a shortened, abducted, externally rotated leg (due to the unopposed pull of the muscles that act across the hip joint) and the inability to straight-leg-raise. With undisplaced fractures, signs are more subtle.
High energy injuries should be treated in line with ATLS principles. All patients should be fluid resuscitated, have adequate pain relief (often with a fascio-iliiaca nerve block), and be optimised for surgery. In addition, elderly patients should be assessed by an orthogeriatrician.

Imaging
Anteroposterior and cross-table lateral plain radiographs are sufficient to diagnose the majority of NOF fractures. If the fracture extends below the level of the lesser trochanter, or there is any possibility of pathological fracture, full length femur views are essential to plan surgery.

Where there is a high index of suspicion of fracture, but plain radiographs are inconclusive, gold standard investigation is MRI. However, if unavailable within 24 hours, or if the patient will not tolerate MRI, CT is appropriate. The majority of fractures can be seen with modern CT techniques, and so this is becoming first line in many hospitals.

Classification
There has been a move away from named classification systems towards descriptive classification systems.
Two main types of NOF exist: Intra-capsular, and extra-capsular. Extra-capsular fractures are further divided into pertrochanteric or subtrochanteric (within 5cm distal to the lesser trochanter). All fractures are then described as undisplaced, minimally displaced, or displaced.
Femoral neck and head blood supply disruption is common with intracapsular NOF fractures, and rare with extracapsular fractures. This fundamental principle underpins the practise of arthroplasty for intracapsular fractures, and fixation for extracapsular fractures.

If you wish to use a named classification system, the most commonly used are below:
Elderly intracapsular - Garden Classification
Young intrasapsular - Pauvels Classification
Intertrochanteric - Evans
Subtrochanteric - Russell Taylor

Treatment
In general, NOF fractures are treated operatively except if the patient is deemed unlikely to survive an anaesthetic. Best Practice Tarif (BPT) dictates that surgery should happen within 36 hours, as delay of greater than 48 hours is associated with increased morbidity and mortality. Below are suggested algorithms for the treatment of NOF. There are some areas of debate/controversy which are detailed below.

  • The priority with the young patient is to retain the femoral head if possible, even with a displaced intracapsular fracture. The risk of avascular necrosis and non-union (and therefore revision surgery) associated with internal fixation needs weighing up against the sequelae of total hip replacement in the young (wear, dislocation, revision). Discussion is necessary with the patient, on a case by case basis.

** Undisplaced fractures in the elderly can be treated with internal fixation, often with cannulated screws. This is appropriate for valgus impacted subcapital fractures which are inherently stable, to prevent secondary displacement. This does still carry the risk of AVN or non-union, and therefore a future revision. For this reason, many surgeons advocate arthroplasty as a single surgery.

*** NICE guidance - patients who fulfil these criteria should be offered total hip replacement which conveys better function and prosthetic survivorship, compared with hemiarthroplasty, but at an increased risk of dislocation.

  • Intertrochanteric fractures vary greatly in their stability. If the trochanter (and therefore lateral wall), and medial calcar is in tact, then the fracture configuration bears stability. This can be treated with a DHS, as collapse of the fracture is predictable. Where either or both structures are involved in the fracture, stability becomes compromised and many surgeons will favour using an intramedullary device. This is an ongoing debate, and difficult to test in an exam setting.

Post operative management
Patients should be mobilised fully weight bearing where possible. Care is multidisciplinary in its delivery. Elderly patients should have orthogeriatrician assessment of comorbidity, and bone health with secondary prevention measures if appropriate. There should be early involvement of physiotherapy and occupational therapy services. For further guidance see sources listed below.

How well did you know this?
1
Not at all
2
3
4
5
Perfectly
153
Q

A 16 year old boy develops a painful swelling of his distal femur. An osteoblastic sarcoma is diagnosed. To which of the following sites is this lesion most likely to metastasise?

Inguinal lymph nodes

Common iliac lymph nodes

Liver

Brain

Lung

A

Sarcomas in which Lymphatic Metastasis is seen:

‘RACE For MS’

R: Rhabdomyosarcoma
A: Angiosarcoma
C: Clear cell sarcoma
E: Epithelial cell sarcoma

For: Fibrosarcoma

M: Malignant fibrous histiocytoma
S: Synovial cell sarcoma

Or

‘SCARE’

Synovial sarcoma
Clear cell sarcoma
Angiosarcoma
Rhabdomyosarcoma
Epithelioid sarcoma
Sarcomas often metastasise via the haematogenous route and the lung is a common site for sarcoma metastasis. The liver and brain are often spared (at least initially). A smaller number may develop lymphatic metastasis

Sarcomas

Malignant tumours of mesenchymal origin

Types
May be either bone or soft tissue in origin.
Bone sarcoma include:
Osteosarcoma
Ewings sarcoma (although non bony sites recognised)
Chondrosarcoma - originate from Chondrocytes

Soft tissue sarcoma are a far more heterogeneous group and include:
Liposarcoma-adipocytes
Rhabdomyosarcoma-striated muscle
Leiomyosarcoma-smooth muscle
Synovial sarcomas- close to joints (cell of origin not known but not synovium)

Malignant fibrous histiocytoma is now referred to as undifferentiated pleomorphic sarcoma. Careful histological assessment of lesions now allows more accurate categorisation of sarcoma subtypes than was previously possible.

Features
Certain features of a mass or swelling should raise suspicion for a sarcoma these include:
Large >5cm soft tissue mass
Deep tissue location or intra muscular location
Rapid growth
Painful lump

Assessment
Imaging of suspicious masses should utilise a combination of MRI, CT and USS. Blind biopsy should not be performed prior to imaging and where required should be done in such a way that the biopsy tract can be subsequently included in any resection.

Ewings sarcoma
Commoner in males
Incidence of 0.3 / 1, 000, 000
Onset typically between 10 and 20 years of age
Location by femoral diaphysis is commonest site
Histologically it is a small round tumour
Blood borne metastasis is common and chemotherapy is often combined with surgery

Osteosarcoma
Mesenchymal cells with osteoblastic differentiation
20% of all primary bone tumours
Incidence of 5 per 1,000,000
Peak age 15-30, commoner in males
Limb preserving surgery may be possible and many patients will receive chemotherapy

Liposarcoma
Malignancy of adipocytes
Rare, approximately 2.5 per 1,000,000. They are the most common soft tissue sarcoma
Typically located in deep locations such as retroperitoneum
Affect older age group usually >40 years of age
May be well differentiated and thus slow growing although may undergo de-differentiation and disease progression
Many tumours will have a pseudocapsule that can misleadingly allow surgeons to feel that they can ‘shell out’ these lesions. In reality, tumour may invade at the edge of the pseudocapsule and result in local recurrence if this strategy is adopted
Usually resistant to radiotherapy, although this is often used in a palliative setting

Malignant Fibrous Histiocytoma
Tumour with large number of histiocytes
Also described as undifferentiated pleomorphic sarcoma NOS (i.e. Cell of origin is not known)
Four major subtypes are recognised: storiform-pleomorphic (70% cases), myxoid (less aggressive), giant cell and inflammatory
Treatment is usually with surgical resection and adjuvant radiotherapy as this reduces the likelihood of local recurrence

How well did you know this?
1
Not at all
2
3
4
5
Perfectly
154
Q

A 63 year old man is recovering following an open extended right hemicolectomy for carcinoma of the colonic splenic flexure. Two days post operatively he develops a persistent pyrexia. What is the least likely cause?

Ileus

Aspiration pneumonia

Anastomotic leak

Wound infection

Urinary tract infection

A

An ileus in itself is seldom a cause of a pyrexia. It may serve as a proxy marker of other complications. In this scenario atelectasis would be the most likely underlying cause, as open extended right hemicolectomies will necessitate a long midline incision. Anastomotic leaks are less common after right sided colonic surgery and the timeframe for it is rather short (but are possible). Both wound infections and UTI’s ,may complicate major abdominal surgery at any stage. We remind you to check the wording of the question, it asks for the ‘least likely’ cause of pyrexia.

Pyrexia- post operative

Many surgical patients will develop a pyrexia post operatively. The cause and investigation depends upon the nature of the infection.
The following scenarios may account for post operative pyrexia:
Cause Features
Anastomotic leak
Swinging pyrexia
Ileus
Increasing abdominal pain
Raised inflammatory markers
Wound infection
Evidence of superficial erythema, discharge of pus or increasing pain
Usually mild pyrexia (unless major or deep seated wound infection)
May be accompanied by evidence of wound dehisence
Inflammatory markers raised
Atelectasis
Usually complicates abdominal surgery
Most common after midline laparotomies (pain impairs ventilation)
Pyrexia usually mild and non swinging
Most patients will have chest signs on examination
Inflammatory markers raised
Central line sepsis
Patients with complex venous access
May have marked pyrexia
Access site may show evidence of erythema
Diagnosis is by blood culture from line, line removal and subsequent tip culture
Groin lines and those for TPN have the highest risk
Inflammatory markers raised
Urinary tract infection
Common in surgical patients
Usually occur in patients with indwelling urinary catheters
Diagnosis is by dipstick and CSU and signs of raised inflammatory markers
Treatment is with antibiotics (to cover hospital acquired organisms)

How well did you know this?
1
Not at all
2
3
4
5
Perfectly
155
Q

An otherwise fit 74 year old man presents with pain in the right hip following minimal trauma. On examination, his leg is shortened and externally rotated. Plain films demonstrate a displaced intracapsular fracture of the femoral neck. What is the best course of action?

Uncemented hemi arthroplasty

Dynamic hip screw

Total hip replacement

Intra medullary nail

Percutaneous pins

A

In otherwise fit patients aged over 70, the best long term functional outcomes are obtained with total hip arthroplasty.

Hip fractures

Background
Neck of femur (NOF) fracture is a common orthopaedic presentation, with over 65000 fractures in the UK per year. Like many orthopaedic injuries, there is a bimodal age distribution. It is imperative to distinguish between the high energy injury in a young patient, and the low energy osteoporotic fracture in the elderly, as their management aims are very different:

Young patient - Usually high energy trauma (e.g road traffic accident, horse riding) and needs treating in accordance with Advanced Trauma Life Support (ATLS) principles. Will often have associated injuries. Aim is to retain the patients own anatomy, and optimise their function.

Elderly patient - Predominantly female, fall from standing height (fragility fracture). Often patients have multiple comorbidities that will ultimately dictate their prognosis. Aim of orthopaedic treatment is to immediately regain patient mobility so that morbidity (infection, thromboembolic events, pressure sores etc) and mortality associated with prolonged bed rest is avoided. Left untreated, a neck of femur fracture can be considered a terminal event. Historically, mortality associated with elderly hip fracture is 10% at one month, and 30% at one year. However, this has been improved in the UK with the introduction of multidisciplinary, orthogeriatric lead care and the National Hip Fracture Database and Best Practice Tariff.

Pertinent anatomy
Osteology - normal neck-shaft angle is 130 +/- 7 degrees, and 10 +/- 7 degrees of neck anteversion.
Vascular supply - The predominant blood supply to the femoral head and neck is from the medial and lateral femoral circumflex arteries (branches of profunda femoris). These anastomose and pierce the joint capsule at the base of the neck, mainly posteriorly. There is a small vascular contribution from the artery of the ligament teres. Understanding the blood supply is fundamental to the decision making process in treating NOF fractures.

Presentation and initial management
Typically, patients present with pain in the hip/groin, a shortened, abducted, externally rotated leg (due to the unopposed pull of the muscles that act across the hip joint) and the inability to straight-leg-raise. With undisplaced fractures, signs are more subtle.
High energy injuries should be treated in line with ATLS principles. All patients should be fluid resuscitated, have adequate pain relief (often with a fascio-iliiaca nerve block), and be optimised for surgery. In addition, elderly patients should be assessed by an orthogeriatrician.

Imaging
Anteroposterior and cross-table lateral plain radiographs are sufficient to diagnose the majority of NOF fractures. If the fracture extends below the level of the lesser trochanter, or there is any possibility of pathological fracture, full length femur views are essential to plan surgery.

Where there is a high index of suspicion of fracture, but plain radiographs are inconclusive, gold standard investigation is MRI. However, if unavailable within 24 hours, or if the patient will not tolerate MRI, CT is appropriate. The majority of fractures can be seen with modern CT techniques, and so this is becoming first line in many hospitals.

Classification
There has been a move away from named classification systems towards descriptive classification systems.
Two main types of NOF exist: Intra-capsular, and extra-capsular. Extra-capsular fractures are further divided into pertrochanteric or subtrochanteric (within 5cm distal to the lesser trochanter). All fractures are then described as undisplaced, minimally displaced, or displaced.
Femoral neck and head blood supply disruption is common with intracapsular NOF fractures, and rare with extracapsular fractures. This fundamental principle underpins the practise of arthroplasty for intracapsular fractures, and fixation for extracapsular fractures.

If you wish to use a named classification system, the most commonly used are below:
Elderly intracapsular - Garden Classification
Young intrasapsular - Pauvels Classification
Intertrochanteric - Evans
Subtrochanteric - Russell Taylor

Treatment
In general, NOF fractures are treated operatively except if the patient is deemed unlikely to survive an anaesthetic. Best Practice Tarif (BPT) dictates that surgery should happen within 36 hours, as delay of greater than 48 hours is associated with increased morbidity and mortality. Below are suggested algorithms for the treatment of NOF. There are some areas of debate/controversy which are detailed below.

  • The priority with the young patient is to retain the femoral head if possible, even with a displaced intracapsular fracture. The risk of avascular necrosis and non-union (and therefore revision surgery) associated with internal fixation needs weighing up against the sequelae of total hip replacement in the young (wear, dislocation, revision). Discussion is necessary with the patient, on a case by case basis.

** Undisplaced fractures in the elderly can be treated with internal fixation, often with cannulated screws. This is appropriate for valgus impacted subcapital fractures which are inherently stable, to prevent secondary displacement. This does still carry the risk of AVN or non-union, and therefore a future revision. For this reason, many surgeons advocate arthroplasty as a single surgery.

*** NICE guidance - patients who fulfil these criteria should be offered total hip replacement which conveys better function and prosthetic survivorship, compared with hemiarthroplasty, but at an increased risk of dislocation.
* Intertrochanteric fractures vary greatly in their stability. If the trochanter (and therefore lateral wall), and medial calcar is in tact, then the fracture configuration bears stability. This can be treated with a DHS, as collapse of the fracture is predictable. Where either or both structures are involved in the fracture, stability becomes compromised and many surgeons will favour using an intramedullary device. This is an ongoing debate, and difficult to test in an exam setting.

Post operative management
Patients should be mobilised fully weight bearing where possible. Care is multidisciplinary in its delivery. Elderly patients should have orthogeriatrician assessment of comorbidity, and bone health with secondary prevention measures if appropriate. There should be early involvement of physiotherapy and occupational therapy services.

How well did you know this?
1
Not at all
2
3
4
5
Perfectly
156
Q

At which of the following vertebral body levels does the common carotid artery typically bifurcate into the external and internal carotid arteries?

C4

C2

C1

C6

C7

A

It terminates at the upper border of the thyroid cartilage, Which is usually located at C4. Common carotid artery

The right common carotid artery arises at the bifurcation of the brachiocephalic trunk, the left common carotid arises from the arch of the aorta. Both terminate at the level of the upper border of the thyroid cartilage (the lower border of the third cervical vertebra) by dividing into the internal and external carotid arteries.

Left common carotid artery
This vessel arises immediately to the left and slightly behind the origin of the brachiocephalic trunk. Its thoracic portion is 2.5- 3.5 cm in length and runs superolaterally to the sternoclavicular joint.

In the thorax
The vessel is in contact, from below upwards, with the trachea, left recurrent laryngeal nerve, left margin of the oesophagus. Anteriorly the left brachiocephalic vein runs across the artery, and the cardiac branches from the left vagus descend in front of it. These structures together with the thymus and the anterior margins of the left lung and pleura separate the artery from the manubrium.

In the neck
The artery runs superiorly deep to sternocleidomastoid and then enters the anterior triangle. At this point it lies within the carotid sheath with the vagus nerve and the internal jugular vein. Posteriorly the sympathetic trunk lies between the vessel and the prevertebral fascia. At the level of C7 the vertebral artery and thoracic duct lie behind it. The anterior tubercle of C6 transverse process is prominent and the artery can be compressed against this structure (it corresponds to the level of the cricoid).
Anteriorly at C6 the omohyoid muscle passes superficial to the artery.
Within the carotid sheath the jugular vein lies lateral to the artery.

Right common carotid artery
The right common carotid arises from the brachiocephalic artery. The right common carotid artery corresponds with the cervical portion of the left common carotid, except that there is no thoracic duct on the right. The oesophagus is less closely related to the right carotid than the left.

Summary points about the carotid anatomy

Path
Passes behind the sternoclavicular joint (12% patients above this level) to the upper border of the thyroid cartilage, to divide into the external (ECA) and internal carotid arteries (ICA).

Relations
Level of 6th cervical vertebra crossed by omohyoid
Then passes deep to the thyrohyoid, sternohyoid, sternomastoid muscles.
Passes anterior to the carotid tubercle (transverse process 6th cervical vertebra)-NB compression here stops haemorrhage.
The inferior thyroid artery passes posterior to the common carotid artery.
Then : Left common carotid artery crosses the thoracic duct, Right common carotid artery crossed by recurrent laryngeal nerve

How well did you know this?
1
Not at all
2
3
4
5
Perfectly
157
Q

A female infant is born by cesarean section at 38 weeks gestation for foetal distress. The attending paediatricians notice that she has a single palmar crease and prominent epicanthic folds of the eyes. Soon after the birth the mother tries to feed the child who has a projectile vomit about 10 minutes after feeding. On examination, she has a soft, non distended abdomen. What is the most likely cause?

Pyloric stenosis

Duodenal atresia

Milk allergy

Meconium ileus

Anal atresia

A

Proximally sited atresia will produce high volume vomits which may or may not be bile stained. Abdominal distension is characteristically absent. Whilst under resuscitated children may be a little dehydrated they are seldom severely ill. The presence of Trisomy 21 (palmar and eye signs) increases the likelihood of duodenal atresia.

Bilious vomiting in neonates

Causes of intestinal obstruction with bilious vomiting in neonates
Disorder Incidence and causation Age at presentation Diagnosis Treatment
Duodenal atresia 1 in 5000 (higher in Downs syndrome) Few hours after birth AXR shows ‘double bubble sign, contrast study may confirm Duodenoduodenostomy
Malrotation with volvulus Usually cause by incomplete rotation during embryogenesis Usually 3-7 days after birth, volvulus with compromised circulation may result in peritoneal signs and haemodynamic instability Upper GI contrast study may show DJ flexure is more medially placed, USS may show abnormal orientation of SMA and SMV Ladd’s procedure
Jejunal/ ileal atresia Usually caused by vascular insufficiency in utero, usually 1 in 3000 Usually within 24 hours of birth AXR will show air-fluid levels Laparotomy with primary resection and anastomosis
Meconium ileus Occurs in between 15 and20% of those babies with cystic fibrosis, otherwise 1 in 5000 Typically in first 24-48 hours of life with abdominal distension and bilious vomiting Air - fluid levels on AXR, sweat test to confirm cystic fibrosis Surgical decompression, serosal damage may require segmental resection
Necrotising enterocolitis Up to 2.4 per 1000 births, risks increased in prematurity and inter-current illness Usually second week of life Dilated bowel loops on AXR, pneumatosis and portal venous air Conservative and supportive for non perforated cases, laparotomy and resection in cases of perforation of ongoing clinical deteriorat

How well did you know this?
1
Not at all
2
3
4
5
Perfectly
158
Q

A 73 year old man develops a lesion affecting the lobule of his ear. Into which of the following lymph node groups will the lesion drain?

Deep cervical

Superficial cervical

Retroauricular nodes

Occipital nodes

Internal jugular nodes

A

The lobule of the ear drains to the superficial cervical nodes. Lymphatic drainage of the auricle

Lymphatic Drainage
The lateral surface of the upper half drains to the superficial parotid lymph nodes
The cranial surface of the superior half drains to the mastoid nodes and deep cervical lymph nodes
The lower half and lobule drain into the superficial cervical lymph nodes.

How well did you know this?
1
Not at all
2
3
4
5
Perfectly
159
Q

In Froment’s test which muscle function is tested?

Flexor pollicis longus

Abductor pollicis longus

Abductor pollicis brevis

Adductor pollicis

Opponens pollicis

A

Fromen’s test, also known as the Fromen (or Froment) sign, is a clinical test used to evaluate the function of the ulnar nerve, specifically assessing the strength of the adductor pollicis muscle. Here is how it is typically performed:

  1. The patient is asked to hold a piece of paper between their thumb and index finger.
  2. The examiner attempts to pull the paper away.

In a positive Fromen’s sign, the patient compensates for a weak adductor pollicis by flexing the thumb’s interphalangeal joint, which is facilitated by the flexor pollicis longus muscle, innervated by the median nerve. This compensation indicates weakness or paralysis of the adductor pollicis muscle due to ulnar nerve dysfunction.

Fromen’s test can be useful in diagnosing conditions that affect the ulnar nerve, such as cubital tunnel syndrome or ulnar neuropathy.

Assess for ulnar nerve palsy
Adductor pollicis muscle function tested
Hold a piece of paper between their thumb and index finger. The object is then pulled away. If ulnar nerve palsy, unable to hold the paper and will flex the flexor pollicis longus to compensate (flexion of thumb at interphalangeal joint).

Phalen’s test

Assess carpal tunnel syndrome
More sensitive than Tinel’s sign
Hold wrist in maximum flexion and the test is positive if there is numbness in the median nerve distribution.

Tinel’s sign

Assess for carpal tunnel syndrome
Tap the median nerve at the wrist and the test is positive if there is tingling/electric-like sensations over the distribution of the median nerve.

How well did you know this?
1
Not at all
2
3
4
5
Perfectly
160
Q

A 33 year old lady is 32 weeks pregnant and it is suspected that there may be a perforated abdominal viscus. What is the most appropriate course of action?

Arrange an abdominal CT scan

Arrange an abdominal MRI scan

Undertake a laparotomy

Undertake a laparoscopy

Undertake diagnostic peritoneal lavage

A

The keyword here is suspected. That being the case the investigation is CT. Although it is known that ionising radiation exposure can lead to cell death, mutation of germ cells, and carcinogenesis, there is no common modern radiographic procedure that results in radiation exposure to a level that threatens embryo or fetal well-being.Radiation exposure of <5 rads (a computed tomographic abdomen/pelvis study carries an exposure of 3.5 rads) has not been associated with fetal defects or loss. Careful shielding of the patient can also minimise exposure. Importantly, the use of ultrasound for diagnosis is clearly safe in pregnancy, although it should be used to evaluate and answer a defined clinical problem. A negative laparoscopy/ laparotomy poses signifcant foetal risks and at either procedure the enlarged uterus can result in missed pathology.

Abdominal imaging in pregnancy

Pregnant patients will often present with abdominal pain and the co-existing physiological leucocytosis can make laboratory assessment less reliable. In the non pregnant patient there is a low threshold for cross sectional imaging with CT scanning. The main foetal risks from cross sectional imaging in pregnancy are delayed rather than immediate. That being the case, where CT is likely to facilitate accurate diagnosis that improves short term foetal outcomes or saves foetal life, it is justified.
At all times, alternatives to CT need to be considered and utilised where possible. MRI is often used though for complex sequences movement artifact can limit the quality of views obtained. The use of shields can also limit radiation exposure to the foetus.

How well did you know this?
1
Not at all
2
3
4
5
Perfectly
161
Q

A 77-year-old man who is a smoker, collapses 12 days following an esophagectomy. Prior to his collapse he complained of a sudden onset of chest pain and dyspnea on examination is blood pressure is 90/60 mmHg. His ECG shows right heart strain. What is the most likely explanation?

Pulmonary embolism

Simple pneumothorax

Tension pneumothorax

Atelectesis

Myocardial infarction

A

The event occurring at day 12 with right sided ECG changes all favor PE. Surgery and cancer are risk factors. Pneumothorax would tend to occur much earlier in the post operative period.

Pulmonary Embolism: ECG changes

  • No changes
    S1, Q3, T3
    Tall R waves: V1
    P pulmonale (peaked P waves): inferior leads
    Right axis deviation, Right bundle branch block
    Atrial arrhythmias
    T wave inversion: V1, V2, V3
    Right ventricular strain: if identified is associated with adverse short-term outcome and adds prognostic value to echocardiographic evidence of right ventricular dysfunction in patients with acute pulmonary embolism and normal blood pressure.
How well did you know this?
1
Not at all
2
3
4
5
Perfectly
162
Q

An obese 12 year old boy is referred with pain in the left knee and hip. On examination, he has an antaglic gait and limitation of internal rotation. His knee has normal range of passive and active movement. What is the most likely diagnosis?

Septic arthritis

Developmental dysplasia of the hip

Perthes disease

Osteoarthritis of the hip

Slipped upper femoral epiphysis

A

Based on the information provided, the most likely diagnosis for this 12-year-old obese boy is:

Slipped Upper Femoral Epiphysis (SUFE)

Here’s why this is the most probable diagnosis:

  1. Age and body habitus: SUFE typically occurs in adolescents, often during growth spurts. It is more common in obese children, which matches the description of this patient.
  2. Symptoms:
    • Pain in the hip and knee: SUFE often presents with hip pain that can be referred to the knee.
    • Antalgic gait: This is a common finding in SUFE, where the patient tries to avoid putting weight on the affected side.
  3. Physical examination:
    • Limitation of internal rotation: This is a classic sign of SUFE. The slippage of the femoral head causes restricted internal rotation of the hip.
    • Normal knee examination: The knee pain is referred from the hip, so the knee itself shows no abnormalities on examination.

Let’s briefly consider why the other options are less likely:

  • Septic arthritis: This would typically present with more acute symptoms, fever, and more severe limitation of movement.
  • Developmental dysplasia of the hip: This is usually diagnosed in infancy or early childhood, not at 12 years old.
  • Perthes disease: While possible, it’s less common in obese children and typically affects younger children (4-8 years old).
  • Osteoarthritis of the hip: This is extremely rare in a 12-year-old and would not be a primary consideration.

Given the patient’s age, obesity, symptoms, and examination findings, Slipped Upper Femoral Epiphysis (SUFE) is the most likely diagnosis. This condition requires prompt recognition and treatment to prevent further slippage and potential complications.

Slipped upper femoral epiphysis is commonest in obese adolescent males. The x-ray will show displacement of the femoral epiphysis inferolaterally. Treatment is usually with rest and non weight bearing crutches.

Paediatric orthopaedics

Diagnosis Mode of presentation Treatment Radiology
Developmental dysplasia of the hip Usually diagnosed in infancy by screening tests. May be bilateral, when disease is unilateral there may be leg length inequality. As disease progresses child may limp and then early onset arthritis. More common in extended breech babies. Splints and harnesses or traction. In later years osteotomy and hip realignment procedures may be needed. In arthritis a joint replacement may be needed. However, this is best deferred if possible as it will almost certainly require revision Initially no obvious change on plain films and USS gives best resolution until 3 months of age. On plain films Shentons line should form a smooth arc
Perthes Disease Hip pain (may be referred to the knee) usually occurring between 5 and 12 years of age. Bilateral disease in 20%. Remove pressure from joint to allow normal development. Physiotherapy. Usually self-limiting if diagnosed and treated promptly. X-rays will show flattened femoral head. Eventually in untreated cases the femoral head will fragment.
Slipped upper femoral epiphysis Typically seen in obese male adolescents. Pain is often referred to the knee. Limitation to internal rotation is usually seen. Knee pain is usually present 2 months prior to hip slipping. Bilateral in 20%. Bed rest and non-weight bearing. Aim to avoid avascular necrosis. If severe slippage or risk of it occurring then percutaneous pinning of the hip may be required. X-rays will show the femoral head displaced and falling inferolaterally (like a melting ice cream cone) The Southwick angle gives indication of disease severity

How well did you know this?
1
Not at all
2
3
4
5
Perfectly
163
Q

In the distal third of the upper arm, where is the musculocutaneous nerve located?

Between the biceps brachii and brachialis muscles

Between the brachialis and brachioradialis muscles

Between the brachioradialis and triceps muscles

Between the brachialis and triceps muscles

Between the humerus and brachialis muscles

A

The musculocutaneous nerve lies between the biceps and brachialis muscles.

Musculocutaneous nerve

Branch of lateral cord of brachial plexus

Path
It penetrates the coracobrachialis muscle
Passes obliquely between the biceps brachii and the brachialis to the lateral side of the arm
Above the elbow it pierces the deep fascia lateral to the tendon of the biceps brachii
Continues into the forearm as the lateral cutaneous nerve of the forearm

Innervates
Coracobrachialis
Biceps brachii
Brachialis

How well did you know this?
1
Not at all
2
3
4
5
Perfectly
164
Q

Which of the following tumours are most likely to give rise to para-aortic nodal metastasis early?

Ovarian

Bladder

Rectal

Caecal

Cervical

A

Ovarian tumours are supplied by the ovarian vessels, these branch directly from the aorta. The cervix drains to the internal and external iliac nodes.

Para-aortic lymphadenopathy

Organ sites that may metastasise (early) to the para-aortic lymph nodes:
Testis
Ovary
Uterine fundus
Many other organs may result in para-aortic nodal disease. However, these deposits will represent a much later stage of the disease, since other nodal stations are involved earlier.

How well did you know this?
1
Not at all
2
3
4
5
Perfectly
165
Q

A 76 year old man complains of symptoms of claudication. The decision is made to measure his ankle brachial pressure index. The signal from the dorsalis pedis artery is auscultated with a hand held doppler device. This vessel is the continuation of which of the following?

Posterior tibial artery

Anterior tibial artery

Peroneal artery

Popliteal artery

None of the above

A

The dorsalis pedis is a continuation of the anterior tibial artery.
Foot- anatomy

Arches of the foot
The foot is conventionally considered to have two arches.
The longitudinal arch is higher on the medial than on the lateral side. The posterior part of the calcaneum forms a posterior pillar to support the arch. The lateral part of this structure passes via the cuboid bone and the lateral two metatarsal bones. The medial part of this structure is more important. The head of the talus marks the summit of this arch, located between the sustentaculum tali and the navicular bone. The anterior pillar of the medial arch is composed of the navicular bone, the three cuneiforms and the medial three metatarsal bones.
The transverse arch is situated on the anterior part of the tarsus and the posterior part of the metatarsus. The cuneiforms and metatarsal bases narrow inferiorly, which contributes to the shape of the arch.

Intertarsal joints
Sub talar joint Formed by the cylindrical facet on the lower surface of the body of the talus and the posterior facet on the upper surface of the calcaneus. The facet on the talus is concave anteroposteriorly, the other is convex. The synovial cavity of this joint does not communicate with any other joint.
Talocalcaneonavicular joint The anterior part of the socket is formed by the concave articular surface of the navicular bone, posteriorly by the upper surface of the sustentaculum tali. The talus sits within this socket
Calcaneocuboid joint Highest point in the lateral part of the longitudinal arch. The lower aspect of this joint is reinforced by the long plantar and plantar calcaneocuboid ligaments.
Transverse tarsal joint The talocalcaneonavicular joint and the calcaneocuboid joint extend across the tarsus in an irregular transverse plane, between the talus and calcaneus behind and the navicular and cuboid bones in front. This plane is termed the transverse tarsal joint.
Cuneonavicular joint Formed between the convex anterior surface of the navicular bone and the concave surface of the the posterior ends of the three cuneiforms.
Intercuneiform joints Between the three cuneiform bones.
Cuneocuboid joint Between the circular facets on the lateral cuneiform bone and the cuboid. This joint contributes to the tarsal part of the transverse arch.
A detailed knowledge of the joints is not required for MRCS Part A. However, the contribution they play to the overall structure of the foot should be appreciated

Muscles of the foot
Muscle Origin Insertion Nerve supply Action
Abductor hallucis Medial side of the calcaneus, flexor retinaculum, plantar aponeurosis Medial side of the base of the proximal phalanx Medial plantar nerve Abducts the great toe
Flexor digitorum brevis Medial process of the calcaneus, plantar eponeurosis. Via 4 tendons into the middle phalanges of the lateral 4 toes. Medial plantar nerve Flexes all the joints of the lateral 4 toes except for the interphalangeal joint.
Abductor digit minimi From the tubercle of the calcaneus and from the plantar aponeurosis Together with flexor digit minimi brevis into the lateral side of the base of the proximal phalanx of the little toe Lateral plantar nerve Abducts the little toe at the metatarsophalangeal joint
Flexor hallucis brevis From the medial side of the plantar surface of the cuboid bone, from the adjacent part of the lateral cuneiform bone and from the tendon of tibialis posterior. Into the proximal phalanx of the great toe, the tendon contains a sesamoid bone Medial plantar nerve Flexes the metatarsophalangeal joint of the great toe.
Adductor hallucis Arises from two heads. The oblique head arises from the sheath of the peroneus longus tendon, and from the plantar surfaces of the bases of the 2nd, 3rd and 4th metatarsal bones. The transverse head arises from the plantar surface of the lateral 4 metatarsophalangeal joints and from the deep transverse metatarsal ligament. Lateral side of the base of the proximal phalanx of the great toe. Lateral plantar nerve Adducts the great toe towards the second toe. Helps maintain the transverse arch of the foot.
Extensor digitorum brevis On the dorsal surface of the foot from the upper surface of the calcaneus and its associated fascia Via four thin tendons which run forward and medially to be inserted into the medial four toes. The lateral three tendons join with hoods of extensor digitorum longus. Deep peroneal Extend the metatarsophalangeal joint of the medial four toes. It is unable to extend the interphalangeal joint without the assistance of the lumbrical muscles.

Nerves in the foot

Lateral plantar nerve
Passes anterolaterally towards the base of the 5th metatarsal between flexor digitorum brevis and flexor accessorius. On the medial aspect of the lateral plantar artery. At the base of the 5th metatarsal it splits into superficial and deep branches.

Medial plantar nerve
Passes forwards with the medial plantar artery under the cover of the flexor retinaculum to the interval between abductor hallucis and flexor digitorum brevis on the sole of the foot.

Plantar arteries
Arise under the cover of the flexor retinaculum, midway between the tip of the medial malleolus and the most prominent part of the medial side of the heel.

Medial plantar artery. Passes forwards medial to medial plantar nerve in the space between abductor hallucis and flexor digitorum brevis.Ends by uniting with a branch of the 1st plantar metatarsal artery.
Lateral plantar artery. Runs obliquely across the sole of the foot. It lies lateral to the lateral plantar nerve. At the base of the 5th metatarsal bone it arches medially across the foot on the metatarsals

Dorsalis pedis artery
This vessel is a direct continuation of the anterior tibial artery. It commences on the front of the ankle joint and runs to the proximal end of the first metatarsal space. Here is gives off the arcuate artery and continues forwards as the first dorsal metatarsal artery. It is accompanied by two veins throughout its length. It is crossed by the extensor hallucis brevis

How well did you know this?
1
Not at all
2
3
4
5
Perfectly
166
Q

Loss of taste sensation from the posterior third of the tongue is most likely the result to an injury to which of the structures listed below?

Hypoglossal nerve

Chorda tympani nerve

Facial nerve

Mandibular branch of the trigeminal nerve

Glossopharyngeal nerve

A

The glossopharyngeal nerve supplies the taste and general sensation to the posterior third of the tongue.

Tongue

The tongue is a muscular organ concerned with both feeding and speech. It is shaped like an inverted boot with a root through which extrinsic muscles pass to attach it to the soft palate, the styloid process, the hyoid bone and the mandible. It has a body that comprises muscle intermingled with lingual salivary glands, connective tissue, nerves, vessels and lymphatic follicles.
The dorsum of the tongue is divided by an ill defined V shaped groove, the sulcus terminalis, into an anterior two thirds and a posterior third.

The arterial blood supply to the tongue is from the paired sets of lingual arteries that arise from the external carotid. The venous drainage is via dorsal lingual veins which then join the lingual veins and eventually into the sublingual vein.

The nerve supply is as follows:
The lingual branch of the mandibular division of the trigeminal nerve supplies common sensation to the anterior two thirds
The chorda tympani branch of the facial nerve supplies taste to the anterior two thirds
The lingual branch of the glossopharyngeal nerve supplies common sensation and taste to the posterior third of the tongue
All intrinsic muscles of the tongue are supplied by the hypoglossal nerve

How well did you know this?
1
Not at all
2
3
4
5
Perfectly
167
Q

A 33 year old female is admitted for varicose vein surgery. She is fit and well. After the procedure she is persistently bleeding. She is known to have menorrhagia. Investigations show a prolonged bleeding time and increased APTT. She has a normal PT and platelet count. What is the most likely cause?

Anti phospholipid syndrome

Haemophilia

Factor V Leiden deficiency

von Willebrands disease

Protein C and S deficiency

A

Bleeding post operatively, epistaxis and menorrhagia may indicate a diagnosis of vWD. Haemoarthroses are rare. The bleeding time is usually normal in haemophilia (X-linked) and vitamin K deficiency.

von Willebrands disease

  • Most common inherited bleeding disorder
    All vWD is caused by mutations in the gene for von Willebrand factor. von Willebrand factor is an adhesive glycoprotein that is secreted by endothelium and megakaryocytes
    von Willebrand factor promotes platelet adhesion to damaged endothelium and other platelets. It is also involved in the transport and stabilization of factor VIII
    There are 7 subtypes of von Willebrand disease. The commonest is type I (autosomal dominant) which accounts for 80% of cases, type 2vWD (autosomal dominant or recessive) accounts for 15% of cases
    There is a significant spectrum of severity ranging from spontaneous bleeding and epistaxis through to troublesome excessive bleeding following minor procedures
    The test that is most commonly used are von willebrand factor assays
    Treatments include administration of tranexamic acid for minor cases undergoing minor procedures. More significant bleeding or more significant procedures respond well to DDAVP. This is most effective in type I, less effective in type 2 and contraindicated in type 2B. Patients with type 3 disease do not respond to DDAVP as they lack the ability to secrete vWF
    Individuals who cannot have DDAVP or in whom it is contra indicated usually receive factor VIII concentrates containing vWF
How well did you know this?
1
Not at all
2
3
4
5
Perfectly
168
Q

Administration of neostigmine to a patient who has received a non depolarizing muscle relaxant is most likely to result in which of the following?

Prolongation of muscle relaxation

Tachycardia

Hypertension

Bradycardia

Decreased gut peristalsis

A

Neostigmine can cause bradycardia and atropine is often administered concomitantly to counter this effect.

Muscle relaxants

Suxamethonium
Depolarising neuromuscular blocker
Inhibits action of acetylcholine at the neuromuscular junction
Degraded by plasma cholinesterase and acetylcholinesterase (affected by lack of acetylcholinesterase)
Fastest onset and shortest duration of action of all muscle relaxants
Produces generalised muscular contraction prior to paralysis
Adverse effects include hyperkalaemia, malignant hyperthermia, delayed recovery
Atracurium
Non depolarising neuromuscular blocking drug
Duration of action usually 30-45 minutes
Generalised histamine release on administration may produce facial flushing, tachycardia and hypotension
Not excreted by liver or kidney, broken down in tissues by hydrolysis
Reversed by neostigmine
Vecuronium
Non depolarising neuromuscular blocking drug
Duration of action approximately 30 - 40 minutes
Degraded by liver and kidney and effects prolonged in organ dysfunction
Effects may be reversed by neostigmine
Pancuronium
Non depolarising neuromuscular blocker
Onset of action approximately 2-3 minutes
Duration of action up to 2 hours
Effects may be partially reversed with drugs such as neostigmine

How well did you know this?
1
Not at all
2
3
4
5
Perfectly
169
Q

A 22 year old is found to have bilateral acoustic neuromas. Which of the disorders listed below is most likely to be present?

Neurofibromatosis Type I

Neurofibromatosis Type II

MEN I

MEN II

Gardner’s syndrome

A

In NF2 bilateral acoustic neuromas are characteristic with a family history of Neurofibroma.

Genetics and surgical disease

Some of the more commonly occurring genetic conditions occurring in surgical patients are presented here.

Li-Fraumeni Syndrome
Autosomal dominant
Consists of germline mutations to p53 tumour suppressor gene
High incidence of malignancies particularly sarcomas and leukaemias
Diagnosed when:

*Individual develops sarcoma under 45 years
*First degree relative diagnosed with any cancer below age 45 years and another family member develops malignancy under 45 years or sarcoma at any age

BRCA 1 and 2
Carried on chromosome 17 (BRCA 1) and Chromosome 13 (BRCA 2)
Linked to developing breast cancer (60%) risk.
Associated risk of developing ovarian cancer (55% with BRCA 1 and 25% with BRCA 2).

Lynch Syndrome
Autosomal dominant
Develop colonic cancer and endometrial cancer at young age
80% of affected individuals will get colonic and/ or endometrial cancer
High risk individuals may be identified using the Amsterdam criteria

Amsterdam criteria
Three or more family members with a confirmed diagnosis of colorectal cancer, one of whom is a first degree (parent, child, sibling) relative of the other two.
Two successive affected generations.
One or more colon cancers diagnosed under age 50 years.
Familial adenomatous polyposis (FAP) has been excluded.

Gardners syndrome
Autosomal dominant familial colorectal polyposis
Multiple colonic polyps
Extra colonic diseases include: skull osteoma, thyroid cancer and epidermoid cysts
Desmoid tumours are seen in 15%
Mutation of APC gene located on chromosome 5
Due to colonic polyps most patients will undergo colectomy to reduce risk of colorectal cancer
Now considered a variant of familial adenomatous polyposis coli

How well did you know this?
1
Not at all
2
3
4
5
Perfectly
170
Q

During an operation for varicose veins the surgeons are mobilising the long saphenous vein. Near its point of entry to the femoral vein an artery is injured and bleeding is encountered. From where is the bleeding most likely to originate?

Femoral artery

Profunda femoris artery

Superficial circumflex iliac artery

Superficial epigastric artery

Deep external pudendal artery

A

The deep external pudendal artery is a branch of the SFA and it runs medially under the long saphenous vein near its point of union with the femoral vein. The superficial external pudendal artery lies superior to the SFJ. Neither vessel is functionally important and if injured they are best ligated.

Femoral triangle anatomy

Boundaries
Superiorly Inguinal ligament
Laterally Sartorius
Medially Adductor longus
Floor Iliopsoas, adductor longus and pectineus
Roof
Fascia lata and Superficial fascia
Superficial inguinal lymph nodes (palpable below the inguinal ligament)
Long saphenous vein

How well did you know this?
1
Not at all
2
3
4
5
Perfectly
171
Q

Which of the nerves listed below is responsible for the innervation of gluteus medius?

Inferior gluteal nerve

Sciatic nerve

Superior gluteal nerve

Perineal nerve

Inferior rectal nerve

A

Superior and inferior gluteal nerve

Superior gluteal nerve
Arises from dorsal surface of the sacral plexus (L4, L5, S1)
Passes into gluteal region together with superior gluteal vessels
Supplies gluteus medius and minimus

Inferior gluteal nerve
Arises from dorsal surface of sacral plexus (L5, S1 and S2)
Runs medial to the posterior femoral cutaneous nerve
Enters gluteal region at inferior border of piriformis
Supplies gluteus maximus

How well did you know this?
1
Not at all
2
3
4
5
Perfectly
172
Q

A 73 year old man is recovering from a stroke but is deemed to have an unsafe swallow. Apart from his CVA his past medical history includes rate controlled atrial fibrillation and a previous oesophagectomy. What is the best option for long term feeding?

Endoscopically inserted PEG tube

Surgically inserted PEG tube

Surgically inserted feeding jejunostomy tube

TPN via peripheral venous access system

TPN via a central line

A

Given the patient’s history of a previous esophagectomy and the current need for long-term feeding due to an unsafe swallow, the best option for long-term feeding would be:

Surgically inserted feeding jejunostomy tube

Here’s why:

  1. Previous Esophagectomy: This history complicates the placement of a percutaneous endoscopic gastrostomy (PEG) tube, whether endoscopic or surgical, as the anatomy may be altered significantly.
  2. Safety Concerns: A feeding jejunostomy tube can bypass the stomach and esophagus altogether, effectively preventing potential complications related to his previous esophagectomy.
  3. Long-term Feeding: A surgically inserted jejunostomy tube is suitable for long-term nutritional support, especially in patients where the upper gastrointestinal tract cannot be used safely.
  4. TPN (Total Parenteral Nutrition): While TPN can be used for nutritional support, it is generally considered as a last resort due to the high risk of complications such as infections, liver dysfunction, and metabolic disturbances, especially over the long term. Peripheral TPN is not suitable for long-term use, and central TPN carries significant risks.

Therefore, the most appropriate and safest option for long-term feeding in this patient is the surgically inserted feeding jejunostomy tube.

How well did you know this?
1
Not at all
2
3
4
5
Perfectly
173
Q

A surgical resection specimen is analysed histologically. The pathologist comments that at the periphery of the resected specimen, oxyphil cells are identified. In which of the structures listed below are these cells typically found?

Thymus

Thyroid gland

Parathyroid gland

Lymph node

Adrenal gland

A

Oxyphil cells are typically found in parathyroid glands. Parathyroid glands- anatomy

Four parathyroid glands
Located posterior to the thyroid gland
They lie within the pretracheal fascia

Embryology
The parathyroids develop from the extremities of the third and fourth pharyngeal pouches. The parathyroids derived from the fourth pharyngeal pouch are located more superiorly and are associated with the thyroid gland. Those derived from the third pharyngeal pouch lie more inferiorly and may become associated with the thymus.

Blood supply
The blood supply to the parathyroid glands is derived from the inferior and superior thyroid arteries[1]. There is a rich anastomosis between the two vessels. Venous drainage is into the thyroid veins.

Relations
Laterally Common carotid
Medially Recurrent laryngeal nerve, trachea
Anterior Thyroid
Posterior Pretracheal fascia

How well did you know this?
1
Not at all
2
3
4
5
Perfectly
174
Q

In which of the conditions described below is Rovsing’s sign most likely to be absent?

Locally advanced caecal cancer

Para ileal appendicitis

Right sided colonic diverticulitis

Retrocaecal appendicitis

Severe terminal ileal Crohns disease

A

Any advanced right iliac fossa pathology can result in a positive Rovsings sign. However, in retrocaecal appendicitis, it may be absent and this fact can contribute to a delayed diagnosis if undue weight is placed on the presence of the sign in making the diagnosis.

How well did you know this?
1
Not at all
2
3
4
5
Perfectly
175
Q

A 22 year old man presents with an aching pain and discomfort in his right testicle. He has felt systemically unwell for the preceding 48 hours. On examination, there is tenderness of the right testicle. He has an exaggerated cremasteric reflex. What is the correct course of action?

Scrotal exploration via a scrotal approach

Testicular inspection via an inguinal approach

Administration of antibiotics

Fine needle aspiration cytology

Reassure and discharge

A

This is likely to represent epididymo-orchitis, this is usually due to infection with gonorrhoea or chlamydia in this age group. In addition to treatment with antibiotics contact tracing and appropriate swabs should also be performed.

Testicular disorders

Testicular cancer
Testicular cancer is the most common malignancy in men aged 20-30 years. Around 95% of cases of testicular cancer are germ-cell tumours. Germ cell tumours may essentially be divided into:

Tumour type Key features Tumour markers Pathology
Seminoma
Commonest subtype (50%)
Average age at diagnosis = 40
Even advanced disease associated with 5 year survival of 73%
AFP usually normal
HCG elevated in 10% seminomas
Lactate dehydrogenase; elevated in 10-20% seminomas (but also in many other conditions)
Sheet like lobular patterns of cells with substantial fibrous component. Fibrous septa contain lymphocytic inclusions and granulomas may be seen.
Non seminomatous germ cell tumours (42%)
Teratoma
Yolk sac tumour
Choriocarcinoma
Mixed germ cell tumours (10%)
Younger age at presentation =20-30 years
Advanced disease carries worse prognosis (48% at 5 years)
Retroperitoneal lymph node dissection may be needed for residual disease after chemotherapy

Risk factors for testicular cancer
Cryptorchidism
Infertility
Family history
Klinefelter’s syndrome
Mumps orchitis

Features
A painless lump is the most common presenting symptom
Pain may also be present in a minority of men
Other possible features include hydrocele, gynaecomastia

Diagnosis
Ultrasound is first-line
CT scanning of the chest/ abdomen and pelvis is used for staging
Tumour markers (see above) should be measured

Management
Orchidectomy (Inguinal approach)
Chemotherapy and radiotherapy may be given depending on staging
Abdominal lesions >1cm following chemotherapy may require retroperitoneal lymph node dissection.

Prognosis is generally excellent
5 year survival for seminomas is around 95% if Stage I
5 year survival for teratomas is around 85% if Stage I

Benign disease

Epididymo-orchitis
Acute epididymitis is an acute inflammation of the epididymis, often involving the testis and usually caused by bacterial infection.
Infection spreads from the urethra or bladder. In men <35 years, gonorrhoea or chlamydia are the usual infections.
Amiodarone is a recognised non infective cause of epididymitis, which resolves on stopping the drug.
Tenderness is usually confined to the epididymis, which may facilitate differentiating it from torsion where pain usually affects the entire testis.

Testicular torsion
Twist of the spermatic cord resulting in testicular ischaemia and necrosis.
Most common in males aged between 10 and 30 (peak incidence 13-15 years)
Pain is usually severe and of sudden onset.
Cremasteric reflex is lost and elevation of the testis does not ease the pain.
Treatment is with surgical exploration. If a torted testis is identified then both testis should be fixed as the condition of bell clapper testis is often bilateral.

Hydrocele
Presents as a mass that transilluminates, usually possible to ‘get above’ it on examination.
In younger men it should be investigated with USS to exclude tumour.
In children it may occur as a result of a patent processus vaginalis.
Treatment in adults is with a Lords or Jabouley procedure.
Treatment in children is with trans inguinal ligation of PPV.

How well did you know this?
1
Not at all
2
3
4
5
Perfectly
176
Q

A 4 year old is admitted with right iliac fossa pain and is due to undergo an appendicectomy. The nursing staff would like to give the child an infusion of intravenous fluid whilst waiting for theatre. Assuming electrolytes are normal, which of the following is an appropriate fluid for infusion in this situation?

10% Dextrose solution

0.9% Saline solution

0.45% saline/ 5% glucose solution

Gelofusin

None of the above

A

0.45% saline/ 5% glucose solutions carry a risk of hyponatraemia and is contra indicated- see below.

Paediatric fluid management

Since 2000 there have been at least 4 reported deaths from fluid induced hyponatraemia in children. This led to the National Patient Safety Agency introducing revised guidelines in 2007. These have been reviewed and extensively updated by NICE in 2015 and further modified in 2020.
Indications for IV fluids include:
Resuscitation and circulatory support
Replacing on-going fluid losses
Maintenance fluids for children for whom oral fluids are not appropriate
Correction of electrolyte disturbances

Fluids to be avoided
Outside the neonatal period saline / glucose solutions should not be given. The greatest risk is with saline 0.18 / glucose 4% solutions. The report states that 0.45% saline / 5% glucose may be used. But preference should be given to isotonic solutions and few indications exist for this solution either.
The key point emphasised in the NICE guidelines in the avoidance of glucose containing solutions and instead of routinely giving glucose IV to children and neonates, the blood glucose levels should be monitored. In neonates in the first few days of life, sodium levels in the range of 131-154 mmol/l may be too high and a neonatologist consulted on a case by case basis.

Fluids to be used
If children and young people need IV fluids for routine maintenance, initially use isotonic crystalloids that contain sodium in the range 131 to 154 mmol/litre

Potassium should be added to maintenance fluids according patients plasma potassium levels (which should be monitored).
Blood glucose levels should be monitored in individuals at risk of hypoglycaemia

Intraoperative fluid management
If children and young people need IV fluids for routine maintenance, initially use isotonic crystalloids that contain sodium in the range 131 to 154 mmol/litre
Blood glucose levels should be monitored

Maintenance fluids
Weight Water requirement/kg/day Na mmol/kg/day K mmol/kg/day
First 10Kg body weight 100ml 2-4 1.5-2.5
Second 10Kg body weight 50ml 1-2 0.5-1.5
Subsequent Kg 20ml 0.5-1.0 0.2-0.7

Blood glucose will need to be monitored

How well did you know this?
1
Not at all
2
3
4
5
Perfectly
177
Q

A 23 year old man is reviewed on the ward 10 days following a laparotomy. The wound is inspected and is healing well. Which of the following processes is least likely to be occurring in the wound at this stage?

Angiogenesis

Synthesis of collagen

Necrosis of fibroblasts

Secretion of matrix metalloproteinases by fibroblasts

Proliferation of fibroblasts

A

Wound Healing Processes at 10 Days Post-Laparotomy

Wound healing is a complex biological process that occurs in several overlapping phases: hemostasis, inflammation, proliferation, and remodeling. By the tenth day following a laparotomy, the wound is typically in the proliferative phase of healing. During this phase, several key processes are actively occurring:

Angiogenesis: This is the formation of new blood vessels from pre-existing ones. It is crucial for supplying nutrients and oxygen to the healing tissue. At 10 days post-surgery, angiogenesis is likely to be well underway as the body attempts to restore blood supply to the area.

Synthesis of Collagen: Fibroblasts play a significant role in synthesizing collagen, which provides structural support to the wound. By this stage, collagen deposition is prominent as part of the body’s efforts to strengthen the newly formed tissue.

Necrosis of Fibroblasts: This refers to cell death due to injury or lack of blood supply. In a healing wound at this stage, necrosis of fibroblasts would be less likely because these cells are essential for wound repair and are actively proliferating and synthesizing extracellular matrix components.

Secretion of Matrix Metalloproteinases (MMPs) by Fibroblasts: MMPs are enzymes that degrade various components of the extracellular matrix and play a role in tissue remodeling. While their secretion can occur during wound healing, it is more prominent later in the remodeling phase rather than at 10 days post-surgery when fibroblast activity focuses on synthesis rather than degradation.

Proliferation of Fibroblasts: This process involves an increase in fibroblast numbers as they migrate into the wound site and begin producing collagen and other extracellular matrix components necessary for repair.

Given these considerations, we can analyze which process is least likely occurring at this stage:

Angiogenesis is expected.
Synthesis of collagen is expected.
Proliferation of fibroblasts is expected.
Secretion of MMPs may occur but not as prominently as other processes.
Necrosis of fibroblasts would be atypical since they are vital for healing.
Based on this analysis, necrosis of fibroblasts is least likely to be occurring in the wound at this stage because fibroblasts are actively involved in healing rather than undergoing necrosis.

Answer: Necrosis of fibroblasts

How well did you know this?
1
Not at all
2
3
4
5
Perfectly
178
Q

A 43 year old man is due to undergo an excision of the sub mandibular gland. Which of the following incisions is the most appropriate for this procedure?

A transversely orientated incision 3cm below the mandible

A transversely orientated incision immediately inferior to the mandible

A vertical incision 3 cm anterior to the angle of the mandible and extending inferiorly

A transversely orientated incision 2cm above the mandible

A transversely orientated incision 12cm below the mandible

A

To access the sub mandibular gland a transverse incision 3cm below the mandible should be made. Incisions located higher than this may damage the marginal mandibular branch of the facial nerve.

Anterior triangle of the neck

Boundaries
Anterior border of the Sternocleidomastoid
Lower border of mandible
Anterior midline

Sub triangles (divided by Digastric above and Omohyoid)
Muscular triangle: Neck strap muscles
Carotid triangle: Carotid sheath
Submandibular Triangle (digastric)

Contents of the anterior triangle
Digastric triangle Submandibular gland
Submandibular nodes
Facial vessels
Hypoglossal nerve
Muscular triangle Strap muscles
Jugular vein
Carotid triangle Carotid sheath (Common carotid, vagus and internal jugular vein)
Ansa cervicalis

Nerve supply to digastric muscle
Anterior: Mylohyoid nerve
Posterior: Facial nerve

How well did you know this?
1
Not at all
2
3
4
5
Perfectly
179
Q

Which of the muscles listed below is not innervated by the median nerve?

Flexor pollicis brevis

Lateral two lumbricals

Pronator teres

Opponens pollicis

Adductor pollicis

A

Adductor pollicis is innervated by the ulnar nerve.
Medial two lumbricals innervated by the ulnar nerve.

Median nerve

The median nerve is formed by the union of a lateral and medial root respectively from the lateral (C5,6,7) and medial (C8 and T1) cords of the brachial plexus; the medial root passes anterior to the third part of the axillary artery. The nerve descends lateral to the brachial artery, crosses to its medial side (usually passing anterior to the artery). It passes deep to the bicipital aponeurosis and the median cubital vein at the elbow.
It passes between the two heads of the pronator teres muscle, and runs on the deep surface of flexor digitorum superficialis (within its fascial sheath).
Near the wrist it becomes superficial between the tendons of flexor digitorum superficialis and flexor carpi radialis, deep to palmaris longus tendon. It passes deep to the flexor retinaculum to enter the palm, but lies anterior to the long flexor tendons within the carpal tunnel.

Branches
Region Branch
Upper arm No branches, although the nerve commonly communicates with the musculocutaneous nerve
Forearm Pronator teres
Pronator quadratus
Flexor carpi radialis
Palmaris longus
Flexor digitorum superficialis
Flexor pollicis longus
Flexor digitorum profundus (only the radial half)
Distal forearm Palmar cutaneous branch
Hand (Motor) Motor supply (LOAF)
Lateral 2 lumbricals
Opponens pollicis
Abductor pollicis brevis
Flexor pollicis brevis
Hand (Sensory)
Over thumb and lateral 2 ½ fingers
On the palmar aspect this projects proximally, on the dorsal aspect only the distal regions are innervated with the radial nerve providing the more proximal cutaneous innervation.

Patterns of damage
Damage at wrist
e.g. carpal tunnel syndrome
paralysis and wasting of thenar eminence muscles and opponens pollicis (ape hand deformity)
sensory loss to palmar aspect of lateral (radial) 2 ½ fingers

Damage at elbow, as above plus:
unable to pronate forearm
weak wrist flexion
ulnar deviation of wrist

Anterior interosseous nerve (branch of median nerve)
leaves just below the elbow
results in loss of pronation of forearm and weakness of long flexors of thumb and index finger

How well did you know this?
1
Not at all
2
3
4
5
Perfectly
180
Q

In which of the following operations are Skew flaps created?

Hindquarter amputation

Above knee amputation

Gritti- Stokes amputation

Below knee amputation

Symes amputation

A

This is one variant of a below knee amputation. The Burgess flap is the other commonly practised approach.

Amputations

Amputations are indicated when the affected limb is one of the following:
Dead non viable
Deadly where it is posing a major threat to life
Dead useless where it is viable but a prosthesis would be preferable

Orthopaedic surgery
Amputation is often undertaken as an option of last resort e.g. Limb salvage has failed and the limb is so non functional that mobility needs would be best met with prosthesis.
Chronic fracture non union or significant limb shortening following trauma would fit into this category. Occasionally following major trauma a primary amputation is preferable. This would be the case in an open fracture with major distal neurovascular compromise and other more life threatening injuries are present.

Vascular surgery
The first two categories are the most prevalent.
Diabetic foot sepsis is often a major cause of sepsis which can spread rapidly in the presence of established peripheral vascular disease.
As a general rule the main issue in vascular surgery is to optimise vascular inflow prior to surgery. The more distal the planned amputation is to be, the more important this rule becomes.
In other situations there has been something such as an embolic event that has not been revascularised in time. In this case the limb shows fixed mottling and an amputation will be needed.

Types of amputations
As the vast majority of commonly performed amputations affect the lower limbs these will be covered here.

The main categories of amputations are:
Pelvic disarticulation (hindquarter)
Above knee amputation
Gritti Stokes (through knee amputation)
Below knee amputation (using either Skew or Burgess flaps)
Syme’s amputation (through ankle)
Amputations of mid foot and digits

Choosing a level of amputation depends on:
The disease process being treated
Desired functional outcome
Co-morbidities of the patient

Above knee amputations
Quick to perform
Heal reliably
Patients regain their general health quickly
For this benefit, a functional price has to be paid and many patients over the age of 70 will never walk on an above knee prosthesis.
Above knee amputations use equal anterior-posterior flaps

Below knee amputations
Technically more challenging to perform
Heal less reliably than their above knee counterparts.
However, many more patients are able to walk using a below knee prosthesis.
In below knee amputations the two main flaps are Skew flaps or the Burgess long posterior flap. Skew flaps result in a less bulky limb that is easier to attach a prosthesis to.

It is worth remembering that whilst it may be technically feasible to offer a below knee amputation there may be circumstances where an above knee option is preferable. For example, in fixed flexion deformities of the lower limb, little functional benefit would be gained from below knee amputation surgery.

How well did you know this?
1
Not at all
2
3
4
5
Perfectly
181
Q

A Medical F1 phones you as he is concerned his patient has had a major internal bleed. The patient is 42 years old and is known to have sickle cell anaemia. His blood results are:

Hb 3.7 g /dl
Reticulocyte count 0.4%

His Hb is normally 7g/dl. What is the diagnosis?

Psoas haemorrhage

Acute sequestration

Parvovirus

Splenic haemorrhage

Acute haemolysis

A

A sudden anemia and a LOW reticulocyte count indicates parvovirus. Acute sequestration and haemolysis causes a high reticulocyte count. There is no clinical indication to suspect a bleed, therefore you can advise the F1 not to panic and to speak to the haematologists!

Sickle cell anaemia

Autosomal recessive
Single base mutation
Deoxygenated cells become sickle in shape
Causes: short red cell survival, obstruction of microvessels and infarction
Sickling is precipitated by: dehydration, infection, hypoxia
Manifest at 6 months age
Africans, Middle East, Indian
Diagnosis: Hb electrophoresis

Sickle crises

Bone pain
Pleuritic chest pain: acute sickle chest syndrome commonest cause of death
CVA, seizures
Papillary necrosis
Splenic infarcts
Priapism
Hepatic pain

Hb does not fall during a crisis, unless there is

Aplasia: parvovirus
Acute sequestration
Haemolysis

Long-term complications

Infections: Streptococcus pnemoniae
Chronic leg ulcers
Gallstones: haemolysis
Aseptic necrosis of bone
Chronic renal disease
Retinal detachment, proliferative retinopathy

Surgical complications

Bowel ischaemia
Cholecystitis
Avascular necrosis

Management

Supportive
Hydroxyurea
Repeated transfusions pre operatively
Exchange transfusion in emergencies

Sickle cell trait

Heterozygous state
Asymptomatic
Symptoms associated with extreme situations ie anaesthesia complications
Protective against Plasmodium falciparum

How well did you know this?
1
Not at all
2
3
4
5
Perfectly
182
Q

A 73 year old man develops disseminated intravascular coagulation following an abdominal aortic aneurysm repair. He receives an infusion of cryoprecipitate. What is the major constituent of this infusion?

Factor VIII

Factor IX

Protein C

Protein S

Factor V

A

A- Cryoprecipitate

  • Blood product made from plasma
    Usually transfused as 6 unit pool
    Indications include massive haemorrhage and uncontrolled bleeding due to haemophilia

Composition
Agent Quantity
Factor VIII 100IU
Fibrinogen 250mg
von Willebrand factor Variable
Factor XIII Variable

How well did you know this?
1
Not at all
2
3
4
5
Perfectly
183
Q

Which of the following local anaesthetics is not an amino amide type?

Lignocaine

Xylocaine

Procaine

Bupivacaine

Prilocaine

A

All local anaesthetics have a chemical bond linking an amine to either an amide or an ester. Most local anaesthetics are of the amino- amide types, these have a more favorable side effect profile and are more stable in solution. Procaine and benzocaine have amino - ester groups, these are metabolised by pseudocholinesterases.

Local anaesthetic agents

Lidocaine
An amide
Local anaesthetic and a less commonly used antiarrhythmic (affects Na channels in the axon)
Hepatic metabolism, protein bound, renally excreted
Toxicity: due to IV or excess administration. Increased risk if liver dysfunction or low protein states. Note acidosis causes lidocaine to detach from protein binding.
Drug interactions: Beta blockers, ciprofloxacin, phenytoin
Features of toxicity: Initial CNS over activity then depression as lidocaine initially blocks inhibitory pathways then blocks both inhibitory and activating pathways. Cardiac arrhythmias.
Increased doses may be used when combined with adrenaline to limit systemic absorption.

Cocaine
Pure cocaine is a salt, usually cocaine hydrochloride. It is supplied for local anaesthetic purposes as a paste.
It is supplied for clinical use in concentrations of 4 and 10%. It may be applied topically to the nasal mucosa. It has a rapid onset of action and has the additional advantage of causing marked vasoconstriction.
It is lipophillic and will readily cross the blood brain barrier. Its systemic effects also include cardiac arrhythmias and tachycardia.
Apart from its limited use in ENT surgery it is otherwise used rarely in mainstream surgical practice.

Bupivacaine
Bupivacaine binds to the intracellular portion of sodium channels and blocks sodium influx into nerve cells, which prevents depolarization.
It has a much longer duration of action than lignocaine and this is of use in that it may be used for topical wound infiltration at the conclusion of surgical procedures with long duration analgesic effect.
It is cardiotoxic and is therefore contra indicated in regional blockage in case the tourniquet fails.
Levobupivacaine (Chirocaine) is less cardiotoxic and causes less vasodilation.

Prilocaine
Similar mechanism of action to other local anaesthetic agents. However, it is far less cardiotoxic and is therefore the agent of choice for intravenous regional anaesthesia e.g. Biers Block.

All local anaesthetic agents dissociate in tissues and this contributes to their therapeutic effect. The dissociation constant shifts in tissues that are acidic e.g. where an abscess is present, and this reduces the efficacy.

Doses of local anaesthetics
Agent Dose plain Dose with adrenaline
Lignocaine 3mg/Kg 7mg/Kg
Bupivacaine 2mg/Kg 2mg/Kg
Prilocaine 6mg/Kg 9mg/Kg
These are a guide only as actual doses depend on site of administration, tissue vascularity and co-morbidities.

Maximum total local anaesthetic doses
Lignocaine 1% plain - 3mg/ Kg - 200mg (20ml)
Lignocaine 1% with 1 in 200,000 adrenaline - 7mg/Kg - 500mg (50ml)
Bupivacaine 0.5% - 2mg/kg- 150mg (30ml)
Maximum doses are based on ideal body weight

Effects of adrenaline
Adrenaline may be added to local anaesthetic drugs. It prolongs the duration of action at the site of injection and permits usage of higher doses (see above). It is contra indicated in patients taking MAOI’s or tricyclic antidepressants. The toxicity of bupivacaine is related to protein binding and addition of adrenaline to this drug does not permit increases in the total dose of bupivacaine, in contrast to the situation with lignocaine.

References
An excellent review is provided by:
French J and Sharp L. Local Anaesthetics. Ann R Coll Surg Engl 2012; 94: 76-80.

How well did you know this?
1
Not at all
2
3
4
5
Perfectly
184
Q

A 22 year old man is involved in a fight and is stabbed in the posterior aspect of his right leg. The knife passes into the popliteal fossa. He sustains an injury to his tibial nerve. Which muscle is least likely to be compromised as a result?

Tibialis posterior

Flexor hallucis longus

Flexor digitorum brevis

Soleus

Peroneus tertius

A

Peroneus tertius is innervated by the deep peroneal nerve.

Tibial nerve

Begins at the upper border of the popliteal fossa and is a branch of the sciatic nerve.

Root values: L4, L5, S1, S2, S3

Muscles innervated
Popliteus
Gastrocnemius
Soleus
Plantaris
Tibialis posterior
Flexor hallucis longus
Flexor digitorum brevis and longus

Terminates by dividing into the medial and lateral plantar nerves.

How well did you know this?
1
Not at all
2
3
4
5
Perfectly
185
Q

You review a 42-year-old woman 8 months following a renal transplant for focal segmental glomerulosclerosis. She is on a combination of tacrolimus, mycophenolate, and prednisolone. She has now presented with a five day history of feeling generally unwell with jaundice, fatigue and arthralgia. On examination she has jaundice, widespread lymphadenopathy and hepatomegaly. What is the most likely diagnosis?

Hepatitis C

Epstein-Barr virus

HIV

Hepatitis B

Cytomegalovirus

A

Post transplant complications

CMV: 4 weeks to 6 months post transplant
EBV: post transplant lymphoproliferative disease. > 6 months post transplant
Post transplant lymphoproliferative disorder is most commonly associated with Epstein-Barr virus. It typically occurs 6 months post transplant and is associated with high dose immunosupressant therapy. Remember cytomegalovirus presents within the first 4 weeks to 6 months post transplant.

Renal transplant:HLA typing and graft failure

The human leucocyte antigen (HLA) system is the name given to the major histocompatibility complex (MHC) in humans. It is coded for on chromosome 6.

Some basic points on the HLA system
Class 1 antigens include A, B and C. Class 2 antigens include DP,DQ and DR
When HLA matching for a renal transplant the relative importance of the HLA antigens are as follows DR > B > A

Graft survival
1 year = 90%, 10 years = 60% for cadaveric transplants
1 year = 95%, 10 years = 70% for living-donor transplants

Post-op problems
ATN of graft
Vascular thrombosis
Urine leakage
UTI

Hyperacute acute rejection
Due to antibodies against donor HLA type 1 antigens
Rarely seen due to HLA matching

Acute graft failure (< 6 months)
Usually due to mismatched HLA
Other causes include cytomegalovirus infection
Management: give steroids, if resistant use monoclonal antibodies

Causes of chronic graft failure (> 6 months)
Chronic allograft nephropathy
Ureteric obstruction
Recurrence of original renal disease (MCGN > IgA > FSGS)

How well did you know this?
1
Not at all
2
3
4
5
Perfectly
186
Q

Which of the following is not a content of the anterior triangle of the neck?

Ansa cervicalis

Vagus nerve

Anterior jugular vein

Transverse cervical nerve

Hypoglossal nerve

A

The transverse cervical nerve lies within the posterior triangle (the transverse cervical nerve divides into superior and inferior branches of the anterior margin of SCM). The anterior jugular vein is formed in the submental region and descends in the superficial fascia near the median plane. It passes inferior to enter the suprasternal space, it is linked to the contralateral anterior jugular vein by the jugular venous arch.

Anterior triangle of the neck

Boundaries
Anterior border of the Sternocleidomastoid
Lower border of mandible
Anterior midline

Sub triangles (divided by Digastric above and Omohyoid)
Muscular triangle: Neck strap muscles
Carotid triangle: Carotid sheath
Submandibular Triangle (digastric)

Contents of the anterior triangle
Digastric triangle Submandibular gland
Submandibular nodes
Facial vessels
Hypoglossal nerve
Muscular triangle Strap muscles
Jugular vein
Carotid triangle Carotid sheath (Common carotid, vagus and internal jugular vein)
Ansa cervicalis

Nerve supply to digastric muscle
Anterior: Mylohyoid nerve
Posterior: Facial nerve

How well did you know this?
1
Not at all
2
3
4
5
Perfectly
187
Q

A 55 year old man with carcinoma of the larynx is undergoing a difficult laryngectomy. The surgeons divide the thyrocervical trunk, from which of the following vessels does this structure most commonly originate?

Subclavian artery

Common carotid artery

Vertebral artery

External carotid artery

Internal carotid artery

A

The thyrocervical trunk is a branch of the subclavian artery. It arises from the first part between the subclavian artery and the inner border of scalenus anterior. It branches off the subclavian distal to the vertebral artery.

Root of the neck

Thoracic Outlet

Where the subclavian artery and vein and the brachial plexus exit the thorax and enter the arm.
They pass over the 1st rib and under the clavicle.
The subclavian vein is the most anterior structure and is immediately anterior to scalenus anterior and its attachment to the first rib.
Then subclavian artery passes between the anterior and middle scalene muscles.
At the level of the first rib, the lower cervical nerve roots combine to form the 3 trunks of the brachial plexus. The lowest trunk is formed by the union of C8 and T1, and this trunk lies directly posterior to the artery and is in contact with the superior surface of the first rib.

Thoracic outlet obstruction causes neurovascular compromise.

How well did you know this?
1
Not at all
2
3
4
5
Perfectly
188
Q

Which type of fracture is seen when a 22 year old drunk man is involved in a fight and injures his thumb when he punches his opponent?

Barton’s

Bennett’s

Galeazzi

Colles’

Smith’s

A

When a person, such as a 22-year-old man, injures his thumb by punching an opponent, the most likely type of fracture to occur is a Bennett’s fracture. This is a fracture at the base of the thumb metacarpal involving the carpometacarpal (CMC) joint, resulting in a clean break with a significant piece of bone broken off. This type of fracture is common in scenarios involving direct trauma to the thumb, such as punching.

Eponymous fractures

Colles’ fracture (dinner fork deformity)
Fall onto extended outstretched hand
Classical Colles’ fractures have the following 3 features:

  1. Transverse fracture of the radius
  2. 1 inch proximal to the radio-carpal joint
  3. Dorsal displacement and angulation

Smith’s fracture (reverse Colles’ fracture)
Volar angulation of distal radius fragment (Garden spade deformity)
Caused by falling backwards onto the palm of an outstretched hand or falling with wrists flexed

Bennett’s fracture
Intra-articular fracture of the first carpometacarpal joint
Impact on flexed metacarpal, caused by fist fights
X-ray: triangular fragment at ulnar base of metacarpal

Monteggia’s fracture
Dislocation of the proximal radioulnar joint in association with an ulna fracture
Fall on outstretched hand with forced pronation
Needs prompt diagnosis to avoid disability

Galeazzi fracture
Radial shaft fracture with associated dislocation of the distal radioulnar joint
Direct blow

Pott’s fracture
Bimalleolar ankle fracture
Forced foot eversion

Barton’s fracture
Distal radius fracture (Colles’/Smith’s) with associated radiocarpal dislocation
Fall onto extended and pronated wrist
Involvement of the joint is a defining feature

Holstein Lewis Fracture
A HolsteinLewis fracture is a fracture of the distal third of the humerus resulting in entrapment of the radial nerve.
The radial nerve is one of the major nerves of the upper limb. It innervates all of the muscles in the extensor compartments of the arm.
Conservative treatment includes reduction and use of a functional brace
Vascular injury may require open surgery

How well did you know this?
1
Not at all
2
3
4
5
Perfectly
189
Q

A 72 year old man is undergoing a left pneumonectomy for carcinoma of the bronchus. As the surgeons approach the root of the lung, which structure will lie most anteriorly (in the anatomical plane)?

Vagus nerve

Phrenic nerve

Bronchus

Pulmonary vein

Pulmonary artery

A

The phrenic nerve is the most anteriorly located structure in the lung root. The vagus nerve lies most posteriorly.

Lung anatomy

The right lung is composed of 3 lobes divided by the oblique and transverse fissures. The left lung has two lobes divided by the oblique fissure.The apex of both lungs is approximately 4cm superior to the sterno-costal joint of the first rib. Immediately below this is a sulcus created by the subclavian artery.

Peripheral contact points of the lung
Base: diaphragm
Costal surface: corresponds to the cavity of the chest
Mediastinal surface: Contacts the mediastinal pleura. Has the cardiac impression. Above and behind this concavity is a triangular depression named the hilum, where the structures which form the root of the lung enter and leave the viscus. These structures are invested by pleura, which, below the hilum and behind the pericardial impression, forms the pulmonary ligament

Right lung
Above the hilum is the azygos vein; Superior to this is the groove for the superior vena cava and right innominate vein; behind this, and nearer the apex, is a furrow for the innominate artery. Behind the hilum and the attachment of the pulmonary ligament is a vertical groove for the oesophagus; In front and to the right of the lower part of the oesophageal groove is a deep concavity for the extrapericardiac portion of the inferior vena cava.

The root of the right lung lies behind the superior vena cava and the right atrium, and below the azygos vein.

The right main bronchus is shorter, wider and more vertical than the left main bronchus and therefore the route taken by most foreign bodies.
Left lung
Above the hilum is the furrow produced by the aortic arch, and then superiorly the groove accommodating the left subclavian artery; Behind the hilum and pulmonary ligament is a vertical groove produced by the descending aorta, and in front of this, near the base of the lung, is the lower part of the oesophagus.

The root of the left lung passes under the aortic arch and in front of the descending aorta.

Inferior borders of both lungs
6th rib in mid clavicular line
8th rib in mid axillary line
10th rib posteriorly
The pleura runs two ribs lower than the corresponding lung level.

Bronchopulmonary segments
Segment number Right lung Left lung
1 Apical Apical
2 Posterior Posterior
3 Anterior Anterior
4 Lateral Superior lingular
5 Medial Inferior lingular
6 Superior (apical) Superior (apical)
7 Medial basal Medial basal
8 Anterior basal Anterior basal
9 Lateral basal Lateral basal
10 Posterior basal Posterior basal

How well did you know this?
1
Not at all
2
3
4
5
Perfectly
190
Q

Which nerve lies medially to the lobes of the thyroid gland, in the groove between the oesophagus and trachea?

Vagus nerve

External laryngeal nerve

Recurrent laryngeal nerve

Ansa cervicalis

Phrenic nerve

A

The recurrent laryngeal nerve may be injured at this site during ligation of the inferior thyroid artery.

Thyroid gland

Right and left lobes connected by isthmus
Surrounded by sheath from pretracheal layer of deep fascia
Apex: Lamina of thyroid cartilage
Base: 4th-5th tracheal ring
Pyramidal lobe: from isthmus
May be attached to foramen caecum at the base of the tongue

Relations
Anteromedially
Sternothyroid
Superior belly of omohyoid
Sternohyoid
Anterior aspect of sternocleidomastoid
Posterolaterally Carotid sheath
Medially
Larynx
Trachea
Pharynx
Oesophagus
Cricothyroid muscle
External laryngeal nerve (near superior thyroid artery)
Recurrent laryngeal nerve (near inferior thyroid artery)
Posterior
Parathyroid glands
Anastomosis of superior and inferior thyroid arteries
Isthmus
Anteriorly: Sternothyroids, sternohyoids, anterior jugular veins
Posteriorly: 2nd, 3rd, 4th tracheal rings (attached via Ligament of Berry)

Blood Supply
Arterial
Superior thyroid artery (1st branch of external carotid)
Inferior thyroid artery (from thyrocervical trunk)
Thyroidea ima (in 10% of population -from brachiocephalic artery or aorta)
Venous
Superior and middle thyroid veins - into the IJV
Inferior thyroid vein - into the brachiocephalic veins

How well did you know this?
1
Not at all
2
3
4
5
Perfectly
191
Q

A 43 year old lady is receiving chemotherapy for the treatment of metastatic breast cancer. You are called because it has become apparent that her doxorubicin infusion has extravasated. What is the most appropriate course of action?

Stop the infusion and administer dexamethasone through the infusion device

Stop the infusion and administer hyaluronidase through the infusion device

Stop the infusion and apply a cold compress to the site

Stop the infusion and apply a warm compress to the site

Stop the infusion and administer sodium bicarbonate through the infusion device

A

The application of cold compresses is indicated in doxorubicin extravasation. Warm compresses increase the risk of doxorubicin ulceration. Hyaluronidase is indicated in the extravasation of contrast media, TPN and vinca alkaloids. However, if administered following doxorubicin extravasation it will dramatically worsen the situation and is contra indicated.
Up to 50% of those sustaining severe injuries will require delayed surgical reconstruction.

Extravasation injury

Chemotherapy may be complicated by extravasation reactions in up to 6% of cases. The following chemotherapy agents are recognised causes of extravasation reactions; doxorubicin, vincristine, vinblastine, cisplatin, mitomycin and mithramycin.
Up to 30% of extravasation reactions may be complicated by the development of ulceration.
When an extravasation reaction is suspected, the infusion should be stopped and the infusing device aspirated. The extremity should be elevated. As a general rule cold compresses have been shown to reduce the incidence of subsequent ulceration with doxorubicin. Warm compresses have been found to be beneficial in extravasation of vinca alkaloids. Dimethylsulfoxide may be infused in some cases, ideally within 5 hours of the event occurring. No conclusive evidence exists to support the use of corticosteroids or sodium bicarbonate for extravasation injuries.

Extravasation of total parenteral nutrition solutions is usually managed by the local administration of hyaluronidase to the infusion site.

How well did you know this?
1
Not at all
2
3
4
5
Perfectly
192
Q

A 68 year old man presents with an ulcerated lesion on his right cheek. It is excised and on histological assessment a squamous cell carcinoma is diagnosed. It measures 25mm in diameter and is 4mm deep. Which of the following statements relating to this condition is false?

In this particular case margins of at least 6mm are required

Use of cryosurgery to treat this patients lesion would have been unsafe

Use of radiotherapy to treat this lesion would have been unsafe

This patients local recurrence rate may approach 15%

The disease usually spreads via lymphatics

A

Analysis of Statements Regarding Squamous Cell Carcinoma

To determine which statement is false regarding the squamous cell carcinoma (SCC) diagnosed in the 68-year-old man, we will analyze each statement based on current medical knowledge and guidelines.

In this particular case margins of at least 6mm are required:

For SCC, especially those that are larger or have certain risk factors, surgical excision with clear margins is critical to minimize the risk of recurrence. The recommended margin for low-risk SCCs can be around 4-5mm, but for high-risk lesions (which may include those that are larger than 2cm or deeper than 4mm), a margin of at least 6mm is often recommended. Given that this lesion measures 25mm in diameter and is 4mm deep, it would be prudent to require a margin of at least 6mm. Therefore, this statement is true.

Use of cryosurgery to treat this patient’s lesion would have been unsafe:

Cryosurgery can be used for superficial skin cancers; however, it is generally not recommended for invasive SCCs due to the potential for incomplete treatment and local recurrence. Given the size and depth of this lesion (25mm and 4mm respectively), using cryosurgery would indeed be considered unsafe as it may not adequately address the invasive nature of the cancer. Thus, this statement is true.

Use of radiotherapy to treat this lesion would have been unsafe:

Radiotherapy can be an effective treatment modality for SCC, particularly in cases where surgery is not feasible or when there are concerns about surgical margins. It can also be used post-operatively if there are positive margins or other high-risk features present. Therefore, stating that radiotherapy would be unsafe in treating this lesion does not align with current practices; thus, this statement is false.
This patient’s local recurrence rate may approach 15%:

Local recurrence rates for SCC can vary widely based on several factors including tumor size, depth, location, and adequacy of surgical margins. For high-risk lesions such as large or deeply invasive SCCs, local recurrence rates can indeed approach or exceed 15%. This aligns with clinical data suggesting higher recurrence rates in such cases; therefore, this statement is true.

The disease usually spreads via lymphatics:

SCC typically spreads through lymphatic channels rather than hematogenous routes (bloodstream). This characteristic behavior is well-documented in medical literature regarding squamous cell carcinoma; hence, this statement is also true.
Based on the analysis above:

The false statement among those provided regarding the management and characteristics of squamous cell carcinoma in this patient scenario is:
Use of radiotherapy to treat this lesion would have been unsafe

Poor prognostic factors in SCC:
Size >20mm (local recurrence rate of up to 15%)
Depth greater than 4mm (risk of metastasis up to 30%)
This man has an SCC with significant risk of metastasis. Although cryotherapy may be used to treat SCC it would be most unsafe in this setting as the lesion extends deeply. However, radiotherapy is a safe treatment modality for SCC and may be used in selected cases. It is unwise to use radiotherapy in areas prone to radionecrosis e.g. the nose.

Squamous cell carcinoma of the skin

Second most common skin malignancy
Derived from epidermal keratinocytes
Commonest in fair skinned individuals in sun exposed sites
May occur in perianal and genital skin especially in association with Human Papilloma Virus 16 and 18 infections.

Groups at high risk
Renal transplant and on immunosuppression
Individuals with HIV
Those who have received psoralen UVA therapy
Chronic wounds (Marjolins ulcer)
Xeroderma pigmentosum
Oculocutaneous albinism

Prognosis
Good Prognosis Poor prognosis
Well differentiated tumours Poorly differentiated tumours
<20mm diameter >20mm in diameter
<2mm deep >4mm deep
No associated diseases Immunosupression for whatever reason

Treatment
Surgical excision with 4mm margins if lesion <20mm in diameter. If tumour >20mm then margins should be 6mm.

How well did you know this?
1
Not at all
2
3
4
5
Perfectly
193
Q

A 20 year old man is admitted to the intensive care unit with an isolated severe head injury. A CT scan shows multiple intracerebral bleeds but no midline shift. He is intubated and ventilated. His pupils are dilated and react sluggishly to light. His heart rate is 50 beats/minute blood pressure 170/110 mmHg and his respiratory rate is set at 10 breaths/minute. The rising blood pressure is likely to be caused by:

Aortic and carotid baroreceptor stimulation

Cortisol stimulation

Renin angiotensin stimulation

Sympathetic stimulation related to blood loss

Sympathetic stimulation related to increased intra cranial pressure

A

Clinical Presentation and Context

In this scenario, we have a 20-year-old man with a severe head injury, as evidenced by the CT scan showing multiple intracerebral bleeds. The clinical signs include dilated pupils that react sluggishly to light, bradycardia (heart rate of 50 beats/minute), hypertension (blood pressure of 170/110 mmHg), and a controlled respiratory rate set at 10 breaths/minute due to intubation and ventilation.

Understanding the Physiological Responses

Intracranial Pressure (ICP): Severe head injuries often lead to increased intracranial pressure due to swelling or bleeding within the cranial cavity. This increase in ICP can stimulate various physiological responses.

Sympathetic Nervous System Activation: In response to elevated ICP, the body may activate the sympathetic nervous system as a compensatory mechanism. This activation can lead to increased systemic vascular resistance and subsequent hypertension.

Cushing’s Reflex: The combination of bradycardia and hypertension is indicative of Cushing’s reflex, which is a physiological response to increased ICP. This reflex involves:

Increased blood pressure due to sympathetic stimulation.
Bradycardia as a result of baroreceptor reflexes responding to high blood pressure.
Baroreceptor Response: While baroreceptors in the carotid arteries and aorta respond to changes in blood pressure, in this case, the primary driver for the observed hypertension is likely related to increased intracranial pressure rather than direct stimulation from baroreceptors alone.

Other Factors Considered:

Cortisol Stimulation: While cortisol can influence blood pressure regulation, it is not the primary factor in acute head trauma scenarios.
Renin-Angiotensin Stimulation: This system typically responds to low blood volume or low blood pressure situations rather than isolated increases in ICP.
Sympathetic Stimulation Related to Blood Loss: There is no indication of significant blood loss in this scenario; thus, this option is less relevant.
Given these considerations, the most plausible explanation for the rising blood pressure in this patient is:

Sympathetic stimulation related to increased intracranial pressure

This conclusion aligns with established medical knowledge regarding responses following severe head injuries and their effects on cardiovascular dynamics.

The changes seen are the result of raised ICP, its rare for head injuries to cause sufficient blood loss to affect the circulating volume.

Cushing reflex

When intra cranial pressure exceeds mean arterial pressure, compression of cerebral arterioles occurs. This results in cerebral ischaemia.
Increases in ICP results in stepwise activation of the sympathetic nervous system initially. This raises peripheral vascular resistance and results in hypertension. Cardiac output is also increased. These haemodynamic changes are detected by aortic arch baroreceptors and this results in activation of the parasympathetic nervous system. These parasympathetic effects comprise the second stage of the Cushing reflex. The Cushing reflex is a serious development that indicates imminent coning or other terminal events if not resolved quickly.

How well did you know this?
1
Not at all
2
3
4
5
Perfectly
194
Q

An 83 year old lady attends the endoscopy department for a flexible sigmoidoscopy. The endoscopist administers 7mg of intravenous midazolam for sedation. Approximately 3 minutes later the patient is noted to be hypoxic and has a respiratory rate of 5 breaths per minute. What is the most useful agent to administer at this point?

Intravenous naloxone

Oral naloxone

Intravenous flumazenil

Intravenous doxapram

Nebulised adrenaline

A

Benzodiazepine overdose with marked suppression of respiratory activity should be managed with IV flumazenil.

Midazolam

Midazolam is a short acting benzodiazepine. It is usually administered intravenously, when administered orally, dosing is less reliable. It is eliminated via the cytochrome P450 pathway in the liver. It is often used for procedural related sedation purposes. It is generally administered in doses of 2-5mg for this purpose. It is often given in combination with a short acting opiate such as fentanyl for analgesic purposes. One of the main reasons that midazolam is preferred for procedural related sedation is due to a degree of amnesia that can accompany it. If administered in excess, most patients can be managed with simple airway support measures an monitoring. However, if there is significant respiratory compromise, it can be reversed with the agent flumazenil which is a selective GABA receptor antagonist.

How well did you know this?
1
Not at all
2
3
4
5
Perfectly
195
Q

A 73 year old lady presents with large bowel obstruction. On examination, she has a rectal cancer 6cm from the anal verge which has occluded the colonic lumen. An abdominal x-ray shows a caecal diameter of 7cm. Which of the management strategies outlined below is the most appropriate?

Construction of a loop ileostomy

Construction of a loop colostomy

Construction of a venting caecostomy

Abdomino-perineal resection of the colon and rectum

Low anterior resection and loop ileostomy

A

Bowel obstruction due to RECTAL cancer should be treated by loop colostomy.
Bowel obstruction due to obstructing left sided COLON cancer is usually treated by resection of the primary lesion and formation of colostomy.
This patient should be defunctioned, definitive surgery should wait until staging is completed. A loop ileostomy will not satisfactorily decompress an acutely obstructed colon. Low rectal cancers that are obstructed should not usually be primarily resected. The obstructed colon that would be used for anastomosis would carry a high risk of anastomotic dehiscence. In addition, as this is an emergency presentation, staging may not be completed, an attempted resection may therefore compromise the circumferential resection margin, with an associated risk of local recurrence.

Colorectal cancer treatment

Patients diagnosed as having colorectal cancer should be completely staged using CT of the chest/ abdomen and pelvis. Their entire colon should have been evaluated with colonoscopy or CT colonography. Patients whose tumours lie below the peritoneal reflection should have their mesorectum evaluated with MRI.

Once their staging is complete patients should be discussed within a dedicated colorectal MDT meeting and a treatment plan formulated.

Treatment of colonic cancer
Cancer of the colon is nearly always treated with surgery. Stents, surgical bypass and diversion stomas may all be used as palliative adjuncts. Resectional surgery is the only option for cure in patients with colon cancer. The procedure is tailored to the patient and the tumour location. The lymphatic drainage of the colon follows the arterial supply and therefore most resections are tailored around the resection of particular lymphatic chains (e.g. ileo-colic pedicle for right sided tumours). Some patients may have confounding factors that will govern the choice of procedure, for example a tumour in a patient from a HNPCC family may be better served with a panproctocolectomy rather than segmental resection. Following resection the decision has to be made regarding restoration of continuity. For an anastomosis to heal the key technical factors include; adequate blood supply, mucosal apposition and no tissue tension. Surrounding sepsis, unstable patients and inexperienced surgeons may compromise these key principles and in such circumstances it may be safer to construct an end stoma rather than attempting an anastomosis.
When a colonic cancer presents with an obstructing lesion; the options are to either stent it or resect. In modern practice it is unusual to simply defunction a colonic tumour with a proximal loop stoma. This differs from the situation in the rectum (see below).
Following resection patients with risk factors for disease recurrence are usually offered chemotherapy, a combination of 5FU and oxaliplatin is common.

Rectal cancer
The management of rectal cancer is slightly different to that of colonic cancer. This reflects the rectum’s anatomical location and the challenges posed as a result. Tumours located in the rectum can be surgically resected with either an anterior resection or an abdomino - perineal resection. The technical aspects governing the choice between these two procedures can be complex to appreciate and the main point to appreciate for the MRCS is that involvement of the sphincter complex or very low tumours require APER. In the rectum a 2cm distal clearance margin is required and this may also impact on the procedure chosen. In addition to excision of the rectal tube an integral part of the procedure is a meticulous dissection of the mesorectal fat and lymph nodes (total mesorectal excision/ TME). In rectal cancer surgery invovlement of the cirumferential resection margin carries a high risk of disease recurrence. Because the rectum is an extraperitoneal structure (until you remove it that is!) it is possible to irradiate it, something which cannot be offered for colonic tumours. This has a major impact in rectal cancer treatment and many patients will be offered neoadjuvent radiotherapy (both long and short course) prior to resectional surgery. Patients with T1, 2 and 3 /N0 disease on imaging do not require irradiation and should proceed straight to surgery. Patients with T4 disease will typically have long course chemo radiotherapy. Patients presenting with large bowel obstruction from rectal cancer should not undergo resectional surgery without staging as primary treatment (very different from colonic cancer). This is because rectal surgery is more technically demanding, the anastomotic leak rate is higher and the danger of a positive resection margin in an unstaged patient is high. Therefore patients with obstructing rectal cancer should have a defunctioning loop colostomy.

Summary of procedures
The operations for cancer are segmental resections based on blood supply and lymphatic drainage. These commonly performed procedures are core knowledge for the MRCS and should be understood.

Site of cancer Type of resection Anastomosis Risk of leak
Right colon Right hemicolectomy Ileo-colic Low <5%
Transverse Extended right hemicolectomy Ileo-colic Low <5%
Splenic flexure Extended right hemicolectomy Ileo-colic Low <5%
Splenic flexure Left hemicolectomy Colo-colon 2-5%
Left colon Left hemicolectomy Colo-colon 2-5%
Sigmoid colon High anterior resection Colo-rectal 5%
Upper rectum Anterior resection (TME) Colo-rectal 5%
Low rectum Anterior resection (Low TME) Colo-rectal
(+/- Defunctioning stoma) 10%
Anal verge Abdomino-perineal excision of colon and rectum None n/a

In the emergency setting, where the bowel has perforated, the risk of an anastomotic breakdown is much greater, particularly when the anastomosis is colon-colon. In this situation, an end colostomy is often safer and can be reversed later. When resection of the sigmoid colon is performed and an end colostomy is fashioned the operation is referred to as a Hartmans procedure. Whilst left sided resections are more risky, ileo-colic anastomoses are relatively safe even in the emergency setting and do not need to be defunctioned.

References
A review of the diagnosis and management of colorectal cancer and a summary of the UK National Institute of Clinical Excellence guidelines is provided in:
Poston G, et al . Diagnosis and management of colorectal cancer: summary of NICE guidance. BMJ 2011: 343: d 6751.

How well did you know this?
1
Not at all
2
3
4
5
Perfectly
196
Q

A 27 year old lady presents with a breast lump. She has previously undergone a breast augmentation with an implant. Which of these imaging techniques is the most appropriate next step?

Ultrasound

CT scanning

MRI

PET CT

Mammography

A

Unless there are concerns about implant rupture, the imaging of a breast lump in a young patient with implants would be USS initially. If this is not conclusive then MRI should be performed. Remember that MRI is the modality of choice where an implant related complication is suspected or there are concerns about malignant or pre malignant changes and none of these apply here.

Breast imaging

Breast imaging is undertaken with a combination of USS and mammography in most women presenting with a palpable lump. In younger patients, the denser breasts encountered in this group may make mammography less informative.

In women who present with breast implants with a palpable lump, the imaging of choice remains the same. However, specialized imaging techniques may be needed to obtain optimal mammographic views. Where there are specific concerns about a breast implant, rather than a lump, the imaging modality of choice is MRI scanning.
MRI scanning may be beneficial in screening younger patients with a family history and also in patients with lobular cancers who are being considered for breast conserving surgery.

How well did you know this?
1
Not at all
2
3
4
5
Perfectly
197
Q

The pathogenicity of the tubercle bacillus is due to which of the following?

Necrosis caused by expanding granulomas

Ability to multiply within fibroblasts

Delayed hypersensitivity reaction against bacteria

Effect of antibody response

Direct toxic effect on host cells

A

Answer: Delayed hypersensitivity reaction against bacteria

Pathogenicity of the Tubercle Bacillus

The pathogenicity of the tubercle bacillus, scientifically known as Mycobacterium tuberculosis, is a complex process that involves several mechanisms. To determine which option best describes the primary cause of its pathogenicity, we will analyze each provided choice in detail.

Necrosis caused by expanding granulomas:

Granulomas are organized structures formed in response to chronic inflammation, typically seen in infections like tuberculosis. They consist of macrophages, lymphocytes, and other immune cells that attempt to contain the infection. While necrosis can occur within granulomas due to cell death and tissue damage, it is not the primary mechanism through which M. tuberculosis exerts its pathogenic effects.
Ability to multiply within fibroblasts:

Fibroblasts are connective tissue cells that play a role in wound healing and tissue repair. However, M. tuberculosis primarily resides and replicates within macrophages rather than fibroblasts. The ability to survive and multiply inside macrophages is crucial for its pathogenicity but does not specifically highlight fibroblasts as a significant factor.
Delayed hypersensitivity reaction against bacteria:

The immune response to M. tuberculosis includes delayed-type hypersensitivity (DTH), which is mediated by T cells and leads to the formation of granulomas. This immune response is an important aspect of how the body attempts to control the infection; however, it is more a consequence of infection rather than a direct cause of pathogenicity.
Effect of antibody response:

Antibodies play a role in the immune response against many pathogens; however, M. tuberculosis has evolved mechanisms to evade antibody-mediated responses effectively. The bacterium’s cell wall contains mycolic acids that protect it from being effectively targeted by antibodies.
Direct toxic effect on host cells:

While M. tuberculosis does not produce traditional toxins like some other bacteria, it can induce cytotoxic effects indirectly through immune-mediated mechanisms and inflammation caused by its presence in host tissues.
After analyzing these options, it becomes clear that while all these factors contribute to the overall disease process associated with tuberculosis, none directly encapsulate the primary mechanism behind the pathogenicity of M. tuberculosis. However, if we consider how this organism survives and thrives within host tissues despite an immune response aimed at eliminating it, we find that:

The most relevant factor among those listed would be related to its ability to evade destruction within macrophages (not explicitly mentioned but implied) leading to chronic infection and delayed hypersensitivity reactions as part of the immune response.
Thus, considering all aspects discussed:

The pathogenicity of the tubercle bacillus is primarily due to its ability to multiply within host cells (macrophages) and elicit a delayed hypersensitivity reaction against itself, but since “ability to multiply within fibroblasts” was one option presented:

Mycobacteria stimulate a specific T cell response of cell mediated immunity. This is effective in reducing the infection, the delayed hypersensitivity also damages tissues. Necrosis occurs in TB but is usually within the granuloma.

Tuberculosis pathology

Is a form of primary chronic inflammation, caused by the inability of macrophages to kill the Mycobacterium tuberculosis.
The macrophages often migrate to regional lymph nodes, the lung lesion plus affected lymph nodes is referred to as a Ghon complex.
This leads to the formation of a granuloma which is a collection of epithelioid histiocytes.
There is the presence of caseous necrosis in the centre.
The inflammatory response is mediated by a type 4 hypersensitivity reaction.
In healthy individuals the disease may be contained, in the immunocompromised disseminated (miliary TB) may occur.

Diagnosis
Waxy membrane of mycobacteria prevents binding with normal stains. Ziehl - Neelsen staining is typically used.
Culture based methods take far longer.

How well did you know this?
1
Not at all
2
3
4
5
Perfectly
198
Q

Which of the following is a recognised feature of ketamine when used as an anaesthetic agent?

Malignant hyperpyrexia

Adrenal suppression

Myocardial depression

Dissociative anaesthesia

Marked respiratory depression

A

Unlike most anaesthetic agents ketamine does not cause myocardial or marked respiratory depression. It is not associated with the adrenal suppression that may occur with etomidate. It is however, associated with a state of dissociative anaesthesia which patients may find distressing.

Anaesthetic agents

The table below summarises some of the more commonly used IV induction agents
Agent Specific features
Propofol
Rapid onset of anaesthesia
Pain on IV injection
Rapidly metabolised with little accumulation of metabolites
Proven anti emetic properties
Moderate myocardial depression
Widely used especially for maintaining sedation on ITU, total IV anaesthesia and for daycase surgery
Sodium thiopentone
Extremely rapid onset of action making it the agent of choice for rapid sequence of induction
Marked myocardial depression may occur
Metabolites build up quickly
Unsuitable for maintenance infusion
Little analgesic effects
Ketamine
May be used for induction of anaesthesia
Has moderate to strong analgesic properties
Produces little myocardial depression making it a suitable agent for anaesthesia in those who are haemodynamically unstable
May induce state of dissociative anaesthesia resulting in nightmares
Etomidate
Has favorable cardiac safety profile with very little haemodynamic instability
No analgesic properties
Unsuitable for maintaining sedation as prolonged (and even brief) use may result in adrenal suppression
Post operative vomiting is common

How well did you know this?
1
Not at all
2
3
4
5
Perfectly
199
Q

During an Ivor Lewis Oesophagectomy for carcinoma of the lower third of the oesophagus which structure is divided to allow mobilisation of the oesophagus?

Vagus nerve

Azygos vein

Right inferior lobar bronchus

Phrenic nerve

Pericardiophrenic artery

A

The azygos vein is routinely divided during an oesophagectomy to allow mobilisation. It arches anteriorly to insert into the SVC on the right hand side.

Treatment of oesophageal cancer

  • In general resections are not offered to those patients with distant metastasis, and usually not to those with N2 disease.
    Local nodal involvement is not in itself a contra indication to resection.
    Surgical resection is the mainstay of treatment.
    Neoadjuvent chemotherapy is given in most cases prior to surgery.
    In situ disease may be managed by endoscopic mucosal resection, although this is still debated.
    In patients with lower third lesions an Ivor - Lewis type procedure is most commonly performed. Very distal tumours may be suitable to a transhiatal procedure. Which is an attractive option as the penetration of two visceral cavities required for an Ivor- Lewis type procedure increases the morbidity considerably.
    More proximal lesions will require a total oesphagectomy (Mckeown type) with anastomosis to the cervical oesophagus.
    Patients with unresectable disease may derive benefit from local ablative procedures, palliative chemotherapy or stent insertion.

Operative details of Ivor- Lewis procedure
Combined laparotomy and right thoracotomy

Indication
Lower and middle third oesophageal tumours

Preparation
Staging with a combination of CT chest abdomen and pelvis- if no metastatic disease detected then patients will undergo a staging laparoscopy to detect peritoneal disease.
If both these modalities are negative then patients will finally undergo a PET CT scan to detect occult metastatic disease. Only in those whom no evidence of advanced disease is detected will proceed to resection.
Patients receive a GA, double lumen endotracheal tube to allow for lung deflation, CVP and arterial monitoring.

Procedure
A rooftop incision is made to access the stomach and duodenum.

Laparotomy To mobilize the stomach
The greater omentum is incised away from its attachment to the right gastroepiploic vessels along the greater curvature of the stomach.
Then the short gastric vessels are ligated and detached from the greater curvature from the spleen.
The lesser omentum is incised, preserving the right gastric artery.
The retroperitoneal attachments of the duodenum in its second and third portions are incised, allowing the pylorus to reach the oesophageal hiatus. Some surgeons perform a pyloroplasty at this point to facilitate gastric emptying.
The left gastric vessels are then ligated, avoiding any injury to the common hepatic or splenic arteries. Care must be taken to avoid inadvertently devascularising the liver owing to variations in anatomy.

Right Thoracotomy Oesophageal resection and oesophagogastric anastomosis
Through 5th intercostal space
Dissection performed 10cm above the tumour
This may involve transection of the azygos vein.
The oesophagus is then removed with the stomach creating a gastric tube.
An anastomosis is created.

The chest is closed with underwater seal drainage and tube drains to the abdominal cavity.

Post operatively
Patients will typically recover in ITU initially.
A nasogastric tube will have been inserted intraoperatively and must remain in place during the early phases of recovery.
Post operatively these patients are at relatively high risk of developing complications:

  • Atelectasis- due to the effects of thoracotomy and lung collapse
  • Anastomotic leakage. The risk is relatively high owing to the presence of a relatively devascularised stomach. Often the only blood supply is from the gastroepiploic artery as all others will have been divided. If a leak does occur then many will attempt to manage conservatively with prolonged nasogastric tube drainage and TPN. The reality is that up to 50% of patients developing an anastomotic leak will not survive to discharge.
  • Delayed gastric emptying (may be avoided by performing a pyloroplasty).
How well did you know this?
1
Not at all
2
3
4
5
Perfectly
200
Q

A 28 year old man has a long history of recurrent chest infections. On examination, he is noted to have no palpable vas deferens. However, both testes are located within the scrotum. What is the most likely underlying disease association?

Kleinfelters syndrome

Kallmann syndrome

Cystic fibrosis

Coeliac disease

Gardners syndrome

A

99% of males with cystic fibrosis will have absent vas.

Underlying Disease Association in a 28-Year-Old Man with Recurrent Chest Infections and No Palpable Vas Deferens

To determine the most likely underlying disease association for the clinical scenario presented, we need to analyze the symptoms and their implications step by step.

Clinical Features

Recurrent Chest Infections: This symptom suggests an underlying issue with respiratory function or immune response. In particular, recurrent infections can be associated with conditions that affect mucous production or ciliary function.

No Palpable Vas Deferens: The absence of the vas deferens is a significant finding. This condition is known as congenital bilateral absence of the vas deferens (CBAVD), which is commonly associated with certain genetic disorders.

Testes Located Within the Scrotum: The presence of both testes in the scrotum indicates that there is no issue with testicular descent, which rules out some conditions like cryptorchidism.

Differential Diagnosis

Now let’s evaluate each of the potential associations provided:

Klinefelter Syndrome: This genetic condition (47,XXY) typically presents with hypogonadism, gynecomastia, and infertility but does not specifically correlate with recurrent chest infections or absent vas deferens.

Kallmann Syndrome: This condition involves hypogonadotropic hypogonadism and anosmia (loss of smell). While it can lead to infertility due to low testosterone levels, it does not typically present with recurrent chest infections or absent vas deferens.

Cystic Fibrosis: Cystic fibrosis is a genetic disorder caused by mutations in the CFTR gene, leading to thickened secretions in various organs including the lungs and pancreas. Patients often experience recurrent respiratory infections due to obstructed airways and may also have CBAVD due to abnormal development of the reproductive tract.

Coeliac Disease: This autoimmune disorder primarily affects the gastrointestinal system and does not typically present with absent vas deferens or recurrent chest infections as primary symptoms.

Gardner’s Syndrome: This is a genetic disorder characterized by multiple polyps in the colon along with other tumors and skin lesions but does not correlate directly with respiratory issues or absent vas deferens.

Conclusion

Given that this patient has recurrent chest infections along with congenital bilateral absence of the vas deferens, cystic fibrosis emerges as the most likely underlying disease association due to its well-documented connection between respiratory complications and reproductive tract anomalies such as CBAVD.

Therefore, based on this analysis:

Cystic fibrosis

Absence of the vas deferens

Absence of the vas may be unilateral or bilateral
Cystic fibrosis CFTR gene mutations are the cause in 40% of cases
Some non CF cases are due to unilateral renal agenesis
Sperm harvesting may allow for assisted conception

How well did you know this?
1
Not at all
2
3
4
5
Perfectly
201
Q

An infant is admitted with symptoms and signs of respiratory infection and is found to have several posterior rib fractures on chest radiograph. He was born prematurely at 37 weeks’ gestation and was observed overnight on the special care baby unit for tachypnoea which settled by the following day. On assessment, it is also apparent that his head circumference has increased at an excessive rate and has crossed 3 centiles since birth. What is the most likely underlying issue?

Accidental fracture

Pagets disease

Myeloproliferative disorder

Non accidental injury

Osteomalacia

A

Assessment of the Clinical Scenario

In this clinical scenario, we have an infant presenting with respiratory infection symptoms and posterior rib fractures identified on a chest radiograph. The infant was born prematurely at 37 weeks’ gestation and had a transient episode of tachypnoea that resolved quickly. Additionally, there is a notable increase in head circumference that has crossed three centiles since birth.

Step 1: Analyzing the Symptoms

Respiratory Infection: This can be common in infants, especially those born prematurely due to underdeveloped lungs.
Posterior Rib Fractures: The presence of rib fractures in an infant raises concern for potential underlying issues. In infants, rib fractures can occur due to various reasons, including accidental trauma or non-accidental injury (abuse).
Increased Head Circumference: A rapid increase in head circumference could indicate conditions such as hydrocephalus or other intracranial pathology.
Step 2: Evaluating Possible Causes

Accidental Fracture: While possible, the specific location of the fractures (posterior ribs) and the context suggest a higher likelihood of non-accidental injury.

Paget’s Disease: This condition primarily affects older adults and is characterized by abnormal bone remodeling. It is not relevant in this case involving an infant.

Myeloproliferative Disorder: These disorders involve excessive production of blood cells but do not typically present with rib fractures or increased head circumference as primary symptoms.

Non-Accidental Injury: Given the combination of posterior rib fractures and rapid increase in head circumference, non-accidental injury (child abuse) becomes a significant consideration. Rib fractures are often associated with physical abuse in infants, particularly when they are located posteriorly.

Osteomalacia: This condition involves softening of bones due to vitamin D deficiency but would not typically present with isolated rib fractures or increased head circumference without other systemic signs.

Step 3: Conclusion

Considering all these factors, the most likely underlying issue for this infant is non-accidental injury. The combination of posterior rib fractures and excessive increase in head circumference strongly suggests that there may be underlying trauma or abuse rather than benign causes.

Final Answer: Non accidental injury

Posterior rib fractures are extremely unusual in neonates. The change in head size may be accounted for by hydrocephalus which may occur as a sequelae from head injury.

Paediatric fractures

Paediatric fracture types
Type Injury pattern
Complete fracture Both sides of cortex are breached
Toddlers fracture Oblique tibial fracture in infants
Plastic deformity Stress on bone resulting in deformity without cortical disruption
Greenstick fracture Unilateral cortical breach only
Buckle fracture Incomplete cortical disruption resulting in periosteal haematoma only

Growth plate fractures
In paediatric practice fractures may also involve the growth plate and these injuries are classified according to the Salter- Harris system (given below):

Type Injury pattern
I Fracture through the physis only (x-ray often normal)
II Fracture through the physis and metaphysis
III Fracture through the physis and epiphyisis to include the joint
IV Fracture involving the physis, metaphysis and epiphysis
V Crush injury involving the physis (x-ray may resemble type I, and appear normal)

As a general rule it is safer to assume that growth plate tenderness is indicative of an underlying fracture even if the x-ray appears normal. Injuries of Types III, IV and V will usually require surgery. Type V injuries are often associated with disruption to growth.

Non accidental injury
Delayed presentation
Delay in attaining milestones
Lack of concordance between proposed and actual mechanism of injury
Multiple injuries
Injuries at sites not commonly exposed to trauma
Children on the at risk register

Pathological fractures
Genetic conditions, such as osteogenesis imperfecta, may cause pathological fractures.

Osteogenesis imperfecta
Defective osteoid formation due to congenital inability to produce adequate intercellular substances like osteoid, collagen and dentine.
Failure of maturation of collagen in all the connective tissues.
Radiology may show translucent bones, multiple fractures, particularly of the long bones, wormian bones (irregular patches of ossification) and a trefoil pelvis.

Subtypes
Type I The collagen is normal quality but insufficient quantity.
Type II- Poor collagen quantity and quality.
Type III- Collagen poorly formed. Normal quantity.
Type IV- Sufficient collagen quantity but poor quality.

Osteopetrosis
Bones become harder and more dense.
Autosomal recessive condition.
It is commonest in young adults.
Radiology reveals a lack of differentiation between the cortex and the medulla described as marble bone.

How well did you know this?
1
Not at all
2
3
4
5
Perfectly
202
Q

Secretions from which of the following will contain the highest levels of potassium?

Rectum

Ileum

Gallbladder

Pancreas

Stomach

A

The rectum has the potential to generate secretions rich in potassium. This is the rationale behind administration of resins for hyperkalaemia and the development of hypokalaemia in patients with villous adenoma of the rectum.

Potassium secretion -GI tract

Potassium secretions
Salivary glands Variable may be up to 60mmol/L
Stomach 10 mmol/L
Bile 5 mmol/L
Pancreas 4-5 mmol/L
Small bowel 10 mmol/L
Rectum 30 mmol/L

The above table provides average figures only and the exact composition varies depending upon the existence of disease, serum aldosterone levels and serum pH.

A key point to remember for the exam is that gastric potassium secretions are low. Hypokalaemia may occur in vomiting, usually as a result of renal wasting of potassium, not because of potassium loss in vomit.

How well did you know this?
1
Not at all
2
3
4
5
Perfectly
203
Q

A 28 year old rugby player injures his right humerus and on examination is noted to have a minor sensory deficit overlying the point of deltoid insertion into the humerus. Which of the nerves listed below is most likely to have been affected?

Radial

Axillary

Musculocutaneous

Median

Subscapular

A

This patch of skin is supplied by the axillary nerve

Axillary nerve

Terminal branch of the posterior cord of the brachial plexus
Root values C5 and C6
Descends posterior to the axillary artery at the lower border of subscapularis and then passes through quadrangular space with the posterior circumflex humeral vessels
Divides into anterior and posterior branches
Innervates deltoid muscle and small patch of skin over deltoid

How well did you know this?
1
Not at all
2
3
4
5
Perfectly
204
Q

A 53 year old man with a chronically infected right kidney is due to undergo a nephrectomy. Which of the following structures would be encountered first during a posterior approach to the hilum of the right kidney?

Right renal artery

Ureter

Right renal vein

Inferior vena cava

Right testicular vein

A

The ureter is the most posterior structure at the hilum of the right kidney and would therefore be encountered first during a posterior approach.

Renal arteries

The right renal artery is longer than the left renal artery
The renal vein/artery/pelvis enter the kidney at the hilum

Relations
Right Anterior- IVC, right renal vein, the head of the pancreas, and the descending part of the duodenum
Left Anterior- left renal vein, the tail of the pancreas

Branches
The renal arteries are direct branches off the aorta (upper border of L2- right side and L1 - left side)
In 30% there may be accessory arteries (mainly left side). Instead of entering the kidney at the hilum, they usually pierce the upper or lower part of the organ.
Before reaching the hilum of the kidney, each artery divides into four or five segmental branches (renal vein anterior and ureter posterior); which then divide within the sinus into lobar arteries supplying each pyramid and cortex.
Each vessel gives off some small inferior suprarenal branches to the suprarenal gland, the ureter, and the surrounding cellular tissue and muscles.

How well did you know this?
1
Not at all
2
3
4
5
Perfectly
205
Q

Which virus is associated with Kaposi’s sarcoma?

Human herpes virus 8

Human papillomavirus 16

Human T-lymphotropic virus 1

Epstein-Barr virus

Human papillomavirus 18

A

Virus Associated with Kaposi’s Sarcoma

Kaposi’s sarcoma (KS) is a type of cancer that primarily affects the skin, but can also involve other organs. It is characterized by the development of tumors that appear as purple or brown lesions on the skin and can be associated with significant morbidity. Understanding the viral etiology of Kaposi’s sarcoma is crucial for both diagnosis and treatment.

Human Herpes Virus 8 (HHV-8)

The primary virus associated with Kaposi’s sarcoma is Human herpes virus 8 (HHV-8), also known as Kaposi’s sarcoma-associated herpesvirus (KSHV). HHV-8 was discovered in 1994 and has since been established as a necessary factor in the development of KS, particularly in immunocompromised individuals, such as those with HIV/AIDS.

Mechanism of Action: HHV-8 infects endothelial cells, which line blood vessels, leading to their proliferation and the formation of vascular tumors characteristic of KS. The virus encodes several proteins that interfere with normal cellular processes, promoting cell survival and proliferation while inhibiting apoptosis (programmed cell death).

Epidemiology: The prevalence of HHV-8 varies geographically and demographically. It is more common in certain populations, including men who have sex with men and individuals from regions such as sub-Saharan Africa where KS is endemic.

Clinical Presentation: In patients who are immunocompromised, particularly those with HIV/AIDS, KS can present aggressively. In contrast, in immunocompetent individuals, KS may remain asymptomatic or present more indolently.

Other Viruses Considered

While HHV-8 is the primary virus linked to Kaposi’s sarcoma, other viruses listed in the question have different associations:

Human Papillomavirus 16 & 18 (HPV): These viruses are primarily associated with cervical cancer and other anogenital cancers but are not directly linked to Kaposi’s sarcoma.

Human T-Lymphotropic Virus 1 (HTLV-1): This virus is associated with adult T-cell leukemia/lymphoma but does not have a direct connection to KS.

Epstein-Barr Virus (EBV): While EBV has been implicated in various lymphoproliferative disorders and some forms of lymphoma, it does not have a direct causal relationship with Kaposi’s sarcoma.

In conclusion, after analyzing the associations between these viruses and Kaposi’s sarcoma:

The correct answer is: Human herpes virus 8

Oncoviruses

Viruses which cause cancer
These may be detected on blood test and prevented by vaccine

These are the main types of oncoviruses and their diseases:

Oncovirus Cancer
Epstein-Barr virus Burkitt’s lymphoma
Hodgkin’s lymphoma
Post transplant lymphoma
Nasopharyngeal carcinoma
Human papillomavirus 16/18 Cervical cancer
Anal cancer
Penile cancer
Vulval cancer
Oropharyneal cancer
Human herpes virus 8 Kaposi’s sarcoma
Hepatitis B virus Hepatocellular carcinoma
Hepatitis C virus Hepatocellular carcinoma
Human T-lymphotropic virus 1 Tropical spastic paraparesis
Adult T cell leukaemia

How well did you know this?
1
Not at all
2
3
4
5
Perfectly
206
Q

A 57 year old lady has suffered from lymphoedema for many years. The left leg is swollen to the mid thigh. Severe limb deformity has developed as a result of process and in spite of compression hosiery. Lymphoscintography shows no patent lymphatics in the proximal leg. The overlying skin is healthy. Which of the following options would give the best long term outcome?

Multilayer compression bandages

Charles operation

Homans operation

Long term loop diuretic therapy

Amputation

A

Surgery is indicated in less than 10% of cases. However, severe deformity is one of the indications for surgery. Lymphovenous anastomosis is indicated where the proximal lymphatics are not patent. When the overlying skin is healthy (and limb deformity a problem), a Homans procedure is a reasonable first line operative option.
Lymphoedema

  • Due to impaired lymphatic drainage in the presence of normal capillary function.
    Lymphoedema causes the accumulation of protein rich fluid, subdermal fibrosis and dermal thickening.
    Characteristically fluid is confined to the epifascial space (skin and subcutaneous tissues); muscle compartments are free of oedema. It involves the foot, unlike other forms of oedema. There may be a ‘buffalo hump’ on the dorsum of the foot and the skin cannot be pinched due to subcutaneous fibrosis.

Causes of lymphoedema

Primary
Congenital < 1 year: sporadic, Milroy’s disease
Onset 1-35 years: sporadic, Meige’s disease
> 35 years: Tarda
Secondary
Bacterial/fungal/parasitic infection (filariasis)
Lymphatic malignancy
Radiotherapy to lymph nodes
Surgical resection of lymph nodes
DVT
Thrombophlebitis

Indications for surgery
Marked disability or deformity from limb swelling
Lymphoedema caused by proximal lymphatic obstruction with patent distal lymphatics suitable for a lymphatic drainage procedure
Lymphocutaneous fistulae and megalymphatics

Procedures
Homans operation Reduction procedure with preservation of overlying skin (which must be in good condition). Skin flaps are raised and the underlying tissue excised. Limb circumference typically reduced by a third.
Charles operation All skin and subcutaneous tissue around the calf are excised down to the deep fascia. Split skin grafts are placed over the site. May be performed if overlying skin is not in good condition. Larger reduction in size than with Homans procedure.
Lymphovenous anastamosis Identifiable lymphatics are anastomosed to sub dermal venules. Usually indicated in 2% of patients with proximal lymphatic obstruction and normal distal lymphatics.

How well did you know this?
1
Not at all
2
3
4
5
Perfectly
207
Q

Which option is not recommended during the management of compartment syndrome?

Anticoagulation

Keep limb level with the body

Intravenous fluids

Pain control

Fasciotomy

A

Anticoagulation will worsen compartment syndrome.

Compartment syndrome

  • This is a particular complication that may occur following fractures (or following ischaemia re-perfusion injury in vascular patients). It is characterised by raised pressure within a closed anatomical space.
    The raised pressure within the compartment will eventually compromise tissue perfusion resulting in necrosis. The two main fractures carrying this complication include supracondylar fractures and tibial shaft injuries.

Symptoms and signs
Pain, especially on movement (even passive)
Parasthesiae
Pallor may be present
Arterial pulsation may still be felt as the necrosis occurs as a result of microvascular compromise
Paralysis of the muscle group may occur

Diagnosis
Is made by measurement of intracompartmental pressure measurements. Pressures in excess of 20mmHg are abnormal and >40mmHg is diagnostic.

Treatment
This is essentially prompt and extensive fasciotomies
In the lower limb the deep muscles may be inadequately decompressed by the inexperienced operator when smaller incisions are performed
Myoglobinuria may occur following fasciotomy and result in renal failure and for this reason these patients require aggressive IV fluids
Where muscle groups are frankly necrotic at fasciotomy they should be debrided and amputation may have to be considered
Death of muscle groups may occur within 4-6 hours

How well did you know this?
1
Not at all
2
3
4
5
Perfectly
208
Q

As it exits the axilla the radial nerve passes under the inferior border of which of the muscles listed below?

Supraspinatus

Infraspinatus

Teres major

Deltoid

Pectoralis major

A

The radial nerve passes through the triangular interval to leave the axilla. The superior border of this is bounded by the teres major muscle to which the radial nerve is closely related.

Radial nerve

Continuation of posterior cord of the brachial plexus (root values C5 to T1)

Path
In the axilla: lies posterior to the axillary artery on subscapularis, latissimus dorsi and teres major.
Enters the arm between the brachial artery and the long head of triceps (medial to humerus).
Spirals around the posterior surface of the humerus in the groove for the radial nerve.
At the distal third of the lateral border of the humerus it then pierces the intermuscular septum and descends in front of the lateral epicondyle.
At the lateral epicondyle it lies deeply between brachialis and brachioradialis where it then divides into a superficial and deep terminal branch.
Deep branch crosses the supinator to become the posterior interosseous nerve.

Regions innervated
Motor (main nerve)
Triceps
Anconeus
Brachioradialis
Extensor carpi radialis
Motor (posterior interosseous branch)
Supinator
Extensor carpi ulnaris
Extensor digitorum
Extensor indicis
Extensor digiti minimi
Extensor pollicis longus and brevis
Abductor pollicis longus
Sensory The area of skin supplying the proximal phalanges on the dorsal aspect of the hand is supplied by the radial nerve (this does not apply to the little finger and part of the ring finger)

Muscular innervation and effect of denervation
Anatomical location Muscle affected Effect of paralysis
Shoulder Long head of triceps Minor effects on shoulder stability in abduction
Arm Triceps Loss of elbow extension
Forearm Supinator
Brachioradialis
Extensor carpi radialis longus and brevis

How well did you know this?
1
Not at all
2
3
4
5
Perfectly
209
Q

Through which of the structures listed below does the axillary nerve pass?

Quadrangular space

Triangular space

Subclavicular space

Sub pectoral space

Intercostal space

A

Axillary nerve passes through the quadrangular space
Axillary nerve

Terminal branch of the posterior cord of the brachial plexus
Root values C5 and C6
Descends posterior to the axillary artery at the lower border of subscapularis and then passes through quadrangular space with the posterior circumflex humeral vessels
Divides into anterior and posterior branches
Innervates deltoid muscle and small patch of skin over deltoid

How well did you know this?
1
Not at all
2
3
4
5
Perfectly
210
Q

A 75 year old man presents with locally advanced carcinoma of the prostate and vertebral body metastasis with impending spinal cord compression. Which of the following agents (if used in isolation) carries the greatest risk of worsening his symptoms in the short term?

Surgical orchidectomy

Cyproterone acetate

Luteinising hormone releasing hormone analogues

Flutamide

None of the above

A

LHRH analogues may cause flare of metastatic disease and anti androgens should be administered to counter this. Surgical orchidectomy reduces testosterone levels within 8 hours (but fails to reduce adrenal androgen release). Cyproterone and flutamide are androgen blockers that may be considered as add on therapy to reduce the risk of tumour flare when commencing treatment with LH RH analogues.
Prostate Cancer

Prostate Cancer
This is a common condition and up to 30,000 men are diagnosed with the condition each year. Up to 9,000 will die in in the UK from the condition per year.

Diagnosis
Early prostate cancers have few symptoms.
Metastatic disease may present as bone pain.
Locally advanced disease may present as pelvic pain or with urinary symptoms.
Prostate specific antigen measurement
Digital rectal examination
Trans rectal USS (+/- biopsy)
MRI/ CT and bone scan for staging.

PSA Test
The normal upper limit for PSA is 4ng/ml. However, in this group will lie patients with benign disease and some with localised prostate cancer. False positives may be due to prostatitis, UTI, BPH, vigorous DRE.
The percentage of free: total PSA may help to distinguish benign disease from cancer. Values of <20% are suggestive of cancer and biopsy is advised.

Pathology
95% adenocarcinoma
In situ malignancy is sometimes found in areas adjacent to cancer. Multiple biopsies needed to call true in situ disease.
Often multifocal- 70% lie in the peripheral zone.
Graded using the Gleason grading system, two grades awarded 1 for most dominant grade (on scale of 1-5) and 2 for second most dominant grade (scale 1-5). The two added together give the Gleason score. Where 2 is best prognosis and 10 the worst.
Lymphatic spread occurs first to the obturator nodes and local extra prostatic spread to the seminal vesicles is associated with distant disease.

Treatment
Watch and wait- Elderly, multiple co-morbidities, low Gleason score
Radiotherapy (External)- Both potentially curative and palliative therapy possible. However, radiation proctitis and rectal malignancy are late problems. Brachytherapy is a modification allowing internal radiotherapy.
Surgery- Radical prostatectomy. Surgical removal of the prostate is the standard treatment for localised disease. The robot is being used increasingly for this procedure. As well as the prostate the obturator nodes are also removed to complement the staging process. Erectile dysfunction is a common side effect. Survival may be better than with radiotherapy (see references). Functional outcomes are better when a robotic approach is used.
Hormonal therapy- Testosterone stimulates prostate tissue and prostatic cancers usually show some degree of testosterone dependence. 95% of testosterone is derived from the testis and bilateral orchidectomy may be used for this reason. Pharmacological alternatives include LHRH analogues and anti androgens (which may be given in combination).
In the UK the National Institute for Clinical Excellence (NICE) suggests that active surveillance is the preferred option for low risk men. It is particularly suitable for men with clinical stage T1c, Gleason score 3+3 and PSA density < 0.15 ng/ml/ml who have cancer in less than 50% of their biopsy cores, with < 10 mm of any core involved.
Candidates for active surveillance should:
have had at least 10 biopsy cores taken
have at least one re-biopsy.
If men on active surveillance show evidence of disease progression, offer radical treatment. Treatment decisions should be made with the man, taking into account co-morbidities and life expectancy.

How well did you know this?
1
Not at all
2
3
4
5
Perfectly
211
Q

During a difficult femoro-popliteal bypass operation the surgeon inadvertently places a clamp across the femoral nerve. It remains there for most of the procedure. At the end of the operation the nerve is inspected, it is in continuity but has evidence of being crushed. Which of the following is most likely to occur over the following weeks?

Wallerian degeneration

Rapid restoration of neuronal function because the axon itself is intact

Normal but delayed neuronal transmission due to disruption of the myelin

Absence of neuroma formation

None of the above

A

A neuronal injury such as this will result in Wallerian degeneration even though the nerve remains in continuity. Neuromas may well form.

Nerve injury

There are 3 types of nerve injury:
Neuropraxia
Nerve intact but electrical conduction is affected
Full recovery
Autonomic function preserved
Wallerian degeneration does not occur
Axonotmesis
Axon is damaged and the myelin sheath is preserved. The connective tissue framework is not affected.
Wallerian degeneration occurs.
Neurotmesis
Disruption of the axon, myelin sheath and surrounding connective tissue.
Wallerian degeneration occurs.

Wallerian Degeneration
Axonal degeneration distal to the site of injury.
Typically begins 24-36 hours following injury.
Axons are excitable prior to degeneration occurring.
Myelin sheath degenerates and is phagocytosed by tissue macrophages.

Nerve repair
Neuronal repair may only occur physiologically where nerves are in direct contact. Where a large defect is present, the process of nerve regeneration is hampered. It may not occur at all or result in the formation of a neuroma. Where nerve regrowth occurs it is typically at a rate of 1mm per day.

How well did you know this?
1
Not at all
2
3
4
5
Perfectly
212
Q

A 25 year old man is undergoing respiratory spirometry. He takes a maximal inspiration and maximally exhales. Which of the following measurements will best illustrate this process?

Functional residual capacity

Vital capacity

Inspiratory capacity

Maximum voluntary ventilation

Tidal volume

A

The maximum voluntary ventilation is the maximal ventilation over the course of 1 minute.

Definitions

Tidal volume (TV)
Is the volume of air inspired and expired during each ventilatory cycle at rest.
It is normally 500mls in males and 340mls in females.
Inspiratory reserve volume (IRV)
Is the maximum volume of air that can be forcibly inhaled following a normal inspiration. 3000mls.
Expiratory reserve volume (ERV)
Is the maximum volume of air that can be forcibly exhaled following a normal expiration. 1000mls.
Residual volume (RV)
Is that volume of air remaining in the lungs after a maximal expiration.
RV = FRC - ERV. 1500mls.
Functional residual capacity (FRC)
Is the volume of air remaining in the lungs at the end of a normal expiration.
FRC = RV + ERV. 2500mls.
Vital capacity (VC)
Is the maximal volume of air that can be forcibly exhaled after a maximal inspiration.
VC = TV + IRV + ERV. 4500mls in males, 3500mls in females.
Total lung capacity (TLC)
Is the volume of air in the lungs at the end of a maximal inspiration.
TLC = FRC + TV + IRV = VC + RV. 5500-6000mls.
Forced vital capacity (FVC)
The volume of air that can be maximally forcefully exhaled.

How well did you know this?
1
Not at all
2
3
4
5
Perfectly
213
Q

Which of the following laryngeal tumours will not typically metastasise to the cervical lymph nodes?

Glottic

Supraglottic

Subglottic

Transglottic

Aryepiglottic fold

A

A-The vocal cords have no lymphatic drainage and therefore this region serves as a lymphatic watershed. The supraglottic part drains to the upper deep cervical nodes through vessels piercing the thyrohyoid membrane. The sub glottic part drains to the pre laryngeal, pre tracheal and inferior deep cervical nodes. The aryepiglottic and vestibular folds have a rich lymphatic drainage and will metastasise early.

Larynx

The larynx lies in the anterior part of the neck at the levels of C3 to C6 vertebral bodies. The laryngeal skeleton consists of a number of cartilagenous segments. Three of these are paired; arytenoid, corniculate and cuneiform. Three are single; thyroid, cricoid and epiglottic. The cricoid cartilage forms a complete ring (the only one to do so).
The laryngeal cavity extends from the laryngeal inlet to the level of the inferior border of the cricoid cartilage.

Divisions of the laryngeal cavity
Laryngeal vestibule Superior to the vestibular folds
Laryngeal ventricle Lies between vestibular folds and superior to the vocal cords
Infraglottic cavity Extends from vocal cords to inferior border of the cricoid cartilage

The vocal folds (true vocal cords) control sound production. The apex of each fold projects medially into the laryngeal cavity. Each vocal fold includes:
Vocal ligament
Vocalis muscle (most medial part of thyroarytenoid muscle)
The glottis is composed of the vocal folds, processes and rima glottidis. The rima glottidis is the narrowest potential site within the larynx, as the vocal cords may be completely opposed, forming a complete barrier.

Muscles of the larynx
Muscle Origin Insertion Innervation Action
Posterior cricoarytenoid Posterior aspect of lamina of cricoid Muscular process of arytenoid Recurrent Laryngeal Abducts vocal fold
Lateral cricoarytenoid Arch of cricoid Muscular process of arytenoid Recurrent laryngeal Adducts vocal fold
Thyroarytenoid Posterior aspect of thyroid cartilage Muscular process of arytenoid Recurrent laryngeal Relaxes vocal fold
Transverse and oblique arytenoids Arytenoid cartilage Contralateral arytenoid Recurrent laryngeal Closure of intercartilagenous part of the rima glottidis
Vocalis Depression between lamina of thyroid cartilage Vocal ligament and vocal process of arytenoid cartilage Recurrent laryngeal Relaxes posterior vocal ligament, tenses anterior part
Cricothyroid Anterolateral part of cricoid Inferior margin and horn of thyroid cartilage External laryngeal Tenses vocal fold

Blood supply
Arterial supply is via the laryngeal arteries, branches of the superior and inferior thyroid arteries. The superior laryngeal artery is closely related to the internal laryngeal nerve. The inferior laryngeal artery is related to the inferior laryngeal nerve. Venous drainage is via superior and inferior laryngeal veins, the former draining into the superior thyroid vein and the latter draining into the middle thyroid vein, or thyroid venous plexus.

Lymphatic drainage
The vocal cords have no lymphatic drainage and this site acts as a lymphatic watershed.
Supraglottic part Upper deep cervical nodes
Subglottic part Prelaryngeal and pretracheal nodes and inferior deep cervical nodes
The aryepiglottic fold and vestibular folds have a dense plexus of lymphatics associated with them and malignancies at these sites have a greater propensity for nodal metastasis.

How well did you know this?
1
Not at all
2
3
4
5
Perfectly
214
Q

How many valves lie between the superior vena cava and the right atrium?

None

One

Two

Three

Four

A

There are no valves which is why it is relatively easy to insert a CVP line from the internal jugular vein into the right atrium.

Superior vena cava

Drainage
Head and neck
Upper limbs
Thorax
Part of abdominal walls

Formation
Subclavian and internal jugular veins unite to form the right and left brachiocephalic veins
These unite to form the SVC
Azygos vein joins the SVC before it enters the right atrium

Relations
Anterior Anterior margins of the right lung and pleura
Posteromedial Trachea and right vagus nerve
Posterolateral Posterior aspects of right lung and pleura
Pulmonary hilum is posterior
Right lateral Right phrenic nerve and pleura
Left lateral Brachiocephalic artery and ascending aorta

Developmental variations
Anomalies of the connection of the SVC are recognised. In some individuals a persistent left sided SVC drains into the right atrium via an enlarged orifice of the coronary sinus. More rarely the left sided vena cava may connect directly with the superior aspect of the left atrium, usually associated with an un-roofing of the coronary sinus. The commonest lesion of the IVC is for its abdominal course to be interrupted, with drainage achieved via the azygos venous system. This may occur in patients with left sided atrial isomerism.

How well did you know this?
1
Not at all
2
3
4
5
Perfectly
215
Q

A cohort study is being designed to look at the relationship between smoking and breast cancer. What is the usual outcome measure in a cohort study?

Odds ratio

Experimental event rate

Relative risk

Absolute risk increase

Numbers needed to harm

A

Cohort studies - relative risk

Study design

The following table highlights the main features of the main types of study:

Randomised controlled trial Participants randomly allocated to intervention or control group (e.g. standard treatment or placebo)

Practical or ethical problems may limit use
Cohort study Observational and prospective. Two (or more) are selected according to their exposure to a particular agent (e.g. medicine, toxin) and followed up to see how many develop a disease or other outcome.

The usual outcome measure is the relative risk.

Examples include Framingham Heart Study
Case-control study Observational and retrospective. Patients with a particular condition (cases) are identified and matched with controls. Data is then collected on past exposure to a possible causal agent for the condition.

The usual outcome measure is the odds ratio.

Inexpensive, produce quick results
Useful for studying rare conditions
Prone to confounding
Cross-sectional survey Provide a ‘snapshot’, sometimes called prevalence studies

Provide weak evidence of cause and effect

How well did you know this?
1
Not at all
2
3
4
5
Perfectly
216
Q

An 8 week old infant is brought to clinic with a history of 18 days of jaundice. The mother is breast feeding. He was a full term baby. There is no family history of liver disease. What is the most appropriate next step?

Liver USS

Unconjugated bilirubin measurement

Conjugated bilirubin measurement

Reassure and discharge

ERCP

A

This baby is a full term and has > 14 days of jaundice, therefore needs an urgent conjugated bilirubin check to rule out biliary atresia. If physiological jaundice the unconjugated bilirubin levels will be increased. Isotope scanning may be used in diagnosis, but a definitive diagnosis is normally made during a laparotomy.

Biliary atresia

1 in 17000 affected
Biliary tree lumen is obliterated by an inflammatory cholangiopathy causing progressive liver damage

Clinical features
Infant well in 1st few weeks of life
No family history of liver disease
Jaundice in infants > 14 days in term infants (>21 days in pre term infants)
Pale stool, yellow urine (colourless in babies)
Associated with cardiac malformations, polysplenia, situs inversus

Investigation
Conjugated bilirubin (prolonged physiological jaundice or breast milk jaundice will cause a rise in unconjugated bilirubin, whereas those with obstructive liver disease will have a rise in conjugated bilirubin)
Ultrasound of the liver (excludes extrahepatic causes, in biliary atresia infant may have tiny or invisible gallbladder)
Hepato-iminodiacetic acid radionuclide scan (good uptake but no excretion usually seen)

Management
Early recognition is important to prevent liver transplantation.
Nutritional support.
Roux-en-Y portojejunostomy (Kasai procedure).
If Kasai procedure fails or late recognition, a liver transplant becomes the only option.

How well did you know this?
1
Not at all
2
3
4
5
Perfectly
217
Q

A patient is due to undergo a right hemicolectomy for a carcinoma of the caecum. Which of the following vessels will require high ligation to provide optimal oncological control?

Middle colic artery

Inferior mesenteric artery

Superior mesenteric artery

Ileo-colic artery

None of the above

A

The ileo - colic artery supplies the caecum and would require high ligation during a right hemicolectomy. The middle colic artery should generally be preserved when resecting a caecal lesion.
This question is essentially asking you to name the vessel supplying the caecum. The SMA does not directly supply the caecum, it is the ileocolic artery which does this.

Caecum

Location
Proximal right colon below the ileocaecal valve
Intraperitoneal
Posterior relations
Psoas
Iliacus
Femoral nerve
Genitofemoral nerve
Gonadal vessels
Anterior relations Greater omentum
Arterial supply Ileocolic artery
Lymphatic drainage Mesenteric nodes accompany the venous drainage

The caecum is the most distensible part of the colon and in complete large bowel obstruction with a competent ileocaecal valve the most likely site of eventual perforation.

How well did you know this?
1
Not at all
2
3
4
5
Perfectly
218
Q

A 33 year old man sustains an injury to his forearm and wrist. When examined in clinic he is unable to adduct his thumb. What is the most likely underlying nerve lesion?

Radial nerve

Superficial branch of the ulnar nerve

Median nerve

Posterior interosseous nerve

Deep branch of the ulnar nerve

A

Damage to the deep branch of the ulnar nerve may result in an inability to adduct the thumb. This is tested clinically by trying to withdraw a piece of paper from a patients hand grasped between thumb and index finger.

Adductor pollicis

Origin Insertion Nerve supply Actions
Tendon sheath of flexor carpi radialis
Bases of second, third and fourth metacarpals
Anterior aspect of the trapezoid and capitate bones
Transverse head comes from the longitudinal ride of the third metacarpal

How well did you know this?
1
Not at all
2
3
4
5
Perfectly
219
Q

A 6 year old boy presents with groin pain. He is known to be disruptive in class. He reports that he is bullied for being short. On examination, he has an antalgic gait and pain on internal rotation of the right hip. What is the most likely diagnosis?

Perthes disease

Transient synovitis

Slipped upper femoral epiphysis

Developmental dysplasia of the hip

Septic arthritis

A

This child is short, has hyperactivity (disruptive behaviour) and is within the age range for Perthes disease. Hyperactivity and short stature are associated with Perthes disease.

Paediatric orthopaedics

Diagnosis Mode of presentation Treatment Radiology

Developmental dysplasia of the hip
Usually diagnosed in infancy by screening tests. May be bilateral, when disease is unilateral there may be leg length inequality. As disease progresses child may limp and then early onset arthritis. More common in extended breech babies. Splints and harnesses or traction. In later years osteotomy and hip realignment procedures may be needed. In arthritis a joint replacement may be needed. However, this is best deferred if possible as it will almost certainly require revision. Initially no obvious change on plain films and USS gives best resolution until 3 months of age. On plain films Shentons line should form a smooth arc

Perthes Disease Hip pain (may be referred to the knee) usually occurring between 5 and 12 years of age. Bilateral disease in 20%. Remove pressure from joint to allow normal development. Physiotherapy. Usually self-limiting if diagnosed and treated promptly. X-rays will show flattened femoral head. Eventually in untreated cases the femoral head will fragment.

Slipped upper femoral epiphysis Typically seen in obese male adolescents. Pain is often referred to the knee. Limitation to internal rotation is usually seen. Knee pain is usually present 2 months prior to hip slipping. Bilateral in 20%. Bed rest and non-weight bearing. Aim to avoid avascular necrosis. If severe slippage or risk of it occurring then percutaneous pinning of the hip may be required. X-rays will show the femoral head displaced and falling inferolaterally (like a melting ice cream cone) The Southwick angle gives indication of disease severity

How well did you know this?
1
Not at all
2
3
4
5
Perfectly
220
Q

Inspection of the left ventricle reveals all except which of the following?

Papillary muscles

Trabeculae carnae

Chordae tendinae

Conus arteriosus

Openings of the venae cordis minimae

A

The conus arteriosus (infundibulum) is the smooth walled outflow tract of the right ventricle leading to the pulmonary trunk.

How well did you know this?
1
Not at all
2
3
4
5
Perfectly
221
Q

A 23 year old lady with troublesome axillary hyperhidrosis is undergoing a thorascopic sympathectomy to treat the condition. Which of the following structures will need to be divided to access the sympathetic trunk?

Intercostal vein

Intercostal artery

Parietal pleura

Visceral pleura

None of the above

A

The sympathetic chain lies posterior to the parietal pleura. During a thorascopic sympathetomy this structure will need to be divided. The intercostal vessels lie posteriorly. They may be damaged with troublesome bleeding but otherwise are best left alone as deliberate division will not improve surgical access

How well did you know this?
1
Not at all
2
3
4
5
Perfectly
222
Q

A 44 year old man undergoes a distal gastrectomy for cancer. He is slightly anaemic and therefore receives a transfusion of 4 units of packed red cells to cover both the existing anaemia and associated perioperative blood loss. He is noted to develop ECG changes that are not consistent with ischaemia. What is the most likely cause?

Hyponatraemia

Hyperkalaemia

Hypercalcaemia

Metabolic alkalosis

Hypernatraemia

A

The transfusion of packed red cells has been shown to increase serum potassium levels. The risk is higher with large volume transfusions and with old blood.

How well did you know this?
1
Not at all
2
3
4
5
Perfectly
223
Q

In which space is a lumbar puncture performed?

Subdural space

Epidural space

Subarachnoid space

Extradural space

Intraventricular space

A

Samples of CSF are normally obtained by inserting a needle between the third and fourth lumbar vertebrae. The tip of the needle lies in the sub arachnoid space, the spinal cord terminates at L1 and is not at risk of injury. Clinical evidence of raised intracranial pressure is a contraindication to lumbar puncture.

How well did you know this?
1
Not at all
2
3
4
5
Perfectly
224
Q

A 56 year old lady with idiopathic thrombocytopenic purpura has a platelet count of 50. She is due to undergo a splenectomy. What is the optimal timing of a platelet transfusion in this case?

24 hours pre-operatively

2 hours pre-operatively

Whilst making the skin incision

After ligation of the splenic artery

On removal of the spleen

A

ITP causes splenic sequestration of platelets. Therefore a platelet transfusion should be carefully timed. Too soon and it will be ineffective. Too late and unnecessary bleeding will occur. The optimal time is after the splenic artery has been ligated.

How well did you know this?
1
Not at all
2
3
4
5
Perfectly
225
Q

Which is true about post-splencetomy changes:

Platelets will rise first (therefore in ITP should be given after splenic artery clamped)

Blood film will change over following weeks, Howell Jolly bodies will appear

Other blood film changes include target cells and Pappenheimer bodies

Increased risk of post splenectomy sepsis, therefore prophylactic antibiotics and pneumococcal vaccine should be given.

All of the above

A

All of the above

How well did you know this?
1
Not at all
2
3
4
5
Perfectly
226
Q

A 21 year old man is stabbed in the antecubital fossa. A decision is made to surgically explore the wound. At operation the surgeon dissects down onto the brachial artery. A nerve is identified medially, which nerve is it likely to be?

Radial

Recurrent branch of median

Anterior interosseous

Ulnar

Median

A

Median Median nerve

The median nerve is formed by the union of a lateral and medial root respectively from the lateral (C5,6,7) and medial (C8 and T1) cords of the brachial plexus; the medial root passes anterior to the third part of the axillary artery. The nerve descends lateral to the brachial artery, crosses to its medial side (usually passing anterior to the artery). It passes deep to the bicipital aponeurosis and the median cubital vein at the elbow.
It passes between the two heads of the pronator teres muscle, and runs on the deep surface of flexor digitorum superficialis (within its fascial sheath).
Near the wrist it becomes superficial between the tendons of flexor digitorum superficialis and flexor carpi radialis, deep to palmaris longus tendon. It passes deep to the flexor retinaculum to enter the palm, but lies anterior to the long flexor tendons within the carpal tunnel.

Branches
Region Branch
Upper arm No branches, although the nerve commonly communicates with the musculocutaneous nerve
Forearm Pronator teres
Pronator quadratus
Flexor carpi radialis
Palmaris longus
Flexor digitorum superficialis
Flexor pollicis longus
Flexor digitorum profundus (only the radial half)
Distal forearm Palmar cutaneous branch
Hand (Motor) Motor supply (LOAF)
Lateral 2 lumbricals
Opponens pollicis
Abductor pollicis brevis
Flexor pollicis brevis
Hand (Sensory)
Over thumb and lateral 2 ½ fingers
On the palmar aspect this projects proximally, on the dorsal aspect only the distal regions are innervated with the radial nerve providing the more proximal cutaneous innervation.

Patterns of damage
Damage at wrist
e.g. carpal tunnel syndrome
paralysis and wasting of thenar eminence muscles and opponens pollicis (ape hand deformity)
sensory loss to palmar aspect of lateral (radial) 2 ½ fingers

Damage at elbow, as above plus:
unable to pronate forearm
weak wrist flexion
ulnar deviation of wrist

Anterior interosseous nerve (branch of median nerve)
leaves just below the elbow
results in loss of pronation of forearm and weakness of long flexors of thumb and index finger

How well did you know this?
1
Not at all
2
3
4
5
Perfectly
227
Q

Survival in HCC:

% 5 at 5 years

% 10 at 5 years

% 15 at 5 years

% 30 at 5 years

A

% 15 at 5 years

How well did you know this?
1
Not at all
2
3
4
5
Perfectly
228
Q

Treatment of cholangiocarcinoma, which is true?

Surgical resection offers the best chance of cure.

Local invasion of peri hilar tumours is a particular problem

Lobar atrophy will often contra indicate surgical resection.

Palliation of jaundice is important,

Metallic stents should be avoided in those considered for resection.

None of the above

All of the above

A

All of the above

How well did you know this?
1
Not at all
2
3
4
5
Perfectly
229
Q

Survival in cholangiocarcinoma:

% 1-5 at 5 years

% 5-10 at 5 years

% 20 at 5 years

% 25 at 5 years

A

% 5-10 at 5 years

How well did you know this?
1
Not at all
2
3
4
5
Perfectly
230
Q

A 56 year old man has long standing chronic pancreatitis and develops pancreatic insufficiency. Which of the following will be absorbed normally?

Fat

Protein

Folic acid

Vitamin B12

None of the above

A

Pancreatic lipase is required for digestion of fat, Proteases facilitate protein and B12 absorption. Folate digestion is independent of the pancreas.

Pancreas exocrine physiology

Composition of pancreatic secretions
Pancreatic secretions are usually 1000-1500ml per 24 hours and have a pH of 8.
Secretion Source Substances secreted
Enzymic Acinar cells Trypsinogen
Procarboxylase
Amylase
Elastase
Aqueous Ductal and Centroacinar cells Sodium
Bicarbonate
Water
Potassium
Chloride
NB: Sodium and potassium reflect their plasma levels; chloride and bicarbonate vary with flow rate

Regulation
The cephalic and gastric phases (neuronal and physical) are less important in regulating the pancreatic secretions. The effect of digested material in the small bowel stimulates CCK release and ACh which stimulate acinar and ductal cells. Of these CCK is the most potent stimulus. In the case of the ductal cells these are potently stimulated by secretin which is released by the S cells of the duodenum. This results in an increase in bicarbonate.

Enzyme activation
Trypsinogen is converted via enterokinase to active trypsin in the duodenum. Trypsin then activates the other inactive enzymes

How well did you know this?
1
Not at all
2
3
4
5
Perfectly
231
Q

A 65 year old man with long standing atrial fibrillation develops an embolus to the lower leg. The decision is made to perform an embolectomy, utilising a trans popliteal approach. After incising the deep fascia, which of the following structures will the surgeons encounter first on exploring the central region of the popliteal fossa?

Popliteal vein

Common peroneal nerve

Popliteal artery

Tibial nerve

None of the above

A

The tibial nerve lies superior to the vessels in the inferior aspect of the popliteal fossa. In the upper part of the fossa the tibial nerve lies lateral to the vessels, it then passes superficial to them to lie medially. The popliteal artery is the deepest structure in the popliteal fossa.

How well did you know this?
1
Not at all
2
3
4
5
Perfectly
232
Q

A 73 year old lady is admitted with a brisk rectal bleed. She is otherwise well and the bleed settles. On examination her abdomen is soft and non tender. Elective colonoscopy shows a small erythematous lesion in the right colon, but no other abnormality. Diagnosis?

A.Haemorrhoids

B.Meckels diverticulum

C.Angiodysplasia

D.Colonic cancer

E.Diverticular bleed

F.Ulcerative colitis

G.Ischaemic colitis

A

The correct answer is Angiodysplasia

Angiodysplasia can be difficult to identify and treat. The colonoscopic stigmata are easily missed by poor bowel preparation.

Lower Gastrointestinal bleeding

Colonic bleeding
This typically presents as bright red or dark red blood per rectum. Colonic bleeding rarely presents as malaena type stool, this is because blood in the colon has a powerful laxative effect and is rarely retained long enough for transformation to occur and because the digestive enzymes present in the small bowel are not present in the colon. Up to 15% of patients presenting with haemochezia will have an upper gastrointestinal source of haemorrhage.

As a general rule right sided bleeds tend to present with darker coloured blood than left sided bleeds. Haemorrhoidal bleeding typically presents as bright red rectal bleeding that occurs post defecation either onto toilet paper or into the toilet pan. It is very unusual for haemorrhoids alone to cause any degree of haemodynamic compromise.

Causes
Cause Presenting features
Colitis Bleeding may be brisk in advanced cases, diarrhoea is commonly present. Abdominal x-ray may show featureless colon.
Diverticular disease Acute diverticulitis often is not complicated by major bleeding and diverticular bleeds often occur sporadically. 75% all will cease spontaneously within 24-48 hours. Bleeding is often dark and of large volume.
Cancer Colonic cancers often bleed and for many patients this may be the first sign of the disease. Major bleeding from early lesions is uncommon
Haemorrhoidal bleeding Typically bright red bleeding occurring post defecation. Although patients may give graphic descriptions bleeding of sufficient volume to cause haemodynamic compromise is rare.
Angiodysplasia Apart from bleeding, which may be massive, these arteriovenous lesions cause little in the way of symptoms. The right side of the colon is more commonly affected.

Management
Prompt correction of any haemodynamic compromise is required. Unlike upper gastrointestinal bleeding the first line management is usually supportive. This is because in the acute setting endoscopy is rarely helpful.
When haemorrhoidal bleeding is suspected a proctosigmoidoscopy is reasonable as attempts at full colonoscopy are usually time consuming and often futile.
In the unstable patient the usual procedure would be an angiogram (either CT or percutaneous), when these are performed during a period of haemodynamic instability they may show a bleeding point and may be the only way of identifying a patch of angiodysplasia.
In others who are more stable the standard procedure would be a colonoscopy in the elective setting. In patients undergoing angiography attempts can be made to address the lesion in question such as coiling. Otherwise surgery will be necessary.
In patients with ulcerative colitis who have significant haemorrhage the standard approach would be a sub total colectomy, particularly if medical management has already been tried and is not effective.

Indications for surgery
Patients > 60 years
Continued bleeding despite endoscopic intervention
Recurrent bleeding
Known cardiovascular disease with poor response to hypotension

Surgery
Selective mesenteric embolisation if life threatening bleeding. This is most helpful if conducted during a period of relative haemodynamic instability. If all haemodynamic parameters are normal then the bleeding is most likely to have stopped and any angiography normal in appearance. In many units a CT angiogram will replace selective angiography but the same caveats will apply.

If the source of colonic bleeding is unclear; perform a laparotomy, on table colonic lavage and following this attempt a resection. A blind sub total colectomy is most unwise, for example bleeding from an small bowel arterio-venous malformation will not be treated by this manoeuvre.

Summary of Acute Lower GI bleeding recommendations
Consider admission if:
* Over 60 years
* Haemodynamically unstable/profuse PR bleeding
* On aspirin or NSAID
* Significant co morbidity

Management
All patients should have a history and examination, PR and proctoscopy
Colonoscopic haemostasis aimed for in post polypectomy or diverticular bleeding

References
http://www.sign.ac.uk/guidelines/fulltext/105/index.html

How well did you know this?
1
Not at all
2
3
4
5
Perfectly
233
Q

A 39 year old man has suffered from terminal ileal Crohns disease for the past 20 years. Which condition is he least likely to develop?

Gallstones

Malabsorption

Pyoderma gangrenosum

Amyloidosis

Feltys syndrome

A

Step 1: Understanding Crohn’s Disease and Its Complications

Crohn’s disease is a type of inflammatory bowel disease (IBD) that can affect any part of the gastrointestinal tract but most commonly affects the terminal ileum. It is characterized by chronic inflammation, which can lead to various complications over time. Patients with Crohn’s disease are at risk for several conditions due to the nature of their illness.

Step 2: Evaluating Each Condition

Gallstones: Patients with Crohn’s disease, particularly those with ileal involvement, are at an increased risk for gallstones. This is primarily due to malabsorption of bile salts, which can lead to changes in bile composition and promote gallstone formation.

Malabsorption: Malabsorption is a common complication in patients with Crohn’s disease, especially when the terminal ileum is affected. The terminal ileum is responsible for absorbing vitamin B12 and bile acids; thus, its dysfunction can lead to significant nutritional deficiencies.

Pyoderma Gangrenosum: This is a rare skin condition associated with IBD, including Crohn’s disease. While it can occur in patients with Crohn’s, it is not as common as other complications.

Amyloidosis: This condition involves the deposition of amyloid proteins in various tissues and organs and can occur in patients with chronic inflammatory diseases like Crohn’s disease. However, it is relatively rare compared to other complications.

Felty’s Syndrome: This syndrome consists of rheumatoid arthritis, splenomegaly, and neutropenia (low white blood cell count). It is primarily associated with rheumatoid arthritis rather than Crohn’s disease or other forms of IBD.

Step 3: Determining the Least Likely Condition

Given the above evaluations:

Gallstones are likely due to malabsorption.
Malabsorption itself is very likely.
Pyoderma gangrenosum may occur but isn’t as prevalent.
Amyloidosis could develop but is less common.
Felty’s syndrome has no direct association with Crohn’s disease and occurs primarily in rheumatoid arthritis patients.
Thus, among these options, Felty’s syndrome stands out as the least likely condition for a patient suffering from terminal ileal Crohn’s disease.

Answer: Felty’s syndrome

How well did you know this?
1
Not at all
2
3
4
5
Perfectly
234
Q

A 56 year old male presents to the acute surgical take with severe abdominal pain. He is normally fit and well. He has no malignancy. The biochemistry laboratory contacts the ward urgently, his corrected calcium result is 3.6 mmol/l. What is the medication of choice to treat this abnormality?

IV Pamidronate

Oral Alendronate

Dexamethasone

Vitamin D

Resonium salts

A

IV Pamidronate is the drug of choice as it most effective and has long lasting effects. Calcitonin would need to be given with another agent, to ensure that the hypercalcaemia is treated once its short term effects wear off. IV zoledronate is preferred in scenarios associated with malignancy.

How well did you know this?
1
Not at all
2
3
4
5
Perfectly
235
Q

A 53 year old man is undergoing a distal pancreatectomy for trauma. Which of the following vessels is responsible for the arterial supply to the tail of the pancreas?

Splenic artery

Pancreaticoduodenal artery

Gastric artery

Hepatic artery

Superior mesenteric artery

A

Pancreatic head is supplied by the pancreaticoduodenal artery
Pancreatic tail is supplied by branches of the splenic artery

There is an arterial watershed in the supply between the head and tail of the pancreas. The head is supplied by the pancreaticoduodenal artery and the tail is supplied by branches of the splenic artery.

How well did you know this?
1
Not at all
2
3
4
5
Perfectly
236
Q

A 43 year old lady presents with varicose veins and undergoes a saphenofemoral disconnection, long saphenous vein stripping to the ankle and isolated hook phlebectomies. Post operatively she notices an area of numbness superior to her ankle. What is the most likely cause for this?

Sural nerve injury

Femoral nerve injury

Saphenous nerve injury

Common peroneal nerve injury

Superficial peroneal nerve injury

A

The most likely cause for the postoperative numbness superior to the ankle in this case is a Saphenous nerve injury. The Saphenous nerve runs alongside the long saphenous vein, which is typically addressed during procedures like saphenofemoral disconnection and vein stripping. Nerve injuries during these surgeries can lead to sensory disturbances such as numbness. It’s advisable for the patient to discuss this with their healthcare provider for a more accurate assessment.

The sural nerve is related to the short saphenous vein. The saphenous nerve is related to the long saphenous vein below the knee and for this reason full length stripping of the vein is no longer advocated.

How well did you know this?
1
Not at all
2
3
4
5
Perfectly
237
Q

Course of the long saphenous, which one is true?

1st digit where the dorsal vein merges with the dorsal venous arch of the foot.

Passes anterior to the medial malleolus and runs up the medial side of the leg.

At the knee it runs over the posterior border of the medial epicondyle of the femur.

Passes laterally to lie on the anterior surface of the thigh before entering the saphenous opening in the fascia lata.

Joins the femoral vein in the femoral triangle at the SFJ

A

All

How well did you know this?
1
Not at all
2
3
4
5
Perfectly
238
Q

Tributaries of the saphenous vein, which one is true?

Medial marginal

Superficial epigastric

Superficial iliac circumflex

Superficial external pudendal veins

A

Tributaries of the Saphenous Vein

The saphenous vein is a major superficial vein in the leg, primarily responsible for draining blood from the lower extremities. It has several tributaries that contribute to its function. To determine which of the listed veins are true tributaries of the saphenous vein, we will analyze each option step by step.

Medial Marginal Vein: The medial marginal vein runs along the medial side of the foot and is known to drain into the great saphenous vein. Therefore, this option is indeed a true tributary.

Superficial Epigastric Vein: This vein arises from the femoral vein and travels upward towards the abdomen, draining into the external iliac vein rather than directly into the saphenous system. Thus, it is not a tributary of the saphenous vein.

Superficial Iliac Circumflex Vein: Similar to the superficial epigastric vein, this vessel also drains into the external iliac vein and does not connect with or drain into the saphenous system. Therefore, it is not a tributary of the saphenous vein.

Superficial External Pudendal Veins: These veins also originate from areas around the groin and drain into either the femoral or external iliac veins rather than directly contributing to the saphenous system. Hence, they are not considered tributaries of the saphenous vein.

After analyzing all four options:

The only true tributary of the saphenous vein among those listed is Medial Marginal Vein.
Thus, based on this detailed examination:

Answer: Medial marginal

How well did you know this?
1
Not at all
2
3
4
5
Perfectly
239
Q

A 58 year old man is reviewed in the clinic following a successful cadaveric renal transplant the previous year. He has been able to return to work as a swimming instructor. Over the past week he reports that he has been suffering from recurrent episodes of diarrhoea. It has made him feel lethargic and exhausted. Stool microscopy shows evidence of cysts.

A.Giardia Infection

B.Cryptosporidium infection

C.Clonorchis sinensis infection

D.Ancylostoma duodenale infection

E.Ascaris lumbricoides infection

A

Cryptosporidium infection

Cryptosporidium is associated with infection, particularly in those who are immunocompromised. Diarrhoea is the main disease. The cysts are typically identified on stool microscopy.

How well did you know this?
1
Not at all
2
3
4
5
Perfectly
240
Q

A 23 year old women has undergone a pan proctocolectomy and ileoanal pouch because she suffers from familial adenomatous polyposis coli. What is the commonest extra colonic lesion in this disorder?

Gastric fundal polyps

Trichilemmomas

Duodenal polyps

Fibrocystic disease of the breast

Skull osteomas

A

Duodenal polyps occur in up to 100% of patients with FAP if follow up is continued for long enough. Duodenal cancer has an incidence of 4-10%.

Duodenal polyps are the commonest extra colonic lesion in FAP. Gastric fundal polyps are seen in 50% of patients. Skull osteomas are seen in Gardeners syndrome which is a variant of FAP.

How well did you know this?
1
Not at all
2
3
4
5
Perfectly
241
Q

The oxygen-haemoglobin dissociation curve is shifted to the right in which of the following scenarios?

Hypothermia

Respiratory alkalosis

Low altitude

Decreased 2,3-DPG in transfused red cells

Chronic iron deficiency anaemia

A

Hypothermia:

Hypothermia typically causes a leftward shift in the oxygen-haemoglobin dissociation curve. This is because lower temperatures increase hemoglobin’s affinity for oxygen, making it hold onto oxygen more tightly rather than releasing it.
Respiratory Alkalosis:

Respiratory alkalosis occurs when there is a decrease in carbon dioxide (CO2) levels due to hyperventilation, leading to an increase in blood pH. This condition also results in a leftward shift of the curve, as higher pH increases hemoglobin’s affinity for oxygen.
Low Altitude:

At low altitudes, the partial pressure of oxygen is higher compared to high altitudes; however, this does not directly cause a rightward shift in the dissociation curve. The body generally maintains normal hemoglobin-oxygen affinity at low altitudes.
Decreased 2,3-DPG in Transfused Red Cells:

2,3-Diphosphoglycerate (2,3-DPG) is a metabolite produced by red blood cells that decreases hemoglobin’s affinity for oxygen when present in higher concentrations. In transfused red cells where 2,3-DPG levels are decreased (as stored blood has lower levels), there would be a leftward shift rather than a rightward one.
Chronic Iron Deficiency Anaemia:

Chronic iron deficiency anemia can lead to changes in hemoglobin structure and function that may result in a rightward shift of the curve due to increased production of 2,3-DPG as the body attempts to enhance tissue oxygen delivery despite reduced overall hemoglobin levels.
Conclusion

Based on the analysis above:

The only scenario among those listed that leads to a rightward shift of the oxygen-haemoglobin dissociation curve is Chronic Iron Deficiency Anaemia.
Thus, the answer is:

Chronic iron deficiency anaemia

Mnemonic to remember causes of right shift of the oxygen dissociation curve:

CADET face RIGHT

C O2
A cidosis
2,3-DPG
E xercise
T emperature

The curve is shifted to the right when there is an increased oxygen requirement by the tissue. This includes:

Increased temperature

Acidosis

Increased DPG:

DPG is found in erythrocytes and is increased during glycolysis. It binds to the Hb molecule, thereby releasing oxygen to tissues. DPG is increased in conditions associated with poor oxygen delivery to tissues, such as anaemia and high altitude.

How well did you know this?
1
Not at all
2
3
4
5
Perfectly
242
Q

A pathologist is examining a histological section and identifies Hassall’s corpuscles. With what are they most commonly associated?

Follicular carcinoma of the thyroid

Medulla of the thymus

Medulla of the spleen

Medulla of the kidney

Fundus of the stomach

A

Hassall’s corpuscles are the concentric ring of epithelial cells seen in the medulla of the thymus.

How well did you know this?
1
Not at all
2
3
4
5
Perfectly
243
Q

A 25 year old man is stabbed in the upper arm. The brachial artery is lacerated at the level of the proximal humerus, and is being repaired. A nerve lying immediately lateral to the brachial artery is also lacerated. Which of the following is the nerve most likely to be?

Ulnar nerve

Median nerve

Radial nerve

Intercostobrachial nerve

Axillary nerve

A

The brachial artery begins at the lower border of teres major and terminates in the cubital fossa by branching into the radial and ulnar arteries. In the upper arm the median nerve lies closest to it in the lateral position. In the cubital fossa it lies medial to it.

How well did you know this?
1
Not at all
2
3
4
5
Perfectly
244
Q

A 63 year old man undergoes an upper GI endoscopy and adrenaline injection for a large actively bleeding duodenal ulcer. He remains stable for 6 hours and the nurses then call because he has passed 400ml malaena and has become tachycardic (pulse rate 120) and hypotensive (Bp 80/40). What is the best option?

Reassure that blood trapped in the upper portion of the gastrointestinal system will pass and that this episode will resolve with phosphate enema

Perform a repeat upper GI endoscopy

Perform a laparotomy and under-running of the ulcer

Administer tranexamic acid and intravenous proton pump inhibitors

Insert a Minnesota tube

A

The decision as to how best to manage patients with re-bleeding is difficult. Whilst it is tempting to offer repeat endoscopy, this intervention is best used on those with small ulcers. Large posteriorly sited duodenal ulcers are at high risk for re-bleeding and the timeframe of this event suggests that primary endoscopic haemostasis was inadequate. Surgery thus represents the safest way forward.

How well did you know this?
1
Not at all
2
3
4
5
Perfectly
245
Q

A 64 year old man presents to the clinic with right upper quadrant discomfort. He has never attended the hospital previously and is usually well. He has just retired from full time employment as a machinist in a PVC factory. CT scanning shows a large irregular tumour in the right lobe of his liver. Which of the following lesions is the most likely?

Liposarcoma

Angiosarcoma

Hamartoma

Hyatid liver disease

Benign angioma

A

Angiosarcoma of the liver is a rare tumour. However, it is linked to working with vinyl chloride, as in this case. Although modern factories minimise the exposure to this agent, this has not always been the case.

How well did you know this?
1
Not at all
2
3
4
5
Perfectly
246
Q

A 45 year old man undergoes a sub total colectomy and formation of end ileostomy. What is the most likely sodium content per litre of ileostomy fluid?

120 mmol

60 mmol

20 mmol

210 mmol

180 mmol

A

Investigators in the 1960’s dehydrated and measured the sodium content of ileostomy effluent and determined this concentration. Not an experiment many would care to repeat! (120)

How well did you know this?
1
Not at all
2
3
4
5
Perfectly
247
Q

Which of the following muscle relaxants will tend to incite neuromuscular excitability following administration?

Atracurium

Suxamethonium

Vecuronium

Pancuronium

None of the above

A

Suxamethonium may induce generalised muscular contractions following administration. This may raise serum potassium levels.

How well did you know this?
1
Not at all
2
3
4
5
Perfectly
248
Q

Which of the following blood products can be administered to a non ABO matched recipient?

Whole blood

Platelets

Packed red cells

Stem cells

Cryoprecipitate

A

In the UK, platelets either come from pooling of the platelet component from four units of whole donated blood, called random donor platelets, or by plasmapharesis from a single donor. The platelets are suspended in 200-300 ml of plasma and may be stored for up to 4 days in the transfusion laboratory where they are continually agitated at 22oC to preserve function. One adult platelet pool raises the normal platelet count by 30,000 to 60,000 platelets litre. ABO identical or compatible platelets are preferred but not necessary in adults; but rhesus compatibility is required in recipients who are children and women of childbearing age to prevent haemolytic disease of the newborn.

How well did you know this?
1
Not at all
2
3
4
5
Perfectly
249
Q

Removal of all plasma from a blood unit and substitution with:

Sodium chloride

Adenine

Anhydrous glucose

Mannitol

A

Up to 4 units of SAG M Blood may be administered. Thereafter whole blood is preferred. After 8 units, clotting factors and platelets should be considered.

How well did you know this?
1
Not at all
2
3
4
5
Perfectly
250
Q

Which of the following is not a content of the cavernous sinus?

Oculomotor nerve

Internal carotid artery

Opthalmic nerve

Abducens nerve

Optic nerve

A

Mnemonic for contents of cavernous sinus:
O TOM CAT

Occulomotor nerve (III)
Trochlear nerve (IV)
Ophthalmic nerve (V1)
Maxillary nerve (V2)
Carotid artery
Abducent nerve (VI)
T

OTOM=lateral wall components
CA= components within sinus

The optic nerve lies above and outside the cavernous sinus.

How well did you know this?
1
Not at all
2
3
4
5
Perfectly
251
Q

A 44 year old man with end stage renal failure undergoes a live donor renal transplant. During the immediate post operative period a good urine output is recorded. However, on return to the ward the nursing staff notice that the urinary catheter is no longer draining. However, the urostomy is continuing to drain urine.

A.Acute tubular necrosis

B.Renal artery thrombosis

C.Bladder occlusion

D.Ureteric occlusion

E.Acute rejection

F.Acute on chronic rejection

G.Hyperacute rejection

A

Bladder occlusion

The most likely explanation for this event is a blocked catheter. This may be the result of blood clot from the ureteric anastomosis. Bladder irrigation will usually resolve the problem.

How well did you know this?
1
Not at all
2
3
4
5
Perfectly
252
Q

A 43 year old man undergoes a live donor renal transplant. The donor’s right kidney is anastomosed to the recipient. On removal of the arterial clamps there is good urinary flow noted and the wounds are closed. On return to the ward the nurses notice that the patient suddenly becomes anuric and irrigation of the bladder does not improve the situation.

A.Acute tubular necrosis

B.Renal artery thrombosis

C.Bladder occlusion

D.Ureteric occlusion

E.Acute rejection

F.Acute on chronic rejection

G.Hyperacute rejection

A

The correct answer is Renal artery thrombosis

Right sided live donor transplants are extremely rare. This is because the vena cava precludes mobilisation of the right renal artery. The short right renal artery that is produced therefore presents a major challenge. The sudden cessation of urine output in this context is highly suggestive of an acute thrombosis. Delay in thrombectomy beyond 1 hour almost inevitably results in graft loss.

How well did you know this?
1
Not at all
2
3
4
5
Perfectly
253
Q

A 73 year old lady is admitted for a laparoscopic cholecystectomy. During her pre-operative assessment it is noted that she is receiving furosemide for the treatment of hypertension. Where is the site of action of this diuretic?

Proximal convoluted tubule

Descending limb of the loop of Henle

Ascending limb of the loop of Henle

Distal convoluted tubule

Collecting ducts

A

Action of furosemide = ascending limb of the loop of Henle

Furosemide and bumetanide are loop diuretics that act by inhibiting the Na-K-Cl cotransporter in the thick ascending limb of the loop of Henle, reducing the absorption of NaCl.

Diuretic agents

The diuretic drugs are divided into three major classes, which are distinguished according to the site at which they impair sodium reabsorption: loop diuretics in the thick ascending loop of Henle, thiazide type diuretics in the distal tubule and connecting segment; and potassium sparing diuretics in the aldosterone - sensitive principal cells in the cortical collecting tubule.
In the kidney, sodium is reabsorbed through Na+/ K+ ATPase pumps located on the basolateral membrane. These pumps return reabsorbed sodium to the circulation and maintain low intracellular sodium levels. This latter effect ensures a constant concentration gradient.

Physiological effects of commonly used diuretics
Site of action Diuretic Carrier or channel inhibited Percentage of filtered sodium excreted
Ascending limb of loop of Henle Frusemide Na+/K+ 2Cl - carrier Up to 25%
Distal tubule and connecting segment Thiazides Na+Cl- carrier Between 3 and 5%
Cortical collecting tubule Spironolactone Na+/K+ ATP ase pump Between 1 and 2%

How well did you know this?
1
Not at all
2
3
4
5
Perfectly
254
Q

A 56 year old man who drinks heavily is found collapsed by friends at his house. He was out drinking the previous night and following this was noted to have vomited repeatedly so his friends brought him home.

A.Schatzki ring

B.Plummer Vinson syndrome

C.Squamous cell carcinoma

D.Barretts oesophagus

E.Pharyngeal pouch

F.Adenocarcinoma

G.Leiomyoma

H.Oesophageal rupture

I.Diffuse oesophageal spasm

J.Hiatus hernia

A

Oesophageal rupture

Spontaneous rupture of the oesophagus may occur following an episode of vomiting. The subsequent mediastinitis can produce severe sepsis and death if not treated promptly. Adequate drainage of sepsis and early surgery are the cornerstones of management.

How well did you know this?
1
Not at all
2
3
4
5
Perfectly
255
Q

A 43 year old man has been troubled with dysphagia for many years. He is known to have achalasia and has had numerous dilatations. Over the past 6 weeks his dysphagia has worsened. At endoscopy a friable mass is noted in the oesophagus.

A.Schatzki ring

B.Plummer Vinson syndrome

C.Squamous cell carcinoma

D.Barretts oesophagus

A

Squamous cell carcinoma

The risk of squamous cell carcinoma of the oesophagus is increased in people with achalasia. The condition often presents late and has a poor prognosis.

How well did you know this?
1
Not at all
2
3
4
5
Perfectly
256
Q

A 73 year old lady is troubled by episodic swallowing difficulty and halitosis. An upper GI endoscopy is attempted and abandoned due to difficulty in achieving intubation.

A.Schatzki ring

B.Plummer Vinson syndrome

C.Squamous cell carcinoma

D.Barretts oesophagus

E.Pharyngeal pouch

A

Pharyngeal pouches occur when a defect occurs in killians dehiscence. Difficulty in intubation is a well recognised consequence and care must be taken to take the correct track during OGD to avoid perforation. Most cases are now treated with endoscopic stapling.

How well did you know this?
1
Not at all
2
3
4
5
Perfectly
257
Q

Surgical occlusion of which of these structures, will result in the greatest reduction in hepatic blood flow?

Portal vein

Common hepatic artery

Right hepatic artery

Coeliac axis

Left hepatic artery

A

The portal vein transports 70% of the blood supply to the liver, while the hepatic artery provides 30%. The portal vein contains the products of digestion. The arterial and venous blood is dispersed by sinusoids to the central veins of the liver lobules; these drain into the hepatic veins and then into the IVC. The caudate lobe drains directly into the IVC rather than into other hepatic veins.

How well did you know this?
1
Not at all
2
3
4
5
Perfectly
258
Q

A 55 year old man presents with symptoms of dyspepsia and on upper GI endoscopy an area of patchy erythematous tissue is identified protruding proximally from the gastro oesophageal junction. A biopsy is diagnostic of Barretts oesophagus with low grade dysplasia. Which of the following is the most appropriate management?

Distal oesophagectomy

Upper GI endoscopy with quadrantic biopsies from the region

Photodynamic therapy

Endoscopic sub mucosal resection of the area

Argon plasma coagulation

A

In Barrett’s surveillance the safest option is quadrantic (i.e. 4 biopsies, one from each quarter of the oesophagus at 2cm intervals)

Low grade dysplasia in conjunction with Barretts oesphagus should be monitored with regular (6 monthly) upper GI endoscopy and quadrantic biopsies. If the disease remains static at 2 years then the screening frequency may be decreased.

How well did you know this?
1
Not at all
2
3
4
5
Perfectly
259
Q

Which of the following would be the optimal fluid management option for a 45 year old man due to undergo an elective right hemicolectomy?

Remain “nil by mouth” for at least 6 hours pre-operatively and avoid intra venous fluids

Remain “nil by mouth” for at least 6 hours pre-operatively and receive supplementary intravenous 5% dextrose to replace lost calories

Allow him free access to oral fluids only until 30 minutes prior to surgery

Administer a carbohydrate based loading drink 3 hours pre operatively, and avoid intravenous fluids

Administer a carbohydrate based loading drink 6 hours pre-operatively and administer 5% dextrose saline thereafter

A

Administer a carbohydrate based loading drink 3 hours pre operatively, and avoid intravenous fluids

Patients for elective surgery should not have solids for 6 hours pre-operatively. However, clear fluids may be given up to 2 hours pre-operatively. Enhanced recovery programmes are now the standard of care in many countries around the world and involve administration of carbohydrate loading drinks.
The routine administration of 5% dextrose in the scenarios given above would convey little in the way of benefit and increase the risks of electrolyte derangement post operatively

How well did you know this?
1
Not at all
2
3
4
5
Perfectly
260
Q

A 23 year old man is undergoing an inguinal hernia repair. The surgeons mobilise the spermatic cord and place it in a hernia ring. A small slender nerve is identified superior to the cord. Which nerve is it most likely to be?

Iliohypogastric nerve

Pudendal nerve

Femoral branch of the genitofemoral nerve

Ilioinguinal nerve

Obturator nerve

A

The ilioinguinal nerve passes through the inguinal canal and is the nerve most commonly identified during hernia surgery. The genitofemoral nerve splits into two branches, the genital branch passes through the inguinal canal within the cord structures. The femoral branch of the genitofemoral nerve enters the thigh posterior to the inguinal ligament, lateral to the femoral artery. The iliohypogastric nerve pierces the external oblique aponeurosis above the superficial inguinal ring.

How well did you know this?
1
Not at all
2
3
4
5
Perfectly
261
Q

Which of the following physiological changes do not occur following tracheostomy?

Alveolar ventilation is increased.

Anatomical dead space is reduced by 50%.

Work of breathing is increased.

Proportion of ciliated epithelial cells in the trachea may decrease.

Splinting of the larynx may lead to swallowing difficulties.

A

Work of breathing is decreased which is one reasons it is popular option for weaning ventilated patients. Humidified air in this setting helps to reduce the viscosity of mucous that forms.

How well did you know this?
1
Not at all
2
3
4
5
Perfectly
262
Q

Where does the spinal cord terminate in neonates?

L1

L2

L3

L4

L5

A

At the 3rd month the foetus’s spinal cord occupies the entire length of the vertebral canal. The vertebral column then grows longer exceeding the growth rate of the spinal cord. This results with the cord being at L3 at birth and L1-2 by adulthood.

How well did you know this?
1
Not at all
2
3
4
5
Perfectly
263
Q

A 45 year old man is undergoing a low anterior resection for a carcinoma of the rectum. Which of the following fascial structures will need to be divided to mobilise the mesorectum from the sacrum and coccyx?

Denonvilliers fascia

Colles fascia

Sibsons fascia

Waldeyers fascia

None of the above

A

Fascial layers surrounding the rectum:

Anteriorly lies the fascia of Denonvilliers

Posteriorly lies Waldeyers fascia

Waldeyers fascia separates the mesorectum from the sacrum and will need to be divided.

How well did you know this?
1
Not at all
2
3
4
5
Perfectly
264
Q

A 68 year old man with type 2 diabetes is admitted to hospital unwell. On examination he has features of septic shock and right upper quadrant tenderness. He is not jaundiced. Imaging shows a normal calibre bile duct and no stones in the gallbladder.

A.Uncomplicated biliary colic

B.Acute cholecystitis

C.Cholangitis

D.Gallbladder abscess

E.Acalculous cholecystitis

A

Acalculous cholecystitis

Acalculous cholecystitis is more common in patients with an underlying co-morbidity. The morbidity and mortality following intervention are higher than in conventional gallstone disease.

How well did you know this?
1
Not at all
2
3
4
5
Perfectly
265
Q

A 43 year old lady with known gallstones is admitted with a high fever and jaundice. On examination, she looks extremely unwell. Her abdomen is generally soft although there is some mild tenderness in the right upper quadrant.

A.Uncomplicated biliary colic

B.Acute cholecystitis

C.Cholangitis

D.Gallbladder abscess

E.Acalculous cholecystitis

A

Cholangitis

Features of jaundice, fever and systemic sepsis are typical of cholangitis.

How well did you know this?
1
Not at all
2
3
4
5
Perfectly
266
Q

A 34 year old lady is admitted with a 3 day history of colicky right upper quadrant pain which radiates to her back. The pain is now more constant. On examination she is not jaundiced, but has a temperature of 38.5oC. She has localised peritonism in the right upper quadrant.

A.Uncomplicated biliary colic

B.Acute cholecystitis

C.Cholangitis

D.Gallbladder abscess

E.Acalculous cholecystitis

F.Pancreatitis

A

Acute cholecystitis

The features of pain and fever with right upper quadrant pain are suggestive of acute cholecystitis. The short nature of the history makes an abscess less likely.

How well did you know this?
1
Not at all
2
3
4
5
Perfectly
267
Q

A 10 year old child has a grommet inserted for a glue ear. What type of epithelium is present on the external aspect of the tympanic membrane?

Stratified squamous

Ciliated columnar

Non ciliated columnar

Non stratified squamous

None of the above

A

The external aspect of the tympanic membrane is lined by stratified squamous epithelium. This is significant clinically in the development of middle ear infections when this type of epithelium may migrate inside the middle ear.

How well did you know this?
1
Not at all
2
3
4
5
Perfectly
268
Q

A 73 year old lady is admitted with acute mesenteric ischaemia. A CT angiogram is performed and a stenotic lesion is noted at the origin of the superior mesenteric artery. At which of the following levels does this branch from the aorta?

L1

L2

L3

L4

L5

A

The SMA leaves the aorta at L1. It passes under the neck of the pancreas prior to giving its first branch the inferior pancreatico-duodenal artery.

How well did you know this?
1
Not at all
2
3
4
5
Perfectly
269
Q

A 42 year old man from Southern India presents with chronic swelling of both lower legs, they are brawny and indurated with marked skin trophic changes. Which of the following organisms is the most likely origin of this disease process?

Loa loa

Wuchereria bancrofti

Trypanosoma cruzi

Trypanosoma gambiense

None of the above

A

W. Bancrofti is the commonest cause of filariasis leading to lymphatic obstruction. Infection with Loa loa typically occurs in the African sub continent and usually results in generalised sub cutaneous infections without lymphatic obstruction. Trypanosomal infections would not produce this clinical picture.

How well did you know this?
1
Not at all
2
3
4
5
Perfectly
270
Q

The following statements relating to the musculocutaneous nerve are true except?

It arises from the lateral cord of the brachial plexus

It provides cutaneous innervation to the lateral side of the forearm

If damaged, then extension of the elbow joint will be impaired

It supplies the biceps muscle

It runs beneath biceps

A

It supplies biceps, brachialis and coracobrachialis. If damaged then elbow flexion rather than extension will be impaired.

How well did you know this?
1
Not at all
2
3
4
5
Perfectly
271
Q

Which of the following structures does not pass through the foramen ovale?

Lesser petrosal nerve

Accessory meningeal artery

Maxillary nerve

Emissary veins

Otic ganglion

A

Mnemonic: OVALE

O tic ganglion
V3 (Mandibular nerve:3rd branch of trigeminal)
A ccessory meningeal artery
L esser petrosal nerve
E missary veins

How well did you know this?
1
Not at all
2
3
4
5
Perfectly
272
Q

Which of the following is not utilised as a descriptive statistic?

Mean

Median

Mode

Z score

Standard deviation

A

The z score is determined using the normal distribution and is not a descriptive statistic

How well did you know this?
1
Not at all
2
3
4
5
Perfectly
273
Q

Which of the cranial nerves listed below is least likely to carry parasympathetic fibres?

III

VII

IX

X

II

A

Cranial nerves carrying parasympathetic fibres
X IX VII III (1973)

The parasympathetic functions served by the cranial nerves include:

III (oculomotor)Pupillary constriction and accommodation

VII (facial)Lacrimal gland, submandibular and sublingual glands

IX (glossopharyngeal)Parotid

X (vagus)Heart and abdominal viscera

The optic nerve carries no parasympathetic fibres.

The cranial preganglionic parasympathetic nerves arise from specific nuclei in the CNS. These synapse at one of four parasympathetic ganglia; otic, pterygopalatine, ciliary and submandibular. From these ganglia the parasympathetic nerves complete their journey to their target tissues via CN V (trigeminal) branches (ophthalmic nerve CNV branch 1, Maxillary nerve CN V branch2, mandibular nerve CN V branch 3)

How well did you know this?
1
Not at all
2
3
4
5
Perfectly
274
Q

A 72 year old man is undergoing an open abdominal aortic aneurysm repair. The aneurysm is located in a juxtarenal location and surgical access to the neck of aneurysm is difficult. Which of the following structures may be divided to improve access?

Cisterna chyli

Transverse colon

Left renal vein

Superior mesenteric artery

Coeliac axis

A

The left renal vein will be stretched over the neck of the anuerysm in this location and is not infrequently divided. This adds to the nephrotoxic insult of juxtarenal aortic surgery as a supra renal clamp is also often applied. Deliberate division of the Cisterna Chyli will not improve access and will result in a chyle leak. Division of the transverse colon will not help at all and would result in a high risk of graft infection. Division of the SMA is pointless for a juxtarenal procedure.

How well did you know this?
1
Not at all
2
3
4
5
Perfectly
275
Q

Please select the most appropriate management for the injury type described. Each option may be used once, more than once or not at all.

A 20 year old woman trips over a step, injuring her ankle. Examination reveals tenderness over the lateral malleolus and an x-ray demonstrates an undisplaced fracture distal to the syndesmosis.

A.Surgical fixation

B.Below knee amputation

C.Application of below knee plaster

D.Application of ankle boot

E.Application of external fixation device

F.Application of compression dressing and physiotherapy

G.Immediate reduction and application of backslab

A

Application of ankle boot

This is a Weber A fracture. It is a stable ankle injury and can therefore be managed conservatively. Whilst this patient could also be treated in a below knee plaster, most clinicians would nowadays treat this injury in an ankle boot. Patients should be advised to mobilise in the ankle boot, as pain allows, and can wean themselves out of the boot as the symptoms improve.

How well did you know this?
1
Not at all
2
3
4
5
Perfectly
276
Q

A 30 year old man injures his ankle playing football. On examination he has tenderness over both medial and lateral malleoli. X-ray demonstrates a bimalleolar fracture with a displaced distal fibula fracture, at the level of the syndesmosis and fracture of the medial malleolus with talar shift. The ankle has been provisionally reduced and splinted in the emergency department.

A.Surgical fixation

B.Below knee amputation

C.Application of below knee plaster

D.Application of ankle boot

E.Application of external fixation device

F.Application of compression dressing and physiotherapy

G.Immediate reduction and application of backslab

A

The correct answer is Surgical fixation

This is an unstable fracture pattern with a Weber B fracture of the distal fibula and a fracture of the medial malleolus. Talar shift indicates loss of ankle mortice congruity. This injury should therefore be treated with surgical fixation.

How well did you know this?
1
Not at all
2
3
4
5
Perfectly
277
Q

A 50 year old female slips on wet floor injuring her ankle. On examination, she has tenderness over the lateral and medial malleolus. X-rays demonstrate an undisplaced fracture of the distal fiibula at the level of the syndesmosis and a congruent ankle mortice.

A.Surgical fixation

B.Below knee amputation

C.Application of below knee plaster

D.Application of ankle boot

E.Application of external fixation device

F.Application of compression dressing and physiotherapy

G.Immediate reduction and application of backslab

A

Application of below knee plaster

This is a Weber B fracture and therefore potentially unstable. Medial malleolar tenderness indicates deltoid ligament injury. As the fracture is currently undisplaced and the ankle mortice is congruent, the injury can be initially managed conservatively in a below knee plaster but the patient should be monitored in the outpatient clinic for fracture displacement in the first few weeks.

How well did you know this?
1
Not at all
2
3
4
5
Perfectly
278
Q

An occlusion of the anterior cerebral artery may compromise the blood supply to the following structures except:

Medial inferior surface of the frontal lobe

Corpus callosum

Medial surface of the frontal lobe

Olfactory bulb

Brocas area

A

Brocas area is usually supplied by branches from the middle cerebral artery.

How well did you know this?
1
Not at all
2
3
4
5
Perfectly
279
Q

A 32 year old Indian lady presents with a diffuse swelling of the left breast. She has a 4 month old child. Clinically, she has jaundice and there is erythema of the left breast.

A.Ductal carcinoma in situ

B.Lobular carcinoma in situ

C.Invasive ductal carcinoma

D.Invasive lobular carcinoma

E.Inflammatory carcinoma

A

Inflammatory carcinoma

Inflammatory breast cancers have an aggressive nature. Dissemination occurs early and is more resistant to adjuvent treatments than other types of breast cancer. Often occurs in pregnancy or lactation.

How well did you know this?
1
Not at all
2
3
4
5
Perfectly
280
Q

A 72 year old female presents with a painless breast lump. Clinically she has a 4cm diameter irregular breast mass, with no other palpable masses.

A.Ductal carcinoma in situ

B.Lobular carcinoma in situ

C.Invasive ductal carcinoma

D.Invasive lobular carcinoma

E.Inflammatory carcinoma

F.Phyllodes tumour

A

The correct answer is Invasive ductal carcinoma

A post menopausal woman is more likely to have a ductal carcinoma and they tend to occur at a single focus within the breast.

How well did you know this?
1
Not at all
2
3
4
5
Perfectly
281
Q

A 72 year old woman presents with 2 breast lumps. She has a history of breast cancer in the opposite breast 5 years ago.

A.Ductal carcinoma in situ

B.Lobular carcinoma in situ

C.Invasive ductal carcinoma

D.Invasive lobular carcinoma

E.Inflammatory carcinoma

F.Phyllodes tumour

G.Paget’s disease of the nipple

A

The correct answer is Invasive lobular carcinoma

This is likely to be an invasive lobular carcinoma, mainly due to the multifocal lesions and the history of previous breast cancer in the opposite breast.

How well did you know this?
1
Not at all
2
3
4
5
Perfectly
282
Q

Parasympathetic fibres innervating the parotid gland originate from which of the following?

Submandibular ganglion

Otic ganglion

Ciliary ganglion

Pterygopalatine ganglion

None of the above

A

Secretion of saliva by the parotid gland is controlled by nerve fibres originating in the inferior salivatory nucleus; these leave the brain via the tympanic nerve (branch of glossopharyngeal nerve (CN IX), travel through the tympanic plexus (located in the middle ear), and then form the lesser petrosal nerve until reaching the otic ganglion. After synapsing in the Otic ganglion, the postganglionic (postsynaptic) fibres travel as part of the auriculotemporal nerve (a branch of the mandibular nerve (V3) to reach the parotid gland.

How well did you know this?
1
Not at all
2
3
4
5
Perfectly
283
Q

Which of the following structures suspends the spinal cord in the dural sheath?

Filum terminale

Conus medullaris

Ligamentum flavum

Denticulate ligaments

Anterior longitudinal ligament

A

The spinal cord is approximately 45cm in men and 43cm in women. The denticulate ligament is a continuation of the pia mater (innermost covering of the spinal cord) which has intermittent lateral projections attaching the spinal cord to the dura mater.

How well did you know this?
1
Not at all
2
3
4
5
Perfectly
284
Q

Where is the ‘safe triangle’ for chest drain insertion located?

4th intercostal space, mid axillary line

5th intercostal space, mid axillary line

4th intercostal space, mid scapular line

5th intercostal space, mid scapular line

4th intercostal space, mid clavicular line

A

‘Safe Triangle’ for chest drain insertion:

5th intercostal space, mid axillary line

How well did you know this?
1
Not at all
2
3
4
5
Perfectly
285
Q

A 73 year old man develops disseminated intravascular coagulation following an abdominal aortic aneurysm repair. He receives an infusion of cryoprecipitate. What is the major constituent of this infusion?

Factor VIII

Factor IX

Protein C

Protein S

Factor V

A

Factor 8

How well did you know this?
1
Not at all
2
3
4
5
Perfectly
286
Q

A 49 -year-old male presents with discomfort in the fingers of his left hand. On examination, the ring and little fingers of his left hand are flexed and unable to extend completely. He is able to make a fist with the hand. Palpation reveals thickened nodules on the medial half of the palm.

A.de Quervain’s tenosynovitis

B.Dupuytren’s contracture

C.Bouchard’s nodes

D.Ganglion

E.Carpal tunnel syndrome

F.Radial nerve injury

G.Ulnar nerve injury

A

Dupuytren’s contracture

Discomfort of the hand is not uncommon in Dupuytrens contracture, true pain is unusual. The disease most commonly affects the ring and little fingers.

How well did you know this?
1
Not at all
2
3
4
5
Perfectly
287
Q

A 62 year old man presents after his wife commented on the unusual shape of his fingers. On examination, he has a hard swelling adjacent to the distal interphalangeal joint of his index finger of the right hand with lateral deviation of the finger tip. There is no sensory disturbance and the swelling is not tender.

A.de Quervain’s tenosynovitis

B.Dupuytren’s contracture

C.Bouchard’s nodes

D.Ganglion

E.Carpal tunnel syndrome

F.Radial nerve injury

G.Ulnar nerve injury

H.Heberden’s nodes

A

Heberden’s nodes

These are bony outgrowths that occur in the distal interphalangeal joint in association with osteoarthritis. They may skew the finger tip sideways. Bouchards nodes are similar, but affect the proximal interphalangeal joint.

How well did you know this?
1
Not at all
2
3
4
5
Perfectly
288
Q

A 57 year - old lady presents with a three month history of pins and needles in the fingers of the right hand, particularly at night. On examination, there is some loss of the sensation over the palmar aspect of the lateral three fingers and wasting of the thenar eminence.

A.de Quervain’s tenosynovitis

B.Dupuytren’s contracture

C.Bouchard’s nodes

D.Ganglion

E.Carpal tunnel syndrome

A

Carpal tunnel syndrome

Carpal tunnel syndrome commonly produces pain at night as the wrists are flexed during sleep. Compromise of the median nerve may produce wasting of the thenar eminence muscles.

How well did you know this?
1
Not at all
2
3
4
5
Perfectly
289
Q

Hard, bony outgrowths or gelatinous cysts on the proximal interphalangeal joints (the middle joints of fingers or toes.) They are a sign of osteoarthritis, and are caused by formation of calcific spurs of the articular cartilage

A.de Quervain’s tenosynovitis

B.Dupuytren’s contracture

C.Bouchard’s nodes

D.Ganglion

E.Carpal tunnel syndrome

F.Radial nerve injury

G.Ulnar nerve injury

H.Heberden’s nodes

I.Tendon sheath infection

A

Bouchards nodes

How well did you know this?
1
Not at all
2
3
4
5
Perfectly
290
Q

Typically develop in middle age, beginning either with a chronic swelling of the affected joints or the sudden painful onset of redness, numbness, and loss of manual dexterity. This initial inflammation and pain eventually subsides, and the patient is left with a permanent bony outgrowth that often skews the fingertip sideways. It typically affects the DIP joint.

A.de Quervain’s tenosynovitis

B.Dupuytren’s contracture

C.Bouchard’s nodes

D.Ganglion

E.Carpal tunnel syndrome

F.Radial nerve injury

G.Ulnar nerve injury

H.Heberden’s nodes

I.Tendon sheath infection

A

Based on the description provided, the condition being described is:

H. Heberden’s nodes

Let’s break down the key features that point to this diagnosis:

  1. Age of onset: The description mentions “typically develop in middle age,” which is consistent with Heberden’s nodes.
  2. Joint affected: The condition “typically affects the DIP joint” (Distal Interphalangeal joint). This is the defining characteristic of Heberden’s nodes.
  3. Presentation:
    • Can begin with “chronic swelling of the affected joints” or “sudden painful onset of redness, numbness, and loss of manual dexterity.”
    • The initial inflammation and pain eventually subsides.
    • Results in “permanent bony outgrowth that often skews the fingertip sideways.”
  4. Nature of the condition: Heberden’s nodes are bony enlargements of the DIP joints, which matches the description of “permanent bony outgrowth.”

Heberden’s nodes are a sign of osteoarthritis in the hands. They are named after William Heberden, an English physician who first described them in 1802. These nodes represent osteophytes (bone spurs) that form as a result of the degenerative process in osteoarthritis.

It’s worth noting that while similar, this condition is different from:

  • Bouchard’s nodes (option C), which affect the proximal interphalangeal (PIP) joints rather than the DIP joints.
  • The other options listed do not match the specific description provided, particularly the involvement of the DIP joint and the characteristic bony outgrowth.​​​​​​​​​​​​​​​​
How well did you know this?
1
Not at all
2
3
4
5
Perfectly
291
Q

Which of the following is not characteristic of a granuloma?

Altered macrophages

Fused macrophages

Epithelioid cells

Mixture of chronic inflammatory cells

Polymorphnuclear leucocytes, cellular debris and fibrin

A

Polymorphnuclear leucocytes, cellular debris and fibrin
These are typical components of an abscess cavity. Polymorphonuclear leucocytes may be found in a granuloma if there is a focus of suppuration.

How well did you know this?
1
Not at all
2
3
4
5
Perfectly
292
Q

A 42 year old man presents with a painless lump in the left testicle that he noticed on self examination. Clinically there is a firm nodule in the left testicle, ultrasound appearances show an irregular mass lesion. His serum AFP and HCG levels are both within normal limits. What is the most likely diagnosis?

Yolk sack tumour

Seminoma

Testicular teratoma

Epididymo-orchitis

Adenomatoid tumour

A

Seminomas typically have normal AFP and HCG. These are usually raised in teratomas and yolk sac tumours

This man’s age, presenting symptoms and normal tumour markers make a seminoma the most likely diagnosis. Epididymo-orchitis does not produce irregular mass lesions which are painless.

Testicular disorders

Testicular cancer
Testicular cancer is the most common malignancy in men aged 20-30 years. Around 95% of cases of testicular cancer are germ-cell tumours. Germ cell tumours may essentially be divided into:

Tumour type Key features Tumour markers Pathology
Seminoma
Commonest subtype (50%)
Average age at diagnosis = 40
Even advanced disease associated with 5 year survival of 73%
AFP usually normal
HCG elevated in 10% seminomas
Lactate dehydrogenase; elevated in 10-20% seminomas (but also in many other conditions)
Sheet like lobular patterns of cells with substantial fibrous component. Fibrous septa contain lymphocytic inclusions and granulomas may be seen.
Non seminomatous germ cell tumours (42%)
Teratoma
Yolk sac tumour
Choriocarcinoma
Mixed germ cell tumours (10%)
Younger age at presentation =20-30 years
Advanced disease carries worse prognosis (48% at 5 years)
Retroperitoneal lymph node dissection may be needed for residual disease after chemotherapy
AFP elevated in up to 70% of cases
HCG elevated in up to 40% of cases
Other markers rarely helpful

Risk factors for testicular cancer
Cryptorchidism
Infertility
Family history
Klinefelter’s syndrome
Mumps orchitis

Features
A painless lump is the most common presenting symptom
Pain may also be present in a minority of men
Other possible features include hydrocele, gynaecomastia

Diagnosis
Ultrasound is first-line
CT scanning of the chest/ abdomen and pelvis is used for staging
Tumour markers (see above) should be measured

Management
Orchidectomy (Inguinal approach)
Chemotherapy and radiotherapy may be given depending on staging
Abdominal lesions >1cm following chemotherapy may require retroperitoneal lymph node dissection.

Prognosis is generally excellent
5 year survival for seminomas is around 95% if Stage I
5 year survival for teratomas is around 85% if Stage I

Benign disease

Epididymo-orchitis
Acute epididymitis is an acute inflammation of the epididymis, often involving the testis and usually caused by bacterial infection.
Infection spreads from the urethra or bladder. In men <35 years, gonorrhoea or chlamydia are the usual infections.
Amiodarone is a recognised non infective cause of epididymitis, which resolves on stopping the drug.
Tenderness is usually confined to the epididymis, which may facilitate differentiating it from torsion where pain usually affects the entire testis.

Testicular torsion
Twist of the spermatic cord resulting in testicular ischaemia and necrosis.
Most common in males aged between 10 and 30 (peak incidence 13-15 years)
Pain is usually severe and of sudden onset.
Cremasteric reflex is lost and elevation of the testis does not ease the pain.
Treatment is with surgical exploration. If a torted testis is identified then both testis should be fixed as the condition of bell clapper testis is often bilateral.

Hydrocele
Presents as a mass that transilluminates, usually possible to ‘get above’ it on examination.
In younger men it should be investigated with USS to exclude tumour.
In children it may occur as a result of a patent processus vaginalis.
Treatment in adults is with a Lords or Jabouley procedure.
Treatment in children is with trans inguinal ligation of PPV.

How well did you know this?
1
Not at all
2
3
4
5
Perfectly
293
Q

A 63 year old lady presents with an obstructing cancer of the sigmoid colon. She is not peritonitic and her imaging demonstrates a solitary liver metastasis.

A.Ileocolic bypass

B.Loop ileostomy

C.High anterior resection

D.Insertion of self expanding metallic stent

E.Left hemicolectomy and on table colonic lavage and primary anastomosis

F.Extended right hemicolectomy and ileocolic anastomosis

A

The correct answer is Insertion of self expanding metallic stent

Ideally, the distant disease should be managed first and then the primary lesion addressed. A self expanding stent is likely to achieve this and avoids a stoma.

How well did you know this?
1
Not at all
2
3
4
5
Perfectly
294
Q

A 65 year old man presents with absolute constipation and abdominal pain. On examination he has marked abdominal distension. A digital rectal examination reveals an empty rectum. A rectal contrast study shows an obstructing lesion of the proximal rectum.

A.Ileocolic bypass

B.Loop ileostomy

C.High anterior resection

D.Insertion of self expanding metallic stent

E.Left hemicolectomy and on table colonic lavage and primary anastomosis

F.Extended right hemicolectomy and ileocolic anastomosis

G.Low anterior resection

H.Loop colostomy of the transverse colon

I.Loop colostomy of the sigmoid colon

J.Right hemicolectomy

A

The correct answer is Loop colostomy of the sigmoid colon

Rectal cancers should not be primarily resected prior to definitive staging and a tumour of this nature is likely to have circumferential margin involvement. Whilst a sigmoid and transverse loop colostomy would both provide an equal relief of obstruction the former procedure has the added benefit of making a subsequent resection safer, since a transverse colostomy would have to be taken down and closed during the course of subsequent surgery.

How well did you know this?
1
Not at all
2
3
4
5
Perfectly
295
Q

A 70 year old lady presents with a two day history of constipation and vomiting. On examination she has right iliac fossa tenderness and little abdominal distension. A CT scan is performed and is suggestive of an obstructing carcinoma of the colonic hepatic flexure (stage T3).

A.Ileocolic bypass

B.Loop ileostomy

C.High anterior resection

D.Insertion of self expanding metallic stent

E.Left hemicolectomy and on table colonic lavage and primary anastomosis

F.Extended right hemicolectomy and ileocolic anastomosis

G.Low anterior resection

H.Loop colostomy of the transverse colon

I.Loop colostomy of the sigmoid colon

J.Right hemicolectomy

A

The correct answer is Right hemicolectomy

This lesion should be amenable to standard right hemicolectomy. Extending the resection to take the middle colic vessels and distal transverse colon is unlikely to provide additional oncological benefit.

How well did you know this?
1
Not at all
2
3
4
5
Perfectly
296
Q

A baby is born by normal vaginal delivery at 39 weeks gestation. Initially all appears well and then the clinical staff become concerned because the baby develops recurrent episodes of cyanosis. These are worse during feeding and improve dramatically when the baby cries. The most likely underlying diagnosis is:

Choanal atresia

Oesophageal reflux

Tetralogy of Fallot

Oesophageal atresia

Congenital diaphragmatic hernia

A

In Choanal atresia the episodes of cyanosis are usually worst during feeding. Improvement may be seen when the baby cries as the oropharyngeal airway is used.

How well did you know this?
1
Not at all
2
3
4
5
Perfectly
297
Q

A 32 year old man presents to the acute surgical unit with acute pancreatitis. Over the next few days he becomes dyspnoeic and his saturations are 89% on air. A CXR shows bilateral pulmonary infiltrates. His CVP pressure is 16mmHg. What is the most likely diagnosis?

Cardiac failure

Pneumococcal pneumonia

Staphylococcal pneumonia

Pneumocystis carinii

Adult respiratory distress syndrome

A

Acute pancreatitis is known to precipitate ARDS. ARDS is characterised by bilateral pulmonary infiltrates and hypoxaemia. Note that pulmonary oedema is excluded by the CVP reading < 18mmHg.

How well did you know this?
1
Not at all
2
3
4
5
Perfectly
298
Q

A 28 year old lady presents with a pigmented lesion on her calf. Excisional biopsy confirms a diagnosis of melanoma measuring 1cm in diameter with a Breslow thickness of 0.1mm. The lesion is less than 1 mm at all resection margins. Which of the following surgical resection margins is acceptable for this lesion?

5 cm

1 cm

0.5 cm

2 cm

3 cm A 28 year old lady presents with a pigmented lesion on her calf. Excisional biopsy confirms a diagnosis of melanoma measuring 1cm in diameter with a Breslow thickness of 0.1mm. The lesion is less than 1 mm at all resection margins. Which of the following surgical resection margins is acceptable for this lesion?

5 cm

1 cm

0.5 cm

2 cm

3 cm

A

1 cm

How well did you know this?
1
Not at all
2
3
4
5
Perfectly
299
Q

A 53 year old lady has undergone a bilateral breast augmentation procedure many years previously. The implants are tense and uncomfortable and are removed. During their removal the surgeon encounters a dense membrane surrounding the implants, it has a coarse granular appearance. The tissue is sent for histology and it demonstrates fibrosis with the presence of calcification. The underlying process responsible for these changes is:

Hyperplasia

Dysplasia

Metastatic calcification

Dystrophic calcification

Necrosis

A

Breast implants often become surrounded by a pseudocapsule and this may secondarily then be subjected to a process of dystrophic calcification.

How well did you know this?
1
Not at all
2
3
4
5
Perfectly
300
Q

A 72 year old female is found to have a malignant lesion in her left arm. She had a mastectomy of the left breast 10 years ago and has chronic lymph oedema of the left arm. What is the most likely cause of the malignancy?

Lymphangiosarcoma

Lymphoma

Myeloma

Angiomyolipoma

Giant cell tumour

A

Lymphangiosarcoma is a rare condition arising as a result of chronic oedema. It is an aggressive malignancy.

How well did you know this?
1
Not at all
2
3
4
5
Perfectly
301
Q

A 45 year old lady undergoes a renal transplant from a living related donor. She is well for several months but on review in the outpatient department is noted to have persistent hypertension and a slight deterioration in renal function.

A.Ureteric anastomotic leak

B.Renal vein thrombosis

C.Acute rejection

D.Chronic allograft nephropathy

E.Renal artery thrombosis

F.Renal artery stenosis

G.Lymphocele

H.Hyperacute rejection

A

Renal artery stenosis

Renal artery stenosis typically occurs over several months and will usually result in the development of hypertension. Most cases can be assessed using duplex scanning and managed with angioplasty.

Complications of renal transplantation

A number of complications may occur following renal transplantation. A critical aspect of post operative care is evaluation of graft function. Post operatively, urine output is the most readily available, and easily measured, indicator of graft function. If an individual was relatively anuric pre-transplant and has a good urine output following surgery then this is more useful than it would be in someone who had a higher volume diuresis prior to transplantation. Recipients can be divided into three main groups following renal transplantation, with regard to their graft function:
Immediate graft function; brisk diuresis and falling serum creatinine
Slow graft function; modest urine output and slowly falling creatinine levels
Delayed graft function; defined as need for dialysis post transplant

Decreased urine output following surgery can be the result of hypovolvaemia or a blocked catheter (commonest causes). Other important causes include rejection, or a vascular complication.

Vascular complications
These may involve the donor vessels, those of the recipient or both. Renal artery thrombosis usually occurs early post transplant, but is uncommon with an incidence of less than 1%. It typically results in graft loss. It usually occurs as a result of a technical problem such a vessel torsion or sub intimal flaps. The usual presenting feature is a sudden cessation of urine output. When suspected, the occlusion is usually well demonstrated with duplex scanning. Ideally immediate surgical re-exploration should occur. Sadly, the graft has usually been lost by this stage and will require graft nephrectomy. Renal vein thrombosis is not as common as arterial graft thrombosis and the usual presenting features include discomfort at the graft site and swelling of the graft associated with loss of urine output. Again, duplex scanning is indicated. Unfortunately, this complication is also associated with a high incidence of graft loss.
Over a longer time frame (typically months) some individuals will develop renal artery stenosis. These individuals will typically develop hypertension and over time graft function will decline as hypertensive nephropathy occurs. It is usually demonstrated by duplex scanning and is usually amenable to endovascular intervention.

Urological complications
Urinary tract complications manifesting as leakage or obstruction are common complications following renal transplantation and occur in up to 10% of patients. The main underlying cause is the relatively poor blood supply to the transplanted ureter. Patients typically present relatively early in the first 5 weeks following transplantation with pain and swelling at the graft site. Imaging with USS is often the initial test. Therapeutic options include surgical re-implantation of the ureter for large leaks and stent insertion and nephrostomy placement for smaller leaks.

Lymphocele
These do not generally occur until 2 weeks or longer after surgery. They are, however, relatively common and may be seen in up to 18% of patients. Symptoms usually occur as a result of mass effect with compression of adjacent structures. These include the vessels supplying both the graft, with deterioration in graft function, the ureter, with alteration in urine output and the recipients lower limb vessels, with development of leg swelling. Creation of a laparoscopic or open peritoneal window is a favored treatment.

Rejection
Four types of graft rejection are recognised; hyperacute, accelerated acute, acute and chronic.

Type of rejection Key features
Hyperacute Occurs within minutes of clamp release
Due to pre formed antibodies
Immediate loss of graft occurs
Accelerated acute Occurs in first few days following surgery
Involved both cellular and antibody mediated injury
Pre-sensitisation of the donor is a common cause
Acute Traditionally the most common type of rejection
Seen days to weeks after surgery
Predominantly a cell mediated process mediated by lymphocytes
Organ biopsy demonstrates cellular infiltrates and graft cell apoptosis
Chronic Increasingly common problem
Typically; graft atrophy and atherosclerosis are seen. Fibrosis often occurs as a late event

How well did you know this?
1
Not at all
2
3
4
5
Perfectly
302
Q

A 39 year old lady undergoes a live related renal transplant. She progresses well. Two weeks following the transplant she is noted to have swelling overlying the transplant site and swelling of the ipsilateral limb.Urine output is acceptable and creatinine unchanged.

A.Ureteric anastomotic leak

B.Renal vein thrombosis

C.Acute rejection

D.Chronic allograft nephropathy

E.Renal artery thrombosis

F.Renal artery stenosis

G.Lymphocele

H.Hyperacute rejection

A

Lymphocele

Swelling over the graft site is often due to a lymphocele and this is further suggested by the normal renal function. They cause symptoms through mass effect and limb swelling may occur. Treatment is often surgical.

How well did you know this?
1
Not at all
2
3
4
5
Perfectly
303
Q

Your consultant decides to perform an open inguinal hernia repair under local anaesthesia. Which of the following dermatomal levels will require blockade?

T10

T12

T11

S1

S2

A

T12

How well did you know this?
1
Not at all
2
3
4
5
Perfectly
304
Q

If a sample is normally distributed which of the following is true?

Mean = standard deviation

Mean = standard error of the mean

Mean = median

Mean = variance

The mode and standard error of the mean have the same value

A

In a normally distributed sample, the mean, median and mode are the same.

How well did you know this?
1
Not at all
2
3
4
5
Perfectly
305
Q

A syndrome consisting of a PTEN mutation and intestinal hamartomas.

A.Peutz-Jeghers syndrome

B.Cowden disease

C.Familial adenomatous polyposis coli

D.Lynch syndrome

E.MYH associated polyposis

A

Cowden disease

PTEN is a tumour supressor gene and loss of function mutations result in up regulation of the mTOR pathway.

How well did you know this?
1
Not at all
2
3
4
5
Perfectly
306
Q

A syndrome which may be present in a patient with multiple intestinal hamartomas and pigmentation spots around the mouth.

A.Peutz-Jeghers syndrome

B.Cowden disease

C.Familial adenomatous polyposis coli

D.Lynch syndrome

E.MYH associated polyposis

A

Peutz-Jeghers syndrome

How well did you know this?
1
Not at all
2
3
4
5
Perfectly
307
Q

A syndrome likely to be present in a 28 year old man who presents with a locally advanced mucinous carcinoma of the caecum. There are scanty polyps in the remaining colon. His father died from colorectal cancer aged 34.

A.Peutz-Jeghers syndrome

B.Cowden disease

C.Familial adenomatous polyposis coli

D.Lynch syndrome

E.MYH associated polyposis

A

The correct answer is Lynch syndrome

Lynch syndrome is likely when right sided colonic cancers occur at a young age. These tumours are often poorly differentiated and mucinous. The Amsterdam criteria can be used to identify families at risk who may benefit from genetic testing

How well did you know this?
1
Not at all
2
3
4
5
Perfectly
308
Q

A 48 year old lady has previously undergone a sigmoid colectomy for carcinoma. On follow up imaging she is found to have a 3cm foci of metastatic disease in segment IV of the liver. What is the most appropriate course of action?

Palliative chemotherapy

External beam radiotherapy

Brachytherapy

Surgical resection alone

Chemotherapy followed by surgical resection

A

The treatment of colorectal liver metastasis is usually with chemotherapy followed by surgical resection. Where surgery is performed for liver metastasis with curative intent, the 5 year survival is 20%. Palliation would generally only be considered if the patient were frail or widespread disease found on imaging. Radiotherapy is not part of the treatment of liver metastasis.

Colorectal cancer treatment

Patients diagnosed as having colorectal cancer should be completely staged using CT of the chest/ abdomen and pelvis. Their entire colon should have been evaluated with colonoscopy or CT colonography. Patients whose tumours lie below the peritoneal reflection should have their mesorectum evaluated with MRI.

Once their staging is complete patients should be discussed within a dedicated colorectal MDT meeting and a treatment plan formulated.

Treatment of colonic cancer
Cancer of the colon is nearly always treated with surgery. Stents, surgical bypass and diversion stomas may all be used as palliative adjuncts. Resectional surgery is the only option for cure in patients with colon cancer. The procedure is tailored to the patient and the tumour location. The lymphatic drainage of the colon follows the arterial supply and therefore most resections are tailored around the resection of particular lymphatic chains (e.g. ileo-colic pedicle for right sided tumours). Some patients may have confounding factors that will govern the choice of procedure, for example a tumour in a patient from a HNPCC family may be better served with a panproctocolectomy rather than segmental resection. Following resection the decision has to be made regarding restoration of continuity. For an anastomosis to heal the key technical factors include; adequate blood supply, mucosal apposition and no tissue tension. Surrounding sepsis, unstable patients and inexperienced surgeons may compromise these key principles and in such circumstances it may be safer to construct an end stoma rather than attempting an anastomosis.
When a colonic cancer presents with an obstructing lesion; the options are to either stent it or resect. In modern practice it is unusual to simply defunction a colonic tumour with a proximal loop stoma. This differs from the situation in the rectum (see below).
Following resection patients with risk factors for disease recurrence are usually offered chemotherapy, a combination of 5FU and oxaliplatin is common.

Rectal cancer
The management of rectal cancer is slightly different to that of colonic cancer. This reflects the rectum’s anatomical location and the challenges posed as a result. Tumours located in the rectum can be surgically resected with either an anterior resection or an abdomino - perineal resection. The technical aspects governing the choice between these two procedures can be complex to appreciate and the main point to appreciate for the MRCS is that involvement of the sphincter complex or very low tumours require APER. In the rectum a 2cm distal clearance margin is required and this may also impact on the procedure chosen. In addition to excision of the rectal tube an integral part of the procedure is a meticulous dissection of the mesorectal fat and lymph nodes (total mesorectal excision/ TME). In rectal cancer surgery invovlement of the cirumferential resection margin carries a high risk of disease recurrence. Because the rectum is an extraperitoneal structure (until you remove it that is!) it is possible to irradiate it, something which cannot be offered for colonic tumours. This has a major impact in rectal cancer treatment and many patients will be offered neoadjuvent radiotherapy (both long and short course) prior to resectional surgery. Patients with T1, 2 and 3 /N0 disease on imaging do not require irradiation and should proceed straight to surgery. Patients with T4 disease will typically have long course chemo radiotherapy. Patients presenting with large bowel obstruction from rectal cancer should not undergo resectional surgery without staging as primary treatment (very different from colonic cancer). This is because rectal surgery is more technically demanding, the anastomotic leak rate is higher and the danger of a positive resection margin in an unstaged patient is high. Therefore patients with obstructing rectal cancer should have a defunctioning loop colostomy.

Summary of procedures
The operations for cancer are segmental resections based on blood supply and lymphatic drainage. These commonly performed procedures are core knowledge for the MRCS and should be understood.

Site of cancer Type of resection Anastomosis Risk of leak
Right colon Right hemicolectomy Ileo-colic Low <5%
Transverse Extended right hemicolectomy Ileo-colic Low <5%
Splenic flexure Extended right hemicolectomy Ileo-colic Low <5%
Splenic flexure Left hemicolectomy Colo-colon 2-5%
Left colon Left hemicolectomy Colo-colon 2-5%
Sigmoid colon High anterior resection Colo-rectal 5%
Upper rectum Anterior resection (TME) Colo-rectal 5%
Low rectum Anterior resection (Low TME) Colo-rectal
(+/- Defunctioning stoma) 10%
Anal verge Abdomino-perineal excision of colon and rectum None n/a

In the emergency setting, where the bowel has perforated, the risk of an anastomotic breakdown is much greater, particularly when the anastomosis is colon-colon. In this situation, an end colostomy is often safer and can be reversed later. When resection of the sigmoid colon is performed and an end colostomy is fashioned the operation is referred to as a Hartmans procedure. Whilst left sided resections are more risky, ileo-colic anastomoses are relatively safe even in the emergency setting and do not need to be defunctioned.

References
A review of the diagnosis and management of colorectal cancer and a summary of the UK National Institute of Clinical Excellence guidelines is provided in:
NICE Guideline NG151. Last updated in 2020.

How well did you know this?
1
Not at all
2
3
4
5
Perfectly
309
Q

Which of the following are not characteristic features of central chemoreceptors in the control of ventilation?

They are located in the medulla oblongata

They are stimulated primarily by venous hypercapnia

They are relatively insensitive to hypoxia

They may be affected by changes in the pH of CSF

During acute hypercapnia the carotid receptors will be stimulated first

A

They are stimulated by arterial carbon dioxide. They take longer to equilibrate than the peripheral chemoreceptors located in the carotid. They are less exposed to acidity due to the blood brain barrier.

How well did you know this?
1
Not at all
2
3
4
5
Perfectly
310
Q

A 48 year old lady with end stage renal failure receives a cadaveric renal transplant. The organ is ABO group matched only. On completion of the vascular anastomoses the surgeons remove the clamps. Over the course of the next twelve minutes the donated kidney becomes dusky and swollen and appears non viable. Which of the following is the most likely process that has caused this event?

IgG anti HLA Class I antibodies in the recipient

IgM anti HLA Class I antibodies in the recipient

IgG anti HLA Class I antibodies from the donor

IgM anti HLA Class I antibodies from the donor

IgM anti HLA Class II antibodies from the recipient

A

Episodes of hyperacute rejection are typically due to preformed antibodies. ABO mismatch is the best example. However, IgG anti HLA Class I antibodies are another potential cause. These events are now seen less commonly because the cross matching process generally takes this possibility into account

IgG anti HLA Class I antibodies in the recipient

How well did you know this?
1
Not at all
2
3
4
5
Perfectly
311
Q

A 45 year old lady develops severe back pain and on examination is found to have clinical evidence of an L5/ S1 radiculopathy. Her symptoms deteriorate and eventually a laminectomy is performed. During a posterior surgical approach the surgeons encounter a tough ligamentous structure lying anterior to the spinous processes. This structure is most likely to be the

Transverse spinal ligament

Supraspinal ligament

Anterior longitudinal ligament

Ligamentum flavum

Posterior longitudinal ligament

A

Ligamentum flavum

How well did you know this?
1
Not at all
2
3
4
5
Perfectly
312
Q

A 30 year old woman presents with pain and swelling of the left shoulder. There is a large radiolucent lesion in the head of the humerus extending to the subchondral plate.

A.Osteosarcoma

B.Osteomalacia

C.Osteoporosis

D.Metastatic carcinoma

E.Osteoblastoma

F.Giant cell tumour

G.Ewing’s sarcoma

A

Giant cell tumour

Giant cell tumours on x-ray have a ‘soap bubble’ appearance. They present as pain or pathological fractures. They commonly metastasize to the lungs.

How well did you know this?
1
Not at all
2
3
4
5
Perfectly
313
Q

A 72 year old woman has a lumbar vertebral crush fracture. She has hypocalcaemia and a low urinary calcium.
A.Osteosarcoma

B.Osteomalacia

C.Osteoporosis

D.Metastatic carcinoma

E.Osteoblastoma

F.Giant cell tumour

G.Ewing’s sarcoma

A

Osteomalacia

Hypocalcemia and low urinary calcium are biochemical features of osteomalacia. Unfortunately surgeons do need to look at some blood results!

How well did you know this?
1
Not at all
2
3
4
5
Perfectly
314
Q

A 16 year old boy presents with severe groin pain after kicking a football. Imaging confirms a pelvic fracture. A previous pelvic x-ray performed 2 weeks ago shows a lytic lesion with ‘onion type’ periosteal reaction.

A.Osteosarcoma

B.Osteomalacia

C.Osteoporosis

D.Metastatic carcinoma

E.Osteoblastoma

F.Giant cell tumour

G.Ewing’s sarcoma

A

Ewing’s sarcoma

A Ewings sarcoma is most common in males between 10-20 years. It can occur in girls. A lytic lesion with a lamellated or onion type periosteal reaction is a classical finding on x-rays. Most patients present with metastatic disease with a 5 year prognosis between 5-10%.

How well did you know this?
1
Not at all
2
3
4
5
Perfectly
315
Q

The cell of origin in virtually all pancreatic carcinomas is which of the following?

The acinar cells

The islet beta cells

The islet alpha cells

The interstitial fibroblasts

The ductular epithelium

A

Over 90% of pancreatic carcinomas are adenocarcinomas and are thus of ductular epithelial origin.

How well did you know this?
1
Not at all
2
3
4
5
Perfectly
316
Q

Which of the following does not pass through the superior orbital fissure?

Lacrimal nerve

Abducens nerve

Opthalmic artery

Trochlear nerve

Superior opthalmic vein

A

Mnemonic for the nerves passing through the supraorbital fissure:

Live Frankly To See Absolutely No Insult

Lacrimal
Frontal
Trochlear
Superior Division of Oculomotor
Abducens
Nasociliary
Inferior Division of Oculomotor nerve

The opthalmic artery arises from the internal carotid immediately after it has pierced the dura and arachnoid. It runs through the optic canal below the optic nerve and within its dural and arachnoid sheaths. It terminates as the supratrochlear and dorsal nasal arteries

How well did you know this?
1
Not at all
2
3
4
5
Perfectly
317
Q

An 18 year old man undergoes a tonsillectomy for attacks of recurrent acute tonsillitis. Whilst in recovery he develops a post operative haemorrhage. Which of the following vessels is the most likely culprit?

Facial vein

External palatine vein

External carotid artery

Internal jugular vein

None of the above

A

The external palatine vein lies immediately lateral to the tonsil and if damaged may be a cause of reactionary haemorrhage following tonsillectomy.

How well did you know this?
1
Not at all
2
3
4
5
Perfectly
318
Q

A 25 year old male presents with epistaxis, the ENT SpR plans to cauterise the bleeding point with silver nitrate.

A.1% xylocaine with 1 in 200,000 adrenaline

B.1% Lignocaine

C.0.5% Bupivacaine with 1 in 200,000 adrenaline

D.0.5% Bupivicaine

E.Prilocaine 1%

F.Procaine 1%

G.Cocaine 25%

H.Cocaine 10%

A

The correct answer is 1% xylocaine with 1 in 200,000 adrenaline

Historically cocaine was popular for the management of epistaxis. Some surgeons will still routinely use cocaine paste for this indication. Its popularity stems from the fact that it causes vasospasm. However, systemic absorption carries the risk of adverse reactions. Where it is used the correct dose is 4%. Topically applied short acting local anaesthetic agents with adrenaline may produce similar effects, with lower risks of toxicity.

How well did you know this?
1
Not at all
2
3
4
5
Perfectly
319
Q

An 18 year old boy requires a Zadeks procedure

A.1% xylocaine with 1 in 200,000 adrenaline

B.1% Lignocaine

C.0.5% Bupivacaine with 1 in 200,000 adrenaline

D.0.5% Bupivicaine

E.Prilocaine 1%

F.Procaine 1%

G.Cocaine 25%

H.Cocaine 10%

A

The correct answer is 1% Lignocaine

This is excision of the toe nail and a fast acting local anaesthetic is indicated. Adrenaline should be avoided in this setting as it can cause digital ischaemia

How well did you know this?
1
Not at all
2
3
4
5
Perfectly
320
Q

A 72 year old woman fractured her distal radius. A Biers Block is planned to facilitate reduction of the fracture.

A.1% xylocaine with 1 in 200,000 adrenaline

B.1% Lignocaine

C.0.5% Bupivacaine with 1 in 200,000 adrenaline

D.0.5% Bupivicaine

E.Prilocaine 1%

F.Procaine 1%

G.Cocaine 25%

H.Cocaine 10%

A

Prilocaine 1%

This is the best local anaesthetic for this. Bupivacaine may cause cardiotoxicity and should be avoided.

How well did you know this?
1
Not at all
2
3
4
5
Perfectly
321
Q

Which of the nerves listed below is responsible for providing innervation to the lower molar teeth?

Greater palatine nerve

Nasopalatine nerve

Inferior alveolar nerve

Zygomatic nerve

Mandibular nerve

A

The branches of the lower molar and premolar teeth are supplied by branches of the inferior alveolar nerve. Those of the canine and incisors by the incisive branch of the same nerve. The gingiva and supporting structures are innervated by the lingual nerve.

Trigeminal nerve

The trigeminal nerve is the main sensory nerve of the head. In addition to its major sensory role, it also innervates the muscles of mastication.

Distribution of the trigeminal nerve
Sensory
Scalp
Face
Oral cavity (and teeth)
Nose and sinuses
Dura mater
Motor
Muscles of mastication
Mylohyoid
Anterior belly of digastric
Tensor tympani
Tensor palati
Autonomic connections (ganglia)
Ciliary
Sphenopalatine
Otic
Submandibular

Path
Originates at the pons
Sensory root forms the large, crescentic trigeminal ganglion within Meckel’s cave, and contains the cell bodies of incoming sensory nerve fibres. Here the 3 branches exit.
The motor root cell bodies are in the pons and the motor fibres are distributed via the mandibular nerve. The motor root is not part of the trigeminal ganglion.

Branches of the trigeminal nerve
Ophthalmic nerve Sensory only
Maxillary nerve Sensory only
Mandibular nerve Sensory and motor

Sensory
Ophthalmic Exits skull via the superior orbital fissure
Sensation of: scalp and forehead, the upper eyelid, the conjunctiva and cornea of the eye, the nose (including the tip of the nose, except alae nasi), the nasal mucosa, the frontal sinuses, and parts of the meninges (the dura and blood vessels).
Maxillary nerve Exit skull via the foramen rotundum
Sensation: lower eyelid and cheek, the nares and upper lip, the upper teeth and gums, the nasal mucosa, the palate and roof of the pharynx, the maxillary, ethmoid and sphenoid sinuses, and parts of the meninges.
Mandibular nerve Exit skull via the foramen ovale
Sensation: lower lip, the lower teeth and gums, the chin and jaw (except the angle of the jaw), parts of the external ear, and parts of the meninges.

Motor
Distributed via the mandibular nerve.
The following muscles of mastication are innervated:
Masseter
Temporalis
Medial pterygoid
Lateral pterygoid

Other muscles innervated include:
Tensor veli palatini
Mylohyoid
Anterior belly of digastric
Tensor tympani

How well did you know this?
1
Not at all
2
3
4
5
Perfectly
322
Q

Which of the anaesthetic agents below is most likely to induce adrenal suppression?

Sodium thiopentone

Midazolam

Propofol

Etomidate

Ketamine

A

Etomidate is a recognised cause of adrenal suppression, this has been associated with increased mortality when used as a sedation agent in the critically ill.

How well did you know this?
1
Not at all
2
3
4
5
Perfectly
323
Q

Rapid onset of anaesthesia

Pain on IV injection

Rapidly metabolised with little accumulation of metabolites

Proven anti emetic properties

Moderate myocardial depression

Widely used especially for maintaining sedation on ITU, total IV anaesthesia and for daycase surgery

A

Propofol

How well did you know this?
1
Not at all
2
3
4
5
Perfectly
324
Q

Extremely rapid onset of action making it the agent of choice for rapid sequence of induction

Marked myocardial depression may occur

Metabolites build up quickly

Unsuitable for maintenance infusion

Little analgesic effects

A

Sodium thiopentone

How well did you know this?
1
Not at all
2
3
4
5
Perfectly
325
Q

May be used for induction of anaesthesia

Has moderate to strong analgesic properties

Produces little myocardial depression making it a suitable agent for anaesthesia in those who are haemodynamically unstable

May induce state of dissociative anaesthesia resulting in nightmares

A

Ketamine

How well did you know this?
1
Not at all
2
3
4
5
Perfectly
326
Q

Has favorable cardiac safety profile with very little haemodynamic instability

No analgesic properties

Unsuitable for maintaining sedation as prolonged (and even brief) use may result in adrenal suppression

Post operative vomiting is common

A

Etomidate

How well did you know this?
1
Not at all
2
3
4
5
Perfectly
327
Q

A 22 year old female undergoes a thyroidectomy. The resected specimen shows a non encapsulated tumour with papillary projections and pale empty nuclei.

A.Follicular carcinoma

B.Anaplastic carcinoma

C.Medullary carcinoma

D.Papillary carcinoma

E.Lymphoma

F.Hashimotos thyroiditis

G.Graves disease

A

Papillary carcinoma

The presence of papillary structures together with the cytoplasmic features described is strongly suggestive of papillary carcinoma. They are seldom encapsulated.

How well did you know this?
1
Not at all
2
3
4
5
Perfectly
328
Q

32 year old lady undergoes a thyroidectomy for a mild goitre. The resected specimen shows an intense lymphocytic infiltrate with acinar destruction and fibrosis.

A.Follicular carcinoma

B.Anaplastic carcinoma

C.Medullary carcinoma

D.Papillary carcinoma

E.Lymphoma

F.Hashimotos thyroiditis

G.Graves disease

A

Hashimotos thyroiditis

Lymphocytic infiltrates and fibrosis are typically seen in Hashimotos thyroiditis. In Lymphoma only dense lymphatic type tissue is usually present.

How well did you know this?
1
Not at all
2
3
4
5
Perfectly
329
Q

A 7 year old boy develops a persistent fever following an open appendicectomy for gangrenous appendicitis. On examination he has erythema of the wound and some abdominal distension. For the following post operative scenarios please select the most appropriate investigation or management.

A.Trans anal ultrasound scan

B.Upper abdominal ultrasound scan

C.Total abdominal ultrasound scan

D.PET CT scan

E.ERCP

F.Small bowel MRI Scan

G.Abdominal CT scan with oral and IV contrast

H.Non contrast abdominal CT scan

I.Laparotomy

J.Laparoscopy

A

Total abdominal ultrasound scan

This patient has risk factors for a wound infection which is not in itself an indication for scanning. However, he also had abdominal distension and this, together with the history of distension would generally attract a recommendation for imaging. A USS will show an abdominal wall collection and more importantly any phrenic or pelvic collections. Unlike adult practice, CT scanning is rarely performed in children.

How well did you know this?
1
Not at all
2
3
4
5
Perfectly
330
Q

A 56 year old man is 8 days following a left hemicolectomy. He has developed a swinging pyrexia over the past 48 hours and has an ileus clinically.

A.Trans anal ultrasound scan

B.Upper abdominal ultrasound scan

C.Total abdominal ultrasound scan

D.PET CT scan

E.ERCP

F.Small bowel MRI Scan

G.Abdominal CT scan with oral and IV contrast

H.Non contrast abdominal CT scan

I.Laparotomy

J.Laparoscopy

A

Abdominal CT scan with oral and IV contrast

This would most likely be the result of an anastomotic leak with abscess formation. Detailed imaging is required to allow accurate diagnosis and planning of management.

How well did you know this?
1
Not at all
2
3
4
5
Perfectly
331
Q

A 43 year old lady underwent an acute cholecystectomy for cholecystitis. A drain is left during the procedure. Over the next 5 days the drain has been accumulating between 100-200ml of bile per 24 hour period.

A.Trans anal ultrasound scan

B.Upper abdominal ultrasound scan

C.Total abdominal ultrasound scan

D.PET CT scan

E.ERCP

F.Small bowel MRI Scan

G.Abdominal CT scan with oral and IV contrast

H.Non contrast abdominal CT scan

I.Laparotomy

J.Laparoscopy

A

ERCP

The most likely cause of a bile leak in this scenario would be a dislodged clip from the cystic duct. Whilst it may be tempting to try and plan to manage this surgically the anatomy is often unfavorable and the duct very difficult to identify. An ERCP has the advantage of demonstrating the cause of the leak and allowing placement of a stent. This will usually allow the resolution of most leaks without the need for surgery.

How well did you know this?
1
Not at all
2
3
4
5
Perfectly
332
Q

At which level does the aorta perforate the diaphragm?

T10

T9

T8

T11

T12

A

Memory aid:
T8 (8 letters) = vena cava
T10 (10 letters) = oesophagus
T12 (12 letters) = aortic hiatus

How well did you know this?
1
Not at all
2
3
4
5
Perfectly
333
Q

A 24 year old lady is stabbed in the buttock. Following the injury the wound is sutured in the emergency department. Eight weeks later she attends the clinic, as she walks into the clinic room she has a waddling gait and difficulty with thigh abduction. On examination she has buttock muscle wasting. Which nerve has been injured?

Superior gluteal nerve

Obturator nerve

Sciatic nerve

Femoral nerve

Inferior gluteal nerve

A

Damage to the superior gluteal nerve will result in a Trendelenburg gait.

How well did you know this?
1
Not at all
2
3
4
5
Perfectly
334
Q

Which of the following does not stimulate insulin release?

Gastrin

Atenolol

Protein

Secretin

Vagal cholinergic action

A

Beta blockers inhibit the release of insulin.

Stimulation of insulin release:

Glucose

Amino acid

Vagal cholinergic

Secretin/Gastrin/CCK

Fatty acids

Beta adrenergic drugs

How well did you know this?
1
Not at all
2
3
4
5
Perfectly
335
Q

During a radical neck dissection, division of which of the following fascial layers will expose the ansa cervicalis?

Pretracheal fascia

Carotid sheath

Prevertebral fascia

Investing layer of fascia

Sibsons fascia

A

The ansa cervicalis lies anterior to the carotid sheath. It may be exposed by division of the pretracheal fascia at the posterolateral aspect of the thyroid gland. The pre vertebral fascia lies more posteriorly and division of the investing layer of fascia will not expose this nerve.

How well did you know this?
1
Not at all
2
3
4
5
Perfectly
336
Q

Which one is true for pheochromacytoma?

10% of cases are bilateral.

10% occur in children.

11% are malignant (higher when tumour is located outside the adrenal).

10% will not be hypertensive.

All of the above

A

All of the above. Rule of 10s in phaeochromocytoma

How well did you know this?
1
Not at all
2
3
4
5
Perfectly
337
Q

An 8 year old boy falls onto an outstretched hand and is brought to the emergency department. He is examined by a doctor and a bony injury is cleared clinically. He re-presents a week later with pain in his hand. What is the most likely underlying injury?

Fracture of the distal radius

Fracture of the scaphoid

Dislocation of the lunate

Rupture of flexor pollicis longus tendon

Bennett’s fracture

A

Scaphoid fractures in children are rare, will usually involve the distal pole and are easily missed. The initial clinical examination (and sometimes x-rays) may be normal and repeated clinical examination and imaging is advised for this reason. Whilst the other injuries may be sustained from a fall onto an outstretched hand they are less likely to be overlooked on clinical examination. In the case of a Bennetts fracture, the injury mechanism is less compatible with this type of injury.

How well did you know this?
1
Not at all
2
3
4
5
Perfectly
338
Q

A 73 year old lady presents with symptoms of faecal incontinence. On examination she has weak anal sphincter muscles. What are the main nerve root values of the nerves supplying the external anal sphincter?

S2,3

L5, S1

S4,5

S5

S2,3,4

A

S2, 3, 4 Keeps the poo off the floor

The external anal sphincter is innervated by the inferior rectal branch of the pudendal nerve, this has root values of S2, 3 and the perineal branch of S4.

How well did you know this?
1
Not at all
2
3
4
5
Perfectly
339
Q

A 22 year old falls over and lands on a shard of glass. It penetrates the palmar aspect of his hand, immediately lateral to the pisiform bone. Which of the following structures is most likely to be injured?

Palmar cutaneous branch of the median nerve

Lateral tendons of flexor digitorum superficialis

Ulnar artery

Flexor carpi radialis tendons

Lateral tendons of flexor digitorum profundus

A

The ulnar nerve and artery are at most immediate risk in this injury. This is illustrated in the image below

How well did you know this?
1
Not at all
2
3
4
5
Perfectly
340
Q

A 63 year old man is admitted with severe headache, nausea and recent epileptic fit. Fundoscopy shows papilloedema. He is also noted to have diplopia.

A.Optic nerve

B.Oculomotor nerve

C.Trigeminal nerve

D.Facial nerve

E.Abducens nerve

F.Glossopharyngeal nerve

G.Vestibulocochlear nerve

H.Accessory nerve

I.Hypoglossal nerve

A

Abducens nerve

The long intracranial course of this nerve makes it susceptible to damage early in the course of raised ICP.

How well did you know this?
1
Not at all
2
3
4
5
Perfectly
341
Q

An 18 year old boy undergoes an uncomplicated tonsillectomy for recurrent attacks of tonsillitis. Post operatively he complains of otalgia.

A.Optic nerve

B.Oculomotor nerve

C.Trigeminal nerve

D.Facial nerve

E.Abducens nerve

F.Glossopharyngeal nerve

G.Vestibulocochlear nerve

H.Accessory nerve

I.Hypoglossal nerve

A

Glossopharyngeal nerve

The glossopharyngeal nerve supplies this area and the ear and otalgia may be the result of referred pain.

How well did you know this?
1
Not at all
2
3
4
5
Perfectly
342
Q

A 63 year old female is referred to the surgical clinic with an iron deficiency anaemia. Her past medical history includes a left hemi colectomy but no other co-morbidities. At what site is most dietary iron absorbed?

Stomach

Duodenum

Proximal ileum

Distal ileum

Colon

A

Iron is best absorbed from the proximal small bowel (duodenum and jejunum) in the Fe 2+ state. Iron is transported across the small bowel mucosa by a divalent membrane transporter protein (hence the improved absorption of Fe 2+). The intestinal cells typically store the bound iron as ferritin. Cells requiring iron will typically then absorb the complex as needed.

How well did you know this?
1
Not at all
2
3
4
5
Perfectly
343
Q

Which of the following structures lies posterior to the femoral nerve in the femoral triangle?

Adductor longus

Pectineus

Psoas major

Iliacus

None of the above

A

The iliacus lies posterior to the femoral nerve in the femoral triangle. The femoral sheath lies anterior to the iliacus and pectineus muscles.

How well did you know this?
1
Not at all
2
3
4
5
Perfectly
344
Q

A 46 year old lady presents with symptoms of diarrhoea, weight loss of 10 Kg and a skin rash of erythematous blisters involving the abdomen and buttocks. The blisters have an irregular border and both intact and ruptured vesicles. What is the most likely diagnosis?

Colonic adenocarcinoma

Pancreatic adenocarcinoma

Tropical sprue

Glucagonoma

Insulinoma

A

Glucagonoma is strongly associated with necrolytic migratory erythema.

How well did you know this?
1
Not at all
2
3
4
5
Perfectly
345
Q

You are assisting in an open right adrenalectomy for a large adrenal adenoma. The consultant is distracted and you helpfully pull the adrenal into the wound to improve the view. Unfortunately this is followed by brisk bleeding. The vessel responsible for this is most likely to be:

Portal vein

Phrenic vein

Right renal vein

Superior mesenteric vein

Inferior vena cava

A

It drains directly via a very short vessel. If the sutures are not carefully tied then it may be avulsed off the IVC. An injury best managed using a Satinsky clamp and a 6/0 prolene suture.

How well did you know this?
1
Not at all
2
3
4
5
Perfectly
346
Q

A 28 year old lady requires an episiotomy for a ventouse vaginal delivery. Which of the nerves listed below will usually be anaesthetised to allow the episiotomy?

Femoral

Ilioinguinal

Pudendal

Genitofemoral

Sacral plexus

A

The pudendal nerve innervates the posterior vulval area and is routinely blocked in procedures such as episiotomy.

How well did you know this?
1
Not at all
2
3
4
5
Perfectly
347
Q

An enthusiastic surgical registrar undertakes his first solo splenectomy. The operation is far more difficult than anticipated and the registrar leaves a tube drain to the splenic bed at the end of the procedure. Over the following 24 hours approximately 500ml of clear fluid has entered the drain. Biochemical testing of the fluid is most likely to reveal:

Elevated creatinine

Elevated triglycerides

Elevated glucagon

Elevated amylase

None of the above

A

During splenectomy the tail of the pancreas may be damaged. The pancreatic duct will then drain into the splenic bed, amylase is the most likely biochemical finding. Glucagon is not secreted into the pancreatic duct.

How well did you know this?
1
Not at all
2
3
4
5
Perfectly
348
Q

A 22 year old man returns to the UK from holiday in India. He presents with painless jaundice. On examination he is not deeply jaundiced and there is no organomegaly.

A.Gilberts syndrome

B.Crigler Najjar syndrome

C.Hepatocellular carcinoma

D.Mirizzi syndrome

E.Hepatitis A

F.Hepatitis E

G.Bile duct stones

H.Multi cystic liver disease

A

Hepatitis A

Infective hepatitis is the most likely cause. In the UK, foreign travel is a common cause of developing infectious hepatitis, of which hepatitis A is the most common.

How well did you know this?
1
Not at all
2
3
4
5
Perfectly
349
Q

A 56 year old man presents with jaundice. He has a long history of alcohol misuse. On examination he is jaundiced and ultrasound shows multiple echo dense lesions in both lobes of the liver. His alpha feto protein is elevated 6 times the normal range

A.Gilberts syndrome

B.Crigler Najjar syndrome

C.Hepatocellular carcinoma

D.Mirizzi syndrome

E.Hepatitis A

F.Hepatitis E

G.Bile duct stones

H.Multi cystic liver disease

A

Hepatocellular carcinoma

HCC may complicate cirrhosis. AFP is often raised in HCC.

How well did you know this?
1
Not at all
2
3
4
5
Perfectly
350
Q

A 32 year old man who has suffered from Crohns disease for many years presents with intermittent jaundice. When it occurs it is obstructive in nature. It then usually resolves spontaneously.

A.Gilberts syndrome

B.Crigler Najjar syndrome

C.Hepatocellular carcinoma

D.Mirizzi syndrome

E.Hepatitis A

F.Hepatitis E

G.Bile duct stones

H.Multi cystic liver disease

A

Bile duct stones

Bile salts are absorbed in the terminal ileum. When this process is impaired as in Crohns the patient may develop gallstones, if these pass into the CBD then obstructive jaundice will result.

How well did you know this?
1
Not at all
2
3
4
5
Perfectly
351
Q

A 48 year old lady is undergoing an axillary node clearance for breast cancer. Which of the structures listed below are most likely to be encountered during the axillary dissection?

Cords of the brachial plexus

Thoracodorsal trunk

Internal mammary artery

Thoracoacromial artery

None of the above

A

Beware of damaging the thoracodorsal trunk if a latissimus dorsi flap reconstruction is planned.

The thoracodorsal trunk runs through the nodes in the axilla. If injured it may compromise the function and blood supply to latissimus dorsi, which is significant if it is to be used as a flap for a reconstructive procedure.

How well did you know this?
1
Not at all
2
3
4
5
Perfectly
352
Q

A 56 year old lady is referred to the colorectal clinic with symptoms of pruritus ani. On examination a polypoidal mass is identified inferior to the dentate line. A biopsy confirms squamous cell carcinoma. To which of the following lymph node groups will the lesion potentially metastasise?

Internal iliac

External iliac

Mesorectal

Inguinal

None of the above

A

Lesions distal to the dentate line drain to the inguinal nodes. Occasionally this will result in the need for a block dissection of the groin.

How well did you know this?
1
Not at all
2
3
4
5
Perfectly
353
Q

A 56 year old man presents with symptoms of neuropathic facial pain and some weakness of the muscles of facial expression on the right side. On examination he has a hard mass approximately 6cm anterior to the right external auditory meatus. What is the most likely diagnosis?

Pleomorphic adenoma

Adenocarcinoma

Mucoepidermoid carcinoma

Adenoid cystic carcinoma

Lymphoma

A

The patient is most likely to have a malignant lesion within the parotid. Of the malignancies listed; adenoid cystic carcinoma has the greatest tendency to perineural invasion.

How well did you know this?
1
Not at all
2
3
4
5
Perfectly
354
Q

A 45 year old women with breast cancer is started on a chemotherapy regime containing epirubicin. What is the primary mode of action of this drug?

Intercalation of DNA

Antimetabolite

Monoclonal antibody to epidermal growth factor

Inhibition of DNA gyrase

Inhibition of topoisomerase 1

A

Intercalation of DNA

How well did you know this?
1
Not at all
2
3
4
5
Perfectly
355
Q

Which of the following cancers is not associated with the human papillomavirus?

Anal cancer

Oropharyngeal cancer

Tracheal cancer

Vulval cancer

Penile cancer

A

HPV is associated with:

Cervical cancer (HPV 16/18 most common)
Anal cancer
Penile cancer
Vulval cancer
Oropharyngeal cancer

How well did you know this?
1
Not at all
2
3
4
5
Perfectly
356
Q

A 28 year old man has a long history of recurrent chest infections. On examination, he is noted to have no palpable vas deferens. However, both testes are located within the scrotum. What is the most likely underlying disease association?

Kleinfelters syndrome

Kallmann syndrome

Cystic fibrosis

Coeliac disease

Gardners syndrome

A

99% of males with cystic fibrosis will have absent vas

How well did you know this?
1
Not at all
2
3
4
5
Perfectly
357
Q

A 63 year old man is admitted with obstructive jaundice that has developed over the past 3 weeks. He was previously well and on examination has a smooth mass in his right upper quadrant.

A.Carcinoma of the head of the pancreas

B.Bile duct stricture

C.Mirizzi syndrome

D.Bile duct stones

E.Chronic cholecystitis

F.Peri hilar lymphadenopathy

G.Fitz - Hugh Curtis syndrome

A

Carcinoma of the head of the pancreas

Carcinoma of the pancreas (Courvoisiers law!). The development of jaundice in association with a smooth right upper quadrant mass is typical of distal biliary obstruction secondary to pancreatic malignancy. A bile duct stricture would not present in this way, all the other choices are related to gallstones and Fitz Hugh Curtis syndrome is a complication of pelvic inflammatory disease.

How well did you know this?
1
Not at all
2
3
4
5
Perfectly
358
Q

A 41 year old lady is admitted with colicky right upper quadrant pain. On clinical examination she has a mild pyrexia and is clinically jaundiced. An ultrasound scan is reported as showing gallstones and the patient is taken to theatre for an open cholecystectomy. At operation, Calots triangle is almost completely impossible to delineate.

A.Carcinoma of the head of the pancreas

B.Bile duct stricture

C.Mirizzi syndrome

D.Bile duct stones

E.Chronic cholecystitis

F.Peri hilar lymphadenopathy

G.Fitz - Hugh Curtis syndrome

A

Mirizzi syndrome

In Mirizzi syndrome the gallstone becomes impacted in Hartmans pouch. Episodes of recurrent inflammation occur and this causes compression of the bile duct. In severe cases this then progresses to fistulation. Surgery is extremely difficult as Calots triangle is often completely obliterated and the risks of causing injury to the CBD are high.

How well did you know this?
1
Not at all
2
3
4
5
Perfectly
359
Q

A 72 year old man undergoes a distal gastrectomy for carcinoma of the stomach. He presents with jaundice approximately 8 months post operatively. Ultrasound of the liver and bile ducts shows no focal liver lesion and normal calibre common bile duct with intra hepatic duct dilatation.

A.Carcinoma of the head of the pancreas

B.Bile duct stricture

C.Mirizzi syndrome

D.Bile duct stones

E.Chronic cholecystitis

F.Peri hilar lymphadenopathy

G.Fitz - Hugh Curtis syndrome

A

Peri hilar lymphadenopathy

Unfortunately metastatic disease is the most likely event. Peri hilar lymphadenopathy would be a common culprit.

How well did you know this?
1
Not at all
2
3
4
5
Perfectly
360
Q

A 70 year old lady presents with a number of skin lesions that she describes as unsightly. On examination she has a number of raised lesions with a greasy surface located over her trunk. Apart from having a greasy surface the lesions also seem to have scattered keratin plugs located within them.

A.Basal cell carcinoma

B.Dermatofibroma

C.Pilar cyst

D.Epidermoid cyst

E.Spitz naevus

F.Seborrhoeic keratosis

G.Atypical naevus

H.Capillary cavernous haemangioma

A

Seborrhoeic keratosis

Seborrhoeic keratosis may have a number of appearances. However, the scaly, thick, greasy surface with scattered keratin plugs makes this the most likely diagnosis.

How well did you know this?
1
Not at all
2
3
4
5
Perfectly
361
Q

A 21 year old lady presents with a nodule on the posterior aspect of her right calf. It has been present at the site for the past 6 months and occurred at the site of a previous insect bite. Although the nodule appears small, on palpation it appears to be nearly twice the size it appears on examination. The overlying skin is faintly pigmented.

A.Basal cell carcinoma

B.Dermatofibroma

C.Pilar cyst

D.Epidermoid cyst

E.Spitz naevus

F.Seborrhoeic keratosis

G.Atypical naevus

H.Capillary cavernous haemangioma

A

Dermatofibroma

Dermatofibromas may be pigmented and are often larger than they appear. They frequently occur at sites of previous trauma.

How well did you know this?
1
Not at all
2
3
4
5
Perfectly
362
Q

Which of the following ligaments contains the artery supplying the head of femur in children?

Transverse ligament

Ligamentum teres

Iliofemoral ligament

Ischiofemoral ligament

Pubofemoral ligament

A

Ligamentum teres

How well did you know this?
1
Not at all
2
3
4
5
Perfectly
363
Q

What is the reciprocal of absolute risk reduction?

Odds ratio

Number needed to treat

False positive

False negative

None of the above

A

In epidemiology, the absolute risk reduction, or risk difference is the decrease in risk of a given activity or treatment in relation to a control activity or treatment. It is the inverse of the number needed to treat.

How well did you know this?
1
Not at all
2
3
4
5
Perfectly
364
Q

A 15 year old boy undergoes an emergency splenectomy for trauma. He makes a full recovery and is discharged home. Eight weeks post operatively the general practitioner performs a full blood count with a blood film. Which of the following is most likely to be present?

Myofibroblasts

Howell-Jolly bodies

Multinucleate giant cells

Reed Sternberg Cells

None of the above

A

Post splenectomy blood film features:
Howell- Jolly bodies
Pappenheimer bodies
Target cells
Irregular contracted erythrocytes

As the filtration function is the spleen is no longer present Howell-Jolly bodies are found.

The loss of splenic tissue results in the inability to readily remove immature or abnormal red blood cells from the circulation. The red cell count does not alter significantly. However, cytoplasmic inclusions may be seen e.g. Howell-Jolly bodies.
In the first few days after splenectomy target cells, siderocytes and reticulocytes will appear in the circulation. Immediately following splenectomy a granulocytosis (mainly composed of neutrophils) is seen, this is replaced by a lymphocytosis and monocytosis over the following weeks.
The platelet count is usually increased and this may be persistent, oral antiplatelet agents may be needed in some patients.

How well did you know this?
1
Not at all
2
3
4
5
Perfectly
365
Q

A 43 year old woman is identified as being a carrier of a BRCA 1 mutation. Apart from breast cancer, which of the following malignancies is she at greatest risk of developing?

Colonic cancer

Ovarian cancer

Follicular carcinoma of the thyroid

Pituitary adenoma

Phaeochromocytoma

A

BRCA 1 mutation patients are 55% more likely to get ovarian cancer. Those with BRCA 2 are 25% more likely. The risk of developing other malignancies is slightly increased but not to the same extent, and not enough to justify screening.

How well did you know this?
1
Not at all
2
3
4
5
Perfectly
366
Q

A 53 year old man is due to undergo a splenectomy as a treatment for refractory haemolytic anaemia. The underlying pathological basis for haemolytic anaemia is thought to be a Type 2 hypersensitivity response. Which of the following mechanisms best describes this process

Deposition of immune complexes

Cell mediated immune response

IgE mediated response

Formation of autoantibodies against cell surface antigens

None of the above

A

Formation of autoantibodies against cell surface antigens

Mnemonic for the reactions and the mediators involved
ACID EGG-T
Type 1 Anaphylactic
Type 2 Cytotoxic
Type 3 Immune complex
Type 4 Delayed type

EGG T (mediators)

IgE
IgG
IgG
T cells

Type 2 hypersensitivity reactions (which includes haemolytic anaemia) are associated with formation of antibody against cell surface antigens.

Hypersensitivity reactions

The Gell and Coombs classification divides hypersensitivity reactions into 4 types

Type I Type II Type III Type IV
Description Anaphylactic Cytotoxic Immune complex Delayed type
Mediator IgE IgG, IgM IgG, Ig A, IgM T-cells
Antigen Exogenous Cell surface Soluble Tissues
Response time Minutes Hours Hours 2-3 days
Examples Asthma
Hay fever Autoimmune haemolytic anaemia
Pemphigus
Goodpasture’s Serum sickness
SLE
Aspergillosis Graft versus host disease
Contact dermatitis

How well did you know this?
1
Not at all
2
3
4
5
Perfectly
367
Q

What is the commonest type of fistula in ano?

Trans-sphincteric

Supra levator

Complex supra levator

Intersphincteric

Suprasphincteric

A

Intersphincteric fistulas are the commonest type and the external opening may be internal or external. These are the classical type of fistula and will have an internal opening near the anal verge and obey Goodsalls rule. Primary fistulotomy in this situation usually poses little risk to continence.

How well did you know this?
1
Not at all
2
3
4
5
Perfectly
368
Q

A 43 year old lady is donating her left kidney to her sister and the surgeons are harvesting the left kidney. Which of the following structures will lie most anteriorly at the hilum of the left kidney?

Left renal artery

Left renal vein

Left ureter

Left ovarian vein

Left ovarian artery

A

The renal veins lie most anteriorly, then artery and ureter lies posteriorly.

How well did you know this?
1
Not at all
2
3
4
5
Perfectly
369
Q

What is the sensory nerve supply to the angle of the jaw?

Maxillary branch of the trigeminal nerve

Mandibular branch of the trigeminal nerve

C3-C4

Greater auricular nerve (C2-C3)

Buccal branch of the facial nerve

A

The trigeminal nerve is the major sensory nerve to the face except over the angle of the jaw. The angle of the jaw is innervated by the greater auricular nerve.

How well did you know this?
1
Not at all
2
3
4
5
Perfectly
370
Q

A 63 year old man is undergoing a coronary artery bypass procedure. During the median sternotomy which structure would routinely require division?

Parietal pleura

Interclavicular ligament

Internal mammary artery

Brachiocephalic vein

Left vagus nerve

A

The interclavicular ligament lies at the upper end of a median sternotomy and is routinely divided to provide access. The pleural reflections are often encountered and should not be intentionally divided, if they are, then a chest drain will need to be inserted on the affected side as collections may then accumulate in the pleural cavity. Other structures encountered include the pectoralis major muscles, again if the incision is truly midline then these should not require formal division. The close relationship of the brachiocephalic vein should be borne in mind and it should be avoided, iatrogenic injury to this structure will result in considerable haemorrhage.

How well did you know this?
1
Not at all
2
3
4
5
Perfectly
371
Q

A 63 year old male is gardening when he trips and lands on a scythe. He sustains a deep laceration of his lateral thigh, it measures 3cm depth by 7cm length, it penetrates down to the bone, but no fracture is evident on imaging or examination. His co- morbidities include type II diabetes mellitus (diet controlled) and polymyalgia rheumatica (takes regular low dose prednisolone).

A.Immediate split thickness skin graft

B.Delayed split thickness skin graft

C.Primary closure

D.Delayed primary closure

E.Compression bandages

F.Myocutaneous flap

G.Random free flap

A

The correct answer is Delayed primary closure

Wounds which are contaminated or have the potential to become so are unsafe for immediate primary closure. The combination of diabetes and steroids makes wound complications more likely. Despite his high risk a primary skin graft or flap is unlikely to be a safer option. Either may be used at a later date in the event that delayed primary closure is unsuccessful.

How well did you know this?
1
Not at all
2
3
4
5
Perfectly
372
Q

A 71 year old lady trips over and falls landing on her left shin. She sustains a large pretibial laceration of her leg.

A.Immediate split thickness skin graft

B.Delayed split thickness skin graft

C.Primary closure

D.Delayed primary closure

E.Compression bandages

F.Myocutaneous flap

G.Random free flap

A

Delayed split thickness skin graft

Pretibial lacerations do not heal well. Simple apposition of skin edges almost always fails due to poor quality dermal tissues and underlying haematoma. Debridement of devitalised tissues prior to grafting usually gives the best results.

How well did you know this?
1
Not at all
2
3
4
5
Perfectly
373
Q

A 73 year old lady presents with an ulcer overlying her medial malleolus. It is painless and has been present for 4 months. She has oedema of the lower limbs and her ABPI measures 0.9.

A.Immediate split thickness skin graft

B.Delayed split thickness skin graft

C.Primary closure

D.Delayed primary closure

E.Compression bandages

F.Myocutaneous flap

G.Random free flap

A

Compression bandages

This is likely to be a venous leg ulcer. These are typically managed using compression bandages. Contra indications to this technique include peripheral vascular disease (not present here).

How well did you know this?
1
Not at all
2
3
4
5
Perfectly
374
Q

Which of the following structures separates the subclavian artery from the subclavian vein?

Scalenus anterior

Scalenus medius

Sternocleidomastoid

Pectoralis major

Pectoralis minor

A

The artery and vein are separated by scalenus anterior. This muscle runs from the transverse processes of C3,4,5 and 6 to insert onto the scalene tubercle of the first rib.

How well did you know this?
1
Not at all
2
3
4
5
Perfectly
375
Q

A 56 year old lady is due to undergo a left hemicolectomy for carcinoma of the splenic flexure. The surgeons decide to perform a high ligation of the inferior mesenteric vein. Into which of the following does this structure usually drain?

Portal vein

Inferior vena cava

Left renal vein

Left iliac vein

Splenic vein

A

Beware of ureteric injury in colonic surgery.

The inferior mesenteric vein drains into the splenic vein, this point of union lies close to the duodenum and this surgical maneouvre is a recognised cause of ileus.

How well did you know this?
1
Not at all
2
3
4
5
Perfectly
376
Q

Which nerve innervates the 1st and 2nd muscles of the thenar eminence?
A. Radial nerve
B. Ulnar nerve
C. Median nerve
D. Musculocutaneous nerve
E. Axillary nerve

A

C The median nerve innervates the 1st and 2nd muscles of the thenar eminence, including the abductor pollicis brevis and opponens pollicis.

How well did you know this?
1
Not at all
2
3
4
5
Perfectly
377
Q

Which part of the sternocleidomastoid muscle is attached to the upper manubrium sterni?
A. Muscular head
B. Mastoid process
C. Lateral area of the superior nuchal line
D. Spinal part of accessory nerve
E. Rounded tendon

A

E The rounded tendon of the sternocleidomastoid muscle is attached to the upper manubrium sterni.

How well did you know this?
1
Not at all
2
3
4
5
Perfectly
378
Q

Which nervous discharge causes erection?
A. Sympathetic discharge
B. Parasympathetic discharge
C. Somatic nerves
D. Autonomic discharge
E. Pelvic plexus

A

B Parasympathetic discharge causes erection, while sympathetic discharge causes ejaculation and detumescence.

How well did you know this?
1
Not at all
2
3
4
5
Perfectly
379
Q

What is the extensor retinaculum?
A. A tendon in the wrist
B. A thickening of the deep fascia across the back of the wrist
C. A ligament connecting the radius and ulnar styloid
D. A branch of the radial nerve
E. A branch of the ulnar nerve

A

B The extensor retinaculum is a thickening of the deep fascia that stretches across the back of the wrist and holds the long extensor tendons in position.

How well did you know this?
1
Not at all
2
3
4
5
Perfectly
380
Q

What is the arterial supply to the adrenal glands?
A. From inferior phrenic artery
B. From aorta
C. From renal arteries
D. From celiac artery
E. From superior mesenteric artery

A

A The arterial supply to the adrenal glands includes superior adrenal arteries from the inferior phrenic artery, middle adrenal arteries from the aorta, and inferior adrenal arteries from the renal arteries.

How well did you know this?
1
Not at all
2
3
4
5
Perfectly
381
Q

The knee joint is supplied by which nerve divisions?
A. Femoral, tibial and common peroneal divisions of the sciatic
B. Obturator and sciatic divisions
C. Femoral and obturator divisions
D. Tibial and common peroneal divisions of the sciatic
E. Femoral and tibial divisions of the sciatic

A

A The knee joint is supplied by the femoral, tibial and common peroneal divisions of the sciatic and by a branch from the obturator nerve.

How well did you know this?
1
Not at all
2
3
4
5
Perfectly
382
Q

A 43 year old lady is due to undergo an axillary node clearance as part of treatment for carcinoma of the breast. Which of the following fascial layers will be divided during the surgical approach to the axilla?

Sibsons fascia

Pre tracheal fascia

Waldayers fascia

Clavipectoral fascia

None of the above

A

The clavipectoral fascia is situated under the clavicular portion of pectoralis major. It protects both the axillary vessels and nodes. During an axillary node clearance for breast cancer the clavipectoral fascia is incised and this allows access to the nodal stations. The nodal stations are; level 1 nodes inferior to pectoralis minor, level 2 lie behind it and level 3 above it. During a Patey Mastectomy surgeons divide pectoralis minor to gain access to level 3 nodes. The use of sentinel node biopsy (and stronger assistants!) have made this procedure far less common.

How well did you know this?
1
Not at all
2
3
4
5
Perfectly
383
Q

What are the boundaries of the ‘safe triangle’ for chest drain insertion?

Bounded by trapezius, latissimus dorsi, and laterally by the vertebral border of the scapula

Bounded by latissimus dorsi, pectoralis major, line superior to the nipple and apex at the axilla

Bounded by latissimus dorsi, serratus anterior, line superior to the nipple and apex at the axilla

Bounded by trapezius, deltoid, rhomboid major and teres minor

Bounded by trapezius, deltoid and latissimus dorsi

A

Bounded by latissimus dorsi, pectoralis major, line superior to the nipple and apex at the axilla

How well did you know this?
1
Not at all
2
3
4
5
Perfectly
384
Q

Which of the following is not an oncogene?

ras

myc

sis

Ki 67

erb-B

A

Ki 67 is a nuclear proliferation marker (used in immunohistochemistry). Although, Ki67 positivity is a marker of malignancy, it is not itself, an oncogene.

Among the options you provided, Ki 67 is not an oncogene 123. Ki 67 is a protein that is expressed in proliferating cells and is used as a marker of cell proliferation 13. It is not a gene that has the potential to cause cancer 13.
On the other hand, ras, myc, sis, and erb-B are all oncogenes 14. Ras is a family of genes that encode proteins involved in cell signaling pathways that regulate cell growth and division 14. Mutations in ras genes can cause them to become permanently activated, leading to uncontrolled cell growth and division, and ultimately cancer 14. Myc is a transcription factor that regulates the expression of genes involved in cell growth and division 14. Overexpression of myc can lead to uncontrolled cell growth and division, and ultimately cancer 14. Sis is a gene that encodes a protein called platelet-derived growth factor (PDGF) 14. PDGF is involved in cell growth and division, and overexpression of sis can lead to uncontrolled cell growth and division, and ultimately cancer 14. Erb-B is a gene that encodes a receptor protein involved in cell signaling pathways that regulate cell growth and division 14. Mutations in erb-B can cause the receptor to become permanently activated, leading to uncontrolled cell growth and division, and ultimately cancer 14.

How well did you know this?
1
Not at all
2
3
4
5
Perfectly
385
Q

An otherwise fit 30 year old male donates 500ml of blood. Which of the processes outlined below is most likely to occur?

Oliguria

Activation of the renin angiotensin system

Sweating

Fall in mean arterial pressure

Tachypnoea

A

The loss of 500ml (assuming a 70 Kg male) will usually be sufficient to activate the renin angiotensin system. It is unlikely that it would cause any other physiological disturbance.

How well did you know this?
1
Not at all
2
3
4
5
Perfectly
386
Q

The vertebral artery traverses all of the following except?

Transverse process of C6

Transverse process of the axis

Vertebral canal

Foramen magnum

Intervertebral foramen

A

The vertebral artery passes through the foramina which are located in the transverse processes of the cervical vertebra, it does not traverse the intervertebral foramen.

The vertebral artery is a major artery in the neck that supplies blood to the brainstem and cerebellum. It arises from the subclavian artery and enters the transverse foramen of the cervical vertebrae, except for the seventh cervical vertebra (C7) 1. After passing through the transverse foramina of the cervical vertebrae, the vertebral artery ascends posteromedially towards the skull 2. It passes through the foramen magnum, which is a large opening in the occipital bone of the skull that allows the spinal cord to connect with the brain 2. The vertebral artery also passes through the subarachnoid space, which is the space between the arachnoid mater and the pia mater that surrounds the brain and spinal cord 3. However, the vertebral artery does not traverse the intervertebral foramen 45. The intervertebral foramen is the opening between adjacent vertebrae through which spinal nerves exit the spinal cord 6

How well did you know this?
1
Not at all
2
3
4
5
Perfectly
387
Q

A 25 year old man is injured in a road traffic accident. His right tibia is fractured and is managed by fasciotomies and application of an external fixator. Over the next 48 hours his serum creatinine rises and urine is sent for microscopy, muddy brown casts are identified. What is the most likely underlying diagnosis?

Acute interstitial nephritis

Acute tubular necrosis

Glomerulonephritis

IgA Nephropathy

Thin basement membrane disease

A

This patient is likely to have had compartment syndrome (tibial fracture + fasciotomies) which may produce myoglobinuria. The presence of worsening renal function, together with muddy brown casts is strongly suggestive of acute tubular necrosis. Acute interstitial nephritis usually arises from drug toxicity and does not usually produce urinary muddy brown casts. Thin basement membrane disease is an autosomal dominant condition that causes persistent microscopic haematuria, but not worsening renal function.

How well did you know this?
1
Not at all
2
3
4
5
Perfectly
388
Q

During an Ivor Lewis Oesophagectomy for carcinoma of the lower third of the oesophagus which structure is divided to allow mobilisation of the oesophagus?

Vagus nerve

Azygos vein

Right inferior lobar bronchus

Phrenic nerve

Pericardiophrenic artery

A

The azygos vein is routinely divided during an oesophagectomy to allow mobilisation. It arches anteriorly to insert into the SVC on the right hand side.

How well did you know this?
1
Not at all
2
3
4
5
Perfectly
389
Q

A 32 year old man is stabbed in the neck and the inferior trunk of his brachial plexus is injured. Which of the modalities listed below is least likely to be affected?

Initiating abduction of the shoulder

Abduction of the fingers

Flexion of the little finger

Sensation on the palmar aspect of the little finger

Gripping a screwdriver

A

Inferior trunk of brachial plexus.
C8 and T1 roots
Contributes to ulnar nerve and part of median nerve
The inferior trunk of the brachial plexus is rarely injured. Nerve roots C8 and T1 are the main contributors to this trunk. Therefore an injury to this site will most consistently affect the ulnar nerve. The inferior trunk also contributes to the median nerve by way of the medial cord and therefore some impairment of grip is almost inevitable.

Brachial plexus

The brachial plexus extends from the neck to the axilla. It is formed by the ventral rami of the fifth to the eighth cervical nerves with the ascending part of the first thoracic nerve.

Location of the plexus
The ventral rami which form the plexus enter the lower part of the posterior triangle of the neck in series with the ventral rami of the cervical plexus. The second part of the subclavian artery lies immediately anterior to the lower two rami. The upper three rami intermingle and pass inferolaterally towards the axilla and subclavian artery. They are enclosed within an extension of the prevertebral fascia. In the neck the plexus lies deep to platysma, the supraclavicular nerves, inferior belly of omohyoid and the transverse cervical artery. It then passes deep to the clavicle and the suprascapular vessels, to enter the axilla, and thence surround the second part of the axillary artery

Composition of the plexus
Ventral rami, the roots of the plexus, lie between scalenus medius and anterior.

As they enter the posterior triangle, the upper two (C5,6) and lower two (C8, T1) roots of the plexus unite to form the upper and lower trunks of the plexus respectively. Meanwhile, C7 continues as the middle trunk. The lower trunk may groove the superior surface of the first rib posterior to the subclavian artery, and the root from the first ventral ramus is always in contact with it.

Each trunk divides into ventral and dorsal divisions which are destined to supply the anterior (flexor) and posterior (extensor) parts of the upper limb.

The cords of the plexus are formed in the axilla. The dorsal divisions unite to form the posterior cord (C5-8). The ventral divisions of the upper and middle trunks unite to form the lateral cord (C5-7), while the ventral divisions of the lower trunk continues as the medial cord (C8-T1). The cords are named according to their relationship to the axillary artery. Each cord terminates by dividing into two main branches at the beginning of the third part of the artery.

Sympathetic communications
The fifth and sixth cervical ventral rami receive grey rami communicantes from the middle cervical ganglion, while the two or more grey rami communicantes pass from the inferior cervical ganglion to the seventh and eighth cervical ventral rami. The first thoracic ventral ramus receives its grey ramus from the cervicothoracic ganglion. Its for this reason that inferior plexus injury can be complicated by a Horners syndrome.

Summary
Origin Anterior rami of C5 to T1
Sections of the plexus
Roots, trunks, divisions, cords, branches
Mnemonic:Real Teenagers Drink Cold Beer
Roots
Located in the posterior triangle
Pass between scalenus anterior and medius
Trunks
Located posterior to middle third of clavicle
Upper and middle trunks related superiorly to the subclavian artery
Lower trunk passes over 1st rib posterior to the subclavian artery
Divisions Apex of axilla
Cords Related to axillary artery

How well did you know this?
1
Not at all
2
3
4
5
Perfectly
390
Q

Brown tumours of bone are associated with which of the following?

Hyperthyroidism

Hypothyroidism

Hyperparathyroidism

Hypoparathyroidism

Osteopetrosis

A

Brown tumors are tumors of bone that arise in settings of excess osteoclast activity, such as hyperparathyroidism, and consist of fibrous tissue, woven bone and supporting vasculature, but no matrix. They are radiolucent on x-ray. The osteoclasts consume the trabecular bone that osteoblasts lay down and this front of reparative bone deposition followed by additional resorption can expand beyond the usual shape of the bone, involving the periosteum thus causing bone pain. They appear brown because haemosiderin is deposited at the site.

Primary hyperparathyroidism

In exams, primary hyperparathyroidism is stereotypically seen in elderly females with an unquenchable thirst and an inappropriately normal or raised parathyroid hormone level. It is most commonly due to a solitary adenoma

Causes of primary hyperparathyroidism
80%: solitary adenoma
15%: hyperplasia
4%: multiple adenoma
1%: carcinoma

Features - ‘bones, stones, abdominal groans and psychic moans’
Polydipsia, polyuria
Peptic ulceration/constipation/pancreatitis
Bone pain/fracture
Renal stones
Depression
Hypertension

Associations
Hypertension
Multiple endocrine neoplasia: MEN I and II

Investigations
Raised calcium, low phosphate
PTH may be raised or normal
Technetium-MIBI subtraction scan

Treatment
Parathyroidectomy, if imaging suggests target gland then a focused approach may be used

How well did you know this?
1
Not at all
2
3
4
5
Perfectly
391
Q

Which of the following muscles does not attach to the radius?

Pronator quadratus

Biceps

Brachioradialis

Supinator

Brachialis

A

The brachialis muscle inserts into the ulna. The other muscles are all inserted onto the radius.

Radius

The radius is one of the two long forearm bones that extends from the lateral side of the elbow to the thumb side of the wrist. It has two expanded ends, of which the distal end is the larger. Key points relating to its topography and relations are outlined below;

Upper end
Articular cartilage- covers medial > lateral side
Articulates with radial notch of the ulna by the annular ligament
Muscle attachment- biceps brachii at the tuberosity

Shaft
Muscle attachment
Upper third of the body Supinator
Flexor digitorum superficialis
Flexor pollicis longus
Middle third of the body Pronator teres
Lower quarter of the body Pronator quadratus
Tendon of supinator longus

Lower end
Quadrilateral
Anterior surface- capsule of wrist joint
Medial surface- head of ulna
Lateral surface- ends in the styloid process
Posterior surface: 3 grooves containing:
1. Tendons of extensor carpi radialis longus and brevis
2. Tendon of extensor pollicis longus
3. Tendon of extensor indicis

How well did you know this?
1
Not at all
2
3
4
5
Perfectly
392
Q

A 27-year-old male presents to urology for investigation of pyelonephritis. He reports malaise, pyrexia, lymphadenopathy and a maculopapular rash. The Monospot test is negative. Given a history of recent high-risk sexual behaviour you are asked to exclude a HIV seroconversion illness. What is the most appropriate investigation?

Antibodies to HIV-2

gp120 polymerase chain reaction

p24 antigen test

CCR5 polymerase chain reaction

Antibodies to HIV-1

A

p24 antigen test

HIV testing

HIV seroconversion is symptomatic in 60-80% of patients and typically presents as a glandular fever type illness. Increased symptomatic severity is associated with poorer long term prognosis. It typically occurs 3-12 weeks after infection

Features
sore throat
lymphadenopathy
malaise, myalgia, arthralgia
diarrhoea
maculopapular rash
mouth ulcers
rarely meningoencephalitis

Diagnosis
antibodies to HIV may not be present
HIV PCR and p24 antigen tests can confirm diagnosis

HIV antibody test
most common and accurate test
usually consists of both a screening ELISA (Enzyme Linked Immuno-Sorbent Assay) test and a confirmatory Western Blot Assay
most people develop antibodies to HIV at 4-6 weeks but 99% do by 3 months

p24 antigen test
usually positive from about 1 week to 3 - 4 weeks after infection with HIV
sometimes used as an additional screening test in blood banks

How well did you know this?
1
Not at all
2
3
4
5
Perfectly
393
Q

An 83 year old lady is admitted with a fractured neck of femur and is managed with surgical fixation. On the day following surgery, she is noted to have a dense hemiparesis. Several days later, an MRI scan of the brain is performed which demonstrates a 3cm area of devitalised tissue within the right frontal lobe. Which of the pathological processes described below is most likely to account for these appearances?

Coagulative necrosis

Colliquative necrosis

Caseous necrosis

Fibrinoid necrosis

Apoptosis

A

Death of CNS tissue through vascular compromise tends to result in colliquative necrosis as the cells have less formal supporting stroma.

Cell death

Cells can die via two mechanisms; necrosis and apoptosis. These are outlined below:

Necrosis
Necrosis is characterised by bioenergetics failure. Loss of tissue perfusion results in hypoxia and an inability to generate ATP. The integrity of the cellular membrane is lost and the loss of ATP results in loss of energy dependent cellular transport mechanisms. There is an influx of water, ionic instability and cellular lysis. The release of intracellular contents may stimulate an inflammatory response. Several types of necrosis are recognised; coagulative, colliquative, caseous, gangrene, fibrinoid and fat necrosis. The type of tissue and the underlying cause determine the predominant necrosis pattern.

Coagulative necrosis
The commonest type, occurs in most organs
Tissue is initially firm, later becomes soft as tissue is digested by macrophages
In the early phases the histological appearances may demonstrate little change
In later stages cellular outlines are seen with loss of intracellular detail

Colliquative necrosis
Occurs in tissues with no supporting stroma
Dominant necrosis pattern in the CNS
Necrotic site may eventually become encysted

Caseous necrosis
No definable structure seen in the necrotic tissue
Amorphous eosinophilic tissue may be seen histologically
Classically seen in tuberculosis

Gangrene
Necrosis with putrefaction of tissue
May complicate ischaemia
Haemoglobin degenerates and results in the deposition of iron sulphide (which is why the tissue is black)
Both wet and dry gangrene may occur, in wet gangrene there is often a liquefactive component and super-added infection (which usually smells!)

Fibrinoid necrosis
Classically seen in arterioles in patients with hypertension (malignant type)
Necrosis of the smooth muscle wall occurs and plasma may extravasate into the media with fibrin deposition

Fat necrosis
Direct trauma to fat can result in rupture of adipocytes
Lipids incite a local inflammatory reaction
Inflammatory cells phagocytose the lipid with eventual fibrosis

Apoptosis
Also known as programmed cell death
Energy dependent pathways are activated via a number of intracellular signalling mechanisms
It is the result of the activation of caspases triggered by the bcl-2 family or the binding of the FAS ligand to its receptor
DNA fragments, mitochondrial function ceases, nuclear and cellular shrinkage occurs
Phagocytosis of the cell does not occur, instead the cell degenerates into apoptotic bodies

How well did you know this?
1
Not at all
2
3
4
5
Perfectly
394
Q

Into which of these veins does the middle thyroid vein drain?

Vertebral

External jugular

Internal jugular

Subclavian

Anterior jugular

A

It drains to the internal jugular vein. Which is one of the reasons why it bleeds so copiously if a ligature slips.

Thyroid gland

Right and left lobes connected by isthmus
Surrounded by sheath from pretracheal layer of deep fascia
Apex: Lamina of thyroid cartilage
Base: 4th-5th tracheal ring
Pyramidal lobe: from isthmus
May be attached to foramen caecum at the base of the tongue

Relations
Anteromedially
Sternothyroid
Superior belly of omohyoid
Sternohyoid
Anterior aspect of sternocleidomastoid
Posterolaterally Carotid sheath
Medially
Larynx
Trachea
Pharynx
Oesophagus
Cricothyroid muscle
External laryngeal nerve (near superior thyroid artery)
Recurrent laryngeal nerve (near inferior thyroid artery)
Posterior
Parathyroid glands
Anastomosis of superior and inferior thyroid arteries
Isthmus
Anteriorly: Sternothyroids, sternohyoids, anterior jugular veins
Posteriorly: 2nd, 3rd, 4th tracheal rings (attached via Ligament of Berry)

Blood Supply
Arterial
Superior thyroid artery (1st branch of external carotid)
Inferior thyroid artery (from thyrocervical trunk)
Thyroidea ima (in 10% of population -from brachiocephalic artery or aorta)
Venous
Superior and middle thyroid veins - into the IJV
Inferior thyroid vein - into the brachiocephalic veins

How well did you know this?
1
Not at all
2
3
4
5
Perfectly
395
Q

A 28 year old man falls and hits his head against a wall. There is a brief loss of consciousness. When assessed in accident and emergency he is alert and orientated with a GCS of 15, imaging shows no fracture of the skull. What is his risk of having an intracranial haematoma requiring removal?

1 in 6000

1 in 40

1 in 4

1 in 50,000

1 in 120

A

Head injury and haematoma

Risk of haematoma (requiring removal) in adults attending accident and emergency units following head injury.
Injury Conscious level Risk of haematoma requiring removal
Concussion, no skull fracture Orientated 1 in 6000
Concussion, no skull fracture Not orientated 1 in 120
Skull fracture Orientated 1 in 32
Skull fracture Not orientated 1 in 4

How well did you know this?
1
Not at all
2
3
4
5
Perfectly
396
Q

A 22 year old lady presents with an episode of renal colic and following investigation is suspected of suffering from MEN IIa. Which of the following abnormalities of the parathyroid glands are most often found in this condition?

Hypertrophy

Hyperplasia

Adenoma

Carcinoma

Metaplasia

A

MEN IIa
Medullary thyroid cancer
Hyperparathyroidism (usually hyperplasia)
Phaeochromocytoma
In MEN IIa the commonest lesion is medullary thyroid cancer, with regards to the parathyroid glands the most common lesion is hyperplasia. In MEN I a parathyroid adenoma is the most common lesion.

Multiple Endocrine Neoplasia

Multiple endocrine neoplasia (MEN) is inherited as an autosomal dominant disorder.

The table below summarises the three main types of MEN:

MEN type I MEN type IIa MEN type IIb
Mnemonic ‘three P’s’:

Parathyroid (95%): Parathyroid adenoma
Pituitary (70%): Prolactinoma/ACTH/Growth Hormone secreting adenoma
Pancreas (50%): Islet cell tumours/Zollinger Ellison syndrome

also: Adrenal (adenoma) and thyroid (adenoma) Phaeochromocytoma
Medullary thyroid cancer (70%)
Hyperparathyroidism (60%) Same as MEN IIa with addition of:
Marfanoid body habitus
Mucosal neuromas
MENIN gene (chromosome 11)

Most common presentation = hypercalcaemia RET oncogene (chromosome 10) RET oncogene (chromosome 10)

How well did you know this?
1
Not at all
2
3
4
5
Perfectly
397
Q

Which of the following is the first vessel to branch from the external carotid artery?

Superior thyroid artery

Inferior thyroid artery

Lingual artery

Facial artery

Occipital artery

A

Superior Thyroid Artery
Mnemonic
(Order in which they branch off)Some (sup thyroid)Attendings (Ascending Pharyngeal)Like (Lingual)Freaking (Facial)Out (Occipital)Potential (Post auricular)Medical (Maxillary)Students (Sup temporal) The first branch of the external carotid artery is the superior thyroid artery. The inferior thyroid artery is derived from the thyrocervical trunk.

External carotid artery

The external carotid commences immediately lateral to the pharyngeal side wall. It ascends and lies anterior to the internal carotid and posterior to the posterior belly of digastric and stylohyoid. More inferiorly it is covered by sternocleidomastoid, passed by hypoglossal nerves, lingual and facial veins.
It then pierces the fascia of the parotid gland finally dividing into its terminal branches within the gland itself.

Surface marking of the carotid
This is an imaginary line drawn from the bifurcation of the common carotid passing behind the angle of the jaw to a point immediately anterior to the tragus of the ear.

Branches of the external carotid artery
It has six main branches, three in front, two behind and one deep.
Three in front Superior thyroid
Lingual
Facial
Two behind Occipital
Posterior auricular
Deep Ascending pharyngeal

How well did you know this?
1
Not at all
2
3
4
5
Perfectly
398
Q

A 52 year old male presents with discomfort in the fingers of his left hand. On examination, the ring and little fingers of his left hand are flexed and unable to extend completely. He is able to make a fist with the hand. Palpation reveals thickened nodules on the medial half of the palm. What is the most likely diagnosis?

de Quervain’s tenosynovitis

Tendon sheath infection

Dupuytren’s contracture

Ganglion

Heberden’s nodes

A

Discomfort of the hand is not uncommon in Dupuytren’s contracture, true pain is unusual. The disease most commonly affects the ring and little fingers.

Hand diseases

Dupuytrens contracture
Fixed flexion contracture of the hand where the fingers bend towards the palm and cannot be fully extended.
Caused by underlying contractures of the palmar aponeurosis . The ring finger and little finger are the fingers most commonly affected. The middle finger may be affected in advanced cases, but the index finger and the thumb are nearly always spared.
Progresses slowly and is usually painless. In patients with this condition, the tissues under the skin on the palm of the hand thicken and shorten so that the tendons connected to the fingers cannot move freely. The palmar aponeurosis becomes hyperplastic and undergoes contracture.
Commonest in males over 40 years of age.
Association with liver cirrhosis and alcoholism. However, many cases are idiopathic.
Treatment is surgical and involves fasciectomy. However, the condition may recur and many surgical therapies are associated with risk of neurovascular damage to the digital nerves and arteries.

Carpal tunnel syndrome
Idiopathic median neuropathy at the carpal tunnel.
Characterised by altered sensation of the lateral 3 fingers.
The condition is commoner in females and is associated with other connective tissue disorders such as rheumatoid disease. It may also occur following trauma to the distal radius.
Symptoms occur mainly at night in early stages of the condition.
Examination may demonstrate wasting of the muscles of the thenar eminence and symptoms may be reproduced by Tinels test (compression of the contents of the carpal tunnel).
Formal diagnosis is usually made by electrophysiological studies.
Treatment is by surgical decompression of the carpal tunnel, a procedure achieved by division of the flexor retinaculum. Non - surgical options include splinting and bracing.

Miscellaneous hand lumps
Osler’s nodes Osler’s nodes are painful, red, raised lesions found on the hands and feet. They are the result of the deposition of immune complexes.
Bouchard’s nodes Hard, bony outgrowths or gelatinous cysts on the proximal interphalangeal joints (the middle joints of fingers or toes.) They are a sign of osteoarthritis, and are caused by formation of calcific spurs of the articular cartilage.
Heberden’s nodes Typically develop in middle age, beginning either with a chronic swelling of the affected joints or the sudden painful onset of redness, numbness, and loss of manual dexterity. This initial inflammation and pain eventually subsides, and the patient is left with a permanent bony outgrowth that often skews the fingertip sideways. It typically affects the DIP joint.
Ganglion Swelling in association with a tendon sheath commonly near a joint. They are common lesions in the wrist and hand. Usually they are asymptomatic and cause little in the way of functional compromise. They are fluid filled although the fluid is similar to synovial fluid it is slightly more viscous. When the cysts are troublesome they may be excised.

How well did you know this?
1
Not at all
2
3
4
5
Perfectly
399
Q

What are the most likely effects of the release of vasopressin from the pituitary?

Vasoconstriction of the afferent glomerular arteriole

Increased permeability of the mesangial cells to glucose

Reduced permeability of the inner medullary portion of the collecting duct to urea

Increased secretion of aldosterone from the macula densa

Increased water permeability of the distal tubule cells of the kidney

A

ADH (vasopressin) results in the insertion of aquaporin channels in apical membrane of the distal tubule and collecting ducts.

Renal Physiology

Overview
Each nephron is supplied with blood from an afferent arteriole that opens onto the glomerular capillary bed.
Blood then flows to an efferent arteriole, supplying the peritubular capillaries and medullary vasa recta.
The kidney receives up to 25% of resting cardiac output.

Control of blood flow
The kidney is able to autoregulate its blood flow between systolic pressures of 80- 180mmHg so there is little variation in renal blood flow.
This is achieved by myogenic control of arteriolar tone, both sympathetic input and hormonal signals (e.g. renin) are responsible.

Glomerular structure and function
Blood inside the glomerulus has considerable hydrostatic pressure.
The basement membrane has pores that will allow free diffusion of smaller solutes, larger negatively charged molecules such as albumin are unable to cross.
The glomerular filtration rate (GFR) is equal to the concentration of a solute in the urine, times the volume of urine produced per minute, divided by the plasma concentration (assuming that the solute is freely diffused e.g. inulin).
In clinical practice creatinine is used because it is subjected to very little proximal tubular secretion.
Although subject to variability, the typical GFR is 125ml per minute.
Glomerular filtration rate = Total volume of plasma per unit time leaving the capillaries and entering the bowman’s capsule
Renal clearance = volume plasma from which a substance is removed per minute by the kidneys

Substances used to measure GFR have the following features:
1. Inert
2. Free filtration from the plasma at the glomerulus (not protein bound)
3. Not absorbed or secreted at the tubules
4. Plasma concentration constant during urine collection

Examples: inulin, creatinine

plasma concentration (mmol/l)

The clearance of a substance is dependent not only on its diffusivity across the basement membrane but also subsequent tubular secretion and / or reabsorption.
So glucose which is freely filtered across the basement membrane is usually reabsorbed from tubules giving a clearance of zero.

Tubular function
Reabsorption and secretion of substances occurs in the tubules.
In the proximal tubule substrates such as glucose, amino acids and phosphate are co-transported with sodium across the semi permeable membrane.
Glucose is reabsorbed by both active and passive processes. The former uses sodium–glucose co-transporters (SGLTs) in the proximal convoluted tubule. The latter achieved through facilitated and passive process through GLUT transporters.
Up to two thirds of filtered water is reabsorbed in the proximal tubules.
This will lead to increase in urea concentration in the distal tubule allowing for its increased diffusion.
Substances to be secreted into the tubules are taken up from the peritubular blood by tubular cells.
Solutes such as paraaminohippuric acid are cleared with a single passage through the kidneys and this is why it is used to measure renal plasma flow. Ions such as calcium and phosphate will have a tubular reabsorption that is influenced by plasma PTH levels.
Potassium may be both secreted and re-absorbed and is co-exchanged with sodium.

Loop of Henle
Approximately 60 litres of water containing 9000mmol sodium enters the descending limb of the loop of Henle in 24 hours.
Loops from the juxtamedullary nephrons run deep into the medulla.
The osmolarity of fluid changes and is greatest at the tip of the papilla.
The thick ascending limb is impermeable to water, but highly permeable to sodium and chloride ions.
This loss means that at the beginning of the thick ascending limb the fluid is hypo osmotic compared with adjacent interstitial fluid.
In the thick ascending limb the reabsorption of sodium and chloride ions occurs by both facilitated and passive diffusion pathways.
The loops of Henle are co-located with vasa recta, these will have similar solute compositions to the surrounding extracellular fluid so preventing the diffusion and subsequent removal of this hypertonic fluid.
The energy dependent reabsorption of sodium and chloride in the thick ascending limb helps to maintain this osmotic gradient.

How well did you know this?
1
Not at all
2
3
4
5
Perfectly
400
Q

A 42 year old man is admitted to surgery with acute appendicitis. He is known to have hypertension, psoriatic arthropathy and polymyalgia rheumatica. His medical therapy includes:
Paracetamol 1g qds
Codeine phosphate 30mg qds
Bendrofluazide 2.5 mg od
Ramipril 10mg od
Methotrexate 7.5mg once a week
Prednisolone 5mg od
You are called by the core surgical trainee to assess this man as he has become delirious and hypotensive 2 hours after surgery. His blood results reveal:

Na+ 132 mmol/l
K+ 5.2 mmol/l
Urea 10 mmol/l
Creatinine 111 µmol/l
Glucose 3.5
CRP 158

Hb 10.2 g/dl
Platelets 156 * 109/l
WBC 14 * 109/l

What is the most likely diagnosis?

Septic shock secondary to appendicitis

Neutropenic sepsis

Phaeochromocytoma

Perforated bowel

Addisonian crisis

A

Features of an addisonian crisis:
Hyponatraemia
Hyperkalaemia
Hypoglycaemia
This man is on steroids for polymyalgia rheumatica. Surgery can precipitate acute adrenal deficiency. The diagnosis is further confirmed by the blood results of hyponatraemia, hyperkalaemia and hypoglycaemia. This patient urgently needs hydrocortisone.

Addisonian crisis

Causes
Sepsis or surgery causing an acute exacerbation of chronic insufficiency (Addison’s, Hypopituitarism)
Adrenal haemorrhage eg Waterhouse-Friderichsen syndrome (fulminant meningococcemia)
Steroid withdrawal

Management
Hydrocortisone 100 mg im or iv
1 litre normal saline infused over 30-60 mins or with dextrose if hypoglycaemic
Continue hydrocortisone 6 hourly until the patient is stable. No fludrocortisone is required because high cortisol exerts weak mineralocorticoid action
Oral replacement may begin after 24 hours and be reduced to maintenance over 3-4 days

How well did you know this?
1
Not at all
2
3
4
5
Perfectly
401
Q

Which of the following conditions is least likely to exhibit the Koebner phenomenon?

Vitiligo

Molluscum contagiosum

Lichen planus

Psoriasis

Lupus vulgaris

A

Lupus vulgaris is not associated with the Koebner phenomenon.

Koebner phenomenon

The Koebner phenomenon describes skin lesions which appear at the site of injury. It is seen in:
Psoriasis
Vitiligo
Warts
Lichen planus
Lichen sclerosus
Molluscum contagiosum

How well did you know this?
1
Not at all
2
3
4
5
Perfectly
402
Q

Which of the following statements relating to the posterior cerebral artery is false?

It supplies the visual cortex

It is closely related to the 3rd cranial nerve

It is a branch of the basilar artery

It is connected to the circle of Willis via the superior cerebellar artery

When occluded may result in contralateral loss of field of vision

A

The posterior cerebral arteries are formed at the bifurcation of the basilar artery and is connected to the circle of Willis via the posterior communicating artery.

The posterior cerebral arteries supply the occipital lobe and part of the temporal lobe.

Circle of Willis

The two internal carotid arteries and two vertebral arteries form an anastomosis known as the Circle of Willis on the inferior surface of the brain. Each half of the circle is formed by:
1. Anterior communicating artery
2. Anterior cerebral artery
3. Internal carotid artery
4. Posterior communicating artery
5. Posterior cerebral arteries and the termination of the basilar artery

The circle and its branches supply; the corpus striatum, internal capsule, diencephalon and midbrain.

Vertebral arteries
Enter the cranial cavity via foramen magnum
Lie in the subarachnoid space
Ascend on anterior surface of medulla oblongata
Unite to form the basilar artery at the base of the pons

Branches:
Posterior spinal artery
Anterior spinal artery
Posterior inferior cerebellar artery

Basilar artery
Branches:
Anterior inferior cerebellar artery
Labyrinthine artery
Pontine arteries
Superior cerebellar artery
Posterior cerebral artery (at the point where it bifurcates)

Internal carotid arteries
Branches:
Posterior communicating artery
Anterior cerebral artery
Middle cerebral artery
Anterior choroid artery

How well did you know this?
1
Not at all
2
3
4
5
Perfectly
403
Q

A 67 year old lady presents with jaundice and abdominal pain. Her investigations show a dilated common bile duct, a carcinoma of the pancreatic head compressing the pancreatic duct. Her liver contains bi-lobar metastasis. What is the most appropriate course of action?

Undertake synchronous resection of liver metastases and pancreatoduodenectomy

Resection of liver metastases, chemotherapy and then resection of the primary lesion

Insertion of endoscopic biliary stent and consideration of palliative chemotherapy

Palliation alone

Insertion of PTC drain and palliation

A

The presence of metastatic disease in the context of pancreatic cancer renders this incurable and resection of metastatic disease is not appropriate.

Pancreatic cancer

  • Adenocarcinoma
    Risk factors: Smoking, diabetes, adenoma, familial adenomatous polyposis
    Mainly occur in the head of the pancreas (70%)
    Spread locally and metastasizes to the liver
    Carcinoma of the pancreas should be differentiated from other periampullary tumours with better prognosis

Clinical features
Weight loss
Painless jaundice
Epigastric discomfort (pain usually due to invasion of the coeliac plexus is a late feature)
Pancreatitis
Trousseau’s sign: migratory superficial thrombophlebitis

Investigations
USS: May miss small lesions
CT Scanning (pancreatic protocol). If unresectable on CT then no further staging needed
PET/CT for those with operable disease on CT alone
ERCP/ MRI for bile duct assessment
Staging laparoscopy to exclude peritoneal disease

Management
Head of pancreas: Whipple’s resection (SE dumping and ulcers). Newer techniques include pylorus preservation and SMA/ SMV resection
Carcinoma body and tail: poor prognosis, distal pancreatectomy, if operable
Usually adjuvent chemotherapy for resectable disease
ERCP and stent for jaundice and palliation
Surgical bypass may be needed for duodenal obstruction

How well did you know this?
1
Not at all
2
3
4
5
Perfectly
404
Q

A motorcyclist is involved in a road traffic accident. He suffers a complex humeral shaft fracture which is plated. Post operatively he complains of an inability to extend his fingers. Which of the following structures is most likely to have been injured?

Ulnar nerve

Radial nerve

Median nerve

Axillary nerve

None of the above

A

Mnemonic for radial nerve muscles: BEST

B rachioradialis
E xtensors
S upinator
T riceps

Radial nerve

Continuation of posterior cord of the brachial plexus (root values C5 to T1)

Path
In the axilla: lies posterior to the axillary artery on subscapularis, latissimus dorsi and teres major.
Enters the arm between the brachial artery and the long head of triceps (medial to humerus).
Spirals around the posterior surface of the humerus in the groove for the radial nerve.
At the distal third of the lateral border of the humerus it then pierces the intermuscular septum and descends in front of the lateral epicondyle.
At the lateral epicondyle it lies deeply between brachialis and brachioradialis where it then divides into a superficial and deep terminal branch.
Deep branch crosses the supinator to become the posterior interosseous nerve.

How well did you know this?
1
Not at all
2
3
4
5
Perfectly
405
Q

Which muscle does not insert on the medial or superomedial surface of the greater trochanter?

Gemelli

Obturator internus

Piriformis

Quadratus femoris

Obturator externus

A

Mnemonic for muscle attachment on greater trochanter is POGO:
Piriformis
Obturator internus
Gemelli
Obturator externus
The quadratus femoris fibres pass laterally to be inserted into the quadrate tubercle on the intertrochanteric crest of the femur. The other muscles all insert on the trochanteric fossa lying medial to the greater trochanter.

Gluteal region

Gluteal muscles
Gluteus maximus: inserts to gluteal tuberosity of the femur and iliotibial tract
Gluteus medius: attach to lateral greater trochanter
Gluteus minimis: attach to anterior greater trochanter
All extend and abduct the hip

Deep lateral hip rotators
Piriformis
Gemelli
Obturator internus
Quadratus femoris

Nerves
Superior gluteal nerve (L4,L5, S1)
Gluteus medius
Gluteus minimis
Tensor fascia lata
Inferior gluteal nerve (L5, S1, S2) Gluteus maximus
Damage to the superior gluteal nerve will result in the patient developing a Trendelenberg gait. Affected patients are unable to abduct the thigh at the hip joint. During the stance phase, the weakened abductor muscles allow the pelvis to tilt down on the opposite side. To compensate, the trunk lurches to the weakened side to attempt to maintain a level pelvis throughout the gait cycle. The pelvis sags on the opposite side of the lesioned superior gluteal nerve.

How well did you know this?
1
Not at all
2
3
4
5
Perfectly
406
Q

Which of the following nerves is responsible for innervation of the triceps muscle?

Radial

Ulnar

Axillary

Median

None of the above

A

To remember nerve roots and their reflexes:

1-2 Ankle (S1-S2)
3-4 Knee (L3-L4)
5-6 Biceps (C5-C6)
7-8 Triceps (C7-C8)
The radial nerve innervates all three heads of triceps, with a separate branch to each head.

Triceps

Origin
Long head- infraglenoid tubercle of the scapula.
Lateral head- dorsal surface of the humerus, lateral and proximal to the groove of the radial nerve
Medial head- posterior surface of the humerus on the inferomedial side of the radial groove and both of the intermuscular septae
Insertion
Olecranon process of the ulna. Here the olecranon bursa is between the triceps tendon and olecranon.
Some fibres insert to the deep fascia of the forearm, posterior capsule of the elbow (preventing the capsule from being trapped between olecranon and olecranon fossa during extension)
Innervation Radial nerve
Blood supply Profunda brachii artery
Action Elbow extension. The long head can adduct the humerus and extend it from a flexed position
Relations The radial nerve and profunda brachii vessels lie between the lateral and medial heads

How well did you know this?
1
Not at all
2
3
4
5
Perfectly
407
Q

A 32 year old man requires venous access for chemotherapy for acute myeloid leukaemia. What is the best option?

Groshong line

Internal jugular central line (triple lumen)

14 G cannula in the dorsum of the hand

18 G cannula in the non dominant hand

Sub cuticular catheter

A

Groshong lines and Hickman lines are similar and knowledge of these systems is needed because surgeons are often called upon to either insert or remove them. Chemotherapy for AML requires long term therapy and multiple blood tests therefore an indwelling device is preferable.

Intravenous access

Venous access
A number of routes for establishing venous access are available.

Peripheral venous cannula
Easy to insert with minimal morbidity. Wide lumen cannulae can provide rapid fluid infusions. When properly managed infections may be promptly identified and the cannula easily re sited. Problems relate to their peripheral sites and they are unsuitable for the administration of vaso active drugs, such as inotropes and irritant drugs such as TPN (except in the very short term setting).

Central lines
Insertion is more difficult and most operators and NICE advocate the use of ultra sound. Coagulopathies may lead to haemorrhage following iatrogenic arterial injury. Femoral lines are easier to insert and iatrogenic injuries easier to manage in this site however they are prone to high infection rates. Internal jugular route is preferred. They have multiple lumens allowing for administration of multiple infusions. The lumens are relatively narrow and thus they do not allow particularly rapid rates of infusion.

Intraosseous access
This is typically undertaken at the anteromedial aspect of the proximal tibia and provides access to the marrow cavity and circulatory system. Although traditionally preferred in paediatric practice they may be used in adults and a wide range of fluids can be infused using these devices.

Tunneled lines
Tunneled lines such as Groshong and Hickman lines are popular devices for patients with long term therapeutic requirements. These devices are usually inserted using ultrasound guidance into the internal jugular vein and then tunneled under the skin. A cuff of woven material is sited near the end and helps to anchor the device into the tissues. These cuffs require formal dissection to allow the device to be removed. Tunneled lines can be linked to injection ports that are located under the skin. These are especially popular in paediatric practice.

Peripherally inserted central cannula
Referred to as PICC lines, these are popular methods for establishing central venous access. Because they are inserted peripherally they are less prone to major complications relating to device insertion than conventional central lines.

How well did you know this?
1
Not at all
2
3
4
5
Perfectly
408
Q

A 73 year old lady presents with a femoral hernia. Which of the following structures forms the lateral wall of the femoral canal?

Pubic tubercle

Femoral vein

Femoral artery

Conjoint tendon

Femoral nerve

A

The canal exists to allow for the physiological expansion of the femoral vein, which lies lateral to it.

Femoral canal

The femoral canal lies at the medial aspect of the femoral sheath. The femoral sheath is a fascial tunnel containing both the femoral artery laterally and femoral vein medially. The canal lies medial to the vein.

Borders of the femoral canal
Laterally Femoral vein
Medially Lacunar ligament
Anteriorly Inguinal ligament
Posteriorly Pectineal ligament

Contents
Lymphatic vessels
Cloquet’s lymph node

Physiological significance
Allows the femoral vein to expand to allow for increased venous return from the lower limbs.

Pathological significance
As a potential space, it is the site of femoral hernias. The relatively tight neck places these at high risk of strangulation.

How well did you know this?
1
Not at all
2
3
4
5
Perfectly
409
Q

A 77 year old man with symptoms of intermittent claudication is due to have his ankle brachial pressure indices measured. The dorsalis pedis artery is impalpable. Which of the following tendinous structures lies medial to it, that may facilitate its identification?

Extensor digitorum longus tendon

Peroneus tertius tendon

Extensor hallucis longus tendon

Extensor digitorum brevis tendon

Flexor digitorum longus tendon

A

The extensor hallucis longus tendon lies medial to the dorsalis pedis artery.

Foot- anatomy

Arches of the foot
The foot is conventionally considered to have two arches.
The longitudinal arch is higher on the medial than on the lateral side. The posterior part of the calcaneum forms a posterior pillar to support the arch. The lateral part of this structure passes via the cuboid bone and the lateral two metatarsal bones. The medial part of this structure is more important. The head of the talus marks the summit of this arch, located between the sustentaculum tali and the navicular bone. The anterior pillar of the medial arch is composed of the navicular bone, the three cuneiforms and the medial three metatarsal bones.
The transverse arch is situated on the anterior part of the tarsus and the posterior part of the metatarsus. The cuneiforms and metatarsal bases narrow inferiorly, which contributes to the shape of the arch.

Intertarsal joints
Sub talar joint Formed by the cylindrical facet on the lower surface of the body of the talus and the posterior facet on the upper surface of the calcaneus. The facet on the talus is concave anteroposteriorly, the other is convex. The synovial cavity of this joint does not communicate with any other joint.
Talocalcaneonavicular joint The anterior part of the socket is formed by the concave articular surface of the navicular bone, posteriorly by the upper surface of the sustentaculum tali. The talus sits within this socket
Calcaneocuboid joint Highest point in the lateral part of the longitudinal arch. The lower aspect of this joint is reinforced by the long plantar and plantar calcaneocuboid ligaments.
Transverse tarsal joint The talocalcaneonavicular joint and the calcaneocuboid joint extend across the tarsus in an irregular transverse plane, between the talus and calcaneus behind and the navicular and cuboid bones in front. This plane is termed the transverse tarsal joint.
Cuneonavicular joint Formed between the convex anterior surface of the navicular bone and the concave surface of the the posterior ends of the three cuneiforms.
Intercuneiform joints Between the three cuneiform bones.
Cuneocuboid joint Between the circular facets on the lateral cuneiform bone and the cuboid. This joint contributes to the tarsal part of the transverse arch.
A detailed knowledge of the joints is not required for MRCS Part A. However, the contribution they play to the overall structure of the foot should be appreciated

Nerves in the foot

Lateral plantar nerve
Passes anterolaterally towards the base of the 5th metatarsal between flexor digitorum brevis and flexor accessorius. On the medial aspect of the lateral plantar artery. At the base of the 5th metatarsal it splits into superficial and deep branches.

Medial plantar nerve
Passes forwards with the medial plantar artery under the cover of the flexor retinaculum to the interval between abductor hallucis and flexor digitorum brevis on the sole of the foot.

Plantar arteries
Arise under the cover of the flexor retinaculum, midway between the tip of the medial malleolus and the most prominent part of the medial side of the heel.

Medial plantar artery. Passes forwards medial to medial plantar nerve in the space between abductor hallucis and flexor digitorum brevis.Ends by uniting with a branch of the 1st plantar metatarsal artery.
Lateral plantar artery. Runs obliquely across the sole of the foot. It lies lateral to the lateral plantar nerve. At the base of the 5th metatarsal bone it arches medially across the foot on the metatarsals

Dorsalis pedis artery
This vessel is a direct continuation of the anterior tibial artery. It commences on the front of the ankle joint and runs to the proximal end of the first metatarsal space. Here is gives off the arcuate artery and continues forwards as the first dorsal metatarsal artery. It is accompanied by two veins throughout its length. It is crossed by the extensor hallucis brevis

How well did you know this?
1
Not at all
2
3
4
5
Perfectly
410
Q

Considering the pituitary gland, which of the following is false?

The anterior pituitary secretes thyroid stimulating hormone

The anterior pituitary develops from Rathkes pouch

Patients with craniopharyngioma may develop bi temporal hemianopia

The pituitary is in direct contact with the optic chiasm

The posterior pituitary releases oxytocin via a positive feedback loop

A

Although the optic chiasm is closely related to the pituitary, and craniopharyngiomas may compress this structure leading to bitemporal hemianopia, it is separated from the chiasm itself by a dural fold.

Pituitary Gland

The pituitary gland is located within the sella turcica within the sphenoid bone in the middle cranial fossa. It is covered by a dural fold and weighs around 0.5g. It is attached to the hypothalamus by the infundibulum. The anterior pituitary receives hormonal stimuli from the hypothalamus by way of the hypothalamo-pituitary portal system. It develops from a depression in the wall of the pharynx (Rathkes pouch).

Anterior pituitary hormones
Growth hormone
Thyroid stimulating hormone
ACTH
Prolactin
LH and FSH
Melanocyte releasing hormone

Posterior pituitary hormones
Oxytocin
Anti diuretic hormone

How well did you know this?
1
Not at all
2
3
4
5
Perfectly
411
Q

Which of the following nerves innervates the long head of the biceps femoris muscle?

Inferior gluteal nerve

Tibial division of sciatic nerve

Superior gluteal nerve

Common peroneal division of sciatic nerve

Obturator nerve

A

The short head of biceps femoris, which may occasionally be absent, is innervated by the common peroneal component of the sciatic nerve. The long head is innervated by the tibial division of the sciatic nerve.

Biceps femoris

The biceps femoris is one of the hamstring group of muscles located in the posterior upper thigh. It has two heads.

Long head
Origin Ischial tuberosity
Insertion Fibular head
Action Knee flexion, lateral rotation tibia, extension hip
Innervation Tibial division of sciatic nerve (L5, S1, S2)
Arterial supply Profunda femoris artery, inferior gluteal artery, and the superior muscular branches of popliteal artery

Short head
Origin Lateral lip of linea aspera, lateral supracondylar ridge of femur
Insertion Fibular head
Action Knee flexion, lateral rotation tibia
Innervation Common peroneal division of sciatic nerve (L5, S1, S2)
Arterial supply Profunda femoris artery, inferior gluteal art

How well did you know this?
1
Not at all
2
3
4
5
Perfectly
412
Q

During a neck dissection, a nerve is noted to pass posterior to the medial aspect of the first rib. Which of the nerves listed below is this most likely to be?

Medial cord of the brachial plexus

Long thoracic nerve

Nerve to subclavius

Medial pectoral nerve

Phrenic nerve

A

The key point is that the phrenic nerve runs posterior to the medial aspect of the first rib. Superiorly, it lies on the surface of scalenus anterior.

Phrenic nerve

Origin
C3,4,5

Supplies
Diaphragm, sensation central diaphragm and pericardium

Path
The phrenic nerve passes with the internal jugular vein across scalenus anterior. It passes superficial to prevertebral fascia of deep cervical fascia.
Left: crosses anterior to the 1st part of the subclavian artery.
Right: Anterior to scalenus anterior and crosses anterior to the 2nd part of the subclavian artery.
On both sides, the phrenic nerve runs posterior to the subclavian vein and posterior to the internal thoracic artery as it enters the thorax.

Right phrenic nerve
In the superior mediastinum: anterior to right vagus and laterally to superior vena cava
Middle mediastinum: right of pericardium
It passes over the right atrium to exit the diaphragm at T8

Left phrenic nerve
Passes lateral to the left subclavian artery, aortic arch and left ventricle
Passes anterior to the root of the lung
Pierces the diaphragm alone

How well did you know this?
1
Not at all
2
3
4
5
Perfectly
413
Q

Which of the following is a branch of the third part of the axillary artery?

Superior thoracic

Lateral thoracic

Dorsal scapular

Thoracoacromial

Posterior circumflex humeral

A

Posterior circumflex humeral

The other branches include:
Subscapular
Anterior circumflex humeral
The dorsal scapular artery arises from the third part of the subclavian artery in most cases

Axilla

Boundaries of the axilla
Medially Chest wall and Serratus anterior
Laterally Humeral head
Floor Subscapularis
Anterior aspect Lateral border of Pectoralis major
Fascia Clavipectoral fascia

Content:
Long thoracic nerve (of Bell) Derived from C5-C7 and passes behind the brachial plexus to enter the axilla. It lies on the medial chest wall and supplies serratus anterior. Its location puts it at risk during axillary surgery and damage will lead to winging of the scapula.
Thoracodorsal nerve and thoracodorsal trunk Innervate and vascularise latissimus dorsi.
Axillary vein Lies at the apex of the axilla, it is the continuation of the basilic vein. Becomes the subclavian vein at the outer border of the first rib.
Intercostobrachial nerves Traverse the axillary lymph nodes and are often divided during axillary surgery. They provide cutaneous sensation to the axillary skin.
Lymph nodes The axilla is the main site of lymphatic drainage for the breast.

Branches of Axillary Artery

FROM FIRST PART
Superior thoracic artery, a very small branch, originates near the subclavius, enters between the pectoralis major and minor muscles, and then these muscles go towards the medial wall of the axilla.
FROM SECOND PART
Thoraco-acromial artery (acromiothoracic artery) : emerges at the upper border of pectoralis minor, pierces clavipectoral fascia and soon breaks up into 4 branches:
(a) pectoral branch,
(b) deltoid branch,
(c) acromial branch, and
(d) clavicular branch.
These branches radiate at right angle to every other. The pectoral branch consists of pectoral muscles, deltoid branch, ends by joining anastomosis over the acromion, clavicular branch supplied sternoclavicular joint.
Lateral thoracic artery : emerges at and runs along the inferior border of pectoralis minor, supplying the branches to pectoralis major and minor and serratus anterior muscles. In the females, the lateral thoracic artery is large and gives significant supply to the breast via it’s lateral mammary branches.
FROM THIRD PART
Subscapular artery, the largest branch of axillary artery, runs along the lower border of the subscapularis and ends near the inferior angle of the scapula. It supplies a large branch, the circumflex scapular artery; which goes through upper triangular intermuscular space, winds round the lateral border of scapula to goes into infraspinous fossa. Additionally, it supplies numerous small branches.
Anterior circumflex humeral artery, a small branch, enters in front of surgical neck of humerus and anastomoses with the posterior circumflex humeral artery to create an arterial circle around the surgical neck of humerus. It supplies an ascending branch, which runs upwards into the intertubercular sulcus of humerus to supply the head of humerus and shoulder joint.
Posterior circumflex humeral artery, larger in relation to the anterior circumflex humeral artery, enters backwards, together with axillary nerve via the quadrangular intermuscular space, crosses the posterior aspect of surgical neck of humerus to anastomose with the anterior circumflex humeral artery. It supplies the deltoid muscle and shoulder joint.

How well did you know this?
1
Not at all
2
3
4
5
Perfectly
414
Q

Which statement relating to phaeochromocytoma is untrue?

They are tumours of chromaffin cells in the adrenal medulla.

They are bilateral in 10% of cases.

When located in an extra adrenal location have a higher incidence of malignancy.

May be associated with an elevated urinary VMA.

Up to 40% may have a blood pressure within the normal range.

A

Normotension is seen in around 10% cases. The remainder show a degree of hypertension.
Phaeochromocytoma and adrenal lesions

Phaeochromocytoma
Neuroendocrine tumour of the chromaffin cells of the adrenal medulla. Hypertension and hyperglycaemia are often found.
10% of cases are bilateral.
10% occur in children.
11% are malignant (higher when tumour is located outside the adrenal).
10% will not be hypertensive.

Familial cases are usually linked to the Multiple endocrine neoplasia syndromes (considered under its own heading).

Most tumours are unilateral (often right sided) and smaller than 10cm.

Diagnosis
Urine and plasma metanephrine levels.

CT and MRI scanning are both used to localise the lesion.

Treatment
Patients require medical therapy first. An irreversible alpha adrenoreceptor blocker should be given, although minority may prefer reversible blockade(1). Labetolol may be co-administered for cardiac chronotropic control. Isolated beta blockade should not be considered as it will lead to unopposed alpha activity.

These patients are often volume depleted and will often require moderate volumes of intra venous normal saline perioperatively.

Once medically optimised the phaeochromocytoma should be removed. Most adrenalectomies can now be performed using a laparoscopic approach(2). The adrenals are highly vascular structures and removal can be complicated by catastrophic haemorrhage in the hands of the inexperienced. This is particularly true of right sided resections where the IVC is perilously close. Should the IVC be damaged a laparotomy will be necessary and the defect enclosed within a Satinsky style vascular clamp and the defect closed with prolene sutures. Attempting to interfere with the IVC using any instruments other than vascular clamps will result in vessel trauma and make a bad situation much worse.

Incidental adrenal lesions
Adrenal lesions may be identified on CT scanning performed for other reasons(3). Factors suggesting benign disease on CT include(4):
Size less than 3cm
Homogeneous texture
Lipid rich tissue
Thin wall to lesion

All patients with incidental lesions should be managed jointly with an endocrinologist and full work up as described above. Patients with functioning lesions or those with adverse radiological features (Particularly size >3cm) should proceed to surgery.

How well did you know this?
1
Not at all
2
3
4
5
Perfectly
415
Q

A 67 year old man is investigated for biliary colic and a 4.8 cm abdominal aortic aneurysm is identified. Which of the following statements relating to this condition is untrue?

The wall will be composed of dense fibrous tissue only

The majority are located inferior to the renal arteries

They occur most often in current or former smokers

He should initially be managed by a process of active surveillance

Aortoduodenal fistula is a recognised complication following repair.

A

They are true aneurysms and have all 3 layers of arterial wall.
Abdominal aorta aneurysm

  • Abdominal aortic aneurysms are a common problem in vascular surgery.
    They may occur as either true or false aneurysm. With the former all 3 layers of the arterial wall are involved, in the latter only a single layer of fibrous tissue forms the aneurysm wall.
    True abdominal aortic aneurysms have an approximate incidence of 0.06 per 1000 people. They are commonest in elderly men and for this reason the UK is now introducing the aneurysm screening program with the aim of performing an abdominal aortic ultrasound measurement in all men aged 65 years.

Causes
Several different groups of patients suffer from aneurysmal disease.
The commonest group is those who suffer from standard arterial disease, i.e. Those who are hypertensive and have been or are smokers.
Other patients such as those suffering from connective tissue diseases such as Marfan’s may also develop aneurysms. In patients with abdominal aortic aneurysms the extracellular matrix becomes disrupted with a change in the balance of collagen and elastic fibres.

Management
Most abdominal aortic aneurysms are an incidental finding.
Symptoms most often relate to rupture or impending rupture.
20% rupture anteriorly into the peritoneal cavity. Very poor prognosis.
80% rupture posteriorly into the retroperitoneal space
The risk of rupture is related to aneurysm size, only 2% of aneurysms measuring less than 4cm in diameter will rupture over a 5 year period. This contrasts with 75% of aneurysms measuring over 7cm in diameter.
This is well explained by Laplaces’ law which relates size to transmural pressure.
For this reason most vascular surgeons will subject patients with an aneurysm size of 5cm or greater to CT scanning of the chest, abdomen and pelvis with the aim of delineating anatomy and planning treatment. Depending upon co-morbidities, surgery is generally offered once the aneurysm is between 5.5cm and 6cm.

Indications for surgery
Symptomatic aneurysms (80% annual mortality if untreated)
Increasing size above 5.5cm if asymptomatic
Rupture (100% mortality without surgery)

Surgical procedures
Abdominal aortic aneurysm repair

Procedure:

GA
Invasive monitoring (A-line, CVP, catheter)
Incision: Midline or transverse
Bowel and distal duodenum mobilised to access aorta.
Aneurysm neck and base dissected out and prepared for cross clamp
Systemic heparinisation
Cross clamp (proximal first)
Longitudinal aortotomy
Atherectomy
Deal with back bleeding from lumbar vessels and inferior mesenteric artery
Insert graft either tube or bifurcated depending upon anatomy
Suture using Prolene (3/0 for proximal , distal anastomosis suture varies according to site)
Clamps off: End tidal CO2 will rise owing to effects of reperfusion, at this point major risk of myocardial events.
Haemostasis
Closure of aneurysm sac to minimise risk of aorto-enteric fistula
Closure: Loop 1 PDS or Prolene to abdominal wall
Skin- surgeons preference

Post operatively:

ITU (Almost all)
Greatest risk of complications following emergency repair
Complications: Embolic- gut and foot infarcts
Cardiac - owing to premorbid states, re-perfusion injury and effects of cross clamp
Wound problems
Later risks related to graft- infection and aorto-enteric fistula

Special groups

Supra renal AAA
These patients will require a supra renal clamp and this carries a far higher risk of complications and risk of renal failure.

Ruptured AAA
Pre-operatively the management depends upon haemodynamic instability. In patients with symptoms of rupture (typical pain, haemodynamic compromise and risk factors) then ideally prompt laparotomy. In those with vague symptoms and haemodynamic stability the ideal test is CT scan to determine whether rupture has occurred or not. Most common rupture site is retroperitoneal 80%. These patients will tend to develop retroperitoneal haematoma. This can be disrupted if Bp is allowed to rise too high so aim for Bp 100mmHg.
Operative details are similar to elective repair although surgery should be swift, blind rushing often makes the situation worse. Plunging vascular clamps blindly into a pool of blood at the aneurysm neck carries the risk of injury the vena cava that these patients do not withstand. Occasionally a supracoeliac clamp is needed to effect temporary control, although leaving this applied for more than 20 minutes tends to carry a dismal outcome.

EVAR
Increasingly patients are now being offered endovascular aortic aneurysm repair. This is undertaken by surgeons and radiologists working jointly. The morphology of the aneurysm is important and not all are suitable. Here is a typical list of those features favoring a suitable aneurysm:
Long neck
Straight iliac vessels
Healthy groin vessels

Clearly few AAA patients possess the above and compromise has to be made. The use of fenestrated grafts can allow supra renal AAA to be treated.

Procedure:

GA
Radiology or theatre
Bilateral groin incisions
Common femoral artery dissected out
Heparinisation
Arteriotomy and insertion of guide wire
Dilation of arteriotomy
Insertion of EVAR Device
Once in satisfactory position it is released
Arteriotomy closed once check angiogram shows good position and no endoleak

Complications:
Endoleaks depending upon site are either Type I or 2. These may necessitate re-intervention and all EVAR patients require follow up . Details are not needed for MRCS.

How well did you know this?
1
Not at all
2
3
4
5
Perfectly
416
Q

A 28 year old man is stabbed outside a nightclub in the upper arm. The median nerve is transected. Which of the following muscles will demonstrate impaired function as a result?

Palmaris brevis

Second and third interossei

Adductor pollicis

Abductor pollicis longus

Abductor pollicis brevis

A

Palmaris brevis - Ulnar nerve
Palmar interossei- Ulnar nerve
Adductor pollicis - Ulnar nerve
Abductor pollicis longus - Posterior interosseous nerve
Abductor pollicis brevis - Median nerve
The median nerve innervates all the short muscles of the thumb except the adductor and the deep head of the short flexor. Palmaris and the interossei are innervated by the ulnar nerve.

Median nerve

The median nerve is formed by the union of a lateral and medial root respectively from the lateral (C5,6,7) and medial (C8 and T1) cords of the brachial plexus; the medial root passes anterior to the third part of the axillary artery. The nerve descends lateral to the brachial artery, crosses to its medial side (usually passing anterior to the artery). It passes deep to the bicipital aponeurosis and the median cubital vein at the elbow.
It passes between the two heads of the pronator teres muscle, and runs on the deep surface of flexor digitorum superficialis (within its fascial sheath).
Near the wrist it becomes superficial between the tendons of flexor digitorum superficialis and flexor carpi radialis, deep to palmaris longus tendon. It passes deep to the flexor retinaculum to enter the palm, but lies anterior to the long flexor tendons within the carpal tunnel.

Branches
Region Branch
Upper arm No branches, although the nerve commonly communicates with the musculocutaneous nerve
Forearm Pronator teres
Pronator quadratus
Flexor carpi radialis
Palmaris longus
Flexor digitorum superficialis
Flexor pollicis longus
Flexor digitorum profundus (only the radial half)
Distal forearm Palmar cutaneous branch
Hand (Motor) Motor supply (LOAF)
Lateral 2 lumbricals
Opponens pollicis
Abductor pollicis brevis
Flexor pollicis brevis
Hand (Sensory)
Over thumb and lateral 2 ½ fingers
On the palmar aspect this projects proximally, on the dorsal aspect only the distal regions are innervated with the radial nerve providing the more proximal cutaneous innervation.

Patterns of damage
Damage at wrist
e.g. carpal tunnel syndrome
paralysis and wasting of thenar eminence muscles and opponens pollicis (ape hand deformity)
sensory loss to palmar aspect of lateral (radial) 2 ½ fingers

Damage at elbow, as above plus:
unable to pronate forearm
weak wrist flexion
ulnar deviation of wrist

Anterior interosseous nerve (branch of median nerve)
leaves just below the elbow
results in loss of pronation of forearm and weakness of long flexors of thumb and index finger

How well did you know this?
1
Not at all
2
3
4
5
Perfectly
417
Q

A 68 year old man presents to the plastics team with severe burns to his hands. He is not distressed by the burns. He has bilateral charcot joints. On examination; there is loss of pain and temperature sensation of the upper limbs.

A. Osteomyelitis
B. Potts disease of the spine
C. Scheuermanns disease
D. Transverse myelitis
E. Tabesdorsalis
F. Subacutedegenerationofthecord G. Brown-Sequard syndrome
H. Syringomyelia
I. Epiduralhaematoma

A

The correct answer is Syringomyelia
This patient has syringomyelia which selectively affects the spinotholamic tracts. Syringomyelia is a disorder in which a cystic cavity forms within the spinal cord. The commonest variant is the Arnold- Chiari malformation in which the cavity connects with a congenital malformation affecting the cerebellum. Acquired forms of the condition may occur as a result of previous meningitis, surgery or tumours. Many neurological manifestations have been reported, although the classical variety spares the dorsal columns and medial lemniscus and affecting only the spinothalamic tract with loss of pain and temperature sensation. The bilateral distribution of this patients symptoms would therefore favor syringomyelia over SCID or Brown Sequard syndrome. Osteomyelitis would tend to present with back pain and fever in addition to any neurological signs. Epidural haematoma large enough to produce neurological impairment will usually have motor symptoms in addition to any selective sensory loss, and the history is usually shorter.

How well did you know this?
1
Not at all
2
3
4
5
Perfectly
418
Q

A 24 year old man presents with localised spinal pain over 2 months which is worsened on movement. He is known to be an IVDU. He has no history suggestive of tuberculosis. The pain is now excruciating at rest and not improving with analgesia. He has a temperature of 39 oC. Which is the most likely diagnosis for the scenario given.

A. Osteomyelitis
B. Potts disease of the spine
C. Scheuermanns disease
D. Transverse myelitis
E. Tabesdorsalis
F. Subacutedegenerationofthecord G. Brown-Sequard syndrome
H. Syringomyelia
I. Epiduralhaematoma

A

The correct answer is Osteomyelitis
In an IVDU with back pain and pyrexia have a high suspicion for osteomylelitis. The most likely organism is staph aureus and the cervical spine is the most common region affected. TB tends to affect the thoracic spine and in other causes of osteomyelitis the lumbar spine is affected.

How well did you know this?
1
Not at all
2
3
4
5
Perfectly
419
Q

A 22 year man is shot in the back, in the lumbar region. He has increased tone and hyper-reflexia of his right leg. He cannot feel his left leg. Which is the most likely diagnosis for the scenario given.

A. Osteomyelitis
B. Potts disease of the spine
C. Scheuermanns disease
D. Transverse myelitis
E. Tabesdorsalis
F. Subacute degeneration of the cord
G. Brown-Sequard syndrome
H. Syringomyelia
I. Epidural haematoma

A

The correct answer is Brown-Sequard syndrome
Theme from January 2012 exam
Brown -Sequard syndrome is caused by hemisection of the spinal cord. It may result from stab injuries or lateral vertebral fractures. It results in ipsilateral paralysis (pyramidal tract) , and also loss of proprioception and fine discrimination (dorsal columns). Pain and temperature sensation are lost on the contra-lateral side. This is because the fibres of the spinothalamic tract have decussated below the level of the cord transection.

Spinal disorders
Dorsal column lesion
Spinothalamic tract lesion Central cord lesion
Infarction spinal cord
Loss vibration and proprioception Tabes dorsalis, SACD
Loss of pain, sensation and temperature Flaccid paralysis of the upper limbs
Dorsal column signs (loss of proprioception and fine discrimination)
Osteomyelitis
Normally progressive
Staph aureus in IVDU, normally cervical region affected Fungal infections in immunocompromised
Thoracic region affected in TB
Cord compression
UMN signs Malignancy Haematoma Fracture
Brown-sequard syndrome
Hemisection of the spinal cord
Ipsilateral paralysis
Ipsilateral loss of proprioception and fine discrimination Contralateral loss of pain and temperature

How well did you know this?
1
Not at all
2
3
4
5
Perfectly
420
Q

A 24 year old man is brought to the emergency department have suffered a crush injury to his forearm. Assessment demonstrates that the arm is tender, red and swollen. There is clinical evidence of an ulnar fracture and the patient cannot move their fingers. Which is the most appropriate course of action?
Application of an external fixation device Closed reduction
Debridement
Discharge and review in fracture clinic Fasciotomy

A

Theme from April 2012
The combination of a crush injury, limb swelling and inability to move digits should raise suspicion of a compartment syndrome that will require a fasciotomy. Paralysis is a very late sign.
Compartment syndrome
This is a particular complication that may occur following fractures (or following ischaemia re-perfusion injury in vascular patients). It is characterised by raised pressure within a closed anatomical space.
The raised pressure within the compartment will eventually compromise tissue perfusion resulting in necrosis. The two main fractures carrying this complication include supracondylar fractures and tibial shaft injuries.
Symptoms and signs
Pain, especially on movement (even passive)
Parasthesiae
Pallor may be present
Arterial pulsation may still be felt as the necrosis occurs as a result of microvascular compromise Paralysis of the muscle group may occur
Diagnosis
Is made by measurement of intracompartmental pressure measurements. Pressures in excess of 20mmHg are abnormal and >40mmHg is diagnostic.
Treatment
This is essentially prompt and extensive fasciotomies
In the lower limb the deep muscles may be inadequately decompressed by the inexperienced operator when smaller incisions are performed Myoglobinuria may occur following fasciotomy and result in renal failure and for this reason these patients require aggressive IV fluids
Where muscle groups are frankly necrotic at fasciotomy they should be debrided and amputation may have to be considered
Death of muscle groups may occur within 4-6 hours

How well did you know this?
1
Not at all
2
3
4
5
Perfectly
421
Q

Which of the following transplants is most susceptible to donor- recipient HLA mismatches?

Autologous skin graft

Renal allograft

Liver allograft

Corneal allograft

Cardiac valve allograft

A

Autologous transplant- same individual (genetically identical)
Allograft - Genetically different
The kidney is highly susceptible to HLA mismatches and hyperacute rejection may occur in patients with IgG anti HLA Class I antibodies. The liver is at far lower risk of rejection of this nature. Although the heart is sensitive to HLA mismatches this is less than the kidney. Cardiac valves and the cornea incite little immunological response.

Organ Transplant

A number of different organ and tissue transplants are now available. In many cases an allograft is performed, where an organ is transplanted from one individual to another. Allografts will elicit an immune response and this is one of the main reasons for organ rejection.

Graft rejection occurs because allografts have allelic differences at genes that code immunohistocompatability complex genes. The main antigens that give rise to rejection are:
ABO blood group
Human leucocyte antigens (HLA)
Minor histocompatability antigens

ABO Matching
ABO incompatibility will result in early organ rejection (hyperacute) because of pre existing antibodies to other groups. Group O donors can give organs to any type of ABO recipient whereas group AB donor can only donate to AB recipient.

HLA System
The four most important HLA alleles are:

HLA A
HLA B
HLA C
HLA DR

An ideal organ match would be one in which all 8 alleles are matched (remember 2 from each parent, four each = 8 alleles). Modern immunosuppressive regimes help to manage the potential rejection due to HLA mismatching. However, the greater the number of mismatches the worse the long term outcome will be. T lymphocytes will recognise antigens bound to HLA molecules and will then become activated. Clonal expansion then occurs with a response directed against that antigen.

Types of organ rejection
Hyperacute. This occurs immediately through presence of pre formed antibodies (such as ABO incompatibility).
Acute. Occurs during the first 6 months and is usually T cell mediated. Usually tissue infiltrates and vascular lesions.
Chronic. Occurs after the first 6 months. Vascular changes predominate.

Hyperacute
Renal transplants at greatest risk and liver transplants at least risk. Although ABO incompatibility and HLA Class I incompatible transplants will all fare worse in long term.

Acute
All organs may undergo acute rejection. Mononuclear cell infiltrates predominate. All types of transplanted organ are susceptible and it may occur in up to 50% cases.

Chronic
Again all transplants with HLA mismatch may suffer this fate. Previous acute rejections and other immunosensitising events all increase the risk. Vascular changes are most prominent with myointimal proliferation leading to organ ischaemia. Organ specific changes are also seen such as loss of acinar cells in pancreas transplants and rapidly progressive coronary artery disease in cardiac transplants.

Surgical overview-Renal transplantation
A brief overview of the steps involved in renal transplantation is given.
Patients with end stage renal failure who are dialysis dependent or likely to become so in the immediate future are considered for transplant. Exclusion criteria include; active malignancy, old age (due to limited organ availability). Patients are medically optimised.
Donor kidneys, these may be taken from live related donors and close family, members may have less HLA mismatch than members of the general population. Laparoscopic donor nephrectomy further minimises the operative morbidity for the donor. Other organs are typically taken from brain dead or dying patients who have a cardiac arrest and in whom resuscitation is futile. The key event is to minimise the warm ischaemic time in the donor phase.

The kidney once removed is usually prepared on the bench in theatre by the transplant surgeon immediately prior to implantation and factors such as accessory renal arteries and vessel length are assessed and managed.

For first time recipients the operation is performed under general anaesthesia. A Rutherford-Morison incision is made on the preferred side. This provides excellent extraperitoneal access to the iliac vessels. The external iliac artery and vein are dissected out and following systemic heparinisation are cross clamped. The vein and artery are anastamosed to the iliacs and the clamps removed. The ureter is then implanted into the bladder and a stent is usually placed to maintain patency. The wounds are then closed and the patient recovered from surgery.

In the immediate phase a common problem encountered in cadaveric kidneys is acute tubular necrosis and this tends to resolve.

Graft survival times from cadaveric donors are typically of the order of 9 years and monozygotic twin transplant (live donor) may survive as long as 25 years.

How well did you know this?
1
Not at all
2
3
4
5
Perfectly
422
Q

Which of the following statements relating to menisceal tears is false?

The medial meniscus is most often affected

True locking of the knee joint may occur

Most established tears will heal with conservative management

In the chronic setting there is typically little to find on examination if the knee is not locked

An arthroscopic approach may be used to treat most lesions

A

Menisci have no nerve or blood supply and thus heal poorly. Established tears with associated symptoms are best managed by arthroscopic menisectomy.

How well did you know this?
1
Not at all
2
3
4
5
Perfectly
423
Q

A 25 year old man is undergoing respiratory spirometry. He takes a maximal inspiration and maximally exhales. Which of the following measurements will best illustrate this process?

Functional residual capacity

Vital capacity

Inspiratory capacity

Maximum voluntary ventilation

Tidal volume

A

Vital capacity

The maximum voluntary ventilation is the maximal ventilation over the course of 1 minute.

How well did you know this?
1
Not at all
2
3
4
5
Perfectly
424
Q

A 58 year old male is referred to endocrinology clinic for a parathyroidectomy by the F1 in medicine. His corrected calcium is 2.85 (2.2-2.6), PTH 7.5 (3-7) and 24h urinary calcium is 1.5 (2.5-7.5). What is the diagnosis?

Primary hyperparathyroidism

Secondary hyperparathyroidism

Tertiary hyperparathyroidism

Familial hypocalciuric hypercalcaemia

Hypercalacemia associated with malignancy

A

This F1 should have spoken to his senior. This patient has familial hypocalciuric hypercalcaemia, which requires no further action. A calcium to creatinine clearance ratio of <0.01 will confirm this diagnosis.

How well did you know this?
1
Not at all
2
3
4
5
Perfectly
425
Q

A 65 year old lady presents with a lesion affecting her right breast. On examination she has a weeping, crusting lesion overlying the right nipple, the areolar region is not involved. There is no palpable mass lesion in the breast, there is a palpable axillary lymph node. The patient’s general practitioner has tried treating the lesion with 1% hydrocortisone cream, with no success. What is the most likely diagnosis?

Infection with Staphylococcus aureus

Pagets disease of the nipple

Phylloides tumour

Nipple eczema

Basal cell carcinoma

A

A weeping, crusty lesion such as this is most likely to represent Pagets disease of the nipple (especially since the areolar region is spared). Although no mass lesion is palpable, a proportion of patients will still have an underlying invasive malignancy (hence the lymphadenopathy).

Pagets disease differs from eczema of the nipple in that it involves the nipple primarily and only latterly spreads to the areolar (the opposite occurs in eczema).

How well did you know this?
1
Not at all
2
3
4
5
Perfectly
426
Q

Which of the following statements relating to quadratus lumborum is false?

Causes flexion of the thoracic spine

Causes the rib cage to be pulled down

Innervated by anterior primary rami of T12 and L1-3

Attached to the iliac crest

Inserts into the 12th rib

A

Quadratus lumborum
Origin: Medial aspect of iliac crest and iliolumbar ligament
Insertion: 12th rib
Action: Pulls the rib cage inferiorly. Lateral flexion.
Nerve supply: Anterior primary rami of T12 and L1-3

The rectus abdominis causes flexion of the thoracic spine and therefore the statement suggesting that quaratus lumborum does so is incorrect.

How well did you know this?
1
Not at all
2
3
4
5
Perfectly
427
Q

A 76 year old man presents with a painful left leg. The pain began suddenly and with no previous history. On examination he has a white left leg with no palpable femoral pulse and loss of sensation. The pulses in the contra lateral limb are normal. It is now three hours since the pain first started.

A.Primary amputation

B.Transfemoral embolectomy with prophylactic fasciotomy

C.Transpopliteal embolectomy without prophylactic fasciotomy

D.Transfemoral embolectomy without prophylactic fasciotomy

E.Transpopliteal embolectomy with prophylactic fasciotomy

F.Angiogram

G.Systemic heparin infusion

H.Peripheral thrombolysis

A

Transfemoral embolectomy without prophylactic fasciotomy

A limb which is acutely ischaemic and with normal contralateral pulses normally indicates an acute embolus. Whilst intra arterial thrombolysis may be an option there is a reasonable argument for immediate surgery. A fasciotomy is unlikely to be required.

How well did you know this?
1
Not at all
2
3
4
5
Perfectly
428
Q

A 56 year old man presents with a painful left leg. The pain has been present for the past 8 hours although it has also been present (though less severe) about a week ago. At that stage he noted that his hallux had turned blue. This resolved spontaneously. On examination he has a weakly palpable femoral pulse on the affected side but no pulses palpable distal to this. His sensation is mildly impaired.

A.Primary amputation

B.Transfemoral embolectomy with prophylactic fasciotomy

C.Transpopliteal embolectomy without prophylactic fasciotomy

D.Transfemoral embolectomy without prophylactic fasciotomy

E.Transpopliteal embolectomy with prophylactic fasciotomy

F.Angiogram

G.Systemic heparin infusion

H.Peripheral thrombolysis

A

The correct answer is Angiogram

The history favors a more chronic process and the great toe cyanotic spell may be indicative of previous embolism from pathology such as an aneurysm. In the ideal scenario a duplex scan would be performed. However, an angiogram would probably supply sufficient information and allow appropriate endovascular therapy.

How well did you know this?
1
Not at all
2
3
4
5
Perfectly
429
Q

A 78 year old lady is found by carers with a severely painful left leg. On examination she has no palpable pulses and the limb is cold, insensate and mottled. The mottling does not blanch with pressure.

A.Primary amputation

B.Transfemoral embolectomy with prophylactic fasciotomy

C.Transpopliteal embolectomy without prophylactic fasciotomy

D.Transfemoral embolectomy without prophylactic fasciotomy

E.Transpopliteal embolectomy with prophylactic fasciotomy

F.Angiogram

G.Systemic heparin infusion

H.Peripheral thrombolysis

A

Primary amputation

This is an unsalvagable limb and is best amputated primarily.

How well did you know this?
1
Not at all
2
3
4
5
Perfectly
430
Q

Which of the following does not decrease the functional residual capacity?

Obesity

Pulmonary fibrosis

Muscle relaxants

Laparoscopic surgery

Upright position

A

Increased FRC:

Erect position

Emphysema

Asthma

Decreased FRC:

Pulmonary fibrosis

Laparoscopic surgery

Obesity

Abdominal swelling

Muscle relaxants

When the patient is upright the diaphragm and abdominal organs put less pressure on the lung bases, allowing for an increase in the functional residual capacity (FRC). Other causes of increased FRC include:

Emphysema

Asthma

In addition to those listed above, causes of reduced FRC include:

Abdominal swelling

Pulmonary oedema

Reduced muscle tone of the diaphragm

Age

How well did you know this?
1
Not at all
2
3
4
5
Perfectly
431
Q

A 23 year old climber falls and fractures his humerus. The surgeons decide upon a posterior approach to the middle third of the bone. Which of the following nerves is at greatest risk in this approach?

Ulnar

Antebrachial

Musculocutaneous

Radial

Intercostobrachial

A

The radial nerve wraps around the humerus and may be injured during a posterior approach. An IM nail may be preferred as it avoids the complex dissection needed for direct bone exposure.

How well did you know this?
1
Not at all
2
3
4
5
Perfectly
432
Q

Which of the following pathological explanations best describes the initial pathological processes occurring in an abdominal aortic aneurysm in an otherwise well 65 year old, hypertensive male?

Loss of elastic fibres from the adventitia

Loss of collagen from the adventitia

Loss of collagen from the media

Loss of elastic fibres from the media

Decreased matrix metalloproteinases in the adventitia

A

Loss of elastic fibres from the media

In established aneurysmal disease there is dilation of all layers of the arterial wall and loss of both elastin and collagen. The primary event is loss of elastic fibres with subsequent degradation of collagen fibres.

How well did you know this?
1
Not at all
2
3
4
5
Perfectly
433
Q

In medical statistics, which of the following does a p value of 0.04 represent?

Risk of type 1 Error

Risk of type 2 Error

Size of power of the study

Sample size

Number of degrees of freedom

A

P values are related to the significance levels of a statistical test and therefore are in effect measuring the risk of a type 1 error.

How well did you know this?
1
Not at all
2
3
4
5
Perfectly
434
Q

Which of the following is the main site of dehydroepiandrosterone release?

Posterior pituitary

Zona reticularis of the adrenal gland

Zona glomerulosa of the adrenal gland

Juxtaglomerular apparatus of the kidney

Zona fasciculata of the adrenal gland

A

The zona fasciculata of the adrenal gland is the primary site responsible for the production of dehydroepiandrosterone (DHEA). This layer of the adrenal cortex synthesizes DHEA, which is an important precursor for sex hormones. On the other hand, the zona glomerulosa is primarily involved in the production of aldosterone, a mineralocorticoid hormone that regulates sodium and potassium balance in the body. So, when it comes to DHEA, think zona fasciculata!

How well did you know this?
1
Not at all
2
3
4
5
Perfectly
435
Q

A 34 year old lady undergoes an elective cholecystectomy for attacks of recurrent cholecystitis due to gallstones. Microscopic assessment of the gallbladder is most likely to show which of the following?

Dysplasia of the fundus

Widespread necrosis

Aschoff-Rokitansky sinuses

Metaplasia of the fundus

None of the above

A

Aschoff-Rokitansky sinuses are the result of hyperplasia and herniation of epithelial cells through the fibromuscular layer of the gallbladder wall. They may be macroscopic or microscopic. Ashoff-Rokitansky sinuses may be identified in cases of chronic cholecystitis and gallstones. Although gallstones may predispose to the development of gallbladder cancer the actual incidence of dysplasia and metaplastic change is rare. In the elective setting described above necrosis would be rare.

How well did you know this?
1
Not at all
2
3
4
5
Perfectly
436
Q

A 1-day-old baby girl is noted to become profoundly cyanotic whilst feeding and crying. A diagnosis of congenital heart disease is suspected. What is the most likely cause?

Transposition of the great arteries

Coarctation of the aorta

Patent ductus arteriosus

Hypoplastic left heart

Ventricular septal defect

A

Congenital heart disease

Cyanotic: TGA most common at birth, Fallot’s most common overall

Acyanotic: VSD most common cause

It is important to be aware of common congenital cardiac abnormalities. The main differentiating factor is whether the patient is cyanotic or acyanotic. In the neonate, TGA is the most common condition presenting with profound cyanosis.

The other options are causes of acyanotic congenital heart disease

How well did you know this?
1
Not at all
2
3
4
5
Perfectly
437
Q

A 15 year-old boy presents to the out-patient clinic with tiredness, recurrent throat and chest infections, and gradual loss of vision. Multiple x-rays show brittle bones with no differentiation between the cortex and the medulla.

A.Rickets

B.Craniocleidodysostosis

C.Achondroplasia

D.Scurvy

E.Pagets disease

F.Multiple myeloma

G.Osteogenesis imperfecta

H.Osteomalacia

I.Osteopetrosis

J.None of the above

A

Osteopetrosis

Osteopetrosis is an autosomal recessive condition. It is commonest in young adults. They may present with symptoms of anaemia or thrombocytopaenia due to decreased marrow space. Radiology reveals a lack of differentiation between the cortex and the medulla described as marble bone. These bones are very dense and brittle.

How well did you know this?
1
Not at all
2
3
4
5
Perfectly
438
Q

A 12 year-old boy who is small for his age presents to the clinic with poor muscular development and hyper-mobile fingers. His x-rays show multiple fractures of the long bones and irregular patches of ossification.

A.Rickets

B.Craniocleidodysostosis

C.Achondroplasia

D.Scurvy

E.Pagets disease

F.Multiple myeloma

G.Osteogenesis imperfecta

H.Osteomalacia

I.Osteopetrosis

J.None of the above

A

Osteogenesis imperfecta

Osteogenesis imperfecta is caused by defective osteoid formation due to congenital inability to produce adequate intercellular substances like osteoid, collagen and dentine. There is a failure of maturation of collagen in all the connective tissues.Radiology may show translucent bones, multiple fractures, particularly of the long bones, wormian bones (irregular patches of ossification) and a trefoil pelvis.

How well did you know this?
1
Not at all
2
3
4
5
Perfectly
439
Q

A 1 year-old is brought to the Emergency Department with a history of failure to thrive. On examination, the child is small for age and has a large head. X-ray shows a cupped appearance of the epiphysis of the wrist.

A.Rickets

B.Craniocleidodysostosis

C.Achondroplasia

D.Scurvy

E.Pagets disease

F.Multiple myeloma

G.Osteogenesis imperfecta

H.Osteomalacia

I.Osteopetrosis

J.None of the above

A

Rickets

Rickets is the childhood form of osteomalacia. It is due to the failure of the osteoid to ossify due to vitamin D deficiency. Symptoms start about the age of one. The child is small for age and there is a history of failure to thrive. Bony deformities include bowing of the femur and tibia, a large head, deformity of the chest wall with thickening of the costochondral junction (rickettary rosary), and a transverse sulcus in the chest caused by the pull of the diaphragm (Harrison’s sulcus). X- Rays show widening and cupping of the epiphysis of the long bones, most readily apparent in the wrist.

How well did you know this?
1
Not at all
2
3
4
5
Perfectly
440
Q

Which of the following are not true of Li-Fraumeni syndrome?

It consists of mutations to the p53 tumour suppressor gene

Is likely to be present in a teenager presenting with a liposarcoma

It has an autosomal dominant inheritance pattern

Affected individuals are unlikely to develop acute myeloid leukaemia

Adrenal malignancies are more common than in normal population

A

They are at high risk of developing leukaemia.

How well did you know this?
1
Not at all
2
3
4
5
Perfectly
441
Q

A 42 year old teacher is admitted with a fall. An x-ray confirms a fracture of the surgical neck of the humerus. Which nerve is at risk?

A.Median nerve

B.Ulnar nerve

C.Radial nerve

D.Posterior interosseous nerve

E.Anterior interosseous nerve

F.Musculocutaneous nerve

G.Axillary nerve

H.Brachial Trunks C5-6

I.Brachial trunks C6-7

J.Brachial Trunks C8-T1

A

Axillary nerve

The Axillary nerve winds around the bone at the neck of the humerus. The axillary nerve is also at risk during shoulder dislocation.

How well did you know this?
1
Not at all
2
3
4
5
Perfectly
442
Q

A 32 year old window cleaner is admitted after falling off the roof. He reports that he had slipped off the top of the roof and was able to cling onto the gutter for a few seconds. The patient has Horner’s syndrome.

A.Median nerve

B.Ulnar nerve

C.Radial nerve

D.Posterior interosseous nerve

E.Anterior interosseous nerve

F.Musculocutaneous nerve

G.Axillary nerve

H.Brachial Trunks C5-6

I.Brachial trunks C6-7

J.Brachial Trunks C8-T1

A

Brachial Trunks C8-T1

The patient has a Klumpke’s paralysis involving brachial trunks C8-T1. Classically there is weakness of the hand intrinsic muscles. Involvement of T1 may cause a Horner’s syndrome. It occurs as a result of traction injuries or during delivery.

How well did you know this?
1
Not at all
2
3
4
5
Perfectly
443
Q

A 32 year old rugby player is hit hard on the shoulder during a rough tackle. Clinically his arm is hanging loose on the side. It is pronated and medially rotated.

A.Median nerve

B.Ulnar nerve

C.Radial nerve

D.Posterior interosseous nerve

E.Anterior interosseous nerve

F.Musculocutaneous nerve

G.Axillary nerve

H.Brachial Trunks C5-6

I.Brachial trunks C6-7

J.Brachial Trunks C8-T1

A

Brachial Trunks C5-6

The patient has an Erb’s palsy involving brachial trunks C5-6.

How well did you know this?
1
Not at all
2
3
4
5
Perfectly
444
Q

A 24 year old motorist is involved in a road traffic accident in which he collides with the wall of a tunnel in a head on car crash, speed 85mph. He is wearing a seatbelt and the airbags have deployed. When rescuers arrive he is lucid and conscious and then dies suddenly.

A.Tension pneumothorax

B.Haemopericardium

C.Haemothorax

D.Aortic transection

E.Ruptured spleen

F.Duodeno-jejunal flexure disruption

G.Aorto iliac disruption

H.Ileo-colic junction disruption

A

Aortic transection

Aortic transections typically occur distal to the ligamentum arteriosum. A temporary haematoma may prevent the immediate death that usually occurs. This is a deceleration injury. A widened mediastinum may be seen on x-ray.

How well did you know this?
1
Not at all
2
3
4
5
Perfectly
445
Q

A 30 year old women is involved in a road traffic accident she is a passenger in a car involved in a head on collision with another vehicle. Her car is travelling at 60mph. She has been haemodynamically stable throughout with only minimal tachycardia. On examination she has marked abdominal tenderness and a large amount of intra abdominal fluid on CT scan

A.Tension pneumothorax

B.Haemopericardium

C.Haemothorax

D.Aortic transection

E.Ruptured spleen

F.Duodeno-jejunal flexure disruption

G.Aorto iliac disruption

H.Ileo-colic junction disruption

A

Duodeno-jejunal flexure disruption

This is another site of sudden deceleration injury. Given the large amount of free fluid, if it were blood, then a greater degree of haemodynamic instability would be expected.

How well did you know this?
1
Not at all
2
3
4
5
Perfectly
446
Q

A 17 year old boy is involved in a motorcycle accident in which he is thrown from his motorcycle. On admission he has distended neck veins and a weak pulse. The trachea is central.

A.Tension pneumothorax

B.Haemopericardium

C.Haemothorax

D.Aortic transection

E.Ruptured spleen

F.Duodeno-jejunal flexure disruption

G.Aorto iliac disruption

H.Ileo-colic junction disruption

A

Haemopericardium

This is most likely a cardiac tamponade produced by haemopericardium. As little as 100ml of blood may result in tamponade as the pericardial sac is not distensible. Diagnosis is suggested by muffled heart sounds, paradoxical pulse and jugular vein distension.

How well did you know this?
1
Not at all
2
3
4
5
Perfectly
447
Q

What is the typical stroke volume in a resting 70 Kg man?

10ml

150ml

125ml

45ml

70ml

A

70ml

Stroke volumes range from 55-100ml.

How well did you know this?
1
Not at all
2
3
4
5
Perfectly
448
Q

Which of the following statements relating to parathyroid neoplasms is incorrect?

15% of cases are due to parathyroid carcinoma

80% of cases are due to parathyroid adenomas

Parathyroid adenomas are often encapsulated

10% of parathyroid adenomas develop in ectopically located glands

85% of cases of primary hyperparathyroidism are due to solitary adenomas

A

Parathyroid carcinomas account for up to 5% of tumours. Adenomas are often encapsulated. Lesions that are fibrotic and densely adherent to the gland may be a carcinoma. 85% cases of primary hyperparathyroidism are due to a single adenoma and this is the reason some surgeons favour a focused parathyroidectomy.

How well did you know this?
1
Not at all
2
3
4
5
Perfectly
449
Q

A 20 year old girl presents with a thyroid cancer, she is otherwise well with no significant family history. On examination she has a nodule in the left lobe of the thyroid with a small discrete mass separate from the gland itself. Which of the following is the most likely cause?

Follicular carcinoma

Anaplastic carcinoma

Medullary carcinoma

Papillary carcinoma

B Cell Lymphoma

A

Papillary carcinoma is the most common subtype and may cause lymph node metastasis (mass separate from the gland itself) that is rare with follicular tumours. Anaplastic carcinoma would cause more local symptoms and would be rare in this age group.

How well did you know this?
1
Not at all
2
3
4
5
Perfectly
450
Q

What are the most likely effects of the release of vasopressin from the pituitary?

Vasoconstriction of the afferent glomerular arteriole

Increased permeability of the mesangial cells to glucose

Reduced permeability of the inner medullary portion of the collecting duct to urea

Increased secretion of aldosterone from the macula densa

Increased water permeability of the distal tubule cells of the kidney

A

Vasopressin, also known as antidiuretic hormone (ADH), is a crucial hormone released from the posterior pituitary gland. Let’s explore its effects:
Vasoconstriction of the Afferent Glomerular Arteriole: True. Vasopressin acts on the afferent arteriole of the glomerulus in the kidneys, causing vasoconstriction. This constriction reduces blood flow into the glomerulus, helping to regulate glomerular filtration rate (GFR) and maintain blood pressure1.
Increased Permeability of Mesangial Cells to Glucose: False. Vasopressin does not directly affect glucose permeability in mesangial cells.
Reduced Permeability of the Inner Medullary Portion of the Collecting Duct to Urea: True. Vasopressin increases the permeability of the collecting ducts in the kidney, allowing for greater reabsorption of water. This process concentrates urine and reduces urea loss2.
Increased Secretion of Aldosterone from the Macula Densa: False. Aldosterone is primarily regulated by the zona glomerulosa of the adrenal cortex, not vasopressin.
Increased Water Permeability of Distal Tubule Cells of the Kidney: True. Vasopressin acts on the distal tubules and collecting ducts to enhance water reabsorption. It promotes the insertion of aquaporin-2 channels in the tubular cells, allowing water to move from the tubules back into the bloodstream1.
In summary, vasopressin primarily affects water balance by regulating water reabsorption in the kidneys, leading to concentrated urine and maintaining blood pressure.

The most likely effect of vasopressin (antidiuretic hormone) is increased water permeability of the distal tubule cells of the kidney. Vasopressin acts on the distal tubules and collecting ducts, promoting the insertion of aquaporin-2 channels in the tubular cells. This allows water to move from the tubules back into the bloodstream, leading to water reabsorption and concentrated urine

How well did you know this?
1
Not at all
2
3
4
5
Perfectly
451
Q

Which of the following hormones is mainly responsible for sodium - potassium exchange in the salivary ducts?

Vasopressin

Angiotensin I

Aldosterone

Somatostatin

Cholecystokinin

A

Aldosterone is responsible for regulating ion exchange in salivary glands. It acts on a sodium / potassium ion exchange pump.It is a mineralocorticoid hormone derived from the zona glomerulosa of the adrenal gland.

How well did you know this?
1
Not at all
2
3
4
5
Perfectly
452
Q

A 44 year old lady presents with a mass in the upper outer quadrant of her right breast. Imaging, histology and clinical examination confirm a 1.5cm malignant mass lesion with no clinical evidence of axillary nodal disease.

A.Simple mastectomy alone

B.Radical mastectomy alone

C.Simple mastectomy and sentinel lymph node biopsy

D.Wide local excision and sentinel lymph node biopsy

E.Simple mastectomy and axillary node clearance

F.Radical mastectomy and axillary node clearance

G.Wide local excision and axillary node clearance

H.Wide local excision alone

A

Wide local excision and sentinel lymph node biopsy

A small peripheral lesion such as this would usually be suitable for breast conserving surgery. Since imaging and clinical examination is not suspicious for axillary disease, a sentinel lymph node biopsy should be performed.

How well did you know this?
1
Not at all
2
3
4
5
Perfectly
453
Q

A 44 year old lady presents with a mass lesion in the upper outer quadrant of the left breast. On clinical examination she has a 2cm mass lesion which on core biopsy is demonstrated to have invasive ductal carcinoma. An FNA of a bulky axillary lymph node contains malignant cells.

A.Simple mastectomy alone

B.Radical mastectomy alone

C.Simple mastectomy and sentinel lymph node biopsy

D.Wide local excision and sentinel lymph node biopsy

E.Simple mastectomy and axillary node clearance

F.Radical mastectomy and axillary node clearance

G.Wide local excision and axillary node clearance

H.Wide local excision alone

A

Wide local excision and axillary node clearance

Although the primary lesion is small enough for breast conserving surgery, the presence of overt axillary lymph node metastasis will attract a recommendation for axillary node clearance

How well did you know this?
1
Not at all
2
3
4
5
Perfectly
454
Q

A 39 year old lady presents with a mass lesion in her right breast. Clinical examination, biopsy and imaging confirm a 2.5 cm lesion in the upper inner quadrant of her right breast and a 1.5 cm lesion at the central aspect of the same breast. Her axilla shows lymphadenopathy and a fine needle aspirate from the node shows malignant cells.

A.Simple mastectomy alone

B.Radical mastectomy alone

C.Simple mastectomy and sentinel lymph node biopsy

D.Wide local excision and sentinel lymph node biopsy

E.Simple mastectomy and axillary node clearance

F.Radical mastectomy and axillary node clearance

G.Wide local excision and axillary node clearance

H.Wide local excision alone

A

Simple mastectomy and axillary node clearance

A combination of established axillary disease and multifocal invasive lesions attracts an indication for mastectomy and axillary clearance. A radical mastectomy is less frequently indicated in modern surgical practice, disease that is locally advanced is often best downstaged using medical therapy, rather than embarking on the operations for breast cancer that were first popularised over 100 years ago.

How well did you know this?
1
Not at all
2
3
4
5
Perfectly
455
Q

A 73 year old male presents with a collapse and is brought to the emergency department. On examination he has a cold, painful left hand and forearm.

A.Proximal brachial artery occlusion secondary to atheroma

B.Distal brachial artery occlusion secondary to atheroma

C.Axillary artery embolus

D.Axillary vein thrombosis

E.Cervical rib

F.Raynaud’s disease

G.Rheumatoid disease

A

Sudden arterial embolus will affect the axillary artery in up to 30% cases. Because of the acute nature of the condition there is no time for the development of a collateral circulation so the limb is usually pale and painful. Emboli occur usually occur as a result of atrial fibrillation. Fast atrial fibrillation can cause syncope and an acute embolus.

How well did you know this?
1
Not at all
2
3
4
5
Perfectly
456
Q

A 23 year old man presents with intermittent symptoms of altered sensation in his arm and discomfort when he uses his hands. He works as an electrician and his symptoms are worst when he is fitting light fixtures.

A.Proximal brachial artery occlusion secondary to atheroma

B.Distal brachial artery occlusion secondary to atheroma

C.Axillary artery embolus

D.Axillary vein thrombosis

E.Cervical rib

F.Raynaud’s disease

G.Rheumatoid disease

A

Cervical rib

Compression of the thoracic outlet by the fibrous band of the “rib” can result in both neurological and circulatory compromise. When manual tasks are performed in which the hand works overhead the signs and symptoms will be maximal and this is the basis of Adsons test.

How well did you know this?
1
Not at all
2
3
4
5
Perfectly
457
Q

A 19 year old lady presents with recurrent episodes of pain in her hands. She notices that her symptoms are worst in cold weather. When she gets the pain she notices that her hands are very pale, they then become dark blue in colour.
A.Proximal brachial artery occlusion secondary to atheroma

B.Distal brachial artery occlusion secondary to atheroma

C.Axillary artery embolus

D.Axillary vein thrombosis

E.Cervical rib

F.Raynaud’s disease

G.Rheumatoid disease

A

Raynaud’s disease

Raynauds disease is characterised by a series of colour changes and discomfort is often present. The young age at presentation coupled with the absence of a smoking history (in most cases) makes occlusive disease unlikely.

How well did you know this?
1
Not at all
2
3
4
5
Perfectly
458
Q

What is the most common site for upper limb emboli to lodge?
A-Brachial artery
B-Axillary artery
C-Radial artery
D-Ulnar artery

A

Brachial artery

How well did you know this?
1
Not at all
2
3
4
5
Perfectly
459
Q

Which muscle does not insert on the medial surface of the greater trochanter?

Gemelli

Obturator internus

Piriformis

Quadratus femoris

Obturator externus

A

The quadratus femoris fibres pass laterally to be inserted into the quadrate tubercle on the intertrochanteric crest of the femur. The other muscles all insert on the trochanteric fossa lying medial to the greater trochanter. The mnemonic for the muscles that insert on the medial surface of the greater trochanter is indeed “POGO”:
P: Piriformis
O: Obturator internus
G: Gemelli
O: Obturator externus

How well did you know this?
1
Not at all
2
3
4
5
Perfectly
460
Q

Which muscle is not innervated by the superior gluteal nerve?

Gluteus medius

Gluteus minimis

Tensor fascia lata

Obturator externus

A

Obturator externus

How well did you know this?
1
Not at all
2
3
4
5
Perfectly
461
Q

During a radical gastrectomy for carcinoma of the stomach the surgeons remove the omentum. What is the main source of its blood supply?

Ileocolic artery

Superior mesenteric artery

Gastroepiploic artery

Middle colic artery

Inferior mesenteric artery

A

The vessels supplying the omentum are the omental branches of the right and left gastro-epiploic arteries. The colonic vessels are not responsible for the arterial supply to the omentum. The left gastro-epiploic artery is a branch of the splenic artery and the right gastro-epiploic artery is a terminal branch of the gastroduodenal artery.

How well did you know this?
1
Not at all
2
3
4
5
Perfectly
462
Q

A 20 year old male presents with a tense, swollen knee joint. There is no history of antecedent trauma. On examination the joint is tense and swollen but there is no sign of injury. Plain x-rays show no fracture or arthritis. What is the most likely explanation?

Rupture of the anterior cruciate ligament

Rupture of the medial collateral ligament

Tibial plateau fracture

Haemophilia A

von Willebrands disease

A

Haemarthrosis without trauma is typically a feature of haemophilia A and B

Without a history of trauma, ligamentous rupture or tibial plateau fractures would be unusual.
Haemarthroses may occur in 45% of patients with severe von Willebrands disease. However, this is rare

How well did you know this?
1
Not at all
2
3
4
5
Perfectly
463
Q

A 38 year old lady is due to undergo a parathyroidectomy for hyperparathyroidism. At operation the inferior parathyroid gland is identified as being enlarged. A vessel is located adjacent to the gland laterally. This vessel is most likely to be the:

External carotid artery

Common carotid artery

Internal carotid artery

External jugular vein

None of the above

A

The common carotid artery is a lateral relation of the inferior parathyroid.

How well did you know this?
1
Not at all
2
3
4
5
Perfectly
464
Q

A 45 year old man has a long femoral line inserted to provide CVP measurements. The catheter passes from the common iliac vein into the inferior vena cava. At which of the following vertebral levels will this occur?

L5

L4

S1

L3

L2

A

The common iliac veins fuse with the IVC at L5.

How well did you know this?
1
Not at all
2
3
4
5
Perfectly
465
Q

Which of the following is not considered a major branch of the descending thoracic aorta?

Bronchial artery

Mediastinal artery

Inferior thyroid artery

Posterior intercostal artery

Oesophageal artery

A

The inferior thyroid artery is usually derived from the thyrocervical trunk, a branch of the subclavian artery.

Thoracic aorta

Origin T4
Terminates T12
Relations
Anteriorly (from top to bottom)-root of the left lung, the pericardium, the oesophagus, and the diaphragm
Posteriorly-vertebral column, azygos vein
Right- azygos veins, thoracic duct
Left- left pleura and lung
Branches
Lateral segmental branches: Posterior intercostal arteries
Lateral visceral: Bronchial arteries supply bronchial walls and lung excluding the alveoli
Midline branches: Oesophageal arteries

How well did you know this?
1
Not at all
2
3
4
5
Perfectly
466
Q

Where are the greatest proportion of musculi pectinati found?

Right ventricle

Left ventricle

Right atrium

Pulmonary valve

Aortic valve

A

The musculi pectinati are found in the atria, hence the reason that the atrial walls in the right atrium are irregular anteriorly.
The musculi pectinati of the atria are internal muscular ridges on the anterolateral surface of the chambers and they are only present in the area derived from the embryological true atrium.

Heart anatomy

The walls of each cardiac chamber comprise:
Epicardium
Myocardium
Endocardium

Cardiac muscle is attached to the cardiac fibrous skeleton.

Relations
The heart and roots of the great vessels within the pericardial sac are related to the posterior aspect of the sternum, medial ends of the 3rd to 5th ribs on the left and their associated costal cartilages. The heart and pericardial sac are situated obliquely two thirds to the left and one third to the right of the median plane.

The pulmonary valve lies at the level of the left third costal cartilage.
The mitral valve lies at the level of the fourth costal cartilage.

Coronary sinus
This lies in the posterior part of the coronary groove and receives blood from the cardiac veins. The great cardiac vein lies at its left and the middle and small cardiac veins lie on its right. The smallest cardiac vein (anterior cardiac vein) drains into the right atrium directly.

Aortic sinus
Right coronary artery arises from the right aortic sinus, the left is derived from the left aortic sinus, which lies posteriorly.

Features of the left ventricle as opposed to the right

Structure Left Ventricle
A-V Valve Mitral (double leaflet)
Walls Twice as thick as right
Trabeculae carnae Much thicker and more numerous

Right coronary artery
The RCA supplies:
Right atrium
Diaphragmatic part of the right ventricle
Usually the posterior third of the interventricular septum
The sino atrial node (60% cases)
The atrio ventricular node (80% cases)

Left coronary artery
The LCA supplies:
Left atrium
Most of left ventricle
Part of the right ventricle
Anterior two thirds of the inter ventricular septum
The sino atrial node (remaining 40% cases)

Innervation of the heart
Autonomic nerve fibres from the superficial and deep cardiac plexus. These lie anterior to the bifurcation of the trachea, posterior to the ascending aorta and superior to the bifurcation of the pulmonary trunk. The parasympathetic supply to the heart is from presynaptic fibres of the vagus nerves.

Valves of the heart
Mitral valve Aortic valve Pulmonary valve Tricuspid valve
2 cusps 3 cusps 3 cusps 3 cusps
First heart sound Second heart sound Second heart sound First heart sound
1 anterior cusp 2 anterior cusps 2 anterior cusps 2 anterior cusps
Attached to chordae tendinae No chordae No chordae Attached to chordae tendinae

How well did you know this?
1
Not at all
2
3
4
5
Perfectly
467
Q

The anterior interosseous nerve is a branch of which of the following?

Ulnar nerve

Superficial branch of the radial nerve

Axillary nerve

Deep branch of the radial nerve

Median nerve

A

Anterior interosseous nerve

The anterior interosseous nerve (volar interosseous nerve) is a branch of the median nerve that supplies the deep muscles on the front of the forearm, except the ulnar half of the flexor digitorum profundus.

It accompanies the anterior interosseous artery along the anterior of the interosseous membrane of the forearm, in the interval between the flexor pollicis longus and flexor digitorum profundus, supplying the whole of the former and (most commonly) the radial half of the latter, and ending below in the pronator quadratus and wrist joint.

Innervation
The anterior interosseous nerve classically innervates 2.5 muscles:

Flexor pollicis longus
Pronator quadratus
The radial half of flexor digitorum profundus (the lateral two out of the four tendons).

These muscles are in the deep level of the anterior compartment of the forearm.

How well did you know this?
1
Not at all
2
3
4
5
Perfectly
468
Q

Which of the structures listed below are most closely related to the axillary nerve within the quadrangular space?

Posterior circumflex humeral vessels

Axillary artery

Anterior circumflex humeral vessels

Radial artery

Acromiothoracic artery

A

The posterior circumflex humeral vessels which are branches of the axillary artery are related to the axillary nerve within the quadrangular space.

Axillary nerve

Terminal branch of the posterior cord of the brachial plexus
Root values C5 and C6
Descends posterior to the axillary artery at the lower border of subscapularis and then passes through quadrangular space with the posterior circumflex humeral vessels
Divides into anterior and posterior branches
Innervates deltoid muscle and small patch of skin over deltoid

How well did you know this?
1
Not at all
2
3
4
5
Perfectly
469
Q

Which of the following is not associated with Epstein-Barr virus?

Burkitt’s lymphoma

Post-transplantation lymphoma

Hodgkin’s lymphoma

Nasopharyngeal carcinoma

Mycosis fungoides

A

Mycosis fungoides

Oncoviruses

Viruses which cause cancer
These may be detected on blood test and prevented by vaccine

These are the main types of oncoviruses and their diseases:

Oncovirus Cancer
Epstein-Barr virus Burkitt’s lymphoma
Hodgkin’s lymphoma
Post transplant lymphoma
Nasopharyngeal carcinoma
Human papillomavirus 16/18 Cervical cancer
Anal cancer
Penile cancer
Vulval cancer
Oropharyneal cancer
Human herpes virus 8 Kaposi’s sarcoma
Hepatitis B virus Hepatocellular carcinoma
Hepatitis C virus Hepatocellular carcinoma
Human T-lymphotropic virus 1 Tropical spastic paraparesis
Adult T cell leukaemia

How well did you know this?
1
Not at all
2
3
4
5
Perfectly
470
Q

An 80 year old lady is brought to the emergency department by her carers. She has been unwell for the past few days. On examination, she has a cold, pulseless leg with fixed mottling, below the knee. A duplex scan shows a stenosis of the profunda femoris and popliteal artery with no flow distal to this. What is the most appropriate course of action.

Femoro-femoro cross over graft

Axillo- femoral bypass graft

Trans femoral amputation

Gritti Stokes amputation

Administration of intravenous unfractionated heparin

A

Fixed mottling is a sign of an unsalvageable limb and mandates either amputation or palliation. A through knee amputation (Gritti Stokes) is unlikely to heal in this setting.

Acute limb ischaemia

  • Thrombosis of a pre-existing site of atherosclerosis is the commonest cause of acute limb ischaemia
    Acute thrombosis of popliteal aneurysms poses the greatest threat to the limb
    Sudden occlusion of a large proximal vessel results in the typical appearances of acute limb ischaemia

Clinical appearances
Less than 6 hours = White leg
At 6 -12 hours = Mottled limb with blanching on pressure
More than 12-24 hours = Fixed mottling

Management of acutely ischaemic leg
Clinical picture Treatment
White leg with sensorimotor deficit Surgery and embolectomy
Dusky leg, mild anaesthesia Angiography
Fixed mottling Primary amputation

Role of thrombolysis
Intra arterial thrombolysis is better than peripheral thrombolysis
Mainly indicated in acute on chronic thrombosis
Avoid if within 2 months of CVA or 2 weeks of surgery
Aspiration of clot may improve success rate if the thrombosis is large

Surgery
Both groins should be prepared
Transverse arteriotomy is easier to close
Poor inflow should be managed with iliac trawl- if this fails to improve then consider a femoro-femoral cross over or axillo-femoral cross over.
A check angiogram should be performed on table and prior to closure
Systemic heparinisation should follow surgery
Fasciotomy should be considered if the time between onset and surgery exceeds 6 hours

How well did you know this?
1
Not at all
2
3
4
5
Perfectly
471
Q

A 17 year old man undergoes an elective right hemicolectomy. Post operatively he receives a total of 6 litres of 0.9% sodium chloride solution, over 24 hours. Which of the following complications may ensue?

Hyperchloraemic acidosis

Hypochloraemic alkalosis

Hyperchloraemic alkalosis

Acute renal failure

None of the above

A

Excessive infusions of any intravenous fluid carry the risk of development of tissue oedema and potentially cardiac failure. Excessive administration of sodium chloride is a recognised cause of hyperchloraemic acidosis and therefore Hartmans solution may be preferred where large volumes of fluid are to be administered.
Post operative fluid management

Composition of commonly used intravenous fluids mmol-1

Na K Cl Bicarbonate Lactate
Plasma 137-147 4-5.5 95-105 22-25 -
0.9% Saline 153 - 153 - -
Dextrose / saline 30.6 - 30.6 - -
Hartmans 130 4 110 - 28

Post operative fluid management
In the UK the GIFTASUP and NICE (CG174 2017) guidelines (see reference below) were devised to try and provide some consensus guidance as to how intravenous fluids should be administered. Some time ago it was a commonly held belief that little harm would occur as a result of excessive administration of normal saline and many oliguric post operative patients received enormous quantities of IV fluids. As a result they developed hyperchloraemic acidosis. With greater understanding of this potential complication, the use of electrolyte balanced solutions (Ringers lactate/ Hartmans) is now favored over normal saline.
The other guidance includes:
Fluids given should be documented clearly and easily available
Assess the patient’s fluid status when they leave theatre
If a patient is haemodynamically stable and euvolaemic, aim to restart oral fluid intake as soon as possible
Review patients whose urinary sodium is < 20
If a patient is oedematous, hypovolaemia if present should be treated first. This should then be followed by a negative balance of sodium and water, monitored using urine Na excretion levels
Solutions such as Dextran 70 should be used in caution in patients with sepsis as there is a risk of developing acute renal injury

How well did you know this?
1
Not at all
2
3
4
5
Perfectly
472
Q

A 52 year female post hysterectomy attends clinic. She reports pain and reduced sensation over the medial aspect of her thigh. Clinically thigh adduction is weak. What is the most likely nerve injury?

Obturator nerve

Sciatic nerve

Femoral nerve

L3 cord compression

Deep peroneal nerve

A

The obturator nerve supplies sensation to the medial aspect of the thigh and causes adduction and internal rotation of the thigh.
Injury occurs during pelvic or abdominal surgery.
L3 cord compression is unlikely.
Obturator nerve

The obturator nerve arises from L2, L3 and L4 by branches from the ventral divisions of each of these nerve roots. L3 forms the main contribution and the second lumbar branch is occasionally absent. These branches unite in the substance of psoas major, descending vertically in its posterior part to emerge from its medial border at the lateral margin of the sacrum. It then crosses the sacroiliac joint to enter the lesser pelvis, it descends on obturator internus to enter the obturator groove. In the lesser pelvis the nerve lies lateral to the internal iliac vessels and ureter, and is joined by the obturator vessels lateral to the ovary or ductus deferens.

Supplies
Medial compartment of thigh
Muscles supplied: external obturator, adductor longus, adductor brevis, adductor magnus (not the lower part-sciatic nerve), gracilis
The cutaneous branch is often absent. When present, it passes between gracilis and adductor longus near the middle part of the thigh, and supplies the skin and fascia of the distal two thirds of the medial aspect.

Obturator canal
Connects the pelvis and thigh: contains the obturator artery, vein, nerve which divides into anterior and posterior branches.

How well did you know this?
1
Not at all
2
3
4
5
Perfectly
473
Q

A 28 year old lady with known von Willebrands disease is bleeding following the excision of a sebaceous cyst. Administration of which of the following agents is most likely to be beneficial?

Desmopressin

Factor IX concentrate

Factor VII concentrate

Factor X concentrate

Vasopressin

A

Desmopressin is useful in managing the bleeding from the commonest type of vWD. Vasopressin is less likely to be disease specific and at most would have a vasoconstrictor approach which is likely to be of brief duration.

von Willebrands disease

  • Most common inherited bleeding disorder
    All vWD is caused by mutations in the gene for von Willebrand factor. von Willebrand factor is an adhesive glycoprotein that is secreted by endothelium and megakaryocytes
    von Willebrand factor promotes platelet adhesion to damaged endothelium and other platelets. It is also involved in the transport and stabilization of factor VIII
    There are 7 subtypes of von Willebrand disease. The commonest is type I (autosomal dominant) which accounts for 80% of cases, type 2vWD (autosomal dominant or recessive) accounts for 15% of cases
    There is a significant spectrum of severity ranging from spontaneous bleeding and epistaxis through to troublesome excessive bleeding following minor procedures
    The test that is most commonly used are von willebrand factor assays
    Treatments include administration of tranexamic acid for minor cases undergoing minor procedures. More significant bleeding or more significant procedures respond well to DDAVP. This is most effective in type I, less effective in type 2 and contraindicated in type 2B. Patients with type 3 disease do not respond to DDAVP as they lack the ability to secrete vWF
    Individuals who cannot have DDAVP or in whom it is contra indicated usually receive factor VIII concentrates containing vWF
How well did you know this?
1
Not at all
2
3
4
5
Perfectly
474
Q

The following are true of the femoral nerve, except:

It is derived from L2, L3 and L4 nerve roots

It supplies sartorius

It supplies quadriceps femoris

It gives cutaneous innervations via the saphenous nerve

It supplies adductor longus

A

Adductor longus is supplied by the obturator nerve.

Femoral nerve

Root values L2, 3, 4
Innervates
Pectineus
Sartorius
Quadriceps femoris
Vastus lateralis/medialis/intermedius
Rectus femoris
Branches
Medial cutaneous nerve of thigh
Saphenous nerve
Intermediate cutaneous nerve of thigh

Path
Penetrates psoas major and exits the pelvis by passing under the inguinal ligament to enter the femoral triangle, lateral to the femoral artery and vein.

Mnemonic for femoral nerve supply

(don’t) M I S V Q Scan for PE
M edial cutaneous nerve of the thigh
I ntermediate cutaneous nerve of the thigh
S aphenous nerve

V astus
Q uadriceps femoris
S artorius

PE ectineus

How well did you know this?
1
Not at all
2
3
4
5
Perfectly
475
Q

A 2 week old baby is referred to the surgical team by the paediatricians. They are concerned because the child has a painful area of macerated tissue at the site of the umbilicus. On examination, a clear- yellowish fluid is seen to be draining from the umbilicus when the baby cries. What is the most likely diagnosis?

Omphalitis

Umbilical granuloma

Persistent vitello-intestinal duct

Patent urachus

Isolated cellular remnants

A

A patent urachus will present with umbilical urinary discharge. The skin may become macerated if not properly cared for. The discharge is most likely to be present when intra-abdominal pressure is raised. It is associated with posterior urethral valves.

Paediatric umbilical disorders

Embryology
During development the umbilicus has two umbilical arteries and one umbilical vein. The arteries are continuous with the internal iliac arteries and the vein is continuous with the falciform ligament (ductus venosus). After birth the cord dessicates and separates and the umbilical ring closes.

Umbilical hernia
Up to 20% of neonates may have an umbilical hernia, it is more common in premature infants. The majority of these hernias will close spontaneously (may take between 12 months and three years). Strangulation is rare.

Paraumbilical hernia
These are due to defects in the linea alba that are in close proximity to the umbilicus. The edges of a paraumbilical hernia are more clearly defined than those of an umbilical hernia. They are less likely to resolve spontaneously than an umbilical hernia.

Omphalitis
This condition consists of infection of the umbilicus. Infection with Staphylococcus aureus is the commonest cause. The condition is potentially serious as infection may spread rapidly through the umbilical vessels in neonates with a risk of portal pyaemia, and portal vein thrombosis. Treatment is usually with a combination of topical and systemic antibiotics.

Umbilical granuloma
These consist of cherry red lesions surrounding the umbilicus, they may bleed on contact and be a site of seropurulent discharge. Infection is unusual and they will often respond favorably to chemical cautery with topically applied silver nitrate.

Persistent urachus
This is characterised by urinary discharge from the umbilicus. It is caused by persistence of the urachus which attaches to the bladder. They are associated with other urogenital abnormalities.

Persistent vitello-intestinal duct
This will typically present as an umbilical discharge that discharges small bowel content. Complete persistence of the duct is a rare condition. Much more common is the persistence of part of the duct (Meckels diverticulum). Persistent vitello-intestinal ducts are best imaged using a contrast study to delineate the anatomy and are managed by laparotomy and surgical closure.

How well did you know this?
1
Not at all
2
3
4
5
Perfectly
476
Q

A 73 year old female is referred to the surgical clinic with an iron deficiency anaemia. As part of the diagnostic work up the doctor requests a serum ferritin level. Which of the conditions listed is most likely to lead to a falsely elevated result?

Locally perforated sigmoid colonic adenocarcinoma

Colonic angiodysplasia

Dieulafoy lesion of the stomach

Transitional cell carcinoma of the bladder

Endometrial adenocarcinoma

A

A locally perforated colonic tumour will typically cause an intense inflammatory response and if peritonitis is not present clinically then at the very least a localised abscess. This inflammatory process is the most likely (from the list) to falsely raise the serum ferritin level. Angiodysplasia and dieulafoy lesions are mucosal arteriovenous malformations and unlikely to result in considerable inflammatory activity.

Ferritin

Ferritin is an intracellular protein that binds iron and stores it to be released in a controlled fashion at sites where iron is required. Because iron and ferritin are bound the total body ferritin levels may be decreased in cases of iron deficiency anaemia. Measurement of serum ferritin levels can be useful in determining whether an apparently low haemoglobin and microcytosis is truly caused by an iron deficiency state.
Ferritin is an acute phase protein and may be synthesised in increased quantities in situations where inflammatory activity is ongoing. Falsely elevated results may therefore be encountered clinically and need to be taken in context of the clinical picture and full blood count results.

How well did you know this?
1
Not at all
2
3
4
5
Perfectly
477
Q

An 18 year old athlete attends orthopaedic clinic reporting pain and swelling over the medial aspect of the knee joint. The pain occurs when climbing the stairs, but is not present when walking on flat ground. Clinically there is pain over the medial, proximal tibia and the McMurray test is negative. What is the most likely cause of this patient’s symptoms?

Anterior cruciate ligament tear

Prepatellar bursitis

Medial meniscus injury

Pes Anserinus Bursitis

Fracture of tibia

A

Pes anserinus: GOOSE’S FOOT

Combination of sartorius, gracilis and semitendinous tendons inserting into the anteromedial proximal tibia.
Pes Anserinus Bursitis is common in sportsmen due to overuse injuries. The main sign is of pain in the medial proximal tibia. As the McMurray test is negative, medial meniscal injury is excluded.

Sartorius

  • Longest strap muscle in the body
    Most superficial muscle in the anterior compartment of the thigh

Origin Anterior superior iliac spine
Insertion Medial surface of the of the body of the tibia (upper part). It inserts anterior to gracilis and semitendinosus
Nerve Supply Femoral nerve (L2,3 -root values for sartorius)
Action
Flexor of the hip and knee, slight abducts the thigh and rotates it laterally
It assists with medial rotation of the tibia on the femur. For example it would play a pivotal role in placing the right heel onto the left knee ( and vice versa)
Important relations The middle third of this muscle, and its strong underlying fascia forms the roof of the adductor canal , in which lie the femoral vessels, the saphenous nerve and the nerve to vastus medialis.

How well did you know this?
1
Not at all
2
3
4
5
Perfectly
478
Q

A patient is referred due to the development of a third nerve palsy associated with a headache. On examination, meningism is present. Which one of the following diagnoses needs to be urgently excluded?

Weber’s syndrome

Internal carotid artery aneurysm

Multiple sclerosis

Posterior communicating artery aneurysm

Anterior communicating artery aneurysm

A

Painful third nerve palsy = posterior communicating artery aneurysm
Given the combination of a headache and third nerve palsy it is important to exclude a posterior communicating artery aneurysm

Third nerve palsy

Features
eye is deviated ‘down and out’
ptosis
pupil may be dilated (sometimes called a ‘surgical’ third nerve palsy)

Causes
diabetes mellitus
vasculitis e.g. temporal arteritis, SLE
false localizing sign* due to uncal herniation through tentorium if raised ICP
posterior communicating artery aneurysm (pupil dilated)
cavernous sinus thrombosis
Weber’s syndrome: ipsilateral third nerve palsy with contralateral hemiplegia -caused by midbrain strokes
other possible causes: amyloid, multiple sclerosis

*this term is usually associated with sixth nerve palsies but it may be used for a variety of neurological presentations

How well did you know this?
1
Not at all
2
3
4
5
Perfectly
479
Q

Which of the following would be a sensible volume for maintenance intravenous fluids in a 3 day old term neonate?

50ml/ kg/ hour

50ml/ kg/ day

100ml/kg/hour

100ml/kg/day

200ml/kg/day

A

Calculate routine maintenance IV fluid rates for children and young people using the HollidaySegar formula (100 ml/kg/day for the first 10 kg of weight, 50 ml/kg/day for the next 10 kg and 20 ml/kg/day for the weight over 20 kg). Be aware that over a 24hour period, males rarely need more than 2500 ml and females rarely need more than 2000 ml of fluids.
From birth to day 1: 50-60 ml/kg/day.
Day 2: 70-80 ml/kg/day.
Day 3: 80-100 ml/kg/day.
Day 4: 100-120 ml/kg/day.
Days 5-8: 120-150 ml/kg/day.

Paediatric fluid management

Since 2000 there have been at least 4 reported deaths from fluid induced hyponatraemia in children. This led to the National Patient Safety Agency introducing revised guidelines in 2007. These have been reviewed and extensively updated by NICE in 2015 and further modified in 2020.
Indications for IV fluids include:
Resuscitation and circulatory support
Replacing on-going fluid losses
Maintenance fluids for children for whom oral fluids are not appropriate
Correction of electrolyte disturbances

Fluids to be avoided
Outside the neonatal period saline / glucose solutions should not be given. The greatest risk is with saline 0.18 / glucose 4% solutions. The report states that 0.45% saline / 5% glucose may be used. But preference should be given to isotonic solutions and few indications exist for this solution either.
The key point emphasised in the NICE guidelines in the avoidance of glucose containing solutions and instead of routinely giving glucose IV to children and neonates, the blood glucose levels should be monitored. In neonates in the first few days of life, sodium levels in the range of 131-154 mmol/l may be too high and a neonatologist consulted on a case by case basis.

Fluids to be used
If children and young people need IV fluids for routine maintenance, initially use isotonic crystalloids that contain sodium in the range 131 to 154 mmol/litre

Potassium should be added to maintenance fluids according patients plasma potassium levels (which should be monitored).
Blood glucose levels should be monitored in individuals at risk of hypoglycaemia

Intraoperative fluid management
If children and young people need IV fluids for routine maintenance, initially use isotonic crystalloids that contain sodium in the range 131 to 154 mmol/litre
Blood glucose levels should be monitored

Maintenance fluids
Weight Water requirement/kg/day Na mmol/kg/day K mmol/kg/day
First 10Kg body weight 100ml 2-4 1.5-2.5
Second 10Kg body weight 50ml 1-2 0.5-1.5
Subsequent Kg 20ml 0.5-1.0 0.2-0.7

Blood glucose will need to be monitored

How well did you know this?
1
Not at all
2
3
4
5
Perfectly
480
Q

A 38 year old man presents to the clinic with shoulder weakness. On examination he has an inability to initiate shoulder abduction. Which of the nerves listed below is least likely to be functioning normally?

Suprascapular nerve

Medial pectoral nerve

Axillary nerve

Median nerve

Radial nerve

A

Suprascapular nerve

The suprascapular nerve arises from the upper trunk of the brachial plexus. It lies superior to the trunks of the brachial plexus and passes inferolaterally parallel to them. It passes through the scapular notch, deep to trapezius. It innervates both supraspinatus and infraspinatus and initiates abduction of the shoulder. If damaged, patients may be able to abduct the shoulder by leaning over the affected side and deltoid can then continue to abduct the shoulder.

How well did you know this?
1
Not at all
2
3
4
5
Perfectly
481
Q

A 70 year old man undergoes a revision total hip replacement. 30 days post operatively the hip dislocates and pus is discharging from the wound. He is systemically unwell with a temperature of 38.5 and WCC 19. What is the most appropriate course of action?

Lay open wound and apply a VAC dressing

Hindquater amputation

Revision arthroplasty

Removal of metalwork and bone grafting

Removal of metalwork and implantation of local antibiotics

A

Removal of metal work implantation of gentamicin beads and delayed revision is the mainstay of managing this complication after 4 weeks. In earlier infections, linear exchanges covered by prolonged courses of antibiotics can be used.

Osteomyelitis

Infection of the bone

Causes
S aureus and occasionally Enterobacter or Streptococcus species
In sickle cell: Salmonella species

Clinical features
Erythema
Pain
Fever

Investigation
X-ray: lytic centre with a ring of sclerosis
Bone biopsy and culture

Treatment
Prolonged antibiotics
Sequestra may need surgical removal

How well did you know this?
1
Not at all
2
3
4
5
Perfectly
482
Q

A 39 year old man notices a swelling in his left hemiscrotum. On examination he has a left sided varicocele. The ipsilateral testis is normal on palpation. What is the most appropriate course of action?

Scrotal exploration and ligation of the varicocele

Abdominal ultrasound

Scrotal ultrasound

Left orchidectomy

Discharge

A

A left sided varicocele is a recognised presenting sign of a renal tumour occluding the renal vein (into which the left testicular vein drains). An abdominal ultrasound should be undertaken to exclude this. Surgery for uncomplicated varicocele is usually unnecessary.
Renal tumours

Renal cell carcinoma
Renal cell carcinoma is an adenocarcinoma of the renal cortex and is believed to arise from the proximal convoluted tubule. They are usually solid lesions, up to 20% may be multifocal, 20% may be calcified and 20% may have either a cystic component or be wholly cystic. They are often circumscribed by a pseudocapsule of compressed normal renal tissue. Spread may occur either by direct extension into the adrenal gland, renal vein or surrounding fascia. More distant disease usually occurs via the haematogenous route to lung, bone or brain.
Renal cell carcinoma comprise up to 85% of all renal malignancies. Males are more commonly affected than females and sporadic tumours typically affect patients in their sixth decade.
Patients may present with a variety of symptoms including; haematuria (50%), loin pain (40%), mass (30%) and up to 25% may have symptoms of metastasis.Less than 10% have the classic triad of haematuria, pain and mass.

Investigation
Many cases will present as haematuria and be discovered during diagnostic work up. Benign renal tumours are rare, so renal masses should be investigated with multislice CT scanning. Most tumours are also characterised with an arterial phase CT, particularly if they may be suitable for partial nephrectomy.

CT scanning of the chest and abdomen to detect distant disease should also be undertaken.

Routine bone scanning is not indicated in the absence of symptoms.

Biopsy should not be performed when a nephrectomy is planned but is mandatory before any ablative therapies are undertaken.

Assessment of the functioning of the contra lateral kidney.

Management
T1 lesions may be managed by partial nephrectomy and this gives equivalent oncological results to total radical nephrectomy. Partial nephrectomy may also be performed when there is inadequate reserve in the remaining kidney. Ablative techniques may also be considered for small T1 lesions in unfit patients. In general, outcomes are less favorable than with surgical resection.

For T2 lesions and above a radical nephrectomy is standard practice and this may be performed via a laparoscopic or open approach. Preoperative embolisation is not indicated nor is resection of uninvolved adrenal glands. During surgery early venous control is mandatory to avoid shedding of tumour cells into the circulation.

Patients with completely resected disease do not benefit from adjuvant therapy with either chemotherapy or biological agents. These should not be administered outside the setting of clinical trials.

Patients with transitional cell cancer will require a nephroureterectomy with disconnection of the ureter at the bladder.

How well did you know this?
1
Not at all
2
3
4
5
Perfectly
483
Q

A 50 year old man is admitted after falling from scaffolding. He has an open fracture of his tibia with a 15 cm wound. He is neurovascularly intact. What is the best initial course of action?

Intravenous antibiotics, photography and application of saline soaked gauze with impermeable dressing

Thorough wound debridement in the emergency department

Combined skeletal and soft tissue reconstruction on a scheduled operating list

Application of external fixator and conversion to internal fixation after two weeks

Immediate skeletal stabilisation and application of negative pressure dressing

A

Note the question asks for initial management

The initial management of open fractures should include administration of intravenous antibiotics, photography of wound and application of a sterile soaked gauze and impermeable film. The wound should only be handled to remove gross contamination. The patient is then likely to require definitive skeletal and soft tissue reconstruction.

Open fractures

The term open fracture refers to a disruption of the bony cortex associated with a breach in the overlying skin. Any wound that is present in the same limb as a fracture should be suspected as being representative of an open fracture. One of the main problems with open fractures is the associated injuries to the surrounding soft tissues. Whilst the skin is usually relatively resistant to trauma, underlying muscle can be damaged or devitalised, nerves, blood vessels and periosteum may all be disrupted the degree to which this occurs correlates with the severity of the injury and the outcome. These can be graded using the Gustilo and Anderson system (see below).

Grade Injury
1 Low energy wound <1cm
2 Greater than 1cm wound with moderate soft tissue damage
3 High energy wound > 10cm with extensive soft tissue damage
3 A (sub group of 3) Adequate soft tissue coverage
3 B (sub group of 3) Inadequate soft tissue coverage
3 C (sub group of 3) Associated arterial injury

In Type IIIc injuries, the mangled extremity scoring system (MESS) can help to predict the need for primary amputation.

Key points for initial management
Remove obvious contaminants from very contaminated wounds in the ED
Wound lavage should usually be undertaken in the OR
Early wound photography should be performed
Consider transfer of complex cases to centres that provide orthoplastic care
All wounds should be managed within 24 hours and high velocity ones within 12 hours, those with vascular compromise should be managed immediately
CT angiography is useful in delineating the extent of concommitant vascular injury

How well did you know this?
1
Not at all
2
3
4
5
Perfectly
484
Q

In relation to operating in the elderly which statement is false?

A 30 minute increment in operation length is associated with increase in mortality in patients over the age of 80

Hypoalbuminaemia is associated with increased mortality

Statins given preoperatively reduce perioperative cardiac events

Elevated brain (or B-type) natriuretic peptide (BNP) levels before undergoing non cardiac surgery is associated with high risk of cardiac mortality and all cause mortality

Beta blockers should be stopped acutely prior to surgery due to risk of perioperative hypotension

A

Beta blockers should not be stopped acutely prior to surgery as there may be a rebound effect associated with increased complications.

Brain natriuretic peptide is a neurohormone synthesized in the cardiac ventricles. Levels have been used to assess prognosis in heart failure and acute coronary syndromes. Preoperative elevated brain natriuretic peptide levels identify patients undergoing non cardiac surgery at high risk of cardiac mortality and all cause mortality.

All patients with peripheral vascular disease should take statins prior to vascular surgery as studies have shown a 50% risk reduction and a reduction in perioperative cardiac events.
Proactive care of older people undergoing surgery (POPS)

Comprehensive geriatric assessment
MDT assessment preoperatively
Main predictors of complications are co-morbidities cardiac disease and reduced functional capacity - preoperative assessment is the key to preventing adverse postoperative outcomes
Patients screened for risk factors (albumin <30, co morbidities)
Management plan made and disseminated to all involved
Patients education: pain relief, post op exercises, nutrition

Outcomes:
Fewer postoperative medical complications
Reduced length of stay by 4.5 days

How well did you know this?
1
Not at all
2
3
4
5
Perfectly
485
Q

With respect to the basilic vein, which statement is false?

Its deep anatomical location makes it unsuitable for use as an arteriovenous access site in fistula surgery

It originates from the dorsal venous network on the hand

It travels up the medial aspect of the forearm

Halfway between the shoulder and the elbow it lies deep to muscle

It is joined by the brachial vein to form the axillary vein

A

It is used in arteriovenous fistula surgery during a procedure known as a basilic vein transposition.

Basilic vein

The basilic and cephalic veins both provide the main pathways of venous drainage for the arm and hand. It is continuous with the palmar venous arch distally and the axillary vein proximally.

Path
Originates on the medial side of the dorsal venous network of the hand, and passes up the forearm and arm.
Most of its course is superficial.
Near the region anterior to the cubital fossa the vein joins the cephalic vein.
Midway up the humerus the basilic vein passes deep under the muscles.
At the lower border of the teres major muscle, the anterior and posterior circumflex humeral veins feed into it.
It is often joined by the medial brachial vein before draining into the axillary vein.

How well did you know this?
1
Not at all
2
3
4
5
Perfectly
486
Q

A 32 year old lady is admitted with weakness, visual disturbance and peri orbital pain. On examination, she is noted to have mydriasis and diminished direct response to light shone into the affected eye. The consensual response is preserved when light is shone into the unaffected eye. Which of the cranial nerves listed below is responsible for the diminished direct response?

Abducens

Oculomotor

Optic

Trigeminal

Hypoglossal

A

This describes a relative afferent pupillary defect (RAPD). RAPD is a defect in the direct response to light. It is due to damage in optic nerve or severe retinal disease. If an optic nerve lesion is present the affected pupil will not constrict to light when light is shone in the that pupil during the swinging flashlight test. However, it will constrict if light is shone in the other eye (consensual response).

The most likely cause for this is an optic neuritis (not really surgical!). Other causes include ischemic optic disease or retinal disease, severe glaucoma causing trauma to optic nerve and direct optic nerve damage (trauma, radiation, tumor).

Cranial nerves

Cranial nerve lesions
Olfactory nerve May be injured in basal skull fractures or involved in frontal lobe tumour extension. Loss of olfactory nerve function in relation to major CNS pathology is seldom an isolated event and thus it is poor localiser of CNS pathology.
Optic nerve Problems with visual acuity may result from intra ocular disorders. Problems with the blood supply such as amaurosis fugax may produce temporary visual distortion. More important surgically is the pupillary response to light. The pupillary size may be altered in a number of disorders. Nerves involved in the resizing of the pupil connect to the pretectal nucleus of the high midbrain, bypassing the lateral geniculate nucleus and the primary visual cortex. From the pretectal nucleus neurones pass to the Edinger - Westphal nucleus, motor axons from here pass along with the oculomotor nerve. They synapse with ciliary ganglion neurones; the parasympathetic axons from this then innervate the iris and produce miosis. The miotic pupil is seen in disorders such as Horner’s syndrome or opiate overdose.
Mydriasis is the dilatation of the pupil in response to disease, trauma, drugs (or the dark!). It is pathological when light fails to induce miosis. The radial muscle is innervated by the sympathetic nervous system. Because the parasympathetic fibres travel with the oculomotor nerve they will be damaged by lesions affecting this nerve (e.g. cranial trauma).
The response to light shone in one eye is usually a constriction of both pupils. This indicates intact direct and consensual light reflexes. When the optic nerve has an afferent defect the light shining on the affected eye will produce a diminished pupillary response in both eyes. Whereas light shone on the unaffected eye will produce a normal pupillary response in both eyes. This is referred to as the Marcus Gunn pupil and is seen in conditions such as optic neuritis. In a total CN II lesion shining the light in the affected eye will produce no response.
Oculomotor nerve The pupillary effects are described above. In addition it supplies all ocular muscles apart from lateral rectus and superior oblique. Thus the affected eye will be deviated inferolaterally. Levator palpebrae superioris may also be impaired resulting in impaired ability to open the eye.
Trochlear nerve The eye will not be able to look down.
Trigeminal nerve Largest cranial nerve. Exits the brainstem at the pons. Branches are ophthalmic, maxillary and mandibular. Only the mandibular branch has both sensory and motor fibres. Branches converge to form the trigeminal ganglion (located in Meckels cave). It supplies the muscles of mastication and also tensor veli palatine, mylohyoid, anterior belly of digastric and tensor tympani. The detailed descriptions of the various sensory functions are described in other areas of the website. The corneal reflex is important and is elicited by applying a small tip of cotton wool to the cornea, a reflex blink should occur if it is intact. It is mediated by: the naso ciliary branch of the ophthalmic branch of the trigeminal (sensory component) and the facial nerve producing the motor response. Lesions of the afferent arc will produce bilateral absent blink and lesions of the efferent arc will result in a unilateral absent blink.
Abducens nerve The affected eye will have a deficit of abduction. This cranial nerve exits the brainstem between the pons and medulla. It thus has a relatively long intra cranial course which renders it susceptible to damage in raised intra cranial pressure.
Facial nerve Emerges from brainstem between pons and medulla. It controls muscles of facial expression and taste from the anterior 2/3 of the tongue. The nerve passes into the petrous temporal bone and into the internal auditory meatus. It then passes through the facial canal and exits at the stylomastoid foramen. It passes through the parotid gland and divides at this point. It does not innervate the parotid gland. Its divisions are considered in other parts of the website. Its motor fibres innervate orbicularis oculi to produce the efferent arm of the corneal reflex. In surgical practice it may be injured during parotid gland surgery or invaded by malignancies of the gland and a lower motor neurone on the ipsilateral side will result.
Vestibulo-cochlear nerve Exits from the pons and then passes through the internal auditory meatus. It is implicated in sensorineural hearing loss. Individuals with sensorineural hearing loss will localise the sound in webers test to the normal ear. Rinnes test will be reduced on the affected side but should still work. These two tests will distinguish sensorineural hearing loss from conductive deafness. In the latter condition webers test will localise to the affected ear and Rinnes test will be impaired on the affected side. Surgical lesions affecting this nerve include CNS tumours and basal skull fractures. It may also be damaged by the administration of ototoxic drugs (of which gentamicin is the most commonly used in surgical practice).
Glossopharyngeal nerve Exits the pons just above the vagus. Receives sensory fibres from posterior 1/3 tongue, tonsils, pharynx and middle ear (otalgia may occur following tonsillectomy). It receives visceral afferents from the carotid bodies. It supplies parasympathetic fibres to the parotid gland via the otic ganglion and motor function to stylopharyngeaus muscle. The sensory function of the nerve is tested using the gag reflex.
Vagus nerve Leaves the medulla between the olivary nucleus and the inferior cerebellar peduncle. Passes through the jugular foramen and into the carotid sheath. Details of the functions of the vagus nerve are covered in the website under relevant organ sub headings.
Accessory nerve Exists from the caudal aspect of the brainstem (multiple branches) supplies trapezius and sternocleidomastoid muscles. The distal portion of this nerve is most prone to injury during surgical procedures.
Hypoglossal nerve Emerges from the medulla at the preolivary sulcus, passes through the hypoglossal canal. It lies on the carotid sheath and passes deep to the posterior belly of digastric to supply muscles of the tongue (except palatoglossus). Its location near the carotid sheath makes it vulnerable during carotid endarterectomy surgery and damage will produce ipsilateral defect in muscle function.

How well did you know this?
1
Not at all
2
3
4
5
Perfectly
487
Q

Which of the following fascial structures encases the apex of the lungs?

Waldeyers fascia

Sibsons fascia

Pretracheal fascia

Clavipectoral fascia

None of the above

A

Sibson’s fascia overlies the apices of both lungs
The suprapleural fascia (Sibson’s fascia) runs from C7 to the first rib and overlies the apex of both lungs.It lies between the parietal pleura and the thoracic cage.

Lung anatomy

The right lung is composed of 3 lobes divided by the oblique and transverse fissures. The left lung has two lobes divided by the oblique fissure.The apex of both lungs is approximately 4cm superior to the sterno-costal joint of the first rib. Immediately below this is a sulcus created by the subclavian artery.

Peripheral contact points of the lung
Base: diaphragm
Costal surface: corresponds to the cavity of the chest
Mediastinal surface: Contacts the mediastinal pleura. Has the cardiac impression. Above and behind this concavity is a triangular depression named the hilum, where the structures which form the root of the lung enter and leave the viscus. These structures are invested by pleura, which, below the hilum and behind the pericardial impression, forms the pulmonary ligament

Right lung
Above the hilum is the azygos vein; Superior to this is the groove for the superior vena cava and right innominate vein; behind this, and nearer the apex, is a furrow for the innominate artery. Behind the hilum and the attachment of the pulmonary ligament is a vertical groove for the oesophagus; In front and to the right of the lower part of the oesophageal groove is a deep concavity for the extrapericardiac portion of the inferior vena cava.

The root of the right lung lies behind the superior vena cava and the right atrium, and below the azygos vein.

The right main bronchus is shorter, wider and more vertical than the left main bronchus and therefore the route taken by most foreign bodies.
Left lung
Above the hilum is the furrow produced by the aortic arch, and then superiorly the groove accommodating the left subclavian artery; Behind the hilum and pulmonary ligament is a vertical groove produced by the descending aorta, and in front of this, near the base of the lung, is the lower part of the oesophagus.

The root of the left lung passes under the aortic arch and in front of the descending aorta.

Inferior borders of both lungs
6th rib in mid clavicular line
8th rib in mid axillary line
10th rib posteriorly
The pleura runs two ribs lower than the corresponding lung level.

Bronchopulmonary segments
Segment number Right lung Left lung
1 Apical Apical
2 Posterior Posterior
3 Anterior Anterior
4 Lateral Superior lingular
5 Medial Inferior lingular
6 Superior (apical) Superior (apical)
7 Medial basal Medial basal
8 Anterior basal Anterior basal
9 Lateral basal Lateral basal
10 Posterior basal Posterior basal

How well did you know this?
1
Not at all
2
3
4
5
Perfectly
488
Q

A 63 year old man is admitted with severe headache, nausea and recent epileptic fit. Fundoscopy shows papilloedema. He is also noted to have diplopia. Which of the cranial nerves listed accounts for the latter?

Abducens

Optic

Oculomotor

Facial

Trigeminal

A

Abducens. The long intracranial course of this nerve makes it susceptible to damage early in the course of raised ICP.

Cranial nerves

Cranial nerve lesions
Olfactory nerve May be injured in basal skull fractures or involved in frontal lobe tumour extension. Loss of olfactory nerve function in relation to major CNS pathology is seldom an isolated event and thus it is poor localiser of CNS pathology.
Optic nerve Problems with visual acuity may result from intra ocular disorders. Problems with the blood supply such as amaurosis fugax may produce temporary visual distortion. More important surgically is the pupillary response to light. The pupillary size may be altered in a number of disorders. Nerves involved in the resizing of the pupil connect to the pretectal nucleus of the high midbrain, bypassing the lateral geniculate nucleus and the primary visual cortex. From the pretectal nucleus neurones pass to the Edinger - Westphal nucleus, motor axons from here pass along with the oculomotor nerve. They synapse with ciliary ganglion neurones; the parasympathetic axons from this then innervate the iris and produce miosis. The miotic pupil is seen in disorders such as Horner’s syndrome or opiate overdose.
Mydriasis is the dilatation of the pupil in response to disease, trauma, drugs (or the dark!). It is pathological when light fails to induce miosis. The radial muscle is innervated by the sympathetic nervous system. Because the parasympathetic fibres travel with the oculomotor nerve they will be damaged by lesions affecting this nerve (e.g. cranial trauma).
The response to light shone in one eye is usually a constriction of both pupils. This indicates intact direct and consensual light reflexes. When the optic nerve has an afferent defect the light shining on the affected eye will produce a diminished pupillary response in both eyes. Whereas light shone on the unaffected eye will produce a normal pupillary response in both eyes. This is referred to as the Marcus Gunn pupil and is seen in conditions such as optic neuritis. In a total CN II lesion shining the light in the affected eye will produce no response.
Oculomotor nerve The pupillary effects are described above. In addition it supplies all ocular muscles apart from lateral rectus and superior oblique. Thus the affected eye will be deviated inferolaterally. Levator palpebrae superioris may also be impaired resulting in impaired ability to open the eye.
Trochlear nerve The eye will not be able to look down.
Trigeminal nerve Largest cranial nerve. Exits the brainstem at the pons. Branches are ophthalmic, maxillary and mandibular. Only the mandibular branch has both sensory and motor fibres. Branches converge to form the trigeminal ganglion (located in Meckels cave). It supplies the muscles of mastication and also tensor veli palatine, mylohyoid, anterior belly of digastric and tensor tympani. The detailed descriptions of the various sensory functions are described in other areas of the website. The corneal reflex is important and is elicited by applying a small tip of cotton wool to the cornea, a reflex blink should occur if it is intact. It is mediated by: the naso ciliary branch of the ophthalmic branch of the trigeminal (sensory component) and the facial nerve producing the motor response. Lesions of the afferent arc will produce bilateral absent blink and lesions of the efferent arc will result in a unilateral absent blink.
Abducens nerve The affected eye will have a deficit of abduction. This cranial nerve exits the brainstem between the pons and medulla. It thus has a relatively long intra cranial course which renders it susceptible to damage in raised intra cranial pressure.
Facial nerve Emerges from brainstem between pons and medulla. It controls muscles of facial expression and taste from the anterior 2/3 of the tongue. The nerve passes into the petrous temporal bone and into the internal auditory meatus. It then passes through the facial canal and exits at the stylomastoid foramen. It passes through the parotid gland and divides at this point. It does not innervate the parotid gland. Its divisions are considered in other parts of the website. Its motor fibres innervate orbicularis oculi to produce the efferent arm of the corneal reflex. In surgical practice it may be injured during parotid gland surgery or invaded by malignancies of the gland and a lower motor neurone on the ipsilateral side will result.
Vestibulo-cochlear nerve Exits from the pons and then passes through the internal auditory meatus. It is implicated in sensorineural hearing loss. Individuals with sensorineural hearing loss will localise the sound in webers test to the normal ear. Rinnes test will be reduced on the affected side but should still work. These two tests will distinguish sensorineural hearing loss from conductive deafness. In the latter condition webers test will localise to the affected ear and Rinnes test will be impaired on the affected side. Surgical lesions affecting this nerve include CNS tumours and basal skull fractures. It may also be damaged by the administration of ototoxic drugs (of which gentamicin is the most commonly used in surgical practice).
Glossopharyngeal nerve Exits the pons just above the vagus. Receives sensory fibres from posterior 1/3 tongue, tonsils, pharynx and middle ear (otalgia may occur following tonsillectomy). It receives visceral afferents from the carotid bodies. It supplies parasympathetic fibres to the parotid gland via the otic ganglion and motor function to stylopharyngeaus muscle. The sensory function of the nerve is tested using the gag reflex.
Vagus nerve Leaves the medulla between the olivary nucleus and the inferior cerebellar peduncle. Passes through the jugular foramen and into the carotid sheath. Details of the functions of the vagus nerve are covered in the website under relevant organ sub headings.
Accessory nerve Exists from the caudal aspect of the brainstem (multiple branches) supplies trapezius and sternocleidomastoid muscles. The distal portion of this nerve is most prone to injury during surgical procedures.
Hypoglossal nerve Emerges from the medulla at the preolivary sulcus, passes through the hypoglossal canal. It lies on the carotid sheath and passes deep to the posterior belly of digastric to supply muscles of the tongue (except palatoglossus). Its location near the carotid sheath makes it vulnerable during carotid endarterectomy surgery and damage will produce ipsilateral defect in muscle function.

How well did you know this?
1
Not at all
2
3
4
5
Perfectly
489
Q

A surgical trainee is incising a groin ‘abscess’ in an intravenous drug abuser. Unfortunately the ‘abscess’ is a false aneurysm and torrential bleeding ensues. In the panic of the situation the doctor then stabs himself in the finger. It transpires that the patient is a Hepatitis B carrier and the doctor is not immunised! What type of virus is Hepatitis B?

Double stranded DNA virus

Single stranded DNA virus

Double stranded RNA virus

Single stranded RNA virus

Retrovirus

A

Hepatitis B

Hepatitis B is a double-stranded DNA virus and is spread through exposure to infected blood or body fluids, including vertical transmission from mother to child. The incubation period is 6-20 weeks.

Immunisation against hepatitis B
Contains HBsAg absorbed onto aluminium hydroxide adjuvant and is prepared from yeast cells using recombinant DNA technology
Most schedules give 3 doses of the vaccine with a recommendation for a one-off booster 5 years following the initial primary vaccination
At risk groups who should be vaccinated include: healthcare workers, intravenous drug users, sex workers, close family contacts of an individual with hepatitis B, individuals receiving blood transfusions regularly, chronic kidney disease patients who may soon require renal replacement therapy, prisoners, chronic liver disease patients
Around 10-15% of adults fail to respond or respond poorly to 3 doses of the vaccine. Risk factors include age over 40 years, obesity, smoking, alcohol excess and immunosuppression
Testing for anti-HBs is only recommended for those at risk of occupational exposure (i.e. Healthcare workers) and patients with chronic kidney disease. In these patients anti-HBs levels should be checked 1-4 months after primary immunisation
The table below shows how to interpret anti-HBs levels:

Anti-HBs level (mIU/ml) Response
> 100 Indicates adequate response, no further testing required. Should still receive booster at 5 years
10 - 100 Suboptimal response - one additional vaccine dose should be given. If immunocompetent no further testing is required
< 10 Non-responder. Test for current or past infection. Give further vaccine course (i.e. 3 doses again) with testing following. If still fails to respond then HBIG would be required for protection if exposed to the virus

Complications of hepatitis B infection
Chronic hepatitis (5-10%)
Fulminant liver failure (1%)
Hepatocellular carcinoma
Glomerulonephritis
Polyarteritis nodosa
Cryoglobulinaemia

Management of hepatitis B
Pegylated interferon-alpha used to be the only treatment available. It reduces viral replication in up to 30% of chronic carriers. A better response is predicted by being female, < 50 years old, low HBV DNA levels, non-Asian, HIV negative, high degree of inflammation on liver biopsy
However, due to the side-effects of pegylated interferon it is now used less commonly in clinical practice. Oral antiviral medication is increasingly used with an aim to suppress viral replication (not in dissimilar way to treating HIV patients)
Examples include lamivudine, tenofovir and entecavir

How well did you know this?
1
Not at all
2
3
4
5
Perfectly
490
Q

A 7 year old boy presents with right iliac fossa pain and there is a clinical suspicion that appendicitis is present. From which of the following embryological structures is the appendix derived?

Vitello-intestinal duct

Urachus

Foregut

Hindgut

Midgut

A

The appendix is derived from the midgut
It is derived from the midgut which is why early appendicitis may present with periumbilical pain.

Appendix

Location: Base of caecum.
Up to 10cm long.
Mainly lymphoid tissue (Hence mesenteric adenitis may mimic appendicitis).
Caecal taenia coli converge at base of appendix and form a longitudinal muscle cover over the appendix. This convergence should facilitate its identification at surgery if it is retrocaecal and difficult to find (which it can be when people start doing appendicectomies!)
Arterial supply: Appendicular artery (branch of the ileocolic).
It is intra peritoneal.

McBurney’s point
1/3 of the way along a line drawn from the Anterior Superior Iliac Spine to the Umbilicus

6 Positions:

Retrocaecal 74%
Pelvic 21%
Postileal
Subcaecal
Paracaecal
Preileal

How well did you know this?
1
Not at all
2
3
4
5
Perfectly
491
Q

Which of the following is not a branch of the posterior cord of the brachial plexus?

Thoracodorsal nerve

Axillary nerve

Radial nerve

Lower subscapular nerve

Musculocutaneous nerve

A

Mnemonic branches off the posterior cord

S ubscapular (upper and lower)
T horacodorsal
A xillary
R adial

The musculocutaneous nerve is a branch off the lateral cord.

Brachial plexus

The brachial plexus extends from the neck to the axilla. It is formed by the ventral rami of the fifth to the eighth cervical nerves with the ascending part of the first thoracic nerve.

Location of the plexus
The ventral rami which form the plexus enter the lower part of the posterior triangle of the neck in series with the ventral rami of the cervical plexus. The second part of the subclavian artery lies immediately anterior to the lower two rami. The upper three rami intermingle and pass inferolaterally towards the axilla and subclavian artery. They are enclosed within an extension of the prevertebral fascia. In the neck the plexus lies deep to platysma, the supraclavicular nerves, inferior belly of omohyoid and the transverse cervical artery. It then passes deep to the clavicle and the suprascapular vessels, to enter the axilla, and thence surround the second part of the axillary artery

Composition of the plexus
Ventral rami, the roots of the plexus, lie between scalenus medius and anterior.

As they enter the posterior triangle, the upper two (C5,6) and lower two (C8, T1) roots of the plexus unite to form the upper and lower trunks of the plexus respectively. Meanwhile, C7 continues as the middle trunk. The lower trunk may groove the superior surface of the first rib posterior to the subclavian artery, and the root from the first ventral ramus is always in contact with it.

Each trunk divides into ventral and dorsal divisions which are destined to supply the anterior (flexor) and posterior (extensor) parts of the upper limb.

The cords of the plexus are formed in the axilla. The dorsal divisions unite to form the posterior cord (C5-8). The ventral divisions of the upper and middle trunks unite to form the lateral cord (C5-7), while the ventral divisions of the lower trunk continues as the medial cord (C8-T1). The cords are named according to their relationship to the axillary artery. Each cord terminates by dividing into two main branches at the beginning of the third part of the artery.

Sympathetic communications
The fifth and sixth cervical ventral rami receive grey rami communicantes from the middle cervical ganglion, while the two or more grey rami communicantes pass from the inferior cervical ganglion to the seventh and eighth cervical ventral rami. The first thoracic ventral ramus receives its grey ramus from the cervicothoracic ganglion. Its for this reason that inferior plexus injury can be complicated by a Horners syndrome.

Summary
Origin Anterior rami of C5 to T1
Sections of the plexus
Roots, trunks, divisions, cords, branches
Mnemonic:Real Teenagers Drink Cold Beer
Roots
Located in the posterior triangle
Pass between scalenus anterior and medius
Trunks
Located posterior to middle third of clavicle
Upper and middle trunks related superiorly to the subclavian artery
Lower trunk passes over 1st rib posterior to the subclavian artery
Divisions Apex of axilla
Cords Related to axillary artery

How well did you know this?
1
Not at all
2
3
4
5
Perfectly
492
Q

A 58 year old man is due to undergo a right hemicolectomy to treat an ascending colon cancer. This is a planned procedure and the surgeon decides to enroll the patient in an enhanced recovery programme following the surgery. Which of the interventions listed below is associated with reduced efficacy of enhanced recovery regimes?

Carbohydrate loading drinks prior to surgery

Early light diet following the procedure

Early mobilisation

IV fluid loading regimes during the procedure

Oral fluids rather than IV fluids post procedure

A

Enhanced recovery regimes try and avoid excessive IV fluids. These increase the rates of ileus in GI surgery. The oral route is preferred where possible.

Enhanced recovery programmes

Many surgical specialties have developed or adopted an enhanced recovery approach following procedures. Whilst the exact aspects differ according to the specialty involved, the broad principles remain the same.

Pre admission
Aim to optimise nutrition and any pre existing conditions. Prehabilitation programmes are used in some centres. Unit visits can be helpful to ensure patients know what to expect following the procedure.

Admission
In many cases admission on the day of the procedure is desirable and as near to the time of the procedure as possible.

Procedure related preparation
Try to minimise starvation times if GA is to be used. In many cases a carbohydrate loading drink is given 2 hours pre procedure. This is usually omitted in diabetic patients who are on complex insulin regimes.

For GI surgery, it is desirable to avoid mechanical bowel preparation unless this is essential.

Procedure
Minimally invasive procedures are preferred and sufficient analgesia to facilitate early ambulation. Avoid excessive IV fluids during the procedure as this can result in interstitial oedema and increase complication rates.

Post procedure
Aim for early ambulation, early removal of drains, catheters and IV lines where possible. Restart oral nutrition as soon as possible. Encourage patients to dress and walk about ward.

How well did you know this?
1
Not at all
2
3
4
5
Perfectly
493
Q

What is the lymphatic drainage of the membranous urethra?

Deep inguinal nodes

Superficial inguinal nodes

Internal iliac nodes

External iliac nodes

Para-aortic nodes

A

The prostatic and membranous urethra drain to the internal iliac nodes.

Urethral anatomy

Female urethra
The female urethra is shorter and more acutely angulated than the male urethra. It is an extra-peritoneal structure and embedded in the endopelvic fascia. The neck of the bladder is subjected to transmitted intra-abdominal pressure and therefore deficiency in this area may result in stress urinary incontinence. Between the layers of the urogenital diaphragm the female urethra is surrounded by the external urethral sphincter, this is innervated by the pudendal nerve. It ultimately lies anterior to the vaginal orifice.

Male urethra
In males the urethra is much longer and is divided into four parts.

Pre-prostatic urethra Extremely short and lies between the bladder and prostate gland.It has a stellate lumen and is between 1 and 1.5cm long.Innervated by sympathetic noradrenergic fibres, as this region is composed of striated muscles bundles they may contract and prevent retrograde ejaculation.
Prostatic urethra This segment is wider than the membranous urethra and contains several openings for the transmission of semen (at the midpoint of the urethral crest).
Membranous urethra Narrowest part of the urethra and surrounded by external sphincter. It traverses the perineal membrane 2.5cm postero-inferior to the symphysis pubis.
Penile urethra Travels through the corpus spongiosum on the underside of the penis. It is the longest urethral segment.It is dilated at its origin as the infrabulbar fossa and again in the glans penis as the navicular fossa. The bulbo-urethral glands open into the spongiose section of the urethra 2.5cm below the perineal membrane.

The urothelium is transitional in nature near to the bladder and becomes squamous more distally.

How well did you know this?
1
Not at all
2
3
4
5
Perfectly
494
Q

What is the most appropriate method of delivering early post-operative analgesia to a 6 month old child following an orchidopexy?

TAP block

Caudal block

Wound analgesic infusion catheter

Spinal block

Epidural block

A

Orchidopexy can be quite uncomfortable immediately following surgery. A caudal block can be a very effective adjunct and provides good analgesia. A spinal block and epidural would be inappropriate. A TAP block may cover the inguinal canal but this is not generally reliable and wound catheters are not used.
Management of pain

World Health Organisation Analgesic Ladder
Initially peripherally acting drugs such as paracetamol or non-steroidal anti-inflammatory drugs (NSAIDs) are given.
If pain control is not achieved, the second part of the ladder is to introduce weak opioid drugs such as codeine or dextropropoxyphene together with appropriate agents to control and minimise side effects.
The final rung of the ladder is to introduce strong opioid drugs such as morphine. Analgesia from peripherally acting drugs may be additive to that from centrally-acting opioids and thus, the two are given together.

The World Federation of Societies of Anaesthesiologists (WFSA) Analgesic Ladder
For management of acute pain
Initially, the pain can be expected to be severe and may need controlling with strong analgesics in combination with local anaesthetic blocks and peripherally acting drugs.
The second rung on the postoperative pain ladder is the restoration of the use of the oral route to deliver analgesia. Strong opioids may no longer be required and adequate analgesia can be obtained by using combinations of peripherally acting agents and weak opioids.
The final step is when the pain can be controlled by peripherally acting agents alone.

Local anaesthetics
Infiltration of a wound with a long-acting local anaesthetic such as Bupivacaine
Analgesia for several hours
Further pain relief can be obtained with repeat injections or by infusions via a thin catheter
Blockade of plexuses or peripheral nerves will provide selective analgesia in those parts of the body supplied by the plexus or nerves
Can either be used to provide anaesthesia for the surgery or specifically for postoperative pain relief
Especially useful where a sympathetic block is needed to improve postoperative blood supply or where central blockade such as spinal or epidural blockade is contraindicated.

Spinal anaesthesia
Provides excellent analgesia for surgery in the lower half of the body and pain relief can last many hours after completion of the operation if long-acting drugs containing vasoconstrictors are used.

  • Side effects of spinal anaesthesia include: hypotension, sensory and motor block, nausea and urinary retention.

Epidural anaesthesia
An indwelling epidural catheter inserted. This can then be used to provide a continuous infusion of analgesic agents. It can provide excellent analgesia. They are still the preferred option following major open abdominal procedures and help prevent post operative respiratory compromise resulting from pain.

  • Disadvantages of epidurals is that they usually confine patients to bed, especially if a motor block is present. In addition an indwelling urinary catheter is required. Which may not only impair mobility but also serve as a conduit for infection. They are contraindicated in coagulopathies.

Transversus Abdominal Plane block (TAP)
In this technique an ultrasound is used to identify the correct muscle plane and local anaesthetic (usually bupivicaine) is injected. The agent diffuses in the plane and blocks many of the spinal nerves. It is an attractive technique as it provides a wide field of blockade but does not require the placement of any indwelling devices. There is no post operative motor impairment. For this reason it is the preferred technique when extensive laparoscopic abdominal procedures are performed. They will then provide analgesia immediately following surgery but as they do not confine the patient to bed, the focus on enhanced recovery can begin sooner.

-The main disadvantage is that their duration of action is limited to the half life of the local anaesthetic agent chosen. In addition some anaesthetists do not have the USS skills required to site the injections.

Patient Controlled Analgesia (PCA)

  • Patients administer their own intravenous analgesia and titrate the dose to their own end-point of pain relief using a small microprocessor - controlled pump. Morphine is the most popular drug used.

Strong Opioids

Severe pain arising from deep or visceral structures requires the use of strong opioids

Morphine
Short half life and poor bioavailability.
Metabolised in the liver and clearance is reduced in patients with liver disease, in the elderly and the debilitated
Side effects include nausea, vomiting, constipation and respiratory depression.
Tolerance may occur with repeated dosage

Pethidine
Synthetic opioid which is structurally different from morphine but which has similar actions. Has 10% potency of morphine.
Short half life and similar bioavailability and clearance to morphine.
Short duration of action and may need to be given hourly.
Pethidine has a toxic metabolite (norpethidine) which is cleared by the kidney, but which accumulates in renal failure or following frequent and prolonged doses and may lead to muscle twitching and convulsions. Extreme caution is advised if pethidine is used over a prolonged period or in patients with renal failure.

Weak opioids
Codeine: markedly less active than morphine, has predictable effects when given orally and is effective against mild to moderate pain.

Non opioid analgesics
- Mild to moderate pain.

Paracetamol
Inhibits prostaglandin synthesis.
Analgesic and antipyretic properties but little anti-inflammatory effect
It is well absorbed orally and is metabolised almost entirely in the liver
Side effects in normal dosage and is widely used for the treatment of minor pain. It causes hepatotoxicity in over dosage by overloading the normal metabolic pathways with the formation of a toxic metabolite.

NSAIDs
Analgesic and anti-inflammatory actions
Inhibition of prostaglandin synthesis by the enzyme Cyclooxygenase which catalyses the conversion of arachidonic acid to the various prostaglandins that are the chief mediators of inflammation. All NSAIDs work in the same way and thus there is no point in giving more than one at a time. .
NSAIDs are, in general, more useful for superficial pain arising from the skin, buccal mucosa, joint surfaces and bone.
Relative contraindications: history of peptic ulceration, gastrointestinal bleeding or bleeding diathesis; operations associated with high blood loss, asthma, moderate to severe renal impairment, dehydration and any history of hypersensitivity to NSAIDs or aspirin.

Neuropathic pain
National Institute of Clinical Excellence (UK) guidelines:
First line: Amitriptyline (Imipramine if cannot tolerate) or pregabalin
Second line: Amitriptyline AND pregabalin
Third line: refer to pain specialist. Give tramadol in the interim (avoid morphine)
If diabetic neuropathic pain: Duloxetine
As of 1 April 2019, pregabalin and gabapentin are Class C controlled substances (under the Misuse of Drugs Act 1971) and scheduled under the Misuse of Drugs Regulations 2001 as Schedule 3. Evaluate patients carefully for a history of drug abuse before prescribing and observe patients for development of signs of abuse and dependence (MHRA, Drug Safety Update April 2019).

How well did you know this?
1
Not at all
2
3
4
5
Perfectly
495
Q

What is the course of the median nerve relative to the brachial artery in the upper arm?

Medial to anterior to lateral

Lateral to posterior to medial

Medial to posterior to lateral

Medial to anterior to medial

Lateral to anterior to medial

A

Relations of median nerve to the brachial artery:
Lateral -> Anterior -> Medial
The median nerve descends lateral to the brachial artery, it usually passes anterior to the artery to lie on its medial side. It passes deep to the bicipital aponeurosis and the median cubital vein at the elbow. It enters the forearm between the two heads of the pronator teres muscle.

Brachial artery

The brachial artery begins at the lower border of teres major as a continuation of the axillary artery. It terminates in the cubital fossa at the level of the neck of the radius by dividing into the radial and ulnar arteries.

Relations
Posterior relations include the long head of triceps with the radial nerve and profunda vessels intervening. Anteriorly it is overlapped by the medial border of biceps.
It is crossed by the median nerve in the middle of the arm.
In the cubital fossa it is separated from the median cubital vein by the bicipital aponeurosis.
The basilic vein is in contact at the most proximal aspect of the cubital fossa and lies medially.

How well did you know this?
1
Not at all
2
3
4
5
Perfectly
496
Q

Which of the following stimulates prolactin release or action?

Leutinising hormone

Dopamine

Thyrotropin releasing hormone

Oestrogen

Follicle stimulating hormone

A

Dopamine: Dopamine, acting as a PRL-inhibitory factor (PIF), inhibits prolactin release. When dopamine levels decrease (such as during suckling), there is a disinhibition effect, leading to prolactin secretion12.
Thyrotropin Releasing Hormone (TRH): TRH, produced by the hypothalamus, stimulates the release of prolactin. Additionally, oestrogens enhance lactotroph sensitivity to TRH and reduce their sensitivity to dopamine inhibition1.
In summary, dopamine suppresses prolactin, while TRH and oestrogens promote its release. The other hormones listed (luteinizing hormone and follicle-stimulating hormone) do not directly influence prolactin13.
Remember, prolactin plays a crucial role in breast development, milk production, and other physiological processes, particularly in females

Prolactin

Prolactin is a peptide hormone released from the anterior pituitary. It is under tonic dopamine inhibition, thyrotropin releasing hormone has a stimulatory effect on release. Prolactin release stimulates milk production but also reduces gonadal activity. It decreases GnRH pulsatility at the hypothalamic level and to a lesser extent, blocks the action of LH on the ovary or testis.

How well did you know this?
1
Not at all
2
3
4
5
Perfectly
497
Q

A 43 year old lady develops a cerebello-pontine angle lesion. Which of the nerves listed below is likely to be affected first?

CN X

CN III

CN V

CN IX

CN XII

A

The most likely lesion to occur in the cerebello-pontine angle is an acoustic neuroma.
The trigeminal nerve has a broad base and involvement of at least part of this nerve is the most likely initial finding. The defect may be subtle such as loss of the ipsilateral corneal reflex. Ipsilateral hearing loss will also occur. Untreated, progressive lesions, may ultimately affect cranial nerve roots in this region.

Cranial nerves

Cranial nerve lesions
Olfactory nerve May be injured in basal skull fractures or involved in frontal lobe tumour extension. Loss of olfactory nerve function in relation to major CNS pathology is seldom an isolated event and thus it is poor localiser of CNS pathology.
Optic nerve Problems with visual acuity may result from intra ocular disorders. Problems with the blood supply such as amaurosis fugax may produce temporary visual distortion. More important surgically is the pupillary response to light. The pupillary size may be altered in a number of disorders. Nerves involved in the resizing of the pupil connect to the pretectal nucleus of the high midbrain, bypassing the lateral geniculate nucleus and the primary visual cortex. From the pretectal nucleus neurones pass to the Edinger - Westphal nucleus, motor axons from here pass along with the oculomotor nerve. They synapse with ciliary ganglion neurones; the parasympathetic axons from this then innervate the iris and produce miosis. The miotic pupil is seen in disorders such as Horner’s syndrome or opiate overdose.
Mydriasis is the dilatation of the pupil in response to disease, trauma, drugs (or the dark!). It is pathological when light fails to induce miosis. The radial muscle is innervated by the sympathetic nervous system. Because the parasympathetic fibres travel with the oculomotor nerve they will be damaged by lesions affecting this nerve (e.g. cranial trauma).
The response to light shone in one eye is usually a constriction of both pupils. This indicates intact direct and consensual light reflexes. When the optic nerve has an afferent defect the light shining on the affected eye will produce a diminished pupillary response in both eyes. Whereas light shone on the unaffected eye will produce a normal pupillary response in both eyes. This is referred to as the Marcus Gunn pupil and is seen in conditions such as optic neuritis. In a total CN II lesion shining the light in the affected eye will produce no response.
Oculomotor nerve The pupillary effects are described above. In addition it supplies all ocular muscles apart from lateral rectus and superior oblique. Thus the affected eye will be deviated inferolaterally. Levator palpebrae superioris may also be impaired resulting in impaired ability to open the eye.
Trochlear nerve The eye will not be able to look down.
Trigeminal nerve Largest cranial nerve. Exits the brainstem at the pons. Branches are ophthalmic, maxillary and mandibular. Only the mandibular branch has both sensory and motor fibres. Branches converge to form the trigeminal ganglion (located in Meckels cave). It supplies the muscles of mastication and also tensor veli palatine, mylohyoid, anterior belly of digastric and tensor tympani. The detailed descriptions of the various sensory functions are described in other areas of the website. The corneal reflex is important and is elicited by applying a small tip of cotton wool to the cornea, a reflex blink should occur if it is intact. It is mediated by: the naso ciliary branch of the ophthalmic branch of the trigeminal (sensory component) and the facial nerve producing the motor response. Lesions of the afferent arc will produce bilateral absent blink and lesions of the efferent arc will result in a unilateral absent blink.
Abducens nerve The affected eye will have a deficit of abduction. This cranial nerve exits the brainstem between the pons and medulla. It thus has a relatively long intra cranial course which renders it susceptible to damage in raised intra cranial pressure.
Facial nerve Emerges from brainstem between pons and medulla. It controls muscles of facial expression and taste from the anterior 2/3 of the tongue. The nerve passes into the petrous temporal bone and into the internal auditory meatus. It then passes through the facial canal and exits at the stylomastoid foramen. It passes through the parotid gland and divides at this point. It does not innervate the parotid gland. Its divisions are considered in other parts of the website. Its motor fibres innervate orbicularis oculi to produce the efferent arm of the corneal reflex. In surgical practice it may be injured during parotid gland surgery or invaded by malignancies of the gland and a lower motor neurone on the ipsilateral side will result.
Vestibulo-cochlear nerve Exits from the pons and then passes through the internal auditory meatus. It is implicated in sensorineural hearing loss. Individuals with sensorineural hearing loss will localise the sound in webers test to the normal ear. Rinnes test will be reduced on the affected side but should still work. These two tests will distinguish sensorineural hearing loss from conductive deafness. In the latter condition webers test will localise to the affected ear and Rinnes test will be impaired on the affected side. Surgical lesions affecting this nerve include CNS tumours and basal skull fractures. It may also be damaged by the administration of ototoxic drugs (of which gentamicin is the most commonly used in surgical practice).
Glossopharyngeal nerve Exits the pons just above the vagus. Receives sensory fibres from posterior 1/3 tongue, tonsils, pharynx and middle ear (otalgia may occur following tonsillectomy). It receives visceral afferents from the carotid bodies. It supplies parasympathetic fibres to the parotid gland via the otic ganglion and motor function to stylopharyngeaus muscle. The sensory function of the nerve is tested using the gag reflex.
Vagus nerve Leaves the medulla between the olivary nucleus and the inferior cerebellar peduncle. Passes through the jugular foramen and into the carotid sheath. Details of the functions of the vagus nerve are covered in the website under relevant organ sub headings.
Accessory nerve Exists from the caudal aspect of the brainstem (multiple branches) supplies trapezius and sternocleidomastoid muscles. The distal portion of this nerve is most prone to injury during surgical procedures.
Hypoglossal nerve Emerges from the medulla at the preolivary sulcus, passes through the hypoglossal canal. It lies on the carotid sheath and passes deep to the posterior belly of digastric to supply muscles of the tongue (except palatoglossus). Its location near the carotid sheath makes it vulnerable during carotid endarterectomy surgery and damage will produce ipsilateral defect in muscle function.

How well did you know this?
1
Not at all
2
3
4
5
Perfectly
498
Q

Which of the agents listed below can be administered via the peripheral intra venous route in the non cardiac arrest setting?

Milrinone

Noradrenaline

Adrenaline

Metaraminol

Dobutamine

A

Metaraminol is an alpha receptor agonist.
As a general rule, inotropes and vasopressors can only be administered via a central vein. Metaraminol is an exception to this as it can be administered via a peripheral line.

Circulatory support of the critically ill

Circulatory support
Impaired tissue oxygenation may occur as a result of circulatory shock. Shock is considered further under its own topic heading.

Patients requiring circulatory support require haemodynamic monitoring. At its simplest level this may simply be in the form of regular urine output measurements and blood pressure monitoring. In addition ECG monitoring will allow the identification of cardiac arrhythmias. Pulse oximeter measurements will allow quick estimation haemoglobin oxygen saturation in arterial blood.

Invasive arterial blood pressure monitoring is undertaken by the use of an indwelling arterial line. Most arterial sites can be used although the radial artery is the commonest. It is important not to cannulate end arteries. The arterial trace can be tracked to ventilation phases and those patients whose systolic pressure varies with changes in intrathoracic pressure may benefit from further intravenous fluids.

Central venous pressure is measured using a CVP line that is usually sited in the superior vena cava via the internal jugular route. The CVP will demonstrate right atrial filling pressure and volume status. When adequate intra vascular volume is present a fluid challenge will typically cause a prolonged rise in CVP (usually greater than 6-8mmHg).

To monitor the cardiac output a Swan-Ganz catheter is traditionally inserted (other devices may be used and are less invasive). Inflation of the distal balloon will provide the pulmonary artery occlusion pressure and the pressure distal to the balloon will equate to the left atrial pressure. This gives a measure of left ventricular preload. Because the Swan-Ganz catheter can measure several variables it can be used to calculate:
Stroke volume
Systemic vascular resistance
Pulmonary artery resistance
Oxygen delivery (and consumption)

Inotropes
In patients with an adequate circulating volume but on-going circulatory compromise a vasoactive drug may be considered. These should usually be administered via the central venous route. Commonly used inotropes include:
Agent Mode of action Effect
Noradrenaline α agonist Vasopressor action, minimal effect on cardiac output
Adrenaline α and β receptor agonist Increases cardiac output and peripheral vascular resistance
Dopamine β1 agonist Increases contractility and rate
Dobutamine β1 and β2 agonist Increases cardiac output and decreases SVR
Milrinone Phosphodiesterase inhibitor Elevation of cAMP levels improves muscular contractility, short half life and acts as vasodilator

How well did you know this?
1
Not at all
2
3
4
5
Perfectly
499
Q

What is the most appropriate management for a 56 year old lady who has shooting pains in her arm following a mastectomy and axillary node clearance?

Carbamazepine

Pregabalin

Oramorph

Diclofenac

Chemical neurectomy

A

Pregabalin is generally the first line agent for neuropathic pain.

Management of pain

World Health Organisation Analgesic Ladder
Initially peripherally acting drugs such as paracetamol or non-steroidal anti-inflammatory drugs (NSAIDs) are given.
If pain control is not achieved, the second part of the ladder is to introduce weak opioid drugs such as codeine or dextropropoxyphene together with appropriate agents to control and minimise side effects.
The final rung of the ladder is to introduce strong opioid drugs such as morphine. Analgesia from peripherally acting drugs may be additive to that from centrally-acting opioids and thus, the two are given together.

The World Federation of Societies of Anaesthesiologists (WFSA) Analgesic Ladder
For management of acute pain
Initially, the pain can be expected to be severe and may need controlling with strong analgesics in combination with local anaesthetic blocks and peripherally acting drugs.
The second rung on the postoperative pain ladder is the restoration of the use of the oral route to deliver analgesia. Strong opioids may no longer be required and adequate analgesia can be obtained by using combinations of peripherally acting agents and weak opioids.
The final step is when the pain can be controlled by peripherally acting agents alone.

Local anaesthetics
Infiltration of a wound with a long-acting local anaesthetic such as Bupivacaine
Analgesia for several hours
Further pain relief can be obtained with repeat injections or by infusions via a thin catheter
Blockade of plexuses or peripheral nerves will provide selective analgesia in those parts of the body supplied by the plexus or nerves
Can either be used to provide anaesthesia for the surgery or specifically for postoperative pain relief
Especially useful where a sympathetic block is needed to improve postoperative blood supply or where central blockade such as spinal or epidural blockade is contraindicated.

Spinal anaesthesia
Provides excellent analgesia for surgery in the lower half of the body and pain relief can last many hours after completion of the operation if long-acting drugs containing vasoconstrictors are used.

  • Side effects of spinal anaesthesia include: hypotension, sensory and motor block, nausea and urinary retention.

Epidural anaesthesia
An indwelling epidural catheter inserted. This can then be used to provide a continuous infusion of analgesic agents. It can provide excellent analgesia. They are still the preferred option following major open abdominal procedures and help prevent post operative respiratory compromise resulting from pain.

  • Disadvantages of epidurals is that they usually confine patients to bed, especially if a motor block is present. In addition an indwelling urinary catheter is required. Which may not only impair mobility but also serve as a conduit for infection. They are contraindicated in coagulopathies.

Transversus Abdominal Plane block (TAP)
In this technique an ultrasound is used to identify the correct muscle plane and local anaesthetic (usually bupivicaine) is injected. The agent diffuses in the plane and blocks many of the spinal nerves. It is an attractive technique as it provides a wide field of blockade but does not require the placement of any indwelling devices. There is no post operative motor impairment. For this reason it is the preferred technique when extensive laparoscopic abdominal procedures are performed. They will then provide analgesia immediately following surgery but as they do not confine the patient to bed, the focus on enhanced recovery can begin sooner.

-The main disadvantage is that their duration of action is limited to the half life of the local anaesthetic agent chosen. In addition some anaesthetists do not have the USS skills required to site the injections.

Patient Controlled Analgesia (PCA)

  • Patients administer their own intravenous analgesia and titrate the dose to their own end-point of pain relief using a small microprocessor - controlled pump. Morphine is the most popular drug used.

Strong Opioids

Severe pain arising from deep or visceral structures requires the use of strong opioids

Morphine
Short half life and poor bioavailability.
Metabolised in the liver and clearance is reduced in patients with liver disease, in the elderly and the debilitated
Side effects include nausea, vomiting, constipation and respiratory depression.
Tolerance may occur with repeated dosage

Pethidine
Synthetic opioid which is structurally different from morphine but which has similar actions. Has 10% potency of morphine.
Short half life and similar bioavailability and clearance to morphine.
Short duration of action and may need to be given hourly.
Pethidine has a toxic metabolite (norpethidine) which is cleared by the kidney, but which accumulates in renal failure or following frequent and prolonged doses and may lead to muscle twitching and convulsions. Extreme caution is advised if pethidine is used over a prolonged period or in patients with renal failure.

Weak opioids
Codeine: markedly less active than morphine, has predictable effects when given orally and is effective against mild to moderate pain.

Non opioid analgesics
- Mild to moderate pain.

Paracetamol
Inhibits prostaglandin synthesis.
Analgesic and antipyretic properties but little anti-inflammatory effect
It is well absorbed orally and is metabolised almost entirely in the liver
Side effects in normal dosage and is widely used for the treatment of minor pain. It causes hepatotoxicity in over dosage by overloading the normal metabolic pathways with the formation of a toxic metabolite.

NSAIDs
Analgesic and anti-inflammatory actions
Inhibition of prostaglandin synthesis by the enzyme Cyclooxygenase which catalyses the conversion of arachidonic acid to the various prostaglandins that are the chief mediators of inflammation. All NSAIDs work in the same way and thus there is no point in giving more than one at a time. .
NSAIDs are, in general, more useful for superficial pain arising from the skin, buccal mucosa, joint surfaces and bone.
Relative contraindications: history of peptic ulceration, gastrointestinal bleeding or bleeding diathesis; operations associated with high blood loss, asthma, moderate to severe renal impairment, dehydration and any history of hypersensitivity to NSAIDs or aspirin.

Neuropathic pain
National Institute of Clinical Excellence (UK) guidelines:
First line: Amitriptyline (Imipramine if cannot tolerate) or pregabalin
Second line: Amitriptyline AND pregabalin
Third line: refer to pain specialist. Give tramadol in the interim (avoid morphine)
If diabetic neuropathic pain: Duloxetine
As of 1 April 2019, pregabalin and gabapentin are Class C controlled substances (under the Misuse of Drugs Act 1971) and scheduled under the Misuse of Drugs Regulations 2001 as Schedule 3. Evaluate patients carefully for a history of drug abuse before prescribing and observe patients for development of signs of abuse and dependence (MHRA, Drug Safety Update April 2019).

References
1. http://guidance.nice.org.uk/CG173/Guidance/pdf/English
2. Lovich-Sapola J, Smith CE, Brandt CP. Post operative pain control. Surg Clin North Am. 2015 Apr;95(2):301-183. Finnerup N et al. Pharmacotherapy for neuropathic pain in adults: a systematic review and meta-analysis. Lancet Neurol. 2015 Feb;14(2):162-73.

How well did you know this?
1
Not at all
2
3
4
5
Perfectly
500
Q

A 32 year old lady presents with a 1.5cm pigmented lesion on her back. The surgeon is concerned that this may be a melanoma. What is the most appropriate course of action?

2mm punch biopsy from the centre of the lesion

4mm punch biopsy from the centre of the lesion

Wide excision of the lesion with 3cm margins

Excisional biopsy of the lesion

Wide excision of the lesion with 1cm margins

A

Suspicious naevi should NOT be partially sampled as histological interpretation is severely compromised. Complete excision is mandatory where lesions fulfil diagnostic criteria. However, wide excision for margins may be deferred until definitive histology is available.
Lesions that are suspicious for melanoma should be excised with complete margins. Radical excision is not routinely undertaken for diagnostic purposes and therefore if subsequent histopathological assessment determines that the lesion is a melanoma a re-exicision of margins may be required. Incisional punch biopsies of potential melanomas makes histological interpretation difficult and is best avoided

Malignant melanoma

The main diagnostic features (major criteria):
Change in size
Change in shape
Change in colour
Secondary features (minor criteria)
Diameter >6mm
Inflammation
Oozing or bleeding
Altered sensation

Treatment
Suspicious lesions should undergo excision biopsy. The lesion should be removed in completely as incision biopsy can make subsequent histopathological assessment difficult.
Once the diagnosis is confirmed the pathology report should be reviewed to determine whether further re-excision of margins is required (see below):

Margins of excision-Related to Breslow thickness
Lesions 0-1mm thick 1cm
Lesions 1-2mm thick 1- 2cm (Depending upon site and pathological features)
Lesions 2-4mm thick 2-3 cm (Depending upon site and pathological features)
Lesions >4 mm thick 3cm
Marsden J et al Revised UK guidelines for management of Melanoma. Br J Dermatol 2010 163:238-256.

Further treatments such as sentinel lymph node mapping, isolated limb perfusion and block dissection of regional lymph node groups should be selectively applied.

Guidelines
The UK NICE guidance is covered by Melanoma: assessment and management NICE guideline [NG14]. However, the complex oncological regimens are not extensively covered in this guidance though more formal guidance is provided by relevant oncological organisations.

How well did you know this?
1
Not at all
2
3
4
5
Perfectly
501
Q

What is the correct embryological origin of the stapes?

First pharyngeal arch

Second pharyngeal arch

Third pharyngeal arch

Fourth pharyngeal arch

Fifth pharyngeal arch

A

Embryological origin stapes = 2nd pharyngeal arch
The ectoderm covering the outer aspect of the second arch originates from a strip of ectoderm lateral to the metencephalic neural fold. The cartilaginous element to this, eponymously known as Reicherts cartilage extends from the otic capsule to the midline on each side. Its dorsal end separates and becomes enclosed in the tympanic cavity as the stapes.
The dorsal ends of the cartilages of the first and second pharyngeal arches articulate superior to the tubotympanic recess. These cartilages form the malleus, incus and stapes. At least part of the malleus is formed from the first arch and the stapes from the second arch. The incus is most likely to arise from the first arch.

Pharyngeal arches

These develop during the fourth week of embryonic growth from a series of mesodermal outpouchings of the developing pharynx.
They develop and fuse in the ventral midline. Pharyngeal pouches form on the endodermal side between the arches.
There are 6 pharyngeal arches, the fifth does not contribute any useful structures and often fuses with the sixth arch.

Pharyngeal arches
Pharyngeal arch Muscular contributions Skeletal contributions Endocrine Artery Nerve
First Muscles of mastication
Anterior belly of digastric
Mylohyoid
Tensor tympanic
Tensor veli palatini Maxilla
Meckels cartilage
Incus
Malleus n/a Maxillary
External carotid Mandibular
Second Buccinator
Platysma
Muscles of facial expression
Stylohyoid
Posterior belly of digastric
Stapedius Stapes
Styloid process
Lesser horn and upper body of hyoid n/a Inferior branch of superior thyroid artery
Stapedial artery Facial
Third Stylopharyngeus Greater horn and lower part of hyoid Thymus
Inferior parathyroids Common and internal carotid Glossopharyngeal
Fourth Cricothyroid
All intrinsic muscles of the soft palate Thyroid and epiglottic cartilages Superior parathyroids Right- subclavian artery, Left-aortic arch Vagus
Sixth All intrinsic muscles of the larynx (except cricothyroid) Cricoid, arytenoid and corniculate cartilages n/a Right -Pulmonary artery, Left- Pulmonary artery and ductus arteriosus Vagus and recurrent laryngeal nerve

How well did you know this?
1
Not at all
2
3
4
5
Perfectly
502
Q

A 20 year old woman sustains a Holstein-Lewis fracture. Which nerve is at risk?

Ulnar

Radial

Median

Musculocutaneous

Axillary

A

Since the distal humerus is affected, the radial is at risk.

Eponymous fractures

Colles’ fracture (dinner fork deformity)
Fall onto extended outstretched hand
Classical Colles’ fractures have the following 3 features:

  1. Transverse fracture of the radius
  2. 1 inch proximal to the radio-carpal joint
  3. Dorsal displacement and angulation

Smith’s fracture (reverse Colles’ fracture)
Volar angulation of distal radius fragment (Garden spade deformity)
Caused by falling backwards onto the palm of an outstretched hand or falling with wrists flexed

Bennett’s fracture
Intra-articular fracture of the first carpometacarpal joint
Impact on flexed metacarpal, caused by fist fights

Monteggia’s fracture
Dislocation of the proximal radioulnar joint in association with an ulna fracture
Fall on outstretched hand with forced pronation
Needs prompt diagnosis to avoid disability

Galeazzi fracture
Radial shaft fracture with associated dislocation of the distal radioulnar joint
Direct blow

Pott’s fracture
Bimalleolar ankle fracture
Forced foot eversion

Barton’s fracture
Distal radius fracture (Colles’/Smith’s) with associated radiocarpal dislocation
Fall onto extended and pronated wrist
Involvement of the joint is a defining feature

Holstein Lewis Fracture
A HolsteinLewis fracture is a fracture of the distal third of the humerus resulting in entrapment of the radial nerve.
The radial nerve is one of the major nerves of the upper limb. It innervates all of the muscles in the extensor compartments of the arm.
Conservative treatment includes reduction and use of a functional brace
Vascular injury may require open surgery

How well did you know this?
1
Not at all
2
3
4
5
Perfectly
503
Q

What is the level of the hyoid bone?

C1

C2

C3

C4

C5

A

Surface anatomy of the neck

In the midline from above down, the following structures are felt
Structure Level
Hyoid C3
Notch of the thyroid cartilage C4
Cricoid cartilage(termination) C6
The lower border of the cricoid cartilage corresponds to the commencement of the trachea and also to the following:

Junction of larynx with trachea
Junction of pharynx with oesophagus
Level at which the inferior thyroid artery enters the thyroid gland
The level at which the vertebral artery enters the transverse foramen in the 6th cervical vertebra
Level at which the superior belly of omohyoid crosses the carotid sheath
The level of the middle cervical sympathetic ganglion
The level at which the carotid artery can be compressed against the transverse process of C6 (carotid tubercle).

How well did you know this?
1
Not at all
2
3
4
5
Perfectly
504
Q

A person is diagnosed with Conns syndrome. If the parotid gland secretions were assayed, what change (if any) will be noted?

Decreased potassium

Decreased sodium

Increased sodium

Potassium levels unchanged, sodium levels increased

Potassium levels raised sodium levels unchanged

A

Decreased sodium.
Aldosterone affects the parotid gland secretions and has the effect of conserving sodium (so salivary sodium reduced) and potassium secretion increased.

Parotid gland secretions

  • Parotid gland secretions account for around 25% of salivary secretions
    Levels of sodium and chloride are lower than plasma, potassium and bicarbonate levels are higher
    Parotid secretions are watery and have high enzyme concentrations
    Parasympathetic stimulation produces a water rich, serous saliva. Sympathetic stimulation leads to the production of a low volume, enzyme-rich saliva.
    Fluid that is secreted within the acini is isotonic with plasma and undergoes modification within the ducts via process of ion exchange
    The ducts are affected by aldosterone and so will retain sodium and secrete potassium when aldosterone levels are elevated
How well did you know this?
1
Not at all
2
3
4
5
Perfectly
505
Q

Which of the following statements relating to the Cavernous Sinus is false?

The pituitary gland lies medially

The internal carotid artery passes through it

The temporal lobe of the brain is a lateral relation

The mandibular branch of the trigeminal and optic nerve lie on the lateral wall

The ophthalmic veins drain into the anterior aspect of the sinus

A

The veins that drain into the sinus are important as sepsis can cause cavernous sinus thrombosis. The maxillary branch of the trigeminal and not the mandibular branches pass through the sinus

Cavernous sinus

The cavernous sinuses are paired and are situated on the body of the sphenoid bone. It runs from the superior orbital fissure to the petrous temporal bone.

Relations
Medial Lateral
Pituitary fossa
Sphenoid sinus Temporal lobe

Contents
Lateral wall components (from top to bottom:)
Oculomotor nerve
Trochlear nerve
Ophthalmic nerve
Maxillary nerve
Contents of the sinus (from medial to lateral:)
Internal carotid artery (and sympathetic plexus)
Abducens nerve

Blood supply
Ophthalmic vein, superficial cortical veins, basilar plexus of veins posteriorly.

Drains into the internal jugular vein via: the superior and inferior petrosal sinuses

How well did you know this?
1
Not at all
2
3
4
5
Perfectly
506
Q

A 25 year old man is diagnosed as having an undisplaced fracture of the proximal pole of the scaphoid. What is the best course of action?

Immobilisation in future splint for 5 weeks

Arrange an MRI scan

Immobilisation in plaster cast for 4 weeks

Surgical fixation

Initial immobisation in plaster cast for 2 weeks with check radiographs at that stage

A

It is generally accepted that proximal pole fractures of the scaphoid should be surgically fixed as non union rates of up to 34% can be seen when cast immobilization alone is attempted.
Scaphoid fractures

  • Incidence of scaphoid fractures in UK ranges from 12.4 per 100,000 to 29 per 100,000
    Surface of scaphoid is covered by articular cartilage with small area available for blood vessels (fracture risks blood supply)
    Forms floor of anatomical snuffbox
    Risk of fracture associated with fall onto outstretched hand (tubercle, waist, or proximal third)
    A series of 4 scaphoid radiographs should be undertaken (PA, pronated oblique, Ziter view and lateral view). The Ziter view is a PA view with the wrist in ulnar deviation and beam angulated at 20 degrees
    Sensitivity of scaphoid radiographs in 1st week of injury is 80%
    Immobilization of scaphoid fractures difficult
    Repeat imaging should be done at 10 days. MRI should be done in cases of diagnostic uncertainty

Classification of scaphoid fractures
Scaphoid tubercle
Distal pole
Waist
Proximal pole

Management
Undisplaced fractures of the waist of the scaphoid and most distal pole fractures can be managed in a cast for 6 weeks with high rates of union.
Displaced scaphoid waist fractures (more than 1-2mm) should be viewed as unstable and surgically fixed.
All proximal pole fractures should be fixed surgically.

Complications
Non union of scaphoid
Avascular necrosis of the scaphoid
Scapholunate disruption and wrist collapse
Degenerative changes of the adjacent joint

How well did you know this?
1
Not at all
2
3
4
5
Perfectly
507
Q

Which of the following does not exit the pelvis through the greater sciatic foramen?

Superior gluteal artery

Internal pudendal vessels

Sciatic nerve

Obturator nerve

Inferior gluteal nerve

A

The obturator nerve exits through the obturator foramen.

Greater sciatic foramen

Contents
Nerves
Sciatic Nerve
Superior and Inferior Gluteal Nerves
Pudendal Nerve
Posterior Femoral Cutaneous Nerve
Nerve to Quadratus Femoris
Nerve to Obturator internus
Vessels
Superior Gluteal Artery and vein
Inferior Gluteal Artery and vein
Internal Pudendal Artery and vein

Piriformis
The piriformis is a landmark for identifying structures passing out of the sciatic notch
Above piriformis: Superior gluteal vessels
Below piriformis: Inferior gluteal vessels, sciatic nerve (10% pass through it, <1% above it), posterior cutaneous nerve of the thigh

Greater sciatic foramen boundaries
Anterolaterally Greater sciatic notch of the ilium
Posteromedially Sacrotuberous ligament
Inferior Sacrospinous ligament and the ischial spine
Superior Anterior sacroiliac ligament

Structures passing between both foramina (Medial to lateral)
Pudendal nerve
Internal pudendal artery
Nerve to obturator internus

Contents of the lesser sciatic foramen
Tendon of the obturator internus
Pudendal nerve
Internal pudendal artery and vein
Nerve to the obturator internus

How well did you know this?
1
Not at all
2
3
4
5
Perfectly
508
Q

Which part of the jugular venous waveform is associated with the closure of the tricuspid valve?

a wave

c wave

x descent

y descent

v wave

A

JVP: C wave - closure of the tricuspid valve
The c wave of the jugular venous waveform is associated with the closure of the tricuspid valve.

Jugular venous pressure

As well as providing information on right atrial pressure, the jugular vein waveform may provide clues to underlying valvular disease. A non-pulsatile JVP is seen in superior vena caval obstruction. Kussmaul’s sign describes a paradoxical rise in JVP during inspiration seen in constrictive pericarditis

‘a’ wave = atrial contraction
large if atrial pressure e.g. tricuspid stenosis, pulmonary stenosis, pulmonary hypertension
absent if in atrial fibrillation

Cannon ‘a’ waves
caused by atrial contractions against a closed tricuspid valve
are seen in complete heart block, ventricular tachycardia/ectopics, nodal rhythm, single chamber ventricular pacing

‘c’ wave
closure of tricuspid valve
not normally visible

‘v’ wave
due to passive filling of blood into the atrium against a closed tricuspid valve
giant v waves in tricuspid regurgitation

‘x’ descent = fall in atrial pressure during ventricular systole

‘y’ descent = opening of tricuspid valve

How well did you know this?
1
Not at all
2
3
4
5
Perfectly
509
Q

A 45 year old patient undergoes a CT scan of the abdomen and is noted to have a 6cm mass in the right adrenal gland. Urinary catecholamines and other endocrine investigations are negative. CT of the chest and remainder of the abdomen is otherwise normal. What is the most appropriate course of action?

Image guided FNAC of the adrenal gland

Image guided core biopsy of the adrenal gland

List the patient for an adrenalectomy

Organise surveillance of the lesion with CT scanning

Organise surveillance of the lesion with USS

A

Most surgeons would excise a mass of this size rather than attempt biopsy. Further information relating to adrenal masses is covered under this topic.

Tissue sampling

Tissue sampling is an important surgical process. Biopsy modalities vary according to the site, experience and subsequent planned therapeutic outcome

The modalities comprise:
-Fine needle aspiration cytology
-Core biopsy
-Excision biopsy
-Tru cut biopsy
-Punch biopsy
-Cytological smears
-Endoscopic or laparoscopic biopsy

When the lesion is superficial the decision needs to be taken as to whether complete excision is desirable or whether excision biopsy is acceptable. In malignant melanoma for example the need for safe margins will mean that a more radical surgical approach needs to be adopted after diagnostic confirmation from excision biopsy than would be the case in basal cell carcinoma. Punch biopsies are useful in gaining histological diagnosis of unclear skin lesions where excision biopsy is undesirable such as in establishing whether a skin lesion is vasculitic or not.

Fine needle aspiration cytology (FNAC) is an operator dependent procedure that may or may not be image guided and essentially involves passing a needle through a lesion whilst suction is applied to a syringe. The material thus obtained is expressed onto a slide and sent for cytological assessment. This test can be limited by operator inexperience and also by the lack of histological architectural information (e.g. Follicular carcinoma of the thyroid). Where a discharge is present a sample may be sent for cytology although in some sites (e.g. Nipple discharge ) the information gleaned may be meaningless.

Tissue samples may be obtained by both core and tru cut biopsy. A core biopsy is obtained by use of a spring loaded gun with a needle passing quickly through the lesion of interest. A tru cut biopsy achieves the same objective but the needle moved by hand. When performing these techniques image guidance may be desirable (e.g. In breast lesions). Consideration needs to be given to any planned surgical resection as it may be necessary to resect the biopsy tract along with the specimen (e.g. In sarcoma surgery).

Visceral lesions may be accessed percutaneously under image guidance such as ultrasound guided biopsy of liver metastases. Or under direct vision such as a colonoscopic biopsy.

How well did you know this?
1
Not at all
2
3
4
5
Perfectly
510
Q

A female neonate who is born at term has an episode of bilious vomiting. An upper GI contrast study is performed and it shows the duodeno-jejunal flexure lies to the right of the midline. What is the most appropriate course of action?

Undertake a Ramstedts pyloromyotomy

Perform a gastrojejunostomy

Undertake a Ladd’s procedure

Undertake a duodeno-duodenostomy

Undertake a Kasai procedure

A

Intestinal malrotation with volvulus is treated with a Ladd’s procedure.

Intestinal malrotation in neonates

Normal embryology
During the fourth week of embryogenesis the intestine moves into the abdomen via the base of the umbilicus. Through a combination of foetal growth and bowel maturation the bowel resides in the abdominal cavity and undergoes a 270o counterclockwise twist. At the conclusion of this process the ligament of Treitz lies to the left of the spine and the caecum in the right lower quadrant. Malrotation occurs when the rotational process described is incomplete. Typically the duodenal loop lies to the left of the caecum and therefore lacks 90 o of its 270o rotation. It becomes fixed in this position with peritoneal attachments (Ladds bands).

Symptoms
Bilious vomiting is the cardinal symptom and sign. In most cases there are no antecedent symptoms. If untreated then gut perfusion may be impaired with resultant development of further symptoms.

Diagnosis
The main problem is that the infant develops mid gut volvulus. In this situation the bowel undergoes a 720o twist, the bowel viability depends upon a narrow mesentery containing the superior mesenteric artery. To ascertain whether this has occurred the two main tests include an abdominal ultrasound scan to determine the relationship between the superior mesenteric artery and vein (normally SMA lies to the left of the SMV). This test is complemented with an upper GI contrast series and this aims to establish that the DJ flexure is correctly sited to the left of the vertebral bodies.

Treatment
Laparotomy and division of adhesional bands (Ladds procedure). The division of the congenital adhesions allows widening of the small bowel mesentery. The bowel is untwisted and assessed for viability. The bowel is returned to the abdominal cavity in the non rotated fashion with the small bowel on the right hand side and the large bowel on the left, the caecum is positioned in the left upper quadrant. Because the caecum is located in this new location many surgeons will also perform an appendicectomy due to the diagnostic difficulty posed in diagnosing appendicitis in this group of patients.

How well did you know this?
1
Not at all
2
3
4
5
Perfectly
511
Q

Which one of the following would cause a rise in the carbon monoxide transfer factor (TLCO)?

Emphysema

Pulmonary embolism

Pulmonary haemorrhage

Pneumonia

Pulmonary fibrosis

A

Transfer factor raised: asthma, haemorrhage, left-to-right shunts, polycythaemia
low: everything else
Where alveolar haemorrhage occurs the TLCO tends to increase due to the enhanced uptake of carbon monoxide by intra-alveolar haemoglobin.

Transfer factor

The transfer factor describes the rate at which a gas will diffuse from alveoli into blood. Carbon monoxide is used to test the rate of diffusion. Results may be given as the total gas transfer (TLCO) or that corrected for lung volume (transfer coefficient, KCO)

Causes of a raised TLCO
asthma
pulmonary haemorrhage (Wegener’s, Goodpasture’s)
left-to-right cardiac shunts
polycythaemia
hyperkinetic states
male gender, exercise
Causes of a lower TLCO
pulmonary fibrosis
pneumonia
pulmonary emboli
pulmonary oedema
emphysema
anaemia
low cardiac output

KCO also tends to increase with age. Some conditions may cause an increased KCO with a normal or reduced TLCO
pneumonectomy/lobectomy
scoliosis/kyphosis
neuromuscular weakness
ankylosis of costovertebral joints e.g. ankylosing spondylitis

How well did you know this?
1
Not at all
2
3
4
5
Perfectly
512
Q

A 75 year old man is admitted with sudden onset severe generalised abdominal pain, vomiting and a single episode of bloody diarrhoea. On examination, he looks unwell and is in uncontrolled atrial fibrillation. Although diffusely tender his abdomen is soft. What is the most likely diagnosis?

Pancreatitis

Infective diarrhoea

Ischaemic colitis

Crohns disease

Mesenteric infarction

A

Pain out of proportion to physical signs, AF and generalized abdominal pain suggest widespread infarction.

Acute abdominal pain-diagnoses

Conditions presenting with acute abdominal pain
Condition Features Investigations Management
Appendicitis History of migratory pain.
Fever.
Anorexia.
Evidence of right iliac fossa tenderness.
Mild pyrexia. Differential white cell count
Pregnancy test
C-Reactive protein
Amylase
Urine dipstick testing Appendicectomy
Mesenteric adenitis Usually recent upper respiratory tract infection.
High fever.
Generalised abdominal discomfort- true localised pain and signs are rare. Full blood count- may show slightly raised white cell count
Urine dipstick often normal
Abdominal ultrasound scan - usually no free fluid Conservative management- appendicectomy if diagnostic doubt
Mittelschmerz Only seen in females
Mid cycle pain
Usually occurs two weeks after last menstrual period
Pain usually has a supra-pubic location
Usually subsides over a 24-48 hour period. Full blood count- normal
Urine dipstick- normal
Abdominal and pelvic ultrasound- may show a trace of pelvic free fluid Manage conservatively if doubt or symptoms fail to settle then laparoscopy
Fitz-Hugh Curtis syndrome Disseminated infection with Chlamydia.
Usually seen in females.
Consists of evidence of pelvic inflammatory disease together with peri-hepatic inflammation and subsequent adhesion formation. Abdominal ultrasound scan- may show free fluid
High vaginal swabs - may show evidence of sexually transmitted infections Usually medically managed- doxycycline or azithromycin
Abdominal aortic aneurysm (ruptured) Sudden onset of abdominal pain radiating to the back in older adults (look for risk factors).
Collapse.
May be moribund on arrival in casualty, more stable if contained haematoma.
Careful clinical assessment may reveal pulsatile mass. Patients who are haemodynamically stable should have a CT scan Unstable patients should undergo immediate surgery (unless it is not in their best interests).
Those with evidence of contained leak on CT should undergo immediate surgery
Increasing unruptured aneurysmal size is an indication for urgent surgical intervention (that can wait until the next working day)
Perforated peptic ulcer Sudden onset of pain (usually epigastric).
Often preceding history of upper abdominal pain.
Soon develop generalised abdominal pain.
On examination may have clinical evidence of peritonitis. Erect CXR may show free air. A CT scan may be indicated where there is diagnostic doubt Laparotomy (laparoscopic surgery for perforated peptic ulcers is both safe and feasible in experienced hands)
Intestinal obstruction Colicky abdominal pain and vomiting (the nature of which depends on the level of the obstruction).
Abdominal distension and constipation (again depending upon site of obstruction).
Features of peritonism may occur where local necrosis of bowel loops is occurring. A plain abdominal film may help with making the diagnosis. A CT scan may be useful where diagnostic uncertainty exists In those with a virgin abdomen a lower and earlier threshold for laparotomy should exist than in those who may have adhesional obstruction
Mesenteric infarction Embolic events present with sudden pain and forceful evacuation.
Acute on chronic events usually have a longer history and previous weight loss.
On examination the pain is typically greater than the physical signs would suggest. Arterial pH and lactate
Arterial phase CT scanning is the most sensitive test

How well did you know this?
1
Not at all
2
3
4
5
Perfectly
513
Q

A 73 year old lady is admitted with brisk rectal bleeding. Despite attempts at resuscitation the bleeding proceeds to cause haemodynamic compromise. An upper GI endoscopy is normal. A mesenteric angiogram is performed and a contrast blush is seen in the region of the sigmoid colon. The radiologist decides to embolise the vessel supplying this area. At what spinal level does it leave the aorta?

L2

L1

L4

L3

T10

A

The inferior mesenteric artery leaves the aorta at L3. It supplies the left colon and sigmoid. Its proximal continuation to communicate with the middle colic artery is via the marginal artery.

Levels

Transpyloric plane
Level of the body of L1

Pylorus stomach
Left kidney hilum (L1- left one!)
Fundus of the gallbladder
Neck of pancreas
Duodenojejunal flexure
Superior mesenteric artery
Portal vein
Left and right colic flexure
Root of the transverse mesocolon
2nd part of the duodenum
Upper part of conus medullaris
Spleen

Can be identified by asking the supine patient to sit up without using their arms. The plane is located where the lateral border of the rectus muscle crosses the costal margin.

Anatomical planes
Subcostal plane Lowest margin of 10th costal cartilage
Intercristal plane Level of body L4 (highest point of iliac crest)
Intertubercular plane Level of body L5

Common level landmarks
Inferior mesenteric artery L3
Bifurcation of aorta into common iliac arteries L4
Formation of IVC L5 (union of common iliac veins)
Diaphragm apertures
Vena cava T8
Oesophagus T10
Aortic hiatus T12

How well did you know this?
1
Not at all
2
3
4
5
Perfectly
514
Q

A 74 year old male is admitted to the Emergency Department with a fall. He is known to have rheumatoid arthritis and is on methotrexate and paracetamol. He lives alone in a bungalow and enjoys playing golf. He is independent with his ADLs. He complains of left groin pain, therefore has a hip x-ray which confirms a displaced intracapsular fracture. What is the best course of action?

Cemented hemiarthroplasty

Uncemented hemiarthroplasty

Total hip replacement

Dynamic hip screw

Intramedullary nail

A

This patient has pre-existing joint disease, good level of activity and a relatively high life expectancy, therefore THR is preferable to hemiarthroplasty.

Hip fractures

Background
Neck of femur (NOF) fracture is a common orthopaedic presentation, with over 65000 fractures in the UK per year. Like many orthopaedic injuries, there is a bimodal age distribution. It is imperative to distinguish between the high energy injury in a young patient, and the low energy osteoporotic fracture in the elderly, as their management aims are very different:

Young patient - Usually high energy trauma (e.g road traffic accident, horse riding) and needs treating in accordance with Advanced Trauma Life Support (ATLS) principles. Will often have associated injuries. Aim is to retain the patients own anatomy, and optimise their function.

Elderly patient - Predominantly female, fall from standing height (fragility fracture). Often patients have multiple comorbidities that will ultimately dictate their prognosis. Aim of orthopaedic treatment is to immediately regain patient mobility so that morbidity (infection, thromboembolic events, pressure sores etc) and mortality associated with prolonged bed rest is avoided. Left untreated, a neck of femur fracture can be considered a terminal event. Historically, mortality associated with elderly hip fracture is 10% at one month, and 30% at one year. However, this has been improved in the UK with the introduction of multidisciplinary, orthogeriatric lead care and the National Hip Fracture Database and Best Practice Tariff.

Pertinent anatomy
Osteology - normal neck-shaft angle is 130 +/- 7 degrees, and 10 +/- 7 degrees of neck anteversion.
Vascular supply - The predominant blood supply to the femoral head and neck is from the medial and lateral femoral circumflex arteries (branches of profunda femoris). These anastomose and pierce the joint capsule at the base of the neck, mainly posteriorly. There is a small vascular contribution from the artery of the ligament teres. Understanding the blood supply is fundamental to the decision making process in treating NOF fractures.

Presentation and initial management
Typically, patients present with pain in the hip/groin, a shortened, abducted, externally rotated leg (due to the unopposed pull of the muscles that act across the hip joint) and the inability to straight-leg-raise. With undisplaced fractures, signs are more subtle.
High energy injuries should be treated in line with ATLS principles. All patients should be fluid resuscitated, have adequate pain relief (often with a fascio-iliiaca nerve block), and be optimised for surgery. In addition, elderly patients should be assessed by an orthogeriatrician.

Imaging
Anteroposterior and cross-table lateral plain radiographs are sufficient to diagnose the majority of NOF fractures. If the fracture extends below the level of the lesser trochanter, or there is any possibility of pathological fracture, full length femur views are essential to plan surgery.

Where there is a high index of suspicion of fracture, but plain radiographs are inconclusive, gold standard investigation is MRI. However, if unavailable within 24 hours, or if the patient will not tolerate MRI, CT is appropriate. The majority of fractures can be seen with modern CT techniques, and so this is becoming first line in many hospitals.

Classification
There has been a move away from named classification systems towards descriptive classification systems.
Two main types of NOF exist: Intra-capsular, and extra-capsular. Extra-capsular fractures are further divided into pertrochanteric or subtrochanteric (within 5cm distal to the lesser trochanter). All fractures are then described as undisplaced, minimally displaced, or displaced.
Femoral neck and head blood supply disruption is common with intracapsular NOF fractures, and rare with extracapsular fractures. This fundamental principle underpins the practise of arthroplasty for intracapsular fractures, and fixation for extracapsular fractures.

If you wish to use a named classification system, the most commonly used are below:
Elderly intracapsular - Garden Classification
Young intrasapsular - Pauvels Classification
Intertrochanteric - Evans
Subtrochanteric - Russell Taylor

Treatment
In general, NOF fractures are treated operatively except if the patient is deemed unlikely to survive an anaesthetic. Best Practice Tarif (BPT) dictates that surgery should happen within 36 hours, as delay of greater than 48 hours is associated with increased morbidity and mortality. Below are suggested algorithms for the treatment of NOF.
* The priority with the young patient is to retain the femoral head if possible, even with a displaced intracapsular fracture. The risk of avascular necrosis and non-union (and therefore revision surgery) associated with internal fixation needs weighing up against the sequelae of total hip replacement in the young (wear, dislocation, revision). Discussion is necessary with the patient, on a case by case basis.

** Undisplaced fractures in the elderly can be treated with internal fixation, often with cannulated screws. This is appropriate for valgus impacted subcapital fractures which are inherently stable, to prevent secondary displacement. This does still carry the risk of AVN or non-union, and therefore a future revision. For this reason, many surgeons advocate arthroplasty as a single surgery.

*** NICE guidance - patients who fulfil these criteria should be offered total hip replacement which conveys better function and prosthetic survivorship, compared with hemiarthroplasty, but at an increased risk of dislocation.

  • Intertrochanteric fractures vary greatly in their stability. If the trochanter (and therefore lateral wall), and medial calcar is in tact, then the fracture configuration bears stability. This can be treated with a DHS, as collapse of the fracture is predictable. Where either or both structures are involved in the fracture, stability becomes compromised and many surgeons will favour using an intramedullary device. This is an ongoing debate, and difficult to test in an exam setting.

Post operative management
Patients should be mobilised fully weight bearing where possible. Care is multidisciplinary in its delivery. Elderly patients should have orthogeriatrician assessment of comorbidity, and bone health with secondary prevention measures if appropriate. There should be early involvement of physiotherapy and occupational therapy services. For further guidance see sources listed below.

NICE clinical guidance on hip fracture: https://www.nice.org.uk/guidance/cg124
Best Practice Tarif: www.nhfd.co.uk/20/hipfractureR…/Best%20Practice%20Tariff%20User%20Guide.pdf National Hip Fracture Database: www.nhfd.co.uk/

How well did you know this?
1
Not at all
2
3
4
5
Perfectly
515
Q

A 43 year old lady undergoes a live related renal transplant. At the conclusion of the operation she has a good urine output and the graft appeared well perfused. On the ward she suddenly becomes anuric. What is the most likely cause?

Renal artery stenosis

Renal vein thrombosis

Renal artery thrombosis

Hyperacute rejection

Acute rejection

A

Sudden loss of urine output is most commonly due to a blocked catheter. However, if this is excluded (and is not included in the options) the most worrisome cause is arterial thrombosis. This will often be a delayed diagnosis and the rate of graft loss is high.
Complications of renal transplantation

A number of complications may occur following renal transplantation. A critical aspect of post operative care is evaluation of graft function. Post operatively, urine output is the most readily available, and easily measured, indicator of graft function. If an individual was relatively anuric pre-transplant and has a good urine output following surgery then this is more useful than it would be in someone who had a higher volume diuresis prior to transplantation. Recipients can be divided into three main groups following renal transplantation, with regard to their graft function:
Immediate graft function; brisk diuresis and falling serum creatinine
Slow graft function; modest urine output and slowly falling creatinine levels
Delayed graft function; defined as need for dialysis post transplant

Decreased urine output following surgery can be the result of hypovolvaemia or a blocked catheter (commonest causes). Other important causes include rejection, or a vascular complication.

Vascular complications
These may involve the donor vessels, those of the recipient or both. Renal artery thrombosis usually occurs early post transplant, but is uncommon with an incidence of less than 1%. It typically results in graft loss. It usually occurs as a result of a technical problem such a vessel torsion or sub intimal flaps. The usual presenting feature is a sudden cessation of urine output. When suspected, the occlusion is usually well demonstrated with duplex scanning. Ideally immediate surgical re-exploration should occur. Sadly, the graft has usually been lost by this stage and will require graft nephrectomy. Renal vein thrombosis is not as common as arterial graft thrombosis and the usual presenting features include discomfort at the graft site and swelling of the graft associated with loss of urine output. Again, duplex scanning is indicated. Unfortunately, this complication is also associated with a high incidence of graft loss.
Over a longer time frame (typically months) some individuals will develop renal artery stenosis. These individuals will typically develop hypertension and over time graft function will decline as hypertensive nephropathy occurs. It is usually demonstrated by duplex scanning and is usually amenable to endovascular intervention.

Urological complications
Urinary tract complications manifesting as leakage or obstruction are common complications following renal transplantation and occur in up to 10% of patients. The main underlying cause is the relatively poor blood supply to the transplanted ureter. Patients typically present relatively early in the first 5 weeks following transplantation with pain and swelling at the graft site. Imaging with USS is often the initial test. Therapeutic options include surgical re-implantation of the ureter for large leaks and stent insertion and nephrostomy placement for smaller leaks.

Lymphocele
These do not generally occur until 2 weeks or longer after surgery. They are, however, relatively common and may be seen in up to 18% of patients. Symptoms usually occur as a result of mass effect with compression of adjacent structures. These include the vessels supplying both the graft, with deterioration in graft function, the ureter, with alteration in urine output and the recipients lower limb vessels, with development of leg swelling. Creation of a laparoscopic or open peritoneal window is a favored treatment.

Rejection
Four types of graft rejection are recognised; hyperacute, accelerated acute, acute and chronic.

Type of rejection Key features
Hyperacute Occurs within minutes of clamp release
Due to pre formed antibodies
Immediate loss of graft occurs
Accelerated acute Occurs in first few days following surgery
Involved both cellular and antibody mediated injury
Pre-sensitisation of the donor is a common cause
Acute Traditionally the most common type of rejection
Seen days to weeks after surgery
Predominantly a cell mediated process mediated by lymphocytes
Organ biopsy demonstrates cellular infiltrates and graft cell apoptosis
Chronic Increasingly common problem
Typically; graft atrophy and atherosclerosis are seen. Fibrosis often occurs as a late event

How well did you know this?
1
Not at all
2
3
4
5
Perfectly
516
Q

Which of the following statements relating to abnormal coagulation is false?

Warfarin affects the synthesis of factor 2,7,9,10

The prothrombin time is prolonged in Haemophilia A

Cholestatic jaundice can cause vitamin K deficiency

Disseminated intravascular coagulation is associated with thrombocytopenia

Massive transfusion is associated with reduced levels of factor 5 and 8

A

In haemophilia A the APTT is prolonged and there is reduced levels of factor 8:C. The bleeding time and PT are normal. Cholestatic jaundice prevents the absorption of the fat soluble vitamin K. Massive transfusion (>10u blood or equivalent to the blood volume of a person) puts the patient at risk of thrombocytopaenia, factor 5 and 8 deficiency.

Abnormal coagulation

Cause Factors affected
Heparin Prevents activation factors 2,9,10,11
Warfarin Affects synthesis of factors 2,7,9,10
DIC Factors 1,2,5,8,11
Liver disease Factors 1,2,5,7,9,10,11

Interpretation blood clotting test results
Disorder APTT PT Bleeding time
Haemophilia Increased Normal Normal
von Willebrand’s disease Increased Normal Increased
Vitamin K deficiency Increased Increased Normal

How well did you know this?
1
Not at all
2
3
4
5
Perfectly
517
Q

A neurosurgeon decides to explore the cavernous sinus via the extended endonasal route. Which of the structures listed below is unlikely to be encountered with such an approach.

Oculomotor nerve

Internal carotid artery

Opthalmic nerve

Abducens nerve

Optic nerve

A

Mnemonic for contents of cavernous sinus:
O TOM CAT

Occulomotor nerve (III)
Trochlear nerve (IV)
Ophthalmic nerve (V1)
Maxillary nerve (V2)
Carotid artery
Abducent nerve (VI)
T

OTOM=lateral wall components
CA= components within sinus
The optic nerve lies above and outside the cavernous sinus.

How well did you know this?
1
Not at all
2
3
4
5
Perfectly
518
Q

A 23 year old man is stabbed in the groin, several structures are injured and the adductor longus muscle has been lacerated. Which of the following nerves is responsible for the innervation of adductor longus?

Femoral nerve

Obturator nerve

Sciatic nerve

Common peroneal nerve

Ilioinguinal nerve

A

The adductors are innervated by the obturator nerve.

Adductor longus

Origin Anterior body of pubis
Insertion Middle third of linea aspera
Action Adducts and flexes the thigh, medially rotate the hip
Innervation Anterior division of obturator nerve (L2, L3, L4)

How well did you know this?
1
Not at all
2
3
4
5
Perfectly
519
Q

Which of the following statements relating to audit and governance is untrue?

An audit standard is a threshold of compliance with an audit criterion

Sample size calculations are an important part of audit planning

Clinical audit is part of clinical governance

Audits should be performed regularly when a novel surgical technique is introduced and where there is little knowledge of anticipated complications or outcomes

An audit criterion is a measurable outcome of care, aspect of practice or capacity

A

6 pillars of clinical governance:
Clinical effectiveness
Research and development
Openness
Risk management
Education and training
Clinical audit
Audits should compare performance against known standards. Where a novel technique is being introduced standards are unlikely to exist, sample sizes cannot therefore be accurately calculated. This is an example of research, which is not an audit.

How well did you know this?
1
Not at all
2
3
4
5
Perfectly
520
Q

A 55 year old man is attending a health screening centre and is concerned about risk factors for the development of gastric cancer. Which of the factors described below carries the least risk of developing the condition subsequently

Polya gastrectomy for antral ulcer

Atrophic gastritis

Intestinal metaplasia of columnar type at the gastric cardia

Gastric polyp showing medium grade dysplasia

Long term therapy with sucralfate

A

Although some acid lowering procedures increase the risk of gastric cancer the use of sucralfate does not, at the present time, seem to increase the risk.

Gastric cancer

Overview
There are 700,000 new cases of gastric cancer worldwide each year. It is most common in Japan and less common in western countries. It is more common in men and incidence rises with increasing age. The exact cause of many sporadic cancer is not known, however, familial cases do occur in HNPCC families. In addition, smoking and smoked or preserved foods increase the risk. Japanese migrants retain their increased risk (decreased in subsequent generations). The distribution of the disease in western countries is changing towards a more proximal location (perhaps due to rising obesity).

Pathology
There is some evidence of support a stepwise progression of the disease through intestinal metaplasia progressing to atrophic gastritis and subsequent dysplasia, through to cancer. The favoured staging system is TNM. The risk of lymph node involvement is related to size and depth of invasion; early cancers confined to submucosa have a 20% incidence of lymph node metastasis. Tumours of the gastro-oesophageal junction are classified as below:

Type 1 True oesophageal cancers and may be associated with Barrett’s oesophagus.
Type 2 Carcinoma of the cardia, arising from cardiac type epithelium
or short segments with intestinal metaplasia at the oesophagogastric junction.
Type 3 Sub cardial cancers that spread across the junction. Involve similar nodal stations to gastric cancer.

Groups for close endoscopic monitoring
Intestinal metaplasia of columnar type
Atrophic gastritis
Low to medium grade dysplasia
Patients who have previously undergone resections for benign peptic ulcer disease (except highly selective vagotomy).

Referral to endoscopy

Patients of any age with dyspepsia and any of the following Patients without dyspepsia Worsening dyspepsia
Chronic gastrointestinal bleeding Dysphagia Barretts oesophagus
Dysphagia Unexplained abdominal pain or weight loss Intestinal metaplasia
Weight loss Vomiting Dysplasia
Iron deficiency anaemia Upper abdominal mass Atrophic gastritis
Upper abdominal mass Jaundice Patient aged over 55 years with unexplained or persistent dyspepsia

Upper GI endoscopy performed for dyspepsia. The addition of dye spraying (as shown in the bottom right) may facilitate identification of smaller tumours

Staging
CT scanning of the chest abdomen and pelvis is the routine first line staging investigation in most centres.
Laparoscopy to identify occult peritoneal disease
PET CT (particularly for junctional tumours)

Treatment
Proximally sited disease greater than 5-10cm from the OG junction may be treated by sub total gastrectomy
Total gastrectomy if tumour is <5cm from OG junction
For type 2 junctional tumours (extending into oesophagus) oesophagogastrectomy is usual
Endoscopic sub mucosal resection may play a role in early gastric cancer confined to the mucosa and perhaps the sub mucosa (this is debated)
Lymphadenectomy should be performed. A D2 lymphadenectomy is widely advocated by the Japanese, the survival advantages of extended lymphadenectomy have been debated. However, the overall recommendation is that a D2 nodal dissection be undertaken.
Most patients will receive chemotherapy either pre or post operatively.

Prognosis

UK Data

Disease extent Percentage 5 year survival
All RO resections 54%
Early gastric cancer 91%
Stage 1 87%
Stage 2 65%
Stage 3 18%

Operative procedure

Total Gastrectomy , lymphadenectomy and Roux en Y anastomosis

General anaesthesia
Prophylactic intravenous antibiotics
Incision: Rooftop.
Perform a thorough laparotomy to identify any occult disease.
Mobilise the left lobe of the liver off the diaphragm and place a large pack over it. Insert a large self retaining retractor e.g. omnitract or Balfour (take time with this, the set up should be perfect). Pack the small bowel away.
Begin by mobilising the omentum off the transverse colon.
Proceed to detach the short gastric vessels.
Mobilise the pylorus and divide it at least 2cm distally using a linear cutter stapling device.
Continue the dissection into the lesser sac taking the lesser omentum and left gastric artery flush at its origin.
The lymph nodes should be removed en bloc with the specimen where possible.
Place 2 stay sutures either side of the distal oesophagus. Ask the anaesthetist to pull back on the nasogastric tube. Divide the distal oesophagus and remove the stomach.
The oesphago jejunal anastomosis should be constructed. Identify the DJ flexure and bring a loop of jejunum up to the oesophagus (to check it will reach). Divide the jejunum at this point. Bring the divided jejunum either retrocolic or antecolic to the oesophagus. Anastamose the oesophagus to the jejunum, using either interrupted 3/0 vicryl or a stapling device. Then create the remainder of the Roux en Y reconstruction distally.
Place a jejunostomy feeding tube.
Wash out the abdomen and insert drains (usually the anastomosis and duodenal stump). Help the anaesthetist insert the nasogastric tube (carefully!)
Close the abdomen and skin.
Enteral feeding may commence on the first post-operative day. However, most surgeons will leave patients on free NG drainage for several days and keep them nil by mouth.

How well did you know this?
1
Not at all
2
3
4
5
Perfectly
521
Q

A 43 year old lady with hypertension is suspected of having a phaeochromocytoma. Which of the following investigations is most likely to be beneficial in this situation?

Dexamethasone suppression test

Urinary 5-Hydroxyindoleacetic Acid (5-HIAA)

Histamine provocation test

Tyramine provocation test

Urinary metanephrines measurement

A

Urinary metanephrine measurements are not completely specific but constitute first line assessment and more accurate than urinary VMA measurements. Stimulation tests of any sort are not justified in first line assessments.
Phaeochromocytoma and adrenal lesions

Phaeochromocytoma
Neuroendocrine tumour of the chromaffin cells of the adrenal medulla. Hypertension and hyperglycaemia are often found.
10% of cases are bilateral.
10% occur in children.
11% are malignant (higher when tumour is located outside the adrenal).
10% will not be hypertensive.

Familial cases are usually linked to the Multiple endocrine neoplasia syndromes (considered under its own heading).

Most tumours are unilateral (often right sided) and smaller than 10cm.

Diagnosis
Urine and plasma metanephrine levels.

CT and MRI scanning are both used to localise the lesion.

Treatment
Patients require medical therapy first. An irreversible alpha adrenoreceptor blocker should be given, although minority may prefer reversible blockade(1). Labetolol may be co-administered for cardiac chronotropic control. Isolated beta blockade should not be considered as it will lead to unopposed alpha activity.

These patients are often volume depleted and will often require moderate volumes of intra venous normal saline perioperatively.

Once medically optimised the phaeochromocytoma should be removed. Most adrenalectomies can now be performed using a laparoscopic approach(2). The adrenals are highly vascular structures and removal can be complicated by catastrophic haemorrhage in the hands of the inexperienced. This is particularly true of right sided resections where the IVC is perilously close. Should the IVC be damaged a laparotomy will be necessary and the defect enclosed within a Satinsky style vascular clamp and the defect closed with prolene sutures. Attempting to interfere with the IVC using any instruments other than vascular clamps will result in vessel trauma and make a bad situation much worse.

Incidental adrenal lesions
Adrenal lesions may be identified on CT scanning performed for other reasons(3). Factors suggesting benign disease on CT include(4):
Size less than 3cm
Homogeneous texture
Lipid rich tissue
Thin wall to lesion

All patients with incidental lesions should be managed jointly with an endocrinologist and full work up as described above. Patients with functioning lesions or those with adverse radiological features (Particularly size >3cm) should proceed to surgery.

How well did you know this?
1
Not at all
2
3
4
5
Perfectly
522
Q

A healthy man has a blood pressure of 120/80 mmHg and an intra cranial pressure of 17 mmHg. What is the approximate cerebral perfusion pressure?

103 mmHg

63 mmHg

83 mmHg

91 mmHg

76 mmHg

A

Cerebral perfusion pressure= Mean arterial pressure - intra cranial pressure
The mean arterial pressure can be calculated as:
MAP= Diastolic pressure+ 0.333(Systolic pressure- Diastolic pressure)
In this situation the MAP = 93.
The ICP is subtracted from this value; 93 - 17 = 76

Cerebral perfusion pressure

The cerebral perfusion pressure (CPP) is defined as being the net pressure gradient causing blood flow to the brain. The CPP is tightly autoregulated to maximise cerebral perfusion. A sharp rise in CPP may result in a rising ICP, a fall in CPP may result in cerebral ischaemia. It may be calculated by the following equation:

CPP= Mean arterial pressure - Intra cranial pressure

Following trauma, the CPP has to be carefully controlled and the may require invasive monitoring of the ICP and MAP.

How well did you know this?
1
Not at all
2
3
4
5
Perfectly
523
Q

Which ligament keeps the head of the radius connected to the radial notch of the ulna?

Annular (orbicular) ligament

Quadrate ligament

Radial collateral ligament of the elbow

Ulnar collateral ligament

Radial collateral ligament

A

The annular ligament connects the radial head to the radial notch of the ulna.
Radius

The radius is one of the two long forearm bones that extends from the lateral side of the elbow to the thumb side of the wrist. It has two expanded ends, of which the distal end is the larger. Key points relating to its topography and relations are outlined below;

Upper end
Articular cartilage- covers medial > lateral side
Articulates with radial notch of the ulna by the annular ligament
Muscle attachment- biceps brachii at the tuberosity

Shaft
Muscle attachment
Upper third of the body Supinator
Flexor digitorum superficialis
Flexor pollicis longus
Middle third of the body Pronator teres
Lower quarter of the body Pronator quadratus
Tendon of supinator longus

Lower end
Quadrilateral
Anterior surface- capsule of wrist joint
Medial surface- head of ulna
Lateral surface- ends in the styloid process
Posterior surface: 3 grooves containing:
1. Tendons of extensor carpi radialis longus and brevis
2. Tendon of extensor pollicis longus
3. Tendon of extensor indicis

How well did you know this?
1
Not at all
2
3
4
5
Perfectly
524
Q

A 34 year old male is being examined in the pre-operative assessment clinic. A murmur is identified in the 4th intercostal space just next to the left side of the sternum. From where is it most likely to have originated?

Mitral valve

Aortic valve

Pulmonary valve

Right ventricular aneurysm

Tricuspid valve

A

The tricuspid valve is generally referred to being best auscultated adjacent to the sternum. The plane of projected sound from the mitral area is best heard in the region of the cardiac apex.

Heart sounds

Sites of auscultation
Valve Site
Pulmonary valve Left second intercostal space, at the upper sternal border
Aortic valve Right second intercostal space, at the upper sternal border
Mitral valve Left fifth intercostal space, just medial to mid clavicular line
Tricuspid valve Left fourth intercostal space, at the lower left sternal border

How well did you know this?
1
Not at all
2
3
4
5
Perfectly
525
Q

A 24 year old man is involved in a fight and his face is cut with a knife. The wound lies immediately anterior to the tragus of the ear and extends anteriorly. The wound is surgically explored and the laceration is found to be mainly superficial. It extends slightly more deeply immediately inferior to the main trunk of the facial nerve. Bleeding is observed, from which of the following is it most likely to originate?

External carotid artery

Retromandibular vein

Occipital artery

Maxillary artery

Ascending pharyngeal artery

A

The retromandibular vein lies slightly more deeply than the facial nerve in the parotid gland. It is formed from the maxillary and superficial temporal vein.

Retromandibular vein

Formed by a union of the maxillary vein and superficial temporal vein
It descends through the parotid gland and bifurcates within it
The anterior division passes forwards to join the facial vein, the posterior division is one of the tributaries of the external jugular vein

How well did you know this?
1
Not at all
2
3
4
5
Perfectly
526
Q

What type of visual field defect is most likely to be noted in a patient with a craniopharyngioma?

Lower bitemporal hemianopia

Upper bitemporal hemianopia

Right superior quadranopia

Right homonymous hemianopia

Left homonymous hemianopia

A

Lesions at the optic chiasm classically produce a bitemporal hemianopia, however note lesions that spread up from below ie pituitary tumours, the defect is worse in the upper fields and if a lesion spreads down from above ie craniopharyngiomas, the visual defect is worse in the lower quadrants. Therefore this patient is likely to have a lower bitemporal hemianopia.

Visual field defects

left homonymous hemianopia means visual field defect to the left, i.e. Lesion of right optic tract
homonymous quadrantanopias: PITS (Parietal-Inferior, Temporal-Superior)
incongruous defects = optic tract lesion; congruous defects = optic radiation lesion or occipital cortex

Homonymous hemianopia
Incongruous defects: lesion of optic tract
Congruous defects: lesion of optic radiation or occipital cortex
Macula sparing: lesion of occipital cortex

Homonymous quadrantanopias
Superior: lesion of temporal lobe
Inferior: lesion of parietal lobe
Mnemonic = PITS (Parietal-Inferior, Temporal-Superior)

Bitemporal hemianopia
Lesion of optic chiasm
Upper quadrant defect > lower quadrant defect = inferior chiasmal compression, commonly a pituitary tumour
Lower quadrant defect > upper quadrant defect = superior chiasmal compression, commonly a craniopharyngioma

How well did you know this?
1
Not at all
2
3
4
5
Perfectly
527
Q

Which of the following fractures names best accounts for the injury seen in a 14 year old boy who jumps off a 10 foot wall and lands on both feet and whose imaging shows a bimalleolar fracture of the right ankle?

Pott’s

Barton’s

Galeazzi

Colles’

Bennett’s

A

Eponymous fractures

Colles’ fracture (dinner fork deformity)
Fall onto extended outstretched hand
Classical Colles’ fractures have the following 3 features:

  1. Transverse fracture of the radius
  2. 1 inch proximal to the radio-carpal joint
  3. Dorsal displacement and angulation

Smith’s fracture (reverse Colles’ fracture)
Volar angulation of distal radius fragment (Garden spade deformity)
Caused by falling backwards onto the palm of an outstretched hand or falling with wrists flexed

Bennett’s fracture
Intra-articular fracture of the first carpometacarpal joint
Impact on flexed metacarpal, caused by fist fights
X-ray: triangular fragment at ulnar base of metacarpal

Monteggia’s fracture
Dislocation of the proximal radioulnar joint in association with an ulna fracture
Fall on outstretched hand with forced pronation
Needs prompt diagnosis to avoid disability

Galeazzi fracture
Radial shaft fracture with associated dislocation of the distal radioulnar joint
Direct blow

Pott’s fracture
Bimalleolar ankle fracture
Forced foot eversion

Barton’s fracture
Distal radius fracture (Colles’/Smith’s) with associated radiocarpal dislocation
Fall onto extended and pronated wrist
Involvement of the joint is a defining feature

Holstein Lewis Fracture
A HolsteinLewis fracture is a fracture of the distal third of the humerus resulting in entrapment of the radial nerve.
The radial nerve is one of the major nerves of the upper limb. It innervates all of the muscles in the extensor compartments of the arm.
Conservative treatment includes reduction and use of a functional brace
Vascular injury may require open surgery

How well did you know this?
1
Not at all
2
3
4
5
Perfectly
528
Q

At which of the following sites is the most water absorbed?

Right colon

Left colon

Stomach

Jejunum

Duodenum

A

Water absorption in the gastrointestinal tract predominantly occurs in the small bowel (jejunum and ileum). The colon is an important site of water absorption, however, its overall contribution is relatively small. The importance of the colonic component to water absorption may increase following extensive small bowel resections.

Water absorption

During a 24 hours period the average person will ingest up to 2000ml of liquid orally. In addition a further 8000ml of fluid will enter the small bowel as gastrointestinal secretions. Intestinal water absorption is a passive process and is related to solute load. In the jejunum the active absorption of glucose and amino acids will create a concentration gradient that water will flow across. In the ileum most water is absorbed by a process of facilitated diffusion (with sodium).
Approximately 150ml of water enters the colon daily, most is absorbed, the colon can adapt to, and increase this amount following resection.

How well did you know this?
1
Not at all
2
3
4
5
Perfectly
529
Q

A 28 year old Indian woman, who is 18 weeks pregnant, presents with increasing shortness of breath, chest pain and coughing clear sputum. She is apyrexial, blood pressure is 140/80 mmHg, heart rate 130 bpm and saturations 94% on 15L oxygen. On examination, there is a mid diastolic murmur, there are bibasal crepitations and mild pedal oedema. She suddenly deteriorates and has a respiratory arrest. Her chest x-ray shows a whiteout of both of her lungs. What is the most likely explanation?

Acute exacerbation asthma

Pulmonary embolus

Mitral regurgitation

Mitral valve stenosis

Aortic dissection

A

Mitral stenosis is the commonest cause of cardiac abnormality occurring in pregnant women. Mitral stenosis is becoming less common in the UK population, however should be considered in women from countries where there is a higher incidence of rheumatic heart disease. Mitral stenosis causes a mid diastolic murmur which may be difficult to auscultate unless the patient is placed into the left lateral position. These patients are at risk of atrial fibrillation (up to 40%), which can also contribute to rapid decompensation such as pulmonary oedema (hence cxr ‘whiteout’ of lungs). Physiological changes in pregnancy may cause an otherwise asymptomatic patient to suddenly deteriorate. Balloon valvuloplasty is the treatment of choice.

How well did you know this?
1
Not at all
2
3
4
5
Perfectly
530
Q

A 25 year old cyclist is hit by a bus traveling at 30mph. He was not wearing a helmet. He arrives with a GCS of 3/15 and is intubated. A CT scan shows evidence of cerebral contusion but no localising clinical signs are present. What is the most appropriate course of action?

Burr hole decompression

Decompressive craniotomy

Insertion of intra cranial pressure monitoring device

Administration of intravenous mannitol

Parietotemporal craniotomy

A

This patient may well develop raised ICP over the next few days and intracranial pressure monitoring will help with management.

Head injury management- NICE Guidelines

Summary of guidelines
All patients should be assessed within 15 minutes on arrival to A&E
Document all 3 components of the GCS
If GCS <8 or = to 8, consider stabilising the airway
Full spine immobilisation until assessment if:

  • GCS < 15
  • neck pain/tenderness
  • paraesthesia extremities
  • focal neurological deficit
  • suspected c-spine injury

If a c-spine injury is suspected a 3 view c-spine x-ray is indicated. CT c-spine is preferred if:
- Intubated
- GCS <13
- Normal x-ray but continued concerns regarding c-spine injury
- Any focal neurology
- A CT head scan is being performed
- Initial plain films are abnormal

Immediate CT head (within 1 hour) if:
GCS < 12 on admission
GCS < 15 2 hours after admission
Suspected open or depressed skull fracture
Suspected skull base fracture (panda eyes, Battle’s sign, CSF from nose/ear, bleeding ear)
Focal neurology
Vomiting > 1 episode
Post traumatic seizure

Contact neurosurgeon if:
Persistent GCS < 8 or = 8
Unexplained confusion > 4h
Reduced GCS after admission
Progressive neurological signs
Incomplete recovery post seizure
Penetrating injury
Cerebrospinal fluid leak

Observations
1/2 hourly GCS until 15

In the latest guidance, there is a move to considering CT in those on anticoagulants rather than mandating it.

Reference
1. NICE Guidance NG 232 (Published 2023).
2. Hodgkinson S et al. Early management of head injury: summary of NICE guidance. BMJ 2014 (348):34-37.

How well did you know this?
1
Not at all
2
3
4
5
Perfectly
531
Q

A 73 year old lady is recovering from recent surgery and is resting. Whilst she is resting, which of these muscles will be the major contributor to ventilatory activity?

Innermost intercostal

Diaphragm

Internal intercostal

Sternocleidomastoid

Scalene

A

The diaphragm is the major muscle involved in this process and it is for this reason that diminished activity following major abdominal surgery (note surgery type not specified here) can result in atelectasis.

Alveolar ventilation

  • Minute ventilation is the total volume of gas ventilated per minute.

MV (ml/min)= tidal volume x Respiratory rate (resps/min).

Dead space ventilation describes the volume of gas not involved in exchange in the blood.

There are 2 types:

  1. Anatomical dead space: 150mls
    Volume of gas in the respiratory tree not involved in gaseous exchange: mouth, pharynx, trachea, bronchi up to terminal bronchioles
    Measured by Fowlers method
    Increased by:
    Standing, increased size of person, increased lung volume and drugs causing bronchodilatation e.g. Adrenaline
  2. Physiological dead space: normal 150 mls, increases in ventilation/perfusion mismatch e.g. PE, COPD, hypotension
    Volume of gas in the alveoli and anatomical dead space not involved in gaseous exchange.

Alveolar ventilation is the volume of fresh air entering the alveoli per minute.

Alveolar ventilation = minute ventilation - Dead space volume

How well did you know this?
1
Not at all
2
3
4
5
Perfectly
532
Q

A 77 year old man is admitted with large bowel obstruction and on investigation with an abdominal CT scan is found to have an obstructing cancer of the sigmoid colon. What is the most appropriate course of action?

Laparotomy, sigmoid colectomy and formation of end colostomy

Laparotomy and loop colostomy

Laparotomy and loop ileostomy

Laparotomy, high anterior resection and colo-rectal anastomosis

Palliation

A

Obstructing sigmoid cancers can be resected or stented. If stented, then the patient may need definitive surgery later. If resected, then a resection and end colostomy (Hartmann’s ) procedure is usually undertaken because of the risks of anastomotic leak in the setting of anastomosing obstructed colon to rectum.

Large bowel obstruction

Colonic obstruction remains a common surgical problem. It is most commonly due to malignancy (60%) and diverticular disease (20%). Volvulus affecting the colon accounts for 5% of cases. Acute colonic pseudo-obstruction remains a potential differential diagnosis in all cases. Intussusception affecting the colon (most often due to tumours in the adult population) remains a rare but recognised cause.
The typical patient will present with gradual onset of progressive abdominal distension, colicky abdominal pain and either obstipation or absolute constipation.
On examination abdominal distension is present, the presence of caecal tenderness (assuming no overt evidence of peritonitis) is a useful sign to elicit. A digital rectal examination and rigid sigmoidoscopy should be performed.
A plain abdominal x-ray is the usual first line test and; the caecal diameter and ileocaecal valve competency should be assessed on this film.

Imaging modalities
Debate long surrounds the use of CT versus gastrograffin enemas. The latter investigation has always been the traditional method of determining whether a structural lesion is indeed present. However, in the UK the use of this technique has declined and in most units a CT scan will be offered as the first line investigation by the majority of radiologists (and is advocated by the ACPGBI). In most cases this will provide sufficient detail to allow operative planning, and since malignancy accounts for most presentations may also stage the disease. In the event that the radiologist cannot provide a clear statement of lesion site, the surgeon should have no hesitation in requesting a contrast enema.

Surgical options
The decision as to when to operate or not is determined firstly by the patients physiological status. Unstable patients require resuscitation prior to surgery and admission to a critical care unit for invasive monitoring and potential inotropic support may be needed. In patients who are otherwise stable the decision then rests on the radiological and clinical findings. As a general rule the old adage that the sun should not rise and set on unrelieved large bowel obstruction still holds true. A caecal diameter of 12cm or more in the presence of complete obstruction with a competent ileocaecal valve and caecal tenderness is a sign of impending perforation and a relative indication for prompt surgery.

Right sided and transverse lesions
Right sided lesions producing large bowel obstruction should generally be treated by right hemicolectomy or its extended variant if the lesion lies in the distal transverse colon or splenic flexure. In these cases an ileocolic anastomosis may be easily constructed and even in the emergency setting has a low risk of anastomotic leak.

Left sided lesions
The options here lie between sub total colectomy and anastomosis, left hemicolectomy with on table lavage and primary anastomosis, left hemicolectomy and end colostomy formation and finally colonic stent insertion.
The usefulness of colonic stents was the subject of a Cochrane review in 2011. The authors concluded that on the basis of the data that they reviewed, there was no benefit from the use of colonic stents over conventional surgical resection with a tendency to better outcomes seen in the surgical group (1). A more recently conducted meta analysis met with the same conclusion (2). However, the recently concluded CREST trial has suggested that self expanding metallic stents can improve outcomes and avoids a stoma.

Rectosigmoid lesions
Lesions below the peritoneal reflection that are causing obstruction should generally be treated with a loop colostomy. Primary resection of unstaged rectal cancer would most likely carry a high CRM positivity rate and cannot be condoned. Where the lesion occupies the distal sigmoid colon the usual practice would be to perform a high anterior resection. The decision surrounding restoration of intestinal continuity would lie with the operating surgeon.

References
1. Sagar J. Colorectal stents for the management of malignant colonic obstructions. Cochrane Database of Systematic Reviews 2011, Issue 11. Art. No.: CD007378. DOI: 10.1002/14651858.CD007378.pub2.
2. Cirrochi et al Safety and efficacy of endoscopic colonic stenting as a bridge to surgery in the management of intestinal obstruction due to left colon and rectal cancer: A systematic review and meta-analysis. Surg Oncol. 2013 Mar;22(1):14-21.

How well did you know this?
1
Not at all
2
3
4
5
Perfectly
533
Q

A 45 year old man has been admitted after being knocked off his bicycle. His ankle is grossly deformed with bilateral malleolar tenderness with severe ankle swelling and tenting of the medial soft tissues. What is the most appropriate initial management?

Application of compression dressing and physiotherapy

Application of external fixation device

Immediate reduction and application of backslab

Surgical fixation

Application of full leg plaster cast

A

This is an unstable ankle injury that is likely to require surgical fixation. The immediate management of a displaced ankle fracture is to reduce the fracture to prevent soft tissues compromise and help reduce swelling. This can be performed before an x-ray is obtained if performing the x-ray will significantly delay reduction.

Ankle injuries

An ankle fracture relates to a fracture around the tibio-talar joint. It generally refers to a fracture involving the lateral, and/or medial and/or posterior malleolus. Pilon and Tillaux fractures are also considered to be ankle fractures, but are not covered here.
Ankle fractures are common. They effect men and women in equal numbers, but men have a higher rate as young adults (sports and contact injuries), and women a higher rate post-menopausal (fragility type fracture).

Osseous anatomy
The ankle (or mortise) joint consists of the distal tibia (tibial plafond and posterior malleolus), the distal fibula (lateral malleolus), and the talus. The main movement at the ankle joint is plantar and dorsiflexion.

Ligamentous anatomy
Medial side: Deltoid ligament. This is divided into superficial and deep portions. It is the primary restraint to valgus tilting of the talus.

Lateral side: Lateral ligament complex consisting from anterior to posterior of the anterior talofibular ligament (ATFL), calcaneofibular ligament (CFL), and the posterior talofibular ligament (PTFL). Together they resist valgus stress to the ankle, and are a restraint to anterior translation of the talus within the mortise joint.

Syndesmosis: The syndesmosis is a ligament complex between the distal tibia and fibula, holding the two bones together. It is fundamental to the integrity of the ankle joint, and its disruption leads to instability. It consists of (from anterior to posterior) the anterior-inferior tibiofibular ligament (AITFL), the transverse tibiofibular ligament (TTFL), the interosseous membrane, and the posterior-inferior tibiofibular ligament (PITFL).

Presentation and initial management
Patients will present following a traumatic event with a painful, swollen ankle, and reluctance/inability to weight bear. The Ottawa rules can be applied to differentiate between an ankle fracture and sprain, but can be unreliable.
In high energy injuries, management should follow ATLS principles to identify more significant injuries first. Neurovascular status of the foot should be documented, and open injuries should be excluded. If an open injury is identified, it should be managed in line with BOAST 4 principles1. If an obvious deformity exists, it should be reduced as soon as possible with appropriate analgesia or conscious sedation. Radiographs of clearly deformed or dislocated joints are not necessary, and removing the pressure on the surrounding soft tissues from the underlying bony deformity is the priority. If the fracture pattern is not clinically obvious then plain radiographs are appropriate and will guide the subsequent manipulation during plaster-of-paris below knee backslab application.

Imaging
AP, lateral and mortise views (20 degrees internal rotation) are essential to evaluate fracture displacement and syndesmotic injury. Decreased tibiofibular overlap, medial joint clear space and lateral talar shift all indicate a syndesmotic injury. (In subtle cases of shift, imaging the uninjured ankle can be helpful as a proportion of the population have little or no tibiotalar overlap-reference 2.)

Where there is suspicion of syndesmosis involvement in the absence of radiographic evidence, stress radiographs can be diagnostic.
Complex fracture patterns (and increasingly posterior malleolar fractures) are best defined using CT.

Classification
The most commonly used classifications are Lauge-Hansen and Danis-Weber.

Lauge-Hansen
Comprises two parts: first part is the foot position, and the second part is the force applied. Useful for understanding the forces involved and therefore predict the ligamentous or bony injury. Results in four injury patterns:
Supination - Adduction (SA) - 10-20%
Supination - External rotation (SER) - 40-75%
Pronation - Abduction (PA) - 5-20%
Pronation - External rotation (PER) - 5-20%

Not often used in clinical practice but good for understanding the principles of ankle fracture.

Danis-Weber
Commonly used. Based on the level of the fibula fracture in relation to the syndesmosis. The more proximal, the greater the risk of syndesmotic injury and therefore fracture instability.
A - fracture below the level of the syndesmosis
B - fracture at the level of the syndesmosis / level of the tibial plafond
C - fracture above the level of the syndesmosis. This includes Maisonneuve fractures (proximal fibula fracture), which can be associated with ankle instability. Beware the high fibula fracture - it may be an ankle fracture!

The Weber classification is based purely on the the lateral side. All injuries can include a medial or posterior bony or ligamentous injury which also dictates fracture stability (bimalleolar and trimalleolar fractures are more unstable).

Treatment
When deciding upon treatment for an ankle fracture, one must consider both the fracture and the patient. Diabetic patients and smokers are at greater risk of post-operative complication, especially wound problems and infection. Likewise, the long term outcome of post-traumatic arthritis from a malunited ankle fracture is extremely important for a young patient, but not as relevant in the elderly. Therefore, normal surgical decision processes apply as with all fractures.

Defining stability of an ankle fracture underpins the treatment decision.
Weber A - Unimalleolar Weber A Weber fractures by definition are stable and therefore can be mobilised fully weight bearing in an ankle boot.

Weber C - Fractures tend to include syndesmotic disruption and are usually bimalleolar (either bony or ligamentous). They are therefore unstable and usually require operative fixation. In addition to the fracture fixation, the syndesmosis usually requires reconstruction/augmentation with screws to restore the joint integrity and function.

Weber B - B fractures vary greatly. They can be part of a trimalleolar injury and therefore extremely unstable, requiring fixation. Alternatively, a uni-malleolar Weber B fracture can be a stable injury, and therefore mobilised immediately in an ankle boot. Defining the stability can be challenging, and often involves stress radiographs, or a trial of mobilisation and repeat radiographs. However, treating undisplaced ankle fractures in a below knee plaster, non-weight bearing for six weeks is still widely practised, and a safe approach.

When operative fixation is appropriate, it is usually via open reduction and internal fixation using plates and screws. It must be carried out when soft tissue swelling has settled in order to minimise the risk of wound problems. This can often take a week to settle.

The use of fibula nails is expanding, but is not yet mainstream. Ankle fractures can also be treated with external fixation, or with a hind foot nail in patients who need fixation but where soft tissue or bone quality is poor.

Post operative management
Ankle fractures generally take 6 weeks to unite enough to prevent secondary displacement. This is therefore an appropriate time period to keep a cast on in a conservatively managed patient. Weight bearing post-operatively depends on the quality of the fixation and bone quality, and preference varies between surgeons, ranging from aggressive early mobilisation to a period of non-weight bearing. Return to activities takes approximately three months, and often requires assistance of a physiotherapist to improve range-of-movement and muscle strengthening.

References
1. http://www.boa.ac.uk/publications/boa-standards-trauma-boasts/
2. Shah AS, Kadakia AR, Tan GJ, Karadsheh MS, Wolter TD, Sabb B. Radiographic evaluation of the normal distal tibiofibular syndesmosis. Foot Ankle Int. 2012;33(10):870-6

How well did you know this?
1
Not at all
2
3
4
5
Perfectly
534
Q

An 83 year old man with a long standing staghorn calculus presents with recurrent haematuria and investigation shows a mass of the left renal pelvis. Of the lesions listed below, the diagnosis is:

Squamous cell carcinoma

Adenocarcinoma

Transitional cell carcinoma

Sarcoma

Transitional metaplasia

A

SCC of the kidney usually arises in an area of chronic inflammation such as a staghorn calculus. They are rare.

Renal tumours

Renal cell carcinoma
Renal cell carcinoma is an adenocarcinoma of the renal cortex and is believed to arise from the proximal convoluted tubule. They are usually solid lesions, up to 20% may be multifocal, 20% may be calcified and 20% may have either a cystic component or be wholly cystic. They are often circumscribed by a pseudocapsule of compressed normal renal tissue. Spread may occur either by direct extension into the adrenal gland, renal vein or surrounding fascia. More distant disease usually occurs via the haematogenous route to lung, bone or brain.
Renal cell carcinoma comprise up to 85% of all renal malignancies. Males are more commonly affected than females and sporadic tumours typically affect patients in their sixth decade.
Patients may present with a variety of symptoms including; haematuria (50%), loin pain (40%), mass (30%) and up to 25% may have symptoms of metastasis.Less than 10% have the classic triad of haematuria, pain and mass.

Investigation
Many cases will present as haematuria and be discovered during diagnostic work up. Benign renal tumours are rare, so renal masses should be investigated with multislice CT scanning. Most tumours are also characterised with an arterial phase CT, particularly if they may be suitable for partial nephrectomy.

CT scanning of the chest and abdomen to detect distant disease should also be undertaken.

Routine bone scanning is not indicated in the absence of symptoms.

Biopsy should not be performed when a nephrectomy is planned but is mandatory before any ablative therapies are undertaken.

Assessment of the functioning of the contra lateral kidney.

Management
T1 lesions may be managed by partial nephrectomy and this gives equivalent oncological results to total radical nephrectomy. Partial nephrectomy may also be performed when there is inadequate reserve in the remaining kidney. Ablative techniques may also be considered for small T1 lesions in unfit patients. In general, outcomes are less favorable than with surgical resection.

For T2 lesions and above a radical nephrectomy is standard practice and this may be performed via a laparoscopic or open approach. Preoperative embolisation is not indicated nor is resection of uninvolved adrenal glands. During surgery early venous control is mandatory to avoid shedding of tumour cells into the circulation.

Patients with completely resected disease do not benefit from adjuvant therapy with either chemotherapy or biological agents. These should not be administered outside the setting of clinical trials.

Patients with transitional cell cancer will require a nephroureterectomy with disconnection of the ureter at the bladder.

References
Capitanio U, Montorsi F. Renal cancer. Lancet. 2016 Feb 27;387(10021):894-906.

Capitanio U et al. Epidemiology of Renal Cell Carcinoma. Eur Urol. 2019 Jan;75(1):74-84.

How well did you know this?
1
Not at all
2
3
4
5
Perfectly
535
Q

Which of the following features are not typically seen in a venous stasis ulcer?

Located above the medial malleolus

Haemosiderin deposits

Variable scarring

Exophytic granulation tissue

Varicose veins

A

Whilst the base of the ulcer may be lined by granulation tissue, this is seldom exophytic. One of the concerns would be that this represents a malignant transformation.

Chronic venous insufficiency and varicose veins

Wide spectrum of disease ranging from minor cosmetic problem through to ulceration and disability. It is commoner in women than men and is worse during pregnancy. Varicose veins are best considered as being a saccular dilation of veins (WHO). Chronic venous insufficiency is a series of tissue changes which occur in relation to pooling of blood in the extremities with associated venous hypertension occurring as a result of incompetent deep vein valves.

The veins of the lower limb consist of an interconnected network of superficial and deep venous systems. Varices occur because of localised weakness in the vein wall resulting in dilatation and reflux of blood due to non union of valve cusps. Histologically, the typical changes include fibrous scar tissue dividing smooth muscle within media in the vessel wall.

Tissue damage in chronic venous insufficiency occurs because of perivascular cytokine leakage resulting in localised tissue damage coupled with impaired lymphatic flow.

Diagnosis
Typical symptoms of varicose veins include:
Cosmetic appearance
Aching
Ankle swelling that worsens as the day progresses
Episodic thrombophlebitis
Bleeding
Itching

Symptoms of chronic venous insufficiency include:
Dependant leg pain
Prominent leg swelling
Oedema extending beyond the ankle
Venous stasis ulcers

The typical venous stasis ulcer is:
Located above the medial malleolus
Indolent appearance with basal granulation tissue
Variable degree of scarring
Non ischaemic edges
Haemosiderin deposition in the gaiter area (and also lipodermatosclerosis).

Differential diagnosis
Lower limb arterial disease
Marjolins ulcer
Claudication
Spinal stenosis
Swelling due to medical causes e.g. CCF.

Exclusion of these differentials is by means of physical examination and ankle brachial pressure index measurement.

Examination
Assess for dilated short saphenous vein (popliteal fossa) and palpate for saphena varix medial to the femoral artery
Brodie-Trendelenburg test: to assess level of incompetence
Perthes’ walking test: assess if deep venous system competent

Investigation
Doppler exam: if incompetent a biphasic signal due to retrograde flow is detected
Duplex scanning: to ensure patent deep venous system (do if DVT or trauma)

All patients should have a Doppler assessment to assess for venous reflux and should be classified as having uncomplicated varicose veins or varicose veins with associated chronic venous insufficiency. In the history establishing a previous thrombotic event (DVT/ lower limb fracture) is important and patients with such a history and all who have evidence of chronic venous insufficiency should have a duplex scan performed.

Owing to litigation patients with saphenopopliteal incompetence should have a duplex scan performed and the site marked by scan on the day of surgery.

Treatment
Indications for surgery:
Cosmetic: majority
Lipodermatosclerosis causing venous ulceration
Recurrent superficial thrombophlebitis
Bleeding from ruptured varix

Condition Therapy
Minor varicose veins - no complications Reassure/ cosmetic therapy
Symptomatic uncomplicated varicose veins In those without deep venous insufficiency options include; endothermal ablation, foam sclerotherapy, saphenofemoral / popliteal disconnection, stripping and avulsions, compression stockings
Varicose veins with skin changes Therapy as above (if compression minimum is formal class I stockings)
Chronic venous insufficiency or ulcers Class 2-3 compression stockings (ensure no arterial disease).

Application of formal compression stockings (usually class II/III). In patients who have suffered ulceration, compression stockings should be worn long term. Where ulceration is present and established saphenofemoral reflux exists this should be addressed surgically for durable relief of symptoms, either at the outset or following ulcer healing.
Injection sclerotherapy (5% Ethanolamine oleate), foam is increasingly popular, though transient blindness has been reported. Endo venous laser therapy is another minimally invasive option
Sapheno-femoral or sapheno-popliteal ligation, in the case of the LSV; stripping and multiple phlebectomies

Current best practice guidance
In the United Kingdom the National Institute of Clinical Excellence guidance on varicose veins suggests that for patients with symptomatic varicose veins the first line procedure of choice should be endothermal ablation (see reference for more information). Where this is unavailable or unsuitable then foam sclerotherapy should be the second line option. Surgery is currently the third line treatment option.

Trendelenburg procedure (sapheno-femoral junction ligation)
Head tilt 15 degrees and legs abducted
Oblique incision 1cm medial from artery
Tributaries ligated (Superficial circumflex iliac vein, Superficial inferior epigastric vein, Superficial and deep external pudendal vein)
SF junction double ligated
Saphenous vein stripped to level of knee/upper calf. NB increased risk of saphenous neuralgia if stripped more distally

References
Marsden G et al. Diagnosis of management of varicose veins in the the legs: summary of NICE guidance. BMJ 2013 (347): 30-31.

How well did you know this?
1
Not at all
2
3
4
5
Perfectly
536
Q

How many compartments are there in the lower leg?

2

1

3

5

4

A

The posterior compartment of the lower leg has both superficial and deep posterior layers, together with the anterior and lateral compartments this allows for four compartments. Decompression of the deep posterior compartment during fasciotomy may be overlooked with significant sequelae.

Fascial compartments of the leg

Compartments of the thigh

Formed by septae passing from the femur to the fascia lata.
Compartment Nerve Muscles Blood supply
Anterior compartment Femoral
Iliacus
Tensor fasciae latae
Sartorius
Quadriceps femoris
Femoral artery
Medial compartment Obturator
Adductor longus/magnus/brevis
Gracilis
Obturator externus
Profunda femoris artery and obturator artery
Posterior compartment (2 layers) Sciatic
Semimembranosus
Semitendinosus
Biceps femoris
Branches of Profunda femoris artery

Compartments of the lower leg
Separated by the interosseous membrane (anterior and posterior compartments), anterior fascial septum (separate anterior and lateral compartments) and posterior fascial septum (separate lateral and posterior compartments)

Compartment Nerve Muscles Blood supply
Anterior compartment Deep peroneal nerve
Tibialis anterior
Extensor digitorum longus
Extensor hallucis longus
Peroneus tertius
Anterior tibial artery
Posterior compartment Tibial
Muscles: deep and superficial compartments (separated by deep transverse fascia)
Deep: Flexor hallucis longus, Flexor digitalis longus, Tibialis posterior, Popliteus
Superficial: Gastrocnemius, Soleus, Plantaris
Posterior tibial
Lateral compartment Superficial peroneal
Peroneus longus/brevis
Peroneal artery

How well did you know this?
1
Not at all
2
3
4
5
Perfectly
537
Q

A 6 year old boy presents with pain in the hip it is present on activity and has been worsening over the past few weeks. There is no history of trauma. He was born by normal vaginal delivery at 38 weeks gestation On examination he has an antalgic gait and limitation of active and passive movement of the hip joint in all directions. C-reactive protein is mildly elevated at 10 but the white cell count is normal. What is the most likely diagnosis?

Perthes disease

Septic arthritis

Slipped upper femoral epiphysis

Developmental dysplasia of the hip

Osteoarthritis

A

Early plain x-ray changes in Perthes Disease:
Widening of the joint space.
Sub chondral linear lucency.
This is a typical presentation for Perthes disease. X-ray may show flattening of the femoral head or fragmentation in more advanced cases.

Paediatric orthopaedics

Diagnosis Mode of presentation Treatment Radiology
Developmental dysplasia of the hip Usually diagnosed in infancy by screening tests. May be bilateral, when disease is unilateral there may be leg length inequality. As disease progresses child may limp and then early onset arthritis. More common in extended breech babies. Splints and harnesses or traction. In later years osteotomy and hip realignment procedures may be needed. In arthritis a joint replacement may be needed. However, this is best deferred if possible as it will almost certainly require revision Initially no obvious change on plain films and USS gives best resolution until 3 months of age. On plain films Shentons line should form a smooth arc
Perthes Disease Hip pain (may be referred to the knee) usually occurring between 5 and 12 years of age. Bilateral disease in 20%. Remove pressure from joint to allow normal development. Physiotherapy. Usually self-limiting if diagnosed and treated promptly. X-rays will show flattened femoral head. Eventually in untreated cases the femoral head will fragment.
Slipped upper femoral epiphysis Typically seen in obese male adolescents. Pain is often referred to the knee. Limitation to internal rotation is usually seen. Knee pain is usually present 2 months prior to hip slipping. Bilateral in 20%. Bed rest and non-weight bearing. Aim to avoid avascular necrosis. If severe slippage or risk of it occurring then percutaneous pinning of the hip may be required. X-rays will show the femoral head displaced and falling inferolaterally (like a melting ice cream cone) The Southwick angle gives indication of disease severity

How well did you know this?
1
Not at all
2
3
4
5
Perfectly
538
Q

Which of the following substances is released from the sympathetic nervous system to stimulate the adrenal medulla?

Noradrenaline

Acetyl choline

Substance P

Tyrosine

Arginine

A

In the autonomic nervous system, noradrenaline is the commonly used neurotransmitter. However, in the adrenal medulla, Acetylcholine is released to stimulate adrenaline release.

Adrenal physiology

Adrenal medulla
The chromaffin cells of the adrenal medulla secrete the catecholamines noradrenaline and adrenaline. The medulla is innervated by the splanchnic nerves; the preganglionic sympathetic fibres secrete acetylcholine causing the chromaffin cells to secrete their contents by exocytosis.
Phaeochromocytomas are derived from these cells and will secrete both adrenaline and nor adrenaline.

Adrenal cortex
Three histologically distinct zones are recognised:

Zone Location Hormone Secreted
Zona glomerulosa Outer zone Aldosterone
Zona fasiculata Middle zone Glucocorticoids
Zona reticularis Inner zone Androgens

The glucocorticoids and aldosterone are mostly bound to plasma proteins in the circulation. Glucocorticoids are inactivated and excreted by the liver.

How well did you know this?
1
Not at all
2
3
4
5
Perfectly
539
Q

A 68 year old man is involved in a road traffic accident that is relatively minor and sustains a minor head injury with small associated scalp laceration. Apart from atrial fibrillation for which he is treated with bisoprolol and dabigatran, he is well. On examination, he is drowsy and GCS is 12 with no localizing neurological signs. What is the most appropriate course of action?

Arrange a skull x-ray

Arrange an MRI scan of the brain

Admit for observation with GCS measurements every 30 minutes

Admit for observation with GCS measurements every 60 minutes

Arrange a CT scan of the head

A

Head injury with anticoagulants= CT head

Head injury management- NICE Guidelines

Summary of guidelines
All patients should be assessed within 15 minutes on arrival to A&E
Document all 3 components of the GCS
If GCS <8 or = to 8, consider stabilising the airway
Full spine immobilisation until assessment if:

  • GCS < 15
  • neck pain/tenderness
  • paraesthesia extremities
  • focal neurological deficit
  • suspected c-spine injury

If a c-spine injury is suspected a 3 view c-spine x-ray is indicated. CT c-spine is preferred if:
- Intubated
- GCS <13
- Normal x-ray but continued concerns regarding c-spine injury
- Any focal neurology
- A CT head scan is being performed
- Initial plain films are abnormal

Immediate CT head (within 1 hour) if:
GCS < 12 on admission
GCS < 15 2 hours after admission
Suspected open or depressed skull fracture
Suspected skull base fracture (panda eyes, Battle’s sign, CSF from nose/ear, bleeding ear)
Focal neurology
Vomiting > 1 episode
Post traumatic seizure

Contact neurosurgeon if:
Persistent GCS < 8 or = 8
Unexplained confusion > 4h
Reduced GCS after admission
Progressive neurological signs
Incomplete recovery post seizure
Penetrating injury
Cerebrospinal fluid leak

Observations
1/2 hourly GCS until 15

In the latest guidance, there is a move to considering CT in those on anticoagulants rather than mandating it.

Reference
1. NICE Guidance NG 232 (Published 2023).
2. Hodgkinson S et al. Early management of head injury: summary of NICE guidance. BMJ 2014 (348):34-37.

How well did you know this?
1
Not at all
2
3
4
5
Perfectly
540
Q

A 43 year old lady presents with varicose veins and undergoes a saphenofemoral disconnection, long saphenous vein stripping to the ankle and isolated hook phlebectomies. Post operatively she notices an area of numbness superior to her ankle. What is the most likely cause for this?

Sural nerve injury

Femoral nerve injury

Saphenous nerve injury

Common peroneal nerve injury

Superficial peroneal nerve injury

A

The sural nerve is related to the short saphenous vein. The saphenous nerve is related to the long saphenous vein below the knee and for this reason full length stripping of the vein is no longer advocated.

Saphenous vein

Long saphenous vein
This vein may be harvested for bypass surgery, or removed as treatment for varicose veins with saphenofemoral junction incompetence.

Originates at the 1st digit where the dorsal vein merges with the dorsal venous arch of the foot
Passes anterior to the medial malleolus and runs up the medial side of the leg
At the knee, it runs over the posterior border of the medial epicondyle of the femur bone
Then passes laterally to lie on the anterior surface of the thigh before entering an opening in the fascia lata called the saphenous opening
It joins with the femoral vein in the region of the femoral triangle at the saphenofemoral junction

Tributaries
Medial marginal
Superficial epigastric
Superficial iliac circumflex
Superficial external pudendal veins

Short saphenous vein
Originates at the 5th digit where the dorsal vein merges with the dorsal venous arch of the foot, which attaches to the great saphenous vein.
It passes around the lateral aspect of the foot (inferior and posterior to the lateral malleolus) and runs along the posterior aspect of the leg (with the sural nerve)
Passes between the heads of the gastrocnemius muscle, and drains into the popliteal vein, approximately at or above the level of the knee joint.

How well did you know this?
1
Not at all
2
3
4
5
Perfectly
541
Q

Which of the following are not typical of Lynch syndrome?

It is inherited in an autosomal recessive manner

Affected patients are more likely to develop right colon mucinous tumours than the general population

Affected individuals have an 80% lifetime risk of colon cancer

Endometrial cancer is seen in 80% of women

Gastric cancers are more common

A

Lynch syndrome is inherited in an autosomal dominant fashion. It is characterised by microsatellite instability in the DNA mismatch repair genes. Colonic tumours in patients with Lynch syndrome are more likely to be right sided tumours and to be poorly differentiated.

Genetics and surgical disease

Some of the more commonly occurring genetic conditions occurring in surgical patients are presented here.

Li-Fraumeni Syndrome
Autosomal dominant
Consists of germline mutations to p53 tumour suppressor gene
High incidence of malignancies particularly sarcomas and leukaemias
Diagnosed when:

*Individual develops sarcoma under 45 years
*First degree relative diagnosed with any cancer below age 45 years and another family member develops malignancy under 45 years or sarcoma at any age

BRCA 1 and 2
Carried on chromosome 17 (BRCA 1) and Chromosome 13 (BRCA 2)
Linked to developing breast cancer (60%) risk.
Associated risk of developing ovarian cancer (55% with BRCA 1 and 25% with BRCA 2).

Lynch Syndrome
Autosomal dominant
Develop colonic cancer and endometrial cancer at young age
80% of affected individuals will get colonic and/ or endometrial cancer
High risk individuals may be identified using the Amsterdam criteria

Amsterdam criteria
Three or more family members with a confirmed diagnosis of colorectal cancer, one of whom is a first degree (parent, child, sibling) relative of the other two.
Two successive affected generations.
One or more colon cancers diagnosed under age 50 years.
Familial adenomatous polyposis (FAP) has been excluded.

Gardners syndrome
Autosomal dominant familial colorectal polyposis
Multiple colonic polyps
Extra colonic diseases include: skull osteoma, thyroid cancer and epidermoid cysts
Desmoid tumours are seen in 15%
Mutation of APC gene located on chromosome 5
Due to colonic polyps most patients will undergo colectomy to reduce risk of colorectal cancer
Now considered a variant of familial adenomatous polyposis coli

How well did you know this?
1
Not at all
2
3
4
5
Perfectly
542
Q

A 60 year old man presents with recurrent renal stones. He is found to have a calcium of 2.72 (elevated) and a PTH of 12 (elevated). What is the most appropriate long term management plan?

Neck exploration and parathyroidectomy

CT scanning of the neck

MRI scanning of the neck

Medical treatment alone

External beam radiotherapy to the neck

A

This patient has primary hyperparathyroidism and nephrolithiasis, which is an indication for parathyroidectomy.

Parathyroid glands and disorders of calcium metabolism

Hyperparathyroidism
Disease type Hormone profile Clinical features Cause
Primary hyperparathyroidism
PTH (Elevated)
Ca2+ (Elevated)
Phosphate (Low)
Urine calcium : creatinine clearance ratio > 0.01
May be asymptomatic if mild
Recurrent abdominal pain (pancreatitis, renal colic)
Changes to emotional or cognitive state
Most cases due to solitary adenoma (80%), multifocal disease occurs in 10-15% and parathyroid carcinoma in 1% or less
Secondary hyperparathyroidism
PTH (Elevated)
Ca2+ (Low or normal)
Phosphate (Elevated)
Vitamin D levels (Low)
May have few symptoms
Eventually may develop bone disease, osteitis fibrosa cystica and soft tissue calcifications
Parathyroid gland hyperplasia occurs as a result of low calcium, almost always in a setting of chronic renal failure
Tertiary hyperparathyroidism
Ca2+ (Normal or high)
PTH (Elevated)
Phosphate levels (Decreased or Normal)
Vitamin D (Normal or decreased)
Alkaline phosphatase (Elevated)
Metastatic calcification
Bone pain and / or fracture
Nephrolithiasis
Pancreatitis
Occurs as a result of ongoing hyperplasia of the parathyroid glands after correction of underlying renal disorder, hyperplasia of all 4 glands is usually the cause

Differential diagnoses
It is important to consider the rare but relatively benign condition of benign familial hypocalciuric hypercalcaemia, caused by an autosomal dominant genetic disorder. Diagnosis is usually made by genetic testing and concordant biochemistry (urine calcium : creatinine clearance ratio <0.01-distinguished from primary hyperparathyroidism).

Treatment

Primary hyperparathyroidism
Indications for surgery
Elevated serum Calcium > 1mg/dL above normal
Hypercalciuria > 400mg/day
Creatinine clearance < 30% compared with normal
Episode of life threatening hypercalcaemia
Nephrolithiasis
Age < 50 years
Neuromuscular symptoms
Reduction in bone mineral density of the femoral neck, lumbar spine, or distal radius of more than 2.5 standard deviations below peak bone mass (T score lower than -2.5)

Secondary hyperparathyroidism
Usually managed with medical therapy.

Indications for surgery in secondary (renal) hyperparathyroidism:
Bone pain
Persistent pruritus
Soft tissue calcifications

Tertiary hyperparathyroidism
Allow 12 months to elapse following transplant as many cases will resolve
The presence of an autonomously functioning parathyroid gland may require surgery. If the culprit gland can be identified then it should be excised. Otherwise total parathyroidectomy and re-implantation of part of the gland may be required.

References
1. Insogna K. Primary Hyperparathyroidism. N Engl J Med. 2018 Sep 13;379(11):1050-1059.

  1. van der Plas W Y et al. Secondary and Tertiary Hyperparathyroidism: A Narrative Review. Scand J Surg. 2020 Dec;109(4):271-278.
How well did you know this?
1
Not at all
2
3
4
5
Perfectly
543
Q

A 34 year old man undergoes a sub total colectomy to treat fulminant ulcerative colitis. What type of stoma is most likely to be fashioned?

End colostomy

Loop colostomy

End ileostomy

Loop ileostomy

End jejunostomy

A

A sub total colectomy involves the removal of the entire right, transverse, left and part of the sigmoid colon. The rectal stump is closed and an end ileostomy fashioned in the right iliac fossa.

Abdominal stomas

Stomas may be sited during a range of abdominal procedures and involve bringing the lumen or visceral contents onto the skin. In most cases this applies to the bowel. However, other organs or their contents may be diverted in case of need.

With bowel stomas the type method of construction and to a lesser extent the site will be determined by the contents of the bowel. In practice, small bowel stomas should be spouted so that their irritant contents are not in contact with the skin. Colonic stomas do not need to be spouted as their contents are less irritant.

In the ideal situation the site of the stoma should be marked with the patient prior to surgery. Stoma siting is important as it will ultimately influence the ability of the patient to manage their stoma and also reduce the risk of leakage. Leakage of stoma contents and subsequent maceration of the surrounding skin can rapidly progress into a spiraling loss of control of stoma contents.

Types of stomas
Name of stoma Use Common sites
Gastrostomy
Gastric decompression or fixation
Feeding
Epigastrium
Loop jejunostomy
Seldom used as very high output
May be used following emergency laparotomy with planned early closure
Any location according to need
Percutaneous jejunostomy
Usually performed for feeding purposes and site in the proximal bowel
Usually left upper quadrant
Loop ileostomy
Defunctioning of colon e.g. following rectal cancer surgery
Does not decompress colon (if ileocaecal valve competent)
Usually right iliac fossa
End ileostomy
Usually following complete excision of colon or where ileo-colic anastomosis is not planned
May be used to defunction colon, but reversal is more difficult
Usually right iliac fossa
End colostomy Where a colon is diverted or resected and anastomosis is not primarily achievable or desirable Either left or right iliac fossa
Loop colostomy
To defunction a distal segment of colon
Since both lumens are present the distal lumen acts as a vent
May be located in any region of the abdomen, depending upon colonic segment used
Caecostomy Stoma of last resort where loop colostomy is not possible Right iliac fossa
Mucous fistula
To decompress a distal segment of bowel following colonic division or resection
Where closure of a distal resection margin is not safe or achievable
May be located in any region of the abdomen according to clinical need

How well did you know this?
1
Not at all
2
3
4
5
Perfectly
544
Q

Which of the following is not an intrinsic muscle of the hand?

Opponens pollicis

Palmaris longus

Flexor pollicis brevis

Flexor digiti minimi brevis

Opponens digiti minimi

A

Mnemonic for intrinsic hand muscles
‘A OF A OF A’

A bductor pollicis brevis
O pponens pollicis
F lexor pollicis brevis
A dductor pollicis (thenar muscles)
O pponens digiti minimi
F lexor digiti minimi brevis
A bductor digiti minimi (hypothenar muscles)
Palmaris longus originates in the forearm.

Hand

Anatomy of the hand
Bones
8 Carpal bones
5 Metacarpals
14 phalanges
Intrinsic Muscles 8 Interossei - Supplied by ulnar nerve
4 palmar-adduct fingers
4 dorsal- abduct fingers
Intrinsic muscles Lumbricals
Flex MCPJ and extend the IPJ.
Origin deep flexor tendon and insertion dorsal extensor hood mechanism.
Innervation: 1st and 2nd- median nerve, 3rd and 4th- deep branch of the ulnar nerve.
Thenar eminence
Abductor pollicis brevis
Opponens pollicis
Flexor pollicis brevis
Hypothenar eminence
Opponens digiti minimi
Flexor digiti minimi brevis
Abductor digiti minimi

Fascia and compartments of the palm
The fascia of the palm is continuous with the antebrachial fascia and the fascia of the dorsum of the hand. The palmar fascia is thin over the thenar and hypothenar eminences. In contrast, the central palmar fascia is relatively thick. The palmar aponeurosis covers the soft tissues and overlies the flexor tendons. The apex of the palmar aponeurosis is continuous with the flexor retinaculum and the palmaris longus tendon. Distally, it forms four longitudinal digital bands that attach to the bases of the proximal phalanges, blending with the fibrous digital sheaths.
A medial fibrous septum extends deeply from the medial border of the palmar aponeurosis to the 5th metacarpal. Lying medial to this are the hypothenar muscles. In a similar fashion, a lateral fibrous septum extends deeply from the lateral border of the palmar aponeurosis to the 3rd metacarpal. The thenar compartment lies lateral to this area.
Lying between the thenar and hypothenar compartments is the central compartment. It contains the flexor tendons and their sheaths, the lumbricals, the superficial palmar arterial arch and the digital vessels and nerves.
The deepest muscular plane is the adductor compartment, which contains adductor pollicis.

Short muscles of the hand
These comprise the lumbricals and interossei. The four slender lumbrical muscles flex the fingers at the metacarpophalangeal joints and extend the interphalangeal joint. The four dorsal interossei are located between the metacarpals and the four palmar interossei lie on the palmar surface of the metacarpals in the interosseous compartment of the hand.

Long flexor tendons and sheaths in the hand
The tendons of FDS and FDP enter the common flexor sheath deep to the flexor retinaculum. The tendons enter the central compartment of the hand and fan out to their respective digital synovial sheaths. Near the base of the proximal phalanx, the tendon of FDS splits to permit the passage of FDP. The FDP tendons are attached to the margins of the anterior aspect of the base of the distal phalanx.
The fibrous digital sheaths contain the flexor tendons and their synovial sheaths. These extend from the heads of the metacarpals to the base of the distal phalanges.

Palmar Interossei
Note that there are 4 palmar interossei. The first is a small slip of muscle which arises from the ulnar side of the base of the first metacarpal and passes between the head of the first dorsal interosseous and the oblique head of adductor pollicis to insert into the ulnar base of the of the proximal phalanx of the thumb. The second arises from the ulnar side of the body of the second metacarpal and is inserted into the ulnar side of the extensor hood of the index. The third and fourth palmar interossei arise from the radial sides of the bodies of the 4th and 5th metacarpals respectively and insert into the radial sides of the extensor hoods of the ring and little fingers.

How well did you know this?
1
Not at all
2
3
4
5
Perfectly
545
Q

Which of the following statements relating to alveolar ventilation is untrue?

Anatomical dead space is measured by helium dilution

Physiological dead space is increased in PE

Alveolar ventilation is defined as the volume of fresh air entering the alveoli per minute

Anatomical dead space is increased by adrenaline

Type 2 pneumocytes in the alveoli secrete surfactant

A

Anatomical dead space is measured by Fowlers method.

A patient inhales 100% oxygen to empty the conducting zone gases of nitrogen and then exhales through a mouthpiece which analyses the nitrogen concentration at the mouth. Initially the exhaled gases contain no nitrogen as this is dead space gas; the nitrogen concentration will increase as the alveolar gases are exhaled. Nitrogen which is measured following the breath of 100% oxygen must then have come only from gas exchanging areas of the lung and not dead space.

Alveolar ventilation

  • Minute ventilation is the total volume of gas ventilated per minute.

MV (ml/min)= tidal volume x Respiratory rate (resps/min).

Dead space ventilation describes the volume of gas not involved in exchange in the blood.

There are 2 types:

  1. Anatomical dead space: 150mls
    Volume of gas in the respiratory tree not involved in gaseous exchange: mouth, pharynx, trachea, bronchi up to terminal bronchioles
    Measured by Fowlers method
    Increased by:
    Standing, increased size of person, increased lung volume and drugs causing bronchodilatation e.g. Adrenaline
  2. Physiological dead space: normal 150 mls, increases in ventilation/perfusion mismatch e.g. PE, COPD, hypotension
    Volume of gas in the alveoli and anatomical dead space not involved in gaseous exchange.

Alveolar ventilation is the volume of fresh air entering the alveoli per minute.

Alveolar ventilation = minute ventilation - Dead space volume Alveolar ventilation

How well did you know this?
1
Not at all
2
3
4
5
Perfectly
546
Q

A 25 year old man sustains a severe middle cranial fossa basal skull fracture. Once he has recovered it is noticed that he has impaired tear secretion. This is most likely to be the result of damage to which of the following?

Stellate ganglion

Ciliary ganglion

Otic ganglion

Trigeminal nerve

Greater petrosal nerve

A

The greater petrosal nerve may be injured and carries fibres for lacrimation (see below).

Lacrimal system

Lacrimal gland
Consists of an orbital part and palpebral part. They are continuous posterolaterally around the concave lateral edge of the levator palpebrae superioris muscle.
The ducts of the lacrimal gland open into the superior fornix. Those from the orbital part penetrate the aponeurosis of levator palpebrae superioris to join those from the palpebral part. Therefore excision of the palpebral part is functionally similar to excision of the entire gland.

Blood supply
Lacrimal branch of the opthalmic artery. Venous drainage is to the superior opthalmic vein.

Innervation
The gland is innervated by the secretomotor parasympathetic fibres from the pterygopalatine ganglion which in turn may reach the gland via the zygomatic or lacrimal branches of the maxillary nerve or pass directly to the gland. The preganglionic fibres travel to the ganglion in the greater petrosal nerve (a branch of the facial nerve at the geniculate ganglion).

Nasolacrimal duct
Descends from the lacrimal sac to open anteriorly in the inferior meatus of the nose.

Lacrimation reflex
Occurs in response to conjunctival irritation (or emotional events). The conjunctiva will send signals via the opthalmic nerve. These then pass to the superior salivary centre. The efferent signals pass via the greater petrosal nerve (parasympathetic preganglionic fibres) and the deep petrosal nerve which carries the post ganglionic sympathetic fibres. The parasympathetic fibres will relay in the pterygopalatine ganglion, the sympathetic fibres do not synapse. They in turn will relay to the lacrimal apparatus.

How well did you know this?
1
Not at all
2
3
4
5
Perfectly
547
Q

A 65 year old lady is recovering from major abdominal surgery in ITU and, as part of her clinical care a clotting screen is performed. The prothrombin time is noted to be increased. Which of the conditions below is unlikely to account for this finding?

Cholestatic jaundice

Disseminated intravascular coagulation

Prolonged antibiotic treatment

Liver disease

Acquired factor 12 deficiency

A

Acquired factor 12 deficiency
Vitamin K deficiency results from cholestatic jaundice and prolonged antibiotic therapy. Acquired factor 12 deficiency causes prolonged APTT.
Abnormal coagulation

Cause Factors affected
Heparin Prevents activation factors 2,9,10,11
Warfarin Affects synthesis of factors 2,7,9,10
DIC Factors 1,2,5,8,11
Liver disease Factors 1,2,5,7,9,10,11

Interpretation blood clotting test results
Disorder APTT PT Bleeding time
Haemophilia Increased Normal Normal
von Willebrand’s disease Increased Normal Increased
Vitamin K deficiency Increased Increased Normal

How well did you know this?
1
Not at all
2
3
4
5
Perfectly
548
Q

A 35 year old type 1 diabetic presents with difficulty mobilising and back pain radiating to the thigh. He has a temperature of 39 oC and has pain on extension of the hip. He is diagnosed with an iliopsoas abscess. Which of the following statements is false in relation to his diagnosis?

Staphylococcus aureus is the most likely primary cause

Recurrence occurs in 60% cases

More common in males

Crohn’s is the most likely secondary cause in patients within the UK

CT guided drainage is preferable first line management

A

Recurrence occurs in 60% cases
Classical features include: a limp, back pain and fever. Recurrence rates are about 15-20%. In the UK, Staphylococcus is the commonest primary cause, others include Streptococcus and E.coli. Management is ideally by CT guided drainage.

Iliopsoas abscess

  • Collection of pus in iliopsoas compartment (iliopsoas and iliacus)
    Causes:

Primary

Haematogenous spread of bacteria
Staphylococcus aureus: most common

Secondary

Crohn’s (commonest cause in this category)
Diverticulitis, Colorectal cancer
UTI, GU cancers
Vertebral osteomyelitis
Femoral catheter, lithotripsy
Endocarditis

Note the mortality rate can be up to 19-20% in secondary iliopsoas abscesses compared with 2.4% in primary abscesses.

Clinical features

Fever
Back/flank pain
Limp
Weight loss

Clinical examination

Patient in the supine position with the knee flexed and the hip mildly externally rotated

Specific tests to diagnose iliopsoas inflammation:

Place hand proximal to the patient’s ipsilateral knee and ask patient to lift thigh against your hand. This will cause pain due to contraction of the psoas muscle.

Lie the patient on the normal side and hyperextend the affected hip. In inflammation this should elicit pain as the psoas muscle is stretched.

Investigation

CT is gold standard

Management

Antibiotics
Percutaneous drainage
Surgery is indicated if:

  1. Failure of percutaneous drainage
  2. Presence of an another intra-abdominal pathology which requires surgery

Surgical approach
The authors technique for draining these collections is given here.
Review the CT scans and plan surgical approach. An extraperitoneal approach is important.
The collection usually extends inferiorly and can be accessed from an incision at a level of L4 on the affected side.

GA
Transverse laterally placed incision.
Incise external oblique.
Split the subsequent muscle layers.
As you approach the peritoneum use blunt dissection to pass laterally around it.
Remember the ureter and gonadal veins lie posterior at this level.
Eventually you will enter the abscess cavity, a large amount of pus is usually released at this point. Drain the area with suction and washout with saline.
Place a corrugated drain well into the abscess cavity.
If you have made a small skin incision it is reasonable to bring the drain up through the skin wound. Otherwise place a lateral exit site and close the skin and external oblique. If you do this ensure that you use interrupted sutures.

Anchor the drain with strong securely tied silk sutures (it is extremely tiresome if it falls out!)

Reference

Iliopsoas abscesses
I H Mallick, M H Thoufeeq, T P Rajendran
Postgrad Med J 2004;80:459-462

How well did you know this?
1
Not at all
2
3
4
5
Perfectly
549
Q

Which of the following statements relating to malignant mesothelioma is false?

It may be treated by extrapleural pneumonectomy.

It is linked to asbestos exposure.

It is linked to cigarette smoking independent of asbestos exposure.

It may occur intra abdominally.

It is relatively resistant to radiotherapy

A

It is not linked to cigarette smoking. When identified at an early stage a radical resection is the favored option. Radiotherapy is often given perioperatively. However, it is not a particularly radiosensitive tumour. Combination chemotherapy gives some of the best results and most regimes are cisplatin based.

Occupational cancers

Occupational cancers accounted for 5.3% cancer deaths in 2005.
In men the main cancers include:
Mesothelioma
Bladder cancer
Non melanoma skin cancer
Lung cancer
Sino nasal cancer

Occupations with high levels of occupational tumours include:
Construction industry
Working with coal tar and pitch
Mining
Metalworkers
Working with asbestos (accounts for 98% of all mesotheliomas)
Working in rubber industry

Shift work has been linked to breast cancer in women (Health and safety executive report RR595).

The latency between exposure and disease is typically 15 years for solid tumours and 20 for leukaemia.

Many occupational cancers are otherwise rare. For example sino nasal cancer is an uncommon tumour, 50% will be SCC. They are linked to conditions such as wood dust exposure and unlike lung cancer is not strongly linked to cigarette smoking. Another typical occupational tumour is angiosarcoma of the liver which is linked to working with vinyl chloride. Again in the non occupational context this is an extremely rare sporadic tumour.

How well did you know this?
1
Not at all
2
3
4
5
Perfectly
550
Q

Where are the reticulo-endothelial cells concentrated within the spleen?

Within the capsule

Within the red pulp

Within the trabeculae

Within the white pulp

Within the splenic cords

A

The reticuloendothelial cells are concerned with the immune functions of the spleen and these are therefore concentrated in the white pulp.

Spleen- function

The spleen is a reticuloendothelial organ, it develops in the dorsal mesogastrium at around 5 weeks gestation. At this stage it has an irregular surface and migrates to the left upper quadrant over the following weeks. In most cases the irregular hillocks on the spleen surface unify, when they fail to do so, accessory spleens may develop and are found in around 20% of people. The spleen is composed of both red and white pulp. In the red pulp, blood filled venous sinuses are found. In the white pulp, reticuloendothelial cords and white lymphoid follicles are present. Blood flows into the spleen at a rate of 150ml per minute. As blood passes through the spleen, the erythrocytes have to pass through fine endothelial fenestrations, older and less deformable erythrocytes are trapped during this process and destroyed. Red cell inclusion bodies such as parasites or residual nuclear components are split off during this process which is termed pitting.

Functions of spleen
Maintenance of the quality of erythrocytes in the red pulp by removal of senescent and dysfunctional cells
Antibody production in the white pulp
Removal of antibody coated bacteria and blood cells from the circulation

Because of these important functions, the spleen is seldom removed. Indications for splenectomy include major trauma and uncontrollable haemorrhage and the treatment of haemolytic anaemia.

The absence of a spleen has minimal long term effects on the haematologic profile. In the immediate post operative period, both leucocytosis and thrombocytosis are seen. In the longer term, the main manifestations are visible on the blood film and include visible nuclear remnants (Howell Jolly bodies), denatured haemoglobin (Heinz bodies), basophilic stippling and occasional nucleated erythrocytes.
The main risk following splenectomy is overwhelming sepsis with encapsulated organisms. Patients should receive the pneumococcal vaccine and long term antibiotic prophylaxis.

How well did you know this?
1
Not at all
2
3
4
5
Perfectly
551
Q

Which of the structures listed below is not typically transmitted via the superior orbital fissure?

Lacrimal nerve

Abducens nerve

Zygomatic nerve

Superior opthalmic vein

Superior division of the oculomotor nerve

A

The zygomatic nerve is transmitted via the inferior orbital fissure.
A useful mnemonic for the structures that pass through the superior orbital fissure:
SUPERIOR ORBITAL FISSURE:
Outside
L - lacrimal F - frontal T - trochlear done
Inside
N - nasociliary A - abducens nerve O - oculomotor nerve
LFT done outside No Abnormality Occurred inside

Orbital foramina
Foramina Structures transmitted
Superior orbital fissure Recurrent meningeal artery*
Lacrimal nerve
Trochlear nerve
Abducens nerve
Superior ophthalmic vein
Superior division of the oculomotor nerve
Inferior orbital fissure Maxillary nerve
Inferior ophthalmic vein
Zygomatic nerve
Optic foramen Optic nerve
Ophthalmic artery

*=It may help to know that it is also termed the anastomotic branch ( of the middle meningeal artery). It enters the orbit lateral in the superior orbital fissure, it anastomoses with the recurrent branch of the lacrimal artery.

How well did you know this?
1
Not at all
2
3
4
5
Perfectly
552
Q

A 53 year old lady is recovering following a difficult mastectomy and axillary nodal clearance for carcinoma of the breast. She complains of shoulder pain and on examination has obvious winging of the scapula. Loss of innervation to which of the following is the most likely underlying cause?

Latissimus dorsi

Serratus anterior

Pectoralis minor

Pectoralis major

Rhomboids

A

Serratus anterior

Winging of the scapula is most commonly the result of long thoracic nerve injury or dysfunction. Iatrogenic damage during the course of the difficult axillary dissection is the most likely cause in this scenario. Damage to the rhomboids may produce winging of the scapula but would be rare in the scenario given.

Long thoracic nerve

Derived from ventral rami of C5, C6, and C7 (close to their emergence from intervertebral foramina)
It runs downward and passes either anterior or posterior to the middle scalene muscle
It reaches upper tip of serratus anterior muscle and descends on outer surface of this muscle, giving branches into it
Winging of Scapula occurs in long thoracic nerve injury (most common) or from spinal accessory nerve injury (which denervates the trapezius) or a dorsal scapular nerve injury

How well did you know this?
1
Not at all
2
3
4
5
Perfectly
553
Q

A 75-year-old man presents with hepatomegaly and ascites. A CT scan shows evidence of post hepatic portal hypertension. The inferior vena cava passes through the diaphragm at which vertebral level?

T5

T8

T9

T10

T11

A

It passes through the diaphragm at T8.

Inferior vena cava

Origin
L5

Path
Left and right common iliac veins merge to form the IVC.
Passes right of midline
Paired segmental lumbar veins drain into the IVC throughout its length
The right gonadal vein empties directly into the cava and the left gonadal vein generally empties into the left renal vein.
The next major veins are the renal veins and the hepatic veins
Pierces the central tendon of diaphragm at T8
Right atrium

Relations
Anteriorly Small bowel, first and third part of duodenum, head of pancreas, liver and bile duct, right common iliac artery, right gonadal artery
Posteriorly Right renal artery, right psoas, right sympathetic chain, coeliac ganglion

Levels
Level Vein
T8 Hepatic vein, inferior phrenic vein, pierces diaphragm
L1 Right suprarenal vein, renal vein
L2 Gonadal vein
L1-5 Lumbar veins
L5 Common iliac vein, formation of IVC

How well did you know this?
1
Not at all
2
3
4
5
Perfectly
554
Q

A man is undergoing excision of a sub mandibular gland. As the gland is mobilised, a vessel is injured lying between the gland and the mandible. Which of the following is this vessel most likely to be?

Lingual artery

Occipital artery

Superior thyroid artery

Facial artery

External jugular vein

A

The high salivary viscosity of submandibular gland secretions favors stone formation. Most stones are radio-opaque.
The marginal mandibular nerve is the most superficial structure.
The facial artery lies between the gland and mandible and is often ligated during excision of the gland. The lingual artery may be encountered but this is usually later in the operative process as Whartons duct is mobilised.

Submandibular gland

Relations of the submandibular gland
Superficial Platysma, deep fascia and mandible
Submandibular lymph nodes
Facial vein (facial artery near mandible)
Marginal mandibular nerve
Cervical branch of the facial nerve
Deep Facial artery (inferior to the mandible)
Mylohyoid muscle
Sub mandibular duct
Hyoglossus muscle
Lingual nerve
Submandibular ganglion
Hypoglossal nerve

Submandibular duct (Wharton’s duct)
Opens lateral to the lingual frenulum on the anterior floor of mouth.
5 cm length
Lingual nerve wraps around Wharton’s duct. As the duct passes forwards it crosses medial to the nerve to lie above it and then crosses back, lateral to it, to reach a position below the nerve.

Innervation
Sympathetic innervation- Derived from superior cervical ganglion
Parasympathetic innervation- Submandibular ganglion via lingual nerve

Arterial supply
Branch of the facial artery. The facial artery passes through the gland to groove its deep surface. It then emerges onto the face by passing between the gland and the mandible.

Venous drainage
Anterior facial vein (lies deep to the Marginal Mandibular nerve)

Lymphatic drainage
Deep cervical and jugular chains of nodes

How well did you know this?
1
Not at all
2
3
4
5
Perfectly
555
Q

A 44 year old man is stabbed in the back and the left kidney is injured. A haematoma forms, which of the following fascial structures will contain the haematoma?

Waldeyers fascia

Sibsons fascia

Bucks fascia

Gerotas fascia

Denonvilliers fascia

A

Waldeyers fascia- Posterior ano-rectum
Sibsons fascia- Lung apex
Bucks fascia- Base of penis
Gerotas fascia- Surrounding kidney
Denonvilliers fascia- Between rectum and prostate

Renal anatomy

Each kidney is about 11cm long, 5cm wide and 3cm thick. They are located in a deep gutter alongside the projecting vertebral bodies, on the anterior surface of psoas major. In most cases the left kidney lies approximately 1.5cm higher than the right. The upper pole of both kidneys approximates with the 11th rib (beware pneumothorax during nephrectomy). On the left hand side the hilum is located at the L1 vertebral level and the right kidney at level L1-2. The lower border of the kidneys is usually alongside L3.

The table below shows the anatomical relations of the kidneys:

Relations
Relations Right Kidney Left Kidney
Posterior Quadratus lumborum, diaphragm, psoas major, transversus abdominis Quadratus lumborum, diaphragm, psoas major, transversus abdominis
Anterior Hepatic flexure of colon Stomach, Pancreatic tail
Superior Liver, adrenal gland Spleen, adrenal gland

Fascial covering
Each kidney and suprarenal gland is enclosed within a common layer of investing fascia, derived from the transversalis fascia. It is divided into anterior and posterior layers (Gerotas fascia).

Renal structure
Kidneys are surrounded by an outer cortex and an inner medulla which usually contains between 6 and 10 pyramidal structures. The papilla marks the innermost apex of these. They terminate at the renal pelvis, into the ureter.
Lying in a hollow within the kidney is the renal sinus. This contains:
1. Branches of the renal artery
2. Tributaries of the renal vein
3. Major and minor calyces’s
4. Fat

Structures at the renal hilum
The renal vein lies most anteriorly, then renal artery (it is an end artery) and the ureter lies most posterior.

How well did you know this?
1
Not at all
2
3
4
5
Perfectly
556
Q

An 82 year old lady is taken to theatre for a common bile duct exploration. She has a stone impacted at the distal aspect of the common bile duct and despite best efforts it proves impossible to remove it. What is the best course of action?

Close the bile duct over a T Tube and arrange for a stent to be placed

Undertake a choledochoduodenostomy

Arrange for a repeat ERCP

Construct a hepaticojejunostomy

Bypass the gallbladder onto the jejunum

A

If a stone cannot be removed at surgery then the chances of succeeding at ERCP are slim. In this case, its probably best to bypass the distal bile duct and a choledochoduodenostomy is the best way of achieving this. There are long term risks of cholangitis which are less of a concern in older patients.

Biliary disease

Diagnosis Typical features Pathogenesis
Gallstones Typically history of biliary colic or episodes of chlolecystitis. Obstructive type history and test results. Usually small calibre gallstones which can pass through the cystic duct. In Mirizzi syndrome the stone may compress the bile duct directly- one of the rare times that cholecystitis may present with jaundice
Cholangitis Usually obstructive and will have Charcot’s triad of symptoms (pain, fever, jaundice) Ascending infection of the bile ducts usually by E. coli and by definition occurring in a pool of stagnant bile.
Pancreatic cancer Typically painless jaundice with palpable gallbladder (Courvoisier’s Law) Direct occlusion of distal bile duct or pancreatic duct by tumour. Sometimes nodal disease at the portal hepatis may be the culprit in which case the bile duct may be of normal calibre.
TPN (total parenteral nutrition) associated jaundice Usually follows long term use and is usually painless with non obstructive features Often due to hepatic dysfunction and fatty liver which may occur with long term TPN usage.
Bile duct injury Depending upon the type of injury may be of sudden or gradual onset and is usually of obstructive type Often due to a difficult laparoscopic cholecystectomy when anatomy in Calots triangle is not appreciated. In the worst scenario the bile duct is excised and jaundice develops rapidly post operatively. More insidious is that of bile duct stenosis which may be caused by clips or diathermy injury.
Cholangiocarcinoma Gradual onset obstructive pattern Direct occlusion by disease and also extrinsic compression by nodal disease at the porta hepatis.
Septic surgical patient Usually hepatic features Combination of impaired biliary excretion and drugs such as ciprofloxacin which may cause cholestasis.
Metastatic disease Mixed hepatic and post hepatic Combination of liver synthetic failure (late) and extrinsic compression by nodal disease and anatomical compression of intra hepatic structures (earlier)

A gallbladder may develop a thickened wall in chronic cholecystitis, microscopically Roikitansky-Aschoff Sinuses may be seen

How well did you know this?
1
Not at all
2
3
4
5
Perfectly
557
Q

A 72 year old man with carcinoma of the lung is undergoing a left pneumonectomy. The left main bronchus is divided. Which of the following thoracic vertebrae lies posterior to this structure?

T3

T7

T6

T10

T1

A

The left main bronchus lies at T6. Topographical anatomy of the thorax is important as it helps surgeons to predict the likely structures to be injured in trauma scenarios (so popular with examiners)

Lung anatomy

The right lung is composed of 3 lobes divided by the oblique and transverse fissures. The left lung has two lobes divided by the oblique fissure.The apex of both lungs is approximately 4cm superior to the sterno-costal joint of the first rib. Immediately below this is a sulcus created by the subclavian artery.

Peripheral contact points of the lung
Base: diaphragm
Costal surface: corresponds to the cavity of the chest
Mediastinal surface: Contacts the mediastinal pleura. Has the cardiac impression. Above and behind this concavity is a triangular depression named the hilum, where the structures which form the root of the lung enter and leave the viscus. These structures are invested by pleura, which, below the hilum and behind the pericardial impression, forms the pulmonary ligament

Right lung
Above the hilum is the azygos vein; Superior to this is the groove for the superior vena cava and right innominate vein; behind this, and nearer the apex, is a furrow for the innominate artery. Behind the hilum and the attachment of the pulmonary ligament is a vertical groove for the oesophagus; In front and to the right of the lower part of the oesophageal groove is a deep concavity for the extrapericardiac portion of the inferior vena cava.

The root of the right lung lies behind the superior vena cava and the right atrium, and below the azygos vein.

The right main bronchus is shorter, wider and more vertical than the left main bronchus and therefore the route taken by most foreign bodies.

Left lung
Above the hilum is the furrow produced by the aortic arch, and then superiorly the groove accommodating the left subclavian artery; Behind the hilum and pulmonary ligament is a vertical groove produced by the descending aorta, and in front of this, near the base of the lung, is the lower part of the oesophagus.

The root of the left lung passes under the aortic arch and in front of the descending aorta.

Inferior borders of both lungs
6th rib in mid clavicular line
8th rib in mid axillary line
10th rib posteriorly
The pleura runs two ribs lower than the corresponding lung level.

Bronchopulmonary segments
Segment number Right lung Left lung
1 Apical Apical
2 Posterior Posterior
3 Anterior Anterior
4 Lateral Superior lingular
5 Medial Inferior lingular
6 Superior (apical) Superior (apical)
7 Medial basal Medial basal
8 Anterior basal Anterior basal
9 Lateral basal Lateral basal
10 Posterior basal Posterior basal

How well did you know this?
1
Not at all
2
3
4
5
Perfectly
558
Q

A 19 year old man undergoes an open inguinal hernia repair. The cord is mobilised and the deep inguinal ring identified. Which of the following structures forms its lateral wall?

External oblique aponeurosis

Transversalis fascia

Conjoint tendon

Inferior epigastric artery

Inferior epigastric vein

A

The transversalis fascia forms the superolateral edge of the deep inguinal ring. The epigastric vessels form its inferomedial wall.
Inguinal canal

Location
Above the inguinal ligament
The inguinal canal is 4cm long
The superficial ring is located anterior to the pubic tubercle
The deep ring is located approximately 1.5-2cm above the half way point between the anterior superior iliac spine and the pubic tubercle

Boundaries of the inguinal canal
Floor
External oblique aponeurosis
Inguinal ligament
Lacunar ligament
Roof
Internal oblique
Transversus abdominis
Anterior wall External oblique aponeurosis
Posterior wall
Transversalis fascia
Conjoint tendon
Laterally
Internal ring
Transversalis fascia
Fibres of internal oblique
Medially
External ring
Conjoint tendon

Contents
Males Spermatic cord and ilioinguinal nerve As it passes through the canal the spermatic cord has 3 coverings:
External spermatic fascia from external oblique aponeurosis
Cremasteric fascia
Internal spermatic fascia
Females Round ligament of uterus and ilioinguinal nerve

How well did you know this?
1
Not at all
2
3
4
5
Perfectly
559
Q

A 55 year old lady presents with discomfort in the right breast. On clinical examination a small lesion is identified and clinical appearances suggest fibroadenoma. Imaging confirms the presence of a fibroadenoma alone. A core biopsy is taken, this confirms the presence of the fibroadenoma. However, the pathologist notices that a small area of lobular carcinoma in situ is also present in the biopsy. What is the most appropriate course of action?

Whole breast irradiation

Simple mastectomy

Mastectomy and sentinal lymph node biopsy

Wide local excision and sentinel lymph node biopsy

Breast MRI scan

A

Lobular carcinoma in situ has a low association with invasive malignancy. It is seldom associated with microcalcification and therefore MRI is the best tool for determining disease extent. Resection of in situ disease is not generally recommended and most surgeons would simply pursue a policy of close clinical and radiological follow up.

Lobular carcinoma of the breast

Lobular breast cancers are less common than their ductal counterparts. They typically present differently, the mass is usually more diffuse and less obvious on the usual imaging modalities of ultrasound and mammography. This is significant since the disease may be understaged resulting in inadequate treatment when wide local excision is undertaken.
In women with invasive lobular carcinoma it is usually safest to perform an MRI scan of the breast, if breast conserving surgery is planned.
Lobular carcinomas are also more likely to be multifocal and metastasise to the contralateral breast.
Lobular carcinoma in situ is occasionally diagnosed incidentally on core biopsies. Unlike DCIS, lobular carcinoma in situ is far less strongly associated with foci of invasion and is usually managed by close monitoring.

How well did you know this?
1
Not at all
2
3
4
5
Perfectly
560
Q

Administration of which of the following may facilitate the identification of parathyroid glands intra operatively?

Patent V dye intravenously

Methylene blue intravenously

Indigocarmine dye intravenously

India ink intravenously

Intravenous rifampicin

A

Methylene blue stains the parathyroid glands and can be useful in facilitating their identification.
Preparation for surgery

Elective and emergency patients require different preparation.

Elective cases
Consider pre admission clinic to address medical issues.
Blood tests including FBC, U+E, LFT’s, Clotting, Group and Save
Urine analysis
Pregnancy test
Sickle cell test
ECG/ Chest x-ray

Exact tests to be performed will depend upon the proposed procedure and patient fitness.

Risk factors for development of deep vein thrombosis should be assessed and a plan for thromboprophylaxis formulated.

Diabetes
Diabetic patients have greater risk of complications.
Poorly controlled diabetes carries high risk of wound infections.
Patients with diet or tablet controlled diabetes may be managed using a policy of omitting medication and checking blood glucose levels regularly. Diabetics who are poorly controlled or who take insulin may require a intravenous sliding scale. Potassium supplementation should also be given.
Diabetic cases should be operated on first.

Emergency cases
Stabilise and resuscitate where needed.
Consider whether antibiotics are needed and when and how they should be administered.
Inform blood bank if major procedures planned particularly where coagulopathies are present at the outset or anticipated (e.g. Ruptured AAA repair)
Don’t forget to consent and inform relatives.

Special preparation
Some procedures require special preparation:
Thyroid surgery; vocal cord check.
Parathyroid surgery; consider methylene blue to identify gland.
Sentinel node biopsy; radioactive marker/ patent blue dye.
Surgery involving the thoracic duct; consider administration of cream.
Pheochromocytoma surgery; will need alpha and beta blockade.
Surgery for carcinoid tumours; will need covering with octreotide.
Colorectal cases; bowel preparation (especially left sided surgery)
Thyrotoxicosis; lugols iodine/ medical therapy.

How well did you know this?
1
Not at all
2
3
4
5
Perfectly
561
Q

A 30 year old man is suspected of having appendicitis. At operation an inflamed Meckels diverticulum is found. Which of the following vessels is responsible for the blood supply to a Meckels diverticulum?

Right colic artery

Vitelline artery

Appendicular artery

Internal iliac artery

External iliac artery

A

The vitelline arteries supply a Meckels these are usually derived from the ileal arcades.

Meckel’s diverticulum

  • Congenital abnormality resulting in incomplete obliteration of the vitello-intestinal duct
    Normally, in the foetus, there is an attachment between the vitello-intestinal duct and the yolk sac.This disappears at 6 weeks gestation.
    The tip is free in majority of cases.
    Associated with enterocystomas, umbilical sinuses, and omphaloileal fistulas.
    Arterial supply: omphalomesenteric artery.
    2% of population, 2 inches long, 2 feet from the ileocaecal valve.
    Typically lined by ileal mucosa but ectopic gastric mucosa can occur, with the risk of peptic ulceration. Pancreatic and jejunal mucosa can also occur.

Clinical
Normally asymptomatic and an incidental finding.
Complications are the result of obstruction, ectopic tissue, or inflammation.
Removal if narrow neck or symptomatic. Options are between wedge excision or formal small bowel resection and anastomosis.

How well did you know this?
1
Not at all
2
3
4
5
Perfectly
562
Q

Which one of the following is least associated with Tetralogy of Fallot?

Right ventricular outflow tract obstruction

Overriding aorta

Pan systolic murmur

Left-to-right shunt

Right ventricular hypertrophy

A

Right-to-left shunting is characteristic of Fallot’s. In some patients there can be bidirectional shunting (if there is mild pulmonary stenosis) and a few patients can even have pink tetralogy when there is a predominant shunt from left to right due to minimal infundibular stenosis.

Tetralogy of Fallot

Tetralogy of Fallot (TOF) is the most common cause of cyanotic congenital heart disease*. It typically presents at around 1-2 months, although may not be picked up until the baby is 6 months old

The four characteristic features are:
ventricular septal defect (VSD)
right ventricular hypertrophy
right ventricular outflow tract obstruction, pulmonary stenosis
overriding aorta

The severity of the right ventricular outflow tract obstruction determines the degree of cyanosis and clinical severity

Other features
cyanosis
causes a right-to-left shunt
ejection systolic murmur due to pulmonary stenosis (the VSD doesn’t usually cause a murmur)
a right-sided aortic arch is seen in 25% of patients
chest x-ray shows a ‘boot-shaped’ heart, ECG shows right ventricular hypertrophy

Management
surgical repair is often undertaken in two parts
cyanotic episodes may be helped by beta-blockers to reduce infundibular spasm

*however, at birth transposition of the great arteries is the more common lesion as patients with TOF generally present at around 1-2 months

How well did you know this?
1
Not at all
2
3
4
5
Perfectly
563
Q

A 48 year old lady undergoes an ERCP for jaundice. 36 hours following the procedure she develops a fever and rigors. A blood culture is taken, which of the following organisms is most likely to be cultured?

Pseudomonas aeruginosa

Streptococcus

Enterobacter

Staphylococcus

Escherichia coli

A

Charcots triad = Surgical emergency.
Patients need: Biliary decompression and broad spectrum antibiotics.
E Coli is the most common organism implicated in cholangitis infections. Whilst enterobacter is occasionally cultured it is not the most common organism.

Cholangitis

  • Combination of bacterial infection and biliary obstruction
    Most common organisms are: (most frequent at top of list)
    Escherichia coli
    Klebsiella species
    Enterococcus species
    Streptococcus species

Clinical features
Charcot’s triad:
Fever (90% cases)
Right upper quadrant pain
Jaundice

Reynolds pentad: Above plus confusion and hypotension

Investigations
USS 1st line
CT scan
ERCP: may be 1st line if high clinical suspicion and suitable for treatment

Treatment
ERCP -usually after 72 hours of antibiotics
Percutaneous transhepatic cholangiogram and biliary drain

How well did you know this?
1
Not at all
2
3
4
5
Perfectly
564
Q

Which of the following anatomical structures lies within the spiral groove of the humerus?

Median nerve

Radial nerve

Tendon of triceps

Musculocutaneous nerve

Axillary nerve

A

The radial nerve lies in this groove and may be compromised by fractures involving the shaft.

Humerus

The humerus extends from the scapula to the elbow joint. It has a body and two ends. It is almost completely covered with muscle but can usually be palpated throughout its length. The smooth rounded surface of the head articulates with the shallow glenoid cavity. The head is connected to the body of the humerus by the anatomical neck. The surgical neck is the region below the head and tubercles and where they join the shaft and is the commonest site of fracture. The capsule of the shoulder joint is attached to the anatomical neck superiorly but extends down to 1.5cm on the surgical neck.

The greater tubercle is the prominence on the lateral side of the upper end of the bone. It merges with the body below and can be felt through the deltoid inferior to the acromion. The tendons of the supraspinatus and infraspinatus are inserted into impressions on its superior aspect. The lesser tubercle is a distinct prominence on the front of the upper end of the bone. It can be palpated through the deltoid just lateral to the tip of the coracoid process.

The intertubercular groove passes on the body between the greater and lesser tubercles, continuing down from the anterior borders of the tubercles to form the edges of the groove. The tendon of biceps within its synovial sheath passes through this groove, held within it by a transverse ligament.

The posterior surface of the body is marked by a spiral groove for the radial nerve which runs obliquely across the upper half of the body to reach the lateral border below the deltoid tuberosity. Within this groove lie the radial nerve and brachial vessels and both may be affected by fractures involving the shaft of the humerus.

The lower end of the humerus is wide and flattened anteroposteriorly, and inclined anteriorly. The middle third of the distal edge forms the trochlea. Superior to this are indentations for the coronoid fossa anteriorly and olecranon fossa posteriorly. Lateral to the trochlea is a rounded capitulum which articulates with the radius.

The medial epicondyle is very prominent with a smooth posterior surface which contains a sulcus for the ulnar nerve and collateral vessels. It’s distal margin gives attachment for the ulnar collateral ligament and, in front of this, the anterior surface has an impression for the common flexor tendon.

How well did you know this?
1
Not at all
2
3
4
5
Perfectly
565
Q

A 72 year old man presents with lower urinary tract symptoms. On digital rectal examination, benign prostatic hyperplasia is suspected. Which of the following treatments is associated with a reduction in the risk of urinary retention?

Alfuzosin

Finasteride

Prazosin

Tamsulosin

Terazosin

A

5 alpha reductase inhibitors reduce the risk of urinary retention.
In the PLESS study, data show a reduction in the risk of urinary retention although the absolute risk reduction was small.
Finasteride is a medication used to treat benign prostatic hyperplasia (BPH) by reducing the size of the prostate gland. It works by inhibiting the conversion of testosterone to dihydrotestosterone (DHT), which is a hormone that contributes to the growth of the prostate gland. Alfuzosin, tamsulosin, and terazosin are medications that belong to a class of drugs called alpha-blockers. These medications work by relaxing the smooth muscle in the prostate gland and bladder neck, which can improve urine flow and decrease urinary symptoms. While alpha-blockers are effective in improving urinary symptoms associated with BPH, they do not reduce the size of the prostate gland and are not associated with a reduction in the risk of urinary retention. Prazosin is an alpha-blocker that is used to treat high blood pressure and post-traumatic stress disorder (PTSD). It is not typically used to treat urinary symptoms associated with BPH
Reference
McConnell J et al. The effect of finasteride on the risk of urinary retention and the need for surgical intervention amongst men with benign prostatic hyperplasia. N Engl J Med 338:557-563

How well did you know this?
1
Not at all
2
3
4
5
Perfectly
566
Q

A 56 year old man is day one following a laparoscopic appendicectomy. He suddenly develops marked dyspnoea following administration of cyclizine to relieve post operative nausea. On arrival, the patient is breathless with a pulse rate of 122, Bp 86/48, oxygen saturations on air are 85%. Which of these agents should be administered initially?

Adrenaline

Chlorpheniramine

IV 0.9% saline

Hydrocortisone

Salbutamol

A

As this patient most likely has anaphylaxis, administration of adrenaline is most appropriate. Anaphylactic shock

  • Suspect if there has been exposure to an allergen

Management
- Remove allergen
- ABCD
- Drugs:

Adrenaline 1:1000 0.5ml INTRAMUSCULARLY (not IV). Repeat after 5 mins if no response.
Consider antihistamine if ongoing local symptoms.

In the refractory anaphylaxis guidance, consideration is given to adrenaline infusions.

Reference
Emergency treatment of anaphylactic reactions. Guidelines for healthcare providers. Working Group of the Resuscitation Council (UK).2021

How well did you know this?
1
Not at all
2
3
4
5
Perfectly
567
Q

Which of the following is not a feature of a Charcot foot?

Bounding foot pulses in the early phases

Often occurs in the complete absence of trauma

Erythema of the foot in the early phase

Autonomic neuropathy

Peripheral neuropathy

A

Do not confuse the early phase of Charcot foot with cellulitis
Trauma (even if only minor) is a prerequisite. Patients cannot usually recall the traumatic event. The associated neuropathy means that patients continue to walk on the affected foot with subsequent deformity developing over time.

Peripheral vascular disease

Indications for surgery to revascularise the lower limb
Intermittent claudication
Critical ischaemia
Ulceration
Gangrene

Intermittent claudication that is not disabling may provide a relative indication, whilst the other complaints are often absolute indications depending upon the frailty of the patient.

Assessment
Clinical examination
Ankle brachial pressure index measurement
Duplex arterial ultrasound
Angiography (standard, CT or MRI): usually performed only if intervention being considered.

Angioplasty
In order for angioplasty to be undertaken successfully the artery has to be accessible. The lesion relatively short and reasonable distal vessel runoff. Longer lesions may be amenable to sub-intimal angioplasty.

Surgery
Surgery will be undertaken where attempts at angioplasty have either failed or are unsuitable. Bypass essentially involves bypassing the affected arterial segment by utilising a graft to run from above the disease to below the disease. As with angioplasty good runoff improves the outcome.

Some key concepts with bypass surgery

Superficial femoral artery occlusion to the above knee popliteal
In the ideal scenario, vein (either in situ or reversed LSV) would be used as a conduit. However, prosthetic material has reasonable 5 year patency rates and some would advocate using this in preference to vein so that vein can be used for other procedures in the future. In general terms either technique is usually associated with an excellent outcome (if run off satisfactory).

Procedure
Artery dissected out, IV heparin 3,000 units given and then the vessels are cross clamped
Longitudinal arteriotomy
Graft cut to size and tunneled to arteriotomy sites
Anastomosis to femoral artery usually with 5/0 ‘double ended’ Prolene suture
Distal anastomosis usually using 6/0 ‘double ended’ Prolene

Distal disease
Femoro-distal bypass surgery takes longer to perform, is more technically challenging and has higher failure rates.
In elderly diabetic patients with poor runoff a primary amputation may well be a safer and more effective option. There is no point in embarking on this type of surgery in patients who are wheelchair bound.
In femorodistal bypasses vein gives superior outcomes to PTFE.

Rules
Vein mapping 1st to see whether there is suitable vein (the preferred conduit). Sub intimal hyperplasia occurs early when PTFE is used for the distal anastomosis and will lead to early graft occlusion and failure.
Essential operative procedure as for above knee fem-pop.
If there is insufficient vein for the entire conduit then vein can be attached to the end of the PTFE graft and then used for the distal anastomosis. This type of ‘vein boot’ is technically referred to as a Miller Cuff and is associated with better patency rates than PTFE alone.
Remember the more distal the arterial anastomosis the lower the success rate.

References
Peach G et al. Diagnosis and management of peripheral arterial disease. BMJ 2012; 345: 36-41.

How well did you know this?
1
Not at all
2
3
4
5
Perfectly
568
Q

What is the main impact of the Montgomery ruling in consenting patients for surgical procedures?

It clearly states that all risks however minor with a frequency of 10% or greater be disclosed

It states whether a reasonable person in the patients position would be likely to attach significance to the risk, or the doctor is or should reasonably be aware that the particular patient would be likely to attach significance to it

It states all procedures should be performed by a consultant

It covers the decision making for minors who need to be consented by proxy

It replaces the Gillick competency test

A

The new legal judgement recognises this individual approach to warning patients about risk. Rather than taking into account the percentage possibility of a risk arising, doctors need to bear in mind the significance of a given risk for that particular patient, and the nature of the risk, such as the effect it would have on the patient’s life if it were to occur. The assessment is therefore considered to be both fact-sensitive and sensitive to the characteristics of the particular patient.

Consent

There are 3 types of consent:

  1. Informed
  2. Expressed
  3. Implied

Consent forms used in UK NHS
Consent Form 1 For competent adults who are able to consent for themselves where consciousness may be impaired (e.g. GA)
Consent Form 2 For an adult consenting on behalf of a child where consciousness is impaired
Consent Form 3 For an adult or child where consciousness is not impaired
Consent Form 4 For adults who lack capacity to provide informed consent

Capacity
Key points include:
1. Understand and retain information
2. Patient believes the information to be true
3. Patient is able to weigh the information to make a decision
All patients must be assumed to have capacity

Consent in minors
Young children and older children who are not Gillick competent cannot consent for themselves. In British law the patients biological mother can always provide consent. The child’s father can consent if the parents are married (and the father is the biological father), or if the father is named on the birth certificate (irrespective of marital status). If parents are not married and the father is not named on the birth certificate then the father cannot consent.

How well did you know this?
1
Not at all
2
3
4
5
Perfectly
569
Q

A 56 year old man is admitted with passage of a large volume of blood per rectum. On examination, he is tachycardic, his abdomen is soft, although he has marked dilated veins on his abdominal wall. Proctoscopy reveals large dilated veins with stigmata of recent haemorrhage. What is the most appropriate treatment?

IV terlipressin

Excisional haemorrhoidectomy

Injection sclerotherapy

Proctectomy

Rectal pack insertion

A

Rectal varices are a recognised complication of portal hypertension. In the first instance they can be managed with medical therapy to lower pressure in the portal venous system. TIPSS may be considered. Whilst band ligation is an option, attempting to inject these in same way as haemorroids would carry a high risk of precipitating further haemorrhage.

Lower Gastrointestinal bleeding

Colonic bleeding
This typically presents as bright red or dark red blood per rectum. Colonic bleeding rarely presents as malaena type stool, this is because blood in the colon has a powerful laxative effect and is rarely retained long enough for transformation to occur and because the digestive enzymes present in the small bowel are not present in the colon. Up to 15% of patients presenting with haemochezia will have an upper gastrointestinal source of haemorrhage.

As a general rule right sided bleeds tend to present with darker coloured blood than left sided bleeds. Haemorrhoidal bleeding typically presents as bright red rectal bleeding that occurs post defecation either onto toilet paper or into the toilet pan. It is very unusual for haemorrhoids alone to cause any degree of haemodynamic compromise.

Causes
Cause Presenting features
Colitis Bleeding may be brisk in advanced cases, diarrhoea is commonly present. Abdominal x-ray may show featureless colon.
Diverticular disease Acute diverticulitis often is not complicated by major bleeding and diverticular bleeds often occur sporadically. 75% all will cease spontaneously within 24-48 hours. Bleeding is often dark and of large volume.
Cancer Colonic cancers often bleed and for many patients this may be the first sign of the disease. Major bleeding from early lesions is uncommon
Haemorrhoidal bleeding Typically bright red bleeding occurring post defecation. Although patients may give graphic descriptions bleeding of sufficient volume to cause haemodynamic compromise is rare.
Angiodysplasia Apart from bleeding, which may be massive, these arteriovenous lesions cause little in the way of symptoms. The right side of the colon is more commonly affected.

Management
Prompt correction of any haemodynamic compromise is required. Unlike upper gastrointestinal bleeding the first line management is usually supportive. This is because in the acute setting endoscopy is rarely helpful.
When haemorrhoidal bleeding is suspected a proctosigmoidoscopy is reasonable as attempts at full colonoscopy are usually time consuming and often futile.
In the unstable patient the usual procedure would be an angiogram (either CT or percutaneous), when these are performed during a period of haemodynamic instability they may show a bleeding point and may be the only way of identifying a patch of angiodysplasia.
In others who are more stable the standard procedure would be a colonoscopy in the elective setting. In patients undergoing angiography attempts can be made to address the lesion in question such as coiling. Otherwise surgery will be necessary.
In patients with ulcerative colitis who have significant haemorrhage the standard approach would be a sub total colectomy, particularly if medical management has already been tried and is not effective.

Indications for surgery
Patients > 60 years
Continued bleeding despite endoscopic intervention
Recurrent bleeding
Known cardiovascular disease with poor response to hypotension

Surgery
Selective mesenteric embolisation if life threatening bleeding. This is most helpful if conducted during a period of relative haemodynamic instability. If all haemodynamic parameters are normal then the bleeding is most likely to have stopped and any angiography normal in appearance. In many units a CT angiogram will replace selective angiography but the same caveats will apply.

If the source of colonic bleeding is unclear; perform a laparotomy, on table colonic lavage and following this attempt a resection. A blind sub total colectomy is most unwise, for example bleeding from an small bowel arterio-venous malformation will not be treated by this manoeuvre.

Summary of Acute Lower GI bleeding recommendations
Consider admission if:
* Over 60 years
* Haemodynamically unstable/profuse PR bleeding
* On aspirin or NSAID
* Significant co morbidity

Management
All patients should have a history and examination, PR and proctoscopy
Colonoscopic haemostasis aimed for in post polypectomy or diverticular bleeding

References
http://www.sign.ac.uk/guidelines/fulltext/105/index.html

How well did you know this?
1
Not at all
2
3
4
5
Perfectly
570
Q

The oxygen-haemoglobin dissociation curve is shifted to the right in which of the following scenarios?

Hypothermia

Respiratory alkalosis

Low altitude

Decreased 2,3-DPG in transfused red cells

Chronic iron deficiency anaemia

A

Chronic iron deficiency anemia
Mnemonic to remember causes of right shift of the oxygen dissociation curve:

CADET face RIGHT

C O2
A cidosis
2,3-DPG
E xercise
T emperature
The curve is shifted to the right when there is an increased oxygen requirement by the tissue. This includes:
Increased temperature
Acidosis
Increased DPG:
DPG is found in erythrocytes and is increased during glycolysis. It binds to the Hb molecule, thereby releasing oxygen to tissues. DPG is increased in conditions associated with poor oxygen delivery to tissues, such as anaemia and high altitude.

How well did you know this?
1
Not at all
2
3
4
5
Perfectly
571
Q

A 45 year old lady presents with a pathological fracture of her femoral shaft. She is a poor historian, but it transpires that she underwent a thyroidectomy 1 year previously. She has no other illness or co-morbidities. What is the most likely underlying diagnosis?

Hyperparathyroidism

Metastatic papillary carcinoma of the thyroid

Metastatic medullary carcinoma of the thyroid

Metastatic follicular carcinoma of the thyroid

None of the above

A

Follicular carcinomas are a recognised cause of bone metastasis. Papillary lesions typically spread via the lymphatics.

Thyroid malignancy

Papillary carcinoma
Commonest sub-type
Accurately diagnosed on fine needle aspiration cytology
Histologically, they may demonstrate psammoma bodies (areas of calcification) and so called ‘orphan Annie’ nuclei
They typically metastasise via the lymphatics and thus laterally located apparently ectopic thyroid tissue is usually a metastasis from a well differentiated papillary carcinoma

Follicular carcinoma
Are less common than papillary lesions
Like papillary tumours, they may present as a discrete nodule. Although they appear to be well encapsulated macroscopically there is invasion on microscopic evaluation
Lymph node metastases are uncommon and these tumours tend to spread haematogenously. This translates into a higher mortality rate
Follicular lesions cannot be accurately diagnosed on fine needle aspiration cytology and thus all follicular FNA’s (THY 3f) will require at least a hemi thyroidectomy

Anaplastic carcinoma
Less common and tend to occur in elderly females
Disease is usually advanced at presentation and often only palliative decompression and radiotherapy can be offered.

Medullary carcinoma
These are tumours of the parafollicular cells ( C Cells) and are of neural crest origin.
The serum calcitonin may be elevated which is of use when monitoring for recurrence.
They may be familial and occur as part of the MEN -2A disease spectrum.
Spread may be either lymphatic or haematogenous and as these tumours are not derived primarily from thyroid cells they are not responsive to radioiodine.

Lymphoma
These respond well to combined chemoradiotherapy
Radical surgery is unnecessary once the disease has been diagnosed on biopsy material. Such biopsy material is not generated by an FNA and thus a core biopsy has to be obtained (with care!).

How well did you know this?
1
Not at all
2
3
4
5
Perfectly
572
Q

A 17 year old male has a suspected testicular torsion and the scrotum is to be explored surgically. The surgeon incises the skin and then the dartos muscle. What is the next tissue layer that will be encountered during the dissection?

Visceral layer of the tunica vaginalis

Cremasteric fascia

Parietal layer of the tunica vaginalis

External spermatic fascia

Internal spermatic fascia

A

The layers that will be encountered are (in order):
1. Skin
2. Dartos fascia and muscle
3. External spermatic fascia
4. Cremasteric muscle and fascia
5. Internal spermatic fascia
6. Parietal layer of the tunica vaginalis

The layers of the spermatic cord and scrotum are a popular topic in the MRCS exam.
A mnemonic which may help:
Some Damned Examiner Called It The Testes (skin dartos external fascia cremaster internal fascia tunica Testes)

Scrotal and testicular anatomy

Spermatic cord
Formed by the vas deferens and is covered by the following structures:
Layer Origin
Internal spermatic fascia Transversalis fascia
Cremasteric fascia From the fascial coverings of internal oblique
External spermatic fascia External oblique aponeurosis

Contents of the cord
Vas deferens Transmits sperm and accessory gland secretions
Testicular artery Branch of abdominal aorta supplies testis and epididymis
Artery of vas deferens Arises from inferior vesical artery
Cremasteric artery Arises from inferior epigastric artery
Pampiniform plexus Venous plexus, drains into right or left testicular vein
Sympathetic nerve fibres Lie on arteries, the parasympathetic fibres lie on the vas
Genital branch of the genitofemoral nerve Supplies cremaster
Lymphatic vessels Drain to lumbar and para-aortic nodes

Scrotum
Composed of skin and closely attached dartos fascia.
Arterial supply from the anterior and posterior scrotal arteries
Lymphatic drainage to the inguinal lymph nodes
Parietal layer of the tunica vaginalis is the innermost layer

Testes
The testes are surrounded by the tunica vaginalis (closed peritoneal sac). The parietal layer of the tunica vaginalis adjacent to the internal spermatic fascia.
The testicular arteries arise from the aorta immediately inferiorly to the renal arteries.
The pampiniform plexus drains into the testicular veins, the left drains into the left renal vein and the right into the inferior vena cava.
Lymphatic drainage is to the para-aortic nodes.

How well did you know this?
1
Not at all
2
3
4
5
Perfectly
573
Q

A 46 year old lady presents with symptoms of diarrhoea, weight loss of 10 Kg and a skin rash of erythematous blisters involving the abdomen and buttocks. The blisters have an irregular border and both intact and ruptured vesicles. What is the most likely diagnosis?

Colonic adenocarcinoma

Pancreatic adenocarcinoma

Tropical sprue

Glucagonoma

Insulinoma

A

Glucagonoma is strongly associated with necrolytic migratory erythema.

Glucagonoma

Rare pancreatic tumours arising from the alpha cells of the pancreas.
Glucagon levels markedly elevated.
Symptoms include diarrhoea, weight loss and necrolytic migratory erythema.
A serum level of glucagon >1000pg/ml usually suggests the diagnosis, imaging with CT scanning is also required.
Treatment is with surgical resection. However, careful staging is required for these tumours are usually malignant and non resectable.

How well did you know this?
1
Not at all
2
3
4
5
Perfectly
574
Q

A 63 year old man with end stage osteoarthritis of the hip is due to undergo a total hip replacement. The skin has been prepared and antibiotics given. What is the single most important modality to reduce the risks of infection?

Laminar flow theatre

Exhaust suits

Skin shaving on the ward

Total body scrubbing of the surgical team

Extended antibiotic chemoprophylaxis as routine

A

A laminar flow is the single most important intervention, many units will also use exhaust suits but these are less essential. Shaving skin on the ward increases infection rates. Extended chemoprophylaxis increases risks of antibiotic associated diarrhea.

Surgical site infection

  • Surgical site infections may occur following a breach in tissue surfaces and allow normal commensals and other pathogens to initiate infection. They are a major cause of morbidity and mortality.
    Surgical site infections (SSI) comprise up to 20% of all healthcare associated infections and at least 5% of patients undergoing surgery will develop an SSI as a result.
    In many cases the organisms are derived from the patient’s own body. Measures that may increase the risk of SSI include:
    Shaving the wound using a razor (disposable clipper preferred)
    Using a non iodine impregnated incise drape if one is deemed to be necessary
    Tissue hypoxia
    Delayed administration of prophylactic antibiotics in tourniquet surgery

Preoperatively
Don’t remove body hair routinely
If hair needs removal, use electrical clippers with single use head (razors increase infection risk)
Antibiotic prophylaxis if:
- placement of prosthesis or valve
- clean-contaminated surgery
- contaminated surgery
Use local formulary
Aim to give single dose IV antibiotic on anaesthesia
If a tourniquet is to be used, give prophylactic antibiotics earlier

Intraoperatively
Prepare the skin with alcoholic chlorhexidine (Lowest incidence of SSI)
Cover surgical site with dressing
A recent meta analysis has confirmed that administration of supplementary oxygen does not reduce the risk of wound infection. In contrast to previous individual RCT’s(1)
Wound edge protectors do not appear to confer benefit (2)

Post operatively
Tissue viability advice for management of surgical wounds healing by secondary intention

Use of diathermy for skin incisions
In the NICE guidelines the use of diathermy for skin incisions is not advocated(3). Several randomised controlled trials have been undertaken and demonstrated no increase in risk of SSI when diathermy is used(4).

References
1. Brar M et al.. Perioperative supplemental oxygen in colorectal patients: a meta analysis. J Surg Res 2011 (166): 227 -235.
2. Pinkney T et al. Impact of wound edge protection devices on surgical site infection after laparotomy: impact of a multicentre randomised controlled trial (ROSSINI Trial). BMJ 2013 (347):10.
3. http://www.nice.org.uk/CG74
4. Ahmad N and Ahmed A. Meta-analysis of the effectiveness of surgical scalpel or diathermy in making abdominal skin incisions. Ann Surg 2011, 253(1):8-13.

How well did you know this?
1
Not at all
2
3
4
5
Perfectly
575
Q

Which of the following pairings of foramina and their contents is not correct?

Superior orbital fissure and the oculomotor nerve

Foramina rotundum and the maxillary nerve

Jugular foramen and the hypoglossal nerve

Foramina spinosum and the middle meningeal artery

Carotid canal and the internal carotid artery

A

The hypoglossal nerve passes through the hypoglossal canal.

Foramina of the base of the skull

Foramen Location Contents
Foramen ovale Sphenoid bone Otic ganglion
V3 (Mandibular nerve:3rd branch of
trigeminal)
Accessory meningeal artery
Lesser petrosal nerve
Emissary veins
Foramen spinosum Sphenoid bone Middle meningeal artery
Meningeal branch of the Mandibular nerve
Foramen rotundum Sphenoid bone Maxillary nerve (V2)
Foramen lacerum/ carotid canal Located between the sphenoid, the apex of the petrous temporal and the basilar part of the occipital Base of the medial pterygoid plate.
Internal carotid artery*
Nerve and artery of the pterygoid canal
Jugular foramen Temporal bone Anterior: inferior petrosal sinus
Intermediate: glossopharyngeal, vagus, and accessory nerves.
Posterior: sigmoid sinus (becoming the internal jugular vein) and some meningeal branches from the occipital and ascending pharyngeal arteries.
Foramen magnum Occipital bone Anterior and posterior spinal arteries
Vertebral arteries
Medulla oblongata
Stylomastoid foramen Temporal bone Stylomastoid artery
Facial nerve
Superior orbital fissure Sphenoid bone Oculomotor nerve (III)
Recurrent meningeal artery
Trochlear nerve (IV)
Lacrimal, frontal and nasociliary branches of opthalmic nerve (V1)
Abducent nerve (VI)
Superior ophthalmic vein

*= In life the foramen lacerum is occluded by a cartilagenous plug. The ICA initially passes into the carotid canal which ascends superomedially to enter the cranial cavity through the foramen lacerum.

How well did you know this?
1
Not at all
2
3
4
5
Perfectly
576
Q

A 26 year old man presents to the emergency department with a swelling over his left elbow after a fall on an outstretched hand. On examination, he has tenderness over the proximal part of his forearm, and has severely restricted supination and pronation movements. What is the most likely injury?

Fracture of the olecranon

Fracture of the radial head

Galeazzi fracture

Fracture of the shaft of the radius and ulnar

Fracture of the coronoid process

A

Fracture of the radial head is common in young adults. It is usually caused by a fall on the outstretched hand. On examination, there is marked local tenderness over the head of the radius, impaired movements at the elbow, and a sharp pain at the lateral side of the elbow at the extremes of rotation (pronation and supination).

Upper limb fractures

Colles’ fracture
Fall onto extended outstretched hands
Described as a dinner fork type deformity
Classical Colles’ fractures have the following 3 features:

Features of the injury
1. Transverse fracture of the radius
2. 1 inch proximal to the radio-carpal joint
3. Dorsal displacement and angulation

Smith’s fracture (reverse Colles’ fracture)
Volar angulation of distal radius fragment (Garden spade deformity)
Caused by falling backwards onto the palm of an outstretched hand or falling with wrists flexed

Bennett’s fracture
Intra-articular fracture of the first carpometacarpal joint
Impact on flexed metacarpal, caused by fist fights
X-ray: triangular fragment at ulnar base of metacarpal

Monteggia’s fracture
Dislocation of the proximal radioulnar joint in association with an ulna fracture
Fall on outstretched hand with forced pronation
Needs prompt diagnosis to avoid disability

Galeazzi fracture
Radial shaft fracture with associated dislocation of the distal radioulnar joint
Occur after a fall on the hand with a rotational force superimposed on it.
On examination, there is bruising, swelling and tenderness over the lower end of the forearm.
X Rays reveal the displaced fracture of the radius and a prominent ulnar head due to dislocation of the inferior radio-ulnar joint.

Barton’s fracture
Distal radius fracture (Colles’/Smith’s) with associated radiocarpal dislocation
Fall onto extended and pronated wrist

Scaphoid fractures
Scaphoid fractures are the commonest carpal fractures.
Surface of scaphoid is covered by articular cartilage with small area available for blood vessels (fracture risks blood supply)
Forms floor of anatomical snuffbox
Risk of fracture associated with fall onto outstretched hand (tubercle, waist, or proximal 1/3)
The main physical signs are swelling and tenderness in the anatomical snuff box, and pain on wrist movements and on longitudinal compression of the thumb.
Ulnar deviation AP needed for visualization of scaphoid
Immobilization of scaphoid fractures difficult

Radial head fracture
Fracture of the radial head is common in young adults.
It is usually caused by a fall on the outstretched hand.
On examination, there is marked local tenderness over the head of the radius, impaired movements at the elbow, and a sharp pain at the lateral side of the elbow at the extremes of rotation (pronation and supination).

How well did you know this?
1
Not at all
2
3
4
5
Perfectly
577
Q

A 35 year old man is admitted to hospital with vomiting, nausea and severe headaches. An MRI scan reveals a tumour of the cerebellopontine angle. Which one of the following pairs of cranial nerves is most likely to be compressed by this tumour?

Accessory and vagus

Facial and vagus

Facial and vestibulocochlear

Glossopharyngeal and vestibulocochlear

Vagus and vestibulocochlear

A

The cerebellopontine angle is located between the superior and inferior limbs of the angular cerebellopontine fissure formed by the petrosal cerebellar surface folding around the pons and middle cerebellar peduncle. The cerebellopontine fissure opens medially and has superior and inferior limbs that meet at a lateral apex. The fourth through the eleventh cranial nerves are located near or within the angular space between the two limbs commonly referred to as the cerebellopontine angle. The commonest lesion to affect this site is an acoustic neuroma. Therefore the vestibulocochlear nerve is commonly compromised. Larger lesions may also affect the facial nerve which lies closest to this site.

Cranial nerves

Cranial nerve lesions
Olfactory nerve May be injured in basal skull fractures or involved in frontal lobe tumour extension. Loss of olfactory nerve function in relation to major CNS pathology is seldom an isolated event and thus it is poor localiser of CNS pathology.
Optic nerve Problems with visual acuity may result from intra ocular disorders. Problems with the blood supply such as amaurosis fugax may produce temporary visual distortion. More important surgically is the pupillary response to light. The pupillary size may be altered in a number of disorders. Nerves involved in the resizing of the pupil connect to the pretectal nucleus of the high midbrain, bypassing the lateral geniculate nucleus and the primary visual cortex. From the pretectal nucleus neurones pass to the Edinger - Westphal nucleus, motor axons from here pass along with the oculomotor nerve. They synapse with ciliary ganglion neurones; the parasympathetic axons from this then innervate the iris and produce miosis. The miotic pupil is seen in disorders such as Horner’s syndrome or opiate overdose.
Mydriasis is the dilatation of the pupil in response to disease, trauma, drugs (or the dark!). It is pathological when light fails to induce miosis. The radial muscle is innervated by the sympathetic nervous system. Because the parasympathetic fibres travel with the oculomotor nerve they will be damaged by lesions affecting this nerve (e.g. cranial trauma).
The response to light shone in one eye is usually a constriction of both pupils. This indicates intact direct and consensual light reflexes. When the optic nerve has an afferent defect the light shining on the affected eye will produce a diminished pupillary response in both eyes. Whereas light shone on the unaffected eye will produce a normal pupillary response in both eyes. This is referred to as the Marcus Gunn pupil and is seen in conditions such as optic neuritis. In a total CN II lesion shining the light in the affected eye will produce no response.
Oculomotor nerve The pupillary effects are described above. In addition it supplies all ocular muscles apart from lateral rectus and superior oblique. Thus the affected eye will be deviated inferolaterally. Levator palpebrae superioris may also be impaired resulting in impaired ability to open the eye.
Trochlear nerve The eye will not be able to look down.
Trigeminal nerve Largest cranial nerve. Exits the brainstem at the pons. Branches are ophthalmic, maxillary and mandibular. Only the mandibular branch has both sensory and motor fibres. Branches converge to form the trigeminal ganglion (located in Meckels cave). It supplies the muscles of mastication and also tensor veli palatine, mylohyoid, anterior belly of digastric and tensor tympani. The detailed descriptions of the various sensory functions are described in other areas of the website. The corneal reflex is important and is elicited by applying a small tip of cotton wool to the cornea, a reflex blink should occur if it is intact. It is mediated by: the naso ciliary branch of the ophthalmic branch of the trigeminal (sensory component) and the facial nerve producing the motor response. Lesions of the afferent arc will produce bilateral absent blink and lesions of the efferent arc will result in a unilateral absent blink.
Abducens nerve The affected eye will have a deficit of abduction. This cranial nerve exits the brainstem between the pons and medulla. It thus has a relatively long intra cranial course which renders it susceptible to damage in raised intra cranial pressure.
Facial nerve Emerges from brainstem between pons and medulla. It controls muscles of facial expression and taste from the anterior 2/3 of the tongue. The nerve passes into the petrous temporal bone and into the internal auditory meatus. It then passes through the facial canal and exits at the stylomastoid foramen. It passes through the parotid gland and divides at this point. It does not innervate the parotid gland. Its divisions are considered in other parts of the website. Its motor fibres innervate orbicularis oculi to produce the efferent arm of the corneal reflex. In surgical practice it may be injured during parotid gland surgery or invaded by malignancies of the gland and a lower motor neurone on the ipsilateral side will result.
Vestibulo-cochlear nerve Exits from the pons and then passes through the internal auditory meatus. It is implicated in sensorineural hearing loss. Individuals with sensorineural hearing loss will localise the sound in webers test to the normal ear. Rinnes test will be reduced on the affected side but should still work. These two tests will distinguish sensorineural hearing loss from conductive deafness. In the latter condition webers test will localise to the affected ear and Rinnes test will be impaired on the affected side. Surgical lesions affecting this nerve include CNS tumours and basal skull fractures. It may also be damaged by the administration of ototoxic drugs (of which gentamicin is the most commonly used in surgical practice).
Glossopharyngeal nerve Exits the pons just above the vagus. Receives sensory fibres from posterior 1/3 tongue, tonsils, pharynx and middle ear (otalgia may occur following tonsillectomy). It receives visceral afferents from the carotid bodies. It supplies parasympathetic fibres to the parotid gland via the otic ganglion and motor function to stylopharyngeaus muscle. The sensory function of the nerve is tested using the gag reflex.
Vagus nerve Leaves the medulla between the olivary nucleus and the inferior cerebellar peduncle. Passes through the jugular foramen and into the carotid sheath. Details of the functions of the vagus nerve are covered in the website under relevant organ sub headings.
Accessory nerve Exists from the caudal aspect of the brainstem (multiple branches) supplies trapezius and sternocleidomastoid muscles. The distal portion of this nerve is most prone to injury during surgical procedures.
Hypoglossal nerve Emerges from the medulla at the preolivary sulcus, passes through the hypoglossal canal. It lies on the carotid sheath and passes deep to the posterior belly of digastric to supply muscles of the tongue (except palatoglossus). Its location near the carotid sheath makes it vulnerable during carotid endarterectomy surgery and damage will produce ipsilateral defect in muscle function.

How well did you know this?
1
Not at all
2
3
4
5
Perfectly
578
Q

A 50 year old male presents with painless frank haematuria. Clinical examination is unremarkable. Routine blood tests reveal a haemoglobin of 18g/dl but are otherwise normal. What is the most likely underlying diagnosis?

Squamous cell carcinoma of the bladder

Adenocarcinoma of the prostate

Adenocarcinoma of the kidney

Wilms tumour

Transitional cell carcinoma of the renal pelvis

A

Polycythaemia is a recognised feature of renal cell carcinoma. Wilms tumours most commonly occur in children.

Haematuria

Causes of haematuria

Trauma
Injury to renal tract
Renal trauma commonly due to blunt injury (others penetrating injuries)
Ureter trauma rare: iatrogenic
Bladder trauma: due to RTA or pelvic fractures
Infection
Remember TB
Malignancy
Renal cell carcinoma (remember paraneoplastic syndromes): painful or painless
Urothelial malignancies: 90% are transitional cell carcinoma, can occur anywhere along the urinary tract. Painless haematuria.
Squamous cell carcinoma and adenocarcinoma: rare bladder tumours
Prostate cancer
Penile cancers: SCC
Renal disease
Glomerulonephritis
Stones
Microscopic haematuria common
Structural abnormalities
Benign prostatic hyperplasia (BPH) causes haematuria due to hypervascularity of the prostate gland
Cystic renal lesions e.g. polycystic kidney disease
Vascular malformations
Renal vein thrombosis due to renal cell carcinoma
Coagulopathy
Causes bleeding of underlying lesions
Drugs
Cause tubular necrosis or interstitial nephritis: aminoglycosides, chemotherapy
Interstitial nephritis: penicillin, sulphonamides, and NSAIDs
Anticoagulants
Benign
Exercise
Gynaecological
Endometriosis: flank pain, dysuria, and haematuria that is cyclical
Iatrogenic
Catheterisation
Radiotherapy; cystitis, severe haemorrhage, bladder necrosis
Pseudohaematuria For example following consumption of beetroot

References
Http://bestpractice.bmj.com/best-practice/monograph/316/overview/aetiology.html

How well did you know this?
1
Not at all
2
3
4
5
Perfectly
579
Q

A surgical unit are conducting a study to determine whether patients who have bowel preparation have a lower risk of colonic anastomotic leakage than those having none. The planned sample size is 25. Which of the tests below is most appropriate?

Paired T test

Unpaired T test

Fishers exact test

Chi squared test

LSD post hoc test

A

It is likely to be underpowered with the number provided. However, it would be possible to classify such data into a 2x2 contingency table. However, when the sample size is small, the Chi squared test is not suitable and in these situations the Fishers exact test is used.

Statistics

Statistics is a topic that generally strikes fear and dread into most surgeons hearts. The MRCS is not an examination designed to test mathematical skill but the examiners do expect you to have working knowledge of commonly used tests so that you can appraise the literature properly.

Data types
Before selecting a method of statistical analysis it is imperative that the type of data to be analysed is correctly categorised. Commonly used terms include nominal, ordinal, interval and continuous.
Term Interpretation
Nominal Data can be allocated a numerical code that is arbitrary. For example allocating people as alive or dead using codes of 0 or 1
Ordinal data Data using numbers that can be used on a scale. Severity of pain is often measured in this way
Interval scale Data is measured numerically. However, the zero point is arbitrary
Continuous Data is measured numerically where the numerical value is a real number and may be any value. Examples include height and weight

Analysing data
Having ascribed the data it is then possible to begin the process of analysis. Nominal data is often tabulated into categories because of the nature of the underlying data sets. Continuous data may be displayed graphically often as individual data points. When the sample size is large enough, continuous data can be analysed to determine the distribution of the data points. Often, but not always these will be in the form of a gaussian distribution. Determining whether data is normally distributed or not is key to making sense of the subsequent statistical tests. Parametric tests are used to test normally distributed data, the T Test is one of the best examples. Data which is not normally distributed cannot be analysed in this way and a non parametric test must be used. Examples of such tests include Chi Squared and Mann Whitney U tests. Chi squared tests often appear in the medical literature. There are some assumptions that are made in relation to Chi squared tests; these include the need to use 2 degrees of freedom (usually) and the minimum sample size. Where the sample size is small then a different test is appropriate and the Fishers exact test is often used.
In situations where data is normally distributed and paired samples are taken from the same individuals (such as following an intervention) then the paired T Test may be used.

Multiple testing and post hoc analysis
In the ideal world statistical analysis is conducted on data that is collected prospectively according to pre set power calculations and defined end points. Occasionally, data does not produce an expected outcome or a certain type of patient appears to have a different result. Subsequent analysis of such groups is termed a post hoc analysis. This can be perfectly legitimate, alternatively it can represent the last ditch attempt of a researcher to try and find any aspect of the data that is worthwhile. This can lead to errors and false rejection of a null hypothesis. A statistically significant result is more likely to occur if the same dataset is subjected to multiple analyses. To counteract this problem some researchers will apply a Bonferroni correction, this adjusts the analysis to allow for multiple testing.

How well did you know this?
1
Not at all
2
3
4
5
Perfectly
580
Q

A 43 year old lady complains of significant varicose veins that are increasingly symptomatic. On examination, there is clinical signs of sapheno femoral junction incompetence and associated truncal varicosities. What is the most appropriate course of action?

Undertake a venous duplex scan

Undertake a venous doppler study

Perform a Trendelenberg test

Offer the patient radiofrequency ablative therapy

Offer the patient surgical excision of the varicosities

A

In modern venous surgery, decision making is based on venous imaging to determine which treatment modality is best. Few surgeons would proceed straight to surgery and excision without this test. A venous doppler would show incompetence of the valve but not more than that.

Vascular investigations

Venous disease

Venous Doppler
The simplest investigation for assessment of venous junctional incompetence is a Doppler assessment. This involves the patient standing and manual compression of the limb distal to the junction of interest. Flow should normally occur in one direction only. Where junctional incompetence is present reverse flow will occur and is relatively easy to identify.

Venograms and duplex scans
Structural venous information is historically obtained using a venogram. This is an invasive test and rarely required in modern clinical practice. The most helpful test is a venous duplex scan which will provide information relating to flow and vessel characteristics. Duplex is also useful in providing vein maps for bypass surgery.

Arterial disease

Ankle-brachial pressure
The ankle brachial pressure index measurement is an important investigation as it will allow classification of the severity of the flow compromise present. False readings may occur in those with calcified vessels such as diabetics and results in such settings should be interpreted with caution. When auscultating the vessel note should be made of the character of the signal. Monophasic signals are associated with a proximal stenosis and reduction in flow. Triphasic signals provide reassurance of a healthy vessel.

Arterial Duplex
As with the vein the duplex scan can provide a substantial amount of information about arterial patency and flow patterns. In skilled hands they can provide insight as to the state of proximal vessels that are anatomically inaccessible to duplex (e.g. Iliacs). Through assessment of distal flow patterns. It is an operator dependent test.

Conventional angiogram
Vessel puncture and catheter angiography is the gold standard method of assessing arteries. High quality information can usually be obtained. Limitations of the technique include the risk of contrast toxicity and risks of vessel damage. Severely calcified vessels may be difficult to puncture and in this situation a remote access site (e.g. brachial) may be used. This technique is particularly useful in providing a distal arterial roadmap prior to femoro-distal bypass.

CT angiography
These tests provide a considerable amount of structural and flow information. They require contrast and thus carry the risks associated with this. They are particularly useful in the setting of GI bleeding as they are rapidly available and can be performed by a non vascular radiologist. However, they lack the facility for endovascular intervention. In general they do not provide high enough resolution for distal arterial surgery.

Magnetic resonance angiography
This has the advantage of being non-invasive and not using nephrotoxic contrast. Movement artifact remains a problem in some sites and distal arterial resolution is imperfect.

How well did you know this?
1
Not at all
2
3
4
5
Perfectly
581
Q

A 22 year old woman has recently undergone a surgical excision of the submandibular gland. She presents to the follow up clinic with a complaint of tongue weakness on the ipsilateral side to her surgery. Which nerve has been damaged?

Hypoglossal nerve

Lingual nerve

Inferior alveolar nerve

Facial nerve

Lesser petrosal nerve

A

Three cranial nerves may be injured during submandibular gland excision.
Marginal mandibular branch of the facial nerve
Lingual nerve
Hypoglossal nerve
Hypoglossal nerve damage may result in paralysis of the ipsilateral aspect of the tongue. The nerve itself lies deep to the capsule surrounding the gland and should not be injured during an intracapsular dissection. The lingual nerve is probably at greater risk of injury. However, the effects of lingual nerve injury are sensory rather than motor.

Submandibular gland

Relations of the submandibular gland
Superficial Platysma, deep fascia and mandible
Submandibular lymph nodes
Facial vein (facial artery near mandible)
Marginal mandibular nerve
Cervical branch of the facial nerve
Deep Facial artery (inferior to the mandible)
Mylohyoid muscle
Sub mandibular duct
Hyoglossus muscle
Lingual nerve
Submandibular ganglion
Hypoglossal nerve

Submandibular duct (Wharton’s duct)
Opens lateral to the lingual frenulum on the anterior floor of mouth.
5 cm length
Lingual nerve wraps around Wharton’s duct. As the duct passes forwards it crosses medial to the nerve to lie above it and then crosses back, lateral to it, to reach a position below the nerve.

Innervation
Sympathetic innervation- Derived from superior cervical ganglion
Parasympathetic innervation- Submandibular ganglion via lingual nerve

Arterial supply
Branch of the facial artery. The facial artery passes through the gland to groove its deep surface. It then emerges onto the face by passing between the gland and the mandible.

Venous drainage
Anterior facial vein (lies deep to the Marginal Mandibular nerve)

Lymphatic drainage
Deep cervical and jugular chains of nodes

How well did you know this?
1
Not at all
2
3
4
5
Perfectly
582
Q

A 68 year old man with poorly controlled diabetes presents with severe otalgia and headaches. On examination, there is granulation tissue within the external auditory meatus. What is the most likely underlying infective agent?

Pseudomonas aeruginosa

Streptococcus pyogenes

Staphylococcus aureus

Actinomyces

Bacteroides fragilis

A

Malignant otitis externa is caused by Pseudomonas aeruginosa
Severe pain, headaches and granulation tissue within the external auditory meatus are key features of malignant otitis externa. Diabetes mellitus is one of the commonest risk factors.

Malignant otitis externa

  • Uncommon type of otitis externa that is found in immunocompromised individuals (90% cases found in diabetics)
    Infective organism is usually Pseudomonas aeruginosa
    Infection commences in the soft tissues of the external auditory meatus, then progresses to involve the soft tissues and into the bony ear canal
    Progresses to temporal bone osteomyelitis

Key features in history
Diabetes (90%) or immunosuppression (illness or treatment related)
Severe, unrelenting, deep-seated otalgia
Temporal headaches
Purulent otorrhea
Possibly dysphagia, hoarseness, and/or facial nerve dysfunction

Treatment
Anti pseudomonal antimicrobial agents
Topical agents
Hyperbaric oxygen is sometimes used in refractory cases

How well did you know this?
1
Not at all
2
3
4
5
Perfectly
583
Q

A 38 year old man undergoes an OGD to investigate dyspepsia. Following intubation of the duodenum, the ampulla of Vater is identified. At which of these sites, is the it most likely to be located?

Inferior aspect of the 1st part of the duodenum

Medial aspect of the 3rd part of the duodenum

Lateral aspect of the 2nd part of the duodenum

Lateral aspect of the 3rd part of the duodenum

Medial aspect of the 2nd part of the duodenum

A

The Ampulla of Vater is usually located 8-10cm from the pylorus and 2-3cm inferior to the opening of the accessory pancreatic duct of the lesser duodenal papilla. Knowledge of the location of the Ampulla is important when cannulating it at ERCP.

Duodenum

This is the first and widest part of the small bowel. It has a diameter of around 4-5cm. Its commencement is immediately distal to the pylorus and it runs for around 25cm where it becomes the jejunum at the region of the duodenojejunal flexure. It comprises four parts, superior, descending, horizontal and ascending. Of these, the horizontal is the longest segment. The first 2-3cm of the superior duodenum are intraperitoneal. The remainder is largely retroperitoneal with the exception of the final 1-2cm.

Medial relations of the duodenum include the superior pancreatico-duodenal artery and the pancreatic head. The descending duodenum is closely related to the commencement of the transverse colon which has little in the way of mesentery at this area. Posterior to the descending duodenum lies the right kidney.

The horizontal part passes transversely to the left with an upward deflection as it does so. From right to left it crosses in front of the right ureter, right psoas major, right gonadal vessels and IVC. It terminates anterior to the aorta. Anteriorly, it’s relations include the superior mesenteric vessels and the root of the small bowel.

The ascending part runs to the left of the aorta and upwards to the level of L2. It terminates by binding abruptly forwards as the duodenojejunal flexure. Posteriorly, are the left sympathetic trunk, left psoas major and left gonadal vessels. Anteriorly, it gives attachment to the root of the mesentery, while the left kidney lies laterally and the uncinate process of the pancreas lies medially. The region of the duodenojenunal flexure is fixed in position by the suspensory muscle of the duodenum. This fibromuscular band blends with the musculature of the flexure and passes upwards deep to the pancreas to gain attachment to the right crus of the diaphragm. It is referred to eponymously as the ligament of Treitz.

How well did you know this?
1
Not at all
2
3
4
5
Perfectly
584
Q

The following statements relating to the ankle joint are true except?

Three groups of ligaments provide mechanical stability

The sural nerve lies medial to the Achilles tendon at its point of insertion

Eversion of the foot occurs at the sub talar joint

The flexor hallucis longus tendon is the most posterior structure at the medial malleolus

The saphenous nerve crosses the ankle joint.

A

The sural nerve lies behind the distal fibula. Inversion and eversion are sub talar movements. The structures passing behind the medial malleolus from anterior to posterior include: tibialis posterior, flexor digitorum longus, posterior tibial vein, posterior tibial artery, nerve, flexor hallucis longus.

Ankle joint

The ankle joint is a synovial joint composed of the tibia and fibula superiorly and the talus inferiorly.

Ligaments of the ankle joint
Deltoid ligament (medially)
Lateral collateral ligament
Talofibular ligaments (both anteriorly and posteriorly)
The calcaneofibular ligament is separate from the fibrous capsule of the joint. The two talofibular ligaments are fused with it.

The components of the syndesmosis are
Antero-inferior tibiofibular ligament
Postero-inferior tibiofibular ligament
Inferior transverse tibiofibular ligament
Interosseous ligament

Movements at the ankle joint
Plantar flexion (55 degrees)
Dorsiflexion (35 degrees)
Inversion and eversion movements occur at the level of the sub talar joint

Nerve supply
Branches of deep peroneal and tibial nerves.

References
Golano P et al. Anatomy of the ankle ligaments: a pictorial essay. Knee Surg Sports Traumatol Arthrosc. 2010 May;18(5):557-69

How well did you know this?
1
Not at all
2
3
4
5
Perfectly
585
Q

In examining a biopsy of a primary tumour, the clearest evidence of malignancy is provided by:

Absence of a capsule

Basophilia of the cytoplasm

Invasion of surrounding structures

Excess of mitoses

Nuclear aberrations

A

Invasion is the hallmark of malignancy. The others may occur in insitu disease or dysplastic lesions.

Tissue sampling

Tissue sampling is an important surgical process. Biopsy modalities vary according to the site, experience and subsequent planned therapeutic outcome

The modalities comprise:
-Fine needle aspiration cytology
-Core biopsy
-Excision biopsy
-Tru cut biopsy
-Punch biopsy
-Cytological smears
-Endoscopic or laparoscopic biopsy

When the lesion is superficial the decision needs to be taken as to whether complete excision is desirable or whether excision biopsy is acceptable. In malignant melanoma for example the need for safe margins will mean that a more radical surgical approach needs to be adopted after diagnostic confirmation from excision biopsy than would be the case in basal cell carcinoma. Punch biopsies are useful in gaining histological diagnosis of unclear skin lesions where excision biopsy is undesirable such as in establishing whether a skin lesion is vasculitic or not.

Fine needle aspiration cytology (FNAC) is an operator dependent procedure that may or may not be image guided and essentially involves passing a needle through a lesion whilst suction is applied to a syringe. The material thus obtained is expressed onto a slide and sent for cytological assessment. This test can be limited by operator inexperience and also by the lack of histological architectural information (e.g. Follicular carcinoma of the thyroid). Where a discharge is present a sample may be sent for cytology although in some sites (e.g. Nipple discharge ) the information gleaned may be meaningless.

Tissue samples may be obtained by both core and tru cut biopsy. A core biopsy is obtained by use of a spring loaded gun with a needle passing quickly through the lesion of interest. A tru cut biopsy achieves the same objective but the needle moved by hand. When performing these techniques image guidance may be desirable (e.g. In breast lesions). Consideration needs to be given to any planned surgical resection as it may be necessary to resect the biopsy tract along with the specimen (e.g. In sarcoma surgery).

Visceral lesions may be accessed percutaneously under image guidance such as ultrasound guided biopsy of liver metastases. Or under direct vision such as a colonoscopic biopsy.

How well did you know this?
1
Not at all
2
3
4
5
Perfectly
586
Q

A 23 year old man undergoes an orchidectomy. The right testicular vein is ligated; into which structure does it drain?

Right renal vein

Inferior vena cava

Common iliac vein

Internal iliac vein

External iliac vein

A

The testicular venous drainage begins in the septa and these veins together with those of the tunica vasculosa converge on the posterior border of the testis as the pampiniform plexus. The pampiniform plexus drains to the testicular vein. The left testicular vein drains into the left renal vein. The right testicular vein drains into the inferior vena cava.

Scrotal and testicular anatomy

Spermatic cord
Formed by the vas deferens and is covered by the following structures:
Layer Origin
Internal spermatic fascia Transversalis fascia
Cremasteric fascia From the fascial coverings of internal oblique
External spermatic fascia External oblique aponeurosis

Contents of the cord
Vas deferens Transmits sperm and accessory gland secretions
Testicular artery Branch of abdominal aorta supplies testis and epididymis
Artery of vas deferens Arises from inferior vesical artery
Cremasteric artery Arises from inferior epigastric artery
Pampiniform plexus Venous plexus, drains into right or left testicular vein
Sympathetic nerve fibres Lie on arteries, the parasympathetic fibres lie on the vas
Genital branch of the genitofemoral nerve Supplies cremaster
Lymphatic vessels Drain to lumbar and para-aortic nodes

Scrotum
Composed of skin and closely attached dartos fascia.
Arterial supply from the anterior and posterior scrotal arteries
Lymphatic drainage to the inguinal lymph nodes
Parietal layer of the tunica vaginalis is the innermost layer

Testes
The testes are surrounded by the tunica vaginalis (closed peritoneal sac). The parietal layer of the tunica vaginalis adjacent to the internal spermatic fascia.
The testicular arteries arise from the aorta immediately inferiorly to the renal arteries.
The pampiniform plexus drains into the testicular veins, the left drains into the left renal vein and the right into the inferior vena cava.
Lymphatic drainage is to the para-aortic nodes.

How well did you know this?
1
Not at all
2
3
4
5
Perfectly
587
Q

Which of the tumour markers listed below is most likely to be elevated in a patient with pancreatic cancer?

CEA

CA19-9

AFP

PSA

CA15-3

A

Tumour markers

Tumour markers may be divided into:
monoclonal antibodies against carbohydrate or glycoprotein tumour antigens
tumour antigens
enzymes (alkaline phosphatase, neurone specific enolase)
hormones (e.g. calcitonin, ADH)

It should be noted that tumour markers usually have a low specificity

Monoclonal antibodies
Tumour marker Association
CA 125 Ovarian cancer
CA 19-9 Pancreatic cancer
CA 15-3 Breast cancer
NB: The breast cancer tumour marker is not specific or sensitive enough to be used routinely.

Tumour antigens
Tumour marker Association
Prostate specific antigen (PSA) Prostatic carcinoma
Alpha-feto protein (AFP) Hepatocellular carcinoma, teratoma
Carcinoembryonic antigen (CEA) Colorectal cancer

How well did you know this?
1
Not at all
2
3
4
5
Perfectly
588
Q

An 18 year old man is cutting some plants when a small piece of vegetable matter enters his eye. His eye becomes watery. Which of the following is responsible for relaying parasympathetic neuronal signals to the lacrimal apparatus?

Pterygopalatine ganglion

Otic ganglion

Submandibular ganglion

Ciliary ganglion

None of the above

A

The parasympathetic fibres to the lacrimal apparatus transit via the pterygopalatine ganglion.

Lacrimal system

Lacrimal gland
Consists of an orbital part and palpebral part. They are continuous posterolaterally around the concave lateral edge of the levator palpebrae superioris muscle.
The ducts of the lacrimal gland open into the superior fornix. Those from the orbital part penetrate the aponeurosis of levator palpebrae superioris to join those from the palpebral part. Therefore excision of the palpebral part is functionally similar to excision of the entire gland.

Blood supply
Lacrimal branch of the opthalmic artery. Venous drainage is to the superior opthalmic vein.

Innervation
The gland is innervated by the secretomotor parasympathetic fibres from the pterygopalatine ganglion which in turn may reach the gland via the zygomatic or lacrimal branches of the maxillary nerve or pass directly to the gland. The preganglionic fibres travel to the ganglion in the greater petrosal nerve (a branch of the facial nerve at the geniculate ganglion).

Nasolacrimal duct
Descends from the lacrimal sac to open anteriorly in the inferior meatus of the nose.

Lacrimation reflex
Occurs in response to conjunctival irritation (or emotional events). The conjunctiva will send signals via the opthalmic nerve. These then pass to the superior salivary centre. The efferent signals pass via the greater petrosal nerve (parasympathetic preganglionic fibres) and the deep petrosal nerve which carries the post ganglionic sympathetic fibres. The parasympathetic fibres will relay in the pterygopalatine ganglion, the sympathetic fibres do not synapse. They in turn will relay to the lacrimal apparatus.

How well did you know this?
1
Not at all
2
3
4
5
Perfectly
589
Q

Which of the following is not a typical feature of neuropraxia?

Transient delay in neuronal transmission

Axonal degeneration distal to the site of injury

Absence of neuroma formation

Preservation of autonomic function

Absence of axonal degeneration proximal to the site of injury

A

Axonal degeneration distal to the site of injury

Full recovery may occur 6-8 weeks after nerve injury in neuropraxia.
Wallerian degeneration does not usually occur in simple neuropraxia.
Autonomic function is usually preserved.

Nerve injury

There are 3 types of nerve injury:
Neuropraxia
Nerve intact but electrical conduction is affected
Full recovery
Autonomic function preserved
Wallerian degeneration does not occur
Axonotmesis
Axon is damaged and the myelin sheath is preserved. The connective tissue framework is not affected.
Wallerian degeneration occurs.
Neurotmesis
Disruption of the axon, myelin sheath and surrounding connective tissue.
Wallerian degeneration occurs.

Wallerian Degeneration
Axonal degeneration distal to the site of injury.
Typically begins 24-36 hours following injury.
Axons are excitable prior to degeneration occurring.
Myelin sheath degenerates and is phagocytosed by tissue macrophages.

Nerve repair
Neuronal repair may only occur physiologically where nerves are in direct contact. Where a large defect is present, the process of nerve regeneration is hampered. It may not occur at all or result in the formation of a neuroma. Where nerve regrowth occurs it is typically at a rate of 1mm per day.

How well did you know this?
1
Not at all
2
3
4
5
Perfectly
590
Q

A 55 year old man presents with a soft, fluctuant lesion overlying his right scapula. The surgeon suspects the lesion may be a lipoma. Which of the following, if present, may be indicative of an alternative diagnosis?

Located in superficial tissues

Size greater than 5cm

Presence of multiple similar lesions at other anatomical sites

Increased mobility of the lesion

Lobulated appearance during surgical excision

A

Lipomas are typically small and mobile lesions. They may be multiple. Lesions >5cm may be indicative of a soft tissue sarcoma and additional diagnostic tests may be required prior to excision.

Lipomata

Benign tumour of adipocytes
Occur in middle aged adults
Smooth, mobile, painless
Subtypes: Angiolipoma, Angiolipoleiomyoma
Malignant transformation is rare

Features suggestive of sarcomatous change
Size >5cm
Increasing size
Pain
Deep anatomical location
In one series the presence of all 4 features was associated with up to 85% being sarcomatous (1).

Lipoma removed surgically. Benign lesions are often small and well encapsulated. Deep seated or lesions larger than 5cm are at increased risk of being associated with sarcomatous change

How well did you know this?
1
Not at all
2
3
4
5
Perfectly
591
Q

Which of the following is not found within the deep perineal pouch in an adult male?

Pudendal nerve

Dorsal nerve of the penis

Sphincter urethrae

Urethral artery

Obturator nerve

A

Obturator nerve

Deep perineal pouch

The perineal pouch is surrounded inferiorly by the inferior fascia of the urogenital diaphragm. This fascial boundary extends laterally to form the medial wall of the ischiorectal fossa. The pouch is bounded superiorly by the superior fascia of the urogenital diaphragm and this lies beneath the levator ani muscle.

Contents of the deep perineal pouch
Urethral sphincter
Transversus perinei
Dorsal nerve of penis, muscular branches of the perineal nerve
Deep and dorsal arteries of penis, stem of origin of artery to the bulb of penis, urethral artery.

How well did you know this?
1
Not at all
2
3
4
5
Perfectly
592
Q

A 22 year old man has a long history of ulcerative colitis. His symptoms are well controlled with steroids. However, attempts at steroid weaning and use of steroid sparing drugs have repeatedly failed. He wishes to avoid a permanent stoma. Which of the following is the best operative option?

Pan proctocolectomy and end ileostomy

Abdomino perineal excision of the colon and rectum and end colostomy

Abdomino perineal excision of the colon and rectum and construction of an ileo anal pouch

Pan proctocolectomy and construction of an ileo anal pouch

Sub total colectomy and construction of an ileo anal pouch

A

Don’t confuse AP resection and proctectomy. The former is a cancer related procedure.
In patients with UC where medical management is not successful, surgical resection may offer a chance of cure. Those patients wishing to avoid a permanent stoma may be considered for an ileoanal pouch. However, this procedure is only offered in the elective setting.

Surgery for inflammatory bowel disease

Patients with inflammatory bowel disease (UC and Crohns) frequently present in surgical practice. Ulcerative colitis may be cured by surgical resection (Proctocolectomy), this is not the case in Crohns disease which may recur and affect other areas of the gastrointestinal tract.

Ulcerative colitis
Elective indications for surgery include disease that is requiring maximal therapy, or prolonged courses of steroids.
Longstanding UC is associated with a risk of malignant transformation. Dysplastic transformation of the colonic epithelium with associated mass lesions is an absolute indication for a proctocolectomy.
Emergency presentations of poorly controlled colitis that fails to respond to medical therapy should usually be managed with a sub total colectomy. Excision of the rectum is a procedure with a higher morbidity and is not generally performed in the emergency setting. An end ileostomy is usually created and the rectum either stapled off and left in situ, or, if the bowel is very oedematous, may be brought to the surface as a mucous fistula.
Patients with IBD have a high incidence of DVT and appropriate thromboprophylaxis is mandatory.
Restorative options in UC include an ileoanal pouch. This procedure can only be performed whilst the rectum is in situ and cannot usually be undertaken as a delayed procedure following proctectomy.
Ileoanal pouch complications include, anastomotic dehiscence, pouchitis and poor physiological function with seepage and soiling.

Crohns disease
Surgical resection of Crohns disease does not equate with cure, but may produce substantial symptomatic improvement.
Indications for surgery include complications such as fistulae, abscess formation and strictures.
Extensive small bowel resections may result in short bowel syndrome and localised stricturoplasty may allow preservation of intestinal length.
Staging of Crohns will usually involve colonoscopy and a small bowel study (e.g. MRI enteroclysis).
Complex perianal fistulae are best managed with long term draining seton sutures, complex attempts at fistula closure e.g. advancement flaps, may be complicated by non healing and fistula recurrence.
Severe perianal and / or rectal Crohns may require proctectomy. Ileoanal pouch reconstruction in Crohns carries a high risk of fistula formation and pouch failure and is not recommended.
Terminal ileal Crohns remains the commonest disease site and these patients may be treated with limited ileocaecal resections.
Terminal ileal Crohns may affect enterohepatic bile salt recycling and increase the risk of gallstones.

How well did you know this?
1
Not at all
2
3
4
5
Perfectly
593
Q

Which of the following is responsible for the rapid depolarisation phase of the myocardial action potential?

Rapid sodium influx

Rapid sodium efflux

Slow efflux of calcium

Efflux of potassium

Rapid calcium influx

A

Electrical activity of the heart

Myocardial action potential
Phase Description Mechanism
0 Rapid depolarisation Rapid sodium influx
These channels automatically deactivate after a few ms
1 Early repolarisation Efflux of potassium
2 Plateau Slow influx of calcium
3 Final repolarisation Efflux of potassium
4 Restoration of ionic concentrations Resting potential is restored by Na+/K+ ATPase
There is slow entry of Na+ into the cell decreasing the potential difference until the threshold potential is reached, triggering a new action potential

NB cardiac muscle remains contracted 10-15 times longer than skeletal muscle

Conduction velocity
Atrial conduction Spreads along ordinary atrial myocardial fibres at 1 m/sec
AV node conduction 0.05 m/sec
Ventricular conduction Purkinje fibres are of large diameter and achieve velocities of 2-4 m/sec (this allows a rapid and coordinated contraction of the ventricles

How well did you know this?
1
Not at all
2
3
4
5
Perfectly
594
Q

A 56 year old man is admitted with acute retention of urine. He has had a recent urinary tract infection. An USS shows bilateral hydronephrosis. What is the best course of action?

Antegrade ureteric stents

Retrograde ureteric stents

Urethral catheter

Bilateral nephrostomy

Suprapubic catheter

A

Establishing bladder drainage will often correct the situation. These patients often have a significant diuresis with associated electrolyte disturbance. The urethral route should be tried first.

Hydronephrosis

Causes of hydronephrosis

Unilateral: PACT
Pelvic-ureteric obstruction (congenital or acquired)
Aberrant renal vessels
Calculi
Tumours of renal pelvis

Bilateral: SUPER
Stenosis of the urethra
Urethral valve
Prostatic enlargement
Extensive bladder tumour
Retro-peritoneal fibrosis

Investigation
USS- identifies presence of hydronephrosis and can assess the kidneys
IVU- assess the position of the obstruction
Antegrade or retrograde pyelography- allows treatment
If renal colic suspected: non contrast CT scan (majority of stones are detected this way)

Management
Remove the obstruction and drainage of urine
Acute upper urinary tract obstruction: Nephrostomy tube
Chronic upper urinary tract obstruction: Ureteric stent or a pyeloplasty

How well did you know this?
1
Not at all
2
3
4
5
Perfectly
595
Q

A 55 year old lady has undergone a wide local excision and sentinel lymph node biopsy for breast cancer. The histology report shows a completely excised 1.3cm grade 1 invasive ductal carcinoma. The sentinel node contained no evidence of metastatic disease. The tumour is oestrogen receptor positive. What is the next course of action?

Monitor in clinic with annual review and mammography

Arrange radiotherapy

Arranged combined chemoradiotherapy

Arrange chemotherapy

Administration of GnRH analogue

A

Radiotherapy is routine following breast conserving surgery. Without irradiation the local recurrence rates are approximately 40%. These rates are potentially lower in older patients who receive endocrine therapy and who have small low grade tumours. Alongside this, an anti oestrogen will be needed.

Breast cancer treatment

Treatment Indication
Endocrine therapy
Oestrogen receptor positive tumours
Downstaging primary lesions
Definitive treatment in old, infirm patients
Irradiation
Wide local excision
Large lesion, high grade or marked vascular invasion following mastectomy
Chemotherapy
Downstaging advanced lesions to facilitate breast conserving surgery
Patients with grade 3 lesions or axillary nodal disease

Endocrine agents
Tamoxifen is used and works as a partial oestrogen receptor agonist. It will typically block activity at the breast. It does, however, stimulate the receptors at other sites and it is this that accounts for its association with endometrial cancer. In post menopausal women the process of aromatisation accounts for most oestrogen production. Therefore in this group aromatase inhibitors are the preferred agents. Women who are perimenopausal start on tamoxifen and switch at 3 years.
More recent studies (aTTom and ATLAS) have demonstrated benefits for continuing the drug for 10 years. In pre-menopausal women, there is increasing preference for the use of Exemestane over tamoxifen.

Chemotherapy
The FEC regime is most commonly used (Fluorouracil, epirubicin and cyclophosphamide). This was found to be superior to the older CMF regime. The Taxanes are commonly used in high risk patients and in this setting a regime of docetaxal, doxorubicin and cyclophosphamide may be used. The anthracycline class drugs have marked cardiotoxicity (a property that they share with trastuzumab) and this can limit their use.

How well did you know this?
1
Not at all
2
3
4
5
Perfectly
596
Q

A 34 year old man presents with symptoms attributable to a fistula in ano. He is examined in the lithotomy position and the external opening of the fistula is identified in the 7 o’clock position. At which of the following locations is the internal opening most likely to be identified?

7 o’clock

12 o’clock

9 o’clock

3 o’clock

6 o’clock

A

Goodsals rule:
Anterior fistulae will tend to have an internal opening opposite the external opening.
Posterior fistulae will tend to have a curved track that passes towards the midline.
According to Goodsalls rule the track of a posteriorly sited fistula will track to the posterior midline (i.e. 6 o’clock)

Fistulas

A fistula is defined as an abnormal connection between two epithelial surfaces.
There are many types ranging from Branchial fistulae in the neck to entero-cutaneous fistulae abdominally.
In general surgical practice the abdominal cavity generates the majority and most of these arise from diverticular disease and Crohn’s.
As a general rule all fistulae will resolve spontaneously as long as there is no distal obstruction. This is particularly true of intestinal fistulae.

The four types of fistulae are:

Enterocutaneous
These link the intestine to the skin. They may be high (>500ml) or low output (<250ml) depending upon source. Duodenal /jejunal fistulae will tend to produce high volume, electrolyte rich secretions which can lead to severe excoriation of the skin. Colo-cutaneous fistulae will tend to leak faeculent material. Both fistulae may result from the spontaneous rupture of an abscess cavity onto the skin (such as following perianal abscess drainage) or may occur as a result of iatrogenic input. In some cases it may even be surgically desirable e.g. mucous fistula following sub total colectomy for colitis.

Suspect if there is excess fluid in the drain.

Enteroenteric or Enterocolic
This is a fistula that involves the large or small intestine. They may originate in a similar manner to enterocutaneous fistulae. A particular problem with this fistula type is that bacterial overgrowth may precipitate malabsorption syndromes. This may be particularly serious in inflammatory bowel disease.

Enterovaginal
Aetiology as above.

Enterovesical
This type of fistula goes to the bladder. These fistulas may result in frequent urinary tract infections, or the passage of gas from the urethra during urination.

Management
Some rules relating to fistula management:
They will heal provided there is no underlying inflammatory bowel disease and no distal obstruction, so conservative measures may be the best option
Where there is skin involvement, protect the overlying skin, often using a well fitted stoma bag- skin damage is difficult to treat
A high output fistula may be rendered more easily managed by the use of octreotide, this will tend to reduce the volume of pancreatic secretions.
Nutritional complications are common especially with high fistula (e.g. high jejunal or duodenal) these may necessitate the use of TPN to provide nutritional support together with the concomitant use of octreotide to reduce volume and protect skin.
When managing perianal fistulae surgeons should avoid probing the fistula where acute inflammation is present, this almost always worsens outcomes.
When perianal fistulae occur secondary to Crohn’s disease the best management option is often to drain acute sepsis and maintain that drainage through the judicious use of setons whilst medical management is implemented.
Always attempt to delineate the fistula anatomy, for abscesses and fistulae that have an intra abdominal source the use of barium and CT studies should show a track. For perianal fistulae surgeons should recall Goodsall’s rule in relation to internal and external openings.

How well did you know this?
1
Not at all
2
3
4
5
Perfectly
597
Q

Which of the following structures suspends the spinal cord in the dural sheath?

Filum terminale

Conus medullaris

Ligamentum flavum

Denticulate ligaments

Anterior longitudinal ligament

A

The spinal cord is approximately 45cm in men and 43cm in women. The denticulate ligament is a continuation of the pia mater (innermost covering of the spinal cord) which has intermittent lateral projections attaching the spinal cord to the dura mater.

Spinal cord

  • Located in a canal within the vertebral column that affords it structural support.
    Rostrally it continues to the medulla oblongata of the brain and caudally it tapers at a level corresponding to the L1-2 interspace (in the adult), a central structure, the filum terminale anchors the cord to the first coccygeal vertebra.
    The spinal cord is characterised by cervico-lumbar enlargements and these, broadly speaking, are the sites which correspond to the brachial and lumbar plexuses respectively.

There are some key points to note when considering the surgical anatomy of the spinal cord:

  • During foetal growth the spinal cord becomes shorter than the spinal canal, hence the adult site of cord termination at the L1-2 level.
  • Due to growth of the vertebral column the spine segmental levels may not always correspond to bony landmarks as they do in the cervical spine.
  • The spinal cord is incompletely divided into two symmetrical halves by a dorsal median sulcus and ventral median fissure. Grey matter surrounds a central canal that is continuous rostrally with the ventricular system of the CNS.
  • The grey matter is sub divided cytoarchitecturally into Rexeds laminae.
  • Afferent fibres entering through the dorsal roots usually terminate near their point of entry but may travel for varying distances in Lissauers tract. In this way they may establish synaptic connections over several levels
  • At the tip of the dorsal horn are afferents associated with nociceptive stimuli. The ventral horn contains neurones that innervate skeletal muscle.

The key point to remember when revising CNS anatomy is to keep a clinical perspective in mind. So it is worth classifying the ways in which the spinal cord may become injured. These include:

Trauma either direct or as a result of disc protrusion
Neoplasia either by direct invasion (rare) or as a result of pathological vertebral fracture
Inflammatory diseases such as Rheumatoid disease, or OA (formation of osteophytes compressing nerve roots etc.
Vascular either as a result of stroke (rare in cord) or as complication of aortic dissection
Infection historically diseases such as TB, epidural abscesses.

The anatomy of the cord will, to an extent dictate the clinical presentation. Some points/ conditions to remember:

Brown- Sequard syndrome-Hemisection of the cord producing ipsilateral loss of proprioception and upper motor neurone signs, plus contralateral loss of pain and temperature sensation. The explanation of this is that the fibres decussate at different levels.
Lesions below L1 will tend to present with lower motor neurone signs

How well did you know this?
1
Not at all
2
3
4
5
Perfectly
598
Q

A footballer sustains a knee injury in a match and is being assessed in the outpatient department. On examination, he has a positive valgus stress test and minimal joint effusion. What is the most likely underlying injury?

Injury to the lateral collateral ligament

Injury to the medial collateral ligament

Injury to the anterior cruciate ligament

Injury to the posterior cruciate ligament

Injury to the patellar tendon

A

A knee injury in the footballer with a positive valgus stress test is usually associated with MCL injury.

Knee collateral ligament

Anatomy
The tibial collateral ligament is a broad, flat band. Its upper end has an extensive attachment to the medial epicondyle of the femur with some fibres projecting onto the adductor magnus tendon. The ligament passes downwards and forwards to the medial side of the tibia. The deepest fibres are fused with the medial meniscus.
The fibular collateral ligament is round and cord like and stands clear of the thin, lateral part of the fibrous capsule. It is enclosed within the fascia lata. It passes from the lateral epicondyle of the femur to the head of the fibula in front of its highest point and splits the tendon of biceps femoris. On the lateral side of the joint the fibres are short and weak and bridge the interval between the femoral and tibial condyles. The popliteus tendon intervenes between the lateral meniscus and the capsule.
The tibial and fibular collateral ligaments prevent disruption of the joint at the sides. They are most tightly stretched in extension, and then their direction- the fibular ligament downwards and backwards, the tibial downwards and forwards- prevents rotation of the tibia laterally or the femur medially. Rotation may be demonstrated in the flexed knee.

Injury
The collateral ligaments are commonly injured, the medial is most often affected. It requires a significant force such as sporting tackle or motor vehicle to strike the side of the leg. Associated injuries to both the tibial plateau or menisci are not uncommon.

Grading and treatment
Grade of injury Features Treatment
1 Minor tearing of ligament fibres
Negative instability tests Conservative (analgesia and physiotherapy)
2 Ligament laxity (seen with knee in 30o flexion)
Knee stable when joint extended Usually splinting or casting for 4-6 weeks
3 Ligament completely torn
Joint instability Surgical ligament reconstruction

How well did you know this?
1
Not at all
2
3
4
5
Perfectly
599
Q

Which of the following statements relating to cerebrospinal fluid is untrue?

The choroid plexus is only present in the lateral ventricles

Total CSF volume is 100-150ml

CSF pressure is usually 10-15mmHg

The cerebral aqueduct connects the third and fourth ventricle

The foramen of Luschka are paired and lie laterally in the fourth ventricle

A

The choroid plexus lies in all ventricles.

Cerebrospinal fluid

The CSF fills the space between the arachnoid mater and pia mater (covering surface of the brain). The total volume of CSF in the brain is approximately 150ml. Approximately 500 ml is produced by the ependymal cells in the choroid plexus (70%), or blood vessels (30%). It is reabsorbed via the arachnoid granulations which project into the venous sinuses.

Circulation
1. Lateral ventricles (via foramen of Munro)
2. 3rd ventricle
3. Cerebral aqueduct (aqueduct of Sylvius)
4. 4th ventricle
5. Subarachnoid space (via foramina of Magendie and Luschka)
6. Reabsorbed into the venous system via arachnoid granulations into superior sagittal sinus

Composition
Glucose: 50-80mg/dl
Protein: 15-40 mg/dl
Red blood cells: Nil
White blood cells: 0-3 cells/ mm3

How well did you know this?
1
Not at all
2
3
4
5
Perfectly
600
Q

Which of the following statements relating to large volume blood loss in trauma is incorrect?

Tranexamic acid reduces the incidence of rebleeding following surgery

Hypocalcaemia may complicate resuscitation

Colloids are preferred initially as they reduce the incidence of coagulopathy

When patients receive over 5 units of whole blood mortality increases when blood products greater than 3 weeks old are utilised

In the battlefield setting a ratio of 1:1:1 for blood, plasma and platelets is used

A

Fresh blood is the fluid of choice when large volume blood loss complicates trauma. Mortality is doubled when blood >3 weeks old is used.

Trauma management

The cornerstone of trauma management is embodied in the principles of ATLS.

Following trauma there is a trimodal death distribution:
Immediately following injury. Typically as result of brain or high spinal injuries, cardiac or great vessel damage. Salvage rate is low.
In early hours following injury. In this group deaths are due to phenomena such as splenic rupture, sub dural haematomas and haemopneumothoraces
In the days following injury. Usually due to sepsis or multi organ failure.

Aspects of trauma management
ABCDE approach.
Tension pneumothoraces will deteriorate with vigorous ventilation attempts.
External haemorrhage is managed as part of the primary survey. As a rule tourniquets should not be used. Blind application of clamps will tend to damage surrounding structures and packing is the preferred method of haemorrhage control.
Urinary catheters and naso gastric tubes may need inserting. Be wary of basal skull fractures and urethral injuries.
Patients with head and neck trauma should be assumed to have a cervical spine injury until proven otherwise.

Thoracic injuries
Simple pneumothorax
Mediastinal traversing wounds
Tracheobronchial tree injury
Haemothorax
Blunt cardiac injury
Diaphragmatic injury
Aortic disruption
Pulmonary contusion

Management of thoracic trauma
Simple pneumothorax insert chest drain. Aspiration is risky in trauma as pneumothorax may be from lung laceration and convert to tension pneumothorax.
Mediastinal traversing wounds These result from situations like stabbings. Exit and entry wounds in separate hemithoraces. The presence of a mediastinal haematoma indicates the likelihood of a great vessel injury. All patients should undergo CT angiogram and oesophageal contrast swallow. Indications for thoracotomy are largely related to blood loss and will be addressed below.
Tracheobronchial tree injury Unusual injuries. In blunt trauma most injuries occur within 4cm of the carina. Features suggesting this injury include haemoptysis and surgical emphysema. These injuries have a very large air leak and may have tension pneumothorax.
Haemothorax Usually caused by laceration of lung vessel or internal mammary artery by rib fracture. Patients should all have a wide bore 36F chest drain. Indications for thoracotomy include loss of more than 1.5L blood initially or ongoing losses of >200ml per hour for >2 hours.
Cardiac contusions Usually cardiac arrhythmias, often overlying sternal fracture. Perform echocardiography to exclude pericardial effusions and tamponade. Risk of arrhythmias falls after 24 hours.
Diaphragmatic injury Usually left sided. Direct surgical repair is performed.
Traumatic aortic disruption Commonest cause of death after RTA or falls. Usually incomplete laceration near ligamentum arteriosum. All survivors will have contained haematoma. Only 1-2% of patients with this injury will have a normal chest x-ray.
Pulmonary contusion Common and lethal. Insidious onset. Early intubation and ventilation.

Abdominal trauma
Deceleration injuries are common.
In blunt trauma requiring laparotomy the spleen is most commonly injured (40%)
Stab wounds traverse structures most commonly liver (40%)
Gunshot wounds have variable effects depending upon bullet type. Small bowel is most commonly injured (50%)
Patients with stab wounds and no peritoneal signs up to 25% will not enter the peritoneal cavity
Blood at urethral meatus suggests a urethral tear
High riding prostate on PR = urethral disruption
Mechanical testing for pelvic stability should only be performed once

Investigations in abdominal trauma

Diagnostic Peritoneal Lavage Abdominal CT scan USS
Indication Document bleeding if hypotensive Document organ injury if normotensive Document fluid if hypotensive
Advantages Early diagnosis and sensitive; 98% accurate Most specific for localising injury; 92 to 98% accurate Early diagnosis, non invasive and repeatable; 86 to 95% accurate
Disadvantages Invasive and may miss retroperitoneal and diaphragmatic injury Location of scanner away from facilities, time taken for reporting, need for contrast Operator dependent and may miss retroperitoneal injury
Amylase may be normal following pancreatic trauma
Urethrography if suspected urethral injury

How well did you know this?
1
Not at all
2
3
4
5
Perfectly
601
Q

A 28 year old woman, who is 30 weeks pregnant, presents with sudden onset chest pain associated with loss of consciousness. Her blood pressure is 170/90 mmHg, saturations on 15L oxygen 93%, heart rate 120 bpm and she is apyrexial. On examination, there is an early diastolic murmur, occasional bibasal creptitations and mild pedal oedema. An ECG shows ST elevation in leads II, III and aVF. What is the most likely diagnosis?

Pulmonary embolism

Aortic dissection

Mitral valve stenosis

Pneumonia

Pneumothorax

A

Aortic dissection is associated with the 3rd trimester of pregnancy, connective tissue disorders (Marfan’s, Ehlers- Danlos) and bicuspid valve. Patients may complain of a tearing chest pain or syncope. Clinically they may be hypertensive. The right coronary artery may become involved in the dissection, causing myocardial infarct in up to 2% cases (hence ST elevation in the inferior leads). An aortic regurgitant murmur may be auscultated.

Chest pain in pregnancy

Aortic dissection
Predisposing factors in pregnancy are hypertension, congenital heart disease and Marfan’s syndrome
Mainly Stanford type A dissections
Sudden tearing chest pain, transient syncope
Patient may be cold and clammy, hypertensive and have an aortic regurgitation murmur
Involvement of the right coronary artery may cause inferior myocardial infarction

Surgical management
Gestational timeframe Management
< 28/40 Aortic repair with the fetus kept in utero
28-32/40 Dependent on fetal condition
> 32/40 Primary Cesarean section followed by aortic repair at the same operation

Mitral stenosis
Most cases associated with rheumatic heart disease
Becoming less common in British women; suspect in women who have moved recently to the country
Commonest cardiac condition in pregnancy
Commonly associated with mortality
Valve surgery; balloon valvuloplasty preferable

Pulmonary embolism
Leading cause of mortality in pregnancy
Half dose scintigraphy; CT chest if underlying lung disease, should aid diagnosis
Treatment with low molecular weight heparin throughout pregnancy and 4-6 weeks after childbirth
Warfarin is contra indicated in pregnancy (though may be continued in women with mechanical heart valves due to the significant risk of thromboembolism)

References
1. Bates S.M. and Ginsberg J.S. How we manage venous thromboembolism during pregnancy. Blood 2002 (100): 3470-3478.

  1. Scarsbrook A.Fand Gleeson V. Investigating suspected pulmonary embolism in pregnancy. BMJ 2007 (326) : 1135 doi: 10.1136/bmj.7399.1135.
  2. Morley C. A. and Lim B. A. Lesson of the Week: The risks of delay in diagnosis of breathlessness in pregnancy. BMJ 1995 (311) : 1083.
How well did you know this?
1
Not at all
2
3
4
5
Perfectly
602
Q

Which of the following drugs is least likely to cause syndrome of inappropriate anti diuretic hormone release?

Haloperidol

Carbamazepine

Amitriptylline

Cyclophosphamide

5 Flurouracil

A

5 Flurouracil
Drugs causing SIADH: ABCD

A nalgesics: opioids, NSAIDs
B arbiturates
C yclophosphamide/ Chlorpromazine/ Carbamazepine
D iuretic (thiazides)

Hyponatraemia

This is commonly tested in the MRCS (despite most surgeons automatically seeking medical advice if this occurs!). The most common cause in surgery is the over administration of 5% dextrose.

Hyponatraemia may be caused by water excess or sodium depletion. Causes of pseudohyponatraemia include hyperlipidaemia (increase in serum volume) or a taking blood from a drip arm. Urinary sodium and osmolarity levels aid making a diagnosis.

Classification
Urinary sodium > 20 mmol/l Sodium depletion, renal loss
Patient often hypovolaemic
Diuretics (thiazides)
Addison’s
Diuretic stage of renal failure
SIADH (serum osmolality low, urine osmolality high, urine Na high)
Patient often euvolaemic
Mnemonic: Syndrome of INAPPropriate Anti-Diuretic Hormone:
In creased
Na (sodium)
PP (urine)
Urinary sodium < 20 mmol/l Sodium depletion, extra-renal loss
Diarrhoea, vomiting, sweating
Burns, adenoma of rectum (if villous lesion and large)
Water excess (patient often hypervolaemic and oedematous)
Secondary hyperaldosteronism: CCF, cirrhosis
Reduced GFR: renal failure
IV dextrose, psychogenic polydipsia

Management

Symptomatic Hyponatremia :

Acute hyponatraemia with Na <120: immediate therapy. Central Pontine Myelinolisis, may occur from overly rapid correction of serum sodium. Aim to correct until the Na is > 125 at a rate of 1 mEq/h. Normal saline with frusemide is an alternative method.

The sodium requirement can be calculated as follows :

(125 - serum sodium) x 0.6 x body weight = required mEq of sodium

How well did you know this?
1
Not at all
2
3
4
5
Perfectly
603
Q

A 39 year old man has suffered from terminal ileal Crohns disease for the past 20 years. Which condition is he least likely to develop?

Gallstones

Malabsorption

Pyoderma gangrenosum

Amyloidosis

Feltys syndrome

A

Feltys syndrome:
Rheumatoid disease
Splenomegaly
Neutropenia
Feltys syndrome is associated with rheumatoid disease. Individuals with long standing Crohns disease are at risk of gallstones because of impairment of the enterohepatic recycling of bile salts. Formation of entero-enteric fistulation may produce malabsorption. Amyloidosis may complicate chronic inflammatory states.

Crohns disease

Crohns disease is a chronic transmural inflammation of a segment(s) of the gastrointestinal tract and may be associated with extra intestinal manifestations. Frequent disease patterns observed include ileal, ileocolic and colonic disease. Peri-anal disease may occur in association with any of these. The disease is often discontinuous in its distribution. Inflammation may cause ulceration, fissures, fistulas and fibrosis with stricturing. Histology reveals a chronic inflammatory infiltrate that is usually patchy and transmural.

Ulcerative colitis Vs Crohns
Crohn’s disease Ulcerative colitis
Distribution Mouth to anus Rectum and colon
Macroscopic changes Cobblestone appearance, apthoid ulceration Contact bleeding
Depth of disease Transmural inflammation Superficial inflammation
Distribution pattern Patchy Continuous
Histological features Granulomas (non caseating epithelioid cell aggregates with Langhans’ giant cells) Crypt abscesses, Inflammatory cells in the lamina propria

Extraintestinal manifestations of Crohns
Related to disease extent Unrelated to disease extent
Aphthous ulcers (10%) Sacroiliiitis (10-15%)
Erythema nodosum (5-10%) Ankylosing spondylitis (1-2%)
Pyoderma gangrenosum (0.5%) Primary sclerosing cholangitis (Rare)
Acute arthropathy (6-12%) Gallstones (up to 30%)
Ocular complications (up to 10%) Renal calculi (up to 10%)

Diarrhoea in Crohns
Diarrhoea in Crohns may be multifactorial since actual inflammation of the colon is not common. Causes therefore include the following:
Bile salt diarrhoea secondary to terminal ileal disease
Entero-colic fistula
Short bowel due to multiple resections
Bacterial overgrowth

Surgical interventions in Crohns disease
The commonest disease pattern in Crohns is stricturing terminal ileal disease and this often culminates in an ileocaecal resection. Other procedures performed include segmental small bowel resections and stricturoplasty. Colonic involvement in patients with Crohns is not common and, where found, distribution is often segmental. However, despite this distribution segmental resections of the colon in patients with Crohns disease are generally not advocated because the recurrence rate in the remaining colon is extremely high. As a result, the standard options of colonic surgery in Crohns patients are generally; sub total colectomy, panproctocolectomy and staged sub total colectomy and proctectomy. Restorative procedures such as ileoanal pouch have no role in therapy.
Crohns disease is notorious for the developmental of intestinal fistulae; these may form between the rectum and skin (peri anal) or the small bowel and skin. Fistulation between loops of bowel may also occur and result in bacterial overgrowth and malabsorption. Management of enterocutaneous fistulae involves controlling sepsis, optimising nutrition, imaging the disease and planning definitive surgical management.

How well did you know this?
1
Not at all
2
3
4
5
Perfectly
604
Q

A 34 year old man presents with varicose veins and it is suspected that these are part of the Klippel-Trenaunay syndrome. Which of the following is not a characteristic of this condition?

Presence of varicose veins

Gigantism of a limb

Long saphenous vein involvement

Port wine stains with clear borders

Low flow rates through capillary anomalies

A

The Klippel-Trenaunay vein is a large, lateral, superficial vein sometimes seen at birth. This vein begins in the foot or the lower leg and travels proximally until it enters the thigh or the gluteal area. Otherwise, varicosities may not be clinically evident until the child begins to ambulate.
Varicosities may be extensive, though they often spare the saphenous distribution. They are seen below the knee, laterally above the knee, and occasionally in the pelvic region. Varicosities may affect the superficial, deep, and perforating venous systems.
Flow rates through the vascular anomalies are low flow rates in contrast to Parkes Weber syndrome where high flow rates are seen.
Surgical exploration has demonstrated atresia and agenesis of deep veins, compression due to fibrous bands, aberrant arteries, abnormal muscles, or venous sheaths.
Rarely, varicosities have been found in the bladder, the colon, and the pulmonary vessels

Klippel-Trenaunay syndrome is a rare condition that affects the development of blood vessels, soft tissues, and bones. The disorder has three characteristic features: a red birthmark called a port-wine stain, abnormal overgrowth of soft tissues and bones, and vein malformations12
The cause of Klippel-Trenaunay syndrome is not well understood, but it is not directly inherited from parents. It may be related to problems with how the blood vessel system develops during pregnancy23
The symptoms of Klippel-Trenaunay syndrome vary from person to person, but they usually affect only one limb, most often the leg. The port-wine stain is often the first sign of the condition, and it may cover a large area of the skin. The affected limb may also be longer and wider than the other, and have varicose veins or lymphatic malformations12
There is no cure for Klippel-Trenaunay syndrome, but treatments can help manage the symptoms and prevent complications. These may include compression garments, laser therapy, surgery, or medication24

How well did you know this?
1
Not at all
2
3
4
5
Perfectly
605
Q

A 6 month old boy is brought to the clinic with difficulty breathing and weight loss. On examination, he is found to have a firm para testicular mass. What is the most likely diagnosis?

Teratoma

Seminoma

Adenocarcinoma

Rhabdomyosarcoma

Malignant fibrous histiocytoma

A

Rhabdomyosarcomas as one of the more common malignant solid tumours in children (though all are rare). They have an aggressive behavior pattern and metastases are common. Teratomas are nearly always benign in younger children. Seminomas are very rare indeed. Malignant fibrous histiocytomas are almost never found in this location.

Sarcomas

Malignant tumours of mesenchymal origin

Types
May be either bone or soft tissue in origin.
Bone sarcoma include:
Osteosarcoma
Ewings sarcoma (although non bony sites recognised)
Chondrosarcoma - originate from Chondrocytes

Soft tissue sarcoma are a far more heterogeneous group and include:
Liposarcoma-adipocytes
Rhabdomyosarcoma-striated muscle
Leiomyosarcoma-smooth muscle
Synovial sarcomas- close to joints (cell of origin not known but not synovium)

Malignant fibrous histiocytoma is now referred to as undifferentiated pleomorphic sarcoma. Careful histological assessment of lesions now allows more accurate categorisation of sarcoma subtypes than was previously possible.

Features
Certain features of a mass or swelling should raise suspicion for a sarcoma these include:
Large >5cm soft tissue mass
Deep tissue location or intra muscular location
Rapid growth
Painful lump

Assessment
Imaging of suspicious masses should utilise a combination of MRI, CT and USS. Blind biopsy should not be performed prior to imaging and where required should be done in such a way that the biopsy tract can be subsequently included in any resection.

Ewings sarcoma
Commoner in males
Incidence of 0.3 / 1, 000, 000
Onset typically between 10 and 20 years of age
Location by femoral diaphysis is commonest site
Histologically it is a small round tumour
Blood borne metastasis is common and chemotherapy is often combined with surgery

Osteosarcoma
Mesenchymal cells with osteoblastic differentiation
20% of all primary bone tumours
Incidence of 5 per 1,000,000
Peak age 15-30, commoner in males
Limb preserving surgery may be possible and many patients will receive chemotherapy

Liposarcoma
Malignancy of adipocytes
Rare, approximately 2.5 per 1,000,000. They are the most common soft tissue sarcoma
Typically located in deep locations such as retroperitoneum
Affect older age group usually >40 years of age
May be well differentiated and thus slow growing although may undergo de-differentiation and disease progression
Many tumours will have a pseudocapsule that can misleadingly allow surgeons to feel that they can ‘shell out’ these lesions. In reality, tumour may invade at the edge of the pseudocapsule and result in local recurrence if this strategy is adopted
Usually resistant to radiotherapy, although this is often used in a palliative setting

Malignant Fibrous Histiocytoma
Tumour with large number of histiocytes
Also described as undifferentiated pleomorphic sarcoma NOS (i.e. Cell of origin is not known)
Four major subtypes are recognised: storiform-pleomorphic (70% cases), myxoid (less aggressive), giant cell and inflammatory
Treatment is usually with surgical resection and adjuvant radiotherapy as this reduces the likelihood of local recurrence

How well did you know this?
1
Not at all
2
3
4
5
Perfectly
606
Q

An ENT surgeon is undertaking a nasendoscopy to evaluate symptoms of recurrent sinusitis. As she performs the procedure, she notices that there is lesion within the sphenoethmoidal recess. Of the structures outlined below, which is most likely to be compromised as a result?

Sphenoidal sinus

Fontal sinus

Maxillary sinus

Anterior ethmoidal sinus

Posterior ethmoidal sinus

A

The sphenoidal sinus drains into the sphenoethmoidal recess and is likely to be affected. The other sinuses drain into the superior, middle and inferior meatus.

Nasal cavity

There are two nasal cavities separated by the median nasal septum. This structure consists of septal cartilage and two bony parts: the vomer and the perpendicular plate of the ethmoid bone. Each cavity is approximately 5cm high and 7cm long, from the nares anteriorly to the choanae posteriorly. The lateral walls of each cavity are mainly made up of the maxilla. However, the lacrimal, ethmoid and palatine bones also contribute. The ethmoid gives rise to the two upper protrusions from the lateral walls, the superior and middle conchae. The inferior concha is a separate bone. Above the superior concha lies the sphenoethmoidal recess. The spaces inferior to conchae are the superior, middle and inferior meatus. The structures draining into each space are shown below:

Sphenoethmoidal recess Sphenoidal sinus
Superior meatus Posterior ethmoidal sinus
Middle meatus Frontal sinus, maxillary sinus ,anterior and middle ethmoidal sinus
Inferior meatus Nasolacrimal duct

The main arterial supply to the septum and lateral walls is the sphenopalatine artery, a branch of the maxillary artery. The olfactory nerve conveys special sensation. Common sensation from the nasal mucosa is conveyed by the trigeminal nerve.

How well did you know this?
1
Not at all
2
3
4
5
Perfectly
607
Q

Which of the following is not contained within the deep posterior compartment of the lower leg?

Tibialis posterior muscle

Posterior tibial artery

Tibial nerve

Sural nerve

Flexor hallucis longus

A

The deep posterior compartment lies anterior to soleus. The sural nerve is superficially sited and therefore not contained within it. Lower limb- Muscular compartments

Anterior compartment
Muscle Nerve Action
Tibialis anterior Deep peroneal nerve Dorsiflexes ankle joint, inverts foot
Extensor digitorum longus Deep peroneal nerve Extends lateral four toes, dorsiflexes ankle joint
Peroneus tertius Deep peroneal nerve Dorsiflexes ankle, everts foot
Extensor hallucis longus Deep peroneal nerve Dorsiflexes ankle joint, extends big toe

Peroneal compartment
Muscle Nerve Action
Peroneus longus Superficial peroneal nerve Everts foot, assists in plantar flexion
Peroneus brevis Superficial peroneal nerve Plantar flexes the ankle joint

Superficial posterior compartment
Nerve Action
Gastrocnemius Tibial nerve Plantar flexes the foot, may also flex the knee
Soleus Tibial nerve Plantar flexor

Deep posterior compartment
Muscle Nerve Action
Flexor digitorum longus Tibial Flexes the lateral four toes
Flexor hallucis longus Tibial Flexes the great toe
Tibialis posterior Tibial Plantar flexor, inverts the foot

How well did you know this?
1
Not at all
2
3
4
5
Perfectly
608
Q

A 52 year old man develops septic shock following a Hartmans procedure for perforated diverticular disease. He is started on an adrenaline infusion. Which of the following is least likely to occur?

Peripheral vasoconstriction

Coronary artery vasospasm

Gluconeogenesis

Lipolysis

Tachycardia

A

It’s cardiac effects are mediated via β 1 receptors. The coronary arteries which have β 2 receptors are unaffected.

Adrenaline

Fight or Flight response
- Catecholamine (phenylalanine and tyrosine)
- Neurotransmitter and hormone
- Released by the adrenal glands
- Effects on α 1 and 2, β 1 and 2 receptors
- Effect on β 2 receptors in skeletal muscle vessels-causing vasodilation
- Increase cardiac output and total peripheral resistance
- Vasoconstriction in the skin and kidneys causing a narrow pulse pressure

Actions
α adrenergic receptors:
Inhibits insulin secretion by the pancreas
Stimulates glycogenolysis in the liver and muscle
Stimulates glycolysis in muscle

β adrenergic receptors:
Stimulates glucagon secretion in the pancreas
Stimulates ACTH
Stimulates lipolysis by adipose tissue

How well did you know this?
1
Not at all
2
3
4
5
Perfectly
609
Q

A 32 year old man is diagnosed as having a carcinoma of the caecum. On questioning, his mother developed uterine cancer at the age of 39 and his maternal uncle died from colonic cancer aged 38. His older brother developed a colonic cancer with micro satellite instability aged 37. What is the most appropriate operative treatment?

Limited ileocaecal resection

Right hemicolectomy

Extended right hemicolectomy

Panproctocolectomy

Sub total colectomy

A

The likely diagnosis is one of a familial cancer syndrome and now that he has developed a colonic cancer the safest operative strategy is a total colectomy and end ileostomy.

Polyposis syndromes

Syndrome Genetic defect Features Screening and management Associated disorders
Familial adenomatous polyposis Mutation of APC gene (80%) cases, dominant Typically over 100 colonic adenomas
Cancer risk of 100%
20% are new mutations If known to be at risk then predictive genetic testing as teenager
Annual flexible sigmoidoscopy from 15 years
If no polyps found then 5 yearly colonoscopy started at age 20
Polyps found = resectional surgery (resection and pouch Vs sub total colectomy and IRA) Gastric fundal polyps (50%).
Duodenal polyps 90%.
If severe duodenal polyposis cancer risk of 30% at 10 years.
Abdominal desmoid tumours.
MYH associated polyposis Biallelic mutation of mut Y human homologue (MYH) on chromosome 1p, recessive Multiple colonic polyps
Later onset right sided cancers more common than in FAP
100% cancer risk by age 60 Once identified resection and ileoanal pouch reconstruction is recommended
Attenuated phenotype - regular colonoscopy Duodenal polyposis in 30%
Associated with increased risk of breast cancer (self examination)
Peutz -Jeghers syndrome STK11 (LKB1) mutation on chromosome 19 in some (but not all) cases, dominant Multiple benign intestinal hamartomas
Episodic obstruction and intussceception
Increased risk of GI cancers (colorectal cancer 20%, gastric 5%)
Increased risk of breast, ovarian, cervical pancreatic and testicular cancers Annual examination
Pan intestinal endoscopy every 2-3 years Malignancies at other sites
Classical pigmentation pattern
Cowden disease Mutation of PTEN gene on chromosome 10q22, dominant Macrocephaly
Multiple intestinal hamartomas
Multiple trichilemmomas
89% risk of cancer at any site
16% risk of colorectal cancer Targeted individualised screening Breast cancer (81% risk)
Thyroid cancer and non toxic goitre
Uterine cancer
HNPCC (Lynch syndrome) Germline mutations of DNA mismatch repair genes Colo rectal cancer 30-70%
Endometrial cancer 30-70%
Gastric cancer 5-10%
Scanty colonic polyps may be present
Colonic tumours likely to be right sided and mucinous Colonoscopy every 1-2 years from age 25
Consideration of prophylactic surgery
Extra colonic surveillance recommended Extra colonic cancers

How well did you know this?
1
Not at all
2
3
4
5
Perfectly
610
Q

A 70 year old man undergoes an oesophagectomy and an oesophagogastric anastomosis is constructed. The arterial supply to the gastric component is mainly provided by which of these vessels?

Short gastric arteries

Left gastric artery

Right gastroepiploic artery

Left gastroepiploic artery

Common hepatic artery

A

The resection of the proximal stomach which is performed in these cases together with the gastric mobilisation required results in the division of all vessels except the right gastro-epiploic artery which then supplies the gastric remnant. It is this precarious supply that can result in conduit necrosis.

Stomach anatomy

The abdominal oesophagus enters the stomach through the cardiac orifice, it tends to lie around the level of T11 although this can vary considerably. The pyloric orifice tends to lie on a level of L1 (the transpyloric plane) although this, too, can vary.

Structurally, the stomach comprises fundic, body and pyloric regions. The body is comprises most of the gastric surface area. The anterior and posterior surfaces of the stomach unite at the greater and lesser curvatures. The peritoneum on the anterior and posterior surfaces of the stomach unites at the greater curvature to form the gastrosplenic ligament above and the anterior layer of the greater omentum below.

There are three muscular layers to the stomach wall; circular muscle fibres, longitudinal fibres and oblique fibres.

Blood supply
Left gastric artery
Short gastric vessels
Gastro-epiploic arteries

The veins draining the stomach accompany the arteries. The right and left gastric veins drain directly into the portal vein. The right gastro-epiploic vein either drains into the SMV or middle colic vein. The left gastro-epiploic and short gastric veins drain into the splenic vein.

Innervation
Sympathetic nerves arise from the coeliac ganglia with post ganglionic fibres accompanying the arteries.
Parasympathetic innervation is from the anterior and posterior vagal trunks

How well did you know this?
1
Not at all
2
3
4
5
Perfectly
611
Q

Which of the following is not linked to excess glucocorticoids?

Osteonecrosis

Osteoporosis

Hypokalaemia

Hyponatraemia

Growth retardation in children

A

D- Hyponatraemia

There are many adverse effects associated with excess glucocorticoids. Thinning of the skin, osteonecrosis and osteoporosis are all common. Steroids are associated with retention of sodium and water. Potassium loss may occur and hypokalaemic alkalosis has been reported.

Cortisol

Glucocorticoid
Released by zona fasiculata of the adrenal gland
90% protein bound; 10% active
Circadian rhythm: High in the mornings
Negative feedback via ACTH

Actions
Glycogenolysis
Gluconeogenesis
Protein catabolism
Lipolysis
Stress response
Anti-inflammatory
Decrease protein in bones
Increase gastric acid
Increases neutrophils/platelets/red blood cells
Inhibits fibroblastic activity

How well did you know this?
1
Not at all
2
3
4
5
Perfectly
612
Q

A 19 year old student is involved in a head on car collision. He complains of severe chest pain. A Chest x-ray performed as part of a trauma series shows widening of the mediastinum. Which is the most likely injury in this scenario?

Rupture of the distal oesophagus

Rupture of the left main bronchus

Rupture of the aorta proximal to the left subclavian artery

Rupture of the aorta distal to the left subclavian artery

Rupture of the inferior vena cava

A

The aorta may be injured in deceleration accidents. In the setting of deceleration injury, chest pain and mediastinal widening the most likely problem is aortic rupture. This will typically occur distal to the left subclavian artery. Rupture of the proximal aorta may occur. However, survival is unlikely. It is important to note that the question uses the term Most likely injury as this is the component that distinguishes an ascending rupture from a descending rupture.

Thoracic aorta rupture

Mechanism of injury: Decelerating force i.e. RTA, fall from a great height
Most people die at scene
Survivors may have an incomplete laceration at the ligamentum arteriosum of the aorta.

Clinical features
Contained haematoma: persistent hypotension
Detected mainly by history, CXR changes

CXR changes
Widened mediastinum
Trachea/Oesophagus to right
Depression of left main stem bronchus
Widened paratracheal stripe/paraspinal interfaces
Space between aorta and pulmonary artery obliterated
Rib fracture/left haemothorax

Diagnosis
Angiography, usually CT aortogram.

Treatment
Repair or replacement. Ideally they should undergo endovascular repair.

How well did you know this?
1
Not at all
2
3
4
5
Perfectly
613
Q

A 40 year old professional singer is admitted for a right thyroid lobectomy. Post operatively, she is unable to sing high notes. Which of the following muscles is likely to demonstrate impaired function?

Thyroarytenoid

Posterior cricoarytenoid

Cricothyroid

Thyrohyoid

Lateral cricoarytenoid

A

The most likely injury is to the superior laryngeal nerve which innervates the cricothyroid muscle. Since this tenses the vocal cords, singing high notes becomes a problem.

Voice production

There are 2 main nerves involved:

Superior laryngeal nerve (SLN)
Innervates the cricothyroid muscle

Since the cricothyroid muscle is involved in adjusting the tension of the vocal fold for high notes during singing, SLN paresis and paralysis result in:

a. Abnormalities in pitch
b. Inability to sing with smooth change to each higher note (glissando or pitch glide)

Recurrent laryngeal nerve (RLN)/Inferior laryngeal nerve
Innervates intrinsic larynx muscles

a. Opening vocal folds (as in breathing, coughing)
b. Closing vocal folds for vocal fold vibration during voice use
c. Closing vocal folds during swallowing

How well did you know this?
1
Not at all
2
3
4
5
Perfectly
614
Q

Which of these structures are linked by the ductus arteriosus?

Iliac vessels and inferior vena cava

Umbilical vein and inferior vena cava

Iliac arteries and umbilical arteries

Pulmonary artery and aorta

Pulmonary vein and inferior vena cava

A

The ductus arteriosus connects the pulmonary artery to the first part of the descending aorta. The ductus venosus connects the umbilical vein to the inferior vena cava and bypasses the liver. The volume transferred by this route decreases with gestation. Persistent patent ductus arteriosus is treated with NSAIDS in most cases.

Ductus arteriosus

The ductus arteriosus is formed from the left 6th aortic arch during embryonic development and attaches to the final part of the aortic arch (the isthmus of aorta) and the first part of the pulmonary artery.
It serves to allow blood in the developing foetus to bypass the lungs. It usually closes soon after birth. Should it not do so, it can be stimulated to close by administration of indomethacin.

How well did you know this?
1
Not at all
2
3
4
5
Perfectly
615
Q

Which of the following types of growth plate fractures may have similar radiological appearances?

Salter Harris types 1 and 5

Salter Harris types 4 and 5

Salter Harris types 3 and 5

Salter Harris types 1 and 2

Salter Harris types 1 and 3

A

Mnemonic: SALTER

S (Type 1): Straight through the growth plate
A (Type 2): Above - through growth plate and Above involving the metaphysis
L (Type 3): Lower -through growth plate and beLow involving the epiphysis
T (Type 4):Through - Through both metaphysis, epiphysis and growth plate
E (Type 5): Everything - Crush / compression injury
R (Type 5): Ruined
As recommended by one of our users
Salter Harris injury types 1 and 5 (transverse fracture through growth plate Vs. Compression fracture) may mimic each other radiologically. Type 5 injuries have the worst outcomes. Radiological signs of type 5 injuries are subtle and may include narrowing of the growth plate.

Epiphyseal fractures

Fractures involving the growth plate in children are classified using the Salter - Harris system.
There are 5 main types.

Salter Harris Classification
Type Description
Type 1 Transverse fracture through the growth plate
Type 2 Fracture through the growth plate to the metaphysis (commonest type)
Type 3 Fracture through the growth plate and the epiphysis with metaphysis spared
Type 4 Fracture involving the growth plate, metaphysis and epiphysis
Type 5 Compression fracture of the growth plate (worst outcome)

Management
Non displaced type 1 injuries can generally be managed conservatively. Unstable or more extensive injuries will usually require surgical reduction and/ or fixation, as proper alignment is crucial.

How well did you know this?
1
Not at all
2
3
4
5
Perfectly
616
Q

A 59 year old man is diagnosed as having carcinoma of the pancreas with two malignant deposits in the right lobe of the liver. What is the most appropriate treatment?

Palliative chemotherapy

Liver resection followed by chemotherapy

Simultaneous resection of liver metastasis and en bloc segmental pancreatic resection

Pancreatic resection followed by liver resection once recovered

Radical radiotherapy followed by surgery

A

Pancreatic cancer has a poor prognosis and most cases have metastatic disease at presentation. There is no role in pancreatic cancer for liver resection together with pancreatic surgery as there is no survival benefit. Most centres will offer palliative chemotherapy which has improved both longevity and quality of life.

Pancreatic cancer

  • Adenocarcinoma
    Risk factors: Smoking, diabetes, adenoma, familial adenomatous polyposis
    Mainly occur in the head of the pancreas (70%)
    Spread locally and metastasizes to the liver
    Carcinoma of the pancreas should be differentiated from other periampullary tumours with better prognosis

Clinical features
Weight loss
Painless jaundice
Epigastric discomfort (pain usually due to invasion of the coeliac plexus is a late feature)
Pancreatitis
Trousseau’s sign: migratory superficial thrombophlebitis

Investigations
USS: May miss small lesions
CT Scanning (pancreatic protocol). If unresectable on CT then no further staging needed
PET/CT for those with operable disease on CT alone
ERCP/ MRI for bile duct assessment
Staging laparoscopy to exclude peritoneal disease

Management
Head of pancreas: Whipple’s resection (SE dumping and ulcers). Newer techniques include pylorus preservation and SMA/ SMV resection
Carcinoma body and tail: poor prognosis, distal pancreatectomy, if operable
Usually adjuvent chemotherapy for resectable disease
ERCP and stent for jaundice and palliation
Surgical bypass may be needed for duodenal obstruction

How well did you know this?
1
Not at all
2
3
4
5
Perfectly
617
Q

A 35-year-old female is admitted to hospital with hypovolaemic shock. CT abdomen reveals a haemorrhagic lesion in the right kidney. Following surgery and biopsy this is shown to be an angiomyolipomata. What is the most likely underlying diagnosis?

Neurofibromatosis

Budd-Chiari syndrome

Hereditary haemorrhagic telangiectasia

Von Hippel-Lindau syndrome

Tuberous sclerosis

A

Tuberous sclerosis

Tuberous sclerosis (TS) is a genetic condition of autosomal dominant inheritance. Like neurofibromatosis, the majority of features seen in TS are neuro-cutaneous

Cutaneous features
depigmented ‘ash-leaf’ spots which fluoresce under UV light
roughened patches of skin over lumbar spine (Shagreen patches)
adenoma sebaceum: butterfly distribution over nose
fibromata beneath nails (subungual fibromata)
café-au-lait spots* may be seen

Neurological features
developmental delay
epilepsy (infantile spasms or partial)
intellectual impairment

Also
retinal hamartomas: dense white areas on retina (phakomata)
rhabdomyomas of the heart
gliomatous changes can occur in the brain lesions
polycystic kidneys, renal angiomyolipomata

*these of course are more commonly associated with neurofibromatosis. However a 1998 study of 106 children with TS found café-au-lait spots in 28% of patients

How well did you know this?
1
Not at all
2
3
4
5
Perfectly
618
Q

Which of the following is the equivalent of cardiac preload?

End diastolic volume

Stroke volume

Systemic vascular resistance

Mean arterial pressure

Peak systolic arterial pressure

A

Preload is the same as end diastolic volume. When it is increased slightly there is an associated increase in cardiac output (Frank Starling principle). When it is markedly increased e.g. over 250ml then cardiac output falls.
Cardiac physiology

  • The heart has four chambers ejecting blood into both low pressure and high pressure systems.
    The pumps generate pressures of between 0-25mmHg on the right side and 0-120 mmHg on the left.
    At rest diastole comprises 2/3 of the cardiac cycle.
    The product of the frequency of heart rate and stroke volume combine to give the cardiac output which is typically 5-6L per minute.

Detailed descriptions of the various waveforms are often not a feature of MRCS A (although they are on the syllabus). However, they are a very popular topic for surgical physiology in the MRCS B exam.

Electrical properties
Intrinsic myogenic rhythm within cardiac myocytes means that even the denervated heart is capable of contraction.
In the normal situation the cardiac impulse is generated in the sino atrial node in the right atrium and conveyed to the ventricles via the atrioventricular node.
The sino atrial node is also capable of spontaneous discharge and in the absence of background vagal tone will typically discharge around 100x per minute. Hence the higher resting heart rate found in cardiac transplant cases. In the SA and AV nodes the resting membrane potential is lower than in surrounding cardiac cells and will slowly depolarise from -70mV to around -50mV at which point an action potential is generated.
Differences in the depolarisation slopes between SA and AV nodes help to explain why the SA node will depolarise first. The cells have a refractory period during which they cannot be re-stimulated and this period allows for adequate ventricular filling. In pathological tachycardic states this time period is overridden and inadequate ventricular filling may then occur, cardiac output falls and syncope may ensue.

Parasympathetic fibres project to the heart via the vagus and will release acetylcholine. Sympathetic fibres release nor adrenaline and circulating adrenaline comes from the adrenal medulla. Noradrenaline binds to β 1 receptors in the SA node and increases the rate of pacemaker potential depolarisation.

Mid diastole: AV valves open. Ventricles hold 80% of final volume. Outflow valves shut. Aortic pressure is high.

Late diastole: Atria contract. Ventricles receive 20% to complete filling. Typical end diastolic volume 130-160ml.

Early systole: AV valves shut. Ventricular pressure rises. Isovolumetric ventricular contraction. AV Valves bulge into atria (c-wave). Aortic and pulmonary pressure exceeded- blood is ejected. Shortening of ventricles pulls atria downwards and drops intra atrial pressure (x-descent).

Late systole: Ventricular muscles relax and ventricular pressures drop. Although ventricular pressure drops the aortic pressure remains constant owing to peripheral vascular resistance and elastic property of the aorta. Brief period of retrograde flow that occurs in aortic recoil shuts the aortic valve. Ventricles will contain 60ml end systolic volume. The average stroke volume is 70ml (i.e. Volume ejected).

Early diastole: All valves are closed. Isovolumetric ventricular relaxation occurs. Pressure wave associated with closure of the aortic valve increases aortic pressure. The pressure dip before this rise can be seen on arterial waveforms and is called the incisura. During systole the atrial pressure increases such that it is now above zero (v- wave). Eventually atrial pressure exceed ventricular pressure and AV valves open - atria empty passively into ventricles and atrial pressure falls (y -descent )

The negative atrial pressures are of clinical importance as they can allow air embolization to occur if the neck veins are exposed to air. This patient positioning is important in head and neck surgery to avoid this occurrence if veins are inadvertently cut, or during CVP line insertion.

Mechanical properties
Preload = end diastolic volume
Afterload = aortic pressure

It is important to understand the principles of Laplace’s law in surgery.
It states that for hollow organs with a circular cross section, the total circumferential wall tension depends upon the circumference of the wall, multiplied by the thickness of the wall and on the wall tension.
The total luminal pressure depends upon the cross sectional area of the lumen and the transmural pressure. Transmural pressure is the internal pressure minus external pressure and at equilibrium the total pressure must counterbalance each other.
In terms of cardiac physiology the law explains that the rise in ventricular pressure that occurs during the ejection phase is due to physical change in heart size. It also explains why a dilated diseased heart will have impaired systolic function.

Starlings law
Increase in end diastolic volume will produce larger stroke volume.
This occurs up to a point beyond which cardiac fibres are excessively stretched and stroke volume will fall once more. It is important for the regulation of cardiac output in cardiac transplant patients who need to increase their cardiac output.

Baroreceptor reflexes
Baroreceptors located in aortic arch and carotid sinus.
Aortic baroreceptor impulses travel via the vagus and from the carotid via the glossopharyngeal nerve.
They are stimulated by arterial stretch.
Even at normal blood pressures they are tonically active.
Increase in baroreceptor discharge causes:

*Increased parasympathetic discharge to the SA node.
*Decreased sympathetic discharge to ventricular muscle causing decreased contractility and fall in stroke volume.
*Decreased sympathetic discharge to venous system causing increased compliance.
*Decreased peripheral arterial vascular resistance

Atrial stretch receptors
Located in atria at junction between pulmonary veins and vena cava.
Stimulated by atrial stretch and are thus low pressure sensors.
Increased blood volume will cause increased parasympathetic activity.
Very rapid infusion of blood will result in increase in heart rate mediated via atrial receptors: the Bainbridge reflex.
Decreases in receptor stimulation results in increased sympathetic activity this will decrease renal blood flow-decreases GFR-decreases urinary sodium excretion-renin secretion by juxtaglomerular apparatus-Increase in angiotensin II.
Increased atrial stretch will also result in increased release of atrial natriuretic peptide.

How well did you know this?
1
Not at all
2
3
4
5
Perfectly
619
Q

Which of the following statements relating to the basilar artery and its branches is false?

The superior cerebellar artery may be decompressed to treat trigeminal neuralgia

Occlusion of the posterior cerebral artery causes contralateral loss of the visual field

The oculomotor nerve lies between the superior cerebellar and posterior cerebral arteries

The posterior inferior cerebellar artery is the largest of the cerebellar arteries arising from the basilar artery

The labyrinthine branch is accompanied by the facial nerve

A

The posterior inferior cerebellar artery is the largest of the cerebellar arteries arising from the vertebral artery. The labyrinthine artery is long and slender and may arise from the lower part of the basilar artery. It accompanies the facial and vestibulocochlear nerves into the internal auditory meatus. The posterior cerebral artery is often larger than the superior cerebellar artery and it is separated from the vessel, near it’s origin, by the oculomotor nerve. Arterial decompression is a well established therapy for trigeminal neuralgia.
Circle of Willis

The two internal carotid arteries and two vertebral arteries form an anastomosis known as the Circle of Willis on the inferior surface of the brain. Each half of the circle is formed by:
1. Anterior communicating artery
2. Anterior cerebral artery
3. Internal carotid artery
4. Posterior communicating artery
5. Posterior cerebral arteries and the termination of the basilar artery

The circle and its branches supply; the corpus striatum, internal capsule, diencephalon and midbrain.

Vertebral arteries
Enter the cranial cavity via foramen magnum
Lie in the subarachnoid space
Ascend on anterior surface of medulla oblongata
Unite to form the basilar artery at the base of the pons

Branches:
Posterior spinal artery
Anterior spinal artery
Posterior inferior cerebellar artery

Basilar artery
Branches:
Anterior inferior cerebellar artery
Labyrinthine artery
Pontine arteries
Superior cerebellar artery
Posterior cerebral artery (at the point where it bifurcates)

Internal carotid arteries
Branches:
Posterior communicating artery
Anterior cerebral artery
Middle cerebral artery
Anterior choroid artery

How well did you know this?
1
Not at all
2
3
4
5
Perfectly
620
Q

A patient has an appendicectomy and a 1.2cm carcinoid tumour is identified in the tip of the appendix. What is the most appropriate management?

Watchful waiting

Discharge

Right hemicolectomy

Limited ileocaecal resection

Radioisotope scan

A

Individuals with small carcinoids can be discharged (<2cm and limited to the appendix). Larger tumours should have a radioisotope scan. Where the resection margin is positive or where the isotope scan suggests lymphatic metastasis a right hemicolectomy should be performed.

Carcinoid syndrome

Carcinoid tumours secrete serotonin
Originate in neuroendocrine cells mainly in the intestine (midgut-distal ileum/appendix)
Can occur in the rectum, bronchi
Hormonal symptoms mainly occur when disease spreads outside the bowel

Clinical features
Onset: insidious over many years
Flushing face
Palpitations
Pulmonary valve stenosis and tricuspid regurgitation causing dyspnoea
Asthma
Severe diarrhoea (secretory, persists despite fasting)

Investigation
5-HIAA in a 24-hour urine collection
Somatostatin receptor scintigraphy
CT scan
Blood testing for chromogranin A

Treatment
Octreotide
Surgical removal

How well did you know this?
1
Not at all
2
3
4
5
Perfectly
621
Q

Which of the nerves listed below provides sensory innervation to the skin overlying the lateral aspect of the nose?

Infratrochlear nerve

Zygomatic nerve

Nasopalatine nerve

Lateral nasal branches of the ethmoidal nerve

Frontal nerve

A

The lateral aspect of the external nose is innervated by lateral nasal branches of the anterior ethmoidal nerve. The ethmoidal nerve is a branch of the nasociliary nerve which is one of the divisions of the trigeminal.
Trigeminal nerve

The trigeminal nerve is the main sensory nerve of the head. In addition to its major sensory role, it also innervates the muscles of mastication.

Distribution of the trigeminal nerve
Sensory
Scalp
Face
Oral cavity (and teeth)
Nose and sinuses
Dura mater
Motor
Muscles of mastication
Mylohyoid
Anterior belly of digastric
Tensor tympani
Tensor palati
Autonomic connections (ganglia)
Ciliary
Sphenopalatine
Otic
Submandibular

Path
Originates at the pons
Sensory root forms the large, crescentic trigeminal ganglion within Meckel’s cave, and contains the cell bodies of incoming sensory nerve fibres. Here the 3 branches exit.
The motor root cell bodies are in the pons and the motor fibres are distributed via the mandibular nerve. The motor root is not part of the trigeminal ganglion.

Branches of the trigeminal nerve
Ophthalmic nerve Sensory only
Maxillary nerve Sensory only
Mandibular nerve Sensory and motor

Sensory
Ophthalmic Exits skull via the superior orbital fissure
Sensation of: scalp and forehead, the upper eyelid, the conjunctiva and cornea of the eye, the nose (including the tip of the nose, except alae nasi), the nasal mucosa, the frontal sinuses, and parts of the meninges (the dura and blood vessels).
Maxillary nerve Exit skull via the foramen rotundum
Sensation: lower eyelid and cheek, the nares and upper lip, the upper teeth and gums, the nasal mucosa, the palate and roof of the pharynx, the maxillary, ethmoid and sphenoid sinuses, and parts of the meninges.
Mandibular nerve Exit skull via the foramen ovale
Sensation: lower lip, the lower teeth and gums, the chin and jaw (except the angle of the jaw), parts of the external ear, and parts of the meninges.

Motor
Distributed via the mandibular nerve.
The following muscles of mastication are innervated:
Masseter
Temporalis
Medial pterygoid
Lateral pterygoid

Other muscles innervated include:
Tensor veli palatini
Mylohyoid
Anterior belly of digastric
Tensor tympani

How well did you know this?
1
Not at all
2
3
4
5
Perfectly
622
Q

A 52 year old male attends renal transplant clinic for a post operative assessment. You note that he is on ciclosporin and that a recent blood test shows that the ciclosporin level is elevated. Of the adverse effects noted below, which is the greatest concern?

Hyperthyroidism

Diabetes

Alopecia

Hypothermia

Nephrotoxicity

A

Ciclosporin- nephrotoxicity
This patient is at risk of nephrotoxicity and should be referred to the renal team as soon as possible. Alopecia is associated with azathioprine and diabetes is associated with long term use.

Organ transplantation: immunosuppressants

A number of drugs are available which help to mitigate the processes resulting in acute rejection. Cyclosporin and tacrolimus are commonly used drugs.

Example regime
Initial: ciclosporin/tacrolimus with a monoclonal antibody
Maintenance: ciclosporin/tacrolimus with MMF or sirolimus
Add steroids if more than one steroid responsive acute rejection episode

Ciclosporin
Inhibits calcineurin, a phosphatase involved in T cell activation
Nephrotoxic
Monitor levels

Azathioprine
Metabolised to form 6 mercaptopurine which inhibits DNA synthesis and cell division
Side effects include myelosupression, alopecia and nausea

Tacrolimus
Lower incidence of acute rejection compared to ciclosporin
Also less hypertension and hyperlipidaemia
However, high incidence of impaired glucose tolerance and diabetes

Mycophenolate mofetil (MMF)
Blocks purine synthesis by inhibition of IMPDH
Therefore inhibits proliferation of B and T cells
Side-effects: GI and marrow suppression

Sirolimus (rapamycin)
Blocks T cell proliferation by blocking the IL-2 receptor
Can cause hyperlipidaemia

Monoclonal antibodies
Selective inhibitors of IL-2 receptor
Daclizumab
Basilximab

How well did you know this?
1
Not at all
2
3
4
5
Perfectly
623
Q

A 22 year man is shot in the back, in the lumbar region. He has increased tone and hyper-reflexia of his right leg. He cannot feel his left leg. What is the most likely explanation?

Epidural haematoma

Osteomyelitis

Transverse myelitis

Brown-Sequard syndrome

Tabes dorsalis

A

Brown -Sequard syndrome is caused by hemisection of the spinal cord. It may result from stab injuries or lateral vertebral fractures. It results in ipsilateral paralysis (pyramidal tract) , and also loss of proprioception and fine discrimination (dorsal columns). Pain and temperature sensation are lost on the contra-lateral side. This is because the fibres of the spinothalamic tract have decussated below the level of the cord transection.

Spinal disorders

Dorsal column lesion
Loss vibration and proprioception
Tabes dorsalis, SACD
Spinothalamic tract lesion
Loss of pain, sensation and temperature
Central cord lesion
Flaccid paralysis of the upper limbs
Osteomyelitis
Normally progressive
Staph aureus in IVDU, normally cervical region affected
Fungal infections in immunocompromised
Thoracic region affected in TB
Infarction spinal cord
Dorsal column signs (loss of proprioception and fine discrimination)
Cord compression
UMN signs
Malignancy
Haematoma
Fracture
Brown-sequard syndrome
Hemisection of the spinal cord
Ipsilateral paralysis
Ipsilateral loss of proprioception and fine discrimination
Contralateral loss of pain and temperature

Dermatomes
C2 to C4 The C2 dermatome covers the occiput and the top part of the neck. C3 covers the lower part of the neck to the clavicle. C4 covers the area just below the clavicle.
C5 to T1 Situated in the arms. C5 covers the lateral arm at and above the elbow. C6 covers the forearm and the radial (thumb) side of the hand. C7 is the middle finger, C8 is the medial aspect of the hand, and T1 covers the medial side of the forearm.
T2 to T12 The thoracic covers the axillary and chest region. T3 to T12 covers the chest and back to the hip girdle. The nipples are situated in the middle of T4. T10 is situated at the umbilicus. T12 ends just above the hip girdle.
L1 to L5 The cutaneous dermatome representing the hip girdle and groin area is innervated by L1 spinal cord. L2 and 3 cover the front part of the thighs. L4 and L5 cover medial and lateral aspects of the lower leg.
S1 to S5 S1 covers the heel and the middle back of the leg. S2 covers the back of the thighs. S3 cover the medial side of the buttocks and S4-5 covers the perineal region. S5 is of course the lowest dermatome and represents the skin immediately at and adjacent to the anus.

Myotomes

Upper limb
Elbow flexors/Biceps C5
Wrist extensors C6
Elbow extensors/Triceps C7
Long finger flexors C8
Small finger abductors T1

Lower limb
Hip flexors (psoas) L1 and L2
Knee extensors (quadriceps) L3
Ankle dorsiflexors (tibialis anterior) L4 and L5
Toe extensors (hallucis longus) L 5
Ankle plantar flexors (gastrocnemius) S1

The anal sphincter is innervated by S2,3,4

How well did you know this?
1
Not at all
2
3
4
5
Perfectly
624
Q

Which of the following statements relating to sternocleidomastoid is untrue?

The external jugular vein lies posteromedially.

It is supplied by the accessory nerve.

It has two heads of origin

It inserts into the lateral aspect of the mastoid process.

It marks the anterior border of the posterior triangle.

A

The external jugular vein lies lateral (i.e. superficial) to the sternocleidomastoid.

Sternocleidomastoid

Anatomy
Origin Rounded tendon attached to upper manubrium sterni and muscular head attached to medial third of the clavicle
Insertion Mastoid process of the temporal bone and lateral area of the superior nuchal line of the occipital bone
Innervation Spinal part of accessory nerve and anterior rami of C2 and C3 (proprioception)
Action
Both: extend the head at atlanto-occipital joint and flex the cervical vertebral column. Accessory muscles of inspiration.
Single: lateral flexion of neck, rotates head so face looks upward to the opposite side

Sternocleidomastoid divides the anterior and posterior triangles of the neck.

How well did you know this?
1
Not at all
2
3
4
5
Perfectly
625
Q

A motor cyclist is involved in a road traffic accident causing severe right shoulder injuries. He is found to have an adducted, medially rotated shoulder. The elbow is fully extended and the forearm pronated. Which is the most likely diagnosis?

C8, T1 root lesion

C5, C6 root lesion

Radial nerve lesion

Ulnar nerve lesion

Axillary nerve lesion

A

Erbs Palsy C5, C6 lesion
The features include:
Waiter’s tip position
Loss of shoulder abduction (deltoid and supraspinatus paralysis)
Loss of external rotation of the shoulder (paralysis of infraspinatus)
Loss of elbow flexion (paralysis of biceps, brachialis and brachioradialis)
Loss of forearm supination (paralysis of Biceps)
The motorcyclist has had an Erb’s palsy (C5, C6 root lesion). This is commonly known to be associated with birth injury when a baby has a shoulder dystocia.

Brachial plexus

The brachial plexus extends from the neck to the axilla. It is formed by the ventral rami of the fifth to the eighth cervical nerves with the ascending part of the first thoracic nerve.

Location of the plexus
The ventral rami which form the plexus enter the lower part of the posterior triangle of the neck in series with the ventral rami of the cervical plexus. The second part of the subclavian artery lies immediately anterior to the lower two rami. The upper three rami intermingle and pass inferolaterally towards the axilla and subclavian artery. They are enclosed within an extension of the prevertebral fascia. In the neck the plexus lies deep to platysma, the supraclavicular nerves, inferior belly of omohyoid and the transverse cervical artery. It then passes deep to the clavicle and the suprascapular vessels, to enter the axilla, and thence surround the second part of the axillary artery

Composition of the plexus
Ventral rami, the roots of the plexus, lie between scalenus medius and anterior.

As they enter the posterior triangle, the upper two (C5,6) and lower two (C8, T1) roots of the plexus unite to form the upper and lower trunks of the plexus respectively. Meanwhile, C7 continues as the middle trunk. The lower trunk may groove the superior surface of the first rib posterior to the subclavian artery, and the root from the first ventral ramus is always in contact with it.

Each trunk divides into ventral and dorsal divisions which are destined to supply the anterior (flexor) and posterior (extensor) parts of the upper limb.

The cords of the plexus are formed in the axilla. The dorsal divisions unite to form the posterior cord (C5-8). The ventral divisions of the upper and middle trunks unite to form the lateral cord (C5-7), while the ventral divisions of the lower trunk continues as the medial cord (C8-T1). The cords are named according to their relationship to the axillary artery. Each cord terminates by dividing into two main branches at the beginning of the third part of the artery.

Sympathetic communications
The fifth and sixth cervical ventral rami receive grey rami communicantes from the middle cervical ganglion, while the two or more grey rami communicantes pass from the inferior cervical ganglion to the seventh and eighth cervical ventral rami. The first thoracic ventral ramus receives its grey ramus from the cervicothoracic ganglion. Its for this reason that inferior plexus injury can be complicated by a Horners syndrome.

Summary
Origin Anterior rami of C5 to T1
Sections of the plexus
Roots, trunks, divisions, cords, branches
Mnemonic:Real Teenagers Drink Cold Beer
Roots
Located in the posterior triangle
Pass between scalenus anterior and medius
Trunks
Located posterior to middle third of clavicle
Upper and middle trunks related superiorly to the subclavian artery
Lower trunk passes over 1st rib posterior to the subclavian artery
Divisions Apex of axilla
Cords Related to axillary artery

How well did you know this?
1
Not at all
2
3
4
5
Perfectly
626
Q

A 55 year old man presents with symptoms of dyspepsia and on upper GI endoscopy an area of patchy erythematous tissue is identified extending proximally from the gastro oesophageal junction. A biopsy is diagnostic of Barretts oesophagus with low grade dysplasia. Which of the following is the most appropriate next step?

Distal oesophagectomy

Upper GI endoscopy with quadrantic biopsies from the region

Photodynamic therapy

Endoscopic sub mucosal resection of the area

Argon plasma coagulation

A

In Barrett’s surveillance the safest option is quadrantic (i.e. 4 biopsies, one from each quarter of the oesophagus at 2cm intervals)
Low grade dysplasia in conjunction with Barretts oesphagus should be monitored with regular (6 monthly) upper GI endoscopy and quadrantic biopsies. If the disease remains static at 2 years then the screening frequency may be decreased.

Barrett’s oesophagus

Barretts oesophagus is a condition characterised by the metaplastic transformation of squamous oesophageal epithelium to columnar gastric type epithelium. Three types of this metaplastic process are recognised; intestinal (high risk), cardiac and fundic. The latter two categories may cause difficulties in diagnosis. The most concrete diagnosis can be made when endoscopic features of Barretts oesophagus are present together with a deep biopsy that demonstrates not just goblet cell metaplasia but also oesophageal glands.

Barrett’s can be sub divided into short (<3cm) and long (>3cm). The length of the affected segment correlates strongly with the chances of identifying metaplasia. The overall prevalence of Barrett’s oesophagus is difficult to determine but may be in the region of 1 in 20 and is identified in up to 12% of those undergoing endoscopy for reflux.

A proportion of patients with metaplasia will progress to dysplasia and for this reason individuals identified as having Barrett’s should undergo endoscopic surveillance (every 2-5 years). Biopsies should be quadrantic and taken at 2-3cm intervals. Biopsies need to be adequate. Where mass lesions are present consideration should be given to endoscopic sub mucosal resection. Up to 40% of patients will be upstaged from high grade dysplasia to invasive malignancy with such techniques.

Treatment
Long term proton pump inhibitor
Consider pH and manometry studies in younger patients who may prefer to consider an anti reflux procedure
Regular endoscopic monitoring (more frequently if moderate dysplasia). With quadrantic biopsies every 2-3 cm
If severe dysplasia be very wary of small foci of cancer

References
A consensus statement of the British approach is provided by:
Bennett C et al Consensus Statements for Management of Barrett’s Dysplasia and Early-Stage Esophageal Adenocarcinoma, Based on a Delphi Process. Gastroenterology Volume 143, Issue 2 , Pages 336-346, August 2012.

How well did you know this?
1
Not at all
2
3
4
5
Perfectly
627
Q

A 74 year old gentleman presents with an obstructing carcinoma of the splenic flexure. Attempts at placement of a colonic stent have failed. Which of the operative options listed below offers the best solution to this problem?

Sub total colectomy

Extended right hemicolectomy

Standard right hemicolectomy

Standard left hemicolectomy

Transverse colectomy

A

Standard right hemicolectomy involves colonic division to the right of the middle colic vessels
Extended right hemicolectomy involves division of the middle colic vessels and usually resection of the splenic flexure as well.
The question always causes confusion and to understand it the information needs to be carefully read. Firstly, the tumour is definitely at the splenic flexure and the second point is that the operation is definitely an extended right hemicolectomy. A left hemicolectomy or even the older operation of a transverse colectomy could be considered if the patient was not obstructed. However, when obstruction is present, an extended right hemicolectomy (which involves an ileocolic anastomosis) is relatively safe even in the obstructed setting.

Colorectal cancer treatment

Patients diagnosed as having colorectal cancer should be completely staged using CT of the chest/ abdomen and pelvis. Their entire colon should have been evaluated with colonoscopy or CT colonography. Patients whose tumours lie below the peritoneal reflection should have their mesorectum evaluated with MRI.

Once their staging is complete patients should be discussed within a dedicated colorectal MDT meeting and a treatment plan formulated.

Treatment of colonic cancer
Cancer of the colon is nearly always treated with surgery. Stents, surgical bypass and diversion stomas may all be used as palliative adjuncts. Resectional surgery is the only option for cure in patients with colon cancer. The procedure is tailored to the patient and the tumour location. The lymphatic drainage of the colon follows the arterial supply and therefore most resections are tailored around the resection of particular lymphatic chains (e.g. ileo-colic pedicle for right sided tumours). Some patients may have confounding factors that will govern the choice of procedure, for example a tumour in a patient from a HNPCC family may be better served with a panproctocolectomy rather than segmental resection. Following resection the decision has to be made regarding restoration of continuity. For an anastomosis to heal the key technical factors include; adequate blood supply, mucosal apposition and no tissue tension. Surrounding sepsis, unstable patients and inexperienced surgeons may compromise these key principles and in such circumstances it may be safer to construct an end stoma rather than attempting an anastomosis.
When a colonic cancer presents with an obstructing lesion; the options are to either stent it or resect. In modern practice it is unusual to simply defunction a colonic tumour with a proximal loop stoma. This differs from the situation in the rectum (see below).
Following resection patients with risk factors for disease recurrence are usually offered chemotherapy, a combination of 5FU and oxaliplatin is common.

Rectal cancer
The management of rectal cancer is slightly different to that of colonic cancer. This reflects the rectum’s anatomical location and the challenges posed as a result. Tumours located in the rectum can be surgically resected with either an anterior resection or an abdomino - perineal resection. The technical aspects governing the choice between these two procedures can be complex to appreciate and the main point to appreciate for the MRCS is that involvement of the sphincter complex or very low tumours require APER. In the rectum a 2cm distal clearance margin is required and this may also impact on the procedure chosen. In addition to excision of the rectal tube an integral part of the procedure is a meticulous dissection of the mesorectal fat and lymph nodes (total mesorectal excision/ TME). In rectal cancer surgery invovlement of the cirumferential resection margin carries a high risk of disease recurrence. Because the rectum is an extraperitoneal structure (until you remove it that is!) it is possible to irradiate it, something which cannot be offered for colonic tumours. This has a major impact in rectal cancer treatment and many patients will be offered neoadjuvent radiotherapy (both long and short course) prior to resectional surgery. Patients with T1, 2 and 3 /N0 disease on imaging do not require irradiation and should proceed straight to surgery. Patients with T4 disease will typically have long course chemo radiotherapy. Patients presenting with large bowel obstruction from rectal cancer should not undergo resectional surgery without staging as primary treatment (very different from colonic cancer). This is because rectal surgery is more technically demanding, the anastomotic leak rate is higher and the danger of a positive resection margin in an unstaged patient is high. Therefore patients with obstructing rectal cancer should have a defunctioning loop colostomy.

Summary of procedures
The operations for cancer are segmental resections based on blood supply and lymphatic drainage. These commonly performed procedures are core knowledge for the MRCS and should be understood.

Site of cancer Type of resection Anastomosis Risk of leak
Right colon Right hemicolectomy Ileo-colic Low <5%
Transverse Extended right hemicolectomy Ileo-colic Low <5%
Splenic flexure Extended right hemicolectomy Ileo-colic Low <5%
Splenic flexure Left hemicolectomy Colo-colon 2-5%
Left colon Left hemicolectomy Colo-colon 2-5%
Sigmoid colon High anterior resection Colo-rectal 5%
Upper rectum Anterior resection (TME) Colo-rectal 5%
Low rectum Anterior resection (Low TME) Colo-rectal
(+/- Defunctioning stoma) 10%
Anal verge Abdomino-perineal excision of colon and rectum None n/a

In the emergency setting, where the bowel has perforated, the risk of an anastomotic breakdown is much greater, particularly when the anastomosis is colon-colon. In this situation, an end colostomy is often safer and can be reversed later. When resection of the sigmoid colon is performed and an end colostomy is fashioned the operation is referred to as a Hartmans procedure. Whilst left sided resections are more risky, ileo-colic anastomoses are relatively safe even in the emergency setting and do not need to be defunctioned.

References
A review of the diagnosis and management of colorectal cancer and a summary of the UK National Institute of Clinical Excellence guidelines is provided in:
Poston G, et al . Diagnosis and management of colorectal cancer: summary of NICE guidance. BMJ 2011: 343: d 6751.

How well did you know this?
1
Not at all
2
3
4
5
Perfectly
628
Q

Which statement is false about the foramina of the skull?

The hypoglossal canal transmits the hypoglossal nerve

The foramen spinosum is at the base of the medial pterygoid plate.

The jugular foramen transmits the accessory nerve

The foramen lacerum is located between the sphenoid and temporal bones

The stylomastoid foramen transmits the facial nerve

A

Foramina of the base of the skull

Foramen Location Contents
Foramen ovale Sphenoid bone Otic ganglion
V3 (Mandibular nerve:3rd branch of
trigeminal)
Accessory meningeal artery
Lesser petrosal nerve
Emissary veins
Foramen spinosum Sphenoid bone Middle meningeal artery
Meningeal branch of the Mandibular nerve
Foramen rotundum Sphenoid bone Maxillary nerve (V2)
Foramen lacerum/ carotid canal Located between the sphenoid, the apex of the petrous temporal and the basilar part of the occipital Base of the medial pterygoid plate.
Internal carotid artery*
Nerve and artery of the pterygoid canal
Jugular foramen Temporal bone Anterior: inferior petrosal sinus
Intermediate: glossopharyngeal, vagus, and accessory nerves.
Posterior: sigmoid sinus (becoming the internal jugular vein) and some meningeal branches from the occipital and ascending pharyngeal arteries.
Foramen magnum Occipital bone Anterior and posterior spinal arteries
Vertebral arteries
Medulla oblongata
Stylomastoid foramen Temporal bone Stylomastoid artery
Facial nerve
Superior orbital fissure Sphenoid bone Oculomotor nerve (III)
Recurrent meningeal artery
Trochlear nerve (IV)
Lacrimal, frontal and nasociliary branches of opthalmic nerve (V1)
Abducent nerve (VI)
Superior ophthalmic vein

*= In life the foramen lacerum is occluded by a cartilagenous plug. The ICA initially passes into the carotid canal which ascends superomedially to enter the cranial cavity through the foramen lacerum.

How well did you know this?
1
Not at all
2
3
4
5
Perfectly
629
Q

Which nerve directly innervates the sinoatrial node?

Superior cardiac nerve

Right vagus nerve

Left vagus nerve

Inferior cardiac nerve

None of the above

A

No single one of the above nerves is responsible for direct cardiac innervation (which those who have handled the heart surgically will appreciate).
The heart receives its nerves from the superficial and deep cardiac plexuses. The cardiac plexuses send small branches to the heart along the major vessels, continuing with the right and left coronary arteries. The vagal efferent fibres emerge from the brainstem in the roots of the vagus and accessory nerves, and run to ganglia in the cardiac plexuses and within the heart itself.

The background vagal discharge serves to limit heart rate, and loss of this background vagal tone accounts for the higher resting heart rate seen following cardiac transplant.

Sinoatrial node

Located in the wall of the right atrium in the upper part of the sulcus terminalis from which it extends anteriorly over the opening of the superior vena cava.
In most cases it is supplied by the right coronary artery.
It has a complicated nerve supply from the cardiac nerve plexus that takes both sympathetic and parasympathetic fibres that run alongside the main vessels.

How well did you know this?
1
Not at all
2
3
4
5
Perfectly
630
Q

A 22 year old lady receives intravenous morphine for acute abdominal pain. Which of the following best accounts for its analgesic properties?

Binding to δ opioid receptors in the brainstem

Binding to δ opioid receptors at peripheral nerve sites

Binding to β opioid receptors within the CNS

Binding to α opioid receptors within the CNS

Binding to µ opioid receptors within the CNS

A

Binding to µ opioid receptors within the CNS
4 Types of opioid receptor:
δ (located in CNS)- Accounts for analgesic and antidepressant effects
k (mainly CNS)- analgesic and dissociative effects
µ (central and peripheral) - causes analgesia, miosis, decreased gut motility
Nociceptin receptor (CNS)- Affect of appetite and tolerance to µ agonists.

Morphine

Strong opiate analgesic. It is a pro- type narcotic drug and its effects mediated via the 4 types of opioid receptor. Its clinical effects stem from binding to these receptor sites within the CNS and gastrointestinal tract. Unwanted side effects include nausea, constipation, respiratory depression and, if used long term, addiction .
It may be administered orally or intravenously. It can be reversed with naloxone.

How well did you know this?
1
Not at all
2
3
4
5
Perfectly
631
Q

A 22 year old intravenous drug user is found to have a femoral abscess. The nursing staff contact the on call doctor as the patient has a temperature of 39oC. He is found to have a pan systolic murmur loudest at the left sternal edge at the 4th intercostal space. What is the most likely underlying lesion?

Aortic regurgitation

Mitral regurgitation

Aortic stenosis

Tricuspid stenosis

Tricuspid regurgitation

A

Intravenous drug users are at high risk of right sided cardiac valvular endocarditis. The character of the murmur fits with a diagnosis of tricuspid valve endocarditis.

Cardiac murmurs

Type of Murmur Conditions
Ejection systolic Aortic stenosis
Pulmonary stenosis, HOCM
ASD, Fallot’s
Pan-systolic Mitral regurgitation
Tricuspid regurgitation
VSD
Late systolic Mitral valve prolapse
Coarctation of aorta
Early diastolic Aortic regurgitation
Graham-Steel murmur (pulmonary regurgitation)
Mid diastolic Mitral stenosis
Austin-Flint murmur (severe aortic regurgitation)

How well did you know this?
1
Not at all
2
3
4
5
Perfectly
632
Q

A 73 year old lady is undergoing surgery to remove the middle lobe of the right lung. Which of these sites should the incision be centered on to gain access to the apex of the right middle lobe?

6th interspace in the mid axillary line

4th interspace in the mid clavicular line

6th interspace posterior to the mid axillary line

4th interspace posterior to the mid axillary line

3rd interspace in the mid axillary line

A

The right middle lobe is a wedge shaped structure. Its apex is posterior and it lies deep to the 6th rib in the mid axillary line. Anteriorly, it is in contact with the 5th, 6th and 7th costal cartiladges.

Lung anatomy

The right lung is composed of 3 lobes divided by the oblique and transverse fissures. The left lung has two lobes divided by the oblique fissure.The apex of both lungs is approximately 4cm superior to the sterno-costal joint of the first rib. Immediately below this is a sulcus created by the subclavian artery.

Peripheral contact points of the lung
Base: diaphragm
Costal surface: corresponds to the cavity of the chest
Mediastinal surface: Contacts the mediastinal pleura. Has the cardiac impression. Above and behind this concavity is a triangular depression named the hilum, where the structures which form the root of the lung enter and leave the viscus. These structures are invested by pleura, which, below the hilum and behind the pericardial impression, forms the pulmonary ligament

Right lung
Above the hilum is the azygos vein; Superior to this is the groove for the superior vena cava and right innominate vein; behind this, and nearer the apex, is a furrow for the innominate artery. Behind the hilum and the attachment of the pulmonary ligament is a vertical groove for the oesophagus; In front and to the right of the lower part of the oesophageal groove is a deep concavity for the extrapericardiac portion of the inferior vena cava.

The root of the right lung lies behind the superior vena cava and the right atrium, and below the azygos vein.

The right main bronchus is shorter, wider and more vertical than the left main bronchus and therefore the route taken by most foreign bodies.
Left lung
Above the hilum is the furrow produced by the aortic arch, and then superiorly the groove accommodating the left subclavian artery; Behind the hilum and pulmonary ligament is a vertical groove produced by the descending aorta, and in front of this, near the base of the lung, is the lower part of the oesophagus.

The root of the left lung passes under the aortic arch and in front of the descending aorta.
Inferior borders of both lungs
6th rib in mid clavicular line
8th rib in mid axillary line
10th rib posteriorly
The pleura runs two ribs lower than the corresponding lung level.

Bronchopulmonary segments
Segment number Right lung Left lung
1 Apical Apical
2 Posterior Posterior
3 Anterior Anterior
4 Lateral Superior lingular
5 Medial Inferior lingular
6 Superior (apical) Superior (apical)
7 Medial basal Medial basal
8 Anterior basal Anterior basal
9 Lateral basal Lateral basal
10 Posterior basal Posterior basal

How well did you know this?
1
Not at all
2
3
4
5
Perfectly
633
Q

A 19 year old man is playing rugby when he suddenly notices a severe pain at the posterolateral aspect of his right thigh. Which of the following muscle groups is most likely to have been injured?

Semimembranosus

Semitendinosus

Long head of biceps femoris

Gastrocnemius

Soleus

A

The biceps femoris is the laterally located hamstring muscle. The semitendinosus and semimembranosus are located medially. Rupture of gastrocnemius and soleus may occur but is less common.

Biceps femoris

The biceps femoris is one of the hamstring group of muscles located in the posterior upper thigh. It has two heads.

Long head
Origin Ischial tuberosity
Insertion Fibular head
Action Knee flexion, lateral rotation tibia, extension hip
Innervation Tibial division of sciatic nerve (L5, S1, S2)
Arterial supply Profunda femoris artery, inferior gluteal artery, and the superior muscular branches of popliteal artery

Short head
Origin Lateral lip of linea aspera, lateral supracondylar ridge of femur
Insertion Fibular head
Action Knee flexion, lateral rotation tibia
Innervation Common peroneal division of sciatic nerve (L5, S1, S2)
Arterial supply Profunda femoris artery, inferior gluteal artery, and the superior muscular branches of popliteal artery

How well did you know this?
1
Not at all
2
3
4
5
Perfectly
634
Q

Which of the following features are not typical of Crohns disease?

Complex fistula in ano

Small bowel strictures

Skip lesions

‘Rose thorn ulcers’ on barium studies

Pseudopolyps on colonoscopy

A

Pseudopolyps are a feature of ulcerative colitis and occur when there is severe mucosal ulceration. The remaining islands of mucosa may then appear to be isolated and almost polypoidal.

Crohns disease

Crohns disease is a chronic transmural inflammation of a segment(s) of the gastrointestinal tract and may be associated with extra intestinal manifestations. Frequent disease patterns observed include ileal, ileocolic and colonic disease. Peri-anal disease may occur in association with any of these. The disease is often discontinuous in its distribution. Inflammation may cause ulceration, fissures, fistulas and fibrosis with stricturing. Histology reveals a chronic inflammatory infiltrate that is usually patchy and transmural.

Ulcerative colitis Vs Crohns
Crohn’s disease Ulcerative colitis
Distribution Mouth to anus Rectum and colon
Macroscopic changes Cobblestone appearance, apthoid ulceration Contact bleeding
Depth of disease Transmural inflammation Superficial inflammation
Distribution pattern Patchy Continuous
Histological features Granulomas (non caseating epithelioid cell aggregates with Langhans’ giant cells) Crypt abscesses, Inflammatory cells in the lamina propria

Extraintestinal manifestations of Crohns
Related to disease extent Unrelated to disease extent
Aphthous ulcers (10%) Sacroiliiitis (10-15%)
Erythema nodosum (5-10%) Ankylosing spondylitis (1-2%)
Pyoderma gangrenosum (0.5%) Primary sclerosing cholangitis (Rare)
Acute arthropathy (6-12%) Gallstones (up to 30%)
Ocular complications (up to 10%) Renal calculi (up to 10%)

Diarrhoea in Crohns
Diarrhoea in Crohns may be multifactorial since actual inflammation of the colon is not common. Causes therefore include the following:
Bile salt diarrhoea secondary to terminal ileal disease
Entero-colic fistula
Short bowel due to multiple resections
Bacterial overgrowth

Surgical interventions in Crohns disease
The commonest disease pattern in Crohns is stricturing terminal ileal disease and this often culminates in an ileocaecal resection. Other procedures performed include segmental small bowel resections and stricturoplasty. Colonic involvement in patients with Crohns is not common and, where found, distribution is often segmental. However, despite this distribution segmental resections of the colon in patients with Crohns disease are generally not advocated because the recurrence rate in the remaining colon is extremely high. As a result, the standard options of colonic surgery in Crohns patients are generally; sub total colectomy, panproctocolectomy and staged sub total colectomy and proctectomy. Restorative procedures such as ileoanal pouch have no role in therapy.
Crohns disease is notorious for the developmental of intestinal fistulae; these may form between the rectum and skin (peri anal) or the small bowel and skin. Fistulation between loops of bowel may also occur and result in bacterial overgrowth and malabsorption. Management of enterocutaneous fistulae involves controlling sepsis, optimising nutrition, imaging the disease and planning definitive surgical management.

How well did you know this?
1
Not at all
2
3
4
5
Perfectly
635
Q

During a right hemicolectomy the caecum is mobilised. As the bowel is retracted medially a vessel is injured, posterior to the colon. Which of the following is the most likely vessel?

Right colic artery

Inferior vena cava

Aorta

External iliac artery

Gonadal vessels

A

The key in this question is that its during the caecal mobilization. The gonadal vessels and ureter are important posterior relations that are at risk during a right hemicolectomy. During latter stages of the procedure, the ileocolic artery and vein are traced along the anterior aspect of the duodenum. At this point it is possible to injure these, the superior mesenteric vein or the middle colic vein, injury to any of these can result in torrential bleeding that is very difficult to control.

Caecum

Location
Proximal right colon below the ileocaecal valve
Intraperitoneal
Posterior relations
Psoas
Iliacus
Femoral nerve
Genitofemoral nerve
Gonadal vessels
Anterior relations Greater omentum
Arterial supply Ileocolic artery
Lymphatic drainage Mesenteric nodes accompany the venous drainage

The caecum is the most distensible part of the colon and in complete large bowel obstruction with a competent ileocaecal valve the most likely site of eventual perforation.

How well did you know this?
1
Not at all
2
3
4
5
Perfectly
636
Q

The oxygen-haemoglobin dissociation curve is shifted to the left in:

With decreased 2,3-DPG in transfused red cells

Respiratory acidosis

Sudden move to high altitude

Pyrexia

Haemolytic anaemia

A

With decreased 2,3-DPG in transfused red cells

S shaped curve
The curve is shifted to the left when there is a decreased oxygen requirement by the tissue. This includes:
1. Hypothermia
2. Alkalosis
3. Reduced levels of DPG:
DPG is found in erythrocytes and is reduced in non exercising muscles, i.e. when there is reduced glycolysis.
4. Polycythaemia

Oxygen Transport

Oxygen transport
Almost all oxygen is transported within erythrocytes. It has limited solubility and only 1% is carried as solution. Therefore the amount of oxygen transported will depend upon haemoglobin concentration and its degree of saturation.

Haemoglobin
Globular protein composed of 4 subunits. Haem consists of a protoporphyrin ring surrounding an iron atom in its ferrous state. The iron can form two additional bonds; one with oxygen and the other with a polypeptide chain. There are two alpha and two beta subunits to this polypeptide chain in an adult and together these form globin. Globin cannot bind oxygen but is able to bind to carbon dioxide and hydrogen ions, the beta chains are able to bind to 2,3 diphosphoglycerate. The oxygenation of haemoglobin is a reversible reaction. The molecular shape of haemoglobin is such that binding of one oxygen molecule facilitates the binding of subsequent molecules.

Oxygen dissociation curve
The oxygen dissociation curve describes the relationship between the percentage of saturated haemoglobin and partial pressure of oxygen in the blood. It is not affected by haemoglobin concentration.
Chronic anaemia causes 2, 3 DPG levels to increase, hence shifting the curve to the right

Haldane effect
Shifts to left = for given oxygen tension there is increased saturation of Hb with oxygen i.e. Decreased oxygen delivery to tissues

Bohr effect
Shifts to right = for given oxygen tension there is reduced saturation of Hb with oxygen i.e. Enhanced oxygen delivery to tissues

Shifts to Left = Lower oxygen delivery
HbF, methaemoglobin, carboxyhaemoglobin
low [H+] (alkali)
low pCO2
low 2,3-DPG
low temperature
Shifts to Right = Raised oxygen delivery
raised [H+] (acidic)
raised pCO2
raised 2,3-DPG*
raised temperature

*2,3-diphosphoglycerate

How well did you know this?
1
Not at all
2
3
4
5
Perfectly
637
Q

A 78 year old man is due to undergo an endarterectomy of the internal carotid artery. Which of the following nervous structures are most at risk during the dissection?

Recurrent laryngeal nerve

Sympathetic chain

Hypoglossal nerve

Phrenic nerve

Lingual nerve

A

Nerves at risk during a carotid endarterectomy:
Hypoglossal nerve
Greater auricular nerve
Superior laryngeal nerve
During a carotid endarterectomy the sternocleidomastoid muscle is dissected, with ligation of the common facial vein and then the internal jugular is dissected exposing the common and the internal carotid arteries. The nerves at risk during the operation include:
Hypoglossal nerve
Greater auricular nerve
Superior laryngeal nerve
The sympathetic chain lies posteriorly and is less prone to injury in this procedure.
Internal carotid artery

The internal carotid artery is formed from the common carotid opposite the upper border of the thyroid cartilage. It extends superiorly to enter the skull via the carotid canal. From the carotid canal it then passes through the cavernous sinus, above which it divides into the anterior and middle cerebral arteries.

Relations in the neck
Posterior
Longus capitis
Pre-vertebral fascia
Sympathetic chain
Superior laryngeal nerve
Medially
External carotid (near origin)
Wall of pharynx
Ascending pharyngeal artery
Laterally
Internal jugular vein (moves posteriorly at entrance to skull)
Vagus nerve (most posterolaterally)
Anteriorly
Sternocleidomastoid
Lingual and facial veins
Hypoglossal nerve

Relations in the carotid canal
Internal carotid plexus
Cochlea and middle ear cavity
Trigeminal ganglion (superiorly)
Leaves canal lies above the foramen lacerum

Path and relations in the cranial cavity
The artery bends sharply forwards in the cavernous sinus, the aducens nerve lies close to its inferolateral aspect. The oculomotor, trochlear, opthalmic and, usually, the maxillary nerves lie in the lateral wall of the sinus. Near the superior orbital fissure it turns posteriorly and passes postero-medially to pierce the roof of the cavernous sinus inferior to the optic nerve. It then passes between the optic and oculomotor nerves to terminate below the anterior perforated substance by dividing into the anterior and middle cerebral arteries.

Branches
Anterior and middle cerebral artery
Ophthalmic artery
Posterior communicating artery
Anterior choroid artery
Meningeal arteries
Hypophyseal arteries

How well did you know this?
1
Not at all
2
3
4
5
Perfectly
638
Q

A 19 year old sportswoman presents with knee pain which is worse on walking down the stairs and when sitting still. On examination, there is wasting of the quadriceps and pseudolocking of the knee. What is the diagnosis?

Osteoarthritis

Quadriceps tendon rupture

Undisplaced fracture patella

Chondromalacia patellae

Osgood Schlatters disease

A

A teenage girl with knee pain on walking down the stairs is characteristic for chondromalacia patellae (anterior knee pain). Most cases are managed with physiotherapy.

Knee injury

Types of injury

Ruptured anterior cruciate ligament
Sport injury
Mechanism: high twisting force applied to a bent knee
Typically presents with: loud crack, pain and RAPID joint swelling (haemoarthrosis)
Poor healing
Management: intense physiotherapy or surgery
Ruptured posterior cruciate ligament
Mechanism: hyperextension injuries
Tibia lies back on the femur
Paradoxical anterior draw test
Rupture of medial collateral ligament
Mechanism: leg forced into valgus via force outside the leg
Knee unstable when put into valgus position
Menisceal tear
Rotational sporting injuries
Delayed knee swelling
Joint locking (Patient may develop skills to ‘unlock’ the knee
Recurrent episodes of pain and effusions are common, often following minor trauma
Chondromalacia patellae
Teenage girls, following an injury to knee e.g. Dislocation patella
Typical history of pain on going downstairs or at rest
Tenderness, quadriceps wasting
Dislocation of the patella
Most commonly occurs as a traumatic primary event, either through direct trauma or through severe contraction of quadriceps with knee stretched in valgus and external rotation
Genu valgum, tibial torsion and high riding patella are risk factors
Skyline x-ray views of patella are required, although displaced patella may be clinically obvious
An osteochondral fracture is present in 5%
The condition has a 20% recurrence rate
Fractured patella
2 types:
i. Direct blow to patella causing undisplaced fragments
ii. Avulsion fracture
Tibial plateau fracture
Occur in the elderly (or following significant trauma in young)
Mechanism: knee forced into valgus or varus, but the knee fractures before the ligaments rupture
Varus injury affects medial plateau and if valgus injury, lateral plateau depressed fracture occurs
Classified using the Schatzker system (see below)

Schatzker Classification system for tibial plateau fractures
Type Anatomical description Features
1 Vertical split of lateral condyle Fracture through dense bone, usually in the young. It may be virtually undisplaced, or the condylar fragment may be pushed inferiorly and tilted
2 Vertical split of the lateral condyle combined with an adjacent load bearing part of the condyle The wedge fragment (which may be of variable size), is displaced laterally; the joint is widened. Untreated, a valgus deformity may develop
3 Depression of the articular surface with intact condylar rim The split does not extend to the edge of the plateau. Depressed fragments may be firmly embedded in subchondral bone, the joint is stable
4 Fragment of the medial tibial condyle Two injuries are seen in this category; (1) a depressed fracture of osteoporotic bone in the elderly. (2) a high energy fracture resulting in a condylar split that runs from the intercondylar eminence to the medial cortex. Associated ligamentous injury may be severe
5 Fracture of both condyles Both condyles fractured but the column of the metaphysis remains in continuity with the tibial shaft
6 Combined condylar and subcondylar fractures High energy fracture with marked comminution

How well did you know this?
1
Not at all
2
3
4
5
Perfectly
639
Q

With which of the following blood products is iatrogenic septicaemia with a gram positive organism most likely?

Cryoprecipitate

Platelets

Packed red cells

Factor VIII concentrate

Factor IX concentrate

A

Platelets are stored at room temperature and must be used soon after collection. This places them at increased risk of culturing gram positive organisms. Iatrogenic infection with gram negative organisms is more likely with packed red cells as these are stored at 4 degrees.
Infections with blood products of this nature are both rare.

Blood products

Whole blood fractions

Fraction Key points
Packed red cells Used for transfusion in chronic anaemia and cases where infusion of large volumes of fluid may result in cardiovascular compromise. Product obtained by centrifugation of whole blood.
Platelet rich plasma Usually administered to patients who are thrombocytopaenic and are bleeding or require surgery. It is obtained by low speed centrifugation.
Platelet concentrate Prepared by high speed centrifugation and administered to patients with thrombocytopaenia.
Fresh frozen plasma
Prepared from single units of blood.
Contains clotting factors, albumin and immunoglobulin.
Unit is usually 200 to 250ml.
Usually used in correcting clotting deficiencies in patients with hepatic synthetic failure who are due to undergo surgery.
Usual dose is 12-15ml/Kg-1.
It should not be used as first line therapy for hypovolaemia.
Cryoprecipitate
Formed from supernatant of FFP.
Rich source of Factor VIII and fibrinogen.
Allows large concentration of factor VIII to be administered in small volume.
SAG-Mannitol Blood Removal of all plasma from a blood unit and substitution with:
Sodium chloride
Adenine
Anhydrous glucose
Mannitol
Up to 4 units of SAG M Blood may be administered. Thereafter whole blood is preferred. After 8 units, clotting factors and platelets should be considered.

Cell saver devices
These collect patients own blood lost during surgery and then re-infuse it. There are two main types:
Those which wash the blood cells prior to re-infusion. These are more expensive to purchase and more complicated to operate. However, they reduce the risk of re-infusing contaminated blood back into the patient.
Those which do not wash the blood prior to re-infusion.
Their main advantage is that they avoid the use of infusion of blood from donors into patients and this may reduce risk of blood borne infection. It may be acceptable to Jehovah’s witnesses. It is contraindicated in malignant disease for risk of facilitating disease dissemination.

Blood products used in warfarin reversal
In some surgical patients the use of warfarin can pose specific problems and may require the use of specialised blood products

Immediate or urgent surgery in patients taking warfarin(1) (2):

  1. Stop warfarin
  2. Vitamin K (reversal within 4-24 hours)
    -IV takes 4-6h to work (at least 5mg)
    -Oral can take 24 hours to be clinically effective
  3. Fresh frozen plasma
    Used less commonly now as 1st line warfarin reversal
    -30ml/kg-1
    -Need to give at least 1L fluid in 70kg person (therefore not appropriate in fluid overload)
    -Need blood group
    -Only use if human prothrombin complex is not available
  4. Human Prothrombin Complex (reversal within 1 hour)
    -Bereplex 50 u/kg
    -Rapid action but factor 6 short half life, therefore give with vitamin K

References
1. Dentali, F., C. Marchesi, et al. (2011). ‘Safety of prothrombin complex concentrates for rapid anticoagulation reversal of vitamin K antagonists. A meta-analysis.’ Thromb Haemost 106(3): 429-438.

  1. http://www.transfusionguidelines.org/docs/pdfs/bbt-03warfarin-reversal-flowchart-2006.pdf
How well did you know this?
1
Not at all
2
3
4
5
Perfectly
640
Q

Which of the following statements relating to avascular necrosis is false?

When associated with fracture may occur despite the radiological evidence of fracture union.

Pain and stiffness will typically precede radiological evidence of the condition.

Drilling of affected bony fragments may be used to facilitate angiogenesis where arthroplasty is not warranted.

The earliest detectable radiological evidence is a radiolucency of the affected area coupled with subchondral collapse.

It is less likely when prompt anatomical alignment of fracture fragments is achieved.

A

Avascular necrosis- radiological changes occur late.
Radiolucency and subchondral collapse are late changes. The earliest evidence on plain films is the affected area appearing as being more radio-opaque due to hyperaemia and resorption of the neighboring area. It may be diagnosed earlier using bone scans and MRI.

Avascular necrosis

Cellular death of bone components due to interruption of the blood supply, causing bone destruction
Main joints affected are hip, scaphoid, lunate and the talus.
It is not the same as non union. The fracture has usually united.
Radiological evidence is slow to appear.
Vascular ingrowth into the affected bone may occur. However, many joints will develop secondary osteoarthritis.

Causes
P ancreatitis
L upus
A lcohol
S teroids
T rauma
I diopathic, infection
C aisson disease, collagen vascular disease
R adiation, rheumatoid arthritis
A myloid
G aucher disease
S ickle cell disease

Presentation
Usually pain. Often despite apparent fracture union.

Investigation
MRI scanning will show changes earlier than plain films.

Treatment
In fractures at high risk sites anticipation is key. Early prompt and accurate reduction is essential.

Non weight bearing may help to facilitate vascular regeneration.

Joint replacement may be necessary, or even the preferred option (e.g. Hip in the elderly).

How well did you know this?
1
Not at all
2
3
4
5
Perfectly
641
Q

Inferior alveolar nerve block may result in?

Numbness of lower lip on injected side

Ineffective block for the lower incisor teeth

Numbness of hard palate

Inability to clench jaws

Transient ipsilateral facial weakness

A

An inferior alveolar nerve block is a nerve block technique that anesthetizes the areas of the mouth and face innervated by one of the inferior alveolar nerves1. It may result in various symptoms, such as numbness of the lower lip and chin, parts of the tongue and gingiva, and transient facial weakness2.
The best answer to your question is:
Numbness of lower lip on injected side
This is because the inferior alveolar nerve block affects the mental nerve, which is a branch of the inferior alveolar nerve that supplies the skin and mucous membranes of the lower lip and chin on one side1. The other options are either incorrect or less likely to occur. For example, an inferior alveolar nerve block is usually effective for the lower incisor teeth, unless there is anatomical variation or accessory innervation3. Numbness of the hard palate is not caused by an inferior alveolar nerve block, but by a maxillary nerve block. Inability to clench jaws is not a common symptom of an inferior alveolar nerve block, but it may happen if the nerve block also affects the motor branches of the mandibular nerve. Transient ipsilateral facial weakness is a rare complication of an inferior alveolar nerve block, and it may occur due to diffusion of the anesthetic agent into the parotid gland, affecting the facial nerve.

How well did you know this?
1
Not at all
2
3
4
5
Perfectly
642
Q

An 88 year old lady presents with a large mass in the upper inner quadrant of her right breast. Investigations confirm an oestrogen receptor positive, invasive ductal carcinoma. She has declined operative treatment. What is the best course of action?

Combined chemoradiotherapy

Radical radiotherapy

Administration of letrozole

Best supportive care

Chemotherapy alone

A

Elderly patients may be managed using endocrine therapy alone. Eventually most will escape hormonal control. In post menopausal women oestrogens are produced by the peripheral aromatization of androgens and aromatase inhibitors are therefore the most popular agent in this age group.

Breast cancer treatment

Treatment Indication
Endocrine therapy
Oestrogen receptor positive tumours
Downstaging primary lesions
Definitive treatment in old, infirm patients
Irradiation
Wide local excision
Large lesion, high grade or marked vascular invasion following mastectomy
Chemotherapy
Downstaging advanced lesions to facilitate breast conserving surgery
Patients with grade 3 lesions or axillary nodal disease

Endocrine agents
Tamoxifen is used and works as a partial oestrogen receptor agonist. It will typically block activity at the breast. It does, however, stimulate the receptors at other sites and it is this that accounts for its association with endometrial cancer. In post menopausal women the process of aromatisation accounts for most oestrogen production. Therefore in this group aromatase inhibitors are the preferred agents. Women who are perimenopausal start on tamoxifen and switch at 3 years.
More recent studies (aTTom and ATLAS) have demonstrated benefits for continuing the drug for 10 years. In pre-menopausal women, there is increasing preference for the use of Exemestane over tamoxifen.

Chemotherapy
The FEC regime is most commonly used (Fluorouracil, epirubicin and cyclophosphamide). This was found to be superior to the older CMF regime. The Taxanes are commonly used in high risk patients and in this setting a regime of docetaxal, doxorubicin and cyclophosphamide may be used. The anthracycline class drugs have marked cardiotoxicity (a property that they share with trastuzumab) and this can limit their use.

How well did you know this?
1
Not at all
2
3
4
5
Perfectly
643
Q

Which of the following is not a content of the porta hepatis?

Portal vein

Hepatic artery

Cystic duct

Lymph nodes

None of the above

A

The cystic duct lies outside the porta hepatis and is an important landmark in laparoscopic cholecystectomy. The structures in the porta hepatis are:
Portal vein
Hepatic artery
Common hepatic duct
These structures divide immediately after or within the porta hepatis to supply the functional left and right lobes of the liver.
The porta hepatis is also surrounded by lymph nodes, that may enlarge to produce obstructive jaundice and parasympathetic nervous fibres that travel along vessels to enter the liver.

Liver

Structure of the liver
Right lobe
Supplied by right hepatic artery
Contains Couinaud segments V to VIII (-/+Sg I)
Left lobe
Supplied by the left hepatic artery
Contains Couinaud segments II to IV (+/- Sg1)
Quadrate lobe
Part of the right lobe anatomically, functionally is part of the left
Couinaud segment IV
Porta hepatis lies behind
On the right lies the gallbladder fossa
On the left lies the fossa for the umbilical vein
Caudate lobe
Supplied by both right and left hepatic arteries
Couinaud segment I
Lies superior to the porta hepatis
Anterior and medial to the inferior vena cava
Bile from the caudate lobe drains into both right and left hepatic ducts

Detailed knowledge of Couinaud segments is not required for MRCS
Between the liver lobules are portal canals which contain the portal triad: Hepatic Artery, Portal Vein, tributary of Bile Duct.

Relations of the liver
Anterior Postero inferiorly
Diaphragm Oesophagus
Xiphoid process Stomach
Duodenum
Hepatic flexure of colon
Right kidney
Gallbladder
Inferior vena cava

Porta hepatis
Location Postero inferior surface, it joins nearly at right angles with the left sagittal fossa, and separates the caudate lobe behind from the quadrate lobe in front
Transmits
Common hepatic duct
Hepatic artery
Portal vein
Sympathetic and parasympathetic nerve fibres
Lymphatic drainage of the liver (and nodes)

Ligaments
Falciform ligament
2 layer fold peritoneum from the umbilicus to anterior liver surface
Contains ligamentum teres (remnant umbilical vein)
On superior liver surface it splits into the coronary and left triangular ligaments
Ligamentum teres Joins the left branch of the portal vein in the porta hepatis
Ligamentum venosum Remnant of ductus venosus

Arterial supply
Hepatic artery

Venous
Hepatic veins
Portal vein

Nervous supply
Sympathetic and parasympathetic trunks of coeliac plexus

How well did you know this?
1
Not at all
2
3
4
5
Perfectly
644
Q

A 55 year old accountant has jaundice and a temperature of 39oC. He is known to have gallstones. Blood cultures have grown a gram negative bacilli. Imaging shows a bile duct measuring 1.2cm in diameter. What is the best treatment option?

PTC and stent

PTC and drain

ERCP and stent

MRCP

USS

A

You should suspect cholangitis in a patient with fevers and jaundice. Charcot’s triad may only be present in 20% of patients.
Note the question states treatment option, this excludes MRCP and USS which are not treatments. Whilst PTC may access the duct, the drains may displace easily and the transduodenal route is the preferred first line access.

Management of Pancreatitis

Management of Acute Pancreatitis in the UK

Diagnosis
Traditionally hyperamylasaemia has been utilised with amylase being elevated three times the normal range.
However, amylase may give both false positive and negative results.
Serum lipase is both more sensitive and specific than serum amylase. It also has a longer half life.
Serum amylase levels do not correlate with disease severity.

Differential causes of hyperamylasaemia
Acute pancreatitis
Pancreatic pseudocyst
Mesenteric infarct
Perforated viscus
Acute cholecystitis
Diabetic ketoacidosis

Assessment of severity
Glasgow, Ranson scoring systems and APACHE II
Biochemical scoring e.g. using CRP

Features that may predict a severe attack within 48 hours of admission to hospital
Initial assessment
Clinical impression of severity
Body mass index >30
Pleural effusion
APACHE score >8
24 hours after admission
Clinical impression of severity
APACHE II >8
Glasgow score of 3 or more
Persisting multiple organ failure
CRP>150
48 hours after admission
Glasgow Score of >3
CRP >150
Persisting or progressive organ failure
Table adapted from UK guidelines for management of acute pancreatitis. GUT 2005, 54 suppl III

Management

Nutrition
There is reasonable evidence to suggest that the use of enteral nutrition does not worsen the outcome in pancreatitis
Most trials to date were underpowered to demonstrate a conclusive benefit.
The rationale behind feeding is that it helps to prevent bacterial translocation from the gut, thereby contributing to the development of infected pancreatic necrosis.

Use of antibiotic therapy
Many UK surgeons administer antibiotics to patients with acute pancreatitis. However, there is very little evidence to support this practice.
A recent Cochrane review highlights the potential benefits of administering Imipenem to patients with established pancreatic necrosis in the hope of averting the progression to infection.
There are concerns that the administration of antibiotics in mild attacks of pancreatitis will not affect outcome and may contribute to antibiotic resistance and increase the risks of antibiotic associated diarrhoea.

Surgery
Patients with acute pancreatitis due to gallstones should undergo early cholecystectomy.
Patients with obstructed biliary system due to stones should undergo early ERCP.
Patients with extensive necrosis where infection is suspected should usually undergo FNA for culture.
Patients with infected necrosis should undergo either radiological drainage or surgical necrosectomy. The choice of procedure depends upon local expertise.

References
www.bsg.org.uk/pdfworddocs/pancreatic.pdf

Antibiotic therapy for prophylaxis against infection of pancreatic necrosis in acute pancreatitis. Villatoro et al. Cochrane Library DOI: 10.1002/14651858.CD002941.pub3. 2010 version.

How well did you know this?
1
Not at all
2
3
4
5
Perfectly
645
Q

You are working as an anatomy demonstrator and the medical students decide to test your knowledge on the Circle of Willis. Which of the following comments is false?

The anterior communicating artery links the right and left sides

Asymmetry of the circle of willis is a risk factor for the development of intracranial aneurysms

Majority of blood passing through the vessels mix together

Includes the anterior communicating artery

The circle surrounds the stalk of the pituitary gland

A

There is minimum mixing of blood passing through the vessels.

Circle of Willis

The two internal carotid arteries and two vertebral arteries form an anastomosis known as the Circle of Willis on the inferior surface of the brain. Each half of the circle is formed by:
1. Anterior communicating artery
2. Anterior cerebral artery
3. Internal carotid artery
4. Posterior communicating artery
5. Posterior cerebral arteries and the termination of the basilar artery

The circle and its branches supply; the corpus striatum, internal capsule, diencephalon and midbrain.

Vertebral arteries
Enter the cranial cavity via foramen magnum
Lie in the subarachnoid space
Ascend on anterior surface of medulla oblongata
Unite to form the basilar artery at the base of the pons

Branches:
Posterior spinal artery
Anterior spinal artery
Posterior inferior cerebellar artery

Basilar artery
Branches:
Anterior inferior cerebellar artery
Labyrinthine artery
Pontine arteries
Superior cerebellar artery
Posterior cerebral artery (at the point where it bifurcates)

Internal carotid arteries
Branches:
Posterior communicating artery
Anterior cerebral artery
Middle cerebral artery
Anterior choroid artery

How well did you know this?
1
Not at all
2
3
4
5
Perfectly
646
Q

A 65 year old lady is admitted with large bowel obstruction. On investigation with CT, she is found to have a tumour of the mid rectum with no evidence of metastatic disease. What is the most appropriate course of action?

Formation of a loop colostomy

Laparotomy and Hartmanns procedure

Pan proctocolectomy and end ileostomy

Low anterior resection and covering loop ileostomy

Low anterior resection and end colostomy

A

Avoid emergency resections in large bowel obstruction due to rectal cancer
This patient has presented with large bowel obstruction. However, in the case of rectal cancer, she is incompletely staged as ability to completely resect the lesion can only be determined with MRI scanning and this information is not provided. Even if the lesion were resectable, in the emergency setting, it is often safer to undertake a simple procedure such as a loop colostomy and then complete surgery at a later date. A low anterior resection and loop ileostomy in this situation would almost certainly leak (and for the reasons outlined above, may be incomplete).

Large bowel obstruction

Colonic obstruction remains a common surgical problem. It is most commonly due to malignancy (60%) and diverticular disease (20%). Volvulus affecting the colon accounts for 5% of cases. Acute colonic pseudo-obstruction remains a potential differential diagnosis in all cases. Intussusception affecting the colon (most often due to tumours in the adult population) remains a rare but recognised cause.
The typical patient will present with gradual onset of progressive abdominal distension, colicky abdominal pain and either obstipation or absolute constipation.
On examination abdominal distension is present, the presence of caecal tenderness (assuming no overt evidence of peritonitis) is a useful sign to elicit. A digital rectal examination and rigid sigmoidoscopy should be performed.
A plain abdominal x-ray is the usual first line test and; the caecal diameter and ileocaecal valve competency should be assessed on this film.

Imaging modalities
Debate long surrounds the use of CT versus gastrograffin enemas. The latter investigation has always been the traditional method of determining whether a structural lesion is indeed present. However, in the UK the use of this technique has declined and in most units a CT scan will be offered as the first line investigation by the majority of radiologists (and is advocated by the ACPGBI). In most cases this will provide sufficient detail to allow operative planning, and since malignancy accounts for most presentations may also stage the disease. In the event that the radiologist cannot provide a clear statement of lesion site, the surgeon should have no hesitation in requesting a contrast enema.

Surgical options
The decision as to when to operate or not is determined firstly by the patients physiological status. Unstable patients require resuscitation prior to surgery and admission to a critical care unit for invasive monitoring and potential inotropic support may be needed. In patients who are otherwise stable the decision then rests on the radiological and clinical findings. As a general rule the old adage that the sun should not rise and set on unrelieved large bowel obstruction still holds true. A caecal diameter of 12cm or more in the presence of complete obstruction with a competent ileocaecal valve and caecal tenderness is a sign of impending perforation and a relative indication for prompt surgery.

Right sided and transverse lesions
Right sided lesions producing large bowel obstruction should generally be treated by right hemicolectomy or its extended variant if the lesion lies in the distal transverse colon or splenic flexure. In these cases an ileocolic anastomosis may be easily constructed and even in the emergency setting has a low risk of anastomotic leak.

Left sided lesions
The options here lie between sub total colectomy and anastomosis, left hemicolectomy with on table lavage and primary anastomosis, left hemicolectomy and end colostomy formation and finally colonic stent insertion.
The usefulness of colonic stents was the subject of a Cochrane review in 2011. The authors concluded that on the basis of the data that they reviewed, there was no benefit from the use of colonic stents over conventional surgical resection with a tendency to better outcomes seen in the surgical group (1). A more recently conducted meta analysis met with the same conclusion (2). However, the recently concluded CREST trial has suggested that self expanding metallic stents can improve outcomes and avoids a stoma.

Rectosigmoid lesions
Lesions below the peritoneal reflection that are causing obstruction should generally be treated with a loop colostomy. Primary resection of unstaged rectal cancer would most likely carry a high CRM positivity rate and cannot be condoned. Where the lesion occupies the distal sigmoid colon the usual practice would be to perform a high anterior resection. The decision surrounding restoration of intestinal continuity would lie with the operating surgeon.

References
1. Sagar J. Colorectal stents for the management of malignant colonic obstructions. Cochrane Database of Systematic Reviews 2011, Issue 11. Art. No.: CD007378. DOI: 10.1002/14651858.CD007378.pub2.
2. Cirrochi et al Safety and efficacy of endoscopic colonic stenting as a bridge to surgery in the management of intestinal obstruction due to left colon and rectal cancer: A systematic review and meta-analysis. Surg Oncol. 2013 Mar;22(1):14-21.

How well did you know this?
1
Not at all
2
3
4
5
Perfectly
647
Q

During a gangland gunfight a man is shot in the chest. The bullet passes through the posterior mediastinum (from left to right). Which of the following structures is least likely to be injured

Thoracic duct

Oesophagus

Vagus nerve

Descending thoracic aorta

Arch of the azygos vein

A

The arch of the azygos vein lies in the middle mediastinum.

Mediastinum

Region between the pulmonary cavities.
It is covered by the mediastinal pleura. It does not contain the lungs.
It extends from the thoracic inlet superiorly to the diaphragm inferiorly.

Mediastinal regions
Superior mediastinum (between manubriosternal angle and T4/5)
Middle mediastinum
Posterior mediastinum
Anterior mediastinum

Region Contents
Superior mediastinum
Superior vena cava
Brachiocephalic veins
Arch of aorta
Thoracic duct
Trachea
Oesophagus
Thymus
Vagus nerve
Left recurrent laryngeal nerve
Phrenic nerve
Anterior mediastinum
Thymic remnants
Lymph nodes
Fat
Middle mediastinum
Pericardium
Heart
Aortic root
Arch of azygos vein
Main bronchi
Posterior mediastinum
Oesophagus
Thoracic aorta
Azygos vein
Thoracic duct
Vagus nerve
Sympathetic nerve trunks
Splanchnic nerves

How well did you know this?
1
Not at all
2
3
4
5
Perfectly
648
Q

A 43 year old female develops severe chest wall cellulitis following a mastectomy. On examination, the skin is markedly erythematous. Which of the acute inflammatory mediators listed below is least likely to produce vasodilation at this site?

Complement component C5a

Lysosomal compounds

Histamine

Serotonin

Prostaglandins

A

Erythema is a classical feature of acute inflammation. Potent mediators of vascular dilatation include; histamine, prostaglandins, nitric oxide, platelet activating factor, complement C5a (and C3a) and lysosomal compounds. Although serotonin is associated with acute inflammation it is a vasoconstrictor. The effects of serotonin are dependent upon the state of the vessels in the tissues. Intact and healthy tissues and vessels will respond to a serotonin infusion with vasodilation (hence the flushing seen in carcinoid syndrome). In contrast it worsens cardiac ischaemia in myocardial infarcts when released from damaged platelets.

Acute inflammation

Inflammation is the reaction of the tissue elements to injury. Vascular changes occur, resulting in the generation of a protein rich exudate. So long as the injury does not totally destroy the existing tissue architecture, the episode may resolve with restoration of original tissue architecture.

Vascular changes
Vasodilation occurs and persists throughout the inflammatory phase.
Inflammatory cells exit the circulation at the site of injury.
The equilibrium that balances Starlings forces within capillary beds is disrupted and a protein rich exudate will form as the vessel walls also become more permeable to proteins.
The high fibrinogen content of the fluid may form a fibrin clot. This has several important immunomodulatory functions.

Sequelae
Resolution
Typically occurs with minimal initial injury
Stimulus removed and normal tissue architecture results
Organisation
Delayed removal of exudate
Tissues undergo organisation and usually fibrosis
Suppuration
Typically formation of an abscess or an empyema
Sequestration of large quantities of dead neutrophils
Progression to chronic inflammation
Coupled inflammatory and reparative activities
Usually occurs when initial infection or suppuration has been inadequately managed

Causes
Infections e.g. Viruses, exotoxins or endotoxins released by bacteria
Chemical agents
Physical agents e.g. Trauma
Hypersensitivity reactions
Tissue necrosis

Presence of neutrophil polymorphs is a histological diagnostic feature of acute inflammation

How well did you know this?
1
Not at all
2
3
4
5
Perfectly
649
Q

The vertebral artery traverses all of the following except?

Transverse process of C6

Transverse process of the axis

Vertebral canal

Foramen magnum

Intervertebral foramen

A

The vertebral artery passes through the foramina which are located in the transverse processes of the cervical vertebra, it does not traverse the intervertebral foramen.

Vertebral artery

The vertebral artery is the first branch of the subclavian artery. Anatomically it is divisible into 4 regions:
The first part runs to the foramen in the transverse process of C6. Anterior to this part lies the vertebral and internal jugular veins. On the left side the thoracic duct is also an anterior relation.
The second part runs superiorly through the foramina of the the transverse processes of the upper 6 cervical vertebrae. Once it has passed through the transverse process of the axis it then turns superolaterally to the atlas. It is accompanied by a venous plexus and the inferior cervical sympathetic ganglion.
The third part runs posteromedially on the lateral mass of the atlas. It enters the sub occipital triangle, in the groove of the upper surface of the posterior arch of the atlas. It then passes anterior to the edge of the posterior atlanto-occipital membrane to enter the vertebral canal.
The fourth part passes through the spinal dura and arachnoid, running superiorly and anteriorly at the lateral aspect of the medulla oblongata. At the lower border of the pons it unites to form the basilar artery.

How well did you know this?
1
Not at all
2
3
4
5
Perfectly
650
Q

A 19 year old motorcyclist is involved in a road traffic accident. His chest movements are irregular. He is found to have multiple rib fractures, with 2 fractures in the 3rd rib and 3 fractures in the 4th rib. What is the underlying diagnosis?

Simple rib fractures

Flail chest injury

Cardiac tamponade

Pneumothorax

Aortic rupture

A

Multiple rib fractures with > or = 2 rib fractures in more than 2 ribs is diagnosed as a flail chest. This is associated with pulmonary contusion.

Thoracic trauma

Key points related to thoracic trauma
Less than 10% of blunt chest trauma and 15-30% of penetrating chest trauma requires operative intervention.
The physiologic consequences of thoracic trauma are hypoxia, hypercarbia, and acidosis. Contusion, hematoma, and alveolar collapse, or changes in intrathoracic pressure relationships (e.g., tension pneumothorax and open pneumothorax) cause hypoxia and lead to metabolic acidosis. Hypercarbia causes respiratory acidosis and most often follows inadequate ventilation caused by changes in intrathoracic pressure relationships and depressed level of consciousness.

Types of thoracic trauma

Tension pneumothorax
Often laceration to lung parenchyma with flap
Pressure develops in thorax
Most common cause is mechanical ventilation in patient with pleural injury
Symptoms overlap with cardiac tamponade, hyper-resonant percussion note is more likely in tension pnemothorax
Flail chest
Chest wall disconnects from thoracic cage
Multiple rib fractures (at least two fractures per rib in at least two ribs)
Associated with pulmonary contusion
Abnormal chest motion
Avoid over hydration and fluid overload
Pneumothorax
Most common cause is lung laceration with air leakage
Most traumatic pneumothoraces should have a chest drain
Patients with traumatic pneumothorax should never be mechanically ventilated until a chest drain is inserted
Haemothorax
Most commonly due to laceration of lung, intercostal vessel or internal mammary artery
Haemothoraces large enough to appear on CXR are treated with large bore chest drain
Surgical exploration is warranted if >1500ml blood drained immediately
Cardiac tamponade
Beck’s triad: elevated venous pressure, reduced arterial pressure, reduced heart sounds
Pulsus paradoxus
May occur with as little as 100ml blood
Pulmonary contusion
Most common potentially lethal chest injury
Arterial blood gases and pulse oximetry important
Early intubation within an hour if significant hypoxia
Blunt cardiac injury
Usually occurs secondary to chest wall injury
ECG may show features of myocardial infarction
Sequelae: hypotension, arrhythmias, cardiac wall motion abnormalities
Aorta disruption
Deceleration injuries
Contained haematoma
Widened mediastinum
Diaphragm disruption
Most due to motor vehicle accidents and blunt trauma causing large radial tears (laceration injuries result in small tears)
More common on left side
Insert gastric tube, may pass into intrathoracic stomach
Mediastinal traversing wounds
Entrance wound in one hemithorax and exit wound/foreign body in opposite hemithorax
Mediastinal haematoma or pleural cap suggests great vessel injury
Mortality is 20%

How well did you know this?
1
Not at all
2
3
4
5
Perfectly
651
Q

During short saphenous vein surgery for varicose veins which of the following nerves is particularly at risk?

Sural nerve

Popliteal nerve

Tibial nerve

Femoral nerve

Saphenous nerve

A

Saphenous vein

Long saphenous vein
This vein may be harvested for bypass surgery, or removed as treatment for varicose veins with saphenofemoral junction incompetence.

Originates at the 1st digit where the dorsal vein merges with the dorsal venous arch of the foot
Passes anterior to the medial malleolus and runs up the medial side of the leg
At the knee, it runs over the posterior border of the medial epicondyle of the femur bone
Then passes laterally to lie on the anterior surface of the thigh before entering an opening in the fascia lata called the saphenous opening
It joins with the femoral vein in the region of the femoral triangle at the saphenofemoral junction

Tributaries
Medial marginal
Superficial epigastric
Superficial iliac circumflex
Superficial external pudendal veins

Short saphenous vein
Originates at the 5th digit where the dorsal vein merges with the dorsal venous arch of the foot, which attaches to the great saphenous vein.
It passes around the lateral aspect of the foot (inferior and posterior to the lateral malleolus) and runs along the posterior aspect of the leg (with the sural nerve)
Passes between the heads of the gastrocnemius muscle, and drains into the popliteal vein, approximately at or above the level of the knee joint.

How well did you know this?
1
Not at all
2
3
4
5
Perfectly
652
Q

Where are sarcomas most likely to be found?

In the thorax

In the abdomen

Around the axial spine

In the extremities

In the head and neck

A

40% of sarcomas are found in the extremities.

Sarcomas

Malignant tumours of mesenchymal origin

Types
May be either bone or soft tissue in origin.
Bone sarcoma include:
Osteosarcoma
Ewings sarcoma (although non bony sites recognised)
Chondrosarcoma - originate from Chondrocytes

Soft tissue sarcoma are a far more heterogeneous group and include:
Liposarcoma-adipocytes
Rhabdomyosarcoma-striated muscle
Leiomyosarcoma-smooth muscle
Synovial sarcomas- close to joints (cell of origin not known but not synovium)

Malignant fibrous histiocytoma is now referred to as undifferentiated pleomorphic sarcoma. Careful histological assessment of lesions now allows more accurate categorisation of sarcoma subtypes than was previously possible.

Features
Certain features of a mass or swelling should raise suspicion for a sarcoma these include:
Large >5cm soft tissue mass
Deep tissue location or intra muscular location
Rapid growth
Painful lump

Assessment
Imaging of suspicious masses should utilise a combination of MRI, CT and USS. Blind biopsy should not be performed prior to imaging and where required should be done in such a way that the biopsy tract can be subsequently included in any resection.

Ewings sarcoma
Commoner in males
Incidence of 0.3 / 1, 000, 000
Onset typically between 10 and 20 years of age
Location by femoral diaphysis is commonest site
Histologically it is a small round tumour
Blood borne metastasis is common and chemotherapy is often combined with surgery

Osteosarcoma
Mesenchymal cells with osteoblastic differentiation
20% of all primary bone tumours
Incidence of 5 per 1,000,000
Peak age 15-30, commoner in males
Limb preserving surgery may be possible and many patients will receive chemotherapy

Liposarcoma
Malignancy of adipocytes
Rare, approximately 2.5 per 1,000,000. They are the most common soft tissue sarcoma
Typically located in deep locations such as retroperitoneum
Affect older age group usually >40 years of age
May be well differentiated and thus slow growing although may undergo de-differentiation and disease progression
Many tumours will have a pseudocapsule that can misleadingly allow surgeons to feel that they can ‘shell out’ these lesions. In reality, tumour may invade at the edge of the pseudocapsule and result in local recurrence if this strategy is adopted
Usually resistant to radiotherapy, although this is often used in a palliative setting

Malignant Fibrous Histiocytoma
Tumour with large number of histiocytes
Also described as undifferentiated pleomorphic sarcoma NOS (i.e. Cell of origin is not known)
Four major subtypes are recognised: storiform-pleomorphic (70% cases), myxoid (less aggressive), giant cell and inflammatory
Treatment is usually with surgical resection and adjuvant radiotherapy as this reduces the likelihood of local recurrence

How well did you know this?
1
Not at all
2
3
4
5
Perfectly
653
Q

A 30 year old male is hit on the side of the head with a bat. He now presents to Emergency Department with odd behaviour and complaining of a headache. Whilst waiting for a CT scan he becomes drowsy and unresponsive. What is the most likely underlying injury?

Intra cerebral haematoma

Sub dural haematoma

Extra dural haematoma

Intraventricular haemorrhage

Sub arachnoid haemorrhage

A

Extra dural haematoma
The middle meningeal artery is prone to damage when the temporal side of the head is hit.
Note that there may NOT be any initial LOC or lucid interval.

Head injury

Patients who suffer head injuries should be managed according to ATLS principles and extra cranial injuries should be managed alongside cranial trauma. Inadequate cardiac output will compromise CNS perfusion irrespective of the nature of the cranial injury.

Types of traumatic brain injury
Extradural haematoma Bleeding into the space between the dura mater and the skull. Often results from acceleration-deceleration trauma or a blow to the side of the head. The majority of extradural haematomas occur in the temporal region where skull fractures cause a rupture of the middle meningeal artery.

Features
Raised intracranial pressure
Some patients may exhibit a lucid interval
Subdural haematoma Bleeding into the outermost meningeal layer. Most commonly occur around the frontal and parietal lobes. May be either acute or chronic.

Risk factors include old age and alcoholism.

Slower onset of symptoms than a extradural haematoma.
Subarachnoid haemorrhage Usually occurs spontaneously in the context of a ruptured cerebral aneurysm, but may be seen in association with other injuries when a patient has sustained a traumatic brain injury.

Pathophysiology
Primary brain injury may be focal (contusion/ haematoma) or diffuse (diffuse axonal injury)
Diffuse axonal injury occurs as a result of mechanical shearing following deceleration, causing disruption and tearing of axons
Intra-cranial haematomas can be extradural, subdural or intracerebral, while contusions may occur adjacent to (coup) or contralateral (contre-coup) to the side of impact
Secondary brain injury occurs when cerebral oedema, ischaemia, infection, tonsillar or tentorial herniation exacerbates the original injury. The normal cerebral auto regulatory processes are disrupted following trauma rendering the brain more susceptible to blood flow changes and hypoxia
The Cushings reflex (hypertension and bradycardia) often occurs late and is usually a pre terminal event

Management
Where there is life threatening rising ICP such as in extra dural haematoma and whilst theatre is prepared or transfer arranged use of IV mannitol/ frusemide may be required.
Diffuse cerebral oedema may require decompressive craniotomy
Exploratory Burr Holes have little management in modern practice except where scanning may be unavailable and to thus facilitate creation of formal craniotomy flap
Depressed skull fractures that are open require formal surgical reduction and debridement, closed injuries may be managed non operatively if there is minimal displacement.
ICP monitoring is appropriate in those who have GCS 3-8 and normal CT scan.
ICP monitoring is mandatory in those who have GCS 3-8 and abnormal CT scan.
Hyponatraemia is most likely to be due to syndrome of inappropriate ADH secretion.
Minimum of cerebral perfusion pressure of 70mmHg in adults.
Minimum cerebral perfusion pressure of between 40 and 70 mmHg in children.

Interpretation of pupillary findings in head injuries
Pupil size Light response Interpretation
Unilaterally dilated Sluggish or fixed 3rd nerve compression secondary to tentorial herniation
Bilaterally dilated Sluggish or fixed
Poor CNS perfusion
Bilateral 3rd nerve palsy
Unilaterally dilated or equal Cross reactive (Marcus - Gunn) Optic nerve injury
Bilaterally constricted May be difficult to assess
Opiates
Pontine lesions
Metabolic encephalopathy
Unilaterally constricted Preserved Sympathetic pathway disruption

How well did you know this?
1
Not at all
2
3
4
5
Perfectly
654
Q

Which of the following statements relating to quadratus lumborum is false?

Causes flexion of the thoracic spine

Causes the rib cage to be pulled down

Innervated by anterior primary rami of T12 and L1-3

Attached to the iliac crest

Inserts into the 12th rib

A

Quadratus lumborum
Origin: Medial aspect of iliac crest and iliolumbar ligament
Insertion: 12th rib
Action: Pulls the rib cage inferiorly. Lateral flexion.
Nerve supply: Anterior primary rami of T12 and L1-3
The rectus abdominis causes flexion of the thoracic spine and therefore the statement suggesting that quadratus lumborum does so is incorrect.

Abdominal wall

The 2 main muscles of the abdominal wall are the rectus abdominis (anterior) and the quadratus lumborum (posterior).
The remaining abdominal wall consists of 3 muscular layers. Each muscle passes from the lateral aspect of the quadratus lumborum posteriorly to the lateral margin of the rectus sheath anteriorly. Each layer is muscular posterolaterally and aponeurotic anteriorly.

Muscles of abdominal wall
External oblique
Lies most superficially
Originates from 5th to 12th ribs
Inserts into the anterior half of the outer aspect of the iliac crest, linea alba and pubic tubercle
More medially and superiorly to the arcuate line, the aponeurotic layer overlaps the rectus abdominis muscle
The lower border forms the inguinal ligament
The triangular expansion of the medial end of the inguinal ligament is the lacunar ligament.
Internal oblique
Arises from the thoracolumbar fascia, the anterior 2/3 of the iliac crest and the lateral 2/3 of the inguinal ligament
The muscle sweeps upwards to insert into the cartilages of the lower 3 ribs
The lower fibres form an aponeurosis that runs from the tenth costal cartilage to the body of the pubis
At its lowermost aspect it joins the fibres of the aponeurosis of transversus abdominis to form the conjoint tendon.
Transversus abdominis
Innermost muscle
Arises from the inner aspect of the costal cartilages of the lower 6 ribs , from the anterior 2/3 of the iliac crest and lateral 1/3 of the inguinal ligament
Its fibres run horizontally around the abdominal wall ending in an aponeurosis. The upper part runs posterior to the rectus abdominis. Lower down the fibres run anteriorly only.
The rectus abdominis lies medially; running from the pubic crest and symphysis to insert into the xiphoid process and 5th, 6th and 7th costal cartilages. The muscles lies in a aponeurosis as described above.
Nerve supply: anterior primary rami of T7-12

Surgical notes
During abdominal surgery it is usually necessary to divide either the muscles or their aponeuroses. During a midline laparotomy it is desirable to divide the aponeurosis. This will leave the rectus sheath intact above the arcuate line and the muscles intact below it. Straying off the midline will often lead to damage to the rectus muscles, particularly below the arcuate line where they may often be in close proximity to each other.

How well did you know this?
1
Not at all
2
3
4
5
Perfectly
655
Q

A 23 year old man who plays rugby for a hobby presents with recurrent anterior dislocation of the shoulder. Which of the following abnormalities is most likely to be present to account for this?

Rotator cuff tear

Biceps tendon rupture

Bankart lesion

Axillary nerve injury

Infraspinatus tendinitis

A

A Bankart lesion is an injury of the anterior (inferior) glenoid labrum of the shoulder due to anterior shoulder dislocation. When this happens, a pocket at the front of the glenoid forms that allows the humeral head to dislocate into it.
Anterior dislocations are the most common. When recurrent, a Bankart lesion is the most common underlying abnormality. This is usually visualised by CT and MRI scanning and often repaired arthroscopically.

Shoulder disorders

Shoulder fractures and dislocations
Fractures
Proximal humerus
Background
Third most common fragility fracture in the elderly.
Results from low energy fall in predominantly elderly females, or from high energy trauma in young males.
Can be associated with nerve injury (commonly axillary), and fracture-dislocation of the humeral head. Detailed neurological assessment is essential for all upper limb injuries.

Anatomy
Osteology
Consists of articular head, greater tuberosity, lesser tuberosity, metaphysis and diaphysis. Between the articular head and the tuberosities is the anatomical neck (previous physis). Between the tuberosities and the metaphysis is the surgical neck.
The supraspinatus, infraspinatus and teres minor muscles attach to the greater tuberosity. The subscapularis muscle attaches to the lesser tuberosity.

Vascular Supply
Humeral head is supplied by the anterior and posterior humeral circumflex arteries. Anatomical neck fractures are at greatest risk of osteonecrosis.

Imaging
Imaging aims to both delineate the fracture pattern, and confirm/exlude the presence of an associated dislocation.
Radiographs - True anteroposterior (AP), axillary lateral and/or scapula Y view.
CT - indicated to better define intra-articular involvement and to aid pre-operative planning. MRI is not useful for fracture imaging.

Classification
Description of the fracture is often more useful than classification. Particular attention should be paid to humeral alignment, fracture displacement, and greater tuberosity position (rotator cuff will pull the GT supero-posterioly, which can cause impingement problems with malunion).
- Neer Classification: Most commonly used. Describes fracture as 2,3,or 4 part depending upon the number main fragments. Also comments on the degree of displacement. Fragments:
-greater tuberosity
-lesser tuberosity
- articular surface
- shaft
Displacement: >1cm or angulation >45 degrees.

Treatment
The vast majority of proximal humeral fractures are minimally displaced, and therefore can be managed conservatively. This involves immobilisation in a polysling, and progressive mobilisation. Pendular exercise can commence at 14 days, and active abduction from 4-6 weeks.

Irreducible fracture dislocation is an indication for operative management. Other indications include large displacement, younger patient, head splitting (intra-articular fractures). However, the recent PROFHER trial (1) has suggested no benefit to operative intervention on patient outcome (it must be applied cautiously as majority of patients were elderly with extraarticular fractures). Options available for surgical management include:

ORIF Most commonly used. Plate and screw fixation. Can reconstruct complex fractures.
Intramedullary nail Suitable for extra-articular configuration, predominantly surgical neck +/- GT fractures.
Hemiarthroplasty Used for un-reconstructable fractures in the older patient who has good glenoid quality.
Total shoulder arthroplasty Unconstructable fractures where high functioning shoulder is required (hemiarthroplasty will cause glenoid erosion)
Reverse shoulder arthroplasty Total shoulder arthroplasty that provides better functional outcome than conventional total shoulder replacement.

Scapula
Background
Uncommon fractures usually associated with high energy trauma. Most commonly involve scapula body or spine (50%), glenoid fossa and glenoid neck. Important to exclude associated life threatening injury.

Imaging
Plain radiographs should include true anteroposterior (AP), axillary lateral and/or scapula Y view. CT scanning is useful for defining intra-articular involvement, displacement and for three dimensional reconstruction.

Classification
Based on the location of the fracture (coracoid, acromion, glenoid neck, glenoid fossa, scapula body). Beware of ipsilateral glenoid neck and clavicle fracture -floating shoulder - where limb is effectively dissociated from axial skeleton.

Treatment
The vast majority of scapula fractures are amenable to conservative management, consisting of sling immobilisation for two weeks followed by early rehabilitation. Floating shoulder will usually require fixation, and consideration of surgery should also be given to intra-articular and displaced/angulated glenoid fractures.

Dislocations

Types
Dislocations around the shoulder joint include glenohumeral dislocation, acromioclavicular joint disruption and sternoclavicular dislocation. Only glenohumeral dislocation will be covered here.

Glenohumeral dislocation
Diagnosis, classification and management are covered here.

Background
Shoulder dislocation is commonly seen in A&E. It has a high recurrence rate that is as high as 80% in teenagers. Initial management requires emergent reduction to prevent lasting chondral damage.

Early assessment and management
Usually a traumatic cause (multi-directional instability in frequent dislocations requires discussion with orthopaedics and is not covered here). Careful history, examination and documentation of neurovascular status of the limb, in particular the axillary nerve (regimental badge sensation). This should be re-assessed post manipulation. Early radiographs to confirm direction of dislocation.

Initial management consists of emergent closed reduction under under entanox and analgesia, but often requires conscious sedation. Arm should then be immobilised in a polysling, and XR to confirm relocation.

Imaging - True anteroposterior (AP), axillary lateral and/or scapula Y view. Reduced humeral head should lie between acromion and coracoid on lateral/scapula view.

Types
Direction Features Cause Examination Reduction techniques
Anterior Most Common >90% Usually traumatic - anterior force on arm when shoulder is abducted, externally rotated Loss of shoulder contour - sulcus sign. Humeral head can be felt anteriorly.
Hippocratic.
Milch.
Stimson.

Kocher not advised due to complication of fracture
Posterior 50% missed in A&E 50% traumatic, but classically post seizure or electrocution Shoulder locked in internal rotation. XR may show lightbulb appearance. Gentle lateral traction to adducted arm.
Inferior Rare Associated with pectorals and rotator cuff tears, and glenoid fracture As for primary injury Management of primary injury
Superior Rare Associated with acrominon/clavicle fracture As for primary injury Management of primary injury

Associated injuries
Bankart lesion - avulsion of the anterior glenoid labrum with an anterior shoulder dislocation (reverse Bankart if poster labrum in posterior dislocation).
Hill Sachs defect - chondral impaction on posteriosuperior humeral head from contact with gleonoid rim. Can be large enough to lock shoulder, requiring open reduction. (Reverse Hill Sachs in posterior dislocation).
Rotator cuff tear - increases with age.
Greater or lesser tuberosity fracture - increases with age.
Humeral neck fracture - shoulder fracture dislocation. More common in high energy trauma and elderly. Should be discussed with orthopaedics prior to any attempted reduction.

Rotator Cuff Disease

Rotator cuff disease is a spectrum of conditions that ranges from subacromial impingement to rotator cuff tears and eventually to rotator cuff arthropathy (arthritis).

Anatomy
The rotator cuff is a group of four muscles that are important in shoulder movements, and maintenance of glenohumeral stability.
Muscle Scapular attachment Humeral attachment Action Innervation
Supraspinatus Supraspinatus fossa Superior facet of greater tuberosity Initiation of abduction of humerus Suprascapular nerve
Infraspinatus Infraspinatus fossa Posterior facet of greater tuberosity External rotation of humerus Suprascapular nerve
Teres Minor Lateral border Inferior facet of greater tuberosity External rotation of humerus Axillary Nerve
Subscapularis Subscapular fossa Lesser tuberosity Internal rotation of humerus Upper and lower subscapular nerve

The inferior rotator cuff muscles (infraspinatus, teres minor, and subscapularis) balance the superior pull of the deltoid. Injury/tear results in upward migration of the humeral head on the glenoid (can be seen on AP radiograph).
Likewise, the anterior muscles (subscapularis) are balanced with the posterior muscles (infraspinatus, teres minor).

Subacromial Impingement

The most common cause of shoulder pain, which results from impingement of the superior cuff on the undersurface of the acromion, and an inflammatory bursitis.
Associated with certain types of acromial morphology (Bigliani classification).
Presents as insidious pain which is exacerbated by overhead activities.

Rotator Cuff Tear

Often presents as an acute event on the background of chronic subacromial impingement in the older patient, but can present as an avulsion injury in younger patients.
Majority of tears are to the superior cuff (supraspinatus, infraspinatus, teres minor), though a tear to subscapularis is associated with subcoracoid impingement.
Tears present as pain and weakness when using the muscles in question.

Rotator Cuff Arthropathy

Defined as shoulder arthritis in the setting of rotator cuff dysfunction. Results from superior migration due to the loss of rotator cuff function and integrity. Unopposed deltoid pulls the humeral head superiorly.
Associated with massive chronic cuff tears.

Imaging

Plain radiographs
AP of the shoulder may show superior migration of the humerus with a cuff tear, and features of arthritis with arthropathy. Other causes of pain may also be identified (e.g. calcific tendonitis/fracture)
Outlet view is useful for defining the acromial morphology

USS
Allows dynamic imaging of the cuff, and is inexpensive. However, it is very user dependent.

MRI
Best imaging modality for cuff pathology.
Also allows imaging of the rest of the shoulder. When intra-articular pathology is suspected, can be combined with an arthrogram for improved sensitivity and specificity.

Treatment

Subacromial impingement
Physiotherapy, oral anti-inflammatory medication
Subacromial steroid injection can settle inflammation
Arthroscopic subacromial decompression by shaving away the undersurface of the acromion, more space is created for the rotator cuff. Cuff integrity is assessed also at time of surgery, and can be repaired if necessary.

Rotator cuff tear
When considering repair of a cuff tear, the age and activity of the patient, the nature of the tear (degenerative vs. acute traumatic), and the size and retraction of the tear should be considered when making a surgical plan.
Mild tears or tears in the elderly can be managed conservatively, as outlined above.
Moderate tears can be repaired arthroscopically. Massive or retracted tears will often require an open repair (occasionally with a tendon transfer). Subacromial decompression is performed at the same time to reduce impingement, symptoms and recurrence.

Calcific tendonitis
Calcific tendonitis involves calcific deposits within tendons anywhere in the body, but most commonly in the rotator cuff (specifically the supraspinatus tendon). When present in the shoulder, it is associated with subacromial impingement and pain.

Pathology
More common in women aged 30-60 years.
Association with diabetes and hypothyroidism

There are three stages of calcification
Formative phase characterized by calcific deposits
Resting phase deposit is stable, but presents with impingement problems
Resorptive phase phagocytic resorption. Most painful stage.

Presentation
Similar in presentation to subacromial impingement, with pain especially with over head activities. Atraumatic in nature.

Imaging
Plain radiographs show calcification of the rotator cuff, usually within 1.5cm of its insertion on the humerus. Supraspinatus outlet views can show level of impingment. Further imaging is rarely needed.

Treatment
Non-operative NSAIDS, steroid injection (controversial, but practiced) and physiotherapy. Approximately 75% will resolve by 6 months with conservative management.
Ultrasound guided or surgical needle barbotage can break down deposits and resolve symptoms. Occasionally surgical excision is required.

Adhesive capsulitis (Frozen Shoulder)
Pain and loss of movement of shoulder joint, which involves fibroplastic proliferation of capsular tissue, causing soft tissue scarring and contracture. Patients present with a painful and decreased arc of motion.
Associated with prolonged immobilization, previous surgery, thyroid disorders (AI) and diabetes
Classically three stages which can take up to two years to resolve:
Stage one the freezing and painful stage
Stage two the frozen and stiff stage
Stage three the thawing stage, where shoulder movement slowly improves

Imaging
Plain radiographs to exclude other causes of a painful shoulder
MRI arthrogram may show capsular contracture, and again may be used to exclude cuff pathology. However, often not performed as diagnosis is largely clinical.

Treatment
Non-operative NSAIDS, steroid injection and physiotherapy. Patience is required as condition can take up to 2 years to improve.
Operative MUA or arthroscopic adhesiolysis (release of adhesions) can expedite recovery, followed by intensive physiotherapy.

Glenohumeral Arthritis
Background
May be osteoarthritis (primary or secondary to cuff tear or trauma), rheumatoid arthritis, or as part of a spondyloarthropathy. Majority of those with RA will develop symptoms.
More common in the elderly
Presents like any other arthritis - pain at night and with movement

Imaging
AP and axillary radiographs will show features of arthritis.
CT/MRI is often useful to classify the shape of the glenoid and extent of bone loss when considering arthroplasty. MRI also essential to asses integrity of rotator cuff if considering shoulder replacement.

Treatment
Like all orthopaedics, start with simple measures:
NSAIDS, management of RA, physiotherapy, steroid injection.
Hemiarthroplasty can sometimes be considered if glenoid is in excellent condition or if patient has large comorbidity.
Arthroscopic debridement is useful if patient has isolated ACJ arthritis, but is rarely used for glenohumeral arthritis.
Total shoulder replacement is shown to produce superior outcome when compared to hemiarthroplasty in terms of pain relief, function and implant survival.
Total shoulder replacement can be anatomical (ball on humerus, with cup on glenoid), or reverse geometry (ball on glenoid, with cup on humerus). Anatomical TSR requires an in tact rotator cuff, so often reverse is preferable when the cuff if questionable in integrity.

References
1. JAMA. 2015;313(10):1037-1047. doi:10.1001/jama.2015.1629

How well did you know this?
1
Not at all
2
3
4
5
Perfectly
656
Q

A 32 year old attends neurology clinic complaining of tingling in his hand. He has radial deviation of his wrist and there is mild clawing of his fingers, with the 4th and 5th digits being relatively spared. What is the most likely lesion?

Ulnar nerve damage at the wrist

Ulnar nerve damage at the elbow

Radial nerve damage at the elbow

Median nerve damage at the wrist

Median nerve damage at the elbow

A

The ulnar paradox- the higher the lesion, the less the clawing of the fingers seen clinically.
At the elbow the ulnar nerve lesion affects the flexor carpi ulnaris and flexor digitorum profundus.
Ulnar nerve

Origin
C8, T1

Supplies (no muscles in the upper arm)
Flexor carpi ulnaris
Flexor digitorum profundus
Flexor digiti minimi
Abductor digiti minimi
Opponens digiti minimi
Adductor pollicis
Interossei muscle
Third and fourth lumbricals
Palmaris brevis

Path
Posteromedial aspect of upper arm to flexor compartment of forearm, then along the ulnar. Passes beneath the flexor carpi ulnaris muscle, then superficially over the flexor retinaculum into the palm of the hand.

Branches
Branch Supplies
Muscular branch Flexor carpi ulnaris
Medial half of the flexor digitorum profundus
Palmar cutaneous branch (Arises near the middle of the forearm) Skin on the medial part of the palm
Dorsal cutaneous branch Dorsal surface of the medial part of the hand
Superficial branch Cutaneous fibres to the anterior surfaces of the medial one and one-half digits
Deep branch Hypothenar muscles
All the interosseous muscles
Third and fourth lumbricals
Adductor pollicis
Medial head of the flexor pollicis brevis

Effects of injury
Damage at the wrist
Wasting and paralysis of intrinsic hand muscles (claw hand)
Wasting and paralysis of hypothenar muscles
Loss of sensation medial 1 and half fingers
Damage at the elbow
Radial deviation of the wrist
Clawing less in 4th and 5th digits

How well did you know this?
1
Not at all
2
3
4
5
Perfectly
657
Q

At what level does the aorta bifurcate into the left and right common iliac arteries?

L1

L2

L3

L4

L5

A

The aorta typically bifurcates at L4. This level is usually fairly constant and is often tested in the exam.

Levels

Transpyloric plane
Level of the body of L1

Pylorus stomach
Left kidney hilum (L1- left one!)
Fundus of the gallbladder
Neck of pancreas
Duodenojejunal flexure
Superior mesenteric artery
Portal vein
Left and right colic flexure
Root of the transverse mesocolon
2nd part of the duodenum
Upper part of conus medullaris
Spleen

Can be identified by asking the supine patient to sit up without using their arms. The plane is located where the lateral border of the rectus muscle crosses the costal margin.

Anatomical planes
Subcostal plane Lowest margin of 10th costal cartilage
Intercristal plane Level of body L4 (highest point of iliac crest)
Intertubercular plane Level of body L5

Common level landmarks
Inferior mesenteric artery L3
Bifurcation of aorta into common iliac arteries L4
Formation of IVC L5 (union of common iliac veins)
Diaphragm apertures
Vena cava T8
Oesophagus T10
Aortic hiatus T12

How well did you know this?
1
Not at all
2
3
4
5
Perfectly
658
Q

A 32 year old man presents with a painful swelling over the volar aspect of his hand after receiving a hard blow to his palm. On examination, he experiences pain on moving the wrist and on longitudinal compression of the thumb. What is the most likely injury?

Bennets fracture

Scaphoid fracture

5th metacarpal fracture

Ganglion

Bursitis

A

Scaphoid fractures usually occur as a result of direct hard blow to the palm or following a fall on the out-stretched hand. The main physical signs are swelling and tenderness in the anatomical snuff box, and pain on wrist movements and on longitudinal compression of the thumb

Upper limb fractures

Colles’ fracture
Fall onto extended outstretched hands
Described as a dinner fork type deformity
Classical Colles’ fractures have the following 3 features:

Features of the injury
1. Transverse fracture of the radius
2. 1 inch proximal to the radio-carpal joint
3. Dorsal displacement and angulation

Smith’s fracture (reverse Colles’ fracture)
Volar angulation of distal radius fragment (Garden spade deformity)
Caused by falling backwards onto the palm of an outstretched hand or falling with wrists flexed

Bennett’s fracture
Intra-articular fracture of the first carpometacarpal joint
Impact on flexed metacarpal, caused by fist fights
X-ray: triangular fragment at ulnar base of metacarpal

Monteggia’s fracture
Dislocation of the proximal radioulnar joint in association with an ulna fracture
Fall on outstretched hand with forced pronation
Needs prompt diagnosis to avoid disability

Galeazzi fracture
Radial shaft fracture with associated dislocation of the distal radioulnar joint
Occur after a fall on the hand with a rotational force superimposed on it.
On examination, there is bruising, swelling and tenderness over the lower end of the forearm.
X Rays reveal the displaced fracture of the radius and a prominent ulnar head due to dislocation of the inferior radio-ulnar joint.

Barton’s fracture
Distal radius fracture (Colles’/Smith’s) with associated radiocarpal dislocation
Fall onto extended and pronated wrist

Scaphoid fractures
Scaphoid fractures are the commonest carpal fractures.
Surface of scaphoid is covered by articular cartilage with small area available for blood vessels (fracture risks blood supply)
Forms floor of anatomical snuffbox
Risk of fracture associated with fall onto outstretched hand (tubercle, waist, or proximal 1/3)
The main physical signs are swelling and tenderness in the anatomical snuff box, and pain on wrist movements and on longitudinal compression of the thumb.
Ulnar deviation AP needed for visualization of scaphoid
Immobilization of scaphoid fractures difficult

Radial head fracture
Fracture of the radial head is common in young adults.
It is usually caused by a fall on the outstretched hand.
On examination, there is marked local tenderness over the head of the radius, impaired movements at the elbow, and a sharp pain at the lateral side of the elbow at the extremes of rotation (pronation and supination).

How well did you know this?
1
Not at all
2
3
4
5
Perfectly
659
Q

A man with lung cancer and bone metastasis in the thoracic spinal vertebral bodies, sustains a pathological fracture at the level of T4. The fracture is unstable and the spinal cord is severely compressed at this level. Which of the findings below will not be present 6 weeks after injury?

Extensor plantar reflexes

Spasticity of the lower limbs

Diminished patellar tendon reflex

Urinary incontinence

Sensory ataxia

A

A thoracic cord lesion causes spastic paraperesis, hyperrflexia and extensor plantar responses (UMN lesion), incontinence, sensory loss below the lesion and ‘sensory’ ataxia.These features typically manifest several weeks later, once spinal shock (in which areflexia predominates) has resolved.

Spinal cord

  • Located in a canal within the vertebral column that affords it structural support.
    Rostrally it continues to the medulla oblongata of the brain and caudally it tapers at a level corresponding to the L1-2 interspace (in the adult), a central structure, the filum terminale anchors the cord to the first coccygeal vertebra.
    The spinal cord is characterised by cervico-lumbar enlargements and these, broadly speaking, are the sites which correspond to the brachial and lumbar plexuses respectively.

There are some key points to note when considering the surgical anatomy of the spinal cord:

  • During foetal growth the spinal cord becomes shorter than the spinal canal, hence the adult site of cord termination at the L1-2 level.
  • Due to growth of the vertebral column the spine segmental levels may not always correspond to bony landmarks as they do in the cervical spine.
  • The spinal cord is incompletely divided into two symmetrical halves by a dorsal median sulcus and ventral median fissure. Grey matter surrounds a central canal that is continuous rostrally with the ventricular system of the CNS.
  • The grey matter is sub divided cytoarchitecturally into Rexeds laminae.
  • Afferent fibres entering through the dorsal roots usually terminate near their point of entry but may travel for varying distances in Lissauers tract. In this way they may establish synaptic connections over several levels
  • At the tip of the dorsal horn are afferents associated with nociceptive stimuli. The ventral horn contains neurones that innervate skeletal muscle.

The key point to remember when revising CNS anatomy is to keep a clinical perspective in mind. So it is worth classifying the ways in which the spinal cord may become injured. These include:

Trauma either direct or as a result of disc protrusion
Neoplasia either by direct invasion (rare) or as a result of pathological vertebral fracture
Inflammatory diseases such as Rheumatoid disease, or OA (formation of osteophytes compressing nerve roots etc.
Vascular either as a result of stroke (rare in cord) or as complication of aortic dissection
Infection historically diseases such as TB, epidural abscesses.

The anatomy of the cord will, to an extent dictate the clinical presentation. Some points/ conditions to remember:

Brown- Sequard syndrome-Hemisection of the cord producing ipsilateral loss of proprioception and upper motor neurone signs, plus contralateral loss of pain and temperature sensation. The explanation of this is that the fibres decussate at different levels.
Lesions below L1 will tend to present with lower motor neurone signs

How well did you know this?
1
Not at all
2
3
4
5
Perfectly
660
Q

A 35 year old farm labourer injures the posterior aspect of his hand with a mechanical scythe. He severs some of his extensor tendons in this injury. How many tunnels lie in the extensor retinaculum that transmit the tendons of the extensor muscles?

One

Three

Four

Five

Six

A

There are six tunnels, each lined by its own synovial sheath.

Extensor retinaculum

The extensor retinaculum is a thickening of the deep fascia that stretches across the back of the wrist and holds the long extensor tendons in position.
Its attachments are:
The pisiform and triquetral medially
The end of the radius laterally

Structures related to the extensor retinaculum
Structures superficial to the retinaculum
Basilic vein
Dorsal cutaneous branch of the ulnar nerve
Cephalic vein
Superficial branch of the radial nerve
Structures passing deep to the extensor retinaculum
Extensor carpi ulnaris tendon
Extensor digiti minimi tendon
Extensor digitorum and extensor indicis tendon
Extensor pollicis longus tendon
Extensor carpi radialis longus tendon
Extensor carpi radialis brevis tendon
Abductor pollicis longus and extensor pollicis brevis tendons

Beneath the extensor retinaculum fibrous septa form six compartments that contain the extensor muscle tendons. Each compartment has its own synovial sheath.

The radial artery
The radial artery passes between the lateral collateral ligament of the wrist joint and the tendons of the abductor pollicis longus and extensor pollicis brevis.

Image illustrating the topography of tendons passing under the extensor retinaculum

How well did you know this?
1
Not at all
2
3
4
5
Perfectly
661
Q

Which of the following cell types is least likely to be found in a wound 1 week following injury?

Macrophages

Fibroblasts

Myofibroblasts

Endothelial cells

Neutrophils

A

Myofibroblasts are differentiated fibroblasts, in which the cytoskeleton contains actin filaments. These cell types facilitate wound contracture and are the hallmark of a mature wound. They are almost never found in wounds less than 1 month old.
Remember the question asks about the cell type asks about which cells are least likely to be found.

Phases of wound healing

Phase Key features Cells Timeframe
Haemostasis
Vasospasm in adjacent vessels
Platelet plug formation and generation of fibrin rich clot
Erythrocytes and platelets Seconds/ Minutes
Inflammation
Neutrophils migrate into wound (function impaired in diabetes).
Growth factors released, including basic fibroblast growth factor and vascular endothelial growth factor.
Fibroblasts replicate within the adjacent matrix and migrate into wound.
Macrophages and fibroblasts couple matrix regeneration and clot substitution.
Neutrophils, fibroblasts and macrophages Days
Regeneration
Platelet derived growth factor and transformation growth factors stimulate fibroblasts and epithelial cells.
Fibroblasts produce a collagen network.
Angiogenesis occurs and wound resembles granulation tissue.
Fibroblasts, endothelial cells, macrophages Weeks
Remodelling
Longest phase of the healing process and may last up to one year (or longer).
During this phase fibroblasts become differentiated (myofibroblasts) and these facilitate wound contraction.
Collagen fibres are remodelled.
Microvessels regress leaving a pale scar.
Myofibroblasts

How well did you know this?
1
Not at all
2
3
4
5
Perfectly
662
Q

Which main group of receptors does dobutamine bind to?

α-1

α-2

ß-1

ß-2

D-1

A

Dobutamine is a sympathomimetic with both alpha- and beta-agonist properties; it displays a considerable selectivity for beta1-cardiac receptors.

Inotropes and cardiovascular receptors

Inotropes are a class of drugs which work primarily by increasing cardiac output. They should be distinguished from vasoconstrictor drugs which are used specifically when the primary problem is peripheral vasodilatation.

Catecholamine type agents are commonly used and work by increasing cAMP levels by adenylate cyclase stimulation. This in turn increases intracellular calcium ion mobilisation and thus the force of contraction. Adrenaline works as a beta adrenergic receptor agonist at lower doses and an alpha receptor agonist at higher doses. Dopamine causes dopamine receptor mediated renal and mesenteric vascular dilatation and beta 1 receptor agonism at higher doses. This results in increased cardiac output. Since both heart rate and blood pressure are raised, there is less overall myocardial ischaemia. Dobutamine is a predominantly beta 1 receptor agonist with weak beta 2 and alpha receptor agonist properties. Noradrenaline is a catecholamine type agent and predominantly acts as an alpha receptor agonist and serves as a peripheral vasoconstrictor.

Phosphodiesterase inhibitors such as milrinone act specifically on the cardiac phosphodiesterase and increase cardiac output.

Inotrope Cardiovascular receptor action
Adrenaline α-1, α-2, β-1, β-2
Noradrenaline α-1,( α-2), (β-1), (β-2)
Dobutamine β-1, (β 2)
Dopamine (α-1), (α-2), (β-1), D-1,D-2
Minor receptor effects in brackets

Effects of receptor binding
α-1, α-2 vasoconstriction
β-1 increased cardiac contractility and HR
β-2 vasodilatation
D-1 renal and spleen vasodilatation
D-2 inhibits release of noradrenaline

How well did you know this?
1
Not at all
2
3
4
5
Perfectly
663
Q

A 66 year old male is admitted to the vascular ward for an amputation. He reports episodes of vertigo and dysarthria to the house officer. He suddenly collapses with a Glasgow Coma Score of 3. What is the most likely diagnosis?

Cerebral haemorrhage in left temporal parietal area

Opiate overdose

Cerebral haemorrhage in right temporal parietal area

Diazepam overdose

Basilar artery occlusion

A

Vertigo and dysarthria suggest a posterior circulation event. In the scenario of a patient complaining of posterior symptoms and a sudden deterioration in consciousness, the main differential diagnosis is of a basilar artery occlusion.

Stroke: types

Primary intracerebral haemorrhage (PICH, c. 10%)
Presents with headache, vomiting, loss of consciousness
Total anterior circulation infarcts (TACI, c. 15%)
Involves middle and anterior cerebral arteries
Hemiparesis/hemisensory loss
Homonymous hemianopia
Higher cognitive dysfunction e.g. Dysphasia
Partial anterior circulation infarcts (PACI, c. 25%)
Involves smaller arteries of anterior circulation e.g. upper or lower division of middle cerebral artery
Higher cognitive dysfunction or two of the three TACI features
Lacunar infarcts (LACI, c. 25%)
Involves perforating arteries around the internal capsule, thalamus and basal ganglia
Present with either isolated hemiparesis, hemisensory loss or hemiparesis with limb ataxia
Posterior circulation infarcts (POCI, c. 25%)
Vertebrobasilar arteries
Presents with features of brainstem damage
Ataxia, disorders of gaze and vision, cranial nerve lesions
Lateral medullary syndrome (posterior inferior cerebellar artery)
Wallenberg’s syndrome
Ipsilateral: ataxia, nystagmus, dysphagia, facial numbness, cranial nerve palsy
Contralateral: limb sensory loss
Weber’s syndrome
Ipsilateral III palsy
Contralateral weakness

Anterior cerebral artery
Contralateral hemiparesis and sensory loss, lower extremity > upper
Disconnection syndrome

Middle cerebral artery
Contralateral hemiparesis and sensory loss, upper extremity > lower
Contralateral hemianopia
Aphasia (Wernicke’s)
Gaze abnormalities

Posterior cerebral artery
Contralateral hemianopia with macular sparing
Disconnection syndrome

Lacunar
Present with either isolated hemiparesis, hemisensory loss or hemiparesis with limb ataxia

Lateral medulla (posterior inferior cerebellar artery)
Ipsilateral: ataxia, nystagmus, dysphagia, facial numbness, cranial nerve palsy e.g.

Horner’s
Contralateral: limb sensory loss

Pontine
VI nerve: horizontal gaze palsy
VII nerve
Contralateral hemiparesis

How well did you know this?
1
Not at all
2
3
4
5
Perfectly
664
Q

Which cranial nerve provides general sensation to the anterior two thirds of the tongue?

Facial

Trigeminal

Hypoglossal

Vagus

Glossopharyngeal

A

Taste to the anterior two thirds of the tongue is supplied by the facial nerve, the trigeminal supplies general sensation, this is mediated by the mandibular branch of the trigeminal nerve (via the lingual nerve).

Cranial nerves

Cranial nerve lesions
Olfactory nerve May be injured in basal skull fractures or involved in frontal lobe tumour extension. Loss of olfactory nerve function in relation to major CNS pathology is seldom an isolated event and thus it is poor localiser of CNS pathology.
Optic nerve Problems with visual acuity may result from intra ocular disorders. Problems with the blood supply such as amaurosis fugax may produce temporary visual distortion. More important surgically is the pupillary response to light. The pupillary size may be altered in a number of disorders. Nerves involved in the resizing of the pupil connect to the pretectal nucleus of the high midbrain, bypassing the lateral geniculate nucleus and the primary visual cortex. From the pretectal nucleus neurones pass to the Edinger - Westphal nucleus, motor axons from here pass along with the oculomotor nerve. They synapse with ciliary ganglion neurones; the parasympathetic axons from this then innervate the iris and produce miosis. The miotic pupil is seen in disorders such as Horner’s syndrome or opiate overdose.
Mydriasis is the dilatation of the pupil in response to disease, trauma, drugs (or the dark!). It is pathological when light fails to induce miosis. The radial muscle is innervated by the sympathetic nervous system. Because the parasympathetic fibres travel with the oculomotor nerve they will be damaged by lesions affecting this nerve (e.g. cranial trauma).
The response to light shone in one eye is usually a constriction of both pupils. This indicates intact direct and consensual light reflexes. When the optic nerve has an afferent defect the light shining on the affected eye will produce a diminished pupillary response in both eyes. Whereas light shone on the unaffected eye will produce a normal pupillary response in both eyes. This is referred to as the Marcus Gunn pupil and is seen in conditions such as optic neuritis. In a total CN II lesion shining the light in the affected eye will produce no response.
Oculomotor nerve The pupillary effects are described above. In addition it supplies all ocular muscles apart from lateral rectus and superior oblique. Thus the affected eye will be deviated inferolaterally. Levator palpebrae superioris may also be impaired resulting in impaired ability to open the eye.
Trochlear nerve The eye will not be able to look down.
Trigeminal nerve Largest cranial nerve. Exits the brainstem at the pons. Branches are ophthalmic, maxillary and mandibular. Only the mandibular branch has both sensory and motor fibres. Branches converge to form the trigeminal ganglion (located in Meckels cave). It supplies the muscles of mastication and also tensor veli palatine, mylohyoid, anterior belly of digastric and tensor tympani. The detailed descriptions of the various sensory functions are described in other areas of the website. The corneal reflex is important and is elicited by applying a small tip of cotton wool to the cornea, a reflex blink should occur if it is intact. It is mediated by: the naso ciliary branch of the ophthalmic branch of the trigeminal (sensory component) and the facial nerve producing the motor response. Lesions of the afferent arc will produce bilateral absent blink and lesions of the efferent arc will result in a unilateral absent blink.
Abducens nerve The affected eye will have a deficit of abduction. This cranial nerve exits the brainstem between the pons and medulla. It thus has a relatively long intra cranial course which renders it susceptible to damage in raised intra cranial pressure.
Facial nerve Emerges from brainstem between pons and medulla. It controls muscles of facial expression and taste from the anterior 2/3 of the tongue. The nerve passes into the petrous temporal bone and into the internal auditory meatus. It then passes through the facial canal and exits at the stylomastoid foramen. It passes through the parotid gland and divides at this point. It does not innervate the parotid gland. Its divisions are considered in other parts of the website. Its motor fibres innervate orbicularis oculi to produce the efferent arm of the corneal reflex. In surgical practice it may be injured during parotid gland surgery or invaded by malignancies of the gland and a lower motor neurone on the ipsilateral side will result.
Vestibulo-cochlear nerve Exits from the pons and then passes through the internal auditory meatus. It is implicated in sensorineural hearing loss. Individuals with sensorineural hearing loss will localise the sound in webers test to the normal ear. Rinnes test will be reduced on the affected side but should still work. These two tests will distinguish sensorineural hearing loss from conductive deafness. In the latter condition webers test will localise to the affected ear and Rinnes test will be impaired on the affected side. Surgical lesions affecting this nerve include CNS tumours and basal skull fractures. It may also be damaged by the administration of ototoxic drugs (of which gentamicin is the most commonly used in surgical practice).
Glossopharyngeal nerve Exits the pons just above the vagus. Receives sensory fibres from posterior 1/3 tongue, tonsils, pharynx and middle ear (otalgia may occur following tonsillectomy). It receives visceral afferents from the carotid bodies. It supplies parasympathetic fibres to the parotid gland via the otic ganglion and motor function to stylopharyngeaus muscle. The sensory function of the nerve is tested using the gag reflex.
Vagus nerve Leaves the medulla between the olivary nucleus and the inferior cerebellar peduncle. Passes through the jugular foramen and into the carotid sheath. Details of the functions of the vagus nerve are covered in the website under relevant organ sub headings.
Accessory nerve Exists from the caudal aspect of the brainstem (multiple branches) supplies trapezius and sternocleidomastoid muscles. The distal portion of this nerve is most prone to injury during surgical procedures.
Hypoglossal nerve Emerges from the medulla at the preolivary sulcus, passes through the hypoglossal canal. It lies on the carotid sheath and passes deep to the posterior belly of digastric to supply muscles of the tongue (except palatoglossus). Its location near the carotid sheath makes it vulnerable during carotid endarterectomy surgery and damage will produce ipsilateral defect in muscle function.

How well did you know this?
1
Not at all
2
3
4
5
Perfectly
665
Q

A 30 year old male is admitted electively for a right inguinal hernia repair under local anaesthesia. He is otherwise well but his grandfather died from a pulmonary embolism. What is the most appropriate form of thromboprophylaxis?

Administration of low dose low molecular weight heparin for 2 weeks

Administration of high dose low molecular weight heparin for 2 weeks

No prophylaxis

Low dose low molecular weight heparin and pneumatic compression stockings

High dose low molecular weight heparin and pneumatic compression stockings

A

Inguinal hernia repairs under local anaesthetic have a short operative time and patients are usually ambulant immediately afterwards. His family history is unlikely to be significant and he is at very low risk.

Thromboprophylaxis in surgical patients

Deep vein thrombosis may develop insidiously in many surgical patients. Untreated it may progress to result in pulmonary embolism.
The following surgical patients are at increased risk of deep vein thrombosis:
Surgery greater than 90 minutes at any site or greater than 60 minutes if the procedure involves the lower limbs or pelvis
Acute admissions with inflammatory process involving the abdominal cavity
Expected significant reduction in mobility
Age over 60 years
Known malignancy
Thrombophilia
Previous thrombosis
BMI >30
Taking hormone replacement therapy or the contraceptive pill
Varicose veins with phlebitis

Mechanical thromboprophylaxis
Early ambulation after surgery is cheap and is effective
Compression stockings (contra -indicated in peripheral arterial disease)
Intermittent pneumatic compression devices
Foot impulse devices

Therapeutic agents
Agent Mode of action Uses
Low molecular weight heparin Binds antithrombin causing inhibition of factor Xa Thromboprophylaxis or treatment of thromboembolic events in those with normal renal function. It is given as once daily subcutaneous injection
Unfractionated heparin Binds antithrombin III affecting thrombin and factor Xa Effective anticoagulation, administered intravenously it has a rapid onset and its therapeutic effects decline quickly on stopping and infusion. Its activity is measured using the APTT. If need be it can be reversed using protamine sulphate
Dabigatran Orally administered direct thrombin inhibitor Used prophylaxis in hip and knee surgery. It does not require therapeutic monitoring. It should not be used in any patient in whom there is a risk of active bleeding or imminent likelihood of surgery. It is reversed using Idarucizumab

How well did you know this?
1
Not at all
2
3
4
5
Perfectly
666
Q

A 23 year old man suffers a thermal injury to his left hand. It becomes red and painful. Which of the following mediators are not involved in this process?

Histamine

Free radicals

Prostaglandins

Leukotrienes

Serotonin

A

Acute inflammation is not mediated by free radicals
Chemical mediators facilitate the spread of inflammation into normal tissue
Chemical mediators include:
Lysosomal compounds
Chemokines such as serotinin and histamine (released by platelets and mast cells)

Other enzyme cascades producing inflammatory mediators include:
Complement, kinin, coagulation system and fibrinolytic system

Acute inflammation

Inflammation is the reaction of the tissue elements to injury. Vascular changes occur, resulting in the generation of a protein rich exudate. So long as the injury does not totally destroy the existing tissue architecture, the episode may resolve with restoration of original tissue architecture.

Vascular changes
Vasodilation occurs and persists throughout the inflammatory phase.
Inflammatory cells exit the circulation at the site of injury.
The equilibrium that balances Starlings forces within capillary beds is disrupted and a protein rich exudate will form as the vessel walls also become more permeable to proteins.
The high fibrinogen content of the fluid may form a fibrin clot. This has several important immunomodulatory functions.

Sequelae
Resolution
Typically occurs with minimal initial injury
Stimulus removed and normal tissue architecture results
Organisation
Delayed removal of exudate
Tissues undergo organisation and usually fibrosis
Suppuration
Typically formation of an abscess or an empyema
Sequestration of large quantities of dead neutrophils
Progression to chronic inflammation
Coupled inflammatory and reparative activities
Usually occurs when initial infection or suppuration has been inadequately managed

Causes
Infections e.g. Viruses, exotoxins or endotoxins released by bacteria
Chemical agents
Physical agents e.g. Trauma
Hypersensitivity reactions
Tissue necrosis

Presence of neutrophil polymorphs is a histological diagnostic feature of acute inflammation

How well did you know this?
1
Not at all
2
3
4
5
Perfectly
667
Q

A baby is found to have a Klumpke’s palsy post delivery. Which of the following is most likely to be present?

Partial loss of flexors of the wrist

Weak elbow flexion

Pronated forearm

Adducted shoulder

Shoulder medially rotated

A

Features of Klumpkes Paralysis
Claw hand (MCP joints extended and IP joints flexed)
Loss of sensation over medial aspect of forearm and hand
Horner’s syndrome
Loss of flexors of the wrist
A C8, T1 root lesion is called Klumpke’s paralysis and is caused by delivery with the arm extended.

Brachial plexus

The brachial plexus extends from the neck to the axilla. It is formed by the ventral rami of the fifth to the eighth cervical nerves with the ascending part of the first thoracic nerve.

Location of the plexus
The ventral rami which form the plexus enter the lower part of the posterior triangle of the neck in series with the ventral rami of the cervical plexus. The second part of the subclavian artery lies immediately anterior to the lower two rami. The upper three rami intermingle and pass inferolaterally towards the axilla and subclavian artery. They are enclosed within an extension of the prevertebral fascia. In the neck the plexus lies deep to platysma, the supraclavicular nerves, inferior belly of omohyoid and the transverse cervical artery. It then passes deep to the clavicle and the suprascapular vessels, to enter the axilla, and thence surround the second part of the axillary artery

Composition of the plexus
Ventral rami, the roots of the plexus, lie between scalenus medius and anterior.

As they enter the posterior triangle, the upper two (C5,6) and lower two (C8, T1) roots of the plexus unite to form the upper and lower trunks of the plexus respectively. Meanwhile, C7 continues as the middle trunk. The lower trunk may groove the superior surface of the first rib posterior to the subclavian artery, and the root from the first ventral ramus is always in contact with it.

Each trunk divides into ventral and dorsal divisions which are destined to supply the anterior (flexor) and posterior (extensor) parts of the upper limb.

The cords of the plexus are formed in the axilla. The dorsal divisions unite to form the posterior cord (C5-8). The ventral divisions of the upper and middle trunks unite to form the lateral cord (C5-7), while the ventral divisions of the lower trunk continues as the medial cord (C8-T1). The cords are named according to their relationship to the axillary artery. Each cord terminates by dividing into two main branches at the beginning of the third part of the artery.

Sympathetic communications
The fifth and sixth cervical ventral rami receive grey rami communicantes from the middle cervical ganglion, while the two or more grey rami communicantes pass from the inferior cervical ganglion to the seventh and eighth cervical ventral rami. The first thoracic ventral ramus receives its grey ramus from the cervicothoracic ganglion. Its for this reason that inferior plexus injury can be complicated by a Horners syndrome.

Summary
Origin Anterior rami of C5 to T1
Sections of the plexus
Roots, trunks, divisions, cords, branches
Mnemonic:Real Teenagers Drink Cold Beer
Roots
Located in the posterior triangle
Pass between scalenus anterior and medius
Trunks
Located posterior to middle third of clavicle
Upper and middle trunks related superiorly to the subclavian artery
Lower trunk passes over 1st rib posterior to the subclavian artery
Divisions Apex of axilla
Cords Related to axillary artery

How well did you know this?
1
Not at all
2
3
4
5
Perfectly
668
Q

A 43 year old lady presents with urinary incontinence. At which of the following locations is Onufs nucleus likely to be found?

Medulla oblongata

Anterior horn of L5 nerve roots

Micturition centre in the Pons

Anterior horn of S2 nerve roots

None of the above

A

Onufs nucleus is located in the anterior horn of S2 and is the origin of neurones to the external urethral sphincter.

Urinary incontinence

Involuntary passage of urine. Most cases are female (80%). It has a prevalence of 11% in those aged greater than 65 years. The commonest variants include:
Stress urinary incontinence (50%)
Urge incontinence (15%)
Mixed (35%)

Males
Males may also suffer from incontinence although it is a much rarer condition in men. A number of anatomical factors contribute to this. Males have 2 powerful sphincters; one at the bladder neck and the other in the urethra. Damage to the bladder neck mechanism is a factor in causing retrograde ejaculation following prostatectomy. The short segment of urethra passing through the urogenital diaphragm consists of striated muscle fibres (the external urethral sphincter) and smooth muscle capable of more sustained contraction. It is the latter mechanism that maintains continence following prostatectomy.

Females
The sphincter complex at the level of bladder neck is poorly developed in females. As a result the external sphincter complex is functionally more important, its composition being similar to that of males. Innervation is via the pudendal nerve and the neuropathy that may accompany obstetric events may compromise this and lead to stress urinary incontinence.

Innervation
Somatic innervation to the bladder is via the pudendal, hypogastric and pelvic nerves. Autonomic nerves travel in these nerve fibres too. Bladder filling leads to detrusor relaxation (sympathetic) coupled with sphincter contraction. The parasympathetic system causes detrusor contraction and sphincter relaxation. Overall control of micturition is centrally mediated via centres in the Pons.

Stress urinary incontinence
50% of cases, especially in females.
Damage (often obstetric) to the supporting structures surrounding the bladder may lead to urethral hypermobility.
Other cases due to sphincter dysfunction, usually from neurological disorders (e.g. Pudendal neuropathy, multiple sclerosis).

Urethral mobility:
Pressure not transmitted appropriately to the urethra resulting in involuntary passage of urine during episodes of raised intra-abdominal pressure.

Sphincter dysfunction:
Sphincter fails to adapt to compress urethra resulting in involuntary passage of urine. When the sphincter completely fails there is often to continuous passage of urine.

Urge incontinence
In these patients there is sense of urgency followed by incontinence. The detrusor muscle in these patients is unstable and urodynamic investigation will demonstrate overactivity of the detrusor muscle at inappropriate times (e.g. Bladder filling). Urgency may be seen in patients with overt neurological disorders and those without. The pathophysiology is not well understood but poor central and peripheral co-ordination of the events surrounding bladder filling are the main processes.

Assessment
Careful history and examination including vaginal examination for cystocele.
Bladder diary for at least 3 days
Consider flow cystometry if unclear symptomatology or surgery considered and diagnosis is unclear.
Exclusion of other organic disease (e.g. Stones, UTI, Cancer)

Management
Conservative measures should be tried first; Stress urinary incontinence or mixed symptoms should undergo 3 months of pelvic floor exercise. Over active bladder should have 6 weeks of bladder retraining.
Drug therapy for women with overactive bladder should be offered oxybutynin (or solifenacin if elderly) if conservative measures fail.
In women with detrusor instability who fail non operative therapy a trial of sacral neuromodulation may be considered, with conversion to permanent implant if good response. Augmentation cystoplasty is an alternative but will involve long term intermittent self catheterisation.
In women with stress urinary incontinence a urethral sling type procedure may be undertaken. Where cystocele is present in association with incontinence it should be repaired particularly if it lies at the introitus.

NICE guidelines
Initial assessment urinary incontinence should be classified as stress/urge/mixed.
At least 3/7 bladder diary if unable to classify easily.
Start conservative treatment before urodynamic studies if a diagnosis is obvious from the history
Urodynamic studies if plans for surgery.
Stress incontinence: Pelvic floor exercises 3/12, if fails consider surgery.
Urge incontinence: Bladder training >6/52, if fails for oxybutynin (antimuscarinic drugs) then sacral nerve stimulation.
Pelvic floor exercises offered to all women in their 1st pregnancy.

How well did you know this?
1
Not at all
2
3
4
5
Perfectly
669
Q

A cervical rib is due to which of the following?

Hyperplasia of the annulus fibrosus

Proliferation of the nucleus pulposus

Fusion of the transverse processes of the 6th and 7th cervical vertebrae

An accessory cervical vertebra

Elongation of the transverse processes of the 7th cervical vertebra

A

Cervical ribs occur as a result of the elongation of the transverse process of the 7th cervical vertebra. It is usually a fibrous band that attaches to the first thoracic rib.

Cervical ribs

0.2-0.4% incidence
Most cases present with neurological symptoms
Consist of an anomalous fibrous band that often originates from C7 and may arc towards, but rarely reaches the sternum
Congenital cases may present around the third decade, some cases are reported to occur following trauma
Bilateral in up to 70%
Compression of the subclavian artery may produce absent radial pulse on clinical examination and in particular may result in a positive Adsons test (lateral flexion of the neck towards the symptomatic side and traction of the symptomatic arm- leads to obliteration of radial pulse)
Treatment is most commonly undertaken when there is evidence of neurovascular compromise. A transaxillary approach is the traditional operative method for excision.

How well did you know this?
1
Not at all
2
3
4
5
Perfectly
670
Q

A 22 year old man suffers a compound fracture of the tibia. During attempted surgical repair the deep peroneal nerve is divided. Which of the following muscles will not be affected as a result?

Tibialis anterior

Peroneus longus

Extensor hallucis longus

Extensor digitorum longus

Peroneus tertius

A

Peroneus longus is innervated by the superficial peroneal nerve (L4, L5, S1).

How well did you know this?
1
Not at all
2
3
4
5
Perfectly
671
Q

A 50 year old lady is admitted having fallen down some stairs sustaining multiple rib fractures 36 hours previously. On examination, she is confused and agitated and has clinical evidence of lateralising signs. She deteriorates further and then dies with no response to resuscitation. What is the most likely explanation?

Intraventricular haemorrhage

Acute sub dural haemorrhage

Chronic sub dural haematoma

Sub arachnoid haemorrhage

Extra dural haematoma

A

The time frame of deterioration of an acute sub dural bleed would fit with this scenario. They are highly lethal and not uncommon injuries. As the bleed enlarges, lateralising signs may be seen and eventually coning and death will occur.

Intra cranial haemorrhage

Extradural haematoma Bleeding into the space between the dura mater and the skull. Often results from acceleration-deceleration trauma or a blow to the side of the head. The majority of extradural haematomas occur in the temporal region where skull fractures cause a rupture of the middle meningeal artery.

Features
Raised intracranial pressure
Some patients may exhibit a lucid interval
Subdural haematoma Bleeding into the outermost meningeal layer. Most commonly occur around the frontal and parietal lobes. May be either acute or chronic.

Risk factors include old age and alcoholism.

Slower onset of symptoms than a extradural haematoma.
Intracerebral haematoma Usually hyperdense lesions on CT scanning. Arise in areas of traumatic contusion which fuse to become a haematoma. Areas of clot and fresh blood may co-exist on the same CT scan (Swirl sign). Large haematomas and those associated with mass effect should be evacuated.
Subarachnoid haemorrhage Usually occurs spontaneously in the context of a ruptured cerebral aneurysm but may be seen in association with other injuries when a patient has sustained a traumatic brain injury
Intraventricular haemorrhage Haemorrhage that occurs into the ventricular system of the brain. It is relatively rare in adult surgical practice and when it does occur, it is typically associated with severe head injuries. In premature neonates it may occur spontaneously. The blood may clot and occlude CSF flow, hydrocephalus may result.
In neonatal practice the vast majority of IVH occur in the first 72 hours after birth, the aetiology is not well understood and it is suggested to occur as a result of birth trauma combined with cellular hypoxia, together with the delicate neonatal CNS.

How well did you know this?
1
Not at all
2
3
4
5
Perfectly
672
Q

An 18 year old man develops a severe spreading sepsis of the hand. The palm is explored surgically and the flexor digiti minimi brevis muscle is mobilised to facilitate drainage of the infection. Which of the following structures is not closely related to this muscle?

The hook of hamate

Median nerve

Superficial palmar arterial arch

Digital nerves arising from the ulnar nerve

None of the above

A

The flexor digiti minimi brevis originates from the Hamate, on its under- surface lie the ulnar contribution to the superficial palmar arterial arch and digital nerves derived from the ulnar nerve. The median nerve overlies the flexor tendons.

Hand

Anatomy of the hand
Bones
8 Carpal bones
5 Metacarpals
14 phalanges
Intrinsic Muscles 8 Interossei - Supplied by ulnar nerve
4 palmar-adduct fingers
4 dorsal- abduct fingers
Intrinsic muscles Lumbricals
Flex MCPJ and extend the IPJ.
Origin deep flexor tendon and insertion dorsal extensor hood mechanism.
Innervation: 1st and 2nd- median nerve, 3rd and 4th- deep branch of the ulnar nerve.
Thenar eminence
Abductor pollicis brevis
Opponens pollicis
Flexor pollicis brevis
Hypothenar eminence
Opponens digiti minimi
Flexor digiti minimi brevis
Abductor digiti minimi

Fascia and compartments of the palm
The fascia of the palm is continuous with the antebrachial fascia and the fascia of the dorsum of the hand. The palmar fascia is thin over the thenar and hypothenar eminences. In contrast, the central palmar fascia is relatively thick. The palmar aponeurosis covers the soft tissues and overlies the flexor tendons. The apex of the palmar aponeurosis is continuous with the flexor retinaculum and the palmaris longus tendon. Distally, it forms four longitudinal digital bands that attach to the bases of the proximal phalanges, blending with the fibrous digital sheaths.
A medial fibrous septum extends deeply from the medial border of the palmar aponeurosis to the 5th metacarpal. Lying medial to this are the hypothenar muscles. In a similar fashion, a lateral fibrous septum extends deeply from the lateral border of the palmar aponeurosis to the 3rd metacarpal. The thenar compartment lies lateral to this area.
Lying between the thenar and hypothenar compartments is the central compartment. It contains the flexor tendons and their sheaths, the lumbricals, the superficial palmar arterial arch and the digital vessels and nerves.
The deepest muscular plane is the adductor compartment, which contains adductor pollicis.

Short muscles of the hand
These comprise the lumbricals and interossei. The four slender lumbrical muscles flex the fingers at the metacarpophalangeal joints and extend the interphalangeal joint. The four dorsal interossei are located between the metacarpals and the four palmar interossei lie on the palmar surface of the metacarpals in the interosseous compartment of the hand.

Long flexor tendons and sheaths in the hand
The tendons of FDS and FDP enter the common flexor sheath deep to the flexor retinaculum. The tendons enter the central compartment of the hand and fan out to their respective digital synovial sheaths. Near the base of the proximal phalanx, the tendon of FDS splits to permit the passage of FDP. The FDP tendons are attached to the margins of the anterior aspect of the base of the distal phalanx.
The fibrous digital sheaths contain the flexor tendons and their synovial sheaths. These extend from the heads of the metacarpals to the base of the distal phalanges.

Palmar Interossei
Note that there are 4 palmar interossei. The first is a small slip of muscle which arises from the ulnar side of the base of the first metacarpal and passes between the head of the first dorsal interosseous and the oblique head of adductor pollicis to insert into the ulnar base of the of the proximal phalanx of the thumb. The second arises from the ulnar side of the body of the second metacarpal and is inserted into the ulnar side of the extensor hood of the index. The third and fourth palmar interossei arise from the radial sides of the bodies of the 4th and 5th metacarpals respectively and insert into the radial sides of the extensor hoods of the ring and little fingers.

How well did you know this?
1
Not at all
2
3
4
5
Perfectly
673
Q

A 78 year old man complains of a long history of shoulder pain and more recently weakness. On examination, active attempts at abduction are impaired. Passive movements are normal. What is the most likely diagnosis?

Rotator cuff tear

Osteoarthritis

Metastatic malignancy

Adhesive capsulitis

Calcific tendonitis

A

Rotator cuff tears are common in elderly people and may occur following minor trauma or as a result of long standing impingement. Tears greater than 2cm should generally be repaired surgically. The length of the history in this scenario is suggestive of a tear complicating impingement.

Shoulder disorders

Shoulder fractures and dislocations
Fractures
Proximal humerus
Background
Third most common fragility fracture in the elderly.
Results from low energy fall in predominantly elderly females, or from high energy trauma in young males.
Can be associated with nerve injury (commonly axillary), and fracture-dislocation of the humeral head. Detailed neurological assessment is essential for all upper limb injuries.

Anatomy
Osteology
Consists of articular head, greater tuberosity, lesser tuberosity, metaphysis and diaphysis. Between the articular head and the tuberosities is the anatomical neck (previous physis). Between the tuberosities and the metaphysis is the surgical neck.
The supraspinatus, infraspinatus and teres minor muscles attach to the greater tuberosity. The subscapularis muscle attaches to the lesser tuberosity.

Vascular Supply
Humeral head is supplied by the anterior and posterior humeral circumflex arteries. Anatomical neck fractures are at greatest risk of osteonecrosis.

Imaging
Imaging aims to both delineate the fracture pattern, and confirm/exlude the presence of an associated dislocation.
Radiographs - True anteroposterior (AP), axillary lateral and/or scapula Y view.
CT - indicated to better define intra-articular involvement and to aid pre-operative planning. MRI is not useful for fracture imaging.

Classification
Description of the fracture is often more useful than classification. Particular attention should be paid to humeral alignment, fracture displacement, and greater tuberosity position (rotator cuff will pull the GT supero-posterioly, which can cause impingement problems with malunion).
- Neer Classification: Most commonly used. Describes fracture as 2,3,or 4 part depending upon the number main fragments. Also comments on the degree of displacement. Fragments:
-greater tuberosity
-lesser tuberosity
- articular surface
- shaft
Displacement: >1cm or angulation >45 degrees.

Treatment
The vast majority of proximal humeral fractures are minimally displaced, and therefore can be managed conservatively. This involves immobilisation in a polysling, and progressive mobilisation. Pendular exercise can commence at 14 days, and active abduction from 4-6 weeks.

Irreducible fracture dislocation is an indication for operative management. Other indications include large displacement, younger patient, head splitting (intra-articular fractures). However, the recent PROFHER trial (1) has suggested no benefit to operative intervention on patient outcome (it must be applied cautiously as majority of patients were elderly with extraarticular fractures). Options available for surgical management include:

ORIF Most commonly used. Plate and screw fixation. Can reconstruct complex fractures.
Intramedullary nail Suitable for extra-articular configuration, predominantly surgical neck +/- GT fractures.
Hemiarthroplasty Used for un-reconstructable fractures in the older patient who has good glenoid quality.
Total shoulder arthroplasty Unconstructable fractures where high functioning shoulder is required (hemiarthroplasty will cause glenoid erosion)
Reverse shoulder arthroplasty Total shoulder arthroplasty that provides better functional outcome than conventional total shoulder replacement.

Scapula
Background
Uncommon fractures usually associated with high energy trauma. Most commonly involve scapula body or spine (50%), glenoid fossa and glenoid neck. Important to exclude associated life threatening injury.

Imaging
Plain radiographs should include true anteroposterior (AP), axillary lateral and/or scapula Y view. CT scanning is useful for defining intra-articular involvement, displacement and for three dimensional reconstruction.

Classification
Based on the location of the fracture (coracoid, acromion, glenoid neck, glenoid fossa, scapula body). Beware of ipsilateral glenoid neck and clavicle fracture -floating shoulder - where limb is effectively dissociated from axial skeleton.

Treatment
The vast majority of scapula fractures are amenable to conservative management, consisting of sling immobilisation for two weeks followed by early rehabilitation. Floating shoulder will usually require fixation, and consideration of surgery should also be given to intra-articular and displaced/angulated glenoid fractures.

Dislocations

Types
Dislocations around the shoulder joint include glenohumeral dislocation, acromioclavicular joint disruption and sternoclavicular dislocation. Only glenohumeral dislocation will be covered here.

Glenohumeral dislocation
Diagnosis, classification and management are covered here.

Background
Shoulder dislocation is commonly seen in A&E. It has a high recurrence rate that is as high as 80% in teenagers. Initial management requires emergent reduction to prevent lasting chondral damage.

Early assessment and management
Usually a traumatic cause (multi-directional instability in frequent dislocations requires discussion with orthopaedics and is not covered here). Careful history, examination and documentation of neurovascular status of the limb, in particular the axillary nerve (regimental badge sensation). This should be re-assessed post manipulation. Early radiographs to confirm direction of dislocation.

Initial management consists of emergent closed reduction under under entanox and analgesia, but often requires conscious sedation. Arm should then be immobilised in a polysling, and XR to confirm relocation.

Imaging - True anteroposterior (AP), axillary lateral and/or scapula Y view. Reduced humeral head should lie between acromion and coracoid on lateral/scapula view.

Types
Direction Features Cause Examination Reduction techniques
Anterior Most Common >90% Usually traumatic - anterior force on arm when shoulder is abducted, externally rotated Loss of shoulder contour - sulcus sign. Humeral head can be felt anteriorly.
Hippocratic.
Milch.
Stimson.

Kocher not advised due to complication of fracture
Posterior 50% missed in A&E 50% traumatic, but classically post seizure or electrocution Shoulder locked in internal rotation. XR may show lightbulb appearance. Gentle lateral traction to adducted arm.
Inferior Rare Associated with pectorals and rotator cuff tears, and glenoid fracture As for primary injury Management of primary injury
Superior Rare Associated with acrominon/clavicle fracture As for primary injury Management of primary injury

Associated injuries
Bankart lesion - avulsion of the anterior glenoid labrum with an anterior shoulder dislocation (reverse Bankart if poster labrum in posterior dislocation).
Hill Sachs defect - chondral impaction on posteriosuperior humeral head from contact with gleonoid rim. Can be large enough to lock shoulder, requiring open reduction. (Reverse Hill Sachs in posterior dislocation).
Rotator cuff tear - increases with age.
Greater or lesser tuberosity fracture - increases with age.
Humeral neck fracture - shoulder fracture dislocation. More common in high energy trauma and elderly. Should be discussed with orthopaedics prior to any attempted reduction.

Rotator Cuff Disease

Rotator cuff disease is a spectrum of conditions that ranges from subacromial impingement to rotator cuff tears and eventually to rotator cuff arthropathy (arthritis).

Anatomy
The rotator cuff is a group of four muscles that are important in shoulder movements, and maintenance of glenohumeral stability.
Muscle Scapular attachment Humeral attachment Action Innervation
Supraspinatus Supraspinatus fossa Superior facet of greater tuberosity Initiation of abduction of humerus Suprascapular nerve
Infraspinatus Infraspinatus fossa Posterior facet of greater tuberosity External rotation of humerus Suprascapular nerve
Teres Minor Lateral border Inferior facet of greater tuberosity External rotation of humerus Axillary Nerve
Subscapularis Subscapular fossa Lesser tuberosity Internal rotation of humerus Upper and lower subscapular nerve

The inferior rotator cuff muscles (infraspinatus, teres minor, and subscapularis) balance the superior pull of the deltoid. Injury/tear results in upward migration of the humeral head on the glenoid (can be seen on AP radiograph).
Likewise, the anterior muscles (subscapularis) are balanced with the posterior muscles (infraspinatus, teres minor).

Subacromial Impingement

The most common cause of shoulder pain, which results from impingement of the superior cuff on the undersurface of the acromion, and an inflammatory bursitis.
Associated with certain types of acromial morphology (Bigliani classification).
Presents as insidious pain which is exacerbated by overhead activities.

Rotator Cuff Tear

Often presents as an acute event on the background of chronic subacromial impingement in the older patient, but can present as an avulsion injury in younger patients.
Majority of tears are to the superior cuff (supraspinatus, infraspinatus, teres minor), though a tear to subscapularis is associated with subcoracoid impingement.
Tears present as pain and weakness when using the muscles in question.

Rotator Cuff Arthropathy

Defined as shoulder arthritis in the setting of rotator cuff dysfunction. Results from superior migration due to the loss of rotator cuff function and integrity. Unopposed deltoid pulls the humeral head superiorly.
Associated with massive chronic cuff tears.

Imaging

Plain radiographs
AP of the shoulder may show superior migration of the humerus with a cuff tear, and features of arthritis with arthropathy. Other causes of pain may also be identified (e.g. calcific tendonitis/fracture)
Outlet view is useful for defining the acromial morphology

USS
Allows dynamic imaging of the cuff, and is inexpensive. However, it is very user dependent.

MRI
Best imaging modality for cuff pathology.
Also allows imaging of the rest of the shoulder. When intra-articular pathology is suspected, can be combined with an arthrogram for improved sensitivity and specificity.

Treatment

Subacromial impingement
Physiotherapy, oral anti-inflammatory medication
Subacromial steroid injection can settle inflammation
Arthroscopic subacromial decompression by shaving away the undersurface of the acromion, more space is created for the rotator cuff. Cuff integrity is assessed also at time of surgery, and can be repaired if necessary.

Rotator cuff tear
When considering repair of a cuff tear, the age and activity of the patient, the nature of the tear (degenerative vs. acute traumatic), and the size and retraction of the tear should be considered when making a surgical plan.
Mild tears or tears in the elderly can be managed conservatively, as outlined above.
Moderate tears can be repaired arthroscopically. Massive or retracted tears will often require an open repair (occasionally with a tendon transfer). Subacromial decompression is performed at the same time to reduce impingement, symptoms and recurrence.

Calcific tendonitis
Calcific tendonitis involves calcific deposits within tendons anywhere in the body, but most commonly in the rotator cuff (specifically the supraspinatus tendon). When present in the shoulder, it is associated with subacromial impingement and pain.

Pathology
More common in women aged 30-60 years.
Association with diabetes and hypothyroidism

There are three stages of calcification
Formative phase characterized by calcific deposits
Resting phase deposit is stable, but presents with impingement problems
Resorptive phase phagocytic resorption. Most painful stage.

Presentation
Similar in presentation to subacromial impingement, with pain especially with over head activities. Atraumatic in nature.

Imaging
Plain radiographs show calcification of the rotator cuff, usually within 1.5cm of its insertion on the humerus. Supraspinatus outlet views can show level of impingment. Further imaging is rarely needed.

Treatment
Non-operative NSAIDS, steroid injection (controversial, but practiced) and physiotherapy. Approximately 75% will resolve by 6 months with conservative management.
Ultrasound guided or surgical needle barbotage can break down deposits and resolve symptoms. Occasionally surgical excision is required.

Adhesive capsulitis (Frozen Shoulder)
Pain and loss of movement of shoulder joint, which involves fibroplastic proliferation of capsular tissue, causing soft tissue scarring and contracture. Patients present with a painful and decreased arc of motion.
Associated with prolonged immobilization, previous surgery, thyroid disorders (AI) and diabetes
Classically three stages which can take up to two years to resolve:
Stage one the freezing and painful stage
Stage two the frozen and stiff stage
Stage three the thawing stage, where shoulder movement slowly improves

Imaging
Plain radiographs to exclude other causes of a painful shoulder
MRI arthrogram may show capsular contracture, and again may be used to exclude cuff pathology. However, often not performed as diagnosis is largely clinical.

Treatment
Non-operative NSAIDS, steroid injection and physiotherapy. Patience is required as condition can take up to 2 years to improve.
Operative MUA or arthroscopic adhesiolysis (release of adhesions) can expedite recovery, followed by intensive physiotherapy.

Glenohumeral Arthritis
Background
May be osteoarthritis (primary or secondary to cuff tear or trauma), rheumatoid arthritis, or as part of a spondyloarthropathy. Majority of those with RA will develop symptoms.
More common in the elderly
Presents like any other arthritis - pain at night and with movement

Imaging
AP and axillary radiographs will show features of arthritis.
CT/MRI is often useful to classify the shape of the glenoid and extent of bone loss when considering arthroplasty. MRI also essential to asses integrity of rotator cuff if considering shoulder replacement.

Treatment
Like all orthopaedics, start with simple measures:
NSAIDS, management of RA, physiotherapy, steroid injection.
Hemiarthroplasty can sometimes be considered if glenoid is in excellent condition or if patient has large comorbidity.
Arthroscopic debridement is useful if patient has isolated ACJ arthritis, but is rarely used for glenohumeral arthritis.
Total shoulder replacement is shown to produce superior outcome when compared to hemiarthroplasty in terms of pain relief, function and implant survival.
Total shoulder replacement can be anatomical (ball on humerus, with cup on glenoid), or reverse geometry (ball on glenoid, with cup on humerus). Anatomical TSR requires an in tact rotator cuff, so often reverse is preferable when the cuff if questionable in integrity.

References
1. JAMA. 2015;313(10):1037-1047. doi:10.1001/jama.2015.1629

How well did you know this?
1
Not at all
2
3
4
5
Perfectly
674
Q

A 21 year old man is involved in a road traffic accident. After a transient period of concussion he is found to have a GCS of 15 by the paramedics. On arrival at hospital he is monitored in a side room of the emergency department. When he is next observed he is noted to have a GCS of 3 and a blown right pupil. Which of the processes below best accounts for this deterioration?

Hydrocephalus

Intraventricular bleed

Sub dural bleed

Trans tentorial herniation

Sub arachnoid haemorrhage

A

The presence of a blown right pupil is a sign of a third cranial nerve compression. The most likely cause is an extradural bleed. However, since this option is not listed the process of trans tentorial herniation would be the most applicable answer. Intraventricular bleeds are typically more common in premature neonates, deterioration due to hydrocephalus is more chronic.

Head injuries

Head injury is the commonest cause of death and disability in people aged 1-40 years in the UK. In the UK 1.4 million people will attend emergency departments each year with a recent head injury.
The typical patterns are described below:

Extradural haematoma Bleeding into the space between the dura mater and the skull. Often results from acceleration-deceleration trauma or a blow to the side of the head. The majority of extradural haematomas occur in the temporal region where skull fractures cause a rupture of the middle meningeal artery.

Features
Raised intracranial pressure
Some patients may exhibit a lucid interval
Subdural haematoma Bleeding into the outermost meningeal layer. Most commonly occur around the frontal and parietal lobes. May be either acute or chronic.

Risk factors include old age and alcoholism.

Slower onset of symptoms than a extradural haematoma.
Subarachnoid haemorrhage Usually occurs spontaneously in the context of a ruptured cerebral aneurysm but may be seen in association with other injuries when a patient has sustained a traumatic brain injury

How well did you know this?
1
Not at all
2
3
4
5
Perfectly
675
Q

Which of the nerves listed below is responsible for the innervation of gluteus maximus?

Inferior gluteal nerve

Superior gluteal nerve

Posterior femoral cutaneous nerve

Sciatic nerve

Perineal nerve

A

Superior and inferior gluteal nerve

Superior gluteal nerve
Arises from dorsal surface of the sacral plexus (L4, L5, S1)
Passes into gluteal region together with superior gluteal vessels
Supplies gluteus medius and minimus

Inferior gluteal nerve
Arises from dorsal surface of sacral plexus (L5, S1 and S2)
Runs medial to the posterior femoral cutaneous nerve
Enters gluteal region at inferior border of piriformis
Supplies gluteus maximus

How well did you know this?
1
Not at all
2
3
4
5
Perfectly
676
Q

Which of the following statements relating to branchial cysts is untrue?

The greater auricular nerve may be divided during excision

They typically occur in young adults

They move upwards on swallowing

They are rare over the age of 40 years

They are usually located in the anterior triangle of the neck

A

Nerves at risk during branchial cyst excision: Mandibular branch of facial nerve, greater auricular nerve and accessory nerve.
They do not move on swallowing. They should be diagnosed with caution in those aged >40 years, as lumps in this age group may in fact be metastatic disease from oropharyngeal cancer.

Neck lumps

The table below gives characteristic exam question features for conditions causing neck lumps:

Reactive lymphadenopathy By far the most common cause of neck swellings. There may be a history of local infection or a generalised viral illness
Lymphoma Rubbery, painless lymphadenopathy
The phenomenon of pain whilst drinking alcohol is very uncommon
There may be associated night sweats and splenomegaly
Thyroid swelling May be hypo-, eu- or hyperthyroid symptomatically
Moves upwards on swallowing
Thyroglossal cyst More common in patients < 20 years old
Usually midline, between the isthmus of the thyroid and the hyoid bone
Moves upwards with protrusion of the tongue
May be painful if infected
Pharyngeal pouch More common in older men
Represents a posteromedial herniation between thyropharyngeus and cricopharyngeus muscles
Usually not seen, but if large then a midline lump in the neck that gurgles on palpation
Typical symptoms are dysphagia, regurgitation, aspiration and chronic cough
Cystic hygroma A congenital lymphatic lesion (lymphangioma) typically found in the neck, classically on the left side
Most are evident at birth, around 90% present before 2 years of age
Branchial cyst An oval, mobile cystic mass that develops between the sternocleidomastoid muscle and the pharynx
Develop due to failure of obliteration of the second branchial cleft in embryonic development
Usually present in early adulthood
Cervical rib More common in adult females
Around 10% develop thoracic outlet syndrome
Carotid aneurysm Pulsatile lateral neck mass which doesn’t move on swallowing

How well did you know this?
1
Not at all
2
3
4
5
Perfectly
677
Q

A 56 year old machinist has his arm entrapped in a steel grinder and is brought to the emergency department. On examination, he is unable to extend his metacarpophalangeal joints and abduct his shoulder. He has weakness of his elbow and wrist. What has been injured?

Ulnar nerve

Axillary nerve

Medial cord of brachial plexus

Lateral cord of brachial plexus

Posterior cord of brachial plexus

A

The posterior cord gives rise to:
Radial nerve ((innervates the triceps, brachioradialis, wrist extensors, and finger extensors)
Axillary nerve (innervates deltoid and teres minor)
Upper subscapular nerve (innervates subscapularis)
Lower subscapular nerve (innervates teres major and subscapularis)
Thoracodorsal nerve (innervates latissimus dorsi)
This is a description of a posterior cord lesion. Remember that the posterior cord gives rise to the axillary and radial nerve.

How well did you know this?
1
Not at all
2
3
4
5
Perfectly
678
Q

A man is stabbed in the chest to the right of the manubriosternal angle. Which structure is least likely to be injured in this case?

Right pleura

The trachea

Right phrenic nerve

Right recurrent laryngeal nerve

Brachiocephalic vein

A

The right recurrent laryngeal nerve branches off the right vagus more proximally and arches posteriorly round the subclavian artery. So of the structures given it is the least likely to be injured.

Mediastinum

Region between the pulmonary cavities.
It is covered by the mediastinal pleura. It does not contain the lungs.
It extends from the thoracic inlet superiorly to the diaphragm inferiorly.

Mediastinal regions
Superior mediastinum (between manubriosternal angle and T4/5)
Middle mediastinum
Posterior mediastinum
Anterior mediastinum

Region Contents
Superior mediastinum
Superior vena cava
Brachiocephalic veins
Arch of aorta
Thoracic duct
Trachea
Oesophagus
Thymus
Vagus nerve
Left recurrent laryngeal nerve
Phrenic nerve
Anterior mediastinum
Thymic remnants
Lymph nodes
Fat
Middle mediastinum
Pericardium
Heart
Aortic root
Arch of azygos vein
Main bronchi
Posterior mediastinum
Oesophagus
Thoracic aorta
Azygos vein
Thoracic duct
Vagus nerve
Sympathetic nerve trunks
Splanchnic nerves

How well did you know this?
1
Not at all
2
3
4
5
Perfectly
679
Q

A 45 year old man has widespread metastatic adenocarcinoma of the colon. Which of these tumour markers is most likely to be elevated?

CA19-9

Carcinoembryonic antigen

Alpha Feto Protein

CA 125

Beta HCG

A

Screening for colonic cancer using CEA is not justified
Carcinoembryonic antigen is elevated in colonic cancer, typically in relation to disease extent with highest serum levels noted in metastatic disease. It is falsely elevated in a number of non-malignant disease states such as cirrhosis and colitis and for this reason it has no role in monitoring colitics for colonic cancer[1].
Reference
1. Sturgeon, C.M., L.C. Lai, and M.J. Duffy. Serum tumour markers: how to order and interpret them. BMJ, 2009. 339: p. b3527.

Colorectal cancer screening and diagnosis

Overview
Most cancers develop from adenomatous polyps. Screening for colorectal cancer has been shown to reduce mortality by 16%
The NHS now has a national screening programme offering screening every 2 years to all men and women aged 60 to 69 years. Patients aged over 70 years may request screening
Eligible patients are sent faecal occult blood (FOB) tests through the post. This is being replaced by FIT testing.
Patients with abnormal results are offered a colonoscopy
- The NHS BOSS flexible sigmoidoscopy screening comprises a single flexible sigmoidoscopy to patients aged 55 years

At colonoscopy, approximately:
5 out of 10 patients will have a normal exam
4 out of 10 patients will be found to have polyps which may be removed due to their premalignant potential
1 out of 10 patients will be found to have cancer

Diagnosis
Essentially the following patients need referral:
- Altered bowel habit for more than six weeks
- New onset of rectal bleeding
- Symptoms of tenesmus

Colonoscopy is the gold standard, provided it is complete and good mucosal visualisation is achieved. Other options include double contrast barium enema and CT colonography.

Staging
Once a malignant diagnosis is made patients with colonic cancer will be staged using chest / abdomen and pelvic CT. Patients with rectal cancer will also undergo evaluation of the mesorectum with pelvic MRI scanning.

For examination purposes the Dukes and TNM systems are preferred.

Tumour markers
Carcinoembryonic antigen (CEA) is the main tumour marker in colorectal cancer. Not all tumours secrete this, and it may be raised in conditions such as IBD. However, absolute levels do correlate (roughly) with disease burden and it is once again being used routinely in follow up.

How well did you know this?
1
Not at all
2
3
4
5
Perfectly
680
Q

A 28 year old man has a carcinoid tumour identified in his appendix. Blood testing for which of the substances listed below is likely to be helpful during follow up?

CA19-9

Alkaline phosphatase

AFP

CEA

Chromogranin A

A

It is important to distinguish between blood and urine tests for carcinoid syndrome. Blood tests usually measure chromogranin A,neuron-specific enolase (NSE), substance P, and gastrin. Urine tests usually measure 5 HIAA, which is a metabolite of serotonin. Sometimes blood tests for 5 hydroxytryptamine (serotonin) are also performed.

Carcinoid syndrome

Carcinoid tumours secrete serotonin
Originate in neuroendocrine cells mainly in the intestine (midgut-distal ileum/appendix)
Can occur in the rectum, bronchi
Hormonal symptoms mainly occur when disease spreads outside the bowel

Clinical features
Onset: insidious over many years
Flushing face
Palpitations
Pulmonary valve stenosis and tricuspid regurgitation causing dyspnoea
Asthma
Severe diarrhoea (secretory, persists despite fasting)

Investigation
5-HIAA in a 24-hour urine collection
Somatostatin receptor scintigraphy
CT scan
Blood testing for chromogranin A

Treatment
Octreotide
Surgical removal

How well did you know this?
1
Not at all
2
3
4
5
Perfectly
681
Q

A 44 year old lady presents with a mass lesion in the upper outer quadrant of the left breast. On clinical examination she has a 2cm mass lesion which on core biopsy is demonstrated to have invasive ductal carcinoma. An FNA of a bulky axillary lymph node contains malignant cells. What is the correct course of action?

Wide local excision and axillary node clearance

Radical mastectomy and axillary node clearance

Simple mastectomy and sentinel node biopsy

Wide local excision and sentinel node biopsy

Excision biopsy and sentinel node biopsy

A

Although the primary lesion is small enough for breast conserving surgery, the presence of overt axillary lymph node metastasis will attract a recommendation for axillary node clearance. Note that an excision biopsy is not appropriate for malignant cases.

Breast cancer management

  • Surgery is performed in most patients suffering from breast cancer.
    Chemotherapy may be used to downstage tumours and allow breast conserving surgery. Hormonal therapy may also be used for the same purposes.
    Radiotherapy is given to most patients who have undergone breast conserving surgery (some older patients receiving hormone treatment and who have small low grade tumours may safely avoid DXT.
    Therapeutic mammoplasty is an option for some patients but requires symmetrizing surgery in most cases.
    Patients who have undergone mastectomy may be offered a reconstructive procedure either in conjunction with their primary resection or as a staged procedure at a later date.

Surgical options
Mastectomy vs Wide local excision

Mastectomy Wide Local Excision
Multifocal tumour Solitary lesion
Central tumour Peripheral tumour
Large lesion in small breast Small lesion in large breast
DCIS >4cm DCIS <4cm
Patient Choice Patient choice

Central lesions may be managed using breast conserving surgery, where an acceptable cosmetic result may be obtained, this is rarely the case in small breasts

Axillary disease
As a minimum, all patients with invasive breast cancer should have their axilla staged. In those who do not have overt evidence of axillary nodal involvement this can be undertaken using sentinel lymph node biopsy.
Patients with a positive sentinel lymph node biopsy or who have imaging and cytological or histological evidence of axillary nodal metastasis should undergo axillary node clearance or axillary irradiation.
Axillary node clearance is associated with the development of lymphoedema, increased risk of cellulitis and frozen shoulder.

How well did you know this?
1
Not at all
2
3
4
5
Perfectly
682
Q

Which of the ABPI measurements shown below is most likely to be found in a 43 year old lady with long standing diabetes who complains of foot pain. It is worse at night and during minor exercise.

> 1.0

0.9

0.3

0.5

0.7

A

Diabetes may be complicated by vessel calcification and neuropathic pain. Therefore individuals may present with pain which is atypical for claudication both in terms of its tempo of onset and location.

Ankle-Brachial pressure index

Measurement of ankle- brachial pressure index (ABPI) is a commonly performed vascular investigation.
Calculated by dividing lower limb pressure by the highest upper limb pressure.

Results of ABPI
1.2 or greater Usually due to vessel calcification
1.0- 1.2 Normal
0.8-1.0 Minor stenotic lesion
Initiate risk factor management
0.50-0.8 Moderate stenotic lesion
Consider duplex
Risk factor management
If mixed ulcers present then avoid tight compression bandages
0.3 - 0.5 Likely significant stenosis
Duplex scanning to delineate lesions needed
Compression bandaging contra indicated
Less than 0.3 Indicative of critical ischaemia
Urgent detailed imaging required

How well did you know this?
1
Not at all
2
3
4
5
Perfectly
683
Q

Which of the following drugs causes hyperkalaemia?

Heparin

Ciprofloxacin

Salbutamol

Levothyroxine

Codeine phosphate

A

Both unfractionated and low-molecular weight heparin can cause hyperkalaemia. This is thought to be caused by inhibition of aldosterone secretion. Salbutamol is a recognised treatment for hyperkalaemia.

Hyperkalaemia

  • Plasma potassium levels are regulated by a number of factors including aldosterone, acid-base balance and insulin levels.
    Metabolic acidosis is associated with hyperkalaemia as hydrogen and potassium ions compete with each other for exchange with sodium ions across cell membranes and in the distal tubule.
    ECG changes seen in hyperkalaemia include tall-tented T waves, small P waves, widened QRS leading to a sinusoidal pattern and asystole

Causes of hyperkalaemia
Acute renal failure
Drugs*: potassium sparing diuretics, ACE inhibitors, angiotensin 2 receptor blockers, spironolactone, ciclosporin, heparin**
Metabolic acidosis
Addison’s
Tissue necrosis/rhabdomyolysis: burns, trauma
Massive blood transfusion

Foods that are high in potassium
Salt substitutes (i.e. Contain potassium rather than sodium)
Bananas, oranges, kiwi fruit, avocado, spinach, tomatoes

*beta-blockers interfere with potassium transport into cells and can potentially cause hyperkalaemia in renal failure patients - remember beta-agonists, e.g. Salbutamol, are sometimes used as emergency treatment

**both unfractionated and low-molecular weight heparin can cause hyperkalaemia. This is thought to be caused by inhibition of aldosterone secretion

How well did you know this?
1
Not at all
2
3
4
5
Perfectly
684
Q

Which of the following is not a risk factor for developing tuberculosis?

Gastrectomy

Solid organ transplantation with immunosupression

Intravenous drug use

Haematological malignancy

Amiodarone

A

Risk factors for developing active tuberculosis include:
silicosis
chronic renal failure
HIV positive
solid organ transplantation with immunosuppression
intravenous drug use
haematological malignancy
anti-TNF treatment
previous gastrectomy

Tuberculosis

Tuberculosis (TB) is an infection caused by Mycobacterium tuberculosis that most commonly affects the lungs. Understanding the pathophysiology of TB can be difficult - the key is to differentiate between primary and secondary disease.

Primary tuberculosis
A non-immune host who is exposed to M. tuberculosis may develop primary infection of the lungs. A small lung lesion known as a Ghon focus develops. The Ghon focus is composed of tubercle-laden macrophages. The combination of a Ghon focus and hilar lymph nodes is known as a Ghon complex

In immunocompetent people the initial lesion usually heals by fibrosis. Those who are immunocompromised may develop disseminated disease (miliary tuberculosis).

Secondary (post-primary) tuberculosis
If the host becomes immunocompromised the initial infection may become reactivated. Reactivation generally occurs in the apex of the lungs and may spread locally or to more distant sites. Possible causes of immunocomprise include:
immunosuppressive drugs including steroids
HIV
malnutrition

The lungs remain the most common site for secondary tuberculosis. Extra-pulmonary infection may occur in the following areas:
central nervous system (tuberculous meningitis - the most serious complication)
vertebral bodies (Pott’s disease)
cervical lymph nodes (scrofuloderma)
renal
gastrointestinal tract

How well did you know this?
1
Not at all
2
3
4
5
Perfectly
685
Q

Which of the following is a content of the adductor canal?

Saphenous nerve

Sural nerve

Femoral nerve

Profunda branch of the femoral artery

Saphenous vein

A

It contains the saphenous nerve, femoral vein and the superficial branch of the femoral artery.

Adductor canal

Also called Hunter’s or subsartorial canal
Immediately distal to the apex of the femoral triangle, lying in the middle third of the thigh. Canal terminates at the adductor hiatus.

Borders Contents
Laterally Vastus medialis muscle Saphenous nerve
Posteriorly Adductor longus, adductor magnus Superficial femoral artery
Roof Sartorius Superficial femoral vein

How well did you know this?
1
Not at all
2
3
4
5
Perfectly
686
Q

Which of these statements relating to the external carotid is false?

It ends by bifurcating into the superficial temporal and ascending pharyngeal artery

Its first branch is the superior thyroid artery

The superior thyroid, lingual and facial arteries all arise from its anterior surface

The ascending pharyngeal artery is a medial branch

Initially it lies anteromedial to the internal carotid

A

It terminates by dividing into the superficial temporal and maxillary branches. The external carotid has eight branches, 3 from its anterior surface ; thyroid, lingual and facial. The pharyngeal artery is a medial branch. The posterior auricular and occipital are posterior branches.
External carotid artery

The external carotid commences immediately lateral to the pharyngeal side wall. It ascends and lies anterior to the internal carotid and posterior to the posterior belly of digastric and stylohyoid. More inferiorly it is covered by sternocleidomastoid, passed by hypoglossal nerves, lingual and facial veins.
It then pierces the fascia of the parotid gland finally dividing into its terminal branches within the gland itself.

Surface marking of the carotid
This is an imaginary line drawn from the bifurcation of the common carotid passing behind the angle of the jaw to a point immediately anterior to the tragus of the ear.

Branches of the external carotid artery
It has six main branches, three in front, two behind and one deep.
Three in front Superior thyroid
Lingual
Facial
Two behind Occipital
Posterior auricular
Deep Ascending pharyngeal

It terminates by dividing into the superficial temporal and maxillary arteries in the parotid gland.

How well did you know this?
1
Not at all
2
3
4
5
Perfectly
687
Q

A 22 year old man sustains a blow to the side of his head with a baseball bat during a fight. He is initially conscious. However, he subsequently loses consciousness and then dies. Post mortem examination shows an extradural haematoma. The most likely culprit vessel is a branch of which of the following?

Middle cerebral artery

Internal carotid artery

Anterior cerebral artery

Maxillary artery

Mandibular artery

A

The middle meningeal artery is the most likely source of the extradural haematoma in this setting. It is a branch of the maxillary artery. The middle cerebral artery does not give rise to the middle meningeal artery. Note that the question is asking for the vessel which gives rise to the middle meningeal artery (‘the likely culprit vessel is a branch of which of the following’)

Middle meningeal artery

Middle meningeal artery is typically the third branch of the first part of the maxillary artery, one of the two terminal branches of the external carotid artery. After branching off the maxillary artery in the infratemporal fossa, it runs through the foramen spinosum to supply the dura mater (the outermost meninges) .
The middle meningeal artery is the largest of the three (paired) arteries which supply the meninges, the others being the anterior meningeal artery and the posterior meningeal artery.
The middle meningeal artery runs beneath the pterion. It is vulnerable to injury at this point, where the skull is thin. Rupture of the artery may give rise to an extra dural hematoma.
In the dry cranium, the middle meningeal, which runs within the dura mater surrounding the brain, makes a deep indention in the calvarium.
The middle meningeal artery is intimately associated with the auriculotemporal nerve which wraps around the artery making the two easily identifiable in the dissection of human cadavers and also easily damaged in surgery.

How well did you know this?
1
Not at all
2
3
4
5
Perfectly
688
Q

A 73 year old man undergoes an excision biopsy of a lymph node that is closely applied to sternocleidomastoid. This muscle is mobilized and a nerve that is present is damaged. Which muscle below is most likely to be affected?

Trapezius

Rhomboid major

Deltoid

Supraspinatus

Rhomboid minor

A

The accessory nerve has a number of lymph nodes applied to it near the sternocleidomastoid muscle. It is particularly at risk if SCM is mobilized. If injured, the trapezius muscle and SCM will be paralysed.

Trapezius

Origin Medial third of the superior nuchal line of the occiput
External occipital protruberance
Ligamentum nuchae
Spines of C7 and all thoracic vertebrae and all intervening interspinous ligaments
Insertion Posterior border of the lateral third of the clavicle
Medial border of the acromion
Upper border of the crest of the spine of the scapula
Nerve supply Spinal portion of the accessory nerve
Actions Elevation of the shoulder girdle
Lateral rotation of the scapula

How well did you know this?
1
Not at all
2
3
4
5
Perfectly
689
Q

A 39 year old man notices a swelling in his left hemiscrotum. On examination he has a left sided varicocele. The ipsilateral testis is normal on palpation. What is the most appropriate course of action?

A-Scrotal exploration and ligation of the varicocele

B-Abdominal ultrasound

C-Scrotal ultrasound

D-Left orchidectomy

E-Discharge

A

B-Abdominal ultrasound
A left sided varicocele is a recognized presenting sign of a renal tumour occluding the renal vein (into which left testicular vein drains). An abdominal ultrasound should be undertaken to exclude this. Surgery for uncomplicated varicocele is usually unnecessary.

How well did you know this?
1
Not at all
2
3
4
5
Perfectly
690
Q

A 3 month old boy is brought to the clinic by his mother who has noticed a swelling in the right inguinal region. On examination, there is a firm mass affecting the right spermatic cord distally, the testis is felt separately from it. What is the most likely diagnosis?
A-Inguino scrotal hernia
B-Rhabdomyosarcoma
C-Lymphatic filariasis
D-Torsion of testicular hydatid
E-Hydrocele

A

B-Rhabdomyosarcoma
Rhabdomyosarcomas are paratesticular tumours with a bimodal distribution. Because the mass is felt separate to the testis, this is the more likely diagnosis.
• 5% of testicular tumors
• Most often arises of distal portion of spermatic cord and may invade testis or surounding tissues
• Bimodal age distribution - 3-4 months - 16 years
• Arises from mesenchymal tissue - 90% embryonal variant (better prognosis) - 30% - 50% have metastasis (usually lymph node) at diagnosis

How well did you know this?
1
Not at all
2
3
4
5
Perfectly
691
Q

An 8 year-old boy of Caribbean descent presents with periumbilical abdominal pain. He has vomited twice and is refusing fluids. His temperature is 38.1oC and blood tests are as follows: Haemoglobin 8 g/dl, WCC 13 x 109/l, with a neutrophilia. What is the most likely diagnosis?

Pancreatitis

Sickle cell crisis

Appendicitis

Intussusception

Spontaneous bacterial peritonitis

A

Anaemia is seldom seen in appendicitis and if present should prompt a search for an alternative underlying diagnosis.
Sickle cell anaemia is characterised by severe chronic haemolytic anaemia resulting from poorly formed erythrocytes. Painful crises result from vaso-occlusive episodes, which may occur spontaneously or may be precipitated by infection. Consider this diagnosis in all children of appropriate ethnic background.

Acute abdominal pain-diagnoses

Conditions presenting with acute abdominal pain
Condition Features Investigations Management
Appendicitis History of migratory pain.
Fever.
Anorexia.
Evidence of right iliac fossa tenderness.
Mild pyrexia. Differential white cell count
Pregnancy test
C-Reactive protein
Amylase
Urine dipstick testing Appendicectomy
Mesenteric adenitis Usually recent upper respiratory tract infection.
High fever.
Generalised abdominal discomfort- true localised pain and signs are rare. Full blood count- may show slightly raised white cell count
Urine dipstick often normal
Abdominal ultrasound scan - usually no free fluid Conservative management- appendicectomy if diagnostic doubt
Mittelschmerz Only seen in females
Mid cycle pain
Usually occurs two weeks after last menstrual period
Pain usually has a supra-pubic location
Usually subsides over a 24-48 hour period. Full blood count- normal
Urine dipstick- normal
Abdominal and pelvic ultrasound- may show a trace of pelvic free fluid Manage conservatively if doubt or symptoms fail to settle then laparoscopy
Fitz-Hugh Curtis syndrome Disseminated infection with Chlamydia.
Usually seen in females.
Consists of evidence of pelvic inflammatory disease together with peri-hepatic inflammation and subsequent adhesion formation. Abdominal ultrasound scan- may show free fluid
High vaginal swabs - may show evidence of sexually transmitted infections Usually medically managed- doxycycline or azithromycin
Abdominal aortic aneurysm (ruptured) Sudden onset of abdominal pain radiating to the back in older adults (look for risk factors).
Collapse.
May be moribund on arrival in casualty, more stable if contained haematoma.
Careful clinical assessment may reveal pulsatile mass. Patients who are haemodynamically stable should have a CT scan Unstable patients should undergo immediate surgery (unless it is not in their best interests).
Those with evidence of contained leak on CT should undergo immediate surgery
Increasing unruptured aneurysmal size is an indication for urgent surgical intervention (that can wait until the next working day)
Perforated peptic ulcer Sudden onset of pain (usually epigastric).
Often preceding history of upper abdominal pain.
Soon develop generalised abdominal pain.
On examination may have clinical evidence of peritonitis. Erect CXR may show free air. A CT scan may be indicated where there is diagnostic doubt Laparotomy (laparoscopic surgery for perforated peptic ulcers is both safe and feasible in experienced hands)
Intestinal obstruction Colicky abdominal pain and vomiting (the nature of which depends on the level of the obstruction).
Abdominal distension and constipation (again depending upon site of obstruction).
Features of peritonism may occur where local necrosis of bowel loops is occurring. A plain abdominal film may help with making the diagnosis. A CT scan may be useful where diagnostic uncertainty exists In those with a virgin abdomen a lower and earlier threshold for laparotomy should exist than in those who may have adhesional obstruction
Mesenteric infarction Embolic events present with sudden pain and forceful evacuation.
Acute on chronic events usually have a longer history and previous weight loss.
On examination the pain is typically greater than the physical signs would suggest. Arterial pH and lactate
Arterial phase CT scanning is the most sensitive test Immediate laparotomy and resection of affected segments, in acute embolic events SMA embolectomy may be needed.

How well did you know this?
1
Not at all
2
3
4
5
Perfectly
692
Q

A 45 year old man with recurrent episodes of confusion is found to have a 1.5cm insulinoma of the pancreatic head. What is the most appropriate management?

Whipples procedure

Total pancreatectomy and en bloc splenectomy

Pylorus preserving pancreatico duodenectomy

Enucleation of the lesion

External beam radiotherapy

A

Most insulinomas are benign and radical resection is therefore not justified.

Insulinoma

  • Insulin producing tumours of the pancreatic β cells
    Incidence of 1 per 1,000,000 per year
    90% of lesions are benign
    Most tumours less than 2cm in size
    Between 5 and 10% have MEN type 1
    75% of patients with MEN 1 will develop pancreatic islet cell tumours

Typical features of insulinoma
Symptomatic hypoglycaemia during fasting
Concomitant blood glucose of less than 3mmol/L
Relief of hypoglycaemia by use of glucose

Testing
When neuroglycopenic symptoms occur blood is taken for serum insulin levels, serum glucose, C-peptide and pro insulin concentrations. The plasma insulin concentration is >10 micro U/ml in patients with the disorder.

Tumour localisation
USS (25% accuracy), endoscopic USS better (75% accuracy)
CT scanning (pancreatic protocol=40% accuracy)
Malignant insulinomas are larger and diagnostic accuracy with MRI is nearly 100% in such cases
Somatostatin receptor scintigraphy (50% accuracy)

Treatment
Since the majority of tumours are benign; the blind segmental resection of the pancreas (e.g. Whipples) cannot be justified, this may be considered acceptable for malignant lesions. The best approach at laparotomy is to corroborate pre operative imaging with intraoperative ultrasonography to identify the lesion. Tumours may be close to the pancreatic duct and this must be appreciated by the operating surgeon. The perioperative use of octreotide reduces the amount of pancreatic drainage, but not overall complications.

How well did you know this?
1
Not at all
2
3
4
5
Perfectly
693
Q

Which of the following is not a change found on an ECG in acute pulmonary embolism?

No changes

J waves

P pulmonale

Right ventricular strain

T wave inversion in the inferior leads

A

S1, Q3, T3
J waves are pathognomonic of hypothermia.

Pulmonary Embolism: ECG changes

  • No changes
    S1, Q3, T3
    Tall R waves: V1
    P pulmonale (peaked P waves): inferior leads
    Right axis deviation, Right bundle branch block
    Atrial arrhythmias
    T wave inversion: V1, V2, V3
    Right ventricular strain: if identified is associated with adverse short-term outcome and adds prognostic value to echocardiographic evidence of right ventricular dysfunction in patients with acute pulmonary embolism and normal blood pressure.

References
Vanni S et al. Prognostic value of ECG among patients with acute pulmonary embolism and normal blood pressure. Am J Med. 2009 Mar;122(3):257-64.

How well did you know this?
1
Not at all
2
3
4
5
Perfectly
694
Q

A 23 year old man presents with blunt abdominal trauma and a splenic bleed is suspected. He is commenced on an infusion of tranexamic acid. Which of the following best describes its mechanism of action?

Inhibition of plasmin

Inhibition of thrombin

Inhibition of factor II

Inhibition of factor Xa

Activation of factor VIII

A

Tranexamic acid inhibits plasmin and this prevents fibrin degradation.

Tranexamic acid

Tranexamic acid is a synthetic derivative of lysine. Its primary mode of action is as an anti fibrinolytic that competitively inhibits the conversion of plasminogen to plasmin. Plasmin degrades fibrin and therefore rendering plasmin inactive slows this process.
The role of tranexamic acid in trauma was investigated in the CRASH 2 trial and has been shown to be of benefit in bleeding trauma when administered in the first 3 hours.

How well did you know this?
1
Not at all
2
3
4
5
Perfectly
695
Q

A 33 year old lady presents to the emergency department following a road traffic accident. Her evaluation demonstrates a mild degree of anaemia and on examination, the spleen is palpable. In her family history, her mother underwent a splenectomy in her fifth decade. Which of the following laboratory test findings is most likely?

Positive Paul Bunnell test

Blood film showing elliptical cells

Blood film showing infection with plasmodium vivax

Reed Sternberg cells within the splenic tissue

Spur cells

A

It is likely that the patient has hereditary spherocytosis. Given the familial nature, EBV infection is unlikely and so a Paul Bunnell test will be negative. Lymphoma would be unusual and Spur cells are seen in association with liver disease and / or renal failure.

Hereditary Spherocytosis

Most common disorder of the red cell membrane, it has an incidence of 1 in 5000. The abnormally shaped erythrocytes are prone to splenic sequestration and destruction. This can result in hyperbilirubinaemia, jaundice and splenomegaly. In older patients an intercurrent illness may increase the rate of red cell destruction resulting in more acute symptoms.
Severe cases may benefit from splenectomy.

How well did you know this?
1
Not at all
2
3
4
5
Perfectly
696
Q

A 62 year old man is identified as having a rectal cancer. Following diagnostic work up no metastatic disease is identified. The tumour is 2cm from the anal verge. On MRI, lesion is T2, N0. Which of the following represents the correct course of action?

Proceed to abdomino-perineal excision of the colon and rectum (ELAPE)

Undertake a low anterior resection and loop ileostomy

Undertake a Hartmanns procedure

Offer radical external beam radiotherapy followed by abdomino-perineal excision of the colon and rectum (ELAPE)

Offer radical radiotherapy followed by low anterior resection and loop ileostomy

A

The tumour is too low for restorative surgery to be considered with an acceptable functional outcome. The tumour will therefore require an ELAPE style abdomino perineal resection. Since the lesion is T2 there is no prognostic benefit from adding radiotherapy which will confer additional morbidity.

Colorectal cancer treatment

Patients diagnosed as having colorectal cancer should be completely staged using CT of the chest/ abdomen and pelvis. Their entire colon should have been evaluated with colonoscopy or CT colonography. Patients whose tumours lie below the peritoneal reflection should have their mesorectum evaluated with MRI.

Once their staging is complete patients should be discussed within a dedicated colorectal MDT meeting and a treatment plan formulated.

Treatment of colonic cancer
Cancer of the colon is nearly always treated with surgery. Stents, surgical bypass and diversion stomas may all be used as palliative adjuncts. Resectional surgery is the only option for cure in patients with colon cancer. The procedure is tailored to the patient and the tumour location. The lymphatic drainage of the colon follows the arterial supply and therefore most resections are tailored around the resection of particular lymphatic chains (e.g. ileo-colic pedicle for right sided tumours). Some patients may have confounding factors that will govern the choice of procedure, for example a tumour in a patient from a HNPCC family may be better served with a panproctocolectomy rather than segmental resection. Following resection the decision has to be made regarding restoration of continuity. For an anastomosis to heal the key technical factors include; adequate blood supply, mucosal apposition and no tissue tension. Surrounding sepsis, unstable patients and inexperienced surgeons may compromise these key principles and in such circumstances it may be safer to construct an end stoma rather than attempting an anastomosis.
When a colonic cancer presents with an obstructing lesion; the options are to either stent it or resect. In modern practice it is unusual to simply defunction a colonic tumour with a proximal loop stoma. This differs from the situation in the rectum (see below).
Following resection patients with risk factors for disease recurrence are usually offered chemotherapy, a combination of 5FU and oxaliplatin is common.

Rectal cancer
The management of rectal cancer is slightly different to that of colonic cancer. This reflects the rectum’s anatomical location and the challenges posed as a result. Tumours located in the rectum can be surgically resected with either an anterior resection or an abdomino - perineal resection. The technical aspects governing the choice between these two procedures can be complex to appreciate and the main point to appreciate for the MRCS is that involvement of the sphincter complex or very low tumours require APER. In the rectum a 2cm distal clearance margin is required and this may also impact on the procedure chosen. In addition to excision of the rectal tube an integral part of the procedure is a meticulous dissection of the mesorectal fat and lymph nodes (total mesorectal excision/ TME). In rectal cancer surgery invovlement of the cirumferential resection margin carries a high risk of disease recurrence. Because the rectum is an extraperitoneal structure (until you remove it that is!) it is possible to irradiate it, something which cannot be offered for colonic tumours. This has a major impact in rectal cancer treatment and many patients will be offered neoadjuvent radiotherapy (both long and short course) prior to resectional surgery. Patients with T1, 2 and 3 /N0 disease on imaging do not require irradiation and should proceed straight to surgery. Patients with T4 disease will typically have long course chemo radiotherapy. Patients presenting with large bowel obstruction from rectal cancer should not undergo resectional surgery without staging as primary treatment (very different from colonic cancer). This is because rectal surgery is more technically demanding, the anastomotic leak rate is higher and the danger of a positive resection margin in an unstaged patient is high. Therefore patients with obstructing rectal cancer should have a defunctioning loop colostomy.

Summary of procedures
The operations for cancer are segmental resections based on blood supply and lymphatic drainage. These commonly performed procedures are core knowledge for the MRCS and should be understood.

Site of cancer Type of resection Anastomosis Risk of leak
Right colon Right hemicolectomy Ileo-colic Low <5%
Transverse Extended right hemicolectomy Ileo-colic Low <5%
Splenic flexure Extended right hemicolectomy Ileo-colic Low <5%
Splenic flexure Left hemicolectomy Colo-colon 2-5%
Left colon Left hemicolectomy Colo-colon 2-5%
Sigmoid colon High anterior resection Colo-rectal 5%
Upper rectum Anterior resection (TME) Colo-rectal 5%
Low rectum Anterior resection (Low TME) Colo-rectal
(+/- Defunctioning stoma) 10%
Anal verge Abdomino-perineal excision of colon and rectum None n/a

In the emergency setting, where the bowel has perforated, the risk of an anastomotic breakdown is much greater, particularly when the anastomosis is colon-colon. In this situation, an end colostomy is often safer and can be reversed later. When resection of the sigmoid colon is performed and an end colostomy is fashioned the operation is referred to as a Hartmans procedure. Whilst left sided resections are more risky, ileo-colic anastomoses are relatively safe even in the emergency setting and do not need to be defunctioned.

References
A review of the diagnosis and management of colorectal cancer and a summary of the UK National Institute of Clinical Excellence guidelines is provided in:
Poston G, et al . Diagnosis and management of colorectal cancer: summary of NICE guidance. BMJ 2011: 343: d 6751.

How well did you know this?
1
Not at all
2
3
4
5
Perfectly
697
Q

Parasympathetic fibres innervating the parotid gland originate from which of the following?

Submandibular ganglion

Otic ganglion

Ciliary ganglion

Pterygopalatine ganglion

None of the above

A

Otic ganglion
Secretion of saliva by the parotid gland is controlled by nerve fibres originating in the inferior salivatory nucleus; these leave the brain via the tympanic nerve (branch of glossopharyngeal nerve (CN IX), travel through the tympanic plexus (located in the middle ear), and then form the lesser petrosal nerve until reaching the otic ganglion. After synapsing in the Otic ganglion, the postganglionic (postsynaptic) fibres travel as part of the auriculotemporal nerve (a branch of the mandibular nerve (V3) to reach the parotid gland.

Parotid gland

Anatomy of the parotid gland
Location Overlying the mandibular ramus; anterior and inferior to the ear.
Salivary duct Crosses the masseter, pierces the buccinator and drains adjacent to the 2nd upper molar tooth (Stensen’s duct).
Structures passing through the gland
Facial nerve (Mnemonic: The Zebra Buggered My Cat; Temporal Zygomatic, Buccal, Mandibular, Cervical)
External carotid artery
Retromandibular vein
Auriculotemporal nerve
Relations
Anterior: masseter, medial pterygoid, superficial temporal and maxillary artery, facial nerve, stylomandibular ligament
Posterior: posterior belly digastric muscle, sternocleidomastoid, stylohyoid, internal carotid artery, mastoid process, styloid process
Arterial supply Branches of external carotid artery
Venous drainage Retromandibular vein
Lymphatic drainage Deep cervical nodes
Nerve innervation
Parasympathetic-Secretomotor
Sympathetic-Superior cervical ganglion
Sensory- Greater auricular nerve

Parasympathetic stimulation produces a water rich, serous saliva. Sympathetic stimulation leads to the production of a low volume, enzyme-rich saliva.

How well did you know this?
1
Not at all
2
3
4
5
Perfectly
698
Q

A 63 year old man undergoes a sub total gastrectomy for carcinoma of the stomach. Which of the sequelae below is least likely to occur?

Metabolic bone disease

Bile reflux

Dumping syndrome

Zinc deficiency

B12 deficiency

A

Zinc is mainly absorbed in the duodenum and jejunum. Bile reflux may occur post gastrectomy. The risk of bile reflux is lower if a Roux en Y reconstruction is used.

Post gastrectomy syndromes

Post gastrectomy syndromes may vary slightly depending upon whether a total or partial gastrectomy is performed. A Roux en Y reconstruction generally gives the best functional outcomes. Where a gastrojejunostomy is performed as reconstruction following a distal gastrectomy the gastric emptying is generally better if the jejunal limbs are tunneled in the retrocolic plane.

The following may occur following gastrectomy:
Small capacity (early satiety)
Dumping syndrome
Bile gastritis
Afferent loop syndrome
Efferent loop syndrome
Anaemia (B12 deficiency)
Metabolic bone disease

How well did you know this?
1
Not at all
2
3
4
5
Perfectly
699
Q

An injured axillary artery is ligated between the thyrocervical trunk of the subclavian and subscapular artery. Subsequent collateral circulation is likely to result in reversal of blood flow in which of the vessels listed below?

Circumflex scapular artery

Transverse cervical artery

Posterior intercostal arteries

Suprascapular artery

Profunda brachii artery

A

It’s an easy question really, we just made the wording difficult (on purpose). It is asking about the branches of the axillary artery and knowledge of the fact that there is an extensive collateral network around the shoulder joint. As a result, the occlusion of the proximal aspect of the circumflex humeral inflow (from the axillary artery) ceases and there is then retrograde flow through it from collaterals.
The circumflex scapular artery is a branch of the subscapular artery and normally supplies the muscle on the dorsal aspect of the scapula. In this instance, flow is reversed in the circumflex scapular and subscapular arteries forming a collateral circulation around the scapula.

Axillary artery

The axillary artery extends from the outer border of the first rib to the lower border of teres major, where it becomes the brachial artery. The vessel is subdivided into three zones; the first part lies above pectoralis minor, the second part is behind the muscle and the third part lies inferior to it.

First part
Together with the axillary vein, the artery is enclosed within the cords of the brachial plexus. Both vessels are contained within the axillary sheath, a prolongation of the prevertebral fascia. Posteriomedial to the sheath lies the first intercostal space, the superior aspect of the serratus anterior and the long thoracic nerve. Within the sheath, the medial cord of the brachial plexus lies behind the artery. Anteriorly lies the clavipectoral fascia. Superolaterally, lie the lateral and posterior cords of the brachial plexus. Inferomedially lies the axillary vein.

Second part
Posterior to the second part lies the posterior cord of the brachial plexus and the subscapularis muscle. Anteriorly, lie pectoralis minor and major. The lateral cord of the brachial plexus lies laterally. Medially, lies the medial cord of the brachial plexus, here it separates the artery from the vein.

Third part
Posterior to the artery lie subscapularis, latissimus dorsi and teres major. Interspersed between the vessel and subscapularis are the axillary and radial nerves. Anterior to the vessel is the medial root of the median nerve. Laterally, the lies the median and musculocutaneous nerves and coracobrachialis. The axillary vein is related medially.

Branches of the axillary artery
Highest thoracic artery
Thoraco-acromial artery
Lateral thoracic artery
Subscapular artery
Posterior circumflex humeral artery
Anterior circumflex humeral artery

How well did you know this?
1
Not at all
2
3
4
5
Perfectly
700
Q

A 64 year old man has a suspected lymphoma and lymph node biopsy from the posterolateral aspect of the right neck is planned. Which of the nerves listed is at greatest risk?

Accessory

Long thoracic

External laryngeal

Facial

Vagus

A

The accessory nerve has a superficial course and is easily injured. It lies under platysma and may be divided during the early part of the procedure.

Accessory nerve

The cranial root of the accessory nerve arises from the caudal two thirds of the nucleus ambiguus and the caudal four fifths of the dorsal nucleus of the vagus. The cranial root emerges as four rootlets from the dorsolateral surface of the medulla oblongata below those of the vagus. It then traverses the jugular foramen. On exiting the jugular foramen it separates from its spinal part. Whereupon its cranial fibres joint those of the vagus to innervate some of the palatal muscles. The fibres arising from the spinal root exit near the junction between the spinal cord and the medulla. The fibres pass rostrally to unite with the cranial roots to exit through the jugular foramen. As outlined above these separate on exiting the foramen. The spinal part then crosses the transverse process of the atlas, and is crossed by the occipital artery as it does so.

It descends obliquely, medial to the styloid process, stylohyoid and the posterior belly of digastric. It then reaches the upper part of sternocleidomastoid to enter its upper surface. It typically exits this muscle a little above the midpoint of the posterior aspect of it. This point is usually 4-6cm below the tip of the mastoid process. It crosses the posterior triangle on the levator scapulae separated from it by the pre vertebral layer of deep cervical fascia. At this point, the nerve is superficial and related to the superficial cervical lymph nodes. Approximately 3-5 cm above the clavicle it passes behind the anterior border of trapezius which it innervates.

How well did you know this?
1
Not at all
2
3
4
5
Perfectly
701
Q

A 26 year old man who smokes heavily develops aching, crampy pains in his legs. On examination, distal limb pulses are diminished. What is the most likely cause?

Vasculitis

Steal syndrome

Clot embolus

Vasospasm

Arterial injury

A

This is likely to represent Buergers disease. It is commonest in young males who smoke heavily.

Arterial occlusions/ insufficiency

Arterial occlusions may occur as a result of a number of processes. The typical clinical scenarios are outlined below.

Cause of occlusion Typical picture
Embolus Sudden onset
Depending upon level of occlusion; limb may show typical features of pain, loss of pulses and pallor. Sensory perceptive changes may also be present
Thrombosis Usually known disease and prodromal symptoms e.g. claudication
Disruption to flow may be incomplete
If background disease process present then collaterals may be present and picture less dramatic
Vasospasm May be due to Raynauds and affect extremities
Symptoms are often temperature related
Discolouration of the hands may occur (pale, dark, red)
Symptoms improve during pregnancy (hyperdynamic circulation)
Steal syndromes Occur secondary to arteriovenous fistula, or partial arterial occlusions (e.g. cervical rib)
Pain and diminished pulses distal to fistula are seen

Vasculitis
Vessel diameter and vasculitis classification
Aorta and branches
Takayasu’s arteritis
Buergers disease
Giant cell arteritis
Large and medium sized arteries
Buergers disease
Giant cell arteritis
Polyarteritis nodosa
Medium sized muscular arteries
Polyarteritis nodosa
Wegeners granulomatosis
Small muscular arteries
Wegeners granulomatosis
Rheumatoid vasculitis

Specific conditions
Takyasu’s arteritis
Inflammatory, obliterative arteritis affecting aorta and branches
Females> Males
Symptoms may include upper limb claudication
Clinical findings include diminished or absent pulses
ESR often affected during the acute phase
Buergers disease
Segmental thrombotic occlusions of the small and medium sized lower limb vessels
Commonest in young male smokers
Proximal pulses usually present, but pedal pulses are lost
An acuter hypercellular occlusive thrombus is often present
Tortuous corkscrew shaped collateral vessels may be seen on angiography
Giant cell arteritis
Systemic granulomatous arteritis that usually affects large and medium sized vessels
Females > Males
Temporal arteritis is commonest type
Granulomatous lesions may be seen on biopsy (although up to 50% are normal)
Polyarteritis nodosa
Systemic necrotising vasculitis affecting small and medium sized muscular arteries
Most common in populations with high prevalence of hepatitis B
Renal disease is seen in 70% cases
Angiography may show saccular or fusiform aneurysms and arterial stenoses
Wegeners granulomatosis
Predominantly affects small and medium sized arteries
Systemic necrotising granulomatous vasculitis
Cutaneous vascular lesions may be seen (ulceration, nodules and purpura)
Sinus imaging may show mucosal thickening and air fluid levels

How well did you know this?
1
Not at all
2
3
4
5
Perfectly
702
Q

A 22 year old man is involved in a road traffic accident. He is found to have a pelvic fracture. While on the ward the nursing staff report that he is complaining of lower abdominal pain. On examination, you find a distended tender bladder and blood at the urethral meatus. What is the best management?

10 Ch foley urethral catheter

Suprapubic catheter

16 Ch foley urethral catheter

18 Ch coude tip urethral catheter

Pain relief and review in 1 hour

A

This patient has possible urethral injury based on the history. Urethral catheterisation is contraindicated in this situation.

Lower genitourinary tract trauma

Most bladder injuries occur due to blunt trauma
85% associated with pelvic fractures
Easily overlooked during assessment in trauma
Up to 10% of male pelvic fractures are associated with urethral or bladder injuries

Types of injury

Urethral injury
Mainly in males
Blood at the meatus (50% cases)
There are 2 types:
i.Bulbar rupture
- most common
- straddle type injury e.g. bicycles
- triad signs: urinary retention, perineal haematoma, blood at the meatus
ii. Membranous rupture
- can be extra or intraperitoneal
- commonly due to pelvic fracture
- Penile or perineal oedema/ hematoma
- PR: prostate displaced upwards (beware co-existing retroperitoneal haematomas as they may make examination difficult)

  • Investigation: ascending urethrogram
  • Management: suprapubic catheter (surgical placement, not percutaneously)
    External genitalia injuries (i.e., the penis and the scrotum)
    Secondary to injuries caused by penetration, blunt trauma, continence- or sexual pleasure-enhancing devices, and mutilation
    Bladder injury
    rupture is intra or extraperitoneal
    presents with haematuria or suprapubic pain
    history of pelvic fracture and inability to void: always suspect bladder or urethral injury
    inability to retrieve all fluid used to irrigate the bladder through a Foley catheter indicates bladder injury
    investigation- IVU or cystogram
    management: laparotomy if intraperitoneal, conservative if extraperitoneal
How well did you know this?
1
Not at all
2
3
4
5
Perfectly
703
Q

A 50 year old female slips on wet floor injuring her ankle. On examination, she has tenderness over the lateral and medial malleolus. X-rays (stress views) demonstrate an undisplaced fracture of the distal fibula at the level of the syndesmosis and a congruent ankle mortise. What is the most appropriate management?

Application of full leg cast

Surgical fixation

Application of below knee walking boot

Application of external fixator

Bed rest, splinting and traction

A

Application of below knee walking boot
This is a Weber B fracture and therefore potentially unstable. Medial malleolar tenderness indicates deltoid ligament injury. As the fracture is currently undisplaced and the ankle mortise is congruent, the injury can be initially managed conservatively in a moon boot but the patient should be monitored in the outpatient clinic for fracture displacement in the first few weeks.

Ankle injuries

An ankle fracture relates to a fracture around the tibio-talar joint. It generally refers to a fracture involving the lateral, and/or medial and/or posterior malleolus. Pilon and Tillaux fractures are also considered to be ankle fractures, but are not covered here.
Ankle fractures are common. They effect men and women in equal numbers, but men have a higher rate as young adults (sports and contact injuries), and women a higher rate post-menopausal (fragility type fracture).

Osseous anatomy
The ankle (or mortise) joint consists of the distal tibia (tibial plafond and posterior malleolus), the distal fibula (lateral malleolus), and the talus. The main movement at the ankle joint is plantar and dorsiflexion.

Ligamentous anatomy
Medial side: Deltoid ligament. This is divided into superficial and deep portions. It is the primary restraint to valgus tilting of the talus.

Lateral side: Lateral ligament complex consisting from anterior to posterior of the anterior talofibular ligament (ATFL), calcaneofibular ligament (CFL), and the posterior talofibular ligament (PTFL). Together they resist valgus stress to the ankle, and are a restraint to anterior translation of the talus within the mortise joint.

Syndesmosis: The syndesmosis is a ligament complex between the distal tibia and fibula, holding the two bones together. It is fundamental to the integrity of the ankle joint, and its disruption leads to instability. It consists of (from anterior to posterior) the anterior-inferior tibiofibular ligament (AITFL), the transverse tibiofibular ligament (TTFL), the interosseous membrane, and the posterior-inferior tibiofibular ligament (PITFL).

Presentation and initial management
Patients will present following a traumatic event with a painful, swollen ankle, and reluctance/inability to weight bear. The Ottawa rules can be applied to differentiate between an ankle fracture and sprain, but can be unreliable.
In high energy injuries, management should follow ATLS principles to identify more significant injuries first. Neurovascular status of the foot should be documented, and open injuries should be excluded. If an open injury is identified, it should be managed in line with BOAST 4 principles1. If an obvious deformity exists, it should be reduced as soon as possible with appropriate analgesia or conscious sedation. Radiographs of clearly deformed or dislocated joints are not necessary, and removing the pressure on the surrounding soft tissues from the underlying bony deformity is the priority. If the fracture pattern is not clinically obvious then plain radiographs are appropriate and will guide the subsequent manipulation during plaster-of-paris below knee backslab application.

Imaging
AP, lateral and mortise views (20 degrees internal rotation) are essential to evaluate fracture displacement and syndesmotic injury. Decreased tibiofibular overlap, medial joint clear space and lateral talar shift all indicate a syndesmotic injury. (In subtle cases of shift, imaging the uninjured ankle can be helpful as a proportion of the population have little or no tibiotalar overlap-reference 2.)

Where there is suspicion of syndesmosis involvement in the absence of radiographic evidence, stress radiographs can be diagnostic.
Complex fracture patterns (and increasingly posterior malleolar fractures) are best defined using CT.

Classification
The most commonly used classifications are Lauge-Hansen and Danis-Weber.

Lauge-Hansen
Comprises two parts: first part is the foot position, and the second part is the force applied. Useful for understanding the forces involved and therefore predict the ligamentous or bony injury. Results in four injury patterns:
Supination - Adduction (SA) - 10-20%
Supination - External rotation (SER) - 40-75%
Pronation - Abduction (PA) - 5-20%
Pronation - External rotation (PER) - 5-20%

Not often used in clinical practice but good for understanding the principles of ankle fracture.

Danis-Weber
Commonly used. Based on the level of the fibula fracture in relation to the syndesmosis. The more proximal, the greater the risk of syndesmotic injury and therefore fracture instability.
A - fracture below the level of the syndesmosis
B - fracture at the level of the syndesmosis / level of the tibial plafond
C - fracture above the level of the syndesmosis. This includes Maisonneuve fractures (proximal fibula fracture), which can be associated with ankle instability. Beware the high fibula fracture - it may be an ankle fracture!

The Weber classification is based purely on the the lateral side. All injuries can include a medial or posterior bony or ligamentous injury which also dictates fracture stability (bimalleolar and trimalleolar fractures are more unstable).

Treatment
When deciding upon treatment for an ankle fracture, one must consider both the fracture and the patient. Diabetic patients and smokers are at greater risk of post-operative complication, especially wound problems and infection. Likewise, the long term outcome of post-traumatic arthritis from a malunited ankle fracture is extremely important for a young patient, but not as relevant in the elderly. Therefore, normal surgical decision processes apply as with all fractures.

Defining stability of an ankle fracture underpins the treatment decision.
Weber A - Unimalleolar Weber A Weber fractures by definition are stable and therefore can be mobilised fully weight bearing in an ankle boot.

Weber C - Fractures tend to include syndesmotic disruption and are usually bimalleolar (either bony or ligamentous). They are therefore unstable and usually require operative fixation. In addition to the fracture fixation, the syndesmosis usually requires reconstruction/augmentation with screws to restore the joint integrity and function.

Weber B - B fractures vary greatly. They can be part of a trimalleolar injury and therefore extremely unstable, requiring fixation. Alternatively, a uni-malleolar Weber B fracture can be a stable injury, and therefore mobilised immediately in an ankle boot. Defining the stability can be challenging, and often involves stress radiographs, or a trial of mobilisation and repeat radiographs. However, treating undisplaced ankle fractures in a below knee plaster, non-weight bearing for six weeks is still widely practised, and a safe approach.

When operative fixation is appropriate, it is usually via open reduction and internal fixation using plates and screws. It must be carried out when soft tissue swelling has settled in order to minimise the risk of wound problems. This can often take a week to settle.

The use of fibula nails is expanding, but is not yet mainstream. Ankle fractures can also be treated with external fixation, or with a hind foot nail in patients who need fixation but where soft tissue or bone quality is poor.

Post operative management
Ankle fractures generally take 6 weeks to unite enough to prevent secondary displacement. This is therefore an appropriate time period to keep a cast on in a conservatively managed patient. Weight bearing post-operatively depends on the quality of the fixation and bone quality, and preference varies between surgeons, ranging from aggressive early mobilisation to a period of non-weight bearing. Return to activities takes approximately three months, and often requires assistance of a physiotherapist to improve range-of-movement and muscle strengthening.

References
1. http://www.boa.ac.uk/publications/boa-standards-trauma-boasts/
2. Shah AS, Kadakia AR, Tan GJ, Karadsheh MS, Wolter TD, Sabb B. Radiographic evaluation of the normal distal tibiofibular syndesmosis. Foot Ankle Int. 2012;33(10):870-6

How well did you know this?
1
Not at all
2
3
4
5
Perfectly
704
Q

A 32 year old female hits her head on the steering wheel during a collision with another car. She has periorbital swelling and a flattened appearance of the face. What is the most likely injury?

Le Fort 1 fracture affecting maxilla

Le Fort 3 fracture affecting the maxilla

Mandibular fracture

Unilateral fracture of the zygoma

Isolated temporal bone injury

A

The flattened appearance of the face is a classical description of the dish/pan face associated with Le fort fracture 2 or 3 of the maxilla.

Craniomaxillofacial injuries

Craniomaxillofacial injuries in the UK are due to:
Interpersonal violence (52%)
Motor vehicle accidents (16%)
Sporting injuries (19%)
Falls (11%)

Le Fort Fractures
Grade Feature
Le Fort 1 The fracture extends from the nasal septum to the lateral pyriform rims, travels horizontally above the teeth apices, crosses below the zygomaticomaxillary junction, and traverses the pterygomaxillary junction to interrupt the pterygoid plates.
Le Fort 2 These fractures have a pyramidal shape and extend from the nasal bridge at or below the nasofrontal suture through the frontal process of the maxilla, inferolaterally through the lacrimal bones and inferior orbital floor and rim through or near the inferior orbital foramen, and inferiorly through the anterior wall of the maxillary sinus; it then travels under the zygoma, across the pterygomaxillary fissure, and through the pterygoid plates.
Le Fort 3 These fractures start at the nasofrontal and frontomaxillary sutures and extend posteriorly along the medial wall of the orbit through the nasolacrimal groove and ethmoid bones. The thicker sphenoid bone posteriorly usually prevents continuation of the fracture into the optic canal. Instead, the fracture continues along the floor of the orbit along the inferior orbital fissure and continues superolaterally through the lateral orbital wall, through the zygomaticofrontal junction and the zygomatic arch. Intranasally, a branch of the fracture extends through the base of the perpendicular plate of the ethmoid, through the vomer, and through the interface of the pterygoid plates to the base of the sphenoid. This type of fracture predisposes the patient to CSF rhinorrhea more commonly than the other types.

Ocular injuries
Superior orbital fissure syndrome
Severe force to the lateral wall of the orbit resulting in compression of neurovascular structures. Results in :
Complete opthalmoplegia and ptosis (Cranial nerves 3, 4, 6 and nerve to levator palpebrae superioris)
Relative afferent pupillary defect
Dilatation of the pupil and loss of accommodation and corneal reflexes
Altered sensation from forehead to vertex (frontal branch of trigeminal nerve)

Orbital blow out fracture
Typically occurs when an object of slightly larger diameter than the orbital rim strikes the incompressible eyeball. The bone fragment is displaced downwards into the antral cavity, remaining attached to the orbital periosteum. Periorbital fat may be herniated through the defect, interfering with the inferior rectus and inferior oblique muscles which are contained within the same fascial sheath. This prevents upward movement and outward rotation of the eye and the patient experiences diplopia on upward gaze. The initial bruising and swelling may make assessment difficult and patients should usually be reviewed 5 days later. Residual defects may require orbital floor reconstruction.

Nasal Fractures
Common injury
Ensure new and not old deformity
Control epistaxis
CSF rhinorrhoea implies that the cribriform plate has been breached and antibiotics will be required.
Usually best to allow bruising and swelling to settle and then review patient clinically. Major persistent deformity requires fracture manipulation, best performed within 10 days of injury.

Retrobulbar haemorrhage
Rare but important ocular emergency. Presents with:
Pain (usually sharp and within the globe)
Proptosis
Pupil reactions are lost
Paralysis (eye movements lost)
Visual acuity is lost (colour vision is lost first)
May be the result of Le Fort type facial fractures.

Management:
Mannitol 1g/Kg as 20% infusion, Osmotic diuretic, Contra-indicated in congestive heart failure and pulmonary oedema
Acetazolamide 500mg IV, (Monitor FBC/U+E) Reduces aqueous pressure by inhibition of carbonic anhydrase (used in glaucoma)
Dexamethasone 8mg orally or intravenously
In a traumatic setting an urgent cantholysis may be needed prior to definitive surgery.

Consider
Papaverine 40mg smooth muscle relaxant
Dextran 40 500mls IV improves perfusion

How well did you know this?
1
Not at all
2
3
4
5
Perfectly
705
Q

A 45 year old man with long standing ulcerative colitis and rectal dysplasia presents with a DALM lesion in the rectum. What is the most appropriate management option?

Snare polypectomy

Repeat endoscopy in 2 years

Discharge

Anterior resection

Panproctocolectomy

A

DALM lesions complicating ulcerative colitis should be managed with panproctocolectomy. An anterior resection is inadequate since it will only remove the rectum and ulcerative colitis affects the entire colon. Since many will be associated with invasion a snare polypectomy is not sufficient either.

Colonic lesions - DALM

The term DALM lesion refers to a Dysplasia Associated Lesion or Mass.
They may complicate dysplasia occurring in patients with longstanding ulcerative colitis.
They have a high incidence of invasive foci.
When they complicate longstanding ulcerative colitis, they should be treated by panproctocolectomy.

How well did you know this?
1
Not at all
2
3
4
5
Perfectly
706
Q

A 48 year old man is recovering on the high dependency unit following a long and complex laparotomy. His preoperative medication includes an ACE inhibitor for blood pressure control. For the past two hours he has been oliguric with a urine output of 10ml/hr-1. What the most appropriate immediate course of action?

Stop the ACE inhibitor

Administer a fluid challenge

Start an infusion of nor adrenaline

Administer intravenous frusemide

Insert a Swann-Ganz Catheter

A

Hypovolaemia is the most likely cause for oliguria and a fluid challenge is the most appropriate action. Blind administration of inotropes to hypovolaemic patients is unwise, with the possible exception of cardiac patients.

Hypovolaemia and the surgical patient

Hypovolaemia often represents the end point of multiple pathological processes. It may be divided into the following categories; overt compensated hypovolaemia, covert compensated hypovolaemia and decompensated hypovolaemia. Of these three categories the covert compensated subtype of hypovolaemia remains the commonest and is accounted for by the fact that class I shock will often produce no overtly discernible clinical signs. This is due, in most cases, to a degree of splanchnic autotransfusion. The most useful diagnostic test for detection of covert compensated hypovolaemia remains urinanalysis. This often shows increased urinary osmolality and decreased sodium concentration.

In overt compensated hypovolaemia the blood pressure is maintained although other haemodynamic parameters may be affected. This correlates to class II shock. In most cases assessment can be determined clinically. Where underlying cardiopulmonary disease may be present the placement of a CVP line may guide fluid resuscitation. Severe pulmonary disease may produce discrepancies between right and left atrial filling pressures. This problem was traditionally overcome through the use of Swann-Ganz catheters.

Untreated, hypovolaemia may ultimately become uncompensated with resultant end organ dysfunction. Microvascular hypoperfusion may result in acidosis with a subsequent myocardial depressive effect, thereby producing a vicious circle.

The treatment of hypovolaemia is with intravenous fluids. In the first instance a fluid challenge such as the rapid infusion of 250ml of crystalloid will often serve as both a diagnostic and resuscitative measure. In the event that this fails to produce the desired response the patient will need to be re-evaluated clinically. More fluid may be needed. However, it is important not to overlook mechanical ureteric obstruction in the anuric, normotensive patient.

How well did you know this?
1
Not at all
2
3
4
5
Perfectly
707
Q

A 56 year old man presents with lethargy, haematuria and haemoptysis. On examination he is hypertensive and has a right loin mass. A CT scan shows a lesion affecting the upper pole of the right kidney, it has a small cystic centre. Which of the options below is the most likely diagnosis?

Squamous cell carcinoma of the kidney

Nephroblastoma

Renal adenocarcinoma

Transitional cell carcinoma of the kidney

Polycystic kidney disease

A

Renal adenocarcinoma are the most common renal tumours. These will typically affect the renal parenchyma. Transitional cell carcinoma will usually affect urothelial surfaces. Nephroblastoma would be very rare in this age group. Renal adenocarcinoma may produce cannon ball metastasis in the lung which cause haemoptysis, this is not a feature of PKD.

Renal tumours

Renal cell carcinoma
Renal cell carcinoma is an adenocarcinoma of the renal cortex and is believed to arise from the proximal convoluted tubule. They are usually solid lesions, up to 20% may be multifocal, 20% may be calcified and 20% may have either a cystic component or be wholly cystic. They are often circumscribed by a pseudocapsule of compressed normal renal tissue. Spread may occur either by direct extension into the adrenal gland, renal vein or surrounding fascia. More distant disease usually occurs via the haematogenous route to lung, bone or brain.
Renal cell carcinoma comprise up to 85% of all renal malignancies. Males are more commonly affected than females and sporadic tumours typically affect patients in their sixth decade.
Patients may present with a variety of symptoms including; haematuria (50%), loin pain (40%), mass (30%) and up to 25% may have symptoms of metastasis.Less than 10% have the classic triad of haematuria, pain and mass.

Investigation
Many cases will present as haematuria and be discovered during diagnostic work up. Benign renal tumours are rare, so renal masses should be investigated with multislice CT scanning. Most tumours are also characterised with an arterial phase CT, particularly if they may be suitable for partial nephrectomy.

CT scanning of the chest and abdomen to detect distant disease should also be undertaken.

Routine bone scanning is not indicated in the absence of symptoms.

Biopsy should not be performed when a nephrectomy is planned but is mandatory before any ablative therapies are undertaken.

Assessment of the functioning of the contra lateral kidney.

Management
T1 lesions may be managed by partial nephrectomy and this gives equivalent oncological results to total radical nephrectomy. Partial nephrectomy may also be performed when there is inadequate reserve in the remaining kidney. Ablative techniques may also be considered for small T1 lesions in unfit patients. In general, outcomes are less favorable than with surgical resection.

For T2 lesions and above a radical nephrectomy is standard practice and this may be performed via a laparoscopic or open approach. Preoperative embolisation is not indicated nor is resection of uninvolved adrenal glands. During surgery early venous control is mandatory to avoid shedding of tumour cells into the circulation.

Patients with completely resected disease do not benefit from adjuvant therapy with either chemotherapy or biological agents. These should not be administered outside the setting of clinical trials.

Patients with transitional cell cancer will require a nephroureterectomy with disconnection of the ureter at the bladder.

References
Capitanio U, Montorsi F. Renal cancer. Lancet. 2016 Feb 27;387(10021):894-906.

Capitanio U et al. Epidemiology of Renal Cell Carcinoma. Eur Urol. 2019 Jan;75(1):74-84.

How well did you know this?
1
Not at all
2
3
4
5
Perfectly
708
Q

A 89 year old woman presents with long standing seborrhoeic warts of her abdominal wall , they have caused troublesome itching. What is the best treatment?

Administration of topical steroids

Shave excision and cautery

Excision and primary closure

Excision and skin graft

Excision biopsy

A

These lesions are often extensive and superficial. Shave excision will suffice, material must be sent for histology.

Skin Diseases

Skin lesions may be referred for surgical assessment, but more commonly will come via a dermatologist for definitive surgical management.

Skin malignancies include basal cell carcinoma, squamous cell carcinoma and malignant melanoma.

Basal Cell Carcinoma
Most common form of skin cancer.
Commonly occur on sun exposed sites apart from the ear.
Sub types include nodular, morphoeic, superficial and pigmented.
Typically slow growing with low metastatic potential.
Standard surgical excision, topical chemotherapy and radiotherapy are all successful.
As a minimum a diagnostic punch biopsy should be taken if treatment other than standard surgical excision is planned.

Squamous Cell Carcinoma
Again related to sun exposure.
May arise in pre - existing solar keratoses.
May metastasize if left.
Immunosupression (e.g. following transplant), increases risk.
Wide local excision is the treatment of choice and where a diagnostic excision biopsy has demonstrated SCC, repeat surgery to gain adequate margins may be required.

Malignant Melanoma
The main diagnostic features (major criteria):
Change in size
Change in shape
Change in colour
Secondary features (minor criteria)
Diameter >6mm
Inflammation
Oozing or bleeding
Altered sensation

Treatment
Suspicious lesions should undergo excision biopsy. The lesion should be removed completely as incision biopsy can make subsequent histopathological assessment difficult.
Once the diagnosis is confirmed the pathology report should be reviewed to determine whether further re-excision of margins is required (see below):

Margins of excision-Related to Breslow thickness
Lesions 0-1mm thick 1cm
Lesions 1-2mm thick 1- 2cm (Depending upon site and pathological features)
Lesions 2-4mm thick 2-3 cm (Depending upon site and pathological features)
Lesions >4 mm thick 3cm
Marsden J et al. Revised UK guidelines for management of Melanoma. Br J Dermatol 2010 163:238-256.

Further treatments such as sentinel lymph node mapping, isolated limb perfusion and block dissection of regional lymph node groups should be selectively applied.

Kaposi Sarcoma
Tumour of vascular and lymphatic endothelium.
Purple cutaneous nodules.
Associated with immuno supression.
Classical form affects elderly males and is slow growing.
Immunosupression form is much more aggressive and tends to affect those with HIV related disease.

Non malignant skin disease

Dermatitis Herpetiformis
Chronic itchy clusters of blisters.
Linked to underlying gluten enteropathy (coeliac disease).

Dermatofibroma
Benign lesion.
Firm elevated nodules.
Usually history of trauma.
Lesion consists of histiocytes, blood vessels and fibrotic changes.

Pyogenic granuloma
Overgrowth of blood vessels.
Red nodules.
Usually follow trauma.
May mimic amelanotic melanoma.

Acanthosis nigricans
Brown to black, poorly defined, velvety hyperpigmentation of the skin.
Usually found in body folds such as the posterior and lateral folds of the neck, the axilla, groin, umbilicus, forehead, and other areas.
The most common cause of acanthosis nigricans is insulin resistance, which leads to increased circulating insulin levels. Insulin spillover into the skin results in its abnormal increase in growth (hyperplasia of the skin).
In the context of a malignant disease, acanthosis nigricans is a paraneoplastic syndrome and is then commonly referred to as acanthosis nigricans maligna. Involvement of mucous membranes is rare and suggests a coexisting malignant condition.

How well did you know this?
1
Not at all
2
3
4
5
Perfectly
709
Q

A 78 year old man is referred to the clinic by his general practitioner. For many years he noticed a smooth swelling approximately 2cm anterior to the tragus of his right ear. Apart from being a heavy smoker he has no co-morbidities. What is the most likely diagnosis?

Pleomorphic adenoma

Liposarcoma

Warthins tumour

Adenocarcinoma

None of the above

A

Warthins tumours are most common in elderly smokers. They have a relatively benign and indolent course.

Parotid gland clinical

Benign neoplasms
Up to 80% of all salivary gland tumours occur in the parotid gland and up to 80% of these are benign. There is no consistent correlation between the rate of growth and the malignant potential of the lesion. However, benign tumours should not invade structures such as the facial nerve.
With the exception of Warthins tumours, they are commoner in women than men. The median age of developing a lesion is in the 5th decade of life.

Benign tumour types
Tumour type Features
Benign pleomorphic adenoma or benign mixed tumor Most common parotid neoplasm (80%)
Proliferation of epithelial and myoepithelial cells of the ducts and an increase in stromal components
Slow growing, lobular, and not well encapsulated
Recurrence rate of 1-5% with appropriate excision (parotidectomy)
Recurrence possibly secondary to capsular disruption during surgery
Malignant degeneration occurring in 2-10% of adenomas observed for long periods, with carcinoma ex-pleomorphic adenoma occurring most frequently as adenocarcinoma
Warthin tumor (papillary cystadenoma lymphoma or adenolymphoma) Second most common benign parotid tumor (5%)
Most common bilateral benign neoplasm of the parotid
Marked male as compared to female predominance
Occurs later in life (sixth and seventh decades)
Presents as a lymphocytic infiltrate and cystic epithelial proliferation
May represent heterotopic salivary gland epithelial tissue trapped within intraparotid lymph nodes
Incidence of bilaterality and multicentricity of 10%
Malignant transformation rare (almost unheard of)
Monomorphic adenoma Account for less than 5% of tumours
Slow growing
Consist of only one morphological cell type (hence term mono)
Include; basal cell adenoma, canalicular adenoma, oncocytoma, myoepitheliomas
Haemangioma Should be considered in the differential of a parotid mass in a child
Accounts for 90% of parotid tumours in children less than 1 year of age
Hypervascular on imaging
Spontaneous regression may occur and malignant transformation is almost unheard of

Malignant salivary gland tumours
Types of malignancy

Mucoepidermoid carcinoma 30% of all parotid malignancies
Usually low potential for local invasiveness and metastasis (depends mainly on grade)
Adenoid cystic carcinoma Unpredictable growth pattern
Tendency for perineural spread
Nerve growth may display skip lesions resulting in incomplete excision
Distant metastasis more common (visceral rather than nodal spread)
5 year survival 35%
Mixed tumours Often a malignancy occurring in a previously benign parotid lesion
Acinic cell carcinoma Intermediate grade malignancy
May show perineural invasion
Low potential for distant metastasis
5 year survival 80%
Adenocarcinoma Develops from secretory portion of gland
Risk of regional nodal and distant metastasis
5 year survival depends upon stage at presentation, may be up to 75% with small lesions with no nodal involvement
Lymphoma Large rubbery lesion, may occur in association with Warthins tumours
Diagnosis should be based on regional nodal biopsy rather than parotid resection
Treatment is with chemotherapy (and radiotherapy)

Diagnostic evaluation
Plain x-rays may be used to exclude calculi
Sialography may be used to delineate ductal anatomy
FNAC is used in most cases
Superficial parotidectomy may be either diagnostic or therapeutic depending upon the nature of the lesion
Where malignancy is suspected the primary approach should be definitive resection rather than excisional biopsy
CT/ MRI may be used in cases of malignancy for staging primary disease

Treatment
For nearly all lesions this consists of surgical resection, for benign disease this will usually consist of a superficial parotidectomy. For malignant disease a radical or extended radical parotidectomy is performed. The facial nerve is included in the resection if involved. The need for neck dissection is determined by the potential for nodal involvement.

Other parotid disorders
HIV infection
Lymphoepithelial cysts associated with HIV occur almost exclusively in the parotid
Typically presents as bilateral, multicystic, symmetrical swelling
Risk of malignant transformation is low and management usually conservative

Sjogren syndrome
Autoimmune disorder characterised by parotid enlargement, xerostomia and keratoconjunctivitis sicca
90% of cases occur in females
Second most common connective tissue disorder
Bilateral, non tender enlargement of the gland is usual
Histologically, the usual findings are of a lymphocytic infiltrate in acinar units and epimyoepithelial islands surrounded by lymphoid stroma
Treatment is supportive
There is an increased risk of subsequent lymphoma

Sarcoid
Parotid involvement occurs in 6% of patients with sarcoid
Bilateral in most cases
Gland is not tender
Xerostomia may occur
Management of isolated parotid disease is usually conservative

How well did you know this?
1
Not at all
2
3
4
5
Perfectly
710
Q

Which of the following associations are incorrect?

Afro-Caribbean ethnicity and keloid scarring

Extensive third degree burns and wound contraction

Chemotherapy and dehiscence of healed wounds

Poor healing at the site of previous radiotherapy

Zinc deficiency and delayed healing

A

The question asks for the incorrect association. Certain ethnic groups are at increased risk of keloid scar formation. Severe burns can be complicated by wound contracture that requires surgery. Radiotherapy and zinc deficiency are both recognized risk factors for delayed healing. However, fully healed wounds cannot breakdown with chemotherapy and this statement is therefore incorrect and represents the correct answer.

Wound healing

Surgical wounds are either incisional or excisional and either clean, clean contaminated or dirty. Although the stages of wound healing are broadly similar their contributions will vary according to the wound type.

The main stages of wound healing include:

Haemostasis
Minutes to hours following injury
Vasospasm in adjacent vessels, platelet plug formation and generation of fibrin rich clot.

Inflammation
Typically days 1-5
Neutrophils migrate into wound (function impaired in diabetes).
Growth factors released, including basic fibroblast growth factor and vascular endothelial growth factor.
Fibroblasts replicate within the adjacent matrix and migrate into wound.
Macrophages and fibroblasts couple matrix regeneration and clot substitution.

Regeneration
Typically days 7 to 56
Platelet derived growth factor and transformation growth factors stimulate fibroblasts and epithelial cells.
Fibroblasts produce a collagen network.
Angiogenesis occurs and wound resembles granulation tissue.

Remodeling
From 6 weeks to 1 year
Longest phase of the healing process and may last up to one year (or longer).
During this phase fibroblasts become differentiated (myofibroblasts) and these facilitate wound contraction.
Collagen fibres are remodeled.
Microvessels regress leaving a pale scar.

The above description represents an idealised scenario. A number of diseases may distort this process. Neovascularisation is an important early process. Endothelial cells may proliferate in the wound bed and recanalise to form a vessel. Vascular disease, shock and sepsis can all compromise microvascular flow and impair healing.

Conditions such as jaundice will impair fibroblast synthetic function and immunity with a detrimental effect in most parts of the healing process.

Problems with scars:

Hypertrophic scars
Excessive amounts of collagen within a scar. Nodules may be present histologically containing randomly arranged fibrils within and parallel fibres on the surface. The tissue itself is confined to the extent of the wound itself and is usually the result of a full thickness dermal injury. They may go on to develop contractures.

Keloid scars
Excessive amounts of collagen within a scar. Typically a keloid scar will pass beyond the boundaries of the original injury. They do not contain nodules and may occur following even trivial injury. They do not regress over time and may recur following removal.

Drugs which impair wound healing:
Non steroidal anti inflammatory drugs
Steroids
Immunosupressive agents
Anti neoplastic drugs

Closure
Delayed primary closure is the anatomically precise closure that is delayed for a few days but before granulation tissue becomes macroscopically evident.

Secondary closure refers to either spontaneous closure or to surgical closure after granulation tissue has formed.

How well did you know this?
1
Not at all
2
3
4
5
Perfectly
711
Q

A 56 year old man is undergoing a carotid endarterectomy. The internal carotid artery is mobilised. How many branches does this vessel give off in the neck?

0

1

2

3

6

A

The internal carotid does not have any branches in the neck.
Internal carotid artery

The internal carotid artery is formed from the common carotid opposite the upper border of the thyroid cartilage. It extends superiorly to enter the skull via the carotid canal. From the carotid canal it then passes through the cavernous sinus, above which it divides into the anterior and middle cerebral arteries.

Relations in the neck
Posterior
Longus capitis
Pre-vertebral fascia
Sympathetic chain
Superior laryngeal nerve
Medially
External carotid (near origin)
Wall of pharynx
Ascending pharyngeal artery
Laterally
Internal jugular vein (moves posteriorly at entrance to skull)
Vagus nerve (most posterolaterally)
Anteriorly
Sternocleidomastoid
Lingual and facial veins
Hypoglossal nerve

Relations in the carotid canal
Internal carotid plexus
Cochlea and middle ear cavity
Trigeminal ganglion (superiorly)
Leaves canal lies above the foramen lacerum

Path and relations in the cranial cavity
The artery bends sharply forwards in the cavernous sinus, the aducens nerve lies close to its inferolateral aspect. The oculomotor, trochlear, opthalmic and, usually, the maxillary nerves lie in the lateral wall of the sinus. Near the superior orbital fissure it turns posteriorly and passes postero-medially to pierce the roof of the cavernous sinus inferior to the optic nerve. It then passes between the optic and oculomotor nerves to terminate below the anterior perforated substance by dividing into the anterior and middle cerebral arteries.

Branches
Anterior and middle cerebral artery
Ophthalmic artery
Posterior communicating artery
Anterior choroid artery
Meningeal arteries
Hypophyseal arteries

How well did you know this?
1
Not at all
2
3
4
5
Perfectly
712
Q

The following muscles are supplied by the recurrent laryngeal nerve except:

Transverse arytenoid

Posterior crico-arytenoid

Cricothyroid

Oblique arytenoid

Thyroarytenoid

A

Innervates: all intrinsic larynx muscles (excluding cricothyroid)
The external branch of the superior laryngeal nerve innervates the cricothyroid muscle.

Recurrent laryngeal nerve

Branch of the vagus nerve

Path

Right
Arises anterior to the subclavian artery and ascends obliquely next to the trachea, behind the common carotid artery
It is either anterior or posterior to the inferior thyroid artery

Left
Arises left to the arch of the aorta
Winds below the aorta
Ascends along the side of the trachea

Then both

Pass in a groove between the trachea and oesophagus
Enters the larynx behind the articulation between the thyroid cartilage and cricoid
Distributed to larynx muscles

Branches to

Cardiac plexus
Mucous membrane and muscular coat of the oesophagus and trachea

Innervates

Intrinsic larynx muscles (excluding cricothyroid)

How well did you know this?
1
Not at all
2
3
4
5
Perfectly
713
Q

A male infant, born at term appears well following delivery. Six hours later, he is noted to have bilious vomiting by the paediatricians. On examination, he seems well and his abdomen is soft and non tender. What is the best course of action?

Arrange an abdominal x-ray

Undertake a test feed

Perform serial abdominal examinations

Arrange an upper GI contrast study

Arrange a laparotomy

A

Bilious vomiting in neonates is a surgical emergency and is intestinal malrotation and volvulus until otherwise proven. It is investigated with an upper GI contrast study. Contrast should be seen to exit the stomach and the location of the DJ flexure is noted (it lies to the left of the midline). If this is not the case, or the study is inconclusive, a laparotomy is performed.

Intestinal malrotation in neonates

Normal embryology
During the fourth week of embryogenesis the intestine moves into the abdomen via the base of the umbilicus. Through a combination of foetal growth and bowel maturation the bowel resides in the abdominal cavity and undergoes a 270o counterclockwise twist. At the conclusion of this process the ligament of Treitz lies to the left of the spine and the caecum in the right lower quadrant. Malrotation occurs when the rotational process described is incomplete. Typically the duodenal loop lies to the left of the caecum and therefore lacks 90 o of its 270o rotation. It becomes fixed in this position with peritoneal attachments (Ladds bands).

Symptoms
Bilious vomiting is the cardinal symptom and sign. In most cases there are no antecedent symptoms. If untreated then gut perfusion may be impaired with resultant development of further symptoms.

Diagnosis
The main problem is that the infant develops mid gut volvulus. In this situation the bowel undergoes a 720o twist, the bowel viability depends upon a narrow mesentery containing the superior mesenteric artery. To ascertain whether this has occurred the two main tests include an abdominal ultrasound scan to determine the relationship between the superior mesenteric artery and vein (normally SMA lies to the left of the SMV). This test is complemented with an upper GI contrast series and this aims to establish that the DJ flexure is correctly sited to the left of the vertebral bodies.

Treatment
Laparotomy and division of adhesional bands (Ladds procedure). The division of the congenital adhesions allows widening of the small bowel mesentery. The bowel is untwisted and assessed for viability. The bowel is returned to the abdominal cavity in the non rotated fashion with the small bowel on the right hand side and the large bowel on the left, the caecum is positioned in the left upper quadrant. Because the caecum is located in this new location many surgeons will also perform an appendicectomy due to the diagnostic difficulty posed in diagnosing appendicitis in this group of patients.

How well did you know this?
1
Not at all
2
3
4
5
Perfectly
714
Q

Which of the following is not typically associated with a degloving injury?

Overlying pallor of the skin

Abnormal motility of the overlying skin

History of friction type injury

Improved results when the degloved segment is left in situ as a temporary closure

Poor results when primary compression treatment is used in preference to skin grafting

A

Improved results when the degloved segment is left in situ as a temporary closure

Degloving injuries typically involve extremities and are usually friction injuries e.g. arm being run over. There is abnormal motility of the overlying skin, pallor, loss of sensation. Early treatment is key and should involve skin grafting which may use the degloved segment. This however, should be formally prepared for the role and simple compression bandaging gives poor results.

Wound healing

Surgical wounds are either incisional or excisional and either clean, clean contaminated or dirty. Although the stages of wound healing are broadly similar their contributions will vary according to the wound type.

The main stages of wound healing include:

Haemostasis
Minutes to hours following injury
Vasospasm in adjacent vessels, platelet plug formation and generation of fibrin rich clot.

Inflammation
Typically days 1-5
Neutrophils migrate into wound (function impaired in diabetes).
Growth factors released, including basic fibroblast growth factor and vascular endothelial growth factor.
Fibroblasts replicate within the adjacent matrix and migrate into wound.
Macrophages and fibroblasts couple matrix regeneration and clot substitution.

Regeneration
Typically days 7 to 56
Platelet derived growth factor and transformation growth factors stimulate fibroblasts and epithelial cells.
Fibroblasts produce a collagen network.
Angiogenesis occurs and wound resembles granulation tissue.

Remodeling
From 6 weeks to 1 year
Longest phase of the healing process and may last up to one year (or longer).
During this phase fibroblasts become differentiated (myofibroblasts) and these facilitate wound contraction.
Collagen fibres are remodeled.
Microvessels regress leaving a pale scar.

The above description represents an idealised scenario. A number of diseases may distort this process. Neovascularisation is an important early process. Endothelial cells may proliferate in the wound bed and recanalise to form a vessel. Vascular disease, shock and sepsis can all compromise microvascular flow and impair healing.

Conditions such as jaundice will impair fibroblast synthetic function and immunity with a detrimental effect in most parts of the healing process.

Problems with scars:

Hypertrophic scars
Excessive amounts of collagen within a scar. Nodules may be present histologically containing randomly arranged fibrils within and parallel fibres on the surface. The tissue itself is confined to the extent of the wound itself and is usually the result of a full thickness dermal injury. They may go on to develop contractures.

How well did you know this?
1
Not at all
2
3
4
5
Perfectly
715
Q

Intra cranial pressure is governed by the principles of the Monroe-Kellie doctrine. To which of the following does this concept not apply?

A 2 month old child

A 2 year old child

A 5 year old child

A 10 year old child

An adult

A

The Monroe-Kelly Doctrine assumes that the cranial cavity is a rigid box. In children with non fused fontanells this is not the case.

Applied neurophysiology

Pressure within the cranium is governed by the Monroe-Kelly doctrine. This considers the skull as a closed box. Increases in mass can be accommodated by loss of CSF. Once a critical point is reached (usually 100- 120ml of CSF lost) there can be no further compensation and ICP rises sharply. The next step is that pressure will begin to equate with MAP and neuronal death will occur. Herniation will also accompany this process.
The CNS can autoregulate its own blood supply. Vaso constriction and dilatation of the cerebral blood vessels is the primary method by which this occurs. Extremes of blood pressure can exceed this capacity resulting in risk of stroke. Other metabolic factors such as hypercapnia will also cause vasodilation, which is of importance in ventilating head injured patients.
The brain can only metabolise glucose, when glucose levels fall, consciousness will be impaired.

How well did you know this?
1
Not at all
2
3
4
5
Perfectly
716
Q

A 2 year old boy presents with vague neurological symptoms and is imaged with an MRI of the brain. This demonstrates a tumour that is located in the floor of the 4th ventricle and extends through the foramen of Magendie. What is the most likely diagnosis?

Glioblastoma

CNS lymphoma

Ependymoma

Schwannoma

Meningioma

A

Glioblastoma is rare in children. Ependymoma account for up to 33% of CNS tumours in those under age of 3. They commonly arise in the 4th ventricle and can grow through the foramina of Luschka and Magendie

CNS tumours

60% = Glioma and metastatic disease
20% = Meningioma
10% = Pituitary lesions
In paediatric practice medulloblastomas (neuroectodermal tumours) were the commonest lesions, astrocytomas now account for the majority.
Tumours arising in right temporal and frontal lobe may reach considerable size before becoming symptomatic. Whereas tumours in the speech and visual areas will typically produce early symptoms.

Most paediatric CNS tumours are infratentorial
Most adult CNS tumours are supratentorial

Diagnosis
MRI Scanning provides the best resolution.

Treatment
Usually surgery, even if tumour cannot be completely resected conditions such as rising ICP can be addressed with tumour debulking and survival and quality of life prolonged.
Curative surgery can usually be undertaken with lesions such as meningiomas. Gliomas have a marked propensity to invade normal brain and resection of these lesions is nearly always incomplete.

How well did you know this?
1
Not at all
2
3
4
5
Perfectly
717
Q

A 43 year old man is stabbed outside a nightclub. He suffers a transection of his median nerve just as it leaves the brachial plexus. Which of these features is least likely to ensue?

Ulnar deviation of the wrist

Complete loss of wrist flexion

Loss of pronation

Loss of flexion at the thumb joint

Inability to oppose the thumb

A

Loss of the median nerve will result in loss of function of the flexor muscles. However, flexor carpi ulnaris will still function and produce ulnar deviation and some residual wrist flexion. High median nerve lesions result in complete loss of flexion at the thumb joint.

Median nerve

The median nerve is formed by the union of a lateral and medial root respectively from the lateral (C5,6,7) and medial (C8 and T1) cords of the brachial plexus; the medial root passes anterior to the third part of the axillary artery. The nerve descends lateral to the brachial artery, crosses to its medial side (usually passing anterior to the artery). It passes deep to the bicipital aponeurosis and the median cubital vein at the elbow.
It passes between the two heads of the pronator teres muscle, and runs on the deep surface of flexor digitorum superficialis (within its fascial sheath).
Near the wrist it becomes superficial between the tendons of flexor digitorum superficialis and flexor carpi radialis, deep to palmaris longus tendon. It passes deep to the flexor retinaculum to enter the palm, but lies anterior to the long flexor tendons within the carpal tunnel.

Branches
Region Branch
Upper arm No branches, although the nerve commonly communicates with the musculocutaneous nerve
Forearm Pronator teres
Pronator quadratus
Flexor carpi radialis
Palmaris longus
Flexor digitorum superficialis
Flexor pollicis longus
Flexor digitorum profundus (only the radial half)
Distal forearm Palmar cutaneous branch
Hand (Motor) Motor supply (LOAF)
Lateral 2 lumbricals
Opponens pollicis
Abductor pollicis brevis
Flexor pollicis brevis
Hand (Sensory)
Over thumb and lateral 2 ½ fingers
On the palmar aspect this projects proximally, on the dorsal aspect only the distal regions are innervated with the radial nerve providing the more proximal cutaneous innervation.

Patterns of damage
Damage at wrist
e.g. carpal tunnel syndrome
paralysis and wasting of thenar eminence muscles and opponens pollicis (ape hand deformity)
sensory loss to palmar aspect of lateral (radial) 2 ½ fingers

Damage at elbow, as above plus:
unable to pronate forearm
weak wrist flexion
ulnar deviation of wrist

Anterior interosseous nerve (branch of median nerve)
leaves just below the elbow
results in loss of pronation of forearm and weakness of long flexors of thumb and index finger

How well did you know this?
1
Not at all
2
3
4
5
Perfectly
718
Q

A 67 year old man is undergoing an angiogram for gastro intestinal bleeding. The radiologist advances the catheter into the coeliac axis. At what spinal level does this vessel typically arise from the aorta?

T10

L3

L4

T12

None of the above

A

The coeliac axis lies at T12, it takes an almost horizontal angle off the aorta. It has three major branches.

Abdominal aortic branches

Branches Level Paired Type
Inferior phrenic T12 (Upper border) Yes Parietal
Coeliac T12 No Visceral
Superior mesenteric L1 No Visceral
Middle suprarenal L1 Yes Visceral
Renal L1-L2 Yes Visceral
Gonadal L2 Yes Visceral
Lumbar L1-L4 Yes Parietal
Inferior mesenteric L3 No Visceral
Median sacral L4 No Parietal
Common iliac L4 Yes Terminal

How well did you know this?
1
Not at all
2
3
4
5
Perfectly
719
Q

Which structure separates the cephalic vein and the brachial artery in the antecubital fossa?

Brachioradialis muscle

Bicipital aponeurosis

Origin of flexor digitorum profundus muscle

Pronator quadratus muscle

Origin of flexor digitorum superficialis muscle

A

The correct answer is bicipital aponeurosis. The bicipital aponeurosis is a thin sheet of connective tissue that extends from the biceps brachii muscle and covers the brachial artery and median nerve in the cubital fossa1. It separates these structures from the median cubital vein, which is a superficial vein that connects the cephalic vein (located laterally) and the basilic vein (located medially)2. The other options are muscles that are either located outside the cubital fossa or form the floor of the fossa, but do not separate the cephalic vein and the brachial artery.

How well did you know this?
1
Not at all
2
3
4
5
Perfectly
720
Q

A 62 year old man presents with dysphagia and on investigation is found to have a stenosing tumour of the mid oesophagus with a single metastasis in the right lobe of the liver (segment VI). What is the most appropriate treatment?

Radical radiotherapy to the oesophagus and liver resection

Insertion of self expanding metallic stent

Liver resection and subsequent oesophageal resection

Oesophageal resection and subsequent liver resection

Combined oesophageal and liver resection if cardiopulmonary exercise testing shows the patient is fit enough

A

Distant disease in patients with oesophageal cancer is a contra indication to a resectional strategy and downstaging with chemotherapy is not routinely undertaken in this age group as the results are poor. An expanding stent will provide rapid and durable palliation.

Oesophageal cancer - treatment

Treatments for SCC’s and adenocarcinomas of the oesophagus differ. This is primarily due to the positive outcomes that are observed when localised SCC’s (particularly of the proximal oesophagus) are treated with radical chemoradiotherapy (obviating the need for surgery).
Only those patients whose staging investigations are negative for metastatic disease should be considered for surgery.

Surgical options
Endoscopic mucosal resection Treatment for early localised adenocarcinoma of the distal oesophagus. Survival mirrors that of surgical resection for Tis and T1 disease
Transhiatal oeosphagectomy Most commonly used for junctional (type II) (1) tumours where limited thoracic oesophageal resection is required. Less morbidity than two field oesophagectomy
Ivor Lewis oesophagectomy Two stage approach for middle and distal tumours. Very commonly performed, intrathoracic anastomosis will result in mediastinitis in event of anastomotic leak. Lower incidence of recurrent laryngeal nerve injury
McKeown oesophagectomy Three field approach, may be useful for proximal tumours. Anastomotic leakage is less serious. Higher incidence of recurrent laryngeal nerve injury

Neoadjuvent and adjuvent treatment
Neoadjuvent radiotherapy alone prior to resection confers little benefit and is not routinely performed (2)
Preoperative chemotherapy is associated with a survival advantage (OE02 trial)
Peri operative (pre and post operative) chemotherapy confers a survival advantage in junctional tumours
Post operative chemotherapy is not generally recommended following oesophageal resections outside clinical trials

Palliation strategies
Combination chemotherapy improves quality of life and survival in non operable disease (3)
Trastuzumab may improve survival in patients with HER 2 positive tumours
Oesophageal intubation with self expanding metal stents is the treatment of choice in patients with occluding tumours >2cm from the cricopharyngeus
Covered metal stents are useful in cases of malignant fistulas
Laser therapy and argon plasma coagulation may be useful as therapies for tumour overgrowth and bleeding
Photodynamic therapy and ethanol injections confer little benefit and should not be routinely used

References
1. Hulscher JB, van Sandick JW, de Boer AG, et al. Extended transthoracic resection compared with limited transhiatal resection for adenocarcinoma of the esophagus. N Engl J Med 2002;347:1662-9
2. Deng HY, Wang WP, Wang YC, Hu WP, Ni PZ, Lin YD, Chen LQ. Neoadjuvant chemoradiotherapy or chemotherapy? A comprehensive systematic review and meta-analysis of the options for neoadjuvant therapy for treating oesophageal cancer. Eur J Cardiothorac Surg. 2017 Mar 1;51(3):421-431. doi: 10.1093/ejcts/ezw315.
3.Khin MO, Bromham N, Harrison M, Eadon H. Assessment and management of oesophago-gastric cancer: summary of NICE guidance. BMJ. 2018 Jan 29;360:k213. doi: 10.1136/bmj.k213.

How well did you know this?
1
Not at all
2
3
4
5
Perfectly
721
Q

A 30 year old male presents with a painless swelling of the testis. Histologically the stroma has a lymphocytic infiltrate. The most likely diagnosis is :

Differentiated teratoma

Malignant undifferentiated teratoma

Classical seminoma

Spermatocytic seminoma

Anaplastic seminoma

A

Seminoma is the commonest type of testicular tumour and is more common in males aged between 30-40 years. Classical seminoma is the commonest subtype and histology shows lymphocytic stromal infiltrate. Other subtypes include:
1. Spermatocytic: tumour cells resemble spermatocytes. Excellent prognosis.
2. Anaplastic
3. Syncytiotrophoblast giant cells: β HCG present in cells
A teratoma is more common in males aged 20-30 years.

Testicular disorders

Testicular cancer
Testicular cancer is the most common malignancy in men aged 20-30 years. Around 95% of cases of testicular cancer are germ-cell tumours. Germ cell tumours may essentially be divided into:

Tumour type Key features Tumour markers Pathology
Seminoma
Commonest subtype (50%)
Average age at diagnosis = 40
Even advanced disease associated with 5 year survival of 73%
AFP usually normal
HCG elevated in 10% seminomas
Lactate dehydrogenase; elevated in 10-20% seminomas (but also in many other conditions)
Sheet like lobular patterns of cells with substantial fibrous component. Fibrous septa contain lymphocytic inclusions and granulomas may be seen.
Non seminomatous germ cell tumours (42%)
Teratoma
Yolk sac tumour
Choriocarcinoma
Mixed germ cell tumours (10%)
Younger age at presentation =20-30 years
Advanced disease carries worse prognosis (48% at 5 years)
Retroperitoneal lymph node dissection may be needed for residual disease after chemotherapy
AFP elevated in up to 70% of cases
HCG elevated in up to 40% of cases
Other markers rarely helpful

Risk factors for testicular cancer
Cryptorchidism
Infertility
Family history
Klinefelter’s syndrome
Mumps orchitis

Features
A painless lump is the most common presenting symptom
Pain may also be present in a minority of men
Other possible features include hydrocele, gynaecomastia

Diagnosis
Ultrasound is first-line
CT scanning of the chest/ abdomen and pelvis is used for staging
Tumour markers (see above) should be measured

Management
Orchidectomy (Inguinal approach)
Chemotherapy and radiotherapy may be given depending on staging
Abdominal lesions >1cm following chemotherapy may require retroperitoneal lymph node dissection.

Prognosis is generally excellent
5 year survival for seminomas is around 95% if Stage I
5 year survival for teratomas is around 85% if Stage I

Benign disease

Epididymo-orchitis
Acute epididymitis is an acute inflammation of the epididymis, often involving the testis and usually caused by bacterial infection.
Infection spreads from the urethra or bladder. In men <35 years, gonorrhoea or chlamydia are the usual infections.
Amiodarone is a recognised non infective cause of epididymitis, which resolves on stopping the drug.
Tenderness is usually confined to the epididymis, which may facilitate differentiating it from torsion where pain usually affects the entire testis.

Testicular torsion
Twist of the spermatic cord resulting in testicular ischaemia and necrosis.
Most common in males aged between 10 and 30 (peak incidence 13-15 years)
Pain is usually severe and of sudden onset.
Cremasteric reflex is lost and elevation of the testis does not ease the pain.
Treatment is with surgical exploration. If a torted testis is identified then both testis should be fixed as the condition of bell clapper testis is often bilateral.

Hydrocele
Presents as a mass that transilluminates, usually possible to ‘get above’ it on examination.
In younger men it should be investigated with USS to exclude tumour.
In children it may occur as a result of a patent processus vaginalis.
Treatment in adults is with a Lords or Jabouley procedure.
Treatment in children is with trans inguinal ligation of PPV.

How well did you know this?
1
Not at all
2
3
4
5
Perfectly
722
Q

A 30 year old lady is found to have a thyroglossal cyst that has been recently infected and the patient requests treatment. What is the most appropriate course of action?

Surgical treatment with resection of cyst, associated track, central portion of the hyoid and wedge of tongue muscle behind the hyoid

Excision of the cyst alone with ligation of the track

Laying open of the track and excision of the cyst

Phenolisation of the cyst

Marsupialisation of the cyst

A

Recurrence following attempted resection of thyroglossal cysts is very common. Complete excision of the cyst and its track and origin is mandatory (Sistrunks procedure).

Thyroglossal cysts

The commonest midline neck mass in children is a congenital cyst of the thyroglossal duct. Embryologically, the cyst can arise at any point along the route of the thyroglossal duct, extending from the foramen caecum to the thyroid gland.
They are typically located below the hyoid and moves both on swallowing and tongue protrusion. Whilst most cysts are asymptomatic, associated infection may result in pain and swelling.
The usual treatment is excision of the cyst and track, this is a called a Sistrunk procedure.

How well did you know this?
1
Not at all
2
3
4
5
Perfectly
723
Q

Which of the following is not a feature of oesphageal atresia in neonates?

High incidence of polyhydramnios

Risk of recurrence in subsequent pregnancies of 80%

Distal tracheoesphageal fistula is the commonest variant

High incidence of associated imperforate anus

Absence of gastric fluid on antenatal ultrasound

A

Most are sporadic and risk in subsequent pregnancies is not increased.

Paediatric Gastrointestinal disorders

Pyloric stenosis
M>F
5-10% Family history in parents
Projectile non bile stained vomiting at 4-6 weeks of life
Diagnosis is made by test feed or USS
Treatment: Ramstedt pyloromyotomy (open or laparoscopic)
Acute appendicitis
Uncommon under 3 years
When occurs may present atypically
Mesenteric adenitis
Central abdominal pain and URTI
Conservative management
Intussusception
Telescoping bowel
Proximal to or at the level of, ileocaecal valve
6-9 months age
Colicky pain, diarrhoea and vomiting, sausage shaped mass, red jelly stool.
Treatment: reduction with air insufflation
Malrotation
High caecum at the midline
Feature in exomphalos, congenital diaphragmatic hernia, intrinsic duodenal atresia
May be complicated by development of volvulus, infant with volvulus may have bile stained vomiting
Diagnosis is made by upper GI contrast study and USS
Treatment is by laparotomy, if volvulus is present (or at high risk of occurring then a ladds procedure is performed
Hirschsprung’s disease
Absence of ganglion cells from myenteric and submucosal plexuses
Occurs in 1/5000 births
Full thickness rectal biopsy for diagnosis
Delayed passage of meconium and abdominal distension
Treatment is with rectal washouts initially, thereafter an anorectal pull through procedure
Oesophageal atresia
Associated with tracheo-oesophageal fistula and polyhydramnios
May present with choking and cyanotic spells following aspiration
VACTERL associations
Meconium ileus
Usually delayed passage of meconium and abdominal distension
Majority have cystic fibrosis
X-Rays may not show a fluid level as the meconium is viscid (depends upon feeding), PR contrast studies may dislodge meconium plugs and be therapeutic
Infants who do not respond to PR contrast and NG N-acetyl cysteine will require surgery to remove the plugs
Biliary atresia
Jaundice > 14 days
Increased conjugated bilirubin
Urgent Kasai procedure
Necrotising enterocolitis
Prematurity is the main risk factor
Early features include abdominal distension and passage of bloody stools
X-Rays may show pneumatosis intestinalis and evidence of free air
Increased risk when empirical antibiotics are given to infants beyond 5 days
Treatment is with total gut rest and TPN, babies with perforations will require laparotomy

How well did you know this?
1
Not at all
2
3
4
5
Perfectly
724
Q

Which of the following is not a feature of oesphageal atresia in neonates?

High incidence of polyhydramnios

Risk of recurrence in subsequent pregnancies of 80%

Distal tracheoesphageal fistula is the commonest variant

High incidence of associated imperforate anus

Absence of gastric fluid on antenatal ultrasound

A

Most are sporadic and risk in subsequent pregnancies is not increased.

Paediatric Gastrointestinal disorders

Pyloric stenosis
M>F
5-10% Family history in parents
Projectile non bile stained vomiting at 4-6 weeks of life
Diagnosis is made by test feed or USS
Treatment: Ramstedt pyloromyotomy (open or laparoscopic)
Acute appendicitis
Uncommon under 3 years
When occurs may present atypically
Mesenteric adenitis
Central abdominal pain and URTI
Conservative management
Intussusception
Telescoping bowel
Proximal to or at the level of, ileocaecal valve
6-9 months age
Colicky pain, diarrhoea and vomiting, sausage shaped mass, red jelly stool.
Treatment: reduction with air insufflation
Malrotation
High caecum at the midline
Feature in exomphalos, congenital diaphragmatic hernia, intrinsic duodenal atresia
May be complicated by development of volvulus, infant with volvulus may have bile stained vomiting
Diagnosis is made by upper GI contrast study and USS
Treatment is by laparotomy, if volvulus is present (or at high risk of occurring then a ladds procedure is performed
Hirschsprung’s disease
Absence of ganglion cells from myenteric and submucosal plexuses
Occurs in 1/5000 births
Full thickness rectal biopsy for diagnosis
Delayed passage of meconium and abdominal distension
Treatment is with rectal washouts initially, thereafter an anorectal pull through procedure
Oesophageal atresia
Associated with tracheo-oesophageal fistula and polyhydramnios
May present with choking and cyanotic spells following aspiration
VACTERL associations
Meconium ileus
Usually delayed passage of meconium and abdominal distension
Majority have cystic fibrosis
X-Rays may not show a fluid level as the meconium is viscid (depends upon feeding), PR contrast studies may dislodge meconium plugs and be therapeutic
Infants who do not respond to PR contrast and NG N-acetyl cysteine will require surgery to remove the plugs
Biliary atresia
Jaundice > 14 days
Increased conjugated bilirubin
Urgent Kasai procedure
Necrotising enterocolitis
Prematurity is the main risk factor
Early features include abdominal distension and passage of bloody stools
X-Rays may show pneumatosis intestinalis and evidence of free air
Increased risk when empirical antibiotics are given to infants beyond 5 days
Treatment is with total gut rest and TPN, babies with perforations will require laparotomy

How well did you know this?
1
Not at all
2
3
4
5
Perfectly
725
Q

Surgical treatment with resection of cyst, associated track, central portion of the hyoid and wedge of tongue muscle behind the hyoid

Excision of the cyst alone with ligation of the track

Laying open of the track and excision of the cyst

Phenolisation of the cyst

Marsupialisation of the cyst

A

Recurrence following attempted resection of thyroglossal cysts is very common. Complete excision of the cyst and its track and origin is mandatory (Sistrunks procedure).

Thyroglossal cysts

The commonest midline neck mass in children is a congenital cyst of the thyroglossal duct. Embryologically, the cyst can arise at any point along the route of the thyroglossal duct, extending from the foramen caecum to the thyroid gland.
They are typically located below the hyoid and moves both on swallowing and tongue protrusion. Whilst most cysts are asymptomatic, associated infection may result in pain and swelling.
The usual treatment is excision of the cyst and track, this is a called a Sistrunk procedure.

How well did you know this?
1
Not at all
2
3
4
5
Perfectly
726
Q

A 28 year old man is reviewed in the clinic. He has suffered from Crohns disease for many years, he has recently undergone a sub total colectomy. However, he has residual Crohns in his rectum and this is the cause of ongoing symptoms. Medical therapy is proving ineffective. What is the best course of action?

Abdomino perineal excision of the colon and rectum

Proctectomy

Hartmanns procedure

Ileo-rectal anastomosis

Formation of ileo-anal pouch

A

An abdomino-perineal excision of the colon and rectum is a cancer procedure and not appropriate in the context of inflammatory bowel disease. The only appropriate surgical option here is a proctectomy to remove the rectal stump and anal canal.

IBD

Ulcerative colitis Vs Crohns

Crohn’s disease Ulcerative colitis
Distribution Mouth to anus Rectum and colon
Macroscopic changes Cobblestone appearance, apthoid ulceration Contact bleeding
Depth of disease Transmural inflammation Superficial inflammation
Distribution pattern Patchy Continuous
Histological features Granulomas (non caseating epithelioid cell aggregates with Langerhans’ giant cells) Crypt abscesses, Inflammatory cells in the lamina propria

Surgical treatment

Ulcerative colitis
In UC the main place for surgery is when medical treatment has failed, in the emergency setting this will be a sub total colectomy, end ileostomy and a mucous fistula. Electively it will be a pan proctocolectomy, an ileoanal pouch may be a selected option for some. Remember that longstanding UC increases colorectal cancer risk.
Crohn’s disease
Unlike UC Crohn’s patients need to avoid surgeons, minimal resections are the rule. They should not have ileoanal pouches as they will do poorly with them. Management of Crohn’s ano rectal sepsis is with a minimal approach, simply drain sepsis and use setons to facilitate drainage. Definitive fistula surgery should be avoided.

How well did you know this?
1
Not at all
2
3
4
5
Perfectly
727
Q

A 74 year old man is admitted for an inguinal hernia repair. His past medical history includes transient ischaemic attacks for which he takes clopidogrel. This is stopped 36 hours prior to surgery. In the recovery room, he is noted to have signs of bleeding from the wound. What is the most likely explanation?

Reduced Factor Xa levels

Platelet dysfunction

Reduced Factor IX levels

Injury to the epigastric vein

Injury to the femoral artery

A

Platelet dysfunction. The clopidogrel should have been stopped sooner. Vascular injuries are usually evident intraoperatively.

Clopidogrel

Clopidogrel is an antiplatelet agent that is usually administered orally in doses of 75mg. It is converted by the P450 enzymes in the liver into the active drug. It decreases ADP induced platelet aggregation persisting for 120 hours after the final dose. This last point is especially relevant for surgeons as it means that it needs to be stopped around 5-7 days prior to surgery. Its effects are very potent and failure to adhere to this can result in significant increases in perioperative bleeding that can be difficult to control.

How well did you know this?
1
Not at all
2
3
4
5
Perfectly
728
Q

A 42 year old woman complains of a burning pain of her anterolateral thigh which worsens on walking. There is a positive tinel sign over the inguinal ligament. Which nerve is affected?

Ilioinguinal nerve

Genitofemoral nerve

Lateral cutaneous nerve of the thigh

Femoral nerve

Saphenous nerve

A

The lateral cutaneous nerve supplies sensation to the anterior and lateral aspect of the thigh. Entrapment is commonly due to intra and extra pelvic causes. Treatment involves local anaesthetic injections.

Lateral cutaneous nerve of the thigh

Cutaneous nerve arising from posterior surface of the second and third lumbar ventral rami
Emerges from the lateral border of psoas major anterior to the iliac crest, and passes between iliacus and iliac fascia
Enters thigh posterior to the lateral end of the inguinal ligament, medial to the anterior superior iliac spine
It pierces the fascia lata 10cm inferior to the anterior superior iliac spine and divides into 2 branches
Anterior branch supplies skin and fascia of the anterolateral surface of the knee
Smaller posterior branch supplies the skin and fascia on the lateral part of the upper leg between the greater trochanter and distal third of the thigh

How well did you know this?
1
Not at all
2
3
4
5
Perfectly
729
Q

A 56 year old man presents with a painless swelling in the upper part of the anterior triangle of his neck. On examination a mass lesion involving the sub mandibular gland is identified. On CT scanning this is shown to be a solid lesion. There is no regional lymphadenopathy. Two fine needle aspirates have failed to be diagnostic. Which of the following is the most appropriate management option?

Sub mandibular gland excision

Incisional biopsy of the mass

Manage conservatively and repeat the CT scan in 6 months

Sub mandibular gland excision and radical neck dissection

Diagnostic excision of the superficial lobe of the submandibular gland

A

There is a 50% risk that this lesion is malignant (in some series up to 70%). Therefore the gland should be excised entirely. At this stage a radical neck dissection is not justified.

Submandibular glands- disease

Physiology
The submandibular glands secrete approximately 800- 1000ml saliva per day. They typically produce mixed seromucinous secretions. When parasympathetic activity is dominant; the secretions will be more serous. The parasympathetic fibres are derived from the chorda tympani nerves and the submandibular ganglion. Sensory fibres are conveyed by the lingual branch of the mandibular nerve.

Sialolithiasis
80% of all salivary gland calculi occur in the submandibular gland
70% of the these calculi are radio-opaque
Stones are usually composed of calcium phosphate or calcium carbonate
Patients typically develop colicky pain and post prandial swelling of the gland
Investigation involves sialography to demonstrate the site of obstruction and associated other stones
Stones impacted in the distal aspect of Whartons duct may be removed orally, other stones and chronic inflammation will usually require gland excision

Sialadenitis
Usually occurs as a result of Staphylococcus aureus infection
Pus may be seen leaking from the duct, erythema may also be noted
Development of a sub mandibular abscess is a serious complication as it may spread through the other deep fascial spaces and occlude the airway

Submandibular tumours
Only 8% of salivary gland tumours affect the sub mandibular gland
Of these 50% are malignant (usually adenoid cystic carcinoma)
Diagnosis usually involves fine needle aspiration cytology
Imaging is with CT and MRI
In view of the high prevalence of malignancy, all masses of the submandibular glands should generally be excised.

How well did you know this?
1
Not at all
2
3
4
5
Perfectly
730
Q

A 43 year old man falls over landing on his left hand. Although there was anatomical snuffbox tenderness; no x-rays either at the time, or subsequently, have shown evidence of scaphoid fracture. He has been immobilised in a futura splint for two weeks and is now asymptomatic. What is the most appropriate course of action?

Application of tubigrip bandage and fracture clinic review

Admission and surgical debridement

Application of futura splint and fracture clinic review

Application of below elbow cast for 6 weeks

Discharge with reassurance

A

This patient is at extremely low risk of having sustained a scaphoid injury and may be discharged.

Scaphoid fractures

  • Incidence of scaphoid fractures in UK ranges from 12.4 per 100,000 to 29 per 100,000
    Surface of scaphoid is covered by articular cartilage with small area available for blood vessels (fracture risks blood supply)
    Forms floor of anatomical snuffbox
    Risk of fracture associated with fall onto outstretched hand (tubercle, waist, or proximal third)
    A series of 4 scaphoid radiographs should be undertaken (PA, pronated oblique, Ziter view and lateral view). The Ziter view is a PA view with the wrist in ulnar deviation and beam angulated at 20 degrees
    Sensitivity of scaphoid radiographs in 1st week of injury is 80%
    Immobilization of scaphoid fractures difficult
    Repeat imaging should be done at 10 days. MRI should be done in cases of diagnostic uncertainty

Classification of scaphoid fractures
Scaphoid tubercle
Distal pole
Waist
Proximal pole

Management
Undisplaced fractures of the waist of the scaphoid and most distal pole fractures can be managed in a cast for 6 weeks with high rates of union.
Displaced scaphoid waist fractures (more than 1-2mm) should be viewed as unstable and surgically fixed.
All proximal pole fractures should be fixed surgically.

Complications
Non union of scaphoid
Avascular necrosis of the scaphoid
Scapholunate disruption and wrist collapse
Degenerative changes of the adjacent joint

Reference
Berber O et al. Fractures of the scaphoid. BMJ 2020 (369): 414-416.

How well did you know this?
1
Not at all
2
3
4
5
Perfectly
731
Q

A 56 year old man is diagnosed with an abdominal aortic aneurysm and undergoes a CT scan to assess the size of the aorta. During the course of his investigations a lesion of the adrenal gland is identified. It measures 1.5 cm in diameter and the gland is otherwise normal. What is the most likely diagnosis?

Adrenal gland metastasis

Adrenal gland arterio-venous malformation

Adrenal cyst

Phaeochromocytoma

Adrenal cortical adenoma

A

Adrenal cortical adenoma

25% of all adrenal lesions >4cm in diameter are malignant
Incidentalomas of the adrenal gland are common and represent the most likely lesion in this scenario. Clearly the other lesions are all possibilities but are unlikely.

Adrenal lesions- Incidental

Incidentaloma of the adrenal glands have become increasingly common as CT scanning of the abdomen is widely undertaken. Prevalences range from 1.5-9% in autopsy studies. Overall, 75% will be non functioning adenomas. Investigation to exclude a functioning lesion is as below:

Investigation
Morning and midnight plasma cortisol measurements
Dexamethasone suppression test
24 hour urinary cortisol excretion
24 hour urinary excretion of catecholamines
Serum potassium, aldosterone and renin levels

Management
The risk of malignancy is related to the size of the lesion and 25% of all masses greater than 4cm will be malignant. Such lesions should usually be excised. Where a lesion is a suspected metastatic deposit a biopsy may be considered.

How well did you know this?
1
Not at all
2
3
4
5
Perfectly
732
Q

A 73 year old man undergoes a sub total oesophagectomy with anastomosis of the stomach to the cervical oesophagus. Which vessel will be primarily responsible for the arterial supply to the oesophageal portion of the anastomosis?

Superior thyroid artery

Internal carotid artery

Direct branches from the thoracic aorta

Inferior thyroid artery

Subclavian artery

A

The cervical oesophagus is supplied by the inferior thyroid artery. The thoracic oesophagus (removed in this case) is supplied by direct branches from the thoracic aorta.

Oesophagus

  • 25cm long
    Starts at C6 vertebra, pierces diaphragm at T10 and ends at T11
    Squamous epithelium

Constrictions of the oesophagus
Structure Distance from incisors
Cricoid cartilage 15cm
Arch of the Aorta 22.5cm
Left principal bronchus 27cm
Diaphragmatic hiatus 40cm

Relations
Anteriorly
Trachea to T4
Recurrent laryngeal nerve
Left bronchus, Left atrium
Diaphragm
Posteriorly
Thoracic duct to left at T5
Hemiazygos to the left T8
Descending aorta
First 2 intercostal branches of aorta
Left
Thoracic duct
Left subclavian artery
Right
Azygos vein

Arterial, venous and lymphatic drainage of the oesophagus
Artery Vein Lymphatics Muscularis externa
Upper third Inferior thyroid Inferior thyroid Deep cervical Striated muscle
Mid third Aortic branches Azygos branches Mediastinal Smooth & striated muscle
Lower third Left gastric Left gastric Gastric Smooth muscle

Nerve supply
Upper half is supplied by recurrent laryngeal nerve
Lower half by oesophageal plexus (vagus)

Histology
Mucosa :Non-keratinized stratified squamous epithelium
Submucosa: glandular tissue
Muscularis externa (muscularis): composition varies. See table
Adventitia

How well did you know this?
1
Not at all
2
3
4
5
Perfectly
733
Q

A 41 year old man is admitted with peritonitis secondary to a perforated appendix. He is treated with a laparoscopic appendicectomy but has a stormy post operative course. He is now developing increasing abdominal pain and has been vomiting. A laparotomy is performed and at operation a large amount of small bowel shows evidence of patchy areas of infarction. What is the most likely cause?

Mesenteric venous thrombosis

Superior mesenteric artery embolus

Acute on chronic mesenteric ischaemia

Vasculitis

Median arcuate ligament syndrome

A

Mesenteric vein thrombosis may complicate severe intra abdominal sepsis and when it progresses may impair bowel perfusion. The serosa is quite resistant to ischaemia so in this case the appearances are usually patchy.

Mesenteric vessel disease

Mesenteric ischaemia accounts for 1 in 1000 acute surgical admissions. It is primarily caused by arterial embolism resulting in infarction of the colon. It is more likely to occur in areas such as the splenic flexure that are located at the borders of the territory supplied by the superior and inferior mesenteric arteries.

Types
Acute mesenteric embolus (commonest 50%)
Sudden onset abdominal pain followed by profuse diarrhoea.
May be associated with vomiting.
Rapid clinical deterioration.
Serological tests: WCC, lactate, amylase may all be abnormal particularly in established disease. These can be normal in the early phases.
Acute on chronic mesenteric ischaemia
Usually longer prodromal history.
Post prandial abdominal discomfort and weight loss are dominant features. Patients will usually present with an acute on chronic event, but otherwise will tend not to present until mesenteric flow is reduced by greater than 80%.
When acute thrombosis occurs presentation may be as above. In the chronic setting the symptoms will often be those of ischaemic colitis (mucosa is the most sensitive area to this insult).
Mesenteric vein thrombosis
Usually a history over weeks.
Overt abdominal signs and symptoms will not occur until venous thrombosis has reached a stage to compromise arterial inflow.
Thrombophilia accounts for 60% of cases.
Low flow mesenteric infarction
This occurs in patients with multiple co morbidities in whom mesenteric perfusion is significantly compromised by overuse of inotropes or background cardiovascular compromise.
The end result is that the bowel is not adequately perfused and infarcts occur from the mucosa outwards.

Diagnosis
Serological tests: WCC, lactate, CRP, amylase (can be normal in early disease).
Cornerstone for diagnosis of arterial AND venous mesenteric disease is CT angiography scanning in the arterial phase with thin slices (<5mm). Venous phase contrast is not helpful.
SMA duplex USS is useful in the evaluation of proximal SMA disease in patients with chronic mesenteric ischaemia.
MRI is of limited use due to gut peristalsis and movement artefact.

Management
Overt signs of peritonism: Laparotomy
Mesenteric vein thrombosis: If no peritonism: Medical management with IV heparin
At operation limited resection of frankly necrotic bowel with view to relook laparotomy at 24-48h. In the interim urgent bowel revascularisation via endovascular (preferred) or surgery.

Prognosis
Overall poor. Best outlook is from an acute ischaemia from an embolic event where surgery occurs within 12h. Survival may be 50%. This falls to 30% with treatment delay. The other conditions carry worse survival figures.

How well did you know this?
1
Not at all
2
3
4
5
Perfectly
734
Q

Which of the following structures passes through the quadrangular space near the humeral head?

Axillary artery

Radial nerve

Axillary nerve

Median nerve

Transverse scapular artery

A

The quadrangular space is bordered by the humerus laterally, subscapularis and teres minor superiorly, teres major inferiorly and the long head of triceps medially. It lies lateral to the triangular space. It transmits the axillary nerve and posterior circumflex humeral artery.

Shoulder joint

Shallow synovial ball and socket type of joint.
It is an inherently unstable joint, but is capable to a wide range of movement.
Stability is provided by muscles of the rotator cuff that pass from the scapula to insert in the greater tuberosity (all except sub scapularis-lesser tuberosity).

Glenoid labrum
Fibrocartilaginous rim attached to the free edge of the glenoid cavity
Tendon of the long head of biceps arises from within the joint from the supraglenoid tubercle, and is fused at this point to the labrum.
The long head of triceps attaches to the infraglenoid tubercle

Fibrous capsule
Attaches to the scapula external to the glenoid labrum and to the labrum itself (postero-superiorly)
Attaches to the humerus at the level of the anatomical neck superiorly and the surgical neck inferiorly
Anteriorly the capsule is in contact with the tendon of subscapularis, superiorly with the supraspinatus tendon, and posteriorly with the tendons of infraspinatus and teres minor. All these blend with the capsule towards their insertion.
Two defects in the fibrous capsule; superiorly for the tendon of biceps. Anteriorly there is a defect beneath the subscapularis tendon.
The inferior extension of the capsule is closely related to the axillary nerve at the surgical neck and this nerve is at risk in anteroinferior dislocations. It also means that proximally sited osteomyelitis may progress to septic arthritis.

Movements and muscles
Flexion Anterior part of deltoid
Pectoralis major
Biceps
Coracobrachialis
Extension Posterior deltoid
Teres major
Latissimus dorsi
Adduction Pectoralis major
Latissimus dorsi
Teres major
Coracobrachialis
Abduction Mid deltoid
Supraspinatus
Medial rotation Subscapularis
Anterior deltoid
Teres major
Latissimus dorsi
Lateral rotation Posterior deltoid
Infraspinatus
Teres minor

Important anatomical relations
Anteriorly Brachial plexus
Axillary artery and vein
Posterior Suprascapular nerve
Suprascapular vessels
Inferior Axillary nerve
Circumflex humeral vessels

How well did you know this?
1
Not at all
2
3
4
5
Perfectly
735
Q

A 35 year old male presents with haematuria. He is found to have bilateral masses in the flanks. He has a history of epilepsy and learning disability. Which of the lesions below is most likely?

Angiomyolipoma

Renal cortical cysts

Transitional cell cancer

Nephroblastomas

Staghorn calculi

A

This patient has tuberous sclerosis. This is associated with angiomyolipoma, which is present in 60-80% patients. It is a benign lesion.

Renal lesions

Lesion Disease specific features Treatment
Renal cell carcinoma
Most present with haematuria (50%)
Common renal tumour (85% cases)
Paraneoplastic features include hypertension and polycythaemia
Most commonly has haematogenous mestastasis
Usually radical or partial nephrectomy
Nephroblastoma
Rare childhood tumour
It accounts for 80% of all genitourinary malignancies in those under the age of 15 years
Up to 90% will have a mass
50% will be hypertensive
Diagnostic work up includes ultrasound and CT scanning
Surgical resection combined with chemotherapy (usually vincristine, actinomycin D and doxorubicin)
Neuroblastoma
Most common extracranial tumour of childhood
80% occur in those under 4 years of age
Tumour of neural crest origin (up to 50% occur in the adrenal gland)
The tumour is usually calcified and may be diagnosed using MIBG scanning
Staging is with CT
Surgical resection, radiotherapy and chemotherapy
Transitional cell carcinoma
Accounts for 90% of lower urinary tract tumours, but only 10% of renal tumours
Males affected 3x more than females
Occupational exposure to industrial dyes and rubber chemicals may increase risk
Up to 80% present with painless haematuria
Diagnosis and staging is with CT IVU
Radical nephroureterectomy
Angiomyolipoma
80% of these hamartoma type lesions occur sporadically, the remainder are seen in those with tuberous sclerosis
Tumour is composed of blood vessels, smooth muscle and fat
Massive bleeding may occur in 10% of cases
50% of patients with lesions >4cm will have symptoms and will require surgical resection

How well did you know this?
1
Not at all
2
3
4
5
Perfectly
736
Q

A 74 year old lady falls and injures her left arm. Following assessment she is found to have an impacted fracture affecting the surgical neck of the humerus. What is the most appropriate course of action?

Reduce the fracture and apply a plate to stabilize the fragments

Perform a hemiarthroplasty

Apply a collar and cuff system for three weeks and then commence physiotherapy

Apply an upper limb cast for 8 weeks

Apply an external fixator system

A

Impacted fractures of the surgical neck are stable injuries and usually heal without complication. It is rare to need to resort to surgery. Its important to start physiotherapy early.

Proximal humerus fractures

Very common injury. Usually through the surgical neck. Number of classification systems though for practical purposes describing the number of fracture fragments is probably easier. Some key points:
It is rare to have fractures through the anatomical neck.
Anatomical neck fractures which are displaced by >1cm carry a risk of avascular necrosis to the humeral head.
In children the commonest injury pattern is a greenstick fracture through the surgical neck.
Impacted fractures of the surgical neck are usually managed with a collar and cuff for 3 weeks followed by physiotherapy.
More significant displaced fractures may require open reduction and fixation or use of an intramedullary device.

How well did you know this?
1
Not at all
2
3
4
5
Perfectly
737
Q

A 45 year old man complains of sharp chest pain. He is due to have elective surgery to replace his left hip. He has been bed bound for 3 months. He suddenly collapses; his blood pressue is 70/40mmHg, heart rate 120 bpm and his saturations are 74% on air. He is deteriorating in front of you. What is the next best management plan?

Aspirin

Thrombolysis with Alteplase

Unfractionated heparin

Thrombolysis with streptokinase

Clopidogrel

A

This man is peri arrest with the diagnosis of pulmonary embolism (chest pain,bedbound, collapse, low saturations). He needs urgent thrombolysis with alteplase (he may not survive if you wait for the medical Spr/ITU to arrive!).

Pulmonary embolism: management

A summary of the British Thoracic Society guidelines

Heparin should be given if intermediate or high clinical probability before imaging.
Unfractionated heparin (UFH) should be considered (a) as a first dose bolus, (b) in massive PE, or (c) where rapid reversal of effect may be needed.
Otherwise, low molecular weight heparin (LMWH) should be considered as preferable to UFH, having equal efficacy and safety and being easier to use.
Oral anticoagulation should only be commenced once VTE has been reliably confirmed.
The target INR should be 2.0-3.0; when this is achieved, heparin can be discontinued.
The standard duration of oral anticoagulation is: 4 to 6 weeks for temporary risk factors, 3 months for first idiopathic, and at least 6 months for other; the risk of bleeding should be balanced with that of further VTE.

Massive PE

CTPA or echocardiography will reliably diagnose clinically massive PE.
Thrombolysis is 1st line for massive PE (ie circulatory failure) and may be instituted on clinical grounds alone if cardiac arrest is imminent; a 50 mg bolus of alteplase is recommended.
Invasive approaches (thrombus fragmentation and IVC filter insertion) should be considered where facilities and expertise are readily available.

How well did you know this?
1
Not at all
2
3
4
5
Perfectly
738
Q

Which of the following is the most sensitive blood test for diagnosis of acute pancreatitis?

Amylase

Lipase

C-peptide

Trypsin

Trysinogen

A

The serum amylase may rise and fall quite quickly and lead to a false negative result. Should the clinical picture not be concordant with the amylase level then serum lipase or a CT Scan should be performed.

Management of Pancreatitis

Management of Acute Pancreatitis in the UK

Diagnosis
Traditionally hyperamylasaemia has been utilised with amylase being elevated three times the normal range.
However, amylase may give both false positive and negative results.
Serum lipase is both more sensitive and specific than serum amylase. It also has a longer half life.
Serum amylase levels do not correlate with disease severity.

Differential causes of hyperamylasaemia
Acute pancreatitis
Pancreatic pseudocyst
Mesenteric infarct
Perforated viscus
Acute cholecystitis
Diabetic ketoacidosis

Assessment of severity
Glasgow, Ranson scoring systems and APACHE II
Biochemical scoring e.g. using CRP

Features that may predict a severe attack within 48 hours of admission to hospital
Initial assessment
Clinical impression of severity
Body mass index >30
Pleural effusion
APACHE score >8
24 hours after admission
Clinical impression of severity
APACHE II >8
Glasgow score of 3 or more
Persisting multiple organ failure
CRP>150
48 hours after admission
Glasgow Score of >3
CRP >150
Persisting or progressive organ failure
Table adapted from UK guidelines for management of acute pancreatitis. GUT 2005, 54 suppl III

Management

Nutrition
There is reasonable evidence to suggest that the use of enteral nutrition does not worsen the outcome in pancreatitis
Most trials to date were underpowered to demonstrate a conclusive benefit.
The rationale behind feeding is that it helps to prevent bacterial translocation from the gut, thereby contributing to the development of infected pancreatic necrosis.

Use of antibiotic therapy
Many UK surgeons administer antibiotics to patients with acute pancreatitis. However, there is very little evidence to support this practice.
A recent Cochrane review highlights the potential benefits of administering Imipenem to patients with established pancreatic necrosis in the hope of averting the progression to infection.
There are concerns that the administration of antibiotics in mild attacks of pancreatitis will not affect outcome and may contribute to antibiotic resistance and increase the risks of antibiotic associated diarrhoea.

Surgery
Patients with acute pancreatitis due to gallstones should undergo early cholecystectomy.
Patients with obstructed biliary system due to stones should undergo early ERCP.
Patients with extensive necrosis where infection is suspected should usually undergo FNA for culture.
Patients with infected necrosis should undergo either radiological drainage or surgical necrosectomy. The choice of procedure depends upon local expertise.

References
www.bsg.org.uk/pdfworddocs/pancreatic.pdf

Antibiotic therapy for prophylaxis against infection of pancreatic necrosis in acute pancreatitis. Villatoro et al. Cochrane Library DOI: 10.1002/14651858.CD002941.pub3. 2010 version.

How well did you know this?
1
Not at all
2
3
4
5
Perfectly
739
Q

In paediatric orthopaedic surgery, which of the following does not fulfill the Kocher criteria for septic arthritis?

ESR > 40mm/h

Positive blood culture

Fever

White cell count > 12, 000

Non weight bearing on the affected side

A

Kocher’s ‘WIFE’ is:

WCC >12
Inability to weight bear
Fever
ESR >40
= >90 chance of septic arthritis
Kocher criteria
1. Non weight bearing on affected side
2. ESR > 40 mm/hr
3. Fever
4. WBC count of >12,000 mm3
- When 4/4 criteria are met, there is a 99% chance that the child has septic arthritis

The Kocher criteria do not consider blood culture results.

Septic arthritis- Paediatric

Septic arthritis
Staph aureus commonest organism
Urgent washout and antibiotics otherwise high risk of joint destruction

Diagnosis
Plain x-rays
Consider aspiration
Utilise the Kocher criteria (see below)

Kocher criteria:
1. Non weight bearing on affected side
2. ESR > 40 mm/hr
3. Fever
4. WBC count of >12,000 mm3
- when 4/4 criteria are met, there is a 99% chance that the child has septic arthritis

Treatment
Surgical drainage of the affected joint is required, this should be done as soon as possible since permanent damage to the joint may occur. In some cases repeated procedures are necessary. Appropriate intravenous antibiotics should be administered.

How well did you know this?
1
Not at all
2
3
4
5
Perfectly
740
Q

A 43 year old lady with a metallic heart valve has just undergone an elective paraumbilical hernia repair. In view of her metallic valve, she is given unfractionated heparin perioperatively. How should the therapeutic efficacy be monitored, assuming her renal function is normal?

Therapeutic monitoring is not required

Measurement of APTT

Measurement of INR

Measurement of Prothromin time

None of the above

A

Unlike low molecular weight heparins that do not require monitoring unfractionated heparin does require monitoring, this is done by measuring the APTT.

Heparin

Causes the formation of complexes between antithrombin and activated thrombin/factors 7,9,10,11 & 12

Advantages of low molecular weight heparin
Better bioavailability
Lower risk of bleeding
Longer half life
Little effect on APTT at prophylactic dosages
Less risk of HIT

Complications
Bleeding
Osteoporosis
Heparin induced thrombocytopenia (HIT): occurs 5-14 days after 1st exposure
Anaphylaxis

In surgical patients that may need a rapid return to theatre, administration of unfractionated heparin is preferred; as low molecular weight heparins have a longer duration of action and are harder to reverse.

How well did you know this?
1
Not at all
2
3
4
5
Perfectly
741
Q

Which of the following is the main site of dehydroepiandrosterone release?

Posterior pituitary

Zona reticularis of the adrenal gland

Zona glomerulosa of the adrenal gland

Juxtaglomerular apparatus of the kidney

Zona fasciculata of the adrenal gland

A

Adrenal cortex mnemonic: GFR - ACD
DHEA possesses some androgenic activity and is almost exclusively released from the adrenal gland.

Renin-angiotensin-aldosterone system

Adrenal cortex (mnemonic GFR - ACD)
Zona glomerulosa (on outside): mineralocorticoids, mainly aldosterone
Zona fasciculata (middle): glucocorticoids, mainly cortisol
Zona reticularis (on inside): androgens, mainly dehydroepiandrosterone (DHEA)

Renin
Released by JGA cells in kidney in response to reduced renal perfusion, low sodium
Hydrolyses angiotensinogen to form angiotensin I

Factors stimulating renin secretion
Low BP
Hyponatraemia
Sympathetic nerve stimulation
Catecholamines
Erect posture

Angiotensin
ACE in lung converts angiotensin I → angiotensin II
Vasoconstriction leads to raised BP
Stimulates thirst
Stimulates aldosterone and ADH release

Aldosterone
Released by the zona glomerulosa in response to raised angiotensin II, potassium, and ACTH levels
Causes retention of Na+ in exchange for K+/H+ in distal tubule

How well did you know this?
1
Not at all
2
3
4
5
Perfectly
742
Q

Which of the following statements in relation to the p53 tumour suppressor protein is false?

It may induce necrosis of cells with non repairable DNA damage

It is affected in Li Fraumeni syndrome

It can induce DNA repair

It can halt the cell cycle

It may inhibit angiogenesis

A

When DNA cannot be repaired it will induce cellular apoptosis (not necrosis)

Genetics and surgical disease

Some of the more commonly occurring genetic conditions occurring in surgical patients are presented here.

Li-Fraumeni Syndrome
Autosomal dominant
Consists of germline mutations to p53 tumour suppressor gene
High incidence of malignancies particularly sarcomas and leukaemias
Diagnosed when:

*Individual develops sarcoma under 45 years
*First degree relative diagnosed with any cancer below age 45 years and another family member develops malignancy under 45 years or sarcoma at any age

BRCA 1 and 2
Carried on chromosome 17 (BRCA 1) and Chromosome 13 (BRCA 2)
Linked to developing breast cancer (60%) risk.
Associated risk of developing ovarian cancer (55% with BRCA 1 and 25% with BRCA 2).

Lynch Syndrome
Autosomal dominant
Develop colonic cancer and endometrial cancer at young age
80% of affected individuals will get colonic and/ or endometrial cancer
High risk individuals may be identified using the Amsterdam criteria

Amsterdam criteria
Three or more family members with a confirmed diagnosis of colorectal cancer, one of whom is a first degree (parent, child, sibling) relative of the other two.
Two successive affected generations.
One or more colon cancers diagnosed under age 50 years.
Familial adenomatous polyposis (FAP) has been excluded.

Gardners syndrome
Autosomal dominant familial colorectal polyposis
Multiple colonic polyps
Extra colonic diseases include: skull osteoma, thyroid cancer and epidermoid cysts
Desmoid tumours are seen in 15%
Mutation of APC gene located on chromosome 5
Due to colonic polyps most patients will undergo colectomy to reduce risk of colorectal cancer
Now considered a variant of familial adenomatous polyposis coli

How well did you know this?
1
Not at all
2
3
4
5
Perfectly
743
Q

A 28 year old man lacerates the posterolateral aspect of his wrist with a knife in an attempted suicide. On arrival in the emergency department the wound is inspected and found to be located over the lateral aspect of the extensor retinaculum (which is intact). Which of the following structures is at greatest risk of injury?

Superficial branch of the radial nerve

Radial artery

Dorsal branch of the ulnar nerve

Tendon of extensor carpi radialis brevis

Tendon of extensor digiti minimi

A

The superficial branch of the radial nerve passes superior to the extensor retinaculum in the position of this laceration and is at greatest risk of injury. The dorsal branch of the ulnar nerve and artery also pass superior to the extensor retinaculum but are located medially.

Extensor retinaculum

The extensor retinaculum is a thickening of the deep fascia that stretches across the back of the wrist and holds the long extensor tendons in position.
Its attachments are:
The pisiform and triquetral medially
The end of the radius laterally

Structures related to the extensor retinaculum
Structures superficial to the retinaculum
Basilic vein
Dorsal cutaneous branch of the ulnar nerve
Cephalic vein
Superficial branch of the radial nerve
Structures passing deep to the extensor retinaculum
Extensor carpi ulnaris tendon
Extensor digiti minimi tendon
Extensor digitorum and extensor indicis tendon
Extensor pollicis longus tendon
Extensor carpi radialis longus tendon
Extensor carpi radialis brevis tendon
Abductor pollicis longus and extensor pollicis brevis tendons

Beneath the extensor retinaculum fibrous septa form six compartments that contain the extensor muscle tendons. Each compartment has its own synovial sheath.

The radial artery
The radial artery passes between the lateral collateral ligament of the wrist joint and the tendons of the abductor pollicis longus and extensor pollicis brevis.

Image illustrating the topography of tendons passing under the extensor retinaculum

How well did you know this?
1
Not at all
2
3
4
5
Perfectly
744
Q

An 18 year old lady with troublesome hyperhidrosis of the hands and arms is due to undergo a sympathectomy to treat the condition. Which of the following should the surgeons divide to most effectively treat her condition?

Sympathetic ganglia at T1, T2 and T3

Sympathetic ganglia at T2 and T3

Sympathetic ganglia at T1 and T2

Stellate ganglion

Superior cervical ganglion

A

To treat hyperhidrosis the sympathetic ganglia at T2 and T3 should be divided. Dividing the other structures listed would either carry a risk of Horners syndrome or be ineffective.
Useful mnemonic: T2 & T3 Make the hands sweat-free

Sympathetic nervous system- anatomy

The cell bodies of the pre-ganglionic efferent neurones lie in the lateral horn of the grey matter of the spinal cord in the thoraco-lumbar regions.
The pre-ganglionic efferents leave the spinal cord at levels T1-L2. These pass to the sympathetic chain.
Lateral branches of the sympathetic chain connect it to every spinal nerve. These post ganglionic nerves will pass to structures that receive sympathetic innervation at the periphery.

Sympathetic chains
These lie on the vertebral column and run from the base of the skull to the coccyx.
Cervical region Lie anterior to the transverse processes of the cervical vertebrae and posterior to the carotid sheath.
Thoracic region Lie anterior to the neck of the upper ribs and and lateral sides of the lower thoracic vertebrae.They are covered by the parietal pleura
Lumbar region Enter by passing posterior to the medial arcuate ligament. Lie anteriorly to the vertebrae and medial to psoas major.

Sympathetic ganglia
Superior cervical ganglion lies anterior to C2 and C3.
Middle cervical ganglion (if present) C6
Stellate ganglion- anterior to transverse process of C7, lies posterior to the subclavian artery, vertebral artery and cervical pleura.
Thoracic ganglia are segmentally arranged.
There are usually 4 lumbar ganglia.

Clinical importance
Interruption of the head and neck supply of the sympathetic nerves will result in an ipsilateral Horners syndrome.
For treatment of hyperhidrosis the sympathetic denervation can be achieved by removing the second and third thoracic ganglia with their rami. Removal of T1 will cause a Horners syndrome and is therefore not performed.
In patients with vascular disease of the lower limbs a lumbar sympathetomy may be performed, either radiologically or (more rarely now) surgically. The ganglia of L2 and below are disrupted. If L1 is removed then ejaculation may be compromised (and little additional benefit conferred as the preganglionic fibres do not arise below L2.

How well did you know this?
1
Not at all
2
3
4
5
Perfectly
745
Q

Which of these muscles is not a component of the rotator cuff?

Subscapularis

Teres minor

Supraspinatus

Infraspinatus

Deltoid

A

Mnemonic for rotator cuff muscles (TISS)
Teres minor
Infraspinatus
Supraspinatus
Subscapularis

Deltoid may abduct the s houlder and is not a rotator cuff muscle.

Muscles of the rotator cuff

Muscle Innervation
Supraspinatus muscle Suprascapular nerve
Infraspinatus muscle Suprascapular nerve
Teres minor muscle Axillary nerve
Subscapularis muscle Superior and inferior subscapular nerves

How well did you know this?
1
Not at all
2
3
4
5
Perfectly
746
Q

A 67 year old patient is due to undergo a femoro-popliteal bypass graft. Which heparin regime should the surgeon ask for prior to cross clamping the femoral artery?

Single therapeutic dose of low molecular weight heparin on the ward prior to coming to theatre

Single therapeutic dose of low molecular weight heparin the night before surgery

Dose of 10,000 units of unfractionated heparin prior to induction of anaesthesia

Dose of 3,000 units of unfractionated heparin, 3 minutes prior to cross clamping

Dose of 30,000 units of unfractionated heparin, 3 minutes prior to cross clamping

A

As a rule most vascular surgeons will administer approximately 3,000 units of systemic heparin 3-5 minutes prior to cross clamping to help prevent further intra arterial thromboses. A dose of 30,000 units is given prior to going on cardiopulmonary bypass. Heparin given at induction will cause bleeding during routine dissection.

Heparin

Causes the formation of complexes between antithrombin and activated thrombin/factors 7,9,10,11 & 12

Advantages of low molecular weight heparin
Better bioavailability
Lower risk of bleeding
Longer half life
Little effect on APTT at prophylactic dosages
Less risk of HIT

Complications
Bleeding
Osteoporosis
Heparin induced thrombocytopenia (HIT): occurs 5-14 days after 1st exposure
Anaphylaxis

In surgical patients that may need a rapid return to theatre, administration of unfractionated heparin is preferred; as low molecular weight heparins have a longer duration of action and are harder to reverse.

How well did you know this?
1
Not at all
2
3
4
5
Perfectly
747
Q

A 55 year old man is admitted with a brisk haematemesis. He is taken to the endoscopy department and an upper GI endoscopy is performed by the gastroenterologist. He identifies an ulcer on the posterior duodenal wall and spends an eternity trying to control the bleeding with all the latest haemostatic techniques. He eventually asks the surgeons for help. A laparotomy and anterior duodenotomy are performed, as the surgeon opens the duodenum a vessel is spurting blood into the duodenal lumen. From which of the following does this vessel arise?

Left gastric artery

Common hepatic artery

Right hepatic artery

Superior mesenteric artery

Splenic artery

A

The vessel will be the gastroduodenal artery, this arises from the common hepatic artery.

Gastroduodenal artery

Supplies
Pylorus, proximal part of the duodenum, and indirectly to the pancreatic head (via the anterior and posterior superior pancreaticoduodenal arteries)

Path
The gastroduodenal artery most commonly arises from the common hepatic artery of the coeliac trunk. It terminates by bifurcating into the right gastroepiploic artery and the superior pancreaticoduodenal artery

How well did you know this?
1
Not at all
2
3
4
5
Perfectly
748
Q

A 25 year old lady presents with a swelling located at the anterior border of the sternocleidomastoid muscle. The swelling is intermittent and on examination it is soft and fluctuant. What is the most likely diagnosis?

Thyroglossal cyst

Branchial cyst

Carotid body tumour

Infection with Bartonella

Dental abscess

A

Branchial cysts are remnants of the branchial cleft. They may become infected.

Neck lumps

The table below gives characteristic exam question features for conditions causing neck lumps:

Reactive lymphadenopathy By far the most common cause of neck swellings. There may be a history of local infection or a generalised viral illness
Lymphoma Rubbery, painless lymphadenopathy
The phenomenon of pain whilst drinking alcohol is very uncommon
There may be associated night sweats and splenomegaly
Thyroid swelling May be hypo-, eu- or hyperthyroid symptomatically
Moves upwards on swallowing
Thyroglossal cyst More common in patients < 20 years old
Usually midline, between the isthmus of the thyroid and the hyoid bone
Moves upwards with protrusion of the tongue
May be painful if infected
Pharyngeal pouch More common in older men
Represents a posteromedial herniation between thyropharyngeus and cricopharyngeus muscles
Usually not seen, but if large then a midline lump in the neck that gurgles on palpation
Typical symptoms are dysphagia, regurgitation, aspiration and chronic cough
Cystic hygroma A congenital lymphatic lesion (lymphangioma) typically found in the neck, classically on the left side
Most are evident at birth, around 90% present before 2 years of age
Branchial cyst An oval, mobile cystic mass that develops between the sternocleidomastoid muscle and the pharynx
Develop due to failure of obliteration of the second branchial cleft in embryonic development
Usually present in early adulthood
Cervical rib More common in adult females
Around 10% develop thoracic outlet syndrome
Carotid aneurysm Pulsatile lateral neck mass which doesn’t move on swallowing

How well did you know this?
1
Not at all
2
3
4
5
Perfectly
749
Q

What type of stoma should be considered in a patient undergoing emergency operative intervention for large bowel obstruction as a result of a carcinoma 5cm from the anal verge?

End colostomy

Loop colostomy

End ileostomy

Loop jejunostomy

Loop ileostomy

A

Don’t confuse loop ileostomy and loop colostomy. A loop colostomy is the only safe option for an obstructing rectal cancer. Loop ileostomy in the context of rectal cancer is performed to mitigate the effects of anastomotic leak following a low anterior resection.
Large bowel obstruction resulting from carcinoma should be resected, stented or defunctioned. The first two options typically apply to tumours above the peritoneal reflection. Lower tumours should be defunctioned with a loop colostomy and then formal staging undertaken prior to definitive surgery. An emergency attempted rectal resection carries a high risk of involvement of the circumferential resection margin and is not recommended.

Abdominal stomas

Stomas may be sited during a range of abdominal procedures and involve bringing the lumen or visceral contents onto the skin. In most cases this applies to the bowel. However, other organs or their contents may be diverted in case of need.

With bowel stomas the type method of construction and to a lesser extent the site will be determined by the contents of the bowel. In practice, small bowel stomas should be spouted so that their irritant contents are not in contact with the skin. Colonic stomas do not need to be spouted as their contents are less irritant.

In the ideal situation the site of the stoma should be marked with the patient prior to surgery. Stoma siting is important as it will ultimately influence the ability of the patient to manage their stoma and also reduce the risk of leakage. Leakage of stoma contents and subsequent maceration of the surrounding skin can rapidly progress into a spiraling loss of control of stoma contents.

Types of stomas
Name of stoma Use Common sites
Gastrostomy
Gastric decompression or fixation
Feeding
Epigastrium
Loop jejunostomy
Seldom used as very high output
May be used following emergency laparotomy with planned early closure
Any location according to need
Percutaneous jejunostomy
Usually performed for feeding purposes and site in the proximal bowel
Usually left upper quadrant
Loop ileostomy
Defunctioning of colon e.g. following rectal cancer surgery
Does not decompress colon (if ileocaecal valve competent)
Usually right iliac fossa
End ileostomy
Usually following complete excision of colon or where ileo-colic anastomosis is not planned
May be used to defunction colon, but reversal is more difficult
Usually right iliac fossa
End colostomy Where a colon is diverted or resected and anastomosis is not primarily achievable or desirable Either left or right iliac fossa
Loop colostomy
To defunction a distal segment of colon
Since both lumens are present the distal lumen acts as a vent
May be located in any region of the abdomen, depending upon colonic segment used
Caecostomy Stoma of last resort where loop colostomy is not possible Right iliac fossa
Mucous fistula
To decompress a distal segment of bowel following colonic division or resection
Where closure of a distal resection margin is not safe or achievable

How well did you know this?
1
Not at all
2
3
4
5
Perfectly
750
Q

A 28 year old man falls on the back of his hand. On x-ray, he has a fractured distal radius demonstrating volar displacement of the fracture. What eponymous term is used to describe this?

Barton’s

Colles’

Smith’s

Pott’s

Galeazzi

A

This is a Smith fracture (reverse Colles’ fracture); unlike a Colles’ this is a high velocity injury and may require surgical correction. Note that Colles’ fractures are usually dorsally displaced.

Eponymous fractures

Colles’ fracture (dinner fork deformity)
Fall onto extended outstretched hand
Classical Colles’ fractures have the following 3 features:

  1. Transverse fracture of the radius
  2. 1 inch proximal to the radio-carpal joint
  3. Dorsal displacement and angulation

Smith’s fracture (reverse Colles’ fracture)
Volar angulation of distal radius fragment (Garden spade deformity)
Caused by falling backwards onto the palm of an outstretched hand or falling with wrists flexed

Bennett’s fracture
Intra-articular fracture of the first carpometacarpal joint
Impact on flexed metacarpal, caused by fist fights
X-ray: triangular fragment at ulnar base of metacarpal

Monteggia’s fracture
Dislocation of the proximal radioulnar joint in association with an ulna fracture
Fall on outstretched hand with forced pronation
Needs prompt diagnosis to avoid disability

Galeazzi fracture
Radial shaft fracture with associated dislocation of the distal radioulnar joint
Direct blow

Pott’s fracture
Bimalleolar ankle fracture
Forced foot eversion

Barton’s fracture
Distal radius fracture (Colles’/Smith’s) with associated radiocarpal dislocation
Fall onto extended and pronated wrist
Involvement of the joint is a defining feature

Holstein Lewis Fracture
A HolsteinLewis fracture is a fracture of the distal third of the humerus resulting in entrapment of the radial nerve.
The radial nerve is one of the major nerves of the upper limb. It innervates all of the muscles in the extensor compartments of the arm.
Conservative treatment includes reduction and use of a functional brace
Vascular injury may require open surgery

How well did you know this?
1
Not at all
2
3
4
5
Perfectly
751
Q

A 23 year old man is undergoing an inguinal hernia repair under local anaesthesia. The surgeon encounters a bleeding site which he manages with diathermy. About a minute or so later the patient complains that he is able to feel the burning pain of the heat at the operative site. Which of the following nerve fibres is responsible for the transmission of this signal?

A α fibres

A β fibres

B fibres

C fibres

None of the above

A

Slow transmission of mechanothermal stimuli is transmitted via C fibres.
A γ fibres transmit information relating to motor proprioception, A β fibres transmit touch and pressure and B fibres are autonomic fibres.

Pain - neuronal transmission

Somatic pain
Peripheral nociceptors are innervated by either small myelinated fibres (A-delta) fibres or by unmyelinated C fibres.
The A gamma fibres register high intensity mechanical stimuli. The C fibres usually register high intensity mechanothermal stimuli.

How well did you know this?
1
Not at all
2
3
4
5
Perfectly
752
Q

A 19 year old female is involved in an athletics event. She has just completed the high jump when she suddenly develops severe back pain and weakness affecting both her legs. On examination, she has a prominent sacrum and her lower back is painful. What is the most likely underlying cause?

Structural scoliosis

Ankylosing spondylitis

Scheuermanns disease

Spondylolisthesis

Spondylolysis

A

Young athletic females are the group most frequently affected by spondylolythesis who have a background of spondylolysis. Whilst the latter condition is a risk factor for spondylolythesis the former condition is most likely in a young athletic female who presents with sudden pain.

Diseases affecting the vertebral column

Ankylosing spondylitis
Chronic inflammatory disorder affecting the axial skeleton
Sacro-ilitis is a usually visible in plain films
Up to 20% of those who are HLA B27 positive will develop the condition
Affected articulations develop bony or fibrous changes
Typical spinal features include loss of the lumbar lordosis and progressive kyphosis of the cervico-thoracic spine
Scheuermann’s disease
Epiphysitis of the vertebral joints is the main pathological process
Predominantly affects adolescents
Symptoms include back pain and stiffness
X-ray changes include epiphyseal plate disturbance and anterior wedging
Clinical features include progressive kyphosis (at least 3 vertebrae must be involved)
Minor cases may be managed with physiotherapy and analgesia, more severe cases may require bracing or surgical stabilisation
Scoliosis
Consists of curvature of the spine in the coronal plane
Divisible into structural and non structural, the latter being commonest in adolescent females who develop minor postural changes only. Postural scoliosis will typically disappear on manoeuvres such as bending forwards
Structural scoliosis affects > 1 vertebral body and is divisible into idiopathic, congential and neuromuscular in origin. It is not correctable by alterations in posture
Within structural scoliosis, idiopathic is the most common type
Severe, or progressive structural disease is often managed surgically with bilateral rod stabilisation of the spine
Spina bifida
Non fusion of the vertebral arches during embryonic development
Three categories; myelomeningocele, spina bifida occulta and meningocele
Myelomeningocele is the most severe type with associated neurological defects that may persist in spite of anatomical closure of the defect
Up to 10% of the population may have spina bifida occulta, in this condition the skin and tissues (but not not bones) may develop over the distal cord. The site may be identifiable by a birth mark or hair patch
The incidence of the condition is reduced by use of folic acid supplements during pregnancy
Spondylolysis
Congenital or acquired deficiency of the pars interarticularis of the neural arch of a particular vertebral body, usually affects L4/ L5
May be asymptomatic and affects up to 5% of the population
Spondylolysis is the commonest cause of spondylolisthesis in children
Asymptomatic cases do not require treatment
Spondylolisthesis
This occurs when one vertebra is displaced relative to its immediate inferior vertebral body
May occur as a result of stress fracture or spondylolysis
Traumatic cases may show the classic ‘Scotty Dog’ appearance on plain films
Treatment depends upon the extent of deformity and associated neurological symptoms, minor cases may be actively monitored. Individuals with radicular symptoms or signs will usually require spinal decompression and stabilisation

How well did you know this?
1
Not at all
2
3
4
5
Perfectly
753
Q

A 23 year old lady with troublesome axillary hyperhidrosis is undergoing a thorascopic sympathectomy to treat the condition. Which of the following structures will need to be divided to access the sympathetic trunk?

Intercostal vein

Intercostal artery

Parietal pleura

Visceral pleura

None of the above

A

The sympathetic chain lies posterior to the parietal pleura. During a thorascopic sympathectomy this structure will need to be divided. The intercostal vessels lie posteriorly. They may be damaged with troublesome bleeding but otherwise are best left alone as deliberate division will not improve surgical access.
Sympathetic nervous system- anatomy

The cell bodies of the pre-ganglionic efferent neurones lie in the lateral horn of the grey matter of the spinal cord in the thoraco-lumbar regions.
The pre-ganglionic efferents leave the spinal cord at levels T1-L2. These pass to the sympathetic chain.
Lateral branches of the sympathetic chain connect it to every spinal nerve. These post ganglionic nerves will pass to structures that receive sympathetic innervation at the periphery.

Sympathetic chains
These lie on the vertebral column and run from the base of the skull to the coccyx.
Cervical region Lie anterior to the transverse processes of the cervical vertebrae and posterior to the carotid sheath.
Thoracic region Lie anterior to the neck of the upper ribs and and lateral sides of the lower thoracic vertebrae.They are covered by the parietal pleura
Lumbar region Enter by passing posterior to the medial arcuate ligament. Lie anteriorly to the vertebrae and medial to psoas major.

Sympathetic ganglia
Superior cervical ganglion lies anterior to C2 and C3.
Middle cervical ganglion (if present) C6
Stellate ganglion- anterior to transverse process of C7, lies posterior to the subclavian artery, vertebral artery and cervical pleura.
Thoracic ganglia are segmentally arranged.
There are usually 4 lumbar ganglia.

Clinical importance
Interruption of the head and neck supply of the sympathetic nerves will result in an ipsilateral Horners syndrome.
For treatment of hyperhidrosis the sympathetic denervation can be achieved by removing the second and third thoracic ganglia with their rami. Removal of T1 will cause a Horners syndrome and is therefore not performed.
In patients with vascular disease of the lower limbs a lumbar sympathetomy may be performed, either radiologically or (more rarely now) surgically. The ganglia of L2 and below are disrupted. If L1 is removed then ejaculation may be compromised (and little additional benefit conferred as the preganglionic fibres do not arise below L2.

How well did you know this?
1
Not at all
2
3
4
5
Perfectly
754
Q

The recurrent laryngeal is connected to which of the following nerves?

Trigeminal

Accessory

Hypoglossal

Vagus

Glossopharyngeal

A

Recurrent laryngeal nerve

Branch of the vagus nerve

Path

Right
Arises anterior to the subclavian artery and ascends obliquely next to the trachea, behind the common carotid artery
It is either anterior or posterior to the inferior thyroid artery

Left
Arises left to the arch of the aorta
Winds below the aorta
Ascends along the side of the trachea

Then both

Pass in a groove between the trachea and oesophagus
Enters the larynx behind the articulation between the thyroid cartilage and cricoid
Distributed to larynx muscles

Branches to

Cardiac plexus
Mucous membrane and muscular coat of the oesophagus and trachea

Innervates

Intrinsic larynx muscles (excluding cricothyroid)

How well did you know this?
1
Not at all
2
3
4
5
Perfectly
755
Q

Stimulation of the parasympathetic nerve supply to the bladder is most likely to result in which of the following actions?

Contraction of the internal urethral sphincter

Relaxation of the external urethral sphincter

Contraction of the external urethral sphincter

Contraction of the detrusor muscle

Relaxation of the detrusor muscle

A

The internal sphincter is formed by the circular arrangement of smooth muscle of the bladder wall at the neck of the bladder. The detrusor muscle and internal sphincter are under autonomic control. Parasympathetic fibres cause contraction of the detrusor muscle and relaxation of the internal sphincter. Sympathetic fibres cause relaxation of the detrusor muscle and contraction of the internal sphincter. The external sphincter is under voluntary control.

Bladder

The empty bladder is contained within the pelvic cavity. It is usually a three sided pyramid. The apex of the bladder points forwards towards the symphysis pubis and the base lies immediately anterior to the rectum or vagina. Continuous with the apex is the median umbilical ligament, during development this was the site of the urachus.
The inferior aspect of the bladder is retroperitoneal and the superior aspect covered by peritoneum. As the bladder distends it will tend to separate the peritoneum from the fascia of transversalis. For this reason a bladder that is distended due to acute urinary retention may be approached with a suprapubic catheter that avoids entry into the peritoneal cavity.
The trigone is the least mobile part of the bladder and forms the site of the ureteric orifices and internal urethral orifice. In the empty bladder the ureteric orifices are approximately 2-3cm apart, this distance may increase to 5cm in the distended bladder.

Arterial supply
The superior and inferior vesical arteries provide the main blood supply to the bladder. These are branches of the internal iliac artery.

Venous drainage
In males the bladder is drained by the vesicoprostatic venous plexus. In females the bladder is drained by the vesicouterine venous plexus. In both sexes this venous plexus will ultimately drain to the internal iliac veins.

Lymphatic drainage
Lymphatic drainage is predominantly to the external iliac nodes, internal iliac and obturator nodes also form sites of bladder lymphatic drainage.

Innervation
Parasympathetic nerve fibres innervate the bladder from the pelvic splanchnic nerves. Sympathetic nerve fibres are derived from L1 and L2 via the hypogastric nerve plexuses. The parasympathetic nerve fibres will typically cause detrusor muscle contraction and result in voiding. The muscle of the trigone is innervated by the sympathetic nervous system. The external urethral sphincter is under conscious control. During bladder filling the rate of firing of nerve impulses to the detrusor muscle is low and receptive relaxation occurs. At higher volumes and increased intra vesical pressures the rate of neuronal firing will increase and eventually voiding will occur.

How well did you know this?
1
Not at all
2
3
4
5
Perfectly
756
Q

A 43 year old lady undergoes a day case laparoscopic cholecystectomy. The operation is more difficult than anticipated and a drain is placed to the operative site. Whilst in recovery, the patient loses 1800ml of frank blood into the drain. Which of the following will not occur?

Release of aldosterone via the Bainbridge reflex

Reduced urinary sodium excretion

Increase in sympathetic discharge to ventricular muscle

Fall in parasympathetic discharge to the sino atrial node

Decreased stimulation from atrial pressure receptors

A

The Bainbridge reflex is the increase in heart rate mediated via atrial stretch receptors that occurs following a rapid infusion of blood. Note the question asks which will not occur and that is why it is A and not the other options.
Cardiac physiology

  • The heart has four chambers ejecting blood into both low pressure and high pressure systems.
    The pumps generate pressures of between 0-25mmHg on the right side and 0-120 mmHg on the left.
    At rest diastole comprises 2/3 of the cardiac cycle.
    The product of the frequency of heart rate and stroke volume combine to give the cardiac output which is typically 5-6L per minute.

Detailed descriptions of the various waveforms are often not a feature of MRCS A (although they are on the syllabus). However, they are a very popular topic for surgical physiology in the MRCS B exam.

Electrical properties
Intrinsic myogenic rhythm within cardiac myocytes means that even the denervated heart is capable of contraction.
In the normal situation the cardiac impulse is generated in the sino atrial node in the right atrium and conveyed to the ventricles via the atrioventricular node.
The sino atrial node is also capable of spontaneous discharge and in the absence of background vagal tone will typically discharge around 100x per minute. Hence the higher resting heart rate found in cardiac transplant cases. In the SA and AV nodes the resting membrane potential is lower than in surrounding cardiac cells and will slowly depolarise from -70mV to around -50mV at which point an action potential is generated.
Differences in the depolarisation slopes between SA and AV nodes help to explain why the SA node will depolarise first. The cells have a refractory period during which they cannot be re-stimulated and this period allows for adequate ventricular filling. In pathological tachycardic states this time period is overridden and inadequate ventricular filling may then occur, cardiac output falls and syncope may ensue.

Parasympathetic fibres project to the heart via the vagus and will release acetylcholine. Sympathetic fibres release nor adrenaline and circulating adrenaline comes from the adrenal medulla. Noradrenaline binds to β 1 receptors in the SA node and increases the rate of pacemaker potential depolarisation.

Mid diastole: AV valves open. Ventricles hold 80% of final volume. Outflow valves shut. Aortic pressure is high.

Late diastole: Atria contract. Ventricles receive 20% to complete filling. Typical end diastolic volume 130-160ml.

Early systole: AV valves shut. Ventricular pressure rises. Isovolumetric ventricular contraction. AV Valves bulge into atria (c-wave). Aortic and pulmonary pressure exceeded- blood is ejected. Shortening of ventricles pulls atria downwards and drops intra atrial pressure (x-descent).

Late systole: Ventricular muscles relax and ventricular pressures drop. Although ventricular pressure drops the aortic pressure remains constant owing to peripheral vascular resistance and elastic property of the aorta. Brief period of retrograde flow that occurs in aortic recoil shuts the aortic valve. Ventricles will contain 60ml end systolic volume. The average stroke volume is 70ml (i.e. Volume ejected).

Early diastole: All valves are closed. Isovolumetric ventricular relaxation occurs. Pressure wave associated with closure of the aortic valve increases aortic pressure. The pressure dip before this rise can be seen on arterial waveforms and is called the incisura. During systole the atrial pressure increases such that it is now above zero (v- wave). Eventually atrial pressure exceed ventricular pressure and AV valves open - atria empty passively into ventricles and atrial pressure falls (y -descent )

The negative atrial pressures are of clinical importance as they can allow air embolization to occur if the neck veins are exposed to air. This patient positioning is important in head and neck surgery to avoid this occurrence if veins are inadvertently cut, or during CVP line insertion.

Mechanical properties
Preload = end diastolic volume
Afterload = aortic pressure

It is important to understand the principles of Laplace’s law in surgery.
It states that for hollow organs with a circular cross section, the total circumferential wall tension depends upon the circumference of the wall, multiplied by the thickness of the wall and on the wall tension.
The total luminal pressure depends upon the cross sectional area of the lumen and the transmural pressure. Transmural pressure is the internal pressure minus external pressure and at equilibrium the total pressure must counterbalance each other.
In terms of cardiac physiology the law explains that the rise in ventricular pressure that occurs during the ejection phase is due to physical change in heart size. It also explains why a dilated diseased heart will have impaired systolic function.

Starlings law
Increase in end diastolic volume will produce larger stroke volume.
This occurs up to a point beyond which cardiac fibres are excessively stretched and stroke volume will fall once more. It is important for the regulation of cardiac output in cardiac transplant patients who need to increase their cardiac output.

Baroreceptor reflexes
Baroreceptors located in aortic arch and carotid sinus.
Aortic baroreceptor impulses travel via the vagus and from the carotid via the glossopharyngeal nerve.
They are stimulated by arterial stretch.
Even at normal blood pressures they are tonically active.
Increase in baroreceptor discharge causes:

*Increased parasympathetic discharge to the SA node.
*Decreased sympathetic discharge to ventricular muscle causing decreased contractility and fall in stroke volume.
*Decreased sympathetic discharge to venous system causing increased compliance.
*Decreased peripheral arterial vascular resistance

Atrial stretch receptors
Located in atria at junction between pulmonary veins and vena cava.
Stimulated by atrial stretch and are thus low pressure sensors.
Increased blood volume will cause increased parasympathetic activity.
Very rapid infusion of blood will result in increase in heart rate mediated via atrial receptors: the Bainbridge reflex.
Decreases in receptor stimulation results in increased sympathetic activity this will decrease renal blood flow-decreases GFR-decreases urinary sodium excretion-renin secretion by juxtaglomerular apparatus-Increase in angiotensin II.
Increased atrial stretch will also result in increased release of atrial natriuretic peptide.

How well did you know this?
1
Not at all
2
3
4
5
Perfectly
757
Q

A 22 year old man is involved in a fight outside a nightclub. He is stabbed in the back, on the left side, approximately 3cm below the 12th rib in the mid scapular line. The structure most likely to be injured first as a result is the:

Spleen

Left kidney

Left adrenal gland

Left ureter

None of the above

A

The left kidney lies in this location and is the most likely structure to be injured. The Spleen lies more superiorly, and the left adrenal and ureter are unlikely to be injured in isolation.
Levels

Transpyloric plane
Level of the body of L1

Pylorus stomach
Left kidney hilum (L1- left one!)
Fundus of the gallbladder
Neck of pancreas
Duodenojejunal flexure
Superior mesenteric artery
Portal vein
Left and right colic flexure
Root of the transverse mesocolon
2nd part of the duodenum
Upper part of conus medullaris
Spleen

Can be identified by asking the supine patient to sit up without using their arms. The plane is located where the lateral border of the rectus muscle crosses the costal margin.

Anatomical planes
Subcostal plane Lowest margin of 10th costal cartilage
Intercristal plane Level of body L4 (highest point of iliac crest)
Intertubercular plane Level of body L5

Common level landmarks
Inferior mesenteric artery L3
Bifurcation of aorta into common iliac arteries L4
Formation of IVC L5 (union of common iliac veins)
Diaphragm apertures
Vena cava T8
Oesophagus T10
Aortic hiatus T12

How well did you know this?
1
Not at all
2
3
4
5
Perfectly
758
Q

Which of the following represents the root values of the sciatic nerve?

L4 to S3

L1 to L4

L3 to S1

S1 to S4

L5 to S1

A

The sciatic nerve most commonly arises from L4 to S3.

Sciatic nerve

The sciatic nerve is formed from the sacral plexus and is the largest nerve in the body. It is the continuation of the main part of the plexus arising from ventral rami of L4 to S3. These rami converge at the inferior border of piriformis to form the nerve itself. It passes through the inferior part of the greater sciatic foramen and emerges beneath piriformis. Medially, lie the inferior gluteal nerve and vessels and the pudendal nerve and vessels. It runs inferolaterally under the cover of gluteus maximus midway between the greater trochanter and ischial tuberosity. It receives its blood supply from the inferior gluteal artery. The nerve provides cutaneous sensation to the skin of the foot and the leg. It also innervates the posterior thigh muscles and the lower leg and foot muscles. The nerve splits into the tibial and common peroneal nerves approximately half way down the posterior thigh. The tibial nerve supplies the flexor muscles and the common peroneal nerve supplies the extensor muscles and the evertor muscles of the foot.

Summary points
Origin Spinal nerves L4 - S3
Articular Branches Hip joint
Muscular branches in upper leg
Semitendinosus
Semimembranosus
Biceps femoris
Part of adductor magnus
Cutaneous sensation
Posterior aspect of thigh (via cutaneous nerves)
Gluteal region
Entire lower leg (except the medial aspect)
Terminates At the upper part of the popliteal fossa by dividing into the tibial and peroneal nerves

The nerve to the short head of the biceps femoris comes from the common peroneal part of the sciatic and the other muscular branches arise from the tibial portion.
The tibial nerve goes on to innervate all muscles of the foot except the extensor digitorum brevis (which is innervated by the common peroneal nerve).

How well did you know this?
1
Not at all
2
3
4
5
Perfectly
759
Q

Which of the structures listed below is not a content of the carotid sheath?

Internal jugular vein

Internal carotid artery

Vagus nerve

Recurrent laryngeal nerve

Common carotid artery

A

Contents of carotid sheath:
Common carotid artery
Internal carotid artery
Internal jugular vein
Vagus nerve

Common carotid artery

The right common carotid artery arises at the bifurcation of the brachiocephalic trunk, the left common carotid arises from the arch of the aorta. Both terminate at the level of the upper border of the thyroid cartilage (the lower border of the third cervical vertebra) by dividing into the internal and external carotid arteries.

Left common carotid artery
This vessel arises immediately to the left and slightly behind the origin of the brachiocephalic trunk. Its thoracic portion is 2.5- 3.5 cm in length and runs superolaterally to the sternoclavicular joint.

In the thorax
The vessel is in contact, from below upwards, with the trachea, left recurrent laryngeal nerve, left margin of the oesophagus. Anteriorly the left brachiocephalic vein runs across the artery, and the cardiac branches from the left vagus descend in front of it. These structures together with the thymus and the anterior margins of the left lung and pleura separate the artery from the manubrium.

In the neck
The artery runs superiorly deep to sternocleidomastoid and then enters the anterior triangle. At this point it lies within the carotid sheath with the vagus nerve and the internal jugular vein. Posteriorly the sympathetic trunk lies between the vessel and the prevertebral fascia. At the level of C7 the vertebral artery and thoracic duct lie behind it. The anterior tubercle of C6 transverse process is prominent and the artery can be compressed against this structure (it corresponds to the level of the cricoid).
Anteriorly at C6 the omohyoid muscle passes superficial to the artery.
Within the carotid sheath the jugular vein lies lateral to the artery.

Right common carotid artery
The right common carotid arises from the brachiocephalic artery. The right common carotid artery corresponds with the cervical portion of the left common carotid, except that there is no thoracic duct on the right. The oesophagus is less closely related to the right carotid than the left.

Summary points about the carotid anatomy

Path
Passes behind the sternoclavicular joint (12% patients above this level) to the upper border of the thyroid cartilage, to divide into the external (ECA) and internal carotid arteries (ICA).

Relations
Level of 6th cervical vertebra crossed by omohyoid
Then passes deep to the thyrohyoid, sternohyoid, sternomastoid muscles.
Passes anterior to the carotid tubercle (transverse process 6th cervical vertebra)-NB compression here stops haemorrhage.
The inferior thyroid artery passes posterior to the common carotid artery.
Then : Left common carotid artery crosses the thoracic duct, Right common carotid artery crossed by recurrent laryngeal nerve

How well did you know this?
1
Not at all
2
3
4
5
Perfectly
760
Q

Which of the following statements relating to necrotising enterocolitis is false?

It has a mortality of 30%

Most frequently presents in premature neonates less than 32 weeks gestation.

Should be managed by early laparotomy and segmental resections in most cases.

Pneumostosis intestinalis may be visible on plain abdominal x-ray.

May be minimised by use of breast milk over formula feeds.

A

Most cases will settle with conservative management with NG decompression and appropriate support. Laparotomy should be undertaken in patients do not who progress despite conservative management or in whom compelling indications for surgery exist (eg free air).

Paediatric Gastrointestinal disorders

Pyloric stenosis
M>F
5-10% Family history in parents
Projectile non bile stained vomiting at 4-6 weeks of life
Diagnosis is made by test feed or USS
Treatment: Ramstedt pyloromyotomy (open or laparoscopic)
Acute appendicitis
Uncommon under 3 years
When occurs may present atypically
Mesenteric adenitis
Central abdominal pain and URTI
Conservative management
Intussusception
Telescoping bowel
Proximal to or at the level of, ileocaecal valve
6-9 months age
Colicky pain, diarrhoea and vomiting, sausage shaped mass, red jelly stool.
Treatment: reduction with air insufflation
Malrotation
High caecum at the midline
Feature in exomphalos, congenital diaphragmatic hernia, intrinsic duodenal atresia
May be complicated by development of volvulus, infant with volvulus may have bile stained vomiting
Diagnosis is made by upper GI contrast study and USS
Treatment is by laparotomy, if volvulus is present (or at high risk of occurring then a ladds procedure is performed
Hirschsprung’s disease
Absence of ganglion cells from myenteric and submucosal plexuses
Occurs in 1/5000 births
Full thickness rectal biopsy for diagnosis
Delayed passage of meconium and abdominal distension
Treatment is with rectal washouts initially, thereafter an anorectal pull through procedure
Oesophageal atresia
Associated with tracheo-oesophageal fistula and polyhydramnios
May present with choking and cyanotic spells following aspiration
VACTERL associations
Meconium ileus
Usually delayed passage of meconium and abdominal distension
Majority have cystic fibrosis
X-Rays may not show a fluid level as the meconium is viscid (depends upon feeding), PR contrast studies may dislodge meconium plugs and be therapeutic
Infants who do not respond to PR contrast and NG N-acetyl cysteine will require surgery to remove the plugs
Biliary atresia
Jaundice > 14 days
Increased conjugated bilirubin
Urgent Kasai procedure
Necrotising enterocolitis
Prematurity is the main risk factor
Early features include abdominal distension and passage of bloody stools
X-Rays may show pneumatosis intestinalis and evidence of free air
Increased risk when empirical antibiotics are given to infants beyond 5 days
Treatment is with total gut rest and TPN, babies with perforations will require laparotomy

How well did you know this?
1
Not at all
2
3
4
5
Perfectly
761
Q

A patient has an arterial blood gas sample which provides the following result:
pH 7.20
pO2 7.5
Bicarbonate 22
pCO2 8.1
Chloride 10meq
What is the most likely cause?

Type II respiratory failure

Metabolic acidosis with increased anion gap

Metabolic alkalosis

Type I respiratory failure

Respiratory alkalosis

A

Type II respiratory failure

This is a sign of acute type 2 respiratory failure (non compensated). This is the result of carbon dioxide retention.

Metabolic acidosis
This is the most common surgical acid - base disorder.
Reduction in plasma bicarbonate levels.
Two mechanisms:
1. Gain of strong acid (e.g. diabetic ketoacidosis)
2. Loss of base (e.g. from bowel in diarrhoea)
- Classified according to the anion gap, this can be calculated by:
(Na+ + K+) - (Cl- + HCO3-).
- If a question supplies the chloride level then this is often a clue that the anion gap should be calculated. The normal range = 10-18 mmol/L

Normal anion gap ( = hyperchloraemic metabolic acidosis)
Gastrointestinal bicarbonate loss: diarrhoea, ureterosigmoidostomy, fistula
Renal tubular acidosis
Drugs: e.g. acetazolamide
Ammonium chloride injection
Addison’s disease

Raised anion gap
Lactate: shock, hypoxia
Ketones: diabetic ketoacidosis, alcohol
Urate: renal failure
Acid poisoning: salicylates, methanol

Metabolic acidosis secondary to high lactate levels may be subdivided into two types:
Lactic acidosis type A: (Perfusion disorders e.g.shock, hypoxia, burns)
Lactic acidosis type B: (Metabolic e.g. metformin toxicity)

Metabolic alkalosis
Usually caused by a rise in plasma bicarbonate levels.
Rise of bicarbonate above 24 mmol/L will typically result in renal excretion of excess bicarbonate.
Caused by a loss of hydrogen ions or a gain of bicarbonate. It is due mainly to problems of the kidney or gastrointestinal tract

Causes
Vomiting / aspiration (e.g. Peptic ulcer leading to pyloric stenosis, nasogastric suction)
Diuretics
Liquorice, carbenoxolone
Hypokalaemia
Primary hyperaldosteronism
Cushing’s syndrome
Bartter’s syndrome
Congenital adrenal hyperplasia

Mechanism of metabolic alkalosis
Activation of renin-angiotensin II-aldosterone (RAA) system is a key factor
Aldosterone causes reabsorption of Na+ in exchange for H+ in the distal convoluted tubule
ECF depletion (vomiting, diuretics) → Na+ and Cl- loss → activation of RAA system → raised aldosterone levels
In hypokalaemia, K+ shift from cells → ECF, alkalosis is caused by shift of H+ into cells to maintain neutrality

Respiratory acidosis
Rise in carbon dioxide levels usually as a result of alveolar hypoventilation
Renal compensation may occur leading to Compensated respiratory acidosis

Causes
COPD
Decompensation in other respiratory conditions e.g. Life-threatening asthma / pulmonary oedema
Sedative drugs: benzodiazepines, opiate overdose

Respiratory alkalosis
Hyperventilation resulting in excess loss of carbon dioxide
This will result in increasing pH

Causes
Psychogenic: anxiety leading to hyperventilation
Hypoxia causing a subsequent hyperventilation: pulmonary embolism, high altitude
Early salicylate poisoning*
CNS stimulation: stroke, subarachnoid haemorrhage, encephalitis
Pregnancy

*Salicylate overdose leads to a mixed respiratory alkalosis and metabolic acidosis. Early stimulation of the respiratory centre leads to a respiratory alkalosis whilst later the direct acid effects of salicylates (combined with acute renal failure) may lead to an acidosis

How well did you know this?
1
Not at all
2
3
4
5
Perfectly
762
Q

What is the most common cause of hypercalcaemia in the UK in hospitalised patients?

Thiazide use

Metastatic malignancy

Primary hyperparathyroidism

Osteogenic sarcoma

Sarcoidosis

A

Metastatic cancer accounts for most cases of hypercalcaemia in hospitalised patients. In the community primary hyperparathyroidism is the commonest cause.

Hypercalcaemia

Main causes
Malignancy (most common cause in hospital in-patients)
Primary hyperparathyroidism (commonest cause in non hospitalised patients)

Less common
Sarcoidosis (extrarenal synthesis of calcitriol )
Thiazides, lithium
Immobilisation
Pagets disease
Vitamin A/D toxicity
Thyrotoxicosis
MEN
Milk alkali syndrome

Clinical features
Stones, bones, abdominal groans, and psychic moans
High serum calcium levels result in decreased neuronal excitability. Therefore sluggish reflexes, muscle weakness and constipation may occur.

How well did you know this?
1
Not at all
2
3
4
5
Perfectly
763
Q

Which of the following features does not put a patient at risk of refeeding syndrome?

BMI < 16 kg/m

Alcohol abuse

Thyrotoxicosis

Chemotherapy

Diuretics

A

Thyrotoxicosis
Diuretics increase the risk of re-feeding syndrome through a process of increasing the risk of depletion of key electrolytes.

Nutrition - Refeeding syndrome

Refeeding syndrome describes the metabolic abnormalities which occur on feeding a person following a period of starvation. The metabolic consequences include:
Hypophosphataemia
Hypokalaemia
Hypomagnesaemia
Abnormal fluid balance
These abnormalities can lead to organ failure.

Re-feeding problems
If patient not eaten for > 5 days, aim to re-feed at < 50% energy and protein levels

High risk for re-feeding problems
If one or more of the following:
BMI < 16 kg/m2
Unintentional weight loss >15% over 3-6 months
Little nutritional intake > 10 days
Hypokalaemia, Hypophosphataemia or hypomagnesaemia prior to feeding (unless high)

If two or more of the following:
BMI < 18.5 kg/m2
Unintentional weight loss > 10% over 3-6 months
Little nutritional intake > 5 days
History of: alcohol abuse, drug therapy including insulin, chemotherapy, diuretics and antacids

Prescription
Start at up to 10 kcal/kg/day increasing to full needs over 4-7 days
Start immediately before and during feeding: oral thiamine 200-300mg/day, vitamin B co strong 1 tds and supplements
Give K+ (2-4 mmol/kg/day), phosphate (0.3-0.6 mmol/kg/day), magnesium (0.2-0.4 mmol/kg/day)

How well did you know this?
1
Not at all
2
3
4
5
Perfectly
764
Q

A 6 year old child presents with colicky abdominal pain, vomiting and the passage of red current jelly stool per rectum. On examination, the child has a tender abdomen and a palpable mass in the right upper quadrant. Imaging shows an intussusception. Which of the conditions below is least recognised as a precipitant?

Inflammation of Peyers patches

Cystic fibrosis

Meckels diverticulum

Mesenteric cyst

Mucosal polyps

A

Mesenteric cysts may be associated with intra abdominal catastrophes where these occur they are typically either intestinal volvulus or intestinal infarction. They seldom cause intussusception. Cystic fibrosis may lead to the formation of meconium ileus equivalent and plugs may occasionally serve as the lead points for an intussusception.

Intussusception- Paediatric

Intussusception typically presents with colicky abdominal pain and vomiting. The telescoping of the bowel produces mucosal ischaemia and bleeding may occur resulting in the passage of red current jelly stools. Recognised causes include lumenal pathologies such as polyps, lymphadenopathy and diseases such as cystic fibrosis. Idiopathic intussceception of the ileocaecal valve and terminal ileum is the most common variant and typically affects young children and toddlers.

The diagnosis is usually made by abdominal ultrasound investigation. The decision as to the optimal treatment is dictated by the patients physiological status and abdominal signs. In general, children who are unstable with localising peritoneal signs should undergo laparotomy, as should those in whom attempted radiological reduction has failed.

In relatively well children, without localising signs, attempted pneumatic reduction under fluroscopic guidance is the usual treatment. This is not done where there are concerns about impending perforation or if there is a small bowel intussusception.

How well did you know this?
1
Not at all
2
3
4
5
Perfectly
765
Q

Which of the following is not an extraintestinal feature of Crohns disease?

Iritis

Clubbing

Aphthous ulcers

Erythema multiforme

Pyoderma gangrenosum

A

Extraintestinal manifestation of inflammatory bowel disease: A PIE SAC

Aphthous ulcers
Pyoderma gangrenosum
Iritis
Erythema nodosum
Sclerosing cholangitis
Arthritis
Clubbing

Crohns disease

Crohns disease is a chronic transmural inflammation of a segment(s) of the gastrointestinal tract and may be associated with extra intestinal manifestations. Frequent disease patterns observed include ileal, ileocolic and colonic disease. Peri-anal disease may occur in association with any of these. The disease is often discontinuous in its distribution. Inflammation may cause ulceration, fissures, fistulas and fibrosis with stricturing. Histology reveals a chronic inflammatory infiltrate that is usually patchy and transmural.

Ulcerative colitis Vs Crohns
Crohn’s disease Ulcerative colitis
Distribution Mouth to anus Rectum and colon
Macroscopic changes Cobblestone appearance, apthoid ulceration Contact bleeding
Depth of disease Transmural inflammation Superficial inflammation
Distribution pattern Patchy Continuous
Histological features Granulomas (non caseating epithelioid cell aggregates with Langhans’ giant cells) Crypt abscesses, Inflammatory cells in the lamina propria

Extraintestinal manifestations of Crohns
Related to disease extent Unrelated to disease extent
Aphthous ulcers (10%) Sacroiliiitis (10-15%)
Erythema nodosum (5-10%) Ankylosing spondylitis (1-2%)
Pyoderma gangrenosum (0.5%) Primary sclerosing cholangitis (Rare)
Acute arthropathy (6-12%) Gallstones (up to 30%)
Ocular complications (up to 10%) Renal calculi (up to 10%)

Diarrhoea in Crohns
Diarrhoea in Crohns may be multifactorial since actual inflammation of the colon is not common. Causes therefore include the following:
Bile salt diarrhoea secondary to terminal ileal disease
Entero-colic fistula
Short bowel due to multiple resections
Bacterial overgrowth

Surgical interventions in Crohns disease
The commonest disease pattern in Crohns is stricturing terminal ileal disease and this often culminates in an ileocaecal resection. Other procedures performed include segmental small bowel resections and stricturoplasty. Colonic involvement in patients with Crohns is not common and, where found, distribution is often segmental. However, despite this distribution segmental resections of the colon in patients with Crohns disease are generally not advocated because the recurrence rate in the remaining colon is extremely high. As a result, the standard options of colonic surgery in Crohns patients are generally; sub total colectomy, panproctocolectomy and staged sub total colectomy and proctectomy. Restorative procedures such as ileoanal pouch have no role in therapy.
Crohns disease is notorious for the developmental of intestinal fistulae; these may form between the rectum and skin (peri anal) or the small bowel and skin. Fistulation between loops of bowel may also occur and result in bacterial overgrowth and malabsorption. Management of enterocutaneous fistulae involves controlling sepsis, optimising nutrition, imaging the disease and planning definitive surgical management.

How well did you know this?
1
Not at all
2
3
4
5
Perfectly
766
Q

A 56 year old man presents with urinary symptoms and on investigation is found to have a 2cm nodule in the left lobe of the prostate. Imaging with whole body MRI and pelvic CT/MRI demonstrates a likely cancer with no distant disease and no nodal metastasis. What is the most appropriate course of action?

Robotic prostatectomy

Open prostatectomy

Transvesical prostatectomy

Prostatic biopsy

Radical pelvic radiotherapy

A

Prostate cancers are typically biopsied prior to treatment.

Prostate Cancer

Prostate Cancer
This is a common condition and up to 30,000 men are diagnosed with the condition each year. Up to 9,000 will die in in the UK from the condition per year.

Diagnosis
Early prostate cancers have few symptoms.
Metastatic disease may present as bone pain.
Locally advanced disease may present as pelvic pain or with urinary symptoms.
Prostate specific antigen measurement
Digital rectal examination
Trans rectal USS (+/- biopsy)
MRI/ CT and bone scan for staging.

PSA Test
The normal upper limit for PSA is 4ng/ml. However, in this group will lie patients with benign disease and some with localised prostate cancer. False positives may be due to prostatitis, UTI, BPH, vigorous DRE.
The percentage of free: total PSA may help to distinguish benign disease from cancer. Values of <20% are suggestive of cancer and biopsy is advised.

Pathology
95% adenocarcinoma
In situ malignancy is sometimes found in areas adjacent to cancer. Multiple biopsies needed to call true in situ disease.
Often multifocal- 70% lie in the peripheral zone.
Graded using the Gleason grading system, two grades awarded 1 for most dominant grade (on scale of 1-5) and 2 for second most dominant grade (scale 1-5). The two added together give the Gleason score. Where 2 is best prognosis and 10 the worst.
Lymphatic spread occurs first to the obturator nodes and local extra prostatic spread to the seminal vesicles is associated with distant disease.

Treatment
Watch and wait- Elderly, multiple co-morbidities, low Gleason score
Radiotherapy (External)- Both potentially curative and palliative therapy possible. However, radiation proctitis and rectal malignancy are late problems. Brachytherapy is a modification allowing internal radiotherapy.
Surgery- Radical prostatectomy. Surgical removal of the prostate is the standard treatment for localised disease. The robot is being used increasingly for this procedure. As well as the prostate the obturator nodes are also removed to complement the staging process. Erectile dysfunction is a common side effect. Survival may be better than with radiotherapy (see references). Functional outcomes are better when a robotic approach is used.
Hormonal therapy- Testosterone stimulates prostate tissue and prostatic cancers usually show some degree of testosterone dependence. 95% of testosterone is derived from the testis and bilateral orchidectomy may be used for this reason. Pharmacological alternatives include LHRH analogues and anti androgens (which may be given in combination).
In the UK the National Institute for Clinical Excellence (NICE) suggests that active surveillance is the preferred option for low risk men. It is particularly suitable for men with clinical stage T1c, Gleason score 3+3 and PSA density < 0.15 ng/ml/ml who have cancer in less than 50% of their biopsy cores, with < 10 mm of any core involved.
Candidates for active surveillance should:
have had at least 10 biopsy cores taken
have at least one re-biopsy.
If men on active surveillance show evidence of disease progression, offer radical treatment. Treatment decisions should be made with the man, taking into account co-morbidities and life expectancy.

References
1. Prostate cancer pathway. NICE.(http://guidance.nice.org.uk/IPG424)
2. Sooriakumaran P et al. Comparative effectiveness of radical prostatectomy and radiotherapy in prostate cancer: observational study of mortality outcomes. BMJ 2014 (348):13. This study shows that in men with localised disease survival was greater in those offered surgery.

How well did you know this?
1
Not at all
2
3
4
5
Perfectly
767
Q

A 63 year old man has a history of claudication that has been present for many years. He is recently evaluated in the clinic and a duplex scan shows that he has an 85% stenosis of the superficial femoral artery. Two weeks later he presents with a 1 hour history of severe pain in his leg. On examination, he has absent pulses in the affected limb and it is much cooler than the contra-lateral limb. Which process best accounts for this presentation?

Thrombosis

Embolus

Atheroma growth

Sub intimal dissection

Anaemia

A

In an existing lesion a complication such as thrombosis is more likely than embolus. These patients should receive heparin and imaging with duplex scanning. Whilst an early surgical bypass or intra-arterial thrombolysis may be indicated, an embolectomy should not generally be performed as the lesion is not an embolus and the operation therefore ineffective.

Claudication

Claudication is a condition in which patients develop pain in a limb during periods of exercise. The underlying disorder is usually that of arterial insufficiency. Atheroma develops in the arterial wall and once this occludes >50-75% of the lumenal diameter the supply to metabolising tissues distally may become compromised. The typical claudicant complains of calf pain that is worse on exercise and relieved by rest. This typical description assumes that the SFA is the site of disease, more proximal disease may present with other symptoms such as buttock claudication and impotence.

The history is usually a progressive one, patients presenting as an emergency with severe pain, diminished sensation, pallor and absent pulses have critical limb ischaemia. This may complicate claudication and usually indicates a plaque related complication, such as thrombosis.

Risk factors
Risk factors for claudication include smoking, diabetes and hyperlipidaemia.

Diagnosis
Diagnostic work -up includes measurement of ankle- brachial pressure indices, duplex scanning and formal angiography.

Treatment
Those with long claudication distances, no ulceration or gangrene may be managed conservatively. Patients with rest pain, ulceration or gangrene will almost always require intervention. All patients should receive an antiplatelet agent and a statin unless there are compelling contra-indications.

How well did you know this?
1
Not at all
2
3
4
5
Perfectly
768
Q

A 17 year old male is admitted with lower abdominal discomfort. He has been suffering from intermittent right iliac fossa pain for the past few months. His past medical history includes a negative colonoscopy and gastroscopy undertaken as investigation for iron deficiency anaemia. The pain is worse after meals. Inflammatory markers are normal. What is the most likely cause?

Appendicitis

Crohns disease

Peptic ulcer disease

Meckels diverticulum

Irritable bowel syndrome

A

This scenario should raise suspicion for Meckels as these may contain ectopic gastric mucosa which may secrete acid with subsequent bleeding and ulceration. The iron deficiency anaemia makes a Meckels more likely than IBS.

Right iliac fossa pain

Differential diagnosis

Appendicitis
Pain radiating to right iliac fossa
Anorexia (very common)
Short history
Diarrhoea and profuse vomiting rare
Crohn’s disease
Often long history
Signs of malnutrition
Change in bowel habit, especially diarrhoea
Mesenteric adenitis
Mainly affects children
Causes include Adenoviruses, Epstein Barr Virus, beta-haemolytic Streptococcus, Staphylococcus spp., Escherichia coli, Streptococcus viridans and Yersinia spp.
Patients have a higher temperature than those with appendicitis
If laparotomy is performed, enlarged mesenteric lymph nodes will be present
Diverticulitis
Both left and right sided disease may present with right iliac fossa pain
Clinical history may be similar, although some change in bowel habit is usual
When suspected, a CT scan may help in refining the diagnosis
Meckel’s diverticulitis
A Meckel’s diverticulum is a congenital abnormality that is present in about 2% of the population
Typically 2 feet proximal to the ileocaecal valve
May be lined by ectopic gastric mucosal tissue and produce bleeding
Perforated peptic ulcer
This usually produces upper quadrant pain but pain may be lower
Perforations typically have a sharp sudden onset of pain in the history
Incarcerated right inguinal or femoral hernia
Usually only right iliac fossa pain if right sided or bowel obstruction.
Bowel perforation secondary to caecal or colon carcinoma
Seldom localised to right iliac fossa, although complete large bowel obstruction with caecal distension may cause pain prior to perforation.
Gynaecological causes
Pelvic inflammatory disease/salpingitis/pelvic abscess/Ectopic pregnancy/Ovarian torsion/Threatened or complete abortion/Mittelschmerz
Urological causes
Ureteric colic/UTI/Testicular torsion
Other causes
TB/Typhoid/Herpes Zoster/AAA/Situs inversus

How well did you know this?
1
Not at all
2
3
4
5
Perfectly
769
Q

A 25 year old man is found to have carcinoid syndrome. Which of the following hormones is released by carcinoids?

Serotonin

Dopamine

Nor adrenaline

Adrenaline

Aldosterone

A

Rule of thirds:

1/3 multiple
1/3 small bowel
1/3 metastasize
1/3 second tumour
Carcinoids secrete serotonin. Carcinoid syndrome will only occur in the presence of liver metastasis as the hormone released from primary lesions will typically be metabolised by the liver.

Carcinoid syndrome

Carcinoid tumours secrete serotonin
Originate in neuroendocrine cells mainly in the intestine (midgut-distal ileum/appendix)
Can occur in the rectum, bronchi
Hormonal symptoms mainly occur when disease spreads outside the bowel

Clinical features
Onset: insidious over many years
Flushing face
Palpitations
Pulmonary valve stenosis and tricuspid regurgitation causing dyspnoea
Asthma
Severe diarrhoea (secretory, persists despite fasting)

Investigation
5-HIAA in a 24-hour urine collection
Somatostatin receptor scintigraphy
CT scan
Blood testing for chromogranin A

Treatment
Octreotide
Surgical removal

How well did you know this?
1
Not at all
2
3
4
5
Perfectly
770
Q

A 59 year old lady is referred from the NHS breast screening program. A recent mammogram is reported as showing linear, branching microcalcification with coarse granules. Which disease process is the most likely underlying cause of these appearances?

Invasive lobular cancer

Lobular carcinoma in situ

Cribriform type ductal carcinoma in situ

Comedo type ductal carcinoma in situ

Fibroadenosis

A

Comedo type DCIS is usually associated with microcalcifications. Cribriform lesions are usually multifocal but less likely to form microcalcifications. Lobular cancers and in situ lesions rarely form microcalcifications and are difficult to detect using mammography.

Breast cancer - In situ disease

Breast cancer that has yet to invade the basement membrane is referred to as in situ disease. Both ductal and lobular in situ variants are recognised.

Ductal carcinoma in situ
Sub types include; comedo, cribriform, micropapillary and solid
Comdeo DCIS is most likely to form microcalcifications
Cribriform and micropapillary are most likely to be multifocal
Most lesions are mixed (composed of multiple subtypes)
High nuclear grade DCIS is associated with more malignant characteristics (loss of p53, increased erbB2 expression)
Local excision of low nuclear grade DCIS will usually produce satisfactory outcomes.
Multifocal lesions, large and high nuclear grade lesions will usually require mastectomy
Whole breast irradiation improves locoregional control when breast conserving surgery is performed

Lobular carcinoma in situ
Much rarer than DCIS
Does not form microcalcifications
Usually single growth pattern
When an invasive component is found it is less likely to be associated with axillary nodal metastasis than with invasive foci found within DCIS
Low grade LCIS is usually treated by monitoring rather than excision

How well did you know this?
1
Not at all
2
3
4
5
Perfectly
771
Q

Which of the following structures is not closely related to the carotid sheath?

Sternothyroid muscle

Sternohyoid muscle

Hypoglossal nerve

Superior belly of omohyoid muscle

Anterior belly of digastric muscle

A

At its lower end the carotid sheath is related to sternohyoid and sternothyroid. Opposite the cricoid cartilage the sheath is crossed by the superior belly of omohyoid. Above this level the sheath is covered by the sternocleidomastoid muscle. Above the level of the hyoid the vessels pass deep to the posterior belly of digastric and stylohyoid. Opposite the hyoid bone the sheath is crossed obliquely by the hypoglossal nerve.

Common carotid artery

The right common carotid artery arises at the bifurcation of the brachiocephalic trunk, the left common carotid arises from the arch of the aorta. Both terminate at the level of the upper border of the thyroid cartilage (the lower border of the third cervical vertebra) by dividing into the internal and external carotid arteries.

Left common carotid artery
This vessel arises immediately to the left and slightly behind the origin of the brachiocephalic trunk. Its thoracic portion is 2.5- 3.5 cm in length and runs superolaterally to the sternoclavicular joint.

In the thorax
The vessel is in contact, from below upwards, with the trachea, left recurrent laryngeal nerve, left margin of the oesophagus. Anteriorly the left brachiocephalic vein runs across the artery, and the cardiac branches from the left vagus descend in front of it. These structures together with the thymus and the anterior margins of the left lung and pleura separate the artery from the manubrium.

In the neck
The artery runs superiorly deep to sternocleidomastoid and then enters the anterior triangle. At this point it lies within the carotid sheath with the vagus nerve and the internal jugular vein. Posteriorly the sympathetic trunk lies between the vessel and the prevertebral fascia. At the level of C7 the vertebral artery and thoracic duct lie behind it. The anterior tubercle of C6 transverse process is prominent and the artery can be compressed against this structure (it corresponds to the level of the cricoid).
Anteriorly at C6 the omohyoid muscle passes superficial to the artery.
Within the carotid sheath the jugular vein lies lateral to the artery.

Right common carotid artery
The right common carotid arises from the brachiocephalic artery. The right common carotid artery corresponds with the cervical portion of the left common carotid, except that there is no thoracic duct on the right. The oesophagus is less closely related to the right carotid than the left.

Summary points about the carotid anatomy

Path
Passes behind the sternoclavicular joint (12% patients above this level) to the upper border of the thyroid cartilage, to divide into the external (ECA) and internal carotid arteries (ICA).

Relations
Level of 6th cervical vertebra crossed by omohyoid
Then passes deep to the thyrohyoid, sternohyoid, sternomastoid muscles.
Passes anterior to the carotid tubercle (transverse process 6th cervical vertebra)-NB compression here stops haemorrhage.
The inferior thyroid artery passes posterior to the common carotid artery.
Then : Left common carotid artery crosses the thoracic duct, Right common carotid artery crossed by recurrent laryngeal nerve

How well did you know this?
1
Not at all
2
3
4
5
Perfectly
772
Q

A 70 year old man falls and fractures his scaphoid bone. The fracture is displaced and the decision is made to insert a screw to fix the fracture. Which of the following structures lies directly medial to the scaphoid?

Lunate

Pisiform

Trapezoid

Trapezium

None of the above

A

The lunate lies medially in the anatomical plane. Fractures of the scaphoid that are associated with high velocity injuries may cause associated lunate dislocation.

Scaphoid bone

The scaphoid has a concave articular surface for the head of the capitate and at the edge of this is a crescentic surface for the corresponding area on the lunate.
Proximally, it has a wide convex articular surface with the radius. It has a distally sited tubercle that can be palpated. The remaining articular surface is to the lateral side of the tubercle. It faces laterally and is associated with the trapezium and trapezoid bones.

The narrow strip between the radial and trapezial surfaces and the tubercle gives rise to the radial collateral carpal ligament.

Blood supply
This is from the scaphoid tubercle vessels that comprise 20% and the dorsal ridge vessels that supply 80%.

How well did you know this?
1
Not at all
2
3
4
5
Perfectly
773
Q

Which of the following forms the medial wall of the femoral canal?

Pectineal ligament

Adductor longus

Sartorius

Lacunar ligament

Inguinal ligament

A

Lacunar ligament
The femoral canal and the femoral triangle are distinct anatomical structures. Do not confuse them, especially in the time pressured exam situation.

Femoral canal

The femoral canal lies at the medial aspect of the femoral sheath. The femoral sheath is a fascial tunnel containing both the femoral artery laterally and femoral vein medially. The canal lies medial to the vein.

Borders of the femoral canal
Laterally Femoral vein
Medially Lacunar ligament
Anteriorly Inguinal ligament
Posteriorly Pectineal ligament Contents
Lymphatic vessels
Cloquet’s lymph node

Physiological significance
Allows the femoral vein to expand to allow for increased venous return from the lower limbs.

Pathological significance
As a potential space, it is the site of femoral hernias. The relatively tight neck places these at high risk of strangulation.

How well did you know this?
1
Not at all
2
3
4
5
Perfectly
774
Q

An 18 year old male presents to casualty with a depressed skull fracture. This is managed surgically. Over the next few days he complains of double vision on walking down stairs and reading. On testing ocular convergence, the left eye faces downwards and medially, but the right side does not do so. Which of the nerves listed below is most likely to be responsible?

Facial

Oculomotor

Abducens

Trochlear

Trigeminal nerve

A

The trochlear nerve has a relatively long intracranial course and this makes it vulnerable to injury in head trauma. Head trauma is the commonest cause of an acute fourth nerve palsy. A 4th nerve palsy is the commonest cause of a vertical diplopia. The diplopia is at its worst when the eye looks medially which it usually does as part of the accommodation reflex when walking down stairs.

Disorders of the oculomotor system

Nerve Path Nerve palsy features
Oculomotor nerve
Large nucleus at the midbrain
Fibres pass through the red nucleus and the pyramidal tract; through the cavernous sinus into the orbit
Ptosis
Eye down and out
Unable to move the eye superiorly, inferiorly, medially
Pupil fixed and dilated
Trochlear nerve
Longest intracranial course
Only nerve to exit the dorsal aspect of brainstem
Nucleus at midbrain, passes between the posterior cerebral and superior cerebellar arteries, through the cavernous sinus into the orbit
Vertical diplopia (diplopia on descending the stairs)
Unable to look down and in
Abducens nerve Nucleus lies in the mid pons Convergence of eyes in primary position
Lateral diplopia towards side of lesion
Eye deviates medially

How well did you know this?
1
Not at all
2
3
4
5
Perfectly
775
Q

Which of the cranial nerves listed below is least likely to carry parasympathetic fibres?

III

VII

IX

X

II

A

Cranial nerves carrying parasympathetic fibres
X IX VII III (1973)
The parasympathetic functions served by the cranial nerves include:
III (oculomotor) Pupillary constriction and accommodation
VII (facial) Lacrimal gland, submandibular and sublingual glands
IX (glossopharyngeal) Parotid
X (vagus) Heart and abdominal viscera
The optic nerve carries no parasympathetic fibres.

The cranial preganglionic parasympathetic nerves arise from specific nuclei in the CNS. These synapse at one of four parasympathetic ganglia; otic, pterygopalatine, ciliary and submandibular. From these ganglia the parasympathetic nerves complete their journey to their target tissues via CN V (trigeminal) branches (ophthalmic nerve CNV branch 1, Maxillary nerve CN V branch2, mandibular nerve CN V branch 3)

Cranial nerves

Cranial nerve lesions
Olfactory nerve May be injured in basal skull fractures or involved in frontal lobe tumour extension. Loss of olfactory nerve function in relation to major CNS pathology is seldom an isolated event and thus it is poor localiser of CNS pathology.
Optic nerve Problems with visual acuity may result from intra ocular disorders. Problems with the blood supply such as amaurosis fugax may produce temporary visual distortion. More important surgically is the pupillary response to light. The pupillary size may be altered in a number of disorders. Nerves involved in the resizing of the pupil connect to the pretectal nucleus of the high midbrain, bypassing the lateral geniculate nucleus and the primary visual cortex. From the pretectal nucleus neurones pass to the Edinger - Westphal nucleus, motor axons from here pass along with the oculomotor nerve. They synapse with ciliary ganglion neurones; the parasympathetic axons from this then innervate the iris and produce miosis. The miotic pupil is seen in disorders such as Horner’s syndrome or opiate overdose.
Mydriasis is the dilatation of the pupil in response to disease, trauma, drugs (or the dark!). It is pathological when light fails to induce miosis. The radial muscle is innervated by the sympathetic nervous system. Because the parasympathetic fibres travel with the oculomotor nerve they will be damaged by lesions affecting this nerve (e.g. cranial trauma).
The response to light shone in one eye is usually a constriction of both pupils. This indicates intact direct and consensual light reflexes. When the optic nerve has an afferent defect the light shining on the affected eye will produce a diminished pupillary response in both eyes. Whereas light shone on the unaffected eye will produce a normal pupillary response in both eyes. This is referred to as the Marcus Gunn pupil and is seen in conditions such as optic neuritis. In a total CN II lesion shining the light in the affected eye will produce no response.
Oculomotor nerve The pupillary effects are described above. In addition it supplies all ocular muscles apart from lateral rectus and superior oblique. Thus the affected eye will be deviated inferolaterally. Levator palpebrae superioris may also be impaired resulting in impaired ability to open the eye.
Trochlear nerve The eye will not be able to look down.
Trigeminal nerve Largest cranial nerve. Exits the brainstem at the pons. Branches are ophthalmic, maxillary and mandibular. Only the mandibular branch has both sensory and motor fibres. Branches converge to form the trigeminal ganglion (located in Meckels cave). It supplies the muscles of mastication and also tensor veli palatine, mylohyoid, anterior belly of digastric and tensor tympani. The detailed descriptions of the various sensory functions are described in other areas of the website. The corneal reflex is important and is elicited by applying a small tip of cotton wool to the cornea, a reflex blink should occur if it is intact. It is mediated by: the naso ciliary branch of the ophthalmic branch of the trigeminal (sensory component) and the facial nerve producing the motor response. Lesions of the afferent arc will produce bilateral absent blink and lesions of the efferent arc will result in a unilateral absent blink.
Abducens nerve The affected eye will have a deficit of abduction. This cranial nerve exits the brainstem between the pons and medulla. It thus has a relatively long intra cranial course which renders it susceptible to damage in raised intra cranial pressure.
Facial nerve Emerges from brainstem between pons and medulla. It controls muscles of facial expression and taste from the anterior 2/3 of the tongue. The nerve passes into the petrous temporal bone and into the internal auditory meatus. It then passes through the facial canal and exits at the stylomastoid foramen. It passes through the parotid gland and divides at this point. It does not innervate the parotid gland. Its divisions are considered in other parts of the website. Its motor fibres innervate orbicularis oculi to produce the efferent arm of the corneal reflex. In surgical practice it may be injured during parotid gland surgery or invaded by malignancies of the gland and a lower motor neurone on the ipsilateral side will result.
Vestibulo-cochlear nerve Exits from the pons and then passes through the internal auditory meatus. It is implicated in sensorineural hearing loss. Individuals with sensorineural hearing loss will localise the sound in webers test to the normal ear. Rinnes test will be reduced on the affected side but should still work. These two tests will distinguish sensorineural hearing loss from conductive deafness. In the latter condition webers test will localise to the affected ear and Rinnes test will be impaired on the affected side. Surgical lesions affecting this nerve include CNS tumours and basal skull fractures. It may also be damaged by the administration of ototoxic drugs (of which gentamicin is the most commonly used in surgical practice).
Glossopharyngeal nerve Exits the pons just above the vagus. Receives sensory fibres from posterior 1/3 tongue, tonsils, pharynx and middle ear (otalgia may occur following tonsillectomy). It receives visceral afferents from the carotid bodies. It supplies parasympathetic fibres to the parotid gland via the otic ganglion and motor function to stylopharyngeaus muscle. The sensory function of the nerve is tested using the gag reflex.
Vagus nerve Leaves the medulla between the olivary nucleus and the inferior cerebellar peduncle. Passes through the jugular foramen and into the carotid sheath. Details of the functions of the vagus nerve are covered in the website under relevant organ sub headings.
Accessory nerve Exists from the caudal aspect of the brainstem (multiple branches) supplies trapezius and sternocleidomastoid muscles. The distal portion of this nerve is most prone to injury during surgical procedures.
Hypoglossal nerve Emerges from the medulla at the preolivary sulcus, passes through the hypoglossal canal. It lies on the carotid sheath and passes deep to the posterior belly of digastric to supply muscles of the tongue (except palatoglossus). Its location near the carotid sheath makes it vulnerable during carotid endarterectomy surgery and damage will produce ipsilateral defect in muscle function.

How well did you know this?
1
Not at all
2
3
4
5
Perfectly
776
Q

A 28 year old African man is admitted with acute severe abdominal pain. He has just flown into the UK long haul and the pain developed whilst in flight. On examination he is tender in the left upper quadrant. His blood tests are as shown.

Hb 6 g/dl
Reticulocyte count 15%.
Ultrasound shows a spleen with a heterogenous texture. The gallbladder shows a few small gallstones but is otherwise normal.
What is the most likely diagnosis?

Pancreatitis

Parvovirus infection

Sickle cell anaemia

Pulmonary embolism

Beta Thalassaemia minor

A

A combination of a high reticulocyte count and severe anaemia indicates sickle cell anaemia, however another differential can be of a transient aplastic crisis due to parvovirus. This is less likely as this causes a reticulocytopenia rather than a reticulocytosis.

Parvovirus B19 infects erythroid progenitor cells in the bone marrow and causes temporary cessation of red blood cell production, patients who have underlying hematologic abnormalities are at risk of cessation of red blood cell production if they become infected. This can result in a transient aplastic crisis. Thus, patients with sickle cell anaemia are at risk. Typically, these patients have a viral prodrome followed by anaemia, often with haemoglobin concentrations falling below 5.0 g/dL and reticulocytopenia.

Sickle cell anaemia

Autosomal recessive
Single base mutation
Deoxygenated cells become sickle in shape
Causes: short red cell survival, obstruction of microvessels and infarction
Sickling is precipitated by: dehydration, infection, hypoxia
Manifest at 6 months age
Africans, Middle East, Indian
Diagnosis: Hb electrophoresis

Sickle crises

Bone pain
Pleuritic chest pain: acute sickle chest syndrome commonest cause of death
CVA, seizures
Papillary necrosis
Splenic infarcts
Priapism
Hepatic pain

Hb does not fall during a crisis, unless there is

Aplasia: parvovirus
Acute sequestration
Haemolysis

Long-term complications

Infections: Streptococcus pnemoniae
Chronic leg ulcers
Gallstones: haemolysis
Aseptic necrosis of bone
Chronic renal disease
Retinal detachment, proliferative retinopathy

Surgical complications

Bowel ischaemia
Cholecystitis
Avascular necrosis

Management

Supportive
Hydroxyurea
Repeated transfusions pre operatively
Exchange transfusion in emergencies

Sickle cell trait

Heterozygous state
Asymptomatic
Symptoms associated with extreme situations ie anaesthesia complications
Protective against Plasmodium falciparum

How well did you know this?
1
Not at all
2
3
4
5
Perfectly
777
Q

Which statement is false about pethidine?

It is thirty times more lipid soluble than morphine

Structurally similar to morphine

Pethidine has a toxic metabolite (norpethidine) which is cleared by the kidney

Pethidine is metabolized by the liver

Can be given intramuscularly

A

It has a different structure. It is much more lipid soluble than morphine. It produces less biliary tract spasm than morphine.

Management of pain

World Health Organisation Analgesic Ladder
Initially peripherally acting drugs such as paracetamol or non-steroidal anti-inflammatory drugs (NSAIDs) are given.
If pain control is not achieved, the second part of the ladder is to introduce weak opioid drugs such as codeine or dextropropoxyphene together with appropriate agents to control and minimise side effects.
The final rung of the ladder is to introduce strong opioid drugs such as morphine. Analgesia from peripherally acting drugs may be additive to that from centrally-acting opioids and thus, the two are given together.

The World Federation of Societies of Anaesthesiologists (WFSA) Analgesic Ladder
For management of acute pain
Initially, the pain can be expected to be severe and may need controlling with strong analgesics in combination with local anaesthetic blocks and peripherally acting drugs.
The second rung on the postoperative pain ladder is the restoration of the use of the oral route to deliver analgesia. Strong opioids may no longer be required and adequate analgesia can be obtained by using combinations of peripherally acting agents and weak opioids.
The final step is when the pain can be controlled by peripherally acting agents alone.

Local anaesthetics
Infiltration of a wound with a long-acting local anaesthetic such as Bupivacaine
Analgesia for several hours
Further pain relief can be obtained with repeat injections or by infusions via a thin catheter
Blockade of plexuses or peripheral nerves will provide selective analgesia in those parts of the body supplied by the plexus or nerves
Can either be used to provide anaesthesia for the surgery or specifically for postoperative pain relief
Especially useful where a sympathetic block is needed to improve postoperative blood supply or where central blockade such as spinal or epidural blockade is contraindicated.

Spinal anaesthesia
Provides excellent analgesia for surgery in the lower half of the body and pain relief can last many hours after completion of the operation if long-acting drugs containing vasoconstrictors are used.

  • Side effects of spinal anaesthesia include: hypotension, sensory and motor block, nausea and urinary retention.

Epidural anaesthesia
An indwelling epidural catheter inserted. This can then be used to provide a continuous infusion of analgesic agents. It can provide excellent analgesia. They are still the preferred option following major open abdominal procedures and help prevent post operative respiratory compromise resulting from pain.

  • Disadvantages of epidurals is that they usually confine patients to bed, especially if a motor block is present. In addition an indwelling urinary catheter is required. Which may not only impair mobility but also serve as a conduit for infection. They are contraindicated in coagulopathies.

Transversus Abdominal Plane block (TAP)
In this technique an ultrasound is used to identify the correct muscle plane and local anaesthetic (usually bupivicaine) is injected. The agent diffuses in the plane and blocks many of the spinal nerves. It is an attractive technique as it provides a wide field of blockade but does not require the placement of any indwelling devices. There is no post operative motor impairment. For this reason it is the preferred technique when extensive laparoscopic abdominal procedures are performed. They will then provide analgesia immediately following surgery but as they do not confine the patient to bed, the focus on enhanced recovery can begin sooner.

-The main disadvantage is that their duration of action is limited to the half life of the local anaesthetic agent chosen. In addition some anaesthetists do not have the USS skills required to site the injections.

Patient Controlled Analgesia (PCA)

  • Patients administer their own intravenous analgesia and titrate the dose to their own end-point of pain relief using a small microprocessor - controlled pump. Morphine is the most popular drug used.

Strong Opioids

Severe pain arising from deep or visceral structures requires the use of strong opioids

Morphine
Short half life and poor bioavailability.
Metabolised in the liver and clearance is reduced in patients with liver disease, in the elderly and the debilitated
Side effects include nausea, vomiting, constipation and respiratory depression.
Tolerance may occur with repeated dosage

Pethidine
Synthetic opioid which is structurally different from morphine but which has similar actions. Has 10% potency of morphine.
Short half life and similar bioavailability and clearance to morphine.
Short duration of action and may need to be given hourly.
Pethidine has a toxic metabolite (norpethidine) which is cleared by the kidney, but which accumulates in renal failure or following frequent and prolonged doses and may lead to muscle twitching and convulsions. Extreme caution is advised if pethidine is used over a prolonged period or in patients with renal failure.

Weak opioids
Codeine: markedly less active than morphine, has predictable effects when given orally and is effective against mild to moderate pain.

Non opioid analgesics
- Mild to moderate pain.

Paracetamol
Inhibits prostaglandin synthesis.
Analgesic and antipyretic properties but little anti-inflammatory effect
It is well absorbed orally and is metabolised almost entirely in the liver
Side effects in normal dosage and is widely used for the treatment of minor pain. It causes hepatotoxicity in over dosage by overloading the normal metabolic pathways with the formation of a toxic metabolite.

NSAIDs
Analgesic and anti-inflammatory actions
Inhibition of prostaglandin synthesis by the enzyme Cyclooxygenase which catalyses the conversion of arachidonic acid to the various prostaglandins that are the chief mediators of inflammation. All NSAIDs work in the same way and thus there is no point in giving more than one at a time. .
NSAIDs are, in general, more useful for superficial pain arising from the skin, buccal mucosa, joint surfaces and bone.
Relative contraindications: history of peptic ulceration, gastrointestinal bleeding or bleeding diathesis; operations associated with high blood loss, asthma, moderate to severe renal impairment, dehydration and any history of hypersensitivity to NSAIDs or aspirin.

Neuropathic pain
National Institute of Clinical Excellence (UK) guidelines:
First line: Amitriptyline (Imipramine if cannot tolerate) or pregabalin
Second line: Amitriptyline AND pregabalin
Third line: refer to pain specialist. Give tramadol in the interim (avoid morphine)
If diabetic neuropathic pain: Duloxetine
As of 1 April 2019, pregabalin and gabapentin are Class C controlled substances (under the Misuse of Drugs Act 1971) and scheduled under the Misuse of Drugs Regulations 2001 as Schedule 3. Evaluate patients carefully for a history of drug abuse before prescribing and observe patients for development of signs of abuse and dependence (MHRA, Drug Safety Update April 2019).

References
1. http://guidance.nice.org.uk/CG173/Guidance/pdf/English
2. Lovich-Sapola J, Smith CE, Brandt CP. Post operative pain control. Surg Clin North Am. 2015 Apr;95(2):301-183. Finnerup N et al. Pharmacotherapy for neuropathic pain in adults: a systematic review and meta-analysis. Lancet Neurol. 2015 Feb;14(2):162-73.

How well did you know this?
1
Not at all
2
3
4
5
Perfectly
778
Q

A 73 year old lady is hit by a car. She suffers a complex fracture of the distal aspect of her humerus with associated injury to the radial nerve. Which of the following movements will be most impaired as a result?

Elbow extension

Elbow flexion

Shoulder abduction

Wrist extension

None of the above

A

The triceps will not be affected so elbow extension will be preserved. Loss of wrist extension will be the most obvious effect.

Radial nerve

Continuation of posterior cord of the brachial plexus (root values C5 to T1)

Path
In the axilla: lies posterior to the axillary artery on subscapularis, latissimus dorsi and teres major.
Enters the arm between the brachial artery and the long head of triceps (medial to humerus).
Spirals around the posterior surface of the humerus in the groove for the radial nerve.
At the distal third of the lateral border of the humerus it then pierces the intermuscular septum and descends in front of the lateral epicondyle.
At the lateral epicondyle it lies deeply between brachialis and brachioradialis where it then divides into a superficial and deep terminal branch.
Deep branch crosses the supinator to become the posterior interosseous nerve.

Regions innervated
Motor (main nerve)
Triceps
Anconeus
Brachioradialis
Extensor carpi radialis
Motor (posterior interosseous branch)
Supinator
Extensor carpi ulnaris
Extensor digitorum
Extensor indicis
Extensor digiti minimi
Extensor pollicis longus and brevis
Abductor pollicis longus
Sensory The area of skin supplying the proximal phalanges on the dorsal aspect of the hand is supplied by the radial nerve (this does not apply to the little finger and part of the ring finger)

Muscular innervation and effect of denervation
Anatomical location Muscle affected Effect of paralysis
Shoulder Long head of triceps Minor effects on shoulder stability in abduction
Arm Triceps Loss of elbow extension
Forearm Supinator
Brachioradialis
Extensor carpi radialis longus and brevis Weakening of supination of prone hand and elbow flexion in mid prone position

How well did you know this?
1
Not at all
2
3
4
5
Perfectly
779
Q

A 55 year old motorcyclist is involved in a road traffic accident and sustained a Gustilo and Anderson IIIc type fracture to the distal tibia. He was trapped in the wreckage for 7 hours during which time he bled profusely from the fracture site. He has an established distal neurovascular deficit. What is the most appropriate course of action?

Amputation

Skeletal traction

Application of external fixator and arterial reconstruction

Insertion of intramedullary nail and arterial reconstruction

Application of plate to tibia and arterial reconstruction

A

This man is unstable, and at 7 hours after extraction, the limb is not viable. The safest option is primary amputation.

Fracture management

Bony injury resulting in a fracture may arise from trauma (excessive forces applied to bone), stress related (repetitive low velocity injury) or pathological (abnormal bone which fractures during normal use of following minimal trauma)
Diagnosis involves not just evaluating the fracture ; such as site and type of injury but also other associated injuries and distal neurovascular deficits. This may entail not just clinical examination but radiographs of proximal and distal joints.
When assessing x-rays it is important to assess for changes in length of the bone, the angulation of the distal bone, rotational effects, presence of material such as glass.

Fracture types
Fracture type Description
Oblique fracture Fracture lies obliquely to long axis of bone
Comminuted fracture >2 fragments
Segmental fracture More than one fracture along a bone
Transverse fracture Perpendicular to long axis of bone
Spiral fracture Severe oblique fracture with rotation along long axis of bone

Open Vs Closed
It is also important to distinguish open from closed injuries. The most common classification system for open fractures is the Gustilo and Anderson classification system (given below):

Grade Injury
1 Low energy wound <1cm
2 Greater than 1cm wound with moderate soft tissue damage
3 High energy wound > 10cm with extensive soft tissue damage
3 A (sub group of 3) Adequate soft tissue coverage
3 B (sub group of 3) Inadequate soft tissue coverage
3 C (sub group of 3) Associated arterial injury

Key points in management of fractures
Immobilise the fracture including the proximal and distal joints
Carefully monitor and document neurovascular status, particularly following reduction and immobilisation
Manage infection including tetanus prophylaxis
IV broad spectrum antibiotics for open injuries
As a general principle all open fractures should be thoroughly debrided ( and internal fixation devices avoided or used with extreme caution)
Open fractures constitute an emergency and should be debrided and lavaged within 6 hours of injury

How well did you know this?
1
Not at all
2
3
4
5
Perfectly
780
Q

Which muscle is responsible for causing flexion of the interphalangeal joint of the thumb?

Flexor pollicis longus

Flexor pollicis brevis

Flexor digitorum superficialis

Flexor digitorum profundus

Adductor pollicis

A

There are 8 muscles:
1. Two flexors (flexor pollicis brevis and flexor pollicis longus)
2. Two extensors (extensor pollicis brevis and longus)
3. Two abductors (abductor pollicis brevis and longus)
4. One adductor (adductor pollicis)
5. One muscle that opposes the thumb by rotating the CMC joint (opponens pollicis).

Flexor and extensor longus insert on the distal phalanx moving both the MCP and IP joints.

Hand

Anatomy of the hand
Bones
8 Carpal bones
5 Metacarpals
14 phalanges
Intrinsic Muscles 8 Interossei - Supplied by ulnar nerve
4 palmar-adduct fingers
4 dorsal- abduct fingers
Intrinsic muscles Lumbricals
Flex MCPJ and extend the IPJ.
Origin deep flexor tendon and insertion dorsal extensor hood mechanism.
Innervation: 1st and 2nd- median nerve, 3rd and 4th- deep branch of the ulnar nerve.
Thenar eminence
Abductor pollicis brevis
Opponens pollicis
Flexor pollicis brevis
Hypothenar eminence
Opponens digiti minimi
Flexor digiti minimi brevis
Abductor digiti minimi

Image sourced from Wikipedia

Fascia and compartments of the palm
The fascia of the palm is continuous with the antebrachial fascia and the fascia of the dorsum of the hand. The palmar fascia is thin over the thenar and hypothenar eminences. In contrast, the central palmar fascia is relatively thick. The palmar aponeurosis covers the soft tissues and overlies the flexor tendons. The apex of the palmar aponeurosis is continuous with the flexor retinaculum and the palmaris longus tendon. Distally, it forms four longitudinal digital bands that attach to the bases of the proximal phalanges, blending with the fibrous digital sheaths.
A medial fibrous septum extends deeply from the medial border of the palmar aponeurosis to the 5th metacarpal. Lying medial to this are the hypothenar muscles. In a similar fashion, a lateral fibrous septum extends deeply from the lateral border of the palmar aponeurosis to the 3rd metacarpal. The thenar compartment lies lateral to this area.
Lying between the thenar and hypothenar compartments is the central compartment. It contains the flexor tendons and their sheaths, the lumbricals, the superficial palmar arterial arch and the digital vessels and nerves.
The deepest muscular plane is the adductor compartment, which contains adductor pollicis.

Short muscles of the hand
These comprise the lumbricals and interossei. The four slender lumbrical muscles flex the fingers at the metacarpophalangeal joints and extend the interphalangeal joint. The four dorsal interossei are located between the metacarpals and the four palmar interossei lie on the palmar surface of the metacarpals in the interosseous compartment of the hand.

Long flexor tendons and sheaths in the hand
The tendons of FDS and FDP enter the common flexor sheath deep to the flexor retinaculum. The tendons enter the central compartment of the hand and fan out to their respective digital synovial sheaths. Near the base of the proximal phalanx, the tendon of FDS splits to permit the passage of FDP. The FDP tendons are attached to the margins of the anterior aspect of the base of the distal phalanx.
The fibrous digital sheaths contain the flexor tendons and their synovial sheaths. These extend from the heads of the metacarpals to the base of the distal phalanges.

Palmar Interossei
Note that there are 4 palmar interossei. The first is a small slip of muscle which arises from the ulnar side of the base of the first metacarpal and passes between the head of the first dorsal interosseous and the oblique head of adductor pollicis to insert into the ulnar base of the of the proximal phalanx of the thumb. The second arises from the ulnar side of the body of the second metacarpal and is inserted into the ulnar side of the extensor hood of the index. The third and fourth palmar interossei arise from the radial sides of the bodies of the 4th and 5th metacarpals respectively and insert into the radial sides of the extensor hoods of the ring and little fingers.

How well did you know this?
1
Not at all
2
3
4
5
Perfectly
781
Q

The pudendal canal is a fascial canal located on the lateral wall of the ischioanal fossa. In this location, it lies on the inferior border of which of the following muscles?

Coccygeus

Obturator internus

Pubococcygeus

Iliococcygeus

Piriformis

A

The coccygeus, pubococcygeus and iliococcygeus form part of the pelvic diaphragm and are not related to it. The piriformis exits the pelvis via the greater sciatic foramen and is not associated with the canal in the ischiorectal fossa.

Pudendal canal

The pudendal canal is located along the lateral wall of the ischioanal fossa at the inferior margin of the obturator internus muscle. It extends from the lesser sciatic foramen to the posterior margin of the urogenital diaphragm. It conveys the internal pudendal vessels and nerve.

How well did you know this?
1
Not at all
2
3
4
5
Perfectly
782
Q

Which of the muscles below does not cause lateral rotation of the hip?

Obturator internus

Quadratus femoris

Gemellus inferior

Piriformis

Pectineus

A

Mnemonic lateral hip rotators: P-GO-GO-Q (top to bottom)

Piriformis
Gemellus superior
Obturator internus
Gemellus inferior
Obturator externus
Quadratus femoris
Pectineus adducts and medially rotates the femur.

Hip joint

Head of femur articulates with acetabulum of the pelvis
Both covered by articular hyaline cartilage
The acetabulum forms at the union of the ilium, pubis, and ischium
The triradiate cartilage (Y-shaped growth plate) separates the pelvic bones
The acetabulum holds the femoral head by the acetabular labrum
Normal angle between femoral head and femoral shaft is 130o

Ligaments
Transverse ligament: joints anterior and posterior ends of the articular cartilage
Head of femur ligament (ligamentum teres): acetabular notch to the fovea. Contains arterial supply to head of femur in children.

Extracapsular ligaments
Iliofemoral ligament: inverted Y shape. Anterior iliac spine to the trochanteric line
Pubofemoral ligament: acetabulum to lesser trochanter
Ischiofemoral ligament: posterior support. Ischium to greater trochanter.

Blood supply
Medial circumflex femoral and lateral circumflex femoral arteries (Branches of profunda femoris). Also from the inferior gluteal artery. These form an anastomosis and travel to up the femoral neck to supply the head.

How well did you know this?
1
Not at all
2
3
4
5
Perfectly
783
Q

A 58 year old woman is admitted with an attack of severe acute pancreatitis. She is managed on the intensive care unit and is making progress. She then deteriorates and a CT scan shows extensive pancreatic necrosis (>40%). There are concerns that this may have become infected. What is the correct course of action?

Undertake a fine needle aspiration of the area

Perform a pancreatic necrosectomy

Perform a Whipples procedure

Arrange an ERCP

Perform a distal pancreatectomy

A

When there are concerns that pancreatic necrosis may have become infected the usual approach is to perform an image guided FNA for culture. There is always the risk of seeding infection with such a strategy so it must be performed with care. Pancreatic necrosectomy is not usually undertaken until the presence of infection is proven.

Management of Pancreatitis

Management of Acute Pancreatitis in the UK

Diagnosis
Traditionally hyperamylasaemia has been utilised with amylase being elevated three times the normal range.
However, amylase may give both false positive and negative results.
Serum lipase is both more sensitive and specific than serum amylase. It also has a longer half life.
Serum amylase levels do not correlate with disease severity.

Differential causes of hyperamylasaemia
Acute pancreatitis
Pancreatic pseudocyst
Mesenteric infarct
Perforated viscus
Acute cholecystitis
Diabetic ketoacidosis

Assessment of severity
Glasgow, Ranson scoring systems and APACHE II
Biochemical scoring e.g. using CRP

Features that may predict a severe attack within 48 hours of admission to hospital
Initial assessment
Clinical impression of severity
Body mass index >30
Pleural effusion
APACHE score >8
24 hours after admission
Clinical impression of severity
APACHE II >8
Glasgow score of 3 or more
Persisting multiple organ failure
CRP>150
48 hours after admission
Glasgow Score of >3
CRP >150
Persisting or progressive organ failure
Table adapted from UK guidelines for management of acute pancreatitis. GUT 2005, 54 suppl III

Management

Nutrition
There is reasonable evidence to suggest that the use of enteral nutrition does not worsen the outcome in pancreatitis
Most trials to date were underpowered to demonstrate a conclusive benefit.
The rationale behind feeding is that it helps to prevent bacterial translocation from the gut, thereby contributing to the development of infected pancreatic necrosis.

Use of antibiotic therapy
Many UK surgeons administer antibiotics to patients with acute pancreatitis. However, there is very little evidence to support this practice.
A recent Cochrane review highlights the potential benefits of administering Imipenem to patients with established pancreatic necrosis in the hope of averting the progression to infection.
There are concerns that the administration of antibiotics in mild attacks of pancreatitis will not affect outcome and may contribute to antibiotic resistance and increase the risks of antibiotic associated diarrhoea.

Surgery
Patients with acute pancreatitis due to gallstones should undergo early cholecystectomy.
Patients with obstructed biliary system due to stones should undergo early ERCP.
Patients with extensive necrosis where infection is suspected should usually undergo FNA for culture.
Patients with infected necrosis should undergo either radiological drainage or surgical necrosectomy. The choice of procedure depends upon local expertise.

References
www.bsg.org.uk/pdfworddocs/pancreatic.pdf

Antibiotic therapy for prophylaxis against infection of pancreatic necrosis in acute pancreatitis. Villatoro et al. Cochrane Library DOI: 10.1002/14651858.CD002941.pub3. 2010 version.

How well did you know this?
1
Not at all
2
3
4
5
Perfectly
784
Q

Which of the nerves below innervates the tensor tympani muscle?

Vestibulocochlear

Facial

Vagus

Trigeminal

Hypoglossal

A

The tensor tympani muscle (in conjunction with stapedius) helps to mitigate the effects of loud sounds. The tensor tympani is innervated by the trigeminal nerve (the stapedius by the facial). In some people with hyperacousia, the tensor tympani muscle does not function normally.
Trigeminal nerve

The trigeminal nerve is the main sensory nerve of the head. In addition to its major sensory role, it also innervates the muscles of mastication.

Distribution of the trigeminal nerve
Sensory
Scalp
Face
Oral cavity (and teeth)
Nose and sinuses
Dura mater
Motor
Muscles of mastication
Mylohyoid
Anterior belly of digastric
Tensor tympani
Tensor palati
Autonomic connections (ganglia)
Ciliary
Sphenopalatine
Otic
Submandibular

Path
Originates at the pons
Sensory root forms the large, crescentic trigeminal ganglion within Meckel’s cave, and contains the cell bodies of incoming sensory nerve fibres. Here the 3 branches exit.
The motor root cell bodies are in the pons and the motor fibres are distributed via the mandibular nerve. The motor root is not part of the trigeminal ganglion.

Branches of the trigeminal nerve
Ophthalmic nerve Sensory only
Maxillary nerve Sensory only
Mandibular nerve Sensory and motor

Sensory
Ophthalmic Exits skull via the superior orbital fissure
Sensation of: scalp and forehead, the upper eyelid, the conjunctiva and cornea of the eye, the nose (including the tip of the nose, except alae nasi), the nasal mucosa, the frontal sinuses, and parts of the meninges (the dura and blood vessels).
Maxillary nerve Exit skull via the foramen rotundum
Sensation: lower eyelid and cheek, the nares and upper lip, the upper teeth and gums, the nasal mucosa, the palate and roof of the pharynx, the maxillary, ethmoid and sphenoid sinuses, and parts of the meninges.
Mandibular nerve Exit skull via the foramen ovale
Sensation: lower lip, the lower teeth and gums, the chin and jaw (except the angle of the jaw), parts of the external ear, and parts of the meninges.

Motor
Distributed via the mandibular nerve.
The following muscles of mastication are innervated:
Masseter
Temporalis
Medial pterygoid
Lateral pterygoid

Other muscles innervated include:
Tensor veli palatini
Mylohyoid
Anterior belly of digastric
Tensor tympani

How well did you know this?
1
Not at all
2
3
4
5
Perfectly
785
Q

During a radical neck dissection, division of which of the following fascial layers will expose the ansa cervicalis?

Pretracheal fascia

Carotid sheath

Prevertebral fascia

Investing layer of fascia

Sibsons fascia

A

The ansa cervicalis lies anterior to the carotid artery and on the carotid sheath. It may be exposed by division of the pretracheal fascia at the posterolateral aspect of the thyroid gland. The pre vertebral fascia lies more posteriorly and division of the investing layer of fascia will not expose this nerve.

Ansa cervicalis

Superior root Branch of C1 anterolateral to carotid sheath
Inferior root Derived from C2 and C3 roots, passes posterolateral to the internal jugular vein (may lie either deep or superficial to it)
Innervation Sternohyoid
Sternothyroid
Omohyoid

The ansa cervicalis lies anterior to the carotid sheath. The nerve supply to the inferior strap muscles enters at their inferior aspect. Therefore when dividing these muscles to expose a large goitre, the muscles should be divided in their upper half.

How well did you know this?
1
Not at all
2
3
4
5
Perfectly
786
Q

A 49 year old man undergoes a distal gastrectomy to treat peptic ulcer disease. Whilst he is recovering from surgery on the ward, which of the effects listed below is most likely to be encountered?

Decreased cardiac output

Bradycardia

Relaxation of the smooth muscle of the GI tract

Vasodilatation

Reduction in angiotensin II levels

A

The stress response to surgery is multimodal and encompasses a number of physiological responses. However, cardiac output is usually increased (sympathetic nervous system induces hypertension and tachycardia). Its also results in GI tract smooth muscle relaxation. The renin angiotensin system is activated resulting in vasoconstriction and increased angiotensin II levels.

Stress response: Endocrine and metabolic changes

  • Surgery precipitates hormonal and metabolic changes causing the stress response.
    Stress response is associated with: substrate mobilization, muscle protein loss, sodium and water retention, suppression of anabolic hormone secretion, activation of the sympathetic nervous system, immunological and haematological changes.
    The hypothalamic-pituitary axis and the sympathetic nervous systems are activated and there is a failure of the normal feedback mechanisms of control of hormone secretion.

A summary of the hormonal changes associated with the stress response:

Increased Decreased No Change
Growth hormone Insulin Thyroid stimulating hormone
Cortisol Testosterone Luteinizing hormone
Renin Oestrogen Follicle stimulating hormone
Adrenocorticotrophic hormone (ACTH)
Aldosterone
Prolactin
Antidiuretic hormone
Glucagon

Sympathetic nervous system
Stimulates catecholamine release
Causes tachycardia and hypertension

Pituitary gland
ACTH and growth hormone (GH) is stimulated by hypothalamic releasing factors, corticotrophin releasing factor (CRF) and somatotrophin (or growth hormone releasing factor)
Perioperative increased prolactin secretion occurs by release of inhibitory control
Secretion of thyroid stimulating hormone (TSH), luteinizing hormone (LH) and follicle stimulating hormone (FSH) does not change significantly
ACTH stimulates cortisol production within a few minutes of the start of surgery. More ACTH is produced than needed to produce a maximum adrenocortical response.

Cortisol
Significant increases within 4-6 hours of surgery (>1000 nmol litre-1).
The usual negative feedback mechanism fails and concentrations of ACTH and cortisol remain persistently increased.
The magnitude and duration of the increase correlate with the severity of stress and the response is not abolished by the administration of corticosteroids.
The metabolic effects of cortisol are enhanced:

Skeletal muscle protein breakdown to provide gluconeogenic precursors and amino acids for protein synthesis in the liver
Stimulation of lipolysis
‘Anti-insulin effect’
Mineralocorticoid effects
Anti-inflammatory effects

Growth hormone
Increased secretion after surgery has a minor role
Most important for preventing muscle protein breakdown and promote tissue repair by insulin growth factors

Alpha Endorphin
Increased

Antidiuretic hormone
An important vasopressor and enhances haemostasis
Renin is released causing the conversion of angiotensinogen to angiotensin I
Angiotensin II formed by ACE on angiotensin 1, which causes the secretion of aldosterone from the adrenal cortex. This increases sodium reabsorption at the distal convoluted tubule

Insulin
Release inhibited by stress
Occurs via the inhibition of the beta cells in the pancreas by the α2-adrenergic inhibitory effects of catecholamines
Insulin resistance by target cells occurs later
The perioperative period is characterized by a state of functional insulin deficiency

Thyroxine (T4) and tri-iodothyronine (T3)
Circulating concentrations are inversely correlated with sympathetic activity and after surgery there is a reduction in thyroid hormone production, which normalises over a few days.

Metabolic effect of endocrine response

Carbohydrate metabolism
Hyperglycaemia is a main feature of the metabolic response to surgery
Due to increase in glucose production and a reduction in glucose utilization
Catecholamines and cortisol promote glycogenolysis and gluconeogenesis
Initial failure of insulin secretion followed by insulin resistance affects the normal responses
The proportion of the hyperglycaemic response reflects the severity of surgery
Hyperglycaemia impairs wound healing and increase infection rates

Protein metabolism
Initially there is inhibition of protein anabolism, followed later, if the stress response is severe, by enhanced catabolism
The amount of protein degradation is influenced by the type of surgery and also by the nutritional status of the patient
Mainly skeletal muscle protein is affected
The amino acids released form acute phase proteins (fibrinogen, C reactive protein, complement proteins, a2-macroglobulin, amyloid A and ceruloplasmin) and are used for gluconeogenesis
Nutritional support has little effect on preventing catabolism

Lipid metabolism
Increased catecholamine, cortisol and glucagon secretion, and insulin deficiency, promotes lipolysis and ketone body production.

Salt and water metabolism
ADH causes water retention, concentrated urine, and potassium loss and may continue for 3 to 5 days after surgery
Renin causes sodium and water retention

Cytokines
Glycoproteins
Interleukins (IL) 1 to 17, interferons, and tumour necrosis factor
Synthesized by activated macrophages, fibroblasts, endothelial and glial cells in response to tissue injury from surgery or trauma
IL-6 main cytokine associated with surgery. Peak 12 to 24 h after surgery and increase by the degree of tissue damage Other effects of cytokines include fever, granulocytosis, haemostasis, tissue damage limitation and promotion of healing.

Modifying the response
Opioids suppress hypothalamic and pituitary hormone secretion
At high doses the hormonal response to pelvic and abdominal surgery is abolished. However, such doses prolong recovery and increase the need for postoperative ventilatory support
Spinal anaesthesia can reduce the glucose, ACTH, cortisol, GH and epinephrine changes, although cytokine responses are unaltered
Cytokine release is reduced in less invasive surgery
Nutrition prevents the adverse effects of the stress response. Enteral feeding improves recovery
Growth hormone and anabolic steroids may improve outcome
Normothermia decreases the metabolic response

References
Vasiliki Manou-Stathopoulou et al. Redefining the perioperative stress response: a narrative review. Br J Anaesth. 2019 Nov;123(5):570-583.

Deborah Burton, Grainne Nicholson, and George Hall
Endocrine and metabolic response to surgery .

Contin Educ Anaesth Crit Care Pain (2004) 4(5): 144-147 doi:10.1093/bjaceaccp/mkh040

How well did you know this?
1
Not at all
2
3
4
5
Perfectly
787
Q

Which of the following are not generally supplied by the right coronary artery?

The sino atrial node

The circumflex artery

The atrioventricular node

Most of the right ventricle

The right atrium

A

The circumflex artery is generally a branch of the left coronary artery.

Heart anatomy

The walls of each cardiac chamber comprise:
Epicardium
Myocardium
Endocardium

Cardiac muscle is attached to the cardiac fibrous skeleton.

Relations
The heart and roots of the great vessels within the pericardial sac are related to the posterior aspect of the sternum, medial ends of the 3rd to 5th ribs on the left and their associated costal cartilages. The heart and pericardial sac are situated obliquely two thirds to the left and one third to the right of the median plane.

The pulmonary valve lies at the level of the left third costal cartilage.
The mitral valve lies at the level of the fourth costal cartilage.

Coronary sinus
This lies in the posterior part of the coronary groove and receives blood from the cardiac veins. The great cardiac vein lies at its left and the middle and small cardiac veins lie on its right. The smallest cardiac vein (anterior cardiac vein) drains into the right atrium directly.

Aortic sinus
Right coronary artery arises from the right aortic sinus, the left is derived from the left aortic sinus, which lies posteriorly.

Features of the left ventricle as opposed to the right

Structure Left Ventricle
A-V Valve Mitral (double leaflet)
Walls Twice as thick as right
Trabeculae carnae Much thicker and more numerous

Right coronary artery
The RCA supplies:
Right atrium
Diaphragmatic part of the right ventricle
Usually the posterior third of the interventricular septum
The sino atrial node (60% cases)
The atrio ventricular node (80% cases)

Left coronary artery
The LCA supplies:
Left atrium
Most of left ventricle
Part of the right ventricle
Anterior two thirds of the inter ventricular septum
The sino atrial node (remaining 40% cases)

Innervation of the heart
Autonomic nerve fibres from the superficial and deep cardiac plexus. These lie anterior to the bifurcation of the trachea, posterior to the ascending aorta and superior to the bifurcation of the pulmonary trunk. The parasympathetic supply to the heart is from presynaptic fibres of the vagus nerves.

Valves of the heart
Mitral valve Aortic valve Pulmonary valve Tricuspid valve
2 cusps 3 cusps 3 cusps 3 cusps
First heart sound Second heart sound Second heart sound First heart sound
1 anterior cusp 2 anterior cusps 2 anterior cusps 2 anterior cusps
Attached to chordae tendinae No chordae No chordae Attached to chordae tendinae

How well did you know this?
1
Not at all
2
3
4
5
Perfectly
788
Q

What is the nerve root value of the external urethral sphincter?

S4

S1, S2, S3

S2, S3, S4

L3, L4, L5

L5, S1, S2

A

The external urethral sphincter is innervated by branches of the pudendal nerve, therefore the root values are S2, S3, S4.

Urethral anatomy

Female urethra
The female urethra is shorter and more acutely angulated than the male urethra. It is an extra-peritoneal structure and embedded in the endopelvic fascia. The neck of the bladder is subjected to transmitted intra-abdominal pressure and therefore deficiency in this area may result in stress urinary incontinence. Between the layers of the urogenital diaphragm the female urethra is surrounded by the external urethral sphincter, this is innervated by the pudendal nerve. It ultimately lies anterior to the vaginal orifice.

Male urethra
In males the urethra is much longer and is divided into four parts.

Pre-prostatic urethra Extremely short and lies between the bladder and prostate gland.It has a stellate lumen and is between 1 and 1.5cm long.Innervated by sympathetic noradrenergic fibres, as this region is composed of striated muscles bundles they may contract and prevent retrograde ejaculation.
Prostatic urethra This segment is wider than the membranous urethra and contains several openings for the transmission of semen (at the midpoint of the urethral crest).
Membranous urethra Narrowest part of the urethra and surrounded by external sphincter. It traverses the perineal membrane 2.5cm postero-inferior to the symphysis pubis.
Penile urethra Travels through the corpus spongiosum on the underside of the penis. It is the longest urethral segment.It is dilated at its origin as the infrabulbar fossa and again in the glans penis as the navicular fossa. The bulbo-urethral glands open into the spongiose section of the urethra 2.5cm below the perineal membrane.

The urothelium is transitional in nature near to the bladder and becomes squamous more distally.

How well did you know this?
1
Not at all
2
3
4
5
Perfectly
789
Q

A 18 year old man presents with an indirect inguinal hernia and undergoes surgery. The deep inguinal ring is exposed and held with a retractor at its medial aspect. Which structure is most likely to lie under the retractor?

Ureter

Inferior epigastric artery

Internal iliac vein

Femoral artery

Lateral border of rectus abdominis

A

Boundaries of the deep inguinal ring:
Superolaterally - transversalis fascia
Inferomedially - inferior epigastric artery
The deep inguinal ring is closely related to the inferior epigastric artery. The inferior epigastric artery forms part of the structure referred to as Hesselbach’s triangle.
Inguinal canal

Location
Above the inguinal ligament
The inguinal canal is 4cm long
The superficial ring is located anterior to the pubic tubercle
The deep ring is located approximately 1.5-2cm above the half way point between the anterior superior iliac spine and the pubic tubercle

Boundaries of the inguinal canal
Floor
External oblique aponeurosis
Inguinal ligament
Lacunar ligament
Roof
Internal oblique
Transversus abdominis
Anterior wall External oblique aponeurosis
Posterior wall
Transversalis fascia
Conjoint tendon
Laterally
Internal ring
Transversalis fascia
Fibres of internal oblique
Medially
External ring
Conjoint tendon

Contents
Males Spermatic cord and ilioinguinal nerve As it passes through the canal the spermatic cord has 3 coverings:
External spermatic fascia from external oblique aponeurosis
Cremasteric fascia
Internal spermatic fascia
Females Round ligament of uterus and ilioinguinal nerve

How well did you know this?
1
Not at all
2
3
4
5
Perfectly
790
Q

A 55-year-old man with a history of type 2 diabetes mellitus, bipolar disorder and chronic obstructive pulmonary disease has bloods taken during a pre operative assessment of an inguinal hernia repair:

Na+ 125 mmol/l
K+ 3.8 mmol/l
Bicarbonate 24 mmol/l
Urea 3.7 mmol/l
Creatinine 92 µmol/l

Due to his smoking history a chest x-ray is ordered which is reported as normal. The Consultant asks you what is the most likely cause for the hyponatraemia?

Metformin

Lithium

Carbamazepine

Sodium valproate

Pioglitazone

A

SIADH - drug causes: carbamazepine, sulfonylureas, SSRIs, tricyclics
Lithium can cause diabetes insipidus but this is generally associated with a high sodium. Lithium only tends to cause raised antidiuretic hormone levels following a severe overdosage.

syndrome of inappropriate antidiuretic hormone (SIADH): causes

Malignancy
especially small cell lung cancer
also: pancreas, prostate

Neurological
stroke
subarachnoid haemorrhage
subdural haemorrhage
meningitis/encephalitis/abscess

Infections
tuberculosis
pneumonia

Drugs
sulfonylureas
SSRIs, tricyclics
carbamazepine
vincristine
cyclophosphamide

Other causes
positive end-expiratory pressure (PEEP)
porphyrias

How well did you know this?
1
Not at all
2
3
4
5
Perfectly
791
Q

You are assisting in an open right adrenalectomy for a large adrenal adenoma. The consultant is distracted and you helpfully pull the adrenal into the wound to improve the view. Unfortunately this is followed by brisk bleeding. The vessel responsible for this is most likely to be:

Portal vein

Phrenic vein

Right renal vein

Superior mesenteric vein

Inferior vena cava

A

It drains directly via a very short vessel. If the sutures are not carefully tied then it may be avulsed off the IVC. An injury best managed using a Satinsky clamp and a 6/0 prolene suture.

Adrenal gland anatomy

Anatomy

Location Superomedially to the upper pole of each kidney
Relationships of the right adrenal Diaphragm-Posteriorly, Kidney-Inferiorly, Vena Cava-Medially, Hepato-renal pouch and bare area of the liver-Anteriorly
Relationships of the left adrenal Crus of the diaphragm-Postero- medially, Pancreas and splenic vessels-Inferiorly, Lesser sac and stomach-Anteriorly
Arterial supply Superior adrenal arteries- from inferior phrenic artery, Middle adrenal arteries - from aorta, Inferior adrenal arteries -from renal arteries
Venous drainage of the right adrenal Via one central vein directly into the IVC
Venous drainage of the left adrenal Via one central vein into the left renal vein

How well did you know this?
1
Not at all
2
3
4
5
Perfectly
792
Q

Which of the following structures separates the posterior cruciate ligament from the popliteal artery?

Oblique popliteal ligament

Transverse ligament

Popliteus tendon

Biceps femoris

Semitendinosus

A

The posterior cruciate ligament is separated from the popliteal vessels at its origin by the oblique popliteal ligament.It is attached above to the upper margin of the intercondyloid fossa and posterior surface of the femur close to the articular margins of the condyles, and below to the posterior margin of the head of the tibia. The transverse ligament is located anteriorly.
Knee joint

The knee joint is a synovial joint, the largest and most complicated. It consists of two condylar joints between the femur and tibia and a sellar joint between the patella and the femur. The tibiofemoral articular surfaces are incongruent, however, this is improved by the presence of the menisci. The degree of congruence is related to the anatomical position of the knee joint and is greatest in full extension.

Knee joint compartments
Tibiofemoral
Comprised of the patella/femur joint, lateral and medial compartments (between femur condyles and tibia)
Synovial membrane and cruciate ligaments partially separate the medial and lateral compartments
Patellofemoral
Ligamentum patellae
Actions: provides joint stability in full extension

Fibrous capsule
The capsule of the knee joint is a complex, composite structure with contributions from adjacent tendons.
Anterior fibres The capsule does not pass proximal to the patella. It blends with the tendinous expansions of vastus medialis and lateralis
Posterior fibres These fibres are vertical and run from the posterior surface of the femoral condyles to the posterior aspect of the tibial condyle
Medial fibres Attach to the femoral and tibial condyles beyond their articular margins, blending with the tibial collateral ligament
Lateral fibres Attach to the femur superior to popliteus, pass over its tendon to head of fibula and tibial condyle

Bursae
Anterior
Subcutaneous prepatellar bursa; between patella and skin
Deep infrapatellar bursa; between tibia and patellar ligament
Subcutaneous infrapatellar bursa; between distal tibial tuberosity and skin
Laterally
Bursa between lateral head of gastrocnemius and joint capsule
Bursa between fibular collateral ligament and tendon of biceps femoris
Bursa between fibular collateral ligament and tendon of popliteus
Medially
Bursa between medial head of gastrocnemius and the fibrous capsule
Bursa between tibial collateral ligament and tendons of sartorius, gracilis and semitendinosus
Bursa between the tendon of semimembranosus and medial tibial condyle and medial head of gastrocnemius
Posterior Highly variable and inconsistent

Ligaments
Medial collateral ligament Medial epicondyle femur to medial tibial condyle: valgus stability
Lateral collateral ligament Lateral epicondyle femur to fibula head: varus stability
Anterior cruciate ligament Anterior tibia to lateral intercondylar notch femur: prevents tibia sliding anteriorly
Posterior cruciate ligament Posterior tibia to medial intercondylar notch femur: prevents tibia sliding posteriorly
Patellar ligament Central band of the tendon of quadriceps femoris, extends from patella to tibial tuberosity

Menisci
Medial and lateral menisci compensate for the incongruence of the femoral and tibial condyles.
Composed of fibrous tissue.
Medial meniscus is attached to the tibial collateral ligament.
Lateral meniscus is attached to the loose fibres at the lateral edge of the joint and is separate from the fibular collateral ligament. The lateral meniscus is crossed by the popliteus tendon.

Nerve supply
The knee joint is supplied by the femoral, tibial and common peroneal divisions of the sciatic and by a branch from the obturator nerve. Hip pathology pain may be referred to the knee.

Blood supply
Genicular branches of the femoral artery, popliteal and anterior tibial arteries all supply the knee joint.

How well did you know this?
1
Not at all
2
3
4
5
Perfectly
793
Q

Which of the following does not cause parotid enlargement?

Liver cirrhosis

Myxoedema

Amiodarone

Tuberculosis

Sjogrens syndrome

A

Amiodarone
Drugs commonly implicated in parotid gland enlargement include:
Thiouracil, isoprenaline, phenylbutazone, high oestrogen dose contraceptive pills.

Parotid gland clinical

Benign neoplasms
Up to 80% of all salivary gland tumours occur in the parotid gland and up to 80% of these are benign. There is no consistent correlation between the rate of growth and the malignant potential of the lesion. However, benign tumours should not invade structures such as the facial nerve.
With the exception of Warthins tumours, they are commoner in women than men. The median age of developing a lesion is in the 5th decade of life.

Benign tumour types
Tumour type Features
Benign pleomorphic adenoma or benign mixed tumor Most common parotid neoplasm (80%)
Proliferation of epithelial and myoepithelial cells of the ducts and an increase in stromal components
Slow growing, lobular, and not well encapsulated
Recurrence rate of 1-5% with appropriate excision (parotidectomy)
Recurrence possibly secondary to capsular disruption during surgery
Malignant degeneration occurring in 2-10% of adenomas observed for long periods, with carcinoma ex-pleomorphic adenoma occurring most frequently as adenocarcinoma
Warthin tumor (papillary cystadenoma lymphoma or adenolymphoma) Second most common benign parotid tumor (5%)
Most common bilateral benign neoplasm of the parotid
Marked male as compared to female predominance
Occurs later in life (sixth and seventh decades)
Presents as a lymphocytic infiltrate and cystic epithelial proliferation
May represent heterotopic salivary gland epithelial tissue trapped within intraparotid lymph nodes
Incidence of bilaterality and multicentricity of 10%
Malignant transformation rare (almost unheard of)
Monomorphic adenoma Account for less than 5% of tumours
Slow growing
Consist of only one morphological cell type (hence term mono)
Include; basal cell adenoma, canalicular adenoma, oncocytoma, myoepitheliomas
Haemangioma Should be considered in the differential of a parotid mass in a child
Accounts for 90% of parotid tumours in children less than 1 year of age
Hypervascular on imaging
Spontaneous regression may occur and malignant transformation is almost unheard of

Malignant salivary gland tumours
Types of malignancy

Mucoepidermoid carcinoma 30% of all parotid malignancies
Usually low potential for local invasiveness and metastasis (depends mainly on grade)
Adenoid cystic carcinoma Unpredictable growth pattern
Tendency for perineural spread
Nerve growth may display skip lesions resulting in incomplete excision
Distant metastasis more common (visceral rather than nodal spread)
5 year survival 35%
Mixed tumours Often a malignancy occurring in a previously benign parotid lesion
Acinic cell carcinoma Intermediate grade malignancy
May show perineural invasion
Low potential for distant metastasis
5 year survival 80%
Adenocarcinoma Develops from secretory portion of gland
Risk of regional nodal and distant metastasis
5 year survival depends upon stage at presentation, may be up to 75% with small lesions with no nodal involvement
Lymphoma Large rubbery lesion, may occur in association with Warthins tumours
Diagnosis should be based on regional nodal biopsy rather than parotid resection
Treatment is with chemotherapy (and radiotherapy)

Diagnostic evaluation
Plain x-rays may be used to exclude calculi
Sialography may be used to delineate ductal anatomy
FNAC is used in most cases
Superficial parotidectomy may be either diagnostic or therapeutic depending upon the nature of the lesion
Where malignancy is suspected the primary approach should be definitive resection rather than excisional biopsy
CT/ MRI may be used in cases of malignancy for staging primary disease

Treatment
For nearly all lesions this consists of surgical resection, for benign disease this will usually consist of a superficial parotidectomy. For malignant disease a radical or extended radical parotidectomy is performed. The facial nerve is included in the resection if involved. The need for neck dissection is determined by the potential for nodal involvement.

Other parotid disorders
HIV infection
Lymphoepithelial cysts associated with HIV occur almost exclusively in the parotid
Typically presents as bilateral, multicystic, symmetrical swelling
Risk of malignant transformation is low and management usually conservative

Sjogren syndrome
Autoimmune disorder characterised by parotid enlargement, xerostomia and keratoconjunctivitis sicca
90% of cases occur in females
Second most common connective tissue disorder
Bilateral, non tender enlargement of the gland is usual
Histologically, the usual findings are of a lymphocytic infiltrate in acinar units and epimyoepithelial islands surrounded by lymphoid stroma
Treatment is supportive
There is an increased risk of subsequent lymphoma

Sarcoid
Parotid involvement occurs in 6% of patients with sarcoid
Bilateral in most cases
Gland is not tender
Xerostomia may occur
Management of isolated parotid disease is usually conservative

How well did you know this?
1
Not at all
2
3
4
5
Perfectly
794
Q

Which of the following is commonest cause of acute abdominal pain in acute unselected surgical ‘take’?

Non specific abdominal pain

Biliary colic

Acute appendicitis

Ureteric colic

Pancreatitis

A

Non specific abdominal pain is a commonly recorded diagnosis for patients presenting with acute abdominal pain. Following careful diagnostic work up, a proportion of patients may be identified with disorders such as coeliac disease and the diagnosis of non specific abdominal pain should not be used lightly.

Acute abdominal pain

Acute abdominal pain is a common cause of admission to hospital. The relative proportions of conditions presenting with abdominal pain is given below:
Non specific abdominal pain (35%)
Appendicitis (17%)
Intestinal obstruction (15%)
Urological disease (6%)
Gallstone disease (5%)
Colonic diverticular disease (4%)
Abdominal trauma (3%)
Perforated peptic ulcer (3%)
Pancreatitis (2%)
(Data derived from Irvin T. Br. J. Surg 1989 76:1121-1125)

Non specific abdominal pain should really be a diagnosis of exclusion and if care is taken in excluding organic disease the proportion of cases labeled such should decline. It should also be appreciated that a proportion of patients may have an underlying medical cause for their symptoms such as pneumonia or diabetic ketoacidosis.

Key points in management
Early administration of adequate analgesia (including opiates).
Abdominal ultrasound is safe, non invasive and cheap and yields significantly more information than plain radiology. However, plain radiology is still the main test for suspected perforated viscus, especially out of hours.
In up to 50% cases with perforated peptic ulcer, the plain x-rays may show no evidence of free air. If clinical signs suggest otherwise, then a CT scan may be a more accurate investigation, if plain films are normal.
Plain film radiology usually cannot detect <1mm free air, and is 33% sensitive for detection of 1-13mm pockets of free air (Stoker et al. Radiology 2009 253: 31-46).
Think of strangulated intestine when there is fever, raised white cell count, tachycardia and peritonism.
In suspected large bowel obstruction a key investigation is either a water soluble contrast enema or CT scan.
Where need for surgery is difficult to define and imaging is inconclusive the use of laparoscopy as a definitive diagnostic test is both safe and sensible.

How well did you know this?
1
Not at all
2
3
4
5
Perfectly
795
Q

The cephalic vein pierces the clavipectoral fascia to terminate in which of the veins listed below?

External jugular

Axillary

Internal jugular

Azygos

Brachial

A

Path
Dorsal venous arch drains laterally into the cephalic vein
Crosses the anatomical snuffbox and travels laterally up the arm
At the antecubital fossa connected to the basilic vein by the median cubital vein
Pierces deep fascia of deltopectoral groove to join axillary vein

How well did you know this?
1
Not at all
2
3
4
5
Perfectly
796
Q

A 52 year man is admitted to the vascular ward for an amputation. The patient complains of unsteadiness. On further examination you detect right facial numbness and right sided nystagmus. There is sensory loss of the left side of the body. What is the most likely cause?

Lateral medullary syndrome

Pontine infarct

Cerebellar infarct

Middle cerebral artery infarct

Posterior cerebral artery infarct

A

A combination of ipsilateral ataxia, nystagmus, dysphagia, facial numbness, cranial nerve palsy with contralateral hemisensory loss indicates this diagnosis.

Stroke: types

Primary intracerebral haemorrhage (PICH, c. 10%)
Presents with headache, vomiting, loss of consciousness
Total anterior circulation infarcts (TACI, c. 15%)
Involves middle and anterior cerebral arteries
Hemiparesis/hemisensory loss
Homonymous hemianopia
Higher cognitive dysfunction e.g. Dysphasia
Partial anterior circulation infarcts (PACI, c. 25%)
Involves smaller arteries of anterior circulation e.g. upper or lower division of middle cerebral artery
Higher cognitive dysfunction or two of the three TACI features
Lacunar infarcts (LACI, c. 25%)
Involves perforating arteries around the internal capsule, thalamus and basal ganglia
Present with either isolated hemiparesis, hemisensory loss or hemiparesis with limb ataxia
Posterior circulation infarcts (POCI, c. 25%)
Vertebrobasilar arteries
Presents with features of brainstem damage
Ataxia, disorders of gaze and vision, cranial nerve lesions
Lateral medullary syndrome (posterior inferior cerebellar artery)
Wallenberg’s syndrome
Ipsilateral: ataxia, nystagmus, dysphagia, facial numbness, cranial nerve palsy
Contralateral: limb sensory loss
Weber’s syndrome
Ipsilateral III palsy
Contralateral weakness

Anterior cerebral artery
Contralateral hemiparesis and sensory loss, lower extremity > upper
Disconnection syndrome

Middle cerebral artery
Contralateral hemiparesis and sensory loss, upper extremity > lower
Contralateral hemianopia
Aphasia (Wernicke’s)
Gaze abnormalities

Posterior cerebral artery
Contralateral hemianopia with macular sparing
Disconnection syndrome

Lacunar
Present with either isolated hemiparesis, hemisensory loss or hemiparesis with limb ataxia

Lateral medulla (posterior inferior cerebellar artery)
Ipsilateral: ataxia, nystagmus, dysphagia, facial numbness, cranial nerve palsy e.g.

Horner’s
Contralateral: limb sensory loss

Pontine
VI nerve: horizontal gaze palsy
VII nerve
Contralateral hemiparesis

How well did you know this?
1
Not at all
2
3
4
5
Perfectly
797
Q

A 43 year old lady is admitted with cholestasis secondary to a stone impacted at the level of the ampulla of vater. Which of the following tests is most likely to be predictive of bleeding diathesis at the time of ERCP in this particular case?

Bleeding time

Prothrombin time

APTT

Platelet count

Factor I levels

A

PT: Vitamin K dependent factors 2, 7, 9, 10
APTT: Factors 8, 9, 11, 12
Jaundice will impair the production of vitamin K dependent clotting factors. This is most accurately tested by measuring the prothrombin time. APTT can be affected by vitamin K deficiency (due to factor 9 deficiency), however this occurs to a lesser extent and is normally associated with severe liver disease. The bleeding time is a measure of platelet function.

Abnormal coagulation

Cause Factors affected
Heparin Prevents activation factors 2,9,10,11
Warfarin Affects synthesis of factors 2,7,9,10
DIC Factors 1,2,5,8,11
Liver disease Factors 1,2,5,7,9,10,11

Interpretation blood clotting test results
Disorder APTT PT Bleeding time
Haemophilia Increased Normal Normal
von Willebrand’s disease Increased Normal Increased
Vitamin K deficiency Increased Increased Normal

How well did you know this?
1
Not at all
2
3
4
5
Perfectly
798
Q

A 28 year old professional footballer is admitted to the emergency department. During a tackle his leg is twisted with his knee flexed. He hears a loud crack and his knee rapidly becomes swollen. Which of the following structures is the main site of injury?

Anterior cruciate ligament

Posterior cruciate ligament

Meniscus

Medial collateral ligament

Lateral collateral ligament

A

This is common in footballers as the football boot studs stick to the ground and high twisting force is applied to a flexed knee. Rapid joint swelling also supports the diagnosis.
Knee injury

Types of injury

Ruptured anterior cruciate ligament
Sport injury
Mechanism: high twisting force applied to a bent knee
Typically presents with: loud crack, pain and RAPID joint swelling (haemoarthrosis)
Poor healing
Management: intense physiotherapy or surgery
Ruptured posterior cruciate ligament
Mechanism: hyperextension injuries
Tibia lies back on the femur
Paradoxical anterior draw test
Rupture of medial collateral ligament
Mechanism: leg forced into valgus via force outside the leg
Knee unstable when put into valgus position
Menisceal tear
Rotational sporting injuries
Delayed knee swelling
Joint locking (Patient may develop skills to ‘unlock’ the knee
Recurrent episodes of pain and effusions are common, often following minor trauma
Chondromalacia patellae
Teenage girls, following an injury to knee e.g. Dislocation patella
Typical history of pain on going downstairs or at rest
Tenderness, quadriceps wasting
Dislocation of the patella
Most commonly occurs as a traumatic primary event, either through direct trauma or through severe contraction of quadriceps with knee stretched in valgus and external rotation
Genu valgum, tibial torsion and high riding patella are risk factors
Skyline x-ray views of patella are required, although displaced patella may be clinically obvious
An osteochondral fracture is present in 5%
The condition has a 20% recurrence rate
Fractured patella
2 types:
i. Direct blow to patella causing undisplaced fragments
ii. Avulsion fracture
Tibial plateau fracture
Occur in the elderly (or following significant trauma in young)
Mechanism: knee forced into valgus or varus, but the knee fractures before the ligaments rupture
Varus injury affects medial plateau and if valgus injury, lateral plateau depressed fracture occurs
Classified using the Schatzker system (see below)

Schatzker Classification system for tibial plateau fractures
Type Anatomical description Features
1 Vertical split of lateral condyle Fracture through dense bone, usually in the young. It may be virtually undisplaced, or the condylar fragment may be pushed inferiorly and tilted
2 Vertical split of the lateral condyle combined with an adjacent load bearing part of the condyle The wedge fragment (which may be of variable size), is displaced laterally; the joint is widened. Untreated, a valgus deformity may develop
3 Depression of the articular surface with intact condylar rim The split does not extend to the edge of the plateau. Depressed fragments may be firmly embedded in subchondral bone, the joint is stable
4 Fragment of the medial tibial condyle Two injuries are seen in this category; (1) a depressed fracture of osteoporotic bone in the elderly. (2) a high energy fracture resulting in a condylar split that runs from the intercondylar eminence to the medial cortex. Associated ligamentous injury may be severe
5 Fracture of both condyles Both condyles fractured but the column of the metaphysis remains in continuity with the tibial shaft
6 Combined condylar and subcondylar fractures High energy fracture with marked comminution

How well did you know this?
1
Not at all
2
3
4
5
Perfectly
799
Q

A 43 year old female has undergone a renal transplant 12 months previously. Over the past few weeks there have been concerns about deteriorating renal function. What is the most useful investigation?

MAG 3 renogram

DMSA scan

Intravenous KUB urogram

Renal CT scan

Micturating cystourethrogram

A

Because it is excreted by renal tubular cells a MAG 3 renogram provides excellent imaging of renal function and is often used in investigating failing transplants.

Functional renal imaging

DMSA scan
Dimercaptosuccinic acid (DMSA) scintigraphy
DMSA localises to the renal cortex with little accumulation in the renal papilla and medulla. It is useful for the identification of cortical defects and ectopic or aberrant kidneys. It does not provide useful information on the ureter of collecting system.

Diethylene-triamine-penta-acetic acid (DTPA)
This is primarily a glomerular filtration agent. It is most useful for the assessment of renal function. Because it is filtered at the level of the glomerulus it provides useful information about the GFR. Image quality may be degraded in patients with chronic renal impairment and derangement of GFR.

MAG 3 renogram
Mercaptoacetyle triglycine is an is extensively protein bound and is primarily secreted by tubular cells rather than filtered at the glomerulus. This makes it the agent of choice for imaging the kidneys of patients with existing renal impairment (where GFR is impaired).

Micturating cystourethrogram (MCUG scan)
This scan provides information relating to bladder reflux and is obtained by filling the bladder with contrast media (via a catheter) and asking the child to void. Images are taken during this phase and the degree of reflux can be calculated

Intra venous urography
This examination is conducted by the administration of intravenous iodinated contrast media. The agent is filtered by the kidneys and excreted and may provide evidence of renal stones or other structural lesions. A rough approximation of renal function may be obtained using the technique. But it is not primarily a technique to be used for this purpose. With the advent of widespread non contrast CT scan protocols for the detection of urinary tract calculi it is now rarely used.

PET/CT
This may be used to evaluate structurally indeterminate lesions in the staging of malignancy.

References
Davis A et al. Investigating urinary tract infections in children. BMJ 2013 (346):35-37.

How well did you know this?
1
Not at all
2
3
4
5
Perfectly
800
Q

When is it mandatory to involve an independent mental capacity advocate ?

When a patient requires a significant intervention, time permits and the patient lacks capacity to decide for themselves

When a patient cannot decide on the best treatment

When parents disagree about treatment of a child

When two doctors disagree about the best treatment option

When a patient has lost trust in their clinician

A

When a patient requires a significant intervention, time permits and the patient lacks capacity to decide for themselves

The Mental Capacity Act (MCA) requires doctors to involve an independent mental capacity advocate (IMCA) for serious medical treatment decisions when:
a best interests decision is required because the doctor has assessed the person as not having the capacity to make the decision themselves
the person does not have family or friends with whom it is appropriate to consult about the decision.

Serious medical treatment is defined in the Mental Capacity Act 2005 (Independent Mental Capacity Advocates) (General) Regulations 2006 as treatment which involves providing, withdrawing or withholding treatment in circumstances where one or more of the following apply:
in a case where a single treatment is being proposed, there is a fine balance between its benefits to the patient and the burdens and risks it is likely to entail for them
in a case where there is a choice of treatments, a decision as to which one to use is finely balanced
what is proposed would be likely to involve serious consequences for the patient.
The MCA Code of Practice says that serious consequences may include treatment options which:
cause serious and prolonged pain, distress or side effects have potentially major consequences for the patient (for example, major surgery or stopping life-sustaining treatment)
have a serious impact on the patients future life choices.

Consent

There are 3 types of consent:

  1. Informed
  2. Expressed
  3. Implied

Consent forms used in UK NHS
Consent Form 1 For competent adults who are able to consent for themselves where consciousness may be impaired (e.g. GA)
Consent Form 2 For an adult consenting on behalf of a child where consciousness is impaired
Consent Form 3 For an adult or child where consciousness is not impaired
Consent Form 4 For adults who lack capacity to provide informed consent

Capacity
Key points include:
1. Understand and retain information
2. Patient believes the information to be true
3. Patient is able to weigh the information to make a decision
All patients must be assumed to have capacity

Consent in minors
Young children and older children who are not Gillick competent cannot consent for themselves. In British law the patients biological mother can always provide consent. The child’s father can consent if the parents are married (and the father is the biological father), or if the father is named on the birth certificate (irrespective of marital status). If parents are not married and the father is not named on the birth certificate then the father cannot consent.

How well did you know this?
1
Not at all
2
3
4
5
Perfectly
801
Q

A 63 year old man undergoes an upper GI endoscopy and adrenaline injection for a large actively bleeding duodenal ulcer. He remains stable for 6 hours and the nurses then call because he has passed 400ml malaena and has become tachycardic (pulse rate 120) and hypotensive (Bp 80/40). What is the best option?

Reassure that blood trapped in the upper portion of the gastrointestinal system will pass and that this episode will resolve with phosphate enema

Perform a repeat upper GI endoscopy

Perform a laparotomy and under-running of the ulcer

Administer tranexamic acid and intravenous proton pump inhibitors

Insert a Minnesota tube

A

The decision as to how best to manage patients with re-bleeding is difficult. Whilst it is tempting to offer repeat endoscopy, this intervention is best used on those with small ulcers. Large posteriorly sited duodenal ulcers are at high risk for re-bleeding and the timeframe of this event suggests that primary endoscopic haemostasis was inadequate. Surgery thus represents the safest way forward.

Upper gastrointestinal bleeding

Patients may present with the following:
Haematemesis and/ or malaena
Epigastric discomfort
Sudden collapse

The extent to which these will occur will depend upon the source. Mortality is higher in patients presenting with haematemesis than malaena alone.

Oesophageal bleeding
Cause Presenting features
Oesophagitis Small volume of fresh blood, often streaking vomit. Malaena rare. Often ceases spontaneously. Usually history of antecedent GORD type symptoms.
Cancer Usually small volume of blood, except as pre terminal event with erosion of major vessels. Often associated symptoms of dysphagia and constitutional symptoms such as weight loss. May be recurrent until malignancy managed.
Mallory Weiss Tear Typically brisk small to moderate volume of bright red blood following bout of repeated vomiting. Malaena rare. Usually ceases spontaneously.
Varices Usually large volume of fresh blood. Swallowed blood may cause malaena. Often associated with haemodynamic compromise. May stop spontaneously but re-bleeds are common until appropriately managed.

Gastric Bleeding
Cause Presenting features
Gastric cancer May be frank haematemesis or altered blood mixed with vomit. Usually prodromal features of dyspepsia and may have constitutional symptoms. Amount of bleeding variable but erosion of major vessel may produce considerable haemorrhage.
Dieulafoy Lesion Often no prodromal features prior to haematemesis and malaena, but this arteriovenous malformation may produce quite considerable haemorrhage and may be difficult to detect endoscopically.
Diffuse erosive gastritis Usually haematemesis and epigastric discomfort. Usually there is an underlying cause such as recent NSAID usage. Large volume haemorrhage may occur with considerable haemodynamic compromise.
Gastric ulcer Small low volume bleeds more common so would tend to present as iron deficiency anaemia. Erosion into a significant vessel may produce considerable haemorrhage and haematemesis.

Duodenum
Most common cause of major haemorrhage is a posteriorly sited duodenal ulcer. However, ulcers at any site in the duodenum may present with haematemesis, malaena and epigastric discomfort. The pain of duodenal ulcer is slightly different to that of gastric ulcers and often occurs several hours after eating. Peri ampullary tumours may bleed but these are rare. In patients with previous abdominal aortic aneurysm surgery aorto-enteric fistulation remains a rare but important cause of major haemorrhage associated with high mortality.

Management
Admission to hospital careful monitoring, cross match blood, check FBC, LFTs, U+E and Clotting (as a minimum)
Patients with on-going bleeding and haemodynamic instability are likely to require O negative blood pending cross matched blood
Early control of airway is vital (e.g. Drowsy patient with liver failure)
Patients with suspected varices should receive terlipressin prior to endoscopy
Ideally all patients admitted with upper gastrointestinal haemorrhage should undergo Upper GI endoscopy within 24 hours of admission. In those who are unstable this should occur immediately after resuscitation or in tandem with it. The endoscopy department is a potentially dangerous place for unstable patients and it may be safer to perform the endoscopy in theatre with an anaesthetist present.
Varices should be banded or subjected to sclerotherapy. If this is not possible owing to active bleeding then a Sengstaken- Blakemore tube (or Minnesota tube) should be inserted. This should be done with care; gastric balloon should be inflated first and oesophageal balloon second. Remember the balloon will need deflating after 12 hours (ideally sooner) to prevent necrosis. Portal pressure should be lowered by combination of medical therapy +/- TIPSS.
Patients with erosive oesophagitis / gastritis should receive a proton pump inhibitor.
Mallory Weiss tears will typically resolve spontaneously
Identifiable bleeding points should receive combination therapy of injection of adrenaline and either a thermal or mechanical treatment. All who have received intervention should receive a continuous infusion of a proton pump inhibitor (IV omeprazole for 72 hours) to reduce the re-bleeding rate.
Patients with diffuse erosive gastritis who cannot be managed endoscopically and continue to bleed may require gastrectomy
Bleeding ulcers that cannot be controlled endoscopically may require laparotomy and ulcer underruning

Indications for surgery
Patients > 60 years
Continued bleeding despite endoscopic intervention
Recurrent bleeding
Known cardiovascular disease with poor response to hypotension

Surgery
Duodenal ulcer
Laparotomy, duodenotomy and under running of the ulcer. If bleeding is brisk then the ulcer is almost always posteriorly sited and will have invaded the gastroduodenal artery. Large bites using 0 Vicryl are taken above and below the ulcer base to occlude the vessel. The duodenotomy should be longitudinal but closed transversely to avoid stenosis.

For gastric ulcer
Under-running of the bleeding site
Partial gastrectomy-antral ulcer
Partial gastrectomy or under running the ulcer- lesser curve ulcer (involving left gastric artery)
Total gastrectomy if bleeding persists

Summary of Acute Upper GI bleeding recommendations:
The need for admission and timing of endoscopic intervention may be predicted by using the Blatchford score. This considers a patients Hb, serum urea, pulse rate and blood pressure. Those patients with a score of 0 are low risk, all others are considered high risk and require admission and endoscopy.
The requirement for pre endoscopic proton pump inhibition is contentious. In the UK the National Institute of Clinical Excellence guidelines suggest the pre endoscopic PPI therapy is unnecessary. Whilst it is accepted that such treatment has no impact on mortality or morbidity a Cochrane review of this practice in 2007 did suggest that it reduced the stigmata of recent haemorrhage at endoscopy. As a result many will still administer PPI to patients prior to endoscopic intervention.
Following endoscopy it is important to calculate the Rockall score for patients to determine their risk of rebleeding and mortality. A score of 3 or less is associated with a rebleeding rate of 4% and a very low risk of mortality and identifies a group of patients suitable for early discharge.

References
1. http://www.sign.ac.uk/guidelines/fulltext/105/index.html
2. Joint Advisory Group on Endoscopy (JAG) Guidelines - http://www.thejag.org.uk
3. NICE Guideline: Management of acute upper GI bleeding. July 2012.

How well did you know this?
1
Not at all
2
3
4
5
Perfectly
802
Q

A 34 year old female presents with a thyroid nodule. She has a family history of thyroid disease and both her sisters have undergone total thyroidectomies. Her past medical history includes hypertension which has been difficult to manage. What is the most likely underlying thyroid lesion?

Papillary thyroid cancer

Follicular adenoma

Medullary thyroid cancer

Anaplastic thyroid cancer

Lymphoma of the thyroid

A

This is a typical scenario for medullary carcinoma in which a phaeochromocytoma may also be present. It may be inherited in an autosomal dominant fashion and affected family members may be offered prophylactic thyroidectomy.

Thyroid disease

Patients may present with a number of different manifestations of thyroid disease. They can be broadly sub classified according to whether they are euthyroid or have clinical signs of thyroid dysfunction. In addition it needs to be established whether they have a mass or not.

Assessment
History
Examination including USS
If a nodule is identified then it should be sampled ideally via an image guided fine needle aspiration
Radionucleotide scanning is of limited use

Thyroid Tumours
Papillary carcinoma
Follicular carcinoma
Anaplastic carcinoma
Medullary carcinoma
Lymphoma’s

Multinodular goitre
One of the most common reasons for presentation
Provided the patient is euthyroid and asymptomatic and no discrete nodules are seen, they can be reassured.
In those with compressive symptoms surgery is required and the best operation is a total thyroidectomy.
Sub total resections were practised in the past and simply result in recurrent disease that requires a difficult revisional resection.

Endocrine dysfunction
In general these patients are managed by physicians initially.
Surgery may be offered alongside radio iodine for patients with Graves disease that fails with medical management or in patients who would prefer not to be irradiated (e.g. pregnant women).
Patients with hypothyroidism do not generally get offered a thyroidectomy. Sometimes people inadvertently get offered resections during the early phase of Hashimotos thyroiditis, however, with time the toxic phase passes and patients can simply be managed with thyroxine.

Complications following surgery
Anatomical such as recurrent laryngeal nerve damage.
Bleeding. Owing to the confined space haematoma’s may rapidly lead to respiratory compromise owing to laryngeal oedema.
Damage to the parathyroid glands resulting in hypocalcaemia.

Further sources of information
1. http://www.acb.org.uk/docs/TFTguidelinefinal.pdf- Association of Clinical Biochemistry guidelines for thyroid function tests.

  1. British association of endocrine surgeons website- http://www.baets.org.uk
How well did you know this?
1
Not at all
2
3
4
5
Perfectly
803
Q

Which of the conditions listed below is least likely to be associated with hypersplenism?

Coeliac disease

Lymphoma

Leukaemia

Portal hypertension

Feltys syndrome

A

Many conditions may result in overactivity of the spleen. It is not seen in coeliac disease. Feltys syndrome occurs in association with rheumatoid disease.

Disorders affecting the spleen

One of the commonest conditions to affect the spleen is traumatic rupture and this is addressed elsewhere.

Splenomegaly
Enlargement of the spleen can commonly accompany a variety of haematological disorders. Of these, chronic conditions such as haemolytic anaemia and chronic leukaemia are often associated with splenomegaly. More acute haematological conditions tend not to cause massive splenic enlargement. Infections with parasites (such as malaria) and viruses such as Epstein Barr virus can cause splenomegaly and this one condition that is associated with sudden spontaneous splenic rupture. Portal hypertension can result in splenomegaly and the finding of an enlarged spleen on abdominal examination should prompt a search for the other stigmata of chronic liver disease.

Splenic atrophy
This typically occurs as a result of conditions that impair the blood flow to the spleen. Division of some of the major vessels can cause shrinkage of the spleen and this is commonly seen when splenic artery aneurysms are coiled for example.

How well did you know this?
1
Not at all
2
3
4
5
Perfectly
804
Q

Which of the following is not classically seen in coning resulting from raised intra cranial pressure?

Coma

Hypotension

Unreactive mid sized pupils

Cheyne Stokes style respiratory efforts

Bradycardia

A

Hypotension
Cushings triad
Widening of the pulse pressure
Respiratory changes
Bradycardia
Due to raised ICP systemic hypertension is usually seen. Compression of the respiratory centre will typically result in Cheyne Stokes style respiration.

Coning

The cranial vault is a confined cavity apart from infants with a non fused fontanelle.
Rises in ICP may be accommodated by shifts of CSF.
Once the CSF shifting has reached its capacity ICP will start to rise briskly.
The brain autoregulates its blood supply, as ICP rises the systemic circulation will display changes to try and meet the perfusion needs of the brain. Usually this will involve hypertension.
As ICP rises further, the brain will be compressed, cranial nerve palsies may be seen and compression of essential centres in the brain stem will occur. When the cardiac centre is involved bradycardia will often develop.

How well did you know this?
1
Not at all
2
3
4
5
Perfectly
805
Q

A 74 year old woman presents with a breast lump. On examination, it has a soft consistency. The lump is removed and sliced apart. Macroscopically there is a grey, gelatinous surface. Which of the following tumour types is most likely?

Sarcoma

Invasive ductal carcinoma

Mucinous carcinoma

DCIS

Lobular carcinoma

A

Mucinous carcinomas comprise 2-3% of all breast cancers. They are one of the special type of carcinomas. These have a better prognosis than is associated with tumours of Non Special Type (NST) and axillary nodal disease is rare in this group.
Breast cancer

  • Commoner in the older age group
    Invasive ductal carcinomas are the most common type. Some may arise as a result of ductal carcinoma in situ (DCIS). There are associated carcinomas of special type e.g. Tubular that may carry better prognosis.
    The pathological assessment involves assessment of the tumour and lymph nodes, sentinel lymph node biopsy is often used to minimise the morbidity of an axillary dissection.
    Treatment, typically this is either wide local excision or mastectomy. There are many sub types of both of these that fall outside of the MRCS. Some key rules to bear in mind.
    Whatever operation is contemplated the final cosmetic outcome does have a bearing. A woman with small breasts and a large tumour will tend to fare better with mastectomy, even if clear pathological and clinical margins can be obtained. Conversely a women with larger breasts may be able to undergo breast conserving surgery even with a relatively large primary lesion (NB tumours >4cm used to attract recommendation for mastectomy). For screen detected and impalpable tumour image guidance will be necessary.
    Reconstruction is always an option following any resectional procedure. However, its exact type must be tailored to age and co-morbidities of the patient. The main operations in common use include latissimus dorsi myocutaneous flap and sub pectoral implants. Women wishing to avoid a prosthesis may be offered TRAM or DIEP flaps.

Surgical options
Mastectomy vs Wide local excision

Mastectomy Wide Local Excision
Multifocal tumour Solitary lesion
Central tumour Peripheral tumour
Large lesion in small breast Small lesion in large breast
DCIS >4cm DCIS <4cm
Patient Choice Patient choice

Central lesions may be managed using breast conserving surgery where an acceptable cosmetic result may be obtained, this is rarely the case in small breasts

Whatever surgical option is chosen the aim should be to have a local recurrence rate of 5% or less at 5 years [1].

Nottingham Prognostic Index
The Nottingham Prognostic Index can be used to give an indication of survival. In this system the tumour size is weighted less heavily than other major prognostic parameters.

Calculation of NPI
Tumour Size x 0.2 + Lymph node score(From table below)+Grade score(From table below).

Score Lymph nodes involved Grade
1 0 1
2 1-3 2
3 >3 3

Prognosis

Score Percentage 5 year survival
2.0 to 2.4 93%
2.5 to 3.4 85%
3.5 to 5.4 70%
>5.4 50%

This data was originally published in 1992. It should be emphasised that other factors such as vascular invasion and receptor status also impact on survival and are not included in this data and account for varying prognoses often cited in the literature.

References
For guidance on how breast cancer is managed in the United Kingdom visit the Association of Breast Surgery website (www.https://associationofbreastsurgery.org.uk/).

How well did you know this?
1
Not at all
2
3
4
5
Perfectly
806
Q

A 28 year old man presents with a burn injury secondary to a house fire. He has sustained 25% mixed full and partial thickness burns to his torso and limbs. Which of the following resuscitation protocols would be most appropriate in this case?

Administration of 2 ml of Hartmanns x weight in kg x 25 to maintain a urine output of 30ml/ hour

Administration of 4 ml of Hartmanns x weight in kg x 25 to maintain a urine output of 100ml/ hour

Administration of 3 ml of Hartmanns x weight in kg x 25 to maintain a urine output of 70ml/ hour

Administration of 2 ml of 5% dextrose x weight in kg x 25 to maintain a urine output of 50ml/ hour

Administration of 2 ml of Dextrose / saline x weight in kg x 25 to maintain a urine output of 30ml/ hour

A

A

Lower infusion volumes are now administered to burn patients to minimize the potential harm caused by excessive fluid administration

Fluid resuscitation burns

Indication: >15% total body area burns in adults (>10% children)
The main aim of resuscitation is to prevent the burn deepening
Most fluid is lost 24 hours after injury
First 8-12 hour fluid shifts are from intravascular to interstitial fluid compartments
Therefore circulatory volume can be compromised. However fluid resuscitation causes more fluid into the interstitial compartment especially colloid (therefore avoided in first 8-24 hours)
Protein loss occurs

There are concerns that use of the traditional Parkland formula resulted in the administration of excessive quantities of intravenous fluids in patients with burns. The current consensus guidelines state that fluid resuscitation should begin at 2 ml of lactated Ringers x patients body weight in kg x % TBSA for second- and third-degree burns.
The calculated fluid volume is initiated in the following manner: one-half of the total fluid is provided in the first 8 hours after the burn injury (for example, a 100-kg man with 80% TBSA burns requires 2 × 80 × 100 = 16,000 mL in 24 hours). One-half of that volume (8,000 mL) should be provided in the first 8 hours, so the patient should be started at a rate of 1000 mL/hr. The remaining one-half of the total fluid is administered during the subsequent 16 hours.
The efficacy of fluid replacement is determined in the main by monitoring urine output. For adults this is in the region of 0.5ml/kg/hr and for children less than 30Kg it is 1ml/kg/hr.

Fluid resuscitation according to burn type
Type of burn Age or weight Fluid rate Urine output
Flames or scalding Adults and children over 14 years 2 ml Hartmanns x kg x % TBSA 0.3-0.5ml/kg/hr OR 30-50ml per hour
Flames or scalding Children less than 14 years 3 ml Hartmanns x kg x % TBSA 1ml/kg/hr
Flames or scalds Children less than 30Kg and infants 3 ml Hartmanns x kg x % TBSA. Plus a sugar-containing solution at maintenance rate 1ml/kg/hr
Electrical injury All ages 4 ml Hartmanns x kg x % TBSA until urine clears 1-1.5 ml/kg/hr until urine clears

Fluid resuscitation in electrical burn patients is different because they have far greater soft tissue involvement in muscle compartments and associated muscle death that can result in myoglobinuria.

After 24 hours
Maintenance crystalloid (usually dextrose-saline) is continued at a rate of 1.5 ml x(burn area)x(body weight)
Colloids are rarely used (e.g. albumin)
Antioxidants, such as vitamin C, can be used to minimize oxidant-mediated contributions to the inflammatory cascade in burns
High tension electrical injuries and inhalation injuries require more fluid
Monitor: packed cell volume, plasma sodium, base excess, and lactate

Reference
ATLS Manual 10th Edition.

How well did you know this?
1
Not at all
2
3
4
5
Perfectly
807
Q

A 53 year old lady undergoes a mastectomy to remove a breast cancer. Microscopic analysis of the tumour shows a pronounced lymphocytic infiltrate. Which of the tumour types listed below is most often associated with this finding?

Invasive ductal carcinoma

Tubular breast cancer

Medullary breast cancer

Mucinous breast cancer

Adenoid cystic carcinoma of the breast

A

Medullary breast cancer is a breast cancer of special type. Histologically, it is characterised by a marked lymphocytic infiltrate. Many of the breast cancers of special type (which comprise less than 5% of all breast cancers) have a very good prognosis. These special type cancers include; medullary, tubular, mucinous, adenoid cystic,

Breast cancer

  • Commoner in the older age group
    Invasive ductal carcinomas are the most common type. Some may arise as a result of ductal carcinoma in situ (DCIS). There are associated carcinomas of special type e.g. Tubular that may carry better prognosis.
    The pathological assessment involves assessment of the tumour and lymph nodes, sentinel lymph node biopsy is often used to minimise the morbidity of an axillary dissection.
    Treatment, typically this is either wide local excision or mastectomy. There are many sub types of both of these that fall outside of the MRCS. Some key rules to bear in mind.
    Whatever operation is contemplated the final cosmetic outcome does have a bearing. A woman with small breasts and a large tumour will tend to fare better with mastectomy, even if clear pathological and clinical margins can be obtained. Conversely a women with larger breasts may be able to undergo breast conserving surgery even with a relatively large primary lesion (NB tumours >4cm used to attract recommendation for mastectomy). For screen detected and impalpable tumour image guidance will be necessary.
    Reconstruction is always an option following any resectional procedure. However, its exact type must be tailored to age and co-morbidities of the patient. The main operations in common use include latissimus dorsi myocutaneous flap and sub pectoral implants. Women wishing to avoid a prosthesis may be offered TRAM or DIEP flaps.

Surgical options
Mastectomy vs Wide local excision

Mastectomy Wide Local Excision
Multifocal tumour Solitary lesion
Central tumour Peripheral tumour
Large lesion in small breast Small lesion in large breast
DCIS >4cm DCIS <4cm
Patient Choice Patient choice

Central lesions may be managed using breast conserving surgery where an acceptable cosmetic result may be obtained, this is rarely the case in small breasts

A compelling indication for mastectomy, a larger tumour that would be unsuitable for breast conserving surgery

Whatever surgical option is chosen the aim should be to have a local recurrence rate of 5% or less at 5 years [1].

Nottingham Prognostic Index
The Nottingham Prognostic Index can be used to give an indication of survival. In this system the tumour size is weighted less heavily than other major prognostic parameters.

Calculation of NPI
Tumour Size x 0.2 + Lymph node score(From table below)+Grade score(From table below).

Score Lymph nodes involved Grade
1 0 1
2 1-3 2
3 >3 3

Prognosis

Score Percentage 5 year survival
2.0 to 2.4 93%
2.5 to 3.4 85%
3.5 to 5.4 70%
>5.4 50%

This data was originally published in 1992. It should be emphasised that other factors such as vascular invasion and receptor status also impact on survival and are not included in this data and account for varying prognoses often cited in the literature.

References
For guidance on how breast cancer is managed in the United Kingdom visit the Association of Breast Surgery website (www.https://associationofbreastsurgery.org.uk/).

How well did you know this?
1
Not at all
2
3
4
5
Perfectly
808
Q

During a carotid endarterectomy the internal carotid artery is cross clamped. Assuming that no shunt is inserted, which of the following vessels will not have diminished or absent flow as a result?

Anterior cerebral artery

Ophthalmic artery

Middle cerebral artery

Maxillary artery

None of the above

A

Mnemonic for branches of the cerebral portion of the internal carotid artery ‘Only Press Carotid Arteries Momentarily’

Only = Opthalmic
Press = Posterior communicating
Carotid = Choroidal
Arteries = Anterior cerebral
Momentarily = Middle cerebral
Internal carotid artery

The internal carotid artery is formed from the common carotid opposite the upper border of the thyroid cartilage. It extends superiorly to enter the skull via the carotid canal. From the carotid canal it then passes through the cavernous sinus, above which it divides into the anterior and middle cerebral arteries.

Relations in the neck
Posterior
Longus capitis
Pre-vertebral fascia
Sympathetic chain
Superior laryngeal nerve
Medially
External carotid (near origin)
Wall of pharynx
Ascending pharyngeal artery
Laterally
Internal jugular vein (moves posteriorly at entrance to skull)
Vagus nerve (most posterolaterally)
Anteriorly
Sternocleidomastoid
Lingual and facial veins
Hypoglossal nerve

Relations in the carotid canal
Internal carotid plexus
Cochlea and middle ear cavity
Trigeminal ganglion (superiorly)
Leaves canal lies above the foramen lacerum

Path and relations in the cranial cavity
The artery bends sharply forwards in the cavernous sinus, the aducens nerve lies close to its inferolateral aspect. The oculomotor, trochlear, opthalmic and, usually, the maxillary nerves lie in the lateral wall of the sinus. Near the superior orbital fissure it turns posteriorly and passes postero-medially to pierce the roof of the cavernous sinus inferior to the optic nerve. It then passes between the optic and oculomotor nerves to terminate below the anterior perforated substance by dividing into the anterior and middle cerebral arteries.

Branches
Anterior and middle cerebral artery
Ophthalmic artery
Posterior communicating artery
Anterior choroid artery
Meningeal arteries
Hypophyseal arteries

How well did you know this?
1
Not at all
2
3
4
5
Perfectly
809
Q

What is the typical stroke volume in a resting 70 Kg man?

10ml

150ml

125ml

45ml

70ml

A

Stroke volumes range from 55-100ml. Stroke volume-Cardiac physiology

The stroke volume equates to the volume of blood ejected from the ventricle during each cycle of cardiac contraction. The volumes for both ventricles are typically equal and equate roughly to 70ml for a 70Kg man. It is calculated by subtracting the end systolic volume from the end diastolic volume.

Factors affecting stroke volume
Cardiac size
Contractility
Preload
Afterload

How well did you know this?
1
Not at all
2
3
4
5
Perfectly
810
Q

Which of these nerves passes through the greater and lesser sciatic foramina?

Pudendal nerve

Sciatic nerve

Superior gluteal nerve

Inferior gluteal nerve

Posterior cutaneous nerve of the thigh

A

Structures passing through the lesser and greater sciatic foramina (medial to lateral): PIN
Pudendal nerve
Internal pudendal artery
Nerve to obturator internus
The pudendal nerve originates from the ventral rami of the second, third, and fourth sacral nerves (S2, S3, S4).

It passes between the piriformis and coccygeus muscles and exits the pelvis through the the greater sciatic foramen. It crosses the spine of the ischium and reenters the pelvis through the lesser sciatic foramen. It passes through the pudendal canal.

The pudendal nerve gives off the inferior rectal nerves. It terminates into 2 branches: perineal nerve, and the dorsal nerve of the penis or the dorsal nerve of the clitoris.

How well did you know this?
1
Not at all
2
3
4
5
Perfectly
811
Q

A 45 year man presents with hand weakness. He is given a piece of paper to hold between his thumb and index finger. When the paper is pulled, the patient has difficulty maintaining a grip. Grip pressure is maintained by flexing the thumb at the interphalangeal joint. What is the most likely nerve lesion?

Posterior interosseous nerve

Deep branch of ulnar nerve

Anterior interosseous nerve

Superficial branch of the ulnar nerve

Radial nerve

A

This is a description of Froment’s sign, which tests for ulnar nerve palsy. It mainly tests for the function of adductor pollicis. This is supplied by the deep branch of the ulnar nerve. Remember the anterior interosseous branch (of the median nerve), which innervates the flexor pollicis longus (hence causing flexion of the thumb IP joint), branches off more proximally to the wrist.

Ulnar nerve

Origin
C8, T1

Supplies (no muscles in the upper arm)
Flexor carpi ulnaris
Flexor digitorum profundus
Flexor digiti minimi
Abductor digiti minimi
Opponens digiti minimi
Adductor pollicis
Interossei muscle
Third and fourth lumbricals
Palmaris brevis

Path
Posteromedial aspect of upper arm to flexor compartment of forearm, then along the ulnar. Passes beneath the flexor carpi ulnaris muscle, then superficially over the flexor retinaculum into the palm of the hand.

Branches
Branch Supplies
Muscular branch Flexor carpi ulnaris
Medial half of the flexor digitorum profundus
Palmar cutaneous branch (Arises near the middle of the forearm) Skin on the medial part of the palm
Dorsal cutaneous branch Dorsal surface of the medial part of the hand
Superficial branch Cutaneous fibres to the anterior surfaces of the medial one and one-half digits
Deep branch Hypothenar muscles
All the interosseous muscles
Third and fourth lumbricals
Adductor pollicis
Medial head of the flexor pollicis brevis

Effects of injury
Damage at the wrist
Wasting and paralysis of intrinsic hand muscles (claw hand)
Wasting and paralysis of hypothenar muscles
Loss of sensation medial 1 and half fingers
Damage at the elbow
Radial deviation of the wrist
Clawing less in 4th and 5th digits

How well did you know this?
1
Not at all
2
3
4
5
Perfectly
812
Q

A 48 year old lady is admitted with abdominal distension. On examination, she is cachectic and has ascites. Her CA19-9 returns highly elevated. What is the most likely cause?

Metastatic ovarian cancer

Metastatic pancreatic cancer

Metastatic gastric cancer

Metastatic colonic cancer

Pseudomyxoma peritoneii

A

Although not specific CA 19-9 in the context of this history is highly suggestive of pancreatic cancer over the other scenarios.

Pancreatic cancer

  • Adenocarcinoma
    Risk factors: Smoking, diabetes, adenoma, familial adenomatous polyposis
    Mainly occur in the head of the pancreas (70%)
    Spread locally and metastasizes to the liver
    Carcinoma of the pancreas should be differentiated from other periampullary tumours with better prognosis

Clinical features
Weight loss
Painless jaundice
Epigastric discomfort (pain usually due to invasion of the coeliac plexus is a late feature)
Pancreatitis
Trousseau’s sign: migratory superficial thrombophlebitis

Investigations
USS: May miss small lesions
CT Scanning (pancreatic protocol). If unresectable on CT then no further staging needed
PET/CT for those with operable disease on CT alone
ERCP/ MRI for bile duct assessment
Staging laparoscopy to exclude peritoneal disease

Management
Head of pancreas: Whipple’s resection (SE dumping and ulcers). Newer techniques include pylorus preservation and SMA/ SMV resection
Carcinoma body and tail: poor prognosis, distal pancreatectomy, if operable
Usually adjuvent chemotherapy for resectable disease
ERCP and stent for jaundice and palliation
Surgical bypass may be needed for duodenal obstruction

How well did you know this?
1
Not at all
2
3
4
5
Perfectly
813
Q

A term neonate is undergoing an open inguinal herniotomy. During the operation, the operating surgeon identifies a small round yellow nodule within the inguinal canal. What is this most likely to represent?

Ectopic lymphatic tissue

Adrenal rest

Encysted hydrocele of the cord

Yolk sac tumour

Hamartoma

A

Adrenal rests are a very common finding in term neonates (50%) and most regress so that by adulthood the population incidence is 1%. The adrenal glands and gonads both develop from the urogenital ridge. In early embryogenesis, the adrenal cortical tissue can migrate with the descending gonads.

Adrenal gland embryology

First detected at 6 weeks’ gestation, the adrenal cortex is derived from the mesoderm of the posterior abdominal wall. Steroid secretion from the fetal cortex begins shortly thereafter. Adult-type zona glomerulosa and fasciculata are detected in fetal life but make up only a small proportion of the gland, and the zona reticularis is not present at all. The fetal cortex predominates throughout fetal life. The adrenal medulla is of ectodermal origin, arising from neural crest cells that migrate to the medial aspect of the developing cortex.

The fetal adrenal gland is relatively large. At 4 months’ gestation, it is 4 times the size of the kidney; however, at birth, it is a third of the size of the kidney. This occurs because of the rapid regression of the fetal cortex at birth. It disappears almost completely by age 1 year; by age 4-5 years, the permanent adult-type adrenal cortex has fully developed.

Anatomic anomalies of the adrenal gland may occur. Because the development of the adrenals is closely associated with that of the kidneys, agenesis of an adrenal gland is usually associated with ipsilateral agenesis of the kidney, and fused adrenal glands (whereby the 2 glands join across the midline posterior to the aorta) are also associated with a fused kidney.

Adrenal hypoplasia occurs in the following 2 forms: (1) hypoplasia or absence of the fetal cortex with a poorly formed medulla and (2) disorganized fetal cortex and medulla with no permanent cortex present. Adrenal heterotopia describes a normal adrenal gland in an abnormal location, such as within the renal or hepatic capsules. Accessory adrenal tissue (adrenal rests), which is usually comprised only of cortex but seen combined with medulla in some cases, is most commonly located in the broad ligament or spermatic cord but can be found anywhere within the abdomen. Even intracranial adrenal rests have been reported.

How well did you know this?
1
Not at all
2
3
4
5
Perfectly
814
Q

What type of stoma is most likely to be encountered in a 56 year old man undergoing a low anterior resection for carcinoma of the rectum with a colorectal anastomosis?

Loop colostomy

End colostomy

End ileostomy

Loop ileostomy

Caecostomy

A

Colonic resections with an anastomosis below the peritoneal reflection may have an anastomotic leak rate (both clinical and radiological) of up to 15%. Therefore most surgeons will defunction such an anastomosis to reduce the clinical severity of an anastomotic leak. A loop ileostomy will achieve this end point and is relatively easy to reverse. Loop colostomy is less popular in this setting as reversal can compromise the blood supply to the anastomosis.

Abdominal stomas

Stomas may be sited during a range of abdominal procedures and involve bringing the lumen or visceral contents onto the skin. In most cases this applies to the bowel. However, other organs or their contents may be diverted in case of need.

With bowel stomas the type method of construction and to a lesser extent the site will be determined by the contents of the bowel. In practice, small bowel stomas should be spouted so that their irritant contents are not in contact with the skin. Colonic stomas do not need to be spouted as their contents are less irritant.

In the ideal situation the site of the stoma should be marked with the patient prior to surgery. Stoma siting is important as it will ultimately influence the ability of the patient to manage their stoma and also reduce the risk of leakage. Leakage of stoma contents and subsequent maceration of the surrounding skin can rapidly progress into a spiraling loss of control of stoma contents.

Types of stomas
Name of stoma Use Common sites
Gastrostomy
Gastric decompression or fixation
Feeding
Epigastrium
Loop jejunostomy
Seldom used as very high output
May be used following emergency laparotomy with planned early closure
Any location according to need
Percutaneous jejunostomy
Usually performed for feeding purposes and site in the proximal bowel
Usually left upper quadrant
Loop ileostomy
Defunctioning of colon e.g. following rectal cancer surgery
Does not decompress colon (if ileocaecal valve competent)
Usually right iliac fossa
End ileostomy
Usually following complete excision of colon or where ileo-colic anastomosis is not planned
May be used to defunction colon, but reversal is more difficult
Usually right iliac fossa
End colostomy Where a colon is diverted or resected and anastomosis is not primarily achievable or desirable Either left or right iliac fossa
Loop colostomy
To defunction a distal segment of colon
Since both lumens are present the distal lumen acts as a vent
May be located in any region of the abdomen, depending upon colonic segment used
Caecostomy Stoma of last resort where loop colostomy is not possible Right iliac fossa
Mucous fistula
To decompress a distal segment of bowel following colonic division or resection
Where closure of a distal resection margin is not safe or achievable
May be located in any region of the abdomen according to clinical need

How well did you know this?
1
Not at all
2
3
4
5
Perfectly
815
Q

Which of the following drugs does not interfere with the laboratory analysis of serum cortisol levels?

Dexamethasone

Prednisolone

Hydrocortisone IV

Hydrocortisone PO

Hydrocortisone IM

A

Prednisolone and it’s metabolites can chemically mimic cortisol in radio-immunoassay techniques of laboratory analysis.
Dexamethasone can be given as glucorticoid replacement during testing for addisons or adrenal insufficiency as it does not interfere with cortisol levels. For example, if you have a patient with polymyalgia rheumatica and they are on long term prednisolone, you can replace the prednisolone with dexamethasone to undertake a short synacthen test.

Cortisol

Glucocorticoid
Released by zona fasiculata of the adrenal gland
90% protein bound; 10% active
Circadian rhythm: High in the mornings
Negative feedback via ACTH

Actions
Glycogenolysis
Gluconeogenesis
Protein catabolism
Lipolysis
Stress response
Anti-inflammatory
Decrease protein in bones
Increase gastric acid
Increases neutrophils/platelets/red blood cells
Inhibits fibroblastic activity

How well did you know this?
1
Not at all
2
3
4
5
Perfectly
816
Q

A 67 year old man undergoes a carotid endarterectomy and seems to recover well following surgery. When he is reviewed on the ward post operatively he complains that his voice is hoarse. What is the most likely cause?

Damage to the accessory nerve

Damage to the cervical plexus

Damage to the glossopharyngeal nerve

Damage to the hypoglossal nerve

Damage to the vagus

A

Many of these nerves are at risk of injury during carotid surgery. However, only damage to the vagus would account for a hoarse voice.

Vagus nerve

The vagus nerve has mixed functions and supplies the structures from the fourth and sixth pharyngeal arches. It also supplies the fore and midgut sections of the embryonic gut tube. It carries afferent fibres from these areas (viz; pharynx, larynx, oesophagus, stomach, lungs, heart and great vessels). The efferent fibres of the vagus are of two main types. The first are preganglionic parasympathetic fibres distributed to the parasympathetic ganglia that innervate smooth muscle of the innervated organs (such as gut). The second type of efferent fibres have direct skeletal muscle innervation, these are largely to the muscles of the larynx and pharynx.

Origin and course
The vagus arises from the lateral surface of the medulla oblongata by a series of rootlets. It is related to the glossopharyngeal nerve cranially and the accessory nerve caudally. It exits through the jugular foramen and is contained within its own dural sheath alongside the accessory nerve. In the neck it descends vertically in the carotid sheath where it is closely related to the internal and common carotid arteries. It leaves the neck and enters the mediastinum. On the right it passes anterior to the first part of the subclavian artery, on the left it lies in the interval between the common carotid and subclavian arteries.
In the mediastinum both nerves pass postero-inferiorly and reach the posterior surface of the corresponding lung root. These then branch into both lungs. At the inferior end of the mediastinum these plexuses reunite to form the formal vagal trunks that pass through the oesophageal hiatus and into the abdomen. The anterior and posterior vagal trunks are formal nerve fibres these then splay out once again sending fibres over the stomach and posteriorly to the coeliac plexus. Branches pass to the liver, spleen and kidney.

Communications and branches
Communication Details
Superior ganglion Located in jugular foramen
Communicates with the superior cervical sympathetic ganglion, accessory nerve
Two branches; meningeal and auricular (the latter may give rise to vagal stimulation following instrumentation of the external auditory meatus)
Inferior ganglion Communicates with the superior cervical sympathetic ganglion, hypoglossal nerve and loop between first and second cervical ventral rami
Two branches; pharyngeal (supplies pharyngeal muscles) and superior laryngeal nerve (inferomedially- deep to both carotid arteries)

Branches in the neck
Branch Detail
Superior and inferior cervical cardiac branches Arise at various points and descend into thorax
On the right these pass posterior to the subclavian artery
On the left the superior branch passes between the arch of the aorta and the trachea to connect with the deep cardiac plexus. The inferior branch descends with the vagus itself.
Right recurrent laryngeal nerve Arises from vagus anterior to the first part of the subclavian artery, hooks under it, and ascends superomedially. It passes close to the common carotid and finally the inferior thyroid artery to insert into the larynx

Branches in the thorax
Branch Details
Left recurrent laryngeal nerve Arises from the vagus on the aortic arch. It hooks around the inferior surface of the arch, posterior to the ligamentum arteriosum and passes upwards through the superior mediastinum and lower part of the neck. It lies in the groove between oesophagus and trachea (supplies both). It passes with the inferior thyroid artery and inserts into the larynx.
Thoracic and cardiac branches There are extensive branches to both the heart and lung roots. These pass throughout both these viscera. The fibres reunite distally prior to passing into the abdomen.

Abdominal branches
After entry into the abdominal cavity the nerves branch extensively. In previous years the extensive network of the distal branches (nerves of Laterjet) over the surface of the distal stomach were important for the operation of highly selective vagotomy. The use of modern PPI’s has reduced the need for such highly selective procedures. Branches pass to the coeliac axis and alongside the vessels to supply the spleen, liver and kidney.

How well did you know this?
1
Not at all
2
3
4
5
Perfectly
817
Q

Which of the following is not a content of the posterior triangle of the neck?

Spinal accessory nerve

Phrenic nerve

External jugular vein

Occipital lymph nodes

Internal jugular vein

A

The IJV does not lie in the posterior triangle. However, the terminal branches of the external jugular vein do.

Posterior triangle of the neck

Boundaries
Apex Sternocleidomastoid and the Trapezius muscles at the Occipital bone
Anterior Posterior border of the Sternocleidomastoid
Posterior Anterior border of the Trapezius
Base Middle third of the clavicle

Contents
Nerves
Accessory nerve
Phrenic nerve
Three trunks of the brachial plexus
Branches of the cervical plexus: Supraclavicular nerve, transverse cervical nerve, great auricular nerve, lesser occipital nerve
Vessels
External jugular vein
Subclavian artery
Muscles
Inferior belly of omohyoid
Scalene
Lymph nodes
Supraclavicular
Occipital

How well did you know this?
1
Not at all
2
3
4
5
Perfectly
818
Q

A 40-year-old man presents with pain in his lower back and ‘sciatica’ for the past three days. He describes bending down to pick up a washing machine when he felt ‘something go’. He now has severe pain radiating from his back down the right leg. On examination, there is paraesthesia over the anterior aspect of the right knee and the medial aspect of his lower leg. Power is intact and the right knee reflex is diminished. The femoral stretch test is positive on the right side. Which nerve or nerve root is most likely to be affected?

Common peroneal nerve

Lateral cutaneous nerve of the thigh

L5

L3

L1

A

L3

Prolapsed disc

A prolapsed lumbar disc usually produces clear dermatomal leg pain associated with neurological deficits.

Features
Leg pain usually worse than back
Pain often worse when sitting

The table below demonstrates the expected features according to the level of compression:

L3 nerve root compression Sensory loss over anterior thigh/knee
Weak quadriceps
Reduced knee reflex
Positive femoral stretch test
L5 nerve root compression Sensory loss dorsum of foot
Weakness in foot and big toe dorsiflexion
Reflexes intact
Positive sciatic nerve stretch test
S1 nerve root compression Sensory loss posterolateral aspect of leg and lateral aspect of foot
Weakness in plantar flexion of foot
Reduced ankle reflex
Positive sciatic nerve stretch test

Management
Similar to that of other musculoskeletal lower back pain: analgesia, physiotherapy, exercises
Persistent symptoms, muscular weakness, bladder or bowel dysfunction are indications for urgent MRI scanning to delineate the disease extent to allow surgical planning
Plain spinal x-rays have no useful role in establishing the extent of disc disease

How well did you know this?
1
Not at all
2
3
4
5
Perfectly
819
Q

The oesophagus is constricted at the following levels apart from:

Cricoid cartilage

Arch of the aorta

Lower oesophageal sphincter

Left main stem bronchus

Diaphragmatic hiatus

A

Constrictions of the oesophagus : ABCD
A- Arch of the Aorta
B- Left main Bronchus
C- Cricoid Cartilage
D- Diaphragmatic Hiatus

The oesophagus is not constricted at the level of the lower oesophageal sphincter.

Oesophagus

  • 25cm long
    Starts at C6 vertebra, pierces diaphragm at T10 and ends at T11
    Squamous epithelium

Constrictions of the oesophagus
Structure Distance from incisors
Cricoid cartilage 15cm
Arch of the Aorta 22.5cm
Left principal bronchus 27cm
Diaphragmatic hiatus 40cm

Relations
Anteriorly
Trachea to T4
Recurrent laryngeal nerve
Left bronchus, Left atrium
Diaphragm
Posteriorly
Thoracic duct to left at T5
Hemiazygos to the left T8
Descending aorta
First 2 intercostal branches of aorta
Left
Thoracic duct
Left subclavian artery
Right
Azygos vein

Arterial, venous and lymphatic drainage of the oesophagus
Artery Vein Lymphatics Muscularis externa
Upper third Inferior thyroid Inferior thyroid Deep cervical Striated muscle
Mid third Aortic branches Azygos branches Mediastinal Smooth & striated muscle
Lower third Left gastric Left gastric Gastric Smooth muscle

Nerve supply
Upper half is supplied by recurrent laryngeal nerve
Lower half by oesophageal plexus (vagus)

Histology
Mucosa :Non-keratinized stratified squamous epithelium
Submucosa: glandular tissue
Muscularis externa (muscularis): composition varies. See table
Adventitia

How well did you know this?
1
Not at all
2
3
4
5
Perfectly
820
Q

A 43 year old lady is recovering on the intensive care unit following a Whipples procedure. She has a central venous line in situ. Which of the following will lead to the y descent on the waveform trace ?

Ventricular contraction

Emptying of the right atrium

Emptying of the right ventricle

Opening of the pulmonary valve

Cardiac tamponade

A

JVP
3 Upward deflections and 2 downward deflections

Upward deflections
a wave = atrial contraction
c wave = ventricular contraction
v wave = atrial venous filling

Downward deflections
x wave = atrium relaxes and tricuspid valve moves down
y wave = ventricular filling
The y descent represents the emptying of the atrium and the filling of the right ventricle.

Cardiac physiology

  • The heart has four chambers ejecting blood into both low pressure and high pressure systems.
    The pumps generate pressures of between 0-25mmHg on the right side and 0-120 mmHg on the left.
    At rest diastole comprises 2/3 of the cardiac cycle.
    The product of the frequency of heart rate and stroke volume combine to give the cardiac output which is typically 5-6L per minute.

Detailed descriptions of the various waveforms are often not a feature of MRCS A (although they are on the syllabus). However, they are a very popular topic for surgical physiology in the MRCS B exam.

Electrical properties
Intrinsic myogenic rhythm within cardiac myocytes means that even the denervated heart is capable of contraction.
In the normal situation the cardiac impulse is generated in the sino atrial node in the right atrium and conveyed to the ventricles via the atrioventricular node.
The sino atrial node is also capable of spontaneous discharge and in the absence of background vagal tone will typically discharge around 100x per minute. Hence the higher resting heart rate found in cardiac transplant cases. In the SA and AV nodes the resting membrane potential is lower than in surrounding cardiac cells and will slowly depolarise from -70mV to around -50mV at which point an action potential is generated.
Differences in the depolarisation slopes between SA and AV nodes help to explain why the SA node will depolarise first. The cells have a refractory period during which they cannot be re-stimulated and this period allows for adequate ventricular filling. In pathological tachycardic states this time period is overridden and inadequate ventricular filling may then occur, cardiac output falls and syncope may ensue.

Parasympathetic fibres project to the heart via the vagus and will release acetylcholine. Sympathetic fibres release nor adrenaline and circulating adrenaline comes from the adrenal medulla. Noradrenaline binds to β 1 receptors in the SA node and increases the rate of pacemaker potential depolarisation.

Cardiac cycle

Mid diastole: AV valves open. Ventricles hold 80% of final volume. Outflow valves shut. Aortic pressure is high.

Late diastole: Atria contract. Ventricles receive 20% to complete filling. Typical end diastolic volume 130-160ml.

Early systole: AV valves shut. Ventricular pressure rises. Isovolumetric ventricular contraction. AV Valves bulge into atria (c-wave). Aortic and pulmonary pressure exceeded- blood is ejected. Shortening of ventricles pulls atria downwards and drops intra atrial pressure (x-descent).

Late systole: Ventricular muscles relax and ventricular pressures drop. Although ventricular pressure drops the aortic pressure remains constant owing to peripheral vascular resistance and elastic property of the aorta. Brief period of retrograde flow that occurs in aortic recoil shuts the aortic valve. Ventricles will contain 60ml end systolic volume. The average stroke volume is 70ml (i.e. Volume ejected).

Early diastole: All valves are closed. Isovolumetric ventricular relaxation occurs. Pressure wave associated with closure of the aortic valve increases aortic pressure. The pressure dip before this rise can be seen on arterial waveforms and is called the incisura. During systole the atrial pressure increases such that it is now above zero (v- wave). Eventually atrial pressure exceed ventricular pressure and AV valves open - atria empty passively into ventricles and atrial pressure falls (y -descent )

The negative atrial pressures are of clinical importance as they can allow air embolization to occur if the neck veins are exposed to air. This patient positioning is important in head and neck surgery to avoid this occurrence if veins are inadvertently cut, or during CVP line insertion.

Mechanical properties
Preload = end diastolic volume
Afterload = aortic pressure

It is important to understand the principles of Laplace’s law in surgery.
It states that for hollow organs with a circular cross section, the total circumferential wall tension depends upon the circumference of the wall, multiplied by the thickness of the wall and on the wall tension.
The total luminal pressure depends upon the cross sectional area of the lumen and the transmural pressure. Transmural pressure is the internal pressure minus external pressure and at equilibrium the total pressure must counterbalance each other.
In terms of cardiac physiology the law explains that the rise in ventricular pressure that occurs during the ejection phase is due to physical change in heart size. It also explains why a dilated diseased heart will have impaired systolic function.

Starlings law
Increase in end diastolic volume will produce larger stroke volume.
This occurs up to a point beyond which cardiac fibres are excessively stretched and stroke volume will fall once more. It is important for the regulation of cardiac output in cardiac transplant patients who need to increase their cardiac output.

Baroreceptor reflexes
Baroreceptors located in aortic arch and carotid sinus.
Aortic baroreceptor impulses travel via the vagus and from the carotid via the glossopharyngeal nerve.
They are stimulated by arterial stretch.
Even at normal blood pressures they are tonically active.
Increase in baroreceptor discharge causes:

*Increased parasympathetic discharge to the SA node.
*Decreased sympathetic discharge to ventricular muscle causing decreased contractility and fall in stroke volume.
*Decreased sympathetic discharge to venous system causing increased compliance.
*Decreased peripheral arterial vascular resistance

Atrial stretch receptors
Located in atria at junction between pulmonary veins and vena cava.
Stimulated by atrial stretch and are thus low pressure sensors.
Increased blood volume will cause increased parasympathetic activity.
Very rapid infusion of blood will result in increase in heart rate mediated via atrial receptors: the Bainbridge reflex.
Decreases in receptor stimulation results in increased sympathetic activity this will decrease renal blood flow-decreases GFR-decreases urinary sodium excretion-renin secretion by juxtaglomerular apparatus-Increase in angiotensin II.
Increased atrial stretch will also result in increased release of atrial natriuretic peptide.

How well did you know this?
1
Not at all
2
3
4
5
Perfectly
821
Q

A 12 year old male is investigated for swallowing difficulties and is found to have dysphagia lusoria. Which of the diagnostic modalities described below is most likely to demonstrate the cause?

CT angiogram

Upper GI endoscopy

Barium swallow

Capsule endoscopy

Oesophageal manometry

A

Dysphagia lusoria is caused by compression of the esophagus from any of several congenital vascular abnormalities. The vascular abnormality is usually an aberrant right subclavian artery arising from the left side of the aortic arch, a double aortic arch, or a right aortic arch with left ligamentum arteriosum. The dysphagia may develop in childhood or later in life as a result of arteriosclerotic changes in the aberrant vessel.
Barium swallow shows the extrinsic compression, but arteriography is necessary for absolute diagnosis.

Dysphagia

Causes of dysphagia

Extrinsic
Mediastinal masses
Cervical spondylosis
Oesophageal wall
Achalasia
Diffuse oesophageal spasm
Hypertensive lower oesophageal sphincter
Intrinsic
Tumours
Strictures
Oesophageal web
Schatzki rings
Neurological
CVA
Parkinson’s disease
Multiple Sclerosis
Brainstem pathology
Myasthenia Gravis

Investigation
All patients require an upper GI endoscopy unless there are compelling reasons for this not to be performed. Motility disorders may be best appreciated by undertaking fluoroscopic swallowing studies.

A full blood count should be performed.

Ambulatory oesophageal pH and manometry studies will be required to evaluate conditions such as achalasia and patients with GORD being considered for fundoplication surgery.

How well did you know this?
1
Not at all
2
3
4
5
Perfectly
822
Q

A patient is brought to the emergency department following a motor vehicle accident. He is unconscious and has a deep scalp laceration. His heart rate is 120/min, blood pressure is 80/40 mmHg, and respiratory rate is 35/min. Despite rapid administration of 2 litres of Hartmans solution, the patient’s vital signs do not change significantly. The injury likely to explain this patient’s hypotension is:

Epidural haematoma

Sub dural haematoma

Intra parenchymal brain haemorrhage

Base of skull fracture

None of the above

A

Raised intracranial pressure (Cushing response)
Hypertension
Bradycardia
Respiratory depression
In the patient described, hypotension and tachycardia should not be uncritically attributed to the head injury, since these findings in the setting of blunt trauma are suggestive of serious thoracic, abdominal, or pelvic hemorrhage. When cardiovascular collapse occurs as a result of rising intracranial pressure, it is generally accompanied by hypertension, bradycardia, and respiratory depression.

How well did you know this?
1
Not at all
2
3
4
5
Perfectly
823
Q

Which of the structures listed below accompanies the aorta as it traverses the aortic hiatus?

Oesophagus

Thoracic duct

Vagal trunks

Right phrenic nerve

Left phrenic nerve

A

The aorta is accompanied by the thoracic duct as it traverses the aortic hiatus. The vagal trunks accompany the oesophagus which passes through the muscular part of the diaphragm on the right. The right phrenic nerve accompanies the IVC as it passes through the caval opening. The left phrenic nerve passes through the muscular part of the diaphragm anterior to the central tendon on the left.

Abdominal aorta

Abdominal aortic topography
Origin T12
Termination L4
Posterior relations L1-L4 Vertebral bodies
Anterior relations Lesser omentum
Liver
Left renal vein
Inferior mesenteric vein
Third part of duodenum
Pancreas
Parietal peritoneum
Peritoneal cavity
Right lateral relations Right crus of the diaphragm
Cisterna chyli
IVC (becomes posterior distally)
Left lateral relations 4th part of duodenum
Duodenal-jejunal flexure
Left sympathetic trunk

How well did you know this?
1
Not at all
2
3
4
5
Perfectly
824
Q

A 48 year old lady with end stage renal failure receives a cadaveric renal transplant. The organ is ABO group matched only. On completion of the vascular anastomoses the surgeons remove the clamps. Over the course of the next twelve minutes the donated kidney becomes dusky and swollen and appears non viable. Which of the following is the most likely process that has caused this event?

IgG anti HLA Class I antibodies in the recipient

IgM anti HLA Class I antibodies in the recipient

IgG anti HLA Class I antibodies from the donor

IgM anti HLA Class I antibodies from the donor

IgM anti HLA Class II antibodies from the recipient

A

Episodes of hyperacute rejection are typically due to preformed antibodies. ABO mismatch is the best example. However, IgG anti HLA Class I antibodies are another potential cause. These events are now seen less commonly because the cross matching process generally takes this possibility into account.

Organ Transplant

A number of different organ and tissue transplants are now available. In many cases an allograft is performed, where an organ is transplanted from one individual to another. Allografts will elicit an immune response and this is one of the main reasons for organ rejection.

Graft rejection occurs because allografts have allelic differences at genes that code immunohistocompatability complex genes. The main antigens that give rise to rejection are:
ABO blood group
Human leucocyte antigens (HLA)
Minor histocompatability antigens

ABO Matching
ABO incompatibility will result in early organ rejection (hyperacute) because of pre existing antibodies to other groups. Group O donors can give organs to any type of ABO recipient whereas group AB donor can only donate to AB recipient.

HLA System
The four most important HLA alleles are:

HLA A
HLA B
HLA C
HLA DR

An ideal organ match would be one in which all 8 alleles are matched (remember 2 from each parent, four each = 8 alleles). Modern immunosuppressive regimes help to manage the potential rejection due to HLA mismatching. However, the greater the number of mismatches the worse the long term outcome will be. T lymphocytes will recognise antigens bound to HLA molecules and will then become activated. Clonal expansion then occurs with a response directed against that antigen.

Types of organ rejection
Hyperacute. This occurs immediately through presence of pre formed antibodies (such as ABO incompatibility).
Acute. Occurs during the first 6 months and is usually T cell mediated. Usually tissue infiltrates and vascular lesions.
Chronic. Occurs after the first 6 months. Vascular changes predominate.

Hyperacute
Renal transplants at greatest risk and liver transplants at least risk. Although ABO incompatibility and HLA Class I incompatible transplants will all fare worse in long term.

Acute
All organs may undergo acute rejection. Mononuclear cell infiltrates predominate. All types of transplanted organ are susceptible and it may occur in up to 50% cases.

Chronic
Again all transplants with HLA mismatch may suffer this fate. Previous acute rejections and other immunosensitising events all increase the risk. Vascular changes are most prominent with myointimal proliferation leading to organ ischaemia. Organ specific changes are also seen such as loss of acinar cells in pancreas transplants and rapidly progressive coronary artery disease in cardiac transplants.

Surgical overview-Renal transplantation
A brief overview of the steps involved in renal transplantation is given.
Patients with end stage renal failure who are dialysis dependent or likely to become so in the immediate future are considered for transplant. Exclusion criteria include; active malignancy, old age (due to limited organ availability). Patients are medically optimised.
Donor kidneys, these may be taken from live related donors and close family, members may have less HLA mismatch than members of the general population. Laparoscopic donor nephrectomy further minimises the operative morbidity for the donor. Other organs are typically taken from brain dead or dying patients who have a cardiac arrest and in whom resuscitation is futile. The key event is to minimise the warm ischaemic time in the donor phase.

The kidney once removed is usually prepared on the bench in theatre by the transplant surgeon immediately prior to implantation and factors such as accessory renal arteries and vessel length are assessed and managed.

For first time recipients the operation is performed under general anaesthesia. A Rutherford-Morison incision is made on the preferred side. This provides excellent extraperitoneal access to the iliac vessels. The external iliac artery and vein are dissected out and following systemic heparinisation are cross clamped. The vein and artery are anastamosed to the iliacs and the clamps removed. The ureter is then implanted into the bladder and a stent is usually placed to maintain patency. The wounds are then closed and the patient recovered from surgery.

In the immediate phase a common problem encountered in cadaveric kidneys is acute tubular necrosis and this tends to resolve.

Graft survival times from cadaveric donors are typically of the order of 9 years and monozygotic twin transplant (live donor) may survive as long as 25 years.

How well did you know this?
1
Not at all
2
3
4
5
Perfectly
825
Q

A 22 year old woman presents with macroscopic haematuria. She is sexually active. She is known to have renal calculi and had a berry aneurysm clipped. What is the most likely cause?

Interstitial nephritis

Membranous glomerulonephritis

Renal vein thrombosis

Endometriosis

Adult polycystic kidney disease

A

APKD is associated with liver cysts (70%), berry aneurysms (25%) and pancreatic cysts (10%). Patients may have a renal mass, hypertension, renal calculi and macroscopic haematuria.

Haematuria

Causes of haematuria

Trauma
Injury to renal tract
Renal trauma commonly due to blunt injury (others penetrating injuries)
Ureter trauma rare: iatrogenic
Bladder trauma: due to RTA or pelvic fractures
Infection
Remember TB
Malignancy
Renal cell carcinoma (remember paraneoplastic syndromes): painful or painless
Urothelial malignancies: 90% are transitional cell carcinoma, can occur anywhere along the urinary tract. Painless haematuria.
Squamous cell carcinoma and adenocarcinoma: rare bladder tumours
Prostate cancer
Penile cancers: SCC
Renal disease
Glomerulonephritis
Stones
Microscopic haematuria common
Structural abnormalities
Benign prostatic hyperplasia (BPH) causes haematuria due to hypervascularity of the prostate gland
Cystic renal lesions e.g. polycystic kidney disease
Vascular malformations
Renal vein thrombosis due to renal cell carcinoma
Coagulopathy
Causes bleeding of underlying lesions
Drugs
Cause tubular necrosis or interstitial nephritis: aminoglycosides, chemotherapy
Interstitial nephritis: penicillin, sulphonamides, and NSAIDs
Anticoagulants
Benign
Exercise
Gynaecological
Endometriosis: flank pain, dysuria, and haematuria that is cyclical
Iatrogenic
Catheterisation
Radiotherapy; cystitis, severe haemorrhage, bladder necrosis
Pseudohaematuria For example following consumption of beetroot

References
Http://bestpractice.bmj.com/best-practice/monograph/316/overview/aetiology.html

How well did you know this?
1
Not at all
2
3
4
5
Perfectly
826
Q

Which of the following nerves conveys sensory information from the laryngeal mucosa?

Glossopharyngeal

Laryngeal branches of the vagus

Ansa cervicalis

Laryngeal branches of the trigeminal

None of the above

A

The laryngeal branches of the vagus supply sensory information from the larynx.

Larynx

The larynx lies in the anterior part of the neck at the levels of C3 to C6 vertebral bodies. The laryngeal skeleton consists of a number of cartilagenous segments. Three of these are paired; arytenoid, corniculate and cuneiform. Three are single; thyroid, cricoid and epiglottic. The cricoid cartilage forms a complete ring (the only one to do so).
The laryngeal cavity extends from the laryngeal inlet to the level of the inferior border of the cricoid cartilage.

Divisions of the laryngeal cavity
Laryngeal vestibule Superior to the vestibular folds
Laryngeal ventricle Lies between vestibular folds and superior to the vocal cords
Infraglottic cavity Extends from vocal cords to inferior border of the cricoid cartilage

The vocal folds (true vocal cords) control sound production. The apex of each fold projects medially into the laryngeal cavity. Each vocal fold includes:
Vocal ligament
Vocalis muscle (most medial part of thyroarytenoid muscle)
The glottis is composed of the vocal folds, processes and rima glottidis. The rima glottidis is the narrowest potential site within the larynx, as the vocal cords may be completely opposed, forming a complete barrier.

Muscles of the larynx
Muscle Origin Insertion Innervation Action
Posterior cricoarytenoid Posterior aspect of lamina of cricoid Muscular process of arytenoid Recurrent Laryngeal Abducts vocal fold
Lateral cricoarytenoid Arch of cricoid Muscular process of arytenoid Recurrent laryngeal Adducts vocal fold
Thyroarytenoid Posterior aspect of thyroid cartilage Muscular process of arytenoid Recurrent laryngeal Relaxes vocal fold
Transverse and oblique arytenoids Arytenoid cartilage Contralateral arytenoid Recurrent laryngeal Closure of intercartilagenous part of the rima glottidis
Vocalis Depression between lamina of thyroid cartilage Vocal ligament and vocal process of arytenoid cartilage Recurrent laryngeal Relaxes posterior vocal ligament, tenses anterior part
Cricothyroid Anterolateral part of cricoid Inferior margin and horn of thyroid cartilage External laryngeal Tenses vocal fold

Blood supply
Arterial supply is via the laryngeal arteries, branches of the superior and inferior thyroid arteries. The superior laryngeal artery is closely related to the internal laryngeal nerve. The inferior laryngeal artery is related to the inferior laryngeal nerve. Venous drainage is via superior and inferior laryngeal veins, the former draining into the superior thyroid vein and the latter draining into the middle thyroid vein, or thyroid venous plexus.

Lymphatic drainage
The vocal cords have no lymphatic drainage and this site acts as a lymphatic watershed.
Supraglottic part Upper deep cervical nodes
Subglottic part Prelaryngeal and pretracheal nodes and inferior deep cervical nodes
The aryepiglottic fold and vestibular folds have a dense plexus of lymphatics associated with them and malignancies at these sites have a greater propensity for nodal metastasis.

How well did you know this?
1
Not at all
2
3
4
5
Perfectly
827
Q

Which of the metastatic bone tumours described below is at the greatest risk of pathological fracture ?

Proximal humeral lesion from a prostate cancer

Vertebral body lesions from a prostate cancer

Peritrochanteric lesion from a carcinoma of the breast

Proximal humeral lesion from a carcinoma of the breast

Peritrochanteric lesion from a prostate cancer

A

Peritrochanteric lesions have the greatest risks of fracture (due to loading). The lesions from breast cancer are usually lytic and therefore at higher risk rather than the sclerotic lesions from prostate cancer.

Metastatic bone disease- risk of fracture

Metastatic bone tumours may be described as blastic, lytic or mixed. Osteoblastic metastatic disease has the lowest risk of spontaneous fracture when compared to osteolytic lesions of a similar size.
Lesions affecting the peritrochanteric region are most prone to spontaneous fracture (because of loading forces at that site).
The factors are incorporated into the Mirel Scoring system to stratify the risk of spontaneous fracture for bone metastasis of varying types.

Mirel Scoring system

Score points Site Radiographic appearance Width of bone involved Pain
1 Upper extremity Blastic Less than 1/3 Mild
2 Lower extremity Mixed 1/3 to 2/3 Moderate
3 Peritrochanteric Lytic More than 2/3 Aggravated by function

Depending upon the score the treatment should be as follows:

Score Risk of fracture Treatment
9 or greater Impending (33%) Prophylactic fixation
8 Borderline Consider fixation
7 or less Not impending (4%) Non operative management

How well did you know this?
1
Not at all
2
3
4
5
Perfectly
828
Q

Which of the following is not a direct branch of the facial nerve?

Greater petrosal nerve

Nerve to stapedius

Auriculotemporal

Chorda tympani

Buccal branch of facial nerve

A

The auriculotemporal nerve is a direct branch of the mandibular nerve.
Other branches of the mandibular nerve include:
Lingual
Inferior alveolar
Nerve to the mylohyoid
Mental

Facial nerve

The facial nerve is the main nerve supplying the structures of the second embryonic branchial arch. It is predominantly an efferent nerve to the muscles of facial expression, digastric muscle and also to many glandular structures. It contains a few afferent fibres which originate in the cells of its genicular ganglion and are concerned with taste.

Supply - ‘face, ear, taste, tear’
Face: muscles of facial expression
Ear: nerve to stapedius
Taste: supplies anterior two-thirds of tongue
Tear: parasympathetic fibres to lacrimal glands, also salivary glands

Path
Subarachnoid path
Origin: motor- pons, sensory- nervus intermedius
Pass through the petrous temporal bone into the internal auditory meatus with the vestibulocochlear nerve. Here they combine to become the facial nerve.

Facial canal path
The canal passes superior to the vestibule of the inner ear
At the medial aspect of the middle ear, it becomes wider and contains the geniculate ganglion.
- 3 branches:
1. greater petrosal nerve
2. nerve to stapedius
3. chorda tympani

Stylomastoid foramen
Passes through the stylomastoid foramen (tympanic cavity anterior and mastoid antrum posteriorly)
Posterior auricular nerve and branch to posterior belly of digastric and stylohyoid muscle

Face
Enters parotid gland and divides into 5 branches:
Temporal branch
Zygomatic branch
Buccal branch
Marginal mandibular branch
Cervical branch

How well did you know this?
1
Not at all
2
3
4
5
Perfectly
829
Q

Which of the following statements relating to the tympanic membrane is false?

The umbo marks the point of attachment of the handle of the malleus to the tympanic membrane

The lateral aspect of the tympanic membrane is lined by stratified squamous epithelium

The chorda tympani nerve runs medial to the pars tensa

The medial aspect of the tympanic membrane is lined by mucous membrane

The tympanic membrane is approximately 1cm in diameter

A

The chorda tympani runs medially to the pars flaccida. Ear- anatomy

The ear is composed of three anatomically distinct regions.

External ear
Auricle is composed of elastic cartilage covered by skin. The lobule has no cartilage and contains fat and fibrous tissue.

External auditory meatus is approximately 2.5cm long.
Lateral third of the external auditory meatus is cartilaginous and the medial two thirds is bony.

The region is innervated by the greater auricular nerve. The auriculotemporal branch of the trigeminal nerve supplies most of the external auditory meatus and the lateral surface of the auricle.

Middle ear
Space between the tympanic membrane and cochlea. The aditus leads to the mastoid air cells is the route through which middle ear infections may cause mastoiditis. Anteriorly the eustacian tube connects the middle ear to the naso pharynx.
The tympanic membrane consists of:
Outer layer of stratified squamous epithelium.
Middle layer of fibrous tissue.
Inner layer of mucous membrane continuous with the middle ear.
The tympanic membrane is approximately 1cm in diameter.
The chorda tympani nerve passes on the medial side of the pars flaccida.

The middle ear is innervated by the glossopharyngeal nerve and pain may radiate to the middle ear following tonsillectomy.

Ossicles
Malleus attaches to the tympanic membrane (the Umbo).
Malleus articulates with the incus (synovial joint).
Incus attaches to stapes (another synovial joint).

Internal ear
Cochlea, semi circular canals and vestibule

Organ of corti is the sense organ of hearing and is located on the inside of the cochlear duct on the basilar membrane.

Vestibule accommodates the utricule and the saccule. These structures contain endolymph and are surrounded by perilymph within the vestibule.

The semicircular canals lie at various angles to the petrous temporal bone. All share a common opening into the vestibule.

How well did you know this?
1
Not at all
2
3
4
5
Perfectly
830
Q

A 22 year old lady has a long history of severe perianal Crohns disease with multiple fistulae. She is keen to avoid a stoma. However, she has progressive disease and multiple episodes of rectal bleeding. A colonoscopy shows rectal disease only and a small bowel study shows no involvement with Crohns. What is the best operative strategy?

Abdomino perineal excision of the colon and rectum

Proctectomy and end stoma

Pan proctocolectomy and ileoanal pouch

Loop colostomy alone

Sub total colectomy

A

Crohns disease is a contra indication to having an ileo-anal pouch as its associated with very poor pouch function and significant complications.
Whilst the patient wishes to avoid a stoma, that’s inevitable here.

Surgery for inflammatory bowel disease

Patients with inflammatory bowel disease (UC and Crohns) frequently present in surgical practice. Ulcerative colitis may be cured by surgical resection (Proctocolectomy), this is not the case in Crohns disease which may recur and affect other areas of the gastrointestinal tract.

Ulcerative colitis
Elective indications for surgery include disease that is requiring maximal therapy, or prolonged courses of steroids.
Longstanding UC is associated with a risk of malignant transformation. Dysplastic transformation of the colonic epithelium with associated mass lesions is an absolute indication for a proctocolectomy.
Emergency presentations of poorly controlled colitis that fails to respond to medical therapy should usually be managed with a sub total colectomy. Excision of the rectum is a procedure with a higher morbidity and is not generally performed in the emergency setting. An end ileostomy is usually created and the rectum either stapled off and left in situ, or, if the bowel is very oedematous, may be brought to the surface as a mucous fistula.
Patients with IBD have a high incidence of DVT and appropriate thromboprophylaxis is mandatory.
Restorative options in UC include an ileoanal pouch. This procedure can only be performed whilst the rectum is in situ and cannot usually be undertaken as a delayed procedure following proctectomy.
Ileoanal pouch complications include, anastomotic dehiscence, pouchitis and poor physiological function with seepage and soiling.

Crohns disease
Surgical resection of Crohns disease does not equate with cure, but may produce substantial symptomatic improvement.
Indications for surgery include complications such as fistulae, abscess formation and strictures.
Extensive small bowel resections may result in short bowel syndrome and localised stricturoplasty may allow preservation of intestinal length.
Staging of Crohns will usually involve colonoscopy and a small bowel study (e.g. MRI enteroclysis).
Complex perianal fistulae are best managed with long term draining seton sutures, complex attempts at fistula closure e.g. advancement flaps, may be complicated by non healing and fistula recurrence.
Severe perianal and / or rectal Crohns may require proctectomy. Ileoanal pouch reconstruction in Crohns carries a high risk of fistula formation and pouch failure and is not recommended.
Terminal ileal Crohns remains the commonest disease site and these patients may be treated with limited ileocaecal resections.
Terminal ileal Crohns may affect enterohepatic bile salt recycling and increase the risk of gallstones.

How well did you know this?
1
Not at all
2
3
4
5
Perfectly
831
Q

A 58 year old man undergoes a difficult colonoscopy for assessment of a caecal cancer. 48 hours after the procedure he is admitted with septicaemia. His abdomen is soft and non tender. Blood cultures grow gram positive cocci. What is the most likely underlying organism?

Staphylococcus epidermidis

Streptococcus pyogenes

Streptococcus bovis

Clostridium difficle

Bacteroides fragilis

A

Streptococcus bovis septicaemia is associated with carcinoma of the colon. It can also cause endocarditis. Streptococcus bovis is a gram positive organism. In most cases, its portal of entry is via the GI tract and for that reason its typically associated with large bowel malignancy.

Surgical Microbiology

An extensive topic so an overview is given here. Organisms causing common surgical infections are reasonable topics in the examination. However, microbiology is less rigorously tested than anatomy, for example.

Common organisms

Staphylococcus aureus
Facultative anaerobe
Gram positive coccus
Haemolysis on blood agar plates
Catalase positive
20% population are long term carriers
Exo and entero toxin may result in toxic shock syndrome and gastroenteritis respectively
Ideally treated with penicillin although many strains now resistant through beta Lactamase production. In the UK less than 5% of isolates are sensitive to penicillin.
Resistance to methicillin (and other antibiotics) is mediated by the mec operon , essentially penicillin binding protein is altered and resistance to this class of antibiotics ensues
Common cause of cutaneous infections and abscesses

Streptococcus pyogenes
Gram positive, forms chain like colonies, Lancefield Group A Streptococcus
Produces beta haemolysis on blood agar plates
Rarely part of normal skin microflora
Catalase negative
Releases a number of proteins/ virulence factors into host including hyaluronidase, streptokinase which allow rapid tissue destruction
Releases superantigens such as pyogenic exotoxin A which results in scarlet fever
Remains sensitive to penicillin, macrolides may be used as an alternative.

Escherichia coli
Gram negative rod
Facultative anaerobe, non sporing
Wide range of subtypes and some are normal gut commensals
Some subtypes such as 0157 may produce lethal toxins resulting in haemolytic-uraemic syndrome
Enterotoxigenic E-Coli produces an enterotoxin (ST enterotoxin) that results in large volume fluid secretion into the gut lumen (Via cGMP activation)
Enteropathogenic E-Coli binds to intestinal cells and cause structural damage, this coupled with a moderate (or in case of enteroinvasive E-Coli significant) invasive component produces enteritis and large volume diarrhoea together with fever.
They are resistant to many antibiotics used to treat gram positive infections and acquire resistance rapidly and are recognised as producing beta lactamases

Campylobacter jejuni
Curved, gram negative, non sporulating bacteria
One of the commonest causes of diarrhoea worldwide
Produces enteritis which is often diffuse and blood may be passed
Remains a differential for right iliac fossa pain with diarrhoea
Self limiting infection so antibiotics are not usually advised. However, the quinolones are often rapidly effective.

Helicobacter pylori
Gram negative, helix shaped rod, microaerophillic
Produces hydrogenase that can derive energy from hydrogen released by intestinal bacteria
Flagellated and mobile
Those carrying the cag A gene may cause ulcers
It secretes urease that breaks down gastric urea> Carbon dioxide and ammonia> ammonium>bicarbonate (simplified!) The bicarbonate can neutralise the gastric acid.
Usually colonises the gastric antrum and irritates resulting in increased gastrin release and higher levels of gastric acid. These patients will develop duodenal ulcers. In those with more diffuse H-Pylori infection gastric acid levels are lower and ulcers develop by local tissue damage from H-Pylori- these patients get gastric ulcers.
Diagnosis may be made by serology (approx. 75% sensitive). Biopsy urease test during endoscopy probably the most sensitive.
In patients who are colonised 10-20% risk of peptic ulcer, 1-2% risk gastric cancer, <1% risk MALT lymphoma.

How well did you know this?
1
Not at all
2
3
4
5
Perfectly
832
Q

Which of these substances is not released from the islets of Langerhans?

Pancreatic polypeptide

Glucagon

Secretin

Somatostatin

Insulin

A

Secretin is released from mucosal cells in the duodenum and jejunum.

Pancreas endocrine physiology

Hormones released from the islets of Langerhans
Beta cells Insulin (70% of total secretions)
Alpha cells Glucagon
Delta cells Somatostatin
F cells Pancreatic polypeptide

How well did you know this?
1
Not at all
2
3
4
5
Perfectly
833
Q

Which of the following statements relating to blood transfusions in surgical patients is false?

Packed red cells typically have a haematocrit of between 55 and 75%

Clotting factor activity in whole blood decreases in samples stored for longer than 7 days

After 3 weeks of storage blood has a pH of 6.9

Gamma irradiated blood products are not required routinely

Patients should be transfused to achieve a target haemoglobin of 10 g/dl and a haematocrit of 30%

A

Patients can generally be managed without transfusion as long as the Hb is 7 or greater. The exact level depends upon patient factors such as co-morbidities. Old blood functions less effectively and should not be used during massive transfusions.

834
Q

A 38 year old man is playing football when he slips over during a tackle. His knee is painful immediately following the fall. Several hours later he notices that the knee has become swollen. Following a course of non steroidal anti inflammatory drugs and rest the situation improves. However, complains of recurrent pain. On assessment in clinic you notice that it is impossible to fully extend the knee, although the patient is able to do so when asked. What is the most likely injury?

Anterior cruciate ligament rupture

Posterior cruciate ligament rupture

Torn meniscus

Medial collateral ligament tear

Chondromalacia patellae

A

Twisting sporting injuries followed by delayed onset of knee swelling and locking are strongly suggestive of a menisceal tear. Arthroscopic menisectomy is the usual treatment.

Knee injury

Types of injury

Ruptured anterior cruciate ligament
Sport injury
Mechanism: high twisting force applied to a bent knee
Typically presents with: loud crack, pain and RAPID joint swelling (haemoarthrosis)
Poor healing
Management: intense physiotherapy or surgery
Ruptured posterior cruciate ligament
Mechanism: hyperextension injuries
Tibia lies back on the femur
Paradoxical anterior draw test
Rupture of medial collateral ligament
Mechanism: leg forced into valgus via force outside the leg
Knee unstable when put into valgus position
Menisceal tear
Rotational sporting injuries
Delayed knee swelling
Joint locking (Patient may develop skills to ‘unlock’ the knee
Recurrent episodes of pain and effusions are common, often following minor trauma
Chondromalacia patellae
Teenage girls, following an injury to knee e.g. Dislocation patella
Typical history of pain on going downstairs or at rest
Tenderness, quadriceps wasting
Dislocation of the patella
Most commonly occurs as a traumatic primary event, either through direct trauma or through severe contraction of quadriceps with knee stretched in valgus and external rotation
Genu valgum, tibial torsion and high riding patella are risk factors
Skyline x-ray views of patella are required, although displaced patella may be clinically obvious
An osteochondral fracture is present in 5%
The condition has a 20% recurrence rate
Fractured patella
2 types:
i. Direct blow to patella causing undisplaced fragments
ii. Avulsion fracture
Tibial plateau fracture
Occur in the elderly (or following significant trauma in young)
Mechanism: knee forced into valgus or varus, but the knee fractures before the ligaments rupture
Varus injury affects medial plateau and if valgus injury, lateral plateau depressed fracture occurs
Classified using the Schatzker system (see below)

Schatzker Classification system for tibial plateau fractures
Type Anatomical description Features
1 Vertical split of lateral condyle Fracture through dense bone, usually in the young. It may be virtually undisplaced, or the condylar fragment may be pushed inferiorly and tilted
2 Vertical split of the lateral condyle combined with an adjacent load bearing part of the condyle The wedge fragment (which may be of variable size), is displaced laterally; the joint is widened. Untreated, a valgus deformity may develop
3 Depression of the articular surface with intact condylar rim The split does not extend to the edge of the plateau. Depressed fragments may be firmly embedded in subchondral bone, the joint is stable
4 Fragment of the medial tibial condyle Two injuries are seen in this category; (1) a depressed fracture of osteoporotic bone in the elderly. (2) a high energy fracture resulting in a condylar split that runs from the intercondylar eminence to the medial cortex. Associated ligamentous injury may be severe
5 Fracture of both condyles Both condyles fractured but the column of the metaphysis remains in continuity with the tibial shaft
6 Combined condylar and subcondylar fractures High energy fracture with marked comminution

835
Q

A 52 year old man with long standing Barretts oesophagus is diagnosed with high grade dysplasia on recent endoscopy. The lesions are multifocal and mainly distally sited. What is the best course of action?

Endoscopic surveillance at 3 monthly intervals

Photodynamic therapy

Nissens fundoplication

Oesophagectomy

External beam radiotherapy

A

Some may argue for local therapy. However, in young patients who are otherwise fit, multifocal disease such as this should probably be resected. (Debatable answer. Perplexity answer at the end)

Treatment of oesophageal cancer

  • In general resections are not offered to those patients with distant metastasis, and usually not to those with N2 disease.
    Local nodal involvement is not in itself a contra indication to resection.
    Surgical resection is the mainstay of treatment.
    Neoadjuvent chemotherapy is given in most cases prior to surgery.
    In situ disease may be managed by endoscopic mucosal resection, although this is still debated.
    In patients with lower third lesions an Ivor - Lewis type procedure is most commonly performed. Very distal tumours may be suitable to a transhiatal procedure. Which is an attractive option as the penetration of two visceral cavities required for an Ivor- Lewis type procedure increases the morbidity considerably.
    More proximal lesions will require a total oesphagectomy (Mckeown type) with anastomosis to the cervical oesophagus.
    Patients with unresectable disease may derive benefit from local ablative procedures, palliative chemotherapy or stent insertion.

Operative details of Ivor- Lewis procedure
Combined laparotomy and right thoracotomy

Indication
Lower and middle third oesophageal tumours

Preparation
Staging with a combination of CT chest abdomen and pelvis- if no metastatic disease detected then patients will undergo a staging laparoscopy to detect peritoneal disease.
If both these modalities are negative then patients will finally undergo a PET CT scan to detect occult metastatic disease. Only in those whom no evidence of advanced disease is detected will proceed to resection.
Patients receive a GA, double lumen endotracheal tube to allow for lung deflation, CVP and arterial monitoring.

Procedure
A rooftop incision is made to access the stomach and duodenum.

Laparotomy To mobilize the stomach
The greater omentum is incised away from its attachment to the right gastroepiploic vessels along the greater curvature of the stomach.
Then the short gastric vessels are ligated and detached from the greater curvature from the spleen.
The lesser omentum is incised, preserving the right gastric artery.
The retroperitoneal attachments of the duodenum in its second and third portions are incised, allowing the pylorus to reach the oesophageal hiatus. Some surgeons perform a pyloroplasty at this point to facilitate gastric emptying.
The left gastric vessels are then ligated, avoiding any injury to the common hepatic or splenic arteries. Care must be taken to avoid inadvertently devascularising the liver owing to variations in anatomy.

Right Thoracotomy Oesophageal resection and oesophagogastric anastomosis
Through 5th intercostal space
Dissection performed 10cm above the tumour
This may involve transection of the azygos vein.
The oesophagus is then removed with the stomach creating a gastric tube.
An anastomosis is created.

The chest is closed with underwater seal drainage and tube drains to the abdominal cavity.

Post operatively
Patients will typically recover in ITU initially.
A nasogastric tube will have been inserted intraoperatively and must remain in place during the early phases of recovery.
Post operatively these patients are at relatively high risk of developing complications:

  • Atelectasis- due to the effects of thoracotomy and lung collapse
  • Anastomotic leakage. The risk is relatively high owing to the presence of a relatively devascularised stomach. Often the only blood supply is from the gastroepiploic artery as all others will have been divided. If a leak does occur then many will attempt to manage conservatively with prolonged nasogastric tube drainage and TPN. The reality is that up to 50% of patients developing an anastomotic leak will not survive to discharge.
  • Delayed gastric emptying (may be avoided by performing a pyloroplasty).

The best course of action for a 52-year-old man with long-standing Barrett’s esophagus diagnosed with multifocal, mainly distally sited high-grade dysplasia is endoscopic surveillance at 3-month intervals. According to a clinical practice update on endoscopic surveillance and management of dysplasia, endoscopic surveillance is recommended for patients with high-grade dysplasia. This approach allows for close monitoring of the condition and enables early intervention if the dysplasia progresses. Other options such as photodynamic therapy, Nissen’s fundoplication, esophagectomy, or external beam radiotherapy may be considered based on individual patient factors and the progression of the dysplasia, but the initial recommended approach is endoscopic surveillance

836
Q

A 20 year old lady presents with pain on the medial aspect of her thigh. Investigations show a large ovarian cyst. Compression of which of the nerves listed below is the most likely underlying cause?

Sciatic

Genitofemoral

Obturator

Ilioinguinal

Femoral cutaneous

A

The cutaneous branch of the obturator nerve is frequently absent. However, the obturator nerve is a recognised contributor to innervation of the medial thigh and large pelvic tumours may compress this nerve with resultant pain radiating distally.

Obturator nerve

The obturator nerve arises from L2, L3 and L4 by branches from the ventral divisions of each of these nerve roots. L3 forms the main contribution and the second lumbar branch is occasionally absent. These branches unite in the substance of psoas major, descending vertically in its posterior part to emerge from its medial border at the lateral margin of the sacrum. It then crosses the sacroiliac joint to enter the lesser pelvis, it descends on obturator internus to enter the obturator groove. In the lesser pelvis the nerve lies lateral to the internal iliac vessels and ureter, and is joined by the obturator vessels lateral to the ovary or ductus deferens.

Supplies
Medial compartment of thigh
Muscles supplied: external obturator, adductor longus, adductor brevis, adductor magnus (not the lower part-sciatic nerve), gracilis
The cutaneous branch is often absent. When present, it passes between gracilis and adductor longus near the middle part of the thigh, and supplies the skin and fascia of the distal two thirds of the medial aspect.

Obturator canal
Connects the pelvis and thigh: contains the obturator artery, vein, nerve which divides into anterior and posterior branches.

Cadaveric cross section demonstrating relationships of the obturator nerve

837
Q

In relation to the middle cranial fossa, which of the following statements relating to the foramina is incorrect?

The foramen rotundum transmits the maxillary nerve

The foramen lacerum is closely related to the internal carotid artery

The foramen spinosum lies posterolateral to the foramen ovale

The foramen ovale transmits the middle meningeal artery

The foramen rotundum lies anteromedial to the foramen ovale

A

The foramen spinosum transmits the middle meningeal artery. The foramen ovale transmits the mandibular nerve. As the foramina weaken the bone, a fracture at this site is not uncommon.

Foramina of the base of the skull

Foramen Location Contents
Foramen ovale Sphenoid bone Otic ganglion
V3 (Mandibular nerve:3rd branch of
trigeminal)
Accessory meningeal artery
Lesser petrosal nerve
Emissary veins
Foramen spinosum Sphenoid bone Middle meningeal artery
Meningeal branch of the Mandibular nerve
Foramen rotundum Sphenoid bone Maxillary nerve (V2)
Foramen lacerum/ carotid canal Located between the sphenoid, the apex of the petrous temporal and the basilar part of the occipital Base of the medial pterygoid plate.
Internal carotid artery*
Nerve and artery of the pterygoid canal
Jugular foramen Temporal bone Anterior: inferior petrosal sinus
Intermediate: glossopharyngeal, vagus, and accessory nerves.
Posterior: sigmoid sinus (becoming the internal jugular vein) and some meningeal branches from the occipital and ascending pharyngeal arteries.
Foramen magnum Occipital bone Anterior and posterior spinal arteries
Vertebral arteries
Medulla oblongata
Stylomastoid foramen Temporal bone Stylomastoid artery
Facial nerve
Superior orbital fissure Sphenoid bone Oculomotor nerve (III)
Recurrent meningeal artery
Trochlear nerve (IV)
Lacrimal, frontal and nasociliary branches of opthalmic nerve (V1)
Abducent nerve (VI)
Superior ophthalmic vein

*= In life the foramen lacerum is occluded by a cartilagenous plug. The ICA initially passes into the carotid canal which ascends superomedially to enter the cranial cavity through the foramen lacerum.

838
Q

In a 70 Kg male, what proportion of total body fluid will be contributed by plasma?

50%

5%

35%

65%

25%

A

% 5
70 Kg male = 42 L water (60% of total body weight)

Fluid compartment physiology

Body fluid compartments comprise intracellular and extracellular compartments. The latter includes interstitial fluid, plasma and transcellular fluid.
Typical figures are based on the 70 Kg male.

Body fluid volumes
Compartment Volume in litres Percentage of total volume
Intracellular 28 L 60-65%
Extracellular 14 L 35-40%
Plasma 3 L 5%
Interstitial 10 L 24%
Transcellular 1 L 3%
Figures are approximate

839
Q

A 58 year old man presents to the plastics team with severe burns to his hands. He is not distressed by the burns. He has bilateral charcot joints. On examination; there is loss of pain and temperature sensation of the upper limbs. What is the most likely diagnosis?

Potts disease of the spine

Tabes dorsalis

Transverse myelitis

Syringomyelia

Subacute degeneration of the cord

A

This patient has syringomyelia which selectively affects the spinotholamic tracts. Syringomyelia is a disorder in which a cystic cavity forms within the spinal cord. The commonest variant is the Arnold- Chiari malformation in which the cavity connects with a congenital malformation affecting the cerebellum. Acquired forms of the condition may occur as a result of previous meningitis, surgery or tumours. Many neurological manifestations have been reported, although the classical variety spares the dorsal columns and medial lemniscus and affecting only the spinothalamic tract with loss of pain and temperature sensation. The bilateral distribution of this patients symptoms would therefore favor syringomyelia over SCID or Brown Sequard syndrome. Osteomyelitis would tend to present with back pain and fever in addition to any neurological signs. Epidural haematoma large enough to produce neurological impairment will usually have motor symptoms in addition to any selective sensory loss, and the history is usually shorter.

840
Q

A 3 year old boy is brought to the clinic by his mother who has noticed a mass in his neck. On examination; he has a smooth mass located on the lateral aspect of his anterior triangle, near to the angle of the mandible. On ultrasound; it has a fluid filled, anechoic, appearance. What is the most likely cause?

Cystic hygroma

Dermoid cyst

Thyroglossal cyst

Branchial cyst

Lymphoma

A

Branchial cysts are usually located laterally and derived from the second branchial cleft. Unless infection has occurred they will usually have an anechoic appearance on ultrasound.

Neck Masses in Children

Thyroglossal cyst
Located in the anterior triangle, usually in the midline and below the hyoid (65% cases)
Derived from remnants of the thyroglossal duct
Thin walled and anechoic on USS (echogenicity suggests infection of cyst)
Branchial cyst
Six branchial arches separated by branchial clefts
Incomplete obliteration of the branchial apparatus may result in cysts, sinuses or fistulae
75% of branchial cysts originate from the second branchial cleft
Usually located anterior to the sternocleidomastoid near the angle of the mandible
Unless infected the fluid of the cyst has a similar consistency to water and is anechoic on USS
Dermoids
Derived from pleuripotent stem cells and are located in the midline
Most commonly in a suprahyoid location
They have heterogeneous appearances on imaging and contain variable amounts of calcium and fat
Thyroid gland
True thyroid lesions are rare in children and usually represent thyroglossal cysts or tumours like lymphoma
Lymphatic malformations
Usually located posterior to the sternocleidomastoid
Cystic hygroma result from occlusion of lymphatic channels
The painless, fluid filled, lesions usually present prior to the age of 2
They are often closely linked to surrounding structures and surgical removal is difficult
They are typically hypoechoic on USS
Infantile haemangioma
May present in either triangle of the neck
Grow rapidly initially and then will often spontaneously regress
Plain x-rays will show a mass lesion, usually containing calcified phleboliths
As involution occurs the fat content of the lesions increases
Lymphadenopathy
Located in either triangle of the neck
May be reactive or neoplastic
Generalised lymphadenopathy usually secondary to infection in children (very common)

841
Q

During a radical gastrectomy for carcinoma of the stomach the surgeons remove the omentum. What is the main source of its blood supply?

Ileocolic artery

Superior mesenteric artery

Gastroepiploic artery

Middle colic artery

Inferior mesenteric artery

A

The vessels supplying the omentum are the omental branches of the right and left gastro-epiploic arteries. The colonic vessels are not responsible for the arterial supply to the omentum. The left gastro-epiploic artery is a branch of the splenic artery and the right gastro-epiploic artery is a terminal branch of the gastroduodenal artery.

Omentum

The omentum is divided into two parts which invest the stomach. Giving rise to the greater and lesser omentum. The greater omentum is attached to the inferolateral border of the stomach and houses the gastro-epiploic arteries.
It is of variable size but is less well developed in children. This is important as the omentum confers protection against visceral perforation (e.g. Appendicitis).
Inferiorly between the omentum and transverse colon is one potential entry point into the lesser sac.
Several malignant processes may involve the omentum of which ovarian cancer is the most notable.

842
Q

A patient with tachycardia and hypotension is to receive vasopressors. Which of the following conditions are most likely to be treated with vasopressors?

Hypovolaemic shock

Septic shock

Neurogenic shock

Cardiogenic shock

None of the above

A

The term septic shock has a precise meaning and refers to refractory systemic arterial hypotension in spite of fluid resuscitation. Patients will therefore usually require vasopressors. Individuals suffering from neurogenic shock will usually receive intravenous fluids to achieve a mean arterial pressure of 90mmHg. If this target cannot be achieved then these patients will receive inotropes. Hypovolaemic shock requires fluids and the management of cardiogenic shock is multifactorial and includes inotropes, vasodilators and intra-aortic balloon pumps.

Inotropes and cardiovascular receptors

Inotropes are a class of drugs which work primarily by increasing cardiac output. They should be distinguished from vasoconstrictor drugs which are used specifically when the primary problem is peripheral vasodilatation.

Catecholamine type agents are commonly used and work by increasing cAMP levels by adenylate cyclase stimulation. This in turn increases intracellular calcium ion mobilisation and thus the force of contraction. Adrenaline works as a beta adrenergic receptor agonist at lower doses and an alpha receptor agonist at higher doses. Dopamine causes dopamine receptor mediated renal and mesenteric vascular dilatation and beta 1 receptor agonism at higher doses. This results in increased cardiac output. Since both heart rate and blood pressure are raised, there is less overall myocardial ischaemia. Dobutamine is a predominantly beta 1 receptor agonist with weak beta 2 and alpha receptor agonist properties. Noradrenaline is a catecholamine type agent and predominantly acts as an alpha receptor agonist and serves as a peripheral vasoconstrictor.

Phosphodiesterase inhibitors such as milrinone act specifically on the cardiac phosphodiesterase and increase cardiac output.

Inotrope Cardiovascular receptor action
Adrenaline α-1, α-2, β-1, β-2
Noradrenaline α-1,( α-2), (β-1), (β-2)
Dobutamine β-1, (β 2)
Dopamine (α-1), (α-2), (β-1), D-1,D-2
Minor receptor effects in brackets

Effects of receptor binding
α-1, α-2 vasoconstriction
β-1 increased cardiac contractility and HR
β-2 vasodilatation
D-1 renal and spleen vasodilatation
D-2 inhibits release of noradrenaline

843
Q

A 32 year male with leukaemia attends the day unit for a blood transfusion. Five days after the transfusion he attends the Emergency Department with a temperature of 38.5, erythroderma and desquamation. What is the most likely explanation?

Graft versus host disease

Acute haemolytic transfusion reaction

Neutropenic sepsis

Transfusion associated lung injury

Neutrophilic febrile reaction

A

This is associated with transfusion of unirradiated blood in immunosuppressed patients. Transfusion associated GVHD can occur 4-30 days after a transfusion and follows a sub acute pathway. Patients may also have diarrhoea and abnormal liver function tests. Management involves steroid therapy.

Blood transfusion reactions

Acute transfusion reactions present as adverse signs or symptoms during or within 24 hours of a blood transfusion. The most frequent reactions are fever, chills, pruritus, or urticaria, which typically resolve promptly without specific treatment or complications. Other signs occurring in temporal relationship with a blood transfusion, such as severe dyspnoea, pyrexia, or loss of consciousness may be the first indication of a more severe potentially fatal reaction.
The causes of adverse reactions are multi-factorial. Immune mediated reactions, some of the most feared, occur as a result of component mismatch, the commonest cause of which is clerical error. More common, non immune mediated, complications may occur as a result of product contamination, this may be bacterial or viral.
Transfusion related lung injury is well recognised and there are two proposed mechanisms which underpin this. One involves the sequestration of primed neutrophils within the recipient pulmonary capillary bed. The other proposed mechanism suggests that HLA mismatches between donor neutrophils and recipient lung tissue is to blame.
The table below summarises the main types of transfusion reaction.

Immune mediated Non immune mediated
Pyrexia Hypocalcaemia
Alloimmunization CCF
Thrombocytopaenia Infections
Transfusion associated lung injury Hyperkalaemia
Graft vs Host disease
Urticaria
Acute or delayed haemolysis
ABO incompatibility
Rhesus incompatibility

844
Q

A 22 year old Chef presents with abdominal pain in the right iliac fossa. There is an associated temperature and diarrhoea. The CT1 takes the patient to theatre for an appendicectomy, but the appendix appears normal. The terminal ileum appears thickened and engorged. Infection with which of the following is most likely?

Yersinia enterocolitica

Yersinia pestis

Salmonella

Vibrio cholera

Entero invasive E Coli

A

Yersinia can be mistaken for acute appendicitis due to mesenteric lymphadenitis and ileitis. Yersinia infection of the terminal ileum typically produces more marked clinical changes of this segment of bowel than infection with campylobacter. Yersinia pestis causes plague.

Bacterial Gastroenteritis

Causative organisms Features
Campylobacter jejuni
Most common cause of acute infective diarrhoea
Spiral, gram negative rods
Usually infects caecum and terminal ileum. Local lymphadenopathy is common
May mimic appendicitis as it has marked right iliac fossa pain
Reactive arthritis is seen in 1-2% of cases
Shigella spp.
Members of the enterobacteriaceae
Gram negative bacilli
Clinically causes dysentery
Shigella soneii is the commonest infective organism (mild illness)
Usually self limiting, ciprofloxacin may be required if individual is in a high risk group
Salmonella spp
Facultatively anaerobic, gram negative, enterobacteriaceae
Infective dose varies according to subtype
Salmonellosis: usually transmitted by infected meat (especially poultry) and eggs
E. coli
Enteropathogenic
Enteroinvasive: dysentery, large bowel necrosis/ulcers
Enterotoxigenic: small intestine, travelers diarrhoea
Enterohaemorrhagic: 0157, cause a haemorrhagic colitis, haemolytic uraemic syndrome and thrombotic thrombocytopaenic purpura
Yersinia enterocolitica
Gram negative, coccobacilli
Typically produces a protracted terminal ileitis that may mimic Crohns disease
Differential diagnosis acute appendicitis
May progress to septicaemia in susceptible individuals
Usually sensitive to quinolone or tetracyclines
Vibrio cholera
Short, gram negative rods
Transmitted by contaminated water, seafood
Symptoms include sudden onset of effortless vomiting and profuse watery diarrhoea
Correction of fluid and electrolyte losses are the mainstay of treatment
Most cases will resolve, antibiotics are not generally indicated

845
Q

Which of the following is associated with poor wound healing?

Jaundice

Patients taking carbamazepine

General anaesthesia using thiopentone

General anaesthesia using ketamine

Multiple sclerosis

A

Mnemonic to remember factors affecting wound healing: DID NOT HEAL

D iabetes
I nfection, irradiation
D rugs eg steroids, chemotherapy

N utritional deficiencies (vitamin A, C & zinc, manganese), Neoplasia
O bject (foreign material)
T issue necrosis

H ypoxia
E xcess tension on wound
A nother wound
L ow temperature, Liver jaundice
Multiple sclerosis is associated with pressure sores, however the cellular healing process is not affected.

Wound healing

Surgical wounds are either incisional or excisional and either clean, clean contaminated or dirty. Although the stages of wound healing are broadly similar their contributions will vary according to the wound type.

The main stages of wound healing include:

Haemostasis
Minutes to hours following injury
Vasospasm in adjacent vessels, platelet plug formation and generation of fibrin rich clot.

Inflammation
Typically days 1-5
Neutrophils migrate into wound (function impaired in diabetes).
Growth factors released, including basic fibroblast growth factor and vascular endothelial growth factor.
Fibroblasts replicate within the adjacent matrix and migrate into wound.
Macrophages and fibroblasts couple matrix regeneration and clot substitution.

Regeneration
Typically days 7 to 56
Platelet derived growth factor and transformation growth factors stimulate fibroblasts and epithelial cells.
Fibroblasts produce a collagen network.
Angiogenesis occurs and wound resembles granulation tissue.

Remodeling
From 6 weeks to 1 year
Longest phase of the healing process and may last up to one year (or longer).
During this phase fibroblasts become differentiated (myofibroblasts) and these facilitate wound contraction.
Collagen fibres are remodeled.
Microvessels regress leaving a pale scar.

The above description represents an idealised scenario. A number of diseases may distort this process. Neovascularisation is an important early process. Endothelial cells may proliferate in the wound bed and recanalise to form a vessel. Vascular disease, shock and sepsis can all compromise microvascular flow and impair healing.

Conditions such as jaundice will impair fibroblast synthetic function and immunity with a detrimental effect in most parts of the healing process.

Problems with scars:

Hypertrophic scars
Excessive amounts of collagen within a scar. Nodules may be present histologically containing randomly arranged fibrils within and parallel fibres on the surface. The tissue itself is confined to the extent of the wound itself and is usually the result of a full thickness dermal injury. They may go on to develop contractures.

846
Q

A 12 month old child is brought to the clinic with a history of a right groin swelling. The parents have a photograph on their mobile phone which looks very much like an inguinal hernia. What is the best course of action?

Arrange an MRI scan

Undertake an open inguinal hernia repair with mesh

Undertake an open inguinal herniotomy

Undertake a laparoscopic hernia repair with mesh

Arrange a herniogram

A

Where the history is strongly suggestive and the parents have a clear image or even description, most surgeons would proceed without confirmatory imaging. Herniograms and MRI would require a GA in a child of this age and are not routine. Herniotomy is the usual procedure and no mesh is used.
Inguinal hernia surgery

Inguinal hernias occur when the abdominal viscera protrude through the anterior abdominal wall into the inguinal canal. They may be classified as being either direct or indirect. The distinction between these two rests on their relation to Hesselbach’s triangle.

Boundaries of Hesselbach’s Triangle
Medial: Rectus abdominis
Lateral: Inferior epigastric vessels
Inferior: Inguinal ligament

Hernias occurring within the triangle tend to be direct and those outside - indirect.

Diagnosis
Most cases are diagnosed clinically, a reducible swelling may be located at the level of the inguinal canal. Large hernia’s may extend down into the male scrotum, these will not trans-illuminate and it is not possible to ‘get above’ the swelling.
Cases that are unclear on examination, but suspected from the history, may be further investigated using ultrasound or by performing a herniogram.

Treatment
Hernias associated with few symptoms may be managed conservatively. Symptomatic hernias or those which are at risk of developing complications are usually treated surgically.
First time hernias may be treated by performing an open inguinal hernia repair; the inguinal canal is opened, the hernia reduced and the defect repaired. A prosthetic mesh may be placed posterior to the cord structures to re-enforce the repair and reduce the risk of recurrence.
Recurrent hernias and those which are bilateral are generally managed with a laparoscopic approach. This may be via an intra or extra peritoneal route. As in open surgery a mesh is deployed. However, it will typically lie posterior to the deep ring.

Inguinal hernia in children
Inguinal hernias in children are almost always of an indirect type and therefore are usually dealt with by herniotomy, rather than herniorraphy. Neonatal hernias especially in those children born prematurely are at highest risk of strangulation and should be repaired urgently. Other hernias may be repaired on an elective basis.

References
The UK Based National Institute of Clinical Excellence has published guidelines relating to the choice between open and laparoscopic inguinal hernia repair. Which users may find interesting:

http://guidance.nice.org.uk/TA83/Guidance/pdf/English

847
Q

Which of the following nerves is responsible for the motor innervation of the sternocleidomastoid muscle?

Ansa cervicalis

Accessory nerve

Hypoglossal nerve

Facial nerve

Vagus nerve

A

The motor supply to the sternocleidomastoid is from the accessory nerve. The ansa cervicalis supplies sensory information from the muscle.

Sternocleidomastoid

Anatomy
Origin Rounded tendon attached to upper manubrium sterni and muscular head attached to medial third of the clavicle
Insertion Mastoid process of the temporal bone and lateral area of the superior nuchal line of the occipital bone
Innervation Spinal part of accessory nerve and anterior rami of C2 and C3 (proprioception)
Action
Both: extend the head at atlanto-occipital joint and flex the cervical vertebral column. Accessory muscles of inspiration.
Single: lateral flexion of neck, rotates head so face looks upward to the opposite side

Sternocleidomastoid divides the anterior and posterior triangles of the neck.

848
Q

A 45 year old man develops a colocutaneous fistula following reversal of a loop colostomy fashioned for the defunctioning of an anterior resection. Pre-operative gastrograffin enema showed no distal obstruction or anastomotic stricture. What is the best course of action?

Make the patient nil by mouth and commence total parenteral nutrition

Provide local wound care and await spontaneous resolution

Undertake a laparotomy and resect the fistula

Construct a loop ileostomy

Re-construct the loop colostomy

A

Fistulas are more likely to heal in the absence of distal luminal obstruction
Colocutaneous fistulae may occur as a result of anastomotic leakage following loop colostomy reversal. In the absence of abdominal signs a laparotomy is not necessarily required. Signs of wound sepsis may require antibiotics. Because there is not any distal obstruction (note normal pre-operative gastrograffin enema) these fistulae will usually close spontaneously.

Fistulas

A fistula is defined as an abnormal connection between two epithelial surfaces.
There are many types ranging from Branchial fistulae in the neck to entero-cutaneous fistulae abdominally.
In general surgical practice the abdominal cavity generates the majority and most of these arise from diverticular disease and Crohn’s.
As a general rule all fistulae will resolve spontaneously as long as there is no distal obstruction. This is particularly true of intestinal fistulae.

The four types of fistulae are:

Enterocutaneous
These link the intestine to the skin. They may be high (>500ml) or low output (<250ml) depending upon source. Duodenal /jejunal fistulae will tend to produce high volume, electrolyte rich secretions which can lead to severe excoriation of the skin. Colo-cutaneous fistulae will tend to leak faeculent material. Both fistulae may result from the spontaneous rupture of an abscess cavity onto the skin (such as following perianal abscess drainage) or may occur as a result of iatrogenic input. In some cases it may even be surgically desirable e.g. mucous fistula following sub total colectomy for colitis.

Suspect if there is excess fluid in the drain.

Enteroenteric or Enterocolic
This is a fistula that involves the large or small intestine. They may originate in a similar manner to enterocutaneous fistulae. A particular problem with this fistula type is that bacterial overgrowth may precipitate malabsorption syndromes. This may be particularly serious in inflammatory bowel disease.

Enterovaginal
Aetiology as above.

Enterovesical
This type of fistula goes to the bladder. These fistulas may result in frequent urinary tract infections, or the passage of gas from the urethra during urination.

Management
Some rules relating to fistula management:
They will heal provided there is no underlying inflammatory bowel disease and no distal obstruction, so conservative measures may be the best option
Where there is skin involvement, protect the overlying skin, often using a well fitted stoma bag- skin damage is difficult to treat
A high output fistula may be rendered more easily managed by the use of octreotide, this will tend to reduce the volume of pancreatic secretions.
Nutritional complications are common especially with high fistula (e.g. high jejunal or duodenal) these may necessitate the use of TPN to provide nutritional support together with the concomitant use of octreotide to reduce volume and protect skin.
When managing perianal fistulae surgeons should avoid probing the fistula where acute inflammation is present, this almost always worsens outcomes.
When perianal fistulae occur secondary to Crohn’s disease the best management option is often to drain acute sepsis and maintain that drainage through the judicious use of setons whilst medical management is implemented.
Always attempt to delineate the fistula anatomy, for abscesses and fistulae that have an intra abdominal source the use of barium and CT studies should show a track. For perianal fistulae surgeons should recall Goodsall’s rule in relation to internal and external openings.

849
Q

Into which of the following structures does the superior part of the fibrous capsule of the shoulder joint insert?

The surgical neck of the humerus

The body of the humerus

The bicipital groove

Immediately distal to the greater tuberosity

The anatomical neck of the humerus

A

The shoulder joint is a shallow joint, hence its great mobility. However, this comes at the expense of stability. The fibrous capsule attaches to the anatomical neck superiorly and the surgical neck inferiorly

Shoulder joint

Shallow synovial ball and socket type of joint.
It is an inherently unstable joint, but is capable to a wide range of movement.
Stability is provided by muscles of the rotator cuff that pass from the scapula to insert in the greater tuberosity (all except sub scapularis-lesser tuberosity).

Glenoid labrum
Fibrocartilaginous rim attached to the free edge of the glenoid cavity
Tendon of the long head of biceps arises from within the joint from the supraglenoid tubercle, and is fused at this point to the labrum.
The long head of triceps attaches to the infraglenoid tubercle

Fibrous capsule
Attaches to the scapula external to the glenoid labrum and to the labrum itself (postero-superiorly)
Attaches to the humerus at the level of the anatomical neck superiorly and the surgical neck inferiorly
Anteriorly the capsule is in contact with the tendon of subscapularis, superiorly with the supraspinatus tendon, and posteriorly with the tendons of infraspinatus and teres minor. All these blend with the capsule towards their insertion.
Two defects in the fibrous capsule; superiorly for the tendon of biceps. Anteriorly there is a defect beneath the subscapularis tendon.
The inferior extension of the capsule is closely related to the axillary nerve at the surgical neck and this nerve is at risk in anteroinferior dislocations. It also means that proximally sited osteomyelitis may progress to septic arthritis.

Movements and muscles
Flexion Anterior part of deltoid
Pectoralis major
Biceps
Coracobrachialis
Extension Posterior deltoid
Teres major
Latissimus dorsi
Adduction Pectoralis major
Latissimus dorsi
Teres major
Coracobrachialis
Abduction Mid deltoid
Supraspinatus
Medial rotation Subscapularis
Anterior deltoid
Teres major
Latissimus dorsi
Lateral rotation Posterior deltoid
Infraspinatus
Teres minor

Important anatomical relations
Anteriorly Brachial plexus
Axillary artery and vein
Posterior Suprascapular nerve
Suprascapular vessels
Inferior Axillary nerve
Circumflex humeral vessels

850
Q

A 65 year old man with long standing atrial fibrillation develops an embolus to the lower leg. The decision is made to perform an embolectomy, utilising a trans popliteal approach. After incising the deep fascia, which of the following structures will the surgeons encounter first on exploring the central region of the popliteal fossa?

Popliteal vein

Common peroneal nerve

Popliteal artery

Tibial nerve

None of the above

A

The tibial nerve lies superior to the vessels in the inferior aspect of the popliteal fossa. In the upper part of the fossa the tibial nerve lies lateral to the vessels, it then passes superficial to them to lie medially. The popliteal artery is the deepest structure in the popliteal fossa.

Popliteal fossa

Boundaries of the popliteal fossa
Laterally Biceps femoris above, lateral head of gastrocnemius and plantaris below
Medially Semimembranosus and semitendinosus above, medial head of gastrocnemius below
Floor Popliteal surface of the femur, posterior ligament of knee joint and popliteus muscle
Roof Superficial and deep fascia

Contents
Popliteal artery and vein
Small saphenous vein
Common peroneal nerve
Tibial nerve
Posterior cutaneous nerve of the thigh
Genicular branch of the obturator nerve
Lymph nodes

851
Q

A 70 year old man is due to undergo an arterial bypass procedure for claudication and foot ulceration. The anterior tibial artery will form the site of the distal arterial anastomosis. Which of the following structures is not closely related to it distally?

Interosseous membrane

Deep peroneal nerve

Tibialis posterior

Extensor hallucis longus

Dorsalis pedis artery

A

As an artery of the anterior compartment, the anterior tibial artery is closely related to tibialis anterior. The tibialis posterior is related to it at its origin.

Anterior tibial artery

Begins opposite the distal border of popliteus
Terminates in front of the ankle, continuing as the dorsalis pedis artery
As it descends it lies on the interosseous membrane, distal part of the tibia and front of the ankle joint
Passes between the tendons of extensor digitorum and extensor hallucis longus distally
It is related to the deep peroneal nerve, it lies anterior to the middle third of the vessel and lateral to it in the lower third

852
Q

A 65 year old man is undergoing a high anterior resection of the rectum for a distally sited sigmoid cancer. As the inferior mesenteric artery is mobilised at its distal aspect some fine nerves are encountered and inadvertently damaged. What is the most likely functional sequelae?

Impaired contraction of the external urethral sphincter

Impaired contraction of the detrusor muscle

Impaired relaxation of the detrusor muscle

Impaired relaxation of the external urethral sphincter

Impaired contraction of the internal urethral sphincter

A

The parasympathetic nerves to the pelvic organs can be encountered in left sided distal colonic resections. If divided, there are notable effects on bladder function and this can result in impaired detrusor muscle function as parasympathetic nerves are responsible for detrusor contraction. The sympathetic nerves cause detrusor relaxation. The external sphincter is not affected. This is one of the reasons that patients may develop urinary retention following colon resections.

Bladder

The empty bladder is contained within the pelvic cavity. It is usually a three sided pyramid. The apex of the bladder points forwards towards the symphysis pubis and the base lies immediately anterior to the rectum or vagina. Continuous with the apex is the median umbilical ligament, during development this was the site of the urachus.
The inferior aspect of the bladder is retroperitoneal and the superior aspect covered by peritoneum. As the bladder distends it will tend to separate the peritoneum from the fascia of transversalis. For this reason a bladder that is distended due to acute urinary retention may be approached with a suprapubic catheter that avoids entry into the peritoneal cavity.
The trigone is the least mobile part of the bladder and forms the site of the ureteric orifices and internal urethral orifice. In the empty bladder the ureteric orifices are approximately 2-3cm apart, this distance may increase to 5cm in the distended bladder.

Arterial supply
The superior and inferior vesical arteries provide the main blood supply to the bladder. These are branches of the internal iliac artery.

Venous drainage
In males the bladder is drained by the vesicoprostatic venous plexus. In females the bladder is drained by the vesicouterine venous plexus. In both sexes this venous plexus will ultimately drain to the internal iliac veins.

Lymphatic drainage
Lymphatic drainage is predominantly to the external iliac nodes, internal iliac and obturator nodes also form sites of bladder lymphatic drainage.

Innervation
Parasympathetic nerve fibres innervate the bladder from the pelvic splanchnic nerves. Sympathetic nerve fibres are derived from L1 and L2 via the hypogastric nerve plexuses. The parasympathetic nerve fibres will typically cause detrusor muscle contraction and result in voiding. The muscle of the trigone is innervated by the sympathetic nervous system. The external urethral sphincter is under conscious control. During bladder filling the rate of firing of nerve impulses to the detrusor muscle is low and receptive relaxation occurs. At higher volumes and increased intra vesical pressures the rate of neuronal firing will increase and eventually voiding will occur.

853
Q

A surgical department wishes to determine whether it is using types of prosthetic mesh material for incisional hernia surgery in the most effective manner. Recently there have been cases of non mesh usage and loss of material as a result of the implants being ‘out of date’. What is the most appropriate method to investigate this?

System based audit

Standards audit

Peer review

Financial audit

Operational audit

A

If the problem was related to how the surgical department is performing the surgeries, rather than stock control and the system by which the materials are used within the theater, then an operational audit would be more appropriate. For example, if the problem was that the surgical department was not using the prosthetic mesh material in the most effective manner during the surgery, such as incorrect placement or incorrect sizing, then an operational audit would be more appropriate. The audit would focus on evaluating the effectiveness and efficiency of the surgical procedures being performed, including the policies and procedures, resources, and performance metrics. The audit would identify any inefficiencies or breakdowns in the day-to-day operations that are contributing to the problem and recommend solutions to improve the effectiveness and efficiency of the surgical procedures. So, in summary, the type of audit that is most appropriate depends on the nature of the problem being investigated. If the problem is related to a breakdown in the system or technology infrastructure, then a system-based audit would be more appropriate. If the problem is related to day-to-day operations, policies, and procedures, then an operational audit would be more appropriate.

Audit categories

Audits may be used in a variety of clinical settings. These range from standards based audits, which will be familiar to most clinicians, through to systems based audits which focus more on the processes within an organisation.

Types of audit
Financial audit A historically oriented, independent evaluation performed for the purpose of attesting to the fairness, accuracy, and reliability of financial data
Operational audit A future-oriented, systematic, and independent evaluation of organizational activities. Financial data may be used, but the primary sources of evidence are the operational policies and achievements related to organizational objectives. Internal controls and efficiencies may be evaluated during this type of review.
Departmental review A current period analysis of administrative functions, to evaluate the adequacy of controls, safeguarding of assets, efficient use of resources, compliance with related laws, regulations and institutional policy and integrity of financial information.
Standards based audit Comparison of care or passage of care against set and widely agreed standards or outcomes.
Systems based audit Evaluation of processes occurring within an institution.

Systems based audits are an integral part of the process of clinical governance.

854
Q

Which of the following inhibits gastric acid secretion?

Histamine

Nausea

Calcium

Parasympathetic vagal stimulation

Gastrin

A

Nausea inhibits gastric secretion via higher cerebral activity and sympathetic innervation.

Gastric secretions

A working knowledge of gastric secretions is important for surgery because peptic ulcers are common, surgeons frequently prescribe anti secretory drugs and because there are still patients around who will have undergone acid lowering procedures (Vagotomy) in the past.

Gastric acid
Is produced by the parietal cells in the stomach
pH of gastric acid is around 2 with acidity being maintained by the H+/K+ ATP ase pump. As part of the process bicarbonate ions will be secreted into the surrounding vessels.
Sodium and chloride ions are actively secreted from the parietal cell into the canaliculus. This sets up a negative potential across the membrane and as a result sodium and potassium ions diffuse across into the canaliculus.
Carbonic anhydrase forms carbonic acid which dissociates and the hydrogen ions formed by dissociation leave the cell via the H+/K+ antiporter pump. At the same time sodium ions are actively absorbed. This leaves hydrogen and chloride ions in the canaliculus these mix and are secreted into the lumen of the oxyntic gland.
Phases of gastric acid secretion
There are 3 phases of gastric secretion:

  1. Cephalic phase (smell / taste of food)
    30% acid produced
    Vagal cholinergic stimulation causing secretion of HCL and gastrin release from G cells
  2. Gastric phase (distension of stomach )
    60% acid produced
    Stomach distension/low H+/peptides causes Gastrin release
  3. Intestinal phase (food in duodenum)
    10% acid produced
    High acidity/distension/hypertonic solutions in the duodenum inhibits gastric acid secretion via enterogastrones (CCK, secretin) and neural reflexes.

Regulation of gastric acid production
Factors increasing production include:
Vagal nerve stimulation
Gastrin release
Histamine release (indirectly following gastrin release) from enterchromaffin like cells

Factors decreasing production include:
Somatostatin (inhibits histamine release)
Cholecystokinin
Secretin

Below is a brief summary of the major hormones involved in food digestion:

Source Stimulus Actions
Gastrin
G cells in antrum of the stomach
Distension of stomach, extrinsic nerves
Inhibited by: low antral pH, somatostatin
Increase HCL, pepsinogen and IF secretion, increases gastric motility, trophic effect on gastric mucosa
CCK
I cells in upper small intestine
Partially digested proteins and triglycerides
Increases secretion of enzyme-rich fluid from pancreas, contraction of gallbladder and relaxation of sphincter of Oddi, decreases gastric emptying, trophic effect on pancreatic acinar cells, induces satiety
Secretin
S cells in upper small intestine
Acidic chyme, fatty acids
Increases secretion of bicarbonate-rich fluid from pancreas and hepatic duct cells, decreases gastric acid secretion, trophic effect on pancreatic acinar cells
VIP
Small intestine, pancreas
Neural
Stimulates secretion by pancreas and intestines, inhibits acid and pepsinogen secretion
Somatostatin
D cells in the pancreas and stomach
Fat, bile salts and glucose in the intestinal lumen
Decreases acid and pepsin secretion, decreases gastrin secretion, decreases pancreatic enzyme secretion, decreases insulin and glucagon secretion
inhibits trophic effects of gastrin, stimulates gastric mucous production

855
Q

A 68-year-old man presents with mechanical hip pain. He undergoes routine plain AP and lateral radiography of the hip. What is the first radiographic sign to appear in osteoarthritis?

A.Narrowing joint space

B.Osteopenia

C.Osteophytes

D.Soft tissue swelling

E.Subchondral sclerosis

A

Narrowing joint space
Osteoarthritis is a degenerative joint disorder, which results from a loss of hyaline cartilage in synovial joints. The first radiographic sign of osteoarthritis is non-uniform joint space narrowing.

Osteopenia
Osteopenia is a condition in which bone mineral density is lower than normal.

Osteophytes
Osteophyte formation is a radiographic sign of osteoarthritis, but it is not the first to appear. As the disease progresses, subluxation may occur and osteophytes may form.

Soft tissue swelling
Rest pain, stiffness and bony swelling are clinical features of osteoarthritis. Soft tissue swelling is more commonly associated with rheumatoid arthritis.

Subchondral sclerosis
Subchrondal sclerosis occurs as cartilage loss increases and appears as an area of increased density on the radiograph.
The classic radiographic signs of osteoarthritis are joint space narrowing, subchondral sclerosis, osteophytes and subchondral cysts. Osteopenia and soft tissue swelling are often associated with rheumatoid arthritis.

856
Q

A 76 years old man presented with nocturia, frequency urgency and frank hematuria. Ultrasound reveal 2cm filling defect at left bladder wall and left hydronephrosis. What is the diagnosis?
A. Transitionalcellcarcinomaofbladder
B. Renal cell carcinoma
C. Leiomyoma
D. Squamouscellcarcinoma
E. Nephroblastoma

A

Based on the information you provided, the most likely diagnosis is A. Transitional cell carcinoma of the bladder. This is a type of cancer that arises from the cells that line the inner surface of the bladder. It is the most common type of bladder cancer and accounts for about 90% of cases1.
Some of the risk factors for transitional cell carcinoma of the bladder include:
Age over 55 years
Male sex
Smoking
Exposure to certain chemicals, such as aromatic amines, benzene, or arsenic
Chronic urinary tract infections or irritation
Family history of bladder cancer12
Some of the signs and symptoms of transitional cell carcinoma of the bladder include:
Hematuria (blood in the urine)
Dysuria (painful or difficult urination)
Frequency (urinating more often than usual)
Urgency (feeling a strong need to urinate)
Nocturia (waking up at night to urinate)
Lower back pain
Pelvic pain
Weight loss
Fatigue12
The diagnosis of transitional cell carcinoma of the bladder is usually confirmed by:
Urinalysis and urine cytology (to check for blood, infection, or abnormal cells in the urine)
Cystoscopy (a procedure that uses a thin, flexible tube with a light and a camera to examine the inside of the bladder)
Biopsy (a procedure that removes a small sample of tissue from the bladder for microscopic examination)
Imaging tests, such as ultrasound, CT scan, or MRI (to check for the size, location, and extent of the tumor and possible spread to other organs)12
The treatment of transitional cell carcinoma of the bladder depends on the stage and grade of the cancer, as well as the patient’s age, overall health, and preferences. Some of the treatment options include:
Surgery (to remove the tumor, part of the bladder, or the entire bladder)
Chemotherapy (to kill cancer cells with drugs)
Immunotherapy (to stimulate the body’s immune system to fight cancer cells)
Radiation therapy (to destroy cancer cells with high-energy rays)
Intravesical therapy (to deliver drugs directly into the bladder through a catheter)

857
Q

The most commonly injured ligament in ankle injury is:
A. Posteriortalofibularligament
B. Anterior talofibular ligament
C. Deltoid ligament
D. Calcaneotalarligament
E. None of the above

A

The correct answer is B. Anterior talofibular ligament. This is the most frequently damaged ligament in the ankle, especially when the foot is forcefully inverted12. The anterior talofibular ligament connects the talus bone to the fibula bone on the outer side of the ankle

858
Q

Which of the following pairings are incorrect?

Aortic bifurcation and L4

Transpyloric plane and L1

Termination of dural sac and L4

Oesophageal passage through diaphragm and T10

Transition between pharynx and oesophagus at C6

A

Vena cava T8 (eight letters)
Oesophagus T10 (ten letters)
Aortic hiatus T12 (twelve letters)
The dura terminates at S2. The spinal cord terminates at L1, which is why it’s safe to undertake an LP at L4/5.

Levels

Transpyloric plane
Level of the body of L1

Pylorus stomach
Left kidney hilum (L1- left one!)
Fundus of the gallbladder
Neck of pancreas
Duodenojejunal flexure
Superior mesenteric artery
Portal vein
Left and right colic flexure
Root of the transverse mesocolon
2nd part of the duodenum
Upper part of conus medullaris
Spleen

Can be identified by asking the supine patient to sit up without using their arms. The plane is located where the lateral border of the rectus muscle crosses the costal margin.

Anatomical planes
Subcostal plane Lowest margin of 10th costal cartilage
Intercristal plane Level of body L4 (highest point of iliac crest)
Intertubercular plane Level of body L5

Common level landmarks
Inferior mesenteric artery L3
Bifurcation of aorta into common iliac arteries L4
Formation of IVC L5 (union of common iliac veins)
Diaphragm apertures
Vena cava T8
Oesophagus T10
Aortic hiatus T12

859
Q

A 45 year old man undergoes a general anaesthetic and a depolarising muscle relaxation agent is used. At the conclusion of the procedure which comprises the removal of a small simple skin lesion, the patient does not resume spontaneous respiration. A deficiency in which of the enzymes listed below is likely to account for this process?

P450 cytochrome oxidase

Lipase

Catalase

Glucose-6-phosphate dehydrogenase

Pseudocholinesterase

A

Pseudocholinesterase

Neuromuscular blocking drugs

These drugs work by preventing acetylcholine interacting with the post synaptic (nicotinic) receptors on the motor end plate of skeletal muscle. They are described as being either depolarising or non depolarising according to their mode of action.

Depolarising neuromuscular blocking drugs
Suxamethonium is the only commonly used drug in this category. Following injection there is a brief period of muscular fasciculation as the membrane is depolarised followed by paralysis. Recovery occurs spontaneously as the suxamethonium is hydrolysed by the enzyme plasma pseudocholinesterase. This process typically takes 4-6 minutes.
Suxamethonium has a number of important side effects which include malignant hyperpyrexia in susceptible patients, increased intra ocular pressure, prolonged apoea in those with pseudocholinesterase deficiency, a rise in serum potassium (which is most marked in burns and crush injury patients).

Pseudocholinesterase deficiency
Approximately 50 in 1000 will have slightly reduced enzyme activity and the paralysis effects may last for up to 20 minutes. In those with true and complete enzyme deficiency the apnoea can lasts for up to 2 hours, the incidence of this is <1 per 1000. Patients who display an enzyme lack will require prolonged ventillatory support subsequently and should be warned of this. This should then be clearly recorded in patient notes.

Non depolarising neuromuscular blocking drugs
These drugs compete with acetylcholine and block its access at the post synaptic receptor sites on the muscle. They do not cause depolarisation. They have a slower duration of onset and a longer duration of action than suxamethonium. In many cases an anticholinesterase may be administered to hasten recovery. This results in an increase in acetylcholine at the receptor site which then competes with the paralysing drug at the receptor site. Reversal drugs are not administered until there is some evidence of returning neuromuscular function to avoid rebound paralysis following recovery from anaesthesia. An antimuscarinic drug is used concomitantly with reversal agents to avoid the parsympathetic surge that would otherwise accompany a rise in AcH levels at the parasympathetic receptors with associated bradycardia and salivation. Neostigmine is the reversal agent in common use and it is often administered with atropine or glycopyrrolate to avoid the adverse effects outline above.

860
Q

A 28 year old man develops an acute paronychia and subsequent spreading sepsis. The tissue exudate has a higher protein content than normal tissue because?

Breakdown of tissue cells release protein

Capillary walls are more permeable

Increased blood flow transports more protein into the area

Intracapillary pressure is raised

Plasma cells release gamma globulin

A

The increased permeability allows the exudation of plasma proteins.

Acute inflammation

Inflammation is the reaction of the tissue elements to injury. Vascular changes occur, resulting in the generation of a protein rich exudate. So long as the injury does not totally destroy the existing tissue architecture, the episode may resolve with restoration of original tissue architecture.

Vascular changes
Vasodilation occurs and persists throughout the inflammatory phase.
Inflammatory cells exit the circulation at the site of injury.
The equilibrium that balances Starlings forces within capillary beds is disrupted and a protein rich exudate will form as the vessel walls also become more permeable to proteins.
The high fibrinogen content of the fluid may form a fibrin clot. This has several important immunomodulatory functions.

Sequelae
Resolution
Typically occurs with minimal initial injury
Stimulus removed and normal tissue architecture results
Organisation
Delayed removal of exudate
Tissues undergo organisation and usually fibrosis
Suppuration
Typically formation of an abscess or an empyema
Sequestration of large quantities of dead neutrophils
Progression to chronic inflammation
Coupled inflammatory and reparative activities
Usually occurs when initial infection or suppuration has been inadequately managed

Causes
Infections e.g. Viruses, exotoxins or endotoxins released by bacteria
Chemical agents
Physical agents e.g. Trauma
Hypersensitivity reactions
Tissue necrosis

Presence of neutrophil polymorphs is a histological diagnostic feature of acute inflammation

861
Q

Which muscle is not innervated by the trigeminal nerve?

Medial pterygoid

Mylohyoid

Stylohyoid

Masseter

Temporalis

A

Stylohyoid is innervated by the facial nerve.

Trigeminal nerve

The trigeminal nerve is the main sensory nerve of the head. In addition to its major sensory role, it also innervates the muscles of mastication.

Distribution of the trigeminal nerve
Sensory
Scalp
Face
Oral cavity (and teeth)
Nose and sinuses
Dura mater
Motor
Muscles of mastication
Mylohyoid
Anterior belly of digastric
Tensor tympani
Tensor palati
Autonomic connections (ganglia)
Ciliary
Sphenopalatine
Otic
Submandibular

Path
Originates at the pons
Sensory root forms the large, crescentic trigeminal ganglion within Meckel’s cave, and contains the cell bodies of incoming sensory nerve fibres. Here the 3 branches exit.
The motor root cell bodies are in the pons and the motor fibres are distributed via the mandibular nerve. The motor root is not part of the trigeminal ganglion.

Branches of the trigeminal nerve
Ophthalmic nerve Sensory only
Maxillary nerve Sensory only
Mandibular nerve Sensory and motor

Sensory
Ophthalmic Exits skull via the superior orbital fissure
Sensation of: scalp and forehead, the upper eyelid, the conjunctiva and cornea of the eye, the nose (including the tip of the nose, except alae nasi), the nasal mucosa, the frontal sinuses, and parts of the meninges (the dura and blood vessels).
Maxillary nerve Exit skull via the foramen rotundum
Sensation: lower eyelid and cheek, the nares and upper lip, the upper teeth and gums, the nasal mucosa, the palate and roof of the pharynx, the maxillary, ethmoid and sphenoid sinuses, and parts of the meninges.
Mandibular nerve Exit skull via the foramen ovale
Sensation: lower lip, the lower teeth and gums, the chin and jaw (except the angle of the jaw), parts of the external ear, and parts of the meninges.

Motor
Distributed via the mandibular nerve.
The following muscles of mastication are innervated:
Masseter
Temporalis
Medial pterygoid
Lateral pterygoid

Other muscles innervated include:
Tensor veli palatini
Mylohyoid
Anterior belly of digastric
Tensor tympani

862
Q

Which of the following is a recognised tributary of the retromandibular vein?

Internal jugular vein

External jugular vein

Anterior temporal diploic vein

Maxillary vein

Inferior opthalmic vein

A

The retromandibular vein is formed from the union of the maxillary and superficial temporal veins.

Retromandibular vein

Formed by a union of the maxillary vein and superficial temporal vein
It descends through the parotid gland and bifurcates within it
The anterior division passes forwards to join the facial vein, the posterior division is one of the tributaries of the external jugular vein

863
Q

A newborn infant is noted to have a unilateral cleft lip only. What is the most likely explanation for this process?

Incomplete fusion of the second branchial arch

Incomplete fusion of the nasolabial muscle rings

Incomplete fusion of the first branchial arch

Incomplete fusion of the third branchial arch

Incomplete fusion of the secondary palate

A

Unilateral isolated cleft lip represents a failure of nasolabial ring fusion. It is not related to branchial arch fusion. Arch disorders have a far more profound phenotype and malformation sequences.

Cleft lip and palate

Cleft lip and palate are the most common congenital deformity affecting the orofacial structures. Whilst they may be an isolated developmental malformation they are also a recognised component of more than 200 birth defects. The incidence is as high as 1 in 600 live births. The commonest variants are:
Isolated cleft lip (15%)
Isolated cleft palate (40%)
Combined cleft lip and palate (45%)
The aetiology of the disorder is multifactorial; both genetic (affected first degree relative increases risk) and environmental factors play a role.

Cleft lip
Cleft lip occurs as a result of disruption of the muscles of the upper lip and nasolabial region. These muscles comprise a chain of muscles viz; nasolabial, bilabial and labiomental. Defects may be unilateral or bilateral.

Cleft palate
The primary palate consists of all anatomical structures anterior to the incisive foramen. The secondary palate lies more posteriorly and is sub divided into the hard and soft palate. Cleft palate occurs as a result of non fusion of the two palatine shelves. Both hard and soft palate may be involved. Complete cases are associated with complete separation of the nasal septum and vomer from the palatine processes.

Treatment
Surgical reconstruction is the mainstay of management. The procedures are planned according to the extent of malformation and child age. Simple defects are managed as a single procedure. Complex malformations are usually corrected in stages. Affected individuals have a higher incidence of hearing and speech problems.

864
Q

A 50 year old lady presents with symptoms of a dry mouth that has been present for the past few months. She also has a sensation of grittiness in her eyes. On examination, she has a diffuse swelling of her parotid gland. There is no evidence of facial nerve palsy. What is the likely underlying diagnosis?

Sarcoid

Sjogrens syndrome

Adenoid cystic carcinoma

Pleomorphic adenoma

Lymphoma

A

Most patients with Sjogrens present in the post menopausal years. Multi system involvement is common.

Parotid gland clinical

Benign neoplasms
Up to 80% of all salivary gland tumours occur in the parotid gland and up to 80% of these are benign. There is no consistent correlation between the rate of growth and the malignant potential of the lesion. However, benign tumours should not invade structures such as the facial nerve.
With the exception of Warthins tumours, they are commoner in women than men. The median age of developing a lesion is in the 5th decade of life.

Benign tumour types
Tumour type Features
Benign pleomorphic adenoma or benign mixed tumor Most common parotid neoplasm (80%)
Proliferation of epithelial and myoepithelial cells of the ducts and an increase in stromal components
Slow growing, lobular, and not well encapsulated
Recurrence rate of 1-5% with appropriate excision (parotidectomy)
Recurrence possibly secondary to capsular disruption during surgery
Malignant degeneration occurring in 2-10% of adenomas observed for long periods, with carcinoma ex-pleomorphic adenoma occurring most frequently as adenocarcinoma
Warthin tumor (papillary cystadenoma lymphoma or adenolymphoma) Second most common benign parotid tumor (5%)
Most common bilateral benign neoplasm of the parotid
Marked male as compared to female predominance
Occurs later in life (sixth and seventh decades)
Presents as a lymphocytic infiltrate and cystic epithelial proliferation
May represent heterotopic salivary gland epithelial tissue trapped within intraparotid lymph nodes
Incidence of bilaterality and multicentricity of 10%
Malignant transformation rare (almost unheard of)
Monomorphic adenoma Account for less than 5% of tumours
Slow growing
Consist of only one morphological cell type (hence term mono)
Include; basal cell adenoma, canalicular adenoma, oncocytoma, myoepitheliomas
Haemangioma Should be considered in the differential of a parotid mass in a child
Accounts for 90% of parotid tumours in children less than 1 year of age
Hypervascular on imaging
Spontaneous regression may occur and malignant transformation is almost unheard of

Malignant salivary gland tumours
Types of malignancy

Mucoepidermoid carcinoma 30% of all parotid malignancies
Usually low potential for local invasiveness and metastasis (depends mainly on grade)
Adenoid cystic carcinoma Unpredictable growth pattern
Tendency for perineural spread
Nerve growth may display skip lesions resulting in incomplete excision
Distant metastasis more common (visceral rather than nodal spread)
5 year survival 35%
Mixed tumours Often a malignancy occurring in a previously benign parotid lesion
Acinic cell carcinoma Intermediate grade malignancy
May show perineural invasion
Low potential for distant metastasis
5 year survival 80%
Adenocarcinoma Develops from secretory portion of gland
Risk of regional nodal and distant metastasis
5 year survival depends upon stage at presentation, may be up to 75% with small lesions with no nodal involvement
Lymphoma Large rubbery lesion, may occur in association with Warthins tumours
Diagnosis should be based on regional nodal biopsy rather than parotid resection
Treatment is with chemotherapy (and radiotherapy)

Diagnostic evaluation
Plain x-rays may be used to exclude calculi
Sialography may be used to delineate ductal anatomy
FNAC is used in most cases
Superficial parotidectomy may be either diagnostic or therapeutic depending upon the nature of the lesion
Where malignancy is suspected the primary approach should be definitive resection rather than excisional biopsy
CT/ MRI may be used in cases of malignancy for staging primary disease

Treatment
For nearly all lesions this consists of surgical resection, for benign disease this will usually consist of a superficial parotidectomy. For malignant disease a radical or extended radical parotidectomy is performed. The facial nerve is included in the resection if involved. The need for neck dissection is determined by the potential for nodal involvement.

Other parotid disorders
HIV infection
Lymphoepithelial cysts associated with HIV occur almost exclusively in the parotid
Typically presents as bilateral, multicystic, symmetrical swelling
Risk of malignant transformation is low and management usually conservative

Sjogren syndrome
Autoimmune disorder characterised by parotid enlargement, xerostomia and keratoconjunctivitis sicca
90% of cases occur in females
Second most common connective tissue disorder
Bilateral, non tender enlargement of the gland is usual
Histologically, the usual findings are of a lymphocytic infiltrate in acinar units and epimyoepithelial islands surrounded by lymphoid stroma
Treatment is supportive
There is an increased risk of subsequent lymphoma

Sarcoid
Parotid involvement occurs in 6% of patients with sarcoid
Bilateral in most cases
Gland is not tender
Xerostomia may occur
Management of isolated parotid disease is usually conservative

865
Q

Which of the following fingers is not a point of attachment for the palmar interossei?

Middle finger

Little finger

Ring finger

Index finger

None of the above

A

The middle finger has no attachment of the palmar interosseous.
Note that there are 4 palmar interossei. The first is a small slip of muscle which arises from the ulnar side of the base of the first metacarpal and passes between the head of the first dorsal interosseous and the oblique head of adductor pollicis to insert into the ulnar base of the of the proximal phalanx of the thumb. The second arises from the ulnar side of the body of the second metacarpal and is inserted into the ulnar side of the extensor hood of the index. The third and fourth palmar interossei arise from the radial sides of the bodies of the 4th and 5th metacarpals respectively and insert into the radial sides of the extensor hoods of the ring and little fingers.

Hand

Anatomy of the hand
Bones
8 Carpal bones
5 Metacarpals
14 phalanges
Intrinsic Muscles 8 Interossei - Supplied by ulnar nerve
4 palmar-adduct fingers
4 dorsal- abduct fingers
Intrinsic muscles Lumbricals
Flex MCPJ and extend the IPJ.
Origin deep flexor tendon and insertion dorsal extensor hood mechanism.
Innervation: 1st and 2nd- median nerve, 3rd and 4th- deep branch of the ulnar nerve.
Thenar eminence
Abductor pollicis brevis
Opponens pollicis
Flexor pollicis brevis
Hypothenar eminence
Opponens digiti minimi
Flexor digiti minimi brevis
Abductor digiti minimi

Fascia and compartments of the palm
The fascia of the palm is continuous with the antebrachial fascia and the fascia of the dorsum of the hand. The palmar fascia is thin over the thenar and hypothenar eminences. In contrast, the central palmar fascia is relatively thick. The palmar aponeurosis covers the soft tissues and overlies the flexor tendons. The apex of the palmar aponeurosis is continuous with the flexor retinaculum and the palmaris longus tendon. Distally, it forms four longitudinal digital bands that attach to the bases of the proximal phalanges, blending with the fibrous digital sheaths.
A medial fibrous septum extends deeply from the medial border of the palmar aponeurosis to the 5th metacarpal. Lying medial to this are the hypothenar muscles. In a similar fashion, a lateral fibrous septum extends deeply from the lateral border of the palmar aponeurosis to the 3rd metacarpal. The thenar compartment lies lateral to this area.
Lying between the thenar and hypothenar compartments is the central compartment. It contains the flexor tendons and their sheaths, the lumbricals, the superficial palmar arterial arch and the digital vessels and nerves.
The deepest muscular plane is the adductor compartment, which contains adductor pollicis.

Short muscles of the hand
These comprise the lumbricals and interossei. The four slender lumbrical muscles flex the fingers at the metacarpophalangeal joints and extend the interphalangeal joint. The four dorsal interossei are located between the metacarpals and the four palmar interossei lie on the palmar surface of the metacarpals in the interosseous compartment of the hand.

Long flexor tendons and sheaths in the hand
The tendons of FDS and FDP enter the common flexor sheath deep to the flexor retinaculum. The tendons enter the central compartment of the hand and fan out to their respective digital synovial sheaths. Near the base of the proximal phalanx, the tendon of FDS splits to permit the passage of FDP. The FDP tendons are attached to the margins of the anterior aspect of the base of the distal phalanx.
The fibrous digital sheaths contain the flexor tendons and their synovial sheaths. These extend from the heads of the metacarpals to the base of the distal phalanges.

Palmar Interossei
Note that there are 4 palmar interossei. The first is a small slip of muscle which arises from the ulnar side of the base of the first metacarpal and passes between the head of the first dorsal interosseous and the oblique head of adductor pollicis to insert into the ulnar base of the of the proximal phalanx of the thumb. The second arises from the ulnar side of the body of the second metacarpal and is inserted into the ulnar side of the extensor hood of the index. The third and fourth palmar interossei arise from the radial sides of the bodies of the 4th and 5th metacarpals respectively and insert into the radial sides of the extensor hoods of the ring and little fingers.

866
Q

A 19 year old man presents with diarrhoea and rectal bleeding that has been present for the past two weeks. In addition, he has noticed that he has had faecal incontinence at night. What is the most likely cause?

Viral gastroenteritis

Inflammatory bowel disease

Intersphincteric abscess

Irritable bowel syndrome and haemorrhoids

Irritable bowel syndrome and a fissure in ano

A

Nocturnal diarrhea and incontinence is a key feature in the history and is strongly suggestive of a diagnosis of IBD. More benign IBS presentations seldom have nocturnal events or a short history.

Rectal bleeding

Rectal bleeding is a common cause for patients to be referred to the surgical clinic. In the clinical history it is useful to try and localise the anatomical source of the blood. Bright red blood is usually of rectal anal canal origin, whilst dark red blood is more suggestive of a proximally sited bleeding source. Blood which has entered the GI tract from a gastro-duodenal source will typically resemble malaena due to the effects of the digestive enzymes on the blood itself.

In the table below we give some typical bleeding scenarios together with physical examination findings and causation.

Cause Type of bleeding Features in history Examination findings
Fissure in ano Bright red rectal bleeding Painful bleeding that occurs post defecation in small volumes. Usually antecedent features of constipation Muco-epithelial defect usually in the midline posteriorly (anterior fissures more likely to be due to underlying disease)
Haemorroids Bright red rectal bleeding Post defecation bleeding noted both on toilet paper and drips into pan. May be alteration of bowel habit and history of straining. No blood mixed with stool. No local pain. Normal colon and rectum. Proctoscopy may show internal haemorrhoids. Internal haemorrhoids are usually impalpable.
Crohns disease Bright red or mixed blood Bleeding that is accompanied by other symptoms such as altered bowel habit, malaise, history of fissures (especially anterior) and abscesses. Perineal inspection may show fissures or fistulae. Proctoscopy may demonstrate indurated mucosa and possibly strictures. Skip lesions may be noted at colonoscopy.
Ulcerative colitis Bright red bleeding often mixed with stool Diarrhoea, weight loss, nocturnal incontinence, passage of mucous PR. Proctitis is the most marked finding. Peri anal disease is usually absent. Colonoscopy will show continuous mucosal lesion.
Rectal cancer Bright red blood mixed volumes Alteration of bowel habit. Tenesmus may be present. Symptoms of metastatic disease. Usually obvious mucosal abnormality. Lesion may be fixed or mobile depending upon disease extent. Surrounding mucosa often normal, although polyps may be present.

Image showing a fissure in ano. Typically these are located posteriorly and in the midline. Fissures at other sites may be associated with underlying disease.

Investigation
All patients presenting with rectal bleeding require digital rectal examination and procto-sigmoidoscopy as a minimal baseline.
Remember that haemorrhoids are typically impalpable and to attribute bleeding to these in the absence of accurate internal inspection is unsatisfactory.
In young patients with no other concerning features in the history a carefully performed sigmoidoscopy that demonstrates clear haemorrhoidal disease may be sufficient. If clear views cannot be obtained then patients require bowel preparation with an enema and a flexible sigmoidscopy performed.
In those presenting with features of altered bowel habit or suspicion of inflammatory bowel disease a colonoscopy is the best test.
Patients with excessive pain who are suspected of having a fissure may require an examination under general or local anaesthesia.
In young patients with external stigmata of fissure and a compatible history it is acceptable to treat medically and defer internal examination until the fissure is healed. If the fissure fails to heal then internal examination becomes necessary along the lines suggested above to exclude internal disease.

Special tests
In patients with a malignancy of the rectum the staging investigations comprise an MRI of the rectum to identify circumferential resection margin compromise and to identify mesorectal nodal disease. In addition to this CT scanning of the chest abdomen and pelvis is necessary to stage for more distant disease. Some centres will still stage the mesorectum with endo rectal ultrasound but this is becoming far less common.

Patients with fissure in ano who are being considered for surgical sphincterotomy and are females who have an obstetric history should probably have ano rectal manometry testing performed together with endo anal ultrasound. As this service is not universally available it is not mandatory but in the absence of such information there are continence issues that may arise following sphincterotomy.

Management

Disease Management
Fissure in ano GTN ointment 0.2% or diltiazem cream applied topically is the usual first line treatment. Botulinum toxin for those who fail to respond. Internal sphincterotomy for those who fail with botox, can be considered earlier in males.
Haemorroids Lifestyle advice, for small internal haemorrhoids can consider injection sclerotherapy or rubber band ligation. For external haemorrhoids consider haemorrhoidectomy. Modern options include HALO procedure and stapled haemorrhoidectomy.
Inflammatory bowel disease Medical management- although surgery may be needed for fistulating Crohns (setons).
Rectal cancer Anterior resection or abdomino-perineal excision of the colon and rectum. Total mesorectal excision is now standard of care. Most resections below the peritoneal reflection will require defunctioning ileostomy. Most patients will require preoperative radiotherapy.

867
Q

Which of the following is true in relation to the sartorius muscle?

Innervated by the deep branch of the femoral nerve

Inserts at the fibula

It is the shortest muscle in the body

Forms the Pes anserinus with Gracilis and semitendinosus muscle

Causes extension of the knee

A

It is innervated by the superficial branch of the femoral nerve. It is a component of the pes anserinus.

Sartorius

  • Longest strap muscle in the body
    Most superficial muscle in the anterior compartment of the thigh

Origin Anterior superior iliac spine
Insertion Medial surface of the of the body of the tibia (upper part). It inserts anterior to gracilis and semitendinosus
Nerve Supply Femoral nerve (L2,3 -root values for sartorius)
Action
Flexor of the hip and knee, slight abducts the thigh and rotates it laterally
It assists with medial rotation of the tibia on the femur. For example it would play a pivotal role in placing the right heel onto the left knee ( and vice versa)
Important relations The middle third of this muscle, and its strong underlying fascia forms the roof of the adductor canal , in which lie the femoral vessels, the saphenous nerve and the nerve to vastus medialis.

868
Q

A 55 year old lady presents with a neck mass. On examination, there is a swelling of the neck and enlarged deep cervical lymph nodes. A biopsy is taken from the nodes and is noted to be CD20 positive. What is the most likely diagnosis?

Hashimotos thyroiditis

Thyroid gland lymphoma

Papillary thyroid cancer

Follicular thyroid cancer

Anaplastic thyroid cancer

A

CD20 is a B cell lymphoma marker and is useful in histochemical analysis of biopsies. Even without knowing that, it is still possible to approach questions like this with a logical exclusion approach. Follicular lesions tend not to have lymphatic involvement and the same is true of Hashimotos thyroiditis. Papillary thyroid cancers do remain a possible tumour as they have lymphatic involvement (its for this that knowledge of the CD20 is required).

Thyroid malignancy

Papillary carcinoma
Commonest sub-type
Accurately diagnosed on fine needle aspiration cytology
Histologically, they may demonstrate psammoma bodies (areas of calcification) and so called ‘orphan Annie’ nuclei
They typically metastasise via the lymphatics and thus laterally located apparently ectopic thyroid tissue is usually a metastasis from a well differentiated papillary carcinoma

Follicular carcinoma
Are less common than papillary lesions
Like papillary tumours, they may present as a discrete nodule. Although they appear to be well encapsulated macroscopically there is invasion on microscopic evaluation
Lymph node metastases are uncommon and these tumours tend to spread haematogenously. This translates into a higher mortality rate
Follicular lesions cannot be accurately diagnosed on fine needle aspiration cytology and thus all follicular FNA’s (THY 3f) will require at least a hemi thyroidectomy

Anaplastic carcinoma
Less common and tend to occur in elderly females
Disease is usually advanced at presentation and often only palliative decompression and radiotherapy can be offered.

Medullary carcinoma
These are tumours of the parafollicular cells ( C Cells) and are of neural crest origin.
The serum calcitonin may be elevated which is of use when monitoring for recurrence.
They may be familial and occur as part of the MEN -2A disease spectrum.
Spread may be either lymphatic or haematogenous and as these tumours are not derived primarily from thyroid cells they are not responsive to radioiodine.

Lymphoma
These respond well to combined chemoradiotherapy
Radical surgery is unnecessary once the disease has been diagnosed on biopsy material. Such biopsy material is not generated by an FNA and thus a core biopsy has to be obtained (with care!).

869
Q

A 78-year-old woman is discharged following a fractured neck of femur. On review, she is making good progress but consideration is given to secondary prevention of further fractures. Unfortunately the orthogeriatricians are all on annual leave and the consultant has asked you to arrange suitable management. Which is the best option?

Alendronate

Alendronate, calcium and vitamin D supplementation

Strontium

Arrange a DEXA scan

Hormone replacement therapy

A

A bisphosphonate, calcium and vitamin D supplementation should be given to all patients aged over 75 years after having a fracture. A DEXA scan is only needed of the patient is aged below 75 years. Hormone replacement therapy has been shown to reduce vertebral and non vertebral fractures, however the risks of cardiovascular disease and breast malignancy make this a less favourable option.

Osteoporosis: secondary prevention

NICE guidelines were updated in 2008 on the secondary prevention of osteoporotic fractures in postmenopausal women.

Key points include
Treatment is indicated following osteoporotic fragility fractures in postmenopausal women who are confirmed to have osteoporosis (a T-score of - 2.5 SD or below).
In women aged 75 years or older, a DEXA scan may not be required if the responsible clinician considers it to be clinically inappropriate or unfeasible
Vitamin D and calcium supplementation should be offered to all women unless the clinician is confident they have adequate calcium intake and are vitamin D replete
Alendronate is first-line
Around 25% of patients cannot tolerate alendronate, usually due to upper gastrointestinal problems. These patients should be offered risedronate or etidronate (see treatment criteria below)
Strontium ranelate and raloxifene are recommended if patients cannot tolerate bisphosphonates (see treatment criteria below)

Supplementary notes on treatment

Bisphosphonates
Alendronate, risedronate and etidronate are all licensed for the prevention and treatment of post-menopausal and glucocorticoid-induced osteoporosis
All three have been shown to reduce the risk of both vertebral and non-vertebral fractures although alendronate, risedronate may be superior to etidronate in preventing hip fractures
Ibandronate is a once-monthly oral bisphosphonate

Vitamin D and calcium
Poor evidence base to suggest reduced fracture rates in the general population at risk of osteoporotic fractures - may reduce rates in frail, housebound patients

Raloxifene - selective oestrogen receptor modulator (SERM)
Has been shown to prevent bone loss and to reduce the risk of vertebral fractures, but has not yet been shown to reduce the risk of non-vertebral fractures
Has been shown to increase bone density in the spine and proximal femur
May worsen menopausal symptoms
Increased risk of thromboembolic events
May decrease risk of breast cancer

Strontium ranelate
‘Dual action bone agent’ - increases deposition of new bone by osteoblasts and reduces the resorption of bone by osteoclasts
Strong evidence base, may be second-line treatment in near future
Increased risk of thromboembolic events

870
Q

A 34 year old lady undergoes a thyroidectomy for Graves disease. Post operatively she develops a tense haematoma in the neck. In which of the following fascial planes will it be contained?

Gerotas fascia

Waldeyers fascia

Pretracheal fascia

Sibsons fascia

Clavipectoral fascia

A

The pretracheal fascia encloses the thyroid and is unyielding. Therefore tense haematomas can develop

Thyroid gland

Right and left lobes connected by isthmus
Surrounded by sheath from pretracheal layer of deep fascia
Apex: Lamina of thyroid cartilage
Base: 4th-5th tracheal ring
Pyramidal lobe: from isthmus
May be attached to foramen caecum at the base of the tongue

Relations
Anteromedially
Sternothyroid
Superior belly of omohyoid
Sternohyoid
Anterior aspect of sternocleidomastoid
Posterolaterally Carotid sheath
Medially
Larynx
Trachea
Pharynx
Oesophagus
Cricothyroid muscle
External laryngeal nerve (near superior thyroid artery)
Recurrent laryngeal nerve (near inferior thyroid artery)
Posterior
Parathyroid glands
Anastomosis of superior and inferior thyroid arteries
Isthmus
Anteriorly: Sternothyroids, sternohyoids, anterior jugular veins
Posteriorly: 2nd, 3rd, 4th tracheal rings (attached via Ligament of Berry)

Blood Supply
Arterial
Superior thyroid artery (1st branch of external carotid)
Inferior thyroid artery (from thyrocervical trunk)
Thyroidea ima (in 10% of population -from brachiocephalic artery or aorta)
Venous
Superior and middle thyroid veins - into the IJV
Inferior thyroid vein - into the brachiocephalic veins

871
Q

A 55 year old man is due to undergo a radical prostatectomy for carcinoma of the prostate gland. Which of the following vessels directly supplies the prostate?

External iliac artery

Common iliac artery

Internal iliac artery

Inferior vesical artery

None of the above

A

The arterial supply to the prostate gland is from the inferior vesical artery, it is a branch of the prostatovesical artery. The prostatovesical artery usually arises from the internal pudendal and inferior gluteal arterial branches of the internal iliac artery.

Prostate gland

The prostate gland is approximately the shape and size of a walnut and is located inferior to the bladder. It is separated from the rectum by Denonvilliers fascia and its blood supply is derived from the internal iliac vessels (via inferior vesical artery). The internal sphincter lies at the apex of the gland and may be damaged during prostatic surgery, affected individuals may complain of retrograde ejaculation.

Summary of prostate gland
Arterial supply Inferior vesical artery (from internal iliac)
Venous drainage Prostatic venous plexus (to paravertebral veins)
Lymphatic drainage Internal iliac nodes
Innervation Inferior hypogastric plexus
Dimensions
Transverse diameter (4cm)
AP diameter (2cm)
Height (3cm)
Lobes
Posterior lobe: posterior to urethra
Median lobe: posterior to urethra, in between ejaculatory ducts
Lateral lobes x 2
Isthmus
Zones
Peripheral zone: subcapsular portion of posterior prostate. Most prostate cancers are here
Central zone
Transition zone
Stroma

Relations
Anterior Pubic symphysis
Prostatic venous plexus
Posterior Denonvilliers fascia
Rectum
Ejaculatory ducts
Lateral Venous plexus (lies on prostate)
Levator ani (immediately below the puboprostatic ligaments)

872
Q

A 28-year-old female undergoes a renal transplant for focal segmental glomerulosclerosis. Within hours of the operation the patient becomes unwell with features consistent with severe systemic inflammatory response syndrome. The patient is immediately taken back to theatre and the transplanted kidney is removed. What type of immunoglobulins are responsible for the graft rejection?

IgE

IgM

IgG

IgD

IgA

A

Hyperacute graft rejection is due to pre-existent antibodies to HLA antigens and is therefore IgG mediated

Renal transplant:HLA typing and graft failure

The human leucocyte antigen (HLA) system is the name given to the major histocompatibility complex (MHC) in humans. It is coded for on chromosome 6.

Some basic points on the HLA system
Class 1 antigens include A, B and C. Class 2 antigens include DP,DQ and DR
When HLA matching for a renal transplant the relative importance of the HLA antigens are as follows DR > B > A

Graft survival
1 year = 90%, 10 years = 60% for cadaveric transplants
1 year = 95%, 10 years = 70% for living-donor transplants

Post-op problems
ATN of graft
Vascular thrombosis
Urine leakage
UTI

Hyperacute acute rejection
Due to antibodies against donor HLA type 1 antigens
Rarely seen due to HLA matching

Acute graft failure (< 6 months)
Usually due to mismatched HLA
Other causes include cytomegalovirus infection
Management: give steroids, if resistant use monoclonal antibodies

Causes of chronic graft failure (> 6 months)
Chronic allograft nephropathy
Ureteric obstruction
Recurrence of original renal disease (MCGN > IgA > FSGS)

873
Q

A 39 year old lady has undergone surgery for breast cancer. As part of the histopathology report the pathologist provides the surgeon with a Nottingham Prognostic Index score of 6.4. He also states that the tumour size is 2cm. Which of the following inferences can be made in relation to this statement?

The tumour is likely to be grade 1

Vascular invasion is present

Lymph node metastasis are definitely present

The tumour is oestrogen receptor positive

None of the above

A

A score of this value is unlikely to be reached with a grade 1 tumour and a size of 2cm. Therefore lymph node metastasis are definitely present. In addition since the maximal score for lymph node metastasis is 3 the tumour is likely be of a higher grade (see below). The Nottingham Prognostic Index provides no information about oestrogen receptor status or the presence or absence of vascular invasion.

Nottingham prognostic index

The Nottingham Prognostic Index can be used to give an indication of survival following breast cancer surgery. In this system, the tumour size is weighted less heavily than other major prognostic parameters.

Calculation of NPI
Tumour Size x 0.2 + Lymph node score(From table below)+Grade score(From table below).

Score Lymph nodes involved Grade
1 0 1
2 1-3 2
3 >3 3

Prognosis

Score Percentage 5 year survival
2.0 to 2.4 93%
2.5 to 3.4 85%
3.5 to 5.4 70%
>5.4 50%

This data was originally published in 1992. It should be emphasised that other factors such as vascular invasion and receptor status also impact on survival and are not included in this data and account for varying prognoses often cited in the literature.

Reference
Galea, M.H., et al., The Nottingham Prognostic Index in primary breast cancer. Breast Cancer Res Treat, 1992. 22(3): p. 207-19.

874
Q

A 28 year old female attends the gynaecology unit for a D+C following an incomplete miscarriage. She has previously had recurrent pulmonary embolic events. After the procedure she is persistently bleeding. Her APTT is 52 (increased). What is the most likely cause?

Antiphospholipid syndrome

DIC

Haemophilia B

von Willebrand disease

Factor V Leiden deficiency

A

A combination of thromboembolism and coagulation issues in a young woman should raise the possibility of antiphospholipid syndrome. Other features may include foetal loss, venous and arterial thrombosis and thrombocytopenia. A Lupus anticoagulant may be present and the APTT is prolonged.

Abnormal coagulation

Cause Factors affected
Heparin Prevents activation factors 2,9,10,11
Warfarin Affects synthesis of factors 2,7,9,10
DIC Factors 1,2,5,8,11
Liver disease Factors 1,2,5,7,9,10,11

Interpretation blood clotting test results
Disorder APTT PT Bleeding time
Haemophilia Increased Normal Normal
von Willebrand’s disease Increased Normal Increased
Vitamin K deficiency Increased Increased Normal

875
Q

Which of the nerves listed below is responsible for providing voluntary control of the urethral sphincter?

Inferior hypogastric plexus

Superior hypogastric plexus

Obturator nerve

Femoral nerve

Pudendal nerve

A

The hypogastric plexuses provide autonomic control of the bladder. However, voluntary control of the urethral sphincter is provided by the pudendal nerve.

Pudendal nerve

The pudendal nerve arises from nerve roots S2, S3 and S4 and exits the pelvis through the greater sciatic foramen. It re-enters the perineum through the lesser sciatic foramen. It travels inferior to give innervation to the anal sphincters and external urethral sphincter. It also provides cutaneous innervation to the region of perineum surrounding the anus and posterior vulva.

Traction and compression of the pudendal nerve by the foetus in late pregnancy may result in late onset pudendal neuropathy which may be part of the process involved in the development of faecal incontinence.

876
Q

Which of the following does not need monitoring during home parenteral nutritional support?

Folate levels

Zinc levels

Vitamin D

Thyroid function

Bone densitometry

A

Nutrition Monitoring-NICE guidelines

Weight: daily if fluid balance concerns, otherwise weekly reducing to monthly
BMI: at start of feeding and then monthly
If weight cannot be obtained: monthly mid arm circumference or triceps skin fold thickness
Daily electrolytes until levels stable. Then once or twice a week.
Weekly glucose, phosphate, magnesium, LFTs, Ca, albumin, FBC, MCV
levels if stable
2-4 weekly Zn, Folate, B12 and Cu levels if stable
3-6 monthly iron and ferritin levels, manganese (if on home parenteral regime)
6 monthly vitamin D
Bone densitometry initially on starting home parenteral nutrition then every 2 years

877
Q

A 23 year old man has a cannula inserted into his cephalic vein. Through which structure does the cephalic vein pass?

Interosseous membrane

Triceps

Pectoralis major

Clavipectoral fascia

Tendon of biceps

A

The cephalic vein is a favored vessel for arteriovenous fistula formation and should be preserved in patients with end stage renal failure
The cephalic vein penetrates the clavipectoral fascia (but not the pectoralis major) prior to terminating in the axillary vein.

Cephalic vein

Path
Dorsal venous arch drains laterally into the cephalic vein
Crosses the anatomical snuffbox and travels laterally up the arm
At the antecubital fossa connected to the basilic vein by the median cubital vein
Pierces deep fascia of deltopectoral groove to join axillary vein

878
Q

Which of the following is not a typical feature of irritable bowel syndrome?

A change in the consistency of stools

Abdominal pain relieved with defecation

A change in frequency of defecation

Abdominal bloating

Pain at a single fixed site

A

The pain or discomfort of IBS is typically migratory and variable in intensity. Pain at a fixed site is suggestive of malignancy.
Abdominal bloating is an extremely common feature.

Irritable bowel syndrome

The diagnosis of irritable bowel syndrome is made according to the ROME IV diagnostic criteria which state:

Recurrent abdominal pain or discomfort at 3 days per month for the past 3 months associated with two or more of the following:
Improvement with defecation.
Onset associated with a change in the frequency of stool.
Onset associated with a change in the form of the stool.

Features such as lethargy, nausea, backache and bladder symptoms may also support the diagnosis

Red flag features should be inquired about:

Rectal bleeding
Unexplained/unintentional weight loss
Family history of bowel or ovarian cancer
Onset after 60 years of age

Suggested investigations are:

Full blood count
ESR/CRP
Coeliac disease screen (tissue transglutaminase antibodies)
Colonoscopy (if worrying symptoms, positive family history)
Thyroid function tests
Glucose (ensure not diabetic)

The NICE criteria state that blood tests alone will suffice in people fulfilling the diagnostic criteria. We would point out that luminal colonic studies should be considered early in patients with altered bowel habit referred to hospital and a diagnosis of IBS should still be largely one of exclusion.

Treatment
Usually reduce fibre intake.
Tailored prescriptions of laxatives or loperamide according to clinical picture.
Dietary modification (caffeine avoidance, less carbonated drinks).
Consider low dose tricyclic antidepressants if pain is a dominant symptom.
Biofeedback may help.

879
Q

A 45 year old man has a long femoral line inserted to provide CVP measurements. The catheter passes from the common iliac vein into the inferior vena cava. At which of the following vertebral levels will this occur?

L5

L4

S1

L3

L2

A

The common iliac veins fuse with the IVC at L5.
Inferior vena cava

Origin
L5

Path
Left and right common iliac veins merge to form the IVC.
Passes right of midline
Paired segmental lumbar veins drain into the IVC throughout its length
The right gonadal vein empties directly into the cava and the left gonadal vein generally empties into the left renal vein.
The next major veins are the renal veins and the hepatic veins
Pierces the central tendon of diaphragm at T8
Right atrium

Relations
Anteriorly Small bowel, first and third part of duodenum, head of pancreas, liver and bile duct, right common iliac artery, right gonadal artery
Posteriorly Right renal artery, right psoas, right sympathetic chain, coeliac ganglion

Levels
Level Vein
T8 Hepatic vein, inferior phrenic vein, pierces diaphragm
L1 Right suprarenal vein, renal vein
L2 Gonadal vein
L1-5 Lumbar veins
L5 Common iliac vein, formation of IVC

880
Q

An elderly lady who presented with weight loss and malabsorption was found to have amyloid of the small bowel. On presentation she was found to have osteomalacia and was hypocalcaemic. Over the past seven days she has received total parenteral nutrition with adequate calcium replacement. Despite this she remained hypocalcaemic. Deficiency of which of the following electrolytes is most likely to account for this process?

Magnesium

Potassium

Sodium

Phosphate

None of the above

A

Patients with malabsorption may develop magnesium deficiency, although her TPN feeds may have contained magnesium it may not have been sufficient to correct her losses. Sodium, phosphate and potassium would not have this effect on serum calcium.

Combined deficiency of magnesium and calcium

Magnesium is required for both PTH secretion and its action on target tissues. Hypomagnesaemia may both cause hypocalcaemia and render patients unresponsive to treatment with calcium and vitamin D supplementation.

Magnesium is the fourth most abundant cation in the body. The body contains 1000mmol, with half contained in bone and the remainder in muscle, soft tissues and extracellular fluid. There is no one specific hormonal control of magnesium and various hormones including PTH and aldosterone affect the renal handling of magnesium.

Magnesium and calcium interact at a cellular level also and as a result decreased magnesium will tend to affect the permeability of cellular membranes to calcium, resulting in hyperexcitability.

881
Q

A 22 year old man is stabbed in the chest at the level of the junction between the sternum and manubrium. Which structure is at greatest risk?

Left atrium

Oesophagus

Thyroid gland

Inferior vena cava

Aortic arch

A

Mnemonic for lower sternal angle:
Red, white, blue and air 2

Red- arch of aorta
White- the thoracic duct crosses at the midline
Bue- azygous joins the svc
Air- tracheal bifurcation
2- costal cartilage of the second rib

At the level of the Angle of Louis (Manubriosternal angle), is the surface marking for the aortic arch. The oesophagus is posteriorly located and at less risk.
Sternal angle

Anatomical structures at the level of the manubrium and upper sternum
Upper part of the manubrium
Left brachiocephalic vein
Brachiocephalic artery
Left common carotid
Left subclavian artery
Lower part of the manubrium/ manubrio-sternal angle
Costal cartilages of the 2nd ribs
Transition point between superior and inferior mediastinum
Arch of the aorta
Tracheal bifurcation
Union of the azygos vein and superior vena cava
The thoracic duct crosses to the midline

882
Q

Which of the following statements relating to neutrophil polymorphs is true?

Produce nitrogen peroxide as a microbicidal agent

Not involved in opsonisation

Deficiency leads to AIDS

Neutrophil disorders may result in chronic granulomatous diseases

Have a lifespan of 9 hours

A

Neutrophils are the main cells of acute inflammation, important action against gram -ve and +ve bacteria
Appearance of segmented nucleus and granulated cytoplasm
Have a lifespan of 1-3 days (shorter when consumed during septic process, though 9 hours is unusual)
Actions include: movement, opsonise microorganisms, phagocytosis & intracellular killing of microorganisms via aerobic (produce HYDROGEN PEROXIDE) & anaerobic mechanisms.
Neutrophil disorders include chronic granulomatous diseases: rare
AIDS associated with T cell deficiency

Acute inflammation

Inflammation is the reaction of the tissue elements to injury. Vascular changes occur, resulting in the generation of a protein rich exudate. So long as the injury does not totally destroy the existing tissue architecture, the episode may resolve with restoration of original tissue architecture.

Vascular changes
Vasodilation occurs and persists throughout the inflammatory phase.
Inflammatory cells exit the circulation at the site of injury.
The equilibrium that balances Starlings forces within capillary beds is disrupted and a protein rich exudate will form as the vessel walls also become more permeable to proteins.
The high fibrinogen content of the fluid may form a fibrin clot. This has several important immunomodulatory functions.

Sequelae
Resolution
Typically occurs with minimal initial injury
Stimulus removed and normal tissue architecture results
Organisation
Delayed removal of exudate
Tissues undergo organisation and usually fibrosis
Suppuration
Typically formation of an abscess or an empyema
Sequestration of large quantities of dead neutrophils
Progression to chronic inflammation
Coupled inflammatory and reparative activities
Usually occurs when initial infection or suppuration has been inadequately managed

Causes
Infections e.g. Viruses, exotoxins or endotoxins released by bacteria
Chemical agents
Physical agents e.g. Trauma
Hypersensitivity reactions
Tissue necrosis

Presence of neutrophil polymorphs is a histological diagnostic feature of acute inflammation

883
Q

A 66 year old female has long standing mixed arteriovenous ulcers of the lower leg. Over the past 6 months one of the ulcers has become much worse and despite a number of different topical therapies is increasing in size. What is the most likely diagnosis?

Neuropathic ulcer

Mixed ulcer

Deep venous insufficiency

Marjolins ulcer

Chronic obliterative arterial disease

A

Marjolin’s ulcer is a squamous cell carcinoma occurring at sites of chronic inflammation or previous injury.

Lower leg ulcers

Venous leg ulcers
Most due to venous hypertension, secondary to chronic venous insufficiency (other causes include calf pump dysfunction or neuromuscular disorders)
Ulcers form due to capillary fibrin cuff or leucocyte sequestration
Features of venous insufficiency include oedema, brown pigmentation, lipodermatosclerosis, eczema
Location above the ankle, painless
Deep venous insufficiency is related to previous DVT and superficial venous insufficiency is associated with varicose veins
Doppler ultrasound looks for presence of reflux and duplex ultrasound looks at the anatomy/ flow of the vein
Management: 4 layer compression banding after exclusion of arterial disease or surgery
If fail to heal after 12 weeks or >10cm2 skin grafting may be needed

Marjolin’s ulcer Squamous cell carcinoma
Occurring at sites of chronic inflammation e.g; burns, osteomyelitis after 10-20 years
Mainly occur on the lower limb

Arterial ulcers
Occur on the toes and heel
Painful
There may be areas of gangrene
Cold with no palpable pulses
Low ABPI measurements

Neuropathic ulcers
Commonly over plantar surface of metatarsal head and plantar surface of hallux
The plantar neuropathic ulcer is the condition that most commonly leads to amputation in diabetic patients
Due to pressure
Management includes cushioned shoes to reduce callus formation

Pyoderma gangrenosum Associated with inflammatory bowel disease/RA
Can occur at stoma sites
Erythematous nodules or pustules which ulcerate

884
Q

A 28 year old lady with hyperthyroidism is started on carbimazole to treat the condition. What is the main mechanism of action?

Inhibition of thyroid stimulating hormone release

Increased degradation of T4 within the thyroid colloid

Increased degradation of T3 within the thyroid colloid

Inhibits release of T3 and T4 into the circulation

Inhibition of thyroid peroxidase enzyme

A

Carbimazole is used to treat hyperthyroidism. Carbimazole is a pro-drug as after absorption it is converted to the active form, methimazole. Methimazole prevents thyroid peroxidase enzyme from iodinating and coupling the tyrosine residues on thyroglobulin, hence reducing the production of the thyroid hormones T3 and T4. Clinical effects take some months to occur as thyroid globulin stores remain for some time after treatment commences.

Thyroid hormones

Hormones of the thyroid gland
Triiodothyronine T3 Major hormone active in target cells
Thyroxine T4 Most prevalent form in plasma, less biologically active than T3
Calcitonin Lowers plasma calcium

Synthesis and secretion of thyroid hormones
Thyroid actively concentrates iodide to twenty five times the plasma concentration.
Iodide is oxidised by peroxidase in the follicular cells to atomic iodine which then iodinates tyrosine residues contained in thyroglobulin.
Iodinated tyrosine residues in thyroglobulin undergo coupling to either T3 or T4.
Process is stimulated by TSH, which stimulates secretion of thyroid hormones.
The normal thyroid has approximately 3 month reserves of thyroid hormones.

LATS and Graves disease
In Graves disease, patients develop IgG antibodies to the TSH receptors on the thyroid gland. This results in chronic and long term stimulation of the gland with release of thyroid hormones. The typically situation is raised thyroid hormones and low TSH. Thyroid receptor autoantibodies should be checked in individuals presenting with hyperthyroidism as they are present in up to 85% cases.

885
Q

A keen surgical trainee is about to embark on her first hemi arthroplasty for a fractured neck of femur. In the anaesthetic room the patient is given 1.2g intravenous co-amoxiclav. There is a possible history of penicillin allergy but the patient has dementia and the history is not checked. The patient then develops severe respiratory compromise and haemodynamic collapse. Which pathological process accounts for this event?

Binding of the drug to circulating IgG class antibodies

Recognition of the drug by IgE receptors on mast cells

Drug initiated formation of hapten-protein complexes

Binding of the drug to circulating IgM class antibodies

None of the above

A

Anaphylactic shock:
Antigen recognised by IgE molecules on the surface of mast cells resulting in rapid degranulation with release of histamine and other inflammatory cytokines.
This is a case of anaphylactic shock. In anaphylaxis the mast cells degranulate.

Anaphylactic shock

  • Suspect if there has been exposure to an allergen

Management
- Remove allergen
- ABCD
- Drugs:

Adrenaline 1:1000 0.5ml INTRAMUSCULARLY (not IV). Repeat after 5 mins if no response.
Consider antihistamine if ongoing local symptoms.

In the refractory anaphylaxis guidance, consideration is given to adrenaline infusions.

Reference
Emergency treatment of anaphylactic reactions. Guidelines for healthcare providers. Working Group of the Resuscitation Council (UK).2021

886
Q

A 58 year old lady undergoes a screening mammogram and appearances are suspicious for ductal carcinoma in situ. A stereotactic core biopsy is performed. If ductal carcinoma in situ is to be diagnosed, which of the following pathological features must not be present?

Nuclear pleomorphism

Coarse chromatin

Abnormal mitoses

Angiogenesis

Dysplastic cells infiltrating the suspensory ligaments of the breast

A

The presence of invasion is a hallmark of invasive disease and thus would not be a feature of DCIS. Angiogenesis may occur in association with high grade DCIS.

Histopathology of malignancy

Abnormal tissue architecture
Coarse chromatin
Invasion of basement membrane*
Abnormal mitoses
Angiogenesis
De-differentiation
Areas of necrosis
Nuclear pleomorphism

*= Those features that distinguish invasive malignancy from in situ disease

887
Q

In patients with an annular pancreas where is the most likely site of obstruction?

The first part of the duodenum

The second part of the duodenum

The fourth part of the duodenum

The third part of the duodenum

The duodeno-jejunal flexure

A

The pancreas develops from two foregut outgrowths (ventral and dorsal). During rotation the ventral bud and adjacent gallbladder and bile duct lie together and fuse. When the pancreas fails to rotate normally it can compress the duodenum with development of obstruction. Usually occurring as a result of associated duodenal malformation. The second part of the duodenum is the commonest site.
Pancreas

The pancreas is a retroperitoneal organ and lies posterior to the stomach. It may be accessed surgically by dividing the peritoneal reflection that connects the greater omentum to the transverse colon. The pancreatic head sits in the curvature of the duodenum. Its tail lies close to the hilum of the spleen, a site of potential injury during splenectomy.

Relations
Posterior to the pancreas
Pancreatic head Inferior vena cava
Common bile duct
Right and left renal veins
Superior mesenteric vein and artery
Pancreatic neck Superior mesenteric vein, portal vein
Pancreatic body- Left renal vein
Crus of diaphragm
Psoas muscle
Adrenal gland
Kidney
Aorta
Pancreatic tail Left kidney

Anterior to the pancreas
Pancreatic head 1st part of the duodenum
Pylorus
SMA and SMV(uncinate process)
Pancreatic body Stomach
Duodenojejunal flexure
Pancreatic tail Splenic hilum

Superior to the pancreas
Coeliac trunk and its branches common hepatic artery and splenic artery

Grooves of the head of the pancreas
2nd and 3rd part of the duodenum

Arterial supply
Head: pancreaticoduodenal artery
Rest: splenic artery

Venous drainage
Head: superior mesenteric vein
Body and tail: splenic vein

Ampulla of Vater
Merge of pancreatic duct and common bile duct
Is an important landmark, halfway along the second part of the duodenum, that marks the anatomical transition from foregut to midgut (also the site of transition between regions supplied by coeliac trunk and SMA).

888
Q

A 12 year old boy undergoes surgery for recurrent mastoid infections. Post operatively he complains of an altered taste sensation. Which of the following nerves has been injured?

Glossopharyngeal

Greater petrosal

Olfactory

Trigeminal

Chorda tympani

A

The chorda tympani branch of the facial nerve passes forwards through its canaliculus into the middle ear, and crosses the medial aspect of the tympanic membrane. It then passes antero-inferiorly in the infratemporal fossa. It distributes taste fibres to the anterior two thirds of the tongue.
The chorda tympani branch of the facial nerve passes forwards through its canaliculus into the middle ear, and crosses the medial aspect of the tympanic membrane. It then passes antero-inferiorly in the infratemporal fossa. It distributes taste fibres to the anterior two thirds of the tongue.

889
Q

A 48 year old man notices that he is becoming increasingly dizzy when he plays squash, in addition he has also developed cramping pain in his left arm. One day he is inflating his car tyre with a hand held pump, he collapses and is brought to hospital. What is the most likely underlying cause?

Aortic coarctation

Cervical rib

Patent ductus arteriosus

Takayasu’s arteritis

Subclavian steal syndrome

A

Subclavian steal syndrome is associated with a stenosis or occlusion of the subclavian artery, proximal to the origin of the vertebral artery. As a result, the increased metabolic needs of the arm then cause retrograde flow and symptoms of CNS vascular insufficiency.

Vascular disease

Aortic dissection
Chest pain (anterior chest pain- ascending aorta, back pain - descending aorta)
Widening of aorta on chest x-ray
Diagnosis made by CT scanning
Treatment is either medical (Type B disease) or surgical (Type A disease)
Cervical rib
Supernumery fibrous band arising from seventh cervical vertebra
Incidence of 1 in 500
May cause thoracic outlet syndrome
Treatment involves surgical division of rib
Subclavian steal syndrome
Due to proximal stenotic lesion of the subclavian artery
Results in retrograte flow through vertebral or internal thoracic arteries
The result is that decrease in cerebral blood flow may occur and produce syncopal symptoms
A duplex scan and/ or angiogram will delineate the lesion and allow treatment to be planned
Takayasu’s arteritis
Large vessel granulomatous vasculitis
Results in intimal narrowing
Most commonly affects young asian females
Patients present with features of mild systemic illness, followed by pulseless phase with symptoms of vascular insufficiency
Treatment is with systemic steroids
Patent ductus arteriosus
Ductus arteriosus is a normal foetal vessel that closes spontaneously after birth
Results in high pressure, oxygenated blood entering the pulmonary circuit
Untreated patients develop symptoms of congestive cardiac failure
Coarctation of the aorta
Aortic stenosis at the site of the ductus arteriosus insertion
More prevalent in boys or females with Turners syndrome
Patients may present with symptoms of arterial insufficiency, such as syncope and claudication
Blood pressure mismatch may be seen, as may mismatch of pulse pressure in the upper and lower limbs
Treatment is either with angioplasty or surgical resection (the former is the most common)

890
Q

Which of the following is seen more commonly with Crohns disease rather than ulcerative colitis?

Mucosal islands at endoscopy

Goblet cell depletion on biopsy

Fat wrapping of the terminal ileum

Attenuated symptoms in smokers

Toxic megacolon

A

Crohns disease is worse in smokers and smoking is an independent risk factor for disease recurrence following resection.
Fat wrapping of the terminal ileum is commonly seen in patients with ileal disease (the commonest disease site). The mesenteric fat in patients with IBD is often dense, hard and prone to considerable haemorrhage during surgery. At endoscopy, the mucosa in patients with Crohns disease is said to resemble cobblestones, mucosal islands (pseudopolyps) are seen in ulcerative colitis.

Crohns disease

Crohns disease is a chronic transmural inflammation of a segment(s) of the gastrointestinal tract and may be associated with extra intestinal manifestations. Frequent disease patterns observed include ileal, ileocolic and colonic disease. Peri-anal disease may occur in association with any of these. The disease is often discontinuous in its distribution. Inflammation may cause ulceration, fissures, fistulas and fibrosis with stricturing. Histology reveals a chronic inflammatory infiltrate that is usually patchy and transmural.

Ulcerative colitis Vs Crohns
Crohn’s disease Ulcerative colitis
Distribution Mouth to anus Rectum and colon
Macroscopic changes Cobblestone appearance, apthoid ulceration Contact bleeding
Depth of disease Transmural inflammation Superficial inflammation
Distribution pattern Patchy Continuous
Histological features Granulomas (non caseating epithelioid cell aggregates with Langhans’ giant cells) Crypt abscesses, Inflammatory cells in the lamina propria

Extraintestinal manifestations of Crohns
Related to disease extent Unrelated to disease extent
Aphthous ulcers (10%) Sacroiliiitis (10-15%)
Erythema nodosum (5-10%) Ankylosing spondylitis (1-2%)
Pyoderma gangrenosum (0.5%) Primary sclerosing cholangitis (Rare)
Acute arthropathy (6-12%) Gallstones (up to 30%)
Ocular complications (up to 10%) Renal calculi (up to 10%)

Diarrhoea in Crohns
Diarrhoea in Crohns may be multifactorial since actual inflammation of the colon is not common. Causes therefore include the following:
Bile salt diarrhoea secondary to terminal ileal disease
Entero-colic fistula
Short bowel due to multiple resections
Bacterial overgrowth

Surgical interventions in Crohns disease
The commonest disease pattern in Crohns is stricturing terminal ileal disease and this often culminates in an ileocaecal resection. Other procedures performed include segmental small bowel resections and stricturoplasty. Colonic involvement in patients with Crohns is not common and, where found, distribution is often segmental. However, despite this distribution segmental resections of the colon in patients with Crohns disease are generally not advocated because the recurrence rate in the remaining colon is extremely high. As a result, the standard options of colonic surgery in Crohns patients are generally; sub total colectomy, panproctocolectomy and staged sub total colectomy and proctectomy. Restorative procedures such as ileoanal pouch have no role in therapy.
Crohns disease is notorious for the developmental of intestinal fistulae; these may form between the rectum and skin (peri anal) or the small bowel and skin. Fistulation between loops of bowel may also occur and result in bacterial overgrowth and malabsorption. Management of enterocutaneous fistulae involves controlling sepsis, optimising nutrition, imaging the disease and planning definitive surgical management.

891
Q

A 45 year old lady has recently undergone a thyroidectomy for treatment of medullary thyroid cancer. Which of the following tumour markers is used clinically to screen for recurrence?

Free T3

Thyroglobulin

Calcitonin

Free T4

Thyroid stimulating hormone

A

Calcitonin is clinically utilised to screen for medullary thyroid cancer recurrence. Thyroid function testing does not form part of either diagnosis or follow up from a malignancy perspective. However, routine assessment of TSH may be needed in patients on thyroxine.

Thyroid malignancy

Papillary carcinoma
Commonest sub-type
Accurately diagnosed on fine needle aspiration cytology
Histologically, they may demonstrate psammoma bodies (areas of calcification) and so called ‘orphan Annie’ nuclei
They typically metastasise via the lymphatics and thus laterally located apparently ectopic thyroid tissue is usually a metastasis from a well differentiated papillary carcinoma

Follicular carcinoma
Are less common than papillary lesions
Like papillary tumours, they may present as a discrete nodule. Although they appear to be well encapsulated macroscopically there is invasion on microscopic evaluation
Lymph node metastases are uncommon and these tumours tend to spread haematogenously. This translates into a higher mortality rate
Follicular lesions cannot be accurately diagnosed on fine needle aspiration cytology and thus all follicular FNA’s (THY 3f) will require at least a hemi thyroidectomy

Anaplastic carcinoma
Less common and tend to occur in elderly females
Disease is usually advanced at presentation and often only palliative decompression and radiotherapy can be offered.

Medullary carcinoma
These are tumours of the parafollicular cells ( C Cells) and are of neural crest origin.
The serum calcitonin may be elevated which is of use when monitoring for recurrence.
They may be familial and occur as part of the MEN -2A disease spectrum.
Spread may be either lymphatic or haematogenous and as these tumours are not derived primarily from thyroid cells they are not responsive to radioiodine.

Lymphoma
These respond well to combined chemoradiotherapy
Radical surgery is unnecessary once the disease has been diagnosed on biopsy material. Such biopsy material is not generated by an FNA and thus a core biopsy has to be obtained (with care!).

892
Q

Which of the following muscles is not innervated by the ansa cervicalis?

Sternohyoid

Mylohyoid

Omohyoid

Sternothyroid

None of the above

A

Ansa cervicalis muscles:

GHost THought SOmeone Stupid Shot Irene

GenioHyoid
ThyroidHyoid
Superior Omohyoid
SternoThyroid
SternoHyoid
Inferior Omohyoid

Mylohyoid is innervated by the mylohyoid branch of the inferior alveolar nerve.

893
Q

A 2 year old boy is identified as having a communicating hydrocele and a decision is made to address this surgically. At the level of the testicle, between which of the following layers will the surgeon find accumulated fluid?

Cremasteric and internal spermatic fascia

Dartos and external spermatic fascia

Ductus deferens and internal spermatic fascia

Internal and external spermatic fascia

Visceral and parietal layers of the tunica vaginalis

A

At its distal end, the patent processes vaginalis will envelop the testis. Both visceral and parietal layers will be present. Fluid of a hydrocele does not collect in the other spaces.

Scrotal and testicular anatomy

Spermatic cord
Formed by the vas deferens and is covered by the following structures:
Layer Origin
Internal spermatic fascia Transversalis fascia
Cremasteric fascia From the fascial coverings of internal oblique
External spermatic fascia External oblique aponeurosis

Contents of the cord
Vas deferens Transmits sperm and accessory gland secretions
Testicular artery Branch of abdominal aorta supplies testis and epididymis
Artery of vas deferens Arises from inferior vesical artery
Cremasteric artery Arises from inferior epigastric artery
Pampiniform plexus Venous plexus, drains into right or left testicular vein
Sympathetic nerve fibres Lie on arteries, the parasympathetic fibres lie on the vas
Genital branch of the genitofemoral nerve Supplies cremaster
Lymphatic vessels Drain to lumbar and para-aortic nodes

Scrotum
Composed of skin and closely attached dartos fascia.
Arterial supply from the anterior and posterior scrotal arteries
Lymphatic drainage to the inguinal lymph nodes
Parietal layer of the tunica vaginalis is the innermost layer

Testes
The testes are surrounded by the tunica vaginalis (closed peritoneal sac). The parietal layer of the tunica vaginalis adjacent to the internal spermatic fascia.
The testicular arteries arise from the aorta immediately inferiorly to the renal arteries.
The pampiniform plexus drains into the testicular veins, the left drains into the left renal vein and the right into the inferior vena cava.
Lymphatic drainage is to the para-aortic nodes.

894
Q

Which of the following statements relating to the root of the neck is false?

The lung projects into the neck beyond the first rib and is constrained by Sibson’s fascia

The subclavian artery arches over the first rib anterior to scalenus anterior

The trunks of the brachial plexus lie posterior to the subclavian artery on the first rib

The roots and trunks of the Brachial plexus lie between scalenus anterior and scalenus medius muscles

The thyrocervical trunk is a branch of the subclavian artery

A

The subclavian artery lies posterior to scalenus anterior, the vein lies in front. Sibson’s fascia is another name for the suprapleural membrane.

Root of the neck

Thoracic Outlet

Where the subclavian artery and vein and the brachial plexus exit the thorax and enter the arm.
They pass over the 1st rib and under the clavicle.
The subclavian vein is the most anterior structure and is immediately anterior to scalenus anterior and its attachment to the first rib.
Then subclavian artery passes between the anterior and middle scalene muscles.
At the level of the first rib, the lower cervical nerve roots combine to form the 3 trunks of the brachial plexus. The lowest trunk is formed by the union of C8 and T1, and this trunk lies directly posterior to the artery and is in contact with the superior surface of the first rib.

Thoracic outlet obstruction causes neurovascular compromise.

895
Q

Which of the following structures separates the intervertebral discs from the spinal cord?

Anterior longitudinal ligament

Posterior longitudinal ligament

Supraspinous ligament

Interspinous ligament

Ligamentum flavum

A

The posterior longitudinal ligament overlies the posterior aspect of the vertebral bodies. It also overlies the posterior aspect of the intervertebral discs.

Intervertebral discs

Consist of an outer annulus fibrosus and an inner nucleus pulposus.
The anulus fibrosus consists of several layers of fibrocartilage.
The nucleus pulposus contains loose fibres suspended in a mucoprotein gel with the consistency of jelly. The nucleus of the disc acts as a shock absorber.
Pressure on the disc causes posterior protrusion of the nucleus pulposus. Most commonly in the lumbrosacral and lower cervical areas.
There is one disc between each pair of vertebrae, except for C1/2 and the sacrococcygeal vertebrae.

896
Q

A patient sustains damage to the median nerve during a carpal tunnel release. Which of the following muscles will be affected?

Abductor digiti minimi

Abductor pollicis brevis

Adductor pollicis

Palmaris brevis

Flexor digiti minimi brevis

A

Of the muscles listed, only the abductor pollicis brevis is innervated by the median. In questions like this one, ensure you don’t become confused between adductor and abductor.

Abductor pollicis brevis

Origin Fleshy fibres from the flexor retinaculum, scaphoid and trapezium
Insertion Via a short tendon into the radial side of the proximal phalanx of the thumb
Nerve Recurrent branch of median nerve in the palm
Actions Abducts the thumb at the carpometacarpal and metacarpophalangeal joints, causing it to travel anteriorly at right angles to the plane of the palm and to rotate medially (e.g. typing). When the thumb is fully abducted there is angulation of around 30 degrees between the proximal phalanx and the metacarpal. Because of the direction of the muscle, abduction involves medial rotation of the metacarpal, and the abductor is used along with opponens pollicis in the initial stages of thumb opposition

897
Q

A 78 year old man presents with unilateral deafness which has been present for the past 3 months. On examination, Webers test localises to the contralateral side and a CT scan of his head shows a thickened calvarium with areas of sclerosis and radiolucency. His blood tests show an elevated alkaline phosphatase, normal serum calcium and normal PTH levels. Which of the following is the most likely underlying diagnosis?

Multiple myeloma with skull involvement

Osteoporosis

Pagets disease with skull involvement

Lung cancer with skull metastasis

Osteopetrosis with skull involvement

A

Of the conditions listed, Pagets disease is the most likely diagnosis (skull vault expansion and sensorineural hearing loss). Multiple myeloma would typically result in multiple areas of radiolucency and usually raised calcium in this setting. Osteopetrosis is a recognised cause of the features described. However, it is a rare inherited disorder and usually presents in children in young adults. Presentation at this stage with no prior symptoms would be extremely rare and therefore this is not the most likely diagnosis.

Pagets disease

Paget’s disease is a disease of increased but uncontrolled bone turnover and is characterised by architecturally abnormal bones. It is thought to be primarily a disorder of osteoclasts, with excessive osteoclastic resorption followed by increased osteoblastic activity causing areas of sclerosis and deformity. Paget’s disease is common (UK prevalence 5%) but symptomatic in only 1 in 20 patients

Predisposing factors
increasing age
male sex
northern latitude
family history

Clinical features
bone pain (e.g. pelvis, lumbar spine, femur)
classical, untreated features: bowing of tibia, bossing of skull
raised alkaline phosphatase (ALP) - calcium* and phosphate are typically normal
skull x-ray: thickened vault, osteoporosis circumscripta

Indications for treatment include bone pain, skull or long bone deformity, fracture, periarticular Paget’s
bisphosphonate (either oral risedronate or IV zoledronate)
calcitonin is less commonly used now

Complications
deafness (cranial nerve entrapment)
bone sarcoma (1% if affected for > 10 years)
fractures
skull thickening
high-output cardiac failure

*usually normal in this condition but hypercalcaemia may occur with prolonged immobilisation

898
Q

A 52 year old obese lady reports a painless grape sized mass in her groin area. She has no medical conditions apart from some varicose veins. There is a cough impulse and the mass disappears on lying down. What is the most likely cause?

Femoral hernia

Inguinal hernia

Saphena varix

False aneurysm of the femoral artery

Arteriovenous malformation

A

The history of varicose veins should indicate a more likely diagnosis of a varix. The varix can enlarge during coughing/sneezing. A blue discolouration may be noted.

Groin masses clinical

Groin masses are common and include:
Herniae
Lipomas
Lymph nodes
Undescended testis
Femoral aneurysm
Saphena varix (more a swelling than a mass!)

In the history, features relating to systemic illness and tempo of onset will often give a clue as to the most likely underlying diagnosis.

Groin lumps- some key questions
Is there a cough impulse
Is it pulsatile AND is it expansile (to distinguish between false and true aneurysm)
Are both testes intra scrotal
Any lesions in the legs such as malignancy or infections (?lymph nodes)
Examine the ano rectum as anal cancer may metastasise to the groin
Is the lump soft, small and very superficial (?lipoma)

Scrotal lumps - some key questions
Is the lump entirely intra scrotal
Does it transilluminate (?hydrocele)
Is there a cough impulse (?hernia)

In most cases a diagnosis can be made clinically. Where it is not clear an ultrasound scan is often the most convenient next investigation.

899
Q

A lung resection is least likely to be utilized in the management of which of the following?

Small cell lung cancer

Adenocarcinoma of the lung

Squamous cell carcinoma of the lung

A solitary lung metastasis from colorectal cancer

A solitary lung metastasis from a low grade sarcoma

A

Note, the question asked for least likely. That being the case, the correct answer is small cell lung cancer as they are nearly always disseminated at diagnosis.

Lung cancer

Lung cancers may be classified according to histological subtypes. The main distinction is between small cell and non small cell lung cancer. Non small cell lung cancer is the most common variant and accounts for 80% of all lung cancers.

Non small cell lung cancer
These share common features of prognosis and management. They comprise the following tumours:
Squamous cell carcinoma (25% cases)
Adenocarcinoma (40% cases)
Large cell carcinoma (10% cases)
Paraneoplastic features and early disease dissemination are less likely than with small cell lung carcinoma. Adenocarcinoma is the most common lung cancer type encountered in never smokers.

Small cell lung carcinoma
Small cell lung carcinomas are comprised of cells with a neuro endocrine differentiation. The neuroendocrine hormones may be released from these cells with a wide range of paraneoplastic associations. These tumours are strongly associated with smoking and will typically arise in the larger airways. They disseminate early in the course of the disease and although they are usually chemosensitive this seldom results in long lasting remissions.

900
Q

Which of the following structures does not pass posteriorly to the medial malleolus?

Posterior tibial artery

Tibial nerve

Tibialis anterior tendon

Tendon of flexor digitorum longus

Tendon of flexor hallucis longus

A

Mnemonic for structures posterior to the medial malleolus:

Tom Dick And Nervous Harry

T ibialis posterior tendon
flexor Digitorum longus
A rtery
N erve
H allucis longus

Medial malleolus

The following structures pass posterior to medial malleolus (in order):

Tibialis posterior tendon
Flexor digitorum longus tendon
Posterior tibial artery
Tibial nerve
Tendon of flexor hallucis longus

901
Q

Which of the following muscles does not recieve any innervation from the sciatic nerve?

Semimembranosus

Quadriceps femoris

Biceps femoris

Semitendinosus

Adductor magnus

A

The sciatic nerve is traditionally viewed as being a nerve of the posterior compartment. It is known to contribute to the innervation of adductor magnus (although the main innervation to this muscle is from the obturator nerve). The quadriceps femoris is nearly always innervated by the femoral nerve.

Sciatic nerve

The sciatic nerve is formed from the sacral plexus and is the largest nerve in the body. It is the continuation of the main part of the plexus arising from ventral rami of L4 to S3. These rami converge at the inferior border of piriformis to form the nerve itself. It passes through the inferior part of the greater sciatic foramen and emerges beneath piriformis. Medially, lie the inferior gluteal nerve and vessels and the pudendal nerve and vessels. It runs inferolaterally under the cover of gluteus maximus midway between the greater trochanter and ischial tuberosity. It receives its blood supply from the inferior gluteal artery. The nerve provides cutaneous sensation to the skin of the foot and the leg. It also innervates the posterior thigh muscles and the lower leg and foot muscles. The nerve splits into the tibial and common peroneal nerves approximately half way down the posterior thigh. The tibial nerve supplies the flexor muscles and the common peroneal nerve supplies the extensor muscles and the evertor muscles of the foot.

Summary points
Origin Spinal nerves L4 - S3
Articular Branches Hip joint
Muscular branches in upper leg
Semitendinosus
Semimembranosus
Biceps femoris
Part of adductor magnus
Cutaneous sensation
Posterior aspect of thigh (via cutaneous nerves)
Gluteal region
Entire lower leg (except the medial aspect)
Terminates At the upper part of the popliteal fossa by dividing into the tibial and peroneal nerves

The nerve to the short head of the biceps femoris comes from the common peroneal part of the sciatic and the other muscular branches arise from the tibial portion.
The tibial nerve goes on to innervate all muscles of the foot except the extensor digitorum brevis (which is innervated by the common peroneal nerve).

902
Q

A 34 year old man is diagnosed with an aggressive caecal adenocarcinoma. His sister died from the same disease at 38 years of age. His mother died from endometrial cancer at the age of 41. What is the most likely underlying abnormality?

Familial adenopolypomatosis coli

Gardners syndrome

Mutation of mismatch repair genes

Deletion of chromosome 6

MYH gene mutation

A

Mutation of mismatch repair genes or Lynch syndrome which is characterised by aggressive right sided colonic malignancy and endometrial cancer is caused by microsatellite instability of DNA repair genes.

Genetics of colorectal cancer

The lifetime risk of colorectal cancer in the UK population is 5%. Up to 5% of newly diagnosed bowel cancers will be in those individuals who have a high genetically acquired risk of bowel cancer. Cancers arising in the low-moderate genetic risk group comprise approximately 30% of newly diagnosed bowel cancer.

Genetics of inherited colorectal cancer syndromes
Syndrome Features Genes implicated
FAP More than 100 adenomatous polyps affecting the colon and rectum. Duodenal and fundic glandular polyps APC (over 90%)
Gardner syndrome As FAP but with desmoid tumours and mandibular osteomas APC
Turcots syndrome Polyposis and colonic tumours and CNS tumours APC +MLH1 and PMS2
HNPCC Colorectal cancer without extensive polyposis. Endometrial cancer, renal and CNS MSH2, MLH1, PMS2 and GTBP
Peutz-Jeghers syndrome Hamartomatous polyps in GI tract and increased risk of GI malignancy LKB1 andSTK11 (in up to 70%)
Cowden disease Multiple hamartomas (see below) PTEN (85%)
MYH associated polyposis Autosomal recessive, multiple adenomatous polyps in GI tract, those in colon having somatic KRAS mutations MYH

FAP
Autosomal dominant condition, affects 1 in 12,000. Accounts for 0.5% of all CRCs. Lifetime incidence of colorectal cancer in untreated FAP =100%. Up to 25% cases are caused by de-novo germ line mutations and show no prior family history. The APC tumour suppressor gene is affected in most cases.

APC in non inherited colorectal cancer
Up to 80% of sporadic colorectal cancers will have somatic mutations that inactivate APC[1]. Both alleles are usually affected. Although the APC protein more than likely has multiple critical cellular functions, the best-established role for APC in the cancer process is as a major binding partner and regulator of the β- catenin protein in the so-called canonical or β- catenin dependent Wnt signaling pathway.

HNPCC (Lynch syndrome)
HNPCC cancers differ from conventional tumours in a number of respects. In the colon the tumours are more likely to be right sided, histologically they are more likely to be mucinous and have dense lymphocytic infiltrates. To be diagnosed as having HNPCC individuals must show typically HNPCC tumours in at least three individuals, (one of whom must be a first degree relative to the other two). In at least two successive generations. At least one cancer must be diagnosed under the age of 50. FAP must be excluded and tumours should be verified by pathological identification (Amsterdam criteria). The genetic changes in HNPCC stem primarily from microsatellite instability affecting DNA mismatch repair genes. In HNPCC the mismatch repair genes most commonly implicated include; MSH2 and MLH1 and these occur in up to 70% of people with HNPCC. The finding of microsatellite instability is unusual in sporadic colorectal cancers. Approximately 60% of individuals who fulfill the Amsterdam criteria will not be found to have evidence of mismatch repair gene defects on genetic testing. The risk of developing colorectal cancer in those who have not demonstrated mutation of the mis match repair genes is increased if they fulfill the Amsterdam criteria, but not
the extent that it is increased in those who fulfill the criteria AND have evidence of mis match repair gene defects.

KRAS Mutations
The RAS family of small G proteins act as molecular switches downstream of growth factor receptors. KRAS and the other two members of the family; HRAS and NRAS, are the site of mutation in approximately 40% of colorectal cancers. When adenomas are examined the proportion of adenomas less than 1cm showing KRAS mutations was only 10% which contrasts with 50% in those lesions greater than 1cm.

p53 mutations
The p53 protein functions as a key transcriptional regulator of genes that encode proteins with functions in cell-cycle checkpoints at the G1/S and G2/M boundaries, in promoting apoptosis, and in restricting angiogenesis . As such, selection for p53 defects at the adenoma-carcinoma transition may reflect the fact that stresses on tumor cells activate cell-cycle arrest, apoptotic, and antiangiogenic pathways in cells with wild-type p53 function. Many colonic tumours will demonstrate changes in the p53 gene that may facilitate tumour progression through from adenoma to carcinoma.

Cowden syndrome
Also known as multiple hamartoma syndrome. Rare autosomal dominant condition with incidence of 1 in 200,000.. It is characterised by multiple mucocutaneous lesions, trichilemmomas, oral papillomas and acral keratosis. Most often diagnosed in third decade of life. Breast carcinoma may occur in up to 50% of patients and conditions such as fibrocystic disease of the breast may occur in 75% of women. Thyroid disease occurs in 75% and may include malignancy. Endoscopic screening will identify disease in up to 85% although the small bowel is rarely involved. There is a 15-20% risk of developing colorectal cancer and regular colonoscopic screening from age 45 is recommended.

Terminology
Oncogene Oncogenes are genes which have the potential to induce cellular proliferation and avoid apoptosis. Oncogene mutations are general gain of function and are therefore dominant. Increased expression of oncogenes are found in most tumours
Tumour suppressor gene These genes generally inhibit cellular proliferation or induce apoptosis. Mutations in tumour suppressor genes are generally loss of function mutations, and are therefore recessive. Mutations in both tumour suppressor gene alleles allow cells to proliferate without restraint

References
1. Fearon, E.R. and B. Vogelstein, A genetic model for colorectal tumorigenesis. Cell, 1990. 61(5): p. 759-67.

903
Q

A patient presents with superior vena caval obstruction. How many collateral circulations exist as alternative pathways of venous return?

None

One

Two

Three

Four

A

There are 4 collateral venous systems:
Azygos venous system
Internal mammary venous pathway
Long thoracic venous system with connections to the femoral and vertebral veins (2 pathways)

Despite this, venous hypertension still occurs.

Superior vena cava

Drainage
Head and neck
Upper limbs
Thorax
Part of abdominal walls

Formation
Subclavian and internal jugular veins unite to form the right and left brachiocephalic veins
These unite to form the SVC
Azygos vein joins the SVC before it enters the right atrium

Relations
Anterior Anterior margins of the right lung and pleura
Posteromedial Trachea and right vagus nerve
Posterolateral Posterior aspects of right lung and pleura
Pulmonary hilum is posterior
Right lateral Right phrenic nerve and pleura
Left lateral Brachiocephalic artery and ascending aorta

Developmental variations
Anomalies of the connection of the SVC are recognised. In some individuals a persistent left sided SVC drains into the right atrium via an enlarged orifice of the coronary sinus. More rarely the left sided vena cava may connect directly with the superior aspect of the left atrium, usually associated with an un-roofing of the coronary sinus. The commonest lesion of the IVC is for its abdominal course to be interrupted, with drainage achieved via the azygos venous system. This may occur in patients with left sided atrial isomerism.

904
Q

During a trans femoral arch aortogram the brachiocephalic artery is entered with an angiography catheter. The radiologist continues to advance the catheter. Into which of the following vessels is it likely to enter?

Left subclavian artery

Left axillary artery

Right subclavian artery

Right axillary artery

None of the above

A

Right subclavian artery

The axillary artery is a branch of the subclavian artery and although developmental anomalies may occur they are rare. The catheter may also enter the right carotid. There is no brachiocephalic artery on the left side.

Brachiocephalic artery

The brachiocephalic artery is the largest branch of the aortic arch. From its aortic origin it ascends superiorly, it initially lies anterior to the trachea and then on its right hand side. It branches into the common carotid and right subclavian arteries at the level of the sternoclavicular joint.

Path
Origin- apex of the midline of the aortic arch
Passes superiorly and posteriorly to the right
Divides into the right subclavian and right common carotid artery

Relations
Anterior
Sternohyoid
Sternothyroid
Thymic remnants
Left brachiocephalic vein
Right inferior thyroid veins
Posterior
Trachea
Right pleura
Right lateral
Right brachiocephalic vein
Superior part of SVC
Left lateral
Thymic remnants
Origin of left common carotid
Inferior thyroid veins
Trachea (higher level)

Branches
Normally none but may have the thyroidea ima artery

905
Q

Which of the following arterial blood gas results would fit with chronic respiratory acidosis with a compensatory metabolic alkalosis?

A-pH 7.36, PaCO
2
7.3, PO
2
8.9 (FiO
2
40%), Bicarbonate 30.2, Base excess +5.3

B- pH 7.32, PCO
2
3.8, PaO
2
22.2 (FiO
2
40%), Bicarbonate 19.1, Base excess -7.9

C-pH 7.14, PaCO
2
7.4, PaO
2
8.9 (FiO
2
40%), Bicarbonate 14 mmol, Base excess -10.6

D-pH 7.57, PaCO
2
3.5, Pa O
2
24.5 (FiO
2
85%), Bicarbonate 23.5, Base excess +1.8 mmol

E-pH 7.19, pCO
2
10.2, pO
2
16 (FiO
2
85%), Bicarbonate 23.8, Base excess -2.2 mmol

A

A

Arterial blood gas interpretation

In advanced life support training, a 5 step approach to arterial blood gas interpretation is advocated.

  1. How is the patient?
  2. Is the patient hypoxaemic?
    The Pa02 on air should be 10.0-13.0 kPa
  3. Is the patient acidaemic (pH <7.35) or alkalaemic (pH >7.45)
  4. What has happened to the PaCO2?
    If there is acidaemia, an elevated PaCO2 will account for this
  5. What is the bicarbonate level or base excess?
    A metabolic acidosis will have a low bicarbonate level and a low base excess (< -2 mmol)
    A metabolic alkalosis will have a high bicarbonate and a high base excess (> +2 mmol)
906
Q

Which of the following agents is most likely to be helpful in controlling refractory hypotension in a 23 year old female with severe pyelonephritis?

Noradrenaline

Adrenaline

Dobutamine

Dopamine

Milrinone

A

A-Noradrenaline

Since the main issue here is vasodilation, a vasoconstrictor is most likely to be helpful.

Circulatory support of the critically ill

Circulatory support
Impaired tissue oxygenation may occur as a result of circulatory shock. Shock is considered further under its own topic heading.

Patients requiring circulatory support require haemodynamic monitoring. At its simplest level this may simply be in the form of regular urine output measurements and blood pressure monitoring. In addition ECG monitoring will allow the identification of cardiac arrhythmias. Pulse oximeter measurements will allow quick estimation haemoglobin oxygen saturation in arterial blood.

Invasive arterial blood pressure monitoring is undertaken by the use of an indwelling arterial line. Most arterial sites can be used although the radial artery is the commonest. It is important not to cannulate end arteries. The arterial trace can be tracked to ventilation phases and those patients whose systolic pressure varies with changes in intrathoracic pressure may benefit from further intravenous fluids.

Central venous pressure is measured using a CVP line that is usually sited in the superior vena cava via the internal jugular route. The CVP will demonstrate right atrial filling pressure and volume status. When adequate intra vascular volume is present a fluid challenge will typically cause a prolonged rise in CVP (usually greater than 6-8mmHg).

To monitor the cardiac output a Swan-Ganz catheter is traditionally inserted (other devices may be used and are less invasive). Inflation of the distal balloon will provide the pulmonary artery occlusion pressure and the pressure distal to the balloon will equate to the left atrial pressure. This gives a measure of left ventricular preload. Because the Swan-Ganz catheter can measure several variables it can be used to calculate:
Stroke volume
Systemic vascular resistance
Pulmonary artery resistance
Oxygen delivery (and consumption)

Inotropes
In patients with an adequate circulating volume but on-going circulatory compromise a vasoactive drug may be considered. These should usually be administered via the central venous route. Commonly used inotropes include:
Agent Mode of action Effect
Noradrenaline α agonist Vasopressor action, minimal effect on cardiac output
Adrenaline α and β receptor agonist Increases cardiac output and peripheral vascular resistance
Dopamine β1 agonist Increases contractility and rate
Dobutamine β1 and β2 agonist Increases cardiac output and decreases SVR
Milrinone Phosphodiesterase inhibitor Elevation of cAMP levels improves muscular contractility, short half life and acts as vasodilator

907
Q

Which of the following is not a carpal bone?

Trapezium

Triquetrum

Trapezoid

Trapezius

Lunate

A

Mnemonic for the Carpal Bones

Sally Likes To Play The Tiny Chrome Harmonica
She Looks Too Pretty Try To Catch Her
Scared Lovers Try Positions That They Can’t Handle
Trapezius is a muscle of the back.

Carpal bones

The wrist is comprised of 8 carpal bones, these are arranged in two rows of 4. It is convex from side to side posteriorly and concave anteriorly.

908
Q

With which of the conditions listed below is a Hill- Sachs lesions classically associated?

Fracture of the surgical neck of the humerus

Glenohumeral dislocation

Supraspinatus tear

Acromioclavicular dislocation

Sternoclavicular dislocation

A

A Hill-Sachs lesion occurs when the cartilage surface of the humerus is in contact with the rim of the glenoid. About 50% of anterior glenohumeral dislocations are associated with this lesion.

Shoulder disorders

Shoulder fractures and dislocations
Fractures
Proximal humerus
Background
Third most common fragility fracture in the elderly.
Results from low energy fall in predominantly elderly females, or from high energy trauma in young males.
Can be associated with nerve injury (commonly axillary), and fracture-dislocation of the humeral head. Detailed neurological assessment is essential for all upper limb injuries.

Anatomy
Osteology
Consists of articular head, greater tuberosity, lesser tuberosity, metaphysis and diaphysis. Between the articular head and the tuberosities is the anatomical neck (previous physis). Between the tuberosities and the metaphysis is the surgical neck.
The supraspinatus, infraspinatus and teres minor muscles attach to the greater tuberosity. The subscapularis muscle attaches to the lesser tuberosity.

Vascular Supply
Humeral head is supplied by the anterior and posterior humeral circumflex arteries. Anatomical neck fractures are at greatest risk of osteonecrosis.

Imaging
Imaging aims to both delineate the fracture pattern, and confirm/exlude the presence of an associated dislocation.
Radiographs - True anteroposterior (AP), axillary lateral and/or scapula Y view.
CT - indicated to better define intra-articular involvement and to aid pre-operative planning. MRI is not useful for fracture imaging.

Classification
Description of the fracture is often more useful than classification. Particular attention should be paid to humeral alignment, fracture displacement, and greater tuberosity position (rotator cuff will pull the GT supero-posterioly, which can cause impingement problems with malunion).
- Neer Classification: Most commonly used. Describes fracture as 2,3,or 4 part depending upon the number main fragments. Also comments on the degree of displacement. Fragments:
-greater tuberosity
-lesser tuberosity
- articular surface
- shaft
Displacement: >1cm or angulation >45 degrees.

Treatment
The vast majority of proximal humeral fractures are minimally displaced, and therefore can be managed conservatively. This involves immobilisation in a polysling, and progressive mobilisation. Pendular exercise can commence at 14 days, and active abduction from 4-6 weeks.

Irreducible fracture dislocation is an indication for operative management. Other indications include large displacement, younger patient, head splitting (intra-articular fractures). However, the recent PROFHER trial (1) has suggested no benefit to operative intervention on patient outcome (it must be applied cautiously as majority of patients were elderly with extraarticular fractures). Options available for surgical management include:

ORIF Most commonly used. Plate and screw fixation. Can reconstruct complex fractures.
Intramedullary nail Suitable for extra-articular configuration, predominantly surgical neck +/- GT fractures.
Hemiarthroplasty Used for un-reconstructable fractures in the older patient who has good glenoid quality.
Total shoulder arthroplasty Unconstructable fractures where high functioning shoulder is required (hemiarthroplasty will cause glenoid erosion)
Reverse shoulder arthroplasty Total shoulder arthroplasty that provides better functional outcome than conventional total shoulder replacement.

Scapula
Background
Uncommon fractures usually associated with high energy trauma. Most commonly involve scapula body or spine (50%), glenoid fossa and glenoid neck. Important to exclude associated life threatening injury.

Imaging
Plain radiographs should include true anteroposterior (AP), axillary lateral and/or scapula Y view. CT scanning is useful for defining intra-articular involvement, displacement and for three dimensional reconstruction.

Classification
Based on the location of the fracture (coracoid, acromion, glenoid neck, glenoid fossa, scapula body). Beware of ipsilateral glenoid neck and clavicle fracture -floating shoulder - where limb is effectively dissociated from axial skeleton.

Treatment
The vast majority of scapula fractures are amenable to conservative management, consisting of sling immobilisation for two weeks followed by early rehabilitation. Floating shoulder will usually require fixation, and consideration of surgery should also be given to intra-articular and displaced/angulated glenoid fractures.

Dislocations

Types
Dislocations around the shoulder joint include glenohumeral dislocation, acromioclavicular joint disruption and sternoclavicular dislocation. Only glenohumeral dislocation will be covered here.

Glenohumeral dislocation
Diagnosis, classification and management are covered here.

Background
Shoulder dislocation is commonly seen in A&E. It has a high recurrence rate that is as high as 80% in teenagers. Initial management requires emergent reduction to prevent lasting chondral damage.

Early assessment and management
Usually a traumatic cause (multi-directional instability in frequent dislocations requires discussion with orthopaedics and is not covered here). Careful history, examination and documentation of neurovascular status of the limb, in particular the axillary nerve (regimental badge sensation). This should be re-assessed post manipulation. Early radiographs to confirm direction of dislocation.

Initial management consists of emergent closed reduction under under entanox and analgesia, but often requires conscious sedation. Arm should then be immobilised in a polysling, and XR to confirm relocation.

Imaging - True anteroposterior (AP), axillary lateral and/or scapula Y view. Reduced humeral head should lie between acromion and coracoid on lateral/scapula view.

Types
Direction Features Cause Examination Reduction techniques
Anterior Most Common >90% Usually traumatic - anterior force on arm when shoulder is abducted, externally rotated Loss of shoulder contour - sulcus sign. Humeral head can be felt anteriorly.
Hippocratic.
Milch.
Stimson.

Kocher not advised due to complication of fracture
Posterior 50% missed in A&E 50% traumatic, but classically post seizure or electrocution Shoulder locked in internal rotation. XR may show lightbulb appearance. Gentle lateral traction to adducted arm.
Inferior Rare Associated with pectorals and rotator cuff tears, and glenoid fracture As for primary injury Management of primary injury
Superior Rare Associated with acrominon/clavicle fracture As for primary injury Management of primary injury

Associated injuries
Bankart lesion - avulsion of the anterior glenoid labrum with an anterior shoulder dislocation (reverse Bankart if poster labrum in posterior dislocation).
Hill Sachs defect - chondral impaction on posteriosuperior humeral head from contact with gleonoid rim. Can be large enough to lock shoulder, requiring open reduction. (Reverse Hill Sachs in posterior dislocation).
Rotator cuff tear - increases with age.
Greater or lesser tuberosity fracture - increases with age.
Humeral neck fracture - shoulder fracture dislocation. More common in high energy trauma and elderly. Should be discussed with orthopaedics prior to any attempted reduction.

Rotator Cuff Disease

Rotator cuff disease is a spectrum of conditions that ranges from subacromial impingement to rotator cuff tears and eventually to rotator cuff arthropathy (arthritis).

Anatomy
The rotator cuff is a group of four muscles that are important in shoulder movements, and maintenance of glenohumeral stability.
Muscle Scapular attachment Humeral attachment Action Innervation
Supraspinatus Supraspinatus fossa Superior facet of greater tuberosity Initiation of abduction of humerus Suprascapular nerve
Infraspinatus Infraspinatus fossa Posterior facet of greater tuberosity External rotation of humerus Suprascapular nerve
Teres Minor Lateral border Inferior facet of greater tuberosity External rotation of humerus Axillary Nerve
Subscapularis Subscapular fossa Lesser tuberosity Internal rotation of humerus Upper and lower subscapular nerve

The inferior rotator cuff muscles (infraspinatus, teres minor, and subscapularis) balance the superior pull of the deltoid. Injury/tear results in upward migration of the humeral head on the glenoid (can be seen on AP radiograph).
Likewise, the anterior muscles (subscapularis) are balanced with the posterior muscles (infraspinatus, teres minor).

Subacromial Impingement

The most common cause of shoulder pain, which results from impingement of the superior cuff on the undersurface of the acromion, and an inflammatory bursitis.
Associated with certain types of acromial morphology (Bigliani classification).
Presents as insidious pain which is exacerbated by overhead activities.

Rotator Cuff Tear

Often presents as an acute event on the background of chronic subacromial impingement in the older patient, but can present as an avulsion injury in younger patients.
Majority of tears are to the superior cuff (supraspinatus, infraspinatus, teres minor), though a tear to subscapularis is associated with subcoracoid impingement.
Tears present as pain and weakness when using the muscles in question.

Rotator Cuff Arthropathy

Defined as shoulder arthritis in the setting of rotator cuff dysfunction. Results from superior migration due to the loss of rotator cuff function and integrity. Unopposed deltoid pulls the humeral head superiorly.
Associated with massive chronic cuff tears.

Imaging

Plain radiographs
AP of the shoulder may show superior migration of the humerus with a cuff tear, and features of arthritis with arthropathy. Other causes of pain may also be identified (e.g. calcific tendonitis/fracture)
Outlet view is useful for defining the acromial morphology

USS
Allows dynamic imaging of the cuff, and is inexpensive. However, it is very user dependent.

MRI
Best imaging modality for cuff pathology.
Also allows imaging of the rest of the shoulder. When intra-articular pathology is suspected, can be combined with an arthrogram for improved sensitivity and specificity.

Treatment

Subacromial impingement
Physiotherapy, oral anti-inflammatory medication
Subacromial steroid injection can settle inflammation
Arthroscopic subacromial decompression by shaving away the undersurface of the acromion, more space is created for the rotator cuff. Cuff integrity is assessed also at time of surgery, and can be repaired if necessary.

Rotator cuff tear
When considering repair of a cuff tear, the age and activity of the patient, the nature of the tear (degenerative vs. acute traumatic), and the size and retraction of the tear should be considered when making a surgical plan.
Mild tears or tears in the elderly can be managed conservatively, as outlined above.
Moderate tears can be repaired arthroscopically. Massive or retracted tears will often require an open repair (occasionally with a tendon transfer). Subacromial decompression is performed at the same time to reduce impingement, symptoms and recurrence.

Calcific tendonitis
Calcific tendonitis involves calcific deposits within tendons anywhere in the body, but most commonly in the rotator cuff (specifically the supraspinatus tendon). When present in the shoulder, it is associated with subacromial impingement and pain.

Pathology
More common in women aged 30-60 years.
Association with diabetes and hypothyroidism

There are three stages of calcification
Formative phase characterized by calcific deposits
Resting phase deposit is stable, but presents with impingement problems
Resorptive phase phagocytic resorption. Most painful stage.

Presentation
Similar in presentation to subacromial impingement, with pain especially with over head activities. Atraumatic in nature.

Imaging
Plain radiographs show calcification of the rotator cuff, usually within 1.5cm of its insertion on the humerus. Supraspinatus outlet views can show level of impingment. Further imaging is rarely needed.

Treatment
Non-operative NSAIDS, steroid injection (controversial, but practiced) and physiotherapy. Approximately 75% will resolve by 6 months with conservative management.
Ultrasound guided or surgical needle barbotage can break down deposits and resolve symptoms. Occasionally surgical excision is required.

Adhesive capsulitis (Frozen Shoulder)
Pain and loss of movement of shoulder joint, which involves fibroplastic proliferation of capsular tissue, causing soft tissue scarring and contracture. Patients present with a painful and decreased arc of motion.
Associated with prolonged immobilization, previous surgery, thyroid disorders (AI) and diabetes
Classically three stages which can take up to two years to resolve:
Stage one the freezing and painful stage
Stage two the frozen and stiff stage
Stage three the thawing stage, where shoulder movement slowly improves

Imaging
Plain radiographs to exclude other causes of a painful shoulder
MRI arthrogram may show capsular contracture, and again may be used to exclude cuff pathology. However, often not performed as diagnosis is largely clinical.

Treatment
Non-operative NSAIDS, steroid injection and physiotherapy. Patience is required as condition can take up to 2 years to improve.
Operative MUA or arthroscopic adhesiolysis (release of adhesions) can expedite recovery, followed by intensive physiotherapy.

Glenohumeral Arthritis
Background
May be osteoarthritis (primary or secondary to cuff tear or trauma), rheumatoid arthritis, or as part of a spondyloarthropathy. Majority of those with RA will develop symptoms.
More common in the elderly
Presents like any other arthritis - pain at night and with movement

Imaging
AP and axillary radiographs will show features of arthritis.
CT/MRI is often useful to classify the shape of the glenoid and extent of bone loss when considering arthroplasty. MRI also essential to asses integrity of rotator cuff if considering shoulder replacement.

Treatment
Like all orthopaedics, start with simple measures:
NSAIDS, management of RA, physiotherapy, steroid injection.
Hemiarthroplasty can sometimes be considered if glenoid is in excellent condition or if patient has large comorbidity.
Arthroscopic debridement is useful if patient has isolated ACJ arthritis, but is rarely used for glenohumeral arthritis.
Total shoulder replacement is shown to produce superior outcome when compared to hemiarthroplasty in terms of pain relief, function and implant survival.
Total shoulder replacement can be anatomical (ball on humerus, with cup on glenoid), or reverse geometry (ball on glenoid, with cup on humerus). Anatomical TSR requires an in tact rotator cuff, so often reverse is preferable when the cuff if questionable in integrity.

References
1. JAMA. 2015;313(10):1037-1047. doi:10.1001/jama.2015.1629

909
Q

In which electrosurgical modality does a sinusoidal, non modulated waveform result in vaporization of the tissues?

Coagulation current

Cutting current

Blended current

Fulguration

Desiccation

A

Cutting current

The high energy levels result in tissue vaporisation and cleavage of tissues.

Electrosurgery

Electrosurgery utilises the heat generated by the passage of high frequency alternating electrical current through living tissues. The application of a voltage across human tissue results in the formation of an electrical circuit between the voltage source and the tissue. The tissue acts as a resistor and the level of resistance is determined by the water content of the tissue. It is this resistance that results in the formation of heat.

An alternating current constantly changes the direction in which the current flows, the speed with which this occurs is measured in Hertz. Most diathermy units operate at a frequency of between 200,000 kHZ to 5MHz. This means that tissue such as nerves and muscles will not depolarise (since this seldom occurs at frequencies above 10,000Hz). The current waveform can be adjusted to deliver three main therapeutic modalities; cutting, coagulation and blend.

Types of current
Cutting
Sinusoidal and non modulated waveform
High average power and current density
Precise cutting without thermal damage
Coagulation
Modulated current with intermittent dampened sine waves of high peak voltage
Evaporation, rather than vaporisation of intracellular fluid occurs
Results in formation of coagulum
Desication
Active electrode in direct contact with tissue
Low current and high voltage system
Results in loss of cellular water but no protein damage
Fulguration
Electrode probe is held away from tissue
Produces spray effect with local, superficial tissue destruction
Low amplitude and high voltage system
Blend
Alternating cutting and coagulation modes
Total average power is less than with cutting

910
Q

Which of the following is not a feature found on a CXR in traumatic aortic disruption?

Widened mediastinum

Trachea deviated to the left

Depression of the left main stem bronchus

Obliteration of the aortic knob

Widened paraspinal interfaces

A

The trachea is normally deviated to the right.

Thoracic aorta rupture

Mechanism of injury: Decelerating force i.e. RTA, fall from a great height
Most people die at scene
Survivors may have an incomplete laceration at the ligamentum arteriosum of the aorta.

Clinical features
Contained haematoma: persistent hypotension
Detected mainly by history, CXR changes

CXR changes
Widened mediastinum
Trachea/Oesophagus to right
Depression of left main stem bronchus
Widened paratracheal stripe/paraspinal interfaces
Space between aorta and pulmonary artery obliterated
Rib fracture/left haemothorax

Diagnosis
Angiography, usually CT aortogram.

Treatment
Repair or replacement. Ideally they should undergo endovascular repair.

911
Q

A 62 year old man presents with arm weakness. On examination, he has a weakness of elbow extension and loss of sensation on the dorsal aspect of the first digit. What is the site of the most likely underlying defect?

Axillary nerve

Median nerve

Ulnar nerve

Radial nerve

Musculocutaneous nerve

A

The loss of extension is secondary to triceps injury which reflects its innervation by the radial nerve.

Radial nerve

Continuation of posterior cord of the brachial plexus (root values C5 to T1)

Path
In the axilla: lies posterior to the axillary artery on subscapularis, latissimus dorsi and teres major.
Enters the arm between the brachial artery and the long head of triceps (medial to humerus).
Spirals around the posterior surface of the humerus in the groove for the radial nerve.
At the distal third of the lateral border of the humerus it then pierces the intermuscular septum and descends in front of the lateral epicondyle.
At the lateral epicondyle it lies deeply between brachialis and brachioradialis where it then divides into a superficial and deep terminal branch.
Deep branch crosses the supinator to become the posterior interosseous nerve.

Regions innervated
Motor (main nerve)
Triceps
Anconeus
Brachioradialis
Extensor carpi radialis
Motor (posterior interosseous branch)
Supinator
Extensor carpi ulnaris
Extensor digitorum
Extensor indicis
Extensor digiti minimi
Extensor pollicis longus and brevis
Abductor pollicis longus
Sensory The area of skin supplying the proximal phalanges on the dorsal aspect of the hand is supplied by the radial nerve (this does not apply to the little finger and part of the ring finger)

Muscular innervation and effect of denervation
Anatomical location Muscle affected Effect of paralysis
Shoulder Long head of triceps Minor effects on shoulder stability in abduction
Arm Triceps Loss of elbow extension
Forearm Supinator
Brachioradialis
Extensor carpi radialis longus and brevis Weakening of supination of prone hand and elbow flexion in mid prone positio

912
Q

Which of the following mechanisms best accounts for the release of adrenaline?

Release from the adrenal medulla in response to increased angiotensin 1 levels

Release from the zona fasiculata from the adrenal gland in response to increased sympathetic discharge

Release from the adrenal medulla in response to increased noradrenaline levels

Release from the adrenal medulla in response to sympathetic stimulation from the splanchnic nerves

None of the above

A

D-The adrenal gland releases adrenaline in response to increased sympathetic discharge from preganglionic sympathetic fibres of the splanchnic nerves. These cause the chromafin cells of the medulla to release adrenaline (which is preformed) by exocytosis.

Adrenaline

Fight or Flight response
- Catecholamine (phenylalanine and tyrosine)
- Neurotransmitter and hormone
- Released by the adrenal glands
- Effects on α 1 and 2, β 1 and 2 receptors
- Effect on β 2 receptors in skeletal muscle vessels-causing vasodilation
- Increase cardiac output and total peripheral resistance
- Vasoconstriction in the skin and kidneys causing a narrow pulse pressure

Actions
α adrenergic receptors:
Inhibits insulin secretion by the pancreas
Stimulates glycogenolysis in the liver and muscle
Stimulates glycolysis in muscle

β adrenergic receptors:
Stimulates glucagon secretion in the pancreas
Stimulates ACTH
Stimulates lipolysis by adipose tissue

913
Q

An intravenous drug user develops a false aneurysm and requires emergency surgery. The procedure is difficult and the femoral nerve is inadvertently transected. Which of the following muscles is least likely to be affected as a result?

Sartorius

Vastus medialis

Pectineus

Quadriceps femoris

Adductor magnus

A

Mnemonic for femoral nerve supply

(don’t) M I S V Q Scan for PE
M edial cutaneous nerve of the thigh
I ntermediate cutaneous nerve of the thigh
S aphenous nerve

V astus
Q uadriceps femoris
S artorius

PE ectineus
Adductor magnus is innervated by the obturator and sciatic nerve. The pectineus muscle is sometimes supplied by the obturator nerve but this is variable. Since the question states least likely, the correct answer is adductor magnus.

914
Q

A 43 year old man suffers a pelvic fracture which is complicated by an injury to the junction of the membranous urethra to the bulbar urethra. In which of the following directions is the extravasated urine most likely to pass?

Posteriorly into extra peritoneal tissues

Laterally into the buttocks

Into the abdomen

Anteriorly into the connective tissues surrounding the scrotum

None of the above

A

D-The superficial perineal pouch is a compartment bounded superficially by the superficial perineal fascia, deep by the perineal membrane (inferior fascia of the urogenital diaphragm), and laterally by the ischiopubic ramus. It contains the crura of the penis or clitoris, muscles, viscera, blood vessels, nerves, the proximal part of the spongy urethra in males, and the greater vestibular glands in females.
When urethral rupture occurs as in this case the urine will tend to pass anteriorly because the fascial condensations will prevent lateral and posterior passage of the urine.

Urogenital triangle

The urogenital triangle is formed by the:
Ischiopubic inferior rami
Ischial tuberosities
A fascial sheet is attached to the sides, forming the inferior fascia of the urogenital diaphragm.

It transmits the urethra in males and both the urethra and vagina in females. The membranous urethra lies deep to this structure and is surrounded by the external urethral sphincter.

Superficial to the urogenital diaphragm lies the superficial perineal pouch. In males this contains:
Bulb of penis
Crura of the penis
Superficial transverse perineal muscle
Posterior scrotal arteries
Posterior scrotal nerves

In females the internal pudendal artery branches to become the posterior labial arteries in the superficial perineal pouch.

915
Q

How many dorsal interossei are there?

1

3

2

4

5

A

Dorsally, the first interosseous occupies the space between the thumb and index finger. The tendon of the second inserts into the radial side of the extensor hood and proximal phalanx of the middle finger. The third and the fourth are inserted into the ulnar sides of the extensor hoods of the middle and ring fingers.

Interossei

Origin and insertion Nerve supply Actions
Four palmar and four dorsal interossei occupy the spaces between the metacarpal bones. Each palmar interossei originates from the metacarpal of the digit on which it acts.
Each dorsal interossei comes from the surface of the adjacent metacarpal on which it acts. As a result the dorsal interossei are twice the size of the palmar ones.
The interossei tendons, except the first palmar, pass to one or other side of the metacarpophalangeal joint posterior to the deep transverse metacarpal ligament. They become inserted into the base of the proximal phalanx and partly into the extensor hood They are all innervated by the ulnar nerve Dorsal interossei abduct the fingers, palmar interossei adduct the fingers

Clinical notes
Along with the lumbricals the interossei flex the metacarpophalangeal joints and extend the proximal and distal interphalangeal joints. They are responsible for fine tuning these movements. When the interossei and lumbricals are paralysed the digits are pulled into hyperextension by extensor digitorum and a claw hand is seen.

916
Q

Which of the following structures are not closely related to the adductor longus muscle?

Long saphenous vein

Tendon of iliacus

The profunda branch of the femoral artery

Pectineus muscle

Femoral nerve

A

Femoral triangle:
Adductor longus medially
Inguinal ligament superiorly
Sartorius muscle laterally
Adductor longus forms the medial border of the femoral triangle. It is closely related to the long saphenous vein which overlies it and the profunda branch of the femoral artery. The femoral nerve is related to it inferiorly. However, the tendon of iliacus inserts proximally and is not in contact with adductor longus.
Adductor longus

Origin Anterior body of pubis
Insertion Middle third of linea aspera
Action Adducts and flexes the thigh, medially rotate the hip
Innervation Anterior division of obturator nerve (L2, L3, L4)

917
Q

The right and left pulmonary arteries are derived from which of the following embryological aortic arches?

Fifth

Second

Sixth

Fourth

Third

A

Pulmonary vessels develop from the sixth arch

Embryological aortic arches

  • The aortic arches are a series of six paired embryological vascular structures which give rise to the great arteries of the neck and head.
    The first and second arches disappear early. A remnant of the 1st arch forms part of the maxillary artery. The external carotid buds from the horns of the aortic sac left behind by the regression of the first two arches.
    The third aortic arch constitutes the commencement of the internal carotid artery, and is therefore named the carotid arch. It contributes to the common carotid artery and the proximal portion of the internal carotid artery.
    The fourth right arch forms the right subclavian as far as the origin of its internal mammary branch. The fourth left arch forms the arch of the aorta between the origin of the left carotid artery and the termination of the ductus arteriosus.
    The fifth arch regresses or forms incompletely.
    The proximal part of the sixth right arch persists as the proximal part of the right pulmonary artery while the distal section degenerates; The sixth left arch gives off the left pulmonary artery and forms the ductus arteriosus.
918
Q

A 19 year old man is stabbed in the chest at a nightclub. He develops a cardiac arrest in casualty following an attempted transfer to the CT scanning room. What is the most appropriate course of action?

Immediate CT scanning with ongoing CPR

Echocardiography

Thoracotomy

Pericardiocentesis

Chest ultrasound

A

Massive hemothorax results from the rapid accumulation of more than 1500 mL of blood or one third or more of the patients blood volume in the chest cavity. It is most commonly caused by a penetrating wound that disrupts the systemic or hilar vessels, although massive hemothorax can also result from blunt trauma. In massive haemothorax the neck veins are typically collapsed and the chest wall remains mobile. These features can help distinguish it from pneumothorax where the neck veins are distended.
Penetrating thoracic trauma that is then followed by cardiac arrest in the department is an indication for ER thoracotomy.

Thoracic trauma

Key points related to thoracic trauma
Less than 10% of blunt chest trauma and 15-30% of penetrating chest trauma requires operative intervention.
The physiologic consequences of thoracic trauma are hypoxia, hypercarbia, and acidosis. Contusion, hematoma, and alveolar collapse, or changes in intrathoracic pressure relationships (e.g., tension pneumothorax and open pneumothorax) cause hypoxia and lead to metabolic acidosis. Hypercarbia causes respiratory acidosis and most often follows inadequate ventilation caused by changes in intrathoracic pressure relationships and depressed level of consciousness.

Types of thoracic trauma

Tension pneumothorax
Often laceration to lung parenchyma with flap
Pressure develops in thorax
Most common cause is mechanical ventilation in patient with pleural injury
Symptoms overlap with cardiac tamponade, hyper-resonant percussion note is more likely in tension pnemothorax
Flail chest
Chest wall disconnects from thoracic cage
Multiple rib fractures (at least two fractures per rib in at least two ribs)
Associated with pulmonary contusion
Abnormal chest motion
Avoid over hydration and fluid overload
Pneumothorax
Most common cause is lung laceration with air leakage
Most traumatic pneumothoraces should have a chest drain
Patients with traumatic pneumothorax should never be mechanically ventilated until a chest drain is inserted
Haemothorax
Most commonly due to laceration of lung, intercostal vessel or internal mammary artery
Haemothoraces large enough to appear on CXR are treated with large bore chest drain
Surgical exploration is warranted if >1500ml blood drained immediately
Cardiac tamponade
Beck’s triad: elevated venous pressure, reduced arterial pressure, reduced heart sounds
Pulsus paradoxus
May occur with as little as 100ml blood
Pulmonary contusion
Most common potentially lethal chest injury
Arterial blood gases and pulse oximetry important
Early intubation within an hour if significant hypoxia
Blunt cardiac injury
Usually occurs secondary to chest wall injury
ECG may show features of myocardial infarction
Sequelae: hypotension, arrhythmias, cardiac wall motion abnormalities
Aorta disruption
Deceleration injuries
Contained haematoma
Widened mediastinum
Diaphragm disruption
Most due to motor vehicle accidents and blunt trauma causing large radial tears (laceration injuries result in small tears)
More common on left side
Insert gastric tube, may pass into intrathoracic stomach
Mediastinal traversing wounds
Entrance wound in one hemithorax and exit wound/foreign body in opposite hemithorax
Mediastinal haematoma or pleural cap suggests great vessel injury
Mortality is 20%

References
ATLS Manual 10th Edition. ISBN 78-0-9968262-3-5

919
Q

An 83 year old lady with a significant cardiac history is admitted with rest pain and bilateral leg ulcers. Imaging demonstrates bilateral occlusion of both common iliac arteries that are unsuitable for stenting. What is the most appropriate intervention?

Femoro-femoral cross over graft

Axillo-bifemoral bypass graft

Bilateral trans femoral amputations

Aorto-bifemoral bypass graft

Femoro-distal bypass

A

In patients with major cardiac co-morbidities the safest option is to choose an axillo-bifemoral bypass graft. The long term patency rates are less good than with aorto-bifemoral bypass grafts, however, the operation is less major.

Peripheral vascular disease

Indications for surgery to revascularise the lower limb
Intermittent claudication
Critical ischaemia
Ulceration
Gangrene

Intermittent claudication that is not disabling may provide a relative indication, whilst the other complaints are often absolute indications depending upon the frailty of the patient.

Assessment
Clinical examination
Ankle brachial pressure index measurement
Duplex arterial ultrasound
Angiography (standard, CT or MRI): usually performed only if intervention being considered.

Angioplasty
In order for angioplasty to be undertaken successfully the artery has to be accessible. The lesion relatively short and reasonable distal vessel runoff. Longer lesions may be amenable to sub-intimal angioplasty.

Surgery
Surgery will be undertaken where attempts at angioplasty have either failed or are unsuitable. Bypass essentially involves bypassing the affected arterial segment by utilising a graft to run from above the disease to below the disease. As with angioplasty good runoff improves the outcome.

Some key concepts with bypass surgery

Superficial femoral artery occlusion to the above knee popliteal
In the ideal scenario, vein (either in situ or reversed LSV) would be used as a conduit. However, prosthetic material has reasonable 5 year patency rates and some would advocate using this in preference to vein so that vein can be used for other procedures in the future. In general terms either technique is usually associated with an excellent outcome (if run off satisfactory).

Procedure
Artery dissected out, IV heparin 3,000 units given and then the vessels are cross clamped
Longitudinal arteriotomy
Graft cut to size and tunneled to arteriotomy sites
Anastomosis to femoral artery usually with 5/0 ‘double ended’ Prolene suture
Distal anastomosis usually using 6/0 ‘double ended’ Prolene

Distal disease
Femoro-distal bypass surgery takes longer to perform, is more technically challenging and has higher failure rates.
In elderly diabetic patients with poor runoff a primary amputation may well be a safer and more effective option. There is no point in embarking on this type of surgery in patients who are wheelchair bound.
In femorodistal bypasses vein gives superior outcomes to PTFE.

Rules
Vein mapping 1st to see whether there is suitable vein (the preferred conduit). Sub intimal hyperplasia occurs early when PTFE is used for the distal anastomosis and will lead to early graft occlusion and failure.
Essential operative procedure as for above knee fem-pop.
If there is insufficient vein for the entire conduit then vein can be attached to the end of the PTFE graft and then used for the distal anastomosis. This type of ‘vein boot’ is technically referred to as a Miller Cuff and is associated with better patency rates than PTFE alone.
Remember the more distal the arterial anastomosis the lower the success rate.

References
Peach G et al. Diagnosis and management of peripheral arterial disease. BMJ 2012; 345: 36-41.

920
Q

In a biopsy taken from the gastro intestinal tract, where is a Paneth cell most likely to be identified?

Intestinal villi

Crypt of Lieberkuhn

Gastric antrum

Lamina propria

Gastric fundus

A

Paneth cells are secretory cells located in the crypts of Lieberkuhn. They are an important contributor to the gut microbiome and have roles in host defence and immunity.

Gastrointestinal secretions

Up to 7 litres of gastrointestinal secretions enter the lumen of the GI tract in a 24 hour period. The absorptive function of the small bowel is such that by the time a formed stool is created, it will contain, on average 200ml water.
The common secretions together with their approximate volumes are demonstrated below:

Origin of secretion Volume in ml / 24 hour period Na + mmol/L K+ mmol/L Cl-mmol/L HCO3
Salivary glands 1500 10 26 10 30
Stomach 1500 60 10 130
Duodenum 100-2000 140 80 80
Pancreas 1000 140 5 70 115
Bile 50-800 145 5 100 35
Jejunum/ileum 3000 140 10 104 30
Colon 100 60 30 40

The regulation of these secretions is dependent upon location. In the salivary glands a complex interaction of flow rate governed by the autonomic nervous system. The exact composition of sodium and potassium is regulated by aldosterone. In the stomach hormones such as gastrin play a role and feedback is both endocrine and neurologically mediated (vagus). In the duodenum CCK is released in response to duodenal distension and this causes contraction of the gallbladder and release of bile.

Pancreatic secretions are affected by somatostatin. The secretions in the small bowel are affected by the osmolality of the lumenal contents. This is in part due to the tightness of cellular junctions and in this regard the jejunum is more permeable than the ileum. The practical implication of this is that if an individual has an extensive intestinal resection and a high output, proximally sited stoma then administration of hypotonic rather than isotonic solutions will result in worsening of electrolyte disturbances as electrolyte rich secretions will enter the jejunum.

In some individuals a colectomy or similar procedure results in formation of an end or loop ileostomy. Ileostomies typically lose between 500 and 1000ml over a 24 hour period and patients with high output ileostomies can rapidly become dehydrated. Ileostomy effluent typically contains 126mmol/L of sodium and 22mmol/L of potassium. Knowledge of this fluid composition should guide fluid prescribing in replacing losses.

921
Q

A 48 year old lady is undergoing an axillary node clearance for breast cancer. Which of the structures listed below are most likely to be encountered during the axillary dissection?

Cords of the brachial plexus

Thoracodorsal trunk

Internal mammary artery

Thoracoacromial artery

None of the above

A

Beware of damaging the thoracodorsal trunk if a latissimus dorsi flap reconstruction is planned.
The thoracodorsal trunk runs through the nodes in the axilla. If injured it may compromise the function and blood supply to latissimus dorsi, which is significant if it is to be used as a flap for a reconstructive procedure.

Axilla

Boundaries of the axilla
Medially Chest wall and Serratus anterior
Laterally Humeral head
Floor Subscapularis
Anterior aspect Lateral border of Pectoralis major
Fascia Clavipectoral fascia

Content:
Long thoracic nerve (of Bell) Derived from C5-C7 and passes behind the brachial plexus to enter the axilla. It lies on the medial chest wall and supplies serratus anterior. Its location puts it at risk during axillary surgery and damage will lead to winging of the scapula.
Thoracodorsal nerve and thoracodorsal trunk Innervate and vascularise latissimus dorsi.
Axillary vein Lies at the apex of the axilla, it is the continuation of the basilic vein. Becomes the subclavian vein at the outer border of the first rib.
Intercostobrachial nerves Traverse the axillary lymph nodes and are often divided during axillary surgery. They provide cutaneous sensation to the axillary skin.
Lymph nodes The axilla is the main site of lymphatic drainage for the breast.

922
Q

A 73 year old lady presents with symptoms of faecal incontinence. On examination she has weak anal sphincter muscles. What are the main nerve root values of the nerves supplying the external anal sphincter?

S2,3

L5, S1

S4,5

S5

S2,3,4

A

S2, 3, 4 Keeps the poo off the floor
The external anal sphincter is innervated by the inferior rectal branch of the pudendal nerve, this has root values of S2, 3 and the perineal branch of S4.

Anal sphincter

Internal anal sphincter composed of smooth muscle continuous with the circular muscle of the rectum. It surrounds the upper two- thirds of the anal canal and is supplied by sympathetic nerves.
External anal sphincter is composed of striated muscle which surrounds the internal sphincter but extends more distally.
The nerve supply of the external anal sphincter is from the inferior rectal branch of the pudendal nerve (S2 and S3) and the perineal branch of the S4 nerve roots.

923
Q

A 48 year old man undergoes a right hemicolectomy for a large caecal polyp. In the immediate post operative period which of the physiological processes described below is least likely to occur?

Glycogenolysis

Increased production of acute phase proteins

Increased cortisol production

Bronchoconstriction

Release of nitric oxide by vessels

A

Bronchoconstriction

Response to surgery

Sympathetic nervous system
Noradrenaline from sympathetic nerves and adrenaline from adrenal medulla
Blood diverted from skin and visceral organs; bronchodilatation, reduced intestinal motility, increased glucagon and glycogenolysis, insulin reduced
Heart rate and myocardial contractility are increased

Acute phase response
TNF-α, IL-1, IL-2, IL-6, interferon and prostaglandins are released
Excess cytokines may cause SIRS
Cytokines increase the release of acute phase proteins

Endocrine response
Hypothalamus, pituitary, adrenal axis
Increases ACTH and cortisol production:
increases protein breakdown
increases blood glucose levels
Aldosterone increases sodium re-absorption
Vasopressin increases water re-absorption and causes vasoconstriction

Vascular endothelium
Nitric oxide produces vasodilatation
Platelet activating factor enhances the cytokine response
Prostaglandins produce vasodilatation and induce platelet aggregation

924
Q

In which space is a lumbar puncture performed?

Subdural space

Epidural space

Subarachnoid space

Extradural space

Intraventricular space

A

Samples of CSF are normally obtained by inserting a needle between the third and fourth lumbar vertebrae. The tip of the needle lies in the sub arachnoid space, the spinal cord terminates at L1 and is not at risk of injury. Clinical evidence of raised intracranial pressure is a contraindication to lumbar puncture.

Cerebrospinal fluid

The CSF fills the space between the arachnoid mater and pia mater (covering surface of the brain). The total volume of CSF in the brain is approximately 150ml. Approximately 500 ml is produced by the ependymal cells in the choroid plexus (70%), or blood vessels (30%). It is reabsorbed via the arachnoid granulations which project into the venous sinuses.

Circulation
1. Lateral ventricles (via foramen of Munro)
2. 3rd ventricle
3. Cerebral aqueduct (aqueduct of Sylvius)
4. 4th ventricle
5. Subarachnoid space (via foramina of Magendie and Luschka)
6. Reabsorbed into the venous system via arachnoid granulations into superior sagittal sinus

Composition
Glucose: 50-80mg/dl
Protein: 15-40 mg/dl
Red blood cells: Nil
White blood cells: 0-3 cells/ mm3

925
Q

A 23 year old man falls and slips at a nightclub. A shard of glass penetrates the skin at the level of the medial epicondyle, which of the following sequelae is least likely to occur?

Atrophy of the first dorsal interosseous muscle

Difficulty in abduction of the the 2nd, 3rd, 4th and 5th fingers

Claw like appearance of the hand

Loss of sensation on the anterior aspect of the 5th finger

Partial denervation of flexor digitorum profundus

A

Injury to the ulnar nerve in the mid to distal forearm will typically produce a claw hand. This consists of flexion of the 4th and 5th interphalangeal joints and extension of the metacarpophalangeal joints. The effects are potentiated when flexor digitorum profundus is not affected, and the clawing is more pronounced.More proximally sited ulnar nerve lesions produce a milder clinical picture owing to the simultaneous paralysis of flexor digitorum profundus (ulnar half).
This is the ‘ulnar paradox’, due to the more proximal level of transection the hand will typically not have a claw like appearance that may be seen following a more distal injury. The first dorsal interosseous muscle will be affected as it is supplied by the ulnar nerve.

Ulnar nerve

Origin
C8, T1

Supplies (no muscles in the upper arm)
Flexor carpi ulnaris
Flexor digitorum profundus
Flexor digiti minimi
Abductor digiti minimi
Opponens digiti minimi
Adductor pollicis
Interossei muscle
Third and fourth lumbricals
Palmaris brevis

Path
Posteromedial aspect of upper arm to flexor compartment of forearm, then along the ulnar. Passes beneath the flexor carpi ulnaris muscle, then superficially over the flexor retinaculum into the palm of the hand.

Branches
Branch Supplies
Muscular branch Flexor carpi ulnaris
Medial half of the flexor digitorum profundus
Palmar cutaneous branch (Arises near the middle of the forearm) Skin on the medial part of the palm
Dorsal cutaneous branch Dorsal surface of the medial part of the hand
Superficial branch Cutaneous fibres to the anterior surfaces of the medial one and one-half digits
Deep branch Hypothenar muscles
All the interosseous muscles
Third and fourth lumbricals
Adductor pollicis
Medial head of the flexor pollicis brevis

Effects of injury
Damage at the wrist
Wasting and paralysis of intrinsic hand muscles (claw hand)
Wasting and paralysis of hypothenar muscles
Loss of sensation medial 1 and half fingers
Damage at the elbow
Radial deviation of the wrist
Clawing less in 4th and 5th digits

926
Q

What is the most significant event to contribute to wound healing immediately following injury?

Neutrophil activation

Platelet degranulation

Endothelial cell proliferation

Migration of tissue macrophages

Proliferation of wound bed fibroblasts

A

Many of these events contribute to healing. However, platelet degranulation is the earliest phase and results in haemostasis, the main event is then allowing the release of cytokines to attract other cells types to the wound and co-ordinate healing.

Phases of wound healing

Phase Key features Cells Timeframe
Haemostasis
Vasospasm in adjacent vessels
Platelet plug formation and generation of fibrin rich clot
Erythrocytes and platelets Seconds/ Minutes
Inflammation
Neutrophils migrate into wound (function impaired in diabetes).
Growth factors released, including basic fibroblast growth factor and vascular endothelial growth factor.
Fibroblasts replicate within the adjacent matrix and migrate into wound.
Macrophages and fibroblasts couple matrix regeneration and clot substitution.
Neutrophils, fibroblasts and macrophages Days
Regeneration
Platelet derived growth factor and transformation growth factors stimulate fibroblasts and epithelial cells.
Fibroblasts produce a collagen network.
Angiogenesis occurs and wound resembles granulation tissue.
Fibroblasts, endothelial cells, macrophages Weeks
Remodelling
Longest phase of the healing process and may last up to one year (or longer).
During this phase fibroblasts become differentiated (myofibroblasts) and these facilitate wound contraction.
Collagen fibres are remodelled.
Microvessels regress leaving a pale scar.
Myofibroblasts

927
Q

If a 2 x 2 cm autologus skin graft is placed on an area of healthy granulation tissue. After about a week, a thin bluish - white margin appears around the graft and spreads at a rate of 1mm per day. What is it?

Epidermis alone

Epidermis and dermis

Dermis alone

Inflammatory exudate

Fibrin

A

Epidermis alone. This is the process of re-epithelialisation.

Wound healing

Surgical wounds are either incisional or excisional and either clean, clean contaminated or dirty. Although the stages of wound healing are broadly similar their contributions will vary according to the wound type.

The main stages of wound healing include:

Haemostasis
Minutes to hours following injury
Vasospasm in adjacent vessels, platelet plug formation and generation of fibrin rich clot.

Inflammation
Typically days 1-5
Neutrophils migrate into wound (function impaired in diabetes).
Growth factors released, including basic fibroblast growth factor and vascular endothelial growth factor.
Fibroblasts replicate within the adjacent matrix and migrate into wound.
Macrophages and fibroblasts couple matrix regeneration and clot substitution.

Regeneration
Typically days 7 to 56
Platelet derived growth factor and transformation growth factors stimulate fibroblasts and epithelial cells.
Fibroblasts produce a collagen network.
Angiogenesis occurs and wound resembles granulation tissue.

Remodeling
From 6 weeks to 1 year
Longest phase of the healing process and may last up to one year (or longer).
During this phase fibroblasts become differentiated (myofibroblasts) and these facilitate wound contraction.
Collagen fibres are remodeled.
Microvessels regress leaving a pale scar.

The above description represents an idealised scenario. A number of diseases may distort this process. Neovascularisation is an important early process. Endothelial cells may proliferate in the wound bed and recanalise to form a vessel. Vascular disease, shock and sepsis can all compromise microvascular flow and impair healing.

Conditions such as jaundice will impair fibroblast synthetic function and immunity with a detrimental effect in most parts of the healing process.

Problems with scars:

Hypertrophic scars
Excessive amounts of collagen within a scar. Nodules may be present histologically containing randomly arranged fibrils within and parallel fibres on the surface. The tissue itself is confined to the extent of the wound itself and is usually the result of a full thickness dermal injury. They may go on to develop contractures.
Keloid scars
Excessive amounts of collagen within a scar. Typically a keloid scar will pass beyond the boundaries of the original injury. They do not contain nodules and may occur following even trivial injury. They do not regress over time and may recur following removal.

Drugs which impair wound healing:
Non steroidal anti inflammatory drugs
Steroids
Immunosupressive agents
Anti neoplastic drugs

Closure
Delayed primary closure is the anatomically precise closure that is delayed for a few days but before granulation tissue becomes macroscopically evident.

Secondary closure refers to either spontaneous closure or to surgical closure after granulation tissue has formed.

928
Q

A 15-month-old girl presents with a three day history of periorbital oedema. She is brought to hospital. On examination she has facial oedema and a tender distended abdomen. Her temperature is 39oC and her blood pressure is 90/45 mmHg. There is clinical evidence of poor peripheral perfusion. What is the most likely diagnosis?

Sickle cell crisis

Intussusception

Spontaneous bacterial peritonitis

Henoch Schonlein purpura

Appendicitis

A

The 15-month-old girl is a patient with nephrotic syndrome. Patients with this condition are at risk of septicaemia and peritonitis from Streptococcus pneumonia, due to the loss of immunoglobulins and opsonins in the urine.

929
Q

A 45 year old man is undergoing a lymph node biopsy from the posterior triangle of his neck. Which structure forms the posterior border of this region?

Trapezius muscle

Diagastric muscle

External jugular vein

Omohyoid muscle

Sternocleidomastoid muscle

A

Trapezius muscle.

Posterior triangle of the neck

Boundaries
Apex Sternocleidomastoid and the Trapezius muscles at the Occipital bone
Anterior Posterior border of the Sternocleidomastoid
Posterior Anterior border of the Trapezius
Base Middle third of the clavicle

Contents
Nerves
Accessory nerve
Phrenic nerve
Three trunks of the brachial plexus
Branches of the cervical plexus: Supraclavicular nerve, transverse cervical nerve, great auricular nerve, lesser occipital nerve
Vessels
External jugular vein
Subclavian artery
Muscles
Inferior belly of omohyoid
Scalene
Lymph nodes
Supraclavicular
Occipital

930
Q

A 56 year old man is left impotent following an abdomino-perineal excision of the colon and rectum. What is the most likely explanation?

Psychosexual issues related to an end colostomy

Damage to the sacral venous plexus during total mesorectal excision

Damage to the left ureter during sigmoid mobilisation

Damage to the hypogastric plexus during mobilisation of the inferior mesenteric artery

Damage to the internal iliac artery during total mesorectal excision

A

D. Autonomic nerve injury is the most common cause.

Nerve lesions during surgery

A variety of different procedures carry the risk of iatrogenic nerve injury. These are important not only from the patients perspective but also from a medicolegal standpoint.

The following operations and their associated nerve lesions are listed here:
Posterior triangle lymph node biopsy and accessory nerve lesion.
Lloyd Davies stirrups and common peroneal nerve.
Thyroidectomy and laryngeal nerve.
Anterior resection of rectum and hypogastric autonomic nerves.
Axillary node clearance; long thoracic nerve, thoracodorsal nerve and intercostobrachial nerve.
Inguinal hernia surgery and ilioinguinal nerve.
Varicose vein surgery- sural and saphenous nerves.
Posterior approach to the hip and sciatic nerve.
Carotid endarterectomy and hypoglossal nerve.

There are many more, with sound anatomical understanding of the commonly performed procedures the incidence of nerve lesions can be minimised. They commonly occur when surgeons operate in an unfamiliar tissue plane or by blind placement of haemostats (not recommended).

931
Q

Mobilisation of the left lobe of the liver will facilitate surgical access to which of the following?

Abdominal oesophagus

Duodenum

Right colic flexure

Right kidney

Pylorus of stomach

A

A. The fundus of the stomach is a posterior relation. The pylorus lies more inferolaterally. During a total gastrectomy division of the ligaments holding the left lobe of the liver will facilitate access to the proximal stomach and abdominal oesophagus. This manoeuvre is seldom beneficial during a distal gastrectomy.

Liver

Structure of the liver
Right lobe
Supplied by right hepatic artery
Contains Couinaud segments V to VIII (-/+Sg I)
Left lobe
Supplied by the left hepatic artery
Contains Couinaud segments II to IV (+/- Sg1)
Quadrate lobe
Part of the right lobe anatomically, functionally is part of the left
Couinaud segment IV
Porta hepatis lies behind
On the right lies the gallbladder fossa
On the left lies the fossa for the umbilical vein
Caudate lobe
Supplied by both right and left hepatic arteries
Couinaud segment I
Lies superior to the porta hepatis
Anterior and medial to the inferior vena cava
Bile from the caudate lobe drains into both right and left hepatic ducts

Detailed knowledge of Couinaud segments is not required for MRCS
Between the liver lobules are portal canals which contain the portal triad: Hepatic Artery, Portal Vein, tributary of Bile Duct.

Relations of the liver
Anterior Postero inferiorly
Diaphragm Oesophagus
Xiphoid process Stomach
Duodenum
Hepatic flexure of colon
Right kidney
Gallbladder
Inferior vena cava

Porta hepatis
Location Postero inferior surface, it joins nearly at right angles with the left sagittal fossa, and separates the caudate lobe behind from the quadrate lobe in front
Transmits
Common hepatic duct
Hepatic artery
Portal vein
Sympathetic and parasympathetic nerve fibres
Lymphatic drainage of the liver (and nodes)

Ligaments
Falciform ligament
2 layer fold peritoneum from the umbilicus to anterior liver surface
Contains ligamentum teres (remnant umbilical vein)
On superior liver surface it splits into the coronary and left triangular ligaments
Ligamentum teres Joins the left branch of the portal vein in the porta hepatis
Ligamentum venosum Remnant of ductus venosus

Arterial supply
Hepatic artery

Venous
Hepatic veins
Portal vein

Nervous supply
Sympathetic and parasympathetic trunks of coeliac plexus

932
Q

Which of the following variables is not included in the Rockall score?

Congestive cardiac failure

Liver failure

Systolic blood pressure < 100mmHg

Aspirin usage

Age

A

Patients should have their Rockall score calculated following endoscopy for upper GI haemorrhage

Mnemonic for Rockall score
ABCDE
A: Age
B: Blood pressure drop (Shock)
C: Co-morbidity
D: Diagnosis
E: Evidence of bleeding

Rockall Score

Applies to upper gastrointestinal bleeding

Variable Score 0 Score 1 Score 2 Score 3
Age <60 60-79 >80
Shock None Pulse >100 Hypotension (systolic <100mmHg)
Co-Morbidity Nil or minimal Major organ disease e.g. IHD, CCF Renal or liver failure, metastatic cancer
Diagnosis Mallory-Weiss All GI Cancer
Evidence of Bleeding None Clot, Blood, spurting vessel

Score <3 = Good prognosis (mortality approx. 2%)

Score >8= High mortality (Mortality approx. 40%)

Upper gastrointestinal bleeding

Patients may present with the following:
Haematemesis and/ or malaena
Epigastric discomfort
Sudden collapse

The extent to which these will occur will depend upon the source. Mortality is higher in patients presenting with haematemesis than malaena alone.

Oesophageal bleeding
Cause Presenting features
Oesophagitis Small volume of fresh blood, often streaking vomit. Malaena rare. Often ceases spontaneously. Usually history of antecedent GORD type symptoms.
Cancer Usually small volume of blood, except as pre terminal event with erosion of major vessels. Often associated symptoms of dysphagia and constitutional symptoms such as weight loss. May be recurrent until malignancy managed.
Mallory Weiss Tear Typically brisk small to moderate volume of bright red blood following bout of repeated vomiting. Malaena rare. Usually ceases spontaneously.
Varices Usually large volume of fresh blood. Swallowed blood may cause malaena. Often associated with haemodynamic compromise. May stop spontaneously but re-bleeds are common until appropriately managed.

Gastric Bleeding
Cause Presenting features
Gastric cancer May be frank haematemesis or altered blood mixed with vomit. Usually prodromal features of dyspepsia and may have constitutional symptoms. Amount of bleeding variable but erosion of major vessel may produce considerable haemorrhage.
Dieulafoy Lesion Often no prodromal features prior to haematemesis and malaena, but this arteriovenous malformation may produce quite considerable haemorrhage and may be difficult to detect endoscopically.
Diffuse erosive gastritis Usually haematemesis and epigastric discomfort. Usually there is an underlying cause such as recent NSAID usage. Large volume haemorrhage may occur with considerable haemodynamic compromise.
Gastric ulcer Small low volume bleeds more common so would tend to present as iron deficiency anaemia. Erosion into a significant vessel may produce considerable haemorrhage and haematemesis.

Duodenum
Most common cause of major haemorrhage is a posteriorly sited duodenal ulcer. However, ulcers at any site in the duodenum may present with haematemesis, malaena and epigastric discomfort. The pain of duodenal ulcer is slightly different to that of gastric ulcers and often occurs several hours after eating. Peri ampullary tumours may bleed but these are rare. In patients with previous abdominal aortic aneurysm surgery aorto-enteric fistulation remains a rare but important cause of major haemorrhage associated with high mortality.

Management
Admission to hospital careful monitoring, cross match blood, check FBC, LFTs, U+E and Clotting (as a minimum)
Patients with on-going bleeding and haemodynamic instability are likely to require O negative blood pending cross matched blood
Early control of airway is vital (e.g. Drowsy patient with liver failure)
Patients with suspected varices should receive terlipressin prior to endoscopy
Ideally all patients admitted with upper gastrointestinal haemorrhage should undergo Upper GI endoscopy within 24 hours of admission. In those who are unstable this should occur immediately after resuscitation or in tandem with it. The endoscopy department is a potentially dangerous place for unstable patients and it may be safer to perform the endoscopy in theatre with an anaesthetist present.
Varices should be banded or subjected to sclerotherapy. If this is not possible owing to active bleeding then a Sengstaken- Blakemore tube (or Minnesota tube) should be inserted. This should be done with care; gastric balloon should be inflated first and oesophageal balloon second. Remember the balloon will need deflating after 12 hours (ideally sooner) to prevent necrosis. Portal pressure should be lowered by combination of medical therapy +/- TIPSS.
Patients with erosive oesophagitis / gastritis should receive a proton pump inhibitor.
Mallory Weiss tears will typically resolve spontaneously
Identifiable bleeding points should receive combination therapy of injection of adrenaline and either a thermal or mechanical treatment. All who have received intervention should receive a continuous infusion of a proton pump inhibitor (IV omeprazole for 72 hours) to reduce the re-bleeding rate.
Patients with diffuse erosive gastritis who cannot be managed endoscopically and continue to bleed may require gastrectomy
Bleeding ulcers that cannot be controlled endoscopically may require laparotomy and ulcer underruning

Indications for surgery
Patients > 60 years
Continued bleeding despite endoscopic intervention
Recurrent bleeding
Known cardiovascular disease with poor response to hypotension

Surgery
Duodenal ulcer
Laparotomy, duodenotomy and under running of the ulcer. If bleeding is brisk then the ulcer is almost always posteriorly sited and will have invaded the gastroduodenal artery. Large bites using 0 Vicryl are taken above and below the ulcer base to occlude the vessel. The duodenotomy should be longitudinal but closed transversely to avoid stenosis.

For gastric ulcer
Under-running of the bleeding site
Partial gastrectomy-antral ulcer
Partial gastrectomy or under running the ulcer- lesser curve ulcer (involving left gastric artery)
Total gastrectomy if bleeding persists

Summary of Acute Upper GI bleeding recommendations:
The need for admission and timing of endoscopic intervention may be predicted by using the Blatchford score. This considers a patients Hb, serum urea, pulse rate and blood pressure. Those patients with a score of 0 are low risk, all others are considered high risk and require admission and endoscopy.
The requirement for pre endoscopic proton pump inhibition is contentious. In the UK the National Institute of Clinical Excellence guidelines suggest the pre endoscopic PPI therapy is unnecessary. Whilst it is accepted that such treatment has no impact on mortality or morbidity a Cochrane review of this practice in 2007 did suggest that it reduced the stigmata of recent haemorrhage at endoscopy. As a result many will still administer PPI to patients prior to endoscopic intervention.
Following endoscopy it is important to calculate the Rockall score for patients to determine their risk of rebleeding and mortality. A score of 3 or less is associated with a rebleeding rate of 4% and a very low risk of mortality and identifies a group of patients suitable for early discharge.

References
1. http://www.sign.ac.uk/guidelines/fulltext/105/index.html
2. Joint Advisory Group on Endoscopy (JAG) Guidelines - http://www.thejag.org.uk
3. NICE Guideline: Management of acute upper GI bleeding. July 2012.

933
Q

A 35 year old woman has undergone a wide local excision. The histology shows an invasive lobular carcinoma present at 3 of the resection margins. Cavity shavings taken at the original operation are also involved. Sentinel node biopsy was negative. What is the most appropriate course of action?

Arrange for re-excision of margins

Arrange for breast radiotherapy alone

Arrange for completion mastectomy alone

Arrange for radical radiotherapy to the breast and axilla

Arrange for completion mastectomy and axillary node clearance

A

This patient has an extensive disease process and lobular cancers are notorious for being multifocal. In this case a mastectomy is the safest next step. Radiotherapy is not appropriate in this setting as the margins are not clear.

Breast cancer management

  • Surgery is performed in most patients suffering from breast cancer.
    Chemotherapy may be used to downstage tumours and allow breast conserving surgery. Hormonal therapy may also be used for the same purposes.
    Radiotherapy is given to most patients who have undergone breast conserving surgery (some older patients receiving hormone treatment and who have small low grade tumours may safely avoid DXT.
    Therapeutic mammoplasty is an option for some patients but requires symmetrizing surgery in most cases.
    Patients who have undergone mastectomy may be offered a reconstructive procedure either in conjunction with their primary resection or as a staged procedure at a later date.

Surgical options
Mastectomy vs Wide local excision

Mastectomy Wide Local Excision
Multifocal tumour Solitary lesion
Central tumour Peripheral tumour
Large lesion in small breast Small lesion in large breast
DCIS >4cm DCIS <4cm
Patient Choice Patient choice

Central lesions may be managed using breast conserving surgery, where an acceptable cosmetic result may be obtained, this is rarely the case in small breasts

Axillary disease
As a minimum, all patients with invasive breast cancer should have their axilla staged. In those who do not have overt evidence of axillary nodal involvement this can be undertaken using sentinel lymph node biopsy.
Patients with a positive sentinel lymph node biopsy or who have imaging and cytological or histological evidence of axillary nodal metastasis should undergo axillary node clearance or axillary irradiation.
Axillary node clearance is associated with the development of lymphoedema, increased risk of cellulitis and frozen shoulder.

934
Q

Which of the following nerves is responsible for the innervation of the posterior belly of the digastric muscle?

Facial nerve

Hypoglossal nerve

Trigeminal nerve

Ansa cervicalis

Mylohyoid nerve

A

The posterior belly of digastric is innervated by the facial nerve and the anterior belly by the mylohoid nerve.

Anterior triangle of the neck

Boundaries
Anterior border of the Sternocleidomastoid
Lower border of mandible
Anterior midline

Sub triangles (divided by Digastric above and Omohyoid)
Muscular triangle: Neck strap muscles
Carotid triangle: Carotid sheath
Submandibular Triangle (digastric)

Contents of the anterior triangle
Digastric triangle Submandibular gland
Submandibular nodes
Facial vessels
Hypoglossal nerve
Muscular triangle Strap muscles
Jugular vein
Carotid triangle Carotid sheath (Common carotid, vagus and internal jugular vein)
Ansa cervicalis

Nerve supply to digastric muscle
Anterior: Mylohyoid nerve
Posterior: Facial nerve

935
Q

Which of the following drugs increases the rate of gastric emptying in the vagotomised stomach?

Ondansetron

Metoclopramide

Cyclizine

Erythromycin

Chloramphenicol

A

Vagotomy seriously compromises gastric emptying which is why either a pyloroplasty or gastro-enterostomy is routinely performed at the same time.
Chloramphenicol has no effect on gastric emptying. Ondansetron slows gastric emptying slightly. Metoclopramide increases the rate of gastric emptying but its effects are mediated via the vagus nerve. Erythromycin enhances gastric emptying by acting via the motilin receptor in the gut.

Gastric emptying

The stomach serves both a mechanical and immunological function. Solid and liquid are retained in the stomach during which time repeated peristaltic activity against a closed pyloric sphincter will cause fragmentation of food bolus material. Contact with gastric acid will help to neutralise any pathogens present.
The amount of time material spends in the stomach is related to its composition and volume. For example a glass of water will empty more quickly than a large meal. The presence of amino acids and fat will all serve to delay gastric emptying.

Controlling factors
Neuronal stimulation of the stomach is mediated via the vagus and the parasympathetic nervous system will tend to favor an increase in gastric motility. It is for this reason that individuals who have undergone truncal vagotomy will tend to routinely require either a pyloroplasty or gastro-enterostomy as they would otherwise have delayed gastric emptying.

The following hormonal factors are all involved:

Delay emptying Increase emptying
Gastric inhibitory peptide Gastrin
Cholecystokinin
Enteroglucagon

Diseases affecting gastric emptying
All diseases that affect gastric emptying may result in bacterial overgrowth, retained food and eventually the formation of bezoars that may occlude the pylorus and make gastric emptying even worse. Fermentation of food may cause dyspepsia, reflux and foul smelling belches of gas.

Iatrogenic
Gastric surgery can have profound effects on gastric emptying. As stated above any procedure that disrupts the vagus can cause delayed emptying. Whilst this is particularly true of vagotomy, this operation is now rarely performed. Surgeons are divided on the importance of vagal disruption that occurs during an oesophagectomy, some will routinely perform a pyloroplasty and others will not.

When a distal gastrectomy is performed, the type of anastomosis performed will impact on emptying. When a gastro-enterostomy is constructed, a posterior, retrocolic gastroenterostomy will empty better than an anterior one.

Diabetic gastroparesis
This is predominantly due to neuropathy affecting the vagus nerve. The stomach empties poorly and patients may have episodes of repeated and protracted vomiting. Diagnosis is made by upper GI endoscopy and contrast studies, in some cases a radio nucleotide scan is needed to demonstrate the abnormality more clearly. In treating these conditions, drugs such as metoclopramide will be less effective as they exert their effect via the vagus nerve. One of the few prokinetic drugs that do not work in this way is the antibiotic erythromycin.

Malignancies
Obviously a distal gastric cancer may obstruct the pylorus and delay emptying. In addition, malignancies of the pancreas may cause extrinsic compression of the duodenum and delay emptying. Treatment in these cases is by gastric decompression using a wide bore nasogastric tube and insertion of a stent or, if that is not possible, by a surgical gastroenterostomy. As a general rule gastroenterostomies constructed for bypass of malignancy are usually placed on the anterior wall of the stomach (in spite of the fact that they empty less well). A Roux en Y bypass may also be undertaken, but the increased number of anastomoses for this, in malignant disease that is being palliated, is probably not justified.

Congenital Hypertrophic Pyloric Stenosis
This is typically a disease of infancy. Most babies will present around 6 weeks of age with projectile non bile stained vomiting. It has an incidence of 2.4 per 1000 live births and is more common in males. Diagnosis is usually made by careful history and examination and a mass may be palpable in the epigastrium (often cited seldom felt!). The most important diagnostic test is an ultrasound that usually demonstrates the hypertrophied pylorus. Blood tests may reveal a hypochloraemic metabolic alkalosis if the vomiting is long standing. Once the diagnosis is made the infant is resuscitated and a pyloromyotomy is performed (either open or laparoscopically). Once treated there are no long term sequelae.

936
Q

What is the origin of the superior gluteal artery?

Internal iliac artery

External iliac artery

Femoral artery

Common iliac artery

Circumflex femoral artery

A

The inferior gluteal artery arises from the anterior trunk of the internal iliac artery
The superior gluteal artery arises from the posterior trunk of the internal iliac artery

Gluteal region

Gluteal muscles
Gluteus maximus: inserts to gluteal tuberosity of the femur and iliotibial tract
Gluteus medius: attach to lateral greater trochanter
Gluteus minimis: attach to anterior greater trochanter
All extend and abduct the hip

Deep lateral hip rotators
Piriformis
Gemelli
Obturator internus
Quadratus femoris

Nerves
Superior gluteal nerve (L4,L5, S1)
Gluteus medius
Gluteus minimis
Tensor fascia lata
Inferior gluteal nerve (L5, S1, S2) Gluteus maximus
Damage to the superior gluteal nerve will result in the patient developing a Trendelenberg gait. Affected patients are unable to abduct the thigh at the hip joint. During the stance phase, the weakened abductor muscles allow the pelvis to tilt down on the opposite side. To compensate, the trunk lurches to the weakened side to attempt to maintain a level pelvis throughout the gait cycle. The pelvis sags on the opposite side of the lesioned superior gluteal nerve.

937
Q

A 73 year old man presents with a tumour at the central aspect of the posterior third of the tongue. To which of the following lymph node groups is it most likely to metastasise?

Submental

Submandibular

Ipsilateral deep cervical nodes

Contralateral deep cervical nodes

Bilateral deep cervical nodes

A

Posterior third tumours of the tongue commonly metastasise to the bilateral deep cervical lymph nodes
Tumours of the posterior third of the tongue will typically metastasise early and bilateral nodal involvement is well recognised, this is most often true of centrally located tumours and those adjacent to the midline as the lymph vessels may cross the median plane at this location.

Lymphatic drainage of the tongue

  • The lymphatic drainage of the anterior two thirds of the tongue shows only minimal communication of lymphatics across the midline, so metastasis to the ipsilateral nodes is usual.
    The lymphatic drainage of the posterior third of the tongue have communicating networks, as a result early bilateral nodal metastases are more common in this area.
    Lymphatics from the tip of the tongue usually pass to the sub mental nodes and from there to the deep cervical nodes.
    Lymphatics from the mid portion of the tongue usually drain to the submandibular nodes and then to the deep cervical nodes. Mid tongue tumours that are laterally located will usually drain to the ipsilateral deep cervical nodes, those from more central regions may have bilateral deep cervical nodal involvement.
938
Q

Which nerve supplies sensation to the nail bed of the index finger?

Median

Ulnar

Radial

Musculocutaneous

Axillary

A

Median nerve

The median nerve is formed by the union of a lateral and medial root respectively from the lateral (C5,6,7) and medial (C8 and T1) cords of the brachial plexus; the medial root passes anterior to the third part of the axillary artery. The nerve descends lateral to the brachial artery, crosses to its medial side (usually passing anterior to the artery). It passes deep to the bicipital aponeurosis and the median cubital vein at the elbow.
It passes between the two heads of the pronator teres muscle, and runs on the deep surface of flexor digitorum superficialis (within its fascial sheath).
Near the wrist it becomes superficial between the tendons of flexor digitorum superficialis and flexor carpi radialis, deep to palmaris longus tendon. It passes deep to the flexor retinaculum to enter the palm, but lies anterior to the long flexor tendons within the carpal tunnel.

Branches
Region Branch
Upper arm No branches, although the nerve commonly communicates with the musculocutaneous nerve
Forearm Pronator teres
Pronator quadratus
Flexor carpi radialis
Palmaris longus
Flexor digitorum superficialis
Flexor pollicis longus
Flexor digitorum profundus (only the radial half)
Distal forearm Palmar cutaneous branch
Hand (Motor) Motor supply (LOAF)
Lateral 2 lumbricals
Opponens pollicis
Abductor pollicis brevis
Flexor pollicis brevis
Hand (Sensory)
Over thumb and lateral 2 ½ fingers
On the palmar aspect this projects proximally, on the dorsal aspect only the distal regions are innervated with the radial nerve providing the more proximal cutaneous innervation.

Patterns of damage
Damage at wrist
e.g. carpal tunnel syndrome
paralysis and wasting of thenar eminence muscles and opponens pollicis (ape hand deformity)
sensory loss to palmar aspect of lateral (radial) 2 ½ fingers

Damage at elbow, as above plus:
unable to pronate forearm
weak wrist flexion
ulnar deviation of wrist

Anterior interosseous nerve (branch of median nerve)
leaves just below the elbow
results in loss of pronation of forearm and weakness of long flexors of thumb and index finger

939
Q

A 34 year old woman is admitted with cholangitis. Her bilirubin is 180 and alkaline phosphatase is 348. She becomes progressively more unwell and develops abdominal pain. The F1 checks her amylase which is elevated at 1080. Standard treatment is initiated and her Glasgow score is 3. What is the most appropriate course of action?

ERCP

PTC

Laparotomy

Pancreatic necrosectomy

Cholecystectomy

A

She requires urgent decompression of her biliary system. An ERCP is the conventional method of performing this. It is important to ensure that her coagulation status is normalised prior to performing this procedure.

Management of Pancreatitis

Management of Acute Pancreatitis in the UK

Diagnosis
Traditionally hyperamylasaemia has been utilised with amylase being elevated three times the normal range.
However, amylase may give both false positive and negative results.
Serum lipase is both more sensitive and specific than serum amylase. It also has a longer half life.
Serum amylase levels do not correlate with disease severity.

Differential causes of hyperamylasaemia
Acute pancreatitis
Pancreatic pseudocyst
Mesenteric infarct
Perforated viscus
Acute cholecystitis
Diabetic ketoacidosis

Assessment of severity
Glasgow, Ranson scoring systems and APACHE II
Biochemical scoring e.g. using CRP

Features that may predict a severe attack within 48 hours of admission to hospital
Initial assessment
Clinical impression of severity
Body mass index >30
Pleural effusion
APACHE score >8
24 hours after admission
Clinical impression of severity
APACHE II >8
Glasgow score of 3 or more
Persisting multiple organ failure
CRP>150
48 hours after admission
Glasgow Score of >3
CRP >150
Persisting or progressive organ failure
Table adapted from UK guidelines for management of acute pancreatitis. GUT 2005, 54 suppl III

Management

Nutrition
There is reasonable evidence to suggest that the use of enteral nutrition does not worsen the outcome in pancreatitis
Most trials to date were underpowered to demonstrate a conclusive benefit.
The rationale behind feeding is that it helps to prevent bacterial translocation from the gut, thereby contributing to the development of infected pancreatic necrosis.

Use of antibiotic therapy
Many UK surgeons administer antibiotics to patients with acute pancreatitis. However, there is very little evidence to support this practice.
A recent Cochrane review highlights the potential benefits of administering Imipenem to patients with established pancreatic necrosis in the hope of averting the progression to infection.
There are concerns that the administration of antibiotics in mild attacks of pancreatitis will not affect outcome and may contribute to antibiotic resistance and increase the risks of antibiotic associated diarrhoea.

Surgery
Patients with acute pancreatitis due to gallstones should undergo early cholecystectomy.
Patients with obstructed biliary system due to stones should undergo early ERCP.
Patients with extensive necrosis where infection is suspected should usually undergo FNA for culture.
Patients with infected necrosis should undergo either radiological drainage or surgical necrosectomy. The choice of procedure depends upon local expertise.

References
www.bsg.org.uk/pdfworddocs/pancreatic.pdf

Antibiotic therapy for prophylaxis against infection of pancreatic necrosis in acute pancreatitis. Villatoro et al. Cochrane Library DOI: 10.1002/14651858.CD002941.pub3. 2010 version.

940
Q

At the level of the wrist joint, which of the statements below best describes the relationship of the ulnar artery to the ulnar nerve?

It lies on its radial side

It lies deep to it

It lies superficial to it

It lies on its ulnar side

None of the above

A

n the middle of the forearm, the artery is overlapped by the flexor carpi ulnaris and on the flexor retinaculum it is covered by a superficial layer from that structure. In its distal two-thirds, flexor digitorum superficialis lies on its radial side, and the ulnar nerve is situated on its ulnar side.
Ulnar artery

Path
Starts: middle of antecubital fossa
Passes obliquely downward, reaching the ulnar side of the forearm at a point about midway between the elbow and the wrist. It follows the ulnar border to the wrist, crossing over the flexor retinaculum. It then divides into the superficial and deep volar arches.

Relations
Deep to- Pronator teres, Flexor carpi ulnaris, Palmaris longus
Lies on- Brachialis and Flexor digitorum profundus
Superficial to the flexor retinaculum at the wrist

The median nerve is in relation with the medial side of the artery for about 2.5 cm. And then crosses the vessel, being separated from it by the ulnar head of the Pronator teres

The ulnar nerve lies medially to the lower two-thirds of the artery

Branch
Anterior interosseous artery

941
Q

Which of the following is the most common childhood brain tumour?

Glioblastoma multiforme

Astrocytoma

Medulloblastoma

Ependymoma

Meningioma

A

Glioblastoma multiforme is rare in childhood. In contrast, astrocytoma is the commonest brain tumour in children. Medulloblastoma is no longer the commonest CNS tumour in children (Cancer research UK)

CNS tumours

60% = Glioma and metastatic disease
20% = Meningioma
10% = Pituitary lesions
In paediatric practice medulloblastomas (neuroectodermal tumours) were the commonest lesions, astrocytomas now account for the majority.
Tumours arising in right temporal and frontal lobe may reach considerable size before becoming symptomatic. Whereas tumours in the speech and visual areas will typically produce early symptoms.

Most paediatric CNS tumours are infratentorial
Most adult CNS tumours are supratentorial

Diagnosis
MRI Scanning provides the best resolution.

Treatment
Usually surgery, even if tumour cannot be completely resected conditions such as rising ICP can be addressed with tumour debulking and survival and quality of life prolonged.
Curative surgery can usually be undertaken with lesions such as meningiomas. Gliomas have a marked propensity to invade normal brain and resection of these lesions is nearly always incomplete.

942
Q

Which of the following renal stone types is most radiodense on a plain x-ray?

Calcium phosphate

Calcium oxalate

Uric acid

Struvite

Cystine

A

Calcium phosphate stones are the most radiodense stones, calcium oxalate stones slightly less so. Uric acid stones are radiolucent (unless they have calcium contained within them).

Renal stones

Type of stones Features Percentage of all calculi
Calcium oxalate Hypercalciuria is a major risk factor (various causes)
Hyperoxaluria may also increase risk
Hypocitraturia increases risk because citrate forms complexes with calcium making it more soluble
Stones are radio-opaque (though less than calcium phosphate stones)
Hyperuricosuria may cause uric acid stones to which calcium oxalate binds 85%
Cystine Inherited recessive disorder of transmembrane cystine transport leading to decreased absorption of cystine from intestine and renal tubule
Multiple stones may form
Relatively radiodense because they contain sulphur 1%
Uric acid Uric acid is a product of purine metabolism
May precipitate when urinary pH low
May be caused by diseases with extensive tissue breakdown e.g. malignancy
More common in children with inborn errors of metabolism
Radiolucent 5-10%
Calcium phosphate May occur in renal tubular acidosis, high urinary pH increases supersaturation of urine with calcium and phosphate
Renal tubular acidosis types 1 and 3 increase risk of stone formation (types 2 and 4 do not)
Radio-dense stones (composition similar to bone) 10%
Struvite Stones formed from magnesium, ammonium and phosphate
Occur as a result of urease producing bacteria (and are thus associated with chronic infections)
Under the alkaline conditions produced, the crystals can precipitate
Slightly radio-opaque 2-20%

Effect of urinary pH on stone formation
Urine pH will show individual variation (from pH 5-7). Post prandially the pH falls as purine metabolism will produce uric acid. Then the urine becomes more alkaline (alkaline tide). When the stone is not available for analysis the pH of urine may help to determine which stone was present.

Stone type Urine acidity Mean urine pH
Calcium phosphate Normal- alkaline >5.5
Calcium oxalate Variable 6
Uric acid Acid 5.5
Struvite Alkaline >7.2
Cystine Normal 6.5

943
Q

Which of the following structures lies deepest in the popliteal fossa?

Popliteal artery

Popliteal vein

Tibial nerve

Common peroneal nerve

Popliteal lymph nodes

A

From superficial to deep:
The common peroneal nerve exits the popliteal fossa along the medial border of the biceps tendon. Then the tibial nerve lies lateral to the popliteal vessels to pass posteriorly and then medially to them. The popliteal vein lies superficial to the popliteal artery, which is the deepest structure in the fossa.
Popliteal fossa

Boundaries of the popliteal fossa
Laterally Biceps femoris above, lateral head of gastrocnemius and plantaris below
Medially Semimembranosus and semitendinosus above, medial head of gastrocnemius below
Floor Popliteal surface of the femur, posterior ligament of knee joint and popliteus muscle
Roof Superficial and deep fascia

Contents
Popliteal artery and vein
Small saphenous vein
Common peroneal nerve
Tibial nerve
Posterior cutaneous nerve of the thigh
Genicular branch of the obturator nerve
Lymph nodes

944
Q

Which of the following are not true of Li-Fraumeni syndrome?

It consists of mutations to the p53 tumour suppressor gene

Is likely to be present in a teenager presenting with a liposarcoma

It has an autosomal dominant inheritance pattern

Affected individuals are unlikely to develop acute myeloid leukaemia

Adrenal malignancies are more common than in normal population

A

The classical LFS malignancies - sarcoma, cancers of the breast, brain and adrenal glands - comprise about 80% of all cancers that occur in this syndrome.
They are at high risk of developing leukaemia.

Genetics and surgical disease

Some of the more commonly occurring genetic conditions occurring in surgical patients are presented here.

Li-Fraumeni Syndrome
Autosomal dominant
Consists of germline mutations to p53 tumour suppressor gene
High incidence of malignancies particularly sarcomas and leukaemias
Diagnosed when:

*Individual develops sarcoma under 45 years
*First degree relative diagnosed with any cancer below age 45 years and another family member develops malignancy under 45 years or sarcoma at any age

BRCA 1 and 2
Carried on chromosome 17 (BRCA 1) and Chromosome 13 (BRCA 2)
Linked to developing breast cancer (60%) risk.
Associated risk of developing ovarian cancer (55% with BRCA 1 and 25% with BRCA 2).

Lynch Syndrome
Autosomal dominant
Develop colonic cancer and endometrial cancer at young age
80% of affected individuals will get colonic and/ or endometrial cancer
High risk individuals may be identified using the Amsterdam criteria

Amsterdam criteria
Three or more family members with a confirmed diagnosis of colorectal cancer, one of whom is a first degree (parent, child, sibling) relative of the other two.
Two successive affected generations.
One or more colon cancers diagnosed under age 50 years.
Familial adenomatous polyposis (FAP) has been excluded.

Gardners syndrome
Autosomal dominant familial colorectal polyposis
Multiple colonic polyps
Extra colonic diseases include: skull osteoma, thyroid cancer and epidermoid cysts
Desmoid tumours are seen in 15%
Mutation of APC gene located on chromosome 5
Due to colonic polyps most patients will undergo colectomy to reduce risk of colorectal cancer
Now considered a variant of familial adenomatous polyposis coli

945
Q

A 70 year old man has a basal cell carcinoma affecting his temple. Following excision and histopathological assessment, which of the factors outlined below has the greatest bearing on prognosis?

Pathological subtype

Completeness of excision

Lymphocytic infiltrate

Lymphovascular invasion

Mitotic index

A

Incompletely excised BCC have upwards of 30% recurrence rates. Systemic spread with these lesions is rare and it is usually local recurrence that is the main problem.

Skin disorders in surgery- malignancy and related lesions

Non melanoma skin cancer (BCC and SCC) are some of the commonest types of human malignancy. Up to 80% of these are BCC’s with approximately 20% comprising SCC’s. The incidence of NMSC’s increases with age and whilst there is a female preponderance in those under 40 years of age, in latter life the sex incidence is roughly equal.
The vast majority of NMSC’s are related to UV light exposure. For SCC’s the major pattern is in chronic long term exposure. For BCC’s, the pattern of sporadic exposure with episodes of burning is more important. Organ transplant recipients have a markedly increased incidence of SCC, risk factors include length of immunosuppression, ethnic origin and associated sunlight exposure. Human papilloma virus DNA is found in the majority of transplant recipient SCC’s. In addition to this increased risk, transplant recipients are also more likely to develop locoregional recurrences following treatment.

Actinic keratosis and SCC
Actinic keratosis is viewed as a premalignant lesion because there are atypical keratinocytes present in the epidermis. In a person with 7 actinic keratosis the risks of subsequent SCC is of the order of 10% at 10 years. The primary lesion is a rough erythematous papule with a white to yellow scale. Lesions are typically clustered at sites of chronic sun exposure.

Squamous cell carcinoma in situ
Also known as Bowens disease the commonest presentation of in situ SCC is with an erythematous scaling patch or elevated plaque arising on sun exposed skin in an elderly patient. Lesions may arise de novo or from pre-existing actinic keratosis.
Pathologically there is full thickness atypia of dermal keratinocytes over a broad zone. Nuclear pleomorphism, apoptosis and abnormal mitoses are all seen.

Invasive SCC
The commonest clinical presentation of SCC is with an erythematous keratotic papule or nodule on a background of sun exposure. Ulceration may occur and both exophytic and endophytic areas may be seen. Regional lymphadenopathy may be present.
Pathologically there is downward proliferation of malignant cells and invasion of the basement membrane. Poorly differentiated lesions may show perineural invasion and require immunohistochemistry with S100 to distinguish them from melanomas (which stain strongly positive with this marker).

Basal cell carcinoma
Nodular BCC Commonest variant (60%)
Raised translucent papule
Usually affect the face
Large nodular BCC’s are locally destructive
Superficial BCC Usually appears as superficial erythematous macule affecting the trunk
Younger age at presentation (mean 57)
May show areas of spontaneous regression
Horizontal growth pattern predominates
High recurrence rate (due to sub clinical lateral spread)
Morpheaform BCC Macroscopically resembles flat, slightly atrophic lesion or plaque without well defined borders
Tumour has sub clinical lateral spread which increases recurrence rates
Cystic BCC Often have clear or blue - grey appearance
Cystic degeneration may not be clinically obvious and tumour may resemble nodular BCC
Basosquamous carcinoma Atypical BCC
Basaloid histological BCC features with eosinophillic squamoid features of SCC
Biologically more aggressive and are more locally destructive
Rare lesion accounts for 1% of all non melanoma skin cancers
Metastatic disease may occur in 9-10% of cases and resemble an SCC

Keratoacanthoma
Dome shaped erythematous lesions that develop over a period of days and grow rapidly. They often contain a central pit of keratin. They then begin to necrose and slough off. They are generally benign lesions although some do view them as precursors of malignancy. They may be treated by curettage and cautery. If there is diagnostic doubt (they can mimic malignancy) then formal excision biopsy is warranted.

Pyogenic granuloma
These present as friable overgrowths of granulation at sites of minor trauma. They may be ulcerated and bleeding on contact is common. They may be treated with curretage and cautery, formal excision may be used if there is diagnostic doubt.

946
Q

A 70 year old lady presents with a painless neck lump. There is a mass noted beneath the sternocleidomastoid muscle. There is a long history and somewhat unkindly her husband remarked on her rather noticeable halitosis. What is the most likely explanation?

Branchial cyst

Branchial fistula

Pharyngeal pouch

Thyroglossal cyst

Dental caries

A

C-Usual history of regurgitation of undigested food or coughing at night. Associated with halitosis and throat infections.
Neck lumps

The table below gives characteristic exam question features for conditions causing neck lumps:

Reactive lymphadenopathy By far the most common cause of neck swellings. There may be a history of local infection or a generalised viral illness
Lymphoma Rubbery, painless lymphadenopathy
The phenomenon of pain whilst drinking alcohol is very uncommon
There may be associated night sweats and splenomegaly
Thyroid swelling May be hypo-, eu- or hyperthyroid symptomatically
Moves upwards on swallowing
Thyroglossal cyst More common in patients < 20 years old
Usually midline, between the isthmus of the thyroid and the hyoid bone
Moves upwards with protrusion of the tongue
May be painful if infected
Pharyngeal pouch More common in older men
Represents a posteromedial herniation between thyropharyngeus and cricopharyngeus muscles
Usually not seen, but if large then a midline lump in the neck that gurgles on palpation
Typical symptoms are dysphagia, regurgitation, aspiration and chronic cough
Cystic hygroma A congenital lymphatic lesion (lymphangioma) typically found in the neck, classically on the left side
Most are evident at birth, around 90% present before 2 years of age
Branchial cyst An oval, mobile cystic mass that develops between the sternocleidomastoid muscle and the pharynx
Develop due to failure of obliteration of the second branchial cleft in embryonic development
Usually present in early adulthood
Cervical rib More common in adult females
Around 10% develop thoracic outlet syndrome
Carotid aneurysm Pulsatile lateral neck mass which doesn’t move on swallowing

947
Q

A 1 year-old is brought to the Emergency Department with a history of failure to thrive. On examination, the child is small for age and has a large head. X-ray shows a cupped appearance of the epiphysis of the wrist. What is the most likely cause?

Osteoporosis

Ehlers Danlos

Marfans

Rickets

Non accidental injury

A

Rickets is the childhood form of osteomalacia. It is due to the failure of the osteoid to ossify due to vitamin D deficiency. Symptoms start about the age of one. The child is small for age and there is a history of failure to thrive. Bony deformities include bowing of the femur and tibia, a large head, deformity of the chest wall with thickening of the costochondral junction (rickettary rosary), and a transverse sulcus in the chest caused by the pull of the diaphragm (Harrison’s sulcus). X- Rays show widening and cupping of the epiphysis of the long bones, most readily apparent in the wrist.

Paediatric fractures

Paediatric fracture types
Type Injury pattern
Complete fracture Both sides of cortex are breached
Toddlers fracture Oblique tibial fracture in infants
Plastic deformity Stress on bone resulting in deformity without cortical disruption
Greenstick fracture Unilateral cortical breach only
Buckle fracture Incomplete cortical disruption resulting in periosteal haematoma only

Growth plate fractures
In paediatric practice fractures may also involve the growth plate and these injuries are classified according to the Salter- Harris system (given below):

Type Injury pattern
I Fracture through the physis only (x-ray often normal)
II Fracture through the physis and metaphysis
III Fracture through the physis and epiphyisis to include the joint
IV Fracture involving the physis, metaphysis and epiphysis
V Crush injury involving the physis (x-ray may resemble type I, and appear normal)

As a general rule it is safer to assume that growth plate tenderness is indicative of an underlying fracture even if the x-ray appears normal. Injuries of Types III, IV and V will usually require surgery. Type V injuries are often associated with disruption to growth.

Non accidental injury
Delayed presentation
Delay in attaining milestones
Lack of concordance between proposed and actual mechanism of injury
Multiple injuries
Injuries at sites not commonly exposed to trauma
Children on the at risk register

Pathological fractures
Genetic conditions, such as osteogenesis imperfecta, may cause pathological fractures.

Osteogenesis imperfecta
Defective osteoid formation due to congenital inability to produce adequate intercellular substances like osteoid, collagen and dentine.
Failure of maturation of collagen in all the connective tissues.
Radiology may show translucent bones, multiple fractures, particularly of the long bones, wormian bones (irregular patches of ossification) and a trefoil pelvis.

Subtypes
Type I The collagen is normal quality but insufficient quantity.
Type II- Poor collagen quantity and quality.
Type III- Collagen poorly formed. Normal quantity.
Type IV- Sufficient collagen quantity but poor quality.

Osteopetrosis
Bones become harder and more dense.
Autosomal recessive condition.
It is commonest in young adults.
Radiology reveals a lack of differentiation between the cortex and the medulla described as marble bone.

948
Q

A 34 year old man presents with a peptic ulcer. Which of the following is responsible for the release of gastric acid?

Chief cells

Parietal cells

Brunners Glands

G Cells

None of the above

A

Parietal cells are responsible for the release of gastric acid. Brunners glands are found in the duodenum.

Gastric secretions

A working knowledge of gastric secretions is important for surgery because peptic ulcers are common, surgeons frequently prescribe anti secretory drugs and because there are still patients around who will have undergone acid lowering procedures (Vagotomy) in the past.

Gastric acid
Is produced by the parietal cells in the stomach
pH of gastric acid is around 2 with acidity being maintained by the H+/K+ ATP ase pump. As part of the process bicarbonate ions will be secreted into the surrounding vessels.
Sodium and chloride ions are actively secreted from the parietal cell into the canaliculus. This sets up a negative potential across the membrane and as a result sodium and potassium ions diffuse across into the canaliculus.
Carbonic anhydrase forms carbonic acid which dissociates and the hydrogen ions formed by dissociation leave the cell via the H+/K+ antiporter pump. At the same time sodium ions are actively absorbed. This leaves hydrogen and chloride ions in the canaliculus these mix and are secreted into the lumen of the oxyntic gland.

Phases of gastric acid secretion
There are 3 phases of gastric secretion:

  1. Cephalic phase (smell / taste of food)
    30% acid produced
    Vagal cholinergic stimulation causing secretion of HCL and gastrin release from G cells
  2. Gastric phase (distension of stomach )
    60% acid produced
    Stomach distension/low H+/peptides causes Gastrin release
  3. Intestinal phase (food in duodenum)
    10% acid produced
    High acidity/distension/hypertonic solutions in the duodenum inhibits gastric acid secretion via enterogastrones (CCK, secretin) and neural reflexes.

Regulation of gastric acid production
Factors increasing production include:
Vagal nerve stimulation
Gastrin release
Histamine release (indirectly following gastrin release) from enterchromaffin like cells

Factors decreasing production include:
Somatostatin (inhibits histamine release)
Cholecystokinin
Secretin

Below is a brief summary of the major hormones involved in food digestion:

Source Stimulus Actions
Gastrin
G cells in antrum of the stomach
Distension of stomach, extrinsic nerves
Inhibited by: low antral pH, somatostatin
Increase HCL, pepsinogen and IF secretion, increases gastric motility, trophic effect on gastric mucosa
CCK
I cells in upper small intestine
Partially digested proteins and triglycerides
Increases secretion of enzyme-rich fluid from pancreas, contraction of gallbladder and relaxation of sphincter of Oddi, decreases gastric emptying, trophic effect on pancreatic acinar cells, induces satiety
Secretin
S cells in upper small intestine
Acidic chyme, fatty acids
Increases secretion of bicarbonate-rich fluid from pancreas and hepatic duct cells, decreases gastric acid secretion, trophic effect on pancreatic acinar cells
VIP
Small intestine, pancreas
Neural
Stimulates secretion by pancreas and intestines, inhibits acid and pepsinogen secretion
Somatostatin
D cells in the pancreas and stomach
Fat, bile salts and glucose in the intestinal lumen
Decreases acid and pepsin secretion, decreases gastrin secretion, decreases pancreatic enzyme secretion, decreases insulin and glucagon secretion
inhibits trophic effects of gastrin, stimulates gastric mucous production

949
Q

A 56 year old man suddenly develops severe back pain. His pain has a radicular pattern. On examination, he is unable to extend his great toe. Which of the spinal levels listed below is most likely to have been affected?

L5

L4

L3

L2

L1

A

Extensor hallucis longus is derived from L5 and loss of EHL function is a useful test to determine whether this level is involved.

Spinal cord

  • Located in a canal within the vertebral column that affords it structural support.
    Rostrally it continues to the medulla oblongata of the brain and caudally it tapers at a level corresponding to the L1-2 interspace (in the adult), a central structure, the filum terminale anchors the cord to the first coccygeal vertebra.
    The spinal cord is characterised by cervico-lumbar enlargements and these, broadly speaking, are the sites which correspond to the brachial and lumbar plexuses respectively.

There are some key points to note when considering the surgical anatomy of the spinal cord:

  • During foetal growth the spinal cord becomes shorter than the spinal canal, hence the adult site of cord termination at the L1-2 level.
  • Due to growth of the vertebral column the spine segmental levels may not always correspond to bony landmarks as they do in the cervical spine.
  • The spinal cord is incompletely divided into two symmetrical halves by a dorsal median sulcus and ventral median fissure. Grey matter surrounds a central canal that is continuous rostrally with the ventricular system of the CNS.
  • The grey matter is sub divided cytoarchitecturally into Rexeds laminae.
  • Afferent fibres entering through the dorsal roots usually terminate near their point of entry but may travel for varying distances in Lissauers tract. In this way they may establish synaptic connections over several levels
  • At the tip of the dorsal horn are afferents associated with nociceptive stimuli. The ventral horn contains neurones that innervate skeletal muscle.

The key point to remember when revising CNS anatomy is to keep a clinical perspective in mind. So it is worth classifying the ways in which the spinal cord may become injured. These include:

Trauma either direct or as a result of disc protrusion
Neoplasia either by direct invasion (rare) or as a result of pathological vertebral fracture
Inflammatory diseases such as Rheumatoid disease, or OA (formation of osteophytes compressing nerve roots etc.
Vascular either as a result of stroke (rare in cord) or as complication of aortic dissection
Infection historically diseases such as TB, epidural abscesses.

The anatomy of the cord will, to an extent dictate the clinical presentation. Some points/ conditions to remember:

Brown- Sequard syndrome-Hemisection of the cord producing ipsilateral loss of proprioception and upper motor neurone signs, plus contralateral loss of pain and temperature sensation. The explanation of this is that the fibres decussate at different levels.
Lesions below L1 will tend to present with lower motor neurone signs

950
Q

A 59 year old man is undergoing an extended right hemicolectomy for a carcinoma of the splenic flexure of the colon. The surgeons divide the middle colic vein close to its origin. Into which of the following structures does this vessel primarily drain?

Superior mesenteric vein

Portal vein

Inferior mesenteric vein

Inferior vena cava

Ileocolic vein

A

The middle colonic vein drains into the SMV, if avulsed during mobilisation then dramatic haemorrhage can occur and be difficult to control.
Transverse colon

The right colon undergoes a sharp turn at the level of the hepatic flexure to become the transverse colon.
At this point it also becomes intraperitoneal.
It is connected to the inferior border of the pancreas by the transverse mesocolon.
The greater omentum is attached to the superior aspect of the transverse colon from which it can easily be separated. The mesentery contains the middle colic artery and vein. The greater omentum remains attached to the transverse colon up to the splenic flexure. At this point the colon undergoes another sharp turn.

Relations
Superior Liver and gall-bladder, the greater curvature of the stomach, and the lower end of the spleen
Inferior Small intestine
Anterior Greater omentum
Posterior From right to left with the descending portion of the duodenum, the head of the pancreas, convolutions of the jejunum and ileum, spleen

951
Q

A 5 year old boy presents with recurrent episodes of sinusitis. The casualty staff are surprised to find his liver lying in the left upper quadrant of the abdomen. What is the underlying diagnosis?

Von Recklinghausen’s disease

Li Fraumeni syndrome

Pierre Robin syndrome

Kartagener’s syndrome

MEN I

A

This is a case of Kartagener’s syndrome. The primary problem is of immotile cilia syndrome. When associated with situs inversus Kartagener’s syndrome is diagnosed.

Genetics and surgical disease

Some of the more commonly occurring genetic conditions occurring in surgical patients are presented here.

Li-Fraumeni Syndrome
Autosomal dominant
Consists of germline mutations to p53 tumour suppressor gene
High incidence of malignancies particularly sarcomas and leukaemias
Diagnosed when:

*Individual develops sarcoma under 45 years
*First degree relative diagnosed with any cancer below age 45 years and another family member develops malignancy under 45 years or sarcoma at any age

BRCA 1 and 2
Carried on chromosome 17 (BRCA 1) and Chromosome 13 (BRCA 2)
Linked to developing breast cancer (60%) risk.
Associated risk of developing ovarian cancer (55% with BRCA 1 and 25% with BRCA 2).

Lynch Syndrome
Autosomal dominant
Develop colonic cancer and endometrial cancer at young age
80% of affected individuals will get colonic and/ or endometrial cancer
High risk individuals may be identified using the Amsterdam criteria

Amsterdam criteria
Three or more family members with a confirmed diagnosis of colorectal cancer, one of whom is a first degree (parent, child, sibling) relative of the other two.
Two successive affected generations.
One or more colon cancers diagnosed under age 50 years.
Familial adenomatous polyposis (FAP) has been excluded.

Gardners syndrome
Autosomal dominant familial colorectal polyposis
Multiple colonic polyps
Extra colonic diseases include: skull osteoma, thyroid cancer and epidermoid cysts
Desmoid tumours are seen in 15%
Mutation of APC gene located on chromosome 5
Due to colonic polyps most patients will undergo colectomy to reduce risk of colorectal cancer
Now considered a variant of familial adenomatous polyposis coli

952
Q

A 5 year old boy is playing in a tree when he falls and lands on his right forearm. He is brought to the emergency department by his parents. On examination he has bony tenderness and bruising. An X-ray is taken and shows unilateral cortical disruption and development of periosteal haematoma. What is the most likely diagnosis?

Buckle fracture

Greenstick fracture

Toddlers fracture

Complete fracture

None of the above

A

Greenstick fractures are common childhood injuries. Unilateral cortical disruption is the main radiological feature, since involvement of both cortices makes the injury a complete fracture. Buckle fractures will show periosteal haematoma formation only.

Paediatric fractures

Paediatric fracture types
Type Injury pattern
Complete fracture Both sides of cortex are breached
Toddlers fracture Oblique tibial fracture in infants
Plastic deformity Stress on bone resulting in deformity without cortical disruption
Greenstick fracture Unilateral cortical breach only
Buckle fracture Incomplete cortical disruption resulting in periosteal haematoma only

Growth plate fractures
In paediatric practice fractures may also involve the growth plate and these injuries are classified according to the Salter- Harris system (given below):

Type Injury pattern
I Fracture through the physis only (x-ray often normal)
II Fracture through the physis and metaphysis
III Fracture through the physis and epiphyisis to include the joint
IV Fracture involving the physis, metaphysis and epiphysis
V Crush injury involving the physis (x-ray may resemble type I, and appear normal)

As a general rule it is safer to assume that growth plate tenderness is indicative of an underlying fracture even if the x-ray appears normal. Injuries of Types III, IV and V will usually require surgery. Type V injuries are often associated with disruption to growth.

Non accidental injury
Delayed presentation
Delay in attaining milestones
Lack of concordance between proposed and actual mechanism of injury
Multiple injuries
Injuries at sites not commonly exposed to trauma
Children on the at risk register

Pathological fractures
Genetic conditions, such as osteogenesis imperfecta, may cause pathological fractures.

Osteogenesis imperfecta
Defective osteoid formation due to congenital inability to produce adequate intercellular substances like osteoid, collagen and dentine.
Failure of maturation of collagen in all the connective tissues.
Radiology may show translucent bones, multiple fractures, particularly of the long bones, wormian bones (irregular patches of ossification) and a trefoil pelvis.

Subtypes
Type I The collagen is normal quality but insufficient quantity.
Type II- Poor collagen quantity and quality.
Type III- Collagen poorly formed. Normal quantity.
Type IV- Sufficient collagen quantity but poor quality.

Osteopetrosis
Bones become harder and more dense.
Autosomal recessive condition.
It is commonest in young adults.
Radiology reveals a lack of differentiation between the cortex and the medulla described as marble bone.

953
Q

A 53 year old man presents with a nodule on his chin. He is concerned because it has grown extremely rapidly over the course of the preceding week. On examination, he has a swollen, red, dome shaped lesion with a central defect that contains a keratinous type material. What is the most likely cause?

Actinic keratosis

Bowens disease

Pyogenic granuloma

Keratoacanthoma

Squamous cell carcinoma

A

Keratoacanthomas are characterised by a rapid growth phase. This may mimic amelanotic melanoma (although such rapid growth is rare even in these lesions). The keratin core is the clue as to the true nature of the lesion.

Skin disorders in surgery- malignancy and related lesions

Non melanoma skin cancer (BCC and SCC) are some of the commonest types of human malignancy. Up to 80% of these are BCC’s with approximately 20% comprising SCC’s. The incidence of NMSC’s increases with age and whilst there is a female preponderance in those under 40 years of age, in latter life the sex incidence is roughly equal.
The vast majority of NMSC’s are related to UV light exposure. For SCC’s the major pattern is in chronic long term exposure. For BCC’s, the pattern of sporadic exposure with episodes of burning is more important. Organ transplant recipients have a markedly increased incidence of SCC, risk factors include length of immunosuppression, ethnic origin and associated sunlight exposure. Human papilloma virus DNA is found in the majority of transplant recipient SCC’s. In addition to this increased risk, transplant recipients are also more likely to develop locoregional recurrences following treatment.

Actinic keratosis and SCC
Actinic keratosis is viewed as a premalignant lesion because there are atypical keratinocytes present in the epidermis. In a person with 7 actinic keratosis the risks of subsequent SCC is of the order of 10% at 10 years. The primary lesion is a rough erythematous papule with a white to yellow scale. Lesions are typically clustered at sites of chronic sun exposure.

Squamous cell carcinoma in situ
Also known as Bowens disease the commonest presentation of in situ SCC is with an erythematous scaling patch or elevated plaque arising on sun exposed skin in an elderly patient. Lesions may arise de novo or from pre-existing actinic keratosis.
Pathologically there is full thickness atypia of dermal keratinocytes over a broad zone. Nuclear pleomorphism, apoptosis and abnormal mitoses are all seen.

Invasive SCC
The commonest clinical presentation of SCC is with an erythematous keratotic papule or nodule on a background of sun exposure. Ulceration may occur and both exophytic and endophytic areas may be seen. Regional lymphadenopathy may be present.
Pathologically there is downward proliferation of malignant cells and invasion of the basement membrane. Poorly differentiated lesions may show perineural invasion and require immunohistochemistry with S100 to distinguish them from melanomas (which stain strongly positive with this marker).

Basal cell carcinoma
Nodular BCC Commonest variant (60%)
Raised translucent papule
Usually affect the face
Large nodular BCC’s are locally destructive
Superficial BCC Usually appears as superficial erythematous macule affecting the trunk
Younger age at presentation (mean 57)
May show areas of spontaneous regression
Horizontal growth pattern predominates
High recurrence rate (due to sub clinical lateral spread)
Morpheaform BCC Macroscopically resembles flat, slightly atrophic lesion or plaque without well defined borders
Tumour has sub clinical lateral spread which increases recurrence rates
Cystic BCC Often have clear or blue - grey appearance
Cystic degeneration may not be clinically obvious and tumour may resemble nodular BCC
Basosquamous carcinoma Atypical BCC
Basaloid histological BCC features with eosinophillic squamoid features of SCC
Biologically more aggressive and are more locally destructive
Rare lesion accounts for 1% of all non melanoma skin cancers
Metastatic disease may occur in 9-10% of cases and resemble an SCC

Keratoacanthoma
Dome shaped erythematous lesions that develop over a period of days and grow rapidly. They often contain a central pit of keratin. They then begin to necrose and slough off. They are generally benign lesions although some do view them as precursors of malignancy. They may be treated by curettage and cautery. If there is diagnostic doubt (they can mimic malignancy) then formal excision biopsy is warranted.

Pyogenic granuloma
These present as friable overgrowths of granulation at sites of minor trauma. They may be ulcerated and bleeding on contact is common. They may be treated with curretage and cautery, formal excision may be used if there is diagnostic doubt.

954
Q

An 18 month old boy presents with recurrent urinary tract infections. As part of the diagnostic work-up he is noted to have abnormal renal function. An ultrasound scan is performed and shows bilateral hydronephrosis and hydroureter. What is the most likely underlying diagnosis?

Posterior urethral valves

Meatal stenosis

Hydronephrosis

Pelvico-ureteric junction obstruction

Benign prostatic hyperplasia

A

A posterior urethral valve is an obstructive, developmental uropathy that usually affects male infants (incidence 1 in 8000). Diagnostic features include bladder wall hypertrophy, hydronephrosis and bladder diverticula.

Urethral valves

Posterior urethral valves are the commonest cause of infravesical outflow obstruction in males. They may be diagnosed on ante natal ultrasonography. Because the bladder has to develop high emptying pressures in utero, the child may develop renal parenchymal damage. This translates to renal impairment noted in 70% of boys at presentation. Treatment is with bladder catheterisation. Endoscopic valvotomy is the definitive treatment of choice with cystoscopic and renal follow up.

955
Q

Which of these muscles is supplied by the musculocutaneous nerve?

Brachialis

Latissimus dorsi

Flexor carpi ulnaris

Teres minor

Triceps

A

Mnemonic

Muscles innervated by the musculocutaneous nerve BBC:

Biceps brachii
Brachialis
Coracobrachialis

Musculocutaneous nerve

Branch of lateral cord of brachial plexus

Path
It penetrates the coracobrachialis muscle
Passes obliquely between the biceps brachii and the brachialis to the lateral side of the arm
Above the elbow it pierces the deep fascia lateral to the tendon of the biceps brachii
Continues into the forearm as the lateral cutaneous nerve of the forearm

Innervates
Coracobrachialis
Biceps brachii
Brachialis

956
Q

A 38 year old sheep farmer presents to the clinic with a 3 month history of malaise and right upper quadrant pain. On examination, he is mildly jaundiced. His liver function tests demonstrate a mild elevation in bilirubin and transaminases, his full blood count shows an elevated eosinophil level. An abdominal x-ray is performed by the senior house officer and demonstrates a calcified lesion in the right upper quadrant of the abdomen. What is the most probably underlying diagnosis?

Mesenchymal hamartoma

Clonorchiasis

Fasciolopsiasis

Liver cell adenoma

Hydatid cyst

A

Hydatid disease is more common in those who work with sheep or dogs. Liver function tests may be abnormal and an eosinophilia is often present. Plain radiographs may reveal a calcified cyst wall. Fasciolopsiasis infection is confined to intestinal wall in most cases.

Benign liver lesions

Benign liver lesions
Haemangioma
Most common benign tumours of mesenchymal origin
Incidence in autopsy series is 8%
Cavernous haemangiomas may be enormous
Clinically they are reddish purple hypervascular lesions
Lesions are normally separated from normal liver by ring of fibrous tissue
On ultrasound they are typically hyperechoic
Liver cell adenoma
90% develop in women in their third to fifth decade
Linked to use of oral contraceptive pill
Lesions are usually solitary
They are usually sharply demarcated from normal liver although they usually lack a fibrous capsule
On ultrasound the appearances are of mixed echoity and heterogeneous texture. On CT most lesions are hypodense when imaged prior to administration of IV contrast agents
In patients with haemorrhage or symptoms removal of the adenoma may be required
Mesenchymal hamartomas Congential and benign, usually present in infants. May compress normal liver
Liver abscess
Biliary sepsis is a major predisposing factor
Structures drained by the portal venous system form the second largest source
Common symptoms include fever, right upper quadrant pain. Jaundice may be seen in 50%
Ultrasound will usually show a fluid filled cavity, hyperechoic walls may be seen in chronic abscesses
Amoebic abscess
Liver abscess is the most common extra intestinal manifestation of amoebiasis
Between 75 and 90% lesions occur in the right lobe
Presenting complaints typically include fever and right upper quadrant pain
Ultrasonography will usually show a fluid filled structure with poorly defined boundaries
Aspiration yield sterile odourless fluid which has an anchovy paste consistency
Treatment is with metronidazole
Hyatid cysts
Seen in cases of Echinococcus infection
Typically an intense fibrotic reaction occurs around sites of infection
The cyst has no epithelial lining
Cysts are commonly unilocular and may grow to 20cm in size. The cyst wall is thick and has an external laminated hilar membrane and an internal enucleated germinal layer
Typically presents with malaise and right upper quadrant pain. Secondary bacterial infection occurs in 10%.
Liver function tests are usually abnormal and eosinophilia is present in 33% cases
Ultrasound may show septa and hyatid sand or daughter cysts.
Percutaneous aspiration was previously contra indicated, it is now incorporated into some treatment regimens
Treatment is by sterilisation of the cyst with mebendazole and may be followed by surgical resection. Hypertonic swabs are packed around the cysts during surgery
Polycystic liver disease
Usually occurs in association with polycystic kidney disease
Autosomal dominant disorder
Symptoms may occur as a result of capsular stretch
Cystadenoma
Rare lesions with malignant potential
Usually solitary multiloculated lesions
Liver function tests usually normal
Ultrasonography typically shows a large anechoic, fluid filled area with irregular margins. Internal echos may result from septa
Surgical resection is indicated in all cases

957
Q

At which of the following levels does the inferior vena cava exit the abdominal cavity?

T6

T7

T10

T8

T12

A

T8.

Levels

Transpyloric plane
Level of the body of L1

Pylorus stomach
Left kidney hilum (L1- left one!)
Fundus of the gallbladder
Neck of pancreas
Duodenojejunal flexure
Superior mesenteric artery
Portal vein
Left and right colic flexure
Root of the transverse mesocolon
2nd part of the duodenum
Upper part of conus medullaris
Spleen

Can be identified by asking the supine patient to sit up without using their arms. The plane is located where the lateral border of the rectus muscle crosses the costal margin.

Anatomical planes
Subcostal plane Lowest margin of 10th costal cartilage
Intercristal plane Level of body L4 (highest point of iliac crest)
Intertubercular plane Level of body L5

Common level landmarks
Inferior mesenteric artery L3
Bifurcation of aorta into common iliac arteries L4
Formation of IVC L5 (union of common iliac veins)
Diaphragm apertures
Vena cava T8
Oesophagus T10
Aortic hiatus T12

958
Q

A 53 year old man is admitted to the vascular ward for a carotid endarterectomy. His CT head report confirms a left parietal lobe infarct. What type of visual field defect might be noted?

Right inferior quadranopia

Right superior quadranopia

Right homonymous hemianopia

Left superior quadranopia

Lower bitemporal hemianopia

A

Superior quadranopia = temporal lobe lesion
Inferior quadranopia = parietal lobe lesion
Parietal lesions cause a contralateral inferior quadranopia.

Visual field defects

left homonymous hemianopia means visual field defect to the left, i.e. Lesion of right optic tract
homonymous quadrantanopias: PITS (Parietal-Inferior, Temporal-Superior)
incongruous defects = optic tract lesion; congruous defects = optic radiation lesion or occipital cortex

Homonymous hemianopia
Incongruous defects: lesion of optic tract
Congruous defects: lesion of optic radiation or occipital cortex
Macula sparing: lesion of occipital cortex

Homonymous quadrantanopias
Superior: lesion of temporal lobe
Inferior: lesion of parietal lobe
Mnemonic = PITS (Parietal-Inferior, Temporal-Superior)

Bitemporal hemianopia
Lesion of optic chiasm
Upper quadrant defect > lower quadrant defect = inferior chiasmal compression, commonly a pituitary tumour
Lower quadrant defect > upper quadrant defect = superior chiasmal compression, commonly a craniopharyngioma

959
Q

Following a carotid endarterectomy a man notices that he has a weakness of his tongue. Damage to which of the following nerves is the most likely explanation for this process?

Hypoglossal

Accessory

Ansa cervicalis

Vagus

Cervical plexus

A

The hypoglossal nerve innervates the tongue and is one of the structures more commonly at risk in carotid surgery.

Internal carotid artery

The internal carotid artery is formed from the common carotid opposite the upper border of the thyroid cartilage. It extends superiorly to enter the skull via the carotid canal. From the carotid canal it then passes through the cavernous sinus, above which it divides into the anterior and middle cerebral arteries.

Relations in the neck
Posterior
Longus capitis
Pre-vertebral fascia
Sympathetic chain
Superior laryngeal nerve
Medially
External carotid (near origin)
Wall of pharynx
Ascending pharyngeal artery
Laterally
Internal jugular vein (moves posteriorly at entrance to skull)
Vagus nerve (most posterolaterally)
Anteriorly
Sternocleidomastoid
Lingual and facial veins
Hypoglossal nerve

Relations in the carotid canal
Internal carotid plexus
Cochlea and middle ear cavity
Trigeminal ganglion (superiorly)
Leaves canal lies above the foramen lacerum

Path and relations in the cranial cavity
The artery bends sharply forwards in the cavernous sinus, the aducens nerve lies close to its inferolateral aspect. The oculomotor, trochlear, opthalmic and, usually, the maxillary nerves lie in the lateral wall of the sinus. Near the superior orbital fissure it turns posteriorly and passes postero-medially to pierce the roof of the cavernous sinus inferior to the optic nerve. It then passes between the optic and oculomotor nerves to terminate below the anterior perforated substance by dividing into the anterior and middle cerebral arteries.

Branches
Anterior and middle cerebral artery
Ophthalmic artery
Posterior communicating artery
Anterior choroid artery
Meningeal arteries
Hypophyseal arteries

960
Q

A 72 year old man presents with haemoptysis and undergoes a bronchoscopy. The carina is noted to be widened. At which level does the trachea bifurcate?

T3

T5

T7

T2

T8

A

The trachea bifurcates at the level of the angle of Louis which is the intervertebral space between the fourth and fifth thoracic vertebra.

Trachea

Trachea
Location C6 vertebra to the upper border of T5 vertebra (bifurcation)
Arterial and venous supply Inferior thyroid arteries and the thyroid venous plexus.
Nerve Branches of vagus, sympathetic and the recurrent nerves

Relations in the neck
Anterior(Superior to inferior)
Isthmus of the thyroid gland
Inferior thyroid veins
Arteria thyroidea ima (when that vessel exists)
Sternothyroid
Sternohyoid
Cervical fascia
Anastomosing branches between the anterior jugular veins
Posterior Oesophagus.
Laterally
Common carotid arteries
Right and left lobes of the thyroid gland
Inferior thyroid arteries
Recurrent laryngeal nerves

Relations in the thorax

Anterior
Manubrium, the remains of the thymus, the aortic arch, left common carotid arteries, and the deep cardiac plexus

Lateral
In the superior mediastinum, on the right side is the pleura and right vagus; on its left side are the left recurrent nerve, the aortic arch, and the left common carotid and subclavian arteries.

961
Q

Which of the following statements relating to gas gangrene is untrue?

There is necrosis with putrefaction

The causative pathogens may be detected on normal perineal skin

Treatment with low dose penicillin is indicated

Hyperbaric oxygen may be beneficial

Clostridium perfringens is a recognised cause

A

Rapid surgery and high dose antibiotics are indicated in the treatment of gas gangrene.

Meleney’s Gangrene and Necrotising Fasciitis

Necrotising fasciitis
Advancing soft tissue infection associated with fascial necrosis
Uncommon, but can be fatal
In many cases there is underlying background immunosuppression e.g. Diabetes
Caused by polymicrobial flora (aerobic and anaerobic) and MRSA is seen increasingly in cases of necrotising fasciitis
Streptococcus is the commonest organism in isolated pathogen infection (15%)

Meleneys gangrene
Meleneys is a similar principle but the infection is more superficially sited than necrotising fasciitis and often confined to the trunk

Fournier gangrene
Necrotising fasciitis affecting the perineum
Polymicrobial with E-coli and Bacteroides acting in synergy

Clinical features
Fever
Pain
Cellulitis
Oedema
Induration
Numbness

Late findings

Purple/black skin discolouration
Blistering
Haemorrhagic bullae
Crepitus
Dirty Dishwater fluid discharge
Septic shock

Diagnosis is mainly clinical

Management
Radical surgical debridement forms the cornerstone of management
Sterile dressing is used to dress the wound
Reconstructive surgery is considered once the infection is completely treated

Reference

Hasham S, Matteucci P, Stanley PR, Hart NB. Necrotising fasciitis. BMJ 2005;330:830-833.

962
Q

A 56 year old man presents with symptoms of nasal pain, anosmia and rhinorrhea. He has been well until recently and has worked as a wood carver for many years. What is the most likely diagnosis?

Ethmoid sinus cancer

Ameloblastoma

Maxillary sinus cancer

Fibrous dysplasia

Haemangiopericytoma

A

Paranasal sinus cancer is strongly associated with wood work. Most cases require an occupational exposure of greater than 10 years and are adenocarcinomas on histology.
Most cases are ethmoidal in origin
References:
Hadfield E. Ann R Coll Surg Engl. 1970 June; 46(6): 301319
Binazzi A et al. Occupational exposure and sinonasal cancer: a systematic review and meta-analysis. BMC Cancer. 2015; 15: 49. This is a comprehensive review of the topic. In particular, it summarises that it is not maxillary sinus cancer in the woodworking subgroup.

Diseases of nose and sinuses

Benign Tumours
Simple papillomas may be an incidental finding or present with obstructive symptoms. Excision under general anaesthesia is sufficient management.
Transitional cell papillomas may be more extensive and produce obstructive symptoms. Erosion of local structures is a recognised complication. These lesions may rarely undergo malignant transformation and therefore careful and complete excision is required, some cases may require partial or total maxillectomy.
Pleomorphic adenomas of the maxillary sinuses are reported but are extremely rare, their symptoms typically include nasal obstruction and pain if the sinus is obstructed. Treatment is by complete surgical excision, the diagnosis is not infrequently made post operatively.
Benign osteomas may develop in the paranasal sinuses, the frontal sinus is the most frequent location of such lesions. Symptoms include; pain, rhinorrhoea and anosmia. Most osteomas may be observed if asymptomatic, sphenoid osteomas should be resected soon after diagnosis as enlargement may compromise visual fields. Many sinus osteomas can now be resected endoscopically, complete surgical resection is required.
Nasal polyps are benign lesions of the ethmoid sinus mucosa. Many patients may also have asthma, cystic fibrosis and a sensitivity to aspirin. Symptoms include watery rhinorrhoea, infection and anosmia. The polyps are usually a semi transparent grey mass. They are rare in childhood. Treatment is either with systemic steroids or surgical resection. The latter should be combined with antral washout. Low dose, nasal, steroid drops may reduce the risk of recurrence.

Malignant disease
Malignancies encountered in the nose and paranasal sinuses include; adenoid cystic carcinoma, squamous cell carcinoma and adenocarcinoma.
Adenocarcinoma of the paranasal sinuses and nasopharynx is strongly linked to exposure to hard wood dust (after >10 years exposure).
Adenoid cystic carcinoma usually originate in the smaller salivary glands.
The majority of cancers (50%) arise from the lateral nasal wall, a smaller number (33%) arise from the maxillary antrum, ethmoid and sphenoid cancers comprise only 7%.
Signs of malignancy on clinical examination include loose teeth, cranial nerve palsies and lymphadenopathy.
Nasopharyngeal cancers are most common in individuals presenting from China and Asia and are linked to viral infection with Epstein Barr Virus. Radiotherapy and chemotherapy are the most commonly used modalities.

Maxillary sinusitis
Common symptoms include post nasal discharge, pain, headache and toothache.
Imaging may show a fluid level in the antrum.
Common organisms include Haemophilus influenzae or Streptococcus pneumoniae.
Treatment with antral lavage may facilitate diagnosis and relieve symptoms. Antimicrobial therapy has to be continued for long periods. Antrostomy may be needed.

Frontoethmoidal sinusitis
Usually presents with frontal headache, nasal obstruction and altered sense of smell.
Inflammation may progress to involve periorbital tissues. Ocular symptoms may occur and secondary CNS involvement brought about by infection entering via emissary veins.
CT scanning is the imaging modality of choice. Early cases may be managed with antibiotics. More severe cases usually require surgical drainage.

963
Q

The common peroneal nerve, or its branches, supply the following muscles except:

Peroneus longus

Tibialis anterior

Extensor hallucis longus

Flexor digitorum brevis

Extensor digitorum longus

A

Flexor digitorum is supplied by the tibial nerve.

Common peroneal nerve

Derived from the dorsal divisions of the sacral plexus (L4, L5, S1 and S2).

This nerve supplies the skin and fascia of the anterolateral surface of the leg and the dorsum of the foot. It also innervates the muscles of the anterior and peroneal compartments of the leg, extensor digitorum brevis as well as the knee, ankle and foot joints.

It is laterally placed within the sciatic nerve. From the bifurcation of the sciatic nerve it passes inferolaterally in the lateral and proximal part of the popliteal fossa, under the cover of biceps femoris and its tendon. To reach the posterior aspect of the fibular head. It ends by dividing into the deep and superficial peroneal nerves at the point where it winds around the lateral surface of the neck of the fibula in the body of peroneus longus, approximately 2cm distal to the apex of the head of the fibula. It is palpable posterior to the head of the fibula.

Branches
In the thigh Nerve to the short head of biceps
Articular branch (knee)
In the popliteal fossa Lateral cutaneous nerve of the calf
Neck of fibula Superficial and deep peroneal nerves

964
Q

A 24 year old male was admitted with bloody diarrhea, cramping abdominal pain and weight loss. Colonoscopy revealed a friable, diffusely red mucosa involving the rectum and sigmoid colon. The mucosa was normal proximal to this. The disease progressed with time to involve most of the entire colon, but not the ileum. Many years later, a colonic biopsy shows high grade epithelial dysplasia. What is the most likely initial diagnosis?

Colonic tuberculosis

Collagenous colitis

Ulcerative colitis

Crohns disease

Ischaemic colitis

A

Ulcerative colitis spreads in a progressive distal to proximal manner. Over time a dysplastic transformation is recognised. Such endoscopic findings mandate a minimum of close endoscopic surveillance and if they occur in association with a colonic mass then usually a panproctocolectomy.

Ulcerative colitis

Ulcerative colitis is a form of inflammatory bowel disease. Inflammation always starts at rectum, does not spread beyond ileocaecal valve (although backwash ileitis may occur) and is continuous. The peak incidence of ulcerative colitis is in people aged 15-25 years and in those aged 55-65 years. It is less common in smokers.

The initial presentation is usually following insidious and intermittent symptoms. Features include:
bloody diarrhoea
urgency
tenesmus
abdominal pain, particularly in the left lower quadrant
extra-intestinal features (see below)

Questions regarding the ‘extra-intestinal’ features of inflammatory bowel disease are common. Extra-intestinal features include sclerosing cholangitis, iritis and ankylosing spondylitis.

Common to both Crohn’s disease (CD) and Ulcerative colitis (UC) Notes
Related to disease activity Arthritis: pauciarticular, asymmetric
Erythema nodosum
Episcleritis
Osteoporosis Arthritis is the most common extra-intestinal feature in both CD and UC
Episcleritis is more common in Crohns disease
Unrelated to disease activity Arthritis: polyarticular, symmetric
Uveitis
Pyoderma gangrenosum
Clubbing
Primary sclerosing cholangitis Primary sclerosing cholangitis is much more common in UC
Uveitis is more common in UC

Pathology
Red, raw mucosa, bleeds easily
No inflammation beyond submucosa (unless fulminant disease)
Widespread superficial ulceration with preservation of adjacent mucosa which has the appearance of polyps (‘pseudopolyps’)
Inflammatory cell infiltrate in lamina propria
Neutrophils migrate through the walls of glands to form crypt abscesses
Depletion of goblet cells and mucin from gland epithelium
Granulomas are infrequent

Barium enema
Loss of haustrations
Superficial ulceration, ‘pseudopolyps’
Long standing disease: colon is narrow and short -‘drainpipe colon’

Endoscopy
Superficial inflammation of the colonic and rectal mucosa
Continuous disease from rectum proximally
Superficial ulceration, mucosal islands, loss of vascular definition and continuous ulceration pattern.

Management
Patients with long term disease are at increased risk of development of malignancy
Acute exacerbations are generally managed with steroids, in chronic patients agents such as azathioprine and infliximab may be used
Individuals with medically unresponsive disease usually require surgery- in the acute phase a sub total colectomy and end ileostomy. In the longer term a proctectomy will be required. An ileoanal pouch is an option for selected patients

References
Ford A et al. Ulcerative colitis. BMJ 2013 (346):29-34.

965
Q

A 50-year-old female with a history of rheumatoid presents with a suspected septic knee joint. A diagnostic aspiration is performed and sent to microbiology. Which of the organisms below is most likely to be responsible?

Staphylococcus aureus

Staphylococcus epidermidis

Escherichia coli

Neisseria gonorrhoeae

Streptococcus pneumoniae

A

Septic arthritis - most common organism: Staphylococcus aureus

Septic arthritis

Overview
Most common organism overall is Staphylococcus aureus
In young adults who are sexually active Neisseria gonorrhoeae should also be considered

Management
Synovial fluid should be obtained before starting treatment
Intravenous antibiotics which cover Gram-positive cocci are indicated. The BNF currently recommends flucloxacillin or clindamycin if penicillin allergic
Antibiotic treatment is normally be given for several weeks (BNF states 6-12 weeks)
Needle aspiration should be used to decompress the joint
Arthroscopic lavage may be required

966
Q

A 73 year old man undergoes a laparotomy for mesenteric infarction. An extensive small bowel resection is undertaken. A planned re-look laparotomy is scheduled to occur in 24 hours time. What is the most appropriate closure strategy in this situation?

Mass closure of the abdomen obeying Jenkins rule using 2 PDS

Application of a Bogota bag

Mass closure of the abdomen obeying Jenkins rule using 2/0 PDS

Mass closure of the abdomen obeying Jenkins rule using 2 nylon

Mass closure of the abdomen obeying Jenkins rule using 2/0 nylon

A

Where there are definite plans for early re-look surgery and it is desirable to visualize the viscera (as it would be in this case), application of a Bogota bag will provide the best closure option.

Abdominal wound dehiscence

  • This is a significant problem facing all surgeons who undertake abdominal surgery on a regular basis. Traditionally, it is said to occur when all layers of an abdominal mass closure fail and the viscera protrude externally (associated with 30% mortality).
    It can be subdivided into superficial, in which the skin wound alone fails and complete, implying failure of all layers.

Factors which increase the risk are:
* Malnutrition
* Vitamin deficiencies
* Jaundice
* Steroid use
* Major wound contamination (e.g. faecal peritonitis)
* Poor surgical technique (Mass closure technique is the preferred method-Jenkins Rule)

When sudden full dehiscence occurs the management is as follows:
* Analgesia
* Intravenous fluids
* Intravenous broad spectrum antibiotics
* Coverage of the wound with saline impregnated gauze (on the ward)
* Arrangements made for a return to theatre

Surgical strategy
Correct the underlying cause (e.g. TPN or NG feed if malnourished)
Determine the most appropriate strategy for managing the wound

Options
Resuturing of the wound This may be an option if the wound edges are healthy and there is enough tissue for sufficient coverage. Deep tension sutures are traditionally used for this purpose.
Application of a wound manager This is a clear dressing with removable front. Particularly suitable when some granulation tissue is present over the viscera or where there is a high output bowel fistula present in the dehisced wound.
Application of a ‘Bogota bag’ This is a clear plastic bag that is cut and sutured to the wound edges and is only a temporary measure to be adopted when the wound cannot be closed and will necessitate a return to theatre for definitive management.
Application of a VAC dressing system These can be safely used BUT ONLY if the correct layer is interposed between the suction device and the bowel. Failure to adhere to this absolute rule will almost invariably result in the development of multiple bowel fistulae and create an extremely difficult management problem.

967
Q

A 34 year old man develops mild hypothermia. Which of the hormones listed below will be released in reduced quantities?

Thyroid stimulating hormone

Corticosterone

Insulin

Glucagon

Aldosterone

A

In mild hypothermia TSH release is increased as is corticosterone, glucagon and aldosterone. The effects on electrolytes are minimal. There is a perceived loss of plasma volume which may reflect microvascular changes within capillary beds.

Hypothermia

Core body temperature below 35oC. Severe hypothermia is present when the core temperature is below 28oC. Hypothermia is associated with a reduction in both respiratory and cardiac activity. Many cases are complicated by the presence of alcohol intoxication.

Stages of hypothermia
Stage Features Classification Temperature oC
1 Awake and shivering Mild 32-35
2 Drowsy and not shivering Moderate 28-32
3 Unconcious and not shivering Severe 20-28
4 No vital signs Profound Less than 20

Management
An organised cardiac rhythm may be converted to fibrillation if CPR is attempted inappropriately so ECG should be analysed with care. The rewarming technique used depends upon the degree of hypothermia and the physiological state of the patient. Mild hypothermia may respond to external rewarming devices. Severe hypothermia may require active core rewarming techniques such as peritoneal lavage, haemodialysis or cardiac bypass. Patients who develop cardiac arrhythmias who are severely hypothermic may respond to bretylium toslyte (sadly no longer available in most centres), but do not generally respond to standard therapies or DC cardioversion.

968
Q

A 34 year old lady suffers from hyperparathyroidism. The right inferior parathyroid is identified as having an adenoma and is scheduled for resection. From which of the following embryological structures is it derived?

Second pharyngeal pouch

Third pharyngeal pouch

Fourth pharyngeal pouch

First pharyngeal pouch

None of the above

A

The inferior parathyroid is a derivative of the third pharyngeal pouch. The superior parathyroid originates from the fourth pharyngeal pouch.

Parathyroid glands- anatomy

Four parathyroid glands
Located posterior to the thyroid gland
They lie within the pretracheal fascia

Embryology
The parathyroids develop from the extremities of the third and fourth pharyngeal pouches. The parathyroids derived from the fourth pharyngeal pouch are located more superiorly and are associated with the thyroid gland. Those derived from the third pharyngeal pouch lie more inferiorly and may become associated with the thymus.

Blood supply
The blood supply to the parathyroid glands is derived from the inferior and superior thyroid arteries[1]. There is a rich anastomosis between the two vessels. Venous drainage is into the thyroid veins.

Relations
Laterally Common carotid
Medially Recurrent laryngeal nerve, trachea
Anterior Thyroid
Posterior Pretracheal fascia

References
1.Nobori, M., et al., Blood supply of the parathyroid gland from the superior thyroid artery. Surgery, 1994. 115(4): p. 417-23.

969
Q

A 62 year old man is admitted with dull lower back pain and abdominal discomfort. On examination, he is hypertensive and a lower abdominal fullness is elicited on examination. An abdominal ultrasound demonstrates hydronephrosis and intravenous urography demonstrated medially displaced ureters. A CT scan shows a periaortic mass. What is the most likely cause?

Abdominal liposarcoma

Metastatic pancreatic cancer

Abdominal aortic aneurysm

Retroperitoneal fibrosis

Colonic cancer

A

Retroperitoneal fibrosis is an uncommon condition and its aetiology is poorly understood. In a significant proportion the ureters are displaced medially. In most retroperitoneal malignancies they are displaced laterally. Hypertension is another common finding. A CT scan will often show a para-aortic mass

Retroperitoneal fibrosis

Retroperitoneal fibrosis or Ormond’s disease is a disease featuring the proliferation of fibrous tissue in the retroperitoneum, the compartment of the body containing the kidneys, aorta, renal tract, and various other structures. It may present with lower back pain, kidney failure, hypertension, deep vein thrombosis, and other obstructive symptoms. It is named after John Kelso Ormond, who rediscovered the condition in 1948
Imaged with CT scanning of the abdomen
Biopsy is usually performed to exclude other causes
Treatment includes steroids and occasionally surgery, ureteric stent insertion is commonly required

970
Q

A 32 year old woman presents with an episode of haemoptysis and is found to have metastatic tumour present within the parenchyma of the lungs. This is biopsied and subsequent histology shows clear cells. What is the most likely primary site?

Kidney

Breast

Liver

Adrenal

Bone

A

Clear cell tumours are a sub type of renal cell cancer it is associated with specific genetic changes localised to chromosome 3.

Renal lesions

Lesion Disease specific features Treatment
Renal cell carcinoma
Most present with haematuria (50%)
Common renal tumour (85% cases)
Paraneoplastic features include hypertension and polycythaemia
Most commonly has haematogenous mestastasis
Usually radical or partial nephrectomy
Nephroblastoma
Rare childhood tumour
It accounts for 80% of all genitourinary malignancies in those under the age of 15 years
Up to 90% will have a mass
50% will be hypertensive
Diagnostic work up includes ultrasound and CT scanning
Surgical resection combined with chemotherapy (usually vincristine, actinomycin D and doxorubicin)
Neuroblastoma
Most common extracranial tumour of childhood
80% occur in those under 4 years of age
Tumour of neural crest origin (up to 50% occur in the adrenal gland)
The tumour is usually calcified and may be diagnosed using MIBG scanning
Staging is with CT
Surgical resection, radiotherapy and chemotherapy
Transitional cell carcinoma
Accounts for 90% of lower urinary tract tumours, but only 10% of renal tumours
Males affected 3x more than females
Occupational exposure to industrial dyes and rubber chemicals may increase risk
Up to 80% present with painless haematuria
Diagnosis and staging is with CT IVU
Radical nephroureterectomy
Angiomyolipoma
80% of these hamartoma type lesions occur sporadically, the remainder are seen in those with tuberous sclerosis
Tumour is composed of blood vessels, smooth muscle and fat
Massive bleeding may occur in 10% of cases
50% of patients with lesions >4cm will have symptoms and will require surgical resection

971
Q

Which of the following structures is not transmitted by the jugular foramen?

Hypoglossal nerve

Accessory nerve

Internal jugular vein

Inferior petrosal sinus

Vagus nerve

A

Contents of the jugular foramen:

Anterior: inferior petrosal sinus
Intermediate: glossopharyngeal, vagus, and accessory nerves
Posterior: sigmoid sinus (becoming the internal jugular vein) and some meningeal branches from the occipital and ascending pharyngeal arteries
The jugular foramen may be divided into three compartments:
Anterior compartment transmits the inferior petrosal sinus
Middle compartment transmits cranial nerves IX, X and XI
Posterior compartment transmits the sigmoid sinus

Foramina of the base of the skull

Foramen Location Contents
Foramen ovale Sphenoid bone Otic ganglion
V3 (Mandibular nerve:3rd branch of
trigeminal)
Accessory meningeal artery
Lesser petrosal nerve
Emissary veins
Foramen spinosum Sphenoid bone Middle meningeal artery
Meningeal branch of the Mandibular nerve
Foramen rotundum Sphenoid bone Maxillary nerve (V2)
Foramen lacerum/ carotid canal Located between the sphenoid, the apex of the petrous temporal and the basilar part of the occipital Base of the medial pterygoid plate.
Internal carotid artery*
Nerve and artery of the pterygoid canal
Jugular foramen Temporal bone Anterior: inferior petrosal sinus
Intermediate: glossopharyngeal, vagus, and accessory nerves.
Posterior: sigmoid sinus (becoming the internal jugular vein) and some meningeal branches from the occipital and ascending pharyngeal arteries.
Foramen magnum Occipital bone Anterior and posterior spinal arteries
Vertebral arteries
Medulla oblongata
Stylomastoid foramen Temporal bone Stylomastoid artery
Facial nerve
Superior orbital fissure Sphenoid bone Oculomotor nerve (III)
Recurrent meningeal artery
Trochlear nerve (IV)
Lacrimal, frontal and nasociliary branches of opthalmic nerve (V1)
Abducent nerve (VI)
Superior ophthalmic vein

*= In life the foramen lacerum is occluded by a cartilagenous plug. The ICA initially passes into the carotid canal which ascends superomedially to enter the cranial cavity through the foramen lacerum.

972
Q

A 66 year old man is admitted with severe angina. There is a lesion of the proximal left anterior descending coronary artery. Which of the following would be the most suitable conduit for bypass?

Long saphenous vein

Short saphenous vein

Cephalic vein

Internal mammary artery

Thoraco-acromial artery

A

The internal mammary artery is an excellent conduit for coronary artery bypass. It has better long term patency rates than venous grafts. The thoraco-acromial artery is seldom used.

Cardiopulmonary bypass

Indications for surgery
Left main stem stenosis or equivalent (proximal LAD and proximal circumflex)
Triple vessel disease
Diffuse disease unsuitable for PCI

The guidelines state that CABG is the preferred treatment in high-risk patients with severe ventricular dysfunction or diabetes mellitus.

Technique
General anaesthesia
Central and arterial lines
Midline sternotomy or left sub mammary incision
Aortic root and pericardium dissected
Heart inspected

Bypass grafting may be performed using a cardiopulmonary bypass circuit with cardiac arrest or using a number of novel ‘off pump’ techniques.

Procedure cardiopulmonary bypass
Aortic root cannulated
Right atrial cannula
Circuit primed and patient fully heparinised (30,000 Units unfractionated heparin) as the circuit is highly thrombogenic
Flow established through circuit
Aortic cross clamp applied
Cardioplegia solution instilled into the aortic root below cross clamp
Heart now asystolic and ready for surgery

Off pump techniques are evolving on a constant basis and details are beyond the scope of the MRCS.

Conduits for bypass
> Internal mammary artery is best. Use of both is associated with increased risk of sternal wound dehiscence. However, many surgeons will use both especially for redo surgery.
> Radial artery harvested from forearm. Ensure ulnar collateral working first!
> Reversed long saphenous vein grafts
Typically anastamosed using 7/0-8/0 prolene sutures (distally) and 6/0 prolene for top end.

Once flow established
Anticoagulation reversed using protamine
Patient is taken off bypass
Inotropes given if needed
Sternum closed using sternal closure device or stainless steel wire

Complications
Post perfusion syndrome: transient cognitive impairment
Non union of the sternum; due to loss of the internal thoracic artery
Myocardial infarction
Late graft stenosis
Acute renal failure
Stroke
Gastrointestinal

Perioperative risk is quantified using the Parsonnet and Euroscores and unit outcomes are audited using this data.

Reference
Eagle KA, Guyton RA, Davidoff R, et al: ACC/AHA 2004 guideline update for coronary artery bypass graft surgery: A report of the American College of Cardiology/American Heart Association Task Force on Practice Guidelines (Committee to Revise the 1999 Guidelines for Coronary Artery Bypass Graft Surgery). Circulation 2004; 110

973
Q

A pregnant woman suddenly develops bilateral leg swelling. Her mother and aunt were troubled by the same problem. What is the most likely underlying abnormality?

Anti endomysial antibodies

Anti nuclear antibodies

Anti cardiolipin antibodies

Anti thyroid antibodies

Anti mitochondrial antibodies

A

Anti phospholipid syndrome= following antibodies
Lupus anticoagulant
Anti-cardiolipin
Anti-β2-glycoprotein
Antiphospholipid syndrome, is an autoimmune, hypercoagulable state caused by antiphospholipid antibodies. APS provokes blood clots (thrombosis) in both arteries and veins as well as pregnancy-related complications such as miscarriage, stillbirth, preterm delivery, or severe preeclampsia.

The diagnostic criteria requires one clinical event, i.e. thrombosis or pregnancy complication, and two positive blood tests spaced at least 3 months apart. These antibodies are: lupus anticoagulant, anti-cardiolipin and anti-β2-glycoprotein.

Antiphospholipid syndrome can be primary or secondary. Primary antiphospholipid syndrome occurs in the absence of any other related disease. Secondary antiphospholipid syndrome occurs with other autoimmune diseases, such as systemic lupus erythematosus (SLE). In rare cases, APS leads to rapid organ failure due to generalised thrombosis; this is termed catastrophic antiphospholipid syndrome (CAPS) and is associated with a high risk of death.

Antiphospholipid syndrome often requires treatment with anticoagulant medication such as heparin to reduce the risk of further episodes of thrombosis and improve the prognosis of pregnancy. Warfarin is not used during pregnancy because it can cross the placenta, unlike heparin, and is teratogenic.

Hypercoagulability

Type of thrombophilia Features
Antithrombin deficiency Antithrombin inactivates thrombin and factor XII a, XIa, IXa and Xa
Rare defect, inherited in autosomal dominant fashion
10x increase in risk of thrombotic events
Heparin may be ineffective because it works via antithrombin
Protein C and S deficiency These are natural anticoagulants (vitamin K dependent synthesis)
Protein C produced by liver
Protein S produced by liver, megakaryocytes, Leydig cells and endothelial cells
Protein C and S bind to form activated complex which binds to factor V
Deficiency accounts for up to 5% of thrombotic episodes
Factor V Leiden Resistance to anticoagulant effect of activated protein C
May account for up to 20% or more of thrombotic episodes
Prevalence of 7% in Europe
Most common genetic defect accounting for DVT
Antiphospholipid syndrome Multi organ disease
Pregnancy involvement common
Arterial and venous thromboses
Either Lupus anticoagulant or Anti cardiolipin antibodies
APTT usually prolonged
Antibodies may be elevated following surgery, drugs or malignancy
Need anticoagulation with INR between 3 and 4

974
Q

A 53 year old male presents with a carcinoma of the transverse colon. Which of the following structures should be ligated close to their origin to maximise clearance of the tumour?

Superior mesenteric artery

Inferior mesenteric artery

Middle colic artery

Ileo-colic artery

Superior rectal artery

A

The middle colic artery supplies the transverse colon and requires high ligation during cancer resections. It is a branch of the superior mesenteric artery.

Transverse colon

The right colon undergoes a sharp turn at the level of the hepatic flexure to become the transverse colon.
At this point it also becomes intraperitoneal.
It is connected to the inferior border of the pancreas by the transverse mesocolon.
The greater omentum is attached to the superior aspect of the transverse colon from which it can easily be separated. The mesentery contains the middle colic artery and vein. The greater omentum remains attached to the transverse colon up to the splenic flexure. At this point the colon undergoes another sharp turn.

Relations
Superior Liver and gall-bladder, the greater curvature of the stomach, and the lower end of the spleen
Inferior Small intestine
Anterior Greater omentum
Posterior From right to left with the descending portion of the duodenum, the head of the pancreas, convolutions of the jejunum and ileum, spleen

975
Q

Which of the following statements related to the coagulation cascade is true?

The intrinsic pathway is the main pathway in coagulation

Heparin inhibits the activation of Factor 8

The activation of factor 8 is the point when the intrinsic and the extrinsic pathways meet

Tissue factor released by damaged tissue initiates the extrinsic pathway

Thrombin converts plasminogen to plasmin

A

The extrinsic pathway is the main path of coagulation. Heparin inhibits the activation of factors 2,9,10,11. The activation of factor 10 is when both pathways meet. Thrombin converts fibrinogen to fibrin. During fibrinolysis plasminogen is converted to plasmin to break down fibrin.

Coagulation cascade

Two pathways lead to fibrin formation

Intrinsic pathway (components already present in the blood)
Minor role in clotting
Subendothelial damage e.g. collagen
Formation of the primary complex on collagen by high-molecular-weight kininogen (HMWK), prekallikrein, and Factor 12
Prekallikrein is converted to kallikrein and Factor 12 becomes activated
Factor 12 activates Factor 11
Factor 11 activates Factor 9, which with its co-factor Factor 8a form the tenase complex which activates Factor 10

Extrinsic pathway (needs tissue factor released by damaged tissue)
Tissue damage
Factor 7 binds to Tissue factor
This complex activates Factor 9
Activated Factor 9 works with Factor 8 to activate Factor 10

Common pathway
Activated Factor 10 causes the conversion of prothrombin to thrombin
Thrombin hydrolyses fibrinogen peptide bonds to form fibrin and also activates factor 13 to form links between fibrin molecules

Fibrinolysis
Plasminogen is converted to plasmin to facilitate clot resorption

Clotting pathway Clotting parameters affected Factors affected
Intrinsic pathway Increased APTT Factors 8,9,11,12
Extrinsic pathway Increased PT Factor 7
Common pathway Increased APTT & PT Factors 2,5,10
Vitamin K dependent Factors 2,7,9,10

976
Q

A 28 year old man injures his knee whilst playing hockey. His right knee sustains direct impact and this results in a deep laceration of the right knee cap. He is taken to the nearest emergency department. As part of his assessment, xrays are taken of his right knee. These show a right sided patella fracture with a small amount of comminution. What is the most appropriate course of action?

Administration of antibiotics, debridement of wound, primary closure and placement in an extensor cast

Administration of antibiotics, debridement of wound, fracture control with encirclage wires and primary closure

Administration of antibiotics, debridement of wound, fracture control with external fixator and delayed primary closure

Administration of antibiotics, wound debridement, patellectomy, tendon advancement and primary closure

Administration of antibiotics, wound debridement, patellectomy, tendon advancement and delayed primary closure

A

B-Even in cases where an open fracture is present, patella fractures can be managed with definitive fixation and encirlage wires represent one such technique. Patella fractures do not lend themselves to external fixator placement. Patella excision would be an unusual immediate option unless severe contamination and or severe comminution were present.

Patella fractures

These are rather uncommon injuries and account for around 1% of fractures overall. Around 7% of them are open injuries. Typical mechanisms include either direct impact of indirect tension style injuries which occur from rapid knee flexion against contracted quadriceps muscle causing failure in tension. Depending on mechanism some are comminuted injuries others can be simple transverse or inferior avulsion style injuries.

Management is determined by mechanism of injury and patient status. Vertical or minimally displaced fractures can be managed by conservative measures. More complex fracture patterns may require operative fixation. There are several operative techniques, plates, cannulated screws and wires are all used. Healing rates of up to 95% are seen. Patients with open fractures see non union rates of up to 10%. Cases which are infected or fail to heal or are significantly infected may require partial or total patellectomy and tendon advancement. These procedures carry associated functional compromise.

977
Q

A 43 year old man is reviewed in the clinic following a cardiac operation. A chest x-ray is performed and a circular radio-opaque structure is noted medial aspect of the 4th intercostal space on the left. Which of the following procedures is the patient most likely to have undergone?

Aortic valve replacement with metallic valve

Tricuspid valve replacement with metallic valve

Tricuspid valve replacement with porcine valve

Pulmonary valve replacement with porcine valve

Mitral valve replacement with metallic valve

A

Prosthetic heart valves on Chest X-rays

The aortic and mitral valves are most commonly replaced and when a metallic valve is used, can be most readily identified on plain x-rays.
The presence of cardiac disease (such as cardiomegaly) may affect the figures quoted here.

Aortic
Usually located medial to the 3rd interspace on the right.

Mitral
Usually located medial to the 4th interspace on the left.

Tricuspid
Usually located medial to the 5th interspace on the right.

Please note that these are the sites at which an artificial valve may be located and are NOT the sites of auscultation.

978
Q

A 43 year old lady is diagnosed with primary hyperparathyroidism. Her serum PTH levels are elevated. An endocrine surgeon performs a parathyroidectomy. How long will it take for the serum PTH levels to fall if the functioning adenoma has been successfully removed?

6 hours

24 hours

2 hours

1 hour

10 minutes

A

PTH has a very short half life usually less than 10 minutes. Therefore a demonstrable drop in serum PTH should be identified within 10 minutes of removing the adenoma. This is useful clinically since it is possible to check the serum PTH intraoperatively prior to skin closure and explore the other glands if levels fail to fall.

Calcium homeostasis

Calcium ions are linked to a wide range of physiological processes. The largest store of bodily calcium is contained within the skeleton. Calcium levels are primarily controlled by parathyroid hormone, vitamin D and calcitonin.

Hormonal regulation of calcium
Hormone Actions
Parathyroid hormone (PTH)
Increase calcium levels and decrease phosphate levels
Increases bone resorption
Immediate action on osteoblasts to increase ca2+ in extracellular fluid
Osteoblasts produce a protein signaling molecule that activate osteoclasts which cause bone resorption
Increases renal tubular reabsorption of calcium
Increases synthesis of 1,25(OH)2D (active form of vitamin D) in the kidney which increases bowel absorption of Ca2+
Decreases renal phosphate reabsorption
1,25-dihydroxycholecalciferol (the active form of vitamin D)
Increases plasma calcium and plasma phosphate
Increases renal tubular reabsorption and gut absorption of calcium
Increases osteoclastic activity at high levels and osteoblasts at low levels
Increases renal phosphate reabsorption
Calcitonin
Secreted by C cells of thyroid
Inhibits intestinal calcium absorption
Inhibits osteoclast activity
Inhibits renal tubular absorption of calcium

Both growth hormone and thyroxine also play a small role in calcium metabolism.

979
Q

A 15 year-old boy presents to the out-patient clinic with tiredness, recurrent throat and chest infections, and gradual loss of vision. Multiple x-rays show brittle bones with no differentiation between the cortex and the medulla. What is the most likely diagnosis?

Rickets

Osteomalacia

Osteopetrosis

Ehlers Danlos type III

Osteogenesis imperfecta

A

Osteopetrosis is an autosomal recessive condition. It is commonest in young adults. They may present with symptoms of anaemia or thrombocytopaenia due to decreased marrow space. Radiology reveals a lack of differentiation between the cortex and the medulla described as marble bone. These bones are very dense and brittle.

Paediatric fractures

Paediatric fracture types
Type Injury pattern
Complete fracture Both sides of cortex are breached
Toddlers fracture Oblique tibial fracture in infants
Plastic deformity Stress on bone resulting in deformity without cortical disruption
Greenstick fracture Unilateral cortical breach only
Buckle fracture Incomplete cortical disruption resulting in periosteal haematoma only

Growth plate fractures
In paediatric practice fractures may also involve the growth plate and these injuries are classified according to the Salter- Harris system (given below):

Type Injury pattern
I Fracture through the physis only (x-ray often normal)
II Fracture through the physis and metaphysis
III Fracture through the physis and epiphyisis to include the joint
IV Fracture involving the physis, metaphysis and epiphysis
V Crush injury involving the physis (x-ray may resemble type I, and appear normal)

As a general rule it is safer to assume that growth plate tenderness is indicative of an underlying fracture even if the x-ray appears normal. Injuries of Types III, IV and V will usually require surgery. Type V injuries are often associated with disruption to growth.

Non accidental injury
Delayed presentation
Delay in attaining milestones
Lack of concordance between proposed and actual mechanism of injury
Multiple injuries
Injuries at sites not commonly exposed to trauma
Children on the at risk register

Pathological fractures
Genetic conditions, such as osteogenesis imperfecta, may cause pathological fractures.

Osteogenesis imperfecta
Defective osteoid formation due to congenital inability to produce adequate intercellular substances like osteoid, collagen and dentine.
Failure of maturation of collagen in all the connective tissues.
Radiology may show translucent bones, multiple fractures, particularly of the long bones, wormian bones (irregular patches of ossification) and a trefoil pelvis.

Subtypes
Type I The collagen is normal quality but insufficient quantity.
Type II- Poor collagen quantity and quality.
Type III- Collagen poorly formed. Normal quantity.
Type IV- Sufficient collagen quantity but poor quality.

Osteopetrosis
Bones become harder and more dense.
Autosomal recessive condition.
It is commonest in young adults.
Radiology reveals a lack of differentiation between the cortex and the medulla described as marble bone.

980
Q

A 53 year old man presents with drooling and facial pain and swelling. Ludwigs angina is suspected. Which of these tissue spaces is predominantly affected by this process?

Submandibular

Retropharyngeal

Buccal

Pre tracheal

Parapharyngeal

A

In most cases the submandibular space is involved. Incision and drainage of this space is often undertaken.

Ludwigs angina

This is a severe form of infection which affects the floor of mouth and surrounding tissues. The underlying cause in many cases is dental caries affecting the third molar tooth. The floor of the mouth and neck usually becomes very swollen and drooling is often seen. The presence of immunosuppression or conditions like diabetes or HIV increases the risks. Staphylococcal organisms and streptococcal organisms account for many cases.

Management comprises careful attention to maintaining airway, prompt administration of antibiotics and surgical drainage of any discrete collections. The underlying cause should also be identified and addressed.

981
Q

A 40 year old lady presents with varicose veins, these are found to originate from the short saphenous vein. As the vein is mobilised which structure is at greatest risk of injury?

Sciatic nerve

Sural nerve

Common peroneal nerve

Tibial nerve

Popliteal artery

A

The sural nerve is closely related and damage to this structure is a major cause of litigation. The other structures may all be injured but the risks are lower.

Popliteal fossa

Boundaries of the popliteal fossa
Laterally Biceps femoris above, lateral head of gastrocnemius and plantaris below
Medially Semimembranosus and semitendinosus above, medial head of gastrocnemius below
Floor Popliteal surface of the femur, posterior ligament of knee joint and popliteus muscle
Roof Superficial and deep fascia

Contents
Popliteal artery and vein
Small saphenous vein
Common peroneal nerve
Tibial nerve
Posterior cutaneous nerve of the thigh
Genicular branch of the obturator nerve
Lymph nodes

982
Q

A 53 year old man from Hong Kong presents with symptoms of fatigue, weight loss and recurrent epistaxis. Clinical examination reveals left sided cervical lymphadenopathy and oropharyngeal examination reveals an ulcerated mass in the naso pharynx. Which of the following viral agents is most commonly implicated in the development of this condition?

Cytomegalovirus

Epstein Barr virus

Coxsackie virus

Herpes simplex virus

None of the above

A

Trotter’s triad (diagnosis of nasopharyngeal carcinoma)
Unilateral conductive hearing loss
Ipsilateral facial & ear pain
Ipsilateral paralysis of soft palate

The clinical scenario is most typical for nasopharyngeal carcinoma. An association with previous Epstein Barr Virus is well established. Infection with the other viruses listed is not a recognised risk factor for the development of the condition.

Nasopharyngeal carcinoma

Squamous cell carcinoma of the nasopharynx
Rare in most parts of the world, apart from individuals from Southern China
Associated with Epstein Barr virus infection

Presenting features
Systemic Local
Cervical lymphadenopathy Otalgia
Unilateral serous otitis media
Nasal obstruction, discharge and/ or epistaxis
Cranial nerve palsies e.g. III-VI

Imaging
Combined CT and MRI.

Treatment
Radiotherapy is first line therapy.

983
Q

What is the most inferior anterior aortic branch?

Median sacral artery

Inferior mesenteric artery

Lumbar artery

Superior mesenteric artery

Gonadal artery

A

The IMA leaves the front of the aorta usually about 3 to 4cm superior to its bifurcation. The median sacral is not an anterior branch.
Abdominal aortic branches

Branches Level Paired Type
Inferior phrenic T12 (Upper border) Yes Parietal
Coeliac T12 No Visceral
Superior mesenteric L1 No Visceral
Middle suprarenal L1 Yes Visceral
Renal L1-L2 Yes Visceral
Gonadal L2 Yes Visceral
Lumbar L1-L4 Yes Parietal
Inferior mesenteric L3 No Visceral
Median sacral L4 No Parietal
Common iliac L4 Yes Terminal

984
Q

Which of the following is least likely to reduce the risk of post operative wound infection?

Electrical clippers to remove body hair

Use of poviodone impregnated drapes

Antibiotic prophylaxis for prosthesis placement

Routine use of mechanical bowel preparation

Chlorhexidine to prepare the skin

A

The routine use of mechanical bowel preparation is not recommended. There is some recent evidence to support the use of selective gut decontamination. However, this is not in mainstream practice at present.

Surgical site infection

  • Surgical site infections may occur following a breach in tissue surfaces and allow normal commensals and other pathogens to initiate infection. They are a major cause of morbidity and mortality.
    Surgical site infections (SSI) comprise up to 20% of all healthcare associated infections and at least 5% of patients undergoing surgery will develop an SSI as a result.
    In many cases the organisms are derived from the patient’s own body. Measures that may increase the risk of SSI include:
    Shaving the wound using a razor (disposable clipper preferred)
    Using a non iodine impregnated incise drape if one is deemed to be necessary
    Tissue hypoxia
    Delayed administration of prophylactic antibiotics in tourniquet surgery

Preoperatively
Don’t remove body hair routinely
If hair needs removal, use electrical clippers with single use head (razors increase infection risk)
Antibiotic prophylaxis if:
- placement of prosthesis or valve
- clean-contaminated surgery
- contaminated surgery
Use local formulary
Aim to give single dose IV antibiotic on anaesthesia
If a tourniquet is to be used, give prophylactic antibiotics earlier

Intraoperatively
Prepare the skin with alcoholic chlorhexidine (Lowest incidence of SSI)
Cover surgical site with dressing
A recent meta analysis has confirmed that administration of supplementary oxygen does not reduce the risk of wound infection. In contrast to previous individual RCT’s(1)
Wound edge protectors do not appear to confer benefit (2)

Post operatively
Tissue viability advice for management of surgical wounds healing by secondary intention

Use of diathermy for skin incisions
In the NICE guidelines the use of diathermy for skin incisions is not advocated(3). Several randomised controlled trials have been undertaken and demonstrated no increase in risk of SSI when diathermy is used(4).

References
1. Brar M et al.. Perioperative supplemental oxygen in colorectal patients: a meta analysis. J Surg Res 2011 (166): 227 -235.
2. Pinkney T et al. Impact of wound edge protection devices on surgical site infection after laparotomy: impact of a multicentre randomised controlled trial (ROSSINI Trial). BMJ 2013 (347):10.
3. http://www.nice.org.uk/CG74
4. Ahmad N and Ahmed A. Meta-analysis of the effectiveness of surgical scalpel or diathermy in making abdominal skin incisions. Ann Surg 2011, 253(1):8-13.

985
Q

A 22 year old man is investigated for weight loss. A duodenal biopsy taken as part of his investigations shows total villous atrophy and lymphocytic infiltrate. He has a skin lesion that has small itchy papules. What is the most likely underlying diagnosis of the skin lesion?

Pyoderma gangrenosum

Dermatitis herpetiformis

Bullous pemphigoid

Acanthosis nigricans

Pemphigus vulgaris

A

The patient has coeliac disease and this is classically associated with dermatitis herpetiformis.

Skin Diseases

Skin lesions may be referred for surgical assessment, but more commonly will come via a dermatologist for definitive surgical management.

Skin malignancies include basal cell carcinoma, squamous cell carcinoma and malignant melanoma.

Basal Cell Carcinoma
Most common form of skin cancer.
Commonly occur on sun exposed sites apart from the ear.
Sub types include nodular, morphoeic, superficial and pigmented.
Typically slow growing with low metastatic potential.
Standard surgical excision, topical chemotherapy and radiotherapy are all successful.
As a minimum a diagnostic punch biopsy should be taken if treatment other than standard surgical excision is planned.

Squamous Cell Carcinoma
Again related to sun exposure.
May arise in pre - existing solar keratoses.
May metastasize if left.
Immunosupression (e.g. following transplant), increases risk.
Wide local excision is the treatment of choice and where a diagnostic excision biopsy has demonstrated SCC, repeat surgery to gain adequate margins may be required.

Malignant Melanoma
The main diagnostic features (major criteria):
Change in size
Change in shape
Change in colour
Secondary features (minor criteria)
Diameter >6mm
Inflammation
Oozing or bleeding
Altered sensation

Treatment
Suspicious lesions should undergo excision biopsy. The lesion should be removed completely as incision biopsy can make subsequent histopathological assessment difficult.
Once the diagnosis is confirmed the pathology report should be reviewed to determine whether further re-excision of margins is required (see below):

Margins of excision-Related to Breslow thickness
Lesions 0-1mm thick 1cm
Lesions 1-2mm thick 1- 2cm (Depending upon site and pathological features)
Lesions 2-4mm thick 2-3 cm (Depending upon site and pathological features)
Lesions >4 mm thick 3cm
Marsden J et al. Revised UK guidelines for management of Melanoma. Br J Dermatol 2010 163:238-256.

Further treatments such as sentinel lymph node mapping, isolated limb perfusion and block dissection of regional lymph node groups should be selectively applied.

Kaposi Sarcoma
Tumour of vascular and lymphatic endothelium.
Purple cutaneous nodules.
Associated with immuno supression.
Classical form affects elderly males and is slow growing.
Immunosupression form is much more aggressive and tends to affect those with HIV related disease.

Non malignant skin disease

Dermatitis Herpetiformis
Chronic itchy clusters of blisters.
Linked to underlying gluten enteropathy (coeliac disease).

Dermatofibroma
Benign lesion.
Firm elevated nodules.
Usually history of trauma.
Lesion consists of histiocytes, blood vessels and fibrotic changes.

Pyogenic granuloma
Overgrowth of blood vessels.
Red nodules.
Usually follow trauma.
May mimic amelanotic melanoma.

Acanthosis nigricans
Brown to black, poorly defined, velvety hyperpigmentation of the skin.
Usually found in body folds such as the posterior and lateral folds of the neck, the axilla, groin, umbilicus, forehead, and other areas.
The most common cause of acanthosis nigricans is insulin resistance, which leads to increased circulating insulin levels. Insulin spillover into the skin results in its abnormal increase in growth (hyperplasia of the skin).
In the context of a malignant disease, acanthosis nigricans is a paraneoplastic syndrome and is then commonly referred to as acanthosis nigricans maligna. Involvement of mucous membranes is rare and suggests a coexisting malignant condition.

986
Q

Which of the intravenous fluid combinations listed below should be considered for replacement of losses from a high output ileostomy in a 2 year old?

0.9% saline with added potassium

Hartmanns solution

10% dextrose with added potassium

0.45% saline with added potassium

2.7% saline with added potassium

A

In children with ongoing losses, these should be replaced with 0.9% sodium chloride with added potassium.

Paediatric fluid management

Since 2000 there have been at least 4 reported deaths from fluid induced hyponatraemia in children. This led to the National Patient Safety Agency introducing revised guidelines in 2007. These have been reviewed and extensively updated by NICE in 2015 and further modified in 2020.
Indications for IV fluids include:
Resuscitation and circulatory support
Replacing on-going fluid losses
Maintenance fluids for children for whom oral fluids are not appropriate
Correction of electrolyte disturbances

Fluids to be avoided
Outside the neonatal period saline / glucose solutions should not be given. The greatest risk is with saline 0.18 / glucose 4% solutions. The report states that 0.45% saline / 5% glucose may be used. But preference should be given to isotonic solutions and few indications exist for this solution either.
The key point emphasised in the NICE guidelines in the avoidance of glucose containing solutions and instead of routinely giving glucose IV to children and neonates, the blood glucose levels should be monitored. In neonates in the first few days of life, sodium levels in the range of 131-154 mmol/l may be too high and a neonatologist consulted on a case by case basis.

Fluids to be used
If children and young people need IV fluids for routine maintenance, initially use isotonic crystalloids that contain sodium in the range 131 to 154 mmol/litre

Potassium should be added to maintenance fluids according patients plasma potassium levels (which should be monitored).
Blood glucose levels should be monitored in individuals at risk of hypoglycaemia

Intraoperative fluid management
If children and young people need IV fluids for routine maintenance, initially use isotonic crystalloids that contain sodium in the range 131 to 154 mmol/litre
Blood glucose levels should be monitored

Maintenance fluids
Weight Water requirement/kg/day Na mmol/kg/day K mmol/kg/day
First 10Kg body weight 100ml 2-4 1.5-2.5
Second 10Kg body weight 50ml 1-2 0.5-1.5
Subsequent Kg 20ml 0.5-1.0 0.2-0.7

Blood glucose will need to be monitored

Reference
NPSA -reducing risk of hyponatraemia when administering intravenous fluids to children. Issue date March 2007. Further references included in this document.

NICE Guidance NG29. Intravenous fluid therapy in children and young people in hospital. Published 2015, reviewed an updated 2020.

987
Q

A 23 year old man presents with delayed diagnosis of appendicitis. The appendix is retrocaecal and has perforated causing a psoas abscess. Into which structure does the psoas major muscle insert?

Greater trochanter of the femur

Linea aspera of the femur

Lesser trochanter of the femur

Iliac crest

None of the above

A

The psoas major inserts into the lesser trochanter.

Psoas Muscle

Origin
The deep part originates from the transverse processes of the five lumbar vertebrae, the superficial part originates from T12 and the first 4 lumbar vertebrae.

Insertion
Lesser trochanter of the femur.

Innervation
Anterior rami of L1 to L3.

Action
Flexion and external rotation of the hip. Bilateral contraction can raise the trunk from the supine position.

988
Q

An 85 year old man presents with a cough and haemoptysis. He has a modest smoking history of 15 pack years. He is found to have a tumour located in the right main bronchus, with no evidence of metastatic disease. He decides not to undergo any treatment and he remains well for a further 12 months before developing symptomatic metastasis. What is the most likely diagnosis?

Adenocarcinoma

Small cell carcinoma

Squamous cell carcinoma

Metastatic renal cancer

Lymphoma

A

Squamous cell carcinomas are reported to be more slow growing and are typically centrally located. Small cell carcinomas are usually centrally located. However, small cell carcinomas would seldom be associated with a survival of a year without treatment.

Lung cancer

Lung cancers may be classified according to histological subtypes. The main distinction is between small cell and non small cell lung cancer. Non small cell lung cancer is the most common variant and accounts for 80% of all lung cancers.

Non small cell lung cancer
These share common features of prognosis and management. They comprise the following tumours:
Squamous cell carcinoma (25% cases)
Adenocarcinoma (40% cases)
Large cell carcinoma (10% cases)
Paraneoplastic features and early disease dissemination are less likely than with small cell lung carcinoma. Adenocarcinoma is the most common lung cancer type encountered in never smokers.

Small cell lung carcinoma
Small cell lung carcinomas are comprised of cells with a neuro endocrine differentiation. The neuroendocrine hormones may be released from these cells with a wide range of paraneoplastic associations. These tumours are strongly associated with smoking and will typically arise in the larger airways. They disseminate early in the course of the disease and although they are usually chemosensitive this seldom results in long lasting remissions.

989
Q

A 54-year-old man is admitted for an elective hip replacement. Three days post operatively you suspect he has had a pulmonary embolism. He has no past medical history of note. Blood pressure is 120/80 mmHg with a pulse of 90/min. The chest x-ray is normal. Following treatment with low-molecular weight heparin, what is the most appropriate initial lung imaging investigation to perform?

Pulmonary angiography

Echocardiogram

MRI thorax

Ventilation-perfusion scan

Computed tomographic pulmonary angiography

A

CTPA is the first line investigation for PE according to current BTS guidelines
Current UK NICE guidance supports CTPA as first line investigation now. Pulmonary angiography is of course the ‘gold standard’ but this is not what the question asks for

Pulmonary embolism: investigation

UK 2020 NICE guidance endorses CTPA is most cases of suspected PE.

Key points from the guidelines include:
computed tomographic pulmonary angiography (CTPA) is now the recommended initial lung-imaging modality for non-massive PE. Advantages compared to V/Q scans include speed, easier to perform out-of-hours, a reduced need for further imaging and the possibility of providing an alternative diagnosis if PE is excluded
if the CTPA is negative then patients do not need further investigations or treatment for PE
ventilation-perfusion scanning may be used initially if appropriate facilities exist, the chest x-ray is normal, and there is no significant symptomatic concurrent cardiopulmonary disease

Some other points
Clinical probability scores based on risk factors and history and now widely used to help decide on further investigation/management

D-dimers
sensitivity = 95-98%, but poor specificity

V/Q scan
sensitivity = 98%; specificity = 40% - high negative predictive value, i.e. if normal virtually excludes PE
other causes of mismatch in V/Q include old pulmonary embolisms, AV malformations, vasculitis, previous radiotherapy
COPD gives matched defects

CTPA
peripheral emboli affecting subsegmental arteries may be missed

Pulmonary angiography
the gold standard
significant complication rate compared to other investigations

990
Q

A 56-year-old man with metastatic prostate cancer comes for review. He is known to have spinal metastases but until now has not had any significant problems with pain control. Unfortunately he is now getting regular back pain despite taking paracetamol 1g qds. Neurological examination is unremarkable. What is the most appropriate next step?

Switch to co-codamol 30/500

Refer for radiotherapy

Add oral bisphosphonate

Add non steroidal anti inflammatory drug

Add dexamethasone

A

Metastatic bone pain may respond to NSAIDs, bisphosphonates or radiotherapy
Bone pain often responds well to NSAIDs. Both radiotherapy and bisphosphonates have a role in managing bony pain but these are not first-line treatments.

Palliative care prescribing: pain

SIGN issued guidance on the control of pain in adults with cancer in 2008. Selected points
the breakthrough dose of morphine is one-sixth the daily dose of morphine
all patients who receive opioids should be prescribed a laxative
opioids should be used with caution in patients with chronic kidney disease. Alfentanil, buprenorphine and fentanyl are preferred
metastatic bone pain may respond to NSAIDs, bisphosphonates or radiotherapy

When increasing the dose of opioids the next dose should be increased by 30-50%.

Conversion between opioids
From To
Oral codeine Oral morphine Divide by 10
Oral tramadol Oral morphine Divide by 5

From To
Oral morphine Oral oxycodone Divide by 2

The BNF states that oral morphine sulphate 80-90mg over 24 hours is approximately equivalent to one ‘25 mcg/hour’ fentanyl patch, therefore product literature should be consulted.

From To
Oral morphine Subcutaneous diamorphine Divide by 3
Oral oxycodone Subcutaneous diamorphine Divide by 1.5

991
Q

A 14 month old child is admitted with colicky abdominal pain and on investigation is found to have an ileo-ileal intussusception. What is the best course of action?

Attempt hydrostatic reduction with barium enema

Attempt pneumatic reduction with air insufflation

Undertake a laparotomy

Undertake a colonoscopy

Undertake a flexible sigmoidoscopy

A

Ileo-ileal intussusception are far less common than the ileo-colic variant. However, where they occur, they require surgery and are not amenable to pneumatic reduction.

Intussusception- Paediatric

Intussusception typically presents with colicky abdominal pain and vomiting. The telescoping of the bowel produces mucosal ischaemia and bleeding may occur resulting in the passage of red current jelly stools. Recognised causes include lumenal pathologies such as polyps, lymphadenopathy and diseases such as cystic fibrosis. Idiopathic intussceception of the ileocaecal valve and terminal ileum is the most common variant and typically affects young children and toddlers.

The diagnosis is usually made by abdominal ultrasound investigation. The decision as to the optimal treatment is dictated by the patients physiological status and abdominal signs. In general, children who are unstable with localising peritoneal signs should undergo laparotomy, as should those in whom attempted radiological reduction has failed.

In relatively well children, without localising signs, attempted pneumatic reduction under fluroscopic guidance is the usual treatment. This is not done where there are concerns about impending perforation or if there is a small bowel intussusception.

992
Q

A 4 year old boy falls and sustains a fracture to the growth plate of his right wrist. Which of the following systems is used to classify the injury?

Salter - Harris system

Weber system

Gustilo - Anderson system

Garden system

None of the above

A

The mnemonic ‘SALTR’ can be used to help remember the first five types. This mnemonic requires the reader to imagine the bones as long bones, with the epiphyses at the base.
I ‘S’ = Slip (separated or straight across). Fracture of the cartilage of the physis (growth plate)
II ‘A’ = Above. The fracture lies above the physis, or Away from the joint.
III ‘L’ = Lower. The fracture is below the physis in the epiphysis.
IV ‘TE’ = Through Everything. The fracture is through the metaphysis, physis, and epiphysis.
V ‘R’ = Rammed (crushed). The physis has been crushed
The Salter - Harris system is most commonly used. The radiological signs in Type 1 and 5 injuries may be identical. Which is unfortunate as type 5 injuries do not do well (and may be missed!). One of our users has helpfully supplied a mnemonic for remembering the types (see above).

993
Q

A 60 year old female is undergoing a Whipples procedure for adenocarcinoma of the pancreas. As the surgeons begin to mobilise the pancreatic head they identify a large vessel passing inferiorly over the anterior aspect of the uncinate process. What is it likely to be?

Superior mesenteric artery

Coeliac axis

Inferior mesenteric artery

Aorta

Left gastric artery

A

The superior mesenteric artery arises from the aorta and passes anterior to the lower part of the pancreas. Invasion of this structure is a relative contra indication to resectional surgery.
Pancreas

The pancreas is a retroperitoneal organ and lies posterior to the stomach. It may be accessed surgically by dividing the peritoneal reflection that connects the greater omentum to the transverse colon. The pancreatic head sits in the curvature of the duodenum. Its tail lies close to the hilum of the spleen, a site of potential injury during splenectomy.

Relations
Posterior to the pancreas
Pancreatic head Inferior vena cava
Common bile duct
Right and left renal veins
Superior mesenteric vein and artery
Pancreatic neck Superior mesenteric vein, portal vein
Pancreatic body- Left renal vein
Crus of diaphragm
Psoas muscle
Adrenal gland
Kidney
Aorta
Pancreatic tail Left kidney

Anterior to the pancreas
Pancreatic head 1st part of the duodenum
Pylorus
SMA and SMV(uncinate process)
Pancreatic body Stomach
Duodenojejunal flexure
Pancreatic tail Splenic hilum

Superior to the pancreas
Coeliac trunk and its branches common hepatic artery and splenic artery

Grooves of the head of the pancreas
2nd and 3rd part of the duodenum

Arterial supply
Head: pancreaticoduodenal artery
Rest: splenic artery

Venous drainage
Head: superior mesenteric vein
Body and tail: splenic vein

Ampulla of Vater
Merge of pancreatic duct and common bile duct
Is an important landmark, halfway along the second part of the duodenum, that marks the anatomical transition from foregut to midgut (also the site of transition between regions supplied by coeliac trunk and SMA).

994
Q

Which of the following is least likely to be associated with hypercalcaemia?

Thiazides

Antacids

Coeliac disease

Sarcoidosis

Zolinger-Ellison syndrome

A

Mnemonic for the causes of hypercalcaemia:

CHIMPANZEES

C alcium supplementation
H yperparathyroidism
I atrogentic (Drugs: Thiazides)
M ilk Alkali syndrome
P aget disease of the bone
A cromegaly and Addison’s Disease
N eoplasia
Z olinger-Ellison Syndrome (MEN Type I)
E xcessive Vitamin D
E xcessive Vitamin A
S arcoidosis
Patients with coeliac disease tend to develop hypocalcaemia due to malabsorption of calcium by the bowel.

995
Q

A 3 day old baby presents with recurrent episodes of choking and cyanotic episodes. There is a history of polyhydramnios. What is the most likely diagnosis?

Meconium ileus

Oesophageal atresia

Duodenal atresia

Pyloric stenosis

Diaphragmatic hernia

A

The history of polyhydramios makes diaphragmatic hernia less likely. The other conditions tend not to cause choking. In the short term this is addressed by passage of a Replogle tube which can be used to remove the oesophageal secretions, pending surgery.

Paediatric Gastrointestinal disorders

Pyloric stenosis
M>F
5-10% Family history in parents
Projectile non bile stained vomiting at 4-6 weeks of life
Diagnosis is made by test feed or USS
Treatment: Ramstedt pyloromyotomy (open or laparoscopic)
Acute appendicitis
Uncommon under 3 years
When occurs may present atypically
Mesenteric adenitis
Central abdominal pain and URTI
Conservative management
Intussusception
Telescoping bowel
Proximal to or at the level of, ileocaecal valve
6-9 months age
Colicky pain, diarrhoea and vomiting, sausage shaped mass, red jelly stool.
Treatment: reduction with air insufflation
Malrotation
High caecum at the midline
Feature in exomphalos, congenital diaphragmatic hernia, intrinsic duodenal atresia
May be complicated by development of volvulus, infant with volvulus may have bile stained vomiting
Diagnosis is made by upper GI contrast study and USS
Treatment is by laparotomy, if volvulus is present (or at high risk of occurring then a ladds procedure is performed
Hirschsprung’s disease
Absence of ganglion cells from myenteric and submucosal plexuses
Occurs in 1/5000 births
Full thickness rectal biopsy for diagnosis
Delayed passage of meconium and abdominal distension
Treatment is with rectal washouts initially, thereafter an anorectal pull through procedure
Oesophageal atresia
Associated with tracheo-oesophageal fistula and polyhydramnios
May present with choking and cyanotic spells following aspiration
VACTERL associations
Meconium ileus
Usually delayed passage of meconium and abdominal distension
Majority have cystic fibrosis
X-Rays may not show a fluid level as the meconium is viscid (depends upon feeding), PR contrast studies may dislodge meconium plugs and be therapeutic
Infants who do not respond to PR contrast and NG N-acetyl cysteine will require surgery to remove the plugs
Biliary atresia
Jaundice > 14 days
Increased conjugated bilirubin
Urgent Kasai procedure
Necrotising enterocolitis
Prematurity is the main risk factor
Early features include abdominal distension and passage of bloody stools
X-Rays may show pneumatosis intestinalis and evidence of free air
Increased risk when empirical antibiotics are given to infants beyond 5 days
Treatment is with total gut rest and TPN, babies with perforations will require laparotomy

996
Q

Which of the muscles listed below is attached to the anterior aspect of the fibrous capsule that encases the elbow joint?

Pronator teres

Biceps

Brachialis

Triceps

Extensor carpi radialis longus

A

The brachialis inserts some of its fibres into the fibrous joint of the elbow capsule and when it contracts, it helps to flex the joint.

Elbow joint

This large synovial joint is of the hinge variety of joint, the bones of the forearm articulate with the lower end of the humerus. The upper ends of the radius and ulnar are bound together by the anular ligament of the radius in such a way as to permit movement between these two bones at what is described as the proximal radio-ulnar joint. The elbow and the proximal radio-ulnar joints have a common fibrous capsule and synovial cavity, and though the anular ligament plays a part in the structure of both joints, it is described with the proximal radio-ulnar joint.

Articular surfaces
The humeral articular surface at the elbow comprises the grooved trochlea, the spheroidal capitulum, and the sulcus between them. This composite surface is covered by a layer of articular cartilage. The capitulum is confined to the anterior and distal aspects of the bone, but the trochea extends round the distal end of the bone from the lower edge of the coronoid fossa on the front of the humerus to the lower edge of the olecranon fossa posteriorly. The articular surface covering the ulnar surface of the elbow joint is interrupted along the deepest part in a transverse line. Then trochlear notch articulates with the trochlea of the humerus forming a saddle shaped joint with it. The radial surface has a slight concavity to the proximal surface of the head which articulates with the capitulum while its raised margin lies on the capitulotrochlear groove. This surface of the head is covered with articular cartilage which is continuous with that round the sides in the radio-ulnar joint. The radial and ulnar surfaces are most fully in contact with the corresponding humeral surfaces when the forearm is in a position midway between full pronation and supination and the elbow is fixed to a right angle.

Fibrous capsule
The joint is encased within a fibrous capsule that is relatively weak anteriorly and posteriorly, its strengthened at the sides to form the radial and ulnar collateral ligaments. The anterior part of the capsule is attached to the front of the humerus immediately superior to the radial and coronoid fossae, to the anterior border of the coronoid process of the ulna, and to the anterior part of the anular ligament of the radius. The brachialis muscle covers the greater part of the front of the capsule, and some of its deep fibres insert into the capsule.
The posterior part of the capsule is very weak in its median part. However, the tendon of triceps inserts at this site and supports it and also draws it upwards in extension.
The radial collateral ligament is a strong triangular shaped thickening of the fibrous capsule. Its apex is attached superiorly to the antero-inferior aspect of the lateral epicondyle of the humerus in close relation to the common extensor muscles whose common origin overlies this site. Distally, the broad base of the ligament blends with the anular ligament of the radius, and is attached both in front and behind to the margins of the radial notch on the ulna.
The ulna collateral ligament is comprised of three capsular condensations which are continuous with one another. An anterior band passes from the front of the medial epicondyle of the humerus to the medial edge of the coronoid process of the ulna; it is closely related to the common origin of the superficial flexor muscles. A posterior band is attached above to the back of the medial epicondyle and below to the medial edge of the olecranon. A transverse band stretches between the attachments of the anterior and posterior bands on the coronoid process and the olecranon. The lower edge of this transverse ligament is free, a small amount of synovial membrane may protrude through the space between this and the underlying bone.

Synovial membrane
The attachments of the synovial membrane generally follow those of the fibrous capsule.

Nerve supply
The elbow joint derives its nerve supply from the musculocutaneous, median, radial and ulnar nerves.

Movements
Movement occurs around a transverse axis, a movement of flexion when the forearm makes anteriorly a diminishing angle with the upper arm and extension when the opposite occurs. The axis of movement passes through the humeral epicondyles and is not at right angles with either the humerus or bones of the forearm. In full extension with the forearm supinated, the arm and forearm form an angle which is more than 180 degrees, the extent to which this angle is exceeded is termed the carrying angle. The carrying angle is masked when the forearm is pronated.

997
Q

A 45 year old male is referred to clinic for consideration of resection of a lung malignancy. He reports shortness of breath and haemoptysis. Investigations reveal a corrected calcium of 2.84 mmol/l, an FEV 1 of 1.9L and histology of a squamous cell carcinoma. The patient is noted to have a hoarse voice. Which one of the following is a contraindication to surgical resection in lung cancer?

Haemoptysis

FEV 1 of 1.9 litres

Histology shows squamous cell cancer

Vocal cord paralysis

Calcium = 2.84 mmol/L

A

Contraindications to lung cancer surgery include SVC obstruction, FEV < 1.5, MALIGNANT pleural effusion, and vocal cord paralysis
Paralysis of a vocal cord implies extracapsular spread to mediastinal nodes and is an indication of inoperability.

Lung cancer: non-small cell management

Management
Only 20% suitable for surgery
Mediastinoscopy performed prior to surgery as CT does not always show mediastinal lymph node involvement
Curative or palliative radiotherapy
Poor response to chemotherapy

Surgery contraindications
Assess general health
Stage IIIb or IV (i.e. metastases present)
FEV1 < 1.5 litres is considered a general cut-off point*
Malignant pleural effusion
Tumour near hilum
Vocal cord paralysis
SVC obstruction

  • However if FEV1 < 1.5 for lobectomy or < 2.0 for pneumonectomy then some authorities advocate further lung function tests as operations may still go ahead based on the results
998
Q

A 42 year old woman is admitted to surgery with acute cholecystitis. She is known to have hypertension, rheumatoid arthritis and polymyalgia rheumatica. Her medical therapy includes:
Paracetamol 1g qds
Codeine phosphate 30mg qds
Bendrofluazide 2.5 mg od
Ramipril 10mg od
Methotrexate 7.5mg once a week
Prednisolone 5mg od
You are called by the CT1 to assess this lady as she has become delirious and hypotensive 2 hours after surgery. Her blood results reveal:

Na+ 132 mmol/l
K+ 5.3 mmol/l
Urea 7 mmol/l
Creatinine 108 µmol/l

Hb 12.4 g/dl
Platelets 178 * 109/l
WBC 15.4 * 109/l

What management is needed immediately?

Ceftriaxone IV

Hydrocortisone 50mg IV

CT scan abdomen

Urgent exploratory laparotomy

Hydrocortisone 100mg IV

A

This patient has acute adrenal insufficiency and urgently needs steroid replacement.

Addisonian crisis

Causes
Sepsis or surgery causing an acute exacerbation of chronic insufficiency (Addison’s, Hypopituitarism)
Adrenal haemorrhage eg Waterhouse-Friderichsen syndrome (fulminant meningococcemia)
Steroid withdrawal

Management
Hydrocortisone 100 mg im or iv
1 litre normal saline infused over 30-60 mins or with dextrose if hypoglycaemic
Continue hydrocortisone 6 hourly until the patient is stable. No fludrocortisone is required because high cortisol exerts weak mineralocorticoid action
Oral replacement may begin after 24 hours and be reduced to maintenance over 3-4 days

999
Q

A 55 year old man develops an acute colonic pseudo-obstruction following a laminectomy. Despite correction of his electrolytes and ongoing supportive care he fails to settle. Which of the drugs listed below may improve the situation?

Buscopan

Neostigmine

Metoclopramide

Mebevrine

Sodium picosulphate

A

Neostigmine affects the degradation of acetylcholine and will therefore stimulate both nicotinic and muscarinic receptors. It may produce symptomatic bradycardia and should therefore only be administered in a monitored environment. In colonic pseudo-obstruction it produces generalised colonic contractions and its onset is usually rapid.

Colonic pseudo-obstruction

Colonic pseudo-obstruction is characterised by the progressive and painless dilation of the colon. The abdomen may become grossly distended and tympanic. Unless a complication such as impending bowel necrosis or perforation occurs, there is usually little pain.
Diagnosis involves excluding a mechanical bowel obstruction with a plain film and contrast enema. The underlying cause is usually electrolyte imbalance and the condition will resolve with correction of this and supportive care.
Patients who do not respond to supportive measures should be treated with attempted colonoscopic decompression and/ or the drug neostigmine. In rare cases surgery may be required.

1000
Q

A 42 year old lady is reviewed in the outpatient clinic following a routine surgical procedure. She complains of diminished sensation at the lateral aspect of her foot. Which of the following nerves is most likely to be affected?

Sural

Superficial peroneal

Deep peroneal

Medial plantar

Lateral plantar

A

The sural nerve supplies the lateral aspect of the foot. It runs alongside the short saphenous vein and may be injured in short saphenous vein surgery.

Foot- Cutaneous sensation

Region Nerve
Lateral plantar Sural
Dorsum (not 1st web space) Superficial peroneal
1st Web space Deep peroneal
Extremities of toes Medial and lateral plantar nerves
Proximal plantar Tibial
Medial plantar Medial plantar nerve
Lateral plantar (inferior aspect) Lateral plantar nerve

1001
Q

A 78 year old man presents with left sided rest pain in his leg and a non healing arterial leg ulcer on the same leg. Imaging shows normal right leg vessels, on the left side there is a long occlusion of the external iliac artery that is unsuitable for stenting. He has a significant cardiac history. What is the most appropriate treatment option?

Aorto-bifemoral bypass

Axillo-bifemoral bypass

Femoro-femoro cross over graft

Femoro-distal bypass

Amputation

A

Femoro-femoral cross over grafts are an option for treatment of iliac occlusions in patients with significant co-morbidities and healthy contralateral vessels. In reality, the idealised situation presented here seldom applies and the opposite vessels usually have some disease and one must be careful not to damage the healthy side.

Peripheral vascular disease

Indications for surgery to revascularise the lower limb
Intermittent claudication
Critical ischaemia
Ulceration
Gangrene

Intermittent claudication that is not disabling may provide a relative indication, whilst the other complaints are often absolute indications depending upon the frailty of the patient.

Assessment
Clinical examination
Ankle brachial pressure index measurement
Duplex arterial ultrasound
Angiography (standard, CT or MRI): usually performed only if intervention being considered.

Angioplasty
In order for angioplasty to be undertaken successfully the artery has to be accessible. The lesion relatively short and reasonable distal vessel runoff. Longer lesions may be amenable to sub-intimal angioplasty.

Surgery
Surgery will be undertaken where attempts at angioplasty have either failed or are unsuitable. Bypass essentially involves bypassing the affected arterial segment by utilising a graft to run from above the disease to below the disease. As with angioplasty good runoff improves the outcome.

Some key concepts with bypass surgery

Superficial femoral artery occlusion to the above knee popliteal
In the ideal scenario, vein (either in situ or reversed LSV) would be used as a conduit. However, prosthetic material has reasonable 5 year patency rates and some would advocate using this in preference to vein so that vein can be used for other procedures in the future. In general terms either technique is usually associated with an excellent outcome (if run off satisfactory).

Procedure
Artery dissected out, IV heparin 3,000 units given and then the vessels are cross clamped
Longitudinal arteriotomy
Graft cut to size and tunneled to arteriotomy sites
Anastomosis to femoral artery usually with 5/0 ‘double ended’ Prolene suture
Distal anastomosis usually using 6/0 ‘double ended’ Prolene

Distal disease
Femoro-distal bypass surgery takes longer to perform, is more technically challenging and has higher failure rates.
In elderly diabetic patients with poor runoff a primary amputation may well be a safer and more effective option. There is no point in embarking on this type of surgery in patients who are wheelchair bound.
In femorodistal bypasses vein gives superior outcomes to PTFE.

Rules
Vein mapping 1st to see whether there is suitable vein (the preferred conduit). Sub intimal hyperplasia occurs early when PTFE is used for the distal anastomosis and will lead to early graft occlusion and failure.
Essential operative procedure as for above knee fem-pop.
If there is insufficient vein for the entire conduit then vein can be attached to the end of the PTFE graft and then used for the distal anastomosis. This type of ‘vein boot’ is technically referred to as a Miller Cuff and is associated with better patency rates than PTFE alone.
Remember the more distal the arterial anastomosis the lower the success rate.

References
Peach G et al. Diagnosis and management of peripheral arterial disease. BMJ 2012; 345: 36-41.

1002
Q

What are the boundaries of the ‘safe triangle’ for chest drain insertion?

Bounded by trapezius, latissimus dorsi, and laterally by the vertebral border of the scapula

Bounded by latissimus dorsi, pectoralis major, line superior to the nipple and apex at the axilla

Bounded by latissimus dorsi, serratus anterior, line superior to the nipple and apex at the axilla

Bounded by trapezius, deltoid, rhomboid major and teres minor

Bounded by trapezius, deltoid and latissimus dorsi

A

B
Chest drains

There are a number of different indications for chest drain insertion. In general terms large bore chest drains are preferred for trauma and haemothorax drainage. Smaller diameter chest drains can be used for pneumothorax or pleural effusion drainage.

Insertion can be performed either using anatomical guidance or through ultrasound guidance. In the exam, the anatomical method is usually tested.

It is advised that chest drains are placed in the ‘safe triangle’. The triangle is located in the mid axillary line of the 5th intercostal space. It is bordered by:
Anterior edge latissimus dorsi, the lateral border of pectoralis major, a line superior to the horizontal level of the nipple, and the apex below the axilla.

Another triangle is situated behind the scapula. It is bounded above by the trapezius, below by the latissimus dorsi, and laterally by the vertebral border of the scapula; the floor is partly formed by the rhomboid major. If the scapula is drawn forward by folding the arms across the chest, and the trunk bent forward, parts of the sixth and seventh ribs and the interspace between them become subcutaneous and available for auscultation. The space is therefore known as the triangle of auscultation.

References
Prof Harold Ellis. The applied anatomy of chest drains insertions. British Journal of hospital medicine 2007; (68): 44-45.

Laws D, Neville E, Duffy J. BTS guidelines for insertion of chest drains. Thorax, 2003; (58): 53-59.

1003
Q

Which of the following structures does not pass through the foramen ovale?

Lesser petrosal nerve

Accessory meningeal artery

Maxillary nerve

Emissary veins

Otic ganglion

A

Mnemonic: OVALE

O tic ganglion
V3 (Mandibular nerve:3rd branch of trigeminal)
A ccessory meningeal artery
L esser petrosal nerve
E missary veins
Foramina of the base of the skull

Foramen Location Contents
Foramen ovale Sphenoid bone Otic ganglion
V3 (Mandibular nerve:3rd branch of
trigeminal)
Accessory meningeal artery
Lesser petrosal nerve
Emissary veins
Foramen spinosum Sphenoid bone Middle meningeal artery
Meningeal branch of the Mandibular nerve
Foramen rotundum Sphenoid bone Maxillary nerve (V2)
Foramen lacerum/ carotid canal Located between the sphenoid, the apex of the petrous temporal and the basilar part of the occipital Base of the medial pterygoid plate.
Internal carotid artery*
Nerve and artery of the pterygoid canal
Jugular foramen Temporal bone Anterior: inferior petrosal sinus
Intermediate: glossopharyngeal, vagus, and accessory nerves.
Posterior: sigmoid sinus (becoming the internal jugular vein) and some meningeal branches from the occipital and ascending pharyngeal arteries.
Foramen magnum Occipital bone Anterior and posterior spinal arteries
Vertebral arteries
Medulla oblongata
Stylomastoid foramen Temporal bone Stylomastoid artery
Facial nerve
Superior orbital fissure Sphenoid bone Oculomotor nerve (III)
Recurrent meningeal artery
Trochlear nerve (IV)
Lacrimal, frontal and nasociliary branches of opthalmic nerve (V1)
Abducent nerve (VI)
Superior ophthalmic vein

*= In life the foramen lacerum is occluded by a cartilagenous plug. The ICA initially passes into the carotid canal which ascends superomedially to enter the cranial cavity through the foramen lacerum.

1004
Q

Which of the following vessels does not drain directly into the inferior vena cava?

Superior mesenteric vein

Right common iliac vein

Right hepatic vein

Left hepatic vein

Right testicular vein

A

The superior mesenteric vein drains into the portal vein. The right and left hepatic veins drain into it directly, this can account for major bleeding in more extensive liver shearing type injuries.
Inferior vena cava

Origin
L5

Path
Left and right common iliac veins merge to form the IVC.
Passes right of midline
Paired segmental lumbar veins drain into the IVC throughout its length
The right gonadal vein empties directly into the cava and the left gonadal vein generally empties into the left renal vein.
The next major veins are the renal veins and the hepatic veins
Pierces the central tendon of diaphragm at T8
Right atrium

Relations
Anteriorly Small bowel, first and third part of duodenum, head of pancreas, liver and bile duct, right common iliac artery, right gonadal artery
Posteriorly Right renal artery, right psoas, right sympathetic chain, coeliac ganglion

Levels
Level Vein
T8 Hepatic vein, inferior phrenic vein, pierces diaphragm
L1 Right suprarenal vein, renal vein
L2 Gonadal vein
L1-5 Lumbar veins
L5 Common iliac vein, formation of IVC

1005
Q

Which of the anaesthetic agents below is most likely to induce adrenal suppression?

Sodium thiopentone

Midazolam

Propofol

Etomidate

Ketamine

A

Etomidate is a recognised cause of adrenal suppression, this has been associated with increased mortality when used as a sedation agent in the critically ill.

Anaesthetic agents

The table below summarises some of the more commonly used IV induction agents
Agent Specific features
Propofol
Rapid onset of anaesthesia
Pain on IV injection
Rapidly metabolised with little accumulation of metabolites
Proven anti emetic properties
Moderate myocardial depression
Widely used especially for maintaining sedation on ITU, total IV anaesthesia and for daycase surgery
Sodium thiopentone
Extremely rapid onset of action making it the agent of choice for rapid sequence of induction
Marked myocardial depression may occur
Metabolites build up quickly
Unsuitable for maintenance infusion
Little analgesic effects
Ketamine
May be used for induction of anaesthesia
Has moderate to strong analgesic properties
Produces little myocardial depression making it a suitable agent for anaesthesia in those who are haemodynamically unstable
May induce state of dissociative anaesthesia resulting in nightmares
Etomidate
Has favorable cardiac safety profile with very little haemodynamic instability
No analgesic properties
Unsuitable for maintaining sedation as prolonged (and even brief) use may result in adrenal suppression
Post operative vomiting is common

1006
Q

A 63 year old man is undergoing a right pneumonectomy for carcinoma of the bronchus. As the surgeons approach the root of the lung, which structure will lie most posteriorly (in the anatomical plane)?

Phrenic nerve

Main Bronchus

Vagus nerve

Pulmonary vein

Pulmonary artery

A

The vagus nerve is the most posteriorly located structure at the lung root. The phrenic nerve lies most anteriorly.

Lung anatomy

The right lung is composed of 3 lobes divided by the oblique and transverse fissures. The left lung has two lobes divided by the oblique fissure.The apex of both lungs is approximately 4cm superior to the sterno-costal joint of the first rib. Immediately below this is a sulcus created by the subclavian artery.

Peripheral contact points of the lung
Base: diaphragm
Costal surface: corresponds to the cavity of the chest
Mediastinal surface: Contacts the mediastinal pleura. Has the cardiac impression. Above and behind this concavity is a triangular depression named the hilum, where the structures which form the root of the lung enter and leave the viscus. These structures are invested by pleura, which, below the hilum and behind the pericardial impression, forms the pulmonary ligament

Right lung
Above the hilum is the azygos vein; Superior to this is the groove for the superior vena cava and right innominate vein; behind this, and nearer the apex, is a furrow for the innominate artery. Behind the hilum and the attachment of the pulmonary ligament is a vertical groove for the oesophagus; In front and to the right of the lower part of the oesophageal groove is a deep concavity for the extrapericardiac portion of the inferior vena cava.

The root of the right lung lies behind the superior vena cava and the right atrium, and below the azygos vein.

The right main bronchus is shorter, wider and more vertical than the left main bronchus and therefore the route taken by most foreign bodies.

Left lung
Above the hilum is the furrow produced by the aortic arch, and then superiorly the groove accommodating the left subclavian artery; Behind the hilum and pulmonary ligament is a vertical groove produced by the descending aorta, and in front of this, near the base of the lung, is the lower part of the oesophagus.

The root of the left lung passes under the aortic arch and in front of the descending aorta.

Inferior borders of both lungs
6th rib in mid clavicular line
8th rib in mid axillary line
10th rib posteriorly
The pleura runs two ribs lower than the corresponding lung level.

Bronchopulmonary segments
Segment number Right lung Left lung
1 Apical Apical
2 Posterior Posterior
3 Anterior Anterior
4 Lateral Superior lingular
5 Medial Inferior lingular
6 Superior (apical) Superior (apical)
7 Medial basal Medial basal
8 Anterior basal Anterior basal
9 Lateral basal Lateral basal
10 Posterior basal Posterior basal

1007
Q

A 6 year old child develops ballooning of the foreskin on micturition and is brought to the clinic by his anxious mother. On examination, the foreskin is non retractile but otherwise normal. By which age are 95% of all foreskins retractile

2 years

16 years

8 years

5 years

10 years

A

By 16 years of age almost all foreskins should be retractile and if they are not circumcision should be considered at around this time.

Paediatric urology- foreskin disorders

Disorders of the foreskin
At birth and in the neonatal period the normal foreskin is non retractile due to the presence of adhesions between the foreskin and glans. In most cases these will separate spontaneously. By the end of puberty 95% of foreskins can be retracted. In some children the non-retractile foreskin may balloon during micturition. This is a normal variant and requires no specific treatment.

Balanitis This is inflammation of the glans penis. It may occur in both circumcised and non-circumcised individuals.
Posthitis This is inflammation of the foreskin. It may occur as a result of infections such as gonorrhoea and other STD’s. It may also complicate diabetes. Posthitis may progress to phimosis and as this may make cleaning of the glans difficult and allow progression to balanoposthitis.
Paraphimosis Prolonged retraction of the foreskin proximal to the glans may allow oedema to occur. This may then make foreskin manipulation difficult. It can usually be managed by compression to reduce the oedema and replacement of the foreskin. Where this fails a dorsal slit may be required and this followed by delayed circumcision.
Phimosis This is inability to retract the foreskin and may be partial or complete. It may occur secondary to balanoposthitis or balanitis xerotica obliterans. Depending upon the severity and symptoms treatment with circumcision may be required.
Balanitis xerotica obliterans This is a dermatological condition in which scarring of the foreskin occurs leading to phimosis. It is rare below the age of 5 years. Treatment is usually with circumcision.

1008
Q

Which of the following surgical procedures will have the greatest long term impact on a patients calcium metabolism?

Distal gastrectomy

Cholecystectomy

Extensive small bowel resection

Sub total colectomy

Gastric banding for obesity

A

Calcium is mainly absorbed from the small bowel and this will have a direct long term impact on calcium metabolism and increase the risk of osteoporosis. Gastric banding and distal gastrectomy may affect a patients dietary choices but any potential deleterious nutritional intake may be counteracted by administration of calcium supplements orally. Only 10% of calcium is absorbed from the colon so that a sub total colectomy will only have a negligible effect.

Calcium homeostasis

Calcium ions are linked to a wide range of physiological processes. The largest store of bodily calcium is contained within the skeleton. Calcium levels are primarily controlled by parathyroid hormone, vitamin D and calcitonin.

Hormonal regulation of calcium
Hormone Actions
Parathyroid hormone (PTH)
Increase calcium levels and decrease phosphate levels
Increases bone resorption
Immediate action on osteoblasts to increase ca2+ in extracellular fluid
Osteoblasts produce a protein signaling molecule that activate osteoclasts which cause bone resorption
Increases renal tubular reabsorption of calcium
Increases synthesis of 1,25(OH)2D (active form of vitamin D) in the kidney which increases bowel absorption of Ca2+
Decreases renal phosphate reabsorption
1,25-dihydroxycholecalciferol (the active form of vitamin D)
Increases plasma calcium and plasma phosphate
Increases renal tubular reabsorption and gut absorption of calcium
Increases osteoclastic activity at high levels and osteoblasts at low levels
Increases renal phosphate reabsorption
Calcitonin
Secreted by C cells of thyroid
Inhibits intestinal calcium absorption
Inhibits osteoclast activity
Inhibits renal tubular absorption of calcium

Both growth hormone and thyroxine also play a small role in calcium metabolism.

1009
Q

A 23 year old woman has undergone a pan proctocolectomy and ileoanal pouch because she suffers from familial adenomatous polyposis coli. What is the commonest extra colonic lesion in this disorder?

Gastric fundal polyps

Trichilemmomas

Duodenal polyps

Fibrocystic disease of the breast

Skull osteomas

A

Duodenal polyps occur in up to 100% of patients with FAP if follow up is continued for long enough. Duodenal cancer has an incidence of 4-10%.
Duodenal polyps are the commonest extra colonic lesion in FAP. Gastric fundal polyps are seen in 50% of patients. Skull osteomas are seen in Gardeners syndrome which is a variant of FAP.

Polyposis syndromes

Syndrome Genetic defect Features Screening and management Associated disorders
Familial adenomatous polyposis Mutation of APC gene (80%) cases, dominant Typically over 100 colonic adenomas
Cancer risk of 100%
20% are new mutations If known to be at risk then predictive genetic testing as teenager
Annual flexible sigmoidoscopy from 15 years
If no polyps found then 5 yearly colonoscopy started at age 20
Polyps found = resectional surgery (resection and pouch Vs sub total colectomy and IRA) Gastric fundal polyps (50%).
Duodenal polyps 90%.
If severe duodenal polyposis cancer risk of 30% at 10 years.
Abdominal desmoid tumours.
MYH associated polyposis Biallelic mutation of mut Y human homologue (MYH) on chromosome 1p, recessive Multiple colonic polyps
Later onset right sided cancers more common than in FAP
100% cancer risk by age 60 Once identified resection and ileoanal pouch reconstruction is recommended
Attenuated phenotype - regular colonoscopy Duodenal polyposis in 30%
Associated with increased risk of breast cancer (self examination)
Peutz -Jeghers syndrome STK11 (LKB1) mutation on chromosome 19 in some (but not all) cases, dominant Multiple benign intestinal hamartomas
Episodic obstruction and intussceception
Increased risk of GI cancers (colorectal cancer 20%, gastric 5%)
Increased risk of breast, ovarian, cervical pancreatic and testicular cancers Annual examination
Pan intestinal endoscopy every 2-3 years Malignancies at other sites
Classical pigmentation pattern
Cowden disease Mutation of PTEN gene on chromosome 10q22, dominant Macrocephaly
Multiple intestinal hamartomas
Multiple trichilemmomas
89% risk of cancer at any site
16% risk of colorectal cancer Targeted individualised screening Breast cancer (81% risk)
Thyroid cancer and non toxic goitre
Uterine cancer
HNPCC (Lynch syndrome) Germline mutations of DNA mismatch repair genes Colo rectal cancer 30-70%
Endometrial cancer 30-70%
Gastric cancer 5-10%
Scanty colonic polyps may be present
Colonic tumours likely to be right sided and mucinous Colonoscopy every 1-2 years from age 25
Consideration of prophylactic surgery
Extra colonic surveillance recommended Extra colonic cancers

1010
Q

A 43 year old typist presents with pain at the dorsal aspect of the upper part of her forearm. She also complains of weakness when extending her fingers. On examination triceps and supinator are both functioning normally. There is weakness of most of the extensor muscles. However, there is no sensory deficit. Which of the following nerves has been affected?

Anterior interosseous

Median

Posterior interosseous

Palmar cutaneous

Ulnar

A

The radial nerve may become entrapped in the arcade of Frohse which is a superficial part of the supinator muscle which overlies the posterior interosseous nerve. This nerve is entirely muscular and articular in its distribution. It passes postero-inferiorly and gives branches to extensor carpi radialis brevis and supinator. It enters supinator and curves around the lateral and posterior surfaces of the radius. On emerging from the supinator the posterior interosseous nerve lies between the superficial extensor muscles and the lowermost fibres of supinator. It then gives branches to the extensors.

Posterior interosseous nerve

Emerges from supinator between the superficial extensor muscles and lowest fibres of supinator
It gives recurrent branches which innervate extensor digitorum, extensor digiti minimi and extensor carpi ulnaris
It then passes with the posterior interosseous artery superficial to the abductor pollicis longus supplying it. It supplies branches to extensor pollicis longus and brevis and extensor indicis and ends as a small gangliform enlargement at the back of the carpus from which the intercarpal joints are supplied.

1011
Q

A 55 year old woman complains of nipple discharge. This was blood stained on one occasion. But not subsequently. Clinical examination shows clear fluid but no discrete lump. Imaging with ultrasound and mammography is normal. What is the best course of action?

Mastectomy

Total duct excision

Microdochectomy

Discharge

Mammographic surveillence

A

Although this is likely to be benign disease, her age coupled with an episode of blood stained discharge would attract a recommendation for microdochectomy. She may have an intraductal papilloma. But the concern would be DCIS.

Nipple discharge

Causes of nipple discharge
Physiological During breast feeding
Galactorrhoea Commonest cause may be response to emotional events, drugs such as histamine receptor antagonists are also implicated
Hyperprolactinaemia
Commonest type of pituitary tumour
Microadenomas <1cm in diameter
Macroadenomas >1cm in diameter
Pressure on optic chiasm may cause bitemporal hemianopia
Mammary duct ectasia
Dilatation breast ducts.
Most common in menopausal women
Discharge typically thick and green in colour
Most common in smokers
Carcinoma
Often blood stained
May be underlying mass or axillary lymphadenopathy
Intraductal papilloma
Commoner in younger patients
May cause blood stained discharge
There is usually no palpable lump

Assessment of patients
Examine breast and determine whether there is mass lesion present
All mass lesions should undergo Triple assessment.

Reporting of investigations
Where a mass lesion is suspected or investigations are requested these are prefixed using a system that denotes the investigation type e.g. M for mammography, followed by a numerical code as shown below:

1 No abnormality
2 Abnormality with benign features
3 Indeterminate probably benign
4 Indeterminate probably malignant
5 Malignant

Management of non malignant nipple discharge
Exclude endocrine disease
Nipple cytology unhelpful
Smoking cessation advice for duct ectasia
For duct ectasia with severe symptoms, total duct excision may be warranted.

1012
Q

A 56 year old lorry driver presents to the emergency department after returning from his usual week long trucking trip with excruciating pain around his anus. The patient admits to drinking beer when not working and notes that his meals usually consist of fast food. He has no allergies, takes no medications, and his clinical observations are normal. On examination, he was found to have a tender lump on the right side of his anus that measures 1 cm in diameter. The lump is bluish with surrounding oedema. It is visible without the aid of a proctoscope . It is soft and tender on palpation. The remainder of the history and physical examination are unremarkable. Which vein drains the vessels responsible for the formation of this lump?

Internal haemorrhoids

Internal pudendal

Inferior mesenteric

Inferior vena cava

Superior rectal

A

Venous drainage of hemorrhoidal tissue mirrors embryologic origin. Internal hemorrhoids drain through the superior rectal vein into the portal system. External hemorrhoids drain through the inferior rectal vein into the inferior vena cava.

Rectum

The rectum is approximately 12 cm long. It is a capacitance organ. It has both intra and extraperitoneal components. The transition between the sigmoid colon is marked by the disappearance of the tenia coli.The extra peritoneal rectum is surrounded by mesorectal fat that also contains lymph nodes. This mesorectal fatty layer is removed surgically during rectal cancer surgery (Total Mesorectal Excision). The fascial layers that surround the rectum are important clinical landmarks, anteriorly lies the fascia of Denonvilliers. Posteriorly lies Waldeyers fascia.

Extra peritoneal rectum
Posterior upper third
Posterior and lateral middle third
Whole lower third

Relations
Anteriorly (Males) Rectovesical pouch
Bladder
Prostate
Seminal vesicles
Anteriorly (Females) Recto-uterine pouch (Douglas)
Cervix
Vaginal wall
Posteriorly Sacrum
Coccyx
Middle sacral artery
Laterally Levator ani
Coccygeus

Arterial supply
Superior rectal artery
Middle rectal artery (from the internal iliac)
Inferior rectal artery (from the pudendal vessels)

Venous drainage
Superior rectal vein
Inferior rectal vein
Note the venous drainage is a site of portosystemic anastomosis.

Lymphatic drainage
Mesorectal lymph nodes (superior to dentate line)
Inguinal nodes (inferior to dentate line)

1013
Q

A baby boy is born by elective cesarean section at 39 weeks gestation. He initially seems to progress well and is discharged from hospital the following day. The parents bring the child to the clinic at 10 days of age and are concerned at the presence of a profuse and foul smelling discharge at the site of the umbilicus. On examination, the umbilicus has some prominent granulation tissue. When the baby cries a small trickle of brownish fluid is seen to pass from the umbilicus. What is the most likely cause?

Omphalitis

Patent vitello-intestinal duct

Patent urachus

Pyogenic granuloma

Umbilical hernia

A

A persistent vitello-intestinal duct may allow the persistent and ongoing discharge of small bowel content from the umbilicus. This fluid may be very irritant to the surrounding skin.

Paediatric umbilical disorders

Embryology
During development the umbilicus has two umbilical arteries and one umbilical vein. The arteries are continuous with the internal iliac arteries and the vein is continuous with the falciform ligament (ductus venosus). After birth the cord dessicates and separates and the umbilical ring closes.

Umbilical hernia
Up to 20% of neonates may have an umbilical hernia, it is more common in premature infants. The majority of these hernias will close spontaneously (may take between 12 months and three years). Strangulation is rare.

Paraumbilical hernia
These are due to defects in the linea alba that are in close proximity to the umbilicus. The edges of a paraumbilical hernia are more clearly defined than those of an umbilical hernia. They are less likely to resolve spontaneously than an umbilical hernia.

Omphalitis
This condition consists of infection of the umbilicus. Infection with Staphylococcus aureus is the commonest cause. The condition is potentially serious as infection may spread rapidly through the umbilical vessels in neonates with a risk of portal pyaemia, and portal vein thrombosis. Treatment is usually with a combination of topical and systemic antibiotics.

Umbilical granuloma
These consist of cherry red lesions surrounding the umbilicus, they may bleed on contact and be a site of seropurulent discharge. Infection is unusual and they will often respond favorably to chemical cautery with topically applied silver nitrate.

Persistent urachus
This is characterised by urinary discharge from the umbilicus. It is caused by persistence of the urachus which attaches to the bladder. They are associated with other urogenital abnormalities.

Persistent vitello-intestinal duct
This will typically present as an umbilical discharge that discharges small bowel content. Complete persistence of the duct is a rare condition. Much more common is the persistence of part of the duct (Meckels diverticulum). Persistent vitello-intestinal ducts are best imaged using a contrast study to delineate the anatomy and are managed by laparotomy and surgical closure.

1014
Q

A 70 year old lady presents with an upper gastrointestinal haemorrhage. This surprises the surgical team as she has been taking a proton pump inhibitor at high dose since a recent admission with epigastric pain. At endoscopy, the duodenum is found to contain multiple ulcers. What is the most likely explanation?

Infection with helicobacter pylori

Gastrinoma

Duodenal lymphoma

Adenocarcinoma of the duodenum

Alcoholism

A

Gastrinomas are typically the source of gastrin in Zollinger Ellison syndrome.

Upper gastrointestinal bleeding

Patients may present with the following:
Haematemesis and/ or malaena
Epigastric discomfort
Sudden collapse

The extent to which these will occur will depend upon the source. Mortality is higher in patients presenting with haematemesis than malaena alone.

Oesophageal bleeding
Cause Presenting features
Oesophagitis Small volume of fresh blood, often streaking vomit. Malaena rare. Often ceases spontaneously. Usually history of antecedent GORD type symptoms.
Cancer Usually small volume of blood, except as pre terminal event with erosion of major vessels. Often associated symptoms of dysphagia and constitutional symptoms such as weight loss. May be recurrent until malignancy managed.
Mallory Weiss Tear Typically brisk small to moderate volume of bright red blood following bout of repeated vomiting. Malaena rare. Usually ceases spontaneously.
Varices Usually large volume of fresh blood. Swallowed blood may cause malaena. Often associated with haemodynamic compromise. May stop spontaneously but re-bleeds are common until appropriately managed.

Gastric Bleeding
Cause Presenting features
Gastric cancer May be frank haematemesis or altered blood mixed with vomit. Usually prodromal features of dyspepsia and may have constitutional symptoms. Amount of bleeding variable but erosion of major vessel may produce considerable haemorrhage.
Dieulafoy Lesion Often no prodromal features prior to haematemesis and malaena, but this arteriovenous malformation may produce quite considerable haemorrhage and may be difficult to detect endoscopically.
Diffuse erosive gastritis Usually haematemesis and epigastric discomfort. Usually there is an underlying cause such as recent NSAID usage. Large volume haemorrhage may occur with considerable haemodynamic compromise.
Gastric ulcer Small low volume bleeds more common so would tend to present as iron deficiency anaemia. Erosion into a significant vessel may produce considerable haemorrhage and haematemesis.

Duodenum
Most common cause of major haemorrhage is a posteriorly sited duodenal ulcer. However, ulcers at any site in the duodenum may present with haematemesis, malaena and epigastric discomfort. The pain of duodenal ulcer is slightly different to that of gastric ulcers and often occurs several hours after eating. Peri ampullary tumours may bleed but these are rare. In patients with previous abdominal aortic aneurysm surgery aorto-enteric fistulation remains a rare but important cause of major haemorrhage associated with high mortality.

Management
Admission to hospital careful monitoring, cross match blood, check FBC, LFTs, U+E and Clotting (as a minimum)
Patients with on-going bleeding and haemodynamic instability are likely to require O negative blood pending cross matched blood
Early control of airway is vital (e.g. Drowsy patient with liver failure)
Patients with suspected varices should receive terlipressin prior to endoscopy
Ideally all patients admitted with upper gastrointestinal haemorrhage should undergo Upper GI endoscopy within 24 hours of admission. In those who are unstable this should occur immediately after resuscitation or in tandem with it. The endoscopy department is a potentially dangerous place for unstable patients and it may be safer to perform the endoscopy in theatre with an anaesthetist present.
Varices should be banded or subjected to sclerotherapy. If this is not possible owing to active bleeding then a Sengstaken- Blakemore tube (or Minnesota tube) should be inserted. This should be done with care; gastric balloon should be inflated first and oesophageal balloon second. Remember the balloon will need deflating after 12 hours (ideally sooner) to prevent necrosis. Portal pressure should be lowered by combination of medical therapy +/- TIPSS.
Patients with erosive oesophagitis / gastritis should receive a proton pump inhibitor.
Mallory Weiss tears will typically resolve spontaneously
Identifiable bleeding points should receive combination therapy of injection of adrenaline and either a thermal or mechanical treatment. All who have received intervention should receive a continuous infusion of a proton pump inhibitor (IV omeprazole for 72 hours) to reduce the re-bleeding rate.
Patients with diffuse erosive gastritis who cannot be managed endoscopically and continue to bleed may require gastrectomy
Bleeding ulcers that cannot be controlled endoscopically may require laparotomy and ulcer underruning

Indications for surgery
Patients > 60 years
Continued bleeding despite endoscopic intervention
Recurrent bleeding
Known cardiovascular disease with poor response to hypotension

Surgery
Duodenal ulcer
Laparotomy, duodenotomy and under running of the ulcer. If bleeding is brisk then the ulcer is almost always posteriorly sited and will have invaded the gastroduodenal artery. Large bites using 0 Vicryl are taken above and below the ulcer base to occlude the vessel. The duodenotomy should be longitudinal but closed transversely to avoid stenosis.

For gastric ulcer
Under-running of the bleeding site
Partial gastrectomy-antral ulcer
Partial gastrectomy or under running the ulcer- lesser curve ulcer (involving left gastric artery)
Total gastrectomy if bleeding persists

Summary of Acute Upper GI bleeding recommendations:
The need for admission and timing of endoscopic intervention may be predicted by using the Blatchford score. This considers a patients Hb, serum urea, pulse rate and blood pressure. Those patients with a score of 0 are low risk, all others are considered high risk and require admission and endoscopy.
The requirement for pre endoscopic proton pump inhibition is contentious. In the UK the National Institute of Clinical Excellence guidelines suggest the pre endoscopic PPI therapy is unnecessary. Whilst it is accepted that such treatment has no impact on mortality or morbidity a Cochrane review of this practice in 2007 did suggest that it reduced the stigmata of recent haemorrhage at endoscopy. As a result many will still administer PPI to patients prior to endoscopic intervention.
Following endoscopy it is important to calculate the Rockall score for patients to determine their risk of rebleeding and mortality. A score of 3 or less is associated with a rebleeding rate of 4% and a very low risk of mortality and identifies a group of patients suitable for early discharge.

References
1. http://www.sign.ac.uk/guidelines/fulltext/105/index.html
2. Joint Advisory Group on Endoscopy (JAG) Guidelines - http://www.thejag.org.uk
3. NICE Guideline: Management of acute upper GI bleeding. July 2012.

1015
Q

Transection of the radial nerve at the level of the axilla will result in all of the following except:

Loss of elbow extension.

Loss of extension of the interphalangeal joints.

Loss of metacarpophalangeal extension.

Loss of triceps reflex.

Loss of sensation overlying the first dorsal interosseous.

A

B- These may still extend by virtue of retained lumbrical muscle function.
Radial nerve

Continuation of posterior cord of the brachial plexus (root values C5 to T1)

Path
In the axilla: lies posterior to the axillary artery on subscapularis, latissimus dorsi and teres major.
Enters the arm between the brachial artery and the long head of triceps (medial to humerus).
Spirals around the posterior surface of the humerus in the groove for the radial nerve.
At the distal third of the lateral border of the humerus it then pierces the intermuscular septum and descends in front of the lateral epicondyle.
At the lateral epicondyle it lies deeply between brachialis and brachioradialis where it then divides into a superficial and deep terminal branch.
Deep branch crosses the supinator to become the posterior interosseous nerve.

egions innervated
Motor (main nerve)
Triceps
Anconeus
Brachioradialis
Extensor carpi radialis
Motor (posterior interosseous branch)
Supinator
Extensor carpi ulnaris
Extensor digitorum
Extensor indicis
Extensor digiti minimi
Extensor pollicis longus and brevis
Abductor pollicis longus
Sensory The area of skin supplying the proximal phalanges on the dorsal aspect of the hand is supplied by the radial nerve (this does not apply to the little finger and part of the ring finger)

Muscular innervation and effect of denervation
Anatomical location Muscle affected Effect of paralysis
Shoulder Long head of triceps Minor effects on shoulder stability in abduction
Arm Triceps Loss of elbow extension
Forearm Supinator
Brachioradialis
Extensor carpi radialis longus and brevis Weakening of supination of prone hand and elbow flexion in mid prone position

1016
Q

Where are the arterial baroreceptors located?

Carotid sinus and aortic arch

Carotid sinus only

Superior vena cava

External carotid artery

None of the above

A

They lie in the carotid sinus and aortic arch.

Cardiac physiology

  • The heart has four chambers ejecting blood into both low pressure and high pressure systems.
    The pumps generate pressures of between 0-25mmHg on the right side and 0-120 mmHg on the left.
    At rest diastole comprises 2/3 of the cardiac cycle.
    The product of the frequency of heart rate and stroke volume combine to give the cardiac output which is typically 5-6L per minute.

Detailed descriptions of the various waveforms are often not a feature of MRCS A (although they are on the syllabus). However, they are a very popular topic for surgical physiology in the MRCS B exam.

Electrical properties
Intrinsic myogenic rhythm within cardiac myocytes means that even the denervated heart is capable of contraction.
In the normal situation the cardiac impulse is generated in the sino atrial node in the right atrium and conveyed to the ventricles via the atrioventricular node.
The sino atrial node is also capable of spontaneous discharge and in the absence of background vagal tone will typically discharge around 100x per minute. Hence the higher resting heart rate found in cardiac transplant cases. In the SA and AV nodes the resting membrane potential is lower than in surrounding cardiac cells and will slowly depolarise from -70mV to around -50mV at which point an action potential is generated.
Differences in the depolarisation slopes between SA and AV nodes help to explain why the SA node will depolarise first. The cells have a refractory period during which they cannot be re-stimulated and this period allows for adequate ventricular filling. In pathological tachycardic states this time period is overridden and inadequate ventricular filling may then occur, cardiac output falls and syncope may ensue.

Parasympathetic fibres project to the heart via the vagus and will release acetylcholine. Sympathetic fibres release nor adrenaline and circulating adrenaline comes from the adrenal medulla. Noradrenaline binds to β 1 receptors in the SA node and increases the rate of pacemaker potential depolarisation.

Cardiac cycle

Image sourced from Wikipedia

Mid diastole: AV valves open. Ventricles hold 80% of final volume. Outflow valves shut. Aortic pressure is high.

Late diastole: Atria contract. Ventricles receive 20% to complete filling. Typical end diastolic volume 130-160ml.

Early systole: AV valves shut. Ventricular pressure rises. Isovolumetric ventricular contraction. AV Valves bulge into atria (c-wave). Aortic and pulmonary pressure exceeded- blood is ejected. Shortening of ventricles pulls atria downwards and drops intra atrial pressure (x-descent).

Late systole: Ventricular muscles relax and ventricular pressures drop. Although ventricular pressure drops the aortic pressure remains constant owing to peripheral vascular resistance and elastic property of the aorta. Brief period of retrograde flow that occurs in aortic recoil shuts the aortic valve. Ventricles will contain 60ml end systolic volume. The average stroke volume is 70ml (i.e. Volume ejected).

Early diastole: All valves are closed. Isovolumetric ventricular relaxation occurs. Pressure wave associated with closure of the aortic valve increases aortic pressure. The pressure dip before this rise can be seen on arterial waveforms and is called the incisura. During systole the atrial pressure increases such that it is now above zero (v- wave). Eventually atrial pressure exceed ventricular pressure and AV valves open - atria empty passively into ventricles and atrial pressure falls (y -descent )

The negative atrial pressures are of clinical importance as they can allow air embolization to occur if the neck veins are exposed to air. This patient positioning is important in head and neck surgery to avoid this occurrence if veins are inadvertently cut, or during CVP line insertion.

Mechanical properties
Preload = end diastolic volume
Afterload = aortic pressure

It is important to understand the principles of Laplace’s law in surgery.
It states that for hollow organs with a circular cross section, the total circumferential wall tension depends upon the circumference of the wall, multiplied by the thickness of the wall and on the wall tension.
The total luminal pressure depends upon the cross sectional area of the lumen and the transmural pressure. Transmural pressure is the internal pressure minus external pressure and at equilibrium the total pressure must counterbalance each other.
In terms of cardiac physiology the law explains that the rise in ventricular pressure that occurs during the ejection phase is due to physical change in heart size. It also explains why a dilated diseased heart will have impaired systolic function.

Starlings law
Increase in end diastolic volume will produce larger stroke volume.
This occurs up to a point beyond which cardiac fibres are excessively stretched and stroke volume will fall once more. It is important for the regulation of cardiac output in cardiac transplant patients who need to increase their cardiac output.

Baroreceptor reflexes
Baroreceptors located in aortic arch and carotid sinus.
Aortic baroreceptor impulses travel via the vagus and from the carotid via the glossopharyngeal nerve.
They are stimulated by arterial stretch.
Even at normal blood pressures they are tonically active.
Increase in baroreceptor discharge causes:

*Increased parasympathetic discharge to the SA node.
*Decreased sympathetic discharge to ventricular muscle causing decreased contractility and fall in stroke volume.
*Decreased sympathetic discharge to venous system causing increased compliance.
*Decreased peripheral arterial vascular resistance

Atrial stretch receptors
Located in atria at junction between pulmonary veins and vena cava.
Stimulated by atrial stretch and are thus low pressure sensors.
Increased blood volume will cause increased parasympathetic activity.
Very rapid infusion of blood will result in increase in heart rate mediated via atrial receptors: the Bainbridge reflex.
Decreases in receptor stimulation results in increased sympathetic activity this will decrease renal blood flow-decreases GFR-decreases urinary sodium excretion-renin secretion by juxtaglomerular apparatus-Increase in angiotensin II.
Increased atrial stretch will also result in increased release of atrial natriuretic peptide.

1017
Q

A 3 month old boy is brought to the clinic by his mother who has noticed a swelling in the right hemiscrotum. On examination, there is a firm mass affecting the right spermatic cord distally, the testis is felt separately from it. What is the most likely diagnosis?

Inguino scrotal hernia

Rhabdomyosarcoma

Leydig cell tumour

Torsion of testicular hydatid

Hydrocele

A

Rhabdomyosarcoma are paratesticular tumours with a bimodal distribution. Because the mass is felt separate to the testis, this is the more likely diagnosis.
5% of testicular tumors
Most often arises in distal portion of spermatic cord and may invade testis of surrounding tissues
60% occur in the first 2 decades of life
Bimodal age distribution - 3-4 months - 16 years
Arises from mesenchymal tissue - 90% embryonal variant (better prognosis) - 30% - 50% have metastasis (usually lymph node) at diagnosis

Testicular disorders

Testicular cancer
Testicular cancer is the most common malignancy in men aged 20-30 years. Around 95% of cases of testicular cancer are germ-cell tumours. Germ cell tumours may essentially be divided into:

Tumour type Key features Tumour markers Pathology
Seminoma
Commonest subtype (50%)
Average age at diagnosis = 40
Even advanced disease associated with 5 year survival of 73%
AFP usually normal
HCG elevated in 10% seminomas
Lactate dehydrogenase; elevated in 10-20% seminomas (but also in many other conditions)
Sheet like lobular patterns of cells with substantial fibrous component. Fibrous septa contain lymphocytic inclusions and granulomas may be seen.
Non seminomatous germ cell tumours (42%)
Teratoma
Yolk sac tumour
Choriocarcinoma
Mixed germ cell tumours (10%)
Younger age at presentation =20-30 years
Advanced disease carries worse prognosis (48% at 5 years)
Retroperitoneal lymph node dissection may be needed for residual disease after chemotherapy
AFP elevated in up to 70% of cases
HCG elevated in up to 40% of cases
Other markers rarely helpful
Heterogenous texture with occasional ectopic tissue such as hair

Risk factors for testicular cancer
Cryptorchidism
Infertility
Family history
Klinefelter’s syndrome
Mumps orchitis

Features
A painless lump is the most common presenting symptom
Pain may also be present in a minority of men
Other possible features include hydrocele, gynaecomastia

Diagnosis
Ultrasound is first-line
CT scanning of the chest/ abdomen and pelvis is used for staging
Tumour markers (see above) should be measured

Management
Orchidectomy (Inguinal approach)
Chemotherapy and radiotherapy may be given depending on staging
Abdominal lesions >1cm following chemotherapy may require retroperitoneal lymph node dissection.

Prognosis is generally excellent
5 year survival for seminomas is around 95% if Stage I
5 year survival for teratomas is around 85% if Stage I

Benign disease

Epididymo-orchitis
Acute epididymitis is an acute inflammation of the epididymis, often involving the testis and usually caused by bacterial infection.
Infection spreads from the urethra or bladder. In men <35 years, gonorrhoea or chlamydia are the usual infections.
Amiodarone is a recognised non infective cause of epididymitis, which resolves on stopping the drug.
Tenderness is usually confined to the epididymis, which may facilitate differentiating it from torsion where pain usually affects the entire testis.

Testicular torsion
Twist of the spermatic cord resulting in testicular ischaemia and necrosis.
Most common in males aged between 10 and 30 (peak incidence 13-15 years)
Pain is usually severe and of sudden onset.
Cremasteric reflex is lost and elevation of the testis does not ease the pain.
Treatment is with surgical exploration. If a torted testis is identified then both testis should be fixed as the condition of bell clapper testis is often bilateral.

Hydrocele
Presents as a mass that transilluminates, usually possible to ‘get above’ it on examination.
In younger men it should be investigated with USS to exclude tumour.
In children it may occur as a result of a patent processus vaginalis.
Treatment in adults is with a Lords or Jabouley procedure.
Treatment in children is with trans inguinal ligation of PPV.

1018
Q

Which of the following statements is false in relation to renal adenocarcinoma?

They account for over 75% cases of renal tumours

Renal biopsy should be performed in all cases considered for radical nephrectomy

They typically spread via the haematogenous route

Patients with completely resected T2 disease should not receive adjuvant chemotherapy

Partial nephrectomy gives equivalent oncological outcomes in patients with T1 disease

A

Routine chemotherapy is not effective in patients with renal adenocarcinoma and should not be used following R0 resections.
Routine renal biopsy should not be performed in cases for nephrectomy. Most cases of malignancy can be accurately classified on imaging.

Renal tumours

Renal cell carcinoma
Renal cell carcinoma is an adenocarcinoma of the renal cortex and is believed to arise from the proximal convoluted tubule. They are usually solid lesions, up to 20% may be multifocal, 20% may be calcified and 20% may have either a cystic component or be wholly cystic. They are often circumscribed by a pseudocapsule of compressed normal renal tissue. Spread may occur either by direct extension into the adrenal gland, renal vein or surrounding fascia. More distant disease usually occurs via the haematogenous route to lung, bone or brain.
Renal cell carcinoma comprise up to 85% of all renal malignancies. Males are more commonly affected than females and sporadic tumours typically affect patients in their sixth decade.
Patients may present with a variety of symptoms including; haematuria (50%), loin pain (40%), mass (30%) and up to 25% may have symptoms of metastasis.Less than 10% have the classic triad of haematuria, pain and mass.

Investigation
Many cases will present as haematuria and be discovered during diagnostic work up. Benign renal tumours are rare, so renal masses should be investigated with multislice CT scanning. Most tumours are also characterised with an arterial phase CT, particularly if they may be suitable for partial nephrectomy.

CT scanning of the chest and abdomen to detect distant disease should also be undertaken.

Routine bone scanning is not indicated in the absence of symptoms.

Biopsy should not be performed when a nephrectomy is planned but is mandatory before any ablative therapies are undertaken.

Assessment of the functioning of the contra lateral kidney.

Management
T1 lesions may be managed by partial nephrectomy and this gives equivalent oncological results to total radical nephrectomy. Partial nephrectomy may also be performed when there is inadequate reserve in the remaining kidney. Ablative techniques may also be considered for small T1 lesions in unfit patients. In general, outcomes are less favorable than with surgical resection.

For T2 lesions and above a radical nephrectomy is standard practice and this may be performed via a laparoscopic or open approach. Preoperative embolisation is not indicated nor is resection of uninvolved adrenal glands. During surgery early venous control is mandatory to avoid shedding of tumour cells into the circulation.

Patients with completely resected disease do not benefit from adjuvant therapy with either chemotherapy or biological agents. These should not be administered outside the setting of clinical trials.

Patients with transitional cell cancer will require a nephroureterectomy with disconnection of the ureter at the bladder.

References
Capitanio U, Montorsi F. Renal cancer. Lancet. 2016 Feb 27;387(10021):894-906.

Capitanio U et al. Epidemiology of Renal Cell Carcinoma. Eur Urol. 2019 Jan;75(1):74-84.

1019
Q

A 24 year old lady from Western India presents with symptoms of lethargy and dizziness, worse on turning her head. On examination, her blood pressure is 176/128. Her pulses are impalpable at all peripheral sites. Auscultation of her chest reveals a systolic heart murmur. What is the most likely diagnosis?

Subclavian steal syndrome

Aortic coarctation

Patent ductus arteriosus

Aortic dissection

Takayasu’s arteritis

A

Takayasu’s arteritis most commonly affects young Asian females. Pulseless peripheries are a classical finding. The CNS symptoms may be variable.

Vascular disease

Aortic dissection
Chest pain (anterior chest pain- ascending aorta, back pain - descending aorta)
Widening of aorta on chest x-ray
Diagnosis made by CT scanning
Treatment is either medical (Type B disease) or surgical (Type A disease)
Cervical rib
Supernumery fibrous band arising from seventh cervical vertebra
Incidence of 1 in 500
May cause thoracic outlet syndrome
Treatment involves surgical division of rib
Subclavian steal syndrome
Due to proximal stenotic lesion of the subclavian artery
Results in retrograte flow through vertebral or internal thoracic arteries
The result is that decrease in cerebral blood flow may occur and produce syncopal symptoms
A duplex scan and/ or angiogram will delineate the lesion and allow treatment to be planned
Takayasu’s arteritis
Large vessel granulomatous vasculitis
Results in intimal narrowing
Most commonly affects young asian females
Patients present with features of mild systemic illness, followed by pulseless phase with symptoms of vascular insufficiency
Treatment is with systemic steroids
Patent ductus arteriosus
Ductus arteriosus is a normal foetal vessel that closes spontaneously after birth
Results in high pressure, oxygenated blood entering the pulmonary circuit
Untreated patients develop symptoms of congestive cardiac failure
Coarctation of the aorta
Aortic stenosis at the site of the ductus arteriosus insertion
More prevalent in boys or females with Turners syndrome
Patients may present with symptoms of arterial insufficiency, such as syncope and claudication
Blood pressure mismatch may be seen, as may mismatch of pulse pressure in the upper and lower limbs
Treatment is either with angioplasty or surgical resection (the former is the most common)

1020
Q

Which of the following relationship descriptions regarding the scalene muscles is incorrect?

The brachial plexus passes anterior to the middle scalene muscle

The phrenic nerve lies anterior to the anterior scalene muscle

The subclavian artery passes posterior to the middle scalene

The subclavian vein lies anterior to the anterior scalene muscle at the level of the first rib

The anterior scalene inserts into the first rib

A

The subclavian artery passes anterior to the middle scalene.

Scalene muscles

The 3 paired muscles are:
Scalenus anterior: Elevate 1st rib and laterally flex the neck to same side
Scalenus medius: Same action as scalenus anterior
Scalenus posterior: Elevate 2nd rib and tilt the cervical spine

Innervation Spinal nerves C4-6
Origin Transverse processes C2 to C7
Insertion First and second ribs
Important relations
The brachial plexus and subclavian artery pass between the anterior and middle scalenes through a space called the scalene hiatus/fissure.
The subclavian vein and phrenic nerve pass anteriorly to the anterior scalene as it crosses over the first rib.

Thoracic outlet syndrome
The scalenes are at risk of adhering to the fascia surrounding the brachial plexus or shortening causing compression of the brachial plexus when it passes between the clavicle and 1st rib causing thoracic outlet syndrome.

1021
Q

A 62 year old man presents after his wife commented on the unusual shape of his fingers. On examination, he has a hard swelling adjacent to the distal interphalangeal joint of his index finger of the right hand with lateral deviation of the finger tip. There is no sensory disturbance and the swelling is not tender. Which pathological process underpins the underlying diagnosis?

Rheumatoid arthritis

Osteoarthritis

Infection with atypical organisms

Deposition of immune complexes

Malignancy
Malignancy

A

The description fits with Heberdens nodes. These are bony outgrowths that occur in the distal interphalangeal joint in association with osteoarthritis. They may skew the finger tip sideways. Bouchards nodes are similar, but affect the proximal interphalangeal joint.

Hand diseases

Dupuytrens contracture
Fixed flexion contracture of the hand where the fingers bend towards the palm and cannot be fully extended.
Caused by underlying contractures of the palmar aponeurosis . The ring finger and little finger are the fingers most commonly affected. The middle finger may be affected in advanced cases, but the index finger and the thumb are nearly always spared.
Progresses slowly and is usually painless. In patients with this condition, the tissues under the skin on the palm of the hand thicken and shorten so that the tendons connected to the fingers cannot move freely. The palmar aponeurosis becomes hyperplastic and undergoes contracture.
Commonest in males over 40 years of age.
Association with liver cirrhosis and alcoholism. However, many cases are idiopathic.
Treatment is surgical and involves fasciectomy. However, the condition may recur and many surgical therapies are associated with risk of neurovascular damage to the digital nerves and arteries.

Carpal tunnel syndrome
Idiopathic median neuropathy at the carpal tunnel.
Characterised by altered sensation of the lateral 3 fingers.
The condition is commoner in females and is associated with other connective tissue disorders such as rheumatoid disease. It may also occur following trauma to the distal radius.
Symptoms occur mainly at night in early stages of the condition.
Examination may demonstrate wasting of the muscles of the thenar eminence and symptoms may be reproduced by Tinels test (compression of the contents of the carpal tunnel).
Formal diagnosis is usually made by electrophysiological studies.
Treatment is by surgical decompression of the carpal tunnel, a procedure achieved by division of the flexor retinaculum. Non - surgical options include splinting and bracing.

Miscellaneous hand lumps
Osler’s nodes Osler’s nodes are painful, red, raised lesions found on the hands and feet. They are the result of the deposition of immune complexes.
Bouchard’s nodes Hard, bony outgrowths or gelatinous cysts on the proximal interphalangeal joints (the middle joints of fingers or toes.) They are a sign of osteoarthritis, and are caused by formation of calcific spurs of the articular cartilage.
Heberden’s nodes Typically develop in middle age, beginning either with a chronic swelling of the affected joints or the sudden painful onset of redness, numbness, and loss of manual dexterity. This initial inflammation and pain eventually subsides, and the patient is left with a permanent bony outgrowth that often skews the fingertip sideways. It typically affects the DIP joint.
Ganglion Swelling in association with a tendon sheath commonly near a joint. They are common lesions in the wrist and hand. Usually they are asymptomatic and cause little in the way of functional compromise. They are fluid filled although the fluid is similar to synovial fluid it is slightly more viscous. When the cysts are troublesome they may be excised.

1022
Q

A 52 year old male presents with central chest pain. On examination, he has an mitral regurgitation murmur. An ECG shows ST elevation in leads V1 to V6. There is no ST elevation in leads II, III and aVF. What is the diagnosis?

Pulmonary embolism

Boerhaaves syndrome

Inferior myocardial infarct

Prinzmetal angina

Anterior myocardial infarct

A

The most likely diagnosis is an anterior MI. As there are no ST changes in the inferior leads, aortic dissection is less likely.

Chest pain

Aortic dissection
This occurs when there is a flap or filling defect within the aortic intima. Blood tracks into the medial layer and splits the tissues with the subsequent creation of a false lumen. It most commonly occurs in the ascending aorta or just distal to the left subclavian artery (less common). It is most common in Afro-carribean males aged 50-70 years.
Patients usually present with a tearing intrascapular pain, which may be similar to the pain of a myocardial infarct.
The dissection may spread either proximally or distally with subsequent disruption to the arterial branches that are encountered.
In the Stanford classification system the disease is classified into lesions with a proximal origin (Type A) and those that commence distal to the left subclavian (Type B).
Diagnosis may be suggested by a chest x-ray showing a widened mediastinum. Confirmation of the diagnosis is usually made by use of CT angiography
Proximal (Type A) lesions are usually treated surgically, type B lesions are usually managed non operatively.

Pulmonary embolism
Typically sudden onset of chest pain, haemoptysis, hypoxia and small pleural effusions may be present.
Most patients will have an underlying deep vein thrombosis
Diagnosis may be suggested by various ECG findings including S waves in lead I, Q waves in lead III and inverted T waves in lead III. Confirmation of the diagnosis is usually made through use of CT pulmonary angiography.
Treatment is with anticoagulation, in those patients who develop a cardiac arrest or severe compromise from their PE, consideration may be given to thrombolysis.

Myocardial infarction
Traditionally described as sudden onset of central, crushing chest pain. It may radiate into the neck and down the left arm. Signs of autonomic dysfunction may be present. The presenting features may be atypical in the elderly and those with diabetes.
Diagnosis is made through identification of new and usually dynamic ECG changes (and cardiac enzyme changes). Inferior and anterior infarcts may be distinguished by the presence of specific ECG changes (usually II, III and aVF for inferior, leads V1-V5 for anterior).
Treatment is with oral antiplatelet agents, primary coronary angioplasty and/ or thrombolysis.

Perforated peptic ulcer
Patients usually develop sudden onset of epigastric abdominal pain, it may be soon followed by generalised abdominal pain.
There may be features of antecendant abdominal discomfort, the pain of gastric ulcer is typically worse immediately after eating.
Diagnosis may be made by erect chest x-ray which may show a small amount of free intra-abdominal air (very large amounts of air are more typically associated with colonic perforation).
Treatment is usually with a laparotomy, small defects may be excised and overlaid with an omental patch, larger defects are best managed with a partial gastrectomy.

Boerhaaves syndrome
Spontaneous rupture of the oesophagus that occurs as a result of repeated episodes of vomiting.
The rupture is usually distally sited and on the left side.
Patients usually give a history of sudden onset of severe chest pain that may complicate severe vomiting.
Severe sepsis occurs secondary to mediastinitis.
Diagnosis is CT contrast swallow.
Treatment is with thoracotomy and lavage, if less than 12 hours after onset then primary repair is usually feasible, surgery delayed beyond 12 hours is best managed by insertion of a T tube to create a controlled fistula between oesophagus and skin.
Delays beyond 24 hours are associated with a very high mortality rate.

1023
Q

A 52 year old male is referred to urology clinic with impotence. He is known to have hypertension. He does not have any morning erections. On further questioning the patient reports pain in his buttocks, this worsens on mobilising. On examination there is some muscle atrophy. The penis and scrotum are normal. What is the most likely diagnosis?

Leriche syndrome

S3-S4 cord lesion

Pudendal nerve lesion

Psychological impotence

Beta blocker induced impotence

A

Leriche syndrome

Classically, it is described in male patients as a triad of symptoms:

  1. Claudication of the buttocks and thighs
  2. Atrophy of the musculature of the legs
  3. Impotence (due to paralysis of the L1 nerve)
    Leriche syndrome, is atherosclerotic occlusive disease involving the abdominal aorta and/or both of the iliac arteries. Management involves correcting underlying risk factors such as hypercholesterolaemia and stopping smoking. Investigation is usually with angiography.

Leriche syndrome

Atheromatous disease involving the iliac vessels. Blood flow to the pelvic viscera is compromised. Patients may present with buttock claudication and impotence (in this particular syndrome). Diagnostic work up will include angiography, where feasible, iliac occlusions are usually treated with endovascular angioplasty and stent insertion.

1024
Q

A 32 year old lady complains of carpal tunnel syndrome. The carpal tunnel is explored surgically. Which of the following structures will lie in closest proximity to the hamate bone within the carpal tunnel?

The tendon of abductor pollicis longus

The tendons of flexor digitorum profundus

The tendons of flexor carpi radialis longus

Median nerve

Radial artery

A

The carpal tunnel contains nine flexor tendons:
Flexor digitorum profundus
Flexor digitorum superficialis
Flexor pollicis longus

The tendon of flexor digitorum profundus lies deepest in the tunnel and will thus lie nearest to the hamate bone.

Carpal bones

The wrist is comprised of 8 carpal bones, these are arranged in two rows of 4. It is convex from side to side posteriorly and concave anteriorly.
Diagrammatic image of carpal bones

Key to image
A Scaphoid
B Lunate
C Triquetrum
D Pisiform
E Trapezium
F Trapezoid
G Capitate
H Hamate
1 Radius
2 Ulna
3 Metacarpals

No tendons attach to: Scaphoid, lunate, triquetrum (stabilised by ligaments)

1025
Q

Which of the following stimulates gastric acid secretion?

Cholecystokinin

Gastric inhibitory peptide

Secretin

Histamine

Somatostatin

A

Gastrin: From G cells: stimulates gastric acid production

Pepsin: Digestion of protein, secretion occurs simultaneously with gastrin

Secretin: From mucosal cells in the duodenum and jejunum: inhibits gastric acid, stimulates bile and pancreatic juice production

Gastric inhibitory peptide: (produced in response to fatty acids) inhibits gastrin release and acid secretion from parietal cells

Cholecystokinin: From mucosal cells in the duodenum and jejunum (produced in response to fatty acids) inhibits acid secretion from parietal cells, causes gallbladder contraction and relaxation of sphincter of Oddi

Somatostatin: From D cells

Histamine released from enterochromaffin like cells then stimulates the acid-making cells (parietal cells) in a paracrine manner to increase gastric acid production.

Hormonal control of gastric acid secretion

Source Stimulus Actions
Gastrin
G cells in antrum of the stomach
Distension of stomach, extrinsic nerves
Inhibited by: low antral pH, somatostatin
Increase HCL, pepsinogen and IF secretion, increases gastric motility, trophic effect on gastric mucosa
CCK
I cells in upper small intestine
Partially digested proteins and triglycerides
Increases secretion of enzyme-rich fluid from pancreas, contraction of gallbladder and relaxation of sphincter of Oddi, decreases gastric emptying, trophic effect on pancreatic acinar cells, induces satiety
Secretin
S cells in upper small intestine
Acidic chyme, fatty acids
Increases secretion of bicarbonate-rich fluid from pancreas and hepatic duct cells, decreases gastric acid secretion, trophic effect on pancreatic acinar cells
VIP
Small intestine, pancreas
Neural
Stimulates secretion by pancreas and intestines, inhibits acid and pepsinogen secretion
Somatostatin
D cells in the pancreas and stomach
Fat, bile salts and glucose in the intestinal lumen
Decreases acid and pepsin secretion, decreases gastrin secretion, decreases pancreatic enzyme secretion, decreases insulin and glucagon secretion
inhibits trophic effects of gastrin, stimulates gastric mucous production

1026
Q

A man develops an infection in his external ear. The infection is extremely painful. Which of the following nerves conveys sensation from this region?

Occipital branch of the trigeminal nerve

Vestibulocochlear nerve

Glossopharyngeal nerve

Auriculotemporal nerve

Maxillary branch of the trigeminal nerve

A

Tensor tympania and stapedius are the only two muscles of the middle ear. Contraction of tensor tympani will tend to dampen the vibrations produced by loud sounds, it is innervated by a branch of the trigeminal nerve. The stapedius dampens movements of the ossicles in response to loud sounds and is innervated by a branch of the facial nerve.
The auriculotemporal nerve, which is derived from the mandibular branch of the trigeminal nerve supplies this area.
Some areas may also be innervated by the vagus.

Ear- anatomy

The ear is composed of three anatomically distinct regions.

External ear
Auricle is composed of elastic cartilage covered by skin. The lobule has no cartilage and contains fat and fibrous tissue.

External auditory meatus is approximately 2.5cm long.
Lateral third of the external auditory meatus is cartilaginous and the medial two thirds is bony.

The region is innervated by the greater auricular nerve. The auriculotemporal branch of the trigeminal nerve supplies most of the external auditory meatus and the lateral surface of the auricle.

Middle ear
Space between the tympanic membrane and cochlea. The aditus leads to the mastoid air cells is the route through which middle ear infections may cause mastoiditis. Anteriorly the eustacian tube connects the middle ear to the naso pharynx.
The tympanic membrane consists of:
Outer layer of stratified squamous epithelium.
Middle layer of fibrous tissue.
Inner layer of mucous membrane continuous with the middle ear.
The tympanic membrane is approximately 1cm in diameter.
The chorda tympani nerve passes on the medial side of the pars flaccida.

The middle ear is innervated by the glossopharyngeal nerve and pain may radiate to the middle ear following tonsillectomy.

Ossicles
Malleus attaches to the tympanic membrane (the Umbo).
Malleus articulates with the incus (synovial joint).
Incus attaches to stapes (another synovial joint).

Internal ear
Cochlea, semi circular canals and vestibule

Organ of corti is the sense organ of hearing and is located on the inside of the cochlear duct on the basilar membrane.

Vestibule accommodates the utricule and the saccule. These structures contain endolymph and are surrounded by perilymph within the vestibule.

The semicircular canals lie at various angles to the petrous temporal bone. All share a common opening into the vestibule.

1027
Q

A 66 year old man with peripheral vascular disease is undergoing a below knee amputation. In which of the lower leg compartments does peroneus brevis lie?

Lateral compartment

Anterior compartment

Superficial posterior compartment

Deep posterior compartment

None of the above

A

The interosseous membrane separates the anterior and posterior compartments. The deep and superficial compartments are separated by the deep transverse fascia. The peroneus brevis is part of the lateral compartment.
Fascial compartments of the leg

Compartments of the thigh

Formed by septae passing from the femur to the fascia lata.
Compartment Nerve Muscles Blood supply
Anterior compartment Femoral
Iliacus
Tensor fasciae latae
Sartorius
Quadriceps femoris
Femoral artery
Medial compartment Obturator
Adductor longus/magnus/brevis
Gracilis
Obturator externus
Profunda femoris artery and obturator artery
Posterior compartment (2 layers) Sciatic
Semimembranosus
Semitendinosus
Biceps femoris
Branches of Profunda femoris artery

Compartments of the lower leg
Separated by the interosseous membrane (anterior and posterior compartments), anterior fascial septum (separate anterior and lateral compartments) and posterior fascial septum (separate lateral and posterior compartments)

Compartment Nerve Muscles Blood supply
Anterior compartment Deep peroneal nerve
Tibialis anterior
Extensor digitorum longus
Extensor hallucis longus
Peroneus tertius
Anterior tibial artery
Posterior compartment Tibial
Muscles: deep and superficial compartments (separated by deep transverse fascia)
Deep: Flexor hallucis longus, Flexor digitalis longus, Tibialis posterior, Popliteus
Superficial: Gastrocnemius, Soleus, Plantaris
Posterior tibial
Lateral compartment Superficial peroneal
Peroneus longus/brevis
Peroneal artery

1028
Q

A 20 year old woman trips over a step, injuring her ankle. Examination reveals tenderness over the lateral malleolus and an x-ray demonstrates an undisplaced fracture distal to the syndesmosis. What is the best course of action?

Application of ankle boot

Surgical fixation

Application of full leg plaster cast

Application of external fixator

Application of Ilizarov frame

A

This is a Weber A fracture. It is a stable ankle injury and can therefore be managed conservatively. Whilst this patient could also be treated in a below knee plaster, most clinicians would nowadays treat this injury in an ankle boot. Patients should be advised to mobilise in the ankle boot, as pain allows, and can wean themselves out of the boot as the symptoms improve.

Ankle injuries

An ankle fracture relates to a fracture around the tibio-talar joint. It generally refers to a fracture involving the lateral, and/or medial and/or posterior malleolus. Pilon and Tillaux fractures are also considered to be ankle fractures, but are not covered here.
Ankle fractures are common. They effect men and women in equal numbers, but men have a higher rate as young adults (sports and contact injuries), and women a higher rate post-menopausal (fragility type fracture).

Osseous anatomy
The ankle (or mortise) joint consists of the distal tibia (tibial plafond and posterior malleolus), the distal fibula (lateral malleolus), and the talus. The main movement at the ankle joint is plantar and dorsiflexion.

Ligamentous anatomy
Medial side: Deltoid ligament. This is divided into superficial and deep portions. It is the primary restraint to valgus tilting of the talus.

Lateral side: Lateral ligament complex consisting from anterior to posterior of the anterior talofibular ligament (ATFL), calcaneofibular ligament (CFL), and the posterior talofibular ligament (PTFL). Together they resist valgus stress to the ankle, and are a restraint to anterior translation of the talus within the mortise joint.

Syndesmosis: The syndesmosis is a ligament complex between the distal tibia and fibula, holding the two bones together. It is fundamental to the integrity of the ankle joint, and its disruption leads to instability. It consists of (from anterior to posterior) the anterior-inferior tibiofibular ligament (AITFL), the transverse tibiofibular ligament (TTFL), the interosseous membrane, and the posterior-inferior tibiofibular ligament (PITFL).

Presentation and initial management
Patients will present following a traumatic event with a painful, swollen ankle, and reluctance/inability to weight bear. The Ottawa rules can be applied to differentiate between an ankle fracture and sprain, but can be unreliable.
In high energy injuries, management should follow ATLS principles to identify more significant injuries first. Neurovascular status of the foot should be documented, and open injuries should be excluded. If an open injury is identified, it should be managed in line with BOAST 4 principles1. If an obvious deformity exists, it should be reduced as soon as possible with appropriate analgesia or conscious sedation. Radiographs of clearly deformed or dislocated joints are not necessary, and removing the pressure on the surrounding soft tissues from the underlying bony deformity is the priority. If the fracture pattern is not clinically obvious then plain radiographs are appropriate and will guide the subsequent manipulation during plaster-of-paris below knee backslab application.

Imaging
AP, lateral and mortise views (20 degrees internal rotation) are essential to evaluate fracture displacement and syndesmotic injury. Decreased tibiofibular overlap, medial joint clear space and lateral talar shift all indicate a syndesmotic injury. (In subtle cases of shift, imaging the uninjured ankle can be helpful as a proportion of the population have little or no tibiotalar overlap-reference 2.)

Where there is suspicion of syndesmosis involvement in the absence of radiographic evidence, stress radiographs can be diagnostic.
Complex fracture patterns (and increasingly posterior malleolar fractures) are best defined using CT.

Classification
The most commonly used classifications are Lauge-Hansen and Danis-Weber.

Lauge-Hansen
Comprises two parts: first part is the foot position, and the second part is the force applied. Useful for understanding the forces involved and therefore predict the ligamentous or bony injury. Results in four injury patterns:
Supination - Adduction (SA) - 10-20%
Supination - External rotation (SER) - 40-75%
Pronation - Abduction (PA) - 5-20%
Pronation - External rotation (PER) - 5-20%

Not often used in clinical practice but good for understanding the principles of ankle fracture.

Danis-Weber
Commonly used. Based on the level of the fibula fracture in relation to the syndesmosis. The more proximal, the greater the risk of syndesmotic injury and therefore fracture instability.
A - fracture below the level of the syndesmosis
B - fracture at the level of the syndesmosis / level of the tibial plafond
C - fracture above the level of the syndesmosis. This includes Maisonneuve fractures (proximal fibula fracture), which can be associated with ankle instability. Beware the high fibula fracture - it may be an ankle fracture!

The Weber classification is based purely on the the lateral side. All injuries can include a medial or posterior bony or ligamentous injury which also dictates fracture stability (bimalleolar and trimalleolar fractures are more unstable).

Treatment
When deciding upon treatment for an ankle fracture, one must consider both the fracture and the patient. Diabetic patients and smokers are at greater risk of post-operative complication, especially wound problems and infection. Likewise, the long term outcome of post-traumatic arthritis from a malunited ankle fracture is extremely important for a young patient, but not as relevant in the elderly. Therefore, normal surgical decision processes apply as with all fractures.

Defining stability of an ankle fracture underpins the treatment decision.
Weber A - Unimalleolar Weber A Weber fractures by definition are stable and therefore can be mobilised fully weight bearing in an ankle boot.

Weber C - Fractures tend to include syndesmotic disruption and are usually bimalleolar (either bony or ligamentous). They are therefore unstable and usually require operative fixation. In addition to the fracture fixation, the syndesmosis usually requires reconstruction/augmentation with screws to restore the joint integrity and function.

Weber B - B fractures vary greatly. They can be part of a trimalleolar injury and therefore extremely unstable, requiring fixation. Alternatively, a uni-malleolar Weber B fracture can be a stable injury, and therefore mobilised immediately in an ankle boot. Defining the stability can be challenging, and often involves stress radiographs, or a trial of mobilisation and repeat radiographs. However, treating undisplaced ankle fractures in a below knee plaster, non-weight bearing for six weeks is still widely practised, and a safe approach.

When operative fixation is appropriate, it is usually via open reduction and internal fixation using plates and screws. It must be carried out when soft tissue swelling has settled in order to minimise the risk of wound problems. This can often take a week to settle.

The use of fibula nails is expanding, but is not yet mainstream. Ankle fractures can also be treated with external fixation, or with a hind foot nail in patients who need fixation but where soft tissue or bone quality is poor.

Post operative management
Ankle fractures generally take 6 weeks to unite enough to prevent secondary displacement. This is therefore an appropriate time period to keep a cast on in a conservatively managed patient. Weight bearing post-operatively depends on the quality of the fixation and bone quality, and preference varies between surgeons, ranging from aggressive early mobilisation to a period of non-weight bearing. Return to activities takes approximately three months, and often requires assistance of a physiotherapist to improve range-of-movement and muscle strengthening.

References
1. http://www.boa.ac.uk/publications/boa-standards-trauma-boasts/
2. Shah AS, Kadakia AR, Tan GJ, Karadsheh MS, Wolter TD, Sabb B. Radiographic evaluation of the normal distal tibiofibular syndesmosis. Foot Ankle Int. 2012;33(10):870-6

1029
Q

Which of these nerves pass through the greater sciatic foramen and innervate the perineum?

Pudendal

Sciatic

Superior gluteal

Inferior gluteal

Posterior cutaneous nerve of the thigh

A

3 divisions of the pudendal nerve:
Rectal nerve
Perineal nerve
Dorsal nerve of penis/ clitoris
All these pass through the greater sciatic foramen.
The pudendal nerve innervates the perineum. It passes between piriformis and coccygeus medial to the sciatic nerve.

Gluteal region

Gluteal muscles
Gluteus maximus: inserts to gluteal tuberosity of the femur and iliotibial tract
Gluteus medius: attach to lateral greater trochanter
Gluteus minimis: attach to anterior greater trochanter
All extend and abduct the hip

Deep lateral hip rotators
Piriformis
Gemelli
Obturator internus
Quadratus femoris

Nerves
Superior gluteal nerve (L4,L5, S1)
Gluteus medius
Gluteus minimis
Tensor fascia lata
Inferior gluteal nerve (L5, S1, S2) Gluteus maximus
Damage to the superior gluteal nerve will result in the patient developing a Trendelenberg gait. Affected patients are unable to abduct the thigh at the hip joint. During the stance phase, the weakened abductor muscles allow the pelvis to tilt down on the opposite side. To compensate, the trunk lurches to the weakened side to attempt to maintain a level pelvis throughout the gait cycle. The pelvis sags on the opposite side of the lesioned superior gluteal nerve.

1030
Q

The proximal aspect of the right subclavian artery is derived from which of the structures listed below:

6th aortic arch

3rd aortic arch

4th aortic arch

1st aortic arch

2nd aortic arch

A

Embryological aortic arches

  • The aortic arches are a series of six paired embryological vascular structures which give rise to the great arteries of the neck and head.
    The first and second arches disappear early. A remnant of the 1st arch forms part of the maxillary artery. The external carotid buds from the horns of the aortic sac left behind by the regression of the first two arches.
    The third aortic arch constitutes the commencement of the internal carotid artery, and is therefore named the carotid arch. It contributes to the common carotid artery and the proximal portion of the internal carotid artery.
    The fourth right arch forms the right subclavian as far as the origin of its internal mammary branch. The fourth left arch forms the arch of the aorta between the origin of the left carotid artery and the termination of the ductus arteriosus.
    The fifth arch regresses or forms incompletely.
    The proximal part of the sixth right arch persists as the proximal part of the right pulmonary artery while the distal section degenerates; The sixth left arch gives off the left pulmonary artery and forms the ductus arteriosus.
1031
Q

A 28 year old man falls and sustains a simple rib fracture. On examination, there is a small pneumothorax. What is the most appropriate course of action?

Discharge with advice to return if symptoms worsen

Insertion of chest drain

Admission for observation

CT scanning of the chest

Thoracocentesis

A

Do not confuse the management of pneumothorax due to trauma with the management of spontaneous uncomplicated pneumothorax.
For a rib fracture to cause a pneumothorax, there must also be laceration to the underlying lung parenchyma. This has the risk of developing into a tension pneumothorax and for this reason a chest drain should be inserted and the patient admitted.

Thoracic trauma

Key points related to thoracic trauma
Less than 10% of blunt chest trauma and 15-30% of penetrating chest trauma requires operative intervention.
The physiologic consequences of thoracic trauma are hypoxia, hypercarbia, and acidosis. Contusion, hematoma, and alveolar collapse, or changes in intrathoracic pressure relationships (e.g., tension pneumothorax and open pneumothorax) cause hypoxia and lead to metabolic acidosis. Hypercarbia causes respiratory acidosis and most often follows inadequate ventilation caused by changes in intrathoracic pressure relationships and depressed level of consciousness.

Types of thoracic trauma

Tension pneumothorax
Often laceration to lung parenchyma with flap
Pressure develops in thorax
Most common cause is mechanical ventilation in patient with pleural injury
Symptoms overlap with cardiac tamponade, hyper-resonant percussion note is more likely in tension pnemothorax
Flail chest
Chest wall disconnects from thoracic cage
Multiple rib fractures (at least two fractures per rib in at least two ribs)
Associated with pulmonary contusion
Abnormal chest motion
Avoid over hydration and fluid overload
Pneumothorax
Most common cause is lung laceration with air leakage
Most traumatic pneumothoraces should have a chest drain
Patients with traumatic pneumothorax should never be mechanically ventilated until a chest drain is inserted
Haemothorax
Most commonly due to laceration of lung, intercostal vessel or internal mammary artery
Haemothoraces large enough to appear on CXR are treated with large bore chest drain
Surgical exploration is warranted if >1500ml blood drained immediately
Cardiac tamponade
Beck’s triad: elevated venous pressure, reduced arterial pressure, reduced heart sounds
Pulsus paradoxus
May occur with as little as 100ml blood
Pulmonary contusion
Most common potentially lethal chest injury
Arterial blood gases and pulse oximetry important
Early intubation within an hour if significant hypoxia
Blunt cardiac injury
Usually occurs secondary to chest wall injury
ECG may show features of myocardial infarction
Sequelae: hypotension, arrhythmias, cardiac wall motion abnormalities
Aorta disruption
Deceleration injuries
Contained haematoma
Widened mediastinum
Diaphragm disruption
Most due to motor vehicle accidents and blunt trauma causing large radial tears (laceration injuries result in small tears)
More common on left side
Insert gastric tube, may pass into intrathoracic stomach
Mediastinal traversing wounds
Entrance wound in one hemithorax and exit wound/foreign body in opposite hemithorax
Mediastinal haematoma or pleural cap suggests great vessel injury
Mortality is 20%

References
ATLS Manual 10th Edition. ISBN 78-0-9968262-3-5

1032
Q

Which of the following is not a branch of the subclavian artery?

Superior thyroid artery
Vertebral artery
Thyrocervical trunk
Internal thoracic artery
Dorsal scapular artery

A

Mnemonic for the branches of the subclavian artery: VIT C & D

V ertebral artery
I nternal thoracic
T hyrocervical trunk

C ostalcervical trunk
D orsal scapular
Superior thyroid artery is a branch of the external carotid artery.

Subclavian artery

Path
The left subclavian comes directly off the arch of aorta
The right subclavian arises from the brachiocephalic artery (trunk) when it bifurcates into the subclavian and the right common carotid artery.
From its origin, the subclavian artery travels laterally, passing between anterior and middle scalene muscles, deep to scalenus anterior and anterior to scalenus medius. As the subclavian artery crosses the lateral border of the first rib, it becomes the axillary artery. At this point it is superficial and within the subclavian triangle.

Branches
Vertebral artery
Internal thoracic artery
Thyrocervical trunk
Costocervical trunk
Dorsal scapular artery

1033
Q

Which investigation is best for initial assessment of recurrence of follicular carcinoma of the thyroid?

Free T4
Thyroid stimulating hormone
Scintigraphy
Serum thyroglobulin
USS thyroid gland

A

Elevated thyroglobulin levels raises suspicion of recurrence.

Thyroid malignancy

Papillary carcinoma
Commonest sub-type
Accurately diagnosed on fine needle aspiration cytology
Histologically, they may demonstrate psammoma bodies (areas of calcification) and so called ‘orphan Annie’ nuclei
They typically metastasise via the lymphatics and thus laterally located apparently ectopic thyroid tissue is usually a metastasis from a well differentiated papillary carcinoma

Follicular carcinoma
Are less common than papillary lesions
Like papillary tumours, they may present as a discrete nodule. Although they appear to be well encapsulated macroscopically there is invasion on microscopic evaluation
Lymph node metastases are uncommon and these tumours tend to spread haematogenously. This translates into a higher mortality rate
Follicular lesions cannot be accurately diagnosed on fine needle aspiration cytology and thus all follicular FNA’s (THY 3f) will require at least a hemi thyroidectomy

Anaplastic carcinoma
Less common and tend to occur in elderly females
Disease is usually advanced at presentation and often only palliative decompression and radiotherapy can be offered.

Medullary carcinoma
These are tumours of the parafollicular cells ( C Cells) and are of neural crest origin.
The serum calcitonin may be elevated which is of use when monitoring for recurrence.
They may be familial and occur as part of the MEN -2A disease spectrum.
Spread may be either lymphatic or haematogenous and as these tumours are not derived primarily from thyroid cells they are not responsive to radioiodine.

Lymphoma
These respond well to combined chemoradiotherapy
Radical surgery is unnecessary once the disease has been diagnosed on biopsy material. Such biopsy material is not generated by an FNA and thus a core biopsy has to be obtained (with care!).

1034
Q

A 24 year female is admitted to A&E with tingling of her hand after a fall. She is found to have a fracture of the medial epicondyle. What is the most likely nerve lesion?

Ulnar nerve
Radial nerve
Median nerve
Axillary nerve
Cutaneous nerve

A

Ulnar nerve. The radial nerve is located near the lateral epicondyle.

Ulnar nerve

Origin
C8, T1

Supplies (no muscles in the upper arm)
Flexor carpi ulnaris
Flexor digitorum profundus
Flexor digiti minimi
Abductor digiti minimi
Opponens digiti minimi
Adductor pollicis
Interossei muscle
Third and fourth lumbricals
Palmaris brevis

Path
Posteromedial aspect of upper arm to flexor compartment of forearm, then along the ulnar. Passes beneath the flexor carpi ulnaris muscle, then superficially over the flexor retinaculum into the palm of the hand.

Branches
Branch Supplies
Muscular branch Flexor carpi ulnaris
Medial half of the flexor digitorum profundus
Palmar cutaneous branch (Arises near the middle of the forearm) Skin on the medial part of the palm
Dorsal cutaneous branch Dorsal surface of the medial part of the hand
Superficial branch Cutaneous fibres to the anterior surfaces of the medial one and one-half digits
Deep branch Hypothenar muscles
All the interosseous muscles
Third and fourth lumbricals
Adductor pollicis
Medial head of the flexor pollicis brevis

Effects of injury
Damage at the wrist
Wasting and paralysis of intrinsic hand muscles (claw hand)
Wasting and paralysis of hypothenar muscles
Loss of sensation medial 1 and half fingers
Damage at the elbow
Radial deviation of the wrist
Clawing less in 4th and 5th digits

1035
Q

An 83 year old lady presents with a femoral hernia and undergoes a femoral hernia repair. Which of the following forms the posterior wall of the femoral canal?

Pectineal ligament
Lacunar ligament
Inguinal ligament
Adductor longus
Sartorius

A

Femoral canal

The femoral canal lies at the medial aspect of the femoral sheath. The femoral sheath is a fascial tunnel containing both the femoral artery laterally and femoral vein medially. The canal lies medial to the vein.

Borders of the femoral canal
Laterally Femoral vein
Medially Lacunar ligament
Anteriorly Inguinal ligament
Posteriorly Pectineal ligament

Contents
Lymphatic vessels
Cloquet’s lymph node

Physiological significance
Allows the femoral vein to expand to allow for increased venous return from the lower limbs.

Pathological significance
As a potential space, it is the site of femoral hernias. The relatively tight neck places these at high risk of strangulation.

1036
Q

Chordoma may typically occur at the following sites, except?

Ribs
Clivus
Sacrum
Lumbar vertebra
Cervical vertebra

A

Ribs. Chordoma is a neoplasm originating from ectopic cellular remnants of the notochord and therefore arises from the midline of the axial skeleton. It accounts for 24% of all primary malignant bone tumours. Chordoma is the second commonest primary malignancy of the spine and accounts for over 50% of primary sacral tumours. The neoplasm has a predilection for the sacrococcygeal (50%) and clival (40%) regions, with other areas of the spine rarely involved. More than one vertebral body can be affected in half the cases. Chordomas most commonly present between 50 and 70 years of age. Sex incidence is equal below 40 years, but men are affected twice as often at older ages, particularly in the sacral region.

The most frequent radiographic appearance of chordoma is that of a destructive lesion of a vertebral body centered in the midline, with a large, associated soft-tissue mass.

Chordoma

Chordoma is a rare slow-growing bone tumour. Their favored origin is remnants of the notochord.

Chordomas can arise anywhere from the skull base to the sacrum. The two most common locations are the skull base and sacrum.

There are three histological variants of chordoma: classical (or ‘conventional’), chondroid and de-differentiated.
The histological appearance of classical chordoma is of a lobulated tumor composed of groups of cells separated by fibrous septa. The cells have small round nuclei and abundant vacuolated cytoplasm.
Chondroid chordomas histologically show features of both chordoma and chondrosarcoma.

The 10-year tumor free survival rate for sacral chordoma was 46%. Chondroid chordomas appear to have a more indolent clinical course.

In most cases, complete surgical resection followed by radiation therapy offers the best chance of long-term control. Unfortunately, the lesion has a close proximity to the spine itself and this can compromise resection margins.

Chordomas are relatively radioresistant, requiring high doses of radiation to be controlled. The proximity of chordomas to vital neurological structures such as the brain stem and nerves limits the dose of radiation that can safely be delivered. Therefore, highly focused radiation such as proton therapy and carbon ion therapy are more effective than conventional x-ray radiation.

1037
Q

A 56 year old man is undergoing a pancreatectomy for carcinoma. During resection of the gland which of the following structures will the surgeon not encounter posterior to the pancreas itself?

Left crus of the diaphragm
Superior mesenteric vein
Common bile duct
Portal vein
Gastroduodenal artery

A

The gastroduodenal artery divides into the gastro-epiploic and pancreaticoduodenal arteries at the superior aspect of the pancreas.

Pancreas

The pancreas is a retroperitoneal organ and lies posterior to the stomach. It may be accessed surgically by dividing the peritoneal reflection that connects the greater omentum to the transverse colon. The pancreatic head sits in the curvature of the duodenum. Its tail lies close to the hilum of the spleen, a site of potential injury during splenectomy.

Relations
Posterior to the pancreas
Pancreatic head Inferior vena cava
Common bile duct
Right and left renal veins
Superior mesenteric vein and artery
Pancreatic neck Superior mesenteric vein, portal vein
Pancreatic body- Left renal vein
Crus of diaphragm
Psoas muscle
Adrenal gland
Kidney
Aorta
Pancreatic tail Left kidney

Anterior to the pancreas
Pancreatic head 1st part of the duodenum
Pylorus
SMA and SMV(uncinate process)
Pancreatic body Stomach
Duodenojejunal flexure
Pancreatic tail Splenic hilum

Superior to the pancreas
Coeliac trunk and its branches common hepatic artery and splenic artery

Grooves of the head of the pancreas
2nd and 3rd part of the duodenum

Arterial supply
Head: pancreaticoduodenal artery
Rest: splenic artery

Venous drainage
Head: superior mesenteric vein
Body and tail: splenic vein

Ampulla of Vater
Merge of pancreatic duct and common bile duct
Is an important landmark, halfway along the second part of the duodenum, that marks the anatomical transition from foregut to midgut (also the site of transition between regions supplied by coeliac trunk and SMA).

1038
Q

A 69 year old male presents with haematuria. He worked in the textile industry. He has a left flank discomfort. A CT IVU shows a lesion of the left renal pelvis. What is the most likely lesion?

Adenocarcinoma
Transitional cell carcinoma
Nephroblastoma
Neuroblastoma
Angiomyolipoma

A

TCC is a rare form of renal cancer, accounting for approximately 7% of all renal tumours. Risk factors include exposure to chemicals in the textile, plastic and rubber industry.

Renal lesions

Lesion Disease specific features Treatment
Renal cell carcinoma
Most present with haematuria (50%)
Common renal tumour (85% cases)
Paraneoplastic features include hypertension and polycythaemia
Most commonly has haematogenous mestastasis
Usually radical or partial nephrectomy
Nephroblastoma
Rare childhood tumour
It accounts for 80% of all genitourinary malignancies in those under the age of 15 years
Up to 90% will have a mass
50% will be hypertensive
Diagnostic work up includes ultrasound and CT scanning
Surgical resection combined with chemotherapy (usually vincristine, actinomycin D and doxorubicin)
Neuroblastoma
Most common extracranial tumour of childhood
80% occur in those under 4 years of age
Tumour of neural crest origin (up to 50% occur in the adrenal gland)
The tumour is usually calcified and may be diagnosed using MIBG scanning
Staging is with CT
Surgical resection, radiotherapy and chemotherapy
Transitional cell carcinoma
Accounts for 90% of lower urinary tract tumours, but only 10% of renal tumours
Males affected 3x more than females
Occupational exposure to industrial dyes and rubber chemicals may increase risk
Up to 80% present with painless haematuria
Diagnosis and staging is with CT IVU
Radical nephroureterectomy
Angiomyolipoma
80% of these hamartoma type lesions occur sporadically, the remainder are seen in those with tuberous sclerosis
Tumour is composed of blood vessels, smooth muscle and fat
Massive bleeding may occur in 10% of cases

1039
Q

A 55 year old man is found to have an anal cancer. His staging investigations show a T2 lesion with no metastatic disease. What is the most appropriate treatment?

Radical abdominoperineal excision of the anus and rectum
Radical chemoradiotherapy
Excision proctectomy
External beam irradiation alone
Chemotherapy alone

A

Combined chemoradiotherapy is the standard treatment for anal cancer
First line treatment for anal cancer (which is very different from rectal cancer) is radical chemoradiotherapy.

Anal cancer

  • Cancers arising from the squamous epithelium of the anal canal
    Arise inferior to the dentate line
    Strongly linked to HPV type 16 infection
    Other risk factors include ano-receptive intercourse, smoking and immunosuppression
    Presenting symptoms include anal discomfort, discharge or pruritus
    Lymphatic spread typically occurs to the inguinal nodes
    Diagnosis is made by EUA and biopsies
    Staging is with CT scanning of the chest, abdomen and pelvis
    First line treatment is typically with chemoradiotherapy
    Second line treatment for non metastatic disease is with salvage radical abdominoperineal excision of the anus and rectum
1040
Q

A 53 year old lady presents with pain and discomfort in her hand. She works as a typist and notices that the pain is worst when she is working. She also suffers symptoms at night. Her little finger is less affected by the pain. Which of the nerves listed below is most likely to be affected?

Radial
Median
Ulnar
Anterior interosseous nerve
Posterior interosseous nerve

A

Median.

Motor supply: LOAF

L ateral 2 lumbricals
O pponens pollicis
A bductor pollicisbrevis
F lexor pollicis brevis
The most likely diagnosis here is carpal tunnel syndrome, the median nerve is compressed in the wrist and symptoms usually affect the fingers and wrist either at night or when the hand is being used (e.g. as a typist).

Median nerve

The median nerve is formed by the union of a lateral and medial root respectively from the lateral (C5,6,7) and medial (C8 and T1) cords of the brachial plexus; the medial root passes anterior to the third part of the axillary artery. The nerve descends lateral to the brachial artery, crosses to its medial side (usually passing anterior to the artery). It passes deep to the bicipital aponeurosis and the median cubital vein at the elbow.
It passes between the two heads of the pronator teres muscle, and runs on the deep surface of flexor digitorum superficialis (within its fascial sheath).
Near the wrist it becomes superficial between the tendons of flexor digitorum superficialis and flexor carpi radialis, deep to palmaris longus tendon. It passes deep to the flexor retinaculum to enter the palm, but lies anterior to the long flexor tendons within the carpal tunnel.

Branches
Region Branch
Upper arm No branches, although the nerve commonly communicates with the musculocutaneous nerve
Forearm Pronator teres
Pronator quadratus
Flexor carpi radialis
Palmaris longus
Flexor digitorum superficialis
Flexor pollicis longus
Flexor digitorum profundus (only the radial half)
Distal forearm Palmar cutaneous branch
Hand (Motor) Motor supply (LOAF)
Lateral 2 lumbricals
Opponens pollicis
Abductor pollicis brevis
Flexor pollicis brevis
Hand (Sensory)
Over thumb and lateral 2 ½ fingers
On the palmar aspect this projects proximally, on the dorsal aspect only the distal regions are innervated with the radial nerve providing the more proximal cutaneous innervation.

Patterns of damage
Damage at wrist
e.g. carpal tunnel syndrome
paralysis and wasting of thenar eminence muscles and opponens pollicis (ape hand deformity)
sensory loss to palmar aspect of lateral (radial) 2 ½ fingers

Damage at elbow, as above plus:
unable to pronate forearm
weak wrist flexion
ulnar deviation of wrist

Anterior interosseous nerve (branch of median nerve)
leaves just below the elbow
results in loss of pronation of forearm and weakness of long flexors of thumb and index finger

1041
Q

A 19 year old man is attacked outside a club and beaten with a baseball bat. He sustains a blow to the right side of his head. He is brought to the emergency department and a policy of observation is adopted. His glasgow coma score deteriorates and he becomes comatose. Which of the following haemodynamic parameters is most likely to be present?

Hypertension and bradycardia
Hypotension and tachycardia
Hypotension and bradycardia
Hypertension and tachycardia
Normotension and bradycardia

A

Hypertension and bradycardia are seen prior to coning. The brain autoregulates its blood supply by controlling systemic blood pressure.

Head injury

Patients who suffer head injuries should be managed according to ATLS principles and extra cranial injuries should be managed alongside cranial trauma. Inadequate cardiac output will compromise CNS perfusion irrespective of the nature of the cranial injury.

Types of traumatic brain injury
Extradural haematoma Bleeding into the space between the dura mater and the skull. Often results from acceleration-deceleration trauma or a blow to the side of the head. The majority of extradural haematomas occur in the temporal region where skull fractures cause a rupture of the middle meningeal artery.

Features
Raised intracranial pressure
Some patients may exhibit a lucid interval
Subdural haematoma Bleeding into the outermost meningeal layer. Most commonly occur around the frontal and parietal lobes. May be either acute or chronic.

Risk factors include old age and alcoholism.

Slower onset of symptoms than a extradural haematoma.
Subarachnoid haemorrhage Usually occurs spontaneously in the context of a ruptured cerebral aneurysm, but may be seen in association with other injuries when a patient has sustained a traumatic brain injury.

Pathophysiology
Primary brain injury may be focal (contusion/ haematoma) or diffuse (diffuse axonal injury)
Diffuse axonal injury occurs as a result of mechanical shearing following deceleration, causing disruption and tearing of axons
Intra-cranial haematomas can be extradural, subdural or intracerebral, while contusions may occur adjacent to (coup) or contralateral (contre-coup) to the side of impact
Secondary brain injury occurs when cerebral oedema, ischaemia, infection, tonsillar or tentorial herniation exacerbates the original injury. The normal cerebral auto regulatory processes are disrupted following trauma rendering the brain more susceptible to blood flow changes and hypoxia
The Cushings reflex (hypertension and bradycardia) often occurs late and is usually a pre terminal event

Management
Where there is life threatening rising ICP such as in extra dural haematoma and whilst theatre is prepared or transfer arranged use of IV mannitol/ frusemide may be required.
Diffuse cerebral oedema may require decompressive craniotomy
Exploratory Burr Holes have little management in modern practice except where scanning may be unavailable and to thus facilitate creation of formal craniotomy flap
Depressed skull fractures that are open require formal surgical reduction and debridement, closed injuries may be managed non operatively if there is minimal displacement.
ICP monitoring is appropriate in those who have GCS 3-8 and normal CT scan.
ICP monitoring is mandatory in those who have GCS 3-8 and abnormal CT scan.
Hyponatraemia is most likely to be due to syndrome of inappropriate ADH secretion.
Minimum of cerebral perfusion pressure of 70mmHg in adults.
Minimum cerebral perfusion pressure of between 40 and 70 mmHg in children.

Interpretation of pupillary findings in head injuries
Pupil size Light response Interpretation
Unilaterally dilated Sluggish or fixed 3rd nerve compression secondary to tentorial herniation
Bilaterally dilated Sluggish or fixed
Poor CNS perfusion
Bilateral 3rd nerve palsy
Unilaterally dilated or equal Cross reactive (Marcus - Gunn) Optic nerve injury
Bilaterally constricted May be difficult to assess
Opiates
Pontine lesions
Metabolic encephalopathy
Unilaterally constricted Preserved Sympathetic pathway disruption

1042
Q

A surgical team wish to conduct a meta analysis of randomised controlled trials of the use of low molecular weight heparins in the prevention of post operative deep vein thrombosis. How would these results be best displayed graphically?

Forest plot
Box Whisker plot
Violin plot
Kaplan Meier graph
None of the above

A

Data from multiple RCT’s are best displayed using Forest plots. Funnel plots may be used to determine the effect of small studies and their overall effect on the data. Violin plots and Box Whisker plots are often used to graphically display non parametric data from single studies and are not generally used to display data from meta analyses.

Forest plots

A Forest plot is a graphical display designed to illustrate the relative strength of treatment effects in multiple quantitative scientific studies, addressing the same question. It is often used to graphically display meta analyses of randomised controlled trials.

The graph may be plotted on a natural logarithmic scale when using odds ratios or other ratio-based effect measures, so that the confidence intervals are symmetrical about the means from each study and to ensure undue emphasis is not given to odds ratios greater than 1 when compared to those less than 1. The area of each square is proportional to the study’s weight in the meta-analysis. The overall meta-analysed measure of effect is often represented on the plot as a vertical line. This meta-analysed measure of effect is commonly plotted as a diamond, the lateral points of which indicate confidence intervals for this estimate.

A vertical line representing no effect is also plotted. If the confidence intervals for individual studies overlap with this line, it demonstrates that at the given level of confidence their effect sizes do not differ from no effect for the individual study. The same applies for the meta-analysed measure of effect: if the points of the diamond overlap the line of no effect the overall meta-analysed result cannot be said to differ from no effect at the given level of confidence.

1043
Q

A 72 year old man has been unwell for many years and following his death a post mortem is performed. Tissue is submitted for microscopic evaluation. Evaluation of sections of the myocardium demonstrates evidence of apple green birefringence with polarised light. What is the most likely diagnosis?

Amyloidosis
SLE
Tuberculosis
Disseminated B cell lymphoma
Systemic sclerosis

A

Amyloidosis = apple green birefringence with polarised light

Amyloid

Amyloid is an extracellular protein deposit which is insoluble. These deposits disrupt normal tissue structure and if excessive may affect function. All types of amyloid consist of a major fibrillar protein that defines the type of amyloid (approximately 90%) plus various minor components.

Amyloid is classified with the prefix A (for amyloid) and the suffix depending upon the fibrillary protein present. The main clinical types are AA and AL amyloidosis. Systemic AA amyloidosis is a long-term complication of several chronic inflammatory disorders - e.g. rheumatoid arthritis, ankylosing spondylitis, Crohn’s disease, malignancies and conditions predisposing to recurrent infections. AL amyloidosis results from extra-cellular deposition of fibril-forming monoclonal immunoglobulin light chains (most commonly of lambda isotype). Most patients have evidence of isolated monoclonal gammopathy or asymptomatic myeloma, and the occurrence of AL amyloidosis in patients with symptomatic multiple myeloma or other B-cell lymphoproliferative disorders is unusual. AL type amyloidosis is the most common variant. The kidney and heart are two of the most commonly affected sites. Diagnosis is based on surgical biopsy and characteristic histological features which consist birefringence under polarised light. Immunohistochemistry is used to delineate the subtype. Treatment is usually targeted at the underlying cause.

1044
Q

What is the risk of wound infection occurring in a 43 year old woman who is undergoing a laparoscopic cholecystectomy for uncomplicated biliary colic?

1%
4%
15%
20%
30%

A

In straightforward uncomplicated laparoscopic cholecystectomy the wound infection rates are around 3-5% (Pasquali et al and Matsui et al). The use of antibiotics in this group is therefore unnecessary. Complex procedures, bile spills and acute cholecystectomy have higher infection rates and antibiotics should be administered to these patients.

References:

S Pasquali , M Boal , E A Griffiths , D Alderson , R S Vohra, CholeS Study Group; West Midlands Research Collaborative. Meta-analysis of perioperative antibiotics in patients undergoing laparoscopic cholecystectomy. Br J Surg. 2016 Jan;103(1):27-34

Yoichi Matsui, Sohei Satoi, Satoshi Hirooka, Hisashi Kosaka, Takayuki Kawaura, and Tomoki Kitawaki.
Reappraisal of previously reported meta-analyses on antibiotic prophylaxis for low-risk laparoscopic cholecystectomy: an overview of systematic reviews. BMJ Open. 2018; 8(3): e016666.

Surgical site infection

  • Surgical site infections may occur following a breach in tissue surfaces and allow normal commensals and other pathogens to initiate infection. They are a major cause of morbidity and mortality.
    Surgical site infections (SSI) comprise up to 20% of all healthcare associated infections and at least 5% of patients undergoing surgery will develop an SSI as a result.
    In many cases the organisms are derived from the patient’s own body. Measures that may increase the risk of SSI include:
    Shaving the wound using a razor (disposable clipper preferred)
    Using a non iodine impregnated incise drape if one is deemed to be necessary
    Tissue hypoxia
    Delayed administration of prophylactic antibiotics in tourniquet surgery

Preoperatively
Don’t remove body hair routinely
If hair needs removal, use electrical clippers with single use head (razors increase infection risk)
Antibiotic prophylaxis if:
- placement of prosthesis or valve
- clean-contaminated surgery
- contaminated surgery
Use local formulary
Aim to give single dose IV antibiotic on anaesthesia
If a tourniquet is to be used, give prophylactic antibiotics earlier

Intraoperatively
Prepare the skin with alcoholic chlorhexidine (Lowest incidence of SSI)
Cover surgical site with dressing
A recent meta analysis has confirmed that administration of supplementary oxygen does not reduce the risk of wound infection. In contrast to previous individual RCT’s(1)
Wound edge protectors do not appear to confer benefit (2)

Post operatively
Tissue viability advice for management of surgical wounds healing by secondary intention

Use of diathermy for skin incisions
In the NICE guidelines the use of diathermy for skin incisions is not advocated(3). Several randomised controlled trials have been undertaken and demonstrated no increase in risk of SSI when diathermy is used(4).

References
1. Brar M et al.. Perioperative supplemental oxygen in colorectal patients: a meta analysis. J Surg Res 2011 (166): 227 -235.
2. Pinkney T et al. Impact of wound edge protection devices on surgical site infection after laparotomy: impact of a multicentre randomised controlled trial (ROSSINI Trial). BMJ 2013 (347):10.
3. http://www.nice.org.uk/CG74
4. Ahmad N and Ahmed A. Meta-analysis of the effectiveness of surgical scalpel or diathermy in making abdominal skin incisions. Ann Surg 2011, 253(1):8-13.

1045
Q

Which of the following statements relating to gastric acid secretions are untrue?

In parietal cells carbonic anhydrase generates hydrogen ions which are then actively secreted
The cephalic phase is abolished following truncal vagotomy
The intestinal phase accounts for 60% of gastric acid produced
Histamine acts in a paracrine manner on H
2
receptors
H
2
receptor antagonists will not completely abolish gastric acid production

A

The intestinal phase of gastric acid secretion accounts for only 10% of gastric acid produced.

Gastric secretions

A working knowledge of gastric secretions is important for surgery because peptic ulcers are common, surgeons frequently prescribe anti secretory drugs and because there are still patients around who will have undergone acid lowering procedures (Vagotomy) in the past.

Gastric acid
Is produced by the parietal cells in the stomach
pH of gastric acid is around 2 with acidity being maintained by the H+/K+ ATP ase pump. As part of the process bicarbonate ions will be secreted into the surrounding vessels.
Sodium and chloride ions are actively secreted from the parietal cell into the canaliculus. This sets up a negative potential across the membrane and as a result sodium and potassium ions diffuse across into the canaliculus.
Carbonic anhydrase forms carbonic acid which dissociates and the hydrogen ions formed by dissociation leave the cell via the H+/K+ antiporter pump. At the same time sodium ions are actively absorbed. This leaves hydrogen and chloride ions in the canaliculus these mix and are secreted into the lumen of the oxyntic gland.
Phases of gastric acid secretion
There are 3 phases of gastric secretion:

  1. Cephalic phase (smell / taste of food)
    30% acid produced
    Vagal cholinergic stimulation causing secretion of HCL and gastrin release from G cells
  2. Gastric phase (distension of stomach )
    60% acid produced
    Stomach distension/low H+/peptides causes Gastrin release
  3. Intestinal phase (food in duodenum)
    10% acid produced
    High acidity/distension/hypertonic solutions in the duodenum inhibits gastric acid secretion via enterogastrones (CCK, secretin) and neural reflexes.

Regulation of gastric acid production
Factors increasing production include:
Vagal nerve stimulation
Gastrin release
Histamine release (indirectly following gastrin release) from enterchromaffin like cells

Factors decreasing production include:
Somatostatin (inhibits histamine release)
Cholecystokinin
Secretin

Below is a brief summary of the major hormones involved in food digestion:

Source Stimulus Actions
Gastrin
G cells in antrum of the stomach
Distension of stomach, extrinsic nerves
Inhibited by: low antral pH, somatostatin
Increase HCL, pepsinogen and IF secretion, increases gastric motility, trophic effect on gastric mucosa
CCK
I cells in upper small intestine
Partially digested proteins and triglycerides
Increases secretion of enzyme-rich fluid from pancreas, contraction of gallbladder and relaxation of sphincter of Oddi, decreases gastric emptying, trophic effect on pancreatic acinar cells, induces satiety
Secretin
S cells in upper small intestine
Acidic chyme, fatty acids
Increases secretion of bicarbonate-rich fluid from pancreas and hepatic duct cells, decreases gastric acid secretion, trophic effect on pancreatic acinar cells
VIP
Small intestine, pancreas
Neural
Stimulates secretion by pancreas and intestines, inhibits acid and pepsinogen secretion
Somatostatin
D cells in the pancreas and stomach
Fat, bile salts and glucose in the intestinal lumen
Decreases acid and pepsin secretion, decreases gastrin secretion, decreases pancreatic enzyme secretion, decreases insulin and glucagon secretion
inhibits trophic effects of gastrin, stimulates gastric mucous production

1046
Q

Which of the following best accounts for the action of PTH in increasing serum calcium levels?

Activation of vitamin D to increase absorption of calcium from the small intestine.
Direct stimulation of osteoclasts to absorb bone with release of calcium.
Stimulation of phosphate absorption at the distal convoluted tubule of the kidney.
Decreased porosity of the vessels at Bowmans capsule to calcium.
Vasospasm of the afferent renal arteriole thereby reducing GFR and calcium urinary loss.

A

PTH increases the activity of 1-α-hydroxylase enzyme, which converts 25-hydroxycholecalciferol to 1,25-dihydroxycholecalciferol, the active form of vitamin D.
Osteoclasts do not have a PTH receptor and effects are mediated via osteoblasts.

Parathyroid hormone

Parathyroid hormone is secreted by the chief cells of the parathyroid glands. It acts to increase serum calcium concentration by stimulation of the PTH receptors in the kidney and bone. PTH has a plasma half life of 4 minutes.

Effects of PTH
Bone Binds to osteoblasts which signal to osteoclasts to cause resorption of bone and release calcium.
Kidney Active reabsorption of calcium and magnesium from the distal convoluted tubule. Decreases reabsorption of phosphate.
Intestine via kidney Increases intestinal calcium absorption by increasing activated vitamin D. Activated vitamin D increases calcium absorption.

1047
Q

You decide to take an arterial blood gas from the femoral artery. Where should the needle be inserted to gain the sample?

1-2 cm inferiorly to the mid point of the inguinal ligament
1-2cm inferiorly to the mid inguinal point
2cm inferomedially to the pubic tubercle
2cm superomedially to the pubic tubercle
3cm inferolaterally to the deep inguinal ring

A

The mid inguinal point is midway between the anterior superior iliac spine and the symphysis pubis
The mid inguinal point in the surface marking for the femoral artery.

Femoral triangle anatomy

Boundaries
Superiorly Inguinal ligament
Laterally Sartorius
Medially Adductor longus
Floor Iliopsoas, adductor longus and pectineus
Roof
Fascia lata and Superficial fascia
Superficial inguinal lymph nodes (palpable below the inguinal ligament)
Long saphenous vein

Contents
Femoral vein (medial to lateral)
Femoral artery-pulse palpated at the mid inguinal point
Femoral nerve
Deep and superficial inguinal lymph nodes
Lateral cutaneous nerve
Great saphenous vein
Femoral branch of the genitofemoral nerve

1048
Q

A 33 year old man is involved in a motor vehicle accident. He is found to have a low GCS and is intubated at the scene. Shortly after intubation, he becomes profoundly hypoxic. What is the most likely explanation?

Slipped endotracheal tube

Trachobronchial injury

Tension pneumothorax

Mucous plugging

Aspiration

A

In trauma, sudden development of hypoxia following intubation suggests a pneumothorax. Where there is concern and time permits, it is usually safer to insert a chest drain prior to intubation and if a trauma patient does require intubation at scene . It is often safer to prophylactically insert a chest drain prior to transfer.

Pneumothorax

Pneumothorax is a common medical problem. In surgical practice patients can be divided into those who sustain thoracic trauma with an associated pneumothorax and those who happen to be surgical patients for another reason and develop a pneumothorax. In some cases this may be a sudden spontaneous event. For others, it may be the result of iatrogenic injury.

Initial approach
ABCD approach
High flow oxygen
Adequate monitoring
Early imaging (portable CXR)

Traumatic pneumothorax
The management of a traumatic pneumothorax is embodied in the ATLS principles. The most important decision to make and to make early, is whether the patient has a tension present.
Signs of tension pneumothorax
Severe respiratory distress
Hyper-expanded chest
Tracheal shift
Hyperressonant percussion note
Distended neck veins
Decreased breath sounds

If so then it must be immediately decompressed using a wide bore needle inserted in the fourth intercostal space in the mid axillary line.
Once this has been undertaken, immediate steps should be taken to insert a chest drain.

Traumatic pneumothoraxes may occur via a number of different mechanisms. Penetrating trauma may create a flap of lung tissue that can predispose to subsequent tension and for that reason, most would proceed to insert a chest drain if even a small pneumothorax were present by the underlying mechanism one of penetrating injury. One potential pitfall is to fail to appreciate that blunt mechanisms of injury to the chest wall may cause a rib fracture which is then able to inflict a penetrating lung injury.

The insertion of chest drains for chest trauma is different from the seldinger sets that are often used by physicians. In trauma it is usual to insert a wide bore chest drain (between 30 and 34Fr) using an open technique (with trochar removed).

Triangle of safety
It is advised that chest drains are placed in the ‘safe triangle’. The triangle is located in the mid axillary line of the 5th intercostal space. It is bordered by:
Anterior edge latissimus dorsi, the lateral border of pectoralis major, a line superior to the horizontal level of the nipple, and the apex below the axilla.

Non traumatic pneumothorax
The management of these is governed by the British Thoracic Society Guidelines (which do not apply to trauma cases). When reviewing the chest x-rays for these cases it is useful to distinguish between large and small pneumothoraces, The differentiation of a large from a small pneumo-thorax continues to be the presence of a visible rim of >2 cm between the lung margin and the chest wall (at the level of the hilum). However, size does not correlate well with symptoms. The presence of small primary pneumothorax in a patient with no compromise may not require any specific treatment. Those with underlying lung disease or large pneumothoraces should be considered for aspiration or chest drain insertion. Where a chest drain is inserted, a small calibre drain is sufficient. Detailed management of these cases is usually provided by physicians with an interest in respiratory medicine and fall outside of the remit of general surgical practice (and thus the MRCS!)

Whenever a chest drain or thoracic aspiration is performed, a check x-ray should be routinely performed following the intervention

1049
Q

A 56 year old lady undergoes a mastectomy as treatment for multifocal ductal carcinoma in situ. Two weeks post operatively she attends the clinic and complains of a diffuse swelling at the surgical site. On examination she has a large, fluctuant area underlying the mastectomy skin flaps. She is otherwise well. What is the most likely cause?

Abscess

Seroma

Haematoma

Disease recurrence

Arteriovenous malformation

A

Seromas are very common after breast surgery. The exposed raw surfaces created during the elevation of the skin flaps are a common cause. Treatment usually involves percutaneous drainage under aseptic conditions.

Complications of breast surgery

Breast surgery may be associated with the following complications:
Long thoracic nerve injury. This may occur during the axillary dissection and result in winging of the scapula.
Intercostobrachial nerve injury. These nerves traverse the axilla. When they are divided (which they often are) the patient will notice an area of parasthesia in the armpit.
Injury to the thoracodorsal trunk. This nerve and vessels supply latissimus dorsi. If they are damaged the functional effects are not too serious, the greatest setback is that a latissimus dorsi flap cannot be used for reconstruction purposes.
Infections. Cellulitis of the chest wall and arm may be a major problem if axillary nodal clearance is undertaken. Infections may run a protracted course and require polytherapy for treatment.
Lymphoedema. Usually complicates axillary node clearance or irradiation. Treatment is with manual lymphatic drainage and compression sleeves.
Seroma. This is an accumulation of fluid at the site of surgery. The fluid is usually straw coloured and may re-accumulate despite drainage. Most will resolve with time.

1050
Q

A 38 year old lady is troubled by lymphoedema that occurred following a block dissection of the groin for malignant melanoma many years previously. Despite therapy with compression bandages she has persistent lower leg swelling impairing her activities of daily living. She has no evidence of recurrent malignancy. Lymphoscintography demonstrates occlusion of the groin lymphatics. However, the distal lymphatic system appears healthy. Which of the following options could be considered?

Lymphovenous anastomosis

Homans procedure

Charles procedure

Loop diurectic therapy long term

Amputation

A

In young patients with proximal disease and healthy distal lymphatics a lymphovenous anastomosis may be considered. Such cases are rare.
Lymphoedema

  • Due to impaired lymphatic drainage in the presence of normal capillary function.
    Lymphoedema causes the accumulation of protein rich fluid, subdermal fibrosis and dermal thickening.
    Characteristically fluid is confined to the epifascial space (skin and subcutaneous tissues); muscle compartments are free of oedema. It involves the foot, unlike other forms of oedema. There may be a ‘buffalo hump’ on the dorsum of the foot and the skin cannot be pinched due to subcutaneous fibrosis.

Causes of lymphoedema

Primary
Congenital < 1 year: sporadic, Milroy’s disease
Onset 1-35 years: sporadic, Meige’s disease
> 35 years: Tarda
Secondary
Bacterial/fungal/parasitic infection (filariasis)
Lymphatic malignancy
Radiotherapy to lymph nodes
Surgical resection of lymph nodes
DVT
Thrombophlebitis

Indications for surgery
Marked disability or deformity from limb swelling
Lymphoedema caused by proximal lymphatic obstruction with patent distal lymphatics suitable for a lymphatic drainage procedure
Lymphocutaneous fistulae and megalymphatics

Procedures
Homans operation Reduction procedure with preservation of overlying skin (which must be in good condition). Skin flaps are raised and the underlying tissue excised. Limb circumference typically reduced by a third.
Charles operation All skin and subcutaneous tissue around the calf are excised down to the deep fascia. Split skin grafts are placed over the site. May be performed if overlying skin is not in good condition. Larger reduction in size than with Homans procedure.
Lymphovenous anastamosis Identifiable lymphatics are anastomosed to sub dermal venules. Usually indicated in 2% of patients with proximal lymphatic obstruction and normal distal lymphatics.

1051
Q

Which of the following structures are at risk of direct injury following a fracture dislocation of the femoral condyles?

Popliteal artery

Sciatic nerve

Plantaris muscle

Tibial artery

Tibial nerve

A

The heads of gastrocnemius will contract to pull the fracture segment posteriorly. The popliteal artery lies against the bone and may be damaged or compressed.

Popliteal fossa

Boundaries of the popliteal fossa
Laterally Biceps femoris above, lateral head of gastrocnemius and plantaris below
Medially Semimembranosus and semitendinosus above, medial head of gastrocnemius below
Floor Popliteal surface of the femur, posterior ligament of knee joint and popliteus muscle
Roof Superficial and deep fascia

Contents
Popliteal artery and vein
Small saphenous vein
Common peroneal nerve
Tibial nerve
Posterior cutaneous nerve of the thigh
Genicular branch of the obturator nerve
Lymph nodes

1052
Q

A 56 year old man is undergoing a right nephrectomy. The surgeons divide the renal artery. At what level does this usually branch off the abdominal aorta?

T9

L2

L3

T10

L4

A

The renal arteries usually branch off the aorta on a level with L2.

Renal arteries

The right renal artery is longer than the left renal artery
The renal vein/artery/pelvis enter the kidney at the hilum

Relations
Right Anterior- IVC, right renal vein, the head of the pancreas, and the descending part of the duodenum
Left Anterior- left renal vein, the tail of the pancreas

Branches
The renal arteries are direct branches off the aorta (upper border of L2- right side and L1 - left side)
In 30% there may be accessory arteries (mainly left side). Instead of entering the kidney at the hilum, they usually pierce the upper or lower part of the organ.
Before reaching the hilum of the kidney, each artery divides into four or five segmental branches (renal vein anterior and ureter posterior); which then divide within the sinus into lobar arteries supplying each pyramid and cortex.
Each vessel gives off some small inferior suprarenal branches to the suprarenal gland, the ureter, and the surrounding cellular tissue and muscles.

1053
Q

What is the first branch of the axillary artery?

Subscapular artery

Lateral thoracic artery

Thoraco acromial artery

Superior thoracic artery

Anterior circumflex humeral artery

A

The superior thoracic artery is the first branch of the axillary artery arises from the first part
Two branches arise from the second part, thoraco acromial and lateral thoracic
Three branches from the third part, subscapular artery, anterior and posterior circumflex humeral arteries
Mnemonic for axillary artery branches
Screw The Lawyer Save A patient
Superior thoracic artery
Thoracoacromial artery
Lateral thoracic artery
Subscapular artery
Anterior humeral circumflex artery
Posterior humeral circumflex artery

1054
Q

A 36-year-old woman who presented with a goitre is diagnosed with autoimmune thyroiditis. Which one of the following types of thyroid cancer is she predisposed to developing?

Anaplastic

Lymphoma

Medullary

Follicular

None of the above

A

Hashimoto’s thyroiditis is associated with thyroid lymphoma
Rarely, a patient with Hashimotos thyroiditis may develop a lymphoma of the thyroid gland. The exact aetiology of thyroid gland lymphoma is unknown. Hashimotos thyroiditis is a definite risk factor. It is possible that the lymphoma may represent the expansion of a clone of immortalised intrathyroidal lymphocytes. Lymphoma of the gland is characterised by rapid thyroid growth despite T4 therapy. Whilst adenocarcinoma of the thyroid may occur in association with thyroiditis there are no studies, to date, showing a correlation between these two conditions. The commonest sequelae of thyroiditis is hypothyroidism.

Thyroiditis

Sub acute thyroiditis
Subacute thyroiditis (also known as De Quervain’s thyroiditis) is thought to occur following viral infection and typically presents with hyperthyroidism

Features
Hyperthyroidism
Painful goitre
Raised ESR
Globally reduced uptake on iodine-131 scan

Management
Usually self-limiting - most patients do not require treatment
Thyroid pain may respond to aspirin or other NSAIDs
In more severe cases steroids are used, particularly if hypothyroidism develops

Hashimotos thyroiditis
Hashimotos thyroiditis is an immunological disorder in which lymphocytes become sensitised to thyroidal antigens. The three most important antibodies include; thyroglobulin, TPO and TSH-R. During the early phase of Hashimotos the the thyroglobulin antibody is markedly elevated and then declines.

Features
Goitre and either euthyroid or mild hypothyroidism
Progressive hypothyroidism (and associated symptoms)

Management
During the hyperthyroid phase of illness beta blockers may manage symptoms
As hypothyroidism develops patients may require thyroxine

1055
Q

Which of the following is not closely related to the capitate bone?

Lunate bone

Scaphoid bone

Ulnar nerve

Hamate bone

Trapezoid bone

A

The ulnar nerve and artery lie adjacent to the pisiform bone. The capitate bone articulates with the lunate, scaphoid, hamate and trapezoid bones, which are therefore closely related to it.

Capitate bone

This is the largest of the carpal bones. It is centrally placed with a rounded head set into the cavities of the lunate and scaphoid bones. Flatter articular surfaces are present for the hamate medially and the trapezoid laterally. Distally the bone articulates predominantly with the middle metacarpal.

1056
Q

A 77 year old morbidly obese man with type 2 diabetes presents with leg pain at rest. His symptoms are worst at night and sometimes improve during the day. He has no areas of ulceration. Which of the ABPI measurements shown below is most likely to be found?

1.0

> 1.2

0.3

0.7

0.5

A

Type 2 diabetics may have vessel calcification. This will result in abnormally high ABPI readings. Pain of this nature in diabetics is usually neuropathic and if a duplex scan is normal then treatment with an agent such as duloxetine is sometimes helpful.
Ankle-Brachial pressure index

Measurement of ankle- brachial pressure index (ABPI) is a commonly performed vascular investigation.
Calculated by dividing lower limb pressure by the highest upper limb pressure.

Results of ABPI
1.2 or greater Usually due to vessel calcification
1.0- 1.2 Normal
0.8-1.0 Minor stenotic lesion
Initiate risk factor management
0.50-0.8 Moderate stenotic lesion
Consider duplex
Risk factor management
If mixed ulcers present then avoid tight compression bandages
0.3 - 0.5 Likely significant stenosis
Duplex scanning to delineate lesions needed
Compression bandaging contra indicated
Less than 0.3 Indicative of critical ischaemia
Urgent detailed imaging required

1057
Q

An 8 year old boy is examined by his doctor as part of a routine clinical examination. The doctor notices a smooth swelling in the right iliac fossa. It is mobile and he is otherwise well. What is the most likely underlying cause?

Meckels diverticulum

Spigelian hernia

Mesenteric cyst

Appendix mass

Liposarcoma

A

Mesenteric cysts are often smooth. Imaging with ultrasound and CT is usually sufficient. Although rare, they most often occur in young children (up to 30% present before the age of 15). Many are asymptomatic and discovered incidentally. Acute presentations are recognised and may occur following cyst torsion, infarction or rupture. Most cysts will be surgically resected.
Spigelian hernias are very rare in children, liposarcomas are not smooth swellings. An appendix mass will usually produce systemic illness.

Right iliac fossa pain

Differential diagnosis

Appendicitis
Pain radiating to right iliac fossa
Anorexia (very common)
Short history
Diarrhoea and profuse vomiting rare
Crohn’s disease
Often long history
Signs of malnutrition
Change in bowel habit, especially diarrhoea
Mesenteric adenitis
Mainly affects children
Causes include Adenoviruses, Epstein Barr Virus, beta-haemolytic Streptococcus, Staphylococcus spp., Escherichia coli, Streptococcus viridans and Yersinia spp.
Patients have a higher temperature than those with appendicitis
If laparotomy is performed, enlarged mesenteric lymph nodes will be present
Diverticulitis
Both left and right sided disease may present with right iliac fossa pain
Clinical history may be similar, although some change in bowel habit is usual
When suspected, a CT scan may help in refining the diagnosis
Meckel’s diverticulitis
A Meckel’s diverticulum is a congenital abnormality that is present in about 2% of the population
Typically 2 feet proximal to the ileocaecal valve
May be lined by ectopic gastric mucosal tissue and produce bleeding
Perforated peptic ulcer
This usually produces upper quadrant pain but pain may be lower
Perforations typically have a sharp sudden onset of pain in the history
Incarcerated right inguinal or femoral hernia
Usually only right iliac fossa pain if right sided or bowel obstruction.
Bowel perforation secondary to caecal or colon carcinoma
Seldom localised to right iliac fossa, although complete large bowel obstruction with caecal distension may cause pain prior to perforation.
Gynaecological causes
Pelvic inflammatory disease/salpingitis/pelvic abscess/Ectopic pregnancy/Ovarian torsion/Threatened or complete abortion/Mittelschmerz
Urological causes
Ureteric colic/UTI/Testicular torsion
Other causes
TB/Typhoid/Herpes Zoster/AAA/Situs inversus

1058
Q

Which of the following structures is not at the level of the transpyloric plane?

Hilum left kidney

Superior mesenteric artery

Fundus of the gallbladder

Cardioesophageal junction

Root of transverse mesocolon

A

Cardiooesophageal junction level = T11
A knowledge of this anatomic level is commonly tested.
The oesophagus extends from C6 (the lower border of the cricoid cartilage) to T11 at the cardioesophageal junction. Note that in the neonate the oesophagus extends from C4 or C5 to T9.

Levels

Transpyloric plane
Level of the body of L1

Pylorus stomach
Left kidney hilum (L1- left one!)
Fundus of the gallbladder
Neck of pancreas
Duodenojejunal flexure
Superior mesenteric artery
Portal vein
Left and right colic flexure
Root of the transverse mesocolon
2nd part of the duodenum
Upper part of conus medullaris
Spleen

Can be identified by asking the supine patient to sit up without using their arms. The plane is located where the lateral border of the rectus muscle crosses the costal margin.

Anatomical planes
Subcostal plane Lowest margin of 10th costal cartilage
Intercristal plane Level of body L4 (highest point of iliac crest)
Intertubercular plane Level of body L5

Common level landmarks
Inferior mesenteric artery L3
Bifurcation of aorta into common iliac arteries L4
Formation of IVC L5 (union of common iliac veins)
Diaphragm apertures
Vena cava T8
Oesophagus T10
Aortic hiatus T12

1059
Q

A 74 year old woman presents with an erythematous rash originating in the nipple. It is spreading to the surrounding areolar area and the associated normal tissue. What is the most likely cause?

Nipple eczema

Pagets disease of the nipple

Fibroadenosis

Invasive lobular carcinoma

Radial scar

A

In most patients with Pagets disease, an underlying mammary carcinoma will be found.
Paget’s is associated with DCIS or invasive carcinoma. Unlike eczema of the nipple which predominantly affects the areolar region, Pagets will usually affect the nipple first and then spread to the areolar area. Diagnosis is made by punch biopsy.

Breast cancer

  • Commoner in the older age group
    Invasive ductal carcinomas are the most common type. Some may arise as a result of ductal carcinoma in situ (DCIS). There are associated carcinomas of special type e.g. Tubular that may carry better prognosis.
    The pathological assessment involves assessment of the tumour and lymph nodes, sentinel lymph node biopsy is often used to minimise the morbidity of an axillary dissection.
    Treatment, typically this is either wide local excision or mastectomy. There are many sub types of both of these that fall outside of the MRCS. Some key rules to bear in mind.
    Whatever operation is contemplated the final cosmetic outcome does have a bearing. A woman with small breasts and a large tumour will tend to fare better with mastectomy, even if clear pathological and clinical margins can be obtained. Conversely a women with larger breasts may be able to undergo breast conserving surgery even with a relatively large primary lesion (NB tumours >4cm used to attract recommendation for mastectomy). For screen detected and impalpable tumour image guidance will be necessary.
    Reconstruction is always an option following any resectional procedure. However, its exact type must be tailored to age and co-morbidities of the patient. The main operations in common use include latissimus dorsi myocutaneous flap and sub pectoral implants. Women wishing to avoid a prosthesis may be offered TRAM or DIEP flaps.

Surgical options
Mastectomy vs Wide local excision

Mastectomy Wide Local Excision
Multifocal tumour Solitary lesion
Central tumour Peripheral tumour
Large lesion in small breast Small lesion in large breast
DCIS >4cm DCIS <4cm
Patient Choice Patient choice

Central lesions may be managed using breast conserving surgery where an acceptable cosmetic result may be obtained, this is rarely the case in small breasts

Whatever surgical option is chosen the aim should be to have a local recurrence rate of 5% or less at 5 years [1].

Nottingham Prognostic Index
The Nottingham Prognostic Index can be used to give an indication of survival. In this system the tumour size is weighted less heavily than other major prognostic parameters.

Calculation of NPI
Tumour Size x 0.2 + Lymph node score(From table below)+Grade score(From table below).

Score Lymph nodes involved Grade
1 0 1
2 1-3 2
3 >3 3

Prognosis

Score Percentage 5 year survival
2.0 to 2.4 93%
2.5 to 3.4 85%
3.5 to 5.4 70%
>5.4 50%

This data was originally published in 1992. It should be emphasised that other factors such as vascular invasion and receptor status also impact on survival and are not included in this data and account for varying prognoses often cited in the literature.

References
For guidance on how breast cancer is managed in the United Kingdom visit the Association of Breast Surgery website (www.https://associationofbreastsurgery.org.uk/).

1060
Q

Which muscle is supplied by the superficial peroneal nerve?

Peroneus tertius

Sartorius

Adductor magnus

Peroneus brevis

Gracilis

A

Superficial peroneal nerve

Supplies
Lateral compartment of leg: peroneus longus, peroneus brevis (action: eversion and plantar flexion)
Sensation over dorsum of the foot (except the first web space, which is innervated by the deep peroneal nerve)

Path
Passes between peroneus longus and peroneus brevis along the length of the proximal one third of the fibula
10-12 cm above the tip of the lateral malleolus, the superficial peroneal nerve pierces the fascia
6-7 cm distal to the fibula, the superficial peroneal nerve bifurcates into intermediate and medial dorsal cutaneous nerves

1061
Q

Which of the following substances related to thyroid function has its effects mediated by a nuclear receptor?

Triiodothyronine

Thyroxine

Thyroglobulin

Thyroid stimulating hormone

Thyroxin binding globulin

A

T3 binds to a receptor on chromatin to induce protein synthesis.

Thyroid hormones

Hormones of the thyroid gland
Triiodothyronine T3 Major hormone active in target cells
Thyroxine T4 Most prevalent form in plasma, less biologically active than T3
Calcitonin Lowers plasma calcium

Synthesis and secretion of thyroid hormones
Thyroid actively concentrates iodide to twenty five times the plasma concentration.
Iodide is oxidised by peroxidase in the follicular cells to atomic iodine which then iodinates tyrosine residues contained in thyroglobulin.
Iodinated tyrosine residues in thyroglobulin undergo coupling to either T3 or T4.
Process is stimulated by TSH, which stimulates secretion of thyroid hormones.
The normal thyroid has approximately 3 month reserves of thyroid hormones.

LATS and Graves disease
In Graves disease, patients develop IgG antibodies to the TSH receptors on the thyroid gland. This results in chronic and long term stimulation of the gland with release of thyroid hormones. The typically situation is raised thyroid hormones and low TSH. Thyroid receptor autoantibodies should be checked in individuals presenting with hyperthyroidism as they are present in up to 85% cases.

1062
Q

At which of the following spinal levels does the oesophagus pass through the diaphragm into the abdominal cavity?

L2

L1

T10

T5

T12

A

The oesophagus passes into the abdomen at T10.

Oesophagus

  • 25cm long
    Starts at C6 vertebra, pierces diaphragm at T10 and ends at T11
    Squamous epithelium

Constrictions of the oesophagus
Structure Distance from incisors
Cricoid cartilage 15cm
Arch of the Aorta 22.5cm
Left principal bronchus 27cm
Diaphragmatic hiatus 40cm

Relations
Anteriorly
Trachea to T4
Recurrent laryngeal nerve
Left bronchus, Left atrium
Diaphragm
Posteriorly
Thoracic duct to left at T5
Hemiazygos to the left T8
Descending aorta
First 2 intercostal branches of aorta
Left
Thoracic duct
Left subclavian artery
Right
Azygos vein

Arterial, venous and lymphatic drainage of the oesophagus
Artery Vein Lymphatics Muscularis externa
Upper third Inferior thyroid Inferior thyroid Deep cervical Striated muscle
Mid third Aortic branches Azygos branches Mediastinal Smooth & striated muscle
Lower third Left gastric Left gastric Gastric Smooth muscle

Nerve supply
Upper half is supplied by recurrent laryngeal nerve
Lower half by oesophageal plexus (vagus)

Histology
Mucosa :Non-keratinized stratified squamous epithelium
Submucosa: glandular tissue
Muscularis externa (muscularis): composition varies. See table
Adventitia

1063
Q

A 44 year old man with end stage renal failure undergoes a live donor renal transplant. During the immediate post operative period a good urine output is recorded. However, on return to the ward the nursing staff notice that the urinary catheter is no longer draining. Which of the interventions listed below is most likely to be required?

Bladder wash out

Revision of the ureteric anastomosis

Revision of the venous anastomosis

Revision of the arterial anastomosis

Graft nephrectomy

A

The most likely explanation for this event is a blocked catheter. This may be the result of blood clot from the ureteric anastomosis. Bladder irrigation will usually resolve the problem.

Complications following renal transplant

Renal transplantation is widely practised. The commonest technical related complications are related to the ureteric anastomosis. The warm ischaemic time is also of considerable importance and graft survival is directly related to this. Long warm ischaemic times increase the risk of acute tubular necrosis which may occur in all types of renal transplanation and provided other insults are minimised, will usually recover. Organ rejection may occur at any phase following the transplantation process.

Immunological complications
Types of organ rejection
Hyperacute. This occurs immediately through presence of pre formed antibody (such as ABO incompatibility).
Acute. Occurs during the first 6 months and is usually T cell mediated. Usually tissue infiltrates and vascular lesions.
Chronic. Occurs after the first 6 months. Vascular changes predominate.

Hyperacute
Renal transplants are most susceptible to this process. Risk factors include major HLA mismatch and ABO incompatibility. The rejection occurs almost immediately and the macroscopic features may become manifest following completion of the vascular anastomosis and removal of clamps. The kidney becomes mottled, dusky and the vessels will thrombose. The only treatment is removal of the graft, if left in situ it will result in abscess formation.

Acute
All organs may undergo acute rejection. Mononuclear cell infiltrates predominate. All types of transplanted organ are susceptible and it may occur in up to 50% cases. Most cases can be managed medically.

Chronic
Again all transplants with HLA mismatch may suffer this fate. Previous acute rejections and other immunosensitising events all increase the risk. Vascular changes are most prominent with myointimal proliferation leading to organ ischaemia. Organ specific changes are also seen such as loss of acinar cells in pancreas transplants and rapidly progressive coronary artery disease in cardiac transplants.

Technical complications
Complication Presenting features Treatment
Renal artery thrombosis Sudden complete loss of urine output Immediate surgery may salvage the graft, delays beyond 30 minutes are associated with a high rate of graft loss
Renal artery stenosis Uncontrolled hypertension, allograft dysfunction and oedema Angioplasty is the treatment of choice
Renal vein thrombosis Pain and swelling over the graft site, haematuria and oliguria The graft is usually lost
Urine leaks Diminished urine output, rising creatinine, fever and abdominal pain USS shows perigraft collection, necrosis of ureter tip is the commonest cause and the anastomosis may need revision
Lymphocele Common complication (occurs in 15%), may present as a mass, if large may compress ureter May be drained with percutaneous technique and sclerotherapy, or intraperitoneal drainage

1064
Q

A 45-year-old man is seen in the Emergency Department with nausea, pallor and lethargy. He has no past medical history of note. A cannula is inserted and serum urea and electrolytes show the following

Na+ 140 mmol/l
K+ 6.7 mmol/l
Bicarbonate 14 mmol/l
Urea 18.2 mmol/l
Creatinine 230 micro mol/l
An ECG shows peaked T waves.
What is the most appropriate initial management?

Nebulised salbutamol

Intravenous bicarbonate

Haemodialysis

Insulin/dextrose infusion

Intravenous calcium gluconate

A

The first priority in this patient is to stabilise the myocardium with intravenous calcium gluconate.

Management of hyperkalaemia

Untreated hyperkalaemia may cause life-threatening arrhythmias. Precipitating factors should be addressed (e.g. acute renal failure) and aggravating drugs stopped (e.g. ACE inhibitors). Management may be categorised by the aims of treatment

Stabilisation of the cardiac membrane
Intravenous calcium gluconate

Short-term shift in potassium from extracellular to intracellular fluid compartment
Combined insulin/dextrose infusion
Nebulised salbutamol

Removal of potassium from the body
Calcium resonium (orally or enema)
Loop diuretics
Dialysis

1065
Q

There is decreased secretion of which one of the following hormones in response to major surgery:

Insulin

Cortisol

Renin

Anti diuretic hormone

Prolactin

A

Endocrine parameters reduced in stress response:
Insulin
Testosterone
Oestrogen
Insulin is often released in decreased quantities following surgery.

Stress response: Endocrine and metabolic changes

  • Surgery precipitates hormonal and metabolic changes causing the stress response.
    Stress response is associated with: substrate mobilization, muscle protein loss, sodium and water retention, suppression of anabolic hormone secretion, activation of the sympathetic nervous system, immunological and haematological changes.
    The hypothalamic-pituitary axis and the sympathetic nervous systems are activated and there is a failure of the normal feedback mechanisms of control of hormone secretion.

A summary of the hormonal changes associated with the stress response:

Increased Decreased No Change
Growth hormone Insulin Thyroid stimulating hormone
Cortisol Testosterone Luteinizing hormone
Renin Oestrogen Follicle stimulating hormone
Adrenocorticotrophic hormone (ACTH)
Aldosterone
Prolactin
Antidiuretic hormone
Glucagon

Sympathetic nervous system
Stimulates catecholamine release
Causes tachycardia and hypertension

Pituitary gland
ACTH and growth hormone (GH) is stimulated by hypothalamic releasing factors, corticotrophin releasing factor (CRF) and somatotrophin (or growth hormone releasing factor)
Perioperative increased prolactin secretion occurs by release of inhibitory control
Secretion of thyroid stimulating hormone (TSH), luteinizing hormone (LH) and follicle stimulating hormone (FSH) does not change significantly
ACTH stimulates cortisol production within a few minutes of the start of surgery. More ACTH is produced than needed to produce a maximum adrenocortical response.

Cortisol
Significant increases within 4-6 hours of surgery (>1000 nmol litre-1).
The usual negative feedback mechanism fails and concentrations of ACTH and cortisol remain persistently increased.
The magnitude and duration of the increase correlate with the severity of stress and the response is not abolished by the administration of corticosteroids.
The metabolic effects of cortisol are enhanced:

Skeletal muscle protein breakdown to provide gluconeogenic precursors and amino acids for protein synthesis in the liver
Stimulation of lipolysis
‘Anti-insulin effect’
Mineralocorticoid effects
Anti-inflammatory effects

Growth hormone
Increased secretion after surgery has a minor role
Most important for preventing muscle protein breakdown and promote tissue repair by insulin growth factors

Alpha Endorphin
Increased

Antidiuretic hormone
An important vasopressor and enhances haemostasis
Renin is released causing the conversion of angiotensinogen to angiotensin I
Angiotensin II formed by ACE on angiotensin 1, which causes the secretion of aldosterone from the adrenal cortex. This increases sodium reabsorption at the distal convoluted tubule

Insulin
Release inhibited by stress
Occurs via the inhibition of the beta cells in the pancreas by the α2-adrenergic inhibitory effects of catecholamines
Insulin resistance by target cells occurs later
The perioperative period is characterized by a state of functional insulin deficiency

Thyroxine (T4) and tri-iodothyronine (T3)
Circulating concentrations are inversely correlated with sympathetic activity and after surgery there is a reduction in thyroid hormone production, which normalises over a few days.

Metabolic effect of endocrine response

Carbohydrate metabolism
Hyperglycaemia is a main feature of the metabolic response to surgery
Due to increase in glucose production and a reduction in glucose utilization
Catecholamines and cortisol promote glycogenolysis and gluconeogenesis
Initial failure of insulin secretion followed by insulin resistance affects the normal responses
The proportion of the hyperglycaemic response reflects the severity of surgery
Hyperglycaemia impairs wound healing and increase infection rates

Protein metabolism
Initially there is inhibition of protein anabolism, followed later, if the stress response is severe, by enhanced catabolism
The amount of protein degradation is influenced by the type of surgery and also by the nutritional status of the patient
Mainly skeletal muscle protein is affected
The amino acids released form acute phase proteins (fibrinogen, C reactive protein, complement proteins, a2-macroglobulin, amyloid A and ceruloplasmin) and are used for gluconeogenesis
Nutritional support has little effect on preventing catabolism

Lipid metabolism
Increased catecholamine, cortisol and glucagon secretion, and insulin deficiency, promotes lipolysis and ketone body production.

Salt and water metabolism
ADH causes water retention, concentrated urine, and potassium loss and may continue for 3 to 5 days after surgery
Renin causes sodium and water retention

Cytokines
Glycoproteins
Interleukins (IL) 1 to 17, interferons, and tumour necrosis factor
Synthesized by activated macrophages, fibroblasts, endothelial and glial cells in response to tissue injury from surgery or trauma
IL-6 main cytokine associated with surgery. Peak 12 to 24 h after surgery and increase by the degree of tissue damage Other effects of cytokines include fever, granulocytosis, haemostasis, tissue damage limitation and promotion of healing.

Modifying the response
Opioids suppress hypothalamic and pituitary hormone secretion
At high doses the hormonal response to pelvic and abdominal surgery is abolished. However, such doses prolong recovery and increase the need for postoperative ventilatory support
Spinal anaesthesia can reduce the glucose, ACTH, cortisol, GH and epinephrine changes, although cytokine responses are unaltered
Cytokine release is reduced in less invasive surgery
Nutrition prevents the adverse effects of the stress response. Enteral feeding improves recovery
Growth hormone and anabolic steroids may improve outcome
Normothermia decreases the metabolic response

References
Vasiliki Manou-Stathopoulou et al. Redefining the perioperative stress response: a narrative review. Br J Anaesth. 2019 Nov;123(5):570-583.

Deborah Burton, Grainne Nicholson, and George Hall
Endocrine and metabolic response to surgery .

Contin Educ Anaesth Crit Care Pain (2004) 4(5): 144-147 doi:10.1093/bjaceaccp/mkh040

1066
Q

A 28 year old female presents with a painless lump in the upper outer quadrant of her left breast. Imaging using ultrasound is indeterminate (U3). Two core biopsies have now been performed and both show normal breast tissue (B1). What is the most appropriate course of action?

Arrange for imaging surveillance at 3 monthly intervals

Undertake a wide local excision of the lump

Reassure the patient and discharge

Undertake an excision biopsy of the lump

Arrange for a breast CT scan

A

Wide local excision and excision biopsy are different procedures.
The imaging and biopsy results are not concordant. At this stage an excision biopsy is the safest option. CT scanning is seldom helpful in breast surgery.

Breast cancer management

  • Surgery is performed in most patients suffering from breast cancer.
    Chemotherapy may be used to downstage tumours and allow breast conserving surgery. Hormonal therapy may also be used for the same purposes.
    Radiotherapy is given to most patients who have undergone breast conserving surgery (some older patients receiving hormone treatment and who have small low grade tumours may safely avoid DXT.
    Therapeutic mammoplasty is an option for some patients but requires symmetrizing surgery in most cases.
    Patients who have undergone mastectomy may be offered a reconstructive procedure either in conjunction with their primary resection or as a staged procedure at a later date.

Surgical options
Mastectomy vs Wide local excision

Mastectomy Wide Local Excision
Multifocal tumour Solitary lesion
Central tumour Peripheral tumour
Large lesion in small breast Small lesion in large breast
DCIS >4cm DCIS <4cm
Patient Choice Patient choice

Central lesions may be managed using breast conserving surgery, where an acceptable cosmetic result may be obtained, this is rarely the case in small breasts

Axillary disease
As a minimum, all patients with invasive breast cancer should have their axilla staged. In those who do not have overt evidence of axillary nodal involvement this can be undertaken using sentinel lymph node biopsy.
Patients with a positive sentinel lymph node biopsy or who have imaging and cytological or histological evidence of axillary nodal metastasis should undergo axillary node clearance or axillary irradiation.
Axillary node clearance is associated with the development of lymphoedema, increased risk of cellulitis and frozen shoulder.

1067
Q

A baby is delivered in the breech position. Barlows and Ortolani tests are normal. What is the most appropriate course of action?

Reassure and discharge the patient

Reassess the patient at 3 years

Arrange a hip USS

Arrange a hip X-ray

Arrange a hip CT scan

A

This child is at risk of developmental dysplasia of the hip (up to 20% will have DDH), so should have the hip joints scanned to exclude this.
Paediatric orthopaedics

Diagnosis Mode of presentation Treatment Radiology
Developmental dysplasia of the hip Usually diagnosed in infancy by screening tests. May be bilateral, when disease is unilateral there may be leg length inequality. As disease progresses child may limp and then early onset arthritis. More common in extended breech babies. Splints and harnesses or traction. In later years osteotomy and hip realignment procedures may be needed. In arthritis a joint replacement may be needed. However, this is best deferred if possible as it will almost certainly require revision Initially no obvious change on plain films and USS gives best resolution until 3 months of age. On plain films Shentons line should form a smooth arc
Perthes Disease Hip pain (may be referred to the knee) usually occurring between 5 and 12 years of age. Bilateral disease in 20%. Remove pressure from joint to allow normal development. Physiotherapy. Usually self-limiting if diagnosed and treated promptly. X-rays will show flattened femoral head. Eventually in untreated cases the femoral head will fragment.
Slipped upper femoral epiphysis Typically seen in obese male adolescents. Pain is often referred to the knee. Limitation to internal rotation is usually seen. Knee pain is usually present 2 months prior to hip slipping. Bilateral in 20%. Bed rest and non-weight bearing. Aim to avoid avascular necrosis. If severe slippage or risk of it occurring then percutaneous pinning of the hip may be required. X-rays will show the femoral head displaced and falling inferolaterally (like a melting ice cream cone) The Southwick angle gives indication of disease severity

1068
Q

A 22 year old man is admitted to hospital with a lower respiratory chest infection. He had a splenectomy after being involved in a car accident. What is the most likely infective organism?

Haemophilus influenzae

Staphylococcus aureus

Rhinovirus

Mycobacterium tuberculosis

Moraxella catarrhalis

A

Organisms causing post splenectomy sepsis:
Streptococcus pneumoniae
Haemophilus influenzae
Meningococci
Encapsulated organisms carry the greatest pathogenic risk following splenectomy. The effects of sepsis following splenectomy are variable. This may be the result of small isolated fragments of splenic tissue that retain some function following splenectomy. These may implant spontaneously following splenic rupture (in trauma) or be surgically implanted at the time of splenectomy.

Post splenectomy sepsis

Hyposplenism may complicate certain medical conditions where splenic atrophy occurs or may be the result of medical intervention such as splenic artery embolization and splenectomy for trauma. Diagnosis of hyposplenism is difficult and whilst there may be peripheral markers of the splenectomised state (e.g. Howell Jolly bodies) these are neither 100% sensitive or specific. The most sensitive test is a radionucleotide labeled red cell scan.
Hyposplenism, by whatever mechanism it occurs dramatically increases the risk of post splenectomy sepsis, particularly with encapsulated organisms. Since these organisms may be opsonised, but this then goes undetected at an immunological level due to loss of the spleen. For this reason individuals are recommended to be vaccinated and have antibiotic prophylaxis.

Key recommendations
All those with hyposplenism or may become so (such as prior to an elective splenectomy) should receive pneumococcal, haemophilus type b and meningococcal type C vaccines. These should be administered 2 weeks prior to splenectomy or two weeks following splenectomy. The vaccine schedule for meningococcal disease essentially consists of a dose of Men C and Hib at 2 weeks and then a dose of the MenACWY vaccine one month later. Those aged under 2 may require a booster at 2 years. A dose of pneumococcal polyvalent polysaccharide vaccine (PPV) is given at two weeks. A conjugated vaccine (PCV) is offered to young children. The PCV is more immunogenic but covers fewer serotypes. Boosting PPV is either guided by serological measurements (where available) or by routine boosting doses at 5 yearly intervals.
Annual influenza vaccination is recommended in all cases
Antibiotic prophylaxis is offered to all. The risk of post splenectomy sepsis is greatest immediately following splenectomy and in those aged less than 16 years or greater than 50 years. Individuals with a poor response to pneumococcal vaccination are another high risk group. High risk individuals should be counselled to take penicillin or macrolide prophylaxis. Those at low risk may choose to discontinue therapy. All patients should be advised about taking antibiotics early in the case of intercurrent infections.
Asplenic individuals traveling to malaria endemic areas are at high risk and should have both pharmacological and mechanical protection.

Dosing
Penicillin V 500mg BD or amoxicillin 250mg BD

References
Davies J et al. Review of guidelines for the prevention and treatment of infection in patients with an absent or dysfunctional spleen: Prepared on behalf of the British Committee for Standards in Haematology by a Working Party of the Haemato-Oncology Task Force. British Journal of Haematology 2011 (155): 308317.

1069
Q

A 48 year old woman is admitted with sepsis secondary to an infected diabetic foot ulcer. She has a necrotic and infected forefoot with necrosis of the heel. There is a boggy indurated swelling anterior to the ankle joint. Pulses are normal. What is the best course of action?

Above knee amputation

Amputation of the foot

Below knee amputation

Incision and drainage of pus

Application of 4 layer bandages

A

A below knee amputation is the best option here. The foot is non salvageable. However, she may ambulate with a prosthesis.

Amputations

Amputations are indicated when the affected limb is one of the following:
Dead non viable
Deadly where it is posing a major threat to life
Dead useless where it is viable but a prosthesis would be preferable

Orthopaedic surgery
Amputation is often undertaken as an option of last resort e.g. Limb salvage has failed and the limb is so non functional that mobility needs would be best met with prosthesis.
Chronic fracture non union or significant limb shortening following trauma would fit into this category. Occasionally following major trauma a primary amputation is preferable. This would be the case in an open fracture with major distal neurovascular compromise and other more life threatening injuries are present.

Vascular surgery
The first two categories are the most prevalent.
Diabetic foot sepsis is often a major cause of sepsis which can spread rapidly in the presence of established peripheral vascular disease.
As a general rule the main issue in vascular surgery is to optimise vascular inflow prior to surgery. The more distal the planned amputation is to be, the more important this rule becomes.
In other situations there has been something such as an embolic event that has not been revascularised in time. In this case the limb shows fixed mottling and an amputation will be needed.

Types of amputations
As the vast majority of commonly performed amputations affect the lower limbs these will be covered here.

The main categories of amputations are:
Pelvic disarticulation (hindquarter)
Above knee amputation
Gritti Stokes (through knee amputation)
Below knee amputation (using either Skew or Burgess flaps)
Syme’s amputation (through ankle)
Amputations of mid foot and digits

Choosing a level of amputation depends on:
The disease process being treated
Desired functional outcome
Co-morbidities of the patient

Above knee amputations
Quick to perform
Heal reliably
Patients regain their general health quickly
For this benefit, a functional price has to be paid and many patients over the age of 70 will never walk on an above knee prosthesis.
Above knee amputations use equal anterior-posterior flaps

Below knee amputations
Technically more challenging to perform
Heal less reliably than their above knee counterparts.
However, many more patients are able to walk using a below knee prosthesis.
In below knee amputations the two main flaps are Skew flaps or the Burgess long posterior flap. Skew flaps result in a less bulky limb that is easier to attach a prosthesis to.

It is worth remembering that whilst it may be technically feasible to offer a below knee amputation there may be circumstances where an above knee option is preferable. For example, in fixed flexion deformities of the lower limb, little functional benefit would be gained from below knee amputation surgery.

1070
Q

Which of the agents listed below is a phosphodiesterase inhibitor?

Milrinone

Metaraminol

Dopamine

Dobutamine

Adrenaline

A

A-Milrinone

Circulatory support of the critically ill

Circulatory support
Impaired tissue oxygenation may occur as a result of circulatory shock. Shock is considered further under its own topic heading.

Patients requiring circulatory support require haemodynamic monitoring. At its simplest level this may simply be in the form of regular urine output measurements and blood pressure monitoring. In addition ECG monitoring will allow the identification of cardiac arrhythmias. Pulse oximeter measurements will allow quick estimation haemoglobin oxygen saturation in arterial blood.

Invasive arterial blood pressure monitoring is undertaken by the use of an indwelling arterial line. Most arterial sites can be used although the radial artery is the commonest. It is important not to cannulate end arteries. The arterial trace can be tracked to ventilation phases and those patients whose systolic pressure varies with changes in intrathoracic pressure may benefit from further intravenous fluids.

Central venous pressure is measured using a CVP line that is usually sited in the superior vena cava via the internal jugular route. The CVP will demonstrate right atrial filling pressure and volume status. When adequate intra vascular volume is present a fluid challenge will typically cause a prolonged rise in CVP (usually greater than 6-8mmHg).

To monitor the cardiac output a Swan-Ganz catheter is traditionally inserted (other devices may be used and are less invasive). Inflation of the distal balloon will provide the pulmonary artery occlusion pressure and the pressure distal to the balloon will equate to the left atrial pressure. This gives a measure of left ventricular preload. Because the Swan-Ganz catheter can measure several variables it can be used to calculate:
Stroke volume
Systemic vascular resistance
Pulmonary artery resistance
Oxygen delivery (and consumption)

Inotropes
In patients with an adequate circulating volume but on-going circulatory compromise a vasoactive drug may be considered. These should usually be administered via the central venous route. Commonly used inotropes include:
Agent Mode of action Effect
Noradrenaline α agonist Vasopressor action, minimal effect on cardiac output
Adrenaline α and β receptor agonist Increases cardiac output and peripheral vascular resistance
Dopamine β1 agonist Increases contractility and rate
Dobutamine β1 and β2 agonist Increases cardiac output and decreases SVR
Milrinone Phosphodiesterase inhibitor Elevation of cAMP levels improves muscular contractility, short half life and acts as vasodilator

1071
Q

A 28 year old man undergoes an ileocaecal resection and end ileostomy for Crohn’s disease. One year later he presents with a deep painful ulcer at his stoma site. What is the most likely diagnosis?

Bullous pemphigoid

Pemphigus vulgaris

Pyoderma gangrenosum

Squamous cell carcinoma

Poorly fitting appliance

A

Pyoderma gangrenosum is associated with inflammatory bowel disease (this patient had a stoma for crohns!). It is commonly found on lower limbs and described as being painful, the size of an insect bite and growing. It looks like a margherita pizza (with a red base and yellow topping!) Treatment involves steroids. It would be rare for a poorly fitting appliance to cause a deep painful ulcer.

Benign skin diseases

Seborrhoeic keratosis
Most commonly arise in patients over the age of 50 years, often idiopathic
Equal sex incidence and prevalence
Usually multiple lesions over face and trunk
Flat, raised, filiform and pedunculated subtypes are recognised
Variable colours and surface may have greasy scale overlying it
Treatment options consist of leaving alone or simple shave excision

Melanocytic naevi
Congenital melanocytic naevi
Typically appear at, or soon after, birth
Usually greater than 1cm diameter
Increased risk of malignant transformation (increased risk greatest for large lesions)
Junctional melanocytic naevi
Circular macules
May have heterogeneous colour even within same lesion
Most naevi of the palms, soles and mucous membranes are of this type
Compound naevi
Domed pigmented nodules up to 1cm in diameter
Arise from junctional naevi, usually have uniform colour and are smooth
Spitz naevus
Usually develop over a few months in children
May be pink or red in colour, most common on face and legs
May grow up to 1cm and growth can be rapid, this usually results in excision
Atypical naevus syndrome
Atypical melanocytic naevi that may be autosomally dominantly inherited
Some individuals are at increased risk of melanoma (usually have mutations of CDKN2A gene)
Many people with atypical naevus syndrome AND a parent sibling with melanoma will develop melanoma

Epidermoid cysts
Common and affect face and trunk
They have a central punctum, they may contain small quantities of sebum
The cyst lining is either normal epidermis (epidermoid cyst) or outer root sheath of hair follicle (pilar cyst)

Dermatofibroma
Solitary dermal nodules
Usually affect extremities of young adults
Lesions feel larger than they appear visually
Histologically they consist of proliferating fibroblasts merging with sparsely cellular dermal tissues

Painful skin lesions
Eccrine spiradenoma
Neuroma
Glomus tumour
Leiomyoma
Angiolipoma
Neurofibroma (rarely painful) and dermatofibroma (rarely painful)

1072
Q

A 16 year old boy presents with renal colic. His parents both have a similar history of the condition. His urine tests positive for blood. A KUB style x-ray shows a relatively radiodense stone in the region of the mid ureter. What is the most likely composition of the stone?

Calcium phosphate stone

Uric acid stone

Struvite stone

Cystine stone

Calcium oxalate stone

A

Cystine stones are associated with an inherited metabolic disorder.
Renal stones

Type of stones Features Percentage of all calculi
Calcium oxalate Hypercalciuria is a major risk factor (various causes)
Hyperoxaluria may also increase risk
Hypocitraturia increases risk because citrate forms complexes with calcium making it more soluble
Stones are radio-opaque (though less than calcium phosphate stones)
Hyperuricosuria may cause uric acid stones to which calcium oxalate binds 85%
Cystine Inherited recessive disorder of transmembrane cystine transport leading to decreased absorption of cystine from intestine and renal tubule
Multiple stones may form
Relatively radiodense because they contain sulphur 1%
Uric acid Uric acid is a product of purine metabolism
May precipitate when urinary pH low
May be caused by diseases with extensive tissue breakdown e.g. malignancy
More common in children with inborn errors of metabolism
Radiolucent 5-10%
Calcium phosphate May occur in renal tubular acidosis, high urinary pH increases supersaturation of urine with calcium and phosphate
Renal tubular acidosis types 1 and 3 increase risk of stone formation (types 2 and 4 do not)
Radio-dense stones (composition similar to bone) 10%
Struvite Stones formed from magnesium, ammonium and phosphate
Occur as a result of urease producing bacteria (and are thus associated with chronic infections)
Under the alkaline conditions produced, the crystals can precipitate
Slightly radio-opaque 2-20%

Effect of urinary pH on stone formation
Urine pH will show individual variation (from pH 5-7). Post prandially the pH falls as purine metabolism will produce uric acid. Then the urine becomes more alkaline (alkaline tide). When the stone is not available for analysis the pH of urine may help to determine which stone was present.

Stone type Urine acidity Mean urine pH
Calcium phosphate Normal- alkaline >5.5
Calcium oxalate Variable 6
Uric acid Acid 5.5
Struvite Alkaline >7.2
Cystine Normal 6.5

1073
Q

A 39 year old lady undergoes a laparoscopic cholecystectomy as a daycase. The operation is more difficult than anticipated and the surgeon places a drain to the liver bed. In recovery 1.5 litres of blood is seen to enter the drain. Which of the following substances is the first to be released in this situation?

Angiotensinogen

Renin

Angiotensin I

Angiotensin II

Aldosterone

A

The decrease in blood pressure will be sensed by the juxtaglomerular cells in the kidney. This will cause renin secretion.

Shock

  • Shock occurs when there is insufficient tissue perfusion.
    The pathophysiology of shock is an important surgical topic and may be divided into the following aetiological groups:
    Septic
    Haemorrhagic
    Neurogenic
    Cardiogenic
    Anaphylactic

Septic shock
Septic shock is a major problem and those patients with severe sepsis have a mortality rate in excess of 40%. In those who are admitted to intensive care mortality ranges from 6% with no organ failure to 65% in those with 4 organ failure.

Sepsis is defined as an infection that triggers a particular Systemic Inflammatory Response Syndrome (SIRS). In 2016, the European Society of Intensive Care Medicine and the Society of Critical Care Medicine (SCCM) created a task force that proposed Sepsis-3, a new definition for sepsis. The new definition excluded the establishment of SIRS criteria to define sepsis and made it more nonspecific as any life-threatening organ dysfunction caused by the dysregulated host response to infection. The task force claimed that sequential organ failure assessment (SOFA) has a better predictive validity for sepsis than SIRS criteria. It has better prognostic accuracy and the ability to predict in-hospital mortality. To reduce the complexity of calculating the SOFA, they introduced q SOFA

Q SOFA

3 component assessment system with:

Systolic blood pressure below 100 mm Hg

Highest respiratory rate exceeding 21

Lowest Glasgow coma score is under 15

Although the validity of q SOFA is limited in an ICU setting, it has consistently outperformed SIRS criteria in predicting organ dysfunction in a non-ICU and ER setting. The use of vasopressors, mechanical ventilation, and aggressive therapeutic interventions in ICU limit the efficacy of q SOFA.

During the septic process there is marked activation of the immune system with extensive cytokine release. This may be coupled with or triggered by systemic circulation of bacterial toxins. These all cause endothelial cell damage and neutrophil adhesion. The overall hallmarks are thus those of excessive inflammation, coagulation and fibrinolytic suppression.

The main management issues with severe infections resulting in organ dysfunction are as follows:
Prompt administration of antibiotics to cover all likely pathogens coupled with a rigorous search for the source of infection.
Haemodynamic stabilisation. Many patients are hypovolaemic and require aggressive fluid administration. Aim for CVP 8-12 cm H2O, MAP >65mmHg.
Modulation of the septic response. This includes manoeuvres to counteract the changes and includes measures such as tight glycaemic control. The routine use of steroids is not advised.

In surgical patients, the main groups with septic shock include those with anastomotic leaks, abscesses and extensive superficial infections such as necrotising fasciitis. When performing surgery the aim should be to undertake the minimum necessary to restore physiology. These patients do not fare well with prolonged surgery. Definitive surgery can be more safely undertaken when physiology is restored and clotting in particular has been normalised.

Haemorrhagic shock
The average adult blood volume comprises 7% of body weight. Thus in the 70 Kg adult this will equate to 5 litres. This changes in children (8-9% body weight) and is slightly lower in the elderly.

The table below outlines the 4 major classes of haemorrhagic shock and their associated physiological sequelae:

Parameter Class I Class II Class III Class IV
Blood loss ml <750ml 750-1500ml 1500-2000ml >2000ml
Blood loss % <15% 15-30% 30-40% >40%
Pulse rate <100 >100 >120 >140
Blood pressure Normal Normal Decreased Decreased
Respiratory rate 14-20 20-30 30-40 >35
Urine output >30ml 20-30ml 5-15ml <5ml
Symptoms Normal Anxious Confused Lethargic

Decreasing blood pressure during haemorrhagic shock causes organ hypoperfusion and relative myocardial ischaemia. The cardiac index gives a numerical value for tissue oxygen delivery and is given by the equation: Cardiac index= Cardiac output/ body surface area. Where Hb is haemoglobin concentration in blood and SaO2 the saturation and PaO2 the partial pressure of oxygen. Detailed knowledge of this equation is required for the MRCS Viva but not for part A, although you should understand the principle.

In patients suffering from trauma the most likely cause of shock is haemorrhage. However, the following may also be the cause or occur concomitantly:

Tension pneumothorax
Spinal cord injury
Myocardial contusion
Cardiac tamponade

When assessing trauma patients it is worth remembering that in order to generate a palpable femoral pulse an arterial pressure of >65mmHg is required.

Once bleeding is controlled and circulating volume normalised the levels of transfusion should be to maintain a Hb of 7-8 in those with no risk factors for tissue hypoxia and Hb 10 for those who have such risk factors.

Neurogenic shock
This occurs most often following a spinal cord transection, usually at a high level. There is resultant interruption of the autonomic nervous system. The result is either decreased sympathetic tone or increased parasympathetic tone, the effect of which is a decrease in peripheral vascular resistance mediated by marked vasodilation.

This results in decreased preload and thus decreased cardiac output (Starlings law). There is decreased peripheral tissue perfusion and shock is thus produced. In contrast with many other types of shock peripheral vasoconstrictors are used to return vascular tone to normal.

Cardiogenic shock
In medical patients the main cause is ischaemic heart disease. In the traumatic setting direct myocardial trauma or contusion is more likely. Evidence of ECG changes and overlying sternal fractures or contusions should raise the suspicion of injury. Treatment is largely supportive and transthoracic echocardiography should be used to determine evidence of pericardial fluid or direct myocardial injury. The measurement of troponin levels in trauma patients may be undertaken but they are less useful in delineating the extent of myocardial trauma than following MI.

When cardiac injury is of a blunt nature and is associated with cardiogenic shock the right side of the heart is the most likely site of injury with chamber and or valve rupture. These patients require surgery to repair these defects and will require cardiopulmonary bypass to achieve this. Some may require intra aortic balloon pump as a bridge to surgery.

Anaphylactic shock
Anaphylaxis may be defined as a severe, life-threatening, generalised or systemic
hypersensitivity reaction.

Anaphylaxis is one of the few times when you would not have time to look up the dose of a medication. The Resuscitation Council guidelines on anaphylaxis have recently been updated. Adrenaline is by far the most important drug in anaphylaxis and should be given as soon as possible. The recommended doses for adrenaline, hydrocortisone and chlorpheniramine (if one wishes to use them) are as follows:

Adrenaline Hydrocortisone Chlorpheniramine
< 6 months 150 mcg (0.15ml 1 in 1,000) 25 mg 250 mcg/kg
6 months - 6 years 150 mcg (0.15ml 1 in 1,000) 50 mg 2.5 mg
6-12 years 300 mcg (0.3ml 1 in 1,000) 100 mg 5 mg
Adult and child 12 years 500 mcg (0.5ml 1 in 1,000) 200 mg 10 mg

Adrenaline can be repeated every 5 minutes if necessary. The best site for IM injection is the anterolateral aspect of the middle third of the thigh.

Common identified causes of anaphylaxis
food (e.g. Nuts) - the most common cause in children
drugs
venom (e.g. Wasp sting)

1074
Q

A 60 year old Tibetan immigrant is referred to the surgical clinic with a painless neck swelling. On examination, it is located on the left side of the neck in the anterior triangle. There are no other abnormalities to find. What is the most likely diagnosis?

Carotid body tumour

Submandibular gland calculus

Carotid artery aneurysm

Fibromatosis colli

Laryngeal cancer

A

The most likely diagnosis is a carotid body tumour and it is likely to be of the hyperplastic type.

Carotid body tumour

These are rare tumours. However, they typically account for around 60% of head and neck paraganglionomas. They are usually tumours of middle age. Around 5% are bilateral and around 5% are malignant. They are rarely found as part of the MEN II or neurofibromatosis type I disease complexes.

They typically present as an asymptomatic neck mass in the anterior triangle of the neck. They are typically slow growing lesions.

Types of carotid body tumour
Sporadic - Accounts for 85% of cases
Familial - Seen in around 10% of cases and usually in younger patients
Hyperplastic - Seen in those at high altitude or in those with COPD

Imaging
They are readily imaged using duplex ultrasonography. CT angiography is sometimes helpful.

Treatment
Typically this comprises surgical resection. This is preceded by embolization in selected cases.

1075
Q

A 56 year old man presents with jaundice. He has a long history of alcohol misuse. On examination he is jaundiced and ultrasound shows multiple echo dense lesions in both lobes of the liver. His alpha feto protein is elevated 6 times the normal range. What is the most likely diagnosis?

Gallstones

Hepatocellular carcinoma

Metastatic pancreatic adenocarcinoma

Mirizzi syndrome

Benign bile duct stricture

A

HCC may complicate cirrhosis. AFP is often raised in HCC.

Hepatocellular carcinoma

Hepatocellular carcinoma is the second leading cause of cancer deaths globally. Up to 750,000 cases are reported annually. Unfortunately the incidence approximates to the death rate so there are few long term survivors[1]. The disease occurs most commonly in those with chronic hepatitis and established liver cirrhosis. Therefore, these individuals should be closely screened for the development of HCC with serum AFP and liver USS every 6-12 months. Rising AFP and liver USS showing a nodule greater than 1cm in diameter makes HCC much more likely and such patients should then undergo MRI scanning.
The presence of adenomas in an otherwise healthy liver is a recognised risk factor for HCC [2, 3] and many surgeons will remove liver adenomas for this reason[4].

Diagnosis
The aim is to avoid unnecessary percutaneous biopsy. Radiologically on CT the classical feature is a suspicious lesion which is highlighted during the arterial phase with washout during the venous phase, this reflects the hypervascularity of the lesions.The risk of tumour seeding as a result of a liver biopsy is 2.7% with a median time interval between biopsy and seeding of 17 months[5].

Barcelona Clinic Liver Classification
There are many classification systems for addressing the management and prognosis, the BCLC system has the convenience of categorising disease extent with treatment and prognostic outcomes. In determining the ideal treatment modality for HCC the key points are not just disease extent, but also the functional state of the liver and patient.

Stage Features Treatment Prognosis-5 yr survival
Stage 0 Child-Pugh A
Single lesion (less than 2cm)
Normal portal pressures Resection 40-70%
Stage A Single nodule greater than 3cm or multiple nodules (no more than 3)
Child Pugh A/ B If associated disease then radiofrequency ablation
If no associated disease then transplantation May be up to 70% in some
Stage B Multiple nodules
Child Pugh A/B Trans arterial chemo-embolisation (usually with doxorubicin) 26% at 3 years
Stage C Advanced tumours
Invasion of portal vein
Child Pugh A/B Sorafenib Usually survive 10.7 months
Stage D Child Pugh stage C
Advanced tumours Best supportive care Less than 6 months survival
[6]
In selected patients the best outcomes are achieved with surgical resection, or transplantation where surgical resection is precluded. Anatomical resections with minimum 2cm margins provide the best outcomes.
At the present time there is no evidence to recommend treatment with adjuvant chemotherapy[6].

Sorafenib
This is an oral multi tyrosine kinase inhibitor. It is the only drug that has been currently demonstrated to extend survival in individuals with advanced hepatocellular cancer[7]. The improvement in survival is from a median of 7 months to 10 months.

References
1. Jemal, A., et al., Global cancer statistics. CA Cancer J Clin, 2011. 61(2): p. 69-90.
2. Leese, T., O. Farges, and H. Bismuth, Liver cell adenomas. A 12-year surgical experience from a specialist hepato-biliary unit. Ann Surg, 1988. 208(5): p. 558-64.
3. Farges, O. and S. Dokmak, Malignant transformation of liver adenoma: an analysis of the literature. Dig Surg, 2010. 27(1): p. 32-8.
4. Ehrl, D., et al., ‘Incidentaloma’ of the liver: management of a diagnostic and therapeutic dilemma. HPB Surg, 2012. 2012: p. 891787.
5. Silva, M.A., et al., Needle track seeding following biopsy of liver lesions in the diagnosis of hepatocellular cancer: a systematic review and meta-analysis. Gut, 2008. 57(11): p. 1592-6.
6. EASL-EORTC clinical practice guidelines: management of hepatocellular carcinoma. J Hepatol, 2012. 56(4): p. 908-43.
7. Abou-Alfa, G.K., et al., Phase II study of sorafenib in patients with advanced hepatocellular carcinoma. J Clin Oncol, 2006. 24(26): p. 4293-300.

1076
Q

A 22 year old lady is admitted to the intensive care unit following a laparotomy. She has a central line, pulmonary artery catheter and arterial lines inserted. The following results are obtained:
Pulmonary artery occlusion pressure Cardiac output Systemic vascular resistance
Low High Low
How may these findings be best interpreted?

Hypovolaemia

Septic shock

Normal

Fluid overload

Cardiogenic shock

A

Decreased SVR is a major feature of sepsis. A hyperdynamic circulation is often present. This is the reason for the use of vasoconstrictors.

Pulmonary artery occlusion pressure monitoring

The pulmonary artery occlusion pressure is an indirect measure of left atrial pressure, and thus filling pressure of the left heart. The low resistance within the pulmonary venous system allows this useful measurement to be made. The most accurate trace is made by inflating the balloon at the catheter tip and ‘floating’ it so that it occludes the vessel. If it is not possible to occlude the vessel in this way then the measurement gained will be the pulmonary artery end diastolic pressure.

Interpretation of PAOP
PAOP mmHg Scenario
Normal 8-12
Low <5 Hypovolaemia
Low with pulmonary oedema <5 ARDS
High >18 Overload

When combined with measurements of systemic vascular resistance and cardiac output it is possible to accurately classify patients.

Systemic vascular resistance
Derived from aortic pressure, right atrial pressure and cardiac output.

SVR=80(mean aortic pressure-mean right atrial pressure)/cardiac output

1077
Q

A 28 year old motorcyclist is involved in a road traffic accident and is brought to the emergency department. On examination, they are noted to have oxygen saturations of 89% and a chest x-ray shows that there is a left sided flail chest injury. What is the most appropriate management strategy?

Administration of oral analgesia

Intercostal nerve block

Intubation and ventilation

Arrange an echocardiogram

Admit for active observation

A

A flail chest injury should usually be managed by intubation if saturations are 90% or less. Beware of the potential for underlying parenchymal lung injury which may require chest drain insertion.

Thoracic trauma

Key points related to thoracic trauma
Less than 10% of blunt chest trauma and 15-30% of penetrating chest trauma requires operative intervention.
The physiologic consequences of thoracic trauma are hypoxia, hypercarbia, and acidosis. Contusion, hematoma, and alveolar collapse, or changes in intrathoracic pressure relationships (e.g., tension pneumothorax and open pneumothorax) cause hypoxia and lead to metabolic acidosis. Hypercarbia causes respiratory acidosis and most often follows inadequate ventilation caused by changes in intrathoracic pressure relationships and depressed level of consciousness.

Types of thoracic trauma

Tension pneumothorax
Often laceration to lung parenchyma with flap
Pressure develops in thorax
Most common cause is mechanical ventilation in patient with pleural injury
Symptoms overlap with cardiac tamponade, hyper-resonant percussion note is more likely in tension pnemothorax
Flail chest
Chest wall disconnects from thoracic cage
Multiple rib fractures (at least two fractures per rib in at least two ribs)
Associated with pulmonary contusion
Abnormal chest motion
Avoid over hydration and fluid overload
Pneumothorax
Most common cause is lung laceration with air leakage
Most traumatic pneumothoraces should have a chest drain
Patients with traumatic pneumothorax should never be mechanically ventilated until a chest drain is inserted
Haemothorax
Most commonly due to laceration of lung, intercostal vessel or internal mammary artery
Haemothoraces large enough to appear on CXR are treated with large bore chest drain
Surgical exploration is warranted if >1500ml blood drained immediately
Cardiac tamponade
Beck’s triad: elevated venous pressure, reduced arterial pressure, reduced heart sounds
Pulsus paradoxus
May occur with as little as 100ml blood
Pulmonary contusion
Most common potentially lethal chest injury
Arterial blood gases and pulse oximetry important
Early intubation within an hour if significant hypoxia
Blunt cardiac injury
Usually occurs secondary to chest wall injury
ECG may show features of myocardial infarction
Sequelae: hypotension, arrhythmias, cardiac wall motion abnormalities
Aorta disruption
Deceleration injuries
Contained haematoma
Widened mediastinum
Diaphragm disruption
Most due to motor vehicle accidents and blunt trauma causing large radial tears (laceration injuries result in small tears)
More common on left side
Insert gastric tube, may pass into intrathoracic stomach
Mediastinal traversing wounds
Entrance wound in one hemithorax and exit wound/foreign body in opposite hemithorax
Mediastinal haematoma or pleural cap suggests great vessel injury
Mortality is 20%

References
ATLS Manual 10th Edition. ISBN 78-0-9968262-3-5

1078
Q

Which of the following bones is related to the cuboid’s distal articular surface?

All metatarsals

5th metatarsal

Calcaneum

Medial cuneiform

3rd metatarsal

A

The cuboid is located at the lateral aspect of the foot between the calcaneus posteriorly and the 4th and 5th metatarsals distally.

Foot- anatomy

Arches of the foot
The foot is conventionally considered to have two arches.
The longitudinal arch is higher on the medial than on the lateral side. The posterior part of the calcaneum forms a posterior pillar to support the arch. The lateral part of this structure passes via the cuboid bone and the lateral two metatarsal bones. The medial part of this structure is more important. The head of the talus marks the summit of this arch, located between the sustentaculum tali and the navicular bone. The anterior pillar of the medial arch is composed of the navicular bone, the three cuneiforms and the medial three metatarsal bones.
The transverse arch is situated on the anterior part of the tarsus and the posterior part of the metatarsus. The cuneiforms and metatarsal bases narrow inferiorly, which contributes to the shape of the arch.

Intertarsal joints
Sub talar joint Formed by the cylindrical facet on the lower surface of the body of the talus and the posterior facet on the upper surface of the calcaneus. The facet on the talus is concave anteroposteriorly, the other is convex. The synovial cavity of this joint does not communicate with any other joint.
Talocalcaneonavicular joint The anterior part of the socket is formed by the concave articular surface of the navicular bone, posteriorly by the upper surface of the sustentaculum tali. The talus sits within this socket
Calcaneocuboid joint Highest point in the lateral part of the longitudinal arch. The lower aspect of this joint is reinforced by the long plantar and plantar calcaneocuboid ligaments.
Transverse tarsal joint The talocalcaneonavicular joint and the calcaneocuboid joint extend across the tarsus in an irregular transverse plane, between the talus and calcaneus behind and the navicular and cuboid bones in front. This plane is termed the transverse tarsal joint.
Cuneonavicular joint Formed between the convex anterior surface of the navicular bone and the concave surface of the the posterior ends of the three cuneiforms.
Intercuneiform joints Between the three cuneiform bones.
Cuneocuboid joint Between the circular facets on the lateral cuneiform bone and the cuboid. This joint contributes to the tarsal part of the transverse arch.
A detailed knowledge of the joints is not required for MRCS Part A. However, the contribution they play to the overall structure of the foot should be appreciated

Ligaments of the ankle joint and foot

Muscles of the foot
Muscle Origin Insertion Nerve supply Action
Abductor hallucis Medial side of the calcaneus, flexor retinaculum, plantar aponeurosis Medial side of the base of the proximal phalanx Medial plantar nerve Abducts the great toe
Flexor digitorum brevis Medial process of the calcaneus, plantar eponeurosis. Via 4 tendons into the middle phalanges of the lateral 4 toes. Medial plantar nerve Flexes all the joints of the lateral 4 toes except for the interphalangeal joint.
Abductor digit minimi From the tubercle of the calcaneus and from the plantar aponeurosis Together with flexor digit minimi brevis into the lateral side of the base of the proximal phalanx of the little toe Lateral plantar nerve Abducts the little toe at the metatarsophalangeal joint
Flexor hallucis brevis From the medial side of the plantar surface of the cuboid bone, from the adjacent part of the lateral cuneiform bone and from the tendon of tibialis posterior. Into the proximal phalanx of the great toe, the tendon contains a sesamoid bone Medial plantar nerve Flexes the metatarsophalangeal joint of the great toe.
Adductor hallucis Arises from two heads. The oblique head arises from the sheath of the peroneus longus tendon, and from the plantar surfaces of the bases of the 2nd, 3rd and 4th metatarsal bones. The transverse head arises from the plantar surface of the lateral 4 metatarsophalangeal joints and from the deep transverse metatarsal ligament. Lateral side of the base of the proximal phalanx of the great toe. Lateral plantar nerve Adducts the great toe towards the second toe. Helps maintain the transverse arch of the foot.
Extensor digitorum brevis On the dorsal surface of the foot from the upper surface of the calcaneus and its associated fascia Via four thin tendons which run forward and medially to be inserted into the medial four toes. The lateral three tendons join with hoods of extensor digitorum longus. Deep peroneal Extend the metatarsophalangeal joint of the medial four toes. It is unable to extend the interphalangeal joint without the assistance of the lumbrical muscles.

Nerves in the foot

Lateral plantar nerve
Passes anterolaterally towards the base of the 5th metatarsal between flexor digitorum brevis and flexor accessorius. On the medial aspect of the lateral plantar artery. At the base of the 5th metatarsal it splits into superficial and deep branches.

Medial plantar nerve
Passes forwards with the medial plantar artery under the cover of the flexor retinaculum to the interval between abductor hallucis and flexor digitorum brevis on the sole of the foot.

Plantar arteries
Arise under the cover of the flexor retinaculum, midway between the tip of the medial malleolus and the most prominent part of the medial side of the heel.

Medial plantar artery. Passes forwards medial to medial plantar nerve in the space between abductor hallucis and flexor digitorum brevis.Ends by uniting with a branch of the 1st plantar metatarsal artery.
Lateral plantar artery. Runs obliquely across the sole of the foot. It lies lateral to the lateral plantar nerve. At the base of the 5th metatarsal bone it arches medially across the foot on the metatarsals

Dorsalis pedis artery
This vessel is a direct continuation of the anterior tibial artery. It commences on the front of the ankle joint and runs to the proximal end of the first metatarsal space. Here is gives off the arcuate artery and continues forwards as the first dorsal metatarsal artery. It is accompanied by two veins throughout its length. It is crossed by the extensor hallucis brevis

1079
Q

Which of the structures listed below are not located within the mediastinum?

Thymus

Heart

Great vessels

Arch of azygos vein

Vertebral bodies

A

The vertebral bodies lie outside of the mediastinum, as do the lungs.

Mediastinum

Region between the pulmonary cavities.
It is covered by the mediastinal pleura. It does not contain the lungs.
It extends from the thoracic inlet superiorly to the diaphragm inferiorly.

Mediastinal regions
Superior mediastinum (between manubriosternal angle and T4/5)
Middle mediastinum
Posterior mediastinum
Anterior mediastinum

Region Contents
Superior mediastinum
Superior vena cava
Brachiocephalic veins
Arch of aorta
Thoracic duct
Trachea
Oesophagus
Thymus
Vagus nerve
Left recurrent laryngeal nerve
Phrenic nerve
Anterior mediastinum
Thymic remnants
Lymph nodes
Fat
Middle mediastinum
Pericardium
Heart
Aortic root
Arch of azygos vein
Main bronchi
Posterior mediastinum
Oesophagus
Thoracic aorta
Azygos vein
Thoracic duct
Vagus nerve
Sympathetic nerve trunks
Splanchnic nerves

1080
Q

A 56 year old lady is undergoing an adrenalectomy for Conns syndrome. During the operation the surgeon damages the middle adrenal artery and haemorrhage ensues. From which of the following structures does this vessel originate?

Aorta

Renal artery

Splenic artery

Coeliac axis

Superior mesenteric artery

A

The middle adrenal artery is usually a branch of the aorta, the lower adrenal artery typically arises from the renal vessels.

Adrenal gland anatomy

Anatomy

Location Superomedially to the upper pole of each kidney
Relationships of the right adrenal Diaphragm-Posteriorly, Kidney-Inferiorly, Vena Cava-Medially, Hepato-renal pouch and bare area of the liver-Anteriorly
Relationships of the left adrenal Crus of the diaphragm-Postero- medially, Pancreas and splenic vessels-Inferiorly, Lesser sac and stomach-Anteriorly
Arterial supply Superior adrenal arteries- from inferior phrenic artery, Middle adrenal arteries - from aorta, Inferior adrenal arteries -from renal arteries
Venous drainage of the right adrenal Via one central vein directly into the IVC
Venous drainage of the left adrenal Via one central vein into the left renal vein

1081
Q

A 13 month old boy is brought to the paediatric clinic by his mother who is concerned that his testis are not palpable. On examination his testis are not palpable either in the scrotum or inguinal region and cannot be visualised on ultrasound either. What is the most appropriate next stage in management?

Laparoscopy

Re-assess at 5 years of age

Re-assess at 13 years of age

Administration of testosterone

Administration of cyproterone acetate

A

Impalpable testes are an indication for laparoscopy. Ultrasound is a relatively unhelpful tool in evaluating cryptorchid patients and most experienced paediatric surgeons would not use it pre-operatively. They may be associated with an intra-abdominal location. Whilst it is reasonable to defer orchidopexy for retractile testis completely absent testes should be investigated further.

Cryptorchidism

A congenital undescended testis is one that has failed to reach the bottom of the scrotum by 3 months of age. At birth up to 5% of boys will have an undescended testis, post natal descent occurs in most and by 3 months the incidence of cryptorchidism falls to 1-2%. In the vast majority of cases the cause of the maldescent is unknown. A proportion may be associated with other congenital defects including:

Patent processus vaginalis
Abnormal epididymis
Cerebral palsy
Developmental delay
Wilms tumour
Abdominal wall defects (e.g. gastroschisis, prune belly syndrome)

Differential diagnosis
These include retractile testes and, in the case of absent bilateral testes the possibility of intersex conditions. A retractile testis can be brought into the scrotum by the clinician and when released remains in the scrotum. If the examining clinician notes the testis to return rapidly into the inguinal canal when released then surgery is probably indicated.

Reasons for correction of cryptorchidism
Reduce risk of infertility
Allows the testes to be examined for testicular cancer
Avoid testicular torsion
Cosmetic appearance
Males with undescended testis are 40 times as likely to develop testicular cancer (seminoma) as males without undescended testis
The location of the undescended testis affects the relative risk of testicular cancer (50% intra-abdominal testes)

Treatment
Orchidopexy at 6- 18 months of age. The operation usually consists of inguinal exploration, mobilisation of the testis and implantation into a dartos pouch.
Intra-abdominal testis should be evaluated laparoscopically and mobilised. Whether this is a single stage or two stage procedure depends upon the exact location.
After the age of 2 years in untreated individuals the Sertoli cells will degrade and those presenting late in teenage years may be better served by orchidectomy than to try and salvage a non functioning testis with an increased risk of malignancy.

1082
Q

A 34 year old man is suffering from septic shock and receives an infusion of Dextran 70. Which of the following complications may potentially ensue?

Anaphylaxis

Vomiting

Acute hepatic failure

Digital necrosis

Deep vein thrombosis

A

Dextran 40 and 70 have higher incidence of anaphylaxis than either gelatins or starches.
Dextrans are branched polysaccharide molecules. Dextran 40 and 70 are available. The higher molecular weight dextran 70 may persist for up to 8 hours. They inhibit platelet aggregation and leucocyte plugging in the microcirculation. Thereby improving flow through the microcirculation, primarily of use in sepsis.
Unlike many other intravenous fluids Dextrans are a recognised cause of anaphylaxis.

Post operative fluid management

Composition of commonly used intravenous fluids mmol-1

Na K Cl Bicarbonate Lactate
Plasma 137-147 4-5.5 95-105 22-25 -
0.9% Saline 153 - 153 - -
Dextrose / saline 30.6 - 30.6 - -
Hartmans 130 4 110 - 28

Post operative fluid management
In the UK the GIFTASUP and NICE (CG174 2017) guidelines (see reference below) were devised to try and provide some consensus guidance as to how intravenous fluids should be administered. Some time ago it was a commonly held belief that little harm would occur as a result of excessive administration of normal saline and many oliguric post operative patients received enormous quantities of IV fluids. As a result they developed hyperchloraemic acidosis. With greater understanding of this potential complication, the use of electrolyte balanced solutions (Ringers lactate/ Hartmans) is now favored over normal saline.
The other guidance includes:
Fluids given should be documented clearly and easily available
Assess the patient’s fluid status when they leave theatre
If a patient is haemodynamically stable and euvolaemic, aim to restart oral fluid intake as soon as possible
Review patients whose urinary sodium is < 20
If a patient is oedematous, hypovolaemia if present should be treated first. This should then be followed by a negative balance of sodium and water, monitored using urine Na excretion levels
Solutions such as Dextran 70 should be used in caution in patients with sepsis as there is a risk of developing acute renal injury

References
NICE guidance CG174. Intravenous fluid therapy in adults. May 2017.
NICE guidance NG29. Intravenous fluid therapy in children and young people in hospital (2020)
British Consensus Guidelines on Intravenous Fluid Therapy for Adult Surgical Patients. GIFTASUP (2009)

1083
Q

What is the approximate volume of pancreatic secretions in a 24 hour period?

100ml

200ml

500ml

1500ml

3000ml

A

Typically the pancreas secretes between 1000 and 1500ml per day.

Pancreas exocrine physiology

Composition of pancreatic secretions
Pancreatic secretions are usually 1000-1500ml per 24 hours and have a pH of 8.
Secretion Source Substances secreted
Enzymic Acinar cells Trypsinogen
Procarboxylase
Amylase
Elastase
Aqueous Ductal and Centroacinar cells Sodium
Bicarbonate
Water
Potassium
Chloride
NB: Sodium and potassium reflect their plasma levels; chloride and bicarbonate vary with flow rate

Regulation
The cephalic and gastric phases (neuronal and physical) are less important in regulating the pancreatic secretions. The effect of digested material in the small bowel stimulates CCK release and ACh which stimulate acinar and ductal cells. Of these CCK is the most potent stimulus. In the case of the ductal cells these are potently stimulated by secretin which is released by the S cells of the duodenum. This results in an increase in bicarbonate.

Enzyme activation
Trypsinogen is converted via enterokinase to active trypsin in the duodenum. Trypsin then activates the other inactive enzymes

1084
Q

An 83 year old lady presents to the clinic with early satiety. On examination, she is found to have massive splenomegaly. What is the most likely underlying cause?

Acute lymphoblastic leukaemia

Acute myelomonocytic leukaemia

Acute monoblastic leukaemia

Chronic granulocytic leukaemia

Epstein Barr virus infection

A

Long standing pathology is usually the cause of massive splenomegaly. Of these, its a common finding in chronic leukaemia. Its rare for EBV infections to cause massive splenomegaly.

Disorders affecting the spleen

One of the commonest conditions to affect the spleen is traumatic rupture and this is addressed elsewhere.

Splenomegaly
Enlargement of the spleen can commonly accompany a variety of haematological disorders. Of these, chronic conditions such as haemolytic anaemia and chronic leukaemia are often associated with splenomegaly. More acute haematological conditions tend not to cause massive splenic enlargement. Infections with parasites (such as malaria) and viruses such as Epstein Barr virus can cause splenomegaly and this one condition that is associated with sudden spontaneous splenic rupture. Portal hypertension can result in splenomegaly and the finding of an enlarged spleen on abdominal examination should prompt a search for the other stigmata of chronic liver disease.

Splenic atrophy
This typically occurs as a result of conditions that impair the blood flow to the spleen. Division of some of the major vessels can cause shrinkage of the spleen and this is commonly seen when splenic artery aneurysms are coiled for example.

1085
Q

Which of the following does not cause an increased anion gap acidosis?

Uraemia

Paraldehyde

Diabetic ketoacidosis

Ethylene glycol

Acetazolamide

A

Causes of increased anion acidosis: MUDPILES

M - Methanol
U - Uraemia
D - DKA/AKA
P - Paraldehyde/phenformin
I - Iron/INH
L - Lactic acidosis
E - Ethylene glycol
S - Salicylates
Uraemia, paraldehyde, diabetic ketoacidosis, and ethylene glycol are all known to cause increased anion gap acidosis. Uraemia is a condition in which there is a buildup of urea and other nitrogenous wastes in the blood, leading to the production of acidic metabolites. Paraldehyde is a medication used to treat alcohol withdrawal, but it can cause metabolic acidosis due to the accumulation of acetaldehyde. Diabetic ketoacidosis is a complication of diabetes in which there is an accumulation of ketone bodies in the blood, leading to metabolic acidosis. Ethylene glycol, which is found in antifreeze, can be metabolized into acidic metabolites that can cause metabolic acidosis. Acetazolamide, on the other hand, is a medication that works by inhibiting carbonic anhydrase, an enzyme that is involved in the production of bicarbonate in the body. As a result, acetazolamide can cause a non-anion gap metabolic acidosis due to the loss of bicarbonate, but it does not cause an increased anion gap acidosis.

1086
Q

A 22 year old man is involved in a fight and sustains a stab wound in his upper forearm. On examination there is a small, but deep laceration. There is an obvious loss of pincer movement involving the thumb and index finger with minimal loss of sensation. The most likely nerve injury is to the:

Ulnar nerve

Radial nerve

Anterior interosseous nerve

Axillary nerve

Median nerve

A

The anterior interosseous nerve is a motor branch of the median nerve just below the elbow. When damaged it classically causes:
Pain in the forearm
Loss of pincer movement of the thumb and index finger (innervates the long flexor muscles of flexor pollicis longus & flexor digitorum profundus of the index and middle finger)
Minimal loss of sensation due to lack of a cutaneous branch

Median nerve

The median nerve is formed by the union of a lateral and medial root respectively from the lateral (C5,6,7) and medial (C8 and T1) cords of the brachial plexus; the medial root passes anterior to the third part of the axillary artery. The nerve descends lateral to the brachial artery, crosses to its medial side (usually passing anterior to the artery). It passes deep to the bicipital aponeurosis and the median cubital vein at the elbow.
It passes between the two heads of the pronator teres muscle, and runs on the deep surface of flexor digitorum superficialis (within its fascial sheath).
Near the wrist it becomes superficial between the tendons of flexor digitorum superficialis and flexor carpi radialis, deep to palmaris longus tendon. It passes deep to the flexor retinaculum to enter the palm, but lies anterior to the long flexor tendons within the carpal tunnel.

Branches
Region Branch
Upper arm No branches, although the nerve commonly communicates with the musculocutaneous nerve
Forearm Pronator teres
Pronator quadratus
Flexor carpi radialis
Palmaris longus
Flexor digitorum superficialis
Flexor pollicis longus
Flexor digitorum profundus (only the radial half)
Distal forearm Palmar cutaneous branch
Hand (Motor) Motor supply (LOAF)
Lateral 2 lumbricals
Opponens pollicis
Abductor pollicis brevis
Flexor pollicis brevis
Hand (Sensory)
Over thumb and lateral 2 ½ fingers
On the palmar aspect this projects proximally, on the dorsal aspect only the distal regions are innervated with the radial nerve providing the more proximal cutaneous innervation.

Patterns of damage
Damage at wrist
e.g. carpal tunnel syndrome
paralysis and wasting of thenar eminence muscles and opponens pollicis (ape hand deformity)
sensory loss to palmar aspect of lateral (radial) 2 ½ fingers

Damage at elbow, as above plus:
unable to pronate forearm
weak wrist flexion
ulnar deviation of wrist

Anterior interosseous nerve (branch of median nerve)
leaves just below the elbow
results in loss of pronation of forearm and weakness of long flexors of thumb and index finger

1087
Q

A 68 year old man with diabetes presents with an area of necrosis of the perineum at the base of the scrotum, there is some surrounding erythema. He is systemically unwell and hypotensive. Which of the following organisms is likely to be accountable?

Clostridium tetani

Staphylococcus epidermidis and streptococcus viridans

Bacteroides and staphylococcus saprophyticus

Streptococcus viridans and bacteroides

E-coli and bacteroides

A

This is likely to be Fournier’s Gangrene. A number of agents are implicated. E-coli and bacteroides are the most commonly isolated organisms. The key point is that both aerobic and anaerobic organisms must be present.

This is likely to be Fournier’s Gangrene. A number of agents are implicated. E-coli and bacteroides are the most commonly isolated organisms. The key point is that both aerobic and anaerobic organisms must be present.

This is likely to be Fournier’s Gangrene. A number of agents are implicated. E-coli and bacteroides are the most commonly isolated organisms. The key point is that both aerobic and anaerobic organisms must be present.

1088
Q

A 63 year old lady is suspected as having sarcoidosis. She is sent to the general surgeons and a lymph node biopsy is performed. Which histological feature is most likely to be identified in a lymph node if sarcoid is present?

Psammoma bodies

Extensive necrosis

Dense eosinophillic infiltrates

Asteroid bodies

None of the above

A

Asteroid bodies are often found in the granulomas of individuals with sarcoid. Unlike the granulomata associated with tuberculosis the granulomas of sarcoid are rarely associated with extensive necrosis.
Psammoma bodies are concentrically lamellated calcific structures that can be found in various tissues, particularly in tumors and lesions. They are typically small, round, and resemble grains of sand under the microscope, hence the name “psammoma,” which is derived from the Greek word for sand.

These bodies are often observed in certain types of tumors, including:

1.	Papillary Thyroid Carcinoma: Psammoma bodies are commonly found in papillary thyroid carcinoma, a type of thyroid cancer.
2.	Meningioma: They can also be present in meningiomas, which are tumors that arise from the meninges, the protective membranes surrounding the brain and spinal cord.
3.	Serous Ovarian Carcinoma: Psammoma bodies can occur in serous ovarian carcinoma, a type of ovarian cancer.

Psammoma bodies are formed through a process called dystrophic calcification, where calcium salts are deposited in tissues that are undergoing degeneration or necrosis. While their exact significance is not fully understood, their presence can aid in the diagnosis of certain tumors when observed in histological specimens.

Understanding the presence of psammoma bodies in tissues can assist pathologists in identifying specific types of tumors and guiding treatment decisions for patients.

1089
Q

From which of these foraminae does the opthalmic branch of the trigeminal nerve exit the skull?

Foramen ovale

Foramen rotundum

Foramen spinosum

Superior orbital fissure

Foramen magnum

A

Mnemonic:

Standing Room Only -Exit of branches of trigeminal nerve from the skull

V1 -Superior orbital fissure
V2 -foramen Rotundum
V3 -foramen Ovale

The opthalmic branch of the trigeminal nerve exits the skull through the superior orbital fissure.
Trigeminal nerve

The trigeminal nerve is the main sensory nerve of the head. In addition to its major sensory role, it also innervates the muscles of mastication.

Distribution of the trigeminal nerve
Sensory
Scalp
Face
Oral cavity (and teeth)
Nose and sinuses
Dura mater
Motor
Muscles of mastication
Mylohyoid
Anterior belly of digastric
Tensor tympani
Tensor palati
Autonomic connections (ganglia)
Ciliary
Sphenopalatine
Otic
Submandibular

Path
Originates at the pons
Sensory root forms the large, crescentic trigeminal ganglion within Meckel’s cave, and contains the cell bodies of incoming sensory nerve fibres. Here the 3 branches exit.
The motor root cell bodies are in the pons and the motor fibres are distributed via the mandibular nerve. The motor root is not part of the trigeminal ganglion.

Branches of the trigeminal nerve
Ophthalmic nerve Sensory only
Maxillary nerve Sensory only
Mandibular nerve Sensory and motor

Sensory
Ophthalmic Exits skull via the superior orbital fissure
Sensation of: scalp and forehead, the upper eyelid, the conjunctiva and cornea of the eye, the nose (including the tip of the nose, except alae nasi), the nasal mucosa, the frontal sinuses, and parts of the meninges (the dura and blood vessels).
Maxillary nerve Exit skull via the foramen rotundum
Sensation: lower eyelid and cheek, the nares and upper lip, the upper teeth and gums, the nasal mucosa, the palate and roof of the pharynx, the maxillary, ethmoid and sphenoid sinuses, and parts of the meninges.
Mandibular nerve Exit skull via the foramen ovale
Sensation: lower lip, the lower teeth and gums, the chin and jaw (except the angle of the jaw), parts of the external ear, and parts of the meninges.

Motor
Distributed via the mandibular nerve.
The following muscles of mastication are innervated:
Masseter
Temporalis
Medial pterygoid
Lateral pterygoid

Other muscles innervated include:
Tensor veli palatini
Mylohyoid
Anterior belly of digastric
Tensor tympani

1090
Q

A 22 year old man develops an infection in the pulp of his little finger. What is the most proximal site to which this infection may migrate?

The metacarpophalangeal joint

The distal interphalangeal joint

The proximal interphalangeal joint

Proximal to the flexor retinaculum

Immediately distal to the carpal tunnel

A

The 5th tendon sheath extends from the little finger to the proximal aspect of the carpal tunnel. This carries a significant risk of allowing infections to migrate proximally.

Hand

Anatomy of the hand
Bones
8 Carpal bones
5 Metacarpals
14 phalanges
Intrinsic Muscles 8 Interossei - Supplied by ulnar nerve
4 palmar-adduct fingers
4 dorsal- abduct fingers
Intrinsic muscles Lumbricals
Flex MCPJ and extend the IPJ.
Origin deep flexor tendon and insertion dorsal extensor hood mechanism.
Innervation: 1st and 2nd- median nerve, 3rd and 4th- deep branch of the ulnar nerve.
Thenar eminence
Abductor pollicis brevis
Opponens pollicis
Flexor pollicis brevis
Hypothenar eminence
Opponens digiti minimi
Flexor digiti minimi brevis
Abductor digiti minimi

Fascia and compartments of the palm
The fascia of the palm is continuous with the antebrachial fascia and the fascia of the dorsum of the hand. The palmar fascia is thin over the thenar and hypothenar eminences. In contrast, the central palmar fascia is relatively thick. The palmar aponeurosis covers the soft tissues and overlies the flexor tendons. The apex of the palmar aponeurosis is continuous with the flexor retinaculum and the palmaris longus tendon. Distally, it forms four longitudinal digital bands that attach to the bases of the proximal phalanges, blending with the fibrous digital sheaths.
A medial fibrous septum extends deeply from the medial border of the palmar aponeurosis to the 5th metacarpal. Lying medial to this are the hypothenar muscles. In a similar fashion, a lateral fibrous septum extends deeply from the lateral border of the palmar aponeurosis to the 3rd metacarpal. The thenar compartment lies lateral to this area.
Lying between the thenar and hypothenar compartments is the central compartment. It contains the flexor tendons and their sheaths, the lumbricals, the superficial palmar arterial arch and the digital vessels and nerves.
The deepest muscular plane is the adductor compartment, which contains adductor pollicis.

Short muscles of the hand
These comprise the lumbricals and interossei. The four slender lumbrical muscles flex the fingers at the metacarpophalangeal joints and extend the interphalangeal joint. The four dorsal interossei are located between the metacarpals and the four palmar interossei lie on the palmar surface of the metacarpals in the interosseous compartment of the hand.

Long flexor tendons and sheaths in the hand
The tendons of FDS and FDP enter the common flexor sheath deep to the flexor retinaculum. The tendons enter the central compartment of the hand and fan out to their respective digital synovial sheaths. Near the base of the proximal phalanx, the tendon of FDS splits to permit the passage of FDP. The FDP tendons are attached to the margins of the anterior aspect of the base of the distal phalanx.
The fibrous digital sheaths contain the flexor tendons and their synovial sheaths. These extend from the heads of the metacarpals to the base of the distal phalanges.

Palmar Interossei
Note that there are 4 palmar interossei. The first is a small slip of muscle which arises from the ulnar side of the base of the first metacarpal and passes between the head of the first dorsal interosseous and the oblique head of adductor pollicis to insert into the ulnar base of the of the proximal phalanx of the thumb. The second arises from the ulnar side of the body of the second metacarpal and is inserted into the ulnar side of the extensor hood of the index. The third and fourth palmar interossei arise from the radial sides of the bodies of the 4th and 5th metacarpals respectively and insert into the radial sides of the extensor hoods of the ring and little fingers.

1091
Q

A 32 year old man who has suffered from Crohns disease for many years presents with intermittent jaundice. When it occurs, it is obstructive in nature. It then usually resolves spontaneously. What is the most likely cause?

Crigler Najjar syndrome

Parasitic infection of the liver

Bile duct stones

Gilberts syndrome

Multi cystic liver disease

A

Bile salts are absorbed in the terminal ileum. When this process is impaired as in Crohns the patient may develop gallstones, if these pass into the CBD then obstructive jaundice will result.

Surgical jaundice

Jaundice can present in a manner of different surgical situations. As with all types of jaundice a careful history and examination will often give clues as to the most likely underlying cause. Liver function tests whilst conveying little in the way of information about liver synthetic function, will often facilitate classification as to whether the jaundice is pre hepatic, hepatic or post hepatic. The typical LFT patterns are given below:

Location Bilirubin ALT/ AST Alkaline phosphatase
Pre hepatic Normal or high Normal Normal
Hepatic High Elevated (often very high) Elevated but seldom to very high levels
Post hepatic High-very high Moderate elevation High- very high

In post hepatic jaundice the stools are often of pale colour and this feature should be specifically addressed in the history.

Modes of presentation
These are addressed in the table below:

Diagnosis Typical features Pathogenesis
Gallstones Typically history of biliary colic or episodes of chlolecystitis. Obstructive type history and test results. Usually small calibre gallstones which can pass through the cystic duct. In Mirizzi syndrome the stone may compress the bile duct directly- one of the rare times that cholecystitis may present with jaundice
Cholangitis Usually obstructive and will have Charcots triad of symptoms (pain, fever, jaundice) Ascending infection of the bile ducts usually by E. coli and by definition occurring in a pool of stagnant bile.
Pancreatic cancer Typically painless jaundice with palpable gallbladder (Courvoisier’s Law) Direct occlusion of distal bile duct or pancreatic duct by tumour. Sometimes nodal disease at the portal hepatis may be the culprit in which case the bile duct may be of normal calibre.
TPN associated jaundice Usually follows long term use and is usually painless with non obstructive features Often due to hepatic dysfunction and fatty liver which may occur with long term TPN usage.
Bile duct injury Depending upon the type of injury may be of sudden or gradual onset and is usually of obstructive type Often due to a difficult cholecystectomy when anatomy in Calots triangle is not appreciated. In the worst scenario the bile duct is excised and jaundice offers rapidly post operatively. More insidious is that of bile duct stenosis which may be caused by clips or diathermy injury.
Cholangiocarcinoma Gradual onset obstructive pattern Direct occlusion by disease and also extrinsic compression by nodal disease at the porta hepatis.
Septic surgical patient Usually hepatic features Combination of impaired biliary excretion and drugs such as ciprofloxacin which may cause cholestasis.
Metastatic disease Mixed hepatic and post hepatic Combination of liver synthetic failure (late) and extrinsic compression by nodal disease and anatomical compression of intra hepatic structures (earlier)

Diagnosis
An ultrasound of the liver and biliary tree is the most commonly used first line test. This will establish bile duct calibre, often ascertain the presence of gallstones, may visualise pancreatic masses and other lesions. The most important clinical question is essentially the extent of biliary dilatation and its distribution.

Where pancreatic neoplasia is suspected, the next test should be a pancreatic protocol CT scan. With liver tumours and cholangiocarcinoma an MRI/ MRCP is often the preferred option. PET scans may be used to stage a number of malignancies but do not routinely form part of first line testing.

Where MRCP fails to give adequate information an ERCP may be necessary. In many cases this may form part of patient management. It is however, invasive and certainly not without risk and highly operator dependent.

Management
Clearly this will depend to an extent upon the underlying cause but relief of jaundice is important, even if surgery forms part of the planned treatment. Patients with unrelieved jaundice have a much higher incidence of septic complications, bleeding and death.

Screen for and address any clotting irregularities

In patients with malignancy a stent will need to be inserted. These come in two main types; metal and plastic. Plastic stents are cheap and easy to replace and should be used if any surgical intervention (e.g. Whipples) is planned. However, they are prone to displacement and blockage. Metal stents are much more expensive and may compromise a surgical resection. However, they are far less prone to displacement and to a lesser extent blockage than their plastic counterparts.

If malignancy is in bile duct/ pancreatic head and stenting has been attempted and has failed, then an alternative strategy is to drain the biliary system percutaneously via a transhepatic route. It may also be possible to insert a stent in this way. One of the main problems with temporary PTC’s is their propensity to displacement, which may result in a bile leak.

In patients who have a bile duct injury surgery will be required to repair the defect. If the bile duct has been inadvertently excised then a hepatico-jejunostomy will need to be created (difficult!)

If gallstones are the culprit, then these may be removed by ERCP and a cholecystectomy performed. Where there is doubt about the efficacy of the ERCP an operative cholangiogram should be performed and bile duct exploration undertaken where stones remain. When the bile duct has been formally opened the options are between closure over a T tube, a choledochoduodenostomy or choledochojejunostomy.

Patients with cholangitis should receive high dose broad spectrum antibiotics via the intravenous route. Biliary decompression should follow soon afterwards, instrumenting the bile duct of these patients will often provoke a septic episode (but should be done anyway).

1092
Q

Which of the anaesthetic agents listed below is associated with hepatotoxicity?

Halothane

Sevoflurane

Propofol

Ketamine

Desflurane

A

Halothane is largely of historical interest and that is because of its hepatotoxicity.

Anaesthetic agents

The table below summarises some of the more commonly used IV induction agents
Agent Specific features
Propofol
Rapid onset of anaesthesia
Pain on IV injection
Rapidly metabolised with little accumulation of metabolites
Proven anti emetic properties
Moderate myocardial depression
Widely used especially for maintaining sedation on ITU, total IV anaesthesia and for daycase surgery
Sodium thiopentone
Extremely rapid onset of action making it the agent of choice for rapid sequence of induction
Marked myocardial depression may occur
Metabolites build up quickly
Unsuitable for maintenance infusion
Little analgesic effects
Ketamine
May be used for induction of anaesthesia
Has moderate to strong analgesic properties
Produces little myocardial depression making it a suitable agent for anaesthesia in those who are haemodynamically unstable
May induce state of dissociative anaesthesia resulting in nightmares
Etomidate
Has favorable cardiac safety profile with very little haemodynamic instability
No analgesic properties
Unsuitable for maintaining sedation as prolonged (and even brief) use may result in adrenal suppression
Post operative vomiting is common

1093
Q

A 23 year old man falls and injures his hand. There are concerns that he may have a scaphoid fracture as there is tenderness in his anatomical snuffbox on clinical examination. Which of the following forms the medial border of this structure?

Basilic vein

Radial artery

Extensor pollicis brevis

Abductor pollicis longus

Extensor pollicis longus

A

It’s boundaries are extensor pollicis longus medially and laterally by the tendons of abductor pollicis longus and extensor pollicis brevis.

Anatomical snuffbox

Posterior border Tendon of extensor pollicis longus
Anterior border Tendons of extensor pollicis brevis and abductor pollicis longus
Proximal border Styloid process of the radius
Distal border Apex of snuffbox triangle
Floor Trapezium and scaphoid
Content Radial artery

1094
Q

A 63 year old lady undergoes an axillary clearance for breast cancer. She makes steady progress. However, 8 weeks post operatively she still suffers from severe shoulder pain. On examination, she has reduced active movements in all planes and loss of passive external rotation. What is the most likely cause?

Metastatic cancer

Adhesive capsulitis

Rotator cuff tear

Osteoarthritis

Rheumatoid disease

A

Frozen shoulder is more likely. Metastatic cancer can cause pain. However, passive movements are normally unaffected.
Frozen shoulder passes through an initial painful stage followed by a period of joint stiffness. With physiotherapy the problem will usually resolve although it may take up to 2 years to do so.

Shoulder disorders

Shoulder fractures and dislocations
Fractures
Proximal humerus
Background
Third most common fragility fracture in the elderly.
Results from low energy fall in predominantly elderly females, or from high energy trauma in young males.
Can be associated with nerve injury (commonly axillary), and fracture-dislocation of the humeral head. Detailed neurological assessment is essential for all upper limb injuries.

Anatomy
Osteology
Consists of articular head, greater tuberosity, lesser tuberosity, metaphysis and diaphysis. Between the articular head and the tuberosities is the anatomical neck (previous physis). Between the tuberosities and the metaphysis is the surgical neck.
The supraspinatus, infraspinatus and teres minor muscles attach to the greater tuberosity. The subscapularis muscle attaches to the lesser tuberosity.

Vascular Supply
Humeral head is supplied by the anterior and posterior humeral circumflex arteries. Anatomical neck fractures are at greatest risk of osteonecrosis.

Imaging
Imaging aims to both delineate the fracture pattern, and confirm/exlude the presence of an associated dislocation.
Radiographs - True anteroposterior (AP), axillary lateral and/or scapula Y view.
CT - indicated to better define intra-articular involvement and to aid pre-operative planning. MRI is not useful for fracture imaging.

Classification
Description of the fracture is often more useful than classification. Particular attention should be paid to humeral alignment, fracture displacement, and greater tuberosity position (rotator cuff will pull the GT supero-posterioly, which can cause impingement problems with malunion).
- Neer Classification: Most commonly used. Describes fracture as 2,3,or 4 part depending upon the number main fragments. Also comments on the degree of displacement. Fragments:
-greater tuberosity
-lesser tuberosity
- articular surface
- shaft
Displacement: >1cm or angulation >45 degrees.

Treatment
The vast majority of proximal humeral fractures are minimally displaced, and therefore can be managed conservatively. This involves immobilisation in a polysling, and progressive mobilisation. Pendular exercise can commence at 14 days, and active abduction from 4-6 weeks.

Irreducible fracture dislocation is an indication for operative management. Other indications include large displacement, younger patient, head splitting (intra-articular fractures). However, the recent PROFHER trial (1) has suggested no benefit to operative intervention on patient outcome (it must be applied cautiously as majority of patients were elderly with extraarticular fractures). Options available for surgical management include:

ORIF Most commonly used. Plate and screw fixation. Can reconstruct complex fractures.
Intramedullary nail Suitable for extra-articular configuration, predominantly surgical neck +/- GT fractures.
Hemiarthroplasty Used for un-reconstructable fractures in the older patient who has good glenoid quality.
Total shoulder arthroplasty Unconstructable fractures where high functioning shoulder is required (hemiarthroplasty will cause glenoid erosion)
Reverse shoulder arthroplasty Total shoulder arthroplasty that provides better functional outcome than conventional total shoulder replacement.

Scapula
Background
Uncommon fractures usually associated with high energy trauma. Most commonly involve scapula body or spine (50%), glenoid fossa and glenoid neck. Important to exclude associated life threatening injury.

Imaging
Plain radiographs should include true anteroposterior (AP), axillary lateral and/or scapula Y view. CT scanning is useful for defining intra-articular involvement, displacement and for three dimensional reconstruction.

Classification
Based on the location of the fracture (coracoid, acromion, glenoid neck, glenoid fossa, scapula body). Beware of ipsilateral glenoid neck and clavicle fracture -floating shoulder - where limb is effectively dissociated from axial skeleton.

Treatment
The vast majority of scapula fractures are amenable to conservative management, consisting of sling immobilisation for two weeks followed by early rehabilitation. Floating shoulder will usually require fixation, and consideration of surgery should also be given to intra-articular and displaced/angulated glenoid fractures.

Dislocations

Types
Dislocations around the shoulder joint include glenohumeral dislocation, acromioclavicular joint disruption and sternoclavicular dislocation. Only glenohumeral dislocation will be covered here.

Glenohumeral dislocation
Diagnosis, classification and management are covered here.

Background
Shoulder dislocation is commonly seen in A&E. It has a high recurrence rate that is as high as 80% in teenagers. Initial management requires emergent reduction to prevent lasting chondral damage.

Early assessment and management
Usually a traumatic cause (multi-directional instability in frequent dislocations requires discussion with orthopaedics and is not covered here). Careful history, examination and documentation of neurovascular status of the limb, in particular the axillary nerve (regimental badge sensation). This should be re-assessed post manipulation. Early radiographs to confirm direction of dislocation.

Initial management consists of emergent closed reduction under under entanox and analgesia, but often requires conscious sedation. Arm should then be immobilised in a polysling, and XR to confirm relocation.

Imaging - True anteroposterior (AP), axillary lateral and/or scapula Y view. Reduced humeral head should lie between acromion and coracoid on lateral/scapula view.

Types
Direction Features Cause Examination Reduction techniques
Anterior Most Common >90% Usually traumatic - anterior force on arm when shoulder is abducted, externally rotated Loss of shoulder contour - sulcus sign. Humeral head can be felt anteriorly.
Hippocratic.
Milch.
Stimson.

Kocher not advised due to complication of fracture
Posterior 50% missed in A&E 50% traumatic, but classically post seizure or electrocution Shoulder locked in internal rotation. XR may show lightbulb appearance. Gentle lateral traction to adducted arm.
Inferior Rare Associated with pectorals and rotator cuff tears, and glenoid fracture As for primary injury Management of primary injury
Superior Rare Associated with acrominon/clavicle fracture As for primary injury Management of primary injury

Associated injuries
Bankart lesion - avulsion of the anterior glenoid labrum with an anterior shoulder dislocation (reverse Bankart if poster labrum in posterior dislocation).
Hill Sachs defect - chondral impaction on posteriosuperior humeral head from contact with gleonoid rim. Can be large enough to lock shoulder, requiring open reduction. (Reverse Hill Sachs in posterior dislocation).
Rotator cuff tear - increases with age.
Greater or lesser tuberosity fracture - increases with age.
Humeral neck fracture - shoulder fracture dislocation. More common in high energy trauma and elderly. Should be discussed with orthopaedics prior to any attempted reduction.

Rotator Cuff Disease

Rotator cuff disease is a spectrum of conditions that ranges from subacromial impingement to rotator cuff tears and eventually to rotator cuff arthropathy (arthritis).

Anatomy
The rotator cuff is a group of four muscles that are important in shoulder movements, and maintenance of glenohumeral stability.
Muscle Scapular attachment Humeral attachment Action Innervation
Supraspinatus Supraspinatus fossa Superior facet of greater tuberosity Initiation of abduction of humerus Suprascapular nerve
Infraspinatus Infraspinatus fossa Posterior facet of greater tuberosity External rotation of humerus Suprascapular nerve
Teres Minor Lateral border Inferior facet of greater tuberosity External rotation of humerus Axillary Nerve
Subscapularis Subscapular fossa Lesser tuberosity Internal rotation of humerus Upper and lower subscapular nerve

The inferior rotator cuff muscles (infraspinatus, teres minor, and subscapularis) balance the superior pull of the deltoid. Injury/tear results in upward migration of the humeral head on the glenoid (can be seen on AP radiograph).
Likewise, the anterior muscles (subscapularis) are balanced with the posterior muscles (infraspinatus, teres minor).

Subacromial Impingement

The most common cause of shoulder pain, which results from impingement of the superior cuff on the undersurface of the acromion, and an inflammatory bursitis.
Associated with certain types of acromial morphology (Bigliani classification).
Presents as insidious pain which is exacerbated by overhead activities.

Rotator Cuff Tear

Often presents as an acute event on the background of chronic subacromial impingement in the older patient, but can present as an avulsion injury in younger patients.
Majority of tears are to the superior cuff (supraspinatus, infraspinatus, teres minor), though a tear to subscapularis is associated with subcoracoid impingement.
Tears present as pain and weakness when using the muscles in question.

Rotator Cuff Arthropathy

Defined as shoulder arthritis in the setting of rotator cuff dysfunction. Results from superior migration due to the loss of rotator cuff function and integrity. Unopposed deltoid pulls the humeral head superiorly.
Associated with massive chronic cuff tears.

Imaging

Plain radiographs
AP of the shoulder may show superior migration of the humerus with a cuff tear, and features of arthritis with arthropathy. Other causes of pain may also be identified (e.g. calcific tendonitis/fracture)
Outlet view is useful for defining the acromial morphology

USS
Allows dynamic imaging of the cuff, and is inexpensive. However, it is very user dependent.

MRI
Best imaging modality for cuff pathology.
Also allows imaging of the rest of the shoulder. When intra-articular pathology is suspected, can be combined with an arthrogram for improved sensitivity and specificity.

Treatment

Subacromial impingement
Physiotherapy, oral anti-inflammatory medication
Subacromial steroid injection can settle inflammation
Arthroscopic subacromial decompression by shaving away the undersurface of the acromion, more space is created for the rotator cuff. Cuff integrity is assessed also at time of surgery, and can be repaired if necessary.

Rotator cuff tear
When considering repair of a cuff tear, the age and activity of the patient, the nature of the tear (degenerative vs. acute traumatic), and the size and retraction of the tear should be considered when making a surgical plan.
Mild tears or tears in the elderly can be managed conservatively, as outlined above.
Moderate tears can be repaired arthroscopically. Massive or retracted tears will often require an open repair (occasionally with a tendon transfer). Subacromial decompression is performed at the same time to reduce impingement, symptoms and recurrence.

Calcific tendonitis
Calcific tendonitis involves calcific deposits within tendons anywhere in the body, but most commonly in the rotator cuff (specifically the supraspinatus tendon). When present in the shoulder, it is associated with subacromial impingement and pain.

Pathology
More common in women aged 30-60 years.
Association with diabetes and hypothyroidism

There are three stages of calcification
Formative phase characterized by calcific deposits
Resting phase deposit is stable, but presents with impingement problems
Resorptive phase phagocytic resorption. Most painful stage.

Presentation
Similar in presentation to subacromial impingement, with pain especially with over head activities. Atraumatic in nature.

Imaging
Plain radiographs show calcification of the rotator cuff, usually within 1.5cm of its insertion on the humerus. Supraspinatus outlet views can show level of impingment. Further imaging is rarely needed.

Treatment
Non-operative NSAIDS, steroid injection (controversial, but practiced) and physiotherapy. Approximately 75% will resolve by 6 months with conservative management.
Ultrasound guided or surgical needle barbotage can break down deposits and resolve symptoms. Occasionally surgical excision is required.

Adhesive capsulitis (Frozen Shoulder)
Pain and loss of movement of shoulder joint, which involves fibroplastic proliferation of capsular tissue, causing soft tissue scarring and contracture. Patients present with a painful and decreased arc of motion.
Associated with prolonged immobilization, previous surgery, thyroid disorders (AI) and diabetes
Classically three stages which can take up to two years to resolve:
Stage one the freezing and painful stage
Stage two the frozen and stiff stage
Stage three the thawing stage, where shoulder movement slowly improves

Imaging
Plain radiographs to exclude other causes of a painful shoulder
MRI arthrogram may show capsular contracture, and again may be used to exclude cuff pathology. However, often not performed as diagnosis is largely clinical.

Treatment
Non-operative NSAIDS, steroid injection and physiotherapy. Patience is required as condition can take up to 2 years to improve.
Operative MUA or arthroscopic adhesiolysis (release of adhesions) can expedite recovery, followed by intensive physiotherapy.

Glenohumeral Arthritis
Background
May be osteoarthritis (primary or secondary to cuff tear or trauma), rheumatoid arthritis, or as part of a spondyloarthropathy. Majority of those with RA will develop symptoms.
More common in the elderly
Presents like any other arthritis - pain at night and with movement

Imaging
AP and axillary radiographs will show features of arthritis.
CT/MRI is often useful to classify the shape of the glenoid and extent of bone loss when considering arthroplasty. MRI also essential to asses integrity of rotator cuff if considering shoulder replacement.

Treatment
Like all orthopaedics, start with simple measures:
NSAIDS, management of RA, physiotherapy, steroid injection.
Hemiarthroplasty can sometimes be considered if glenoid is in excellent condition or if patient has large comorbidity.
Arthroscopic debridement is useful if patient has isolated ACJ arthritis, but is rarely used for glenohumeral arthritis.
Total shoulder replacement is shown to produce superior outcome when compared to hemiarthroplasty in terms of pain relief, function and implant survival.
Total shoulder replacement can be anatomical (ball on humerus, with cup on glenoid), or reverse geometry (ball on glenoid, with cup on humerus). Anatomical TSR requires an in tact rotator cuff, so often reverse is preferable when the cuff if questionable in integrity.

References
1. JAMA. 2015;313(10):1037-1047. doi:10.1001/jama.2015.1629

1095
Q

An 18 year old man is stabbed in the neck and has to undergo repair of a laceration to the internal carotid artery. Post operatively he is noted to have a Horners syndrome. Which of the following will not be present?

Apparent enopthalmos

Loss of sweating on the entire ipsilateral side of the face

Constricted pupil

Mild ptosis

Normal sympathetic activity in the torso

A

The anhidrosis will be mild as this is a distal lesion and at worst only a very limited area of the ipsilateral face will be anhidrotic.

Horners syndrome

Horners syndrome, clinical features:
Ptosis
Miosis
Enopthalmos
Anhydrosis

Primarily a disorder of the sympathetic nervous system. Extent of symptoms depends upon the anatomical site of the lesion.

Proximal lesions occur along the hypothalamospinal tract

Distal lesions are usually post ganglionic e.g. at level of internal carotid artery or beyond.

1096
Q

Which of the drugs listed below confers the greatest risk of malignant hyperthermia?

Decamethonium halides

Suxamethonium

Benzquinonium

Gallamine

Vecuronium

A

Suxamethonium may cause malignant hyperthermia and 1 in 2800 will have abnormal cholinesterase enzyme and prolonged clinical effect.

Muscle relaxants

Suxamethonium
Depolarising neuromuscular blocker
Inhibits action of acetylcholine at the neuromuscular junction
Degraded by plasma cholinesterase and acetylcholinesterase (affected by lack of acetylcholinesterase)
Fastest onset and shortest duration of action of all muscle relaxants
Produces generalised muscular contraction prior to paralysis
Adverse effects include hyperkalaemia, malignant hyperthermia, delayed recovery
Atracurium
Non depolarising neuromuscular blocking drug
Duration of action usually 30-45 minutes
Generalised histamine release on administration may produce facial flushing, tachycardia and hypotension
Not excreted by liver or kidney, broken down in tissues by hydrolysis
Reversed by neostigmine
Vecuronium
Non depolarising neuromuscular blocking drug
Duration of action approximately 30 - 40 minutes
Degraded by liver and kidney and effects prolonged in organ dysfunction
Effects may be reversed by neostigmine
Pancuronium
Non depolarising neuromuscular blocker
Onset of action approximately 2-3 minutes
Duration of action up to 2 hours
Effects may be partially reversed with drugs such as neostigmine

1097
Q

A child is brought to casualty complaining of a headache and a sensation of pressure between the eyes. On examination, she is febrile with a smooth swelling overlying the superomedial aspect of the right eye. The eye is uncomfortable and there is a purulent discharge from the inner canthus. What is the most likely cause?

Maxillary sinusitis

Ethmoidal sinusitis

Nasal polyps

Ethmoid sinus cancer

Maxillary sinus cancer

A

Ethmoidal sinusitis may spread to the periorbital tissues resulting in periorbital cellulitis. The superomedial distribution makes a maxillary sinusitis less likely.

Diseases of nose and sinuses

Benign Tumours
Simple papillomas may be an incidental finding or present with obstructive symptoms. Excision under general anaesthesia is sufficient management.
Transitional cell papillomas may be more extensive and produce obstructive symptoms. Erosion of local structures is a recognised complication. These lesions may rarely undergo malignant transformation and therefore careful and complete excision is required, some cases may require partial or total maxillectomy.
Pleomorphic adenomas of the maxillary sinuses are reported but are extremely rare, their symptoms typically include nasal obstruction and pain if the sinus is obstructed. Treatment is by complete surgical excision, the diagnosis is not infrequently made post operatively.
Benign osteomas may develop in the paranasal sinuses, the frontal sinus is the most frequent location of such lesions. Symptoms include; pain, rhinorrhoea and anosmia. Most osteomas may be observed if asymptomatic, sphenoid osteomas should be resected soon after diagnosis as enlargement may compromise visual fields. Many sinus osteomas can now be resected endoscopically, complete surgical resection is required.
Nasal polyps are benign lesions of the ethmoid sinus mucosa. Many patients may also have asthma, cystic fibrosis and a sensitivity to aspirin. Symptoms include watery rhinorrhoea, infection and anosmia. The polyps are usually a semi transparent grey mass. They are rare in childhood. Treatment is either with systemic steroids or surgical resection. The latter should be combined with antral washout. Low dose, nasal, steroid drops may reduce the risk of recurrence.

Malignant disease
Malignancies encountered in the nose and paranasal sinuses include; adenoid cystic carcinoma, squamous cell carcinoma and adenocarcinoma.
Adenocarcinoma of the paranasal sinuses and nasopharynx is strongly linked to exposure to hard wood dust (after >10 years exposure).
Adenoid cystic carcinoma usually originate in the smaller salivary glands.
The majority of cancers (50%) arise from the lateral nasal wall, a smaller number (33%) arise from the maxillary antrum, ethmoid and sphenoid cancers comprise only 7%.
Signs of malignancy on clinical examination include loose teeth, cranial nerve palsies and lymphadenopathy.
Nasopharyngeal cancers are most common in individuals presenting from China and Asia and are linked to viral infection with Epstein Barr Virus. Radiotherapy and chemotherapy are the most commonly used modalities.

Maxillary sinusitis
Common symptoms include post nasal discharge, pain, headache and toothache.
Imaging may show a fluid level in the antrum.
Common organisms include Haemophilus influenzae or Streptococcus pneumoniae.
Treatment with antral lavage may facilitate diagnosis and relieve symptoms. Antimicrobial therapy has to be continued for long periods. Antrostomy may be needed.

Frontoethmoidal sinusitis
Usually presents with frontal headache, nasal obstruction and altered sense of smell.
Inflammation may progress to involve periorbital tissues. Ocular symptoms may occur and secondary CNS involvement brought about by infection entering via emissary veins.
CT scanning is the imaging modality of choice. Early cases may be managed with antibiotics. More severe cases usually require surgical drainage.

1098
Q

Which of the following is not contained within the middle mediastinum?

Main bronchi

Arch of the azygos vein

Thoracic duct

Pericardium

Aortic root

A

The thoracic duct lies within the posterior and superior mediastinum.

Mediastinum

Region between the pulmonary cavities.
It is covered by the mediastinal pleura. It does not contain the lungs.
It extends from the thoracic inlet superiorly to the diaphragm inferiorly.

Mediastinal regions
Superior mediastinum (between manubriosternal angle and T4/5)
Middle mediastinum
Posterior mediastinum
Anterior mediastinum

Region Contents
Superior mediastinum
Superior vena cava
Brachiocephalic veins
Arch of aorta
Thoracic duct
Trachea
Oesophagus
Thymus
Vagus nerve
Left recurrent laryngeal nerve
Phrenic nerve
Anterior mediastinum
Thymic remnants
Lymph nodes
Fat
Middle mediastinum
Pericardium
Heart
Aortic root
Arch of azygos vein
Main bronchi
Posterior mediastinum
Oesophagus
Thoracic aorta
Azygos vein
Thoracic duct
Vagus nerve
Sympathetic nerve trunks
Splanchnic nerves

1099
Q

A 37 year old lady presents with right upper quadrant pain and nausea. She is otherwise well and her only medical therapy is the oral contraceptive pill which she has taken for many years with no ill effects. Her liver function tests and serum alpha feto protein are normal. An ultrasound examination demonstrates a 4cm non encapsulated lesion in the right lobe of the liver which has a mixed echoity and heterogeneous texture. What is the underlying lesion most likely to be?

Liver cell adenoma

Hydatid cyst

Hepatocellular carcinoma

Cystadenoma

Mesenchymal hamartoma

A

Liver cell adenomas are linked to OCP use and 90% of patients with liver cell adenomas have used the OCP. Liver function tests are often normal. The lesions will typically have a mixed echoity and heterogeneous texture.

Benign liver lesions

Benign liver lesions
Haemangioma
Most common benign tumours of mesenchymal origin
Incidence in autopsy series is 8%
Cavernous haemangiomas may be enormous
Clinically they are reddish purple hypervascular lesions
Lesions are normally separated from normal liver by ring of fibrous tissue
On ultrasound they are typically hyperechoic
Liver cell adenoma
90% develop in women in their third to fifth decade
Linked to use of oral contraceptive pill
Lesions are usually solitary
They are usually sharply demarcated from normal liver although they usually lack a fibrous capsule
On ultrasound the appearances are of mixed echoity and heterogeneous texture. On CT most lesions are hypodense when imaged prior to administration of IV contrast agents
In patients with haemorrhage or symptoms removal of the adenoma may be required
Mesenchymal hamartomas Congential and benign, usually present in infants. May compress normal liver
Liver abscess
Biliary sepsis is a major predisposing factor
Structures drained by the portal venous system form the second largest source
Common symptoms include fever, right upper quadrant pain. Jaundice may be seen in 50%
Ultrasound will usually show a fluid filled cavity, hyperechoic walls may be seen in chronic abscesses
Amoebic abscess
Liver abscess is the most common extra intestinal manifestation of amoebiasis
Between 75 and 90% lesions occur in the right lobe
Presenting complaints typically include fever and right upper quadrant pain
Ultrasonography will usually show a fluid filled structure with poorly defined boundaries
Aspiration yield sterile odourless fluid which has an anchovy paste consistency
Treatment is with metronidazole
Hyatid cysts
Seen in cases of Echinococcus infection
Typically an intense fibrotic reaction occurs around sites of infection
The cyst has no epithelial lining
Cysts are commonly unilocular and may grow to 20cm in size. The cyst wall is thick and has an external laminated hilar membrane and an internal enucleated germinal layer
Typically presents with malaise and right upper quadrant pain. Secondary bacterial infection occurs in 10%.
Liver function tests are usually abnormal and eosinophilia is present in 33% cases
Ultrasound may show septa and hyatid sand or daughter cysts.
Percutaneous aspiration was previously contra indicated, it is now incorporated into some treatment regimens
Treatment is by sterilisation of the cyst with mebendazole and may be followed by surgical resection. Hypertonic swabs are packed around the cysts during surgery
Polycystic liver disease
Usually occurs in association with polycystic kidney disease
Autosomal dominant disorder
Symptoms may occur as a result of capsular stretch
Cystadenoma
Rare lesions with malignant potential
Usually solitary multiloculated lesions
Liver function tests usually normal
Ultrasonography typically shows a large anechoic, fluid filled area with irregular margins. Internal echos may result from septa
Surgical resection is indicated in all cases

1100
Q

A 6 year old girl undergoes a transanal excision of of fleshy polyp 8cm from the anal verge. What type of polyp is most likely to be seen?

Hyperplastic polyp

Sessile serrated lesion

Adenoma

Cloacogenic polyp

Hamartoma

A

The child is most likely to have a juvenile polyp, these are hamartomatous lesions. Cloacogenic inflammatory polyps correspond to a rare subtype of prolapsing mucosal polyps that arise around the anal transitional zone. They can rarely mimic neoplasia. Adenomatous lesions are rarely seen in children.

Lower GI polyps

Colonic polyps are a common condition and there are a number of different types. The type of polyps found varies according to the age of the patient and the aetiology of the polyp. Isolated, small proximal polyps are often an incidental. Multiple, distal or large polyps may give rise to symptoms such as bleeding, intussceception or systemic disturbance owing to electrolyte derangement.

Polyp types
Type Key features Management
Juvenile polyp Often single hamartomatous lesions
Rectum commonest site
Multiple lesions associated with increased risk of malignancy Excision (usually per anal or endoscopic)
Hyperplastic polyps Account for 90% of all polyps
Usually less than 5mm in diameter
Rectosigmoid is commonest location If multiple polyps (more than 20) increased risk of malignancy
Isolated hyperplastic polyps are benign and can be left
Peutz-Jeghers syndrome Hamartomatous polyps
Mucocutaneous hyperpigmentation
Family history (autosomal dominant) Polyps benign but may intusscecept
Surveillance as increased risk of malignancy
Adenoma Comprise around 10% of all colonic polyps
90% less than 1cm in diameter
Increased risk of malignancy in those over 1cm in diameter
3 types villous, tubulovillous and tubular
Serrated adenomas are a combination of adenomatous and hyperplastic features and are also associated with increased risk of malignancy Full colonoscopic evaluation
Colonoscopic polypectomy
Follow up depending upon polyp burden and risk factors
Inflammatory pseudopolyp Found in patients with IBD
Co-existing inflammation is present Management directed at underlying cause

Polyposis syndromes
Syndrome Genetic defect Features Screening and management Associated disorders
Familial adenomatous polyposis Mutation of APC gene (80%) cases, dominant Typically over 100 colonic adenomas
Cancer risk of 100%
20% are new mutations If known to be at risk then predictive genetic testing as teenager
Annual flexible sigmoidoscopy from 15 years
If no polyps found then 5 yearly colonoscopy started at age 20
Polyps found = resectional surgery (resection and pouch Vs sub total colectomy and IRA) Gastric fundal polyps (50%).
Duodenal polyps 90%.
If severe duodenal polyposis cancer risk of 30% at 10 years.
Abdominal desmoid tumours
MYH associated polyposis Biallelic mutation of mut Y human homologue (MYH) on chromosome 1p, recessive Multiple colonic polyps
Later onset right sided cancers more common than in FAP
100% cancer risk by age 60 Once identified resection and ileoanal pouch reconstruction is recommended
Attenuated phenotype - regular colonoscopy Duodenal polyposis in 30%
Associated with increased risk of breast cancer (self examination)
Peutz -Jeghers syndrome STK11 (LKB1) mutation on chromosome 19 in some (but not all) cases, dominant Multiple benign intestinal hamartomas
Episodic obstruction and intussceception
Increased risk of GI cancers (colorectal cancer 20%, gastric 5%)
Increased risk of breast, ovarian, cervical pancreatic and testicular cancers Annual examination
Pan intestinal endoscopy every 2-3 years Malignancies at other sites
Classical pigmentation pattern
Cowden disease Mutation of PTEN gene on chromosome 10q22, dominant Macrocephaly
Multiple intestinal hamartomas
Multiple trichilemmomas
89% risk of cancer at any site
16% risk of colorectal cancer Targeted individualised screening Breast cancer (81% risk)
Thyroid cancer and non toxic goitre
Uterine cancer
HNPCC (Lynch syndrome) Germline mutations of DNA mismatch repair genes Colo rectal cancer 30-70%
Endometrial cancer 30-70%
Gastric cancer 5-10%
Scanty colonic polyps may be present
Colonic tumours likely to be right sided and mucinous Colonoscopy every 1-2 years from age 25
Consideration of prophylactic surgery
Extra colonic surveillance recommended Extra colonic cancers

1101
Q

Which of the statistical tests listed below is most appropriate to use with data which follows a normal distribution?

T Test

Fishers exact test

Spearmans rank test

Mann Whitney U test

Bonferroni test

A

A T Test is most suited to data which follows the normal distribution as the others are non parametric tests. The Bonferroni test is used to provide correction when multiple tests are used.
Qualitative and quantitative data

Qualitative and quantitative data
Qualitative (categorical) data refers to different descriptions of a characteristic, although it may be possible to allocate a number it has no scale.
Quantitative data is associated with numerical values on a numerical scale.

Since quantitative data is based on a numerical scale it can be organised to create a distribution curve. The central tendency may be estimated using the mode, median and mean. The standard deviation gives an estimation of the spread of data.

1102
Q

How many unpaired branches leave the abdominal aorta to supply the abdominal viscera?

One

Two

Three

Four

Five

A

There are three unpaired branches to the abdominal viscera. These include the coeliac axis, the SMA and IMA. Branches to the adrenals, renal arteries and gonadal vessels are paired. The fourth unpaired branch of the abdominal aorta, the median sacral artery, does not directly supply the abdominal viscera.
Abdominal aortic branches

Branches Level Paired Type
Inferior phrenic T12 (Upper border) Yes Parietal
Coeliac T12 No Visceral
Superior mesenteric L1 No Visceral
Middle suprarenal L1 Yes Visceral
Renal L1-L2 Yes Visceral
Gonadal L2 Yes Visceral
Lumbar L1-L4 Yes Parietal
Inferior mesenteric L3 No Visceral
Median sacral L4 No Parietal
Common iliac L4 Yes Terminal

1103
Q

A 19 year old male presents with axillary lymphadenopathy and symptoms suggestive of Hodgkins lymphoma. What is the most appropriate investigation?

Fine needle aspiration of the lymph nodes

Freehand needle core biopsy of the lymph nodes

Image guided core biopsy of the lymph nodes

Excision biopsy of a lymph node

Axillary node clearance

A

When a diagnosis of lymphoma is suspected, the correct investigation is excision biopsy of a complete lymph node to confirm the diagnosis.
Excision of a single node is appropriate. Lymphoma is usually treated with chemotherapy and axillary node clearance is therefore inappropriate. FNA and core biopsy will not allow accurate diagnosis and are therefore not appropriate.

Hodgkins lymphoma

Presenting features
Asymptomatic lympadenopathy
Cough, Pel Ebstein fever, haemoptysis, dyspnoea
B Symptoms - 10% weight loss, fever, night sweats

Staging
All patients are staged with CT scanning of the chest, abdomen and pelvis
The Ann Arbor staging system is commonly used
Stage Features
I Single lymph node region
II Two or more regions on the same side of the diaphragm
III Involvement of lymph node regions on both sides of the diaphragm
IV Involvement of extra nodal sites

Sub types
Classical Hodgkin lymphoma is classified into the following 4 types:

Nodular sclerosing Hodgkin lymphoma (NSHL)
Mixed-cellularity Hodgkin lymphoma (MCHL)
Lymphocyte-depleted Hodgkin lymphoma (LDHL)
Lymphocyte-rich classical Hodgkin lymphoma (LRHL)

Reed Sternberg cells may be identified histologically.

Treatment
This may be multimodal and both chemo and radiotherapy are used.

Diagnosis
This is made by excision of a complete lymph node that is then submitted for detailed histological evaluation.

Pathogenesis
Infection with Ebstein Barr virus is linked to the condition (particularly mixed cellularity lymphoma).

Prognosis
Stage I disease is associated with survival figures of up to 85% at 5 years. Nodular sclerosing has the best prognosis. Lymphocyte depleted Hodgkins lymphoma, advancing age, male sex and stage IV disease are all associated with a worsening of prognosis.

Reference
Shanbhag S and Ambinder R. Hodgkin Lymphoma: a review and update on recent progress. CA Cancer J Clin . 2018 March ; 68(2): 116132

1104
Q

Which of the following upper limb muscles is not innervated by the radial nerve?

Extensor carpi ulnaris

Abductor digiti minimi

Anconeus

Supinator

Brachioradialis

A

Mnemonic for radial nerve muscles: BEST

B rachioradialis
E xtensors
S upinator
T riceps
Abductor digiti minimi is innervated by the ulnar nerve.
Radial nerve

Continuation of posterior cord of the brachial plexus (root values C5 to T1)

Path
In the axilla: lies posterior to the axillary artery on subscapularis, latissimus dorsi and teres major.
Enters the arm between the brachial artery and the long head of triceps (medial to humerus).
Spirals around the posterior surface of the humerus in the groove for the radial nerve.
At the distal third of the lateral border of the humerus it then pierces the intermuscular septum and descends in front of the lateral epicondyle.
At the lateral epicondyle it lies deeply between brachialis and brachioradialis where it then divides into a superficial and deep terminal branch.
Deep branch crosses the supinator to become the posterior interosseous nerve.

Regions innervated
Motor (main nerve)
Triceps
Anconeus
Brachioradialis
Extensor carpi radialis
Motor (posterior interosseous branch)
Supinator
Extensor carpi ulnaris
Extensor digitorum
Extensor indicis
Extensor digiti minimi
Extensor pollicis longus and brevis
Abductor pollicis longus
Sensory The area of skin supplying the proximal phalanges on the dorsal aspect of the hand is supplied by the radial nerve (this does not apply to the little finger and part of the ring finger)

Muscular innervation and effect of denervation
Anatomical location Muscle affected Effect of paralysis
Shoulder Long head of triceps Minor effects on shoulder stability in abduction
Arm Triceps Loss of elbow extension
Forearm Supinator
Brachioradialis
Extensor carpi radialis longus and brevis Weakening of supination of prone hand and elbow flexion in mid prone position

1105
Q

A 65 year old man is admitted for a below knee amputation. He is taking digoxin. Clinically the patient has an irregularly irregular pulse. What would you expect to see when you examine the jugular venous pressure?

Absent y waves

Slow y descent

Cannon waves

Steep y descent

Absent a waves

A

Jugular venous pressure

Absent a waves = Atrial fibrillation
Large a waves = Any cause of right ventricular hypertrophy, tricuspid stenosis
Cannon waves (extra large a waves) = Complete heart block
Prominent v waves = Tricuspid regurgitation
Slow y descent = Tricuspid stenosis, right atrial myxoma
Steep y descent = Right ventricular failure, constrictive pericarditis, tricuspid regurgitation
This patient has atrial fibrillation and is most likely to have absent a waves.

Jugular venous pressure

As well as providing information on right atrial pressure, the jugular vein waveform may provide clues to underlying valvular disease. A non-pulsatile JVP is seen in superior vena caval obstruction. Kussmaul’s sign describes a paradoxical rise in JVP during inspiration seen in constrictive pericarditis

‘a’ wave = atrial contraction
large if atrial pressure e.g. tricuspid stenosis, pulmonary stenosis, pulmonary hypertension
absent if in atrial fibrillation

Cannon ‘a’ waves
caused by atrial contractions against a closed tricuspid valve
are seen in complete heart block, ventricular tachycardia/ectopics, nodal rhythm, single chamber ventricular pacing

‘c’ wave
closure of tricuspid valve
not normally visible

‘v’ wave
due to passive filling of blood into the atrium against a closed tricuspid valve
giant v waves in tricuspid regurgitation

‘x’ descent = fall in atrial pressure during ventricular systole

‘y’ descent = opening of tricuspid valve

1106
Q

A 77-year-old female presents with a non-healing ulcer on her right foot. Blood cultures grow MRSA. Which antibiotic would you consider in addition to vancomycin to cover this?

Flucloxacillin

Ceftazidime

Ciprofloxacin

Metronidazole

Rifampicin

A

The MRSA would or may be resistant to Other antibiotics. Rifampicin is normally given in combination with another antibiotic.

MRSA

Methicillin-resistant Staphylococcus aureus (MRSA) was one of the first organisms which highlighted the dangers of hospital-acquired infections.

Who should be screened for MRSA?
all patients awaiting elective admissions (exceptions include day patients having terminations of pregnancy and ophthalmic surgery. Patients admitted to mental health trusts are also excluded)
in the UK all emergency admissions are currently screened

How should a patient be screened for MRSA?
nasal swab and skin lesions or wounds
the swab should be wiped around the inside rim of a patient’s nose for 5 seconds
the microbiology form must be labelled ‘MRSA screen’

Suppression of MRSA from a carrier once identified
nose: mupirocin 2% in white soft paraffin, tds for 5 days
skin: chlorhexidine gluconate, od for 5 days. Apply all over but particularly to the axilla, groin and perineum

The following antibiotics are commonly used in the treatment of MRSA infections:
vancomycin
teicoplanin

Some strains may be sensitive to the antibiotics listed below but they should not generally be used alone because resistance may develop:
rifampicin
macrolides
tetracyclines
aminoglycosides
clindamycin

Relatively new antibiotics such as linezolid, quinupristin/dalfopristin combinations and tigecycline have activity against MRSA but should be reserved for resistant cases

1107
Q

A 23 year old lady with sialolithiasis of the submandibular gland is undergoing excision of the gland. Which of the following nerves is at risk as the duct is mobilised?

Lingual nerve

Buccal nerve

Facial nerve

Glossopharyngeal

Vagus

A

The lingual nerve wraps around Whartons duct. The lingual nerve provides sensory supply to the anterior 2/3 of the tongue.

Submandibular gland

Relations of the submandibular gland
Superficial Platysma, deep fascia and mandible
Submandibular lymph nodes
Facial vein (facial artery near mandible)
Marginal mandibular nerve
Cervical branch of the facial nerve
Deep Facial artery (inferior to the mandible)
Mylohyoid muscle
Sub mandibular duct
Hyoglossus muscle
Lingual nerve
Submandibular ganglion
Hypoglossal nerve

Submandibular duct (Wharton’s duct)
Opens lateral to the lingual frenulum on the anterior floor of mouth.
5 cm length
Lingual nerve wraps around Wharton’s duct. As the duct passes forwards it crosses medial to the nerve to lie above it and then crosses back, lateral to it, to reach a position below the nerve.

Innervation
Sympathetic innervation- Derived from superior cervical ganglion
Parasympathetic innervation- Submandibular ganglion via lingual nerve

Arterial supply
Branch of the facial artery. The facial artery passes through the gland to groove its deep surface. It then emerges onto the face by passing between the gland and the mandible.

Venous drainage
Anterior facial vein (lies deep to the Marginal Mandibular nerve)

Lymphatic drainage
Deep cervical and jugular chains of nodes

1108
Q

A 21 year old woman presents with right iliac fossa pain. She reports some bloodstained vaginal discharge. On examination, she is afebrile and has a pulse rate of 97 bpm, normal blood pressure. She has diffuse lower abdominal tenderness. What is the most appropriate course of action?

Laparotomy

Laparoscopy

Abdominal and pelvic MRI scan

Abdominal and pelvic CT scan

Abdominal and pelvic USS

A

The history of blood stained discharge and tenderness makes an ectopic pregnancy a strong possibility, a USS should be performed and a pregnancy test undertaken. If the beta HCG is high then an intra uterine pregnancy should be found. If it is not, then an ectopic pregnancy is likely and surgery should be considered.

Right iliac fossa pain

Differential diagnosis

Appendicitis
Pain radiating to right iliac fossa
Anorexia (very common)
Short history
Diarrhoea and profuse vomiting rare
Crohn’s disease
Often long history
Signs of malnutrition
Change in bowel habit, especially diarrhoea
Mesenteric adenitis
Mainly affects children
Causes include Adenoviruses, Epstein Barr Virus, beta-haemolytic Streptococcus, Staphylococcus spp., Escherichia coli, Streptococcus viridans and Yersinia spp.
Patients have a higher temperature than those with appendicitis
If laparotomy is performed, enlarged mesenteric lymph nodes will be present
Diverticulitis
Both left and right sided disease may present with right iliac fossa pain
Clinical history may be similar, although some change in bowel habit is usual
When suspected, a CT scan may help in refining the diagnosis
Meckel’s diverticulitis
A Meckel’s diverticulum is a congenital abnormality that is present in about 2% of the population
Typically 2 feet proximal to the ileocaecal valve
May be lined by ectopic gastric mucosal tissue and produce bleeding
Perforated peptic ulcer
This usually produces upper quadrant pain but pain may be lower
Perforations typically have a sharp sudden onset of pain in the history
Incarcerated right inguinal or femoral hernia
Usually only right iliac fossa pain if right sided or bowel obstruction.
Bowel perforation secondary to caecal or colon carcinoma
Seldom localised to right iliac fossa, although complete large bowel obstruction with caecal distension may cause pain prior to perforation.
Gynaecological causes
Pelvic inflammatory disease/salpingitis/pelvic abscess/Ectopic pregnancy/Ovarian torsion/Threatened or complete abortion/Mittelschmerz
Urological causes
Ureteric colic/UTI/Testicular torsion
Other causes
TB/Typhoid/Herpes Zoster/AAA/Situs inversus

1109
Q

A 50 year old female presents with bilateral parotid gland swelling and symptoms of a dry mouth. On examination, she has bilateral facial nerve palsies. This improved following steroid treatment. What is the likely underlying diagnosis?

Pleomorphic adenoma

Sarcoid

Warthins tumour

Sialolithiasis

Adenoid cystic carcinoma

A

Sarcoid occurs bilaterally in 70% of cases and facial nerve involvement is recognised. Treatment is conservative in most cases although individuals with facial nerve palsy will usually receive steroids with good effect.

Parotid gland clinical

Benign neoplasms
Up to 80% of all salivary gland tumours occur in the parotid gland and up to 80% of these are benign. There is no consistent correlation between the rate of growth and the malignant potential of the lesion. However, benign tumours should not invade structures such as the facial nerve.
With the exception of Warthins tumours, they are commoner in women than men. The median age of developing a lesion is in the 5th decade of life.

Benign tumour types
Tumour type Features
Benign pleomorphic adenoma or benign mixed tumor Most common parotid neoplasm (80%)
Proliferation of epithelial and myoepithelial cells of the ducts and an increase in stromal components
Slow growing, lobular, and not well encapsulated
Recurrence rate of 1-5% with appropriate excision (parotidectomy)
Recurrence possibly secondary to capsular disruption during surgery
Malignant degeneration occurring in 2-10% of adenomas observed for long periods, with carcinoma ex-pleomorphic adenoma occurring most frequently as adenocarcinoma
Warthin tumor (papillary cystadenoma lymphoma or adenolymphoma) Second most common benign parotid tumor (5%)
Most common bilateral benign neoplasm of the parotid
Marked male as compared to female predominance
Occurs later in life (sixth and seventh decades)
Presents as a lymphocytic infiltrate and cystic epithelial proliferation
May represent heterotopic salivary gland epithelial tissue trapped within intraparotid lymph nodes
Incidence of bilaterality and multicentricity of 10%
Malignant transformation rare (almost unheard of)
Monomorphic adenoma Account for less than 5% of tumours
Slow growing
Consist of only one morphological cell type (hence term mono)
Include; basal cell adenoma, canalicular adenoma, oncocytoma, myoepitheliomas
Haemangioma Should be considered in the differential of a parotid mass in a child
Accounts for 90% of parotid tumours in children less than 1 year of age
Hypervascular on imaging
Spontaneous regression may occur and malignant transformation is almost unheard of

Malignant salivary gland tumours
Types of malignancy

Mucoepidermoid carcinoma 30% of all parotid malignancies
Usually low potential for local invasiveness and metastasis (depends mainly on grade)
Adenoid cystic carcinoma Unpredictable growth pattern
Tendency for perineural spread
Nerve growth may display skip lesions resulting in incomplete excision
Distant metastasis more common (visceral rather than nodal spread)
5 year survival 35%
Mixed tumours Often a malignancy occurring in a previously benign parotid lesion
Acinic cell carcinoma Intermediate grade malignancy
May show perineural invasion
Low potential for distant metastasis
5 year survival 80%
Adenocarcinoma Develops from secretory portion of gland
Risk of regional nodal and distant metastasis
5 year survival depends upon stage at presentation, may be up to 75% with small lesions with no nodal involvement
Lymphoma Large rubbery lesion, may occur in association with Warthins tumours
Diagnosis should be based on regional nodal biopsy rather than parotid resection
Treatment is with chemotherapy (and radiotherapy)

Diagnostic evaluation
Plain x-rays may be used to exclude calculi
Sialography may be used to delineate ductal anatomy
FNAC is used in most cases
Superficial parotidectomy may be either diagnostic or therapeutic depending upon the nature of the lesion
Where malignancy is suspected the primary approach should be definitive resection rather than excisional biopsy
CT/ MRI may be used in cases of malignancy for staging primary disease

Treatment
For nearly all lesions this consists of surgical resection, for benign disease this will usually consist of a superficial parotidectomy. For malignant disease a radical or extended radical parotidectomy is performed. The facial nerve is included in the resection if involved. The need for neck dissection is determined by the potential for nodal involvement.

Other parotid disorders
HIV infection
Lymphoepithelial cysts associated with HIV occur almost exclusively in the parotid
Typically presents as bilateral, multicystic, symmetrical swelling
Risk of malignant transformation is low and management usually conservative

Sjogren syndrome
Autoimmune disorder characterised by parotid enlargement, xerostomia and keratoconjunctivitis sicca
90% of cases occur in females
Second most common connective tissue disorder
Bilateral, non tender enlargement of the gland is usual
Histologically, the usual findings are of a lymphocytic infiltrate in acinar units and epimyoepithelial islands surrounded by lymphoid stroma
Treatment is supportive
There is an increased risk of subsequent lymphoma

Sarcoid
Parotid involvement occurs in 6% of patients with sarcoid
Bilateral in most cases
Gland is not tender
Xerostomia may occur
Management of isolated parotid disease is usually conservative

1110
Q

At which of the following anatomical locations does the common peroneal nerve bifurcate into the superficial and deep peroneal nerves?

Immediately anterior to the linea aspera

At the lateral aspect of the neck of the fibula

Within the substance of tibialis anterior muscle

At the inferomedial aspect of the popliteal fossa

Under the medial head of gastrocnemius

A

The common peroneal nerve bifurcates at the neck of the fibula (where it is most likely to be injured).

Common peroneal nerve

Derived from the dorsal divisions of the sacral plexus (L4, L5, S1 and S2).

This nerve supplies the skin and fascia of the anterolateral surface of the leg and the dorsum of the foot. It also innervates the muscles of the anterior and peroneal compartments of the leg, extensor digitorum brevis as well as the knee, ankle and foot joints.

It is laterally placed within the sciatic nerve. From the bifurcation of the sciatic nerve it passes inferolaterally in the lateral and proximal part of the popliteal fossa, under the cover of biceps femoris and its tendon. To reach the posterior aspect of the fibular head. It ends by dividing into the deep and superficial peroneal nerves at the point where it winds around the lateral surface of the neck of the fibula in the body of peroneus longus, approximately 2cm distal to the apex of the head of the fibula. It is palpable posterior to the head of the fibula.

Branches
In the thigh Nerve to the short head of biceps
Articular branch (knee)
In the popliteal fossa Lateral cutaneous nerve of the calf
Neck of fibula Superficial and deep peroneal nerves

1111
Q

A 74 year old male with colon cancer sustains an iatrogenic splenic injury during surgery. He is bleeding profusely. What is the most appropriate course of action?

Infusion of human prothrombin complex

Infusion of packed red cells

Infusion of blood from the cell saver salvaged during surgery

Infusion of factor VIII concentrate

Infusion of gelofusine

A

The cell saver is inappropriate because the cells will be contaminated with malignant cells and faecal matter from the open bowel.

Blood products

Whole blood fractions

Fraction Key points
Packed red cells Used for transfusion in chronic anaemia and cases where infusion of large volumes of fluid may result in cardiovascular compromise. Product obtained by centrifugation of whole blood.
Platelet rich plasma Usually administered to patients who are thrombocytopaenic and are bleeding or require surgery. It is obtained by low speed centrifugation.
Platelet concentrate Prepared by high speed centrifugation and administered to patients with thrombocytopaenia.
Fresh frozen plasma
Prepared from single units of blood.
Contains clotting factors, albumin and immunoglobulin.
Unit is usually 200 to 250ml.
Usually used in correcting clotting deficiencies in patients with hepatic synthetic failure who are due to undergo surgery.
Usual dose is 12-15ml/Kg-1.
It should not be used as first line therapy for hypovolaemia.
Cryoprecipitate
Formed from supernatant of FFP.
Rich source of Factor VIII and fibrinogen.
Allows large concentration of factor VIII to be administered in small volume.
SAG-Mannitol Blood Removal of all plasma from a blood unit and substitution with:
Sodium chloride
Adenine
Anhydrous glucose
Mannitol
Up to 4 units of SAG M Blood may be administered. Thereafter whole blood is preferred. After 8 units, clotting factors and platelets should be considered.

Cell saver devices
These collect patients own blood lost during surgery and then re-infuse it. There are two main types:
Those which wash the blood cells prior to re-infusion. These are more expensive to purchase and more complicated to operate. However, they reduce the risk of re-infusing contaminated blood back into the patient.
Those which do not wash the blood prior to re-infusion.
Their main advantage is that they avoid the use of infusion of blood from donors into patients and this may reduce risk of blood borne infection. It may be acceptable to Jehovah’s witnesses. It is contraindicated in malignant disease for risk of facilitating disease dissemination.

Blood products used in warfarin reversal
In some surgical patients the use of warfarin can pose specific problems and may require the use of specialised blood products

Immediate or urgent surgery in patients taking warfarin(1) (2):

  1. Stop warfarin
  2. Vitamin K (reversal within 4-24 hours)
    -IV takes 4-6h to work (at least 5mg)
    -Oral can take 24 hours to be clinically effective
  3. Fresh frozen plasma
    Used less commonly now as 1st line warfarin reversal
    -30ml/kg-1
    -Need to give at least 1L fluid in 70kg person (therefore not appropriate in fluid overload)
    -Need blood group
    -Only use if human prothrombin complex is not available
  4. Human Prothrombin Complex (reversal within 1 hour)
    -Bereplex 50 u/kg
    -Rapid action but factor 6 short half life, therefore give with vitamin K

References
1. Dentali, F., C. Marchesi, et al. (2011). ‘Safety of prothrombin complex concentrates for rapid anticoagulation reversal of vitamin K antagonists. A meta-analysis.’ Thromb Haemost 106(3): 429-438.

  1. http://www.transfusionguidelines.org/docs/pdfs/bbt-03warfarin-reversal-flowchart-2006.pdf
1112
Q

An 82 year old lady presents with a carcinoma of the caecum. Approximately what proportion of patients presenting with this diagnosis will have synchronous cancer?

<1%

60%

50%

20%

5%

A

Synchronous colonic tumours are seen in 5% cases and all patients having a flexible sigmoidoscopy should have completion colonoscopy if tumours or polyps are found
Synchronous lesions may occur in up to 5% of patients with colorectal cancer. A full and complete lumenal study with either colonoscopy, CT cologram or barium enema is mandatory in all patients being considered for surgery.

Colorectal cancer

Annually, about 150,000 new cases are diagnosed and 50,000 deaths from the disease
About 75% will have sporadic disease and 25% will have a family history
Colorectal tumours comprise a spectrum of disease ranging from adenomas through to polyp cancers and frank malignancy.
Polyps may be categorised into: neoplastic polyps, adenomatous polyps and non neoplastic polyps.
The majority of adenomas are polypoidal lesions, although flat lesions do occur and may prove to be dysplastic.
Non-neoplastic polyps include hyperplastic, juvenile, hamartomatous, inflammatory, and lymphoid polyps, which have not generally been thought of as precursors of cancer.
Three characteristics of adenomas that correlate with malignant potential have been characterised. These include increased size, villous architecture and dysplasia. For this reason most polyps identified at colonoscopy should be removed.
The transformation from polyp to cancer is described by the adenoma - carcinoma sequence and its principles should be appreciated. Essentially genetic changes accompany the transition from adenoma to carcinoma; key changes include APC, c-myc, K RAS mutations and p53 deletions.

1113
Q

A 56 year old man is investigated for anaemia and a large exophytic growth is identified in the distal stomach. What pathological finding is most likely?

Squamous cell carcinoma

Adenocarcinoma

Lymphoma

Leiomyosarcoma

Rhadomyosarcoma

A

Most gastric malignancies are adenocarcinomas.

Gastric cancer

Overview
There are 700,000 new cases of gastric cancer worldwide each year. It is most common in Japan and less common in western countries. It is more common in men and incidence rises with increasing age. The exact cause of many sporadic cancer is not known, however, familial cases do occur in HNPCC families. In addition, smoking and smoked or preserved foods increase the risk. Japanese migrants retain their increased risk (decreased in subsequent generations). The distribution of the disease in western countries is changing towards a more proximal location (perhaps due to rising obesity).

Pathology
There is some evidence of support a stepwise progression of the disease through intestinal metaplasia progressing to atrophic gastritis and subsequent dysplasia, through to cancer. The favoured staging system is TNM. The risk of lymph node involvement is related to size and depth of invasion; early cancers confined to submucosa have a 20% incidence of lymph node metastasis. Tumours of the gastro-oesophageal junction are classified as below:

Type 1 True oesophageal cancers and may be associated with Barrett’s oesophagus.
Type 2 Carcinoma of the cardia, arising from cardiac type epithelium
or short segments with intestinal metaplasia at the oesophagogastric junction.
Type 3 Sub cardial cancers that spread across the junction. Involve similar nodal stations to gastric cancer.

Groups for close endoscopic monitoring
Intestinal metaplasia of columnar type
Atrophic gastritis
Low to medium grade dysplasia
Patients who have previously undergone resections for benign peptic ulcer disease (except highly selective vagotomy).

Referral to endoscopy

Patients of any age with dyspepsia and any of the following Patients without dyspepsia Worsening dyspepsia
Chronic gastrointestinal bleeding Dysphagia Barretts oesophagus
Dysphagia Unexplained abdominal pain or weight loss Intestinal metaplasia
Weight loss Vomiting Dysplasia
Iron deficiency anaemia Upper abdominal mass Atrophic gastritis
Upper abdominal mass Jaundice Patient aged over 55 years with unexplained or persistent dyspepsia

Staging
CT scanning of the chest abdomen and pelvis is the routine first line staging investigation in most centres.
Laparoscopy to identify occult peritoneal disease
PET CT (particularly for junctional tumours)

Treatment
Proximally sited disease greater than 5-10cm from the OG junction may be treated by sub total gastrectomy
Total gastrectomy if tumour is <5cm from OG junction
For type 2 junctional tumours (extending into oesophagus) oesophagogastrectomy is usual
Endoscopic sub mucosal resection may play a role in early gastric cancer confined to the mucosa and perhaps the sub mucosa (this is debated)
Lymphadenectomy should be performed. A D2 lymphadenectomy is widely advocated by the Japanese, the survival advantages of extended lymphadenectomy have been debated. However, the overall recommendation is that a D2 nodal dissection be undertaken.
Most patients will receive chemotherapy either pre or post operatively.

Prognosis

UK Data

Disease extent Percentage 5 year survival
All RO resections 54%
Early gastric cancer 91%
Stage 1 87%
Stage 2 65%
Stage 3 18%

Operative procedure

Total Gastrectomy , lymphadenectomy and Roux en Y anastomosis

General anaesthesia
Prophylactic intravenous antibiotics
Incision: Rooftop.
Perform a thorough laparotomy to identify any occult disease.
Mobilise the left lobe of the liver off the diaphragm and place a large pack over it. Insert a large self retaining retractor e.g. omnitract or Balfour (take time with this, the set up should be perfect). Pack the small bowel away.
Begin by mobilising the omentum off the transverse colon.
Proceed to detach the short gastric vessels.
Mobilise the pylorus and divide it at least 2cm distally using a linear cutter stapling device.
Continue the dissection into the lesser sac taking the lesser omentum and left gastric artery flush at its origin.
The lymph nodes should be removed en bloc with the specimen where possible.
Place 2 stay sutures either side of the distal oesophagus. Ask the anaesthetist to pull back on the nasogastric tube. Divide the distal oesophagus and remove the stomach.
The oesphago jejunal anastomosis should be constructed. Identify the DJ flexure and bring a loop of jejunum up to the oesophagus (to check it will reach). Divide the jejunum at this point. Bring the divided jejunum either retrocolic or antecolic to the oesophagus. Anastamose the oesophagus to the jejunum, using either interrupted 3/0 vicryl or a stapling device. Then create the remainder of the Roux en Y reconstruction distally.
Place a jejunostomy feeding tube.
Wash out the abdomen and insert drains (usually the anastomosis and duodenal stump). Help the anaesthetist insert the nasogastric tube (carefully!)
Close the abdomen and skin.
Enteral feeding may commence on the first post-operative day. However, most surgeons will leave patients on free NG drainage for several days and keep them nil by mouth.

1114
Q

A 42 year old man presents with epigastric pain. At endoscopy, he is found to have a punched out ulcer on the anterior wall of the stomach. It is shallow and measures 1cm in diameter. What is the most likely diagnosis?

Acute peptic ulcer

Chronic peptic ulcer

Adenocarcinoma

Lymphoma

Dieulafoy lesion

A

The absence of fibrosis coupled with small size suggests a more acute ulcer. Management should include biopsy, PPI and repeat endoscopy at 6 weeks.

Upper gastrointestinal bleeding

Patients may present with the following:
Haematemesis and/ or malaena
Epigastric discomfort
Sudden collapse

The extent to which these will occur will depend upon the source. Mortality is higher in patients presenting with haematemesis than malaena alone.

Oesophageal bleeding
Cause Presenting features
Oesophagitis Small volume of fresh blood, often streaking vomit. Malaena rare. Often ceases spontaneously. Usually history of antecedent GORD type symptoms.
Cancer Usually small volume of blood, except as pre terminal event with erosion of major vessels. Often associated symptoms of dysphagia and constitutional symptoms such as weight loss. May be recurrent until malignancy managed.
Mallory Weiss Tear Typically brisk small to moderate volume of bright red blood following bout of repeated vomiting. Malaena rare. Usually ceases spontaneously.
Varices Usually large volume of fresh blood. Swallowed blood may cause malaena. Often associated with haemodynamic compromise. May stop spontaneously but re-bleeds are common until appropriately managed.

Gastric Bleeding
Cause Presenting features
Gastric cancer May be frank haematemesis or altered blood mixed with vomit. Usually prodromal features of dyspepsia and may have constitutional symptoms. Amount of bleeding variable but erosion of major vessel may produce considerable haemorrhage.
Dieulafoy Lesion Often no prodromal features prior to haematemesis and malaena, but this arteriovenous malformation may produce quite considerable haemorrhage and may be difficult to detect endoscopically.
Diffuse erosive gastritis Usually haematemesis and epigastric discomfort. Usually there is an underlying cause such as recent NSAID usage. Large volume haemorrhage may occur with considerable haemodynamic compromise.
Gastric ulcer Small low volume bleeds more common so would tend to present as iron deficiency anaemia. Erosion into a significant vessel may produce considerable haemorrhage and haematemesis.

Duodenum
Most common cause of major haemorrhage is a posteriorly sited duodenal ulcer. However, ulcers at any site in the duodenum may present with haematemesis, malaena and epigastric discomfort. The pain of duodenal ulcer is slightly different to that of gastric ulcers and often occurs several hours after eating. Peri ampullary tumours may bleed but these are rare. In patients with previous abdominal aortic aneurysm surgery aorto-enteric fistulation remains a rare but important cause of major haemorrhage associated with high mortality.

Management
Admission to hospital careful monitoring, cross match blood, check FBC, LFTs, U+E and Clotting (as a minimum)
Patients with on-going bleeding and haemodynamic instability are likely to require O negative blood pending cross matched blood
Early control of airway is vital (e.g. Drowsy patient with liver failure)
Patients with suspected varices should receive terlipressin prior to endoscopy
Ideally all patients admitted with upper gastrointestinal haemorrhage should undergo Upper GI endoscopy within 24 hours of admission. In those who are unstable this should occur immediately after resuscitation or in tandem with it. The endoscopy department is a potentially dangerous place for unstable patients and it may be safer to perform the endoscopy in theatre with an anaesthetist present.
Varices should be banded or subjected to sclerotherapy. If this is not possible owing to active bleeding then a Sengstaken- Blakemore tube (or Minnesota tube) should be inserted. This should be done with care; gastric balloon should be inflated first and oesophageal balloon second. Remember the balloon will need deflating after 12 hours (ideally sooner) to prevent necrosis. Portal pressure should be lowered by combination of medical therapy +/- TIPSS.
Patients with erosive oesophagitis / gastritis should receive a proton pump inhibitor.
Mallory Weiss tears will typically resolve spontaneously
Identifiable bleeding points should receive combination therapy of injection of adrenaline and either a thermal or mechanical treatment. All who have received intervention should receive a continuous infusion of a proton pump inhibitor (IV omeprazole for 72 hours) to reduce the re-bleeding rate.
Patients with diffuse erosive gastritis who cannot be managed endoscopically and continue to bleed may require gastrectomy
Bleeding ulcers that cannot be controlled endoscopically may require laparotomy and ulcer underruning

Indications for surgery
Patients > 60 years
Continued bleeding despite endoscopic intervention
Recurrent bleeding
Known cardiovascular disease with poor response to hypotension

Surgery
Duodenal ulcer
Laparotomy, duodenotomy and under running of the ulcer. If bleeding is brisk then the ulcer is almost always posteriorly sited and will have invaded the gastroduodenal artery. Large bites using 0 Vicryl are taken above and below the ulcer base to occlude the vessel. The duodenotomy should be longitudinal but closed transversely to avoid stenosis.

For gastric ulcer
Under-running of the bleeding site
Partial gastrectomy-antral ulcer
Partial gastrectomy or under running the ulcer- lesser curve ulcer (involving left gastric artery)
Total gastrectomy if bleeding persists

Summary of Acute Upper GI bleeding recommendations:
The need for admission and timing of endoscopic intervention may be predicted by using the Blatchford score. This considers a patients Hb, serum urea, pulse rate and blood pressure. Those patients with a score of 0 are low risk, all others are considered high risk and require admission and endoscopy.
The requirement for pre endoscopic proton pump inhibition is contentious. In the UK the National Institute of Clinical Excellence guidelines suggest the pre endoscopic PPI therapy is unnecessary. Whilst it is accepted that such treatment has no impact on mortality or morbidity a Cochrane review of this practice in 2007 did suggest that it reduced the stigmata of recent haemorrhage at endoscopy. As a result many will still administer PPI to patients prior to endoscopic intervention.
Following endoscopy it is important to calculate the Rockall score for patients to determine their risk of rebleeding and mortality. A score of 3 or less is associated with a rebleeding rate of 4% and a very low risk of mortality and identifies a group of patients suitable for early discharge.

References
1. http://www.sign.ac.uk/guidelines/fulltext/105/index.html
2. Joint Advisory Group on Endoscopy (JAG) Guidelines - http://www.thejag.org.uk
3. NICE Guideline: Management of acute upper GI bleeding. July 2012.

1115
Q

A 76 year old man is undergoing an abdominal aortic aneurysm repair. The surgeons occlude the aorta with two clamps, the inferior clamp being placed at the point of aortic bifurcation. Which of the following vertebral bodies will lie posterior to the clamp at this level?

L1

T10

L4

L5

L2

A

The aorta bifurcates at L4. An important landmark that is tested frequently.

Abdominal aorta

Abdominal aortic topography
Origin T12
Termination L4
Posterior relations L1-L4 Vertebral bodies
Anterior relations Lesser omentum
Liver
Left renal vein
Inferior mesenteric vein
Third part of duodenum
Pancreas
Parietal peritoneum
Peritoneal cavity
Right lateral relations Right crus of the diaphragm
Cisterna chyli
IVC (becomes posterior distally)
Left lateral relations 4th part of duodenum
Duodenal-jejunal flexure
Left sympathetic trunk

1116
Q

An 82 year old man presents with dysphagia. He is investigated and found to have an adenocarcinoma of the distal oesophagus. His staging investigations have revealed a solitary metastatic lesion in the right lobe of his liver. What is the best course of action?

Arrange a PET CT scan

Arrange an endoscopic ultrasound

Assess fitness for liver resection prior to oesophagectomy

Assess fitness for oesophagectomy followed by liver resection

Insertion of metallic stent

A

The presence of distant disease in the context of oesophageal cancer renders him incurable. Further staging is not needed and surgery is not an option. Palliation is the preferred option and a metallic stent will achieve this.

Treatment of oesophageal cancer

  • In general resections are not offered to those patients with distant metastasis, and usually not to those with N2 disease.
    Local nodal involvement is not in itself a contra indication to resection.
    Surgical resection is the mainstay of treatment.
    Neoadjuvent chemotherapy is given in most cases prior to surgery.
    In situ disease may be managed by endoscopic mucosal resection, although this is still debated.
    In patients with lower third lesions an Ivor - Lewis type procedure is most commonly performed. Very distal tumours may be suitable to a transhiatal procedure. Which is an attractive option as the penetration of two visceral cavities required for an Ivor- Lewis type procedure increases the morbidity considerably.
    More proximal lesions will require a total oesphagectomy (Mckeown type) with anastomosis to the cervical oesophagus.
    Patients with unresectable disease may derive benefit from local ablative procedures, palliative chemotherapy or stent insertion.

Operative details of Ivor- Lewis procedure
Combined laparotomy and right thoracotomy

Indication
Lower and middle third oesophageal tumours

Preparation
Staging with a combination of CT chest abdomen and pelvis- if no metastatic disease detected then patients will undergo a staging laparoscopy to detect peritoneal disease.
If both these modalities are negative then patients will finally undergo a PET CT scan to detect occult metastatic disease. Only in those whom no evidence of advanced disease is detected will proceed to resection.
Patients receive a GA, double lumen endotracheal tube to allow for lung deflation, CVP and arterial monitoring.

Procedure
A rooftop incision is made to access the stomach and duodenum.

Laparotomy To mobilize the stomach
The greater omentum is incised away from its attachment to the right gastroepiploic vessels along the greater curvature of the stomach.
Then the short gastric vessels are ligated and detached from the greater curvature from the spleen.
The lesser omentum is incised, preserving the right gastric artery.
The retroperitoneal attachments of the duodenum in its second and third portions are incised, allowing the pylorus to reach the oesophageal hiatus. Some surgeons perform a pyloroplasty at this point to facilitate gastric emptying.
The left gastric vessels are then ligated, avoiding any injury to the common hepatic or splenic arteries. Care must be taken to avoid inadvertently devascularising the liver owing to variations in anatomy.

Right Thoracotomy Oesophageal resection and oesophagogastric anastomosis
Through 5th intercostal space
Dissection performed 10cm above the tumour
This may involve transection of the azygos vein.
The oesophagus is then removed with the stomach creating a gastric tube.
An anastomosis is created.

The chest is closed with underwater seal drainage and tube drains to the abdominal cavity.

Post operatively
Patients will typically recover in ITU initially.
A nasogastric tube will have been inserted intraoperatively and must remain in place during the early phases of recovery.
Post operatively these patients are at relatively high risk of developing complications:

  • Atelectasis- due to the effects of thoracotomy and lung collapse
  • Anastomotic leakage. The risk is relatively high owing to the presence of a relatively devascularised stomach. Often the only blood supply is from the gastroepiploic artery as all others will have been divided. If a leak does occur then many will attempt to manage conservatively with prolonged nasogastric tube drainage and TPN. The reality is that up to 50% of patients developing an anastomotic leak will not survive to discharge.
  • Delayed gastric emptying (may be avoided by performing a pyloroplasty).
1117
Q

Where does Stensens duct primarily open?

Immediately lateral to the foramen caecum

Floor of mouth

Opposite the second molar tooth

Opposite the fifth molar tooth

Into the post nasal space

A

Stensens duct conveys secretions from the parotid gland and these enter the oral cavity at the level of the second molar tooth.

Parotid gland

Anatomy of the parotid gland
Location Overlying the mandibular ramus; anterior and inferior to the ear.
Salivary duct Crosses the masseter, pierces the buccinator and drains adjacent to the 2nd upper molar tooth (Stensen’s duct).
Structures passing through the gland
Facial nerve (Mnemonic: The Zebra Buggered My Cat; Temporal Zygomatic, Buccal, Mandibular, Cervical)
External carotid artery
Retromandibular vein
Auriculotemporal nerve
Relations
Anterior: masseter, medial pterygoid, superficial temporal and maxillary artery, facial nerve, stylomandibular ligament
Posterior: posterior belly digastric muscle, sternocleidomastoid, stylohyoid, internal carotid artery, mastoid process, styloid process
Arterial supply Branches of external carotid artery
Venous drainage Retromandibular vein
Lymphatic drainage Deep cervical nodes
Nerve innervation
Parasympathetic-Secretomotor
Sympathetic-Superior cervical ganglion
Sensory- Greater auricular nerve

Parasympathetic stimulation produces a water rich, serous saliva. Sympathetic stimulation leads to the production of a low volume, enzyme-rich saliva.

1118
Q

A 74 year old man presents with a painful right leg. The pain developed suddenly the preceeding evening. However, he wondered if it might resolve overnight, instead, it has got worse. On examination, his right leg is cold and white with diminished distal sensation. A CT angiogram shows a thrombus occluding the external iliac artery with no atheromatous disease. What is the most appropriate course of action?

Femoral embolectomy and below knee fasciotomy

Femoral embolectomy alone

Femoral popliteal bypass graft without fasciotomy

Femoral embolectomy with above and below knee fasciotomy

Femoro-femoro cross over graft with above and below knee fasciotomy

A

Delayed limb re-perfusion = Risk of compartment syndrome
Delayed surgery for limb ischaemic carries a risk of re-perfusion injury and compartment syndrome. Where surgery is delayed beyond 6 hours, most surgeons would perform a fasciotomy at the same time as the embolectomy. However, whilst the compartments below the knee are vulnerable, its very rare for this to occur in the thigh and so, as a rule, the thigh is not treated in this manner.

Acute limb ischaemia

  • Thrombosis of a pre-existing site of atherosclerosis is the commonest cause of acute limb ischaemia
    Acute thrombosis of popliteal aneurysms poses the greatest threat to the limb
    Sudden occlusion of a large proximal vessel results in the typical appearances of acute limb ischaemia

Clinical appearances
Less than 6 hours = White leg
At 6 -12 hours = Mottled limb with blanching on pressure
More than 12-24 hours = Fixed mottling

Management of acutely ischaemic leg
Clinical picture Treatment
White leg with sensorimotor deficit Surgery and embolectomy
Dusky leg, mild anaesthesia Angiography
Fixed mottling Primary amputation

Role of thrombolysis
Intra arterial thrombolysis is better than peripheral thrombolysis
Mainly indicated in acute on chronic thrombosis
Avoid if within 2 months of CVA or 2 weeks of surgery
Aspiration of clot may improve success rate if the thrombosis is large

Surgery
Both groins should be prepared
Transverse arteriotomy is easier to close
Poor inflow should be managed with iliac trawl- if this fails to improve then consider a femoro-femoral cross over or axillo-femoral cross over.
A check angiogram should be performed on table and prior to closure
Systemic heparinisation should follow surgery
Fasciotomy should be considered if the time between onset and surgery exceeds 6 hours

1119
Q

A 34 year old lady presents to her general practitioner with peri anal discomfort. The general practitioner diagnoses pruritus ani, which of the following is least associated with the condition?

Hyperbilirubinaemia

Anal fissure

Leukaemia

Syphilis

Tuberculosis

A

Causes:

Systemic (DM, Hyperbilirubinaemia, aplastic anaemia)
Mechanical (diarrhoea, constipation, anal fissure)
Infections (STDs)
Dermatological
Drugs (quinidine, colchicine)
Topical agents

Ano rectal disease

Haemorrhoids Location: 3, 7, 11 o’clock position
Internal or external
Treatment: Conservative, Rubber band ligation, Haemorrhoidectomy
Fissure in ano Location: midline 6 (posterior midline 90%) and 12 o’clock position. Distal to the dentate line
Chronic fissure > 6/52: triad: Ulcer, sentinel pile, enlarged anal papillae
Proctitis Causes: Crohn’s, ulcerative colitis, Clostridium difficile
Ano rectal abscess E.coli, staph aureus
Positions: Perianal, Ischiorectal, Pelvirectal, Intersphincteric
Anal fistula Usually due to previous ano-rectal abscess
Intersphincteric, transsphincteric, suprasphincteric, and extrasphincteric. Goodsalls rule determines location
Rectal prolapse Associated with childbirth and rectal intussceception. May be internal or external
Pruritus ani Systemic and local causes
Anal neoplasm Squamous cell carcinoma commonest unlike adenocarcinoma in rectum
Solitary rectal ulcer Associated with chronic straining and constipation. Histology shows mucosal thickening, lamina propria replaced with collagen and smooth muscle (fibromuscular obliteration)

Rectal prolapse
Common especially in multiparous women.
May be internal or external.
Internal rectal prolapse can present insidiously.
External prolapse can ulcerate and in long term impair continence.
Diagnostic work up includes colonoscopy, defecating proctogram, ano rectal manometry studies and if doubt exists an examination under anaesthesia.

Treatments for prolapse
In the acute setting reduce it (covering it with sugar may reduce swelling.
Delormes procedure which excises mucosa and plicates the rectum (high recurrence rates) may be used for external prolapse.
Altmeirs procedure which resects the colon via the perineal route has lower recurrence rates but carries the risk of anastamotic leak.
Rectopexy is an abdominal procedure in which the rectum is elevated and usually supported at the level of the sacral promontory. Post operative constipation may be reduced by limiting the dissection to the anterior plane (laparoscopic ventral mesh rectopexy).

Pruritus ani
Extremely common.
Check not secondary to altered bowel habits (e.g. Diarrhoea)
Associated with underlying diseases such as haemorrhoids.
Examine to look for causes such as worms.
Proctosigmoidoscopy to identify associated haemorrhoids and exclude cancer.
Treatment is largely supportive and patients should avoid using perfumed products around the area.

Fissure in ano
Typically painful PR bleeding (bright red).
Nearly always in the posterior midline.
Usually solitary.

Treatment
Stool softeners.
Topical diltiazem (or GTN).
If topical treatments fail then botulinum toxin should be injected.
If botulinum toxin fails then males should probably undergo lateral internal sphincterotomy.
Females who do not respond to botulinum toxin should undergo ano rectal manometry studies and endo anal USS prior to being offered surgery such as sphincterotomy.

1120
Q

A 63 year nurse falls on an extended and pronated wrist. An x-ray shows a distal radial fracture with radiocarpal dislocation. Which type of fracture is most likely?

Bennett’s

Barton’s

Pott’s

Smith’s

Monteggia’s

A

Barton’s fractures tend to have intra-articular involvement and dislocation may sometimes be present.

Eponymous fractures

Colles’ fracture (dinner fork deformity)
Fall onto extended outstretched hand
Classical Colles’ fractures have the following 3 features:

  1. Transverse fracture of the radius
  2. 1 inch proximal to the radio-carpal joint
  3. Dorsal displacement and angulation

Smith’s fracture (reverse Colles’ fracture)
Volar angulation of distal radius fragment (Garden spade deformity)
Caused by falling backwards onto the palm of an outstretched hand or falling with wrists flexed

Bennett’s fracture
Intra-articular fracture of the first carpometacarpal joint
Impact on flexed metacarpal, caused by fist fights
X-ray: triangular fragment at ulnar base of metacarpal

Monteggia’s fracture
Dislocation of the proximal radioulnar joint in association with an ulna fracture
Fall on outstretched hand with forced pronation
Needs prompt diagnosis to avoid disability

Galeazzi fracture
Radial shaft fracture with associated dislocation of the distal radioulnar joint
Direct blow

Pott’s fracture
Bimalleolar ankle fracture
Forced foot eversion

Barton’s fracture
Distal radius fracture (Colles’/Smith’s) with associated radiocarpal dislocation
Fall onto extended and pronated wrist
Involvement of the joint is a defining feature

Holstein Lewis Fracture
A HolsteinLewis fracture is a fracture of the distal third of the humerus resulting in entrapment of the radial nerve.
The radial nerve is one of the major nerves of the upper limb. It innervates all of the muscles in the extensor compartments of the arm.
Conservative treatment includes reduction and use of a functional brace
Vascular injury may require open surgery

1121
Q

A 73 year old lady suffers a fracture at the surgical neck of the humerus. The decision is made to operate. There are difficulties in reducing the fracture and a vessel lying posterior to the surgical neck is injured. Which of the following is this vessel most likely to be?

Axillary artery

Brachial artery

Thoracoacromial artery

Transverse scapular artery

Posterior circumflex humeral artery

A

The circumflex humeral arteries lie at the surgical neck and is this scenario the posterior circumflex is likely to be injured. The thoracoacromial and transverse scapular arteries lie more superomedially. The posterior circumflex humeral artery is a branch of the axillary artery.

Shoulder joint

Shallow synovial ball and socket type of joint.
It is an inherently unstable joint, but is capable to a wide range of movement.
Stability is provided by muscles of the rotator cuff that pass from the scapula to insert in the greater tuberosity (all except sub scapularis-lesser tuberosity).

Glenoid labrum
Fibrocartilaginous rim attached to the free edge of the glenoid cavity
Tendon of the long head of biceps arises from within the joint from the supraglenoid tubercle, and is fused at this point to the labrum.
The long head of triceps attaches to the infraglenoid tubercle

Fibrous capsule
Attaches to the scapula external to the glenoid labrum and to the labrum itself (postero-superiorly)
Attaches to the humerus at the level of the anatomical neck superiorly and the surgical neck inferiorly
Anteriorly the capsule is in contact with the tendon of subscapularis, superiorly with the supraspinatus tendon, and posteriorly with the tendons of infraspinatus and teres minor. All these blend with the capsule towards their insertion.
Two defects in the fibrous capsule; superiorly for the tendon of biceps. Anteriorly there is a defect beneath the subscapularis tendon.
The inferior extension of the capsule is closely related to the axillary nerve at the surgical neck and this nerve is at risk in anteroinferior dislocations. It also means that proximally sited osteomyelitis may progress to septic arthritis.

Movements and muscles
Flexion Anterior part of deltoid
Pectoralis major
Biceps
Coracobrachialis
Extension Posterior deltoid
Teres major
Latissimus dorsi
Adduction Pectoralis major
Latissimus dorsi
Teres major
Coracobrachialis
Abduction Mid deltoid
Supraspinatus
Medial rotation Subscapularis
Anterior deltoid
Teres major
Latissimus dorsi
Lateral rotation Posterior deltoid
Infraspinatus
Teres minor

Important anatomical relations
Anteriorly Brachial plexus
Axillary artery and vein
Posterior Suprascapular nerve
Suprascapular vessels
Inferior Axillary nerve
Circumflex humeral vessels

1122
Q

Which of the following conditions is most likely to be associated with these arterial blood gas sample results:
pH 7.48
pO2 10.1
HCO3 30
pCO2 4.5
Cl- 96mEq

Respiratory alkalosis

Metabolic acidosis with normal anion gap

Metabolic alkalosis

Metabolic acidosis with increased anion gap

Type 2 respiratory failure

A

These arterial blood gas results are classically seen in situations where there is metabolic alkalosis such as may occur following prolonged vomiting.

Disorders of acid - base balance

Disorders of acid- base balance are often covered in the MRCS part A.

Metabolic acidosis
This is the most common surgical acid - base disorder.
Reduction in plasma bicarbonate levels.
Two mechanisms:
1. Gain of strong acid (e.g. diabetic ketoacidosis)
2. Loss of base (e.g. from bowel in diarrhoea)
- Classified according to the anion gap, this can be calculated by:
(Na+ + K+) - (Cl- + HCO3-).
- If a question supplies the chloride level then this is often a clue that the anion gap should be calculated. The normal range = 10-18 mmol/L

Normal anion gap ( = hyperchloraemic metabolic acidosis)
Gastrointestinal bicarbonate loss: diarrhoea, ureterosigmoidostomy, fistula
Renal tubular acidosis
Drugs: e.g. acetazolamide
Ammonium chloride injection
Addison’s disease

Raised anion gap
Lactate: shock, hypoxia
Ketones: diabetic ketoacidosis, alcohol
Urate: renal failure
Acid poisoning: salicylates, methanol

Metabolic acidosis secondary to high lactate levels may be subdivided into two types:
Lactic acidosis type A: (Perfusion disorders e.g.shock, hypoxia, burns)
Lactic acidosis type B: (Metabolic e.g. metformin toxicity)

Metabolic alkalosis
Usually caused by a rise in plasma bicarbonate levels.
Rise of bicarbonate above 24 mmol/L will typically result in renal excretion of excess bicarbonate.
Caused by a loss of hydrogen ions or a gain of bicarbonate. It is due mainly to problems of the kidney or gastrointestinal tract

Causes
Vomiting / aspiration (e.g. Peptic ulcer leading to pyloric stenosis, nasogastric suction)
Diuretics
Liquorice, carbenoxolone
Hypokalaemia
Primary hyperaldosteronism
Cushing’s syndrome
Bartter’s syndrome
Congenital adrenal hyperplasia

Mechanism of metabolic alkalosis
Activation of renin-angiotensin II-aldosterone (RAA) system is a key factor
Aldosterone causes reabsorption of Na+ in exchange for H+ in the distal convoluted tubule
ECF depletion (vomiting, diuretics) → Na+ and Cl- loss → activation of RAA system → raised aldosterone levels
In hypokalaemia, K+ shift from cells → ECF, alkalosis is caused by shift of H+ into cells to maintain neutrality

Respiratory acidosis
Rise in carbon dioxide levels usually as a result of alveolar hypoventilation
Renal compensation may occur leading to Compensated respiratory acidosis

Causes
COPD
Decompensation in other respiratory conditions e.g. Life-threatening asthma / pulmonary oedema
Sedative drugs: benzodiazepines, opiate overdose

Respiratory alkalosis
Hyperventilation resulting in excess loss of carbon dioxide
This will result in increasing pH

Causes
Psychogenic: anxiety leading to hyperventilation
Hypoxia causing a subsequent hyperventilation: pulmonary embolism, high altitude
Early salicylate poisoning*
CNS stimulation: stroke, subarachnoid haemorrhage, encephalitis
Pregnancy

*Salicylate overdose leads to a mixed respiratory alkalosis and metabolic acidosis. Early stimulation of the respiratory centre leads to a respiratory alkalosis whilst later the direct acid effects of salicylates (combined with acute renal failure) may lead to an acidosis

1123
Q

A 77 year old man presents to pre operative clinic for a total knee replacement. He is on furosemide for hypertension. He is known to have multiple myeloma. He is found to have the following test results:
Na 120
Serum osmolality 280 (normal)
Urine osmolality normal
Urine Na normal
What is the most likely cause?

Pseudohyponatraemia

Syndrome of inappropriate ADH secretion (SIADH)

Hypotonic hypovolaemic hyponatraemia

Psychogenic polydipsia

Hypertonic hypovolaemic hyponatraemia

A

Hyperlipidaemia and multiple myeloma are known to cause a pseudohyponatraemia, this is due to raised protein.

Hyponatraemia

This is commonly tested in the MRCS (despite most surgeons automatically seeking medical advice if this occurs!). The most common cause in surgery is the over administration of 5% dextrose.

Hyponatraemia may be caused by water excess or sodium depletion. Causes of pseudohyponatraemia include hyperlipidaemia (increase in serum volume) or a taking blood from a drip arm. Urinary sodium and osmolarity levels aid making a diagnosis.

Classification
Urinary sodium > 20 mmol/l Sodium depletion, renal loss
Patient often hypovolaemic
Diuretics (thiazides)
Addison’s
Diuretic stage of renal failure
SIADH (serum osmolality low, urine osmolality high, urine Na high)
Patient often euvolaemic
Mnemonic: Syndrome of INAPPropriate Anti-Diuretic Hormone:
In creased
Na (sodium)
PP (urine)
Urinary sodium < 20 mmol/l Sodium depletion, extra-renal loss
Diarrhoea, vomiting, sweating
Burns, adenoma of rectum (if villous lesion and large)
Water excess (patient often hypervolaemic and oedematous)
Secondary hyperaldosteronism: CCF, cirrhosis
Reduced GFR: renal failure
IV dextrose, psychogenic polydipsia

Management

Symptomatic Hyponatremia :

Acute hyponatraemia with Na <120: immediate therapy. Central Pontine Myelinolisis, may occur from overly rapid correction of serum sodium. Aim to correct until the Na is > 125 at a rate of 1 mEq/h. Normal saline with frusemide is an alternative method.

The sodium requirement can be calculated as follows :

(125 - serum sodium) x 0.6 x body weight = required mEq of sodium

1124
Q

An 18 year old lady presents with extensive varicose veins of her left leg. There is associated port wine staining. What is the most likely diagnosis?

Type 1 diabetes

Osler syndrome

Gardner’s syndrome

Proteus syndrome

Klippel-Trenaunay-Weber syndrome

A

A less common cause of venous insufficiency is Klippel-Trenaunay-Weber (KTW) syndrome, which involves port-wine stains, varicose veins, and bony or soft-tissue hypertrophy.

Lower leg ulcers

Venous leg ulcers
Most due to venous hypertension, secondary to chronic venous insufficiency (other causes include calf pump dysfunction or neuromuscular disorders)
Ulcers form due to capillary fibrin cuff or leucocyte sequestration
Features of venous insufficiency include oedema, brown pigmentation, lipodermatosclerosis, eczema
Location above the ankle, painless
Deep venous insufficiency is related to previous DVT and superficial venous insufficiency is associated with varicose veins
Doppler ultrasound looks for presence of reflux and duplex ultrasound looks at the anatomy/ flow of the vein
Management: 4 layer compression banding after exclusion of arterial disease or surgery
If fail to heal after 12 weeks or >10cm2 skin grafting may be needed

Marjolin’s ulcer

Squamous cell carcinoma
Occurring at sites of chronic inflammation e.g; burns, osteomyelitis after 10-20 years
Mainly occur on the lower limb

Arterial ulcers
Occur on the toes and heel
Painful
There may be areas of gangrene
Cold with no palpable pulses
Low ABPI measurements

Neuropathic ulcers
Commonly over plantar surface of metatarsal head and plantar surface of hallux
The plantar neuropathic ulcer is the condition that most commonly leads to amputation in diabetic patients
Due to pressure
Management includes cushioned shoes to reduce callus formation

Pyoderma gangrenosum

Associated with inflammatory bowel disease/RA
Can occur at stoma sites
Erythematous nodules or pustules which ulcerate

1125
Q

A 56 year old man underwent a truncal vagotomy operation many years ago. The direct release of which of these hormones will be most affected as a result?

Cholecystokinin

Secretin

Motilin

Glucose-dependent insulinotropic peptide

Gastrin

A

Motilin is released by the small bowel in response to alkaline contents in the duodenum/ jejunum and increases the migrating motor complexes in the small bowel
Cholecystokinin, secretin and GIP are all released in response to changes in the luminal contents. Gastrin is released from G cells and whilst the luminal contents affect its release, it is also directly released in response to vagal stimuli.

Hormonal control of gastric acid secretion

Source Stimulus Actions
Gastrin
G cells in antrum of the stomach
Distension of stomach, extrinsic nerves
Inhibited by: low antral pH, somatostatin
Increase HCL, pepsinogen and IF secretion, increases gastric motility, trophic effect on gastric mucosa
CCK
I cells in upper small intestine
Partially digested proteins and triglycerides
Increases secretion of enzyme-rich fluid from pancreas, contraction of gallbladder and relaxation of sphincter of Oddi, decreases gastric emptying, trophic effect on pancreatic acinar cells, induces satiety
Secretin
S cells in upper small intestine
Acidic chyme, fatty acids
Increases secretion of bicarbonate-rich fluid from pancreas and hepatic duct cells, decreases gastric acid secretion, trophic effect on pancreatic acinar cells
VIP
Small intestine, pancreas
Neural
Stimulates secretion by pancreas and intestines, inhibits acid and pepsinogen secretion
Somatostatin
D cells in the pancreas and stomach
Fat, bile salts and glucose in the intestinal lumen
Decreases acid and pepsin secretion, decreases gastrin secretion, decreases pancreatic enzyme secretion, decreases insulin and glucagon secretion
inhibits trophic effects of gastrin, stimulates gastric mucous production

1126
Q

A 22 year old lady presents with symptoms and signs of hyperthyroidism. Her diagnostic work up results in a diagnosis of Graves disease. Which of these statements best describes the pathophysiology of the condition?

Formation of IgG antibodies to the TSH receptors on the thyroid gland

Formation of IgG antibodies to the TRH receptors on the anterior pituitary

Formation of IgM antibodies to the TSH receptors on the thyroid gland

Formation of IgA antibodies to the TSH receptors on the thyroid gland

Formation of IgM antibodies to the TRH receptors on the anterior pituitary

A

Usually IgG antibodies are formed against the TSH receptors on the thyroid gland. Which is why the TSH level is often very low in Graves disease.

Thyroid hormones

Hormones of the thyroid gland
Triiodothyronine T3 Major hormone active in target cells
Thyroxine T4 Most prevalent form in plasma, less biologically active than T3
Calcitonin Lowers plasma calcium

Synthesis and secretion of thyroid hormones
Thyroid actively concentrates iodide to twenty five times the plasma concentration.
Iodide is oxidised by peroxidase in the follicular cells to atomic iodine which then iodinates tyrosine residues contained in thyroglobulin.
Iodinated tyrosine residues in thyroglobulin undergo coupling to either T3 or T4.
Process is stimulated by TSH, which stimulates secretion of thyroid hormones.
The normal thyroid has approximately 3 month reserves of thyroid hormones.

LATS and Graves disease
In Graves disease, patients develop IgG antibodies to the TSH receptors on the thyroid gland. This results in chronic and long term stimulation of the gland with release of thyroid hormones. The typically situation is raised thyroid hormones and low TSH. Thyroid receptor autoantibodies should be checked in individuals presenting with hyperthyroidism as they are present in up to 85% cases.

1127
Q

Which of the following nerve roots contribute nerve fibres to the ansa cervicalis?

C1 only

C1, C2 and C3

C2, C3 and C6

C2, C4 and C5

C4, C5 and C6

A

The ansa cervicalis is composed of a superior and inferior root, derived from C1, C2 and C3. The superior root arises where the nerve crosses the internal carotid artery. It descends anterior to the carotid sheath in the anterior triangle. It is joined in the region of the mid neck by the inferior root. The inferior root may pass either superficially or deep to the internal jugular vein.

Ansa cervicalis

Superior root Branch of C1 anterolateral to carotid sheath
Inferior root Derived from C2 and C3 roots, passes posterolateral to the internal jugular vein (may lie either deep or superficial to it)
Innervation Sternohyoid
Sternothyroid
Omohyoid

The ansa cervicalis lies anterior to the carotid sheath. The nerve supply to the inferior strap muscles enters at their inferior aspect. Therefore when dividing these muscles to expose a large goitre, the muscles should be divided in their upper half.

1128
Q

What is the arterial blood supply to the lacrimal apparatus?

Nasociliary artery

Supra orbital artery

Internal carotid artery

Ophthalmic artery

Supra trochlear artery

A

The ophthalmic artery supplies the gland

Lacrimal apparatus

Comprises the lacrimal gland and its ducts, lacrimal canaliculi, lacrimal sac and naso lacrimal duct. It lies anteriorly in the superolateral region of the orbit and is divided into 2 parts by the levator palpebrae superioris. Numerous ducts empty glandular secretions into the lateral part of the superior fornix of the conjunctiva. The fluid so produced finally accumulates in the lacrimal lake from which it drains via the lacrimal canaliculi, one with each eyelid. Passing medially, the lacrimal canaliculi eventually join the lacrimal sac between the anterior and posterior lacrimal crests, posterior to the medial palpebral ligament and anterior to the lacrimal part of the orbicularis oculi muscle. When the orbicularis oculi muscle contracts during blinking, the small lacrimal part of the muscle dilates the lacrimal sac and draws tears into it.

Innervation
Secretomotor fibres from the parasympathetic nervous system. The preganglionic parasympathetic neurons leave the CNS in the facial nerve, enter the greater petrosal nerve and continue with this nerve until it becomes the nerve of the pterygoid canal. The nerve of the pterygoid canal eventually joins the pterygopalatine ganglion where the pre-ganglionic parasympathetic neurons synapse on post ganglionic parasympathetic neurons. The post ganglionic neurons join the maxillary nerve and continue with it until the zygomatic nerve branches from it, and travels with the zygomatic nerve until it gives off the zygomaticotemporal nerve which eventually distributes the post ganglionic parasympathetic fibres in a small branch that joins the lacrimal nerve.

Sympathetic innervation follows a similar path to the parasympathetic path described above. Post ganglionic sympathetic fibres originating in the superior cervical ganglion travel along the plexus surrounding the internal carotid. They leave this plexus as the deep petrosal nerve and join the parasympathetic fibres in the nerve of the pterygoid canal. Passing through the pterygopalatine ganglion, the parasympathetic fibres from this point onwards follow the same path as the parasympathetic fibres to the lacrimal gland.

Vessels
Arterial supply is from branches of the ophthalmic artery and venous drainage is to the ophthalmic veins

1129
Q

During a block dissection of the groin, the sartorius muscle is identified. What is the nerve supply to this muscle?

Obturator nerve

Posterior division of the femoral nerve

Anterior division of the femoral nerve

Sciatic nerve

Genitofemoral nerve

A

Sartorius is supplied by the anterior (superficial) branch of the femoral nerve.
Most of the branches of the superficial femoral nerve are cutaneous. However, it does also supply sartorius.
Sartorius

  • Longest strap muscle in the body
    Most superficial muscle in the anterior compartment of the thigh

Origin Anterior superior iliac spine
Insertion Medial surface of the of the body of the tibia (upper part). It inserts anterior to gracilis and semitendinosus
Nerve Supply Femoral nerve (L2,3 -root values for sartorius)
Action
Flexor of the hip and knee, slight abducts the thigh and rotates it laterally
It assists with medial rotation of the tibia on the femur. For example it would play a pivotal role in placing the right heel onto the left knee ( and vice versa)
Important relations The middle third of this muscle, and its strong underlying fascia forms the roof of the adductor canal , in which lie the femoral vessels, the saphenous nerve and the nerve to vastus medialis.

1130
Q

A 30 year old woman, with a recent diagnosis of Graves disease, underwent incision and drainage of a pilonidal abscess three hours ago. She has become agitated, confused and is noted to be jaundiced and sweaty. Her temperature is 39 oC, pulse 152 beats / minute and blood pressure 95/60 mmHg. An ECG shows an irregular ventricular rate with absent p waves. After resuscitation what is the most appropriate next step in management?

Bile acid sequestrants and glucocorticoids

Bile acid sequestrants and iodides

Beta adrenoceptor blockers and thionamides

Beta adrenoceptor blockers and iodides

Thionamides and thyroidectomy

A

In the first instance, control of cardiac rate and administration of antithyroid drugs are typically used. Development of jaundice in people with a thyrotoxic storm is well described and the aetiology multifactorial.

Management of thyroid disease

Management of thyroid disease

Lesion Management
Large multinodular goitre Surgery for pressure symptoms. Total thyroidectomy is treatment of choice
Toxic nodule Hemithyroidectomy
Follicular lesion (THY 3f) Hemithyroidectomy to establish diagnosis
Papillary thyroid cancer Total thyroidectomy and central compartment nodal dissection (extended lymphadenectomy as required)
Follicular thyroid cancer Total thyroidectomy (usually completion as already had hemithyroidectomy)
Anaplastic thyroid cancer Palliative radiotherapy
Medullary thyroid cancer Total thyroidectomy and central compartment nodal dissection (screen for other MEN tumours)
Lymphoma of the thyroid Consider core biopsy
Persistent refilling cysts Injection sclerotherapy and surgery if this fails
Graves disease with significant eye signs Total thyroidectomy
Graves disease without significant eye signs Patient choice radioiodine Vs surgery

1131
Q

A 78 year old man develops a carcinoma of the scrotum. To which of the following lymph node groups may the tumour initially metastasise?

Para aortic

Obturator

Inguinal

Meso rectal

None of the above

A

The scrotum is drained by the inguinal nodes.

Scrotal and testicular anatomy

Spermatic cord
Formed by the vas deferens and is covered by the following structures:
Layer Origin
Internal spermatic fascia Transversalis fascia
Cremasteric fascia From the fascial coverings of internal oblique
External spermatic fascia External oblique aponeurosis

Contents of the cord
Vas deferens Transmits sperm and accessory gland secretions
Testicular artery Branch of abdominal aorta supplies testis and epididymis
Artery of vas deferens Arises from inferior vesical artery
Cremasteric artery Arises from inferior epigastric artery
Pampiniform plexus Venous plexus, drains into right or left testicular vein
Sympathetic nerve fibres Lie on arteries, the parasympathetic fibres lie on the vas
Genital branch of the genitofemoral nerve Supplies cremaster
Lymphatic vessels Drain to lumbar and para-aortic nodes

Scrotum
Composed of skin and closely attached dartos fascia.
Arterial supply from the anterior and posterior scrotal arteries
Lymphatic drainage to the inguinal lymph nodes
Parietal layer of the tunica vaginalis is the innermost layer

Testes
The testes are surrounded by the tunica vaginalis (closed peritoneal sac). The parietal layer of the tunica vaginalis adjacent to the internal spermatic fascia.
The testicular arteries arise from the aorta immediately inferiorly to the renal arteries.
The pampiniform plexus drains into the testicular veins, the left drains into the left renal vein and the right into the inferior vena cava.
Lymphatic drainage is to the para-aortic nodes.

1132
Q

A 34 year old male donates a unit of blood. It is stored at 4 oC. After 72 hours which of the following clotting factors will be most affected?

Factor IV

Factor II

Factor VIII

Factor IX

Factor XI

A

Factors V and VIII are sensitive to temperature which is the reason why FFP is frozen soon after collection.

Blood products

Whole blood fractions

Fraction Key points
Packed red cells Used for transfusion in chronic anaemia and cases where infusion of large volumes of fluid may result in cardiovascular compromise. Product obtained by centrifugation of whole blood.
Platelet rich plasma Usually administered to patients who are thrombocytopaenic and are bleeding or require surgery. It is obtained by low speed centrifugation.
Platelet concentrate Prepared by high speed centrifugation and administered to patients with thrombocytopaenia.
Fresh frozen plasma
Prepared from single units of blood.
Contains clotting factors, albumin and immunoglobulin.
Unit is usually 200 to 250ml.
Usually used in correcting clotting deficiencies in patients with hepatic synthetic failure who are due to undergo surgery.
Usual dose is 12-15ml/Kg-1.
It should not be used as first line therapy for hypovolaemia.
Cryoprecipitate
Formed from supernatant of FFP.
Rich source of Factor VIII and fibrinogen.
Allows large concentration of factor VIII to be administered in small volume.
SAG-Mannitol Blood Removal of all plasma from a blood unit and substitution with:
Sodium chloride
Adenine
Anhydrous glucose
Mannitol
Up to 4 units of SAG M Blood may be administered. Thereafter whole blood is preferred. After 8 units, clotting factors and platelets should be considered.

Cell saver devices
These collect patients own blood lost during surgery and then re-infuse it. There are two main types:
Those which wash the blood cells prior to re-infusion. These are more expensive to purchase and more complicated to operate. However, they reduce the risk of re-infusing contaminated blood back into the patient.
Those which do not wash the blood prior to re-infusion.
Their main advantage is that they avoid the use of infusion of blood from donors into patients and this may reduce risk of blood borne infection. It may be acceptable to Jehovah’s witnesses. It is contraindicated in malignant disease for risk of facilitating disease dissemination.

Blood products used in warfarin reversal
In some surgical patients the use of warfarin can pose specific problems and may require the use of specialised blood products

Immediate or urgent surgery in patients taking warfarin(1) (2):

  1. Stop warfarin
  2. Vitamin K (reversal within 4-24 hours)
    -IV takes 4-6h to work (at least 5mg)
    -Oral can take 24 hours to be clinically effective
  3. Fresh frozen plasma
    Used less commonly now as 1st line warfarin reversal
    -30ml/kg-1
    -Need to give at least 1L fluid in 70kg person (therefore not appropriate in fluid overload)
    -Need blood group
    -Only use if human prothrombin complex is not available
  4. Human Prothrombin Complex (reversal within 1 hour)
    -Bereplex 50 u/kg
    -Rapid action but factor 6 short half life, therefore give with vitamin K

References
1. Dentali, F., C. Marchesi, et al. (2011). ‘Safety of prothrombin complex concentrates for rapid anticoagulation reversal of vitamin K antagonists. A meta-analysis.’ Thromb Haemost 106(3): 429-438.

  1. http://www.transfusionguidelines.org/docs/pdfs/bbt-03warfarin-reversal-flowchart-2006.pdf
1133
Q

A 67 year old man has an abdominal aortic aneurysm which displaces the left renal vein. Which branch of the aorta is most likely to be affected at this level?

Inferior mesenteric artery

Superior mesenteric artery

Coeliac axis

Testicular artery

None of the above

A

The left renal vein lies behind of the SMA as it branches off the aorta. Whilst juxtarenal AAA may sometimes require the division of the left renal vein, direct involvement of the SMA may require a hybrid surgical bypass and subsequent endovascular occlusion.

Abdominal aortic branches

Branches Level Paired Type
Inferior phrenic T12 (Upper border) Yes Parietal
Coeliac T12 No Visceral
Superior mesenteric L1 No Visceral
Middle suprarenal L1 Yes Visceral
Renal L1-L2 Yes Visceral
Gonadal L2 Yes Visceral
Lumbar L1-L4 Yes Parietal
Inferior mesenteric L3 No Visceral
Median sacral L4 No Parietal
Common iliac L4 Yes Terminal

1134
Q

A 20 year old man is hit over the head with a mallet. On arrival in the accident and emergency department he opens his eyes to pain and groans or grunts. On application of a painful stimulus to his hands, he extends his arm at the elbow. What is his Glasgow coma score

10

6

3

7

4

A

E=2, V= 2, M=2.

Glasgow coma scale

Modality Options
Eye opening
Spontaneous
To speech
To pain
None
Verbal response
Orientated
Confused
Words
Sounds
None
Motor response
Obeys commands
Localises to pain
Withdraws from pain
Abnormal flexion to pain (decorticate posture)
Extending to pain
None

Responses are taken from each category (marks in descending order) to produce an overall score. Severe brain injuries are generally associated with GCS <8.

1135
Q

A 34 year old man undergoes excision of a sarcoma from the right buttock. During the procedure the sciatic nerve is sacrificed. Which of the following will not occur as a result of this process?

Loss of extension at the knee joint

Foot drop

Inability to extend extensor hallucis longus

Impaired function of gastrocnemius

Loss of sensation to the posterior aspect of the lower leg

A

Extension of the knee joint is caused by the obturator and femoral nerves. Sensation to the posterior aspect of the thigh is via the posterior cutaneous nerve of the thigh and this is a direct branch from the plexus itself.

Sciatic nerve

The sciatic nerve is formed from the sacral plexus and is the largest nerve in the body. It is the continuation of the main part of the plexus arising from ventral rami of L4 to S3. These rami converge at the inferior border of piriformis to form the nerve itself. It passes through the inferior part of the greater sciatic foramen and emerges beneath piriformis. Medially, lie the inferior gluteal nerve and vessels and the pudendal nerve and vessels. It runs inferolaterally under the cover of gluteus maximus midway between the greater trochanter and ischial tuberosity. It receives its blood supply from the inferior gluteal artery. The nerve provides cutaneous sensation to the skin of the foot and the leg. It also innervates the posterior thigh muscles and the lower leg and foot muscles. The nerve splits into the tibial and common peroneal nerves approximately half way down the posterior thigh. The tibial nerve supplies the flexor muscles and the common peroneal nerve supplies the extensor muscles and the evertor muscles of the foot.

Summary points
Origin Spinal nerves L4 - S3
Articular Branches Hip joint
Muscular branches in upper leg
Semitendinosus
Semimembranosus
Biceps femoris
Part of adductor magnus
Cutaneous sensation
Posterior aspect of thigh (via cutaneous nerves)
Gluteal region
Entire lower leg (except the medial aspect)
Terminates At the upper part of the popliteal fossa by dividing into the tibial and peroneal nerves

The nerve to the short head of the biceps femoris comes from the common peroneal part of the sciatic and the other muscular branches arise from the tibial portion.
The tibial nerve goes on to innervate all muscles of the foot except the extensor digitorum brevis (which is innervated by the common peroneal nerve).

1136
Q

A 28 year old man presents with musculoskeletal pain. He undergoes a radiological assessment and his x-rays demonstrate widespread osteopaenia with well circumscribed lucent areas. Which of the following imaging modalities is most likely to demonstrate the cause?

Transrectal USS of the prostate gland

USS of the neck

CT scan of the kidney with contrast

MRI of the pancreas

Small bowel MRI scan

A

The description provided would fit with bone lesions that are seen in hyperparathyroidism and this would account for the generalised osteopaenia. Malignancies of the prostate present in older adults and the lesions are different. A GI cause may cause malabsorption but not create lucent areas. Renal tumours are often very hypervascular but there is not osteopaenia.

Primary hyperparathyroidism

In exams, primary hyperparathyroidism is stereotypically seen in elderly females with an unquenchable thirst and an inappropriately normal or raised parathyroid hormone level. It is most commonly due to a solitary adenoma

Causes of primary hyperparathyroidism
80%: solitary adenoma
15%: hyperplasia
4%: multiple adenoma
1%: carcinoma

Features - ‘bones, stones, abdominal groans and psychic moans’
Polydipsia, polyuria
Peptic ulceration/constipation/pancreatitis
Bone pain/fracture
Renal stones
Depression
Hypertension

Associations
Hypertension
Multiple endocrine neoplasia: MEN I and II

Investigations
Raised calcium, low phosphate
PTH may be raised or normal
Technetium-MIBI subtraction scan

Treatment
Parathyroidectomy, if imaging suggests target gland then a focused approach may be used

1137
Q

A 79 year old lady develops sudden onset of abdominal pain and collapses, she has passed a large amount of diarrhoea. In casualty her pH is 7.35 and WCC is 18. What is the most likely cause?

Mesenteric venous thrombosis

Acute mesenteric embolus

Acute on chronic mesenteric ischaemia

Vasculitis

Myocardial infarct

A

Although mesenteric infarct may raise the lactate the pH may be raised often secondary to vomiting.

Mesenteric vessel disease

Mesenteric ischaemia accounts for 1 in 1000 acute surgical admissions. It is primarily caused by arterial embolism resulting in infarction of the colon. It is more likely to occur in areas such as the splenic flexure that are located at the borders of the territory supplied by the superior and inferior mesenteric arteries.

Types
Acute mesenteric embolus (commonest 50%)
Sudden onset abdominal pain followed by profuse diarrhoea.
May be associated with vomiting.
Rapid clinical deterioration.
Serological tests: WCC, lactate, amylase may all be abnormal particularly in established disease. These can be normal in the early phases.
Acute on chronic mesenteric ischaemia
Usually longer prodromal history.
Post prandial abdominal discomfort and weight loss are dominant features. Patients will usually present with an acute on chronic event, but otherwise will tend not to present until mesenteric flow is reduced by greater than 80%.
When acute thrombosis occurs presentation may be as above. In the chronic setting the symptoms will often be those of ischaemic colitis (mucosa is the most sensitive area to this insult).
Mesenteric vein thrombosis
Usually a history over weeks.
Overt abdominal signs and symptoms will not occur until venous thrombosis has reached a stage to compromise arterial inflow.
Thrombophilia accounts for 60% of cases.
Low flow mesenteric infarction
This occurs in patients with multiple co morbidities in whom mesenteric perfusion is significantly compromised by overuse of inotropes or background cardiovascular compromise.
The end result is that the bowel is not adequately perfused and infarcts occur from the mucosa outwards.

Diagnosis
Serological tests: WCC, lactate, CRP, amylase (can be normal in early disease).
Cornerstone for diagnosis of arterial AND venous mesenteric disease is CT angiography scanning in the arterial phase with thin slices (<5mm). Venous phase contrast is not helpful.
SMA duplex USS is useful in the evaluation of proximal SMA disease in patients with chronic mesenteric ischaemia.
MRI is of limited use due to gut peristalsis and movement artefact.

Management
Overt signs of peritonism: Laparotomy
Mesenteric vein thrombosis: If no peritonism: Medical management with IV heparin
At operation limited resection of frankly necrotic bowel with view to relook laparotomy at 24-48h. In the interim urgent bowel revascularisation via endovascular (preferred) or surgery.

Prognosis
Overall poor. Best outlook is from an acute ischaemia from an embolic event where surgery occurs within 12h. Survival may be 50%. This falls to 30% with treatment delay. The other conditions carry worse survival figures.

1138
Q

A 63 year old man finds that he has to stop walking after 100 yards due to bilateral calf pain. He finds that bending forwards and walking up hill helps. He is able to ride a bike without any pain. What is the most likely underlying cause?

Lumbar canal stenosis

Diabetic neuropathy

Aorto-iliac occlusion

Occlusion of the superficial femoral artery

Pelvic rheumatoid arthritis

A

The positional nature of the pain and the fact that improves with walking uphill makes an underlying vascular aetiology far less likely.

Lumbar spinal stenosis

Lumbar spinal stenosis is a condition in which the central canal is narrowed by tumour, disk prolapse or other similar degenerative changes.
Patients may present with a combination of back pain, neuropathic pain and symptoms mimicking claudication. One of the main features that may help to differentiate it from true claudication in the history is the positional element to the pain. Sitting is better than standing and patients may find it easier to walk uphill rather than downhill. The neurogenic claudication type history makes lumbar spinal stenosis a likely underlying diagnosis, the absence of such symptoms makes it far less likely.

Pathology
Degenerative disease is the commonest underlying cause. Degeneration is believed to begin in the intervertebral disk where biochemical changes such as cell death and loss of proteoglycan and water content lead to progressive disk bulging and collapse. This process leads to an increased stress transfer to the posterior facet joints, which accelerates cartilaginous degeneration, hypertrophy, and osteophyte formation; this is associated with thickening and distortion of the ligamentum flavum. The combination of the ventral disk bulging, osteophyte formation at the dorsal facet, and ligamentum flavum hyptertrophy combine to circumferentially narrow the spinal canal and the space available for the neural elements. The compression of the nerve roots of the cauda equina leads to the characteristic clinical signs and symptoms of lumbar spinal stenosis.

Diagnosis
MRI scanning is the best modality for demonstrating the canal narrowing. Historically a bicycle test was used as true vascular claudicants could not complete the test.

Treatment
Laminectomy

1139
Q

A baby is born by normal vaginal delivery at 39 weeks gestation. Initially all appears well and then the clinical staff become concerned because the baby develops recurrent episodes of cyanosis. These are worse during feeding and improve dramatically when the baby cries. The most likely underlying diagnosis is:

Choanal atresia

Oesophageal reflux

Tetralogy of Fallot

Oesophageal atresia

Congenital diaphragmatic hernia

A

In Choanal atresia the episodes of cyanosis are usually worst during feeding. Improvement may be seen when the baby cries as the oropharyngeal airway is used.

Choanal atresia

Congenital disorder with an incidence of 1 in 7000 births.
Posterior nasal airway occluded by soft tissue or bone.
Associated with other congenital malformations e.g. coloboma
Babies with unilateral disease may go unnoticed.
Babies with bilateral disease will present early in life as they can then only breathe through their mouth.
Treatment is with fenestration procedures designed to restore patency.

1140
Q

A 29 year old woman has had a Pfannenstiel incision. She has pain over the inguinal ligament which radiates to the lower abdomen. There is tenderness when the inguinal canal is compressed. Which of the following is most likely to have been affected?

Genitofemoral nerve

Ilioinguinal nerve

Lateral cutaneous nerve of the thigh

Iliohypogastric nerve

Saphenous nerve

A

Ilioinguinal nerve

Nerve lesions during surgery

A variety of different procedures carry the risk of iatrogenic nerve injury. These are important not only from the patients perspective but also from a medicolegal standpoint.

The following operations and their associated nerve lesions are listed here:
Posterior triangle lymph node biopsy and accessory nerve lesion.
Lloyd Davies stirrups and common peroneal nerve.
Thyroidectomy and laryngeal nerve.
Anterior resection of rectum and hypogastric autonomic nerves.
Axillary node clearance; long thoracic nerve, thoracodorsal nerve and intercostobrachial nerve.
Inguinal hernia surgery and ilioinguinal nerve.
Varicose vein surgery- sural and saphenous nerves.
Posterior approach to the hip and sciatic nerve.
Carotid endarterectomy and hypoglossal nerve.

There are many more, with sound anatomical understanding of the commonly performed procedures the incidence of nerve lesions can be minimised. They commonly occur when surgeons operate in an unfamiliar tissue plane or by blind placement of haemostats (not recommended).

1141
Q

The motor nucleus of cranial nerve V supplies all except which of the following muscles?

Masseter

Posterior belly of digastric

Temporalis

Tensor tympani

Tensor veli palatini

A

The posterior belly of digastric is supplied by the branchial motor component of the facial nerve. All of the other muscles are supplied by axons from the motor nucleus of cranial nerve V.
Trigeminal nerve

The trigeminal nerve is the main sensory nerve of the head. In addition to its major sensory role, it also innervates the muscles of mastication.

Distribution of the trigeminal nerve
Sensory
Scalp
Face
Oral cavity (and teeth)
Nose and sinuses
Dura mater
Motor
Muscles of mastication
Mylohyoid
Anterior belly of digastric
Tensor tympani
Tensor palati
Autonomic connections (ganglia)
Ciliary
Sphenopalatine
Otic
Submandibular

Path
Originates at the pons
Sensory root forms the large, crescentic trigeminal ganglion within Meckel’s cave, and contains the cell bodies of incoming sensory nerve fibres. Here the 3 branches exit.
The motor root cell bodies are in the pons and the motor fibres are distributed via the mandibular nerve. The motor root is not part of the trigeminal ganglion.

Branches of the trigeminal nerve
Ophthalmic nerve Sensory only
Maxillary nerve Sensory only
Mandibular nerve Sensory and motor

Sensory
Ophthalmic Exits skull via the superior orbital fissure
Sensation of: scalp and forehead, the upper eyelid, the conjunctiva and cornea of the eye, the nose (including the tip of the nose, except alae nasi), the nasal mucosa, the frontal sinuses, and parts of the meninges (the dura and blood vessels).
Maxillary nerve Exit skull via the foramen rotundum
Sensation: lower eyelid and cheek, the nares and upper lip, the upper teeth and gums, the nasal mucosa, the palate and roof of the pharynx, the maxillary, ethmoid and sphenoid sinuses, and parts of the meninges.
Mandibular nerve Exit skull via the foramen ovale
Sensation: lower lip, the lower teeth and gums, the chin and jaw (except the angle of the jaw), parts of the external ear, and parts of the meninges.

Motor
Distributed via the mandibular nerve.
The following muscles of mastication are innervated:
Masseter
Temporalis
Medial pterygoid
Lateral pterygoid

Other muscles innervated include:
Tensor veli palatini
Mylohyoid
Anterior belly of digastric
Tensor tympani

1142
Q

What vessel is the origin of the middle rectal artery?

Aorta

Inferior mesenteric artery

Superior mesenteric artery

Internal iliac artery

Internal pudendal artery

A

The rectum is supplied by 3 main vessels
Superior rectal artery from inferior mesenteric artery
Middle rectal artery from the internal iliac artery
Inferior rectal artery from the internal pudendal artery
The rectum has a dual blood supply. The superior rectal artery is a branch of the inferior mesenteric artery. Inferiorly, it derives a blood supply from branches of the internal iliac artery.

Rectum

The rectum is approximately 12 cm long. It is a capacitance organ. It has both intra and extraperitoneal components. The transition between the sigmoid colon is marked by the disappearance of the tenia coli.The extra peritoneal rectum is surrounded by mesorectal fat that also contains lymph nodes. This mesorectal fatty layer is removed surgically during rectal cancer surgery (Total Mesorectal Excision). The fascial layers that surround the rectum are important clinical landmarks, anteriorly lies the fascia of Denonvilliers. Posteriorly lies Waldeyers fascia.

Extra peritoneal rectum
Posterior upper third
Posterior and lateral middle third
Whole lower third

Relations
Anteriorly (Males) Rectovesical pouch
Bladder
Prostate
Seminal vesicles
Anteriorly (Females) Recto-uterine pouch (Douglas)
Cervix
Vaginal wall
Posteriorly Sacrum
Coccyx
Middle sacral artery
Laterally Levator ani
Coccygeus

Arterial supply
Superior rectal artery
Middle rectal artery (from the internal iliac)
Inferior rectal artery (from the pudendal vessels)

Venous drainage
Superior rectal vein
Inferior rectal vein
Note the venous drainage is a site of portosystemic anastomosis.

Lymphatic drainage
Mesorectal lymph nodes (superior to dentate line)
Inguinal nodes (inferior to dentate line)

1143
Q

A new hernia mesh designed to prevent the risk of infection undergoes clinical trials. 100 patients are given the new mesh. During a three month period 10 of the patients have an episode of infection. In the control group there are 300 patients who are given a placebo. In this group 50 people have an infection during the same time period. What is the relative risk of having an infection when the new mesh is used?

0.8

0.2

1.66

0.6

0.06

A

Experimental event rate, EER = 10 / 100 = 0.10

Control event rate, CER = 50 / 300 = 0.166

Therefore the relative risk = EER / CER = 0.1 / 0.166 = 0.6

Relative risk

Relative risk (RR) is the ratio of risk in the experimental group (experimental event rate, EER) to risk in the control group (control event rate, CER)

To recap
EER = rate at which events occur in the experimental group
CER = rate at which events occur in the control group

For example, if we look at a trial comparing the use of paracetamol for back pain compared to placebo we may get the following results

Total number of patients Experienced significant pain relief
Paracetamol 100 60
Placebo 80 20

Experimental event rate, EER = 60 / 100 = 0.6

Control event rate, CER = 20 / 80 = 0.25

Therefore the relative risk = EER / CER = 0.6 / 0.25 = 2.4

If the risk ratio is > 1 then the rate of an event (in this case experiencing significant pain relief) is increased compared to controls. It is therefore appropriate to calculate the relative risk increase if necessary (see below).

If the risk ratio is < 1 then the rate of an event is decreased compared to controls. The relative risk reduction should therefore be calculated (see below).

Relative risk reduction (RRR) or relative risk increase (RRI) is calculated by dividing the absolute risk change by the control event rate

Using the above data, RRI = (EER - CER) / CER = (0.6 - 0.25) / 0.25 = 1.4 = 140%

1144
Q

Which of the following muscles is penetrated by the parotid duct?

Medial pterygoid

Buccinator

Levator anguli oris

Temporalis

Masseter

A

The duct crosses the masseter muscle and buccal fat pad and then penetrates the buccinator muscle to enter the oral cavity opposite the second upper molar tooth.

Parotid gland

Anatomy of the parotid gland
Location Overlying the mandibular ramus; anterior and inferior to the ear.
Salivary duct Crosses the masseter, pierces the buccinator and drains adjacent to the 2nd upper molar tooth (Stensen’s duct).
Structures passing through the gland
Facial nerve (Mnemonic: The Zebra Buggered My Cat; Temporal Zygomatic, Buccal, Mandibular, Cervical)
External carotid artery
Retromandibular vein
Auriculotemporal nerve
Relations
Anterior: masseter, medial pterygoid, superficial temporal and maxillary artery, facial nerve, stylomandibular ligament
Posterior: posterior belly digastric muscle, sternocleidomastoid, stylohyoid, internal carotid artery, mastoid process, styloid process
Arterial supply Branches of external carotid artery
Venous drainage Retromandibular vein
Lymphatic drainage Deep cervical nodes
Nerve innervation
Parasympathetic-Secretomotor
Sympathetic-Superior cervical ganglion
Sensory- Greater auricular nerve

Parasympathetic stimulation produces a water rich, serous saliva. Sympathetic stimulation leads to the production of a low volume, enzyme-rich saliva.

1145
Q

Which of the following statements relating the fluid physiology of a physiologically normal 70 Kg adult male is false?

He will have more water per unit of body weight than a female of similar weight

Plasma will comprise 25% of his body weight

Interstitial fluid will account for up to 24% of body water

Approximately 65% of total body water is intracellular

60% of his body weight is composed of water

A

The 60-40-20 rule:
60% total body weight is water
40% of total body weight is intracellular fluids
20% of body weight is extracellular fluids

Plasma typically accounts for 4-6% of body weight in healthy individuals.
Males typically have more water per unit weight than females, as females have a higher fat content.

Fluid compartment physiology

Body fluid compartments comprise intracellular and extracellular compartments. The latter includes interstitial fluid, plasma and transcellular fluid.
Typical figures are based on the 70 Kg male.

Body fluid volumes
Compartment Volume in litres Percentage of total volume
Intracellular 28 L 60-65%
Extracellular 14 L 35-40%
Plasma 3 L 5%
Interstitial 10 L 24%
Transcellular 1 L 3%
Figures are approximate

1146
Q

When the brachial plexus is injured in the axilla as a result of a crutch palsy, which of the nerves listed is most commonly affected?

Thoracodorsal nerve

Suprascapular nerve

Radial nerve

Ulnar nerve

Long thoracic nerve

A

The radial nerve is most commonly injured and results in a wrist drop. The ulnar nerve arises from the medial cord and is rarely affected as a result of this injury mechanism.

Brachial plexus

The brachial plexus extends from the neck to the axilla. It is formed by the ventral rami of the fifth to the eighth cervical nerves with the ascending part of the first thoracic nerve.

Location of the plexus
The ventral rami which form the plexus enter the lower part of the posterior triangle of the neck in series with the ventral rami of the cervical plexus. The second part of the subclavian artery lies immediately anterior to the lower two rami. The upper three rami intermingle and pass inferolaterally towards the axilla and subclavian artery. They are enclosed within an extension of the prevertebral fascia. In the neck the plexus lies deep to platysma, the supraclavicular nerves, inferior belly of omohyoid and the transverse cervical artery. It then passes deep to the clavicle and the suprascapular vessels, to enter the axilla, and thence surround the second part of the axillary artery

Composition of the plexus
Ventral rami, the roots of the plexus, lie between scalenus medius and anterior.

As they enter the posterior triangle, the upper two (C5,6) and lower two (C8, T1) roots of the plexus unite to form the upper and lower trunks of the plexus respectively. Meanwhile, C7 continues as the middle trunk. The lower trunk may groove the superior surface of the first rib posterior to the subclavian artery, and the root from the first ventral ramus is always in contact with it.

Each trunk divides into ventral and dorsal divisions which are destined to supply the anterior (flexor) and posterior (extensor) parts of the upper limb.

The cords of the plexus are formed in the axilla. The dorsal divisions unite to form the posterior cord (C5-8). The ventral divisions of the upper and middle trunks unite to form the lateral cord (C5-7), while the ventral divisions of the lower trunk continues as the medial cord (C8-T1). The cords are named according to their relationship to the axillary artery. Each cord terminates by dividing into two main branches at the beginning of the third part of the artery.

Sympathetic communications
The fifth and sixth cervical ventral rami receive grey rami communicantes from the middle cervical ganglion, while the two or more grey rami communicantes pass from the inferior cervical ganglion to the seventh and eighth cervical ventral rami. The first thoracic ventral ramus receives its grey ramus from the cervicothoracic ganglion. Its for this reason that inferior plexus injury can be complicated by a Horners syndrome.

Summary
Origin Anterior rami of C5 to T1
Sections of the plexus
Roots, trunks, divisions, cords, branches
Mnemonic:Real Teenagers Drink Cold Beer
Roots
Located in the posterior triangle
Pass between scalenus anterior and medius
Trunks
Located posterior to middle third of clavicle
Upper and middle trunks related superiorly to the subclavian artery
Lower trunk passes over 1st rib posterior to the subclavian artery
Divisions Apex of axilla
Cords Related to axillary artery

1147
Q

A 58 year old man is reviewed in the clinic following a successful cadaveric renal transplant the previous year. He has been able to return to work as a swimming instructor. Over the past week he reports that he has been suffering from recurrent episodes of diarrhoea. It has made him feel lethargic and exhausted. Stool microscopy shows evidence of cysts. What is the most likely source of infection?

Enterobius vermicularis

Clonorchis sinensis

Giardia

Ascaris lumbricoides

Cryptosporidium

A

Cryptosporidium is associated with infection, particularly in those who are immunocompromised. Diarrhoea is the main disease. The cysts are typically identified on stool microscopy.

Gastro intestinal parasitic infections

Common infections

Enterobiasis
Due to organism Enterobius vermicularis
Common cause of pruritus ani
Diagnosis usually made by placing scotch tape at the anus, this will trap eggs that can then be viewed microscopically
Treatment is with mebendazole
Ancylostoma duodenale
Hookworms that anchor in proximal small bowel
Most infections are asymptomatic although may cause iron deficiency anaemia
Larvae may be found in stools left at ambient temperature, otherwise infection is difficult to diagnose
Infection occurs as a result of cutaneous penetration, migrates to lungs, coughed up and then swallowed
Treatment is with mebendazole
Ascariasis
Due to infection with roundworm Ascaris lumbricoides
Infections begin in gut following ingestion, then penetrate duodenal wall to migrate to lungs, coughed up and swallowed, cycle begins again
Diagnosis is made by identification of worm or eggs within faeces
Treatment is with mebendazole
Strongyloidiasis
Due to infection with Strongyloides stercoralis
Rare in west
Organism is a nematode living in duodenum of host
Initial infection is via skin penetration. They then migrate to lungs and are coughed up and swallowed. Then mature in small bowel are excreted and cycle begins again
An auto infective cycle is also recognised where larvae will penetrate colonic wall
Individuals may be asymptomatic, although they may also have respiratory disease and skin lesions
Diagnosis is usually made by stool microscopy
In the UK mebendazole is used for treatment
Cryptosporidium
Protozoal infection
Organisms produce cysts which are excreted and thereby cause new infections
Symptoms consist of diarrhoea and cramping abdominal pains. Symptoms are worse in immunosuppressed people
Cysts may be identified in stools
Treatment is with metronidazole
Giardiasis
Diarrhoeal infection caused by Giardia lamblia (protozoan)
Infections occur as a result of ingestion of cysts
Symptoms are usually gastrointestinal with abdominal pain, bloating and passage of soft or loose stools
Diagnosis is by serology or stool microscopy
First line treatment is with metronidazole

1148
Q

Which of the following statements relating to use of tourniquets in surgery is false?

The use of an esmarch bandage tourniquet to exsanguinate the limb reduces the incidence of neuropraxia.

Excessive inflation pressures are amongst the commonest causes of nerve injury related to tourniquet use.

Tourniquet deflation causes a fall in CVP.

Children require lower inflation pressures than adults.

In patients developing neuropraxia related to tourniquets the radial nerve is most frequently affected.

A

The use of esmarch bandage tourniquet increases the risk of nerve injury as it increases pressure in the limb. Limb elevation is safer.

Tourniquets

Tourniquets are used during surgery to minimise blood loss and ensure a clear operative field. They must be correctly applied and monitored. They are applied to extremities and in most cases are inflated using a pressure monitoring system.

There are a number of systemic effects that can accompany tourniquet use, these can be divided into those which occur following inflation and those that occur once the tourniquet is deflated.

Post inflation
Increased systemic vascular resistance, increased CVP and increased BP
Slower gradual increase in BP over time
Induced hypercoagulable state
Slow increase in core temperature

Post deflation
Fall in CVP, BP and SVR
Increased end tidal carbon dioxide
Enhanced fibrinolysis
Fall in core temperature
Raised serum potassium and lactate levels

Contra indications
Absolute Relative
AV fistula
Severe peripheral vascular disease
Previous vascular surgery
Bone fracture or thrombosis at the site of tourniquet application Sickle cell disease
History of thromboembolic events
Skin grafts
Localised infection
Lymphoedema

Local complications
Damage to skin
Damage to muscle (rarely compartment syndrome)
Damage to vessels
Neuropraxia

1149
Q

A patient has a chest drain insertion. There is fresh blood at the chest drain insertion area. Which vessel has been damaged?

Pericardiophrenic artery

Intercostal vein

Right ventricle

Vagus artery

Intercostal artery

A

The intercostal vein is more superior than the artery and is thus slightly less susceptible to injury.
Within the intercostal spaces there are thin, strong muscles, intercostal vessels, nerves and lymphatics. There are 3 intercostal muscle layers corresponding to the lateral abdominal wall; external, internal, innermost intercostals. At the mid axillary line there are thin intracostals which is an extension of the internal intercostal muscle. In each intercostal space lies the neurovascular bundle, comprising, from superior to inferiorly; the posterior intercostal vein, artery and nerve, lying protected in the subcostal groove of the rib above and situated between the second and third layer of the intercostal muscles. These blood vessels anastomose anteriorly with the anterior intercostal vessels, which arise from the internal thoracic artery and vein.

Chest drains

There are a number of different indications for chest drain insertion. In general terms large bore chest drains are preferred for trauma and haemothorax drainage. Smaller diameter chest drains can be used for pneumothorax or pleural effusion drainage.

Insertion can be performed either using anatomical guidance or through ultrasound guidance. In the exam, the anatomical method is usually tested.

It is advised that chest drains are placed in the ‘safe triangle’. The triangle is located in the mid axillary line of the 5th intercostal space. It is bordered by:
Anterior edge latissimus dorsi, the lateral border of pectoralis major, a line superior to the horizontal level of the nipple, and the apex below the axilla.

Another triangle is situated behind the scapula. It is bounded above by the trapezius, below by the latissimus dorsi, and laterally by the vertebral border of the scapula; the floor is partly formed by the rhomboid major. If the scapula is drawn forward by folding the arms across the chest, and the trunk bent forward, parts of the sixth and seventh ribs and the interspace between them become subcutaneous and available for auscultation. The space is therefore known as the triangle of auscultation.

References
Prof Harold Ellis. The applied anatomy of chest drains insertions. British Journal of hospital medicine 2007; (68): 44-45.

Laws D, Neville E, Duffy J. BTS guidelines for insertion of chest drains. Thorax, 2003; (58): 53-59.

1150
Q

A 28 year old lady presents with a pigmented lesion on her calf. Excisional biopsy confirms a diagnosis of melanoma measuring 1cm in diameter with a Breslow thickness of 0.1mm. The lesion is less than 1 mm at all resection margins. Which of the following surgical resection margins is acceptable for this lesion?

5 cm

1 cm

0.5 cm

2 cm

3 cm

A

Malignant melanoma

The main diagnostic features (major criteria):
Change in size
Change in shape
Change in colour
Secondary features (minor criteria)
Diameter >6mm
Inflammation
Oozing or bleeding
Altered sensation

Treatment
Suspicious lesions should undergo excision biopsy. The lesion should be removed in completely as incision biopsy can make subsequent histopathological assessment difficult.
Once the diagnosis is confirmed the pathology report should be reviewed to determine whether further re-excision of margins is required (see below):

Margins of excision-Related to Breslow thickness
Lesions 0-1mm thick 1cm
Lesions 1-2mm thick 1- 2cm (Depending upon site and pathological features)
Lesions 2-4mm thick 2-3 cm (Depending upon site and pathological features)
Lesions >4 mm thick 3cm
Marsden J et al Revised UK guidelines for management of Melanoma. Br J Dermatol 2010 163:238-256.

Further treatments such as sentinel lymph node mapping, isolated limb perfusion and block dissection of regional lymph node groups should be selectively applied.

Guidelines
The UK NICE guidance is covered by Melanoma: assessment and management NICE guideline [NG14]. However, the complex oncological regimens are not extensively covered in this guidance though more formal guidance is provided by relevant oncological organisations.

1151
Q

An ENT surgeon is performing a radical neck dissection. She wishes to fully expose the external carotid artery. To do so she inserts a self retaining retractor close to its origin. Which of the following structures lies posterolaterally to the external carotid at this point?

Superior thyroid artery

Internal carotid artery

Lingual artery

Facial artery

None of the above

A

The internal carotid artery lies posterolaterally to the external carotid artery at it’s origin from the common carotid. The superior thyroid, lingual and facial arteries all arise from its anterior surface.

External carotid artery

The external carotid commences immediately lateral to the pharyngeal side wall. It ascends and lies anterior to the internal carotid and posterior to the posterior belly of digastric and stylohyoid. More inferiorly it is covered by sternocleidomastoid, passed by hypoglossal nerves, lingual and facial veins.
It then pierces the fascia of the parotid gland finally dividing into its terminal branches within the gland itself.

Surface marking of the carotid
This is an imaginary line drawn from the bifurcation of the common carotid passing behind the angle of the jaw to a point immediately anterior to the tragus of the ear.

Branches of the external carotid artery
It has six main branches, three in front, two behind and one deep.
Three in front Superior thyroid
Lingual
Facial
Two behind Occipital
Posterior auricular
Deep Ascending pharyngeal

It terminates by dividing into the superficial temporal and maxillary arteries in the parotid gland.

1152
Q

A 50 year old alcoholic man attends the emergency department. His main reason for presenting is that he has no home to go to. On examination, he has no evidence of involvement in recent trauma, a skull x-ray fails to show any evidence of skull fracture. He is admitted and twelve hours following admission he develops sudden onset headache, becomes comatose and then dies. What is the most likely cause?

Acute extra dural haematoma

Chronic sub dural haematoma

Sub arachnoid haemorrhage

Intraventricular haemorrhage

Acute sub dural haematoma

A

The absence of trauma here makes an acute sub dural and extra dural bleed unlikely. Chronic sub dural bleeds would usually cause a more gradual deterioration than is seen here. The absence of any skull fracture also makes an underlying intra cranial bleed less likely. Sudden onset headaches, together with sudden deterioration in neurological function are typical of a sub arachnoid haemorrhage.

Intra cranial haemorrhage

Extradural haematoma Bleeding into the space between the dura mater and the skull. Often results from acceleration-deceleration trauma or a blow to the side of the head. The majority of extradural haematomas occur in the temporal region where skull fractures cause a rupture of the middle meningeal artery.

Features
Raised intracranial pressure
Some patients may exhibit a lucid interval
Subdural haematoma Bleeding into the outermost meningeal layer. Most commonly occur around the frontal and parietal lobes. May be either acute or chronic.

Risk factors include old age and alcoholism.

Slower onset of symptoms than a extradural haematoma.
Intracerebral haematoma Usually hyperdense lesions on CT scanning. Arise in areas of traumatic contusion which fuse to become a haematoma. Areas of clot and fresh blood may co-exist on the same CT scan (Swirl sign). Large haematomas and those associated with mass effect should be evacuated.
Subarachnoid haemorrhage Usually occurs spontaneously in the context of a ruptured cerebral aneurysm but may be seen in association with other injuries when a patient has sustained a traumatic brain injury
Intraventricular haemorrhage Haemorrhage that occurs into the ventricular system of the brain. It is relatively rare in adult surgical practice and when it does occur, it is typically associated with severe head injuries. In premature neonates it may occur spontaneously. The blood may clot and occlude CSF flow, hydrocephalus may result.
In neonatal practice the vast majority of IVH occur in the first 72 hours after birth, the aetiology is not well understood and it is suggested to occur as a result of birth trauma combined with cellular hypoxia, together with the delicate neonatal CNS.

1153
Q

A 40 year old woman presents as an emergency with a painful mass underneath her right mandible. The mass has appeared over the previous week with the pain worsening as the lump has increased in size. On examination, there is a 4cm mass underneath her mandible, there is no associated lymphadenopathy. What is the most likely diagnosis?

Submandibular gland cancer

Submandibular gland calculus

Carotid body tumour

Papilloma of Whartons duct

Carotid body aneurysm

A

The sub mandibular gland is the most common site for salivary calculi. Patients will usually complain of pain, which is worse on eating. When the lesion is located distally the duct may be laid open and the stone excised. Otherwise the gland will require removal. In this case the increase in size will be the result of infection occurring in a blocked duct.

Neck lumps

The table below gives characteristic exam question features for conditions causing neck lumps:

Reactive lymphadenopathy By far the most common cause of neck swellings. There may be a history of local infection or a generalised viral illness
Lymphoma Rubbery, painless lymphadenopathy
The phenomenon of pain whilst drinking alcohol is very uncommon
There may be associated night sweats and splenomegaly
Thyroid swelling May be hypo-, eu- or hyperthyroid symptomatically
Moves upwards on swallowing
Thyroglossal cyst More common in patients < 20 years old
Usually midline, between the isthmus of the thyroid and the hyoid bone
Moves upwards with protrusion of the tongue
May be painful if infected
Pharyngeal pouch More common in older men
Represents a posteromedial herniation between thyropharyngeus and cricopharyngeus muscles
Usually not seen, but if large then a midline lump in the neck that gurgles on palpation
Typical symptoms are dysphagia, regurgitation, aspiration and chronic cough
Cystic hygroma A congenital lymphatic lesion (lymphangioma) typically found in the neck, classically on the left side
Most are evident at birth, around 90% present before 2 years of age
Branchial cyst An oval, mobile cystic mass that develops between the sternocleidomastoid muscle and the pharynx
Develop due to failure of obliteration of the second branchial cleft in embryonic development
Usually present in early adulthood
Cervical rib More common in adult females
Around 10% develop thoracic outlet syndrome
Carotid aneurysm Pulsatile lateral neck mass which doesn’t move on swallowing

1154
Q

A 44 year old man is involved in a road traffic accident. He suffers significant injuries to his thorax, he has bilateral haemopneumothoraces and a suspected haemopericardium. He is to undergo surgery, what is the best method of accessing these injuries?

Bilateral thoracoscopy and mediastinoscopy

Midline sternotomy

Bilateral posterolateral thoracotomy

Clam shell thoracotomy

None of the above

A

Patients with significant mediastinal and lung injuries are best operated on using a Clam shell thoracotomy. All modes of access involve a degree of compromise. A sternotomy would give good access to the heart. However, it takes longer to perform and does not provide good access to the lungs. Trauma should not be managed using laparoscopy.

Trauma management

The cornerstone of trauma management is embodied in the principles of ATLS.

Following trauma there is a trimodal death distribution:
Immediately following injury. Typically as result of brain or high spinal injuries, cardiac or great vessel damage. Salvage rate is low.
In early hours following injury. In this group deaths are due to phenomena such as splenic rupture, sub dural haematomas and haemopneumothoraces
In the days following injury. Usually due to sepsis or multi organ failure.

Aspects of trauma management
ABCDE approach.
Tension pneumothoraces will deteriorate with vigorous ventilation attempts.
External haemorrhage is managed as part of the primary survey. As a rule tourniquets should not be used. Blind application of clamps will tend to damage surrounding structures and packing is the preferred method of haemorrhage control.
Urinary catheters and naso gastric tubes may need inserting. Be wary of basal skull fractures and urethral injuries.
Patients with head and neck trauma should be assumed to have a cervical spine injury until proven otherwise.

Thoracic injuries
Simple pneumothorax
Mediastinal traversing wounds
Tracheobronchial tree injury
Haemothorax
Blunt cardiac injury
Diaphragmatic injury
Aortic disruption
Pulmonary contusion

Management of thoracic trauma
Simple pneumothorax insert chest drain. Aspiration is risky in trauma as pneumothorax may be from lung laceration and convert to tension pneumothorax.
Mediastinal traversing wounds These result from situations like stabbings. Exit and entry wounds in separate hemithoraces. The presence of a mediastinal haematoma indicates the likelihood of a great vessel injury. All patients should undergo CT angiogram and oesophageal contrast swallow. Indications for thoracotomy are largely related to blood loss and will be addressed below.
Tracheobronchial tree injury Unusual injuries. In blunt trauma most injuries occur within 4cm of the carina. Features suggesting this injury include haemoptysis and surgical emphysema. These injuries have a very large air leak and may have tension pneumothorax.
Haemothorax Usually caused by laceration of lung vessel or internal mammary artery by rib fracture. Patients should all have a wide bore 36F chest drain. Indications for thoracotomy include loss of more than 1.5L blood initially or ongoing losses of >200ml per hour for >2 hours.
Cardiac contusions Usually cardiac arrhythmias, often overlying sternal fracture. Perform echocardiography to exclude pericardial effusions and tamponade. Risk of arrhythmias falls after 24 hours.
Diaphragmatic injury Usually left sided. Direct surgical repair is performed.
Traumatic aortic disruption Commonest cause of death after RTA or falls. Usually incomplete laceration near ligamentum arteriosum. All survivors will have contained haematoma. Only 1-2% of patients with this injury will have a normal chest x-ray.
Pulmonary contusion Common and lethal. Insidious onset. Early intubation and ventilation.

Abdominal trauma
Deceleration injuries are common.
In blunt trauma requiring laparotomy the spleen is most commonly injured (40%)
Stab wounds traverse structures most commonly liver (40%)
Gunshot wounds have variable effects depending upon bullet type. Small bowel is most commonly injured (50%)
Patients with stab wounds and no peritoneal signs up to 25% will not enter the peritoneal cavity
Blood at urethral meatus suggests a urethral tear
High riding prostate on PR = urethral disruption
Mechanical testing for pelvic stability should only be performed once

Investigations in abdominal trauma

Diagnostic Peritoneal Lavage Abdominal CT scan USS
Indication Document bleeding if hypotensive Document organ injury if normotensive Document fluid if hypotensive
Advantages Early diagnosis and sensitive; 98% accurate Most specific for localising injury; 92 to 98% accurate Early diagnosis, non invasive and repeatable; 86 to 95% accurate
Disadvantages Invasive and may miss retroperitoneal and diaphragmatic injury Location of scanner away from facilities, time taken for reporting, need for contrast Operator dependent and may miss retroperitoneal injury
Amylase may be normal following pancreatic trauma
Urethrography if suspected urethral injury

1155
Q

Which of the skin lesions listed below are most likely to resemble a malignant melanoma?

Spitz naevus

Dermatofibroma

Squamous cell carcinoma

Bowens disease

Seborrhoeic wart

A

Spitz naevi can often resemble melanoma and are best excised.

Benign skin diseases

Seborrhoeic keratosis
Most commonly arise in patients over the age of 50 years, often idiopathic
Equal sex incidence and prevalence
Usually multiple lesions over face and trunk
Flat, raised, filiform and pedunculated subtypes are recognised
Variable colours and surface may have greasy scale overlying it
Treatment options consist of leaving alone or simple shave excision

Melanocytic naevi
Congenital melanocytic naevi
Typically appear at, or soon after, birth
Usually greater than 1cm diameter
Increased risk of malignant transformation (increased risk greatest for large lesions)
Junctional melanocytic naevi
Circular macules
May have heterogeneous colour even within same lesion
Most naevi of the palms, soles and mucous membranes are of this type
Compound naevi
Domed pigmented nodules up to 1cm in diameter
Arise from junctional naevi, usually have uniform colour and are smooth
Spitz naevus
Usually develop over a few months in children
May be pink or red in colour, most common on face and legs
May grow up to 1cm and growth can be rapid, this usually results in excision
Atypical naevus syndrome
Atypical melanocytic naevi that may be autosomally dominantly inherited
Some individuals are at increased risk of melanoma (usually have mutations of CDKN2A gene)
Many people with atypical naevus syndrome AND a parent sibling with melanoma will develop melanoma

Epidermoid cysts
Common and affect face and trunk
They have a central punctum, they may contain small quantities of sebum
The cyst lining is either normal epidermis (epidermoid cyst) or outer root sheath of hair follicle (pilar cyst)

Dermatofibroma
Solitary dermal nodules
Usually affect extremities of young adults
Lesions feel larger than they appear visually
Histologically they consist of proliferating fibroblasts merging with sparsely cellular dermal tissues

Painful skin lesions
Eccrine spiradenoma
Neuroma
Glomus tumour
Leiomyoma
Angiolipoma
Neurofibroma (rarely painful) and dermatofibroma (rarely painful)

1156
Q

A 43 year old male with long standing chronic hepatitis is being followed up. Recently his AFP is noted to be increased and an abdominal USS demonstrates a 2cm lesion in segment V of the liver. What is the most appropriate course of action?

PET CT scan

Liver MRI

USS guided liver biopsy

Laparoscopic biopsy

Segmental resection of segment V

A

Liver lesions that are suspicious of HCC should be scanned prior to resection as there is a risk of multifocal lesions that would either preclude or otherwise affect the decision to proceed with segmental resection.

Hepatocellular carcinoma

Hepatocellular carcinoma is the second leading cause of cancer deaths globally. Up to 750,000 cases are reported annually. Unfortunately the incidence approximates to the death rate so there are few long term survivors[1]. The disease occurs most commonly in those with chronic hepatitis and established liver cirrhosis. Therefore, these individuals should be closely screened for the development of HCC with serum AFP and liver USS every 6-12 months. Rising AFP and liver USS showing a nodule greater than 1cm in diameter makes HCC much more likely and such patients should then undergo MRI scanning.
The presence of adenomas in an otherwise healthy liver is a recognised risk factor for HCC [2, 3] and many surgeons will remove liver adenomas for this reason[4].

Diagnosis
The aim is to avoid unnecessary percutaneous biopsy. Radiologically on CT the classical feature is a suspicious lesion which is highlighted during the arterial phase with washout during the venous phase, this reflects the hypervascularity of the lesions.The risk of tumour seeding as a result of a liver biopsy is 2.7% with a median time interval between biopsy and seeding of 17 months[5].

Barcelona Clinic Liver Classification
There are many classification systems for addressing the management and prognosis, the BCLC system has the convenience of categorising disease extent with treatment and prognostic outcomes. In determining the ideal treatment modality for HCC the key points are not just disease extent, but also the functional state of the liver and patient.

Stage Features Treatment Prognosis-5 yr survival
Stage 0 Child-Pugh A
Single lesion (less than 2cm)
Normal portal pressures Resection 40-70%
Stage A Single nodule greater than 3cm or multiple nodules (no more than 3)
Child Pugh A/ B If associated disease then radiofrequency ablation
If no associated disease then transplantation May be up to 70% in some
Stage B Multiple nodules
Child Pugh A/B Trans arterial chemo-embolisation (usually with doxorubicin) 26% at 3 years
Stage C Advanced tumours
Invasion of portal vein
Child Pugh A/B Sorafenib Usually survive 10.7 months
Stage D Child Pugh stage C
Advanced tumours Best supportive care Less than 6 months survival
[6]
In selected patients the best outcomes are achieved with surgical resection, or transplantation where surgical resection is precluded. Anatomical resections with minimum 2cm margins provide the best outcomes.
At the present time there is no evidence to recommend treatment with adjuvant chemotherapy[6].

Sorafenib
This is an oral multi tyrosine kinase inhibitor. It is the only drug that has been currently demonstrated to extend survival in individuals with advanced hepatocellular cancer[7]. The improvement in survival is from a median of 7 months to 10 months.

References
1. Jemal, A., et al., Global cancer statistics. CA Cancer J Clin, 2011. 61(2): p. 69-90.
2. Leese, T., O. Farges, and H. Bismuth, Liver cell adenomas. A 12-year surgical experience from a specialist hepato-biliary unit. Ann Surg, 1988. 208(5): p. 558-64.
3. Farges, O. and S. Dokmak, Malignant transformation of liver adenoma: an analysis of the literature. Dig Surg, 2010. 27(1): p. 32-8.
4. Ehrl, D., et al., ‘Incidentaloma’ of the liver: management of a diagnostic and therapeutic dilemma. HPB Surg, 2012. 2012: p. 891787.
5. Silva, M.A., et al., Needle track seeding following biopsy of liver lesions in the diagnosis of hepatocellular cancer: a systematic review and meta-analysis. Gut, 2008. 57(11): p. 1592-6.
6. EASL-EORTC clinical practice guidelines: management of hepatocellular carcinoma. J Hepatol, 2012. 56(4): p. 908-43.
7. Abou-Alfa, G.K., et al., Phase II study of sorafenib in patients with advanced hepatocellular carcinoma. J Clin Oncol, 2006. 24(26): p. 4293-300.

1157
Q

At which of the following levels does the inferior thyroid artery enter the thyroid gland?

C6

C2

C4

C3

C5

A

It enters the gland at C6.

Surface anatomy of the neck

In the midline from above down, the following structures are felt
Structure Level
Hyoid C3
Notch of the thyroid cartilage C4
Cricoid cartilage(termination) C6
The lower border of the cricoid cartilage corresponds to the commencement of the trachea and also to the following:

Junction of larynx with trachea
Junction of pharynx with oesophagus
Level at which the inferior thyroid artery enters the thyroid gland
The level at which the vertebral artery enters the transverse foramen in the 6th cervical vertebra
Level at which the superior belly of omohyoid crosses the carotid sheath
The level of the middle cervical sympathetic ganglion
The level at which the carotid artery can be compressed against the transverse process of C6 (carotid tubercle).

1158
Q

A 78 year old man is walking to the bus stop when he suddenly develops severe back pain and collapses. On examination he has a blood pressure of 90/40 and pulse rate of 110. His abdomen is distended and he is obese. Though tender his abdomen itself is soft. What is the most likely diagnosis?

Ruptured abdominal aortic aneurysm

Perforated peptic ulcer

Appendicitis

Mesenteric infarction

Perforated diverticulitis

A

This will be a retroperitoneal rupture (anterior ones generally don’t survive to hospital). The debate regarding CT varies, it is the authors opinion that a systolic BP of <100mmHg at presentation mandates immediate laparotomy.

Acute abdominal pain-diagnoses

Conditions presenting with acute abdominal pain
Condition Features Investigations Management
Appendicitis History of migratory pain.
Fever.
Anorexia.
Evidence of right iliac fossa tenderness.
Mild pyrexia. Differential white cell count
Pregnancy test
C-Reactive protein
Amylase
Urine dipstick testing Appendicectomy
Mesenteric adenitis Usually recent upper respiratory tract infection.
High fever.
Generalised abdominal discomfort- true localised pain and signs are rare. Full blood count- may show slightly raised white cell count
Urine dipstick often normal
Abdominal ultrasound scan - usually no free fluid Conservative management- appendicectomy if diagnostic doubt
Mittelschmerz Only seen in females
Mid cycle pain
Usually occurs two weeks after last menstrual period
Pain usually has a supra-pubic location
Usually subsides over a 24-48 hour period. Full blood count- normal
Urine dipstick- normal
Abdominal and pelvic ultrasound- may show a trace of pelvic free fluid Manage conservatively if doubt or symptoms fail to settle then laparoscopy
Fitz-Hugh Curtis syndrome Disseminated infection with Chlamydia.
Usually seen in females.
Consists of evidence of pelvic inflammatory disease together with peri-hepatic inflammation and subsequent adhesion formation. Abdominal ultrasound scan- may show free fluid
High vaginal swabs - may show evidence of sexually transmitted infections Usually medically managed- doxycycline or azithromycin
Abdominal aortic aneurysm (ruptured) Sudden onset of abdominal pain radiating to the back in older adults (look for risk factors).
Collapse.
May be moribund on arrival in casualty, more stable if contained haematoma.
Careful clinical assessment may reveal pulsatile mass. Patients who are haemodynamically stable should have a CT scan Unstable patients should undergo immediate surgery (unless it is not in their best interests).
Those with evidence of contained leak on CT should undergo immediate surgery
Increasing unruptured aneurysmal size is an indication for urgent surgical intervention (that can wait until the next working day)
Perforated peptic ulcer Sudden onset of pain (usually epigastric).
Often preceding history of upper abdominal pain.
Soon develop generalised abdominal pain.
On examination may have clinical evidence of peritonitis. Erect CXR may show free air. A CT scan may be indicated where there is diagnostic doubt Laparotomy (laparoscopic surgery for perforated peptic ulcers is both safe and feasible in experienced hands)
Intestinal obstruction Colicky abdominal pain and vomiting (the nature of which depends on the level of the obstruction).
Abdominal distension and constipation (again depending upon site of obstruction).
Features of peritonism may occur where local necrosis of bowel loops is occurring. A plain abdominal film may help with making the diagnosis. A CT scan may be useful where diagnostic uncertainty exists In those with a virgin abdomen a lower and earlier threshold for laparotomy should exist than in those who may have adhesional obstruction
Mesenteric infarction Embolic events present with sudden pain and forceful evacuation.
Acute on chronic events usually have a longer history and previous weight loss.
On examination the pain is typically greater than the physical signs would suggest. Arterial pH and lactate
Arterial phase CT scanning is the most sensitive test Immediate laparotomy and resection of affected segments, in acute embolic events SMA embolectomy may be needed.

1159
Q

A premature neonate is born by emergency cesarean section at 29 weeks gestation. He initially seems to be progressing well. However, the team are concerned because he has become septic and on examination has a swollen and erythematous umbilicus. What is the most likely diagnosis?

Patent urachus

Patent vitello–intestinal duct

Omphalitis

Umbilical granuloma

Pyogenic granuloma

A

Infection from omphalitis may spread rapidly and cause severe sepsis especially in immunologically compromised, premature neonates.

Paediatric umbilical disorders

Embryology
During development the umbilicus has two umbilical arteries and one umbilical vein. The arteries are continuous with the internal iliac arteries and the vein is continuous with the falciform ligament (ductus venosus). After birth the cord dessicates and separates and the umbilical ring closes.

Umbilical hernia
Up to 20% of neonates may have an umbilical hernia, it is more common in premature infants. The majority of these hernias will close spontaneously (may take between 12 months and three years). Strangulation is rare.

Paraumbilical hernia
These are due to defects in the linea alba that are in close proximity to the umbilicus. The edges of a paraumbilical hernia are more clearly defined than those of an umbilical hernia. They are less likely to resolve spontaneously than an umbilical hernia.

Omphalitis
This condition consists of infection of the umbilicus. Infection with Staphylococcus aureus is the commonest cause. The condition is potentially serious as infection may spread rapidly through the umbilical vessels in neonates with a risk of portal pyaemia, and portal vein thrombosis. Treatment is usually with a combination of topical and systemic antibiotics.

Umbilical granuloma
These consist of cherry red lesions surrounding the umbilicus, they may bleed on contact and be a site of seropurulent discharge. Infection is unusual and they will often respond favorably to chemical cautery with topically applied silver nitrate.

Persistent urachus
This is characterised by urinary discharge from the umbilicus. It is caused by persistence of the urachus which attaches to the bladder. They are associated with other urogenital abnormalities.

Persistent vitello-intestinal duct
This will typically present as an umbilical discharge that discharges small bowel content. Complete persistence of the duct is a rare condition. Much more common is the persistence of part of the duct (Meckels diverticulum). Persistent vitello-intestinal ducts are best imaged using a contrast study to delineate the anatomy and are managed by laparotomy and surgical closure.

1160
Q

Which of the following statements relating to a burst abdomen is false?

Is seen in 1-2% of modern laparotomies

Is more common in faecal peritonitis

Is less common when a ‘mass closure’ technique is used

When it does occur is most common at 15 days

Is similar in incidence regardless of whether 1/0 polydiaxone or 1/0 polypropylene are used

A

When it does occur, a burst abdomen is most common at 6 days and is usually the result of technical error when Jenkins rule is not followed and sutures are placed in the zone of collagenolysis. The choice of materials given above does not influence dehisence rates.

Abdominal wound dehiscence

  • This is a significant problem facing all surgeons who undertake abdominal surgery on a regular basis. Traditionally, it is said to occur when all layers of an abdominal mass closure fail and the viscera protrude externally (associated with 30% mortality).
    It can be subdivided into superficial, in which the skin wound alone fails and complete, implying failure of all layers.

Factors which increase the risk are:
* Malnutrition
* Vitamin deficiencies
* Jaundice
* Steroid use
* Major wound contamination (e.g. faecal peritonitis)
* Poor surgical technique (Mass closure technique is the preferred method-Jenkins Rule)

When sudden full dehiscence occurs the management is as follows:
* Analgesia
* Intravenous fluids
* Intravenous broad spectrum antibiotics
* Coverage of the wound with saline impregnated gauze (on the ward)
* Arrangements made for a return to theatre

Surgical strategy
Correct the underlying cause (e.g. TPN or NG feed if malnourished)
Determine the most appropriate strategy for managing the wound

Options
Resuturing of the wound This may be an option if the wound edges are healthy and there is enough tissue for sufficient coverage. Deep tension sutures are traditionally used for this purpose.
Application of a wound manager This is a clear dressing with removable front. Particularly suitable when some granulation tissue is present over the viscera or where there is a high output bowel fistula present in the dehisced wound.
Application of a ‘Bogota bag’ This is a clear plastic bag that is cut and sutured to the wound edges and is only a temporary measure to be adopted when the wound cannot be closed and will necessitate a return to theatre for definitive management.
Application of a VAC dressing system These can be safely used BUT ONLY if the correct layer is interposed between the suction device and the bowel. Failure to adhere to this absolute rule will almost invariably result in the development of multiple bowel fistulae and create an extremely difficult management problem.

1161
Q

A 35 year old man presents to the surgical clinic with a suspected direct inguinal hernia. These will pass through Hesselbach’s triangle. Which of the following forms the medial edge of this structure?

External oblique aponeurosis

Inferior epigastric artery

Rectus abdominis muscle

Inferior epigastric vein

Obturator nerve

A

Direct inguinal hernias pass through Hesselbachs triangle (although this is of minimal clinical significance!). Its medial boundary is the rectus muscle.

Hesselbach’s triangle

Direct hernias pass through Hesselbachs triangle.

Superolaterally Epigastric vessels
Medially Lateral edge of rectus muscle
Inferiorly Inguinal ligament

1162
Q

A 25 year old man undergoes an excision of a pelvic chondrosarcoma, during the operation the obturator nerve is sacrificed. Which of the following muscles is least likely to be affected as a result?

Adductor longus

Pectineus

Adductor magnus

Sartorius

Gracilis

A

Sartorius is supplied by the femoral nerve. In approximately 20% of the population, pectineus is supplied by the accessory obturator nerve. Adductor magnus has a composite supply from the obturator and sciatic nerves.
Obturator nerve

The obturator nerve arises from L2, L3 and L4 by branches from the ventral divisions of each of these nerve roots. L3 forms the main contribution and the second lumbar branch is occasionally absent. These branches unite in the substance of psoas major, descending vertically in its posterior part to emerge from its medial border at the lateral margin of the sacrum. It then crosses the sacroiliac joint to enter the lesser pelvis, it descends on obturator internus to enter the obturator groove. In the lesser pelvis the nerve lies lateral to the internal iliac vessels and ureter, and is joined by the obturator vessels lateral to the ovary or ductus deferens.

Supplies
Medial compartment of thigh
Muscles supplied: external obturator, adductor longus, adductor brevis, adductor magnus (not the lower part-sciatic nerve), gracilis
The cutaneous branch is often absent. When present, it passes between gracilis and adductor longus near the middle part of the thigh, and supplies the skin and fascia of the distal two thirds of the medial aspect.

Obturator canal
Connects the pelvis and thigh: contains the obturator artery, vein, nerve which divides into anterior and posterior branches.

1163
Q

A 67 year old man is due to undergo a revisional total hip replacement using a posterior approach. After dividing gluteus maximus in the line of its fibres there is brisk arterial bleeding. Which of the following vessels is likely to be responsible?

Profunda femoris artery

External iliac artery

Internal iliac artery

Obturator artery

Inferior gluteal artery

A

The inferior gluteal artery runs on the deep surface of the gluteus maximus muscle. It is a branch of the internal iliac artery. It is commonly divided during the posterior approach to the hip joint.

Hip joint

Head of femur articulates with acetabulum of the pelvis
Both covered by articular hyaline cartilage
The acetabulum forms at the union of the ilium, pubis, and ischium
The triradiate cartilage (Y-shaped growth plate) separates the pelvic bones
The acetabulum holds the femoral head by the acetabular labrum
Normal angle between femoral head and femoral shaft is 130o

Ligaments
Transverse ligament: joints anterior and posterior ends of the articular cartilage
Head of femur ligament (ligamentum teres): acetabular notch to the fovea. Contains arterial supply to head of femur in children.

Hip joint

Head of femur articulates with acetabulum of the pelvis
Both covered by articular hyaline cartilage
The acetabulum forms at the union of the ilium, pubis, and ischium
The triradiate cartilage (Y-shaped growth plate) separates the pelvic bones
The acetabulum holds the femoral head by the acetabular labrum
Normal angle between femoral head and femoral shaft is 130o

Ligaments
Transverse ligament: joints anterior and posterior ends of the articular cartilage
Head of femur ligament (ligamentum teres): acetabular notch to the fovea. Contains arterial supply to head of femur in children.

Extracapsular ligaments
Iliofemoral ligament: inverted Y shape. Anterior iliac spine to the trochanteric line
Pubofemoral ligament: acetabulum to lesser trochanter
Ischiofemoral ligament: posterior support. Ischium to greater trochanter.

Blood supply
Medial circumflex femoral and lateral circumflex femoral arteries (Branches of profunda femoris). Also from the inferior gluteal artery. These form an anastomosis and travel to up the femoral neck to supply the head.

1164
Q

A 22 year old man is shot in the groin. On examination, he has weak hip flexion, weak knee extension, and impaired quadriceps tendon reflex, as well as sensory deficit in the anteromedial aspect of the thigh. Which structure has been affected?

Femoral nerve

Sciatic nerve

Superior gluteal nerve

Ilioinguinal nerve

Genitofemoral nerve

A

This is a classical description of a femoral nerve injury.
Femoral nerve

Root values L2, 3, 4
Innervates
Pectineus
Sartorius
Quadriceps femoris
Vastus lateralis/medialis/intermedius
Rectus femoris
Branches
Medial cutaneous nerve of thigh
Saphenous nerve
Intermediate cutaneous nerve of thigh

Path
Penetrates psoas major and exits the pelvis by passing under the inguinal ligament to enter the femoral triangle, lateral to the femoral artery and vein.

Mnemonic for femoral nerve supply

(don’t) M I S V Q Scan for PE
M edial cutaneous nerve of the thigh
I ntermediate cutaneous nerve of the thigh
S aphenous nerve

V astus
Q uadriceps femoris
S artorius

PE ectineus

1165
Q

A 34 year old lady is admitted with pancreatitis. The aetiology is unclear and it is classified as an attack of moderate severity according to the Glasgow criteria. Her imaging shows no gallstones and fluid around the pancreas. Which of the following is the most appropriate initial management option?

Laparotomy

Laparoscopy

Radiological aspiration of the fluid

Active observation

Administration of octreotide

A

LEARN THIS!

Mnemonic for the assessment of the severity of pancreatitis: PANCREAS
(Ann R Coll Surg Engl 2000; 82: 16-17

P a02 < 60 mmHg
A ge > 55 years
N eutrophils > 15 x 10/l
C alcium < 2 mmol/l
R aised urea > 16 mmol/l
E nzyme (lactate dehydrogenase) > 600 units/l
A lbumin < 32 g/l
S ugar (glucose) > 10 mmol/l

> 3 positive criteria indicates severe pancreatitis.
Acute early fluid collections are seen in 25% of patients with pancreatitis and require no specific treatment. Attempts at drainage may introduce infection and result in pancreatic abscess formation.

Management of Pancreatitis

Management of Acute Pancreatitis in the UK

Diagnosis
Traditionally hyperamylasaemia has been utilised with amylase being elevated three times the normal range.
However, amylase may give both false positive and negative results.
Serum lipase is both more sensitive and specific than serum amylase. It also has a longer half life.
Serum amylase levels do not correlate with disease severity.

Differential causes of hyperamylasaemia
Acute pancreatitis
Pancreatic pseudocyst
Mesenteric infarct
Perforated viscus
Acute cholecystitis
Diabetic ketoacidosis

Assessment of severity
Glasgow, Ranson scoring systems and APACHE II
Biochemical scoring e.g. using CRP

Features that may predict a severe attack within 48 hours of admission to hospital
Initial assessment
Clinical impression of severity
Body mass index >30
Pleural effusion
APACHE score >8
24 hours after admission
Clinical impression of severity
APACHE II >8
Glasgow score of 3 or more
Persisting multiple organ failure
CRP>150
48 hours after admission
Glasgow Score of >3
CRP >150
Persisting or progressive organ failure
Table adapted from UK guidelines for management of acute pancreatitis. GUT 2005, 54 suppl III

Management

Nutrition
There is reasonable evidence to suggest that the use of enteral nutrition does not worsen the outcome in pancreatitis
Most trials to date were underpowered to demonstrate a conclusive benefit.
The rationale behind feeding is that it helps to prevent bacterial translocation from the gut, thereby contributing to the development of infected pancreatic necrosis.

Use of antibiotic therapy
Many UK surgeons administer antibiotics to patients with acute pancreatitis. However, there is very little evidence to support this practice.
A recent Cochrane review highlights the potential benefits of administering Imipenem to patients with established pancreatic necrosis in the hope of averting the progression to infection.
There are concerns that the administration of antibiotics in mild attacks of pancreatitis will not affect outcome and may contribute to antibiotic resistance and increase the risks of antibiotic associated diarrhoea.

Surgery
Patients with acute pancreatitis due to gallstones should undergo early cholecystectomy.
Patients with obstructed biliary system due to stones should undergo early ERCP.
Patients with extensive necrosis where infection is suspected should usually undergo FNA for culture.
Patients with infected necrosis should undergo either radiological drainage or surgical necrosectomy. The choice of procedure depends upon local expertise.

References
www.bsg.org.uk/pdfworddocs/pancreatic.pdf

Antibiotic therapy for prophylaxis against infection of pancreatic necrosis in acute pancreatitis. Villatoro et al. Cochrane Library DOI: 10.1002/14651858.CD002941.pub3. 2010 version.

1166
Q

A 52 year old lady develops lower leg swelling following redo varicose vein surgery. There is evidence of swelling of the left leg up to the knee. The overlying skin appears healthy. What is the best management option?

Homans operation

Multilayer compression bandages

High dose frusemide

High dose bumetanide

Lymphovenous anastomosis

A

Unfortunately, lymphoedema may complicate redo varicose vein surgery (in 0.5% of cases). As the presentation is mild, she should be managed using compression hosiery. Diuretics do not help in cases of true lymphoedema and a dramatic response suggests an alternative underlying cause.
Lymphoedema

  • Due to impaired lymphatic drainage in the presence of normal capillary function.
    Lymphoedema causes the accumulation of protein rich fluid, subdermal fibrosis and dermal thickening.
    Characteristically fluid is confined to the epifascial space (skin and subcutaneous tissues); muscle compartments are free of oedema. It involves the foot, unlike other forms of oedema. There may be a ‘buffalo hump’ on the dorsum of the foot and the skin cannot be pinched due to subcutaneous fibrosis.

Causes of lymphoedema

Primary
Congenital < 1 year: sporadic, Milroy’s disease
Onset 1-35 years: sporadic, Meige’s disease
> 35 years: Tarda
Secondary
Bacterial/fungal/parasitic infection (filariasis)
Lymphatic malignancy
Radiotherapy to lymph nodes
Surgical resection of lymph nodes
DVT
Thrombophlebitis

Indications for surgery
Marked disability or deformity from limb swelling
Lymphoedema caused by proximal lymphatic obstruction with patent distal lymphatics suitable for a lymphatic drainage procedure
Lymphocutaneous fistulae and megalymphatics

Procedures
Homans operation Reduction procedure with preservation of overlying skin (which must be in good condition). Skin flaps are raised and the underlying tissue excised. Limb circumference typically reduced by a third.
Charles operation All skin and subcutaneous tissue around the calf are excised down to the deep fascia. Split skin grafts are placed over the site. May be performed if overlying skin is not in good condition. Larger reduction in size than with Homans procedure.
Lymphovenous anastamosis Identifiable lymphatics are anastomosed to sub dermal venules. Usually indicated in 2% of patients with proximal lymphatic obstruction and normal distal lymphatics.

1167
Q

A 30 year old male presents with gynaecomastia. Clinically, he is noted to have a nodule in the left testis. What is the most likely diagnosis?

Oestrogen abuse

Seminoma with syncytiotrophoblast giant cells

Teratoma

Sarcoma

Leydig cell tumour

A

Leydig cell tumours are rare testicular sex cord stromal tumours (which also include sertoli cell tumours) which are associated with hormonal activity.
Patients with Leydig cell tumours may present with gynaecomastia before they notice testicular enlargement.
Majority are benign
Histology: eosinophilic cells in columns
Testicular disorders

Testicular cancer
Testicular cancer is the most common malignancy in men aged 20-30 years. Around 95% of cases of testicular cancer are germ-cell tumours. Germ cell tumours may essentially be divided into:

Tumour type Key features Tumour markers Pathology
Seminoma
Commonest subtype (50%)
Average age at diagnosis = 40
Even advanced disease associated with 5 year survival of 73%
AFP usually normal
HCG elevated in 10% seminomas
Lactate dehydrogenase; elevated in 10-20% seminomas (but also in many other conditions)
Sheet like lobular patterns of cells with substantial fibrous component. Fibrous septa contain lymphocytic inclusions and granulomas may be seen.
Non seminomatous germ cell tumours (42%)
Teratoma
Yolk sac tumour
Choriocarcinoma
Mixed germ cell tumours (10%)
Younger age at presentation =20-30 years
Advanced disease carries worse prognosis (48% at 5 years)
Retroperitoneal lymph node dissection may be needed for residual disease after chemotherapy
AFP elevated in up to 70% of cases
HCG elevated in up to 40% of cases
Other markers rarely helpful
Heterogenous texture with occasional ectopic tissue such as hair

Risk factors for testicular cancer
Cryptorchidism
Infertility
Family history
Klinefelter’s syndrome
Mumps orchitis

Features
A painless lump is the most common presenting symptom
Pain may also be present in a minority of men
Other possible features include hydrocele, gynaecomastia

Diagnosis
Ultrasound is first-line
CT scanning of the chest/ abdomen and pelvis is used for staging
Tumour markers (see above) should be measured

Management
Orchidectomy (Inguinal approach)
Chemotherapy and radiotherapy may be given depending on staging
Abdominal lesions >1cm following chemotherapy may require retroperitoneal lymph node dissection.

Prognosis is generally excellent
5 year survival for seminomas is around 95% if Stage I
5 year survival for teratomas is around 85% if Stage I

Benign disease

Epididymo-orchitis
Acute epididymitis is an acute inflammation of the epididymis, often involving the testis and usually caused by bacterial infection.
Infection spreads from the urethra or bladder. In men <35 years, gonorrhoea or chlamydia are the usual infections.
Amiodarone is a recognised non infective cause of epididymitis, which resolves on stopping the drug.
Tenderness is usually confined to the epididymis, which may facilitate differentiating it from torsion where pain usually affects the entire testis.

Testicular torsion
Twist of the spermatic cord resulting in testicular ischaemia and necrosis.
Most common in males aged between 10 and 30 (peak incidence 13-15 years)
Pain is usually severe and of sudden onset.
Cremasteric reflex is lost and elevation of the testis does not ease the pain.
Treatment is with surgical exploration. If a torted testis is identified then both testis should be fixed as the condition of bell clapper testis is often bilateral.

Hydrocele
Presents as a mass that transilluminates, usually possible to ‘get above’ it on examination.
In younger men it should be investigated with USS to exclude tumour.
In children it may occur as a result of a patent processus vaginalis.
Treatment in adults is with a Lords or Jabouley procedure.
Treatment in children is with trans inguinal ligation of PPV.

1168
Q

From the list below, which drug is known to cause haemorrhagic cystitis?

Rifampicin

Methotrexate

Dexamethasone

Leflunomide

Cyclophosphamide

A

Cyclophosphamide is metabolised into a toxic metabolite acrolein. The effects may be attenuated by administration of large volumes of intravenous fluids and mesna (which neutralises the metabolite). The condition may be managed initially by bladder catheterisation and irrigation.

Haematuria

Causes of haematuria

Trauma
Injury to renal tract
Renal trauma commonly due to blunt injury (others penetrating injuries)
Ureter trauma rare: iatrogenic
Bladder trauma: due to RTA or pelvic fractures
Infection
Remember TB
Malignancy
Renal cell carcinoma (remember paraneoplastic syndromes): painful or painless
Urothelial malignancies: 90% are transitional cell carcinoma, can occur anywhere along the urinary tract. Painless haematuria.
Squamous cell carcinoma and adenocarcinoma: rare bladder tumours
Prostate cancer
Penile cancers: SCC
Renal disease
Glomerulonephritis
Stones
Microscopic haematuria common
Structural abnormalities
Benign prostatic hyperplasia (BPH) causes haematuria due to hypervascularity of the prostate gland
Cystic renal lesions e.g. polycystic kidney disease
Vascular malformations
Renal vein thrombosis due to renal cell carcinoma
Coagulopathy
Causes bleeding of underlying lesions
Drugs
Cause tubular necrosis or interstitial nephritis: aminoglycosides, chemotherapy
Interstitial nephritis: penicillin, sulphonamides, and NSAIDs
Anticoagulants
Benign
Exercise
Gynaecological
Endometriosis: flank pain, dysuria, and haematuria that is cyclical
Iatrogenic
Catheterisation
Radiotherapy; cystitis, severe haemorrhage, bladder necrosis
Pseudohaematuria For example following consumption of beetroot

References
Http://bestpractice.bmj.com/best-practice/monograph/316/overview/aetiology.html

1169
Q

A 10 year old boy is admitted to the emergency department following a fall. On examination, there is deformity and swelling of the forearm. The ability to flex the fingers of the affected limb is impaired. However, there is no sensory impairment. Imaging confirms a displaced forearm fracture. Which of the nerves listed below is likely to have been affected?

Ulnar

Posterior interosseous nerve

Axillary

Radial

A

Forearm fractures may be complicated by neurovascular compromise. The anterior interosseous nerve may be affected. It has no sensory supply so the defect is motor alone.

Anterior interosseous nerve

The anterior interosseous nerve (volar interosseous nerve) is a branch of the median nerve that supplies the deep muscles on the front of the forearm, except the ulnar half of the flexor digitorum profundus.

It accompanies the anterior interosseous artery along the anterior of the interosseous membrane of the forearm, in the interval between the flexor pollicis longus and flexor digitorum profundus, supplying the whole of the former and (most commonly) the radial half of the latter, and ending below in the pronator quadratus and wrist joint.

Innervation
The anterior interosseous nerve classically innervates 2.5 muscles:

Flexor pollicis longus
Pronator quadratus
The radial half of flexor digitorum profundus (the lateral two out of the four tendons).

These muscles are in the deep level of the anterior compartment of the forearm.

1170
Q

A 43 year old lady is due to undergo an axillary node clearance as part of treatment for carcinoma of the breast. Which of the following fascial layers will be divided during the surgical approach to the axilla?

Sibsons fascia

Pre tracheal fascia

Waldayers fascia

Clavipectoral fascia

None of the above

A

The clavipectoral fascia is situated under the clavicular portion of pectoralis major. It protects both the axillary vessels and nodes. During an axillary node clearance for breast cancer the clavipectoral fascia is incised and this allows access to the nodal stations. The nodal stations are; level 1 nodes inferior to pectoralis minor, level 2 lie behind it and level 3 above it. During a Patey Mastectomy surgeons divide pectoralis minor to gain access to level 3 nodes. The use of sentinel node biopsy (and stronger assistants!) have made this procedure far less common.

Axilla

Boundaries of the axilla
Medially Chest wall and Serratus anterior
Laterally Humeral head
Floor Subscapularis
Anterior aspect Lateral border of Pectoralis major
Fascia Clavipectoral fascia

Content:
Long thoracic nerve (of Bell) Derived from C5-C7 and passes behind the brachial plexus to enter the axilla. It lies on the medial chest wall and supplies serratus anterior. Its location puts it at risk during axillary surgery and damage will lead to winging of the scapula.
Thoracodorsal nerve and thoracodorsal trunk Innervate and vascularise latissimus dorsi.
Axillary vein Lies at the apex of the axilla, it is the continuation of the basilic vein. Becomes the subclavian vein at the outer border of the first rib.
Intercostobrachial nerves Traverse the axillary lymph nodes and are often divided during axillary surgery. They provide cutaneous sensation to the axillary skin.
Lymph nodes The axilla is the main site of lymphatic drainage for the breast.

1171
Q

A 74 year old lady has a long standing venous leg ulcer overlying her medial malleolus. Which of the following statements relating to the management of this condition is false?

Pentoxifylline may speed ulcer healing

Treatment with daily low dose flucloxacillin may speed ulcer healing

Multilayer bandages may provide compression equivalent to 40mmHg

Large ulcers may be considered for skin grafting

They should not be treated with compression stockings if the ankle / brachial pressure index is 0.4

A

Routine use of antibiotics is not advised as this may predispose to resistant organisms. Pentoxifylline was subjected to a Cochrane review in 2007 and shown to improve healing rates.

Lower leg ulcers

Venous leg ulcers
Most due to venous hypertension, secondary to chronic venous insufficiency (other causes include calf pump dysfunction or neuromuscular disorders)
Ulcers form due to capillary fibrin cuff or leucocyte sequestration
Features of venous insufficiency include oedema, brown pigmentation, lipodermatosclerosis, eczema
Location above the ankle, painless
Deep venous insufficiency is related to previous DVT and superficial venous insufficiency is associated with varicose veins
Doppler ultrasound looks for presence of reflux and duplex ultrasound looks at the anatomy/ flow of the vein
Management: 4 layer compression banding after exclusion of arterial disease or surgery
If fail to heal after 12 weeks or >10cm2 skin grafting may be needed

Marjolin’s ulcer

Squamous cell carcinoma
Occurring at sites of chronic inflammation e.g; burns, osteomyelitis after 10-20 years
Mainly occur on the lower limb

Arterial ulcers
Occur on the toes and heel
Painful
There may be areas of gangrene
Cold with no palpable pulses
Low ABPI measurements

Neuropathic ulcers
Commonly over plantar surface of metatarsal head and plantar surface of hallux
The plantar neuropathic ulcer is the condition that most commonly leads to amputation in diabetic patients
Due to pressure
Management includes cushioned shoes to reduce callus formation

Pyoderma gangrenosum

Associated with inflammatory bowel disease/RA
Can occur at stoma sites
Erythematous nodules or pustules which ulcerate

1172
Q

A 25 year old man is involved in a road traffic accident. He is evacuated from the scene by helicopter. En route to the regional trauma centre, he becomes more unstable and hypotensive. Due to agitation, the decision is made to intubate him to achieve a secure airway. Which of these agents is likely to be required first?

Propofol

Ketamine

Vecuronium

Suxamethonium

Etomidate

A

It is standard practice to administer a sedative drug prior to achieving paralysis. Ketamine causes the least haemodynamic instability and would usually be used first. A paralysing agent is then used.

Anaesthetic agents

The table below summarises some of the more commonly used IV induction agents
Agent Specific features
Propofol
Rapid onset of anaesthesia
Pain on IV injection
Rapidly metabolised with little accumulation of metabolites
Proven anti emetic properties
Moderate myocardial depression
Widely used especially for maintaining sedation on ITU, total IV anaesthesia and for daycase surgery
Sodium thiopentone
Extremely rapid onset of action making it the agent of choice for rapid sequence of induction
Marked myocardial depression may occur
Metabolites build up quickly
Unsuitable for maintenance infusion
Little analgesic effects
Ketamine
May be used for induction of anaesthesia
Has moderate to strong analgesic properties
Produces little myocardial depression making it a suitable agent for anaesthesia in those who are haemodynamically unstable
May induce state of dissociative anaesthesia resulting in nightmares
Etomidate
Has favorable cardiac safety profile with very little haemodynamic instability
No analgesic properties
Unsuitable for maintaining sedation as prolonged (and even brief) use may result in adrenal suppression
Post operative vomiting is common

1173
Q

A 75 year old lady presents with weight loss, pain and a swelling over her left knee. She has been treated for Pagets disease of the bone for some time. What is the most likely diagnosis?

Ewings sarcoma

Osteosarcoma

Myeloma

Septic arthritis

Osteoclastoma

A

Osteosarcoma may complicate Pagets disease of bone in up to 10% cases. Radiological appearances include bone destruction coupled with new bone formation, periosteal elevation may also occur. Surgical resection is the main treatment.

Sarcomas

Malignant tumours of mesenchymal origin

Types
May be either bone or soft tissue in origin.
Bone sarcoma include:
Osteosarcoma
Ewings sarcoma (although non bony sites recognised)
Chondrosarcoma - originate from Chondrocytes

Soft tissue sarcoma are a far more heterogeneous group and include:
Liposarcoma-adipocytes
Rhabdomyosarcoma-striated muscle
Leiomyosarcoma-smooth muscle
Synovial sarcomas- close to joints (cell of origin not known but not synovium)

Malignant fibrous histiocytoma is now referred to as undifferentiated pleomorphic sarcoma. Careful histological assessment of lesions now allows more accurate categorisation of sarcoma subtypes than was previously possible.

Features
Certain features of a mass or swelling should raise suspicion for a sarcoma these include:
Large >5cm soft tissue mass
Deep tissue location or intra muscular location
Rapid growth
Painful lump

Assessment
Imaging of suspicious masses should utilise a combination of MRI, CT and USS. Blind biopsy should not be performed prior to imaging and where required should be done in such a way that the biopsy tract can be subsequently included in any resection.

Ewings sarcoma
Commoner in males
Incidence of 0.3 / 1, 000, 000
Onset typically between 10 and 20 years of age
Location by femoral diaphysis is commonest site
Histologically it is a small round tumour
Blood borne metastasis is common and chemotherapy is often combined with surgery

Osteosarcoma
Mesenchymal cells with osteoblastic differentiation
20% of all primary bone tumours
Incidence of 5 per 1,000,000
Peak age 15-30, commoner in males
Limb preserving surgery may be possible and many patients will receive chemotherapy

Liposarcoma
Malignancy of adipocytes
Rare, approximately 2.5 per 1,000,000. They are the most common soft tissue sarcoma
Typically located in deep locations such as retroperitoneum
Affect older age group usually >40 years of age
May be well differentiated and thus slow growing although may undergo de-differentiation and disease progression
Many tumours will have a pseudocapsule that can misleadingly allow surgeons to feel that they can ‘shell out’ these lesions. In reality, tumour may invade at the edge of the pseudocapsule and result in local recurrence if this strategy is adopted
Usually resistant to radiotherapy, although this is often used in a palliative setting

Malignant Fibrous Histiocytoma
Tumour with large number of histiocytes
Also described as undifferentiated pleomorphic sarcoma NOS (i.e. Cell of origin is not known)
Four major subtypes are recognised: storiform-pleomorphic (70% cases), myxoid (less aggressive), giant cell and inflammatory
Treatment is usually with surgical resection and adjuvant radiotherapy as this reduces the likelihood of local recurrence

1174
Q

A 46 year old man is undergoing a colonoscopy under sedation using a combination of midazolam and fentanyl sedation. The procedure is complicated by the development of a significant sigmoid loop and becomes increasingly uncomfortable. When the endoscope is in the region of the colonic splenic flexure, the patients asks for the procedure to stop. What is the best course of action?

Administration of additional midazolam and proceeding to complete the procedure

Withdrawal of the endoscope and arranging an alternative test

Call an anaesthetist to administer an general anaesthetic

Administration of intravenous fentantyl and proceeding to complete the procedure

Turn the patient prone and complete the procedure

A

During endoscopic procedures, many patients have conscious sedation and retain decision making ability. If they request the procedure to be stopped this must be adhered to.

Consent

There are 3 types of consent:

  1. Informed
  2. Expressed
  3. Implied

Consent forms used in UK NHS
Consent Form 1 For competent adults who are able to consent for themselves where consciousness may be impaired (e.g. GA)
Consent Form 2 For an adult consenting on behalf of a child where consciousness is impaired
Consent Form 3 For an adult or child where consciousness is not impaired
Consent Form 4 For adults who lack capacity to provide informed consent

Capacity
Key points include:
1. Understand and retain information
2. Patient believes the information to be true
3. Patient is able to weigh the information to make a decision
All patients must be assumed to have capacity

Consent in minors
Young children and older children who are not Gillick competent cannot consent for themselves. In British law the patients biological mother can always provide consent. The child’s father can consent if the parents are married (and the father is the biological father), or if the father is named on the birth certificate (irrespective of marital status). If parents are not married and the father is not named on the birth certificate then the father cannot consent.

1175
Q

A 72 year old man develops a foot drop after a revision total hip replacement. Which nerve is likely to have been affected?

Sciatic

Femoral

Obturator

Superior gluteal

Inferior gluteal

A

Whilst many of these nerves can be injured in hip surgery, for a foot drop to develop in this context (i.e. revision THR), the sciatic nerve is the most likely.

Sciatic nerve

The sciatic nerve is formed from the sacral plexus and is the largest nerve in the body. It is the continuation of the main part of the plexus arising from ventral rami of L4 to S3. These rami converge at the inferior border of piriformis to form the nerve itself. It passes through the inferior part of the greater sciatic foramen and emerges beneath piriformis. Medially, lie the inferior gluteal nerve and vessels and the pudendal nerve and vessels. It runs inferolaterally under the cover of gluteus maximus midway between the greater trochanter and ischial tuberosity. It receives its blood supply from the inferior gluteal artery. The nerve provides cutaneous sensation to the skin of the foot and the leg. It also innervates the posterior thigh muscles and the lower leg and foot muscles. The nerve splits into the tibial and common peroneal nerves approximately half way down the posterior thigh. The tibial nerve supplies the flexor muscles and the common peroneal nerve supplies the extensor muscles and the evertor muscles of the foot.

Summary points
Origin Spinal nerves L4 - S3
Articular Branches Hip joint
Muscular branches in upper leg
Semitendinosus
Semimembranosus
Biceps femoris
Part of adductor magnus
Cutaneous sensation
Posterior aspect of thigh (via cutaneous nerves)
Gluteal region
Entire lower leg (except the medial aspect)
Terminates At the upper part of the popliteal fossa by dividing into the tibial and peroneal nerves

The nerve to the short head of the biceps femoris comes from the common peroneal part of the sciatic and the other muscular branches arise from the tibial portion.
The tibial nerve goes on to innervate all muscles of the foot except the extensor digitorum brevis (which is innervated by the common peroneal nerve).

1176
Q

A 15 year old boy develops sudden onset of pain in the left hemiscrotum. He has no other urinary symptoms. On examination, the superior pole of the testis is tender and the cremasteric reflex is particularly marked. What is the most likely underlying diagnosis?

Epididymo-orchitis

Torsion of the testis

Torsion of a testicular hydatid

Viral orchitis

Testicular malignancy

A

The cremasteric reflex is usually preserved when the torsion affects the appendage only. Testicular disorders

Testicular cancer
Testicular cancer is the most common malignancy in men aged 20-30 years. Around 95% of cases of testicular cancer are germ-cell tumours. Germ cell tumours may essentially be divided into:

Tumour type Key features Tumour markers Pathology
Seminoma
Commonest subtype (50%)
Average age at diagnosis = 40
Even advanced disease associated with 5 year survival of 73%
AFP usually normal
HCG elevated in 10% seminomas
Lactate dehydrogenase; elevated in 10-20% seminomas (but also in many other conditions)
Sheet like lobular patterns of cells with substantial fibrous component. Fibrous septa contain lymphocytic inclusions and granulomas may be seen.
Non seminomatous germ cell tumours (42%)
Teratoma
Yolk sac tumour
Choriocarcinoma
Mixed germ cell tumours (10%)
Younger age at presentation =20-30 years
Advanced disease carries worse prognosis (48% at 5 years)
Retroperitoneal lymph node dissection may be needed for residual disease after chemotherapy
AFP elevated in up to 70% of cases
HCG elevated in up to 40% of cases
Other markers rarely helpful

Risk factors for testicular cancer
Cryptorchidism
Infertility
Family history
Klinefelter’s syndrome
Mumps orchitis

Features
A painless lump is the most common presenting symptom
Pain may also be present in a minority of men
Other possible features include hydrocele, gynaecomastia

Diagnosis
Ultrasound is first-line
CT scanning of the chest/ abdomen and pelvis is used for staging
Tumour markers (see above) should be measured

Management
Orchidectomy (Inguinal approach)
Chemotherapy and radiotherapy may be given depending on staging
Abdominal lesions >1cm following chemotherapy may require retroperitoneal lymph node dissection.

Prognosis is generally excellent
5 year survival for seminomas is around 95% if Stage I
5 year survival for teratomas is around 85% if Stage I

Benign disease

Epididymo-orchitis
Acute epididymitis is an acute inflammation of the epididymis, often involving the testis and usually caused by bacterial infection.
Infection spreads from the urethra or bladder. In men <35 years, gonorrhoea or chlamydia are the usual infections.
Amiodarone is a recognised non infective cause of epididymitis, which resolves on stopping the drug.
Tenderness is usually confined to the epididymis, which may facilitate differentiating it from torsion where pain usually affects the entire testis.

Testicular torsion
Twist of the spermatic cord resulting in testicular ischaemia and necrosis.
Most common in males aged between 10 and 30 (peak incidence 13-15 years)
Pain is usually severe and of sudden onset.
Cremasteric reflex is lost and elevation of the testis does not ease the pain.
Treatment is with surgical exploration. If a torted testis is identified then both testis should be fixed as the condition of bell clapper testis is often bilateral.

Hydrocele
Presents as a mass that transilluminates, usually possible to ‘get above’ it on examination.
In younger men it should be investigated with USS to exclude tumour.
In children it may occur as a result of a patent processus vaginalis.
Treatment in adults is with a Lords or Jabouley procedure.
Treatment in children is with trans inguinal ligation of PPV.

1177
Q

A 23 year old rugby player sustains a Smiths Fracture. On examination, opposition of the thumb is markedly weakened. Which of the following nerves is least likely to be working normally?

Ulnar

Median

Radial

Musculocutaneous

Palmar cutaneous

A

This high velocity injury can often produce significant angulation and displacement. Both of these may impair the function of the median nerve with loss of function of the muscles of the thenar eminence.
Median nerve

The median nerve is formed by the union of a lateral and medial root respectively from the lateral (C5,6,7) and medial (C8 and T1) cords of the brachial plexus; the medial root passes anterior to the third part of the axillary artery. The nerve descends lateral to the brachial artery, crosses to its medial side (usually passing anterior to the artery). It passes deep to the bicipital aponeurosis and the median cubital vein at the elbow.
It passes between the two heads of the pronator teres muscle, and runs on the deep surface of flexor digitorum superficialis (within its fascial sheath).
Near the wrist it becomes superficial between the tendons of flexor digitorum superficialis and flexor carpi radialis, deep to palmaris longus tendon. It passes deep to the flexor retinaculum to enter the palm, but lies anterior to the long flexor tendons within the carpal tunnel.

Branches
Region Branch
Upper arm No branches, although the nerve commonly communicates with the musculocutaneous nerve
Forearm Pronator teres
Pronator quadratus
Flexor carpi radialis
Palmaris longus
Flexor digitorum superficialis
Flexor pollicis longus
Flexor digitorum profundus (only the radial half)
Distal forearm Palmar cutaneous branch
Hand (Motor) Motor supply (LOAF)
Lateral 2 lumbricals
Opponens pollicis
Abductor pollicis brevis
Flexor pollicis brevis
Hand (Sensory)
Over thumb and lateral 2 ½ fingers
On the palmar aspect this projects proximally, on the dorsal aspect only the distal regions are innervated with the radial nerve providing the more proximal cutaneous innervation.

Patterns of damage
Damage at wrist
e.g. carpal tunnel syndrome
paralysis and wasting of thenar eminence muscles and opponens pollicis (ape hand deformity)
sensory loss to palmar aspect of lateral (radial) 2 ½ fingers

Damage at elbow, as above plus:
unable to pronate forearm
weak wrist flexion
ulnar deviation of wrist

Anterior interosseous nerve (branch of median nerve)
leaves just below the elbow
results in loss of pronation of forearm and weakness of long flexors of thumb and index finger

1178
Q

A motorcyclist is injured in a road traffic accident and is not wearing a helmet. He suffers a severe closed head injury and develops raised intracranial pressure. The first cranial nerve to be affected by this process is likely to be:

Oculomotor

Hypoglossal

Motor branch of the trigeminal

Abducens

Sensory branch of the trigeminal

A

The abducens nerve (CN VI) has a long intra cranial course and is thus susceptible to raised intra cranial pressure. It also passes over the petrous temporal bone and 6th nerve palsies are also seen in mastoiditis.

Cranial nerves

Cranial nerve lesions
Olfactory nerve May be injured in basal skull fractures or involved in frontal lobe tumour extension. Loss of olfactory nerve function in relation to major CNS pathology is seldom an isolated event and thus it is poor localiser of CNS pathology.
Optic nerve Problems with visual acuity may result from intra ocular disorders. Problems with the blood supply such as amaurosis fugax may produce temporary visual distortion. More important surgically is the pupillary response to light. The pupillary size may be altered in a number of disorders. Nerves involved in the resizing of the pupil connect to the pretectal nucleus of the high midbrain, bypassing the lateral geniculate nucleus and the primary visual cortex. From the pretectal nucleus neurones pass to the Edinger - Westphal nucleus, motor axons from here pass along with the oculomotor nerve. They synapse with ciliary ganglion neurones; the parasympathetic axons from this then innervate the iris and produce miosis. The miotic pupil is seen in disorders such as Horner’s syndrome or opiate overdose.
Mydriasis is the dilatation of the pupil in response to disease, trauma, drugs (or the dark!). It is pathological when light fails to induce miosis. The radial muscle is innervated by the sympathetic nervous system. Because the parasympathetic fibres travel with the oculomotor nerve they will be damaged by lesions affecting this nerve (e.g. cranial trauma).
The response to light shone in one eye is usually a constriction of both pupils. This indicates intact direct and consensual light reflexes. When the optic nerve has an afferent defect the light shining on the affected eye will produce a diminished pupillary response in both eyes. Whereas light shone on the unaffected eye will produce a normal pupillary response in both eyes. This is referred to as the Marcus Gunn pupil and is seen in conditions such as optic neuritis. In a total CN II lesion shining the light in the affected eye will produce no response.
Oculomotor nerve The pupillary effects are described above. In addition it supplies all ocular muscles apart from lateral rectus and superior oblique. Thus the affected eye will be deviated inferolaterally. Levator palpebrae superioris may also be impaired resulting in impaired ability to open the eye.
Trochlear nerve The eye will not be able to look down.
Trigeminal nerve Largest cranial nerve. Exits the brainstem at the pons. Branches are ophthalmic, maxillary and mandibular. Only the mandibular branch has both sensory and motor fibres. Branches converge to form the trigeminal ganglion (located in Meckels cave). It supplies the muscles of mastication and also tensor veli palatine, mylohyoid, anterior belly of digastric and tensor tympani. The detailed descriptions of the various sensory functions are described in other areas of the website. The corneal reflex is important and is elicited by applying a small tip of cotton wool to the cornea, a reflex blink should occur if it is intact. It is mediated by: the naso ciliary branch of the ophthalmic branch of the trigeminal (sensory component) and the facial nerve producing the motor response. Lesions of the afferent arc will produce bilateral absent blink and lesions of the efferent arc will result in a unilateral absent blink.
Abducens nerve The affected eye will have a deficit of abduction. This cranial nerve exits the brainstem between the pons and medulla. It thus has a relatively long intra cranial course which renders it susceptible to damage in raised intra cranial pressure.
Facial nerve Emerges from brainstem between pons and medulla. It controls muscles of facial expression and taste from the anterior 2/3 of the tongue. The nerve passes into the petrous temporal bone and into the internal auditory meatus. It then passes through the facial canal and exits at the stylomastoid foramen. It passes through the parotid gland and divides at this point. It does not innervate the parotid gland. Its divisions are considered in other parts of the website. Its motor fibres innervate orbicularis oculi to produce the efferent arm of the corneal reflex. In surgical practice it may be injured during parotid gland surgery or invaded by malignancies of the gland and a lower motor neurone on the ipsilateral side will result.
Vestibulo-cochlear nerve Exits from the pons and then passes through the internal auditory meatus. It is implicated in sensorineural hearing loss. Individuals with sensorineural hearing loss will localise the sound in webers test to the normal ear. Rinnes test will be reduced on the affected side but should still work. These two tests will distinguish sensorineural hearing loss from conductive deafness. In the latter condition webers test will localise to the affected ear and Rinnes test will be impaired on the affected side. Surgical lesions affecting this nerve include CNS tumours and basal skull fractures. It may also be damaged by the administration of ototoxic drugs (of which gentamicin is the most commonly used in surgical practice).
Glossopharyngeal nerve Exits the pons just above the vagus. Receives sensory fibres from posterior 1/3 tongue, tonsils, pharynx and middle ear (otalgia may occur following tonsillectomy). It receives visceral afferents from the carotid bodies. It supplies parasympathetic fibres to the parotid gland via the otic ganglion and motor function to stylopharyngeaus muscle. The sensory function of the nerve is tested using the gag reflex.
Vagus nerve Leaves the medulla between the olivary nucleus and the inferior cerebellar peduncle. Passes through the jugular foramen and into the carotid sheath. Details of the functions of the vagus nerve are covered in the website under relevant organ sub headings.
Accessory nerve Exists from the caudal aspect of the brainstem (multiple branches) supplies trapezius and sternocleidomastoid muscles. The distal portion of this nerve is most prone to injury during surgical procedures.
Hypoglossal nerve Emerges from the medulla at the preolivary sulcus, passes through the hypoglossal canal. It lies on the carotid sheath and passes deep to the posterior belly of digastric to supply muscles of the tongue (except palatoglossus). Its location near the carotid sheath makes it vulnerable during carotid endarterectomy surgery and damage will produce ipsilateral defect in muscle function.

1179
Q

A 48 year old motor cyclist sustains a complex lower limb fracture in a motor accident. For a time the popliteal artery is occluded and eventually repaired. Subsequently he develops a compartment syndrome and the anterior and superficial posterior compartments of the lower leg are decompressed. Unfortunately, the operating surgeon neglects to decompress the deep posterior compartment. Which of the following muscles is least likely to be affected as a result?

Flexor digitorum longus

Plantaris

Tibialis posterior

Flexor hallucis longus

None of the above

A

Muscles of the deep posterior compartment:
Tibialis posterior
Flexor hallucis longus
Flexor digitorum longus
Popliteus
The plantaris muscle lies within the superficial posterior compartment of the lower leg.

Lower limb- Muscular compartments

Anterior compartment
Muscle Nerve Action
Tibialis anterior Deep peroneal nerve Dorsiflexes ankle joint, inverts foot
Extensor digitorum longus Deep peroneal nerve Extends lateral four toes, dorsiflexes ankle joint
Peroneus tertius Deep peroneal nerve Dorsiflexes ankle, everts foot
Extensor hallucis longus Deep peroneal nerve Dorsiflexes ankle joint, extends big toe

Peroneal compartment
Muscle Nerve Action
Peroneus longus Superficial peroneal nerve Everts foot, assists in plantar flexion
Peroneus brevis Superficial peroneal nerve Plantar flexes the ankle joint

Superficial posterior compartment
Nerve Action
Gastrocnemius Tibial nerve Plantar flexes the foot, may also flex the knee
Soleus Tibial nerve Plantar flexor

Deep posterior compartment
Muscle Nerve Action
Flexor digitorum longus Tibial Flexes the lateral four toes
Flexor hallucis longus Tibial Flexes the great toe
Tibialis posterior Tibial Plantar flexor, inverts the foot

1180
Q

A 60-year-old man develops palpitations while on the acute surgical unit. An ECG shows a broad complex tachycardia at a rate of 150 bpm. His blood pressure is 124/82 mmHg and there is no evidence of heart failure. The surgical consultant wants to give rate control (the medical team are not answering their bleeps). Which one of the following is least appropriate to give?

Procainamide

Lidocaine

Amiodarone

Adenosine

Verapamil

A

Ventricular tachycardia - verapamil is contraindicated
Verapamil should never be given to a patient with a broad complex tachycardia as it may precipitate ventricular fibrillation in patients with ventricular tachycardia. Adenosine is sometimes given in this situation as a ‘trial’ if there is a strong suspicion the underlying rhythm is a supraventricular tachycardia with aberrant conduction

Ventricular tachycardia: management

Whilst a broad complex tachycardia may result from a supraventricular rhythm with aberrant conduction, the European Resuscitation Council advise that in a peri-arrest situation it is assumed to be ventricular in origin

If the patient has adverse signs (systolic BP < 90 mmHg, chest pain, heart failure or rate > 150 beats/min) then immediate cardioversion is indicated. In the absence of such signs antiarrhythmics may be used. If these fail, then electrical cardioversion may be needed with synchronised DC shocks

Drug therapy
amiodarone: ideally administered through a central line
lidocaine: use with caution in severe left ventricular impairment
procainamide

Verapamil should NOT be used in VT

If drug therapy fails
electrophysiological study (EPS)
implant able cardioverter-defibrillator (ICD) - this is particularly indicated in patients with significantly impaired LV function

1181
Q

A 23 year old man is involved in a fight outside a nightclub and sustains a laceration to his right arm. On examination, he has lost extension of the fingers in his right hand. Which of the nerves listed below is most likely to have been divided?

Median

Musculocutaneous

Radial

Ulnar

Axillary

A

The radial nerve supplies the extensor muscle group.
Radial nerve

Continuation of posterior cord of the brachial plexus (root values C5 to T1)

Path
In the axilla: lies posterior to the axillary artery on subscapularis, latissimus dorsi and teres major.
Enters the arm between the brachial artery and the long head of triceps (medial to humerus).
Spirals around the posterior surface of the humerus in the groove for the radial nerve.
At the distal third of the lateral border of the humerus it then pierces the intermuscular septum and descends in front of the lateral epicondyle.
At the lateral epicondyle it lies deeply between brachialis and brachioradialis where it then divides into a superficial and deep terminal branch.
Deep branch crosses the supinator to become the posterior interosseous nerve.

Regions innervated
Motor (main nerve)
Triceps
Anconeus
Brachioradialis
Extensor carpi radialis
Motor (posterior interosseous branch)
Supinator
Extensor carpi ulnaris
Extensor digitorum
Extensor indicis
Extensor digiti minimi
Extensor pollicis longus and brevis
Abductor pollicis longus
Sensory The area of skin supplying the proximal phalanges on the dorsal aspect of the hand is supplied by the radial nerve (this does not apply to the little finger and part of the ring finger)

Muscular innervation and effect of denervation
Anatomical location Muscle affected Effect of paralysis
Shoulder Long head of triceps Minor effects on shoulder stability in abduction
Arm Triceps Loss of elbow extension
Forearm Supinator
Brachioradialis
Extensor carpi radialis longus and brevis Weakening of supination of prone hand and elbow flexion in mid prone position

1182
Q

Which of the following is not an effect of cholecystokinin?

It causes gallbladder contraction

It increases the rate of gastric emptying

It relaxes the sphincter of oddi

It stimulates pancreatic acinar cells

It has a trophic effect on pancreatic acinar cells

A

It decreases the rate of gastric emptying.

Gastric secretions

A working knowledge of gastric secretions is important for surgery because peptic ulcers are common, surgeons frequently prescribe anti secretory drugs and because there are still patients around who will have undergone acid lowering procedures (Vagotomy) in the past.

Gastric acid
Is produced by the parietal cells in the stomach
pH of gastric acid is around 2 with acidity being maintained by the H+/K+ ATP ase pump. As part of the process bicarbonate ions will be secreted into the surrounding vessels.
Sodium and chloride ions are actively secreted from the parietal cell into the canaliculus. This sets up a negative potential across the membrane and as a result sodium and potassium ions diffuse across into the canaliculus.
Carbonic anhydrase forms carbonic acid which dissociates and the hydrogen ions formed by dissociation leave the cell via the H+/K+ antiporter pump. At the same time sodium ions are actively absorbed. This leaves hydrogen and chloride ions in the canaliculus these mix and are secreted into the lumen of the oxyntic gland.

Phases of gastric acid secretion
There are 3 phases of gastric secretion:

  1. Cephalic phase (smell / taste of food)
    30% acid produced
    Vagal cholinergic stimulation causing secretion of HCL and gastrin release from G cells
  2. Gastric phase (distension of stomach )
    60% acid produced
    Stomach distension/low H+/peptides causes Gastrin release
  3. Intestinal phase (food in duodenum)
    10% acid produced
    High acidity/distension/hypertonic solutions in the duodenum inhibits gastric acid secretion via enterogastrones (CCK, secretin) and neural reflexes.

Regulation of gastric acid production
Factors increasing production include:
Vagal nerve stimulation
Gastrin release
Histamine release (indirectly following gastrin release) from enterchromaffin like cells

Factors decreasing production include:
Somatostatin (inhibits histamine release)
Cholecystokinin
Secretin

Below is a brief summary of the major hormones involved in food digestion:

Source Stimulus Actions
Gastrin
G cells in antrum of the stomach
Distension of stomach, extrinsic nerves
Inhibited by: low antral pH, somatostatin
Increase HCL, pepsinogen and IF secretion, increases gastric motility, trophic effect on gastric mucosa
CCK
I cells in upper small intestine
Partially digested proteins and triglycerides
Increases secretion of enzyme-rich fluid from pancreas, contraction of gallbladder and relaxation of sphincter of Oddi, decreases gastric emptying, trophic effect on pancreatic acinar cells, induces satiety
Secretin
S cells in upper small intestine
Acidic chyme, fatty acids
Increases secretion of bicarbonate-rich fluid from pancreas and hepatic duct cells, decreases gastric acid secretion, trophic effect on pancreatic acinar cells
VIP
Small intestine, pancreas
Neural
Stimulates secretion by pancreas and intestines, inhibits acid and pepsinogen secretion
Somatostatin
D cells in the pancreas and stomach
Fat, bile salts and glucose in the intestinal lumen
Decreases acid and pepsin secretion, decreases gastrin secretion, decreases pancreatic enzyme secretion, decreases insulin and glucagon secretion
inhibits trophic effects of gastrin, stimulates gastric mucous production

1183
Q

Where does spironolactone act in the kidney?

Glomerulus

Proximal convoluted tubule

Descending limb of the loop of Henle

Ascending limb of the loop of Henle

Distal convoluted tubule

A

E- Potassium sparing diuretics

Potassium-sparing diuretics may be divided into the epithelial sodium channel blockers (amiloride and triamterene) and aldosterone antagonists (spironolactone and eplerenone).

Amiloride is a weak diuretic which blocks the epithelial sodium channel in the distal convoluted tubule.

Usually given with thiazides or loop diuretics as an alternative to potassium supplementation.

Spironolactone is an aldosterone antagonist which acts at in the distal convoluted tubule.

Indications
ascites: patients with cirrhosis develop a secondary hyperaldosteronism. Relatively large doses such as 100 or 200mg are often used
heart failure
nephrotic syndrome
Conn’s syndrome

1184
Q

Which of the following surgical energy devices would be most appropriate for the dissection of the posteriomedial aspect of the thyroid gland during thyroidectomy?

Harmonic ACE

Monopolar diathermy in coagulation mode

Bipolar diathermy

CUSA device

Argon plasma coagulation device

A

This will minimise thermal trauma to the recurrent laryngeal nerve

Diathermy

  • Diathermy devices are used by surgeons in all branches of surgery.
    Use electric currents to produce local heat and thereby facilitate haemostasis or surgical dissection.
    Consist of a generator unit that is located outside the patient and can be set to the level of power required by the surgeon.
    There are two major types of diathermy machine;

Monopolar
The current flows through the diathermy unit into a handheld device that is controlled by the surgeon. Electricity can flow from the tip of the device into the patient. The earth electrode is located some distance away. The relatively narrow tip of the diathermy device produces local heat and this can be used to vaporise and fulgurate tissues. The current can be adjusted in terms of frequency so that different actions can be effected. In cutting mode sufficient power is applied to the tissues to vaporise their water content. In coagulation mode the power level is reduced so that a coagulum is formed instead. Some diathermy machines can utilise a setting known as blend that alternates cutting and coagulation functions, these tend to be used during procedures such as colonoscopic polypectomy.

Bipolar
The electric current flows from one electrode to another however, both electrodes are usually contained within the same device e.g. a pair of forceps. The result is that heating is localised to the area between the two electrodes and surrounding tissue damage is minimised.

Ultrasound based devices
These include CUSA and Harmonic scalpel. They generate high frequency oscillations that seal and coagulate tissues. They have different energy settings that allow them to dissect and simultaneously seal vessels if required. The CUSA device leaves vessels intact that may then be divided.

Ligasure device
Delivers tailored energy levels to allow simultaneous haemostasis and dissection. The device senses the impedance of the tissues and tailors energy levels accordingly.

Hazards of diathermy
Inadvertent patient burn. This may result of careless handling of the device or in the case of monopolar devices forgetting to apply a return electrode plate, In this situation patients may develop a contact burn when electricity flows to earth
Explosion or fire. This may occur when volatile anaesthetic gases or skin preparation fluid have been used

1185
Q

A 53 year old man presents with unilateral hyperacousia. Which of the following nerves is least likely to be functioning normally?

Vestibulocochlear

Glossopharyngeal

Facial

Trochlear

Vagus

A

Facial. Damage to the nerve in the bony canal may result in impaired innervation to stapedius and therefore sounds are no longer dampened. Another cause of hyperacusis is increased activity in the tensor tympani muscle, this is innervated by the trigeminal nerve.

Facial nerve

The facial nerve is the main nerve supplying the structures of the second embryonic branchial arch. It is predominantly an efferent nerve to the muscles of facial expression, digastric muscle and also to many glandular structures. It contains a few afferent fibres which originate in the cells of its genicular ganglion and are concerned with taste.

Supply - ‘face, ear, taste, tear’
Face: muscles of facial expression
Ear: nerve to stapedius
Taste: supplies anterior two-thirds of tongue
Tear: parasympathetic fibres to lacrimal glands, also salivary glands

Path
Subarachnoid path
Origin: motor- pons, sensory- nervus intermedius
Pass through the petrous temporal bone into the internal auditory meatus with the vestibulocochlear nerve. Here they combine to become the facial nerve.

Facial canal path
The canal passes superior to the vestibule of the inner ear
At the medial aspect of the middle ear, it becomes wider and contains the geniculate ganglion.
- 3 branches:
1. greater petrosal nerve
2. nerve to stapedius
3. chorda tympani

Stylomastoid foramen
Passes through the stylomastoid foramen (tympanic cavity anterior and mastoid antrum posteriorly)
Posterior auricular nerve and branch to posterior belly of digastric and stylohyoid muscle

Face
Enters parotid gland and divides into 5 branches:
Temporal branch
Zygomatic branch
Buccal branch
Marginal mandibular branch
Cervical branch

1186
Q

A 53 year old man with a carcinoma of the lower third of the oesophagus is undergoing an oesophagogastrectomy. As the surgeons mobilise the mid part of the oesophagus, where are they most likely to encounter the thoracic duct?

Anterior to the oesophagus

On the left side of the oesophagus

On the right side of the oesophagus

Immediately anterior to the azygos vein

Posterior to the oesophagus

A

The thoracic duct lies posterior to the oesophagus and passes to the left at the level of the Angle of Louis. It enters the thorax at T12 alongside with the aorta.

Thoracic duct

Continuation of the cisterna chyli in the abdomen.
Enters the thorax at T12.
Lies posterior to the oesophagus for most of its intrathoracic course. Passes to the left at T5.
Lymphatics draining the left side of the head and neck join the thoracic duct prior to its insertion into the junction between left subclavian and internal jugular veins.
Lymphatics draining the right side of the head and neck drain via the subclavian and jugular trunks into the right lymphatic duct and thence into the mediastinal trunk and eventually the right brachiocephalic vein.
Its location in the thorax makes it prone to injury during oesophageal surgery. Some surgeons administer cream to patients prior to oesophagectomy so that it is easier to identify the cut ends of the duct.

1187
Q

A 44 year old lady has undergone a mastectomy and axillary node clearance. Post operatively, she notices a patch of anaesthesia of her axillary skin when she applies an underarm deodorant. Which nerve has most likely been affected?

Axillary

Intercostobrachial

Long thoracic

Thoracodorsal

Accessory

A

The intercostobrachial nerves traverse the axilla and innervate the overlying skin. These can be injured or divided during axillary surgery and the result is anaesthesia of the overlying skin.
Axilla

Boundaries of the axilla
Medially Chest wall and Serratus anterior
Laterally Humeral head
Floor Subscapularis
Anterior aspect Lateral border of Pectoralis major
Fascia Clavipectoral fascia

Content:
Long thoracic nerve (of Bell) Derived from C5-C7 and passes behind the brachial plexus to enter the axilla. It lies on the medial chest wall and supplies serratus anterior. Its location puts it at risk during axillary surgery and damage will lead to winging of the scapula.
Thoracodorsal nerve and thoracodorsal trunk Innervate and vascularise latissimus dorsi.
Axillary vein Lies at the apex of the axilla, it is the continuation of the basilic vein. Becomes the subclavian vein at the outer border of the first rib.
Intercostobrachial nerves Traverse the axillary lymph nodes and are often divided during axillary surgery. They provide cutaneous sensation to the axillary skin.
Lymph nodes The axilla is the main site of lymphatic drainage for the breast.

1188
Q

Infusion with which of the following blood products is most likely to result in an urticarial reaction?

Packed red cells

Fresh frozen plasma

Platelets

Cryoprecipitate

Factor VIII concentrate

A

Pyrexia is the most common adverse event in transfusing packed red cells
Urticaria is the most common adverse event following infusion of FFP
Blood transfusion reactions

Acute transfusion reactions present as adverse signs or symptoms during or within 24 hours of a blood transfusion. The most frequent reactions are fever, chills, pruritus, or urticaria, which typically resolve promptly without specific treatment or complications. Other signs occurring in temporal relationship with a blood transfusion, such as severe dyspnoea, pyrexia, or loss of consciousness may be the first indication of a more severe potentially fatal reaction.
The causes of adverse reactions are multi-factorial. Immune mediated reactions, some of the most feared, occur as a result of component mismatch, the commonest cause of which is clerical error. More common, non immune mediated, complications may occur as a result of product contamination, this may be bacterial or viral.
Transfusion related lung injury is well recognised and there are two proposed mechanisms which underpin this. One involves the sequestration of primed neutrophils within the recipient pulmonary capillary bed. The other proposed mechanism suggests that HLA mismatches between donor neutrophils and recipient lung tissue is to blame.
The table below summarises the main types of transfusion reaction.

Immune mediated Non immune mediated
Pyrexia Hypocalcaemia
Alloimmunization CCF
Thrombocytopaenia Infections
Transfusion associated lung injury Hyperkalaemia
Graft vs Host disease
Urticaria
Acute or delayed haemolysis
ABO incompatibility
Rhesus incompatibility

1189
Q

A 56 year old lady is due to undergo a left hemicolectomy for carcinoma of the splenic flexure. The surgeons decide to perform a high ligation of the inferior mesenteric vein. Into which of the following does this structure usually drain?

Portal vein

Inferior vena cava

Left renal vein

Left iliac vein

Splenic vein

A

Beware of ureteric injury in colonic surgery.
The inferior mesenteric vein drains into the splenic vein, this point of union lies close to the duodenum and this surgical maneouvre is a recognised cause of ileus.

Left colon

Position
As the left colon passes inferiorly its posterior aspect becomes extraperitoneal, and the ureter and gonadal vessels are close posterior relations that may become involved in disease processes
At a level of L3-4 (variable) the left colon becomes the sigmoid colon and wholly intraperitoneal once again
The sigmoid colon is a highly mobile structure and may even lie on the right side of the abdomen
It passes towards the midline, the taenia blend and this marks the transition between sigmoid colon and upper rectum

Blood supply
Inferior mesenteric artery
However, the marginal artery (from the right colon) contributes, this contribution becomes clinically significant when the IMA is divided surgically (e.g. During AAA repair)

1190
Q

A 56 year old motorcyclist is involved in a road traffic accident and sustains a displaced femoral shaft fracture. No other injuries are identified on the primary or secondary surveys. The fracture is treated with closed, antegrade intramedullary nailing. The following day the patient becomes increasingly agitated and confused. On examination, he is pyrexial, hypoxic SaO2 90% on 6 litres O2, tachycardic and normotensive. Systemic examination demonstrates a non blanching petechial rash present over the torso. What is the most likely explanation for this?

Pulmonary embolism with paradoxical embolus

Fat embolism

Meningococcal sepsis

Alcohol withdrawl

Chronic sub dural haematoma

A

Triad of symptoms:
Respiratory
Neurological
Petechial rash (tends to occur after the first 2 symptoms)
This man has a recent injury and physical signs that would be concordant with fat embolism syndrome. Meningococcal sepsis is not usually associated with hypoxia initially. Pulmonary emboli are not typically associated with pyrexia.

Fat embolism

Diagnosis and clinical features
System Feature
Respiratory
Early persistent tachycardia
Tachypnoea, dyspnoea, hypoxia usually 72 hours following injury
Pyrexia
Dermatological
Red/ brown impalpable petechial rash (usually only in 25-50%)
Subconjunctival and oral haemorrhage/ petechiae
CNS
Confusion and agitation
Retinal haemorrhages and intra-arterial fat globules on fundoscopy

Imaging
May be normal
Fat emboli tend to lodge distally and therefore CTPA may not show any vascular occlusion, a ground glass appearance may be seen at the periphery

Treatment
Prompt fixation of long bone fractures
Some debate regarding benefit Vs. risk of medullary reaming in femoral shaft/ tibial fractures in terms of increasing risk (probably does not).
DVT prophylaxis
General supportive care

1191
Q

A 53 year old teacher is admitted to the vascular ward for a carotid endarterectomy. Your FY1 does a preoperative assessment and notes that there is a right homonymous hemianopia. There is no other neurology. What is the most likely cause?

Lateral medullary syndrome

Middle cerebral artery infarct

Anterior cerebral artery infarct

Cerebellar infarct

Posterior cerebral artery infarct

A

This patient has had a left occipital infarct, as there is only a homonymous hemianopia. If this patient had a temporal or parietal lobe infarct, there would be associated hemiparesis and higher cortical dysfunction. This is important to differentiate, as the carotid endarterectomy is inappropriate in this patient as the lesion is in the posterior cerebral artery.

Stroke: types

Primary intracerebral haemorrhage (PICH, c. 10%)
Presents with headache, vomiting, loss of consciousness
Total anterior circulation infarcts (TACI, c. 15%)
Involves middle and anterior cerebral arteries
Hemiparesis/hemisensory loss
Homonymous hemianopia
Higher cognitive dysfunction e.g. Dysphasia
Partial anterior circulation infarcts (PACI, c. 25%)
Involves smaller arteries of anterior circulation e.g. upper or lower division of middle cerebral artery
Higher cognitive dysfunction or two of the three TACI features
Lacunar infarcts (LACI, c. 25%)
Involves perforating arteries around the internal capsule, thalamus and basal ganglia
Present with either isolated hemiparesis, hemisensory loss or hemiparesis with limb ataxia
Posterior circulation infarcts (POCI, c. 25%)
Vertebrobasilar arteries
Presents with features of brainstem damage
Ataxia, disorders of gaze and vision, cranial nerve lesions
Lateral medullary syndrome (posterior inferior cerebellar artery)
Wallenberg’s syndrome
Ipsilateral: ataxia, nystagmus, dysphagia, facial numbness, cranial nerve palsy
Contralateral: limb sensory loss
Weber’s syndrome
Ipsilateral III palsy
Contralateral weakness

Anterior cerebral artery
Contralateral hemiparesis and sensory loss, lower extremity > upper
Disconnection syndrome

Middle cerebral artery
Contralateral hemiparesis and sensory loss, upper extremity > lower
Contralateral hemianopia
Aphasia (Wernicke’s)
Gaze abnormalities

Posterior cerebral artery
Contralateral hemianopia with macular sparing
Disconnection syndrome

Lacunar
Present with either isolated hemiparesis, hemisensory loss or hemiparesis with limb ataxia

Lateral medulla (posterior inferior cerebellar artery)
Ipsilateral: ataxia, nystagmus, dysphagia, facial numbness, cranial nerve palsy e.g.

Horner’s
Contralateral: limb sensory loss

Pontine
VI nerve: horizontal gaze palsy
VII nerve
Contralateral hemiparesis

1192
Q

A patient presents with a facial nerve palsy. This occurred following repeat excision of a facial lump. The histology report remarks on the biphasic appearance of the lesion and mucinous connective tissue. What is the most likely underlying lesion?

Pleomorphic adenoma

Adenolymphoma

Anaplastic carcinoma

Adenoid cystic carcinoma

Schwannoma

A

The histological features are as described with a classic biphasic (mixed stromal and epithelial elements), although benign local recurrence can complicate incomplete excision. As this is a benign lesion direct extension into the facial nerve is unlikely to occur. Facial nerve injury can happen during repeat parotid surgery.

Neck lumps

The table below gives characteristic exam question features for conditions causing neck lumps:

Reactive lymphadenopathy By far the most common cause of neck swellings. There may be a history of local infection or a generalised viral illness
Lymphoma Rubbery, painless lymphadenopathy
The phenomenon of pain whilst drinking alcohol is very uncommon
There may be associated night sweats and splenomegaly
Thyroid swelling May be hypo-, eu- or hyperthyroid symptomatically
Moves upwards on swallowing
Thyroglossal cyst More common in patients < 20 years old
Usually midline, between the isthmus of the thyroid and the hyoid bone
Moves upwards with protrusion of the tongue
May be painful if infected
Pharyngeal pouch More common in older men
Represents a posteromedial herniation between thyropharyngeus and cricopharyngeus muscles
Usually not seen, but if large then a midline lump in the neck that gurgles on palpation
Typical symptoms are dysphagia, regurgitation, aspiration and chronic cough
Cystic hygroma A congenital lymphatic lesion (lymphangioma) typically found in the neck, classically on the left side
Most are evident at birth, around 90% present before 2 years of age
Branchial cyst An oval, mobile cystic mass that develops between the sternocleidomastoid muscle and the pharynx
Develop due to failure of obliteration of the second branchial cleft in embryonic development
Usually present in early adulthood
Cervical rib More common in adult females
Around 10% develop thoracic outlet syndrome
Carotid aneurysm Pulsatile lateral neck mass which doesn’t move on swallowing

1193
Q

A 43 year old lady presents in a state of cardiovascular collapse. She is stabilized by the ambulance crew prior to transfer to hospital. In the emergency department, a CT scan of the chest, abdomen and pelvis is performed. The scan shows a large volume of blood in the abdomen the cause of which is not immediately apparent. An infusion of packed red cells is ongoing. Her current observations demonstrate a pulse rate of 120 beats per minute. Her blood pressure is 80/60 mm Hg, respiratory rate is 23 breaths per minute. What is the most appropriate course of action?

Immediate laparotomy

Administration of intravenous volume replacement to achieve a systolic blood pressure of >100 mmHg and then proceed to laparotomy

Undertake a diagnostic laparoscopy

Administration of intravenous volume replacement to achieve a systolic blood pressure of >100mmHg and then proceed to diagnostic laparoscopy

Undertake a mesenteric angiogram

A

The patient has large volume blood loss and is unstable in spite of volume replacement. A laparoscopy is unlikely to demonstrate clear operative views to allow assessment. At laparotomy, there will need to be evacuation of blood (the use of a cell saver can be considered). Damage control principles should be followed in such cases.

Massive haemorrhage

Definition
This is the loss of one blood volume in a 24 hour period or the loss of 50% of the circulating blood volume in 3 hours. A blood loss of 150ml/ minute is also included. The normal adult blood volume is 7% of total adult body weight. The blood volume equates to between 8 and 9% of a child’s body weight.

Complications of massive transfusion
Complication Key points
Hypothermia Blood is refrigerated
Hypothermic blood impairs homeostasis
Shifts Bohr curve to the left
Hypocalcaemia Both FFP and platelets contain citrate anticoagulant, this may chelate calcium
Hyperkalaemia Plasma of red cells stored for 4-5 weeks contains 5-10 mmol K+
Delayed type transfusion reactions Due to minor incompatibility issues especially if urgent or non cross matched blood used
Transfusion related lung injury Acute onset non cardiogenic pulmonary oedema
Leading cause of transfusion related deaths
Greatest risk posed with plasma components
Occurs as a result of leucocyte antibodies in transfused plasma
Aggregation and degranulation of leucocytes in lung tissue accounts for lung injury
Coagulopathy Anticipate once circulating blood volume transfused
1 blood volume usually drops platelet count to 100 or less
1 blood volume will both dilute and not replace clotting factors
Fibrinogen concentration halves per 0.75 blood volume transfused

References
Stanworth S et al. Haematological management of major haemorrhage: a British Society for Haematology Guideline British Journal of Haematology 2022 (198): 654 to 667.

1194
Q

Which of the structures listed below lies posterior to the carotid sheath at the level of the 6th cervical vertebra?

Hypoglossal nerve

Vagus nerve

Cervical sympathetic chain

Ansa cervicalis

Glossopharyngeal nerve

A

The carotid sheath is crossed anteriorly by the hypoglossal nerves and the ansa cervicalis. The vagus lies within it. The cervical sympathetic chain lies posteriorly between the sheath and the prevertebral fascia.

Common carotid artery

The right common carotid artery arises at the bifurcation of the brachiocephalic trunk, the left common carotid arises from the arch of the aorta. Both terminate at the level of the upper border of the thyroid cartilage (the lower border of the third cervical vertebra) by dividing into the internal and external carotid arteries.

Left common carotid artery
This vessel arises immediately to the left and slightly behind the origin of the brachiocephalic trunk. Its thoracic portion is 2.5- 3.5 cm in length and runs superolaterally to the sternoclavicular joint.

In the thorax
The vessel is in contact, from below upwards, with the trachea, left recurrent laryngeal nerve, left margin of the oesophagus. Anteriorly the left brachiocephalic vein runs across the artery, and the cardiac branches from the left vagus descend in front of it. These structures together with the thymus and the anterior margins of the left lung and pleura separate the artery from the manubrium.

In the neck
The artery runs superiorly deep to sternocleidomastoid and then enters the anterior triangle. At this point it lies within the carotid sheath with the vagus nerve and the internal jugular vein. Posteriorly the sympathetic trunk lies between the vessel and the prevertebral fascia. At the level of C7 the vertebral artery and thoracic duct lie behind it. The anterior tubercle of C6 transverse process is prominent and the artery can be compressed against this structure (it corresponds to the level of the cricoid).
Anteriorly at C6 the omohyoid muscle passes superficial to the artery.
Within the carotid sheath the jugular vein lies lateral to the artery.

Right common carotid artery
The right common carotid arises from the brachiocephalic artery. The right common carotid artery corresponds with the cervical portion of the left common carotid, except that there is no thoracic duct on the right. The oesophagus is less closely related to the right carotid than the left.

Summary points about the carotid anatomy

Path
Passes behind the sternoclavicular joint (12% patients above this level) to the upper border of the thyroid cartilage, to divide into the external (ECA) and internal carotid arteries (ICA).

Relations
Level of 6th cervical vertebra crossed by omohyoid
Then passes deep to the thyrohyoid, sternohyoid, sternomastoid muscles.
Passes anterior to the carotid tubercle (transverse process 6th cervical vertebra)-NB compression here stops haemorrhage.
The inferior thyroid artery passes posterior to the common carotid artery.
Then : Left common carotid artery crosses the thoracic duct, Right common carotid artery crossed by recurrent laryngeal nerve

1195
Q

An 8 year old boy presents with a 4 hour history of right iliac fossa pain with nausea and vomiting. He has been back at school for two days after being kept home with a flu like illness. On examination, he is tender in the right iliac fossa, although his abdomen is soft. Temperature is 39.3oC. Blood tests show a CRP of 40 and a WCC of 8.1. What is the most appropriate course of action?

Abdominal MRI scan

Abdominal CT scan

Diagnostic laparoscopy

Active observation

Colonoscopy

A

The key point in the history is the preceding flu like illness and absence of abdominal signs. These make mesenteric adenitis the most likely diagnosis. The patient should have a period of active observation, if this were to represent early appendicitis, then the clinical picture may change and this would be detected with serial examination.

Right iliac fossa pain

Differential diagnosis

Appendicitis
Pain radiating to right iliac fossa
Anorexia (very common)
Short history
Diarrhoea and profuse vomiting rare
Crohn’s disease
Often long history
Signs of malnutrition
Change in bowel habit, especially diarrhoea
Mesenteric adenitis
Mainly affects children
Causes include Adenoviruses, Epstein Barr Virus, beta-haemolytic Streptococcus, Staphylococcus spp., Escherichia coli, Streptococcus viridans and Yersinia spp.
Patients have a higher temperature than those with appendicitis
If laparotomy is performed, enlarged mesenteric lymph nodes will be present
Diverticulitis
Both left and right sided disease may present with right iliac fossa pain
Clinical history may be similar, although some change in bowel habit is usual
When suspected, a CT scan may help in refining the diagnosis
Meckel’s diverticulitis
A Meckel’s diverticulum is a congenital abnormality that is present in about 2% of the population
Typically 2 feet proximal to the ileocaecal valve
May be lined by ectopic gastric mucosal tissue and produce bleeding
Perforated peptic ulcer
This usually produces upper quadrant pain but pain may be lower
Perforations typically have a sharp sudden onset of pain in the history
Incarcerated right inguinal or femoral hernia
Usually only right iliac fossa pain if right sided or bowel obstruction.
Bowel perforation secondary to caecal or colon carcinoma
Seldom localised to right iliac fossa, although complete large bowel obstruction with caecal distension may cause pain prior to perforation.
Gynaecological causes
Pelvic inflammatory disease/salpingitis/pelvic abscess/Ectopic pregnancy/Ovarian torsion/Threatened or complete abortion/Mittelschmerz
Urological causes
Ureteric colic/UTI/Testicular torsion
Other causes
TB/Typhoid/Herpes Zoster/AAA/Situs inversus

1196
Q

A 53 year old man has a 1cm polyp identified and completely removed during a colonoscopy. Histology confirms a low grade adenoma. What is the correct follow up?

Suggest participation in bowel cancer screening but no further routine endoscopy

Repeat endoscopy in 5 years.

Repeat endoscopy in 3 years.

Segmental resection of the affected area.

Barium enema at 5 years.

A

In the UK, the guidance has now changed and patients like this are managed expectantly with suggestion that they participate in bowel cancer screening programmes.

Colonic polyps

Colonic Polyps
May occur in isolation, or greater numbers as part of the polyposis syndromes. In FAP greater than 100 polyps are typically present. The risk of malignancy in association with adenomas is related to size, and is the order of 10% in a 1cm adenoma. Isolated adenomas seldom give risk of symptoms (unless large and distal). Distally sited villous lesions may produce mucous and if very large, electrolyte disturbances may occur.

Follow up of colonic polyps
Group Action
Colorectal cancer Colonoscopy 1 year post resection
Large non pedunculated colorectal polyps (LNPCP), R0 resection One off scope at 3 years
Large non pedunculated colorectal polyps (LNPCP) R1 or non en bloc resection Site check at 2-6 months and then a further scope at 12 months
High risk findings at baseline colonoscopy One off surveillance at 3 years
No high risk findings at baseline colonoscopy No colonoscopic surveillance and invite participation in NHSBCSP programme when due

High risk findings
More than 2 premalignant polyps including 1 or more advanced colorectal polyps
OR
More than 5 pre malignant polyps

Exceptions to guidelines
If patient more than 10 years younger than lower screening age and has polyps but no high risk findings, consider colonoscopy at 5 or 10 years.

Segmental resection or complete colectomy should be considered when:

  1. Incomplete excision of malignant polyp
  2. Malignant sessile polyp
  3. Malignant pedunculated polyp with submucosal invasion
  4. Polyps with poorly differentiated carcinoma
  5. Familial polyposis coli
    -Screening from teenager up to 40 years by 2 yearly sigmoidoscopy/colonoscopy
    -Panproctocolectomy and Ileostomy or Restorative Panproctocolectomy.

Rectal polypoidal lesions may be amenable to trans anal endoscopic microsurgery.

References
Rutter MD et al. British Society of Gastroenterology/Association of Coloproctology of Great Britain and Ireland/Public Health England post- polypectomy and post- colorectal cancer resection surveillance guidelines. Gut 2019;0:123.

1197
Q

A 25 year-old lady presents to her GP complaining of a two day history of right upper quadrant pain, fever and a white vaginal discharge. She has seen the GP twice in 12 weeks complaining of pelvic pain and dyspareunia. What is the most likely cause?

Appendicitis

Adnexial torsion

Endometriosis

Pelvic inflammatory disease

Ruptured ectopic pregnancy

A

The most likely diagnosis is pelvic inflammatory disease. Right upper quadrant pain occurs as part of the Fitz Hugh Curtis syndrome in which peri hepatic inflammation occurs.

Gynaecological causes of abdominal pain

A number of women will present with abdominal pain and subsequently be diagnosed with a gynaecological disorder. In addition to routine diagnostic work up of abdominal pain, all female patients should also undergo a bimanual vaginal examination, urine pregnancy test and consideration given to abdominal and pelvic ultrasound scanning.
When diagnostic doubt persists a laparoscopy provides a reliable method of assessing suspected tubulo-ovarian pathology.

Differential diagnoses of abdominal pain in females
Diagnosis Features Investigation Treatment
Mittelschmerz Usually mid cycle pain.
Often sharp onset.
Little systemic disturbance.
May have recurrent episodes.
Usually settles over 24-48 hours. Full blood count- usually normal
Ultrasound- may show small quantity of free fluid Conservative
Endometriosis 25% asymptomatic, in a further 25% associated with other pelvic organ pathology.
Remaining 50% may have menstrual irregularity, infertility, pain and deep dyspareunia.
Complex disease may result in pelvic adhesional formation with episodes of intermittent small bowel obstruction.
Intra-abdominal bleeding may produce localised peritoneal inflammation.
Recurrent episodes are common. Ultrasound- may show free fluid
Laparoscopy will usually show lesions Usually managed medically, complex disease will often require surgery and some patients will even require formal colonic and rectal resections if these areas are involved
Ovarian torsion Usually sudden onset of deep seated colicky abdominal pain.
Associated with vomiting and distress.
Vaginal examination may reveal adnexial tenderness. Ultrasound may show free fluid
Laparoscopy is usually both diagnostic and therapeutic Laparoscopy
Ectopic gestation Symptoms of pregnancy without evidence of intra uterine gestation.
Present as an emergency with evidence of rupture or impending rupture.
Open tubular ruptures may have sudden onset of abdominal pain and circulatory collapse, in other the symptoms may be more prolonged and less marked.
Small amount of vaginal discharge is common.
There is usually adnexial tenderness. Ultrasound showing no intra uterine pregnancy and beta HCG that is elevated
May show intra abdominal free fluid Laparoscopy or laparotomy is haemodynamically unstable. A salphingectomy is usually performed.
Pelvic inflammatory disease Bilateral lower abdominal pain associated with vaginal discharge.
Dysuria may also be present.
Peri-hepatic inflammation secondary to Chlamydia (Fitz Hugh Curtis Syndrome) may produce right upper quadrant discomfort.
Fever >38o Full blood count- Leucocytosis
Pregnancy test negative (Although infection and pregnancy may co-exist)
Amylase - usually normal or slightly raised
High vaginal and urethral swabs Usually medical management

1198
Q

A 56 year old man from Ghana has suffered from recurrent attacks of haematuria for many years. He presents with suprapubic discomfort and at cystoscopy is found to have a mass lesion within the bladder. What is the most likely diagnosis?

Squamous cell carcinoma

Transitional cell papilloma

Adenocarcinoma

Leiomyosarcoma

Rhabdomyosarcoma

A

In Ghana, Schistosomiasis is more common than in the UK and may cause recurrent episodes of haematuria. In those affected with the condition who develop a bladder neoplasm, an SCC is the most common type.

Bladder cancer

Bladder cancer is the second most common urological cancer. It most commonly affects males aged between 50 and 80 years of age. Those who are current, or previous (within 20 years), smokers have a 2-5 fold increased risk of the disease. Exposure to hydrocarbons such as 2-Naphthylamine increases the risk. Although rare in the UK, chronic bladder inflammation arising from Schistosomiasis infection remains a common cause of squamous cell carcinomas, in those countries where the disease is endemic.

Benign tumours
Benign tumours of the bladder including inverted urothelial papilloma and nephrogenic adenoma are uncommon.

Bladder malignancies
Transitional cell carcinoma (>90% of cases)
Squamous cell carcinoma ( 1-7% -except in regions affected by schistosomiasis)
Adenocarcinoma (2%)

Transitional cell carcinomas may arise as solitary lesions, or may be multifocal, owing to the effect of ‘field change’ within the urothelium. Up to 70% of TCC’s will have a papillary growth pattern. These tumours are usually superficial in location and accordingly have a better prognosis. The remaining tumours show either mixed papillary and solid growth or pure solid growths. These tumours are typically more prone to local invasion and may be of higher grade, the prognosis is therefore worse. Those with T3 disease or worse have a 30% (or higher) risk of regional or distant lymph node metastasis.

TNM Staging
Stage Description
T0 No evidence of tumour
Ta Non invasive papillary carcinoma
T1 Tumour invades sub epithelial connective tissue
T2a Tumor invades superficial muscularis propria (inner half)
T2b Tumor invades deep muscularis propria (outer half)
T3 Tumour extends to perivesical fat
T4 Tumor invades any of the following: prostatic stroma, seminal vesicles, uterus, vagina
T4a Invasion of uterus, prostate or bowel
T4b Invasion of pelvic sidewall or abdominal wall
N0 No nodal disease
N1 Single regional lymph node metastasis in the true pelvis (hypogastric, obturator, external iliac, or presacral lymph node)
N2 Multiple regional lymph node metastasis in the true pelvis (hypogastric, obturator, external iliac, or presacral lymph node metastasis)
N3 Lymph node metastasis to the common iliac lymph nodes
M0 No distant metastasis
M1 Distant disease

Presentation
Most patients (85%) will present with painless, macroscopic haematuria. In those patients with incidental microscopic haematuria, up to 10% of females aged over 50 will be found to have a malignancy (once infection excluded).

Staging
Most will undergo a cystoscopy and biopsies or TURBT, this provides histological diagnosis and information relating to depth of invasion. Locoregional spread is best determined using pelvic MRI and distant disease CT scanning. Nodes of uncertain significance may be investigated using PET CT.

Treatment
Those with superficial lesions may be managed using TURBT in isolation. Those with recurrences or higher grade/ risk on histology may be offered intravesical chemotherapy. Those with T2 disease are usually offered either surgery (radical cystectomy and ileal conduit) or radical radiotherapy.

Prognosis
T1 90%
T2 60%
T3 35%
T4a 10-25%
Any T, N1-N2 30%

1199
Q

A 62 year old man presents with dyspepsia and a tumour of the gastric cardia is diagnosed. He has no evidence of metastatic disease. What is the most appropriate treatment option?

Sub total gastrectomy and Roux en Y reconstruction

Total gastrectomy and Roux en Y reconstruction

Total gastrectomy and loop jejunostomy reconstruction

Proximal gastrectomy and gastric pull up

External beam radical radiotherapy

A

This will require a total gastrectomy. Retention of a gastric remnant is unlikely to achieve acceptable resection margins. Resection of the proximal stomach with pull up of the stomach is not standard oncological practice for gastric cancer.

Gastric cancer

Overview
There are 700,000 new cases of gastric cancer worldwide each year. It is most common in Japan and less common in western countries. It is more common in men and incidence rises with increasing age. The exact cause of many sporadic cancer is not known, however, familial cases do occur in HNPCC families. In addition, smoking and smoked or preserved foods increase the risk. Japanese migrants retain their increased risk (decreased in subsequent generations). The distribution of the disease in western countries is changing towards a more proximal location (perhaps due to rising obesity).

Pathology
There is some evidence of support a stepwise progression of the disease through intestinal metaplasia progressing to atrophic gastritis and subsequent dysplasia, through to cancer. The favoured staging system is TNM. The risk of lymph node involvement is related to size and depth of invasion; early cancers confined to submucosa have a 20% incidence of lymph node metastasis. Tumours of the gastro-oesophageal junction are classified as below:

Type 1 True oesophageal cancers and may be associated with Barrett’s oesophagus.
Type 2 Carcinoma of the cardia, arising from cardiac type epithelium
or short segments with intestinal metaplasia at the oesophagogastric junction.
Type 3 Sub cardial cancers that spread across the junction. Involve similar nodal stations to gastric cancer.

Groups for close endoscopic monitoring
Intestinal metaplasia of columnar type
Atrophic gastritis
Low to medium grade dysplasia
Patients who have previously undergone resections for benign peptic ulcer disease (except highly selective vagotomy).

Referral to endoscopy

Patients of any age with dyspepsia and any of the following Patients without dyspepsia Worsening dyspepsia
Chronic gastrointestinal bleeding Dysphagia Barretts oesophagus
Dysphagia Unexplained abdominal pain or weight loss Intestinal metaplasia
Weight loss Vomiting Dysplasia
Iron deficiency anaemia Upper abdominal mass Atrophic gastritis
Upper abdominal mass Jaundice Patient aged over 55 years with unexplained or persistent dyspepsia

Staging
CT scanning of the chest abdomen and pelvis is the routine first line staging investigation in most centres.
Laparoscopy to identify occult peritoneal disease
PET CT (particularly for junctional tumours)

Treatment
Proximally sited disease greater than 5-10cm from the OG junction may be treated by sub total gastrectomy
Total gastrectomy if tumour is <5cm from OG junction
For type 2 junctional tumours (extending into oesophagus) oesophagogastrectomy is usual
Endoscopic sub mucosal resection may play a role in early gastric cancer confined to the mucosa and perhaps the sub mucosa (this is debated)
Lymphadenectomy should be performed. A D2 lymphadenectomy is widely advocated by the Japanese, the survival advantages of extended lymphadenectomy have been debated. However, the overall recommendation is that a D2 nodal dissection be undertaken.
Most patients will receive chemotherapy either pre or post operatively.

Prognosis

UK Data

Disease extent Percentage 5 year survival
All RO resections 54%
Early gastric cancer 91%
Stage 1 87%
Stage 2 65%
Stage 3 18%

Operative procedure

Total Gastrectomy , lymphadenectomy and Roux en Y anastomosis

General anaesthesia
Prophylactic intravenous antibiotics
Incision: Rooftop.
Perform a thorough laparotomy to identify any occult disease.
Mobilise the left lobe of the liver off the diaphragm and place a large pack over it. Insert a large self retaining retractor e.g. omnitract or Balfour (take time with this, the set up should be perfect). Pack the small bowel away.
Begin by mobilising the omentum off the transverse colon.
Proceed to detach the short gastric vessels.
Mobilise the pylorus and divide it at least 2cm distally using a linear cutter stapling device.
Continue the dissection into the lesser sac taking the lesser omentum and left gastric artery flush at its origin.
The lymph nodes should be removed en bloc with the specimen where possible.
Place 2 stay sutures either side of the distal oesophagus. Ask the anaesthetist to pull back on the nasogastric tube. Divide the distal oesophagus and remove the stomach.
The oesphago jejunal anastomosis should be constructed. Identify the DJ flexure and bring a loop of jejunum up to the oesophagus (to check it will reach). Divide the jejunum at this point. Bring the divided jejunum either retrocolic or antecolic to the oesophagus. Anastamose the oesophagus to the jejunum, using either interrupted 3/0 vicryl or a stapling device. Then create the remainder of the Roux en Y reconstruction distally.
Place a jejunostomy feeding tube.
Wash out the abdomen and insert drains (usually the anastomosis and duodenal stump). Help the anaesthetist insert the nasogastric tube (carefully!)
Close the abdomen and skin.
Enteral feeding may commence on the first post-operative day. However, most surgeons will leave patients on free NG drainage for several days and keep them nil by mouth.

1200
Q

A 63 year old man undergoes a radical cystectomy for carcinoma of the bladder. During the procedure there is considerable venous bleeding. What is the primary site of venous drainage of the urinary bladder?

Vesicoprostatic venous plexus

Internal iliac vein

External iliac vein

Gonadal vein

Common iliac vein

A

The urinary bladder has a rich venous plexus surrounding it, this drains subsequently into the internal iliac vein. The vesicoprostatic plexus may be a site of considerable venous bleeding during cystectomy.

Bladder

The empty bladder is contained within the pelvic cavity. It is usually a three sided pyramid. The apex of the bladder points forwards towards the symphysis pubis and the base lies immediately anterior to the rectum or vagina. Continuous with the apex is the median umbilical ligament, during development this was the site of the urachus.
The inferior aspect of the bladder is retroperitoneal and the superior aspect covered by peritoneum. As the bladder distends it will tend to separate the peritoneum from the fascia of transversalis. For this reason a bladder that is distended due to acute urinary retention may be approached with a suprapubic catheter that avoids entry into the peritoneal cavity.
The trigone is the least mobile part of the bladder and forms the site of the ureteric orifices and internal urethral orifice. In the empty bladder the ureteric orifices are approximately 2-3cm apart, this distance may increase to 5cm in the distended bladder.

Arterial supply
The superior and inferior vesical arteries provide the main blood supply to the bladder. These are branches of the internal iliac artery.

Venous drainage
In males the bladder is drained by the vesicoprostatic venous plexus. In females the bladder is drained by the vesicouterine venous plexus. In both sexes this venous plexus will ultimately drain to the internal iliac veins.

Lymphatic drainage
Lymphatic drainage is predominantly to the external iliac nodes, internal iliac and obturator nodes also form sites of bladder lymphatic drainage.

Innervation
Parasympathetic nerve fibres innervate the bladder from the pelvic splanchnic nerves. Sympathetic nerve fibres are derived from L1 and L2 via the hypogastric nerve plexuses. The parasympathetic nerve fibres will typically cause detrusor muscle contraction and result in voiding. The muscle of the trigone is innervated by the sympathetic nervous system. The external urethral sphincter is under conscious control. During bladder filling the rate of firing of nerve impulses to the detrusor muscle is low and receptive relaxation occurs. At higher volumes and increased intra vesical pressures the rate of neuronal firing will increase and eventually voiding will occur.

1201
Q

A 48 year old type 2 diabetic complains of numbness in his left arm and leg. Otherwise there are no other neurological signs. What is the most likely cause?

Middle cerebral artery infarct

Lacunar infarct

Pontine infarct

Anterior cerebral artery infarct

Posterior cerebral artery infarct

A

Isolated hemisensory loss is a feature of a lacunar infarct.

Stroke: types

Primary intracerebral haemorrhage (PICH, c. 10%)
Presents with headache, vomiting, loss of consciousness
Total anterior circulation infarcts (TACI, c. 15%)
Involves middle and anterior cerebral arteries
Hemiparesis/hemisensory loss
Homonymous hemianopia
Higher cognitive dysfunction e.g. Dysphasia
Partial anterior circulation infarcts (PACI, c. 25%)
Involves smaller arteries of anterior circulation e.g. upper or lower division of middle cerebral artery
Higher cognitive dysfunction or two of the three TACI features
Lacunar infarcts (LACI, c. 25%)
Involves perforating arteries around the internal capsule, thalamus and basal ganglia
Present with either isolated hemiparesis, hemisensory loss or hemiparesis with limb ataxia
Posterior circulation infarcts (POCI, c. 25%)
Vertebrobasilar arteries
Presents with features of brainstem damage
Ataxia, disorders of gaze and vision, cranial nerve lesions
Lateral medullary syndrome (posterior inferior cerebellar artery)
Wallenberg’s syndrome
Ipsilateral: ataxia, nystagmus, dysphagia, facial numbness, cranial nerve palsy
Contralateral: limb sensory loss
Weber’s syndrome
Ipsilateral III palsy
Contralateral weakness

Anterior cerebral artery
Contralateral hemiparesis and sensory loss, lower extremity > upper
Disconnection syndrome

Middle cerebral artery
Contralateral hemiparesis and sensory loss, upper extremity > lower
Contralateral hemianopia
Aphasia (Wernicke’s)
Gaze abnormalities

Posterior cerebral artery
Contralateral hemianopia with macular sparing
Disconnection syndrome

Lacunar
Present with either isolated hemiparesis, hemisensory loss or hemiparesis with limb ataxia

Lateral medulla (posterior inferior cerebellar artery)
Ipsilateral: ataxia, nystagmus, dysphagia, facial numbness, cranial nerve palsy e.g.

Horner’s
Contralateral: limb sensory loss

Pontine
VI nerve: horizontal gaze palsy
VII nerve
Contralateral hemiparesis

1202
Q

A 42 year old woman with known multiple gastric ulcers attends the surgical out patient unit. She has not improved despite 2 months of proton pump inhibitor treatment. She is found to have a gastrinoma. Where are these most often located?

Duodenum

Pancreatic head

Pancreatic tail

Jejunum

Gastric antrum

A

Most commonly found in the duodenum (in up to 50% patients), then the pancreas (approximately 20%). Other ectopic areas include stomach, spleen, gallbladder and ovary
Greater than 4/5 of gastrinomas are found within the triangle bounded by:
Cystic and common bile duct (Top)
2nd and 3rd part of the duodenum (Bottom)
Neck and body of pancreas (Medial)
Pancreatic gastrinomas are normally solitary and display malignant behaviour.
Zollinger Ellison syndrome is composed of the triad of:
1. Non beta islet cell tumours of the pancreas
2. Hypergastrinaemia
3. Severe ulcer disease

Clinical features related to peptic ulcer disease. Diagnosis is based on 3 criteria:
1. Fasting hypergastrinaemia
2. Increased basal acid output
3. Secretin stimulation test positive

Management
Resection if localised disease

Pancreatic cancer

  • Adenocarcinoma
    Risk factors: Smoking, diabetes, adenoma, familial adenomatous polyposis
    Mainly occur in the head of the pancreas (70%)
    Spread locally and metastasizes to the liver
    Carcinoma of the pancreas should be differentiated from other periampullary tumours with better prognosis

Clinical features
Weight loss
Painless jaundice
Epigastric discomfort (pain usually due to invasion of the coeliac plexus is a late feature)
Pancreatitis
Trousseau’s sign: migratory superficial thrombophlebitis

Investigations
USS: May miss small lesions
CT Scanning (pancreatic protocol). If unresectable on CT then no further staging needed
PET/CT for those with operable disease on CT alone
ERCP/ MRI for bile duct assessment
Staging laparoscopy to exclude peritoneal disease

Management
Head of pancreas: Whipple’s resection (SE dumping and ulcers). Newer techniques include pylorus preservation and SMA/ SMV resection
Carcinoma body and tail: poor prognosis, distal pancreatectomy, if operable
Usually adjuvent chemotherapy for resectable disease
ERCP and stent for jaundice and palliation
Surgical bypass may be needed for duodenal obstruction

1203
Q

A 18 year old boy falls off a balcony and hits the right side of the head. He is admitted to the emergency department and has a GCS of 10. He is admitted for observation, and over the following twelve hours develops an increasing headache and confusion. A CT scan shows a crescent shaped collection of fluid between the brain and the dura with associated midline shift. What is the most likely diagnosis?

Acute sub dural haematoma

Chronic sub dural haematoma

Acute extra dural haematoma

Chronic extra dural haematoma

Sub arachnoid haemorrhage

A

Sub dural haematomas are the commonest intracranial mass lesions resulting from trauma. They are classified as acute, sub acute or chronic according to tempo of onset. Acute sub dural haematomas will present within 72 hours of the original injury and have hyperdense, crescent shaped appearances on CT scanning.

Head injury

Patients who suffer head injuries should be managed according to ATLS principles and extra cranial injuries should be managed alongside cranial trauma. Inadequate cardiac output will compromise CNS perfusion irrespective of the nature of the cranial injury.

Types of traumatic brain injury
Extradural haematoma Bleeding into the space between the dura mater and the skull. Often results from acceleration-deceleration trauma or a blow to the side of the head. The majority of extradural haematomas occur in the temporal region where skull fractures cause a rupture of the middle meningeal artery.

Features
Raised intracranial pressure
Some patients may exhibit a lucid interval
Subdural haematoma Bleeding into the outermost meningeal layer. Most commonly occur around the frontal and parietal lobes. May be either acute or chronic.

Risk factors include old age and alcoholism.

Slower onset of symptoms than a extradural haematoma.
Subarachnoid haemorrhage Usually occurs spontaneously in the context of a ruptured cerebral aneurysm, but may be seen in association with other injuries when a patient has sustained a traumatic brain injury.

Pathophysiology
Primary brain injury may be focal (contusion/ haematoma) or diffuse (diffuse axonal injury)
Diffuse axonal injury occurs as a result of mechanical shearing following deceleration, causing disruption and tearing of axons
Intra-cranial haematomas can be extradural, subdural or intracerebral, while contusions may occur adjacent to (coup) or contralateral (contre-coup) to the side of impact
Secondary brain injury occurs when cerebral oedema, ischaemia, infection, tonsillar or tentorial herniation exacerbates the original injury. The normal cerebral auto regulatory processes are disrupted following trauma rendering the brain more susceptible to blood flow changes and hypoxia
The Cushings reflex (hypertension and bradycardia) often occurs late and is usually a pre terminal event

Management
Where there is life threatening rising ICP such as in extra dural haematoma and whilst theatre is prepared or transfer arranged use of IV mannitol/ frusemide may be required.
Diffuse cerebral oedema may require decompressive craniotomy
Exploratory Burr Holes have little management in modern practice except where scanning may be unavailable and to thus facilitate creation of formal craniotomy flap
Depressed skull fractures that are open require formal surgical reduction and debridement, closed injuries may be managed non operatively if there is minimal displacement.
ICP monitoring is appropriate in those who have GCS 3-8 and normal CT scan.
ICP monitoring is mandatory in those who have GCS 3-8 and abnormal CT scan.
Hyponatraemia is most likely to be due to syndrome of inappropriate ADH secretion.
Minimum of cerebral perfusion pressure of 70mmHg in adults.
Minimum cerebral perfusion pressure of between 40 and 70 mmHg in children.

Interpretation of pupillary findings in head injuries
Pupil size Light response Interpretation
Unilaterally dilated Sluggish or fixed 3rd nerve compression secondary to tentorial herniation
Bilaterally dilated Sluggish or fixed
Poor CNS perfusion
Bilateral 3rd nerve palsy
Unilaterally dilated or equal Cross reactive (Marcus - Gunn) Optic nerve injury
Bilaterally constricted May be difficult to assess
Opiates
Pontine lesions
Metabolic encephalopathy
Unilaterally constricted Preserved Sympathetic pathway disruption

1204
Q

A 42 year old lady who has systemic lupus erythematosus presents to the clinic with a 5 day history of a painful purple lesion on her index finger. On examination, she has a tender red lesion on the index finger. What is the diagnosis?

Heberdens nodes

Oslers nodes

Bouchards nodes

Malignant fibrous histiocytoma

Osteoclastoma

A

Osler nodes are normally described as tender, purple/red raised lesions with a pale centre. These lesions occur as a result of immune complex deposition. These occur most often in association with endocarditis. However, other causes include SLE, gonorrhoea, typhoid and haemolytic anaemia.

Hand diseases

Dupuytrens contracture
Fixed flexion contracture of the hand where the fingers bend towards the palm and cannot be fully extended.
Caused by underlying contractures of the palmar aponeurosis . The ring finger and little finger are the fingers most commonly affected. The middle finger may be affected in advanced cases, but the index finger and the thumb are nearly always spared.
Progresses slowly and is usually painless. In patients with this condition, the tissues under the skin on the palm of the hand thicken and shorten so that the tendons connected to the fingers cannot move freely. The palmar aponeurosis becomes hyperplastic and undergoes contracture.
Commonest in males over 40 years of age.
Association with liver cirrhosis and alcoholism. However, many cases are idiopathic.
Treatment is surgical and involves fasciectomy. However, the condition may recur and many surgical therapies are associated with risk of neurovascular damage to the digital nerves and arteries.

Carpal tunnel syndrome
Idiopathic median neuropathy at the carpal tunnel.
Characterised by altered sensation of the lateral 3 fingers.
The condition is commoner in females and is associated with other connective tissue disorders such as rheumatoid disease. It may also occur following trauma to the distal radius.
Symptoms occur mainly at night in early stages of the condition.
Examination may demonstrate wasting of the muscles of the thenar eminence and symptoms may be reproduced by Tinels test (compression of the contents of the carpal tunnel).
Formal diagnosis is usually made by electrophysiological studies.
Treatment is by surgical decompression of the carpal tunnel, a procedure achieved by division of the flexor retinaculum. Non - surgical options include splinting and bracing.

Miscellaneous hand lumps
Osler’s nodes Osler’s nodes are painful, red, raised lesions found on the hands and feet. They are the result of the deposition of immune complexes.
Bouchard’s nodes Hard, bony outgrowths or gelatinous cysts on the proximal interphalangeal joints (the middle joints of fingers or toes.) They are a sign of osteoarthritis, and are caused by formation of calcific spurs of the articular cartilage.
Heberden’s nodes Typically develop in middle age, beginning either with a chronic swelling of the affected joints or the sudden painful onset of redness, numbness, and loss of manual dexterity. This initial inflammation and pain eventually subsides, and the patient is left with a permanent bony outgrowth that often skews the fingertip sideways. It typically affects the DIP joint.
Ganglion Swelling in association with a tendon sheath commonly near a joint. They are common lesions in the wrist and hand. Usually they are asymptomatic and cause little in the way of functional compromise. They are fluid filled although the fluid is similar to synovial fluid it is slightly more viscous. When the cysts are troublesome they may be excised.

1205
Q

A 16 year-old boy presents to his GP with loss of weight, pain and fever. On examination, a mass is palpable over the mid-thigh region. What is the most likely diagnosis?

Fibrosarcoma

Osteosarcoma

Ewing sarcoma

Osteoid osteoma

Synovial sarcoma

A

Ewing’s sarcoma is a malignant round cell tumour occurring in the diaphysis of the long bones in the children. These are not confined to the ends of long bones. X- Rays often show a large soft-tissue mass with concentric layers of new bone formation ( ‘onion-peel’ sign). The ESR may be elevated, thus suggesting an inflammatory or an infective cause such as osteomyelitis; although osteomyelitis usually affects the metaphyseal region in children. Treatment is with chemotherapy and surgical excision, an endoprosthesis may be used to conserve the limb.

Sarcomas

Malignant tumours of mesenchymal origin

Types
May be either bone or soft tissue in origin.
Bone sarcoma include:
Osteosarcoma
Ewings sarcoma (although non bony sites recognised)
Chondrosarcoma - originate from Chondrocytes

Soft tissue sarcoma are a far more heterogeneous group and include:
Liposarcoma-adipocytes
Rhabdomyosarcoma-striated muscle
Leiomyosarcoma-smooth muscle
Synovial sarcomas- close to joints (cell of origin not known but not synovium)

Malignant fibrous histiocytoma is now referred to as undifferentiated pleomorphic sarcoma. Careful histological assessment of lesions now allows more accurate categorisation of sarcoma subtypes than was previously possible.

Features
Certain features of a mass or swelling should raise suspicion for a sarcoma these include:
Large >5cm soft tissue mass
Deep tissue location or intra muscular location
Rapid growth
Painful lump

Assessment
Imaging of suspicious masses should utilise a combination of MRI, CT and USS. Blind biopsy should not be performed prior to imaging and where required should be done in such a way that the biopsy tract can be subsequently included in any resection.

Ewings sarcoma
Commoner in males
Incidence of 0.3 / 1, 000, 000
Onset typically between 10 and 20 years of age
Location by femoral diaphysis is commonest site
Histologically it is a small round tumour
Blood borne metastasis is common and chemotherapy is often combined with surgery

Osteosarcoma
Mesenchymal cells with osteoblastic differentiation
20% of all primary bone tumours
Incidence of 5 per 1,000,000
Peak age 15-30, commoner in males
Limb preserving surgery may be possible and many patients will receive chemotherapy

Liposarcoma
Malignancy of adipocytes
Rare, approximately 2.5 per 1,000,000. They are the most common soft tissue sarcoma
Typically located in deep locations such as retroperitoneum
Affect older age group usually >40 years of age
May be well differentiated and thus slow growing although may undergo de-differentiation and disease progression
Many tumours will have a pseudocapsule that can misleadingly allow surgeons to feel that they can ‘shell out’ these lesions. In reality, tumour may invade at the edge of the pseudocapsule and result in local recurrence if this strategy is adopted
Usually resistant to radiotherapy, although this is often used in a palliative setting

Malignant Fibrous Histiocytoma
Tumour with large number of histiocytes
Also described as undifferentiated pleomorphic sarcoma NOS (i.e. Cell of origin is not known)
Four major subtypes are recognised: storiform-pleomorphic (70% cases), myxoid (less aggressive), giant cell and inflammatory
Treatment is usually with surgical resection and adjuvant radiotherapy as this reduces the likelihood of local recurrence

1206
Q

Which of the following is not found on a blood film post splenectomy?

Pappenheimer bodies

Stipple cells

Erythrocyte containing siderotic granules

Howell-Jolly bodies

Target cells

A

Stipple cells are found in lead poisoning/haemoglobinopathies.

Blood film in hyposplenism:

Howell-Jolly bodies
Pappenheimer bodies
Poikilocytes (Target cells)
Erythrocyte containing siderotic granules
Heinz bodies

Post splenectomy blood film changes

The loss of splenic tissue results in the inability to readily remove immature or abnormal red blood cells from the circulation. The red cell count does not alter significantly. However, cytoplasmic inclusions may be seen e.g. Howell-Jolly bodies.
In the first few days after splenectomy target cells, siderocytes and reticulocytes will appear in the circulation. Immediately following splenectomy a granulocytosis (mainly composed of neutrophils) is seen, this is replaced by a lymphocytosis and monocytosis over the following weeks.
The platelet count is usually increased and this may be persistent, oral antiplatelet agents may be needed in some patients.

1207
Q

A 22 year old teacher is admitted with severe epigastric pain. Serum amylase is normal. You wish to exclude a perforated viscus, and determine whether pancreatitis is present. What is the best course of action?

Order an erect CXR and plain abdominal film

Request an abdominal USS

Request a CT scan of the abdomen and pelvis with intravenous contrast

Request a non contrast CT scan of the abdomen and pelvis

Request a magnetic resonance cholangiopancreatography scan

A

A CT scan with IV contrast is needed, without contrast only the perforated viscus question can be answered.

Management of Pancreatitis

Management of Acute Pancreatitis in the UK

Diagnosis
Traditionally hyperamylasaemia has been utilised with amylase being elevated three times the normal range.
However, amylase may give both false positive and negative results.
Serum lipase is both more sensitive and specific than serum amylase. It also has a longer half life.
Serum amylase levels do not correlate with disease severity.

Differential causes of hyperamylasaemia
Acute pancreatitis
Pancreatic pseudocyst
Mesenteric infarct
Perforated viscus
Acute cholecystitis
Diabetic ketoacidosis

Assessment of severity
Glasgow, Ranson scoring systems and APACHE II
Biochemical scoring e.g. using CRP

Features that may predict a severe attack within 48 hours of admission to hospital
Initial assessment
Clinical impression of severity
Body mass index >30
Pleural effusion
APACHE score >8
24 hours after admission
Clinical impression of severity
APACHE II >8
Glasgow score of 3 or more
Persisting multiple organ failure
CRP>150
48 hours after admission
Glasgow Score of >3
CRP >150
Persisting or progressive organ failure
Table adapted from UK guidelines for management of acute pancreatitis. GUT 2005, 54 suppl III

Management

Nutrition
There is reasonable evidence to suggest that the use of enteral nutrition does not worsen the outcome in pancreatitis
Most trials to date were underpowered to demonstrate a conclusive benefit.
The rationale behind feeding is that it helps to prevent bacterial translocation from the gut, thereby contributing to the development of infected pancreatic necrosis.

Use of antibiotic therapy
Many UK surgeons administer antibiotics to patients with acute pancreatitis. However, there is very little evidence to support this practice.
A recent Cochrane review highlights the potential benefits of administering Imipenem to patients with established pancreatic necrosis in the hope of averting the progression to infection.
There are concerns that the administration of antibiotics in mild attacks of pancreatitis will not affect outcome and may contribute to antibiotic resistance and increase the risks of antibiotic associated diarrhoea.

Surgery
Patients with acute pancreatitis due to gallstones should undergo early cholecystectomy.
Patients with obstructed biliary system due to stones should undergo early ERCP.
Patients with extensive necrosis where infection is suspected should usually undergo FNA for culture.
Patients with infected necrosis should undergo either radiological drainage or surgical necrosectomy. The choice of procedure depends upon local expertise.

References
www.bsg.org.uk/pdfworddocs/pancreatic.pdf

Antibiotic therapy for prophylaxis against infection of pancreatic necrosis in acute pancreatitis. Villatoro et al. Cochrane Library DOI: 10.1002/14651858.CD002941.pub3. 2010 version.

1208
Q

A 14-year-old boy is admitted to the acute surgical unit with appendicitis. He is normally fit and well. Apart from metoclopramide, the patient has had no other medications. The nursing staff contact you as the patient is acting strange. On examination he is agitated, has a clenched jaw and his eyes are deviated upwards. What is the most likely diagnosis?

Functional disorder

Malignant hyperthermia

Oculogyric crisis

Epilepsy

Serotonin syndrome

A

This is a classic description of an oculogyric crisis, a form of extrapyramidal disorder. An oculogyric crisis is an acute dystonic reaction. This is precipitated by antipsychotics (haloperidol) and metoclopramide in susceptible individuals with a genetic predisposition to this. Treatment is with procyclidine IM.

Oculogyric crisis

An oculogyric crisis is a dystonic reaction to certain drugs or medical conditions

Features
Restlessness, agitation
Involuntary upward deviation of the eyes

Causes
Phenothiazines
Haloperidol
Metoclopramide
Postencephalitic Parkinson’s disease

Management
Procyclidine

1209
Q

A man sustains a laceration between the base of the little finger and wrist. Several weeks after the injury there is loss of thumb adduction power. Which nerve is most likely to have been injured?

Superficial ulnar nerve

Deep ulnar nerve

Median nerve

Radial nerve

Recurrent branch of median nerve

A

Ulnar nerve injury at wrist

Branches of the ulnar nerve in the wrist and hand
At the wrist the ulnar nerve divides into superficial and deep branches. The superficial branch lies deep to the palmaris brevis. It divides into two; to produce digital nerves, which innervate the skin of the medial third of the palm and the palmar surface of one and a half fingers.
The deep branch arises from the nerve on the flexor retinaculum lateral to the pisiform bone. It passes posteriorly between the abductor and short flexor of the little finger supplying them, and supplying and piercing the opponens digiti minimi near its origin from the flexor retinaculum, turns laterally over the distal surface of the Hook of the Hamate bone. It eventually passes between the two heads of adductor pollicis with the deep palmar arch and ends in the first dorsal interosseous muscle. In the palm the deep branch also innervates the lumbricals and interosseous muscles.

1210
Q

A 25 year old man is admitted with left sided loin pain that radiates to his groin. His investigations demonstrate a 9mm left sided calculus within the proximal ureter. His renal function is normal. What is the most appropriate course of action?

Arrange a percutaneous nephrolithotomy

Arrange extra corporeal shock wave lithotripsy

Review the patient in 4 weeks with KUB x-ray on arrival

Arrange for ureteroscopy and stent insertion

Arrange for open ureteric stone extraction

A

For ureteric stones with a maximum diameter of less than 10mm the first-line treatment is extracorporeal shock wave lithotripsy (ESWL). If ESWL fails or if the stone is impacted in the upper ureter then ureteroscopy can be performed.

Management of renal stones

Urolithiasis will affect up to 15% of the worldwide population. The development of sudden onset loin to groin pain which is colicky in nature is a classic feature in the history. It is nearly always associated with haematuria that is either visible or non-visible but detectable on urine dipstick testing. Where the diagnosis is suspected the most sensitive and specific investigation is helical, non-contrast, computerised tomographic (CT) scanning.

Management
Most renal stones measuring less than 5mm in maximum diameter will typically pass within 4 weeks of symptom onset(1)(2). More urgent treatment is indicated in the presence of ureteric obstruction, renal developmental abnormalities such as horseshoe kidney and previous renal transplant. Ureteric obstruction together with infection is a surgical emergency and the system must be decompressed. Options include nephrostomy tube placement or ureteric stent placement via cystoscopy.

In the non-emergency setting the preferred options for treatment of stone disease include extracorporeal shock wave lithotripsy (ESWL), percutaneous nephrolithotomy (PCNL) and ureteroscopy (URS). These minimally invasive options are the most popular first line treatments. Open surgery remains an option in a few selected cases.

Extracorporeal shock wave lithotripsy
A shock wave is generated external to the patient. Internally, cavitation bubbles and mechanical stress lead to stone fragmentation. The passage of shock waves can occasionally result in the development of solid organ injury. Fragmentation of larger stones may result in the development of ureteric obstruction. The procedure can be uncomfortable for patients and analgesia is often required during the procedure and afterwards. ESWL is contra-indicated in pregnant females and patients with significant vascular calcification.

Ureteroscopy
A ureteroscope is passed retrograde via the urethra and bladder, into the ureter and renal pelvis. Laser or pneumatic fragmentation (lithoclasty) is performed using the ureteroscope and stone fragments are extracted(3). In some cases a stent is left in situ after the procedure.

Percutaneous nephrolithotomy
In this procedure percutaneous access is gained to the renal collecting system. Once access is achieved, endoscopic intra-corporeal lithotripsy, lithoclasty or laser stone fragmentation is performed and stone fragments removed.

Therapeutic selection

Renal stones
Size First line option
Less than 5mm and asymptomatic Watchful waiting
Less than 10mm ESWL
10 20mm ESWL or ureteroscopy
Greater than 20mm (including staghorn calculi) PCNL

Ureteric stones
Size First line option
Less than 5mm Watchful waiting
5-10mm ESWL (if upper ureter)
10-20mm Ureteroscopy

Reference
1. Bultitude M, Rees J. Management of renal colic. BMJ. 2012;345(7872):18.
2. Shah TT, Gao C, Peters M, Manning T, Cashman S, Nambiar A, et al. Factors associated with spontaneous stone passage in a contemporary cohort of patients presenting with acute ureteric colic: results from the Multi-centre cohort study evaluating the role of Inflammatory Markers In patients presenting with acute ureteric . BJU Int. 2019 Apr.
3. BAUS. Ureteroscopy (Telescopic surgery for stone removal) (Internet). BAUS patient information leaflet. 2017 (cited 2019 Jun 27). p. 16. Available from: https://www.baus.org.uk/userfiles/pages/files/Patients/Leaflets/Ureteroscopy for stone.pdf
4. BAUS. Percutaneous nephrolithotomy (keyhole surgery for kidney stones). BAUS patient information leaflet. 2017. p. 16. Available from: https://www.baus.org.uk/userfiles/pages/files/Patients/Leaflets/PCNL.pdf
5. NICE. Renal and ureteric stones: assessment and management. NICE Guide[NG118]. 2019;(January):112.

1211
Q

A 23 year old man is readmitted following a difficult appendicectomy. His wound is erythematous and, on incision, foul smelling pus is drained. Which of the organisms listed below is responsible?

Steptococcus bovis

Bacteroides fragilis

Staphylococcus aureus

Streptococcus pyogenes

Clostridium perfringens

A

Bacteroides is commonly present in severe peritoneal infections and as it is facultatively anaerobic may be present in pus. It has a pungent aroma.

Surgical Microbiology

An extensive topic so an overview is given here. Organisms causing common surgical infections are reasonable topics in the examination. However, microbiology is less rigorously tested than anatomy, for example.

Common organisms

Staphylococcus aureus
Facultative anaerobe
Gram positive coccus
Haemolysis on blood agar plates
Catalase positive
20% population are long term carriers
Exo and entero toxin may result in toxic shock syndrome and gastroenteritis respectively
Ideally treated with penicillin although many strains now resistant through beta Lactamase production. In the UK less than 5% of isolates are sensitive to penicillin.
Resistance to methicillin (and other antibiotics) is mediated by the mec operon , essentially penicillin binding protein is altered and resistance to this class of antibiotics ensues
Common cause of cutaneous infections and abscesses

Streptococcus pyogenes
Gram positive, forms chain like colonies, Lancefield Group A Streptococcus
Produces beta haemolysis on blood agar plates
Rarely part of normal skin microflora
Catalase negative
Releases a number of proteins/ virulence factors into host including hyaluronidase, streptokinase which allow rapid tissue destruction
Releases superantigens such as pyogenic exotoxin A which results in scarlet fever
Remains sensitive to penicillin, macrolides may be used as an alternative.

Escherichia coli
Gram negative rod
Facultative anaerobe, non sporing
Wide range of subtypes and some are normal gut commensals
Some subtypes such as 0157 may produce lethal toxins resulting in haemolytic-uraemic syndrome
Enterotoxigenic E-Coli produces an enterotoxin (ST enterotoxin) that results in large volume fluid secretion into the gut lumen (Via cGMP activation)
Enteropathogenic E-Coli binds to intestinal cells and cause structural damage, this coupled with a moderate (or in case of enteroinvasive E-Coli significant) invasive component produces enteritis and large volume diarrhoea together with fever.
They are resistant to many antibiotics used to treat gram positive infections and acquire resistance rapidly and are recognised as producing beta lactamases

Campylobacter jejuni
Curved, gram negative, non sporulating bacteria
One of the commonest causes of diarrhoea worldwide
Produces enteritis which is often diffuse and blood may be passed
Remains a differential for right iliac fossa pain with diarrhoea
Self limiting infection so antibiotics are not usually advised. However, the quinolones are often rapidly effective.

Helicobacter pylori
Gram negative, helix shaped rod, microaerophillic
Produces hydrogenase that can derive energy from hydrogen released by intestinal bacteria
Flagellated and mobile
Those carrying the cag A gene may cause ulcers
It secretes urease that breaks down gastric urea> Carbon dioxide and ammonia> ammonium>bicarbonate (simplified!) The bicarbonate can neutralise the gastric acid.
Usually colonises the gastric antrum and irritates resulting in increased gastrin release and higher levels of gastric acid. These patients will develop duodenal ulcers. In those with more diffuse H-Pylori infection gastric acid levels are lower and ulcers develop by local tissue damage from H-Pylori- these patients get gastric ulcers.
Diagnosis may be made by serology (approx. 75% sensitive). Biopsy urease test during endoscopy probably the most sensitive.
In patients who are colonised 10-20% risk of peptic ulcer, 1-2% risk gastric cancer, <1% risk MALT lymphoma.

1212
Q

A 65 year old man presents with significant lower urinary tract symptoms and is diagnosed as having benign prostatic hyperplasia. Which of the following drug treatments will produce the slowest clinical response?

Tamsulosin

Alfuzosin

Doxazosin

Finasteride

Terazosin

A

5 alpha reductase inhibitors have a more favorable side effect profile than α blockers.
Alpha blockers have a faster onset of action (but lower reduction of complications from BPH) than 5 α reductase inhibitors.

Benign Prostatic Hyperplasia

Benign prostatic hyperplasia occurs via an increase in the epithelial and stromal cell numbers in the peri-urethral zone of the prostate. BPH is very common and 90% of men aged over 80 will have at least microscopic evidence of benign prostatic hyperplasia. The causes of BPH are still not well understood, but the importance of androgens remains appreciated even if the exact role by which they induce BPH is elusive.

Presentation
The vast majority of men will present with lower urinary tract symptoms. These will typically be:

Poor flow
Nocturia
Hesitancy
Incomplete and double voiding
Terminal dribbling
Urgency
Incontinence

Investigation
Digital rectal examination to assess prostatic size and morphology.
Urine dipstick for infections and haematuria.
Uroflowmetry (a flow rate of >15ml/second helps to exclude BOO)
Bladder pressure studies may help identify detrusor failure and whilst may not form part of first line investigations should be included in those with atypical symptoms and prior to redo surgery.
Bladder scanning to demonstrate residual volumes. USS if high pressure chronic retention.

Management
Lifestyle changes such as stopping smoking and altering fluid intake may help those with mild symptoms.
Medical therapy includes alpha blockers and 5 α reductase inhibitors. The former work quickly on receptor zones located at the bladder neck. Cardiovascular side effects are well documented. The latter work on testosterone metabolising enzymes. Although they have a slower onset of action, the 5 α reductase inhibitors may prevent acute urinary retention.
Surgical therapy includes transurethral resection of the prostate and is the treatment of choice in those with severe symptoms and those who fail to respond to medical therapy. More tailored bladder neck incision procedures may be considered in those with small prostates. Retrograde ejaculation may occur following surgery. The change in the type of irrigation solutions used has helped to minimise the TURP syndrome of electrolyte disturbances.

1213
Q

A 73 year old lady is diagnosed with hyperaldosteronism. From which of the following structures is aldosterone released?

Zona fasciculata of the adrenal gland

Juxtaglomerular apparatus of the kidney

Zona reticularis of the adrenal gland

Adrenal medulla

Zona glomerulosa of the adrenal cortex

A

Aldosterone serves to conserve sodium and water. It is produced in the zona glomerulosa of the adrenal cortex.

Aldosterone

Aldosterone is secreted by the zona glomerulosa of the adrenal cortex. It is a mineralocorticoid hormone. Secretion is regulated by the renin- angiotensin system, and by plasma levels of sodium and potassium. Aldosterone conserves sodium by stimulating the reabsorption of sodium in the distal nephron in exchange for potassium. Lack of aldosterone release will result in hyperkalaemia and hyponatraemia.

1214
Q

A 34 year old man is involved in the road traffic accident and suffers bilateral tibial fractures and a ruptured spleen. He undergoes an emergency laparotomy and splenectomy soon after the incident. A blood test is taken in the recovery room and the following results are obtained:
Hb 91
WCC 5.6
Platelets 230
Na 132
K 6.5
Urea 6.1
Creatinine 83
What is the most appropriate course of action?

CT scan of the abdomen

Measurement of abdominal compartment pressures

Measurement of leg compartment pressures

Re-opening the abdomen

Insertion of a nephrostomy

A

It is likely that the patient has developed lower leg compartment syndrome and as cell death occurs, hyperkalaemia results. The time taken for the laparotomy would have contributed to this event and provided a distraction. It is less likely that the patient will have developed intra abdominal compartment syndrome in this time frame.

Compartment syndrome

  • This is a particular complication that may occur following fractures (or following ischaemia re-perfusion injury in vascular patients). It is characterised by raised pressure within a closed anatomical space.
    The raised pressure within the compartment will eventually compromise tissue perfusion resulting in necrosis. The two main fractures carrying this complication include supracondylar fractures and tibial shaft injuries.

Symptoms and signs
Pain, especially on movement (even passive)
Parasthesiae
Pallor may be present
Arterial pulsation may still be felt as the necrosis occurs as a result of microvascular compromise
Paralysis of the muscle group may occur

Diagnosis
Is made by measurement of intracompartmental pressure measurements. Pressures in excess of 20mmHg are abnormal and >40mmHg is diagnostic.

Treatment
This is essentially prompt and extensive fasciotomies
In the lower limb the deep muscles may be inadequately decompressed by the inexperienced operator when smaller incisions are performed
Myoglobinuria may occur following fasciotomy and result in renal failure and for this reason these patients require aggressive IV fluids
Where muscle groups are frankly necrotic at fasciotomy they should be debrided and amputation may have to be considered
Death of muscle groups may occur within 4-6 hours

1215
Q

A 56 year old man has undergone a radical nephrectomy. The pathologist bisects the kidney and identifies a pink fleshy tumour in the renal pelvis. What is the most likely disease?

Renal cell carcinoma

Transitional cell carcinoma

Angiomyolipoma

Phaeochromocytoma

Renal adenoma

A

Most renal tumours are yellow or brown in colour. TCC’s are one of the few tumours to appear pink.
The finding of a TCC in the renal pelvis mandates a nephroureterectomy.

Renal lesions

Lesion Disease specific features Treatment
Renal cell carcinoma
Most present with haematuria (50%)
Common renal tumour (85% cases)
Paraneoplastic features include hypertension and polycythaemia
Most commonly has haematogenous mestastasis
Usually radical or partial nephrectomy
Nephroblastoma
Rare childhood tumour
It accounts for 80% of all genitourinary malignancies in those under the age of 15 years
Up to 90% will have a mass
50% will be hypertensive
Diagnostic work up includes ultrasound and CT scanning
Surgical resection combined with chemotherapy (usually vincristine, actinomycin D and doxorubicin)
Neuroblastoma
Most common extracranial tumour of childhood
80% occur in those under 4 years of age
Tumour of neural crest origin (up to 50% occur in the adrenal gland)
The tumour is usually calcified and may be diagnosed using MIBG scanning
Staging is with CT
Surgical resection, radiotherapy and chemotherapy
Transitional cell carcinoma
Accounts for 90% of lower urinary tract tumours, but only 10% of renal tumours
Males affected 3x more than females
Occupational exposure to industrial dyes and rubber chemicals may increase risk
Up to 80% present with painless haematuria
Diagnosis and staging is with CT IVU
Radical nephroureterectomy
Angiomyolipoma
80% of these hamartoma type lesions occur sporadically, the remainder are seen in those with tuberous sclerosis
Tumour is composed of blood vessels, smooth muscle and fat
Massive bleeding may occur in 10% of cases
50% of patients with lesions >4cm will have symptoms and will require surgical resection

1216
Q

A 22 year female who is 24 weeks pregnant presents with brisk frank haematuria. She is sexually active. She has had a previous pregnancy resulting in caesarean section. What is the most likely cause?

Renal vein thrombosis

Placenta percreta

Placenta praevia

Membranous glomerulonephritis

Endometriosis

A

Pregnancy and frank haematuria, especially if there is a history of placenta previa or prior caesarean section, should indicate this diagnosis. There is invasive placental implantation into the myometrium, which can rarely extend into the bladder causing severe bleeding.

Haematuria

Causes of haematuria

Trauma
Injury to renal tract
Renal trauma commonly due to blunt injury (others penetrating injuries)
Ureter trauma rare: iatrogenic
Bladder trauma: due to RTA or pelvic fractures
Infection
Remember TB
Malignancy
Renal cell carcinoma (remember paraneoplastic syndromes): painful or painless
Urothelial malignancies: 90% are transitional cell carcinoma, can occur anywhere along the urinary tract. Painless haematuria.
Squamous cell carcinoma and adenocarcinoma: rare bladder tumours
Prostate cancer
Penile cancers: SCC
Renal disease
Glomerulonephritis
Stones
Microscopic haematuria common
Structural abnormalities
Benign prostatic hyperplasia (BPH) causes haematuria due to hypervascularity of the prostate gland
Cystic renal lesions e.g. polycystic kidney disease
Vascular malformations
Renal vein thrombosis due to renal cell carcinoma
Coagulopathy
Causes bleeding of underlying lesions
Drugs
Cause tubular necrosis or interstitial nephritis: aminoglycosides, chemotherapy
Interstitial nephritis: penicillin, sulphonamides, and NSAIDs
Anticoagulants
Benign
Exercise
Gynaecological
Endometriosis: flank pain, dysuria, and haematuria that is cyclical
Iatrogenic
Catheterisation
Radiotherapy; cystitis, severe haemorrhage, bladder necrosis
Pseudohaematuria For example following consumption of beetroot

References
Http://bestpractice.bmj.com/best-practice/monograph/316/overview/aetiology.html

1217
Q

Which of the following best accounts for the mechanism of action of glucocorticoids?

Binding of cell wall receptors and intracellular tyrosine kinase activation

Binding of intracellular receptors that migrate to the nucleus to then affect gene transcription

Activation of transmembrane tyrosine kinase systems to affect intranuclear gene transcription

Induces post translational modification of intracellular proteins

Direct binding of inflammatory cells inducing apoptosis

A

Glucocorticoids exert their effects by binding intracellular receptors that are then transported to the nucleus where they affect gene transcription. There are some questions in the MRCS now that seem to test common pharmacology and so we have decided to include this. A detailed knowledge of the mechanisms by which these effects occur is not needed.

Corticosteroids

Synthesised from cholesterol within the adrenal cortex
Bind to specific intracellular glucocorticoid receptors located on the nucleus

Effects of glucocorticoids
Metabolic
Decreased uptake and utilisation of glucose, increased gluconeogenesis and hyperglycaemia
Increased protein catabolism
Permissive effect on lipolytic hormones

Regulatory actions
Has a negative feedback action on the hypothalamus causing reduced release of endogenous glucocorticoids
Within the CVS they cause decreased vasodilation and decreased fluid exudation
They decrease osteoblastic activity and increase osteoclastic activity
Decrease acute and chronic inflammation (decrease in influx and activity of leukocytes), decreased clonal expansion of B and T lymphocytes.

1218
Q

An 83 year old man is admitted for an abdomino-perineal excision of the colon and rectum for a distal rectal tumour. His co-morbidities include diabetes. His renal function is normal. What is the best form of thromboprophylaxis?

None

Compression stockings alone for 4 weeks

Daily low dose low molecular weight heparin for 4 weeks

Daily high dose low molecular weight heparin for 4 weeks

Daily administration of high dose unfractionated heparin until discharge

A

There is good evidence to support the use of extended thromboprophylaxis after pelvic cancer surgery. Don’t confuse low dose and high dose low molecular weight heparins, re read the options if you got the answer wrong.

Thromboprophylaxis in surgical patients

Deep vein thrombosis may develop insidiously in many surgical patients. Untreated it may progress to result in pulmonary embolism.
The following surgical patients are at increased risk of deep vein thrombosis:
Surgery greater than 90 minutes at any site or greater than 60 minutes if the procedure involves the lower limbs or pelvis
Acute admissions with inflammatory process involving the abdominal cavity
Expected significant reduction in mobility
Age over 60 years
Known malignancy
Thrombophilia
Previous thrombosis
BMI >30
Taking hormone replacement therapy or the contraceptive pill
Varicose veins with phlebitis

Mechanical thromboprophylaxis
Early ambulation after surgery is cheap and is effective
Compression stockings (contra -indicated in peripheral arterial disease)
Intermittent pneumatic compression devices
Foot impulse devices

Therapeutic agents
Agent Mode of action Uses
Low molecular weight heparin Binds antithrombin causing inhibition of factor Xa Thromboprophylaxis or treatment of thromboembolic events in those with normal renal function. It is given as once daily subcutaneous injection
Unfractionated heparin Binds antithrombin III affecting thrombin and factor Xa Effective anticoagulation, administered intravenously it has a rapid onset and its therapeutic effects decline quickly on stopping and infusion. Its activity is measured using the APTT. If need be it can be reversed using protamine sulphate
Dabigatran Orally administered direct thrombin inhibitor Used prophylaxis in hip and knee surgery. It does not require therapeutic monitoring. It should not be used in any patient in whom there is a risk of active bleeding or imminent likelihood of surgery. It is reversed using Idarucizumab

1219
Q

A 53 year old woman presents with a bloody nipple discharge. On mammography, there is calcification behind the nipple areolar complex. A core biopsy shows background benign change, but cells that show comedo necrosis which have not breached the basement membrane are also seen. Which of the lesions below is most likely?

Fibrocystic disease of the breast

Ductal carcinoma in situ

Invasive ductal carcinoma

Atypical ductal hyperplasia

Lobular carcinoma in situ

A

Comedo necrosis is a feature of high nuclear grade ductal carcinoma in situ. It has a high risk of being associated with foci of invasion.

Breast cancer

  • Commoner in the older age group
    Invasive ductal carcinomas are the most common type. Some may arise as a result of ductal carcinoma in situ (DCIS). There are associated carcinomas of special type e.g. Tubular that may carry better prognosis.
    The pathological assessment involves assessment of the tumour and lymph nodes, sentinel lymph node biopsy is often used to minimise the morbidity of an axillary dissection.
    Treatment, typically this is either wide local excision or mastectomy. There are many sub types of both of these that fall outside of the MRCS. Some key rules to bear in mind.
    Whatever operation is contemplated the final cosmetic outcome does have a bearing. A woman with small breasts and a large tumour will tend to fare better with mastectomy, even if clear pathological and clinical margins can be obtained. Conversely a women with larger breasts may be able to undergo breast conserving surgery even with a relatively large primary lesion (NB tumours >4cm used to attract recommendation for mastectomy). For screen detected and impalpable tumour image guidance will be necessary.
    Reconstruction is always an option following any resectional procedure. However, its exact type must be tailored to age and co-morbidities of the patient. The main operations in common use include latissimus dorsi myocutaneous flap and sub pectoral implants. Women wishing to avoid a prosthesis may be offered TRAM or DIEP flaps.

Surgical options
Mastectomy vs Wide local excision

Mastectomy Wide Local Excision
Multifocal tumour Solitary lesion
Central tumour Peripheral tumour
Large lesion in small breast Small lesion in large breast
DCIS >4cm DCIS <4cm
Patient Choice Patient choice

Central lesions may be managed using breast conserving surgery where an acceptable cosmetic result may be obtained, this is rarely the case in small breasts

Whatever surgical option is chosen the aim should be to have a local recurrence rate of 5% or less at 5 years [1].

Nottingham Prognostic Index
The Nottingham Prognostic Index can be used to give an indication of survival. In this system the tumour size is weighted less heavily than other major prognostic parameters.

Calculation of NPI
Tumour Size x 0.2 + Lymph node score(From table below)+Grade score(From table below).

Score Lymph nodes involved Grade
1 0 1
2 1-3 2
3 >3 3

Prognosis

Score Percentage 5 year survival
2.0 to 2.4 93%
2.5 to 3.4 85%
3.5 to 5.4 70%
>5.4 50%

This data was originally published in 1992. It should be emphasised that other factors such as vascular invasion and receptor status also impact on survival and are not included in this data and account for varying prognoses often cited in the literature.

References
For guidance on how breast cancer is managed in the United Kingdom visit the Association of Breast Surgery website (www.https://associationofbreastsurgery.org.uk/).

1220
Q

A male infant is born at 28 weeks gestation by emergency cesarean section. He is taken to theatre for a colostomy due to an imperforate anus. He initially seems to be progressing well. However, he begins to develop decerebrate posturing and is becoming increasingly obtunded. What is the most likely underlying problem?

Extra dural haematoma

Sub arachnoid haemorrhage

Acute sub dural haemorrhage

Intraventricular haemorrhage

Chronic sub dural haematoma

A

Acute neurological deterioration in premature neonates is usually due to intraventricular haemorrhage. Diagnosis is made by cranial ultrasound. Development of hydrocephalus may necessitate surgery.

Intra cranial haemorrhage

Extradural haematoma Bleeding into the space between the dura mater and the skull. Often results from acceleration-deceleration trauma or a blow to the side of the head. The majority of extradural haematomas occur in the temporal region where skull fractures cause a rupture of the middle meningeal artery.

Features
Raised intracranial pressure
Some patients may exhibit a lucid interval
Subdural haematoma Bleeding into the outermost meningeal layer. Most commonly occur around the frontal and parietal lobes. May be either acute or chronic.

Risk factors include old age and alcoholism.

Slower onset of symptoms than a extradural haematoma.
Intracerebral haematoma Usually hyperdense lesions on CT scanning. Arise in areas of traumatic contusion which fuse to become a haematoma. Areas of clot and fresh blood may co-exist on the same CT scan (Swirl sign). Large haematomas and those associated with mass effect should be evacuated.
Subarachnoid haemorrhage Usually occurs spontaneously in the context of a ruptured cerebral aneurysm but may be seen in association with other injuries when a patient has sustained a traumatic brain injury
Intraventricular haemorrhage Haemorrhage that occurs into the ventricular system of the brain. It is relatively rare in adult surgical practice and when it does occur, it is typically associated with severe head injuries. In premature neonates it may occur spontaneously. The blood may clot and occlude CSF flow, hydrocephalus may result.
In neonatal practice the vast majority of IVH occur in the first 72 hours after birth, the aetiology is not well understood and it is suggested to occur as a result of birth trauma combined with cellular hypoxia, together with the delicate neonatal CNS.

1221
Q

A wheelchair bound 78 year old woman with ischaemic heart disease secondary to long smoking history and longstanding type II diabetes presents with rest pain and a non healing ulcer on the dorsum of her foot. Angiogram shows reasonable superficial femoral artery and iliacs. At the level of the popliteal artery there is an occlusion. Below this there is a short area of patent posterior tibial artery and this reconstitutes lower down the leg to flow to the foot. What is the best treatment option?

Long sub intimal angioplasty

Femoro-distal bypass graft with PTFE

Above knee amputation

Below knee amputation

Axillo-femoral bypass

A

A femoro-distal bypass graft would carry a high risk of failure and risk of peri-operative myocardial infarct. This lady would be well suited to primary amputation as she is not ambulant.

Peripheral vascular disease

Indications for surgery to revascularise the lower limb
Intermittent claudication
Critical ischaemia
Ulceration
Gangrene

Intermittent claudication that is not disabling may provide a relative indication, whilst the other complaints are often absolute indications depending upon the frailty of the patient.

Assessment
Clinical examination
Ankle brachial pressure index measurement
Duplex arterial ultrasound
Angiography (standard, CT or MRI): usually performed only if intervention being considered.

Angioplasty
In order for angioplasty to be undertaken successfully the artery has to be accessible. The lesion relatively short and reasonable distal vessel runoff. Longer lesions may be amenable to sub-intimal angioplasty.

Surgery
Surgery will be undertaken where attempts at angioplasty have either failed or are unsuitable. Bypass essentially involves bypassing the affected arterial segment by utilising a graft to run from above the disease to below the disease. As with angioplasty good runoff improves the outcome.

Some key concepts with bypass surgery

Superficial femoral artery occlusion to the above knee popliteal
In the ideal scenario, vein (either in situ or reversed LSV) would be used as a conduit. However, prosthetic material has reasonable 5 year patency rates and some would advocate using this in preference to vein so that vein can be used for other procedures in the future. In general terms either technique is usually associated with an excellent outcome (if run off satisfactory).

Procedure
Artery dissected out, IV heparin 3,000 units given and then the vessels are cross clamped
Longitudinal arteriotomy
Graft cut to size and tunneled to arteriotomy sites
Anastomosis to femoral artery usually with 5/0 ‘double ended’ Prolene suture
Distal anastomosis usually using 6/0 ‘double ended’ Prolene

Distal disease
Femoro-distal bypass surgery takes longer to perform, is more technically challenging and has higher failure rates.
In elderly diabetic patients with poor runoff a primary amputation may well be a safer and more effective option. There is no point in embarking on this type of surgery in patients who are wheelchair bound.
In femorodistal bypasses vein gives superior outcomes to PTFE.

Rules
Vein mapping 1st to see whether there is suitable vein (the preferred conduit). Sub intimal hyperplasia occurs early when PTFE is used for the distal anastomosis and will lead to early graft occlusion and failure.
Essential operative procedure as for above knee fem-pop.
If there is insufficient vein for the entire conduit then vein can be attached to the end of the PTFE graft and then used for the distal anastomosis. This type of ‘vein boot’ is technically referred to as a Miller Cuff and is associated with better patency rates than PTFE alone.
Remember the more distal the arterial anastomosis the lower the success rate.

References
Peach G et al. Diagnosis and management of peripheral arterial disease. BMJ 2012; 345: 36-41.

1222
Q

A 23 year old man is due to undergo a mitral valve repair for mitral regurgitation. Which of the following is a feature of the mitral valve?

Its closure is marked by the first heart sound

It has two anterior cusps

The chordae tendinae attach to the anterior cusps only

The chordae tendinae anchor the valve directly to the wall of the left ventricle

It is best auscultated in the left third interspace

A

The mitral valve is best auscultated over the cardiac apex, where its closure marks the first heart sound. It has only two cusps. These are attached to chordae tendinae which themselves are linked to the wall of the ventricle by the papillary muscles.

Heart anatomy

The walls of each cardiac chamber comprise:
Epicardium
Myocardium
Endocardium

Cardiac muscle is attached to the cardiac fibrous skeleton.

Relations
The heart and roots of the great vessels within the pericardial sac are related to the posterior aspect of the sternum, medial ends of the 3rd to 5th ribs on the left and their associated costal cartilages. The heart and pericardial sac are situated obliquely two thirds to the left and one third to the right of the median plane.

The pulmonary valve lies at the level of the left third costal cartilage.
The mitral valve lies at the level of the fourth costal cartilage.

Coronary sinus
This lies in the posterior part of the coronary groove and receives blood from the cardiac veins. The great cardiac vein lies at its left and the middle and small cardiac veins lie on its right. The smallest cardiac vein (anterior cardiac vein) drains into the right atrium directly.

Aortic sinus
Right coronary artery arises from the right aortic sinus, the left is derived from the left aortic sinus, which lies posteriorly.

Features of the left ventricle as opposed to the right

Structure Left Ventricle
A-V Valve Mitral (double leaflet)
Walls Twice as thick as right
Trabeculae carnae Much thicker and more numerous

Right coronary artery
The RCA supplies:
Right atrium
Diaphragmatic part of the right ventricle
Usually the posterior third of the interventricular septum
The sino atrial node (60% cases)
The atrio ventricular node (80% cases)

Left coronary artery
The LCA supplies:
Left atrium
Most of left ventricle
Part of the right ventricle
Anterior two thirds of the inter ventricular septum
The sino atrial node (remaining 40% cases)

Innervation of the heart
Autonomic nerve fibres from the superficial and deep cardiac plexus. These lie anterior to the bifurcation of the trachea, posterior to the ascending aorta and superior to the bifurcation of the pulmonary trunk. The parasympathetic supply to the heart is from presynaptic fibres of the vagus nerves.

Valves of the heart
Mitral valve Aortic valve Pulmonary valve Tricuspid valve
2 cusps 3 cusps 3 cusps 3 cusps
First heart sound Second heart sound Second heart sound First heart sound
1 anterior cusp 2 anterior cusps 2 anterior cusps 2 anterior cusps
Attached to chordae tendinae No chordae No chordae Attached to chordae tendinae

1223
Q

A 60 year old man is brought to the emergency department collapsed. He is found to be in a coma and imaging of the CNS shows an intracerebral haemorrhage. On the intensive care unit consideration is given as to whether he has brain stem death. Which of the criteria below form part of the testing for brain stem death?

Hypothermia

Absence of pupillary reflexes

Hypoxia on disconnection from the ventilator

Positive brachialis reflex

Positive knee jerk reflex

A

The criteria for brainstem death testing are pre-requisites which must be met before testing can occur, these include excluding hypothyroidism, presence of sedatives, hypothermia. The tests themselves comprise:
a torch is shone into both eyes to see if they react to the light
the eye, is stroked with a tissue or piece of cotton wool to see if it reacts
pressure is applied to the forehead and the nose is pinched to see if there’s any movement in response
ice-cold water is inserted into each ear, which would usually cause the eyes to move
a thin plastic tube is placed down the windpipe to see if it provokes gagging or coughing
the person is disconnected from the ventilator for a short period of time to see if they make any attempt to breathe on their own, they are pre-oxygenated to avoid hypoxia

Brain stem death testing

Conditions for brainstem death testing
1. There must be an identifiable pathology causing irremediable brain damage. This may be intra - or extra - cranial.

  1. The patient must be deeply unconscious.
    a. Hypothermia must be excluded as the cause of unconsciousness and the patients core temperature should be over 34oC.
    b. There should be no evidence that the patients state is due to depressant drugs. This refers to narcotics, hypnotics and tranquillisers as well as neuromuscular blocking drugs. A
    careful drug history is required, whilst drug levels and antagonists may need to be used.
    c. Potentially reversible circulatory, metabolic and endocrine disturbances must have been
    excluded as the cause of the continuing unconsciousness. Some of these disturbances may occur as a result of the condition rather than the cause and these do not preclude the diagnosis of brain stem death.
  2. The patient must be apnoeic, needing mechanical ventilation. This condition must not be secondary to the effect of sedative drugs of neuromuscular blockade. This may require testing with a nerve stimulator to show intact neuromuscular transmission. Alternatively, demonstration of tendon reflexes can also demonstrate intact transmission

Criteria for brain death
Fixed pupils which do not respond to sharp changes in the intensity of incident light
No corneal reflex
Absent oculo-vestibular reflexes - no eye movements following the slow injection of at least 50ml of ice-cold water into each ear in turn (the caloric test)
No response to supraorbital pressure
No cough reflex to bronchial stimulation or gagging response to pharyngeal stimulation
No observed respiratory effort in response to disconnection of the ventilator for long enough (typically 5 minutes) to ensure elevation of the arterial partial pressure of carbon dioxide to at least 6.0 kPa (6.5 kPa in patients with chronic carbon dioxide retention). Adequate oxygenation is ensured by pre-oxygenation and diffusion oxygenation during the disconnection (so the brain stem respiratory centre is not challenged by the ultimate, anoxic, drive stimulus)

The test should be undertaken by two appropriately experienced doctors on two separate occasions.

1224
Q

A 55 year old man with a long history of achalasia is successfully treated by a Hellers Cardiomyotomy. Several years later he develops an oesophageal malignancy. Which of the following lesions is most likely to be present?

Adenocarcinoma

Gastrointestinal stromal tumour

Leiomyosarcoma

Rhabdomyosarcoma

Squamous cell carcinoma

A

Achalasia is a rare condition. However, even once treated there is an increased risk of malignancy. When it does occur it is most likely to be of squamous cell type.

Oesophageal cancer

  • Incidence is increasing
    In most cases in the Western world this increase is accounted for by a rise in the number of cases of adenocarcinoma. In the UK adenocarcinomas account for 65% of cases.
    Barretts oesophagus is a major risk factor for most cases of oesophageal adenocarcinoma.
    In other regions of the world squamous cancer is more common and is linked to smoking, alcohol intake, diets rich in nitrosamines and achalasia.
    Surveillance of Barretts is important, as it imparts a 30 fold increase in cancer risk and if invasive malignancy is diagnosed early then survival may approach 85% at 5 years.

Diagnosis
Upper GI endoscopy is the first line test
Contrast swallow may be of benefit in classifying benign motility disorders but has no place in the assessment of tumours
Staging is initially undertaken with CT scanning of the chest, abdomen and pelvis. If overt metastatic disease is identified using this modality then further complex imaging is unnecessary
If CT does not show metastatic disease, then local stage may be more accurately assessed by use of endoscopic ultrasound.
Staging laparoscopy is performed to detect occult peritoneal disease. PET CT is performed in those with negative laparoscopy. Thoracoscopy is not routinely performed.

Treatment
Operable disease is best managed by surgical resection. The most standard procedure is an Ivor- Lewis type oesophagectomy. This procedure involves the mobilisation of the stomach and division of the oesophageal hiatus. The abdomen is closed and a right sided thoracotomy performed. The stomach is brought into the chest and the oesophagus mobilised further. An intrathoracic oesophagogastric anastomosis is constructed. Alternative surgical strategies include a transhiatal resection (for distal lesions), a left thoraco-abdominal resection (difficult access due to thoracic aorta) and a total oesophagectomy (McKeown) with a cervical oesophagogastric anastomosis.
The biggest surgical challenge is that of anastomotic leak, with an intrathoracic anastomosis this will result in mediastinitis. With high mortality. The McKeown technique has an intrinsically lower systemic insult in the event of anastomotic leakage.

In addition to surgical resection many patients will be treated with adjuvant chemotherapy.

1225
Q

A 52 year old man falls off his bike. He is found to have a pelvic fracture. On examination, he is found to have perineal oedema and on PR the prostate is not palpable. What is the most likely underlying problem?

Bulbar urethral rupture

Bladder outflow obstruction

Bladder rupture

Bladder contusion

Membranous urethral rupture

A

A pelvic fracture and highly displaced prostate should indicate a diagnosis of membranous urethral rupture.

Lower genitourinary tract trauma

Most bladder injuries occur due to blunt trauma
85% associated with pelvic fractures
Easily overlooked during assessment in trauma
Up to 10% of male pelvic fractures are associated with urethral or bladder injuries

Types of injury

Urethral injury
Mainly in males
Blood at the meatus (50% cases)
There are 2 types:
i.Bulbar rupture
- most common
- straddle type injury e.g. bicycles
- triad signs: urinary retention, perineal haematoma, blood at the meatus
ii. Membranous rupture
- can be extra or intraperitoneal
- commonly due to pelvic fracture
- Penile or perineal oedema/ hematoma
- PR: prostate displaced upwards (beware co-existing retroperitoneal haematomas as they may make examination difficult)

  • Investigation: ascending urethrogram
  • Management: suprapubic catheter (surgical placement, not percutaneously)
    External genitalia injuries (i.e., the penis and the scrotum)
    Secondary to injuries caused by penetration, blunt trauma, continence- or sexual pleasure-enhancing devices, and mutilation
    Bladder injury
    rupture is intra or extraperitoneal
    presents with haematuria or suprapubic pain
    history of pelvic fracture and inability to void: always suspect bladder or urethral injury
    inability to retrieve all fluid used to irrigate the bladder through a Foley catheter indicates bladder injury
    investigation- IVU or cystogram
    management: laparotomy if intraperitoneal, conservative if extraperitoneal
1226
Q

A 35 year tennis player attends clinic reporting tingling down his arm. He says that his ‘funny bone’ was hit very hard by a tennis ball. There is weakness of abduction and adduction of his extended fingers. Which nerve has been affected?

Ulnar

Anterior interosseous

Posterior interosseous

Median

Musculocutaneous

A

The ulnar nerve arises from the medial cord of the brachial plexus (C8, T1 and contribution from C7). The nerve descends between the axillary artery and vein, posterior to the cutaneous nerve of the forearm and then lies anterior to triceps on the medial side of the brachial artery. In the distal half of the arm it passes through the medial intermuscular septum, and continues between this structure and the medial head of triceps to enter the forearm between the medial epicondyle of the humerus and the olecranon. It may be injured at this site in this scenario.
Ulnar nerve

Origin
C8, T1

Supplies (no muscles in the upper arm)
Flexor carpi ulnaris
Flexor digitorum profundus
Flexor digiti minimi
Abductor digiti minimi
Opponens digiti minimi
Adductor pollicis
Interossei muscle
Third and fourth lumbricals
Palmaris brevis

Path
Posteromedial aspect of upper arm to flexor compartment of forearm, then along the ulnar. Passes beneath the flexor carpi ulnaris muscle, then superficially over the flexor retinaculum into the palm of the hand.

Branches
Branch Supplies
Muscular branch Flexor carpi ulnaris
Medial half of the flexor digitorum profundus
Palmar cutaneous branch (Arises near the middle of the forearm) Skin on the medial part of the palm
Dorsal cutaneous branch Dorsal surface of the medial part of the hand
Superficial branch Cutaneous fibres to the anterior surfaces of the medial one and one-half digits
Deep branch Hypothenar muscles
All the interosseous muscles
Third and fourth lumbricals
Adductor pollicis
Medial head of the flexor pollicis brevis

Effects of injury
Damage at the wrist
Wasting and paralysis of intrinsic hand muscles (claw hand)
Wasting and paralysis of hypothenar muscles
Loss of sensation medial 1 and half fingers
Damage at the elbow
Radial deviation of the wrist
Clawing less in 4th and 5th digits

1227
Q

You are the specialist trainee in endocrinology clinic. The medical team have referred a man for a parathyroidectomy who has a corrected calcium of 2.82 (elevated) and a PTH of 11 (elevated). Which of the following is not an indication for parathyroidectomy in this case?

Nephrolithiasis

Reduction in bone mineral density of the femoral neck, lumbar spine, or distal radius of more than 2.5 standard deviations below peak bone mass

Age < 50 years

Episode of life threatening hypercalcaemia

None of the above

A

All of the situations listed are indications for parathyroidectomy. See below for more information.

Parathyroid glands and disorders of calcium metabolism

Hyperparathyroidism
Disease type Hormone profile Clinical features Cause
Primary hyperparathyroidism
PTH (Elevated)
Ca2+ (Elevated)
Phosphate (Low)
Urine calcium : creatinine clearance ratio > 0.01
May be asymptomatic if mild
Recurrent abdominal pain (pancreatitis, renal colic)
Changes to emotional or cognitive state
Most cases due to solitary adenoma (80%), multifocal disease occurs in 10-15% and parathyroid carcinoma in 1% or less
Secondary hyperparathyroidism
PTH (Elevated)
Ca2+ (Low or normal)
Phosphate (Elevated)
Vitamin D levels (Low)
May have few symptoms
Eventually may develop bone disease, osteitis fibrosa cystica and soft tissue calcifications
Parathyroid gland hyperplasia occurs as a result of low calcium, almost always in a setting of chronic renal failure
Tertiary hyperparathyroidism
Ca2+ (Normal or high)
PTH (Elevated)
Phosphate levels (Decreased or Normal)
Vitamin D (Normal or decreased)
Alkaline phosphatase (Elevated)
Metastatic calcification
Bone pain and / or fracture
Nephrolithiasis
Pancreatitis
Occurs as a result of ongoing hyperplasia of the parathyroid glands after correction of underlying renal disorder, hyperplasia of all 4 glands is usually the cause

Differential diagnoses
It is important to consider the rare but relatively benign condition of benign familial hypocalciuric hypercalcaemia, caused by an autosomal dominant genetic disorder. Diagnosis is usually made by genetic testing and concordant biochemistry (urine calcium : creatinine clearance ratio <0.01-distinguished from primary hyperparathyroidism).

Treatment

Primary hyperparathyroidism
Indications for surgery
Elevated serum Calcium > 1mg/dL above normal
Hypercalciuria > 400mg/day
Creatinine clearance < 30% compared with normal
Episode of life threatening hypercalcaemia
Nephrolithiasis
Age < 50 years
Neuromuscular symptoms
Reduction in bone mineral density of the femoral neck, lumbar spine, or distal radius of more than 2.5 standard deviations below peak bone mass (T score lower than -2.5)

Secondary hyperparathyroidism
Usually managed with medical therapy.

Indications for surgery in secondary (renal) hyperparathyroidism:
Bone pain
Persistent pruritus
Soft tissue calcifications

Tertiary hyperparathyroidism
Allow 12 months to elapse following transplant as many cases will resolve
The presence of an autonomously functioning parathyroid gland may require surgery. If the culprit gland can be identified then it should be excised. Otherwise total parathyroidectomy and re-implantation of part of the gland may be required.

References
1. Insogna K. Primary Hyperparathyroidism. N Engl J Med. 2018 Sep 13;379(11):1050-1059.

  1. van der Plas W Y et al. Secondary and Tertiary Hyperparathyroidism: A Narrative Review. Scand J Surg. 2020 Dec;109(4):271-278.
1228
Q

A 23 year old rugby player falls directly onto his shoulder. There is pain and swelling of the shoulder joint. The clavicle is prominent and there appears to be a step deformity. What is the most likely diagnosis?

Acromioclavicular joint dislocation

Glenohumeral dislocation

Sternoclavicular dislocation

Supraspinatus tear

Infra spinatus tear

A

Acromioclavicular joint (ACJ) dislocation normally occurs secondary to direct injury to the superior aspect of the acromion. Loss of shoulder contour and prominent clavicle are key features. Note; rotator cuff tears rarely occur in the third decade.

Shoulder disorders

Shoulder fractures and dislocations
Fractures
Proximal humerus
Background
Third most common fragility fracture in the elderly.
Results from low energy fall in predominantly elderly females, or from high energy trauma in young males.
Can be associated with nerve injury (commonly axillary), and fracture-dislocation of the humeral head. Detailed neurological assessment is essential for all upper limb injuries.

Anatomy
Osteology
Consists of articular head, greater tuberosity, lesser tuberosity, metaphysis and diaphysis. Between the articular head and the tuberosities is the anatomical neck (previous physis). Between the tuberosities and the metaphysis is the surgical neck.
The supraspinatus, infraspinatus and teres minor muscles attach to the greater tuberosity. The subscapularis muscle attaches to the lesser tuberosity.

Vascular Supply
Humeral head is supplied by the anterior and posterior humeral circumflex arteries. Anatomical neck fractures are at greatest risk of osteonecrosis.

Imaging
Imaging aims to both delineate the fracture pattern, and confirm/exlude the presence of an associated dislocation.
Radiographs - True anteroposterior (AP), axillary lateral and/or scapula Y view.
CT - indicated to better define intra-articular involvement and to aid pre-operative planning. MRI is not useful for fracture imaging.

Classification
Description of the fracture is often more useful than classification. Particular attention should be paid to humeral alignment, fracture displacement, and greater tuberosity position (rotator cuff will pull the GT supero-posterioly, which can cause impingement problems with malunion).
- Neer Classification: Most commonly used. Describes fracture as 2,3,or 4 part depending upon the number main fragments. Also comments on the degree of displacement. Fragments:
-greater tuberosity
-lesser tuberosity
- articular surface
- shaft
Displacement: >1cm or angulation >45 degrees.

Treatment
The vast majority of proximal humeral fractures are minimally displaced, and therefore can be managed conservatively. This involves immobilisation in a polysling, and progressive mobilisation. Pendular exercise can commence at 14 days, and active abduction from 4-6 weeks.

Irreducible fracture dislocation is an indication for operative management. Other indications include large displacement, younger patient, head splitting (intra-articular fractures). However, the recent PROFHER trial (1) has suggested no benefit to operative intervention on patient outcome (it must be applied cautiously as majority of patients were elderly with extraarticular fractures). Options available for surgical management include:

ORIF Most commonly used. Plate and screw fixation. Can reconstruct complex fractures.
Intramedullary nail Suitable for extra-articular configuration, predominantly surgical neck +/- GT fractures.
Hemiarthroplasty Used for un-reconstructable fractures in the older patient who has good glenoid quality.
Total shoulder arthroplasty Unconstructable fractures where high functioning shoulder is required (hemiarthroplasty will cause glenoid erosion)
Reverse shoulder arthroplasty Total shoulder arthroplasty that provides better functional outcome than conventional total shoulder replacement.

Scapula
Background
Uncommon fractures usually associated with high energy trauma. Most commonly involve scapula body or spine (50%), glenoid fossa and glenoid neck. Important to exclude associated life threatening injury.

Imaging
Plain radiographs should include true anteroposterior (AP), axillary lateral and/or scapula Y view. CT scanning is useful for defining intra-articular involvement, displacement and for three dimensional reconstruction.

Classification
Based on the location of the fracture (coracoid, acromion, glenoid neck, glenoid fossa, scapula body). Beware of ipsilateral glenoid neck and clavicle fracture -floating shoulder - where limb is effectively dissociated from axial skeleton.

Treatment
The vast majority of scapula fractures are amenable to conservative management, consisting of sling immobilisation for two weeks followed by early rehabilitation. Floating shoulder will usually require fixation, and consideration of surgery should also be given to intra-articular and displaced/angulated glenoid fractures.

Dislocations

Types
Dislocations around the shoulder joint include glenohumeral dislocation, acromioclavicular joint disruption and sternoclavicular dislocation. Only glenohumeral dislocation will be covered here.

Glenohumeral dislocation
Diagnosis, classification and management are covered here.

Background
Shoulder dislocation is commonly seen in A&E. It has a high recurrence rate that is as high as 80% in teenagers. Initial management requires emergent reduction to prevent lasting chondral damage.

Early assessment and management
Usually a traumatic cause (multi-directional instability in frequent dislocations requires discussion with orthopaedics and is not covered here). Careful history, examination and documentation of neurovascular status of the limb, in particular the axillary nerve (regimental badge sensation). This should be re-assessed post manipulation. Early radiographs to confirm direction of dislocation.

Initial management consists of emergent closed reduction under under entanox and analgesia, but often requires conscious sedation. Arm should then be immobilised in a polysling, and XR to confirm relocation.

Imaging - True anteroposterior (AP), axillary lateral and/or scapula Y view. Reduced humeral head should lie between acromion and coracoid on lateral/scapula view.

Types
Direction Features Cause Examination Reduction techniques
Anterior Most Common >90% Usually traumatic - anterior force on arm when shoulder is abducted, externally rotated Loss of shoulder contour - sulcus sign. Humeral head can be felt anteriorly.
Hippocratic.
Milch.
Stimson.

Kocher not advised due to complication of fracture
Posterior 50% missed in A&E 50% traumatic, but classically post seizure or electrocution Shoulder locked in internal rotation. XR may show lightbulb appearance. Gentle lateral traction to adducted arm.
Inferior Rare Associated with pectorals and rotator cuff tears, and glenoid fracture As for primary injury Management of primary injury
Superior Rare Associated with acrominon/clavicle fracture As for primary injury Management of primary injury

Associated injuries
Bankart lesion - avulsion of the anterior glenoid labrum with an anterior shoulder dislocation (reverse Bankart if poster labrum in posterior dislocation).
Hill Sachs defect - chondral impaction on posteriosuperior humeral head from contact with gleonoid rim. Can be large enough to lock shoulder, requiring open reduction. (Reverse Hill Sachs in posterior dislocation).
Rotator cuff tear - increases with age.
Greater or lesser tuberosity fracture - increases with age.
Humeral neck fracture - shoulder fracture dislocation. More common in high energy trauma and elderly. Should be discussed with orthopaedics prior to any attempted reduction.

Rotator Cuff Disease

Rotator cuff disease is a spectrum of conditions that ranges from subacromial impingement to rotator cuff tears and eventually to rotator cuff arthropathy (arthritis).

Anatomy
The rotator cuff is a group of four muscles that are important in shoulder movements, and maintenance of glenohumeral stability.
Muscle Scapular attachment Humeral attachment Action Innervation
Supraspinatus Supraspinatus fossa Superior facet of greater tuberosity Initiation of abduction of humerus Suprascapular nerve
Infraspinatus Infraspinatus fossa Posterior facet of greater tuberosity External rotation of humerus Suprascapular nerve
Teres Minor Lateral border Inferior facet of greater tuberosity External rotation of humerus Axillary Nerve
Subscapularis Subscapular fossa Lesser tuberosity Internal rotation of humerus Upper and lower subscapular nerve

The inferior rotator cuff muscles (infraspinatus, teres minor, and subscapularis) balance the superior pull of the deltoid. Injury/tear results in upward migration of the humeral head on the glenoid (can be seen on AP radiograph).
Likewise, the anterior muscles (subscapularis) are balanced with the posterior muscles (infraspinatus, teres minor).

Subacromial Impingement

The most common cause of shoulder pain, which results from impingement of the superior cuff on the undersurface of the acromion, and an inflammatory bursitis.
Associated with certain types of acromial morphology (Bigliani classification).
Presents as insidious pain which is exacerbated by overhead activities.

Rotator Cuff Tear

Often presents as an acute event on the background of chronic subacromial impingement in the older patient, but can present as an avulsion injury in younger patients.
Majority of tears are to the superior cuff (supraspinatus, infraspinatus, teres minor), though a tear to subscapularis is associated with subcoracoid impingement.
Tears present as pain and weakness when using the muscles in question.

Rotator Cuff Arthropathy

Defined as shoulder arthritis in the setting of rotator cuff dysfunction. Results from superior migration due to the loss of rotator cuff function and integrity. Unopposed deltoid pulls the humeral head superiorly.
Associated with massive chronic cuff tears.

Imaging

Plain radiographs
AP of the shoulder may show superior migration of the humerus with a cuff tear, and features of arthritis with arthropathy. Other causes of pain may also be identified (e.g. calcific tendonitis/fracture)
Outlet view is useful for defining the acromial morphology

USS
Allows dynamic imaging of the cuff, and is inexpensive. However, it is very user dependent.

MRI
Best imaging modality for cuff pathology.
Also allows imaging of the rest of the shoulder. When intra-articular pathology is suspected, can be combined with an arthrogram for improved sensitivity and specificity.

Treatment

Subacromial impingement
Physiotherapy, oral anti-inflammatory medication
Subacromial steroid injection can settle inflammation
Arthroscopic subacromial decompression by shaving away the undersurface of the acromion, more space is created for the rotator cuff. Cuff integrity is assessed also at time of surgery, and can be repaired if necessary.

Rotator cuff tear
When considering repair of a cuff tear, the age and activity of the patient, the nature of the tear (degenerative vs. acute traumatic), and the size and retraction of the tear should be considered when making a surgical plan.
Mild tears or tears in the elderly can be managed conservatively, as outlined above.
Moderate tears can be repaired arthroscopically. Massive or retracted tears will often require an open repair (occasionally with a tendon transfer). Subacromial decompression is performed at the same time to reduce impingement, symptoms and recurrence.

Calcific tendonitis
Calcific tendonitis involves calcific deposits within tendons anywhere in the body, but most commonly in the rotator cuff (specifically the supraspinatus tendon). When present in the shoulder, it is associated with subacromial impingement and pain.

Pathology
More common in women aged 30-60 years.
Association with diabetes and hypothyroidism

There are three stages of calcification
Formative phase characterized by calcific deposits
Resting phase deposit is stable, but presents with impingement problems
Resorptive phase phagocytic resorption. Most painful stage.

Presentation
Similar in presentation to subacromial impingement, with pain especially with over head activities. Atraumatic in nature.

Imaging
Plain radiographs show calcification of the rotator cuff, usually within 1.5cm of its insertion on the humerus. Supraspinatus outlet views can show level of impingment. Further imaging is rarely needed.

Treatment
Non-operative NSAIDS, steroid injection (controversial, but practiced) and physiotherapy. Approximately 75% will resolve by 6 months with conservative management.
Ultrasound guided or surgical needle barbotage can break down deposits and resolve symptoms. Occasionally surgical excision is required.

Adhesive capsulitis (Frozen Shoulder)
Pain and loss of movement of shoulder joint, which involves fibroplastic proliferation of capsular tissue, causing soft tissue scarring and contracture. Patients present with a painful and decreased arc of motion.
Associated with prolonged immobilization, previous surgery, thyroid disorders (AI) and diabetes
Classically three stages which can take up to two years to resolve:
Stage one the freezing and painful stage
Stage two the frozen and stiff stage
Stage three the thawing stage, where shoulder movement slowly improves

Imaging
Plain radiographs to exclude other causes of a painful shoulder
MRI arthrogram may show capsular contracture, and again may be used to exclude cuff pathology. However, often not performed as diagnosis is largely clinical.

Treatment
Non-operative NSAIDS, steroid injection and physiotherapy. Patience is required as condition can take up to 2 years to improve.
Operative MUA or arthroscopic adhesiolysis (release of adhesions) can expedite recovery, followed by intensive physiotherapy.

Glenohumeral Arthritis
Background
May be osteoarthritis (primary or secondary to cuff tear or trauma), rheumatoid arthritis, or as part of a spondyloarthropathy. Majority of those with RA will develop symptoms.
More common in the elderly
Presents like any other arthritis - pain at night and with movement

Imaging
AP and axillary radiographs will show features of arthritis.
CT/MRI is often useful to classify the shape of the glenoid and extent of bone loss when considering arthroplasty. MRI also essential to asses integrity of rotator cuff if considering shoulder replacement.

Treatment
Like all orthopaedics, start with simple measures:
NSAIDS, management of RA, physiotherapy, steroid injection.
Hemiarthroplasty can sometimes be considered if glenoid is in excellent condition or if patient has large comorbidity.
Arthroscopic debridement is useful if patient has isolated ACJ arthritis, but is rarely used for glenohumeral arthritis.
Total shoulder replacement is shown to produce superior outcome when compared to hemiarthroplasty in terms of pain relief, function and implant survival.
Total shoulder replacement can be anatomical (ball on humerus, with cup on glenoid), or reverse geometry (ball on glenoid, with cup on humerus). Anatomical TSR requires an in tact rotator cuff, so often reverse is preferable when the cuff if questionable in integrity.

References
1. JAMA. 2015;313(10):1037-1047. doi:10.1001/jama.2015.1629

1229
Q

Of the list below, which is not a cause of avascular necrosis?

Steroids

Sickle cell disease

Radiotherapy

Myeloma

Caisson disease

A

Causes of avascular necrosis
P ancreatitis
L upus
A lcohol
S teroids
T rauma
I diopathic, infection
C aisson disease, collagen vascular disease
R adiation, rheumatoid arthritis
A myloid
G aucher disease
S ickle cell disease
Steroid containing therapy for myeloma may induce avascular necrosis, however the disease itself does not cause it. Caisson disease as may occur in deep sea divers is a recognised cause.

1230
Q

A 72 year old man has a fall. He is found to have a fractured neck of femur and goes on to have a left hip hemiarthroplasty. Two months post operatively he is found to have an odd gait. When standing on his left leg his pelvis dips on the right side. There is no foot drop. What is the cause?

Sciatic nerve damage

L5 radiculopathy

Inferior gluteal nerve damage

Previous poliomyelitis

Superior gluteal nerve damage

A

This patient has a trendelenburg gait caused by damage to the superior gluteal nerve causing weakness of the abductor muscles. Classically a patient is asked to stand on one leg and the pelvis dips on the opposite side. The absence of a foot drop excludes the possibility of polio or L5 radiculopathy.

Gluteal region

Gluteal muscles
Gluteus maximus: inserts to gluteal tuberosity of the femur and iliotibial tract
Gluteus medius: attach to lateral greater trochanter
Gluteus minimis: attach to anterior greater trochanter
All extend and abduct the hip

Deep lateral hip rotators
Piriformis
Gemelli
Obturator internus
Quadratus femoris

Nerves
Superior gluteal nerve (L4,L5, S1)
Gluteus medius
Gluteus minimis
Tensor fascia lata
Inferior gluteal nerve (L5, S1, S2) Gluteus maximus
Damage to the superior gluteal nerve will result in the patient developing a Trendelenberg gait. Affected patients are unable to abduct the thigh at the hip joint. During the stance phase, the weakened abductor muscles allow the pelvis to tilt down on the opposite side. To compensate, the trunk lurches to the weakened side to attempt to maintain a level pelvis throughout the gait cycle. The pelvis sags on the opposite side of the lesioned superior gluteal nerve.

1231
Q

A 78 year old man presents with symptoms consistent with intermittent claudication. To assess the severity of his disease you decide to measure his ankle brachial pressure index. To do this you will identify the dorsalis pedis artery. Which of the following statements relating to this vessel is false?

It originates from the peroneal artery

It is crossed by the tendon of extensor hallucis brevis

Two veins are usually closely related to it

It passes under the inferior extensor retinaculum

The tendon of extensor digitorum longus lies lateral to it

A

The dorsalis pedis artery is a direct continuation of the anterior tibial artery.

Foot- anatomy

Arches of the foot
The foot is conventionally considered to have two arches.
The longitudinal arch is higher on the medial than on the lateral side. The posterior part of the calcaneum forms a posterior pillar to support the arch. The lateral part of this structure passes via the cuboid bone and the lateral two metatarsal bones. The medial part of this structure is more important. The head of the talus marks the summit of this arch, located between the sustentaculum tali and the navicular bone. The anterior pillar of the medial arch is composed of the navicular bone, the three cuneiforms and the medial three metatarsal bones.
The transverse arch is situated on the anterior part of the tarsus and the posterior part of the metatarsus. The cuneiforms and metatarsal bases narrow inferiorly, which contributes to the shape of the arch.

Intertarsal joints
Sub talar joint Formed by the cylindrical facet on the lower surface of the body of the talus and the posterior facet on the upper surface of the calcaneus. The facet on the talus is concave anteroposteriorly, the other is convex. The synovial cavity of this joint does not communicate with any other joint.
Talocalcaneonavicular joint The anterior part of the socket is formed by the concave articular surface of the navicular bone, posteriorly by the upper surface of the sustentaculum tali. The talus sits within this socket
Calcaneocuboid joint Highest point in the lateral part of the longitudinal arch. The lower aspect of this joint is reinforced by the long plantar and plantar calcaneocuboid ligaments.
Transverse tarsal joint The talocalcaneonavicular joint and the calcaneocuboid joint extend across the tarsus in an irregular transverse plane, between the talus and calcaneus behind and the navicular and cuboid bones in front. This plane is termed the transverse tarsal joint.
Cuneonavicular joint Formed between the convex anterior surface of the navicular bone and the concave surface of the the posterior ends of the three cuneiforms.
Intercuneiform joints Between the three cuneiform bones.
Cuneocuboid joint Between the circular facets on the lateral cuneiform bone and the cuboid. This joint contributes to the tarsal part of the transverse arch.
A detailed knowledge of the joints is not required for MRCS Part A. However, the contribution they play to the overall structure of the foot should be appreciated

Muscles of the foot
Muscle Origin Insertion Nerve supply Action
Abductor hallucis Medial side of the calcaneus, flexor retinaculum, plantar aponeurosis Medial side of the base of the proximal phalanx Medial plantar nerve Abducts the great toe
Flexor digitorum brevis Medial process of the calcaneus, plantar eponeurosis. Via 4 tendons into the middle phalanges of the lateral 4 toes. Medial plantar nerve Flexes all the joints of the lateral 4 toes except for the interphalangeal joint.
Abductor digit minimi From the tubercle of the calcaneus and from the plantar aponeurosis Together with flexor digit minimi brevis into the lateral side of the base of the proximal phalanx of the little toe Lateral plantar nerve Abducts the little toe at the metatarsophalangeal joint
Flexor hallucis brevis From the medial side of the plantar surface of the cuboid bone, from the adjacent part of the lateral cuneiform bone and from the tendon of tibialis posterior. Into the proximal phalanx of the great toe, the tendon contains a sesamoid bone Medial plantar nerve Flexes the metatarsophalangeal joint of the great toe.
Adductor hallucis Arises from two heads. The oblique head arises from the sheath of the peroneus longus tendon, and from the plantar surfaces of the bases of the 2nd, 3rd and 4th metatarsal bones. The transverse head arises from the plantar surface of the lateral 4 metatarsophalangeal joints and from the deep transverse metatarsal ligament. Lateral side of the base of the proximal phalanx of the great toe. Lateral plantar nerve Adducts the great toe towards the second toe. Helps maintain the transverse arch of the foot.
Extensor digitorum brevis On the dorsal surface of the foot from the upper surface of the calcaneus and its associated fascia Via four thin tendons which run forward and medially to be inserted into the medial four toes. The lateral three tendons join with hoods of extensor digitorum longus. Deep peroneal Extend the metatarsophalangeal joint of the medial four toes. It is unable to extend the interphalangeal joint without the assistance of the lumbrical muscles.

Nerves in the foot

Lateral plantar nerve
Passes anterolaterally towards the base of the 5th metatarsal between flexor digitorum brevis and flexor accessorius. On the medial aspect of the lateral plantar artery. At the base of the 5th metatarsal it splits into superficial and deep branches.

Medial plantar nerve
Passes forwards with the medial plantar artery under the cover of the flexor retinaculum to the interval between abductor hallucis and flexor digitorum brevis on the sole of the foot.

Plantar arteries
Arise under the cover of the flexor retinaculum, midway between the tip of the medial malleolus and the most prominent part of the medial side of the heel.

Medial plantar artery. Passes forwards medial to medial plantar nerve in the space between abductor hallucis and flexor digitorum brevis.Ends by uniting with a branch of the 1st plantar metatarsal artery.
Lateral plantar artery. Runs obliquely across the sole of the foot. It lies lateral to the lateral plantar nerve. At the base of the 5th metatarsal bone it arches medially across the foot on the metatarsals

Dorsalis pedis artery
This vessel is a direct continuation of the anterior tibial artery. It commences on the front of the ankle joint and runs to the proximal end of the first metatarsal space. Here is gives off the arcuate artery and continues forwards as the first dorsal metatarsal artery. It is accompanied by two veins throughout its length. It is crossed by the extensor hallucis brevis

1232
Q

Which of the ABPI measurements shown below is most likely to be found in a 45 year old man who develops calf pain after walking 600 yards. It resolves during periods of rest.

> 1.2

1.0

0.7

0.5

0.3

A

Since this is a long claudication distance it may be that only a minor lesion is present. Whilst resting ABPI may be normal, they are usually abnormal following exercise.

Ankle-Brachial pressure index

Measurement of ankle- brachial pressure index (ABPI) is a commonly performed vascular investigation.
Calculated by dividing lower limb pressure by the highest upper limb pressure.

Results of ABPI
1.2 or greater Usually due to vessel calcification
1.0- 1.2 Normal
0.8-1.0 Minor stenotic lesion
Initiate risk factor management
0.50-0.8 Moderate stenotic lesion
Consider duplex
Risk factor management
If mixed ulcers present then avoid tight compression bandages
0.3 - 0.5 Likely significant stenosis
Duplex scanning to delineate lesions needed
Compression bandaging contra indicated
Less than 0.3 Indicative of critical ischaemia
Urgent detailed imaging required

1233
Q

A 19 year old female presents to the clinic with progressive pain in her neck and back. The condition has been progressively worsening over the past 6 months. She has not presented previously because she was an inpatient with a disease flare of ulcerative colitis. On examination, she has a stiff back with limited spinal extension on bending forwards. What is the most likely explanation for this process?

Spondylolysis

Spondylolisthesis

Functional scoliosis

Scheuermanns disease

Ankylosing spondylitis

A

Ankylosing spondylitis is associated with HLA B27, there is a strong association with ulcerative colitis in such individuals. The clinical findings are usually of a kyphosis affecting the cervical and thoracic spine. Considerable symptomatic benefit may be obtained using non steroidal anti inflammatory drugs. These should be used carefully in patients with inflammatory bowel disease who may be taking steroids.

Diseases affecting the vertebral column

Ankylosing spondylitis
Chronic inflammatory disorder affecting the axial skeleton
Sacro-ilitis is a usually visible in plain films
Up to 20% of those who are HLA B27 positive will develop the condition
Affected articulations develop bony or fibrous changes
Typical spinal features include loss of the lumbar lordosis and progressive kyphosis of the cervico-thoracic spine
Scheuermann’s disease
Epiphysitis of the vertebral joints is the main pathological process
Predominantly affects adolescents
Symptoms include back pain and stiffness
X-ray changes include epiphyseal plate disturbance and anterior wedging
Clinical features include progressive kyphosis (at least 3 vertebrae must be involved)
Minor cases may be managed with physiotherapy and analgesia, more severe cases may require bracing or surgical stabilisation
Scoliosis
Consists of curvature of the spine in the coronal plane
Divisible into structural and non structural, the latter being commonest in adolescent females who develop minor postural changes only. Postural scoliosis will typically disappear on manoeuvres such as bending forwards
Structural scoliosis affects > 1 vertebral body and is divisible into idiopathic, congential and neuromuscular in origin. It is not correctable by alterations in posture
Within structural scoliosis, idiopathic is the most common type
Severe, or progressive structural disease is often managed surgically with bilateral rod stabilisation of the spine
Spina bifida
Non fusion of the vertebral arches during embryonic development
Three categories; myelomeningocele, spina bifida occulta and meningocele
Myelomeningocele is the most severe type with associated neurological defects that may persist in spite of anatomical closure of the defect
Up to 10% of the population may have spina bifida occulta, in this condition the skin and tissues (but not not bones) may develop over the distal cord. The site may be identifiable by a birth mark or hair patch
The incidence of the condition is reduced by use of folic acid supplements during pregnancy
Spondylolysis
Congenital or acquired deficiency of the pars interarticularis of the neural arch of a particular vertebral body, usually affects L4/ L5
May be asymptomatic and affects up to 5% of the population
Spondylolysis is the commonest cause of spondylolisthesis in children
Asymptomatic cases do not require treatment
Spondylolisthesis
This occurs when one vertebra is displaced relative to its immediate inferior vertebral body
May occur as a result of stress fracture or spondylolysis
Traumatic cases may show the classic ‘Scotty Dog’ appearance on plain films
Treatment depends upon the extent of deformity and associated neurological symptoms, minor cases may be actively monitored. Individuals with radicular symptoms or signs will usually require spinal decompression and stabilisation

1234
Q

A 50 year old lady is commenced on tamoxifen for the treatment of an oestrogen receptor positive breast cancer. Which of the following malignancies are associated with tamoxifen use?

Adenocarcinoma of the colon

Hodgkins lymphoma

Adenocarcinoma of the lung

Ovarian cancer

Endometrial cancer

A

Tamoxifen is an oestrogen receptor antagonist in breast tissues. However, at other sites, such as the endometrium it may act as an agonist. Hence the reason for increasing risk of endometrial cancer.

Tamoxifen

Synthetic partial oestrogen agonist, acts primarily by binding to the oestrogen receptor.
Half life of 7 days, takes 4 weeks for drug to reach plasma steady state.
Should usually be considered in patients with oestrogen receptor positive tumours (alternative agents may be preferred in some groups).
Although antagonistic with respects to breast tissue tamoxifen may serve as an agonist at other sites. Therefore risk of endometrial cancer is increased, preservation of bone density and decreased cardiovascular risks.
Climateric side effects are common, 3% stop taking the drug because of these.
Aromatase inhibitors are an alternative class of drugs, these work by blocking the peripheral aromatization of androgens (post menopausal women produce oestrogens in this way). They may treat cancers for which tamoxifen is no longer effective.

1235
Q

A 45 year old woman with a thyroid carcinoma undergoes a total thyroidectomy. The post operative histology report shows a final diagnosis of medullary type thyroid cancer. Which of the tests below is most likely to be of clinical use in screening for disease recurrence?

Serum CA 19-9 Levels

Serum thyroglobulin levels

Serum PTH levels

Serum calcitonin levels

Serum TSH levels

A

Medullary thyroid cancers often secrete calcitonin and monitoring the serum levels of this hormone is useful in detecting sub clinical recurrence.

Thyroid neoplasms

Lesion Common features
Follicular adenoma
Usually present as a solitary thyroid nodule
Malignancy can only be excluded on formal histological assessment
Papillary carcinoma
Usually contain a mixture of papillary and colloidal filled follicles
Histologically tumour has papillary projections and pale empty nuclei
Seldom encapsulated
Lymph node metastasis predominate
Haematogenous metastasis rare
Account for 60% of thyroid cancers
Follicular carcinoma
May appear macroscopically encapsulated, microscopically capsular invasion is seen. Without this finding the lesion is a follicular adenoma.
Vascular invasion predominates
Multifocal disease rare
Account for 20% of all thyroid cancers
Anaplastic carcinoma
Most common in elderly females
Local invasion is a common feature
Account for 10% of thyroid cancers
Treatment is by resection where possible, palliation may be achieved through isthmusectomy and radiotherapy. Chemotherapy is ineffective.
Medullary carcinoma
Tumours of the parafollicular cells (C Cells)
C cells derived from neural crest and not thyroid tissue
Serum calcitonin levels often raised
Familial genetic disease accounts for up to 20% cases
Both lymphatic and haematogenous metastasis are recognised, nodal disease is associated with a very poor prognosis.

1236
Q

Which of the following statements relating to menisceal tears is false?

The medial meniscus is most often affected

True locking of the knee joint may occur

Most established tears will heal with conservative management

In the chronic setting there is typically little to find on examination if the knee is not locked

An arthroscopic approach may be used to treat most lesions

A

Menisci have no nerve or blood supply and thus heal poorly. Established tears with associated symptoms are best managed by arthroscopic menisectomy.

Knee injury

Types of injury

Ruptured anterior cruciate ligament
Sport injury
Mechanism: high twisting force applied to a bent knee
Typically presents with: loud crack, pain and RAPID joint swelling (haemoarthrosis)
Poor healing
Management: intense physiotherapy or surgery
Ruptured posterior cruciate ligament
Mechanism: hyperextension injuries
Tibia lies back on the femur
Paradoxical anterior draw test
Rupture of medial collateral ligament
Mechanism: leg forced into valgus via force outside the leg
Knee unstable when put into valgus position
Menisceal tear
Rotational sporting injuries
Delayed knee swelling
Joint locking (Patient may develop skills to ‘unlock’ the knee
Recurrent episodes of pain and effusions are common, often following minor trauma
Chondromalacia patellae
Teenage girls, following an injury to knee e.g. Dislocation patella
Typical history of pain on going downstairs or at rest
Tenderness, quadriceps wasting
Dislocation of the patella
Most commonly occurs as a traumatic primary event, either through direct trauma or through severe contraction of quadriceps with knee stretched in valgus and external rotation
Genu valgum, tibial torsion and high riding patella are risk factors
Skyline x-ray views of patella are required, although displaced patella may be clinically obvious
An osteochondral fracture is present in 5%
The condition has a 20% recurrence rate
Fractured patella
2 types:
i. Direct blow to patella causing undisplaced fragments
ii. Avulsion fracture
Tibial plateau fracture
Occur in the elderly (or following significant trauma in young)
Mechanism: knee forced into valgus or varus, but the knee fractures before the ligaments rupture
Varus injury affects medial plateau and if valgus injury, lateral plateau depressed fracture occurs
Classified using the Schatzker system (see below)

Schatzker Classification system for tibial plateau fractures
Type Anatomical description Features
1 Vertical split of lateral condyle Fracture through dense bone, usually in the young. It may be virtually undisplaced, or the condylar fragment may be pushed inferiorly and tilted
2 Vertical split of the lateral condyle combined with an adjacent load bearing part of the condyle The wedge fragment (which may be of variable size), is displaced laterally; the joint is widened. Untreated, a valgus deformity may develop
3 Depression of the articular surface with intact condylar rim The split does not extend to the edge of the plateau. Depressed fragments may be firmly embedded in subchondral bone, the joint is stable
4 Fragment of the medial tibial condyle Two injuries are seen in this category; (1) a depressed fracture of osteoporotic bone in the elderly. (2) a high energy fracture resulting in a condylar split that runs from the intercondylar eminence to the medial cortex. Associated ligamentous injury may be severe
5 Fracture of both condyles Both condyles fractured but the column of the metaphysis remains in continuity with the tibial shaft
6 Combined condylar and subcondylar fractures High energy fracture with marked comminution

1237
Q

A 25 year old man returns from a backpacking holiday in India. He presents with symptoms of coughing and also of episodic abdominal discomfort. Peri anal examination is normal. Stool microscopy demonstrates both worms and eggs within the faeces. What is the most likely infective organism?

Cryptosporidium

Ancylostoma duodenale

Clonorchis sinensis

Ascaris lumbricoides

Enterobius vermicularis

A

Infection with Ascaris lumbricoides usually occurs after individuals have visited places like sub Saharan Africa or the far east. Unlike ancylostoma duodenale infection there is usually evidence of both worms and eggs in the stool. The absence of pruritus makes enterobius less likely. The presence of coughing may be due to the migration of the larva through the lungs.

Gastro intestinal parasitic infections

Common infections

Enterobiasis
Due to organism Enterobius vermicularis
Common cause of pruritus ani
Diagnosis usually made by placing scotch tape at the anus, this will trap eggs that can then be viewed microscopically
Treatment is with mebendazole
Ancylostoma duodenale
Hookworms that anchor in proximal small bowel
Most infections are asymptomatic although may cause iron deficiency anaemia
Larvae may be found in stools left at ambient temperature, otherwise infection is difficult to diagnose
Infection occurs as a result of cutaneous penetration, migrates to lungs, coughed up and then swallowed
Treatment is with mebendazole
Ascariasis
Due to infection with roundworm Ascaris lumbricoides
Infections begin in gut following ingestion, then penetrate duodenal wall to migrate to lungs, coughed up and swallowed, cycle begins again
Diagnosis is made by identification of worm or eggs within faeces
Treatment is with mebendazole
Strongyloidiasis
Due to infection with Strongyloides stercoralis
Rare in west
Organism is a nematode living in duodenum of host
Initial infection is via skin penetration. They then migrate to lungs and are coughed up and swallowed. Then mature in small bowel are excreted and cycle begins again
An auto infective cycle is also recognised where larvae will penetrate colonic wall
Individuals may be asymptomatic, although they may also have respiratory disease and skin lesions
Diagnosis is usually made by stool microscopy
In the UK mebendazole is used for treatment
Cryptosporidium
Protozoal infection
Organisms produce cysts which are excreted and thereby cause new infections
Symptoms consist of diarrhoea and cramping abdominal pains. Symptoms are worse in immunosuppressed people
Cysts may be identified in stools
Treatment is with metronidazole
Giardiasis
Diarrhoeal infection caused by Giardia lamblia (protozoan)
Infections occur as a result of ingestion of cysts
Symptoms are usually gastrointestinal with abdominal pain, bloating and passage of soft or loose stools
Diagnosis is by serology or stool microscopy
First line treatment is with metronidazole

1238
Q

A 23 year old lady presents with an itchy, bleeding pigmented lesion on her right thigh. What is the best course of action?

Core biopsy

Punch biopsy

Incisional biopsy

Excision biopsy

Wide excision

A

Avoid incisional or punch biopsy of lesions suspected to be melanoma.
This may represent a malignant melanoma. Complete excision is required to allow accurate histological assessment. If the diagnosis is confirmed then re-excision of margins may be required. Clearly if the lesion is benign then no further action is required.

Treatment of suspicious skin lesions

Skin lesions may be referred to surgeons for treatment or discovered incidentally. The table below outlines the various therapeutic options:

Method Indication
Tru-cut biopsy Most often used for percutaneous sampling of deep seated lesions or used intra operatively for visceral lesions
5mm punch biopsy Used for diagnostic confirmation of lesions that are suspected to be benign or where the definitive management is unlikely to be surgical. Of limited usefulness in pigmented lesions where they do not include sufficient tissue for accurate diagnosis. May be used in non melanoma type skin disease to establish diagnosis prior to more extensive resection.
Wide excision Where the complete excision of the lesion (with healthy margins) is the main objective. In cosmetically sensitive sites, or where the defect is large, this may need to be complemented with plastic surgical techniques
Incisional biopsy Used mainly for deep seated or extensive lesions where there is diagnostic doubt (usually following core or tru-cut biopsy). Used rarely for skin lesions.
Diagnostic excision Primarily used for lesions that are suspicious for melanoma, the lesion is excised with a rim of normal tissue. Excision of margins may be required subsequently.

1239
Q

A 1 day old neonate is intolerant of feeds. On examination, she has a pan systolic murmur and her forearms have not developed properly. What is the most likely underlying problem?

Oesophageal atresia

Duodenal atresia

Pyloric stenosis

Ileal atresia

Anal atresia

A

This child has VACTERL, which is a combination of Vertebral, Ano-rectal, Cardiac, Tracheo-oesophageal, Renal and Radial limb anomalies. Half of babies with oesophageal atresia will have VACTERL. Problems that occur during feeding are more suggestive of proximal pathology.

Paediatric Gastrointestinal disorders

Pyloric stenosis
M>F
5-10% Family history in parents
Projectile non bile stained vomiting at 4-6 weeks of life
Diagnosis is made by test feed or USS
Treatment: Ramstedt pyloromyotomy (open or laparoscopic)
Acute appendicitis
Uncommon under 3 years
When occurs may present atypically
Mesenteric adenitis
Central abdominal pain and URTI
Conservative management
Intussusception
Telescoping bowel
Proximal to or at the level of, ileocaecal valve
6-9 months age
Colicky pain, diarrhoea and vomiting, sausage shaped mass, red jelly stool.
Treatment: reduction with air insufflation
Malrotation
High caecum at the midline
Feature in exomphalos, congenital diaphragmatic hernia, intrinsic duodenal atresia
May be complicated by development of volvulus, infant with volvulus may have bile stained vomiting
Diagnosis is made by upper GI contrast study and USS
Treatment is by laparotomy, if volvulus is present (or at high risk of occurring then a ladds procedure is performed
Hirschsprung’s disease
Absence of ganglion cells from myenteric and submucosal plexuses
Occurs in 1/5000 births
Full thickness rectal biopsy for diagnosis
Delayed passage of meconium and abdominal distension
Treatment is with rectal washouts initially, thereafter an anorectal pull through procedure
Oesophageal atresia
Associated with tracheo-oesophageal fistula and polyhydramnios
May present with choking and cyanotic spells following aspiration
VACTERL associations
Meconium ileus
Usually delayed passage of meconium and abdominal distension
Majority have cystic fibrosis
X-Rays may not show a fluid level as the meconium is viscid (depends upon feeding), PR contrast studies may dislodge meconium plugs and be therapeutic
Infants who do not respond to PR contrast and NG N-acetyl cysteine will require surgery to remove the plugs
Biliary atresia
Jaundice > 14 days
Increased conjugated bilirubin
Urgent Kasai procedure
Necrotising enterocolitis
Prematurity is the main risk factor
Early features include abdominal distension and passage of bloody stools
X-Rays may show pneumatosis intestinalis and evidence of free air
Increased risk when empirical antibiotics are given to infants beyond 5 days
Treatment is with total gut rest and TPN, babies with perforations will require laparotomy

1240
Q

A 67 year old male is diagnosed as having a 7cm infra renal abdominal aortic aneurysm. What is the likely risk of rupture over the next 5 years?

<10%

20%

25%

75%

35%

A

Risks of abdominal aortic aneurysm rupture (over 5 years):
5-5.9cm = 25%
6-6.9cm= 35%
7cm and over = 75%
Aneurysms greater than 5cm in diameter on USS should be formally assessed using CT scanning with arterial phases to delineate anatomy and facilitate surgical planning.

Abdominal aorta aneurysm

  • Abdominal aortic aneurysms are a common problem in vascular surgery.
    They may occur as either true or false aneurysm. With the former all 3 layers of the arterial wall are involved, in the latter only a single layer of fibrous tissue forms the aneurysm wall.
    True abdominal aortic aneurysms have an approximate incidence of 0.06 per 1000 people. They are commonest in elderly men and for this reason the UK is now introducing the aneurysm screening program with the aim of performing an abdominal aortic ultrasound measurement in all men aged 65 years.

Causes
Several different groups of patients suffer from aneurysmal disease.
The commonest group is those who suffer from standard arterial disease, i.e. Those who are hypertensive and have been or are smokers.
Other patients such as those suffering from connective tissue diseases such as Marfan’s may also develop aneurysms. In patients with abdominal aortic aneurysms the extracellular matrix becomes disrupted with a change in the balance of collagen and elastic fibres.

Management
Most abdominal aortic aneurysms are an incidental finding.
Symptoms most often relate to rupture or impending rupture.
20% rupture anteriorly into the peritoneal cavity. Very poor prognosis.
80% rupture posteriorly into the retroperitoneal space
The risk of rupture is related to aneurysm size, only 2% of aneurysms measuring less than 4cm in diameter will rupture over a 5 year period. This contrasts with 75% of aneurysms measuring over 7cm in diameter.
This is well explained by Laplaces’ law which relates size to transmural pressure.
For this reason most vascular surgeons will subject patients with an aneurysm size of 5cm or greater to CT scanning of the chest, abdomen and pelvis with the aim of delineating anatomy and planning treatment. Depending upon co-morbidities, surgery is generally offered once the aneurysm is between 5.5cm and 6cm.

Indications for surgery
Symptomatic aneurysms (80% annual mortality if untreated)
Increasing size above 5.5cm if asymptomatic
Rupture (100% mortality without surgery)

Surgical procedures
Abdominal aortic aneurysm repair

Procedure:

GA
Invasive monitoring (A-line, CVP, catheter)
Incision: Midline or transverse
Bowel and distal duodenum mobilised to access aorta.
Aneurysm neck and base dissected out and prepared for cross clamp
Systemic heparinisation
Cross clamp (proximal first)
Longitudinal aortotomy
Atherectomy
Deal with back bleeding from lumbar vessels and inferior mesenteric artery
Insert graft either tube or bifurcated depending upon anatomy
Suture using Prolene (3/0 for proximal , distal anastomosis suture varies according to site)
Clamps off: End tidal CO2 will rise owing to effects of reperfusion, at this point major risk of myocardial events.
Haemostasis
Closure of aneurysm sac to minimise risk of aorto-enteric fistula
Closure: Loop 1 PDS or Prolene to abdominal wall
Skin- surgeons preference

Post operatively:

ITU (Almost all)
Greatest risk of complications following emergency repair
Complications: Embolic- gut and foot infarcts
Cardiac - owing to premorbid states, re-perfusion injury and effects of cross clamp
Wound problems
Later risks related to graft- infection and aorto-enteric fistula

Special groups

Supra renal AAA
These patients will require a supra renal clamp and this carries a far higher risk of complications and risk of renal failure.

Ruptured AAA
Pre-operatively the management depends upon haemodynamic instability. In patients with symptoms of rupture (typical pain, haemodynamic compromise and risk factors) then ideally prompt laparotomy. In those with vague symptoms and haemodynamic stability the ideal test is CT scan to determine whether rupture has occurred or not. Most common rupture site is retroperitoneal 80%. These patients will tend to develop retroperitoneal haematoma. This can be disrupted if Bp is allowed to rise too high so aim for Bp 100mmHg.
Operative details are similar to elective repair although surgery should be swift, blind rushing often makes the situation worse. Plunging vascular clamps blindly into a pool of blood at the aneurysm neck carries the risk of injury the vena cava that these patients do not withstand. Occasionally a supracoeliac clamp is needed to effect temporary control, although leaving this applied for more than 20 minutes tends to carry a dismal outcome.

EVAR
Increasingly patients are now being offered endovascular aortic aneurysm repair. This is undertaken by surgeons and radiologists working jointly. The morphology of the aneurysm is important and not all are suitable. Here is a typical list of those features favoring a suitable aneurysm:
Long neck
Straight iliac vessels
Healthy groin vessels

Clearly few AAA patients possess the above and compromise has to be made. The use of fenestrated grafts can allow supra renal AAA to be treated.

Procedure:

GA
Radiology or theatre
Bilateral groin incisions
Common femoral artery dissected out
Heparinisation
Arteriotomy and insertion of guide wire
Dilation of arteriotomy
Insertion of EVAR Device
Once in satisfactory position it is released
Arteriotomy closed once check angiogram shows good position and no endoleak

Complications:
Endoleaks depending upon site are either Type I or 2. These may necessitate re-intervention and all EVAR patients require follow up . Details are not needed for MRCS.

References
A reasonable review is provided by:
Sakalihasan N, Limet R, Defawe O. Abdominal aortic aneurysm. Lancet 2005 (365):1577- 1589

1241
Q

A 28 year teacher reports difficulty with writing. There is no sensory loss. She is known to have an aberrant Gantzer muscle. Which of the following nerves has been affected?

Posterior interosseous

Anterior interosseous

Median

Ulnar

Musculocutaneous

A

Anterior interosseous lesions occur due to fracture, or rarely due to compression. The Gantzer muscle is an aberrant accessory of the flexor pollicis longus and is a risk factor for anterior interosseous nerve compression. Remember loss of pincer grip and normal sensation indicates an interosseous nerve lesion.

Anterior interosseous nerve

The anterior interosseous nerve (volar interosseous nerve) is a branch of the median nerve that supplies the deep muscles on the front of the forearm, except the ulnar half of the flexor digitorum profundus.

It accompanies the anterior interosseous artery along the anterior of the interosseous membrane of the forearm, in the interval between the flexor pollicis longus and flexor digitorum profundus, supplying the whole of the former and (most commonly) the radial half of the latter, and ending below in the pronator quadratus and wrist joint.

Innervation
The anterior interosseous nerve classically innervates 2.5 muscles:

Flexor pollicis longus
Pronator quadratus
The radial half of flexor digitorum profundus (the lateral two out of the four tendons).

These muscles are in the deep level of the anterior compartment of the forearm.

1242
Q

A 24 year old man from Malawi presents with a lymphadenopathy and weight loss. A diagnosis of tuberculosis is suspected and a lymph node biopsy is performed. Staining with which of the agents below is most likely to facilitate identification of the causative organism?

Gram stain

Ziehl-Neelsen stain

Von Kossa stain

Van Gieson stain

Masson Trichrome stain

A

Ziehl-Neelsen stain is typically used to identify mycobacteria. They are not stained in the Gram staining process. Van Gieson and Masson trichrome are histological staining methods for identification of connective tissues. The Von Kossa technique is useful for identifying tissue mineralisation.
Tuberculosis pathology

Is a form of primary chronic inflammation, caused by the inability of macrophages to kill the Mycobacterium tuberculosis.
The macrophages often migrate to regional lymph nodes, the lung lesion plus affected lymph nodes is referred to as a Ghon complex.
This leads to the formation of a granuloma which is a collection of epithelioid histiocytes.
There is the presence of caseous necrosis in the centre.
The inflammatory response is mediated by a type 4 hypersensitivity reaction.
In healthy individuals the disease may be contained, in the immunocompromised disseminated (miliary TB) may occur.

Diagnosis
Waxy membrane of mycobacteria prevents binding with normal stains. Ziehl - Neelsen staining is typically used.
Culture based methods take far longer.

1243
Q

A 72 year old man presents with a large nodule on his face. It is friable. There is no regional lymphadenopathy. He is lost to follow up and re-attends several months later. On this occasion the lesion has been noted to resolve with scarring. What is the most likely lesion?

Pyogenic granuloma

Keratoacanthoma

Melanoma

Basal cell carcinoma

Dermatitis artifacta

A

Keratoacanthomas may reach a considerable size prior to sloughing off and scarring.

Skin Diseases

Skin lesions may be referred for surgical assessment, but more commonly will come via a dermatologist for definitive surgical management.

Skin malignancies include basal cell carcinoma, squamous cell carcinoma and malignant melanoma.

Basal Cell Carcinoma
Most common form of skin cancer.
Commonly occur on sun exposed sites apart from the ear.
Sub types include nodular, morphoeic, superficial and pigmented.
Typically slow growing with low metastatic potential.
Standard surgical excision, topical chemotherapy and radiotherapy are all successful.
As a minimum a diagnostic punch biopsy should be taken if treatment other than standard surgical excision is planned.

Squamous Cell Carcinoma
Again related to sun exposure.
May arise in pre - existing solar keratoses.
May metastasize if left.
Immunosupression (e.g. following transplant), increases risk.
Wide local excision is the treatment of choice and where a diagnostic excision biopsy has demonstrated SCC, repeat surgery to gain adequate margins may be required.

Malignant Melanoma
The main diagnostic features (major criteria):
Change in size
Change in shape
Change in colour
Secondary features (minor criteria)
Diameter >6mm
Inflammation
Oozing or bleeding
Altered sensation

Treatment
Suspicious lesions should undergo excision biopsy. The lesion should be removed completely as incision biopsy can make subsequent histopathological assessment difficult.
Once the diagnosis is confirmed the pathology report should be reviewed to determine whether further re-excision of margins is required (see below):

Margins of excision-Related to Breslow thickness
Lesions 0-1mm thick 1cm
Lesions 1-2mm thick 1- 2cm (Depending upon site and pathological features)
Lesions 2-4mm thick 2-3 cm (Depending upon site and pathological features)
Lesions >4 mm thick 3cm
Marsden J et al. Revised UK guidelines for management of Melanoma. Br J Dermatol 2010 163:238-256.

Further treatments such as sentinel lymph node mapping, isolated limb perfusion and block dissection of regional lymph node groups should be selectively applied.

Kaposi Sarcoma
Tumour of vascular and lymphatic endothelium.
Purple cutaneous nodules.
Associated with immuno supression.
Classical form affects elderly males and is slow growing.
Immunosupression form is much more aggressive and tends to affect those with HIV related disease.

Non malignant skin disease

Dermatitis Herpetiformis
Chronic itchy clusters of blisters.
Linked to underlying gluten enteropathy (coeliac disease).

Dermatofibroma
Benign lesion.
Firm elevated nodules.
Usually history of trauma.
Lesion consists of histiocytes, blood vessels and fibrotic changes.

Pyogenic granuloma
Overgrowth of blood vessels.
Red nodules.
Usually follow trauma.
May mimic amelanotic melanoma.

Acanthosis nigricans
Brown to black, poorly defined, velvety hyperpigmentation of the skin.
Usually found in body folds such as the posterior and lateral folds of the neck, the axilla, groin, umbilicus, forehead, and other areas.
The most common cause of acanthosis nigricans is insulin resistance, which leads to increased circulating insulin levels. Insulin spillover into the skin results in its abnormal increase in growth (hyperplasia of the skin).
In the context of a malignant disease, acanthosis nigricans is a paraneoplastic syndrome and is then commonly referred to as acanthosis nigricans maligna. Involvement of mucous membranes is rare and suggests a coexisting malignant condition.

1244
Q

How many fissures are present within the right lung?

One

Three

Two

Four

Five

A

The right lung has an oblique and horizontal fissure. The upper oblique fissure separates the inferior from the middle and upper lobes. The short horizontal fissure separates the superior and middle lobes.

Lung anatomy

The right lung is composed of 3 lobes divided by the oblique and transverse fissures. The left lung has two lobes divided by the oblique fissure.The apex of both lungs is approximately 4cm superior to the sterno-costal joint of the first rib. Immediately below this is a sulcus created by the subclavian artery.

Peripheral contact points of the lung
Base: diaphragm
Costal surface: corresponds to the cavity of the chest
Mediastinal surface: Contacts the mediastinal pleura. Has the cardiac impression. Above and behind this concavity is a triangular depression named the hilum, where the structures which form the root of the lung enter and leave the viscus. These structures are invested by pleura, which, below the hilum and behind the pericardial impression, forms the pulmonary ligament

Right lung
Above the hilum is the azygos vein; Superior to this is the groove for the superior vena cava and right innominate vein; behind this, and nearer the apex, is a furrow for the innominate artery. Behind the hilum and the attachment of the pulmonary ligament is a vertical groove for the oesophagus; In front and to the right of the lower part of the oesophageal groove is a deep concavity for the extrapericardiac portion of the inferior vena cava.

The root of the right lung lies behind the superior vena cava and the right atrium, and below the azygos vein.

The right main bronchus is shorter, wider and more vertical than the left main bronchus and therefore the route taken by most foreign bodies.

Left lung
Above the hilum is the furrow produced by the aortic arch, and then superiorly the groove accommodating the left subclavian artery; Behind the hilum and pulmonary ligament is a vertical groove produced by the descending aorta, and in front of this, near the base of the lung, is the lower part of the oesophagus.

The root of the left lung passes under the aortic arch and in front of the descending aorta.
Inferior borders of both lungs
6th rib in mid clavicular line
8th rib in mid axillary line
10th rib posteriorly
The pleura runs two ribs lower than the corresponding lung level.

Bronchopulmonary segments
Segment number Right lung Left lung
1 Apical Apical
2 Posterior Posterior
3 Anterior Anterior
4 Lateral Superior lingular
5 Medial Inferior lingular
6 Superior (apical) Superior (apical)
7 Medial basal Medial basal
8 Anterior basal Anterior basal
9 Lateral basal Lateral basal
10 Posterior basal Posterior basal

1245
Q

During a thyroidectomy the surgeons ligate the inferior thyroid artery. From which vessel does this structure usually originate?

External carotid artery

Thyrocervical trunk

Internal carotid artery

Subclavian artery

Vertebral artery

A

The inferior thyroid artery originates from the thyrocervical trunk. This is a branch of the subclavian artery.

Thyroid gland

Right and left lobes connected by isthmus
Surrounded by sheath from pretracheal layer of deep fascia
Apex: Lamina of thyroid cartilage
Base: 4th-5th tracheal ring
Pyramidal lobe: from isthmus
May be attached to foramen caecum at the base of the tongue

Relations
Anteromedially
Sternothyroid
Superior belly of omohyoid
Sternohyoid
Anterior aspect of sternocleidomastoid
Posterolaterally Carotid sheath
Medially
Larynx
Trachea
Pharynx
Oesophagus
Cricothyroid muscle
External laryngeal nerve (near superior thyroid artery)
Recurrent laryngeal nerve (near inferior thyroid artery)
Posterior
Parathyroid glands
Anastomosis of superior and inferior thyroid arteries
Isthmus
Anteriorly: Sternothyroids, sternohyoids, anterior jugular veins
Posteriorly: 2nd, 3rd, 4th tracheal rings (attached via Ligament of Berry)

Blood Supply
Arterial
Superior thyroid artery (1st branch of external carotid)
Inferior thyroid artery (from thyrocervical trunk)
Thyroidea ima (in 10% of population -from brachiocephalic artery or aorta)
Venous
Superior and middle thyroid veins - into the IJV
Inferior thyroid vein - into the brachiocephalic veins

1246
Q

Which of the following methods is most effective at destroying spores of the tubercle bacilli?

Immersion in 0.5% chlorhexidine in alcohol

Immersion in aqueous iodine

Heating in a hot air oven

Immersion in 0.1% sodium hypochlorite

Autoclaving

A

The tubercle bacilli has a waxy outer membrane that renders it more resistant to sterilisation and cleaning methods. Whilst 0.1% sodium hypochlorite will destroy many microbes it is less reliable in destroying tubercle bacilli. Hot air ovens provide less reliable pathogen destruction than autoclaving, but may be indicated in situations where the equipment is sensitive to the autoclaving process. From the list of options above, autoclaving will most reliably destroy tubercle bacilli.

Sterilisation

Surgical equipment has to be cleaned and sterilised prior to use. The extent to which these processes will be required varies according to the type of equipment and the purpose for which it will be used. In general, the three processes are relevant; cleaning, disinfection and sterilisation.
Cleaning refers to removal of physical debris.
Disinfection refers to reduction in numbers of viable organisms.
Sterilisation is removal of all organisms and spores.

Methods
Method Details Indication
Autoclaving Air removed and high pressure steam used (usually 134 oC for 3 minutes) Most reusable surgical equipment, must be physically cleaned prior to autoclaving, unsuitable for fragile items
Glutaraldehyde solution (2%) Colourless oily liquid, directly cytocidal and virucidal even at low temperatures Specifically used for endoscopes and some laparoscopic items, staff can rapidly develop allergy to this substance which has limited its more widespread use
Ethylene oxide 3% mixture of gas with carbon dioxide used Used for packaged materials that cannot be heated, the gas is explosive and environmentally toxic, it is used mainly in the industrial setting
Gamma irradiation Gamma rays emitted from radioactive substance such as cobalt 60 or caesium 137 Suitable for batch treatment of relatively thermostable items, typically an industrial process

1247
Q

A 53 year old man is referred because he was being investigated with an ultrasound scan for gallstone disease. Whilst a large gallstone was seen in a thin walled gallbladder, a lesion was also seen in the left lobe of the liver. On MRI this has been categorised as a 6cm hepatocellular adenoma. What is the most appropriate course of action?

Discharge the patient

Surveillance with USS and measurement of alpha fetoprotein at 6 monthly intervals

Surgical resection

Further USS at 12 months and discharge if no changes seen

CT guided biopsy of the lesion

A

Hepatocellular adenomas in males have a greater risk of malignant transformation and resection should be considered.

Hepatocellular adenoma

90% develop in women in their third to fifth decade
Linked to use of oral contraceptive pill
Lesions are usually solitary
They are usually sharply demarcated from normal liver although they usually lack a fibrous capsule
On ultrasound the appearances are of mixed echoity and heterogeneous texture. On CT most lesions are hypodense when imaged prior to administration of IV contrast agents
In patients with haemorrhage or symptoms removal of the adenoma may be required
Asymptomatic adenomas >5cm are usually excised
Adenomas in males are likely to be smaller but have a greater risk of malignant transformation
- Risk of rupture is greater in lesions larger than 5cm and those which are exophytic. Mortality rates from spontaneous rupture are 5-10%.

1248
Q

A 42 year old lady presents with right upper quadrant pain and a sensation of abdominal fullness. An ultrasound scan demonstrates a 6.5 cm hyperechoic lesion in the right lobe of the liver. Serum AFP is normal. What is the most likely underlying lesion?

Liver cyst

Haemangioma

Hepatocellular carcinoma

Liver metastasis

Liver cell adenoma

A

A large hyperechoic lesion in the presence of normal AFP is likely to be a haemangioma. An HCC of equivalent size will almost always result in rise in AFP.

Benign liver lesions

Benign liver lesions
Haemangioma
Most common benign tumours of mesenchymal origin
Incidence in autopsy series is 8%
Cavernous haemangiomas may be enormous
Clinically they are reddish purple hypervascular lesions
Lesions are normally separated from normal liver by ring of fibrous tissue
On ultrasound they are typically hyperechoic
Liver cell adenoma
90% develop in women in their third to fifth decade
Linked to use of oral contraceptive pill
Lesions are usually solitary
They are usually sharply demarcated from normal liver although they usually lack a fibrous capsule
On ultrasound the appearances are of mixed echoity and heterogeneous texture. On CT most lesions are hypodense when imaged prior to administration of IV contrast agents
In patients with haemorrhage or symptoms removal of the adenoma may be required
Mesenchymal hamartomas Congential and benign, usually present in infants. May compress normal liver
Liver abscess
Biliary sepsis is a major predisposing factor
Structures drained by the portal venous system form the second largest source
Common symptoms include fever, right upper quadrant pain. Jaundice may be seen in 50%
Ultrasound will usually show a fluid filled cavity, hyperechoic walls may be seen in chronic abscesses
Amoebic abscess
Liver abscess is the most common extra intestinal manifestation of amoebiasis
Between 75 and 90% lesions occur in the right lobe
Presenting complaints typically include fever and right upper quadrant pain
Ultrasonography will usually show a fluid filled structure with poorly defined boundaries
Aspiration yield sterile odourless fluid which has an anchovy paste consistency
Treatment is with metronidazole
Hyatid cysts
Seen in cases of Echinococcus infection
Typically an intense fibrotic reaction occurs around sites of infection
The cyst has no epithelial lining
Cysts are commonly unilocular and may grow to 20cm in size. The cyst wall is thick and has an external laminated hilar membrane and an internal enucleated germinal layer
Typically presents with malaise and right upper quadrant pain. Secondary bacterial infection occurs in 10%.
Liver function tests are usually abnormal and eosinophilia is present in 33% cases
Ultrasound may show septa and hyatid sand or daughter cysts.
Percutaneous aspiration was previously contra indicated, it is now incorporated into some treatment regimens
Treatment is by sterilisation of the cyst with mebendazole and may be followed by surgical resection. Hypertonic swabs are packed around the cysts during surgery
Polycystic liver disease
Usually occurs in association with polycystic kidney disease
Autosomal dominant disorder
Symptoms may occur as a result of capsular stretch
Cystadenoma
Rare lesions with malignant potential
Usually solitary multiloculated lesions
Liver function tests usually normal
Ultrasonography typically shows a large anechoic, fluid filled area with irregular margins. Internal echos may result from septa
Surgical resection is indicated in all cases

1249
Q

A 55 year old man presents with an episode of frank haematuria and on investigation is found to have a T2 transitional cell carcinoma of the bladder. His staging investigations are negative for metastatic disease. What is the most appropriate treatment?

Radical cystectomy

Palliative radiotherapy

Intravesical BCG

Intravesical mitomycin C

Intravesical cisplatin

A

T2 lesions in a young fit patient are best managed surgically. Up to 25 % patients may develop perioperative complications. However, palliative treatments and intravesical chemotherapy (which does NOT include cisplatin) are not used curatively in this situation.

Bladder cancer

Bladder cancer is the second most common urological cancer. It most commonly affects males aged between 50 and 80 years of age. Those who are current, or previous (within 20 years), smokers have a 2-5 fold increased risk of the disease. Exposure to hydrocarbons such as 2-Naphthylamine increases the risk. Although rare in the UK, chronic bladder inflammation arising from Schistosomiasis infection remains a common cause of squamous cell carcinomas, in those countries where the disease is endemic.

Benign tumours
Benign tumours of the bladder including inverted urothelial papilloma and nephrogenic adenoma are uncommon.

Bladder malignancies
Transitional cell carcinoma (>90% of cases)
Squamous cell carcinoma ( 1-7% -except in regions affected by schistosomiasis)
Adenocarcinoma (2%)

Transitional cell carcinomas may arise as solitary lesions, or may be multifocal, owing to the effect of ‘field change’ within the urothelium. Up to 70% of TCC’s will have a papillary growth pattern. These tumours are usually superficial in location and accordingly have a better prognosis. The remaining tumours show either mixed papillary and solid growth or pure solid growths. These tumours are typically more prone to local invasion and may be of higher grade, the prognosis is therefore worse. Those with T3 disease or worse have a 30% (or higher) risk of regional or distant lymph node metastasis.

TNM Staging
Stage Description
T0 No evidence of tumour
Ta Non invasive papillary carcinoma
T1 Tumour invades sub epithelial connective tissue
T2a Tumor invades superficial muscularis propria (inner half)
T2b Tumor invades deep muscularis propria (outer half)
T3 Tumour extends to perivesical fat
T4 Tumor invades any of the following: prostatic stroma, seminal vesicles, uterus, vagina
T4a Invasion of uterus, prostate or bowel
T4b Invasion of pelvic sidewall or abdominal wall
N0 No nodal disease
N1 Single regional lymph node metastasis in the true pelvis (hypogastric, obturator, external iliac, or presacral lymph node)
N2 Multiple regional lymph node metastasis in the true pelvis (hypogastric, obturator, external iliac, or presacral lymph node metastasis)
N3 Lymph node metastasis to the common iliac lymph nodes
M0 No distant metastasis
M1 Distant disease

Presentation
Most patients (85%) will present with painless, macroscopic haematuria. In those patients with incidental microscopic haematuria, up to 10% of females aged over 50 will be found to have a malignancy (once infection excluded).

Staging
Most will undergo a cystoscopy and biopsies or TURBT, this provides histological diagnosis and information relating to depth of invasion. Locoregional spread is best determined using pelvic MRI and distant disease CT scanning. Nodes of uncertain significance may be investigated using PET CT.

Treatment
Those with superficial lesions may be managed using TURBT in isolation. Those with recurrences or higher grade/ risk on histology may be offered intravesical chemotherapy. Those with T2 disease are usually offered either surgery (radical cystectomy and ileal conduit) or radical radiotherapy.

Prognosis
T1 90%
T2 60%
T3 35%
T4a 10-25%
Any T, N1-N2 30%

1250
Q

A 69 year old man presents with a purple lesion on his forearm. It is excised and a 3 cm Merkel cell tumour is diagnosed. Which of the following statements relating to this diagnosis is false?

He should undergo a sentinel lymph node biopsy.

Lymphovascular invasion is typically seen histologically

They are more common in immunosupressed patients

Histologically they may resemble pyogenic granuloma

They are associated with visceral metastasis

A

Merkel cell tumours are rare cutaneous tumours. Histologically they consist of sheets and nodules of hyperchromatic epithelial cells, with high rates of mitosis and apoptosis. As such they are relatively easy to distinguish from pyogenic granuloma which has no features of malignancy and would not show lymphovascular invasion.

Merkel cell tumours of the skin

Rare but aggressive tumour.
Develops from intra epidermal Merkel cells.
Usually presents on elderly, sun damaged skin. The periorbital area is the commonest site.
Histologically these tumours appear within the dermis and subcutis. The lesions consist of sheets and nodules of small hyperchromatic epithelial cells with high rates of mitosis and apoptosis. Lymphovascular invasion is commonly seen.
Pre-existing infection with Merkel Cell Polyomavirus is seen in 80% cases.

Treatment
Surgical excision is first line. Margins of 1cm are required. Lesions >10mm in diameter should undergo sentinel lymph node biopsy. Adjuvant radiotherapy is often given to reduce the risk of local recurrence.

Prognosis
With lymph node metastasis 5 year survival is 50% or less.
Small lesions without nodal spread are usually associated with a 5 year survival of 80%.

1251
Q

A 45 year old man is stabbed in the abdomen and the inferior vena cava is injured. How many functional valves does this vessel usually have?

0

1

3

2

4

A

Mnemonic for the Inferior vena cava tributaries: I Like To Rise So High:

Iliacs
Lumbar
Testicular
Renal
Suprarenal
Hepatic vein
The lack of valves in the IVC is important clinically when it is cannulated during cardiopulmonary bypass, using separate SVC and IVC catheters, such as when the right atrium is to be opened. Note that there is a non functional valve between the right atrium and inferior vena cava.
Inferior vena cava

Origin
L5

Path
Left and right common iliac veins merge to form the IVC.
Passes right of midline
Paired segmental lumbar veins drain into the IVC throughout its length
The right gonadal vein empties directly into the cava and the left gonadal vein generally empties into the left renal vein.
The next major veins are the renal veins and the hepatic veins
Pierces the central tendon of diaphragm at T8
Right atrium

Relations
Anteriorly Small bowel, first and third part of duodenum, head of pancreas, liver and bile duct, right common iliac artery, right gonadal artery
Posteriorly Right renal artery, right psoas, right sympathetic chain, coeliac ganglion

Levels
Level Vein
T8 Hepatic vein, inferior phrenic vein, pierces diaphragm
L1 Right suprarenal vein, renal vein
L2 Gonadal vein
L1-5 Lumbar veins
L5 Common iliac vein, formation of IVC

1252
Q

A 55 year old man presents with tenesmus and rectal bleeding. On examination he has a large bulky rectal cancer at 5cm from the anal verge with tethering to the prostate gland. Imaging shows no distant disease. What is the most appropriate initial treatment modality?

Abdomino-perineral resection of the colon and rectum

Pelvic exenteration

Abdomino-perineal excision of the colon and rectum with prostatectomy

Long course chemoradiotherapy

Short course radiotherapy

A

Rectal cancers with threatened resection margins are managed with radiotherapy and chemotherapy initially. This is not the case with colonic cancers which are usually primarily resected.
T4 rectal cancers are managed with long course chemoradiotherapy. A dramatic response is not uncommon. To embark on attempted resection at this stage is to court failure.

Colorectal cancer treatment

Patients diagnosed as having colorectal cancer should be completely staged using CT of the chest/ abdomen and pelvis. Their entire colon should have been evaluated with colonoscopy or CT colonography. Patients whose tumours lie below the peritoneal reflection should have their mesorectum evaluated with MRI.

Once their staging is complete patients should be discussed within a dedicated colorectal MDT meeting and a treatment plan formulated.

Treatment of colonic cancer
Cancer of the colon is nearly always treated with surgery. Stents, surgical bypass and diversion stomas may all be used as palliative adjuncts. Resectional surgery is the only option for cure in patients with colon cancer. The procedure is tailored to the patient and the tumour location. The lymphatic drainage of the colon follows the arterial supply and therefore most resections are tailored around the resection of particular lymphatic chains (e.g. ileo-colic pedicle for right sided tumours). Some patients may have confounding factors that will govern the choice of procedure, for example a tumour in a patient from a HNPCC family may be better served with a panproctocolectomy rather than segmental resection. Following resection the decision has to be made regarding restoration of continuity. For an anastomosis to heal the key technical factors include; adequate blood supply, mucosal apposition and no tissue tension. Surrounding sepsis, unstable patients and inexperienced surgeons may compromise these key principles and in such circumstances it may be safer to construct an end stoma rather than attempting an anastomosis.
When a colonic cancer presents with an obstructing lesion; the options are to either stent it or resect. In modern practice it is unusual to simply defunction a colonic tumour with a proximal loop stoma. This differs from the situation in the rectum (see below).
Following resection patients with risk factors for disease recurrence are usually offered chemotherapy, a combination of 5FU and oxaliplatin is common.

Rectal cancer
The management of rectal cancer is slightly different to that of colonic cancer. This reflects the rectum’s anatomical location and the challenges posed as a result. Tumours located in the rectum can be surgically resected with either an anterior resection or an abdomino - perineal resection. The technical aspects governing the choice between these two procedures can be complex to appreciate and the main point to appreciate for the MRCS is that involvement of the sphincter complex or very low tumours require APER. In the rectum a 2cm distal clearance margin is required and this may also impact on the procedure chosen. In addition to excision of the rectal tube an integral part of the procedure is a meticulous dissection of the mesorectal fat and lymph nodes (total mesorectal excision/ TME). In rectal cancer surgery invovlement of the cirumferential resection margin carries a high risk of disease recurrence. Because the rectum is an extraperitoneal structure (until you remove it that is!) it is possible to irradiate it, something which cannot be offered for colonic tumours. This has a major impact in rectal cancer treatment and many patients will be offered neoadjuvent radiotherapy (both long and short course) prior to resectional surgery. Patients with T1, 2 and 3 /N0 disease on imaging do not require irradiation and should proceed straight to surgery. Patients with T4 disease will typically have long course chemo radiotherapy. Patients presenting with large bowel obstruction from rectal cancer should not undergo resectional surgery without staging as primary treatment (very different from colonic cancer). This is because rectal surgery is more technically demanding, the anastomotic leak rate is higher and the danger of a positive resection margin in an unstaged patient is high. Therefore patients with obstructing rectal cancer should have a defunctioning loop colostomy.

Summary of procedures
The operations for cancer are segmental resections based on blood supply and lymphatic drainage. These commonly performed procedures are core knowledge for the MRCS and should be understood.

Site of cancer Type of resection Anastomosis Risk of leak
Right colon Right hemicolectomy Ileo-colic Low <5%
Transverse Extended right hemicolectomy Ileo-colic Low <5%
Splenic flexure Extended right hemicolectomy Ileo-colic Low <5%
Splenic flexure Left hemicolectomy Colo-colon 2-5%
Left colon Left hemicolectomy Colo-colon 2-5%
Sigmoid colon High anterior resection Colo-rectal 5%
Upper rectum Anterior resection (TME) Colo-rectal 5%
Low rectum Anterior resection (Low TME) Colo-rectal
(+/- Defunctioning stoma) 10%
Anal verge Abdomino-perineal excision of colon and rectum None n/a

In the emergency setting, where the bowel has perforated, the risk of an anastomotic breakdown is much greater, particularly when the anastomosis is colon-colon. In this situation, an end colostomy is often safer and can be reversed later. When resection of the sigmoid colon is performed and an end colostomy is fashioned the operation is referred to as a Hartmans procedure. Whilst left sided resections are more risky, ileo-colic anastomoses are relatively safe even in the emergency setting and do not need to be defunctioned.

References
A review of the diagnosis and management of colorectal cancer and a summary of the UK National Institute of Clinical Excellence guidelines is provided in:
NICE Guideline NG151. Last updated in 2020.

1253
Q

A 24 year old motor cyclist is involved in a road traffic accident. He suffers a tibial fracture which is treated with an intra medullary nail. Post operatively he develops a compartment syndrome. Surgical decompression of the anterior compartment will relieve pressure on all of the following muscles except?

Peroneus brevis

Peroneus tertius

Extensor digitorum longus

Tibialis anterior

None of the above

A

A- The anterior compartment contains:
Tibialis anterior
Extensor digitorum longus
Peroneus tertius
Extensor hallucis longus
Anterior tibial artery
All the muscles are innervated by the deep peroneal nerve.

Lower limb- Muscular compartments

Anterior compartment
Muscle Nerve Action
Tibialis anterior Deep peroneal nerve Dorsiflexes ankle joint, inverts foot
Extensor digitorum longus Deep peroneal nerve Extends lateral four toes, dorsiflexes ankle joint
Peroneus tertius Deep peroneal nerve Dorsiflexes ankle, everts foot
Extensor hallucis longus Deep peroneal nerve Dorsiflexes ankle joint, extends big toe

Peroneal compartment
Muscle Nerve Action
Peroneus longus Superficial peroneal nerve Everts foot, assists in plantar flexion
Peroneus brevis Superficial peroneal nerve Plantar flexes the ankle joint

Superficial posterior compartment
Nerve Action
Gastrocnemius Tibial nerve Plantar flexes the foot, may also flex the knee
Soleus Tibial nerve Plantar flexor

Deep posterior compartment
Muscle Nerve Action
Flexor digitorum longus Tibial Flexes the lateral four toes
Flexor hallucis longus Tibial Flexes the great toe
Tibialis posterior Tibial Plantar flexor, inverts the foot

1254
Q

Which of the following is not a branch of the hepatic artery?

Pancreatic artery

Cystic artery

Right gastric artery

Right hepatic artery

Gastroduodenal artery

A

The pancreatic artery is a branch of the splenic artery.

Coeliac axis

The coeliac axis has three main branches.
Left gastric
Hepatic: branches-Right Gastric, Gastroduodenal, Superior Pancreaticoduodenal, Cystic (occasionally).
Splenic: branches- Pancreatic, Short Gastric, Left Gastroepiploic

It occasionally gives off one of the inferior phrenic arteries.

Relations
Anteriorly Lesser omentum
Right Right coeliac ganglion and caudate process of liver
Left Left coeliac ganglion and gastric cardia
Inferiorly Upper border of pancreas and renal vein

1255
Q

A 45 year old man presents with a facial swelling. On examination, he has a swelling that lies inferolaterally to the nose. When the area is palpated, it feels like the underlying bone is cracking. What is the most likely diagnosis?

Maxillary sinus cancer

Ameloblastoma

Nasal polyps

Maxillary sinusitis

Mucocele

A

Ameloblastomas are rare tumours of the odontogenic epithelium. They are slow growing and expand with a rim of periosteum that surrounds them. It is the palpation and disruption of this layer that gives rise to the crepitus.

Diseases of nose and sinuses

Benign Tumours
Simple papillomas may be an incidental finding or present with obstructive symptoms. Excision under general anaesthesia is sufficient management.
Transitional cell papillomas may be more extensive and produce obstructive symptoms. Erosion of local structures is a recognised complication. These lesions may rarely undergo malignant transformation and therefore careful and complete excision is required, some cases may require partial or total maxillectomy.
Pleomorphic adenomas of the maxillary sinuses are reported but are extremely rare, their symptoms typically include nasal obstruction and pain if the sinus is obstructed. Treatment is by complete surgical excision, the diagnosis is not infrequently made post operatively.
Benign osteomas may develop in the paranasal sinuses, the frontal sinus is the most frequent location of such lesions. Symptoms include; pain, rhinorrhoea and anosmia. Most osteomas may be observed if asymptomatic, sphenoid osteomas should be resected soon after diagnosis as enlargement may compromise visual fields. Many sinus osteomas can now be resected endoscopically, complete surgical resection is required.
Nasal polyps are benign lesions of the ethmoid sinus mucosa. Many patients may also have asthma, cystic fibrosis and a sensitivity to aspirin. Symptoms include watery rhinorrhoea, infection and anosmia. The polyps are usually a semi transparent grey mass. They are rare in childhood. Treatment is either with systemic steroids or surgical resection. The latter should be combined with antral washout. Low dose, nasal, steroid drops may reduce the risk of recurrence.

Malignant disease
Malignancies encountered in the nose and paranasal sinuses include; adenoid cystic carcinoma, squamous cell carcinoma and adenocarcinoma.
Adenocarcinoma of the paranasal sinuses and nasopharynx is strongly linked to exposure to hard wood dust (after >10 years exposure).
Adenoid cystic carcinoma usually originate in the smaller salivary glands.
The majority of cancers (50%) arise from the lateral nasal wall, a smaller number (33%) arise from the maxillary antrum, ethmoid and sphenoid cancers comprise only 7%.
Signs of malignancy on clinical examination include loose teeth, cranial nerve palsies and lymphadenopathy.
Nasopharyngeal cancers are most common in individuals presenting from China and Asia and are linked to viral infection with Epstein Barr Virus. Radiotherapy and chemotherapy are the most commonly used modalities.

Maxillary sinusitis
Common symptoms include post nasal discharge, pain, headache and toothache.
Imaging may show a fluid level in the antrum.
Common organisms include Haemophilus influenzae or Streptococcus pneumoniae.
Treatment with antral lavage may facilitate diagnosis and relieve symptoms. Antimicrobial therapy has to be continued for long periods. Antrostomy may be needed.

Frontoethmoidal sinusitis
Usually presents with frontal headache, nasal obstruction and altered sense of smell.
Inflammation may progress to involve periorbital tissues. Ocular symptoms may occur and secondary CNS involvement brought about by infection entering via emissary veins.
CT scanning is the imaging modality of choice. Early cases may be managed with antibiotics. More severe cases usually require surgical drainage.

1256
Q

Which of the following is not a feature of Wallerian Degeneration?

May result from an axonotmesis

Typically occurs in the peripheral nervous systems

The axon remains excitable throughout the whole process

The distal neuronal stump is affected

Is a component of the healing process following neuronal injury

A

The axon loses its excitability once the process is established.

Wallerian degeneration

  • Is the process that occurs when a nerve is cut or crushed.
  • It occurs when the part of the axon separated from the neuron’s cell nucleus degenerates.
  • It usually begins 24 hours following neuronal injury and the distal axon remains excitable up until this time.
  • The degeneration of the axon is followed by breakdown of the myelin sheath, a process that occurs by infiltration of the site with macrophages.
  • Eventually regeneration of the nerve may occur although recovery will depend on the extent and manner of injury
1257
Q

A 45 year old man with end stage renal failure undergoes a cadaveric renal transplant. The transplanted organ has a cold ischaemic time of 26 hours and a warm ischaemic time of 54 minutes. Post operatively the patient receives immunosuppressive therapy. Ten days later the patient has gained weight, becomes oliguric and feels systemically unwell. He also complains of swelling over the transplant site that is painful. What is the most likely cause?

Acute tubular necrosis

Hyperacute rejection

Ureteric occlusion

Acute on chronic rejection

Acute rejection

A

The features described are those of worsening graft function and acute rejection. The fact that there is a 10 day delay goes against hyperacute rejection. Cold ischaemic times are a major factor for delayed graft function. However, even 26 hours is not incompatible with graft survival.

Complications following renal transplant

Renal transplantation is widely practised. The commonest technical related complications are related to the ureteric anastomosis. The warm ischaemic time is also of considerable importance and graft survival is directly related to this. Long warm ischaemic times increase the risk of acute tubular necrosis which may occur in all types of renal transplanation and provided other insults are minimised, will usually recover. Organ rejection may occur at any phase following the transplantation process.

Immunological complications
Types of organ rejection
Hyperacute. This occurs immediately through presence of pre formed antibody (such as ABO incompatibility).
Acute. Occurs during the first 6 months and is usually T cell mediated. Usually tissue infiltrates and vascular lesions.
Chronic. Occurs after the first 6 months. Vascular changes predominate.

Hyperacute
Renal transplants are most susceptible to this process. Risk factors include major HLA mismatch and ABO incompatibility. The rejection occurs almost immediately and the macroscopic features may become manifest following completion of the vascular anastomosis and removal of clamps. The kidney becomes mottled, dusky and the vessels will thrombose. The only treatment is removal of the graft, if left in situ it will result in abscess formation.

Acute
All organs may undergo acute rejection. Mononuclear cell infiltrates predominate. All types of transplanted organ are susceptible and it may occur in up to 50% cases. Most cases can be managed medically.

Chronic
Again all transplants with HLA mismatch may suffer this fate. Previous acute rejections and other immunosensitising events all increase the risk. Vascular changes are most prominent with myointimal proliferation leading to organ ischaemia. Organ specific changes are also seen such as loss of acinar cells in pancreas transplants and rapidly progressive coronary artery disease in cardiac transplants.

Technical complications
Complication Presenting features Treatment
Renal artery thrombosis Sudden complete loss of urine output Immediate surgery may salvage the graft, delays beyond 30 minutes are associated with a high rate of graft loss
Renal artery stenosis Uncontrolled hypertension, allograft dysfunction and oedema Angioplasty is the treatment of choice
Renal vein thrombosis Pain and swelling over the graft site, haematuria and oliguria The graft is usually lost
Urine leaks Diminished urine output, rising creatinine, fever and abdominal pain USS shows perigraft collection, necrosis of ureter tip is the commonest cause and the anastomosis may need revision
Lymphocele Common complication (occurs in 15%), may present as a mass, if large may compress ureter May be drained with percutaneous technique and sclerotherapy, or intraperitoneal drainage

1258
Q

Which of the interventions listed below would not be conventional treatment for a venous ulcer?

Compression bandages

Limb elevation to control oedema

Optimisation of glycaemic control of diabetic

Use of skin grafts in selected cases

Excision and closure in selected cases

A

Most venous ulcers are managed with compression bandages. Skin grafting is sometimes used. However, ulcer excision is not standard practice and closure would be impossible.

Chronic venous insufficiency and varicose veins

Wide spectrum of disease ranging from minor cosmetic problem through to ulceration and disability. It is commoner in women than men and is worse during pregnancy. Varicose veins are best considered as being a saccular dilation of veins (WHO). Chronic venous insufficiency is a series of tissue changes which occur in relation to pooling of blood in the extremities with associated venous hypertension occurring as a result of incompetent deep vein valves.

The veins of the lower limb consist of an interconnected network of superficial and deep venous systems. Varices occur because of localised weakness in the vein wall resulting in dilatation and reflux of blood due to non union of valve cusps. Histologically, the typical changes include fibrous scar tissue dividing smooth muscle within media in the vessel wall.

Tissue damage in chronic venous insufficiency occurs because of perivascular cytokine leakage resulting in localised tissue damage coupled with impaired lymphatic flow.

Diagnosis
Typical symptoms of varicose veins include:
Cosmetic appearance
Aching
Ankle swelling that worsens as the day progresses
Episodic thrombophlebitis
Bleeding
Itching

Symptoms of chronic venous insufficiency include:
Dependant leg pain
Prominent leg swelling
Oedema extending beyond the ankle
Venous stasis ulcers

The typical venous stasis ulcer is:
Located above the medial malleolus
Indolent appearance with basal granulation tissue
Variable degree of scarring
Non ischaemic edges
Haemosiderin deposition in the gaiter area (and also lipodermatosclerosis).

Differential diagnosis
Lower limb arterial disease
Marjolins ulcer
Claudication
Spinal stenosis
Swelling due to medical causes e.g. CCF.

Exclusion of these differentials is by means of physical examination and ankle brachial pressure index measurement.

Examination
Assess for dilated short saphenous vein (popliteal fossa) and palpate for saphena varix medial to the femoral artery
Brodie-Trendelenburg test: to assess level of incompetence
Perthes’ walking test: assess if deep venous system competent

Investigation
Doppler exam: if incompetent a biphasic signal due to retrograde flow is detected
Duplex scanning: to ensure patent deep venous system (do if DVT or trauma)

All patients should have a Doppler assessment to assess for venous reflux and should be classified as having uncomplicated varicose veins or varicose veins with associated chronic venous insufficiency. In the history establishing a previous thrombotic event (DVT/ lower limb fracture) is important and patients with such a history and all who have evidence of chronic venous insufficiency should have a duplex scan performed.

Owing to litigation patients with saphenopopliteal incompetence should have a duplex scan performed and the site marked by scan on the day of surgery.

Treatment
Indications for surgery:
Cosmetic: majority
Lipodermatosclerosis causing venous ulceration
Recurrent superficial thrombophlebitis
Bleeding from ruptured varix

Condition Therapy
Minor varicose veins - no complications Reassure/ cosmetic therapy
Symptomatic uncomplicated varicose veins In those without deep venous insufficiency options include; endothermal ablation, foam sclerotherapy, saphenofemoral / popliteal disconnection, stripping and avulsions, compression stockings
Varicose veins with skin changes Therapy as above (if compression minimum is formal class I stockings)
Chronic venous insufficiency or ulcers Class 2-3 compression stockings (ensure no arterial disease).

Application of formal compression stockings (usually class II/III). In patients who have suffered ulceration, compression stockings should be worn long term. Where ulceration is present and established saphenofemoral reflux exists this should be addressed surgically for durable relief of symptoms, either at the outset or following ulcer healing.
Injection sclerotherapy (5% Ethanolamine oleate), foam is increasingly popular, though transient blindness has been reported. Endo venous laser therapy is another minimally invasive option
Sapheno-femoral or sapheno-popliteal ligation, in the case of the LSV; stripping and multiple phlebectomies

Current best practice guidance
In the United Kingdom the National Institute of Clinical Excellence guidance on varicose veins suggests that for patients with symptomatic varicose veins the first line procedure of choice should be endothermal ablation (see reference for more information). Where this is unavailable or unsuitable then foam sclerotherapy should be the second line option. Surgery is currently the third line treatment option.

Trendelenburg procedure (sapheno-femoral junction ligation)
Head tilt 15 degrees and legs abducted
Oblique incision 1cm medial from artery
Tributaries ligated (Superficial circumflex iliac vein, Superficial inferior epigastric vein, Superficial and deep external pudendal vein)
SF junction double ligated
Saphenous vein stripped to level of knee/upper calf. NB increased risk of saphenous neuralgia if stripped more distally

References
Marsden G et al. Diagnosis of management of varicose veins in the the legs: summary of NICE guidance. BMJ 2013 (347): 30-31.

1259
Q

Which of the following is true in connection with the phrenic nerves?

They both lie anterior to the hilum of the lungs

They are derived from spinal roots C 2,3,4

They pierce the diaphragm at the level of T7

They consist of motor fibres only

None of the above

A

C3,4,5
Keeps the diaphragm alive!
They both lie anterior to the hilum of the lung. The phrenic nerves have both motor and sensory functions. For this reason sub diaphragmatic pathology may cause referred pain to the shoulder.

Phrenic nerve

Origin
C3,4,5

Supplies
Diaphragm, sensation central diaphragm and pericardium

Path
The phrenic nerve passes with the internal jugular vein across scalenus anterior. It passes superficial to prevertebral fascia of deep cervical fascia.
Left: crosses anterior to the 1st part of the subclavian artery.
Right: Anterior to scalenus anterior and crosses anterior to the 2nd part of the subclavian artery.
On both sides, the phrenic nerve runs posterior to the subclavian vein and posterior to the internal thoracic artery as it enters the thorax.

Right phrenic nerve
In the superior mediastinum: anterior to right vagus and laterally to superior vena cava
Middle mediastinum: right of pericardium
It passes over the right atrium to exit the diaphragm at T8

Left phrenic nerve
Passes lateral to the left subclavian artery, aortic arch and left ventricle
Passes anterior to the root of the lung
Pierces the diaphragm alone

1260
Q

A 63 year old man undergoes a subtotal colectomy and iatrogenic injury to both ureters is sustained. He develops renal failure and his serum potassium is found to be elevated at 6.9 mmol/L. An ECG is performed, what is the most likely finding?

Increased PR interval

Prominent U waves

Narrow QRS complexes

Peaked T waves

Low ST segments

A

Peaked T waves are the first and most common finding in hyperkalaemia.

ECG features in hyperkalaemia

Peaking of T waves (occurs first)
Loss of P waves
Broad QRS complexes
Ventricullar fibrillation

1261
Q

A dentist treating a woman with rheumatoid arthritis for recurrent episodes of dental sepsis notices that both parotid and submandibular glands are symmetrically enlarged. What is the most likely diagnosis?

Sjogren’s syndrome

Mikulicz’s syndrome

Alcoholism

Liver cirrhosis

Metastatic cancer

A

Sjogren’s is associated with autoimmune disorders. Mikulicz’s is similar but there is no sicca or arthritis.

Neck lumps

The table below gives characteristic exam question features for conditions causing neck lumps:

Reactive lymphadenopathy By far the most common cause of neck swellings. There may be a history of local infection or a generalised viral illness
Lymphoma Rubbery, painless lymphadenopathy
The phenomenon of pain whilst drinking alcohol is very uncommon
There may be associated night sweats and splenomegaly
Thyroid swelling May be hypo-, eu- or hyperthyroid symptomatically
Moves upwards on swallowing
Thyroglossal cyst More common in patients < 20 years old
Usually midline, between the isthmus of the thyroid and the hyoid bone
Moves upwards with protrusion of the tongue
May be painful if infected
Pharyngeal pouch More common in older men
Represents a posteromedial herniation between thyropharyngeus and cricopharyngeus muscles
Usually not seen, but if large then a midline lump in the neck that gurgles on palpation
Typical symptoms are dysphagia, regurgitation, aspiration and chronic cough
Cystic hygroma A congenital lymphatic lesion (lymphangioma) typically found in the neck, classically on the left side
Most are evident at birth, around 90% present before 2 years of age
Branchial cyst An oval, mobile cystic mass that develops between the sternocleidomastoid muscle and the pharynx
Develop due to failure of obliteration of the second branchial cleft in embryonic development
Usually present in early adulthood
Cervical rib More common in adult females
Around 10% develop thoracic outlet syndrome
Carotid aneurysm Pulsatile lateral neck mass which doesn’t move on swallowing

1262
Q

A 44 year old man is undergoing a parotidectomy and the surgeon is carefully preserving the facial nerve. Unfortunately his trainee then proceeds to divide it. Which of the following will not be affected as a result?

Taste sensation from anterior two thirds of the tongue

Closing the ipsilateral eyelid

Raising the ipsilateral side of the lip

Ipsilateral corneal reflex

Buccinator muscle

A

The chorda tympani branches inside the facial canal and will therefore be unaffected by this most unfortunate event! The corneal reflex is mediated by the opthalmic branch of the trigeminal nerve sensing the stimulus on the cornea, lid or conjunctiva; the facial nerve initiates the motor response of the reflex.

Facial nerve

The facial nerve is the main nerve supplying the structures of the second embryonic branchial arch. It is predominantly an efferent nerve to the muscles of facial expression, digastric muscle and also to many glandular structures. It contains a few afferent fibres which originate in the cells of its genicular ganglion and are concerned with taste.

Supply - ‘face, ear, taste, tear’
Face: muscles of facial expression
Ear: nerve to stapedius
Taste: supplies anterior two-thirds of tongue
Tear: parasympathetic fibres to lacrimal glands, also salivary glands

Path
Subarachnoid path
Origin: motor- pons, sensory- nervus intermedius
Pass through the petrous temporal bone into the internal auditory meatus with the vestibulocochlear nerve. Here they combine to become the facial nerve.

Facial canal path
The canal passes superior to the vestibule of the inner ear
At the medial aspect of the middle ear, it becomes wider and contains the geniculate ganglion.
- 3 branches:
1. greater petrosal nerve
2. nerve to stapedius
3. chorda tympani

Stylomastoid foramen
Passes through the stylomastoid foramen (tympanic cavity anterior and mastoid antrum posteriorly)
Posterior auricular nerve and branch to posterior belly of digastric and stylohyoid muscle

Face
Enters parotid gland and divides into 5 branches:
Temporal branch
Zygomatic branch
Buccal branch
Marginal mandibular branch
Cervical branch

1263
Q

What is the substrate of renin?

Aldosterone

Angiotensinogen

Angiotensin converting enzyme

Angiotensin I

Angiotensin II

A

Renin hydrolyses angiotensinogen to form angiotensin I.

Renin

Renin is secreted by juxtaglomerular cells and hydrolyses angiotensinogen to produce angiotensin I

Factors stimulating renin secretion
Hypotension causing reduced renal perfusion
Hyponatraemia
Sympathetic nerve stimulation
Catecholamines
Erect posture

Factors reducing renin secretion
Drugs: beta-blockers, NSAIDs

1264
Q

A 60 year old alcoholic presents with worsening confusion over 2 weeks. He has weakness of the left side of the body. What is the least likely explanation?

Wernicke’s encephalopathy and CVA

Extra dural haematoma

Decompensated liver failure and CVA

Sub dural haematoma

Vascular dementia

A

If you answered the question incorrectly, check you have read it properly as it asks for the least likely cause.
Note the question asks for the least likely cause. There are many reasons why an alcoholic may develop neurology. However, an extra dural bleed would not typically present such a long latent period.

Head injury

Patients who suffer head injuries should be managed according to ATLS principles and extra cranial injuries should be managed alongside cranial trauma. Inadequate cardiac output will compromise CNS perfusion irrespective of the nature of the cranial injury.

Types of traumatic brain injury
Extradural haematoma Bleeding into the space between the dura mater and the skull. Often results from acceleration-deceleration trauma or a blow to the side of the head. The majority of extradural haematomas occur in the temporal region where skull fractures cause a rupture of the middle meningeal artery.

Features
Raised intracranial pressure
Some patients may exhibit a lucid interval
Subdural haematoma Bleeding into the outermost meningeal layer. Most commonly occur around the frontal and parietal lobes. May be either acute or chronic.

Risk factors include old age and alcoholism.

Slower onset of symptoms than a extradural haematoma.
Subarachnoid haemorrhage Usually occurs spontaneously in the context of a ruptured cerebral aneurysm, but may be seen in association with other injuries when a patient has sustained a traumatic brain injury.

Pathophysiology
Primary brain injury may be focal (contusion/ haematoma) or diffuse (diffuse axonal injury)
Diffuse axonal injury occurs as a result of mechanical shearing following deceleration, causing disruption and tearing of axons
Intra-cranial haematomas can be extradural, subdural or intracerebral, while contusions may occur adjacent to (coup) or contralateral (contre-coup) to the side of impact
Secondary brain injury occurs when cerebral oedema, ischaemia, infection, tonsillar or tentorial herniation exacerbates the original injury. The normal cerebral auto regulatory processes are disrupted following trauma rendering the brain more susceptible to blood flow changes and hypoxia
The Cushings reflex (hypertension and bradycardia) often occurs late and is usually a pre terminal event

Management
Where there is life threatening rising ICP such as in extra dural haematoma and whilst theatre is prepared or transfer arranged use of IV mannitol/ frusemide may be required.
Diffuse cerebral oedema may require decompressive craniotomy
Exploratory Burr Holes have little management in modern practice except where scanning may be unavailable and to thus facilitate creation of formal craniotomy flap
Depressed skull fractures that are open require formal surgical reduction and debridement, closed injuries may be managed non operatively if there is minimal displacement.
ICP monitoring is appropriate in those who have GCS 3-8 and normal CT scan.
ICP monitoring is mandatory in those who have GCS 3-8 and abnormal CT scan.
Hyponatraemia is most likely to be due to syndrome of inappropriate ADH secretion.
Minimum of cerebral perfusion pressure of 70mmHg in adults.
Minimum cerebral perfusion pressure of between 40 and 70 mmHg in children.

Interpretation of pupillary findings in head injuries
Pupil size Light response Interpretation
Unilaterally dilated Sluggish or fixed 3rd nerve compression secondary to tentorial herniation
Bilaterally dilated Sluggish or fixed
Poor CNS perfusion
Bilateral 3rd nerve palsy
Unilaterally dilated or equal Cross reactive (Marcus - Gunn) Optic nerve injury
Bilaterally constricted May be difficult to assess
Opiates
Pontine lesions
Metabolic encephalopathy
Unilaterally constricted Preserved Sympathetic pathway disruption

1265
Q

Which of the following is not true of gastric cancer?

There is an association with blood group A

Adenocarcinoma is the most common subtype

Individuals with histological evidence of signet ring cells have a lower incidence of lymph node metastasis

Lymphomas account for 5% cases

In Western Countries a more proximal disease distribution has been noted

A

Signet ring cells are features of poorly differentiated gastric cancer associated with a increased risk of metastatic disease.

Gastric cancer

Overview
There are 700,000 new cases of gastric cancer worldwide each year. It is most common in Japan and less common in western countries. It is more common in men and incidence rises with increasing age. The exact cause of many sporadic cancer is not known, however, familial cases do occur in HNPCC families. In addition, smoking and smoked or preserved foods increase the risk. Japanese migrants retain their increased risk (decreased in subsequent generations). The distribution of the disease in western countries is changing towards a more proximal location (perhaps due to rising obesity).

Pathology
There is some evidence of support a stepwise progression of the disease through intestinal metaplasia progressing to atrophic gastritis and subsequent dysplasia, through to cancer. The favoured staging system is TNM. The risk of lymph node involvement is related to size and depth of invasion; early cancers confined to submucosa have a 20% incidence of lymph node metastasis. Tumours of the gastro-oesophageal junction are classified as below:

Type 1 True oesophageal cancers and may be associated with Barrett’s oesophagus.
Type 2 Carcinoma of the cardia, arising from cardiac type epithelium
or short segments with intestinal metaplasia at the oesophagogastric junction.
Type 3 Sub cardial cancers that spread across the junction. Involve similar nodal stations to gastric cancer.

Groups for close endoscopic monitoring
Intestinal metaplasia of columnar type
Atrophic gastritis
Low to medium grade dysplasia
Patients who have previously undergone resections for benign peptic ulcer disease (except highly selective vagotomy).

Referral to endoscopy

Patients of any age with dyspepsia and any of the following Patients without dyspepsia Worsening dyspepsia
Chronic gastrointestinal bleeding Dysphagia Barretts oesophagus
Dysphagia Unexplained abdominal pain or weight loss Intestinal metaplasia
Weight loss Vomiting Dysplasia
Iron deficiency anaemia Upper abdominal mass Atrophic gastritis
Upper abdominal mass Jaundice Patient aged over 55 years with unexplained or persistent dyspepsia

Staging
CT scanning of the chest abdomen and pelvis is the routine first line staging investigation in most centres.
Laparoscopy to identify occult peritoneal disease
PET CT (particularly for junctional tumours)

Treatment
Proximally sited disease greater than 5-10cm from the OG junction may be treated by sub total gastrectomy
Total gastrectomy if tumour is <5cm from OG junction
For type 2 junctional tumours (extending into oesophagus) oesophagogastrectomy is usual
Endoscopic sub mucosal resection may play a role in early gastric cancer confined to the mucosa and perhaps the sub mucosa (this is debated)
Lymphadenectomy should be performed. A D2 lymphadenectomy is widely advocated by the Japanese, the survival advantages of extended lymphadenectomy have been debated. However, the overall recommendation is that a D2 nodal dissection be undertaken.
Most patients will receive chemotherapy either pre or post operatively.

Prognosis

UK Data

Disease extent Percentage 5 year survival
All RO resections 54%
Early gastric cancer 91%
Stage 1 87%
Stage 2 65%
Stage 3 18%

Operative procedure

Total Gastrectomy , lymphadenectomy and Roux en Y anastomosis

General anaesthesia
Prophylactic intravenous antibiotics
Incision: Rooftop.
Perform a thorough laparotomy to identify any occult disease.
Mobilise the left lobe of the liver off the diaphragm and place a large pack over it. Insert a large self retaining retractor e.g. omnitract or Balfour (take time with this, the set up should be perfect). Pack the small bowel away.
Begin by mobilising the omentum off the transverse colon.
Proceed to detach the short gastric vessels.
Mobilise the pylorus and divide it at least 2cm distally using a linear cutter stapling device.
Continue the dissection into the lesser sac taking the lesser omentum and left gastric artery flush at its origin.
The lymph nodes should be removed en bloc with the specimen where possible.
Place 2 stay sutures either side of the distal oesophagus. Ask the anaesthetist to pull back on the nasogastric tube. Divide the distal oesophagus and remove the stomach.
The oesphago jejunal anastomosis should be constructed. Identify the DJ flexure and bring a loop of jejunum up to the oesophagus (to check it will reach). Divide the jejunum at this point. Bring the divided jejunum either retrocolic or antecolic to the oesophagus. Anastamose the oesophagus to the jejunum, using either interrupted 3/0 vicryl or a stapling device. Then create the remainder of the Roux en Y reconstruction distally.
Place a jejunostomy feeding tube.
Wash out the abdomen and insert drains (usually the anastomosis and duodenal stump). Help the anaesthetist insert the nasogastric tube (carefully!)
Close the abdomen and skin.
Enteral feeding may commence on the first post-operative day. However, most surgeons will leave patients on free NG drainage for several days and keep them nil by mouth.

1266
Q

A lady has undergone breast cancer surgery and the pathological analysis of the specimen is completed. Which of the Nottingham Prognostic Index scores shown below would correlate with the best long term outcome?

5.4

2.2

2.5

3.0

6.0

A

The Nottingham prognostic index may be used to stratify patients into various prognostic groups (see below). An excellent prognosis is seen with a score of <2.4. Scores of over 5 equate to a greatly reduced survival rate.

Nottingham prognostic index

The Nottingham Prognostic Index can be used to give an indication of survival following breast cancer surgery. In this system, the tumour size is weighted less heavily than other major prognostic parameters.

Calculation of NPI
Tumour Size x 0.2 + Lymph node score(From table below)+Grade score(From table below).

Score Lymph nodes involved Grade
1 0 1
2 1-3 2
3 >3 3

Prognosis

Score Percentage 5 year survival
2.0 to 2.4 93%
2.5 to 3.4 85%
3.5 to 5.4 70%
>5.4 50%

This data was originally published in 1992. It should be emphasised that other factors such as vascular invasion and receptor status also impact on survival and are not included in this data and account for varying prognoses often cited in the literature.

Reference
Galea, M.H., et al., The Nottingham Prognostic Index in primary breast cancer. Breast Cancer Res Treat, 1992. 22(3): p. 207-19.

1267
Q

A 63 year old man is undergoing an upper GI endoscopy for dysphagia. At 33 cm (from the incisors) a malignant looking stricture is encountered. The endoscopist attempts a balloon dilatation.Unfortunately the tumour splits through the oesophageal wall. Into which region will the oesophageal contents now drain?

Superior mediastinum

Posterior mediastinum

Middle mediastinum

Anterior mediastinum

Peritoneal cavity

A

At this position the oesophagus is still likely to be intrathoracic and located in the posterior mediastinum.

Mediastinum

Region between the pulmonary cavities.
It is covered by the mediastinal pleura. It does not contain the lungs.
It extends from the thoracic inlet superiorly to the diaphragm inferiorly.

Mediastinal regions
Superior mediastinum (between manubriosternal angle and T4/5)
Middle mediastinum
Posterior mediastinum
Anterior mediastinum

Region Contents
Superior mediastinum
Superior vena cava
Brachiocephalic veins
Arch of aorta
Thoracic duct
Trachea
Oesophagus
Thymus
Vagus nerve
Left recurrent laryngeal nerve
Phrenic nerve
Anterior mediastinum
Thymic remnants
Lymph nodes
Fat
Middle mediastinum
Pericardium
Heart
Aortic root
Arch of azygos vein
Main bronchi
Posterior mediastinum
Oesophagus
Thoracic aorta
Azygos vein
Thoracic duct
Vagus nerve
Sympathetic nerve trunks
Splanchnic nerves

1268
Q

Which of the following nerves is not contained within the posterior triangle of the neck?

Accessory nerve

Phrenic nerve

Greater auricular nerve

Ansa cervicalis

Lesser occipital nerve

A

Ansa cervicalis is a content of the anterior triangle of the neck.

Posterior triangle of the neck

Boundaries
Apex Sternocleidomastoid and the Trapezius muscles at the Occipital bone
Anterior Posterior border of the Sternocleidomastoid
Posterior Anterior border of the Trapezius
Base Middle third of the clavicle

Contents
Nerves
Accessory nerve
Phrenic nerve
Three trunks of the brachial plexus
Branches of the cervical plexus: Supraclavicular nerve, transverse cervical nerve, great auricular nerve, lesser occipital nerve
Vessels
External jugular vein
Subclavian artery
Muscles
Inferior belly of omohyoid
Scalene
Lymph nodes
Supraclavicular
Occipital

1269
Q

Which of the agents listed below is most likely to help a 22 year old lady with severe peri anal Crohns disease and multiple anal fistulae? The acute sepsis has been drained and setons are in place. She is already receiving standard non biological therapy.

Trastuzumab

Bevacizumab

Imatinib

Cetuximab

Infliximab

A

Infliximab is a popular choice in managing complex peri anal Crohns. It is absolutely vital that all sepsis is drained prior to starting therapy.

Biological agents

Agents Target Uses
Adalimumab
Infliximab
Etanercept TNF alpha inhibitor Crohns disease
Rheumatoid disease
Bevacizumab Anti VEGF (anti angiogenic) Colorectal cancer
Renal
Glioblastoma
Trastuzumab HER receptor Breast cancer
Imatinib Tyrosine kinase inhibitor Gastrointestinal stromal tumours
Chronic myeloid leukaemia
Basiliximab IL2 binding site Renal transplants
Cetuximab Epidermal growth factor inhibitor EGF positive colorectal cancers

Detailed understanding of the actions of biological agents is well beyond the scope of the MRCS syllabus. However, many of these drugs are being frequently encountered in surgical patients.

1270
Q

An 80 year old woman has a hip fracture. Her calcium is normal. She has never been given a diagnosis of osteoporosis. Apart from treating the hip fracture what additional intervention should be considered?

Vitamin D and calcium supplements alone

Vitamin D, calcium supplements and bisphosphonates

Vitamin D alone

Calcium supplements alone

DEXA scan

A

The patient has a frailty fracture and by definition therefore will have osteoporosis and should therefore commence treatment, a DEXA scan will not change this decision.
The osteoporosis guidelines state if a postmenopausal woman has a fracture she should be put on bisphosphonates (there is no need for a DEXA scan).

Osteoporosis

Osteoporosis is a condition of bone atrophy. It is a lesion in which the volume of bone tissue per unit volume of anatomical bone is reduced. It may be generalized (most common) or localized (following pressure or disuse). The hallmark is a reduction in the amount of osteoid matrix, which, however, remains normally mineralized. It is therefore distinct from osteomalacia in which there is abundant osteoid which is poorly calcified. In osteoporosis the bony trabeculae are greatly thinned which significantly reduces the tensile strength of the bone and renders it at increased risk of pathological fracture.
It is only visible on plain films when the calcium content is approximately halved. More subtle changes can be appreciated by use of DEXA scanning.

Treatment
Fractures are treated according to their site and mechanism of injury. Those deemed to occur as a result of osteoporosis should also be addressed medically to treat the underlying osteoporosis.
Drugs that can be prescribed to prevent fragility fractures include bisphosphonates (alendronate, ibandronate, risedronate and zoledronic acid) and non-bisphosphonates (raloxifene, denosumab, teriparatide, calcitriol and hormone replacement therapy).
Calcium and vitamin D supplements are also administered.

Individuals at risk of fragility fractures should also be considered for prophylactic medical treatment as outlined above.

1271
Q

A 32 year old man is involved in a house fire and sustains extensive partial thickness burns to his torso and thigh. Two weeks post incident he develops oedema of both lower legs. The most likely cause of this is:

Iliofemoral deep vein thrombosis

Venous obstruction due to scarring

Hypoalbuminaemia

Excessive administration of intravenous fluids

None of the above

A

Loss of plasma proteins is the most common cause of oedema developing in this time frame.

Burns pathology

Extensive burns
Haemolysis due to damage of erythrocytes by heat and microangiopathy
Loss of capillary membrane integrity causing plasma leakage into interstitial space
Extravasation of fluids from the burn site causing hypovolaemic shock (up to 48h after injury)- decreased blood volume and increased haematocrit
Protein loss
Secondary infection e.g. Staphylococcus aureus
ARDS
Risk of Curlings ulcer (acute peptic stress ulcers)
Danger of full thickness circumferential burns in an extremity as these may develop compartment syndrome

Healing
Superficial burns: keratinocytes migrate to form a new layer over the burn site
Full thickness burns: dermal scarring. Usually need keratinocytes from skin grafts to provide optimal coverage.

1272
Q

A 28 year old man is playing tennis when he suddenly collapses and has a GCS of 4 when examined. What is the most likely cause?

Intraventricular haemorrhage

Acute sub dural haematoma

Sub arachnoid haemorrhage

Chronic sub dural haematoma

Acute extra dural haematoma

A

A sudden collapse and loss of consciousness is most likely to be due to a sub arachnoid haemorrhage. The other potential causes in the list usually occur as a sequel to a traumatic event, which has not occurred here.

Intra cranial haemorrhage

Extradural haematoma Bleeding into the space between the dura mater and the skull. Often results from acceleration-deceleration trauma or a blow to the side of the head. The majority of extradural haematomas occur in the temporal region where skull fractures cause a rupture of the middle meningeal artery.

Features
Raised intracranial pressure
Some patients may exhibit a lucid interval
Subdural haematoma Bleeding into the outermost meningeal layer. Most commonly occur around the frontal and parietal lobes. May be either acute or chronic.

Risk factors include old age and alcoholism.

Slower onset of symptoms than a extradural haematoma.
Intracerebral haematoma Usually hyperdense lesions on CT scanning. Arise in areas of traumatic contusion which fuse to become a haematoma. Areas of clot and fresh blood may co-exist on the same CT scan (Swirl sign). Large haematomas and those associated with mass effect should be evacuated.
Subarachnoid haemorrhage Usually occurs spontaneously in the context of a ruptured cerebral aneurysm but may be seen in association with other injuries when a patient has sustained a traumatic brain injury
Intraventricular haemorrhage Haemorrhage that occurs into the ventricular system of the brain. It is relatively rare in adult surgical practice and when it does occur, it is typically associated with severe head injuries. In premature neonates it may occur spontaneously. The blood may clot and occlude CSF flow, hydrocephalus may result.
In neonatal practice the vast majority of IVH occur in the first 72 hours after birth, the aetiology is not well understood and it is suggested to occur as a result of birth trauma combined with cellular hypoxia, together with the delicate neonatal CNS.

1273
Q

Which one of the following cells secretes the majority of tumour necrosis factor in humans?

Neutrophils

Macrophages

Natural killer cells

Killer-T cells

Helper-T cells

A

Tumour necrosis factor

Tumour necrosis factor (TNF) is a pro-inflammatory cytokine with multiple roles in the immune system

TNF is secreted mainly by macrophages and has a number of effects on the immune system, acting mainly in a paracrine fashion:
activates macrophages and neutrophils
acts as costimulator for T cell activation
key mediator of body’s response to Gram negative septicaemia
similar properties to IL-1
anti-tumour effect (e.g. phospholipase activation)

TNF-alpha binds to both the p55 and p75 receptor. These receptors can induce apoptosis. It also cause activation of NFkB

Endothelial effects include increase expression of selectins and increased production of platelet activating factor, IL-1 and prostaglandins

TNF promotes the proliferation of fibroblasts and their production of protease and collagenase. It is thought fragments of receptors act as binding points in serum

Systemic effects include pyrexia, increased acute phase proteins and disordered metabolism leading to cachexia

TNF is important in the pathogenesis of rheumatoid arthritis - TNF blockers (e.g. infliximab, etanercept) are now licensed for treatment of severe rheumatoid

1274
Q

A 63 year old lady is undergoing an ERCP and the main pancreatic duct is cannulated. What is the embryological origin of this structure?

The duct of the ventral outgrowth of the duodenum

The duct of the dorsal outgrowth of the duodenum

As an outgrowth of the embryological hepatic diverticulum

As a projection of the vitellointestinal duct

As a projection from the yolk sac

A

The main pancreatic duct is largely formed from the duct of the ventral pancreas having fused with the dorsal part of the gland.

Pancreatic embryology

The pancreas develops from a ventral and dorsal entodermal outgrowth from the duodenum. The ventral arises close to, or in common with, the hepatic diverticulum, and the larger, dorsal outgrowth arises slightly cranial to the ventral and extends into the mesoduodenum and dorsal mesogastrium. As a result of the differential growth in the duodenal wall, the point of attachment of the ventral outgrowth with the undivided stem of the hepatic diverticulum and the ventral mesentery are displaced around the right hand side of the duodenum to a dorsal position. As a result, the two pancreatic remnants eventually lie together and then become fused. In this process the ventral rudiment forms the uncinate process and the adjacent region of the head, while the dorsal forms the anterior part of the head, the body and the tail of the pancreas.
At the time of fusion, the ducts become joined. This occurs in a manner that results in the dorsal remnant being drained by the duct of the ventral remnant, this composite duct being named the pancreatic duct.

1275
Q

A 44 year old lady presents with a pathological fracture of the left femur. She has previously undergone a renal transplant for end stage renal failure. Her blood test results are as follows:

Serum Ca2+ 2.80
PTH 88pg/ml
Phosphate 0.30

A surgeon decides to perform a parathyroidectomy on the basis of these results. When the glands are assessed histologically, which of the appearances is most likely to be identified?

Metaplasia the gland

Hypertrophy of the gland

Hyperplasia of the gland

Parathyroid carcinoma

Necrosis of the parathyroid gland

A

This is likely to be a case of tertiary hyperparathyroidism (high Calcium, high PTH, low phosphate). Therefore the glands will be hyperplastic. Hypertrophy is not correct as this implies an increase in size without an increase in cellularity. This mistake has cost many candidates marks in the MRCS exams over the years!

Parathyroid glands and disorders of calcium metabolism

Hyperparathyroidism
Disease type Hormone profile Clinical features Cause
Primary hyperparathyroidism
PTH (Elevated)
Ca2+ (Elevated)
Phosphate (Low)
Urine calcium : creatinine clearance ratio > 0.01
May be asymptomatic if mild
Recurrent abdominal pain (pancreatitis, renal colic)
Changes to emotional or cognitive state
Most cases due to solitary adenoma (80%), multifocal disease occurs in 10-15% and parathyroid carcinoma in 1% or less
Secondary hyperparathyroidism
PTH (Elevated)
Ca2+ (Low or normal)
Phosphate (Elevated)
Vitamin D levels (Low)
May have few symptoms
Eventually may develop bone disease, osteitis fibrosa cystica and soft tissue calcifications
Parathyroid gland hyperplasia occurs as a result of low calcium, almost always in a setting of chronic renal failure
Tertiary hyperparathyroidism
Ca2+ (Normal or high)
PTH (Elevated)
Phosphate levels (Decreased or Normal)
Vitamin D (Normal or decreased)
Alkaline phosphatase (Elevated)
Metastatic calcification
Bone pain and / or fracture
Nephrolithiasis
Pancreatitis
Occurs as a result of ongoing hyperplasia of the parathyroid glands after correction of underlying renal disorder, hyperplasia of all 4 glands is usually the cause

Differential diagnoses
It is important to consider the rare but relatively benign condition of benign familial hypocalciuric hypercalcaemia, caused by an autosomal dominant genetic disorder. Diagnosis is usually made by genetic testing and concordant biochemistry (urine calcium : creatinine clearance ratio <0.01-distinguished from primary hyperparathyroidism).

Treatment

Primary hyperparathyroidism
Indications for surgery
Elevated serum Calcium > 1mg/dL above normal
Hypercalciuria > 400mg/day
Creatinine clearance < 30% compared with normal
Episode of life threatening hypercalcaemia
Nephrolithiasis
Age < 50 years
Neuromuscular symptoms
Reduction in bone mineral density of the femoral neck, lumbar spine, or distal radius of more than 2.5 standard deviations below peak bone mass (T score lower than -2.5)

Secondary hyperparathyroidism
Usually managed with medical therapy.

Indications for surgery in secondary (renal) hyperparathyroidism:
Bone pain
Persistent pruritus
Soft tissue calcifications

Tertiary hyperparathyroidism
Allow 12 months to elapse following transplant as many cases will resolve
The presence of an autonomously functioning parathyroid gland may require surgery. If the culprit gland can be identified then it should be excised. Otherwise total parathyroidectomy and re-implantation of part of the gland may be required.

References
1. Insogna K. Primary Hyperparathyroidism. N Engl J Med. 2018 Sep 13;379(11):1050-1059.

  1. van der Plas W Y et al. Secondary and Tertiary Hyperparathyroidism: A Narrative Review. Scand J Surg. 2020 Dec;109(4):271-278.
1276
Q

A 22 year old man presents with a 5 day history of sore throat, malaise and fatigue. On examination, he has a large peritonsillar abscess. What is the most likely underlying infective organism?

Epstein Barr
Virus

Streptococcus pyogenes

Cytomegalovirus

Moraxella catarrhalis

Streptococcus viridans

A

Streptococcus pyogenes

Quinsy usually occurs as a result of bacterial tonsillitis and the most common cause of bacterial tonsillitis is streptococcal organisms.
Acute tonsillitis

Characterised by pharyngitis, fever, malaise and lymphadenopathy.
Over half of all cases are bacterial with Streptococcus pyogenes the most common organism
The tonsils are typically oedematous and yellow or white pustules may be present
Infectious mononucleosis may mimic the condition.
Treatment with penicillin type antibiotics is indicated for bacterial tonsillitis.
Bacterial tonsillitis may result in local abscess formation (quinsy)

1277
Q

A 34-year-old man is taken immediately to theatre with aortic dissection. You note he is tall with pectus excavatum and arachnodactyly. His condition is primarily due to a defect in which one of the following proteins?

Polycystin-1

Fibrillin

Type IV collagen

Type I collagen

Elastin

A

Although fibrillin is the primary protein affected (due to a defect in the fibrillin-1 gene) it should be noted that fibrillin is used as a substrate of elastin.

Marfan’s syndrome

Marfan’s syndrome is an autosomal dominant connective tissue disorder. It is caused by a defect in the fibrillin-1 gene on chromosome 15 and affects around 1 in 3,000 people.

Features
tall stature with arm span to height ratio > 1.05
high-arched palate
arachnodactyly
pectus excavatum
pes planus
scoliosis of > 20 degrees
heart: dilation of the aortic sinuses (seen in 90%) which may lead to aortic aneurysm, aortic dissection, aortic regurgitation, mitral valve prolapse (75%),
lungs: repeated pneumothoraces
eyes: upwards lens dislocation (superotemporal ectopia lentis), blue sclera, myopia
dural ectasia (ballooning of the dural sac at the lumbosacral level)

The life expectancy of patients used to be around 40-50 years. With the advent of regular echocardiography monitoring and beta-blocker/ACE-inhibitor therapy this has improved significantly over recent years. Aortic dissection and other cardiovascular problems remain the leading cause of death however.

1278
Q

A 29 year old presents with a new onset headache and a neurological exam is undertaken. Which of the lesions listed below is most likely to result in a positive Babinski sign?

Lateral cerebral sulcus lesions

Cerebellar lesions

Basal ganglia lesions

Lesion of the vestibular nuclei

Lesion of the pyramidal tract

A

In the Babinski sign, the normal flexor sign becomes extensor and pyramidal tract lesions can cause UMN signs.

Babinski sign

Joseph Babinski (1857–1932), a French neurologist of Polish descent, first described the Babinski sign, the best known neurological eponym and one of the most important signs in clinical neurology, in 1896.
It is assessed as follows; the first line to be stroked begins a few centimeters distal to the heel and is situated at the junction of the dorsal and plantar surfaces of the foot. The line extends to a point just behind the toes and then turns medially across the transverse arch of the foot. It is a slow stroke, taking 5 or 6 seconds to complete the motion. The response is pathological if the toe is moved upwards by contraction of extensor hallucis longus muscle. Its is a sign of upper motor neurone pathology.

1279
Q

A 32 year old motorcyclist is involved in a road traffic accident. His humerus is fractured and severely displaced. At the time of surgical repair the surgeon notes that the radial nerve has been injured. Which of the following muscles is least likely to be affected by an injury at this site?

Extensor carpi radialis brevis

Brachioradialis

Abductor pollicis longus

Extensor pollicis brevis

None of the above

A

Muscles supplied by the radial nerve

BEST
Brachioradialis
Extensors
Supinator
Triceps
The radial nerve supplies the extensor muscles, abductor pollicis longus and extensor pollicis brevis (the latter two being innervated by the posterior interosseous branch of the radial nerve).

1280
Q

A 52-year-old woman with a history of gastrectomy reports lethargy and a sore tongue. Blood tests are reported as follows:

Hb 10.7 g/dl
MCV 121 fl
Plt 177 * 10^9/l
WBC 5.4 * 10^9/l

What is the most likely cause?

Vitamin B12 deficiency

Vitamin C deficiency

Iron deficiency anaemia

Anaemia of chronic disease

Vitamin E deficiency

A

A history of gastrectomy and a macrocytic anaemia should indicate a diagnosis of B12 deficiency.

Vitamin B12 deficiency

Vitamin B12 is mainly used in the body for red blood cell development and also maintenance of the nervous system. It is absorbed after binding to intrinsic factor (secreted from parietal cells in the stomach) and is actively absorbed in the terminal ileum. A small amount of vitamin B12 is passively absorbed without being bound to intrinsic factor.

Causes of vitamin B12 deficiency
pernicious anaemia
post gastrectomy
poor diet
disorders of terminal ileum (site of absorption): Crohn’s, blind-loop etc

Features of vitamin B12 deficiency
macrocytic anaemia
sore tongue and mouth
neurological symptoms: e.g. Ataxia
neuropsychiatric symptoms: e.g. Mood disturbances

Management
if no neurological involvement 1 mg of IM hydroxocobalamin 3 times each week for 2 weeks, then once every 3 months
if a patient is also deficient in folic acid then it is important to treat the B12 deficiency first to avoid precipitating subacute combined degeneration of the cord

1281
Q

A 52 year old male attends the stroke unit with dizziness and vertigo while playing tennis. He is known to have hypertension and a previous myocardial infarct. He now complains of right arm pain. What is the most likely diagnosis?

Posterior circulation infarct

Vertebrobasilar aneurysm

Dissection of thoracic aorta

Subclavian steal syndrome

Left middle cerebral artery infarct

A

Subclavian steal syndrome characteristically presents with posterior circulation symptoms, such as dizziness and vertigo, during exertion of an arm. There is subclavian artery steno-occlusive disease proximal to the origin of the vertebral artery and is associated with flow reversal in the vertebral artery. Management involves percutaneous transluminal angioplasty or a stent.

Subclavian artery

Path
The left subclavian comes directly off the arch of aorta
The right subclavian arises from the brachiocephalic artery (trunk) when it bifurcates into the subclavian and the right common carotid artery.
From its origin, the subclavian artery travels laterally, passing between anterior and middle scalene muscles, deep to scalenus anterior and anterior to scalenus medius. As the subclavian artery crosses the lateral border of the first rib, it becomes the axillary artery. At this point it is superficial and within the subclavian triangle.

Branches
Vertebral artery
Internal thoracic artery
Thyrocervical trunk
Costocervical trunk
Dorsal scapular artery

1282
Q

A 72 year old man with prostate cancer is admitted to urology with urinary retention. He complains of back pain which is not responding to ward analgesia. Imaging shows several lumbar vertebral body metastasis. What is the most appropriate management?

Commence a bisphosphonate

Arrange radiotherapy to the lumbar spine

Surgical resection and reconstruction of the vertebral body

Administration of pregabalin

Chemotherapy

A

The fact that ward based analgesia has been tried suggests that the patient be considered for palliative radiotherapy. Surgical resection of prostate cancer bony metastatic disease is not generally performed.

Management of pain

World Health Organisation Analgesic Ladder
Initially peripherally acting drugs such as paracetamol or non-steroidal anti-inflammatory drugs (NSAIDs) are given.
If pain control is not achieved, the second part of the ladder is to introduce weak opioid drugs such as codeine or dextropropoxyphene together with appropriate agents to control and minimise side effects.
The final rung of the ladder is to introduce strong opioid drugs such as morphine. Analgesia from peripherally acting drugs may be additive to that from centrally-acting opioids and thus, the two are given together.

The World Federation of Societies of Anaesthesiologists (WFSA) Analgesic Ladder
For management of acute pain
Initially, the pain can be expected to be severe and may need controlling with strong analgesics in combination with local anaesthetic blocks and peripherally acting drugs.
The second rung on the postoperative pain ladder is the restoration of the use of the oral route to deliver analgesia. Strong opioids may no longer be required and adequate analgesia can be obtained by using combinations of peripherally acting agents and weak opioids.
The final step is when the pain can be controlled by peripherally acting agents alone.

Local anaesthetics
Infiltration of a wound with a long-acting local anaesthetic such as Bupivacaine
Analgesia for several hours
Further pain relief can be obtained with repeat injections or by infusions via a thin catheter
Blockade of plexuses or peripheral nerves will provide selective analgesia in those parts of the body supplied by the plexus or nerves
Can either be used to provide anaesthesia for the surgery or specifically for postoperative pain relief
Especially useful where a sympathetic block is needed to improve postoperative blood supply or where central blockade such as spinal or epidural blockade is contraindicated.

Spinal anaesthesia
Provides excellent analgesia for surgery in the lower half of the body and pain relief can last many hours after completion of the operation if long-acting drugs containing vasoconstrictors are used.

  • Side effects of spinal anaesthesia include: hypotension, sensory and motor block, nausea and urinary retention.

Epidural anaesthesia
An indwelling epidural catheter inserted. This can then be used to provide a continuous infusion of analgesic agents. It can provide excellent analgesia. They are still the preferred option following major open abdominal procedures and help prevent post operative respiratory compromise resulting from pain.

  • Disadvantages of epidurals is that they usually confine patients to bed, especially if a motor block is present. In addition an indwelling urinary catheter is required. Which may not only impair mobility but also serve as a conduit for infection. They are contraindicated in coagulopathies.

Transversus Abdominal Plane block (TAP)
In this technique an ultrasound is used to identify the correct muscle plane and local anaesthetic (usually bupivicaine) is injected. The agent diffuses in the plane and blocks many of the spinal nerves. It is an attractive technique as it provides a wide field of blockade but does not require the placement of any indwelling devices. There is no post operative motor impairment. For this reason it is the preferred technique when extensive laparoscopic abdominal procedures are performed. They will then provide analgesia immediately following surgery but as they do not confine the patient to bed, the focus on enhanced recovery can begin sooner.

-The main disadvantage is that their duration of action is limited to the half life of the local anaesthetic agent chosen. In addition some anaesthetists do not have the USS skills required to site the injections.

Patient Controlled Analgesia (PCA)

  • Patients administer their own intravenous analgesia and titrate the dose to their own end-point of pain relief using a small microprocessor - controlled pump. Morphine is the most popular drug used.

Strong Opioids

Severe pain arising from deep or visceral structures requires the use of strong opioids

Morphine
Short half life and poor bioavailability.
Metabolised in the liver and clearance is reduced in patients with liver disease, in the elderly and the debilitated
Side effects include nausea, vomiting, constipation and respiratory depression.
Tolerance may occur with repeated dosage

Pethidine
Synthetic opioid which is structurally different from morphine but which has similar actions. Has 10% potency of morphine.
Short half life and similar bioavailability and clearance to morphine.
Short duration of action and may need to be given hourly.
Pethidine has a toxic metabolite (norpethidine) which is cleared by the kidney, but which accumulates in renal failure or following frequent and prolonged doses and may lead to muscle twitching and convulsions. Extreme caution is advised if pethidine is used over a prolonged period or in patients with renal failure.

Weak opioids
Codeine: markedly less active than morphine, has predictable effects when given orally and is effective against mild to moderate pain.

Non opioid analgesics
- Mild to moderate pain.

Paracetamol
Inhibits prostaglandin synthesis.
Analgesic and antipyretic properties but little anti-inflammatory effect
It is well absorbed orally and is metabolised almost entirely in the liver
Side effects in normal dosage and is widely used for the treatment of minor pain. It causes hepatotoxicity in over dosage by overloading the normal metabolic pathways with the formation of a toxic metabolite.

NSAIDs
Analgesic and anti-inflammatory actions
Inhibition of prostaglandin synthesis by the enzyme Cyclooxygenase which catalyses the conversion of arachidonic acid to the various prostaglandins that are the chief mediators of inflammation. All NSAIDs work in the same way and thus there is no point in giving more than one at a time. .
NSAIDs are, in general, more useful for superficial pain arising from the skin, buccal mucosa, joint surfaces and bone.
Relative contraindications: history of peptic ulceration, gastrointestinal bleeding or bleeding diathesis; operations associated with high blood loss, asthma, moderate to severe renal impairment, dehydration and any history of hypersensitivity to NSAIDs or aspirin.

Neuropathic pain
National Institute of Clinical Excellence (UK) guidelines:
First line: Amitriptyline (Imipramine if cannot tolerate) or pregabalin
Second line: Amitriptyline AND pregabalin
Third line: refer to pain specialist. Give tramadol in the interim (avoid morphine)
If diabetic neuropathic pain: Duloxetine
As of 1 April 2019, pregabalin and gabapentin are Class C controlled substances (under the Misuse of Drugs Act 1971) and scheduled under the Misuse of Drugs Regulations 2001 as Schedule 3. Evaluate patients carefully for a history of drug abuse before prescribing and observe patients for development of signs of abuse and dependence (MHRA, Drug Safety Update April 2019).

References
1. http://guidance.nice.org.uk/CG173/Guidance/pdf/English
2. Lovich-Sapola J, Smith CE, Brandt CP. Post operative pain control. Surg Clin North Am. 2015 Apr;95(2):301-183. Finnerup N et al. Pharmacotherapy for neuropathic pain in adults: a systematic review and meta-analysis. Lancet Neurol. 2015 Feb;14(2):162-73.

1283
Q

A 66 year old man is undergoing a difficult laparotomy and there is bleeding. His blood pressure is controlled by administration of blood products and fluids. An adrenaline infusion is started. What will be the effects of this on renin?

It will stimulate its release from the juxtaglomerular cells

It will inhibit its release from the juxtaglomerular cells

It will inhibit its interaction with angiotensinogen I

It renders it biologically inactive

It accelerates its cellular breakdown

A

The sympathetic nervous system stimulates renin release from juxtaglomerular cells via the beta adrenoreceptor cAMP pathway

Renin

Renin is secreted by juxtaglomerular cells and hydrolyses angiotensinogen to produce angiotensin I

Factors stimulating renin secretion
Hypotension causing reduced renal perfusion
Hyponatraemia
Sympathetic nerve stimulation
Catecholamines
Erect posture

Factors reducing renin secretion
Drugs: beta-blockers, NSAIDs

1284
Q

A 23 year old man is stabbed in the right upper quadrant and is haemodynamically unstable. A laparotomy is performed and the liver has some extensive superficial lacerations and is bleeding profusely. The patient becomes progressively more haemodynamically unstable. What is the best management option?

Pack the liver and close the abdomen with Bogota bag system

Occlude the hepatic inflow with a pringles manoeuvre and suture the defects

Occlude vascular inflow and resect the most severely affected area anatomically

Perform a portosystemic shunt procedure

Suture the defects without vascular occlusion

A

Packing of the liver is the safest option and resection or repair considered later when the physiology is normalised. Often when the packs are removed all the bleeding has ceased and the abdomen can be closed without further action. Definitive attempts at suturing or resection at the primary laparotomy are often complicated by severe bleeding.

Trauma management

The cornerstone of trauma management is embodied in the principles of ATLS.

Following trauma there is a trimodal death distribution:
Immediately following injury. Typically as result of brain or high spinal injuries, cardiac or great vessel damage. Salvage rate is low.
In early hours following injury. In this group deaths are due to phenomena such as splenic rupture, sub dural haematomas and haemopneumothoraces
In the days following injury. Usually due to sepsis or multi organ failure.

Aspects of trauma management
ABCDE approach.
Tension pneumothoraces will deteriorate with vigorous ventilation attempts.
External haemorrhage is managed as part of the primary survey. As a rule tourniquets should not be used. Blind application of clamps will tend to damage surrounding structures and packing is the preferred method of haemorrhage control.
Urinary catheters and naso gastric tubes may need inserting. Be wary of basal skull fractures and urethral injuries.
Patients with head and neck trauma should be assumed to have a cervical spine injury until proven otherwise.

Thoracic injuries
Simple pneumothorax
Mediastinal traversing wounds
Tracheobronchial tree injury
Haemothorax
Blunt cardiac injury
Diaphragmatic injury
Aortic disruption
Pulmonary contusion

Management of thoracic trauma
Simple pneumothorax insert chest drain. Aspiration is risky in trauma as pneumothorax may be from lung laceration and convert to tension pneumothorax.
Mediastinal traversing wounds These result from situations like stabbings. Exit and entry wounds in separate hemithoraces. The presence of a mediastinal haematoma indicates the likelihood of a great vessel injury. All patients should undergo CT angiogram and oesophageal contrast swallow. Indications for thoracotomy are largely related to blood loss and will be addressed below.
Tracheobronchial tree injury Unusual injuries. In blunt trauma most injuries occur within 4cm of the carina. Features suggesting this injury include haemoptysis and surgical emphysema. These injuries have a very large air leak and may have tension pneumothorax.
Haemothorax Usually caused by laceration of lung vessel or internal mammary artery by rib fracture. Patients should all have a wide bore 36F chest drain. Indications for thoracotomy include loss of more than 1.5L blood initially or ongoing losses of >200ml per hour for >2 hours.
Cardiac contusions Usually cardiac arrhythmias, often overlying sternal fracture. Perform echocardiography to exclude pericardial effusions and tamponade. Risk of arrhythmias falls after 24 hours.
Diaphragmatic injury Usually left sided. Direct surgical repair is performed.
Traumatic aortic disruption Commonest cause of death after RTA or falls. Usually incomplete laceration near ligamentum arteriosum. All survivors will have contained haematoma. Only 1-2% of patients with this injury will have a normal chest x-ray.
Pulmonary contusion Common and lethal. Insidious onset. Early intubation and ventilation.

Abdominal trauma
Deceleration injuries are common.
In blunt trauma requiring laparotomy the spleen is most commonly injured (40%)
Stab wounds traverse structures most commonly liver (40%)
Gunshot wounds have variable effects depending upon bullet type. Small bowel is most commonly injured (50%)
Patients with stab wounds and no peritoneal signs up to 25% will not enter the peritoneal cavity
Blood at urethral meatus suggests a urethral tear
High riding prostate on PR = urethral disruption
Mechanical testing for pelvic stability should only be performed once

Investigations in abdominal trauma

Diagnostic Peritoneal Lavage Abdominal CT scan USS
Indication Document bleeding if hypotensive Document organ injury if normotensive Document fluid if hypotensive
Advantages Early diagnosis and sensitive; 98% accurate Most specific for localising injury; 92 to 98% accurate Early diagnosis, non invasive and repeatable; 86 to 95% accurate
Disadvantages Invasive and may miss retroperitoneal and diaphragmatic injury Location of scanner away from facilities, time taken for reporting, need for contrast Operator dependent and may miss retroperitoneal injury
Amylase may be normal following pancreatic trauma
Urethrography if suspected urethral injury

1285
Q

A patient undergoes a femoral hernia repair and at operation the surgeon decides to enter the abdominal cavity to resect small bowel. She makes a transverse incision two thirds of the way between umbilicus and the symphysis pubis. Which of the structures listed below is least likely to be divided?

Rectus abdominis

External oblique aponeurosis

Peritoneum

Fascia transversalis

Posterior lamina of the rectus sheath

A

An incision at this level lies below the arcuate line and the posterior wall of the rectus sheath is deficient at this level.

Rectus abdominis muscle

The rectus sheath is formed by the aponeuroses of the lateral abdominal wall muscles. The rectus sheath has a composition that varies according to anatomical level.

  1. Above the costal margin the anterior sheath is composed of external oblique aponeurosis, the costal cartilages are posterior to it.
  2. From the costal margin to the arcuate line, the anterior rectus sheath is composed of external oblique aponeurosis and the anterior part of the internal oblique aponeurosis. The posterior part of the internal oblique aponeurosis and transversus abdominis form the posterior rectus sheath.
  3. Below the arcuate line the aponeuroses of all the abdominal muscles lie in anterior aspect of the rectus sheath. Posteriorly lies the transversalis fascia and peritoneum.

The arcuate line is the point at which the inferior epigastric vessels enter the rectus sheath.

1286
Q

A 28 year old man complains of pain and weakness in the shoulder. He has recently been unwell with glandular fever from which he is fully recovered. On examination there is some evidence of muscle wasting and a degree of winging of the scapula. Power during active movements is impaired. What is the most likely cause?

Parsonage-Turner syndrome

Adhesive capsulitis

Rotator cuff tear

Osteoarthritis

Calcific tendonitis

A

This is a peripheral neuropathy that may complicate viral illnesses and usually resolves spontaneously.

Shoulder disorders

Shoulder fractures and dislocations
Fractures
Proximal humerus
Background
Third most common fragility fracture in the elderly.
Results from low energy fall in predominantly elderly females, or from high energy trauma in young males.
Can be associated with nerve injury (commonly axillary), and fracture-dislocation of the humeral head. Detailed neurological assessment is essential for all upper limb injuries.

Anatomy
Osteology
Consists of articular head, greater tuberosity, lesser tuberosity, metaphysis and diaphysis. Between the articular head and the tuberosities is the anatomical neck (previous physis). Between the tuberosities and the metaphysis is the surgical neck.
The supraspinatus, infraspinatus and teres minor muscles attach to the greater tuberosity. The subscapularis muscle attaches to the lesser tuberosity.

Vascular Supply
Humeral head is supplied by the anterior and posterior humeral circumflex arteries. Anatomical neck fractures are at greatest risk of osteonecrosis.

Imaging
Imaging aims to both delineate the fracture pattern, and confirm/exlude the presence of an associated dislocation.
Radiographs - True anteroposterior (AP), axillary lateral and/or scapula Y view.
CT - indicated to better define intra-articular involvement and to aid pre-operative planning. MRI is not useful for fracture imaging.

Classification
Description of the fracture is often more useful than classification. Particular attention should be paid to humeral alignment, fracture displacement, and greater tuberosity position (rotator cuff will pull the GT supero-posterioly, which can cause impingement problems with malunion).
- Neer Classification: Most commonly used. Describes fracture as 2,3,or 4 part depending upon the number main fragments. Also comments on the degree of displacement. Fragments:
-greater tuberosity
-lesser tuberosity
- articular surface
- shaft
Displacement: >1cm or angulation >45 degrees.

Treatment
The vast majority of proximal humeral fractures are minimally displaced, and therefore can be managed conservatively. This involves immobilisation in a polysling, and progressive mobilisation. Pendular exercise can commence at 14 days, and active abduction from 4-6 weeks.

Irreducible fracture dislocation is an indication for operative management. Other indications include large displacement, younger patient, head splitting (intra-articular fractures). However, the recent PROFHER trial (1) has suggested no benefit to operative intervention on patient outcome (it must be applied cautiously as majority of patients were elderly with extraarticular fractures). Options available for surgical management include:

ORIF Most commonly used. Plate and screw fixation. Can reconstruct complex fractures.
Intramedullary nail Suitable for extra-articular configuration, predominantly surgical neck +/- GT fractures.
Hemiarthroplasty Used for un-reconstructable fractures in the older patient who has good glenoid quality.
Total shoulder arthroplasty Unconstructable fractures where high functioning shoulder is required (hemiarthroplasty will cause glenoid erosion)
Reverse shoulder arthroplasty Total shoulder arthroplasty that provides better functional outcome than conventional total shoulder replacement.

Scapula
Background
Uncommon fractures usually associated with high energy trauma. Most commonly involve scapula body or spine (50%), glenoid fossa and glenoid neck. Important to exclude associated life threatening injury.

Imaging
Plain radiographs should include true anteroposterior (AP), axillary lateral and/or scapula Y view. CT scanning is useful for defining intra-articular involvement, displacement and for three dimensional reconstruction.

Classification
Based on the location of the fracture (coracoid, acromion, glenoid neck, glenoid fossa, scapula body). Beware of ipsilateral glenoid neck and clavicle fracture -floating shoulder - where limb is effectively dissociated from axial skeleton.

Treatment
The vast majority of scapula fractures are amenable to conservative management, consisting of sling immobilisation for two weeks followed by early rehabilitation. Floating shoulder will usually require fixation, and consideration of surgery should also be given to intra-articular and displaced/angulated glenoid fractures.

Dislocations

Types
Dislocations around the shoulder joint include glenohumeral dislocation, acromioclavicular joint disruption and sternoclavicular dislocation. Only glenohumeral dislocation will be covered here.

Glenohumeral dislocation
Diagnosis, classification and management are covered here.

Background
Shoulder dislocation is commonly seen in A&E. It has a high recurrence rate that is as high as 80% in teenagers. Initial management requires emergent reduction to prevent lasting chondral damage.

Early assessment and management
Usually a traumatic cause (multi-directional instability in frequent dislocations requires discussion with orthopaedics and is not covered here). Careful history, examination and documentation of neurovascular status of the limb, in particular the axillary nerve (regimental badge sensation). This should be re-assessed post manipulation. Early radiographs to confirm direction of dislocation.

Initial management consists of emergent closed reduction under under entanox and analgesia, but often requires conscious sedation. Arm should then be immobilised in a polysling, and XR to confirm relocation.

Imaging - True anteroposterior (AP), axillary lateral and/or scapula Y view. Reduced humeral head should lie between acromion and coracoid on lateral/scapula view.

Types
Direction Features Cause Examination Reduction techniques
Anterior Most Common >90% Usually traumatic - anterior force on arm when shoulder is abducted, externally rotated Loss of shoulder contour - sulcus sign. Humeral head can be felt anteriorly.
Hippocratic.
Milch.
Stimson.

Kocher not advised due to complication of fracture
Posterior 50% missed in A&E 50% traumatic, but classically post seizure or electrocution Shoulder locked in internal rotation. XR may show lightbulb appearance. Gentle lateral traction to adducted arm.
Inferior Rare Associated with pectorals and rotator cuff tears, and glenoid fracture As for primary injury Management of primary injury
Superior Rare Associated with acrominon/clavicle fracture As for primary injury Management of primary injury

Associated injuries
Bankart lesion - avulsion of the anterior glenoid labrum with an anterior shoulder dislocation (reverse Bankart if poster labrum in posterior dislocation).
Hill Sachs defect - chondral impaction on posteriosuperior humeral head from contact with gleonoid rim. Can be large enough to lock shoulder, requiring open reduction. (Reverse Hill Sachs in posterior dislocation).
Rotator cuff tear - increases with age.
Greater or lesser tuberosity fracture - increases with age.
Humeral neck fracture - shoulder fracture dislocation. More common in high energy trauma and elderly. Should be discussed with orthopaedics prior to any attempted reduction.

Rotator Cuff Disease

Rotator cuff disease is a spectrum of conditions that ranges from subacromial impingement to rotator cuff tears and eventually to rotator cuff arthropathy (arthritis).

Anatomy
The rotator cuff is a group of four muscles that are important in shoulder movements, and maintenance of glenohumeral stability.
Muscle Scapular attachment Humeral attachment Action Innervation
Supraspinatus Supraspinatus fossa Superior facet of greater tuberosity Initiation of abduction of humerus Suprascapular nerve
Infraspinatus Infraspinatus fossa Posterior facet of greater tuberosity External rotation of humerus Suprascapular nerve
Teres Minor Lateral border Inferior facet of greater tuberosity External rotation of humerus Axillary Nerve
Subscapularis Subscapular fossa Lesser tuberosity Internal rotation of humerus Upper and lower subscapular nerve

The inferior rotator cuff muscles (infraspinatus, teres minor, and subscapularis) balance the superior pull of the deltoid. Injury/tear results in upward migration of the humeral head on the glenoid (can be seen on AP radiograph).
Likewise, the anterior muscles (subscapularis) are balanced with the posterior muscles (infraspinatus, teres minor).

Subacromial Impingement

The most common cause of shoulder pain, which results from impingement of the superior cuff on the undersurface of the acromion, and an inflammatory bursitis.
Associated with certain types of acromial morphology (Bigliani classification).
Presents as insidious pain which is exacerbated by overhead activities.

Rotator Cuff Tear

Often presents as an acute event on the background of chronic subacromial impingement in the older patient, but can present as an avulsion injury in younger patients.
Majority of tears are to the superior cuff (supraspinatus, infraspinatus, teres minor), though a tear to subscapularis is associated with subcoracoid impingement.
Tears present as pain and weakness when using the muscles in question.

Rotator Cuff Arthropathy

Defined as shoulder arthritis in the setting of rotator cuff dysfunction. Results from superior migration due to the loss of rotator cuff function and integrity. Unopposed deltoid pulls the humeral head superiorly.
Associated with massive chronic cuff tears.

Imaging

Plain radiographs
AP of the shoulder may show superior migration of the humerus with a cuff tear, and features of arthritis with arthropathy. Other causes of pain may also be identified (e.g. calcific tendonitis/fracture)
Outlet view is useful for defining the acromial morphology

USS
Allows dynamic imaging of the cuff, and is inexpensive. However, it is very user dependent.

MRI
Best imaging modality for cuff pathology.
Also allows imaging of the rest of the shoulder. When intra-articular pathology is suspected, can be combined with an arthrogram for improved sensitivity and specificity.

Treatment

Subacromial impingement
Physiotherapy, oral anti-inflammatory medication
Subacromial steroid injection can settle inflammation
Arthroscopic subacromial decompression by shaving away the undersurface of the acromion, more space is created for the rotator cuff. Cuff integrity is assessed also at time of surgery, and can be repaired if necessary.

Rotator cuff tear
When considering repair of a cuff tear, the age and activity of the patient, the nature of the tear (degenerative vs. acute traumatic), and the size and retraction of the tear should be considered when making a surgical plan.
Mild tears or tears in the elderly can be managed conservatively, as outlined above.
Moderate tears can be repaired arthroscopically. Massive or retracted tears will often require an open repair (occasionally with a tendon transfer). Subacromial decompression is performed at the same time to reduce impingement, symptoms and recurrence.

Calcific tendonitis
Calcific tendonitis involves calcific deposits within tendons anywhere in the body, but most commonly in the rotator cuff (specifically the supraspinatus tendon). When present in the shoulder, it is associated with subacromial impingement and pain.

Pathology
More common in women aged 30-60 years.
Association with diabetes and hypothyroidism

There are three stages of calcification
Formative phase characterized by calcific deposits
Resting phase deposit is stable, but presents with impingement problems
Resorptive phase phagocytic resorption. Most painful stage.

Presentation
Similar in presentation to subacromial impingement, with pain especially with over head activities. Atraumatic in nature.

Imaging
Plain radiographs show calcification of the rotator cuff, usually within 1.5cm of its insertion on the humerus. Supraspinatus outlet views can show level of impingment. Further imaging is rarely needed.

Treatment
Non-operative NSAIDS, steroid injection (controversial, but practiced) and physiotherapy. Approximately 75% will resolve by 6 months with conservative management.
Ultrasound guided or surgical needle barbotage can break down deposits and resolve symptoms. Occasionally surgical excision is required.

Adhesive capsulitis (Frozen Shoulder)
Pain and loss of movement of shoulder joint, which involves fibroplastic proliferation of capsular tissue, causing soft tissue scarring and contracture. Patients present with a painful and decreased arc of motion.
Associated with prolonged immobilization, previous surgery, thyroid disorders (AI) and diabetes
Classically three stages which can take up to two years to resolve:
Stage one the freezing and painful stage
Stage two the frozen and stiff stage
Stage three the thawing stage, where shoulder movement slowly improves

Imaging
Plain radiographs to exclude other causes of a painful shoulder
MRI arthrogram may show capsular contracture, and again may be used to exclude cuff pathology. However, often not performed as diagnosis is largely clinical.

Treatment
Non-operative NSAIDS, steroid injection and physiotherapy. Patience is required as condition can take up to 2 years to improve.
Operative MUA or arthroscopic adhesiolysis (release of adhesions) can expedite recovery, followed by intensive physiotherapy.

Glenohumeral Arthritis
Background
May be osteoarthritis (primary or secondary to cuff tear or trauma), rheumatoid arthritis, or as part of a spondyloarthropathy. Majority of those with RA will develop symptoms.
More common in the elderly
Presents like any other arthritis - pain at night and with movement

Imaging
AP and axillary radiographs will show features of arthritis.
CT/MRI is often useful to classify the shape of the glenoid and extent of bone loss when considering arthroplasty. MRI also essential to asses integrity of rotator cuff if considering shoulder replacement.

Treatment
Like all orthopaedics, start with simple measures:
NSAIDS, management of RA, physiotherapy, steroid injection.
Hemiarthroplasty can sometimes be considered if glenoid is in excellent condition or if patient has large comorbidity.
Arthroscopic debridement is useful if patient has isolated ACJ arthritis, but is rarely used for glenohumeral arthritis.
Total shoulder replacement is shown to produce superior outcome when compared to hemiarthroplasty in terms of pain relief, function and implant survival.
Total shoulder replacement can be anatomical (ball on humerus, with cup on glenoid), or reverse geometry (ball on glenoid, with cup on humerus). Anatomical TSR requires an in tact rotator cuff, so often reverse is preferable when the cuff if questionable in integrity.

References
1. JAMA. 2015;313(10):1037-1047. doi:10.1001/jama.2015.1629

1287
Q

A laceration of the wrist produces a median nerve transection. The wound is clean and seen immediately after injury. Collateral soft tissue damage is absent. The patient asks what the prognosis is. You indicate that the nerve should regrow at approximately:

0.1 mm per day

1 mm per day

5 mm per day

1 cm per day

None of the above

A

Transection of a peripheral nerve results in hemorrhage and retraction of the severed nerve ends. Almost immediately, degeneration of the axon distal to the injury begins. Degeneration also occurs in the proximal fragment back to the first node of Ranvier. Phagocytosis of the degenerated axonal fragments leaves neurilemmal sheath with empty cylindrical spaces where the axons were. Several days following the injury, axons from the proximal fragment begin to regrow. If they make contact with the distal neurilemmal sheath, regrowth occurs at about the rate of 1 mm/day. However, if associated trauma, fracture, infection, or separation of neurilemmal sheath ends precludes contact between axons, growth is haphazard and a traumatic neuroma is formed. When neural transection is associated with widespread soft tissue damage and hemorrhage (with increased probability of infection), many surgeons choose to delay reapproximation of the severed nerve end for 3 to 4 weeks.

Nerve injury

There are 3 types of nerve injury:
Neuropraxia
Nerve intact but electrical conduction is affected
Full recovery
Autonomic function preserved
Wallerian degeneration does not occur
Axonotmesis
Axon is damaged and the myelin sheath is preserved. The connective tissue framework is not affected.
Wallerian degeneration occurs.
Neurotmesis
Disruption of the axon, myelin sheath and surrounding connective tissue.
Wallerian degeneration occurs.

Wallerian Degeneration
Axonal degeneration distal to the site of injury.
Typically begins 24-36 hours following injury.
Axons are excitable prior to degeneration occurring.
Myelin sheath degenerates and is phagocytosed by tissue macrophages.

Nerve repair
Neuronal repair may only occur physiologically where nerves are in direct contact. Where a large defect is present, the process of nerve regeneration is hampered. It may not occur at all or result in the formation of a neuroma. Where nerve regrowth occurs it is typically at a rate of 1mm per day.

1288
Q

A 78 year old man is undergoing a femoro-popliteal bypass graft. The operation is not progressing well and the surgeon is complaining of poor access. Retraction of which of the following structures will improve access to the femoral artery distally?

Quadriceps

Adductor longus

Adductor magnus

Pectineus

Sartorius

A

At the lower border of the femoral triangle the femoral artery passes under the sartorius muscle. This can be retracted to improve access.

Femoral triangle anatomy

Boundaries
Superiorly Inguinal ligament
Laterally Sartorius
Medially Adductor longus
Floor Iliopsoas, adductor longus and pectineus
Roof
Fascia lata and Superficial fascia
Superficial inguinal lymph nodes (palpable below the inguinal ligament)
Long saphenous vein

Contents
Femoral vein (medial to lateral)
Femoral artery-pulse palpated at the mid inguinal point
Femoral nerve
Deep and superficial inguinal lymph nodes
Lateral cutaneous nerve
Great saphenous vein
Femoral branch of the genitofemoral nerve

1289
Q

Which of the following is not a branch of the external carotid artery?

Facial artery

Lingual artery

Superior thyroid artery

Mandibular artery

Maxillary artery

A

External carotid artery branches mnemonic:

‘Some Angry Lady Figured Out PMS’

Superior thyroid (superior laryngeal artery branch)
Ascending pharyngeal
Lingual
Facial (tonsillar and labial artery)
Occipital
Posterior auricular
Maxillary (inferior alveolar artery, middle meningeal artery)
Superficial temporal

External carotid artery

The external carotid commences immediately lateral to the pharyngeal side wall. It ascends and lies anterior to the internal carotid and posterior to the posterior belly of digastric and stylohyoid. More inferiorly it is covered by sternocleidomastoid, passed by hypoglossal nerves, lingual and facial veins.
It then pierces the fascia of the parotid gland finally dividing into its terminal branches within the gland itself.

Surface marking of the carotid
This is an imaginary line drawn from the bifurcation of the common carotid passing behind the angle of the jaw to a point immediately anterior to the tragus of the ear.

Branches of the external carotid artery
It has six main branches, three in front, two behind and one deep.
Three in front Superior thyroid
Lingual
Facial
Two behind Occipital
Posterior auricular
Deep Ascending pharyngeal

It terminates by dividing into the superficial temporal and maxillary arteries in the parotid gland.

1290
Q

Which of the disorders listed below is characterized by a PTEN mutation and intestinal hamartomas?

Familial adenomatous polyposis coli

MYH associated polyposis

Peutz-Jeghers syndrome

Cowden disease

Lynch syndrome

A

D-Cowden Disease

PTEN is a tumour suppressor gene and loss of function mutations result in up regulation of the mTOR pathway.

Polyposis syndromes

Syndrome Genetic defect Features Screening and management Associated disorders
Familial adenomatous polyposis Mutation of APC gene (80%) cases, dominant Typically over 100 colonic adenomas
Cancer risk of 100%
20% are new mutations If known to be at risk then predictive genetic testing as teenager
Annual flexible sigmoidoscopy from 15 years
If no polyps found then 5 yearly colonoscopy started at age 20
Polyps found = resectional surgery (resection and pouch Vs sub total colectomy and IRA) Gastric fundal polyps (50%).
Duodenal polyps 90%.
If severe duodenal polyposis cancer risk of 30% at 10 years.
Abdominal desmoid tumours.
MYH associated polyposis Biallelic mutation of mut Y human homologue (MYH) on chromosome 1p, recessive Multiple colonic polyps
Later onset right sided cancers more common than in FAP
100% cancer risk by age 60 Once identified resection and ileoanal pouch reconstruction is recommended
Attenuated phenotype - regular colonoscopy Duodenal polyposis in 30%
Associated with increased risk of breast cancer (self examination)
Peutz -Jeghers syndrome STK11 (LKB1) mutation on chromosome 19 in some (but not all) cases, dominant Multiple benign intestinal hamartomas
Episodic obstruction and intussceception
Increased risk of GI cancers (colorectal cancer 20%, gastric 5%)
Increased risk of breast, ovarian, cervical pancreatic and testicular cancers Annual examination
Pan intestinal endoscopy every 2-3 years Malignancies at other sites
Classical pigmentation pattern
Cowden disease Mutation of PTEN gene on chromosome 10q22, dominant Macrocephaly
Multiple intestinal hamartomas
Multiple trichilemmomas
89% risk of cancer at any site
16% risk of colorectal cancer Targeted individualised screening Breast cancer (81% risk)
Thyroid cancer and non toxic goitre
Uterine cancer
HNPCC (Lynch syndrome) Germline mutations of DNA mismatch repair genes Colo rectal cancer 30-70%
Endometrial cancer 30-70%
Gastric cancer 5-10%
Scanty colonic polyps may be present
Colonic tumours likely to be right sided and mucinous Colonoscopy every 1-2 years from age 25
Consideration of prophylactic surgery
Extra colonic surveillance recommended Extra colonic cancers

1291
Q

Which of the following structures separates the subclavian artery and vein?

Digastric muscle

Prevertebral fascia

Anterior scalene muscle

Middle scalene muscle

Omohyoid

A

The anterior scalene muscle is an important anatomical landmark and separates the subclavian vein (anterior) from the subclavian artery (posterior).

Scalene muscles

The 3 paired muscles are:
Scalenus anterior: Elevate 1st rib and laterally flex the neck to same side
Scalenus medius: Same action as scalenus anterior
Scalenus posterior: Elevate 2nd rib and tilt the cervical spine

Innervation Spinal nerves C4-6
Origin Transverse processes C2 to C7
Insertion First and second ribs
Important relations
The brachial plexus and subclavian artery pass between the anterior and middle scalenes through a space called the scalene hiatus/fissure.
The subclavian vein and phrenic nerve pass anteriorly to the anterior scalene as it crosses over the first rib.

Thoracic outlet syndrome
The scalenes are at risk of adhering to the fascia surrounding the brachial plexus or shortening causing compression of the brachial plexus when it passes between the clavicle and 1st rib causing thoracic outlet syndrome.

1292
Q

A 5 year old boy presents with a painful limp. The symptoms have been present for 8 weeks. Two hip x-rays have been performed and appear normal. What is the best course of action?

Arrange a hip USS

Arrange a hip CT scan

Arrange a hip MRI

Arrange a further hip X-ray

Discharge and reassure

A

Perthes disease should be suspected in boys over 4 years old presenting with a limp. Early disease can be missed on x-ray. An MRI will often demonstrate areas of hypoperfusion and subtle changes that allow for earlier diagnosis. A bone scan is an alternative option.
Paediatric orthopaedics

Diagnosis Mode of presentation Treatment Radiology
Developmental dysplasia of the hip Usually diagnosed in infancy by screening tests. May be bilateral, when disease is unilateral there may be leg length inequality. As disease progresses child may limp and then early onset arthritis. More common in extended breech babies. Splints and harnesses or traction. In later years osteotomy and hip realignment procedures may be needed. In arthritis a joint replacement may be needed. However, this is best deferred if possible as it will almost certainly require revision Initially no obvious change on plain films and USS gives best resolution until 3 months of age. On plain films Shentons line should form a smooth arc
Perthes Disease Hip pain (may be referred to the knee) usually occurring between 5 and 12 years of age. Bilateral disease in 20%. Remove pressure from joint to allow normal development. Physiotherapy. Usually self-limiting if diagnosed and treated promptly. X-rays will show flattened femoral head. Eventually in untreated cases the femoral head will fragment.
Slipped upper femoral epiphysis Typically seen in obese male adolescents. Pain is often referred to the knee. Limitation to internal rotation is usually seen. Knee pain is usually present 2 months prior to hip slipping. Bilateral in 20%. Bed rest and non-weight bearing. Aim to avoid avascular necrosis. If severe slippage or risk of it occurring then percutaneous pinning of the hip may be required. X-rays will show the femoral head displaced and falling inferolaterally (like a melting ice cream cone) The Southwick angle gives indication of disease severity

1293
Q

In which of the following cranial bones does the foramen spinosum lie?

Sphenoid bone

Frontal bone

Temporal bone

Occipital bone

Parietal bone

A

The foramen spinosum (which transmits the middle meningeal artery and vein) lies in the sphenoid bone.

Foramina of the base of the skull

Foramen Location Contents
Foramen ovale Sphenoid bone Otic ganglion
V3 (Mandibular nerve:3rd branch of
trigeminal)
Accessory meningeal artery
Lesser petrosal nerve
Emissary veins
Foramen spinosum Sphenoid bone Middle meningeal artery
Meningeal branch of the Mandibular nerve
Foramen rotundum Sphenoid bone Maxillary nerve (V2)
Foramen lacerum/ carotid canal Located between the sphenoid, the apex of the petrous temporal and the basilar part of the occipital Base of the medial pterygoid plate.
Internal carotid artery*
Nerve and artery of the pterygoid canal
Jugular foramen Temporal bone Anterior: inferior petrosal sinus
Intermediate: glossopharyngeal, vagus, and accessory nerves.
Posterior: sigmoid sinus (becoming the internal jugular vein) and some meningeal branches from the occipital and ascending pharyngeal arteries.
Foramen magnum Occipital bone Anterior and posterior spinal arteries
Vertebral arteries
Medulla oblongata
Stylomastoid foramen Temporal bone Stylomastoid artery
Facial nerve
Superior orbital fissure Sphenoid bone Oculomotor nerve (III)
Recurrent meningeal artery
Trochlear nerve (IV)
Lacrimal, frontal and nasociliary branches of opthalmic nerve (V1)
Abducent nerve (VI)
Superior ophthalmic vein

*= In life the foramen lacerum is occluded by a cartilagenous plug. The ICA initially passes into the carotid canal which ascends superomedially to enter the cranial cavity through the foramen lacerum.

1294
Q

A 22 year old lady is breastfeeding her first child. One week post partum she presents with a tender indurated mass in the right breast. Infection with which of the following is most likely?

Staphylococcus aureus

Streptococcus pyogenes

Streptococcus bovis

Klebsiella

Streptococcus viridans

A

Staphylococcus aureus is the commonest cause of lactational mastitis.

Mastitis

Mastitis refers to infection within the breast, the commonest variant, lactational mastitis is related to breast feeding and occurs as a result of inoculation of the breast tissue (which may have breaks in epithelial integrity) with staphylococcus aureus that is carried in the infants oropharynx. The result is a tender erythematous breast. Fever is common. Treatment is usually with encouraging breast drainage (e.g. breast pumps) and antibiotics. Imaging with USS will demonstrate any underlying abscess. The preferred treatment for this complication is percutaneous aspiration where this is possible. Where the overlying epithelium is non viable, debridement may be needed, there is a risk that this may be complicated by the development of a subsequent mammary duct fistula.

1295
Q

A 42 year old singer is admitted for a thyroidectomy. Post operatively she is only able to make a gargling noise. Her voice sounds breathy. What is the most likely explanation?

Bilateral inferior laryngeal nerve injury

Unilateral inferior laryngeal nerve injury

Unilateral superior laryngeal nerve injury

Bilateral superior laryngeal nerve injury

Lacunar infarct

A

This patient has diplophonia which causes a gargling sound. This is associated with dysphagia. This can also be caused by a vagus nerve lesion, but the recurrent laryngeal nerve is more at risk of damage. In bilateral injury, the airway is at risk and therefore this takes precedence.

Voice production

There are 2 main nerves involved:

Superior laryngeal nerve (SLN)
Innervates the cricothyroid muscle

Since the cricothyroid muscle is involved in adjusting the tension of the vocal fold for high notes during singing, SLN paresis and paralysis result in:

a. Abnormalities in pitch
b. Inability to sing with smooth change to each higher note (glissando or pitch glide)

Recurrent laryngeal nerve (RLN)/Inferior laryngeal nerve
Innervates intrinsic larynx muscles

a. Opening vocal folds (as in breathing, coughing)
b. Closing vocal folds for vocal fold vibration during voice use
c. Closing vocal folds during swallowing

1296
Q

A 19 year old lady is admitted to ITU with severe meningococcal sepsis. She is on maximal inotropic support and a CT scan of her chest and abdomen is performed. The adrenal glands show evidence of diffuse haemorrhage. What is the most likely explanation?

Waterhouse- Friderichsen syndrome

Nelsons syndrome

Walker - Warburg syndrome

Spontaneous bleed into a malignant lesion

Spontaneous bleed into a cyst

A

Waterhouse - Friderichsen syndrome is defined as adrenal gland failure due to bleeding into the adrenal glands. It is caused by severe bacterial infection (most commonly the meningococcus Neisseria meningitidis).

The bacterial infection leads to massive hemorrhage into one or (usually) both adrenal glands. It is characterised by overwhelming bacterial infection meningococcemia leading to massive blood invasion, organ failure, coma, haemodynamic shock, disseminated intravascular coagulation with widespread purpura, rapidly developing adrenocortical insufficiency and death.

Note the scenarios relating to spontaneous bleeding related to a unilateral process and cannot therefore be the correct answer.

Phaeochromocytoma and adrenal lesions

Phaeochromocytoma
Neuroendocrine tumour of the chromaffin cells of the adrenal medulla. Hypertension and hyperglycaemia are often found.
10% of cases are bilateral.
10% occur in children.
11% are malignant (higher when tumour is located outside the adrenal).
10% will not be hypertensive.

Familial cases are usually linked to the Multiple endocrine neoplasia syndromes (considered under its own heading).

Most tumours are unilateral (often right sided) and smaller than 10cm.

Diagnosis
Urine and plasma metanephrine levels.

CT and MRI scanning are both used to localise the lesion.

Treatment
Patients require medical therapy first. An irreversible alpha adrenoreceptor blocker should be given, although minority may prefer reversible blockade(1). Labetolol may be co-administered for cardiac chronotropic control. Isolated beta blockade should not be considered as it will lead to unopposed alpha activity.

These patients are often volume depleted and will often require moderate volumes of intra venous normal saline perioperatively.

Once medically optimised the phaeochromocytoma should be removed. Most adrenalectomies can now be performed using a laparoscopic approach(2). The adrenals are highly vascular structures and removal can be complicated by catastrophic haemorrhage in the hands of the inexperienced. This is particularly true of right sided resections where the IVC is perilously close. Should the IVC be damaged a laparotomy will be necessary and the defect enclosed within a Satinsky style vascular clamp and the defect closed with prolene sutures. Attempting to interfere with the IVC using any instruments other than vascular clamps will result in vessel trauma and make a bad situation much worse.

Incidental adrenal lesions
Adrenal lesions may be identified on CT scanning performed for other reasons(3). Factors suggesting benign disease on CT include(4):
Size less than 3cm
Homogeneous texture
Lipid rich tissue
Thin wall to lesion

All patients with incidental lesions should be managed jointly with an endocrinologist and full work up as described above. Patients with functioning lesions or those with adverse radiological features (Particularly size >3cm) should proceed to surgery.

References
1. Weingarten TN, Cata JP, O’Hara JF, Prybilla DJ, Pike TL, Thompson GB, et al. Comparison of two preoperative medical management strategies for laparoscopic resection of pheochromocytoma. Urology. 2010 Aug;76(2):508 e6-11.

  1. Nguyen PH, Keller JE, Novitsky YW, Heniford BT, Kercher KW. Laparoscopic approach to adrenalectomy: review of perioperative outcomes in a single center. Am Surg. 2011 May;77(5):592-6.
  2. Ng VW, Ma RC, So WY, Choi KC, Kong AP, Cockram CS, et al. Evaluation of functional and malignant adrenal incidentalomas. Arch Intern Med. 2010 Dec 13;170(22):2017-20.
  3. Muth A, Hammarstedt L, Hellstrom M, Sigurjonsdottir HA, Almqvist E, Wangberg B. Cohort study of patients with adrenal lesions discovered incidentally. Br J Surg. 2011 May 27.
1297
Q

Where are accessory spleens not found?

Gonads

Tail of pancreas

Greater omentum

Splenorenal ligament

Ureter

A

Accessory spleens

  • 10% population
  • 1 cm size
  • locations: hilum of the spleen, tail of the pancreas, along the splenic vessels, in the gastrosplenic ligament, the splenorenal ligament, the walls of the stomach or intestines, the greater omentum, the mesentery, the gonads

Spleen

The spleen is located in the left upper quadrant of the abdomen and its size can vary depending upon the amount of blood it contains. The typical adult spleen is 12.5cm long and 7.5cm wide. The usual weight of the adult spleen is 150g.
The exact position of the spleen can vary with respiratory activity, posture and the state of surrounding viscera. It usually lies obliquely with its long axis aligned to the 9th, 10th and 11th ribs. It is separated from these ribs by both diaphragm and pleural cavity. The normal spleen is not palpable.

The shape of the spleen is influenced by the state of the colon and stomach. Gastric distension will cause the spleen to resemble the shape of an orange segment. Colonic distension will cause it to become more tetrahedral.

The spleen is almost entirely covered by peritoneum, which adheres firmly to its capsule. Recesses of the greater sac separate it from the stomach and kidney. It develops from the upper dorsal mesogastrium, remaining connected to the posterior abdominal wall and stomach by two folds of peritoneum; the lienorenal ligament and gastrosplenic ligament. The lienorenal ligament is derived from peritoneum where the wall of the general peritoneum meets the omental bursa between the left kidney and spleen; the splenic vessels lie in its layers. The gastrosplenic ligament also has two layers, formed by the meeting of the walls of the greater sac and omental bursa between spleen and stomach, the short gastric and left gastroepiploic branches of the splenic artery pass in its layers. Laterally, the spleen is in contact with the phrenicocolic ligament.

Relations
Superiorly Diaphragm
Anteriorly Gastric impression
Posteriorly Kidney
Inferiorly Colon
Hilum Tail of pancreas and splenic vessels (splenic artery divides here, branches pass to the white pulp transporting plasma)

Contents
White pulp Immune function. Contains central trabecular artery. The germinal centres are supplied by arterioles called penicilliary radicles.
Red pulp Filters abnormal red blood cells.

Function
Filtration of abnormal blood cells and foreign bodies such as bacteria.
Immunity: IgM. Production of properdin, and tuftsin which help target fungi and bacteria for phagocytosis.
Haematopoiesis: up to 5th month gestation or in haematological disorders.
Pooling: storage of 40% platelets.
Iron reutilisation
Storage monocytes

Disorders of the spleen
Massive splenomegaly
Myelofibrosis
Chronic myeloid leukaemia
Visceral leishmaniasis (kala-azar)
Malaria
Gaucher’s syndrome

Other causes (as above plus)
Portal hypertension e.g. secondary to cirrhosis
Lymphoproliferative disease e.g. CLL, Hodgkin’s
Haemolytic anaemia
Infection: hepatitis, glandular fever
Infective endocarditis
Sickle-cell*, thalassaemia
Rheumatoid arthritis (Felty’s syndrome)

*the majority of adult patients with sickle-cell will have an atrophied spleen due to repeated infarction

1298
Q

Adult lung volumes. Which statement is false?

In restrictive lung disease the FEV1/FVC ratio is normal

Residual volume is increased in emphysema

Functional residual capacity is measured by helium dilution test

The tidal volume is approximately 340mls in females

The vital capacity is increased in Guillain Barre syndrome

A

E-
FEV1/FVC is normal or >80% in restrictive lung disease such as pulmonary fibrosis. The ratio is reduced in obstructive airways disease.

The functional residual capacity, residual volume and the total lung capacity cannot be measured with spirometry. They can only be measured by helium dilution.
The vital capacity is reduced in:
1. Pulmonary fibrosis/infiltration/oedema/effusions
2. Weak respiratory muscles e.g. MG, GBS, myopathies
3. Skeletal abnormalities e.g. chest wall abnormalities
Definitions

Tidal volume (TV)
Is the volume of air inspired and expired during each ventilatory cycle at rest.
It is normally 500mls in males and 340mls in females.
Inspiratory reserve volume (IRV)
Is the maximum volume of air that can be forcibly inhaled following a normal inspiration. 3000mls.
Expiratory reserve volume (ERV)
Is the maximum volume of air that can be forcibly exhaled following a normal expiration. 1000mls.
Residual volume (RV)
Is that volume of air remaining in the lungs after a maximal expiration.
RV = FRC - ERV. 1500mls.
Functional residual capacity (FRC)
Is the volume of air remaining in the lungs at the end of a normal expiration.
FRC = RV + ERV. 2500mls.
Vital capacity (VC)
Is the maximal volume of air that can be forcibly exhaled after a maximal inspiration.
VC = TV + IRV + ERV. 4500mls in males, 3500mls in females.
Total lung capacity (TLC)
Is the volume of air in the lungs at the end of a maximal inspiration.
TLC = FRC + TV + IRV = VC + RV. 5500-6000mls.
Forced vital capacity (FVC)
The volume of air that can be maximally forcefully exhaled.

1299
Q

A 60 year old female attends the preoperative hernia clinic. She reports some visual difficulty. On examination she is noted to have a homonymous hemianopia. Where is the lesion most likely to be?

Frontal lobe

Pituitary gland

Parietal lobe

Optic chiasm

Optic tract

A

Lesions before optic chiasm:
Monocular vision loss = Optic nerve lesion
Bitemporal hemianopia = Optic chiasm lesion

Lesions after the optic chiasm:
Homonymous hemianopia = Optic tract lesion
Upper quadranopia = Temporal lobe lesion
Lower quadranopia = Parietal lobe lesion
Unfortunately we thought as surgeons we could forget about visual field defects! However the college seem to like them. Remember a homonymous hemianopia is indicative of an optic tract lesion. Parietal lobe lesions tend to cause inferior quadranopias and there is a bitemporal hemianopia with optic chiasm lesion or pituitary tumours.

Visual field defects

left homonymous hemianopia means visual field defect to the left, i.e. Lesion of right optic tract
homonymous quadrantanopias: PITS (Parietal-Inferior, Temporal-Superior)
incongruous defects = optic tract lesion; congruous defects = optic radiation lesion or occipital cortex

Homonymous hemianopia
Incongruous defects: lesion of optic tract
Congruous defects: lesion of optic radiation or occipital cortex
Macula sparing: lesion of occipital cortex

Homonymous quadrantanopias
Superior: lesion of temporal lobe
Inferior: lesion of parietal lobe
Mnemonic = PITS (Parietal-Inferior, Temporal-Superior)

Bitemporal hemianopia
Lesion of optic chiasm
Upper quadrant defect > lower quadrant defect = inferior chiasmal compression, commonly a pituitary tumour
Lower quadrant defect > upper quadrant defect = superior chiasmal compression, commonly a craniopharyngioma

1300
Q

A 34 year old teacher attends A&E with a swollen leg. She has been in England for 2 weeks having previously lived in the Democratic Republic of the Congo. She lives in an area prevalent with mosquitoes and where there is poor sanitation. What is the most likely diagnosis?

Meige’s disease

Lymphoma

Milroy’s disease

Filariasis

Malaria

A

Filariasis is caused by the nematode Wuchereria bancrofti, which is mainly spread by mosquito. The oedema can be gross leading to elephantitis. Treatment is with diethylcarbamazine.
Lymphoedema

  • Due to impaired lymphatic drainage in the presence of normal capillary function.
    Lymphoedema causes the accumulation of protein rich fluid, subdermal fibrosis and dermal thickening.
    Characteristically fluid is confined to the epifascial space (skin and subcutaneous tissues); muscle compartments are free of oedema. It involves the foot, unlike other forms of oedema. There may be a ‘buffalo hump’ on the dorsum of the foot and the skin cannot be pinched due to subcutaneous fibrosis.

Causes of lymphoedema

Primary
Congenital < 1 year: sporadic, Milroy’s disease
Onset 1-35 years: sporadic, Meige’s disease
> 35 years: Tarda
Secondary
Bacterial/fungal/parasitic infection (filariasis)
Lymphatic malignancy
Radiotherapy to lymph nodes
Surgical resection of lymph nodes
DVT
Thrombophlebitis

Indications for surgery
Marked disability or deformity from limb swelling
Lymphoedema caused by proximal lymphatic obstruction with patent distal lymphatics suitable for a lymphatic drainage procedure
Lymphocutaneous fistulae and megalymphatics

Procedures
Homans operation Reduction procedure with preservation of overlying skin (which must be in good condition). Skin flaps are raised and the underlying tissue excised. Limb circumference typically reduced by a third.
Charles operation All skin and subcutaneous tissue around the calf are excised down to the deep fascia. Split skin grafts are placed over the site. May be performed if overlying skin is not in good condition. Larger reduction in size than with Homans procedure.
Lymphovenous anastamosis Identifiable lymphatics are anastomosed to sub dermal venules. Usually indicated in 2% of patients with proximal lymphatic obstruction and normal distal lymphatics.

1301
Q

A 45 year old woman complains of painful tingling in her fingers. The pain is relieved by hanging the arm over the side of the bed. She has a positive Tinel’s sign at the wrist. Which of the following is most likely to contribute to her diagnosis?

Methotrexate use

Crohn’s disease

Hyperthyroidism

Tuberculosis

Rheumatoid arthritis

A

This woman has a diagnosis of carpal tunnel syndrome. Rheumatological disorders are a common cause. Clinical examination should focus on identifying stigmata of rheumatoid arthritis, such as rheumatoid nodules, vasculitic lesions and metacarpophalangeal joint arthritis.

Carpal tunnel syndrome

Carpal tunnel syndrome is caused by compression of median nerve in the carpal tunnel

History
pain/pins and needles in thumb, index, middle finger e.g. at night
patient flicks hand to obtain relief

Examination
weakness of thumb abduction
wasting of thenar eminence (NOT hypothenar)
Tinel’s sign: tapping causes paraesthesia
Phalen’s sign: flexion of wrist causes symptoms

Causes of carpal tunnel syndrome

MEDIAN TRAP Mnemonic

Myxoedema
Edema premenstrually
Diabetes
Idiopathic
Acromegaly
Neoplasm
Trauma
Rheumatoid arthritis
Amyloidosis
Pregnancy

Management
Non surgical treatment Surgery
May resolve spontaneously
Avoid precipitants and reassurance
Night-time splints
Local steroid injections Complete division of the flexor retinaculum and decompression of the tunnel (successful in approximately 80% of patients)

1302
Q

A 30 year old lady presents to the emergency department with epistaxis originating from her left nostril. She recently had a sinus infection that was addressed with topical nasal sprays. Which arteries contribute to the vascular bed most likely affected in this patient?

Ascending pharyngeal, facial and maxillary

Ascending pharyngeal, lingual and ophthalmic

Facial, great auricular and lingual

Facial, maxillary and ophthalmic

Inferior alveolar, maxillary and ophthalmic

A

These three arteries coalesce at Littles area which is a common site of epistaxis.

Nasal cavity

There are two nasal cavities separated by the median nasal septum. This structure consists of septal cartilage and two bony parts: the vomer and the perpendicular plate of the ethmoid bone. Each cavity is approximately 5cm high and 7cm long, from the nares anteriorly to the choanae posteriorly. The lateral walls of each cavity are mainly made up of the maxilla. However, the lacrimal, ethmoid and palatine bones also contribute. The ethmoid gives rise to the two upper protrusions from the lateral walls, the superior and middle conchae. The inferior concha is a separate bone. Above the superior concha lies the sphenoethmoidal recess. The spaces inferior to conchae are the superior, middle and inferior meatus. The structures draining into each space are shown below:

Sphenoethmoidal recess Sphenoidal sinus
Superior meatus Posterior ethmoidal sinus
Middle meatus Frontal sinus, maxillary sinus ,anterior and middle ethmoidal sinus
Inferior meatus Nasolacrimal duct

The main arterial supply to the septum and lateral walls is the sphenopalatine artery, a branch of the maxillary artery. The olfactory nerve conveys special sensation. Common sensation from the nasal mucosa is conveyed by the trigeminal nerve.

1303
Q

An 18 year old man is stabbed in the axilla during a fight. His axillary artery is lacerated and repaired. However, the surgeon neglects to repair an associated injury to the upper trunk of the brachial plexus. Which of the following muscles is least likely to demonstrate impaired function as a result?

Palmar interossei

Infraspinatus

Brachialis

Supinator brevis

None of the above

A

The palmar interossei are supplied by the ulnar nerve. Which lies inferiorly and is therefore less likely to be injured.

Brachial plexus

The brachial plexus extends from the neck to the axilla. It is formed by the ventral rami of the fifth to the eighth cervical nerves with the ascending part of the first thoracic nerve.

Location of the plexus
The ventral rami which form the plexus enter the lower part of the posterior triangle of the neck in series with the ventral rami of the cervical plexus. The second part of the subclavian artery lies immediately anterior to the lower two rami. The upper three rami intermingle and pass inferolaterally towards the axilla and subclavian artery. They are enclosed within an extension of the prevertebral fascia. In the neck the plexus lies deep to platysma, the supraclavicular nerves, inferior belly of omohyoid and the transverse cervical artery. It then passes deep to the clavicle and the suprascapular vessels, to enter the axilla, and thence surround the second part of the axillary artery

Composition of the plexus
Ventral rami, the roots of the plexus, lie between scalenus medius and anterior.

As they enter the posterior triangle, the upper two (C5,6) and lower two (C8, T1) roots of the plexus unite to form the upper and lower trunks of the plexus respectively. Meanwhile, C7 continues as the middle trunk. The lower trunk may groove the superior surface of the first rib posterior to the subclavian artery, and the root from the first ventral ramus is always in contact with it.

Each trunk divides into ventral and dorsal divisions which are destined to supply the anterior (flexor) and posterior (extensor) parts of the upper limb.

The cords of the plexus are formed in the axilla. The dorsal divisions unite to form the posterior cord (C5-8). The ventral divisions of the upper and middle trunks unite to form the lateral cord (C5-7), while the ventral divisions of the lower trunk continues as the medial cord (C8-T1). The cords are named according to their relationship to the axillary artery. Each cord terminates by dividing into two main branches at the beginning of the third part of the artery.

Sympathetic communications
The fifth and sixth cervical ventral rami receive grey rami communicantes from the middle cervical ganglion, while the two or more grey rami communicantes pass from the inferior cervical ganglion to the seventh and eighth cervical ventral rami. The first thoracic ventral ramus receives its grey ramus from the cervicothoracic ganglion. Its for this reason that inferior plexus injury can be complicated by a Horners syndrome.

Summary
Origin Anterior rami of C5 to T1
Sections of the plexus
Roots, trunks, divisions, cords, branches
Mnemonic:Real Teenagers Drink Cold Beer
Roots
Located in the posterior triangle
Pass between scalenus anterior and medius
Trunks
Located posterior to middle third of clavicle
Upper and middle trunks related superiorly to the subclavian artery
Lower trunk passes over 1st rib posterior to the subclavian artery
Divisions Apex of axilla
Cords Related to axillary artery

1304
Q

Which of the following statements relating to gastric cancer is untrue?

It is associated with chronic helicobacter pylori infection

5% of gastric malignancies are due to lymphoma

In the Lauren classification the diffuse type of adenocarcinoma typically presents as a large exophytic growth in the antrum

Smoking is a risk factor

It is associated with acanthosis nigricans

A

Barium meal appearances of linitis plastica:
Due to the increased rigidity of the wall, the stomach cannot be adequately distended, with only a narrow lumen identified. The normal mucosal fold pattern is absent, either distorted, thickened or nodular.
The Lauren classification describes a diffuse type of adenocarcinoma (Linitis plastica type lesion) and an intestinal type. The diffuse type is often deeply infiltrative and may be difficult to detect on endoscopy. Barium meal appearances can be characteristic.

Gastric cancer

Overview
There are 700,000 new cases of gastric cancer worldwide each year. It is most common in Japan and less common in western countries. It is more common in men and incidence rises with increasing age. The exact cause of many sporadic cancer is not known, however, familial cases do occur in HNPCC families. In addition, smoking and smoked or preserved foods increase the risk. Japanese migrants retain their increased risk (decreased in subsequent generations). The distribution of the disease in western countries is changing towards a more proximal location (perhaps due to rising obesity).

Pathology
There is some evidence of support a stepwise progression of the disease through intestinal metaplasia progressing to atrophic gastritis and subsequent dysplasia, through to cancer. The favoured staging system is TNM. The risk of lymph node involvement is related to size and depth of invasion; early cancers confined to submucosa have a 20% incidence of lymph node metastasis. Tumours of the gastro-oesophageal junction are classified as below:

Type 1 True oesophageal cancers and may be associated with Barrett’s oesophagus.
Type 2 Carcinoma of the cardia, arising from cardiac type epithelium
or short segments with intestinal metaplasia at the oesophagogastric junction.
Type 3 Sub cardial cancers that spread across the junction. Involve similar nodal stations to gastric cancer.

Groups for close endoscopic monitoring
Intestinal metaplasia of columnar type
Atrophic gastritis
Low to medium grade dysplasia
Patients who have previously undergone resections for benign peptic ulcer disease (except highly selective vagotomy).

Referral to endoscopy

Patients of any age with dyspepsia and any of the following Patients without dyspepsia Worsening dyspepsia
Chronic gastrointestinal bleeding Dysphagia Barretts oesophagus
Dysphagia Unexplained abdominal pain or weight loss Intestinal metaplasia
Weight loss Vomiting Dysplasia
Iron deficiency anaemia Upper abdominal mass Atrophic gastritis
Upper abdominal mass Jaundice Patient aged over 55 years with unexplained or persistent dyspepsia

Staging
CT scanning of the chest abdomen and pelvis is the routine first line staging investigation in most centres.
Laparoscopy to identify occult peritoneal disease
PET CT (particularly for junctional tumours)

Treatment
Proximally sited disease greater than 5-10cm from the OG junction may be treated by sub total gastrectomy
Total gastrectomy if tumour is <5cm from OG junction
For type 2 junctional tumours (extending into oesophagus) oesophagogastrectomy is usual
Endoscopic sub mucosal resection may play a role in early gastric cancer confined to the mucosa and perhaps the sub mucosa (this is debated)
Lymphadenectomy should be performed. A D2 lymphadenectomy is widely advocated by the Japanese, the survival advantages of extended lymphadenectomy have been debated. However, the overall recommendation is that a D2 nodal dissection be undertaken.
Most patients will receive chemotherapy either pre or post operatively.

Prognosis

UK Data

Disease extent Percentage 5 year survival
All RO resections 54%
Early gastric cancer 91%
Stage 1 87%
Stage 2 65%
Stage 3 18%

Operative procedure

Total Gastrectomy , lymphadenectomy and Roux en Y anastomosis

General anaesthesia
Prophylactic intravenous antibiotics
Incision: Rooftop.
Perform a thorough laparotomy to identify any occult disease.
Mobilise the left lobe of the liver off the diaphragm and place a large pack over it. Insert a large self retaining retractor e.g. omnitract or Balfour (take time with this, the set up should be perfect). Pack the small bowel away.
Begin by mobilising the omentum off the transverse colon.
Proceed to detach the short gastric vessels.
Mobilise the pylorus and divide it at least 2cm distally using a linear cutter stapling device.
Continue the dissection into the lesser sac taking the lesser omentum and left gastric artery flush at its origin.
The lymph nodes should be removed en bloc with the specimen where possible.
Place 2 stay sutures either side of the distal oesophagus. Ask the anaesthetist to pull back on the nasogastric tube. Divide the distal oesophagus and remove the stomach.
The oesphago jejunal anastomosis should be constructed. Identify the DJ flexure and bring a loop of jejunum up to the oesophagus (to check it will reach). Divide the jejunum at this point. Bring the divided jejunum either retrocolic or antecolic to the oesophagus. Anastamose the oesophagus to the jejunum, using either interrupted 3/0 vicryl or a stapling device. Then create the remainder of the Roux en Y reconstruction distally.
Place a jejunostomy feeding tube.
Wash out the abdomen and insert drains (usually the anastomosis and duodenal stump). Help the anaesthetist insert the nasogastric tube (carefully!)
Close the abdomen and skin.
Enteral feeding may commence on the first post-operative day. However, most surgeons will leave patients on free NG drainage for several days and keep them nil by mouth.

1305
Q

What is the lymphatic drainage of the upper ureter?

Common iliac nodes

Para aortic nodes

External iliac nodes

Internal iliac nodes

Meso colic nodes

A

The upper ureter drains to the para-aortic nodes, the lower ureter drains to the common iliac nodes.

Ureter

25-35 cm long
Muscular tube lined by transitional epithelium
Surrounded by thick muscular coat. Becomes 3 muscular layers as it crosses the bony pelvis
Retroperitoneal structure overlying transverse processes L2-L5
Lies anterior to bifurcation of iliac vessels
Blood supply is segmental; renal artery, aortic branches, gonadal branches, common iliac and internal iliac
Lies beneath the uterine artery

1306
Q

A 43 year old woman has had recurrent episodes of breast infections. She has received multiple courses of antibiotics and is troubled by persisting green nipple discharge. Clinical examination reveals green nipple discharge, but no discrete lump. Imaging with mammography and ultrasound is reassuring (U2, M2). What is the best course of action?

Undertake a Hadfields procedure

Undertake a microdochectomy

Arrange a breast MRI scan

Express fluid for cytology

Prescribe danazol

A

A- Multiple duct pathology= total duct excision
Single duct pathology= microdochectomy
This woman has troublesome duct ectasia and total duct excision is warranted.

Nipple discharge

Causes of nipple discharge
Physiological During breast feeding
Galactorrhoea Commonest cause may be response to emotional events, drugs such as histamine receptor antagonists are also implicated
Hyperprolactinaemia
Commonest type of pituitary tumour
Microadenomas <1cm in diameter
Macroadenomas >1cm in diameter
Pressure on optic chiasm may cause bitemporal hemianopia
Mammary duct ectasia
Dilatation breast ducts.
Most common in menopausal women
Discharge typically thick and green in colour
Most common in smokers
Carcinoma
Often blood stained
May be underlying mass or axillary lymphadenopathy
Intraductal papilloma
Commoner in younger patients
May cause blood stained discharge
There is usually no palpable lump

Assessment of patients
Examine breast and determine whether there is mass lesion present
All mass lesions should undergo Triple assessment.

Reporting of investigations
Where a mass lesion is suspected or investigations are requested these are prefixed using a system that denotes the investigation type e.g. M for mammography, followed by a numerical code as shown below:

1 No abnormality
2 Abnormality with benign features
3 Indeterminate probably benign
4 Indeterminate probably malignant
5 Malignant

Management of non malignant nipple discharge
Exclude endocrine disease
Nipple cytology unhelpful
Smoking cessation advice for duct ectasia
For duct ectasia with severe symptoms, total duct excision may be warranted.

1307
Q

A 35 year old mechanic is hit by a fork lift truck. He sustains a Gustilo and Anderson type IIIA fracture of the shaft of the left femur. What is the most appropriate course of action?

Amputation

Debridement and external fixation

Open reduction and fixation

Debridement and placement of intramedullary nail

Debridement and placement of long leg plaster cast

A

As there is no associated vascular injury the patient may be suitable for debridement of the area and external fixation. If debridement leaves a tissue defect then plastic surgical repair will be needed at a later stage. With open fractures, its best not to place intramedullary metalwork as this may become infected.

Fracture management

Bony injury resulting in a fracture may arise from trauma (excessive forces applied to bone), stress related (repetitive low velocity injury) or pathological (abnormal bone which fractures during normal use of following minimal trauma)
Diagnosis involves not just evaluating the fracture ; such as site and type of injury but also other associated injuries and distal neurovascular deficits. This may entail not just clinical examination but radiographs of proximal and distal joints.
When assessing x-rays it is important to assess for changes in length of the bone, the angulation of the distal bone, rotational effects, presence of material such as glass.

Fracture types
Fracture type Description
Oblique fracture Fracture lies obliquely to long axis of bone
Comminuted fracture >2 fragments
Segmental fracture More than one fracture along a bone
Transverse fracture Perpendicular to long axis of bone
Spiral fracture Severe oblique fracture with rotation along long axis of bone

Open Vs Closed
It is also important to distinguish open from closed injuries. The most common classification system for open fractures is the Gustilo and Anderson classification system (given below):

Grade Injury
1 Low energy wound <1cm
2 Greater than 1cm wound with moderate soft tissue damage
3 High energy wound > 10cm with extensive soft tissue damage
3 A (sub group of 3) Adequate soft tissue coverage
3 B (sub group of 3) Inadequate soft tissue coverage
3 C (sub group of 3) Associated arterial injury

Key points in management of fractures
Immobilise the fracture including the proximal and distal joints
Carefully monitor and document neurovascular status, particularly following reduction and immobilisation
Manage infection including tetanus prophylaxis
IV broad spectrum antibiotics for open injuries
As a general principle all open fractures should be thoroughly debrided ( and internal fixation devices avoided or used with extreme caution)
Open fractures constitute an emergency and should be debrided and lavaged within 6 hours of injury

1308
Q

A group of surgeons conduct a meta analysis of randomised controlled trials comparing the use of analgesic regimes following laparoscopic cholecystectomy. What level of evidence is provided by such an analysis?

V

I

II

III

IV

A

Study design: evidence and recommendations

Levels of evidence
I - evidence from meta-analysis of randomised controlled trials
II - evidence from at least one well designed controlled trial which is not randomised
III - evidence from correlation and comparative studies or use of historical controls
IV - evidence from case series
V - Expert opinion or founded on basic principles

Knowledge of the sub groups of the levels of evidence are not routinely tested in MRCS Part A.

Grading of recommendation
Grade A - based on evidence from at least one randomised controlled trial (i.e. Ia or Ib)
Grade B - based on evidence from non-randomised controlled trials (i.e. IIa, IIb or III)
Grade C - based on evidence from a panel of experts (i.e. IV)

1309
Q

Which of the following muscles is not innervated by the deep branch of the ulnar nerve?

Adductor pollicis

Hypothenar muscles

All the interosseous muscles

Extensor indicis

Third and fourth lumbricals

A

The extensor muscles are innervated by the radial nerve.

Ulnar nerve

Origin
C8, T1

Supplies (no muscles in the upper arm)
Flexor carpi ulnaris
Flexor digitorum profundus
Flexor digiti minimi
Abductor digiti minimi
Opponens digiti minimi
Adductor pollicis
Interossei muscle
Third and fourth lumbricals
Palmaris brevis

Path
Posteromedial aspect of upper arm to flexor compartment of forearm, then along the ulnar. Passes beneath the flexor carpi ulnaris muscle, then superficially over the flexor retinaculum into the palm of the hand.

Branches
Branch Supplies
Muscular branch Flexor carpi ulnaris
Medial half of the flexor digitorum profundus
Palmar cutaneous branch (Arises near the middle of the forearm) Skin on the medial part of the palm
Dorsal cutaneous branch Dorsal surface of the medial part of the hand
Superficial branch Cutaneous fibres to the anterior surfaces of the medial one and one-half digits
Deep branch Hypothenar muscles
All the interosseous muscles
Third and fourth lumbricals
Adductor pollicis
Medial head of the flexor pollicis brevis

Effects of injury
Damage at the wrist
Wasting and paralysis of intrinsic hand muscles (claw hand)
Wasting and paralysis of hypothenar muscles
Loss of sensation medial 1 and half fingers
Damage at the elbow
Radial deviation of the wrist
Clawing less in 4th and 5th digits

1310
Q

A 56 year old man presents with odynophagia and on investigation is found to have a squamous cell carcinoma of the upper third of the oesophagus. Staging investigations are negative for metastatic disease. What is the most appropriate course of action?

Segmental resection of the proximal oesophagus

Radical radiotherapy alone

Combined radical radiotherapy and chemotherapy

Ivor Lewis oesophagectomy

Photodynamic therapy

A

Oesophageal SCC can be managed with radical chemoradiotherapy. Radiotherapy alone is not usually curative. Segmental resection of the upper oesophagus is not practiced. An Ivor Lewis oesophagectomy would only address distal disease.

Oesophageal cancer - treatment

Treatments for SCC’s and adenocarcinomas of the oesophagus differ. This is primarily due to the positive outcomes that are observed when localised SCC’s (particularly of the proximal oesophagus) are treated with radical chemoradiotherapy (obviating the need for surgery).
Only those patients whose staging investigations are negative for metastatic disease should be considered for surgery.

Surgical options
Endoscopic mucosal resection Treatment for early localised adenocarcinoma of the distal oesophagus. Survival mirrors that of surgical resection for Tis and T1 disease
Transhiatal oeosphagectomy Most commonly used for junctional (type II) (1) tumours where limited thoracic oesophageal resection is required. Less morbidity than two field oesophagectomy
Ivor Lewis oesophagectomy Two stage approach for middle and distal tumours. Very commonly performed, intrathoracic anastomosis will result in mediastinitis in event of anastomotic leak. Lower incidence of recurrent laryngeal nerve injury
McKeown oesophagectomy Three field approach, may be useful for proximal tumours. Anastomotic leakage is less serious. Higher incidence of recurrent laryngeal nerve injury

Neoadjuvent and adjuvent treatment
Neoadjuvent radiotherapy alone prior to resection confers little benefit and is not routinely performed (2)
Preoperative chemotherapy is associated with a survival advantage (OE02 trial)
Peri operative (pre and post operative) chemotherapy confers a survival advantage in junctional tumours
Post operative chemotherapy is not generally recommended following oesophageal resections outside clinical trials

Palliation strategies
Combination chemotherapy improves quality of life and survival in non operable disease (3)
Trastuzumab may improve survival in patients with HER 2 positive tumours
Oesophageal intubation with self expanding metal stents is the treatment of choice in patients with occluding tumours >2cm from the cricopharyngeus
Covered metal stents are useful in cases of malignant fistulas
Laser therapy and argon plasma coagulation may be useful as therapies for tumour overgrowth and bleeding
Photodynamic therapy and ethanol injections confer little benefit and should not be routinely used

References
1. Hulscher JB, van Sandick JW, de Boer AG, et al. Extended transthoracic resection compared with limited transhiatal resection for adenocarcinoma of the esophagus. N Engl J Med 2002;347:1662-9
2. Deng HY, Wang WP, Wang YC, Hu WP, Ni PZ, Lin YD, Chen LQ. Neoadjuvant chemoradiotherapy or chemotherapy? A comprehensive systematic review and meta-analysis of the options for neoadjuvant therapy for treating oesophageal cancer. Eur J Cardiothorac Surg. 2017 Mar 1;51(3):421-431. doi: 10.1093/ejcts/ezw315.
3.Khin MO, Bromham N, Harrison M, Eadon H. Assessment and management of oesophago-gastric cancer: summary of NICE guidance. BMJ. 2018 Jan 29;360:k213. doi: 10.1136/bmj.k213

1311
Q

At which level is the hilum of the left kidney located?

L1

L2

T12

T11

L3

A

Remember L1 (‘left one’) is the level of the hilum of the left kidney

Renal anatomy

Each kidney is about 11cm long, 5cm wide and 3cm thick. They are located in a deep gutter alongside the projecting vertebral bodies, on the anterior surface of psoas major. In most cases the left kidney lies approximately 1.5cm higher than the right. The upper pole of both kidneys approximates with the 11th rib (beware pneumothorax during nephrectomy). On the left hand side the hilum is located at the L1 vertebral level and the right kidney at level L1-2. The lower border of the kidneys is usually alongside L3.

The table below shows the anatomical relations of the kidneys:

Relations
Relations Right Kidney Left Kidney
Posterior Quadratus lumborum, diaphragm, psoas major, transversus abdominis Quadratus lumborum, diaphragm, psoas major, transversus abdominis
Anterior Hepatic flexure of colon Stomach, Pancreatic tail
Superior Liver, adrenal gland Spleen, adrenal gland

Fascial covering
Each kidney and suprarenal gland is enclosed within a common layer of investing fascia, derived from the transversalis fascia. It is divided into anterior and posterior layers (Gerotas fascia).

Renal structure
Kidneys are surrounded by an outer cortex and an inner medulla which usually contains between 6 and 10 pyramidal structures. The papilla marks the innermost apex of these. They terminate at the renal pelvis, into the ureter.
Lying in a hollow within the kidney is the renal sinus. This contains:
1. Branches of the renal artery
2. Tributaries of the renal vein
3. Major and minor calyces’s
4. Fat

Structures at the renal hilum
The renal vein lies most anteriorly, then renal artery (it is an end artery) and the ureter lies most posterior.

1312
Q

Which of the structures listed below overlies the cephalic vein?

Extensor retinaculum

Bicipital aponeurosis

Biceps muscle

Antebrachial fascia

None of the above

A

The cephalic vein is superficially located in the upper limb and overlies most the fascial planes. It pierces the coracoid membrane (continuation of the clavipectoral fascia) to terminate in the axillary vein. It lies anterolaterally to biceps.

Cephalic vein

Path
Dorsal venous arch drains laterally into the cephalic vein
Crosses the anatomical snuffbox and travels laterally up the arm
At the antecubital fossa connected to the basilic vein by the median cubital vein
Pierces deep fascia of deltopectoral groove to join axillary vein

1313
Q

A 32 year old male presents with a swollen right scrotum which has developed over 3 weeks after being kicked in the groin area. There is a non tense swelling of the right scrotum and the underlying testis cannot be easily palpated. A dipstick is positive for nitrates only. What is the most likely diagnosis?

Epididymo-orchitis

Testicular torsion

Torsion of testicular appendage

Hydrocele

Haematocele

A

Haematoceles develop over minutes/ hours and occur after the event
This is a secondary hydrocele which occurs in patients aged 20-40 years. It develops over days/ weeks and there may not be a tense swelling. The underlying testis is NOT palpated therefore indicating a hydrocele. Causes include trauma, infection and tumour.

Scrotal swelling

Differential diagnosis
Inguinal hernia If inguinoscrotal swelling; cannot ‘get above it’ on examination
Cough impulse may be present
May be reducible
Testicular tumours Often discrete testicular nodule (may have associated hydrocele)
Symptoms of metastatic disease may be present
USS scrotum and serum AFP and β HCG required
Acute epididymo-orchitis Often history of dysuria and urethral discharge
Swelling may be tender and eased by elevating testis
Most cases due to Chlamydia
Infections with other gram negative organisms may be associated with underlying structural abnormality
Epididymal cysts Single or multiple cysts
May contain clear or opalescent fluid (spermatoceles)
Usually occur over 40 years of age
Painless
Lie above and behind testis
It is usually possible to ‘get above the lump’ on examination
Hydrocele Non painful, soft fluctuant swelling
Often possible to ‘get above it’ on examination
Usually contain clear fluid
Will often transilluminate
May be presenting feature of testicular cancer in young men
Testicular torsion Severe, sudden onset testicular pain
Risk factors include abnormal testicular lie
Typically affects adolescents and young males
On examination testis is tender and pain not eased by elevation
Urgent surgery is indicated, the contra lateral testis should also be fixed
Varicocele Varicosities of the pampiniform plexus
Typically occur on left (because testicular vein drains into renal vein)
May be presenting feature of renal cell carcinoma
Affected testis may be smaller and bilateral varicoceles may affect fertility

Management
Testicular malignancy is always treated with orchidectomy via an inguinal approach. This allows high ligation of the testicular vessels and avoids exposure of another lymphatic field to the tumour.
Torsion is commonest in young teenagers and the history in older children can be difficult to elicit. Intermittent torsion is a recognised problem. The treatment is prompt surgical exploration and testicular fixation. This can be achieved using sutures or by placement of the testis in a Dartos pouch.
Varicoceles are usually managed conservatively. If there are concerns about testicular function of infertility then surgery or radiological management can be considered.
Epididymal cysts can be excised using a scrotal approach
Hydroceles are managed differently in children where the underlying pathology is a patent processus vaginalis and therefore an inguinal approach is used in children so that the processus can be ligated. In adults a scrotal approach is preferred and the hydrocele sac excised or plicated.

1314
Q

The following are true of carcinoid tumours except:

When present in the appendix tip and measure less than 2 cm have an excellent prognosis

Even when metastatic disease is present it tends to follow a protracted course

When present in the appendix body tend to present with carcinoid syndrome even when liver metastases are not present

May be imaged using somatostatin receptor scintigraphy scanning

Advanced appendiceal carcinoids may require right hemicolectomy

A

Rule of thirds:

1/3 multiple
1/3 small bowel
1/3 metastasize
1/3 second tumour
Liver metastases are necessary for the presence of carcinoid syndrome.

Carcinoid syndrome

Carcinoid tumours secrete serotonin
Originate in neuroendocrine cells mainly in the intestine (midgut-distal ileum/appendix)
Can occur in the rectum, bronchi
Hormonal symptoms mainly occur when disease spreads outside the bowel

Clinical features
Onset: insidious over many years
Flushing face
Palpitations
Pulmonary valve stenosis and tricuspid regurgitation causing dyspnoea
Asthma
Severe diarrhoea (secretory, persists despite fasting)

Investigation
5-HIAA in a 24-hour urine collection
Somatostatin receptor scintigraphy
CT scan
Blood testing for chromogranin A

Treatment
Octreotide
Surgical removal

1315
Q

Which of the following muscle relaxants is an agent that is degraded by hydrolysis and may produce histamine release?

Atracurium

Panciuronium

Curare

Suxamethonium

Vecuronium

A

Atracurium is degraded by a process of ester hydrolysis. This uses non specific plasma esterases.

Muscle relaxants

Suxamethonium
Depolarising neuromuscular blocker
Inhibits action of acetylcholine at the neuromuscular junction
Degraded by plasma cholinesterase and acetylcholinesterase (affected by lack of acetylcholinesterase)
Fastest onset and shortest duration of action of all muscle relaxants
Produces generalised muscular contraction prior to paralysis
Adverse effects include hyperkalaemia, malignant hyperthermia, delayed recovery
Atracurium
Non depolarising neuromuscular blocking drug
Duration of action usually 30-45 minutes
Generalised histamine release on administration may produce facial flushing, tachycardia and hypotension
Not excreted by liver or kidney, broken down in tissues by hydrolysis
Reversed by neostigmine
Vecuronium
Non depolarising neuromuscular blocking drug
Duration of action approximately 30 - 40 minutes
Degraded by liver and kidney and effects prolonged in organ dysfunction
Effects may be reversed by neostigmine
Pancuronium
Non depolarising neuromuscular blocker
Onset of action approximately 2-3 minutes
Duration of action up to 2 hours
Effects may be partially reversed with drugs such as neostigmine

1316
Q

Which of the following structures is not closely related to the piriformis muscle?

Superior gluteal nerve

Sciatic nerve

Inferior gluteal artery

Inferior gluteal nerve

Medial femoral circumflex artery

A

Nerve supply of lateral hip rotators

Piriformis: ventral rami S1, S2
Obturator internus: nerve to obturator internus
Superior gemellus: nerve to obturator internus
Inferior gemellus: nerve to quadratus femoris
Quadrator femoris: nerve to quadrator femoris
The piriformis muscle is an important anatomical landmark in the gluteal region. The following structures are closely related:
Sciatic nerve
Inferior gluteal artery and nerve
Superior gluteal artery and nerve

The medial femoral circumflex artery runs deep to quadratus femoris.

1317
Q

A 49-year-old male sustained a severe blunt injury just below the bridge of the nose with industrial machinery. Imaging demonstrates a fracture involving the superior orbital fissure. On examination an ipsilateral pupillary defect is present and loss of the corneal reflexes. In addition to these examination findings, which of the following will not be present?

Altered cutaneous sensation from the forehead to the vertex

Ptosis

Complete opthalmoplegia

Nystagmus

Ipsilateral visual loss

A

Orbital apex syndrome
This is an extension of superior orbital fissure syndrome and includes compression of the optic nerve passing through the optic foramen. It is indicated by features of superior orbital fissure syndrome and ipsilateral afferent pupillary defect.
This type of injury will result in the orbital apex syndrome (See above). As such opthalmoplegia will be present and nystagmus cannot occur.

Craniomaxillofacial injuries

Craniomaxillofacial injuries in the UK are due to:
Interpersonal violence (52%)
Motor vehicle accidents (16%)
Sporting injuries (19%)
Falls (11%)

Le Fort Fractures
Grade Feature
Le Fort 1 The fracture extends from the nasal septum to the lateral pyriform rims, travels horizontally above the teeth apices, crosses below the zygomaticomaxillary junction, and traverses the pterygomaxillary junction to interrupt the pterygoid plates.
Le Fort 2 These fractures have a pyramidal shape and extend from the nasal bridge at or below the nasofrontal suture through the frontal process of the maxilla, inferolaterally through the lacrimal bones and inferior orbital floor and rim through or near the inferior orbital foramen, and inferiorly through the anterior wall of the maxillary sinus; it then travels under the zygoma, across the pterygomaxillary fissure, and through the pterygoid plates.
Le Fort 3 These fractures start at the nasofrontal and frontomaxillary sutures and extend posteriorly along the medial wall of the orbit through the nasolacrimal groove and ethmoid bones. The thicker sphenoid bone posteriorly usually prevents continuation of the fracture into the optic canal. Instead, the fracture continues along the floor of the orbit along the inferior orbital fissure and continues superolaterally through the lateral orbital wall, through the zygomaticofrontal junction and the zygomatic arch. Intranasally, a branch of the fracture extends through the base of the perpendicular plate of the ethmoid, through the vomer, and through the interface of the pterygoid plates to the base of the sphenoid. This type of fracture predisposes the patient to CSF rhinorrhea more commonly than the other types.

Ocular injuries
Superior orbital fissure syndrome
Severe force to the lateral wall of the orbit resulting in compression of neurovascular structures. Results in :
Complete opthalmoplegia and ptosis (Cranial nerves 3, 4, 6 and nerve to levator palpebrae superioris)
Relative afferent pupillary defect
Dilatation of the pupil and loss of accommodation and corneal reflexes
Altered sensation from forehead to vertex (frontal branch of trigeminal nerve)

Orbital blow out fracture
Typically occurs when an object of slightly larger diameter than the orbital rim strikes the incompressible eyeball. The bone fragment is displaced downwards into the antral cavity, remaining attached to the orbital periosteum. Periorbital fat may be herniated through the defect, interfering with the inferior rectus and inferior oblique muscles which are contained within the same fascial sheath. This prevents upward movement and outward rotation of the eye and the patient experiences diplopia on upward gaze. The initial bruising and swelling may make assessment difficult and patients should usually be reviewed 5 days later. Residual defects may require orbital floor reconstruction.

Nasal Fractures
Common injury
Ensure new and not old deformity
Control epistaxis
CSF rhinorrhoea implies that the cribriform plate has been breached and antibiotics will be required.
Usually best to allow bruising and swelling to settle and then review patient clinically. Major persistent deformity requires fracture manipulation, best performed within 10 days of injury.

Retrobulbar haemorrhage
Rare but important ocular emergency. Presents with:
Pain (usually sharp and within the globe)
Proptosis
Pupil reactions are lost
Paralysis (eye movements lost)
Visual acuity is lost (colour vision is lost first)
May be the result of Le Fort type facial fractures.

Management:
Mannitol 1g/Kg as 20% infusion, Osmotic diuretic, Contra-indicated in congestive heart failure and pulmonary oedema
Acetazolamide 500mg IV, (Monitor FBC/U+E) Reduces aqueous pressure by inhibition of carbonic anhydrase (used in glaucoma)
Dexamethasone 8mg orally or intravenously
In a traumatic setting an urgent cantholysis may be needed prior to definitive surgery.

Consider
Papaverine 40mg smooth muscle relaxant
Dextran 40 500mls IV improves perfusion

1318
Q

A 20 year old lady has undergone a total thyroidectomy for a well differentiated papillary carcinoma. She attends clinic and is well and the surgeon wishes to screen for disease recurrence. Which of these blood tests is most likely to be helpful?

Calcitonin

Antibodies to TSH receptor

Antibodies to parafollicular c cells

Thyroid peroxidase antibodies

Thyroglobulin levels

A

Thyroglobulin, the major constituent of colloid and precursor of thyroid hormones may be elevated in those with metastatic or recurrent thyroid cancer. Results may be erroneous in those with other thyroid disorders.

Blood testing in thyroid disease

Assay Usage
Thyroid peroxidase (microsomal) antibodies
Found in autoimmune disease affecting the thyroid (Hashimotos 100%) and Graves (70%)
Antibodies to TSH receptor
Individuals with Graves disease (95%)
Thyroglobulin
Not useful for clinically distinguishing between different types of thyroid disease, may be used as part of thyroid cancer follow up
Calcitonin
Released from the parafollicular cells
Usually found in patients with medullary carcinoma of the thyroid

1319
Q

A 10 year old boy who has learning difficulties, is reported as having a difference in size between his two legs. What is the underlying disorder?

Neurofibromatosis Type I

Neurofibromatosis Type II

Kartagener’s syndrome

Multiple endocrine neoplasia type II

Multiple endocrine neoplasia type I

A

Neurofibromatosis type I. A hallmark finding is a plexiform neurofibroma, which is a sheet of neurofibromatosis tissue which encases major nerves. In children this attracts extra blood circulation, which can accelerate growth of the affected limb.
Other features include:
Schwannoma, > 6
Cafe au lait spots, axillary freckling, Lisch nodules, Optic glioma. Meningiomas, Glioma, or Schwannoma.

Genetics and surgical disease

Some of the more commonly occurring genetic conditions occurring in surgical patients are presented here.

Li-Fraumeni Syndrome
Autosomal dominant
Consists of germline mutations to p53 tumour suppressor gene
High incidence of malignancies particularly sarcomas and leukaemias
Diagnosed when:

*Individual develops sarcoma under 45 years
*First degree relative diagnosed with any cancer below age 45 years and another family member develops malignancy under 45 years or sarcoma at any age

BRCA 1 and 2
Carried on chromosome 17 (BRCA 1) and Chromosome 13 (BRCA 2)
Linked to developing breast cancer (60%) risk.
Associated risk of developing ovarian cancer (55% with BRCA 1 and 25% with BRCA 2).

Lynch Syndrome
Autosomal dominant
Develop colonic cancer and endometrial cancer at young age
80% of affected individuals will get colonic and/ or endometrial cancer
High risk individuals may be identified using the Amsterdam criteria

Amsterdam criteria
Three or more family members with a confirmed diagnosis of colorectal cancer, one of whom is a first degree (parent, child, sibling) relative of the other two.
Two successive affected generations.
One or more colon cancers diagnosed under age 50 years.
Familial adenomatous polyposis (FAP) has been excluded.

Gardners syndrome
Autosomal dominant familial colorectal polyposis
Multiple colonic polyps
Extra colonic diseases include: skull osteoma, thyroid cancer and epidermoid cysts
Desmoid tumours are seen in 15%
Mutation of APC gene located on chromosome 5
Due to colonic polyps most patients will undergo colectomy to reduce risk of colorectal cancer
Now considered a variant of familial adenomatous polyposis coli

1320
Q

A 4 year old boy is admitted to hospital with night sweats and is found to be generally unwell. As part of his investigations, a spinal MRI is scan is performed and discitis is suspected. Which of these infectious agents represents the most likely underlying pathogen?

Staphylococcus aureus

Brucella

Mycobacterium tuberculosis

Kigella kingae

Candida spp

A

In children aged 6 months to 4 years, Kigella is the most likely infecting organism. S Aureus affects smaller babies.

Discitis in children

In children, infectious discitis (D) and infectious spondylodiscitis (SD) are rare diseases that can cause significant clinical problems, including spinal deformities and segmental instabilities. Moreover, when the infection spreads into the spinal channel, D and SD can cause devastating neurologic complications. Early diagnosis and treatment may reduce these risks. Several different bacterial pathogens can cause D and SD, and, in some cases, particularly those due to Staphylococcus aureus, Kingella kingae, Mycobacterium tuberculosis, Brucella spp., the appropriate choice of drug is critical to achieve cure. In most cases the infections spreads to the site haematogenously from a separate site. Less commonly, it arises as a result of direct inoculation. A triphasic age distribution is described; the first peak incidence is in children aged only a few weeks or months old, the second in those between 6 months and the end of the preschool period, and the third in school-aged children.
Initial treatment is with broad spectrum intravenous antibiotics that cover s aureus, children aged 4 months to 6 years need cover for Kigella. More tailored therapy can be added later depending on progress and culture results.

1321
Q

Which of the following structures separates the ulnar artery from the median nerve?

Brachioradialis

Pronator teres

Tendon of biceps brachii

Flexor carpi ulnaris

Brachialis

A

It lies deep to pronator teres and this separates it from the median nerve.

Ulnar artery

Path
Starts: middle of antecubital fossa
Passes obliquely downward, reaching the ulnar side of the forearm at a point about midway between the elbow and the wrist. It follows the ulnar border to the wrist, crossing over the flexor retinaculum. It then divides into the superficial and deep volar arches.

Relations
Deep to- Pronator teres, Flexor carpi ulnaris, Palmaris longus
Lies on- Brachialis and Flexor digitorum profundus
Superficial to the flexor retinaculum at the wrist

The median nerve is in relation with the medial side of the artery for about 2.5 cm. And then crosses the vessel, being separated from it by the ulnar head of the Pronator teres

The ulnar nerve lies medially to the lower two-thirds of the artery

Branch
Anterior interosseous artery

1322
Q

A 22 year old man undergoes a superficial parotidectomy for a pleomorphic adenoma. The operation does not proceed well and a diathermy malfunction results in division of the buccal branch of the facial nerve. Which of the following muscles will not demonstrate impaired function as a result?

Zygomaticus minor

Mentalis

Buccinator

Levator anguli oris

Orbicularis oris

A

B-Mentalis

Buccal branch supplies
Zygomaticus minor Elevates upper lip
Buccinator Pulls corner of mouth backward and compresses cheek
Levator anguli oris Pulls angles of mouth upward and toward midline
Orbicularis Closes and tightens lips together
Nasalis Flares nostrils and compresses nostrils

The orbicularis oris has innervation by both the buccal and marginal mandibular branches of the facial nerve.

Facial nerve

The facial nerve is the main nerve supplying the structures of the second embryonic branchial arch. It is predominantly an efferent nerve to the muscles of facial expression, digastric muscle and also to many glandular structures. It contains a few afferent fibres which originate in the cells of its genicular ganglion and are concerned with taste.

Supply - ‘face, ear, taste, tear’
Face: muscles of facial expression
Ear: nerve to stapedius
Taste: supplies anterior two-thirds of tongue
Tear: parasympathetic fibres to lacrimal glands, also salivary glands

Path
Subarachnoid path
Origin: motor- pons, sensory- nervus intermedius
Pass through the petrous temporal bone into the internal auditory meatus with the vestibulocochlear nerve. Here they combine to become the facial nerve.

Facial canal path
The canal passes superior to the vestibule of the inner ear
At the medial aspect of the middle ear, it becomes wider and contains the geniculate ganglion.
- 3 branches:
1. greater petrosal nerve
2. nerve to stapedius
3. chorda tympani

Stylomastoid foramen
Passes through the stylomastoid foramen (tympanic cavity anterior and mastoid antrum posteriorly)
Posterior auricular nerve and branch to posterior belly of digastric and stylohyoid muscle

Face
Enters parotid gland and divides into 5 branches:
Temporal branch
Zygomatic branch
Buccal branch
Marginal mandibular branch
Cervical branch

1323
Q

Which of the structures listed below articulates with the head of the radius superiorly?

Capitulum

Trochlea

Lateral epicondyle

Ulna

Medial epicondyle

A

A useful revision aid : ‘Capital Radio’ is the articulation of the radial head superiorly.
The head of the radius articulates with the capitulum of the humerus.

Radius

The radius is one of the two long forearm bones that extends from the lateral side of the elbow to the thumb side of the wrist. It has two expanded ends, of which the distal end is the larger. Key points relating to its topography and relations are outlined below;

Upper end
Articular cartilage- covers medial > lateral side
Articulates with radial notch of the ulna by the annular ligament
Muscle attachment- biceps brachii at the tuberosity

Shaft
Muscle attachment
Upper third of the body Supinator
Flexor digitorum superficialis
Flexor pollicis longus
Middle third of the body Pronator teres
Lower quarter of the body Pronator quadratus
Tendon of supinator longus

Lower end
Quadrilateral
Anterior surface- capsule of wrist joint
Medial surface- head of ulna
Lateral surface- ends in the styloid process
Posterior surface: 3 grooves containing:
1. Tendons of extensor carpi radialis longus and brevis
2. Tendon of extensor pollicis longus
3. Tendon of extensor indicis

1324
Q

A 36 year old man develops cellulitis of the hand. This condition will involve the complement immunological system. Which of the cells types listed below provides the greatest contribution to complement protein and glycoprotein production?

T Cells

B Cells

Hepatocytes

Endothelial cells

Polymorphonucleocytes

A

Whilst all the cell types listed can produce complement proteins, the greatest contribution is made from the hepatocytes. Indeed, most soluble proteins are synthesized by the liver.

Complement system

The complement system plays a crucial role in the innate defense against common pathogens. Activation of complement leads to robust and efficient proteolytic cascades, which terminate in opsonization and lysis of the pathogen as well as in the generation of the classical inflammatory response through the production of potent proinflammatory molecules.

Proteolytic cascades include: potent proinflammatory mediators (anaphylatoxins), opsonization (coating) of the pathogenic surface through various complement opsonins (e.g., C3b), and targeted lysis of the pathogenic surface through the assembly of membrane-penetrating pores known as the membrane attack complex (MAC).

Complement is activated via one of three major pathways; classical, lectin and alternative.
The final output of the system is inflammation, lysis or opsonization.

1325
Q

Concerning proximal aortic dissection (Debakey types 1 and 2/ Stanford type A) which statement is false?

The intimal tear is typically >50% of the aortic circumference

It is usually treated using an endovascular approach

They have a 50% mortality in the first 2 days

Arch reconstructions may require deep hypothermic circulatory arrest

Target systolic pressure of around 110mmHg should be maintained

A

Usually open surgery is required for these lesions as customised grafts are not usually available for this type of repair yet.

Aortic dissection

More common than rupture of the abdominal aorta
33% of patients die within the first 24 hours, and 50% die within 48 hours if no treatment received
Associated with hypertension
Features of aortic dissection: tear in the intimal layer, followed by formation and propagation of a subintimal hematoma. Cystic medial necrosis (Marfan’s)
Most common site of dissection: 90% occurring within 10 centimetres of the aortic valve

Stanford Classification
Type Location Treatment
A Ascending aorta/ aortic root Surgery- aortic root replacement
B Descending aorta Medical therapy with antihypertensives

DeBakey classification
Type Site affected
I Ascending aorta, aortic arch, descending aorta
II Ascending aorta only
III Descending aorta distal to left subclavian artery

Clinical features
Tearing, sudden onset chest pain (painless 10%)
Hypertension or Hypotension
A blood pressure difference (in each arm) greater than 20 mm Hg
Neurologic deficits (20%)

Investigations
CXR: widened mediastinum, abnormal aortic knob, ring sign, deviation of the trachea/oesophagus
CT angiography of the thoracic aorta
MRI angiography
Conventional angiography (now rarely used diagnostically)

Management
Beta-blockers: aim HR 60-80 bpm and systolic BP 100-120 mm Hg
For type A dissections the standard of care is aortic root replacement

1326
Q

A 6 day old child is suspected of having a malrotation and requires urgent abdominal exploration. What is the most appropriate surgical approach?

Midline abdominal incision

Paramedian incision

Transverse supra umbilical abdominal incision

Transverse infra umbilical abdominal incision

Battle incision

A

In young children, laparotomy is performed via transverse supra umbilical incision. Access via midline incisions is very poor and they should not be used.

Abdominal incisions

Midline incision
Commonest approach to the abdomen
Structures divided: linea alba, transversalis fascia, extraperitoneal fat, peritoneum (avoid falciform ligament above the umbilicus)
Bladder can be accessed via an extraperitoneal approach through the space of Retzius
Paramedian incision
Parallel to the midline (about 3-4cm)
Structures divided/retracted: anterior rectus sheath, rectus (retracted), posterior rectus sheath, transversalis fascia, extraperitoneal fat, peritoneum
Incision is closed in layers
Battle
Similar location to paramedian but rectus displaced medially (and thus denervated)
Now seldom used
Kocher’s Incision under right subcostal margin e.g. Cholecystectomy (open)
Lanz Incision in right iliac fossa e.g. Appendicectomy
Gridiron Oblique incision centered over McBurneys point- usually appendicectomy (less cosmetically acceptable than Lanz
Gable Rooftop incision
Pfannenstiel’s Transverse supra pubic, primarily used to access pelvic organs
McEvedy’s Groin incision e.g. Emergency repair strangulated femoral hernia
Rutherford Morrison Extraperitoneal approach to left or right lower quadrants. Gives excellent access to iliac vessels and is the approach of choice for first time renal transplantation.

1327
Q

A 24 year old man falls and sustains a fracture through his scaphoid bone. From which of the following areas does the scaphoid derive the majority of its blood supply?

From its proximal medial border

From its proximal lateral border

From its proximal posterior surface

Superficial palmar arch vessels

Dorsal carpal branch vessels

A

The dorsal carpal branch vessels supply 80% of the scaphoid via retrograde flow. There is a minor supply from the superficial palmar arch vessels that supplies the distal 20% of the scaphoid.
Scaphoid bone

The scaphoid has a concave articular surface for the head of the capitate and at the edge of this is a crescentic surface for the corresponding area on the lunate.
Proximally, it has a wide convex articular surface with the radius. It has a distally sited tubercle that can be palpated. The remaining articular surface is to the lateral side of the tubercle. It faces laterally and is associated with the trapezium and trapezoid bones.

The narrow strip between the radial and trapezial surfaces and the tubercle gives rise to the radial collateral carpal ligament.

Blood supply
This is from the scaphoid tubercle vessels that comprise 20% and the dorsal ridge vessels that supply 80%.

1328
Q

Which of the muscles listed below would be affected if the radial nerve were injured in a humeral shaft fracture?

Supinator

Flexor digitorum profundus

Brachialis

Abductor pollicis brevis

Abductor digiti minimi

A

A-Supinator

Nerve lesions during surgery

A variety of different procedures carry the risk of iatrogenic nerve injury. These are important not only from the patients perspective but also from a medicolegal standpoint.

The following operations and their associated nerve lesions are listed here:
Posterior triangle lymph node biopsy and accessory nerve lesion.
Lloyd Davies stirrups and common peroneal nerve.
Thyroidectomy and laryngeal nerve.
Anterior resection of rectum and hypogastric autonomic nerves.
Axillary node clearance; long thoracic nerve, thoracodorsal nerve and intercostobrachial nerve.
Inguinal hernia surgery and ilioinguinal nerve.
Varicose vein surgery- sural and saphenous nerves.
Posterior approach to the hip and sciatic nerve.
Carotid endarterectomy and hypoglossal nerve.

There are many more, with sound anatomical understanding of the commonly performed procedures the incidence of nerve lesions can be minimised. They commonly occur when surgeons operate in an unfamiliar tissue plane or by blind placement of haemostats (not recommended).

1329
Q

A 48 year old lady with thyrotoxicosis is referred to the clinic, she was poorly controlled on carbimazole and has received orbital radiotherapy for severe proptosis. This has improved matters but she relapsed on stopping her carbimazole. What would be the best intervention to consider at this stage?

Administration of radio-iodine

Sub total thyroidectomy

External beam irradiation to the thyroid

Hemithyroidectomy

Total thyroidectomy

A

The recurrence of symptoms following medical therapy attracts a recommendation for definitive treatment. Since radio-iodine can worsen eye signs, this is not a wise choice. Surgery in the form of a total thyroidectomy would be curative.

Management of thyroid disease

Management of thyroid disease

Lesion Management
Large multinodular goitre Surgery for pressure symptoms. Total thyroidectomy is treatment of choice
Toxic nodule Hemithyroidectomy
Follicular lesion (THY 3f) Hemithyroidectomy to establish diagnosis
Papillary thyroid cancer Total thyroidectomy and central compartment nodal dissection (extended lymphadenectomy as required)
Follicular thyroid cancer Total thyroidectomy (usually completion as already had hemithyroidectomy)
Anaplastic thyroid cancer Palliative radiotherapy
Medullary thyroid cancer Total thyroidectomy and central compartment nodal dissection (screen for other MEN tumours)
Lymphoma of the thyroid Consider core biopsy
Persistent refilling cysts Injection sclerotherapy and surgery if this fails
Graves disease with significant eye signs Total thyroidectomy
Graves disease without significant eye signs Patient choice radioiodine Vs surgery

1330
Q

A 52 year old male presents with tearing central chest pain. On examination, he has an aortic regurgitation murmur. An ECG shows ST elevation in leads II, III and aVF. What is the likely explanation?

Distal aortic dissection

Anterior myocardial infarct

Inferior myocardial infarct

Proximal aortic dissection

Pulmonary embolism

A

An inferior myocardial infarction and AR murmur should raise suspicions of an ascending aorta dissection rather than an inferior myocardial infarction alone. Also the history is more suggestive of a dissection. Other features may include pericardial effusion, carotid dissection and absent subclavian pulse.

Chest pain

Aortic dissection
This occurs when there is a flap or filling defect within the aortic intima. Blood tracks into the medial layer and splits the tissues with the subsequent creation of a false lumen. It most commonly occurs in the ascending aorta or just distal to the left subclavian artery (less common). It is most common in Afro-carribean males aged 50-70 years.
Patients usually present with a tearing intrascapular pain, which may be similar to the pain of a myocardial infarct.
The dissection may spread either proximally or distally with subsequent disruption to the arterial branches that are encountered.
In the Stanford classification system the disease is classified into lesions with a proximal origin (Type A) and those that commence distal to the left subclavian (Type B).
Diagnosis may be suggested by a chest x-ray showing a widened mediastinum. Confirmation of the diagnosis is usually made by use of CT angiography
Proximal (Type A) lesions are usually treated surgically, type B lesions are usually managed non operatively.

Pulmonary embolism
Typically sudden onset of chest pain, haemoptysis, hypoxia and small pleural effusions may be present.
Most patients will have an underlying deep vein thrombosis
Diagnosis may be suggested by various ECG findings including S waves in lead I, Q waves in lead III and inverted T waves in lead III. Confirmation of the diagnosis is usually made through use of CT pulmonary angiography.
Treatment is with anticoagulation, in those patients who develop a cardiac arrest or severe compromise from their PE, consideration may be given to thrombolysis.

Myocardial infarction
Traditionally described as sudden onset of central, crushing chest pain. It may radiate into the neck and down the left arm. Signs of autonomic dysfunction may be present. The presenting features may be atypical in the elderly and those with diabetes.
Diagnosis is made through identification of new and usually dynamic ECG changes (and cardiac enzyme changes). Inferior and anterior infarcts may be distinguished by the presence of specific ECG changes (usually II, III and aVF for inferior, leads V1-V5 for anterior).
Treatment is with oral antiplatelet agents, primary coronary angioplasty and/ or thrombolysis.

Perforated peptic ulcer
Patients usually develop sudden onset of epigastric abdominal pain, it may be soon followed by generalised abdominal pain.
There may be features of antecendant abdominal discomfort, the pain of gastric ulcer is typically worse immediately after eating.
Diagnosis may be made by erect chest x-ray which may show a small amount of free intra-abdominal air (very large amounts of air are more typically associated with colonic perforation).
Treatment is usually with a laparotomy, small defects may be excised and overlaid with an omental patch, larger defects are best managed with a partial gastrectomy.

Boerhaaves syndrome
Spontaneous rupture of the oesophagus that occurs as a result of repeated episodes of vomiting.
The rupture is usually distally sited and on the left side.
Patients usually give a history of sudden onset of severe chest pain that may complicate severe vomiting.
Severe sepsis occurs secondary to mediastinitis.
Diagnosis is CT contrast swallow.
Treatment is with thoracotomy and lavage, if less than 12 hours after onset then primary repair is usually feasible, surgery delayed beyond 12 hours is best managed by insertion of a T tube to create a controlled fistula between oesophagus and skin.
Delays beyond 24 hours are associated with a very high mortality rate.

1331
Q

A 22 year old man suffers a blunt head injury. He is drowsy and has a GCS of 7 on admission. Which of the following is the major determinant of cerebral blood flow in this situation?

Systemic blood pressure

Mean arterial pressure

Intra cranial pressure

Hypoxaemia

Acidosis

A

Hypoxaemia and acidosis may both affect cerebral blood flow. However, in the traumatic situation increases in intracranial pressure are far more likely to occur especially when GCS is low. This will adversely affect cerebral blood flow.

Cerebral blood flow

  • CNS autoregulates its own blood supply
    Factors affecting the cerebral pressure include; systemic carbon dioxide levels, CNS metabolism, CNS trauma, CNS pressure
    The PaCO2 is the most potent mediator
    Acidosis and hypoxaemia will increase cerebral blood flow but to a lesser degree
    Intra cranial pressure may increase in patients with head injuries and this can result in impaired blood flow
    Intra cerebral pressure is governed by Monroe-Kelly Doctrine which considers skull as closed box, changes in pressure are offset by loss of CSF. When this is no longer possible ICP rises
1332
Q

A 66 year old lady presents with pain in her right hip. It has been increasing over the previous three weeks and waking her from sleep. On examination, she is tender on internal rotation. Blood tests reveal a markedly elevated serum calcium and alkaline phosphatase levels. What is the likely cause?

Metastatic breast cancer

Chondrosarcoma

Osteoporosis

Pagets disease

Rickets

A

Increasing pain at rest, together with increased serum calcium and alkaline phosphatase are most likely to represent metastatic tumour to bone. Chondrosarcomas do occur in the pelvis but are not associated with increased serum calcium and typically have a longer history.

Bone disease

Disease Features Treatment
Pagets
Focal bone resorption followed by excessive and chaotic bone deposition
Affects (in order): spine, skull, pelvis and femur
Serum alkaline phosphatase raised (other parameters normal)
Abnormal thickened, sclerotic bone on x-rays
Risk of cardiac failure with >15% bony involvement
Small risk of sarcomatous change
Bisphosphonates
Osteoporosis
Excessive bone resorption resulting in demineralised bone
Commoner in old age
Increased risk of pathological fracture, otherwise asymptomatic
Alkaline phosphatase normal, calcium normal
Bisphosphonates, calcium and vitamin D
Secondary bone tumours
Bone destruction and tumour infiltration
Mirel scoring used to predict risk of fracture
Appearances depend on primary (e.g.sclerotic - prostate, lytic - breast)
Elevated serum calcium and alkaline phosphatase may be seen
Radiotherapy, prophylactic fixation and analgesia

1333
Q

In patients with multiple endocrine neoplasia type IIb which of the following clinical appearances is the patient most likely to display?

Acromegalic facies

Turners type features

Profound kyphoscoliosis

Multiple bony exostoses

Marfanoid features

A

Patients with MEN IIb may display Marfanoid features. It is unclear at the present time whether they have discrete changes in the microfibrils of elastic fibres that are present in Marfans.

Multiple Endocrine Neoplasia

Multiple endocrine neoplasia (MEN) is inherited as an autosomal dominant disorder.

The table below summarises the three main types of MEN:

MEN type I MEN type IIa MEN type IIb
Mnemonic ‘three P’s’:

Parathyroid (95%): Parathyroid adenoma
Pituitary (70%): Prolactinoma/ACTH/Growth Hormone secreting adenoma
Pancreas (50%): Islet cell tumours/Zollinger Ellison syndrome

also: Adrenal (adenoma) and thyroid (adenoma) Phaeochromocytoma
Medullary thyroid cancer (70%)
Hyperparathyroidism (60%) Same as MEN IIa with addition of:
Marfanoid body habitus
Mucosal neuromas
MENIN gene (chromosome 11)

Most common presentation = hypercalcaemia RET oncogene (chromosome 10) RET oncogene (chromosome 10)

1334
Q

A 68 year old man with type 2 diabetes is admitted to hospital unwell. On examination, he has features of septic shock and right upper quadrant tenderness. He is not jaundiced. Imaging shows a normal calibre bile duct and no stones in the gallbladder. What is the most likely diagnosis?

Acute calculous cholecystitis

Acute acalculous cholecystitis

Cholangitis

Mirizzi syndrome

Sphincter of oddi dysfunction

A

This is the classic description of acalculous cholecystitis and its commonest in patients with type 2 diabetes. If you answered it incorrectly, ensure that you were not caught out by the acute calculous cholecystitis as this is a common exam mistake if options are mis read in a rush.

Gallstones

Up to 24% of women and 12% of men may have gallstones. Of these up to 30% may develop local infection and cholecystitis. In patients subjected to surgery 12% will have stones contained within the common bile duct. The majority of gallstones are of a mixed composition (50%) with pure cholesterol stones accounting for 20% of cases.
The aetiology of CBD stones differs in the world, in the West most CBD stones are the result of migration. In the East a far higher proportion arise in the CBD de novo.
The classical symptoms are of colicky right upper quadrant pain that occurs post prandially. The symptoms are usually worst following a fatty meal when cholecystokinin levels are highest and gallbladder contraction is maximal.

Investigation
In almost all suspected cases the standard diagnostic work up consists of abdominal ultrasound and liver function tests. Of patients who have stones within the bile duct, 60% will have at least one abnormal result on LFT’s. Ultrasound is an important test, but is operator dependent and therefore may occasionally need to be repeated if a negative result is at odds with the clinical picture. Where stones are suspected in the bile duct, the options lie between magnetic resonance cholangiography and intraoperative imaging. The choice between these two options is determined by the skills and experience of the surgeon. The advantages of intra operative imaging are less useful in making therapeutic decisions if the operator is unhappy about proceeding the bile duct exploration, and in such circumstances pre operative MRCP is probably a better option.

Specific gallstone and gallbladder related disease
Disease Features Management
Biliary colic Colicky abdominal pain, worse post prandially, worse after fatty foods If imaging shows gallstones and history compatible then laparoscopic cholecystectomy
Acute cholecystitis
Right upper quadrant pain
Fever
Murphys sign on examination
Occasionally mildly deranged LFT’s (especially if Mirizzi syndrome)
Imaging (USS) and cholecystectomy (ideally within 48 hours of presentation) (2)
Gallbladder abscess
Usually prodromal illness and right upper quadrant pain
Swinging pyrexia
Patient may be systemically unwell
Generalised peritonism not present
Imaging with USS +/- CT Scanning
Ideally surgery, sub total cholecystectomy may be needed if Calots triangle is hostile
In unfit patients percutaneous drainage may be considered
Cholangitis
Patient severely septic and unwell
Jaundice
Right upper quadrant pain
Fluid resuscitation
Broad spectrum intravenous antibiotics
Correct any coagulopathy
Early ERCP
Gallstone ileus
Patients may have a history of previous cholecystitis and known gallstones
Small bowel obstruction (may be intermittent)
Laparotomy and removal of gallstone from small bowel, the enterotomy must be made proximal to the site of obstruction and not at the site of obstruction. The fistula between the gallbladder and duodenum should not be interfered with.
Acalculous cholecystitis
Patients with inter current illness (e.g. diabetes, organ failure)
Patient of systemically unwell
Gallbladder inflammation in absence of stones
High fever
If patient fit then cholecystectomy, if unfit then percutaneous cholecystostomy

Treatment
Patients with asymptomatic gallstones rarely develop symptoms related to them (less than 2% per year) and may therefore be managed expectantly. In almost all cases of symptomatic gallstones the treatment of choice is cholecystectomy performed via the laparoscopic route. In the very frail patient there is sometimes a role for selective use of ultrasound guided cholecystostomy.

During the course of the procedure some surgeons will routinely perform either intra operative cholangiography or laparoscopic USS to either confirm anatomy or to exclude CBD stones. The latter may be more easily achieved by use of laparoscopic ultrasound. If stones are found then the options lie between early ERCP in the day or so following surgery or immediate surgical exploration of the bile duct. When performed via the trans cystic route this adds little in the way of morbidity and certainly results in faster recovery. Where transcystic exploration fails the alternative strategy is that of formal choledochotomy. The exploration of a small duct is challenging and ducts of less than 8mm should not be explored. Small stones that measure less than 5mm may be safely left and most will pass spontaneously.

Risks of ERCP(1)
Bleeding 0.9% (rises to 1.5% if sphincterotomy performed)
Duodenal perforation 0.4%
Cholangitis 1.1%
Pancreatitis 1.5%

References
1. Williams E et al. Guidelines on the management of common bile duct stones (CBDS)Gut 2008;57:10041021

  1. Gurusamy KS, Samraj K. Early versus delayed laparoscopic cholecystectomy for acute cholecystitis. Cochrane Database Syst Rev. 2006 Oct 18;(4):CD005440.
  2. Gurusamy K and Davidson B. Gallstones. BMJ 2014 (348):27-30.
1335
Q

A 78 year old man has undergone a hemi-arthroplasty for an intracapsular hip fracture. Post operatively he develops electrolyte derangement and receives intravenous fluids. Over the next 24 hours he develops marked abdominal distension. On examination, he has a tense, tympanic abdomen which is not painful. A contrast enema shows flow of contrast through to the caecum and through the ileocaecal valve. What is the most likely cause?

Ogilvies syndrome

Diverticular stricture

Malignant stricture

Volvulus

Adhesive obstruction

A

Patients with electrolyte disturbance and previous surgery may develop colonic pseudo-obstruction (Ogilvies syndrome). The diagnosis is made using a contrast enema and treatment is usually directed at the underlying cause with colonic decompression if indicated.

Colonic obstruction

Cause Features Treatment
Cancer
Usually insidious onset
History of progressive constipation
Systemic features (e.g. anaemia)
Abdominal distension
Absence of bowel gas distal to site of obstruction
Establish diagnosis (e.g. contrast enema/ endoscopy)
Laparotomy and resection, stenting, defunctioning colostomy or bypass
Diverticular stricture
Usually history of previous acute diverticulitis
Long history of altered bowel habit
Evidence of diverticulosis on imaging or endoscopy
Once diagnosis established, usually surgical resection
Colonic stenting should not be performed for benign disease
Volvulus
Twisting of bowel around its mesentery
Sigmoid colon affected in 76% cases
Patients usually present with abdominal pain, bloating and constipation
Examination usually shows asymmetrical distension
Plain X-rays usually show massively dilated sigmoid colon, loss of haustra and U shape are typical, the loop may contain fluid levels
Initial treatment is to untwist the loop, a flexible sigmoidoscopy may be needed
Those with clinical evidence of ischaemia should undergo surgery
Patient with recurrent volvulus should undergo resection
Acute colonic pseudo-obstruction
Symptoms and signs of large bowel obstruction with no lesion
Usually associated with metabolic disorders
Usually a cut off in the left colon (82% cases)
Although abdomen tense and distended, it is usually not painful
All patients should undergo contrast enema (may be therapeutic)
Colonoscopic decompression
Correct metabolic disorders
IV neostigmine
Surgery

1336
Q

A 73 year old man develops sudden onset abdominal pain and collapses. On examination he has a tender pulsatile mass in his upper abdomen. He has a blood pressure of 90/60mmHg and pulse rate of 105 beats per minute. Which of the following intravenous fluid regimens is most appropriate, whilst waiting for operative repair?

1 Litre of pentastarch over 15 minutes

1 litre of Hartmans solution over 4 hours

1 litre of gelofusin over 30 minutes

1 litre of Hartmans solution over 30 minutes

1 litre of blood over 15 minutes

A

This man will have a contained haematoma and is awaiting surgery. Rapid, high volume infusions may cause this to dislodge with disastrous consequences. In emergency aortic surgery a policy of permissive hypotension is utilised to reduce the risk.

Abdominal aorta aneurysm

  • Abdominal aortic aneurysms are a common problem in vascular surgery.
    They may occur as either true or false aneurysm. With the former all 3 layers of the arterial wall are involved, in the latter only a single layer of fibrous tissue forms the aneurysm wall.
    True abdominal aortic aneurysms have an approximate incidence of 0.06 per 1000 people. They are commonest in elderly men and for this reason the UK is now introducing the aneurysm screening program with the aim of performing an abdominal aortic ultrasound measurement in all men aged 65 years.

Causes
Several different groups of patients suffer from aneurysmal disease.
The commonest group is those who suffer from standard arterial disease, i.e. Those who are hypertensive and have been or are smokers.
Other patients such as those suffering from connective tissue diseases such as Marfan’s may also develop aneurysms. In patients with abdominal aortic aneurysms the extracellular matrix becomes disrupted with a change in the balance of collagen and elastic fibres.

Management
Most abdominal aortic aneurysms are an incidental finding.
Symptoms most often relate to rupture or impending rupture.
20% rupture anteriorly into the peritoneal cavity. Very poor prognosis.
80% rupture posteriorly into the retroperitoneal space
The risk of rupture is related to aneurysm size, only 2% of aneurysms measuring less than 4cm in diameter will rupture over a 5 year period. This contrasts with 75% of aneurysms measuring over 7cm in diameter.
This is well explained by Laplaces’ law which relates size to transmural pressure.
For this reason most vascular surgeons will subject patients with an aneurysm size of 5cm or greater to CT scanning of the chest, abdomen and pelvis with the aim of delineating anatomy and planning treatment. Depending upon co-morbidities, surgery is generally offered once the aneurysm is between 5.5cm and 6cm.

Indications for surgery
Symptomatic aneurysms (80% annual mortality if untreated)
Increasing size above 5.5cm if asymptomatic
Rupture (100% mortality without surgery)

Surgical procedures
Abdominal aortic aneurysm repair

Procedure:

GA
Invasive monitoring (A-line, CVP, catheter)
Incision: Midline or transverse
Bowel and distal duodenum mobilised to access aorta.
Aneurysm neck and base dissected out and prepared for cross clamp
Systemic heparinisation
Cross clamp (proximal first)
Longitudinal aortotomy
Atherectomy
Deal with back bleeding from lumbar vessels and inferior mesenteric artery
Insert graft either tube or bifurcated depending upon anatomy
Suture using Prolene (3/0 for proximal , distal anastomosis suture varies according to site)
Clamps off: End tidal CO2 will rise owing to effects of reperfusion, at this point major risk of myocardial events.
Haemostasis
Closure of aneurysm sac to minimise risk of aorto-enteric fistula
Closure: Loop 1 PDS or Prolene to abdominal wall
Skin- surgeons preference

Post operatively:

ITU (Almost all)
Greatest risk of complications following emergency repair
Complications: Embolic- gut and foot infarcts
Cardiac - owing to premorbid states, re-perfusion injury and effects of cross clamp
Wound problems
Later risks related to graft- infection and aorto-enteric fistula

Special groups

Supra renal AAA
These patients will require a supra renal clamp and this carries a far higher risk of complications and risk of renal failure.

Ruptured AAA
Pre-operatively the management depends upon haemodynamic instability. In patients with symptoms of rupture (typical pain, haemodynamic compromise and risk factors) then ideally prompt laparotomy. In those with vague symptoms and haemodynamic stability the ideal test is CT scan to determine whether rupture has occurred or not. Most common rupture site is retroperitoneal 80%. These patients will tend to develop retroperitoneal haematoma. This can be disrupted if Bp is allowed to rise too high so aim for Bp 100mmHg.
Operative details are similar to elective repair although surgery should be swift, blind rushing often makes the situation worse. Plunging vascular clamps blindly into a pool of blood at the aneurysm neck carries the risk of injury the vena cava that these patients do not withstand. Occasionally a supracoeliac clamp is needed to effect temporary control, although leaving this applied for more than 20 minutes tends to carry a dismal outcome.

EVAR
Increasingly patients are now being offered endovascular aortic aneurysm repair. This is undertaken by surgeons and radiologists working jointly. The morphology of the aneurysm is important and not all are suitable. Here is a typical list of those features favoring a suitable aneurysm:
Long neck
Straight iliac vessels
Healthy groin vessels

Clearly few AAA patients possess the above and compromise has to be made. The use of fenestrated grafts can allow supra renal AAA to be treated.

Procedure:

GA
Radiology or theatre
Bilateral groin incisions
Common femoral artery dissected out
Heparinisation
Arteriotomy and insertion of guide wire
Dilation of arteriotomy
Insertion of EVAR Device
Once in satisfactory position it is released
Arteriotomy closed once check angiogram shows good position and no endoleak

Complications:
Endoleaks depending upon site are either Type I or 2. These may necessitate re-intervention and all EVAR patients require follow up . Details are not needed for MRCS.

References
A reasonable review is provided by:
Sakalihasan N, Limet R, Defawe O. Abdominal aortic aneurysm. Lancet 2005 (365):1577- 1589

1337
Q

A 1 year old child has been unwell with a sore throat and fever for several days. He progresses to develop periumbilical abdominal discomfort and passes diarrhoea. The paediatricians call you because the ultrasound has shown a ‘target sign’ on the right side of the abdomen. What is the best initial course of action?

Obtain intravenous access, administer fluids and antibiotics

Undertake urgent fluoroscopic reduction

Undertake urgent hydrostatic reduction

Undertake a colonoscopy

Undertake a laparotomy

A

Always ensure that children with intussusception are resuscitated first. Administration of antibiotics is also important. This should precede any intervention.

Intussusception- Paediatric

Intussusception typically presents with colicky abdominal pain and vomiting. The telescoping of the bowel produces mucosal ischaemia and bleeding may occur resulting in the passage of red current jelly stools. Recognised causes include lumenal pathologies such as polyps, lymphadenopathy and diseases such as cystic fibrosis. Idiopathic intussceception of the ileocaecal valve and terminal ileum is the most common variant and typically affects young children and toddlers.

The diagnosis is usually made by abdominal ultrasound investigation. The decision as to the optimal treatment is dictated by the patients physiological status and abdominal signs. In general, children who are unstable with localising peritoneal signs should undergo laparotomy, as should those in whom attempted radiological reduction has failed.

In relatively well children, without localising signs, attempted pneumatic reduction under fluroscopic guidance is the usual treatment. This is not done where there are concerns about impending perforation or if there is a small bowel intussusception.

1338
Q

A 42 year old skier falls and impacts his hand on his ski pole. On examination, he is tender in the anatomical snuffbox and on bimanual palpation. X-rays with scaphoid views show no evidence of fracture. What is the most appropriate course of action?

Admission and surgical debridement

Application of tubigrip bandage and fracture clinic review

Application of futura splint and fracture clinic review

Admission for open reduction and fixation

Discharge with reassurance

A

A fracture may still be present and should be immobilised until repeat imaging can be performed. If clinical suspicion persists then subsequent imaging should be with MRI scanning or CT if MRI is contra-indicated.

Scaphoid fractures

  • Incidence of scaphoid fractures in UK ranges from 12.4 per 100,000 to 29 per 100,000
    Surface of scaphoid is covered by articular cartilage with small area available for blood vessels (fracture risks blood supply)
    Forms floor of anatomical snuffbox
    Risk of fracture associated with fall onto outstretched hand (tubercle, waist, or proximal third)
    A series of 4 scaphoid radiographs should be undertaken (PA, pronated oblique, Ziter view and lateral view). The Ziter view is a PA view with the wrist in ulnar deviation and beam angulated at 20 degrees
    Sensitivity of scaphoid radiographs in 1st week of injury is 80%
    Immobilization of scaphoid fractures difficult
    Repeat imaging should be done at 10 days. MRI should be done in cases of diagnostic uncertainty

Classification of scaphoid fractures
Scaphoid tubercle
Distal pole
Waist
Proximal pole

Management
Undisplaced fractures of the waist of the scaphoid and most distal pole fractures can be managed in a cast for 6 weeks with high rates of union.
Displaced scaphoid waist fractures (more than 1-2mm) should be viewed as unstable and surgically fixed.
All proximal pole fractures should be fixed surgically.

Complications
Non union of scaphoid
Avascular necrosis of the scaphoid
Scapholunate disruption and wrist collapse
Degenerative changes of the adjacent joint

Reference
Berber O et al. Fractures of the scaphoid. BMJ 2020 (369): 414-416.

1339
Q

A 35 year old Singaporean female attends a varicose vein pre operative clinic. On auscultation, a mid diastolic murmur is noted at the apex. The murmur is enhanced when the patient lies in the left lateral position. What is the most likely underlying lesion?

Pulmonary valve stenosis

Aortic valve stenosis

Aortic sclerosis

Mitral valve stenosis

Tricuspid regurgitation

A

A mid diastolic murmur at the apex is a classical description of a mitral stenosis murmur. The most common cause is rheumatic heart disease. Complications of mitral stenosis include atrial fibrillation, stroke, myocardial infarction and infective endocarditis.

Cardiac murmurs

Type of Murmur Conditions
Ejection systolic Aortic stenosis
Pulmonary stenosis, HOCM
ASD, Fallot’s
Pan-systolic Mitral regurgitation
Tricuspid regurgitation
VSD
Late systolic Mitral valve prolapse
Coarctation of aorta
Early diastolic Aortic regurgitation
Graham-Steel murmur (pulmonary regurgitation)
Mid diastolic Mitral stenosis
Austin-Flint murmur (severe aortic regurgitation)

1340
Q

A 27 year old man sustains a single gunshot wound to the left thigh. In the emergency department, he is noted to have a large haematoma of his medial thigh. He complains of parasthesia in his foot. On examination, there are weak pulses palpable distal to the injury and the patient is unable to move his foot. The appropriate initial management of this patient is:

Conventional angiography

Immediate exploration and repair

Fasciotomy of the anterior compartment

Observation for resolution of spasm

Local wound exploration

A

B- Immediate exploration and repair

The five P’s of arterial injury include pain, parasthesias, pallor, pulselessness and paralysis. In the extremities, the tissues most sensitive to anoxia are the peripheral nerves and striated muscle. The early developments of paresthesias and paralysis are signals that there is significant ischemia present, and immediate exploration and repair are warranted. The presence of palpable pulse does not exclude an arterial injury because this presence may represent a transmitted pulsation through a blood clot. When severe ischemia is present, the repair must be completed within 6 to 8 h to prevent irreversible muscle ischemia and loss of limb function. Delay to obtain a conventional angiogram or to observe for change needlessly prolongs the ischemic time. A CT angiogram may be a reasonable alternative. Fasciotomy may be required but should be done in conjunction with and after re-establishment of arterial flow. Local wound exploration is not recommended because brisk hemorrhage may be encountered without the securing of prior vascular control.

Vascular trauma

Peripheral and central vessels may be injured by blunt, penetrating or shearing injuries. Fractures of bones close to vessels may also be associated with vascular injury or vessel occlusion.

Assessment
Check for signs of distal perfusion
Doppler signal distally (monophasic/ biphasic or triphasic)
Anatomical location (which vessel is likely to be involved)
Duplex scanning and angiography are ‘gold standard’ tests but may not be immediately available in the trauma setting

Management
Almost always operative.
Obtaining proximal and distal control of affected vessels is crucial.
Simple lacerations of arteries may be directly closed, or a vein patch applied if there is a risk of subsequent stenosis.
Transection of the vessel should be treated by either end to end anastomosis (often not possible) or an interposition vein graft.
Use of PTFE in traumatic open injuries will invariably result in infection.

1341
Q

What is the lymphatic drainage of the ovaries?

Internal iliac nodes

Common iliac nodes

Para-aortic nodes

Para uterine nodes

Inguinal nodes

A

The lymphatic drainage of the ovary follows the gonadal vessels and drainage is therefore to the para-aortic nodes.

Lymphatic drainage of the ovaries, uterus and cervix

The ovaries drain to the para-aortic lymphatics via the gonadal vessels.
The uterine fundus has a lymphatic drainage that runs with the ovarian vessels and may thus drain to the para-aortic nodes. Some drainage may also pass along the round ligament to the inguinal nodes.
The body of the uterus drains through lymphatics contained within the broad ligament to the iliac lymph nodes.
The cervix drains into three potential nodal stations; laterally through the broad ligament to the external iliac nodes, along the lymphatics of the uterosacral fold to the presacral nodes and posterolaterally along lymphatics lying alongside the uterine vessels to the internal iliac nodes.

1342
Q

A man develops xerostomia. What proportion of salivary gland secretions is contributed by the parotid gland?

25%

50%

75%

80%

5%

A

Parotid gland secretions account for around 25% of saliva. Most comes from the submandibular gland.

Parotid gland secretions

  • Parotid gland secretions account for around 25% of salivary secretions
    Levels of sodium and chloride are lower than plasma, potassium and bicarbonate levels are higher
    Parotid secretions are watery and have high enzyme concentrations
    Parasympathetic stimulation produces a water rich, serous saliva. Sympathetic stimulation leads to the production of a low volume, enzyme-rich saliva.
    Fluid that is secreted within the acini is isotonic with plasma and undergoes modification within the ducts via process of ion exchange
    The ducts are affected by aldosterone and so will retain sodium and secrete potassium when aldosterone levels are elevated
1343
Q

A 48 year old woman suffers blunt trauma to the head and develops respiratory compromise. As a result she develops hypercapnia. Which of the following effects is most likely to ensue?

Cerebral vasoconstriction

Cerebral vasodilation

Cerebral blood flow will remain unchanged

Shunting of blood to peripheral tissues will occur in preference to CNS perfusion

None of the above

A

Hypercapnia will tend to produce cerebral vasodilation. This is of considerable importance in patients with cranial trauma as it may increase intracranial pressure.

Applied neurophysiology

Pressure within the cranium is governed by the Monroe-Kelly doctrine. This considers the skull as a closed box. Increases in mass can be accommodated by loss of CSF. Once a critical point is reached (usually 100- 120ml of CSF lost) there can be no further compensation and ICP rises sharply. The next step is that pressure will begin to equate with MAP and neuronal death will occur. Herniation will also accompany this process.
The CNS can autoregulate its own blood supply. Vaso constriction and dilatation of the cerebral blood vessels is the primary method by which this occurs. Extremes of blood pressure can exceed this capacity resulting in risk of stroke. Other metabolic factors such as hypercapnia will also cause vasodilation, which is of importance in ventilating head injured patients.
The brain can only metabolise glucose, when glucose levels fall, consciousness will be impaired.

1344
Q

Which structure is least likely to be found at the level of the sternal angle?

Left brachiocephalic vein

Intervertebral discs T4-T5

Start of aortic arch

2nd pair of costal cartilages

Bifurcation of the trachea into left and right bronchi

A

The left brachiocephalic vein lies posterior to the manubrium, at the level of its upper border. The sternal angle refers to the transition between manubrium and sternum and therefore will not include the left brachiocephalic vein.

Sternal angle

Anatomical structures at the level of the manubrium and upper sternum
Upper part of the manubrium
Left brachiocephalic vein
Brachiocephalic artery
Left common carotid
Left subclavian artery
Lower part of the manubrium/ manubrio-sternal angle
Costal cartilages of the 2nd ribs
Transition point between superior and inferior mediastinum
Arch of the aorta
Tracheal bifurcation
Union of the azygos vein and superior vena cava
The thoracic duct crosses to the midline

1345
Q

A 19 year old female is admitted with suspected meningitis. The House Officer is due to perform a lumbar puncture. What is the most likely structure first encountered when the needle is inserted?

Ligamentum flavum

Denticulate ligament

Dural sheath

Pia Mater

Supraspinous ligament

A

E-Supraspinous ligament

Lumbar puncture

Lumbar punctures are performed to obtain cerebrospinal fluid. In adults, the procedure is best performed at the level of L3/L4 or L4/5 interspace. These regions are below the termination of the spinal cord at L1.

During the procedure the needle passes through:
The supraspinous ligament which connects the tips of spinous processes and the interspinous ligaments between adjacent borders of spinous processes
Then the needle passes through the ligamentum flavum, which may cause a give as it is penetrated
A second give represents penetration of the needle through the dura mater into the subarachnoid space. Clear CSF should be obtained at this point

References
Boon et al Lumbar Puncture: Anatomical Review of a Clinical Skill. Clinical Anatomy 17:544553 (2004)

1346
Q

A 20 year old lady is referred to the vascular clinic. She has been feeling generally unwell for the past six weeks. She works as a typist and has noticed increasing pain in her forearms whilst working. On examination, she has absent upper limb pulses. Her ESR is measured and mildly elevated. What is the most likely diagnosis?

Polyarteritis nodosa

Granulomatosis with polyangiitis

Giant cell arteritis

Takayasu’s arteritis

Buergers disease

A

Takayasus arteritis may be divided into acute systemic phases and the chronic pulseless phase. In the latter part of the disease process the patient may complain of symptoms such as upper limb claudication. In the later stages of the condition the vessels will typically show changes of intimal proliferation, together with band fibrosis of the intima and media.

Vasculitis

The vasculitides are a group of conditions characterised by inflammation of the blood vessel walls. This may, in turn, compromise vessel integrity. Constitutional symptoms may be present. Whilst certain disease subtypes are reported to affect specific vessels, there is often a degree of overlap clinically.

Vessel diameter and vasculitis classification
Aorta and branches
Takayasu’s arteritis
Buergers disease
Giant cell arteritis
Large and medium sized arteries
Buergers disease
Giant cell arteritis
Polyarteritis nodosa
Medium sized muscular arteries
Polyarteritis nodosa
Wegeners granulomatosis
Small muscular arteries
Granulomatosis with polyangiitis
Rheumatoid vasculitis

Specific conditions
Takyasu’s arteritis
Inflammatory, obliterative arteritis affecting aorta and branches
Females> Males
Symptoms may include upper limb claudication
Clinical findings include diminished or absent pulses
ESR often affected during the acute phase
Buergers disease
Segmental thrombotic occlusions of the small and medium sized lower limb vessels
Commonest in young male smokers
Proximal pulses usually present, but pedal pulses are lost
An acuter hypercellular occlusive thrombus is often present
Tortuous corkscrew shaped collateral vessels may be seen on angiography
Giant cell arteritis
Systemic granulomatous arteritis that usually affects large and medium sized vessels
Females > Males
Temporal arteritis is commonest type
Granulomatous lesions may be seen on biopsy (although up to 50% are normal)
Polyarteritis nodosa
Systemic necrotising vasculitis affecting small and medium sized muscular arteries
Most common in populations with high prevalence of hepatitis B
Renal disease is seen in 70% cases
Angiography may show saccular or fusiform aneurysms and arterial stenoses
Granulomatosis with polyangiitis
Predominantly affects small and medium sized arteries
Systemic necrotising granulomatous vasculitis
Cutaneous vascular lesions may be seen (ulceration, nodules and purpura)
Sinus imaging may show mucosal thickening and air fluid levels

Treatment
Conditions such as Buergers disease are markedly helped by smoking cessation. Immunosupression is the main treatment for vasculitides.

1347
Q

Which of the following is not a feature of normal cerebrospinal fluid?

It has a pressure of between 10 and 15 mmHg.

It usually contains a small amount of glucose.

It may normally contain up to 5 red blood cells per mm

It may normally contain up to 3 white blood cells per mm

None of the above

A

It should not contain red blood cells.

Cerebrospinal fluid

The CSF fills the space between the arachnoid mater and pia mater (covering surface of the brain). The total volume of CSF in the brain is approximately 150ml. Approximately 500 ml is produced by the ependymal cells in the choroid plexus (70%), or blood vessels (30%). It is reabsorbed via the arachnoid granulations which project into the venous sinuses.

Circulation
1. Lateral ventricles (via foramen of Munro)
2. 3rd ventricle
3. Cerebral aqueduct (aqueduct of Sylvius)
4. 4th ventricle
5. Subarachnoid space (via foramina of Magendie and Luschka)
6. Reabsorbed into the venous system via arachnoid granulations into superior sagittal sinus

Composition
Glucose: 50-80mg/dl
Protein: 15-40 mg/dl
Red blood cells: Nil
White blood cells: 0-3 cells/ mm3

1348
Q

A 48 year old lady has a metallic heart valve and requires a paraumbilical hernia repair. Perioperatively she is receiving intra venous unfractionated heparin. To perform the surgery safely a normal coagulation state is required. Which of the following strategies is routine standard practice?

Administration of 10 mg of vitamin K the night prior to surgery and stopping the heparin infusion 6 hours pre operatively

Stopping the heparin infusion 6 hours pre operatively

Stop the heparin infusion on induction of anaesthesia

Stopping the heparin infusion 6 hours pre operatively and administration of intravenous protamine sulphate on commencing the operation

None of the above

A

Patients with metallic heart valves will generally stop unfractionated heparin 6 hours pre operatively. Unfractionated heparin is generally cleared from the circulation within 2 hours so this will allow plenty of time and is the method of choice in the elective setting. Protamine sulphate will reverse heparin but is associated with risks of anaphylaxis and is thus not generally used unless immediate reversal of anticoagulation is needed, e.g. coming off bypass.

Heparin

Causes the formation of complexes between antithrombin and activated thrombin/factors 7,9,10,11 & 12

Advantages of low molecular weight heparin
Better bioavailability
Lower risk of bleeding
Longer half life
Little effect on APTT at prophylactic dosages
Less risk of HIT

Complications
Bleeding
Osteoporosis
Heparin induced thrombocytopenia (HIT): occurs 5-14 days after 1st exposure
Anaphylaxis

In surgical patients that may need a rapid return to theatre, administration of unfractionated heparin is preferred; as low molecular weight heparins have a longer duration of action and are harder to reverse.

1349
Q

What is measured to obtain renal plasma flow?

Creatinine

Para-amino hippuric acid (PAH)

Inulin

Glucose

Protein

A

Renal plasma flow = (amount of PAH in urine per unit time) / (difference in PAH concentration in the renal artery or vein)

Normal value = 660ml/min

Renal Physiology

Overview
Each nephron is supplied with blood from an afferent arteriole that opens onto the glomerular capillary bed.
Blood then flows to an efferent arteriole, supplying the peritubular capillaries and medullary vasa recta.
The kidney receives up to 25% of resting cardiac output.

Control of blood flow
The kidney is able to autoregulate its blood flow between systolic pressures of 80- 180mmHg so there is little variation in renal blood flow.
This is achieved by myogenic control of arteriolar tone, both sympathetic input and hormonal signals (e.g. renin) are responsible.

Glomerular structure and function
Blood inside the glomerulus has considerable hydrostatic pressure.
The basement membrane has pores that will allow free diffusion of smaller solutes, larger negatively charged molecules such as albumin are unable to cross.
The glomerular filtration rate (GFR) is equal to the concentration of a solute in the urine, times the volume of urine produced per minute, divided by the plasma concentration (assuming that the solute is freely diffused e.g. inulin).
In clinical practice creatinine is used because it is subjected to very little proximal tubular secretion.
Although subject to variability, the typical GFR is 125ml per minute.
Glomerular filtration rate = Total volume of plasma per unit time leaving the capillaries and entering the bowman’s capsule
Renal clearance = volume plasma from which a substance is removed per minute by the kidneys

Substances used to measure GFR have the following features:
1. Inert
2. Free filtration from the plasma at the glomerulus (not protein bound)
3. Not absorbed or secreted at the tubules
4. Plasma concentration constant during urine collection

Examples: inulin, creatinine

plasma concentration (mmol/l)

The clearance of a substance is dependent not only on its diffusivity across the basement membrane but also subsequent tubular secretion and / or reabsorption.
So glucose which is freely filtered across the basement membrane is usually reabsorbed from tubules giving a clearance of zero.

Tubular function
Reabsorption and secretion of substances occurs in the tubules.
In the proximal tubule substrates such as glucose, amino acids and phosphate are co-transported with sodium across the semi permeable membrane.
Glucose is reabsorbed by both active and passive processes. The former uses sodium–glucose co-transporters (SGLTs) in the proximal convoluted tubule. The latter achieved through facilitated and passive process through GLUT transporters.
Up to two thirds of filtered water is reabsorbed in the proximal tubules.
This will lead to increase in urea concentration in the distal tubule allowing for its increased diffusion.
Substances to be secreted into the tubules are taken up from the peritubular blood by tubular cells.
Solutes such as paraaminohippuric acid are cleared with a single passage through the kidneys and this is why it is used to measure renal plasma flow. Ions such as calcium and phosphate will have a tubular reabsorption that is influenced by plasma PTH levels.
Potassium may be both secreted and re-absorbed and is co-exchanged with sodium.

Loop of Henle
Approximately 60 litres of water containing 9000mmol sodium enters the descending limb of the loop of Henle in 24 hours.
Loops from the juxtamedullary nephrons run deep into the medulla.
The osmolarity of fluid changes and is greatest at the tip of the papilla.
The thick ascending limb is impermeable to water, but highly permeable to sodium and chloride ions.
This loss means that at the beginning of the thick ascending limb the fluid is hypo osmotic compared with adjacent interstitial fluid.
In the thick ascending limb the reabsorption of sodium and chloride ions occurs by both facilitated and passive diffusion pathways.
The loops of Henle are co-located with vasa recta, these will have similar solute compositions to the surrounding extracellular fluid so preventing the diffusion and subsequent removal of this hypertonic fluid.
The energy dependent reabsorption of sodium and chloride in the thick ascending limb helps to maintain this osmotic gradient.

1350
Q

A 48 year old lady is admitted with crampy abdominal pain and diarrhoea. She has been unwell for the past 12 hours. In the history she complains that her milk bottles have been pecked repeatedly by birds, she otherwise has had no dietary changes. Which of the following is the most likely causative organism?

Staphylococcus aureus

Campylobacter jejuni

Clostridium difficile

Norovirus

Clostridium botulinum

A

Birds are a recognised reservoir of campylobacter.

Bacterial Gastroenteritis

Causative organisms Features
Campylobacter jejuni
Most common cause of acute infective diarrhoea
Spiral, gram negative rods
Usually infects caecum and terminal ileum. Local lymphadenopathy is common
May mimic appendicitis as it has marked right iliac fossa pain
Reactive arthritis is seen in 1-2% of cases
Shigella spp.
Members of the enterobacteriaceae
Gram negative bacilli
Clinically causes dysentery
Shigella soneii is the commonest infective organism (mild illness)
Usually self limiting, ciprofloxacin may be required if individual is in a high risk group
Salmonella spp
Facultatively anaerobic, gram negative, enterobacteriaceae
Infective dose varies according to subtype
Salmonellosis: usually transmitted by infected meat (especially poultry) and eggs
E. coli
Enteropathogenic
Enteroinvasive: dysentery, large bowel necrosis/ulcers
Enterotoxigenic: small intestine, travelers diarrhoea
Enterohaemorrhagic: 0157, cause a haemorrhagic colitis, haemolytic uraemic syndrome and thrombotic thrombocytopaenic purpura
Yersinia enterocolitica
Gram negative, coccobacilli
Typically produces a protracted terminal ileitis that may mimic Crohns disease
Differential diagnosis acute appendicitis
May progress to septicaemia in susceptible individuals
Usually sensitive to quinolone or tetracyclines
Vibrio cholera
Short, gram negative rods
Transmitted by contaminated water, seafood
Symptoms include sudden onset of effortless vomiting and profuse watery diarrhoea
Correction of fluid and electrolyte losses are the mainstay of treatment
Most cases will resolve, antibiotics are not generally indicated

1351
Q

The foramen marking the termination of the adductor canal is located in which of the following?

Adductor longus

Adductor magnus

Adductor brevis

Sartorius

Semimembranosus

A

The foramen marking the distal limit of the adductor canal is contained within adductor magnus. The vessel passes through this region to enter the popliteal fossa.

Adductor canal

Also called Hunter’s or subsartorial canal
Immediately distal to the apex of the femoral triangle, lying in the middle third of the thigh. Canal terminates at the adductor hiatus.

Borders Contents
Laterally Vastus medialis muscle Saphenous nerve
Posteriorly Adductor longus, adductor magnus Superficial femoral artery
Roof Sartorius Superficial femoral vein

1352
Q

What is the half life of insulin in the circulation of a normal healthy adult?

Less than 30 minutes

Between 1 and 2 hours

Between 2 and 3 hours

Between 4 and 5 hours

Over 6 hours

A

Insulin is degraded by enzymes in the circulation. It typically has a half life of less than 30 minutes. Abnormalities of the clearance of insulin may occur in type 2 diabetes.

Insulin

Insulin is a peptide hormone, produced by beta cells of the pancreas, and is central to regulating carbohydrate and fat metabolism in the body. Insulin causes cells in the liver, skeletal muscles, and fat tissue to absorb glucose from the blood. In the liver and skeletal muscles, glucose is stored as glycogen, and in fat cells (adipocytes) it is stored as triglycerides.

Structure
The human insulin protein is composed of 51 amino acids, and has a molecular weight of 5808 Da. It is a dimer of an A-chain and a B-chain, which are linked together by disulfide bonds.

Synthesis
Pro-insulin is formed by the rough endoplasmic reticulum in pancreatic beta cells. Then pro-insulin is cleaved to form insulin and C-peptide. Insulin is stored in secretory granules and released in response to Ca2+.

Function
Secreted in response to hyperglycaemia
Glucose utilisation and glycogen synthesis
Inhibits lipolysis
Reduces muscle protein loss

1353
Q

A 31 year old lady is struck by a car and is 32 weeks pregnant. On arrival in the emergency department she has a systolic blood pressure of 105mmHg and a pulse rate of 126 beats per minute. Abdominal examination demonstrates a diffusely tender abdomen and some left sided flank bruising. A FAST scan is normal. What is the most appropriate course of action?

Arrange a departmental abdominal USS scan

Arrange an urgent abdominal MRI scan

Perform a laparotomy

Perform diagnostic peritoneal lavage

Arrange an urgent abdominal CT scan

A

The patient’s mechanism of injury makes a solid organ injury likely. FAST scanning is associated with a false negative rate in pregnancy which makes the negative result less reassuring. CT scanning remains the gold standard.

Imaging in the pregnant trauma patient

Sonography and FAST scanning are established in pregnancy and have the advantage of avoiding ionising radiation. However, the sensitivity of the FAST scan is reduced in pregnancy especially with advanced gestational age. Sensitivity of FAST scanning is 60-80% across all trimesters and 90% in the first. CT scanning remains the first line investigation in major trauma where significant visceral injury is suspected. The maximum permitted safe dose of radiation in pregnancy is 5mSv. A pelvic CT scan would fall below this level. That said, early exposure to radiation will increase the risk of developmental anomalies and foetal loss. Late exposure increases the risk of childhood cancer twofold. CT scanning remains the most sensitive test for identifying complications such as placental abruption in this group.

1354
Q

Which of the following is not a characteristic of the proximal convoluted tubule in the kidney?

Up to 95% of filtered amino acids will be reabsorbed at this site

It is a risk of damage in a patient with compartment syndrome due to a tibial fracture

It is responsible for absorbing more than 50% of filtered water

Its secretory function is most effective at low systolic blood pressures (typically less than 100mmHg)

Glucose is reabsorbed by combined active and passive (facilitated diffusion processes)

A

The proximal convoluted tubule may undergo necrosis in situations such as compartment syndrome. It is responsible for reabsorbing up to two thirds of filtered water. Low systolic blood pressures (below the renal autoregulatory range) are a risk factor for acute tubular necrosis. Within the autoregulatory range the absolute value of systolic BP has little effect. The process of facilitated diffusion of glucose refers to its co-transport with sodium.

Renal Physiology

Overview
Each nephron is supplied with blood from an afferent arteriole that opens onto the glomerular capillary bed.
Blood then flows to an efferent arteriole, supplying the peritubular capillaries and medullary vasa recta.
The kidney receives up to 25% of resting cardiac output.

Control of blood flow
The kidney is able to autoregulate its blood flow between systolic pressures of 80- 180mmHg so there is little variation in renal blood flow.
This is achieved by myogenic control of arteriolar tone, both sympathetic input and hormonal signals (e.g. renin) are responsible.

Glomerular structure and function
Blood inside the glomerulus has considerable hydrostatic pressure.
The basement membrane has pores that will allow free diffusion of smaller solutes, larger negatively charged molecules such as albumin are unable to cross.
The glomerular filtration rate (GFR) is equal to the concentration of a solute in the urine, times the volume of urine produced per minute, divided by the plasma concentration (assuming that the solute is freely diffused e.g. inulin).
In clinical practice creatinine is used because it is subjected to very little proximal tubular secretion.
Although subject to variability, the typical GFR is 125ml per minute.
Glomerular filtration rate = Total volume of plasma per unit time leaving the capillaries and entering the bowman’s capsule
Renal clearance = volume plasma from which a substance is removed per minute by the kidneys

Substances used to measure GFR have the following features:
1. Inert
2. Free filtration from the plasma at the glomerulus (not protein bound)
3. Not absorbed or secreted at the tubules
4. Plasma concentration constant during urine collection

Examples: inulin, creatinine

plasma concentration (mmol/l)

The clearance of a substance is dependent not only on its diffusivity across the basement membrane but also subsequent tubular secretion and / or reabsorption.
So glucose which is freely filtered across the basement membrane is usually reabsorbed from tubules giving a clearance of zero.

Tubular function
Reabsorption and secretion of substances occurs in the tubules.
In the proximal tubule substrates such as glucose, amino acids and phosphate are co-transported with sodium across the semi permeable membrane.
Glucose is reabsorbed by both active and passive processes. The former uses sodium–glucose co-transporters (SGLTs) in the proximal convoluted tubule. The latter achieved through facilitated and passive process through GLUT transporters.
Up to two thirds of filtered water is reabsorbed in the proximal tubules.
This will lead to increase in urea concentration in the distal tubule allowing for its increased diffusion.
Substances to be secreted into the tubules are taken up from the peritubular blood by tubular cells.
Solutes such as paraaminohippuric acid are cleared with a single passage through the kidneys and this is why it is used to measure renal plasma flow. Ions such as calcium and phosphate will have a tubular reabsorption that is influenced by plasma PTH levels.
Potassium may be both secreted and re-absorbed and is co-exchanged with sodium.

Loop of Henle
Approximately 60 litres of water containing 9000mmol sodium enters the descending limb of the loop of Henle in 24 hours.
Loops from the juxtamedullary nephrons run deep into the medulla.
The osmolarity of fluid changes and is greatest at the tip of the papilla.
The thick ascending limb is impermeable to water, but highly permeable to sodium and chloride ions.
This loss means that at the beginning of the thick ascending limb the fluid is hypo osmotic compared with adjacent interstitial fluid.
In the thick ascending limb the reabsorption of sodium and chloride ions occurs by both facilitated and passive diffusion pathways.
The loops of Henle are co-located with vasa recta, these will have similar solute compositions to the surrounding extracellular fluid so preventing the diffusion and subsequent removal of this hypertonic fluid.
The energy dependent reabsorption of sodium and chloride in the thick ascending limb helps to maintain this osmotic gradient.

1355
Q

During a Patey mastectomy, the pectoralis minor muscle is divided to gain access to the level III axillary nodes. What is the nerve supply to this structure?

Axillary nerve

Medial pectoral nerve

Lateral pectoral nerve

Long thoracic nerve

Thoracodorsal nerve

A

Pectoralis minor is supplied by the medial pectoral nerve

Pectoral muscles

Pectoralis major
Origin From the medial two thirds of the clavicle, manubrium and sternocostal angle
Insertion Lateral edge of the bicipital groove of the humerus
Nerve supply Medial and lateral pectoral nerves
Actions Adductor and medial rotator of the humerus

Pectoralis minor
Origin Upper margins of third to fifth ribs and intercostal fascia
Insertion Medial border and upper surface of the coracoid process
Nerve supply Medial pectoral nerve
Actions Assists serratus anterior in drawing scapula forwards and depresses the point of the shoulder

1356
Q

A child presents to the emergency department with sudden onset scrotal pain 4 hours previously. On examination, the testis is elevated and tender. The child has eaten some chips in the waiting room. What is the best course of action?

Administration of analgesic block and manipulation of the testis

Fast for 3 hours and then take to theatre for scrotal exploration

Immediate scrotal exploration under general anaesthesia

Arrange urgent scrotal ultrasound

Administration of regional local anaesthetic block and explore scrotum immediately

A

This is a likely torsion so surgery cannot be deferred. Local anaesthetic scrotal exploration is likely to be very poorly tolerated. Delay of surgery and USS are not appropriate. Immediate exploration under GA with appropriate airway protection in place.

Testicular disorders-paediatric

Testicular disorders
Testicular disorders are some of the commonest conditions present in paediatric urological practice.

Cryptorchidism
The embryological descent of the testicle from within the abdominal cavity may be subject to a number of variations. Distinctions need to be made clinically from a non descended testis and a testis that is retractile.
Testis that lie outside the normal path of embryological descent are termed ectopic testis. Undescended testis occurs in 1% of male infants. Where the testis does not lie in an intra scrotal location, its location should be ascertained (by laparoscopy in first instance). Where both testes are absent, the infant may be intersex.
MRI scanning may reveal intra-abdominal testes; however a GA is often needed to perform this investigation in this age group.
Testes that are undescended should be placed in the scrotum after 1 year of age, as the testosterone surge that may facilitate descent occurs at 6 months of age.
Where the testes lie distally e.g. Superficial inguinal pouch an open orchidopexy is the procedure of choice.
With abdominal testes a laparoscopy should be performed. The risk of seminoma is increased in individuals with a non descended testes and this risk is not reduced by orchidopexy.

Testicular torsion
Typically the patient has severe sudden onset of scrotal pain. The difficulty in paediatric practice is the lack of clear history.
On examination the testis is tender and enlarged.
Management is by surgical exploration.
Delay beyond 6 hours is associated with low salvage rates.
A torted hyatid produces pain that is far more localised and the testis itself should feel normal. However, diagnostic doubt often exists and in such cases surgical exploration is warranted.

Hydrocele
Occur secondary to patent processus vaginalis
Present as fluid filling in scrotum or as cyst of the spermatic cord
Communicating hydroceles are treated by a trans inguinal ligation of the PPV
Cystic hydroceles in older children may be treated with scrotal exploration

1357
Q

A 28 year old man has a pleomorphic adenoma and the decision is made to resect this surgically. Which of the following structures is least likely to be encountered during surgical resection of the parotid gland?

External carotid artery

Retromandibular vein

Auriculotemporal nerve

Mandibular branch of trigeminal nerve

Zygomatic branch of the facial nerve

A

D-Mandibular branch of trigeminal nerve

Structures passing through the parotid gland
Facial nerve and branches
External carotid artery (and its branches; the maxillary and superficial temporal)
Retromandibular vein
Auriculotemporal nerve
The mandibular nerve is well separated from the parotid gland.
The maxillary vein joins to the superficial temporal vein and they form the retromandibular vein which then runs through the parotid gland.
The auriculotemporal nerve runs through the gland. Following a parotidectomy this nerve may be damaged and during neuronal regrowth may then attach to sweat glands in this region. This can then cause gustatory sweating (Freys Syndrome).
The facial nerve branch is the marginal mandibular branch and this is related to the gland.

1358
Q

A 2 day old baby is noted to have voiding difficulties and on closer inspection is noted to have hypospadias. Which of the following abnormalities is most commonly associated with the condition?

Cryptorchidism

Diaphragmatic hernia

Ventricular - septal defect

Bronchogenic cyst

Atrial septal defect

A

Hypospadias most commonly occurs as an isolated disorder. Associated urological abnormalities may be seen in up to 40% of infants, of these cryptorchidism is the most frequent (10%).

Hypospadias

The urethral meatus opens on the ventral surface of the penis. There is also a ventral deficiency of the foreskin. The urethral meatus may open more proximally in the more severe variants. However, 75% of the openings are distally located. The incidence is 1 in 300 male births.

Features include:

Absent frenular artery
Ventrally opened glans
Skin tethering to hypoplastic urethra
Splayed columns of spongiosum tissue distal to the meatus
Deficiency of the foreskin ventrally

Management:

No routine cultural circumcisions
Urethroplasty
Penile reconstruction

The foreskin is often utilised in the reconstructive process. In boys with very distal disease no treatment may be needed.

1359
Q

Via which structure does cerebrospinal fluid enter the subarachnoid space?

Cisterna magna

Cerebral aqueduct

Foramen of Luschka

Cisterna chyli

Arachnoid granulations

A

CSF leaves the 4th ventricle via the laterally placed foramen of Luschka and also via the midline foramen of Magendie.

Cerebrospinal fluid

The CSF fills the space between the arachnoid mater and pia mater (covering surface of the brain). The total volume of CSF in the brain is approximately 150ml. Approximately 500 ml is produced by the ependymal cells in the choroid plexus (70%), or blood vessels (30%). It is reabsorbed via the arachnoid granulations which project into the venous sinuses.

Circulation
1. Lateral ventricles (via foramen of Munro)
2. 3rd ventricle
3. Cerebral aqueduct (aqueduct of Sylvius)
4. 4th ventricle
5. Subarachnoid space (via foramina of Magendie and Luschka)
6. Reabsorbed into the venous system via arachnoid granulations into superior sagittal sinus

Composition
Glucose: 50-80mg/dl
Protein: 15-40 mg/dl
Red blood cells: Nil
White blood cells: 0-3 cells/ mm3

1360
Q

Which of the following most commonly arises from the brachiocephalic artery?

Vertebral artery

Subscapular artery

Thyroidea ima artery

Left Subclavian artery

None of the above

A

C-Thyroidea ima artery

Other occasional branches include the thymic and bronchial branch.

Brachiocephalic artery

The brachiocephalic artery is the largest branch of the aortic arch. From its aortic origin it ascends superiorly, it initially lies anterior to the trachea and then on its right hand side. It branches into the common carotid and right subclavian arteries at the level of the sternoclavicular joint.

Path
Origin- apex of the midline of the aortic arch
Passes superiorly and posteriorly to the right
Divides into the right subclavian and right common carotid artery

Relations
Anterior
Sternohyoid
Sternothyroid
Thymic remnants
Left brachiocephalic vein
Right inferior thyroid veins
Posterior
Trachea
Right pleura
Right lateral
Right brachiocephalic vein
Superior part of SVC
Left lateral
Thymic remnants
Origin of left common carotid
Inferior thyroid veins
Trachea (higher level)

Branches
Normally none but may have the thyroidea ima artery

1361
Q

A pathologist is examining a histological section and identifies Hassall’s corpuscles. With what are they most commonly associated?

Follicular carcinoma of the thyroid

Medulla of the thymus

Medulla of the spleen

Medulla of the kidney

Fundus of the stomach

A

Hassall’s corpuscles are the concentric ring of epithelial cells seen in the medulla of the thymus.

Thymus

The thymus develops from the third and fourth pharyngeal pouches. It descends to lie in the anterior superior mediastinum. It is encapsulated and is subdivided into lobules, these consist of a cortex and a medulla. The cortex is composed of tightly packed lymphocytes, the medulla consists largely of epithelial cells. The medullary epithelial cells are concentrically arranged and may surround a keratinised centre, known as Hassall’s corpuscles.
The inferior parathyroid glands also develop from the third pharyngeal pouch and may also be located with the thymus gland.
Its arterial supply is from the internal mammary artery or pericardiophrenic arteries. Venous drainage is to the left brachiocephalic vein.

Hassall’s corpuscles stained with H+E

1362
Q

A 22 year old man is undergoing a wedge excision of his great toenail. As the surgeon passes a needle into the area to administer local anaesthetic, the patient notices a sharp pain. By which pathway will this sensation be conveyed to the central nervous system?

Anterior corticospinal tract

Posterior spinocerebellar tract

Cuneate fasciculus

Vestibulospinal tract

Spinothalamic tract

A

Spinothalamic tract- Pain and temperature
Vestibulospinal tract- Motor neuronal signals relating to posture
Cuneate fasciculus- Fine touch, pressure and proprioception
Posterior spinocerebellar tract- Proprioceptive signals to cerebellum
Anterior corticospinal tract- Conveys motor signals from precentral gyrus to motor cells within the cord
Pain impulses are transmitted via the spinothalamic tract

Spinothalamic tract

The spinothalamic tract transmits impulses from receptors which measure crude touch, pain and temperature. The spinothalamic tract comprises the lateral and anterior spinothalamic tracts, the former typically transmits pain and temperature and the latter crude touch and pressure. Neurones transmitting these signals will typically ascend by one or two vertebral levels in Lissaurs tract prior to decussating in the spinal cord itself. Neurones then pass rostrally in the cord to connect at the thalamus.

1363
Q

Which one of the following genes protects against neoplasms?

sis

p53

ras

myc

src

A

p53 is a tumour supressor gene and located on chromosome 17. It plays an important role in causing cells that are undergoing neoplastic changes to enter an apoptotic pathway.

Oncogenes

Oncogenes are cancer promoting genes that are derived from normal genes (proto-oncogenes). Proto-oncogenes play an important physiological role in cellular growth. They are implicated in the development of up to 20% of human cancers.

Proto-oncogenes may become oncogenes via the following processes:
Mutation (point mutation)
Chromosomal translocation
Increased protein expression
Only one mutated copy of the gene is needed for cancer to occur - a dominant effect

Classification of oncogenes
Growth factors e.g. Sis
Transcription factors e.g. Myc
Receptor tyrosine kinase e.g. RET
Cytoplasmic tyrosine kinase e.g. Src
Regulatory GTPases e.g. Ras

Tumour suppressor genes
Tumour suppressor genes restrict or repress cellular proliferation in normal cells. Their inactivation through mutation or germ line incorporation is implicated in renal, colonic, breast, bladder and many other cancers. One of the best known tumour suppressor genes is p53. p53 gene offers protection by causing apoptosis of damaged cells. Other well known genes include BRCA 1 and 2.

1364
Q

Which of the following does not lead to relaxation of the lower oesophageal sphincter?

Metoclopramide

Botulinum toxin type A

Nicotine

Alcohol

Theophylline

A

Metoclopramide acts directly on the smooth muscle of the LOS to cause it to contract.
Theophylline is a phosphodiesterase inhibitor (mimics action of prostaglandin E1) which causes relaxation of the LOS.

Peristalsis

Circular smooth muscle contracts behind the food bolus and longitudinal smooth muscle propels the food through the oesophagus
Primary peristalsis spontaneously moves the food from the oesophagus into the stomach (9 seconds)
Secondary peristalsis occurs when food, which doesn’t enter the stomach, stimulates stretch receptors to cause peristalsis
In the small intestine each peristalsis waves slows to a few seconds and causes mixture of chyme
In the colon three main types of peristaltic activity are recognised (see below)

Colonic peristalsis
Segmentation contractions Localised contractions in which the bolus is subjected to local forces to maximise mucosal absorption
Antiperistaltic contractions towards ileum Localised reverse peristaltic waves to slow entry into colon and maximise absorption
Mass movements Waves migratory peristaltic waves along the entire colon to empty the organ prior to the next ingestion of food bolus

1365
Q

A 23 year old lady is undergoing a trendelenberg procedure for varicose veins. During the dissection of the saphenofemoral junction, which of the structures listed below is most liable to injury?

Superficial circumflex iliac artery

Superficial circumflex iliac vein

Femoral artery

Femoral nerve

Deep external pudendal artery

A

The deep external pudendal artery runs under the long saphenous vein close to its origin and may be injured. It is at greatest risk of injury during the flush ligation of the saphenofemoral junction. Provided an injury is identified and vessel ligated, injury is seldom associated with any serious adverse sequelae.

Saphenous vein

Long saphenous vein
This vein may be harvested for bypass surgery, or removed as treatment for varicose veins with saphenofemoral junction incompetence.

Originates at the 1st digit where the dorsal vein merges with the dorsal venous arch of the foot
Passes anterior to the medial malleolus and runs up the medial side of the leg
At the knee, it runs over the posterior border of the medial epicondyle of the femur bone
Then passes laterally to lie on the anterior surface of the thigh before entering an opening in the fascia lata called the saphenous opening
It joins with the femoral vein in the region of the femoral triangle at the saphenofemoral junction

Tributaries
Medial marginal
Superficial epigastric
Superficial iliac circumflex
Superficial external pudendal veins

Short saphenous vein
Originates at the 5th digit where the dorsal vein merges with the dorsal venous arch of the foot, which attaches to the great saphenous vein.
It passes around the lateral aspect of the foot (inferior and posterior to the lateral malleolus) and runs along the posterior aspect of the leg (with the sural nerve)
Passes between the heads of the gastrocnemius muscle, and drains into the popliteal vein, approximately at or above the level of the knee joint.

1366
Q

What is the sensory nerve supply to the skin overlying the angle of the jaw?

Maxillary branch of the trigeminal nerve

Mandibular branch of the trigeminal nerve

C3-C4

Greater auricular nerve (C2-C3)

Buccal branch of the facial nerve

A

The trigeminal nerve is the major sensory nerve to the face except over the angle of the jaw. The angle of the jaw is innervated by the greater auricular nerve.

Trigeminal nerve

The trigeminal nerve is the main sensory nerve of the head. In addition to its major sensory role, it also innervates the muscles of mastication.

Distribution of the trigeminal nerve
Sensory
Scalp
Face
Oral cavity (and teeth)
Nose and sinuses
Dura mater
Motor
Muscles of mastication
Mylohyoid
Anterior belly of digastric
Tensor tympani
Tensor palati
Autonomic connections (ganglia)
Ciliary
Sphenopalatine
Otic
Submandibular

Path
Originates at the pons
Sensory root forms the large, crescentic trigeminal ganglion within Meckel’s cave, and contains the cell bodies of incoming sensory nerve fibres. Here the 3 branches exit.
The motor root cell bodies are in the pons and the motor fibres are distributed via the mandibular nerve. The motor root is not part of the trigeminal ganglion.

Branches of the trigeminal nerve
Ophthalmic nerve Sensory only
Maxillary nerve Sensory only
Mandibular nerve Sensory and motor

Sensory
Ophthalmic Exits skull via the superior orbital fissure
Sensation of: scalp and forehead, the upper eyelid, the conjunctiva and cornea of the eye, the nose (including the tip of the nose, except alae nasi), the nasal mucosa, the frontal sinuses, and parts of the meninges (the dura and blood vessels).
Maxillary nerve Exit skull via the foramen rotundum
Sensation: lower eyelid and cheek, the nares and upper lip, the upper teeth and gums, the nasal mucosa, the palate and roof of the pharynx, the maxillary, ethmoid and sphenoid sinuses, and parts of the meninges.
Mandibular nerve Exit skull via the foramen ovale
Sensation: lower lip, the lower teeth and gums, the chin and jaw (except the angle of the jaw), parts of the external ear, and parts of the meninges.

Motor
Distributed via the mandibular nerve.
The following muscles of mastication are innervated:
Masseter
Temporalis
Medial pterygoid
Lateral pterygoid

Other muscles innervated include:
Tensor veli palatini
Mylohyoid
Anterior belly of digastric
Tensor tympani

1367
Q

A 4 year boy presents with an abnormal gait. He has a history of recent viral illness. His WCC is 11 and ESR is 30. What is the most likely cause?

Perthes disease

Transient synovitis

Septic arthritis

Slipped upper femoral epiphysis

Osteomyelitis

A

Viral illnesses can be associated with transient synovitis. The WCC should ideally be > 12 and the ESR > 40 to suggest septic arthritis.

Paediatric orthopaedics

Diagnosis Mode of presentation Treatment Radiology
Developmental dysplasia of the hip Usually diagnosed in infancy by screening tests. May be bilateral, when disease is unilateral there may be leg length inequality. As disease progresses child may limp and then early onset arthritis. More common in extended breech babies. Splints and harnesses or traction. In later years osteotomy and hip realignment procedures may be needed. In arthritis a joint replacement may be needed. However, this is best deferred if possible as it will almost certainly require revision Initially no obvious change on plain films and USS gives best resolution until 3 months of age. On plain films Shentons line should form a smooth arc
Perthes Disease Hip pain (may be referred to the knee) usually occurring between 5 and 12 years of age. Bilateral disease in 20%. Remove pressure from joint to allow normal development. Physiotherapy. Usually self-limiting if diagnosed and treated promptly. X-rays will show flattened femoral head. Eventually in untreated cases the femoral head will fragment.
Slipped upper femoral epiphysis Typically seen in obese male adolescents. Pain is often referred to the knee. Limitation to internal rotation is usually seen. Knee pain is usually present 2 months prior to hip slipping. Bilateral in 20%. Bed rest and non-weight bearing. Aim to avoid avascular necrosis. If severe slippage or risk of it occurring then percutaneous pinning of the hip may be required. X-rays will show the femoral head displaced and falling inferolaterally (like a melting ice cream cone) The Southwick angle gives indication of disease severity

1368
Q

A 28 year old man with Crohn’s disease has undergone a number of resections. His BMI is currently 18 and his albumin is 18. He feels well but does have a small localised perforation of his small bowel. The gastroenterologists are giving azathioprine. What is the most appropriate advice regarding feeding?

Nil by mouth

Nil by mouth and continuous intra venous fluids until surgery

Enteral feeding

Parenteral feeding

Nutritional supplements

A

This man is malnourished, although he is likely to require surgery it is best for him to be nutritionally optimised first. As he may have reduced surface area for absorption and has a localised perforation TPN is likely to be the best feeding modality.

Parenteral feeding-NICE guidelines

Parenteral nutrition: NICE guidelines summary

Identify patients as malnourished or at risk

Patients identified as being malnourished-
BMI < 18.5 kg/m2
unintentional weight loss of > 10% over 3-6/12
BMI < 20 kg/m2 and unintentional weight loss of > 5% over 3-6/12

AT RISK of malnutrition-
eaten nothing or little > 5 days, who are likely to eat little for a further 5 days
poor absorptive capacity
high nutrient losses
high metabolism

Identify unsafe/inadequate oral intake OR a non functional GI tract/perforation/inaccessible

Consider parenteral nutrition:
for feeding < 14 days consider feeding via a peripheral venous catheter
for feeding > 30 days use a tunneled subclavian line
continuous administration in severely unwell patients
if feed needed > 2 weeks consider changing from continuous to cyclical feeding
don’t give > 50% of daily regime to unwell patients in first 24-48 hours

Surgical patients: if malnourished with unsafe swallow OR a non functional GI tract/perforation/inaccessible then consider peri operative parenteral feeding.

1369
Q

A 73 year old man with rest pain and ulceration of the foot undergoes a femoro-distal bypass graft with a PTFE graft. At the end of the procedure there are good distal foot pulses and a warm pink foot. Over the ensuing 60 days the foot becomes progressively cooler and the pulses diminish. What is the most likely underlying explanation for this process?

Embolus

Neo-intimal flap

Neo-intimal hyperplasia

Polyarteritis

Steal syndrome

A

Neo-intimal hyperplasia in distal arterial anastamoses may be reduced by use of a Miller Cuff when PTFE is the bypass conduit.
PTFE may induce neo-intimal hyperplasia with subsequent occlusion of the distal anastomosis. In more proximal arterial bypass surgery the process of neo-intimal hyperplasia is not sufficient to cause anastomotic occlusion. However, distal bypasses are at greater risk and if vein cannot be used as a conduit then the distal end of the PTFE should anastomosed to a vein cuff to minimise the risk of neo-intimal hyperplasia.

Anastomoses

  • A wide variety of anastomoses are constructed in surgical practice. Essentially the term refers to the restoration of luminal continuity. As such they are a feature of both abdominal and vascular surgery.

Visceral anastomoses

For an anastomosis to heal three criteria need to be fulfilled:

Adequate blood supply
Mucosal apposition
Minimal tension

When these are compromised the anastomosis may break down. Even in the best surgical hands some anastomoses are more prone to dehiscence than others. Oesophageal and rectal anastomoses are more prone to leakage and reported leak rates following oesophageal and rectal surgery can be as high as 20%. This figure includes radiological leaks and those with a clinically significant leak will be of a lower order of magnitude. As a rule small bowel anastomoses heal most reliably.

The decision as to how best to achieve mucosal apposition is one for each surgeon. Some will prefer the use of stapling devices as they are quicker to use, others will prefer to perform a sutured anastomosis. The attention to surgical technique is more important than the method chosen and a poorly constructed stapled anastomosis in thickened tissue is far more prone to leakage than a hand sewn anastomosis in the same circumstances.

If an anastomosis looks unsafe then it may be best not to construct one at all. In colonic surgery this is relatively clear cut and most surgeons would bring out an end colostomy. In situations such as oesophageal surgery this is far more problematic and colonic interposition may be required in this situation.

Vascular anastomoses

Most arterial surgery involving bypasses or aneurysm repairs will require construction of an arterial anastomosis. Technique is important and for small diameter distal arterial surgery the intimal hyperplasia resulting from a badly constructed anastomosis may render the whole operation futile before the patient leaves hospital.

Some key points about vascular anastomoses:

Always use non absorbable monofilament suture (e.g. Polypropylene).
Round bodied needle.
Correct size for anastamosis ( i.e. 6/0 prolene for bottom end of a femoro-distal bypass).
Suture should be continuous and from inside to outside of artery to avoid raising an intimal flap.

1370
Q

A 67 year old lady is suspected of having Pagets disease of the nipple. Mammography and USS are normal. What is the most appropriate next step in her management?

Arrange a core biopsy

Arrange FNAC of the area

Arrange a punch biopsy

Undertake a mastectomy

Arrange for focused radiotherapy

A

This is a relatively clear indication for a punch biopsy. If cellular atypia is present on punch biopsy then any in situ malignancy should be considered. FNAC would be unsuitable.

Tissue sampling

Tissue sampling is an important surgical process. Biopsy modalities vary according to the site, experience and subsequent planned therapeutic outcome

The modalities comprise:
-Fine needle aspiration cytology
-Core biopsy
-Excision biopsy
-Tru cut biopsy
-Punch biopsy
-Cytological smears
-Endoscopic or laparoscopic biopsy

When the lesion is superficial the decision needs to be taken as to whether complete excision is desirable or whether excision biopsy is acceptable. In malignant melanoma for example the need for safe margins will mean that a more radical surgical approach needs to be adopted after diagnostic confirmation from excision biopsy than would be the case in basal cell carcinoma. Punch biopsies are useful in gaining histological diagnosis of unclear skin lesions where excision biopsy is undesirable such as in establishing whether a skin lesion is vasculitic or not.

Fine needle aspiration cytology (FNAC) is an operator dependent procedure that may or may not be image guided and essentially involves passing a needle through a lesion whilst suction is applied to a syringe. The material thus obtained is expressed onto a slide and sent for cytological assessment. This test can be limited by operator inexperience and also by the lack of histological architectural information (e.g. Follicular carcinoma of the thyroid). Where a discharge is present a sample may be sent for cytology although in some sites (e.g. Nipple discharge ) the information gleaned may be meaningless.

Tissue samples may be obtained by both core and tru cut biopsy. A core biopsy is obtained by use of a spring loaded gun with a needle passing quickly through the lesion of interest. A tru cut biopsy achieves the same objective but the needle moved by hand. When performing these techniques image guidance may be desirable (e.g. In breast lesions). Consideration needs to be given to any planned surgical resection as it may be necessary to resect the biopsy tract along with the specimen (e.g. In sarcoma surgery).

Visceral lesions may be accessed percutaneously under image guidance such as ultrasound guided biopsy of liver metastases. Or under direct vision such as a colonoscopic biopsy.

1371
Q

A 40 year old woman had the anterior lobe of the pituitary removed because of a tumour. Without postoperative supplements, which of the following could occur?

Failure to produce adequate amounts of thyroxine

Failure to produce parathyroid hormone in response to hypocalcaemia

Failure to secrete catecholamine in response to stress

Failure to secrete insulin in hyperglycemia

Inability to concentrate urine in response to water deprivation

A

Its an anterior pituitary lesion and that is the reason thyroxine secretion is affected. ADH is released by the posterior pituitary.

Pituitary Gland

The pituitary gland is located within the sella turcica within the sphenoid bone in the middle cranial fossa. It is covered by a dural fold and weighs around 0.5g. It is attached to the hypothalamus by the infundibulum. The anterior pituitary receives hormonal stimuli from the hypothalamus by way of the hypothalamo-pituitary portal system. It develops from a depression in the wall of the pharynx (Rathkes pouch).

Anterior pituitary hormones
Growth hormone
Thyroid stimulating hormone
ACTH
Prolactin
LH and FSH
Melanocyte releasing hormone

Posterior pituitary hormones
Oxytocin
Anti diuretic hormone

1372
Q

In which of the following conditions is a Ghon complex most likely to be found?

Actinomycosis

Infection with mycobacterium tuberculosis

Mycosis fungoides

Lewy body dementia

Chronic lymphocytic leukaemia

A

Ghon focus is primary infection
Ghon complex is primary infection and regional nodal involvement

Tuberculosis pathology

Is a form of primary chronic inflammation, caused by the inability of macrophages to kill the Mycobacterium tuberculosis.
The macrophages often migrate to regional lymph nodes, the lung lesion plus affected lymph nodes is referred to as a Ghon complex.
This leads to the formation of a granuloma which is a collection of epithelioid histiocytes.
There is the presence of caseous necrosis in the centre.
The inflammatory response is mediated by a type 4 hypersensitivity reaction.
In healthy individuals the disease may be contained, in the immunocompromised disseminated (miliary TB) may occur.

Diagnosis
Waxy membrane of mycobacteria prevents binding with normal stains. Ziehl - Neelsen staining is typically used.
Culture based methods take far longer.

1373
Q

A 67 year old male is admitted to the surgical unit with acute abdominal pain. He is found to have a right sided pneumonia. The nursing staff put him onto 15L O2 via a non rebreathe mask. After 30 minutes the patient is found moribund, sweaty and agitated by the nursing staff. An arterial blood gas reveals:
pH 7.15
pCO2 10.2
pO2 8
Bicarbonate 32
Base excess - 5.2

What is the most likely cause for this patients deterioration?

Acute respiratory alkalosis secondary to hyperventilation

Over administration of oxygen in a COPD patient

Metabolic acidosis secondary to severe pancreatitis

Metabolic alkalosis secondary to hypokalaemia

Acute respiratory acidosis secondary to pneumonia

A

This patient has an acute respiratory acidosis, however this is on a background of chronic respiratory acidosis (due to COPD) with a compensatory metabolic alkalosis (the elevated bicarbonate is the main clue to the chronic nature of the respiratory acidosis). This blood gas picture is typical in a COPD patient who has received too much oxygen; these patients lose their hypoxic drive for respiration, therefore retain CO2 and subsequently hypoventilate leading to respiratory arrest. If the bicarbonate was normal, then the answer would be acute respiratory acidosis secondary to pneumonia.

Arterial blood gas interpretation

In advanced life support training, a 5 step approach to arterial blood gas interpretation is advocated.

  1. How is the patient?
  2. Is the patient hypoxaemic?
    The Pa02 on air should be 10.0-13.0 kPa
  3. Is the patient acidaemic (pH <7.35) or alkalaemic (pH >7.45)
  4. What has happened to the PaCO2?
    If there is acidaemia, an elevated PaCO2 will account for this
  5. What is the bicarbonate level or base excess?
    A metabolic acidosis will have a low bicarbonate level and a low base excess (< -2 mmol)
    A metabolic alkalosis will have a high bicarbonate and a high base excess (> +2 mmol)
1374
Q

A male infant is born prematurely at 26 weeks gestation by emergency cesarean section. Following the birth he develops respiratory distress syndrome and is ventilated. He begins to improve twelve days after birth. Then he becomes unwell and develops abdominal distension and passes bloody stools and vomits a small quantity of bile stained vomit. What is the most likely cause?

Ileal atresia

Hirschprungs disease

Pyloric stenosis

Necrotising enterocolitis

Meconium ileus

A

Necrotising enterocolitis often has a delayed presentation and affected infants will typically pass bloody stools. Plain films may show air in the intestinal wall (Pneumatosis).
Bilious vomiting in neonates

Causes of intestinal obstruction with bilious vomiting in neonates
Disorder Incidence and causation Age at presentation Diagnosis Treatment
Duodenal atresia 1 in 5000 (higher in Downs syndrome) Few hours after birth AXR shows ‘double bubble sign, contrast study may confirm Duodenoduodenostomy
Malrotation with volvulus Usually cause by incomplete rotation during embryogenesis Usually 3-7 days after birth, volvulus with compromised circulation may result in peritoneal signs and haemodynamic instability Upper GI contrast study may show DJ flexure is more medially placed, USS may show abnormal orientation of SMA and SMV Ladd’s procedure
Jejunal/ ileal atresia Usually caused by vascular insufficiency in utero, usually 1 in 3000 Usually within 24 hours of birth AXR will show air-fluid levels Laparotomy with primary resection and anastomosis
Meconium ileus Occurs in between 15 and20% of those babies with cystic fibrosis, otherwise 1 in 5000 Typically in first 24-48 hours of life with abdominal distension and bilious vomiting Air - fluid levels on AXR, sweat test to confirm cystic fibrosis Surgical decompression, serosal damage may require segmental resection
Necrotising enterocolitis Up to 2.4 per 1000 births, risks increased in prematurity and inter-current illness Usually second week of life Dilated bowel loops on AXR, pneumatosis and portal venous air Conservative and supportive for non perforated cases, laparotomy and resection in cases of perforation of ongoing clinical deterioration

1375
Q

Which of the following statements relating to biliary atresia is untrue?

It most commonly presents as prolonged conjugated jaundice in the neonatal period.

Evidence of portal hypertension at diagnosis is seldom present in the UK.

It may be confused with Alagille syndrome.

The Kasai procedure is best performed in the first 8 weeks of life.

Survival following a successful Kasai procedure is approximately 45% at 5 years.

A

Alagille syndrome autosomal dominant disorder characterised by presence of paucity of bile ducts and cardiac defects. Only the embryonic form of biliary atresia is associated with cardiac and other embryological defects.
Biliary atresia usually presents with obstructed jaundice. A Kasai procedure is best performed in the first 8 weeks of life. If a Kasai procedure is successful most patients will not require liver transplantation. 45% of patients post Kasai procedure will require transplantation. However, overall survival following a successful Kasai procedure is 80%.

Biliary atresia

1 in 17000 affected
Biliary tree lumen is obliterated by an inflammatory cholangiopathy causing progressive liver damage

Clinical features
Infant well in 1st few weeks of life
No family history of liver disease
Jaundice in infants > 14 days in term infants (>21 days in pre term infants)
Pale stool, yellow urine (colourless in babies)
Associated with cardiac malformations, polysplenia, situs inversus

Investigation
Conjugated bilirubin (prolonged physiological jaundice or breast milk jaundice will cause a rise in unconjugated bilirubin, whereas those with obstructive liver disease will have a rise in conjugated bilirubin)
Ultrasound of the liver (excludes extrahepatic causes, in biliary atresia infant may have tiny or invisible gallbladder)
Hepato-iminodiacetic acid radionuclide scan (good uptake but no excretion usually seen)

Management
Early recognition is important to prevent liver transplantation.
Nutritional support.
Roux-en-Y portojejunostomy (Kasai procedure).
If Kasai procedure fails or late recognition, a liver transplant becomes the only option.

1376
Q

An 86 year old retired pharmacist is admitted to Emergency Department following a fall. She complains of right hip pain. She is known to have hypertension and is currently on bendrofluazide. She lives alone and mobilises with a Zimmer frame. Her right leg is shortened and externally rotated. A hip x-ray confirms a displaced intracapsular fracture. What is the best management option?

Dynamic hip screw

Gamma nail

Total hip replacement

Hemiarthroplasty

Percutaneous pinning

A

Hemiarthroplasty is offered to older, less mobile individuals compared to fracture reduction and fixation in younger patients.

Hip fractures

Background
Neck of femur (NOF) fracture is a common orthopaedic presentation, with over 65000 fractures in the UK per year. Like many orthopaedic injuries, there is a bimodal age distribution. It is imperative to distinguish between the high energy injury in a young patient, and the low energy osteoporotic fracture in the elderly, as their management aims are very different:

Young patient - Usually high energy trauma (e.g road traffic accident, horse riding) and needs treating in accordance with Advanced Trauma Life Support (ATLS) principles. Will often have associated injuries. Aim is to retain the patients own anatomy, and optimise their function.

Elderly patient - Predominantly female, fall from standing height (fragility fracture). Often patients have multiple comorbidities that will ultimately dictate their prognosis. Aim of orthopaedic treatment is to immediately regain patient mobility so that morbidity (infection, thromboembolic events, pressure sores etc) and mortality associated with prolonged bed rest is avoided. Left untreated, a neck of femur fracture can be considered a terminal event. Historically, mortality associated with elderly hip fracture is 10% at one month, and 30% at one year. However, this has been improved in the UK with the introduction of multidisciplinary, orthogeriatric lead care and the National Hip Fracture Database and Best Practice Tariff.

Pertinent anatomy
Osteology - normal neck-shaft angle is 130 +/- 7 degrees, and 10 +/- 7 degrees of neck anteversion.
Vascular supply - The predominant blood supply to the femoral head and neck is from the medial and lateral femoral circumflex arteries (branches of profunda femoris). These anastomose and pierce the joint capsule at the base of the neck, mainly posteriorly. There is a small vascular contribution from the artery of the ligament teres. Understanding the blood supply is fundamental to the decision making process in treating NOF fractures.

Presentation and initial management
Typically, patients present with pain in the hip/groin, a shortened, abducted, externally rotated leg (due to the unopposed pull of the muscles that act across the hip joint) and the inability to straight-leg-raise. With undisplaced fractures, signs are more subtle.
High energy injuries should be treated in line with ATLS principles. All patients should be fluid resuscitated, have adequate pain relief (often with a fascio-iliiaca nerve block), and be optimised for surgery. In addition, elderly patients should be assessed by an orthogeriatrician.

Imaging
Anteroposterior and cross-table lateral plain radiographs are sufficient to diagnose the majority of NOF fractures. If the fracture extends below the level of the lesser trochanter, or there is any possibility of pathological fracture, full length femur views are essential to plan surgery.

Where there is a high index of suspicion of fracture, but plain radiographs are inconclusive, gold standard investigation is MRI. However, if unavailable within 24 hours, or if the patient will not tolerate MRI, CT is appropriate. The majority of fractures can be seen with modern CT techniques, and so this is becoming first line in many hospitals.

Classification
There has been a move away from named classification systems towards descriptive classification systems.
Two main types of NOF exist: Intra-capsular, and extra-capsular. Extra-capsular fractures are further divided into pertrochanteric or subtrochanteric (within 5cm distal to the lesser trochanter). All fractures are then described as undisplaced, minimally displaced, or displaced.
Femoral neck and head blood supply disruption is common with intracapsular NOF fractures, and rare with extracapsular fractures. This fundamental principle underpins the practise of arthroplasty for intracapsular fractures, and fixation for extracapsular fractures.

If you wish to use a named classification system, the most commonly used are below:
Elderly intracapsular - Garden Classification
Young intrasapsular - Pauvels Classification
Intertrochanteric - Evans
Subtrochanteric - Russell Taylor

Treatment
In general, NOF fractures are treated operatively except if the patient is deemed unlikely to survive an anaesthetic. Best Practice Tarif (BPT) dictates that surgery should happen within 36 hours, as delay of greater than 48 hours is associated with increased morbidity and mortality. Below are suggested algorithms for the treatment of NOF.

  • The priority with the young patient is to retain the femoral head if possible, even with a displaced intracapsular fracture. The risk of avascular necrosis and non-union (and therefore revision surgery) associated with internal fixation needs weighing up against the sequelae of total hip replacement in the young (wear, dislocation, revision). Discussion is necessary with the patient, on a case by case basis.

** Undisplaced fractures in the elderly can be treated with internal fixation, often with cannulated screws. This is appropriate for valgus impacted subcapital fractures which are inherently stable, to prevent secondary displacement. This does still carry the risk of AVN or non-union, and therefore a future revision. For this reason, many surgeons advocate arthroplasty as a single surgery.

*** NICE guidance - patients who fulfil these criteria should be offered total hip replacement which conveys better function and prosthetic survivorship, compared with hemiarthroplasty, but at an increased risk of dislocation.

  • Intertrochanteric fractures vary greatly in their stability. If the trochanter (and therefore lateral wall), and medial calcar is in tact, then the fracture configuration bears stability. This can be treated with a DHS, as collapse of the fracture is predictable. Where either or both structures are involved in the fracture, stability becomes compromised and many surgeons will favour using an intramedullary device. This is an ongoing debate, and difficult to test in an exam setting.

Post operative management
Patients should be mobilised fully weight bearing where possible. Care is multidisciplinary in its delivery. Elderly patients should have orthogeriatrician assessment of comorbidity, and bone health with secondary prevention measures if appropriate. There should be early involvement of physiotherapy and occupational therapy services. For further guidance see sources listed below.

NICE clinical guidance on hip fracture: https://www.nice.org.uk/guidance/cg124
Best Practice Tarif: www.nhfd.co.uk/20/hipfractureR…/Best%20Practice%20Tariff%20User%20Guide.pdf National Hip Fracture Database: www.nhfd.co.uk/

1377
Q

A 2 year old boy presents with a right renal mass. On examination, he has an irregular mass arising from the right flank and is hypertensive. A CT scan shows a non calcified irregular lesion affecting the apex of the right kidney and the right adrenal gland. What is the most likely diagnosis?

Neuroblastoma

Nephroblastoma

Renal cell carcinoma

Transitional cell carcinoma

Lymphoma

A

Wilm’s tumour of the kidney is the most common renal tumour in children. Both nephroblastoma and neuroblastoma may occupy the adrenal and apex of the kidney. In the case of neuroblastoma the lesion will have arisen from the adrenal, in the case of nephroblastoma the lesion will have arisen from the kidney. Hypertension is more commonly associated with nephroblastoma. Neuroblastomas are usually calcified, whereas nephroblastomas are not and this may be of diagnostic usefulness pre operatively.

Renal lesions

Lesion Disease specific features Treatment
Renal cell carcinoma
Most present with haematuria (50%)
Common renal tumour (85% cases)
Paraneoplastic features include hypertension and polycythaemia
Most commonly has haematogenous mestastasis
Usually radical or partial nephrectomy
Nephroblastoma
Rare childhood tumour
It accounts for 80% of all genitourinary malignancies in those under the age of 15 years
Up to 90% will have a mass
50% will be hypertensive
Diagnostic work up includes ultrasound and CT scanning
Surgical resection combined with chemotherapy (usually vincristine, actinomycin D and doxorubicin)
Neuroblastoma
Most common extracranial tumour of childhood
80% occur in those under 4 years of age
Tumour of neural crest origin (up to 50% occur in the adrenal gland)
The tumour is usually calcified and may be diagnosed using MIBG scanning
Staging is with CT
Surgical resection, radiotherapy and chemotherapy
Transitional cell carcinoma
Accounts for 90% of lower urinary tract tumours, but only 10% of renal tumours
Males affected 3x more than females
Occupational exposure to industrial dyes and rubber chemicals may increase risk
Up to 80% present with painless haematuria
Diagnosis and staging is with CT IVU
Radical nephroureterectomy
Angiomyolipoma
80% of these hamartoma type lesions occur sporadically, the remainder are seen in those with tuberous sclerosis
Tumour is composed of blood vessels, smooth muscle and fat
Massive bleeding may occur in 10% of cases
50% of patients with lesions >4cm will have symptoms and will require surgical resection

1378
Q

A 44 year old man has a malignant melanoma and is undergoing a block dissection of the groin. The femoral triangle is being explored for intra operative bleeding. Which of the following forms the medial border of the femoral triangle?

Femoral artery

Biceps femoris

Adductor longus

Sartorius

Adductor magnus

A

Adductor longus forms the medial boundary of the femoral triangle (see below).

Femoral triangle anatomy

Boundaries
Superiorly Inguinal ligament
Laterally Sartorius
Medially Adductor longus
Floor Iliopsoas, adductor longus and pectineus
Roof
Fascia lata and Superficial fascia
Superficial inguinal lymph nodes (palpable below the inguinal ligament)
Long saphenous vein

Contents
Femoral vein (medial to lateral)
Femoral artery-pulse palpated at the mid inguinal point
Femoral nerve
Deep and superficial inguinal lymph nodes
Lateral cutaneous nerve
Great saphenous vein
Femoral branch of the genitofemoral nerve

1379
Q

A 38 year old lady presents with a recent episode of renal colic. As part of her investigations the following results are obtained:
Corrected Calcium 3.84 mmol/l
PTH 88pg/ml (increased)
Her serum urea and electrolytes are normal.
What is the most likely diagnosis?

Carcinoma of the bronchus

Secondary hyperparathyroidism

Primary hyperparathyroidism

Tertiary hyperparathyroidism

Carcinoma of the breast

A

In this situation the most likely diagnosis is primary hyperparathyroidism. The question mentions that serum urea and electrolytes are normal, which makes tertiary hyperparathyroidism unlikely.

Primary hyperparathyroidism

In exams, primary hyperparathyroidism is stereotypically seen in elderly females with an unquenchable thirst and an inappropriately normal or raised parathyroid hormone level. It is most commonly due to a solitary adenoma

Causes of primary hyperparathyroidism
80%: solitary adenoma
15%: hyperplasia
4%: multiple adenoma
1%: carcinoma

Features - ‘bones, stones, abdominal groans and psychic moans’
Polydipsia, polyuria
Peptic ulceration/constipation/pancreatitis
Bone pain/fracture
Renal stones
Depression
Hypertension

Associations
Hypertension
Multiple endocrine neoplasia: MEN I and II

Investigations
Raised calcium, low phosphate
PTH may be raised or normal
Technetium-MIBI subtraction scan

Treatment
Parathyroidectomy, if imaging suggests target gland then a focused approach may be used

1380
Q

A 65 year old male with known nasopharyngeal carcinoma presents with double vision over a few weeks. On examination he is found to have left eye proptosis and it is down and out. He reports pain on attempting to move the eye. There is an absent corneal reflex. What is the most likely diagnosis?

Posterior communicating artery aneurysm

Cavernous sinus syndrome

Optic nerve tumour

Migraine

Cerebral metastases

A

Cavernous sinus syndrome is most commonly caused by cavernous sinus tumours. In this case, the nasopharyngeal malignancy has locally invaded the left cavernous sinus. Diagnosis is based on signs of pain, opthalmoplegia, proptosis, trigeminal nerve lesion (opthalmic branch) and Horner’s syndrome.

Cavernous sinus

The cavernous sinuses are paired and are situated on the body of the sphenoid bone. It runs from the superior orbital fissure to the petrous temporal bone.

Relations
Medial Lateral
Pituitary fossa
Sphenoid sinus Temporal lobe

Contents
Lateral wall components (from top to bottom:)
Oculomotor nerve
Trochlear nerve
Ophthalmic nerve
Maxillary nerve
Contents of the sinus (from medial to lateral:)
Internal carotid artery (and sympathetic plexus)
Abducens nerve

Blood supply
Ophthalmic vein, superficial cortical veins, basilar plexus of veins posteriorly.

Drains into the internal jugular vein via: the superior and inferior petrosal sinuses

1381
Q

Which of the blood tests listed below is most likely to be abnormal in a 33 year old lady who presents with a recently diagnosed goitre and a diagnosis of Hashimotos thyroiditis is suspected?

TSH receptor antibodies

Thyroid peroxidase antibodies

Thyroglobulin antibodies

Antibodies to parafollicular c cells

Calcitonin

A

Antibodies to thyroid peroxidase are found in most patients with Graves disease or Hashimotos thyroiditis.

Blood testing in thyroid disease

Assay Usage
Thyroid peroxidase (microsomal) antibodies
Found in autoimmune disease affecting the thyroid (Hashimotos 100%) and Graves (70%)
Antibodies to TSH receptor
Individuals with Graves disease (95%)
Thyroglobulin
Not useful for clinically distinguishing between different types of thyroid disease, may be used as part of thyroid cancer follow up
Calcitonin
Released from the parafollicular cells
Usually found in patients with medullary carcinoma of the thyroid

1382
Q

A 74 year old man is assessed as having oesophageal varices at endoscopy. What is the venous drainage of these varices?

Left gastric vein

Splenic vein

Portal vein

Hemiazygos vein

Renal vein

A

Remember that with impairment of flow through the portal system. The feeding vessels will be the gastric veins and drainage superiorly to the azygos system (i.e. reverse flow).

Oesophagus

  • 25cm long
    Starts at C6 vertebra, pierces diaphragm at T10 and ends at T11
    Squamous epithelium

Constrictions of the oesophagus
Structure Distance from incisors
Cricoid cartilage 15cm
Arch of the Aorta 22.5cm
Left principal bronchus 27cm
Diaphragmatic hiatus 40cm

Relations
Anteriorly
Trachea to T4
Recurrent laryngeal nerve
Left bronchus, Left atrium
Diaphragm
Posteriorly
Thoracic duct to left at T5
Hemiazygos to the left T8
Descending aorta
First 2 intercostal branches of aorta
Left
Thoracic duct
Left subclavian artery
Right
Azygos vein

Arterial, venous and lymphatic drainage of the oesophagus
Artery Vein Lymphatics Muscularis externa
Upper third Inferior thyroid Inferior thyroid Deep cervical Striated muscle
Mid third Aortic branches Azygos branches Mediastinal Smooth & striated muscle
Lower third Left gastric Left gastric Gastric Smooth muscle

Nerve supply
Upper half is supplied by recurrent laryngeal nerve
Lower half by oesophageal plexus (vagus)

Histology
Mucosa :Non-keratinized stratified squamous epithelium
Submucosa: glandular tissue
Muscularis externa (muscularis): composition varies. See table
Adventitia

1383
Q

A 52 year old woman with known Hashimotos thyroiditis presents with a neck swelling. She describes it as rapidly increasing in size over 3 months and she complains of dysphagia to solids. On examination, there is an asymmetrical swelling of the thyroid gland. What is the most likely diagnosis?

Anaplastic thyroid cancer

Follicular thyroid cancer

Papillary thyroid cancer

Lymphoma

Toxic adenoma

A

Thyroid lymphoma (Non Hodgkin’s B cell lymphoma) is rare. It should be considered in patients with a background of Hashimoto’s thyroiditis and a rapid growth in size of the thyroid gland. Diagnosis can be made with core needle biopsy; however an incisional biopsy may be needed. Radiotherapy is the main treatment option.

Thyroiditis

Sub acute thyroiditis
Subacute thyroiditis (also known as De Quervain’s thyroiditis) is thought to occur following viral infection and typically presents with hyperthyroidism

Features
Hyperthyroidism
Painful goitre
Raised ESR
Globally reduced uptake on iodine-131 scan

Management
Usually self-limiting - most patients do not require treatment
Thyroid pain may respond to aspirin or other NSAIDs
In more severe cases steroids are used, particularly if hypothyroidism develops

Hashimotos thyroiditis
Hashimotos thyroiditis is an immunological disorder in which lymphocytes become sensitised to thyroidal antigens. The three most important antibodies include; thyroglobulin, TPO and TSH-R. During the early phase of Hashimotos the the thyroglobulin antibody is markedly elevated and then declines.

Features
Goitre and either euthyroid or mild hypothyroidism
Progressive hypothyroidism (and associated symptoms)

Management
During the hyperthyroid phase of illness beta blockers may manage symptoms
As hypothyroidism develops patients may require thyroxine

1384
Q

A 56 year old man presents with his first attack of diverticulitis. Which of these complications is least likely to ensue?

Formation of colonic strictures

Malignant transformation

Development of colovesical fistula

Formation of a pericolic abscess

Formation of a phlegmon

A

Diverticulitis may result in a number of complications. However, whilst malignant disease may coincide with diverticulitis it is not, in itself, a risk factor for colonic cancer.

Diverticular disease

Diverticular disease is a common surgical problem. It consists of herniation of colonic mucosa through the muscular wall of the colon. The usual site is between the taenia coli where vessels pierce the muscle to supply the mucosa. For this reason, the rectum, which lacks taenia, is often spared.

Symptoms
Altered bowel habit
Bleeding
Abdominal pain

Complications
Diverticulitis
Haemorrhage
Development of fistula
Perforation and faecal peritonitis
Perforation and development of abscess
Development of diverticular phlegmon

Diagnosis
Patients presenting in clinic will typically undergo either a colonoscopy, CT cologram or barium enema as part of their diagnostic work up. All tests can identify diverticular disease. It can be far more difficult to confidently exclude cancer, particularly in diverticular strictures.

Acutely unwell surgical patients should be investigated in a systematic way. Plain abdominal films and an erect chest x-ray will identify perforation. An abdominal CT scan (not a CT cologram) with oral and intravenous contrast will help to identify whether acute inflammation is present but also the presence of local complications such as abscess formation.

Severity Classification- Hinchey
I Para-colonic abscess
II Pelvic abscess
III Purulent peritonitis
IV Faecal peritonitis

Treatment
Increase dietary fibre intake.
Mild attacks of diverticulitis may be managed conservatively with antibiotics.
Peri colonic abscesses should be drained either surgically or radiologically.
Recurrent episodes of acute diverticulitis requiring hospitalisation are a relative indication for a segmental resection.
Hinchey IV perforations (generalised faecal peritonitis) will require a resection and usually a stoma. This group have a very high risk of post operative complications and usually require HDU admission.

1385
Q

A 63 year old man has been on the intensive care unit for a week with adult respiratory distress syndrome complicating acute pancreatitis. He has required ventilation and is still being mechanically ventilated. What is the best option for maintenance of his airway?

Nasotracheal tube

Endotracheal tube

Tracheostomy

Guedel airway

Laryngeal mask

A

Tracheostomy is often used to facilitate long term weaning. The percutaneous devices are popular. These involve a seldinger type insertion of the tube. A second operator inserts a bronchoscope to ensure the device is not advanced through the posterior wall of the trachea. Complications include damage to adjacent structures and bleeding (contra indication in coagulopathy).
Airway management

Oropharyngeal airway
Easy to insert and use
No paralysis required
Ideal for very short procedures
Most often used as bridge to more definitive airway
Laryngeal mask
Widely used
Very easy to insert
Device sits in pharynx and aligns to cover the airway
Poor control against reflux of gastric contents
Paralysis not usually required
Commonly used for wide range of anaesthetic uses, especially in day surgery
Not suitable for high pressure ventilation (small amount of PEEP often possible)
Tracheostomy
Reduces the work of breathing (and dead space)
May be useful in slow weaning
Percutaneous tracheostomy widely used in ITU
Dries secretions, humidified air usually required
Endotracheal tube
Provides optimal control of the airway once cuff inflated
May be used for long or short term ventilation
Errors in insertion may result in oesophageal intubation (therefore end tidal CO2 usually measured)
Paralysis often required
Higher ventilation pressures can be used

1386
Q

A 23 year old man is admitted to hospital with diarrhoea and severe abdominal pain. He was previously well and his illness has lasted 18 hours. What is the likely cause?

Laxative abuse

Clostridium difficile
infection

Salmonella
gastroenteritis infection

Campylobacter jejuni
infection

Ulcerative colitis

A

Severe abdominal pain tends to favour Campylobacter infection.

Diarrhoea

World Health Organisation definitions
Diarrhoea: > 3 loose or watery stool per day
Acute diarrhoea < 14 days
Chronic diarrhoea > 14 days

Acute Diarrhoea
Gastroenteritis May be accompanied by abdominal pain or nausea/vomiting
Diverticulitis Classically causes left lower quadrant pain, diarrhoea and fever
Antibiotic therapy More common with broad spectrum antibiotics
Clostridium difficile is also seen with antibiotic use
Constipation causing overflow A history of alternating diarrhoea and constipation may be given
May lead to faecal incontinence in the elderly

Chronic Diarrhoea
Irritable bowel syndrome Extremely common. The most consistent features are abdominal pain, bloating and change in bowel habit. Patients may be divided into those with diarrhoea predominant IBS and those with constipation predominant IBS.
Features such as lethargy, nausea, backache and bladder symptoms may also be present
Ulcerative colitis Bloody diarrhoea may be seen. Crampy abdominal pain and weight loss are also common. Faecal urgency and tenesmus may occur
Crohn’s disease Crampy abdominal pains and diarrhoea. Bloody diarrhoea less common than in ulcerative colitis. Other features include malabsorption, mouth ulcers perianal disease and intestinal obstruction
Colorectal cancer Symptoms depend on the site of the lesion but include diarrhoea, rectal bleeding, anaemia and constitutional symptoms e.g. Weight loss and anorexia
Coeliac disease
In children may present with failure to thrive, diarrhoea and abdominal distension
In adults lethargy, anaemia, diarrhoea and weight loss are seen. Other autoimmune conditions may coexist

Other conditions associated with diarrhoea include:
Thyrotoxicosis
Laxative abuse
Appendicitis with pelvic abscess or pelvic appendix
Radiation enteritis

Diagnosis
Stool culture
Abdominal and digital rectal examination
Consider colonoscopy (radiological studies unhelpful)
Thyroid function tests, serum calcium, anti endomysial antibodies, glucose

1387
Q

A 2 month old baby is unwell and suspected to have discitis. Infection with which of these organisms is more likely?

Staphylococcus aureus

Brucella

Mycobacterium tuberculosis

Kigella kingae

Candida spp

A

At 2 months of age, s aureus is most likely. In children aged 6 months to 4 years, Kigella is the most likely infecting organism.

Discitis in children

In children, infectious discitis (D) and infectious spondylodiscitis (SD) are rare diseases that can cause significant clinical problems, including spinal deformities and segmental instabilities. Moreover, when the infection spreads into the spinal channel, D and SD can cause devastating neurologic complications. Early diagnosis and treatment may reduce these risks. Several different bacterial pathogens can cause D and SD, and, in some cases, particularly those due to Staphylococcus aureus, Kingella kingae, Mycobacterium tuberculosis, Brucella spp., the appropriate choice of drug is critical to achieve cure. In most cases the infections spreads to the site haematogenously from a separate site. Less commonly, it arises as a result of direct inoculation. A triphasic age distribution is described; the first peak incidence is in children aged only a few weeks or months old, the second in those between 6 months and the end of the preschool period, and the third in school-aged children.
Initial treatment is with broad spectrum intravenous antibiotics that cover s aureus, children aged 4 months to 6 years need cover for Kigella. More tailored therapy can be added later depending on progress and culture results.

1388
Q

Which of the following is responsible for the release and synthesis of calcitonin?

Parathyroid glands

Anterior pituitary

Thyroid gland

Posterior pituitary

Adrenal glands

A

Calcitonin has the opposite effect of PTH and is released from the thyroid gland.

Calcium homeostasis

Calcium ions are linked to a wide range of physiological processes. The largest store of bodily calcium is contained within the skeleton. Calcium levels are primarily controlled by parathyroid hormone, vitamin D and calcitonin.

Hormonal regulation of calcium
Hormone Actions
Parathyroid hormone (PTH)
Increase calcium levels and decrease phosphate levels
Increases bone resorption
Immediate action on osteoblasts to increase ca2+ in extracellular fluid
Osteoblasts produce a protein signaling molecule that activate osteoclasts which cause bone resorption
Increases renal tubular reabsorption of calcium
Increases synthesis of 1,25(OH)2D (active form of vitamin D) in the kidney which increases bowel absorption of Ca2+
Decreases renal phosphate reabsorption
1,25-dihydroxycholecalciferol (the active form of vitamin D)
Increases plasma calcium and plasma phosphate
Increases renal tubular reabsorption and gut absorption of calcium
Increases osteoclastic activity at high levels and osteoblasts at low levels
Increases renal phosphate reabsorption
Calcitonin
Secreted by C cells of thyroid
Inhibits intestinal calcium absorption
Inhibits osteoclast activity
Inhibits renal tubular absorption of calcium

Both growth hormone and thyroxine also play a small role in calcium metabolism.

1389
Q

A 23 year old man is shot in the chest during a robbery. The left lung is lacerated and is bleeding. An emergency thoracotomy is performed. The surgeons place a clamp over the hilum of the left lung. Which of the following structures lies most anteriorly at this level?

Vagus nerve

Oesophagus

Descending aorta

Phrenic nerve

Azygos vein

A

The phrenic nerve lies anteriorly at this point. The vagus passes anteriorly and then arches backwards immediately superior to the root of the left bronchus, giving off the recurrent laryngeal nerve as it does so.
Lung anatomy

The right lung is composed of 3 lobes divided by the oblique and transverse fissures. The left lung has two lobes divided by the oblique fissure.The apex of both lungs is approximately 4cm superior to the sterno-costal joint of the first rib. Immediately below this is a sulcus created by the subclavian artery.

Peripheral contact points of the lung
Base: diaphragm
Costal surface: corresponds to the cavity of the chest
Mediastinal surface: Contacts the mediastinal pleura. Has the cardiac impression. Above and behind this concavity is a triangular depression named the hilum, where the structures which form the root of the lung enter and leave the viscus. These structures are invested by pleura, which, below the hilum and behind the pericardial impression, forms the pulmonary ligament

Right lung
Above the hilum is the azygos vein; Superior to this is the groove for the superior vena cava and right innominate vein; behind this, and nearer the apex, is a furrow for the innominate artery. Behind the hilum and the attachment of the pulmonary ligament is a vertical groove for the oesophagus; In front and to the right of the lower part of the oesophageal groove is a deep concavity for the extrapericardiac portion of the inferior vena cava.

The root of the right lung lies behind the superior vena cava and the right atrium, and below the azygos vein.

The right main bronchus is shorter, wider and more vertical than the left main bronchus and therefore the route taken by most foreign bodies.

Left lung
Above the hilum is the furrow produced by the aortic arch, and then superiorly the groove accommodating the left subclavian artery; Behind the hilum and pulmonary ligament is a vertical groove produced by the descending aorta, and in front of this, near the base of the lung, is the lower part of the oesophagus.

The root of the left lung passes under the aortic arch and in front of the descending aorta.

Inferior borders of both lungs
6th rib in mid clavicular line
8th rib in mid axillary line
10th rib posteriorly
The pleura runs two ribs lower than the corresponding lung level.

Bronchopulmonary segments
Segment number Right lung Left lung
1 Apical Apical
2 Posterior Posterior
3 Anterior Anterior
4 Lateral Superior lingular
5 Medial Inferior lingular
6 Superior (apical) Superior (apical)
7 Medial basal Medial basal
8 Anterior basal Anterior basal
9 Lateral basal Lateral basal
10 Posterior basal Posterior basal

1390
Q

A 32 year old lady has previously undergone a wide local excision and axillary node clearance (5 nodes positive) for an invasive ductal carcinoma. It is oestrogen receptor negative, HER 2 positive, vascular invasion is present. She has a lesion suspicious for metastatic disease in the left lobe of her liver. Of the agents listed below, which is the most likely to be beneficial in this setting?

Cetuximab

Bevacizumab

Trastuzumab

Basiliximab

Imatinib

A

This lady’s young age, coupled with ER negativity and extensive nodal disease with suspicion of metastatic disease makes her a candidate for treatment with trastuzumab (herceptin).

Biological agents

Agents Target Uses
Adalimumab
Infliximab
Etanercept TNF alpha inhibitor Crohns disease
Rheumatoid disease
Bevacizumab Anti VEGF (anti angiogenic) Colorectal cancer
Renal
Glioblastoma
Trastuzumab HER receptor Breast cancer
Imatinib Tyrosine kinase inhibitor Gastrointestinal stromal tumours
Chronic myeloid leukaemia
Basiliximab IL2 binding site Renal transplants
Cetuximab Epidermal growth factor inhibitor EGF positive colorectal cancers

Detailed understanding of the actions of biological agents is well beyond the scope of the MRCS syllabus. However, many of these drugs are being frequently encountered in surgical patients.

1391
Q

A 23 year old lady presents with a posteriorly sited fissure in ano. Treatment with stool softeners and topical GTN has failed to improve matters. Which of the following would be the most appropriate next management step?

Lords procedure

Injection of botulinum toxin

Lateral internal sphincterotomy

Endoanal advancement flap

Surgical division of the external anal sphincter

A

The next most appropriate management option when GTN or other topical nitrates has failed is to consider botulinum toxin injection. In males a lateral internal sphincterotomy would be an acceptable alternative. In a female who has yet to conceive this may predispose to delayed increased risk of sphincter dysfunction. Division of the external sphincter will result in faecal incontinence and is not a justified treatment for fissure.
Rectal bleeding

Rectal bleeding is a common cause for patients to be referred to the surgical clinic. In the clinical history it is useful to try and localise the anatomical source of the blood. Bright red blood is usually of rectal anal canal origin, whilst dark red blood is more suggestive of a proximally sited bleeding source. Blood which has entered the GI tract from a gastro-duodenal source will typically resemble malaena due to the effects of the digestive enzymes on the blood itself.

In the table below we give some typical bleeding scenarios together with physical examination findings and causation.

Cause Type of bleeding Features in history Examination findings
Fissure in ano Bright red rectal bleeding Painful bleeding that occurs post defecation in small volumes. Usually antecedent features of constipation Muco-epithelial defect usually in the midline posteriorly (anterior fissures more likely to be due to underlying disease)
Haemorroids Bright red rectal bleeding Post defecation bleeding noted both on toilet paper and drips into pan. May be alteration of bowel habit and history of straining. No blood mixed with stool. No local pain. Normal colon and rectum. Proctoscopy may show internal haemorrhoids. Internal haemorrhoids are usually impalpable.
Crohns disease Bright red or mixed blood Bleeding that is accompanied by other symptoms such as altered bowel habit, malaise, history of fissures (especially anterior) and abscesses. Perineal inspection may show fissures or fistulae. Proctoscopy may demonstrate indurated mucosa and possibly strictures. Skip lesions may be noted at colonoscopy.
Ulcerative colitis Bright red bleeding often mixed with stool Diarrhoea, weight loss, nocturnal incontinence, passage of mucous PR. Proctitis is the most marked finding. Peri anal disease is usually absent. Colonoscopy will show continuous mucosal lesion.
Rectal cancer Bright red blood mixed volumes Alteration of bowel habit. Tenesmus may be present. Symptoms of metastatic disease. Usually obvious mucosal abnormality. Lesion may be fixed or mobile depending upon disease extent. Surrounding mucosa often normal, although polyps may be present.
Investigation
All patients presenting with rectal bleeding require digital rectal examination and procto-sigmoidoscopy as a minimal baseline.
Remember that haemorrhoids are typically impalpable and to attribute bleeding to these in the absence of accurate internal inspection is unsatisfactory.
In young patients with no other concerning features in the history a carefully performed sigmoidoscopy that demonstrates clear haemorrhoidal disease may be sufficient. If clear views cannot be obtained then patients require bowel preparation with an enema and a flexible sigmoidscopy performed.
In those presenting with features of altered bowel habit or suspicion of inflammatory bowel disease a colonoscopy is the best test.
Patients with excessive pain who are suspected of having a fissure may require an examination under general or local anaesthesia.
In young patients with external stigmata of fissure and a compatible history it is acceptable to treat medically and defer internal examination until the fissure is healed. If the fissure fails to heal then internal examination becomes necessary along the lines suggested above to exclude internal disease.

Special tests
In patients with a malignancy of the rectum the staging investigations comprise an MRI of the rectum to identify circumferential resection margin compromise and to identify mesorectal nodal disease. In addition to this CT scanning of the chest abdomen and pelvis is necessary to stage for more distant disease. Some centres will still stage the mesorectum with endo rectal ultrasound but this is becoming far less common.

Patients with fissure in ano who are being considered for surgical sphincterotomy and are females who have an obstetric history should probably have ano rectal manometry testing performed together with endo anal ultrasound. As this service is not universally available it is not mandatory but in the absence of such information there are continence issues that may arise following sphincterotomy.

Management

Disease Management
Fissure in ano GTN ointment 0.2% or diltiazem cream applied topically is the usual first line treatment. Botulinum toxin for those who fail to respond. Internal sphincterotomy for those who fail with botox, can be considered earlier in males.
Haemorroids Lifestyle advice, for small internal haemorrhoids can consider injection sclerotherapy or rubber band ligation. For external haemorrhoids consider haemorrhoidectomy. Modern options include HALO procedure and stapled haemorrhoidectomy.
Inflammatory bowel disease Medical management- although surgery may be needed for fistulating Crohns (setons).
Rectal cancer Anterior resection or abdomino-perineal excision of the colon and rectum. Total mesorectal excision is now standard of care. Most resections below the peritoneal reflection will require defunctioning ileostomy. Most patients will require preoperative radiotherapy.

1392
Q

A 33 year old lady is admitted with recurrent discharging fistula in ano. She is also known to have ano rectal Crohns disease. On examination, she is found to have a low anal fistula with involvement of a very small amount of the external anal sphincter muscle. What is the most appropriate course of action?

Insertion of a loose seton

Fistulotomy

Core fistulectomy

Core fistulectomy and advancement flap

Insertion of a cutting seton

A

Fistula in ano in patients with Crohns disease should be managed with insertion of seton.
In patients with IBD, management of fistula should be minimalistic and complex procedures best avoided. Laying open fistulas in this situation is likely to result in a chronic and non healing wound.

Anal fistula

Fistula in ano is the most common form of ano rectal sepsis. Fistulae will have both an internal opening and external opening, these will be connected by tract(s). Complexity arises because of the potential for multiple entry and exit sites, together with multiple tracts. Fistulae are classified into four main groups according to anatomical location and the degree of sphincter involvement. Simple uncomplicated fistulae are low and do not involve more than 30% of the external sphincter. Complex fistulae involve the sphincter, have multiple branches or are non cryptoglandular in origin[1]

Assessment
Examination of the perineum for signs of trauma, external openings or the stigmata of IBD is important. Digital rectal examination may reveal the cord linking the internal and external openings. At the same time the integrity of the sphincter mechanism can be assessed. Low, uncomplicated fistulas may not require any further assessment, other groups will usually require more detailed investigation. For the fistula, the use of endo-anal USS with instillation of hydrogen peroxide into the fistula tract may be helpful. Ano-rectal MRI scanning is also a useful tool, it is sensitive and specific for the identification of fistula anatomy, branching tracts and identifying occult sphincter involvement[2].

Identification of the internal opening
Fistulas with an external opening less than 3cm from the anal verge will typically obey Goodsalls rule

Therapies
Seton suture
A seton is a piece of material that is passed through the fistula between the internal and external openings that allows the drainage of sepsis. This is important as undrained septic foci may drain along the path of least resistance, which may result in the development of accessory tracts and openings. Their main use is in treating complex fistula. Two types of seton are recognised, simple and cutting. Simple setons lie within the fistula tract and encourage both drainage and fibrosis. A cutting seton is inserted and the skin incised. The suture is tightened and re-tightened at regular intervals. This may convert a high fistula to a low fistula. Since the tissue will scar surrounding the fistula it is hoped that this technique will minimise incontinence[3]. Unfortunately, a large retrospective review of the literature related to the use of cutting setons has found that they are associated with a 12% long term incontinence rate [4]

Fistulotomy
Low fistulas, that are simple should be treated by fistulotomy once the acute sepsis has been controlled. Fistulotomy (where safe) provides the highest healing rates [5]. Because fistulotomy is regarded as having a high cure rate, there are some who prefer to use this technique with more extensive sphincter involvement. In these patients the fistulotomy is performed as for a low fistula. However, the muscle that is encountered is then divided and reconstructed with an overlapping sphincter repair. A price is paid in terms of incontinence with this technique and up to 12.5% of patients who were continent pre-operatively will have issues relating to continence post procedure[6]. The same group also randomised between fistulotomy and sphincter reconstruction and ano-rectal advancement flaps for the treatment of complex cryptoglandular fistulas and reported similar outcomes in terms of recurrence (>90%) and disturbances to continence (20%)[7].
Other authors have found adverse outcomes following fistulotomy in patients who have undergone previous surgery, are of female gender or who have high internal openings [8], in these patients careful assessment of pre-operative sphincter function should be considered mandatory prior to fistulotomy.

Anal fistula plugs and fibrin glue
The desire to avoid injury to the sphincter complex has led to surgeons using both fibrin glue and plugs to try and improve fistula healing. Meticulous preparation of the tract and prior use of a draining seton is likely to improve chances of success.
The use of anal fistula plugs in high transphincteric fistula of cryptoglandular origin is to be discouraged because of the high incidence of non response in patients treated with such devices [9]In most patients septic complications are the reasons for failure [10]. Fibrin glue is a popular option for the treatment of fistula. There is variability of reported healing rates In some cases initial success rates of up to 50% healing at six months are reported (in patients with complex cryptogenic fistula). Of these successes 25% suffer a long term recurrence of fistula [11]. There are, however, no obvious cases of damage to the sphincter complex and the use of the devices does not appear to adversely impact on subsequent surgical options.

Ano-rectal advancement flaps
This procedure is primarily directed at high fistulae, and is considered attractive as a sphincter saving operation. The procedure is performed either with the patient in the prone jack knife position or in lithotomy (depending upon the site of the fistula). The dissection is commenced in the sub mucosal plane (which may be infiltrated with dilute adrenaline solution to ease dissection). The dissection is continued into healthy proximal tissue. This is brought down and sutured over the defect.
Follow up of patients with cryptoglandular fistulas treated with advancement flaps shows a success in up to 80% patients[12-14]. With most recurrences occurring in the first 6 months following surgery[12]. Continence was affected in some patients, with up to 10% describing major continence issues post operatively.

Ligation of the intersphincteric tract procedure
In this procedure an incision is made in the intersphincteric groove and the fistula tract dissected out in this plane and divided. A greater than 90% cure rate within 4 weeks was initially reported[15]. Others have subsequently performed similar studies on larger numbers of patients with similar success rates.

Fistulotomy at the time of abscess drainage?
A Cochrane review conducted in 2010 suggests that primary fistulotomy for low, uncomplicated fistula in ano may be safe and associated with better outcomes in relation to long term chronic sepsis[16]. However, there is a danger that such surgery performed by non specialists may result in a higher complication rate and therefore the traditional teaching is that primary treatment of acute sepsis is incision and drainage only. All agree that high/ complex fistulae should never be subject to primary fistulotomy in the acute setting.

References
1. Parks, A.G., P.H. Gordon, and J.D. Hardcastle, A classification of fistula-in-ano. Br J Surg, 1976. 63(1): p. 1-12.
2. Lunniss, P.J., et al., Magnetic resonance imaging of fistula-in-ano. Dis Colon Rectum, 1994. 37(7): p. 708-18.
3. Misra, M.C. and B.M. Kapur, A new non-operative approach to fistula in ano. Br J Surg, 1988. 75(11): p. 1093-4.
4. Ritchie, R.D., J.M. Sackier, and J.P. Hodde, Incontinence rates after cutting seton treatment for anal fistula. Colorectal Dis, 2009. 11(6): p. 564-71.
5. Tyler, K.M., C.B. Aarons, and S.M. Sentovich, Successful sphincter-sparing surgery for all anal fistulas. Dis Colon Rectum, 2007. 50(10): p. 1535-9.
6. Perez, F., et al., Prospective clinical and manometric study of fistulotomy with primary sphincter reconstruction in the management of recurrent complex fistula-in-ano. Int J Colorectal Dis, 2006. 21(6): p. 522-6.
7. Perez, F., et al., Randomized clinical and manometric study of advancement flap versus fistulotomy with sphincter reconstruction in the management of complex fistula-in-ano. Am J Surg, 2006. 192(1): p. 34-40.
8. Garcia-Aguilar, J., et al., Anal fistula surgery. Factors associated with recurrence and incontinence. Dis Colon Rectum, 1996. 39(7): p. 723-9.
9. Ortiz, H., et al., Randomized clinical trial of anal fistula plug versus endorectal advancement flap for the treatment of high cryptoglandular fistula in ano. Br J Surg, 2009. 96(6): p. 608-12.
10. El-Gazzaz, G., M. Zutshi, and T. Hull, A retrospective review of chronic anal fistulae treated by anal fistulae plug. Colorectal Dis, 2010. 12(5): p. 442-7.
11. Haim, N., et al., Long-term results of fibrin glue treatment for cryptogenic perianal fistulas: a multicenter study. Dis Colon Rectum, 2011. 54(10): p. 1279-83.
12. Ortiz, H., et al., Length of follow-up after fistulotomy and fistulectomy associated with endorectal advancement flap repair for fistula in ano. Br J Surg, 2008. 95(4): p. 484-7.
13. Kodner, I.J., et al., Endorectal advancement flap repair of rectovaginal and other complicated anorectal fistulas. Surgery, 1993. 114(4): p. 682-9; discussion 689-90.
14. Abbas, M.A., R. Lemus-Rangel, and A. Hamadani, Long-term outcome of endorectal advancement flap for complex anorectal fistulae. Am Surg, 2008. 74(10): p. 921-4.
15. Rojanasakul, A., et al., Total anal sphincter saving technique for fistula-in-ano; the ligation of intersphincteric fistula tract. J Med Assoc Thai, 2007. 90(3): p. 581-6.
16. Malik, A.I., R.L. Nelson, and S. Tou, Incision and drainage of perianal abscess with or without treatment of anal fistula. Cochrane Database Syst Rev, 2010(7): p. CD006827.

1393
Q

Which operation below is an amputation of the lower limb in which the femoral condyles are removed and the patella retained?

Transfemoral amputation

Gritti Stokes amputation

Symes amputation

Below knee amputation with Burgess flap

Below knee amputation with Skew flap

A

This is a Gritti - Stokes amputation. During a Gritti - Stokes operation the patella is conserved and swung posteriorly to cover the distal femoral surface.

Amputations

Amputations are indicated when the affected limb is one of the following:
Dead non viable
Deadly where it is posing a major threat to life
Dead useless where it is viable but a prosthesis would be preferable

Orthopaedic surgery
Amputation is often undertaken as an option of last resort e.g. Limb salvage has failed and the limb is so non functional that mobility needs would be best met with prosthesis.
Chronic fracture non union or significant limb shortening following trauma would fit into this category. Occasionally following major trauma a primary amputation is preferable. This would be the case in an open fracture with major distal neurovascular compromise and other more life threatening injuries are present.

Vascular surgery
The first two categories are the most prevalent.
Diabetic foot sepsis is often a major cause of sepsis which can spread rapidly in the presence of established peripheral vascular disease.
As a general rule the main issue in vascular surgery is to optimise vascular inflow prior to surgery. The more distal the planned amputation is to be, the more important this rule becomes.
In other situations there has been something such as an embolic event that has not been revascularised in time. In this case the limb shows fixed mottling and an amputation will be needed.

Types of amputations
As the vast majority of commonly performed amputations affect the lower limbs these will be covered here.

The main categories of amputations are:
Pelvic disarticulation (hindquarter)
Above knee amputation
Gritti Stokes (through knee amputation)
Below knee amputation (using either Skew or Burgess flaps)
Syme’s amputation (through ankle)
Amputations of mid foot and digits

Choosing a level of amputation depends on:
The disease process being treated
Desired functional outcome
Co-morbidities of the patient

Above knee amputations
Quick to perform
Heal reliably
Patients regain their general health quickly
For this benefit, a functional price has to be paid and many patients over the age of 70 will never walk on an above knee prosthesis.
Above knee amputations use equal anterior-posterior flaps

Below knee amputations
Technically more challenging to perform
Heal less reliably than their above knee counterparts.
However, many more patients are able to walk using a below knee prosthesis.
In below knee amputations the two main flaps are Skew flaps or the Burgess long posterior flap. Skew flaps result in a less bulky limb that is easier to attach a prosthesis to.

It is worth remembering that whilst it may be technically feasible to offer a below knee amputation there may be circumstances where an above knee option is preferable. For example, in fixed flexion deformities of the lower limb, little functional benefit would be gained from below knee amputation surgery.

1394
Q

A 72 year old retired teacher is admitted to A&E with a fall and hip pain. He is normally fit and well. He lives with his son in a detached, 2 storey house. A hip x-ray and femur views confirm a sub trochanteric fracture. What is the best course of action?

Insertion of intra medullary nail

Total hip replacement

Uncemented hemiarthroplasty

Percutaneous pinning

Hip arthrodesis

A

Intramedullary devices are normally recommended for reverse oblique, transverse subtrochanteric fractures.
Hip fractures

Background
Neck of femur (NOF) fracture is a common orthopaedic presentation, with over 65000 fractures in the UK per year. Like many orthopaedic injuries, there is a bimodal age distribution. It is imperative to distinguish between the high energy injury in a young patient, and the low energy osteoporotic fracture in the elderly, as their management aims are very different:

Young patient - Usually high energy trauma (e.g road traffic accident, horse riding) and needs treating in accordance with Advanced Trauma Life Support (ATLS) principles. Will often have associated injuries. Aim is to retain the patients own anatomy, and optimise their function.

Elderly patient - Predominantly female, fall from standing height (fragility fracture). Often patients have multiple comorbidities that will ultimately dictate their prognosis. Aim of orthopaedic treatment is to immediately regain patient mobility so that morbidity (infection, thromboembolic events, pressure sores etc) and mortality associated with prolonged bed rest is avoided. Left untreated, a neck of femur fracture can be considered a terminal event. Historically, mortality associated with elderly hip fracture is 10% at one month, and 30% at one year. However, this has been improved in the UK with the introduction of multidisciplinary, orthogeriatric lead care and the National Hip Fracture Database and Best Practice Tariff.

Pertinent anatomy
Osteology - normal neck-shaft angle is 130 +/- 7 degrees, and 10 +/- 7 degrees of neck anteversion.
Vascular supply - The predominant blood supply to the femoral head and neck is from the medial and lateral femoral circumflex arteries (branches of profunda femoris). These anastomose and pierce the joint capsule at the base of the neck, mainly posteriorly. There is a small vascular contribution from the artery of the ligament teres. Understanding the blood supply is fundamental to the decision making process in treating NOF fractures.

Presentation and initial management
Typically, patients present with pain in the hip/groin, a shortened, abducted, externally rotated leg (due to the unopposed pull of the muscles that act across the hip joint) and the inability to straight-leg-raise. With undisplaced fractures, signs are more subtle.
High energy injuries should be treated in line with ATLS principles. All patients should be fluid resuscitated, have adequate pain relief (often with a fascio-iliiaca nerve block), and be optimised for surgery. In addition, elderly patients should be assessed by an orthogeriatrician.

Imaging
Anteroposterior and cross-table lateral plain radiographs are sufficient to diagnose the majority of NOF fractures. If the fracture extends below the level of the lesser trochanter, or there is any possibility of pathological fracture, full length femur views are essential to plan surgery.

Where there is a high index of suspicion of fracture, but plain radiographs are inconclusive, gold standard investigation is MRI. However, if unavailable within 24 hours, or if the patient will not tolerate MRI, CT is appropriate. The majority of fractures can be seen with modern CT techniques, and so this is becoming first line in many hospitals.

Classification
There has been a move away from named classification systems towards descriptive classification systems.
Two main types of NOF exist: Intra-capsular, and extra-capsular. Extra-capsular fractures are further divided into pertrochanteric or subtrochanteric (within 5cm distal to the lesser trochanter). All fractures are then described as undisplaced, minimally displaced, or displaced.
Femoral neck and head blood supply disruption is common with intracapsular NOF fractures, and rare with extracapsular fractures. This fundamental principle underpins the practise of arthroplasty for intracapsular fractures, and fixation for extracapsular fractures.

If you wish to use a named classification system, the most commonly used are below:
Elderly intracapsular - Garden Classification
Young intrasapsular - Pauvels Classification
Intertrochanteric - Evans
Subtrochanteric - Russell Taylor

Treatment
In general, NOF fractures are treated operatively except if the patient is deemed unlikely to survive an anaesthetic. Best Practice Tarif (BPT) dictates that surgery should happen within 36 hours, as delay of greater than 48 hours is associated with increased morbidity and mortality. Below are suggested algorithms for the treatment of NOF. There are some areas of debate/controversy which are detailed below.

  • The priority with the young patient is to retain the femoral head if possible, even with a displaced intracapsular fracture. The risk of avascular necrosis and non-union (and therefore revision surgery) associated with internal fixation needs weighing up against the sequelae of total hip replacement in the young (wear, dislocation, revision). Discussion is necessary with the patient, on a case by case basis.

** Undisplaced fractures in the elderly can be treated with internal fixation, often with cannulated screws. This is appropriate for valgus impacted subcapital fractures which are inherently stable, to prevent secondary displacement. This does still carry the risk of AVN or non-union, and therefore a future revision. For this reason, many surgeons advocate arthroplasty as a single surgery.

*** NICE guidance - patients who fulfil these criteria should be offered total hip replacement which conveys better function and prosthetic survivorship, compared with hemiarthroplasty, but at an increased risk of dislocation.

  • Intertrochanteric fractures vary greatly in their stability. If the trochanter (and therefore lateral wall), and medial calcar is in tact, then the fracture configuration bears stability. This can be treated with a DHS, as collapse of the fracture is predictable. Where either or both structures are involved in the fracture, stability becomes compromised and many surgeons will favour using an intramedullary device. This is an ongoing debate, and difficult to test in an exam setting.

Post operative management
Patients should be mobilised fully weight bearing where possible. Care is multidisciplinary in its delivery. Elderly patients should have orthogeriatrician assessment of comorbidity, and bone health with secondary prevention measures if appropriate. There should be early involvement of physiotherapy and occupational therapy services. For further guidance see sources listed below.

NICE clinical guidance on hip fracture: https://www.nice.org.uk/guidance/cg124
Best Practice Tarif: www.nhfd.co.uk/20/hipfractureR…/Best%20Practice%20Tariff%20User%20Guide.pdf National Hip Fracture Database: www.nhfd.co.uk/

1395
Q

A 30 year old man presents with back pain and the surgeon tests the ankle reflex. Which of the following nerve roots are tested in this manoeuvre?

S3 and S4

L4 and L5

L3 and L4

S1 and S2

S4 only

A

Ankle reflex

The ankle reflex is elicited by tapping the Achilles tendon with a tendon hammer. It tests the S1 and S2 nerve roots. It is typically delayed in L5 and S1 disk prolapses.

1396
Q

The Cori cycle is important in lactate metabolism in the septic surgical patient. It is used to describe a pathway in which glucose is metabolised anaerobically to lactate in one tissue and the lactate is converted back to glucose in another. Which one of the following relies on this cycle to meet all of its energy needs?

Hepatocyte

Leucocyte

Erythrocyte

Pneumocyte

Goblet cells

A

Erythrocytes lack a mitochondria and therefore they generate energy via glycolytic pathways only. All the other cell types have mitochondria and will therefore use the Krebs cycle unless true anaerobic conditions prevail.

Cellular metabolism

  • Circulating glucose enters the cell, subsequently a glycolytic process results in the generation of ATP and pyruvate
    In the presence of oxygen, the pyruvate from the glycolytic process then enters the Krebs cycle
    When oxygen is limited or absent, pyruvate enters an anaerobic pathway. In these reactions, pyruvate can be converted into lactic acid. In addition to generating an additional ATP, this pathway serves to keep the pyruvate concentration low so glycolysis continues, and it oxidizes NADH into the NAD+ needed by glycolysis. In this reaction, lactic acid replaces oxygen as the final electron acceptor. Anaerobic respiration occurs in most cells of the body when oxygen is limited or mitochondria are absent or nonfunctional. For example, because erythrocytes (red blood cells) lack mitochondria, they must produce their ATP from anaerobic respiration. This is an effective pathway of ATP production for short periods of time, ranging from seconds to a few minutes. The lactic acid produced diffuses into the plasma and is carried to the liver, where it is converted back into pyruvate or glucose via the Cori cycle
    In the presence of oxygen, pyruvate can enter the Krebs cycle where additional energy is extracted as electrons are transferred from the pyruvate to the receptors NAD+, GDP, and FAD, with carbon dioxide being a by product. The NADH and FADH2 pass electrons on to the electron transport chain, which uses the transferred energy to produce ATP. As the terminal step in the electron transport chain, oxygen is the terminal electron acceptor and creates water inside the mitochondria.
    The oxidative pathways eventually yield a total of 36 ATP molecules and are therefore far better at generating energy than anaerobic pathways.
1397
Q

A 30 year old man is rescued from a house fire. On examination, his pulse rate is 80bpm and he is normotensive. His oxygen saturations are 96% on air. and respiratory rate is 14. He has some areas of superficial thickness burns that amount to no more than 2% body surface area. His nasal hairs are noted to be burnt and there is some soot visible in the oropharynx. What is the best course of action?

Admit for observation

Transfer to a burns unit

Admit to high dependency care unit for observation and invasive monitoring

Intubate the patient

Administer nebulised steroids and high flow nasal oxygen

A

Airway injuries can develop marked laryngeal swelling and early intubation is essential to protect the airway.

Burns

Burns may be thermal, chemical or electrical. In the former category are burns which occur as a result of heat. Chemical burns occur when the skin is exposed to an extremely caustic or alkaline substance. Electrical burns occur following exposure to electrical current. The immediate management includes removal of the burning source which usually includes irrigation of the burned area. A detailed assessment then needs to be made of the extent of the burns and a number of charts are available for recording this information. The degree of injury relates to the temperature and duration of exposure. Most domestic burns are mainly scalds in young children.

Following the burn, there is a local response with progressive tissue loss and release of inflammatory cytokines. Systemically, there are cardiovascular effects resulting from fluid loss and sequestration of fluid into the third space. There is a marked catabolic response. Immunosuppression is common with large burns and bacterial translocation from the gut lumen is a recognised event. Sepsis is a common cause of death following major burns.

Types of burn

Type of burn Skin layers affected Skin appearance Blanching Management
Epidermal/Superficial Epidermis Red, moist Yes
Superficial partial thickness Epidermis and part of papillary dermis affected Pale, dry Yes Normally heals with no intervention
Deep partial thickness Epidermis, whole papillary dermis affected Mottled red colour No Needs surgical intervention (depending on site)
Full thickness Whole skin layer and subcutaneous tissue affected Dry, leathery hard wound No Burns centre

Depth of burn assessment
Bleeding on needle prick
Sensation
Appearance
Blanching to pressure

Percentage burn estimation
Lund Browder chart: most accurate even in children
Wallace rule of nines
Palmar surface: surface area palm = 0.8% burn

> 15% body surface area burns in adults needs urgent burn fluid resuscitation

Transfer to burn centre if:
Need burn shock resuscitation
Face/hands/genitals affected
Deep partial thickness or full thickness burns
Significant electrical/chemical burns

Management
The initial aim is to stop the burning process and resuscitate the patient. Intravenous fluids will be required for children with burns greater than 10% of total body surface area. Adults with burns greater than 15% of total body surface area will also require IV fluids. The fluids are calculated using the Parkland formula which is; volume of fluid= total body surface area of the burn % x weight (Kg) x2-4 (preference for lower amount i.e. 2ml to avoid excessive fluid overload). Half of the fluid is administered in the first 8 hours. A urinary catheter should be inserted. Analgesia should be given. Complex burns, burns involving the hand perineum and face and burns >10% in adults and >5% in children should be transferred to a burns unit.

Circumferential burns affecting a limb or severe torso burns impeding respiration may require escharotomy to divide the burnt tissue.

Conservative management is appropriate for superficial burns and mixed superficial burns that will heal in 2 weeks. More complex burns may require excision and skin grafting. Excision and primary closure is not generally practised as there is a high risk of infection.

There is no evidence to support the use of anti microbial prophylaxis or topical antibiotics in burn patients.

Escharotomies
Indicated in circumferential full thickness burns to the torso or limbs.
Careful division of the encasing band of burn tissue will potentially improve ventilation (if the burn involves the torso), or relieve compartment syndrome and oedema (where a limb is involved)

References
www.euroburn.org/e107files/downloads/guidelinesburncare.pdf

Barajas-Nava LA, López-Alcalde J, Roqué i Figuls M, Solà I, Bonfill Cosp X. Antibiotic prophylaxis for preventing burn wound infection. Cochrane Database of Systematic Reviews 2013, Issue 6. Art. No.: CD008738. DOI: 10.1002/14651858.CD008738.pub2.

Hettiaratchy S & Papini R. Initial management of a major burn: assessment and resuscitation. BMJ 2004;329:101-103

1398
Q

A 23 year old man presents to the surgical clinic with an inguinal hernia. On examination he has a small direct hernia. However, you also notice that he has pigmented spots around his mouth, on his palms and soles. In his history he underwent a reduction of an intussusception aged 12 years. Which of the following lesions is most likely to be identified if a colonoscopy were performed?

Hamartomas

Tubulovillous adenoma

Colorectal cancer

Crohns disease

Hyperplastic polyps

A

He is most likely to have Peutz-Jeghers syndrome which is associated with Hamartomas.

Peutz-Jeghers syndrome

Peutz-Jeghers syndrome is an autosomal dominant condition characterised by numerous benign hamartomatous polyps in the gastrointestinal tract. It is also associated with pigmented freckles on the lips, face, palms and soles. Around 50% of patients will have died from a gastrointestinal tract cancer by the age of 60 years.

Genetics
Autosomal dominant
Responsible gene encodes serine threonine kinase LKB1 or STK11

Features
Hamartomatous polyps in GI tract (mainly small bowel)
Pigmented lesions on lips, oral mucosa, face, palms and soles
Intestinal obstruction e.g. intussusception (which may lead to diagnosis)
Gastrointestinal bleeding

Management
Conservative unless complications develop

1399
Q

A 53 year old man presents with a full thickness external rectal prolapse. Which of the following procedures would be the most suitable surgical option?

Rectopexy

Delormes

Altmeirs

Thirsch tape

Abdomino-perineal excision of the rectum

A

As this man is relatively young and has full thickness prolapse a rectopexy is the most appropriate procedure. It will give the lowest recurrence rates. This could be a sutured rectopexy or ventral mesh rectopexy. If the latter procedure is adopted, it is important to counsel the patient around the risks of sexual dysfunction (close to prostate).

Rectal prolapse

Rectal prolapse may be divided into internal and external prolapse. Patients with the former condition may have internal intussceception of the rectum and present with constipation, obstructed defecation and occasionally faecal incontinence. Patients with external rectal prolapse have a full thickness external protrusion of the rectum. Risk factors for the condition include multiparity, pelvic floor trauma and connective tissue disorders.

Diagnosis
External prolapse is usually evident. Internal prolapse may be identified by defecating proctography and examination under anaesthesia.
Sinister pathology should be excluded with endoscopy

Treatment
Perineal approaches include the Delormes operation, this avoids resection and is relatively safe but is associated with high recurrence rates. An Altmeirs operation involves a perineal excision of the sigmoid colon and rectum, it may be a more effective procedure than a Delormes but carries the risk of anastomotic leak.
Rectopexy - this is an abdominal procedure. The rectum is mobilised and fixed onto the sacral promontary. A prosthetic mesh may be inserted. The recurrence rates are low and the procedure is well tolerated (particularly if performed laparoscopically). Risks with ventral mesh rectopexy include chronic pain and visceral mesh erosions.
Thirsch tape- this is a largely historical procedure and involves encircling the rectum with tape or wire. It may be of use in a palliative setting.

1400
Q

A 46-year-old woman is referred to endocrine surgery for a possible thyroidectomy. She has a tender neck swelling. Blood results are as follows:

TSH <0.1 mU/l
T4 188 nmol/l

Hb 14.2 g/dl
Plt 377 * 10^9/l
WBC 6.4 * 10^9/l

ESR 65 mm/hr

Technetium thyroid scan shows decreased uptake globally

What is the most likely diagnosis?

Sick thyroid syndrome

Acute bacterial thyroiditis

Hashimoto’s thyroiditis

Subacute thyroiditis

Toxic multinodular goitre

A

This patient does not need surgery! Subacute thyroiditis is suggested by the tender goitre, hyperthyroidism and raised ESR. The globally reduced uptake on technetium thyroid scan is also typical. This should resolve without any active intervention.

Thyroiditis

Sub acute thyroiditis
Subacute thyroiditis (also known as De Quervain’s thyroiditis) is thought to occur following viral infection and typically presents with hyperthyroidism

Features
Hyperthyroidism
Painful goitre
Raised ESR
Globally reduced uptake on iodine-131 scan

Management
Usually self-limiting - most patients do not require treatment
Thyroid pain may respond to aspirin or other NSAIDs
In more severe cases steroids are used, particularly if hypothyroidism develops

Hashimotos thyroiditis
Hashimotos thyroiditis is an immunological disorder in which lymphocytes become sensitised to thyroidal antigens. The three most important antibodies include; thyroglobulin, TPO and TSH-R. During the early phase of Hashimotos the the thyroglobulin antibody is markedly elevated and then declines.

Features
Goitre and either euthyroid or mild hypothyroidism
Progressive hypothyroidism (and associated symptoms)

Management
During the hyperthyroid phase of illness beta blockers may manage symptoms
As hypothyroidism develops patients may require thyroxine

1401
Q

An athletic 15 year old boy presents with knee pain of 3 weeks duration. It is worst during activity and settles with rest. On examination, there is tenderness overlying the tibial tuberosity and an associated swelling at this site. What is the diagnosis?

Chondromalacia patellae

Avulsion fracture of the tibial tubercle

Osgood Schlatters disease

Quadriceps tendon rupture

Undisplaced fracture patella

A

Athletic boys and girls may develop this condition in their teenage years. It is caused by multiple micro fractures at the point of insertion of the tendon into the tibial tuberosity. Most cases settle with physiotherapy and rest.

Knee injury

Types of injury

Ruptured anterior cruciate ligament
Sport injury
Mechanism: high twisting force applied to a bent knee
Typically presents with: loud crack, pain and RAPID joint swelling (haemoarthrosis)
Poor healing
Management: intense physiotherapy or surgery
Ruptured posterior cruciate ligament
Mechanism: hyperextension injuries
Tibia lies back on the femur
Paradoxical anterior draw test
Rupture of medial collateral ligament
Mechanism: leg forced into valgus via force outside the leg
Knee unstable when put into valgus position
Menisceal tear
Rotational sporting injuries
Delayed knee swelling
Joint locking (Patient may develop skills to ‘unlock’ the knee
Recurrent episodes of pain and effusions are common, often following minor trauma
Chondromalacia patellae
Teenage girls, following an injury to knee e.g. Dislocation patella
Typical history of pain on going downstairs or at rest
Tenderness, quadriceps wasting
Dislocation of the patella
Most commonly occurs as a traumatic primary event, either through direct trauma or through severe contraction of quadriceps with knee stretched in valgus and external rotation
Genu valgum, tibial torsion and high riding patella are risk factors
Skyline x-ray views of patella are required, although displaced patella may be clinically obvious
An osteochondral fracture is present in 5%
The condition has a 20% recurrence rate
Fractured patella
2 types:
i. Direct blow to patella causing undisplaced fragments
ii. Avulsion fracture
Tibial plateau fracture
Occur in the elderly (or following significant trauma in young)
Mechanism: knee forced into valgus or varus, but the knee fractures before the ligaments rupture
Varus injury affects medial plateau and if valgus injury, lateral plateau depressed fracture occurs
Classified using the Schatzker system (see below)

Schatzker Classification system for tibial plateau fractures
Type Anatomical description Features
1 Vertical split of lateral condyle Fracture through dense bone, usually in the young. It may be virtually undisplaced, or the condylar fragment may be pushed inferiorly and tilted
2 Vertical split of the lateral condyle combined with an adjacent load bearing part of the condyle The wedge fragment (which may be of variable size), is displaced laterally; the joint is widened. Untreated, a valgus deformity may develop
3 Depression of the articular surface with intact condylar rim The split does not extend to the edge of the plateau. Depressed fragments may be firmly embedded in subchondral bone, the joint is stable
4 Fragment of the medial tibial condyle Two injuries are seen in this category; (1) a depressed fracture of osteoporotic bone in the elderly. (2) a high energy fracture resulting in a condylar split that runs from the intercondylar eminence to the medial cortex. Associated ligamentous injury may be severe
5 Fracture of both condyles Both condyles fractured but the column of the metaphysis remains in continuity with the tibial shaft
6 Combined condylar and subcondylar fractures High energy fracture with marked comminution

1402
Q

A 20 year old man is hit with a hammer on the right side of the head. He dies on arrival in the emergency department. Which of these features is most likely to be found at post mortem?

Hydrocephalus

Supra tentorial herniation

Laceration of the middle meningeal artery

Sub dural haematoma

Posterior fossa haematoma

A

C-Laceration of the middle meningeal artery
This will account for the scenario given where there is a brief delay prior to death. The other options are less acute and a supratentorial herniation would not occur in this setting.
Head injury

Patients who suffer head injuries should be managed according to ATLS principles and extra cranial injuries should be managed alongside cranial trauma. Inadequate cardiac output will compromise CNS perfusion irrespective of the nature of the cranial injury.

Types of traumatic brain injury
Extradural haematoma Bleeding into the space between the dura mater and the skull. Often results from acceleration-deceleration trauma or a blow to the side of the head. The majority of extradural haematomas occur in the temporal region where skull fractures cause a rupture of the middle meningeal artery.

Features
Raised intracranial pressure
Some patients may exhibit a lucid interval
Subdural haematoma Bleeding into the outermost meningeal layer. Most commonly occur around the frontal and parietal lobes. May be either acute or chronic.

Risk factors include old age and alcoholism.

Slower onset of symptoms than a extradural haematoma.
Subarachnoid haemorrhage Usually occurs spontaneously in the context of a ruptured cerebral aneurysm, but may be seen in association with other injuries when a patient has sustained a traumatic brain injury.

Pathophysiology
Primary brain injury may be focal (contusion/ haematoma) or diffuse (diffuse axonal injury)
Diffuse axonal injury occurs as a result of mechanical shearing following deceleration, causing disruption and tearing of axons
Intra-cranial haematomas can be extradural, subdural or intracerebral, while contusions may occur adjacent to (coup) or contralateral (contre-coup) to the side of impact
Secondary brain injury occurs when cerebral oedema, ischaemia, infection, tonsillar or tentorial herniation exacerbates the original injury. The normal cerebral auto regulatory processes are disrupted following trauma rendering the brain more susceptible to blood flow changes and hypoxia
The Cushings reflex (hypertension and bradycardia) often occurs late and is usually a pre terminal event

Management
Where there is life threatening rising ICP such as in extra dural haematoma and whilst theatre is prepared or transfer arranged use of IV mannitol/ frusemide may be required.
Diffuse cerebral oedema may require decompressive craniotomy
Exploratory Burr Holes have little management in modern practice except where scanning may be unavailable and to thus facilitate creation of formal craniotomy flap
Depressed skull fractures that are open require formal surgical reduction and debridement, closed injuries may be managed non operatively if there is minimal displacement.
ICP monitoring is appropriate in those who have GCS 3-8 and normal CT scan.
ICP monitoring is mandatory in those who have GCS 3-8 and abnormal CT scan.
Hyponatraemia is most likely to be due to syndrome of inappropriate ADH secretion.
Minimum of cerebral perfusion pressure of 70mmHg in adults.
Minimum cerebral perfusion pressure of between 40 and 70 mmHg in children.

Interpretation of pupillary findings in head injuries
Pupil size Light response Interpretation
Unilaterally dilated Sluggish or fixed 3rd nerve compression secondary to tentorial herniation
Bilaterally dilated Sluggish or fixed
Poor CNS perfusion
Bilateral 3rd nerve palsy
Unilaterally dilated or equal Cross reactive (Marcus - Gunn) Optic nerve injury
Bilaterally constricted May be difficult to assess
Opiates
Pontine lesions
Metabolic encephalopathy
Unilaterally constricted Preserved Sympathetic pathway disruption

1403
Q

A 10 year old girl has right iliac fossa pain and an appendicectomy is to be performed. What is the best incision for this procedure?

Paramedian

Midline abdominal

Kochers

Lanz

Gridiron

A

Both Lanz and Gridiron incisions can be used to perform an open appendicectomy. However, a Lanz incision provides for a superior cosmetic scar and is more easily extended than a Gridiron incision.

Abdominal incisions

Midline incision
Commonest approach to the abdomen
Structures divided: linea alba, transversalis fascia, extraperitoneal fat, peritoneum (avoid falciform ligament above the umbilicus)
Bladder can be accessed via an extraperitoneal approach through the space of Retzius
Paramedian incision
Parallel to the midline (about 3-4cm)
Structures divided/retracted: anterior rectus sheath, rectus (retracted), posterior rectus sheath, transversalis fascia, extraperitoneal fat, peritoneum
Incision is closed in layers
Battle
Similar location to paramedian but rectus displaced medially (and thus denervated)
Now seldom used
Kocher’s Incision under right subcostal margin e.g. Cholecystectomy (open)
Lanz Incision in right iliac fossa e.g. Appendicectomy
Gridiron Oblique incision centered over McBurneys point- usually appendicectomy (less cosmetically acceptable than Lanz
Gable Rooftop incision
Pfannenstiel’s Transverse supra pubic, primarily used to access pelvic organs
McEvedy’s Groin incision e.g. Emergency repair strangulated femoral hernia
Rutherford Morrison Extraperitoneal approach to left or right lower quadrants. Gives excellent access to iliac vessels and is the approach of choice for first time renal transplantation.

1404
Q

A homeless 42 year old male had an emergency inguinal hernia repair 24 hours previously. He has a BMI of 15. His electrolytes are normal. What is the best initial feeding regime?

Give 10 kcal/kg/day initially, oral thiamine 200-300mg/day, vitamin B co strong1 tds and supplements.

Give 35 kcal/kg/day initially, oral thiamine 200-300mg/day, vitamin B co strong 1 tds and supplements.

No change to diet needed

Oral thiamine 200-300mg/day, vitamin B co strong1 tds and supplements.

Give 35 kcal/kg/day initially

A

A-This patient is at high risk of refeeding syndrome.

Nutrition - Refeeding syndrome

Refeeding syndrome describes the metabolic abnormalities which occur on feeding a person following a period of starvation. The metabolic consequences include:
Hypophosphataemia
Hypokalaemia
Hypomagnesaemia
Abnormal fluid balance
These abnormalities can lead to organ failure.

Re-feeding problems
If patient not eaten for > 5 days, aim to re-feed at < 50% energy and protein levels

High risk for re-feeding problems
If one or more of the following:
BMI < 16 kg/m2
Unintentional weight loss >15% over 3-6 months
Little nutritional intake > 10 days
Hypokalaemia, Hypophosphataemia or hypomagnesaemia prior to feeding (unless high)

If two or more of the following:
BMI < 18.5 kg/m2
Unintentional weight loss > 10% over 3-6 months
Little nutritional intake > 5 days
History of: alcohol abuse, drug therapy including insulin, chemotherapy, diuretics and antacids

Prescription
Start at up to 10 kcal/kg/day increasing to full needs over 4-7 days
Start immediately before and during feeding: oral thiamine 200-300mg/day, vitamin B co strong 1 tds and supplements
Give K+ (2-4 mmol/kg/day), phosphate (0.3-0.6 mmol/kg/day), magnesium (0.2-0.4 mmol/kg/day)

1405
Q

Which of the following does not cause red urine?

Rifampicin

Phosphaturia

Beetroot

Rhubarb

Blackberries

A

Phosphaturia causes cloudy urine.

Haematuria

Causes of haematuria

Trauma
Injury to renal tract
Renal trauma commonly due to blunt injury (others penetrating injuries)
Ureter trauma rare: iatrogenic
Bladder trauma: due to RTA or pelvic fractures
Infection
Remember TB
Malignancy
Renal cell carcinoma (remember paraneoplastic syndromes): painful or painless
Urothelial malignancies: 90% are transitional cell carcinoma, can occur anywhere along the urinary tract. Painless haematuria.
Squamous cell carcinoma and adenocarcinoma: rare bladder tumours
Prostate cancer
Penile cancers: SCC
Renal disease
Glomerulonephritis
Stones
Microscopic haematuria common
Structural abnormalities
Benign prostatic hyperplasia (BPH) causes haematuria due to hypervascularity of the prostate gland
Cystic renal lesions e.g. polycystic kidney disease
Vascular malformations
Renal vein thrombosis due to renal cell carcinoma
Coagulopathy
Causes bleeding of underlying lesions
Drugs
Cause tubular necrosis or interstitial nephritis: aminoglycosides, chemotherapy
Interstitial nephritis: penicillin, sulphonamides, and NSAIDs
Anticoagulants
Benign
Exercise
Gynaecological
Endometriosis: flank pain, dysuria, and haematuria that is cyclical
Iatrogenic
Catheterisation
Radiotherapy; cystitis, severe haemorrhage, bladder necrosis
Pseudohaematuria For example following consumption of beetroot

References
Http://bestpractice.bmj.com/best-practice/monograph/316/overview/aetiology.html

1406
Q

A 4 year old girl presents with symptoms of right sided loin pain, lethargy and haematuria. On examination she is pyrexial and has a large mass in the right upper quadrant. The most likely underlying diagnosis is:

Perinephric abscess

Nephroblastoma

Renal cortical adenoma

Grawitz tumour

Squamous cell carcinoma of the kidney

A

In a child of this age, with the symptoms described a nephroblastoma is the most likely diagnosis. A perinephric abscess is most unlikely. If an abscess were to occur it would be confined to Gerotas fascia in the first instance, and hence anterior extension would be unlikely.

Nephroblastoma

Nephroblastoma (Wilms tumours)
Usually present in first 4 years of life
May often present as a mass associated with haematuria (pyrexia may occur in 50%)
Often metastasise early (usually to lung)
Treated by nephrectomy
Younger children have better prognosis (<1 year of age =80% overall 5 year survival)

1407
Q

Which of the following does not pass through the superior orbital fissure?

Lacrimal nerve

Abducens nerve

Opthalmic artery

Trochlear nerve

Superior opthalmic vein

A

Mnemonic for the nerves passing through the supraorbital fissure:

Live Frankly To See Absolutely No Insult

Lacrimal
Frontal
Trochlear
Superior Division of Oculomotor
Abducens
Nasociliary
Inferior Division of Oculomotor nerve
The opthalmic artery arises from the internal carotid immediately after it has pierced the dura and arachnoid. It runs through the optic canal below the optic nerve and within its dural and arachnoid sheaths. It terminates as the supratrochlear and dorsal nasal arteries.
Foramina of the base of the skull

Foramen Location Contents
Foramen ovale Sphenoid bone Otic ganglion
V3 (Mandibular nerve:3rd branch of
trigeminal)
Accessory meningeal artery
Lesser petrosal nerve
Emissary veins
Foramen spinosum Sphenoid bone Middle meningeal artery
Meningeal branch of the Mandibular nerve
Foramen rotundum Sphenoid bone Maxillary nerve (V2)
Foramen lacerum/ carotid canal Located between the sphenoid, the apex of the petrous temporal and the basilar part of the occipital Base of the medial pterygoid plate.
Internal carotid artery*
Nerve and artery of the pterygoid canal
Jugular foramen Temporal bone Anterior: inferior petrosal sinus
Intermediate: glossopharyngeal, vagus, and accessory nerves.
Posterior: sigmoid sinus (becoming the internal jugular vein) and some meningeal branches from the occipital and ascending pharyngeal arteries.
Foramen magnum Occipital bone Anterior and posterior spinal arteries
Vertebral arteries
Medulla oblongata
Stylomastoid foramen Temporal bone Stylomastoid artery
Facial nerve
Superior orbital fissure Sphenoid bone Oculomotor nerve (III)
Recurrent meningeal artery
Trochlear nerve (IV)
Lacrimal, frontal and nasociliary branches of opthalmic nerve (V1)
Abducent nerve (VI)
Superior ophthalmic vein

*= In life the foramen lacerum is occluded by a cartilagenous plug. The ICA initially passes into the carotid canal which ascends superomedially to enter the cranial cavity through the foramen lacerum.

1408
Q

Of the agents listed below, which is the most appropriate to give a 65 year old man with a locally unresectable gastrointestinal stromal tumour. Biopsies confirm that it is KIT positive.

Imatinib

Infliximab

Trastuzumab

Bevacizumab

Cetuximab

A

Imatinib is licensed for treatment of GIST in the United Kingdom for this situation. The guidance from the National Institute of Clinical evidence is that patients be reviewed at 12 weeks after initiating therapy.

Biological agents

Agents Target Uses
Adalimumab
Infliximab
Etanercept TNF alpha inhibitor Crohns disease
Rheumatoid disease
Bevacizumab Anti VEGF (anti angiogenic) Colorectal cancer
Renal
Glioblastoma
Trastuzumab HER receptor Breast cancer
Imatinib Tyrosine kinase inhibitor Gastrointestinal stromal tumours
Chronic myeloid leukaemia
Basiliximab IL2 binding site Renal transplants
Cetuximab Epidermal growth factor inhibitor EGF positive colorectal cancers

Detailed understanding of the actions of biological agents is well beyond the scope of the MRCS syllabus. However, many of these drugs are being frequently encountered in surgical patients.

1409
Q

A 21 year old man undergoes surgical removal of an impacted 3rd molar. Post operatively, he is noted to have anaesthesia on the anterolateral aspect of the tongue. What is the most likely explanation?

Injury to the hypoglossal nerve

Injury to the inferior alveolar nerve

Injury to the lingual nerve

Injury to the mandibular branch of the facial nerve

Injury to the glossopharyngeal nerve

A

The lingual nerve is closely related to the third molar and up to 10% of patients undergoing surgical extraction of these teeth may subsequently develop a lingual neuropraxia. The result is anaesthesia of the ipsilateral anterior aspect of the tongue. The inferior alveolar nerve innervates the teeth themselves.

Lingual nerve

  • Sensory nerve to the mucosa of the presulcal part of the tongue, floor of mouth and mandibular lingual gingivae
    Arises from posterior trunk of the mandibular nerve (branch of trigeminal)
    Course runs past tensor veli palatini and lateral pterygoid (where it is joined by the chorda tympani branch of the facial nerve). Emerging from the cover of the lateral pterygoid it proceeds antero inferiorly lying on the surface of the medial pterygoid and lies close to the medial aspect of the mandibular ramus. At the junction of the vertical and horizontal rami of the mandible it is anterior to the inferior alveolar nerve. It then passes below the mandibular attachment of the superior pharyngeal constrictor. Eventually, it lies on the periosteum of the root of the third molar tooth. It then passes medial to the mandibular origin of mylohyoid and then passes forwards on the inferior surface of this muscle
1410
Q

A neonate born at term has an episode of bilious vomiting and on investigation is found to have a DJ flexure displaced to the right. What procedure are they most likely to require?

Ladds procedure

Gastrojejunostomy

Kasai procedure

Duoduodenostomy

Ramstedts procedure

A

Intestinal malrotation with volvulus is treated with a Ladd’s procedure

Bilious vomiting in neonates

Causes of intestinal obstruction with bilious vomiting in neonates
Disorder Incidence and causation Age at presentation Diagnosis Treatment
Duodenal atresia 1 in 5000 (higher in Downs syndrome) Few hours after birth AXR shows ‘double bubble sign, contrast study may confirm Duodenoduodenostomy
Malrotation with volvulus Usually cause by incomplete rotation during embryogenesis Usually 3-7 days after birth, volvulus with compromised circulation may result in peritoneal signs and haemodynamic instability Upper GI contrast study may show DJ flexure is more medially placed, USS may show abnormal orientation of SMA and SMV Ladd’s procedure
Jejunal/ ileal atresia Usually caused by vascular insufficiency in utero, usually 1 in 3000 Usually within 24 hours of birth AXR will show air-fluid levels Laparotomy with primary resection and anastomosis
Meconium ileus Occurs in between 15 and20% of those babies with cystic fibrosis, otherwise 1 in 5000 Typically in first 24-48 hours of life with abdominal distension and bilious vomiting Air - fluid levels on AXR, sweat test to confirm cystic fibrosis Surgical decompression, serosal damage may require segmental resection
Necrotising enterocolitis Up to 2.4 per 1000 births, risks increased in prematurity and inter-current illness Usually second week of life Dilated bowel loops on AXR, pneumatosis and portal venous air Conservative and supportive for non perforated cases, laparotomy and resection in cases of perforation of ongoing clinical deterioration

1411
Q

Which of the nerves listed below is directly responsible for the innervation of the lateral aspect of flexor digitorum profundus?

Ulnar nerve

Anterior interosseous nerve

Radial nerve

Median nerve

Posterior interosseous nerve

A

The anterior interosseous nerve is a branch of the median nerve and is responsible for innervation of the lateral aspect of the flexor digitorum profundus.

Forearm flexor muscles

Muscle Origin Insertion Nerve supply Action
Flexor carpi radialis Common flexor origin and surrounding fascia Front of bases of second and third metacarpals Median Flexes and abducts the carpus, part flexes the elbow and part pronates forearm
Palmaris longus Common flexor origin Apex of palmar aponeurosis Median Wrist flexor
Flexor carpi ulnaris Small humeral head arises from the common flexor origin and adjacent fascia. Ulnar head comes from medial border of olecranon and posterior border of ulna Pisiform and base of the fifth metacarpal Ulnar nerve Flexes and adducts the carpus
Flexor digitorum superficialis Long linear origin from common flexor tendon, adjacent fascia and septa and medial border of the coronoid process Via tendons in the fibrous flexor sheath. At the level of the metacarpophalangeal joint each tendon split into two, these bands pass distally to their insertions Median Flexor of metacarpophalangeal joint and proximal interphalangeal joint
Flexor digitorum profundus Upper two thirds of the medial and anterior surface of the ulna, medial side of the olecranon, medial half of the interosseous membrane Via tendons that lie deep to those of flexor digitorum superficialis to insert into the distal phalanx Medial part= ulnar, lateral part=anterior interosseous nerve Flexes the distal interphalangeal joints and the wrist

1412
Q

A 63 year old lady is undergoing an axillary sentinel lymph node biopsy as part of her breast cancer treatment. Which of the structures listed below is most likely to be encountered?

Subclavian artery

Intercostobrachial nerve

Upper cord of the brachial plexus

Lower cord of the brachial plexus

Axillary nerve

A

This can be a challenging question. A particularly careless surgeon could encounter all of these. However, during a routine level 1 axillary exploration which is where the majority of sentinel nodes will be located, the nerves most commonly encountered are the intercostobrachial nerves.

Axilla

Boundaries of the axilla
Medially Chest wall and Serratus anterior
Laterally Humeral head
Floor Subscapularis
Anterior aspect Lateral border of Pectoralis major
Fascia Clavipectoral fascia

Content:
Long thoracic nerve (of Bell) Derived from C5-C7 and passes behind the brachial plexus to enter the axilla. It lies on the medial chest wall and supplies serratus anterior. Its location puts it at risk during axillary surgery and damage will lead to winging of the scapula.
Thoracodorsal nerve and thoracodorsal trunk Innervate and vascularise latissimus dorsi.
Axillary vein Lies at the apex of the axilla, it is the continuation of the basilic vein. Becomes the subclavian vein at the outer border of the first rib.
Intercostobrachial nerves Traverse the axillary lymph nodes and are often divided during axillary surgery. They provide cutaneous sensation to the axillary skin.
Lymph nodes The axilla is the main site of lymphatic drainage for the breast.

1413
Q

A 34 year old man presents with localised spinal pain over 2 months which is worsened on movement. He is known to be an IVDU. He has no history suggestive of tuberculosis. The pain is now excruciating at rest and not improving with analgesia. He has a temperature of 39 oC. What is the most likely diagnosis?

Transverse myelitis

Osteomyelitis

Potts disease of the spine

Epidural haematoma

Brown-Sequard syndrome

A

In an IVDU with back pain and pyrexia have a high suspicion for osteomylelitis. The most likely organism is staph aureus and the cervical spine is the most common region affected. TB tends to affect the thoracic spine and in other causes of osteomyelitis the lumbar spine is affected.

Spinal disorders

Dorsal column lesion
Loss vibration and proprioception
Tabes dorsalis, SACD
Spinothalamic tract lesion
Loss of pain, sensation and temperature
Central cord lesion
Flaccid paralysis of the upper limbs
Osteomyelitis
Normally progressive
Staph aureus in IVDU, normally cervical region affected
Fungal infections in immunocompromised
Thoracic region affected in TB
Infarction spinal cord
Dorsal column signs (loss of proprioception and fine discrimination)
Cord compression
UMN signs
Malignancy
Haematoma
Fracture
Brown-sequard syndrome
Hemisection of the spinal cord
Ipsilateral paralysis
Ipsilateral loss of proprioception and fine discrimination
Contralateral loss of pain and temperature

Dermatomes
C2 to C4 The C2 dermatome covers the occiput and the top part of the neck. C3 covers the lower part of the neck to the clavicle. C4 covers the area just below the clavicle.
C5 to T1 Situated in the arms. C5 covers the lateral arm at and above the elbow. C6 covers the forearm and the radial (thumb) side of the hand. C7 is the middle finger, C8 is the medial aspect of the hand, and T1 covers the medial side of the forearm.
T2 to T12 The thoracic covers the axillary and chest region. T3 to T12 covers the chest and back to the hip girdle. The nipples are situated in the middle of T4. T10 is situated at the umbilicus. T12 ends just above the hip girdle.
L1 to L5 The cutaneous dermatome representing the hip girdle and groin area is innervated by L1 spinal cord. L2 and 3 cover the front part of the thighs. L4 and L5 cover medial and lateral aspects of the lower leg.
S1 to S5 S1 covers the heel and the middle back of the leg. S2 covers the back of the thighs. S3 cover the medial side of the buttocks and S4-5 covers the perineal region. S5 is of course the lowest dermatome and represents the skin immediately at and adjacent to the anus.

Myotomes

Upper limb
Elbow flexors/Biceps C5
Wrist extensors C6
Elbow extensors/Triceps C7
Long finger flexors C8
Small finger abductors T1

Lower limb
Hip flexors (psoas) L1 and L2
Knee extensors (quadriceps) L3
Ankle dorsiflexors (tibialis anterior) L4 and L5
Toe extensors (hallucis longus) L 5
Ankle plantar flexors (gastrocnemius) S1

The anal sphincter is innervated by S2,3,4

1414
Q

A 30 year old woman is injured in a skiing accident. She suffers a blow to the occiput and is concussed for 5 minutes. On arrival in ED she is confused with GCS 10/15. A CT scan shows no evidence of acute bleed or fracture but some evidence of oedema with the beginnings of mass effect. What is the best initial course of action?

Administration of intravenous mannitol

Administration of intravenous frusemide

Parietotemporal craniotomy

Posterior fossa craniotomy

Burr hole decompression

A

This woman has raised ICP and mannitol will help reduce this in the acute phase.

Head injury management- NICE Guidelines

Summary of guidelines
All patients should be assessed within 15 minutes on arrival to A&E
Document all 3 components of the GCS
If GCS <8 or = to 8, consider stabilising the airway
Full spine immobilisation until assessment if:

  • GCS < 15
  • neck pain/tenderness
  • paraesthesia extremities
  • focal neurological deficit
  • suspected c-spine injury

If a c-spine injury is suspected a 3 view c-spine x-ray is indicated. CT c-spine is preferred if:
- Intubated
- GCS <13
- Normal x-ray but continued concerns regarding c-spine injury
- Any focal neurology
- A CT head scan is being performed
- Initial plain films are abnormal

Immediate CT head (within 1 hour) if:
GCS < 12 on admission
GCS < 15 2 hours after admission
Suspected open or depressed skull fracture
Suspected skull base fracture (panda eyes, Battle’s sign, CSF from nose/ear, bleeding ear)
Focal neurology
Vomiting > 1 episode
Post traumatic seizure

Contact neurosurgeon if:
Persistent GCS < 8 or = 8
Unexplained confusion > 4h
Reduced GCS after admission
Progressive neurological signs
Incomplete recovery post seizure
Penetrating injury
Cerebrospinal fluid leak

Observations
1/2 hourly GCS until 15

In the latest guidance, there is a move to considering CT in those on anticoagulants rather than mandating it.

Reference
1. NICE Guidance NG 232 (Published 2023).
2. Hodgkinson S et al. Early management of head injury: summary of NICE guidance. BMJ 2014 (348):34-37.

1415
Q

A 22 year old man is brought to the emergency department with swallowing difficulty and pain on rotation of the head. Around an hour earlier, he received a kick to the neck in a karate competition. He undergoes imaging with a CT scan of the neck which demonstrates a fracture of the right greater cornu of the hyoid. Injury to which of the muscles listed below accounts for the swallowing difficulty?

Geniohyoid

Middle pharyngeal constrictor

Omohyoid

Sternohyoid

Thyrohyoid

A

The swallowing issues here arise because the function of the middle pharyngeal constrictor is impaired. This muscle takes its origin from the lesser and greater cornu of the hyoid. During swallowing, the pharyngeal constrictor muscles force the food bolus into the oesophagus.

Anterior anatomy of the neck

Anterior triangle
Boundaries
Anterior border of the Sternocleidomastoid
Lower border of mandible
Anterior midline

Sub triangles (divided by Digastric above and Omohyoid)
Muscular triangle: Neck strap muscles
Carotid triangle: Carotid sheath
Submandibular Triangle (digastric)

Contents of the anterior triangle
Digastric triangle Submandibular gland
Submandibular nodes
Facial vessels
Hypoglossal nerve
Muscular triangle Strap muscles
External jugular vein
Carotid triangle Carotid sheath (Common carotid, vagus and internal jugular vein)
Ansa cervicalis

Nerve supply to digastric muscle
Anterior: Mylohyoid nerve
Posterior: Facial nerve

Veins
Veins of the neck are either superficial or deep to the deep fascia of the neck.Superficial tributaries of the external jugular vein drain a much smaller tissue volume than the tributaries of the internal jugular vein.

The external jugular veins drains the superficial aspects of the scalp and neck. It commences as the union of the posterior divisions of the retromandibular and posterior auricular veins near the angle of the mandible either below or within the parotid. It then descends superficially from there to the mid aspect of the clavicle. It is covered by the platysma, superficial fascia and skin.

The anterior jugular vein commences near the hyoid bone by the confluence of the superficial mandibular veins. It descends between the midline and the anterior border of the sternocleidomastoid. It eventually descends deep to this muscle to join either the external jugular or the subclavian vein.

The internal jugular vein commences at the cranial base in the posterior compartment of the jugular foramen and is continuous with the sigmoid sinus. The vein is contained within the carotid sheath and eventually unites with the subclavian vein posterior to the sternal end of the clavicle.

Surface marking of the internal jugular vein
This is from the ear lobe to the medial end of the clavicle. The inferior bulb lies in a depression between the sternal and clavicular heads of the sternocleidomastoid, the lesser clavicular fossa.

External carotid artery
The external carotid commences immediately lateral to the pharyngeal side wall. It ascends and lies anterior to the internal carotid and posterior to the posterior belly of digastric and stylohyoid. More inferiorly it is covered by sternocleidomastoid, passed by hypoglossal nerves, lingual and facial veins.
It then pierces the fascia of the parotid gland finally dividing into its terminal branches within the gland itself.

Surface marking of the carotid
This is an imaginary line drawn from the bifurcation of the common carotid passing behind the angle of the jaw to a point immediately anterior to the tragus of the ear.

Branches of the external carotid artery
It has six branches, three in front, two behind and one deep.
Three in front Superior thyroid
Lingual
Facial
Two behind Occipital
Posterior auricular
Deep Ascending pharyngeal

It terminates by dividing into the maxillary and superficial temporal arteries.

1416
Q

A 67 year old man is due to undergo a Whipples procedure to resect a pancreatic adenocarcinoma. What is the most appropriate surgical incision to address this?

Rooftop

Kochers

Paramedian

Battle

Thoracoabdominal

A

A rooftop incision is typically used to access the pancreas for resectional surgery.

Abdominal incisions

Midline incision
Commonest approach to the abdomen
Structures divided: linea alba, transversalis fascia, extraperitoneal fat, peritoneum (avoid falciform ligament above the umbilicus)
Bladder can be accessed via an extraperitoneal approach through the space of Retzius
Paramedian incision
Parallel to the midline (about 3-4cm)
Structures divided/retracted: anterior rectus sheath, rectus (retracted), posterior rectus sheath, transversalis fascia, extraperitoneal fat, peritoneum
Incision is closed in layers
Battle
Similar location to paramedian but rectus displaced medially (and thus denervated)
Now seldom used
Kocher’s Incision under right subcostal margin e.g. Cholecystectomy (open)
Lanz Incision in right iliac fossa e.g. Appendicectomy
Gridiron Oblique incision centered over McBurneys point- usually appendicectomy (less cosmetically acceptable than Lanz
Gable Rooftop incision
Pfannenstiel’s Transverse supra pubic, primarily used to access pelvic organs
McEvedy’s Groin incision e.g. Emergency repair strangulated femoral hernia
Rutherford Morrison Extraperitoneal approach to left or right lower quadrants. Gives excellent access to iliac vessels and is the approach of choice for first time renal transplantation.

1417
Q

At what level does the sciatic nerve usually bifurcate into the tibial and common peroneal nerves?

At the superior aspect of the popliteal fossa

At the inferior aspect of the popliteal fossa

At the inferior border of gluteus maximus

At the inferior border of the piriformis muscle

In the pelvis

A

The sciatic nerve passes vertically downwards over the posterior surface of the obturator internus and quadratus femoris to the hamstring compartment of the thigh, here it is crossed posteriorly by the long head of biceps femoris. In the buttock it lies under the cover of gluteus maximus. It separates into its tibial and common peroneal components at the upper aspect of the popliteal fossa.

Sciatic nerve

The sciatic nerve is formed from the sacral plexus and is the largest nerve in the body. It is the continuation of the main part of the plexus arising from ventral rami of L4 to S3. These rami converge at the inferior border of piriformis to form the nerve itself. It passes through the inferior part of the greater sciatic foramen and emerges beneath piriformis. Medially, lie the inferior gluteal nerve and vessels and the pudendal nerve and vessels. It runs inferolaterally under the cover of gluteus maximus midway between the greater trochanter and ischial tuberosity. It receives its blood supply from the inferior gluteal artery. The nerve provides cutaneous sensation to the skin of the foot and the leg. It also innervates the posterior thigh muscles and the lower leg and foot muscles. The nerve splits into the tibial and common peroneal nerves approximately half way down the posterior thigh. The tibial nerve supplies the flexor muscles and the common peroneal nerve supplies the extensor muscles and the evertor muscles of the foot.

Summary points
Origin Spinal nerves L4 - S3
Articular Branches Hip joint
Muscular branches in upper leg
Semitendinosus
Semimembranosus
Biceps femoris
Part of adductor magnus
Cutaneous sensation
Posterior aspect of thigh (via cutaneous nerves)
Gluteal region
Entire lower leg (except the medial aspect)
Terminates At the upper part of the popliteal fossa by dividing into the tibial and peroneal nerves

The nerve to the short head of the biceps femoris comes from the common peroneal part of the sciatic and the other muscular branches arise from the tibial portion.
The tibial nerve goes on to innervate all muscles of the foot except the extensor digitorum brevis (which is innervated by the common peroneal nerve).

1418
Q

The femoral nerve is transected by a rather careless surgeon during a botched femoro-popliteal bypass operation. Which of the following actions will be impaired?

Extension of the great toe

Adduction of the thigh

Flexion of the knee joint

Extension of the knee joint

Eversion of the foot

A

The femoral nerve supplies the quadriceps muscle which is responsible for extension at the knee joint.

Femoral nerve

Root values L2, 3, 4
Innervates
Pectineus
Sartorius
Quadriceps femoris
Vastus lateralis/medialis/intermedius
Rectus femoris
Branches
Medial cutaneous nerve of thigh
Saphenous nerve
Intermediate cutaneous nerve of thigh

Path
Penetrates psoas major and exits the pelvis by passing under the inguinal ligament to enter the femoral triangle, lateral to the femoral artery and vein.
Mnemonic for femoral nerve supply

(don’t) M I S V Q Scan for PE
M edial cutaneous nerve of the thigh
I ntermediate cutaneous nerve of the thigh
S aphenous nerve

V astus
Q uadriceps femoris
S artorius

PE ectineus

1419
Q

Which of the following anatomical planes separates the prostate from the rectum?

Sibsons fascia

Denonvilliers fascia

Levator ani muscle

Waldeyers fascia

None of the above

A

The Denonvilliers fascia separates the rectum from the prostate. Waldeyers fascia separates the rectum from the sacrum
Prostate gland

The prostate gland is approximately the shape and size of a walnut and is located inferior to the bladder. It is separated from the rectum by Denonvilliers fascia and its blood supply is derived from the internal iliac vessels (via inferior vesical artery). The internal sphincter lies at the apex of the gland and may be damaged during prostatic surgery, affected individuals may complain of retrograde ejaculation.

Summary of prostate gland
Arterial supply Inferior vesical artery (from internal iliac)
Venous drainage Prostatic venous plexus (to paravertebral veins)
Lymphatic drainage Internal iliac nodes
Innervation Inferior hypogastric plexus
Dimensions
Transverse diameter (4cm)
AP diameter (2cm)
Height (3cm)
Lobes
Posterior lobe: posterior to urethra
Median lobe: posterior to urethra, in between ejaculatory ducts
Lateral lobes x 2
Isthmus
Zones
Peripheral zone: subcapsular portion of posterior prostate. Most prostate cancers are here
Central zone
Transition zone
Stroma

Relations
Anterior Pubic symphysis
Prostatic venous plexus
Posterior Denonvilliers fascia
Rectum
Ejaculatory ducts
Lateral Venous plexus (lies on prostate)
Levator ani (immediately below the puboprostatic ligaments)

1420
Q

A 20 year old man is suspected of having an inflamed Meckels diverticulum. At which of the following locations is it most likely to be found?

Approximately 60 cm distal to the ileo-caecal valve

Approximately 60 cm proximal to the ileocaecal valve

Approximately 200cm distal to the ileocaecal valve

Approximately 200cm proximal to the ileocaecal valve

50cm distal to the DJ flexure

A

Rule of 2’s

2% of population
2 inches (5cm) long
2 feet (60 cm) from the ileocaecal valve
2 x’s more common in men
2 tissue types involved
They are typically found 2 feet proximal to the ileocaecal valve (or approximately 60cm).

Meckel’s diverticulum

  • Congenital abnormality resulting in incomplete obliteration of the vitello-intestinal duct
    Normally, in the foetus, there is an attachment between the vitello-intestinal duct and the yolk sac.This disappears at 6 weeks gestation.
    The tip is free in majority of cases.
    Associated with enterocystomas, umbilical sinuses, and omphaloileal fistulas.
    Arterial supply: omphalomesenteric artery.
    2% of population, 2 inches long, 2 feet from the ileocaecal valve.
    Typically lined by ileal mucosa but ectopic gastric mucosa can occur, with the risk of peptic ulceration. Pancreatic and jejunal mucosa can also occur.

Clinical
Normally asymptomatic and an incidental finding.
Complications are the result of obstruction, ectopic tissue, or inflammation.
Removal if narrow neck or symptomatic. Options are between wedge excision or formal small bowel resection and anastomosis.

1421
Q

Which of the tumour markers listed below is most likely to be elevated in a patient with colorectal cancer?

CEA

CA19-9

IgG 4

AFP

PSA

A

Tumour markers

Tumour markers may be divided into:
monoclonal antibodies against carbohydrate or glycoprotein tumour antigens
tumour antigens
enzymes (alkaline phosphatase, neurone specific enolase)
hormones (e.g. calcitonin, ADH)

It should be noted that tumour markers usually have a low specificity

Monoclonal antibodies
Tumour marker Association
CA 125 Ovarian cancer
CA 19-9 Pancreatic cancer
CA 15-3 Breast cancer
NB: The breast cancer tumour marker is not specific or sensitive enough to be used routinely.

Tumour antigens
Tumour marker Association
Prostate specific antigen (PSA) Prostatic carcinoma
Alpha-feto protein (AFP) Hepatocellular carcinoma, teratoma
Carcinoembryonic antigen (CEA) Colorectal cancer

1422
Q

A 55 year old man undergoes a colonoscopy and a colonic polyp is identified. It has a lobular appearance and is located on a stalk in the sigmoid colon. Which of the processes below best accounts for this finding?

Apoptosis

Metaplasia

Dysplasia

Calcification

Degeneration

A

Most colonic polyps described above are adenomas. These may have associated dysplasia. The more high grade the dysplasia the greater the level of clinical concern.

Colonic polyps

Colonic Polyps
May occur in isolation, or greater numbers as part of the polyposis syndromes. In FAP greater than 100 polyps are typically present. The risk of malignancy in association with adenomas is related to size, and is the order of 10% in a 1cm adenoma. Isolated adenomas seldom give risk of symptoms (unless large and distal). Distally sited villous lesions may produce mucous and if very large, electrolyte disturbances may occur.

Follow up of colonic polyps
Group Action
Colorectal cancer Colonoscopy 1 year post resection
Large non pedunculated colorectal polyps (LNPCP), R0 resection One off scope at 3 years
Large non pedunculated colorectal polyps (LNPCP) R1 or non en bloc resection Site check at 2-6 months and then a further scope at 12 months
High risk findings at baseline colonoscopy One off surveillance at 3 years
No high risk findings at baseline colonoscopy No colonoscopic surveillance and invite participation in NHSBCSP programme when due

High risk findings
More than 2 premalignant polyps including 1 or more advanced colorectal polyps
OR
More than 5 pre malignant polyps

Exceptions to guidelines
If patient more than 10 years younger than lower screening age and has polyps but no high risk findings, consider colonoscopy at 5 or 10 years.

Segmental resection or complete colectomy should be considered when:

  1. Incomplete excision of malignant polyp
  2. Malignant sessile polyp
  3. Malignant pedunculated polyp with submucosal invasion
  4. Polyps with poorly differentiated carcinoma
  5. Familial polyposis coli
    -Screening from teenager up to 40 years by 2 yearly sigmoidoscopy/colonoscopy
    -Panproctocolectomy and Ileostomy or Restorative Panproctocolectomy.

Rectal polypoidal lesions may be amenable to trans anal endoscopic microsurgery.

References
Rutter MD et al. British Society of Gastroenterology/Association of Coloproctology of Great Britain and Ireland/Public Health England post- polypectomy and post- colorectal cancer resection surveillance guidelines. Gut 2019;0:123.

1423
Q

During a revision hip arthroplasty procedure, the inferior gluteal nerve is injured. Which of the muscles listed below is most likely to be affected?

Gluteus medius

Gluteus minimis

Gluteus maximus

Piriformis

Tensor fasciae latae

A

The gluteus maximus is supplied by the inferior gluteal nerve.
Anatomy of the pelvis

The pelvic girdle formed by the union of the two innominate bones anteriorly at the pubis and posteriorly with their connection to the sacrum. The innominate bone is comprised of three main parts; ilium, ischium and pubis. In the young, these are connected by cartilagenous tissue. In adults, they unite as one bone. The dimensions of the bony pelvis differ in the two sexes, being shallower and wider in females and narrower and deeper in males.

Pelvic ligaments
The pelvic bony structures are interconnected by a series of ligaments. These assume clinical significance during pregnancy and childbirth as they develop increased laxity at this time.

Potential exits
Sites at which important anatomical structures exit the abdomen through the pelvis are commonly examined. These include the obturator, inguinal and femoral canals anteriorly. Inferiorly, the sciatic foramina also transmit structures. These are all covered separately in the resource.

Gluteal region
Gluteal muscles
Gluteus maximus: inserts to gluteal tuberosity of the femur and iliotibial tract
Gluteus medius: attach to lateral greater trochanter
Gluteus minimis: attach to anterior greater trochanter
All extend and abduct the hip

Deep lateral hip rotators
Piriformis
Gemelli
Obturator internus
Quadratus femoris

Nerves
Superior gluteal nerve (L5, S1)
Gluteus medius
Gluteus minimis
Tensor fascia lata
Inferior gluteal nerve Gluteus maximus
Damage to the superior gluteal nerve will result in the patient developing a Trendelenberg gait. Affected patients are unable to abduct the thigh at the hip joint. During the stance phase, the weakened abductor muscles allow the pelvis to tilt down on the opposite side. To compensate, the trunk lurches to the weakened side to attempt to maintain a level pelvis throughout the gait cycle. The pelvis sags on the opposite side of the lesioned superior gluteal nerve.

Anatomy of the sciatic foramina is covered separately

1424
Q

A 25 year old man is shot in the abdomen and is transferred to the operating theatre following arrival in the emergency department, as he is unstable and a FAST scan is positive. At operation there is an extensive laceration to the right lobe of the liver and involvement of the IVC. There is massive haemorrhage. What is the most appropriate initial approach to blood component therapy?

Use Factor VIII concentrates early

Avoid use of ‘o’ negative blood

Transfuse packed cells, FFP and platelets in fixed ratios of 1:1:1

Transfuse packed cells and FFP in a fixed ratio of 4:1

Perform goal directed transfusion based on the Hb, PT and TEG studies

A

There is strong evidence to support the use of haemostatic transfusion in the setting of major haemorrhage due to trauma. This advocates the use of 1:1:1 ratios.

Transfusions in major trauma

Uncontrolled haemorrhage accounts for up to 39% of all trauma related death. In the UK approximately 2% of all trauma patients will need massive transfusion.Massive transfusion is defined as the replacement of a patient’s total blood volume in less than 24 hours, or as the acute administration of more than half the patient’s estimated blood volume per hour. In haemorrhaging patients following trauma there is evidence to support the initial administration of tranexamic acid (CRASH study). During acute bleeding the practice of haemostatic resuscitation has been shown to reduce mortality rates. The principle of haemostatic resuscitation is that blood components are transfused in fixed ratios. For example; packed red cells, FFP and platelets are administered in a ratio of 1:1:1.

The typical therapeutic end points include:
Hb: 8-10 g/dl
Platelets > 100
PT (INR) and APTT < 1.5
Fibrinogen > 1.0 g/l
Ca2+ > 1 mmol/l
pH: 7.35-7.45
BE: +/- 2
ToC > 36 °C

Reference
Weymouth W et al. Whole Blood in Trauma: A Review for Emergency Clinicians. J Emerg Med. 2019 May;56(5):491-498.

1425
Q

A 40 year old man presents with obstructive jaundice and dysphagia. Twenty years previously he underwent a right hemicolectomy for a mucinous right sided colonic carcinoma. He was subsequently diagnosed as having Lynch syndrome. A recent colonoscopy was normal. What is the most likely cause of his jaundice?

Hepatocellular carcinoma

Liver metastasis from colonic cancer

Pancreatic carcinoma

Duodenal carcinoma

Metastatic gastric carcinoma

A

E-Metastatic gastric carcinoma

Mnemonic 3-2-1 for HNPCC.
3 individuals, 2 generations and one must be younger than 50 yrs.
Lynch syndrome usually results in colonic cancer which is right sided and mucinous. The next most common site to be affected is the uterus. The stomach is at particular risk and this risk is up to 10 times greater in HNPCC (Lynch) patients than the general population. Duodenal adenomas (and rarely carcinoma) are usually seen in association with FAP. Whilst pancreatic carcinoma is associated with HNPCC it is far less likely to occur than gastric cancer.
We are often asked how these patients become jaundiced, this occurs as a result of nodal spread along the hepatoduodenal ligament nodes to occlude the porta hepatis resulting in jaundice.

Genetics of colorectal cancer

The lifetime risk of colorectal cancer in the UK population is 5%. Up to 5% of newly diagnosed bowel cancers will be in those individuals who have a high genetically acquired risk of bowel cancer. Cancers arising in the low-moderate genetic risk group comprise approximately 30% of newly diagnosed bowel cancer.

Genetics of inherited colorectal cancer syndromes
Syndrome Features Genes implicated
FAP More than 100 adenomatous polyps affecting the colon and rectum. Duodenal and fundic glandular polyps APC (over 90%)
Gardner syndrome As FAP but with desmoid tumours and mandibular osteomas APC
Turcots syndrome Polyposis and colonic tumours and CNS tumours APC +MLH1 and PMS2
HNPCC Colorectal cancer without extensive polyposis. Endometrial cancer, renal and CNS MSH2, MLH1, PMS2 and GTBP
Peutz-Jeghers syndrome Hamartomatous polyps in GI tract and increased risk of GI malignancy LKB1 andSTK11 (in up to 70%)
Cowden disease Multiple hamartomas (see below) PTEN (85%)
MYH associated polyposis Autosomal recessive, multiple adenomatous polyps in GI tract, those in colon having somatic KRAS mutations MYH

FAP
Autosomal dominant condition, affects 1 in 12,000. Accounts for 0.5% of all CRCs. Lifetime incidence of colorectal cancer in untreated FAP =100%. Up to 25% cases are caused by de-novo germ line mutations and show no prior family history. The APC tumour suppressor gene is affected in most cases.

APC in non inherited colorectal cancer
Up to 80% of sporadic colorectal cancers will have somatic mutations that inactivate APC[1]. Both alleles are usually affected. Although the APC protein more than likely has multiple critical cellular functions, the best-established role for APC in the cancer process is as a major binding partner and regulator of the β- catenin protein in the so-called canonical or β- catenin dependent Wnt signaling pathway.

HNPCC (Lynch syndrome)
HNPCC cancers differ from conventional tumours in a number of respects. In the colon the tumours are more likely to be right sided, histologically they are more likely to be mucinous and have dense lymphocytic infiltrates. To be diagnosed as having HNPCC individuals must show typically HNPCC tumours in at least three individuals, (one of whom must be a first degree relative to the other two). In at least two successive generations. At least one cancer must be diagnosed under the age of 50. FAP must be excluded and tumours should be verified by pathological identification (Amsterdam criteria). The genetic changes in HNPCC stem primarily from microsatellite instability affecting DNA mismatch repair genes. In HNPCC the mismatch repair genes most commonly implicated include; MSH2 and MLH1 and these occur in up to 70% of people with HNPCC. The finding of microsatellite instability is unusual in sporadic colorectal cancers. Approximately 60% of individuals who fulfill the Amsterdam criteria will not be found to have evidence of mismatch repair gene defects on genetic testing. The risk of developing colorectal cancer in those who have not demonstrated mutation of the mis match repair genes is increased if they fulfill the Amsterdam criteria, but not
the extent that it is increased in those who fulfill the criteria AND have evidence of mis match repair gene defects.

KRAS Mutations
The RAS family of small G proteins act as molecular switches downstream of growth factor receptors. KRAS and the other two members of the family; HRAS and NRAS, are the site of mutation in approximately 40% of colorectal cancers. When adenomas are examined the proportion of adenomas less than 1cm showing KRAS mutations was only 10% which contrasts with 50% in those lesions greater than 1cm.

p53 mutations
The p53 protein functions as a key transcriptional regulator of genes that encode proteins with functions in cell-cycle checkpoints at the G1/S and G2/M boundaries, in promoting apoptosis, and in restricting angiogenesis . As such, selection for p53 defects at the adenoma-carcinoma transition may reflect the fact that stresses on tumor cells activate cell-cycle arrest, apoptotic, and antiangiogenic pathways in cells with wild-type p53 function. Many colonic tumours will demonstrate changes in the p53 gene that may facilitate tumour progression through from adenoma to carcinoma.

Cowden syndrome
Also known as multiple hamartoma syndrome. Rare autosomal dominant condition with incidence of 1 in 200,000.. It is characterised by multiple mucocutaneous lesions, trichilemmomas, oral papillomas and acral keratosis. Most often diagnosed in third decade of life. Breast carcinoma may occur in up to 50% of patients and conditions such as fibrocystic disease of the breast may occur in 75% of women. Thyroid disease occurs in 75% and may include malignancy. Endoscopic screening will identify disease in up to 85% although the small bowel is rarely involved. There is a 15-20% risk of developing colorectal cancer and regular colonoscopic screening from age 45 is recommended.

Terminology
Oncogene Oncogenes are genes which have the potential to induce cellular proliferation and avoid apoptosis. Oncogene mutations are general gain of function and are therefore dominant. Increased expression of oncogenes are found in most tumours
Tumour suppressor gene These genes generally inhibit cellular proliferation or induce apoptosis. Mutations in tumour suppressor genes are generally loss of function mutations, and are therefore recessive. Mutations in both tumour suppressor gene alleles allow cells to proliferate without restraint

References
1. Fearon, E.R. and B. Vogelstein, A genetic model for colorectal tumorigenesis. Cell, 1990. 61(5): p. 759-67.

1426
Q

A 46 year old man is admitted to hospital with a femoral shaft fracture that occurred suddenly whilst he was out walking his dog. On examination, there is no neurovascular deficit distal to the fracture site. He has a large firm nodule in the left lobe of the thyroid, there is no associated lymphadenopathy. What is the most likely underlying cause?

Papillary thyroid cancer

Follicular thyroid cancer

Thyroid gland lymphoma

Anaplastic thyroid cancer

Medullary thyroid cancer

A

Follicular carcinomas may metastasise haematogenously (often to bone) where they may give rise to pathological fractures as in this case.

Thyroid disease

Patients may present with a number of different manifestations of thyroid disease. They can be broadly sub classified according to whether they are euthyroid or have clinical signs of thyroid dysfunction. In addition it needs to be established whether they have a mass or not.

Assessment
History
Examination including USS
If a nodule is identified then it should be sampled ideally via an image guided fine needle aspiration
Radionucleotide scanning is of limited use

Thyroid Tumours
Papillary carcinoma
Follicular carcinoma
Anaplastic carcinoma
Medullary carcinoma
Lymphoma’s

Multinodular goitre
One of the most common reasons for presentation
Provided the patient is euthyroid and asymptomatic and no discrete nodules are seen, they can be reassured.
In those with compressive symptoms surgery is required and the best operation is a total thyroidectomy.
Sub total resections were practised in the past and simply result in recurrent disease that requires a difficult revisional resection.

Endocrine dysfunction
In general these patients are managed by physicians initially.
Surgery may be offered alongside radio iodine for patients with Graves disease that fails with medical management or in patients who would prefer not to be irradiated (e.g. pregnant women).
Patients with hypothyroidism do not generally get offered a thyroidectomy. Sometimes people inadvertently get offered resections during the early phase of Hashimotos thyroiditis, however, with time the toxic phase passes and patients can simply be managed with thyroxine.

Complications following surgery
Anatomical such as recurrent laryngeal nerve damage.
Bleeding. Owing to the confined space haematoma’s may rapidly lead to respiratory compromise owing to laryngeal oedema.
Damage to the parathyroid glands resulting in hypocalcaemia.

Further sources of information
1. http://www.acb.org.uk/docs/TFTguidelinefinal.pdf- Association of Clinical Biochemistry guidelines for thyroid function tests.

  1. British association of endocrine surgeons website- http://www.baets.org.uk
1427
Q

Patients with suspected temporal arteritis are often sent for temporal artery biopsy. Which statement is true?

Temporal artery biopsy is only diagnostic if there is visual loss

Biopsy is typically taken from the non-symptomatic side to avoid the risk of blindness

Pre-operative localisation with duplex is mandatory

Biopsies may be non diagnostic in over 50% of cases

Biopsies are usually performed under general anaesthesia

A

Temporal artery biopsies are frequently non diagnostic. They should be taken from the symptomatic side and though not mandatory a duplex ultrasound is a helpful investigation, particularly if they mark the artery. It is usually performed under local anaesthetic.

Temporal artery biopsy

Superficial temporal artery is a terminal branch of the external carotid artery

Main indication
Temporal arteritis

American College of Rheumatology guidelines recommend a temporal artery biopsy if:

Age of onset older than 50 years
New-onset headache or localized head pain
Temporal artery tenderness to palpation or reduced pulsation
ESR > 50 mm/h

Histopathology
Vessel wall granulomatous arteritis with mononuclear cell infiltrates and giant cell formation

Procedure
Position: supine, head 45 degrees
USS doppler to locate the superficial temporal artery or palpate
Local anaesthetic
Artery within temporoparietal fascia
Clamp and ligate the vessel
Cut 3-5cm
Ligate the remaining ends with absorbable suture
Close the skin

Contraindication
Glucocorticoid therapy > 30 days

Risks
Injury to facial or auriculotemporal nerve

1428
Q

What is the least likely examination finding in patients with Le Fort II fractures?

Excessive mobility of the palate

Paraesthesia in the region supplied by the inferior alveolar nerve

Malocclusion of the teeth

Enopthalmos

Parasthesia in the region supplied by the infraorbital nerve

A

Le Fort II fractures have a pyramidal shape. The fracture line involves the orbit and extends to involve the bridge of the nose and the ethmoids. In continues to involve the infraorbital rim and usually through the infraorbital foramen. As a result infraorbital parasthesia, palatal mobility and malocclusion are common findings. Severe fractures may result in enopthalmos. However, the fracture does not, by definition, involve the inferior alveolar nerve.

Craniomaxillofacial injuries

Craniomaxillofacial injuries in the UK are due to:
Interpersonal violence (52%)
Motor vehicle accidents (16%)
Sporting injuries (19%)
Falls (11%)

Le Fort Fractures
Grade Feature
Le Fort 1 The fracture extends from the nasal septum to the lateral pyriform rims, travels horizontally above the teeth apices, crosses below the zygomaticomaxillary junction, and traverses the pterygomaxillary junction to interrupt the pterygoid plates.
Le Fort 2 These fractures have a pyramidal shape and extend from the nasal bridge at or below the nasofrontal suture through the frontal process of the maxilla, inferolaterally through the lacrimal bones and inferior orbital floor and rim through or near the inferior orbital foramen, and inferiorly through the anterior wall of the maxillary sinus; it then travels under the zygoma, across the pterygomaxillary fissure, and through the pterygoid plates.
Le Fort 3 These fractures start at the nasofrontal and frontomaxillary sutures and extend posteriorly along the medial wall of the orbit through the nasolacrimal groove and ethmoid bones. The thicker sphenoid bone posteriorly usually prevents continuation of the fracture into the optic canal. Instead, the fracture continues along the floor of the orbit along the inferior orbital fissure and continues superolaterally through the lateral orbital wall, through the zygomaticofrontal junction and the zygomatic arch. Intranasally, a branch of the fracture extends through the base of the perpendicular plate of the ethmoid, through the vomer, and through the interface of the pterygoid plates to the base of the sphenoid. This type of fracture predisposes the patient to CSF rhinorrhea more commonly than the other types.

Ocular injuries
Superior orbital fissure syndrome
Severe force to the lateral wall of the orbit resulting in compression of neurovascular structures. Results in :
Complete opthalmoplegia and ptosis (Cranial nerves 3, 4, 6 and nerve to levator palpebrae superioris)
Relative afferent pupillary defect
Dilatation of the pupil and loss of accommodation and corneal reflexes
Altered sensation from forehead to vertex (frontal branch of trigeminal nerve)

Orbital blow out fracture
Typically occurs when an object of slightly larger diameter than the orbital rim strikes the incompressible eyeball. The bone fragment is displaced downwards into the antral cavity, remaining attached to the orbital periosteum. Periorbital fat may be herniated through the defect, interfering with the inferior rectus and inferior oblique muscles which are contained within the same fascial sheath. This prevents upward movement and outward rotation of the eye and the patient experiences diplopia on upward gaze. The initial bruising and swelling may make assessment difficult and patients should usually be reviewed 5 days later. Residual defects may require orbital floor reconstruction.

Nasal Fractures
Common injury
Ensure new and not old deformity
Control epistaxis
CSF rhinorrhoea implies that the cribriform plate has been breached and antibiotics will be required.
Usually best to allow bruising and swelling to settle and then review patient clinically. Major persistent deformity requires fracture manipulation, best performed within 10 days of injury.

Retrobulbar haemorrhage
Rare but important ocular emergency. Presents with:
Pain (usually sharp and within the globe)
Proptosis
Pupil reactions are lost
Paralysis (eye movements lost)
Visual acuity is lost (colour vision is lost first)
May be the result of Le Fort type facial fractures.

Management:
Mannitol 1g/Kg as 20% infusion, Osmotic diuretic, Contra-indicated in congestive heart failure and pulmonary oedema
Acetazolamide 500mg IV, (Monitor FBC/U+E) Reduces aqueous pressure by inhibition of carbonic anhydrase (used in glaucoma)
Dexamethasone 8mg orally or intravenously
In a traumatic setting an urgent cantholysis may be needed prior to definitive surgery.

Consider
Papaverine 40mg smooth muscle relaxant
Dextran 40 500mls IV improves perfusion

1429
Q

During a tricuspid valve repair the right atrium is opened, following establishment of cardiopulmonary bypass. Which of the following structures do not lie within the right atrium?

Crista terminalis

Tricuspid valve

Fossa ovalis

Trabeculae carnae

Musculi pectinati

A

Structures within the right atrium:
Musculi pectinati
Crista terminalis
Opening of the coronary sinus
Fossa ovalis
The trabeculae carnae are located in the right ventricle.

Heart anatomy

The walls of each cardiac chamber comprise:
Epicardium
Myocardium
Endocardium

Cardiac muscle is attached to the cardiac fibrous skeleton.

Relations
The heart and roots of the great vessels within the pericardial sac are related to the posterior aspect of the sternum, medial ends of the 3rd to 5th ribs on the left and their associated costal cartilages. The heart and pericardial sac are situated obliquely two thirds to the left and one third to the right of the median plane.

The pulmonary valve lies at the level of the left third costal cartilage.
The mitral valve lies at the level of the fourth costal cartilage.

Coronary sinus
This lies in the posterior part of the coronary groove and receives blood from the cardiac veins. The great cardiac vein lies at its left and the middle and small cardiac veins lie on its right. The smallest cardiac vein (anterior cardiac vein) drains into the right atrium directly.

Aortic sinus
Right coronary artery arises from the right aortic sinus, the left is derived from the left aortic sinus, which lies posteriorly.

Features of the left ventricle as opposed to the right

Structure Left Ventricle
A-V Valve Mitral (double leaflet)
Walls Twice as thick as right
Trabeculae carnae Much thicker and more numerous

Right coronary artery
The RCA supplies:
Right atrium
Diaphragmatic part of the right ventricle
Usually the posterior third of the interventricular septum
The sino atrial node (60% cases)
The atrio ventricular node (80% cases)

Left coronary artery
The LCA supplies:
Left atrium
Most of left ventricle
Part of the right ventricle
Anterior two thirds of the inter ventricular septum
The sino atrial node (remaining 40% cases)

Innervation of the heart
Autonomic nerve fibres from the superficial and deep cardiac plexus. These lie anterior to the bifurcation of the trachea, posterior to the ascending aorta and superior to the bifurcation of the pulmonary trunk. The parasympathetic supply to the heart is from presynaptic fibres of the vagus nerves.

Valves of the heart
Mitral valve Aortic valve Pulmonary valve Tricuspid valve
2 cusps 3 cusps 3 cusps 3 cusps
First heart sound Second heart sound Second heart sound First heart sound
1 anterior cusp 2 anterior cusps 2 anterior cusps 2 anterior cusps
Attached to chordae tendinae No chordae No chordae Attached to chordae tendinae

1430
Q

The cords of the brachial plexus are most closely related to which of the following vessels?

Subclavian artery

Axillary artery

Axillary vein

Subclavian vein

Brachial artery

A

The trunks are related to the subclavian artery superiorly. The cords of the plexus surround the axillary artery, they are named according to their positions relative to this structure.
Brachial plexus

The brachial plexus extends from the neck to the axilla. It is formed by the ventral rami of the fifth to the eighth cervical nerves with the ascending part of the first thoracic nerve.

Location of the plexus
The ventral rami which form the plexus enter the lower part of the posterior triangle of the neck in series with the ventral rami of the cervical plexus. The second part of the subclavian artery lies immediately anterior to the lower two rami. The upper three rami intermingle and pass inferolaterally towards the axilla and subclavian artery. They are enclosed within an extension of the prevertebral fascia. In the neck the plexus lies deep to platysma, the supraclavicular nerves, inferior belly of omohyoid and the transverse cervical artery. It then passes deep to the clavicle and the suprascapular vessels, to enter the axilla, and thence surround the second part of the axillary artery

Composition of the plexus
Ventral rami, the roots of the plexus, lie between scalenus medius and anterior.

As they enter the posterior triangle, the upper two (C5,6) and lower two (C8, T1) roots of the plexus unite to form the upper and lower trunks of the plexus respectively. Meanwhile, C7 continues as the middle trunk. The lower trunk may groove the superior surface of the first rib posterior to the subclavian artery, and the root from the first ventral ramus is always in contact with it.

Each trunk divides into ventral and dorsal divisions which are destined to supply the anterior (flexor) and posterior (extensor) parts of the upper limb.

The cords of the plexus are formed in the axilla. The dorsal divisions unite to form the posterior cord (C5-8). The ventral divisions of the upper and middle trunks unite to form the lateral cord (C5-7), while the ventral divisions of the lower trunk continues as the medial cord (C8-T1). The cords are named according to their relationship to the axillary artery. Each cord terminates by dividing into two main branches at the beginning of the third part of the artery.

Sympathetic communications
The fifth and sixth cervical ventral rami receive grey rami communicantes from the middle cervical ganglion, while the two or more grey rami communicantes pass from the inferior cervical ganglion to the seventh and eighth cervical ventral rami. The first thoracic ventral ramus receives its grey ramus from the cervicothoracic ganglion. Its for this reason that inferior plexus injury can be complicated by a Horners syndrome.

Summary
Origin Anterior rami of C5 to T1
Sections of the plexus
Roots, trunks, divisions, cords, branches
Mnemonic:Real Teenagers Drink Cold Beer
Roots
Located in the posterior triangle
Pass between scalenus anterior and medius
Trunks
Located posterior to middle third of clavicle
Upper and middle trunks related superiorly to the subclavian artery
Lower trunk passes over 1st rib posterior to the subclavian artery
Divisions Apex of axilla
Cords Related to axillary artery

1431
Q

A sprinter attends A&E with severe leg pain. He had forgotten to warm up and ran a 100m sprint race. Towards the end of the race he experienced pain in the posterior aspect of his thigh. The pain worsens, localising to the lateral aspect of the knee. The sprinter is unable to flex the knee. What structure has been injured?

Anterior cruciate ligament

Posterior cruciate ligament

Semimembranosus tendon

Semitendinosus tendon

Biceps femoris tendon

A

The biceps femoris is commonly injured in sports that require explosive bending of the knee as seen in sprinting, especially if the athlete has not warmed up first. Avulsion most commonly occurs where the long head attaches to the ischial tuberosity. Injuries to biceps femoris are more common than to the other hamstrings.

Biceps femoris

The biceps femoris is one of the hamstring group of muscles located in the posterior upper thigh. It has two heads.

Long head
Origin Ischial tuberosity
Insertion Fibular head
Action Knee flexion, lateral rotation tibia, extension hip
Innervation Tibial division of sciatic nerve (L5, S1, S2)
Arterial supply Profunda femoris artery, inferior gluteal artery, and the superior muscular branches of popliteal artery

Short head
Origin Lateral lip of linea aspera, lateral supracondylar ridge of femur
Insertion Fibular head
Action Knee flexion, lateral rotation tibia
Innervation Common peroneal division of sciatic nerve (L5, S1, S2)
Arterial supply Profunda femoris artery, inferior gluteal artery, and the superior muscular branches of popliteal artery

1432
Q

A 72 year old man with non reconstructible arterial disease is undergoing an above knee amputation. The posterior compartment muscles are divided. Which of the following muscles does not lie in the posterior compartment of the thigh?

Biceps femoris

Quadriceps femoris

Semitendinosus

Semimembranosus

None of the above

A

The quadriceps femoris lies in the anterior compartment.
Fascial compartments of the leg

Compartments of the thigh

Formed by septae passing from the femur to the fascia lata.
Compartment Nerve Muscles Blood supply
Anterior compartment Femoral
Iliacus
Tensor fasciae latae
Sartorius
Quadriceps femoris
Femoral artery
Medial compartment Obturator
Adductor longus/magnus/brevis
Gracilis
Obturator externus
Profunda femoris artery and obturator artery
Posterior compartment (2 layers) Sciatic
Semimembranosus
Semitendinosus
Biceps femoris
Branches of Profunda femoris artery

Compartments of the lower leg
Separated by the interosseous membrane (anterior and posterior compartments), anterior fascial septum (separate anterior and lateral compartments) and posterior fascial septum (separate lateral and posterior compartments)

Compartment Nerve Muscles Blood supply
Anterior compartment Deep peroneal nerve
Tibialis anterior
Extensor digitorum longus
Extensor hallucis longus
Peroneus tertius
Anterior tibial artery
Posterior compartment Tibial
Muscles: deep and superficial compartments (separated by deep transverse fascia)
Deep: Flexor hallucis longus, Flexor digitalis longus, Tibialis posterior, Popliteus
Superficial: Gastrocnemius, Soleus, Plantaris
Posterior tibial
Lateral compartment Superficial peroneal
Peroneus longus/brevis
Peroneal artery

1433
Q

At what age should children with otherwise uncomplicated umbilical hernias be considered for surgery?

Within the first few days of life

6 months of age

12 months of age

18 months of age

After 3 years of age

A

Many umbilical hernias will close in the first year of life. Defer surgery until the child is 3 years or older.

Paediatric umbilical disorders

Embryology
During development the umbilicus has two umbilical arteries and one umbilical vein. The arteries are continuous with the internal iliac arteries and the vein is continuous with the falciform ligament (ductus venosus). After birth the cord dessicates and separates and the umbilical ring closes.

Umbilical hernia
Up to 20% of neonates may have an umbilical hernia, it is more common in premature infants. The majority of these hernias will close spontaneously (may take between 12 months and three years). Strangulation is rare.

Paraumbilical hernia
These are due to defects in the linea alba that are in close proximity to the umbilicus. The edges of a paraumbilical hernia are more clearly defined than those of an umbilical hernia. They are less likely to resolve spontaneously than an umbilical hernia.

Omphalitis
This condition consists of infection of the umbilicus. Infection with Staphylococcus aureus is the commonest cause. The condition is potentially serious as infection may spread rapidly through the umbilical vessels in neonates with a risk of portal pyaemia, and portal vein thrombosis. Treatment is usually with a combination of topical and systemic antibiotics.

Umbilical granuloma
These consist of cherry red lesions surrounding the umbilicus, they may bleed on contact and be a site of seropurulent discharge. Infection is unusual and they will often respond favorably to chemical cautery with topically applied silver nitrate.

Persistent urachus
This is characterised by urinary discharge from the umbilicus. It is caused by persistence of the urachus which attaches to the bladder. They are associated with other urogenital abnormalities.

Persistent vitello-intestinal duct
This will typically present as an umbilical discharge that discharges small bowel content. Complete persistence of the duct is a rare condition. Much more common is the persistence of part of the duct (Meckels diverticulum). Persistent vitello-intestinal ducts are best imaged using a contrast study to delineate the anatomy and are managed by laparotomy and surgical closure.

1434
Q

A 34 year old man with a submandibular gland stone is undergoing excision of the submandibular gland. The incision is sited transversely approximately 4cm below the mandible. After incising the skin, platysma and deep fascia which of the following structures is most likely to be encountered?

Facial artery

Facial vein

Lingual nerve

Hypoglossal nerve

Glossopharyngeal nerve

A

When approaching the submandibular gland the facial vein and submandibular lymph nodes are the most superficially encountered structures. Each sub mandibular gland has a superficial and deep part, separated by the mylohyoid muscle. The facial artery passes in a groove on the superficial aspect of the gland. It then emerges onto the surface of the face by passing between the gland and the mandible. The facial vein is encountered first in this surgical approach because the incision is made 4cm below the mandible (to avoid injury to the marginal mandibular nerve).

Submandibular gland

Relations of the submandibular gland
Superficial Platysma, deep fascia and mandible
Submandibular lymph nodes
Facial vein (facial artery near mandible)
Marginal mandibular nerve
Cervical branch of the facial nerve
Deep Facial artery (inferior to the mandible)
Mylohyoid muscle
Sub mandibular duct
Hyoglossus muscle
Lingual nerve
Submandibular ganglion
Hypoglossal nerve

Submandibular duct (Wharton’s duct)
Opens lateral to the lingual frenulum on the anterior floor of mouth.
5 cm length
Lingual nerve wraps around Wharton’s duct. As the duct passes forwards it crosses medial to the nerve to lie above it and then crosses back, lateral to it, to reach a position below the nerve.

Innervation
Sympathetic innervation- Derived from superior cervical ganglion
Parasympathetic innervation- Submandibular ganglion via lingual nerve

Arterial supply
Branch of the facial artery. The facial artery passes through the gland to groove its deep surface. It then emerges onto the face by passing between the gland and the mandible.

Venous drainage
Anterior facial vein (lies deep to the Marginal Mandibular nerve)

Lymphatic drainage
Deep cervical and jugular chains of nodes

1435
Q

A middle aged lady is brought to the clinic by her husband who has noted a change in her appearance. She finds removal of rings difficult, her shoe size has changed and photographs show a marked change in her appearance. Which of the following is most likely to be identified on neurological examination?

Bi nasal hemianopia

Bi temporal hemianopia

Inferior quadrantanopia

Homonymous hemianopia

Unilateral loss of vision

A

B-Bi temporal hemianopia
The patient is most likely to have developed acromegaly. Since a pituitary lesion is likely to be present; compression of the optic chiasm may occur.

Visual field defects

left homonymous hemianopia means visual field defect to the left, i.e. Lesion of right optic tract
homonymous quadrantanopias: PITS (Parietal-Inferior, Temporal-Superior)
incongruous defects = optic tract lesion; congruous defects = optic radiation lesion or occipital cortex

Homonymous hemianopia
Incongruous defects: lesion of optic tract
Congruous defects: lesion of optic radiation or occipital cortex
Macula sparing: lesion of occipital cortex

Homonymous quadrantanopias
Superior: lesion of temporal lobe
Inferior: lesion of parietal lobe
Mnemonic = PITS (Parietal-Inferior, Temporal-Superior)

Bitemporal hemianopia
Lesion of optic chiasm
Upper quadrant defect > lower quadrant defect = inferior chiasmal compression, commonly a pituitary tumour
Lower quadrant defect > upper quadrant defect = superior chiasmal compression, commonly a craniopharyngioma

1436
Q

A 48 year old lady is being prepared for a Whipples procedure. A right sided subclavian line is inserted and then anaesthesia is induced. Following intubation the patient becomes progressively hypoxic and haemodynamically unstable. What is the most likely underlying explanation?

Drug allergy

Simple pneumothorax

Tension pneumothorax

Halothane toxicity

Haemothorax

A

Central lines (and particularly subclavian lines) are risk factors for the development of pneumothorax. In the context of positive pressure ventilation a tension pneumothorax is a strong possibility and would be associated with haemodynamic instability.

Intravenous access

Venous access
A number of routes for establishing venous access are available.

Peripheral venous cannula
Easy to insert with minimal morbidity. Wide lumen cannulae can provide rapid fluid infusions. When properly managed infections may be promptly identified and the cannula easily re sited. Problems relate to their peripheral sites and they are unsuitable for the administration of vaso active drugs, such as inotropes and irritant drugs such as TPN (except in the very short term setting).

Central lines
Insertion is more difficult and most operators and NICE advocate the use of ultra sound. Coagulopathies may lead to haemorrhage following iatrogenic arterial injury. Femoral lines are easier to insert and iatrogenic injuries easier to manage in this site however they are prone to high infection rates. Internal jugular route is preferred. They have multiple lumens allowing for administration of multiple infusions. The lumens are relatively narrow and thus they do not allow particularly rapid rates of infusion.

Intraosseous access
This is typically undertaken at the anteromedial aspect of the proximal tibia and provides access to the marrow cavity and circulatory system. Although traditionally preferred in paediatric practice they may be used in adults and a wide range of fluids can be infused using these devices.

Tunneled lines
Tunneled lines such as Groshong and Hickman lines are popular devices for patients with long term therapeutic requirements. These devices are usually inserted using ultrasound guidance into the internal jugular vein and then tunneled under the skin. A cuff of woven material is sited near the end and helps to anchor the device into the tissues. These cuffs require formal dissection to allow the device to be removed. Tunneled lines can be linked to injection ports that are located under the skin. These are especially popular in paediatric practice.

Peripherally inserted central cannula
Referred to as PICC lines, these are popular methods for establishing central venous access. Because they are inserted peripherally they are less prone to major complications relating to device insertion than conventional central lines.

1437
Q

A 43 year old lady has recently undergone a wide local excision and sentinel lymph node biopsy for carcinoma of the breast. Of the factors listed below, which will provide the most important prognostic information?

Mitotic number

Grade

Nodal status

Size

Oestrogen receptor status

A

Nodal status is the single most important prognostic factor in breast cancer.
Nodal status is important because it serves as a marker of tumour metastatic potential. This translates to survival advantages of up to 40% at five years. Both grade and size are of secondary importance as they both less concerning in the absence of nodal involvement.

Breast cancer

  • Commoner in the older age group
    Invasive ductal carcinomas are the most common type. Some may arise as a result of ductal carcinoma in situ (DCIS). There are associated carcinomas of special type e.g. Tubular that may carry better prognosis.
    The pathological assessment involves assessment of the tumour and lymph nodes, sentinel lymph node biopsy is often used to minimise the morbidity of an axillary dissection.
    Treatment, typically this is either wide local excision or mastectomy. There are many sub types of both of these that fall outside of the MRCS. Some key rules to bear in mind.
    Whatever operation is contemplated the final cosmetic outcome does have a bearing. A woman with small breasts and a large tumour will tend to fare better with mastectomy, even if clear pathological and clinical margins can be obtained. Conversely a women with larger breasts may be able to undergo breast conserving surgery even with a relatively large primary lesion (NB tumours >4cm used to attract recommendation for mastectomy). For screen detected and impalpable tumour image guidance will be necessary.
    Reconstruction is always an option following any resectional procedure. However, its exact type must be tailored to age and co-morbidities of the patient. The main operations in common use include latissimus dorsi myocutaneous flap and sub pectoral implants. Women wishing to avoid a prosthesis may be offered TRAM or DIEP flaps.

Surgical options
Mastectomy vs Wide local excision

Mastectomy Wide Local Excision
Multifocal tumour Solitary lesion
Central tumour Peripheral tumour
Large lesion in small breast Small lesion in large breast
DCIS >4cm DCIS <4cm
Patient Choice Patient choice

Central lesions may be managed using breast conserving surgery where an acceptable cosmetic result may be obtained, this is rarely the case in small breasts

Whatever surgical option is chosen the aim should be to have a local recurrence rate of 5% or less at 5 years [1].

Nottingham Prognostic Index
The Nottingham Prognostic Index can be used to give an indication of survival. In this system the tumour size is weighted less heavily than other major prognostic parameters.

Calculation of NPI
Tumour Size x 0.2 + Lymph node score(From table below)+Grade score(From table below).

Score Lymph nodes involved Grade
1 0 1
2 1-3 2
3 >3 3

Prognosis

Score Percentage 5 year survival
2.0 to 2.4 93%
2.5 to 3.4 85%
3.5 to 5.4 70%
>5.4 50%

This data was originally published in 1992. It should be emphasised that other factors such as vascular invasion and receptor status also impact on survival and are not included in this data and account for varying prognoses often cited in the literature.

References
For guidance on how breast cancer is managed in the United Kingdom visit the Association of Breast Surgery website (www.https://associationofbreastsurgery.org.uk/).

1438
Q

In matching donated kidneys to the most appropriate recipient, apart from ABO matching, which of the following is most important?

HLA DR

Rhesus

HLA A

HLA B

Duffy antigen

A

The rhesus group is not important in matching donor and recipient kidneys.

Renal transplant:HLA typing and graft failure

The human leucocyte antigen (HLA) system is the name given to the major histocompatibility complex (MHC) in humans. It is coded for on chromosome 6.

Some basic points on the HLA system
Class 1 antigens include A, B and C. Class 2 antigens include DP,DQ and DR
When HLA matching for a renal transplant the relative importance of the HLA antigens are as follows DR > B > A

Graft survival
1 year = 90%, 10 years = 60% for cadaveric transplants
1 year = 95%, 10 years = 70% for living-donor transplants

Post-op problems
ATN of graft
Vascular thrombosis
Urine leakage
UTI

Hyperacute acute rejection
Due to antibodies against donor HLA type 1 antigens
Rarely seen due to HLA matching

Acute graft failure (< 6 months)
Usually due to mismatched HLA
Other causes include cytomegalovirus infection
Management: give steroids, if resistant use monoclonal antibodies

Causes of chronic graft failure (> 6 months)
Chronic allograft nephropathy
Ureteric obstruction
Recurrence of original renal disease (MCGN > IgA > FSGS)

1439
Q

A 34 year old woman trips over and falls into a bonfire whilst intoxicated at a party. She suffers burns to her arms, torso and face. These are calculated to be 25% body surface area. She is otherwise stable. The burns to the torso are superficial, her left forearm has a full thickness burn and the burns to her face are superficial. There is no airway compromise. She has received 1000ml of intravenous Hartman’s solution, with a further 1000ml prescribed to run over 4 hours. What should be the next course of action?

Undertake an escharotomy

Undertake debridement and skin grafting

Transfer to a regional burns unit
6
Intubate and admit to intensive care

Discharge home with daily review

A

This woman has been resuscitated and requires transfer for specialist management.

Burns

Burns may be thermal, chemical or electrical. In the former category are burns which occur as a result of heat. Chemical burns occur when the skin is exposed to an extremely caustic or alkaline substance. Electrical burns occur following exposure to electrical current. The immediate management includes removal of the burning source which usually includes irrigation of the burned area. A detailed assessment then needs to be made of the extent of the burns and a number of charts are available for recording this information. The degree of injury relates to the temperature and duration of exposure. Most domestic burns are mainly scalds in young children.

Following the burn, there is a local response with progressive tissue loss and release of inflammatory cytokines. Systemically, there are cardiovascular effects resulting from fluid loss and sequestration of fluid into the third space. There is a marked catabolic response. Immunosuppression is common with large burns and bacterial translocation from the gut lumen is a recognised event. Sepsis is a common cause of death following major burns.

Types of burn

Type of burn Skin layers affected Skin appearance Blanching Management
Epidermal/Superficial Epidermis Red, moist Yes
Superficial partial thickness Epidermis and part of papillary dermis affected Pale, dry Yes Normally heals with no intervention
Deep partial thickness Epidermis, whole papillary dermis affected Mottled red colour No Needs surgical intervention (depending on site)
Full thickness Whole skin layer and subcutaneous tissue affected Dry, leathery hard wound No Burns centre

Depth of burn assessment
Bleeding on needle prick
Sensation
Appearance
Blanching to pressure

Percentage burn estimation
Lund Browder chart: most accurate even in children
Wallace rule of nines
Palmar surface: surface area palm = 0.8% burn

> 15% body surface area burns in adults needs urgent burn fluid resuscitation

Transfer to burn centre if:
Need burn shock resuscitation
Face/hands/genitals affected
Deep partial thickness or full thickness burns
Significant electrical/chemical burns

Management
The initial aim is to stop the burning process and resuscitate the patient. Intravenous fluids will be required for children with burns greater than 10% of total body surface area. Adults with burns greater than 15% of total body surface area will also require IV fluids. The fluids are calculated using the Parkland formula which is; volume of fluid= total body surface area of the burn % x weight (Kg) x2-4 (preference for lower amount i.e. 2ml to avoid excessive fluid overload). Half of the fluid is administered in the first 8 hours. A urinary catheter should be inserted. Analgesia should be given. Complex burns, burns involving the hand perineum and face and burns >10% in adults and >5% in children should be transferred to a burns unit.

Circumferential burns affecting a limb or severe torso burns impeding respiration may require escharotomy to divide the burnt tissue.

Conservative management is appropriate for superficial burns and mixed superficial burns that will heal in 2 weeks. More complex burns may require excision and skin grafting. Excision and primary closure is not generally practised as there is a high risk of infection.

There is no evidence to support the use of anti microbial prophylaxis or topical antibiotics in burn patients.

Escharotomies
Indicated in circumferential full thickness burns to the torso or limbs.
Careful division of the encasing band of burn tissue will potentially improve ventilation (if the burn involves the torso), or relieve compartment syndrome and oedema (where a limb is involved)

References
www.euroburn.org/e107files/downloads/guidelinesburncare.pdf

Barajas-Nava LA, López-Alcalde J, Roqué i Figuls M, Solà I, Bonfill Cosp X. Antibiotic prophylaxis for preventing burn wound infection. Cochrane Database of Systematic Reviews 2013, Issue 6. Art. No.: CD008738. DOI: 10.1002/14651858.CD008738.pub2.

Hettiaratchy S & Papini R. Initial management of a major burn: assessment and resuscitation. BMJ 2004;329:101-103

1440
Q

A 68 year old man with critical limb ischaemia is undergoing a femoro-distal bypass graft. During mobilisation of the proximal part of the posterior tibial artery which of the following is at greatest risk of injury?

Tibial nerve

Sciatic nerve

Saphenous nerve

Common peroneal nerve

Medial superior genicular artery

A

The tibial nerve is closely related to the posterior tibial artery. The tibial nerve crosses the vessel posteriorly approximately 2.5cm distal to its origin. At its origin the nerve lies medial and then lateral after it crosses the vessel as described.
Posterior tibial artery

Larger terminal branch of the popliteal artery
Terminates by dividing into the medial and lateral plantar arteries
Accompanied by two veins throughout its length
Position of the artery corresponds to a line drawn from the lower angle of the popliteal fossa, at the level of the neck of the fibula, to a point midway between the medial malleolus and the most prominent part of the heel

Relations of the posterior tibial artery
Proximal to distal
Anteriorly Tibialis posterior
Flexor digitorum longus
Posterior surface of tibia and ankle joint
Posterior Tibial nerve 2.5 cm distal to its origin
Fascia overlying the deep muscular layer
Proximal part covered by gastrocnemius and soleus
Distal part covered by skin and fascia

1441
Q

A 72 year old man undergoes a ray amputation of the foot for an infection complicating long standing diabetes. What is the best method for healing the resultant wound?

Compression bandaging

Vacuum assisted closure system

Split thickness graft

Primary closure with 2/0 polypropylene

Full thickness skin graft

A

Ray amputations for diabetic foot infections do not heal well and should never be primarily closed. The use of vacuum assisted closure devices has been shown to improve healing rates.

Methods of wound closure

Method of closure Indication
Primary closure
Clean wound, usually surgically created or following minor trauma
Standard suturing methods will usually suffice
Wound heals by primary intention
Delayed primary closure
Similar methods of actual closure to primary closure
May be used in situations where primary closure is either not achievable or not advisable e.g. infection
Vacuum assisted closure
Uses negative pressure therapy to facilitate wound closure
Sponge is inserted into wound cavity and then negative pressure applied
Advantages include removal of exudate and versatility
Disadvantages include cost and risk of fistulation if used incorrectly on sites such as bowel
Split thickness skin grafts
Superficial dermis removed with Watson knife or dermatome (commonly from thigh)
Remaining epithelium regenerates from dermal appendages
Coverage may be increased by meshing
Full thickness skin grafts
Whole dermal thickness is removed
Sub dermal fat is then removed and graft placed over donor site
Better cosmesis and flexibility at recipient site
Donor site morbidity
Flaps
Viable tissue with a blood supply
May be pedicled or free
Pedicled flaps are more reliable, but limited in range
Free flaps have greater range but carry greater risk of breakdown as they require vascular anastomosis

1442
Q

Which of the following statements relating to the regulation of renal blood flow is untrue?

In a healthy 70Kg male, the glomerular filtration rate will be the same at a systolic blood pressure of 120mmHg as a systolic blood pressure of 95 mmHg

Over 90% of the blood supply to the kidney is distributed to the cortex

Both kidneys receives approximately 25% of the total cardiac output at rest

A decrease in renal perfusion pressure will cause the juxtaglomerular cells to secrete renin

Systolic blood pressures of less than 65mmHg will cause the mesangial cells to secrete aldosterone

A

The kidney autoregulates its blood supply over a range of systolic blood pressures. Drop in arterial pressure is sensed by the juxtaglomerular cells and renin is released leading to the activation of the renin-angiontensin system. Mesangial cells are contractile cells that are located in the tubule and have no direct endocrine function.

Renal Physiology

Overview
Each nephron is supplied with blood from an afferent arteriole that opens onto the glomerular capillary bed.
Blood then flows to an efferent arteriole, supplying the peritubular capillaries and medullary vasa recta.
The kidney receives up to 25% of resting cardiac output.

Control of blood flow
The kidney is able to autoregulate its blood flow between systolic pressures of 80- 180mmHg so there is little variation in renal blood flow.
This is achieved by myogenic control of arteriolar tone, both sympathetic input and hormonal signals (e.g. renin) are responsible.

Glomerular structure and function
Blood inside the glomerulus has considerable hydrostatic pressure.
The basement membrane has pores that will allow free diffusion of smaller solutes, larger negatively charged molecules such as albumin are unable to cross.
The glomerular filtration rate (GFR) is equal to the concentration of a solute in the urine, times the volume of urine produced per minute, divided by the plasma concentration (assuming that the solute is freely diffused e.g. inulin).
In clinical practice creatinine is used because it is subjected to very little proximal tubular secretion.
Although subject to variability, the typical GFR is 125ml per minute.
Glomerular filtration rate = Total volume of plasma per unit time leaving the capillaries and entering the bowman’s capsule
Renal clearance = volume plasma from which a substance is removed per minute by the kidneys

Substances used to measure GFR have the following features:
1. Inert
2. Free filtration from the plasma at the glomerulus (not protein bound)
3. Not absorbed or secreted at the tubules
4. Plasma concentration constant during urine collection

Examples: inulin, creatinine

plasma concentration (mmol/l)

The clearance of a substance is dependent not only on its diffusivity across the basement membrane but also subsequent tubular secretion and / or reabsorption.
So glucose which is freely filtered across the basement membrane is usually reabsorbed from tubules giving a clearance of zero.

Tubular function
Reabsorption and secretion of substances occurs in the tubules.
In the proximal tubule substrates such as glucose, amino acids and phosphate are co-transported with sodium across the semi permeable membrane.
Glucose is reabsorbed by both active and passive processes. The former uses sodium–glucose co-transporters (SGLTs) in the proximal convoluted tubule. The latter achieved through facilitated and passive process through GLUT transporters.
Up to two thirds of filtered water is reabsorbed in the proximal tubules.
This will lead to increase in urea concentration in the distal tubule allowing for its increased diffusion.
Substances to be secreted into the tubules are taken up from the peritubular blood by tubular cells.
Solutes such as paraaminohippuric acid are cleared with a single passage through the kidneys and this is why it is used to measure renal plasma flow. Ions such as calcium and phosphate will have a tubular reabsorption that is influenced by plasma PTH levels.
Potassium may be both secreted and re-absorbed and is co-exchanged with sodium.

Loop of Henle
Approximately 60 litres of water containing 9000mmol sodium enters the descending limb of the loop of Henle in 24 hours.
Loops from the juxtamedullary nephrons run deep into the medulla.
The osmolarity of fluid changes and is greatest at the tip of the papilla.
The thick ascending limb is impermeable to water, but highly permeable to sodium and chloride ions.
This loss means that at the beginning of the thick ascending limb the fluid is hypo osmotic compared with adjacent interstitial fluid.
In the thick ascending limb the reabsorption of sodium and chloride ions occurs by both facilitated and passive diffusion pathways.
The loops of Henle are co-located with vasa recta, these will have similar solute compositions to the surrounding extracellular fluid so preventing the diffusion and subsequent removal of this hypertonic fluid.
The energy dependent reabsorption of sodium and chloride in the thick ascending limb helps to maintain this osmotic gradient.

1443
Q

Which of the following is the first radiological change likely to be apparent in a plain radiograph of a 12 year old presenting with suspected Perthes disease

Multiple bone cysts

Sclerosis of the femoral head

Loss of bone density

Joint space narrowing

Collapse of the femoral head

A

In Catterall stage I disease there may be no radiological abnormality at all. In Stage II disease there may be sclerosis of the femoral head.

Indication for treatment (aide memoire):Half a dozen, half a head
Those aged greater than 6 years with >50% involvement of the femoral head should almost always be treated.

Perthes disease

Perthes disease
Idiopathic avascular necrosis of the femoral epiphysis of the femoral head
Impaired blood supply to femoral head, causing bone infarction. New vessels develop and ossification occurs. The bone either heals or a subchondral fracture occurs.

Clinical features
Males 4x’s greater than females
Age between 2-12 years (the younger the age of onset, the better the prognosis)
Limp
Hip pain
Bilateral in 20%

Diagnosis
Plain x-ray, Technetium bone scan or magnetic resonance imaging if normal x-ray and symptoms persist.

Catterall staging
Stage Features
Stage 1 Clinical and histological features only
Stage 2 Sclerosis with or without cystic changes and preservation of the articular surface
Stage 3 Loss of structural integrity of the femoral head
Stage 4 Loss of acetabular integrity

Management
To keep the femoral head within the acetabulum: cast, braces
If less than 6 years: observation
Older: surgical management with moderate results
Operate on severe deformities

Prognosis
Most cases will resolve with conservative management. Early diagnosis improves outcomes.

1444
Q

A 23 year old man presents with appendicitis. A decision is made to perform an appendicectomy. The operation commences with a 5cm incision centered on McBurneys point. Which of the following structures will be encountered first during the dissection?

External oblique aponeurosis

Internal oblique muscle

Transversalis fascia

Rectus sheath

Peritoneum

A

The external oblique will be encountered first in this location. The rectus sheath lies more medially.
The external oblique muscle is the most superficial of the abdominal wall muscles. It originates from the 5th to 12th ribs and passes inferomedially to insert into the linea alba, pubic tubercle and anterior half of the iliac crest. It is innervated by the thoracoabdominal nerves (T7-T11) and sub costal nerves.

Abdominal wall

The 2 main muscles of the abdominal wall are the rectus abdominis (anterior) and the quadratus lumborum (posterior).
The remaining abdominal wall consists of 3 muscular layers. Each muscle passes from the lateral aspect of the quadratus lumborum posteriorly to the lateral margin of the rectus sheath anteriorly. Each layer is muscular posterolaterally and aponeurotic anteriorly.

Muscles of abdominal wall
External oblique
Lies most superficially
Originates from 5th to 12th ribs
Inserts into the anterior half of the outer aspect of the iliac crest, linea alba and pubic tubercle
More medially and superiorly to the arcuate line, the aponeurotic layer overlaps the rectus abdominis muscle
The lower border forms the inguinal ligament
The triangular expansion of the medial end of the inguinal ligament is the lacunar ligament.
Internal oblique
Arises from the thoracolumbar fascia, the anterior 2/3 of the iliac crest and the lateral 2/3 of the inguinal ligament
The muscle sweeps upwards to insert into the cartilages of the lower 3 ribs
The lower fibres form an aponeurosis that runs from the tenth costal cartilage to the body of the pubis
At its lowermost aspect it joins the fibres of the aponeurosis of transversus abdominis to form the conjoint tendon.
Transversus abdominis
Innermost muscle
Arises from the inner aspect of the costal cartilages of the lower 6 ribs , from the anterior 2/3 of the iliac crest and lateral 1/3 of the inguinal ligament
Its fibres run horizontally around the abdominal wall ending in an aponeurosis. The upper part runs posterior to the rectus abdominis. Lower down the fibres run anteriorly only.
The rectus abdominis lies medially; running from the pubic crest and symphysis to insert into the xiphoid process and 5th, 6th and 7th costal cartilages. The muscles lies in a aponeurosis as described above.
Nerve supply: anterior primary rami of T7-12

Surgical notes
During abdominal surgery it is usually necessary to divide either the muscles or their aponeuroses. During a midline laparotomy it is desirable to divide the aponeurosis. This will leave the rectus sheath intact above the arcuate line and the muscles intact below it. Straying off the midline will often lead to damage to the rectus muscles, particularly below the arcuate line where they may often be in close proximity to each other.

1445
Q

What proportion of patients presenting for cholecystectomy for treatment of biliary colic due to gallstones will have stones in the common bile duct?

10%

30%

2%

50%

25%

A

Up to 10% of all patients may have stones in the CBD. Therefore, all patients should have their liver function tests checked prior to embarking on a cholecystectomy.
Biliary disease

Diagnosis Typical features Pathogenesis
Gallstones Typically history of biliary colic or episodes of chlolecystitis. Obstructive type history and test results. Usually small calibre gallstones which can pass through the cystic duct. In Mirizzi syndrome the stone may compress the bile duct directly- one of the rare times that cholecystitis may present with jaundice
Cholangitis Usually obstructive and will have Charcot’s triad of symptoms (pain, fever, jaundice) Ascending infection of the bile ducts usually by E. coli and by definition occurring in a pool of stagnant bile.
Pancreatic cancer Typically painless jaundice with palpable gallbladder (Courvoisier’s Law) Direct occlusion of distal bile duct or pancreatic duct by tumour. Sometimes nodal disease at the portal hepatis may be the culprit in which case the bile duct may be of normal calibre.
TPN (total parenteral nutrition) associated jaundice Usually follows long term use and is usually painless with non obstructive features Often due to hepatic dysfunction and fatty liver which may occur with long term TPN usage.
Bile duct injury Depending upon the type of injury may be of sudden or gradual onset and is usually of obstructive type Often due to a difficult laparoscopic cholecystectomy when anatomy in Calots triangle is not appreciated. In the worst scenario the bile duct is excised and jaundice develops rapidly post operatively. More insidious is that of bile duct stenosis which may be caused by clips or diathermy injury.
Cholangiocarcinoma Gradual onset obstructive pattern Direct occlusion by disease and also extrinsic compression by nodal disease at the porta hepatis.
Septic surgical patient Usually hepatic features Combination of impaired biliary excretion and drugs such as ciprofloxacin which may cause cholestasis.
Metastatic disease Mixed hepatic and post hepatic Combination of liver synthetic failure (late) and extrinsic compression by nodal disease and anatomical compression of intra hepatic structures (earlier)

A gallbladder may develop a thickened wall in chronic cholecystitis, microscopically Roikitansky-Aschoff Sinuses may be seen

1446
Q

A 36 year old man is admitted following a major electrical injury. He is assessed as having a second degree burn which affects 30% of the total body surface area and full thickness burn which affects 5% of the total body surface area. He is catheterized and the catheter drains 50ml of dark coloured urine. Which of the resuscitation protocols below is most appropriate in this case?

Administration of 4ml/kg/%total body surface area of Hartmanns solution to achieve a urine output of 100ml per hour

Administration of 2ml/kg/%total body surface area of Hartmanns solution to achieve a urine output of 50ml per hour

Administration of 1ml/kg/%total body surface area of Hartmanns solution to achieve a urine output of 70ml per hour

Administration of 2ml/kg/%total body surface area of Hartmanns solution to achieve a urine output of 100ml per hour

Administration of 10ml/kg/%total body surface area of Hartmanns solution to achieve a urine output of 500ml per hour

A

Electrical burn infusion protocols are different to those for flame or scald burns as higher volumes of fluid are given with increased targets for urine output to minimize the effects of rhabdomyolysis.

Fluid resuscitation burns

Indication: >15% total body area burns in adults (>10% children)
The main aim of resuscitation is to prevent the burn deepening
Most fluid is lost 24 hours after injury
First 8-12 hour fluid shifts are from intravascular to interstitial fluid compartments
Therefore circulatory volume can be compromised. However fluid resuscitation causes more fluid into the interstitial compartment especially colloid (therefore avoided in first 8-24 hours)
Protein loss occurs

There are concerns that use of the traditional Parkland formula resulted in the administration of excessive quantities of intravenous fluids in patients with burns. The current consensus guidelines state that fluid resuscitation should begin at 2 ml of lactated Ringers x patients body weight in kg x % TBSA for second- and third-degree burns.
The calculated fluid volume is initiated in the following manner: one-half of the total fluid is provided in the first 8 hours after the burn injury (for example, a 100-kg man with 80% TBSA burns requires 2 × 80 × 100 = 16,000 mL in 24 hours). One-half of that volume (8,000 mL) should be provided in the first 8 hours, so the patient should be started at a rate of 1000 mL/hr. The remaining one-half of the total fluid is administered during the subsequent 16 hours.
The efficacy of fluid replacement is determined in the main by monitoring urine output. For adults this is in the region of 0.5ml/kg/hr and for children less than 30Kg it is 1ml/kg/hr.

Fluid resuscitation according to burn type
Type of burn Age or weight Fluid rate Urine output
Flames or scalding Adults and children over 14 years 2 ml Hartmanns x kg x % TBSA 0.3-0.5ml/kg/hr OR 30-50ml per hour
Flames or scalding Children less than 14 years 3 ml Hartmanns x kg x % TBSA 1ml/kg/hr
Flames or scalds Children less than 30Kg and infants 3 ml Hartmanns x kg x % TBSA. Plus a sugar-containing solution at maintenance rate 1ml/kg/hr
Electrical injury All ages 4 ml Hartmanns x kg x % TBSA until urine clears 1-1.5 ml/kg/hr until urine clears

Fluid resuscitation in electrical burn patients is different because they have far greater soft tissue involvement in muscle compartments and associated muscle death that can result in myoglobinuria.

After 24 hours
Maintenance crystalloid (usually dextrose-saline) is continued at a rate of 1.5 ml x(burn area)x(body weight)
Colloids are rarely used (e.g. albumin)
Antioxidants, such as vitamin C, can be used to minimize oxidant-mediated contributions to the inflammatory cascade in burns
High tension electrical injuries and inhalation injuries require more fluid
Monitor: packed cell volume, plasma sodium, base excess, and lactate

Reference
ATLS Manual 10th Edition.

1447
Q

Where does the spinal cord terminate in neonates?

L1

L2

L3

L4

L5

A

At the 3rd month the foetus’s spinal cord occupies the entire length of the vertebral canal. The vertebral column then grows longer exceeding the growth rate of the spinal cord. This results with the cord being at L3 at birth and L1-2 by adulthood.

Spinal cord

  • Located in a canal within the vertebral column that affords it structural support.
    Rostrally it continues to the medulla oblongata of the brain and caudally it tapers at a level corresponding to the L1-2 interspace (in the adult), a central structure, the filum terminale anchors the cord to the first coccygeal vertebra.
    The spinal cord is characterised by cervico-lumbar enlargements and these, broadly speaking, are the sites which correspond to the brachial and lumbar plexuses respectively.

There are some key points to note when considering the surgical anatomy of the spinal cord:

  • During foetal growth the spinal cord becomes shorter than the spinal canal, hence the adult site of cord termination at the L1-2 level.
  • Due to growth of the vertebral column the spine segmental levels may not always correspond to bony landmarks as they do in the cervical spine.
  • The spinal cord is incompletely divided into two symmetrical halves by a dorsal median sulcus and ventral median fissure. Grey matter surrounds a central canal that is continuous rostrally with the ventricular system of the CNS.
  • The grey matter is sub divided cytoarchitecturally into Rexeds laminae.
  • Afferent fibres entering through the dorsal roots usually terminate near their point of entry but may travel for varying distances in Lissauers tract. In this way they may establish synaptic connections over several levels
  • At the tip of the dorsal horn are afferents associated with nociceptive stimuli. The ventral horn contains neurones that innervate skeletal muscle.

The key point to remember when revising CNS anatomy is to keep a clinical perspective in mind. So it is worth classifying the ways in which the spinal cord may become injured. These include:

Trauma either direct or as a result of disc protrusion
Neoplasia either by direct invasion (rare) or as a result of pathological vertebral fracture
Inflammatory diseases such as Rheumatoid disease, or OA (formation of osteophytes compressing nerve roots etc.
Vascular either as a result of stroke (rare in cord) or as complication of aortic dissection
Infection historically diseases such as TB, epidural abscesses.

The anatomy of the cord will, to an extent dictate the clinical presentation. Some points/ conditions to remember:

Brown- Sequard syndrome-Hemisection of the cord producing ipsilateral loss of proprioception and upper motor neurone signs, plus contralateral loss of pain and temperature sensation. The explanation of this is that the fibres decussate at different levels.
Lesions below L1 will tend to present with lower motor neurone signs

1448
Q

A 42 year old motorcyclist is involved in a road traffic accident. A FAST scan in the emergency department shows free intrabdominal fluid and a laparotomy is performed. At operation there is evidence of small liver laceration that has stopped bleeding and a tear to the inferior pole of the spleen. What is the best course of action?

Proceed to splenectomy

Attempt measures to conserve the spleen

Resection of the inferior pole of the spleen

Ligate the splenic vein alone

Fully mobilise the spleen to inspect it

A

Mobilising the spleen will result in removal. Splenic injuries like this are amenable to conservation.

Splenic trauma

  • The spleen is one of the more commonly injured intra abdominal organs
    In most cases the spleen can be conserved. The management is dictated by the associated injuries, haemodynamic status and extent of direct splenic injury.

Management of splenic trauma
Conservative Small subcapsular haematoma
Minimal intra abdominal blood
No hilar disruption
Laparotomy with conservation Increased amounts of intraabdominal blood
Moderate haemodynamic compromise
Tears or lacerations affecting <50%
Resection Hilar injuries
Major haemorrhage
Major associated injuries

Splenectomy
Technique
Trauma
GA
Long midline incision
If time permits insert a self retaining retractor (e.g. Balfour/ omnitract)
Large amount of free blood is usually present. Pack all 4 quadrants of the abdomen. Allow the anaesthetist to ‘catch up’
Remove the packs and assess the viability of the spleen. Hilar injuries and extensive parenchymal lacerations will usually require splenectomy.
Divide the short gastric vessels and ligate them.
Clamp the splenic artery and vein. Two clamps on the patient side are better and allow for double ligation and serve as a safety net if your assistant does not release the clamp smoothly.
Be careful not to damage the tail of the pancreas, if you do then this will need to be formally removed and the pancreatic duct closed.
Wash out the abdomen and place a tube drain to the splenic bed.
Some surgeons implant a portion of spleen into the omentum, whether you decide to do this is a matter of personal choice.
Post operatively the patient will require prophylactic penicillin V and pneumococcal vaccine.

Elective
Elective splenectomy is a very different operation from that performed in the emergency setting. The spleen is often large (sometimes massive). Most cases can be performed laparoscopically. The spleen will often be macerated inside a specimen bag to facilitate extraction.

Complications
Haemorrhage (may be early and either from short gastrics or splenic hilar vessels
Pancreatic fistula (from iatrogenic damage to pancreatic tail)
Thrombocytosis: prophylactic aspirin
Encapsulated bacteria infection e.g. Strep. pneumoniae, Haemophilus influenzae and Neisseria meningitidis

1449
Q

A cachectic 32 year old man with severe perineal Crohns disease is receiving treatment with intravenous antibiotics. Over the past 72 hours he has complained of intermittent dysphagia and odynophagia. What is the most likely cause?

Oesophageal Crohns disease

Oesophageal candidiasis

Plummer Vinson syndrome

Globus

Achalasia

A

Treatment with systemic antibiotics may result in development of candidiasis. Patients may present with odynophagia and episodic dysphagia. Endoscopic appearances are usually diagnostic and treatment is with an oral anti fungal agent.

Dysphagia

Causes of dysphagia

Extrinsic
Mediastinal masses
Cervical spondylosis
Oesophageal wall
Achalasia
Diffuse oesophageal spasm
Hypertensive lower oesophageal sphincter
Intrinsic
Tumours
Strictures
Oesophageal web
Schatzki rings
Neurological
CVA
Parkinson’s disease
Multiple Sclerosis
Brainstem pathology
Myasthenia Gravis

Investigation
All patients require an upper GI endoscopy unless there are compelling reasons for this not to be performed. Motility disorders may be best appreciated by undertaking fluoroscopic swallowing studies.

A full blood count should be performed.

Ambulatory oesophageal pH and manometry studies will be required to evaluate conditions such as achalasia and patients with GORD being considered for fundoplication surgery.

1450
Q

A patient is found to have an ischaemic left colon. Which artery arising from the aorta at around the level of L3 is most likely to account for this situation?

Superior mesenteric artery

Inferior mesenteric artery

Superior rectal artery

Ileocolic artery

Middle colic artery

A

Only the IMA is likely to affect the left side of the colon and originate at L3.

Inferior mesenteric artery

The IMA is the main arterial supply of the embryonic hindgut and originates approximately 3-4 cm superior to the aortic bifurcation. From its aortic origin it passes immediately inferiorly across the anterior aspect of the aorta to eventually lie on its left hand side. At the level of the left common iliac artery it becomes the superior rectal artery.

Branches
The left colic artery arises from the IMA near its origin. More distally up to three sigmoid arteries will exit the IMA to supply the sigmoid colon.

1451
Q

A 48 year old woman with end stage renal failure is undergoing a live donor renal transplant. The surgeon decides to implant the kidney in the left iliac fossa via a Rutherford Morison incision. To which of the following vessels should the transplanted kidney be anastomosed?

Aorta and inferior vena cava

Internal iliac artery and vein

Common iliac artery and vein

External iliac artery and vein

Inferior epigastric artery and vein

A

First time renal tranplants are typically implanted in the left or right iliac fossae. The vessels are usually joined to the external iliac artery and vein as these are the most easily accessible. The Rutherford Morison incision provides access to the external iliac vessels.

Organ Transplant

A number of different organ and tissue transplants are now available. In many cases an allograft is performed, where an organ is transplanted from one individual to another. Allografts will elicit an immune response and this is one of the main reasons for organ rejection.

Graft rejection occurs because allografts have allelic differences at genes that code immunohistocompatability complex genes. The main antigens that give rise to rejection are:
ABO blood group
Human leucocyte antigens (HLA)
Minor histocompatability antigens

ABO Matching
ABO incompatibility will result in early organ rejection (hyperacute) because of pre existing antibodies to other groups. Group O donors can give organs to any type of ABO recipient whereas group AB donor can only donate to AB recipient.

HLA System
The four most important HLA alleles are:

HLA A
HLA B
HLA C
HLA DR

An ideal organ match would be one in which all 8 alleles are matched (remember 2 from each parent, four each = 8 alleles). Modern immunosuppressive regimes help to manage the potential rejection due to HLA mismatching. However, the greater the number of mismatches the worse the long term outcome will be. T lymphocytes will recognise antigens bound to HLA molecules and will then become activated. Clonal expansion then occurs with a response directed against that antigen.

Types of organ rejection
Hyperacute. This occurs immediately through presence of pre formed antibodies (such as ABO incompatibility).
Acute. Occurs during the first 6 months and is usually T cell mediated. Usually tissue infiltrates and vascular lesions.
Chronic. Occurs after the first 6 months. Vascular changes predominate.

Hyperacute
Renal transplants at greatest risk and liver transplants at least risk. Although ABO incompatibility and HLA Class I incompatible transplants will all fare worse in long term.

Acute
All organs may undergo acute rejection. Mononuclear cell infiltrates predominate. All types of transplanted organ are susceptible and it may occur in up to 50% cases.

Chronic
Again all transplants with HLA mismatch may suffer this fate. Previous acute rejections and other immunosensitising events all increase the risk. Vascular changes are most prominent with myointimal proliferation leading to organ ischaemia. Organ specific changes are also seen such as loss of acinar cells in pancreas transplants and rapidly progressive coronary artery disease in cardiac transplants.

Surgical overview-Renal transplantation
A brief overview of the steps involved in renal transplantation is given.
Patients with end stage renal failure who are dialysis dependent or likely to become so in the immediate future are considered for transplant. Exclusion criteria include; active malignancy, old age (due to limited organ availability). Patients are medically optimised.
Donor kidneys, these may be taken from live related donors and close family, members may have less HLA mismatch than members of the general population. Laparoscopic donor nephrectomy further minimises the operative morbidity for the donor. Other organs are typically taken from brain dead or dying patients who have a cardiac arrest and in whom resuscitation is futile. The key event is to minimise the warm ischaemic time in the donor phase.

The kidney once removed is usually prepared on the bench in theatre by the transplant surgeon immediately prior to implantation and factors such as accessory renal arteries and vessel length are assessed and managed.

For first time recipients the operation is performed under general anaesthesia. A Rutherford-Morison incision is made on the preferred side. This provides excellent extraperitoneal access to the iliac vessels. The external iliac artery and vein are dissected out and following systemic heparinisation are cross clamped. The vein and artery are anastamosed to the iliacs and the clamps removed. The ureter is then implanted into the bladder and a stent is usually placed to maintain patency. The wounds are then closed and the patient recovered from surgery.

In the immediate phase a common problem encountered in cadaveric kidneys is acute tubular necrosis and this tends to resolve.

Graft survival times from cadaveric donors are typically of the order of 9 years and monozygotic twin transplant (live donor) may survive as long as 25 years.

1452
Q

A 32 year old male is receiving a blood transfusion after being involved in a road traffic accident. A few minutes after the transfusion he complains of loin pain. His observations show temperature 39 oC, HR 130bpm and blood pressure is 95/40mmHg. What is the best test to confirm his diagnosis?

USS abdomen

Direct Coomb’s test

Blood cultures

Blood film

Sickle cell test

A

Mnemonic for transfusion reactions:

Got a bad unit

G raft vs. Host disease
O verload
T hrombocytopaenia

A lloimmunization

B lood pressure unstable
A cute haemolytic reaction
D elayed haemolytic reaction

U rticaria
N eutrophilia
I nfection
T ransfusion associated lung injury
The diagnosis is of an acute haemolytic transfusion reaction, normally due to ABO incompatibility. Haemolysis of the transfused cells occurs causing the combination of shock, haemoglobinaemia and loin pain. This may subsequently lead to disseminated intravascular coagulation. A Coomb’s test should confirm haemolysis. Other tests for haemolysis include: unconjugated bilirubin, haptoglobin, serum and urine free haemoglobin.

Note that delayed haemolytic reactions are normally associated with antibodies to the Rh system and occur 5-10 days after transfusion.

Blood transfusion reactions

Acute transfusion reactions present as adverse signs or symptoms during or within 24 hours of a blood transfusion. The most frequent reactions are fever, chills, pruritus, or urticaria, which typically resolve promptly without specific treatment or complications. Other signs occurring in temporal relationship with a blood transfusion, such as severe dyspnoea, pyrexia, or loss of consciousness may be the first indication of a more severe potentially fatal reaction.
The causes of adverse reactions are multi-factorial. Immune mediated reactions, some of the most feared, occur as a result of component mismatch, the commonest cause of which is clerical error. More common, non immune mediated, complications may occur as a result of product contamination, this may be bacterial or viral.
Transfusion related lung injury is well recognised and there are two proposed mechanisms which underpin this. One involves the sequestration of primed neutrophils within the recipient pulmonary capillary bed. The other proposed mechanism suggests that HLA mismatches between donor neutrophils and recipient lung tissue is to blame.
The table below summarises the main types of transfusion reaction.

Immune mediated Non immune mediated
Pyrexia Hypocalcaemia
Alloimmunization CCF
Thrombocytopaenia Infections
Transfusion associated lung injury Hyperkalaemia
Graft vs Host disease
Urticaria
Acute or delayed haemolysis
ABO incompatibility
Rhesus incompatibility

1453
Q

A 19 year old man presents with painful rectal bleeding and is found to have an anal fissure. Which of the following is least associated with this condition?

Leukaemia

Syphilis

Tuberculosis

Sickle cell disease

Crohn’s disease

A

Anal fissures are associated with:

Sexually transmitted diseases (syphilis, HIV)
Inflammatory bowel disease (Crohn’s up to 50%)
Leukaemia (25% of patients)
Tuberculosis
Previous anal surgery

Ano rectal disease

Haemorrhoids Location: 3, 7, 11 o’clock position
Internal or external
Treatment: Conservative, Rubber band ligation, Haemorrhoidectomy
Fissure in ano Location: midline 6 (posterior midline 90%) and 12 o’clock position. Distal to the dentate line
Chronic fissure > 6/52: triad: Ulcer, sentinel pile, enlarged anal papillae
Proctitis Causes: Crohn’s, ulcerative colitis, Clostridium difficile
Ano rectal abscess E.coli, staph aureus
Positions: Perianal, Ischiorectal, Pelvirectal, Intersphincteric
Anal fistula Usually due to previous ano-rectal abscess
Intersphincteric, transsphincteric, suprasphincteric, and extrasphincteric. Goodsalls rule determines location
Rectal prolapse Associated with childbirth and rectal intussceception. May be internal or external
Pruritus ani Systemic and local causes
Anal neoplasm Squamous cell carcinoma commonest unlike adenocarcinoma in rectum
Solitary rectal ulcer Associated with chronic straining and constipation. Histology shows mucosal thickening, lamina propria replaced with collagen and smooth muscle (fibromuscular obliteration)

Rectal prolapse
Common especially in multiparous women.
May be internal or external.
Internal rectal prolapse can present insidiously.
External prolapse can ulcerate and in long term impair continence.
Diagnostic work up includes colonoscopy, defecating proctogram, ano rectal manometry studies and if doubt exists an examination under anaesthesia.

Treatments for prolapse
In the acute setting reduce it (covering it with sugar may reduce swelling.
Delormes procedure which excises mucosa and plicates the rectum (high recurrence rates) may be used for external prolapse.
Altmeirs procedure which resects the colon via the perineal route has lower recurrence rates but carries the risk of anastamotic leak.
Rectopexy is an abdominal procedure in which the rectum is elevated and usually supported at the level of the sacral promontory. Post operative constipation may be reduced by limiting the dissection to the anterior plane (laparoscopic ventral mesh rectopexy).

Pruritus ani
Extremely common.
Check not secondary to altered bowel habits (e.g. Diarrhoea)
Associated with underlying diseases such as haemorrhoids.
Examine to look for causes such as worms.
Proctosigmoidoscopy to identify associated haemorrhoids and exclude cancer.
Treatment is largely supportive and patients should avoid using perfumed products around the area.

Fissure in ano
Typically painful PR bleeding (bright red).
Nearly always in the posterior midline.
Usually solitary.

Treatment
Stool softeners.
Topical diltiazem (or GTN).
If topical treatments fail then botulinum toxin should be injected.
If botulinum toxin fails then males should probably undergo lateral internal sphincterotomy.
Females who do not respond to botulinum toxin should undergo ano rectal manometry studies and endo anal USS prior to being offered surgery such as sphincterotomy.

1454
Q

A 45 year old man presents with an episode of alcoholic pancreatitis. He makes slow but steady progress. He is reviewed clinically at 6 weeks following admission. He has a diffuse fullness of his upper abdomen and on imaging a collection of fluid is found to be located behind the stomach. His serum amylase is mildly elevated. Which of the following is the most likely explanation?

Early fluid collection

Pancreatic abscess

Peripancreatic necrosis

Pseudocyst

Sterile necrosis

A

Pseudocysts are unlikely to be present less than 4 weeks after an attack of acute pancreatitis. However, they are more common at this stage and are associated with a raised amylase.

Pancreatitis: sequelae

Peripancreatic fluid collections
Occur in 25% cases
Located in or near the pancreas and lack a wall of granulation or fibrous tissue
May resolve or develop into pseudocysts or abscesses
Since most resolve aspiration and drainage is best avoided as it may precipitate infection

Pseudocysts
In acute pancreatitis result from organisation of peripancreatic fluid collection. They may or may not communicate with the ductal system.
The collection is walled by fibrous or granulation tissue and typically occurs 4 weeks or more after an attack of acute pancreatitis
Most are retrogastric
75% are associated with persistent mild elevation of amylase
Investigation is with CT, ERCP and MRI or Endoscopic USS
Symptomatic cases may be observed for 12 weeks as up to 50% resolve
Treatment is either with endoscopic or surgical cystogastrostomy or aspiration

Pancreatic necrosis
Pancreatic necrosis may involve both the pancreatic parenchyma and surrounding fat
Complications are directly linked to extent of parenchymal necrosis and extent of necrosis overall
Early necrosectomy is associated with a high mortality rate (and should be avoided unless compelling indications for surgery exist)
Sterile necrosis should be managed conservatively (at least initially)
Some centres will perform fine needle aspiration sampling of necrotic tissue if infection is suspected. False negatives may occur. The extent of sepsis and organ dysfunction may be a better guide to surgery

Pancreatic abscess
Intra abdominal collection of pus associated with pancreas but in the absence of necrosis
Typically occur as a result of infected pseudocyst
They are usually managed by placement of percutaneous drains

Haemorrhage
Infected necrosis may involve vascular structures with resultant haemorrhage that may occur de novo or as a result of surgical necrosectomy.
When retroperitoneal haemorrhage occurs Grey Turners sign may be identified

1455
Q

A male infant is born by emergency cesarean section at 39 weeks gestation for foetal distress. Soon after the birth the baby becomes progressively hypoxic and on examination is found to have a scaphoid abdomen. What is the most likely underlying diagnosis?

Intestinal malrotation

Hiatus hernia

Foramen of Bochdalek hernia

Foramen of Morgagni hernia

Tracheo-oesphageal fistula

A

The finding of a scaphoid abdomen and respiratory distress suggests extensive intra thoracic herniation of the abdominal contents. This is seen most frequently with Bochdalek hernias. Morgagni hernias seldom present in such a dramatic fashion. The other options do not typically present with the symptoms and signs described.

Embryology of the diaphragm and diaphragmatic hernia

Embryology
The diaphragm is formed between the 5th and 7th weeks of gestation through the progressive fusion of the septum transversum, pleuroperitoneal folds and via lateral muscular ingrowth. The muscular origins of the diaphragm are somites located in cervical segments 3 to 5, which accounts for the long path taken by the phrenic nerve. The components contribute to the following diaphragmatic segments:
Septum transversum - Central tendon
Pleuroperitoneal membranes - Parietal membranes surrounding viscera
Cervical somites C3 to C5 - Muscular component of the diaphragm

Diaphragmatic hernia
Type of hernia Features
Morgagni Anteriorly located
Minimal compromise on lung development
Minimal signs on antenatal ultrasound
Usually present later
Usually good prognosis
Bochdalek hernia Posteriorly located
Larger defect
Often diagnosed antenatally
Associated with pulmonary hypoplasia
Poor prognosis

The posterior hernias of Bochdalek are the most common type and if not diagnosed antenatally will typically present soon after birth with respiratory distress. The classical finding is that of a scaphoid abdomen on clinical examination because of herniation of the abdominal contents into the chest. Bochdalek hernias are associated with a number of chromosomal abnormalities such as Trisomy 21 and 18. Infants have considerable respiratory distress due to hypoplasia of the developing lung. Historically this was considered to be due to direct compression of the lung by herniated viscera. This view over simplifies the situation and the pulmonary hypoplasia occurs concomitantly with the hernial development, rather than as a direct result of it. The pulmonary hypoplasia is associated with pulmonary hypertension and abnormalities of pulmonary vasculature. The pulmonary hypertension renders infants at risk of right to left shunting (resulting in progressive and worsening hypoxia).
Diagnostic work up of these infants includes chest x-rays/ abdominal ultrasound scans and cardiac echo.
Surgery forms the mainstay of treatment and both thoracic and abdominal approaches may be utilised. Following reduction of the hernial contents a careful search needs to be made for a hernial sac as failure to recognise and correct this will result in a high recurrence rate. Smaller defects may be primarily closed, larger defects may require a patch to close the defect. Malrotation of the viscera is a recognised association and may require surgical correction at the same procedure (favoring an abdominal approach).
The mortality rate is 50-75% and is related to the degree of lung compromise and age at presentation (considerably better in infants >24 hours old).

1456
Q

Which of the following statements are not typically true in hypokalaemia?

It may occur as a result of mechanical bowel preparation

Chronic vomiting may increase renal potassium losses

It may be associated with aciduria

It may cause hyponatraemia

It often accompanies acidosis

A

Potassium depletion occurs either through the gastrointestinal tract or the kidney. Chronic vomiting in itself is less prone to induce potassium loss than diarrhoea as gastric secretions contain less potassium than those in the lower GI tract. If vomiting produces a metabolic alkalosis then renal potassium wasting may occur as potassium is excreted in preference to hydrogen ions. The converse may occur in potassium depletion resulting in acid urine.

Hypokalemia is very commonly associated with metabolic alkalosis. This is due to 2 factors: 1) the common causes of metabolic alkalosis (vomiting, diuretics) directly induce H+ and K loss (via aldosterone) and thus also cause hypokalemia and 2) hypokalemia is a very important cause of metabolic alkalosis by three mechanisms. The initial effect is by causing a transcellular shift in which K leaves and H+ enters the cells, thereby raising the extracellular pH. The second effect is by causing a transcellular shift in the cells of the proximal tubules resulting in an intracellular acidosis, which promotes ammonium production and excretion. Thirdly, in the presence of hypokalemia, hydrogen secretion in the proximal and distal tubules increases. This leads to further reabsorption of HCO3-. The net effect is an increase in the net acid excretion.

Hypokalaemia

Potassium and hydrogen can be thought of as competitors. Hyperkalaemia tends to be associated with acidosis because as potassium levels rise fewer hydrogen ions can enter the cells

Hypokalaemia with alkalosis
Vomiting
Diuretics
Cushing’s syndrome
Conn’s syndrome (primary hyperaldosteronism)

Hypokalaemia with acidosis
Diarrhoea
Renal tubular acidosis
Acetazolamide
Partially treated diabetic ketoacidosis

1457
Q

A 56 year old man requires long term parenteral nutrition and the decision is made to insert a PICC line for long term venous access. This is inserted into the basilic vein at the region of the elbow. As the catheter is advanced, into which venous structure is the tip of the catheter most likely to pass from the basilic vein?

Subclavian vein

Axillary vein

Posterior circumflex humeral vein

Cephalic vein

Superior vena cava

A

The basilic vein drains into the axillary vein and although PICC lines may end up in a variety of fascinating locations the axillary vein is usually the commonest site following from the basilic. The posterior circumflex humeral vein is encountered prior to the axillary vein. However, a PICC line is unlikely to enter this structure because of its angle of entry into the basilic vein.

Basilic vein

The basilic and cephalic veins both provide the main pathways of venous drainage for the arm and hand. It is continuous with the palmar venous arch distally and the axillary vein proximally.

Path
Originates on the medial side of the dorsal venous network of the hand, and passes up the forearm and arm.
Most of its course is superficial.
Near the region anterior to the cubital fossa the vein joins the cephalic vein.
Midway up the humerus the basilic vein passes deep under the muscles.
At the lower border of the teres major muscle, the anterior and posterior circumflex humeral veins feed into it.
It is often joined by the medial brachial vein before draining into the axillary vein.

1458
Q

When performing minor surgery in the scalp, which of the following regions is considered a danger area as regards spread of infection into the CNS?

Aponeurosis epicranialis

Skin

Pericranium

Connective tissue

Loose areolar tissue

A

This area is most dangerous as infections can spread easily. The emissary veins that drain this area may allow sepsis to spread to the cranial cavity.

Head injury

Patients who suffer head injuries should be managed according to ATLS principles and extra cranial injuries should be managed alongside cranial trauma. Inadequate cardiac output will compromise CNS perfusion irrespective of the nature of the cranial injury.

Types of traumatic brain injury
Extradural haematoma Bleeding into the space between the dura mater and the skull. Often results from acceleration-deceleration trauma or a blow to the side of the head. The majority of extradural haematomas occur in the temporal region where skull fractures cause a rupture of the middle meningeal artery.

Features
Raised intracranial pressure
Some patients may exhibit a lucid interval
Subdural haematoma Bleeding into the outermost meningeal layer. Most commonly occur around the frontal and parietal lobes. May be either acute or chronic.

Risk factors include old age and alcoholism.

Slower onset of symptoms than a extradural haematoma.
Subarachnoid haemorrhage Usually occurs spontaneously in the context of a ruptured cerebral aneurysm, but may be seen in association with other injuries when a patient has sustained a traumatic brain injury.

Pathophysiology
Primary brain injury may be focal (contusion/ haematoma) or diffuse (diffuse axonal injury)
Diffuse axonal injury occurs as a result of mechanical shearing following deceleration, causing disruption and tearing of axons
Intra-cranial haematomas can be extradural, subdural or intracerebral, while contusions may occur adjacent to (coup) or contralateral (contre-coup) to the side of impact
Secondary brain injury occurs when cerebral oedema, ischaemia, infection, tonsillar or tentorial herniation exacerbates the original injury. The normal cerebral auto regulatory processes are disrupted following trauma rendering the brain more susceptible to blood flow changes and hypoxia
The Cushings reflex (hypertension and bradycardia) often occurs late and is usually a pre terminal event

Management
Where there is life threatening rising ICP such as in extra dural haematoma and whilst theatre is prepared or transfer arranged use of IV mannitol/ frusemide may be required.
Diffuse cerebral oedema may require decompressive craniotomy
Exploratory Burr Holes have little management in modern practice except where scanning may be unavailable and to thus facilitate creation of formal craniotomy flap
Depressed skull fractures that are open require formal surgical reduction and debridement, closed injuries may be managed non operatively if there is minimal displacement.
ICP monitoring is appropriate in those who have GCS 3-8 and normal CT scan.
ICP monitoring is mandatory in those who have GCS 3-8 and abnormal CT scan.
Hyponatraemia is most likely to be due to syndrome of inappropriate ADH secretion.
Minimum of cerebral perfusion pressure of 70mmHg in adults.
Minimum cerebral perfusion pressure of between 40 and 70 mmHg in children.

Interpretation of pupillary findings in head injuries
Pupil size Light response Interpretation
Unilaterally dilated Sluggish or fixed 3rd nerve compression secondary to tentorial herniation
Bilaterally dilated Sluggish or fixed
Poor CNS perfusion
Bilateral 3rd nerve palsy
Unilaterally dilated or equal Cross reactive (Marcus - Gunn) Optic nerve injury
Bilaterally constricted May be difficult to assess
Opiates
Pontine lesions
Metabolic encephalopathy
Unilaterally constricted Preserved Sympathetic pathway disruption

1459
Q

A 14 year old develops abdominal pain and anaemia and a colonoscopy and terminal ileal intubation are performed. The terminal ileum shows normal mucosa but quite a lot of fresh blood. An OGD is normal. Which of these pathological processes is likely to be present within the causative lesion?

Heterotopia

Metaplasia

Dysplasia

Anaplasia

Atrophy

A

This is a knowledge application question. Fresh blood in the small bowel of a child/ adolescent with normal mucosa is likely to reflect a bleeding Meckels diverticulum. These are not usually seen at endoscopy as they are too proximally sited but the normal mucosa seen tends to exclude Crohns disease. For a Meckels to bleed, it must be lined by ectopic gastric mucosa which is an example of heterotopia.

Heterotopia

Heterotopia is the presence of a specified normal tissue type at a non-physiological site, but usually co-existing with original tissue in its correct anatomical location. It implies ectopic tissue, in addition to retention of the original tissue type. In neuropathology, for example, gray matter heterotopia is the presence of gray matter within the cerebral white matter or ventricles. Heterotopia within the brain is often divided into three groups: subependymal heterotopia, focal cortical heterotopia and band heterotopia. Another example is a Meckel’s diverticulum, which may contain heterotopic gastric or pancreatic tissue.
It is the fact that the tissue is itself that distinguishes heterotopia from a malignant process.

1460
Q

A 23 year old man complains of severe groin pain several weeks after a difficult inguinal hernia repair. Which nerve is most likely to have been involved?

Genitofemoral

Ilioinguinal

Femoral

Iliohypogastric

Pudendal

A

The ilioinguinal nerve may have been entrapped in the mesh causing a neuroma.

Ilioinguinal nerve

Arises from the first lumbar ventral ramus with the iliohypogastric nerve. It passes inferolaterally through the substance of psoas major and over the anterior surface of quadratus lumborum. It pierces the internal oblique muscle and passes deep to the aponeurosis of the external oblique muscle. It enters the inguinal canal and then passes through the superficial inguinal ring to reach the skin.

Branches
To supply those muscles of the abdominal wall through which it passes.
Skin and fascia over the pubic symphysis, superomedial part of the femoral triangle, surface of the scrotum, root and dorsum of penis or labium majus in females.

1461
Q

A 53 year old man is undergoing a radical gastrectomy for carcinoma of the stomach. Which of these structures will need to be divided to gain access to the coeliac axis?

Lesser omentum

Greater omentum

Falciform ligament

Median arcuate ligament

Gastrosplenic ligament

A

The lesser omentum will need to be divided. During a radical gastrectomy this forms one of the nodal stations that will need to be taken.

Coeliac axis

The coeliac axis has three main branches.
Left gastric
Hepatic: branches-Right Gastric, Gastroduodenal, Superior Pancreaticoduodenal, Cystic (occasionally).
Splenic: branches- Pancreatic, Short Gastric, Left Gastroepiploic

It occasionally gives off one of the inferior phrenic arteries.

Relations
Anteriorly Lesser omentum
Right Right coeliac ganglion and caudate process of liver
Left Left coeliac ganglion and gastric cardia
Inferiorly Upper border of pancreas and renal vein

1462
Q

A 66 year old man is undergoing a left nephro-ureterectomy. The surgeons remove the ureter, which of the following is responsible for the blood supply to the proximal ureter?

Branches of the renal artery

External iliac artery

Internal iliac artery

Direct branches from the aorta

Common iliac artery

A

The proximal ureter is supplied by branches from the renal artery. For the other feeding vessels - see below.

Ureter

25-35 cm long
Muscular tube lined by transitional epithelium
Surrounded by thick muscular coat. Becomes 3 muscular layers as it crosses the bony pelvis
Retroperitoneal structure overlying transverse processes L2-L5
Lies anterior to bifurcation of iliac vessels
Blood supply is segmental; renal artery, aortic branches, gonadal branches, common iliac and internal iliac
Lies beneath the uterine artery

1463
Q

Which of the following is most important in providing support to the duodenojejunal flexure?

Mesentery of the transverse colon

Uncinate process of the pancreas

Gerotas fascia

Ligament of Treves

Ligament of Treitz

A

The suspensory muscle of the duodenum which is referred to as the ligament of Treitz is most important. The ligament of Treves is located between the ileum and caecum.

Duodenum

This is the first and widest part of the small bowel. It has a diameter of around 4-5cm. Its commencement is immediately distal to the pylorus and it runs for around 25cm where it becomes the jejunum at the region of the duodenojejunal flexure. It comprises four parts, superior, descending, horizontal and ascending. Of these, the horizontal is the longest segment. The first 2-3cm of the superior duodenum are intraperitoneal. The remainder is largely retroperitoneal with the exception of the final 1-2cm.

Medial relations of the duodenum include the superior pancreatico-duodenal artery and the pancreatic head. The descending duodenum is closely related to the commencement of the transverse colon which has little in the way of mesentery at this area. Posterior to the descending duodenum lies the right kidney.

The horizontal part passes transversely to the left with an upward deflection as it does so. From right to left it crosses in front of the right ureter, right psoas major, right gonadal vessels and IVC. It terminates anterior to the aorta. Anteriorly, it’s relations include the superior mesenteric vessels and the root of the small bowel.

The ascending part runs to the left of the aorta and upwards to the level of L2. It terminates by binding abruptly forwards as the duodenojejunal flexure. Posteriorly, are the left sympathetic trunk, left psoas major and left gonadal vessels. Anteriorly, it gives attachment to the root of the mesentery, while the left kidney lies laterally and the uncinate process of the pancreas lies medially. The region of the duodenojenunal flexure is fixed in position by the suspensory muscle of the duodenum. This fibromuscular band blends with the musculature of the flexure and passes upwards deep to the pancreas to gain attachment to the right crus of the diaphragm. It is referred to eponymously as the ligament of Treitz.

1464
Q

A patient is diagnosed with a Klatskin tumour which of the structures listed below is affected by this disease process?

Kidney

Stomach

Colon

Bile duct

Spleen

A

A Klatskin tumor (or hilar cholangiocarcinoma) is a cholangiocarcinoma (cancer of the biliary tree) occurring at the confluence of the right and left hepatic bile ducts.

Liver tumours

Primary liver tumours
The most common primary tumours are cholangiocarcinoma and hepatocellular carcinoma. Overall metastatic disease accounts for 95% of all liver malignancies making the primary liver tumours comparatively rare.

Primary liver tumours include:
Cholangiocarcinoma
Hepatocellular carcinoma
Hepatoblastoma
Sarcomas (Rare)
Lymphomas
Carcinoids (most often secondary although primary may occur)

Hepatocellular carcinoma
These account for the bulk of primary liver tumours (75% cases). Its worldwide incidence reflects its propensity to occur on a background of chronic inflammatory activity. Most cases arise in cirrhotic livers or those with chronic hepatitis B infection, especially where viral replication is actively occurring. In the UK it accounts for less than 5% of all cancers, although in parts of Asia its incidence is 100 per 100,000.
The majority of patients (80%) present with existing liver cirrhosis, with a mass discovered on screening ultrasound.

Diagnosis
CT/ MRI (usually both) are the imaging modalities of choice
a-fetoprotein is elevated in almost all cases
Biopsy should be avoided as it seeds tumours cells through a resection plane.
In cases of diagnostic doubt serial CT and αFP measurements are the preferred strategy.

Treatment
Patients should be staged with liver MRI and chest, abdomen and pelvic CT scan.
The testis should be examined in males (testicular tumours may cause raised AFP). PET CT may be used to identify occult nodal disease.
Surgical resection is the mainstay of treatment in operable cases. In patients with a small primary tumour in a cirrhotic liver whose primary disease process is controlled, consideration may be given to primary whole liver resection and transplantation.
Liver resections are an option but since most cases occur in an already diseased liver the operative risks and post-operative hepatic dysfunction are far greater than is seen following metastectomy.
These tumours are not particularly chemo or radiosensitive however, both may be used in a palliative setting. Tumour ablation is a more popular strategy.

Survival
Poor, overall survival is 15% at 5 years.

Cholangiocarcinoma
This is the second most common type of primary liver malignancy. As its name suggests these tumours arise in the bile ducts. Up to 80% of tumours arise in the extra hepatic biliary tree. Most patients present with jaundice and by this stage the majority will have disease that is not resectable.
Primary sclerosing cholangitis is the main risk factor. In deprived countries typhoid and liver flukes are also major risk factors.

Diagnosis
Patients will typically have an obstructive picture on liver function tests.
CA 19-9, CEA and CA 125 are often elevated
CT/ MRI and MRCP are the imaging methods of choice.

Treatment
Surgical resection offers the best chance of cure. Local invasion of peri hilar tumours is a particular problem and this coupled with lobar atrophy will often contra indicate surgical resection.
Palliation of jaundice is important, although metallic stents should be avoided in those considered for resection.

Survival
Is poor, approximately 5-10% 5 year survival.

1465
Q

During a colonoscopy, a patient is found to have a colonic cancer in the caecum and a 1cm polyp (which looks adenomatous) in the sigmoid colon. What is the correct management of the sigmoid polyp?

Undertake a snare polypectomy

Leave in situ until the cancer has been resected

Perform a hot biopsy

Perform a cold biopsy

Resect the sigmoid at the same time as the cancer resection

A

Dysplasia and cancer are not the same disease. All colonic adenomas are dysplastic. Adenomas greater than 2cm may harbor foci of malignancy within them. However, many have dysplastic cells only. These do not require segmental resection.
When a cancer has been identified during endoscopy, it is safest to avoid undertaking polyp interventions as there is a risk of seeding.
In summary, do NOT remove polyps until after the cancer has been resected.

Colonic polyps

Colonic Polyps
May occur in isolation, or greater numbers as part of the polyposis syndromes. In FAP greater than 100 polyps are typically present. The risk of malignancy in association with adenomas is related to size, and is the order of 10% in a 1cm adenoma. Isolated adenomas seldom give risk of symptoms (unless large and distal). Distally sited villous lesions may produce mucous and if very large, electrolyte disturbances may occur.

Follow up of colonic polyps
Group Action
Colorectal cancer Colonoscopy 1 year post resection
Large non pedunculated colorectal polyps (LNPCP), R0 resection One off scope at 3 years
Large non pedunculated colorectal polyps (LNPCP) R1 or non en bloc resection Site check at 2-6 months and then a further scope at 12 months
High risk findings at baseline colonoscopy One off surveillance at 3 years
No high risk findings at baseline colonoscopy No colonoscopic surveillance and invite participation in NHSBCSP programme when due

High risk findings
More than 2 premalignant polyps including 1 or more advanced colorectal polyps
OR
More than 5 pre malignant polyps

Exceptions to guidelines
If patient more than 10 years younger than lower screening age and has polyps but no high risk findings, consider colonoscopy at 5 or 10 years.

Segmental resection or complete colectomy should be considered when:

  1. Incomplete excision of malignant polyp
  2. Malignant sessile polyp
  3. Malignant pedunculated polyp with submucosal invasion
  4. Polyps with poorly differentiated carcinoma
  5. Familial polyposis coli
    -Screening from teenager up to 40 years by 2 yearly sigmoidoscopy/colonoscopy
    -Panproctocolectomy and Ileostomy or Restorative Panproctocolectomy.

Rectal polypoidal lesions may be amenable to trans anal endoscopic microsurgery.

References
Rutter MD et al. British Society of Gastroenterology/Association of Coloproctology of Great Britain and Ireland/Public Health England post- polypectomy and post- colorectal cancer resection surveillance guidelines. Gut 2019;0:123.

1466
Q

A 40 year old women presented with a mass on her forehead. On examination, she had a fluctuant pulsatile mass on her head. Examination of her neck revealed a mass inferior to the hyoid with a positive Berry’s sign. What is the most likely underlying diagnosis?

Follicular thyroid cancer

Medullary thyroid cancer

Papillary thyroid cancer

Anaplastic thyroid cancer

Parathyroid gland cancer

A

A
Berrys sign= Absence of carotid pulse due to malignant thyromegaly.
Papillary thyroid cancers will tend to spread via lymphatics and present with disease that is nearly always confined to the neck. Follicular carcinomas may metastasise haematogenously and the skull may be the presenting site of disease in between 2 and 8% of patients.

Thyroid malignancy

Papillary carcinoma
Commonest sub-type
Accurately diagnosed on fine needle aspiration cytology
Histologically, they may demonstrate psammoma bodies (areas of calcification) and so called ‘orphan Annie’ nuclei
They typically metastasise via the lymphatics and thus laterally located apparently ectopic thyroid tissue is usually a metastasis from a well differentiated papillary carcinoma

Follicular carcinoma
Are less common than papillary lesions
Like papillary tumours, they may present as a discrete nodule. Although they appear to be well encapsulated macroscopically there is invasion on microscopic evaluation
Lymph node metastases are uncommon and these tumours tend to spread haematogenously. This translates into a higher mortality rate
Follicular lesions cannot be accurately diagnosed on fine needle aspiration cytology and thus all follicular FNA’s (THY 3f) will require at least a hemi thyroidectomy

Anaplastic carcinoma
Less common and tend to occur in elderly females
Disease is usually advanced at presentation and often only palliative decompression and radiotherapy can be offered.

Medullary carcinoma
These are tumours of the parafollicular cells ( C Cells) and are of neural crest origin.
The serum calcitonin may be elevated which is of use when monitoring for recurrence.
They may be familial and occur as part of the MEN -2A disease spectrum.
Spread may be either lymphatic or haematogenous and as these tumours are not derived primarily from thyroid cells they are not responsive to radioiodine.

Lymphoma
These respond well to combined chemoradiotherapy
Radical surgery is unnecessary once the disease has been diagnosed on biopsy material. Such biopsy material is not generated by an FNA and thus a core biopsy has to be obtained (with care!).

1467
Q

A 56 year old man is diagnosed as having a glioma. From which of the following cell types do these tumours usually originate?

Schwann cells

Oligodendrocytes

Ependymal cells

Squamous cells

Neuroglial cells

A

Gliomas originate from glial (otherwise known as neuroglial) cells. These serve a structural function in the CNS. The tumours produced may resemble a number of CNS cell types. Tumours are therefore named according to the cells they resemble rather than the origin. Where this is not possible they are termed gliomas.

Glioma

Glioma is a tumour that is typically found in the CNS. These tumours arise from glial cells. They are sub categorised according to the cell type they most closely resemble.

Glioma sub types
Ependymomas- Ependymal cells
Astocytomas- Astrocytes (including glioblastoma)
Oligodendrogliomas- Oligodendrocytes
Mixed- e.g. oligoastrocytomas

Gliomas are categorised as being either high or low grade lesions (the former has the worse prognosis). They may be either supra or infra tentorial. Their symptoms will typically reflect their site of origin. Glioblastoma multiforme has the worst prognosis and few patients will survive beyond 12 months.

1468
Q

Which nerve supplies the skin at the medial aspect of the palm?

Ulnar

Radial

Median

Musculocutaneous

Anterior interosseous

A

Ulnar nerve

Origin
C8, T1

Supplies (no muscles in the upper arm)
Flexor carpi ulnaris
Flexor digitorum profundus
Flexor digiti minimi
Abductor digiti minimi
Opponens digiti minimi
Adductor pollicis
Interossei muscle
Third and fourth lumbricals
Palmaris brevis

Path
Posteromedial aspect of upper arm to flexor compartment of forearm, then along the ulnar. Passes beneath the flexor carpi ulnaris muscle, then superficially over the flexor retinaculum into the palm of the hand.

Branches
Branch Supplies
Muscular branch Flexor carpi ulnaris
Medial half of the flexor digitorum profundus
Palmar cutaneous branch (Arises near the middle of the forearm) Skin on the medial part of the palm
Dorsal cutaneous branch Dorsal surface of the medial part of the hand
Superficial branch Cutaneous fibres to the anterior surfaces of the medial one and one-half digits
Deep branch Hypothenar muscles
All the interosseous muscles
Third and fourth lumbricals
Adductor pollicis
Medial head of the flexor pollicis brevis

Effects of injury
Damage at the wrist
Wasting and paralysis of intrinsic hand muscles (claw hand)
Wasting and paralysis of hypothenar muscles
Loss of sensation medial 1 and half fingers
Damage at the elbow
Radial deviation of the wrist
Clawing less in 4th and 5th digits

1469
Q

A 30 year old man presents with severe pain in the left hip it has been present on and off for many years. He was born at 39 weeks gestation by emergency caesarean section after a long obstructed breech delivery. He was slow to walk and as a child was noted to have an antalgic gait. He was a frequent attender at the primary care centre and the pains dismissed as growing pains. X-rays show almost complete destruction of the femoral head and a narrow acetabulum. What is the most likely underlying disease process?

Developmental dysplasia of the hip

Slipped upper femoral epiphysis

Extra capsular fracture of the femoral neck

Rheumatoid arthritis

Perthes disease

A

Developmental dysplasia of the hip. Usually diagnosed by Barlow and Ortolani tests in early childhood. Most Breech deliveries are also routinely subjected to USS of the hip joint. At this young age an arthrodesis may be preferable to hip replacement.

Paediatric orthopaedics

Diagnosis Mode of presentation Treatment Radiology
Developmental dysplasia of the hip Usually diagnosed in infancy by screening tests. May be bilateral, when disease is unilateral there may be leg length inequality. As disease progresses child may limp and then early onset arthritis. More common in extended breech babies. Splints and harnesses or traction. In later years osteotomy and hip realignment procedures may be needed. In arthritis a joint replacement may be needed. However, this is best deferred if possible as it will almost certainly require revision Initially no obvious change on plain films and USS gives best resolution until 3 months of age. On plain films Shentons line should form a smooth arc
Perthes Disease Hip pain (may be referred to the knee) usually occurring between 5 and 12 years of age. Bilateral disease in 20%. Remove pressure from joint to allow normal development. Physiotherapy. Usually self-limiting if diagnosed and treated promptly. X-rays will show flattened femoral head. Eventually in untreated cases the femoral head will fragment.
Slipped upper femoral epiphysis Typically seen in obese male adolescents. Pain is often referred to the knee. Limitation to internal rotation is usually seen. Knee pain is usually present 2 months prior to hip slipping. Bilateral in 20%. Bed rest and non-weight bearing. Aim to avoid avascular necrosis. If severe slippage or risk of it occurring then percutaneous pinning of the hip may be required. X-rays will show the femoral head displaced and falling inferolaterally (like a melting ice cream cone) The Southwick angle gives indication of disease severity

1470
Q

A 67 year old man has had multiple episodes of fever and left iliac fossa pain. These have usually resolved with courses of intravenous antibiotics. He is admitted with a history of increasing constipation and abdominal distension. A contrast x-ray is performed which shows flow of contrast to the sigmoid colon, here the contrast flows through a long narrow segment of colon into dilated proximal bowel. What is the most likely cause?

Diverticular stricture

Malignant stricture

Ischaemic stricture

Volvulus

Crohns stricture

A

The long history of left iliac fossa pain and development of bowel obstruction suggests a diverticular stricture. These may contain a malignancy and most will require resection. Whilst colonic Crohns strictures can occur, they would be quite rare in this age group, with this history as an isolated finding.

Colonic obstruction

Cause Features Treatment
Cancer
Usually insidious onset
History of progressive constipation
Systemic features (e.g. anaemia)
Abdominal distension
Absence of bowel gas distal to site of obstruction
Establish diagnosis (e.g. contrast enema/ endoscopy)
Laparotomy and resection, stenting, defunctioning colostomy or bypass
Diverticular stricture
Usually history of previous acute diverticulitis
Long history of altered bowel habit
Evidence of diverticulosis on imaging or endoscopy
Once diagnosis established, usually surgical resection
Colonic stenting should not be performed for benign disease
Volvulus
Twisting of bowel around its mesentery
Sigmoid colon affected in 76% cases
Patients usually present with abdominal pain, bloating and constipation
Examination usually shows asymmetrical distension
Plain X-rays usually show massively dilated sigmoid colon, loss of haustra and U shape are typical, the loop may contain fluid levels
Initial treatment is to untwist the loop, a flexible sigmoidoscopy may be needed
Those with clinical evidence of ischaemia should undergo surgery
Patient with recurrent volvulus should undergo resection
Acute colonic pseudo-obstruction
Symptoms and signs of large bowel obstruction with no lesion
Usually associated with metabolic disorders
Usually a cut off in the left colon (82% cases)
Although abdomen tense and distended, it is usually not painful
All patients should undergo contrast enema (may be therapeutic)

Colonoscopic decompression
Correct metabolic disorders
IV neostigmine
Surgery

1471
Q

A 28 year old woman, who is 18 weeks pregnant, presents with sudden chest pain. Her blood pressure is 150/70 mmHg, saturations are 92% on 15L oxygen and her heart rate is 130 bpm. There are no murmurs and her chest is clear. There are signs of thrombophlebitis in the left leg. What is the most likely problem?

Pulmonary embolism

Aortic dissection

Mitral valve regurgitation

Myocardial infarct

Myocarditis

A

Chest pain, hypoxia and clear chest on auscultation in pregnancy should lead to a high suspicion of pulmonary embolism.

Chest pain in pregnancy

Aortic dissection
Predisposing factors in pregnancy are hypertension, congenital heart disease and Marfan’s syndrome
Mainly Stanford type A dissections
Sudden tearing chest pain, transient syncope
Patient may be cold and clammy, hypertensive and have an aortic regurgitation murmur
Involvement of the right coronary artery may cause inferior myocardial infarction

Surgical management
Gestational timeframe Management
< 28/40 Aortic repair with the fetus kept in utero
28-32/40 Dependent on fetal condition
> 32/40 Primary Cesarean section followed by aortic repair at the same operation

Mitral stenosis
Most cases associated with rheumatic heart disease
Becoming less common in British women; suspect in women who have moved recently to the country
Commonest cardiac condition in pregnancy
Commonly associated with mortality
Valve surgery; balloon valvuloplasty preferable

Pulmonary embolism
Leading cause of mortality in pregnancy
Half dose scintigraphy; CT chest if underlying lung disease, should aid diagnosis
Treatment with low molecular weight heparin throughout pregnancy and 4-6 weeks after childbirth
Warfarin is contra indicated in pregnancy (though may be continued in women with mechanical heart valves due to the significant risk of thromboembolism)

References
1. Bates S.M. and Ginsberg J.S. How we manage venous thromboembolism during pregnancy. Blood 2002 (100): 3470-3478.

  1. Scarsbrook A.Fand Gleeson V. Investigating suspected pulmonary embolism in pregnancy. BMJ 2007 (326) : 1135 doi: 10.1136/bmj.7399.1135.
  2. Morley C. A. and Lim B. A. Lesson of the Week: The risks of delay in diagnosis of breathlessness in pregnancy. BMJ 1995 (311) : 1083.
1472
Q

A 73 year old lady sustains a distal radius fracture and this is manipulated using a Biers block with prilocaine as the local anaesthetic agent. During the procedure the occlusion cuff deflates and the patient becomes progressively cyanosed. What is the treatment of choice?

Intravenous calcium gluconate

Exchange transfusion

Intravenous methylene blue

Intravenous sodium thiosulphate

Intravenous gelofusine

A

Prilocaine is a recognised cause of methaemoglobinaemia, this is characterised by the development of cyanosis and dyspnoea. This disorder occurs because of the change haemoglobin to a ferric subtype rather than ferrous (Fe2+). This type of change shifts the oxygen dissociation curve to the left and tissue hypoxia occurs. Methylene blue will revert the haemoglobin to the ferrous type and reverse this effect.

Local anaesthetic toxicity

Toxicity results from either accidental intravascular injection (rapid onset of symptoms-usually correct dose), or from excessive dosage (slower onset). Local anaesthetic agents not only exert a membrane stabilising effect on peripheral nerves but will also act on excitable membranes within the CNS and Heart. The sensory neurones in the CNS are suppressed before the motor ones. As a result the early symptoms will typically be those of circumoral paraesthesia and tinnitus, followed by falling GCS and eventually coma.

Management of toxicity
Stop injecting the anaesthetic agent
High flow 100% oxygen via face mask
Cardiovascular monitoring
Administer lipid emulsion (Intralipid 20%) at 1.5ml/Kg over 1 minute as a bolus
Consider lipid emulsion infusion, at 0.25ml/ Kg/ minute
If toxicity due to prilocaine then administer methylene blue

Safe doses
10ml of lignocaine 1% contains 100mg of drug, this would constitute 70% of the maximum safe dose in a 50 kg patient. Up to 7mg / kg can be administered if adrenaline is added to the solution.

Doses of local anaesthetics
Agent Dose plain Dose with adrenaline
Lignocaine 3mg/Kg 7mg/Kg
Bupivicane 2mg/Kg 2mg/Kg
Prilocaine 6mg/Kg 9mg/Kg
These are a guide only as actual doses depend on site of administration, tissue vascularity and co-morbidities.

1473
Q

A 43 year old lady undergoes a live donor related renal transplant. Over the next few years it is noted that her renal function progressively deteriorates. What is the most likely underlying explanation?

Type I hypersensitivity reaction

Type III hypersensitivity reaction

Type II hypersensitivity reaction

Type IV hypersensitivity reaction

None of the above

A

Chronic rejection of renal transplants is mediated via T lymphocytes and is therefore a type IV hypersensitivity reaction. This process can be mitigated by immunosupression.

Organ Transplant

A number of different organ and tissue transplants are now available. In many cases an allograft is performed, where an organ is transplanted from one individual to another. Allografts will elicit an immune response and this is one of the main reasons for organ rejection.

Graft rejection occurs because allografts have allelic differences at genes that code immunohistocompatability complex genes. The main antigens that give rise to rejection are:
ABO blood group
Human leucocyte antigens (HLA)
Minor histocompatability antigens

ABO Matching
ABO incompatibility will result in early organ rejection (hyperacute) because of pre existing antibodies to other groups. Group O donors can give organs to any type of ABO recipient whereas group AB donor can only donate to AB recipient.

HLA System
The four most important HLA alleles are:

HLA A
HLA B
HLA C
HLA DR

An ideal organ match would be one in which all 8 alleles are matched (remember 2 from each parent, four each = 8 alleles). Modern immunosuppressive regimes help to manage the potential rejection due to HLA mismatching. However, the greater the number of mismatches the worse the long term outcome will be. T lymphocytes will recognise antigens bound to HLA molecules and will then become activated. Clonal expansion then occurs with a response directed against that antigen.

Types of organ rejection
Hyperacute. This occurs immediately through presence of pre formed antibodies (such as ABO incompatibility).
Acute. Occurs during the first 6 months and is usually T cell mediated. Usually tissue infiltrates and vascular lesions.
Chronic. Occurs after the first 6 months. Vascular changes predominate.

Hyperacute
Renal transplants at greatest risk and liver transplants at least risk. Although ABO incompatibility and HLA Class I incompatible transplants will all fare worse in long term.

Acute
All organs may undergo acute rejection. Mononuclear cell infiltrates predominate. All types of transplanted organ are susceptible and it may occur in up to 50% cases.

Chronic
Again all transplants with HLA mismatch may suffer this fate. Previous acute rejections and other immunosensitising events all increase the risk. Vascular changes are most prominent with myointimal proliferation leading to organ ischaemia. Organ specific changes are also seen such as loss of acinar cells in pancreas transplants and rapidly progressive coronary artery disease in cardiac transplants.

Surgical overview-Renal transplantation
A brief overview of the steps involved in renal transplantation is given.
Patients with end stage renal failure who are dialysis dependent or likely to become so in the immediate future are considered for transplant. Exclusion criteria include; active malignancy, old age (due to limited organ availability). Patients are medically optimised.
Donor kidneys, these may be taken from live related donors and close family, members may have less HLA mismatch than members of the general population. Laparoscopic donor nephrectomy further minimises the operative morbidity for the donor. Other organs are typically taken from brain dead or dying patients who have a cardiac arrest and in whom resuscitation is futile. The key event is to minimise the warm ischaemic time in the donor phase.

The kidney once removed is usually prepared on the bench in theatre by the transplant surgeon immediately prior to implantation and factors such as accessory renal arteries and vessel length are assessed and managed.

For first time recipients the operation is performed under general anaesthesia. A Rutherford-Morison incision is made on the preferred side. This provides excellent extraperitoneal access to the iliac vessels. The external iliac artery and vein are dissected out and following systemic heparinisation are cross clamped. The vein and artery are anastamosed to the iliacs and the clamps removed. The ureter is then implanted into the bladder and a stent is usually placed to maintain patency. The wounds are then closed and the patient recovered from surgery.

In the immediate phase a common problem encountered in cadaveric kidneys is acute tubular necrosis and this tends to resolve.

Graft survival times from cadaveric donors are typically of the order of 9 years and monozygotic twin transplant (live donor) may survive as long as 25 years.

1474
Q

A 73 year old lady with long standing atrial fibrillation develops a cold and pulseless white arm. A brachial embolus is suspected and a brachial embolectomy is performed. Which of the following structures is at greatest risk of injury during this procedure?

Radial nerve

Cephalic vein

Ulnar nerve

Median nerve

None of the above

A

The median nerve lies close to the brachial artery in the antecubital fossa. This is the usual site of surgical access to the brachial artery for an embolectomy procedure. The median nerve may be damaged during clumsy application of vascular clamps to the artery.

Brachial artery

The brachial artery begins at the lower border of teres major as a continuation of the axillary artery. It terminates in the cubital fossa at the level of the neck of the radius by dividing into the radial and ulnar arteries.

Relations
Posterior relations include the long head of triceps with the radial nerve and profunda vessels intervening. Anteriorly it is overlapped by the medial border of biceps.
It is crossed by the median nerve in the middle of the arm.
In the cubital fossa it is separated from the median cubital vein by the bicipital aponeurosis.
The basilic vein is in contact at the most proximal aspect of the cubital fossa and lies medially.

1475
Q

A 73 year old man is due to undergo a radical prostatectomy for carcinoma of the prostate gland. To which of these lymph nodes will the tumour drain primarily?

Para aortic

Internal iliac

Superficial inguinal

Meso rectal

None of the above

A

The prostate lymphatic drainage is primarily to the internal iliac nodes and also the sacral nodes. Although internal iliac is the first site.

Prostate gland

The prostate gland is approximately the shape and size of a walnut and is located inferior to the bladder. It is separated from the rectum by Denonvilliers fascia and its blood supply is derived from the internal iliac vessels (via inferior vesical artery). The internal sphincter lies at the apex of the gland and may be damaged during prostatic surgery, affected individuals may complain of retrograde ejaculation.

Summary of prostate gland
Arterial supply Inferior vesical artery (from internal iliac)
Venous drainage Prostatic venous plexus (to paravertebral veins)
Lymphatic drainage Internal iliac nodes
Innervation Inferior hypogastric plexus
Dimensions
Transverse diameter (4cm)
AP diameter (2cm)
Height (3cm)
Lobes
Posterior lobe: posterior to urethra
Median lobe: posterior to urethra, in between ejaculatory ducts
Lateral lobes x 2
Isthmus
Zones
Peripheral zone: subcapsular portion of posterior prostate. Most prostate cancers are here
Central zone
Transition zone
Stroma

Relations
Anterior Pubic symphysis
Prostatic venous plexus
Posterior Denonvilliers fascia
Rectum
Ejaculatory ducts
Lateral Venous plexus (lies on prostate)
Levator ani (immediately below the puboprostatic ligaments)

1476
Q

A 58 year old lady presents with a mass in the upper outer quadrant of the right breast. Which of the following statements relating to the breast is untrue?

The internal mammary artery provides the majority of its arterial supply

Nipple retraction may occur as a result of tumour infiltration of the clavipectoral fascia

The internal mammary artery is a branch of the subclavian artery

Up to 70% of lymphatic drainage is to the ipsilateral axillary nodes

None of the above

A

Both skin dimpling and nipple retraction are features of breast malignancy. However, they usually occur as a result of tumour infiltration of the breast ligaments and ducts respectively. The clavipectoral fascia encases the axillary contents. The lymphatic drainage of the breast is to the axilla and also to the internal mammary chain. The breast is well vascularised and the internal mammary artery is a branch of the subclavian artery.

Breast

The breast itself lies on a layer of pectoral fascia and the following muscles:
1. Pectoralis major
2. Serratus anterior
3. External oblique

Breast anatomy
Nerve supply Branches of intercostal nerves from T4-T6.
Arterial supply
Internal mammary (thoracic) artery
External mammary artery (laterally)
Anterior intercostal arteries
Thoraco-acromial artery
Venous drainage Superficial venous plexus to subclavian, axillary and intercostal veins.
Lymphatic drainage
70% Axillary nodes
Internal mammary chain
Other lymphatic sites such as deep cervical and supraclavicular fossa (later in disease)

1477
Q

A 1 year old boy is referred because of an impalpable testis in the left scrotum. Which of the following positions describes an ectopic testis?

Intracanalicular

At the superficial ring

At the base of the penis

Upper scrotum

At the deep inguinal ring

A

A testis at the base of the penis is ectopic, not listed is the superficial inguinal pouch (one of the commonest ectopic sites). A testis located at deep or superficial rings or intra canalicular is not ectopically located.

Testicular embryology

Until the end of foetal life the testicles are located within the abdominal cavity. They are initially located on the posterior abdominal wall on a level with the upper lumbar vertebrae (L2). Attached to the inferior aspect of the testis is the gubernaculum testis which extends caudally to the inguinal region, through the canal and down to the superficial skin. Both the testis and the gubernaculum are extra-peritoneal.
As the foetus grows the gubernaculum becomes progressively shorter. It carries the peritoneum of the anterior abdominal wall (the processus vaginalis). As the processus vaginalis descends the testis is guided by the gubernaculum down the posterior abdominal wall and the back of the processus vaginalis into the scrotum.
By the third month of foetal life the testes are located in the iliac fossae, by the seventh they lie at the level of the deep inguinal ring.

The processus vaginalis usually closes after birth, but may persist and be the site of indirect hernias. Part closure may result in development of cysts on the cord.

1478
Q

A 34 year old man has a tissue defect measuring 3 cm by 1 cm following an excision of a lipoma from the scapula. What is the best option for managing the wound?

Direct primary closure

Delayed primary closure

Pedicled skin graft

Free flap

Rotational flap

A

This wound should be amenable to primary closure. There is minimal associated tissue loss and the surgery is minor and uncontaminated.

Tissue reconstruction

Skin Grafts and Flaps
Skin flaps or grafts may be required where primary wound closure cannot be achieved or would entail either significant cosmetic defect or considerable functional disturbance as a result of wound contraction.

Reconstructive ladder
Method Types
Direct closure The simplest option where possible
Grafting techniques
Split thickness
Full thickness
Skin Substitute
Composite
Flap technique Local:
Transposition
Pivot
Alphabetoplasty (e.g. Z-Y)
Regional:
Myocutaneous
Fasciocutaneous
Neurocutaneous
Distant:
Free tissue transfer
Prelamination techniques Allows creation of specialised flaps e.g. buccal mucosa
Tissue expansion Involves placement of tissue expanders to increase amount of tissue at donor sites

Skin Grafts Vs. Flaps
Skin Grafts Flaps
No size limit (Split)/ Relative size limit (full thickness) Size limited by territory of blood supply
Rely on wound bed for blood supply Tissue has its own blood supply
Take better on clean well vascularised wound beds Will survive independent of the wound bed
Split skin graft donor site typically heals in 12 days Direct closure of donor site or secondary skin graft
Donor site may be reused Donor site cannot be reused

Split thickness skin grafts
Available in range of thicknesses.
Thigh is the commonest donor site
Size may be increased by meshing the graft. However this comes with compromise on cosmesis.
Donor sites, especially if thin grafts are taken can be reused following re-epithelialisation

Full thickness grafts
Most commonly used for facial reconstruction
Include dermal appendages
Provide superior cosmetic result

Composite grafts
These are grafts containing more than one tissue type, such as skin and fat. They are usually used to cover small defects in cosmetically important areas.

Flaps
Flaps have their own blood supply and may be pedicled or free.
May have multiple components e.g. skin, skin + fat, skin + fat + muscle.
They will have the ability to take regardless of the underlying tissue bed.
The type of intrinsic blood supply is important. For example in breast surgery pedicled latissimus dorsi flaps will be less prone to failure than microsvascular anastomosed free Diep flaps.

1479
Q

You are the cardiothoracic surgical registrar reviewing a patient referred for an aortic valve replacement. The 40-year-old man is being investigated for progressive breathlessness in a previous respiratory clinic. The notes show he has smoked for the past 25 years. Pulmonary function tests reveal the following:

FEV1 1.4 L
FVC 1.7 L
FEV1/FVC 82%

What is the most likely explanation?

Asthma

Bronchiectasis

Kyphoscoliosis

Chronic obstructive pulmonary disease

Laryngeal malignancy

A

These results show a restrictive picture, which may result from a number of conditions including kyphoscoliosis. The other answers cause an obstructive picture.

We note that most people have chosen COPD as the answer. In COPD the FEV1/FVC would show an obstructive picture with the FEV1/FVC value being low (approximately less than 70%). In restrictive conditions the FEV1/FVC is normal or increased (greater than 70%). With the FEV1/FVC being over 70% the most likely answer is kyphoscoliosis.

Pulmonary function tests

Pulmonary function tests can be used to determine whether a respiratory disease is obstructive or restrictive. The table below summarises the main findings and gives some example conditions:

Obstructive lung disease Restrictive lung disease
FEV1 - significantly reduced
FVC - reduced or normal
FEV1% (FEV1/FVC) - reduced (less than approx. 70%) FEV1 - reduced
FVC - significantly reduced
FEV1% (FEV1/FVC) - normal or increased (over approx. 70%)
Asthma
COPD
Bronchiectasis
Bronchiolitis obliterans Pulmonary fibrosis
Asbestosis
Sarcoidosis
Acute respiratory distress syndrome
Infant respiratory distress syndrome
Kyphoscoliosis
Neuromuscular disorders

If you would like to read more about pulmonary function tests, here is a link that may be useful:

Interpreting pulmonary function tests: Recognize the pattern, and the diagnosis will follow http://www.ccjm.org/content/70/10/866.full.pdf

1480
Q

Which of the following is not well absorbed following a gastrectomy?

Vitamin c

Zinc

Vitamin B12

Copper

Molybdenum

A

Vitamin B12. The others are unaffected

Post gastrectomy syndrome
Rapid emptying of food from stomach into the duodenum: diarrhoea, abdominal pain, hypoglycaemia
Complications: Vitamin B12 and iron malabsorption, osteoporosis
Treatment: High protein, low carbohydrate diet. Replace B12/Fe/Ca

Gastric emptying

The stomach serves both a mechanical and immunological function. Solid and liquid are retained in the stomach during which time repeated peristaltic activity against a closed pyloric sphincter will cause fragmentation of food bolus material. Contact with gastric acid will help to neutralise any pathogens present.
The amount of time material spends in the stomach is related to its composition and volume. For example a glass of water will empty more quickly than a large meal. The presence of amino acids and fat will all serve to delay gastric emptying.

Controlling factors
Neuronal stimulation of the stomach is mediated via the vagus and the parasympathetic nervous system will tend to favor an increase in gastric motility. It is for this reason that individuals who have undergone truncal vagotomy will tend to routinely require either a pyloroplasty or gastro-enterostomy as they would otherwise have delayed gastric emptying.

The following hormonal factors are all involved:

Delay emptying Increase emptying
Gastric inhibitory peptide Gastrin
Cholecystokinin
Enteroglucagon

Diseases affecting gastric emptying
All diseases that affect gastric emptying may result in bacterial overgrowth, retained food and eventually the formation of bezoars that may occlude the pylorus and make gastric emptying even worse. Fermentation of food may cause dyspepsia, reflux and foul smelling belches of gas.

Iatrogenic
Gastric surgery can have profound effects on gastric emptying. As stated above any procedure that disrupts the vagus can cause delayed emptying. Whilst this is particularly true of vagotomy, this operation is now rarely performed. Surgeons are divided on the importance of vagal disruption that occurs during an oesophagectomy, some will routinely perform a pyloroplasty and others will not.

When a distal gastrectomy is performed, the type of anastomosis performed will impact on emptying. When a gastro-enterostomy is constructed, a posterior, retrocolic gastroenterostomy will empty better than an anterior one.

Diabetic gastroparesis
This is predominantly due to neuropathy affecting the vagus nerve. The stomach empties poorly and patients may have episodes of repeated and protracted vomiting. Diagnosis is made by upper GI endoscopy and contrast studies, in some cases a radio nucleotide scan is needed to demonstrate the abnormality more clearly. In treating these conditions, drugs such as metoclopramide will be less effective as they exert their effect via the vagus nerve. One of the few prokinetic drugs that do not work in this way is the antibiotic erythromycin.

Malignancies
Obviously a distal gastric cancer may obstruct the pylorus and delay emptying. In addition, malignancies of the pancreas may cause extrinsic compression of the duodenum and delay emptying. Treatment in these cases is by gastric decompression using a wide bore nasogastric tube and insertion of a stent or, if that is not possible, by a surgical gastroenterostomy. As a general rule gastroenterostomies constructed for bypass of malignancy are usually placed on the anterior wall of the stomach (in spite of the fact that they empty less well). A Roux en Y bypass may also be undertaken, but the increased number of anastomoses for this, in malignant disease that is being palliated, is probably not justified.

Congenital Hypertrophic Pyloric Stenosis
This is typically a disease of infancy. Most babies will present around 6 weeks of age with projectile non bile stained vomiting. It has an incidence of 2.4 per 1000 live births and is more common in males. Diagnosis is usually made by careful history and examination and a mass may be palpable in the epigastrium (often cited seldom felt!). The most important diagnostic test is an ultrasound that usually demonstrates the hypertrophied pylorus. Blood tests may reveal a hypochloraemic metabolic alkalosis if the vomiting is long standing. Once the diagnosis is made the infant is resuscitated and a pyloromyotomy is performed (either open or laparoscopically). Once treated there are no long term sequelae.

1481
Q

78-year-old woman with emphysema receiving 28% oxygen by mask has the following blood gas results The most likely interpretation is:

PH Finding 7.28 (7.35-7.45), p02 6.2 kPa ( 11.9-13.3), pCo2 8 КРА (4.7-6.0), Bicarbonate 36 mmol/L (22-26), Base Excess +5 (-2 to +2)

a. mixed respiratory and metabolic acidosis
b. partially compensated metabolic acidosis
c. partially compensated respiratory acidosis
d. uncompensated metabolic acidosis
e. uncompensated respiratory acidosis

A

c. partially compensated respiratory acidosis

Uncompensated: Coz or HCos normal
Partially Compensated: Nothing is normal
Compensated : PH is normal (7.4 baseline/neutral)

Based on the blood gas results provided, the interpretation suggests a combination of respiratory and metabolic acidosis. Let’s break down the findings:

1.	pH: The pH is below the normal range, indicating acidosis.
2.	pO2: Lower than normal, indicating inadequate oxygenation, which is consistent with emphysema.
3.	pCO2: Elevated, indicating respiratory acidosis, likely due to hypoventilation caused by emphysema.
4.	Bicarbonate: Elevated, indicating metabolic compensation for the respiratory acidosis, which is consistent with the kidneys attempting to retain bicarbonate to buffer the acidic environment. Also high bicarbonate levels reveals that is a chronic condition.
5.	Base Excess: Positive, further indicating metabolic compensation for the acidosis. Metabolic compensation wouldn’t be enough to compensate respiratory acidosis, because pH is still asidotic.
•	Blood Gas Interpretation: https://www.ncbi.nlm.nih.gov/books/NBK482250/
1482
Q

23-year-old man presents three months following extraction of carious teeth with a plaque like infiltration over the upper jaw with sinus formation. Yellow granules are present in the discharging pus. What is the most likely causative organism?

Actinomyces
b. Aspergillus
c. Blastomyces
d. Cryptococcus
e. Histoplasma

A

The most likely causative organism in this scenario is “a. Actinomyces.”

Actinomyces is a genus of bacteria commonly associated with the formation of dental plaques and can cause chronic granulomatous infections, including sinus tract formation, known as actinomycosis. The presence of yellow granules in the discharging pus, known as sulfur granules, is characteristic of actinomycosis.

Source:

•	Actinomycosis: https://www.ncbi.nlm.nih.gov/books/NBK482243/
1483
Q

An 82-year-old man has complete occlusion of his inferior mesenteric artery on angiography but no symptoms or signs of colonic ischemia. Which of the following arteries is the most likely additional source of blood supply to the territory of the inferior mesenteric artery?

a. Left colic
b. Left gastroepiploic
c. Middle colic
d. Splenic
e. Superior rectal

A

When the inferior mesenteric artery (IMA) is completely occluded, collateral circulation becomes essential to supply blood to the affected territory, which includes the descending colon, sigmoid colon, and rectum. One of the major collateral pathways is through the middle colic artery.

The middle colic artery arises from the superior mesenteric artery (SMA) and supplies blood to the transverse colon. When the IMA is occluded, blood from the SMA can flow retrograde into the middle colic artery, providing additional blood supply to the descending colon via collateral circulation.

So, the most likely additional source of blood supply to the territory of the occluded IMA is indeed the “c. Middle colic artery.”

Source:

•	Collateral Circulation: https://www.ncbi.nlm.nih.gov/books/NBK537222/
1484
Q

After being stabbed in the left groin, a previously healthy 20-year-old man was admitted to the emergency Department. He developed hemorrhagic shock. During resuscitation, 10 units of type O negative red cells and 6 L of colloid were administered; Twenty-four hours later, he noted to have severe dyspnea for which he required intubation and ventilation on the intensive care unit. Which of the following would be the most likely cause of his respiratory insufficiency?
a. ABO blood incompatibility
b. Adult respiratory distress syndrome (ARDS)
c. Congestive cardiac tailure
d. Tension pneumothorax
e. Viral pneumonitis

A

Given the scenario described, the most likely cause of the patient’s respiratory insufficiency 24 hours after admission is b. Adult respiratory distress syndrome (ARDS).

ARDS is a severe lung condition characterized by widespread inflammation in the lungs. It can develop as a complication of various traumatic injuries, including hemorrhagic shock and significant blood loss. In this case, the patient’s presentation of severe dyspnea requiring intubation and ventilation, following a traumatic injury and resuscitation with blood products and colloid, is consistent with ARDS.

Other options can be ruled out based on the clinical presentation:

a. ABO blood incompatibility: This typically manifests soon after blood transfusion with symptoms such as fever, chills, chest pain, and hemolysis, rather than delayed respiratory insufficiency.

c. Congestive cardiac failure: While possible, given the patient’s young age and the acute onset of symptoms following trauma, ARDS is a more likely cause of respiratory insufficiency.

d. Tension pneumothorax: This would typically present with acute onset dyspnea and hypoxia, but it is less likely 24 hours after admission and resuscitation.

e. Viral pneumonitis: While viral infections can cause respiratory insufficiency, the patient’s history of trauma and resuscitation makes ARDS a more likely diagnosis.

ARDS is a life-threatening condition that requires prompt recognition and management, including mechanical ventilation and supportive care. Close monitoring of the patient’s oxygenation status and respiratory function is essential in managing ARDS.

Source:

•	Medscape. Acute Respiratory Distress Syndrome (ARDS). https://emedicine.medscape.com/article/165139-overview
1485
Q

An 80-year-old man with a history of hypertension, controlled with thiazide diuretics undergoes transurethral resection of prostate under general anesthesia. The surgery is prolonged and in the recovery room he complains of nausea and a headache. He later becomes agitated and confused. Which of the following is the most likely diagnosis?
a. Hypercalcaemia
b. Hyperuricemia
c. Hypoglycemia
d. Hypokalemia
e. Hyponatremia

A

Based on the patient’s history, surgical procedure, and symptoms, the most likely diagnosis is e. Hyponatremia.

Hyponatremia, or low sodium levels in the blood, can occur due to various reasons, including prolonged surgery with excessive intravenous fluids, especially if those fluids are hypotonic. Thiazide diuretics, which the patient is taking for hypertension, can also contribute to hyponatremia by causing increased sodium loss through urine.

The reason of hyponatremia in that scenario is TURP Syndrome. TURP SYNDROME
• Rapid absorption of a large-volume irrigation solution.
• Can occur 15 min after resection or upto 24 hrs postop.
• Incidence: 1 - 8%
• Characterized by intravascular volume shifts and plasma-solute (osmolarity) effects:
• Circulatory overload
• Water intoxication
• Hyponatremia
• Hypoosmolality
Hyperglycinemia
• Hyperammonemia
• Hemolysis

The symptoms described—nausea, headache, agitation, and confusion—are consistent with the neurological manifestations of hyponatremia. In severe cases, hyponatremia can lead to cerebral edema and potentially life-threatening complications such as seizures and coma.

The other options are less likely:

a. Hypercalcemia: Unlikely given the patient’s presentation and medical history.

b. Hyperuricemia: Typically does not present with symptoms such as nausea, headache, agitation, and confusion.

c. Hypoglycemia: Possible but less likely, especially considering the patient’s history of hypertension and thiazide diuretic use. Hypoglycemia would typically be addressed perioperatively to prevent such complications.

d. Hypokalemia: Less likely given that thiazide diuretics can lead to potassium loss, but the symptoms described are more indicative of hyponatremia.

Management of hyponatremia involves careful assessment of fluid status and electrolyte levels, followed by appropriate correction based on the severity of symptoms and underlying cause.

Source:

•	Medscape. Hyponatremia. https://emedicine.medscape.com/article/242166-overview
1486
Q

An 80-year-old man with a history of hypertension, controlled with thiazide diuretics undergoes transurethral resection of prostate under general anesthesia. The surgery is prolonged and in the recovery room he complains of nausea and a headache. He later becomes agitated and confused. Which of the following is the most likely diagnosis?
a. Hypercalcaemia
b. Hyperuricemia
c. Hypoglycemia
d. Hypokalemia
e. Hyponatremia

A

Based on the patient’s history, surgical procedure, and symptoms, the most likely diagnosis is e. Hyponatremia.

Hyponatremia, or low sodium levels in the blood, can occur due to various reasons, including prolonged surgery with excessive intravenous fluids, especially if those fluids are hypotonic. Thiazide diuretics, which the patient is taking for hypertension, can also contribute to hyponatremia by causing increased sodium loss through urine.

The reason of hyponatremia in that scenario is TURP Syndrome. TURP SYNDROME
• Rapid absorption of a large-volume irrigation solution.
• Can occur 15 min after resection or upto 24 hrs postop.
• Incidence: 1 - 8%
• Characterized by intravascular volume shifts and plasma-solute (osmolarity) effects:
• Circulatory overload
• Water intoxication
• Hyponatremia
• Hypoosmolality
Hyperglycinemia
• Hyperammonemia
• Hemolysis

The symptoms described—nausea, headache, agitation, and confusion—are consistent with the neurological manifestations of hyponatremia. In severe cases, hyponatremia can lead to cerebral edema and potentially life-threatening complications such as seizures and coma.

The other options are less likely:

a. Hypercalcemia: Unlikely given the patient’s presentation and medical history.

b. Hyperuricemia: Typically does not present with symptoms such as nausea, headache, agitation, and confusion.

c. Hypoglycemia: Possible but less likely, especially considering the patient’s history of hypertension and thiazide diuretic use. Hypoglycemia would typically be addressed perioperatively to prevent such complications.

d. Hypokalemia: Less likely given that thiazide diuretics can lead to potassium loss, but the symptoms described are more indicative of hyponatremia.

Management of hyponatremia involves careful assessment of fluid status and electrolyte levels, followed by appropriate correction based on the severity of symptoms and underlying cause.

Source:

•	Medscape. Hyponatremia. https://emedicine.medscape.com/article/242166-overview
1487
Q

A 35-year-old man is admitted to hospital with vomiting, nausea and severe headaches. An MRI scan reveals a tumor of the cerebellopontine angle. Which one of the following pairs of cranial nerves is most likely to be compressed by this tumour?

a.Accessory and vagus
b. Facial and vagus
c. Facial and vestibulocochlear
d. Glossopharyngeal and vestibulocochlear
e. Vagus and

A

The most likely pair of cranial nerves to be compressed by a tumor of the cerebellopontine angle is c. Facial and vestibulocochlear.

Tumors in the cerebellopontine angle can compress the nerves located in this region. The facial nerve (CN VII) and vestibulocochlear nerve (CN VIII) are commonly affected due to their proximity to this area. Compression of the facial nerve can lead to facial weakness or paralysis, while compression of the vestibulocochlear nerve can cause symptoms such as hearing loss, tinnitus, and vertigo.

Options a, b, d, and e involve cranial nerves that are less likely to be affected by a tumor in the cerebellopontine angle.

Source:

•	UpToDate. Overview of Tumors of the Cerebellopontine Angle. https://www.uptodate.com/contents/overview-of-tumors-of-the-cerebellopontine-angle
1488
Q

After resection of d rectal tumour a patient experiences erectile dysfunction. Which of the following nerves is most likely to have been damaged in surgery?
A. Genitofemoral nerve
b. Lumbosacral plexus
c. Pelvic splanchnic nerves
d. Perineal branch of S4
e. Pudendal nerve

A

Given the presentation of erectile dysfunction following rectal tumor resection, the most likely nerve to have been damaged in surgery is c. Pelvic splanchnic nerves. (Aka Hypogastric)

The pelvic splanchnic nerves, also known as the pelvic autonomic nerves or parasympathetic pelvic nerves, are crucial for sexual function. Damage to these nerves during rectal surgery can lead to erectile dysfunction due to disruption of the parasympathetic innervation to the genitalia.

1489
Q

A 45-year-old woman presents with pain, swelling and stiffness affecting all her metacarpophalangeal joints. She has noticed weakness in her grip and her handwriting has changed. She is unable to extend her fingers fully. The index, middle and ring fingers in both hands show an ulnar deviation. The most likely diagnosis is:

a. Duptuyren’s contracture
b. gouty arthritis
c. osteoarthritis
d. radial nerve palsy
e. rheumatoid arthritis

A

The most likely diagnosis for the 45-year-old woman with pain, swelling, stiffness, weakness in grip, and inability to fully extend her fingers, along with ulnar deviation in the index, middle, and ring fingers in both hands is e. rheumatoid arthritis. This condition typically presents with symmetrical joint involvement, deformities like ulnar deviation, and systemic symptoms affecting multiple joints[5]. Dupuytren’s contracture primarily affects the palm and fingers with a different presentation involving bending of the fingers towards the palm without joint inflammation or systemic symptoms[4]. Gouty arthritis is characterized by sudden attacks of severe pain, redness, and swelling in a single joint due to uric acid crystal deposits[5]. Osteoarthritis is more common in older individuals and affects specific joints without systemic symptoms like those described[5]. Radial nerve palsy would lead to specific weakness and sensory changes in the distribution of the radial nerve, not the described symptoms[5].

Sources
[1] Dupuytren’s contracture - NHS https://www.nhs.uk/conditions/dupuytrens-contracture/
[2] Dupuytren contracture - Symptoms and causes - Mayo Clinic https://www.mayoclinic.org/diseases-conditions/dupuytrens-contracture/symptoms-causes/syc-20371943
[3] Dupuytren’s Disease - Dupuytren’s Contracture - OrthoInfo - AAOS https://orthoinfo.aaos.org/en/diseases–conditions/dupuytrens-disease/
[4] Dupuytren’s Contracture: What is it? Symptoms, Causes & Treatment https://www.assh.org/handcare/condition/dupuytrens-contracture
[5] Dupuytren’s disease | The British Society for Surgery of the Hand https://www.bssh.ac.uk/patients/conditions/25/dupuytrens_disease

1490
Q

A 22-year-old man arrives to the Emergency Department with sudden breath less ness due to a large pneumothorax. A chest drain is inserted into the fifth left intercostal space in 5th mid-axillary line. There is hemorrhage into the drainage bottle. Which of the following structures is the most likely cause of this acute hemorrhage?

a. Intercostal artery
b. Left pericardiophrenic artery
C. Lingula of the lung
d. Right ventricle of the heart
e. Spleen

A

The most likely cause of acute hemorrhage in a 22-year-old man with a large pneumothorax after chest drain insertion into the fifth left intercostal space is the intercostal artery[1][2].

Sources
[1] Management of a life-threatening intercostal artery bleeding, difficult … https://www.ncbi.nlm.nih.gov/pmc/articles/PMC7602520/
[2] Iatrogenic injury to the intercostal artery: aetiology, diagnosis and … https://thorax.bmj.com/content/70/8/802
[3] Control of bleeding from intercostal artery laceration - ScienceDirect https://www.sciencedirect.com/science/article/pii/S2213007122002052
[4] Massive hemorrhage from the posterior intercostal artery following … https://cardiothoracicsurgery.biomedcentral.com/articles/10.1186/s13019-021-01718-1
[5] Lethal intercostal artery injury after a fall - Shen https://qims.amegroups.org/article/view/85565/html

1491
Q

A 38-year-oid man in end-stage renal failure resulting from polycystic kidney disease receives a cadaveric renal transplant. Good renal function is established but four weeks later deteriorates, the serum creatinine rising by 25%. Which of the following processes is most likely to be responsible for this deterioration?
a. B-cell mediated rejection
b. Circulating immune complex disease
c. IgG antibody mediated rejection
d. Post-transplant lymphoproliferative disorder
e. T-cell mediated rejection

A

Based on the provided sources, the process most likely responsible for the deterioration in renal function after a cadaveric renal transplant in a 38-year-old man with polycystic kidney disease is e. T-cell mediated rejection. T-cell mediated rejection is a significant factor in graft failure and can lead to a decline in kidney function over time[5][5]. This type of rejection involves T cells recognizing the transplanted kidney as foreign and initiating an immune response against it, contributing to tissue damage and loss of function[5][5]. Understanding T-cell mediated rejection is crucial in managing post-transplant complications and ensuring the long-term success of renal transplants[5][5].

Sources
[1] Safety of CAR T-cell therapy in kidney transplant recipients - PMC - NCBI https://www.ncbi.nlm.nih.gov/pmc/articles/PMC8109014/
[2] Immunosuppression Considerations for Older Kidney Transplant … https://link.springer.com/article/10.1007/s40472-021-00321-6
[3] Combined Bone Marrow and Kidney Transplantation for the Induction of … https://www.ncbi.nlm.nih.gov/pmc/articles/PMC4867066/
[4] Transplanting the Elderly: Mandatory Age- and Minimal Histocompatibility … https://www.frontiersin.org/journals/immunology/articles/10.3389/fimmu.2020.00359/full
[5] The development of adult T cell leukemia/lymphoma in renal … https://rrtjournal.biomedcentral.com/articles/10.1186/s41100-023-00480-5

1492
Q

In order to expose the right axillary artery, a transverse skin incision is typically made below the clavicle from a point just lateral to the sternal end of the clavicle to the deltopectoral groove. Which of the following structures would be encountered in the dissection down to the vessel?
a.Lateral thoracic artery
b. Phrenic nerve
c. Suprascapular artery
d. Thoracic duct
e. Thoraco-acromial artery

A

In the dissection down to the right axillary artery, the first structure encountered would be the Thoracoacromial artery[1][2][3][4]. This artery arises from the second part of the axillary artery and contributes to the blood supply of the shoulder and pectoral region. It passes inferiorly along the medial border of the pectoralis minor muscle before dividing into four terminal branches: Pectoral, Acromial, Clavicular, and Deltoid. Therefore, as you dissect down to the right axillary artery from a point just lateral to the sternal end of the clavicle to the deltopectoral groove, you would encounter the thoracoacromial artery first.

Sources
[1] The Thoracoacromial Artery - Course - Supply - TeachMeAnatomy https://teachmeanatomy.info/encyclopaedia/t/thoracoacromial-artery/
[2] Thoracoacromial artery - Wikipedia https://en.wikipedia.org/wiki/Thoracoacromial_artery
[3] Thoracoacromial artery | Radiology Reference Article | Radiopaedia.org https://radiopaedia.org/articles/thoracoacromial-artery?lang=gb
[4] Thoracoacromial artery: Anatomy, branches, supply - Kenhub https://www.kenhub.com/en/library/anatomy/thoracoacromial-artery
[5] The Lateral Thoracic Vessels: A Useful Recipient in Immediate … https://www.thieme-connect.com/products/ejournals/pdf/10.1055/s-0035-1570535.pdf

1493
Q

4-year-old boy presents to the Emergency Department with a two-day history of headache vomiting and drowsiness. A CT scan reveals dilatation of both his lateral ventricles and his third cerebral ventricle. His fourth ventricle was of normal size. It is suspected that he has an obstruction to his cerebrospinal fluid flow. At which of the following sites is the obstruction most likely to be?
a. Cerebral aqueduct (of Sylvius)
b. Infundibular recess
c. Interventricular foramen (of Monro)
d. Lateral foramen of fourth ventricle (foramen of Luschka)
e. Median foramen of fourth ventricle (foramen of Magendie)

A

The obstruction in the 4-year-old boy with hydrocephalus is most likely to be at the Cerebral aqueduct (of Sylvius) [4]. This site is a common location for obstructions that lead to hydrocephalus, causing dilatation of the lateral and third ventricles while the fourth ventricle remains normal in size.

Sources
[1] Hydrocephalus - Symptoms and causes - Mayo Clinic https://www.mayoclinic.org/diseases-conditions/hydrocephalus/symptoms-causes/syc-20373604
[2] Hydrocephalus | National Institute of Neurological Disorders and Stroke https://www.ninds.nih.gov/health-information/disorders/hydrocephalus
[3] Hydrocephalus | Johns Hopkins Medicine https://www.hopkinsmedicine.org/health/conditions-and-diseases/hydrocephalus
[4] Obstructive Hydrocephalus - Birmingham, AL https://neurosurgicalassociatespc.com/obstructive-hydrocephalus/
[5] Hydrocephalus and CSF Disorders - Diseases of the Brain, Head … - NCBI https://www.ncbi.nlm.nih.gov/books/NBK554339/

1494
Q

A 12-year-old boy presents to the Emergency Department two hours after helping his father cut the grass. He complains of rhinorrhea, itchy eyes, sneezing and a blocked nose. He is apyrexial with a haemoglobin of 12.2 g/dl and white blood cell count of 6.8 * 109/L with a raised eosinophil count. Chest X-ray is clear. Which immunoglobulin is most likely to cause this reaction?

a.IgA
b. IgD
c. IgE
d. IgG
e. IgM

A

The immunoglobulin most likely to cause the allergic reaction described in the scenario of a 12-year-old boy with symptoms after cutting grass is IgE. IgE is associated with allergic reactions and plays a crucial role in allergic responses, triggering the release of histamine and other chemicals that lead to allergy symptoms like rhinorrhea, itchy eyes, sneezing, and nasal congestion.
ACID (Acute-Cytotoxic-Immune Mediated-Delayed—->EGGT (IgE-IgG-IgG-T cell)

Sources
[1] Biochemistry, Immunoglobulin A - StatPearls - NCBI Bookshelf https://www.ncbi.nlm.nih.gov/books/NBK551516/
[2] Immunoglobulin A Deficiency | Johns Hopkins Medicine https://www.hopkinsmedicine.org/health/conditions-and-diseases/immunoglobulin-a-deficiency
[3] Immunoglobulin A - Wikipedia https://en.wikipedia.org/wiki/Immunoglobulin_A
[4] Adverse Reactions to Intravenous Immunoglobulins - Our Experience https://www.ncbi.nlm.nih.gov/pmc/articles/PMC6311486/
[5] Immunoglobulin A (IgA) | British Society for Immunology https://www.immunology.org/public-information/bitesized-immunology/receptors-molecules/immunoglobulin-iga

1495
Q

A 56-year-old man with chronic emphysema is on the high dependency unit (HDU), ten days after anterior resection. He has developed acute shortness of breath and hypotension.
Which of the following landmarks would be the most appropriate to use to obtain a femoral arterial blood gas sample?
a. A point midway between the anterior superior iliac spine and the pubic tubercle
b. A point midway between the anterior superior iliac-spine and the’ pubic symphysis
c. A point midway between the greater trochanter and the pubic symphysis
d. A point 1 cm inferior and 4 cm lateral to the pubic tubercle
e. A point 2 cm lateral to the midway point between the anterior superior iliac spine and pubic symphysis

A

In this scenario, given the patient’s condition and the need for a femoral arterial blood gas sample, the most appropriate landmark to use would be:

“A point midway between the anterior superior iliac spine and the pubic symphysis.”

This location is commonly referred to as the femoral triangle and is a standard site for arterial puncture procedures due to its accessibility and proximity to the femoral artery. Properly identifying this landmark ensures accurate placement of the needle for the arterial blood gas sampling procedure.

Always ensure to adhere to proper aseptic technique and consult institutional guidelines or healthcare professionals for specific protocols related to arterial puncture procedures.

For further reference:

•	https://www.ncbi.nlm.nih.gov/books/NBK430789/
•	https://www.ncbi.nlm.nih.gov/pmc/articles/PMC5542529/
1496
Q

A 70-year-old man complains of persistent numbness since an inguinal hernia repair one year previously. The numbness affects the top of the scrotum, root of penis, and a small area below the medial part of the inguinal ligament. Which nerve is likely to have been damaged?

a. Femoral branch of the genitofemoral nerve
b. Genital branch of the genitofemoral nerve
c. lliohypogastric nerve
d. Ilioinguinal nerve
e. Medial cutaneous branch of the femoral nerve

A

The likely nerve that has been damaged in the 70-year-old man experiencing persistent numbness following inguinal hernia repair is the Ilioinguinal nerve[3]. This nerve is commonly encountered during hernia repair surgeries and can be affected, leading to symptoms like numbness in specific areas such as the top of the scrotum, root of the penis, and below the medial part of the inguinal ligament. The ilioinguinal nerve is one of the nerves identified during open inguinal hernia repair, with an identification rate of 82% in surgeries[3]. Damage to this nerve can result in sensory disturbances and persistent numbness in the described regions.

Sources
[1] Inguinal hernia repair - NHS https://www.nhs.uk/conditions/inguinal-hernia-repair/
[2] Management of chronic pain after hernia repair - PMC - NCBI https://www.ncbi.nlm.nih.gov/pmc/articles/PMC5896652/
[3] Nerve identification during open inguinal hernia repair: a system… https://www.ncbi.nlm.nih.gov/pmc/articles/PMC10598160/
[4] Post Operative Pain: Inguinal Hernia Surgery with Mesh https://www.anthonyechomd.com/blog/post-operative-pain-inguinal-hernia-surgery-with-mesh
[5] Relief for Post-Surgery Hernia Pain - Verywell Health https://www.verywellhealth.com/chronic-post-hernia-repair-pain-2564628

1497
Q

A 34-year-old pregnant woman develops a swollen leg. Her mother and maternal aunt also had similar problems during their pregnancies. Which of the following tests is likely to be positive?
a. Antiendomysial antibodies
b. Antimitochondrial, antibodies
c. Antinuclear antibodies
d. Antiphospholipid antibodies
e. Antithyroglobulin antibodies

A

In this scenario, the most likely positive test would be for d. Antiphospholipid antibodies.

Antiphospholipid syndrome (APS) is an autoimmune disorder characterized by the presence of antiphospholipid antibodies in the blood, which can lead to a tendency to develop blood clots. Pregnant women with APS are at an increased risk of complications such as deep vein thrombosis (DVT) or pulmonary embolism (PE), which could result in swollen legs. Additionally, APS is known to have a familial predisposition, which aligns with the history of similar problems during pregnancy in the patient’s mother and maternal aunt.

The other options listed are associated with different autoimmune conditions and are less likely to be relevant in this context:

a. Antiendomysial antibodies: Associated with celiac disease, a condition involving gluten intolerance.
b. Antimitochondrial antibodies: Typically found in primary biliary cholangitis, a chronic liver disease.
c. Antinuclear antibodies: Often present in systemic autoimmune diseases like systemic lupus erythematosus (SLE).
e. Antithyroglobulin antibodies: Associated with autoimmune thyroid disorders such as Hashimoto’s thyroiditis.

Therefore, the most appropriate choice is d. Antiphospholipid antibodies.

Antiphospholipid antibody syndrome (APS)
• Recurrent arterial or venous thromboses and/or pregnancy morbidity with the persistent presence of anticardiolipin antibodies or lupus anticoagulant
Major types of antiphospholipid antibody
• False positive serologic test for syphilis
• Lupus anticoagulant : positive in 30% of lupus
• Anticardiolipin antibody : positive in 40-47% of lupus
• Anti-ß2 - glycoprotein I antibody

1498
Q

The “fight or flight”, response produces a release of epinephrine (adrenaline). What is the primary metabolic effect of epinephrine?
a. Alanine shunt activation
b. Cortisol release
c. Glycolysis
d. Tachycardia
e. Vasoconstriction

A

The primary metabolic effect of epinephrine is c. Glycolysis.

Epinephrine stimulates the breakdown of glycogen into glucose, which is then utilized for energy production via glycolysis. This process provides a quick source of energy to support the increased metabolic demands associated with the “fight or flight” response. Glycolysis is essential for rapidly generating ATP (adenosine triphosphate), the primary energy currency of cells, to fuel various physiological processes needed during times of stress or physical exertion.

While epinephrine can also lead to effects such as tachycardia (increased heart rate) and vasoconstriction (narrowing of blood vessels), these are primarily related to its role in increasing cardiac output and redirecting blood flow to vital organs during stressful situations. However, the primary metabolic effect, particularly in terms of energy production, is the promotion of glycolysis.

1499
Q

A 58-year-old-man underwent an emergency appendectomy. Which of the following physiological parameters are consistent with sepsis?
A- Heart Rate decreased / Systemic vascular resistance decreased / Cardiac output increased
B-Heart Rate increased / Systemic vascular resistance increased / Cardiac output decreased
C-Heart Rate increased / Systemic vascular resistance decreased / Cardiac output increased
D-Heart Rate decreased / Systemic vascular resistance increased / Cardiac output decreased
E-Heart Rate increased / Systemic vascular resistance decreased / Cardiac output decreased

A

The physiological parameters consistent with sepsis are:

C-Heart Rate increased / Systemic vascular resistance decreased / Cardiac output increased

In sepsis, there is typically an increase in heart rate (tachycardia) as the body’s response to the infection. Systemic vascular resistance tends to decrease due to the systemic inflammatory response, leading to vasodilation. Consequently, cardiac output often increases initially to compensate for the decreased vascular resistance and to maintain tissue perfusion. This pattern of increased heart rate, decreased systemic vascular resistance, and increased cardiac output is characteristic of the hyperdynamic state seen in sepsis.

So, option C is the correct choice.

1500
Q

A 50-year-old man with a 30-year history of pancolitis undergoes surveillance colonoscopy which reveals a plaque-like lesion in the descending colon. Biopsy reveals a pre-malignant change. What is the name of this pre-malignant change?
a. Anaplasia
b. Dysplasia
c. Hyperplasia
d. Metaplasia
e. Neoplasia

A

The pre-malignant change described in the scenario is:

b. Dysplasia

Dysplasia refers to the abnormal growth or development of cells within a tissue or organ. In the context of inflammatory bowel disease (IBD), such as pancolitis, patients are at increased risk of developing dysplasia, which can progress to colorectal cancer if left untreated. Therefore, surveillance colonoscopies are recommended for early detection of dysplastic changes in patients with long-standing IBD.

1501
Q

A 60-year-old man undergoes cystectomy for a bladder carcinoma. During surgery, the ureters are identified. On which region of the bladder do the ureters pierce the bladder wall?
a. Anterior surface
b. Apex
c. Lateral surfaces
d. Neck
e. Posterior surface

A

During a cystectomy for bladder carcinoma, the ureters pierce the bladder wall at the posterior surface[2].

Sources
[1] Cystectomy (bladder removal) - surgery for bladder cancer https://www.macmillan.org.uk/cancer-information-and-support/treatments-and-drugs/cystectomy-removing-the-bladder
[2] Bladder removal surgery (cystectomy) - Mayo Clinic https://www.mayoclinic.org/tests-procedures/cystectomy/about/pac-20385108
[3] Bladder reconstruction (neobladder) - Macmillan Canc… https://www.macmillan.org.uk/cancer-information-and-support/impacts-of-cancer/bladder-reconstruction
[4] Surgery to remove the bladder (cystectomy) - Cancer Research UK https://www.cancerresearchuk.org/about-cancer/bladder-cancer/treatment/invasive/surgery/removing-bladder
[5] Cystectomy (Bladder Removal): Procedure, Risks & Recovery https://my.clevelandclinic.org/health/treatments/21049-cystectomy

1502
Q

A year-old patient in the intensive care unit has a tracheostomy performed via the second, third and fourth tracheal rings. Which intervening structure is most likely to require transaction ?
a. Anterior jugular vein
b. Inferior thyroid veins
c. Sternothyroid muscle
d. Thymus
e. Thyroid isthmus

A

When performing a tracheostomy via the second, third, and fourth tracheal rings, the structure most likely to require transaction is the thyroid isthmus, which is typically located between the second and third tracheal rings.

So, the correct answer is:

e. Thyroid isthmus

This is because the thyroid isthmus is usually positioned just below the cricoid cartilage and may need to be divided or retracted during the tracheostomy procedure to access the trachea properly.

1503
Q

Which of the following anaesthetic agents is least likely to be associated with depression of myocardial contractility?

Propofol

Etomidate

Sodium thiopentone

Ether

None of the above

A

Of the agents mentioned, etomidate has the most favorable cardiac safety profile.

Anaesthetic agents

The table below summarises some of the more commonly used IV induction agents
Agent Specific features
Propofol
Rapid onset of anaesthesia
Pain on IV injection
Rapidly metabolised with little accumulation of metabolites
Proven anti emetic properties
Moderate myocardial depression
Widely used especially for maintaining sedation on ITU, total IV anaesthesia and for daycase surgery
Sodium thiopentone
Extremely rapid onset of action making it the agent of choice for rapid sequence of induction
Marked myocardial depression may occur
Metabolites build up quickly
Unsuitable for maintenance infusion
Little analgesic effects
Ketamine
May be used for induction of anaesthesia
Has moderate to strong analgesic properties
Produces little myocardial depression making it a suitable agent for anaesthesia in those who are haemodynamically unstable
May induce state of dissociative anaesthesia resulting in nightmares
Etomidate
Has favorable cardiac safety profile with very little haemodynamic instability
No analgesic properties
Unsuitable for maintaining sedation as prolonged (and even brief) use may result in adrenal suppression
Post operative vomiting is common

1504
Q

A surgeon is due to perform a laparotomy for perforated duodenal ulcer. An upper midline incision is to be performed. Which of the following structures is the incision most likely to divide?

Rectus abdominis muscle

External oblique muscle

Linea alba

Internal oblique muscle

None of the above

A

Upper midline abdominal incisions will involve the division of the linea alba. Division of muscles will not usually improve access in this approach and they would not be routinely encountered during this incision.

Abdominal incisions

Midline incision
Commonest approach to the abdomen
Structures divided: linea alba, transversalis fascia, extraperitoneal fat, peritoneum (avoid falciform ligament above the umbilicus)
Bladder can be accessed via an extraperitoneal approach through the space of Retzius
Paramedian incision
Parallel to the midline (about 3-4cm)
Structures divided/retracted: anterior rectus sheath, rectus (retracted), posterior rectus sheath, transversalis fascia, extraperitoneal fat, peritoneum
Incision is closed in layers
Battle
Similar location to paramedian but rectus displaced medially (and thus denervated)
Now seldom used
Kocher’s Incision under right subcostal margin e.g. Cholecystectomy (open)
Lanz Incision in right iliac fossa e.g. Appendicectomy
Gridiron Oblique incision centered over McBurneys point- usually appendicectomy (less cosmetically acceptable than Lanz
Gable Rooftop incision
Pfannenstiel’s Transverse supra pubic, primarily used to access pelvic organs
McEvedy’s Groin incision e.g. Emergency repair strangulated femoral hernia
Rutherford Morrison Extraperitoneal approach to left or right lower quadrants. Gives excellent access to iliac vessels and is the approach of choice for first time renal transplantation.

1505
Q

A 43 year old lady underwent an attempted placement of a central line into the internal jugular vein. Unfortunately, the doctor damaged the carotid artery and this necessitated surgical exploration. As the surgeons incise the carotid sheath a nerve is identified lying between the internal jugular vein and the carotid artery. Which of the following is this nerve most likely to be?

Glossopharyngeal nerve

Hypoglossal nerve

Superior laryngeal nerve

Recurrent laryngeal nerve

Vagus

A

The vagus lies in the carotid sheath. The hypoglossal nerve crosses the sheath, but does not lie within it.

Common carotid artery

The right common carotid artery arises at the bifurcation of the brachiocephalic trunk, the left common carotid arises from the arch of the aorta. Both terminate at the level of the upper border of the thyroid cartilage (the lower border of the third cervical vertebra) by dividing into the internal and external carotid arteries.

Left common carotid artery
This vessel arises immediately to the left and slightly behind the origin of the brachiocephalic trunk. Its thoracic portion is 2.5- 3.5 cm in length and runs superolaterally to the sternoclavicular joint.

In the thorax
The vessel is in contact, from below upwards, with the trachea, left recurrent laryngeal nerve, left margin of the oesophagus. Anteriorly the left brachiocephalic vein runs across the artery, and the cardiac branches from the left vagus descend in front of it. These structures together with the thymus and the anterior margins of the left lung and pleura separate the artery from the manubrium.

In the neck
The artery runs superiorly deep to sternocleidomastoid and then enters the anterior triangle. At this point it lies within the carotid sheath with the vagus nerve and the internal jugular vein. Posteriorly the sympathetic trunk lies between the vessel and the prevertebral fascia. At the level of C7 the vertebral artery and thoracic duct lie behind it. The anterior tubercle of C6 transverse process is prominent and the artery can be compressed against this structure (it corresponds to the level of the cricoid).
Anteriorly at C6 the omohyoid muscle passes superficial to the artery.
Within the carotid sheath the jugular vein lies lateral to the artery.

Right common carotid artery
The right common carotid arises from the brachiocephalic artery. The right common carotid artery corresponds with the cervical portion of the left common carotid, except that there is no thoracic duct on the right. The oesophagus is less closely related to the right carotid than the left.

Summary points about the carotid anatomy

Path
Passes behind the sternoclavicular joint (12% patients above this level) to the upper border of the thyroid cartilage, to divide into the external (ECA) and internal carotid arteries (ICA).

Relations
Level of 6th cervical vertebra crossed by omohyoid
Then passes deep to the thyrohyoid, sternohyoid, sternomastoid muscles.
Passes anterior to the carotid tubercle (transverse process 6th cervical vertebra)-NB compression here stops haemorrhage.
The inferior thyroid artery passes posterior to the common carotid artery.
Then : Left common carotid artery crosses the thoracic duct, Right common carotid artery crossed by recurrent laryngeal nerve

1506
Q

An unusually tall 43 year old lady presents to the surgical clinic with bilateral inguinal hernias. She develops chest pain and collapses. As part of her investigations a chest x-ray shows evidence of mediastinal widening. What is the most likely underlying diagnosis?

Pulmonary embolus

Aortic dissection

Tietze syndrome

Boerhaaves syndrome

Myocardial infarct

A

Marfans syndrome may present with a variety of connective tissue disorders such as bilateral inguinal hernia. They are at high risk of aortic dissection, as in this case.

Aortic dissection

More common than rupture of the abdominal aorta
33% of patients die within the first 24 hours, and 50% die within 48 hours if no treatment received
Associated with hypertension
Features of aortic dissection: tear in the intimal layer, followed by formation and propagation of a subintimal hematoma. Cystic medial necrosis (Marfan’s)
Most common site of dissection: 90% occurring within 10 centimetres of the aortic valve

Stanford Classification
Type Location Treatment
A Ascending aorta/ aortic root Surgery- aortic root replacement
B Descending aorta Medical therapy with antihypertensives

DeBakey classification
Type Site affected
I Ascending aorta, aortic arch, descending aorta
II Ascending aorta only
III Descending aorta distal to left subclavian artery

Clinical features
Tearing, sudden onset chest pain (painless 10%)
Hypertension or Hypotension
A blood pressure difference (in each arm) greater than 20 mm Hg
Neurologic deficits (20%)

Investigations
CXR: widened mediastinum, abnormal aortic knob, ring sign, deviation of the trachea/oesophagus
CT angiography of the thoracic aorta
MRI angiography
Conventional angiography (now rarely used diagnostically)

Management
Beta-blockers: aim HR 60-80 bpm and systolic BP 100-120 mm Hg
For type A dissections the standard of care is aortic root replacement

1507
Q

A 56 year old lady presents with a painful swelling over the lower end of the forearm following a fall. Imaging reveals a distal radial fracture with disruption of the distal radio-ulnar joint. What is the most likely fracture?

Fracture of the distal humerus

Fracture of the shaft of the radius and ulnar

Fracture of the coronoid process

Galeazzi fracture

Fracture of the radial head

A

Galeazzi fractures occur after a fall on the hand with a rotational force superimposed on it. On examination, there is bruising, swelling and tenderness over the lower end of the forearm. X- Rays reveal a displaced fracture of the radius and a prominent ulnar head due to dislocation of the inferior radio-ulnar joint.

Upper limb fractures

Colles’ fracture
Fall onto extended outstretched hands
Described as a dinner fork type deformity
Classical Colles’ fractures have the following 3 features:

Features of the injury
1. Transverse fracture of the radius
2. 1 inch proximal to the radio-carpal joint
3. Dorsal displacement and angulation

Smith’s fracture (reverse Colles’ fracture)
Volar angulation of distal radius fragment (Garden spade deformity)
Caused by falling backwards onto the palm of an outstretched hand or falling with wrists flexed

Bennett’s fracture
Intra-articular fracture of the first carpometacarpal joint
Impact on flexed metacarpal, caused by fist fights
X-ray: triangular fragment at ulnar base of metacarpal

Monteggia’s fracture
Dislocation of the proximal radioulnar joint in association with an ulna fracture
Fall on outstretched hand with forced pronation
Needs prompt diagnosis to avoid disability

Galeazzi fracture
Radial shaft fracture with associated dislocation of the distal radioulnar joint
Occur after a fall on the hand with a rotational force superimposed on it.
On examination, there is bruising, swelling and tenderness over the lower end of the forearm.
X Rays reveal the displaced fracture of the radius and a prominent ulnar head due to dislocation of the inferior radio-ulnar joint.

Barton’s fracture
Distal radius fracture (Colles’/Smith’s) with associated radiocarpal dislocation
Fall onto extended and pronated wrist

Scaphoid fractures
Scaphoid fractures are the commonest carpal fractures.
Surface of scaphoid is covered by articular cartilage with small area available for blood vessels (fracture risks blood supply)
Forms floor of anatomical snuffbox
Risk of fracture associated with fall onto outstretched hand (tubercle, waist, or proximal 1/3)
The main physical signs are swelling and tenderness in the anatomical snuff box, and pain on wrist movements and on longitudinal compression of the thumb.
Ulnar deviation AP needed for visualization of scaphoid
Immobilization of scaphoid fractures difficult

Radial head fracture
Fracture of the radial head is common in young adults.
It is usually caused by a fall on the outstretched hand.
On examination, there is marked local tenderness over the head of the radius, impaired movements at the elbow, and a sharp pain at the lateral side of the elbow at the extremes of rotation (pronation and supination).

1508
Q

How many phalanges are there in the hand?

14

12

13

10

8

A

Each digit has 3 phalanges, the thumb has two the total is therefore 14.

Anatomy of the forearm and hand

Forearm muscles
Muscle Origin Insertion Nerve supply Action
Flexor carpi radialis Common flexor origin and surrounding fascia Front of bases of second and third metacarpals Median Flexes and abducts the carpus, part flexes the elbow and part pronates forearm
Palmaris longus Common flexor origin Apex of palmar aponeurosis Median Wrist flexor
Flexor carpi ulnaris Small humeral head arises from the common flexor origin and adjacent fascia. Ulnar head comes from medial border of olecranon and posterior border of ulna Pisiform and base of the fifth metacarpal Ulnar nerve Flexes and adducts the carpus
Flexor digitorum superficialis Long linear origin from common flexor tendon, adjacent fascia and septa and medial border of the coronoid process Via tendons in the fibrous flexor sheath. At the level of the metacarpophalangeal joint each tendon split into two, these bands pass distally to their insertions Median Flexor of metacarpophalangeal joint and proximal interphalangeal joint
Flexor digitorum profundus Upper two thirds of the medial and anterior surface of the ulna, medial side of the olecranon, medial half of the interosseous membrane Via tendons that lie deep to those of flexor digitorum superficialis to insert into the distal phalanx Medial part= ulnar, lateral part=anterior interosseous nerve Flexes the distal interphalangeal joints and the wrist

Anatomy of the hand
Bones
8 Carpal bones
5 Metacarpals
14 phalanges
Intrinsic Muscles 8 Interossei - Supplied by ulnar nerve
4 palmar-adduct fingers
4 dorsal- abduct fingers
Intrinsic muscles Lumbricals
Flex MCPJ and extend the IPJ.
Origin deep flexor tendon and insertion dorsal extensor hood mechanism.
Innervation: 1st and 2nd- median nerve, 3rd and 4th- deep branch of the ulnar nerve.
Thenar eminence
Abductor pollicis brevis
Opponens pollicis
Flexor pollicis brevis
Hypothenar eminence
Opponens digiti minimi
Flexor digiti minimi brevis
Abductor digiti minimi

Fascia and compartments of the palm
The fascia of the palm is continuous with the antebrachial fascia and the fascia of the dorsum of the hand. The palmar fascia is thin over the thenar and hypothenar eminences. In contrast the palmar fascia is relatively thick. The palmar aponeurosis covers the soft tissues and overlies the flexor tendons. The apex of the palmar aponeurosis is continuous with the flexor retinaculum and the palmaris longus tendon. Distally, it forms four longitudinal digital bands that attach to the bases of the proximal phalanges, blending with the fibrous digital sheaths.
A medial fibrous septum extends deeply from the medial border of the palmar aponeurosis to the 5th metacarpal. Lying medial to this are the hypothenar muscles. In a similar fashion, a lateral fibrous septum extends deeply from the lateral border of the palmar aponeurosis to the 3rd metacarpal. The thenar compartment lies lateral to this area.
Lying between the thenar and hypothenar compartments is the central compartment. It contains the flexor tendons and their sheaths, the lumbricals, the superficial palmar arterial arch and the digital vessels and nerves.
The deepest muscular plane is the adductor compartment, which contains adductor pollicis.

Short muscles of the hand
These comprise the lumbricals and interossei. The four slender lumbrical muscles flex the fingers at the metacarpophalangeal joints and extend the interphalangeal joint. The four dorsal interossei are located between the metacarpals and the four palmar interossei lie on the palmar surface of the metacarpals in the interosseous compartment of the hand.

Long flexor tendons and sheaths in the hand
The tendons of FDS and FDP enter the common flexor sheath deep to the flexor retinaculum. The tendons enter the central compartment of the hand and fan out to their respective digital synovial sheaths. Near the base of the proximal phalanx, the tendon of FDS splits to permit the passage of FDP. The FDP tendons are attached to the margins of the anterior aspect of the base of the distal phalanx.
The fibrous digital sheaths contain the flexor tendons and their synovial sheaths. These extend from the heads of the metacarpals to the base of the distal phalanges.

1509
Q

A 23 year old man is injured during a game of rugby. He suffers a fracture of the distal third of his clavicle, it is a compound fracture and there is evidence of arterial haemorrhage. Which of the following vessels is most likely to be encountered first during subsequent surgical exploration?

Posterior circumflex humeral artery

Axillary artery

Thoracoacromial artery

Sub scapular artery

Lateral thoracic artery

A

The thoracoacromial artery arises from the second part of the axillary artery. It is a short, wide trunk, which pierces the clavipectoral fascia, and ends, deep to pectoralis major by dividing into four branches.

Thoracoacromial artery

The thoracoacromial artery (acromiothoracic artery; thoracic axis) is a short trunk, which arises from the forepart of the axillary artery, its origin being generally overlapped by the upper edge of the Pectoralis minor.

Projecting forward to the upper border of the Pectoralis minor, it pierces the coracoclavicular fascia and divides into four branches: pectoral, acromial, clavicular, and deltoid.

Branch Description
Pectoral branch Descends between the two Pectoral muscles, and is distributed to them and to the breast, anastomosing with the intercostal branches of the internal thoracic artery and with the lateral thoracic.
Acromial branch Runs laterally over the coracoid process and under the Deltoid, to which it gives branches; it then pierces that muscle and ends on the acromion in an arterial network formed by branches from the suprascapular, thoracoacromial, and posterior humeral circumflex arteries.
Clavicular branch Runs upwards and medially to the sternoclavicular joint, supplying this articulation, and the Subclavius.
Deltoid branch Arising with the acromial, it crosses over the Pectoralis minor and passes in the same groove as the cephalic vein, between the Pectoralis major and Deltoid, and gives branches to both muscles.

1510
Q

A 25 year old man from the far east presents with a fever and right upper quadrant pain. As part of his investigations a CT scan shows an ill defined lesion in the right lobe of the liver. What is the most probable cause?

Hydatid cyst

Amoebic abscess

Hepatocellular carcinoma

Simple liver cyst

Liver cell adenoma

A

B-Amoebic abscess

Benign liver lesions
Haemangioma
Most common benign tumours of mesenchymal origin
Incidence in autopsy series is 8%
Cavernous haemangiomas may be enormous
Clinically they are reddish purple hypervascular lesions
Lesions are normally separated from normal liver by ring of fibrous tissue
On ultrasound they are typically hyperechoic
Liver cell adenoma
90% develop in women in their third to fifth decade
Linked to use of oral contraceptive pill
Lesions are usually solitary
They are usually sharply demarcated from normal liver although they usually lack a fibrous capsule
On ultrasound the appearances are of mixed echoity and heterogeneous texture. On CT most lesions are hypodense when imaged prior to administration of IV contrast agents
In patients with haemorrhage or symptoms removal of the adenoma may be required
Mesenchymal hamartomas Congential and benign, usually present in infants. May compress normal liver
Liver abscess
Biliary sepsis is a major predisposing factor
Structures drained by the portal venous system form the second largest source
Common symptoms include fever, right upper quadrant pain. Jaundice may be seen in 50%
Ultrasound will usually show a fluid filled cavity, hyperechoic walls may be seen in chronic abscesses
Amoebic abscess
Liver abscess is the most common extra intestinal manifestation of amoebiasis
Between 75 and 90% lesions occur in the right lobe
Presenting complaints typically include fever and right upper quadrant pain
Ultrasonography will usually show a fluid filled structure with poorly defined boundaries
Aspiration yield sterile odourless fluid which has an anchovy paste consistency
Treatment is with metronidazole
Hyatid cysts
Seen in cases of Echinococcus infection
Typically an intense fibrotic reaction occurs around sites of infection
The cyst has no epithelial lining
Cysts are commonly unilocular and may grow to 20cm in size. The cyst wall is thick and has an external laminated hilar membrane and an internal enucleated germinal layer
Typically presents with malaise and right upper quadrant pain. Secondary bacterial infection occurs in 10%.
Liver function tests are usually abnormal and eosinophilia is present in 33% cases
Ultrasound may show septa and hyatid sand or daughter cysts.
Percutaneous aspiration was previously contra indicated, it is now incorporated into some treatment regimens
Treatment is by sterilisation of the cyst with mebendazole and may be followed by surgical resection. Hypertonic swabs are packed around the cysts during surgery
Polycystic liver disease
Usually occurs in association with polycystic kidney disease
Autosomal dominant disorder
Symptoms may occur as a result of capsular stretch
Cystadenoma
Rare lesions with malignant potential
Usually solitary multiloculated lesions
Liver function tests usually normal
Ultrasonography typically shows a large anechoic, fluid filled area with irregular margins. Internal echos may result from septa
Surgical resection is indicated in all cases

1511
Q

Which of the following cranial foramina pairings are incorrect?

Carotid canal and internal carotid artery.

Foramen ovale and mandibular nerve.

Optic canal and ophthalmic artery.

Optic canal and ophthalmic nerve.

Foramen rotundum and maxillary nerve.

A

The optic canal transmits the optic nerve. The ophthalmic nerve traverses the superior orbital fissure.

Foramina of the base of the skull

Foramen Location Contents
Foramen ovale Sphenoid bone Otic ganglion
V3 (Mandibular nerve:3rd branch of
trigeminal)
Accessory meningeal artery
Lesser petrosal nerve
Emissary veins
Foramen spinosum Sphenoid bone Middle meningeal artery
Meningeal branch of the Mandibular nerve
Foramen rotundum Sphenoid bone Maxillary nerve (V2)
Foramen lacerum/ carotid canal Located between the sphenoid, the apex of the petrous temporal and the basilar part of the occipital Base of the medial pterygoid plate.
Internal carotid artery*
Nerve and artery of the pterygoid canal
Jugular foramen Temporal bone Anterior: inferior petrosal sinus
Intermediate: glossopharyngeal, vagus, and accessory nerves.
Posterior: sigmoid sinus (becoming the internal jugular vein) and some meningeal branches from the occipital and ascending pharyngeal arteries.
Foramen magnum Occipital bone Anterior and posterior spinal arteries
Vertebral arteries
Medulla oblongata
Stylomastoid foramen Temporal bone Stylomastoid artery
Facial nerve
Superior orbital fissure Sphenoid bone Oculomotor nerve (III)
Recurrent meningeal artery
Trochlear nerve (IV)
Lacrimal, frontal and nasociliary branches of opthalmic nerve (V1)
Abducent nerve (VI)
Superior ophthalmic vein

*= In life the foramen lacerum is occluded by a cartilagenous plug. The ICA initially passes into the carotid canal which ascends superomedially to enter the cranial cavity through the foramen lacerum.

1512
Q

A 65 year old lady presents with fatigue and is found to have an iron deficiency anaemia. Which of these investigations should be undertaken first?

Colonoscopy

Gastroscopy

Capsule endoscopy

Small bowel MRI scan

Mesenteric CT angiogram

A

Colorectal cancer is a major cause of iron deficiency anaemia and the colon is therefore investigated first before other differentials are considered.

Lower Gastrointestinal bleeding

Colonic bleeding
This typically presents as bright red or dark red blood per rectum. Colonic bleeding rarely presents as malaena type stool, this is because blood in the colon has a powerful laxative effect and is rarely retained long enough for transformation to occur and because the digestive enzymes present in the small bowel are not present in the colon. Up to 15% of patients presenting with haemochezia will have an upper gastrointestinal source of haemorrhage.

As a general rule right sided bleeds tend to present with darker coloured blood than left sided bleeds. Haemorrhoidal bleeding typically presents as bright red rectal bleeding that occurs post defecation either onto toilet paper or into the toilet pan. It is very unusual for haemorrhoids alone to cause any degree of haemodynamic compromise.

Causes
Cause Presenting features
Colitis Bleeding may be brisk in advanced cases, diarrhoea is commonly present. Abdominal x-ray may show featureless colon.
Diverticular disease Acute diverticulitis often is not complicated by major bleeding and diverticular bleeds often occur sporadically. 75% all will cease spontaneously within 24-48 hours. Bleeding is often dark and of large volume.
Cancer Colonic cancers often bleed and for many patients this may be the first sign of the disease. Major bleeding from early lesions is uncommon
Haemorrhoidal bleeding Typically bright red bleeding occurring post defecation. Although patients may give graphic descriptions bleeding of sufficient volume to cause haemodynamic compromise is rare.
Angiodysplasia Apart from bleeding, which may be massive, these arteriovenous lesions cause little in the way of symptoms. The right side of the colon is more commonly affected.

Management
Prompt correction of any haemodynamic compromise is required. Unlike upper gastrointestinal bleeding the first line management is usually supportive. This is because in the acute setting endoscopy is rarely helpful.
When haemorrhoidal bleeding is suspected a proctosigmoidoscopy is reasonable as attempts at full colonoscopy are usually time consuming and often futile.
In the unstable patient the usual procedure would be an angiogram (either CT or percutaneous), when these are performed during a period of haemodynamic instability they may show a bleeding point and may be the only way of identifying a patch of angiodysplasia.
In others who are more stable the standard procedure would be a colonoscopy in the elective setting. In patients undergoing angiography attempts can be made to address the lesion in question such as coiling. Otherwise surgery will be necessary.
In patients with ulcerative colitis who have significant haemorrhage the standard approach would be a sub total colectomy, particularly if medical management has already been tried and is not effective.

Indications for surgery
Patients > 60 years
Continued bleeding despite endoscopic intervention
Recurrent bleeding
Known cardiovascular disease with poor response to hypotension

Surgery
Selective mesenteric embolisation if life threatening bleeding. This is most helpful if conducted during a period of relative haemodynamic instability. If all haemodynamic parameters are normal then the bleeding is most likely to have stopped and any angiography normal in appearance. In many units a CT angiogram will replace selective angiography but the same caveats will apply.

If the source of colonic bleeding is unclear; perform a laparotomy, on table colonic lavage and following this attempt a resection. A blind sub total colectomy is most unwise, for example bleeding from an small bowel arterio-venous malformation will not be treated by this manoeuvre.

Summary of Acute Lower GI bleeding recommendations
Consider admission if:
* Over 60 years
* Haemodynamically unstable/profuse PR bleeding
* On aspirin or NSAID
* Significant co morbidity

Management
All patients should have a history and examination, PR and proctoscopy
Colonoscopic haemostasis aimed for in post polypectomy or diverticular bleeding

References
http://www.sign.ac.uk/guidelines/fulltext/105/index.html

1513
Q

Which of the following treatments is not utilized in the treatment of overactive bladder syndrome?

Sacral neuromodulation

Botulinum toxin injections

Bladder drill

Oxybutinin

Colposuspension

A

Overactive bladder syndrome is very common and first line management includes the use of anticholinergics and bladder drill whereby voiding is deferred. Refractory cases can be treated with SNS or botulinum toxin injections. A Burch Colposuspension is used to treat stress urinary incontinence.

Urinary incontinence

Involuntary passage of urine. Most cases are female (80%). It has a prevalence of 11% in those aged greater than 65 years. The commonest variants include:
Stress urinary incontinence (50%)
Urge incontinence (15%)
Mixed (35%)

Males
Males may also suffer from incontinence although it is a much rarer condition in men. A number of anatomical factors contribute to this. Males have 2 powerful sphincters; one at the bladder neck and the other in the urethra. Damage to the bladder neck mechanism is a factor in causing retrograde ejaculation following prostatectomy. The short segment of urethra passing through the urogenital diaphragm consists of striated muscle fibres (the external urethral sphincter) and smooth muscle capable of more sustained contraction. It is the latter mechanism that maintains continence following prostatectomy.

Females
The sphincter complex at the level of bladder neck is poorly developed in females. As a result the external sphincter complex is functionally more important, its composition being similar to that of males. Innervation is via the pudendal nerve and the neuropathy that may accompany obstetric events may compromise this and lead to stress urinary incontinence.

Innervation
Somatic innervation to the bladder is via the pudendal, hypogastric and pelvic nerves. Autonomic nerves travel in these nerve fibres too. Bladder filling leads to detrusor relaxation (sympathetic) coupled with sphincter contraction. The parasympathetic system causes detrusor contraction and sphincter relaxation. Overall control of micturition is centrally mediated via centres in the Pons.

Stress urinary incontinence
50% of cases, especially in females.
Damage (often obstetric) to the supporting structures surrounding the bladder may lead to urethral hypermobility.
Other cases due to sphincter dysfunction, usually from neurological disorders (e.g. Pudendal neuropathy, multiple sclerosis).

Urethral mobility:
Pressure not transmitted appropriately to the urethra resulting in involuntary passage of urine during episodes of raised intra-abdominal pressure.

Sphincter dysfunction:
Sphincter fails to adapt to compress urethra resulting in involuntary passage of urine. When the sphincter completely fails there is often to continuous passage of urine.

Urge incontinence
In these patients there is sense of urgency followed by incontinence. The detrusor muscle in these patients is unstable and urodynamic investigation will demonstrate overactivity of the detrusor muscle at inappropriate times (e.g. Bladder filling). Urgency may be seen in patients with overt neurological disorders and those without. The pathophysiology is not well understood but poor central and peripheral co-ordination of the events surrounding bladder filling are the main processes.

Assessment
Careful history and examination including vaginal examination for cystocele.
Bladder diary for at least 3 days
Consider flow cystometry if unclear symptomatology or surgery considered and diagnosis is unclear.
Exclusion of other organic disease (e.g. Stones, UTI, Cancer)

Management
Conservative measures should be tried first; Stress urinary incontinence or mixed symptoms should undergo 3 months of pelvic floor exercise. Over active bladder should have 6 weeks of bladder retraining.
Drug therapy for women with overactive bladder should be offered oxybutynin (or solifenacin if elderly) if conservative measures fail.
In women with detrusor instability who fail non operative therapy a trial of sacral neuromodulation may be considered, with conversion to permanent implant if good response. Augmentation cystoplasty is an alternative but will involve long term intermittent self catheterisation.
In women with stress urinary incontinence a urethral sling type procedure may be undertaken. Where cystocele is present in association with incontinence it should be repaired particularly if it lies at the introitus.

NICE guidelines
Initial assessment urinary incontinence should be classified as stress/urge/mixed.
At least 3/7 bladder diary if unable to classify easily.
Start conservative treatment before urodynamic studies if a diagnosis is obvious from the history
Urodynamic studies if plans for surgery.
Stress incontinence: Pelvic floor exercises 3/12, if fails consider surgery.
Urge incontinence: Bladder training >6/52, if fails for oxybutynin (antimuscarinic drugs) then sacral nerve stimulation.
Pelvic floor exercises offered to all women in their 1st pregnancy.

1514
Q

A 45 year old lady is undergoing a Whipples procedure for carcinoma of the pancreatic head. The bile duct is transected. Which of the following vessels is mainly responsible for the blood supply to the bile duct remnant?

Cystic artery

Hepatic artery

Portal vein

Left gastric artery

None of the above

A

Do not confuse the blood supply of the bile duct with that of the cystic duct.
The bile duct has an axial blood supply which is derived from the hepatic artery and from retroduodenal branches of the gastroduodenal artery. Unlike the liver there is no contribution by the portal vein to the blood supply of the bile duct. Damage to the hepatic artery during a difficult cholecystectomy is a recognised cause of bile duct strictures. In this scenario the distal vessels have been removed as the patient is undergoing a resection.

Gallbladder

  • Fibromuscular sac with capacity of 50ml
    Columnar epithelium

Relations of the gallbladder
Anterior Liver
Posterior
Covered by peritoneum
Transverse colon
1st part of the duodenum
Laterally Right lobe of liver
Medially Quadrate lobe of liver

Arterial supply
Cystic artery (branch of Right hepatic artery)

Venous drainage
Directly to the liver

Nerve supply
Sympathetic- mid thoracic spinal cord, Parasympathetic- anterior vagal trunk

Common bile duct

Origin Confluence of cystic and common hepatic ducts
Relations at origin
Medially - Hepatic artery
Posteriorly- Portal vein
Relations distally
Duodenum - anteriorly
Pancreas - medially and laterally
Right renal vein - posteriorly
Arterial supply Branches of hepatic artery and retroduodenal branches of gastroduodenal artery

Hepatobiliary triangle

Medially Common hepatic duct
Inferiorly Cystic duct
Superiorly Inferior edge of liver
Contents Cystic artery

1515
Q

Which of the conditions listed below is most likely to account for the following arterial blood gas result:
pH 7.49
pO2 8.5
Bicarbonate 22
pCO2 2.4
Chloride 12meq

Respiratory alkalosis

Metabolic alkalosis

Metabolic acidosis

Type II respiratory failure

Metabolic acidosis with increased anion gap

A

The hyperventilation results in decreased carbon dioxide levels, causing a respiratory alkalosis (non compensated).

Metabolic acidosis
This is the most common surgical acid - base disorder.
Reduction in plasma bicarbonate levels.
Two mechanisms:
1. Gain of strong acid (e.g. diabetic ketoacidosis)
2. Loss of base (e.g. from bowel in diarrhoea)
- Classified according to the anion gap, this can be calculated by:
(Na+ + K+) - (Cl- + HCO3-).
- If a question supplies the chloride level then this is often a clue that the anion gap should be calculated. The normal range = 10-18 mmol/L

Normal anion gap ( = hyperchloraemic metabolic acidosis)
Gastrointestinal bicarbonate loss: diarrhoea, ureterosigmoidostomy, fistula
Renal tubular acidosis
Drugs: e.g. acetazolamide
Ammonium chloride injection
Addison’s disease

Raised anion gap
Lactate: shock, hypoxia
Ketones: diabetic ketoacidosis, alcohol
Urate: renal failure
Acid poisoning: salicylates, methanol

Metabolic acidosis secondary to high lactate levels may be subdivided into two types:
Lactic acidosis type A: (Perfusion disorders e.g.shock, hypoxia, burns)
Lactic acidosis type B: (Metabolic e.g. metformin toxicity)

Metabolic alkalosis
Usually caused by a rise in plasma bicarbonate levels.
Rise of bicarbonate above 24 mmol/L will typically result in renal excretion of excess bicarbonate.
Caused by a loss of hydrogen ions or a gain of bicarbonate. It is due mainly to problems of the kidney or gastrointestinal tract

Causes
Vomiting / aspiration (e.g. Peptic ulcer leading to pyloric stenosis, nasogastric suction)
Diuretics
Liquorice, carbenoxolone
Hypokalaemia
Primary hyperaldosteronism
Cushing’s syndrome
Bartter’s syndrome
Congenital adrenal hyperplasia

Mechanism of metabolic alkalosis
Activation of renin-angiotensin II-aldosterone (RAA) system is a key factor
Aldosterone causes reabsorption of Na+ in exchange for H+ in the distal convoluted tubule
ECF depletion (vomiting, diuretics) → Na+ and Cl- loss → activation of RAA system → raised aldosterone levels
In hypokalaemia, K+ shift from cells → ECF, alkalosis is caused by shift of H+ into cells to maintain neutrality

Respiratory acidosis
Rise in carbon dioxide levels usually as a result of alveolar hypoventilation
Renal compensation may occur leading to Compensated respiratory acidosis

Causes
COPD
Decompensation in other respiratory conditions e.g. Life-threatening asthma / pulmonary oedema
Sedative drugs: benzodiazepines, opiate overdose

Respiratory alkalosis
Hyperventilation resulting in excess loss of carbon dioxide
This will result in increasing pH

Causes
Psychogenic: anxiety leading to hyperventilation
Hypoxia causing a subsequent hyperventilation: pulmonary embolism, high altitude
Early salicylate poisoning*
CNS stimulation: stroke, subarachnoid haemorrhage, encephalitis
Pregnancy

*Salicylate overdose leads to a mixed respiratory alkalosis and metabolic acidosis. Early stimulation of the respiratory centre leads to a respiratory alkalosis whilst later the direct acid effects of salicylates (combined with acute renal failure) may lead to an acidosis

1516
Q

A 22 year old man has a full thickness burn of his leg after being trapped in a burning car. There are no fractures of the limb. The burn is circumferential. After 2 hours he complains of tingling of his leg and it appears dusky. What is the best management for this?

Fasciotomy

Escharotomy

Angioplasty

Pain control

Anticoagulation

A

The full thickness burn has oedema which is affecting the peripheral circulation. Therefore the burn needs to be divided (not the fascia) to allow normal circulation to return.

Burns

Burns may be thermal, chemical or electrical. In the former category are burns which occur as a result of heat. Chemical burns occur when the skin is exposed to an extremely caustic or alkaline substance. Electrical burns occur following exposure to electrical current. The immediate management includes removal of the burning source which usually includes irrigation of the burned area. A detailed assessment then needs to be made of the extent of the burns and a number of charts are available for recording this information. The degree of injury relates to the temperature and duration of exposure. Most domestic burns are mainly scalds in young children.

Following the burn, there is a local response with progressive tissue loss and release of inflammatory cytokines. Systemically, there are cardiovascular effects resulting from fluid loss and sequestration of fluid into the third space. There is a marked catabolic response. Immunosuppression is common with large burns and bacterial translocation from the gut lumen is a recognised event. Sepsis is a common cause of death following major burns.

Types of burn

Type of burn Skin layers affected Skin appearance Blanching Management
Epidermal/Superficial Epidermis Red, moist Yes
Superficial partial thickness Epidermis and part of papillary dermis affected Pale, dry Yes Normally heals with no intervention
Deep partial thickness Epidermis, whole papillary dermis affected Mottled red colour No Needs surgical intervention (depending on site)
Full thickness Whole skin layer and subcutaneous tissue affected Dry, leathery hard wound No Burns centre

Depth of burn assessment
Bleeding on needle prick
Sensation
Appearance
Blanching to pressure

Percentage burn estimation
Lund Browder chart: most accurate even in children
Wallace rule of nines
Palmar surface: surface area palm = 0.8% burn

> 15% body surface area burns in adults needs urgent burn fluid resuscitation

Transfer to burn centre if:
Need burn shock resuscitation
Face/hands/genitals affected
Deep partial thickness or full thickness burns
Significant electrical/chemical burns

Management
The initial aim is to stop the burning process and resuscitate the patient. Intravenous fluids will be required for children with burns greater than 10% of total body surface area. Adults with burns greater than 15% of total body surface area will also require IV fluids. The fluids are calculated using the Parkland formula which is; volume of fluid= total body surface area of the burn % x weight (Kg) x2-4 (preference for lower amount i.e. 2ml to avoid excessive fluid overload). Half of the fluid is administered in the first 8 hours. A urinary catheter should be inserted. Analgesia should be given. Complex burns, burns involving the hand perineum and face and burns >10% in adults and >5% in children should be transferred to a burns unit.

Circumferential burns affecting a limb or severe torso burns impeding respiration may require escharotomy to divide the burnt tissue.

Conservative management is appropriate for superficial burns and mixed superficial burns that will heal in 2 weeks. More complex burns may require excision and skin grafting. Excision and primary closure is not generally practised as there is a high risk of infection.

There is no evidence to support the use of anti microbial prophylaxis or topical antibiotics in burn patients.

Escharotomies
Indicated in circumferential full thickness burns to the torso or limbs.
Careful division of the encasing band of burn tissue will potentially improve ventilation (if the burn involves the torso), or relieve compartment syndrome and oedema (where a limb is involved)

References
www.euroburn.org/e107files/downloads/guidelinesburncare.pdf

Barajas-Nava LA, López-Alcalde J, Roqué i Figuls M, Solà I, Bonfill Cosp X. Antibiotic prophylaxis for preventing burn wound infection. Cochrane Database of Systematic Reviews 2013, Issue 6. Art. No.: CD008738. DOI: 10.1002/14651858.CD008738.pub2.

Hettiaratchy S & Papini R. Initial management of a major burn: assessment and resuscitation. BMJ 2004;329:101-103

1517
Q

Which is the characteristic finding on a blood film post splenectomy?

Stipple cell

Tear drop cell

Reticulocytes

Howell-Jolly bodies

Schistocyte

A

Blood film in hyposplenism:

Howell-Jolly bodies
Pappenheimer bodies
Poikilocytes (Target cells)
Erythrocyte containing siderotic granules
Heinz bodies

Splenectomy

Indications
Trauma: 1/4 are iatrogenic
Spontaneous rupture: EBV
Hypersplenism: hereditary spherocytosis or elliptocytosis etc
Malignancy: lymphoma or leukaemia
Splenic cysts, hydatid cysts, splenic abscesses

Splenectomy
Technique
Trauma
GA
Long midline incision
If time permits insert a self retaining retractor (e.g. Balfour/ omnitract)
Large amount of free blood is usually present. Pack all 4 quadrants of the abdomen. Allow the anaesthetist to ‘catch up’
Remove the packs and assess the viability of the spleen. Hilar injuries and extensive parenchymal lacerations will usually require splenectomy.
Divide the short gastric vessels and ligate them.
Clamp the splenic artery and vein. Two clamps on the patient side are better and allow for double ligation and serve as a safety net if your assistant does not release the clamp smoothly.
Be careful not to damage the tail of the pancreas, if you do then this will need to be formally removed and the pancreatic duct closed.
Wash out the abdomen and place a tube drain to the splenic bed.
Some surgeons implant a portion of spleen into the omentum, whether you decide to do this is a matter of personal choice.
Post operatively the patient will require prophylactic penicillin V and pneumococcal vaccine.

Elective
Elective splenectomy is a very different operation from that performed in the emergency setting. The spleen is often large (sometimes massive). Most cases can be performed laparoscopically. The spleen will often be macerated inside a specimen bag to facilitate extraction.

Complications
Haemorrhage (may be early and either from short gastrics or splenic hilar vessels
Pancreatic fistula (from iatrogenic damage to pancreatic tail)
Thrombocytosis: prophylactic aspirin
Encapsulated bacteria infection e.g. Strep. pneumoniae, Haemophilus influenzae and Neisseria meningitidis

Post splenectomy changes
Platelets will rise first (therefore in ITP should be given after splenic artery clamped)
Blood film will change over following weeks, Howell Jolly bodies will appear
Other blood film changes include target cells and Pappenheimer bodies
Increased risk of post splenectomy sepsis, therefore prophylactic antibiotics and pneumococcal vaccine should be given.

Post splenectomy sepsis
Typically occurs with encapsulated organisms
Opsonisation occurs but then not recognised

1518
Q

A 64 year old obese female presents with a breast lump. She was hit on the breast by a cricket ball when playing with her grandson. What is the most likely explanation?

Invasive ductal carcinoma

Ductal carcinoma in situ

Fat necrosis

Fibrocystic disease

Duct ectasia

A

An obese, post menopausal woman, with a history of trauma points towards fat necrosis. Trauma causes inflammation of fat cells, leading to formation of a lump. Mammography, USS and biopsy are usually needed to differentiate it from breast malignancy.

Benign breast lesions

Lesion Features Treatment
Fibroadenoma
Develop from a whole lobule
Mobile, firm breast lumps
12% of all breast masses
Over a 2 year period up to 30% will get smaller
No increase in risk of malignancy
If >3cm surgical excision is usual, Phyllodes tumours should be widely excised (mastectomy if the lesion is large)
Breast cyst
7% of all Western females will present with a breast cyst
Usually presents as a smooth discrete lump (may be fluctuant)
Small increased risk of breast cancer (especially if younger)
Cysts should be aspirated, those which are blood stained or persistently refill should be biopsied or excised
Sclerosing adenosis, (radial scars and complex sclerosing lesions)
Usually presents as a breast lump or breast pain
Causes mammographic changes which may mimic carcinoma
Cause distortion of the distal lobular unit, without hyperplasia (complex lesions will show hyperplasia)
Considered a disorder of involution, no increase in malignancy risk
Lesions should be biopsied, excision is not mandatory
Epithelial hyperplasia
Variable clinical presentation ranging from generalised lumpiness through to discrete lump
Disorder consists of increased cellularity of terminal lobular unit, atypical features may be present
Atypical features and family history of breast cancer confers greatly increased risk of malignancy
If no atypical features then conservative, those with atypical features require either close monitoring or surgical resection
Fat necrosis
Up to 40% cases usually have a traumatic aetiology
Physical features usually mimic carcinoma
Mass may increase in size initially
Imaging and core biopsy
Duct papilloma
Usually present with nipple discharge
Large papillomas may present with a mass
The discharge usually originates from a single duct
No increase risk of malignancy

1519
Q

A 4 year old child is brought to the clinic by his father. They are concerned because the child has been noted to have a small epithelial defect anterior to the left ear and is has been noted to discharge foul smelling material for the past 2 days. What is the most likely explanation?

Otitis externa

Otitis media

Pre auricular sinus

Cholesteatoma

Long standing pars flaccida perforation

A

Pre auricular sinuses that are deeper may accumulate secretions and produce foul smelling discharge.

Disorders affecting the ear

Otitis externa
Variant Cause Features Treatment
Acute otitis externa Boil in external auditory meatus Acute pain on moving the pinna
Conductive hearing loss if lesion is large
When rupture occurs pus will flow from ear Ear packs may be used
Topical antibiotics
Operative debridement may be needed in severe cases
Chronic otitis externa Chronic combined infection in the external auditory meatus usually combined staphylococcal and fungal infection Chronic discharge from affected ear, hearing loss and severe pain rare Cleansing of the external ear and treatment with antifungal and antibacterial ear drops

Otitis media
Variant Cause Features Treatment
Acute suppurative otitis media Viral induced middle ear effusions secondary to eustacian tube dysfunction Most common in children and rare in adults
May present with symptoms elsewhere (e.g. vomiting) in children
Severe pain and sometimes fever
May present with discharge if tympanic rupture occurs Antibiotics (usually amoxycillin)
Chronic suppurative otitis media May occur with or without cholesteatoma
Those without cholesteatoma have a perforation of the pars tensa
Those with cholesteatoma have a perforation of the pars flaccida Those without cholesteatoma may complain of intermittent discharge (non offensive)
Those with cholesteatoma have impaired hearing and foul smelling discharge Simple pars tensa perforations may be managed non operatively or a myringoplasty considered if symptoms troublesome.
Pars flaccida perforations will usually require a modified radical mastoidectomy

Otosclerosis
Progressive conductive deafness
Secondary to fixation of the stapes in the oval window
Treatment is with stapedectomy and insertion of a prosthesis

Acoustic neuroma
Symptoms of gradually progressive unilateral perceptive deafness and tinnitus
Involvement of the vestibular nerve may cause vertigo
Extension to involve the facial nerve may cause weakness and then paralysis.

Pre auricular sinus
Common congenital condition in which an epithelial defect forms around the external ear
Small sinuses require no treatment
Deeper sinuses may become blocked and develop episodes of infection, they may be closely related to the facial nerve and are challenging to excise

1520
Q

Which of these structures are not contained within the rectus sheath?

Pyramidalis

Superior epigastric artery

Inferior epigastric vein

Internal iliac artery

Rectus abdominis

A

The rectus sheath also contains:
superior epigastric vein
inferior epigastric artery

Abdominal wall

The 2 main muscles of the abdominal wall are the rectus abdominis (anterior) and the quadratus lumborum (posterior).
The remaining abdominal wall consists of 3 muscular layers. Each muscle passes from the lateral aspect of the quadratus lumborum posteriorly to the lateral margin of the rectus sheath anteriorly. Each layer is muscular posterolaterally and aponeurotic anteriorly.

Muscles of abdominal wall
External oblique
Lies most superficially
Originates from 5th to 12th ribs
Inserts into the anterior half of the outer aspect of the iliac crest, linea alba and pubic tubercle
More medially and superiorly to the arcuate line, the aponeurotic layer overlaps the rectus abdominis muscle
The lower border forms the inguinal ligament
The triangular expansion of the medial end of the inguinal ligament is the lacunar ligament.
Internal oblique
Arises from the thoracolumbar fascia, the anterior 2/3 of the iliac crest and the lateral 2/3 of the inguinal ligament
The muscle sweeps upwards to insert into the cartilages of the lower 3 ribs
The lower fibres form an aponeurosis that runs from the tenth costal cartilage to the body of the pubis
At its lowermost aspect it joins the fibres of the aponeurosis of transversus abdominis to form the conjoint tendon.
Transversus abdominis
Innermost muscle
Arises from the inner aspect of the costal cartilages of the lower 6 ribs , from the anterior 2/3 of the iliac crest and lateral 1/3 of the inguinal ligament
Its fibres run horizontally around the abdominal wall ending in an aponeurosis. The upper part runs posterior to the rectus abdominis. Lower down the fibres run anteriorly only.
The rectus abdominis lies medially; running from the pubic crest and symphysis to insert into the xiphoid process and 5th, 6th and 7th costal cartilages. The muscles lies in a aponeurosis as described above.
Nerve supply: anterior primary rami of T7-12

Surgical notes
During abdominal surgery it is usually necessary to divide either the muscles or their aponeuroses. During a midline laparotomy it is desirable to divide the aponeurosis. This will leave the rectus sheath intact above the arcuate line and the muscles intact below it. Straying off the midline will often lead to damage to the rectus muscles, particularly below the arcuate line where they may often be in close proximity to each other.

1521
Q

A 73 year old man undergoes an emergency amputation for severe lower limb sepsis and gangrene. Post operatively he develops disseminated intravascular coagulation. Which of the following clotting factors will be most rapidly consumed in this process?

Factor V and VIII

Factor I

Factor I and III

Factor III and VII

Factor VI and VIII

A

D-I-S-S-E-M-I-N-A-T-E-D

D-Dx: D dimer
I-Immune complexes
S-Snakebite, shock, heatstroke
S-SLE
E-Eclampsia, HELLP syndrome
M-Massive tissue damage
I-Infections: viral and bacterial
N-Neoplasms
A-Acute promyelocytic leukemia
T-Tumor products: Tissue Factor (TF) and TF-like factors released by carcinomas of pancreas, prostate, lung,
colon, stomach
E-Endotoxins (bacterial)
D-Dead fetus (retained)
DIC Will tend to consume factors five and eight intially (and platelets).

Disseminated intravascular coagulation

Simultaneous coagulation and haemorrhage caused by initially formation of thrombi which consume clotting factors (factors 5,8) and platelets, ultimately leading to bleeding

Causes include:
Infection
Malignancy
Trauma e.g. major surgery, burns, shock, dissecting aortic aneurysm
Liver disease
Obstetric complications

Key points
Clinically bleeding is usually a dominant feature, bruising, ischaemia and organ failure
Blood tests: prolonged clotting times, thrombocytopenia, decreased fibrinogen, increased fibrinogen degradation products
Treat the underlying cause and supportive management

1522
Q

What is the most useful test to clinically distinguish between an upper and lower motor neurone lesion of the facial nerve?

Blow cheeks out

Loss of chin reflex

Close eye

Raise eyebrow

Open mouth against resistance

A

Upper motor neurone lesions of the facial nerve- Paralysis of the lower half of face.
Lower motor neurone lesion- Paralysis of the entire ipsilateral face.

Upper Vs Lower motor neurone lesions - Facial nerve

The nucleus of the facial nerve is located in the caudal aspect of the ventrolateral pontine tegmentum. Its axons exit the ventral pons medial to the spinal trigeminal nucleus.

Any lesion occurring within or affecting the corticobulbar tract is known as an upper motor neuron lesion. Any lesion affecting the individual branches (temporal, zygomatic, buccal, mandibular and cervical) is known as a lower motor neuron lesion.

Branches of the facial nerve leaving the facial motor nucleus (FMN) for the muscles do so via both left and right posterior (dorsal) and anterior (ventral) routes. In other words, this means lower motor neurons of the facial nerve can leave either from the left anterior, left posterior, right anterior or right posterior facial motor nucleus. The temporal branch travels out from the left and right posterior components. The inferior four branches do so via the left and right anterior components. The left and right branches supply their respective sides of the face (ipsilateral innervation). Accordingly, the posterior components receive motor input from both hemispheres of the cerebral cortex (bilaterally), whereas the anterior components receive strictly contra-lateral input. This means that the temporal branch of the facial nerve receives motor input from both hemispheres of the cerebral cortex whereas the zygomatic, buccal, mandibular and cervical branches receive information from only contralateral hemispheres.

Now, because the anterior FMN receives only contralateral cortical input whereas the posterior receives that which is bilateral, a corticobulbar lesion (UMN lesion) occurring in the left hemisphere would eliminate motor input to the right anterior FMN component, thus removing signaling to the inferior four facial nerve branches, thereby paralyzing the right mid- and lower-face. The posterior component, however, although now only receiving input from the right hemisphere, is still able to allow the temporal branch to sufficiently innervate the entire forehead. This means that the forehead will not be paralyzed.

The same mechanism applies for an upper motor neuron lesion in the right hemisphere. The left anterior FMN component no longer receives cortical motor input due to its strict contralateral innervation, whereas the posterior component is still sufficiently supplied by the left hemisphere. The result is paralysis of the left mid- and lower-face with an unaffected forehead.

On the other hand, a lower motor neuron lesion is a bit different.

A lesion on either the left or right side would affect both the anterior and posterior routes on that side because of their close physical proximity to one another. So, a lesion on the left side would inhibit muscle innervation from both the left posterior and anterior routes, thus paralyzing the whole left side of the face (Bells Palsy). With this type of lesion, the bilateral and contalateral inputs of the posterior and anterior routes, respectively, become irrelevant because the lesion is below the level of the medulla and the facial motor nucleus. Whereas at a level above the medulla a lesion occurring in one hemisphere would mean that the other hemisphere could still sufficiently innervate the posterior facial motor nucleus, a lesion affecting a lower motor neuron would eliminate innervation altogether because the nerves no longer have a means to receive compensatory contralateral input at a downstream decussation.

1523
Q

Which one of the following serum proteins is most likely to increase in a patient with severe sepsis?

Transferrin

Transthyretin

Ferritin

Albumin

Cortisol binding protein

A

Ferritin can be markedly increased during acute illness. The other parameters tend to decrease during an acute phase response.

Acute phase proteins

Acute phase proteins
CRP
procalcitonin
ferritin
fibrinogen
alpha-1 antitrypsin
caeruloplasmin
serum amyloid A
haptoglobin
complement

During the acute phase response the liver decreases the production of other proteins (sometimes referred to as negative acute phase proteins). Examples include:
albumin
transthyretin (formerly known as prealbumin)
transferrin
retinol binding protein
cortisol binding protein

Levels of CRP are commonly measured in acutely unwell patients. CRP is a protein synthesised in the liver and binds to phosphocholine in bacterial cells and on those cells undergoing apoptosis. In binding to these cells it is then able to activate the complement system. CRP levels are known to rise in patients following surgery. However, levels of greater than 150 at 48 hours post operatively are suggestive of evolving complications.

1524
Q

Which of these tumour markers is most helpful in identifying an individual with hepatocellular carcinoma?

Serum AFP

Serum CA19-9

CEA

Beta HCG

CA125

A

Hepatocellular carcinoma is commonly diagnosed with imaging and an elevated alpha fetoprotein. Biopsy may seed the tumour and should be avoided. Up to 80% of hepatocellular carcinoma arise in cirrhotic livers.

Liver tumours

Primary liver tumours
The most common primary tumours are cholangiocarcinoma and hepatocellular carcinoma. Overall metastatic disease accounts for 95% of all liver malignancies making the primary liver tumours comparatively rare.

Primary liver tumours include:
Cholangiocarcinoma
Hepatocellular carcinoma
Hepatoblastoma
Sarcomas (Rare)
Lymphomas
Carcinoids (most often secondary although primary may occur)

Hepatocellular carcinoma
These account for the bulk of primary liver tumours (75% cases). Its worldwide incidence reflects its propensity to occur on a background of chronic inflammatory activity. Most cases arise in cirrhotic livers or those with chronic hepatitis B infection, especially where viral replication is actively occurring. In the UK it accounts for less than 5% of all cancers, although in parts of Asia its incidence is 100 per 100,000.
The majority of patients (80%) present with existing liver cirrhosis, with a mass discovered on screening ultrasound.

Diagnosis
CT/ MRI (usually both) are the imaging modalities of choice
a-fetoprotein is elevated in almost all cases
Biopsy should be avoided as it seeds tumours cells through a resection plane.
In cases of diagnostic doubt serial CT and αFP measurements are the preferred strategy.

Treatment
Patients should be staged with liver MRI and chest, abdomen and pelvic CT scan.
The testis should be examined in males (testicular tumours may cause raised AFP). PET CT may be used to identify occult nodal disease.
Surgical resection is the mainstay of treatment in operable cases. In patients with a small primary tumour in a cirrhotic liver whose primary disease process is controlled, consideration may be given to primary whole liver resection and transplantation.
Liver resections are an option but since most cases occur in an already diseased liver the operative risks and post-operative hepatic dysfunction are far greater than is seen following metastectomy.
These tumours are not particularly chemo or radiosensitive however, both may be used in a palliative setting. Tumour ablation is a more popular strategy.

Survival
Poor, overall survival is 15% at 5 years.

Cholangiocarcinoma
This is the second most common type of primary liver malignancy. As its name suggests these tumours arise in the bile ducts. Up to 80% of tumours arise in the extra hepatic biliary tree. Most patients present with jaundice and by this stage the majority will have disease that is not resectable.
Primary sclerosing cholangitis is the main risk factor. In deprived countries typhoid and liver flukes are also major risk factors.

Diagnosis
Patients will typically have an obstructive picture on liver function tests.
CA 19-9, CEA and CA 125 are often elevated
CT/ MRI and MRCP are the imaging methods of choice.

Treatment
Surgical resection offers the best chance of cure. Local invasion of peri hilar tumours is a particular problem and this coupled with lobar atrophy will often contra indicate surgical resection.
Palliation of jaundice is important, although metallic stents should be avoided in those considered for resection.

Survival
Is poor, approximately 5-10% 5 year survival.

1525
Q

Which of the following statements relating to omphalocele is false?

The herniated organs lie outside the peritoneal sac

Cardiac abnormalities co-exist in 25%

The caecum is usually right sided

The defect occurs through the umbilicus

Mortality may be as high as 15%

A

Gastroschisis: Isolated abnormality, bowel lies outside abdominal wall through defect located to right of umbilicus.
Exomphalos: Liver and gut remain covered with membranous sac connected to umbilical cord. It is associated with other developmental defects.
They are contained within the peritoneal sac and therefore do not have the fluid losses seen in gastroschisis. True malrotation is unusual and minor variants may not result in a requirement for surgery.

Paediatric Gastrointestinal disorders

Pyloric stenosis
M>F
5-10% Family history in parents
Projectile non bile stained vomiting at 4-6 weeks of life
Diagnosis is made by test feed or USS
Treatment: Ramstedt pyloromyotomy (open or laparoscopic)
Acute appendicitis
Uncommon under 3 years
When occurs may present atypically
Mesenteric adenitis
Central abdominal pain and URTI
Conservative management
Intussusception
Telescoping bowel
Proximal to or at the level of, ileocaecal valve
6-9 months age
Colicky pain, diarrhoea and vomiting, sausage shaped mass, red jelly stool.
Treatment: reduction with air insufflation
Malrotation
High caecum at the midline
Feature in exomphalos, congenital diaphragmatic hernia, intrinsic duodenal atresia
May be complicated by development of volvulus, infant with volvulus may have bile stained vomiting
Diagnosis is made by upper GI contrast study and USS
Treatment is by laparotomy, if volvulus is present (or at high risk of occurring then a ladds procedure is performed
Hirschsprung’s disease
Absence of ganglion cells from myenteric and submucosal plexuses
Occurs in 1/5000 births
Full thickness rectal biopsy for diagnosis
Delayed passage of meconium and abdominal distension
Treatment is with rectal washouts initially, thereafter an anorectal pull through procedure
Oesophageal atresia
Associated with tracheo-oesophageal fistula and polyhydramnios
May present with choking and cyanotic spells following aspiration
VACTERL associations
Meconium ileus
Usually delayed passage of meconium and abdominal distension
Majority have cystic fibrosis
X-Rays may not show a fluid level as the meconium is viscid (depends upon feeding), PR contrast studies may dislodge meconium plugs and be therapeutic
Infants who do not respond to PR contrast and NG N-acetyl cysteine will require surgery to remove the plugs
Biliary atresia
Jaundice > 14 days
Increased conjugated bilirubin
Urgent Kasai procedure
Necrotising enterocolitis
Prematurity is the main risk factor
Early features include abdominal distension and passage of bloody stools
X-Rays may show pneumatosis intestinalis and evidence of free air
Increased risk when empirical antibiotics are given to infants beyond 5 days
Treatment is with total gut rest and TPN, babies with perforations will require laparotomy

1526
Q

A 6 day old baby was born prematurely at 33 weeks. He has been suffering from respiratory distress syndrome and has been receiving ventilatory support on NICU. He has developed abdominal distension and is increasingly septic. Ultrasound of the abdomen shows free fluid and evidence of small bowel dilatation. His blood pressure has remained labile despite inotropic support. What is the best course of action?

Laparoscopy

Laparotomy

Contrast enema

Upper GI contrast study

MRI abdomen

A

He has necrotising enterocolitis and whilst this is often initially managed medically, a laparotomy is required if the situation deteriorates.

Paediatric Gastrointestinal disorders

Pyloric stenosis
M>F
5-10% Family history in parents
Projectile non bile stained vomiting at 4-6 weeks of life
Diagnosis is made by test feed or USS
Treatment: Ramstedt pyloromyotomy (open or laparoscopic)
Acute appendicitis
Uncommon under 3 years
When occurs may present atypically
Mesenteric adenitis
Central abdominal pain and URTI
Conservative management
Intussusception
Telescoping bowel
Proximal to or at the level of, ileocaecal valve
6-9 months age
Colicky pain, diarrhoea and vomiting, sausage shaped mass, red jelly stool.
Treatment: reduction with air insufflation
Malrotation
High caecum at the midline
Feature in exomphalos, congenital diaphragmatic hernia, intrinsic duodenal atresia
May be complicated by development of volvulus, infant with volvulus may have bile stained vomiting
Diagnosis is made by upper GI contrast study and USS
Treatment is by laparotomy, if volvulus is present (or at high risk of occurring then a ladds procedure is performed
Hirschsprung’s disease
Absence of ganglion cells from myenteric and submucosal plexuses
Occurs in 1/5000 births
Full thickness rectal biopsy for diagnosis
Delayed passage of meconium and abdominal distension
Treatment is with rectal washouts initially, thereafter an anorectal pull through procedure
Oesophageal atresia
Associated with tracheo-oesophageal fistula and polyhydramnios
May present with choking and cyanotic spells following aspiration
VACTERL associations
Meconium ileus
Usually delayed passage of meconium and abdominal distension
Majority have cystic fibrosis
X-Rays may not show a fluid level as the meconium is viscid (depends upon feeding), PR contrast studies may dislodge meconium plugs and be therapeutic
Infants who do not respond to PR contrast and NG N-acetyl cysteine will require surgery to remove the plugs
Biliary atresia
Jaundice > 14 days
Increased conjugated bilirubin
Urgent Kasai procedure
Necrotising enterocolitis
Prematurity is the main risk factor
Early features include abdominal distension and passage of bloody stools
X-Rays may show pneumatosis intestinalis and evidence of free air
Increased risk when empirical antibiotics are given to infants beyond 5 days
Treatment is with total gut rest and TPN, babies with perforations will require laparotomy

1527
Q

A 68 year old woman with previous rheumatic fever is admitted with pyrexia of unknown origin. Her blood cultures are unhelpful but transoesophageal echocardiography reveals vegetations on the mitral valve. Infection with which of the following organisms is most likely?

Staphylococcus aureus

Streptococcus pyogenes

Streptococcus viridans

Staphyloccus epidermidis

Staphylococcus saprophyticus

A

This is the most common organism affecting previously abnormal heart valves.

Surgical Microbiology

An extensive topic so an overview is given here. Organisms causing common surgical infections are reasonable topics in the examination. However, microbiology is less rigorously tested than anatomy, for example.

Common organisms

Staphylococcus aureus
Facultative anaerobe
Gram positive coccus
Haemolysis on blood agar plates
Catalase positive
20% population are long term carriers
Exo and entero toxin may result in toxic shock syndrome and gastroenteritis respectively
Ideally treated with penicillin although many strains now resistant through beta Lactamase production. In the UK less than 5% of isolates are sensitive to penicillin.
Resistance to methicillin (and other antibiotics) is mediated by the mec operon , essentially penicillin binding protein is altered and resistance to this class of antibiotics ensues
Common cause of cutaneous infections and abscesses

Streptococcus pyogenes
Gram positive, forms chain like colonies, Lancefield Group A Streptococcus
Produces beta haemolysis on blood agar plates
Rarely part of normal skin microflora
Catalase negative
Releases a number of proteins/ virulence factors into host including hyaluronidase, streptokinase which allow rapid tissue destruction
Releases superantigens such as pyogenic exotoxin A which results in scarlet fever
Remains sensitive to penicillin, macrolides may be used as an alternative.

Escherichia coli
Gram negative rod
Facultative anaerobe, non sporing
Wide range of subtypes and some are normal gut commensals
Some subtypes such as 0157 may produce lethal toxins resulting in haemolytic-uraemic syndrome
Enterotoxigenic E-Coli produces an enterotoxin (ST enterotoxin) that results in large volume fluid secretion into the gut lumen (Via cGMP activation)
Enteropathogenic E-Coli binds to intestinal cells and cause structural damage, this coupled with a moderate (or in case of enteroinvasive E-Coli significant) invasive component produces enteritis and large volume diarrhoea together with fever.
They are resistant to many antibiotics used to treat gram positive infections and acquire resistance rapidly and are recognised as producing beta lactamases

Campylobacter jejuni
Curved, gram negative, non sporulating bacteria
One of the commonest causes of diarrhoea worldwide
Produces enteritis which is often diffuse and blood may be passed
Remains a differential for right iliac fossa pain with diarrhoea
Self limiting infection so antibiotics are not usually advised. However, the quinolones are often rapidly effective.

Helicobacter pylori
Gram negative, helix shaped rod, microaerophillic
Produces hydrogenase that can derive energy from hydrogen released by intestinal bacteria
Flagellated and mobile
Those carrying the cag A gene may cause ulcers
It secretes urease that breaks down gastric urea> Carbon dioxide and ammonia> ammonium>bicarbonate (simplified!) The bicarbonate can neutralise the gastric acid.
Usually colonises the gastric antrum and irritates resulting in increased gastrin release and higher levels of gastric acid. These patients will develop duodenal ulcers. In those with more diffuse H-Pylori infection gastric acid levels are lower and ulcers develop by local tissue damage from H-Pylori- these patients get gastric ulcers.
Diagnosis may be made by serology (approx. 75% sensitive). Biopsy urease test during endoscopy probably the most sensitive.
In patients who are colonised 10-20% risk of peptic ulcer, 1-2% risk gastric cancer, <1% risk MALT lymphoma.

1528
Q

A 65 year old man presents, for the first time, with pain at the back of his calves when he mobilises 10 metres. He is known to have hypertension. What is the most appropriate investigation?

Arterial duplex scan

MRI angiogram

DSA angiogram

Aortic USS

Knee x-ray

A

An arterial duplex should be performed first, before progression to an angiography.

Vascular investigations

Venous disease

Venous Doppler
The simplest investigation for assessment of venous junctional incompetence is a Doppler assessment. This involves the patient standing and manual compression of the limb distal to the junction of interest. Flow should normally occur in one direction only. Where junctional incompetence is present reverse flow will occur and is relatively easy to identify.

Venograms and duplex scans
Structural venous information is historically obtained using a venogram. This is an invasive test and rarely required in modern clinical practice. The most helpful test is a venous duplex scan which will provide information relating to flow and vessel characteristics. Duplex is also useful in providing vein maps for bypass surgery.

Arterial disease

Ankle-brachial pressure
The ankle brachial pressure index measurement is an important investigation as it will allow classification of the severity of the flow compromise present. False readings may occur in those with calcified vessels such as diabetics and results in such settings should be interpreted with caution. When auscultating the vessel note should be made of the character of the signal. Monophasic signals are associated with a proximal stenosis and reduction in flow. Triphasic signals provide reassurance of a healthy vessel.

Arterial Duplex
As with the vein the duplex scan can provide a substantial amount of information about arterial patency and flow patterns. In skilled hands they can provide insight as to the state of proximal vessels that are anatomically inaccessible to duplex (e.g. Iliacs). Through assessment of distal flow patterns. It is an operator dependent test.

Conventional angiogram
Vessel puncture and catheter angiography is the gold standard method of assessing arteries. High quality information can usually be obtained. Limitations of the technique include the risk of contrast toxicity and risks of vessel damage. Severely calcified vessels may be difficult to puncture and in this situation a remote access site (e.g. brachial) may be used. This technique is particularly useful in providing a distal arterial roadmap prior to femoro-distal bypass.

CT angiography
These tests provide a considerable amount of structural and flow information. They require contrast and thus carry the risks associated with this. They are particularly useful in the setting of GI bleeding as they are rapidly available and can be performed by a non vascular radiologist. However, they lack the facility for endovascular intervention. In general they do not provide high enough resolution for distal arterial surgery.

Magnetic resonance angiography
This has the advantage of being non-invasive and not using nephrotoxic contrast. Movement artifact remains a problem in some sites and distal arterial resolution is imperfect.

1529
Q

A 30 year old man injures his ankle playing football. On examination, he has tenderness over both medial and lateral malleoli. X-ray demonstrates a bimalleolar fracture with a displaced distal fibula fracture, at the level of the syndesmosis and fracture of the medial malleolus with talar shift. The ankle has been provisionally reduced and splinted in the emergency department. What is the most appropriate management?

Application of external fixation device

Application of compression dressing and physiotherapy

Application of ankle boot

Surgical fixation

Below knee amputation

A

This is an unstable fracture pattern with a Weber B fracture of the distal fibula and a fracture of the medial malleolus. Talar shift indicates loss of ankle mortice congruity. This injury should therefore be treated with surgical fixation.

Ankle injuries

An ankle fracture relates to a fracture around the tibio-talar joint. It generally refers to a fracture involving the lateral, and/or medial and/or posterior malleolus. Pilon and Tillaux fractures are also considered to be ankle fractures, but are not covered here.
Ankle fractures are common. They effect men and women in equal numbers, but men have a higher rate as young adults (sports and contact injuries), and women a higher rate post-menopausal (fragility type fracture).

Osseous anatomy
The ankle (or mortise) joint consists of the distal tibia (tibial plafond and posterior malleolus), the distal fibula (lateral malleolus), and the talus. The main movement at the ankle joint is plantar and dorsiflexion.

Ligamentous anatomy
Medial side: Deltoid ligament. This is divided into superficial and deep portions. It is the primary restraint to valgus tilting of the talus.

Lateral side: Lateral ligament complex consisting from anterior to posterior of the anterior talofibular ligament (ATFL), calcaneofibular ligament (CFL), and the posterior talofibular ligament (PTFL). Together they resist valgus stress to the ankle, and are a restraint to anterior translation of the talus within the mortise joint.

Syndesmosis: The syndesmosis is a ligament complex between the distal tibia and fibula, holding the two bones together. It is fundamental to the integrity of the ankle joint, and its disruption leads to instability. It consists of (from anterior to posterior) the anterior-inferior tibiofibular ligament (AITFL), the transverse tibiofibular ligament (TTFL), the interosseous membrane, and the posterior-inferior tibiofibular ligament (PITFL).

Presentation and initial management
Patients will present following a traumatic event with a painful, swollen ankle, and reluctance/inability to weight bear. The Ottawa rules can be applied to differentiate between an ankle fracture and sprain, but can be unreliable.
In high energy injuries, management should follow ATLS principles to identify more significant injuries first. Neurovascular status of the foot should be documented, and open injuries should be excluded. If an open injury is identified, it should be managed in line with BOAST 4 principles1. If an obvious deformity exists, it should be reduced as soon as possible with appropriate analgesia or conscious sedation. Radiographs of clearly deformed or dislocated joints are not necessary, and removing the pressure on the surrounding soft tissues from the underlying bony deformity is the priority. If the fracture pattern is not clinically obvious then plain radiographs are appropriate and will guide the subsequent manipulation during plaster-of-paris below knee backslab application.

Imaging
AP, lateral and mortise views (20 degrees internal rotation) are essential to evaluate fracture displacement and syndesmotic injury. Decreased tibiofibular overlap, medial joint clear space and lateral talar shift all indicate a syndesmotic injury. (In subtle cases of shift, imaging the uninjured ankle can be helpful as a proportion of the population have little or no tibiotalar overlap-reference 2.)

Where there is suspicion of syndesmosis involvement in the absence of radiographic evidence, stress radiographs can be diagnostic.
Complex fracture patterns (and increasingly posterior malleolar fractures) are best defined using CT.

Classification
The most commonly used classifications are Lauge-Hansen and Danis-Weber.

Lauge-Hansen
Comprises two parts: first part is the foot position, and the second part is the force applied. Useful for understanding the forces involved and therefore predict the ligamentous or bony injury. Results in four injury patterns:
Supination - Adduction (SA) - 10-20%
Supination - External rotation (SER) - 40-75%
Pronation - Abduction (PA) - 5-20%
Pronation - External rotation (PER) - 5-20%

Not often used in clinical practice but good for understanding the principles of ankle fracture.

Danis-Weber
Commonly used. Based on the level of the fibula fracture in relation to the syndesmosis. The more proximal, the greater the risk of syndesmotic injury and therefore fracture instability.
A - fracture below the level of the syndesmosis
B - fracture at the level of the syndesmosis / level of the tibial plafond
C - fracture above the level of the syndesmosis. This includes Maisonneuve fractures (proximal fibula fracture), which can be associated with ankle instability. Beware the high fibula fracture - it may be an ankle fracture!

The Weber classification is based purely on the the lateral side. All injuries can include a medial or posterior bony or ligamentous injury which also dictates fracture stability (bimalleolar and trimalleolar fractures are more unstable).

Treatment
When deciding upon treatment for an ankle fracture, one must consider both the fracture and the patient. Diabetic patients and smokers are at greater risk of post-operative complication, especially wound problems and infection. Likewise, the long term outcome of post-traumatic arthritis from a malunited ankle fracture is extremely important for a young patient, but not as relevant in the elderly. Therefore, normal surgical decision processes apply as with all fractures.

Defining stability of an ankle fracture underpins the treatment decision.
Weber A - Unimalleolar Weber A Weber fractures by definition are stable and therefore can be mobilised fully weight bearing in an ankle boot.

Weber C - Fractures tend to include syndesmotic disruption and are usually bimalleolar (either bony or ligamentous). They are therefore unstable and usually require operative fixation. In addition to the fracture fixation, the syndesmosis usually requires reconstruction/augmentation with screws to restore the joint integrity and function.

Weber B - B fractures vary greatly. They can be part of a trimalleolar injury and therefore extremely unstable, requiring fixation. Alternatively, a uni-malleolar Weber B fracture can be a stable injury, and therefore mobilised immediately in an ankle boot. Defining the stability can be challenging, and often involves stress radiographs, or a trial of mobilisation and repeat radiographs. However, treating undisplaced ankle fractures in a below knee plaster, non-weight bearing for six weeks is still widely practised, and a safe approach.

When operative fixation is appropriate, it is usually via open reduction and internal fixation using plates and screws. It must be carried out when soft tissue swelling has settled in order to minimise the risk of wound problems. This can often take a week to settle.

The use of fibula nails is expanding, but is not yet mainstream. Ankle fractures can also be treated with external fixation, or with a hind foot nail in patients who need fixation but where soft tissue or bone quality is poor.

Post operative management
Ankle fractures generally take 6 weeks to unite enough to prevent secondary displacement. This is therefore an appropriate time period to keep a cast on in a conservatively managed patient. Weight bearing post-operatively depends on the quality of the fixation and bone quality, and preference varies between surgeons, ranging from aggressive early mobilisation to a period of non-weight bearing. Return to activities takes approximately three months, and often requires assistance of a physiotherapist to improve range-of-movement and muscle strengthening.

References
1. http://www.boa.ac.uk/publications/boa-standards-trauma-boasts/
2. Shah AS, Kadakia AR, Tan GJ, Karadsheh MS, Wolter TD, Sabb B. Radiographic evaluation of the normal distal tibiofibular syndesmosis. Foot Ankle Int. 2012;33(10):870-6

1530
Q

A 23 year old lady is admitted with right upper quadrant pain. On examination, she is tender in the right upper quadrant. Imaging shows signs of acute cholecystitis due to gallstones. The CBD appears normal. Liver function tests are normal. What is the most appropriate course of action?

Laparoscopic cholecystectomy during the next 24-48 hours

Open cholecystectomy during the next 24-48 hours

Laparoscopic cholecystectomy 3 months following resolution of the attack

Open cholecystectomy 3 months following the attack

Laparoscopic cholecystectomy after 5 days of intravenous antibiotics

A

In most cases the treatment of choice for acute cholecystitis is an acute cholecystectomy performed early in the illness. Delayed surgery particularly around 5- 7 days after presentation is much more technically challenging and is often best deferred. In most cases the procedure can be performed laparoscopically, even when acute inflammation is present.

Gallstones

Up to 24% of women and 12% of men may have gallstones. Of these up to 30% may develop local infection and cholecystitis. In patients subjected to surgery 12% will have stones contained within the common bile duct. The majority of gallstones are of a mixed composition (50%) with pure cholesterol stones accounting for 20% of cases.
The aetiology of CBD stones differs in the world, in the West most CBD stones are the result of migration. In the East a far higher proportion arise in the CBD de novo.
The classical symptoms are of colicky right upper quadrant pain that occurs post prandially. The symptoms are usually worst following a fatty meal when cholecystokinin levels are highest and gallbladder contraction is maximal.

Investigation
In almost all suspected cases the standard diagnostic work up consists of abdominal ultrasound and liver function tests. Of patients who have stones within the bile duct, 60% will have at least one abnormal result on LFT’s. Ultrasound is an important test, but is operator dependent and therefore may occasionally need to be repeated if a negative result is at odds with the clinical picture. Where stones are suspected in the bile duct, the options lie between magnetic resonance cholangiography and intraoperative imaging. The choice between these two options is determined by the skills and experience of the surgeon. The advantages of intra operative imaging are less useful in making therapeutic decisions if the operator is unhappy about proceeding the bile duct exploration, and in such circumstances pre operative MRCP is probably a better option.

Specific gallstone and gallbladder related disease
Disease Features Management
Biliary colic Colicky abdominal pain, worse post prandially, worse after fatty foods If imaging shows gallstones and history compatible then laparoscopic cholecystectomy
Acute cholecystitis
Right upper quadrant pain
Fever
Murphys sign on examination
Occasionally mildly deranged LFT’s (especially if Mirizzi syndrome)
Imaging (USS) and cholecystectomy (ideally within 48 hours of presentation) (2)
Gallbladder abscess
Usually prodromal illness and right upper quadrant pain
Swinging pyrexia
Patient may be systemically unwell
Generalised peritonism not present
Imaging with USS +/- CT Scanning
Ideally surgery, sub total cholecystectomy may be needed if Calots triangle is hostile
In unfit patients percutaneous drainage may be considered
Cholangitis
Patient severely septic and unwell
Jaundice
Right upper quadrant pain
Fluid resuscitation
Broad spectrum intravenous antibiotics
Correct any coagulopathy
Early ERCP
Gallstone ileus
Patients may have a history of previous cholecystitis and known gallstones
Small bowel obstruction (may be intermittent)
Laparotomy and removal of gallstone from small bowel, the enterotomy must be made proximal to the site of obstruction and not at the site of obstruction. The fistula between the gallbladder and duodenum should not be interfered with.
Acalculous cholecystitis
Patients with inter current illness (e.g. diabetes, organ failure)
Patient of systemically unwell
Gallbladder inflammation in absence of stones
High fever
If patient fit then cholecystectomy, if unfit then percutaneous cholecystostomy

Treatment
Patients with asymptomatic gallstones rarely develop symptoms related to them (less than 2% per year) and may therefore be managed expectantly. In almost all cases of symptomatic gallstones the treatment of choice is cholecystectomy performed via the laparoscopic route. In the very frail patient there is sometimes a role for selective use of ultrasound guided cholecystostomy.

During the course of the procedure some surgeons will routinely perform either intra operative cholangiography or laparoscopic USS to either confirm anatomy or to exclude CBD stones. The latter may be more easily achieved by use of laparoscopic ultrasound. If stones are found then the options lie between early ERCP in the day or so following surgery or immediate surgical exploration of the bile duct. When performed via the trans cystic route this adds little in the way of morbidity and certainly results in faster recovery. Where transcystic exploration fails the alternative strategy is that of formal choledochotomy. The exploration of a small duct is challenging and ducts of less than 8mm should not be explored. Small stones that measure less than 5mm may be safely left and most will pass spontaneously.

Risks of ERCP(1)
Bleeding 0.9% (rises to 1.5% if sphincterotomy performed)
Duodenal perforation 0.4%
Cholangitis 1.1%
Pancreatitis 1.5%

References
1. Williams E et al. Guidelines on the management of common bile duct stones (CBDS)Gut 2008;57:10041021

  1. Gurusamy KS, Samraj K. Early versus delayed laparoscopic cholecystectomy for acute cholecystitis. Cochrane Database Syst Rev. 2006 Oct 18;(4):CD005440.
  2. Gurusamy K and Davidson B. Gallstones. BMJ 2014 (348):27-30.
1531
Q

A patient presents to the clinic following a surgical procedure. She complains that she is unable to shrug her shoulder. What is the most likely underlying nerve injury?

Accessory nerve

Cervical plexus

Ansa cervicalis

Long thoracic nerve

Axillary nerve

A

The accessory nerve may be injured in operations in the posterior triangle. Injury will affect trapezius.

Cranial nerves

Cranial nerve lesions
Olfactory nerve May be injured in basal skull fractures or involved in frontal lobe tumour extension. Loss of olfactory nerve function in relation to major CNS pathology is seldom an isolated event and thus it is poor localiser of CNS pathology.
Optic nerve Problems with visual acuity may result from intra ocular disorders. Problems with the blood supply such as amaurosis fugax may produce temporary visual distortion. More important surgically is the pupillary response to light. The pupillary size may be altered in a number of disorders. Nerves involved in the resizing of the pupil connect to the pretectal nucleus of the high midbrain, bypassing the lateral geniculate nucleus and the primary visual cortex. From the pretectal nucleus neurones pass to the Edinger - Westphal nucleus, motor axons from here pass along with the oculomotor nerve. They synapse with ciliary ganglion neurones; the parasympathetic axons from this then innervate the iris and produce miosis. The miotic pupil is seen in disorders such as Horner’s syndrome or opiate overdose.
Mydriasis is the dilatation of the pupil in response to disease, trauma, drugs (or the dark!). It is pathological when light fails to induce miosis. The radial muscle is innervated by the sympathetic nervous system. Because the parasympathetic fibres travel with the oculomotor nerve they will be damaged by lesions affecting this nerve (e.g. cranial trauma).
The response to light shone in one eye is usually a constriction of both pupils. This indicates intact direct and consensual light reflexes. When the optic nerve has an afferent defect the light shining on the affected eye will produce a diminished pupillary response in both eyes. Whereas light shone on the unaffected eye will produce a normal pupillary response in both eyes. This is referred to as the Marcus Gunn pupil and is seen in conditions such as optic neuritis. In a total CN II lesion shining the light in the affected eye will produce no response.
Oculomotor nerve The pupillary effects are described above. In addition it supplies all ocular muscles apart from lateral rectus and superior oblique. Thus the affected eye will be deviated inferolaterally. Levator palpebrae superioris may also be impaired resulting in impaired ability to open the eye.
Trochlear nerve The eye will not be able to look down.
Trigeminal nerve Largest cranial nerve. Exits the brainstem at the pons. Branches are ophthalmic, maxillary and mandibular. Only the mandibular branch has both sensory and motor fibres. Branches converge to form the trigeminal ganglion (located in Meckels cave). It supplies the muscles of mastication and also tensor veli palatine, mylohyoid, anterior belly of digastric and tensor tympani. The detailed descriptions of the various sensory functions are described in other areas of the website. The corneal reflex is important and is elicited by applying a small tip of cotton wool to the cornea, a reflex blink should occur if it is intact. It is mediated by: the naso ciliary branch of the ophthalmic branch of the trigeminal (sensory component) and the facial nerve producing the motor response. Lesions of the afferent arc will produce bilateral absent blink and lesions of the efferent arc will result in a unilateral absent blink.
Abducens nerve The affected eye will have a deficit of abduction. This cranial nerve exits the brainstem between the pons and medulla. It thus has a relatively long intra cranial course which renders it susceptible to damage in raised intra cranial pressure.
Facial nerve Emerges from brainstem between pons and medulla. It controls muscles of facial expression and taste from the anterior 2/3 of the tongue. The nerve passes into the petrous temporal bone and into the internal auditory meatus. It then passes through the facial canal and exits at the stylomastoid foramen. It passes through the parotid gland and divides at this point. It does not innervate the parotid gland. Its divisions are considered in other parts of the website. Its motor fibres innervate orbicularis oculi to produce the efferent arm of the corneal reflex. In surgical practice it may be injured during parotid gland surgery or invaded by malignancies of the gland and a lower motor neurone on the ipsilateral side will result.
Vestibulo-cochlear nerve Exits from the pons and then passes through the internal auditory meatus. It is implicated in sensorineural hearing loss. Individuals with sensorineural hearing loss will localise the sound in webers test to the normal ear. Rinnes test will be reduced on the affected side but should still work. These two tests will distinguish sensorineural hearing loss from conductive deafness. In the latter condition webers test will localise to the affected ear and Rinnes test will be impaired on the affected side. Surgical lesions affecting this nerve include CNS tumours and basal skull fractures. It may also be damaged by the administration of ototoxic drugs (of which gentamicin is the most commonly used in surgical practice).
Glossopharyngeal nerve Exits the pons just above the vagus. Receives sensory fibres from posterior 1/3 tongue, tonsils, pharynx and middle ear (otalgia may occur following tonsillectomy). It receives visceral afferents from the carotid bodies. It supplies parasympathetic fibres to the parotid gland via the otic ganglion and motor function to stylopharyngeaus muscle. The sensory function of the nerve is tested using the gag reflex.
Vagus nerve Leaves the medulla between the olivary nucleus and the inferior cerebellar peduncle. Passes through the jugular foramen and into the carotid sheath. Details of the functions of the vagus nerve are covered in the website under relevant organ sub headings.
Accessory nerve Exists from the caudal aspect of the brainstem (multiple branches) supplies trapezius and sternocleidomastoid muscles. The distal portion of this nerve is most prone to injury during surgical procedures.
Hypoglossal nerve Emerges from the medulla at the preolivary sulcus, passes through the hypoglossal canal. It lies on the carotid sheath and passes deep to the posterior belly of digastric to supply muscles of the tongue (except palatoglossus). Its location near the carotid sheath makes it vulnerable during carotid endarterectomy surgery and damage will produce ipsilateral defect in muscle function.

1532
Q

A 56 year old lady with metastatic breast cancer develops an osteolytic deposit in the proximal femur. One morning whilst getting out of bed she notices severe groin pain. X-rays show that the lesser trochanter has been avulsed. Which muscle is the most likely culprit?

Vastus lateralis

Psoas major

Piriformis

Gluteus maximus

Gluteus medius

A

The psoas major inserts into the lesser trochanter and contracts when raising the trunk from the supine position. When osteolytic lesions are present in the femur the lesser trochanter may be avulsed.

Psoas Muscle

Origin
The deep part originates from the transverse processes of the five lumbar vertebrae, the superficial part originates from T12 and the first 4 lumbar vertebrae.

Insertion
Lesser trochanter of the femur.

Innervation
Anterior rami of L1 to L3.

Action
Flexion and external rotation of the hip. Bilateral contraction can raise the trunk from the supine position.

1533
Q

Beta-naphthalamine is associated with which of the following malignancies?

Lung cancer

Bowel cancer

Bladder cancer

Liver cancer

Renal cancer

A

Beta-naphthalamine is used in the rubber industry.

The following factors are associated with the development of bladder cancer:
smoking
occupational: aniline dyes used in printing and textile industry, rubber manufacture
schistosomiasis
drugs: cyclophosphamide

Occupational cancers

Occupational cancers accounted for 5.3% cancer deaths in 2005.
In men the main cancers include:
Mesothelioma
Bladder cancer
Non melanoma skin cancer
Lung cancer
Sino nasal cancer

Occupations with high levels of occupational tumours include:
Construction industry
Working with coal tar and pitch
Mining
Metalworkers
Working with asbestos (accounts for 98% of all mesotheliomas)
Working in rubber industry

Shift work has been linked to breast cancer in women (Health and safety executive report RR595).

The latency between exposure and disease is typically 15 years for solid tumours and 20 for leukaemia.

Many occupational cancers are otherwise rare. For example sino nasal cancer is an uncommon tumour, 50% will be SCC. They are linked to conditions such as wood dust exposure and unlike lung cancer is not strongly linked to cigarette smoking. Another typical occupational tumour is angiosarcoma of the liver which is linked to working with vinyl chloride. Again in the non occupational context this is an extremely rare sporadic tumour.

1534
Q

A 32 year old rugby player is crushed in a scrum. He is briefly concussed but then regains consciousness. He then collapses and is brought to ED. His GCS on arrival is 6/15 and his left pupil is dilated. What is the best course of definitive management?

Burr Hole decompression

Parietotemporal craniotomy

Intravenous mannitol

Posterior fossa craniotomy

Insertion of intra cranial pressure bolt monitor

A

This man needs urgent decompression and extradural haematoma is the most likely event, from a lacerated middle meningeal artery. Most neurosurgeons would perform a craniotomy. However, rural units and those units without neurosurgical kit facing this emergency may resort to Burr Holes.

Head injury management- NICE Guidelines

Summary of guidelines
All patients should be assessed within 15 minutes on arrival to A&E
Document all 3 components of the GCS
If GCS <8 or = to 8, consider stabilising the airway
Full spine immobilisation until assessment if:

  • GCS < 15
  • neck pain/tenderness
  • paraesthesia extremities
  • focal neurological deficit
  • suspected c-spine injury

If a c-spine injury is suspected a 3 view c-spine x-ray is indicated. CT c-spine is preferred if:
- Intubated
- GCS <13
- Normal x-ray but continued concerns regarding c-spine injury
- Any focal neurology
- A CT head scan is being performed
- Initial plain films are abnormal

Immediate CT head (within 1 hour) if:
GCS < 12 on admission
GCS < 15 2 hours after admission
Suspected open or depressed skull fracture
Suspected skull base fracture (panda eyes, Battle’s sign, CSF from nose/ear, bleeding ear)
Focal neurology
Vomiting > 1 episode
Post traumatic seizure

Contact neurosurgeon if:
Persistent GCS < 8 or = 8
Unexplained confusion > 4h
Reduced GCS after admission
Progressive neurological signs
Incomplete recovery post seizure
Penetrating injury
Cerebrospinal fluid leak

Observations
1/2 hourly GCS until 15

In the latest guidance, there is a move to considering CT in those on anticoagulants rather than mandating it.

Reference
1. NICE Guidance NG 232 (Published 2023).
2. Hodgkinson S et al. Early management of head injury: summary of NICE guidance. BMJ 2014 (348):34-37.

1535
Q

A 75 year old man undergoes a Hartman’s procedure for sigmoid diverticular disease with pericolic abscess and colovesical fistula. What type of drain should be inserted following this procedure?

T Tube drain

Wallace Robinson drain (non suction)

Redivac drain

Corrugated drain

No drain

A

A non suction drain is the preferred option here.

Surgical drains

Drains are inserted in many surgical procedures and are of many types.
As a broad rule they can be divided into those using suction and those which do not.
The diameter of the drain will depend upon the substance being drained, for example smaller lumen drain for pneumothoraces vs haemothorax.
Drains can be associated with complications and these begin with insertion when there may be iatrogenic damage. When in situ they serve as a route for infections. In some specific situations they may cause other complications, for example suction drains left in contact with bowel for long periods may carry a risk of inducing fistulation.
Drains should be inserted for a defined purpose and removed once the need has passed.

A brief overview of types of drain and sites is given below

CNS
Low suction drain or free drainage systems may be used for situations such as drainage of sub dural haematomas.

CVS
Following cardiothoracic procedures of thoracic trauma underwater seal drains are often placed. These should be carefully secured. When an air leak is present they may be placed on suction whilst the air leak settles

Orthopaedics and trauma
In this setting drains are usually used to prevent haematoma formation (with associated risk of infection). Some orthopaedic drains may also be specially adapted to allow the drained blood to be auto transfused.

Gastro-intestinal surgery
Surgeons often place abdominal drains either to prevent or drain abscesses, or to turn an anticipated complication into one that can be easily controlled such as a bile leak following cholecystectomy. The type of drain used will depend upon the indication.

Drain types
Type of drain Features
Redivac
Suction type of drain
Closed drainage system
High pressure vacuum system
Low pressure drainage systems
Consist of small systems such as the lantern style drain that may be used for short term drainage of small wounds and cavities
Larger systems are sometimes used following abdominal surgery, they have a lower pressure than the redivac system, which decreases the risks of fistulation
May be emptied and re-pressurised
Latex tube drains
May be shaped (e.g. T Tube) or straight
Usually used in non pressurised systems and act as sump drains
Most often used when it is desirable to generate fibrosis along the drain track (e.g. following exploration of the CBD)
Chest drains
May be large or small diameter (depending on the indication)
Connected to underwater seal system to ensure one way flow of air
Corrugated drain
Thin, wide sheet of plastic, usually soft
Contains corrugations, along which fluids can track

1536
Q

A 45-year-old man presents to surgical outpatients with a long history of recurrent abdominal pain and vomiting. He is noted to have a peripheral motor neuropathy on examination. What is the most likely diagnosis?

Huntington’s disease

Myeloma

Acute intermittent porphyria

Lawrence-Moon-Biedl syndrome

Friedreich’s ataxia

A

Neurological signs combined with abdominal pain is acute intermittent porphyria or lead poisoning until proven otherwise.

Acute intermittent porphyria

Acute intermittent porphyria (AIP) is a rare autosomal dominant condition caused by a defect in porphobilinogen deaminase, an enzyme involved in the biosynthesis of haem. The results in the toxic accumulation of delta aminolaevulinic acid and porphobilinogen. It characteristically presents with abdominal and neuropsychiatric symptoms in 20-40 year olds. AIP is more common in females (5:1)

Features
abdominal: abdominal pain, vomiting
neurological: motor neuropathy
psychiatric: e.g. depression
hypertension and tachycardia common

Diagnosis
classically urine turns deep red on standing
raised urinary porphobilinogen (elevated between attacks and to a greater extent during acute attacks)
assay of red cells for porphobilinogen deaminase
raised serum levels of delta aminolaevulinic acid and porphobilinogen

1537
Q

A 28 year old lady presents with benign cyclical mastalgia. Which of the following is not a recognised treatment for the condition?

Evening primrose oil

Bromocriptine

Methotrexate

Danazol

Tamoxifen

A

Surgical excision of tender breast tissue is inappropriate
Methotrexate is used for the treatment of breast cancer. Whilst the use of tamoxifen is of benefit other agents such as flaxseed oil or evening primrose oil should be tried first. Danazol is effective, but many women dislike the side effects.

Benign cyclical mastalgia

Benign cyclical mastalgia is a common cause of breast pain in younger females. It varies in intensity according to the phase of the menstrual cycle. It is not associated with point tenderness of the chest wall (more likely to be Tietze’s syndrome).

The underlying cause is difficult to pinpoint, examination should focus on identifying focal lesions (such as cysts) that may be treated to provide symptomatic benefit. Women should be advised to wear a supportive bra. Conservative treatments include flax seed oil and evening primrose oil. There is slightly more evidence in favor of flax seed oil, though neither has performed much better than placebo in RCT’s.

Hormonal agents such as bromocriptine and danazol may be more effective. However, many women discontinue these therapies due to adverse effects.

1538
Q

A 49 year old man undergoes a low anterior resection for cancer. He is assessed in the outpatient clinic post operatively. His wounds are well healed. However, he complains of impotence. Which of the following best explains this problem?

Sciatic nerve injury

Damage to the internal iliac artery

Damage to the nervi erigentes

Damage to the vas

Damage to the genitofemoral nerve

A

The penis takes autonomic nerves from the nervi erigentes that lie near the seminal vesicles. These may be compromised by direct surgical trauma (such as use of diathermy in this area) and also by radiotherapy that is used in these patients pre operatively. The result is that up to 50% of patients may develop impotence following rectal cancer surgery.

Penile erection

Physiology of erection
Autonomic
Sympathetic nerves originate from T11-L2 and parasympathetic nerves from S2-4 join to form pelvic plexus.
Parasympathetic discharge causes erection, sympathetic discharge causes ejaculation and detumescence.
Somatic nerves Supplied by dorsal penile and pudendal nerves. Efferent signals are relayed from Onufs nucleus (S2-4) to innervate ischiocavernosus and bulbocavernosus muscles.

Autonomic discharge to the penis will trigger the veno-occlusive mechanism which triggers the flow of arterial blood into the penile sinusoidal spaces. As the inflow increases the increased volume in this space will secondarily lead to compression of the subtunical venous plexus with reduced venous return. During the detumesence phase the arteriolar constriction will reduce arterial inflow and thereby allow venous return to normalise.

Priapism
Prolonged unwanted erection, in the absence of sexual desire, lasting more than 4 hours.

Classification of priapism
Low flow priapism Due to veno-occlusion (high intracavernosal pressures).
Most common type
Often painful
Often low cavernosal flow
If present for >4 hours requires emergency treatment
High flow priapism Due to unregulated arterial blood flow.
Usually presents as semi rigid painless erection
Recurrent priapism Typically seen in sickle cell disease, most commonly of high flow type.

Causes
Intracavernosal drug therapies (e.g. for erectile dysfunction>
Blood disorders such as leukaemia and sickle cell disease
Neurogenic disorders such as spinal cord transection
Trauma to penis resulting in arterio-venous malformations

Tests
Exclude sickle cell/ leukaemia
Consider blood sampling from cavernosa to determine whether high or low flow (low flow is often hypoxic)

Management
Ice packs/ cold showers
If due to low flow then blood may be aspirated from copora or try intracavernosal alpha adrenergic agonists.
Delayed therapy of low flow priapism may result in erectile dysfunction.

1539
Q

Which receptor does noradrenaline mainly bind to?

α 1 receptors

α 2 receptors

β 1 receptors

β 2 receptors

G receptors

A

Noradrenaline is the precursor of adrenaline. It is a powerful α 1 stimulant (although it will increase myocardial contractility). Infusions will produce vasoconstriction and an increase in total peripheral resistance. It is the inotrope of choice in septic shock.

Inotropes and cardiovascular receptors

Inotropes are a class of drugs which work primarily by increasing cardiac output. They should be distinguished from vasoconstrictor drugs which are used specifically when the primary problem is peripheral vasodilatation.

Catecholamine type agents are commonly used and work by increasing cAMP levels by adenylate cyclase stimulation. This in turn increases intracellular calcium ion mobilisation and thus the force of contraction. Adrenaline works as a beta adrenergic receptor agonist at lower doses and an alpha receptor agonist at higher doses. Dopamine causes dopamine receptor mediated renal and mesenteric vascular dilatation and beta 1 receptor agonism at higher doses. This results in increased cardiac output. Since both heart rate and blood pressure are raised, there is less overall myocardial ischaemia. Dobutamine is a predominantly beta 1 receptor agonist with weak beta 2 and alpha receptor agonist properties. Noradrenaline is a catecholamine type agent and predominantly acts as an alpha receptor agonist and serves as a peripheral vasoconstrictor.

Phosphodiesterase inhibitors such as milrinone act specifically on the cardiac phosphodiesterase and increase cardiac output.

Inotrope Cardiovascular receptor action
Adrenaline α-1, α-2, β-1, β-2
Noradrenaline α-1,( α-2), (β-1), (β-2)
Dobutamine β-1, (β 2)
Dopamine (α-1), (α-2), (β-1), D-1,D-2
Minor receptor effects in brackets

Effects of receptor binding
α-1, α-2 vasoconstriction
β-1 increased cardiac contractility and HR
β-2 vasodilatation
D-1 renal and spleen vasodilatation
D-2 inhibits release of noradrenaline

1540
Q

Which of the energy devices below would be most suited for resection of a parietal lobe tumour?

Monopolar diathermy in coagulation mode

Harmonic ACE

CUSA device

Monopolar diathermy in cutting mode

Ligasure system

A

CUSA systems minimize thermal injury and current flow to the surrounding areas.

Diathermy

  • Diathermy devices are used by surgeons in all branches of surgery.
    Use electric currents to produce local heat and thereby facilitate haemostasis or surgical dissection.
    Consist of a generator unit that is located outside the patient and can be set to the level of power required by the surgeon.
    There are two major types of diathermy machine;

Monopolar
The current flows through the diathermy unit into a handheld device that is controlled by the surgeon. Electricity can flow from the tip of the device into the patient. The earth electrode is located some distance away. The relatively narrow tip of the diathermy device produces local heat and this can be used to vaporise and fulgurate tissues. The current can be adjusted in terms of frequency so that different actions can be effected. In cutting mode sufficient power is applied to the tissues to vaporise their water content. In coagulation mode the power level is reduced so that a coagulum is formed instead. Some diathermy machines can utilise a setting known as blend that alternates cutting and coagulation functions, these tend to be used during procedures such as colonoscopic polypectomy.

Bipolar
The electric current flows from one electrode to another however, both electrodes are usually contained within the same device e.g. a pair of forceps. The result is that heating is localised to the area between the two electrodes and surrounding tissue damage is minimised.

Ultrasound based devices
These include CUSA and Harmonic scalpel. They generate high frequency oscillations that seal and coagulate tissues. They have different energy settings that allow them to dissect and simultaneously seal vessels if required. The CUSA device leaves vessels intact that may then be divided.

Ligasure device
Delivers tailored energy levels to allow simultaneous haemostasis and dissection. The device senses the impedance of the tissues and tailors energy levels accordingly.

Hazards of diathermy
Inadvertent patient burn. This may result of careless handling of the device or in the case of monopolar devices forgetting to apply a return electrode plate, In this situation patients may develop a contact burn when electricity flows to earth
Explosion or fire. This may occur when volatile anaesthetic gases or skin preparation fluid have been used

1541
Q

A 52 year old woman presents with an acutely ischaemic right arm. She is found to have fast atrial fibrillation. Her blood results reveal a free T4 level of 20 and a TSH of < 0.01. What is the most likely diagnosis?

Poor compliance with thyroid medication

Thyrotoxicosis

Multinodular goitre

Sick euthyroid syndrome

Hashimotos thyroiditis

A

The diagnosis is thyrotoxicosis. An elevated T4 and a low TSH should indicate this diagnosis. Remember atrial fibrillation and its complications i.e acute ischaemic limbs can be precipitated by hyperthyroid disorders.

Thyroid function tests

The interpretation of thyroid function tests is usually straightforward:

Disorder TSH Free T4
Thyrotoxicosis (e.g. Graves’ disease) Low High In T3 thyrotoxicosis the free T4 will be normal
Primary hypothyroidism (primary atrophic hypothyroidism) High Low
Secondary hypothyroidism Low Low Replacement steroid therapy is required prior to thyroxine
Sick euthyroid syndrome* Low** Low Common in hospital inpatients
Poor compliance with thyroxine High Normal / high
Steroid therapy Low Normal

*now referred to as non-thyroidal illness

**TSH may be normal in some cases

1542
Q

A 17 year old man is referred to the urology clinic. As a child he was diagnosed as having a right sided PUJ obstruction. However, he was lost to follow up. Over the past 7 months he has been complaining of recurrent episodes of right loin pain. A CT scan shows considerable renal scarring. What is the most useful investigation?

DMSA scan

MAG 3 renogram

CT scan of the kidney

CT KUB

Renal USS

A

In patients with long standing PUJ obstruction and renal scarring the main diagnostic question is whether the individual has sufficient renal function to consider a pyeloplasty or whether a primary nephrectomy is preferable. Since the CT has demonstrated scarring there is no use in obtaining a DMSA scan. Of the investigations listed both a DMSA and MAG 3 renogram will allow assessment of renal function. However, MAG 3 is superior in the assessment of renal function in damaged kidneys (as it is subjected to tubular secretion).

Functional renal imaging

DMSA scan
Dimercaptosuccinic acid (DMSA) scintigraphy
DMSA localises to the renal cortex with little accumulation in the renal papilla and medulla. It is useful for the identification of cortical defects and ectopic or aberrant kidneys. It does not provide useful information on the ureter of collecting system.

Diethylene-triamine-penta-acetic acid (DTPA)
This is primarily a glomerular filtration agent. It is most useful for the assessment of renal function. Because it is filtered at the level of the glomerulus it provides useful information about the GFR. Image quality may be degraded in patients with chronic renal impairment and derangement of GFR.

MAG 3 renogram
Mercaptoacetyle triglycine is an is extensively protein bound and is primarily secreted by tubular cells rather than filtered at the glomerulus. This makes it the agent of choice for imaging the kidneys of patients with existing renal impairment (where GFR is impaired).

Micturating cystourethrogram (MCUG scan)
This scan provides information relating to bladder reflux and is obtained by filling the bladder with contrast media (via a catheter) and asking the child to void. Images are taken during this phase and the degree of reflux can be calculated

Intra venous urography
This examination is conducted by the administration of intravenous iodinated contrast media. The agent is filtered by the kidneys and excreted and may provide evidence of renal stones or other structural lesions. A rough approximation of renal function may be obtained using the technique. But it is not primarily a technique to be used for this purpose. With the advent of widespread non contrast CT scan protocols for the detection of urinary tract calculi it is now rarely used.

PET/CT
This may be used to evaluate structurally indeterminate lesions in the staging of malignancy.

References
Davis A et al. Investigating urinary tract infections in children. BMJ 2013 (346):35-37.

1543
Q

What is the most important structure involved in supporting the uterus?

Round ligament

Broad ligament

Uterosacral ligaments

Cardinal ligaments

Central perineal tendon

A

The central perineal tendon provides the main structural support to the uterus. Damage to this structure is commonly associated with the development of pelvic organ prolapse, even when other structures are intact.

Uterus

The non pregnant uterus resides entirely within the pelvis. The peritoneum invests the uterus and the structure is contained within the peritoneal cavity. The blood supply to the uterine body is via the uterine artery (branch of the internal iliac). The uterine artery passes from the inferior aspect of the uterus (lateral to the cervix) and runs alongside the uterus. It frequently anastomoses with the ovarian artery superiorly. Inferolaterally, the ureter is a close relation and ureteric injuries are a recognised complication when pathology brings these structures into close proximity.

The supports of the uterus include the central perineal tendon (the most important). The lateral cervical, round and uterosacral ligaments are condensations of the endopelvic fascia and provide additional structural support.

1544
Q

Which of the following changes are not typically seen in established dehydration?

Rising haematocrit

Urinary sodium <20mmol/ litre

Metabolic acidosis

Decreased serum urea to creatinine ratio

Hypernatraemia

A

Diagnosing dehydration can be complicated, laboratory features include:
Hypernatraemia
Rising haematocrit
Metabolic acidosis
Rising lactate
Increased serum urea to creatinine ratio
Urinary sodium <20 mmol/litre
Urine osmolality approaching 1200mosmol/kg

Pre operative fluid management

Fluid management has been described in the British Consensus guidelines on IV fluid therapy for Adult Surgical patients (GIFTASUP) and by NICE (CG174 December 2013 updated May 2017)

The Recommendations include:
Use Ringer’s lactate or Hartmann’s when a crystalloid is needed for resuscitation or replacement of fluids. Avoid 0.9% N. Saline (due to risk of hyperchloraemic acidosis) unless patient vomiting or has gastric drainage.
Use 4%/0.18% dextrose saline or 5% dextrose in maintenance fluids. It should not be used in resuscitation or as replacement fluids.
Adult maintenance fluid requirements are: Na 50-100 mmol/day and K 40-80 mmol/day in 1.5-2.5L fluid per day.
Patients for elective surgery should NOT be nil by mouth for >2 hours (unless has disorder of gastric emptying).
Patients for elective surgery should be given carbohydrate rich drinks 2-3h before. Ideally this should form part of a normal pre op plan to facilitate recovery.
Avoid mechanical bowel preparation.
If bowel prep is used, simultaneous administration of Hartmann’s or Ringer’s lactate should be considered.
Excessive fluid losses from vomiting should be treated with a crystalloid with potassium replacement. 0.9% N. Saline should be given if there is hypochloraemia. Otherwise Hartmann’s or Ringer lactate should be given for diarrhoea/ileostomy/ileus/obstruction. Hartmann’s should also be given in sodium losses secondary to diuretics.
High risk patients should receive fluids and inotropes.
An attempt should be made to detect pre or operative hypovolaemia using flow based measurements. If this is not available, then clinical evaluation is needed i.e. JVP, pulse volume etc.
In Blood loss or infection causing hypovolaemia should be treated with a balanced crystalloid or colloid (or until blood available in blood loss). A critically ill patient is unable to excrete Na or H20 leading to a 5% risk of interstitial oedema. Therefore 5% dextrose as well as colloid should be given.
If patients need IV fluid resuscitation, use crystalloids that contain sodium in the range 130-154 mmol/l, with a bolus of 500 ml over less than 15 minutes (NICE Guidance CG 174).

1545
Q

Which statement relating to the peri operative management of patients with diabetes mellitus is false?

They should be placed first on the operating list

An intravenous sliding scale should be used in all cases

Potassium supplementation is likely to be required in diabetics on a sliding scale

Electrolyte abnormalities are more common after major visceral resections

Blood glucose monitoring is required during general anaesthesia

A

B-An intravenous sliding scale should be used in all cases

A sliding scale (or the real name: variable-rate intravenous insulin infusion, sometimes abbreviated to ‘VRIII’) is an intravenous drip with insulin and glucose, two separate lines which enter the hand or wrist via one intravenous needle

This is not the case and some type 2 diabetics may be managed using a watch and wait policy with regular blood glucose monitoring. The cellular shifts of potassium with sliding scales may cause problems with electrolyte management which should be anticipated.

Preparation for surgery

Elective and emergency patients require different preparation.

Elective cases
Consider pre admission clinic to address medical issues.
Blood tests including FBC, U+E, LFT’s, Clotting, Group and Save
Urine analysis
Pregnancy test
Sickle cell test
ECG/ Chest x-ray

Exact tests to be performed will depend upon the proposed procedure and patient fitness.

Risk factors for development of deep vein thrombosis should be assessed and a plan for thromboprophylaxis formulated.

Diabetes
Diabetic patients have greater risk of complications.
Poorly controlled diabetes carries high risk of wound infections.
Patients with diet or tablet controlled diabetes may be managed using a policy of omitting medication and checking blood glucose levels regularly. Diabetics who are poorly controlled or who take insulin may require a intravenous sliding scale. Potassium supplementation should also be given.
Diabetic cases should be operated on first.

Emergency cases
Stabilise and resuscitate where needed.
Consider whether antibiotics are needed and when and how they should be administered.
Inform blood bank if major procedures planned particularly where coagulopathies are present at the outset or anticipated (e.g. Ruptured AAA repair)
Don’t forget to consent and inform relatives.

Special preparation
Some procedures require special preparation:
Thyroid surgery; vocal cord check.
Parathyroid surgery; consider methylene blue to identify gland.
Sentinel node biopsy; radioactive marker/ patent blue dye.
Surgery involving the thoracic duct; consider administration of cream.
Pheochromocytoma surgery; will need alpha and beta blockade.
Surgery for carcinoid tumours; will need covering with octreotide.
Colorectal cases; bowel preparation (especially left sided surgery)
Thyrotoxicosis; lugols iodine/ medical therapy.

References
Management of adults with diabetes undergoing surgery and elective procedures. NHS Diabetes. April 2011.

1546
Q

Which of the features below, following a head injury, is not an indication for an immediate CT head scan in children?

Drowsiness

A single, discrete episode of vomiting

A 9 month old child with a 6cm haematoma on the head

Numb left arm

Suspicion of a non accidental head injury

A

Whilst not an indication for immediate CT there should be a low threshold for admission and observation.

Head injury paediatrics

Criteria for immediate request for CT scan of the head (children)
* Loss of consciousness lasting more than 5 minutes (witnessed)
* Amnesia (antegrade or retrograde) lasting more than 5 minutes
* Abnormal drowsiness
* Three or more discrete episodes of vomiting
* Clinical suspicion of non-accidental injury
* Post-traumatic seizure but no history of epilepsy
* GCS less than 14, or for a baby under 1 year GCS (paediatric) less than 15, on assessment in the emergency department
* Suspicion of open or depressed skull injury or tense fontanelle
* Any sign of basal skull fracture (haemotympanum, panda’ eyes, cerebrospinal fluid leakage from the ear or nose, Battle’s sign)
* Focal neurological deficit
* If under 1 year, presence of bruise, swelling or laceration of more than 5 cm on the head
* Dangerous mechanism of injury (high-speed road traffic accident either as pedestrian, cyclist or vehicle occupant, fall from a height of greater than 3 m, high-speed injury from a projectile or an object)

1547
Q

A 26 year old electrician suffers a full thickness high voltage burn to his leg. On routine urine analysis he has + blood. His U+E’s show mild hyperkalaemia and a CK of 3000. What is the most likely explanation?

Deep vein thrombosis

Disseminated intra vascular coagulation

Rhabdomyolysis

Myocardial infarct

Glomerulonephritis

A

Electrical high voltage burns are associated with rhabdomyolysis. Acute tubular necrosis may occur. Aggressive IV fluids should be given.

Burns

Burns may be thermal, chemical or electrical. In the former category are burns which occur as a result of heat. Chemical burns occur when the skin is exposed to an extremely caustic or alkaline substance. Electrical burns occur following exposure to electrical current. The immediate management includes removal of the burning source which usually includes irrigation of the burned area. A detailed assessment then needs to be made of the extent of the burns and a number of charts are available for recording this information. The degree of injury relates to the temperature and duration of exposure. Most domestic burns are mainly scalds in young children.

Following the burn, there is a local response with progressive tissue loss and release of inflammatory cytokines. Systemically, there are cardiovascular effects resulting from fluid loss and sequestration of fluid into the third space. There is a marked catabolic response. Immunosuppression is common with large burns and bacterial translocation from the gut lumen is a recognised event. Sepsis is a common cause of death following major burns.

Types of burn

Type of burn Skin layers affected Skin appearance Blanching Management
Epidermal/Superficial Epidermis Red, moist Yes
Superficial partial thickness Epidermis and part of papillary dermis affected Pale, dry Yes Normally heals with no intervention
Deep partial thickness Epidermis, whole papillary dermis affected Mottled red colour No Needs surgical intervention (depending on site)
Full thickness Whole skin layer and subcutaneous tissue affected Dry, leathery hard wound No Burns centre

Depth of burn assessment
Bleeding on needle prick
Sensation
Appearance
Blanching to pressure

Percentage burn estimation
Lund Browder chart: most accurate even in children
Wallace rule of nines
Palmar surface: surface area palm = 0.8% burn

> 15% body surface area burns in adults needs urgent burn fluid resuscitation

Transfer to burn centre if:
Need burn shock resuscitation
Face/hands/genitals affected
Deep partial thickness or full thickness burns
Significant electrical/chemical burns

Management
The initial aim is to stop the burning process and resuscitate the patient. Intravenous fluids will be required for children with burns greater than 10% of total body surface area. Adults with burns greater than 15% of total body surface area will also require IV fluids. The fluids are calculated using the Parkland formula which is; volume of fluid= total body surface area of the burn % x weight (Kg) x2-4 (preference for lower amount i.e. 2ml to avoid excessive fluid overload). Half of the fluid is administered in the first 8 hours. A urinary catheter should be inserted. Analgesia should be given. Complex burns, burns involving the hand perineum and face and burns >10% in adults and >5% in children should be transferred to a burns unit.

Circumferential burns affecting a limb or severe torso burns impeding respiration may require escharotomy to divide the burnt tissue.

Conservative management is appropriate for superficial burns and mixed superficial burns that will heal in 2 weeks. More complex burns may require excision and skin grafting. Excision and primary closure is not generally practised as there is a high risk of infection.

There is no evidence to support the use of anti microbial prophylaxis or topical antibiotics in burn patients.

Escharotomies
Indicated in circumferential full thickness burns to the torso or limbs.
Careful division of the encasing band of burn tissue will potentially improve ventilation (if the burn involves the torso), or relieve compartment syndrome and oedema (where a limb is involved)

References
www.euroburn.org/e107files/downloads/guidelinesburncare.pdf

Barajas-Nava LA, López-Alcalde J, Roqué i Figuls M, Solà I, Bonfill Cosp X. Antibiotic prophylaxis for preventing burn wound infection. Cochrane Database of Systematic Reviews 2013, Issue 6. Art. No.: CD008738. DOI: 10.1002/14651858.CD008738.pub2.

Hettiaratchy S & Papini R. Initial management of a major burn: assessment and resuscitation. BMJ 2004;329:101-103

1548
Q

Which of the following muscle relaxants will tend to incite neuromuscular excitability following administration?

Atracurium

Suxamethonium

Vecuronium

Pancuronium

None of the above

A

Suxamethonium may induce generalised muscular contractions following administration. This may raise serum potassium levels.

Muscle relaxants

Suxamethonium
Depolarising neuromuscular blocker
Inhibits action of acetylcholine at the neuromuscular junction
Degraded by plasma cholinesterase and acetylcholinesterase (affected by lack of acetylcholinesterase)
Fastest onset and shortest duration of action of all muscle relaxants
Produces generalised muscular contraction prior to paralysis
Adverse effects include hyperkalaemia, malignant hyperthermia, delayed recovery
Atracurium
Non depolarising neuromuscular blocking drug
Duration of action usually 30-45 minutes
Generalised histamine release on administration may produce facial flushing, tachycardia and hypotension
Not excreted by liver or kidney, broken down in tissues by hydrolysis
Reversed by neostigmine
Vecuronium
Non depolarising neuromuscular blocking drug
Duration of action approximately 30 - 40 minutes
Degraded by liver and kidney and effects prolonged in organ dysfunction
Effects may be reversed by neostigmine
Pancuronium
Non depolarising neuromuscular blocker
Onset of action approximately 2-3 minutes
Duration of action up to 2 hours
Effects may be partially reversed with drugs such as neostigmine

1549
Q

Which of these fluids is not an intravenous colloid?

Gelofusine

Dextran 40

Human albumin solution

Hydroxyethyl starch

Bicarbonate 8.4%

A

Bicarbonate is a crystalloid.

Pre operative fluid management

Fluid management has been described in the British Consensus guidelines on IV fluid therapy for Adult Surgical patients (GIFTASUP) and by NICE (CG174 December 2013 updated May 2017)

The Recommendations include:
Use Ringer’s lactate or Hartmann’s when a crystalloid is needed for resuscitation or replacement of fluids. Avoid 0.9% N. Saline (due to risk of hyperchloraemic acidosis) unless patient vomiting or has gastric drainage.
Use 4%/0.18% dextrose saline or 5% dextrose in maintenance fluids. It should not be used in resuscitation or as replacement fluids.
Adult maintenance fluid requirements are: Na 50-100 mmol/day and K 40-80 mmol/day in 1.5-2.5L fluid per day.
Patients for elective surgery should NOT be nil by mouth for >2 hours (unless has disorder of gastric emptying).
Patients for elective surgery should be given carbohydrate rich drinks 2-3h before. Ideally this should form part of a normal pre op plan to facilitate recovery.
Avoid mechanical bowel preparation.
If bowel prep is used, simultaneous administration of Hartmann’s or Ringer’s lactate should be considered.
Excessive fluid losses from vomiting should be treated with a crystalloid with potassium replacement. 0.9% N. Saline should be given if there is hypochloraemia. Otherwise Hartmann’s or Ringer lactate should be given for diarrhoea/ileostomy/ileus/obstruction. Hartmann’s should also be given in sodium losses secondary to diuretics.
High risk patients should receive fluids and inotropes.
An attempt should be made to detect pre or operative hypovolaemia using flow based measurements. If this is not available, then clinical evaluation is needed i.e. JVP, pulse volume etc.
In Blood loss or infection causing hypovolaemia should be treated with a balanced crystalloid or colloid (or until blood available in blood loss). A critically ill patient is unable to excrete Na or H20 leading to a 5% risk of interstitial oedema. Therefore 5% dextrose as well as colloid should be given.
If patients need IV fluid resuscitation, use crystalloids that contain sodium in the range 130-154 mmol/l, with a bolus of 500 ml over less than 15 minutes (NICE Guidance CG 174).

1550
Q

A 62 year old diabetic man presents with long standing plantar ulcer. He has clinical evidence of a charcot foot. What is the most likely diagnosis?

Mixed ulcer

Chronic obliterative arterial disease

Neuropathic ulcer

Squamous cell carcinoma

Deep venous insufficiency

A

Plantar ulcers in association with peripheral neuropathy are often neuropathic. They classically occur at pressure points.

Lower leg ulcers

Venous leg ulcers
Most due to venous hypertension, secondary to chronic venous insufficiency (other causes include calf pump dysfunction or neuromuscular disorders)
Ulcers form due to capillary fibrin cuff or leucocyte sequestration
Features of venous insufficiency include oedema, brown pigmentation, lipodermatosclerosis, eczema
Location above the ankle, painless
Deep venous insufficiency is related to previous DVT and superficial venous insufficiency is associated with varicose veins
Doppler ultrasound looks for presence of reflux and duplex ultrasound looks at the anatomy/ flow of the vein
Management: 4 layer compression banding after exclusion of arterial disease or surgery
If fail to heal after 12 weeks or >10cm2 skin grafting may be needed

Marjolin’s ulcer

Squamous cell carcinoma
Occurring at sites of chronic inflammation e.g; burns, osteomyelitis after 10-20 years
Mainly occur on the lower limb

Arterial ulcers
Occur on the toes and heel
Painful
There may be areas of gangrene
Cold with no palpable pulses
Low ABPI measurements

Neuropathic ulcers
Commonly over plantar surface of metatarsal head and plantar surface of hallux
The plantar neuropathic ulcer is the condition that most commonly leads to amputation in diabetic patients
Due to pressure
Management includes cushioned shoes to reduce callus formation

Pyoderma gangrenosum

Associated with inflammatory bowel disease/RA
Can occur at stoma sites
Erythematous nodules or pustules which ulcerate

1551
Q

A 25 year old builder presents with a reducible swelling in the right groin, it is becoming larger and has not been operated on previously. What is the best course of action?

Open inguinal herniotomy

Laparoscopic inguinal herniotomy

Open Bassini repair

Laparoscopic Bassini repair

Open Lichtenstein repair

A

The hernia can be repaired by either open or laparoscopic techniques. However, herniotomy is not performed as an isolated procedure in adults. The Bassini darn repair has a high recurrence rate. An open Lichtenstein repair using mesh is appropriate. There is a 0.77% recurrence rate with this technique. A Shouldice repair is an acceptable alternative if the surgeon is experienced

Inguinal hernia surgery

Inguinal hernias occur when the abdominal viscera protrude through the anterior abdominal wall into the inguinal canal. They may be classified as being either direct or indirect. The distinction between these two rests on their relation to Hesselbach’s triangle.

Boundaries of Hesselbach’s Triangle
Medial: Rectus abdominis
Lateral: Inferior epigastric vessels
Inferior: Inguinal ligament

Hernias occurring within the triangle tend to be direct and those outside - indirect.

Diagnosis
Most cases are diagnosed clinically, a reducible swelling may be located at the level of the inguinal canal. Large hernia’s may extend down into the male scrotum, these will not trans-illuminate and it is not possible to ‘get above’ the swelling.
Cases that are unclear on examination, but suspected from the history, may be further investigated using ultrasound or by performing a herniogram.

Treatment
Hernias associated with few symptoms may be managed conservatively. Symptomatic hernias or those which are at risk of developing complications are usually treated surgically.
First time hernias may be treated by performing an open inguinal hernia repair; the inguinal canal is opened, the hernia reduced and the defect repaired. A prosthetic mesh may be placed posterior to the cord structures to re-enforce the repair and reduce the risk of recurrence.
Recurrent hernias and those which are bilateral are generally managed with a laparoscopic approach. This may be via an intra or extra peritoneal route. As in open surgery a mesh is deployed. However, it will typically lie posterior to the deep ring.

Inguinal hernia in children
Inguinal hernias in children are almost always of an indirect type and therefore are usually dealt with by herniotomy, rather than herniorraphy. Neonatal hernias especially in those children born prematurely are at highest risk of strangulation and should be repaired urgently. Other hernias may be repaired on an elective basis.

References
The UK Based National Institute of Clinical Excellence has published guidelines relating to the choice between open and laparoscopic inguinal hernia repair. Which users may find interesting:

http://guidance.nice.org.uk/TA83/Guidance/pdf/English

1552
Q

A 72 year old man present in the vascular clinic with calf pain present on walking 100 yards. He is an ex-smoker and lives alone. On examination, he has reasonable leg pulses. His right dorsalis pedis pulse gives a monophasic doppler signal with an ankle brachial pressure index measurement of 0.7. All other pressures are acceptable. There is no evidence of ulceration or gangrene. What is the best course of action?

Conservative management with medical therapy and exercise

Angioplasty

Arterial bypass surgery with PTFE graft

Arterial bypass surgery with vein graft

Watch and wait

A

Structured exercise programmes combined with medical therapy will improve many patients. Should his symptoms worsen or fail to improve then a more comprehensive work up should be considered.

Peripheral vascular disease

Indications for surgery to revascularise the lower limb
Intermittent claudication
Critical ischaemia
Ulceration
Gangrene

Intermittent claudication that is not disabling may provide a relative indication, whilst the other complaints are often absolute indications depending upon the frailty of the patient.

Assessment
Clinical examination
Ankle brachial pressure index measurement
Duplex arterial ultrasound
Angiography (standard, CT or MRI): usually performed only if intervention being considered.

Angioplasty
In order for angioplasty to be undertaken successfully the artery has to be accessible. The lesion relatively short and reasonable distal vessel runoff. Longer lesions may be amenable to sub-intimal angioplasty.

Surgery
Surgery will be undertaken where attempts at angioplasty have either failed or are unsuitable. Bypass essentially involves bypassing the affected arterial segment by utilising a graft to run from above the disease to below the disease. As with angioplasty good runoff improves the outcome.

Some key concepts with bypass surgery

Superficial femoral artery occlusion to the above knee popliteal
In the ideal scenario, vein (either in situ or reversed LSV) would be used as a conduit. However, prosthetic material has reasonable 5 year patency rates and some would advocate using this in preference to vein so that vein can be used for other procedures in the future. In general terms either technique is usually associated with an excellent outcome (if run off satisfactory).

Procedure
Artery dissected out, IV heparin 3,000 units given and then the vessels are cross clamped
Longitudinal arteriotomy
Graft cut to size and tunneled to arteriotomy sites
Anastomosis to femoral artery usually with 5/0 ‘double ended’ Prolene suture
Distal anastomosis usually using 6/0 ‘double ended’ Prolene

Distal disease
Femoro-distal bypass surgery takes longer to perform, is more technically challenging and has higher failure rates.
In elderly diabetic patients with poor runoff a primary amputation may well be a safer and more effective option. There is no point in embarking on this type of surgery in patients who are wheelchair bound.
In femorodistal bypasses vein gives superior outcomes to PTFE.

Rules
Vein mapping 1st to see whether there is suitable vein (the preferred conduit). Sub intimal hyperplasia occurs early when PTFE is used for the distal anastomosis and will lead to early graft occlusion and failure.
Essential operative procedure as for above knee fem-pop.
If there is insufficient vein for the entire conduit then vein can be attached to the end of the PTFE graft and then used for the distal anastomosis. This type of ‘vein boot’ is technically referred to as a Miller Cuff and is associated with better patency rates than PTFE alone.
Remember the more distal the arterial anastomosis the lower the success rate.

References
Peach G et al. Diagnosis and management of peripheral arterial disease. BMJ 2012; 345: 36-41.

1553
Q

What is the most useful test for a 5 year old who has vesicoureteric reflux in whom there are concerns about the potential of renal scarring?

Intravenous urogram

Renal CT scan

DMSA scan

Micturating cystourethrogram

Retrograde ureterogram

A

Although MAG 3 renograms may provide some information relating to the structural integrity of the kidney, many still consider a DMSA scan to be the gold standard for the detection of renal scarring (which is the main concern in PUJ obstruction and infections).

Functional renal imaging

DMSA scan
Dimercaptosuccinic acid (DMSA) scintigraphy
DMSA localises to the renal cortex with little accumulation in the renal papilla and medulla. It is useful for the identification of cortical defects and ectopic or aberrant kidneys. It does not provide useful information on the ureter of collecting system.

Diethylene-triamine-penta-acetic acid (DTPA)
This is primarily a glomerular filtration agent. It is most useful for the assessment of renal function. Because it is filtered at the level of the glomerulus it provides useful information about the GFR. Image quality may be degraded in patients with chronic renal impairment and derangement of GFR.

MAG 3 renogram
Mercaptoacetyle triglycine is an is extensively protein bound and is primarily secreted by tubular cells rather than filtered at the glomerulus. This makes it the agent of choice for imaging the kidneys of patients with existing renal impairment (where GFR is impaired).

Micturating cystourethrogram (MCUG scan)
This scan provides information relating to bladder reflux and is obtained by filling the bladder with contrast media (via a catheter) and asking the child to void. Images are taken during this phase and the degree of reflux can be calculated

Intra venous urography
This examination is conducted by the administration of intravenous iodinated contrast media. The agent is filtered by the kidneys and excreted and may provide evidence of renal stones or other structural lesions. A rough approximation of renal function may be obtained using the technique. But it is not primarily a technique to be used for this purpose. With the advent of widespread non contrast CT scan protocols for the detection of urinary tract calculi it is now rarely used.

PET/CT
This may be used to evaluate structurally indeterminate lesions in the staging of malignancy.

References
Davis A et al. Investigating urinary tract infections in children. BMJ 2013 (346):35-37.

1554
Q

Which of the positions listed below best describes the location of the coeliac autonomic plexus?

Anterior to the aorta

Posterolateral to the aorta

Anterolateral to the sympathetic chain

Anteromedial to the sympathetic chain

Posterior to L1

A

Coeliac plexus

The coeliac plexus is the largest of the autonomic plexuses. It is located on a level of the last thoracic and first lumbar vertebrae. It surrounds the coeliac axis and the SMA. It lies posterior to the stomach and the lesser sac. It lies anterior to the crura of the diaphragm and the aorta. The plexus and ganglia are joined by the greater and lesser splanchnic nerves on both sides and branches from both the vagus and phrenic nerves.

1555
Q

The pathogenesis of osteopetrosis is best explained by a defect in which of the following?

Osteoclast function

PTH receptors

Osteoblast function

Calcium resorption in proximal tubule

Calcium absorption

A

Osteopetrosis

Overview
Also known as marble bone disease
Rare disorder of defective osteoclast function resulting in failure of normal bone resorption
Stem cell transplant and interferon-gamma have been used for treatment

1556
Q

A 13 month old boy is brought to the surgical clinic by his mother because his left testicle is not located in the scrotum. At which of the following sites would the testicle be located if it were an ectopic testis?

Canalicular

Inguinal

External inguinal ring

Superficial inguinal pouch

High scrotal

A

Ectopic testes are those that come to lie outside the normal range of embryological descent (i.e. in the superficial inguinal pouch). Other sites of ectopic testes include; base of penis, femoral triangle and perineum.

Testicular disorders-paediatric

Testicular disorders
Testicular disorders are some of the commonest conditions present in paediatric urological practice.

Cryptorchidism
The embryological descent of the testicle from within the abdominal cavity may be subject to a number of variations. Distinctions need to be made clinically from a non descended testis and a testis that is retractile.
Testis that lie outside the normal path of embryological descent are termed ectopic testis. Undescended testis occurs in 1% of male infants. Where the testis does not lie in an intra scrotal location, its location should be ascertained (by laparoscopy in first instance). Where both testes are absent, the infant may be intersex.
MRI scanning may reveal intra-abdominal testes; however a GA is often needed to perform this investigation in this age group.
Testes that are undescended should be placed in the scrotum after 1 year of age, as the testosterone surge that may facilitate descent occurs at 6 months of age.
Where the testes lie distally e.g. Superficial inguinal pouch an open orchidopexy is the procedure of choice.
With abdominal testes a laparoscopy should be performed. The risk of seminoma is increased in individuals with a non descended testes and this risk is not reduced by orchidopexy.

Testicular torsion
Typically the patient has severe sudden onset of scrotal pain. The difficulty in paediatric practice is the lack of clear history.
On examination the testis is tender and enlarged.
Management is by surgical exploration.
Delay beyond 6 hours is associated with low salvage rates.
A torted hyatid produces pain that is far more localised and the testis itself should feel normal. However, diagnostic doubt often exists and in such cases surgical exploration is warranted.

Hydrocele
Occur secondary to patent processus vaginalis
Present as fluid filling in scrotum or as cyst of the spermatic cord
Communicating hydroceles are treated by a trans inguinal ligation of the PPV
Cystic hydroceles in older children may be treated with scrotal exploration

1557
Q

Which of the following muscles is not within the posterior compartment of the lower leg?

Peroneus brevis

Flexor digitorum longus

Soleus

Popliteus

Flexor hallucis longus

A

Peroneus brevis lies in the lateral compartment.

Fascial compartments of the leg

Compartments of the thigh

Formed by septae passing from the femur to the fascia lata.
Compartment Nerve Muscles Blood supply
Anterior compartment Femoral
Iliacus
Tensor fasciae latae
Sartorius
Quadriceps femoris
Femoral artery
Medial compartment Obturator
Adductor longus/magnus/brevis
Gracilis
Obturator externus
Profunda femoris artery and obturator artery
Posterior compartment (2 layers) Sciatic
Semimembranosus
Semitendinosus
Biceps femoris
Branches of Profunda femoris artery

Compartments of the lower leg
Separated by the interosseous membrane (anterior and posterior compartments), anterior fascial septum (separate anterior and lateral compartments) and posterior fascial septum (separate lateral and posterior compartments)

Compartment Nerve Muscles Blood supply
Anterior compartment Deep peroneal nerve
Tibialis anterior
Extensor digitorum longus
Extensor hallucis longus
Peroneus tertius
Anterior tibial artery
Posterior compartment Tibial
Muscles: deep and superficial compartments (separated by deep transverse fascia)
Deep: Flexor hallucis longus, Flexor digitalis longus, Tibialis posterior, Popliteus
Superficial: Gastrocnemius, Soleus, Plantaris
Posterior tibial
Lateral compartment Superficial peroneal
Peroneus longus/brevis
Peroneal artery

1558
Q

Which of the tumour markers listed below is most likely to be elevated in a patient with ovarian cancer?

IgG 4

CEA

CA125

CA19-9

AFP

A

Tumour markers

Tumour markers may be divided into:
monoclonal antibodies against carbohydrate or glycoprotein tumour antigens
tumour antigens
enzymes (alkaline phosphatase, neurone specific enolase)
hormones (e.g. calcitonin, ADH)

It should be noted that tumour markers usually have a low specificity

Monoclonal antibodies
Tumour marker Association
CA 125 Ovarian cancer
CA 19-9 Pancreatic cancer
CA 15-3 Breast cancer
NB: The breast cancer tumour marker is not specific or sensitive enough to be used routinely.

Tumour antigens
Tumour marker Association
Prostate specific antigen (PSA) Prostatic carcinoma
Alpha-feto protein (AFP) Hepatocellular carcinoma, teratoma
Carcinoembryonic antigen (CEA) Colorectal cancer

1559
Q

A 75 year old lady is admitted with a 12 hour history of absolute constipation, vomiting and colicky abdominal pain. On examination, her abdomen is distended and she has right sided tenderness. Imaging demonstrates an obstructing hepatic flexure tumour with a caecal diameter of 11cm. What is the best course of action?

Undertake an immediate laparotomy

Commence resuscitation with intravenous fluids and then undertake a laparotomy 2-4 hours later

Administer antibiotics and intravenous fluids and schedule surgery for the following day

Arrange a colonoscopy

Administer oral sodium picosulphate

A

The sun should not rise and set on unrelieved large bowel obstruction! This patient has a competent ileocaecal valve. As a result lack of surgery would result in caecal perforation leading to faecal peritonitis with and associated high mortality rate.

Large bowel obstruction

Colonic obstruction remains a common surgical problem. It is most commonly due to malignancy (60%) and diverticular disease (20%). Volvulus affecting the colon accounts for 5% of cases. Acute colonic pseudo-obstruction remains a potential differential diagnosis in all cases. Intussusception affecting the colon (most often due to tumours in the adult population) remains a rare but recognised cause.
The typical patient will present with gradual onset of progressive abdominal distension, colicky abdominal pain and either obstipation or absolute constipation.
On examination abdominal distension is present, the presence of caecal tenderness (assuming no overt evidence of peritonitis) is a useful sign to elicit. A digital rectal examination and rigid sigmoidoscopy should be performed.
A plain abdominal x-ray is the usual first line test and; the caecal diameter and ileocaecal valve competency should be assessed on this film.

Imaging modalities
Debate long surrounds the use of CT versus gastrograffin enemas. The latter investigation has always been the traditional method of determining whether a structural lesion is indeed present. However, in the UK the use of this technique has declined and in most units a CT scan will be offered as the first line investigation by the majority of radiologists (and is advocated by the ACPGBI). In most cases this will provide sufficient detail to allow operative planning, and since malignancy accounts for most presentations may also stage the disease. In the event that the radiologist cannot provide a clear statement of lesion site, the surgeon should have no hesitation in requesting a contrast enema.

Surgical options
The decision as to when to operate or not is determined firstly by the patients physiological status. Unstable patients require resuscitation prior to surgery and admission to a critical care unit for invasive monitoring and potential inotropic support may be needed. In patients who are otherwise stable the decision then rests on the radiological and clinical findings. As a general rule the old adage that the sun should not rise and set on unrelieved large bowel obstruction still holds true. A caecal diameter of 12cm or more in the presence of complete obstruction with a competent ileocaecal valve and caecal tenderness is a sign of impending perforation and a relative indication for prompt surgery.

Right sided and transverse lesions
Right sided lesions producing large bowel obstruction should generally be treated by right hemicolectomy or its extended variant if the lesion lies in the distal transverse colon or splenic flexure. In these cases an ileocolic anastomosis may be easily constructed and even in the emergency setting has a low risk of anastomotic leak.

Left sided lesions
The options here lie between sub total colectomy and anastomosis, left hemicolectomy with on table lavage and primary anastomosis, left hemicolectomy and end colostomy formation and finally colonic stent insertion.
The usefulness of colonic stents was the subject of a Cochrane review in 2011. The authors concluded that on the basis of the data that they reviewed, there was no benefit from the use of colonic stents over conventional surgical resection with a tendency to better outcomes seen in the surgical group (1). A more recently conducted meta analysis met with the same conclusion (2). However, the recently concluded CREST trial has suggested that self expanding metallic stents can improve outcomes and avoids a stoma.

Rectosigmoid lesions
Lesions below the peritoneal reflection that are causing obstruction should generally be treated with a loop colostomy. Primary resection of unstaged rectal cancer would most likely carry a high CRM positivity rate and cannot be condoned. Where the lesion occupies the distal sigmoid colon the usual practice would be to perform a high anterior resection. The decision surrounding restoration of intestinal continuity would lie with the operating surgeon.

References
1. Sagar J. Colorectal stents for the management of malignant colonic obstructions. Cochrane Database of Systematic Reviews 2011, Issue 11. Art. No.: CD007378. DOI: 10.1002/14651858.CD007378.pub2.
2. Cirrochi et al Safety and efficacy of endoscopic colonic stenting as a bridge to surgery in the management of intestinal obstruction due to left colon and rectal cancer: A systematic review and meta-analysis. Surg Oncol. 2013 Mar;22(1):14-21.
3. Johnson W and Hawkins A. Large bowel obstruction. Clin Colon Rectal Surg. 2021 Jul;34(4):233-241. doi: 10.1055/s-0041-1729927.

1560
Q

A 72 year old man is admitted with an episode of brisk haematemesis. Following resuscitation an upper GI endoscopy is performed and a prominent blood vessel is identified in the sub mucosa approximately 6 cm from the O-G junction on the lesser curve of the stomach. What is the most likely explanation for the bleeding?

Gastric cancer

Portal gastropathy

Dieulafoy lesion

Linitis plastica

Gastritis

A

These small arterial lesions are a rare cause of bleeding and are characterised by a single large tortuous arteriole in the sub mucosa. Extra gastric lesions may occur.

Upper gastrointestinal bleeding

Patients may present with the following:
Haematemesis and/ or malaena
Epigastric discomfort
Sudden collapse

The extent to which these will occur will depend upon the source. Mortality is higher in patients presenting with haematemesis than malaena alone.

Oesophageal bleeding
Cause Presenting features
Oesophagitis Small volume of fresh blood, often streaking vomit. Malaena rare. Often ceases spontaneously. Usually history of antecedent GORD type symptoms.
Cancer Usually small volume of blood, except as pre terminal event with erosion of major vessels. Often associated symptoms of dysphagia and constitutional symptoms such as weight loss. May be recurrent until malignancy managed.
Mallory Weiss Tear Typically brisk small to moderate volume of bright red blood following bout of repeated vomiting. Malaena rare. Usually ceases spontaneously.
Varices Usually large volume of fresh blood. Swallowed blood may cause malaena. Often associated with haemodynamic compromise. May stop spontaneously but re-bleeds are common until appropriately managed.

Gastric Bleeding
Cause Presenting features
Gastric cancer May be frank haematemesis or altered blood mixed with vomit. Usually prodromal features of dyspepsia and may have constitutional symptoms. Amount of bleeding variable but erosion of major vessel may produce considerable haemorrhage.
Dieulafoy Lesion Often no prodromal features prior to haematemesis and malaena, but this arteriovenous malformation may produce quite considerable haemorrhage and may be difficult to detect endoscopically.
Diffuse erosive gastritis Usually haematemesis and epigastric discomfort. Usually there is an underlying cause such as recent NSAID usage. Large volume haemorrhage may occur with considerable haemodynamic compromise.
Gastric ulcer Small low volume bleeds more common so would tend to present as iron deficiency anaemia. Erosion into a significant vessel may produce considerable haemorrhage and haematemesis.

Duodenum
Most common cause of major haemorrhage is a posteriorly sited duodenal ulcer. However, ulcers at any site in the duodenum may present with haematemesis, malaena and epigastric discomfort. The pain of duodenal ulcer is slightly different to that of gastric ulcers and often occurs several hours after eating. Peri ampullary tumours may bleed but these are rare. In patients with previous abdominal aortic aneurysm surgery aorto-enteric fistulation remains a rare but important cause of major haemorrhage associated with high mortality.

Management
Admission to hospital careful monitoring, cross match blood, check FBC, LFTs, U+E and Clotting (as a minimum)
Patients with on-going bleeding and haemodynamic instability are likely to require O negative blood pending cross matched blood
Early control of airway is vital (e.g. Drowsy patient with liver failure)
Patients with suspected varices should receive terlipressin prior to endoscopy
Ideally all patients admitted with upper gastrointestinal haemorrhage should undergo Upper GI endoscopy within 24 hours of admission. In those who are unstable this should occur immediately after resuscitation or in tandem with it. The endoscopy department is a potentially dangerous place for unstable patients and it may be safer to perform the endoscopy in theatre with an anaesthetist present.
Varices should be banded or subjected to sclerotherapy. If this is not possible owing to active bleeding then a Sengstaken- Blakemore tube (or Minnesota tube) should be inserted. This should be done with care; gastric balloon should be inflated first and oesophageal balloon second. Remember the balloon will need deflating after 12 hours (ideally sooner) to prevent necrosis. Portal pressure should be lowered by combination of medical therapy +/- TIPSS.
Patients with erosive oesophagitis / gastritis should receive a proton pump inhibitor.
Mallory Weiss tears will typically resolve spontaneously
Identifiable bleeding points should receive combination therapy of injection of adrenaline and either a thermal or mechanical treatment. All who have received intervention should receive a continuous infusion of a proton pump inhibitor (IV omeprazole for 72 hours) to reduce the re-bleeding rate.
Patients with diffuse erosive gastritis who cannot be managed endoscopically and continue to bleed may require gastrectomy
Bleeding ulcers that cannot be controlled endoscopically may require laparotomy and ulcer underruning

Indications for surgery
Patients > 60 years
Continued bleeding despite endoscopic intervention
Recurrent bleeding
Known cardiovascular disease with poor response to hypotension

Surgery
Duodenal ulcer
Laparotomy, duodenotomy and under running of the ulcer. If bleeding is brisk then the ulcer is almost always posteriorly sited and will have invaded the gastroduodenal artery. Large bites using 0 Vicryl are taken above and below the ulcer base to occlude the vessel. The duodenotomy should be longitudinal but closed transversely to avoid stenosis.

For gastric ulcer
Under-running of the bleeding site
Partial gastrectomy-antral ulcer
Partial gastrectomy or under running the ulcer- lesser curve ulcer (involving left gastric artery)
Total gastrectomy if bleeding persists

Summary of Acute Upper GI bleeding recommendations:
The need for admission and timing of endoscopic intervention may be predicted by using the Blatchford score. This considers a patients Hb, serum urea, pulse rate and blood pressure. Those patients with a score of 0 are low risk, all others are considered high risk and require admission and endoscopy.
The requirement for pre endoscopic proton pump inhibition is contentious. In the UK the National Institute of Clinical Excellence guidelines suggest the pre endoscopic PPI therapy is unnecessary. Whilst it is accepted that such treatment has no impact on mortality or morbidity a Cochrane review of this practice in 2007 did suggest that it reduced the stigmata of recent haemorrhage at endoscopy. As a result many will still administer PPI to patients prior to endoscopic intervention.
Following endoscopy it is important to calculate the Rockall score for patients to determine their risk of rebleeding and mortality. A score of 3 or less is associated with a rebleeding rate of 4% and a very low risk of mortality and identifies a group of patients suitable for early discharge.

References
1. http://www.sign.ac.uk/guidelines/fulltext/105/index.html
2. Joint Advisory Group on Endoscopy (JAG) Guidelines - http://www.thejag.org.uk
3. NICE Guideline: Management of acute upper GI bleeding. July 2012.

1561
Q

A 73 year old lady is admitted with right iliac fossa pain. A plain abdominal x-ray is taken and the caecal diameter measured. Which of the following caecal diameters is pathological?

4cm

5cm

6cm

7cm

10cm

A

8 cm is still within normal limits. However, caecal diameters of 9 and 10 are pathological and should prompt further investigation.

Right colon

Ileocaecal valve
Entry point of the terminal ileum to the caecum
An important colonoscopic landmark
The ileocaecal valve is not always competent and this may allow partial decompression of an obstructed colon

Appendix
At the base of the caecum the taenia coalesce to mark the base of the appendix
This is a reliable way of locating the appendix surgically and is a constant landmark
The appendix has a small mesentery (the mesoappendix) and in this runs the appendiceal artery, a branch of the ileocolic artery.

The posterior aspect of the right colon is extra peritoneal and the anterior aspect intraperitoneal.

Relations

Posterior
Iliacus, Iliolumbar ligament, Quadratus lumborum, Transverse abdominis, Diaphragm at the tip of the last rib; Lateral cutaneous, ilioinguinal, and iliohypogastric nerves; the iliac branches of the iliolumbar vessels, the fourth lumbar artery, gonadal vessels, ureter and the right kidney.

Superior
Right kidney which is embedded in the perinephric fat

Medial
Mesentery which contains the ileocolic artery that supplies the right colon and terminal ileum. A further branch , the right colic artery, also contributes to supply the hepatic flexure and proximal transverse colon. Medially these pass through the mesentery to join the SMA. This occurs near to the head of the pancreas and care has to be taken when ligating the ileocolic artery near to its origin in cancer cases for fear of impinging on the SMA.

  • Anterior
    Coils of small intestine, the right edge of the greater omentum, and the anterior abdominal wall.

Nerve supply
Parasympathetic fibres of the vagus nerve (CN X)

Arterial supply
Ileocolic artery and right colic artery, both branches of the SMA. While the ileocolic artery is almost always present, the right colic can be absent in 5-15% of individuals.

1562
Q

A 4 year old child presents with a 4-5 day history of feeling generally unwell and also of having a sore throat. On examination, there is marked cervical lymphadenopathy, the oropharynx is covered with a thick grey membrane which bleeds following attempted removal. What is the most likely diagnosis?

Acute streptococcal pharyngitis

Infection with Epstein Barr virus

Diptheria

Ludwigs angina

Ingestion of caustic soda

A

Infection with diphtheria classically causes a systemic illness that lasts several days. The tonsils or pharynx can be covered in a thick grey membrane which bleeds on attempted removal. There is often quite marked cervical adenopathy and some individuals can have a bulls neck appearance. Death can occur through airway compromise, which is why the often described attempted removal of the pseudomembrane so beloved of examiners, is, in practice rather a foolish thing to attempt in a young child!

Acute tonsillitis

Characterised by pharyngitis, fever, malaise and lymphadenopathy.
Over half of all cases are bacterial with Streptococcus pyogenes the most common organism
The tonsils are typically oedematous and yellow or white pustules may be present
Infectious mononucleosis may mimic the condition.
Treatment with penicillin type antibiotics is indicated for bacterial tonsillitis.
Bacterial tonsillitis may result in local abscess formation (quinsy)

1563
Q

Which of the following muscles lies medial to the long thoracic nerve?

Serratus anterior

Latissimus dorsi

Coracobrachialis

Pectoralis minor

None of the above

A

The serratus anterior lies medial, the pectoral muscles being related more anteriorly.

Long thoracic nerve

Derived from ventral rami of C5, C6, and C7 (close to their emergence from intervertebral foramina)
It runs downward and passes either anterior or posterior to the middle scalene muscle
It reaches upper tip of serratus anterior muscle and descends on outer surface of this muscle, giving branches into it
Winging of Scapula occurs in long thoracic nerve injury (most common) or from spinal accessory nerve injury (which denervates the trapezius) or a dorsal scapular nerve injury

1564
Q

A 28 year old lady has a malignant melanoma removed from her calf. Which of the following pathological criteria carries the greatest prognostic weighting?

Vascular invasion

Abnormal mitoses

Breslow thickness

Perineural invasion

Lymphocytic infiltrates

A

The Breslow thickness has considerable prognostic importance. Lymphocytic infiltrates may be associated with an improved prognosis, but do not carry nearly the same weight as increased thickness.

Malignant melanoma

The main diagnostic features (major criteria):
Change in size
Change in shape
Change in colour
Secondary features (minor criteria)
Diameter >6mm
Inflammation
Oozing or bleeding
Altered sensation

Treatment
Suspicious lesions should undergo excision biopsy. The lesion should be removed in completely as incision biopsy can make subsequent histopathological assessment difficult.
Once the diagnosis is confirmed the pathology report should be reviewed to determine whether further re-excision of margins is required (see below):

Margins of excision-Related to Breslow thickness
Lesions 0-1mm thick 1cm
Lesions 1-2mm thick 1- 2cm (Depending upon site and pathological features)
Lesions 2-4mm thick 2-3 cm (Depending upon site and pathological features)
Lesions >4 mm thick 3cm
Marsden J et al Revised UK guidelines for management of Melanoma. Br J Dermatol 2010 163:238-256.

Further treatments such as sentinel lymph node mapping, isolated limb perfusion and block dissection of regional lymph node groups should be selectively applied.

Guidelines
The UK NICE guidance is covered by Melanoma: assessment and management NICE guideline [NG14]. However, the complex oncological regimens are not extensively covered in this guidance though more formal guidance is provided by relevant oncological organisations.

1565
Q

Which statement relating to actinomycosis is false?

They are gram positive bacilli

They are strict aerobes

It may be a cause of chronic multiple abscesses

Abdominal cases may develop in the appendix

Open biopsy of the lesions is the best diagnostic test

A

They are facultative anaerobes and may be difficult to culture. Direct visualisation of organisms and sulphur granules from lesions themselves is the easiest way to make a diagnosis. It remains a differential of conditions such as hydradenitis supprativa, particularly if it is occurring in odd locations and with deeper abscesses than usual.

Surgical Microbiology

An extensive topic so an overview is given here. Organisms causing common surgical infections are reasonable topics in the examination. However, microbiology is less rigorously tested than anatomy, for example.

Common organisms

Staphylococcus aureus
Facultative anaerobe
Gram positive coccus
Haemolysis on blood agar plates
Catalase positive
20% population are long term carriers
Exo and entero toxin may result in toxic shock syndrome and gastroenteritis respectively
Ideally treated with penicillin although many strains now resistant through beta Lactamase production. In the UK less than 5% of isolates are sensitive to penicillin.
Resistance to methicillin (and other antibiotics) is mediated by the mec operon , essentially penicillin binding protein is altered and resistance to this class of antibiotics ensues
Common cause of cutaneous infections and abscesses

Streptococcus pyogenes
Gram positive, forms chain like colonies, Lancefield Group A Streptococcus
Produces beta haemolysis on blood agar plates
Rarely part of normal skin microflora
Catalase negative
Releases a number of proteins/ virulence factors into host including hyaluronidase, streptokinase which allow rapid tissue destruction
Releases superantigens such as pyogenic exotoxin A which results in scarlet fever
Remains sensitive to penicillin, macrolides may be used as an alternative.

Escherichia coli
Gram negative rod
Facultative anaerobe, non sporing
Wide range of subtypes and some are normal gut commensals
Some subtypes such as 0157 may produce lethal toxins resulting in haemolytic-uraemic syndrome
Enterotoxigenic E-Coli produces an enterotoxin (ST enterotoxin) that results in large volume fluid secretion into the gut lumen (Via cGMP activation)
Enteropathogenic E-Coli binds to intestinal cells and cause structural damage, this coupled with a moderate (or in case of enteroinvasive E-Coli significant) invasive component produces enteritis and large volume diarrhoea together with fever.
They are resistant to many antibiotics used to treat gram positive infections and acquire resistance rapidly and are recognised as producing beta lactamases

Campylobacter jejuni
Curved, gram negative, non sporulating bacteria
One of the commonest causes of diarrhoea worldwide
Produces enteritis which is often diffuse and blood may be passed
Remains a differential for right iliac fossa pain with diarrhoea
Self limiting infection so antibiotics are not usually advised. However, the quinolones are often rapidly effective.

Helicobacter pylori
Gram negative, helix shaped rod, microaerophillic
Produces hydrogenase that can derive energy from hydrogen released by intestinal bacteria
Flagellated and mobile
Those carrying the cag A gene may cause ulcers
It secretes urease that breaks down gastric urea> Carbon dioxide and ammonia> ammonium>bicarbonate (simplified!) The bicarbonate can neutralise the gastric acid.
Usually colonises the gastric antrum and irritates resulting in increased gastrin release and higher levels of gastric acid. These patients will develop duodenal ulcers. In those with more diffuse H-Pylori infection gastric acid levels are lower and ulcers develop by local tissue damage from H-Pylori- these patients get gastric ulcers.
Diagnosis may be made by serology (approx. 75% sensitive). Biopsy urease test during endoscopy probably the most sensitive.
In patients who are colonised 10-20% risk of peptic ulcer, 1-2% risk gastric cancer, <1% risk MALT lymphoma.

1566
Q

The following statements regarding the rectus abdominis muscle are true except:

It runs from the symphysis pubis to the xiphoid process

Its nerve supply is from the ventral rami of the lower 6 thoracic nerves

It has collateral supply from both superior and inferior epigastric vessels

It lies in a muscular aponeurosis throughout its length

It has a number of tendinous intersections that penetrate through the anterior layer of the muscle

A

Rectus abdominis
Arises from the pubis.
Inserts into 5th, 6th, 7th costal cartilages.
The muscle lies in the rectal sheath, which also contains the superior and inferior epigastric artery and vein.
Action: flexion of thoracic and lumbar spine.
Nerve supply: anterior primary rami of T7-12.
The aponeurosis is deficient below the arcuate line.

Abdominal wall

The 2 main muscles of the abdominal wall are the rectus abdominis (anterior) and the quadratus lumborum (posterior).
The remaining abdominal wall consists of 3 muscular layers. Each muscle passes from the lateral aspect of the quadratus lumborum posteriorly to the lateral margin of the rectus sheath anteriorly. Each layer is muscular posterolaterally and aponeurotic anteriorly.

Muscles of abdominal wall
External oblique
Lies most superficially
Originates from 5th to 12th ribs
Inserts into the anterior half of the outer aspect of the iliac crest, linea alba and pubic tubercle
More medially and superiorly to the arcuate line, the aponeurotic layer overlaps the rectus abdominis muscle
The lower border forms the inguinal ligament
The triangular expansion of the medial end of the inguinal ligament is the lacunar ligament.
Internal oblique
Arises from the thoracolumbar fascia, the anterior 2/3 of the iliac crest and the lateral 2/3 of the inguinal ligament
The muscle sweeps upwards to insert into the cartilages of the lower 3 ribs
The lower fibres form an aponeurosis that runs from the tenth costal cartilage to the body of the pubis
At its lowermost aspect it joins the fibres of the aponeurosis of transversus abdominis to form the conjoint tendon.
Transversus abdominis
Innermost muscle
Arises from the inner aspect of the costal cartilages of the lower 6 ribs , from the anterior 2/3 of the iliac crest and lateral 1/3 of the inguinal ligament
Its fibres run horizontally around the abdominal wall ending in an aponeurosis. The upper part runs posterior to the rectus abdominis. Lower down the fibres run anteriorly only.
The rectus abdominis lies medially; running from the pubic crest and symphysis to insert into the xiphoid process and 5th, 6th and 7th costal cartilages. The muscles lies in a aponeurosis as described above.
Nerve supply: anterior primary rami of T7-12

Surgical notes
During abdominal surgery it is usually necessary to divide either the muscles or their aponeuroses. During a midline laparotomy it is desirable to divide the aponeurosis. This will leave the rectus sheath intact above the arcuate line and the muscles intact below it. Straying off the midline will often lead to damage to the rectus muscles, particularly below the arcuate line where they may often be in close proximity to each other.

1567
Q

A 20 year old male notices a mild painful swelling of his right scrotum. He also complains of abdominal pain. Clinically, the patient is found to have a swollen right testicle. Apart from a supraclavicular node, there is no obvious lymphadenopathy. What is the best course of action?

Orchidectomy via a scrotal approach

Tru cut biopsy of the testis

FNAC of the testis

Orchidectomy via an inguinal approach

Administration of antibiotics

A

The patient is likely to have a teratoma which has metastasized to the supraclavicular nodes. There is suspicion of spread to the para-aortic nodes due to the abdominal pain. He will need orchidectomy and combination chemotherapy. There is no role for orchidectomy via scrotal approach in malignancy.

Scrotal swelling

Differential diagnosis
Inguinal hernia If inguinoscrotal swelling; cannot ‘get above it’ on examination
Cough impulse may be present
May be reducible
Testicular tumours Often discrete testicular nodule (may have associated hydrocele)
Symptoms of metastatic disease may be present
USS scrotum and serum AFP and β HCG required
Acute epididymo-orchitis Often history of dysuria and urethral discharge
Swelling may be tender and eased by elevating testis
Most cases due to Chlamydia
Infections with other gram negative organisms may be associated with underlying structural abnormality
Epididymal cysts Single or multiple cysts
May contain clear or opalescent fluid (spermatoceles)
Usually occur over 40 years of age
Painless
Lie above and behind testis
It is usually possible to ‘get above the lump’ on examination
Hydrocele Non painful, soft fluctuant swelling
Often possible to ‘get above it’ on examination
Usually contain clear fluid
Will often transilluminate
May be presenting feature of testicular cancer in young men
Testicular torsion Severe, sudden onset testicular pain
Risk factors include abnormal testicular lie
Typically affects adolescents and young males
On examination testis is tender and pain not eased by elevation
Urgent surgery is indicated, the contra lateral testis should also be fixed
Varicocele Varicosities of the pampiniform plexus
Typically occur on left (because testicular vein drains into renal vein)
May be presenting feature of renal cell carcinoma
Affected testis may be smaller and bilateral varicoceles may affect fertility

Management
Testicular malignancy is always treated with orchidectomy via an inguinal approach. This allows high ligation of the testicular vessels and avoids exposure of another lymphatic field to the tumour.
Torsion is commonest in young teenagers and the history in older children can be difficult to elicit. Intermittent torsion is a recognised problem. The treatment is prompt surgical exploration and testicular fixation. This can be achieved using sutures or by placement of the testis in a Dartos pouch.
Varicoceles are usually managed conservatively. If there are concerns about testicular function of infertility then surgery or radiological management can be considered.
Epididymal cysts can be excised using a scrotal approach
Hydroceles are managed differently in children where the underlying pathology is a patent processus vaginalis and therefore an inguinal approach is used in children so that the processus can be ligated. In adults a scrotal approach is preferred and the hydrocele sac excised or plicated.

1568
Q

What is the commonest cause of an amoebic liver abscess?

Hydatid disease

Clonorchiasis

Entamoeba histolytica

Fasicola hepatica

Giardia

A

Entamoeba histiolytica is the only agent on the list that would produce an amoebic abscess.

Benign liver lesions

Benign liver lesions
Haemangioma
Most common benign tumours of mesenchymal origin
Incidence in autopsy series is 8%
Cavernous haemangiomas may be enormous
Clinically they are reddish purple hypervascular lesions
Lesions are normally separated from normal liver by ring of fibrous tissue
On ultrasound they are typically hyperechoic
Liver cell adenoma
90% develop in women in their third to fifth decade
Linked to use of oral contraceptive pill
Lesions are usually solitary
They are usually sharply demarcated from normal liver although they usually lack a fibrous capsule
On ultrasound the appearances are of mixed echoity and heterogeneous texture. On CT most lesions are hypodense when imaged prior to administration of IV contrast agents
In patients with haemorrhage or symptoms removal of the adenoma may be required
Mesenchymal hamartomas Congential and benign, usually present in infants. May compress normal liver
Liver abscess
Biliary sepsis is a major predisposing factor
Structures drained by the portal venous system form the second largest source
Common symptoms include fever, right upper quadrant pain. Jaundice may be seen in 50%
Ultrasound will usually show a fluid filled cavity, hyperechoic walls may be seen in chronic abscesses
Amoebic abscess
Liver abscess is the most common extra intestinal manifestation of amoebiasis
Between 75 and 90% lesions occur in the right lobe
Presenting complaints typically include fever and right upper quadrant pain
Ultrasonography will usually show a fluid filled structure with poorly defined boundaries
Aspiration yield sterile odourless fluid which has an anchovy paste consistency
Treatment is with metronidazole
Hyatid cysts
Seen in cases of Echinococcus infection
Typically an intense fibrotic reaction occurs around sites of infection
The cyst has no epithelial lining
Cysts are commonly unilocular and may grow to 20cm in size. The cyst wall is thick and has an external laminated hilar membrane and an internal enucleated germinal layer
Typically presents with malaise and right upper quadrant pain. Secondary bacterial infection occurs in 10%.
Liver function tests are usually abnormal and eosinophilia is present in 33% cases
Ultrasound may show septa and hyatid sand or daughter cysts.
Percutaneous aspiration was previously contra indicated, it is now incorporated into some treatment regimens
Treatment is by sterilisation of the cyst with mebendazole and may be followed by surgical resection. Hypertonic swabs are packed around the cysts during surgery
Polycystic liver disease
Usually occurs in association with polycystic kidney disease
Autosomal dominant disorder
Symptoms may occur as a result of capsular stretch
Cystadenoma
Rare lesions with malignant potential
Usually solitary multiloculated lesions
Liver function tests usually normal
Ultrasonography typically shows a large anechoic, fluid filled area with irregular margins. Internal echos may result from septa
Surgical resection is indicated in all cases

1569
Q

A 65-year-old Asian female presents with an extracapsular neck of femur fracture. Investigations show:

Calcium 2.07 mmol/l (2.20-2.60 mmol/l)
Phosphate 0.66 mmol/l (0.8-1.40 mmol/l)
ALP 256 IU/l (44-147 IU/l)

What is the most likely diagnosis?

Bone tuberculosis

Hypoparathyroidism

Myeloma

Osteomalacia

Paget’s disease

A

Osteomalacia
low: calcium, phosphate
raised: alkaline phosphatase
The low calcium and phosphate combined with the raised alkaline phosphatase point towards osteomalacia.

1570
Q

Which nerve is at risk during submandibular gland excision?

Maxillary nerve

Buccal nerve

Zygomatic nerve

Marginal mandibular nerve

Cervical nerve

A

The marginal mandibular nerve lies deep to platysma. It supplies the depressor anguli oris and the depressor labii inferioris. If injured it may lead to facial asymmetry and dribbling.

Submandibular gland

Relations of the submandibular gland
Superficial Platysma, deep fascia and mandible
Submandibular lymph nodes
Facial vein (facial artery near mandible)
Marginal mandibular nerve
Cervical branch of the facial nerve
Deep Facial artery (inferior to the mandible)
Mylohyoid muscle
Sub mandibular duct
Hyoglossus muscle
Lingual nerve
Submandibular ganglion
Hypoglossal nerve

Submandibular duct (Wharton’s duct)
Opens lateral to the lingual frenulum on the anterior floor of mouth.
5 cm length
Lingual nerve wraps around Wharton’s duct. As the duct passes forwards it crosses medial to the nerve to lie above it and then crosses back, lateral to it, to reach a position below the nerve.

Innervation
Sympathetic innervation- Derived from superior cervical ganglion
Parasympathetic innervation- Submandibular ganglion via lingual nerve

Arterial supply
Branch of the facial artery. The facial artery passes through the gland to groove its deep surface. It then emerges onto the face by passing between the gland and the mandible.

Venous drainage
Anterior facial vein (lies deep to the Marginal Mandibular nerve)

Lymphatic drainage
Deep cervical and jugular chains of nodes

1571
Q

A 73 year old heavy smoker presents with haemoptysis. On examination, he is cachectic and shows evidence of clubbing. Imaging shows a main bronchial tumour with massive mediastinal lymphadenopathy together with widespread visceral metastases. What is the most likely diagnosis?

Metastatic prostate cancer

Small cell lung cancer

Lymphoma

Adenocarcinoma

Squamous cell carcinoma

A

Small cell carcinoma is often associated with disseminated disease at presentation in the majority of cases. Most cases occur in the main airways and paraneoplastic features are common.

Lung cancer

Lung cancers may be classified according to histological subtypes. The main distinction is between small cell and non small cell lung cancer. Non small cell lung cancer is the most common variant and accounts for 80% of all lung cancers.

Non small cell lung cancer
These share common features of prognosis and management. They comprise the following tumours:
Squamous cell carcinoma (25% cases)
Adenocarcinoma (40% cases)
Large cell carcinoma (10% cases)
Paraneoplastic features and early disease dissemination are less likely than with small cell lung carcinoma. Adenocarcinoma is the most common lung cancer type encountered in never smokers.

Small cell lung carcinoma
Small cell lung carcinomas are comprised of cells with a neuro endocrine differentiation. The neuroendocrine hormones may be released from these cells with a wide range of paraneoplastic associations. These tumours are strongly associated with smoking and will typically arise in the larger airways. They disseminate early in the course of the disease and although they are usually chemosensitive this seldom results in long lasting remissions.

1572
Q

A 43 year old woman is admitted with acute cholecystitis and fails to settle. A laparoscopic cholecystectomy is being performed. At operation, the gallbladder has evidence of an empyema and Calots triangle is inflamed and the surgeon suspects that a Mirizzi syndrome has occurred. What is the best course of action?

Undertake an operative cholecystostomy

Perform a sub total cholecystectomy

Perform a laparotomy

Perform an exploration of the common bile duct

Continue to explore Calots triangle and complete the operation

A

This will address the acute sepsis and resolve the situation. Attempts at completing the surgery at this stage, even in expert hands carries a very high risk of bile duct injury. Do not confuse cholecystostomy with cholecystectomy.

Biliary disease

Diagnosis Typical features Pathogenesis
Gallstones Typically history of biliary colic or episodes of chlolecystitis. Obstructive type history and test results. Usually small calibre gallstones which can pass through the cystic duct. In Mirizzi syndrome the stone may compress the bile duct directly- one of the rare times that cholecystitis may present with jaundice
Cholangitis Usually obstructive and will have Charcot’s triad of symptoms (pain, fever, jaundice) Ascending infection of the bile ducts usually by E. coli and by definition occurring in a pool of stagnant bile.
Pancreatic cancer Typically painless jaundice with palpable gallbladder (Courvoisier’s Law) Direct occlusion of distal bile duct or pancreatic duct by tumour. Sometimes nodal disease at the portal hepatis may be the culprit in which case the bile duct may be of normal calibre.
TPN (total parenteral nutrition) associated jaundice Usually follows long term use and is usually painless with non obstructive features Often due to hepatic dysfunction and fatty liver which may occur with long term TPN usage.
Bile duct injury Depending upon the type of injury may be of sudden or gradual onset and is usually of obstructive type Often due to a difficult laparoscopic cholecystectomy when anatomy in Calots triangle is not appreciated. In the worst scenario the bile duct is excised and jaundice develops rapidly post operatively. More insidious is that of bile duct stenosis which may be caused by clips or diathermy injury.
Cholangiocarcinoma Gradual onset obstructive pattern Direct occlusion by disease and also extrinsic compression by nodal disease at the porta hepatis.
Septic surgical patient Usually hepatic features Combination of impaired biliary excretion and drugs such as ciprofloxacin which may cause cholestasis.
Metastatic disease Mixed hepatic and post hepatic Combination of liver synthetic failure (late) and extrinsic compression by nodal disease and anatomical compression of intra hepatic structures (earlier)

1573
Q

A 66 year old man has been suffering from weight loss and develops severe abdominal pain. He is admitted to hospital and undergoes a laparotomy. At operation, the entire small bowel is infarcted and only the left colon is viable. What is the most likely explanation?

Mesenteric venous thrombosis

Acute on chronic mesenteric ischaemia

Acute embolus to the inferior mesenteric artery

Acute embolus affecting the coeliac axis

Arcuate ligament compression syndrome

A

This man is likely to have underlying chronic mesenteric vascular disease. Only 15% of emboli will occlude SMA orifice leading to entire small bowel infarct. The background history of weight loss also favors an acute on chronic event.

Mesenteric vessel disease

Mesenteric ischaemia accounts for 1 in 1000 acute surgical admissions. It is primarily caused by arterial embolism resulting in infarction of the colon. It is more likely to occur in areas such as the splenic flexure that are located at the borders of the territory supplied by the superior and inferior mesenteric arteries.

Types
Acute mesenteric embolus (commonest 50%)
Sudden onset abdominal pain followed by profuse diarrhoea.
May be associated with vomiting.
Rapid clinical deterioration.
Serological tests: WCC, lactate, amylase may all be abnormal particularly in established disease. These can be normal in the early phases.
Acute on chronic mesenteric ischaemia
Usually longer prodromal history.
Post prandial abdominal discomfort and weight loss are dominant features. Patients will usually present with an acute on chronic event, but otherwise will tend not to present until mesenteric flow is reduced by greater than 80%.
When acute thrombosis occurs presentation may be as above. In the chronic setting the symptoms will often be those of ischaemic colitis (mucosa is the most sensitive area to this insult).
Mesenteric vein thrombosis
Usually a history over weeks.
Overt abdominal signs and symptoms will not occur until venous thrombosis has reached a stage to compromise arterial inflow.
Thrombophilia accounts for 60% of cases.
Low flow mesenteric infarction
This occurs in patients with multiple co morbidities in whom mesenteric perfusion is significantly compromised by overuse of inotropes or background cardiovascular compromise.
The end result is that the bowel is not adequately perfused and infarcts occur from the mucosa outwards.

Diagnosis
Serological tests: WCC, lactate, CRP, amylase (can be normal in early disease).
Cornerstone for diagnosis of arterial AND venous mesenteric disease is CT angiography scanning in the arterial phase with thin slices (<5mm). Venous phase contrast is not helpful.
SMA duplex USS is useful in the evaluation of proximal SMA disease in patients with chronic mesenteric ischaemia.
MRI is of limited use due to gut peristalsis and movement artefact.

Management
Overt signs of peritonism: Laparotomy
Mesenteric vein thrombosis: If no peritonism: Medical management with IV heparin
At operation limited resection of frankly necrotic bowel with view to relook laparotomy at 24-48h. In the interim urgent bowel revascularisation via endovascular (preferred) or surgery.

Prognosis
Overall poor. Best outlook is from an acute ischaemia from an embolic event where surgery occurs within 12h. Survival may be 50%. This falls to 30% with treatment delay. The other conditions carry worse survival figures.

1574
Q

A 22 year old fit and well male undergoes an emergency appendicectomy. He is given suxamethonium. An inflamed appendix is removed and the patient is returned to recovery. On arrival in the recovery area; the patient develops a tachycardia of 120 bpm and a temperature of 40 oC. He has generalised muscular rigidity. What is the most likely diagnosis?

Acute dystonic reaction

Malignant hyperthermia

Pelvic abscess

Epilepsy

Serotonin syndrome

A

Anaesthetic agents, such as suxamethonium, can cause malignant hyperthermia in patients with a genetic defect. Acute dystonic reaction normally is associated with antipsychotics (haloperidol) and metoclopramide. These lead to marked extrapyramidal effects. Serotonin syndrome is associated with the antidepressants selective serotonin reuptake inhibitors (SSRIs) and selective serotonin/norepinephrine reuptake inhibitors (SSNRIs). This causes a syndrome of agitation, tachycardia, hallucinations and hyper-reflexia.

Malignant hyperthermia

Overview
Condition seen following administration of anaesthetic agents ( rate of 1 in 15,000)
Characterised by hyperpyrexia and muscle rigidity
Cause by excessive release of Ca2+ from the sarcoplasmic reticulum of skeletal muscle
Associated with defects in a gene on chromosome 19 encoding the ryanodine receptor, which controls Ca2+ release from the sarcoplasmic reticulum
Neuroleptic malignant syndrome may have a similar aetiology

Causative agents
Halothane
Suxamethonium
Other drugs: antipsychotics (neuroleptic malignant syndrome)

Investigations
CK raised
Contracture tests with halothane and caffeine

Management
Dantrolene - prevents Ca2+ release from the sarcoplasmic reticulum

1575
Q

From which of the following cell types do giant cells most commonly originate?

Neutrophils

Myofibroblasts

Fibroblasts

Macrophages

Goblet cells

A

Although many cell types may give rise to giant cells, macrophages remain the most common.
Although many cell types may give rise to giant cells, macrophages remain the most common.
Giant cells

A giant cell is a mass formed by the union of several distinct types of cells
They are most commonly comprised of macrophages
They are different to granulomas although causative agents may overlap

1576
Q

A 28 year old man is shot in the abdomen and haemorrhages. Which of the following substances will produce vasoconstriction in response to this process?

Renin

Angiotensin I

Angiotensin II

Aldosterone

None of the above

A

Renin does not cause vasoconstriction. Angiotensin I is biologically inactive. Aldosterone will increase blood pressure but does not have direct vasospastic effects.

Shock

  • Shock occurs when there is insufficient tissue perfusion.
    The pathophysiology of shock is an important surgical topic and may be divided into the following aetiological groups:
    Septic
    Haemorrhagic
    Neurogenic
    Cardiogenic
    Anaphylactic

Septic shock
Septic shock is a major problem and those patients with severe sepsis have a mortality rate in excess of 40%. In those who are admitted to intensive care mortality ranges from 6% with no organ failure to 65% in those with 4 organ failure.

Sepsis is defined as an infection that triggers a particular Systemic Inflammatory Response Syndrome (SIRS). In 2016, the European Society of Intensive Care Medicine and the Society of Critical Care Medicine (SCCM) created a task force that proposed Sepsis-3, a new definition for sepsis. The new definition excluded the establishment of SIRS criteria to define sepsis and made it more nonspecific as any life-threatening organ dysfunction caused by the dysregulated host response to infection. The task force claimed that sequential organ failure assessment (SOFA) has a better predictive validity for sepsis than SIRS criteria. It has better prognostic accuracy and the ability to predict in-hospital mortality. To reduce the complexity of calculating the SOFA, they introduced q SOFA

Q SOFA

3 component assessment system with:

Systolic blood pressure below 100 mm Hg

Highest respiratory rate exceeding 21

Lowest Glasgow coma score is under 15

Although the validity of q SOFA is limited in an ICU setting, it has consistently outperformed SIRS criteria in predicting organ dysfunction in a non-ICU and ER setting. The use of vasopressors, mechanical ventilation, and aggressive therapeutic interventions in ICU limit the efficacy of q SOFA.

During the septic process there is marked activation of the immune system with extensive cytokine release. This may be coupled with or triggered by systemic circulation of bacterial toxins. These all cause endothelial cell damage and neutrophil adhesion. The overall hallmarks are thus those of excessive inflammation, coagulation and fibrinolytic suppression.

The main management issues with severe infections resulting in organ dysfunction are as follows:
Prompt administration of antibiotics to cover all likely pathogens coupled with a rigorous search for the source of infection.
Haemodynamic stabilisation. Many patients are hypovolaemic and require aggressive fluid administration. Aim for CVP 8-12 cm H2O, MAP >65mmHg.
Modulation of the septic response. This includes manoeuvres to counteract the changes and includes measures such as tight glycaemic control. The routine use of steroids is not advised.

In surgical patients, the main groups with septic shock include those with anastomotic leaks, abscesses and extensive superficial infections such as necrotising fasciitis. When performing surgery the aim should be to undertake the minimum necessary to restore physiology. These patients do not fare well with prolonged surgery. Definitive surgery can be more safely undertaken when physiology is restored and clotting in particular has been normalised.

Haemorrhagic shock
The average adult blood volume comprises 7% of body weight. Thus in the 70 Kg adult this will equate to 5 litres. This changes in children (8-9% body weight) and is slightly lower in the elderly.

The table below outlines the 4 major classes of haemorrhagic shock and their associated physiological sequelae:

Parameter Class I Class II Class III Class IV
Blood loss ml <750ml 750-1500ml 1500-2000ml >2000ml
Blood loss % <15% 15-30% 30-40% >40%
Pulse rate <100 >100 >120 >140
Blood pressure Normal Normal Decreased Decreased
Respiratory rate 14-20 20-30 30-40 >35
Urine output >30ml 20-30ml 5-15ml <5ml
Symptoms Normal Anxious Confused Lethargic

Decreasing blood pressure during haemorrhagic shock causes organ hypoperfusion and relative myocardial ischaemia. The cardiac index gives a numerical value for tissue oxygen delivery and is given by the equation: Cardiac index= Cardiac output/ body surface area. Where Hb is haemoglobin concentration in blood and SaO2 the saturation and PaO2 the partial pressure of oxygen. Detailed knowledge of this equation is required for the MRCS Viva but not for part A, although you should understand the principle.

In patients suffering from trauma the most likely cause of shock is haemorrhage. However, the following may also be the cause or occur concomitantly:

Tension pneumothorax
Spinal cord injury
Myocardial contusion
Cardiac tamponade

When assessing trauma patients it is worth remembering that in order to generate a palpable femoral pulse an arterial pressure of >65mmHg is required.

Once bleeding is controlled and circulating volume normalised the levels of transfusion should be to maintain a Hb of 7-8 in those with no risk factors for tissue hypoxia and Hb 10 for those who have such risk factors.

Neurogenic shock
This occurs most often following a spinal cord transection, usually at a high level. There is resultant interruption of the autonomic nervous system. The result is either decreased sympathetic tone or increased parasympathetic tone, the effect of which is a decrease in peripheral vascular resistance mediated by marked vasodilation.

This results in decreased preload and thus decreased cardiac output (Starlings law). There is decreased peripheral tissue perfusion and shock is thus produced. In contrast with many other types of shock peripheral vasoconstrictors are used to return vascular tone to normal.

Cardiogenic shock
In medical patients the main cause is ischaemic heart disease. In the traumatic setting direct myocardial trauma or contusion is more likely. Evidence of ECG changes and overlying sternal fractures or contusions should raise the suspicion of injury. Treatment is largely supportive and transthoracic echocardiography should be used to determine evidence of pericardial fluid or direct myocardial injury. The measurement of troponin levels in trauma patients may be undertaken but they are less useful in delineating the extent of myocardial trauma than following MI.

When cardiac injury is of a blunt nature and is associated with cardiogenic shock the right side of the heart is the most likely site of injury with chamber and or valve rupture. These patients require surgery to repair these defects and will require cardiopulmonary bypass to achieve this. Some may require intra aortic balloon pump as a bridge to surgery.

Anaphylactic shock
Anaphylaxis may be defined as a severe, life-threatening, generalised or systemic
hypersensitivity reaction.

Anaphylaxis is one of the few times when you would not have time to look up the dose of a medication. The Resuscitation Council guidelines on anaphylaxis have recently been updated. Adrenaline is by far the most important drug in anaphylaxis and should be given as soon as possible. The recommended doses for adrenaline, hydrocortisone and chlorpheniramine (if one wishes to use them) are as follows:

Adrenaline Hydrocortisone Chlorpheniramine
< 6 months 150 mcg (0.15ml 1 in 1,000) 25 mg 250 mcg/kg
6 months - 6 years 150 mcg (0.15ml 1 in 1,000) 50 mg 2.5 mg
6-12 years 300 mcg (0.3ml 1 in 1,000) 100 mg 5 mg
Adult and child 12 years 500 mcg (0.5ml 1 in 1,000) 200 mg 10 mg

Adrenaline can be repeated every 5 minutes if necessary. The best site for IM injection is the anterolateral aspect of the middle third of the thigh.

Common identified causes of anaphylaxis
food (e.g. Nuts) - the most common cause in children
drugs
venom (e.g. Wasp sting)

1577
Q

A 28 year old man is shot in the abdomen and haemorrhages. Which of the following substances will produce vasoconstriction in response to this process?

Renin

Angiotensin I

Angiotensin II

Aldosterone

None of the above

A

Renin does not cause vasoconstriction. Angiotensin I is biologically inactive. Aldosterone will increase blood pressure but does not have direct vasospastic effects.

Shock

  • Shock occurs when there is insufficient tissue perfusion.
    The pathophysiology of shock is an important surgical topic and may be divided into the following aetiological groups:
    Septic
    Haemorrhagic
    Neurogenic
    Cardiogenic
    Anaphylactic

Septic shock
Septic shock is a major problem and those patients with severe sepsis have a mortality rate in excess of 40%. In those who are admitted to intensive care mortality ranges from 6% with no organ failure to 65% in those with 4 organ failure.

Sepsis is defined as an infection that triggers a particular Systemic Inflammatory Response Syndrome (SIRS). In 2016, the European Society of Intensive Care Medicine and the Society of Critical Care Medicine (SCCM) created a task force that proposed Sepsis-3, a new definition for sepsis. The new definition excluded the establishment of SIRS criteria to define sepsis and made it more nonspecific as any life-threatening organ dysfunction caused by the dysregulated host response to infection. The task force claimed that sequential organ failure assessment (SOFA) has a better predictive validity for sepsis than SIRS criteria. It has better prognostic accuracy and the ability to predict in-hospital mortality. To reduce the complexity of calculating the SOFA, they introduced q SOFA

Q SOFA

3 component assessment system with:

Systolic blood pressure below 100 mm Hg

Highest respiratory rate exceeding 21

Lowest Glasgow coma score is under 15

Although the validity of q SOFA is limited in an ICU setting, it has consistently outperformed SIRS criteria in predicting organ dysfunction in a non-ICU and ER setting. The use of vasopressors, mechanical ventilation, and aggressive therapeutic interventions in ICU limit the efficacy of q SOFA.

During the septic process there is marked activation of the immune system with extensive cytokine release. This may be coupled with or triggered by systemic circulation of bacterial toxins. These all cause endothelial cell damage and neutrophil adhesion. The overall hallmarks are thus those of excessive inflammation, coagulation and fibrinolytic suppression.

The main management issues with severe infections resulting in organ dysfunction are as follows:
Prompt administration of antibiotics to cover all likely pathogens coupled with a rigorous search for the source of infection.
Haemodynamic stabilisation. Many patients are hypovolaemic and require aggressive fluid administration. Aim for CVP 8-12 cm H2O, MAP >65mmHg.
Modulation of the septic response. This includes manoeuvres to counteract the changes and includes measures such as tight glycaemic control. The routine use of steroids is not advised.

In surgical patients, the main groups with septic shock include those with anastomotic leaks, abscesses and extensive superficial infections such as necrotising fasciitis. When performing surgery the aim should be to undertake the minimum necessary to restore physiology. These patients do not fare well with prolonged surgery. Definitive surgery can be more safely undertaken when physiology is restored and clotting in particular has been normalised.

Haemorrhagic shock
The average adult blood volume comprises 7% of body weight. Thus in the 70 Kg adult this will equate to 5 litres. This changes in children (8-9% body weight) and is slightly lower in the elderly.

The table below outlines the 4 major classes of haemorrhagic shock and their associated physiological sequelae:

Parameter Class I Class II Class III Class IV
Blood loss ml <750ml 750-1500ml 1500-2000ml >2000ml
Blood loss % <15% 15-30% 30-40% >40%
Pulse rate <100 >100 >120 >140
Blood pressure Normal Normal Decreased Decreased
Respiratory rate 14-20 20-30 30-40 >35
Urine output >30ml 20-30ml 5-15ml <5ml
Symptoms Normal Anxious Confused Lethargic

Decreasing blood pressure during haemorrhagic shock causes organ hypoperfusion and relative myocardial ischaemia. The cardiac index gives a numerical value for tissue oxygen delivery and is given by the equation: Cardiac index= Cardiac output/ body surface area. Where Hb is haemoglobin concentration in blood and SaO2 the saturation and PaO2 the partial pressure of oxygen. Detailed knowledge of this equation is required for the MRCS Viva but not for part A, although you should understand the principle.

In patients suffering from trauma the most likely cause of shock is haemorrhage. However, the following may also be the cause or occur concomitantly:

Tension pneumothorax
Spinal cord injury
Myocardial contusion
Cardiac tamponade

When assessing trauma patients it is worth remembering that in order to generate a palpable femoral pulse an arterial pressure of >65mmHg is required.

Once bleeding is controlled and circulating volume normalised the levels of transfusion should be to maintain a Hb of 7-8 in those with no risk factors for tissue hypoxia and Hb 10 for those who have such risk factors.

Neurogenic shock
This occurs most often following a spinal cord transection, usually at a high level. There is resultant interruption of the autonomic nervous system. The result is either decreased sympathetic tone or increased parasympathetic tone, the effect of which is a decrease in peripheral vascular resistance mediated by marked vasodilation.

This results in decreased preload and thus decreased cardiac output (Starlings law). There is decreased peripheral tissue perfusion and shock is thus produced. In contrast with many other types of shock peripheral vasoconstrictors are used to return vascular tone to normal.

Cardiogenic shock
In medical patients the main cause is ischaemic heart disease. In the traumatic setting direct myocardial trauma or contusion is more likely. Evidence of ECG changes and overlying sternal fractures or contusions should raise the suspicion of injury. Treatment is largely supportive and transthoracic echocardiography should be used to determine evidence of pericardial fluid or direct myocardial injury. The measurement of troponin levels in trauma patients may be undertaken but they are less useful in delineating the extent of myocardial trauma than following MI.

When cardiac injury is of a blunt nature and is associated with cardiogenic shock the right side of the heart is the most likely site of injury with chamber and or valve rupture. These patients require surgery to repair these defects and will require cardiopulmonary bypass to achieve this. Some may require intra aortic balloon pump as a bridge to surgery.

Anaphylactic shock
Anaphylaxis may be defined as a severe, life-threatening, generalised or systemic
hypersensitivity reaction.

Anaphylaxis is one of the few times when you would not have time to look up the dose of a medication. The Resuscitation Council guidelines on anaphylaxis have recently been updated. Adrenaline is by far the most important drug in anaphylaxis and should be given as soon as possible. The recommended doses for adrenaline, hydrocortisone and chlorpheniramine (if one wishes to use them) are as follows:

Adrenaline Hydrocortisone Chlorpheniramine
< 6 months 150 mcg (0.15ml 1 in 1,000) 25 mg 250 mcg/kg
6 months - 6 years 150 mcg (0.15ml 1 in 1,000) 50 mg 2.5 mg
6-12 years 300 mcg (0.3ml 1 in 1,000) 100 mg 5 mg
Adult and child 12 years 500 mcg (0.5ml 1 in 1,000) 200 mg 10 mg

Adrenaline can be repeated every 5 minutes if necessary. The best site for IM injection is the anterolateral aspect of the middle third of the thigh.

Common identified causes of anaphylaxis
food (e.g. Nuts) - the most common cause in children
drugs
venom (e.g. Wasp sting)

1578
Q

Loss of taste sensation from the posterior third of the tongue is most likely the result to an injury to which of the structures listed below?

Hypoglossal nerve

Chorda tympani nerve

Facial nerve

Mandibular branch of the trigeminal nerve

Glossopharyngeal nerve

A

The glossopharyngeal nerve supplies the taste and general sensation to the posterior third of the tongue.

Tongue

The tongue is a muscular organ concerned with both feeding and speech. It is shaped like an inverted boot with a root through which extrinsic muscles pass to attach it to the soft palate, the styloid process, the hyoid bone and the mandible. It has a body that comprises muscle intermingled with lingual salivary glands, connective tissue, nerves, vessels and lymphatic follicles.
The dorsum of the tongue is divided by an ill defined V shaped groove, the sulcus terminalis, into an anterior two thirds and a posterior third.

The arterial blood supply to the tongue is from the paired sets of lingual arteries that arise from the external carotid. The venous drainage is via dorsal lingual veins which then join the lingual veins and eventually into the sublingual vein.

The nerve supply is as follows:
The lingual branch of the mandibular division of the trigeminal nerve supplies common sensation to the anterior two thirds
The chorda tympani branch of the facial nerve supplies taste to the anterior two thirds
The lingual branch of the glossopharyngeal nerve supplies common sensation and taste to the posterior third of the tongue
All intrinsic muscles of the tongue are supplied by the hypoglossal nerve

1579
Q

Which of the following blood gas results would fit with metabolic acidosis with a compensatory respiratory alkalosis?

pH 7.36, PaCO2 7.3, PO2 8.9 (FiO2 40%), Bicarbonate 30.2, Base excess +5.3

pH 7.14, PaCO2 7.4, PaO28.9 (FiO2
40%), Bicarbonate 14 mmol, Base excess -10.6

pH 7.57, PaCO23.5, PaO2 24.5 (FiO2
85%), Bicarbonate 23.5, Base excess +1.8 mmol

pH 7.32, PCO2 3.8, PaO2 22.2 (FiO2
40%), Bicarbonate 19.1, Base excess -7.9

pH 7.19, pCO2 10.2, pO2 16 (FiO2
85%), Bicarbonate 23.8, Base excess -2.2 mmol

A

D-pH 7.32, PCO2 3.8, PaO2 22.2 (FiO2
40%), Bicarbonate 19.1, Base excess -7.9

Arterial blood gas interpretation

In advanced life support training, a 5 step approach to arterial blood gas interpretation is advocated.

  1. How is the patient?
  2. Is the patient hypoxaemic?
    The Pa02 on air should be 10.0-13.0 kPa
  3. Is the patient acidaemic (pH <7.35) or alkalaemic (pH >7.45)
  4. What has happened to the PaCO2?
    If there is acidaemia, an elevated PaCO2 will account for this
  5. What is the bicarbonate level or base excess?
    A metabolic acidosis will have a low bicarbonate level and a low base excess (< -2 mmol)
    A metabolic alkalosis will have a high bicarbonate and a high base excess (> +2 mmol)
1580
Q

A 48 year old lady undergoes a redo thyroidectomy for a multinodular goitre. 24 hours post operatively she develops oculogyric crises and diffuse muscle spasm. What is the most appropriate course of action?

Arrange a CT scan of the head

Administration of intravenous magnesium

Administration of intravenous calcium

Return the patient to theatre for wound exploration

Call an anaesthetist to paralyse and ventilate the patient

A

She has most likely developed hypocalcaemic tetany and will require immediate calcium supplementation.

Complications following thyroid surgery

The major immediate risk following thyroidectomy is haemorrhage. A tension haematoma deep to the cervical fascia is usually the result of reactionary haemorrhage from an arterial source. This results in the development of laryngeal oedema and airway compromise. The treatment is by urgent wound decompression of all layers prior to return to theatre for haemostasis. Sub cutnaeous haematomas and seromas may accumulate under skin flaps and can often be managed conservatively or by simple aspiration.

In patients with longstanding large goitres there is the risk of tracheomalacia. This is characterised by the development of flacidity of the tracheal cartilage. This can result in airway compromise. In the normal situation the tracheal diameter increases slightly during inspiration and narrows during expiration. In tracheomalacia these processes are exaggerated and the trachea may collapse in expiration resulting in stridor. In the immediate situation an endotracheal tube will need to be inserted.

Recurrent laryngeal nerve injury is recognised following thyroid surgery and may be unilateral or bilateral depending upon the procedure performed. The risk is 1.8% at one month which declines to 0.5% at three months following first time explorations. It is rare for nerve injury alone to result in airway compromise. However, it may occur when nerve injury is associated with minor degrees of laryngeal oedema (such as following intubation).

Hypoparathyroidism is a recognised complication following thyroid surgery and damage to the blood supply to the parathyroid glands is probably the commonest cause. The incidence of permanent hypoparathyroidism is in the region of 1 to 3%. It can present dramatically in the first 2-5 post operative days. In the emergency setting treatment is with intravenous calcium gluconate. Oral calcium carbonate is used in the longer term.

1581
Q

An 18 year old female presents with 3 nodules in the right lobe of the thyroid. Clinically she is euthyroid and there is associated cervical lymphadenopathy. She has no family history of thyroid disease. What is the most likely cause?

Follicular thyroid cancer

Follicular thyroid adenoma

Papillary thyroid cancer

Hashimotos thyroiditis

Toxic nodular goitre

A

Papillary thyroid cancers are the most common type of thyroid cancer and are the more common in females (M:F=1:3). Papillary tumours are more likely to develop lymphatic spread than follicular tumours.

Thyroid disease

Patients may present with a number of different manifestations of thyroid disease. They can be broadly sub classified according to whether they are euthyroid or have clinical signs of thyroid dysfunction. In addition it needs to be established whether they have a mass or not.

Assessment
History
Examination including USS
If a nodule is identified then it should be sampled ideally via an image guided fine needle aspiration
Radionucleotide scanning is of limited use

Thyroid Tumours
Papillary carcinoma
Follicular carcinoma
Anaplastic carcinoma
Medullary carcinoma
Lymphoma’s

Multinodular goitre
One of the most common reasons for presentation
Provided the patient is euthyroid and asymptomatic and no discrete nodules are seen, they can be reassured.
In those with compressive symptoms surgery is required and the best operation is a total thyroidectomy.
Sub total resections were practised in the past and simply result in recurrent disease that requires a difficult revisional resection.

Endocrine dysfunction
In general these patients are managed by physicians initially.
Surgery may be offered alongside radio iodine for patients with Graves disease that fails with medical management or in patients who would prefer not to be irradiated (e.g. pregnant women).
Patients with hypothyroidism do not generally get offered a thyroidectomy. Sometimes people inadvertently get offered resections during the early phase of Hashimotos thyroiditis, however, with time the toxic phase passes and patients can simply be managed with thyroxine.

Complications following surgery
Anatomical such as recurrent laryngeal nerve damage.
Bleeding. Owing to the confined space haematoma’s may rapidly lead to respiratory compromise owing to laryngeal oedema.
Damage to the parathyroid glands resulting in hypocalcaemia.

Further sources of information
1. http://www.acb.org.uk/docs/TFTguidelinefinal.pdf- Association of Clinical Biochemistry guidelines for thyroid function tests.

  1. British association of endocrine surgeons website- http://www.baets.org.uk
1582
Q

What is the mechanism of action of ciprofloxacin?

Inhibition of DNA gyrase

Direct injury to the bacterial cell wall

Osmotic damage to the cell

Inhibition of reverse transcriptase

Destruction of bacterial aquaporin proteins

A

Antibiotics: mechanism of action

The lists below summarise the site of action of the commonly used antibiotics

Inhibit cell wall formation
penicillins
cephalosporins

Inhibit protein synthesis
aminoglycosides (cause misreading of mRNA)
chloramphenicol
macrolides (e.g. erythromycin)
tetracyclines
fusidic acid

Inhibit DNA synthesis
quinolones (e.g. ciprofloxacin)
metronidazole
sulphonamides
trimethoprim

Inhibit RNA synthesis
rifampicin

1583
Q

A 54-year-old man is brought to the Emergency Department after being found collapsed in the street. He is known to have a history of alcoholic liver disease. Blood tests reveal the following:

Calcium 1.62 mmol/l
Albumin 33 g/l

Which one of the following is the most appropriate management of the calcium result?

10ml of 10% calcium carbonate over 10 minutes

20% albumin infusion

10ml of 10% calcium gluconate over 10 minutes

No action

10ml of 10% calcium chloride over 4 hours

A

Even after correction for the low albumin level this patient has significant hypocalcaemia which should be corrected. Both calcium chloride and gluconate can be used. Currently, gluconate is the favored agent.

Hypocalcaemia: causes and management

The clinical history combined with parathyroid hormone levels will reveal the cause of hypocalcaemia in the majority of cases

Causes
Vitamin D deficiency (osteomalacia)
Acute pancreatitis
Chronic renal failure
Hypoparathyroidism (e.g. post thyroid/parathyroid surgery)
Pseudohypoparathyroidism (target cells insensitive to PTH)
Rhabdomyolysis (initial stages)
Magnesium deficiency (due to end organ PTH resistance)

Management
Acute management of severe hypocalcaemia is with intravenous replacement. The preferred method is with intravenous calcium gluconate, 10ml of 10% solution over 10 minutes
ECG monitoring is recommended
Further management depends on the underlying cause
Calcium and bicarbonate should not be administered via the same route

1584
Q

A 3 day old baby develops dyspneoa. A chest x-ray is performed and shows a radio-opaque shadow with an air-fluid level in the chest. It is located immediately anterior to the 6th hemivertebra. Which of the following is the most likely underlying diagnosis?

Bronchogenic cyst

Congenital diaphragmatic hernia

Infection with Staphylococcus aureus

Oesphageal duplication cyst

Hiatus hernia

A

A midline cystic mass of an infant in this age group is most likely to be a bronchogenic cyst. Hiatus hernia is unusual in the neonatal period. Oesophageal duplication cysts are very rare and respiratory symptoms are less common than with bronchogenic cysts. Midline congenital hernias are rare and would usually not include an air/ fluid level in the rare event that they occur at this site.

Bronchogenic cysts

Overview
Bronchogenic cysts most commonly arise as a result of anomalous development of the ventral foregut. They are most commonly single, although multiple cysts are described.

They often lie near the midline and most frequently occur in the region of the carina. They may be attached to the tracheobronchial tree, although they are seldom in direct connection with it.

Cases may be asymptomatic or present with respiratory symptoms early in the neonatal period.

They are the second most common type of foregut cysts (after enterogenous cysts) in the middle mediastinum. Up to 50% of cases are diagnosed prior to 15 years of age.

Investigation
Many cases are diagnosed on antenatal ultrasound. Others may be detected on conventional chest radiography as a midline spherical mass or cystic structure. Once the diagnosis is suspected a CT scan should be performed.

Treatment
Thorascopic resection is the ideal treatment. Very young babies can be operated on once they reach six weeks of age.

1585
Q

In which of the following operations are Skew flaps created?

Hindquarter amputation

Above knee amputation

Gritti- Stokes amputation

Below knee amputation

Symes amputation

A

This is one variant of a below knee amputation. The Burgess flap is the other commonly practised approach.

Amputations

Amputations are indicated when the affected limb is one of the following:
Dead non viable
Deadly where it is posing a major threat to life
Dead useless where it is viable but a prosthesis would be preferable

Orthopaedic surgery
Amputation is often undertaken as an option of last resort e.g. Limb salvage has failed and the limb is so non functional that mobility needs would be best met with prosthesis.
Chronic fracture non union or significant limb shortening following trauma would fit into this category. Occasionally following major trauma a primary amputation is preferable. This would be the case in an open fracture with major distal neurovascular compromise and other more life threatening injuries are present.

Vascular surgery
The first two categories are the most prevalent.
Diabetic foot sepsis is often a major cause of sepsis which can spread rapidly in the presence of established peripheral vascular disease.
As a general rule the main issue in vascular surgery is to optimise vascular inflow prior to surgery. The more distal the planned amputation is to be, the more important this rule becomes.
In other situations there has been something such as an embolic event that has not been revascularised in time. In this case the limb shows fixed mottling and an amputation will be needed.

Types of amputations
As the vast majority of commonly performed amputations affect the lower limbs these will be covered here.

The main categories of amputations are:
Pelvic disarticulation (hindquarter)
Above knee amputation
Gritti Stokes (through knee amputation)
Below knee amputation (using either Skew or Burgess flaps)
Syme’s amputation (through ankle)
Amputations of mid foot and digits

Choosing a level of amputation depends on:
The disease process being treated
Desired functional outcome
Co-morbidities of the patient

Above knee amputations
Quick to perform
Heal reliably
Patients regain their general health quickly
For this benefit, a functional price has to be paid and many patients over the age of 70 will never walk on an above knee prosthesis.
Above knee amputations use equal anterior-posterior flaps

Below knee amputations
Technically more challenging to perform
Heal less reliably than their above knee counterparts.
However, many more patients are able to walk using a below knee prosthesis.
In below knee amputations the two main flaps are Skew flaps or the Burgess long posterior flap. Skew flaps result in a less bulky limb that is easier to attach a prosthesis to.

It is worth remembering that whilst it may be technically feasible to offer a below knee amputation there may be circumstances where an above knee option is preferable. For example, in fixed flexion deformities of the lower limb, little functional benefit would be gained from below knee amputation surgery.

1586
Q

A 24 year old lady presents with a history of severe epigastric pain that is worse post prandially. On examination, the abdomen is soft and non tender with no palpable masses, there is a bruit in the epigastrium. Imaging with USS shows no gallstones and an OGD is normal. What is the most likely diagnosis?

Sphincter of oddi dysfunction

Irritable bowel syndrome

Median arcuate ligament syndrome

Mesenteric vein thrombosis

GORD

A

Median arcuate ligament syndrome is largely a diagnosis of exclusion. The classic signs of epigastric pain with an audible bruit are only found in a minority. Where the condition is suspected, the diagnosis is usually apparent on duplex scanning (in thin patients) or with CT angiography.

Mesenteric vessel disease

Mesenteric ischaemia accounts for 1 in 1000 acute surgical admissions. It is primarily caused by arterial embolism resulting in infarction of the colon. It is more likely to occur in areas such as the splenic flexure that are located at the borders of the territory supplied by the superior and inferior mesenteric arteries.

Types
Acute mesenteric embolus (commonest 50%)
Sudden onset abdominal pain followed by profuse diarrhoea.
May be associated with vomiting.
Rapid clinical deterioration.
Serological tests: WCC, lactate, amylase may all be abnormal particularly in established disease. These can be normal in the early phases.
Acute on chronic mesenteric ischaemia
Usually longer prodromal history.
Post prandial abdominal discomfort and weight loss are dominant features. Patients will usually present with an acute on chronic event, but otherwise will tend not to present until mesenteric flow is reduced by greater than 80%.
When acute thrombosis occurs presentation may be as above. In the chronic setting the symptoms will often be those of ischaemic colitis (mucosa is the most sensitive area to this insult).
Mesenteric vein thrombosis
Usually a history over weeks.
Overt abdominal signs and symptoms will not occur until venous thrombosis has reached a stage to compromise arterial inflow.
Thrombophilia accounts for 60% of cases.
Low flow mesenteric infarction
This occurs in patients with multiple co morbidities in whom mesenteric perfusion is significantly compromised by overuse of inotropes or background cardiovascular compromise.
The end result is that the bowel is not adequately perfused and infarcts occur from the mucosa outwards.

Diagnosis
Serological tests: WCC, lactate, CRP, amylase (can be normal in early disease).
Cornerstone for diagnosis of arterial AND venous mesenteric disease is CT angiography scanning in the arterial phase with thin slices (<5mm). Venous phase contrast is not helpful.
SMA duplex USS is useful in the evaluation of proximal SMA disease in patients with chronic mesenteric ischaemia.
MRI is of limited use due to gut peristalsis and movement artefact.

Management
Overt signs of peritonism: Laparotomy
Mesenteric vein thrombosis: If no peritonism: Medical management with IV heparin
At operation limited resection of frankly necrotic bowel with view to relook laparotomy at 24-48h. In the interim urgent bowel revascularisation via endovascular (preferred) or surgery.

Prognosis
Overall poor. Best outlook is from an acute ischaemia from an embolic event where surgery occurs within 12h. Survival may be 50%. This falls to 30% with treatment delay. The other conditions carry worse survival figures.

1587
Q

A 65 year old man presents with expressive dysphasia and right sided weakness over 4 hours. His symptoms have now completely resolved. An MRI scan of his head is normal. What is the most useful investigation?

Arch aortogram

Arterial duplex scan

24 hour cardiac tape

Implantation of a reveal device

MRI brain

A

This patient has had a transient ischaemic attack. He urgently needs carotid duplex scanning to assess if he needs a carotid endarterectomy.

Vascular investigations

Venous disease

Venous Doppler
The simplest investigation for assessment of venous junctional incompetence is a Doppler assessment. This involves the patient standing and manual compression of the limb distal to the junction of interest. Flow should normally occur in one direction only. Where junctional incompetence is present reverse flow will occur and is relatively easy to identify.

Venograms and duplex scans
Structural venous information is historically obtained using a venogram. This is an invasive test and rarely required in modern clinical practice. The most helpful test is a venous duplex scan which will provide information relating to flow and vessel characteristics. Duplex is also useful in providing vein maps for bypass surgery.

Arterial disease

Ankle-brachial pressure
The ankle brachial pressure index measurement is an important investigation as it will allow classification of the severity of the flow compromise present. False readings may occur in those with calcified vessels such as diabetics and results in such settings should be interpreted with caution. When auscultating the vessel note should be made of the character of the signal. Monophasic signals are associated with a proximal stenosis and reduction in flow. Triphasic signals provide reassurance of a healthy vessel.

Arterial Duplex
As with the vein the duplex scan can provide a substantial amount of information about arterial patency and flow patterns. In skilled hands they can provide insight as to the state of proximal vessels that are anatomically inaccessible to duplex (e.g. Iliacs). Through assessment of distal flow patterns. It is an operator dependent test.

Conventional angiogram
Vessel puncture and catheter angiography is the gold standard method of assessing arteries. High quality information can usually be obtained. Limitations of the technique include the risk of contrast toxicity and risks of vessel damage. Severely calcified vessels may be difficult to puncture and in this situation a remote access site (e.g. brachial) may be used. This technique is particularly useful in providing a distal arterial roadmap prior to femoro-distal bypass.

CT angiography
These tests provide a considerable amount of structural and flow information. They require contrast and thus carry the risks associated with this. They are particularly useful in the setting of GI bleeding as they are rapidly available and can be performed by a non vascular radiologist. However, they lack the facility for endovascular intervention. In general they do not provide high enough resolution for distal arterial surgery.

Magnetic resonance angiography
This has the advantage of being non-invasive and not using nephrotoxic contrast. Movement artifact remains a problem in some sites and distal arterial resolution is imperfect.

1588
Q

A 43 year old man undergoes a live donor renal transplant. The donor’s right kidney is anastomosed to the recipient. On removal of the arterial clamps there is good urinary flow noted and the wounds are closed. On return to the ward the nurses notice that the patient suddenly becomes anuric and irrigation of the bladder does not improve the situation. What is the most likely cause?

Renal artery thrombosis

Acute rejection

Blocked catheter

Renal vein thrombosis

Chronic rejection

A

Right sided live donor transplants are extremely rare. This is because the vena cava precludes mobilisation of the right renal artery. The short right renal artery that is produced therefore presents a major challenge. The sudden cessation of urine output in this context is highly suggestive of an acute thrombosis. Delay in thrombectomy beyond 1 hour almost inevitably results in graft loss.

Complications following renal transplant

Renal transplantation is widely practised. The commonest technical related complications are related to the ureteric anastomosis. The warm ischaemic time is also of considerable importance and graft survival is directly related to this. Long warm ischaemic times increase the risk of acute tubular necrosis which may occur in all types of renal transplanation and provided other insults are minimised, will usually recover. Organ rejection may occur at any phase following the transplantation process.

Immunological complications
Types of organ rejection
Hyperacute. This occurs immediately through presence of pre formed antibody (such as ABO incompatibility).
Acute. Occurs during the first 6 months and is usually T cell mediated. Usually tissue infiltrates and vascular lesions.
Chronic. Occurs after the first 6 months. Vascular changes predominate.

Hyperacute
Renal transplants are most susceptible to this process. Risk factors include major HLA mismatch and ABO incompatibility. The rejection occurs almost immediately and the macroscopic features may become manifest following completion of the vascular anastomosis and removal of clamps. The kidney becomes mottled, dusky and the vessels will thrombose. The only treatment is removal of the graft, if left in situ it will result in abscess formation.

Acute
All organs may undergo acute rejection. Mononuclear cell infiltrates predominate. All types of transplanted organ are susceptible and it may occur in up to 50% cases. Most cases can be managed medically.

Chronic
Again all transplants with HLA mismatch may suffer this fate. Previous acute rejections and other immunosensitising events all increase the risk. Vascular changes are most prominent with myointimal proliferation leading to organ ischaemia. Organ specific changes are also seen such as loss of acinar cells in pancreas transplants and rapidly progressive coronary artery disease in cardiac transplants.

Technical complications
Complication Presenting features Treatment
Renal artery thrombosis Sudden complete loss of urine output Immediate surgery may salvage the graft, delays beyond 30 minutes are associated with a high rate of graft loss
Renal artery stenosis Uncontrolled hypertension, allograft dysfunction and oedema Angioplasty is the treatment of choice
Renal vein thrombosis Pain and swelling over the graft site, haematuria and oliguria The graft is usually lost
Urine leaks Diminished urine output, rising creatinine, fever and abdominal pain USS shows perigraft collection, necrosis of ureter tip is the commonest cause and the anastomosis may need revision
Lymphocele Common complication (occurs in 15%), may present as a mass, if large may compress ureter May be drained with percutaneous technique and sclerotherapy, or intraperitoneal drainage

1589
Q

A 63 year old man is admitted with a hematemesis. An upper GI endoscopy is performed and an ulcer is seen at the greater curvature of the stomach that is actively bleeding. What vessel is most likely to be involved?

Gastroduodenal artery

Gastroepiploic artery

Short gastric artery

Left gastric artery

Pancreaticoduodenal artery

A

The greater curvature of the stomach is closely related to the gastroepiploic artery. The short gastric vessels are closely related to the fundus.

Upper gastrointestinal bleeding

Patients may present with the following:
Haematemesis and/ or malaena
Epigastric discomfort
Sudden collapse

The extent to which these will occur will depend upon the source. Mortality is higher in patients presenting with haematemesis than malaena alone.

Oesophageal bleeding
Cause Presenting features
Oesophagitis Small volume of fresh blood, often streaking vomit. Malaena rare. Often ceases spontaneously. Usually history of antecedent GORD type symptoms.
Cancer Usually small volume of blood, except as pre terminal event with erosion of major vessels. Often associated symptoms of dysphagia and constitutional symptoms such as weight loss. May be recurrent until malignancy managed.
Mallory Weiss Tear Typically brisk small to moderate volume of bright red blood following bout of repeated vomiting. Malaena rare. Usually ceases spontaneously.
Varices Usually large volume of fresh blood. Swallowed blood may cause malaena. Often associated with haemodynamic compromise. May stop spontaneously but re-bleeds are common until appropriately managed.

Gastric Bleeding
Cause Presenting features
Gastric cancer May be frank haematemesis or altered blood mixed with vomit. Usually prodromal features of dyspepsia and may have constitutional symptoms. Amount of bleeding variable but erosion of major vessel may produce considerable haemorrhage.
Dieulafoy Lesion Often no prodromal features prior to haematemesis and malaena, but this arteriovenous malformation may produce quite considerable haemorrhage and may be difficult to detect endoscopically.
Diffuse erosive gastritis Usually haematemesis and epigastric discomfort. Usually there is an underlying cause such as recent NSAID usage. Large volume haemorrhage may occur with considerable haemodynamic compromise.
Gastric ulcer Small low volume bleeds more common so would tend to present as iron deficiency anaemia. Erosion into a significant vessel may produce considerable haemorrhage and haematemesis.

Duodenum
Most common cause of major haemorrhage is a posteriorly sited duodenal ulcer. However, ulcers at any site in the duodenum may present with haematemesis, malaena and epigastric discomfort. The pain of duodenal ulcer is slightly different to that of gastric ulcers and often occurs several hours after eating. Peri ampullary tumours may bleed but these are rare. In patients with previous abdominal aortic aneurysm surgery aorto-enteric fistulation remains a rare but important cause of major haemorrhage associated with high mortality.

Management
Admission to hospital careful monitoring, cross match blood, check FBC, LFTs, U+E and Clotting (as a minimum)
Patients with on-going bleeding and haemodynamic instability are likely to require O negative blood pending cross matched blood
Early control of airway is vital (e.g. Drowsy patient with liver failure)
Patients with suspected varices should receive terlipressin prior to endoscopy
Ideally all patients admitted with upper gastrointestinal haemorrhage should undergo Upper GI endoscopy within 24 hours of admission. In those who are unstable this should occur immediately after resuscitation or in tandem with it. The endoscopy department is a potentially dangerous place for unstable patients and it may be safer to perform the endoscopy in theatre with an anaesthetist present.
Varices should be banded or subjected to sclerotherapy. If this is not possible owing to active bleeding then a Sengstaken- Blakemore tube (or Minnesota tube) should be inserted. This should be done with care; gastric balloon should be inflated first and oesophageal balloon second. Remember the balloon will need deflating after 12 hours (ideally sooner) to prevent necrosis. Portal pressure should be lowered by combination of medical therapy +/- TIPSS.
Patients with erosive oesophagitis / gastritis should receive a proton pump inhibitor.
Mallory Weiss tears will typically resolve spontaneously
Identifiable bleeding points should receive combination therapy of injection of adrenaline and either a thermal or mechanical treatment. All who have received intervention should receive a continuous infusion of a proton pump inhibitor (IV omeprazole for 72 hours) to reduce the re-bleeding rate.
Patients with diffuse erosive gastritis who cannot be managed endoscopically and continue to bleed may require gastrectomy
Bleeding ulcers that cannot be controlled endoscopically may require laparotomy and ulcer underruning

Indications for surgery
Patients > 60 years
Continued bleeding despite endoscopic intervention
Recurrent bleeding
Known cardiovascular disease with poor response to hypotension

Surgery
Duodenal ulcer
Laparotomy, duodenotomy and under running of the ulcer. If bleeding is brisk then the ulcer is almost always posteriorly sited and will have invaded the gastroduodenal artery. Large bites using 0 Vicryl are taken above and below the ulcer base to occlude the vessel. The duodenotomy should be longitudinal but closed transversely to avoid stenosis.

For gastric ulcer
Under-running of the bleeding site
Partial gastrectomy-antral ulcer
Partial gastrectomy or under running the ulcer- lesser curve ulcer (involving left gastric artery)
Total gastrectomy if bleeding persists

Summary of Acute Upper GI bleeding recommendations:
The need for admission and timing of endoscopic intervention may be predicted by using the Blatchford score. This considers a patients Hb, serum urea, pulse rate and blood pressure. Those patients with a score of 0 are low risk, all others are considered high risk and require admission and endoscopy.
The requirement for pre endoscopic proton pump inhibition is contentious. In the UK the National Institute of Clinical Excellence guidelines suggest the pre endoscopic PPI therapy is unnecessary. Whilst it is accepted that such treatment has no impact on mortality or morbidity a Cochrane review of this practice in 2007 did suggest that it reduced the stigmata of recent haemorrhage at endoscopy. As a result many will still administer PPI to patients prior to endoscopic intervention.
Following endoscopy it is important to calculate the Rockall score for patients to determine their risk of rebleeding and mortality. A score of 3 or less is associated with a rebleeding rate of 4% and a very low risk of mortality and identifies a group of patients suitable for early discharge.

References
1. http://www.sign.ac.uk/guidelines/fulltext/105/index.html
2. Joint Advisory Group on Endoscopy (JAG) Guidelines - http://www.thejag.org.uk
3. NICE Guideline: Management of acute upper GI bleeding. July 2012.

1590
Q

A 13 year old boy falls onto an outstretched hand and is brought to the emergency department. He is examined by a doctor and a bony injury is cleared clinically. He re-presents a week later with pain in his hand. What is the most likely underlying injury?

Fracture of the distal radius

Fracture of the scaphoid

Dislocation of the lunate

Rupture of flexor pollicis longus tendon

Bennett’s fracture

A

Scaphoid fractures in children are rare, will usually involve the distal pole and are easily missed. The initial clinical examination (and sometimes x-rays) may be normal and repeated clinical examination and imaging is advised for this reason. Whilst the other injuries may be sustained from a fall onto an outstretched hand they are less likely to be overlooked on clinical examination. In the case of a Bennetts fracture, the injury mechanism is less compatible with this type of injury.

Scaphoid fractures

  • Incidence of scaphoid fractures in UK ranges from 12.4 per 100,000 to 29 per 100,000
    Surface of scaphoid is covered by articular cartilage with small area available for blood vessels (fracture risks blood supply)
    Forms floor of anatomical snuffbox
    Risk of fracture associated with fall onto outstretched hand (tubercle, waist, or proximal third)
    A series of 4 scaphoid radiographs should be undertaken (PA, pronated oblique, Ziter view and lateral view). The Ziter view is a PA view with the wrist in ulnar deviation and beam angulated at 20 degrees
    Sensitivity of scaphoid radiographs in 1st week of injury is 80%
    Immobilization of scaphoid fractures difficult
    Repeat imaging should be done at 10 days. MRI should be done in cases of diagnostic uncertainty

Classification of scaphoid fractures
Scaphoid tubercle
Distal pole
Waist
Proximal pole

Management
Undisplaced fractures of the waist of the scaphoid and most distal pole fractures can be managed in a cast for 6 weeks with high rates of union.
Displaced scaphoid waist fractures (more than 1-2mm) should be viewed as unstable and surgically fixed.
All proximal pole fractures should be fixed surgically.

Complications
Non union of scaphoid
Avascular necrosis of the scaphoid
Scapholunate disruption and wrist collapse
Degenerative changes of the adjacent joint

Reference
Berber O et al. Fractures of the scaphoid. BMJ 2020 (369): 414-416.

1591
Q

A 72 year old man develops a hydrocele which is being surgically managed. As part of the procedure the surgeons divide the tunica vaginalis. From which of the following is this structure derived?

Peritoneum

External oblique aponeurosis

Internal oblique aponeurosis

Transversalis fascia

Rectus sheath

A

The tunica vaginalis is derived from peritoneum, it secretes the fluid that fills the hydrocele cavity.

Scrotal and testicular anatomy

Spermatic cord
Formed by the vas deferens and is covered by the following structures:
Layer Origin
Internal spermatic fascia Transversalis fascia
Cremasteric fascia From the fascial coverings of internal oblique
External spermatic fascia External oblique aponeurosis

Contents of the cord
Vas deferens Transmits sperm and accessory gland secretions
Testicular artery Branch of abdominal aorta supplies testis and epididymis
Artery of vas deferens Arises from inferior vesical artery
Cremasteric artery Arises from inferior epigastric artery
Pampiniform plexus Venous plexus, drains into right or left testicular vein
Sympathetic nerve fibres Lie on arteries, the parasympathetic fibres lie on the vas
Genital branch of the genitofemoral nerve Supplies cremaster
Lymphatic vessels Drain to lumbar and para-aortic nodes

Scrotum
Composed of skin and closely attached dartos fascia.
Arterial supply from the anterior and posterior scrotal arteries
Lymphatic drainage to the inguinal lymph nodes
Parietal layer of the tunica vaginalis is the innermost layer

Testes
The testes are surrounded by the tunica vaginalis (closed peritoneal sac). The parietal layer of the tunica vaginalis adjacent to the internal spermatic fascia.
The testicular arteries arise from the aorta immediately inferiorly to the renal arteries.
The pampiniform plexus drains into the testicular veins, the left drains into the left renal vein and the right into the inferior vena cava.
Lymphatic drainage is to the para-aortic nodes.

1592
Q

A 56 year old man is having a long venous line inserted via the femoral vein into the right atrium for CVP measurements. The catheter is advanced through the IVC. At which of the following levels does this vessel enter the thorax?

L2

T10

L1

T8

T6

A

The IVC passes through the diaphragm at T8.

Inferior vena cava

Origin
L5

Path
Left and right common iliac veins merge to form the IVC.
Passes right of midline
Paired segmental lumbar veins drain into the IVC throughout its length
The right gonadal vein empties directly into the cava and the left gonadal vein generally empties into the left renal vein.
The next major veins are the renal veins and the hepatic veins
Pierces the central tendon of diaphragm at T8
Right atrium

Relations
Anteriorly Small bowel, first and third part of duodenum, head of pancreas, liver and bile duct, right common iliac artery, right gonadal artery
Posteriorly Right renal artery, right psoas, right sympathetic chain, coeliac ganglion

Levels
Level Vein
T8 Hepatic vein, inferior phrenic vein, pierces diaphragm
L1 Right suprarenal vein, renal vein
L2 Gonadal vein
L1-5 Lumbar veins
L5 Common iliac vein, formation of IVC

1593
Q

A 22 year old man is involved in a fight and is stabbed in the posterior aspect of his right leg. The knife passes into the popliteal fossa. He sustains an injury to his tibial nerve. Which muscle is least likely to be compromised as a result?

Tibialis posterior

Flexor hallucis longus

Flexor digitorum brevis

Soleus

Peroneus tertius

A

Peroneus tertius is innervated by the deep peroneal nerve.

Tibial nerve

Begins at the upper border of the popliteal fossa and is a branch of the sciatic nerve.

Root values: L4, L5, S1, S2, S3

Muscles innervated
Popliteus
Gastrocnemius
Soleus
Plantaris
Tibialis posterior
Flexor hallucis longus
Flexor digitorum brevis and longus

Terminates by dividing into the medial and lateral plantar nerves.

1594
Q

A 43 year old lady has undergone a total thyroidectomy for multinodular goitre. You are called to see her because of respiratory distress. On examination she has a marked stridor, her wound seems healthy but there is a swelling within the operative site. What is the most likely explanation for this problem?

Bilateral superior laryngeal nerve injury

Hypocalcaemic tetany

Anxiety

Contained haematoma

Unilateral recurrent laryngeal nerve injury

A

In this setting a contained haematoma is the most likely cause. This will impair venous return resulting in laryngeal oedema and respiratory compromise.

Thyroid disease

Patients may present with a number of different manifestations of thyroid disease. They can be broadly sub classified according to whether they are euthyroid or have clinical signs of thyroid dysfunction. In addition it needs to be established whether they have a mass or not.

Assessment
History
Examination including USS
If a nodule is identified then it should be sampled ideally via an image guided fine needle aspiration
Radionucleotide scanning is of limited use

Thyroid Tumours
Papillary carcinoma
Follicular carcinoma
Anaplastic carcinoma
Medullary carcinoma
Lymphoma’s

Multinodular goitre
One of the most common reasons for presentation
Provided the patient is euthyroid and asymptomatic and no discrete nodules are seen, they can be reassured.
In those with compressive symptoms surgery is required and the best operation is a total thyroidectomy.
Sub total resections were practised in the past and simply result in recurrent disease that requires a difficult revisional resection.

Endocrine dysfunction
In general these patients are managed by physicians initially.
Surgery may be offered alongside radio iodine for patients with Graves disease that fails with medical management or in patients who would prefer not to be irradiated (e.g. pregnant women).
Patients with hypothyroidism do not generally get offered a thyroidectomy. Sometimes people inadvertently get offered resections during the early phase of Hashimotos thyroiditis, however, with time the toxic phase passes and patients can simply be managed with thyroxine.

Complications following surgery
Anatomical such as recurrent laryngeal nerve damage.
Bleeding. Owing to the confined space haematoma’s may rapidly lead to respiratory compromise owing to laryngeal oedema.
Damage to the parathyroid glands resulting in hypocalcaemia.

Further sources of information
1. http://www.acb.org.uk/docs/TFTguidelinefinal.pdf- Association of Clinical Biochemistry guidelines for thyroid function tests.

  1. British association of endocrine surgeons website- http://www.baets.org.uk
1595
Q

A patient with locally advanced pancreatic cancer has persistent back pain. Which of the following is the appropriate treatment?

Systemic steroids

Nerve block

NSAIDS

Paracetamol

Transcutaneous electric nerve stimulation

A

Pancreatic cancer can cause severe pain as a result of retroperitoneal nerve infiltration. It can be managed with chemical neurectomy/ nerve blocks.

Management of pain

World Health Organisation Analgesic Ladder
Initially peripherally acting drugs such as paracetamol or non-steroidal anti-inflammatory drugs (NSAIDs) are given.
If pain control is not achieved, the second part of the ladder is to introduce weak opioid drugs such as codeine or dextropropoxyphene together with appropriate agents to control and minimise side effects.
The final rung of the ladder is to introduce strong opioid drugs such as morphine. Analgesia from peripherally acting drugs may be additive to that from centrally-acting opioids and thus, the two are given together.

The World Federation of Societies of Anaesthesiologists (WFSA) Analgesic Ladder
For management of acute pain
Initially, the pain can be expected to be severe and may need controlling with strong analgesics in combination with local anaesthetic blocks and peripherally acting drugs.
The second rung on the postoperative pain ladder is the restoration of the use of the oral route to deliver analgesia. Strong opioids may no longer be required and adequate analgesia can be obtained by using combinations of peripherally acting agents and weak opioids.
The final step is when the pain can be controlled by peripherally acting agents alone.

Local anaesthetics
Infiltration of a wound with a long-acting local anaesthetic such as Bupivacaine
Analgesia for several hours
Further pain relief can be obtained with repeat injections or by infusions via a thin catheter
Blockade of plexuses or peripheral nerves will provide selective analgesia in those parts of the body supplied by the plexus or nerves
Can either be used to provide anaesthesia for the surgery or specifically for postoperative pain relief
Especially useful where a sympathetic block is needed to improve postoperative blood supply or where central blockade such as spinal or epidural blockade is contraindicated.

Spinal anaesthesia
Provides excellent analgesia for surgery in the lower half of the body and pain relief can last many hours after completion of the operation if long-acting drugs containing vasoconstrictors are used.

  • Side effects of spinal anaesthesia include: hypotension, sensory and motor block, nausea and urinary retention.

Epidural anaesthesia
An indwelling epidural catheter inserted. This can then be used to provide a continuous infusion of analgesic agents. It can provide excellent analgesia. They are still the preferred option following major open abdominal procedures and help prevent post operative respiratory compromise resulting from pain.

  • Disadvantages of epidurals is that they usually confine patients to bed, especially if a motor block is present. In addition an indwelling urinary catheter is required. Which may not only impair mobility but also serve as a conduit for infection. They are contraindicated in coagulopathies.

Transversus Abdominal Plane block (TAP)
In this technique an ultrasound is used to identify the correct muscle plane and local anaesthetic (usually bupivicaine) is injected. The agent diffuses in the plane and blocks many of the spinal nerves. It is an attractive technique as it provides a wide field of blockade but does not require the placement of any indwelling devices. There is no post operative motor impairment. For this reason it is the preferred technique when extensive laparoscopic abdominal procedures are performed. They will then provide analgesia immediately following surgery but as they do not confine the patient to bed, the focus on enhanced recovery can begin sooner.

-The main disadvantage is that their duration of action is limited to the half life of the local anaesthetic agent chosen. In addition some anaesthetists do not have the USS skills required to site the injections.

Patient Controlled Analgesia (PCA)

  • Patients administer their own intravenous analgesia and titrate the dose to their own end-point of pain relief using a small microprocessor - controlled pump. Morphine is the most popular drug used.

Strong Opioids

Severe pain arising from deep or visceral structures requires the use of strong opioids

Morphine
Short half life and poor bioavailability.
Metabolised in the liver and clearance is reduced in patients with liver disease, in the elderly and the debilitated
Side effects include nausea, vomiting, constipation and respiratory depression.
Tolerance may occur with repeated dosage

Pethidine
Synthetic opioid which is structurally different from morphine but which has similar actions. Has 10% potency of morphine.
Short half life and similar bioavailability and clearance to morphine.
Short duration of action and may need to be given hourly.
Pethidine has a toxic metabolite (norpethidine) which is cleared by the kidney, but which accumulates in renal failure or following frequent and prolonged doses and may lead to muscle twitching and convulsions. Extreme caution is advised if pethidine is used over a prolonged period or in patients with renal failure.

Weak opioids
Codeine: markedly less active than morphine, has predictable effects when given orally and is effective against mild to moderate pain.

Non opioid analgesics
- Mild to moderate pain.

Paracetamol
Inhibits prostaglandin synthesis.
Analgesic and antipyretic properties but little anti-inflammatory effect
It is well absorbed orally and is metabolised almost entirely in the liver
Side effects in normal dosage and is widely used for the treatment of minor pain. It causes hepatotoxicity in over dosage by overloading the normal metabolic pathways with the formation of a toxic metabolite.

NSAIDs
Analgesic and anti-inflammatory actions
Inhibition of prostaglandin synthesis by the enzyme Cyclooxygenase which catalyses the conversion of arachidonic acid to the various prostaglandins that are the chief mediators of inflammation. All NSAIDs work in the same way and thus there is no point in giving more than one at a time. .
NSAIDs are, in general, more useful for superficial pain arising from the skin, buccal mucosa, joint surfaces and bone.
Relative contraindications: history of peptic ulceration, gastrointestinal bleeding or bleeding diathesis; operations associated with high blood loss, asthma, moderate to severe renal impairment, dehydration and any history of hypersensitivity to NSAIDs or aspirin.

Neuropathic pain
National Institute of Clinical Excellence (UK) guidelines:
First line: Amitriptyline (Imipramine if cannot tolerate) or pregabalin
Second line: Amitriptyline AND pregabalin
Third line: refer to pain specialist. Give tramadol in the interim (avoid morphine)
If diabetic neuropathic pain: Duloxetine
As of 1 April 2019, pregabalin and gabapentin are Class C controlled substances (under the Misuse of Drugs Act 1971) and scheduled under the Misuse of Drugs Regulations 2001 as Schedule 3. Evaluate patients carefully for a history of drug abuse before prescribing and observe patients for development of signs of abuse and dependence (MHRA, Drug Safety Update April 2019).

References
1. http://guidance.nice.org.uk/CG173/Guidance/pdf/English
2. Lovich-Sapola J, Smith CE, Brandt CP. Post operative pain control. Surg Clin North Am. 2015 Apr;95(2):301-183. Finnerup N et al. Pharmacotherapy for neuropathic pain in adults: a systematic review and meta-analysis. Lancet Neurol. 2015 Feb;14(2):162-73.

1596
Q

A 73 year old lady presents with an ulcer overlying her medial malleolus. It is painless and has been present for 4 months. She has oedema of the lower limbs and her ABPI measures 0.9. What is the best management option?

Compression bandages

Non compression bandaging

Split thickness skin grafting

Full thickness skin grafting

Angioplasty and non compression bandages

A

This is likely to be a venous leg ulcer. These are typically managed using compression bandages. Contra indications to this technique include peripheral vascular disease (not present here).

Methods of wound closure

Method of closure Indication
Primary closure
Clean wound, usually surgically created or following minor trauma
Standard suturing methods will usually suffice
Wound heals by primary intention
Delayed primary closure
Similar methods of actual closure to primary closure
May be used in situations where primary closure is either not achievable or not advisable e.g. infection
Vacuum assisted closure
Uses negative pressure therapy to facilitate wound closure
Sponge is inserted into wound cavity and then negative pressure applied
Advantages include removal of exudate and versatility
Disadvantages include cost and risk of fistulation if used incorrectly on sites such as bowel
Split thickness skin grafts
Superficial dermis removed with Watson knife or dermatome (commonly from thigh)
Remaining epithelium regenerates from dermal appendages
Coverage may be increased by meshing
Full thickness skin grafts
Whole dermal thickness is removed
Sub dermal fat is then removed and graft placed over donor site
Better cosmesis and flexibility at recipient site
Donor site morbidity
Flaps
Viable tissue with a blood supply
May be pedicled or free
Pedicled flaps are more reliable, but limited in range
Free flaps have greater range but carry greater risk of breakdown as they require vascular anastomosis

1597
Q

A 73 year old man develops disseminated intravascular coagulation following an abdominal aortic aneurysm repair. He receives an infusion of cryoprecipitate. What is the major constituent of this infusion?

Factor VIII

Factor IX

Protein C

Protein S

Factor V

A

Cryoprecipitate

  • Blood product made from plasma
    Usually transfused as 6 unit pool
    Indications include massive haemorrhage and uncontrolled bleeding due to haemophilia

Composition
Agent Quantity
Factor VIII 100IU
Fibrinogen 250mg
von Willebrand factor Variable
Factor XIII Variable

1598
Q

A 33 year old lady develops a thunderclap headache and collapses. A CT scan shows that she has developed a subarachnoid haemorrhage. She currently has no evidence of raised intracranial pressure. Which of the following drugs should be administered?

None

Atenotol

Labetolol

Nimodipine

Mannitol

A

Nimodipine is a calcium channel blocker. It reduces cerebral vasospasm and improves outcomes. It is administered to most cases of sub arachnoid haemorrhage.

Sub arachnoid haemorrhage

Spontaneous intracranial haemorrhage
Most commonly sub arachnoid haemorrhage. It is due to intra cranial aneurysm in 85% cases. Approximately 10% of cases will have normal angiography and the cause will remain unclear. Patients with inherited connective tissue disorders are at higher risk although most cases are sporadic.
>95% cases will have headache (often thunderclap)
>15% will have coma

Investigation
CT scan for all (although as CSF blood clears the sensitivity declines)
Lumbar puncture if CT normal (very unlikely if normal)
CT angiogram to look for aneurysms.

Management
Supportive treatment, optimising BP (not too high if untreated aneurysm) and ventilation if needed.
Nimodipine reduces cerebral vasospasm and reduces poor outcomes.
Untreated patients most likely to rebleed in first 2 weeks.
Patients developing hydrocephalus will need a V-P shunt (external ventricular drain acutely).
Electrolytes require careful monitoring and hyponatraemia is common.

Treatment of aneurysm
>80% aneuryms arise from the anterior circulation
Craniotomy and clipping of aneurysm is the traditional treatment, alternatively suitable lesions may be coiled using an endovascular approach. Where both options are suitable data suggests that outcomes are better with coiling than surgery.

1599
Q

An elderly man falls and lands on his hip. On examination, his hip is tender to palpation and x-rays are taken. There are concerns that he may have an intertrochanteric fracture. What is the normal angle between the femoral neck and the femoral shaft?

90

105

80

130

180

A

The normal angle in males between the femoral head and shaft is 130o. Changes to this angle may occur as a result of disease or pathology and should be investigated.
Females have an angle that is around 110-120o

Hip joint

Head of femur articulates with acetabulum of the pelvis
Both covered by articular hyaline cartilage
The acetabulum forms at the union of the ilium, pubis, and ischium
The triradiate cartilage (Y-shaped growth plate) separates the pelvic bones
The acetabulum holds the femoral head by the acetabular labrum
Normal angle between femoral head and femoral shaft is 130o

Ligaments
Transverse ligament: joints anterior and posterior ends of the articular cartilage
Head of femur ligament (ligamentum teres): acetabular notch to the fovea. Contains arterial supply to head of femur in children.

Extracapsular ligaments
Iliofemoral ligament: inverted Y shape. Anterior iliac spine to the trochanteric line
Pubofemoral ligament: acetabulum to lesser trochanter
Ischiofemoral ligament: posterior support. Ischium to greater trochanter.

Blood supply
Medial circumflex femoral and lateral circumflex femoral arteries (Branches of profunda femoris). Also from the inferior gluteal artery. These form an anastomosis and travel to up the femoral neck to supply the head.

1600
Q

Which of the following is not a content of the anterior triangle of the neck?

Ansa cervicalis

Vagus nerve

Anterior jugular vein

Transverse cervical nerve

Hypoglossal nerve

A

The transverse cervical nerve lies within the posterior triangle (the transverse cervical nerve divides into superior and inferior branches of the anterior margin of SCM). The anterior jugular vein is formed in the submental region and descends in the superficial fascia near the median plane. It passes inferior to enter the suprasternal space, it is linked to the contralateral anterior jugular vein by the jugular venous arch.

Anterior triangle of the neck

Boundaries
Anterior border of the Sternocleidomastoid
Lower border of mandible
Anterior midline

Sub triangles (divided by Digastric above and Omohyoid)
Muscular triangle: Neck strap muscles
Carotid triangle: Carotid sheath
Submandibular Triangle (digastric)

Contents of the anterior triangle
Digastric triangle Submandibular gland
Submandibular nodes
Facial vessels
Hypoglossal nerve
Muscular triangle Strap muscles
Jugular vein
Carotid triangle Carotid sheath (Common carotid, vagus and internal jugular vein)
Ansa cervicalis

Nerve supply to digastric muscle
Anterior: Mylohyoid nerve
Posterior: Facial nerve

1601
Q

A 43 year old man has suffered from small bowel Crohns disease for 15 years. Following a recent stricturoplasty he develops an enterocutaneous fistula which is high output. Small bowel follow through shows it to be 15 cm from the DJ flexure. His overlying skin is becoming excoriated. What is the best course of action?

Undertake a further laparotomy and construct a proximal diverting stoma

Commence high dose steroids

Commence TPN and octreotide

Undertake a laparotomy and resect the affected segment

Perform a small bowel bypass procedure

A

This man has a high output and anatomically high fistula. Drying up the fistula with octreotide will not suffice, his nutrition is compromised and TPN will help.

Fistulas

A fistula is defined as an abnormal connection between two epithelial surfaces.
There are many types ranging from Branchial fistulae in the neck to entero-cutaneous fistulae abdominally.
In general surgical practice the abdominal cavity generates the majority and most of these arise from diverticular disease and Crohn’s.
As a general rule all fistulae will resolve spontaneously as long as there is no distal obstruction. This is particularly true of intestinal fistulae.

The four types of fistulae are:

Enterocutaneous
These link the intestine to the skin. They may be high (>500ml) or low output (<250ml) depending upon source. Duodenal /jejunal fistulae will tend to produce high volume, electrolyte rich secretions which can lead to severe excoriation of the skin. Colo-cutaneous fistulae will tend to leak faeculent material. Both fistulae may result from the spontaneous rupture of an abscess cavity onto the skin (such as following perianal abscess drainage) or may occur as a result of iatrogenic input. In some cases it may even be surgically desirable e.g. mucous fistula following sub total colectomy for colitis.

Suspect if there is excess fluid in the drain.

Enteroenteric or Enterocolic
This is a fistula that involves the large or small intestine. They may originate in a similar manner to enterocutaneous fistulae. A particular problem with this fistula type is that bacterial overgrowth may precipitate malabsorption syndromes. This may be particularly serious in inflammatory bowel disease.

Enterovaginal
Aetiology as above.

Enterovesical
This type of fistula goes to the bladder. These fistulas may result in frequent urinary tract infections, or the passage of gas from the urethra during urination.

Management
Some rules relating to fistula management:
They will heal provided there is no underlying inflammatory bowel disease and no distal obstruction, so conservative measures may be the best option
Where there is skin involvement, protect the overlying skin, often using a well fitted stoma bag- skin damage is difficult to treat
A high output fistula may be rendered more easily managed by the use of octreotide, this will tend to reduce the volume of pancreatic secretions.
Nutritional complications are common especially with high fistula (e.g. high jejunal or duodenal) these may necessitate the use of TPN to provide nutritional support together with the concomitant use of octreotide to reduce volume and protect skin.
When managing perianal fistulae surgeons should avoid probing the fistula where acute inflammation is present, this almost always worsens outcomes.
When perianal fistulae occur secondary to Crohn’s disease the best management option is often to drain acute sepsis and maintain that drainage through the judicious use of setons whilst medical management is implemented.
Always attempt to delineate the fistula anatomy, for abscesses and fistulae that have an intra abdominal source the use of barium and CT studies should show a track. For perianal fistulae surgeons should recall Goodsall’s rule in relation to internal and external openings.

1602
Q

Which of the following will increase the volume of pancreatic exocrine secretions?

Octreotide

Cholecystokinin

Aldosterone

Adrenaline

None of the above

A

Cholecystokinin will often increase the volume of pancreatic secretions.

Pancreas exocrine physiology

Composition of pancreatic secretions
Pancreatic secretions are usually 1000-1500ml per 24 hours and have a pH of 8.
Secretion Source Substances secreted
Enzymic Acinar cells Trypsinogen
Procarboxylase
Amylase
Elastase
Aqueous Ductal and Centroacinar cells Sodium
Bicarbonate
Water
Potassium
Chloride
NB: Sodium and potassium reflect their plasma levels; chloride and bicarbonate vary with flow rate

Regulation
The cephalic and gastric phases (neuronal and physical) are less important in regulating the pancreatic secretions. The effect of digested material in the small bowel stimulates CCK release and ACh which stimulate acinar and ductal cells. Of these CCK is the most potent stimulus. In the case of the ductal cells these are potently stimulated by secretin which is released by the S cells of the duodenum. This results in an increase in bicarbonate.

Enzyme activation
Trypsinogen is converted via enterokinase to active trypsin in the duodenum. Trypsin then activates the other inactive enzymes

1603
Q

A 73 year old lady presents with severe abdominal pain and a laparotomy is performed. At operation there is widespread necrosis of the small bowel and right colon. What is the most likely explanation for this finding?

Mesenteric vein thrombosis

Acute embolism affecting the superior mesenteric artery

Acute on chronic thrombus of the superior mesenteric artery

Sub intimal dissection of the superior mesenteric artery

Proximal migration of abdominal aortic aneurysm

A

Acute embolic events account for up to 50% of cases of mesenteric infarcts. These may occur as a result of long standing atrial fibrillation, ventricular anuerysms and post myocardial infarction.

Mesenteric vessel disease

Mesenteric ischaemia accounts for 1 in 1000 acute surgical admissions. It is primarily caused by arterial embolism resulting in infarction of the colon. It is more likely to occur in areas such as the splenic flexure that are located at the borders of the territory supplied by the superior and inferior mesenteric arteries.

Types
Acute mesenteric embolus (commonest 50%)
Sudden onset abdominal pain followed by profuse diarrhoea.
May be associated with vomiting.
Rapid clinical deterioration.
Serological tests: WCC, lactate, amylase may all be abnormal particularly in established disease. These can be normal in the early phases.
Acute on chronic mesenteric ischaemia
Usually longer prodromal history.
Post prandial abdominal discomfort and weight loss are dominant features. Patients will usually present with an acute on chronic event, but otherwise will tend not to present until mesenteric flow is reduced by greater than 80%.
When acute thrombosis occurs presentation may be as above. In the chronic setting the symptoms will often be those of ischaemic colitis (mucosa is the most sensitive area to this insult).
Mesenteric vein thrombosis
Usually a history over weeks.
Overt abdominal signs and symptoms will not occur until venous thrombosis has reached a stage to compromise arterial inflow.
Thrombophilia accounts for 60% of cases.
Low flow mesenteric infarction
This occurs in patients with multiple co morbidities in whom mesenteric perfusion is significantly compromised by overuse of inotropes or background cardiovascular compromise.
The end result is that the bowel is not adequately perfused and infarcts occur from the mucosa outwards.

Diagnosis
Serological tests: WCC, lactate, CRP, amylase (can be normal in early disease).
Cornerstone for diagnosis of arterial AND venous mesenteric disease is CT angiography scanning in the arterial phase with thin slices (<5mm). Venous phase contrast is not helpful.
SMA duplex USS is useful in the evaluation of proximal SMA disease in patients with chronic mesenteric ischaemia.
MRI is of limited use due to gut peristalsis and movement artefact.

Management
Overt signs of peritonism: Laparotomy
Mesenteric vein thrombosis: If no peritonism: Medical management with IV heparin
At operation limited resection of frankly necrotic bowel with view to relook laparotomy at 24-48h. In the interim urgent bowel revascularisation via endovascular (preferred) or surgery.

Prognosis
Overall poor. Best outlook is from an acute ischaemia from an embolic event where surgery occurs within 12h. Survival may be 50%. This falls to 30% with treatment delay. The other conditions carry worse survival figures.

1604
Q

A 3 month old boy is suspected of having hypospadias. At which of the following locations is the urethral opening most frequently located in boys suffering from the condition?

On the distal ventral surface of the penis

On the proximal ventral surface of the penis

On the distal dorsal surface of the penis

On the proximal dorsal surface of the penis

At the base of the scrotum

A

The defect is located ventrally and most often distally. Proximally located urethral openings are well recognised. Circumcision may compromise reconstruction.

Hypospadias

The urethral meatus opens on the ventral surface of the penis. There is also a ventral deficiency of the foreskin. The urethral meatus may open more proximally in the more severe variants. However, 75% of the openings are distally located. The incidence is 1 in 300 male births.

Features include:

Absent frenular artery
Ventrally opened glans
Skin tethering to hypoplastic urethra
Splayed columns of spongiosum tissue distal to the meatus
Deficiency of the foreskin ventrally

Management:

No routine cultural circumcisions
Urethroplasty
Penile reconstruction

The foreskin is often utilised in the reconstructive process. In boys with very distal disease no treatment may be needed.

1605
Q

Which nerve supplies the 1st web space of the foot?

Popliteal nerve

Superficial peroneal nerve

Deep peroneal nerve

Tibial nerve

Saphenous nerve

A

The first web space is innervated by the deep peroneal nerve.

Deep peroneal nerve

Origin From the common peroneal nerve, at the lateral aspect of the fibula, deep to peroneus longus
Nerve root values L4, L5, S1, S2
Course and relation
Pierces the anterior intermuscular septum to enter the anterior compartment of the lower leg
Passes anteriorly down to the ankle joint, midway between the two malleoli
Terminates In the dorsum of the foot
Muscles innervated
Tibialis anterior
Extensor hallucis longus
Extensor digitorum longus
Peroneus tertius
Extensor digitorum brevis
Cutaneous innervation Web space of the first and second toes
Actions
Dorsiflexion of ankle joint
Extension of all toes (extensor hallucis longus and extensor digitorum longus)
Eversion of the foot

After its bifurcation past the ankle joint, the lateral branch of the deep peroneal nerve innervates the extensor digitorum brevis and the extensor hallucis brevis
The medial branch supplies the web space between the first and second digits.

1606
Q

A 53 year old man presents with an ulcerated mass at the anal verge. A biopsy is taken and the histology demonstrates as squamous cell carcinoma. Infection with which of the viruses below is most likely to have contributed to the development of the condition?

Human papillomavirus 7

Human immunodeficiency virus 1

Human immunodeficiency virus 2

Human papillomavirus 16

Human T-lymphotropic virus 1

A

Infection with human papilloma virus 16 is a risk factor for the development of intra epithelial dysplasia of the anal skin with subsequent increased risk of invasive malignancy.

Oncoviruses

Viruses which cause cancer
These may be detected on blood test and prevented by vaccine

These are the main types of oncoviruses and their diseases:

Oncovirus Cancer
Epstein-Barr virus Burkitt’s lymphoma
Hodgkin’s lymphoma
Post transplant lymphoma
Nasopharyngeal carcinoma
Human papillomavirus 16/18 Cervical cancer
Anal cancer
Penile cancer
Vulval cancer
Oropharyneal cancer
Human herpes virus 8 Kaposi’s sarcoma
Hepatitis B virus Hepatocellular carcinoma
Hepatitis C virus Hepatocellular carcinoma
Human T-lymphotropic virus 1 Tropical spastic paraparesis
Adult T cell leukaemia

1607
Q

A 23 year old woman is admitted with loin pain and a fever, she has noticed haematuria for the past week accompanied by dysuria, this was treated empirically with trimethoprim. What is the most likely cause?

Stone disease

Cystitis

Pyelonephritis

Renal cancer

Detrusor instability

A

This is most likely pyelonephritis and partially treated cystitis is a common cause.

Haematuria

Causes of haematuria

Trauma
Injury to renal tract
Renal trauma commonly due to blunt injury (others penetrating injuries)
Ureter trauma rare: iatrogenic
Bladder trauma: due to RTA or pelvic fractures
Infection
Remember TB
Malignancy
Renal cell carcinoma (remember paraneoplastic syndromes): painful or painless
Urothelial malignancies: 90% are transitional cell carcinoma, can occur anywhere along the urinary tract. Painless haematuria.
Squamous cell carcinoma and adenocarcinoma: rare bladder tumours
Prostate cancer
Penile cancers: SCC
Renal disease
Glomerulonephritis
Stones
Microscopic haematuria common
Structural abnormalities
Benign prostatic hyperplasia (BPH) causes haematuria due to hypervascularity of the prostate gland
Cystic renal lesions e.g. polycystic kidney disease
Vascular malformations
Renal vein thrombosis due to renal cell carcinoma
Coagulopathy
Causes bleeding of underlying lesions
Drugs
Cause tubular necrosis or interstitial nephritis: aminoglycosides, chemotherapy
Interstitial nephritis: penicillin, sulphonamides, and NSAIDs
Anticoagulants
Benign
Exercise
Gynaecological
Endometriosis: flank pain, dysuria, and haematuria that is cyclical
Iatrogenic
Catheterisation
Radiotherapy; cystitis, severe haemorrhage, bladder necrosis
Pseudohaematuria For example following consumption of beetroot

References
Http://bestpractice.bmj.com/best-practice/monograph/316/overview/aetiology.html

1608
Q

A 28 year old female has suffered from diffuse abdominal pain for the past 2 weeks since she was started on the contraceptive pill. The pain has increased significantly over the past 10 hours and has been associated with vomiting. A pregnancy test is negative. What is the most likely diagnosis?

Mesenteric venous thrombosis

Acute mesenteric embolus

Chronic mesenteric ischaemia

Ruptured ectopic pregnancy

Inflammatory bowel disease

A

Mesenteric venous thrombosis is the likely underlying cause and an angiogram is the sensible step as it will also facilitate the identification of areas of infarcted bowel , similar to that which may occur in the leg when massive DVT is present.

Acute abdominal pain-diagnoses

Conditions presenting with acute abdominal pain
Condition Features Investigations Management
Appendicitis History of migratory pain.
Fever.
Anorexia.
Evidence of right iliac fossa tenderness.
Mild pyrexia. Differential white cell count
Pregnancy test
C-Reactive protein
Amylase
Urine dipstick testing Appendicectomy
Mesenteric adenitis Usually recent upper respiratory tract infection.
High fever.
Generalised abdominal discomfort- true localised pain and signs are rare. Full blood count- may show slightly raised white cell count
Urine dipstick often normal
Abdominal ultrasound scan - usually no free fluid Conservative management- appendicectomy if diagnostic doubt
Mittelschmerz Only seen in females
Mid cycle pain
Usually occurs two weeks after last menstrual period
Pain usually has a supra-pubic location
Usually subsides over a 24-48 hour period. Full blood count- normal
Urine dipstick- normal
Abdominal and pelvic ultrasound- may show a trace of pelvic free fluid Manage conservatively if doubt or symptoms fail to settle then laparoscopy
Fitz-Hugh Curtis syndrome Disseminated infection with Chlamydia.
Usually seen in females.
Consists of evidence of pelvic inflammatory disease together with peri-hepatic inflammation and subsequent adhesion formation. Abdominal ultrasound scan- may show free fluid
High vaginal swabs - may show evidence of sexually transmitted infections Usually medically managed- doxycycline or azithromycin
Abdominal aortic aneurysm (ruptured) Sudden onset of abdominal pain radiating to the back in older adults (look for risk factors).
Collapse.
May be moribund on arrival in casualty, more stable if contained haematoma.
Careful clinical assessment may reveal pulsatile mass. Patients who are haemodynamically stable should have a CT scan Unstable patients should undergo immediate surgery (unless it is not in their best interests).
Those with evidence of contained leak on CT should undergo immediate surgery
Increasing unruptured aneurysmal size is an indication for urgent surgical intervention (that can wait until the next working day)
Perforated peptic ulcer Sudden onset of pain (usually epigastric).
Often preceding history of upper abdominal pain.
Soon develop generalised abdominal pain.
On examination may have clinical evidence of peritonitis. Erect CXR may show free air. A CT scan may be indicated where there is diagnostic doubt Laparotomy (laparoscopic surgery for perforated peptic ulcers is both safe and feasible in experienced hands)
Intestinal obstruction Colicky abdominal pain and vomiting (the nature of which depends on the level of the obstruction).
Abdominal distension and constipation (again depending upon site of obstruction).
Features of peritonism may occur where local necrosis of bowel loops is occurring. A plain abdominal film may help with making the diagnosis. A CT scan may be useful where diagnostic uncertainty exists In those with a virgin abdomen a lower and earlier threshold for laparotomy should exist than in those who may have adhesional obstruction
Mesenteric infarction Embolic events present with sudden pain and forceful evacuation.
Acute on chronic events usually have a longer history and previous weight loss.
On examination the pain is typically greater than the physical signs would suggest. Arterial pH and lactate
Arterial phase CT scanning is the most sensitive test Immediate laparotomy and resection of affected segments, in acute embolic events SMA embolectomy may be needed.

1609
Q

A 33 year old lady attends the clinic with a 3 month history of palpitations and irritability. Her thyroid function, PTH and calcium are measured:
Thyroid function
Free T4 40 pmol/L
TSH < 0.1 miu/L
Free T3 25 p mol/L (normal 3.5-7.7 p mol/L)
PTH 10pg/ml (normal 10-55pg/ml)
Ca 2.12 mmol/L
(Normal values listed in reference range link)
What is the most likely diagnosis?

Hypothyroidism

Hyperthyroidism

Hypoparathyroidism

Hyperparathyroidism

Euthyroid

A

B- Hyperthyroidism

Elevated T4 and suppressed TSH makes this the most likely diagnosis. The PTH level is normal.

Hyperthyroidism

Causes of hyperthyroidism include:
Diffuse toxic goitre (Graves Disease)
Toxic nodular goitre
Toxic nodule
Rare causes

Graves disease
Graves disease is characterised by a diffuse vascular goitre that appears at the same time as the clinical manifestations of hyperthyroidism. It is commonest in younger females and may be associated with eye signs. Thyrotoxic symptoms will predominate. Up to 50% of patients will have a familial history of autoimmune disorders. The glandular hypertrophy and hyperplasia occur as a result of the thyroid stimulating effects of the TSH receptor antibodies.

Toxic nodular goitre
In this disorder the goitre is present for a long period of time prior to the development of clinical symptoms. In most goitres the nodules are inactive and in some cases it is the internodular tissue that is responsible for the hyperthyroidism.

Toxic nodule
Overactive, autonomously functioning nodule. It may occur as part of generalised nodularity or be a true toxic adenoma. The TSH levels are usually low as the autonomously functioning thyroid tissue will exert a negative feedback effect.

Signs and symptoms
Symptoms Signs
Lethargy Tachycardia
Emotionally labile Agitation
Heat intolerance Hot, moist palms
Weight loss Exopthalmos
Excessive appetite Thyroid goitre and bruit
Palpitations Lid lag/retraction

Diagnosis
The most sensitive test for diagnosing hyperthyroidism is plasma T3 (which is raised). Note in hypothyroidism the plasma T4 and TSH are the most sensitive tests. A TSH level of <0.5U/L suggests hyperthyroidism. TSH receptor antibodies may be tested for in the diagnosis of Graves.

Treatment
First line treatment for Graves disease is usually medical and the block and replace regime is the favored option. Carbimazole is administered at higher doses and thyroxine is administered orally. Patients are maintained on this regime for between 6 and 12 months. Attempts are then made to wean off medication. Where relapse then occurs the options are between ongoing medical therapy, radioiodine or surgery.

1610
Q

A 56 year old male presents to the acute surgical take with severe abdominal pain. He is normally fit and well. He has no malignancy. The biochemistry laboratory contacts the ward urgently, his corrected calcium result is 3.6 mmol/l. What is the medication of choice to treat this abnormality?

IV Pamidronate

Oral Alendronate

Dexamethasone

Vitamin D

Resonium salts

A

IV Pamidronate is the drug of choice as it most effective and has long lasting effects. Calcitonin would need to be given with another agent, to ensure that the hypercalcaemia is treated once its short term effects wear off. IV zoledronate is preferred in scenarios associated with malignancy.

Management of hypercalcaemia

Free Ca is affected by pH (increased in acidosis) and plasma albumin concentration
ECG changes include: Shortening of QTc interval
Urgent management is indicated if:

Calcium > 3.5 mmol/l
Reduced consciousness
Severe abdominal pain
Pre renal failure

Management:
Airway Breathing Circulation
Intravenous fluid resuscitation with 3-6L of 0.9% Normal saline in 24 hours
Concurrent administration of calcitonin will also help lower calcium levels
Medical therapy (usually if Corrected calcium >3.0mmol/l)

Bisphosphonates
Analogues of pryrophosphate
Prevent osteoclast attachment to bone matrix and interfere with osteoclast activity
Inhibit bone resorption.

Agents

Drug Side effects Notes
IV Pamidronate pyrexia, leucopaenia Most potent agent
IV Zoledronate response lasts 30 days Used for malignancy associated hypercalcaemia

Calcitonin
Quickest onset of action however short duration (tachyphylaxis) therefore only given with a second agent.

Prednisolone
May be given in hypercalcaemia related to sarcoidosis, myeloma or vitamin D intoxication.

1611
Q

A 3 year old boy is brought to the clinic with symptoms of urinary hesitancy and poor stream. Which of the following is the most likely underlying diagnosis?

Benign prostatic hypertrophy

Posterior urethral valves

Neurogenic bladder

Urethral calculus

Hypospadias

A

Posterior urethral valves are one of the commonest causes of poor urinary stream and hesitancy in children. Prostatic disorders are rare.
Hypospadias is associated with urine that is difficult to control, but should not produce hesitancy.

Urethral valves

Posterior urethral valves are the commonest cause of infravesical outflow obstruction in males. They may be diagnosed on ante natal ultrasonography. Because the bladder has to develop high emptying pressures in utero, the child may develop renal parenchymal damage. This translates to renal impairment noted in 70% of boys at presentation. Treatment is with bladder catheterisation. Endoscopic valvotomy is the definitive treatment of choice with cystoscopic and renal follow up.

1612
Q

A 42 year old lady is investigated for symptoms of irritability and altered bowel habit. On examination she is noted to have a smooth enlargement of the thyroid gland. As part of her investigations thyroid function tests are requested, these are as follows:
TSH 0.1 mug/l
Free T4 35 pmol/l
The most likely underlying diagnosis is:

Multinodular goitre

Follicular carcinoma of the thyroid gland

Graves disease

Pregnancy

None of the above

A

TSH receptor antibodies will cause stimulation of the thyroid to synthesise T4. However, this will have a negative feedback effect on the pituitary causing decrease in TSH levels.
Where hyperthyroidism occurs secondary to pregnancy the TSH is typically elevated.

Thyroid disease

Patients may present with a number of different manifestations of thyroid disease. They can be broadly sub classified according to whether they are euthyroid or have clinical signs of thyroid dysfunction. In addition it needs to be established whether they have a mass or not.

Assessment
History
Examination including USS
If a nodule is identified then it should be sampled ideally via an image guided fine needle aspiration
Radionucleotide scanning is of limited use

Thyroid Tumours
Papillary carcinoma
Follicular carcinoma
Anaplastic carcinoma
Medullary carcinoma
Lymphoma’s

Multinodular goitre
One of the most common reasons for presentation
Provided the patient is euthyroid and asymptomatic and no discrete nodules are seen, they can be reassured.
In those with compressive symptoms surgery is required and the best operation is a total thyroidectomy.
Sub total resections were practised in the past and simply result in recurrent disease that requires a difficult revisional resection.

Endocrine dysfunction
In general these patients are managed by physicians initially.
Surgery may be offered alongside radio iodine for patients with Graves disease that fails with medical management or in patients who would prefer not to be irradiated (e.g. pregnant women).
Patients with hypothyroidism do not generally get offered a thyroidectomy. Sometimes people inadvertently get offered resections during the early phase of Hashimotos thyroiditis, however, with time the toxic phase passes and patients can simply be managed with thyroxine.

Complications following surgery
Anatomical such as recurrent laryngeal nerve damage.
Bleeding. Owing to the confined space haematoma’s may rapidly lead to respiratory compromise owing to laryngeal oedema.
Damage to the parathyroid glands resulting in hypocalcaemia.

Further sources of information
1. http://www.acb.org.uk/docs/TFTguidelinefinal.pdf- Association of Clinical Biochemistry guidelines for thyroid function tests.

  1. British association of endocrine surgeons website- http://www.baets.org.uk
1613
Q

A 74 year old lady is diagnosed as having a chronic venous leg ulcer. Which of these features either in the history or on examination would be suggestive of an alternative diagnosis?

Heaped raised borders if the ulcer has been present more than 5 years

Evidence of surrounding lipodermatosclerosis

Irregular shape to the ulcer

Previous history of deep vein thrombosis

Haemosiderin deposits in surrounding skin

A

The borders of the ulcer are often well defined even though they may be irregular. Heaped or raised borders should raise suspicion of a marjolins ulcer.

Lower leg ulcers

Venous leg ulcers
Most due to venous hypertension, secondary to chronic venous insufficiency (other causes include calf pump dysfunction or neuromuscular disorders)
Ulcers form due to capillary fibrin cuff or leucocyte sequestration
Features of venous insufficiency include oedema, brown pigmentation, lipodermatosclerosis, eczema
Location above the ankle, painless
Deep venous insufficiency is related to previous DVT and superficial venous insufficiency is associated with varicose veins
Doppler ultrasound looks for presence of reflux and duplex ultrasound looks at the anatomy/ flow of the vein
Management: 4 layer compression banding after exclusion of arterial disease or surgery
If fail to heal after 12 weeks or >10cm2 skin grafting may be needed

Marjolin’s ulcer

Squamous cell carcinoma
Occurring at sites of chronic inflammation e.g; burns, osteomyelitis after 10-20 years
Mainly occur on the lower limb

Arterial ulcers
Occur on the toes and heel
Painful
There may be areas of gangrene
Cold with no palpable pulses
Low ABPI measurements

Neuropathic ulcers
Commonly over plantar surface of metatarsal head and plantar surface of hallux
The plantar neuropathic ulcer is the condition that most commonly leads to amputation in diabetic patients
Due to pressure
Management includes cushioned shoes to reduce callus formation

Pyoderma gangrenosum

Associated with inflammatory bowel disease/RA
Can occur at stoma sites
Erythematous nodules or pustules which ulcerate

1614
Q

An 83 year old lady attends the endoscopy department for a flexible sigmoidoscopy. The endoscopist administers 7mg of intravenous midazolam for sedation. Approximately 3 minutes later the patient is noted to be hypoxic and has a respiratory rate of 5 breaths per minute. What is the most useful agent to administer at this point?

Intravenous naloxone

Oral naloxone

Intravenous flumazenil

Intravenous doxapram

Nebulised adrenaline

A

Benzodiazepine overdose with marked suppression of respiratory activity should be managed with IV flumazenil.

Midazolam

Midazolam is a short acting benzodiazepine. It is usually administered intravenously, when administered orally, dosing is less reliable. It is eliminated via the cytochrome P450 pathway in the liver. It is often used for procedural related sedation purposes. It is generally administered in doses of 2-5mg for this purpose. It is often given in combination with a short acting opiate such as fentanyl for analgesic purposes. One of the main reasons that midazolam is preferred for procedural related sedation is due to a degree of amnesia that can accompany it. If administered in excess, most patients can be managed with simple airway support measures an monitoring. However, if there is significant respiratory compromise, it can be reversed with the agent flumazenil which is a selective GABA receptor antagonist.

1615
Q

A 20 year old man is admitted to the intensive care unit with an isolated severe head injury. A CT scan shows multiple intracerebral bleeds but no midline shift. He is intubated and ventilated. His pupils are dilated and react sluggishly to light. His heart rate is 50 beats/minute blood pressure 170/110 mmHg and his respiratory rate is set at 10 breaths/minute. The rising blood pressure is likely to be caused by:

Aortic and carotid baroreceptor stimulation

Cortisol stimulation

Renin angiotensin stimulation

Sympathetic stimulation related to blood loss

Sympathetic stimulation related to increased intra cranial pressure

A

The changes seen are the result of raised ICP, its rare for head injuries to cause sufficient blood loss to affect the circulating volume.

Cushing reflex

When intra cranial pressure exceeds mean arterial pressure, compression of cerebral arterioles occurs. This results in cerebral ischaemia.
Increases in ICP results in stepwise activation of the sympathetic nervous system initially. This raises peripheral vascular resistance and results in hypertension. Cardiac output is also increased. These haemodynamic changes are detected by aortic arch baroreceptors and this results in activation of the parasympathetic nervous system. These parasympathetic effects comprise the second stage of the Cushing reflex. The Cushing reflex is a serious development that indicates imminent coning or other terminal events if not resolved quickly.

1616
Q

A 32 year old woman presents with a tender breast lump. She has a 2 month old child. Clinically, there is a tender, fluctuant mass of the breast. What is the most likely explanation for this process?

Duct ectasia

Intraductal papilloma

Breast abscess

Breast cancer

Fibroadenoma

A

This lady is likely to be breast feeding and is at risk of mastitis. This may lead to an abscess if not treated. Staphylococcus aureus is usually the causative organism.
Non malignant breast disease

Duct ectasia
Mammary duct ectasia may be seen in up to 25% of normal female breasts
Patients usually present with nipple discharge, which may be from single or multiple ducts (usually present age >50 years)
The discharge is often thick and green
Duct ectasia is a normal variant of breast involution and is not the same condition as periductal mastitis
Mass develops behind nipple

Periductal mastitis
Present at younger age than duct ectasia
May present with features of inflammation, abscess or mammary duct fistula
Strongly associated with smoking
Usually treated with antibiotics, abscess will require drainage
Mass develops around nipple

Intraductal papilloma
Growth of papilloma in a single duct
Usually presents with clear or blood stained discharge originating from a single duct
No increase in risk of malignancy

Breast abscess
Lactational mastitis is common
Infection is usually with Staphylococcus aureus
On examination there is usually a tender fluctuant mass
Treatment is with antibiotics and ultrasound guided aspiration
Overlying skin necrosis is an indication for surgical debridement, which may be complicated by the development of a subsequent mammary duct fistula.

Tuberculosis
Rare in western countries, usually secondary TB
Affects women later in child bearing period
Chronic breast or axillary sinus is present in up to 50% cases
Diagnosis is by biopsy culture and histology

1617
Q

What is the most appropriate analgesic to administer to a term neonate who is recovering following an inguinal herniotomy?

Co-codamol

Paracetamol

Ibuprofen

Carbamazepine

Codeine

A

Paracetamol is an effective analgesic in children and pain following herniotomy is relatively minor. Note that codeine is contra indicated in neonates. The child is too young to receive ibuprofen.

Management of pain

World Health Organisation Analgesic Ladder
Initially peripherally acting drugs such as paracetamol or non-steroidal anti-inflammatory drugs (NSAIDs) are given.
If pain control is not achieved, the second part of the ladder is to introduce weak opioid drugs such as codeine or dextropropoxyphene together with appropriate agents to control and minimise side effects.
The final rung of the ladder is to introduce strong opioid drugs such as morphine. Analgesia from peripherally acting drugs may be additive to that from centrally-acting opioids and thus, the two are given together.

The World Federation of Societies of Anaesthesiologists (WFSA) Analgesic Ladder
For management of acute pain
Initially, the pain can be expected to be severe and may need controlling with strong analgesics in combination with local anaesthetic blocks and peripherally acting drugs.
The second rung on the postoperative pain ladder is the restoration of the use of the oral route to deliver analgesia. Strong opioids may no longer be required and adequate analgesia can be obtained by using combinations of peripherally acting agents and weak opioids.
The final step is when the pain can be controlled by peripherally acting agents alone.

Local anaesthetics
Infiltration of a wound with a long-acting local anaesthetic such as Bupivacaine
Analgesia for several hours
Further pain relief can be obtained with repeat injections or by infusions via a thin catheter
Blockade of plexuses or peripheral nerves will provide selective analgesia in those parts of the body supplied by the plexus or nerves
Can either be used to provide anaesthesia for the surgery or specifically for postoperative pain relief
Especially useful where a sympathetic block is needed to improve postoperative blood supply or where central blockade such as spinal or epidural blockade is contraindicated.

Spinal anaesthesia
Provides excellent analgesia for surgery in the lower half of the body and pain relief can last many hours after completion of the operation if long-acting drugs containing vasoconstrictors are used.

  • Side effects of spinal anaesthesia include: hypotension, sensory and motor block, nausea and urinary retention.

Epidural anaesthesia
An indwelling epidural catheter inserted. This can then be used to provide a continuous infusion of analgesic agents. It can provide excellent analgesia. They are still the preferred option following major open abdominal procedures and help prevent post operative respiratory compromise resulting from pain.

  • Disadvantages of epidurals is that they usually confine patients to bed, especially if a motor block is present. In addition an indwelling urinary catheter is required. Which may not only impair mobility but also serve as a conduit for infection. They are contraindicated in coagulopathies.

Transversus Abdominal Plane block (TAP)
In this technique an ultrasound is used to identify the correct muscle plane and local anaesthetic (usually bupivicaine) is injected. The agent diffuses in the plane and blocks many of the spinal nerves. It is an attractive technique as it provides a wide field of blockade but does not require the placement of any indwelling devices. There is no post operative motor impairment. For this reason it is the preferred technique when extensive laparoscopic abdominal procedures are performed. They will then provide analgesia immediately following surgery but as they do not confine the patient to bed, the focus on enhanced recovery can begin sooner.

-The main disadvantage is that their duration of action is limited to the half life of the local anaesthetic agent chosen. In addition some anaesthetists do not have the USS skills required to site the injections.

Patient Controlled Analgesia (PCA)

  • Patients administer their own intravenous analgesia and titrate the dose to their own end-point of pain relief using a small microprocessor - controlled pump. Morphine is the most popular drug used.

Strong Opioids

Severe pain arising from deep or visceral structures requires the use of strong opioids

Morphine
Short half life and poor bioavailability.
Metabolised in the liver and clearance is reduced in patients with liver disease, in the elderly and the debilitated
Side effects include nausea, vomiting, constipation and respiratory depression.
Tolerance may occur with repeated dosage

Pethidine
Synthetic opioid which is structurally different from morphine but which has similar actions. Has 10% potency of morphine.
Short half life and similar bioavailability and clearance to morphine.
Short duration of action and may need to be given hourly.
Pethidine has a toxic metabolite (norpethidine) which is cleared by the kidney, but which accumulates in renal failure or following frequent and prolonged doses and may lead to muscle twitching and convulsions. Extreme caution is advised if pethidine is used over a prolonged period or in patients with renal failure.

Weak opioids
Codeine: markedly less active than morphine, has predictable effects when given orally and is effective against mild to moderate pain.

Non opioid analgesics
- Mild to moderate pain.

Paracetamol
Inhibits prostaglandin synthesis.
Analgesic and antipyretic properties but little anti-inflammatory effect
It is well absorbed orally and is metabolised almost entirely in the liver
Side effects in normal dosage and is widely used for the treatment of minor pain. It causes hepatotoxicity in over dosage by overloading the normal metabolic pathways with the formation of a toxic metabolite.

NSAIDs
Analgesic and anti-inflammatory actions
Inhibition of prostaglandin synthesis by the enzyme Cyclooxygenase which catalyses the conversion of arachidonic acid to the various prostaglandins that are the chief mediators of inflammation. All NSAIDs work in the same way and thus there is no point in giving more than one at a time. .
NSAIDs are, in general, more useful for superficial pain arising from the skin, buccal mucosa, joint surfaces and bone.
Relative contraindications: history of peptic ulceration, gastrointestinal bleeding or bleeding diathesis; operations associated with high blood loss, asthma, moderate to severe renal impairment, dehydration and any history of hypersensitivity to NSAIDs or aspirin.

Neuropathic pain
National Institute of Clinical Excellence (UK) guidelines:
First line: Amitriptyline (Imipramine if cannot tolerate) or pregabalin
Second line: Amitriptyline AND pregabalin
Third line: refer to pain specialist. Give tramadol in the interim (avoid morphine)
If diabetic neuropathic pain: Duloxetine
As of 1 April 2019, pregabalin and gabapentin are Class C controlled substances (under the Misuse of Drugs Act 1971) and scheduled under the Misuse of Drugs Regulations 2001 as Schedule 3. Evaluate patients carefully for a history of drug abuse before prescribing and observe patients for development of signs of abuse and dependence (MHRA, Drug Safety Update April 2019).

References
1. http://guidance.nice.org.uk/CG173/Guidance/pdf/English
2. Lovich-Sapola J, Smith CE, Brandt CP. Post operative pain control. Surg Clin North Am. 2015 Apr;95(2):301-183. Finnerup N et al. Pharmacotherapy for neuropathic pain in adults: a systematic review and meta-analysis. Lancet Neurol. 2015 Feb;14(2):162-73.

1618
Q

A 25 year old male pedestrian is involved in a road traffic accident. He sustains multiple injuries and is admitted to the intensive care unit, intubated and ventilated. Over the next week he develops adult respiratory distress syndrome. What is the main reason for hypoxaemia in this condition?

Increased lung compliance

Reduced diffusion

Reduced surfactant

Reduced elastase

Left to right shunt

A

The diffuse lung injury, which is associated with loss of surfactant and increased elastase release from neutrophils, results in fluid accumulation. This leads to reduced diffusion, which is the main reason for hypoxaemia.

Adult respiratory distress syndrome

Defined as an acute condition characterized by bilateral pulmonary infiltrates and severe hypoxemia (PaO2/FiO2 ratio < 200) in the absence of evidence for cardiogenic pulmonary oedema (clinically or pulmonary capillary wedge pressure of less than 18 mm Hg).
It is subdivided into two stages. Early stages consist of an exudative phase of injury with associated oedema. The later stage is one of repair and consists of fibroproliferative changes. Subsequent scarring may result in poor lung function.

Causes
Sepsis
Direct lung injury
Trauma
Acute pancreatitis
Long bone fracture or multiple fractures (through fat embolism)
Head injury (causes sympathetic nervous stimulation which leads to acute pulmonary hypertension)

Clinical features
Acute dyspnoea and hypoxaemia hours/days after event
Multi organ failure
Rising ventilatory pressures

Management
Treat the underlying cause
Antibiotics (if signs of sepsis)
Negative fluid balance i.e. Diuretics
Recruitment manoeuvres such as prone ventilation, use of positive end expiratory pressure
Mechanical ventilation strategy using low tidal volumes, as conventional tidal volumes may cause lung injury (only treatment found to improve survival rates)

1619
Q

An over enthusiastic medical student decides to ask you questions about ECGs. Rather than admitting your dwindling knowledge on this topic, you bravely attempt to answer her questions! One question is what component of the ECG represents ventricular repolarization?

QRS complex

Q-T interval

P wave

T wave

S-T segment

A

The T wave represents ventricular repolarization. The common sense approach to remembering this, is to acknowledge that ventricular repolarization is the last phase of cardiac contraction and should therefore correspond the the last part of the QRS complex.

P wave
Represents the wave of depolarization that spreads from the SA node throughout the atria
Lasts 0.08 to 0.1 seconds (80-100 ms)
The isoelectric period after the P wave represents the time in which the impulse is traveling within the AV node

P-R interval
Time from the onset of the P wave to the beginning of the QRS complex
Ranges from 0.12 to 0.20 seconds in duration
Represents the time between the onset of atrial depolarization and the onset of ventricular depolarization

QRS complex
Represents ventricular depolarization
Duration of the QRS complex is normally 0.06 to 0.1 seconds

ST segment
Isoelectric period following the QRS
Represents period which the entire ventricle is depolarized and roughly corresponds to the plateau phase of the ventricular action potential

T wave
Represents ventricular repolarization and is longer in duration than depolarization
A small positive U wave may follow the T wave which represents the last remnants of ventricular repolarization.

Q-T interval
Represents the time for both ventricular depolarization and repolarization to occur, and therefore roughly estimates the duration of an average ventricular action potential.
Interval ranges from 0.2 to 0.4 seconds depending upon heart rate.
At high heart rates, ventricular action potentials shorten in duration, which decreases the Q-T interval. Therefore the Q-T interval is expressed as a ‘corrected Q-T (QTc)’ by taking the Q-T interval and dividing it by the square root of the R-R interval (interval between ventricular depolarizations). This allows an assessment of the Q-T interval that is independent of heart rate.
Normal corrected Q-Tc interval is less than 0.44 seconds.

1620
Q

A 43 year old female presents with recurrent urinary tract infections. She describes blood and frothy urine. She is 6 weeks post operative for a left hemicolectomy for crohn’s disease. What is the most likely reason for this presentation?

Colovesical fistula

Enterovesical fistula

Entero-entero fistula

Entero-colic fistula

Vesico-cutaneous fistula

A

The commonest event here is an anastomotic leak. It is possible that the patient (who has Crohns) originally had a colovesical fistula that was then addressed with a resection. In the event that these leak, the fistula reforms.

Fistulas

A fistula is defined as an abnormal connection between two epithelial surfaces.
There are many types ranging from Branchial fistulae in the neck to entero-cutaneous fistulae abdominally.
In general surgical practice the abdominal cavity generates the majority and most of these arise from diverticular disease and Crohn’s.
As a general rule all fistulae will resolve spontaneously as long as there is no distal obstruction. This is particularly true of intestinal fistulae.

The four types of fistulae are:

Enterocutaneous
These link the intestine to the skin. They may be high (>500ml) or low output (<250ml) depending upon source. Duodenal /jejunal fistulae will tend to produce high volume, electrolyte rich secretions which can lead to severe excoriation of the skin. Colo-cutaneous fistulae will tend to leak faeculent material. Both fistulae may result from the spontaneous rupture of an abscess cavity onto the skin (such as following perianal abscess drainage) or may occur as a result of iatrogenic input. In some cases it may even be surgically desirable e.g. mucous fistula following sub total colectomy for colitis.

Suspect if there is excess fluid in the drain.

Enteroenteric or Enterocolic
This is a fistula that involves the large or small intestine. They may originate in a similar manner to enterocutaneous fistulae. A particular problem with this fistula type is that bacterial overgrowth may precipitate malabsorption syndromes. This may be particularly serious in inflammatory bowel disease.

Enterovaginal
Aetiology as above.

Enterovesical
This type of fistula goes to the bladder. These fistulas may result in frequent urinary tract infections, or the passage of gas from the urethra during urination.

Management
Some rules relating to fistula management:
They will heal provided there is no underlying inflammatory bowel disease and no distal obstruction, so conservative measures may be the best option
Where there is skin involvement, protect the overlying skin, often using a well fitted stoma bag- skin damage is difficult to treat
A high output fistula may be rendered more easily managed by the use of octreotide, this will tend to reduce the volume of pancreatic secretions.
Nutritional complications are common especially with high fistula (e.g. high jejunal or duodenal) these may necessitate the use of TPN to provide nutritional support together with the concomitant use of octreotide to reduce volume and protect skin.
When managing perianal fistulae surgeons should avoid probing the fistula where acute inflammation is present, this almost always worsens outcomes.
When perianal fistulae occur secondary to Crohn’s disease the best management option is often to drain acute sepsis and maintain that drainage through the judicious use of setons whilst medical management is implemented.
Always attempt to delineate the fistula anatomy, for abscesses and fistulae that have an intra abdominal source the use of barium and CT studies should show a track. For perianal fistulae surgeons should recall Goodsall’s rule in relation to internal and external openings.

1621
Q

A baby is brought to casualty unconscious and in a vegetative state. She has cigarette burns on her legs. What is the most likely underlying injury?

Acute sub dural haematoma

Chronic sub dural haematoma

Acute extra dural haematoma

Diffuse axonal injury

Sub arachnoid haemorrhage

A

The baby is likely to be a victim of shaken baby syndrome. This may result in diffuse axonal injury causing extensive lesions in the white matter.

Head injury

Patients who suffer head injuries should be managed according to ATLS principles and extra cranial injuries should be managed alongside cranial trauma. Inadequate cardiac output will compromise CNS perfusion irrespective of the nature of the cranial injury.

Types of traumatic brain injury
Extradural haematoma Bleeding into the space between the dura mater and the skull. Often results from acceleration-deceleration trauma or a blow to the side of the head. The majority of extradural haematomas occur in the temporal region where skull fractures cause a rupture of the middle meningeal artery.

Features
Raised intracranial pressure
Some patients may exhibit a lucid interval
Subdural haematoma Bleeding into the outermost meningeal layer. Most commonly occur around the frontal and parietal lobes. May be either acute or chronic.

Risk factors include old age and alcoholism.

Slower onset of symptoms than a extradural haematoma.
Subarachnoid haemorrhage Usually occurs spontaneously in the context of a ruptured cerebral aneurysm, but may be seen in association with other injuries when a patient has sustained a traumatic brain injury.

Pathophysiology
Primary brain injury may be focal (contusion/ haematoma) or diffuse (diffuse axonal injury)
Diffuse axonal injury occurs as a result of mechanical shearing following deceleration, causing disruption and tearing of axons
Intra-cranial haematomas can be extradural, subdural or intracerebral, while contusions may occur adjacent to (coup) or contralateral (contre-coup) to the side of impact
Secondary brain injury occurs when cerebral oedema, ischaemia, infection, tonsillar or tentorial herniation exacerbates the original injury. The normal cerebral auto regulatory processes are disrupted following trauma rendering the brain more susceptible to blood flow changes and hypoxia
The Cushings reflex (hypertension and bradycardia) often occurs late and is usually a pre terminal event

Management
Where there is life threatening rising ICP such as in extra dural haematoma and whilst theatre is prepared or transfer arranged use of IV mannitol/ frusemide may be required.
Diffuse cerebral oedema may require decompressive craniotomy
Exploratory Burr Holes have little management in modern practice except where scanning may be unavailable and to thus facilitate creation of formal craniotomy flap
Depressed skull fractures that are open require formal surgical reduction and debridement, closed injuries may be managed non operatively if there is minimal displacement.
ICP monitoring is appropriate in those who have GCS 3-8 and normal CT scan.
ICP monitoring is mandatory in those who have GCS 3-8 and abnormal CT scan.
Hyponatraemia is most likely to be due to syndrome of inappropriate ADH secretion.
Minimum of cerebral perfusion pressure of 70mmHg in adults.
Minimum cerebral perfusion pressure of between 40 and 70 mmHg in children.

Interpretation of pupillary findings in head injuries
Pupil size Light response Interpretation
Unilaterally dilated Sluggish or fixed 3rd nerve compression secondary to tentorial herniation
Bilaterally dilated Sluggish or fixed
Poor CNS perfusion
Bilateral 3rd nerve palsy
Unilaterally dilated or equal Cross reactive (Marcus - Gunn) Optic nerve injury
Bilaterally constricted May be difficult to assess
Opiates
Pontine lesions
Metabolic encephalopathy
Unilaterally constricted Preserved Sympathetic pathway disruption

1622
Q

A 39 year old lady presents with a mass lesion in her right breast. Clinical examination, biopsy and imaging confirm a 2.5 cm lesion in the upper inner quadrant of her right breast and a 1.5 cm lesion at the central aspect of the same breast. Her axilla shows lymphadenopathy and a fine needle aspirate from the node shows malignant cells. What is the best course of action?

Simple mastectomy and axillary node clearance

Wide local excisions of the two lesions and axillary node clearance

Simple mastectomy and sentinel node biopsy

Wide local excisions of the two lesions and sentinel node biopsy

Radical mastectomy and axillary node clearance

A

Multifocal breast cancer is best treated with mastectomy
A combination of established axillary disease and multifocal invasive lesions attracts an indication for mastectomy and axillary clearance. A radical mastectomy is less frequently indicated in modern surgical practice, disease that is locally advanced is often best downstaged using medical therapy, rather than embarking on the operations for breast cancer that were first popularised over 100 years ago.

Breast cancer management

  • Surgery is performed in most patients suffering from breast cancer.
    Chemotherapy may be used to downstage tumours and allow breast conserving surgery. Hormonal therapy may also be used for the same purposes.
    Radiotherapy is given to most patients who have undergone breast conserving surgery (some older patients receiving hormone treatment and who have small low grade tumours may safely avoid DXT.
    Therapeutic mammoplasty is an option for some patients but requires symmetrizing surgery in most cases.
    Patients who have undergone mastectomy may be offered a reconstructive procedure either in conjunction with their primary resection or as a staged procedure at a later date.

Surgical options
Mastectomy vs Wide local excision

Mastectomy Wide Local Excision
Multifocal tumour Solitary lesion
Central tumour Peripheral tumour
Large lesion in small breast Small lesion in large breast
DCIS >4cm DCIS <4cm
Patient Choice Patient choice

Central lesions may be managed using breast conserving surgery, where an acceptable cosmetic result may be obtained, this is rarely the case in small breasts

Axillary disease
As a minimum, all patients with invasive breast cancer should have their axilla staged. In those who do not have overt evidence of axillary nodal involvement this can be undertaken using sentinel lymph node biopsy.
Patients with a positive sentinel lymph node biopsy or who have imaging and cytological or histological evidence of axillary nodal metastasis should undergo axillary node clearance or axillary irradiation.
Axillary node clearance is associated with the development of lymphoedema, increased risk of cellulitis and frozen shoulder.

1623
Q

A 45 year old lady presented with a 2cm mobile breast mass. A mammogram is indeterminate (M3), USS shows benign changes (U2), clinical examination is also indeterminate (P3). What is the next most appropriate course of action?

Re-assure and discharge

Fine needle aspiration cytology

Excision biopsy

Image guided core biopsy

Wide local excision

A

Core biopsy Vs fine needle aspiration cytology
Core biopsy is preferred over FNAC by most surgeons. The reason for this is that FNAC often yielded inadequate tissue for assessment. When FNAC demonstrated benign changes, it had to be repeated at least once to confirm this. If it yielded cells that were indeterminate, then a core biopsy was needed. A core biopsy removes many of these stages and is thus more reliable.
All discrete breast lumps, including those that seem benign, should have a confirmed histological diagnosis. In this case, a core biopsy has not yet been performed. This may yield a diagnosis that is concordant with imaging findings. In which case, this concludes the investigative process (if benign). If it remains unclear, excision biopsy will be needed.

1624
Q

A 28 year old man has a long history of recurrent chest infections. On examination, he is noted to have no palpable vas deferens. However, both testes are located within the scrotum. What is the most likely underlying disease association?

Kleinfelters syndrome

Kallmann syndrome

Cystic fibrosis

Coeliac disease

Gardners syndrome

A

99% of males with cystic fibrosis will have absent vas.

Absence of the vas deferens

Absence of the vas may be unilateral or bilateral
Cystic fibrosis CFTR gene mutations are the cause in 40% of cases
Some non CF cases are due to unilateral renal agenesis
Sperm harvesting may allow for assisted conception

1625
Q

A 73 year old lady is admitted for a laparoscopic cholecystectomy. During her pre-operative assessment it is noted that she is receiving furosemide for the treatment of hypertension. Approximately what proportion of the sodium that is filtered at the glomerulus will be subsequently excreted?

Up to 25%

Up to 75%

Between 3 and 5%

<2%

Between 1 and 2%

A

The loop diuretics can lead to marked increases in the amount of sodium excreted. They act in the medullary and cortical aspects of the thick ascending limb of the loop of Henle. This results in a decreased medullary concentration gradient and increases free water excretion (as well as loss of sodium). Because loop diuretics result in the loss of both sodium and water they are less frequently associated with hyponatraemia than thiazide diuretics (these latter agents act in the cortex and do not affect urine concentrating ability).

Diuretic agents

The diuretic drugs are divided into three major classes, which are distinguished according to the site at which they impair sodium reabsorption: loop diuretics in the thick ascending loop of Henle, thiazide type diuretics in the distal tubule and connecting segment; and potassium sparing diuretics in the aldosterone - sensitive principal cells in the cortical collecting tubule.
In the kidney, sodium is reabsorbed through Na+/ K+ ATPase pumps located on the basolateral membrane. These pumps return reabsorbed sodium to the circulation and maintain low intracellular sodium levels. This latter effect ensures a constant concentration gradient.

Physiological effects of commonly used diuretics
Site of action Diuretic Carrier or channel inhibited Percentage of filtered sodium excreted
Ascending limb of loop of Henle Frusemide Na+/K+ 2Cl - carrier Up to 25%
Distal tubule and connecting segment Thiazides Na+Cl- carrier Between 3 and 5%
Cortical collecting tubule Spironolactone Na+/K+ ATP ase pump Between 1 and 2%

1626
Q

During an Ivor Lewis Oesophagectomy for carcinoma of the lower third of the oesophagus which structure is divided to allow mobilisation of the oesophagus?

Vagus nerve

Azygos vein

Right inferior lobar bronchus

Phrenic nerve

Pericardiophrenic artery

A

The azygos vein is routinely divided during an oesophagectomy to allow mobilisation. It arches anteriorly to insert into the SVC on the right hand side.

Treatment of oesophageal cancer

  • In general resections are not offered to those patients with distant metastasis, and usually not to those with N2 disease.
    Local nodal involvement is not in itself a contra indication to resection.
    Surgical resection is the mainstay of treatment.
    Neoadjuvent chemotherapy is given in most cases prior to surgery.
    In situ disease may be managed by endoscopic mucosal resection, although this is still debated.
    In patients with lower third lesions an Ivor - Lewis type procedure is most commonly performed. Very distal tumours may be suitable to a transhiatal procedure. Which is an attractive option as the penetration of two visceral cavities required for an Ivor- Lewis type procedure increases the morbidity considerably.
    More proximal lesions will require a total oesphagectomy (Mckeown type) with anastomosis to the cervical oesophagus.
    Patients with unresectable disease may derive benefit from local ablative procedures, palliative chemotherapy or stent insertion.

Operative details of Ivor- Lewis procedure
Combined laparotomy and right thoracotomy

Indication
Lower and middle third oesophageal tumours

Preparation
Staging with a combination of CT chest abdomen and pelvis- if no metastatic disease detected then patients will undergo a staging laparoscopy to detect peritoneal disease.
If both these modalities are negative then patients will finally undergo a PET CT scan to detect occult metastatic disease. Only in those whom no evidence of advanced disease is detected will proceed to resection.
Patients receive a GA, double lumen endotracheal tube to allow for lung deflation, CVP and arterial monitoring.

Procedure
A rooftop incision is made to access the stomach and duodenum.

Laparotomy To mobilize the stomach
The greater omentum is incised away from its attachment to the right gastroepiploic vessels along the greater curvature of the stomach.
Then the short gastric vessels are ligated and detached from the greater curvature from the spleen.
The lesser omentum is incised, preserving the right gastric artery.
The retroperitoneal attachments of the duodenum in its second and third portions are incised, allowing the pylorus to reach the oesophageal hiatus. Some surgeons perform a pyloroplasty at this point to facilitate gastric emptying.
The left gastric vessels are then ligated, avoiding any injury to the common hepatic or splenic arteries. Care must be taken to avoid inadvertently devascularising the liver owing to variations in anatomy.

Right Thoracotomy Oesophageal resection and oesophagogastric anastomosis
Through 5th intercostal space
Dissection performed 10cm above the tumour
This may involve transection of the azygos vein.
The oesophagus is then removed with the stomach creating a gastric tube.
An anastomosis is created.

The chest is closed with underwater seal drainage and tube drains to the abdominal cavity.

Post operatively
Patients will typically recover in ITU initially.
A nasogastric tube will have been inserted intraoperatively and must remain in place during the early phases of recovery.
Post operatively these patients are at relatively high risk of developing complications:

  • Atelectasis- due to the effects of thoracotomy and lung collapse
  • Anastomotic leakage. The risk is relatively high owing to the presence of a relatively devascularised stomach. Often the only blood supply is from the gastroepiploic artery as all others will have been divided. If a leak does occur then many will attempt to manage conservatively with prolonged nasogastric tube drainage and TPN. The reality is that up to 50% of patients developing an anastomotic leak will not survive to discharge.
  • Delayed gastric emptying (may be avoided by performing a pyloroplasty).
1627
Q

Which of the following is a recognised feature of ketamine when used as an anaesthetic agent?

Malignant hyperpyrexia

Adrenal suppression

Myocardial depression

Dissociative anaesthesia

Marked respiratory depression

A

Unlike most anaesthetic agents ketamine does not cause myocardial or marked respiratory depression. It is not associated with the adrenal suppression that may occur with etomidate. It is however, associated with a state of dissociative anaesthesia which patients may find distressing.

Anaesthetic agents

The table below summarises some of the more commonly used IV induction agents
Agent Specific features
Propofol
Rapid onset of anaesthesia
Pain on IV injection
Rapidly metabolised with little accumulation of metabolites
Proven anti emetic properties
Moderate myocardial depression
Widely used especially for maintaining sedation on ITU, total IV anaesthesia and for daycase surgery
Sodium thiopentone
Extremely rapid onset of action making it the agent of choice for rapid sequence of induction
Marked myocardial depression may occur
Metabolites build up quickly
Unsuitable for maintenance infusion
Little analgesic effects
Ketamine
May be used for induction of anaesthesia
Has moderate to strong analgesic properties
Produces little myocardial depression making it a suitable agent for anaesthesia in those who are haemodynamically unstable
May induce state of dissociative anaesthesia resulting in nightmares
Etomidate
Has favorable cardiac safety profile with very little haemodynamic instability
No analgesic properties
Unsuitable for maintaining sedation as prolonged (and even brief) use may result in adrenal suppression
Post operative vomiting is common

1628
Q

A 15 year old boy is admitted with colicky abdominal pain of 6 hours duration. On examination he has a soft abdomen, on systemic examination he has brownish spots around his mouth, feet and hands. His mother underwent surgery for intussusception, aged 12, and has similar lesions. What is the most likely underlying diagnosis?

Li Fraumeni syndrome

Peutz-Jeghers syndrome

Addisons disease

McCune -Albright syndrome

Appendicitis

A

This is most likely to be Peutz-Jeghers syndrome. Addisons and McCune Albright syndrome may produce similar skin changes but the intussusception resulting from polyps combined with the autosomal inheritance pattern makes this the most likely diagnosis.

Peutz-Jeghers syndrome

Peutz-Jeghers syndrome is an autosomal dominant condition characterised by numerous benign hamartomatous polyps in the gastrointestinal tract. It is also associated with pigmented freckles on the lips, face, palms and soles. Around 50% of patients will have died from a gastrointestinal tract cancer by the age of 60 years.

Genetics
Autosomal dominant
Responsible gene encodes serine threonine kinase LKB1 or STK11

Features
Hamartomatous polyps in GI tract (mainly small bowel)
Pigmented lesions on lips, oral mucosa, face, palms and soles
Intestinal obstruction e.g. intussusception (which may lead to diagnosis)
Gastrointestinal bleeding

Management
Conservative unless complications develop

1629
Q

A 43 year old lady is recovering following a live donor related renal transplant. She has significant abdominal pain. Which of the following analgesic drugs should be avoided?

Paracetamol

Morphine

Nefopam

Diclofenac

Co-codamol

A

Non steroidal anti inflammatory drugs may be nephrotoxic and therefore are usually avoided in patients who have undergone renal transplants. Paracetamol and morphine are metabolised predominantly in the liver. There is some renal contribution to morphine metabolism and excretion and the drug should be administered in reduced doses or avoided if the transplanted kidney stops functioning.

Organ Transplant

A number of different organ and tissue transplants are now available. In many cases an allograft is performed, where an organ is transplanted from one individual to another. Allografts will elicit an immune response and this is one of the main reasons for organ rejection.

Graft rejection occurs because allografts have allelic differences at genes that code immunohistocompatability complex genes. The main antigens that give rise to rejection are:
ABO blood group
Human leucocyte antigens (HLA)
Minor histocompatability antigens

ABO Matching
ABO incompatibility will result in early organ rejection (hyperacute) because of pre existing antibodies to other groups. Group O donors can give organs to any type of ABO recipient whereas group AB donor can only donate to AB recipient.

HLA System
The four most important HLA alleles are:

HLA A
HLA B
HLA C
HLA DR

An ideal organ match would be one in which all 8 alleles are matched (remember 2 from each parent, four each = 8 alleles). Modern immunosuppressive regimes help to manage the potential rejection due to HLA mismatching. However, the greater the number of mismatches the worse the long term outcome will be. T lymphocytes will recognise antigens bound to HLA molecules and will then become activated. Clonal expansion then occurs with a response directed against that antigen.

Types of organ rejection
Hyperacute. This occurs immediately through presence of pre formed antibodies (such as ABO incompatibility).
Acute. Occurs during the first 6 months and is usually T cell mediated. Usually tissue infiltrates and vascular lesions.
Chronic. Occurs after the first 6 months. Vascular changes predominate.

Hyperacute
Renal transplants at greatest risk and liver transplants at least risk. Although ABO incompatibility and HLA Class I incompatible transplants will all fare worse in long term.

Acute
All organs may undergo acute rejection. Mononuclear cell infiltrates predominate. All types of transplanted organ are susceptible and it may occur in up to 50% cases.

Chronic
Again all transplants with HLA mismatch may suffer this fate. Previous acute rejections and other immunosensitising events all increase the risk. Vascular changes are most prominent with myointimal proliferation leading to organ ischaemia. Organ specific changes are also seen such as loss of acinar cells in pancreas transplants and rapidly progressive coronary artery disease in cardiac transplants.

Surgical overview-Renal transplantation
A brief overview of the steps involved in renal transplantation is given.
Patients with end stage renal failure who are dialysis dependent or likely to become so in the immediate future are considered for transplant. Exclusion criteria include; active malignancy, old age (due to limited organ availability). Patients are medically optimised.
Donor kidneys, these may be taken from live related donors and close family, members may have less HLA mismatch than members of the general population. Laparoscopic donor nephrectomy further minimises the operative morbidity for the donor. Other organs are typically taken from brain dead or dying patients who have a cardiac arrest and in whom resuscitation is futile. The key event is to minimise the warm ischaemic time in the donor phase.

The kidney once removed is usually prepared on the bench in theatre by the transplant surgeon immediately prior to implantation and factors such as accessory renal arteries and vessel length are assessed and managed.

For first time recipients the operation is performed under general anaesthesia. A Rutherford-Morison incision is made on the preferred side. This provides excellent extraperitoneal access to the iliac vessels. The external iliac artery and vein are dissected out and following systemic heparinisation are cross clamped. The vein and artery are anastamosed to the iliacs and the clamps removed. The ureter is then implanted into the bladder and a stent is usually placed to maintain patency. The wounds are then closed and the patient recovered from surgery.

In the immediate phase a common problem encountered in cadaveric kidneys is acute tubular necrosis and this tends to resolve.

Graft survival times from cadaveric donors are typically of the order of 9 years and monozygotic twin transplant (live donor) may survive as long as 25 years.

1630
Q

A 60 year old male is admitted to the emergency room with a fall. He lives with his wife and still works as a restaurant manager. He has a past history of benign prostatic hyperplasia and is currently taking tamsulosin. He is otherwise fit and healthy. On examination, there is right hip tenderness on movement in all directions. A hip x-ray confirms an intertrochanteric fracture. What is the best management option?

Cemented hemiarthroplasty

Total hip replacement

Dynamic hip screw

Percutaneous pinning

Leg traction

A

The blood supply to the femoral head may be intact and the fracture should heal with compression type devices such as gamma nails or dynamic hip screws. The latter device being the most commonly performed therapeutic intervention.

Hip fractures

Background
Neck of femur (NOF) fracture is a common orthopaedic presentation, with over 65000 fractures in the UK per year. Like many orthopaedic injuries, there is a bimodal age distribution. It is imperative to distinguish between the high energy injury in a young patient, and the low energy osteoporotic fracture in the elderly, as their management aims are very different:

Young patient - Usually high energy trauma (e.g road traffic accident, horse riding) and needs treating in accordance with Advanced Trauma Life Support (ATLS) principles. Will often have associated injuries. Aim is to retain the patients own anatomy, and optimise their function.

Elderly patient - Predominantly female, fall from standing height (fragility fracture). Often patients have multiple comorbidities that will ultimately dictate their prognosis. Aim of orthopaedic treatment is to immediately regain patient mobility so that morbidity (infection, thromboembolic events, pressure sores etc) and mortality associated with prolonged bed rest is avoided. Left untreated, a neck of femur fracture can be considered a terminal event. Historically, mortality associated with elderly hip fracture is 10% at one month, and 30% at one year. However, this has been improved in the UK with the introduction of multidisciplinary, orthogeriatric lead care and the National Hip Fracture Database and Best Practice Tariff.

Pertinent anatomy
Osteology - normal neck-shaft angle is 130 +/- 7 degrees, and 10 +/- 7 degrees of neck anteversion.
Vascular supply - The predominant blood supply to the femoral head and neck is from the medial and lateral femoral circumflex arteries (branches of profunda femoris). These anastomose and pierce the joint capsule at the base of the neck, mainly posteriorly. There is a small vascular contribution from the artery of the ligament teres. Understanding the blood supply is fundamental to the decision making process in treating NOF fractures.

Presentation and initial management
Typically, patients present with pain in the hip/groin, a shortened, abducted, externally rotated leg (due to the unopposed pull of the muscles that act across the hip joint) and the inability to straight-leg-raise. With undisplaced fractures, signs are more subtle.
High energy injuries should be treated in line with ATLS principles. All patients should be fluid resuscitated, have adequate pain relief (often with a fascio-iliiaca nerve block), and be optimised for surgery. In addition, elderly patients should be assessed by an orthogeriatrician.

Imaging
Anteroposterior and cross-table lateral plain radiographs are sufficient to diagnose the majority of NOF fractures. If the fracture extends below the level of the lesser trochanter, or there is any possibility of pathological fracture, full length femur views are essential to plan surgery.

Where there is a high index of suspicion of fracture, but plain radiographs are inconclusive, gold standard investigation is MRI. However, if unavailable within 24 hours, or if the patient will not tolerate MRI, CT is appropriate. The majority of fractures can be seen with modern CT techniques, and so this is becoming first line in many hospitals.

Classification
There has been a move away from named classification systems towards descriptive classification systems.
Two main types of NOF exist: Intra-capsular, and extra-capsular. Extra-capsular fractures are further divided into pertrochanteric or subtrochanteric (within 5cm distal to the lesser trochanter). All fractures are then described as undisplaced, minimally displaced, or displaced.
Femoral neck and head blood supply disruption is common with intracapsular NOF fractures, and rare with extracapsular fractures. This fundamental principle underpins the practise of arthroplasty for intracapsular fractures, and fixation for extracapsular fractures.

If you wish to use a named classification system, the most commonly used are below:
Elderly intracapsular - Garden Classification
Young intrasapsular - Pauvels Classification
Intertrochanteric - Evans
Subtrochanteric - Russell Taylor

Treatment
In general, NOF fractures are treated operatively except if the patient is deemed unlikely to survive an anaesthetic. Best Practice Tarif (BPT) dictates that surgery should happen within 36 hours, as delay of greater than 48 hours is associated with increased morbidity and mortality. Below are suggested algorithms for the treatment of NOF. There are some areas of debate/controversy which are detailed below.

  • The priority with the young patient is to retain the femoral head if possible, even with a displaced intracapsular fracture. The risk of avascular necrosis and non-union (and therefore revision surgery) associated with internal fixation needs weighing up against the sequelae of total hip replacement in the young (wear, dislocation, revision). Discussion is necessary with the patient, on a case by case basis.

** Undisplaced fractures in the elderly can be treated with internal fixation, often with cannulated screws. This is appropriate for valgus impacted subcapital fractures which are inherently stable, to prevent secondary displacement. This does still carry the risk of AVN or non-union, and therefore a future revision. For this reason, many surgeons advocate arthroplasty as a single surgery.

*** NICE guidance - patients who fulfil these criteria should be offered total hip replacement which conveys better function and prosthetic survivorship, compared with hemiarthroplasty, but at an increased risk of dislocation.

  • Intertrochanteric fractures vary greatly in their stability. If the trochanter (and therefore lateral wall), and medial calcar is in tact, then the fracture configuration bears stability. This can be treated with a DHS, as collapse of the fracture is predictable. Where either or both structures are involved in the fracture, stability becomes compromised and many surgeons will favour using an intramedullary device. This is an ongoing debate, and difficult to test in an exam setting.

Post operative management
Patients should be mobilised fully weight bearing where possible. Care is multidisciplinary in its delivery. Elderly patients should have orthogeriatrician assessment of comorbidity, and bone health with secondary prevention measures if appropriate. There should be early involvement of physiotherapy and occupational therapy services. For further guidance see sources listed below.

NICE clinical guidance on hip fracture: https://www.nice.org.uk/guidance/cg124
Best Practice Tarif: www.nhfd.co.uk/20/hipfractureR…/Best%20Practice%20Tariff%20User%20Guide.pdf National Hip Fracture Database: www.nhfd.co.uk/

1631
Q

A 56 year old lady presents with a 6 month history of dysphagia to solids. She has a long history of retrosternal chest pain that is worse on lying flat and bending forwards. She undergoes an upper GI endoscopy where a smooth stricture is identified. What is the most likely diagnosis?

Globus

Adenocarcinoma

Leiomyoma

Benign stricture

Squamous cell carcinoma

A

A six month history of dysphagia is a relatively long history and makes malignancy less likely. The lesion should be biopsied for histological confirmation. Long standing oesophagitis may be complicated by the development of strictures, Barretts oesophagus or both.

Oesophageal disease

Disorder Features
Mallory-Weiss Tear Usually history of antecedent vomiting. This is then followed by the vomiting of a small amount of blood. There is usually little in the way of systemic disturbance or prior symptoms.
Hiatus hernia of gastric cardia Often longstanding history of dyspepsia, patients are often overweight. Uncomplicated hiatus hernias should not be associated with dysphagia or haematemesis.
Oesophageal rupture Complete disruption of the oesophageal wall in absence of pre-existing pathology. Left postero-lateral oesophageal is commonest site (2-3cm from OG junction). Suspect in patients with severe chest pain without cardiac diagnosis and signs suggestive of pneumonia without convincing history, where there is history of vomiting. Erect CXR shows infiltrate or effusion in 90% of cases(1).
Squamous cell carcinoma of the oesophagus History of progressive dysphagia. Often signs of weight loss. Usually little or no history of previous GORD type symptoms.
Adenocarcinoma of the oesophagus Progressive dysphagia, may have previous symptoms of GORD or Barretts oesophagus.
Peptic stricture Longer history of dysphagia, often not progressive. Usually symptoms of GORD. Often lack systemic features seen with malignancy
Dysmotility disorder May have dysphagia that is episodic and non progressive. Retrosternal pain may accompany the episodes.

Diagnosis
Most of the differential diagnoses listed above can be accurately categorised by upper GI endoscopy (usually most patients). Where this fails to demonstrate a mechanical stricture the use of pH and manometry studies together with radiological contrast swallows will facilitate the diagnosis.

References
Blencowe N et al. Spontaneous oesophageal rupture. BMJ 2013 (346):38-39.

1632
Q

A 21 year old man undergoes an uncomplicated tonsillectomy for recurrent attacks of tonsillitis. Post operatively he complains of otalgia. Which nerve is responsible?

Trigeminal

Hypoglossal

Glossopharyngeal

Facial

Vagus

A

The glossopharyngeal nerve supplies this area and the ear and otalgia may be the result of referred pain.

Cranial nerves

Cranial nerve lesions
Olfactory nerve May be injured in basal skull fractures or involved in frontal lobe tumour extension. Loss of olfactory nerve function in relation to major CNS pathology is seldom an isolated event and thus it is poor localiser of CNS pathology.
Optic nerve Problems with visual acuity may result from intra ocular disorders. Problems with the blood supply such as amaurosis fugax may produce temporary visual distortion. More important surgically is the pupillary response to light. The pupillary size may be altered in a number of disorders. Nerves involved in the resizing of the pupil connect to the pretectal nucleus of the high midbrain, bypassing the lateral geniculate nucleus and the primary visual cortex. From the pretectal nucleus neurones pass to the Edinger - Westphal nucleus, motor axons from here pass along with the oculomotor nerve. They synapse with ciliary ganglion neurones; the parasympathetic axons from this then innervate the iris and produce miosis. The miotic pupil is seen in disorders such as Horner’s syndrome or opiate overdose.
Mydriasis is the dilatation of the pupil in response to disease, trauma, drugs (or the dark!). It is pathological when light fails to induce miosis. The radial muscle is innervated by the sympathetic nervous system. Because the parasympathetic fibres travel with the oculomotor nerve they will be damaged by lesions affecting this nerve (e.g. cranial trauma).
The response to light shone in one eye is usually a constriction of both pupils. This indicates intact direct and consensual light reflexes. When the optic nerve has an afferent defect the light shining on the affected eye will produce a diminished pupillary response in both eyes. Whereas light shone on the unaffected eye will produce a normal pupillary response in both eyes. This is referred to as the Marcus Gunn pupil and is seen in conditions such as optic neuritis. In a total CN II lesion shining the light in the affected eye will produce no response.
Oculomotor nerve The pupillary effects are described above. In addition it supplies all ocular muscles apart from lateral rectus and superior oblique. Thus the affected eye will be deviated inferolaterally. Levator palpebrae superioris may also be impaired resulting in impaired ability to open the eye.
Trochlear nerve The eye will not be able to look down.
Trigeminal nerve Largest cranial nerve. Exits the brainstem at the pons. Branches are ophthalmic, maxillary and mandibular. Only the mandibular branch has both sensory and motor fibres. Branches converge to form the trigeminal ganglion (located in Meckels cave). It supplies the muscles of mastication and also tensor veli palatine, mylohyoid, anterior belly of digastric and tensor tympani. The detailed descriptions of the various sensory functions are described in other areas of the website. The corneal reflex is important and is elicited by applying a small tip of cotton wool to the cornea, a reflex blink should occur if it is intact. It is mediated by: the naso ciliary branch of the ophthalmic branch of the trigeminal (sensory component) and the facial nerve producing the motor response. Lesions of the afferent arc will produce bilateral absent blink and lesions of the efferent arc will result in a unilateral absent blink.
Abducens nerve The affected eye will have a deficit of abduction. This cranial nerve exits the brainstem between the pons and medulla. It thus has a relatively long intra cranial course which renders it susceptible to damage in raised intra cranial pressure.
Facial nerve Emerges from brainstem between pons and medulla. It controls muscles of facial expression and taste from the anterior 2/3 of the tongue. The nerve passes into the petrous temporal bone and into the internal auditory meatus. It then passes through the facial canal and exits at the stylomastoid foramen. It passes through the parotid gland and divides at this point. It does not innervate the parotid gland. Its divisions are considered in other parts of the website. Its motor fibres innervate orbicularis oculi to produce the efferent arm of the corneal reflex. In surgical practice it may be injured during parotid gland surgery or invaded by malignancies of the gland and a lower motor neurone on the ipsilateral side will result.
Vestibulo-cochlear nerve Exits from the pons and then passes through the internal auditory meatus. It is implicated in sensorineural hearing loss. Individuals with sensorineural hearing loss will localise the sound in webers test to the normal ear. Rinnes test will be reduced on the affected side but should still work. These two tests will distinguish sensorineural hearing loss from conductive deafness. In the latter condition webers test will localise to the affected ear and Rinnes test will be impaired on the affected side. Surgical lesions affecting this nerve include CNS tumours and basal skull fractures. It may also be damaged by the administration of ototoxic drugs (of which gentamicin is the most commonly used in surgical practice).
Glossopharyngeal nerve Exits the pons just above the vagus. Receives sensory fibres from posterior 1/3 tongue, tonsils, pharynx and middle ear (otalgia may occur following tonsillectomy). It receives visceral afferents from the carotid bodies. It supplies parasympathetic fibres to the parotid gland via the otic ganglion and motor function to stylopharyngeaus muscle. The sensory function of the nerve is tested using the gag reflex.
Vagus nerve Leaves the medulla between the olivary nucleus and the inferior cerebellar peduncle. Passes through the jugular foramen and into the carotid sheath. Details of the functions of the vagus nerve are covered in the website under relevant organ sub headings.
Accessory nerve Exists from the caudal aspect of the brainstem (multiple branches) supplies trapezius and sternocleidomastoid muscles. The distal portion of this nerve is most prone to injury during surgical procedures.
Hypoglossal nerve Emerges from the medulla at the preolivary sulcus, passes through the hypoglossal canal. It lies on the carotid sheath and passes deep to the posterior belly of digastric to supply muscles of the tongue (except palatoglossus). Its location near the carotid sheath makes it vulnerable during carotid endarterectomy surgery and damage will produce ipsilateral defect in muscle function.

1633
Q

An 18 month old boy is brought to the emergency room by his parents. He was found in bed with a nappy filled with dark red blood. He is haemodynamically unstable and requires a blood transfusion. Prior to this episode he was well with no prior medical history. What is the most likely cause?

Necrotising enterocolitis

Anal fissure

Oesophageal varices

Meckels diverticulum

Crohns disease

A

Meckels diverticulum is the number one cause of painless massive GI bleeding requiring a transfusion in children between the ages of 1 and 2 years.
Other causes of GI bleeding include:
Site Newborn 1 month to 1 year 1 to 2 years Older than 2 years
Upper GI tract Haemorrhagic disease, swallowed maternal blood Oesophagitis/ gastritis Peptic ulcer disease Varices
Lower GI tract Anal fissure/ NEC Anal fissure/ Intussusception Polyps / Meckels diverticulum IBD/ Polyps/ Intussusception

Meckel’s diverticulum

  • Congenital abnormality resulting in incomplete obliteration of the vitello-intestinal duct
    Normally, in the foetus, there is an attachment between the vitello-intestinal duct and the yolk sac.This disappears at 6 weeks gestation.
    The tip is free in majority of cases.
    Associated with enterocystomas, umbilical sinuses, and omphaloileal fistulas.
    Arterial supply: omphalomesenteric artery.
    2% of population, 2 inches long, 2 feet from the ileocaecal valve.
    Typically lined by ileal mucosa but ectopic gastric mucosa can occur, with the risk of peptic ulceration. Pancreatic and jejunal mucosa can also occur.

Clinical
Normally asymptomatic and an incidental finding.
Complications are the result of obstruction, ectopic tissue, or inflammation.
Removal if narrow neck or symptomatic. Options are between wedge excision or formal small bowel resection and anastomosis.

1634
Q

The thebesian veins contribute to the venous drainage of the heart. Into which of the following structures do they primarily drain?

Great cardiac vein

Atrium

Superior vena cava

Oblique vein

Small cardiac vein

A

The thebesian veins are numerous small veins running over the surface of the heart they drain into the heart itself. Usually this is to the atrium directly.

Heart anatomy

The walls of each cardiac chamber comprise:
Epicardium
Myocardium
Endocardium

Cardiac muscle is attached to the cardiac fibrous skeleton.

Relations
The heart and roots of the great vessels within the pericardial sac are related to the posterior aspect of the sternum, medial ends of the 3rd to 5th ribs on the left and their associated costal cartilages. The heart and pericardial sac are situated obliquely two thirds to the left and one third to the right of the median plane.

The pulmonary valve lies at the level of the left third costal cartilage.
The mitral valve lies at the level of the fourth costal cartilage.

Coronary sinus
This lies in the posterior part of the coronary groove and receives blood from the cardiac veins. The great cardiac vein lies at its left and the middle and small cardiac veins lie on its right. The smallest cardiac vein (anterior cardiac vein) drains into the right atrium directly.

Aortic sinus
Right coronary artery arises from the right aortic sinus, the left is derived from the left aortic sinus, which lies posteriorly.

Features of the left ventricle as opposed to the right

Structure Left Ventricle
A-V Valve Mitral (double leaflet)
Walls Twice as thick as right
Trabeculae carnae Much thicker and more numerous

Right coronary artery
The RCA supplies:
Right atrium
Diaphragmatic part of the right ventricle
Usually the posterior third of the interventricular septum
The sino atrial node (60% cases)
The atrio ventricular node (80% cases)

Left coronary artery
The LCA supplies:
Left atrium
Most of left ventricle
Part of the right ventricle
Anterior two thirds of the inter ventricular septum
The sino atrial node (remaining 40% cases)

Innervation of the heart
Autonomic nerve fibres from the superficial and deep cardiac plexus. These lie anterior to the bifurcation of the trachea, posterior to the ascending aorta and superior to the bifurcation of the pulmonary trunk. The parasympathetic supply to the heart is from presynaptic fibres of the vagus nerves.

Valves of the heart
Mitral valve Aortic valve Pulmonary valve Tricuspid valve
2 cusps 3 cusps 3 cusps 3 cusps
First heart sound Second heart sound Second heart sound First heart sound
1 anterior cusp 2 anterior cusps 2 anterior cusps 2 anterior cusps
Attached to chordae tendinae No chordae No chordae Attached to chordae tendinae

1635
Q

A 53 year old man is undergoing a left hemicolectomy for carcinoma of the descending colon. From which embryological structure is this region of the gastrointestinal tract derived?

Vitellino-intestinal duct

Hind gut

Mid gut

Fore gut

Woolffian duct

A

The left colon is embryologically part of the hind gut. Which accounts for its separate blood supply via the IMA.

Colon anatomy

The colon commences with the caecum. This represents the most dilated segment of the human colon and its base (which is intraperitoneal) is marked by the convergence of teniae coli. At this point is located the vermiform appendix. The colon continues as the ascending colon, the posterior aspect of which is retroperitoneal. The line of demarcation between the intra and retro peritoneal right colon is visible as a white line, in the living, and forms the line of incision for colonic resections.

The ascending colon becomes the transverse colon after passing the hepatic flexure. At this location the colon becomes wholly intra peritoneal once again. The superior aspect of the transverse colon is the point of attachment of the transverse colon to the greater omentum. This is an important anatomical site since division of these attachments permits entry into the lesser sac. Separation of the greater omentum from the transverse colon is a routine operative step in both gastric and colonic resections.

At the left side of the abdomen the transverse colon passes to the left upper quadrant and makes an oblique inferior turn at the splenic flexure. Following this, the posterior aspect becomes retroperitoneal once again.

At the level of approximately L4 the descending colon becomes wholly intraperitoneal and becomes the sigmoid colon. Whilst the sigmoid is wholly intraperitoneal there are usually attachments laterally between the sigmoid and the lateral pelvic sidewall. These small congenital adhesions are not formal anatomical attachments but frequently require division during surgical resections.

At its distal end the sigmoid passes to the midline and at the region around the sacral promontary it becomes the upper rectum. This transition is visible macroscopically as the point where the teniae fuse. More distally the rectum passes through the peritoneum at the region of the peritoneal reflection and becomes extraperitoneal.

Arterial supply
Superior mesenteric artery and inferior mesenteric artery: linked by the marginal artery.
Ascending colon: ileocolic and right colic arteries
Transverse colon: middle colic artery
Descending and sigmoid colon: inferior mesenteric artery

Venous drainage
From regional veins (that accompany arteries) to superior and inferior mesenteric vein

Lymphatic drainage
Initially along nodal chains that accompany supplying arteries, then para-aortic nodes.

Embryology
Midgut- Second part of duodenum to 2/3 transverse colon
Hindgut- Distal 1/3 transverse colon to anus

Peritoneal location
The right and left colon are part intraperitoneal and part extraperitoneal. The sigmoid and transverse colon are generally wholly intraperitoneal. This has implications for the sequelae of perforations, which will tend to result in generalised peritonitis in the wholly intra peritoneal segments.

Colonic relations
Region of colon Relation
Caecum/ right colon Right ureter, gonadal vessels
Hepatic flexure Gallbladder (medially)
Splenic flexure Spleen and tail of pancreas
Distal sigmoid/ upper rectum Left ureter
Rectum Ureters, autonomic nerves, seminal vesicles, prostate, urethra (distally)

1636
Q

Which option is not recommended during the management of compartment syndrome?

Anticoagulation

Keep limb level with the body

Intravenous fluids

Pain control

Fasciotomy

A

Anticoagulation will worsen compartment syndrome.

Compartment syndrome

  • This is a particular complication that may occur following fractures (or following ischaemia re-perfusion injury in vascular patients). It is characterised by raised pressure within a closed anatomical space.
    The raised pressure within the compartment will eventually compromise tissue perfusion resulting in necrosis. The two main fractures carrying this complication include supracondylar fractures and tibial shaft injuries.

Symptoms and signs
Pain, especially on movement (even passive)
Parasthesiae
Pallor may be present
Arterial pulsation may still be felt as the necrosis occurs as a result of microvascular compromise
Paralysis of the muscle group may occur

Diagnosis
Is made by measurement of intracompartmental pressure measurements. Pressures in excess of 20mmHg are abnormal and >40mmHg is diagnostic.

Treatment
This is essentially prompt and extensive fasciotomies
In the lower limb the deep muscles may be inadequately decompressed by the inexperienced operator when smaller incisions are performed
Myoglobinuria may occur following fasciotomy and result in renal failure and for this reason these patients require aggressive IV fluids
Where muscle groups are frankly necrotic at fasciotomy they should be debrided and amputation may have to be considered
Death of muscle groups may occur within 4-6 hours

1637
Q

A 57 year old lady has suffered from lymphoedema for many years. The left leg is swollen to the mid thigh. Severe limb deformity has developed as a result of process and in spite of compression hosiery. Lymphoscintography shows no patent lymphatics in the proximal leg. The overlying skin is healthy. Which of the following options would give the best long term outcome?

Multilayer compression bandages

Charles operation

Homans operation

Long term loop diuretic therapy

Amputation

A

Surgery is indicated in less than 10% of cases. However, severe deformity is one of the indications for surgery. Lymphovenous anastomosis is indicated where the proximal lymphatics are not patent. When the overlying skin is healthy (and limb deformity a problem), a Homans procedure is a reasonable first line operative option.
Lymphoedema

  • Due to impaired lymphatic drainage in the presence of normal capillary function.
    Lymphoedema causes the accumulation of protein rich fluid, subdermal fibrosis and dermal thickening.
    Characteristically fluid is confined to the epifascial space (skin and subcutaneous tissues); muscle compartments are free of oedema. It involves the foot, unlike other forms of oedema. There may be a ‘buffalo hump’ on the dorsum of the foot and the skin cannot be pinched due to subcutaneous fibrosis.

Causes of lymphoedema

Primary
Congenital < 1 year: sporadic, Milroy’s disease
Onset 1-35 years: sporadic, Meige’s disease
> 35 years: Tarda
Secondary
Bacterial/fungal/parasitic infection (filariasis)
Lymphatic malignancy
Radiotherapy to lymph nodes
Surgical resection of lymph nodes
DVT
Thrombophlebitis

Indications for surgery
Marked disability or deformity from limb swelling
Lymphoedema caused by proximal lymphatic obstruction with patent distal lymphatics suitable for a lymphatic drainage procedure
Lymphocutaneous fistulae and megalymphatics

Procedures
Homans operation Reduction procedure with preservation of overlying skin (which must be in good condition). Skin flaps are raised and the underlying tissue excised. Limb circumference typically reduced by a third.
Charles operation All skin and subcutaneous tissue around the calf are excised down to the deep fascia. Split skin grafts are placed over the site. May be performed if overlying skin is not in good condition. Larger reduction in size than with Homans procedure.
Lymphovenous anastamosis Identifiable lymphatics are anastomosed to sub dermal venules. Usually indicated in 2% of patients with proximal lymphatic obstruction and normal distal lymphatics.

1638
Q

A 28 year old female presents with a small nodule located on the back of her neck. It is excised for cosmetic reasons and the histology report states that the lesion consists of a sebum filled lesion surrounded by the outer root sheath of a hair follicle. What is the most likely cause?

Pilar cyst

Seborrheoic keratosis

Dermatofibroma

Atypical naevus

Spitz naevus

A

Pilar cysts may contain foul smelling cheesy material and are surrounded by the outer part of a hair follicle. Because of their histological appearances they are more correctly termed pilar cysts than sebaceous cysts.

Benign skin diseases

Seborrhoeic keratosis
Most commonly arise in patients over the age of 50 years, often idiopathic
Equal sex incidence and prevalence
Usually multiple lesions over face and trunk
Flat, raised, filiform and pedunculated subtypes are recognised
Variable colours and surface may have greasy scale overlying it
Treatment options consist of leaving alone or simple shave excision

Melanocytic naevi
Congenital melanocytic naevi
Typically appear at, or soon after, birth
Usually greater than 1cm diameter
Increased risk of malignant transformation (increased risk greatest for large lesions)
Junctional melanocytic naevi
Circular macules
May have heterogeneous colour even within same lesion
Most naevi of the palms, soles and mucous membranes are of this type
Compound naevi
Domed pigmented nodules up to 1cm in diameter
Arise from junctional naevi, usually have uniform colour and are smooth
Spitz naevus
Usually develop over a few months in children
May be pink or red in colour, most common on face and legs
May grow up to 1cm and growth can be rapid, this usually results in excision
Atypical naevus syndrome
Atypical melanocytic naevi that may be autosomally dominantly inherited
Some individuals are at increased risk of melanoma (usually have mutations of CDKN2A gene)
Many people with atypical naevus syndrome AND a parent sibling with melanoma will develop melanoma

Epidermoid cysts
Common and affect face and trunk
They have a central punctum, they may contain small quantities of sebum
The cyst lining is either normal epidermis (epidermoid cyst) or outer root sheath of hair follicle (pilar cyst)

Dermatofibroma
Solitary dermal nodules
Usually affect extremities of young adults
Lesions feel larger than they appear visually
Histologically they consist of proliferating fibroblasts merging with sparsely cellular dermal tissues

Painful skin lesions
Eccrine spiradenoma
Neuroma
Glomus tumour
Leiomyoma
Angiolipoma
Neurofibroma (rarely painful) and dermatofibroma (rarely painful)

1639
Q

A 63 year old man is commenced on an infusion of packed red cells following a total hip replacement. Which of the following adverse events is most likely?

ABO mismatching

Immune mediated intolerance of rhesus incompatible blood

Pyrexia

Jaundice

Graft versus host disease

A

Mild pyrexia during blood transfusion is the most common event and commonly occurs during transfusion.

Blood transfusion reactions

Acute transfusion reactions present as adverse signs or symptoms during or within 24 hours of a blood transfusion. The most frequent reactions are fever, chills, pruritus, or urticaria, which typically resolve promptly without specific treatment or complications. Other signs occurring in temporal relationship with a blood transfusion, such as severe dyspnoea, pyrexia, or loss of consciousness may be the first indication of a more severe potentially fatal reaction.
The causes of adverse reactions are multi-factorial. Immune mediated reactions, some of the most feared, occur as a result of component mismatch, the commonest cause of which is clerical error. More common, non immune mediated, complications may occur as a result of product contamination, this may be bacterial or viral.
Transfusion related lung injury is well recognised and there are two proposed mechanisms which underpin this. One involves the sequestration of primed neutrophils within the recipient pulmonary capillary bed. The other proposed mechanism suggests that HLA mismatches between donor neutrophils and recipient lung tissue is to blame.
The table below summarises the main types of transfusion reaction.

Immune mediated Non immune mediated
Pyrexia Hypocalcaemia
Alloimmunization CCF
Thrombocytopaenia Infections
Transfusion associated lung injury Hyperkalaemia
Graft vs Host disease
Urticaria
Acute or delayed haemolysis
ABO incompatibility
Rhesus incompatibility

1640
Q

A 40 year old female is diagnosed as having Dercums disease. Which of the lesions listed below are most likely to be identified on physical examination?

Lipomas

Neuromas

Hamartomas

Arteriovenous malformations

Histiocytomas

A

Dercums disease is characterized by multiple lipomas. It is also referred to as adiposis dolorosa.

Dercums disease

Condition in which there is generalized obesity and multiple fatty tumours within adipose tissue
Lesions are usually multiple and found around extremity joints
Commonest in females
Typically presents in middle age
Treatments include analgesia and occasionally resection of the lesions, recurrence, however, is common

1641
Q

A 14 year old female is admitted with sudden onset right iliac fossa pain. She is otherwise well and on examination has some right iliac fossa tenderness but no guarding. She is afebrile. Urinary dipstick is normal. Her previous menstrual period two weeks ago was normal and pregnancy test is negative. What is the most likely underlying diagnosis?

Mittelschmerz

Endometriosis

Appendicitis

Crohns disease

Pelvic abscess

A

The timing of the pain and the fact that it is mid cycle makes Mittelschmerz the most likely cause. When follicular cysts rupture, there is sometimes associated bleeding of small volume. The cyst contents and blood if present can be very irritant and pain can sometimes mimic appendicitis or pelvic inflammatory disease. However, normal inflammatory markers makes the former less likely and unless the 14 year old was sexually active, PID is also relatively rare in this group. Crohns disease and pelvic abscesses would typically present with a more protracted history and raised inflammatory markers and fever.

Right iliac fossa pain

Differential diagnosis

Appendicitis
Pain radiating to right iliac fossa
Anorexia (very common)
Short history
Diarrhoea and profuse vomiting rare
Crohn’s disease
Often long history
Signs of malnutrition
Change in bowel habit, especially diarrhoea
Mesenteric adenitis
Mainly affects children
Causes include Adenoviruses, Epstein Barr Virus, beta-haemolytic Streptococcus, Staphylococcus spp., Escherichia coli, Streptococcus viridans and Yersinia spp.
Patients have a higher temperature than those with appendicitis
If laparotomy is performed, enlarged mesenteric lymph nodes will be present
Diverticulitis
Both left and right sided disease may present with right iliac fossa pain
Clinical history may be similar, although some change in bowel habit is usual
When suspected, a CT scan may help in refining the diagnosis
Meckel’s diverticulitis
A Meckel’s diverticulum is a congenital abnormality that is present in about 2% of the population
Typically 2 feet proximal to the ileocaecal valve
May be lined by ectopic gastric mucosal tissue and produce bleeding
Perforated peptic ulcer
This usually produces upper quadrant pain but pain may be lower
Perforations typically have a sharp sudden onset of pain in the history
Incarcerated right inguinal or femoral hernia
Usually only right iliac fossa pain if right sided or bowel obstruction.
Bowel perforation secondary to caecal or colon carcinoma
Seldom localised to right iliac fossa, although complete large bowel obstruction with caecal distension may cause pain prior to perforation.
Gynaecological causes
Pelvic inflammatory disease/salpingitis/pelvic abscess/Ectopic pregnancy/Ovarian torsion/Threatened or complete abortion/Mittelschmerz
Urological causes
Ureteric colic/UTI/Testicular torsion
Other causes
TB/Typhoid/Herpes Zoster/AAA/Situs inversus

1642
Q

A 25 year old man undergoes an appendicetomy for appendicitis. The appendix is submitted for histopathological evaluation. Which of the following is most likely to be identified microscopically within and on the serosa?

Macrophages

Neutrophils

Fibroblasts

Lymphocytes

Stem cells

A

Neutrophil polymorphs are the cell type most commonly encountered in acute inflammation.

Acute inflammation

Inflammation is the reaction of the tissue elements to injury. Vascular changes occur, resulting in the generation of a protein rich exudate. So long as the injury does not totally destroy the existing tissue architecture, the episode may resolve with restoration of original tissue architecture.

Vascular changes
Vasodilation occurs and persists throughout the inflammatory phase.
Inflammatory cells exit the circulation at the site of injury.
The equilibrium that balances Starlings forces within capillary beds is disrupted and a protein rich exudate will form as the vessel walls also become more permeable to proteins.
The high fibrinogen content of the fluid may form a fibrin clot. This has several important immunomodulatory functions.

Sequelae
Resolution
Typically occurs with minimal initial injury
Stimulus removed and normal tissue architecture results
Organisation
Delayed removal of exudate
Tissues undergo organisation and usually fibrosis
Suppuration
Typically formation of an abscess or an empyema
Sequestration of large quantities of dead neutrophils
Progression to chronic inflammation
Coupled inflammatory and reparative activities
Usually occurs when initial infection or suppuration has been inadequately managed

Causes
Infections e.g. Viruses, exotoxins or endotoxins released by bacteria
Chemical agents
Physical agents e.g. Trauma
Hypersensitivity reactions
Tissue necrosis

Presence of neutrophil polymorphs is a histological diagnostic feature of acute inflammation

1643
Q

A 19 year old soldier has just returned from a prolonged marching exercise and presents with a sudden onset, severe pain, in the forefoot. Clinical examination reveals tenderness along the second metatarsal. Plain x-rays are taken of the area, these demonstrate callus surrounding the shaft of the second metatarsal. What is the most likely diagnosis?

Stress fracture

Mortons neuroma

Osteochondroma

Acute osteomyelitis

Freiberg’s disease

A

A short history of pain together with clinical examination and radiological signs affecting the second metatarsal favour a stress fracture. The fact that callus is present suggests that immobilisation is unlikely to be beneficial. Freibergs disease is an anterior metatarsalgia affecting the head of the second metarsal, it typically occurs in the pubertal growth spurt. The initial injury was thought to be due to stress microfractures at the growth plate. The key feature in the history which distinguishes the injury as being stress fracture is the radiology. In Freibergs disease the x-ray changes include; joint space widening, formation of bony spurs, sclerosis and flattening of the metatarsal head.

Stress fractures

Repetitive activity and loading of normal bone may result in small hairline fractures. Whilst these may be painful they are seldom displaced. Surrounding soft tissue injury is unusual. They may present late following the injury, in which case callus formation may be identified on radiographs. Such cases may not require formal immobilisation, injuries associated with severe pain and presenting at an earlier stage may benefit from immobilisation tailored to the site of injury.

1644
Q

The first root of the brachial plexus commonly arises at which of the following levels?

C6

C5

C3

C2

C8

A

It begins at C5 and has 5 roots. It ends with a total of 15 nerves of these 5 are the main nerves to the upper limb (axillary, radial, ulnar, musculocutaneous and median)

Brachial plexus

The brachial plexus extends from the neck to the axilla. It is formed by the ventral rami of the fifth to the eighth cervical nerves with the ascending part of the first thoracic nerve.

Location of the plexus
The ventral rami which form the plexus enter the lower part of the posterior triangle of the neck in series with the ventral rami of the cervical plexus. The second part of the subclavian artery lies immediately anterior to the lower two rami. The upper three rami intermingle and pass inferolaterally towards the axilla and subclavian artery. They are enclosed within an extension of the prevertebral fascia. In the neck the plexus lies deep to platysma, the supraclavicular nerves, inferior belly of omohyoid and the transverse cervical artery. It then passes deep to the clavicle and the suprascapular vessels, to enter the axilla, and thence surround the second part of the axillary artery

Composition of the plexus
Ventral rami, the roots of the plexus, lie between scalenus medius and anterior.

As they enter the posterior triangle, the upper two (C5,6) and lower two (C8, T1) roots of the plexus unite to form the upper and lower trunks of the plexus respectively. Meanwhile, C7 continues as the middle trunk. The lower trunk may groove the superior surface of the first rib posterior to the subclavian artery, and the root from the first ventral ramus is always in contact with it.

Each trunk divides into ventral and dorsal divisions which are destined to supply the anterior (flexor) and posterior (extensor) parts of the upper limb.

The cords of the plexus are formed in the axilla. The dorsal divisions unite to form the posterior cord (C5-8). The ventral divisions of the upper and middle trunks unite to form the lateral cord (C5-7), while the ventral divisions of the lower trunk continues as the medial cord (C8-T1). The cords are named according to their relationship to the axillary artery. Each cord terminates by dividing into two main branches at the beginning of the third part of the artery.

Sympathetic communications
The fifth and sixth cervical ventral rami receive grey rami communicantes from the middle cervical ganglion, while the two or more grey rami communicantes pass from the inferior cervical ganglion to the seventh and eighth cervical ventral rami. The first thoracic ventral ramus receives its grey ramus from the cervicothoracic ganglion. Its for this reason that inferior plexus injury can be complicated by a Horners syndrome.

Summary
Origin Anterior rami of C5 to T1
Sections of the plexus
Roots, trunks, divisions, cords, branches
Mnemonic:Real Teenagers Drink Cold Beer
Roots
Located in the posterior triangle
Pass between scalenus anterior and medius
Trunks
Located posterior to middle third of clavicle
Upper and middle trunks related superiorly to the subclavian artery
Lower trunk passes over 1st rib posterior to the subclavian artery
Divisions Apex of axilla
Cords Related to axillary artery

1645
Q

A 42 year old woman is due to undergo a left nephroureterectomy for a transitional cell carcinoma involving the ureter. Which of the following structures is not related to the left ureter?

Round ligament of the uterus

Internal iliac artery

Ovarian artery

Peritoneum

Sigmoid mesocolon

A

The ureter is not related to the round ligament of the uterus, it is related to the broad ligament and is within 1.5cm of the supravaginal part of the cervix.

Ureter

25-35 cm long
Muscular tube lined by transitional epithelium
Surrounded by thick muscular coat. Becomes 3 muscular layers as it crosses the bony pelvis
Retroperitoneal structure overlying transverse processes L2-L5
Lies anterior to bifurcation of iliac vessels
Blood supply is segmental; renal artery, aortic branches, gonadal branches, common iliac and internal iliac
Lies beneath the uterine artery

1646
Q

A 42 year old woman is admitted to the vascular ward for an endarterectomy. Her CT report confirms a left temporal lobe infarct. What is the most likely visual defect to be encountered?

Right homonymous hemianopia

Right superior quadranopia

Right inferior quadranopia

Left superior quadranopia

Left homonymous hemianopia

A

Temporal lesions cause a contralateral superior quadranopia. Think temporal area is at the top of the head i.e. superior quadranopia.
Another useful mnemonic is: PITS: Parietal Inferior, Temporal superior quadratinopia

Visual field defects

left homonymous hemianopia means visual field defect to the left, i.e. Lesion of right optic tract
homonymous quadrantanopias: PITS (Parietal-Inferior, Temporal-Superior)
incongruous defects = optic tract lesion; congruous defects = optic radiation lesion or occipital cortex

Homonymous hemianopia
Incongruous defects: lesion of optic tract
Congruous defects: lesion of optic radiation or occipital cortex
Macula sparing: lesion of occipital cortex

Homonymous quadrantanopias
Superior: lesion of temporal lobe
Inferior: lesion of parietal lobe
Mnemonic = PITS (Parietal-Inferior, Temporal-Superior)

Bitemporal hemianopia
Lesion of optic chiasm
Upper quadrant defect > lower quadrant defect = inferior chiasmal compression, commonly a pituitary tumour
Lower quadrant defect > upper quadrant defect = superior chiasmal compression, commonly a craniopharyngioma

1647
Q

Which cranial nerve supplies general sensation to the posterior third of the tongue?

Facial

Trigeminal

Vagus

Hypoglossal

Glossopharyngeal

A

The glossopharyngeal nerve supplies general sensation to the posterior third of the tongue and contributes to the gag reflex.

Cranial nerves

Cranial nerve lesions
Olfactory nerve May be injured in basal skull fractures or involved in frontal lobe tumour extension. Loss of olfactory nerve function in relation to major CNS pathology is seldom an isolated event and thus it is poor localiser of CNS pathology.
Optic nerve Problems with visual acuity may result from intra ocular disorders. Problems with the blood supply such as amaurosis fugax may produce temporary visual distortion. More important surgically is the pupillary response to light. The pupillary size may be altered in a number of disorders. Nerves involved in the resizing of the pupil connect to the pretectal nucleus of the high midbrain, bypassing the lateral geniculate nucleus and the primary visual cortex. From the pretectal nucleus neurones pass to the Edinger - Westphal nucleus, motor axons from here pass along with the oculomotor nerve. They synapse with ciliary ganglion neurones; the parasympathetic axons from this then innervate the iris and produce miosis. The miotic pupil is seen in disorders such as Horner’s syndrome or opiate overdose.
Mydriasis is the dilatation of the pupil in response to disease, trauma, drugs (or the dark!). It is pathological when light fails to induce miosis. The radial muscle is innervated by the sympathetic nervous system. Because the parasympathetic fibres travel with the oculomotor nerve they will be damaged by lesions affecting this nerve (e.g. cranial trauma).
The response to light shone in one eye is usually a constriction of both pupils. This indicates intact direct and consensual light reflexes. When the optic nerve has an afferent defect the light shining on the affected eye will produce a diminished pupillary response in both eyes. Whereas light shone on the unaffected eye will produce a normal pupillary response in both eyes. This is referred to as the Marcus Gunn pupil and is seen in conditions such as optic neuritis. In a total CN II lesion shining the light in the affected eye will produce no response.
Oculomotor nerve The pupillary effects are described above. In addition it supplies all ocular muscles apart from lateral rectus and superior oblique. Thus the affected eye will be deviated inferolaterally. Levator palpebrae superioris may also be impaired resulting in impaired ability to open the eye.
Trochlear nerve The eye will not be able to look down.
Trigeminal nerve Largest cranial nerve. Exits the brainstem at the pons. Branches are ophthalmic, maxillary and mandibular. Only the mandibular branch has both sensory and motor fibres. Branches converge to form the trigeminal ganglion (located in Meckels cave). It supplies the muscles of mastication and also tensor veli palatine, mylohyoid, anterior belly of digastric and tensor tympani. The detailed descriptions of the various sensory functions are described in other areas of the website. The corneal reflex is important and is elicited by applying a small tip of cotton wool to the cornea, a reflex blink should occur if it is intact. It is mediated by: the naso ciliary branch of the ophthalmic branch of the trigeminal (sensory component) and the facial nerve producing the motor response. Lesions of the afferent arc will produce bilateral absent blink and lesions of the efferent arc will result in a unilateral absent blink.
Abducens nerve The affected eye will have a deficit of abduction. This cranial nerve exits the brainstem between the pons and medulla. It thus has a relatively long intra cranial course which renders it susceptible to damage in raised intra cranial pressure.
Facial nerve Emerges from brainstem between pons and medulla. It controls muscles of facial expression and taste from the anterior 2/3 of the tongue. The nerve passes into the petrous temporal bone and into the internal auditory meatus. It then passes through the facial canal and exits at the stylomastoid foramen. It passes through the parotid gland and divides at this point. It does not innervate the parotid gland. Its divisions are considered in other parts of the website. Its motor fibres innervate orbicularis oculi to produce the efferent arm of the corneal reflex. In surgical practice it may be injured during parotid gland surgery or invaded by malignancies of the gland and a lower motor neurone on the ipsilateral side will result.
Vestibulo-cochlear nerve Exits from the pons and then passes through the internal auditory meatus. It is implicated in sensorineural hearing loss. Individuals with sensorineural hearing loss will localise the sound in webers test to the normal ear. Rinnes test will be reduced on the affected side but should still work. These two tests will distinguish sensorineural hearing loss from conductive deafness. In the latter condition webers test will localise to the affected ear and Rinnes test will be impaired on the affected side. Surgical lesions affecting this nerve include CNS tumours and basal skull fractures. It may also be damaged by the administration of ototoxic drugs (of which gentamicin is the most commonly used in surgical practice).
Glossopharyngeal nerve Exits the pons just above the vagus. Receives sensory fibres from posterior 1/3 tongue, tonsils, pharynx and middle ear (otalgia may occur following tonsillectomy). It receives visceral afferents from the carotid bodies. It supplies parasympathetic fibres to the parotid gland via the otic ganglion and motor function to stylopharyngeaus muscle. The sensory function of the nerve is tested using the gag reflex.
Vagus nerve Leaves the medulla between the olivary nucleus and the inferior cerebellar peduncle. Passes through the jugular foramen and into the carotid sheath. Details of the functions of the vagus nerve are covered in the website under relevant organ sub headings.
Accessory nerve Exists from the caudal aspect of the brainstem (multiple branches) supplies trapezius and sternocleidomastoid muscles. The distal portion of this nerve is most prone to injury during surgical procedures.
Hypoglossal nerve Emerges from the medulla at the preolivary sulcus, passes through the hypoglossal canal. It lies on the carotid sheath and passes deep to the posterior belly of digastric to supply muscles of the tongue (except palatoglossus). Its location near the carotid sheath makes it vulnerable during carotid endarterectomy surgery and damage will produce ipsilateral defect in muscle function.

1648
Q

A builder falls off a ladder whilst laying roof tiles. He sustains a burst fracture of L2. The MRI scan shows complete nerve transection at this level, as a result of the injury. Which clinical sign will not be present initially?

Flaccid paralysis of the legs

Extensor plantar response

Sensory loss in the legs

Incontinence

Areflexia

A

In lower motor neuron lesions everything is reduced

The main purpose of this question is to differentiate the features of an UMN lesion and a LMN lesion. The features of a LMN lesion include:
Flaccid paralysis of muscles supplied
Atrophy of muscles supplied.
Loss of reflexes of muscles supplied.
Muscles fasciculation
For lesions below L1 LMN signs will occur. Hence in an L3 lesion, there will be loss of the patella reflex but there will be no extensor plantar reflex.

Spinal cord

  • Located in a canal within the vertebral column that affords it structural support.
    Rostrally it continues to the medulla oblongata of the brain and caudally it tapers at a level corresponding to the L1-2 interspace (in the adult), a central structure, the filum terminale anchors the cord to the first coccygeal vertebra.
    The spinal cord is characterised by cervico-lumbar enlargements and these, broadly speaking, are the sites which correspond to the brachial and lumbar plexuses respectively.

There are some key points to note when considering the surgical anatomy of the spinal cord:

  • During foetal growth the spinal cord becomes shorter than the spinal canal, hence the adult site of cord termination at the L1-2 level.
  • Due to growth of the vertebral column the spine segmental levels may not always correspond to bony landmarks as they do in the cervical spine.
  • The spinal cord is incompletely divided into two symmetrical halves by a dorsal median sulcus and ventral median fissure. Grey matter surrounds a central canal that is continuous rostrally with the ventricular system of the CNS.
  • The grey matter is sub divided cytoarchitecturally into Rexeds laminae.
  • Afferent fibres entering through the dorsal roots usually terminate near their point of entry but may travel for varying distances in Lissauers tract. In this way they may establish synaptic connections over several levels
  • At the tip of the dorsal horn are afferents associated with nociceptive stimuli. The ventral horn contains neurones that innervate skeletal muscle.

The key point to remember when revising CNS anatomy is to keep a clinical perspective in mind. So it is worth classifying the ways in which the spinal cord may become injured. These include:

Trauma either direct or as a result of disc protrusion
Neoplasia either by direct invasion (rare) or as a result of pathological vertebral fracture
Inflammatory diseases such as Rheumatoid disease, or OA (formation of osteophytes compressing nerve roots etc.
Vascular either as a result of stroke (rare in cord) or as complication of aortic dissection
Infection historically diseases such as TB, epidural abscesses.

The anatomy of the cord will, to an extent dictate the clinical presentation. Some points/ conditions to remember:

Brown- Sequard syndrome-Hemisection of the cord producing ipsilateral loss of proprioception and upper motor neurone signs, plus contralateral loss of pain and temperature sensation. The explanation of this is that the fibres decussate at different levels.
Lesions below L1 will tend to present with lower motor neurone signs

1649
Q

A 73 year old lady presents with a mass in her neck. For many years she had worked as a radiographer. Imaging is highly suggestive of thyroid gland malignancy. Which of the tumour types listed below represents the most likely underlying diagnosis?

Follicular thyroid cancer

Medullary thyroid cancer

Papillary thyroid cancer

Anaplastic thyroid cancer

Thyroid gland lymphoma

A

Non therapeutic irradiation of the neck is most closely linked to papillary gland cancer.

Thyroid gland disorders-cancer

Thyroid cancer is a relatively rare condition and overall will comprise approximately 1-2% of all solid tumours in a 1 year period. They are classified as being papillary, follicular, anaplastic, lymphoma and sarcomatous.

Presenting features
Typically presents as a discrete nodule or mass
Lymph node or haematogenous metastasis may be present
Locally advanced disease may invade/ compress adjacent structures with symptoms of dysphagia, dyspnoea or speech changes arising from nerve involvement

Key points in the history
Pattern of symptom onset
Any association with pain (cancers are usually painless)
Patients age (cancers most common in those aged >60 years)
History of radiation exposure is significant as this increases the risk
Family history of endocrine disease (MEN I/II)

Examination
Adequate exposure of the neck
Assess peripheral stigmata of thyroid status (most cancers occur in euthyroid patients)
Any overt stigmata of MEN
Is the mass visible?
Palpate the patient’s neck stood behind them
Palpate the mass whilst asking the patient to swallow to determine its relationship to the thyroid
Examine carefully for palpable neck nodes
Attempts to elicit Pembertons sign by having the patient elevate their arms may be useful in establishing the presence of a sub sternal goitre

Diagnosis
In all cases a USS must be performed
Where a lesion is identified an FNAC is the usual first line management
FNAC cannot accurately diagnose follicular lesions or lymphoma
Staging is with chest x-ray and sometimes bone scans. Where an adverse prognosis is suspected CT scanning is usually performed.

Lesions
Lesion Common features
Follicular adenoma
Usually present as a solitary thyroid nodule
Malignancy can only be excluded on formal histological assessment
Papillary carcinoma
Usually contain a mixture of papillary and colloidal filled follicles
Histologically tumour has papillary projections and pale empty nuclei
Seldom encapsulated
Lymph node metastasis predominate
Haematogenous metastasis rare
Account for 60% of thyroid cancers
Follicular carcinoma
May appear macroscopically encapsulated, microscopically capsular invasion is seen. Without this finding the lesion is a follicular adenoma.
Vascular invasion predominates
Multifocal disease rare
Account for 20% of all thyroid cancers
Anaplastic carcinoma
Most common in elderly females
Local invasion is a common feature
Account for 10% of thyroid cancers
Treatment is by resection where possible, palliation may be achieved through isthmusectomy and radiotherapy. Chemotherapy is ineffective.
Medullary carcinoma
Tumours of the parafollicular cells (C Cells)
C cells derived from neural crest and not thyroid tissue
Serum calcitonin levels often raised
Familial genetic disease accounts for up to 20% cases
Both lymphatic and haematogenous metastasis are recognised, nodal disease is associated with a very poor prognosis.

Management
Lesion Usual management
Lymphoma Chemotherapy
Anaplastic cancer Local debulking surgery, palliative radiotherapy
Papillary thyroid cancer Hemithyroidectomy (T1 disease) or total thyroidectomy and central compartmental nodal dissection (T2 or greater)
Follicular thyroid cancer Total thyroidectomy
Follicular adenoma Hemithyroidectomy (the diagnosis is based on resected tissue analysis)
Medullary thyroid cancer Total thyroidectomy

1650
Q

A 32 year old man falls from scaffolding and sustains an injury to his forearm. Clinical examination and x-ray shows that he has sustained a radial fracture with dislocation of the distal radio-ulna joint. What eponymous name is used to describe this injury?

Galeazzi

Monteggia’s

Smith’s

Colles’

Barton’s

A

Isolated fracture of the radius alone can occur but is rare. Always check for associated injury.

Eponymous fractures

Colles’ fracture (dinner fork deformity)
Fall onto extended outstretched hand
Classical Colles’ fractures have the following 3 features:

  1. Transverse fracture of the radius
  2. 1 inch proximal to the radio-carpal joint
  3. Dorsal displacement and angulation

Smith’s fracture (reverse Colles’ fracture)
Volar angulation of distal radius fragment (Garden spade deformity)
Caused by falling backwards onto the palm of an outstretched hand or falling with wrists flexed

Bennett’s fracture
Intra-articular fracture of the first carpometacarpal joint
Impact on flexed metacarpal, caused by fist fights
X-ray: triangular fragment at ulnar base of metacarpal

Monteggia’s fracture
Dislocation of the proximal radioulnar joint in association with an ulna fracture
Fall on outstretched hand with forced pronation
Needs prompt diagnosis to avoid disability

Galeazzi fracture
Radial shaft fracture with associated dislocation of the distal radioulnar joint
Direct blow

Pott’s fracture
Bimalleolar ankle fracture
Forced foot eversion

Barton’s fracture
Distal radius fracture (Colles’/Smith’s) with associated radiocarpal dislocation
Fall onto extended and pronated wrist
Involvement of the joint is a defining feature

Holstein Lewis Fracture
A HolsteinLewis fracture is a fracture of the distal third of the humerus resulting in entrapment of the radial nerve.
The radial nerve is one of the major nerves of the upper limb. It innervates all of the muscles in the extensor compartments of the arm.
Conservative treatment includes reduction and use of a functional brace
Vascular injury may require open surgery

1651
Q

A 34 year old gardener presents with a lesion affecting the dorsum of his right hand. It has been present for the past 10 days and occurred after he had been pruning rose bushes. On examination, he has a raised ulcerated lesion which bleeds easily on contact. What is the diagnosis?

Keratoacanthoma

Pyogenic granuloma

Basal cell carcinoma

Squamous cell carcinoma

Amelanotic melanoma

A

Trauma is a common precipitant of pyogenic granuloma and contact bleeding and ulceration are common.

Skin disorders in surgery- malignancy and related lesions

Non melanoma skin cancer (BCC and SCC) are some of the commonest types of human malignancy. Up to 80% of these are BCC’s with approximately 20% comprising SCC’s. The incidence of NMSC’s increases with age and whilst there is a female preponderance in those under 40 years of age, in latter life the sex incidence is roughly equal.
The vast majority of NMSC’s are related to UV light exposure. For SCC’s the major pattern is in chronic long term exposure. For BCC’s, the pattern of sporadic exposure with episodes of burning is more important. Organ transplant recipients have a markedly increased incidence of SCC, risk factors include length of immunosuppression, ethnic origin and associated sunlight exposure. Human papilloma virus DNA is found in the majority of transplant recipient SCC’s. In addition to this increased risk, transplant recipients are also more likely to develop locoregional recurrences following treatment.

Actinic keratosis and SCC
Actinic keratosis is viewed as a premalignant lesion because there are atypical keratinocytes present in the epidermis. In a person with 7 actinic keratosis the risks of subsequent SCC is of the order of 10% at 10 years. The primary lesion is a rough erythematous papule with a white to yellow scale. Lesions are typically clustered at sites of chronic sun exposure.

Squamous cell carcinoma in situ
Also known as Bowens disease the commonest presentation of in situ SCC is with an erythematous scaling patch or elevated plaque arising on sun exposed skin in an elderly patient. Lesions may arise de novo or from pre-existing actinic keratosis.
Pathologically there is full thickness atypia of dermal keratinocytes over a broad zone. Nuclear pleomorphism, apoptosis and abnormal mitoses are all seen.

Invasive SCC
The commonest clinical presentation of SCC is with an erythematous keratotic papule or nodule on a background of sun exposure. Ulceration may occur and both exophytic and endophytic areas may be seen. Regional lymphadenopathy may be present.
Pathologically there is downward proliferation of malignant cells and invasion of the basement membrane. Poorly differentiated lesions may show perineural invasion and require immunohistochemistry with S100 to distinguish them from melanomas (which stain strongly positive with this marker).

Basal cell carcinoma
Nodular BCC Commonest variant (60%)
Raised translucent papule
Usually affect the face
Large nodular BCC’s are locally destructive
Superficial BCC Usually appears as superficial erythematous macule affecting the trunk
Younger age at presentation (mean 57)
May show areas of spontaneous regression
Horizontal growth pattern predominates
High recurrence rate (due to sub clinical lateral spread)
Morpheaform BCC Macroscopically resembles flat, slightly atrophic lesion or plaque without well defined borders
Tumour has sub clinical lateral spread which increases recurrence rates
Cystic BCC Often have clear or blue - grey appearance
Cystic degeneration may not be clinically obvious and tumour may resemble nodular BCC
Basosquamous carcinoma Atypical BCC
Basaloid histological BCC features with eosinophillic squamoid features of SCC
Biologically more aggressive and are more locally destructive
Rare lesion accounts for 1% of all non melanoma skin cancers
Metastatic disease may occur in 9-10% of cases and resemble an SCC

Keratoacanthoma
Dome shaped erythematous lesions that develop over a period of days and grow rapidly. They often contain a central pit of keratin. They then begin to necrose and slough off. They are generally benign lesions although some do view them as precursors of malignancy. They may be treated by curettage and cautery. If there is diagnostic doubt (they can mimic malignancy) then formal excision biopsy is warranted.

Pyogenic granuloma
These present as friable overgrowths of granulation at sites of minor trauma. They may be ulcerated and bleeding on contact is common. They may be treated with curretage and cautery, formal excision may be used if there is diagnostic doubt.

1652
Q

Why is a 73 year old lady receiving a course of intravesical BCG therapy?

Bladder tuberculosis

Transitional cell carcinoma of the bladder

Adenocarcinoma of the bladder

Squamous cell carcinoma of the bladder

Rhabdomyosarcoma of the bladder

A

Transitional cell carcinoma of the bladder may be treated with intravesical BCG therapy.

Haematuria

Causes of haematuria

Trauma
Injury to renal tract
Renal trauma commonly due to blunt injury (others penetrating injuries)
Ureter trauma rare: iatrogenic
Bladder trauma: due to RTA or pelvic fractures
Infection
Remember TB
Malignancy
Renal cell carcinoma (remember paraneoplastic syndromes): painful or painless
Urothelial malignancies: 90% are transitional cell carcinoma, can occur anywhere along the urinary tract. Painless haematuria.
Squamous cell carcinoma and adenocarcinoma: rare bladder tumours
Prostate cancer
Penile cancers: SCC
Renal disease
Glomerulonephritis
Stones
Microscopic haematuria common
Structural abnormalities
Benign prostatic hyperplasia (BPH) causes haematuria due to hypervascularity of the prostate gland
Cystic renal lesions e.g. polycystic kidney disease
Vascular malformations
Renal vein thrombosis due to renal cell carcinoma
Coagulopathy
Causes bleeding of underlying lesions
Drugs
Cause tubular necrosis or interstitial nephritis: aminoglycosides, chemotherapy
Interstitial nephritis: penicillin, sulphonamides, and NSAIDs
Anticoagulants
Benign
Exercise
Gynaecological
Endometriosis: flank pain, dysuria, and haematuria that is cyclical
Iatrogenic
Catheterisation
Radiotherapy; cystitis, severe haemorrhage, bladder necrosis
Pseudohaematuria For example following consumption of beetroot

References
Http://bestpractice.bmj.com/best-practice/monograph/316/overview/aetiology.html

1653
Q

Which of the following muscles is supplied by the external laryngeal nerve?

Transverse arytenoid

Cricothyroid

Thyro-arytenoid

Posterior crico-arytenoid

Oblique arytenoid

A

The others are all supplied by the recurrent laryngeal nerve.

Larynx

The larynx lies in the anterior part of the neck at the levels of C3 to C6 vertebral bodies. The laryngeal skeleton consists of a number of cartilagenous segments. Three of these are paired; arytenoid, corniculate and cuneiform. Three are single; thyroid, cricoid and epiglottic. The cricoid cartilage forms a complete ring (the only one to do so).
The laryngeal cavity extends from the laryngeal inlet to the level of the inferior border of the cricoid cartilage.

Divisions of the laryngeal cavity
Laryngeal vestibule Superior to the vestibular folds
Laryngeal ventricle Lies between vestibular folds and superior to the vocal cords
Infraglottic cavity Extends from vocal cords to inferior border of the cricoid cartilage

The vocal folds (true vocal cords) control sound production. The apex of each fold projects medially into the laryngeal cavity. Each vocal fold includes:
Vocal ligament
Vocalis muscle (most medial part of thyroarytenoid muscle)
The glottis is composed of the vocal folds, processes and rima glottidis. The rima glottidis is the narrowest potential site within the larynx, as the vocal cords may be completely opposed, forming a complete barrier.

Muscles of the larynx
Muscle Origin Insertion Innervation Action
Posterior cricoarytenoid Posterior aspect of lamina of cricoid Muscular process of arytenoid Recurrent Laryngeal Abducts vocal fold
Lateral cricoarytenoid Arch of cricoid Muscular process of arytenoid Recurrent laryngeal Adducts vocal fold
Thyroarytenoid Posterior aspect of thyroid cartilage Muscular process of arytenoid Recurrent laryngeal Relaxes vocal fold
Transverse and oblique arytenoids Arytenoid cartilage Contralateral arytenoid Recurrent laryngeal Closure of intercartilagenous part of the rima glottidis
Vocalis Depression between lamina of thyroid cartilage Vocal ligament and vocal process of arytenoid cartilage Recurrent laryngeal Relaxes posterior vocal ligament, tenses anterior part
Cricothyroid Anterolateral part of cricoid Inferior margin and horn of thyroid cartilage External laryngeal Tenses vocal fold

Blood supply
Arterial supply is via the laryngeal arteries, branches of the superior and inferior thyroid arteries. The superior laryngeal artery is closely related to the internal laryngeal nerve. The inferior laryngeal artery is related to the inferior laryngeal nerve. Venous drainage is via superior and inferior laryngeal veins, the former draining into the superior thyroid vein and the latter draining into the middle thyroid vein, or thyroid venous plexus.

Lymphatic drainage
The vocal cords have no lymphatic drainage and this site acts as a lymphatic watershed.
Supraglottic part Upper deep cervical nodes
Subglottic part Prelaryngeal and pretracheal nodes and inferior deep cervical nodes
The aryepiglottic fold and vestibular folds have a dense plexus of lymphatics associated with them and malignancies at these sites have a greater propensity for nodal metastasis.

1654
Q

A 30-year-old lady presents with a swelling over her right knee. Movements of her knee are restricted. A plain x-ray of the affected site shows multiple lytic and lucent lesions with clearly defined borders. What is the most likely diagnosis?

Osteoclastoma

Osteoblastoma

Osteosarcoma

Ewings sarcoma

Osteoid osteoma

A

Osteoclastoma has a characteristic appearance on x-ray with multiple lytic and lucent areas (Soap bubble) appearances. Pathological fractures may occur. The disease is usually indolent.

Bone tumours

Benign bone tumours
These are relatively common and typically occur in the young. Most will cease growth once skeletal maturity is attained. A key aspect to managing benign bone tumours lies in the exclusion of a malignant bone tumour. Plain films, CT, MRI and sometimes a bone biopsy may be required.

Benign tumours
Tumour Features
Osteoid osteoma M>F
Commonest between 10 and 25 years
Severe pain that responds to NSAIDS
Femur and tibia usually affected
Lesion affects cortex and radiologically consists of a lucent centre surrounded by reactive sclerosis
Usually smaller than 1cm
Chondroma Common benign tumour
Endochondroma are central lesions that may cause cortical thinning
Ecchondroma project beyond the cortex
May cause pathological fracture
Rarely found in large bones
Osteochondroma One of the commonest benign bone tumours
Usually occurs on the surface of a metaphysis (usually femur, proximal humerus or tibia)
Pedunculated lesion arising from metaphysis
Long standing lesions
Do not grow once skeletal maturity reached
Bone cyst Commonest in adolescent boys
Occur in proximal femur and humerus
May result in pathological fracture
Does not affect growth plate
On imaging appears as ovoid radiolucent area with surrounding cortical thinning

Giant cell tumour of bone
These lesions sit between benign and malignant. Most are benign. They are relatively rare. They consist of multinucleated giant cells (osteoclast like). Malignancy in these lesions is rare and is of the order of 2%. They account for 20% of benign bone tumours. The lesions may present as an enlarging mass, with pain or pathological fracture. On imaging, these lesions have an epiphyseal location and have a characteristic ‘soap bubble’ appearance. They do not normally demonstrate sclerosis and have a sharply defined border. Histologically, they demonstrate multinucleated giant cells. Treatment is challenging since these lesions are locally aggressive and have a high rate of local recurrence following excision.

Malignant bone tumours
The commonest malignant bone tumour is bone metastasis from another solid organ malignancy. Metastatic lesions to bone are significantly more common than primary bone tumours which are extremely rare.

Osteosarcoma
This is the most common form of primary bone tumour. It typically affects males in their second decade. They typically present as a painful expanding mass affecting a long bone such as the femur, in close proximity to a joint. Imaging demonstrates a lesion with mixed sclerotic and lucent zones in the metaphysis bone formation within the tumour is often visible. All patients will be fully staged at presentation with CT, MRI and often PET scanning. There is a high rate of metastasis to the lungs and most patients will receive chemotherapy. Surgical excision is the main treatment.

Ewings sarcoma
Ewings sarcomas typically affect long bones and typically occurs during the first two decades of life. They are complex tumours and have overlap with primitive neuroectodermal tumours. They have a predilection for the lower extremity. They typically present as a painful swelling affecting a long bone. Inflammatory markers such as the ESR are sometimes raised and this is associated with a poorer prognosis. On plain films, the tumour shows evidence of bone destruction with a periosteal reaction (onion skin) appearance. As the condition progresses the periosteal findings can change as the tumour invades the periosteum. Treatment is with excisional surgery and chemotherapy.

Secondary malignant tumours of bone
Metastatic bone tumours may be described as blastic, lytic or mixed. Osteoblastic metastatic disease has the lowest risk of spontaneous fracture when compared to osteolytic lesions of a similar size.
Lesions affecting the peritrochanteric region are most prone to spontaneous fracture (because of loading forces at that site).
The factors are incorporated into the Mirel Scoring system to stratify the risk of spontaneous fracture for bone metastasis of varying types.

Mirel Scoring system

Score points Site Radiographic appearance Width of bone involved Pain
1 Upper extremity Blastic Less than 1/3 Mild
2 Lower extremity Mixed 1/3 to 2/3 Moderate
3 Peritrochanteric Lytic More than 2/3 Aggravated by function

Depending upon the score the treatment should be as follows:

Score Risk of fracture Treatment
9 or greater Impending (33%) Prophylactic fixation
8 Borderline Consider fixation
7 or less Not impending (4%) Non operative management

1655
Q

A 34 year old lady is admitted with recurrent episodes of non-specific abdominal pain. On each admission all blood investigations are normal, as are her observations. On this admission a CT scan was performed. This demonstrates a 1.5cm nodule in the right adrenal gland. This is associated with a lipid rich core. Urinary VMA is within normal limits. Other hormonal studies are normal. What is the most likely diagnosis?

Malignant adrenal adenoma

Phaeochromocytoma

Metastatic lesion

Benign non functional adenoma

Benign functional adenoma

A

This is typical for a benign non functional adenoma. Benign adenomas often have a lipid rich core that is readily identifiable on CT scanning. In addition the nodules are often well circumscribed.

Phaeochromocytoma and adrenal lesions

Phaeochromocytoma
Neuroendocrine tumour of the chromaffin cells of the adrenal medulla. Hypertension and hyperglycaemia are often found.
10% of cases are bilateral.
10% occur in children.
11% are malignant (higher when tumour is located outside the adrenal).
10% will not be hypertensive.

Familial cases are usually linked to the Multiple endocrine neoplasia syndromes (considered under its own heading).

Most tumours are unilateral (often right sided) and smaller than 10cm.

Diagnosis
Urine and plasma metanephrine levels.

CT and MRI scanning are both used to localise the lesion.

Treatment
Patients require medical therapy first. An irreversible alpha adrenoreceptor blocker should be given, although minority may prefer reversible blockade(1). Labetolol may be co-administered for cardiac chronotropic control. Isolated beta blockade should not be considered as it will lead to unopposed alpha activity.

These patients are often volume depleted and will often require moderate volumes of intra venous normal saline perioperatively.

Once medically optimised the phaeochromocytoma should be removed. Most adrenalectomies can now be performed using a laparoscopic approach(2). The adrenals are highly vascular structures and removal can be complicated by catastrophic haemorrhage in the hands of the inexperienced. This is particularly true of right sided resections where the IVC is perilously close. Should the IVC be damaged a laparotomy will be necessary and the defect enclosed within a Satinsky style vascular clamp and the defect closed with prolene sutures. Attempting to interfere with the IVC using any instruments other than vascular clamps will result in vessel trauma and make a bad situation much worse.

Incidental adrenal lesions
Adrenal lesions may be identified on CT scanning performed for other reasons(3). Factors suggesting benign disease on CT include(4):
Size less than 3cm
Homogeneous texture
Lipid rich tissue
Thin wall to lesion

All patients with incidental lesions should be managed jointly with an endocrinologist and full work up as described above. Patients with functioning lesions or those with adverse radiological features (Particularly size >3cm) should proceed to surgery.

References
1. Weingarten TN, Cata JP, O’Hara JF, Prybilla DJ, Pike TL, Thompson GB, et al. Comparison of two preoperative medical management strategies for laparoscopic resection of pheochromocytoma. Urology. 2010 Aug;76(2):508 e6-11.

  1. Nguyen PH, Keller JE, Novitsky YW, Heniford BT, Kercher KW. Laparoscopic approach to adrenalectomy: review of perioperative outcomes in a single center. Am Surg. 2011 May;77(5):592-6.
  2. Ng VW, Ma RC, So WY, Choi KC, Kong AP, Cockram CS, et al. Evaluation of functional and malignant adrenal incidentalomas. Arch Intern Med. 2010 Dec 13;170(22):2017-20.
  3. Muth A, Hammarstedt L, Hellstrom M, Sigurjonsdottir HA, Almqvist E, Wangberg B. Cohort study of patients with adrenal lesions discovered incidentally. Br J Surg. 2011 May 27.
1656
Q

A 48 year old lady is admitted with attacks of biliary colic and investigations show gallstones. A laparoscopic cholecystectomy is performed. The operation is technically challenging due to a large stone impacted in Hartmans pouch. Following the operation she fails to settle and becomes jaundiced and has bile draining into a drain placed at the surgical site. What is the best course of action?

Arrange an ERCP

Undertake a laparotomy

Arrange an abdominal USS

Arrange an MRCP

Arrange an abdominal CT scan

A

In this scenario it must be assumed that the bile duct has been damaged. In most cases an ERCP is the most appropriate investigation. This can also allow the passage of a stent if this is deemed to be safe and sensible. Other imaging modalities will add little to overall management and simply add delay.

Complications following cholecystectomy

Complications following cholecystectomy
Open conversion
One of the largest series, published by Shea in 1996 with a total of more than 70,000 cholecystectomies analysed by pooled studies tends to list most complications as having an incidence of <1%. This includes open conversion which they identified as occurring in less than 1%. One of the interesting points raised by this data is that it covers an early period in laparoscopy and there is highly likely to be an element of selection bias in the cases put forward for laparoscopy. Indeed, some surgeons have noted their own institutions conversion rates increase (in this case 7%) as they take on increasingly complex cases.

Bile leaks
Where these occur they are usually the result of leakage from biliary radicles or cystic stump leaks. Many patients with a cystic stump leak can be managed with ERCP , sphincterotomy and stent. They are not common complications and have an incidence of around 1%.

Bile duct injury
Cholecystectomy remains the commonest cause of bile duct injury. Reported incidences range from 0.3-0.7%. The main reason is that biliary anatomy is not appreciated. Where they occur, the definitive management is reconstruction. If the operating surgeon does not regularly practise this type of surgery then the area should be drained and the patient transferred to an HPB unit.

1657
Q

A 34 year old lady presents with a long standing offensive discharge from the ear and on examination is noted to have a reduction in her hearing of 40 decibels compared to the opposite side. What is the most likely cause?

Otosclerosis

Acute otitis media

Cholesteatoma

Long standing perforation of the pars tensa

Otitis externa

A

The combination of long history, offensive discharge and hearing loss is strongly suggestive of cholesteatoma.

Disorders affecting the ear

Otitis externa
Variant Cause Features Treatment
Acute otitis externa Boil in external auditory meatus Acute pain on moving the pinna
Conductive hearing loss if lesion is large
When rupture occurs pus will flow from ear Ear packs may be used
Topical antibiotics
Operative debridement may be needed in severe cases
Chronic otitis externa Chronic combined infection in the external auditory meatus usually combined staphylococcal and fungal infection Chronic discharge from affected ear, hearing loss and severe pain rare Cleansing of the external ear and treatment with antifungal and antibacterial ear drops

Otitis media
Variant Cause Features Treatment
Acute suppurative otitis media Viral induced middle ear effusions secondary to eustacian tube dysfunction Most common in children and rare in adults
May present with symptoms elsewhere (e.g. vomiting) in children
Severe pain and sometimes fever
May present with discharge if tympanic rupture occurs Antibiotics (usually amoxycillin)
Chronic suppurative otitis media May occur with or without cholesteatoma
Those without cholesteatoma have a perforation of the pars tensa
Those with cholesteatoma have a perforation of the pars flaccida Those without cholesteatoma may complain of intermittent discharge (non offensive)
Those with cholesteatoma have impaired hearing and foul smelling discharge Simple pars tensa perforations may be managed non operatively or a myringoplasty considered if symptoms troublesome.
Pars flaccida perforations will usually require a modified radical mastoidectomy

Otosclerosis
Progressive conductive deafness
Secondary to fixation of the stapes in the oval window
Treatment is with stapedectomy and insertion of a prosthesis

Acoustic neuroma
Symptoms of gradually progressive unilateral perceptive deafness and tinnitus
Involvement of the vestibular nerve may cause vertigo
Extension to involve the facial nerve may cause weakness and then paralysis.

Pre auricular sinus
Common congenital condition in which an epithelial defect forms around the external ear
Small sinuses require no treatment
Deeper sinuses may become blocked and develop episodes of infection, they may be closely related to the facial nerve and are challenging to excise

1658
Q

A 21 year old lady presents with a nodule on the posterior aspect of her right calf. It has been present at the site for the past 6 months and occurred at the site of a previous insect bite. Although the nodule appears small, on palpation it appears to be nearly twice the size it appears on examination. The overlying skin is faintly pigmented. What is the most likely cause?

Pilar cyst

Dermatofibroma

Spitz naevus

Seborrhoeic wart

Basal cell carcinoma

A

Dermatofibromas may be pigmented and are often larger than they appear. They frequently occur at sites of previous trauma.

Benign skin diseases

Seborrhoeic keratosis
Most commonly arise in patients over the age of 50 years, often idiopathic
Equal sex incidence and prevalence
Usually multiple lesions over face and trunk
Flat, raised, filiform and pedunculated subtypes are recognised
Variable colours and surface may have greasy scale overlying it
Treatment options consist of leaving alone or simple shave excision

Melanocytic naevi
Congenital melanocytic naevi
Typically appear at, or soon after, birth
Usually greater than 1cm diameter
Increased risk of malignant transformation (increased risk greatest for large lesions)
Junctional melanocytic naevi
Circular macules
May have heterogeneous colour even within same lesion
Most naevi of the palms, soles and mucous membranes are of this type
Compound naevi
Domed pigmented nodules up to 1cm in diameter
Arise from junctional naevi, usually have uniform colour and are smooth
Spitz naevus
Usually develop over a few months in children
May be pink or red in colour, most common on face and legs
May grow up to 1cm and growth can be rapid, this usually results in excision
Atypical naevus syndrome
Atypical melanocytic naevi that may be autosomally dominantly inherited
Some individuals are at increased risk of melanoma (usually have mutations of CDKN2A gene)
Many people with atypical naevus syndrome AND a parent sibling with melanoma will develop melanoma

Epidermoid cysts
Common and affect face and trunk
They have a central punctum, they may contain small quantities of sebum
The cyst lining is either normal epidermis (epidermoid cyst) or outer root sheath of hair follicle (pilar cyst)

Dermatofibroma
Solitary dermal nodules
Usually affect extremities of young adults
Lesions feel larger than they appear visually
Histologically they consist of proliferating fibroblasts merging with sparsely cellular dermal tissues

Painful skin lesions
Eccrine spiradenoma
Neuroma
Glomus tumour
Leiomyoma
Angiolipoma
Neurofibroma (rarely painful) and dermatofibroma (rarely painful)

1659
Q

A 56 year old woman has undergone a wide local excision and axillary node sample. The histology shows a 2.5cm invasive ductal carcinoma grade 1 which is completely excised. None of the axillary lymph nodes show evidence of metastatic disease. What is the most appropriate course of action?

Arrange routine surgical follow up

Completion mastectomy

Radiotherapy to breast and axilla

Administration of tamoxifen

Radiotherapy to breast alone

A

This woman has a good prognosis. In younger women, DXT following breast conserving surgery is still the standard of care. In some older patients, it may be omitted.

Breast cancer management

  • Surgery is performed in most patients suffering from breast cancer.
    Chemotherapy may be used to downstage tumours and allow breast conserving surgery. Hormonal therapy may also be used for the same purposes.
    Radiotherapy is given to most patients who have undergone breast conserving surgery (some older patients receiving hormone treatment and who have small low grade tumours may safely avoid DXT.
    Therapeutic mammoplasty is an option for some patients but requires symmetrizing surgery in most cases.
    Patients who have undergone mastectomy may be offered a reconstructive procedure either in conjunction with their primary resection or as a staged procedure at a later date.

Surgical options
Mastectomy vs Wide local excision

Mastectomy Wide Local Excision
Multifocal tumour Solitary lesion
Central tumour Peripheral tumour
Large lesion in small breast Small lesion in large breast
DCIS >4cm DCIS <4cm
Patient Choice Patient choice

Central lesions may be managed using breast conserving surgery, where an acceptable cosmetic result may be obtained, this is rarely the case in small breasts

Axillary disease
As a minimum, all patients with invasive breast cancer should have their axilla staged. In those who do not have overt evidence of axillary nodal involvement this can be undertaken using sentinel lymph node biopsy.
Patients with a positive sentinel lymph node biopsy or who have imaging and cytological or histological evidence of axillary nodal metastasis should undergo axillary node clearance or axillary irradiation.
Axillary node clearance is associated with the development of lymphoedema, increased risk of cellulitis and frozen shoulder.

1660
Q

Which of the tumour types listed below is found most frequently in a person with aggressive fibromatosis?

Medullary thyroid cancer

Basal cell carcinoma of the skin

Desmoid tumours

Dermoid tumours

Malignant melanoma

A

Aggressive fibromatosis

Aggressive fibromatosis is a disorder consisting of desmoid tumours, which behave in a locally aggressive manner. Desmoid tumours may be identified in both abdominal and extra-abdominal locations. Metastatic disease is rare. The main risk factor (for abdominal desmoids) is having APC variant of familial adenomatous polyposis coli. Most cases are sporadic.
Treatment is by surgical excision.

1661
Q

A 17 year old lady presents with right iliac fossa pain and diagnosed as having acute appendicitis. You take her to theatre to perform a laparoscopic appendicectomy. During the procedure the scrub nurse distracts you and you inadvertently avulse the appendicular artery. The ensuing haemorrhage is likely to be supplied directly from which vessel?

Inferior mesenteric artery

Superior mesenteric artery

Ileo-colic artery

Internal iliac artery

None of the above

A

The appendicular artery is a branch of the ileocolic artery.

Appendix

Location: Base of caecum.
Up to 10cm long.
Mainly lymphoid tissue (Hence mesenteric adenitis may mimic appendicitis).
Caecal taenia coli converge at base of appendix and form a longitudinal muscle cover over the appendix. This convergence should facilitate its identification at surgery if it is retrocaecal and difficult to find (which it can be when people start doing appendicectomies!)
Arterial supply: Appendicular artery (branch of the ileocolic).
It is intra peritoneal.

McBurney’s point
1/3 of the way along a line drawn from the Anterior Superior Iliac Spine to the Umbilicus

6 Positions:

Retrocaecal 74%
Pelvic 21%
Postileal
Subcaecal
Paracaecal
Preileal

1662
Q

A 72 year old woman fractured her distal radius. A Biers Block is planned to facilitate reduction of the fracture. Which of the following agents should be used?

1% lignocaine

1% lignocaine with 0.5% bupivacaine mixed in 50:50 mixture

1% prilocaine

1% lignocaine with 1 in 200,000 adrenaline

1% prilocaine with 1 in 200,000 adrenaline

A

This is the best local anaesthetic for this. Bupivacaine may cause cardiotoxicity and should be avoided.

Local anaesthetic agents

Lidocaine
An amide
Local anaesthetic and a less commonly used antiarrhythmic (affects Na channels in the axon)
Hepatic metabolism, protein bound, renally excreted
Toxicity: due to IV or excess administration. Increased risk if liver dysfunction or low protein states. Note acidosis causes lidocaine to detach from protein binding.
Drug interactions: Beta blockers, ciprofloxacin, phenytoin
Features of toxicity: Initial CNS over activity then depression as lidocaine initially blocks inhibitory pathways then blocks both inhibitory and activating pathways. Cardiac arrhythmias.
Increased doses may be used when combined with adrenaline to limit systemic absorption.

Cocaine
Pure cocaine is a salt, usually cocaine hydrochloride. It is supplied for local anaesthetic purposes as a paste.
It is supplied for clinical use in concentrations of 4 and 10%. It may be applied topically to the nasal mucosa. It has a rapid onset of action and has the additional advantage of causing marked vasoconstriction.
It is lipophillic and will readily cross the blood brain barrier. Its systemic effects also include cardiac arrhythmias and tachycardia.
Apart from its limited use in ENT surgery it is otherwise used rarely in mainstream surgical practice.

Bupivacaine
Bupivacaine binds to the intracellular portion of sodium channels and blocks sodium influx into nerve cells, which prevents depolarization.
It has a much longer duration of action than lignocaine and this is of use in that it may be used for topical wound infiltration at the conclusion of surgical procedures with long duration analgesic effect.
It is cardiotoxic and is therefore contra indicated in regional blockage in case the tourniquet fails.
Levobupivacaine (Chirocaine) is less cardiotoxic and causes less vasodilation.

Prilocaine
Similar mechanism of action to other local anaesthetic agents. However, it is far less cardiotoxic and is therefore the agent of choice for intravenous regional anaesthesia e.g. Biers Block.

All local anaesthetic agents dissociate in tissues and this contributes to their therapeutic effect. The dissociation constant shifts in tissues that are acidic e.g. where an abscess is present, and this reduces the efficacy.

Doses of local anaesthetics
Agent Dose plain Dose with adrenaline
Lignocaine 3mg/Kg 7mg/Kg
Bupivacaine 2mg/Kg 2mg/Kg
Prilocaine 6mg/Kg 9mg/Kg
These are a guide only as actual doses depend on site of administration, tissue vascularity and co-morbidities.

Maximum total local anaesthetic doses
Lignocaine 1% plain - 3mg/ Kg - 200mg (20ml)
Lignocaine 1% with 1 in 200,000 adrenaline - 7mg/Kg - 500mg (50ml)
Bupivacaine 0.5% - 2mg/kg- 150mg (30ml)
Maximum doses are based on ideal body weight

Effects of adrenaline
Adrenaline may be added to local anaesthetic drugs. It prolongs the duration of action at the site of injection and permits usage of higher doses (see above). It is contra indicated in patients taking MAOI’s or tricyclic antidepressants. The toxicity of bupivacaine is related to protein binding and addition of adrenaline to this drug does not permit increases in the total dose of bupivacaine, in contrast to the situation with lignocaine.

References
An excellent review is provided by:
French J and Sharp L. Local Anaesthetics. Ann R Coll Surg Engl 2012; 94: 76-80.

1663
Q

A 75 year old lady reports urinary incontinence when coughing and sneezing. She has had 2 children with no complications. She has no significant past medical history and is on no medications. What is the most appropriate initial management?

Trial of sacral neuromodulation

Urodynamic studies

Pelvic floor exercises for 3 months

Bladder drill training for 6 weeks

Administration of oxybutinin

A

A diagnosis of stress incontinence is obvious from the history, therefore there is no need for a bladder diary or urodynamic studies. Pelvic floor exercises would be the first line management.

Urinary incontinence

Involuntary passage of urine. Most cases are female (80%). It has a prevalence of 11% in those aged greater than 65 years. The commonest variants include:
Stress urinary incontinence (50%)
Urge incontinence (15%)
Mixed (35%)

Males
Males may also suffer from incontinence although it is a much rarer condition in men. A number of anatomical factors contribute to this. Males have 2 powerful sphincters; one at the bladder neck and the other in the urethra. Damage to the bladder neck mechanism is a factor in causing retrograde ejaculation following prostatectomy. The short segment of urethra passing through the urogenital diaphragm consists of striated muscle fibres (the external urethral sphincter) and smooth muscle capable of more sustained contraction. It is the latter mechanism that maintains continence following prostatectomy.

Females
The sphincter complex at the level of bladder neck is poorly developed in females. As a result the external sphincter complex is functionally more important, its composition being similar to that of males. Innervation is via the pudendal nerve and the neuropathy that may accompany obstetric events may compromise this and lead to stress urinary incontinence.

Innervation
Somatic innervation to the bladder is via the pudendal, hypogastric and pelvic nerves. Autonomic nerves travel in these nerve fibres too. Bladder filling leads to detrusor relaxation (sympathetic) coupled with sphincter contraction. The parasympathetic system causes detrusor contraction and sphincter relaxation. Overall control of micturition is centrally mediated via centres in the Pons.

Stress urinary incontinence
50% of cases, especially in females.
Damage (often obstetric) to the supporting structures surrounding the bladder may lead to urethral hypermobility.
Other cases due to sphincter dysfunction, usually from neurological disorders (e.g. Pudendal neuropathy, multiple sclerosis).

Urethral mobility:
Pressure not transmitted appropriately to the urethra resulting in involuntary passage of urine during episodes of raised intra-abdominal pressure.

Sphincter dysfunction:
Sphincter fails to adapt to compress urethra resulting in involuntary passage of urine. When the sphincter completely fails there is often to continuous passage of urine.

Urge incontinence
In these patients there is sense of urgency followed by incontinence. The detrusor muscle in these patients is unstable and urodynamic investigation will demonstrate overactivity of the detrusor muscle at inappropriate times (e.g. Bladder filling). Urgency may be seen in patients with overt neurological disorders and those without. The pathophysiology is not well understood but poor central and peripheral co-ordination of the events surrounding bladder filling are the main processes.

Assessment
Careful history and examination including vaginal examination for cystocele.
Bladder diary for at least 3 days
Consider flow cystometry if unclear symptomatology or surgery considered and diagnosis is unclear.
Exclusion of other organic disease (e.g. Stones, UTI, Cancer)

Management
Conservative measures should be tried first; Stress urinary incontinence or mixed symptoms should undergo 3 months of pelvic floor exercise. Over active bladder should have 6 weeks of bladder retraining.
Drug therapy for women with overactive bladder should be offered oxybutynin (or solifenacin if elderly) if conservative measures fail.
In women with detrusor instability who fail non operative therapy a trial of sacral neuromodulation may be considered, with conversion to permanent implant if good response. Augmentation cystoplasty is an alternative but will involve long term intermittent self catheterisation.
In women with stress urinary incontinence a urethral sling type procedure may be undertaken. Where cystocele is present in association with incontinence it should be repaired particularly if it lies at the introitus.

NICE guidelines
Initial assessment urinary incontinence should be classified as stress/urge/mixed.
At least 3/7 bladder diary if unable to classify easily.
Start conservative treatment before urodynamic studies if a diagnosis is obvious from the history
Urodynamic studies if plans for surgery.
Stress incontinence: Pelvic floor exercises 3/12, if fails consider surgery.
Urge incontinence: Bladder training >6/52, if fails for oxybutynin (antimuscarinic drugs) then sacral nerve stimulation.
Pelvic floor exercises offered to all women in their 1st pregnancy.

1664
Q

A male infant, born at term by normal vaginal delivery is well. However, 72 hours following delivery, he has still not passed meconium. On examination, his abdomen is soft and not particularly distended. He has a normally sited anus. What is the best course of action?

Undertake a colonoscopy and pan colonic biopsies

Undertake a flexible sigmoidoscopy and left sided colonic biopsies

Undertake full thickness rectal biopsies

Undertake a loop colostomy

Undertake a loop ileostomy

A

Delayed passage of meconium is suggestive of Hirschsprung’s disease and the investigation of choice is full thickness suction rectal biopsy. The condition is managed with washouts initially. At between 9 and 12 months of age, definitive surgery (usually resection and primary anastomosis) is performed.

Paediatric Gastrointestinal disorders

Pyloric stenosis
M>F
5-10% Family history in parents
Projectile non bile stained vomiting at 4-6 weeks of life
Diagnosis is made by test feed or USS
Treatment: Ramstedt pyloromyotomy (open or laparoscopic)
Acute appendicitis
Uncommon under 3 years
When occurs may present atypically
Mesenteric adenitis
Central abdominal pain and URTI
Conservative management
Intussusception
Telescoping bowel
Proximal to or at the level of, ileocaecal valve
6-9 months age
Colicky pain, diarrhoea and vomiting, sausage shaped mass, red jelly stool.
Treatment: reduction with air insufflation
Malrotation
High caecum at the midline
Feature in exomphalos, congenital diaphragmatic hernia, intrinsic duodenal atresia
May be complicated by development of volvulus, infant with volvulus may have bile stained vomiting
Diagnosis is made by upper GI contrast study and USS
Treatment is by laparotomy, if volvulus is present (or at high risk of occurring then a ladds procedure is performed
Hirschsprung’s disease
Absence of ganglion cells from myenteric and submucosal plexuses
Occurs in 1/5000 births
Full thickness rectal biopsy for diagnosis
Delayed passage of meconium and abdominal distension
Treatment is with rectal washouts initially, thereafter an anorectal pull through procedure
Oesophageal atresia
Associated with tracheo-oesophageal fistula and polyhydramnios
May present with choking and cyanotic spells following aspiration
VACTERL associations
Meconium ileus
Usually delayed passage of meconium and abdominal distension
Majority have cystic fibrosis
X-Rays may not show a fluid level as the meconium is viscid (depends upon feeding), PR contrast studies may dislodge meconium plugs and be therapeutic
Infants who do not respond to PR contrast and NG N-acetyl cysteine will require surgery to remove the plugs
Biliary atresia
Jaundice > 14 days
Increased conjugated bilirubin
Urgent Kasai procedure
Necrotising enterocolitis
Prematurity is the main risk factor
Early features include abdominal distension and passage of bloody stools
X-Rays may show pneumatosis intestinalis and evidence of free air
Increased risk when empirical antibiotics are given to infants beyond 5 days
Treatment is with total gut rest and TPN, babies with perforations will require laparotomy

1665
Q

A 6 year old boy presents with symptoms of recurrent pruritus ani. On examination, there is evidence of a small worm like structure protruding from the anus. What is the most likely infective organism?

Echinococcus granulosus

Enterobius vermicularis

Ascaris lumbricoides

Ancylostoma duodenale

Clonorchis sinensis

A

Infection with enterobius is extremely common. Pruritus is the main symptom, as there is a lack of tissue invasion it is rare for individuals to have any signs of systemic sepsis.

Gastro intestinal parasitic infections

Common infections

Enterobiasis
Due to organism Enterobius vermicularis
Common cause of pruritus ani
Diagnosis usually made by placing scotch tape at the anus, this will trap eggs that can then be viewed microscopically
Treatment is with mebendazole
Ancylostoma duodenale
Hookworms that anchor in proximal small bowel
Most infections are asymptomatic although may cause iron deficiency anaemia
Larvae may be found in stools left at ambient temperature, otherwise infection is difficult to diagnose
Infection occurs as a result of cutaneous penetration, migrates to lungs, coughed up and then swallowed
Treatment is with mebendazole
Ascariasis
Due to infection with roundworm Ascaris lumbricoides
Infections begin in gut following ingestion, then penetrate duodenal wall to migrate to lungs, coughed up and swallowed, cycle begins again
Diagnosis is made by identification of worm or eggs within faeces
Treatment is with mebendazole
Strongyloidiasis
Due to infection with Strongyloides stercoralis
Rare in west
Organism is a nematode living in duodenum of host
Initial infection is via skin penetration. They then migrate to lungs and are coughed up and swallowed. Then mature in small bowel are excreted and cycle begins again
An auto infective cycle is also recognised where larvae will penetrate colonic wall
Individuals may be asymptomatic, although they may also have respiratory disease and skin lesions
Diagnosis is usually made by stool microscopy
In the UK mebendazole is used for treatment
Cryptosporidium
Protozoal infection
Organisms produce cysts which are excreted and thereby cause new infections
Symptoms consist of diarrhoea and cramping abdominal pains. Symptoms are worse in immunosuppressed people
Cysts may be identified in stools
Treatment is with metronidazole
Giardiasis
Diarrhoeal infection caused by Giardia lamblia (protozoan)
Infections occur as a result of ingestion of cysts
Symptoms are usually gastrointestinal with abdominal pain, bloating and passage of soft or loose stools
Diagnosis is by serology or stool microscopy
First line treatment is with metronidazole

1666
Q

A 51 year old man is shot in the abdomen and sustains a significant intra abdominal injury. A laparotomy, bowel resection and end colostomy are performed. An associated vascular injury necessitates a 6 unit blood transfusion. He has a prolonged recovery and is paralysed and ventilated for 2 weeks on intensive care. He receives total parenteral nutrition and is eventually weaned from the ventilator and transferred to the ward. On reviewing his routine blood tests the following results are noted:

Full blood count
Hb 11.3 g/dl
Platelets 267 x 109/l
WBC 10.1 x109/l

Urea and electrolytes
Na+ 131 mmol/l
K+ 4.6 mmol/l
Urea 2.3 mmol/l
Creatinine 78 µmol/l

Liver function tests
Bilirubin 25 µmol/l
ALP 445 u/l
ALT 89 u/l
γGT 103 u/l

What is the most likely underlying cause for the abnormalities noted?

Delayed type blood transfusion reaction

Bile leak

Anastomotic leak

Total parenteral nutrition

Gallstones
9%

A

TPN is known to result in derangement of liver function tests. Although, cholestasis may result from TPN, it would be very unusual for gallstones to form and result in the picture above. Blood transfusion reactions typically present earlier and with changes in the haemoglobin and although they may cause hepatitis this is rare nowadays.

Total parenteral nutrition

Commonly used in nutritionally compromised surgical patients.
Bags contain combinations of glucose, lipids and essential electrolytes, the exact composition is determined by the patients nutritional requirements.
Although it may be infused peripherally, this may result in thrombophlebitis.
Longer term infusions should be administered into a central vein (preferably via a PICC line).
Complications are related to sepsis, re-feeding syndromes and hepatic dysfunction.

1667
Q

A 23 year old man has taken an opiate overdose. Of the following structures, which will be most affected by opiates to produce a reduction in respiratory rate?

Carotid chemoreceptors

Central chemoreceptors

Medullary respiratory centre

Pneumotaxic centre

Juxtacapillary (J) receptors

A

Opiates typically affect the medullary respiratory centre to depress respiratory activity.

Control of ventilation

  • Control of ventilation is coordinated by the respiratory centres, chemoreceptors, lung receptors and muscles.
    Automatic, involuntary control of respiration occurs from the medulla.
    The respiratory centres control the respiratory rate and the depth of respiration.

Respiratory centres
Medullary respiratory centre Inspiratory and expiratory neurones. Has ventral group which controls forced voluntary expiration and the dorsal group controls inspiration. Depressed by opiates.
Apneustic centre Lower pons
Stimulates inspiration - activates and prolongs inhalation
Overridden by pneumotaxic control to end inspiration
Pneumotaxic centre Upper pons, inhibits inspiration at a certain point. Fine tunes the respiratory rate.

Ventillatory variables
Levels of pCO2 most important in ventilation control
Levels of O2 are less important.
Peripheral chemoreceptors: located in the bifurcation of carotid arteries and arch of the aorta. They respond to changes in reduced pO2, increased H+ and increased pCO2 in ARTERIAL BLOOD.
Central chemoreceptors: located in the medulla. Respond to increased H+ in BRAIN INTERSTITIAL FLUID to increase ventilation. NB the central receptors are NOT influenced by O2 levels.

Lung receptors include:
Stretch receptors: respond to lung stretching causing a reduced respiratory rate
Irritant receptors: respond to smoke etc causing bronchospasm
J (juxtacapillary) receptors

1668
Q

A 42 year old man is admitted to surgery with acute appendicitis. He is known to have hypertension, psoriatic arthropathy and polymyalgia rheumatica. His medical therapy includes:
Paracetamol 1g qds
Codeine phosphate 30mg qds
Bendrofluazide 2.5 mg od
Ramipril 10mg od
Methotrexate 7.5mg once a week
Prednisolone 5mg od
You are called by the core surgical trainee to assess this man as he has become delirious and hypotensive 2 hours after surgery. His blood results reveal:

Na+ 132 mmol/l
K+ 5.2 mmol/l
Urea 10 mmol/l
Creatinine 111 µmol/l
Glucose 3.5
CRP 158

Hb 10.2 g/dl
Platelets 156 * 109/l
WBC 14 * 109/l

What is the most likely diagnosis?

Septic shock secondary to appendicitis

Neutropenic sepsis

Phaeochromocytoma

Perforated bowel

Addisonian crisis

A

Features of an addisonian crisis:
Hyponatraemia
Hyperkalaemia
Hypoglycaemia
This man is on steroids for polymyalgia rheumatica. Surgery can precipitate acute adrenal deficiency. The diagnosis is further confirmed by the blood results of hyponatraemia, hyperkalaemia and hypoglycaemia. This patient urgently needs hydrocortisone.

Addisonian crisis

Causes
Sepsis or surgery causing an acute exacerbation of chronic insufficiency (Addison’s, Hypopituitarism)
Adrenal haemorrhage eg Waterhouse-Friderichsen syndrome (fulminant meningococcemia)
Steroid withdrawal

Management
Hydrocortisone 100 mg im or iv
1 litre normal saline infused over 30-60 mins or with dextrose if hypoglycaemic
Continue hydrocortisone 6 hourly until the patient is stable. No fludrocortisone is required because high cortisol exerts weak mineralocorticoid action
Oral replacement may begin after 24 hours and be reduced to maintenance over 3-4 days

1669
Q

Which of the variables listed below is not considered in the sequential organ failure assessment (SOFA) tool?

Bilirubin

Urea

Mean arterial pressure

Platelet count

Creatinine

A

Urea is not one of the variables considered.

Severe infections

Sepsis is a multifaceted syndrome which arises as a result of an infective process. Historically, the main focus was on the pro-inflammatory nature of the process, accordingly, 2 of the 4 SIRS criteria were related solely to inflammatory excesses. More modern systems take into account the fact that some of the effects of sepsis are suppressive from an immunological perspective and effects on organ function can be widespread. Sepsis is now defined as life-threatening organ dysfunction caused by a dysregulated host response to infection. This new definition emphasizes the primacy of the non homeostatic host response to infection, the potential lethality that is considerably in excess of a straightforward infection, and the need for urgent recognition. To help identify and categorise patients the Sequential (Sepsis-Related) Organ Failure Assessment Score (SOFA) is increasingly used. The score grades abnormality by organ system and accounts for clinical interventions. However, laboratory variables, namely, PaO2, platelet count, creatinine level, and bilirubin level, are needed for full computation.

SOFA Score
System Score 0 Score 1 Score 2 Score 3 Score 4
PaO2 /FI O2 >400 <400 <300 <200 <100
Platelets x103 microlitres >150 <150 <100 <50 <20
Bilirubin micro Mol/L 20 20-32 33-101 102-204 >204
Cardiovascular MAP >70mmHg MAP 70mmHg Dopamine <5 or dobutamine (any dose) Dopamine 5.1-15
or epinephrine 0.1
or norepinephrine 0.1 Dopamine >15 or
epinephrine >0.1
or norepinephrine >0.1
GCS 15 13-14 10-12 6-9 <6
Creatinine micro mol/L <110 110-170 171-299 300-440 >440
Urine output ml/day >500 >500 >500 <500 <200

A SOFA score of 2 or more reflects an overall mortality risk of approximately 10% in a general hospital population with suspected infection. Even patients presenting with modest dysfunction can deteriorate further, emphasizing the seriousness of this condition and the need for prompt and appropriate intervention, if not already being instituted.

The qSOFA score (also known as quickSOFA) is a bedside prompt that may identify patients with suspected infection who are at greater risk for a poor outcome outside the intensive care unit (ICU). It uses three criteria, assigning one point for low blood pressure (SBP<100 mmHg), high respiratory rate (>22 breaths per min), or altered mentation (Glasgow coma scale<15).

qSOFA score
Respiratory rate 22/min
Altered mentation
Systolic blood pressure 100 mm Hg

Patients with septic shock can be identified with a clinical construct of sepsis with persisting hypotension requiring vasopressors to maintain MAP 65 mm Hg and having a serum lactate level >2 mmol/L (18mg/dL) despite adequate volume resuscitation. With these criteria, hospital mortality is in excess of 40%.

Resuscitation goals
CVP 8-12mmHg
MAP >65mmHg
Urine output >0.5ml/kg per hour
Superior vena cava oxygen saturation >70%
Normal lactate

References
Singer M, Deutschman C, Seymour C et al. The third international consensus definitions for sepsis and septic shock (Sepsis 3). JAMA 2016; 315(8):801-810.

1670
Q

An 11 month-old girl develops sudden onset abdominal pain. Her mother stated that she had not been her usual self for the past 48 hours. Assessing her, she has a high pitched scream and draws up her legs. Her BP is 90/40 mm/Hg, her pulse 118/min and abdominal examination is normal. What is the most likely diagnosis?

Mid gut volvulus

Intussusception

Appendicitis

Mesenteric adenitis

Spontaneous bacterial peritonitis

A

Intussusception should be considered in toddlers and infants presenting with screaming attacks. The child often has a history of being unwell for one to three days prior to presentation. The child may pass bloody mucus stool, which is a late sign. Examination of the abdomen is often normal as the sausage mass in the right upper quadrant is difficult to feel.

Acute abdominal pain-diagnoses

Conditions presenting with acute abdominal pain
Condition Features Investigations Management
Appendicitis History of migratory pain.
Fever.
Anorexia.
Evidence of right iliac fossa tenderness.
Mild pyrexia. Differential white cell count
Pregnancy test
C-Reactive protein
Amylase
Urine dipstick testing Appendicectomy
Mesenteric adenitis Usually recent upper respiratory tract infection.
High fever.
Generalised abdominal discomfort- true localised pain and signs are rare. Full blood count- may show slightly raised white cell count
Urine dipstick often normal
Abdominal ultrasound scan - usually no free fluid Conservative management- appendicectomy if diagnostic doubt
Mittelschmerz Only seen in females
Mid cycle pain
Usually occurs two weeks after last menstrual period
Pain usually has a supra-pubic location
Usually subsides over a 24-48 hour period. Full blood count- normal
Urine dipstick- normal
Abdominal and pelvic ultrasound- may show a trace of pelvic free fluid Manage conservatively if doubt or symptoms fail to settle then laparoscopy
Fitz-Hugh Curtis syndrome Disseminated infection with Chlamydia.
Usually seen in females.
Consists of evidence of pelvic inflammatory disease together with peri-hepatic inflammation and subsequent adhesion formation. Abdominal ultrasound scan- may show free fluid
High vaginal swabs - may show evidence of sexually transmitted infections Usually medically managed- doxycycline or azithromycin
Abdominal aortic aneurysm (ruptured) Sudden onset of abdominal pain radiating to the back in older adults (look for risk factors).
Collapse.
May be moribund on arrival in casualty, more stable if contained haematoma.
Careful clinical assessment may reveal pulsatile mass. Patients who are haemodynamically stable should have a CT scan Unstable patients should undergo immediate surgery (unless it is not in their best interests).
Those with evidence of contained leak on CT should undergo immediate surgery
Increasing unruptured aneurysmal size is an indication for urgent surgical intervention (that can wait until the next working day)
Perforated peptic ulcer Sudden onset of pain (usually epigastric).
Often preceding history of upper abdominal pain.
Soon develop generalised abdominal pain.
On examination may have clinical evidence of peritonitis. Erect CXR may show free air. A CT scan may be indicated where there is diagnostic doubt Laparotomy (laparoscopic surgery for perforated peptic ulcers is both safe and feasible in experienced hands)
Intestinal obstruction Colicky abdominal pain and vomiting (the nature of which depends on the level of the obstruction).
Abdominal distension and constipation (again depending upon site of obstruction).
Features of peritonism may occur where local necrosis of bowel loops is occurring. A plain abdominal film may help with making the diagnosis. A CT scan may be useful where diagnostic uncertainty exists In those with a virgin abdomen a lower and earlier threshold for laparotomy should exist than in those who may have adhesional obstruction
Mesenteric infarction Embolic events present with sudden pain and forceful evacuation.
Acute on chronic events usually have a longer history and previous weight loss.
On examination the pain is typically greater than the physical signs would suggest. Arterial pH and lactate
Arterial phase CT scanning is the most sensitive test

1671
Q

You are called to the acute surgical unit. A patient who has short gut syndrome has developed a broad complex tachycardia. You suspect a diagnosis of ventricular tachycardia. What is the most likely precipitant?

Hypoglycaemia

Bisoprolol

Hypomagnesaemia

Dehydration

Hyperthyroidism

A

Ventricular tachycardia

Ventricular tachycardia (VT)is broad-complex tachycardia originating from a ventricular ectopic focus. It has the potential to precipitate ventricular fibrillation and hence requires urgent treatment.

There are two main types of VT:
monomorphic VT: most commonly caused by myocardial infarction
polymorphic VT: A subtype of polymorphic VT is torsades de pointes which is precipitated by prolongation of the QT interval. The causes of a long QT interval are listed below

Causes of a prolonged QT interval

Congenital
Jervell-Lange-Nielsen syndrome (includes deafness and is due to an abnormal potassium channel)
Romano-Ward syndrome (no deafness)
Drugs
amiodarone, sotalol, class 1a antiarrhythmic drugs
tricyclic antidepressants, fluoxetine
chloroquine
terfenadine
erythromycin
Other
electrolyte: hypocalcaemia, hypokalaemia, hypomagnesaemia
acute myocardial infarction
myocarditis
hypothermia
subarachnoid haemorrhage

1672
Q

An enthusiastic surgical registrar undertakes his first solo splenectomy. The operation is far more difficult than anticipated and the registrar leaves a tube drain to the splenic bed at the end of the procedure. Over the following 24 hours approximately 500ml of clear fluid has entered the drain. Biochemical testing of the fluid is most likely to reveal:

Elevated creatinine

Elevated triglycerides

Elevated glucagon

Elevated amylase

None of the above

A

During splenectomy the tail of the pancreas may be damaged. The pancreatic duct will then drain into the splenic bed, amylase is the most likely biochemical finding. Glucagon is not secreted into the pancreatic duct.

Splenic anatomy

The spleen is the largest lymphoid organ in the body. It is an intraperitoneal organ, the peritoneal attachments condense at the hilum where the vessels enter the spleen. Its blood supply is from the splenic artery (derived from the coeliac axis) and the splenic vein (which is joined by the IMV and unites with the SMV).

Embryology: derived from mesenchymal tissue
Shape: clenched fist
Position: below 9th-12th ribs
Weight: 75-150g

Relations
Superiorly- diaphragm
Anteriorly- gastric impression
Posteriorly- kidney
Inferiorly- colon
Hilum: tail of pancreas and splenic vessels
Forms apex of lesser sac (containing short gastric vessels)

1673
Q

Which of the following structures is not located in the superficial perineal space in females?

Posterior labial arteries

Pudendal nerve

Superficial transverse perineal muscle

Greater vestibular glands

None of the above

A

The pudendal nerve is located in the deep perineal space and then branches to innervate more superficial structures.

Urogenital triangle

The urogenital triangle is formed by the:
Ischiopubic inferior rami
Ischial tuberosities
A fascial sheet is attached to the sides, forming the inferior fascia of the urogenital diaphragm.

It transmits the urethra in males and both the urethra and vagina in females. The membranous urethra lies deep to this structure and is surrounded by the external urethral sphincter.

Superficial to the urogenital diaphragm lies the superficial perineal pouch. In males this contains:
Bulb of penis
Crura of the penis
Superficial transverse perineal muscle
Posterior scrotal arteries
Posterior scrotal nerves

In females the internal pudendal artery branches to become the posterior labial arteries in the superficial perineal pouch.

1674
Q

An 82 year old woman from a nursing home is admitted to the orthopaedic ward with a hip fracture. She is acutely confused and agitated. Her Calcium is 2.95 (elevated) and her PTH is 12 (elevated). What is the best course of action?

Administration of intravenous dextrose

Administration of intravenous crystalloid

Sestamibi scan

Administration of intravenous colloid

Emergency parathyroidectomy

A

Whilst she may have an underlying parathyroid lesion, resuscitation should occur first. Initially, this should be with crystalloid.

Parathyroid glands and disorders of calcium metabolism

Hyperparathyroidism
Disease type Hormone profile Clinical features Cause
Primary hyperparathyroidism
PTH (Elevated)
Ca2+ (Elevated)
Phosphate (Low)
Urine calcium : creatinine clearance ratio > 0.01
May be asymptomatic if mild
Recurrent abdominal pain (pancreatitis, renal colic)
Changes to emotional or cognitive state
Most cases due to solitary adenoma (80%), multifocal disease occurs in 10-15% and parathyroid carcinoma in 1% or less
Secondary hyperparathyroidism
PTH (Elevated)
Ca2+ (Low or normal)
Phosphate (Elevated)
Vitamin D levels (Low)
May have few symptoms
Eventually may develop bone disease, osteitis fibrosa cystica and soft tissue calcifications
Parathyroid gland hyperplasia occurs as a result of low calcium, almost always in a setting of chronic renal failure
Tertiary hyperparathyroidism
Ca2+ (Normal or high)
PTH (Elevated)
Phosphate levels (Decreased or Normal)
Vitamin D (Normal or decreased)
Alkaline phosphatase (Elevated)
Metastatic calcification
Bone pain and / or fracture
Nephrolithiasis
Pancreatitis
Occurs as a result of ongoing hyperplasia of the parathyroid glands after correction of underlying renal disorder, hyperplasia of all 4 glands is usually the cause

Differential diagnoses
It is important to consider the rare but relatively benign condition of benign familial hypocalciuric hypercalcaemia, caused by an autosomal dominant genetic disorder. Diagnosis is usually made by genetic testing and concordant biochemistry (urine calcium : creatinine clearance ratio <0.01-distinguished from primary hyperparathyroidism).

Treatment

Primary hyperparathyroidism
Indications for surgery
Elevated serum Calcium > 1mg/dL above normal
Hypercalciuria > 400mg/day
Creatinine clearance < 30% compared with normal
Episode of life threatening hypercalcaemia
Nephrolithiasis
Age < 50 years
Neuromuscular symptoms
Reduction in bone mineral density of the femoral neck, lumbar spine, or distal radius of more than 2.5 standard deviations below peak bone mass (T score lower than -2.5)

Secondary hyperparathyroidism
Usually managed with medical therapy.

Indications for surgery in secondary (renal) hyperparathyroidism:
Bone pain
Persistent pruritus
Soft tissue calcifications

Tertiary hyperparathyroidism
Allow 12 months to elapse following transplant as many cases will resolve
The presence of an autonomously functioning parathyroid gland may require surgery. If the culprit gland can be identified then it should be excised. Otherwise total parathyroidectomy and re-implantation of part of the gland may be required.

References
1. Insogna K. Primary Hyperparathyroidism. N Engl J Med. 2018 Sep 13;379(11):1050-1059.

  1. van der Plas W Y et al. Secondary and Tertiary Hyperparathyroidism: A Narrative Review. Scand J Surg. 2020 Dec;109(4):271-278.
1675
Q

Following an uncomplicated laparoscopic cholecystectomy a patient is found to have a bile leak. An ERCP is performed and leakage is noted from the cystic duct. What is the most appropriate course of action?

Undertake a sphincterotomy and place a stent in the common bile duct

Undertake a laparoscopy and place a T tube into the bile duct

Start the patient on TPN and keep them nil by mouth

Transfer the patient for a biliary bypass procedure

Place a transhepatic biliary stent

A

Cystic stump leaks following cholecystectomy can be managed with ERCP and stent

Complications following cholecystectomy

Complications following cholecystectomy
Open conversion
One of the largest series, published by Shea in 1996 with a total of more than 70,000 cholecystectomies analysed by pooled studies tends to list most complications as having an incidence of <1%. This includes open conversion which they identified as occurring in less than 1%. One of the interesting points raised by this data is that it covers an early period in laparoscopy and there is highly likely to be an element of selection bias in the cases put forward for laparoscopy. Indeed, some surgeons have noted their own institutions conversion rates increase (in this case 7%) as they take on increasingly complex cases.

Bile leaks
Where these occur they are usually the result of leakage from biliary radicles or cystic stump leaks. Many patients with a cystic stump leak can be managed with ERCP , sphincterotomy and stent. They are not common complications and have an incidence of around 1%.

Bile duct injury
Cholecystectomy remains the commonest cause of bile duct injury. Reported incidences range from 0.3-0.7%. The main reason is that biliary anatomy is not appreciated. Where they occur, the definitive management is reconstruction. If the operating surgeon does not regularly practise this type of surgery then the area should be drained and the patient transferred to an HPB unit.

1676
Q

A 28 year old man undergoes an ileocaecal resection to treat terminal ileal Crohns disease. Post operatively he attends the clinic and complains of diarrhoea. His CRP is within normal limits and small bowel enteroclysis shows no focal changes. Which of the following interventions is most likely to be beneficial?

5 ASA drugs

Azathioprine

Pulsed methylprednisolone

Infliximab

Oral cholestyramine

A

The question is about high output diarrhoea following terminal ileal resection and the most likely cause is malabsorption of bile salts. The administration of cholestyramine (bile salt binding agent) will counter this and thats why its the correct answer.
Malabsorption of bile salts is a common cause of diarrhoea following ileal resection. A normal small bowel study and CRP effectively excludes active Crohns disease and therefore immunomodulator drugs are not appropriate.

Ileum

Anatomical overview
The terminal ileum comprises 20% of the ileum and has a diameter that is smaller than that of the jejunum (typically 2cm at the ileocaecal valve). The ileum is attached to the abdominal wall by a mesentery that contains more fat than that of the jejunum. The blood supply of the ileum is derived from branches of the superior mesenteric artery, the vascular arcades of the ileum are more densely packed than those of the jejunum. The wall of the ileum contains Peyers patches which are aggregations of lymphoid tissue.

Function
The main function of the terminal ileum is absorption of vitamin B12 and bile salts. The neuroendocrine cells in the wall of the ileum may secrete hormones. In surgical patients, resection of the terminal ileum is a common procedure for conditions such as terminal ileal Crohns disease. Where a significant proportion of the ileum is removed, patients are at increased risk of bile salt malabsorption with the development of bile salt diarrhoea and increased risk of gallstones. The lack of vitamin B12 may pre-dispose to macrocytic anaemia.

1677
Q

A 52 year old female underwent a cadaveric renal transplant and recovers well post operatively. Her immunosupression regime consists of tacrolimus. Which of the substances listed below should be avoided?

Paracetamol

Apple juice

Penicillin

Prune juice

Grapefruit juice

A

Tacrolimus is metabolised by the P450 enzyme system. This is inhibited by a number of naturally occurring substances, these include grapefruit, watercress and St.Johns Wort. These should all be avoided in immunosupressed patients taking tacrolimus.

Organ transplantation: immunosuppressants

A number of drugs are available which help to mitigate the processes resulting in acute rejection. Cyclosporin and tacrolimus are commonly used drugs.

Example regime
Initial: ciclosporin/tacrolimus with a monoclonal antibody
Maintenance: ciclosporin/tacrolimus with MMF or sirolimus
Add steroids if more than one steroid responsive acute rejection episode

Ciclosporin
Inhibits calcineurin, a phosphatase involved in T cell activation
Nephrotoxic
Monitor levels

Azathioprine
Metabolised to form 6 mercaptopurine which inhibits DNA synthesis and cell division
Side effects include myelosupression, alopecia and nausea

Tacrolimus
Lower incidence of acute rejection compared to ciclosporin
Also less hypertension and hyperlipidaemia
However, high incidence of impaired glucose tolerance and diabetes

Mycophenolate mofetil (MMF)
Blocks purine synthesis by inhibition of IMPDH
Therefore inhibits proliferation of B and T cells
Side-effects: GI and marrow suppression

Sirolimus (rapamycin)
Blocks T cell proliferation by blocking the IL-2 receptor
Can cause hyperlipidaemia

Monoclonal antibodies
Selective inhibitors of IL-2 receptor
Daclizumab
Basilximab

1678
Q

A 21 year old develops tonsillitis. He is in considerable pain. Which of the following nerves is responsible for the sensory innervation of the tonsillar fossa?

Facial nerve

Trigeminal nerve

Glossopharyngeal nerve

Hypoglossal nerve

Vagus

A

The glossopharyngeal nerve is the main sensory nerve for the tonsillar fossa. A lesser contribution is made by the lesser palatine nerve. Because of this otalgia may occur following tonsillectomy.

Tonsil

Anatomy
Each palatine tonsil has two surfaces, a medial surface which projects into the pharynx and a lateral surface that is embedded in the wall of the pharynx.
They are usually 25mm tall by 15mm wide, although this varies according to age and may be almost completely atrophied in the elderly.
Their arterial supply is from the tonsillar artery, a branch of the facial artery.
Its veins pierce the constrictor muscle to join the external palatine or facial veins. The external palatine vein is immediately lateral to the tonsil, which may result in haemorrhage during tonsillectomy.
Lymphatic drainage is the jugulodigastric node and the deep cervical nodes.

Tonsillitis
Usually bacterial (50%)- group A Streptococcus. Remainder viral.
May be complicated by development of abscess (quinsy). This may distort the uvula.
- Indications for tonsillectomy include recurrent acute tonsillitis, suspected malignancy, enlargement causing sleep apnoea.
- Dissection tonsillectomy is the preferred technique with haemorrhage being the commonest complication. Delayed otalgia may occur owing to irritation of the glossopharyngeal nerve.

1679
Q

Which of the syndromes listed below may be present in a patient with multiple intestinal hamartomas and pigmentation spots around the mouth?

Familial adenomatous polyposis syndrome

Cowden disease

MYH associated polyposis

Peutz-Jeghers syndrome

Lynch syndrome

A

Polyposis syndromes

Syndrome Genetic defect Features Screening and management Associated disorders
Familial adenomatous polyposis Mutation of APC gene (80%) cases, dominant Typically over 100 colonic adenomas
Cancer risk of 100%
20% are new mutations If known to be at risk then predictive genetic testing as teenager
Annual flexible sigmoidoscopy from 15 years
If no polyps found then 5 yearly colonoscopy started at age 20
Polyps found = resectional surgery (resection and pouch Vs sub total colectomy and IRA) Gastric fundal polyps (50%).
Duodenal polyps 90%.
If severe duodenal polyposis cancer risk of 30% at 10 years.
Abdominal desmoid tumours.
MYH associated polyposis Biallelic mutation of mut Y human homologue (MYH) on chromosome 1p, recessive Multiple colonic polyps
Later onset right sided cancers more common than in FAP
100% cancer risk by age 60 Once identified resection and ileoanal pouch reconstruction is recommended
Attenuated phenotype - regular colonoscopy Duodenal polyposis in 30%
Associated with increased risk of breast cancer (self examination)
Peutz -Jeghers syndrome STK11 (LKB1) mutation on chromosome 19 in some (but not all) cases, dominant Multiple benign intestinal hamartomas
Episodic obstruction and intussceception
Increased risk of GI cancers (colorectal cancer 20%, gastric 5%)
Increased risk of breast, ovarian, cervical pancreatic and testicular cancers Annual examination
Pan intestinal endoscopy every 2-3 years Malignancies at other sites
Classical pigmentation pattern
Cowden disease Mutation of PTEN gene on chromosome 10q22, dominant Macrocephaly
Multiple intestinal hamartomas
Multiple trichilemmomas
89% risk of cancer at any site
16% risk of colorectal cancer Targeted individualised screening Breast cancer (81% risk)
Thyroid cancer and non toxic goitre
Uterine cancer
HNPCC (Lynch syndrome) Germline mutations of DNA mismatch repair genes Colo rectal cancer 30-70%
Endometrial cancer 30-70%
Gastric cancer 5-10%
Scanty colonic polyps may be present
Colonic tumours likely to be right sided and mucinous Colonoscopy every 1-2 years from age 25
Consideration of prophylactic surgery
Extra colonic surveillance recommended

1680
Q

A 72 year old man has just undergone an emergency repair for a ruptured abdominal aortic aneurysm. Pre operatively he was taking aspirin, clopidogrel and warfarin. Intra operatively he received 5000 units of unfractionated heparin prior to application of the aortic cross clamp. His blood results on admission to the critical care unit are as follows:
Full blood count
Hb 8 g/dl
Platelets 40 * 109/l
WBC 7.1 * 109/l
His fibrin degradation products are measured and found to be markedly elevated. Which of the following accounts for these results?

Anastomotic leak

Disseminated intravascular coagulation

Heparin induced thrombocytopenia

Adverse effect of warfarin

Adverse effects of antiplatelet agents

A

The combination of low platelet counts and raised FDP in this setting make DIC the most likely diagnosis.

Disseminated intravascular coagulation - Diagnosis

Under homeostatic conditions, coagulation and fibrinolysis are coupled. The activation of the coagulation cascade yields thrombin that converts fibrinogen to fibrin; the stable fibrin clot being the final product of hemostasis. The fibrinolytic system breaks down fibrinogen and fibrin. Activation of the fibrinolytic system generates plasmin (in the presence of thrombin), which is responsible for the lysis of fibrin clots. The breakdown of fibrinogen and fibrin results in polypeptides (fibrin degradation products). In a state of homeostasis, the presence of plasmin is critical, as it is the central proteolytic enzyme of coagulation and is also necessary for fibrinolysis.

In DIC, the processes of coagulation and fibrinolysis are dysregulated, and the result is widespread clotting with resultant bleeding. Regardless of the triggering event of DIC, once initiated, the pathophysiology of DIC is similar in all conditions. One critical mediator of DIC is the release of a transmembrane glycoprotein (tissue factor =TF). TF is present on the surface of many cell types (including endothelial cells, macrophages, and monocytes) and is not normally in contact with the general circulation, but is exposed to the circulation after vascular damage. For example, TF is released in response to exposure to cytokines (particularly interleukin 1), tumor necrosis factor, and endotoxin. This plays a major role in the development of DIC in septic conditions. TF is also abundant in tissues of the lungs, brain, and placenta. This helps to explain why DIC readily develops in patients with extensive trauma. Upon activation, TF binds with coagulation factors that then triggers the extrinsic pathway (via Factor VII) which subsequently triggers the intrinsic pathway (XII to XI to IX) of coagulation.

Diagnosis
Fibrin degradation products are often raised.

Disorder Prothrombin time APTT Bleeding time Platelet count
Warfarin administration Prolonged Normal Normal Normal
Aspirin administration Normal Normal Prolonged Normal
Heparin Often normal (may be prolonged) Prolonged Normal Normal
DIC Prolonged Prolonged Prolonged Low

1681
Q

A 32 year old man has a glomerular filtration rate of 110ml / minute at a systolic blood pressure of 120/80. If his blood pressure were to fall to 100/70 what would glomerular filtration rate be?

110ml / minute

100ml/ minute

55ml/ minute

25ml/ minute

75ml/ minute

A

The proposed drop in blood pressure falls within the range within which the kidney autoregulates its blood supply. GFR will therefore remain unchanged.

Renal Physiology

Overview
Each nephron is supplied with blood from an afferent arteriole that opens onto the glomerular capillary bed.
Blood then flows to an efferent arteriole, supplying the peritubular capillaries and medullary vasa recta.
The kidney receives up to 25% of resting cardiac output.

Control of blood flow
The kidney is able to autoregulate its blood flow between systolic pressures of 80- 180mmHg so there is little variation in renal blood flow.
This is achieved by myogenic control of arteriolar tone, both sympathetic input and hormonal signals (e.g. renin) are responsible.

Glomerular structure and function
Blood inside the glomerulus has considerable hydrostatic pressure.
The basement membrane has pores that will allow free diffusion of smaller solutes, larger negatively charged molecules such as albumin are unable to cross.
The glomerular filtration rate (GFR) is equal to the concentration of a solute in the urine, times the volume of urine produced per minute, divided by the plasma concentration (assuming that the solute is freely diffused e.g. inulin).
In clinical practice creatinine is used because it is subjected to very little proximal tubular secretion.
Although subject to variability, the typical GFR is 125ml per minute.
Glomerular filtration rate = Total volume of plasma per unit time leaving the capillaries and entering the bowman’s capsule
Renal clearance = volume plasma from which a substance is removed per minute by the kidneys

Substances used to measure GFR have the following features:
1. Inert
2. Free filtration from the plasma at the glomerulus (not protein bound)
3. Not absorbed or secreted at the tubules
4. Plasma concentration constant during urine collection

Examples: inulin, creatinine

plasma concentration (mmol/l)

The clearance of a substance is dependent not only on its diffusivity across the basement membrane but also subsequent tubular secretion and / or reabsorption.
So glucose which is freely filtered across the basement membrane is usually reabsorbed from tubules giving a clearance of zero.

Tubular function
Reabsorption and secretion of substances occurs in the tubules.
In the proximal tubule substrates such as glucose, amino acids and phosphate are co-transported with sodium across the semi permeable membrane.
Glucose is reabsorbed by both active and passive processes. The former uses sodium–glucose co-transporters (SGLTs) in the proximal convoluted tubule. The latter achieved through facilitated and passive process through GLUT transporters.
Up to two thirds of filtered water is reabsorbed in the proximal tubules.
This will lead to increase in urea concentration in the distal tubule allowing for its increased diffusion.
Substances to be secreted into the tubules are taken up from the peritubular blood by tubular cells.
Solutes such as paraaminohippuric acid are cleared with a single passage through the kidneys and this is why it is used to measure renal plasma flow. Ions such as calcium and phosphate will have a tubular reabsorption that is influenced by plasma PTH levels.
Potassium may be both secreted and re-absorbed and is co-exchanged with sodium.

Loop of Henle
Approximately 60 litres of water containing 9000mmol sodium enters the descending limb of the loop of Henle in 24 hours.
Loops from the juxtamedullary nephrons run deep into the medulla.
The osmolarity of fluid changes and is greatest at the tip of the papilla.
The thick ascending limb is impermeable to water, but highly permeable to sodium and chloride ions.
This loss means that at the beginning of the thick ascending limb the fluid is hypo osmotic compared with adjacent interstitial fluid.
In the thick ascending limb the reabsorption of sodium and chloride ions occurs by both facilitated and passive diffusion pathways.
The loops of Henle are co-located with vasa recta, these will have similar solute compositions to the surrounding extracellular fluid so preventing the diffusion and subsequent removal of this hypertonic fluid.
The energy dependent reabsorption of sodium and chloride in the thick ascending limb helps to maintain this osmotic gradient.

1682
Q

A 62 year old man is undergoing a left hemicolectomy for carcinoma of the descending colon. The registrar commences mobilisation of the left colon by pulling downwards and medially. Blood soon appears in the left paracolic gutter. The most likely source of bleeding is the:

Marginal artery

Left testicular artery

Spleen

Left renal vein

None of the above

A

The spleen is commonly torn by traction injuries in colonic surgery. The other structures are associated with bleeding during colonic surgery but would not manifest themselves as blood in the paracolic gutter prior to incision of the paracolonic peritoneal edge.

Left colon

Position
As the left colon passes inferiorly its posterior aspect becomes extraperitoneal, and the ureter and gonadal vessels are close posterior relations that may become involved in disease processes
At a level of L3-4 (variable) the left colon becomes the sigmoid colon and wholly intraperitoneal once again
The sigmoid colon is a highly mobile structure and may even lie on the right side of the abdomen
It passes towards the midline, the taenia blend and this marks the transition between sigmoid colon and upper rectum

Blood supply
Inferior mesenteric artery
However, the marginal artery (from the right colon) contributes, this contribution becomes clinically significant when the IMA is divided surgically (e.g. During AAA repair)

1683
Q

A 28 year old man with poorly controlled Crohns disease is nutritionally compromised. The decision is made to start TPN, via which of the following routes should it be best administered?

Internal jugular vein via a central venous catheter

Internal carotid artery

Cephalic vein via peripheral cannula

Basilic vein via peripheral cannula

Common femoral vein via a central venous catheter

A

Since TPN solutions are irritant to veins they are best administered via a central line. The femoral route has a higher incidence of line associated sepsis and is thus best avoided in this setting.

Parenteral feeding-NICE guidelines

Parenteral nutrition: NICE guidelines summary

Identify patients as malnourished or at risk

Patients identified as being malnourished-
BMI < 18.5 kg/m2
unintentional weight loss of > 10% over 3-6/12
BMI < 20 kg/m2 and unintentional weight loss of > 5% over 3-6/12

AT RISK of malnutrition-
eaten nothing or little > 5 days, who are likely to eat little for a further 5 days
poor absorptive capacity
high nutrient losses
high metabolism

Identify unsafe/inadequate oral intake OR a non functional GI tract/perforation/inaccessible

Consider parenteral nutrition:
for feeding < 14 days consider feeding via a peripheral venous catheter
for feeding > 30 days use a tunneled subclavian line
continuous administration in severely unwell patients
if feed needed > 2 weeks consider changing from continuous to cyclical feeding
don’t give > 50% of daily regime to unwell patients in first 24-48 hours

Surgical patients: if malnourished with unsafe swallow OR a non functional GI tract/perforation/inaccessible then consider peri operative parenteral feeding.

1684
Q

A 50 year old woman presents with a diffuse swelling in the region of her right parotid together with facial pain. On examination, she has a right sided facial nerve palsy. What is the most likely underlying cause?

Pleomorphic adenoma

Adenoid cystic carcinoma

Warthins tumour

Lymphoma

Sarcoid

A

Adenoid cystic carcinoma commonly infiltrates the facial nerve and may cause neuropathy and facial pain.

Parotid gland clinical

Benign neoplasms
Up to 80% of all salivary gland tumours occur in the parotid gland and up to 80% of these are benign. There is no consistent correlation between the rate of growth and the malignant potential of the lesion. However, benign tumours should not invade structures such as the facial nerve.
With the exception of Warthins tumours, they are commoner in women than men. The median age of developing a lesion is in the 5th decade of life.

Benign tumour types
Tumour type Features
Benign pleomorphic adenoma or benign mixed tumor Most common parotid neoplasm (80%)
Proliferation of epithelial and myoepithelial cells of the ducts and an increase in stromal components
Slow growing, lobular, and not well encapsulated
Recurrence rate of 1-5% with appropriate excision (parotidectomy)
Recurrence possibly secondary to capsular disruption during surgery
Malignant degeneration occurring in 2-10% of adenomas observed for long periods, with carcinoma ex-pleomorphic adenoma occurring most frequently as adenocarcinoma
Warthin tumor (papillary cystadenoma lymphoma or adenolymphoma) Second most common benign parotid tumor (5%)
Most common bilateral benign neoplasm of the parotid
Marked male as compared to female predominance
Occurs later in life (sixth and seventh decades)
Presents as a lymphocytic infiltrate and cystic epithelial proliferation
May represent heterotopic salivary gland epithelial tissue trapped within intraparotid lymph nodes
Incidence of bilaterality and multicentricity of 10%
Malignant transformation rare (almost unheard of)
Monomorphic adenoma Account for less than 5% of tumours
Slow growing
Consist of only one morphological cell type (hence term mono)
Include; basal cell adenoma, canalicular adenoma, oncocytoma, myoepitheliomas
Haemangioma Should be considered in the differential of a parotid mass in a child
Accounts for 90% of parotid tumours in children less than 1 year of age
Hypervascular on imaging
Spontaneous regression may occur and malignant transformation is almost unheard of

Malignant salivary gland tumours
Types of malignancy

Mucoepidermoid carcinoma 30% of all parotid malignancies
Usually low potential for local invasiveness and metastasis (depends mainly on grade)
Adenoid cystic carcinoma Unpredictable growth pattern
Tendency for perineural spread
Nerve growth may display skip lesions resulting in incomplete excision
Distant metastasis more common (visceral rather than nodal spread)
5 year survival 35%
Mixed tumours Often a malignancy occurring in a previously benign parotid lesion
Acinic cell carcinoma Intermediate grade malignancy
May show perineural invasion
Low potential for distant metastasis
5 year survival 80%
Adenocarcinoma Develops from secretory portion of gland
Risk of regional nodal and distant metastasis
5 year survival depends upon stage at presentation, may be up to 75% with small lesions with no nodal involvement
Lymphoma Large rubbery lesion, may occur in association with Warthins tumours
Diagnosis should be based on regional nodal biopsy rather than parotid resection
Treatment is with chemotherapy (and radiotherapy)

Diagnostic evaluation
Plain x-rays may be used to exclude calculi
Sialography may be used to delineate ductal anatomy
FNAC is used in most cases
Superficial parotidectomy may be either diagnostic or therapeutic depending upon the nature of the lesion
Where malignancy is suspected the primary approach should be definitive resection rather than excisional biopsy
CT/ MRI may be used in cases of malignancy for staging primary disease

Treatment
For nearly all lesions this consists of surgical resection, for benign disease this will usually consist of a superficial parotidectomy. For malignant disease a radical or extended radical parotidectomy is performed. The facial nerve is included in the resection if involved. The need for neck dissection is determined by the potential for nodal involvement.

Other parotid disorders
HIV infection
Lymphoepithelial cysts associated with HIV occur almost exclusively in the parotid
Typically presents as bilateral, multicystic, symmetrical swelling
Risk of malignant transformation is low and management usually conservative

Sjogren syndrome
Autoimmune disorder characterised by parotid enlargement, xerostomia and keratoconjunctivitis sicca
90% of cases occur in females
Second most common connective tissue disorder
Bilateral, non tender enlargement of the gland is usual
Histologically, the usual findings are of a lymphocytic infiltrate in acinar units and epimyoepithelial islands surrounded by lymphoid stroma
Treatment is supportive
There is an increased risk of subsequent lymphoma

Sarcoid
Parotid involvement occurs in 6% of patients with sarcoid
Bilateral in most cases
Gland is not tender
Xerostomia may occur
Management of isolated parotid disease is usually conservative

1685
Q

A 53 year old lady has recently entered the UK from the far East and she undergoes a thyroidectomy for a very large multinodular goitre with retrosternal extension. The operation is relatively straightforward otherwise. The patient is returned to the recovery area and suddenly becomes profoundly dyspnoeic and hypoxic. On examination, the wound is healthy and dry and the drain empty. What is the most likely cause?

Contained haematoma

Unilateral recurrent laryngeal nerve injury

Unilateral superior laryngeal nerve injury

Tracheomalacia

Hypocalcaemia

A

With long standing very large goitre, there is a risk of tracheomalacia. In the UK this is very rarely seen. However, patients from other countries can have enourmous goitres and this then becomes a concern. The fact that the wound is satisfactory makes a bleed less likely and whilst total thyroidectomy can be a risk factor for hypocalcaemia, this is a little early.

Complications following thyroid surgery

The major immediate risk following thyroidectomy is haemorrhage. A tension haematoma deep to the cervical fascia is usually the result of reactionary haemorrhage from an arterial source. This results in the development of laryngeal oedema and airway compromise. The treatment is by urgent wound decompression of all layers prior to return to theatre for haemostasis. Sub cutnaeous haematomas and seromas may accumulate under skin flaps and can often be managed conservatively or by simple aspiration.

In patients with longstanding large goitres there is the risk of tracheomalacia. This is characterised by the development of flacidity of the tracheal cartilage. This can result in airway compromise. In the normal situation the tracheal diameter increases slightly during inspiration and narrows during expiration. In tracheomalacia these processes are exaggerated and the trachea may collapse in expiration resulting in stridor. In the immediate situation an endotracheal tube will need to be inserted.

Recurrent laryngeal nerve injury is recognised following thyroid surgery and may be unilateral or bilateral depending upon the procedure performed. The risk is 1.8% at one month which declines to 0.5% at three months following first time explorations. It is rare for nerve injury alone to result in airway compromise. However, it may occur when nerve injury is associated with minor degrees of laryngeal oedema (such as following intubation).

Hypoparathyroidism is a recognised complication following thyroid surgery and damage to the blood supply to the parathyroid glands is probably the commonest cause. The incidence of permanent hypoparathyroidism is in the region of 1 to 3%. It can present dramatically in the first 2-5 post operative days. In the emergency setting treatment is with intravenous calcium gluconate. Oral calcium carbonate is used in the longer term.

1686
Q

As the SpR in general surgery you wish to determine whether your breast cancer unit is complying with the British Association of Surgical Oncology guidelines for management of high grade ductal carcinoma in situ. What is the best course of action?

Undertake a clinical audit

Undertake a clinical review

Undertake a service evaluation

Discuss the case with the ethics committee

Discuss selected cases at joint histopathology meetings

A

Since this comparison with practice against an agreed standard, the correct course of action is to undertake an audit.

Audit and Research

Clinical audit
Quality improvement process that seeks to improve patient care and outcomes through systematic review of care against explicit criteria and the implementation of change. Aspects of the structure, processes, and outcomes of care are selected and systematically evaluated against explicit criteria. Where indicated, changes are implemented at an individual, team, or service level and further monitoring is used to confirm improvement in healthcare delivery. (NICE).

Research
Aims to derive new knowledge which is potentially generalisable or transferable.

1687
Q

A 63 year old lady is undergoing colonoscopy with midazolam sedation. Her respiratory rate slows and she becomes hypoxic and the decision is made to reverse her sedation. What is the most appropriate agent to administer?

Flumazenil

Doxapram

Naloxone

Procyclidine

Etomidate

A

Flumazenil antagonises the effects of benzodiazepines by competition at GABA binding sites. Since many benzodiazepines have longer half lives than flumazenil patients still require close monitoring after receiving the drug. Whilst doxapram would increase the respiratory rate, it is not an agent for reversal of midazolam.

Anaesthetic agents

The table below summarises some of the more commonly used IV induction agents
Agent Specific features
Propofol
Rapid onset of anaesthesia
Pain on IV injection
Rapidly metabolised with little accumulation of metabolites
Proven anti emetic properties
Moderate myocardial depression
Widely used especially for maintaining sedation on ITU, total IV anaesthesia and for daycase surgery
Sodium thiopentone
Extremely rapid onset of action making it the agent of choice for rapid sequence of induction
Marked myocardial depression may occur
Metabolites build up quickly
Unsuitable for maintenance infusion
Little analgesic effects
Ketamine
May be used for induction of anaesthesia
Has moderate to strong analgesic properties
Produces little myocardial depression making it a suitable agent for anaesthesia in those who are haemodynamically unstable
May induce state of dissociative anaesthesia resulting in nightmares
Etomidate
Has favorable cardiac safety profile with very little haemodynamic instability
No analgesic properties
Unsuitable for maintaining sedation as prolonged (and even brief) use may result in adrenal suppression
Post operative vomiting is common

1688
Q

A 78 year old man has a long history of nocturia, urinary frequency and terminal dribbling. He was admitted with urinary retention and was catheterised. On removal of the catheter he has noticed some haematuria. What is the most likely cause?

Transitional cell carcinoma of the bladder

Squamous cell carcinoma of the bladder

Polycystic kidney disease

Renal cell carcinoma

Benign prostatic hyperplasia

A

The symptoms are typical for prostatic disease.

Benign prostatic disease

BPH is increasingly common with advancing age and is present in 50% over the age of 60 and nearly 90% of men by the age of 90. It occurs as a result of hyperplasia of the periurethral glands in the transitional zone of the prostate. Androgens play a role in the development and progression of BPH. Testosterone diffuses into prostatic and stromal cells. Within epithelial cells it binds to the androgen receptor. In prostatic stromal cells, a small proportion binds directly to the androgen receptor, the majority binds to the 5 alpha reductase type II receptor on the nuclear membrane. This is converted to dihyroxytestosterone and then binds to the androgen receptor. Dihydroxytestosterone has even greater affinity for the androgen receptor than testosterone does. The end result is stimulation of these cells and proliferation.
This proliferative activity results in varying degrees of obstruction and results in lower urinary tract obstructive symptoms. The clinical diagnosis of BPH thus comprise a degree of lower urinary tract symptoms, palpable prostatic enlargement and evidence of impaired voiding on urodynamic assessment.

Presentation
Some patients have minimal symptoms, yet, on examination, have a palpable bladder and obstructive post renal failure. LUTS can be divided into two main groups; obstructive symptoms with voiding that include hesitancy, poor stream, straining, prolonged micturition and dribbling; others develop irritation symptoms which include pain during bladder filling, frequency, urgency and nocturia. Some present with retention and haematuria.

Diagnosis
Abdominal and rectal examination
Symptoms scoring
Urodynamic studies ( a post void volume of >100ml is significant)

Management
Conservative
Alpha adrenergic antagonists. These block the action of noradrenaline on prostatic smooth muscle causing relaxation and improved bladder emptying.
5 alpha reductase inhibitors. Finasteride blocks the enzyme 5 alpha reductase which inhibits the conversion of testosterone to DHT. This in turn reduces intracellular activity and decreases prostatic volume
Surgery- TURP is the gold standard. Occasionally, an open retropubic prostatectomy may be considered for a large gland.

1689
Q

A 43 year old lady presents with 24 hour history of generalised right upper quadrant pain. On admission, she is septic and jaundiced and there is tenderness in the right upper quadrant. What is the most likely diagnosis?

Cholecystitis

Cholangitis

Gallbladder empyema

Gallbladder abscess

Liver abscess

A

A combination of sepsis and jaundice generally favors a diagnosis of cholangitis. Conditions such as empyema may cause sepsis but not marked jaundice, the same is also true of the other differentials listed.

Gallstones

Up to 24% of women and 12% of men may have gallstones. Of these up to 30% may develop local infection and cholecystitis. In patients subjected to surgery 12% will have stones contained within the common bile duct. The majority of gallstones are of a mixed composition (50%) with pure cholesterol stones accounting for 20% of cases.
The aetiology of CBD stones differs in the world, in the West most CBD stones are the result of migration. In the East a far higher proportion arise in the CBD de novo.
The classical symptoms are of colicky right upper quadrant pain that occurs post prandially. The symptoms are usually worst following a fatty meal when cholecystokinin levels are highest and gallbladder contraction is maximal.

Investigation
In almost all suspected cases the standard diagnostic work up consists of abdominal ultrasound and liver function tests. Of patients who have stones within the bile duct, 60% will have at least one abnormal result on LFT’s. Ultrasound is an important test, but is operator dependent and therefore may occasionally need to be repeated if a negative result is at odds with the clinical picture. Where stones are suspected in the bile duct, the options lie between magnetic resonance cholangiography and intraoperative imaging. The choice between these two options is determined by the skills and experience of the surgeon. The advantages of intra operative imaging are less useful in making therapeutic decisions if the operator is unhappy about proceeding the bile duct exploration, and in such circumstances pre operative MRCP is probably a better option.

Specific gallstone and gallbladder related disease
Disease Features Management
Biliary colic Colicky abdominal pain, worse post prandially, worse after fatty foods If imaging shows gallstones and history compatible then laparoscopic cholecystectomy
Acute cholecystitis
Right upper quadrant pain
Fever
Murphys sign on examination
Occasionally mildly deranged LFT’s (especially if Mirizzi syndrome)
Imaging (USS) and cholecystectomy (ideally within 48 hours of presentation) (2)
Gallbladder abscess
Usually prodromal illness and right upper quadrant pain
Swinging pyrexia
Patient may be systemically unwell
Generalised peritonism not present
Imaging with USS +/- CT Scanning
Ideally surgery, sub total cholecystectomy may be needed if Calots triangle is hostile
In unfit patients percutaneous drainage may be considered
Cholangitis
Patient severely septic and unwell
Jaundice
Right upper quadrant pain
Fluid resuscitation
Broad spectrum intravenous antibiotics
Correct any coagulopathy
Early ERCP
Gallstone ileus
Patients may have a history of previous cholecystitis and known gallstones
Small bowel obstruction (may be intermittent)
Laparotomy and removal of gallstone from small bowel, the enterotomy must be made proximal to the site of obstruction and not at the site of obstruction. The fistula between the gallbladder and duodenum should not be interfered with.
Acalculous cholecystitis
Patients with inter current illness (e.g. diabetes, organ failure)
Patient of systemically unwell
Gallbladder inflammation in absence of stones
High fever
If patient fit then cholecystectomy, if unfit then percutaneous cholecystostomy

Treatment
Patients with asymptomatic gallstones rarely develop symptoms related to them (less than 2% per year) and may therefore be managed expectantly. In almost all cases of symptomatic gallstones the treatment of choice is cholecystectomy performed via the laparoscopic route. In the very frail patient there is sometimes a role for selective use of ultrasound guided cholecystostomy.

During the course of the procedure some surgeons will routinely perform either intra operative cholangiography or laparoscopic USS to either confirm anatomy or to exclude CBD stones. The latter may be more easily achieved by use of laparoscopic ultrasound. If stones are found then the options lie between early ERCP in the day or so following surgery or immediate surgical exploration of the bile duct. When performed via the trans cystic route this adds little in the way of morbidity and certainly results in faster recovery. Where transcystic exploration fails the alternative strategy is that of formal choledochotomy. The exploration of a small duct is challenging and ducts of less than 8mm should not be explored. Small stones that measure less than 5mm may be safely left and most will pass spontaneously.

Risks of ERCP(1)
Bleeding 0.9% (rises to 1.5% if sphincterotomy performed)
Duodenal perforation 0.4%
Cholangitis 1.1%
Pancreatitis 1.5%

References
1. Williams E et al. Guidelines on the management of common bile duct stones (CBDS)Gut 2008;57:10041021

  1. Gurusamy KS, Samraj K. Early versus delayed laparoscopic cholecystectomy for acute cholecystitis. Cochrane Database Syst Rev. 2006 Oct 18;(4):CD005440.
  2. Gurusamy K and Davidson B. Gallstones. BMJ 2014 (348):27-30.
1690
Q

A 24 year old man is involved in a road traffic accident. His right leg is trapped for 6 hours whilst he is moved. On examination his foot is insensate and a dorsalis pedis pulse is only weakly felt. Which of the biochemical abnormalities listed below is most likely to be present?

Alkalosis

Hypercalcaemia

Hypocalcaemia

Hyperkalaemia

Hyponatraemia

A

In this scenario the patient will have a compartment syndrome, delayed diagnosis and muscle death. The effect of muscle death will result in the release of potassium. It is also highly likely that there will be a degree of renal impairment, the result of which is that the serum potassium is likely to be high.

In this scenario the patient will have a compartment syndrome, delayed diagnosis and muscle death. The effect of muscle death will result in the release of potassium. It is also highly likely that there will be a degree of renal impairment, the result of which is that the serum potassium is likely to be high.

1691
Q

A 22 year old man is involved in a road traffic accident. He is found to have a pelvic fracture. While on the ward the nursing staff report that he is complaining of lower abdominal pain. On examination, you find a distended tender bladder. What is the most likely diagnosis?

Bladder rupture

Ureter injury

Urethral injury

Clot retention

Prostate rupture

A

Pelvic fractures may cause laceration of the urethra. Urinary retention, blood at the urethral meatus and a high riding prostate on digital rectal examination are the typical features.

Lower genitourinary tract trauma

Most bladder injuries occur due to blunt trauma
85% associated with pelvic fractures
Easily overlooked during assessment in trauma
Up to 10% of male pelvic fractures are associated with urethral or bladder injuries

Types of injury

Urethral injury
Mainly in males
Blood at the meatus (50% cases)
There are 2 types:
i.Bulbar rupture
- most common
- straddle type injury e.g. bicycles
- triad signs: urinary retention, perineal haematoma, blood at the meatus
ii. Membranous rupture
- can be extra or intraperitoneal
- commonly due to pelvic fracture
- Penile or perineal oedema/ hematoma
- PR: prostate displaced upwards (beware co-existing retroperitoneal haematomas as they may make examination difficult)

  • Investigation: ascending urethrogram
  • Management: suprapubic catheter (surgical placement, not percutaneously)
    External genitalia injuries (i.e., the penis and the scrotum)
    Secondary to injuries caused by penetration, blunt trauma, continence- or sexual pleasure-enhancing devices, and mutilation
    Bladder injury
    rupture is intra or extraperitoneal
    presents with haematuria or suprapubic pain
    history of pelvic fracture and inability to void: always suspect bladder or urethral injury
    inability to retrieve all fluid used to irrigate the bladder through a Foley catheter indicates bladder injury
    investigation- IVU or cystogram
    management: laparotomy if intraperitoneal, conservative if extraperitoneal
1692
Q

A 63 year old man who previously smoked 20 cigarettes a day and has newly diagnosed type II diabetes. He presents with rest pain. A diagnostic angiogram demonstrates occlusion of the distal superficial femoral artery continuing below the knee. He has reasonable posterior tibial artery below this level which branches to give good runoff to the foot. He has varicose veins. Which of the strategies presented below is most appropriate?

Femoro-distal bypass graft using long saphenous vein

Femoro-above knee popliteal bypass graft using PTFE

Femoro-distal bypass graft using PTFE

Femoro-distal bypass graft using PTFE and Miller cuff

Above knee amputation

A

This man needs a bypass operation. Using PTFE alone will not give a good result as sub intimal hyperplasia will give poor outcome early. Using a vein cuff (Miller cuff) at the end of a PTFE graft will improve the situation. Varicose veins means that vein cannot be reliably used as a conduit. However, the construction of a Miller vein cuff can be done with vein from another anatomical site.

Peripheral vascular disease

Indications for surgery to revascularise the lower limb
Intermittent claudication
Critical ischaemia
Ulceration
Gangrene

Intermittent claudication that is not disabling may provide a relative indication, whilst the other complaints are often absolute indications depending upon the frailty of the patient.

Assessment
Clinical examination
Ankle brachial pressure index measurement
Duplex arterial ultrasound
Angiography (standard, CT or MRI): usually performed only if intervention being considered.

Angioplasty
In order for angioplasty to be undertaken successfully the artery has to be accessible. The lesion relatively short and reasonable distal vessel runoff. Longer lesions may be amenable to sub-intimal angioplasty.

Surgery
Surgery will be undertaken where attempts at angioplasty have either failed or are unsuitable. Bypass essentially involves bypassing the affected arterial segment by utilising a graft to run from above the disease to below the disease. As with angioplasty good runoff improves the outcome.

Some key concepts with bypass surgery

Superficial femoral artery occlusion to the above knee popliteal
In the ideal scenario, vein (either in situ or reversed LSV) would be used as a conduit. However, prosthetic material has reasonable 5 year patency rates and some would advocate using this in preference to vein so that vein can be used for other procedures in the future. In general terms either technique is usually associated with an excellent outcome (if run off satisfactory).

Procedure
Artery dissected out, IV heparin 3,000 units given and then the vessels are cross clamped
Longitudinal arteriotomy
Graft cut to size and tunneled to arteriotomy sites
Anastomosis to femoral artery usually with 5/0 ‘double ended’ Prolene suture
Distal anastomosis usually using 6/0 ‘double ended’ Prolene

Distal disease
Femoro-distal bypass surgery takes longer to perform, is more technically challenging and has higher failure rates.
In elderly diabetic patients with poor runoff a primary amputation may well be a safer and more effective option. There is no point in embarking on this type of surgery in patients who are wheelchair bound.
In femorodistal bypasses vein gives superior outcomes to PTFE.

Rules
Vein mapping 1st to see whether there is suitable vein (the preferred conduit). Sub intimal hyperplasia occurs early when PTFE is used for the distal anastomosis and will lead to early graft occlusion and failure.
Essential operative procedure as for above knee fem-pop.
If there is insufficient vein for the entire conduit then vein can be attached to the end of the PTFE graft and then used for the distal anastomosis. This type of ‘vein boot’ is technically referred to as a Miller Cuff and is associated with better patency rates than PTFE alone.
Remember the more distal the arterial anastomosis the lower the success rate.

References
Peach G et al. Diagnosis and management of peripheral arterial disease. BMJ 2012; 345: 36-41.

1693
Q

A homeless 42 year old male had an emergency inguinal hernia repair 24 hours previously. He has a BMI of 15. He has been put on a nasogastric feeding regime of 35 kcal/kg/day with no additional medications. The nursing staff contact you as he has become confused and unsteady. On examination the patient is disorientated to place, has diplopia and nystagmus. What is the most likely diagnosis?

Cerebellar stroke

Acute dystonic reaction

Cerebrovascular accident

Parkinsonism

Wernickes encephalopathy

A

Triad of Wernicke encephalopathy:
Acute confusion
Ataxia
Ophthalmoplegia
This patient has received a carbohydrate rich diet without any thiamine or vitamin B co strong replacement. This has led to Wernickes encephalopathy, which classically presents with confusion, ataxia and ophthalmoplegia. Characteristically it is associated with chronic alcoholism, however it is also known to occur post bariatric surgery.

Nutrition - Refeeding syndrome

Refeeding syndrome describes the metabolic abnormalities which occur on feeding a person following a period of starvation. The metabolic consequences include:
Hypophosphataemia
Hypokalaemia
Hypomagnesaemia
Abnormal fluid balance
These abnormalities can lead to organ failure.

Re-feeding problems
If patient not eaten for > 5 days, aim to re-feed at < 50% energy and protein levels

High risk for re-feeding problems
If one or more of the following:
BMI < 16 kg/m2
Unintentional weight loss >15% over 3-6 months
Little nutritional intake > 10 days
Hypokalaemia, Hypophosphataemia or hypomagnesaemia prior to feeding (unless high)

If two or more of the following:
BMI < 18.5 kg/m2
Unintentional weight loss > 10% over 3-6 months
Little nutritional intake > 5 days
History of: alcohol abuse, drug therapy including insulin, chemotherapy, diuretics and antacids

Prescription
Start at up to 10 kcal/kg/day increasing to full needs over 4-7 days
Start immediately before and during feeding: oral thiamine 200-300mg/day, vitamin B co strong 1 tds and supplements
Give K+ (2-4 mmol/kg/day), phosphate (0.3-0.6 mmol/kg/day), magnesium (0.2-0.4 mmol/kg/day)

1694
Q

Which one of the following is associated with increased lung compliance?

Kyphosis

Pulmonary oedema

Emphysema

Pulmonary fibrosis

Pneumonectomy

A

Respiratory physiology: lung compliance

Lung compliance is defined as change in lung volume per unit change in airway pressure

Causes of increased compliance
age
emphysema - this is due to loss of alveolar walls and associated elastic tissue

Causes of decreased compliance
pulmonary oedema
pulmonary fibrosis
pneumonectomy
kyphosis

1695
Q

Which of the following inhibits the secretion of insulin?

Adrenaline

Lipids

Gastrin

Arginine

Vagal cholinergic activity

A

Inhibition of insulin release:

Alpha adrenergic drugs
Beta blockers
Sympathetic nerves

Insulin

Insulin is a peptide hormone, produced by beta cells of the pancreas, and is central to regulating carbohydrate and fat metabolism in the body. Insulin causes cells in the liver, skeletal muscles, and fat tissue to absorb glucose from the blood. In the liver and skeletal muscles, glucose is stored as glycogen, and in fat cells (adipocytes) it is stored as triglycerides.

Structure
The human insulin protein is composed of 51 amino acids, and has a molecular weight of 5808 Da. It is a dimer of an A-chain and a B-chain, which are linked together by disulfide bonds.

Synthesis
Pro-insulin is formed by the rough endoplasmic reticulum in pancreatic beta cells. Then pro-insulin is cleaved to form insulin and C-peptide. Insulin is stored in secretory granules and released in response to Ca2+.

Function
Secreted in response to hyperglycaemia
Glucose utilisation and glycogen synthesis
Inhibits lipolysis
Reduces muscle protein loss

1696
Q

A 34-year-old HIV positive man is referred to the surgical out patient department with jaundice and abnormal liver function tests. Liver function tests are as follows:

Albumin 34 g/l
ALP 540 iu/l
Bilirubin 67 µmol/l
ALT 45 iu/l

What is the most likely diagnosis?

Hepatic abscess

Fungal obstruction of the bile duct

Duodenal adenoma

Primary biliary cirrhosis

Sclerosing cholangitis

A

The LFTs clearly show a cholestatic picture. Given the background of HIV the most likely cause is sclerosing cholangitis.

HIV: biliary and pancreatic disease

The most common cause of biliary disease in patients with HIV is sclerosing cholangitis due to infections such as CMV, Cryptosporidium and Microsporidia

Pancreatitis in the context of HIV infection may be secondary to anti-retroviral treatment (especially didanosine) or by opportunistic infections e.g. CMV

1697
Q

Which of the following patients should be referred for 1st line bariatric surgery?

BMI 30 kg/m
2
and hypertension

BMI 28 kg/m
2

BMI 35 kg/m
2
and type 2 diabetes

BMI 32kg/m
2

BMI 70kg/m
2
, COPD and type 2 diabetes

A

C-With all the other options the patient should have conservative management for a minimum of 6 months first. The patient with COPD should be considered for a less invasive intervention first e.g. intra gastric balloon.

Bariatric surgery

Obesity is a major health problem in the Western world. Surgical solutions to the problem have evolved dramatically over the past few years. Randomised controlled trials have shown that dramatic weight loss can be achieved following surgical interventions compared with standard medical therapy. The weight loss process is also more durable following surgery than with non surgical interventions.

Case selection
BMI >/= 40 kg/m2 or between 35-40 kg/m2 and other significant disease (for example, type 2 diabetes, hypertension) that could be improved with weight loss.

Pre-requisites to surgery (NICE UK Guidelines)
All non-surgical measures have failed to achieve or maintain adequate clinically beneficial weight loss for at least 6 months.
Will receive intensive specialist management
They are generally fit for anaesthesia and surgery
They commit to the need for long-term follow-up
First-line option for adults with a BMI > 40 kg/m2 in whom surgical intervention is considered appropriate; consider orlistat if there is a long waiting list.

Surgical options
Adjustable gastric band
Laparoscopic placement of adjustable band around proximal stomach.
Contains an adjustable filling port
Effective method for lifestyle control
Reversible
Takes longer to achieve target weight
Complications such as band erosion (rare), slippage or loss of efficacy may require re-intervention
Gastric bypass
Combines changes to reservoir size with malabsorptive procedure for more enduring weight loss.
Technically more challenging
Risks related to anastomoses (2% leak rate)
Irreversible
Up to 50% may become B12 deficient
Sleeve gastrectomy
Resection of stomach using stapling devices
Less popular now as initial promising results not sustained

References
1. National Institute for Health and Clinical Excellence. Obesity: the prevention, identification, assessment and management of overweight and obesity in adults and children. www.nice.org.uk/CG189

  1. Leff DR, Heath D. Surgery for obesity in adulthood. BMJ 2009;339:b3402
1698
Q

A 65 year old man presents with a 5cm ulcerated area over his medial malleolus. It has been present for many years and the nurses have complained that it is not healing with simple bandages. What is the best course of action?

Perform a core biopsy

Perform a punch biopsy

Arrange a split thickness skin graft

Undertake an excisional biopsy

Arrange for wide surgical excision and skin grafting

A

This is likely to be a venous ulcer and should usually be managed with compression bandaging if there is no arterial compromise. Long standing lesions may be complicated by the development of malignancy and for this reason a punch biopsy of long standing or non healing lesions is advisable.

Treatment of suspicious skin lesions

Skin lesions may be referred to surgeons for treatment or discovered incidentally. The table below outlines the various therapeutic options:

Method Indication
Tru-cut biopsy Most often used for percutaneous sampling of deep seated lesions or used intra operatively for visceral lesions
5mm punch biopsy Used for diagnostic confirmation of lesions that are suspected to be benign or where the definitive management is unlikely to be surgical. Of limited usefulness in pigmented lesions where they do not include sufficient tissue for accurate diagnosis. May be used in non melanoma type skin disease to establish diagnosis prior to more extensive resection.
Wide excision Where the complete excision of the lesion (with healthy margins) is the main objective. In cosmetically sensitive sites, or where the defect is large, this may need to be complemented with plastic surgical techniques
Incisional biopsy Used mainly for deep seated or extensive lesions where there is diagnostic doubt (usually following core or tru-cut biopsy). Used rarely for skin lesions.
Diagnostic excision Primarily used for lesions that are suspicious for melanoma, the lesion is excised with a rim of normal tissue. Excision of margins may be required subsequently.

1699
Q

Which of the following would be a sensible volume for maintenance intravenous fluids in a 3 day old term neonate?

50ml/ kg/ hour

50ml/ kg/ day

100ml/kg/hour

100ml/kg/day

200ml/kg/day

A

Calculate routine maintenance IV fluid rates for children and young people using the HollidaySegar formula (100 ml/kg/day for the first 10 kg of weight, 50 ml/kg/day for the next 10 kg and 20 ml/kg/day for the weight over 20 kg). Be aware that over a 24hour period, males rarely need more than 2500 ml and females rarely need more than 2000 ml of fluids.
From birth to day 1: 50-60 ml/kg/day.
Day 2: 70-80 ml/kg/day.
Day 3: 80-100 ml/kg/day.
Day 4: 100-120 ml/kg/day.
Days 5-8: 120-150 ml/kg/day.

Paediatric fluid management

Since 2000 there have been at least 4 reported deaths from fluid induced hyponatraemia in children. This led to the National Patient Safety Agency introducing revised guidelines in 2007. These have been reviewed and extensively updated by NICE in 2015 and further modified in 2020.
Indications for IV fluids include:
Resuscitation and circulatory support
Replacing on-going fluid losses
Maintenance fluids for children for whom oral fluids are not appropriate
Correction of electrolyte disturbances

Fluids to be avoided
Outside the neonatal period saline / glucose solutions should not be given. The greatest risk is with saline 0.18 / glucose 4% solutions. The report states that 0.45% saline / 5% glucose may be used. But preference should be given to isotonic solutions and few indications exist for this solution either.
The key point emphasised in the NICE guidelines in the avoidance of glucose containing solutions and instead of routinely giving glucose IV to children and neonates, the blood glucose levels should be monitored. In neonates in the first few days of life, sodium levels in the range of 131-154 mmol/l may be too high and a neonatologist consulted on a case by case basis.

Fluids to be used
If children and young people need IV fluids for routine maintenance, initially use isotonic crystalloids that contain sodium in the range 131 to 154 mmol/litre

Potassium should be added to maintenance fluids according patients plasma potassium levels (which should be monitored).
Blood glucose levels should be monitored in individuals at risk of hypoglycaemia

Intraoperative fluid management
If children and young people need IV fluids for routine maintenance, initially use isotonic crystalloids that contain sodium in the range 131 to 154 mmol/litre
Blood glucose levels should be monitored

Maintenance fluids
Weight Water requirement/kg/day Na mmol/kg/day K mmol/kg/day
First 10Kg body weight 100ml 2-4 1.5-2.5
Second 10Kg body weight 50ml 1-2 0.5-1.5
Subsequent Kg 20ml 0.5-1.0 0.2-0.7

Blood glucose will need to be monitored

Reference
NPSA -reducing risk of hyponatraemia when administering intravenous fluids to children. Issue date March 2007. Further references included in this document.

NICE Guidance NG29. Intravenous fluid therapy in children and young people in hospital. Published 2015, reviewed an updated 2020.

1700
Q

A 23 year old lady is persistently vomiting following a laparoscopic appendicectomy for a perforated gangrenous appendicitis. Imaging shows some dilated small bowel loops. What is the most appropriate course of action?

Insertion of wide bore nasogastric tube

Insertion of narrow bore nasogastric tube

Administration of intravenous cyclizine

Administration of metoclopramide

Arrange a laparotomy

A

This patient is likely to have a paralytic ileus and the administration of anti emetic drugs in this situation achieves very little. It’s important to decompress the stomach and this can be achieved with a wide bore nasogastric tube.

Vomiting

Reflex oral expulsion of gastric (and sometimes intestinal) contents - reverse peristalsis and abdominal contraction

The vomiting centre is in part of the medulla oblongata and is triggered by receptors in several locations:

Labyrinthine receptors of ear (motion sickness)
Over distention receptors of duodenum and stomach
Trigger zone of CNS - many drugs (e.g., opiates) act here
Touch receptors in throat

1701
Q

The following are true of the femoral nerve, except:

It is derived from L2, L3 and L4 nerve roots

It supplies sartorius

It supplies quadriceps femoris

It gives cutaneous innervations via the saphenous nerve

It supplies adductor longus

A

Adductor longus is supplied by the obturator nerve.

Femoral nerve

Root values L2, 3, 4
Innervates
Pectineus
Sartorius
Quadriceps femoris
Vastus lateralis/medialis/intermedius
Rectus femoris
Branches
Medial cutaneous nerve of thigh
Saphenous nerve
Intermediate cutaneous nerve of thigh

Path
Penetrates psoas major and exits the pelvis by passing under the inguinal ligament to enter the femoral triangle, lateral to the femoral artery and vein.

Mnemonic for femoral nerve supply

(don’t) M I S V Q Scan for PE
M edial cutaneous nerve of the thigh
I ntermediate cutaneous nerve of the thigh
S aphenous nerve

V astus
Q uadriceps femoris
S artorius

PE ectineus

1702
Q

A 73 year old female is referred to the surgical clinic with an iron deficiency anaemia. As part of the diagnostic work up the doctor requests a serum ferritin level. Which of the conditions listed is most likely to lead to a falsely elevated result?

Locally perforated sigmoid colonic adenocarcinoma

Colonic angiodysplasia

Dieulafoy lesion of the stomach

Transitional cell carcinoma of the bladder

Endometrial adenocarcinoma

A

A locally perforated colonic tumour will typically cause an intense inflammatory response and if peritonitis is not present clinically then at the very least a localised abscess. This inflammatory process is the most likely (from the list) to falsely raise the serum ferritin level. Angiodysplasia and dieulafoy lesions are mucosal arteriovenous malformations and unlikely to result in considerable inflammatory activity.

Ferritin

Ferritin is an intracellular protein that binds iron and stores it to be released in a controlled fashion at sites where iron is required. Because iron and ferritin are bound the total body ferritin levels may be decreased in cases of iron deficiency anaemia. Measurement of serum ferritin levels can be useful in determining whether an apparently low haemoglobin and microcytosis is truly caused by an iron deficiency state.
Ferritin is an acute phase protein and may be synthesised in increased quantities in situations where inflammatory activity is ongoing. Falsely elevated results may therefore be encountered clinically and need to be taken in context of the clinical picture and full blood count results.

1703
Q

A 23 year old man presents with weight loss fatigue and lymphadenopathy. He is diagnosed with tuberculosis. Which of the following processes most closely matches the underlying pathological process?

Type 1 hypersensitivity reaction

Type 2 hypersensitivity reaction

Type 3 hypersensitivity reaction

Type 4 hypersensitivity reaction

None of the above

A

Granulomas (which occur in tuberculosis) are a feature of Type 4 hypersensitivity reactions.

Hypersensitivity reactions

The Gell and Coombs classification divides hypersensitivity reactions into 4 types

Type I Type II Type III Type IV
Description Anaphylactic Cytotoxic Immune complex Delayed type
Mediator IgE IgG, IgM IgG, Ig A, IgM T-cells
Antigen Exogenous Cell surface Soluble Tissues
Response time Minutes Hours Hours 2-3 days
Examples Asthma
Hay fever Autoimmune haemolytic anaemia
Pemphigus
Goodpasture’s Serum sickness
SLE
Aspergillosis Graft versus host disease
Contact dermatitis

1704
Q

A 10 year old girl presents with epistaxis. From which of the following regions is the bleeding most likely to originate?

Posterior nasal space

Alar rim

Kiesselbach’s plexus

Cribriform plate

None of the above

A

Kiesselbachs plexus has an arterial supply derived from both the internal and external carotid arteries and is the commonest area for bleeding in idiopathic epistaxis.

Epistaxis

Usually trivial and insignificant but severe haemorrhage may compromise airway and pose a risk to life.

Anatomy:

Arterial supply
From internal and external carotid
An arterial plexus exists at Little’s area and is the source of bleeding in 90% cases
Major arterial supply is from the sphenopalatine and greater palatine arteries (branches of the maxillary artery)
The facial artery supplies the more anterior aspect of the nose
Ethmoidal arteries are branches of the ophthalmic artery. They supply the posterosuperior nasal cavity

Venous drainage follows the arterial pattern

Classification
Primary idiopathic epistaxis accounts for 75% of all cases
Secondary cases arise as a result of events such as anticoagulants, trauma and coagulopathy
Classification into anterior and posterior epistaxis may help to locate the source and becomes more important when invasive treatment is required

Management
Resuscitate if required
Subject should sit upright and pinch nose firmly
Nasal cavity should be examined using a headlight
Simple anterior epistaxis may be managed using silver nitrate cautery. If difficult to manage then custom manufactured packs may be inserted
Posterior packing or tamponade may be achieved by passing a balloon tamponade device and inflating it. This is indicated where anterior packing alone has failed to achieve haemostasis.
Post nasal pack patients should receive antibiotics
Failure of these methods will require more invasive therapy. Where a vascular radiology suite is available, consideration may be given to angiographic techniques. Direct ligation of the nasal arterial supply may also be undertaken. Of the arterial ligation techniques available, the endo nasal sphenopalatine arterial ligation procedure is most popular.

1705
Q

A 32 year old man is admitted with a distended tense abdomen. He previously underwent a difficult appendicectomy 1 year previously and was discharged. At laparotomy the abdomen is filled with a gelatinous substance. What is the most likely diagnosis?

Appendiceal sarcoma

Pseudomyxoma peritoneii

Appendiceal adenocarcinoma

Appendiceal carcinoid

Caecal adenocarcinoma

A

Pseudomyxoma is classically associated with mucin production and the appendix is the commonest source.

Pseudomyxoma Peritonei

Rare mucinous tumour
Most commonly arising from the appendix (other abdominal viscera are also recognised as primary sites)
Incidence of 1-2/1,000,000 per year
The disease is characterised by the accumulation of large amounts of mucinous material in the abdominal cavity

Treatment
Is usually surgical and consists of cytoreductive surgery (and often peritonectomy c.f Sugarbaker procedure) combined with intra peritoneal chemotherapy with mitomycin C.

Survival is related to the quality of primary treatment and in Sugarbakers own centre 5 year survival rates of 75% have been quoted. Patients with disseminated intraperitoneal malignancy from another source fare far worse.
In selected patients a second look laparotomy is advocated and some practice this routinely.

1706
Q

A 72 year old man presents with haematuria which is recurrent. On investigation, a retrograde pyelogram shows multiple ureteric filling defects and the renal pelvis is irregular. What is the most likely cause?

Squamous cell carcinoma of the renal pelvis

Transitional cell carcinoma

Adenocarcinoma of the kidney

Retroperitoneal sarcoma

Retroperitoneal fibrosis

A

TCC of the renal pelvis may seed down the ureter.

Renal tumours

Renal cell carcinoma
Renal cell carcinoma is an adenocarcinoma of the renal cortex and is believed to arise from the proximal convoluted tubule. They are usually solid lesions, up to 20% may be multifocal, 20% may be calcified and 20% may have either a cystic component or be wholly cystic. They are often circumscribed by a pseudocapsule of compressed normal renal tissue. Spread may occur either by direct extension into the adrenal gland, renal vein or surrounding fascia. More distant disease usually occurs via the haematogenous route to lung, bone or brain.
Renal cell carcinoma comprise up to 85% of all renal malignancies. Males are more commonly affected than females and sporadic tumours typically affect patients in their sixth decade.
Patients may present with a variety of symptoms including; haematuria (50%), loin pain (40%), mass (30%) and up to 25% may have symptoms of metastasis.Less than 10% have the classic triad of haematuria, pain and mass.

Investigation
Many cases will present as haematuria and be discovered during diagnostic work up. Benign renal tumours are rare, so renal masses should be investigated with multislice CT scanning. Most tumours are also characterised with an arterial phase CT, particularly if they may be suitable for partial nephrectomy.

CT scanning of the chest and abdomen to detect distant disease should also be undertaken.

Routine bone scanning is not indicated in the absence of symptoms.

Biopsy should not be performed when a nephrectomy is planned but is mandatory before any ablative therapies are undertaken.

Assessment of the functioning of the contra lateral kidney.

Management
T1 lesions may be managed by partial nephrectomy and this gives equivalent oncological results to total radical nephrectomy. Partial nephrectomy may also be performed when there is inadequate reserve in the remaining kidney. Ablative techniques may also be considered for small T1 lesions in unfit patients. In general, outcomes are less favorable than with surgical resection.

For T2 lesions and above a radical nephrectomy is standard practice and this may be performed via a laparoscopic or open approach. Preoperative embolisation is not indicated nor is resection of uninvolved adrenal glands. During surgery early venous control is mandatory to avoid shedding of tumour cells into the circulation.

Patients with completely resected disease do not benefit from adjuvant therapy with either chemotherapy or biological agents. These should not be administered outside the setting of clinical trials.

Patients with transitional cell cancer will require a nephroureterectomy with disconnection of the ureter at the bladder.

References
Capitanio U, Montorsi F. Renal cancer. Lancet. 2016 Feb 27;387(10021):894-906.

Capitanio U et al. Epidemiology of Renal Cell Carcinoma. Eur Urol. 2019 Jan;75(1):74-84.

1707
Q

A 4 year old boy presents with haematuria and on examination is found to have a right sided renal mass. What is the most likely cause?

Wilms tumour

Glomerulonephritis

Horseshoe kidney

Renal adenocarcinoma

Ureteric transitional cell carcinoma

A

Wilms tumours (nephroblastoma) usually present in the first 4 years of life and may cause lung metastases.

Haematuria

Causes of haematuria

Trauma
Injury to renal tract
Renal trauma commonly due to blunt injury (others penetrating injuries)
Ureter trauma rare: iatrogenic
Bladder trauma: due to RTA or pelvic fractures
Infection
Remember TB
Malignancy
Renal cell carcinoma (remember paraneoplastic syndromes): painful or painless
Urothelial malignancies: 90% are transitional cell carcinoma, can occur anywhere along the urinary tract. Painless haematuria.
Squamous cell carcinoma and adenocarcinoma: rare bladder tumours
Prostate cancer
Penile cancers: SCC
Renal disease
Glomerulonephritis
Stones
Microscopic haematuria common
Structural abnormalities
Benign prostatic hyperplasia (BPH) causes haematuria due to hypervascularity of the prostate gland
Cystic renal lesions e.g. polycystic kidney disease
Vascular malformations
Renal vein thrombosis due to renal cell carcinoma
Coagulopathy
Causes bleeding of underlying lesions
Drugs
Cause tubular necrosis or interstitial nephritis: aminoglycosides, chemotherapy
Interstitial nephritis: penicillin, sulphonamides, and NSAIDs
Anticoagulants
Benign
Exercise
Gynaecological
Endometriosis: flank pain, dysuria, and haematuria that is cyclical
Iatrogenic
Catheterisation
Radiotherapy; cystitis, severe haemorrhage, bladder necrosis
Pseudohaematuria For example following consumption of beetroot

References
Http://bestpractice.bmj.com/best-practice/monograph/316/overview/aetiology.html

1708
Q

A 70 year old lady presents with a number of skin lesions that she describes as unsightly. On examination she has a number of raised lesions with a greasy surface located over her trunk. Apart from having a greasy surface the lesions also seem to have scattered keratin plugs located within them. What is the most likely diagnosis?

Keratoacanthoma

Pilar cysts

Dermatofibroma

Seborrhoeic keratosis

Epidermoid cyst

A

Seborrhoeic keratosis may have a number of appearances. However, the scaly, thick, greasy surface with scattered keratin plugs makes this the most likely diagnosis.

Benign skin diseases

Seborrhoeic keratosis
Most commonly arise in patients over the age of 50 years, often idiopathic
Equal sex incidence and prevalence
Usually multiple lesions over face and trunk
Flat, raised, filiform and pedunculated subtypes are recognised
Variable colours and surface may have greasy scale overlying it
Treatment options consist of leaving alone or simple shave excision

Melanocytic naevi
Congenital melanocytic naevi
Typically appear at, or soon after, birth
Usually greater than 1cm diameter
Increased risk of malignant transformation (increased risk greatest for large lesions)
Junctional melanocytic naevi
Circular macules
May have heterogeneous colour even within same lesion
Most naevi of the palms, soles and mucous membranes are of this type
Compound naevi
Domed pigmented nodules up to 1cm in diameter
Arise from junctional naevi, usually have uniform colour and are smooth
Spitz naevus
Usually develop over a few months in children
May be pink or red in colour, most common on face and legs
May grow up to 1cm and growth can be rapid, this usually results in excision
Atypical naevus syndrome
Atypical melanocytic naevi that may be autosomally dominantly inherited
Some individuals are at increased risk of melanoma (usually have mutations of CDKN2A gene)
Many people with atypical naevus syndrome AND a parent sibling with melanoma will develop melanoma

Epidermoid cysts
Common and affect face and trunk
They have a central punctum, they may contain small quantities of sebum
The cyst lining is either normal epidermis (epidermoid cyst) or outer root sheath of hair follicle (pilar cyst)

Dermatofibroma
Solitary dermal nodules
Usually affect extremities of young adults
Lesions feel larger than they appear visually
Histologically they consist of proliferating fibroblasts merging with sparsely cellular dermal tissues

Painful skin lesions
Eccrine spiradenoma
Neuroma
Glomus tumour
Leiomyoma
Angiolipoma
Neurofibroma (rarely painful) and dermatofibroma (rarely painful)

1709
Q

A 39 year old man notices a swelling in his left hemiscrotum. On examination lying supine, he has a left sided varicocele. The ipsilateral testis is normal on palpation. What is the most appropriate course of action?

Scrotal exploration and ligation of the varicocele

Abdominal ultrasound

Scrotal ultrasound

Left orchidectomy

Discharge

A

A left sided varicocele is a recognised presenting sign of a renal tumour occluding the renal vein (into which the left testicular vein drains). An abdominal ultrasound should be undertaken to exclude this. Surgery for uncomplicated varicocele is usually unnecessary.

Renal tumours

Renal cell carcinoma
Renal cell carcinoma is an adenocarcinoma of the renal cortex and is believed to arise from the proximal convoluted tubule. They are usually solid lesions, up to 20% may be multifocal, 20% may be calcified and 20% may have either a cystic component or be wholly cystic. They are often circumscribed by a pseudocapsule of compressed normal renal tissue. Spread may occur either by direct extension into the adrenal gland, renal vein or surrounding fascia. More distant disease usually occurs via the haematogenous route to lung, bone or brain.
Renal cell carcinoma comprise up to 85% of all renal malignancies. Males are more commonly affected than females and sporadic tumours typically affect patients in their sixth decade.
Patients may present with a variety of symptoms including; haematuria (50%), loin pain (40%), mass (30%) and up to 25% may have symptoms of metastasis.Less than 10% have the classic triad of haematuria, pain and mass.

Investigation
Many cases will present as haematuria and be discovered during diagnostic work up. Benign renal tumours are rare, so renal masses should be investigated with multislice CT scanning. Most tumours are also characterised with an arterial phase CT, particularly if they may be suitable for partial nephrectomy.

CT scanning of the chest and abdomen to detect distant disease should also be undertaken.

Routine bone scanning is not indicated in the absence of symptoms.

Biopsy should not be performed when a nephrectomy is planned but is mandatory before any ablative therapies are undertaken.

Assessment of the functioning of the contra lateral kidney.

Management
T1 lesions may be managed by partial nephrectomy and this gives equivalent oncological results to total radical nephrectomy. Partial nephrectomy may also be performed when there is inadequate reserve in the remaining kidney. Ablative techniques may also be considered for small T1 lesions in unfit patients. In general, outcomes are less favorable than with surgical resection.

For T2 lesions and above a radical nephrectomy is standard practice and this may be performed via a laparoscopic or open approach. Preoperative embolisation is not indicated nor is resection of uninvolved adrenal glands. During surgery early venous control is mandatory to avoid shedding of tumour cells into the circulation.

Patients with completely resected disease do not benefit from adjuvant therapy with either chemotherapy or biological agents. These should not be administered outside the setting of clinical trials.

Patients with transitional cell cancer will require a nephroureterectomy with disconnection of the ureter at the bladder.

References
Capitanio U, Montorsi F. Renal cancer. Lancet. 2016 Feb 27;387(10021):894-906.

Capitanio U et al. Epidemiology of Renal Cell Carcinoma. Eur Urol. 2019 Jan;75(1):74-84.

1710
Q

In relation to operating in the elderly which statement is false?

A 30 minute increment in operation length is associated with increase in mortality in patients over the age of 80

Hypoalbuminaemia is associated with increased mortality

Statins given preoperatively reduce perioperative cardiac events

Elevated brain (or B-type) natriuretic peptide (BNP) levels before undergoing non cardiac surgery is associated with high risk of cardiac mortality and all cause mortality

Beta blockers should be stopped acutely prior to surgery due to risk of perioperative hypotension

A

Beta blockers should not be stopped acutely prior to surgery as there may be a rebound effect associated with increased complications.

Brain natriuretic peptide is a neurohormone synthesized in the cardiac ventricles. Levels have been used to assess prognosis in heart failure and acute coronary syndromes. Preoperative elevated brain natriuretic peptide levels identify patients undergoing non cardiac surgery at high risk of cardiac mortality and all cause mortality.

All patients with peripheral vascular disease should take statins prior to vascular surgery as studies have shown a 50% risk reduction and a reduction in perioperative cardiac events.

Reference
1. Dernellis J, Panaretou M. Assessment of cardiac risk before non-cardiac surgery: brain natriuretic peptide in 1590 patients. Heart 2006;92:1645-1650
2. Poldermans, D et al Fluvastatin and Perioperative Events in Patients Undergoing Vascular Surgery. NEJM 2009; 361:980-989

Proactive care of older people undergoing surgery (POPS)

Comprehensive geriatric assessment
MDT assessment preoperatively
Main predictors of complications are co-morbidities cardiac disease and reduced functional capacity - preoperative assessment is the key to preventing adverse postoperative outcomes
Patients screened for risk factors (albumin <30, co morbidities)
Management plan made and disseminated to all involved
Patients education: pain relief, post op exercises, nutrition

Outcomes:
Fewer postoperative medical complications
Reduced length of stay by 4.5 days

References
Proactive care of older people undergoing surgery (POPS)
Danielle Harari et al.
Age and Ageing 2007 36(2):190-196

1711
Q

A 52 year old female renal patient needs a femoral catheter to allow for haemodialysis. Which of the structures listed below is least likely to be encountered during its insertion?

Great saphenous vein

Deep circumflex iliac artery

Superficial circumflex iliac artery

Femoral vein

Femoral branch of the genitofemoral nerve

A

Femoral access catheters are typically inserted in the region of the femoral triangle. Therefore the physician may encounter the femoral, vein, nerve, branches of the femoral artery and tributaries of the femoral vein. The deep circumflex iliac artery arises above the inguinal ligament and is therefore less likely to be encountered than the superficial circumflex iliac artery which arises below the inguinal ligament.

Femoral triangle anatomy

Boundaries
Superiorly Inguinal ligament
Laterally Sartorius
Medially Adductor longus
Floor Iliopsoas, adductor longus and pectineus
Roof
Fascia lata and Superficial fascia
Superficial inguinal lymph nodes (palpable below the inguinal ligament)
Long saphenous vein

Contents
Femoral vein (medial to lateral)
Femoral artery-pulse palpated at the mid inguinal point
Femoral nerve
Deep and superficial inguinal lymph nodes
Lateral cutaneous nerve
Great saphenous vein
Femoral branch of the genitofemoral nerve

1712
Q

A 21 year old man has an inguinal hernia and is undergoing a surgical repair. As the surgeons approach the inguinal canal they expose the superficial inguinal ring. Which of the following forms the lateral edge of this structure?

Inferior epigastric artery

Conjoint tendon

Rectus abdominis muscle

External oblique aponeurosis

Transversalis fascia

A

The external oblique aponeurosis forms the anterior wall of the inguinal canal and also the lateral edge of the superficial inguinal ring. The rectus abdominis lies posteromedially and the transversalis posterior to this.

Inguinal canal

Location
Above the inguinal ligament
The inguinal canal is 4cm long
The superficial ring is located anterior to the pubic tubercle
The deep ring is located approximately 1.5-2cm above the half way point between the anterior superior iliac spine and the pubic tubercle

Boundaries of the inguinal canal
Floor
External oblique aponeurosis
Inguinal ligament
Lacunar ligament
Roof
Internal oblique
Transversus abdominis
Anterior wall External oblique aponeurosis
Posterior wall
Transversalis fascia
Conjoint tendon
Laterally
Internal ring
Transversalis fascia
Fibres of internal oblique
Medially
External ring
Conjoint tendon

Contents
Males Spermatic cord and ilioinguinal nerve As it passes through the canal the spermatic cord has 3 coverings:
External spermatic fascia from external oblique aponeurosis
Cremasteric fascia
Internal spermatic fascia
Females Round ligament of uterus and ilioinguinal nerve

Related anatomy of the inguinal region
The boundaries of Hesselbachs triangle are commonly tested and illustrated below:

1713
Q

A 33 year old man develops a complex tibial and associated fibular fracture that is managed in a cast for 6 weeks. On removal of the cast the patient is noted to have a foot drop. Which nerve is most likely to be compromised?

Obturator

Sciatic

Tibial

Common peroneal

Medial plantar

A

The cast is most likely to affect the common peroneal nerve as it wraps around the fibular head. A foot drop is the typical end result of such an injury.

Common peroneal nerve

Derived from the dorsal divisions of the sacral plexus (L4, L5, S1 and S2).

This nerve supplies the skin and fascia of the anterolateral surface of the leg and the dorsum of the foot. It also innervates the muscles of the anterior and peroneal compartments of the leg, extensor digitorum brevis as well as the knee, ankle and foot joints.

It is laterally placed within the sciatic nerve. From the bifurcation of the sciatic nerve it passes inferolaterally in the lateral and proximal part of the popliteal fossa, under the cover of biceps femoris and its tendon. To reach the posterior aspect of the fibular head. It ends by dividing into the deep and superficial peroneal nerves at the point where it winds around the lateral surface of the neck of the fibula in the body of peroneus longus, approximately 2cm distal to the apex of the head of the fibula. It is palpable posterior to the head of the fibula.

Branches
In the thigh Nerve to the short head of biceps
Articular branch (knee)
In the popliteal fossa Lateral cutaneous nerve of the calf
Neck of fibula Superficial and deep peroneal nerves

1714
Q

Which surgical energy device is most appropriate for performing a sphincterotomy during an ERCP?

Monopolar diathermy device in cutting mode

Bipolar diathermy

Monopolar diathermy in coagulation mode

Harmonic ACE

CUSA

A

The cutting monopolar mode is needed, that’s why this can be complicated by bleeding.

Diathermy

  • Diathermy devices are used by surgeons in all branches of surgery.
    Use electric currents to produce local heat and thereby facilitate haemostasis or surgical dissection.
    Consist of a generator unit that is located outside the patient and can be set to the level of power required by the surgeon.
    There are two major types of diathermy machine;

Monopolar
The current flows through the diathermy unit into a handheld device that is controlled by the surgeon. Electricity can flow from the tip of the device into the patient. The earth electrode is located some distance away. The relatively narrow tip of the diathermy device produces local heat and this can be used to vaporise and fulgurate tissues. The current can be adjusted in terms of frequency so that different actions can be effected. In cutting mode sufficient power is applied to the tissues to vaporise their water content. In coagulation mode the power level is reduced so that a coagulum is formed instead. Some diathermy machines can utilise a setting known as blend that alternates cutting and coagulation functions, these tend to be used during procedures such as colonoscopic polypectomy.

Bipolar
The electric current flows from one electrode to another however, both electrodes are usually contained within the same device e.g. a pair of forceps. The result is that heating is localised to the area between the two electrodes and surrounding tissue damage is minimised.

Ultrasound based devices
These include CUSA and Harmonic scalpel. They generate high frequency oscillations that seal and coagulate tissues. They have different energy settings that allow them to dissect and simultaneously seal vessels if required. The CUSA device leaves vessels intact that may then be divided.

Ligasure device
Delivers tailored energy levels to allow simultaneous haemostasis and dissection. The device senses the impedance of the tissues and tailors energy levels accordingly.

Hazards of diathermy
Inadvertent patient burn. This may result of careless handling of the device or in the case of monopolar devices forgetting to apply a return electrode plate, In this situation patients may develop a contact burn when electricity flows to earth
Explosion or fire. This may occur when volatile anaesthetic gases or skin preparation fluid have been used

1715
Q

At which of the following sites is the development of diverticulosis least likely?

Caecum

Ascending colon

Transverse colon

Sigmoid colon

Rectum

A

Rectal involvement with diverticular disease almost never occurs.
Because the rectum has a longitudinal muscle coat (blending of the tenia marks the recto-sigmoid junction), diverticular disease almost never occurs here. Right sided colonic diverticular disease is well recognised (though less common than left sided).

Diverticular disease

Diverticular disease is a common surgical problem. It consists of herniation of colonic mucosa through the muscular wall of the colon. The usual site is between the taenia coli where vessels pierce the muscle to supply the mucosa. For this reason, the rectum, which lacks taenia, is often spared.

Symptoms
Altered bowel habit
Bleeding
Abdominal pain

Complications
Diverticulitis
Haemorrhage
Development of fistula
Perforation and faecal peritonitis
Perforation and development of abscess
Development of diverticular phlegmon

Diagnosis
Patients presenting in clinic will typically undergo either a colonoscopy, CT cologram or barium enema as part of their diagnostic work up. All tests can identify diverticular disease. It can be far more difficult to confidently exclude cancer, particularly in diverticular strictures.

Acutely unwell surgical patients should be investigated in a systematic way. Plain abdominal films and an erect chest x-ray will identify perforation. An abdominal CT scan (not a CT cologram) with oral and intravenous contrast will help to identify whether acute inflammation is present but also the presence of local complications such as abscess formation.

Severity Classification- Hinchey
I Para-colonic abscess
II Pelvic abscess
III Purulent peritonitis
IV Faecal peritonitis

Treatment
Increase dietary fibre intake.
Mild attacks of diverticulitis may be managed conservatively with antibiotics.
Peri colonic abscesses should be drained either surgically or radiologically.
Recurrent episodes of acute diverticulitis requiring hospitalisation are a relative indication for a segmental resection.
Hinchey IV perforations (generalised faecal peritonitis) will require a resection and usually a stoma. This group have a very high risk of post operative complications and usually require HDU admission.

1716
Q

A 43 year old lady presents with jaundice and is diagnosed as having a carcinoma of the head of the pancreas. Although she is deeply jaundiced, her staging investigations are negative for metastatic disease. What is the best method of biliary decompression in this case?

ERCP and sphincterotomy alone

ERCP alone

ERCP and placement of stent

Cholecystostomy

Choledochoduodenostomy

A

A stent is the best option for biliary decompression in resectable disease. Surgical bypasses have no place in the management of operable malignancy as a bridge to definitive surgery.

Pancreatic stents

Both benign and malignant biliary obstruction may be treated by placement of stents. These may be either plastic tubes or self expanding metallic stents. They can be placed either percutaneously, at ERCP, or, less commonly now, open surgery. Complications include blockage, displacement and those related to the method of insertion.

Metallic Vs Plastic stents
Metallic stents Plastic stents
Expensive Cheap
Embed in surrounding tissues Do not usually embed
Displacement rare Displacement common
Blockage rare Blockage common

1717
Q

Which of the following anaesthetic agents has the strongest analgesic effect?

Sodium thiopentone

Ketamine

Midazolam

Etomidate

None of the above

A

Ketamine has a moderate to strong analgesic effect. It may be used for emergency procedures outside the hospital environment to induce anaesthesia for procedures such as emergency amputation.

Anaesthetic agents

The table below summarises some of the more commonly used IV induction agents
Agent Specific features
Propofol
Rapid onset of anaesthesia
Pain on IV injection
Rapidly metabolised with little accumulation of metabolites
Proven anti emetic properties
Moderate myocardial depression
Widely used especially for maintaining sedation on ITU, total IV anaesthesia and for daycase surgery
Sodium thiopentone
Extremely rapid onset of action making it the agent of choice for rapid sequence of induction
Marked myocardial depression may occur
Metabolites build up quickly
Unsuitable for maintenance infusion
Little analgesic effects
Ketamine
May be used for induction of anaesthesia
Has moderate to strong analgesic properties
Produces little myocardial depression making it a suitable agent for anaesthesia in those who are haemodynamically unstable
May induce state of dissociative anaesthesia resulting in nightmares
Etomidate
Has favorable cardiac safety profile with very little haemodynamic instability
No analgesic properties
Unsuitable for maintaining sedation as prolonged (and even brief) use may result in adrenal suppression
Post operative vomiting is common

1718
Q

A 6 week old baby boy is brought to the clinic by his mother. She is concerned because although the left testis is present in the scrotum the right testis is absent. She reports that it is sometimes palpable when she bathes the child. on examination the right testis is palpable at the level of the superficial inguinal ring. What is the most appropriate management?

Discharge

Re-assess in 5 years

Laparoscopy

Re-assess in 6 months

Orchidopexy

A

Undescended testes are not uncommon in young children. They may be present in 4% of term infants, but only in 1.3% children at 3 months of age. In this scenario the testis is retractile and can be managed expectantly.

Cryptorchidism

A congenital undescended testis is one that has failed to reach the bottom of the scrotum by 3 months of age. At birth up to 5% of boys will have an undescended testis, post natal descent occurs in most and by 3 months the incidence of cryptorchidism falls to 1-2%. In the vast majority of cases the cause of the maldescent is unknown. A proportion may be associated with other congenital defects including:

Patent processus vaginalis
Abnormal epididymis
Cerebral palsy
Developmental delay
Wilms tumour
Abdominal wall defects (e.g. gastroschisis, prune belly syndrome)

Differential diagnosis
These include retractile testes and, in the case of absent bilateral testes the possibility of intersex conditions. A retractile testis can be brought into the scrotum by the clinician and when released remains in the scrotum. If the examining clinician notes the testis to return rapidly into the inguinal canal when released then surgery is probably indicated.

Reasons for correction of cryptorchidism
Reduce risk of infertility
Allows the testes to be examined for testicular cancer
Avoid testicular torsion
Cosmetic appearance
Males with undescended testis are 40 times as likely to develop testicular cancer (seminoma) as males without undescended testis
The location of the undescended testis affects the relative risk of testicular cancer (50% intra-abdominal testes)

Treatment
Orchidopexy at 6- 18 months of age. The operation usually consists of inguinal exploration, mobilisation of the testis and implantation into a dartos pouch.
Intra-abdominal testis should be evaluated laparoscopically and mobilised. Whether this is a single stage or two stage procedure depends upon the exact location.
After the age of 2 years in untreated individuals the Sertoli cells will degrade and those presenting late in teenage years may be better served by orchidectomy than to try and salvage a non functioning testis with an increased risk of malignancy.

1719
Q

During an open Watsons Fundoplication, the inferior pole of the spleen is injured causing troublesome bleeding. What is the best course of action?

Removal of the entire spleen

Partial splenectomy

Use of argon plasma coagulation system

Sutured splenorrhaphy

Sutured ligation of the splenic hilar vessels

A

The argon plasma coagulation system is very good for managing splenic bleeding. Alternatives include topical haemostatic agents. Its not necessary to ligate the hilar vessels, if this is required, a splenectomy is the usual outcome.

Management of bleeding

Bleeding is a process that is encountered in all branches of surgery. The decision as to how best to manage bleeding depends upon the site, vessel and circumstances.

Management of superficial dermal bleeding
This will usually cease spontaneously. If it is troublesome then direct use of monopolar or bipolar cautery devices will usually control the situation. Scalp wounds are a notable exception and the bleeding from these may be brisk. In this situation the use of mattress sutures as a wound closure method will usually address the problem.

Superficial arterial bleeding
If the vessel can be safely identified then the easiest method is to apply a haemostatic clip and ligate the vessel.

Major arterial bleeding
If the vessel can be clearly identified and is accessible then it may be possible to apply a clip and ligate the vessel. If the vessel is located in a pool of blood then blind application of haemostatic clips is highly dangerous and may result in collateral injury. In this situation evacuating the clot and packing the area is often safer. The pack can then be carefully removed when the required instruments are available. Some vessels may retract and bleeding may then be controlled by dissection of surrounding structures or under-running the bleeding point.

Major venous bleeding
The safest initial course of action is to apply digital pressure to the bleeding point. To control the bleeding the surgeon will need a working suction device. Divided veins may require ligation. Incomplete lacerations of major veins (e.g. IVC) are best repaired. In order to do this it is safest to apply a Satinsky type vascular clamp and repair the defect with 5/0 prolene.

Bleeding from raw surfaces
This may be mixed bleeding and can be troublesome. Spray diathermy and argon plasma coagulation are both useful agents. Certain topical haemostatic agents such as surgicell are useful in encouraging clot formation and may be used in conjunction with, or instead of, the above agents.

1720
Q

A 12 year old boy is brought to the clinic with history of weight loss and bloody diarrhoea. Abdominal examination is unremarkable. What is the most likely diagnosis?

Colorectal cancer

Inflammatory bowel disease

Irritable bowel syndrome

Munchausen syndrome

Viral gastroenteritis

A

The systemic features in the history are strongly suggestive of inflammatory bowel disease rather than the other causes. Colorectal cancer is almost unheard of at this age.

Paediatric proctology

Children may present with altered bowel habit and/ or rectal bleeding. Classical haemorroidal disease is relatively rare in children. Painful bright red rectal bleeding is much more common since constipation is a relatively common childhood disorder. The hard stool causes a tear of the ano-rectal mucosa with subsequent fissure. The pain from the fissure must be addressed promptly or the child will delay defecation and this fissure will worsen.

Inflammatory bowel disease may present in a similar pattern in paediatric practice with altered bowel habit (usually diarrhoea) and bleeding. Systemic features may be present and investigation with an endoscopy may be required.

Children with intussceception usually present at a relatively young age and the history is usually one of colicky abdominal pain, together with a mass on clinical examination.The often cited red current jelly type stool is a rare but classical feature.

Juvenile polyps may occur as part of the familial polyposis coli syndromes. The lesions, which are hamartomas, are often cherry red if they protrude externally.

1721
Q

A 52 year old man is having a blood transfusion after losing blood from a total knee replacement. He is normally fit and well. Three hours into the transfusion he complains of sudden onset abdominal pain and nausea. His temperature is 39 degrees, Blood pressure 98/42 mmHg, HR 105 bpm and saturations 94% air. His urine appears dark. What is the most likely diagnosis?

Delayed haemolytic transfusion reaction

Neutrophilic febrile reaction

Acute haemolytic transfusion reaction

Sickle cell crisis

Transfusion associated lung injury

A

Rapid intravascular haemolysis leading to shock, DIC and death can occur with this reaction.
Blood transfusion reactions

Acute transfusion reactions present as adverse signs or symptoms during or within 24 hours of a blood transfusion. The most frequent reactions are fever, chills, pruritus, or urticaria, which typically resolve promptly without specific treatment or complications. Other signs occurring in temporal relationship with a blood transfusion, such as severe dyspnoea, pyrexia, or loss of consciousness may be the first indication of a more severe potentially fatal reaction.
The causes of adverse reactions are multi-factorial. Immune mediated reactions, some of the most feared, occur as a result of component mismatch, the commonest cause of which is clerical error. More common, non immune mediated, complications may occur as a result of product contamination, this may be bacterial or viral.
Transfusion related lung injury is well recognised and there are two proposed mechanisms which underpin this. One involves the sequestration of primed neutrophils within the recipient pulmonary capillary bed. The other proposed mechanism suggests that HLA mismatches between donor neutrophils and recipient lung tissue is to blame.
The table below summarises the main types of transfusion reaction.

Immune mediated Non immune mediated
Pyrexia Hypocalcaemia
Alloimmunization CCF
Thrombocytopaenia Infections
Transfusion associated lung injury Hyperkalaemia
Graft vs Host disease
Urticaria
Acute or delayed haemolysis
ABO incompatibility
Rhesus incompatibility

1722
Q

A 24-year-old female is referred to the acute surgical team as she is noted to have an absent left radial pulse. Apart from some dizziness and lethargy, the patient does not have any features suggestive of an acute ischaemic limb. Blood tests are as follows:

Na+ 136 mmol/l
K+ 4.1 mmol/l
Urea 2.3 mmol/l
Creatinine 77 µmol/l

ESR 66 mm/hr

What is the most likely diagnosis?

Turner’s syndrome

Takayasu’s arteritis

Kawasaki disease

Coarctation of the aorta

Breast carcinoma with local spread

A

Inflammatory, obliterative arteritis affecting aorta and branches
Females> Males
Symptoms may include upper limb claudication
Clinical findings include diminished or absent pulses
ESR often affected during the acute phase

akayasu’s arteritis

Takayasu’s arteritis is a large vessel vasculitis. It typically causes occlusion of the aorta and questions commonly refer to an absent limb pulse. It is more common in females and Asian people

Associations
renal artery stenosis

Management
steroids

1723
Q

Which of the following statements in relation to fistula in ano is untrue?

High fistulae are safest treated with a seton insertion

Low fistulae may be laid open

They are typically probed with Lockhart Mummery probes

When discovered during incision and drainage of peri anal abscess; should always be probed to locate the internal opening

When complicating Crohns disease, may respond to infliximab

A

Probing fistulae during acute sepsis is associated with a high complication rate and should not be undertaken routinely.

Fistulas

A fistula is defined as an abnormal connection between two epithelial surfaces.
There are many types ranging from Branchial fistulae in the neck to entero-cutaneous fistulae abdominally.
In general surgical practice the abdominal cavity generates the majority and most of these arise from diverticular disease and Crohn’s.
As a general rule all fistulae will resolve spontaneously as long as there is no distal obstruction. This is particularly true of intestinal fistulae.

The four types of fistulae are:

Enterocutaneous
These link the intestine to the skin. They may be high (>500ml) or low output (<250ml) depending upon source. Duodenal /jejunal fistulae will tend to produce high volume, electrolyte rich secretions which can lead to severe excoriation of the skin. Colo-cutaneous fistulae will tend to leak faeculent material. Both fistulae may result from the spontaneous rupture of an abscess cavity onto the skin (such as following perianal abscess drainage) or may occur as a result of iatrogenic input. In some cases it may even be surgically desirable e.g. mucous fistula following sub total colectomy for colitis.

Suspect if there is excess fluid in the drain.

Enteroenteric or Enterocolic
This is a fistula that involves the large or small intestine. They may originate in a similar manner to enterocutaneous fistulae. A particular problem with this fistula type is that bacterial overgrowth may precipitate malabsorption syndromes. This may be particularly serious in inflammatory bowel disease.

Enterovaginal
Aetiology as above.

Enterovesical
This type of fistula goes to the bladder. These fistulas may result in frequent urinary tract infections, or the passage of gas from the urethra during urination.

Management
Some rules relating to fistula management:
They will heal provided there is no underlying inflammatory bowel disease and no distal obstruction, so conservative measures may be the best option
Where there is skin involvement, protect the overlying skin, often using a well fitted stoma bag- skin damage is difficult to treat
A high output fistula may be rendered more easily managed by the use of octreotide, this will tend to reduce the volume of pancreatic secretions.
Nutritional complications are common especially with high fistula (e.g. high jejunal or duodenal) these may necessitate the use of TPN to provide nutritional support together with the concomitant use of octreotide to reduce volume and protect skin.
When managing perianal fistulae surgeons should avoid probing the fistula where acute inflammation is present, this almost always worsens outcomes.
When perianal fistulae occur secondary to Crohn’s disease the best management option is often to drain acute sepsis and maintain that drainage through the judicious use of setons whilst medical management is implemented.
Always attempt to delineate the fistula anatomy, for abscesses and fistulae that have an intra abdominal source the use of barium and CT studies should show a track. For perianal fistulae surgeons should recall Goodsall’s rule in relation to internal and external openings.

1724
Q

Which of the following local anaesthetic preparations would be most suitable for an 18 year old male undergoing a unilateral Zadeks procedure?

Ring block with 0.5% bupivacaine with 1 in 200,000 adrenaline

Application of topical amethocaine

Ring block with 1% lignocaine alone

Ring block with 1% lignocaine and 1 in 200, 000 adrenaline

Ring block with 0.25% bupivacaine with 1 in 80,000 adrenaline

A

Local anaesthetics: avoid use of adrenaline in extremities
This is excision of the toe nail and a fast acting local anaesthetic is indicated. Whilst use of adrenaline in digits is now sometimes done, there is little additional benefit in this case.

Local anaesthetic agents

Lidocaine
An amide
Local anaesthetic and a less commonly used antiarrhythmic (affects Na channels in the axon)
Hepatic metabolism, protein bound, renally excreted
Toxicity: due to IV or excess administration. Increased risk if liver dysfunction or low protein states. Note acidosis causes lidocaine to detach from protein binding.
Drug interactions: Beta blockers, ciprofloxacin, phenytoin
Features of toxicity: Initial CNS over activity then depression as lidocaine initially blocks inhibitory pathways then blocks both inhibitory and activating pathways. Cardiac arrhythmias.
Increased doses may be used when combined with adrenaline to limit systemic absorption.

Cocaine
Pure cocaine is a salt, usually cocaine hydrochloride. It is supplied for local anaesthetic purposes as a paste.
It is supplied for clinical use in concentrations of 4 and 10%. It may be applied topically to the nasal mucosa. It has a rapid onset of action and has the additional advantage of causing marked vasoconstriction.
It is lipophillic and will readily cross the blood brain barrier. Its systemic effects also include cardiac arrhythmias and tachycardia.
Apart from its limited use in ENT surgery it is otherwise used rarely in mainstream surgical practice.

Bupivacaine
Bupivacaine binds to the intracellular portion of sodium channels and blocks sodium influx into nerve cells, which prevents depolarization.
It has a much longer duration of action than lignocaine and this is of use in that it may be used for topical wound infiltration at the conclusion of surgical procedures with long duration analgesic effect.
It is cardiotoxic and is therefore contra indicated in regional blockage in case the tourniquet fails.
Levobupivacaine (Chirocaine) is less cardiotoxic and causes less vasodilation.

Prilocaine
Similar mechanism of action to other local anaesthetic agents. However, it is far less cardiotoxic and is therefore the agent of choice for intravenous regional anaesthesia e.g. Biers Block.

All local anaesthetic agents dissociate in tissues and this contributes to their therapeutic effect. The dissociation constant shifts in tissues that are acidic e.g. where an abscess is present, and this reduces the efficacy.

Doses of local anaesthetics
Agent Dose plain Dose with adrenaline
Lignocaine 3mg/Kg 7mg/Kg
Bupivacaine 2mg/Kg 2mg/Kg
Prilocaine 6mg/Kg 9mg/Kg
These are a guide only as actual doses depend on site of administration, tissue vascularity and co-morbidities.

Maximum total local anaesthetic doses
Lignocaine 1% plain - 3mg/ Kg - 200mg (20ml)
Lignocaine 1% with 1 in 200,000 adrenaline - 7mg/Kg - 500mg (50ml)
Bupivacaine 0.5% - 2mg/kg- 150mg (30ml)
Maximum doses are based on ideal body weight

Effects of adrenaline
Adrenaline may be added to local anaesthetic drugs. It prolongs the duration of action at the site of injection and permits usage of higher doses (see above). It is contra indicated in patients taking MAOI’s or tricyclic antidepressants. The toxicity of bupivacaine is related to protein binding and addition of adrenaline to this drug does not permit increases in the total dose of bupivacaine, in contrast to the situation with lignocaine.

References
An excellent review is provided by:
French J and Sharp L. Local Anaesthetics. Ann R Coll Surg Engl 2012; 94: 76-80.

1725
Q

A 58 year old lady presents with changes that are suspicious of lichen sclerosus of the perineum. What is the best course of action?

Wide excision of the area and split thickness skin grafts

Punch biopsy

Excision of area with 0.5cm margin

Excision of the area and full thickness skin grafts

Excision of the area with 2cm margin

A

Punch biopsies are a useful option for obtaining a full thickness tissues sample with minimal tissue disruption. In this situation, the other differential would be AIN or VIN and punch biopsies would be useful in distinguishing these.

Skin Diseases

Skin lesions may be referred for surgical assessment, but more commonly will come via a dermatologist for definitive surgical management.

Skin malignancies include basal cell carcinoma, squamous cell carcinoma and malignant melanoma.

Basal Cell Carcinoma
Most common form of skin cancer.
Commonly occur on sun exposed sites apart from the ear.
Sub types include nodular, morphoeic, superficial and pigmented.
Typically slow growing with low metastatic potential.
Standard surgical excision, topical chemotherapy and radiotherapy are all successful.
As a minimum a diagnostic punch biopsy should be taken if treatment other than standard surgical excision is planned.

Squamous Cell Carcinoma
Again related to sun exposure.
May arise in pre - existing solar keratoses.
May metastasize if left.
Immunosupression (e.g. following transplant), increases risk.
Wide local excision is the treatment of choice and where a diagnostic excision biopsy has demonstrated SCC, repeat surgery to gain adequate margins may be required.

Malignant Melanoma
The main diagnostic features (major criteria):
Change in size
Change in shape
Change in colour
Secondary features (minor criteria)
Diameter >6mm
Inflammation
Oozing or bleeding
Altered sensation

Treatment
Suspicious lesions should undergo excision biopsy. The lesion should be removed completely as incision biopsy can make subsequent histopathological assessment difficult.
Once the diagnosis is confirmed the pathology report should be reviewed to determine whether further re-excision of margins is required (see below):

Margins of excision-Related to Breslow thickness
Lesions 0-1mm thick 1cm
Lesions 1-2mm thick 1- 2cm (Depending upon site and pathological features)
Lesions 2-4mm thick 2-3 cm (Depending upon site and pathological features)
Lesions >4 mm thick 3cm
Marsden J et al. Revised UK guidelines for management of Melanoma. Br J Dermatol 2010 163:238-256.

Further treatments such as sentinel lymph node mapping, isolated limb perfusion and block dissection of regional lymph node groups should be selectively applied.

Kaposi Sarcoma
Tumour of vascular and lymphatic endothelium.
Purple cutaneous nodules.
Associated with immuno supression.
Classical form affects elderly males and is slow growing.
Immunosupression form is much more aggressive and tends to affect those with HIV related disease.

Non malignant skin disease

Dermatitis Herpetiformis
Chronic itchy clusters of blisters.
Linked to underlying gluten enteropathy (coeliac disease).

Dermatofibroma
Benign lesion.
Firm elevated nodules.
Usually history of trauma.
Lesion consists of histiocytes, blood vessels and fibrotic changes.

Pyogenic granuloma
Overgrowth of blood vessels.
Red nodules.
Usually follow trauma.
May mimic amelanotic melanoma.

Acanthosis nigricans
Brown to black, poorly defined, velvety hyperpigmentation of the skin.
Usually found in body folds such as the posterior and lateral folds of the neck, the axilla, groin, umbilicus, forehead, and other areas.
The most common cause of acanthosis nigricans is insulin resistance, which leads to increased circulating insulin levels. Insulin spillover into the skin results in its abnormal increase in growth (hyperplasia of the skin).
In the context of a malignant disease, acanthosis nigricans is a paraneoplastic syndrome and is then commonly referred to as acanthosis nigricans maligna. Involvement of mucous membranes is rare and suggests a coexisting malignant condition.

1726
Q

A 63 year old man is admitted with rest pain and foot ulceration. An angiogram shows a 3 cm area of occlusion of the distal superficial femoral artery with 3 vessel run off. His ankle - brachial pressure index is 0.4. What is the most appropriate course of action?

Conservative management with medical therapy and exercise

Arterial bypass surgery using PTFE

Arterial bypass surgery using vein

Primary amputation

Angioplasty

A

Short segment disease and good run off with tissue loss is a compelling indication for angioplasty. He should receive aspirin and a statin if not already taking them.

Peripheral vascular disease

Indications for surgery to revascularise the lower limb
Intermittent claudication
Critical ischaemia
Ulceration
Gangrene

Intermittent claudication that is not disabling may provide a relative indication, whilst the other complaints are often absolute indications depending upon the frailty of the patient.

Assessment
Clinical examination
Ankle brachial pressure index measurement
Duplex arterial ultrasound
Angiography (standard, CT or MRI): usually performed only if intervention being considered.

Angioplasty
In order for angioplasty to be undertaken successfully the artery has to be accessible. The lesion relatively short and reasonable distal vessel runoff. Longer lesions may be amenable to sub-intimal angioplasty.

Surgery
Surgery will be undertaken where attempts at angioplasty have either failed or are unsuitable. Bypass essentially involves bypassing the affected arterial segment by utilising a graft to run from above the disease to below the disease. As with angioplasty good runoff improves the outcome.

Some key concepts with bypass surgery

Superficial femoral artery occlusion to the above knee popliteal
In the ideal scenario, vein (either in situ or reversed LSV) would be used as a conduit. However, prosthetic material has reasonable 5 year patency rates and some would advocate using this in preference to vein so that vein can be used for other procedures in the future. In general terms either technique is usually associated with an excellent outcome (if run off satisfactory).

Procedure
Artery dissected out, IV heparin 3,000 units given and then the vessels are cross clamped
Longitudinal arteriotomy
Graft cut to size and tunneled to arteriotomy sites
Anastomosis to femoral artery usually with 5/0 ‘double ended’ Prolene suture
Distal anastomosis usually using 6/0 ‘double ended’ Prolene

Distal disease
Femoro-distal bypass surgery takes longer to perform, is more technically challenging and has higher failure rates.
In elderly diabetic patients with poor runoff a primary amputation may well be a safer and more effective option. There is no point in embarking on this type of surgery in patients who are wheelchair bound.
In femorodistal bypasses vein gives superior outcomes to PTFE.

Rules
Vein mapping 1st to see whether there is suitable vein (the preferred conduit). Sub intimal hyperplasia occurs early when PTFE is used for the distal anastomosis and will lead to early graft occlusion and failure.
Essential operative procedure as for above knee fem-pop.
If there is insufficient vein for the entire conduit then vein can be attached to the end of the PTFE graft and then used for the distal anastomosis. This type of ‘vein boot’ is technically referred to as a Miller Cuff and is associated with better patency rates than PTFE alone.
Remember the more distal the arterial anastomosis the lower the success rate.

References
Peach G et al. Diagnosis and management of peripheral arterial disease. BMJ 2012; 345: 36-41.

1727
Q

A 45 year old man is admitted after his clothing caught fire. He suffers a full thickness circumferential burn to his lower thigh. He complains of increasing pain in lower leg and on examination there is parasthesia and severe pain in the lower leg. Foot pulses are normal. What is the most likely explanation?

Deep vein thrombosis

Compartment syndrome

Rhabdomyolysis

Synergistic spreading infection

Nerve injury

A

In compartment syndrome the loss of pulses is a very late sign indeed and only seen when compartment pressure exceeds arterial pressure
Circumferential burns may constrict the limb and cause a compartment syndrome to develop. Eshcarotomy is required, and compartmental decompression.

Burns

Burns may be thermal, chemical or electrical. In the former category are burns which occur as a result of heat. Chemical burns occur when the skin is exposed to an extremely caustic or alkaline substance. Electrical burns occur following exposure to electrical current. The immediate management includes removal of the burning source which usually includes irrigation of the burned area. A detailed assessment then needs to be made of the extent of the burns and a number of charts are available for recording this information. The degree of injury relates to the temperature and duration of exposure. Most domestic burns are mainly scalds in young children.

Following the burn, there is a local response with progressive tissue loss and release of inflammatory cytokines. Systemically, there are cardiovascular effects resulting from fluid loss and sequestration of fluid into the third space. There is a marked catabolic response. Immunosuppression is common with large burns and bacterial translocation from the gut lumen is a recognised event. Sepsis is a common cause of death following major burns.

Types of burn

Type of burn Skin layers affected Skin appearance Blanching Management
Epidermal/Superficial Epidermis Red, moist Yes
Superficial partial thickness Epidermis and part of papillary dermis affected Pale, dry Yes Normally heals with no intervention
Deep partial thickness Epidermis, whole papillary dermis affected Mottled red colour No Needs surgical intervention (depending on site)
Full thickness Whole skin layer and subcutaneous tissue affected Dry, leathery hard wound No Burns centre

Depth of burn assessment
Bleeding on needle prick
Sensation
Appearance
Blanching to pressure

Percentage burn estimation
Lund Browder chart: most accurate even in children
Wallace rule of nines
Palmar surface: surface area palm = 0.8% burn

> 15% body surface area burns in adults needs urgent burn fluid resuscitation

Transfer to burn centre if:
Need burn shock resuscitation
Face/hands/genitals affected
Deep partial thickness or full thickness burns
Significant electrical/chemical burns

Management
The initial aim is to stop the burning process and resuscitate the patient. Intravenous fluids will be required for children with burns greater than 10% of total body surface area. Adults with burns greater than 15% of total body surface area will also require IV fluids. The fluids are calculated using the Parkland formula which is; volume of fluid= total body surface area of the burn % x weight (Kg) x2-4 (preference for lower amount i.e. 2ml to avoid excessive fluid overload). Half of the fluid is administered in the first 8 hours. A urinary catheter should be inserted. Analgesia should be given. Complex burns, burns involving the hand perineum and face and burns >10% in adults and >5% in children should be transferred to a burns unit.

Circumferential burns affecting a limb or severe torso burns impeding respiration may require escharotomy to divide the burnt tissue.

Conservative management is appropriate for superficial burns and mixed superficial burns that will heal in 2 weeks. More complex burns may require excision and skin grafting. Excision and primary closure is not generally practised as there is a high risk of infection.

There is no evidence to support the use of anti microbial prophylaxis or topical antibiotics in burn patients.

Escharotomies
Indicated in circumferential full thickness burns to the torso or limbs.
Careful division of the encasing band of burn tissue will potentially improve ventilation (if the burn involves the torso), or relieve compartment syndrome and oedema (where a limb is involved)

References
www.euroburn.org/e107files/downloads/guidelinesburncare.pdf

Barajas-Nava LA, López-Alcalde J, Roqué i Figuls M, Solà I, Bonfill Cosp X. Antibiotic prophylaxis for preventing burn wound infection. Cochrane Database of Systematic Reviews 2013, Issue 6. Art. No.: CD008738. DOI: 10.1002/14651858.CD008738.pub2.

Hettiaratchy S & Papini R. Initial management of a major burn: assessment and resuscitation. BMJ 2004;329:101-103

1728
Q

Which of the following is not a branch of the posterior cord of the brachial plexus?

Thoracodorsal nerve

Axillary nerve

Radial nerve

Lower subscapular nerve

Musculocutaneous nerve

A

Mnemonic branches off the posterior cord

S ubscapular (upper and lower)
T horacodorsal
A xillary
R adial
The musculocutaneous nerve is a branch off the lateral cord.
Brachial plexus

The brachial plexus extends from the neck to the axilla. It is formed by the ventral rami of the fifth to the eighth cervical nerves with the ascending part of the first thoracic nerve.

Location of the plexus
The ventral rami which form the plexus enter the lower part of the posterior triangle of the neck in series with the ventral rami of the cervical plexus. The second part of the subclavian artery lies immediately anterior to the lower two rami. The upper three rami intermingle and pass inferolaterally towards the axilla and subclavian artery. They are enclosed within an extension of the prevertebral fascia. In the neck the plexus lies deep to platysma, the supraclavicular nerves, inferior belly of omohyoid and the transverse cervical artery. It then passes deep to the clavicle and the suprascapular vessels, to enter the axilla, and thence surround the second part of the axillary artery

Composition of the plexus
Ventral rami, the roots of the plexus, lie between scalenus medius and anterior.

As they enter the posterior triangle, the upper two (C5,6) and lower two (C8, T1) roots of the plexus unite to form the upper and lower trunks of the plexus respectively. Meanwhile, C7 continues as the middle trunk. The lower trunk may groove the superior surface of the first rib posterior to the subclavian artery, and the root from the first ventral ramus is always in contact with it.

Each trunk divides into ventral and dorsal divisions which are destined to supply the anterior (flexor) and posterior (extensor) parts of the upper limb.

The cords of the plexus are formed in the axilla. The dorsal divisions unite to form the posterior cord (C5-8). The ventral divisions of the upper and middle trunks unite to form the lateral cord (C5-7), while the ventral divisions of the lower trunk continues as the medial cord (C8-T1). The cords are named according to their relationship to the axillary artery. Each cord terminates by dividing into two main branches at the beginning of the third part of the artery.

Sympathetic communications
The fifth and sixth cervical ventral rami receive grey rami communicantes from the middle cervical ganglion, while the two or more grey rami communicantes pass from the inferior cervical ganglion to the seventh and eighth cervical ventral rami. The first thoracic ventral ramus receives its grey ramus from the cervicothoracic ganglion. Its for this reason that inferior plexus injury can be complicated by a Horners syndrome.

Summary
Origin Anterior rami of C5 to T1
Sections of the plexus
Roots, trunks, divisions, cords, branches
Mnemonic:Real Teenagers Drink Cold Beer
Roots
Located in the posterior triangle
Pass between scalenus anterior and medius
Trunks
Located posterior to middle third of clavicle
Upper and middle trunks related superiorly to the subclavian artery
Lower trunk passes over 1st rib posterior to the subclavian artery
Divisions Apex of axilla
Cords Related to axillary artery

1729
Q

Which of the following is not an oncogene?

ras

myc

sis

Ki 67

erb-B

A

Ki 67 is a nuclear proliferation marker (used in immunohistochemistry). Although, Ki67 positivity is a marker of malignancy, it is not itself, an oncogene.

Oncogenes

Oncogenes are cancer promoting genes that are derived from normal genes (proto-oncogenes). Proto-oncogenes play an important physiological role in cellular growth. They are implicated in the development of up to 20% of human cancers.

Proto-oncogenes may become oncogenes via the following processes:
Mutation (point mutation)
Chromosomal translocation
Increased protein expression
Only one mutated copy of the gene is needed for cancer to occur - a dominant effect

Classification of oncogenes
Growth factors e.g. Sis
Transcription factors e.g. Myc
Receptor tyrosine kinase e.g. RET
Cytoplasmic tyrosine kinase e.g. Src
Regulatory GTPases e.g. Ras

Tumour suppressor genes
Tumour suppressor genes restrict or repress cellular proliferation in normal cells. Their inactivation through mutation or germ line incorporation is implicated in renal, colonic, breast, bladder and many other cancers. One of the best known tumour suppressor genes is p53. p53 gene offers protection by causing apoptosis of damaged cells. Other well known genes include BRCA 1 and 2.

1730
Q

A 19 year old male presents with bilateral gynaecomastia, poor vision and nipple discharge. Which of the following blood tests is most likely to be abnormal?

Oestrogen

Testosterone

β HCG

Prolactin

Calcitonin

A

A combination of nipple discharge, gynaecomastia and poor vision may well be associated with a prolactinoma. The poor vision results from compression of the optic chiasm resulting in bi temporal hemianopia.

Gynaecomastia

Gynaecomastia describes an abnormal amount of breast tissue in males and is usually caused by an increased oestrogen:androgen ratio. It is important to differentiate the causes of galactorrhoea (due to the actions of prolactin on breast tissue) from those of gynaecomastia

Causes of gynaecomastia

physiological: normal in puberty
syndromes with androgen deficiency: Kallman’s, Klinefelter’s
testicular failure: e.g. Mumps
liver disease
testicular cancer e.g. Seminoma secreting HCG
ectopic tumour secretion
hyperthyroidism
haemodialysis
drugs: see below

Drug causes of gynaecomastia

spironolactone (most common drug cause)
cimetidine
digoxin
cannabis
finasteride
oestrogens, anabolic steroids

Very rare drug causes of gynaecomastia
tricyclics
isoniazid
calcium channel blockers
heroin
busulfan
methyldopa

Treatment options
identify and manage any overt underlying cause
liposuction provides the best cosmetic outcome

1731
Q

Which of the following structures does not pass behind the lateral malleolus?

Peroneus brevis tendon

Sural nerve

Short saphenous vein

Peroneus longus tendon

Tibialis anterior tendon

A

Tibialis anterior tendon passes anterior to the medial malleolus.

Lateral malleolus

Structures posterior to the lateral malleolus and superficial to superior peroneal retinaculum
Sural nerve
Short saphenous vein

Structures posterior to the lateral malleolus and deep to superior peroneal retinaculum
Peroneus longus tendon
Peroneus brevis tendon

The calcaneofibular ligament is attached at the lateral malleolus

1732
Q

A 42 year old lady presents with a small mass in the right lobe of the thyroid gland. Following excision, the histopathologist assess the lesion and makes remark about the significant amount of dystrophic calcification associated with the lesion? What is the most likely underlying definitive diagnosis?

Follicular adenoma

Follicular carcinoma

Anaplastic thyroid carcinoma

Papillary carcinoma

Toxic nodule

A

Calcification is likely to be Psammoma body seen in papillary lesions.

Thyroid malignancy

Papillary carcinoma
Commonest sub-type
Accurately diagnosed on fine needle aspiration cytology
Histologically, they may demonstrate psammoma bodies (areas of calcification) and so called ‘orphan Annie’ nuclei
They typically metastasise via the lymphatics and thus laterally located apparently ectopic thyroid tissue is usually a metastasis from a well differentiated papillary carcinoma

Follicular carcinoma
Are less common than papillary lesions
Like papillary tumours, they may present as a discrete nodule. Although they appear to be well encapsulated macroscopically there is invasion on microscopic evaluation
Lymph node metastases are uncommon and these tumours tend to spread haematogenously. This translates into a higher mortality rate
Follicular lesions cannot be accurately diagnosed on fine needle aspiration cytology and thus all follicular FNA’s (THY 3f) will require at least a hemi thyroidectomy

Anaplastic carcinoma
Less common and tend to occur in elderly females
Disease is usually advanced at presentation and often only palliative decompression and radiotherapy can be offered.

Medullary carcinoma
These are tumours of the parafollicular cells ( C Cells) and are of neural crest origin.
The serum calcitonin may be elevated which is of use when monitoring for recurrence.
They may be familial and occur as part of the MEN -2A disease spectrum.
Spread may be either lymphatic or haematogenous and as these tumours are not derived primarily from thyroid cells they are not responsive to radioiodine.

Lymphoma
These respond well to combined chemoradiotherapy
Radical surgery is unnecessary once the disease has been diagnosed on biopsy material. Such biopsy material is not generated by an FNA and thus a core biopsy has to be obtained (with care!).

1733
Q

Which of the vessels listed below is the most inferiorly sited single aortic branch?

Common iliac artery

Inferior mesenteric artery

Superior mesenteric artery

Gonadal artery

Median sacral artery

A

The median sacral artery leaves the aorta a little above its bifurcation. It descends in the midline anterior to L4 and L5.

Abdominal aortic branches

Branches Level Paired Type
Inferior phrenic T12 (Upper border) Yes Parietal
Coeliac T12 No Visceral
Superior mesenteric L1 No Visceral
Middle suprarenal L1 Yes Visceral
Renal L1-L2 Yes Visceral
Gonadal L2 Yes Visceral
Lumbar L1-L4 Yes Parietal
Inferior mesenteric L3 No Visceral
Median sacral L4 No Parietal
Common iliac L4 Yes Terminal

1734
Q

A 22 year old man is operated on for a left inguinal hernia, at operation the sac is opened to reveal a large Meckels diverticulum. What type of hernia is this?

Richters hernia

Morgagni hernia

Littres hernia

Spigelian hernia

Bochdalek hernia

A

Hernia containing Meckels diverticulum is termed a Littres hernia.

Hernia

Hernias occur when a viscus or part of it protrudes from within its normal anatomical cavity. Specific hernias are covered under their designated titles the remainder are addressed here.

Spigelian hernia
Interparietal hernia occurring at the level of the arcuate line
Rare
May lie beneath internal oblique muscle. Usually between internal and external oblique
Equal sex distribution
Position is lateral to rectus abdominis
Both open and laparoscopic repair are possible, the former in cases of strangulation

Lumbar hernia
The lumbar triangle (through which these may occur) is bounded by:
Crest of ilium (inferiorly)
External oblique (laterally)
Latissimus dorsi (medially)
Primary lumbar herniae are rare, most are incisional hernias following renal surgery
- Direct anatomical repair with or without mesh re-enforcement is the procedure of choice

Obturator hernia
Herniation through the obturator canal
Commoner in females
Usually lies behind pectineus muscle
Elective diagnosis is unusual most will present acutely with obstruction
When presenting acutely most cases with require laparotomy or laparoscopy (and small bowel resection if indicated)

Richters hernia
Condition in which part of the wall of the small bowel (usually the anti mesenteric border) is strangulated within a hernia (of any type)
They do not present with typical features of intestinal obstruction as lumenal patency is preserved
Where vomiting is prominent it usually occurs as a result of paralytic ileus from peritonitis (as these hernias may perforate)

Incisional hernia
Occur through sites of surgical access into the abdominal cavity
Most common following surgical wound infection
To minimise following midline laparotomy Jenkins Rule should be followed and this necessitates a suture length 4x length of incision with bites taken at 1cm intervals, 1 cm from the wound edge
Repair may be performed either at open surgery or laparoscopically and a wide variety of techniques are described

Bochdalek hernia
Typically congenital diaphragmatic hernia
85% cases are located in the left hemi diaphragm
Associated with lung hypoplasia on the affected side
More common in males
Associated with other birth defects
May contain stomach
May be treated by direct anatomical apposition or placement of mesh. In infants that have severe respiratory compromise mechanical ventilation may be needed and mortality rate is high

Morgagni Hernia
Rare type of diaphragmatic hernia (approx 2% cases)
Herniation through foramen of Morgagni
Usually located on the right and tend to be less symptomatic
More advanced cases may contain transverse colon
As defects are small pulmonary hypoplasia is less common
Direct anatomical repair is performed

Umbilical hernia
Hernia through weak umbilicus
Usually presents in childhood
Often symptomatic
Equal sex incidence
95% will resolve by the age of 2 years
Surgery performed after the third birthday

Paraumbilical hernia
Usually a condition of adulthood
Defect is in the linea alba
More common in females
Multiparity and obesity are risk factors
Traditionally repaired using Mayos technique - overlapping repair, mesh may be used though not if small bowel resection is required owing to acute strangulation

Littres hernia
Hernia containing Meckels diverticulum
Resection of the diverticulum is usually required and this will preclude a mesh repair

1735
Q

A 65 year old type 2 diabetic with poor glycaemic control is admitted with forefoot cellulitis. X-ray of the foot shows some evidence of osteomyelitis affecting the 2nd ray but overlying skin is healthy. What is the best treatment initially?

Intravenous antibiotics

Below knee amputation

Transfemoral amputation

Midfoot amputation

Ray amputation

A

It is worth attempting to try and resolve this situation with antibiotics at first presentation. A primary amputation will not heal well and may result in progressive surgery.

Osteomyelitis

Infection of the bone

Causes
S aureus and occasionally Enterobacter or Streptococcus species
In sickle cell: Salmonella species

Clinical features
Erythema
Pain
Fever

Investigation
X-ray: lytic centre with a ring of sclerosis
Bone biopsy and culture

Treatment
Prolonged antibiotics
Sequestra may need surgical removal

1736
Q

A 28 year old chef presents to the medical team with profuse bloody diarrhoea. On further questioning he describes tenesmus. They arrange a sigmoidoscopy which reveals necrosis and ulceration of the descending colon mucosa. What is the most likely cause?

Ischaemic colitis

Infection with Yersinia pestis

Infection with entero invasive E Coli

Infection with campylobacter

Infection with vibrio cholera

A

Necrosis and ulcers of the large bowel are a feature of enteroinvasive E.coli. It presents with a dysentery type illness similar to shigellosis.

Bacterial Gastroenteritis

Causative organisms Features
Campylobacter jejuni
Most common cause of acute infective diarrhoea
Spiral, gram negative rods
Usually infects caecum and terminal ileum. Local lymphadenopathy is common
May mimic appendicitis as it has marked right iliac fossa pain
Reactive arthritis is seen in 1-2% of cases
Shigella spp.
Members of the enterobacteriaceae
Gram negative bacilli
Clinically causes dysentery
Shigella soneii is the commonest infective organism (mild illness)
Usually self limiting, ciprofloxacin may be required if individual is in a high risk group
Salmonella spp
Facultatively anaerobic, gram negative, enterobacteriaceae
Infective dose varies according to subtype
Salmonellosis: usually transmitted by infected meat (especially poultry) and eggs
E. coli
Enteropathogenic
Enteroinvasive: dysentery, large bowel necrosis/ulcers
Enterotoxigenic: small intestine, travelers diarrhoea
Enterohaemorrhagic: 0157, cause a haemorrhagic colitis, haemolytic uraemic syndrome and thrombotic thrombocytopaenic purpura
Yersinia enterocolitica
Gram negative, coccobacilli
Typically produces a protracted terminal ileitis that may mimic Crohns disease
Differential diagnosis acute appendicitis
May progress to septicaemia in susceptible individuals
Usually sensitive to quinolone or tetracyclines
Vibrio cholera
Short, gram negative rods
Transmitted by contaminated water, seafood
Symptoms include sudden onset of effortless vomiting and profuse watery diarrhoea
Correction of fluid and electrolyte losses are the mainstay of treatment
Most cases will resolve, antibiotics are not generally indicated

1737
Q

Which of the following is not characteristic of a granuloma?

Altered macrophages

Fused macrophages

Epithelioid cells

Mixture of chronic inflammatory cells

Polymorphnuclear leucocytes, cellular debris and fibrin

A

These are typical components of an abscess cavity. Polymorphonuclear leucocytes may be found in a granuloma if there is a focus of suppuration.

Chronic inflammation

Overview
Chronic inflammation may occur secondary to acute inflammation.In most cases chronic inflammation occurs as a primary process. These may be broadly viewed as being one of three main processes:
Persisting infection with certain organisms such as Mycobacterium tuberculosis which results in delayed type hypersensitivity reactions and inflammation.
Prolonged exposure to non-biodegradable substances such as silica or suture materials which may induce an inflammatory response.
Autoimmune conditions involving antibodies formed against host antigens.

Acute vs. Chronic inflammation
Acute inflammation Chronic inflammation
Changes to existing vascular structure and increased permeability of endothelial cells Angiogenesis predominates
Infiltration of neutrophils Macrophages, plasma cells and lymphocytes predominate
Process may resolve with:
Suppuration
Complete resolution
Abscess formation
Progression to chronic inflammation
Healing by fibrosis
Healing by fibrosis is the main result

Granulomatous inflammation
A granuloma consists of a microscopic aggregation of macrophages (with epithelial type arrangement =epithelioid). Large giant cells may be found at the periphery of granulomas.

Mediators
Growth factors released by activated macrophages include agents such as interferon and fibroblast growth factor (plus many more). Some of these such as interferons may have systemic features resulting in systemic symptoms and signs, which may be present in individuals with long standing chronic inflammation.

1738
Q

A 35 year old woman who is a heavy smoker has long standing stool frequency and crampy abdominal pain. A colonoscopy is performed (which is macroscopically normal) and pan colonic biopsies are taken. Histological analysis of the biopsies demonstrates a thickened sub apical collagen layer together with an increase in lymphocytes in the lamina propria. What is the most likely diagnosis?

Microscopic colitis

Crohns disease

Ulcerative colitis

Pseudomembranous colitis

Irritable bowel syndrome

A

Microscopic colitis is a common condition characterised by normal endoscopic appearances, microscopic features of colonic inflammation and thickening of the sub epithelial collagen layer. Features such as granulomas are absent. It is the normal endoscopic appearance that makes the other options less likely. IBS is a diagnosis of exclusion and the features of inflammation activity would favor a different diagnosis. Severe cases can be treated with steroids. Other agents such as cholestyramine may be beneficial. It is important to exclude infection with a stool culture if this has not been done, not least because the precipitant of many cases of microscopic colitis is an episode of infective diarrhoea.

Diarrhoea

World Health Organisation definitions
Diarrhoea: > 3 loose or watery stool per day
Acute diarrhoea < 14 days
Chronic diarrhoea > 14 days

Acute Diarrhoea
Gastroenteritis May be accompanied by abdominal pain or nausea/vomiting
Diverticulitis Classically causes left lower quadrant pain, diarrhoea and fever
Antibiotic therapy More common with broad spectrum antibiotics
Clostridium difficile is also seen with antibiotic use
Constipation causing overflow A history of alternating diarrhoea and constipation may be given
May lead to faecal incontinence in the elderly

Chronic Diarrhoea
Irritable bowel syndrome Extremely common. The most consistent features are abdominal pain, bloating and change in bowel habit. Patients may be divided into those with diarrhoea predominant IBS and those with constipation predominant IBS.
Features such as lethargy, nausea, backache and bladder symptoms may also be present
Ulcerative colitis Bloody diarrhoea may be seen. Crampy abdominal pain and weight loss are also common. Faecal urgency and tenesmus may occur
Crohn’s disease Crampy abdominal pains and diarrhoea. Bloody diarrhoea less common than in ulcerative colitis. Other features include malabsorption, mouth ulcers perianal disease and intestinal obstruction
Colorectal cancer Symptoms depend on the site of the lesion but include diarrhoea, rectal bleeding, anaemia and constitutional symptoms e.g. Weight loss and anorexia
Coeliac disease
In children may present with failure to thrive, diarrhoea and abdominal distension
In adults lethargy, anaemia, diarrhoea and weight loss are seen. Other autoimmune conditions may coexist

Other conditions associated with diarrhoea include:
Thyrotoxicosis
Laxative abuse
Appendicitis with pelvic abscess or pelvic appendix
Radiation enteritis

Diagnosis
Stool culture
Abdominal and digital rectal examination
Consider colonoscopy (radiological studies unhelpful)
Thyroid function tests, serum calcium, anti endomysial antibodies, glucose

1739
Q

A 72 year old man with peripheral vascular disease develops a gangrenous toe. This becomes infected and there is evidence of infection in the surrounding tissues. On clinical palpation there is crepitus present within the tissues. What is the most likely infective organism?

Staphylococcus aureus

Streptococcus pyogenes

Clostridium difficile

Clostridium perfringens

Clostridium botulinum

A

Clostridium perfringens is the most likely pathogen to be associated with gangrene.

Surgical Microbiology

An extensive topic so an overview is given here. Organisms causing common surgical infections are reasonable topics in the examination. However, microbiology is less rigorously tested than anatomy, for example.

Common organisms

Staphylococcus aureus
Facultative anaerobe
Gram positive coccus
Haemolysis on blood agar plates
Catalase positive
20% population are long term carriers
Exo and entero toxin may result in toxic shock syndrome and gastroenteritis respectively
Ideally treated with penicillin although many strains now resistant through beta Lactamase production. In the UK less than 5% of isolates are sensitive to penicillin.
Resistance to methicillin (and other antibiotics) is mediated by the mec operon , essentially penicillin binding protein is altered and resistance to this class of antibiotics ensues
Common cause of cutaneous infections and abscesses

Streptococcus pyogenes
Gram positive, forms chain like colonies, Lancefield Group A Streptococcus
Produces beta haemolysis on blood agar plates
Rarely part of normal skin microflora
Catalase negative
Releases a number of proteins/ virulence factors into host including hyaluronidase, streptokinase which allow rapid tissue destruction
Releases superantigens such as pyogenic exotoxin A which results in scarlet fever
Remains sensitive to penicillin, macrolides may be used as an alternative.

Escherichia coli
Gram negative rod
Facultative anaerobe, non sporing
Wide range of subtypes and some are normal gut commensals
Some subtypes such as 0157 may produce lethal toxins resulting in haemolytic-uraemic syndrome
Enterotoxigenic E-Coli produces an enterotoxin (ST enterotoxin) that results in large volume fluid secretion into the gut lumen (Via cGMP activation)
Enteropathogenic E-Coli binds to intestinal cells and cause structural damage, this coupled with a moderate (or in case of enteroinvasive E-Coli significant) invasive component produces enteritis and large volume diarrhoea together with fever.
They are resistant to many antibiotics used to treat gram positive infections and acquire resistance rapidly and are recognised as producing beta lactamases

Campylobacter jejuni
Curved, gram negative, non sporulating bacteria
One of the commonest causes of diarrhoea worldwide
Produces enteritis which is often diffuse and blood may be passed
Remains a differential for right iliac fossa pain with diarrhoea
Self limiting infection so antibiotics are not usually advised. However, the quinolones are often rapidly effective.

Helicobacter pylori
Gram negative, helix shaped rod, microaerophillic
Produces hydrogenase that can derive energy from hydrogen released by intestinal bacteria
Flagellated and mobile
Those carrying the cag A gene may cause ulcers
It secretes urease that breaks down gastric urea> Carbon dioxide and ammonia> ammonium>bicarbonate (simplified!) The bicarbonate can neutralise the gastric acid.
Usually colonises the gastric antrum and irritates resulting in increased gastrin release and higher levels of gastric acid. These patients will develop duodenal ulcers. In those with more diffuse H-Pylori infection gastric acid levels are lower and ulcers develop by local tissue damage from H-Pylori- these patients get gastric ulcers.
Diagnosis may be made by serology (approx. 75% sensitive). Biopsy urease test during endoscopy probably the most sensitive.
In patients who are colonised 10-20% risk of peptic ulcer, 1-2% risk gastric cancer, <1% risk MALT lymphoma.

1740
Q

A 44 year old lady presents with a mass in the upper outer quadrant of her right breast. Imaging, histology and clinical examination confirm a 1.5cm malignant mass lesion with no clinical evidence of axillary nodal disease. What is the most appropriate treatment?

Radical chemoradiotherapy alone

Excision biopsy and sentinel node biopsy

Wide local excision and sentinel node biopsy

Mastectomy and axillary node sampling

Wide local excision and axillary node sampling

A

Small peripheral breast cancers are well suited to breast conserving surgical techniques.
A small peripheral lesion such as this would usually be suitable for breast conserving surgery. Since imaging and clinical examination is not suspicious for axillary disease, a sentinel lymph node biopsy should be performed. Axillary node sampling is a procedure that become obsolete with the introduction of sentinel node biopsy.

Breast cancer management

  • Surgery is performed in most patients suffering from breast cancer.
    Chemotherapy may be used to downstage tumours and allow breast conserving surgery. Hormonal therapy may also be used for the same purposes.
    Radiotherapy is given to most patients who have undergone breast conserving surgery (some older patients receiving hormone treatment and who have small low grade tumours may safely avoid DXT.
    Therapeutic mammoplasty is an option for some patients but requires symmetrizing surgery in most cases.
    Patients who have undergone mastectomy may be offered a reconstructive procedure either in conjunction with their primary resection or as a staged procedure at a later date.

Surgical options
Mastectomy vs Wide local excision

Mastectomy Wide Local Excision
Multifocal tumour Solitary lesion
Central tumour Peripheral tumour
Large lesion in small breast Small lesion in large breast
DCIS >4cm DCIS <4cm
Patient Choice Patient choice

Central lesions may be managed using breast conserving surgery, where an acceptable cosmetic result may be obtained, this is rarely the case in small breasts

Axillary disease
As a minimum, all patients with invasive breast cancer should have their axilla staged. In those who do not have overt evidence of axillary nodal involvement this can be undertaken using sentinel lymph node biopsy.
Patients with a positive sentinel lymph node biopsy or who have imaging and cytological or histological evidence of axillary nodal metastasis should undergo axillary node clearance or axillary irradiation.
Axillary node clearance is associated with the development of lymphoedema, increased risk of cellulitis and frozen shoulder.

1741
Q

A group of surgeons wish to determine whether patients are receiving adequate deep vein thrombosis prophylaxis following surgery. What is the most appropriate process to determine this?

Departmental review

Peer review

Financial audit

Systems based audit

Standards based audit

A

This type of audit is widely undertaken in most trusts in the UK.

Audit categories

Audits may be used in a variety of clinical settings. These range from standards based audits, which will be familiar to most clinicians, through to systems based audits which focus more on the processes within an organisation.

Types of audit
Financial audit A historically oriented, independent evaluation performed for the purpose of attesting to the fairness, accuracy, and reliability of financial data
Operational audit A future-oriented, systematic, and independent evaluation of organizational activities. Financial data may be used, but the primary sources of evidence are the operational policies and achievements related to organizational objectives. Internal controls and efficiencies may be evaluated during this type of review.
Departmental review A current period analysis of administrative functions, to evaluate the adequacy of controls, safeguarding of assets, efficient use of resources, compliance with related laws, regulations and institutional policy and integrity of financial information.
Standards based audit Comparison of care or passage of care against set and widely agreed standards or outcomes.
Systems based audit Evaluation of processes occurring within an institution.

Systems based audits are an integral part of the process of clinical governance.

1742
Q

Which of the agents listed below is associated with the strongest anti emetic properties?

Sodium thiopentone

Propofol

Etomidate

Ketamine

Sevoflurane

A

Propofol has anti emetic properties which is of considerable advantage in day case anaesthesia.
Anaesthetic agents

The table below summarises some of the more commonly used IV induction agents
Agent Specific features
Propofol
Rapid onset of anaesthesia
Pain on IV injection
Rapidly metabolised with little accumulation of metabolites
Proven anti emetic properties
Moderate myocardial depression
Widely used especially for maintaining sedation on ITU, total IV anaesthesia and for daycase surgery
Sodium thiopentone
Extremely rapid onset of action making it the agent of choice for rapid sequence of induction
Marked myocardial depression may occur
Metabolites build up quickly
Unsuitable for maintenance infusion
Little analgesic effects
Ketamine
May be used for induction of anaesthesia
Has moderate to strong analgesic properties
Produces little myocardial depression making it a suitable agent for anaesthesia in those who are haemodynamically unstable
May induce state of dissociative anaesthesia resulting in nightmares
Etomidate
Has favorable cardiac safety profile with very little haemodynamic instability
No analgesic properties
Unsuitable for maintaining sedation as prolonged (and even brief) use may result in adrenal suppression
Post operative vomiting is common

1743
Q

A 44 year old man receives a large volume transfusion of whole blood. The whole blood is two weeks old. Which of the following best describes its handling of oxygen?

It will have a low affinity for oxygen

Its affinity for oxygen is unchanged

It will more readily release oxygen in metabolically active tissues than fresh blood

The release of oxygen in metabolically active tissues will be the same as fresh blood

It will have an increased affinity for oxygen

A

Stored blood has less 2,3 DPG and therefore has a higher affinity for oxygen, this reduces its ability to release it at metabolising tissues.

Oxygen Transport

Oxygen transport
Almost all oxygen is transported within erythrocytes. It has limited solubility and only 1% is carried as solution. Therefore the amount of oxygen transported will depend upon haemoglobin concentration and its degree of saturation.

Haemoglobin
Globular protein composed of 4 subunits. Haem consists of a protoporphyrin ring surrounding an iron atom in its ferrous state. The iron can form two additional bonds; one with oxygen and the other with a polypeptide chain. There are two alpha and two beta subunits to this polypeptide chain in an adult and together these form globin. Globin cannot bind oxygen but is able to bind to carbon dioxide and hydrogen ions, the beta chains are able to bind to 2,3 diphosphoglycerate. The oxygenation of haemoglobin is a reversible reaction. The molecular shape of haemoglobin is such that binding of one oxygen molecule facilitates the binding of subsequent molecules.

Oxygen dissociation curve
The oxygen dissociation curve describes the relationship between the percentage of saturated haemoglobin and partial pressure of oxygen in the blood. It is not affected by haemoglobin concentration.
Chronic anaemia causes 2, 3 DPG levels to increase, hence shifting the curve to the right

Haldane effect
Shifts to left = for given oxygen tension there is increased saturation of Hb with oxygen i.e. Decreased oxygen delivery to tissues

Bohr effect
Shifts to right = for given oxygen tension there is reduced saturation of Hb with oxygen i.e. Enhanced oxygen delivery to tissues

Shifts to Left = Lower oxygen delivery
HbF, methaemoglobin, carboxyhaemoglobin
low [H+] (alkali)
low pCO2
low 2,3-DPG
low temperature
Shifts to Right = Raised oxygen delivery
raised [H+] (acidic)
raised pCO2
raised 2,3-DPG*
raised temperature

*2,3-diphosphoglycerate

1744
Q

What is the most appropriate management for a 56 year old lady who has shooting pains in her arm following a mastectomy and axillary node clearance?

Carbamazepine

Pregabalin

Oramorph

Diclofenac

Chemical neurectomy

A

Pregabalin is generally the first line agent for neuropathic pain.

Management of pain

World Health Organisation Analgesic Ladder
Initially peripherally acting drugs such as paracetamol or non-steroidal anti-inflammatory drugs (NSAIDs) are given.
If pain control is not achieved, the second part of the ladder is to introduce weak opioid drugs such as codeine or dextropropoxyphene together with appropriate agents to control and minimise side effects.
The final rung of the ladder is to introduce strong opioid drugs such as morphine. Analgesia from peripherally acting drugs may be additive to that from centrally-acting opioids and thus, the two are given together.

The World Federation of Societies of Anaesthesiologists (WFSA) Analgesic Ladder
For management of acute pain
Initially, the pain can be expected to be severe and may need controlling with strong analgesics in combination with local anaesthetic blocks and peripherally acting drugs.
The second rung on the postoperative pain ladder is the restoration of the use of the oral route to deliver analgesia. Strong opioids may no longer be required and adequate analgesia can be obtained by using combinations of peripherally acting agents and weak opioids.
The final step is when the pain can be controlled by peripherally acting agents alone.

Local anaesthetics
Infiltration of a wound with a long-acting local anaesthetic such as Bupivacaine
Analgesia for several hours
Further pain relief can be obtained with repeat injections or by infusions via a thin catheter
Blockade of plexuses or peripheral nerves will provide selective analgesia in those parts of the body supplied by the plexus or nerves
Can either be used to provide anaesthesia for the surgery or specifically for postoperative pain relief
Especially useful where a sympathetic block is needed to improve postoperative blood supply or where central blockade such as spinal or epidural blockade is contraindicated.

Spinal anaesthesia
Provides excellent analgesia for surgery in the lower half of the body and pain relief can last many hours after completion of the operation if long-acting drugs containing vasoconstrictors are used.

  • Side effects of spinal anaesthesia include: hypotension, sensory and motor block, nausea and urinary retention.

Epidural anaesthesia
An indwelling epidural catheter inserted. This can then be used to provide a continuous infusion of analgesic agents. It can provide excellent analgesia. They are still the preferred option following major open abdominal procedures and help prevent post operative respiratory compromise resulting from pain.

  • Disadvantages of epidurals is that they usually confine patients to bed, especially if a motor block is present. In addition an indwelling urinary catheter is required. Which may not only impair mobility but also serve as a conduit for infection. They are contraindicated in coagulopathies.

Transversus Abdominal Plane block (TAP)
In this technique an ultrasound is used to identify the correct muscle plane and local anaesthetic (usually bupivicaine) is injected. The agent diffuses in the plane and blocks many of the spinal nerves. It is an attractive technique as it provides a wide field of blockade but does not require the placement of any indwelling devices. There is no post operative motor impairment. For this reason it is the preferred technique when extensive laparoscopic abdominal procedures are performed. They will then provide analgesia immediately following surgery but as they do not confine the patient to bed, the focus on enhanced recovery can begin sooner.

-The main disadvantage is that their duration of action is limited to the half life of the local anaesthetic agent chosen. In addition some anaesthetists do not have the USS skills required to site the injections.

Patient Controlled Analgesia (PCA)

  • Patients administer their own intravenous analgesia and titrate the dose to their own end-point of pain relief using a small microprocessor - controlled pump. Morphine is the most popular drug used.

Strong Opioids

Severe pain arising from deep or visceral structures requires the use of strong opioids

Morphine
Short half life and poor bioavailability.
Metabolised in the liver and clearance is reduced in patients with liver disease, in the elderly and the debilitated
Side effects include nausea, vomiting, constipation and respiratory depression.
Tolerance may occur with repeated dosage

Pethidine
Synthetic opioid which is structurally different from morphine but which has similar actions. Has 10% potency of morphine.
Short half life and similar bioavailability and clearance to morphine.
Short duration of action and may need to be given hourly.
Pethidine has a toxic metabolite (norpethidine) which is cleared by the kidney, but which accumulates in renal failure or following frequent and prolonged doses and may lead to muscle twitching and convulsions. Extreme caution is advised if pethidine is used over a prolonged period or in patients with renal failure.

Weak opioids
Codeine: markedly less active than morphine, has predictable effects when given orally and is effective against mild to moderate pain.

Non opioid analgesics
- Mild to moderate pain.

Paracetamol
Inhibits prostaglandin synthesis.
Analgesic and antipyretic properties but little anti-inflammatory effect
It is well absorbed orally and is metabolised almost entirely in the liver
Side effects in normal dosage and is widely used for the treatment of minor pain. It causes hepatotoxicity in over dosage by overloading the normal metabolic pathways with the formation of a toxic metabolite.

NSAIDs
Analgesic and anti-inflammatory actions
Inhibition of prostaglandin synthesis by the enzyme Cyclooxygenase which catalyses the conversion of arachidonic acid to the various prostaglandins that are the chief mediators of inflammation. All NSAIDs work in the same way and thus there is no point in giving more than one at a time. .
NSAIDs are, in general, more useful for superficial pain arising from the skin, buccal mucosa, joint surfaces and bone.
Relative contraindications: history of peptic ulceration, gastrointestinal bleeding or bleeding diathesis; operations associated with high blood loss, asthma, moderate to severe renal impairment, dehydration and any history of hypersensitivity to NSAIDs or aspirin.

Neuropathic pain
National Institute of Clinical Excellence (UK) guidelines:
First line: Amitriptyline (Imipramine if cannot tolerate) or pregabalin
Second line: Amitriptyline AND pregabalin
Third line: refer to pain specialist. Give tramadol in the interim (avoid morphine)
If diabetic neuropathic pain: Duloxetine
As of 1 April 2019, pregabalin and gabapentin are Class C controlled substances (under the Misuse of Drugs Act 1971) and scheduled under the Misuse of Drugs Regulations 2001 as Schedule 3. Evaluate patients carefully for a history of drug abuse before prescribing and observe patients for development of signs of abuse and dependence (MHRA, Drug Safety Update April 2019).

References
1. http://guidance.nice.org.uk/CG173/Guidance/pdf/English
2. Lovich-Sapola J, Smith CE, Brandt CP. Post operative pain control. Surg Clin North Am. 2015 Apr;95(2):301-183. Finnerup N et al. Pharmacotherapy for neuropathic pain in adults: a systematic review and meta-analysis. Lancet Neurol. 2015 Feb;14(2):162-73.

1745
Q

A 35 year old man falls and sustains a fracture to the medial third of his clavicle. Which vessel is at greatest risk of injury?

Subclavian vein

Subclavian artery

External carotid artery

Internal carotid artery

Vertebral artery

A

The subclavian vein lies behind subclavius and the medial part of the clavicle. It rests on the first rib, below and in front of the third part of the subclavian artery, and then on scalenus anterior which separates it from the second part of the artery (posteriorly).

Clavicle

The clavicle extends from the sternum to the acromion and helps prevent the shoulder falling forwards and downwards.
The inferior surface is irregular and strongly marked by ligaments at each end. Laterally, lies the trapezoid line and this runs anterolaterally. Posteriorly, lies the conoid tubercle. These give attachment to the conoid and trapezoid parts of the coracoclavicular ligament. The medial part of the inferior surface has an irregular surface which marks the surface attachment of the costoclavicular ligament. The intermediate portion is marked by a groove for the subclavius muscle.
Medially, the superior part of the bone has a raised surface which gives attachment to the clavicular head of sternocleidomastoid. Sternohyoid gains attachment to the posterior surface.
Laterally there is an oval articular facet for the acromion and a disk lies between the clavicle and acromion. The capsule of the joint is attached to the ridge on the margin of the facet.

1746
Q

Which of the following does not decrease the functional residual capacity?

Obesity

Pulmonary fibrosis

Muscle relaxants

Laparoscopic surgery

Upright position

A

Increased FRC:
Erect position
Emphysema
Asthma

Decreased FRC:
Pulmonary fibrosis
Laparoscopic surgery
Obesity
Abdominal swelling
Muscle relaxants
When the patient is upright the diaphragm and abdominal organs put less pressure on the lung bases, allowing for an increase in the functional residual capacity (FRC). Other causes of increased FRC include:
Emphysema
Asthma

In addition to those listed above, causes of reduced FRC include:
Abdominal swelling
Pulmonary oedema
Reduced muscle tone of the diaphragm
Age
Definitions

Tidal volume (TV)
Is the volume of air inspired and expired during each ventilatory cycle at rest.
It is normally 500mls in males and 340mls in females.
Inspiratory reserve volume (IRV)
Is the maximum volume of air that can be forcibly inhaled following a normal inspiration. 3000mls.
Expiratory reserve volume (ERV)
Is the maximum volume of air that can be forcibly exhaled following a normal expiration. 1000mls.
Residual volume (RV)
Is that volume of air remaining in the lungs after a maximal expiration.
RV = FRC - ERV. 1500mls.
Functional residual capacity (FRC)
Is the volume of air remaining in the lungs at the end of a normal expiration.
FRC = RV + ERV. 2500mls.
Vital capacity (VC)
Is the maximal volume of air that can be forcibly exhaled after a maximal inspiration.
VC = TV + IRV + ERV. 4500mls in males, 3500mls in females.
Total lung capacity (TLC)
Is the volume of air in the lungs at the end of a maximal inspiration.
TLC = FRC + TV + IRV = VC + RV. 5500-6000mls.
Forced vital capacity (FVC)
The volume of air that can be maximally forcefully exhaled.

1747
Q

Which one of the following is least associated with the development of colorectal cancer in patients with ulcerative colitis?

Unremitting disease

Disease duration > 10 years

Onset before 15 years old

Poor compliance to treatment

Disease confined to the rectum

A

Ulcerative colitis and colorectal cancer

Overview
risk of colorectal cancer is 10-20 times that of general population
the increased risk is mainly related to chronic inflammation
worse prognosis than patients without ulcerative colitis (partly due to delayed diagnosis)
lesions may be multifocal

Factors increasing risk of cancer
disease duration > 10 years
patients with pancolitis
onset before 15 years old
unremitting disease
poor compliance to treatment

1748
Q

A 63 year old lady is diagnosed as having an endometrial carcinoma arising from the uterine body. To which nodal region will the tumour initially metastasise?

Para aortic nodes

Iliac lymph nodes

Inguinal nodes

Pre sacral nodes

Mesorectal lymph nodes

A

Tumours of the uterine body will tend to spread to the iliac nodes initially. Tumour expansion crossing different nodal margins this is of considerable clinical significance, if nodal clearance is performed during a Wertheims type hysterectomy.

Lymphatic drainage of the ovaries, uterus and cervix

The ovaries drain to the para-aortic lymphatics via the gonadal vessels.
The uterine fundus has a lymphatic drainage that runs with the ovarian vessels and may thus drain to the para-aortic nodes. Some drainage may also pass along the round ligament to the inguinal nodes.
The body of the uterus drains through lymphatics contained within the broad ligament to the iliac lymph nodes.
The cervix drains into three potential nodal stations; laterally through the broad ligament to the external iliac nodes, along the lymphatics of the uterosacral fold to the presacral nodes and posterolaterally along lymphatics lying alongside the uterine vessels to the internal iliac nodes.

1749
Q

A 56 year old lady is admitted with colicky abdominal pain. A plain x-ray is performed. Which of the following should not show fluid levels on a plain abdominal film?

Stomach

Jejunum

Ileum

Caecum

Descending colon

A

Fluid levels in the distal colon are nearly always pathological. In general contents of the left colon transit quickly and are seldom held in situ for long periods, the content is also more solid.

Abdominal radiology

Plain abdominal x-rays are often used as a first line investigation in patients with acute abdominal pain. A plain abdominal film may demonstrate free air, evidence of bowel obstruction and possibly other causes of pain (e.g. renal or gallbladder stones).
Investigation of potential visceral perforation is usually best performed by obtaining an erect chest x-ray, as this is a more sensitive investigation for suspected visceral perforation than recumbent films.

Features which are usually abnormal
Large amounts of free air (colonic perforation), smaller volumes seen with more proximal perforations.
A positive Riglers sign (gas on both sides of the bowel wall).
Caecal diameter of >8cm
Fluid levels in the colon
Ground glass appearance to film (usually due to large amounts of free fluid).
Sentinel loop in patients with inflammation of other organs (e.g. pancreatitis).

Features which should be expected/ or occur without pathology
In Chiladitis sign, a loop of bowel may be interposed between the liver and diaphragm, giving the mistaken impression that free air is present.
Following ERCP (and sphincterotomy) air may be identified in the biliary tree.
Free intra abdominal air following laparoscopy / laparotomy, although usually dissipates after 48-72 hours.

1750
Q

The oxygen-haemoglobin dissociation curve is shifted to the right in which of the following scenarios?

Hypothermia

Respiratory alkalosis

Low altitude

Decreased 2,3-DPG in transfused red cells

Chronic iron deficiency anaemia

A

Mnemonic to remember causes of right shift of the oxygen dissociation curve:

CADET face RIGHT

C O2
A cidosis
2,3-DPG
E xercise
T emperature
The curve is shifted to the right when there is an increased oxygen requirement by the tissue. This includes:
Increased temperature
Acidosis
Increased DPG:
DPG is found in erythrocytes and is increased during glycolysis. It binds to the Hb molecule, thereby releasing oxygen to tissues. DPG is increased in conditions associated with poor oxygen delivery to tissues, such as anaemia and high altitude.

Oxygen Transport

Oxygen transport
Almost all oxygen is transported within erythrocytes. It has limited solubility and only 1% is carried as solution. Therefore the amount of oxygen transported will depend upon haemoglobin concentration and its degree of saturation.

Haemoglobin
Globular protein composed of 4 subunits. Haem consists of a protoporphyrin ring surrounding an iron atom in its ferrous state. The iron can form two additional bonds; one with oxygen and the other with a polypeptide chain. There are two alpha and two beta subunits to this polypeptide chain in an adult and together these form globin. Globin cannot bind oxygen but is able to bind to carbon dioxide and hydrogen ions, the beta chains are able to bind to 2,3 diphosphoglycerate. The oxygenation of haemoglobin is a reversible reaction. The molecular shape of haemoglobin is such that binding of one oxygen molecule facilitates the binding of subsequent molecules.

Oxygen dissociation curve
The oxygen dissociation curve describes the relationship between the percentage of saturated haemoglobin and partial pressure of oxygen in the blood. It is not affected by haemoglobin concentration.
Chronic anaemia causes 2, 3 DPG levels to increase, hence shifting the curve to the right

Haldane effect
Shifts to left = for given oxygen tension there is increased saturation of Hb with oxygen i.e. Decreased oxygen delivery to tissues

Bohr effect
Shifts to right = for given oxygen tension there is reduced saturation of Hb with oxygen i.e. Enhanced oxygen delivery to tissues

Shifts to Left = Lower oxygen delivery
HbF, methaemoglobin, carboxyhaemoglobin
low [H+] (alkali)
low pCO2
low 2,3-DPG
low temperature
Shifts to Right = Raised oxygen delivery
raised [H+] (acidic)
raised pCO2
raised 2,3-DPG*
raised temperature

1751
Q

A 72 year old man presents with symptoms and signs of benign prostatic hyperplasia. Which of the following structures is most likely to be enlarged?

Posterior lobe of the prostate

Median lobe of the prostate

Right lateral lobe of the prostate

Left lateral lobe of the prostate

Anterior lobe of the prostate

A

Carcinoma of the prostate typically occurs in the posterior lobe. The median lobe is usually enlarged in BPH. The anterior lobe has little in the way of glandular tissue and is seldom enlarged.

Benign Prostatic Hyperplasia

Benign prostatic hyperplasia occurs via an increase in the epithelial and stromal cell numbers in the peri-urethral zone of the prostate. BPH is very common and 90% of men aged over 80 will have at least microscopic evidence of benign prostatic hyperplasia. The causes of BPH are still not well understood, but the importance of androgens remains appreciated even if the exact role by which they induce BPH is elusive.

Presentation
The vast majority of men will present with lower urinary tract symptoms. These will typically be:

Poor flow
Nocturia
Hesitancy
Incomplete and double voiding
Terminal dribbling
Urgency
Incontinence

Investigation
Digital rectal examination to assess prostatic size and morphology.
Urine dipstick for infections and haematuria.
Uroflowmetry (a flow rate of >15ml/second helps to exclude BOO)
Bladder pressure studies may help identify detrusor failure and whilst may not form part of first line investigations should be included in those with atypical symptoms and prior to redo surgery.
Bladder scanning to demonstrate residual volumes. USS if high pressure chronic retention.

Management
Lifestyle changes such as stopping smoking and altering fluid intake may help those with mild symptoms.
Medical therapy includes alpha blockers and 5 α reductase inhibitors. The former work quickly on receptor zones located at the bladder neck. Cardiovascular side effects are well documented. The latter work on testosterone metabolising enzymes. Although they have a slower onset of action, the 5 α reductase inhibitors may prevent acute urinary retention.
Surgical therapy includes transurethral resection of the prostate and is the treatment of choice in those with severe symptoms and those who fail to respond to medical therapy. More tailored bladder neck incision procedures may be considered in those with small prostates. Retrograde ejaculation may occur following surgery. The change in the type of irrigation solutions used has helped to minimise the TURP syndrome of electrolyte disturbances.

1752
Q

A 22 year old man presents with a peri anal abscess, which is managed by incision and drainage. The perineal wound measures 3cm by 3cm. Which of the following is best management option?

Primary closure with interrupted mattress sutures

Delayed primary closure with interrupted mattress sutures

Allow the wound to heal by secondary intention

Insert a seton through the cavity into the rectum to allow a mature fistula track to develop

Perform a V-Y flap 2 weeks later

A

Peri anal abscess are typically managed by secondary intention healing. Any attempt at early closure is at best futile and at worst dangerous. Insertion of a seton may be considered by an experienced colorectal surgeon, and only if the tract is clearly identifiable with minimal probing. There is seldom a need for flaps, ongoing discharge usually indicates a fistula (managed separately).

Wound healing

Surgical wounds are either incisional or excisional and either clean, clean contaminated or dirty. Although the stages of wound healing are broadly similar their contributions will vary according to the wound type.

The main stages of wound healing include:

Haemostasis
Minutes to hours following injury
Vasospasm in adjacent vessels, platelet plug formation and generation of fibrin rich clot.

Inflammation
Typically days 1-5
Neutrophils migrate into wound (function impaired in diabetes).
Growth factors released, including basic fibroblast growth factor and vascular endothelial growth factor.
Fibroblasts replicate within the adjacent matrix and migrate into wound.
Macrophages and fibroblasts couple matrix regeneration and clot substitution.

Regeneration
Typically days 7 to 56
Platelet derived growth factor and transformation growth factors stimulate fibroblasts and epithelial cells.
Fibroblasts produce a collagen network.
Angiogenesis occurs and wound resembles granulation tissue.

Remodeling
From 6 weeks to 1 year
Longest phase of the healing process and may last up to one year (or longer).
During this phase fibroblasts become differentiated (myofibroblasts) and these facilitate wound contraction.
Collagen fibres are remodeled.
Microvessels regress leaving a pale scar.

The above description represents an idealised scenario. A number of diseases may distort this process. Neovascularisation is an important early process. Endothelial cells may proliferate in the wound bed and recanalise to form a vessel. Vascular disease, shock and sepsis can all compromise microvascular flow and impair healing.

Conditions such as jaundice will impair fibroblast synthetic function and immunity with a detrimental effect in most parts of the healing process.

Problems with scars:

Hypertrophic scars
Excessive amounts of collagen within a scar. Nodules may be present histologically containing randomly arranged fibrils within and parallel fibres on the surface. The tissue itself is confined to the extent of the wound itself and is usually the result of a full thickness dermal injury. They may go on to develop contractures.

Image of hypertrophic scarring. Note that it remains confined to the boundaries of the original wound: Keloid scars
Excessive amounts of collagen within a scar. Typically a keloid scar will pass beyond the boundaries of the original injury. They do not contain nodules and may occur following even trivial injury. They do not regress over time and may recur following removal.

Image of a keloid scar. Note the extension beyond the boundaries of the original incision: Drugs which impair wound healing:
Non steroidal anti inflammatory drugs
Steroids
Immunosupressive agents
Anti neoplastic drugs

Closure
Delayed primary closure is the anatomically precise closure that is delayed for a few days but before granulation tissue becomes macroscopically evident.

Secondary closure refers to either spontaneous closure or to surgical closure after granulation tissue has formed.

1753
Q

Which of the following blood products can be administered to a non ABO matched recipient?

Whole blood

Platelets

Packed red cells

Stem cells

Irradiated whole blood

A

In the UK, platelets either come from pooling of the platelet component from four units of whole donated blood, called random donor platelets, or by plasmapharesis from a single donor. The platelets are suspended in 200-300 ml of plasma and may be stored for up to 4 days in the transfusion laboratory where they are continually agitated at 22oC to preserve function. One adult platelet pool raises the normal platelet count by 30,000 to 60,000 platelets litre. ABO identical or compatible platelets are preferred but not necessary in adults; but rhesus compatibility is required in recipients who are children and women of childbearing age to prevent haemolytic disease of the newborn.

Blood products - cross matching

Whole blood fractions

Fraction Key points
Packed red cells Used for transfusion in chronic anaemia and cases where infusion of large volumes of fluid may result in cardiovascular compromise. Product obtained by centrifugation of whole blood.
Platelet rich plasma Usually administered to patients who are thrombocytopaenic and are bleeding or require surgery. It is obtained by low speed centrifugation.
Platelet concentrate Prepared by high speed centrifugation and administered to patients with thrombocytopaenia.
Fresh frozen plasma
Prepared from single units of blood.
Contains clotting factors, albumin and immunoglobulin.
Unit is usually 200 to 250ml.
Usually used in correcting clotting deficiencies in patients with hepatic synthetic failure who are due to undergo surgery.
Usual dose is 12-15ml/Kg-1.
It should not be used as first line therapy for hypovolaemia.
Cryoprecipitate
Formed from supernatant of FFP.
Rich source of Factor VIII and fibrinogen.
Allows large concentration of factor VIII to be administered in small volume.
SAG-Mannitol Blood Removal of all plasma from a blood unit and substitution with:
Sodium chloride
Adenine
Anhydrous glucose
Mannitol
Up to 4 units of SAG M Blood may be administered. Thereafter whole blood is preferred. After 8 units, clotting factors and platelets should be considered.

Cross matching
Must be cross matched Can be ABO incompatible in adults
Packed red cells Platelets
Whole blood FFP

1754
Q

A 28 year old man undergoes a laparotomy for perforated duodenal ulcer and broad spectrum antibiotics are administered. Post operatively he has hearing impairment. Which of the following agents is the most likely underlying culprit?

Gentamicin

Ciprofloxacin

Metronidazole

Ampicillin

Co-trimoxazole

A

Ototoxicity is a recognised adverse reaction with the aminoglycoside antibiotics.

Antibiotics: mechanism of action

The lists below summarise the site of action of the commonly used antibiotics

Inhibit cell wall formation
penicillins
cephalosporins

Inhibit protein synthesis
aminoglycosides (cause misreading of mRNA)
chloramphenicol
macrolides (e.g. erythromycin)
tetracyclines
fusidic acid

Inhibit DNA synthesis
quinolones (e.g. ciprofloxacin)
metronidazole
sulphonamides
trimethoprim

Inhibit RNA synthesis
rifampicin

1755
Q

A 43 year old lady with repeated episodes of abdominal pain is admitted with small bowel obstruction. A laparotomy is performed and at surgery a gallstone ileus is identified. What is the most appropriate course of action?

Remove the gallstone via a proximally sited terminal ileal enterotomy and decompress the small bowel. Leave the gallbladder in situ.

Remove the gallstone via a proximally sited terminal ileal enterotomy and decompress the small bowel. Remove the gallbladder.

Remove the gallstone via a proximally sited terminal ileal enterotomy and decompress the small bowel. Perform a choledochoduodenostomy.

Remove the gallstone from an enterotomy at the site of the obstruction and leave the gallbladder in situ.

Remove the gallstone from an enterotomy at the site of the obstruction and remove the gallbladder.

A

Gallstone ileus occurs as a result of the fistula developing between the gallbladder and the duodenum. These tend to become impacted somewhat proximal to the ileocaecal valve and cause small bowel obstruction. The correct management is to remove the gallstone from an enterotomy proximal to the site of stone impaction. The bowel at the site of impaction itself may not heal well and an enterotomy performed at this site may well result in the need for a resection. The standard surgical teaching is that under almost all circumstances the gallbladder should be left in situ, as the anatomy in this area is often hostile and unpredictable. Disconnecting it from the duodenum leaves a large defect that is difficult to close.

Gallstones

Up to 24% of women and 12% of men may have gallstones. Of these up to 30% may develop local infection and cholecystitis. In patients subjected to surgery 12% will have stones contained within the common bile duct. The majority of gallstones are of a mixed composition (50%) with pure cholesterol stones accounting for 20% of cases.
The aetiology of CBD stones differs in the world, in the West most CBD stones are the result of migration. In the East a far higher proportion arise in the CBD de novo.
The classical symptoms are of colicky right upper quadrant pain that occurs post prandially. The symptoms are usually worst following a fatty meal when cholecystokinin levels are highest and gallbladder contraction is maximal.

Investigation
In almost all suspected cases the standard diagnostic work up consists of abdominal ultrasound and liver function tests. Of patients who have stones within the bile duct, 60% will have at least one abnormal result on LFT’s. Ultrasound is an important test, but is operator dependent and therefore may occasionally need to be repeated if a negative result is at odds with the clinical picture. Where stones are suspected in the bile duct, the options lie between magnetic resonance cholangiography and intraoperative imaging. The choice between these two options is determined by the skills and experience of the surgeon. The advantages of intra operative imaging are less useful in making therapeutic decisions if the operator is unhappy about proceeding the bile duct exploration, and in such circumstances pre operative MRCP is probably a better option.

Specific gallstone and gallbladder related disease
Disease Features Management
Biliary colic Colicky abdominal pain, worse post prandially, worse after fatty foods If imaging shows gallstones and history compatible then laparoscopic cholecystectomy
Acute cholecystitis
Right upper quadrant pain
Fever
Murphys sign on examination
Occasionally mildly deranged LFT’s (especially if Mirizzi syndrome)
Imaging (USS) and cholecystectomy (ideally within 48 hours of presentation) (2)
Gallbladder abscess
Usually prodromal illness and right upper quadrant pain
Swinging pyrexia
Patient may be systemically unwell
Generalised peritonism not present
Imaging with USS +/- CT Scanning
Ideally surgery, sub total cholecystectomy may be needed if Calots triangle is hostile
In unfit patients percutaneous drainage may be considered
Cholangitis
Patient severely septic and unwell
Jaundice
Right upper quadrant pain
Fluid resuscitation
Broad spectrum intravenous antibiotics
Correct any coagulopathy
Early ERCP
Gallstone ileus
Patients may have a history of previous cholecystitis and known gallstones
Small bowel obstruction (may be intermittent)
Laparotomy and removal of gallstone from small bowel, the enterotomy must be made proximal to the site of obstruction and not at the site of obstruction. The fistula between the gallbladder and duodenum should not be interfered with.
Acalculous cholecystitis
Patients with inter current illness (e.g. diabetes, organ failure)
Patient of systemically unwell
Gallbladder inflammation in absence of stones
High fever
If patient fit then cholecystectomy, if unfit then percutaneous cholecystostomy

Treatment
Patients with asymptomatic gallstones rarely develop symptoms related to them (less than 2% per year) and may therefore be managed expectantly. In almost all cases of symptomatic gallstones the treatment of choice is cholecystectomy performed via the laparoscopic route. In the very frail patient there is sometimes a role for selective use of ultrasound guided cholecystostomy.

During the course of the procedure some surgeons will routinely perform either intra operative cholangiography or laparoscopic USS to either confirm anatomy or to exclude CBD stones. The latter may be more easily achieved by use of laparoscopic ultrasound. If stones are found then the options lie between early ERCP in the day or so following surgery or immediate surgical exploration of the bile duct. When performed via the trans cystic route this adds little in the way of morbidity and certainly results in faster recovery. Where transcystic exploration fails the alternative strategy is that of formal choledochotomy. The exploration of a small duct is challenging and ducts of less than 8mm should not be explored. Small stones that measure less than 5mm may be safely left and most will pass spontaneously.

Risks of ERCP(1)
Bleeding 0.9% (rises to 1.5% if sphincterotomy performed)
Duodenal perforation 0.4%
Cholangitis 1.1%
Pancreatitis 1.5%

References
1. Williams E et al. Guidelines on the management of common bile duct stones (CBDS)Gut 2008;57:10041021

  1. Gurusamy KS, Samraj K. Early versus delayed laparoscopic cholecystectomy for acute cholecystitis. Cochrane Database Syst Rev. 2006 Oct 18;(4):CD005440.
  2. Gurusamy K and Davidson B. Gallstones. BMJ 2014 (348):27-30.
1756
Q

A surgeon is undertaking a cholecystectomy for cholecystitis secondary to gallstones. The operation is difficult and views of Calots triangle are poor. Unfortunately the distal aspect of the bile duct is transected. What is the most appropriate course of action?

Perform a primary sutured repair

Repair the defect over a T tube

Place drains and close the wounds and discuss with a hepatobiliary surgeon

Repair the defect with a Roux en Y choledochojejunostomy

Repair the defect with a choledochoduodenostomy

A

Bile duct injuries have worse outcomes if the repair is performed by the surgeon who created the injury and also if the surgeon is not an HPB surgeon.

Biliary disease

Diagnosis Typical features Pathogenesis
Gallstones Typically history of biliary colic or episodes of chlolecystitis. Obstructive type history and test results. Usually small calibre gallstones which can pass through the cystic duct. In Mirizzi syndrome the stone may compress the bile duct directly- one of the rare times that cholecystitis may present with jaundice
Cholangitis Usually obstructive and will have Charcot’s triad of symptoms (pain, fever, jaundice) Ascending infection of the bile ducts usually by E. coli and by definition occurring in a pool of stagnant bile.
Pancreatic cancer Typically painless jaundice with palpable gallbladder (Courvoisier’s Law) Direct occlusion of distal bile duct or pancreatic duct by tumour. Sometimes nodal disease at the portal hepatis may be the culprit in which case the bile duct may be of normal calibre.
TPN (total parenteral nutrition) associated jaundice Usually follows long term use and is usually painless with non obstructive features Often due to hepatic dysfunction and fatty liver which may occur with long term TPN usage.
Bile duct injury Depending upon the type of injury may be of sudden or gradual onset and is usually of obstructive type Often due to a difficult laparoscopic cholecystectomy when anatomy in Calots triangle is not appreciated. In the worst scenario the bile duct is excised and jaundice develops rapidly post operatively. More insidious is that of bile duct stenosis which may be caused by clips or diathermy injury.
Cholangiocarcinoma Gradual onset obstructive pattern Direct occlusion by disease and also extrinsic compression by nodal disease at the porta hepatis.
Septic surgical patient Usually hepatic features Combination of impaired biliary excretion and drugs such as ciprofloxacin which may cause cholestasis.
Metastatic disease Mixed hepatic and post hepatic Combination of liver synthetic failure (late) and extrinsic compression by nodal disease and anatomical compression of intra hepatic structures (earlier)

1757
Q

A 63 year old man is reviewed in the vascular clinic as he is noted to have a pulsatile swelling between the greater trochanter and pubic tubercle which vessel is more likely to be affected?

Internal iliac artery

Inferior epigastric artery

Common iliac artery

External iliac artery

Femoral artery

A

The level specified would lie within the femoral triangle and therefore represents the femoral artery.

Iliac arteries

The common iliac arteries are terminal branches of the abdominal aorta. They begin opposite the middle body of the fourth lumbar vertebra. The superior hypogastric plexus lies anterior to the proximal end and more distally, it is crossed by the ureter. The last two lumbar vertebral bodies, and intervening disc, the sympathetic trunk and the medial part of psoas major lie behind each artery. They are separated from the right artery by the beginning of the inferior vena cava. The only branches of the common iliac arteries are the internal and external iliac arteries.

Internal iliac artery
Anteromedially, each internal iliac artery is covered by peritoneum, beneath which the ureter descends on the anterior surface of each artery. On the left, the vessel is overlapped by the sigmoid colon and, on the right, the terminal ileum overlies it. Posteriorly, the internal iliac vein and commencement of the common iliac vein are closely related to the artery. The posterior division of the internal iliac artery has parietal branches and the anterior division has both parietal and visceral branches.

Branches of the posterior division of the internal iliac
Lateral sacral, superior gluteal, iliolumbar artery.

Branches of the anterior division of the internal iliac artery
Obturator artery, internal pudendal artery, inferior gluteal, inferior vesical artery, artery the ductus deferens, middle rectal, vaginal artery, uterine artery.

External iliac artery
The external iliac artery extends from a point opposite the sacroiliac joint at the level of the lumbosacral disc to the midinguinal point behind the inguinal ligament, midway between the anterior superior iliac spine and the pubic symphysis.

1758
Q

A 56 year old man presents with epigastric discomfort and episodes of migratory thrombophlebitis. On examination he is mildly jaundiced. A CT scan shows lymphadenopathy at the porta hepatis and a mass in the pancreatic head. Which of the following is the most likely underlying diagnosis?

Squamous cell carcinoma of the pancreas

Adenocarcinoma of the pancreas

Insulinoma

Glucagonoma

Gastrinoma

A

Adenocarcinoma of the pancreas is the most likely diagnosis and migratory thrombophlebitis is associated with the condition. Squamous cells carcinoma is extremely uncommon in the pancreas. Gastrinoma are extremely rare and thus not the most likely diagnosis.

Pancreatic cancer

  • Adenocarcinoma
    Risk factors: Smoking, diabetes, adenoma, familial adenomatous polyposis
    Mainly occur in the head of the pancreas (70%)
    Spread locally and metastasizes to the liver
    Carcinoma of the pancreas should be differentiated from other periampullary tumours with better prognosis

Clinical features
Weight loss
Painless jaundice
Epigastric discomfort (pain usually due to invasion of the coeliac plexus is a late feature)
Pancreatitis
Trousseau’s sign: migratory superficial thrombophlebitis

Investigations
USS: May miss small lesions
CT Scanning (pancreatic protocol). If unresectable on CT then no further staging needed
PET/CT for those with operable disease on CT alone
ERCP/ MRI for bile duct assessment
Staging laparoscopy to exclude peritoneal disease

Management
Head of pancreas: Whipple’s resection (SE dumping and ulcers). Newer techniques include pylorus preservation and SMA/ SMV resection
Carcinoma body and tail: poor prognosis, distal pancreatectomy, if operable
Usually adjuvent chemotherapy for resectable disease
ERCP and stent for jaundice and palliation
Surgical bypass may be needed for duodenal obstruction

1759
Q

A 22 year old man is kicked in the head during a rugby match. He is temporarily concussed, but then regains consciousness. Half an hour later he develops slurred speech, ataxia and loses consciousnesses. On arrival in hospital he is intubated and ventilated. A CT Scan is performed which shows an extradural haematoma. What is the most likely cause?

Basilar artery laceration

Middle meningeal artery laceration

Laceration of the sigmoid sinus

Laceration of the anterior cerebral artery

Laceration of the middle cerebral artery

A

The most likely vessel from those in the list to cause an acute extra dural haemorrhage is the middle meningeal artery. The anterior and middle cerebral arteries may cause acute sub dural haemorrhage. Acute sub dural haemorrhages usually take slightly longer to evolve than acute extra dural haemorrhages.

Middle meningeal artery

Middle meningeal artery is typically the third branch of the first part of the maxillary artery, one of the two terminal branches of the external carotid artery. After branching off the maxillary artery in the infratemporal fossa, it runs through the foramen spinosum to supply the dura mater (the outermost meninges) .
The middle meningeal artery is the largest of the three (paired) arteries which supply the meninges, the others being the anterior meningeal artery and the posterior meningeal artery.
The middle meningeal artery runs beneath the pterion. It is vulnerable to injury at this point, where the skull is thin. Rupture of the artery may give rise to an extra dural hematoma.
In the dry cranium, the middle meningeal, which runs within the dura mater surrounding the brain, makes a deep indention in the calvarium.
The middle meningeal artery is intimately associated with the auriculotemporal nerve which wraps around the artery making the two easily identifiable in the dissection of human cadavers and also easily damaged in surgery.

1760
Q

A 73 year old lady presents with constipation and no organic disease is identified on investigation. Which of the following types of laxatives works by direct bowel stimulation?

Magnesium sulphate

Lactulose

Potassium sodium tartrate

Methylcellulose

Senna

A

Senna contains glycosides. It passes unchanged into the colon where bacteria hydrolyse the glycosidic bond, releasing the anthracene derivatives. These stimulate the myenteric plexus.
Laxatives

Bulk forming laxatives
Bran
Psyllium
Methylcellulose
Osmotic laxatives
Magnesium sulphate
Magnesium citrate
Sodium phosphate
Sodium sulphate
Potassium sodium tatrate
Polyethylene glycol
Docusate
Stimulant laxatives
Bisacodyl
Sodium picosulphate
Senna
Ricinoleic acid

1761
Q

A 23 year old man is undergoing an inguinal hernia repair. The surgeons mobilise the spermatic cord and place it in a hernia ring. A small slender nerve is identified superior to the cord. Which nerve is it most likely to be?

Iliohypogastric nerve

Pudendal nerve

Femoral branch of the genitofemoral nerve

Ilioinguinal nerve

Obturator nerve

A

The ilioinguinal nerve passes through the inguinal canal and is the nerve most commonly identified during hernia surgery. The genitofemoral nerve splits into two branches, the genital branch passes through the inguinal canal within the cord structures. The femoral branch of the genitofemoral nerve enters the thigh posterior to the inguinal ligament, lateral to the femoral artery. The iliohypogastric nerve pierces the external oblique aponeurosis above the superficial inguinal ring.

Ilioinguinal nerve

Arises from the first lumbar ventral ramus with the iliohypogastric nerve. It passes inferolaterally through the substance of psoas major and over the anterior surface of quadratus lumborum. It pierces the internal oblique muscle and passes deep to the aponeurosis of the external oblique muscle. It enters the inguinal canal and then passes through the superficial inguinal ring to reach the skin.

Branches
To supply those muscles of the abdominal wall through which it passes.
Skin and fascia over the pubic symphysis, superomedial part of the femoral triangle, surface of the scrotum, root and dorsum of penis or labium majus in females.

1762
Q

Which of the following hormonal agents will increase secretions of water and electrolytes in pancreatic juice?

Secretin

Aldosterone

Somatostatin

Cholecystokinin

Adrenaline

A

Secretin causes secretion of water and electrolytes
Cholecystokinin causes enzyme secretion
While secretin will typically increase electrolyte and water volume of secretions, the enzyme content is increased by cholecystokinin. Somatostatin will decrease the volume of secretions. Aldosterone will tend to conserve electrolytes.

Pancreas exocrine physiology

Composition of pancreatic secretions
Pancreatic secretions are usually 1000-1500ml per 24 hours and have a pH of 8.
Secretion Source Substances secreted
Enzymic Acinar cells Trypsinogen
Procarboxylase
Amylase
Elastase
Aqueous Ductal and Centroacinar cells Sodium
Bicarbonate
Water
Potassium
Chloride
NB: Sodium and potassium reflect their plasma levels; chloride and bicarbonate vary with flow rate

Regulation
The cephalic and gastric phases (neuronal and physical) are less important in regulating the pancreatic secretions. The effect of digested material in the small bowel stimulates CCK release and ACh which stimulate acinar and ductal cells. Of these CCK is the most potent stimulus. In the case of the ductal cells these are potently stimulated by secretin which is released by the S cells of the duodenum. This results in an increase in bicarbonate.

Enzyme activation
Trypsinogen is converted via enterokinase to active trypsin in the duodenum. Trypsin then activates the other inactive enzymes

1763
Q

A 32 year old rugby player is hit hard on the shoulder during a rough tackle. Clinically, his arm is hanging loose on the side. It is pronated and medially rotated. What structure is most likely to have been compromised?

Brachial trunks C5-6

Brachial trunks C6-7

Brachial trunks C8 - T1

Anterior interosseous nerve

Posterior interosseous nerve

A

The patient has an Erb’s palsy involving brachial trunks C5-6.
Brachial plexus

The brachial plexus extends from the neck to the axilla. It is formed by the ventral rami of the fifth to the eighth cervical nerves with the ascending part of the first thoracic nerve.

Location of the plexus
The ventral rami which form the plexus enter the lower part of the posterior triangle of the neck in series with the ventral rami of the cervical plexus. The second part of the subclavian artery lies immediately anterior to the lower two rami. The upper three rami intermingle and pass inferolaterally towards the axilla and subclavian artery. They are enclosed within an extension of the prevertebral fascia. In the neck the plexus lies deep to platysma, the supraclavicular nerves, inferior belly of omohyoid and the transverse cervical artery. It then passes deep to the clavicle and the suprascapular vessels, to enter the axilla, and thence surround the second part of the axillary artery

Composition of the plexus
Ventral rami, the roots of the plexus, lie between scalenus medius and anterior.

As they enter the posterior triangle, the upper two (C5,6) and lower two (C8, T1) roots of the plexus unite to form the upper and lower trunks of the plexus respectively. Meanwhile, C7 continues as the middle trunk. The lower trunk may groove the superior surface of the first rib posterior to the subclavian artery, and the root from the first ventral ramus is always in contact with it.

Each trunk divides into ventral and dorsal divisions which are destined to supply the anterior (flexor) and posterior (extensor) parts of the upper limb.

The cords of the plexus are formed in the axilla. The dorsal divisions unite to form the posterior cord (C5-8). The ventral divisions of the upper and middle trunks unite to form the lateral cord (C5-7), while the ventral divisions of the lower trunk continues as the medial cord (C8-T1). The cords are named according to their relationship to the axillary artery. Each cord terminates by dividing into two main branches at the beginning of the third part of the artery.

Sympathetic communications
The fifth and sixth cervical ventral rami receive grey rami communicantes from the middle cervical ganglion, while the two or more grey rami communicantes pass from the inferior cervical ganglion to the seventh and eighth cervical ventral rami. The first thoracic ventral ramus receives its grey ramus from the cervicothoracic ganglion. Its for this reason that inferior plexus injury can be complicated by a Horners syndrome.

Summary
Origin Anterior rami of C5 to T1
Sections of the plexus
Roots, trunks, divisions, cords, branches
Mnemonic:Real Teenagers Drink Cold Beer
Roots
Located in the posterior triangle
Pass between scalenus anterior and medius
Trunks
Located posterior to middle third of clavicle
Upper and middle trunks related superiorly to the subclavian artery
Lower trunk passes over 1st rib posterior to the subclavian artery
Divisions Apex of axilla
Cords Related to axillary artery

1764
Q

Which of the following is not a feature of a Charcot foot?

Bounding foot pulses in the early phases

Often occurs in the complete absence of trauma

Erythema of the foot in the early phase

Autonomic neuropathy

Peripheral neuropathy

A

Do not confuse the early phase of Charcot foot with cellulitis
Trauma (even if only minor) is a prerequisite. Patients cannot usually recall the traumatic event. The associated neuropathy means that patients continue to walk on the affected foot with subsequent deformity developing over time.

Peripheral vascular disease

Indications for surgery to revascularise the lower limb
Intermittent claudication
Critical ischaemia
Ulceration
Gangrene

Intermittent claudication that is not disabling may provide a relative indication, whilst the other complaints are often absolute indications depending upon the frailty of the patient.

Assessment
Clinical examination
Ankle brachial pressure index measurement
Duplex arterial ultrasound
Angiography (standard, CT or MRI): usually performed only if intervention being considered.

Angioplasty
In order for angioplasty to be undertaken successfully the artery has to be accessible. The lesion relatively short and reasonable distal vessel runoff. Longer lesions may be amenable to sub-intimal angioplasty.

Surgery
Surgery will be undertaken where attempts at angioplasty have either failed or are unsuitable. Bypass essentially involves bypassing the affected arterial segment by utilising a graft to run from above the disease to below the disease. As with angioplasty good runoff improves the outcome.

Some key concepts with bypass surgery

Superficial femoral artery occlusion to the above knee popliteal
In the ideal scenario, vein (either in situ or reversed LSV) would be used as a conduit. However, prosthetic material has reasonable 5 year patency rates and some would advocate using this in preference to vein so that vein can be used for other procedures in the future. In general terms either technique is usually associated with an excellent outcome (if run off satisfactory).

Procedure
Artery dissected out, IV heparin 3,000 units given and then the vessels are cross clamped
Longitudinal arteriotomy
Graft cut to size and tunneled to arteriotomy sites
Anastomosis to femoral artery usually with 5/0 ‘double ended’ Prolene suture
Distal anastomosis usually using 6/0 ‘double ended’ Prolene

Distal disease
Femoro-distal bypass surgery takes longer to perform, is more technically challenging and has higher failure rates.
In elderly diabetic patients with poor runoff a primary amputation may well be a safer and more effective option. There is no point in embarking on this type of surgery in patients who are wheelchair bound.
In femorodistal bypasses vein gives superior outcomes to PTFE.

Rules
Vein mapping 1st to see whether there is suitable vein (the preferred conduit). Sub intimal hyperplasia occurs early when PTFE is used for the distal anastomosis and will lead to early graft occlusion and failure.
Essential operative procedure as for above knee fem-pop.
If there is insufficient vein for the entire conduit then vein can be attached to the end of the PTFE graft and then used for the distal anastomosis. This type of ‘vein boot’ is technically referred to as a Miller Cuff and is associated with better patency rates than PTFE alone.
Remember the more distal the arterial anastomosis the lower the success rate.

References
Peach G et al. Diagnosis and management of peripheral arterial disease. BMJ 2012; 345: 36-41.

1765
Q

A 72-year-old woman is admitted to the acute surgical unit with profuse vomiting. Admission bloods show the following:

Na+ 131 mmol/l
K+ 2.2 mmol/l
Urea 3.1 mmol/l
Creatinine 56 micro mol/l
Glucose 4.3 mmol/l

Which one of the following ECG features is most likely to be seen?

Short PR interval

Short QT interval

Flattened P waves

J waves

U waves

A

Hypokalaemia - U waves on ECG
J waves are seen in hypothermia whilst delta waves are associated with Wolff Parkinson White syndrome.

ECG features in hypokalemia

U waves
Small or absent T waves (occasionally inversion)
Prolonged PR interval
ST depression
Long QT interval

One registered user suggests the following rhyme!
In Hypokalaemia, U have no Pot and no T, but a long PR and a long QT!

1766
Q

A 65 year old man is admitted with cholangitis and investigations demonstrate a carcinoma of the pancreatic head. An ERCP is attempted but the surgeon is unable to cannulate the ampulla. What is the best course of action?

Whipples procedure

Laparotomy and formation of hepatico-jejunostomy

Percutaneous transhepatic cholangiogram and drain

External beam radiotherapy

Laparotomy and duodenoduodenostomy

A

Cancer of the pancreatic head will cause obstructive jaundice and intrahepatic duct dilatation. When an ERCP has failed the most appropriate option is to attempt a PTC. This procedure is always preceded by an ultrasound (which presumably this patient has already had or they would not be undergoing an ERCP). Prior to performing the PTC it is important to stage the disease and establish resectability or not. This is because the PTC drains frequently dislodge and fall out. It is usually desirable to pass a stent at the time of doing the PTC to mitigate the effects of this problem.

Surgical jaundice

Jaundice can present in a manner of different surgical situations. As with all types of jaundice a careful history and examination will often give clues as to the most likely underlying cause. Liver function tests whilst conveying little in the way of information about liver synthetic function, will often facilitate classification as to whether the jaundice is pre hepatic, hepatic or post hepatic. The typical LFT patterns are given below:

Location Bilirubin ALT/ AST Alkaline phosphatase
Pre hepatic Normal or high Normal Normal
Hepatic High Elevated (often very high) Elevated but seldom to very high levels
Post hepatic High-very high Moderate elevation High- very high

In post hepatic jaundice the stools are often of pale colour and this feature should be specifically addressed in the history.

Modes of presentation
These are addressed in the table below:

Diagnosis Typical features Pathogenesis
Gallstones Typically history of biliary colic or episodes of chlolecystitis. Obstructive type history and test results. Usually small calibre gallstones which can pass through the cystic duct. In Mirizzi syndrome the stone may compress the bile duct directly- one of the rare times that cholecystitis may present with jaundice
Cholangitis Usually obstructive and will have Charcots triad of symptoms (pain, fever, jaundice) Ascending infection of the bile ducts usually by E. coli and by definition occurring in a pool of stagnant bile.
Pancreatic cancer Typically painless jaundice with palpable gallbladder (Courvoisier’s Law) Direct occlusion of distal bile duct or pancreatic duct by tumour. Sometimes nodal disease at the portal hepatis may be the culprit in which case the bile duct may be of normal calibre.
TPN associated jaundice Usually follows long term use and is usually painless with non obstructive features Often due to hepatic dysfunction and fatty liver which may occur with long term TPN usage.
Bile duct injury Depending upon the type of injury may be of sudden or gradual onset and is usually of obstructive type Often due to a difficult cholecystectomy when anatomy in Calots triangle is not appreciated. In the worst scenario the bile duct is excised and jaundice offers rapidly post operatively. More insidious is that of bile duct stenosis which may be caused by clips or diathermy injury.
Cholangiocarcinoma Gradual onset obstructive pattern Direct occlusion by disease and also extrinsic compression by nodal disease at the porta hepatis.
Septic surgical patient Usually hepatic features Combination of impaired biliary excretion and drugs such as ciprofloxacin which may cause cholestasis.
Metastatic disease Mixed hepatic and post hepatic Combination of liver synthetic failure (late) and extrinsic compression by nodal disease and anatomical compression of intra hepatic structures (earlier)

Diagnosis
An ultrasound of the liver and biliary tree is the most commonly used first line test. This will establish bile duct calibre, often ascertain the presence of gallstones, may visualise pancreatic masses and other lesions. The most important clinical question is essentially the extent of biliary dilatation and its distribution.

Where pancreatic neoplasia is suspected, the next test should be a pancreatic protocol CT scan. With liver tumours and cholangiocarcinoma an MRI/ MRCP is often the preferred option. PET scans may be used to stage a number of malignancies but do not routinely form part of first line testing.

Where MRCP fails to give adequate information an ERCP may be necessary. In many cases this may form part of patient management. It is however, invasive and certainly not without risk and highly operator dependent.

Management
Clearly this will depend to an extent upon the underlying cause but relief of jaundice is important, even if surgery forms part of the planned treatment. Patients with unrelieved jaundice have a much higher incidence of septic complications, bleeding and death.

Screen for and address any clotting irregularities

In patients with malignancy a stent will need to be inserted. These come in two main types; metal and plastic. Plastic stents are cheap and easy to replace and should be used if any surgical intervention (e.g. Whipples) is planned. However, they are prone to displacement and blockage. Metal stents are much more expensive and may compromise a surgical resection. However, they are far less prone to displacement and to a lesser extent blockage than their plastic counterparts.

If malignancy is in bile duct/ pancreatic head and stenting has been attempted and has failed, then an alternative strategy is to drain the biliary system percutaneously via a transhepatic route. It may also be possible to insert a stent in this way. One of the main problems with temporary PTC’s is their propensity to displacement, which may result in a bile leak.

In patients who have a bile duct injury surgery will be required to repair the defect. If the bile duct has been inadvertently excised then a hepatico-jejunostomy will need to be created (difficult!)

If gallstones are the culprit, then these may be removed by ERCP and a cholecystectomy performed. Where there is doubt about the efficacy of the ERCP an operative cholangiogram should be performed and bile duct exploration undertaken where stones remain. When the bile duct has been formally opened the options are between closure over a T tube, a choledochoduodenostomy or choledochojejunostomy.

Patients with cholangitis should receive high dose broad spectrum antibiotics via the intravenous route. Biliary decompression should follow soon afterwards, instrumenting the bile duct of these patients will often provoke a septic episode (but should be done anyway).

1767
Q

Administration of which of the following may facilitate the identification of parathyroid glands intra operatively?

Patent V dye intravenously

Methylene blue intravenously

Indigocarmine dye intravenously

India ink intravenously

Intravenous rifampicin

A

Methylene blue stains the parathyroid glands and can be useful in facilitating their identification.

Preparation for surgery

Elective and emergency patients require different preparation.

Elective cases
Consider pre admission clinic to address medical issues.
Blood tests including FBC, U+E, LFT’s, Clotting, Group and Save
Urine analysis
Pregnancy test
Sickle cell test
ECG/ Chest x-ray

Exact tests to be performed will depend upon the proposed procedure and patient fitness.

Risk factors for development of deep vein thrombosis should be assessed and a plan for thromboprophylaxis formulated.

Diabetes
Diabetic patients have greater risk of complications.
Poorly controlled diabetes carries high risk of wound infections.
Patients with diet or tablet controlled diabetes may be managed using a policy of omitting medication and checking blood glucose levels regularly. Diabetics who are poorly controlled or who take insulin may require a intravenous sliding scale. Potassium supplementation should also be given.
Diabetic cases should be operated on first.

Emergency cases
Stabilise and resuscitate where needed.
Consider whether antibiotics are needed and when and how they should be administered.
Inform blood bank if major procedures planned particularly where coagulopathies are present at the outset or anticipated (e.g. Ruptured AAA repair)
Don’t forget to consent and inform relatives.

Special preparation
Some procedures require special preparation:
Thyroid surgery; vocal cord check.
Parathyroid surgery; consider methylene blue to identify gland.
Sentinel node biopsy; radioactive marker/ patent blue dye.
Surgery involving the thoracic duct; consider administration of cream.
Pheochromocytoma surgery; will need alpha and beta blockade.
Surgery for carcinoid tumours; will need covering with octreotide.
Colorectal cases; bowel preparation (especially left sided surgery)
Thyrotoxicosis; lugols iodine/ medical therapy.

References
Management of adults with diabetes undergoing surgery and elective procedures. NHS Diabetes. April 2011.

1768
Q

Use of which of the following muscle relaxants is least likely to result in histamine release?

Atracurium

Tubocurarine

Doxacurium

Mivacurium

Suxamethonium

A

Drugs in the tetrahydroisoquinoline such as atracurium cause histamine release. Vecuronium and suxamethonium do not do this.
Muscle relaxants

Suxamethonium
Depolarising neuromuscular blocker
Inhibits action of acetylcholine at the neuromuscular junction
Degraded by plasma cholinesterase and acetylcholinesterase (affected by lack of acetylcholinesterase)
Fastest onset and shortest duration of action of all muscle relaxants
Produces generalised muscular contraction prior to paralysis
Adverse effects include hyperkalaemia, malignant hyperthermia, delayed recovery
Atracurium
Non depolarising neuromuscular blocking drug
Duration of action usually 30-45 minutes
Generalised histamine release on administration may produce facial flushing, tachycardia and hypotension
Not excreted by liver or kidney, broken down in tissues by hydrolysis
Reversed by neostigmine
Vecuronium
Non depolarising neuromuscular blocking drug
Duration of action approximately 30 - 40 minutes
Degraded by liver and kidney and effects prolonged in organ dysfunction
Effects may be reversed by neostigmine
Pancuronium
Non depolarising neuromuscular blocker
Onset of action approximately 2-3 minutes
Duration of action up to 2 hours
Effects may be partially reversed with drugs such as neostigmine

1769
Q

Which of the following infection control modalities would be standard practice for a 53 year old male undergoing a Mayo repair of a paraumbilical hernia?

Sodium hypochlorite solution applied to the skin

Administration of clindamycin

Alcoholic chlorhexidine applied to the skin

Administration of gentamicin

Pre operative shaving

A

The patient will require skin preparation. However, use of glutaraldehyde or sodium hypochlorite would be an inappropriate choice. As the Mayo repair does not involve implantation of prosthetic mesh the use of antibiotics is not appropriate.

Surgical site infection

  • Surgical site infections may occur following a breach in tissue surfaces and allow normal commensals and other pathogens to initiate infection. They are a major cause of morbidity and mortality.
    Surgical site infections (SSI) comprise up to 20% of all healthcare associated infections and at least 5% of patients undergoing surgery will develop an SSI as a result.
    In many cases the organisms are derived from the patient’s own body. Measures that may increase the risk of SSI include:
    Shaving the wound using a razor (disposable clipper preferred)
    Using a non iodine impregnated incise drape if one is deemed to be necessary
    Tissue hypoxia
    Delayed administration of prophylactic antibiotics in tourniquet surgery

Preoperatively
Don’t remove body hair routinely
If hair needs removal, use electrical clippers with single use head (razors increase infection risk)
Antibiotic prophylaxis if:
- placement of prosthesis or valve
- clean-contaminated surgery
- contaminated surgery
Use local formulary
Aim to give single dose IV antibiotic on anaesthesia
If a tourniquet is to be used, give prophylactic antibiotics earlier

Intraoperatively
Prepare the skin with alcoholic chlorhexidine (Lowest incidence of SSI)
Cover surgical site with dressing
A recent meta analysis has confirmed that administration of supplementary oxygen does not reduce the risk of wound infection. In contrast to previous individual RCT’s(1)
Wound edge protectors do not appear to confer benefit (2)

Post operatively
Tissue viability advice for management of surgical wounds healing by secondary intention

Use of diathermy for skin incisions
In the NICE guidelines the use of diathermy for skin incisions is not advocated(3). Several randomised controlled trials have been undertaken and demonstrated no increase in risk of SSI when diathermy is used(4).

References
1. Brar M et al.. Perioperative supplemental oxygen in colorectal patients: a meta analysis. J Surg Res 2011 (166): 227 -235.
2. Pinkney T et al. Impact of wound edge protection devices on surgical site infection after laparotomy: impact of a multicentre randomised controlled trial (ROSSINI Trial). BMJ 2013 (347):10.
3. http://www.nice.org.uk/CG74
4. Ahmad N and Ahmed A. Meta-analysis of the effectiveness of surgical scalpel or diathermy in making abdominal skin incisions. Ann Surg 2011, 253(1):8-13.

1770
Q

A 32 year old man presents to the vascular clinic with symptoms of foot pain during exertion. He is a heavy smoker and has recently tried to stop smoking. On examination, he has normal pulses to the level of the popliteal. However, foot pulses are absent. A diagnostic angiogram is performed which shows an abrupt cut off at the level of the anterior tibial artery, together with the formation of corkscrew shaped collateral vessels distally. What is the most likely diagnosis?

Buergers disease

Granulomatosis with polyangiitis

Giant cell arteritis

Takayasu’s arteritis

Polyarteritis nodosa

A

Buergers disease is most common in young male smokers. This demographic is changing in those areas where young female smokers are more common. In the acute lesion the internal elastic lamina of the vessels is usually intact. As the disease progresses the changes progress to hypercellular occlusive thrombus. Tortuous corkscrew collaterals may reconstitute patent segments of the distal tibial or pedal vessels.

Vasculitis

The vasculitides are a group of conditions characterised by inflammation of the blood vessel walls. This may, in turn, compromise vessel integrity. Constitutional symptoms may be present. Whilst certain disease subtypes are reported to affect specific vessels, there is often a degree of overlap clinically.

Vessel diameter and vasculitis classification
Aorta and branches
Takayasu’s arteritis
Buergers disease
Giant cell arteritis
Large and medium sized arteries
Buergers disease
Giant cell arteritis
Polyarteritis nodosa
Medium sized muscular arteries
Polyarteritis nodosa
Wegeners granulomatosis
Small muscular arteries
Granulomatosis with polyangiitis
Rheumatoid vasculitis

Specific conditions
Takyasu’s arteritis
Inflammatory, obliterative arteritis affecting aorta and branches
Females> Males
Symptoms may include upper limb claudication
Clinical findings include diminished or absent pulses
ESR often affected during the acute phase
Buergers disease
Segmental thrombotic occlusions of the small and medium sized lower limb vessels
Commonest in young male smokers
Proximal pulses usually present, but pedal pulses are lost
An acuter hypercellular occlusive thrombus is often present
Tortuous corkscrew shaped collateral vessels may be seen on angiography
Giant cell arteritis
Systemic granulomatous arteritis that usually affects large and medium sized vessels
Females > Males
Temporal arteritis is commonest type
Granulomatous lesions may be seen on biopsy (although up to 50% are normal)
Polyarteritis nodosa
Systemic necrotising vasculitis affecting small and medium sized muscular arteries
Most common in populations with high prevalence of hepatitis B
Renal disease is seen in 70% cases
Angiography may show saccular or fusiform aneurysms and arterial stenoses
Granulomatosis with polyangiitis
Predominantly affects small and medium sized arteries
Systemic necrotising granulomatous vasculitis
Cutaneous vascular lesions may be seen (ulceration, nodules and purpura)
Sinus imaging may show mucosal thickening and air fluid levels

Treatment
Conditions such as Buergers disease are markedly helped by smoking cessation. Immunosupression is the main treatment for vasculitides.

1771
Q

A 56 year old lady is referred to the colorectal clinic with symptoms of pruritus ani. On examination, a polypoidal mass is identified inferior to the dentate line. A biopsy confirms squamous cell carcinoma. To which of the following lymph node groups will the lesion potentially metastasise?

Internal iliac

External iliac

Mesorectal

Inguinal

None of the above

A

Lesions distal to the dentate line drain to the inguinal nodes. Occasionally this will result in the need for a block dissection of the groin.

Rectum

The rectum is approximately 12 cm long. It is a capacitance organ. It has both intra and extraperitoneal components. The transition between the sigmoid colon is marked by the disappearance of the tenia coli.The extra peritoneal rectum is surrounded by mesorectal fat that also contains lymph nodes. This mesorectal fatty layer is removed surgically during rectal cancer surgery (Total Mesorectal Excision). The fascial layers that surround the rectum are important clinical landmarks, anteriorly lies the fascia of Denonvilliers. Posteriorly lies Waldeyers fascia.

Extra peritoneal rectum
Posterior upper third
Posterior and lateral middle third
Whole lower third

Relations
Anteriorly (Males) Rectovesical pouch
Bladder
Prostate
Seminal vesicles
Anteriorly (Females) Recto-uterine pouch (Douglas)
Cervix
Vaginal wall
Posteriorly Sacrum
Coccyx
Middle sacral artery
Laterally Levator ani
Coccygeus

Arterial supply
Superior rectal artery
Middle rectal artery (from the internal iliac)
Inferior rectal artery (from the pudendal vessels)

Venous drainage
Superior rectal vein
Inferior rectal vein
Note the venous drainage is a site of portosystemic anastomosis.

Lymphatic drainage
Mesorectal lymph nodes (superior to dentate line)
Inguinal nodes (inferior to dentate line)

1772
Q

A 19 year old man undergoes an open inguinal hernia repair. The cord is mobilised and the deep inguinal ring identified. Which of the following structures forms its lateral wall?

External oblique aponeurosis

Transversalis fascia

Conjoint tendon

Inferior epigastric artery

Inferior epigastric vein

A

The transversalis fascia forms the superolateral edge of the deep inguinal ring. The epigastric vessels form its inferomedial wall.

Inguinal canal

Location
Above the inguinal ligament
The inguinal canal is 4cm long
The superficial ring is located anterior to the pubic tubercle
The deep ring is located approximately 1.5-2cm above the half way point between the anterior superior iliac spine and the pubic tubercle

Boundaries of the inguinal canal
Floor
External oblique aponeurosis
Inguinal ligament
Lacunar ligament
Roof
Internal oblique
Transversus abdominis
Anterior wall External oblique aponeurosis
Posterior wall
Transversalis fascia
Conjoint tendon
Laterally
Internal ring
Transversalis fascia
Fibres of internal oblique
Medially
External ring
Conjoint tendon

Contents
Males Spermatic cord and ilioinguinal nerve As it passes through the canal the spermatic cord has 3 coverings:
External spermatic fascia from external oblique aponeurosis
Cremasteric fascia
Internal spermatic fascia
Females Round ligament of uterus and ilioinguinal nerve

1773
Q

A 44 year old man is stabbed in the back and the left kidney is injured. A haematoma forms, which of the following fascial structures will contain the haematoma?

Waldeyers fascia

Sibsons fascia

Bucks fascia

Gerotas fascia

Denonvilliers fascia

A

Waldeyers fascia- Posterior ano-rectum
Sibsons fascia- Lung apex
Bucks fascia- Base of penis
Gerotas fascia- Surrounding kidney
Denonvilliers fascia- Between rectum and prostate

Renal anatomy

Each kidney is about 11cm long, 5cm wide and 3cm thick. They are located in a deep gutter alongside the projecting vertebral bodies, on the anterior surface of psoas major. In most cases the left kidney lies approximately 1.5cm higher than the right. The upper pole of both kidneys approximates with the 11th rib (beware pneumothorax during nephrectomy). On the left hand side the hilum is located at the L1 vertebral level and the right kidney at level L1-2. The lower border of the kidneys is usually alongside L3.

The table below shows the anatomical relations of the kidneys:

Relations
Relations Right Kidney Left Kidney
Posterior Quadratus lumborum, diaphragm, psoas major, transversus abdominis Quadratus lumborum, diaphragm, psoas major, transversus abdominis
Anterior Hepatic flexure of colon Stomach, Pancreatic tail
Superior Liver, adrenal gland Spleen, adrenal gland

Fascial covering
Each kidney and suprarenal gland is enclosed within a common layer of investing fascia, derived from the transversalis fascia. It is divided into anterior and posterior layers (Gerotas fascia).

Renal structure
Kidneys are surrounded by an outer cortex and an inner medulla which usually contains between 6 and 10 pyramidal structures. The papilla marks the innermost apex of these. They terminate at the renal pelvis, into the ureter.
Lying in a hollow within the kidney is the renal sinus. This contains:
1. Branches of the renal artery
2. Tributaries of the renal vein
3. Major and minor calyces’s
4. Fat

Structures at the renal hilum
The renal vein lies most anteriorly, then renal artery (it is an end artery) and the ureter lies most posterior.

1774
Q

A 30 year old man is trapped in a house fire and sustains 30% partial and full thickness burns to his torso and limbs. Three days following admission he has a brisk haematemesis. Which of the following is the most likely explanation for this event?

Dieulafoy lesion

Curlings ulcers

Mallory Weiss tear

Depletion of platelets

Depletion of clotting factors

A

Curlings ulcers typically occur secondary to thermal injuries and are caused by loss of GI protective mechanisms. They are at greater risk of perforation than stress ulcers and may also haemorrhage.

Burns

Burns may be thermal, chemical or electrical. In the former category are burns which occur as a result of heat. Chemical burns occur when the skin is exposed to an extremely caustic or alkaline substance. Electrical burns occur following exposure to electrical current. The immediate management includes removal of the burning source which usually includes irrigation of the burned area. A detailed assessment then needs to be made of the extent of the burns and a number of charts are available for recording this information. The degree of injury relates to the temperature and duration of exposure. Most domestic burns are mainly scalds in young children.

Following the burn, there is a local response with progressive tissue loss and release of inflammatory cytokines. Systemically, there are cardiovascular effects resulting from fluid loss and sequestration of fluid into the third space. There is a marked catabolic response. Immunosuppression is common with large burns and bacterial translocation from the gut lumen is a recognised event. Sepsis is a common cause of death following major burns.

Types of burn

Type of burn Skin layers affected Skin appearance Blanching Management
Epidermal/Superficial Epidermis Red, moist Yes
Superficial partial thickness Epidermis and part of papillary dermis affected Pale, dry Yes Normally heals with no intervention
Deep partial thickness Epidermis, whole papillary dermis affected Mottled red colour No Needs surgical intervention (depending on site)
Full thickness Whole skin layer and subcutaneous tissue affected Dry, leathery hard wound No Burns centre

Depth of burn assessment
Bleeding on needle prick
Sensation
Appearance
Blanching to pressure

Percentage burn estimation
Lund Browder chart: most accurate even in children
Wallace rule of nines
Palmar surface: surface area palm = 0.8% burn

> 15% body surface area burns in adults needs urgent burn fluid resuscitation

Transfer to burn centre if:
Need burn shock resuscitation
Face/hands/genitals affected
Deep partial thickness or full thickness burns
Significant electrical/chemical burns

Management
The initial aim is to stop the burning process and resuscitate the patient. Intravenous fluids will be required for children with burns greater than 10% of total body surface area. Adults with burns greater than 15% of total body surface area will also require IV fluids. The fluids are calculated using the Parkland formula which is; volume of fluid= total body surface area of the burn % x weight (Kg) x2-4 (preference for lower amount i.e. 2ml to avoid excessive fluid overload). Half of the fluid is administered in the first 8 hours. A urinary catheter should be inserted. Analgesia should be given. Complex burns, burns involving the hand perineum and face and burns >10% in adults and >5% in children should be transferred to a burns unit.

Circumferential burns affecting a limb or severe torso burns impeding respiration may require escharotomy to divide the burnt tissue.

Conservative management is appropriate for superficial burns and mixed superficial burns that will heal in 2 weeks. More complex burns may require excision and skin grafting. Excision and primary closure is not generally practised as there is a high risk of infection.

There is no evidence to support the use of anti microbial prophylaxis or topical antibiotics in burn patients.

Escharotomies
Indicated in circumferential full thickness burns to the torso or limbs.
Careful division of the encasing band of burn tissue will potentially improve ventilation (if the burn involves the torso), or relieve compartment syndrome and oedema (where a limb is involved)

References
www.euroburn.org/e107files/downloads/guidelinesburncare.pdf

Barajas-Nava LA, López-Alcalde J, Roqué i Figuls M, Solà I, Bonfill Cosp X. Antibiotic prophylaxis for preventing burn wound infection. Cochrane Database of Systematic Reviews 2013, Issue 6. Art. No.: CD008738. DOI: 10.1002/14651858.CD008738.pub2.

Hettiaratchy S & Papini R. Initial management of a major burn: assessment and resuscitation. BMJ 2004;329:101-103

1775
Q

A patient involved in a national research trial of surgery versus conservative management for ruptured abdominal aortic aneurysms dies during surgery because the surgeon decided to deviate from the study protocol and implant a different type of graft material. What is the correct course of action?

Refer the surgeon to the GMC

Report the case to the ethics committee that gave a favorable opinion for the study

Report it to the responsible officer for the hospital

Report it to the Caldicott guardian for the hospital

Report it to the media

A

Where a research participant experiences an adverse outcome, the correct course of action is to refer to the ethics committee that approved the study. Minor adverse effects are usually reviewed by the data monitoring committee and reported annually. However, a serious event is reported immediately. It would not be appropriate to refer to the GMC though, it may be that the surgeon deviated from the protocol for good reason.

Audit and Research

Clinical audit
Quality improvement process that seeks to improve patient care and outcomes through systematic review of care against explicit criteria and the implementation of change. Aspects of the structure, processes, and outcomes of care are selected and systematically evaluated against explicit criteria. Where indicated, changes are implemented at an individual, team, or service level and further monitoring is used to confirm improvement in healthcare delivery. (NICE).

Research
Aims to derive new knowledge which is potentially generalisable or transferable.

1776
Q

A 43 year old woman undergoes a sigmoid colectomy for carcinoma. The histology report shows pT3, pN1, systemic staging is M0. She is otherwise well. What is the most appropriate course of action?

Active surveillance with CT scans at 6 and 12 months and monthly CEA measurement

Referral for chemotherapy

Radiotherapy to the resection site

Active surveillance with colonoscopy at 12 months and CT scan at 6 months and 3 monthly CEA measurement

Discharge

A

Chemotherapy for colonic cancer is offered when patients have nodal disease.

Colorectal cancer treatment

Patients diagnosed as having colorectal cancer should be completely staged using CT of the chest/ abdomen and pelvis. Their entire colon should have been evaluated with colonoscopy or CT colonography. Patients whose tumours lie below the peritoneal reflection should have their mesorectum evaluated with MRI.

Once their staging is complete patients should be discussed within a dedicated colorectal MDT meeting and a treatment plan formulated.

Treatment of colonic cancer
Cancer of the colon is nearly always treated with surgery. Stents, surgical bypass and diversion stomas may all be used as palliative adjuncts. Resectional surgery is the only option for cure in patients with colon cancer. The procedure is tailored to the patient and the tumour location. The lymphatic drainage of the colon follows the arterial supply and therefore most resections are tailored around the resection of particular lymphatic chains (e.g. ileo-colic pedicle for right sided tumours). Some patients may have confounding factors that will govern the choice of procedure, for example a tumour in a patient from a HNPCC family may be better served with a panproctocolectomy rather than segmental resection. Following resection the decision has to be made regarding restoration of continuity. For an anastomosis to heal the key technical factors include; adequate blood supply, mucosal apposition and no tissue tension. Surrounding sepsis, unstable patients and inexperienced surgeons may compromise these key principles and in such circumstances it may be safer to construct an end stoma rather than attempting an anastomosis.
When a colonic cancer presents with an obstructing lesion; the options are to either stent it or resect. In modern practice it is unusual to simply defunction a colonic tumour with a proximal loop stoma. This differs from the situation in the rectum (see below).
Following resection patients with risk factors for disease recurrence are usually offered chemotherapy, a combination of 5FU and oxaliplatin is common.

Rectal cancer
The management of rectal cancer is slightly different to that of colonic cancer. This reflects the rectum’s anatomical location and the challenges posed as a result. Tumours located in the rectum can be surgically resected with either an anterior resection or an abdomino - perineal resection. The technical aspects governing the choice between these two procedures can be complex to appreciate and the main point to appreciate for the MRCS is that involvement of the sphincter complex or very low tumours require APER. In the rectum a 2cm distal clearance margin is required and this may also impact on the procedure chosen. In addition to excision of the rectal tube an integral part of the procedure is a meticulous dissection of the mesorectal fat and lymph nodes (total mesorectal excision/ TME). In rectal cancer surgery invovlement of the cirumferential resection margin carries a high risk of disease recurrence. Because the rectum is an extraperitoneal structure (until you remove it that is!) it is possible to irradiate it, something which cannot be offered for colonic tumours. This has a major impact in rectal cancer treatment and many patients will be offered neoadjuvent radiotherapy (both long and short course) prior to resectional surgery. Patients with T1, 2 and 3 /N0 disease on imaging do not require irradiation and should proceed straight to surgery. Patients with T4 disease will typically have long course chemo radiotherapy. Patients presenting with large bowel obstruction from rectal cancer should not undergo resectional surgery without staging as primary treatment (very different from colonic cancer). This is because rectal surgery is more technically demanding, the anastomotic leak rate is higher and the danger of a positive resection margin in an unstaged patient is high. Therefore patients with obstructing rectal cancer should have a defunctioning loop colostomy.

Summary of procedures
The operations for cancer are segmental resections based on blood supply and lymphatic drainage. These commonly performed procedures are core knowledge for the MRCS and should be understood.

Site of cancer Type of resection Anastomosis Risk of leak
Right colon Right hemicolectomy Ileo-colic Low <5%
Transverse Extended right hemicolectomy Ileo-colic Low <5%
Splenic flexure Extended right hemicolectomy Ileo-colic Low <5%
Splenic flexure Left hemicolectomy Colo-colon 2-5%
Left colon Left hemicolectomy Colo-colon 2-5%
Sigmoid colon High anterior resection Colo-rectal 5%
Upper rectum Anterior resection (TME) Colo-rectal 5%
Low rectum Anterior resection (Low TME) Colo-rectal
(+/- Defunctioning stoma) 10%
Anal verge Abdomino-perineal excision of colon and rectum None n/a

In the emergency setting, where the bowel has perforated, the risk of an anastomotic breakdown is much greater, particularly when the anastomosis is colon-colon. In this situation, an end colostomy is often safer and can be reversed later. When resection of the sigmoid colon is performed and an end colostomy is fashioned the operation is referred to as a Hartmans procedure. Whilst left sided resections are more risky, ileo-colic anastomoses are relatively safe even in the emergency setting and do not need to be defunctioned.

References
A review of the diagnosis and management of colorectal cancer and a summary of the UK National Institute of Clinical Excellence guidelines is provided in:
NICE Guideline NG151. Last updated in 2020.

1777
Q

A 23 year old woman is admitted with sepsis and right loin pain. She has a history of a UTI that was treated by the GP with a course of trimethoprim that was commenced 24 hours previously. Which of the organisms listed below is the most likely cause?

Bacteroides fragilis

E-Coli

Clostridium difficile

Candida albicans

Mycobacterium tuberculosis

A

Ascending infection of the genitourinary tract is most commonly caused by E-Coli. Other organisms may be accountable, however, these are less common

Surgical Microbiology

An extensive topic so an overview is given here. Organisms causing common surgical infections are reasonable topics in the examination. However, microbiology is less rigorously tested than anatomy, for example.

Common organisms

Staphylococcus aureus
Facultative anaerobe
Gram positive coccus
Haemolysis on blood agar plates
Catalase positive
20% population are long term carriers
Exo and entero toxin may result in toxic shock syndrome and gastroenteritis respectively
Ideally treated with penicillin although many strains now resistant through beta Lactamase production. In the UK less than 5% of isolates are sensitive to penicillin.
Resistance to methicillin (and other antibiotics) is mediated by the mec operon , essentially penicillin binding protein is altered and resistance to this class of antibiotics ensues
Common cause of cutaneous infections and abscesses

Streptococcus pyogenes
Gram positive, forms chain like colonies, Lancefield Group A Streptococcus
Produces beta haemolysis on blood agar plates
Rarely part of normal skin microflora
Catalase negative
Releases a number of proteins/ virulence factors into host including hyaluronidase, streptokinase which allow rapid tissue destruction
Releases superantigens such as pyogenic exotoxin A which results in scarlet fever
Remains sensitive to penicillin, macrolides may be used as an alternative.

Escherichia coli
Gram negative rod
Facultative anaerobe, non sporing
Wide range of subtypes and some are normal gut commensals
Some subtypes such as 0157 may produce lethal toxins resulting in haemolytic-uraemic syndrome
Enterotoxigenic E-Coli produces an enterotoxin (ST enterotoxin) that results in large volume fluid secretion into the gut lumen (Via cGMP activation)
Enteropathogenic E-Coli binds to intestinal cells and cause structural damage, this coupled with a moderate (or in case of enteroinvasive E-Coli significant) invasive component produces enteritis and large volume diarrhoea together with fever.
They are resistant to many antibiotics used to treat gram positive infections and acquire resistance rapidly and are recognised as producing beta lactamases

Campylobacter jejuni
Curved, gram negative, non sporulating bacteria
One of the commonest causes of diarrhoea worldwide
Produces enteritis which is often diffuse and blood may be passed
Remains a differential for right iliac fossa pain with diarrhoea
Self limiting infection so antibiotics are not usually advised. However, the quinolones are often rapidly effective.

Helicobacter pylori
Gram negative, helix shaped rod, microaerophillic
Produces hydrogenase that can derive energy from hydrogen released by intestinal bacteria
Flagellated and mobile
Those carrying the cag A gene may cause ulcers
It secretes urease that breaks down gastric urea> Carbon dioxide and ammonia> ammonium>bicarbonate (simplified!) The bicarbonate can neutralise the gastric acid.
Usually colonises the gastric antrum and irritates resulting in increased gastrin release and higher levels of gastric acid. These patients will develop duodenal ulcers. In those with more diffuse H-Pylori infection gastric acid levels are lower and ulcers develop by local tissue damage from H-Pylori- these patients get gastric ulcers.
Diagnosis may be made by serology (approx. 75% sensitive). Biopsy urease test during endoscopy probably the most sensitive.
In patients who are colonised 10-20% risk of peptic ulcer, 1-2% risk gastric cancer, <1% risk MALT lymphoma.

1778
Q

A 32 year old Indian lady presents with a diffuse swelling of the left breast. She has a 4 month old child. Clinically, she has jaundice and there is erythema of the left breast. Which of the following lesions is most likely?

Phyllodes tumour

Radial scar

Invasive ductal carcinoma

Invasive lobular carcinoma

Inflammatory carcinoma

A

Inflammatory breast cancers have an aggressive nature. Dissemination occurs early and is more resistant to adjuvant treatments than other types of breast cancer. Often occurs in pregnancy or lactation.

Breast cancer

  • Commoner in the older age group
    Invasive ductal carcinomas are the most common type. Some may arise as a result of ductal carcinoma in situ (DCIS). There are associated carcinomas of special type e.g. Tubular that may carry better prognosis.
    The pathological assessment involves assessment of the tumour and lymph nodes, sentinel lymph node biopsy is often used to minimise the morbidity of an axillary dissection.
    Treatment, typically this is either wide local excision or mastectomy. There are many sub types of both of these that fall outside of the MRCS. Some key rules to bear in mind.
    Whatever operation is contemplated the final cosmetic outcome does have a bearing. A woman with small breasts and a large tumour will tend to fare better with mastectomy, even if clear pathological and clinical margins can be obtained. Conversely a women with larger breasts may be able to undergo breast conserving surgery even with a relatively large primary lesion (NB tumours >4cm used to attract recommendation for mastectomy). For screen detected and impalpable tumour image guidance will be necessary.
    Reconstruction is always an option following any resectional procedure. However, its exact type must be tailored to age and co-morbidities of the patient. The main operations in common use include latissimus dorsi myocutaneous flap and sub pectoral implants. Women wishing to avoid a prosthesis may be offered TRAM or DIEP flaps.

Surgical options
Mastectomy vs Wide local excision

Mastectomy Wide Local Excision
Multifocal tumour Solitary lesion
Central tumour Peripheral tumour
Large lesion in small breast Small lesion in large breast
DCIS >4cm DCIS <4cm
Patient Choice Patient choice

Central lesions may be managed using breast conserving surgery where an acceptable cosmetic result may be obtained, this is rarely the case in small breasts

A compelling indication for mastectomy, a larger tumour that would be unsuitable for breast conserving surgery

Whatever surgical option is chosen the aim should be to have a local recurrence rate of 5% or less at 5 years [1].

Nottingham Prognostic Index
The Nottingham Prognostic Index can be used to give an indication of survival. In this system the tumour size is weighted less heavily than other major prognostic parameters.

Calculation of NPI
Tumour Size x 0.2 + Lymph node score(From table below)+Grade score(From table below).

Score Lymph nodes involved Grade
1 0 1
2 1-3 2
3 >3 3

Prognosis

Score Percentage 5 year survival
2.0 to 2.4 93%
2.5 to 3.4 85%
3.5 to 5.4 70%
>5.4 50%

This data was originally published in 1992. It should be emphasised that other factors such as vascular invasion and receptor status also impact on survival and are not included in this data and account for varying prognoses often cited in the literature.

References
For guidance on how breast cancer is managed in the United Kingdom visit the Association of Breast Surgery website (www.https://associationofbreastsurgery.org.uk/).

1779
Q

What is the most important urinary acid base buffer?

Albumin

Phosphate

Nitrate

Bicarbonate

Aspartime

A

Phosphate is the most important urinary buffer. Its concentration is raised relative to that of plasma. While phosphate is a minor extracellular buffer, under ordinary circumstances it is the single most important urinary buffer. Since, pH of urine is commonly close to the pK of the phosphate buffer pair, it functions as an ideal urinary buffer. Proteins cannot act as urinary buffers as they are not filtered.

Renal Physiology

Overview
Each nephron is supplied with blood from an afferent arteriole that opens onto the glomerular capillary bed.
Blood then flows to an efferent arteriole, supplying the peritubular capillaries and medullary vasa recta.
The kidney receives up to 25% of resting cardiac output.

Control of blood flow
The kidney is able to autoregulate its blood flow between systolic pressures of 80- 180mmHg so there is little variation in renal blood flow.
This is achieved by myogenic control of arteriolar tone, both sympathetic input and hormonal signals (e.g. renin) are responsible.

Glomerular structure and function
Blood inside the glomerulus has considerable hydrostatic pressure.
The basement membrane has pores that will allow free diffusion of smaller solutes, larger negatively charged molecules such as albumin are unable to cross.
The glomerular filtration rate (GFR) is equal to the concentration of a solute in the urine, times the volume of urine produced per minute, divided by the plasma concentration (assuming that the solute is freely diffused e.g. inulin).
In clinical practice creatinine is used because it is subjected to very little proximal tubular secretion.
Although subject to variability, the typical GFR is 125ml per minute.
Glomerular filtration rate = Total volume of plasma per unit time leaving the capillaries and entering the bowman’s capsule
Renal clearance = volume plasma from which a substance is removed per minute by the kidneys

Substances used to measure GFR have the following features:
1. Inert
2. Free filtration from the plasma at the glomerulus (not protein bound)
3. Not absorbed or secreted at the tubules
4. Plasma concentration constant during urine collection

Examples: inulin, creatinine

plasma concentration (mmol/l)

The clearance of a substance is dependent not only on its diffusivity across the basement membrane but also subsequent tubular secretion and / or reabsorption.
So glucose which is freely filtered across the basement membrane is usually reabsorbed from tubules giving a clearance of zero.

Tubular function
Reabsorption and secretion of substances occurs in the tubules.
In the proximal tubule substrates such as glucose, amino acids and phosphate are co-transported with sodium across the semi permeable membrane.
Glucose is reabsorbed by both active and passive processes. The former uses sodium–glucose co-transporters (SGLTs) in the proximal convoluted tubule. The latter achieved through facilitated and passive process through GLUT transporters.
Up to two thirds of filtered water is reabsorbed in the proximal tubules.
This will lead to increase in urea concentration in the distal tubule allowing for its increased diffusion.
Substances to be secreted into the tubules are taken up from the peritubular blood by tubular cells.
Solutes such as paraaminohippuric acid are cleared with a single passage through the kidneys and this is why it is used to measure renal plasma flow. Ions such as calcium and phosphate will have a tubular reabsorption that is influenced by plasma PTH levels.
Potassium may be both secreted and re-absorbed and is co-exchanged with sodium.

Loop of Henle
Approximately 60 litres of water containing 9000mmol sodium enters the descending limb of the loop of Henle in 24 hours.
Loops from the juxtamedullary nephrons run deep into the medulla.
The osmolarity of fluid changes and is greatest at the tip of the papilla.
The thick ascending limb is impermeable to water, but highly permeable to sodium and chloride ions.
This loss means that at the beginning of the thick ascending limb the fluid is hypo osmotic compared with adjacent interstitial fluid.
In the thick ascending limb the reabsorption of sodium and chloride ions occurs by both facilitated and passive diffusion pathways.
The loops of Henle are co-located with vasa recta, these will have similar solute compositions to the surrounding extracellular fluid so preventing the diffusion and subsequent removal of this hypertonic fluid.
The energy dependent reabsorption of sodium and chloride in the thick ascending limb helps to maintain this osmotic gradient.

1780
Q

A 70 year old lady with metastatic breast cancer is admitted to hospital confused and acutely unwell, with nausea and vomiting. Her vital signs are within normal limits. What abnormality is most likely to contribute to her clinical condition?

Hyponatraemia

Hypernatraemia

Hypocalcaemia

Hypercalcaemia

Hyperkalaemia

A

The commonest site of metastatic breast cancer is to a bony site (often the axial skeleton) and so hypercalcaemia is often present in those with metastatic disease who are acutely unwell.

Hypercalcaemia

Main causes
Malignancy (most common cause in hospital in-patients)
Primary hyperparathyroidism (commonest cause in non hospitalised patients)

Less common
Sarcoidosis (extrarenal synthesis of calcitriol )
Thiazides, lithium
Immobilisation
Pagets disease
Vitamin A/D toxicity
Thyrotoxicosis
MEN
Milk alkali syndrome

Clinical features
Stones, bones, abdominal groans, and psychic moans
High serum calcium levels result in decreased neuronal excitability. Therefore sluggish reflexes, muscle weakness and constipation may occur.

1781
Q

A 29 year old female undergoes a sub total thyroidectomy. Five days post operatively the wound becomes erythematous and discharges pus. What is the most likely causative organism?

Streptococcus pyogenes

Haemophilus influenzae

Pseudomonas aeruginosa

Staphylococcus aureus

Proteus mirabilis

A

In this setting Staphylococcus aureus Infection is the most likely cause. In the UK between 2010 and 2011 the commonest cause of wound infection was enterobacter infections (usually following cardiac or colonic surgery). 23% of infections were due to Staph aureus, which fits the scenario above. Infection with the other organisms including strep pyogenes was much rarer.

Surgical site infection

  • Surgical site infections may occur following a breach in tissue surfaces and allow normal commensals and other pathogens to initiate infection. They are a major cause of morbidity and mortality.
    Surgical site infections (SSI) comprise up to 20% of all healthcare associated infections and at least 5% of patients undergoing surgery will develop an SSI as a result.
    In many cases the organisms are derived from the patient’s own body. Measures that may increase the risk of SSI include:
    Shaving the wound using a razor (disposable clipper preferred)
    Using a non iodine impregnated incise drape if one is deemed to be necessary
    Tissue hypoxia
    Delayed administration of prophylactic antibiotics in tourniquet surgery

Preoperatively
Don’t remove body hair routinely
If hair needs removal, use electrical clippers with single use head (razors increase infection risk)
Antibiotic prophylaxis if:
- placement of prosthesis or valve
- clean-contaminated surgery
- contaminated surgery
Use local formulary
Aim to give single dose IV antibiotic on anaesthesia
If a tourniquet is to be used, give prophylactic antibiotics earlier

Intraoperatively
Prepare the skin with alcoholic chlorhexidine (Lowest incidence of SSI)
Cover surgical site with dressing
A recent meta analysis has confirmed that administration of supplementary oxygen does not reduce the risk of wound infection. In contrast to previous individual RCT’s(1)
Wound edge protectors do not appear to confer benefit (2)

Post operatively
Tissue viability advice for management of surgical wounds healing by secondary intention

Use of diathermy for skin incisions
In the NICE guidelines the use of diathermy for skin incisions is not advocated(3). Several randomised controlled trials have been undertaken and demonstrated no increase in risk of SSI when diathermy is used(4).

References
1. Brar M et al.. Perioperative supplemental oxygen in colorectal patients: a meta analysis. J Surg Res 2011 (166): 227 -235.
2. Pinkney T et al. Impact of wound edge protection devices on surgical site infection after laparotomy: impact of a multicentre randomised controlled trial (ROSSINI Trial). BMJ 2013 (347):10.
3. http://www.nice.org.uk/CG74
4. Ahmad N and Ahmed A. Meta-analysis of the effectiveness of surgical scalpel or diathermy in making abdominal skin incisions. Ann Surg 2011, 253(1):8-13.

1782
Q

A 16 year old female presents to the emergency department with a 12 hour history of pelvic discomfort. She is otherwise well and her last normal menstrual period was 2 weeks ago. On examination, she has a soft abdomen with some mild supra pubic discomfort. What is the most likely underlying cause?

Ruptured ectopic pregnancy

Degenerating fibroid

Pelvic inflammatory disease

Appendicitis

Mittelschmerz

A

Mid cycle pain is very common and is due to the small amount of fluid released during ovulation. Inflammatory markers are usually normal and the pain typically subsides over the next 24-48 hours

Gynaecological causes of abdominal pain

A number of women will present with abdominal pain and subsequently be diagnosed with a gynaecological disorder. In addition to routine diagnostic work up of abdominal pain, all female patients should also undergo a bimanual vaginal examination, urine pregnancy test and consideration given to abdominal and pelvic ultrasound scanning.
When diagnostic doubt persists a laparoscopy provides a reliable method of assessing suspected tubulo-ovarian pathology.

Differential diagnoses of abdominal pain in females
Diagnosis Features Investigation Treatment
Mittelschmerz Usually mid cycle pain.
Often sharp onset.
Little systemic disturbance.
May have recurrent episodes.
Usually settles over 24-48 hours. Full blood count- usually normal
Ultrasound- may show small quantity of free fluid Conservative
Endometriosis 25% asymptomatic, in a further 25% associated with other pelvic organ pathology.
Remaining 50% may have menstrual irregularity, infertility, pain and deep dyspareunia.
Complex disease may result in pelvic adhesional formation with episodes of intermittent small bowel obstruction.
Intra-abdominal bleeding may produce localised peritoneal inflammation.
Recurrent episodes are common. Ultrasound- may show free fluid
Laparoscopy will usually show lesions Usually managed medically, complex disease will often require surgery and some patients will even require formal colonic and rectal resections if these areas are involved
Ovarian torsion Usually sudden onset of deep seated colicky abdominal pain.
Associated with vomiting and distress.
Vaginal examination may reveal adnexial tenderness. Ultrasound may show free fluid
Laparoscopy is usually both diagnostic and therapeutic Laparoscopy
Ectopic gestation Symptoms of pregnancy without evidence of intra uterine gestation.
Present as an emergency with evidence of rupture or impending rupture.
Open tubular ruptures may have sudden onset of abdominal pain and circulatory collapse, in other the symptoms may be more prolonged and less marked.
Small amount of vaginal discharge is common.
There is usually adnexial tenderness. Ultrasound showing no intra uterine pregnancy and beta HCG that is elevated
May show intra abdominal free fluid Laparoscopy or laparotomy is haemodynamically unstable. A salphingectomy is usually performed.
Pelvic inflammatory disease Bilateral lower abdominal pain associated with vaginal discharge.
Dysuria may also be present.
Peri-hepatic inflammation secondary to Chlamydia (Fitz Hugh Curtis Syndrome) may produce right upper quadrant discomfort.
Fever >38o Full blood count- Leucocytosis
Pregnancy test negative (Although infection and pregnancy may co-exist)
Amylase - usually normal or slightly raised
High vaginal and urethral swabs

1783
Q

A 72 year old man has discharge from a healed abdomino-perineal resection wound. On examination, it has almost completely healed but there is prominent granulation tissue at the apex of the wound. There is no evidence of an underlying collection and he is otherwise well. What is the best course of action?

Excision of the area

Cryotherapy

Application of topical silver nitrate

Compression dressings

Application of alginate dressings

A

Silver nitrate will cauterise the exuberant granulation tissue and promote healing.

Methods of wound closure

Method of closure Indication
Primary closure
Clean wound, usually surgically created or following minor trauma
Standard suturing methods will usually suffice
Wound heals by primary intention
Delayed primary closure
Similar methods of actual closure to primary closure
May be used in situations where primary closure is either not achievable or not advisable e.g. infection
Vacuum assisted closure
Uses negative pressure therapy to facilitate wound closure
Sponge is inserted into wound cavity and then negative pressure applied
Advantages include removal of exudate and versatility
Disadvantages include cost and risk of fistulation if used incorrectly on sites such as bowel
Split thickness skin grafts
Superficial dermis removed with Watson knife or dermatome (commonly from thigh)
Remaining epithelium regenerates from dermal appendages
Coverage may be increased by meshing
Full thickness skin grafts
Whole dermal thickness is removed
Sub dermal fat is then removed and graft placed over donor site
Better cosmesis and flexibility at recipient site
Donor site morbidity
Flaps
Viable tissue with a blood supply
May be pedicled or free
Pedicled flaps are more reliable, but limited in range
Free flaps have greater range but carry greater risk of breakdown as they require vascular anastomosis

1784
Q

A 23 year old man presents with intermittent symptoms of altered sensation in his arm and discomfort when he uses his hand. He works as an electrician and his symptoms are worst when he is fitting light fixtures. What is the most likely cause?

Cervical rib

Axillary artery embolus

Axillary vein thrombosis

Raynaud’s

Atheroma of the subclavian artery

A

Compression of the thoracic outlet by the fibrous band of the rib can result in both neurological and circulatory compromise. When manual tasks are performed in which the hand works overhead the signs and symptoms will be maximal and this is the basis of Adsons test.

Vascular disorders of the upper limb

Upper limb arterial disease is less common than lesions causing symptoms in the lower limb. The upper limb circulation may be affected by embolic events, stenotic lesions (both internal and extrinsic), inflammatory disorders and venous diseases.
The anatomy of the collateral circulation of the arterial inflow may impact on the history and nature of disease presentation. In the region of the subclavian and axillary arteries the collateral vessels passing around the shoulder joint may provide pathways for flow if the main vessels are stenotic or occluded. During periods of increased metabolic demand the collateral flow is not sufficient and the vertebral arteries may have diminished flow. This may result in diminished flow to the brain with neurological sequelae such as syncope.

Vascular disease of the upper limb
Condition Features
Axillary/ brachial embolus
50% of upper limb emboli will lodge in the brachial artery
30% of upper limb emboli will lodge in the axillary artery
Sudden onset of symptoms; pain, pallor, paresis, pulselessness, paraesthesia
Sources are left atrium with cardiac arrhythmia (mainly AF), mural thrombus
Cardiac arrhythmias may result in impaired consciousness in addition to the embolus
Arterial occlusions
Those resulting from atheroma are the most common, trauma may result in vascular changes and long term occlusion but this is rare
Features may include claudication, ulceration and gangrene. Proximally sited lesions may result in subclavian steal syndrome
The progressive nature of the disease allows development of collaterals, acute ischaemia may occur as a result of acute thrombosis
Raynaud’s disease
Idiopathic condition affecting young females
Usually affects hands > feet
Digits become: white →blue →red
Treatment is with calcium antagonists
Upper limb venous thrombosis
Gradual onset of upper limb swelling and discomfort.
Sensation and motor function are normal
Condition may complicate pre-existing malignancy (especially breast cancer) or arise as a result of repetitive use of the limb in a task such as painting a ceiling
The condition is diagnosed with duplex ultrasound and treatment is with anticoagulation
Cervical rib
0.2-0.4% incidence
Consist of an anomalous fibrous band that often originates from C7 and may arc towards, but rarely reaches the sternum
Congenital cases may present around the third decade, some cases are reported to occur following trauma
Bilateral in up to 70%
Compression of the subclavian artery may produce absent radial pulse on clinical examination and in particular may result in a positive Adsons test (lateral flexion of the neck away from symptomatic side and traction of the symptomatic arm- leads to obliteration of radial pulse)
Treatment is most commonly undertaken when there is evidence of neurovascular compromise. A transaxillary approach is the traditional operative method for excision

1785
Q

A 40 year old man presents with a long standing inguinal hernia. On examination he has a small, direct inguinal hernia. He inquires as to the risk of strangulation over the next twelve months should he decide not to undergo surgery. Which of the following most closely matches the likely risk of strangulation over the next 12 months?

50%

40%

25%

15%

<5%

A

The annual probability of strangulation is up to 3% and is more common in indirect hernias. Elective repair poses few risks. However, emergency repair is associated with increased mortality, particularly in the elderly.

Inguinal hernia surgery

Inguinal hernias occur when the abdominal viscera protrude through the anterior abdominal wall into the inguinal canal. They may be classified as being either direct or indirect. The distinction between these two rests on their relation to Hesselbach’s triangle.

Boundaries of Hesselbach’s Triangle
Medial: Rectus abdominis
Lateral: Inferior epigastric vessels
Inferior: Inguinal ligament

Hernias occurring within the triangle tend to be direct and those outside - indirect.

Diagnosis
Most cases are diagnosed clinically, a reducible swelling may be located at the level of the inguinal canal. Large hernia’s may extend down into the male scrotum, these will not trans-illuminate and it is not possible to ‘get above’ the swelling.
Cases that are unclear on examination, but suspected from the history, may be further investigated using ultrasound or by performing a herniogram.

Treatment
Hernias associated with few symptoms may be managed conservatively. Symptomatic hernias or those which are at risk of developing complications are usually treated surgically.
First time hernias may be treated by performing an open inguinal hernia repair; the inguinal canal is opened, the hernia reduced and the defect repaired. A prosthetic mesh may be placed posterior to the cord structures to re-enforce the repair and reduce the risk of recurrence.
Recurrent hernias and those which are bilateral are generally managed with a laparoscopic approach. This may be via an intra or extra peritoneal route. As in open surgery a mesh is deployed. However, it will typically lie posterior to the deep ring.

Inguinal hernia in children
Inguinal hernias in children are almost always of an indirect type and therefore are usually dealt with by herniotomy, rather than herniorraphy. Neonatal hernias especially in those children born prematurely are at highest risk of strangulation and should be repaired urgently. Other hernias may be repaired on an elective basis.

References
The UK Based National Institute of Clinical Excellence has published guidelines relating to the choice between open and laparoscopic inguinal hernia repair. Which users may find interesting:

http://guidance.nice.org.uk/TA83/Guidance/pdf/English

1786
Q

A 22 year old man is undergoing a daycase excision of a sebaceous cyst. He is needle phobic and as the surgeon approaches with the needle the patient begins to hyperventilate. He soon develops circumoral parasthesia and muscular twitching. Which of the following is the most likely explanation for this event?

Temporal lobe epilepsy

Reduction in ionised calcium levels

Increase in ionised calcium levels

Fall in serum PTH levels

Rise in serum PTH levels

A

50% of plasma calcium is ionised. Hyperventilation will induce a state of alkalosis which will lower ionised plasma calcium levels.

Calcium homeostasis

Calcium ions are linked to a wide range of physiological processes. The largest store of bodily calcium is contained within the skeleton. Calcium levels are primarily controlled by parathyroid hormone, vitamin D and calcitonin.

Hormonal regulation of calcium
Hormone Actions
Parathyroid hormone (PTH)
Increase calcium levels and decrease phosphate levels
Increases bone resorption
Immediate action on osteoblasts to increase ca2+ in extracellular fluid
Osteoblasts produce a protein signaling molecule that activate osteoclasts which cause bone resorption
Increases renal tubular reabsorption of calcium
Increases synthesis of 1,25(OH)2D (active form of vitamin D) in the kidney which increases bowel absorption of Ca2+
Decreases renal phosphate reabsorption
1,25-dihydroxycholecalciferol (the active form of vitamin D)
Increases plasma calcium and plasma phosphate
Increases renal tubular reabsorption and gut absorption of calcium
Increases osteoclastic activity at high levels and osteoblasts at low levels
Increases renal phosphate reabsorption
Calcitonin
Secreted by C cells of thyroid
Inhibits intestinal calcium absorption
Inhibits osteoclast activity
Inhibits renal tubular absorption of calcium

Both growth hormone and thyroxine also play a small role in calcium metabolism.

1787
Q

A 20 year old male is referred to the clinic. He has undergone genetic testing because his father died from colorectal cancer at the age of 21. His testing revealed a mutation of the APC gene. A colonoscopy is proposed. What is the most likely finding?

Multiple colonic hamartomas

Carpet villous adenoma of the rectum

Caecal carcinoma

Multiple colonic adenomas

Multiple colonic hyperplastic polyps

A

APC mutations are found in familial adenomatous polyposis coli. These have multiple colonic adenomas.
Polyposis syndromes

Syndrome Genetic defect Features Screening and management Associated disorders
Familial adenomatous polyposis Mutation of APC gene (80%) cases, dominant Typically over 100 colonic adenomas
Cancer risk of 100%
20% are new mutations If known to be at risk then predictive genetic testing as teenager
Annual flexible sigmoidoscopy from 15 years
If no polyps found then 5 yearly colonoscopy started at age 20
Polyps found = resectional surgery (resection and pouch Vs sub total colectomy and IRA) Gastric fundal polyps (50%).
Duodenal polyps 90%.
If severe duodenal polyposis cancer risk of 30% at 10 years.
Abdominal desmoid tumours.
MYH associated polyposis Biallelic mutation of mut Y human homologue (MYH) on chromosome 1p, recessive Multiple colonic polyps
Later onset right sided cancers more common than in FAP
100% cancer risk by age 60 Once identified resection and ileoanal pouch reconstruction is recommended
Attenuated phenotype - regular colonoscopy Duodenal polyposis in 30%
Associated with increased risk of breast cancer (self examination)
Peutz -Jeghers syndrome STK11 (LKB1) mutation on chromosome 19 in some (but not all) cases, dominant Multiple benign intestinal hamartomas
Episodic obstruction and intussceception
Increased risk of GI cancers (colorectal cancer 20%, gastric 5%)
Increased risk of breast, ovarian, cervical pancreatic and testicular cancers Annual examination
Pan intestinal endoscopy every 2-3 years Malignancies at other sites
Classical pigmentation pattern
Cowden disease Mutation of PTEN gene on chromosome 10q22, dominant Macrocephaly
Multiple intestinal hamartomas
Multiple trichilemmomas
89% risk of cancer at any site
16% risk of colorectal cancer Targeted individualised screening Breast cancer (81% risk)
Thyroid cancer and non toxic goitre
Uterine cancer
HNPCC (Lynch syndrome) Germline mutations of DNA mismatch repair genes Colo rectal cancer 30-70%
Endometrial cancer 30-70%
Gastric cancer 5-10%
Scanty colonic polyps may be present
Colonic tumours likely to be right sided and mucinous Colonoscopy every 1-2 years from age 25
Consideration of prophylactic surgery
Extra colonic surveillance recommended Extra colonic cancers

1788
Q

From which of the following structures does the long head of the triceps muscle arise?

Coracoid process

Acromion

Infraglenoid tubercle

Coraco-acromial ligament

Coraco-humeral ligament

A

The long head arises from the infraglenoid tubercle. The fleshy lateral and medial heads are attached to the posterior aspect of the humerus between the insertion of the teres minor and the olecranon fossa.

Triceps

Origin
Long head- infraglenoid tubercle of the scapula.
Lateral head- dorsal surface of the humerus, lateral and proximal to the groove of the radial nerve
Medial head- posterior surface of the humerus on the inferomedial side of the radial groove and both of the intermuscular septae
Insertion
Olecranon process of the ulna. Here the olecranon bursa is between the triceps tendon and olecranon.
Some fibres insert to the deep fascia of the forearm, posterior capsule of the elbow (preventing the capsule from being trapped between olecranon and olecranon fossa during extension)
Innervation Radial nerve
Blood supply Profunda brachii artery
Action Elbow extension. The long head can adduct the humerus and extend it from a flexed position
Relations The radial nerve and profunda brachii vessels lie between the lateral and medial heads

1789
Q

A 45 year old man sustains a closed head injury. He is initially alert, however, his level of consciousness deteriorates on arrival at hospital. An intra cranial pressure monitor is inserted. What is the normal intracranial pressure?

35 - 45mm Hg

45 - 55mm Hg

<15mm Hg

25 - 35mm Hg

25 - 30 mm Hg

A

The normal intracranial pressure is between 7 and 15 mm Hg. The brain can accommodate increases up to 24 mm Hg, thereafter clinical features will become evident.

Applied neurophysiology

Pressure within the cranium is governed by the Monroe-Kelly doctrine. This considers the skull as a closed box. Increases in mass can be accommodated by loss of CSF. Once a critical point is reached (usually 100- 120ml of CSF lost) there can be no further compensation and ICP rises sharply. The next step is that pressure will begin to equate with MAP and neuronal death will occur. Herniation will also accompany this process.
The CNS can autoregulate its own blood supply. Vaso constriction and dilatation of the cerebral blood vessels is the primary method by which this occurs. Extremes of blood pressure can exceed this capacity resulting in risk of stroke. Other metabolic factors such as hypercapnia will also cause vasodilation, which is of importance in ventilating head injured patients.
The brain can only metabolise glucose, when glucose levels fall, consciousness will be impaired.

1790
Q

A 6 year old boy presents with pain in the hip it is present on activity and has been worsening over the past few weeks. There is no history of trauma. He was born by normal vaginal delivery at 38 weeks gestation On examination he has an antalgic gait and limitation of active and passive movement of the hip joint in all directions. C-reactive protein is mildly elevated at 10 but the white cell count is normal. What is the most likely diagnosis?

Perthes disease

Septic arthritis

Slipped upper femoral epiphysis

Developmental dysplasia of the hip

Osteoarthritis

A

Early plain x-ray changes in Perthes Disease:
Widening of the joint space.
Sub chondral linear lucency.
This is a typical presentation for Perthes disease. X-ray may show flattening of the femoral head or fragmentation in more advanced cases.

Paediatric orthopaedics

Diagnosis Mode of presentation Treatment Radiology
Developmental dysplasia of the hip Usually diagnosed in infancy by screening tests. May be bilateral, when disease is unilateral there may be leg length inequality. As disease progresses child may limp and then early onset arthritis. More common in extended breech babies. Splints and harnesses or traction. In later years osteotomy and hip realignment procedures may be needed. In arthritis a joint replacement may be needed. However, this is best deferred if possible as it will almost certainly require revision Initially no obvious change on plain films and USS gives best resolution until 3 months of age. On plain films Shentons line should form a smooth arc
Perthes Disease Hip pain (may be referred to the knee) usually occurring between 5 and 12 years of age. Bilateral disease in 20%. Remove pressure from joint to allow normal development. Physiotherapy. Usually self-limiting if diagnosed and treated promptly. X-rays will show flattened femoral head. Eventually in untreated cases the femoral head will fragment.
Slipped upper femoral epiphysis Typically seen in obese male adolescents. Pain is often referred to the knee. Limitation to internal rotation is usually seen. Knee pain is usually present 2 months prior to hip slipping. Bilateral in 20%. Bed rest and non-weight bearing. Aim to avoid avascular necrosis. If severe slippage or risk of it occurring then percutaneous pinning of the hip may be required. X-rays will show the femoral head displaced and falling inferolaterally (like a melting ice cream cone) The Southwick angle gives indication of disease severity

1791
Q

A 6 year old sustains a supracondylar fracture of the distal humerus. There are concerns that the radial nerve may have been injured. What is the relationship of the radial nerve to the humerus at this point?

Anterolateral

Anteromedial

Posterolateral

Posteromedial

Immediately anterior

A

The radial nerve lies anterolateral to the humerus in the supracondylar area.
Radial nerve

Continuation of posterior cord of the brachial plexus (root values C5 to T1)

Path
In the axilla: lies posterior to the axillary artery on subscapularis, latissimus dorsi and teres major.
Enters the arm between the brachial artery and the long head of triceps (medial to humerus).
Spirals around the posterior surface of the humerus in the groove for the radial nerve.
At the distal third of the lateral border of the humerus it then pierces the intermuscular septum and descends in front of the lateral epicondyle.
At the lateral epicondyle it lies deeply between brachialis and brachioradialis where it then divides into a superficial and deep terminal branch.
Deep branch crosses the supinator to become the posterior interosseous nerve.
Regions innervated
Motor (main nerve)
Triceps
Anconeus
Brachioradialis
Extensor carpi radialis
Motor (posterior interosseous branch)
Supinator
Extensor carpi ulnaris
Extensor digitorum
Extensor indicis
Extensor digiti minimi
Extensor pollicis longus and brevis
Abductor pollicis longus
Sensory The area of skin supplying the proximal phalanges on the dorsal aspect of the hand is supplied by the radial nerve (this does not apply to the little finger and part of the ring finger)

Muscular innervation and effect of denervation
Anatomical location Muscle affected Effect of paralysis
Shoulder Long head of triceps Minor effects on shoulder stability in abduction
Arm Triceps Loss of elbow extension
Forearm Supinator
Brachioradialis
Extensor carpi radialis longus and brevis Weakening of supination of prone hand and elbow flexion in mid prone position

1792
Q

A 73 year old man presents with a tumour at the tip of his tongue. To which of the following regions will the tumour initially metastasise?

Sub mental nodes

Ipsilateral deep cervical nodes

Tonsil

Ipsilateral superficial cervical nodes

Contralateral deep cervical nodes

A

Lymphatic drainage of the tongue

  • The lymphatic drainage of the anterior two thirds of the tongue shows only minimal communication of lymphatics across the midline, so metastasis to the ipsilateral nodes is usual.
    The lymphatic drainage of the posterior third of the tongue have communicating networks, as a result early bilateral nodal metastases are more common in this area.
    Lymphatics from the tip of the tongue usually pass to the sub mental nodes and from there to the deep cervical nodes.
    Lymphatics from the mid portion of the tongue usually drain to the submandibular nodes and then to the deep cervical nodes. Mid tongue tumours that are laterally located will usually drain to the ipsilateral deep cervical nodes, those from more central regions may have bilateral deep cervical nodal involvement.
1793
Q

A 45 year old man has recurrent colicky abdominal pain. As part of a series of investigations he undergoes a CT scan and this demonstrates a hernia lateral to the rectus muscle at the level of the arcuate line. What type of hernia is this?

Littres

Richters

Spigelian

Morgagni

Incisional

A

This is the site for a spigelian hernia.

Hernia

Hernias occur when a viscus or part of it protrudes from within its normal anatomical cavity. Specific hernias are covered under their designated titles the remainder are addressed here.

Spigelian hernia
Interparietal hernia occurring at the level of the arcuate line
Rare
May lie beneath internal oblique muscle. Usually between internal and external oblique
Equal sex distribution
Position is lateral to rectus abdominis
Both open and laparoscopic repair are possible, the former in cases of strangulation

Lumbar hernia
The lumbar triangle (through which these may occur) is bounded by:
Crest of ilium (inferiorly)
External oblique (laterally)
Latissimus dorsi (medially)
Primary lumbar herniae are rare, most are incisional hernias following renal surgery
- Direct anatomical repair with or without mesh re-enforcement is the procedure of choice

Obturator hernia
Herniation through the obturator canal
Commoner in females
Usually lies behind pectineus muscle
Elective diagnosis is unusual most will present acutely with obstruction
When presenting acutely most cases with require laparotomy or laparoscopy (and small bowel resection if indicated)

Richters hernia
Condition in which part of the wall of the small bowel (usually the anti mesenteric border) is strangulated within a hernia (of any type)
They do not present with typical features of intestinal obstruction as lumenal patency is preserved
Where vomiting is prominent it usually occurs as a result of paralytic ileus from peritonitis (as these hernias may perforate)

Incisional hernia
Occur through sites of surgical access into the abdominal cavity
Most common following surgical wound infection
To minimise following midline laparotomy Jenkins Rule should be followed and this necessitates a suture length 4x length of incision with bites taken at 1cm intervals, 1 cm from the wound edge
Repair may be performed either at open surgery or laparoscopically and a wide variety of techniques are described

Bochdalek hernia
Typically congenital diaphragmatic hernia
85% cases are located in the left hemi diaphragm
Associated with lung hypoplasia on the affected side
More common in males
Associated with other birth defects
May contain stomach
May be treated by direct anatomical apposition or placement of mesh. In infants that have severe respiratory compromise mechanical ventilation may be needed and mortality rate is high

Morgagni Hernia
Rare type of diaphragmatic hernia (approx 2% cases)
Herniation through foramen of Morgagni
Usually located on the right and tend to be less symptomatic
More advanced cases may contain transverse colon
As defects are small pulmonary hypoplasia is less common
Direct anatomical repair is performed

Umbilical hernia
Hernia through weak umbilicus
Usually presents in childhood
Often symptomatic
Equal sex incidence
95% will resolve by the age of 2 years
Surgery performed after the third birthday

Paraumbilical hernia
Usually a condition of adulthood
Defect is in the linea alba
More common in females
Multiparity and obesity are risk factors
Traditionally repaired using Mayos technique - overlapping repair, mesh may be used though not if small bowel resection is required owing to acute strangulation

Littres hernia
Hernia containing Meckels diverticulum
Resection of the diverticulum is usually required and this will preclude a mesh repair

1794
Q

A 54 year old man presents to the vascular clinic with severe rest pain and an ulcer on his right foot that is not healing. On examination he has bilateral absent femoral pulses. Imaging demonstrates a bilateral occlusion of the common iliac arteries that is not suitable for stenting. What is the most appropriate intervention?

Aorto-bifemoral bypass

Axillo-bifemoral bypass

Bilateral transfermoral amputation

Femoro-femoro cross over graft

Femoro-popliteal bypass

A

In a young patient consideration should be given to aorto-bifemoral bypass grafts as these have the best long term functional outcome compared with an axillo-bifemoral bypass graft.

Peripheral vascular disease

Indications for surgery to revascularise the lower limb
Intermittent claudication
Critical ischaemia
Ulceration
Gangrene

Intermittent claudication that is not disabling may provide a relative indication, whilst the other complaints are often absolute indications depending upon the frailty of the patient.

Assessment
Clinical examination
Ankle brachial pressure index measurement
Duplex arterial ultrasound
Angiography (standard, CT or MRI): usually performed only if intervention being considered.

Angioplasty
In order for angioplasty to be undertaken successfully the artery has to be accessible. The lesion relatively short and reasonable distal vessel runoff. Longer lesions may be amenable to sub-intimal angioplasty.

Surgery
Surgery will be undertaken where attempts at angioplasty have either failed or are unsuitable. Bypass essentially involves bypassing the affected arterial segment by utilising a graft to run from above the disease to below the disease. As with angioplasty good runoff improves the outcome.

Some key concepts with bypass surgery

Superficial femoral artery occlusion to the above knee popliteal
In the ideal scenario, vein (either in situ or reversed LSV) would be used as a conduit. However, prosthetic material has reasonable 5 year patency rates and some would advocate using this in preference to vein so that vein can be used for other procedures in the future. In general terms either technique is usually associated with an excellent outcome (if run off satisfactory).

Procedure
Artery dissected out, IV heparin 3,000 units given and then the vessels are cross clamped
Longitudinal arteriotomy
Graft cut to size and tunneled to arteriotomy sites
Anastomosis to femoral artery usually with 5/0 ‘double ended’ Prolene suture
Distal anastomosis usually using 6/0 ‘double ended’ Prolene

Distal disease
Femoro-distal bypass surgery takes longer to perform, is more technically challenging and has higher failure rates.
In elderly diabetic patients with poor runoff a primary amputation may well be a safer and more effective option. There is no point in embarking on this type of surgery in patients who are wheelchair bound.
In femorodistal bypasses vein gives superior outcomes to PTFE.

Rules
Vein mapping 1st to see whether there is suitable vein (the preferred conduit). Sub intimal hyperplasia occurs early when PTFE is used for the distal anastomosis and will lead to early graft occlusion and failure.
Essential operative procedure as for above knee fem-pop.
If there is insufficient vein for the entire conduit then vein can be attached to the end of the PTFE graft and then used for the distal anastomosis. This type of ‘vein boot’ is technically referred to as a Miller Cuff and is associated with better patency rates than PTFE alone.
Remember the more distal the arterial anastomosis the lower the success rate.

References
Peach G et al. Diagnosis and management of peripheral arterial disease. BMJ 2012; 345: 36-41.

1795
Q

A 1 day old child is born by emergency cesarean section for foetal distress. On examination, he has decreased air entry on the left side of his chest and a displaced apex beat. Abdominal examination demonstrates a scaphoid abdomen but is otherwise unremarkable. What is the most likely cause?

Pyloric stenosis

Congenital diaphragmatic hernia

Tracheo-oesophageal fistula

Ileal atresia

Meconium ileus

A

Displaced apex beat and decreased air entry are suggestive of diaphragmatic hernia. The abdomen may well be scaphoid in some cases. The underlying lung may be hypoplastic and this correlates directly with prognosis.

Paediatric Gastrointestinal disorders

Pyloric stenosis
M>F
5-10% Family history in parents
Projectile non bile stained vomiting at 4-6 weeks of life
Diagnosis is made by test feed or USS
Treatment: Ramstedt pyloromyotomy (open or laparoscopic)
Acute appendicitis
Uncommon under 3 years
When occurs may present atypically
Mesenteric adenitis
Central abdominal pain and URTI
Conservative management
Intussusception
Telescoping bowel
Proximal to or at the level of, ileocaecal valve
6-9 months age
Colicky pain, diarrhoea and vomiting, sausage shaped mass, red jelly stool.
Treatment: reduction with air insufflation
Malrotation
High caecum at the midline
Feature in exomphalos, congenital diaphragmatic hernia, intrinsic duodenal atresia
May be complicated by development of volvulus, infant with volvulus may have bile stained vomiting
Diagnosis is made by upper GI contrast study and USS
Treatment is by laparotomy, if volvulus is present (or at high risk of occurring then a ladds procedure is performed
Hirschsprung’s disease
Absence of ganglion cells from myenteric and submucosal plexuses
Occurs in 1/5000 births
Full thickness rectal biopsy for diagnosis
Delayed passage of meconium and abdominal distension
Treatment is with rectal washouts initially, thereafter an anorectal pull through procedure
Oesophageal atresia
Associated with tracheo-oesophageal fistula and polyhydramnios
May present with choking and cyanotic spells following aspiration
VACTERL associations
Meconium ileus
Usually delayed passage of meconium and abdominal distension
Majority have cystic fibrosis
X-Rays may not show a fluid level as the meconium is viscid (depends upon feeding), PR contrast studies may dislodge meconium plugs and be therapeutic
Infants who do not respond to PR contrast and NG N-acetyl cysteine will require surgery to remove the plugs
Biliary atresia
Jaundice > 14 days
Increased conjugated bilirubin
Urgent Kasai procedure
Necrotising enterocolitis
Prematurity is the main risk factor
Early features include abdominal distension and passage of bloody stools
X-Rays may show pneumatosis intestinalis and evidence of free air
Increased risk when empirical antibiotics are given to infants beyond 5 days
Treatment is with total gut rest and TPN, babies with perforations will require laparotomy

1796
Q

A 73 year old lady is recovering from recent surgery and is resting. Whilst she is resting, which of these muscles will be the major contributor to ventilatory activity?

Innermost intercostal

Diaphragm

Internal intercostal

Sternocleidomastoid

Scalene

A

The diaphragm is the major muscle involved in this process and it is for this reason that diminished activity following major abdominal surgery (note surgery type not specified here) can result in atelectasis.

Alveolar ventilation

  • Minute ventilation is the total volume of gas ventilated per minute.

MV (ml/min)= tidal volume x Respiratory rate (resps/min).

Dead space ventilation describes the volume of gas not involved in exchange in the blood.

There are 2 types:

  1. Anatomical dead space: 150mls
    Volume of gas in the respiratory tree not involved in gaseous exchange: mouth, pharynx, trachea, bronchi up to terminal bronchioles
    Measured by Fowlers method
    Increased by:
    Standing, increased size of person, increased lung volume and drugs causing bronchodilatation e.g. Adrenaline
  2. Physiological dead space: normal 150 mls, increases in ventilation/perfusion mismatch e.g. PE, COPD, hypotension
    Volume of gas in the alveoli and anatomical dead space not involved in gaseous exchange.

Alveolar ventilation is the volume of fresh air entering the alveoli per minute.

Alveolar ventilation = minute ventilation - Dead space volume

1797
Q

A 21 year old post man notices leg pain after 5 minutes walking during his round. It improves 3 minutes after stopping. Clinically, he is noted to have reduced hair of the lower limb and his calf muscles appear atrophied. There is a weak popliteal pulse, but it is still present when the knee is fully extended. What is the most likely diagnosis?

Occlusive arterial disease caused by atherosclerosis

Popliteal fossa entrapment

Cerebral vascular accident

Diabetes mellitus

Adductor canal compression syndrome

A

Adductor canal compression syndrome most commonly presents in young males and is an important differential diagnosis in men presenting with symptoms of acute limb ischaemia on exertion. It is caused by compression of the femoral artery by the musculotendinous band from adductor magnus muscle.
The treatment consists of the division of the abnormal band and restoration of the arterial circulation. Popliteal fossa entrapment is the main differential diagnosis, however the pulse disappears when the knee is fully extended.

Adductor canal

Also called Hunter’s or subsartorial canal
Immediately distal to the apex of the femoral triangle, lying in the middle third of the thigh. Canal terminates at the adductor hiatus.

Borders Contents
Laterally Vastus medialis muscle Saphenous nerve
Posteriorly Adductor longus, adductor magnus Superficial femoral artery
Roof Sartorius Superficial femoral vein

1798
Q

An individual is noted to have a left sided superior vena cava. By which pathway is blood from this system most likely to enter the heart?

Via the coronary sinus

Via the azygos venous system and into the superior vena cava

Via anomalies in the pumonary vascular bed

Via the left atrium and persistent foramen ovale

Directly into the roof of the right atrium

A

Via the coronary sinus

Persistent left superior vena cava is the most common anomaly of the thoracic venous system. It is prevalent in 0.3% of the population and is a benign entity of failed involution during embryogenesis.

Superior vena cava

Drainage
Head and neck
Upper limbs
Thorax
Part of abdominal walls

Formation
Subclavian and internal jugular veins unite to form the right and left brachiocephalic veins
These unite to form the SVC
Azygos vein joins the SVC before it enters the right atrium

Relations
Anterior Anterior margins of the right lung and pleura
Posteromedial Trachea and right vagus nerve
Posterolateral Posterior aspects of right lung and pleura
Pulmonary hilum is posterior
Right lateral Right phrenic nerve and pleura
Left lateral Brachiocephalic artery and ascending aorta

Developmental variations
Anomalies of the connection of the SVC are recognised. In some individuals a persistent left sided SVC drains into the right atrium via an enlarged orifice of the coronary sinus. More rarely the left sided vena cava may connect directly with the superior aspect of the left atrium, usually associated with an un-roofing of the coronary sinus. The commonest lesion of the IVC is for its abdominal course to be interrupted, with drainage achieved via the azygos venous system. This may occur in patients with left sided atrial isomerism.

1799
Q

A 6 year old is hit by a car and is brought to the emergency department. He is haemodynamically unstable with bilateral femoral shaft fractures and concerns that he may have a ruptured spleen. The parents have refused blood transfusions on religious grounds. What is the correct course of action?

Proceed with treatment

Do not proceed with treatment

Apply to court

Apply for an IMCA

Contact the PALS service

A

In the UK, the GMC and common law advises that emergency life saving treatment can be given to a child irrespective of the parents views. There is clearly insufficient time here to apply to a court.

Consent

There are 3 types of consent:

  1. Informed
  2. Expressed
  3. Implied

Consent forms used in UK NHS
Consent Form 1 For competent adults who are able to consent for themselves where consciousness may be impaired (e.g. GA)
Consent Form 2 For an adult consenting on behalf of a child where consciousness is impaired
Consent Form 3 For an adult or child where consciousness is not impaired
Consent Form 4 For adults who lack capacity to provide informed consent

Capacity
Key points include:
1. Understand and retain information
2. Patient believes the information to be true
3. Patient is able to weigh the information to make a decision
All patients must be assumed to have capacity

Consent in minors
Young children and older children who are not Gillick competent cannot consent for themselves. In British law the patients biological mother can always provide consent. The child’s father can consent if the parents are married (and the father is the biological father), or if the father is named on the birth certificate (irrespective of marital status). If parents are not married and the father is not named on the birth certificate then the father cannot consent.

1800
Q

A 53 year old man presents with dyspepsia. At upper GI endoscopy he has a punched out ulcer on the lesser curve of the stomach. It measures approximately 2cm in diameter and is seen to penetrate muscle with fibrosis present at the base. What is the most likely diagnosis?

Curlings ulcer

Cushings ulcer

Chronic ulcer

Adenocarcinoma

Gastric lymphoma

A

Fibrosis is usually a sign of chronic ulcer. It should be biopsied carefully, a proton pump inhibitor started and re endoscopy should occur at 6 weeks.

Upper gastrointestinal bleeding

Patients may present with the following:
Haematemesis and/ or malaena
Epigastric discomfort
Sudden collapse

The extent to which these will occur will depend upon the source. Mortality is higher in patients presenting with haematemesis than malaena alone.

Oesophageal bleeding
Cause Presenting features
Oesophagitis Small volume of fresh blood, often streaking vomit. Malaena rare. Often ceases spontaneously. Usually history of antecedent GORD type symptoms.
Cancer Usually small volume of blood, except as pre terminal event with erosion of major vessels. Often associated symptoms of dysphagia and constitutional symptoms such as weight loss. May be recurrent until malignancy managed.
Mallory Weiss Tear Typically brisk small to moderate volume of bright red blood following bout of repeated vomiting. Malaena rare. Usually ceases spontaneously.
Varices Usually large volume of fresh blood. Swallowed blood may cause malaena. Often associated with haemodynamic compromise. May stop spontaneously but re-bleeds are common until appropriately managed.

Gastric Bleeding
Cause Presenting features
Gastric cancer May be frank haematemesis or altered blood mixed with vomit. Usually prodromal features of dyspepsia and may have constitutional symptoms. Amount of bleeding variable but erosion of major vessel may produce considerable haemorrhage.
Dieulafoy Lesion Often no prodromal features prior to haematemesis and malaena, but this arteriovenous malformation may produce quite considerable haemorrhage and may be difficult to detect endoscopically.
Diffuse erosive gastritis Usually haematemesis and epigastric discomfort. Usually there is an underlying cause such as recent NSAID usage. Large volume haemorrhage may occur with considerable haemodynamic compromise.
Gastric ulcer Small low volume bleeds more common so would tend to present as iron deficiency anaemia. Erosion into a significant vessel may produce considerable haemorrhage and haematemesis.

Duodenum
Most common cause of major haemorrhage is a posteriorly sited duodenal ulcer. However, ulcers at any site in the duodenum may present with haematemesis, malaena and epigastric discomfort. The pain of duodenal ulcer is slightly different to that of gastric ulcers and often occurs several hours after eating. Peri ampullary tumours may bleed but these are rare. In patients with previous abdominal aortic aneurysm surgery aorto-enteric fistulation remains a rare but important cause of major haemorrhage associated with high mortality.

Management
Admission to hospital careful monitoring, cross match blood, check FBC, LFTs, U+E and Clotting (as a minimum)
Patients with on-going bleeding and haemodynamic instability are likely to require O negative blood pending cross matched blood
Early control of airway is vital (e.g. Drowsy patient with liver failure)
Patients with suspected varices should receive terlipressin prior to endoscopy
Ideally all patients admitted with upper gastrointestinal haemorrhage should undergo Upper GI endoscopy within 24 hours of admission. In those who are unstable this should occur immediately after resuscitation or in tandem with it. The endoscopy department is a potentially dangerous place for unstable patients and it may be safer to perform the endoscopy in theatre with an anaesthetist present.
Varices should be banded or subjected to sclerotherapy. If this is not possible owing to active bleeding then a Sengstaken- Blakemore tube (or Minnesota tube) should be inserted. This should be done with care; gastric balloon should be inflated first and oesophageal balloon second. Remember the balloon will need deflating after 12 hours (ideally sooner) to prevent necrosis. Portal pressure should be lowered by combination of medical therapy +/- TIPSS.
Patients with erosive oesophagitis / gastritis should receive a proton pump inhibitor.
Mallory Weiss tears will typically resolve spontaneously
Identifiable bleeding points should receive combination therapy of injection of adrenaline and either a thermal or mechanical treatment. All who have received intervention should receive a continuous infusion of a proton pump inhibitor (IV omeprazole for 72 hours) to reduce the re-bleeding rate.
Patients with diffuse erosive gastritis who cannot be managed endoscopically and continue to bleed may require gastrectomy
Bleeding ulcers that cannot be controlled endoscopically may require laparotomy and ulcer underruning

Indications for surgery
Patients > 60 years
Continued bleeding despite endoscopic intervention
Recurrent bleeding
Known cardiovascular disease with poor response to hypotension

Surgery
Duodenal ulcer
Laparotomy, duodenotomy and under running of the ulcer. If bleeding is brisk then the ulcer is almost always posteriorly sited and will have invaded the gastroduodenal artery. Large bites using 0 Vicryl are taken above and below the ulcer base to occlude the vessel. The duodenotomy should be longitudinal but closed transversely to avoid stenosis.

For gastric ulcer
Under-running of the bleeding site
Partial gastrectomy-antral ulcer
Partial gastrectomy or under running the ulcer- lesser curve ulcer (involving left gastric artery)
Total gastrectomy if bleeding persists

Summary of Acute Upper GI bleeding recommendations:
The need for admission and timing of endoscopic intervention may be predicted by using the Blatchford score. This considers a patients Hb, serum urea, pulse rate and blood pressure. Those patients with a score of 0 are low risk, all others are considered high risk and require admission and endoscopy.
The requirement for pre endoscopic proton pump inhibition is contentious. In the UK the National Institute of Clinical Excellence guidelines suggest the pre endoscopic PPI therapy is unnecessary. Whilst it is accepted that such treatment has no impact on mortality or morbidity a Cochrane review of this practice in 2007 did suggest that it reduced the stigmata of recent haemorrhage at endoscopy. As a result many will still administer PPI to patients prior to endoscopic intervention.
Following endoscopy it is important to calculate the Rockall score for patients to determine their risk of rebleeding and mortality. A score of 3 or less is associated with a rebleeding rate of 4% and a very low risk of mortality and identifies a group of patients suitable for early discharge.

References
1. http://www.sign.ac.uk/guidelines/fulltext/105/index.html
2. Joint Advisory Group on Endoscopy (JAG) Guidelines - http://www.thejag.org.uk
3. NICE Guideline: Management of acute upper GI bleeding. July 2012.

1801
Q

A 53 year old man has an arterial blood gas sample taken and the following results are obtained, he is breathing room air.
pH 7.49
pO2 8.5
HCO3 22
pCO2 2.4
Cl- <10meq
Which of the conditions listed below is most likely to account for these findings?

Respiratory alkalosis

Type 2 respiratory failure

Metabolic acidosis with increased anion gap

Metabolic alkalosis

Metabolic acidosis with normal anion gap

A

The hyperventilation results in decreased carbon dioxide levels, causing a respiratory alkalosis (non compensated).

1802
Q

Which of the following amino acids is present in all types of collagen?

Alanine

Aspartime

Glycine

Tyrosine

Cysteine

A

Collagen has a generic structure of Glycine- X- Y, where X and Y are variable sub units. The relatively small size of the glycine molecule enables collagen to form a tight helical structure.

Collagen

Collagen is one of the most important structural proteins within the extracellular matrix, collagen together with components such as elastin and glycosaminoglycans determine the properties of all tissues.
Composed of 3 polypeptide strands that are woven into a helix, usually a combination of glycine with either proline or hydroxyproline plus another amino acid
Numerous hydrogen bonds exist within molecule to provide additional strength
Many sub types but commonest sub type is I (90% of bodily collagen), tissues with increased levels of flexibility have increased levels of type III collagen
Vitamin c is important in establishing cross links
Synthesised by fibroblasts

Collagen Diseases
Disorders of collagen range from relatively common, acquired defects (typically aging), through to rarer congenital disorders. The latter are exemplified by conditions such as osteogenesis imperfecta and Ehlers Danlos syndromes.

Osteogenesis imperfecta:
-8 Subtypes
-Defect of type I collagen
-In type I the collagen is normal quality but insufficient quantity
-Type II- poor quantity and quality
-Type III- Collagen poorly formed, normal quantity
-Type IV- Sufficient quantity but poor quality
Patients have bones which fracture easily, loose joint and multiple other defects depending upon which sub type they suffer from.

Ehlers Danlos:
-Multiple sub types
-Abnormality of types 1 and 3 collagen
-Patients have features of hypermobility.
-Individuals are prone to joint dislocations and pelvic organ prolapse. In addition to many other diseases related to connective tissue defects.

1803
Q

A man is admitted after a period of prolonged self, induced starvation. Naso gastric feeding is planned. Which of the following is least likely to occur?

Hypokalaemia

Increased risk of cardiac arrhythmias

His haemoglobin will have decreased affinity for oxygen

Hypophosphataemia

Hypoalbuminaemia

A

The process of starvation may lower DPG levels, in practice this is unlikely to occur early as it is generated during glycolysis. Altered metabolism in starvation may be more acidotic and this would also tend to impair oxygen carriage.

Nutrition - Refeeding syndrome

Refeeding syndrome describes the metabolic abnormalities which occur on feeding a person following a period of starvation. The metabolic consequences include:
Hypophosphataemia
Hypokalaemia
Hypomagnesaemia
Abnormal fluid balance
These abnormalities can lead to organ failure.

Re-feeding problems
If patient not eaten for > 5 days, aim to re-feed at < 50% energy and protein levels

High risk for re-feeding problems
If one or more of the following:
BMI < 16 kg/m2
Unintentional weight loss >15% over 3-6 months
Little nutritional intake > 10 days
Hypokalaemia, Hypophosphataemia or hypomagnesaemia prior to feeding (unless high)

If two or more of the following:
BMI < 18.5 kg/m2
Unintentional weight loss > 10% over 3-6 months
Little nutritional intake > 5 days
History of: alcohol abuse, drug therapy including insulin, chemotherapy, diuretics and antacids

Prescription
Start at up to 10 kcal/kg/day increasing to full needs over 4-7 days
Start immediately before and during feeding: oral thiamine 200-300mg/day, vitamin B co strong 1 tds and supplements
Give K+ (2-4 mmol/kg/day), phosphate (0.3-0.6 mmol/kg/day), magnesium (0.2-0.4 mmol/kg/day)

1804
Q

A 34 year old man is shot in the postero- inferior aspect of his thigh. Which of the following lies at the most lateral aspect of the popliteal fossa?

Popliteal artery

Popliteal vein

Common peroneal nerve

Tibial nerve

Small saphenous vein

A

The contents of the popliteal fossa are (from medial to lateral):
Popliteal artery
Popliteal vein
Tibial nerve
Common peroneal nerve

The sural nerve is a branch of the tibial nerve and usually arises at the inferior aspect of the popliteal fossa. However, its anatomy is variable.

Popliteal fossa

Boundaries of the popliteal fossa
Laterally Biceps femoris above, lateral head of gastrocnemius and plantaris below
Medially Semimembranosus and semitendinosus above, medial head of gastrocnemius below
Floor Popliteal surface of the femur, posterior ligament of knee joint and popliteus muscle
Roof Superficial and deep fascia

Contents
Popliteal artery and vein
Small saphenous vein
Common peroneal nerve
Tibial nerve
Posterior cutaneous nerve of the thigh
Genicular branch of the obturator nerve
Lymph nodes

1805
Q

A 53 year old man presents with a mass lesion slightly inferior to the tragus of his right ear. An FNA and USS are performed and a 2cm pleomorphic adenoma is diagnosed. What is the most appropriate course of action?

USS guided core biopsy

Radical parotidectomy

Superficial parotidectomy

Discharge

MRI scanning of the region

A

Pleomorphic adenoma of the parotid= surgical excision
Pleomorphic adenomas a usually benign tumours. However, they will enlarge over time and a proportion can undergo malignant transformation. Therefore, all pleomorphic adenomas are excised and a superficial parotidectomy is generally the procedure of choice. The facial nerve is preserved. More recently, there has been a move towards partial superficial parotidectomy. However, complete resection of the lesion is still mandatory.

Parotid gland clinical

Benign neoplasms
Up to 80% of all salivary gland tumours occur in the parotid gland and up to 80% of these are benign. There is no consistent correlation between the rate of growth and the malignant potential of the lesion. However, benign tumours should not invade structures such as the facial nerve.
With the exception of Warthins tumours, they are commoner in women than men. The median age of developing a lesion is in the 5th decade of life.

Benign tumour types
Tumour type Features
Benign pleomorphic adenoma or benign mixed tumor Most common parotid neoplasm (80%)
Proliferation of epithelial and myoepithelial cells of the ducts and an increase in stromal components
Slow growing, lobular, and not well encapsulated
Recurrence rate of 1-5% with appropriate excision (parotidectomy)
Recurrence possibly secondary to capsular disruption during surgery
Malignant degeneration occurring in 2-10% of adenomas observed for long periods, with carcinoma ex-pleomorphic adenoma occurring most frequently as adenocarcinoma
Warthin tumor (papillary cystadenoma lymphoma or adenolymphoma) Second most common benign parotid tumor (5%)
Most common bilateral benign neoplasm of the parotid
Marked male as compared to female predominance
Occurs later in life (sixth and seventh decades)
Presents as a lymphocytic infiltrate and cystic epithelial proliferation
May represent heterotopic salivary gland epithelial tissue trapped within intraparotid lymph nodes
Incidence of bilaterality and multicentricity of 10%
Malignant transformation rare (almost unheard of)
Monomorphic adenoma Account for less than 5% of tumours
Slow growing
Consist of only one morphological cell type (hence term mono)
Include; basal cell adenoma, canalicular adenoma, oncocytoma, myoepitheliomas
Haemangioma Should be considered in the differential of a parotid mass in a child
Accounts for 90% of parotid tumours in children less than 1 year of age
Hypervascular on imaging
Spontaneous regression may occur and malignant transformation is almost unheard of

Malignant salivary gland tumours
Types of malignancy

Mucoepidermoid carcinoma 30% of all parotid malignancies
Usually low potential for local invasiveness and metastasis (depends mainly on grade)
Adenoid cystic carcinoma Unpredictable growth pattern
Tendency for perineural spread
Nerve growth may display skip lesions resulting in incomplete excision
Distant metastasis more common (visceral rather than nodal spread)
5 year survival 35%
Mixed tumours Often a malignancy occurring in a previously benign parotid lesion
Acinic cell carcinoma Intermediate grade malignancy
May show perineural invasion
Low potential for distant metastasis
5 year survival 80%
Adenocarcinoma Develops from secretory portion of gland
Risk of regional nodal and distant metastasis
5 year survival depends upon stage at presentation, may be up to 75% with small lesions with no nodal involvement
Lymphoma Large rubbery lesion, may occur in association with Warthins tumours
Diagnosis should be based on regional nodal biopsy rather than parotid resection
Treatment is with chemotherapy (and radiotherapy)

Diagnostic evaluation
Plain x-rays may be used to exclude calculi
Sialography may be used to delineate ductal anatomy
FNAC is used in most cases
Superficial parotidectomy may be either diagnostic or therapeutic depending upon the nature of the lesion
Where malignancy is suspected the primary approach should be definitive resection rather than excisional biopsy
CT/ MRI may be used in cases of malignancy for staging primary disease

Treatment
For nearly all lesions this consists of surgical resection, for benign disease this will usually consist of a superficial parotidectomy. For malignant disease a radical or extended radical parotidectomy is performed. The facial nerve is included in the resection if involved. The need for neck dissection is determined by the potential for nodal involvement.

Other parotid disorders
HIV infection
Lymphoepithelial cysts associated with HIV occur almost exclusively in the parotid
Typically presents as bilateral, multicystic, symmetrical swelling
Risk of malignant transformation is low and management usually conservative

Sjogren syndrome
Autoimmune disorder characterised by parotid enlargement, xerostomia and keratoconjunctivitis sicca
90% of cases occur in females
Second most common connective tissue disorder
Bilateral, non tender enlargement of the gland is usual
Histologically, the usual findings are of a lymphocytic infiltrate in acinar units and epimyoepithelial islands surrounded by lymphoid stroma
Treatment is supportive
There is an increased risk of subsequent lymphoma

Sarcoid
Parotid involvement occurs in 6% of patients with sarcoid
Bilateral in most cases
Gland is not tender
Xerostomia may occur
Management of isolated parotid disease is usually conservative

1806
Q

Which of the following statements relating to the vertebral column is false?

There are 7 cervical vertebrae

The cervical and lumbar lordosis are secondary curves developing after birth due to change in shape of the intervertebral discs

The lumbar vertebrae do not have a transverse process foramina

The lumbar vertebrae receive blood directly from the aorta

The spinous process is formed by the junction of the pedicles posteriorly

A

The spinous process is formed by 2 laminae posteriorly.

Vertebral column

There are 7 cervical, 12 thoracic, 5 lumbar, and 5 sacral vertebrae.
The spinal cord segmental levels do not necessarily correspond to the vertebral segments. For example, while the C1 cord is located at the C1 vertebra, the C8 cord is situated at the C7 vertebra. While the T1 cord is situated at the T1 vertebra, the T12 cord is situated at the T8 vertebra. The lumbar cord is situated between T9 and T11 vertebrae. The sacral cord is situated between the T12 to L2 vertebrae.

Cervical vertebrae
The interface between the first and second vertebra is called the atlanto-axis junction. The C3 cord contains the phrenic nucleus.
Muscle Nerve root value
Deltoid C5,6
Biceps C5,6
Wrist extensors C6-8
Triceps C6-8
Wrist flexors C6-T1
Hand muscles C8-T1

Thoracic vertebrae
The thoracic vertebral segments are defined by those that have a rib. The spinal roots form the intercostal nerves that run on the bottom side of the ribs and these nerves control the intercostal muscles and associated dermatomes.

Lumbosacral vertebrae
Form the remainder of the segments below the vertebrae of the thorax. The lumbosacral spinal cord, however, starts at about T9 and continues only to L2. It contains most of the segments that innervate the hip and legs, as well as the buttocks and anal regions.

Cauda Equina
The spinal cord ends at L1-L2 vertebral level. The tip of the spinal cord is called the conus. Below the conus, there is a spray of spinal roots that is called the cauda equina. Injuries below L2 represent injuries to spinal roots rather than the spinal cord proper.

1807
Q

A 23 year old man is stabbed in the chest approximately 10cm below the right nipple. In the emergency department a abdominal ultrasound scan shows a large amount of intraperitoneal blood. Which of the following statements relating to the likely site of injury is untrue?

Part of its posterior surface is devoid of peritoneum.

The quadrate lobe is contained within the functional right lobe.

Its nerve supply is from the coeliac plexus.

The hepatic flexure of the colon lies posterio-inferiorly.

The right kidney is closely related posteriorly.

A

The right lobe of the liver is the most likely site of injury. Therefore the answer is B as the quadrate lobe is functionally part of the left lobe of the liver. The liver is largely covered in peritoneum. Posteriorly there is an area devoid of peritoneum (the bare area of the liver). The right lobe of the liver has the largest bare area (and is larger than the left lobe).

Liver

Structure of the liver
Right lobe
Supplied by right hepatic artery
Contains Couinaud segments V to VIII (-/+Sg I)
Left lobe
Supplied by the left hepatic artery
Contains Couinaud segments II to IV (+/- Sg1)
Quadrate lobe
Part of the right lobe anatomically, functionally is part of the left
Couinaud segment IV
Porta hepatis lies behind
On the right lies the gallbladder fossa
On the left lies the fossa for the umbilical vein
Caudate lobe
Supplied by both right and left hepatic arteries
Couinaud segment I
Lies superior to the porta hepatis
Anterior and medial to the inferior vena cava
Bile from the caudate lobe drains into both right and left hepatic ducts

Detailed knowledge of Couinaud segments is not required for MRCS
Between the liver lobules are portal canals which contain the portal triad: Hepatic Artery, Portal Vein, tributary of Bile Duct.

Relations of the liver
Anterior Postero inferiorly
Diaphragm Oesophagus
Xiphoid process Stomach
Duodenum
Hepatic flexure of colon
Right kidney
Gallbladder
Inferior vena cava

Porta hepatis
Location Postero inferior surface, it joins nearly at right angles with the left sagittal fossa, and separates the caudate lobe behind from the quadrate lobe in front
Transmits
Common hepatic duct
Hepatic artery
Portal vein
Sympathetic and parasympathetic nerve fibres
Lymphatic drainage of the liver (and nodes)

Ligaments
Falciform ligament
2 layer fold peritoneum from the umbilicus to anterior liver surface
Contains ligamentum teres (remnant umbilical vein)
On superior liver surface it splits into the coronary and left triangular ligaments
Ligamentum teres Joins the left branch of the portal vein in the porta hepatis
Ligamentum venosum Remnant of ductus venosus

Arterial supply
Hepatic artery

Venous
Hepatic veins
Portal vein

Nervous supply
Sympathetic and parasympathetic trunks of coeliac plexus

1808
Q

A 67 year old man is undergoing a transurethral resection of a bladder tumour using diathermy. Suddenly during the procedure the patient’s thigh begins to twitch. Stimulation of which of the following nerves is the most likely cause?

Femoral

Pudendal

Sciatic

Obturator

Gluteal

A

The obturator nerve is most closely related to the bladder
Obturator nerve

The obturator nerve arises from L2, L3 and L4 by branches from the ventral divisions of each of these nerve roots. L3 forms the main contribution and the second lumbar branch is occasionally absent. These branches unite in the substance of psoas major, descending vertically in its posterior part to emerge from its medial border at the lateral margin of the sacrum. It then crosses the sacroiliac joint to enter the lesser pelvis, it descends on obturator internus to enter the obturator groove. In the lesser pelvis the nerve lies lateral to the internal iliac vessels and ureter, and is joined by the obturator vessels lateral to the ovary or ductus deferens.

Supplies
Medial compartment of thigh
Muscles supplied: external obturator, adductor longus, adductor brevis, adductor magnus (not the lower part-sciatic nerve), gracilis
The cutaneous branch is often absent. When present, it passes between gracilis and adductor longus near the middle part of the thigh, and supplies the skin and fascia of the distal two thirds of the medial aspect.

Obturator canal
Connects the pelvis and thigh: contains the obturator artery, vein, nerve which divides into anterior and posterior branches.

1809
Q

A 25 year old man has an inguinal hernia, which of the following structures must be divided (at open surgery) to gain access to the inguinal canal?

Transversalis fascia

External oblique aponeurosis

Conjoint tendon

Rectus abdominis

Inferior epigastric artery

A

This question is asking what structure forms the anterior wall of the inguinal canal. The anterior wall is formed by the external oblique aponeurosis. Once this is divided the canal is entered, the cord can be mobilised and a hernia repair performed. The transversalis fascia and conjoint tendons form the posterior wall and would not routinely be divided to gain access to the inguinal canal itself.

External oblique muscle

External oblique forms the outermost muscle of the three muscles comprising the anterolateral aspect of the abdominal wall. Its aponeurosis comprises the anterior wall of the inguinal canal.

Origin Outer surfaces of the lowest eight ribs
Insertion
Anterior two thirds of the outer lip of the iliac crest.
The remainder becomes the aponeurosis that fuses with the linea alba in the midline.
Nerve supply Ventral rami of the lower six thoracic nerves
Actions Contains the abdominal viscera, may contract to raise intra abdominal pressure. Moves trunk to one side.

1810
Q

Which of the following changes are most likely to be identified in the aortic wall of a 38 year old lady with a Marfans syndrome and a dissecting aortic aneurysm?

Transmural aortitis

Cystic medial necrosis

Foamy macrophages

Dense dystrophic calcification

None of the above

A

Cystic medial necrosis ( or cystic medial degeneration) occurs when basophils and mucoid material lie in between the intimal elastic fibres of the aorta. It is typically found in the aortic degeneration of Marfans syndrome, but may also be seen in aortic degeneration in older adults.

Aortic dissection

More common than rupture of the abdominal aorta
33% of patients die within the first 24 hours, and 50% die within 48 hours if no treatment received
Associated with hypertension
Features of aortic dissection: tear in the intimal layer, followed by formation and propagation of a subintimal hematoma. Cystic medial necrosis (Marfan’s)
Most common site of dissection: 90% occurring within 10 centimetres of the aortic valve

Stanford Classification
Type Location Treatment
A Ascending aorta/ aortic root Surgery- aortic root replacement
B Descending aorta Medical therapy with antihypertensives

DeBakey classification
Type Site affected
I Ascending aorta, aortic arch, descending aorta
II Ascending aorta only
III Descending aorta distal to left subclavian artery

Clinical features
Tearing, sudden onset chest pain (painless 10%)
Hypertension or Hypotension
A blood pressure difference (in each arm) greater than 20 mm Hg
Neurologic deficits (20%)

Investigations
CXR: widened mediastinum, abnormal aortic knob, ring sign, deviation of the trachea/oesophagus
CT angiography of the thoracic aorta
MRI angiography
Conventional angiography (now rarely used diagnostically)

Management
Beta-blockers: aim HR 60-80 bpm and systolic BP 100-120 mm Hg
For type A dissections the standard of care is aortic root replacement

1811
Q

Which of the following events is most classically seen in the arterioles of patients with malignant hypertension?

Caseous necrosis

Coagulative necrosis

Fibrinoid necrosis

Apoptosis

Colliquative necrosis

A

Fibrinoid necrosis is seen in the arterioles of patients with malignant hypertension.

Cell death

Cells can die via two mechanisms; necrosis and apoptosis. These are outlined below:

Necrosis
Necrosis is characterised by bioenergetics failure. Loss of tissue perfusion results in hypoxia and an inability to generate ATP. The integrity of the cellular membrane is lost and the loss of ATP results in loss of energy dependent cellular transport mechanisms. There is an influx of water, ionic instability and cellular lysis. The release of intracellular contents may stimulate an inflammatory response. Several types of necrosis are recognised; coagulative, colliquative, caseous, gangrene, fibrinoid and fat necrosis. The type of tissue and the underlying cause determine the predominant necrosis pattern.

Coagulative necrosis
The commonest type, occurs in most organs
Tissue is initially firm, later becomes soft as tissue is digested by macrophages
In the early phases the histological appearances may demonstrate little change
In later stages cellular outlines are seen with loss of intracellular detail

Colliquative necrosis
Occurs in tissues with no supporting stroma
Dominant necrosis pattern in the CNS
Necrotic site may eventually become encysted

Caseous necrosis
No definable structure seen in the necrotic tissue
Amorphous eosinophilic tissue may be seen histologically
Classically seen in tuberculosis

Gangrene
Necrosis with putrefaction of tissue
May complicate ischaemia
Haemoglobin degenerates and results in the deposition of iron sulphide (which is why the tissue is black)
Both wet and dry gangrene may occur, in wet gangrene there is often a liquefactive component and super-added infection (which usually smells!)

Fibrinoid necrosis
Classically seen in arterioles in patients with hypertension (malignant type)
Necrosis of the smooth muscle wall occurs and plasma may extravasate into the media with fibrin deposition

Fat necrosis
Direct trauma to fat can result in rupture of adipocytes
Lipids incite a local inflammatory reaction
Inflammatory cells phagocytose the lipid with eventual fibrosis

Apoptosis
Also known as programmed cell death
Energy dependent pathways are activated via a number of intracellular signalling mechanisms
It is the result of the activation of caspases triggered by the bcl-2 family or the binding of the FAS ligand to its receptor
DNA fragments, mitochondrial function ceases, nuclear and cellular shrinkage occurs
Phagocytosis of the cell does not occur, instead the cell degenerates into apoptotic bodies

1812
Q

Which substance can be used to achieve the most accurate measurement of the glomerular filtration rate?

Glucose

Protein

Inulin

Creatinine

Para-amino hippuric acid

A

Creatinine declines with age due to decline in renal function and muscle mass. Glucose, protein (amino acids) and PAH are reabsorbed by the kidney.

Renal Physiology

Overview
Each nephron is supplied with blood from an afferent arteriole that opens onto the glomerular capillary bed.
Blood then flows to an efferent arteriole, supplying the peritubular capillaries and medullary vasa recta.
The kidney receives up to 25% of resting cardiac output.

Control of blood flow
The kidney is able to autoregulate its blood flow between systolic pressures of 80- 180mmHg so there is little variation in renal blood flow.
This is achieved by myogenic control of arteriolar tone, both sympathetic input and hormonal signals (e.g. renin) are responsible.

Glomerular structure and function
Blood inside the glomerulus has considerable hydrostatic pressure.
The basement membrane has pores that will allow free diffusion of smaller solutes, larger negatively charged molecules such as albumin are unable to cross.
The glomerular filtration rate (GFR) is equal to the concentration of a solute in the urine, times the volume of urine produced per minute, divided by the plasma concentration (assuming that the solute is freely diffused e.g. inulin).
In clinical practice creatinine is used because it is subjected to very little proximal tubular secretion.
Although subject to variability, the typical GFR is 125ml per minute.
Glomerular filtration rate = Total volume of plasma per unit time leaving the capillaries and entering the bowman’s capsule
Renal clearance = volume plasma from which a substance is removed per minute by the kidneys

Substances used to measure GFR have the following features:
1. Inert
2. Free filtration from the plasma at the glomerulus (not protein bound)
3. Not absorbed or secreted at the tubules
4. Plasma concentration constant during urine collection

Examples: inulin, creatinine

plasma concentration (mmol/l)

The clearance of a substance is dependent not only on its diffusivity across the basement membrane but also subsequent tubular secretion and / or reabsorption.
So glucose which is freely filtered across the basement membrane is usually reabsorbed from tubules giving a clearance of zero.

Tubular function
Reabsorption and secretion of substances occurs in the tubules.
In the proximal tubule substrates such as glucose, amino acids and phosphate are co-transported with sodium across the semi permeable membrane.
Glucose is reabsorbed by both active and passive processes. The former uses sodium–glucose co-transporters (SGLTs) in the proximal convoluted tubule. The latter achieved through facilitated and passive process through GLUT transporters.
Up to two thirds of filtered water is reabsorbed in the proximal tubules.
This will lead to increase in urea concentration in the distal tubule allowing for its increased diffusion.
Substances to be secreted into the tubules are taken up from the peritubular blood by tubular cells.
Solutes such as paraaminohippuric acid are cleared with a single passage through the kidneys and this is why it is used to measure renal plasma flow. Ions such as calcium and phosphate will have a tubular reabsorption that is influenced by plasma PTH levels.
Potassium may be both secreted and re-absorbed and is co-exchanged with sodium.

Loop of Henle
Approximately 60 litres of water containing 9000mmol sodium enters the descending limb of the loop of Henle in 24 hours.
Loops from the juxtamedullary nephrons run deep into the medulla.
The osmolarity of fluid changes and is greatest at the tip of the papilla.
The thick ascending limb is impermeable to water, but highly permeable to sodium and chloride ions.
This loss means that at the beginning of the thick ascending limb the fluid is hypo osmotic compared with adjacent interstitial fluid.
In the thick ascending limb the reabsorption of sodium and chloride ions occurs by both facilitated and passive diffusion pathways.
The loops of Henle are co-located with vasa recta, these will have similar solute compositions to the surrounding extracellular fluid so preventing the diffusion and subsequent removal of this hypertonic fluid.
The energy dependent reabsorption of sodium and chloride in the thick ascending limb helps to maintain this osmotic gradient.

1813
Q

A 74-year-old woman with thyroid cancer is admitted due to shortness of breath. What is the best investigation to assess for possible compression of the upper airways?

Arterial blood gases

Forced vital capacity

Transfer factor

Peak expiratory flow rate

Flow volume loop

A

Flow volume loop is the best investigation to assess for possible compression of the upper airways. It is a graphical representation of the rate and volume of air flow during inhalation and exhalation, which can reveal any obstruction or narrowing of the airways. In the case of upper airway compression, the flow volume loop will show a flattened expiratory curve and decreased peak expiratory flow rate. Arterial blood gases, forced vital capacity, and transfer factor are not useful in assessing upper airway compression.

Flow volume loop

A normal flow volume loop is often described as a ‘triangle on top of a semi circle’

Flow volume loops are the most suitable way of assessing compression of the upper airway

1814
Q

Which of the following is true regarding the Salmonella species?

Rose spots appear in all patients with typhoid

They are normally present in the gut as commensals

Subsequent chronic biliary infection occurs in 75% of cases

A relative bradycardia is often seen in typhoid fever

Salmonella typhi can be categorised into type A, B and C

A

Salmonella

The Salmonella group contains many members, most of which cause diarrhoeal diseases. They are facultative anaerobes, Gram negative rods which are not normally present as commensals in the gut.

Typhoid and paratyphoid are caused by Salmonella typhi and Salmonella paratyphi (types A, B & C) respectively. They are often termed enteric fevers, producing systemic symptoms such as headache, fever, arthralgia

Features
initially systemic upset as above
relative bradycardia
abdominal pain, distension
constipation: although Salmonella is a recognised cause of diarrhoea, constipation is more common in typhoid
rose spots: present on the trunk in 40% of patients, and are more common in paratyphoid

Possible complications include
osteomyelitis (especially in sickle cell disease where Salmonella is one of the most common pathogens)
GI bleed/perforation
meningitis
cholecystitis
chronic carriage (1%, more likely if adult females)

1815
Q

A 40 year old lady trips and falls through a glass door and sustains a severe laceration to her left arm. Amongst her injuries it is noticed that she has lost the ability to adduct the fingers of her left hand. Injury to which of the following nerves is most likely to account for her examination findings?

Ulnar

Median

Radial

Musculocutaneous

Axillary

A

The interossei are supplied by the ulnar nerve.
Ulnar nerve

Origin
C8, T1

Supplies (no muscles in the upper arm)
Flexor carpi ulnaris
Flexor digitorum profundus
Flexor digiti minimi
Abductor digiti minimi
Opponens digiti minimi
Adductor pollicis
Interossei muscle
Third and fourth lumbricals
Palmaris brevis

Path
Posteromedial aspect of upper arm to flexor compartment of forearm, then along the ulnar. Passes beneath the flexor carpi ulnaris muscle, then superficially over the flexor retinaculum into the palm of the hand.

Branches
Branch Supplies
Muscular branch Flexor carpi ulnaris
Medial half of the flexor digitorum profundus
Palmar cutaneous branch (Arises near the middle of the forearm) Skin on the medial part of the palm
Dorsal cutaneous branch Dorsal surface of the medial part of the hand
Superficial branch Cutaneous fibres to the anterior surfaces of the medial one and one-half digits
Deep branch Hypothenar muscles
All the interosseous muscles
Third and fourth lumbricals
Adductor pollicis
Medial head of the flexor pollicis brevis

Effects of injury
Damage at the wrist
Wasting and paralysis of intrinsic hand muscles (claw hand)
Wasting and paralysis of hypothenar muscles
Loss of sensation medial 1 and half fingers
Damage at the elbow
Radial deviation of the wrist
Clawing less in 4th and 5th digits

1816
Q

Which of the following statements relating to the knee joint is false?

It is the largest synovial joint in the body

When the knee is fully extended all ligaments (bar the anterolateral aspect of the posterior cruciate ligament) of the knee joint are taut

Rupture of the anterior cruciate ligament may result in haemarthrosis

The posterior aspect of the patella is extrasynovial

The joint is innervated by the femoral, sciatic and obturator nerves

A

The posterior aspect is intrasynovial and the knee itself comprises the largest synovial joint in the body. It may swell considerably following trauma such as ACL injury. Which may be extremely painful owing to rich innervation from femoral, sciatic and ( a smaller) contribution from the obturator nerve. During full extension all ligaments are taut and the knee is locked.

Knee joint

The knee joint is a synovial joint, the largest and most complicated. It consists of two condylar joints between the femur and tibia and a sellar joint between the patella and the femur. The tibiofemoral articular surfaces are incongruent, however, this is improved by the presence of the menisci. The degree of congruence is related to the anatomical position of the knee joint and is greatest in full extension.

Knee joint compartments
Tibiofemoral
Comprised of the patella/femur joint, lateral and medial compartments (between femur condyles and tibia)
Synovial membrane and cruciate ligaments partially separate the medial and lateral compartments
Patellofemoral
Ligamentum patellae
Actions: provides joint stability in full extension

Fibrous capsule
The capsule of the knee joint is a complex, composite structure with contributions from adjacent tendons.
Anterior fibres The capsule does not pass proximal to the patella. It blends with the tendinous expansions of vastus medialis and lateralis
Posterior fibres These fibres are vertical and run from the posterior surface of the femoral condyles to the posterior aspect of the tibial condyle
Medial fibres Attach to the femoral and tibial condyles beyond their articular margins, blending with the tibial collateral ligament
Lateral fibres Attach to the femur superior to popliteus, pass over its tendon to head of fibula and tibial condyle

Bursae
Anterior
Subcutaneous prepatellar bursa; between patella and skin
Deep infrapatellar bursa; between tibia and patellar ligament
Subcutaneous infrapatellar bursa; between distal tibial tuberosity and skin
Laterally
Bursa between lateral head of gastrocnemius and joint capsule
Bursa between fibular collateral ligament and tendon of biceps femoris
Bursa between fibular collateral ligament and tendon of popliteus
Medially
Bursa between medial head of gastrocnemius and the fibrous capsule
Bursa between tibial collateral ligament and tendons of sartorius, gracilis and semitendinosus
Bursa between the tendon of semimembranosus and medial tibial condyle and medial head of gastrocnemius
Posterior Highly variable and inconsistent

Ligaments
Medial collateral ligament Medial epicondyle femur to medial tibial condyle: valgus stability
Lateral collateral ligament Lateral epicondyle femur to fibula head: varus stability
Anterior cruciate ligament Anterior tibia to lateral intercondylar notch femur: prevents tibia sliding anteriorly
Posterior cruciate ligament Posterior tibia to medial intercondylar notch femur: prevents tibia sliding posteriorly
Patellar ligament Central band of the tendon of quadriceps femoris, extends from patella to tibial tuberosity

Menisci
Medial and lateral menisci compensate for the incongruence of the femoral and tibial condyles.
Composed of fibrous tissue.
Medial meniscus is attached to the tibial collateral ligament.
Lateral meniscus is attached to the loose fibres at the lateral edge of the joint and is separate from the fibular collateral ligament. The lateral meniscus is crossed by the popliteus tendon.

Nerve supply
The knee joint is supplied by the femoral, tibial and common peroneal divisions of the sciatic and by a branch from the obturator nerve. Hip pathology pain may be referred to the knee.

Blood supply
Genicular branches of the femoral artery, popliteal and anterior tibial arteries all supply the knee joint.

1817
Q

A 45 year old man sustains a significant head injury and a craniotomy is performed. The sigmoid sinus is bleeding profusely, into which of the following structures does it drain?

Internal jugular vein

Straight sinus

Petrosal sinus

Inferior sagittal sinus

External jugular vein

A

The sigmoid sinus is joined by the inferior petrosal sinus to drain into the internal jugular vein.
Cranial venous sinuses

The cranial venous sinuses are located within the dura mater. They have no valves which is important in the potential for spreading sepsis. They eventually drain into the internal jugular vein.

They are:

Superior sagittal sinus
Inferior sagittal sinus
Straight sinus
Transverse sinus
Sigmoid sinus
Confluence of sinuses
Occipital sinus
Cavernous sinus

1818
Q

A 10 year old boy is playing with a firework which explodes and he sustains a full thickness burn to his left arm. Which of the following statements is not characteristic of this situation?

They have a leathery appearance

The burn area is extremely painful until skin grafted

They always heal with scarring

Blanching does not occur under pressure

Absence of,or few, blisters

A

Full thickness burns involve complete injury to the dermis and sub dermal appendages. They have a leathery, often white appearance. They are initially insensate although pain often occurs during healing following skin grafting. They do not blanch under pressure.

Burns

Burns may be thermal, chemical or electrical. In the former category are burns which occur as a result of heat. Chemical burns occur when the skin is exposed to an extremely caustic or alkaline substance. Electrical burns occur following exposure to electrical current. The immediate management includes removal of the burning source which usually includes irrigation of the burned area. A detailed assessment then needs to be made of the extent of the burns and a number of charts are available for recording this information. The degree of injury relates to the temperature and duration of exposure. Most domestic burns are mainly scalds in young children.

Following the burn, there is a local response with progressive tissue loss and release of inflammatory cytokines. Systemically, there are cardiovascular effects resulting from fluid loss and sequestration of fluid into the third space. There is a marked catabolic response. Immunosuppression is common with large burns and bacterial translocation from the gut lumen is a recognised event. Sepsis is a common cause of death following major burns.

Types of burn

Type of burn Skin layers affected Skin appearance Blanching Management
Epidermal/Superficial Epidermis Red, moist Yes
Superficial partial thickness Epidermis and part of papillary dermis affected Pale, dry Yes Normally heals with no intervention
Deep partial thickness Epidermis, whole papillary dermis affected Mottled red colour No Needs surgical intervention (depending on site)
Full thickness Whole skin layer and subcutaneous tissue affected Dry, leathery hard wound No Burns centre

Depth of burn assessment
Bleeding on needle prick
Sensation
Appearance
Blanching to pressure

Percentage burn estimation
Lund Browder chart: most accurate even in children
Wallace rule of nines
Palmar surface: surface area palm = 0.8% burn

> 15% body surface area burns in adults needs urgent burn fluid resuscitation

Transfer to burn centre if:
Need burn shock resuscitation
Face/hands/genitals affected
Deep partial thickness or full thickness burns
Significant electrical/chemical burns

Management
The initial aim is to stop the burning process and resuscitate the patient. Intravenous fluids will be required for children with burns greater than 10% of total body surface area. Adults with burns greater than 15% of total body surface area will also require IV fluids. The fluids are calculated using the Parkland formula which is; volume of fluid= total body surface area of the burn % x weight (Kg) x2-4 (preference for lower amount i.e. 2ml to avoid excessive fluid overload). Half of the fluid is administered in the first 8 hours. A urinary catheter should be inserted. Analgesia should be given. Complex burns, burns involving the hand perineum and face and burns >10% in adults and >5% in children should be transferred to a burns unit.

Circumferential burns affecting a limb or severe torso burns impeding respiration may require escharotomy to divide the burnt tissue.

Conservative management is appropriate for superficial burns and mixed superficial burns that will heal in 2 weeks. More complex burns may require excision and skin grafting. Excision and primary closure is not generally practised as there is a high risk of infection.

There is no evidence to support the use of anti microbial prophylaxis or topical antibiotics in burn patients.

Escharotomies
Indicated in circumferential full thickness burns to the torso or limbs.
Careful division of the encasing band of burn tissue will potentially improve ventilation (if the burn involves the torso), or relieve compartment syndrome and oedema (where a limb is involved)

References
www.euroburn.org/e107files/downloads/guidelinesburncare.pdf

Barajas-Nava LA, López-Alcalde J, Roqué i Figuls M, Solà I, Bonfill Cosp X. Antibiotic prophylaxis for preventing burn wound infection. Cochrane Database of Systematic Reviews 2013, Issue 6. Art. No.: CD008738. DOI: 10.1002/14651858.CD008738.pub2.

Hettiaratchy S & Papini R. Initial management of a major burn: assessment and resuscitation. BMJ 2004;329:101-103

1819
Q

Which muscle inserts onto the lesser trochanter?

Psoas minor

Psoas major

Sartorius

Obturator externus

Gemellus

A

Psoas major inserts onto the lesser trochanter.
Femur

  • Extends from a rounded head, which articulates with the acetabulum down to the knee joint where the two large condyles at it’s inferior aspect articulate with the tibia.
    The superior aspect comprises a head and neck which pass inferolaterally to the body and the two trochanters. These lie at the junction between the neck and the body.
    The neck meets the body of the femur at an angle of 125o.
    Developmentally, the neck is part of the body but is demarcated from it by a wide rough intertrochanteric crest, this continues inferomedially as a spiral line that runs below the lesser trochanter. Medially, the intertrochanteric line gives attachment to the inferior end of the iliofemoral ligament. The neck is covered by synovial membrane up to the intertrochanteric line. The posterior aspect of the neck is demarcated from the shaft by the intertrochanteric crest and only it’s medial aspect is covered by synovium and the joint capsule.
    The greater trochanter has discernible surfaces that form the site of attachment of the gluteal muscles.Laterally, the greater trochanter overhangs the body and this forms part of the origin of vastus lateralis
    Viewed anteriorly, the body of the femur appears rounded. Viewed laterally, it has an anterior concavity which gives fullness to the anterior thigh. Posteriorly, there is a ridge of bone, the linea aspera. The surface of the anterior aspect of the body forms the origin of the vastus intermedius. More medially, it forms the origin of vastus medialis.
    The upper and middle aspects of the linea aspera form part of the origin of the attachments of the thigh adductors. Inferiorly, it spans out to form the bony floor of the popliteal fossa. At the inferior aspect of the popliteal surface the surface curves posteriorly to form the femoral condyles.
    The structures that are attached to the inferior aspect of the linea aspera split with it as it approaches the popliteal fossa. Thus the vastus medialis and adductor magnus continue with the medial split and the biceps femoris and vastus intermedius along the lateral split.

Blood supply
The femur has a rich blood supply and numerous vascular foramina exist throughout it’s length. The blood supply to the femoral head is clinically important and is provided by the medial circumflex femoral and lateral circumflex femoral arteries (Branches of profunda femoris). Also from the inferior gluteal artery. These form an anastomosis and travel to up the femoral neck to supply the head.

1820
Q

A 42 year old teacher is admitted with a fall. An x-ray confirms a fracture of the surgical neck of the humerus. Which nerve is at risk?

Median

Radial

Axillary

Muscuculocutaneous

Subscapular

A

The Axillary nerve winds around the bone at the neck of the humerus. The axillary nerve is also at risk during shoulder dislocation.

Axillary nerve

Terminal branch of the posterior cord of the brachial plexus
Root values C5 and C6
Descends posterior to the axillary artery at the lower border of subscapularis and then passes through quadrangular space with the posterior circumflex humeral vessels
Divides into anterior and posterior branches
Innervates deltoid muscle and small patch of skin over deltoid

1821
Q

A 63 year old man undergoes a salvage abdominoperineal excision of the anus and rectum for recurrent anal cancer. He has previously been treated with radical chemoradiotherapy. At the conclusion of the procedure, there is a 10cm x 10cm perineal skin defect. What is the most appropriate option for providing closure?

Use of a VAC wound management system

Rotational skin flap

Deep tension sutures and primary closure

Pedicled myocutaneous flap

Delayed primary closure

A

The use of previous radiotherapy means that the wound will not heal well. A myocutaneous flap will mean that non irradiated tissue is interposed into the wound bed. Rotational skin flaps will comprise irradiated tissue and won’t heal.

Methods of wound closure

Method of closure Indication
Primary closure
Clean wound, usually surgically created or following minor trauma
Standard suturing methods will usually suffice
Wound heals by primary intention
Delayed primary closure
Similar methods of actual closure to primary closure
May be used in situations where primary closure is either not achievable or not advisable e.g. infection
Vacuum assisted closure
Uses negative pressure therapy to facilitate wound closure
Sponge is inserted into wound cavity and then negative pressure applied
Advantages include removal of exudate and versatility
Disadvantages include cost and risk of fistulation if used incorrectly on sites such as bowel
Split thickness skin grafts
Superficial dermis removed with Watson knife or dermatome (commonly from thigh)
Remaining epithelium regenerates from dermal appendages
Coverage may be increased by meshing
Full thickness skin grafts
Whole dermal thickness is removed
Sub dermal fat is then removed and graft placed over donor site
Better cosmesis and flexibility at recipient site
Donor site morbidity
Flaps
Viable tissue with a blood supply
May be pedicled or free
Pedicled flaps are more reliable, but limited in range
Free flaps have greater range but carry greater risk of breakdown as they require vascular anastomosis

1822
Q

A professional musician visits his physician after a morning concert. He complains of painless swelling in his right cheek when he plays his tuba. Physical examination of the patient reveals slight facial asymmetry due to minor swelling on the right side of the face. The skin over the swelling is smooth without any secondary changes. Palpation reveals a soft and non-tender swelling. The oral opening is normal without any trismus. Further examination reveals swelling of the left buccal mucosa extending from the first to the third molar. Bedside ultrasound shows small areas of high echogenicity consistent with pneumoparotid. Which nerve is associated with motor function to prevent air from entering the affected duct in this patient?

V2 – Maxillary nerve

V3 – Mandibular nerve

CN VII – Zygomatic branch

CN VII – Buccal branch

CN VII – Marginal mandibular branch

A

D- CN VII – Buccal branch

Pneumoparotid is a very rare condition of parotid gland, often complicating with a subcutaneous emphysema, causing swelling of the parotid lodge. This pathology usually occurred due no physiological stagnation of air in parotid parenchyma. Pneumoparotid is usually associated with a retrograde insufflation of air and saliva via Stensens duct into the secondary ducts and glandular acini. Hypotonia of the buccinator muscle, hypertrophy of the masseter muscle or temporary obstruction of the Stensens duct by mucous are described as possible risk factors.
The opening of the Stensens duct lies near to the second upper molar tooth bilaterally. The normal anatomy of duct preventing the reflux of air and saliva into the parotid gland are three fold:
The diameter of the duct orifice is smaller than that of the duct itself;
The duct opening is covered by redundant mucosal layer, covering the duct orifice when there is increased intraoral pressure.
The Stensens duct is compressed in its lateral course along the masseter muscle and its passage through the buccinator muscle with an increase in oral pressure.
The buccinator is the muscle of interest here. Motor innervation is from the buccal branch of the facial nerve (cranial nerve VII). Sensory innervation is supplied by the buccal branch (one of the muscular branches) of the mandibular part of the trigeminal (cranial nerve V).

Facial nerve

The facial nerve is the main nerve supplying the structures of the second embryonic branchial arch. It is predominantly an efferent nerve to the muscles of facial expression, digastric muscle and also to many glandular structures. It contains a few afferent fibres which originate in the cells of its genicular ganglion and are concerned with taste.

Supply - ‘face, ear, taste, tear’
Face: muscles of facial expression
Ear: nerve to stapedius
Taste: supplies anterior two-thirds of tongue
Tear: parasympathetic fibres to lacrimal glands, also salivary glands

Path
Subarachnoid path
Origin: motor- pons, sensory- nervus intermedius
Pass through the petrous temporal bone into the internal auditory meatus with the vestibulocochlear nerve. Here they combine to become the facial nerve.

Facial canal path
The canal passes superior to the vestibule of the inner ear
At the medial aspect of the middle ear, it becomes wider and contains the geniculate ganglion.
- 3 branches:
1. greater petrosal nerve
2. nerve to stapedius
3. chorda tympani

Stylomastoid foramen
Passes through the stylomastoid foramen (tympanic cavity anterior and mastoid antrum posteriorly)
Posterior auricular nerve and branch to posterior belly of digastric and stylohyoid muscle

Face
Enters parotid gland and divides into 5 branches:
Temporal branch
Zygomatic branch
Buccal branch
Marginal mandibular branch
Cervical branch

1823
Q

A 73 year old man presents with haemoptysis and is suspected of suffering from lung cancer. On examination he has an enlarged supraclavicular lymph node. Which of the following features is most likely to be present on histological examination?

Increased mitoses

Apoptosis

Barr Bodies

Multinucleate giant cells

Granuloma

A

Increased mitoses are commonly seen in association with malignant transformation of cells. Apoptosis is not a common feature of metastatic cancer. Barr Bodies are formed during X chromosome inactivation in female somatic cells.

Histopathology of malignancy

Abnormal tissue architecture
Coarse chromatin
Invasion of basement membrane*
Abnormal mitoses
Angiogenesis
De-differentiation
Areas of necrosis
Nuclear pleomorphism

*= Those features that distinguish invasive malignancy from in situ disease

1824
Q

A 56 year old lady undergoes a Hartmann’s style resection of the sigmoid colon, with ligation of the vessels close to the colon. Which of the following vessels will be responsible to supplying the rectal stump directly?

Superior mesenteric artery

Middle colic artery

Superior rectal artery

Inferior mesenteric artery

External iliac artery

A

This question is addressing the blood supply to the rectum. Which is supplied by the superior rectal artery. High ligation of the IMA may compromise this structure. However, the question states that during the Hartmans procedure the vessels were ligated close to the bowel. Implying that the superior rectal was preserved.

Rectum

The rectum is approximately 12 cm long. It is a capacitance organ. It has both intra and extraperitoneal components. The transition between the sigmoid colon is marked by the disappearance of the tenia coli.The extra peritoneal rectum is surrounded by mesorectal fat that also contains lymph nodes. This mesorectal fatty layer is removed surgically during rectal cancer surgery (Total Mesorectal Excision). The fascial layers that surround the rectum are important clinical landmarks, anteriorly lies the fascia of Denonvilliers. Posteriorly lies Waldeyers fascia.

Extra peritoneal rectum
Posterior upper third
Posterior and lateral middle third
Whole lower third

Relations
Anteriorly (Males) Rectovesical pouch
Bladder
Prostate
Seminal vesicles
Anteriorly (Females) Recto-uterine pouch (Douglas)
Cervix
Vaginal wall
Posteriorly Sacrum
Coccyx
Middle sacral artery
Laterally Levator ani
Coccygeus

Arterial supply
Superior rectal artery
Middle rectal artery (from the internal iliac)
Inferior rectal artery (from the pudendal vessels)

Venous drainage
Superior rectal vein
Inferior rectal vein
Note the venous drainage is a site of portosystemic anastomosis.

Lymphatic drainage
Mesorectal lymph nodes (superior to dentate line)
Inguinal nodes (inferior to dentate line)

1825
Q

A 63 year old man presents with episodic rectal bleeding the blood tends to be dark in colour and may be mixed with stool. His bowel habit has been erratic since an abdominal aortic aneurysm repair 6 weeks previously. What is the most likely cause?

Ischaemic colitis

Diverticulitis

Angiodysplasia

Cancer

Ulcerative colitis

A

The inferior mesenteric artery may have been ligated and being an arteriopath collateral flow through the marginal may be imperfect.

Lower Gastrointestinal bleeding

Colonic bleeding
This typically presents as bright red or dark red blood per rectum. Colonic bleeding rarely presents as malaena type stool, this is because blood in the colon has a powerful laxative effect and is rarely retained long enough for transformation to occur and because the digestive enzymes present in the small bowel are not present in the colon. Up to 15% of patients presenting with haemochezia will have an upper gastrointestinal source of haemorrhage.

As a general rule right sided bleeds tend to present with darker coloured blood than left sided bleeds. Haemorrhoidal bleeding typically presents as bright red rectal bleeding that occurs post defecation either onto toilet paper or into the toilet pan. It is very unusual for haemorrhoids alone to cause any degree of haemodynamic compromise.

Causes
Cause Presenting features
Colitis Bleeding may be brisk in advanced cases, diarrhoea is commonly present. Abdominal x-ray may show featureless colon.
Diverticular disease Acute diverticulitis often is not complicated by major bleeding and diverticular bleeds often occur sporadically. 75% all will cease spontaneously within 24-48 hours. Bleeding is often dark and of large volume.
Cancer Colonic cancers often bleed and for many patients this may be the first sign of the disease. Major bleeding from early lesions is uncommon
Haemorrhoidal bleeding Typically bright red bleeding occurring post defecation. Although patients may give graphic descriptions bleeding of sufficient volume to cause haemodynamic compromise is rare.
Angiodysplasia Apart from bleeding, which may be massive, these arteriovenous lesions cause little in the way of symptoms. The right side of the colon is more commonly affected.

Management
Prompt correction of any haemodynamic compromise is required. Unlike upper gastrointestinal bleeding the first line management is usually supportive. This is because in the acute setting endoscopy is rarely helpful.
When haemorrhoidal bleeding is suspected a proctosigmoidoscopy is reasonable as attempts at full colonoscopy are usually time consuming and often futile.
In the unstable patient the usual procedure would be an angiogram (either CT or percutaneous), when these are performed during a period of haemodynamic instability they may show a bleeding point and may be the only way of identifying a patch of angiodysplasia.
In others who are more stable the standard procedure would be a colonoscopy in the elective setting. In patients undergoing angiography attempts can be made to address the lesion in question such as coiling. Otherwise surgery will be necessary.
In patients with ulcerative colitis who have significant haemorrhage the standard approach would be a sub total colectomy, particularly if medical management has already been tried and is not effective.

Indications for surgery
Patients > 60 years
Continued bleeding despite endoscopic intervention
Recurrent bleeding
Known cardiovascular disease with poor response to hypotension

Surgery
Selective mesenteric embolisation if life threatening bleeding. This is most helpful if conducted during a period of relative haemodynamic instability. If all haemodynamic parameters are normal then the bleeding is most likely to have stopped and any angiography normal in appearance. In many units a CT angiogram will replace selective angiography but the same caveats will apply.

If the source of colonic bleeding is unclear; perform a laparotomy, on table colonic lavage and following this attempt a resection. A blind sub total colectomy is most unwise, for example bleeding from an small bowel arterio-venous malformation will not be treated by this manoeuvre.

Summary of Acute Lower GI bleeding recommendations
Consider admission if:
* Over 60 years
* Haemodynamically unstable/profuse PR bleeding
* On aspirin or NSAID
* Significant co morbidity

Management
All patients should have a history and examination, PR and proctoscopy
Colonoscopic haemostasis aimed for in post polypectomy or diverticular bleeding

References
http://www.sign.ac.uk/guidelines/fulltext/105/index.html

1826
Q

A 40 year old woman is admitted with 1 day history of abdominal pain. She has suffered from repeated episodes of this colicky right upper quadrant pain. On examination, she is pyrexial with right upper quadrant peritonism. Her blood tests show a white cell count of 23. However, the liver function tests are normal. An abdominal ultrasound scan done the day of admission, shows multiple gallstones in a thick walled gallbladder, the bile duct measures 4mm. What is the best course of action?

Administration of broad spectrum intravenous antibiotics and perform a delayed open cholecystectomy in 3 months

Arrange a radiological cholecystotomy

Undertake a laparoscopic cholecystectomy

Undertake an open cholcystectomy

Administration of broad spectrum intravenous antibiotics and perform a delayed laparoscopic cholecystectomy in 3 months

A

This lady has acute cholecystitis and needs an acute cholecystectomy. This operation should usually be performed within 72 hours of admission. Delay beyond this timeframe will usually result in increased operative complications and most surgeons would administer antibiotics and perform an interval cholecystectomy if the early window for an acute procedure is missed. A bile duct measuring 4mm is usually normal.

Biliary disease

Diagnosis Typical features Pathogenesis
Gallstones Typically history of biliary colic or episodes of chlolecystitis. Obstructive type history and test results. Usually small calibre gallstones which can pass through the cystic duct. In Mirizzi syndrome the stone may compress the bile duct directly- one of the rare times that cholecystitis may present with jaundice
Cholangitis Usually obstructive and will have Charcot’s triad of symptoms (pain, fever, jaundice) Ascending infection of the bile ducts usually by E. coli and by definition occurring in a pool of stagnant bile.
Pancreatic cancer Typically painless jaundice with palpable gallbladder (Courvoisier’s Law) Direct occlusion of distal bile duct or pancreatic duct by tumour. Sometimes nodal disease at the portal hepatis may be the culprit in which case the bile duct may be of normal calibre.
TPN (total parenteral nutrition) associated jaundice Usually follows long term use and is usually painless with non obstructive features Often due to hepatic dysfunction and fatty liver which may occur with long term TPN usage.
Bile duct injury Depending upon the type of injury may be of sudden or gradual onset and is usually of obstructive type Often due to a difficult laparoscopic cholecystectomy when anatomy in Calots triangle is not appreciated. In the worst scenario the bile duct is excised and jaundice develops rapidly post operatively. More insidious is that of bile duct stenosis which may be caused by clips or diathermy injury.
Cholangiocarcinoma Gradual onset obstructive pattern Direct occlusion by disease and also extrinsic compression by nodal disease at the porta hepatis.
Septic surgical patient Usually hepatic features Combination of impaired biliary excretion and drugs such as ciprofloxacin which may cause cholestasis.
Metastatic disease Mixed hepatic and post hepatic Combination of liver synthetic failure (late) and extrinsic compression by nodal disease and anatomical compression of intra hepatic structures (earlier)

A gallbladder may develop a thickened wall in chronic cholecystitis, microscopically Roikitansky-Aschoff Sinuses may be seen

1827
Q

A 32 year old lady undergoes a thyroidectomy for a mild goitre. The resected specimen shows an intense lymphocytic infiltrate with acinar destruction and fibrosis. What is the underlying lesion?

Anaplastic carcinoma

Rhadomyosarcoma

Lymphoma

Hashimotos thyroiditis

Graves disease

A

Lymphocytic infiltrates and fibrosis are typically seen in Hashimotos thyroiditis. In lymphoma only dense lymphatic type tissue is usually present.

Thyroid neoplasms

Lesion Common features
Follicular adenoma
Usually present as a solitary thyroid nodule
Malignancy can only be excluded on formal histological assessment
Papillary carcinoma
Usually contain a mixture of papillary and colloidal filled follicles
Histologically tumour has papillary projections and pale empty nuclei
Seldom encapsulated
Lymph node metastasis predominate
Haematogenous metastasis rare
Account for 60% of thyroid cancers
Follicular carcinoma
May appear macroscopically encapsulated, microscopically capsular invasion is seen. Without this finding the lesion is a follicular adenoma.
Vascular invasion predominates
Multifocal disease rare
Account for 20% of all thyroid cancers
Anaplastic carcinoma
Most common in elderly females
Local invasion is a common feature
Account for 10% of thyroid cancers
Treatment is by resection where possible, palliation may be achieved through isthmusectomy and radiotherapy. Chemotherapy is ineffective.
Medullary carcinoma
Tumours of the parafollicular cells (C Cells)
C cells derived from neural crest and not thyroid tissue
Serum calcitonin levels often raised
Familial genetic disease accounts for up to 20% cases
Both lymphatic and haematogenous metastasis are recognised, nodal disease is associated with a very poor prognosis.

1828
Q

A 68 year old man has a TCC of the bladder. He has a right hydronephrosis detected on ultrasound and deteriorating renal function. A DMSA scan shows a non functioning left kidney. At cystoscopy the ureteric orifice cannot be readily accessed. What is the best course of action?

Insertion of antegrade ureteric stent

Insertion of retrograde ureteric stent

Cystectomy and ileal conduit

Radiotherapy

Instillation of intravesical BCG

A

Antegrade ureteric stents pass from the kidney to the bladder
Retrograde stents pass from the bladder to the kidney
A TCC occluding the ureteric orifice will obscure its identification during surgery, so that passage of a retrograde stent is difficult. Therefore passage of a stent from the renal pelvis is preferable.

Hydronephrosis

Causes of hydronephrosis

Unilateral: PACT
Pelvic-ureteric obstruction (congenital or acquired)
Aberrant renal vessels
Calculi
Tumours of renal pelvis

Bilateral: SUPER
Stenosis of the urethra
Urethral valve
Prostatic enlargement
Extensive bladder tumour
Retro-peritoneal fibrosis

Investigation
USS- identifies presence of hydronephrosis and can assess the kidneys
IVU- assess the position of the obstruction
Antegrade or retrograde pyelography- allows treatment
If renal colic suspected: non contrast CT scan (majority of stones are detected this way)

Management
Remove the obstruction and drainage of urine
Acute upper urinary tract obstruction: Nephrostomy tube
Chronic upper urinary tract obstruction: Ureteric stent or a pyeloplasty

1829
Q

During a pneumonectomy for carcinoma of the bronchus, there is an injury to the pulmonary vein with blood loss of approximately 1750mls. Of the physiological changes described below, which is not in keeping with this event?

Decreased cardiac output

Increased heart rate

Reduced left ventricle filling pressures

Reduced blood pressure

Reduced systemic vascular resistance

A

Cardiogenic Shock:
e.g. MI, valve abnormality

increased SVR (vasoconstriction in response to low BP)
increased HR (sympathetic response)
decreased cardiac output
decreased blood pressure

Hypovolaemic shock:
blood volume depletion
e.g. haemorrhage, vomiting, diarrhoea, dehydration, third-space losses during major operations

increased SVR
increased HR
decreased cardiac output
decreased blood pressure

Septic shock:
occurs when the peripheral vascular dilatation causes a fall in SVR
similar response may occur in anaphylactic shock, neurogenic shock

reduced SVR
increased HR
normal/increased cardiac output
decreased blood pressure
SVR will typically increase in haemorrhagic shock.

1830
Q

A 20 year old lady presents with a thyroid cancer, she is otherwise well with no significant family history. On examination, she has a nodule in the left lobe of the thyroid, with a small discrete mass separate from the gland itself. Which of the following is the most likely cause?

Follicular carcinoma

Anaplastic carcinoma

Medullary carcinoma

Papillary carcinoma

B Cell Lymphoma

A

Papillary carcinoma is the most common subtype and may cause lymph node metastasis (mass separate from the gland itself) that is rare with follicular tumours. Anaplastic carcinoma would cause more local symptoms and would be rare in this age group.

Thyroid malignancy

Papillary carcinoma
Commonest sub-type
Accurately diagnosed on fine needle aspiration cytology
Histologically, they may demonstrate psammoma bodies (areas of calcification) and so called ‘orphan Annie’ nuclei
They typically metastasise via the lymphatics and thus laterally located apparently ectopic thyroid tissue is usually a metastasis from a well differentiated papillary carcinoma

Follicular carcinoma
Are less common than papillary lesions
Like papillary tumours, they may present as a discrete nodule. Although they appear to be well encapsulated macroscopically there is invasion on microscopic evaluation
Lymph node metastases are uncommon and these tumours tend to spread haematogenously. This translates into a higher mortality rate
Follicular lesions cannot be accurately diagnosed on fine needle aspiration cytology and thus all follicular FNA’s (THY 3f) will require at least a hemi thyroidectomy

Anaplastic carcinoma
Less common and tend to occur in elderly females
Disease is usually advanced at presentation and often only palliative decompression and radiotherapy can be offered.

Medullary carcinoma
These are tumours of the parafollicular cells ( C Cells) and are of neural crest origin.
The serum calcitonin may be elevated which is of use when monitoring for recurrence.
They may be familial and occur as part of the MEN -2A disease spectrum.
Spread may be either lymphatic or haematogenous and as these tumours are not derived primarily from thyroid cells they are not responsive to radioiodine.

Lymphoma
These respond well to combined chemoradiotherapy
Radical surgery is unnecessary once the disease has been diagnosed on biopsy material. Such biopsy material is not generated by an FNA and thus a core biopsy has to be obtained (with care!).

1831
Q

The cords of the brachial plexus are most closely related to which of the following vessels?

Subclavian artery

Axillary artery

Axillary vein

Subclavian vein

Brachial artery

A

The trunks are related to the subclavian artery superiorly. The cords of the plexus surround the axillary artery, they are named according to their positions relative to this structure.

Brachial plexus

The brachial plexus extends from the neck to the axilla. It is formed by the ventral rami of the fifth to the eighth cervical nerves with the ascending part of the first thoracic nerve.

Location of the plexus
The ventral rami which form the plexus enter the lower part of the posterior triangle of the neck in series with the ventral rami of the cervical plexus. The second part of the subclavian artery lies immediately anterior to the lower two rami. The upper three rami intermingle and pass inferolaterally towards the axilla and subclavian artery. They are enclosed within an extension of the prevertebral fascia. In the neck the plexus lies deep to platysma, the supraclavicular nerves, inferior belly of omohyoid and the transverse cervical artery. It then passes deep to the clavicle and the suprascapular vessels, to enter the axilla, and thence surround the second part of the axillary artery

Composition of the plexus
Ventral rami, the roots of the plexus, lie between scalenus medius and anterior.

As they enter the posterior triangle, the upper two (C5,6) and lower two (C8, T1) roots of the plexus unite to form the upper and lower trunks of the plexus respectively. Meanwhile, C7 continues as the middle trunk. The lower trunk may groove the superior surface of the first rib posterior to the subclavian artery, and the root from the first ventral ramus is always in contact with it.

Each trunk divides into ventral and dorsal divisions which are destined to supply the anterior (flexor) and posterior (extensor) parts of the upper limb.

The cords of the plexus are formed in the axilla. The dorsal divisions unite to form the posterior cord (C5-8). The ventral divisions of the upper and middle trunks unite to form the lateral cord (C5-7), while the ventral divisions of the lower trunk continues as the medial cord (C8-T1). The cords are named according to their relationship to the axillary artery. Each cord terminates by dividing into two main branches at the beginning of the third part of the artery.

Sympathetic communications
The fifth and sixth cervical ventral rami receive grey rami communicantes from the middle cervical ganglion, while the two or more grey rami communicantes pass from the inferior cervical ganglion to the seventh and eighth cervical ventral rami. The first thoracic ventral ramus receives its grey ramus from the cervicothoracic ganglion. Its for this reason that inferior plexus injury can be complicated by a Horners syndrome.

Summary
Origin Anterior rami of C5 to T1
Sections of the plexus
Roots, trunks, divisions, cords, branches
Mnemonic:Real Teenagers Drink Cold Beer
Roots
Located in the posterior triangle
Pass between scalenus anterior and medius
Trunks
Located posterior to middle third of clavicle
Upper and middle trunks related superiorly to the subclavian artery
Lower trunk passes over 1st rib posterior to the subclavian artery
Divisions Apex of axilla
Cords Related to axillary artery

1832
Q

A 72 year old man attends vascular clinic after having an amputation 2 months ago. He is having difficulty sleeping at night due to persistent tingling at the amputation site. He is known to have orthostatic hypotension. What is the most appropriate analgesic modality?

Amitriptylline

Pregabalin

Duloxetine

Morphine

Diclofenac

A

This patient has phantom limb pain which is a neuropathic pain. First line management is with amitriptylline or pregabalin. However this patient has orthostatic hypotension, which is a side effect of amitriptylline, therefore pregabalin is the treatment of choice.

Management of pain

World Health Organisation Analgesic Ladder
Initially peripherally acting drugs such as paracetamol or non-steroidal anti-inflammatory drugs (NSAIDs) are given.
If pain control is not achieved, the second part of the ladder is to introduce weak opioid drugs such as codeine or dextropropoxyphene together with appropriate agents to control and minimise side effects.
The final rung of the ladder is to introduce strong opioid drugs such as morphine. Analgesia from peripherally acting drugs may be additive to that from centrally-acting opioids and thus, the two are given together.

The World Federation of Societies of Anaesthesiologists (WFSA) Analgesic Ladder
For management of acute pain
Initially, the pain can be expected to be severe and may need controlling with strong analgesics in combination with local anaesthetic blocks and peripherally acting drugs.
The second rung on the postoperative pain ladder is the restoration of the use of the oral route to deliver analgesia. Strong opioids may no longer be required and adequate analgesia can be obtained by using combinations of peripherally acting agents and weak opioids.
The final step is when the pain can be controlled by peripherally acting agents alone.

Local anaesthetics
Infiltration of a wound with a long-acting local anaesthetic such as Bupivacaine
Analgesia for several hours
Further pain relief can be obtained with repeat injections or by infusions via a thin catheter
Blockade of plexuses or peripheral nerves will provide selective analgesia in those parts of the body supplied by the plexus or nerves
Can either be used to provide anaesthesia for the surgery or specifically for postoperative pain relief
Especially useful where a sympathetic block is needed to improve postoperative blood supply or where central blockade such as spinal or epidural blockade is contraindicated.

Spinal anaesthesia
Provides excellent analgesia for surgery in the lower half of the body and pain relief can last many hours after completion of the operation if long-acting drugs containing vasoconstrictors are used.

  • Side effects of spinal anaesthesia include: hypotension, sensory and motor block, nausea and urinary retention.

Epidural anaesthesia
An indwelling epidural catheter inserted. This can then be used to provide a continuous infusion of analgesic agents. It can provide excellent analgesia. They are still the preferred option following major open abdominal procedures and help prevent post operative respiratory compromise resulting from pain.

  • Disadvantages of epidurals is that they usually confine patients to bed, especially if a motor block is present. In addition an indwelling urinary catheter is required. Which may not only impair mobility but also serve as a conduit for infection. They are contraindicated in coagulopathies.

Transversus Abdominal Plane block (TAP)
In this technique an ultrasound is used to identify the correct muscle plane and local anaesthetic (usually bupivicaine) is injected. The agent diffuses in the plane and blocks many of the spinal nerves. It is an attractive technique as it provides a wide field of blockade but does not require the placement of any indwelling devices. There is no post operative motor impairment. For this reason it is the preferred technique when extensive laparoscopic abdominal procedures are performed. They will then provide analgesia immediately following surgery but as they do not confine the patient to bed, the focus on enhanced recovery can begin sooner.

-The main disadvantage is that their duration of action is limited to the half life of the local anaesthetic agent chosen. In addition some anaesthetists do not have the USS skills required to site the injections.

Patient Controlled Analgesia (PCA)

  • Patients administer their own intravenous analgesia and titrate the dose to their own end-point of pain relief using a small microprocessor - controlled pump. Morphine is the most popular drug used.

Strong Opioids

Severe pain arising from deep or visceral structures requires the use of strong opioids

Morphine
Short half life and poor bioavailability.
Metabolised in the liver and clearance is reduced in patients with liver disease, in the elderly and the debilitated
Side effects include nausea, vomiting, constipation and respiratory depression.
Tolerance may occur with repeated dosage

Pethidine
Synthetic opioid which is structurally different from morphine but which has similar actions. Has 10% potency of morphine.
Short half life and similar bioavailability and clearance to morphine.
Short duration of action and may need to be given hourly.
Pethidine has a toxic metabolite (norpethidine) which is cleared by the kidney, but which accumulates in renal failure or following frequent and prolonged doses and may lead to muscle twitching and convulsions. Extreme caution is advised if pethidine is used over a prolonged period or in patients with renal failure.

Weak opioids
Codeine: markedly less active than morphine, has predictable effects when given orally and is effective against mild to moderate pain.

Non opioid analgesics
- Mild to moderate pain.

Paracetamol
Inhibits prostaglandin synthesis.
Analgesic and antipyretic properties but little anti-inflammatory effect
It is well absorbed orally and is metabolised almost entirely in the liver
Side effects in normal dosage and is widely used for the treatment of minor pain. It causes hepatotoxicity in over dosage by overloading the normal metabolic pathways with the formation of a toxic metabolite.

NSAIDs
Analgesic and anti-inflammatory actions
Inhibition of prostaglandin synthesis by the enzyme Cyclooxygenase which catalyses the conversion of arachidonic acid to the various prostaglandins that are the chief mediators of inflammation. All NSAIDs work in the same way and thus there is no point in giving more than one at a time. .
NSAIDs are, in general, more useful for superficial pain arising from the skin, buccal mucosa, joint surfaces and bone.
Relative contraindications: history of peptic ulceration, gastrointestinal bleeding or bleeding diathesis; operations associated with high blood loss, asthma, moderate to severe renal impairment, dehydration and any history of hypersensitivity to NSAIDs or aspirin.

Neuropathic pain
National Institute of Clinical Excellence (UK) guidelines:
First line: Amitriptyline (Imipramine if cannot tolerate) or pregabalin
Second line: Amitriptyline AND pregabalin
Third line: refer to pain specialist. Give tramadol in the interim (avoid morphine)
If diabetic neuropathic pain: Duloxetine
As of 1 April 2019, pregabalin and gabapentin are Class C controlled substances (under the Misuse of Drugs Act 1971) and scheduled under the Misuse of Drugs Regulations 2001 as Schedule 3. Evaluate patients carefully for a history of drug abuse before prescribing and observe patients for development of signs of abuse and dependence (MHRA, Drug Safety Update April 2019).

References
1. http://guidance.nice.org.uk/CG173/Guidance/pdf/English
2. Lovich-Sapola J, Smith CE, Brandt CP. Post operative pain control. Surg Clin North Am. 2015 Apr;95(2):301-183. Finnerup N et al. Pharmacotherapy for neuropathic pain in adults: a systematic review and meta-analysis. Lancet Neurol. 2015 Feb;14(2):162-73.

1833
Q

A 43 year old man is undergoing a right hemicolectomy and the ileo-colic artery is ligated. From which of the following vessels is it derived?

Inferior mesenteric artery

Superior mesenteric artery

Coeliac axis

Aorta

None of the above

A

The ileocolic artery is a branch of the SMA and supplies the right colon and terminal ileum. The transverse colon is supplied by the middle colic artery. As veins accompany arteries in the mesentery and are lined by lymphatics, high ligation is the norm in cancer resections. The ileo-colic artery branches off the SMA near the duodenum.

Colon anatomy

The colon commences with the caecum. This represents the most dilated segment of the human colon and its base (which is intraperitoneal) is marked by the convergence of teniae coli. At this point is located the vermiform appendix. The colon continues as the ascending colon, the posterior aspect of which is retroperitoneal. The line of demarcation between the intra and retro peritoneal right colon is visible as a white line, in the living, and forms the line of incision for colonic resections.

The ascending colon becomes the transverse colon after passing the hepatic flexure. At this location the colon becomes wholly intra peritoneal once again. The superior aspect of the transverse colon is the point of attachment of the transverse colon to the greater omentum. This is an important anatomical site since division of these attachments permits entry into the lesser sac. Separation of the greater omentum from the transverse colon is a routine operative step in both gastric and colonic resections.

At the left side of the abdomen the transverse colon passes to the left upper quadrant and makes an oblique inferior turn at the splenic flexure. Following this, the posterior aspect becomes retroperitoneal once again.

At the level of approximately L4 the descending colon becomes wholly intraperitoneal and becomes the sigmoid colon. Whilst the sigmoid is wholly intraperitoneal there are usually attachments laterally between the sigmoid and the lateral pelvic sidewall. These small congenital adhesions are not formal anatomical attachments but frequently require division during surgical resections.

At its distal end the sigmoid passes to the midline and at the region around the sacral promontary it becomes the upper rectum. This transition is visible macroscopically as the point where the teniae fuse. More distally the rectum passes through the peritoneum at the region of the peritoneal reflection and becomes extraperitoneal.

Arterial supply
Superior mesenteric artery and inferior mesenteric artery: linked by the marginal artery.
Ascending colon: ileocolic and right colic arteries
Transverse colon: middle colic artery
Descending and sigmoid colon: inferior mesenteric artery

Venous drainage
From regional veins (that accompany arteries) to superior and inferior mesenteric vein

Lymphatic drainage
Initially along nodal chains that accompany supplying arteries, then para-aortic nodes.

Embryology
Midgut- Second part of duodenum to 2/3 transverse colon
Hindgut- Distal 1/3 transverse colon to anus

Peritoneal location
The right and left colon are part intraperitoneal and part extraperitoneal. The sigmoid and transverse colon are generally wholly intraperitoneal. This has implications for the sequelae of perforations, which will tend to result in generalised peritonitis in the wholly intra peritoneal segments.

Colonic relations
Region of colon Relation
Caecum/ right colon Right ureter, gonadal vessels
Hepatic flexure Gallbladder (medially)
Splenic flexure Spleen and tail of pancreas
Distal sigmoid/ upper rectum Left ureter
Rectum Ureters, autonomic nerves, seminal vesicles, prostate, urethra (distally)

1834
Q

A 72 year old man is due to undergo an inguinal hernia repair. He suffers from COPD and has an exercise tolerance of 10 yards. He also has pitting oedema to the thighs. What is his ASA?

5

1

3

4

2

A

Severe systemic disease of this nature is a constant threat to life. Especially as he also has evidence of cardiac failure.
American Society of anesthesiologists physical status scoring system (ASA)

ASA grade Description
1 No organic physiological, biochemical or psychiatric disturbance. The surgical pathology is localised and has not invoked systemic disturbance
2 Mild or moderate systemic disruption caused either by the surgical disease process or though underlying pre-existing disease
3 Severe systemic disruption caused either by the surgical pathology or pre-existing disease
4 Patient has severe systemic disease that is a constant threat to life
5 A patient who is moribund and will not survive without surgery

1835
Q

A sprinter attends A&E with severe leg pain. He had forgotten to warm up and ran a 100m sprint race. Towards the end of the race he experienced pain in the posterior aspect of his thigh. The pain worsens, localising to the lateral aspect of the knee. The sprinter is unable to flex the knee. What structure has been injured?

Anterior cruciate ligament

Posterior cruciate ligament

Semimembranosus tendon

Semitendinosus tendon

Biceps femoris tendon

A

The biceps femoris is commonly injured in sports that require explosive bending of the knee as seen in sprinting, especially if the athlete has not warmed up first. Avulsion most commonly occurs where the long head attaches to the ischial tuberosity. Injuries to biceps femoris are more common than to the other hamstrings.

Biceps femoris

The biceps femoris is one of the hamstring group of muscles located in the posterior upper thigh. It has two heads.

Long head
Origin Ischial tuberosity
Insertion Fibular head
Action Knee flexion, lateral rotation tibia, extension hip
Innervation Tibial division of sciatic nerve (L5, S1, S2)
Arterial supply Profunda femoris artery, inferior gluteal artery, and the superior muscular branches of popliteal artery

Short head
Origin Lateral lip of linea aspera, lateral supracondylar ridge of femur
Insertion Fibular head
Action Knee flexion, lateral rotation tibia
Innervation Common peroneal division of sciatic nerve (L5, S1, S2)
Arterial supply Profunda femoris artery, inferior gluteal artery, and the superior muscular branches of popliteal artery

1836
Q

A 72 year old man with non reconstructible arterial disease is undergoing an above knee amputation. The posterior compartment muscles are divided. Which of the following muscles does not lie in the posterior compartment of the thigh?

Biceps femoris

Quadriceps femoris

Semitendinosus

Semimembranosus

None of the above

A

The quadriceps femoris lies in the anterior compartment.

Fascial compartments of the leg

Compartments of the thigh

Formed by septae passing from the femur to the fascia lata.
Compartment Nerve Muscles Blood supply
Anterior compartment Femoral
Iliacus
Tensor fasciae latae
Sartorius
Quadriceps femoris
Femoral artery
Medial compartment Obturator
Adductor longus/magnus/brevis
Gracilis
Obturator externus
Profunda femoris artery and obturator artery
Posterior compartment (2 layers) Sciatic
Semimembranosus
Semitendinosus
Biceps femoris
Branches of Profunda femoris artery

Compartments of the lower leg
Separated by the interosseous membrane (anterior and posterior compartments), anterior fascial septum (separate anterior and lateral compartments) and posterior fascial septum (separate lateral and posterior compartments)

Compartment Nerve Muscles Blood supply
Anterior compartment Deep peroneal nerve
Tibialis anterior
Extensor digitorum longus
Extensor hallucis longus
Peroneus tertius
Anterior tibial artery
Posterior compartment Tibial
Muscles: deep and superficial compartments (separated by deep transverse fascia)
Deep: Flexor hallucis longus, Flexor digitalis longus, Tibialis posterior, Popliteus
Superficial: Gastrocnemius, Soleus, Plantaris
Posterior tibial
Lateral compartment Superficial peroneal
Peroneus longus/brevis
Peroneal artery

1837
Q

Which of the following is not a typical feature of acute appendicitis?

Neutrophilia

Profuse vomiting

Anorexia

Low grade pyrexia

Small amounts of protein on urine analysis

A

Profuse vomiting and diarrhoea are rare in early appendicitis
Whilst patients may vomit once or twice, profuse vomiting is unusual, and would fit more with gastroenteritis or an ileus. A trace of protein is not an uncommon occurrence in acute appendicitis. A free lying pelvic appendix may result in localised bladder irritation, with inflammation occurring as a secondary phenomena. This latter feature may result in patients being incorrectly diagnosed as having a urinary tract infection. A urine dipstick test is useful in differentiating between the two conditions.
Appendicitis

History
Peri umbilical abdominal pain (visceral stretching of appendix lumen and appendix is mid gut structure) radiating to the right iliac fossa due to localised parietal peritoneal inflammation.
Vomit once or twice but marked and persistent vomiting is unusual.
Diarrhoea is rare. However, pelvic appendicitis may cause localised rectal irritation and some loose stools. A pelvic abscess may also cause diarrhoea.
Mild pyrexia is common - temperature is usually 37.5 -38oC. Higher temperatures are more typical of conditions like mesenteric adenitis.
Anorexia is very common. It is very unusual for patients with appendicitis to be hungry.

Examination
Generalised peritonitis if perforation has occurred or localised peritonism.
Retrocaecal appendicitis may have relatively few signs.
Digital rectal examination may reveal boggy sensation if pelvic abscess is present, or even tenderness with a pelvic appendix.

Diagnosis
Typically raised inflammatory markers coupled with compatible history and examination findings should be enough to justify appendicectomy.
Urine analysis may show mild leucocytosis but no nitrites.
Ultrasound is useful in females where pelvic organ pathology is suspected. Although it is not always possible to visualise the appendix on ultrasound, the presence of free fluid (always pathological in males) should raise suspicion.
Treatment
Appendicectomy which can be performed via either an open or laparoscopic approach.
Administration of metronidazole reduces wound infection rates.
Patients with perforated appendicitis require copious abdominal lavage.
Patients without peritonitis who have an appendix mass should receive broad spectrum antibiotics and consideration given to performing an interval appendicectomy.
Be wary in the older patients who may have either an underlying caecal malignancy or perforated sigmoid diverticular disease.

1838
Q

What is the least likely examination finding in patients with Le Fort II fractures?

Excessive mobility of the palate

Paraesthesia in the region supplied by the inferior alveolar nerve

Malocclusion of the teeth

Enopthalmos

Parasthesia in the region supplied by the infraorbital nerve

A

Le Fort II fractures have a pyramidal shape. The fracture line involves the orbit and extends to involve the bridge of the nose and the ethmoids. In continues to involve the infraorbital rim and usually through the infraorbital foramen. As a result infraorbital parasthesia, palatal mobility and malocclusion are common findings. Severe fractures may result in enopthalmos. However, the fracture does not, by definition, involve the inferior alveolar nerve.

Craniomaxillofacial injuries

Craniomaxillofacial injuries in the UK are due to:
Interpersonal violence (52%)
Motor vehicle accidents (16%)
Sporting injuries (19%)
Falls (11%)

Le Fort Fractures
Grade Feature
Le Fort 1 The fracture extends from the nasal septum to the lateral pyriform rims, travels horizontally above the teeth apices, crosses below the zygomaticomaxillary junction, and traverses the pterygomaxillary junction to interrupt the pterygoid plates.
Le Fort 2 These fractures have a pyramidal shape and extend from the nasal bridge at or below the nasofrontal suture through the frontal process of the maxilla, inferolaterally through the lacrimal bones and inferior orbital floor and rim through or near the inferior orbital foramen, and inferiorly through the anterior wall of the maxillary sinus; it then travels under the zygoma, across the pterygomaxillary fissure, and through the pterygoid plates.
Le Fort 3 These fractures start at the nasofrontal and frontomaxillary sutures and extend posteriorly along the medial wall of the orbit through the nasolacrimal groove and ethmoid bones. The thicker sphenoid bone posteriorly usually prevents continuation of the fracture into the optic canal. Instead, the fracture continues along the floor of the orbit along the inferior orbital fissure and continues superolaterally through the lateral orbital wall, through the zygomaticofrontal junction and the zygomatic arch. Intranasally, a branch of the fracture extends through the base of the perpendicular plate of the ethmoid, through the vomer, and through the interface of the pterygoid plates to the base of the sphenoid. This type of fracture predisposes the patient to CSF rhinorrhea more commonly than the other types.

Ocular injuries
Superior orbital fissure syndrome
Severe force to the lateral wall of the orbit resulting in compression of neurovascular structures. Results in :
Complete opthalmoplegia and ptosis (Cranial nerves 3, 4, 6 and nerve to levator palpebrae superioris)
Relative afferent pupillary defect
Dilatation of the pupil and loss of accommodation and corneal reflexes
Altered sensation from forehead to vertex (frontal branch of trigeminal nerve)

Orbital blow out fracture
Typically occurs when an object of slightly larger diameter than the orbital rim strikes the incompressible eyeball. The bone fragment is displaced downwards into the antral cavity, remaining attached to the orbital periosteum. Periorbital fat may be herniated through the defect, interfering with the inferior rectus and inferior oblique muscles which are contained within the same fascial sheath. This prevents upward movement and outward rotation of the eye and the patient experiences diplopia on upward gaze. The initial bruising and swelling may make assessment difficult and patients should usually be reviewed 5 days later. Residual defects may require orbital floor reconstruction.

Nasal Fractures
Common injury
Ensure new and not old deformity
Control epistaxis
CSF rhinorrhoea implies that the cribriform plate has been breached and antibiotics will be required.
Usually best to allow bruising and swelling to settle and then review patient clinically. Major persistent deformity requires fracture manipulation, best performed within 10 days of injury.

Retrobulbar haemorrhage
Rare but important ocular emergency. Presents with:
Pain (usually sharp and within the globe)
Proptosis
Pupil reactions are lost
Paralysis (eye movements lost)
Visual acuity is lost (colour vision is lost first)
May be the result of Le Fort type facial fractures.

Management:
Mannitol 1g/Kg as 20% infusion, Osmotic diuretic, Contra-indicated in congestive heart failure and pulmonary oedema
Acetazolamide 500mg IV, (Monitor FBC/U+E) Reduces aqueous pressure by inhibition of carbonic anhydrase (used in glaucoma)
Dexamethasone 8mg orally or intravenously
In a traumatic setting an urgent cantholysis may be needed prior to definitive surgery.

Consider
Papaverine 40mg smooth muscle relaxant
Dextran 40 500mls IV improves perfusion

1839
Q

A 20 year old man develops acute appendicitis, his appendix is removed and he makes a full recovery. Which of the following pathological processes is least likely to be present in the acutely inflamed tissues?

Altered Starlings forces

Sequestration of neutrophils

Formation of fluid exudate

Formation of granulomas

None of the above

A

Neutrophil polymorphs=Acute inflammation.
Granuloma = Chronic inflammation.
Acute inflammation:
3 phases
1. Changes in blood vessel and flow: flush, flare, wheal
2. Fluid exudates (rich in protein i.e. Ig, coagulation factors) produced via increased vascular permeability
3. Cellular exudates mainly containing neutrophil polymorphs pass into extravascular space.

Neutrophils are then transported to tissues via:

a. Margination of neutrophils to the peripheral plasmatic of the vessel rather than the central axial stream
b. Pavementing: Adhesion of neutrophils to endothelial cells in venules at site of acute inflammation
c. Emigration: neutrophils pass between endothelial cells into the tissue

Acute inflammation

Inflammation is the reaction of the tissue elements to injury. Vascular changes occur, resulting in the generation of a protein rich exudate. So long as the injury does not totally destroy the existing tissue architecture, the episode may resolve with restoration of original tissue architecture.

Vascular changes
Vasodilation occurs and persists throughout the inflammatory phase.
Inflammatory cells exit the circulation at the site of injury.
The equilibrium that balances Starlings forces within capillary beds is disrupted and a protein rich exudate will form as the vessel walls also become more permeable to proteins.
The high fibrinogen content of the fluid may form a fibrin clot. This has several important immunomodulatory functions.

Sequelae
Resolution
Typically occurs with minimal initial injury
Stimulus removed and normal tissue architecture results
Organisation
Delayed removal of exudate
Tissues undergo organisation and usually fibrosis
Suppuration
Typically formation of an abscess or an empyema
Sequestration of large quantities of dead neutrophils
Progression to chronic inflammation
Coupled inflammatory and reparative activities
Usually occurs when initial infection or suppuration has been inadequately managed

Causes
Infections e.g. Viruses, exotoxins or endotoxins released by bacteria
Chemical agents
Physical agents e.g. Trauma
Hypersensitivity reactions
Tissue necrosis

Presence of neutrophil polymorphs is a histological diagnostic feature of acute inflammation

1840
Q

An 80 year old lady is being investigated for dysphagia of long duration. An OGD is attempted. The procedure is complicated and there is significant difficulty in intubating the oesophagus. Eventually, the procedure is abandoned and the patient returns to the ward. Here it is noted that a significant amount of surgical emphysema is present. Which of the diagnoses listed below is most likely to explain this presentation?

Globus

Barretts oesophagus

Squamous cell carcinoma

Pharyngeal pouch

Schatzki ring

A

The early difficulty in intubation is most likely to be the result of pharyngeal pouch. Schatzki rings can cause problems but can usually be visualized. Whilst a squamous cell carcinoma can be perforated, it would be unusual for this to occur spontaneously as perforation of cancer typically occurs after attempted therapeutic intervention.

Oesophageal disease

Disorder Features
Mallory-Weiss Tear Usually history of antecedent vomiting. This is then followed by the vomiting of a small amount of blood. There is usually little in the way of systemic disturbance or prior symptoms.
Hiatus hernia of gastric cardia Often longstanding history of dyspepsia, patients are often overweight. Uncomplicated hiatus hernias should not be associated with dysphagia or haematemesis.
Oesophageal rupture Complete disruption of the oesophageal wall in absence of pre-existing pathology. Left postero-lateral oesophageal is commonest site (2-3cm from OG junction). Suspect in patients with severe chest pain without cardiac diagnosis and signs suggestive of pneumonia without convincing history, where there is history of vomiting. Erect CXR shows infiltrate or effusion in 90% of cases(1).
Squamous cell carcinoma of the oesophagus History of progressive dysphagia. Often signs of weight loss. Usually little or no history of previous GORD type symptoms.
Adenocarcinoma of the oesophagus Progressive dysphagia, may have previous symptoms of GORD or Barretts oesophagus.
Peptic stricture Longer history of dysphagia, often not progressive. Usually symptoms of GORD. Often lack systemic features seen with malignancy
Dysmotility disorder May have dysphagia that is episodic and non progressive. Retrosternal pain may accompany the episodes.

Diagnosis
Most of the differential diagnoses listed above can be accurately categorised by upper GI endoscopy (usually most patients). Where this fails to demonstrate a mechanical stricture the use of pH and manometry studies together with radiological contrast swallows will facilitate the diagnosis.

References
Blencowe N et al. Spontaneous oesophageal rupture. BMJ 2013 (346):38-39.

1841
Q

From which embryological structure is the ureter derived?

Urachus

Cloaca

Vitello-intestinal duct

Mesonephric duct

None of the above

A

The ureter develops from an outpouching that arises from the mesonephric duct. The mesonephric duct is associated with the metanephric duct that develops within the metenephrogenic blastema. This forms the site of the ureteric bud which branches off the mesonephric duct.

Ureter

25-35 cm long
Muscular tube lined by transitional epithelium
Surrounded by thick muscular coat. Becomes 3 muscular layers as it crosses the bony pelvis
Retroperitoneal structure overlying transverse processes L2-L5
Lies anterior to bifurcation of iliac vessels
Blood supply is segmental; renal artery, aortic branches, gonadal branches, common iliac and internal iliac
Lies beneath the uterine artery

1842
Q

A 56 year old man is undergoing a radical nephrectomy via a posterior approach. Which of the following structures is most likely to be encountered during the operative approach?

8th rib

10th rib

6th rib

12th rib

9th rib

A

The 11th and 12th ribs lie posterior to the kidneys and may be encountered during a posterior approach. A pneumothorax is a recognised complication of this type of surgery.

Renal anatomy

Each kidney is about 11cm long, 5cm wide and 3cm thick. They are located in a deep gutter alongside the projecting vertebral bodies, on the anterior surface of psoas major. In most cases the left kidney lies approximately 1.5cm higher than the right. The upper pole of both kidneys approximates with the 11th rib (beware pneumothorax during nephrectomy). On the left hand side the hilum is located at the L1 vertebral level and the right kidney at level L1-2. The lower border of the kidneys is usually alongside L3.

The table below shows the anatomical relations of the kidneys:

Relations
Relations Right Kidney Left Kidney
Posterior Quadratus lumborum, diaphragm, psoas major, transversus abdominis Quadratus lumborum, diaphragm, psoas major, transversus abdominis
Anterior Hepatic flexure of colon Stomach, Pancreatic tail
Superior Liver, adrenal gland Spleen, adrenal gland

Fascial covering
Each kidney and suprarenal gland is enclosed within a common layer of investing fascia, derived from the transversalis fascia. It is divided into anterior and posterior layers (Gerotas fascia).

Renal structure
Kidneys are surrounded by an outer cortex and an inner medulla which usually contains between 6 and 10 pyramidal structures. The papilla marks the innermost apex of these. They terminate at the renal pelvis, into the ureter.
Lying in a hollow within the kidney is the renal sinus. This contains:
1. Branches of the renal artery
2. Tributaries of the renal vein
3. Major and minor calyces’s
4. Fat

Structures at the renal hilum
The renal vein lies most anteriorly, then renal artery (it is an end artery) and the ureter lies most posterior.

1843
Q

Which of the following anatomical planes separates the prostate from the rectum?

Sibsons fascia

Denonvilliers fascia

Levator ani muscle

Waldeyers fascia

None of the above

A

The Denonvilliers fascia separates the rectum from the prostate. Waldeyers fascia separates the rectum from the sacrum
Prostate gland

The prostate gland is approximately the shape and size of a walnut and is located inferior to the bladder. It is separated from the rectum by Denonvilliers fascia and its blood supply is derived from the internal iliac vessels (via inferior vesical artery). The internal sphincter lies at the apex of the gland and may be damaged during prostatic surgery, affected individuals may complain of retrograde ejaculation.

Summary of prostate gland
Arterial supply Inferior vesical artery (from internal iliac)
Venous drainage Prostatic venous plexus (to paravertebral veins)
Lymphatic drainage Internal iliac nodes
Innervation Inferior hypogastric plexus
Dimensions
Transverse diameter (4cm)
AP diameter (2cm)
Height (3cm)
Lobes
Posterior lobe: posterior to urethra
Median lobe: posterior to urethra, in between ejaculatory ducts
Lateral lobes x 2
Isthmus
Zones
Peripheral zone: subcapsular portion of posterior prostate. Most prostate cancers are here
Central zone
Transition zone
Stroma

Relations
Anterior Pubic symphysis
Prostatic venous plexus
Posterior Denonvilliers fascia
Rectum
Ejaculatory ducts
Lateral Venous plexus (lies on prostate)
Levator ani (immediately below the puboprostatic ligaments)

1844
Q

Through which of the following foramina does the genital branch of the genitofemoral nerve exit the abdominal cavity?

Superficial inguinal ring

Sciatic notch

Obturator foramen

Femoral canal

Deep inguinal ring

A

The genitofemoral nerve divides into two branches as it approaches the inguinal ligament. The genital branch passes anterior to the external iliac artery through the deep inguinal ring into the inguinal canal. It communicates with the ilioinguinal nerve in the inguinal canal (though this is seldom of clinical significance).

Genitofemoral nerve

Supplies
Small area of the upper medial thigh.

Path
Arises from the first and second lumbar nerves.
Passes obliquely through psoas major, and emerges from its medial border opposite the fibrocartilage between the third and fourth lumbar vertebrae.
It then descends on the surface of psoas major, under cover of the peritoneum
Divides into genital and femoral branches.
The genital branch passes through the inguinal canal, within the spermatic cord, to supply the skin and fascia of the scrotum. The femoral branch enters the thigh posterior to the inguinal ligament, lateral to the femoral artery. It supplies an area of skin and fascia over the femoral triangle.
It may be injured during abdominal or pelvic surgery, or during inguinal hernia repairs.

1845
Q

The femoral nerve is transected by a rather careless surgeon during a botched femoro-popliteal bypass operation. Which of the following actions will be impaired?

Extension of the great toe

Adduction of the thigh

Flexion of the knee joint

Extension of the knee joint

Eversion of the foot

A

The femoral nerve supplies the quadriceps muscle which is responsible for extension at the knee joint.

Femoral nerve

Root values L2, 3, 4
Innervates
Pectineus
Sartorius
Quadriceps femoris
Vastus lateralis/medialis/intermedius
Rectus femoris
Branches
Medial cutaneous nerve of thigh
Saphenous nerve
Intermediate cutaneous nerve of thigh

Path
Penetrates psoas major and exits the pelvis by passing under the inguinal ligament to enter the femoral triangle, lateral to the femoral artery and vein.
Mnemonic for femoral nerve supply

(don’t) M I S V Q Scan for PE
M edial cutaneous nerve of the thigh
I ntermediate cutaneous nerve of the thigh
S aphenous nerve

V astus
Q uadriceps femoris
S artorius

PE ectineus

1846
Q

Which of the nerves listed below is at greatest risk of injury with a laceration to the upper lateral margin of the popliteal fossa?

Common peroneal nerve

Sural nerve

Sciatic nerve

Saphenous nerve

Tibial nerve

A

The sural nerve exits at the lower infero-lateral aspect of the fossa and is more at risk in short saphenous vein surgery. The tibial nerve lies more medially and is even less likely to be injured in this location.

Popliteal fossa

Boundaries of the popliteal fossa
Laterally Biceps femoris above, lateral head of gastrocnemius and plantaris below
Medially Semimembranosus and semitendinosus above, medial head of gastrocnemius below
Floor Popliteal surface of the femur, posterior ligament of knee joint and popliteus muscle
Roof Superficial and deep fascia

Contents
Popliteal artery and vein
Small saphenous vein
Common peroneal nerve
Tibial nerve
Posterior cutaneous nerve of the thigh
Genicular branch of the obturator nerve
Lymph nodes

1847
Q

A 39 year old lady has recurrent attacks of biliary colic. What is the approximate volume of bile to enter the duodenum per 24 hours?

500 mL

50 mL

100 mL

2000 mL

150 mL

A

Between 500 mL and 1.5 L of bile enters the small bowel daily. Most bile salts are recycled by the enterohepatic circulation. When the gallbladder contracts the lumenal pressure is approximately 25cm water, which is why biliary colic may be so painful.

Bile

Bile is produced at a rate of between 500ml and 1500mL per day. Bile is composed of bile salts, bicarbonate, cholesterol, steroids and water. There are three main factors regulating bile flow; hepatic secretion, gall bladder contraction and sphincter of oddi resistance. Bile salts are absorbed in the terminal ileum (and recycled to the liver). Over 90% of all bile salts are recycled in this way, such that the total pool of bile salts is recycled up to six times a day.

Primary bile salts
Cholate and chenodeoxycholate.

Secondary bile salts
Formed by bacterial action on primary bile salts. These are deoxycholate and lithocholate. Of these deoxycholate is reabsorbed, whilst lithocholate is insoluble and excreted.

Pathophysiology of gallstones
Bile salts have a detergent action. They aggregate to form micelles and these have a lipid centre in which fats may be transported. Excessive quantities of cholesterol cannot be transported in this way and will tend to precipitate, resulting in the formation of cholesterol rich gallstones.

1848
Q

A 45 year old man undergoes an upper gastrointestinal endoscopy for a benign oesophageal stricture. This is dilated and he suffers an iatrogenic perforation at the site. His imaging shows a small contained leak and a small amount of surgical emphysema. What is the most appropriate nutritional option?

Nil by mouth and intravenous fluids alone

Intravenous fluids and sips orally

Total parenteral nutrition

Nasogastric feeding

PEG tube feeding

A

Iatrogenic perforations of the oesophagus may be managed non operatively. This usually involves a nil by mouth regime, tube thoracostomy may be needed. Total parenteral nutrition is the safest option. Insertion of NG feeding tubes and PEG tubes may complicate the process or allow feed to enter the perforation site.

Nutrition options in surgical patients

Oral intake
Easiest option
May be supplemented by calorie rich dietary supplements
May contra indicated following certain procedures
Naso gastric feeding
Usually administered via fine bore naso gastric feeding tube
Complications relate to aspiration of feed or misplaced tube
May be safe to use in patients with impaired swallow
Often contra indicated following head injury due to risks associated with tube insertion
Naso jejunal feeding
Avoids problems of feed pooling in stomach (and risk of aspiration)
Insertion of feeding tube more technically complicated (easiest if done intra operatively)
Safe to use following oesophagogastric surgery
Feeding jejunostomy
Surgically sited feeding tube
May be used for long term feeding
Low risk of aspiration and thus safe for long term feeding following upper GI surgery
Main risks are those of tube displacement and peritubal leakage immediately following insertion, which carries a risk of peritonitis
Percutaneous endoscopic gastrostomy
Combined endoscopic and percutaneous tube insertion
May not be technically possible in those patients who cannot undergo successful endoscopy
Risks include aspiration and leakage at the insertion site
Total parenteral nutrition
The definitive option in those patients in whom enteral feeding is contra indicated
Individualised prescribing and monitoring needed
Should be administered via a central vein as it is strongly phlebitic
Long term use is associated with fatty liver and deranged LFT’s

1849
Q

A neonate born at term has an episode of bilious vomiting and on investigation is found to have a DJ flexure displaced to the right. What procedure are they most likely to require?

Ladds procedure

Gastrojejunostomy

Kasai procedure

Duoduodenostomy

Ramstedts procedure

A

Intestinal malrotation with volvulus is treated with a Ladd’s procedure

Kasai procedure
A surgery that removes damaged and blocked bile ducts and replaces the structures with a segment of the small intestine. Often used as a bridge towards liver transplant. It is the treatment of choice for neonates with biliary atresia.

A Pyloromyotomy or Ramstedt’s procedure is where the muscle of the pylorus (at the end of the stomach) is divided to allow normal stomach emptying.

Bilious vomiting in neonates

Causes of intestinal obstruction with bilious vomiting in neonates
Disorder Incidence and causation Age at presentation Diagnosis Treatment
Duodenal atresia 1 in 5000 (higher in Downs syndrome) Few hours after birth AXR shows ‘double bubble sign, contrast study may confirm Duodenoduodenostomy
Malrotation with volvulus Usually cause by incomplete rotation during embryogenesis Usually 3-7 days after birth, volvulus with compromised circulation may result in peritoneal signs and haemodynamic instability Upper GI contrast study may show DJ flexure is more medially placed, USS may show abnormal orientation of SMA and SMV Ladd’s procedure
Jejunal/ ileal atresia Usually caused by vascular insufficiency in utero, usually 1 in 3000 Usually within 24 hours of birth AXR will show air-fluid levels Laparotomy with primary resection and anastomosis
Meconium ileus Occurs in between 15 and20% of those babies with cystic fibrosis, otherwise 1 in 5000 Typically in first 24-48 hours of life with abdominal distension and bilious vomiting Air - fluid levels on AXR, sweat test to confirm cystic fibrosis Surgical decompression, serosal damage may require segmental resection
Necrotising enterocolitis Up to 2.4 per 1000 births, risks increased in prematurity and inter-current illness Usually second week of life Dilated bowel loops on AXR, pneumatosis and portal venous air Conservative and supportive for non perforated cases, laparotomy and resection in cases of perforation of ongoing clinical deterioration

1850
Q

A 28 year old man presents with hypertension and haematuria. Haematological investigations show polycythaemia but otherwise no abnormality. CT scanning shows a left renal mass. What is the most likely cause?

Wilms tumour

Renal adenocarcinoma

Renal transitional cell carcinoma

Staghorn calculus

Renal cyst

A

Renal adenocarcinoma is the most common variant and is associated with polycythaemia.

Haematuria

Causes of haematuria

Trauma
Injury to renal tract
Renal trauma commonly due to blunt injury (others penetrating injuries)
Ureter trauma rare: iatrogenic
Bladder trauma: due to RTA or pelvic fractures
Infection
Remember TB
Malignancy
Renal cell carcinoma (remember paraneoplastic syndromes): painful or painless
Urothelial malignancies: 90% are transitional cell carcinoma, can occur anywhere along the urinary tract. Painless haematuria.
Squamous cell carcinoma and adenocarcinoma: rare bladder tumours
Prostate cancer
Penile cancers: SCC
Renal disease
Glomerulonephritis
Stones
Microscopic haematuria common
Structural abnormalities
Benign prostatic hyperplasia (BPH) causes haematuria due to hypervascularity of the prostate gland
Cystic renal lesions e.g. polycystic kidney disease
Vascular malformations
Renal vein thrombosis due to renal cell carcinoma
Coagulopathy
Causes bleeding of underlying lesions
Drugs
Cause tubular necrosis or interstitial nephritis: aminoglycosides, chemotherapy
Interstitial nephritis: penicillin, sulphonamides, and NSAIDs
Anticoagulants
Benign
Exercise
Gynaecological
Endometriosis: flank pain, dysuria, and haematuria that is cyclical
Iatrogenic
Catheterisation
Radiotherapy; cystitis, severe haemorrhage, bladder necrosis
Pseudohaematuria For example following consumption of beetroot

References
Http://bestpractice.bmj.com/best-practice/monograph/316/overview/aetiology.html

1851
Q

A 63 year old lady is undergoing an axillary sentinel lymph node biopsy as part of her breast cancer treatment. Which of the structures listed below is most likely to be encountered?

Subclavian artery

Intercostobrachial nerve

Upper cord of the brachial plexus

Lower cord of the brachial plexus

Axillary nerve

A

This can be a challenging question. A particularly careless surgeon could encounter all of these. However, during a routine level 1 axillary exploration which is where the majority of sentinel nodes will be located, the nerves most commonly encountered are the intercostobrachial nerves.

Axilla

Boundaries of the axilla
Medially Chest wall and Serratus anterior
Laterally Humeral head
Floor Subscapularis
Anterior aspect Lateral border of Pectoralis major
Fascia Clavipectoral fascia

Content:
Long thoracic nerve (of Bell) Derived from C5-C7 and passes behind the brachial plexus to enter the axilla. It lies on the medial chest wall and supplies serratus anterior. Its location puts it at risk during axillary surgery and damage will lead to winging of the scapula.
Thoracodorsal nerve and thoracodorsal trunk Innervate and vascularise latissimus dorsi.
Axillary vein Lies at the apex of the axilla, it is the continuation of the basilic vein. Becomes the subclavian vein at the outer border of the first rib.
Intercostobrachial nerves Traverse the axillary lymph nodes and are often divided during axillary surgery. They provide cutaneous sensation to the axillary skin.
Lymph nodes The axilla is the main site of lymphatic drainage for the breast.

1852
Q

A 73 year old woman collapses with sudden onset of abdominal pain and she passes a large amount of diarrhoea. On admission, she is vomiting repeatedly. She has recently been discharged from hospital following a myocardial infarct but recovered well. What is the most likely cause?

Acute superior mesenteric artery embolus

Acute inferior mesenteric artery embolus

Pancreatitis

Infection with clostridium difficile

Myocarditis

A

Sudden onset of abdominal pain and forceful bowel evacuation are features of acute mesenteric infarct. The SMA is more likely to be involved than the IMA and that makes this the most likely diagnosis. Severe vomiting is not commonly seen with clostridium difficile infection.

Mesenteric vessel disease

Mesenteric ischaemia accounts for 1 in 1000 acute surgical admissions. It is primarily caused by arterial embolism resulting in infarction of the colon. It is more likely to occur in areas such as the splenic flexure that are located at the borders of the territory supplied by the superior and inferior mesenteric arteries.

Types
Acute mesenteric embolus (commonest 50%)
Sudden onset abdominal pain followed by profuse diarrhoea.
May be associated with vomiting.
Rapid clinical deterioration.
Serological tests: WCC, lactate, amylase may all be abnormal particularly in established disease. These can be normal in the early phases.
Acute on chronic mesenteric ischaemia
Usually longer prodromal history.
Post prandial abdominal discomfort and weight loss are dominant features. Patients will usually present with an acute on chronic event, but otherwise will tend not to present until mesenteric flow is reduced by greater than 80%.
When acute thrombosis occurs presentation may be as above. In the chronic setting the symptoms will often be those of ischaemic colitis (mucosa is the most sensitive area to this insult).
Mesenteric vein thrombosis
Usually a history over weeks.
Overt abdominal signs and symptoms will not occur until venous thrombosis has reached a stage to compromise arterial inflow.
Thrombophilia accounts for 60% of cases.
Low flow mesenteric infarction
This occurs in patients with multiple co morbidities in whom mesenteric perfusion is significantly compromised by overuse of inotropes or background cardiovascular compromise.
The end result is that the bowel is not adequately perfused and infarcts occur from the mucosa outwards.

Diagnosis
Serological tests: WCC, lactate, CRP, amylase (can be normal in early disease).
Cornerstone for diagnosis of arterial AND venous mesenteric disease is CT angiography scanning in the arterial phase with thin slices (<5mm). Venous phase contrast is not helpful.
SMA duplex USS is useful in the evaluation of proximal SMA disease in patients with chronic mesenteric ischaemia.
MRI is of limited use due to gut peristalsis and movement artefact.

Management
Overt signs of peritonism: Laparotomy
Mesenteric vein thrombosis: If no peritonism: Medical management with IV heparin
At operation limited resection of frankly necrotic bowel with view to relook laparotomy at 24-48h. In the interim urgent bowel revascularisation via endovascular (preferred) or surgery.

Prognosis
Overall poor. Best outlook is from an acute ischaemia from an embolic event where surgery occurs within 12h. Survival may be 50%. This falls to 30% with treatment delay. The other conditions carry worse survival figures.

1853
Q

Which of the following does not pass through the superior orbital fissure?

Lacrimal nerve

Abducens nerve

Opthalmic artery

Trochlear nerve

Superior opthalmic vein

A

Mnemonic for the nerves passing through the supraorbital fissure:

Live Frankly To See Absolutely No Insult

Lacrimal
Frontal
Trochlear
Superior Division of Oculomotor
Abducens
Nasociliary
Inferior Division of Oculomotor nerve
The opthalmic artery arises from the internal carotid immediately after it has pierced the dura and arachnoid. It runs through the optic canal below the optic nerve and within its dural and arachnoid sheaths. It terminates as the supratrochlear and dorsal nasal arteries.

1854
Q

A 4 year old girl presents with symptoms of right sided loin pain, lethargy and haematuria. On examination she is pyrexial and has a large mass in the right upper quadrant. The most likely underlying diagnosis is:

Perinephric abscess

Nephroblastoma

Renal cortical adenoma

Grawitz tumour

Squamous cell carcinoma of the kidney

A

In a child of this age, with the symptoms described a nephroblastoma is the most likely diagnosis. A perinephric abscess is most unlikely. If an abscess were to occur it would be confined to Gerotas fascia in the first instance, and hence anterior extension would be unlikely.

Nephroblastoma

Nephroblastoma (Wilms tumours)
Usually present in first 4 years of life
May often present as a mass associated with haematuria (pyrexia may occur in 50%)
Often metastasise early (usually to lung)
Treated by nephrectomy
Younger children have better prognosis (<1 year of age =80% overall 5 year survival)

1855
Q

A 56 year old man with chronic schizophrenia undergoes a cholecystectomy. He receives metoclopramide for post operative nausea. Twenty minutes later he becomes agitated and develops marked oculogyric crises and oromandibular dystonia. Which of the following drugs may best alleviate his symptoms?

Procyclidine

Lorazepam

Chlorpromazine

Haloperidol

Sulpiride

A

This man has developed an acute dystonic reaction. Administration of further anti dopaminergic drugs will worsen the situation. Procyclidine will help to reverse the event. This is most likely to have occurred because the patient is on long term anti psychotics and has then received metoclopramide.

Acute dystonic reaction

The anti dopaminergic drugs (such as antipsychotics) may result in extrapyramidal side effects. These may range from mild parkinsonian symptoms such as resting tremor and bradykinesia. Through to acute dystonic reactions which are characterised by abnormal and involuntary facial and bodily movements, such as spasmodic torticollis, oculogyric crisis and oromandibular dystonia.

Chronic cases are generally only encountered in psychiatric units. In surgical practice the administration of the anti dopaminergic drug metoclopramide may be sufficient to precipitate an attack.

Treatment may be required if symptoms are sufficiently troublesome; benzhexol and procyclidine are two drugs which may be used.

1856
Q

A 53 year old man presents with a full thickness external rectal prolapse. Which of the following procedures would be the most suitable surgical option?

Rectopexy

Delormes

Altmeirs

Thirsch tape

Abdomino-perineal excision of the rectum

A

As this man is relatively young and has full thickness prolapse a rectopexy is the most appropriate procedure. It will give the lowest recurrence rates. This could be a sutured rectopexy or ventral mesh rectopexy. If the latter procedure is adopted, it is important to counsel the patient around the risks of sexual dysfunction (close to prostate).

Rectal prolapse

Rectal prolapse may be divided into internal and external prolapse. Patients with the former condition may have internal intussceception of the rectum and present with constipation, obstructed defecation and occasionally faecal incontinence. Patients with external rectal prolapse have a full thickness external protrusion of the rectum. Risk factors for the condition include multiparity, pelvic floor trauma and connective tissue disorders.

Diagnosis
External prolapse is usually evident. Internal prolapse may be identified by defecating proctography and examination under anaesthesia.
Sinister pathology should be excluded with endoscopy

Treatment
Perineal approaches include the Delormes operation, this avoids resection and is relatively safe but is associated with high recurrence rates. An Altmeirs operation involves a perineal excision of the sigmoid colon and rectum, it may be a more effective procedure than a Delormes but carries the risk of anastomotic leak.
Rectopexy - this is an abdominal procedure. The rectum is mobilised and fixed onto the sacral promontary. A prosthetic mesh may be inserted. The recurrence rates are low and the procedure is well tolerated (particularly if performed laparoscopically). Risks with ventral mesh rectopexy include chronic pain and visceral mesh erosions.
Thirsch tape- this is a largely historical procedure and involves encircling the rectum with tape or wire. It may be of use in a palliative setting.

1857
Q

Of the surgical incisions listed below, which is most suitable for a 45 year old female undergoing a first time renal transplant?

Abdominal midline

Paramedian

Battle

Rutherford Morrison

Pfannenstiel

A

A Rutherford Morrison incision is the traditional approach for a renal transplant and provides extra peritoneal access to the iliac vessels.
Abdominal incisions

Midline incision
Commonest approach to the abdomen
Structures divided: linea alba, transversalis fascia, extraperitoneal fat, peritoneum (avoid falciform ligament above the umbilicus)
Bladder can be accessed via an extraperitoneal approach through the space of Retzius
Paramedian incision
Parallel to the midline (about 3-4cm)
Structures divided/retracted: anterior rectus sheath, rectus (retracted), posterior rectus sheath, transversalis fascia, extraperitoneal fat, peritoneum
Incision is closed in layers
Battle
Similar location to paramedian but rectus displaced medially (and thus denervated)
Now seldom used
Kocher’s Incision under right subcostal margin e.g. Cholecystectomy (open)
Lanz Incision in right iliac fossa e.g. Appendicectomy
Gridiron Oblique incision centered over McBurneys point- usually appendicectomy (less cosmetically acceptable than Lanz
Gable Rooftop incision
Pfannenstiel’s Transverse supra pubic, primarily used to access pelvic organs
McEvedy’s Groin incision e.g. Emergency repair strangulated femoral hernia
Rutherford Morrison Extraperitoneal approach to left or right lower quadrants. Gives excellent access to iliac vessels and is the approach of choice for first time renal transplantation.

1858
Q

A 73 year old lady presents with pain in her left hip. She was walking around the house when she tripped over a rug and fell over. Apart from temporal arteritis which is well controlled with prednisolone she is otherwise well. On examination, her leg is shorted and externally rotated. Her serum alkaline phosphatase and calcium are normal. What is the likely underlying disease process?

Pagets disease

Metastatic renal cancer

Osteoporosis

Osteopetrosis

Osteoclastoma

A

The combination of age, female gender and steroids coupled with hip pain on minor trauma are strongly suggestive of osteoporosis complicated by pathological fracture.

Bone disease

Disease Features Treatment
Pagets
Focal bone resorption followed by excessive and chaotic bone deposition
Affects (in order): spine, skull, pelvis and femur
Serum alkaline phosphatase raised (other parameters normal)
Abnormal thickened, sclerotic bone on x-rays
Risk of cardiac failure with >15% bony involvement
Small risk of sarcomatous change
Bisphosphonates
Osteoporosis
Excessive bone resorption resulting in demineralised bone
Commoner in old age
Increased risk of pathological fracture, otherwise asymptomatic
Alkaline phosphatase normal, calcium normal
Bisphosphonates, calcium and vitamin D
Secondary bone tumours
Bone destruction and tumour infiltration
Mirel scoring used to predict risk of fracture
Appearances depend on primary (e.g.sclerotic - prostate, lytic - breast)
Elevated serum calcium and alkaline phosphatase may be seen
Radiotherapy, prophylactic fixation and analgesia

1859
Q

An athletic 15 year old boy presents with knee pain of 3 weeks duration. It is worst during activity and settles with rest. On examination, there is tenderness overlying the tibial tuberosity and an associated swelling at this site. What is the diagnosis?

Chondromalacia patellae

Avulsion fracture of the tibial tubercle

Osgood Schlatters disease

Quadriceps tendon rupture

Undisplaced fracture patella

A

Athletic boys and girls may develop this condition in their teenage years. It is caused by multiple micro fractures at the point of insertion of the tendon into the tibial tuberosity. Most cases settle with physiotherapy and rest.

Knee injury

Types of injury

Ruptured anterior cruciate ligament
Sport injury
Mechanism: high twisting force applied to a bent knee
Typically presents with: loud crack, pain and RAPID joint swelling (haemoarthrosis)
Poor healing
Management: intense physiotherapy or surgery
Ruptured posterior cruciate ligament
Mechanism: hyperextension injuries
Tibia lies back on the femur
Paradoxical anterior draw test
Rupture of medial collateral ligament
Mechanism: leg forced into valgus via force outside the leg
Knee unstable when put into valgus position
Menisceal tear
Rotational sporting injuries
Delayed knee swelling
Joint locking (Patient may develop skills to ‘unlock’ the knee
Recurrent episodes of pain and effusions are common, often following minor trauma
Chondromalacia patellae
Teenage girls, following an injury to knee e.g. Dislocation patella
Typical history of pain on going downstairs or at rest
Tenderness, quadriceps wasting
Dislocation of the patella
Most commonly occurs as a traumatic primary event, either through direct trauma or through severe contraction of quadriceps with knee stretched in valgus and external rotation
Genu valgum, tibial torsion and high riding patella are risk factors
Skyline x-ray views of patella are required, although displaced patella may be clinically obvious
An osteochondral fracture is present in 5%
The condition has a 20% recurrence rate
Fractured patella
2 types:
i. Direct blow to patella causing undisplaced fragments
ii. Avulsion fracture
Tibial plateau fracture
Occur in the elderly (or following significant trauma in young)
Mechanism: knee forced into valgus or varus, but the knee fractures before the ligaments rupture
Varus injury affects medial plateau and if valgus injury, lateral plateau depressed fracture occurs
Classified using the Schatzker system (see below)

Schatzker Classification system for tibial plateau fractures
Type Anatomical description Features
1 Vertical split of lateral condyle Fracture through dense bone, usually in the young. It may be virtually undisplaced, or the condylar fragment may be pushed inferiorly and tilted
2 Vertical split of the lateral condyle combined with an adjacent load bearing part of the condyle The wedge fragment (which may be of variable size), is displaced laterally; the joint is widened. Untreated, a valgus deformity may develop
3 Depression of the articular surface with intact condylar rim The split does not extend to the edge of the plateau. Depressed fragments may be firmly embedded in subchondral bone, the joint is stable
4 Fragment of the medial tibial condyle Two injuries are seen in this category; (1) a depressed fracture of osteoporotic bone in the elderly. (2) a high energy fracture resulting in a condylar split that runs from the intercondylar eminence to the medial cortex. Associated ligamentous injury may be severe
5 Fracture of both condyles Both condyles fractured but the column of the metaphysis remains in continuity with the tibial shaft
6 Combined condylar and subcondylar fractures High energy fracture with marked comminution

1860
Q

A 53 year old alcoholic develops acute pancreatitis and is making slow but reasonable progress. He is troubled by persisting ileus and for this reason a CT scan is undertaken. This demonstrates a large pancreatic pseudocyst. This is monitored by repeat CT scanning which shows no resolution and he is now complaining of early satiety. What is the best course of action?

Pancreatectomy

Emergency cystogastrostomy

Elective cystogastrostomy

ERCP

Staging laparotomy to assess severity

A

A pseudocyst is a late complication of pancreatitis and is managed in the elective setting.
Drainage of this man’s pseudocyst is required. This could be accomplished radiologically or endoscopically or surgically. As the other options are not on the list this is the best option from those available.

Management of Pancreatitis

Management of Acute Pancreatitis in the UK

Diagnosis
Traditionally hyperamylasaemia has been utilised with amylase being elevated three times the normal range.
However, amylase may give both false positive and negative results.
Serum lipase is both more sensitive and specific than serum amylase. It also has a longer half life.
Serum amylase levels do not correlate with disease severity.

Differential causes of hyperamylasaemia
Acute pancreatitis
Pancreatic pseudocyst
Mesenteric infarct
Perforated viscus
Acute cholecystitis
Diabetic ketoacidosis

Assessment of severity
Glasgow, Ranson scoring systems and APACHE II
Biochemical scoring e.g. using CRP

Features that may predict a severe attack within 48 hours of admission to hospital
Initial assessment
Clinical impression of severity
Body mass index >30
Pleural effusion
APACHE score >8
24 hours after admission
Clinical impression of severity
APACHE II >8
Glasgow score of 3 or more
Persisting multiple organ failure
CRP>150
48 hours after admission
Glasgow Score of >3
CRP >150
Persisting or progressive organ failure
Table adapted from UK guidelines for management of acute pancreatitis. GUT 2005, 54 suppl III

Management

Nutrition
There is reasonable evidence to suggest that the use of enteral nutrition does not worsen the outcome in pancreatitis
Most trials to date were underpowered to demonstrate a conclusive benefit.
The rationale behind feeding is that it helps to prevent bacterial translocation from the gut, thereby contributing to the development of infected pancreatic necrosis.

Use of antibiotic therapy
Many UK surgeons administer antibiotics to patients with acute pancreatitis. However, there is very little evidence to support this practice.
A recent Cochrane review highlights the potential benefits of administering Imipenem to patients with established pancreatic necrosis in the hope of averting the progression to infection.
There are concerns that the administration of antibiotics in mild attacks of pancreatitis will not affect outcome and may contribute to antibiotic resistance and increase the risks of antibiotic associated diarrhoea.

Surgery
Patients with acute pancreatitis due to gallstones should undergo early cholecystectomy.
Patients with obstructed biliary system due to stones should undergo early ERCP.
Patients with extensive necrosis where infection is suspected should usually undergo FNA for culture.
Patients with infected necrosis should undergo either radiological drainage or surgical necrosectomy. The choice of procedure depends upon local expertise.

References
www.bsg.org.uk/pdfworddocs/pancreatic.pdf

Antibiotic therapy for prophylaxis against infection of pancreatic necrosis in acute pancreatitis. Villatoro et al. Cochrane Library DOI: 10.1002/14651858.CD002941.pub3. 2010 version.

1861
Q

A 45 year old man has a 4 week history of epigastric discomfort which is relieved by eating. He develops haematemesis and undergoes an upper GI endoscopy. An actively bleeding ulcer is noted in the first part of the duodenum. What is the best management?

Whipples procedure

Truncal vagotomy and drainage

Distal gastrectomy

Injection with tranexamic acid

Injection with adrenaline

A

Current guidance is that bleeding peptic ulcers should be treated with dual therapeutic modalities. Adrenaline injection should be augmented with an additional therapy such as endoscopic clipping where this is available.
Bleeding duodenal ulcers will usually undergo adrenaline injection. This may be augmented by the placement of endoscopic clips or heat therapy with endoscopic heater probes. Following these interventions patients should receive a proton pump inhibitor infusion. Those who re-bleed, may require surgery. For ulcers in this location, laparotomy, duodenotomy and under-running of the ulcer is usually performed.

Upper gastrointestinal bleeding

Patients may present with the following:
Haematemesis and/ or malaena
Epigastric discomfort
Sudden collapse

The extent to which these will occur will depend upon the source. Mortality is higher in patients presenting with haematemesis than malaena alone.

Oesophageal bleeding
Cause Presenting features
Oesophagitis Small volume of fresh blood, often streaking vomit. Malaena rare. Often ceases spontaneously. Usually history of antecedent GORD type symptoms.
Cancer Usually small volume of blood, except as pre terminal event with erosion of major vessels. Often associated symptoms of dysphagia and constitutional symptoms such as weight loss. May be recurrent until malignancy managed.
Mallory Weiss Tear Typically brisk small to moderate volume of bright red blood following bout of repeated vomiting. Malaena rare. Usually ceases spontaneously.
Varices Usually large volume of fresh blood. Swallowed blood may cause malaena. Often associated with haemodynamic compromise. May stop spontaneously but re-bleeds are common until appropriately managed.

Gastric Bleeding
Cause Presenting features
Gastric cancer May be frank haematemesis or altered blood mixed with vomit. Usually prodromal features of dyspepsia and may have constitutional symptoms. Amount of bleeding variable but erosion of major vessel may produce considerable haemorrhage.
Dieulafoy Lesion Often no prodromal features prior to haematemesis and malaena, but this arteriovenous malformation may produce quite considerable haemorrhage and may be difficult to detect endoscopically.
Diffuse erosive gastritis Usually haematemesis and epigastric discomfort. Usually there is an underlying cause such as recent NSAID usage. Large volume haemorrhage may occur with considerable haemodynamic compromise.
Gastric ulcer Small low volume bleeds more common so would tend to present as iron deficiency anaemia. Erosion into a significant vessel may produce considerable haemorrhage and haematemesis.

Duodenum
Most common cause of major haemorrhage is a posteriorly sited duodenal ulcer. However, ulcers at any site in the duodenum may present with haematemesis, malaena and epigastric discomfort. The pain of duodenal ulcer is slightly different to that of gastric ulcers and often occurs several hours after eating. Peri ampullary tumours may bleed but these are rare. In patients with previous abdominal aortic aneurysm surgery aorto-enteric fistulation remains a rare but important cause of major haemorrhage associated with high mortality.

Management
Admission to hospital careful monitoring, cross match blood, check FBC, LFTs, U+E and Clotting (as a minimum)
Patients with on-going bleeding and haemodynamic instability are likely to require O negative blood pending cross matched blood
Early control of airway is vital (e.g. Drowsy patient with liver failure)
Patients with suspected varices should receive terlipressin prior to endoscopy
Ideally all patients admitted with upper gastrointestinal haemorrhage should undergo Upper GI endoscopy within 24 hours of admission. In those who are unstable this should occur immediately after resuscitation or in tandem with it. The endoscopy department is a potentially dangerous place for unstable patients and it may be safer to perform the endoscopy in theatre with an anaesthetist present.
Varices should be banded or subjected to sclerotherapy. If this is not possible owing to active bleeding then a Sengstaken- Blakemore tube (or Minnesota tube) should be inserted. This should be done with care; gastric balloon should be inflated first and oesophageal balloon second. Remember the balloon will need deflating after 12 hours (ideally sooner) to prevent necrosis. Portal pressure should be lowered by combination of medical therapy +/- TIPSS.
Patients with erosive oesophagitis / gastritis should receive a proton pump inhibitor.
Mallory Weiss tears will typically resolve spontaneously
Identifiable bleeding points should receive combination therapy of injection of adrenaline and either a thermal or mechanical treatment. All who have received intervention should receive a continuous infusion of a proton pump inhibitor (IV omeprazole for 72 hours) to reduce the re-bleeding rate.
Patients with diffuse erosive gastritis who cannot be managed endoscopically and continue to bleed may require gastrectomy
Bleeding ulcers that cannot be controlled endoscopically may require laparotomy and ulcer underruning

Indications for surgery
Patients > 60 years
Continued bleeding despite endoscopic intervention
Recurrent bleeding
Known cardiovascular disease with poor response to hypotension

Surgery
Duodenal ulcer
Laparotomy, duodenotomy and under running of the ulcer. If bleeding is brisk then the ulcer is almost always posteriorly sited and will have invaded the gastroduodenal artery. Large bites using 0 Vicryl are taken above and below the ulcer base to occlude the vessel. The duodenotomy should be longitudinal but closed transversely to avoid stenosis.

For gastric ulcer
Under-running of the bleeding site
Partial gastrectomy-antral ulcer
Partial gastrectomy or under running the ulcer- lesser curve ulcer (involving left gastric artery)
Total gastrectomy if bleeding persists

Summary of Acute Upper GI bleeding recommendations:
The need for admission and timing of endoscopic intervention may be predicted by using the Blatchford score. This considers a patients Hb, serum urea, pulse rate and blood pressure. Those patients with a score of 0 are low risk, all others are considered high risk and require admission and endoscopy.
The requirement for pre endoscopic proton pump inhibition is contentious. In the UK the National Institute of Clinical Excellence guidelines suggest the pre endoscopic PPI therapy is unnecessary. Whilst it is accepted that such treatment has no impact on mortality or morbidity a Cochrane review of this practice in 2007 did suggest that it reduced the stigmata of recent haemorrhage at endoscopy. As a result many will still administer PPI to patients prior to endoscopic intervention.
Following endoscopy it is important to calculate the Rockall score for patients to determine their risk of rebleeding and mortality. A score of 3 or less is associated with a rebleeding rate of 4% and a very low risk of mortality and identifies a group of patients suitable for early discharge.

References
1. http://www.sign.ac.uk/guidelines/fulltext/105/index.html
2. Joint Advisory Group on Endoscopy (JAG) Guidelines - http://www.thejag.org.uk
3. NICE Guideline: Management of acute upper GI bleeding. July 2012.

1862
Q

A 73 year old lady is admitted with acute mesenteric ischaemia. A CT angiogram is performed and a stenotic lesion is noted at the origin of the superior mesenteric artery. At which of the following levels does this branch from the aorta?

L1

L2

L3

L4

L5

A

The SMA leaves the aorta at L1. It passes under the neck of the pancreas prior to giving its first branch the inferior pancreatico-duodenal artery.

Superior mesenteric artery

Branches off aorta at L1
Supplies small bowel from duodenum (distal to ampulla of vater) through to mid transverse colon
Takes more oblique angle from aorta and thus more likely to recieve emboli than coeliac axis

Relations of superior mesenteric artery
Superiorly Neck of pancreas
Postero-inferiorly Third part of duodenum
Uncinate process
Posteriorly Left renal vein
Right Superior mesenteric vein

Branches of the superior mesenteric artery
Inferior pancreatico-duodenal artery
Jejunal and ileal arcades
Ileo-colic artery
Right colic artery
Middle colic artery

1863
Q

An elderly diabetic male presents with a severe deep seated otalgia and a facial nerve palsy, he has completed a course of amoxycillin with no benefit. What is the most likely diagnosis?

Malignant otitis externa

Otosclerosis

Acoustic neuroma

Meniers disease

Viral illness

A

A combination of severe otalgia and facial nerve palsy in a diabetic should raise suspicion of malignant otitis externa. This is a condition caused by pseudomonas. It commences as otitis externa and then progresses to involve the temporal bone. Spread of the disease outside the external auditory canal occurs through the fissures of Santorini and the osseocartilaginous junction.

Disorders affecting the ear

Otitis externa
Variant Cause Features Treatment
Acute otitis externa Boil in external auditory meatus Acute pain on moving the pinna
Conductive hearing loss if lesion is large
When rupture occurs pus will flow from ear Ear packs may be used
Topical antibiotics
Operative debridement may be needed in severe cases
Chronic otitis externa Chronic combined infection in the external auditory meatus usually combined staphylococcal and fungal infection Chronic discharge from affected ear, hearing loss and severe pain rare Cleansing of the external ear and treatment with antifungal and antibacterial ear drops

Otitis media
Variant Cause Features Treatment
Acute suppurative otitis media Viral induced middle ear effusions secondary to eustacian tube dysfunction Most common in children and rare in adults
May present with symptoms elsewhere (e.g. vomiting) in children
Severe pain and sometimes fever
May present with discharge if tympanic rupture occurs Antibiotics (usually amoxycillin)
Chronic suppurative otitis media May occur with or without cholesteatoma
Those without cholesteatoma have a perforation of the pars tensa
Those with cholesteatoma have a perforation of the pars flaccida Those without cholesteatoma may complain of intermittent discharge (non offensive)
Those with cholesteatoma have impaired hearing and foul smelling discharge Simple pars tensa perforations may be managed non operatively or a myringoplasty considered if symptoms troublesome.
Pars flaccida perforations will usually require a modified radical mastoidectomy

Otosclerosis
Progressive conductive deafness
Secondary to fixation of the stapes in the oval window
Treatment is with stapedectomy and insertion of a prosthesis

Acoustic neuroma
Symptoms of gradually progressive unilateral perceptive deafness and tinnitus
Involvement of the vestibular nerve may cause vertigo
Extension to involve the facial nerve may cause weakness and then paralysis.

Pre auricular sinus
Common congenital condition in which an epithelial defect forms around the external ear
Small sinuses require no treatment
Deeper sinuses may become blocked and develop episodes of infection, they may be closely related to the facial nerve and are challenging to excise

1864
Q

An 83 year old man is admitted on the acute surgical take. His presenting symptom is of painless, profuse rectal bleeding of dark blood. His medical history comprises a previous TIA for which he takes clopidogrel and a statin. What is the most likely underlying cause?

Meckels diverticulum

Colonic cancer

Diverticular bleed

Ischaemic colitis

Diverticulitis

A

The majority of patients with colonic bleeding will be found to have bleeding secondary to diverticular disease. Of note, inflammation (i.e. diverticulitis) is not seen in such cases. Around 70% will stop bleeding spontaneously. Anti platelet and anti coagulants are sometimes complicating factors and may make bleeding less likely to cease spontaneously. Ischaemic colitis often has more dominant colitis symptoms.
Rectal bleeding

Rectal bleeding is a common cause for patients to be referred to the surgical clinic. In the clinical history it is useful to try and localise the anatomical source of the blood. Bright red blood is usually of rectal anal canal origin, whilst dark red blood is more suggestive of a proximally sited bleeding source. Blood which has entered the GI tract from a gastro-duodenal source will typically resemble malaena due to the effects of the digestive enzymes on the blood itself.

In the table below we give some typical bleeding scenarios together with physical examination findings and causation.

Cause Type of bleeding Features in history Examination findings
Fissure in ano Bright red rectal bleeding Painful bleeding that occurs post defecation in small volumes. Usually antecedent features of constipation Muco-epithelial defect usually in the midline posteriorly (anterior fissures more likely to be due to underlying disease)
Haemorroids Bright red rectal bleeding Post defecation bleeding noted both on toilet paper and drips into pan. May be alteration of bowel habit and history of straining. No blood mixed with stool. No local pain. Normal colon and rectum. Proctoscopy may show internal haemorrhoids. Internal haemorrhoids are usually impalpable.
Crohns disease Bright red or mixed blood Bleeding that is accompanied by other symptoms such as altered bowel habit, malaise, history of fissures (especially anterior) and abscesses. Perineal inspection may show fissures or fistulae. Proctoscopy may demonstrate indurated mucosa and possibly strictures. Skip lesions may be noted at colonoscopy.
Ulcerative colitis Bright red bleeding often mixed with stool Diarrhoea, weight loss, nocturnal incontinence, passage of mucous PR. Proctitis is the most marked finding. Peri anal disease is usually absent. Colonoscopy will show continuous mucosal lesion.
Rectal cancer Bright red blood mixed volumes Alteration of bowel habit. Tenesmus may be present. Symptoms of metastatic disease. Usually obvious mucosal abnormality. Lesion may be fixed or mobile depending upon disease extent. Surrounding mucosa often normal, although polyps may be present.
Investigation
All patients presenting with rectal bleeding require digital rectal examination and procto-sigmoidoscopy as a minimal baseline.
Remember that haemorrhoids are typically impalpable and to attribute bleeding to these in the absence of accurate internal inspection is unsatisfactory.
In young patients with no other concerning features in the history a carefully performed sigmoidoscopy that demonstrates clear haemorrhoidal disease may be sufficient. If clear views cannot be obtained then patients require bowel preparation with an enema and a flexible sigmoidscopy performed.
In those presenting with features of altered bowel habit or suspicion of inflammatory bowel disease a colonoscopy is the best test.
Patients with excessive pain who are suspected of having a fissure may require an examination under general or local anaesthesia.
In young patients with external stigmata of fissure and a compatible history it is acceptable to treat medically and defer internal examination until the fissure is healed. If the fissure fails to heal then internal examination becomes necessary along the lines suggested above to exclude internal disease.

Special tests
In patients with a malignancy of the rectum the staging investigations comprise an MRI of the rectum to identify circumferential resection margin compromise and to identify mesorectal nodal disease. In addition to this CT scanning of the chest abdomen and pelvis is necessary to stage for more distant disease. Some centres will still stage the mesorectum with endo rectal ultrasound but this is becoming far less common.

Patients with fissure in ano who are being considered for surgical sphincterotomy and are females who have an obstetric history should probably have ano rectal manometry testing performed together with endo anal ultrasound. As this service is not universally available it is not mandatory but in the absence of such information there are continence issues that may arise following sphincterotomy.

Management

Disease Management
Fissure in ano GTN ointment 0.2% or diltiazem cream applied topically is the usual first line treatment. Botulinum toxin for those who fail to respond. Internal sphincterotomy for those who fail with botox, can be considered earlier in males.
Haemorroids Lifestyle advice, for small internal haemorrhoids can consider injection sclerotherapy or rubber band ligation. For external haemorrhoids consider haemorrhoidectomy. Modern options include HALO procedure and stapled haemorrhoidectomy.
Inflammatory bowel disease Medical management- although surgery may be needed for fistulating Crohns (setons).
Rectal cancer Anterior resection or abdomino-perineal excision of the colon and rectum. Total mesorectal excision is now standard of care. Most resections below the peritoneal reflection will require defunctioning ileostomy. Most patients will require preoperative radiotherapy.

1865
Q

A 2 day old premature neonate is born by emergency cesarean section for maternal illness. The baby is noted to become floppy and unresponsive. What is the most likely cause?

Intraventricular haemorrhage

Sub arachnoid haemorrhage

Acute sub dural haematoma

Acute extra dural haemhorrage

Chronic sub dural haematoma

A

Neonatal deterioration in premature babies is not infrequently due to intra ventricular haemorrhage. In extreme prematurity the prognosis can be very poor.

Intra cranial haemorrhage

Extradural haematoma Bleeding into the space between the dura mater and the skull. Often results from acceleration-deceleration trauma or a blow to the side of the head. The majority of extradural haematomas occur in the temporal region where skull fractures cause a rupture of the middle meningeal artery.

Features
Raised intracranial pressure
Some patients may exhibit a lucid interval
Subdural haematoma Bleeding into the outermost meningeal layer. Most commonly occur around the frontal and parietal lobes. May be either acute or chronic.

Risk factors include old age and alcoholism.

Slower onset of symptoms than a extradural haematoma.
Intracerebral haematoma Usually hyperdense lesions on CT scanning. Arise in areas of traumatic contusion which fuse to become a haematoma. Areas of clot and fresh blood may co-exist on the same CT scan (Swirl sign). Large haematomas and those associated with mass effect should be evacuated.
Subarachnoid haemorrhage Usually occurs spontaneously in the context of a ruptured cerebral aneurysm but may be seen in association with other injuries when a patient has sustained a traumatic brain injury
Intraventricular haemorrhage Haemorrhage that occurs into the ventricular system of the brain. It is relatively rare in adult surgical practice and when it does occur, it is typically associated with severe head injuries. In premature neonates it may occur spontaneously. The blood may clot and occlude CSF flow, hydrocephalus may result.
In neonatal practice the vast majority of IVH occur in the first 72 hours after birth, the aetiology is not well understood and it is suggested to occur as a result of birth trauma combined with cellular hypoxia, together with the delicate neonatal CNS.

1866
Q

Which operation below is an amputation of the lower limb in which the femoral condyles are removed and the patella retained?

Transfemoral amputation

Gritti Stokes amputation

Symes amputation

Below knee amputation with Burgess flap

Below knee amputation with Skew flap

A

This is a Gritti - Stokes amputation. During a Gritti - Stokes operation the patella is conserved and swung posteriorly to cover the distal femoral surface.

Amputations

Amputations are indicated when the affected limb is one of the following:
Dead non viable
Deadly where it is posing a major threat to life
Dead useless where it is viable but a prosthesis would be preferable

Orthopaedic surgery
Amputation is often undertaken as an option of last resort e.g. Limb salvage has failed and the limb is so non functional that mobility needs would be best met with prosthesis.
Chronic fracture non union or significant limb shortening following trauma would fit into this category. Occasionally following major trauma a primary amputation is preferable. This would be the case in an open fracture with major distal neurovascular compromise and other more life threatening injuries are present.

Vascular surgery
The first two categories are the most prevalent.
Diabetic foot sepsis is often a major cause of sepsis which can spread rapidly in the presence of established peripheral vascular disease.
As a general rule the main issue in vascular surgery is to optimise vascular inflow prior to surgery. The more distal the planned amputation is to be, the more important this rule becomes.
In other situations there has been something such as an embolic event that has not been revascularised in time. In this case the limb shows fixed mottling and an amputation will be needed.

Types of amputations
As the vast majority of commonly performed amputations affect the lower limbs these will be covered here.

The main categories of amputations are:
Pelvic disarticulation (hindquarter)
Above knee amputation
Gritti Stokes (through knee amputation)
Below knee amputation (using either Skew or Burgess flaps)
Syme’s amputation (through ankle)
Amputations of mid foot and digits

Choosing a level of amputation depends on:
The disease process being treated
Desired functional outcome
Co-morbidities of the patient

Above knee amputations
Quick to perform
Heal reliably
Patients regain their general health quickly
For this benefit, a functional price has to be paid and many patients over the age of 70 will never walk on an above knee prosthesis.
Above knee amputations use equal anterior-posterior flaps

Below knee amputations
Technically more challenging to perform
Heal less reliably than their above knee counterparts.
However, many more patients are able to walk using a below knee prosthesis.
In below knee amputations the two main flaps are Skew flaps or the Burgess long posterior flap. Skew flaps result in a less bulky limb that is easier to attach a prosthesis to.

It is worth remembering that whilst it may be technically feasible to offer a below knee amputation there may be circumstances where an above knee option is preferable. For example, in fixed flexion deformities of the lower limb, little functional benefit would be gained from below knee amputation surgery.

1867
Q

A 6 year old boy with rhinorrhoea, nasal congestion and a cough is brought to the emergency department. They are assessed by one of the clinicians. The nasal cavity is examined with an otoscope, to visualise the cavity, the childs head is tilted posteriorly. As the scope advances past the nostrils, which of the structures listed below will be visualised first?

Choana

Inferior nasal concha

Middle nasal concha

Torus tubarius

Superior nasal concha

A

The inferior nasal concha is the most inferior and anterior and would be seen first. The choana is located more posteriorly. The torus tubaris is the tubal elevation and is an elevation of the cartilage and mucous membrane that covers the Eustachian tube.

Nasal cavity

There are two nasal cavities separated by the median nasal septum. This structure consists of septal cartilage and two bony parts: the vomer and the perpendicular plate of the ethmoid bone. Each cavity is approximately 5cm high and 7cm long, from the nares anteriorly to the choanae posteriorly. The lateral walls of each cavity are mainly made up of the maxilla. However, the lacrimal, ethmoid and palatine bones also contribute. The ethmoid gives rise to the two upper protrusions from the lateral walls, the superior and middle conchae. The inferior concha is a separate bone. Above the superior concha lies the sphenoethmoidal recess. The spaces inferior to conchae are the superior, middle and inferior meatus. The structures draining into each space are shown below:

Sphenoethmoidal recess Sphenoidal sinus
Superior meatus Posterior ethmoidal sinus
Middle meatus Frontal sinus, maxillary sinus ,anterior and middle ethmoidal sinus
Inferior meatus Nasolacrimal duct

The main arterial supply to the septum and lateral walls is the sphenopalatine artery, a branch of the maxillary artery. The olfactory nerve conveys special sensation. Common sensation from the nasal mucosa is conveyed by the trigeminal nerve.

1868
Q

A 53 year old lady presents with a creamy nipple discharge. On examination, she has discharge originating from multiple ducts and associated nipple inversion. What is the most likely cause?

Ductal papilloma

Duct ectasia

DCIS

LCIS

Tuberculosis

A

Duct ectasia is common during the period of breast involution that occurs during the menopausal period. As the ducts shorten they may contain inspissated material. The discharge will often discharge from several ducts.

Non malignant breast disease

Duct ectasia
Mammary duct ectasia may be seen in up to 25% of normal female breasts
Patients usually present with nipple discharge, which may be from single or multiple ducts (usually present age >50 years)
The discharge is often thick and green
Duct ectasia is a normal variant of breast involution and is not the same condition as periductal mastitis
Mass develops behind nipple

Periductal mastitis
Present at younger age than duct ectasia
May present with features of inflammation, abscess or mammary duct fistula
Strongly associated with smoking
Usually treated with antibiotics, abscess will require drainage
Mass develops around nipple

Intraductal papilloma
Growth of papilloma in a single duct
Usually presents with clear or blood stained discharge originating from a single duct
No increase in risk of malignancy

Breast abscess
Lactational mastitis is common
Infection is usually with Staphylococcus aureus
On examination there is usually a tender fluctuant mass
Treatment is with antibiotics and ultrasound guided aspiration
Overlying skin necrosis is an indication for surgical debridement, which may be complicated by the development of a subsequent mammary duct fistula.

Tuberculosis
Rare in western countries, usually secondary TB
Affects women later in child bearing period
Chronic breast or axillary sinus is present in up to 50% cases
Diagnosis is by biopsy culture and histology

1869
Q

A parent brings her 4 year old child to the surgical clinic. She has noticed an intermittent swelling in the right scrotum that is worse in the evening. On examination he has a soft fluctuant swelling in the right scrotum that cannot be separated from the testis. It transilluminates when a pen torch is held against it. What is the best course of action?

Ligation of patent processus vaginalis via inguinal approach

Ligation of patent processus vaginalis via a scrotal approach

Jaboulay procedure via scrotal approach

Lords procedure via scrotal approach

Aspiration

A

In children, a hydrocele is most commonly due to a persistent processus vaginalis. The swelling is intermittent and in most cases that are diagnosed in infancy the hydrocele resolves. Cases that persist beyond two years of age are best managed surgically and the surgical approach is usually made via the inguinal canal where the patent processus is identified and ligated.

Scrotal swelling

Differential diagnosis
Inguinal hernia If inguinoscrotal swelling; cannot ‘get above it’ on examination
Cough impulse may be present
May be reducible
Testicular tumours Often discrete testicular nodule (may have associated hydrocele)
Symptoms of metastatic disease may be present
USS scrotum and serum AFP and β HCG required
Acute epididymo-orchitis Often history of dysuria and urethral discharge
Swelling may be tender and eased by elevating testis
Most cases due to Chlamydia
Infections with other gram negative organisms may be associated with underlying structural abnormality
Epididymal cysts Single or multiple cysts
May contain clear or opalescent fluid (spermatoceles)
Usually occur over 40 years of age
Painless
Lie above and behind testis
It is usually possible to ‘get above the lump’ on examination
Hydrocele Non painful, soft fluctuant swelling
Often possible to ‘get above it’ on examination
Usually contain clear fluid
Will often transilluminate
May be presenting feature of testicular cancer in young men
Testicular torsion Severe, sudden onset testicular pain
Risk factors include abnormal testicular lie
Typically affects adolescents and young males
On examination testis is tender and pain not eased by elevation
Urgent surgery is indicated, the contra lateral testis should also be fixed
Varicocele Varicosities of the pampiniform plexus
Typically occur on left (because testicular vein drains into renal vein)
May be presenting feature of renal cell carcinoma
Affected testis may be smaller and bilateral varicoceles may affect fertility

Management
Testicular malignancy is always treated with orchidectomy via an inguinal approach. This allows high ligation of the testicular vessels and avoids exposure of another lymphatic field to the tumour.
Torsion is commonest in young teenagers and the history in older children can be difficult to elicit. Intermittent torsion is a recognised problem. The treatment is prompt surgical exploration and testicular fixation. This can be achieved using sutures or by placement of the testis in a Dartos pouch.
Varicoceles are usually managed conservatively. If there are concerns about testicular function of infertility then surgery or radiological management can be considered.
Epididymal cysts can be excised using a scrotal approach
Hydroceles are managed differently in children where the underlying pathology is a patent processus vaginalis and therefore an inguinal approach is used in children so that the processus can be ligated. In adults a scrotal approach is preferred and the hydrocele sac excised or plicated.

1870
Q

A 23 year old man is admitted to the emergency room with a sudden collapse. A FAST scan shows a large amount of free blood and an enlarged spleen. Which of the conditions below is most likely to account for this?

Acute rubella infection

Mumps infection

Infection with Epstein Barr virus

Infection with influenza

Infection with COVID 19

A

EBV infection is the classic cause of sudden spontaneous splenic rupture. Of the many conditions linked to COVID 19, splenic rupture is not one of them.

Disorders affecting the spleen

One of the commonest conditions to affect the spleen is traumatic rupture and this is addressed elsewhere.

Splenomegaly
Enlargement of the spleen can commonly accompany a variety of haematological disorders. Of these, chronic conditions such as haemolytic anaemia and chronic leukaemia are often associated with splenomegaly. More acute haematological conditions tend not to cause massive splenic enlargement. Infections with parasites (such as malaria) and viruses such as Epstein Barr virus can cause splenomegaly and this one condition that is associated with sudden spontaneous splenic rupture. Portal hypertension can result in splenomegaly and the finding of an enlarged spleen on abdominal examination should prompt a search for the other stigmata of chronic liver disease.

Splenic atrophy
This typically occurs as a result of conditions that impair the blood flow to the spleen. Division of some of the major vessels can cause shrinkage of the spleen and this is commonly seen when splenic artery aneurysms are coiled for example.

1871
Q

Surgical occlusion of which of these structures, will result in the greatest reduction in hepatic blood flow?

Portal vein

Common hepatic artery

Right hepatic artery

Coeliac axis

Left hepatic artery

A

The portal vein transports 70% of the blood supply to the liver, while the hepatic artery provides 30%. The portal vein contains the products of digestion. The arterial and venous blood is dispersed by sinusoids to the central veins of the liver lobules; these drain into the hepatic veins and then into the IVC. The caudate lobe drains directly into the IVC rather than into other hepatic veins.

Liver

Structure of the liver
Right lobe
Supplied by right hepatic artery
Contains Couinaud segments V to VIII (-/+Sg I)
Left lobe
Supplied by the left hepatic artery
Contains Couinaud segments II to IV (+/- Sg1)
Quadrate lobe
Part of the right lobe anatomically, functionally is part of the left
Couinaud segment IV
Porta hepatis lies behind
On the right lies the gallbladder fossa
On the left lies the fossa for the umbilical vein
Caudate lobe
Supplied by both right and left hepatic arteries
Couinaud segment I
Lies superior to the porta hepatis
Anterior and medial to the inferior vena cava
Bile from the caudate lobe drains into both right and left hepatic ducts

Detailed knowledge of Couinaud segments is not required for MRCS
Between the liver lobules are portal canals which contain the portal triad: Hepatic Artery, Portal Vein, tributary of Bile Duct.

Relations of the liver
Anterior Postero inferiorly
Diaphragm Oesophagus
Xiphoid process Stomach
Duodenum
Hepatic flexure of colon
Right kidney
Gallbladder
Inferior vena cava

Porta hepatis
Location Postero inferior surface, it joins nearly at right angles with the left sagittal fossa, and separates the caudate lobe behind from the quadrate lobe in front
Transmits
Common hepatic duct
Hepatic artery
Portal vein
Sympathetic and parasympathetic nerve fibres
Lymphatic drainage of the liver (and nodes)

Ligaments
Falciform ligament
2 layer fold peritoneum from the umbilicus to anterior liver surface
Contains ligamentum teres (remnant umbilical vein)
On superior liver surface it splits into the coronary and left triangular ligaments
Ligamentum teres Joins the left branch of the portal vein in the porta hepatis
Ligamentum venosum Remnant of ductus venosus

Arterial supply
Hepatic artery

Venous
Hepatic veins
Portal vein

Nervous supply
Sympathetic and parasympathetic trunks of coeliac plexus

1872
Q

Which of the following is responsible for the release and synthesis of calcitonin?

Parathyroid glands

Anterior pituitary

Thyroid gland

Posterior pituitary

Adrenal glands

A

Calcitonin has the opposite effect of PTH and is released from the thyroid gland.

Calcium homeostasis

Calcium ions are linked to a wide range of physiological processes. The largest store of bodily calcium is contained within the skeleton. Calcium levels are primarily controlled by parathyroid hormone, vitamin D and calcitonin.

Hormonal regulation of calcium
Hormone Actions
Parathyroid hormone (PTH)
Increase calcium levels and decrease phosphate levels
Increases bone resorption
Immediate action on osteoblasts to increase ca2+ in extracellular fluid
Osteoblasts produce a protein signaling molecule that activate osteoclasts which cause bone resorption
Increases renal tubular reabsorption of calcium
Increases synthesis of 1,25(OH)2D (active form of vitamin D) in the kidney which increases bowel absorption of Ca2+
Decreases renal phosphate reabsorption
1,25-dihydroxycholecalciferol (the active form of vitamin D)
Increases plasma calcium and plasma phosphate
Increases renal tubular reabsorption and gut absorption of calcium
Increases osteoclastic activity at high levels and osteoblasts at low levels
Increases renal phosphate reabsorption
Calcitonin
Secreted by C cells of thyroid
Inhibits intestinal calcium absorption
Inhibits osteoclast activity
Inhibits renal tubular absorption of calcium

Both growth hormone and thyroxine also play a small role in calcium metabolism.

1873
Q

A 47-year-old woman presents with loin pain and haematuria. Urine dipstick demonstrates:

Blood ++++
Nitrites POS
Leucocytes +++
Protein ++

Urine culture shows a Proteus infection. An x-ray demonstrates a stag-horn calculus in the left renal pelvis. What is the most likely composition of the renal stone?

Xanthine

Calcium oxalate

Struvite

Cystine

Urate

A

Stag-horn calculi are composed of struvite and form in alkaline urine (ammonia producing bacteria therefore predispose)

Renal stones on x-ray
cystine stones: semi-opaque
urate + xanthine stones: radio-lucent

Renal stones: imaging

The table below summarises the appearance of different types of renal stone on x-ray

Type Frequency Radiograph appearance
Calcium oxalate 40% Opaque
Mixed calcium oxalate/phosphate stones 25% Opaque
Triple phosphate stones* 10% Opaque
Calcium phosphate 10% Opaque
Urate stones 5-10% Radio-lucent
Cystine stones 1% Semi-opaque, ‘ground-glass’ appearance
Xanthine stones <1% Radio-lucent

*stag-horn calculi involve the renal pelvis and extend into at least 2 calyces. They develop in alkaline urine and are composed of struvite (ammonium magnesium phosphate, triple phosphate). Ureaplasma urealyticum and Proteus infections predispose to their formation

1874
Q

A 56 year old man is investigated with an abdominal CT scan for a change of bowel habit towards constipation. It shows no colonic lesions. However, a right sided adrenal lesion is noted and measures 2.5cm in diameter. What is the most appropriate course of action?

Arrange an MRI of the adrenal gland

Arrange an adrenal USS

Arrange an image guided core biopsy

Undertake an adrenalectomy

Arrange a hormonal assay

A

The vast majority of small adrenal lesions are incidental, benign and non functioning adenomas. Apart from minimal workup, no further investigation is needed. Of note, if there are concerns about malignancy, the only surgical option is adrenalectomy.

Adrenal lesions- Incidental

Incidentaloma of the adrenal glands have become increasingly common as CT scanning of the abdomen is widely undertaken. Prevalences range from 1.5-9% in autopsy studies. Overall, 75% will be non functioning adenomas. Investigation to exclude a functioning lesion is as below:

Investigation
Morning and midnight plasma cortisol measurements
Dexamethasone suppression test
24 hour urinary cortisol excretion
24 hour urinary excretion of catecholamines
Serum potassium, aldosterone and renin levels

Management
The risk of malignancy is related to the size of the lesion and 25% of all masses greater than 4cm will be malignant. Such lesions should usually be excised. Where a lesion is a suspected metastatic deposit a biopsy may be considered.

1875
Q

A 44 year old man has a malignant melanoma and is undergoing a block dissection of the groin. The femoral triangle is being explored for intra operative bleeding. Which of the following forms the medial border of the femoral triangle?

Femoral artery

Biceps femoris

Adductor longus

Sartorius

Adductor magnus

A

Adductor longus forms the medial boundary of the femoral triangle (see below).

Femoral triangle anatomy

Boundaries
Superiorly Inguinal ligament
Laterally Sartorius
Medially Adductor longus
Floor Iliopsoas, adductor longus and pectineus
Roof
Fascia lata and Superficial fascia
Superficial inguinal lymph nodes (palpable below the inguinal ligament)
Long saphenous vein

This is displayed as an annotated cadaveric image below, the terms FV, FA, FN refer to the femoral artery, vein and nerve respectively and the SFJ is the saphenofemoral junction.

Contents
Femoral vein (medial to lateral)
Femoral artery-pulse palpated at the mid inguinal point
Femoral nerve
Deep and superficial inguinal lymph nodes
Lateral cutaneous nerve
Great saphenous vein
Femoral branch of the genitofemoral nerve

1876
Q

A 29 year old man has had a sore throat for the past 5 days. Over the past 24 hours he has noticed increasingly severe throbbing pain in the region of his right tonsil. He is pyrexial and on examination he is noted to have a swelling of this area. What is the most likely cause?

Tonsillar cancer

Lymphoma

Quinsy

Glandular fever

Common cold

A

Unilateral swelling and fever is usually indicative of quinsy. Surgical drainage usually produces prompt resolution of symptoms.

Acute tonsillitis

Characterised by pharyngitis, fever, malaise and lymphadenopathy.
Over half of all cases are bacterial with Streptococcus pyogenes the most common organism
The tonsils are typically oedematous and yellow or white pustules may be present
Infectious mononucleosis may mimic the condition.
Treatment with penicillin type antibiotics is indicated for bacterial tonsillitis.
Bacterial tonsillitis may result in local abscess formation (quinsy)

1877
Q

A male infant is born prematurely at 26 weeks gestation by emergency cesarean section. Following the birth he develops respiratory distress syndrome and is ventilated. He begins to improve twelve days after birth. Then he becomes unwell and develops abdominal distension and passes bloody stools and vomits a small quantity of bile stained vomit. What is the most likely cause?

Ileal atresia

Hirschprungs disease

Pyloric stenosis

Necrotising enterocolitis

Meconium ileus

A

Necrotising enterocolitis often has a delayed presentation and affected infants will typically pass bloody stools. Plain films may show air in the intestinal wall (Pneumatosis).

Bilious vomiting in neonates

Causes of intestinal obstruction with bilious vomiting in neonates
Disorder Incidence and causation Age at presentation Diagnosis Treatment
Duodenal atresia 1 in 5000 (higher in Downs syndrome) Few hours after birth AXR shows ‘double bubble sign, contrast study may confirm Duodenoduodenostomy
Malrotation with volvulus Usually cause by incomplete rotation during embryogenesis Usually 3-7 days after birth, volvulus with compromised circulation may result in peritoneal signs and haemodynamic instability Upper GI contrast study may show DJ flexure is more medially placed, USS may show abnormal orientation of SMA and SMV Ladd’s procedure
Jejunal/ ileal atresia Usually caused by vascular insufficiency in utero, usually 1 in 3000 Usually within 24 hours of birth AXR will show air-fluid levels Laparotomy with primary resection and anastomosis
Meconium ileus Occurs in between 15 and20% of those babies with cystic fibrosis, otherwise 1 in 5000 Typically in first 24-48 hours of life with abdominal distension and bilious vomiting Air - fluid levels on AXR, sweat test to confirm cystic fibrosis Surgical decompression, serosal damage may require segmental resection
Necrotising enterocolitis Up to 2.4 per 1000 births, risks increased in prematurity and inter-current illness Usually second week of life Dilated bowel loops on AXR, pneumatosis and portal venous air Conservative and supportive for non perforated cases, laparotomy and resection in cases of perforation of ongoing clinical deterioration

1878
Q

In which of the following conditions is a Ghon complex most likely to be found?

Actinomycosis

Infection with mycobacterium tuberculosis

Mycosis fungoides

Lewy body dementia

Chronic lymphocytic leukaemia

A

Ghon focus is primary infection
Ghon complex is primary infection and regional nodal involvement

Tuberculosis pathology

Is a form of primary chronic inflammation, caused by the inability of macrophages to kill the Mycobacterium tuberculosis.
The macrophages often migrate to regional lymph nodes, the lung lesion plus affected lymph nodes is referred to as a Ghon complex.
This leads to the formation of a granuloma which is a collection of epithelioid histiocytes.
There is the presence of caseous necrosis in the centre.
The inflammatory response is mediated by a type 4 hypersensitivity reaction.
In healthy individuals the disease may be contained, in the immunocompromised disseminated (miliary TB) may occur.

Diagnosis
Waxy membrane of mycobacteria prevents binding with normal stains. Ziehl - Neelsen staining is typically used.
Culture based methods take far longer.

Image showing acid- alcohol fast mycobacteria stained using the Ziehl- Neelsen method

1879
Q

The inferior aspect of the vagina drains to which of the following lymph node groups?

Superficial inguinal nodes

Internal iliac nodes

Para-aortic nodes

Obturator nodes

Meso-rectal nodes

A

Lymphatic drainage of the vagina

The lymph vessels from the superior aspect of the vagina join the internal and external iliac nodes, those from the inferior aspect of the vagina drain to the superficial inguinal nodes.

1880
Q

A 40 year old woman had the anterior lobe of the pituitary removed because of a tumour. Without postoperative supplements, which of the following could occur?

Failure to produce adequate amounts of thyroxine

Failure to produce parathyroid hormone in response to hypocalcaemia

Failure to secrete catecholamine in response to stress

Failure to secrete insulin in hyperglycemia

Inability to concentrate urine in response to water deprivation

A

Its an anterior pituitary lesion and that is the reason thyroxine secretion is affected. ADH is released by the posterior pituitary.

Pituitary Gland

The pituitary gland is located within the sella turcica within the sphenoid bone in the middle cranial fossa. It is covered by a dural fold and weighs around 0.5g. It is attached to the hypothalamus by the infundibulum. The anterior pituitary receives hormonal stimuli from the hypothalamus by way of the hypothalamo-pituitary portal system. It develops from a depression in the wall of the pharynx (Rathkes pouch).

Anterior pituitary hormones
Growth hormone
Thyroid stimulating hormone
ACTH
Prolactin
LH and FSH
Melanocyte releasing hormone

Posterior pituitary hormones
Oxytocin
Anti diuretic hormone

1881
Q

Which of the following statements relating to biliary atresia is untrue?

It most commonly presents as prolonged conjugated jaundice in the neonatal period.

Evidence of portal hypertension at diagnosis is seldom present in the UK.

It may be confused with Alagille syndrome.

The Kasai procedure is best performed in the first 8 weeks of life.

Survival following a successful Kasai procedure is approximately 45% at 5 years.

A

Alagille syndrome autosomal dominant disorder characterised by presence of paucity of bile ducts and cardiac defects. Only the embryonic form of biliary atresia is associated with cardiac and other embryological defects.
Biliary atresia usually presents with obstructed jaundice. A Kasai procedure is best performed in the first 8 weeks of life. If a Kasai procedure is successful most patients will not require liver transplantation. 45% of patients post Kasai procedure will require transplantation. However, overall survival following a successful Kasai procedure is 80%.

Biliary atresia

1 in 17000 affected
Biliary tree lumen is obliterated by an inflammatory cholangiopathy causing progressive liver damage

Clinical features
Infant well in 1st few weeks of life
No family history of liver disease
Jaundice in infants > 14 days in term infants (>21 days in pre term infants)
Pale stool, yellow urine (colourless in babies)
Associated with cardiac malformations, polysplenia, situs inversus

Investigation
Conjugated bilirubin (prolonged physiological jaundice or breast milk jaundice will cause a rise in unconjugated bilirubin, whereas those with obstructive liver disease will have a rise in conjugated bilirubin)
Ultrasound of the liver (excludes extrahepatic causes, in biliary atresia infant may have tiny or invisible gallbladder)
Hepato-iminodiacetic acid radionuclide scan (good uptake but no excretion usually seen)

Management
Early recognition is important to prevent liver transplantation.
Nutritional support.
Roux-en-Y portojejunostomy (Kasai procedure).
If Kasai procedure fails or late recognition, a liver transplant becomes the only option.

1882
Q

Which of these factors does not increase the risk of abdominal wound dehiscence following laparotomy?

Jaundice

Abdominal compartment syndrome

Poorly controlled diabetes mellitus

Administration of intravenous steroids

Use of Ketamine as an anaesthetic agent

A

Ketamine does not affect healing. All the other situations in the list carry a strong association with poor healing and risk of dehisence.

Abdominal wound dehiscence

  • This is a significant problem facing all surgeons who undertake abdominal surgery on a regular basis. Traditionally, it is said to occur when all layers of an abdominal mass closure fail and the viscera protrude externally (associated with 30% mortality).
    It can be subdivided into superficial, in which the skin wound alone fails and complete, implying failure of all layers.

Factors which increase the risk are:
* Malnutrition
* Vitamin deficiencies
* Jaundice
* Steroid use
* Major wound contamination (e.g. faecal peritonitis)
* Poor surgical technique (Mass closure technique is the preferred method-Jenkins Rule)

When sudden full dehiscence occurs the management is as follows:
* Analgesia
* Intravenous fluids
* Intravenous broad spectrum antibiotics
* Coverage of the wound with saline impregnated gauze (on the ward)
* Arrangements made for a return to theatre

Surgical strategy
Correct the underlying cause (e.g. TPN or NG feed if malnourished)
Determine the most appropriate strategy for managing the wound

Options
Resuturing of the wound This may be an option if the wound edges are healthy and there is enough tissue for sufficient coverage. Deep tension sutures are traditionally used for this purpose.
Application of a wound manager This is a clear dressing with removable front. Particularly suitable when some granulation tissue is present over the viscera or where there is a high output bowel fistula present in the dehisced wound.
Application of a ‘Bogota bag’ This is a clear plastic bag that is cut and sutured to the wound edges and is only a temporary measure to be adopted when the wound cannot be closed and will necessitate a return to theatre for definitive management.
Application of a VAC dressing system These can be safely used BUT ONLY if the correct layer is interposed between the suction device and the bowel. Failure to adhere to this absolute rule will almost invariably result in the development of multiple bowel fistulae and create an extremely difficult management problem.

1883
Q

Which of the following is least likely to occur in association with severe atrophic gastritis?

Gastric ulcers

Gastric cancer

Anaemia

Duodenal ulcers

Gastric polyps

A

Achlorhydria would make the formation of duodenal ulcers unlikely. Note the question states ‘least likely’.
Due to the loss of gastric acid a duodenal ulcer is unlikely. Note that gastric polyps may form (see below).

Gastritis

Type of gastritis Features
Type A Autoimmune
Circulating antibodies to parietal cells, causes reduction in cell mass and hypochlorhydria
Loss of parietal cells = loss of intrinsic factor = B12 malabsorption
Absence of antral involvement
Hypochlorhydria causes elevated gastrin levels- stimulating enterochromaffin cells and adenomas may form
Type B Antral gastritis
Associated with infection with helicobacter pylori infection
Intestinal metaplasia may occur in stomach and require surveillance endoscopy
Peptic ulceration may occur
Reflux gastritis Bile refluxes into stomach, either post surgical or due to failure of pyloric function
Histologically, evidence of chronic inflammation, and foveolar hyperplasia
May respond to therapy with prokinetics
Erosive gastritis Agents disrupt the gastric mucosal barrier
Most commonly due to NSAIDs and alcohol
With NSAIDs the effects occur secondary to COX 1 inhibition
Stress ulceration This occurs as a result of mucosal ischaemia during hypotension or hypovolaemia
The stomach is the most sensitive organ in the GI tract to ischaemia following hypovolaemia
Diffuse ulceration may occur
Prophylaxis with acid lowering therapy and sucralfate may minimise complications
Menetriers disease Gross hypertrophy of the gastric mucosal folds, excessive mucous production and hypochlorhydria
Pre malignant condition

References
Whiting J et al. The long term results of endoscopic surveillance of premalignant gastric lesions. Gut 2002; 50 :378381.

Dixon M et al. Reflux gastritis: distinct histopathological entity?J Clin Pathol 1986; 39 : 524-530.

1884
Q

A 38 year old lady presents with abdominal pain. On investigation, her serum calcium is found to be 3.5mmol/L. What is the most appropriate initial management?

Intravenous bisphosphonates

Oral bisphosphonates

Intravenous calcitonin

Intravenous 0.9% sodium chloride

Intravenous frusemide

A

The immediate treatment of hypercalcaemia involves intravenous fluid resuscitation. This may be complemented with the use of bisphosphonates and sometimes diuretics. However, fluids are administered first and because this question asks what the most appropriate initial treatment is, intravenous fluids are the most appropriate answer. Normal saline is usually preferred for this over dextrose containing solutions.

Management of hypercalcaemia

Free Ca is affected by pH (increased in acidosis) and plasma albumin concentration
ECG changes include: Shortening of QTc interval
Urgent management is indicated if:

Calcium > 3.5 mmol/l
Reduced consciousness
Severe abdominal pain
Pre renal failure

Management:
Airway Breathing Circulation
Intravenous fluid resuscitation with 3-6L of 0.9% Normal saline in 24 hours
Concurrent administration of calcitonin will also help lower calcium levels
Medical therapy (usually if Corrected calcium >3.0mmol/l)

Bisphosphonates
Analogues of pryrophosphate
Prevent osteoclast attachment to bone matrix and interfere with osteoclast activity
Inhibit bone resorption.

Agents

Drug Side effects Notes
IV Pamidronate pyrexia, leucopaenia Most potent agent
IV Zoledronate response lasts 30 days Used for malignancy associated hypercalcaemia

Calcitonin
Quickest onset of action however short duration (tachyphylaxis) therefore only given with a second agent.

Prednisolone
May be given in hypercalcaemia related to sarcoidosis, myeloma or vitamin D intoxication.

1885
Q

A 30 year old aid worker becomes unwell whilst helping at the scene of a recent earthquake. He develops vomiting and soon afterwards a diarrhoea that is loose and extremely watery. What is the most likely infective organism?

Campylobacter jejuni

Vibrio cholera

Enterohaemorragic E Coli

Clostridium perfringens

Clostridium botulinum

A

The passage of very loose and watery stools distinguishes cholera. Most gastroenteric infections do not produce such watery motions.

Bacterial Gastroenteritis

Causative organisms Features
Campylobacter jejuni
Most common cause of acute infective diarrhoea
Spiral, gram negative rods
Usually infects caecum and terminal ileum. Local lymphadenopathy is common
May mimic appendicitis as it has marked right iliac fossa pain
Reactive arthritis is seen in 1-2% of cases
Shigella spp.
Members of the enterobacteriaceae
Gram negative bacilli
Clinically causes dysentery
Shigella soneii is the commonest infective organism (mild illness)
Usually self limiting, ciprofloxacin may be required if individual is in a high risk group
Salmonella spp
Facultatively anaerobic, gram negative, enterobacteriaceae
Infective dose varies according to subtype
Salmonellosis: usually transmitted by infected meat (especially poultry) and eggs
E. coli
Enteropathogenic
Enteroinvasive: dysentery, large bowel necrosis/ulcers
Enterotoxigenic: small intestine, travelers diarrhoea
Enterohaemorrhagic: 0157, cause a haemorrhagic colitis, haemolytic uraemic syndrome and thrombotic thrombocytopaenic purpura
Yersinia enterocolitica
Gram negative, coccobacilli
Typically produces a protracted terminal ileitis that may mimic Crohns disease
Differential diagnosis acute appendicitis
May progress to septicaemia in susceptible individuals
Usually sensitive to quinolone or tetracyclines
Vibrio cholera
Short, gram negative rods
Transmitted by contaminated water, seafood
Symptoms include sudden onset of effortless vomiting and profuse watery diarrhoea
Correction of fluid and electrolyte losses are the mainstay of treatment
Most cases will resolve, antibiotics are not generally indicated

1886
Q

A 4 year boy presents with an abnormal gait. He has a history of recent viral illness. His WCC is 11 and ESR is 30. What is the most likely cause?

Perthes disease

Transient synovitis

Septic arthritis

Slipped upper femoral epiphysis

Osteomyelitis

A

Viral illnesses can be associated with transient synovitis. The WCC should ideally be > 12 and the ESR > 40 to suggest septic arthritis.
Paediatric orthopaedics

Diagnosis Mode of presentation Treatment Radiology
Developmental dysplasia of the hip Usually diagnosed in infancy by screening tests. May be bilateral, when disease is unilateral there may be leg length inequality. As disease progresses child may limp and then early onset arthritis. More common in extended breech babies. Splints and harnesses or traction. In later years osteotomy and hip realignment procedures may be needed. In arthritis a joint replacement may be needed. However, this is best deferred if possible as it will almost certainly require revision Initially no obvious change on plain films and USS gives best resolution until 3 months of age. On plain films Shentons line should form a smooth arc
Perthes Disease Hip pain (may be referred to the knee) usually occurring between 5 and 12 years of age. Bilateral disease in 20%. Remove pressure from joint to allow normal development. Physiotherapy. Usually self-limiting if diagnosed and treated promptly. X-rays will show flattened femoral head. Eventually in untreated cases the femoral head will fragment.
Slipped upper femoral epiphysis Typically seen in obese male adolescents. Pain is often referred to the knee. Limitation to internal rotation is usually seen. Knee pain is usually present 2 months prior to hip slipping. Bilateral in 20%. Bed rest and non-weight bearing. Aim to avoid avascular necrosis. If severe slippage or risk of it occurring then percutaneous pinning of the hip may be required. X-rays will show the femoral head displaced and falling inferolaterally (like a melting ice cream cone) The Southwick angle gives indication of disease severity

1887
Q

A patient with Crohns disease has a purple coloured skin lesion near an ileostomy site. What is the most likely diagnosis?

Pyoderma gangrenosum

Pemphigus vulgaris

Metastatic Crohns disease

Dermatitis herpetiformis

Bullous pemphigoid

A

Pyoderma gangrenosum may occur in Crohns disease. Cutaneous involvement of granulomas a.k.a metastatic crohns is recognized but very rare and much less likely and therefore not the correct answer.

Skin Diseases

Skin lesions may be referred for surgical assessment, but more commonly will come via a dermatologist for definitive surgical management.

Skin malignancies include basal cell carcinoma, squamous cell carcinoma and malignant melanoma.

Basal Cell Carcinoma
Most common form of skin cancer.
Commonly occur on sun exposed sites apart from the ear.
Sub types include nodular, morphoeic, superficial and pigmented.
Typically slow growing with low metastatic potential.
Standard surgical excision, topical chemotherapy and radiotherapy are all successful.
As a minimum a diagnostic punch biopsy should be taken if treatment other than standard surgical excision is planned.

Squamous Cell Carcinoma
Again related to sun exposure.
May arise in pre - existing solar keratoses.
May metastasize if left.
Immunosupression (e.g. following transplant), increases risk.
Wide local excision is the treatment of choice and where a diagnostic excision biopsy has demonstrated SCC, repeat surgery to gain adequate margins may be required.

Malignant Melanoma
The main diagnostic features (major criteria):
Change in size
Change in shape
Change in colour
Secondary features (minor criteria)
Diameter >6mm
Inflammation
Oozing or bleeding
Altered sensation

Treatment
Suspicious lesions should undergo excision biopsy. The lesion should be removed completely as incision biopsy can make subsequent histopathological assessment difficult.
Once the diagnosis is confirmed the pathology report should be reviewed to determine whether further re-excision of margins is required (see below):

Margins of excision-Related to Breslow thickness
Lesions 0-1mm thick 1cm
Lesions 1-2mm thick 1- 2cm (Depending upon site and pathological features)
Lesions 2-4mm thick 2-3 cm (Depending upon site and pathological features)
Lesions >4 mm thick 3cm
Marsden J et al. Revised UK guidelines for management of Melanoma. Br J Dermatol 2010 163:238-256.

Further treatments such as sentinel lymph node mapping, isolated limb perfusion and block dissection of regional lymph node groups should be selectively applied.

Kaposi Sarcoma
Tumour of vascular and lymphatic endothelium.
Purple cutaneous nodules.
Associated with immuno supression.
Classical form affects elderly males and is slow growing.
Immunosupression form is much more aggressive and tends to affect those with HIV related disease.

Non malignant skin disease

Dermatitis Herpetiformis
Chronic itchy clusters of blisters.
Linked to underlying gluten enteropathy (coeliac disease).

Dermatofibroma
Benign lesion.
Firm elevated nodules.
Usually history of trauma.
Lesion consists of histiocytes, blood vessels and fibrotic changes.

Pyogenic granuloma
Overgrowth of blood vessels.
Red nodules.
Usually follow trauma.
May mimic amelanotic melanoma.

Acanthosis nigricans
Brown to black, poorly defined, velvety hyperpigmentation of the skin.
Usually found in body folds such as the posterior and lateral folds of the neck, the axilla, groin, umbilicus, forehead, and other areas.
The most common cause of acanthosis nigricans is insulin resistance, which leads to increased circulating insulin levels. Insulin spillover into the skin results in its abnormal increase in growth (hyperplasia of the skin).
In the context of a malignant disease, acanthosis nigricans is a paraneoplastic syndrome and is then commonly referred to as acanthosis nigricans maligna. Involvement of mucous membranes is rare and suggests a coexisting malignant condition.

1888
Q

What is the sensory nerve supply to the skin overlying the angle of the jaw?

Maxillary branch of the trigeminal nerve

Mandibular branch of the trigeminal nerve

C3-C4

Greater auricular nerve (C2-C3)

Buccal branch of the facial nerve

A

The trigeminal nerve is the major sensory nerve to the face except over the angle of the jaw. The angle of the jaw is innervated by the greater auricular nerve.

Trigeminal nerve

The trigeminal nerve is the main sensory nerve of the head. In addition to its major sensory role, it also innervates the muscles of mastication.

Distribution of the trigeminal nerve
Sensory
Scalp
Face
Oral cavity (and teeth)
Nose and sinuses
Dura mater
Motor
Muscles of mastication
Mylohyoid
Anterior belly of digastric
Tensor tympani
Tensor palati
Autonomic connections (ganglia)
Ciliary
Sphenopalatine
Otic
Submandibular

Path
Originates at the pons
Sensory root forms the large, crescentic trigeminal ganglion within Meckel’s cave, and contains the cell bodies of incoming sensory nerve fibres. Here the 3 branches exit.
The motor root cell bodies are in the pons and the motor fibres are distributed via the mandibular nerve. The motor root is not part of the trigeminal ganglion.

Branches of the trigeminal nerve
Ophthalmic nerve Sensory only
Maxillary nerve Sensory only
Mandibular nerve Sensory and motor

Sensory
Ophthalmic Exits skull via the superior orbital fissure
Sensation of: scalp and forehead, the upper eyelid, the conjunctiva and cornea of the eye, the nose (including the tip of the nose, except alae nasi), the nasal mucosa, the frontal sinuses, and parts of the meninges (the dura and blood vessels).
Maxillary nerve Exit skull via the foramen rotundum
Sensation: lower eyelid and cheek, the nares and upper lip, the upper teeth and gums, the nasal mucosa, the palate and roof of the pharynx, the maxillary, ethmoid and sphenoid sinuses, and parts of the meninges.
Mandibular nerve Exit skull via the foramen ovale
Sensation: lower lip, the lower teeth and gums, the chin and jaw (except the angle of the jaw), parts of the external ear, and parts of the meninges.

Motor
Distributed via the mandibular nerve.
The following muscles of mastication are innervated:
Masseter
Temporalis
Medial pterygoid
Lateral pterygoid

Other muscles innervated include:
Tensor veli palatini
Mylohyoid
Anterior belly of digastric
Tensor tympani

1889
Q

A 23 year old lady is undergoing a trendelenberg procedure for varicose veins. During the dissection of the saphenofemoral junction, which of the structures listed below is most liable to injury?

Superficial circumflex iliac artery

Superficial circumflex iliac vein

Femoral artery

Femoral nerve

Deep external pudendal artery

A

The deep external pudendal artery runs under the long saphenous vein close to its origin and may be injured. It is at greatest risk of injury during the flush ligation of the saphenofemoral junction. Provided an injury is identified and vessel ligated, injury is seldom associated with any serious adverse sequelae.

Saphenous vein

Long saphenous vein
This vein may be harvested for bypass surgery, or removed as treatment for varicose veins with saphenofemoral junction incompetence.

Originates at the 1st digit where the dorsal vein merges with the dorsal venous arch of the foot
Passes anterior to the medial malleolus and runs up the medial side of the leg
At the knee, it runs over the posterior border of the medial epicondyle of the femur bone
Then passes laterally to lie on the anterior surface of the thigh before entering an opening in the fascia lata called the saphenous opening
It joins with the femoral vein in the region of the femoral triangle at the saphenofemoral junction

Tributaries
Medial marginal
Superficial epigastric
Superficial iliac circumflex
Superficial external pudendal veins

Short saphenous vein
Originates at the 5th digit where the dorsal vein merges with the dorsal venous arch of the foot, which attaches to the great saphenous vein.
It passes around the lateral aspect of the foot (inferior and posterior to the lateral malleolus) and runs along the posterior aspect of the leg (with the sural nerve)
Passes between the heads of the gastrocnemius muscle, and drains into the popliteal vein, approximately at or above the level of the knee joint.

1890
Q

A 2 day old baby is noted to have voiding difficulties and on closer inspection is noted to have hypospadias. Which of the following abnormalities is most commonly associated with the condition?

Cryptorchidism

Diaphragmatic hernia

Ventricular - septal defect

Bronchogenic cyst

Atrial septal defect

A

Hypospadias most commonly occurs as an isolated disorder. Associated urological abnormalities may be seen in up to 40% of infants, of these cryptorchidism is the most frequent (10%).

Hypospadias

The urethral meatus opens on the ventral surface of the penis. There is also a ventral deficiency of the foreskin. The urethral meatus may open more proximally in the more severe variants. However, 75% of the openings are distally located. The incidence is 1 in 300 male births.

Features include:

Absent frenular artery
Ventrally opened glans
Skin tethering to hypoplastic urethra
Splayed columns of spongiosum tissue distal to the meatus
Deficiency of the foreskin ventrally

Management:

No routine cultural circumcisions
Urethroplasty
Penile reconstruction

The foreskin is often utilised in the reconstructive process. In boys with very distal disease no treatment may be needed.

1891
Q

A pathologist is examining a histological section and identifies Hassall’s corpuscles. With what are they most commonly associated?

Follicular carcinoma of the thyroid

Medulla of the thymus

Medulla of the spleen

Medulla of the kidney

Fundus of the stomach

A

Hassall’s corpuscles are the concentric ring of epithelial cells seen in the medulla of the thymus.

Thymus

The thymus develops from the third and fourth pharyngeal pouches. It descends to lie in the anterior superior mediastinum. It is encapsulated and is subdivided into lobules, these consist of a cortex and a medulla. The cortex is composed of tightly packed lymphocytes, the medulla consists largely of epithelial cells. The medullary epithelial cells are concentrically arranged and may surround a keratinised centre, known as Hassall’s corpuscles.
The inferior parathyroid glands also develop from the third pharyngeal pouch and may also be located with the thymus gland.
Its arterial supply is from the internal mammary artery or pericardiophrenic arteries. Venous drainage is to the left brachiocephalic vein.

1892
Q

A 17 year old male presents to the clinic. He complains of difficulty using his left hand. It has been a persistent problem since he sustained a distal humerus fracture as a child. On examination, there is diminished sensation overlying the hypothenar eminence and medial one and half fingers. What is the most likely nerve lesion?

Anterior interosseous nerve

Posterior interosseous nerve

Ulnar nerve

Median nerve

Radial nerve

A

This sensory deficit pattern is most consistent with ulnar nerve injury.

Ulnar nerve

Origin
C8, T1

Supplies (no muscles in the upper arm)
Flexor carpi ulnaris
Flexor digitorum profundus
Flexor digiti minimi
Abductor digiti minimi
Opponens digiti minimi
Adductor pollicis
Interossei muscle
Third and fourth lumbricals
Palmaris brevis

Path
Posteromedial aspect of upper arm to flexor compartment of forearm, then along the ulnar. Passes beneath the flexor carpi ulnaris muscle, then superficially over the flexor retinaculum into the palm of the hand.

Branches
Branch Supplies
Muscular branch Flexor carpi ulnaris
Medial half of the flexor digitorum profundus
Palmar cutaneous branch (Arises near the middle of the forearm) Skin on the medial part of the palm
Dorsal cutaneous branch Dorsal surface of the medial part of the hand
Superficial branch Cutaneous fibres to the anterior surfaces of the medial one and one-half digits
Deep branch Hypothenar muscles
All the interosseous muscles
Third and fourth lumbricals
Adductor pollicis
Medial head of the flexor pollicis brevis

Effects of injury
Damage at the wrist
Wasting and paralysis of intrinsic hand muscles (claw hand)
Wasting and paralysis of hypothenar muscles
Loss of sensation medial 1 and half fingers
Damage at the elbow
Radial deviation of the wrist
Clawing less in 4th and 5th digits

1893
Q

A 52 year old woman presents with a neck swelling. Her GP reports that her TSH value is low at 0.01. A scintigraphy demonstrates a hot nodule. What is the most likely diagnosis?

Graves disease

Toxic adenoma

Papillary thyroid cancer

Hashimotos thyroiditis

De Quervains thyroiditis

A

This lady has thyrotoxicosis (low TSH) and a hot solitary nodule indicating a toxic adenoma. Thyroid cancer rarely causes thyrotoxicosis or hot nodules.

Hyperthyroidism

Causes of hyperthyroidism include:
Diffuse toxic goitre (Graves Disease)
Toxic nodular goitre
Toxic nodule
Rare causes

Graves disease
Graves disease is characterised by a diffuse vascular goitre that appears at the same time as the clinical manifestations of hyperthyroidism. It is commonest in younger females and may be associated with eye signs. Thyrotoxic symptoms will predominate. Up to 50% of patients will have a familial history of autoimmune disorders. The glandular hypertrophy and hyperplasia occur as a result of the thyroid stimulating effects of the TSH receptor antibodies.

Toxic nodular goitre
In this disorder the goitre is present for a long period of time prior to the development of clinical symptoms. In most goitres the nodules are inactive and in some cases it is the internodular tissue that is responsible for the hyperthyroidism.

Toxic nodule
Overactive, autonomously functioning nodule. It may occur as part of generalised nodularity or be a true toxic adenoma. The TSH levels are usually low as the autonomously functioning thyroid tissue will exert a negative feedback effect.

Signs and symptoms
Symptoms Signs
Lethargy Tachycardia
Emotionally labile Agitation
Heat intolerance Hot, moist palms
Weight loss Exopthalmos
Excessive appetite Thyroid goitre and bruit
Palpitations Lid lag/retraction

Diagnosis
The most sensitive test for diagnosing hyperthyroidism is plasma T3 (which is raised). Note in hypothyroidism the plasma T4 and TSH are the most sensitive tests. A TSH level of <0.5U/L suggests hyperthyroidism. TSH receptor antibodies may be tested for in the diagnosis of Graves.

Treatment
First line treatment for Graves disease is usually medical and the block and replace regime is the favored option. Carbimazole is administered at higher doses and thyroxine is administered orally. Patients are maintained on this regime for between 6 and 12 months. Attempts are then made to wean off medication. Where relapse then occurs the options are between ongoing medical therapy, radioiodine or surgery.

1894
Q

A surgical junior doctor decides to take an arterial blood sample from the femoral artery. What anatomical landmark best corresponds to the commencement of this vessel?

Mid inguinal point

Mid point of the inguinal ligament

Apex of the femoral triangle

Inferomedially to the pubic tubercle

Inferomedially to the symphysis pubis

A

Mid inguinal point= surface marking for femoral artery
Mid point of the inguinal ligament = surface marking for deep inguinal ring.

Iliac arteries

The common iliac arteries are terminal branches of the abdominal aorta. They begin opposite the middle body of the fourth lumbar vertebra. The superior hypogastric plexus lies anterior to the proximal end and more distally, it is crossed by the ureter. The last two lumbar vertebral bodies, and intervening disc, the sympathetic trunk and the medial part of psoas major lie behind each artery. They are separated from the right artery by the beginning of the inferior vena cava. The only branches of the common iliac arteries are the internal and external iliac arteries.

Internal iliac artery
Anteromedially, each internal iliac artery is covered by peritoneum, beneath which the ureter descends on the anterior surface of each artery. On the left, the vessel is overlapped by the sigmoid colon and, on the right, the terminal ileum overlies it. Posteriorly, the internal iliac vein and commencement of the common iliac vein are closely related to the artery. The posterior division of the internal iliac artery has parietal branches and the anterior division has both parietal and visceral branches.

Branches of the posterior division of the internal iliac
Lateral sacral, superior gluteal, iliolumbar artery.

Branches of the anterior division of the internal iliac artery
Obturator artery, internal pudendal artery, inferior gluteal, inferior vesical artery, artery the ductus deferens, middle rectal, vaginal artery, uterine artery.

External iliac artery
The external iliac artery extends from a point opposite the sacroiliac joint at the level of the lumbosacral disc to the midinguinal point behind the inguinal ligament, midway between the anterior superior iliac spine and the pubic symphysis.

1895
Q

What is measured to obtain renal plasma flow?

Creatinine

Para-amino hippuric acid (PAH)

Inulin

Glucose

Protein

A

Renal plasma flow = (amount of PAH in urine per unit time) / (difference in PAH concentration in the renal artery or vein)

Normal value = 660ml/min

Renal Physiology

Overview
Each nephron is supplied with blood from an afferent arteriole that opens onto the glomerular capillary bed.
Blood then flows to an efferent arteriole, supplying the peritubular capillaries and medullary vasa recta.
The kidney receives up to 25% of resting cardiac output.

Control of blood flow
The kidney is able to autoregulate its blood flow between systolic pressures of 80- 180mmHg so there is little variation in renal blood flow.
This is achieved by myogenic control of arteriolar tone, both sympathetic input and hormonal signals (e.g. renin) are responsible.

Glomerular structure and function
Blood inside the glomerulus has considerable hydrostatic pressure.
The basement membrane has pores that will allow free diffusion of smaller solutes, larger negatively charged molecules such as albumin are unable to cross.
The glomerular filtration rate (GFR) is equal to the concentration of a solute in the urine, times the volume of urine produced per minute, divided by the plasma concentration (assuming that the solute is freely diffused e.g. inulin).
In clinical practice creatinine is used because it is subjected to very little proximal tubular secretion.
Although subject to variability, the typical GFR is 125ml per minute.
Glomerular filtration rate = Total volume of plasma per unit time leaving the capillaries and entering the bowman’s capsule
Renal clearance = volume plasma from which a substance is removed per minute by the kidneys

Substances used to measure GFR have the following features:
1. Inert
2. Free filtration from the plasma at the glomerulus (not protein bound)
3. Not absorbed or secreted at the tubules
4. Plasma concentration constant during urine collection

Examples: inulin, creatinine

plasma concentration (mmol/l)

The clearance of a substance is dependent not only on its diffusivity across the basement membrane but also subsequent tubular secretion and / or reabsorption.
So glucose which is freely filtered across the basement membrane is usually reabsorbed from tubules giving a clearance of zero.

Tubular function
Reabsorption and secretion of substances occurs in the tubules.
In the proximal tubule substrates such as glucose, amino acids and phosphate are co-transported with sodium across the semi permeable membrane.
Glucose is reabsorbed by both active and passive processes. The former uses sodium–glucose co-transporters (SGLTs) in the proximal convoluted tubule. The latter achieved through facilitated and passive process through GLUT transporters.
Up to two thirds of filtered water is reabsorbed in the proximal tubules.
This will lead to increase in urea concentration in the distal tubule allowing for its increased diffusion.
Substances to be secreted into the tubules are taken up from the peritubular blood by tubular cells.
Solutes such as paraaminohippuric acid are cleared with a single passage through the kidneys and this is why it is used to measure renal plasma flow. Ions such as calcium and phosphate will have a tubular reabsorption that is influenced by plasma PTH levels.
Potassium may be both secreted and re-absorbed and is co-exchanged with sodium.

Loop of Henle
Approximately 60 litres of water containing 9000mmol sodium enters the descending limb of the loop of Henle in 24 hours.
Loops from the juxtamedullary nephrons run deep into the medulla.
The osmolarity of fluid changes and is greatest at the tip of the papilla.
The thick ascending limb is impermeable to water, but highly permeable to sodium and chloride ions.
This loss means that at the beginning of the thick ascending limb the fluid is hypo osmotic compared with adjacent interstitial fluid.
In the thick ascending limb the reabsorption of sodium and chloride ions occurs by both facilitated and passive diffusion pathways.
The loops of Henle are co-located with vasa recta, these will have similar solute compositions to the surrounding extracellular fluid so preventing the diffusion and subsequent removal of this hypertonic fluid.
The energy dependent reabsorption of sodium and chloride in the thick ascending limb helps to maintain this osmotic gradient.

1896
Q

Which of the following is not a characteristic of the proximal convoluted tubule in the kidney?

Up to 95% of filtered amino acids will be reabsorbed at this site

It is a risk of damage in a patient with compartment syndrome due to a tibial fracture

It is responsible for absorbing more than 50% of filtered water

Its secretory function is most effective at low systolic blood pressures (typically less than 100mmHg)

Glucose is reabsorbed by combined active and passive (facilitated diffusion processes)

A

D-Its secretory function is most effective at low systolic blood pressures (typically less than 100mmHg)

The proximal convoluted tubule may undergo necrosis in situations such as compartment syndrome. It is responsible for reabsorbing up to two thirds of filtered water. Low systolic blood pressures (below the renal autoregulatory range) are a risk factor for acute tubular necrosis. Within the autoregulatory range the absolute value of systolic BP has little effect. The process of facilitated diffusion of glucose refers to its co-transport with sodium.

Renal Physiology

Overview
Each nephron is supplied with blood from an afferent arteriole that opens onto the glomerular capillary bed.
Blood then flows to an efferent arteriole, supplying the peritubular capillaries and medullary vasa recta.
The kidney receives up to 25% of resting cardiac output.

Control of blood flow
The kidney is able to autoregulate its blood flow between systolic pressures of 80- 180mmHg so there is little variation in renal blood flow.
This is achieved by myogenic control of arteriolar tone, both sympathetic input and hormonal signals (e.g. renin) are responsible.

Glomerular structure and function
Blood inside the glomerulus has considerable hydrostatic pressure.
The basement membrane has pores that will allow free diffusion of smaller solutes, larger negatively charged molecules such as albumin are unable to cross.
The glomerular filtration rate (GFR) is equal to the concentration of a solute in the urine, times the volume of urine produced per minute, divided by the plasma concentration (assuming that the solute is freely diffused e.g. inulin).
In clinical practice creatinine is used because it is subjected to very little proximal tubular secretion.
Although subject to variability, the typical GFR is 125ml per minute.
Glomerular filtration rate = Total volume of plasma per unit time leaving the capillaries and entering the bowman’s capsule
Renal clearance = volume plasma from which a substance is removed per minute by the kidneys

Substances used to measure GFR have the following features:
1. Inert
2. Free filtration from the plasma at the glomerulus (not protein bound)
3. Not absorbed or secreted at the tubules
4. Plasma concentration constant during urine collection

Examples: inulin, creatinine

plasma concentration (mmol/l)

The clearance of a substance is dependent not only on its diffusivity across the basement membrane but also subsequent tubular secretion and / or reabsorption.
So glucose which is freely filtered across the basement membrane is usually reabsorbed from tubules giving a clearance of zero.

Tubular function
Reabsorption and secretion of substances occurs in the tubules.
In the proximal tubule substrates such as glucose, amino acids and phosphate are co-transported with sodium across the semi permeable membrane.
Glucose is reabsorbed by both active and passive processes. The former uses sodium–glucose co-transporters (SGLTs) in the proximal convoluted tubule. The latter achieved through facilitated and passive process through GLUT transporters.
Up to two thirds of filtered water is reabsorbed in the proximal tubules.
This will lead to increase in urea concentration in the distal tubule allowing for its increased diffusion.
Substances to be secreted into the tubules are taken up from the peritubular blood by tubular cells.
Solutes such as paraaminohippuric acid are cleared with a single passage through the kidneys and this is why it is used to measure renal plasma flow. Ions such as calcium and phosphate will have a tubular reabsorption that is influenced by plasma PTH levels.
Potassium may be both secreted and re-absorbed and is co-exchanged with sodium.

Loop of Henle
Approximately 60 litres of water containing 9000mmol sodium enters the descending limb of the loop of Henle in 24 hours.
Loops from the juxtamedullary nephrons run deep into the medulla.
The osmolarity of fluid changes and is greatest at the tip of the papilla.
The thick ascending limb is impermeable to water, but highly permeable to sodium and chloride ions.
This loss means that at the beginning of the thick ascending limb the fluid is hypo osmotic compared with adjacent interstitial fluid.
In the thick ascending limb the reabsorption of sodium and chloride ions occurs by both facilitated and passive diffusion pathways.
The loops of Henle are co-located with vasa recta, these will have similar solute compositions to the surrounding extracellular fluid so preventing the diffusion and subsequent removal of this hypertonic fluid.
The energy dependent reabsorption of sodium and chloride in the thick ascending limb helps to maintain this osmotic gradient.

1897
Q

A splenectomy increases the risk of infection from all the following organisms except?

Pneumococcus

Klebsiella

Haemophilus influenzae

Staphylococcus aureus

Neisseria meningitidis

A

Mnemonic used to remember some encapsulated pathogens is: ‘Even Some Super Killers Have Pretty Nice Big Capsules’
Escherichia coli, Streptococcus pneumoniae, Salmonella, Klebsiella pneumoniae, Haemophilus influenzae, Pseudomonas aeruginosa, Neisseria meningitidis, Bacteroides fragilis, and the yeast Cryptococcus neoformans
Staphylococcus aureus infection following splenectomy is no more common than in non splenectomised individuals. The other organisms are encapsulated, which is why they are more likely to cause overwhelming post splenectomy sepsis.

Post splenectomy sepsis

Hyposplenism may complicate certain medical conditions where splenic atrophy occurs or may be the result of medical intervention such as splenic artery embolization and splenectomy for trauma. Diagnosis of hyposplenism is difficult and whilst there may be peripheral markers of the splenectomised state (e.g. Howell Jolly bodies) these are neither 100% sensitive or specific. The most sensitive test is a radionucleotide labeled red cell scan.
Hyposplenism, by whatever mechanism it occurs dramatically increases the risk of post splenectomy sepsis, particularly with encapsulated organisms. Since these organisms may be opsonised, but this then goes undetected at an immunological level due to loss of the spleen. For this reason individuals are recommended to be vaccinated and have antibiotic prophylaxis.

Key recommendations
All those with hyposplenism or may become so (such as prior to an elective splenectomy) should receive pneumococcal, haemophilus type b and meningococcal type C vaccines. These should be administered 2 weeks prior to splenectomy or two weeks following splenectomy. The vaccine schedule for meningococcal disease essentially consists of a dose of Men C and Hib at 2 weeks and then a dose of the MenACWY vaccine one month later. Those aged under 2 may require a booster at 2 years. A dose of pneumococcal polyvalent polysaccharide vaccine (PPV) is given at two weeks. A conjugated vaccine (PCV) is offered to young children. The PCV is more immunogenic but covers fewer serotypes. Boosting PPV is either guided by serological measurements (where available) or by routine boosting doses at 5 yearly intervals.
Annual influenza vaccination is recommended in all cases
Antibiotic prophylaxis is offered to all. The risk of post splenectomy sepsis is greatest immediately following splenectomy and in those aged less than 16 years or greater than 50 years. Individuals with a poor response to pneumococcal vaccination are another high risk group. High risk individuals should be counselled to take penicillin or macrolide prophylaxis. Those at low risk may choose to discontinue therapy. All patients should be advised about taking antibiotics early in the case of intercurrent infections.
Asplenic individuals traveling to malaria endemic areas are at high risk and should have both pharmacological and mechanical protection.

Dosing
Penicillin V 500mg BD or amoxicillin 250mg BD

References
Davies J et al. Review of guidelines for the prevention and treatment of infection in patients with an absent or dysfunctional spleen: Prepared on behalf of the British Committee for Standards in Haematology by a Working Party of the Haemato-Oncology Task Force. British Journal of Haematology 2011 (155): 308317.

1898
Q

A 21 year old man presents with a unilateral facial nerve palsy after being hit on the head. On examination, he has a right sided facial nerve palsy and a watery discharge from his nose. What is the most likely cause?

Pleomorphic adenoma

Cerebrovascular accident

Occipital bone fracture

Petrous temporal fracture

Sarcoidosis

A

Nasal discharge of clear fluid and recent head injury makes a basal skull fracture the most likely underlying diagnosis.

Facial nerve palsy

Sarcoid Facial nerve palsy is the most frequent neurological manifestation of sarcoid
Affects right and left side with equal frequency, may be bilateral
Typically resolves in up to 80% of cases
Cholesteatoma Destructive and expanding growth of keratinised squamous epithelium
Patients often complain of chronic ear discharge
Infection with Pseudomonas may occur resulting in foul smell to discharge
Aquired lesions usually arise from the Pars flaccida region of the tympanic membrane
Surgical removal and mastoidectomy may be needed
Recurrence rates of 20% may be seen following surgery
Basal skull fracture History of head injury
Presence of features such as Battles sign on examination
Clinical presence of CSF leak strongly supports diagnosis
Assessment is by CT and MRI scan
Prophylactic antibiotics should be given in cases of CSF leak

1899
Q

A 71 year old man presents with a painful lower calf ulcer, mild pitting oedema and an ABPI of 0.3. What is the most likely cause?

Mixed ulcer

Deep venous insufficiency

Chronic obliterative arterial disease

Squamous cell carcinoma

Neuropathic ulcer

A

Painful ulcers associated with a low ABPI are usually arterial in nature. The question does not indicate that features of chronic venous insufficiency are present. Patients may have mild pitting oedema as many vascular patients will also have ischaemic heart disease and elevated right heart pressures. The absence of more compelling signs of venous insufficiency makes a mixed ulcer less likely.

Lower leg ulcers

Venous leg ulcers
Most due to venous hypertension, secondary to chronic venous insufficiency (other causes include calf pump dysfunction or neuromuscular disorders)
Ulcers form due to capillary fibrin cuff or leucocyte sequestration
Features of venous insufficiency include oedema, brown pigmentation, lipodermatosclerosis, eczema
Location above the ankle, painless
Deep venous insufficiency is related to previous DVT and superficial venous insufficiency is associated with varicose veins
Doppler ultrasound looks for presence of reflux and duplex ultrasound looks at the anatomy/ flow of the vein
Management: 4 layer compression banding after exclusion of arterial disease or surgery
If fail to heal after 12 weeks or >10cm2 skin grafting may be needed

Marjolin’s ulcer

Squamous cell carcinoma
Occurring at sites of chronic inflammation e.g; burns, osteomyelitis after 10-20 years
Mainly occur on the lower limb

Arterial ulcers
Occur on the toes and heel
Painful
There may be areas of gangrene
Cold with no palpable pulses
Low ABPI measurements

Neuropathic ulcers
Commonly over plantar surface of metatarsal head and plantar surface of hallux
The plantar neuropathic ulcer is the condition that most commonly leads to amputation in diabetic patients
Due to pressure
Management includes cushioned shoes to reduce callus formation

Pyoderma gangrenosum

Associated with inflammatory bowel disease/RA
Can occur at stoma sites
Erythematous nodules or pustules which ulcerate

1900
Q

During embryological development, which of the following represent the correct origin of the pancreas?

Ventral and dorsal endodermal outgrowths of the duodenum

Ventral and dorsal outgrowths of mesenchymal tissue from the posterior abdominal wall

Ventral and dorsal outgrowths of the vitellointestinal duct

Ventral and dorsal biliary tract diverticulae

Buds from the inferior aspect of the caudate lobe

A

The pancreas develops from a ventral and dorsal endodermal outgrowth of the duodenum. The ventral arises close to, or in common with the hepatic diverticulum, and the larger, dorsal outgrowth arises slightly cranial to the ventral extending into the mesoduodenum and mesogastrium. When the buds eventually fuse the duct of the ventral rudiment becomes the main pancreatic duct.

Pancreas

The pancreas is a retroperitoneal organ and lies posterior to the stomach. It may be accessed surgically by dividing the peritoneal reflection that connects the greater omentum to the transverse colon. The pancreatic head sits in the curvature of the duodenum. Its tail lies close to the hilum of the spleen, a site of potential injury during splenectomy.

Relations
Posterior to the pancreas
Pancreatic head Inferior vena cava
Common bile duct
Right and left renal veins
Superior mesenteric vein and artery
Pancreatic neck Superior mesenteric vein, portal vein
Pancreatic body- Left renal vein
Crus of diaphragm
Psoas muscle
Adrenal gland
Kidney
Aorta
Pancreatic tail Left kidney

Anterior to the pancreas
Pancreatic head 1st part of the duodenum
Pylorus
SMA and SMV(uncinate process)
Pancreatic body Stomach
Duodenojejunal flexure
Pancreatic tail Splenic hilum

Superior to the pancreas
Coeliac trunk and its branches common hepatic artery and splenic artery

Grooves of the head of the pancreas
2nd and 3rd part of the duodenum

Arterial supply
Head: pancreaticoduodenal artery
Rest: splenic artery

Venous drainage
Head: superior mesenteric vein
Body and tail: splenic vein

Ampulla of Vater
Merge of pancreatic duct and common bile duct
Is an important landmark, halfway along the second part of the duodenum, that marks the anatomical transition from foregut to midgut (also the site of transition between regions supplied by coeliac trunk and SMA).

1901
Q

Which of the following structures is not directly related to the right adrenal gland?

Diaphragm posteriorly

Kidney inferiorly

Right renal vein

Inferior vena cava

Hepato-renal pouch

A

The right renal vein is very short and lies more inferiorly.

Adrenal gland anatomy

Anatomy

Location Superomedially to the upper pole of each kidney
Relationships of the right adrenal Diaphragm-Posteriorly, Kidney-Inferiorly, Vena Cava-Medially, Hepato-renal pouch and bare area of the liver-Anteriorly
Relationships of the left adrenal Crus of the diaphragm-Postero- medially, Pancreas and splenic vessels-Inferiorly, Lesser sac and stomach-Anteriorly
Arterial supply Superior adrenal arteries- from inferior phrenic artery, Middle adrenal arteries - from aorta, Inferior adrenal arteries -from renal arteries
Venous drainage of the right adrenal Via one central vein directly into the IVC
Venous drainage of the left adrenal Via one central vein into the left renal vein

1902
Q

Which muscle is responsible for causing flexion of the distal interphalangeal joint of the ring finger?

Flexor digitorum superficialis

Lumbricals

Palmar interossei

Flexor digitorum profundus

Flexor digiti minimi brevis

A

D-Flexor digitorum profundus

Flexor digitorum superficialis and flexor digitorum profundus are responsible for causing flexion. The superficialis tendons insert on the bases of the middle phalanges; the profundus tendons insert on the bases of the distal phalanges. Both tendons flex the wrist, MCP and PIP joints; however, only the profundus tendons flex the DIP joints.

Hand

Anatomy of the hand
Bones
8 Carpal bones
5 Metacarpals
14 phalanges
Intrinsic Muscles 8 Interossei - Supplied by ulnar nerve
4 palmar-adduct fingers
4 dorsal- abduct fingers
Intrinsic muscles Lumbricals
Flex MCPJ and extend the IPJ.
Origin deep flexor tendon and insertion dorsal extensor hood mechanism.
Innervation: 1st and 2nd- median nerve, 3rd and 4th- deep branch of the ulnar nerve.
Thenar eminence
Abductor pollicis brevis
Opponens pollicis
Flexor pollicis brevis
Hypothenar eminence
Opponens digiti minimi
Flexor digiti minimi brevis
Abductor digiti minimi

Fascia and compartments of the palm
The fascia of the palm is continuous with the antebrachial fascia and the fascia of the dorsum of the hand. The palmar fascia is thin over the thenar and hypothenar eminences. In contrast, the central palmar fascia is relatively thick. The palmar aponeurosis covers the soft tissues and overlies the flexor tendons. The apex of the palmar aponeurosis is continuous with the flexor retinaculum and the palmaris longus tendon. Distally, it forms four longitudinal digital bands that attach to the bases of the proximal phalanges, blending with the fibrous digital sheaths.
A medial fibrous septum extends deeply from the medial border of the palmar aponeurosis to the 5th metacarpal. Lying medial to this are the hypothenar muscles. In a similar fashion, a lateral fibrous septum extends deeply from the lateral border of the palmar aponeurosis to the 3rd metacarpal. The thenar compartment lies lateral to this area.
Lying between the thenar and hypothenar compartments is the central compartment. It contains the flexor tendons and their sheaths, the lumbricals, the superficial palmar arterial arch and the digital vessels and nerves.
The deepest muscular plane is the adductor compartment, which contains adductor pollicis.

Short muscles of the hand
These comprise the lumbricals and interossei. The four slender lumbrical muscles flex the fingers at the metacarpophalangeal joints and extend the interphalangeal joint. The four dorsal interossei are located between the metacarpals and the four palmar interossei lie on the palmar surface of the metacarpals in the interosseous compartment of the hand.

Long flexor tendons and sheaths in the hand
The tendons of FDS and FDP enter the common flexor sheath deep to the flexor retinaculum. The tendons enter the central compartment of the hand and fan out to their respective digital synovial sheaths. Near the base of the proximal phalanx, the tendon of FDS splits to permit the passage of FDP. The FDP tendons are attached to the margins of the anterior aspect of the base of the distal phalanx.
The fibrous digital sheaths contain the flexor tendons and their synovial sheaths. These extend from the heads of the metacarpals to the base of the distal phalanges.

Palmar Interossei
Note that there are 4 palmar interossei. The first is a small slip of muscle which arises from the ulnar side of the base of the first metacarpal and passes between the head of the first dorsal interosseous and the oblique head of adductor pollicis to insert into the ulnar base of the of the proximal phalanx of the thumb. The second arises from the ulnar side of the body of the second metacarpal and is inserted into the ulnar side of the extensor hood of the index. The third and fourth palmar interossei arise from the radial sides of the bodies of the 4th and 5th metacarpals respectively and insert into the radial sides of the extensor hoods of the ring and little fingers.

1903
Q

Which of the following muscles does not adduct the shoulder?

Teres major

Pectoralis major

Coracobrachialis

Supraspinatus

Latissimus dorsi

A

Supraspinatus is an abductor of the shoulder.
Shoulder joint

Shallow synovial ball and socket type of joint.
It is an inherently unstable joint, but is capable to a wide range of movement.
Stability is provided by muscles of the rotator cuff that pass from the scapula to insert in the greater tuberosity (all except sub scapularis-lesser tuberosity).

Glenoid labrum
Fibrocartilaginous rim attached to the free edge of the glenoid cavity
Tendon of the long head of biceps arises from within the joint from the supraglenoid tubercle, and is fused at this point to the labrum.
The long head of triceps attaches to the infraglenoid tubercle

Fibrous capsule
Attaches to the scapula external to the glenoid labrum and to the labrum itself (postero-superiorly)
Attaches to the humerus at the level of the anatomical neck superiorly and the surgical neck inferiorly
Anteriorly the capsule is in contact with the tendon of subscapularis, superiorly with the supraspinatus tendon, and posteriorly with the tendons of infraspinatus and teres minor. All these blend with the capsule towards their insertion.
Two defects in the fibrous capsule; superiorly for the tendon of biceps. Anteriorly there is a defect beneath the subscapularis tendon.
The inferior extension of the capsule is closely related to the axillary nerve at the surgical neck and this nerve is at risk in anteroinferior dislocations. It also means that proximally sited osteomyelitis may progress to septic arthritis.

Movements and muscles
Flexion Anterior part of deltoid
Pectoralis major
Biceps
Coracobrachialis
Extension Posterior deltoid
Teres major
Latissimus dorsi
Adduction Pectoralis major
Latissimus dorsi
Teres major
Coracobrachialis
Abduction Mid deltoid
Supraspinatus
Medial rotation Subscapularis
Anterior deltoid
Teres major
Latissimus dorsi
Lateral rotation Posterior deltoid
Infraspinatus
Teres minor

Important anatomical relations
Anteriorly Brachial plexus
Axillary artery and vein
Posterior Suprascapular nerve
Suprascapular vessels
Inferior Axillary nerve
Circumflex humeral vessels

1904
Q

The histopathology report indicates that cells are identified with features of coarse chromatin and abnormal mitoses. The cells are confined to the superficial epithelial layer only. Which of the following accounts for this process?

Metaplasia

Apoptosis

Autoimmune oesophagitis

Dysplasia

Infection with
Helicobacter pylori

A

Dysplasia = pre cancerous
Dysplasia tends to develop as a result of prolonged stimulation by precipitants. Removal of these precipitants may possibly reverse these changes. Replacement of differentiated cells with another cell type describes metaplasia rather than dysplasia. The absence of invasion distinguishes this from malignancy.

Dysplasia

Premalignant condition
Disordered growth and differentiation of cells
Alteration in size, shape, and organization of cells
Features increased abnormal cell growth (increased number of mitoses/abnormal mitoses and cellular differentiation)
Underlying connective tissue is not invaded
Causes include smoking, Helicobacter pylori, Human papilloma virus
Main differences to metaplasia is that dysplasia is considered to be part of carcinogenesis (pre cancerous) and is associated with a delay in maturation of cells rather than differentiated cells replacing one another
The absence of invasion differentiates dysplasia from invasive malignancy
Severe dysplasia with foci of invasion are well recognised

1905
Q

A 40 year old lady presents with a gastric carcinoma of the greater curvature of the stomach. Her staging investigations are negative for metastatic disease. What is the most appropriate treatment option?

Radical radiotherapy

Sub total gastrectomy, D2 lymphadenectomy and Roux en Y reconstruction

Sub total gastrectomy, D2 lymphadenectomy and anterior gastrojejunostomy

Oesophagogastrectomy

Sleeve gastrectomy

A

This is amenable to potentially curative resection. The proximal stomach can be conserved.
Reconstruction with an anterior gastrojejunostomy will not provide optimal function. A Sleeve gastrectomy is not performed for malignant disease.
Gastric cancer

Overview
There are 700,000 new cases of gastric cancer worldwide each year. It is most common in Japan and less common in western countries. It is more common in men and incidence rises with increasing age. The exact cause of many sporadic cancer is not known, however, familial cases do occur in HNPCC families. In addition, smoking and smoked or preserved foods increase the risk. Japanese migrants retain their increased risk (decreased in subsequent generations). The distribution of the disease in western countries is changing towards a more proximal location (perhaps due to rising obesity).

Pathology
There is some evidence of support a stepwise progression of the disease through intestinal metaplasia progressing to atrophic gastritis and subsequent dysplasia, through to cancer. The favoured staging system is TNM. The risk of lymph node involvement is related to size and depth of invasion; early cancers confined to submucosa have a 20% incidence of lymph node metastasis. Tumours of the gastro-oesophageal junction are classified as below:

Type 1 True oesophageal cancers and may be associated with Barrett’s oesophagus.
Type 2 Carcinoma of the cardia, arising from cardiac type epithelium
or short segments with intestinal metaplasia at the oesophagogastric junction.
Type 3 Sub cardial cancers that spread across the junction. Involve similar nodal stations to gastric cancer.

Groups for close endoscopic monitoring
Intestinal metaplasia of columnar type
Atrophic gastritis
Low to medium grade dysplasia
Patients who have previously undergone resections for benign peptic ulcer disease (except highly selective vagotomy).

Referral to endoscopy

Patients of any age with dyspepsia and any of the following Patients without dyspepsia Worsening dyspepsia
Chronic gastrointestinal bleeding Dysphagia Barretts oesophagus
Dysphagia Unexplained abdominal pain or weight loss Intestinal metaplasia
Weight loss Vomiting Dysplasia
Iron deficiency anaemia Upper abdominal mass Atrophic gastritis
Upper abdominal mass Jaundice Patient aged over 55 years with unexplained or persistent dyspepsia

Staging
CT scanning of the chest abdomen and pelvis is the routine first line staging investigation in most centres.
Laparoscopy to identify occult peritoneal disease
PET CT (particularly for junctional tumours)

Treatment
Proximally sited disease greater than 5-10cm from the OG junction may be treated by sub total gastrectomy
Total gastrectomy if tumour is <5cm from OG junction
For type 2 junctional tumours (extending into oesophagus) oesophagogastrectomy is usual
Endoscopic sub mucosal resection may play a role in early gastric cancer confined to the mucosa and perhaps the sub mucosa (this is debated)
Lymphadenectomy should be performed. A D2 lymphadenectomy is widely advocated by the Japanese, the survival advantages of extended lymphadenectomy have been debated. However, the overall recommendation is that a D2 nodal dissection be undertaken.
Most patients will receive chemotherapy either pre or post operatively.

Prognosis

UK Data

Disease extent Percentage 5 year survival
All RO resections 54%
Early gastric cancer 91%
Stage 1 87%
Stage 2 65%
Stage 3 18%

Operative procedure

Total Gastrectomy , lymphadenectomy and Roux en Y anastomosis

General anaesthesia
Prophylactic intravenous antibiotics
Incision: Rooftop.
Perform a thorough laparotomy to identify any occult disease.
Mobilise the left lobe of the liver off the diaphragm and place a large pack over it. Insert a large self retaining retractor e.g. omnitract or Balfour (take time with this, the set up should be perfect). Pack the small bowel away.
Begin by mobilising the omentum off the transverse colon.
Proceed to detach the short gastric vessels.
Mobilise the pylorus and divide it at least 2cm distally using a linear cutter stapling device.
Continue the dissection into the lesser sac taking the lesser omentum and left gastric artery flush at its origin.
The lymph nodes should be removed en bloc with the specimen where possible.
Place 2 stay sutures either side of the distal oesophagus. Ask the anaesthetist to pull back on the nasogastric tube. Divide the distal oesophagus and remove the stomach.
The oesphago jejunal anastomosis should be constructed. Identify the DJ flexure and bring a loop of jejunum up to the oesophagus (to check it will reach). Divide the jejunum at this point. Bring the divided jejunum either retrocolic or antecolic to the oesophagus. Anastamose the oesophagus to the jejunum, using either interrupted 3/0 vicryl or a stapling device. Then create the remainder of the Roux en Y reconstruction distally.
Place a jejunostomy feeding tube.
Wash out the abdomen and insert drains (usually the anastomosis and duodenal stump). Help the anaesthetist insert the nasogastric tube (carefully!)
Close the abdomen and skin.
Enteral feeding may commence on the first post-operative day. However, most surgeons will leave patients on free NG drainage for several days and keep them nil by mouth.

1906
Q

Which structure is least likely to be found at the level of the sternal angle?

Left brachiocephalic vein

Intervertebral discs T4-T5

Start of aortic arch

2nd pair of costal cartilages

Bifurcation of the trachea into left and right bronchi

A

The left brachiocephalic vein lies posterior to the manubrium, at the level of its upper border. The sternal angle refers to the transition between manubrium and sternum and therefore will not include the left brachiocephalic vein.
Sternal angle

Anatomical structures at the level of the manubrium and upper sternum
Upper part of the manubrium
Left brachiocephalic vein
Brachiocephalic artery
Left common carotid
Left subclavian artery
Lower part of the manubrium/ manubrio-sternal angle
Costal cartilages of the 2nd ribs
Transition point between superior and inferior mediastinum
Arch of the aorta
Tracheal bifurcation
Union of the azygos vein and superior vena cava
The thoracic duct crosses to the midline

1907
Q

A 43 year old lady presents with severe chest pain. Investigations demonstrate a dissecting aneurysm of the ascending aorta which originates at the aortic valve. What is the optimal treatment?

Endovascular stent

Medical therapy with beta blockers

Medical therapy with ACE inhibitors

Sutured aortic repair

Aortic root replacement

A

Proximal aortic dissections are generally managed with surgical aortic root replacement. The proximal origin of the dissection together with chest pain (which may occur in all types of aortic dissection) raises concerns about the possibility of coronary ostial involvement (which precludes stenting). There is no role for attempted suture repair in this situation.

Aortic dissection

More common than rupture of the abdominal aorta
33% of patients die within the first 24 hours, and 50% die within 48 hours if no treatment received
Associated with hypertension
Features of aortic dissection: tear in the intimal layer, followed by formation and propagation of a subintimal hematoma. Cystic medial necrosis (Marfan’s)
Most common site of dissection: 90% occurring within 10 centimetres of the aortic valve

Stanford Classification
Type Location Treatment
A Ascending aorta/ aortic root Surgery- aortic root replacement
B Descending aorta Medical therapy with antihypertensives

DeBakey classification
Type Site affected
I Ascending aorta, aortic arch, descending aorta
II Ascending aorta only
III Descending aorta distal to left subclavian artery

Clinical features
Tearing, sudden onset chest pain (painless 10%)
Hypertension or Hypotension
A blood pressure difference (in each arm) greater than 20 mm Hg
Neurologic deficits (20%)

Investigations
CXR: widened mediastinum, abnormal aortic knob, ring sign, deviation of the trachea/oesophagus
CT angiography of the thoracic aorta
MRI angiography
Conventional angiography (now rarely used diagnostically)

Management
Beta-blockers: aim HR 60-80 bpm and systolic BP 100-120 mm Hg
For type A dissections the standard of care is aortic root replacement

1908
Q

A 34 year old man presents to the surgical clinic 8 months following a laparotomy for a ruptured spleen. He complains of a nodule in the centre of his laparotomy wound. This is explored surgically and a stitch granuloma is found and excised. From which of the following cell types do granulomata arise?

Polymorpho nucleocytes

Plasma cells

Reed- Sternberg cells

Platelets

Macrophages

A

Granulomas are organised collections of macrophages
Macrophages give origin to granulomas.

Chronic inflammation

Overview
Chronic inflammation may occur secondary to acute inflammation.In most cases chronic inflammation occurs as a primary process. These may be broadly viewed as being one of three main processes:
Persisting infection with certain organisms such as Mycobacterium tuberculosis which results in delayed type hypersensitivity reactions and inflammation.
Prolonged exposure to non-biodegradable substances such as silica or suture materials which may induce an inflammatory response.
Autoimmune conditions involving antibodies formed against host antigens.

Acute vs. Chronic inflammation
Acute inflammation Chronic inflammation
Changes to existing vascular structure and increased permeability of endothelial cells Angiogenesis predominates
Infiltration of neutrophils Macrophages, plasma cells and lymphocytes predominate
Process may resolve with:
Suppuration
Complete resolution
Abscess formation
Progression to chronic inflammation
Healing by fibrosis
Healing by fibrosis is the main result

Granulomatous inflammation
A granuloma consists of a microscopic aggregation of macrophages (with epithelial type arrangement =epithelioid). Large giant cells may be found at the periphery of granulomas.

Mediators
Growth factors released by activated macrophages include agents such as interferon and fibroblast growth factor (plus many more). Some of these such as interferons may have systemic features resulting in systemic symptoms and signs, which may be present in individuals with long standing chronic inflammation.

The finding of granulomas is pathognomonic of chronic inflammation, as illustrated in this biopsy from a patient with colonic Crohns disease

1909
Q

A 55 year old man from Hong Kong presents with left sided otalgia and recurrent episodes of epistaxis. On examination, his pharynx appears normal. Examination of his neck reveals left sided cervical lymphadenopathy. What is the most likely underlying diagnosis?

Antrochoanal polyp

Nasopharyngeal carcinoma

Adenocarcinoma of the tonsil

Angiofibroma

Globus syndrome

A

Given this mans ethnic origin and presenting features a nasopharyngeal carcinoma is the most likely underlying diagnosis.

Nasopharyngeal carcinoma

Squamous cell carcinoma of the nasopharynx
Rare in most parts of the world, apart from individuals from Southern China
Associated with Epstein Barr virus infection

Presenting features
Systemic Local
Cervical lymphadenopathy Otalgia
Unilateral serous otitis media
Nasal obstruction, discharge and/ or epistaxis
Cranial nerve palsies e.g. III-VI

Imaging
Combined CT and MRI.

Treatment
Radiotherapy is first line therapy.

1910
Q

A 23 year old man is involved in a fight, during the dispute he sustains a laceration to the posterior aspect of his right arm, approximately 2cm proximal to the olecranon process. On assessment in the emergency department he is unable to extend his elbow joint. Which of the following tendons is most likely to have been cut?

Triceps

Pronator teres

Brachioradialis

Brachialis

Biceps

A

The triceps muscle extends the elbow joint. The other muscles listed all produce flexion of the elbow joint.

Triceps

Origin
Long head- infraglenoid tubercle of the scapula.
Lateral head- dorsal surface of the humerus, lateral and proximal to the groove of the radial nerve
Medial head- posterior surface of the humerus on the inferomedial side of the radial groove and both of the intermuscular septae
Insertion
Olecranon process of the ulna. Here the olecranon bursa is between the triceps tendon and olecranon.
Some fibres insert to the deep fascia of the forearm, posterior capsule of the elbow (preventing the capsule from being trapped between olecranon and olecranon fossa during extension)
Innervation Radial nerve
Blood supply Profunda brachii artery
Action Elbow extension. The long head can adduct the humerus and extend it from a flexed position
Relations The radial nerve and profunda brachii vessels lie between the lateral and medial heads

1911
Q

Which one of the following is most associated with an increased risk of venous thromboembolism?

Diabetes

Cannula

Hyperthyroidism

Tamoxifen

Amiodarone

A

Consider thromboembolism in breast cancer patients on tamoxifen!

Venous thromboembolism: risk factors

Common predisposing factors include malignancy, pregnancy and the period following an operation. The comprehensive list below is partly based on the 2010 SIGN venous thromboembolism (VTE) guidelines:

General
increased risk with advancing age
obesity
family history of VTE
pregnancy (especially puerperium)
immobility
hospitalisation
anaesthesia
central venous catheter: femoral&raquo_space; subclavian

Underlying conditions
malignancy
thrombophilia: e.g. Activated protein C resistance, protein C and S deficiency
heart failure
antiphospholipid syndrome
Behcet’s
polycythaemia
nephrotic syndrome
sickle cell disease
paroxysmal nocturnal haemoglobinuria
hyperviscosity syndrome
homocystinuria

Medication
combined oral contraceptive pill: 3rd generation more than 2nd generation
hormone replacement therapy
raloxifene and tamoxifen
antipsychotics (especially olanzapine) have recently been shown to be a risk factor

SIGN also state that the following are risk factors for recurrent VTE:
previous unprovoked VTE
male sex
obesity
thrombophilias

1912
Q

An 89 year old man presents with hypotension and collapse and is found by the staff in the toilet of his care home. He is moribund and unable to give a clear history. He had suffered a cardiac arrest in the ambulance but has since been resuscitated and now has a Bp of 95 systolic. He has an obviously palpable AAA. What is the best course of action?

Immediate CT scanning of the abdominal aorta

Immediate laparotomy

Immediate endovascular aortic aneurysm repair

Palliation

USS of aorta

A

The fact that he is frail (care home resident) and has already suffered a cardiac arrest means that attempts at surgery will invariably fail. At 89 years of age the functional outcomes were it to be ‘successful’ would be very poor. Palliation is therefore the best option, imaging will not change this management decision.
Abdominal aorta aneurysm

  • Abdominal aortic aneurysms are a common problem in vascular surgery.
    They may occur as either true or false aneurysm. With the former all 3 layers of the arterial wall are involved, in the latter only a single layer of fibrous tissue forms the aneurysm wall.
    True abdominal aortic aneurysms have an approximate incidence of 0.06 per 1000 people. They are commonest in elderly men and for this reason the UK is now introducing the aneurysm screening program with the aim of performing an abdominal aortic ultrasound measurement in all men aged 65 years.

Causes
Several different groups of patients suffer from aneurysmal disease.
The commonest group is those who suffer from standard arterial disease, i.e. Those who are hypertensive and have been or are smokers.
Other patients such as those suffering from connective tissue diseases such as Marfan’s may also develop aneurysms. In patients with abdominal aortic aneurysms the extracellular matrix becomes disrupted with a change in the balance of collagen and elastic fibres.

Management
Most abdominal aortic aneurysms are an incidental finding.
Symptoms most often relate to rupture or impending rupture.
20% rupture anteriorly into the peritoneal cavity. Very poor prognosis.
80% rupture posteriorly into the retroperitoneal space
The risk of rupture is related to aneurysm size, only 2% of aneurysms measuring less than 4cm in diameter will rupture over a 5 year period. This contrasts with 75% of aneurysms measuring over 7cm in diameter.
This is well explained by Laplaces’ law which relates size to transmural pressure.
For this reason most vascular surgeons will subject patients with an aneurysm size of 5cm or greater to CT scanning of the chest, abdomen and pelvis with the aim of delineating anatomy and planning treatment. Depending upon co-morbidities, surgery is generally offered once the aneurysm is between 5.5cm and 6cm.

Indications for surgery
Symptomatic aneurysms (80% annual mortality if untreated)
Increasing size above 5.5cm if asymptomatic
Rupture (100% mortality without surgery)

Surgical procedures
Abdominal aortic aneurysm repair

Procedure:

GA
Invasive monitoring (A-line, CVP, catheter)
Incision: Midline or transverse
Bowel and distal duodenum mobilised to access aorta.
Aneurysm neck and base dissected out and prepared for cross clamp
Systemic heparinisation
Cross clamp (proximal first)
Longitudinal aortotomy
Atherectomy
Deal with back bleeding from lumbar vessels and inferior mesenteric artery
Insert graft either tube or bifurcated depending upon anatomy
Suture using Prolene (3/0 for proximal , distal anastomosis suture varies according to site)
Clamps off: End tidal CO2 will rise owing to effects of reperfusion, at this point major risk of myocardial events.
Haemostasis
Closure of aneurysm sac to minimise risk of aorto-enteric fistula
Closure: Loop 1 PDS or Prolene to abdominal wall
Skin- surgeons preference

Post operatively:

ITU (Almost all)
Greatest risk of complications following emergency repair
Complications: Embolic- gut and foot infarcts
Cardiac - owing to premorbid states, re-perfusion injury and effects of cross clamp
Wound problems
Later risks related to graft- infection and aorto-enteric fistula

Special groups

Supra renal AAA
These patients will require a supra renal clamp and this carries a far higher risk of complications and risk of renal failure.

Ruptured AAA
Pre-operatively the management depends upon haemodynamic instability. In patients with symptoms of rupture (typical pain, haemodynamic compromise and risk factors) then ideally prompt laparotomy. In those with vague symptoms and haemodynamic stability the ideal test is CT scan to determine whether rupture has occurred or not. Most common rupture site is retroperitoneal 80%. These patients will tend to develop retroperitoneal haematoma. This can be disrupted if Bp is allowed to rise too high so aim for Bp 100mmHg.
Operative details are similar to elective repair although surgery should be swift, blind rushing often makes the situation worse. Plunging vascular clamps blindly into a pool of blood at the aneurysm neck carries the risk of injury the vena cava that these patients do not withstand. Occasionally a supracoeliac clamp is needed to effect temporary control, although leaving this applied for more than 20 minutes tends to carry a dismal outcome.

EVAR
Increasingly patients are now being offered endovascular aortic aneurysm repair. This is undertaken by surgeons and radiologists working jointly. The morphology of the aneurysm is important and not all are suitable. Here is a typical list of those features favoring a suitable aneurysm:
Long neck
Straight iliac vessels
Healthy groin vessels

Clearly few AAA patients possess the above and compromise has to be made. The use of fenestrated grafts can allow supra renal AAA to be treated.

Procedure:

GA
Radiology or theatre
Bilateral groin incisions
Common femoral artery dissected out
Heparinisation
Arteriotomy and insertion of guide wire
Dilation of arteriotomy
Insertion of EVAR Device
Once in satisfactory position it is released
Arteriotomy closed once check angiogram shows good position and no endoleak

Complications:
Endoleaks depending upon site are either Type I or 2. These may necessitate re-intervention and all EVAR patients require follow up . Details are not needed for MRCS.

References
A reasonable review is provided by:
Sakalihasan N, Limet R, Defawe O. Abdominal aortic aneurysm. Lancet 2005 (365):1577- 1589

1913
Q

A 56 year old surgeon has been successfully operating for many years. Over the past few weeks she has begun to notice that her hands are becoming blistering and weepy. A latex allergy is diagnosed. Which of the following pathological processes accounts for this scenario?

Type 1 hypersensitivity reaction

Type 2 hypersensitivity reaction

Type 4 hypersensitivity reaction

Type 3 hypersensitivity reaction

None of the above

A

Hypersensitivity reactions: ACID

type 1 –Anaphylactic
type 2 –Cytotoxic
type 3 –Immune complex
type 4 –Delayed hypersensitivity
Contact dermatitis of a chronic nature is an example of a type 4 hypersensitivity reaction. Type 4 hypersensitivity reactions are cell mediated rather than antibody mediated.

Hypersensitivity reactions

The Gell and Coombs classification divides hypersensitivity reactions into 4 types

Type I Type II Type III Type IV
Description Anaphylactic Cytotoxic Immune complex Delayed type
Mediator IgE IgG, IgM IgG, Ig A, IgM T-cells
Antigen Exogenous Cell surface Soluble Tissues
Response time Minutes Hours Hours 2-3 days
Examples Asthma
Hay fever Autoimmune haemolytic anaemia
Pemphigus
Goodpasture’s Serum sickness
SLE
Aspergillosis Graft versus host disease
Contact dermatitis

1914
Q

Which of the following nerves is the primary source of innervation to the anterior scrotal skin?

Iliohypogastric nerve

Pudendal nerve

Ilioinguinal nerve

Femoral branch of the genitofemoral nerve

Obturator nerve

A

The pudendal nerve may innervate the posterior skin of the scrotum. The anterior innervation of the scrotum is primarily provided by the ilioinguinal nerve. The genital branch of the genitofemoral nerve provides a smaller contribution.

Scrotal sensation

The scrotum is innervated by the ilioinguinal nerve and the pudendal nerve. The ilioinguinal nerve arises from L1 and pierces the internal oblique muscle. It eventually passes through the superficial inguinal ring to innervate the anterior skin of the scrotum.

The pudendal nerve is the principal nerve of the perineum. It arises in the pelvis from 3 nerve roots. It passes through both greater and lesser sciatic foramina to enter the perineal region. The perineal branches pass anteromedially and divide into posterior scrotal branches. The posterior scrotal branches pass superficially to supply the skin and fascia of the perineum. It cross communicates with the inferior rectal nerve.

1915
Q

An 8 year old boy presents with symptoms of right knee pain. The pain has been present on most occasions for the past three months and the pain typically lasts for several hours at a time. On examination; he walks with an antalgic gait and has apparent right leg shortening. What is the most likely diagnosis?

Perthes Disease

Osteosarcoma of the femur

Osteoarthritis of the hip

Transient synovitis of the hip

Torn medial meniscus

A

There are many causes of the irritable hip in the 10-14 year age group. Many of these may cause both hip pain or knee pain. Transient synovitis of the hip the commonest disorder but does not typically last for 3 months. An osteosarcoma would not usually present with apparent limb shortening unless pathological fracture had occurred. A slipped upper femoral epiphysis can cause a similar presentation although it typically presents later and with different patient characteristics.

Perthes disease

Perthes disease
Idiopathic avascular necrosis of the femoral epiphysis of the femoral head
Impaired blood supply to femoral head, causing bone infarction. New vessels develop and ossification occurs. The bone either heals or a subchondral fracture occurs.

Clinical features
Males 4x’s greater than females
Age between 2-12 years (the younger the age of onset, the better the prognosis)
Limp
Hip pain
Bilateral in 20%

Diagnosis
Plain x-ray, Technetium bone scan or magnetic resonance imaging if normal x-ray and symptoms persist.

Catterall staging
Stage Features
Stage 1 Clinical and histological features only
Stage 2 Sclerosis with or without cystic changes and preservation of the articular surface
Stage 3 Loss of structural integrity of the femoral head
Stage 4 Loss of acetabular integrity

Management
To keep the femoral head within the acetabulum: cast, braces
If less than 6 years: observation
Older: surgical management with moderate results
Operate on severe deformities

Prognosis
Most cases will resolve with conservative management. Early diagnosis improves outcomes.

1916
Q

A 56 year old lady reports incontinence mainly when walking the dog. A bladder diary is inconclusive. What is the most appropriate investigation?

Intravenous urography

Urodynamic studies

Flexible cystoscopy

Micturating cystourethrogram

Rigid cystoscopy

A

Urodynamic studies are indicated when there is diagnostic uncertainty or plans for surgery.

Urinary incontinence

Involuntary passage of urine. Most cases are female (80%). It has a prevalence of 11% in those aged greater than 65 years. The commonest variants include:
Stress urinary incontinence (50%)
Urge incontinence (15%)
Mixed (35%)

Males
Males may also suffer from incontinence although it is a much rarer condition in men. A number of anatomical factors contribute to this. Males have 2 powerful sphincters; one at the bladder neck and the other in the urethra. Damage to the bladder neck mechanism is a factor in causing retrograde ejaculation following prostatectomy. The short segment of urethra passing through the urogenital diaphragm consists of striated muscle fibres (the external urethral sphincter) and smooth muscle capable of more sustained contraction. It is the latter mechanism that maintains continence following prostatectomy.

Females
The sphincter complex at the level of bladder neck is poorly developed in females. As a result the external sphincter complex is functionally more important, its composition being similar to that of males. Innervation is via the pudendal nerve and the neuropathy that may accompany obstetric events may compromise this and lead to stress urinary incontinence.

Innervation
Somatic innervation to the bladder is via the pudendal, hypogastric and pelvic nerves. Autonomic nerves travel in these nerve fibres too. Bladder filling leads to detrusor relaxation (sympathetic) coupled with sphincter contraction. The parasympathetic system causes detrusor contraction and sphincter relaxation. Overall control of micturition is centrally mediated via centres in the Pons.

Stress urinary incontinence
50% of cases, especially in females.
Damage (often obstetric) to the supporting structures surrounding the bladder may lead to urethral hypermobility.
Other cases due to sphincter dysfunction, usually from neurological disorders (e.g. Pudendal neuropathy, multiple sclerosis).

Urethral mobility:
Pressure not transmitted appropriately to the urethra resulting in involuntary passage of urine during episodes of raised intra-abdominal pressure.

Sphincter dysfunction:
Sphincter fails to adapt to compress urethra resulting in involuntary passage of urine. When the sphincter completely fails there is often to continuous passage of urine.

Urge incontinence
In these patients there is sense of urgency followed by incontinence. The detrusor muscle in these patients is unstable and urodynamic investigation will demonstrate overactivity of the detrusor muscle at inappropriate times (e.g. Bladder filling). Urgency may be seen in patients with overt neurological disorders and those without. The pathophysiology is not well understood but poor central and peripheral co-ordination of the events surrounding bladder filling are the main processes.

Assessment
Careful history and examination including vaginal examination for cystocele.
Bladder diary for at least 3 days
Consider flow cystometry if unclear symptomatology or surgery considered and diagnosis is unclear.
Exclusion of other organic disease (e.g. Stones, UTI, Cancer)

Management
Conservative measures should be tried first; Stress urinary incontinence or mixed symptoms should undergo 3 months of pelvic floor exercise. Over active bladder should have 6 weeks of bladder retraining.
Drug therapy for women with overactive bladder should be offered oxybutynin (or solifenacin if elderly) if conservative measures fail.
In women with detrusor instability who fail non operative therapy a trial of sacral neuromodulation may be considered, with conversion to permanent implant if good response. Augmentation cystoplasty is an alternative but will involve long term intermittent self catheterisation.
In women with stress urinary incontinence a urethral sling type procedure may be undertaken. Where cystocele is present in association with incontinence it should be repaired particularly if it lies at the introitus.

NICE guidelines
Initial assessment urinary incontinence should be classified as stress/urge/mixed.
At least 3/7 bladder diary if unable to classify easily.
Start conservative treatment before urodynamic studies if a diagnosis is obvious from the history
Urodynamic studies if plans for surgery.
Stress incontinence: Pelvic floor exercises 3/12, if fails consider surgery.
Urge incontinence: Bladder training >6/52, if fails for oxybutynin (antimuscarinic drugs) then sacral nerve stimulation.
Pelvic floor exercises offered to all women in their 1st pregnancy.

1917
Q

Which of the following is not considered a risk factor for the development of oesophageal malignancy?

Oesophageal metaplasia

Smoking

Excessive intake of alcoholic spirits

Achalasia

Blood group O

A

Blood group O is not a risk factor for oesophageal cancer. Achalasia is associated with the risk of developing squamous cell carcinoma of the oesophagus.

Oesophageal cancer

  • Incidence is increasing
    In most cases in the Western world this increase is accounted for by a rise in the number of cases of adenocarcinoma. In the UK adenocarcinomas account for 65% of cases.
    Barretts oesophagus is a major risk factor for most cases of oesophageal adenocarcinoma.
    In other regions of the world squamous cancer is more common and is linked to smoking, alcohol intake, diets rich in nitrosamines and achalasia.
    Surveillance of Barretts is important, as it imparts a 30 fold increase in cancer risk and if invasive malignancy is diagnosed early then survival may approach 85% at 5 years.

Diagnosis
Upper GI endoscopy is the first line test
Contrast swallow may be of benefit in classifying benign motility disorders but has no place in the assessment of tumours
Staging is initially undertaken with CT scanning of the chest, abdomen and pelvis. If overt metastatic disease is identified using this modality then further complex imaging is unnecessary
If CT does not show metastatic disease, then local stage may be more accurately assessed by use of endoscopic ultrasound.
Staging laparoscopy is performed to detect occult peritoneal disease. PET CT is performed in those with negative laparoscopy. Thoracoscopy is not routinely performed.

Treatment
Operable disease is best managed by surgical resection. The most standard procedure is an Ivor- Lewis type oesophagectomy. This procedure involves the mobilisation of the stomach and division of the oesophageal hiatus. The abdomen is closed and a right sided thoracotomy performed. The stomach is brought into the chest and the oesophagus mobilised further. An intrathoracic oesophagogastric anastomosis is constructed. Alternative surgical strategies include a transhiatal resection (for distal lesions), a left thoraco-abdominal resection (difficult access due to thoracic aorta) and a total oesophagectomy (McKeown) with a cervical oesophagogastric anastomosis.
The biggest surgical challenge is that of anastomotic leak, with an intrathoracic anastomosis this will result in mediastinitis. With high mortality. The McKeown technique has an intrinsically lower systemic insult in the event of anastomotic leakage.

In addition to surgical resection many patients will be treated with adjuvant chemotherapy.

1918
Q

A 25 year old female presents with a lump in her neck. On examination, she has a discrete nodule in the right lobe of the thyroid. A fine needle aspirate shows papillary cells. An adjacent nodule is also sampled which shows similar well differentiated papillary cells. What is the most likely cause?

Hashimotos thyroiditis

Papillary carcinoma

Follicular carcinoma

Medullary carcinoma

Thyroid B Cell lymphoma

A

Multifocal disease is a recognised feature of papillary lesions. Papillary adenomas are not really recognised and most well differentiated lesions are papillary carcinomas.

Thyroid neoplasms

Lesion Common features
Follicular adenoma
Usually present as a solitary thyroid nodule
Malignancy can only be excluded on formal histological assessment
Papillary carcinoma
Usually contain a mixture of papillary and colloidal filled follicles
Histologically tumour has papillary projections and pale empty nuclei
Seldom encapsulated
Lymph node metastasis predominate
Haematogenous metastasis rare
Account for 60% of thyroid cancers
Follicular carcinoma
May appear macroscopically encapsulated, microscopically capsular invasion is seen. Without this finding the lesion is a follicular adenoma.
Vascular invasion predominates
Multifocal disease rare
Account for 20% of all thyroid cancers
Anaplastic carcinoma
Most common in elderly females
Local invasion is a common feature
Account for 10% of thyroid cancers
Treatment is by resection where possible, palliation may be achieved through isthmusectomy and radiotherapy. Chemotherapy is ineffective.
Medullary carcinoma
Tumours of the parafollicular cells (C Cells)
C cells derived from neural crest and not thyroid tissue
Serum calcitonin levels often raised
Familial genetic disease accounts for up to 20% cases
Both lymphatic and haematogenous metastasis are recognised, nodal disease is associated with a very poor prognosis.

1919
Q

A 43 year old lady is undergoing an axillary node clearance for breast cancer. The nodal disease is bulky. During clearance of the level 3 nodes there is suddenly brisk haemorrhage. The most likely vessel responsible is:

Thoracoacromial artery

Cephalic vein

Thoracodorsal trunk

Internal mammary artery

Posterior circumflex humeral artery

A

The thoracoacromial artery pierces the pectoralis major and gives off branches within this space. The level 3 axillary nodes lie between pectoralis major and minor.Although the thoracodorsal trunk may be injured during an axillary dissection it does not lie within the level 3 nodes.
Thoracoacromial artery

The thoracoacromial artery (acromiothoracic artery; thoracic axis) is a short trunk, which arises from the forepart of the axillary artery, its origin being generally overlapped by the upper edge of the Pectoralis minor.

Projecting forward to the upper border of the Pectoralis minor, it pierces the coracoclavicular fascia and divides into four branches: pectoral, acromial, clavicular, and deltoid.

Branch Description
Pectoral branch Descends between the two Pectoral muscles, and is distributed to them and to the breast, anastomosing with the intercostal branches of the internal thoracic artery and with the lateral thoracic.
Acromial branch Runs laterally over the coracoid process and under the Deltoid, to which it gives branches; it then pierces that muscle and ends on the acromion in an arterial network formed by branches from the suprascapular, thoracoacromial, and posterior humeral circumflex arteries.
Clavicular branch Runs upwards and medially to the sternoclavicular joint, supplying this articulation, and the Subclavius.
Deltoid branch Arising with the acromial, it crosses over the Pectoralis minor and passes in the same groove as the cephalic vein, between the Pectoralis major and Deltoid, and gives branches to both muscles.

1920
Q

What is the most common cause of osteolytic bone metastasis in children?

Osteosarcoma

Neuroblastoma

Leukaemia

Rhabdomyosarcoma

Medulloblastoma

A

Neuroblastomas are a relatively common childhood tumour and have a strong tendency to developing widespread lytic metastasis. It is unusual for CNS tumours to spread to involve the skeleton.

Secondary malignant tumours of bone

Metastatic lesions affecting bone are more common than primary bone tumours.

The typical tumours that spread to bone include:
Breast
Bronchus
Renal
Thyroid
Prostate

75% cases will affect those over the age of 50

The commonest bone sites affected are:
Vertebrae (usually thoracic)
Proximal femur
Ribs
Sternum
Pelvis
Skull

Pathological fracture
Osteolytic lesions are the greatest risk for pathological fracture
The risk and load required to produce fracture varies according to bone site. Bones with lesions that occupy 50% or less will be prone to fracture under loading (Harrington). When 75% of the bone is affected the process of torsion about a bony fulcrum may produce a fracture.

The Mirel scoring[1] system may be used to help determine the risk of fracture and is more systematic than the Harrington system described above.

Mirel Scoring system

Score points Site Radiographic appearance Width of bone involved Pain
1 Upper extremity Blastic Less than 1/3 Mild
2 Lower extremity Mixed 1/3 to 2/3 Moderate
3 Peritrochanteric Lytic More than 2/3 Aggravated by function

Depending upon the score the treatment should be as follows:

Score Risk of fracture Treatment
9 or greater Impending (33%) Prophylactic fixation
8 Borderline Consider fixation
7 or less Not impending (4%) Non operative management

Where the lesion is an isolated metastatic deposit consideration should be given to excision and reconstruction as the outcome is better [2].

Non operative treatments
Hypercalcaemia- Treat with re hydration and bisphosphonates.
Pain- Opiate analgesics and radiotherapy.
Some tumours such as breast and prostate will benefit from chemotherapy and or hormonal agents.

References
1. Mirels, H. Metastatic disease in long bones. A proposed scoring system for diagnosing impending pathologic fractures. Clin Orthop Relat Res, 1989(249): p. 256-64.
2. Mavrogenis, A.F. et al. Survival analysis of patients with femoral metastases. J Surg Oncol, 2011.

1921
Q

Which of the following is not a typical feature of talipes equinovarus?

Adducted and inverted calcaneus

Medial displacement of the navicular bone

It is nearly always unilateral

Wedge shaped head of talus

Severe Tibio-talar plantar flexion

A

It is bilateral in 50% of cases.

Talipes Equinovarus

Congenital talipes equinovarus.
Features:
Equinus of the hindfoot
Adduction and varus of the midfoot
High arch

Most cases in developing countries. Incidence in UK is 1 per 1000 live births. It is more common in males and is bilateral in 50% cases. There is a strong familial link(1). It may also be associated with other developmental disorders such as Down’s syndrome.

Key anatomical deformities (2):
Adducted and inverted calcaneus
Wedge shaped distal calcaneal articular surface
Severe Tibio-talar plantar flexion
Medial Talar neck inclination
Displacement of the navicular bone (medially)
Wedge shaped head of talus
Displacement of the cuboid (medially)

Management
Conservative first, the Ponseti method is best described and gives comparable results to surgery. It consists of serial casting to mold the foot into correct shape. Following casting around 90% will require a Achilles tenotomy. This is then followed by a phase of walking braces to maintain the correction.

Surgical correction is reserved for those cases that fail to respond to conservative measures. The procedures involve multiple tenotomies and lengthening procedures. In patients who fail to respond surgically an Ilizarov frame reconstruction may be attempted and gives good results.

References
1. Wynne-Davies R, Littlejohn A, Gormley J. Aetiology and interrelationship of some common skeletal deformities. (Talipes equinovarus and calcaneovalgus, metatarsus varus, congenital dislocation of the hip, and infantile idiopathic scoliosis). J Med Genet. 1982 Oct;19(5):321-8.
2. Horn BD, Davidson RS. Current treatment of clubfoot in infancy and childhood. Foot Ankle Clin. 2010 Jun;15(2):235-43.
3. Clarke NM, Uglow MG, Valentine KM. Comparison of Ponseti Versus Surgical Treatment in Congenital Talipes Equinovarus. J Foot Ankle Surg. 2011 Jun 14.

1922
Q

A 72 year old man collapses with sudden onset abdominal pain. He has been suffering from back pain recently and has been taking ibuprofen. What is the most likely cause?

Acute mesenteric artery embolus

Peritonitis due to peptic ulcer disease

Ruptured abdominal aortic aneurysm

Bleeding Dieulafoy lesion

Pancreatitis

A

The key feature here is the sudden nature of the collapse, this suggests a significant intra abdominal catastrophe. The history of back pain is also suggestive of recent aneurysm expansion.
Abdominal aorta aneurysm

  • Abdominal aortic aneurysms are a common problem in vascular surgery.
    They may occur as either true or false aneurysm. With the former all 3 layers of the arterial wall are involved, in the latter only a single layer of fibrous tissue forms the aneurysm wall.
    True abdominal aortic aneurysms have an approximate incidence of 0.06 per 1000 people. They are commonest in elderly men and for this reason the UK is now introducing the aneurysm screening program with the aim of performing an abdominal aortic ultrasound measurement in all men aged 65 years.

Causes
Several different groups of patients suffer from aneurysmal disease.
The commonest group is those who suffer from standard arterial disease, i.e. Those who are hypertensive and have been or are smokers.
Other patients such as those suffering from connective tissue diseases such as Marfan’s may also develop aneurysms. In patients with abdominal aortic aneurysms the extracellular matrix becomes disrupted with a change in the balance of collagen and elastic fibres.

Management
Most abdominal aortic aneurysms are an incidental finding.
Symptoms most often relate to rupture or impending rupture.
20% rupture anteriorly into the peritoneal cavity. Very poor prognosis.
80% rupture posteriorly into the retroperitoneal space
The risk of rupture is related to aneurysm size, only 2% of aneurysms measuring less than 4cm in diameter will rupture over a 5 year period. This contrasts with 75% of aneurysms measuring over 7cm in diameter.
This is well explained by Laplaces’ law which relates size to transmural pressure.
For this reason most vascular surgeons will subject patients with an aneurysm size of 5cm or greater to CT scanning of the chest, abdomen and pelvis with the aim of delineating anatomy and planning treatment. Depending upon co-morbidities, surgery is generally offered once the aneurysm is between 5.5cm and 6cm.

Indications for surgery
Symptomatic aneurysms (80% annual mortality if untreated)
Increasing size above 5.5cm if asymptomatic
Rupture (100% mortality without surgery)

Surgical procedures
Abdominal aortic aneurysm repair

Procedure:

GA
Invasive monitoring (A-line, CVP, catheter)
Incision: Midline or transverse
Bowel and distal duodenum mobilised to access aorta.
Aneurysm neck and base dissected out and prepared for cross clamp
Systemic heparinisation
Cross clamp (proximal first)
Longitudinal aortotomy
Atherectomy
Deal with back bleeding from lumbar vessels and inferior mesenteric artery
Insert graft either tube or bifurcated depending upon anatomy
Suture using Prolene (3/0 for proximal , distal anastomosis suture varies according to site)
Clamps off: End tidal CO2 will rise owing to effects of reperfusion, at this point major risk of myocardial events.
Haemostasis
Closure of aneurysm sac to minimise risk of aorto-enteric fistula
Closure: Loop 1 PDS or Prolene to abdominal wall
Skin- surgeons preference

Post operatively:

ITU (Almost all)
Greatest risk of complications following emergency repair
Complications: Embolic- gut and foot infarcts
Cardiac - owing to premorbid states, re-perfusion injury and effects of cross clamp
Wound problems
Later risks related to graft- infection and aorto-enteric fistula

Special groups

Supra renal AAA
These patients will require a supra renal clamp and this carries a far higher risk of complications and risk of renal failure.

Ruptured AAA
Pre-operatively the management depends upon haemodynamic instability. In patients with symptoms of rupture (typical pain, haemodynamic compromise and risk factors) then ideally prompt laparotomy. In those with vague symptoms and haemodynamic stability the ideal test is CT scan to determine whether rupture has occurred or not. Most common rupture site is retroperitoneal 80%. These patients will tend to develop retroperitoneal haematoma. This can be disrupted if Bp is allowed to rise too high so aim for Bp 100mmHg.
Operative details are similar to elective repair although surgery should be swift, blind rushing often makes the situation worse. Plunging vascular clamps blindly into a pool of blood at the aneurysm neck carries the risk of injury the vena cava that these patients do not withstand. Occasionally a supracoeliac clamp is needed to effect temporary control, although leaving this applied for more than 20 minutes tends to carry a dismal outcome.

EVAR
Increasingly patients are now being offered endovascular aortic aneurysm repair. This is undertaken by surgeons and radiologists working jointly. The morphology of the aneurysm is important and not all are suitable. Here is a typical list of those features favoring a suitable aneurysm:
Long neck
Straight iliac vessels
Healthy groin vessels

Clearly few AAA patients possess the above and compromise has to be made. The use of fenestrated grafts can allow supra renal AAA to be treated.

Procedure:

GA
Radiology or theatre
Bilateral groin incisions
Common femoral artery dissected out
Heparinisation
Arteriotomy and insertion of guide wire
Dilation of arteriotomy
Insertion of EVAR Device
Once in satisfactory position it is released
Arteriotomy closed once check angiogram shows good position and no endoleak

Complications:
Endoleaks depending upon site are either Type I or 2. These may necessitate re-intervention and all EVAR patients require follow up . Details are not needed for MRCS.

References
A reasonable review is provided by:
Sakalihasan N, Limet R, Defawe O. Abdominal aortic aneurysm. Lancet 2005 (365):1577- 1589

1923
Q

The parents of a 1 year old child are concerned after he develops a lump in his neck. On examination, there is a swelling in the subcutaneous tissue of the posterior triangle which transilluminates. What is the most likely cause?

Branchial cyst

Branchial sinus

Cystic hygroma

Lipoma

Pharyngeal pouch

A

This is a classical description. Collection of dilated lymphatic sacs which are fluctuant and brilliantly transilluminable. Recurrence following surgical excision is not uncommon.

Neck lumps

The table below gives characteristic exam question features for conditions causing neck lumps:

Reactive lymphadenopathy By far the most common cause of neck swellings. There may be a history of local infection or a generalised viral illness
Lymphoma Rubbery, painless lymphadenopathy
The phenomenon of pain whilst drinking alcohol is very uncommon
There may be associated night sweats and splenomegaly
Thyroid swelling May be hypo-, eu- or hyperthyroid symptomatically
Moves upwards on swallowing
Thyroglossal cyst More common in patients < 20 years old
Usually midline, between the isthmus of the thyroid and the hyoid bone
Moves upwards with protrusion of the tongue
May be painful if infected
Pharyngeal pouch More common in older men
Represents a posteromedial herniation between thyropharyngeus and cricopharyngeus muscles
Usually not seen, but if large then a midline lump in the neck that gurgles on palpation
Typical symptoms are dysphagia, regurgitation, aspiration and chronic cough
Cystic hygroma A congenital lymphatic lesion (lymphangioma) typically found in the neck, classically on the left side
Most are evident at birth, around 90% present before 2 years of age
Branchial cyst An oval, mobile cystic mass that develops between the sternocleidomastoid muscle and the pharynx
Develop due to failure of obliteration of the second branchial cleft in embryonic development
Usually present in early adulthood
Cervical rib More common in adult females
Around 10% develop thoracic outlet syndrome
Carotid aneurysm Pulsatile lateral neck mass which doesn’t move on swallowing

1924
Q

Which of the following muscle relaxants is an agent that is degraded by hydrolysis and may produce histamine release?

Atracurium

Panciuronium

Curare

Suxamethonium

Vecuronium

A

Atracurium is degraded by a process of ester hydrolysis. This uses non specific plasma esterases.

Muscle relaxants

Suxamethonium
Depolarising neuromuscular blocker
Inhibits action of acetylcholine at the neuromuscular junction
Degraded by plasma cholinesterase and acetylcholinesterase (affected by lack of acetylcholinesterase)
Fastest onset and shortest duration of action of all muscle relaxants
Produces generalised muscular contraction prior to paralysis
Adverse effects include hyperkalaemia, malignant hyperthermia, delayed recovery
Atracurium
Non depolarising neuromuscular blocking drug
Duration of action usually 30-45 minutes
Generalised histamine release on administration may produce facial flushing, tachycardia and hypotension
Not excreted by liver or kidney, broken down in tissues by hydrolysis
Reversed by neostigmine
Vecuronium
Non depolarising neuromuscular blocking drug
Duration of action approximately 30 - 40 minutes
Degraded by liver and kidney and effects prolonged in organ dysfunction
Effects may be reversed by neostigmine
Pancuronium
Non depolarising neuromuscular blocker
Onset of action approximately 2-3 minutes
Duration of action up to 2 hours
Effects may be partially reversed with drugs such as neostigmine

1925
Q

A 20 year old man falls over and bangs his head whilst intoxicated. On arrival in the emergency department he opens his eyes in response to speech, and is able to speak, although he is disorientated. He obeys motor commands. What is his Glasgow coma score?

10

8

15

13

11

A

E=3, V=4, M=6.

Glasgow coma scale

Modality Options
Eye opening
Spontaneous
To speech
To pain
None
Verbal response
Orientated
Confused
Words
Sounds
None
Motor response
Obeys commands
Localises to pain
Withdraws from pain
Abnormal flexion to pain (decorticate posture)
Extending to pain
None

Responses are taken from each category (marks in descending order) to produce an overall score. Severe brain injuries are generally associated with GCS <8.

1926
Q

Cortisol is predominantly produced by which of the following?

Zona fasciculata of the adrenal

Zona glomerulosa of the adrenal

Zona reticularis of the adrenal

Adrenal medulla

Posterior lobe of the pituitary

A

Relative Glucocorticoid activity:

Hydrocortisone = 1
Prednisolone = 4
Dexamethasone = 25
Cortisol is produced by the zona fasciculata of the adrenal gland.

Cortisol

Glucocorticoid
Released by zona fasiculata of the adrenal gland
90% protein bound; 10% active
Circadian rhythm: High in the mornings
Negative feedback via ACTH

Actions
Glycogenolysis
Gluconeogenesis
Protein catabolism
Lipolysis
Stress response
Anti-inflammatory
Decrease protein in bones
Increase gastric acid
Increases neutrophils/platelets/red blood cells
Inhibits fibroblastic activity

1927
Q

A 37 year old man is involved in a road traffic accident and his spleen is ruptured. There is major haemorrhage and he requires a splenectomy. In the days following surgery, his renal function is noted to deteriorate. What is the most likely reason for this?

Glomerulonephritis

Nephrotic syndrome

Acute tubular necrosis

Interstitial nephritis

Nephrosclerosis

A

Acute haemorrhage and associated shock exceed the renovascular compensatory mechanisms. This results in renovascular vasoconstriction of glomerulotubular hypoperfusion. This can result is acute tubular necrosis. An acute inflammatory nephritis is less likely in this setting.

Acute Renal Failure

  • Final pathway is tubular cell death.
    Renal medulla is a relatively hypoxic environment making it susceptible to renal tubular hypoxia.
    Renovascular autoregulation maintains renal blood flow across a range of arterial pressures.
    Estimates of GFR are best indices of level of renal function. Useful clinical estimates can be obtained by considering serum creatinine, age, race, gender and body size. eGFR calculations such as the Cockcroft and Gault equation are less reliable in populations with high GFR’s.
    Nephrotoxic stimuli such as aminoglycosides and radiological contrast media induce apoptosis. Myoglobinuria and haemolysis result in necrosis. Overlap exists and proinflammatory cytokines play an important role in potentiating ongoing damage.
    Post-operative renal failure is more likely to occur in patients who are elderly, have peripheral vascular disease, high BMI, have COPD, receive vasopressors, are on nephrotoxic medication or undergo emergency surgery.
    Avoiding hypotension will reduce risk of renal tubular damage.
    There is no evidence that administration of ACE inhibitors or dopamine reduces the incidence of post-operative renal failure.
1928
Q

A 42 year old female presents with symptoms of biliary colic and on investigation is identified as having gallstones. Of the procedures listed below, which is most likely to increase the risk of gallstone formation?

Partial gastrectomy

Jejunal resection

Liver lobectomy

Ileal resection

Left hemicolectomy

A

Bile salt reabsorption occurs at the ileum. Therefore cholesterol gallstones form as a result of ileal resection.
Biliary disease

Diagnosis Typical features Pathogenesis
Gallstones Typically history of biliary colic or episodes of chlolecystitis. Obstructive type history and test results. Usually small calibre gallstones which can pass through the cystic duct. In Mirizzi syndrome the stone may compress the bile duct directly- one of the rare times that cholecystitis may present with jaundice
Cholangitis Usually obstructive and will have Charcot’s triad of symptoms (pain, fever, jaundice) Ascending infection of the bile ducts usually by E. coli and by definition occurring in a pool of stagnant bile.
Pancreatic cancer Typically painless jaundice with palpable gallbladder (Courvoisier’s Law) Direct occlusion of distal bile duct or pancreatic duct by tumour. Sometimes nodal disease at the portal hepatis may be the culprit in which case the bile duct may be of normal calibre.
TPN (total parenteral nutrition) associated jaundice Usually follows long term use and is usually painless with non obstructive features Often due to hepatic dysfunction and fatty liver which may occur with long term TPN usage.
Bile duct injury Depending upon the type of injury may be of sudden or gradual onset and is usually of obstructive type Often due to a difficult laparoscopic cholecystectomy when anatomy in Calots triangle is not appreciated. In the worst scenario the bile duct is excised and jaundice develops rapidly post operatively. More insidious is that of bile duct stenosis which may be caused by clips or diathermy injury.
Cholangiocarcinoma Gradual onset obstructive pattern Direct occlusion by disease and also extrinsic compression by nodal disease at the porta hepatis.
Septic surgical patient Usually hepatic features Combination of impaired biliary excretion and drugs such as ciprofloxacin which may cause cholestasis.
Metastatic disease Mixed hepatic and post hepatic Combination of liver synthetic failure (late) and extrinsic compression by nodal disease and anatomical compression of intra hepatic structures (earlier)

1929
Q

An 18 year old man is involved in a road traffic accident. He arrives haemodynamically unstable. A CT scan shows disruption of the splenic hilum and a moderate sized perisplenic haematoma. What is the best course of action?

Manage conservatively

Arrange USS

Transfer to centre for interventional radiology

Discharge

Splenectomy

A

Hilar injuries usually mandate splenectomy. The main risk with conservative management here is that he will rebleed and with hilar injuries this can be dramatic.

Splenic trauma

  • The spleen is one of the more commonly injured intra abdominal organs
    In most cases the spleen can be conserved. The management is dictated by the associated injuries, haemodynamic status and extent of direct splenic injury.

Management of splenic trauma
Conservative Small subcapsular haematoma
Minimal intra abdominal blood
No hilar disruption
Laparotomy with conservation Increased amounts of intraabdominal blood
Moderate haemodynamic compromise
Tears or lacerations affecting <50%
Resection Hilar injuries
Major haemorrhage
Major associated injuries

Splenectomy
Technique
Trauma
GA
Long midline incision
If time permits insert a self retaining retractor (e.g. Balfour/ omnitract)
Large amount of free blood is usually present. Pack all 4 quadrants of the abdomen. Allow the anaesthetist to ‘catch up’
Remove the packs and assess the viability of the spleen. Hilar injuries and extensive parenchymal lacerations will usually require splenectomy.
Divide the short gastric vessels and ligate them.
Clamp the splenic artery and vein. Two clamps on the patient side are better and allow for double ligation and serve as a safety net if your assistant does not release the clamp smoothly.
Be careful not to damage the tail of the pancreas, if you do then this will need to be formally removed and the pancreatic duct closed.
Wash out the abdomen and place a tube drain to the splenic bed.
Some surgeons implant a portion of spleen into the omentum, whether you decide to do this is a matter of personal choice.
Post operatively the patient will require prophylactic penicillin V and pneumococcal vaccine.

Elective
Elective splenectomy is a very different operation from that performed in the emergency setting. The spleen is often large (sometimes massive). Most cases can be performed laparoscopically. The spleen will often be macerated inside a specimen bag to facilitate extraction.

Complications
Haemorrhage (may be early and either from short gastrics or splenic hilar vessels
Pancreatic fistula (from iatrogenic damage to pancreatic tail)
Thrombocytosis: prophylactic aspirin
Encapsulated bacteria infection e.g. Strep. pneumoniae, Haemophilus influenzae and Neisseria meningitidis

1930
Q

A 6 year old boy pulls over a kettle and suffers superficial partial thickness burns to his legs. Which of the following will not occur?

Preservation of hair follicles

Formation of vesicles or bullae

Damage to sweat glands

Healing by re-epithelialisation

Pain at the burn site

A

Partial thickness burns are divided into superficial and deep burns, however, this is often not possible on initial assessment and it may be a week or more before the distinction is clear cut. Dermal appendages are, by definition, intact. Superficial partial thickness burns will typically heal by re-epithelialisation, deeper burns will heal with scarring.

Burns

Burns may be thermal, chemical or electrical. In the former category are burns which occur as a result of heat. Chemical burns occur when the skin is exposed to an extremely caustic or alkaline substance. Electrical burns occur following exposure to electrical current. The immediate management includes removal of the burning source which usually includes irrigation of the burned area. A detailed assessment then needs to be made of the extent of the burns and a number of charts are available for recording this information. The degree of injury relates to the temperature and duration of exposure. Most domestic burns are mainly scalds in young children.

Following the burn, there is a local response with progressive tissue loss and release of inflammatory cytokines. Systemically, there are cardiovascular effects resulting from fluid loss and sequestration of fluid into the third space. There is a marked catabolic response. Immunosuppression is common with large burns and bacterial translocation from the gut lumen is a recognised event. Sepsis is a common cause of death following major burns.

Types of burn

Type of burn Skin layers affected Skin appearance Blanching Management
Epidermal/Superficial Epidermis Red, moist Yes
Superficial partial thickness Epidermis and part of papillary dermis affected Pale, dry Yes Normally heals with no intervention
Deep partial thickness Epidermis, whole papillary dermis affected Mottled red colour No Needs surgical intervention (depending on site)
Full thickness Whole skin layer and subcutaneous tissue affected Dry, leathery hard wound No Burns centre

Depth of burn assessment
Bleeding on needle prick
Sensation
Appearance
Blanching to pressure

Percentage burn estimation
Lund Browder chart: most accurate even in children
Wallace rule of nines
Palmar surface: surface area palm = 0.8% burn

> 15% body surface area burns in adults needs urgent burn fluid resuscitation

Transfer to burn centre if:
Need burn shock resuscitation
Face/hands/genitals affected
Deep partial thickness or full thickness burns
Significant electrical/chemical burns

Management
The initial aim is to stop the burning process and resuscitate the patient. Intravenous fluids will be required for children with burns greater than 10% of total body surface area. Adults with burns greater than 15% of total body surface area will also require IV fluids. The fluids are calculated using the Parkland formula which is; volume of fluid= total body surface area of the burn % x weight (Kg) x2-4 (preference for lower amount i.e. 2ml to avoid excessive fluid overload). Half of the fluid is administered in the first 8 hours. A urinary catheter should be inserted. Analgesia should be given. Complex burns, burns involving the hand perineum and face and burns >10% in adults and >5% in children should be transferred to a burns unit.

Circumferential burns affecting a limb or severe torso burns impeding respiration may require escharotomy to divide the burnt tissue.

Conservative management is appropriate for superficial burns and mixed superficial burns that will heal in 2 weeks. More complex burns may require excision and skin grafting. Excision and primary closure is not generally practised as there is a high risk of infection.

There is no evidence to support the use of anti microbial prophylaxis or topical antibiotics in burn patients.

Escharotomies
Indicated in circumferential full thickness burns to the torso or limbs.
Careful division of the encasing band of burn tissue will potentially improve ventilation (if the burn involves the torso), or relieve compartment syndrome and oedema (where a limb is involved)

References
www.euroburn.org/e107files/downloads/guidelinesburncare.pdf

Barajas-Nava LA, López-Alcalde J, Roqué i Figuls M, Solà I, Bonfill Cosp X. Antibiotic prophylaxis for preventing burn wound infection. Cochrane Database of Systematic Reviews 2013, Issue 6. Art. No.: CD008738. DOI: 10.1002/14651858.CD008738.pub2.

Hettiaratchy S & Papini R. Initial management of a major burn: assessment and resuscitation. BMJ 2004;329:101-103

1931
Q

A 22 year old female undergoes a thyroidectomy. The resected specimen shows a non encapsulated tumour with papillary projections and pale empty nuclei. What is the underlying diagnosis?

Papillary carcinoma

Follicular carcinoma

Medullary carcinoma

Anaplastic carcinoma

B Cell lymphoma

A

The presence of papillary structures together with the cytoplasmic features described is strongly suggestive of papillary carcinoma. They are seldom encapsulated.

Thyroid neoplasms

Lesion Common features
Follicular adenoma
Usually present as a solitary thyroid nodule
Malignancy can only be excluded on formal histological assessment
Papillary carcinoma
Usually contain a mixture of papillary and colloidal filled follicles
Histologically tumour has papillary projections and pale empty nuclei
Seldom encapsulated
Lymph node metastasis predominate
Haematogenous metastasis rare
Account for 60% of thyroid cancers
Follicular carcinoma
May appear macroscopically encapsulated, microscopically capsular invasion is seen. Without this finding the lesion is a follicular adenoma.
Vascular invasion predominates
Multifocal disease rare
Account for 20% of all thyroid cancers
Anaplastic carcinoma
Most common in elderly females
Local invasion is a common feature
Account for 10% of thyroid cancers
Treatment is by resection where possible, palliation may be achieved through isthmusectomy and radiotherapy. Chemotherapy is ineffective.
Medullary carcinoma
Tumours of the parafollicular cells (C Cells)
C cells derived from neural crest and not thyroid tissue
Serum calcitonin levels often raised
Familial genetic disease accounts for up to 20% cases
Both lymphatic and haematogenous metastasis are recognised, nodal disease is associated with a very poor prognosis.

1932
Q

A 45 year old man is admitted to the intensive care unit following a laparotomy. He has a central line, pulmonary artery catheter and arterial lines inserted. The following results are obtained:
Pulmonary artery occlusion pressure Cardiac output Systemic vascular resistance
Low Low High
What is the most likely interpretation of this?

Normal

Cardiogenic shock

Septic shock

Fluid overload

Hypovolaemia

A

Cardiac output is lowered in hypovolaemia due to decreased preload.

Pulmonary artery occlusion pressure monitoring

The pulmonary artery occlusion pressure is an indirect measure of left atrial pressure, and thus filling pressure of the left heart. The low resistance within the pulmonary venous system allows this useful measurement to be made. The most accurate trace is made by inflating the balloon at the catheter tip and ‘floating’ it so that it occludes the vessel. If it is not possible to occlude the vessel in this way then the measurement gained will be the pulmonary artery end diastolic pressure.

Interpretation of PAOP
PAOP mmHg Scenario
Normal 8-12
Low <5 Hypovolaemia
Low with pulmonary oedema <5 ARDS
High >18 Overload

When combined with measurements of systemic vascular resistance and cardiac output it is possible to accurately classify patients.

Systemic vascular resistance
Derived from aortic pressure, right atrial pressure and cardiac output.

SVR=80(mean aortic pressure-mean right atrial pressure)/cardiac output

1933
Q

A 73 year old lady presents with rectal bleeding and after extensive investigation, a diagnosis of rectal amyloidosis is suspected. A biopsy is taken for histopathological evaluation. This is stained with congo red stain and evaluated microscopically. Which of the findings described below would be consistent with a diagnosis of amyloidosis?

Negative birefringence and bright red staining of tissue fibres under polarized light

Positive birefringence and bright red fibres under polarized light

Positive birefringence and white under polarized light

Positive birefringence and green staining of fibres under polarized light

Negative birefringence and blue staining of fibres under polarized light

A

Amyloid is best assessed with a congo red stain. The tissues are positively birefringent under polarized light and typically stain green.

Amyloid

Amyloid is an extracellular protein deposit which is insoluble. These deposits disrupt normal tissue structure and if excessive may affect function. All types of amyloid consist of a major fibrillar protein that defines the type of amyloid (approximately 90%) plus various minor components.

Amyloid is classified with the prefix A (for amyloid) and the suffix depending upon the fibrillary protein present. The main clinical types are AA and AL amyloidosis. Systemic AA amyloidosis is a long-term complication of several chronic inflammatory disorders - e.g. rheumatoid arthritis, ankylosing spondylitis, Crohn’s disease, malignancies and conditions predisposing to recurrent infections. AL amyloidosis results from extra-cellular deposition of fibril-forming monoclonal immunoglobulin light chains (most commonly of lambda isotype). Most patients have evidence of isolated monoclonal gammopathy or asymptomatic myeloma, and the occurrence of AL amyloidosis in patients with symptomatic multiple myeloma or other B-cell lymphoproliferative disorders is unusual. AL type amyloidosis is the most common variant. The kidney and heart are two of the most commonly affected sites. Diagnosis is based on surgical biopsy and characteristic histological features which consist birefringence under polarised light. Immunohistochemistry is used to delineate the subtype. Treatment is usually targeted at the underlying cause.

1934
Q

Which of the following has the greatest impact on the positive predictive value of a test?

Prevalence

Subjects who are true negatives

Specificity

Relative risk

None of the above

A

The positive predictive value (PPV) is the probability that an individual with a positive screening result has the disease. The sensitivity is the probability that an individual with the disease is screened positive and the specificity is the probability that an individual without the disease is screened negative.
Its value depends upon the prevalence of the condition being tested for and the sensitivity of the test used.
It may be calculated by dividing the number of true positives by the number of true positives and the number of false positives.

Positive predictive values

Screening tests
Sensitivity: proportion of true positives identified by a test
Specificity: proportion of true negatives correctly identified by a test
Positive predictive value: proportion of those who have a positive test who actually have the disease
Negative predictive value: proportion of those who test negative who do not have the disease

Predictive values are dependent on the prevalence

Likelihood ratio for a positive test result = sensitivity/(1-specificity)
Likelihood ratio for a negative test result = (1-sensitivity)/specificity

Likelihood ratios are not prevalence dependent

1935
Q

A 28 year old lady is breast feeding her first child. She presents with discomfort of the right breast. Clinical examination demonstrates erythema and an area that is fluctuant. Aspiration and culture of the fluid is most likely to demonstrate infection with which of the following organisms?

Lactobacillus caseii

Staphylococcus aureus

Streptococcus pyogenes

Staphylococcus epidermidis

Actinomycosis

A

Staphylococcus aureus is the commonest cause. The infants mouth is usually the source as it damages the nipple areolar complex allowing entry of bacteria.

Breast abscess

In lactational women Staphylococcus aureus is the most common cause
Typical presentation is with a tender, fluctuant mass in a lactating women
Diagnosis and treatment is performed using USS and associated drainage of the abscess cavity. Antibiotics should also be administered
Where there is necrotic skin overlying the abscess, the patient should undergo surgery

1936
Q

Which of these options represents the most common abdominal emergency in children under 1 year of age?

Appendicitis

Intussusception

Intestinal malrotation

Pyloric stenosis

Mid gut volvulus

A

The commonest emergency in this age group is inguinal hernia followed by intussusception. Appendicitis is commoner in those older than 1 year of age.

Paediatric Gastrointestinal disorders

Pyloric stenosis
M>F
5-10% Family history in parents
Projectile non bile stained vomiting at 4-6 weeks of life
Diagnosis is made by test feed or USS
Treatment: Ramstedt pyloromyotomy (open or laparoscopic)
Acute appendicitis
Uncommon under 3 years
When occurs may present atypically
Mesenteric adenitis
Central abdominal pain and URTI
Conservative management
Intussusception
Telescoping bowel
Proximal to or at the level of, ileocaecal valve
6-9 months age
Colicky pain, diarrhoea and vomiting, sausage shaped mass, red jelly stool.
Treatment: reduction with air insufflation
Malrotation
High caecum at the midline
Feature in exomphalos, congenital diaphragmatic hernia, intrinsic duodenal atresia
May be complicated by development of volvulus, infant with volvulus may have bile stained vomiting
Diagnosis is made by upper GI contrast study and USS
Treatment is by laparotomy, if volvulus is present (or at high risk of occurring then a ladds procedure is performed
Hirschsprung’s disease
Absence of ganglion cells from myenteric and submucosal plexuses
Occurs in 1/5000 births
Full thickness rectal biopsy for diagnosis
Delayed passage of meconium and abdominal distension
Treatment is with rectal washouts initially, thereafter an anorectal pull through procedure
Oesophageal atresia
Associated with tracheo-oesophageal fistula and polyhydramnios
May present with choking and cyanotic spells following aspiration
VACTERL associations
Meconium ileus
Usually delayed passage of meconium and abdominal distension
Majority have cystic fibrosis
X-Rays may not show a fluid level as the meconium is viscid (depends upon feeding), PR contrast studies may dislodge meconium plugs and be therapeutic
Infants who do not respond to PR contrast and NG N-acetyl cysteine will require surgery to remove the plugs
Biliary atresia
Jaundice > 14 days
Increased conjugated bilirubin
Urgent Kasai procedure
Necrotising enterocolitis
Prematurity is the main risk factor
Early features include abdominal distension and passage of bloody stools
X-Rays may show pneumatosis intestinalis and evidence of free air
Increased risk when empirical antibiotics are given to infants beyond 5 days
Treatment is with total gut rest and TPN, babies with perforations will require laparotomy

1937
Q

Which of the following statements relating to Keloid scars is untrue?

They have a predilection for sternal , mandibular and deltoid area wounds

They are confined to the margins of the original injury

They often recur following excision

May occur even after superficial injury

They may be treated by injection of triamcinolone

A

Hypertrophic scars remain confined to the wound edges.
Keloids (by definition) will tend to extend beyond the margins of the wound and in wounds of any depth.

Wound healing

Surgical wounds are either incisional or excisional and either clean, clean contaminated or dirty. Although the stages of wound healing are broadly similar their contributions will vary according to the wound type.

The main stages of wound healing include:

Haemostasis
Minutes to hours following injury
Vasospasm in adjacent vessels, platelet plug formation and generation of fibrin rich clot.

Inflammation
Typically days 1-5
Neutrophils migrate into wound (function impaired in diabetes).
Growth factors released, including basic fibroblast growth factor and vascular endothelial growth factor.
Fibroblasts replicate within the adjacent matrix and migrate into wound.
Macrophages and fibroblasts couple matrix regeneration and clot substitution.

Regeneration
Typically days 7 to 56
Platelet derived growth factor and transformation growth factors stimulate fibroblasts and epithelial cells.
Fibroblasts produce a collagen network.
Angiogenesis occurs and wound resembles granulation tissue.

Remodeling
From 6 weeks to 1 year
Longest phase of the healing process and may last up to one year (or longer).
During this phase fibroblasts become differentiated (myofibroblasts) and these facilitate wound contraction.
Collagen fibres are remodeled.
Microvessels regress leaving a pale scar.

The above description represents an idealised scenario. A number of diseases may distort this process. Neovascularisation is an important early process. Endothelial cells may proliferate in the wound bed and recanalise to form a vessel. Vascular disease, shock and sepsis can all compromise microvascular flow and impair healing.

Conditions such as jaundice will impair fibroblast synthetic function and immunity with a detrimental effect in most parts of the healing process.

Problems with scars:

Hypertrophic scars
Excessive amounts of collagen within a scar. Nodules may be present histologically containing randomly arranged fibrils within and parallel fibres on the surface. The tissue itself is confined to the extent of the wound itself and is usually the result of a full thickness dermal injury. They may go on to develop contractures.

Image of hypertrophic scarring. Note that it remains confined to the boundaries of the original wound:

1938
Q

A 78 year old lady presents with colicky abdominal pain and a tender mass in her groin. On examination; there is a small firm mass below and lateral to the pubic tubercle. Which of the following is the most likely underlying diagnosis?

Incarcerated inguinal hernia

Thrombophlebitis of a saphena varix

Incarcerated femoral hernia

Incarcerated obturator hernia

Deep vein thrombosis

A

Femoral hernia = High risk of strangulation (repair urgently)
Femoral herniae account for <10% of all groin hernias. In the scenario the combination of symptoms of intestinal compromise with a mass in the region of the femoral canal points to femoral hernia as the most likely cause.
Femoral canal

The femoral canal lies at the medial aspect of the femoral sheath. The femoral sheath is a fascial tunnel containing both the femoral artery laterally and femoral vein medially. The canal lies medial to the vein.

Borders of the femoral canal
Laterally Femoral vein
Medially Lacunar ligament
Anteriorly Inguinal ligament
Posteriorly Pectineal ligament

Contents
Lymphatic vessels
Cloquet’s lymph node

Physiological significance
Allows the femoral vein to expand to allow for increased venous return from the lower limbs.

Pathological significance
As a potential space, it is the site of femoral hernias. The relatively tight neck places these at high risk of strangulation.

1939
Q

A 45 year old female is diagnosed as having a carcinoma of the caecum. She undergoes a CT scan which shows a tumour invading the muscularis propria with some regional lymphadenopathy. What is the most appropriate initial treatment?

Right hemicolectomy

External beam radiotherapy

Chemotherapy

Combined long course chemo radiotherapy

Referral for palliative care

A

Right sided colonic cancers should proceed straight to surgery. Radiotherapy to this area is poorly tolerated and almost never offered as first line treatment. The decision as to whether or not chemotherapy is given is dependent upon the final histology.

Colorectal cancer treatment

Patients diagnosed as having colorectal cancer should be completely staged using CT of the chest/ abdomen and pelvis. Their entire colon should have been evaluated with colonoscopy or CT colonography. Patients whose tumours lie below the peritoneal reflection should have their mesorectum evaluated with MRI.

Once their staging is complete patients should be discussed within a dedicated colorectal MDT meeting and a treatment plan formulated.

Treatment of colonic cancer
Cancer of the colon is nearly always treated with surgery. Stents, surgical bypass and diversion stomas may all be used as palliative adjuncts. Resectional surgery is the only option for cure in patients with colon cancer. The procedure is tailored to the patient and the tumour location. The lymphatic drainage of the colon follows the arterial supply and therefore most resections are tailored around the resection of particular lymphatic chains (e.g. ileo-colic pedicle for right sided tumours). Some patients may have confounding factors that will govern the choice of procedure, for example a tumour in a patient from a HNPCC family may be better served with a panproctocolectomy rather than segmental resection. Following resection the decision has to be made regarding restoration of continuity. For an anastomosis to heal the key technical factors include; adequate blood supply, mucosal apposition and no tissue tension. Surrounding sepsis, unstable patients and inexperienced surgeons may compromise these key principles and in such circumstances it may be safer to construct an end stoma rather than attempting an anastomosis.
When a colonic cancer presents with an obstructing lesion; the options are to either stent it or resect. In modern practice it is unusual to simply defunction a colonic tumour with a proximal loop stoma. This differs from the situation in the rectum (see below).
Following resection patients with risk factors for disease recurrence are usually offered chemotherapy, a combination of 5FU and oxaliplatin is common.

Rectal cancer
The management of rectal cancer is slightly different to that of colonic cancer. This reflects the rectum’s anatomical location and the challenges posed as a result. Tumours located in the rectum can be surgically resected with either an anterior resection or an abdomino - perineal resection. The technical aspects governing the choice between these two procedures can be complex to appreciate and the main point to appreciate for the MRCS is that involvement of the sphincter complex or very low tumours require APER. In the rectum a 2cm distal clearance margin is required and this may also impact on the procedure chosen. In addition to excision of the rectal tube an integral part of the procedure is a meticulous dissection of the mesorectal fat and lymph nodes (total mesorectal excision/ TME). In rectal cancer surgery invovlement of the cirumferential resection margin carries a high risk of disease recurrence. Because the rectum is an extraperitoneal structure (until you remove it that is!) it is possible to irradiate it, something which cannot be offered for colonic tumours. This has a major impact in rectal cancer treatment and many patients will be offered neoadjuvent radiotherapy (both long and short course) prior to resectional surgery. Patients with T1, 2 and 3 /N0 disease on imaging do not require irradiation and should proceed straight to surgery. Patients with T4 disease will typically have long course chemo radiotherapy. Patients presenting with large bowel obstruction from rectal cancer should not undergo resectional surgery without staging as primary treatment (very different from colonic cancer). This is because rectal surgery is more technically demanding, the anastomotic leak rate is higher and the danger of a positive resection margin in an unstaged patient is high. Therefore patients with obstructing rectal cancer should have a defunctioning loop colostomy.

Summary of procedures
The operations for cancer are segmental resections based on blood supply and lymphatic drainage. These commonly performed procedures are core knowledge for the MRCS and should be understood.

Site of cancer Type of resection Anastomosis Risk of leak
Right colon Right hemicolectomy Ileo-colic Low <5%
Transverse Extended right hemicolectomy Ileo-colic Low <5%
Splenic flexure Extended right hemicolectomy Ileo-colic Low <5%
Splenic flexure Left hemicolectomy Colo-colon 2-5%
Left colon Left hemicolectomy Colo-colon 2-5%
Sigmoid colon High anterior resection Colo-rectal 5%
Upper rectum Anterior resection (TME) Colo-rectal 5%
Low rectum Anterior resection (Low TME) Colo-rectal
(+/- Defunctioning stoma) 10%
Anal verge Abdomino-perineal excision of colon and rectum None n/a

In the emergency setting, where the bowel has perforated, the risk of an anastomotic breakdown is much greater, particularly when the anastomosis is colon-colon. In this situation, an end colostomy is often safer and can be reversed later. When resection of the sigmoid colon is performed and an end colostomy is fashioned the operation is referred to as a Hartmans procedure. Whilst left sided resections are more risky, ileo-colic anastomoses are relatively safe even in the emergency setting and do not need to be defunctioned.

References
A review of the diagnosis and management of colorectal cancer and a summary of the UK National Institute of Clinical Excellence guidelines is provided in:
NICE Guideline NG151. Last updated in 2020.

1940
Q

A 19 year old man has a skin lesion excised from his back. He is reviewed clinically at 4 months post procedure and the surgeon notes that the scar has begun to contract. Which of the following facilitates this process?

Myofibroblasts

Neutrophils

Granuloma formation

Macrophages

Fibroblasts

A

As wounds mature the fibroblast population differentiates into myofibroblasts (usually 6 weeks and beyond), these have a contractile phenotype and therefore help in contracting the wound. Immature fibroblasts, though able to adhere to the ECM, do not have this ability.

Wound healing

Surgical wounds are either incisional or excisional and either clean, clean contaminated or dirty. Although the stages of wound healing are broadly similar their contributions will vary according to the wound type.

The main stages of wound healing include:

Haemostasis
Minutes to hours following injury
Vasospasm in adjacent vessels, platelet plug formation and generation of fibrin rich clot.

Inflammation
Typically days 1-5
Neutrophils migrate into wound (function impaired in diabetes).
Growth factors released, including basic fibroblast growth factor and vascular endothelial growth factor.
Fibroblasts replicate within the adjacent matrix and migrate into wound.
Macrophages and fibroblasts couple matrix regeneration and clot substitution.

Regeneration
Typically days 7 to 56
Platelet derived growth factor and transformation growth factors stimulate fibroblasts and epithelial cells.
Fibroblasts produce a collagen network.
Angiogenesis occurs and wound resembles granulation tissue.

Remodeling
From 6 weeks to 1 year
Longest phase of the healing process and may last up to one year (or longer).
During this phase fibroblasts become differentiated (myofibroblasts) and these facilitate wound contraction.
Collagen fibres are remodeled.
Microvessels regress leaving a pale scar.

The above description represents an idealised scenario. A number of diseases may distort this process. Neovascularisation is an important early process. Endothelial cells may proliferate in the wound bed and recanalise to form a vessel. Vascular disease, shock and sepsis can all compromise microvascular flow and impair healing.

Conditions such as jaundice will impair fibroblast synthetic function and immunity with a detrimental effect in most parts of the healing process.

Problems with scars:

Hypertrophic scars
Excessive amounts of collagen within a scar. Nodules may be present histologically containing randomly arranged fibrils within and parallel fibres on the surface. The tissue itself is confined to the extent of the wound itself and is usually the result of a full thickness dermal injury. They may go on to develop contractures.

Image of hypertrophic scarring. Note that it remains confined to the boundaries of the original wound:

1941
Q

A 23 year old man is undergoing a hernia repair and the mesh is to be sutured to the inguinal ligament. From which of the following does the inguinal ligament arise?

Transversus abdominis fascia

Internal oblique

Rectus sheath

Rectus abdominis muscle

External oblique aponeurosis

A

The inguinal ligament is formed by the external oblique aponeurosis. It runs from the pubic tubercle to the anterior superior iliac spine.

Abdominal wall

The 2 main muscles of the abdominal wall are the rectus abdominis (anterior) and the quadratus lumborum (posterior).
The remaining abdominal wall consists of 3 muscular layers. Each muscle passes from the lateral aspect of the quadratus lumborum posteriorly to the lateral margin of the rectus sheath anteriorly. Each layer is muscular posterolaterally and aponeurotic anteriorly.

Muscles of abdominal wall
External oblique
Lies most superficially
Originates from 5th to 12th ribs
Inserts into the anterior half of the outer aspect of the iliac crest, linea alba and pubic tubercle
More medially and superiorly to the arcuate line, the aponeurotic layer overlaps the rectus abdominis muscle
The lower border forms the inguinal ligament
The triangular expansion of the medial end of the inguinal ligament is the lacunar ligament.
Internal oblique
Arises from the thoracolumbar fascia, the anterior 2/3 of the iliac crest and the lateral 2/3 of the inguinal ligament
The muscle sweeps upwards to insert into the cartilages of the lower 3 ribs
The lower fibres form an aponeurosis that runs from the tenth costal cartilage to the body of the pubis
At its lowermost aspect it joins the fibres of the aponeurosis of transversus abdominis to form the conjoint tendon.
Transversus abdominis
Innermost muscle
Arises from the inner aspect of the costal cartilages of the lower 6 ribs , from the anterior 2/3 of the iliac crest and lateral 1/3 of the inguinal ligament
Its fibres run horizontally around the abdominal wall ending in an aponeurosis. The upper part runs posterior to the rectus abdominis. Lower down the fibres run anteriorly only.
The rectus abdominis lies medially; running from the pubic crest and symphysis to insert into the xiphoid process and 5th, 6th and 7th costal cartilages. The muscles lies in a aponeurosis as described above.
Nerve supply: anterior primary rami of T7-12

Surgical notes
During abdominal surgery it is usually necessary to divide either the muscles or their aponeuroses. During a midline laparotomy it is desirable to divide the aponeurosis. This will leave the rectus sheath intact above the arcuate line and the muscles intact below it. Straying off the midline will often lead to damage to the rectus muscles, particularly below the arcuate line where they may often be in close proximity to each other.

1942
Q

At which of the following anatomical sites does dormant tuberculosis most frequently reactivate?

Apex of the lung

Base of the lung

Brain

Terminal ileum

Lumbar spine

A

TB reactivation most commonly occurs at the lung apex. This site is better oxygenated than elsewhere allowing the mycobacteria to multiply more rapidly and then spread both locally and distantly.

Tuberculosis pathology

Is a form of primary chronic inflammation, caused by the inability of macrophages to kill the Mycobacterium tuberculosis.
The macrophages often migrate to regional lymph nodes, the lung lesion plus affected lymph nodes is referred to as a Ghon complex.
This leads to the formation of a granuloma which is a collection of epithelioid histiocytes.
There is the presence of caseous necrosis in the centre.
The inflammatory response is mediated by a type 4 hypersensitivity reaction.
In healthy individuals the disease may be contained, in the immunocompromised disseminated (miliary TB) may occur.

Diagnosis
Waxy membrane of mycobacteria prevents binding with normal stains. Ziehl - Neelsen staining is typically used.
Culture based methods take far longer.

Image showing acid- alcohol fast mycobacteria stained using the Ziehl- Neelsen method

1943
Q

A 73 year old lady presents with progressive dysphagia and is diagnosed with oesophageal cancer and liver metastases, it is located 8cm proximal to the gastro-oesophageal junction. Which of the following treatment options would be the most appropriate management?

Insertion of Celestin tube

Insertion of Minnesota tube

Insertion of self expanding metal stent

Photodynamic therapy

Trans hiatal oesphagectomy

A

Most cases of malignant oesophageal obstruction can be managed by the placement of self expanding metal stents. The Celestin tube requires a laparotomy and is therefore obsolete. A resectional procedure would be inappropriate in the presence of liver metastasis. The main contra indication to metallic stent placement are very proximal tumours as it can be difficult to get proximal control in this situation and chemotherapy may be more appropriate.

Treatment of oesophageal cancer

  • In general resections are not offered to those patients with distant metastasis, and usually not to those with N2 disease.
    Local nodal involvement is not in itself a contra indication to resection.
    Surgical resection is the mainstay of treatment.
    Neoadjuvent chemotherapy is given in most cases prior to surgery.
    In situ disease may be managed by endoscopic mucosal resection, although this is still debated.
    In patients with lower third lesions an Ivor - Lewis type procedure is most commonly performed. Very distal tumours may be suitable to a transhiatal procedure. Which is an attractive option as the penetration of two visceral cavities required for an Ivor- Lewis type procedure increases the morbidity considerably.
    More proximal lesions will require a total oesphagectomy (Mckeown type) with anastomosis to the cervical oesophagus.
    Patients with unresectable disease may derive benefit from local ablative procedures, palliative chemotherapy or stent insertion.

Operative details of Ivor- Lewis procedure
Combined laparotomy and right thoracotomy

Indication
Lower and middle third oesophageal tumours

Preparation
Staging with a combination of CT chest abdomen and pelvis- if no metastatic disease detected then patients will undergo a staging laparoscopy to detect peritoneal disease.
If both these modalities are negative then patients will finally undergo a PET CT scan to detect occult metastatic disease. Only in those whom no evidence of advanced disease is detected will proceed to resection.
Patients receive a GA, double lumen endotracheal tube to allow for lung deflation, CVP and arterial monitoring.

Procedure
A rooftop incision is made to access the stomach and duodenum.

Laparotomy To mobilize the stomach
The greater omentum is incised away from its attachment to the right gastroepiploic vessels along the greater curvature of the stomach.
Then the short gastric vessels are ligated and detached from the greater curvature from the spleen.
The lesser omentum is incised, preserving the right gastric artery.
The retroperitoneal attachments of the duodenum in its second and third portions are incised, allowing the pylorus to reach the oesophageal hiatus. Some surgeons perform a pyloroplasty at this point to facilitate gastric emptying.
The left gastric vessels are then ligated, avoiding any injury to the common hepatic or splenic arteries. Care must be taken to avoid inadvertently devascularising the liver owing to variations in anatomy.

Right Thoracotomy Oesophageal resection and oesophagogastric anastomosis
Through 5th intercostal space
Dissection performed 10cm above the tumour
This may involve transection of the azygos vein.
The oesophagus is then removed with the stomach creating a gastric tube.
An anastomosis is created.

The chest is closed with underwater seal drainage and tube drains to the abdominal cavity.

Post operatively
Patients will typically recover in ITU initially.
A nasogastric tube will have been inserted intraoperatively and must remain in place during the early phases of recovery.
Post operatively these patients are at relatively high risk of developing complications:

  • Atelectasis- due to the effects of thoracotomy and lung collapse
  • Anastomotic leakage. The risk is relatively high owing to the presence of a relatively devascularised stomach. Often the only blood supply is from the gastroepiploic artery as all others will have been divided. If a leak does occur then many will attempt to manage conservatively with prolonged nasogastric tube drainage and TPN. The reality is that up to 50% of patients developing an anastomotic leak will not survive to discharge.
  • Delayed gastric emptying (may be avoided by performing a pyloroplasty).
1944
Q

Which of the following is not directly affected by warfarin?

Protein C

Factor II

Factor VII

Factor IX

Factor VIII

A

Warfarin affects synthesis of factors II, VII, IX, X and protein C.

Warfarin

Warfarin is an oral anticoagulant which inhibits the reduction of vitamin K to its active hydroquinone form, which in turn acts as a cofactor in the formation of clotting factor II, VII, IX and X (mnemonic = 1972) and protein C

Factors that may potentiate warfarin
Liver disease
P450 enzyme inhibitors, e.g.: amiodarone, ciprofloxacin
Cranberry juice
Drugs which displace warfarin from plasma albumin, e.g. NSAIDs
Inhibit platelet function: NSAIDs

Side-effects
Haemorrhage
Teratogenic
Skin necrosis: when warfarin is first started biosynthesis of protein C is reduced. This results in a temporary procoagulant state after initially starting warfarin, normally avoided by concurrent heparin administration. Thrombosis may occur in venules leading to skin necrosis.

1945
Q

A 34 year old woman presents with recurrent peptic ulceration. She is on proton pump inhibitors and previously received Helicobacter pylori eradication therapy three months ago. Which of the following is likely to be raised on venous blood testing?

Secretin

Cholecystokinin

Gastrin

Amylase

Histamine

A

It is likely that this patient has an MEN I type gastrinoma (female, young age). As such, the serum gastrin levels are likely to be elevated.

Zollinger - Ellison syndrome

  • Gastrin-secreting tumor (gastrinoma) of pancreas or duodenum. Acid hypersecretion causes recurrent ulcers in duodenum and jejunum. Presents with abdominal pain (peptic ulcer disease, distal ulcers), diarrhoea (malabsorption)
    75% are sporadic and the 25% associated with MEN I
    Mean age of presentation is 50 years (earlier in MEN I)
    60% are malignant
    They are typically found in the gastrinoma triangle. This is formed by the junction of cystic duct and CBD, junction of D2 and D3, junction of neck and body of pancreas.
    Imaging is with triple phase CT and endoscopic USS
    Serum sampling of serum gastrin levels will show these to be elevated. A secretin stimulation test is sometimes performed as gastrin levels will remain elevated following administration of secretin which usually suppresses gastrin.
    Treatment is with resection.
1946
Q

In relation to patients with type 1 diabetes mellitus undergoing surgery, which of the following statements is untrue?

They should not receive oral carbohydrate loading drinks as part of enhanced recovery programmes

When a variable rate insulin infusion is required 0.45% sodium chloride and 5% dextrose with either 0.15% or 0.3% potassium are the fluids of choice

Hourly intraoperative blood glucose measurements are required

Insulin infusions are only required in patients who will miss more than two meals or who are nil by mouth for greater than 12 hours

Blood glucose levels persistently greater than 12 should initiate a change in therapy

A

Insulin should not be stopped in patients with type 1 diabetes and omission of more than one meal will usually require a variable rate insulin infusion
Type 1 diabetics who take insulin should have this continued through the perioperative period.
Fluid guidelines in diabetics differ and are not well covered in NPSA fluid guidelines.

Preparation for surgery

Elective and emergency patients require different preparation.

Elective cases
Consider pre admission clinic to address medical issues.
Blood tests including FBC, U+E, LFT’s, Clotting, Group and Save
Urine analysis
Pregnancy test
Sickle cell test
ECG/ Chest x-ray

Exact tests to be performed will depend upon the proposed procedure and patient fitness.

Risk factors for development of deep vein thrombosis should be assessed and a plan for thromboprophylaxis formulated.

Diabetes
Diabetic patients have greater risk of complications.
Poorly controlled diabetes carries high risk of wound infections.
Patients with diet or tablet controlled diabetes may be managed using a policy of omitting medication and checking blood glucose levels regularly. Diabetics who are poorly controlled or who take insulin may require a intravenous sliding scale. Potassium supplementation should also be given.
Diabetic cases should be operated on first.

Emergency cases
Stabilise and resuscitate where needed.
Consider whether antibiotics are needed and when and how they should be administered.
Inform blood bank if major procedures planned particularly where coagulopathies are present at the outset or anticipated (e.g. Ruptured AAA repair)
Don’t forget to consent and inform relatives.

Special preparation
Some procedures require special preparation:
Thyroid surgery; vocal cord check.
Parathyroid surgery; consider methylene blue to identify gland.
Sentinel node biopsy; radioactive marker/ patent blue dye.
Surgery involving the thoracic duct; consider administration of cream.
Pheochromocytoma surgery; will need alpha and beta blockade.
Surgery for carcinoid tumours; will need covering with octreotide.
Colorectal cases; bowel preparation (especially left sided surgery)
Thyrotoxicosis; lugols iodine/ medical therapy.

References
Management of adults with diabetes undergoing surgery and elective procedures. NHS Diabetes. April 2011.

1947
Q

Which of the following is not a diagnostic criteria for brain death?

No response to sound

No corneal reflex

Absent oculo-vestibular reflexes

No response to supraorbital pressure

No cough reflex to bronchial stimulation

A

Brain death

Criteria for brain stem death testing
Deep coma of known aetiology.
Reversible causes excluded
No sedation
Normal electrolytes

Testing for brain death
Fixed pupils which do not respond to sharp changes in the intensity of incident light
No corneal reflex
Absent oculo-vestibular reflexes - no eye movements following the slow injection of at least 50ml of ice-cold water into each ear in turn (the caloric test)
No response to supraorbital pressure
No cough reflex to bronchial stimulation or gagging response to pharyngeal stimulation
No observed respiratory effort in response to disconnection of the ventilator for long enough (typically 5 minutes) to ensure elevation of the arterial partial pressure of carbon dioxide to at least 6.0 kPa (6.5 kPa in patients with chronic carbon dioxide retention). Adequate oxygenation is ensured by pre-oxygenation and diffusion oxygenation during the disconnection (so the brain stem respiratory centre is not challenged by the ultimate, anoxic, drive stimulus)

The test should be undertaken by two appropriately experienced doctors on two separate occasions. Both should be experienced in performing brain stem death testing and have at least 5 years post graduate experience. One of them must be a consultant. Neither can be a member of the transplant team (if organ donation contemplated).

1948
Q

A 40 year old woman is being investigated for haematuria. She was living with her sister who has just died from a sub arachnoid haemorrhage. The haematuria is painless and she has mild renal impairment. What is the most likely cause?

Adenocarcinoma of the bladder

Bladder stones

Polycystic kidney disease

Renal cancer

Pyelonephritis

A

This is likely to be polycystic kidney disease as she has renal failure and family history of sub arachnoid haemorrhage.

Haematuria

Causes of haematuria

Trauma
Injury to renal tract
Renal trauma commonly due to blunt injury (others penetrating injuries)
Ureter trauma rare: iatrogenic
Bladder trauma: due to RTA or pelvic fractures
Infection
Remember TB
Malignancy
Renal cell carcinoma (remember paraneoplastic syndromes): painful or painless
Urothelial malignancies: 90% are transitional cell carcinoma, can occur anywhere along the urinary tract. Painless haematuria.
Squamous cell carcinoma and adenocarcinoma: rare bladder tumours
Prostate cancer
Penile cancers: SCC
Renal disease
Glomerulonephritis
Stones
Microscopic haematuria common
Structural abnormalities
Benign prostatic hyperplasia (BPH) causes haematuria due to hypervascularity of the prostate gland
Cystic renal lesions e.g. polycystic kidney disease
Vascular malformations
Renal vein thrombosis due to renal cell carcinoma
Coagulopathy
Causes bleeding of underlying lesions
Drugs
Cause tubular necrosis or interstitial nephritis: aminoglycosides, chemotherapy
Interstitial nephritis: penicillin, sulphonamides, and NSAIDs
Anticoagulants
Benign
Exercise
Gynaecological
Endometriosis: flank pain, dysuria, and haematuria that is cyclical
Iatrogenic
Catheterisation
Radiotherapy; cystitis, severe haemorrhage, bladder necrosis
Pseudohaematuria For example following consumption of beetroot

References
Http://bestpractice.bmj.com/best-practice/monograph/316/overview/aetiology.html

1949
Q

A 38 year old window cleaner falls from his ladder. He lands on his left arm and notices an obvious injury. An x-ray and clinical examination demonstrate that he has a fracture of the proximal ulna and associated radial dislocation. What eponymous name is used to describe this injury?

Galeazzi

Smith’s

Bennett’s

Pott’s

Monteggia’s

A

This constellation of injuries is referred to as a Monteggia’s fracture.

Eponymous fractures

Colles’ fracture (dinner fork deformity)
Fall onto extended outstretched hand
Classical Colles’ fractures have the following 3 features:

  1. Transverse fracture of the radius
  2. 1 inch proximal to the radio-carpal joint
  3. Dorsal displacement and angulation

Smith’s fracture (reverse Colles’ fracture)
Volar angulation of distal radius fragment (Garden spade deformity)
Caused by falling backwards onto the palm of an outstretched hand or falling with wrists flexed

Bennett’s fracture
Intra-articular fracture of the first carpometacarpal joint
Impact on flexed metacarpal, caused by fist fights
X-ray: triangular fragment at ulnar base of metacarpal

Monteggia’s fracture
Dislocation of the proximal radioulnar joint in association with an ulna fracture
Fall on outstretched hand with forced pronation
Needs prompt diagnosis to avoid disability

Galeazzi fracture
Radial shaft fracture with associated dislocation of the distal radioulnar joint
Direct blow

Pott’s fracture
Bimalleolar ankle fracture
Forced foot eversion

Barton’s fracture
Distal radius fracture (Colles’/Smith’s) with associated radiocarpal dislocation
Fall onto extended and pronated wrist
Involvement of the joint is a defining feature

Holstein Lewis Fracture
A HolsteinLewis fracture is a fracture of the distal third of the humerus resulting in entrapment of the radial nerve.
The radial nerve is one of the major nerves of the upper limb. It innervates all of the muscles in the extensor compartments of the arm.
Conservative treatment includes reduction and use of a functional brace
Vascular injury may require open surgery

1950
Q

Which of the following aortic branches leaves the aorta approximately 1cm below the coeliac axis?

Renal artery

Inferior mesenteric artery

Superior mesenteric artery

Lumbar artery

Gonadal artery

A

The SMA leaves the aorta approximately 1cm below the coeliac axis. This is usually a level of L1. It is crossed anteriorly by the splenic vein and the body of the pancreas. It runs downwards and forwards anterior to the uncinate process.
Abdominal aortic branches

Branches Level Paired Type
Inferior phrenic T12 (Upper border) Yes Parietal
Coeliac T12 No Visceral
Superior mesenteric L1 No Visceral
Middle suprarenal L1 Yes Visceral
Renal L1-L2 Yes Visceral
Gonadal L2 Yes Visceral
Lumbar L1-L4 Yes Parietal
Inferior mesenteric L3 No Visceral
Median sacral L4 No Parietal
Common iliac L4 Yes Terminal

1951
Q

Which one of the following cells secretes the majority of tumour necrosis factor in humans?

Neutrophils

Macrophages

Natural killer cells

Killer-T cells

Helper-T cells

A

Tumour necrosis factor

Tumour necrosis factor (TNF) is a pro-inflammatory cytokine with multiple roles in the immune system

TNF is secreted mainly by macrophages and has a number of effects on the immune system, acting mainly in a paracrine fashion:
activates macrophages and neutrophils
acts as costimulator for T cell activation
key mediator of body’s response to Gram negative septicaemia
similar properties to IL-1
anti-tumour effect (e.g. phospholipase activation)

TNF-alpha binds to both the p55 and p75 receptor. These receptors can induce apoptosis. It also cause activation of NFkB

Endothelial effects include increase expression of selectins and increased production of platelet activating factor, IL-1 and prostaglandins

TNF promotes the proliferation of fibroblasts and their production of protease and collagenase. It is thought fragments of receptors act as binding points in serum

Systemic effects include pyrexia, increased acute phase proteins and disordered metabolism leading to cachexia

TNF is important in the pathogenesis of rheumatoid arthritis - TNF blockers (e.g. infliximab, etanercept) are now licensed for treatment of severe rheumatoid

1952
Q

A 28 year old man is involved in a house fire and sustains severe burns to his torso. Whilst on the burns unit, he is noted to develop significant generalised oedema. What is the most likely explanation for this event?

Raised pressure of fluid within the capillary bed

Decreased interstitial fluid pressure

Increased plasma oncotic pressure

Increased interstitial fluid pressure

Reduction in capillary oncotic pressure

A

The significant burns will result in generalised loss of high molecular weight proteins thereby reducing the capillary oncotic pressure. As a result, the fluid is likely to remain in the interstitium and thereby cause oedema.

Starlings forces

About one sixth of the body consists of spaces between cells. The fluid within these spaces is termed interstitial fluid. This fluid is formed by filtration and diffusion from capillaries. It contains almost the same constituents as plasma with the exception of proteins as these are not filtered due to their high relative molecular mass. The pressure in the capillaries tends to force fluid and its dissolved substances through capillary pores and into this space. In contrast the osmotic pressure exerted by plasma proteins (colloid osmotic pressure) tends to favor movement from the interstitial spaces back into the vascular compartments (capillaries) themselves. The lymphatic system returns back to the circulation the small amounts of protein that do leak into the interstitium.
There are four primary forces that determine fluid movement through a capillary membrane (Starlings forces):
Capillary pressure - forces fluid out of the capillary
Interstitial fluid pressure- which tends to force fluid inwards through the capillary membrane (when it is positive)
Plasma colloid osmotic pressure- favors influx into the capillary
Interstitial fluid osmotic pressure- favors efflux from the capillary into the interstitium

1953
Q

A 28 year old lady requires an episiotomy for a forceps vaginal delivery. Which of the nerves listed below will usually be anaesthetised to allow the episiotomy?

Femoral

Ilioinguinal

Pudendal

Genitofemoral

Sacral plexus

A

The pudendal nerve innervates the posterior vulval area and is routinely blocked in procedures such as episiotomy.

Pudendal nerve

The pudendal nerve arises from nerve roots S2, S3 and S4 and exits the pelvis through the greater sciatic foramen. It re-enters the perineum through the lesser sciatic foramen. It travels inferior to give innervation to the anal sphincters and external urethral sphincter. It also provides cutaneous innervation to the region of perineum surrounding the anus and posterior vulva.

Traction and compression of the pudendal nerve by the foetus in late pregnancy may result in late onset pudendal neuropathy which may be part of the process involved in the development of faecal incontinence.

1954
Q

A 12 day old infant is brought to the emergency department by his anxious mother who notices that he has developed a right sided groin swelling. On examination the testes are correctly located but it is evident that the child has a right sided inguinal hernia. It is soft and easily reduced. What is the most appropriate management?

Surgery over the next few days

Reassure and discharge

Surgery at 1 year of age

Surgery once the child is 6 months old

Application of a hernia truss

A

Inguinal hernia in infants = Urgent surgery
The high incidence of strangulation necessitates an urgent herniotomy be performed. In infants with a reducible hernia this can be performed on a daycase list during the same week. Deferring surgery on the basis of age is not justified.

Paediatric inguinal hernia

Inguinal hernias are a common disorder in children. They are commoner in males as the testis migrates from its location on the posterior abdominal wall, down through the inguinal canal. A patent processus vaginalis may persist and be the site of subsequent hernia development.
Children presenting in the first few months of life are at the highest risk of strangulation and the hernia should be repaired urgently. Children over 1 year of age are at lower risk and surgery may be performed electively. For paediatric hernias a herniotomy without implantation of mesh is sufficient. Most cases are performed as day cases, neonates and premature infants are kept in hospital overnight as there is a recognised increased risk of post operative apnoea.

1955
Q

Which of the following drugs does not cause pseudohaematuria?

Rifampicin

Quinine

Noradrenaline

Levodopa

Phenytoin

A

Rifampicin, phenytoin, levodopa, methyldopa, and quinine all cause pseudohaematuria.

Haematuria

Causes of haematuria

Trauma
Injury to renal tract
Renal trauma commonly due to blunt injury (others penetrating injuries)
Ureter trauma rare: iatrogenic
Bladder trauma: due to RTA or pelvic fractures
Infection
Remember TB
Malignancy
Renal cell carcinoma (remember paraneoplastic syndromes): painful or painless
Urothelial malignancies: 90% are transitional cell carcinoma, can occur anywhere along the urinary tract. Painless haematuria.
Squamous cell carcinoma and adenocarcinoma: rare bladder tumours
Prostate cancer
Penile cancers: SCC
Renal disease
Glomerulonephritis
Stones
Microscopic haematuria common
Structural abnormalities
Benign prostatic hyperplasia (BPH) causes haematuria due to hypervascularity of the prostate gland
Cystic renal lesions e.g. polycystic kidney disease
Vascular malformations
Renal vein thrombosis due to renal cell carcinoma
Coagulopathy
Causes bleeding of underlying lesions
Drugs
Cause tubular necrosis or interstitial nephritis: aminoglycosides, chemotherapy
Interstitial nephritis: penicillin, sulphonamides, and NSAIDs
Anticoagulants
Benign
Exercise
Gynaecological
Endometriosis: flank pain, dysuria, and haematuria that is cyclical
Iatrogenic
Catheterisation
Radiotherapy; cystitis, severe haemorrhage, bladder necrosis
Pseudohaematuria For example following consumption of beetroot

References
Http://bestpractice.bmj.com/best-practice/monograph/316/overview/aetiology.html

1956
Q

As part of a research project you are trying to ascertain whether the use of dummies in infants is linked to sudden infant death syndrome. What is the most appropriate form of study design?

Randomised controlled trial

Cross-over trial

Cross-sectional survey

Case-control study

Cohort study

A

As sudden infant death syndrome is relatively rare a case-control design is more appropriate than a cohort study.

Study design

The following table highlights the main features of the main types of study:

Randomised controlled trial Participants randomly allocated to intervention or control group (e.g. standard treatment or placebo)

Practical or ethical problems may limit use
Cohort study Observational and prospective. Two (or more) are selected according to their exposure to a particular agent (e.g. medicine, toxin) and followed up to see how many develop a disease or other outcome.

The usual outcome measure is the relative risk.

Examples include Framingham Heart Study
Case-control study Observational and retrospective. Patients with a particular condition (cases) are identified and matched with controls. Data is then collected on past exposure to a possible causal agent for the condition.

The usual outcome measure is the odds ratio.

Inexpensive, produce quick results
Useful for studying rare conditions
Prone to confounding
Cross-sectional survey Provide a ‘snapshot’, sometimes called prevalence studies

Provide weak evidence of cause and effect

1957
Q

A 73 year old lifelong heavy smoker presents to the vascular clinic with symptoms of foot ulceration and rest pain. On examination, her foot has areas of gangrene and pulses are impalpable. Which of the ABPI measurements shown below is most compatible with this process?

> 1.2

1.0

0.3

0.8

0.5

A

This is critical limb ischaemia. Values of 0.3 are typical in this setting and urgent further imaging is needed. Debridement of necrosis prior to improving arterial inflow carries a high risk of limb loss.

Ankle-Brachial pressure index

Measurement of ankle- brachial pressure index (ABPI) is a commonly performed vascular investigation.
Calculated by dividing lower limb pressure by the highest upper limb pressure.

Results of ABPI
1.2 or greater Usually due to vessel calcification
1.0- 1.2 Normal
0.8-1.0 Minor stenotic lesion
Initiate risk factor management
0.50-0.8 Moderate stenotic lesion
Consider duplex
Risk factor management
If mixed ulcers present then avoid tight compression bandages
0.3 - 0.5 Likely significant stenosis
Duplex scanning to delineate lesions needed
Compression bandaging contra indicated
Less than 0.3 Indicative of critical ischaemia
Urgent detailed imaging required

1958
Q

A 45 year old man presents with a lipoma located posterior to the posterior border of the sternocleidomastoid muscle, approximately 4cm superior to the middle third of the clavicle. During surgical excision of the lesion troublesome bleeding is encountered. Which of the following is the most likely source?

Internal jugular vein

External jugular vein

Common carotid artery

Vertebral artery

Second part of the subclavian artery

A

The external jugular vein runs obliquely in the superficial fascia of the posterior triangle. It drains into the subclavian vein. During surgical exploration of this area the external jugular vein may be injured and troublesome bleeding may result. The internal jugular vein and carotid arteries are located in the anterior triangle. The third, and not the second, part of the subclavian artery is also a content of the posterior triangle

Posterior triangle of the neck

Boundaries
Apex Sternocleidomastoid and the Trapezius muscles at the Occipital bone
Anterior Posterior border of the Sternocleidomastoid
Posterior Anterior border of the Trapezius
Base Middle third of the clavicle

Contents
Nerves
Accessory nerve
Phrenic nerve
Three trunks of the brachial plexus
Branches of the cervical plexus: Supraclavicular nerve, transverse cervical nerve, great auricular nerve, lesser occipital nerve
Vessels
External jugular vein
Subclavian artery
Muscles
Inferior belly of omohyoid
Scalene
Lymph nodes
Supraclavicular
Occipital

1959
Q

A 59 year old man presents with recurrent episodes of urinary sepsis. In his history he mentions that he has suffered from recurrent attacks of left iliac fossa pain over the past few months. He has also notices bubbles in his urine. He undergoes a CT scan which shows a large inflammatory mass in the left iliac fossa. No other abnormality is detected. The most likely diagnosis is:

Ulcerative colitis

Crohns disease

Mesenteric ischaemia

Diverticular disease

Rectal cancer

A

Diverticular disease is one of the commonest causes of colovesical fistula
Recurrent attacks of diverticulitis may cause the development of local abscesses which may erode into the bladder resulting in urinary sepsis and pneumaturia. This would be an unusual presentation from Crohns disease and rectal cancer would be more distally sited and generally evidence of extra colonic disease would be present if the case were malignant and this advanced.

Diverticular disease

Diverticular disease is a common surgical problem. It consists of herniation of colonic mucosa through the muscular wall of the colon. The usual site is between the taenia coli where vessels pierce the muscle to supply the mucosa. For this reason, the rectum, which lacks taenia, is often spared.

Symptoms
Altered bowel habit
Bleeding
Abdominal pain

Complications
Diverticulitis
Haemorrhage
Development of fistula
Perforation and faecal peritonitis
Perforation and development of abscess
Development of diverticular phlegmon

Diagnosis
Patients presenting in clinic will typically undergo either a colonoscopy, CT cologram or barium enema as part of their diagnostic work up. All tests can identify diverticular disease. It can be far more difficult to confidently exclude cancer, particularly in diverticular strictures.

Acutely unwell surgical patients should be investigated in a systematic way. Plain abdominal films and an erect chest x-ray will identify perforation. An abdominal CT scan (not a CT cologram) with oral and intravenous contrast will help to identify whether acute inflammation is present but also the presence of local complications such as abscess formation.

Severity Classification- Hinchey
I Para-colonic abscess
II Pelvic abscess
III Purulent peritonitis
IV Faecal peritonitis

Treatment
Increase dietary fibre intake.
Mild attacks of diverticulitis may be managed conservatively with antibiotics.
Peri colonic abscesses should be drained either surgically or radiologically.
Recurrent episodes of acute diverticulitis requiring hospitalisation are a relative indication for a segmental resection.
Hinchey IV perforations (generalised faecal peritonitis) will require a resection and usually a stoma. This group have a very high risk of post operative complications and usually require HDU admission.

1960
Q

A 1 month old baby girl presents with bile stained vomiting. She had an exomphalos and a congenital diaphragmatic hernia. What is the most likely underlying explanation?

Duodenal atresia

Intestinal malrotation

Duplication cysts

Annular pancreas

Ileal atresia

A

Exomphalos and diaphragmatic herniae are commonly associated with malrotation.

Intestinal malrotation in neonates

Normal embryology
During the fourth week of embryogenesis the intestine moves into the abdomen via the base of the umbilicus. Through a combination of foetal growth and bowel maturation the bowel resides in the abdominal cavity and undergoes a 270o counterclockwise twist. At the conclusion of this process the ligament of Treitz lies to the left of the spine and the caecum in the right lower quadrant. Malrotation occurs when the rotational process described is incomplete. Typically the duodenal loop lies to the left of the caecum and therefore lacks 90 o of its 270o rotation. It becomes fixed in this position with peritoneal attachments (Ladds bands).

Symptoms
Bilious vomiting is the cardinal symptom and sign. In most cases there are no antecedent symptoms. If untreated then gut perfusion may be impaired with resultant development of further symptoms.

Diagnosis
The main problem is that the infant develops mid gut volvulus. In this situation the bowel undergoes a 720o twist, the bowel viability depends upon a narrow mesentery containing the superior mesenteric artery. To ascertain whether this has occurred the two main tests include an abdominal ultrasound scan to determine the relationship between the superior mesenteric artery and vein (normally SMA lies to the left of the SMV). This test is complemented with an upper GI contrast series and this aims to establish that the DJ flexure is correctly sited to the left of the vertebral bodies.

Treatment
Laparotomy and division of adhesional bands (Ladds procedure). The division of the congenital adhesions allows widening of the small bowel mesentery. The bowel is untwisted and assessed for viability. The bowel is returned to the abdominal cavity in the non rotated fashion with the small bowel on the right hand side and the large bowel on the left, the caecum is positioned in the left upper quadrant. Because the caecum is located in this new location many surgeons will also perform an appendicectomy due to the diagnostic difficulty posed in diagnosing appendicitis in this group of patients.

1961
Q

A 58 year old woman presented to the Emergency Department with a large fluctuant swelling the site of a recent insect bite. She is anxious, tachycardic, and pyrexial. An ECG shows atrial fibrillation. She is noted to have a goitre. The swelling at the site of the bite requires surgical drainage. Which of the following classes of drug would be most appropriate as part of her preoperative preparation for surgery?

Alpha 1 adrenoceptor agonist

Alpha 2 adrenoceptor agonist

Alpha adrenoceptor blocker

Beta adrenoceptor agonist

Beta adrenoceptor blocker

A

The issue here is an interplay between sepsis and thyroid disease. A degree of rate control is required to facilitate anaesthesia and this is best achieved with beta blockade in this particular setting.

Hyperthyroidism

Causes of hyperthyroidism include:
Diffuse toxic goitre (Graves Disease)
Toxic nodular goitre
Toxic nodule
Rare causes

Graves disease
Graves disease is characterised by a diffuse vascular goitre that appears at the same time as the clinical manifestations of hyperthyroidism. It is commonest in younger females and may be associated with eye signs. Thyrotoxic symptoms will predominate. Up to 50% of patients will have a familial history of autoimmune disorders. The glandular hypertrophy and hyperplasia occur as a result of the thyroid stimulating effects of the TSH receptor antibodies.

Toxic nodular goitre
In this disorder the goitre is present for a long period of time prior to the development of clinical symptoms. In most goitres the nodules are inactive and in some cases it is the internodular tissue that is responsible for the hyperthyroidism.

Toxic nodule
Overactive, autonomously functioning nodule. It may occur as part of generalised nodularity or be a true toxic adenoma. The TSH levels are usually low as the autonomously functioning thyroid tissue will exert a negative feedback effect.

Signs and symptoms
Symptoms Signs
Lethargy Tachycardia
Emotionally labile Agitation
Heat intolerance Hot, moist palms
Weight loss Exopthalmos
Excessive appetite Thyroid goitre and bruit
Palpitations Lid lag/retraction

Diagnosis
The most sensitive test for diagnosing hyperthyroidism is plasma T3 (which is raised). Note in hypothyroidism the plasma T4 and TSH are the most sensitive tests. A TSH level of <0.5U/L suggests hyperthyroidism. TSH receptor antibodies may be tested for in the diagnosis of Graves.

Treatment
First line treatment for Graves disease is usually medical and the block and replace regime is the favored option. Carbimazole is administered at higher doses and thyroxine is administered orally. Patients are maintained on this regime for between 6 and 12 months. Attempts are then made to wean off medication. Where relapse then occurs the options are between ongoing medical therapy, radioiodine or surgery.

1962
Q

Which of the following is not caused by cortisol in the stress response?

Anti-inflammatory effects

Hypoglycaemia

Skeletal muscle protein breakdown

Stimulation of lipolysis

Mineralocorticoid effects

A

An ‘anti insulin’ effect occurs leading to hyperglycaemia.

Stress response: Endocrine and metabolic changes

  • Surgery precipitates hormonal and metabolic changes causing the stress response.
    Stress response is associated with: substrate mobilization, muscle protein loss, sodium and water retention, suppression of anabolic hormone secretion, activation of the sympathetic nervous system, immunological and haematological changes.
    The hypothalamic-pituitary axis and the sympathetic nervous systems are activated and there is a failure of the normal feedback mechanisms of control of hormone secretion.

A summary of the hormonal changes associated with the stress response:

Increased Decreased No Change
Growth hormone Insulin Thyroid stimulating hormone
Cortisol Testosterone Luteinizing hormone
Renin Oestrogen Follicle stimulating hormone
Adrenocorticotrophic hormone (ACTH)
Aldosterone
Prolactin
Antidiuretic hormone
Glucagon

Sympathetic nervous system
Stimulates catecholamine release
Causes tachycardia and hypertension

Pituitary gland
ACTH and growth hormone (GH) is stimulated by hypothalamic releasing factors, corticotrophin releasing factor (CRF) and somatotrophin (or growth hormone releasing factor)
Perioperative increased prolactin secretion occurs by release of inhibitory control
Secretion of thyroid stimulating hormone (TSH), luteinizing hormone (LH) and follicle stimulating hormone (FSH) does not change significantly
ACTH stimulates cortisol production within a few minutes of the start of surgery. More ACTH is produced than needed to produce a maximum adrenocortical response.

Cortisol
Significant increases within 4-6 hours of surgery (>1000 nmol litre-1).
The usual negative feedback mechanism fails and concentrations of ACTH and cortisol remain persistently increased.
The magnitude and duration of the increase correlate with the severity of stress and the response is not abolished by the administration of corticosteroids.
The metabolic effects of cortisol are enhanced:

Skeletal muscle protein breakdown to provide gluconeogenic precursors and amino acids for protein synthesis in the liver
Stimulation of lipolysis
‘Anti-insulin effect’
Mineralocorticoid effects
Anti-inflammatory effects

Growth hormone
Increased secretion after surgery has a minor role
Most important for preventing muscle protein breakdown and promote tissue repair by insulin growth factors

Alpha Endorphin
Increased

Antidiuretic hormone
An important vasopressor and enhances haemostasis
Renin is released causing the conversion of angiotensinogen to angiotensin I
Angiotensin II formed by ACE on angiotensin 1, which causes the secretion of aldosterone from the adrenal cortex. This increases sodium reabsorption at the distal convoluted tubule

Insulin
Release inhibited by stress
Occurs via the inhibition of the beta cells in the pancreas by the α2-adrenergic inhibitory effects of catecholamines
Insulin resistance by target cells occurs later
The perioperative period is characterized by a state of functional insulin deficiency

Thyroxine (T4) and tri-iodothyronine (T3)
Circulating concentrations are inversely correlated with sympathetic activity and after surgery there is a reduction in thyroid hormone production, which normalises over a few days.

Metabolic effect of endocrine response

Carbohydrate metabolism
Hyperglycaemia is a main feature of the metabolic response to surgery
Due to increase in glucose production and a reduction in glucose utilization
Catecholamines and cortisol promote glycogenolysis and gluconeogenesis
Initial failure of insulin secretion followed by insulin resistance affects the normal responses
The proportion of the hyperglycaemic response reflects the severity of surgery
Hyperglycaemia impairs wound healing and increase infection rates

Protein metabolism
Initially there is inhibition of protein anabolism, followed later, if the stress response is severe, by enhanced catabolism
The amount of protein degradation is influenced by the type of surgery and also by the nutritional status of the patient
Mainly skeletal muscle protein is affected
The amino acids released form acute phase proteins (fibrinogen, C reactive protein, complement proteins, a2-macroglobulin, amyloid A and ceruloplasmin) and are used for gluconeogenesis
Nutritional support has little effect on preventing catabolism

Lipid metabolism
Increased catecholamine, cortisol and glucagon secretion, and insulin deficiency, promotes lipolysis and ketone body production.

Salt and water metabolism
ADH causes water retention, concentrated urine, and potassium loss and may continue for 3 to 5 days after surgery
Renin causes sodium and water retention

Cytokines
Glycoproteins
Interleukins (IL) 1 to 17, interferons, and tumour necrosis factor
Synthesized by activated macrophages, fibroblasts, endothelial and glial cells in response to tissue injury from surgery or trauma
IL-6 main cytokine associated with surgery. Peak 12 to 24 h after surgery and increase by the degree of tissue damage Other effects of cytokines include fever, granulocytosis, haemostasis, tissue damage limitation and promotion of healing.

Modifying the response
Opioids suppress hypothalamic and pituitary hormone secretion
At high doses the hormonal response to pelvic and abdominal surgery is abolished. However, such doses prolong recovery and increase the need for postoperative ventilatory support
Spinal anaesthesia can reduce the glucose, ACTH, cortisol, GH and epinephrine changes, although cytokine responses are unaltered
Cytokine release is reduced in less invasive surgery
Nutrition prevents the adverse effects of the stress response. Enteral feeding improves recovery
Growth hormone and anabolic steroids may improve outcome
Normothermia decreases the metabolic response

References
Vasiliki Manou-Stathopoulou et al. Redefining the perioperative stress response: a narrative review. Br J Anaesth. 2019 Nov;123(5):570-583.

Deborah Burton, Grainne Nicholson, and George Hall
Endocrine and metabolic response to surgery .

Contin Educ Anaesth Crit Care Pain (2004) 4(5): 144-147 doi:10.1093/bjaceaccp/mkh040

1963
Q

A 62 year old woman presents with acute bowel obstruction. She has been vomiting up to 15 times a day and is taking erythromycin. She suddenly complains of dizziness. Her ECG shows torsades de pointes. What is the management of choice?

IV Atropine

IV Potassium

IV Magnesium sulphate

IV Bicarbonate

IV Adrenaline

A

Torsades de pointes: Treatment IV magnesium sulphate
This woman is likely to have hypokalaemia and hypomagnasaemia as a result of vomiting. In addition to this, the erythromycin will predispose her to torsades de pointes. The patient needs Magnesium 2g over 10 minutes. Knowledge of the management of this peri arrest diagnosis is hence important in surgical practice.

Torsades de pointes

Torsades de pointes (‘twisting of the points’) is a rare arrhythmia associated with a long QT interval. It may deteriorate into ventricular fibrillation and hence lead to sudden death

Causes of long QT interval
congenital: Jervell-Lange-Nielsen syndrome, Romano-Ward syndrome
antiarrhythmics: amiodarone, sotalol, class 1a antiarrhythmic drugs
tricyclic antidepressants
antipsychotics
chloroquine
terfenadine
erythromycin
electrolyte: hypocalcaemia, hypokalaemia, hypomagnesaemia
myocarditis
hypothermia
subarachnoid haemorrhage

Management
IV magnesium sulphate

1964
Q

Where is a gomphoses type of fibrous joint typically found?

Teeth

Skull

Manubriosternum

Ribs

Femur

A

Types of joint

There are three main types of joint, fibrous, cartilaginous and synovial

Type of joint Features Example
Fibrous Comprise sutures, gomphoses and syndesmoses
Tough connective tissues unite the joints Sutures- skull
Gomphoses-Peg to socket e.g. teeth
Syndesmosis-tibiofibular joint
Cartilaginous Joints united by layer of hyaline cartilage Epiphyseal growth plates
Symphysis pubis
Synovial joints Bone ends permitted free movement
Bone ends covered by cartilage and surrounded by fluid

1965
Q

What is embryological origin of the pulmonary artery?

First pharyngeal arch

Second pharyngeal arch

Fourth pharyngeal arch

Fifth pharyngeal arch

Sixth pharyngeal arch

A

The proximal part of the sixth right pharyngeal arch gives origin to the right pulmonary artery. The distal part gives origin to the left pulmonary artery and the ductus arteriosus.

Pharyngeal arches

These develop during the fourth week of embryonic growth from a series of mesodermal outpouchings of the developing pharynx.
They develop and fuse in the ventral midline. Pharyngeal pouches form on the endodermal side between the arches.
There are 6 pharyngeal arches, the fifth does not contribute any useful structures and often fuses with the sixth arch.

Pharyngeal arches
Pharyngeal arch Muscular contributions Skeletal contributions Endocrine Artery Nerve
First Muscles of mastication
Anterior belly of digastric
Mylohyoid
Tensor tympanic
Tensor veli palatini Maxilla
Meckels cartilage
Incus
Malleus n/a Maxillary
External carotid Mandibular
Second Buccinator
Platysma
Muscles of facial expression
Stylohyoid
Posterior belly of digastric
Stapedius Stapes
Styloid process
Lesser horn and upper body of hyoid n/a Inferior branch of superior thyroid artery
Stapedial artery Facial
Third Stylopharyngeus Greater horn and lower part of hyoid Thymus
Inferior parathyroids Common and internal carotid Glossopharyngeal
Fourth Cricothyroid
All intrinsic muscles of the soft palate Thyroid and epiglottic cartilages Superior parathyroids Right- subclavian artery, Left-aortic arch Vagus
Sixth All intrinsic muscles of the larynx (except cricothyroid) Cricoid, arytenoid and corniculate cartilages n/a Right -Pulmonary artery, Left- Pulmonary artery and ductus arteriosus Vagus and recurrent laryngeal nerve

1966
Q

A 65 year old diabetic female presents with a painless ulcer at the medial malleolus, it has been present for the past 16 years. On examination, she has evidence of truncal varicosities and a brownish discolouration of the skin overlying the affected area. What is the most likely cause?

Chronic obliterative arterial disease

Superficial venous insufficiency

Vasculitis

Mixed ulcer

Neuropathic ulcer

A

Venous ulcers are usually associated with features of venous insufficiency. These include haemosiderin deposition and varicose veins. Neuropathic ulcers will tend to present at sites of pressure, which is not typically at the medial malleolus.
Lower leg ulcers

Venous leg ulcers
Most due to venous hypertension, secondary to chronic venous insufficiency (other causes include calf pump dysfunction or neuromuscular disorders)
Ulcers form due to capillary fibrin cuff or leucocyte sequestration
Features of venous insufficiency include oedema, brown pigmentation, lipodermatosclerosis, eczema
Location above the ankle, painless
Deep venous insufficiency is related to previous DVT and superficial venous insufficiency is associated with varicose veins
Doppler ultrasound looks for presence of reflux and duplex ultrasound looks at the anatomy/ flow of the vein
Management: 4 layer compression banding after exclusion of arterial disease or surgery
If fail to heal after 12 weeks or >10cm2 skin grafting may be needed

1967
Q

A 40 year old male is found to have multiple colonic polyps during a colonoscopy. He mentions that he has extra teeth. What is the underlying diagnosis?

Gardner’s syndrome

Lynch syndrome

FAP

Neurofibromatosis Type I

Neurofibromatosis Type II

A

Gardner’s syndrome is an AD disorder, characterised by: Colonic polyps, supernumerary teeth, jaw osteomas, congenital hypertrophy of retinal pigment. osteomas of the skull, thyroid cancer, epidermoid cysts, fibromas and sebaceous cysts.

Genetics and surgical disease

Some of the more commonly occurring genetic conditions occurring in surgical patients are presented here.

Li-Fraumeni Syndrome
Autosomal dominant
Consists of germline mutations to p53 tumour suppressor gene
High incidence of malignancies particularly sarcomas and leukaemias
Diagnosed when:

*Individual develops sarcoma under 45 years
*First degree relative diagnosed with any cancer below age 45 years and another family member develops malignancy under 45 years or sarcoma at any age

BRCA 1 and 2
Carried on chromosome 17 (BRCA 1) and Chromosome 13 (BRCA 2)
Linked to developing breast cancer (60%) risk.
Associated risk of developing ovarian cancer (55% with BRCA 1 and 25% with BRCA 2).

Lynch Syndrome
Autosomal dominant
Develop colonic cancer and endometrial cancer at young age
80% of affected individuals will get colonic and/ or endometrial cancer
High risk individuals may be identified using the Amsterdam criteria

Amsterdam criteria
Three or more family members with a confirmed diagnosis of colorectal cancer, one of whom is a first degree (parent, child, sibling) relative of the other two.
Two successive affected generations.
One or more colon cancers diagnosed under age 50 years.
Familial adenomatous polyposis (FAP) has been excluded.

Gardners syndrome
Autosomal dominant familial colorectal polyposis
Multiple colonic polyps
Extra colonic diseases include: skull osteoma, thyroid cancer and epidermoid cysts
Desmoid tumours are seen in 15%
Mutation of APC gene located on chromosome 5
Due to colonic polyps most patients will undergo colectomy to reduce risk of colorectal cancer
Now considered a variant of familial adenomatous polyposis coli

1968
Q

Which muscle initiates abduction of the shoulder?

Infraspinatus

Latissimus dorsi

Supraspinatus

Deltoid

Teres major

A

The intermediate portion of the deltoid muscle is the chief abductor of the humerus. However, it can only do this after the movement has been initiated by supraspinatus. Damage to the tendon of supraspinatus is a common form of rotator cuff disease.

Shoulder joint

Shallow synovial ball and socket type of joint.
It is an inherently unstable joint, but is capable to a wide range of movement.
Stability is provided by muscles of the rotator cuff that pass from the scapula to insert in the greater tuberosity (all except sub scapularis-lesser tuberosity).

Glenoid labrum
Fibrocartilaginous rim attached to the free edge of the glenoid cavity
Tendon of the long head of biceps arises from within the joint from the supraglenoid tubercle, and is fused at this point to the labrum.
The long head of triceps attaches to the infraglenoid tubercle

Fibrous capsule
Attaches to the scapula external to the glenoid labrum and to the labrum itself (postero-superiorly)
Attaches to the humerus at the level of the anatomical neck superiorly and the surgical neck inferiorly
Anteriorly the capsule is in contact with the tendon of subscapularis, superiorly with the supraspinatus tendon, and posteriorly with the tendons of infraspinatus and teres minor. All these blend with the capsule towards their insertion.
Two defects in the fibrous capsule; superiorly for the tendon of biceps. Anteriorly there is a defect beneath the subscapularis tendon.
The inferior extension of the capsule is closely related to the axillary nerve at the surgical neck and this nerve is at risk in anteroinferior dislocations. It also means that proximally sited osteomyelitis may progress to septic arthritis.

Movements and muscles
Flexion Anterior part of deltoid
Pectoralis major
Biceps
Coracobrachialis
Extension Posterior deltoid
Teres major
Latissimus dorsi
Adduction Pectoralis major
Latissimus dorsi
Teres major
Coracobrachialis
Abduction Mid deltoid
Supraspinatus
Medial rotation Subscapularis
Anterior deltoid
Teres major
Latissimus dorsi
Lateral rotation Posterior deltoid
Infraspinatus
Teres minor

Important anatomical relations
Anteriorly Brachial plexus
Axillary artery and vein
Posterior Suprascapular nerve
Suprascapular vessels
Inferior Axillary nerve
Circumflex humeral vessels

1969
Q

A 49 year old man presents with a single episode of haematuria. Investigations demonstrate adenocarcinoma of the prostate gland. Imaging shows T2 disease and no evidence of metastasis. What is the best treatment option?

Chemotherapy alone

Radical prostatectomy

Trasvesical prostatectomy

TURP

External beam radiotherapy

A

In a young patient with local disease only a radical prostatectomy is the best chance of cure. Radiotherapy may be given instead but has long term sequelae (and inferior survival outcomes). A transvesical prostatectomy is a largely historical operation performed for BPH before TURP was established.

Prostate Cancer

Prostate Cancer
This is a common condition and up to 30,000 men are diagnosed with the condition each year. Up to 9,000 will die in in the UK from the condition per year.

Diagnosis
Early prostate cancers have few symptoms.
Metastatic disease may present as bone pain.
Locally advanced disease may present as pelvic pain or with urinary symptoms.
Prostate specific antigen measurement
Digital rectal examination
Trans rectal USS (+/- biopsy)
MRI/ CT and bone scan for staging.

PSA Test
The normal upper limit for PSA is 4ng/ml. However, in this group will lie patients with benign disease and some with localised prostate cancer. False positives may be due to prostatitis, UTI, BPH, vigorous DRE.
The percentage of free: total PSA may help to distinguish benign disease from cancer. Values of <20% are suggestive of cancer and biopsy is advised.

Pathology
95% adenocarcinoma
In situ malignancy is sometimes found in areas adjacent to cancer. Multiple biopsies needed to call true in situ disease.
Often multifocal- 70% lie in the peripheral zone.
Graded using the Gleason grading system, two grades awarded 1 for most dominant grade (on scale of 1-5) and 2 for second most dominant grade (scale 1-5). The two added together give the Gleason score. Where 2 is best prognosis and 10 the worst.
Lymphatic spread occurs first to the obturator nodes and local extra prostatic spread to the seminal vesicles is associated with distant disease.

Treatment
Watch and wait- Elderly, multiple co-morbidities, low Gleason score
Radiotherapy (External)- Both potentially curative and palliative therapy possible. However, radiation proctitis and rectal malignancy are late problems. Brachytherapy is a modification allowing internal radiotherapy.
Surgery- Radical prostatectomy. Surgical removal of the prostate is the standard treatment for localised disease. The robot is being used increasingly for this procedure. As well as the prostate the obturator nodes are also removed to complement the staging process. Erectile dysfunction is a common side effect. Survival may be better than with radiotherapy (see references). Functional outcomes are better when a robotic approach is used.
Hormonal therapy- Testosterone stimulates prostate tissue and prostatic cancers usually show some degree of testosterone dependence. 95% of testosterone is derived from the testis and bilateral orchidectomy may be used for this reason. Pharmacological alternatives include LHRH analogues and anti androgens (which may be given in combination).
In the UK the National Institute for Clinical Excellence (NICE) suggests that active surveillance is the preferred option for low risk men. It is particularly suitable for men with clinical stage T1c, Gleason score 3+3 and PSA density < 0.15 ng/ml/ml who have cancer in less than 50% of their biopsy cores, with < 10 mm of any core involved.
Candidates for active surveillance should:
have had at least 10 biopsy cores taken
have at least one re-biopsy.
If men on active surveillance show evidence of disease progression, offer radical treatment. Treatment decisions should be made with the man, taking into account co-morbidities and life expectancy.

References
1. Prostate cancer pathway. NICE. NG131 last updated December 2021.
2. Sooriakumaran P et al. Comparative effectiveness of radical prostatectomy and radiotherapy in prostate cancer: observational study of mortality outcomes. BMJ 2014 (348):13. This study shows that in men with localised disease survival was greater in those offered surgery.

1970
Q

A 22 month old child is brought to the clinic by her mother who is concerned that she has developed a swelling in her neck. On examination; she has a soft lesion located in the posterior triangle that transilluminates. What is the most likely cause?

Lymphoma

Cystic hygroma

Rhabdomyosarcoma

Branchial cyst

Dermoid

A

Cystic hygromas are soft and transilluminate. Most are located in the posterior triangle.

Neck Masses in Children

Thyroglossal cyst
Located in the anterior triangle, usually in the midline and below the hyoid (65% cases)
Derived from remnants of the thyroglossal duct
Thin walled and anechoic on USS (echogenicity suggests infection of cyst)
Branchial cyst
Six branchial arches separated by branchial clefts
Incomplete obliteration of the branchial apparatus may result in cysts, sinuses or fistulae
75% of branchial cysts originate from the second branchial cleft
Usually located anterior to the sternocleidomastoid near the angle of the mandible
Unless infected the fluid of the cyst has a similar consistency to water and is anechoic on USS
Dermoids
Derived from pleuripotent stem cells and are located in the midline
Most commonly in a suprahyoid location
They have heterogeneous appearances on imaging and contain variable amounts of calcium and fat
Thyroid gland
True thyroid lesions are rare in children and usually represent thyroglossal cysts or tumours like lymphoma
Lymphatic malformations
Usually located posterior to the sternocleidomastoid
Cystic hygroma result from occlusion of lymphatic channels
The painless, fluid filled, lesions usually present prior to the age of 2
They are often closely linked to surrounding structures and surgical removal is difficult
They are typically hypoechoic on USS
Infantile haemangioma
May present in either triangle of the neck
Grow rapidly initially and then will often spontaneously regress
Plain x-rays will show a mass lesion, usually containing calcified phleboliths
As involution occurs the fat content of the lesions increases
Lymphadenopathy
Located in either triangle of the neck
May be reactive or neoplastic
Generalised lymphadenopathy usually secondary to infection in children (very common)

1971
Q

An obese 14 year old boy presents with difficulty running and mild knee and hip pain. There is no antecedent history of trauma. On examination, internal rotation is restricted but the knee is normal with full range of passive movement possible and no evidence of effusions. Both the C-reactive protein and white cell count are normal. What is the most likely cause?

Perthes disease

Slipped upper femoral epiphysis

Non accidental injury

Septic arthritis

Osteoarthritis

A

Slipped upper femoral epiphysis is the commonest adolescent hip disorder. It occurs most commonly in obese males. It may often present as knee pain which is usually referred from the ipsilateral hip. The knee itself is normal. The hip often limits internal rotation. The diagnosis is easily missed. X-rays will show displacement of the femoral epiphysis and the degree of its displacement may be calculated using the Southwick angle. Treatment is directed at preventing further slippage which may result in avascular necrosis of the femoral head.

Paediatric orthopaedics

Diagnosis Mode of presentation Treatment Radiology
Developmental dysplasia of the hip Usually diagnosed in infancy by screening tests. May be bilateral, when disease is unilateral there may be leg length inequality. As disease progresses child may limp and then early onset arthritis. More common in extended breech babies. Splints and harnesses or traction. In later years osteotomy and hip realignment procedures may be needed. In arthritis a joint replacement may be needed. However, this is best deferred if possible as it will almost certainly require revision Initially no obvious change on plain films and USS gives best resolution until 3 months of age. On plain films Shentons line should form a smooth arc
Perthes Disease Hip pain (may be referred to the knee) usually occurring between 5 and 12 years of age. Bilateral disease in 20%. Remove pressure from joint to allow normal development. Physiotherapy. Usually self-limiting if diagnosed and treated promptly. X-rays will show flattened femoral head. Eventually in untreated cases the femoral head will fragment.
Slipped upper femoral epiphysis Typically seen in obese male adolescents. Pain is often referred to the knee. Limitation to internal rotation is usually seen. Knee pain is usually present 2 months prior to hip slipping. Bilateral in 20%. Bed rest and non-weight bearing. Aim to avoid avascular necrosis. If severe slippage or risk of it occurring then percutaneous pinning of the hip may be required. X-rays will show the femoral head displaced and falling inferolaterally (like a melting ice cream cone) The Southwick angle gives indication of disease severity

1972
Q

A 24 year old man is due to undergo an excision of a sebaceous cyst of his scalp which of the agents below should be used for local anaesthesia?

1% lignocaine with 1 in 200,000 adrenaline

1% lignocaine alone

1% prilocaine

0.5% bupivicaine

0.25% bupivacaine with 1 in 200,000 adrenaline

A

Scalp wounds often bleed and the addition of adrenaline is therefore desirable. Lignocaine is fast acting and the preferred agent.

Local anaesthetic agents

Lidocaine
An amide
Local anaesthetic and a less commonly used antiarrhythmic (affects Na channels in the axon)
Hepatic metabolism, protein bound, renally excreted
Toxicity: due to IV or excess administration. Increased risk if liver dysfunction or low protein states. Note acidosis causes lidocaine to detach from protein binding.
Drug interactions: Beta blockers, ciprofloxacin, phenytoin
Features of toxicity: Initial CNS over activity then depression as lidocaine initially blocks inhibitory pathways then blocks both inhibitory and activating pathways. Cardiac arrhythmias.
Increased doses may be used when combined with adrenaline to limit systemic absorption.

Cocaine
Pure cocaine is a salt, usually cocaine hydrochloride. It is supplied for local anaesthetic purposes as a paste.
It is supplied for clinical use in concentrations of 4 and 10%. It may be applied topically to the nasal mucosa. It has a rapid onset of action and has the additional advantage of causing marked vasoconstriction.
It is lipophillic and will readily cross the blood brain barrier. Its systemic effects also include cardiac arrhythmias and tachycardia.
Apart from its limited use in ENT surgery it is otherwise used rarely in mainstream surgical practice.

Bupivacaine
Bupivacaine binds to the intracellular portion of sodium channels and blocks sodium influx into nerve cells, which prevents depolarization.
It has a much longer duration of action than lignocaine and this is of use in that it may be used for topical wound infiltration at the conclusion of surgical procedures with long duration analgesic effect.
It is cardiotoxic and is therefore contra indicated in regional blockage in case the tourniquet fails.
Levobupivacaine (Chirocaine) is less cardiotoxic and causes less vasodilation.

Prilocaine
Similar mechanism of action to other local anaesthetic agents. However, it is far less cardiotoxic and is therefore the agent of choice for intravenous regional anaesthesia e.g. Biers Block.

All local anaesthetic agents dissociate in tissues and this contributes to their therapeutic effect. The dissociation constant shifts in tissues that are acidic e.g. where an abscess is present, and this reduces the efficacy.

Doses of local anaesthetics
Agent Dose plain Dose with adrenaline
Lignocaine 3mg/Kg 7mg/Kg
Bupivacaine 2mg/Kg 2mg/Kg
Prilocaine 6mg/Kg 9mg/Kg
These are a guide only as actual doses depend on site of administration, tissue vascularity and co-morbidities.

Maximum total local anaesthetic doses
Lignocaine 1% plain - 3mg/ Kg - 200mg (20ml)
Lignocaine 1% with 1 in 200,000 adrenaline - 7mg/Kg - 500mg (50ml)
Bupivacaine 0.5% - 2mg/kg- 150mg (30ml)
Maximum doses are based on ideal body weight

Effects of adrenaline
Adrenaline may be added to local anaesthetic drugs. It prolongs the duration of action at the site of injection and permits usage of higher doses (see above). It is contra indicated in patients taking MAOI’s or tricyclic antidepressants. The toxicity of bupivacaine is related to protein binding and addition of adrenaline to this drug does not permit increases in the total dose of bupivacaine, in contrast to the situation with lignocaine.

References
An excellent review is provided by:
French J and Sharp L. Local Anaesthetics. Ann R Coll Surg Engl 2012; 94: 76-80.

1973
Q

A 38 year old man is noted to have a blood pressure of 175/110 on routine screening. On examination there are no physical abnormalities of note. CT scanning shows a left sided adrenal mass. Plasma metanephrines are elevated. What is the most likely cause?

Benign adrenal adenoma

Metastatic lesion

Phaeochromocytoma

Adrenal cyst

Conn’s syndrome

A

Hypertension in a young patient without any obvious cause should be investigated. Urinary VMA and plasma metanephrines are typically elevated.

Phaeochromocytoma and adrenal lesions

Phaeochromocytoma
Neuroendocrine tumour of the chromaffin cells of the adrenal medulla. Hypertension and hyperglycaemia are often found.
10% of cases are bilateral.
10% occur in children.
11% are malignant (higher when tumour is located outside the adrenal).
10% will not be hypertensive.

Familial cases are usually linked to the Multiple endocrine neoplasia syndromes (considered under its own heading).

Most tumours are unilateral (often right sided) and smaller than 10cm.

Diagnosis
Urine and plasma metanephrine levels.

CT and MRI scanning are both used to localise the lesion.

Treatment
Patients require medical therapy first. An irreversible alpha adrenoreceptor blocker should be given, although minority may prefer reversible blockade(1). Labetolol may be co-administered for cardiac chronotropic control. Isolated beta blockade should not be considered as it will lead to unopposed alpha activity.

These patients are often volume depleted and will often require moderate volumes of intra venous normal saline perioperatively.

Once medically optimised the phaeochromocytoma should be removed. Most adrenalectomies can now be performed using a laparoscopic approach(2). The adrenals are highly vascular structures and removal can be complicated by catastrophic haemorrhage in the hands of the inexperienced. This is particularly true of right sided resections where the IVC is perilously close. Should the IVC be damaged a laparotomy will be necessary and the defect enclosed within a Satinsky style vascular clamp and the defect closed with prolene sutures. Attempting to interfere with the IVC using any instruments other than vascular clamps will result in vessel trauma and make a bad situation much worse.

Incidental adrenal lesions
Adrenal lesions may be identified on CT scanning performed for other reasons(3). Factors suggesting benign disease on CT include(4):
Size less than 3cm
Homogeneous texture
Lipid rich tissue
Thin wall to lesion

All patients with incidental lesions should be managed jointly with an endocrinologist and full work up as described above. Patients with functioning lesions or those with adverse radiological features (Particularly size >3cm) should proceed to surgery.

References
1. Weingarten TN, Cata JP, O’Hara JF, Prybilla DJ, Pike TL, Thompson GB, et al. Comparison of two preoperative medical management strategies for laparoscopic resection of pheochromocytoma. Urology. 2010 Aug;76(2):508 e6-11.

  1. Nguyen PH, Keller JE, Novitsky YW, Heniford BT, Kercher KW. Laparoscopic approach to adrenalectomy: review of perioperative outcomes in a single center. Am Surg. 2011 May;77(5):592-6.
  2. Ng VW, Ma RC, So WY, Choi KC, Kong AP, Cockram CS, et al. Evaluation of functional and malignant adrenal incidentalomas. Arch Intern Med. 2010 Dec 13;170(22):2017-20.
  3. Muth A, Hammarstedt L, Hellstrom M, Sigurjonsdottir HA, Almqvist E, Wangberg B. Cohort study of patients with adrenal lesions discovered incidentally. Br J Surg. 2011 May 27.
1974
Q

Which cranial nerve supplies the motor fibres of styloglossus?

Facial

Trigeminal

Vagus

Hypoglossal

Glossopharyngeal

A

The hypoglossal nerve supplies motor innervation to all extrinsic and intrinsic muscles of the tongue. The only possible exception to this is palatoglossus, which is innervated by the vagus.

Cranial nerves

Cranial nerve lesions
Olfactory nerve May be injured in basal skull fractures or involved in frontal lobe tumour extension. Loss of olfactory nerve function in relation to major CNS pathology is seldom an isolated event and thus it is poor localiser of CNS pathology.
Optic nerve Problems with visual acuity may result from intra ocular disorders. Problems with the blood supply such as amaurosis fugax may produce temporary visual distortion. More important surgically is the pupillary response to light. The pupillary size may be altered in a number of disorders. Nerves involved in the resizing of the pupil connect to the pretectal nucleus of the high midbrain, bypassing the lateral geniculate nucleus and the primary visual cortex. From the pretectal nucleus neurones pass to the Edinger - Westphal nucleus, motor axons from here pass along with the oculomotor nerve. They synapse with ciliary ganglion neurones; the parasympathetic axons from this then innervate the iris and produce miosis. The miotic pupil is seen in disorders such as Horner’s syndrome or opiate overdose.
Mydriasis is the dilatation of the pupil in response to disease, trauma, drugs (or the dark!). It is pathological when light fails to induce miosis. The radial muscle is innervated by the sympathetic nervous system. Because the parasympathetic fibres travel with the oculomotor nerve they will be damaged by lesions affecting this nerve (e.g. cranial trauma).
The response to light shone in one eye is usually a constriction of both pupils. This indicates intact direct and consensual light reflexes. When the optic nerve has an afferent defect the light shining on the affected eye will produce a diminished pupillary response in both eyes. Whereas light shone on the unaffected eye will produce a normal pupillary response in both eyes. This is referred to as the Marcus Gunn pupil and is seen in conditions such as optic neuritis. In a total CN II lesion shining the light in the affected eye will produce no response.
Oculomotor nerve The pupillary effects are described above. In addition it supplies all ocular muscles apart from lateral rectus and superior oblique. Thus the affected eye will be deviated inferolaterally. Levator palpebrae superioris may also be impaired resulting in impaired ability to open the eye.
Trochlear nerve The eye will not be able to look down.
Trigeminal nerve Largest cranial nerve. Exits the brainstem at the pons. Branches are ophthalmic, maxillary and mandibular. Only the mandibular branch has both sensory and motor fibres. Branches converge to form the trigeminal ganglion (located in Meckels cave). It supplies the muscles of mastication and also tensor veli palatine, mylohyoid, anterior belly of digastric and tensor tympani. The detailed descriptions of the various sensory functions are described in other areas of the website. The corneal reflex is important and is elicited by applying a small tip of cotton wool to the cornea, a reflex blink should occur if it is intact. It is mediated by: the naso ciliary branch of the ophthalmic branch of the trigeminal (sensory component) and the facial nerve producing the motor response. Lesions of the afferent arc will produce bilateral absent blink and lesions of the efferent arc will result in a unilateral absent blink.
Abducens nerve The affected eye will have a deficit of abduction. This cranial nerve exits the brainstem between the pons and medulla. It thus has a relatively long intra cranial course which renders it susceptible to damage in raised intra cranial pressure.
Facial nerve Emerges from brainstem between pons and medulla. It controls muscles of facial expression and taste from the anterior 2/3 of the tongue. The nerve passes into the petrous temporal bone and into the internal auditory meatus. It then passes through the facial canal and exits at the stylomastoid foramen. It passes through the parotid gland and divides at this point. It does not innervate the parotid gland. Its divisions are considered in other parts of the website. Its motor fibres innervate orbicularis oculi to produce the efferent arm of the corneal reflex. In surgical practice it may be injured during parotid gland surgery or invaded by malignancies of the gland and a lower motor neurone on the ipsilateral side will result.
Vestibulo-cochlear nerve Exits from the pons and then passes through the internal auditory meatus. It is implicated in sensorineural hearing loss. Individuals with sensorineural hearing loss will localise the sound in webers test to the normal ear. Rinnes test will be reduced on the affected side but should still work. These two tests will distinguish sensorineural hearing loss from conductive deafness. In the latter condition webers test will localise to the affected ear and Rinnes test will be impaired on the affected side. Surgical lesions affecting this nerve include CNS tumours and basal skull fractures. It may also be damaged by the administration of ototoxic drugs (of which gentamicin is the most commonly used in surgical practice).
Glossopharyngeal nerve Exits the pons just above the vagus. Receives sensory fibres from posterior 1/3 tongue, tonsils, pharynx and middle ear (otalgia may occur following tonsillectomy). It receives visceral afferents from the carotid bodies. It supplies parasympathetic fibres to the parotid gland via the otic ganglion and motor function to stylopharyngeaus muscle. The sensory function of the nerve is tested using the gag reflex.
Vagus nerve Leaves the medulla between the olivary nucleus and the inferior cerebellar peduncle. Passes through the jugular foramen and into the carotid sheath. Details of the functions of the vagus nerve are covered in the website under relevant organ sub headings.
Accessory nerve Exists from the caudal aspect of the brainstem (multiple branches) supplies trapezius and sternocleidomastoid muscles. The distal portion of this nerve is most prone to injury during surgical procedures.
Hypoglossal nerve Emerges from the medulla at the preolivary sulcus, passes through the hypoglossal canal. It lies on the carotid sheath and passes deep to the posterior belly of digastric to supply muscles of the tongue (except palatoglossus). Its location near the carotid sheath makes it vulnerable during carotid endarterectomy surgery and damage will produce ipsilateral defect in muscle function.

1975
Q

Which of the following nerves supplies the majority of the skin on the palmar aspect of the thumb?

Ulnar

Median

Radial

Musculocutaneous

None of the above

A

The median nerve supplies cutaneous sensation to this region.
Median nerve

The median nerve is formed by the union of a lateral and medial root respectively from the lateral (C5,6,7) and medial (C8 and T1) cords of the brachial plexus; the medial root passes anterior to the third part of the axillary artery. The nerve descends lateral to the brachial artery, crosses to its medial side (usually passing anterior to the artery). It passes deep to the bicipital aponeurosis and the median cubital vein at the elbow.
It passes between the two heads of the pronator teres muscle, and runs on the deep surface of flexor digitorum superficialis (within its fascial sheath).
Near the wrist it becomes superficial between the tendons of flexor digitorum superficialis and flexor carpi radialis, deep to palmaris longus tendon. It passes deep to the flexor retinaculum to enter the palm, but lies anterior to the long flexor tendons within the carpal tunnel.

Branches
Region Branch
Upper arm No branches, although the nerve commonly communicates with the musculocutaneous nerve
Forearm Pronator teres
Pronator quadratus
Flexor carpi radialis
Palmaris longus
Flexor digitorum superficialis
Flexor pollicis longus
Flexor digitorum profundus (only the radial half)
Distal forearm Palmar cutaneous branch
Hand (Motor) Motor supply (LOAF)
Lateral 2 lumbricals
Opponens pollicis
Abductor pollicis brevis
Flexor pollicis brevis
Hand (Sensory)
Over thumb and lateral 2 ½ fingers
On the palmar aspect this projects proximally, on the dorsal aspect only the distal regions are innervated with the radial nerve providing the more proximal cutaneous innervation.

Patterns of damage
Damage at wrist
e.g. carpal tunnel syndrome
paralysis and wasting of thenar eminence muscles and opponens pollicis (ape hand deformity)
sensory loss to palmar aspect of lateral (radial) 2 ½ fingers

Damage at elbow, as above plus:
unable to pronate forearm
weak wrist flexion
ulnar deviation of wrist

Anterior interosseous nerve (branch of median nerve)
leaves just below the elbow
results in loss of pronation of forearm and weakness of long flexors of thumb and index finger

1976
Q

A 36 year old male is admitted for elective surgery for a lymph node biopsy in the supraclavicular region. Post operatively the patient has difficulty shrugging his left shoulder. What is the most likely reason?

Phrenic nerve lesion

Axillary nerve lesion

C5, C6 root lesion

C8, T1 root lesion

Accessory nerve lesion

A

The accessory nerve lies in the posterior triangle and may be injured in this region. Apart from problems with shrugging the shoulder, he may also have difficulty lifting his arm above his head.

Posterior triangle of the neck

Boundaries
Apex Sternocleidomastoid and the Trapezius muscles at the Occipital bone
Anterior Posterior border of the Sternocleidomastoid
Posterior Anterior border of the Trapezius
Base Middle third of the clavicle

Contents
Nerves
Accessory nerve
Phrenic nerve
Three trunks of the brachial plexus
Branches of the cervical plexus: Supraclavicular nerve, transverse cervical nerve, great auricular nerve, lesser occipital nerve
Vessels
External jugular vein
Subclavian artery
Muscles
Inferior belly of omohyoid
Scalene
Lymph nodes
Supraclavicular
Occipital

1977
Q

A 32 year old man presents with an inguinal hernia and undergoes an open surgical repair. The surgeons decide to place a mesh on the posterior wall of the inguinal canal to complete the repair, which of the following structures will lie posterior to the mesh?

Transversalis fascia

External oblique

Rectus abdominis

Obturator nerve

None of the above

A

Inguinal canal walls: ‘MALT: 2M, 2A, 2L, 2T’:

Starting from superior, moving around in order to posterior:
Superior wall (roof): 2 Muscles:Internal oblique, transversus abdominis
Anterior wall: 2 Aponeuroses: Aponeurosis of external oblique, Aponeurosis of internal oblique
Lower wall (floor): 2 Ligaments: Inguinal Ligament, Lacunar Ligament Posterior wall: 2Ts: Transversalis fascia, Conjoint Tendon
This is actually quite a straightforward question. It is simply asking for the structure that forms the posterior wall of the inguinal canal. This is composed of the transversalis fascia, the conjoint tendon and more laterally the deep inguinal ring.

1978
Q

Which of the following is not a typical feature of an arterial leg ulcer?

Well demarcated edges

A grey - white base to the ulcer

Men are affected more than women

Painful

Ankle swelling

A

Ankle swelling is often absent. If present it may be due to mixed arteriovenous disease. With mixed disease the arterial component is treated first.

Peripheral vascular disease

Indications for surgery to revascularise the lower limb
Intermittent claudication
Critical ischaemia
Ulceration
Gangrene

Intermittent claudication that is not disabling may provide a relative indication, whilst the other complaints are often absolute indications depending upon the frailty of the patient.

Assessment
Clinical examination
Ankle brachial pressure index measurement
Duplex arterial ultrasound
Angiography (standard, CT or MRI): usually performed only if intervention being considered.

Angioplasty
In order for angioplasty to be undertaken successfully the artery has to be accessible. The lesion relatively short and reasonable distal vessel runoff. Longer lesions may be amenable to sub-intimal angioplasty.

Surgery
Surgery will be undertaken where attempts at angioplasty have either failed or are unsuitable. Bypass essentially involves bypassing the affected arterial segment by utilising a graft to run from above the disease to below the disease. As with angioplasty good runoff improves the outcome.

Some key concepts with bypass surgery

Superficial femoral artery occlusion to the above knee popliteal
In the ideal scenario, vein (either in situ or reversed LSV) would be used as a conduit. However, prosthetic material has reasonable 5 year patency rates and some would advocate using this in preference to vein so that vein can be used for other procedures in the future. In general terms either technique is usually associated with an excellent outcome (if run off satisfactory).

Procedure
Artery dissected out, IV heparin 3,000 units given and then the vessels are cross clamped
Longitudinal arteriotomy
Graft cut to size and tunneled to arteriotomy sites
Anastomosis to femoral artery usually with 5/0 ‘double ended’ Prolene suture
Distal anastomosis usually using 6/0 ‘double ended’ Prolene

Distal disease
Femoro-distal bypass surgery takes longer to perform, is more technically challenging and has higher failure rates.
In elderly diabetic patients with poor runoff a primary amputation may well be a safer and more effective option. There is no point in embarking on this type of surgery in patients who are wheelchair bound.
In femorodistal bypasses vein gives superior outcomes to PTFE.

Rules
Vein mapping 1st to see whether there is suitable vein (the preferred conduit). Sub intimal hyperplasia occurs early when PTFE is used for the distal anastomosis and will lead to early graft occlusion and failure.
Essential operative procedure as for above knee fem-pop.
If there is insufficient vein for the entire conduit then vein can be attached to the end of the PTFE graft and then used for the distal anastomosis. This type of ‘vein boot’ is technically referred to as a Miller Cuff and is associated with better patency rates than PTFE alone.
Remember the more distal the arterial anastomosis the lower the success rate.

References
Peach G et al. Diagnosis and management of peripheral arterial disease. BMJ 2012; 345: 36-41.

1979
Q

A 43 year old man presents with dyspepsia and undergoes an upper GI endoscopy. During the procedure diffuse gastric and duodenal ulcers are identified. A Clo test confirms the presence of Helicobacter pylori infection. What is the most likely explanation for the ulcers?

Decreased gastric motility

Increased urease activity

Decreased release of mucous and bicarbonate

Decreased gastrin levels

Increased acid production

A

H-Pylori has a number of pathological effects. In this question the main issue is by what mechanism the organism is able to induce both gastric and duodenal ulceration. Without modestly elevated acid levels, the duodenum would not undergo gastric metaplasia. H-Pylori cannot colonise duodenal mucosa and therefore the development of ulcers at this site can only occur in those who have undergone metaplastic transformation (mediated by increased acidity).

Helicobacter Pylori

Infection with Helicobacter Pylori is implicated in many cases of duodenal ulceration and up to 60% of patients with gastric ulceration.

It is a gram negative, helical shaped rod with microaerophillic requirements. It has the ability to produce a urease enzyme that will hydrolyse urea resulting in the production of ammonia. The effect of ammonia on antral G cells is to cause release of gastrin via a negative feedback loop.

Once infection is established the organism releases enzymes that disrupt the gastric mucous layer. Certain subtypes release cytotoxins cag A and vac A gene products. The organism incites a classical chronic inflammatory process of the gastric epithelium. This accounts for the development of gastric ulcers. The mildly increased acidity may induce a process of duodenal gastric metaplasia. Whilst duodenal mucosa cannot be colonised by H-Pylori, mucosa that has undergone metaplastic change to the gastric epithelial type may be colonised by H- Pylori with subsequent inflammation and development of duodenitis and ulcers.

In patients who are colonized, there is a 10-20% risk of peptic ulcer, 1-2% risk gastric cancer and <1% risk MALT lymphoma.

1980
Q

Which of the following statements relating to menisceal tears is false?

The medial meniscus is most often affected

True locking of the knee joint may occur

Most established tears will heal with conservative management

In the chronic setting there is typically little to find on examination if the knee is not locked

An arthroscopic approach may be used to treat most lesions

A

Menisci have no nerve or blood supply and thus heal poorly. Established tears with associated symptoms are best managed by arthroscopic menisectomy.

Knee injury

Types of injury

Ruptured anterior cruciate ligament
Sport injury
Mechanism: high twisting force applied to a bent knee
Typically presents with: loud crack, pain and RAPID joint swelling (haemoarthrosis)
Poor healing
Management: intense physiotherapy or surgery
Ruptured posterior cruciate ligament
Mechanism: hyperextension injuries
Tibia lies back on the femur
Paradoxical anterior draw test
Rupture of medial collateral ligament
Mechanism: leg forced into valgus via force outside the leg
Knee unstable when put into valgus position
Menisceal tear
Rotational sporting injuries
Delayed knee swelling
Joint locking (Patient may develop skills to ‘unlock’ the knee
Recurrent episodes of pain and effusions are common, often following minor trauma
Chondromalacia patellae
Teenage girls, following an injury to knee e.g. Dislocation patella
Typical history of pain on going downstairs or at rest
Tenderness, quadriceps wasting
Dislocation of the patella
Most commonly occurs as a traumatic primary event, either through direct trauma or through severe contraction of quadriceps with knee stretched in valgus and external rotation
Genu valgum, tibial torsion and high riding patella are risk factors
Skyline x-ray views of patella are required, although displaced patella may be clinically obvious
An osteochondral fracture is present in 5%
The condition has a 20% recurrence rate
Fractured patella
2 types:
i. Direct blow to patella causing undisplaced fragments
ii. Avulsion fracture
Tibial plateau fracture
Occur in the elderly (or following significant trauma in young)
Mechanism: knee forced into valgus or varus, but the knee fractures before the ligaments rupture
Varus injury affects medial plateau and if valgus injury, lateral plateau depressed fracture occurs
Classified using the Schatzker system (see below)

Schatzker Classification system for tibial plateau fractures
Type Anatomical description Features
1 Vertical split of lateral condyle Fracture through dense bone, usually in the young. It may be virtually undisplaced, or the condylar fragment may be pushed inferiorly and tilted
2 Vertical split of the lateral condyle combined with an adjacent load bearing part of the condyle The wedge fragment (which may be of variable size), is displaced laterally; the joint is widened. Untreated, a valgus deformity may develop
3 Depression of the articular surface with intact condylar rim The split does not extend to the edge of the plateau. Depressed fragments may be firmly embedded in subchondral bone, the joint is stable
4 Fragment of the medial tibial condyle Two injuries are seen in this category; (1) a depressed fracture of osteoporotic bone in the elderly. (2) a high energy fracture resulting in a condylar split that runs from the intercondylar eminence to the medial cortex. Associated ligamentous injury may be severe
5 Fracture of both condyles Both condyles fractured but the column of the metaphysis remains in continuity with the tibial shaft
6 Combined condylar and subcondylar fractures High energy fracture with marked comminution

1981
Q

A 45 year old man is recovering in hospital following a total hip replacement. He develops a profuse and watery diarrhoea. Several other patients have been suffering from similar symptoms. Infection with which of the following is the most likely underlying cause?

Clostridium perfringens

Clostridium botulinim

Clostridium difficile

Clostridium welchi

Clostridium tetani

A

Clostridium difficile can spread rapidly on surgical wards. The use of broad spectrum prophylactic antibiotics during arthroplasty surgery can increase the risk.

Surgical Microbiology

An extensive topic so an overview is given here. Organisms causing common surgical infections are reasonable topics in the examination. However, microbiology is less rigorously tested than anatomy, for example.

Common organisms

Staphylococcus aureus
Facultative anaerobe
Gram positive coccus
Haemolysis on blood agar plates
Catalase positive
20% population are long term carriers
Exo and entero toxin may result in toxic shock syndrome and gastroenteritis respectively
Ideally treated with penicillin although many strains now resistant through beta Lactamase production. In the UK less than 5% of isolates are sensitive to penicillin.
Resistance to methicillin (and other antibiotics) is mediated by the mec operon , essentially penicillin binding protein is altered and resistance to this class of antibiotics ensues
Common cause of cutaneous infections and abscesses

Streptococcus pyogenes
Gram positive, forms chain like colonies, Lancefield Group A Streptococcus
Produces beta haemolysis on blood agar plates
Rarely part of normal skin microflora
Catalase negative
Releases a number of proteins/ virulence factors into host including hyaluronidase, streptokinase which allow rapid tissue destruction
Releases superantigens such as pyogenic exotoxin A which results in scarlet fever
Remains sensitive to penicillin, macrolides may be used as an alternative.

Escherichia coli
Gram negative rod
Facultative anaerobe, non sporing
Wide range of subtypes and some are normal gut commensals
Some subtypes such as 0157 may produce lethal toxins resulting in haemolytic-uraemic syndrome
Enterotoxigenic E-Coli produces an enterotoxin (ST enterotoxin) that results in large volume fluid secretion into the gut lumen (Via cGMP activation)
Enteropathogenic E-Coli binds to intestinal cells and cause structural damage, this coupled with a moderate (or in case of enteroinvasive E-Coli significant) invasive component produces enteritis and large volume diarrhoea together with fever.
They are resistant to many antibiotics used to treat gram positive infections and acquire resistance rapidly and are recognised as producing beta lactamases

Campylobacter jejuni
Curved, gram negative, non sporulating bacteria
One of the commonest causes of diarrhoea worldwide
Produces enteritis which is often diffuse and blood may be passed
Remains a differential for right iliac fossa pain with diarrhoea
Self limiting infection so antibiotics are not usually advised. However, the quinolones are often rapidly effective.

Helicobacter pylori
Gram negative, helix shaped rod, microaerophillic
Produces hydrogenase that can derive energy from hydrogen released by intestinal bacteria
Flagellated and mobile
Those carrying the cag A gene may cause ulcers
It secretes urease that breaks down gastric urea> Carbon dioxide and ammonia> ammonium>bicarbonate (simplified!) The bicarbonate can neutralise the gastric acid.
Usually colonises the gastric antrum and irritates resulting in increased gastrin release and higher levels of gastric acid. These patients will develop duodenal ulcers. In those with more diffuse H-Pylori infection gastric acid levels are lower and ulcers develop by local tissue damage from H-Pylori- these patients get gastric ulcers.
Diagnosis may be made by serology (approx. 75% sensitive). Biopsy urease test during endoscopy probably the most sensitive.
In patients who are colonised 10-20% risk of peptic ulcer, 1-2% risk gastric cancer, <1% risk MALT lymphoma.

1982
Q

A 6 year old child requires long term drug therapy to treat a rare genetic disorder. The drug must be given intravenously. Recently, there have been issues with the child pulling at the current system of the Hickman line and the parents are requesting an alternative. What is the best option?

Triple lumen subclavian line

Portacath device

Intermittent cannulation as needed

PICC line

Broviac line

A

Portacaths are a good choice for children as they only need to be accessed when they are used. Broviacs would pose the same core problems as a Hickman.

Intravenous access

Venous access
A number of routes for establishing venous access are available.

Peripheral venous cannula
Easy to insert with minimal morbidity. Wide lumen cannulae can provide rapid fluid infusions. When properly managed infections may be promptly identified and the cannula easily re sited. Problems relate to their peripheral sites and they are unsuitable for the administration of vaso active drugs, such as inotropes and irritant drugs such as TPN (except in the very short term setting).

Central lines
Insertion is more difficult and most operators and NICE advocate the use of ultra sound. Coagulopathies may lead to haemorrhage following iatrogenic arterial injury. Femoral lines are easier to insert and iatrogenic injuries easier to manage in this site however they are prone to high infection rates. Internal jugular route is preferred. They have multiple lumens allowing for administration of multiple infusions. The lumens are relatively narrow and thus they do not allow particularly rapid rates of infusion.

Intraosseous access
This is typically undertaken at the anteromedial aspect of the proximal tibia and provides access to the marrow cavity and circulatory system. Although traditionally preferred in paediatric practice they may be used in adults and a wide range of fluids can be infused using these devices.

Tunneled lines
Tunneled lines such as Groshong and Hickman lines are popular devices for patients with long term therapeutic requirements. These devices are usually inserted using ultrasound guidance into the internal jugular vein and then tunneled under the skin. A cuff of woven material is sited near the end and helps to anchor the device into the tissues. These cuffs require formal dissection to allow the device to be removed. Tunneled lines can be linked to injection ports that are located under the skin. These are especially popular in paediatric practice.

Peripherally inserted central cannula
Referred to as PICC lines, these are popular methods for establishing central venous access. Because they are inserted peripherally they are less prone to major complications relating to device insertion than conventional central lines.

1983
Q

A 73 year old lady presents with peritonitis and tenderness of the left groin. At operation, she has a left femoral hernia with perforation of the anti mesenteric border of ileum associated with the hernia. What type of hernia is this?

Richters hernia

Littres hernia

Morgagni hernia

Spigelian hernia

Bochdalek hernia

A

When part of the bowel wall is trapped in a hernia such as this it is termed a Richters hernia and may complicate any hernia although femoral and obturator hernias are most typically implicated.

Hernia

Hernias occur when a viscus or part of it protrudes from within its normal anatomical cavity. Specific hernias are covered under their designated titles the remainder are addressed here.

Spigelian hernia
Interparietal hernia occurring at the level of the arcuate line
Rare
May lie beneath internal oblique muscle. Usually between internal and external oblique
Equal sex distribution
Position is lateral to rectus abdominis
Both open and laparoscopic repair are possible, the former in cases of strangulation

Lumbar hernia
The lumbar triangle (through which these may occur) is bounded by:
Crest of ilium (inferiorly)
External oblique (laterally)
Latissimus dorsi (medially)
Primary lumbar herniae are rare, most are incisional hernias following renal surgery
- Direct anatomical repair with or without mesh re-enforcement is the procedure of choice

Obturator hernia
Herniation through the obturator canal
Commoner in females
Usually lies behind pectineus muscle
Elective diagnosis is unusual most will present acutely with obstruction
When presenting acutely most cases with require laparotomy or laparoscopy (and small bowel resection if indicated)

Richters hernia
Condition in which part of the wall of the small bowel (usually the anti mesenteric border) is strangulated within a hernia (of any type)
They do not present with typical features of intestinal obstruction as lumenal patency is preserved
Where vomiting is prominent it usually occurs as a result of paralytic ileus from peritonitis (as these hernias may perforate)

Incisional hernia
Occur through sites of surgical access into the abdominal cavity
Most common following surgical wound infection
To minimise following midline laparotomy Jenkins Rule should be followed and this necessitates a suture length 4x length of incision with bites taken at 1cm intervals, 1 cm from the wound edge
Repair may be performed either at open surgery or laparoscopically and a wide variety of techniques are described

Bochdalek hernia
Typically congenital diaphragmatic hernia
85% cases are located in the left hemi diaphragm
Associated with lung hypoplasia on the affected side
More common in males
Associated with other birth defects
May contain stomach
May be treated by direct anatomical apposition or placement of mesh. In infants that have severe respiratory compromise mechanical ventilation may be needed and mortality rate is high

Morgagni Hernia
Rare type of diaphragmatic hernia (approx 2% cases)
Herniation through foramen of Morgagni
Usually located on the right and tend to be less symptomatic
More advanced cases may contain transverse colon
As defects are small pulmonary hypoplasia is less common
Direct anatomical repair is performed

Umbilical hernia
Hernia through weak umbilicus
Usually presents in childhood
Often symptomatic
Equal sex incidence
95% will resolve by the age of 2 years
Surgery performed after the third birthday

Paraumbilical hernia
Usually a condition of adulthood
Defect is in the linea alba
More common in females
Multiparity and obesity are risk factors
Traditionally repaired using Mayos technique - overlapping repair, mesh may be used though not if small bowel resection is required owing to acute strangulation

Littres hernia
Hernia containing Meckels diverticulum
Resection of the diverticulum is usually required and this will preclude a mesh repair

1984
Q

During an inguinal hernia repair the surgeon identifies a small nerve whilst mobilising the cord structures at the level of the superficial inguinal ring. Which nerve is this most likely to be?

Subcostal

Iliohypogastric

Ilioinguinal

Obturator

Pudendal

A

Ilioinguinal nerve entrapment may be a cause of neuropathic pain following inguinal hernia surgery.
The ilioinguinal nerve passes through the superfical inguinal ring and is routinely encountered when exploring the inguinal canal during hernia surgery. The iliohypogastric nerve pierces the aponeurosis of the external oblique muscle superior to the superficial inguinal ring.

Ilioinguinal nerve

Arises from the first lumbar ventral ramus with the iliohypogastric nerve. It passes inferolaterally through the substance of psoas major and over the anterior surface of quadratus lumborum. It pierces the internal oblique muscle and passes deep to the aponeurosis of the external oblique muscle. It enters the inguinal canal and then passes through the superficial inguinal ring to reach the skin.

Branches
To supply those muscles of the abdominal wall through which it passes.
Skin and fascia over the pubic symphysis, superomedial part of the femoral triangle, surface of the scrotum, root and dorsum of penis or labium majus in females.

1985
Q

A 63 year old male is admitted to the surgical ward for an elective femoral popliteal bypass. He suddenly starts complaining of central, crushing chest pain which is radiating to the left arm. An ECG shows some ischaemic changes. The Nursing staff start high flow oxygen and give a spray of glyceryl trinitrate spray. Unfortunately there is no relief of symptoms. What is the next agent to be administered?

Aspirin 75mg

Clopidogrel 75mg

Aspirin 300mg

Clopidogrel 300mg

Direct admission to angiography suite

A

Aspirin 300mg should be given as soon as possible. If the patient has a moderate to high risk of myocardial infarction, then Clopidogrel should be given with a low molecular weight heparin. Thromboloysis or urgent percutaneous intervention should be given if there are significant ECG changes.

Management of acute coronary syndrome

NICE produced guidelines in 2020 on the management of unstable angina and non-ST elevation myocardial infarction (NSTEMI). They advocate managing patients based on the early risk assessment using a recognised scoring system such as GRACE (Global Registry of Acute Cardiac Events) to calculate a predicted 6 month mortality.

All patients should receive
- aspirin 300mg
- nitrates or morphine to relieve chest pain if required

Supplementary oxygen should be routinely given unless there are compelling reasons not to do so.

Antithrombin treatment. Low molecular weight heparin should be offered to patients who are not at a high risk of bleeding and who are not having angiography within the next 24 hours. If angiography is likely within 24 hours or a patients creatinine is > 265 umol/l unfractionated heparin should be given.

Clopidogrel 300mg should be given to patients with a predicted 6 month mortality of more than 1.5% or patients who may undergo percutaneous coronary intervention within 24 hours of admission to hospital. Clopidogrel should be continued for 12 months. Dual antiplatelet therapy should be given for patients who have undergone PCI and this should be continued for 12 months.

Intravenous glycoprotein IIb/IIIa receptor antagonists (eptifibatide or tirofiban) should be given to patients who have an intermediate or higher risk of adverse cardiovascular events (predicted 6-month mortality above 3.0%), and who are scheduled to undergo angiography within 96 hours of hospital admission. These agents should be not be administered to those patients who are being considered for PCI as their primary intervention.

Coronary angiography should be considered within 96 hours of first admission
to hospital to patients who have a predicted 6-month mortality above 3.0%. It should also be performed as soon as possible in patients who are clinically unstable. Coronary angiography is the preferred modality for those patients within 12 hours of acute STEMI.

1986
Q

A 22 year old man has undergone an inguinal hernia repair. Seven days later he presents with an erythematous and tender wound that is discharging a purulent material. What is the most likely cause?

Infection with Staphylococcus aureus

Discharging haematoma

Infection with Pseudomonas

Infection with Streptococcus pyogenes

Infection with Bacteroides

A

In this setting Staphylococcus aureus Infection is the most likely cause. In the UK between 2010 and 2011 the commonest cause of wound infection was enterobacter infections (usually following cardiac or colonic surgery). 23% of infections were due to Staph aureus, which fits the scenario above. Infection with the other organisms including strep pyogenes was much rarer.

Surgical site infection

  • Surgical site infections may occur following a breach in tissue surfaces and allow normal commensals and other pathogens to initiate infection. They are a major cause of morbidity and mortality.
    Surgical site infections (SSI) comprise up to 20% of all healthcare associated infections and at least 5% of patients undergoing surgery will develop an SSI as a result.
    In many cases the organisms are derived from the patient’s own body. Measures that may increase the risk of SSI include:
    Shaving the wound using a razor (disposable clipper preferred)
    Using a non iodine impregnated incise drape if one is deemed to be necessary
    Tissue hypoxia
    Delayed administration of prophylactic antibiotics in tourniquet surgery

Preoperatively
Don’t remove body hair routinely
If hair needs removal, use electrical clippers with single use head (razors increase infection risk)
Antibiotic prophylaxis if:
- placement of prosthesis or valve
- clean-contaminated surgery
- contaminated surgery
Use local formulary
Aim to give single dose IV antibiotic on anaesthesia
If a tourniquet is to be used, give prophylactic antibiotics earlier

Intraoperatively
Prepare the skin with alcoholic chlorhexidine (Lowest incidence of SSI)
Cover surgical site with dressing
A recent meta analysis has confirmed that administration of supplementary oxygen does not reduce the risk of wound infection. In contrast to previous individual RCT’s(1)
Wound edge protectors do not appear to confer benefit (2)

Post operatively
Tissue viability advice for management of surgical wounds healing by secondary intention

Use of diathermy for skin incisions
In the NICE guidelines the use of diathermy for skin incisions is not advocated(3). Several randomised controlled trials have been undertaken and demonstrated no increase in risk of SSI when diathermy is used(4).

References
1. Brar M et al.. Perioperative supplemental oxygen in colorectal patients: a meta analysis. J Surg Res 2011 (166): 227 -235.
2. Pinkney T et al. Impact of wound edge protection devices on surgical site infection after laparotomy: impact of a multicentre randomised controlled trial (ROSSINI Trial). BMJ 2013 (347):10.
3. http://www.nice.org.uk/CG74
4. Ahmad N and Ahmed A. Meta-analysis of the effectiveness of surgical scalpel or diathermy in making abdominal skin incisions. Ann Surg 2011, 253(1):8-13.

1987
Q

Approximately what proportion of basal salivary secretions are provided by the submandibular glands?

10%

70%

40%

90%

20%

A

Although they are small, the submandibular glands provide the bulk of salivary secretions and contribute 70%, the sublingual glands provide 5% and the remainder from the parotid. During food consumption the contribution of the parotid secretions is greater.

Submandibular gland

Relations of the submandibular gland
Superficial Platysma, deep fascia and mandible
Submandibular lymph nodes
Facial vein (facial artery near mandible)
Marginal mandibular nerve
Cervical branch of the facial nerve
Deep Facial artery (inferior to the mandible)
Mylohyoid muscle
Sub mandibular duct
Hyoglossus muscle
Lingual nerve
Submandibular ganglion
Hypoglossal nerve

Submandibular duct (Wharton’s duct)
Opens lateral to the lingual frenulum on the anterior floor of mouth.
5 cm length
Lingual nerve wraps around Wharton’s duct. As the duct passes forwards it crosses medial to the nerve to lie above it and then crosses back, lateral to it, to reach a position below the nerve.

Innervation
Sympathetic innervation- Derived from superior cervical ganglion
Parasympathetic innervation- Submandibular ganglion via lingual nerve

Arterial supply
Branch of the facial artery. The facial artery passes through the gland to groove its deep surface. It then emerges onto the face by passing between the gland and the mandible.

Venous drainage
Anterior facial vein (lies deep to the Marginal Mandibular nerve)

Lymphatic drainage
Deep cervical and jugular chains of nodes

1988
Q

A 72 year old female presents with a painless breast lump. Clinically, she has a 4cm diameter irregular breast mass, with no other palpable masses.
What is the most likely lesion?

Invasive ductal carcinoma

Ductal carcinoma in situ

Lobular carcinoma in situ

Invasive lobular carcinoma

Atypical ductal hyperplasia

A

A post menopausal woman is more likely to have a ductal carcinoma and they tend to occur at a single focus within the breast.
Breast cancer

  • Commoner in the older age group
    Invasive ductal carcinomas are the most common type. Some may arise as a result of ductal carcinoma in situ (DCIS). There are associated carcinomas of special type e.g. Tubular that may carry better prognosis.
    The pathological assessment involves assessment of the tumour and lymph nodes, sentinel lymph node biopsy is often used to minimise the morbidity of an axillary dissection.
    Treatment, typically this is either wide local excision or mastectomy. There are many sub types of both of these that fall outside of the MRCS. Some key rules to bear in mind.
    Whatever operation is contemplated the final cosmetic outcome does have a bearing. A woman with small breasts and a large tumour will tend to fare better with mastectomy, even if clear pathological and clinical margins can be obtained. Conversely a women with larger breasts may be able to undergo breast conserving surgery even with a relatively large primary lesion (NB tumours >4cm used to attract recommendation for mastectomy). For screen detected and impalpable tumour image guidance will be necessary.
    Reconstruction is always an option following any resectional procedure. However, its exact type must be tailored to age and co-morbidities of the patient. The main operations in common use include latissimus dorsi myocutaneous flap and sub pectoral implants. Women wishing to avoid a prosthesis may be offered TRAM or DIEP flaps.

Surgical options
Mastectomy vs Wide local excision

Mastectomy Wide Local Excision
Multifocal tumour Solitary lesion
Central tumour Peripheral tumour
Large lesion in small breast Small lesion in large breast
DCIS >4cm DCIS <4cm
Patient Choice Patient choice

Central lesions may be managed using breast conserving surgery where an acceptable cosmetic result may be obtained, this is rarely the case in small breasts

A compelling indication for mastectomy, a larger tumour that would be unsuitable for breast conserving surgery

1989
Q

Which of the following intravenous fluid solutions has the greatest chloride content?

Dextrose / saline

Normal saline

Hartmanns solution

Ringers lactate

5% dextrose

A

Normal saline has the highest chloride content and excessive administration of normal saline is a recognised risk factor for the development of hyperchloraemic metabolic acidosis.

1990
Q

A 32 year old lady is referred to the clinic with recurrent infections and abscesses in the neck. On examination, she has a midline defect with an overlying scab, which moves upwards on tongue protrusion. What is the most likely diagnosis?

Branchial cyst

Branchial fistula

Thyroglossal cyst

Sebaceous cyst

Dermoid cyst

A

Thyroglossal cyst is always located in the midline as it is this route that the thyroid takes during its embryological descent. Its connection with the foramen caecum means it will move on tongue protrusion.

Neck lumps

The table below gives characteristic exam question features for conditions causing neck lumps:

Reactive lymphadenopathy By far the most common cause of neck swellings. There may be a history of local infection or a generalised viral illness
Lymphoma Rubbery, painless lymphadenopathy
The phenomenon of pain whilst drinking alcohol is very uncommon
There may be associated night sweats and splenomegaly
Thyroid swelling May be hypo-, eu- or hyperthyroid symptomatically
Moves upwards on swallowing
Thyroglossal cyst More common in patients < 20 years old
Usually midline, between the isthmus of the thyroid and the hyoid bone
Moves upwards with protrusion of the tongue
May be painful if infected
Pharyngeal pouch More common in older men
Represents a posteromedial herniation between thyropharyngeus and cricopharyngeus muscles
Usually not seen, but if large then a midline lump in the neck that gurgles on palpation
Typical symptoms are dysphagia, regurgitation, aspiration and chronic cough
Cystic hygroma A congenital lymphatic lesion (lymphangioma) typically found in the neck, classically on the left side
Most are evident at birth, around 90% present before 2 years of age
Branchial cyst An oval, mobile cystic mass that develops between the sternocleidomastoid muscle and the pharynx
Develop due to failure of obliteration of the second branchial cleft in embryonic development
Usually present in early adulthood
Cervical rib More common in adult females
Around 10% develop thoracic outlet syndrome
Carotid aneurysm Pulsatile lateral neck mass which doesn’t move on swallowing

1991
Q

A 30 year old man cuts the corner of his lip whilst shaving. Over the next few days a large purplish lesion appears at the site which bleeds on contact. What is the most likely diagnosis?

Pyogenic granuloma

Keratoacanthoma

Melanoma

Squamous cell carcinoma

Adenocarcinoma

A

Pyogenic granulomas often appear at sites of trauma.

Skin Diseases

Skin lesions may be referred for surgical assessment, but more commonly will come via a dermatologist for definitive surgical management.

Skin malignancies include basal cell carcinoma, squamous cell carcinoma and malignant melanoma.

Basal Cell Carcinoma
Most common form of skin cancer.
Commonly occur on sun exposed sites apart from the ear.
Sub types include nodular, morphoeic, superficial and pigmented.
Typically slow growing with low metastatic potential.
Standard surgical excision, topical chemotherapy and radiotherapy are all successful.
As a minimum a diagnostic punch biopsy should be taken if treatment other than standard surgical excision is planned.

Squamous Cell Carcinoma
Again related to sun exposure.
May arise in pre - existing solar keratoses.
May metastasize if left.
Immunosupression (e.g. following transplant), increases risk.
Wide local excision is the treatment of choice and where a diagnostic excision biopsy has demonstrated SCC, repeat surgery to gain adequate margins may be required.

Malignant Melanoma
The main diagnostic features (major criteria):
Change in size
Change in shape
Change in colour
Secondary features (minor criteria)
Diameter >6mm
Inflammation
Oozing or bleeding
Altered sensation

Treatment
Suspicious lesions should undergo excision biopsy. The lesion should be removed completely as incision biopsy can make subsequent histopathological assessment difficult.
Once the diagnosis is confirmed the pathology report should be reviewed to determine whether further re-excision of margins is required (see below):

Margins of excision-Related to Breslow thickness
Lesions 0-1mm thick 1cm
Lesions 1-2mm thick 1- 2cm (Depending upon site and pathological features)
Lesions 2-4mm thick 2-3 cm (Depending upon site and pathological features)
Lesions >4 mm thick 3cm
Marsden J et al. Revised UK guidelines for management of Melanoma. Br J Dermatol 2010 163:238-256.

Further treatments such as sentinel lymph node mapping, isolated limb perfusion and block dissection of regional lymph node groups should be selectively applied.

Kaposi Sarcoma
Tumour of vascular and lymphatic endothelium.
Purple cutaneous nodules.
Associated with immuno supression.
Classical form affects elderly males and is slow growing.
Immunosupression form is much more aggressive and tends to affect those with HIV related disease.

Non malignant skin disease

Dermatitis Herpetiformis
Chronic itchy clusters of blisters.
Linked to underlying gluten enteropathy (coeliac disease).

Dermatofibroma
Benign lesion.
Firm elevated nodules.
Usually history of trauma.
Lesion consists of histiocytes, blood vessels and fibrotic changes.

Pyogenic granuloma
Overgrowth of blood vessels.
Red nodules.
Usually follow trauma.
May mimic amelanotic melanoma.

Acanthosis nigricans
Brown to black, poorly defined, velvety hyperpigmentation of the skin.
Usually found in body folds such as the posterior and lateral folds of the neck, the axilla, groin, umbilicus, forehead, and other areas.
The most common cause of acanthosis nigricans is insulin resistance, which leads to increased circulating insulin levels. Insulin spillover into the skin results in its abnormal increase in growth (hyperplasia of the skin).
In the context of a malignant disease, acanthosis nigricans is a paraneoplastic syndrome and is then commonly referred to as acanthosis nigricans maligna. Involvement of mucous membranes is rare and suggests a coexisting malignant condition.

1992
Q

A 65 year old man presents with significant lower urinary tract symptoms and is diagnosed as having benign prostatic hyperplasia. Which of the following drug treatments will produce the slowest clinical response?

Tamsulosin

Alfuzosin

Doxazosin

Finasteride

Terazosin

A

5 alpha reductase inhibitors have a more favorable side effect profile than α blockers.
Alpha blockers have a faster onset of action (but lower reduction of complications from BPH) than 5 α reductase inhibitors. Benign Prostatic Hyperplasia

Benign prostatic hyperplasia occurs via an increase in the epithelial and stromal cell numbers in the peri-urethral zone of the prostate. BPH is very common and 90% of men aged over 80 will have at least microscopic evidence of benign prostatic hyperplasia. The causes of BPH are still not well understood, but the importance of androgens remains appreciated even if the exact role by which they induce BPH is elusive.

Presentation
The vast majority of men will present with lower urinary tract symptoms. These will typically be:

Poor flow
Nocturia
Hesitancy
Incomplete and double voiding
Terminal dribbling
Urgency
Incontinence

Investigation
Digital rectal examination to assess prostatic size and morphology.
Urine dipstick for infections and haematuria.
Uroflowmetry (a flow rate of >15ml/second helps to exclude BOO)
Bladder pressure studies may help identify detrusor failure and whilst may not form part of first line investigations should be included in those with atypical symptoms and prior to redo surgery.
Bladder scanning to demonstrate residual volumes. USS if high pressure chronic retention.

Management
Lifestyle changes such as stopping smoking and altering fluid intake may help those with mild symptoms.
Medical therapy includes alpha blockers and 5 α reductase inhibitors. The former work quickly on receptor zones located at the bladder neck. Cardiovascular side effects are well documented. The latter work on testosterone metabolising enzymes. Although they have a slower onset of action, the 5 α reductase inhibitors may prevent acute urinary retention.
Surgical therapy includes transurethral resection of the prostate and is the treatment of choice in those with severe symptoms and those who fail to respond to medical therapy. More tailored bladder neck incision procedures may be considered in those with small prostates. Retrograde ejaculation may occur following surgery. The change in the type of irrigation solutions used has helped to minimise the TURP syndrome of electrolyte disturbances.

1993
Q

A 52 year old lady is referred to the breast clinic with symptoms of nipple discharge. The discharge is usually thick and green. Which of the following statements relating to the most likely underlying diagnosis is untrue?

The majority of patients will be smokers

Typically produces blood stained nipple discharge

It is not associated with increased risk of breast cancer

May result in development of mammary duct fistula

May require total duct excision (Hadfields operation) if it fails to resolve

A

Blood stained nipple discharge should always be investigated.
Nipple fluid cytology is generally unhelpful.
Discharge of this type of material is most likely to be due to duct ectasia. Green or brown discharge is most common. Blood stained discharge should raise concern of intraductal papilloma or cancer.

Nipple discharge

Causes of nipple discharge
Physiological During breast feeding
Galactorrhoea Commonest cause may be response to emotional events, drugs such as histamine receptor antagonists are also implicated
Hyperprolactinaemia
Commonest type of pituitary tumour
Microadenomas <1cm in diameter
Macroadenomas >1cm in diameter
Pressure on optic chiasm may cause bitemporal hemianopia
Mammary duct ectasia
Dilatation breast ducts.
Most common in menopausal women
Discharge typically thick and green in colour
Most common in smokers
Carcinoma
Often blood stained
May be underlying mass or axillary lymphadenopathy
Intraductal papilloma
Commoner in younger patients
May cause blood stained discharge
There is usually no palpable lump

Assessment of patients
Examine breast and determine whether there is mass lesion present
All mass lesions should undergo Triple assessment.

Reporting of investigations
Where a mass lesion is suspected or investigations are requested these are prefixed using a system that denotes the investigation type e.g. M for mammography, followed by a numerical code as shown below:

1 No abnormality
2 Abnormality with benign features
3 Indeterminate probably benign
4 Indeterminate probably malignant
5 Malignant

Management of non malignant nipple discharge
Exclude endocrine disease
Nipple cytology unhelpful
Smoking cessation advice for duct ectasia
For duct ectasia with severe symptoms, total duct excision may be warranted.

1994
Q

A 38 year old lady who smokes heavily presents with recurrent episodes of infection in the right breast. On examination she has an indurated area at the lateral aspect of the nipple areaolar complex. Imaging shows no mass lesions. What is the most likely diagnosis?

Duct ectasia

Periductal mastitis

Pagets disease of the nipple

Mondors disease of the breast

Radial scar

A

Periductal mastitis is common in smokers and may present with recurrent infections. Treatment is with co-amoxiclav. Mondors disease of the breast is a localised thrombophlebitis of a breast vein.

Duct ectasia

Duct ectasia is a dilatation and shortening of the terminal breast ducts within 3cm of the nipple. It is common and the incidence increases with age. It typically presents with nipple retraction and occasionally creamy nipple discharge. It may be confused with periductal mastitis, which presents in younger women, the vast majority of which are smokers. Periductal mastitis typically presents with periareolar or sub areolar infections and may be recurrent.
Patients with troublesome nipple discharge may be treated by microdochectomy (if young) or total duct excision (if older).

1995
Q

Which of the following is not found on a blood film post splenectomy?

Pappenheimer bodies

Stipple cells

Erythrocyte containing siderotic granules

Howell-Jolly bodies

Target cells

A

Stipple cells are found in lead poisoning/haemoglobinopathies.

Blood film in hyposplenism:

Howell-Jolly bodies
Pappenheimer bodies
Poikilocytes (Target cells)
Erythrocyte containing siderotic granules
Heinz bodies

Post splenectomy blood film changes

The loss of splenic tissue results in the inability to readily remove immature or abnormal red blood cells from the circulation. The red cell count does not alter significantly. However, cytoplasmic inclusions may be seen e.g. Howell-Jolly bodies.
In the first few days after splenectomy target cells, siderocytes and reticulocytes will appear in the circulation. Immediately following splenectomy a granulocytosis (mainly composed of neutrophils) is seen, this is replaced by a lymphocytosis and monocytosis over the following weeks.
The platelet count is usually increased and this may be persistent, oral antiplatelet agents may be needed in some patients.

1996
Q

A group of surgeons review a meta-analysis of a series of randomised controlled trials on the Cochrane database and decide that one type of hip replacement is superior to another. What level of evidence is this?

II

III

I

V

IV

A

A meta- analysis of more than one well designed trials will typically represent level I evidence. It does, of course, depend on how well the trials were conducted and reported.

Levels of evidence

The level of evidence refers to the study design used by investigators to minimise bias.

Level of evidence Source
I Evidence obtained from systematic review of all relevant randomised controlled trials
II Evidence derived from at least one properly designed randomised controlled trial
III Evidence derived from well designed pseudo-randomised controlled trials (e.g. alternate allocation) or historical controls
IV Evidence derived from case series or case reports
V Panel or expert opinion

Many of the categories contain sub groups, detailed knowledge of these are not required for MRCS Part A.

References
Interested users will find further information at www.cebm.net

1997
Q

A 22 year old man presents with symptoms of lethargy and bilateral facial nerve palsy. On examination he has bilateral parotid gland enlargement. What is the most likely cause?

Pleomorphic adenoma

Cerebrovascular accident

Sarcoidosis

Warthins tumour

Adenoid cystic carcinoma

A

Facial nerve palsy is the commonest neurological manifestation of sarcoid. It usually resolves. The absence of a discrete lesion on palpation is against the other causes.

Parotid gland clinical

Benign neoplasms
Up to 80% of all salivary gland tumours occur in the parotid gland and up to 80% of these are benign. There is no consistent correlation between the rate of growth and the malignant potential of the lesion. However, benign tumours should not invade structures such as the facial nerve.
With the exception of Warthins tumours, they are commoner in women than men. The median age of developing a lesion is in the 5th decade of life.

Benign tumour types
Tumour type Features
Benign pleomorphic adenoma or benign mixed tumor Most common parotid neoplasm (80%)
Proliferation of epithelial and myoepithelial cells of the ducts and an increase in stromal components
Slow growing, lobular, and not well encapsulated
Recurrence rate of 1-5% with appropriate excision (parotidectomy)
Recurrence possibly secondary to capsular disruption during surgery
Malignant degeneration occurring in 2-10% of adenomas observed for long periods, with carcinoma ex-pleomorphic adenoma occurring most frequently as adenocarcinoma
Warthin tumor (papillary cystadenoma lymphoma or adenolymphoma) Second most common benign parotid tumor (5%)
Most common bilateral benign neoplasm of the parotid
Marked male as compared to female predominance
Occurs later in life (sixth and seventh decades)
Presents as a lymphocytic infiltrate and cystic epithelial proliferation
May represent heterotopic salivary gland epithelial tissue trapped within intraparotid lymph nodes
Incidence of bilaterality and multicentricity of 10%
Malignant transformation rare (almost unheard of)
Monomorphic adenoma Account for less than 5% of tumours
Slow growing
Consist of only one morphological cell type (hence term mono)
Include; basal cell adenoma, canalicular adenoma, oncocytoma, myoepitheliomas
Haemangioma Should be considered in the differential of a parotid mass in a child
Accounts for 90% of parotid tumours in children less than 1 year of age
Hypervascular on imaging
Spontaneous regression may occur and malignant transformation is almost unheard of

Malignant salivary gland tumours
Types of malignancy

Mucoepidermoid carcinoma 30% of all parotid malignancies
Usually low potential for local invasiveness and metastasis (depends mainly on grade)
Adenoid cystic carcinoma Unpredictable growth pattern
Tendency for perineural spread
Nerve growth may display skip lesions resulting in incomplete excision
Distant metastasis more common (visceral rather than nodal spread)
5 year survival 35%
Mixed tumours Often a malignancy occurring in a previously benign parotid lesion
Acinic cell carcinoma Intermediate grade malignancy
May show perineural invasion
Low potential for distant metastasis
5 year survival 80%
Adenocarcinoma Develops from secretory portion of gland
Risk of regional nodal and distant metastasis
5 year survival depends upon stage at presentation, may be up to 75% with small lesions with no nodal involvement
Lymphoma Large rubbery lesion, may occur in association with Warthins tumours
Diagnosis should be based on regional nodal biopsy rather than parotid resection
Treatment is with chemotherapy (and radiotherapy)

Diagnostic evaluation
Plain x-rays may be used to exclude calculi
Sialography may be used to delineate ductal anatomy
FNAC is used in most cases
Superficial parotidectomy may be either diagnostic or therapeutic depending upon the nature of the lesion
Where malignancy is suspected the primary approach should be definitive resection rather than excisional biopsy
CT/ MRI may be used in cases of malignancy for staging primary disease

Treatment
For nearly all lesions this consists of surgical resection, for benign disease this will usually consist of a superficial parotidectomy. For malignant disease a radical or extended radical parotidectomy is performed. The facial nerve is included in the resection if involved. The need for neck dissection is determined by the potential for nodal involvement.

Other parotid disorders
HIV infection
Lymphoepithelial cysts associated with HIV occur almost exclusively in the parotid
Typically presents as bilateral, multicystic, symmetrical swelling
Risk of malignant transformation is low and management usually conservative

Sjogren syndrome
Autoimmune disorder characterised by parotid enlargement, xerostomia and keratoconjunctivitis sicca
90% of cases occur in females
Second most common connective tissue disorder
Bilateral, non tender enlargement of the gland is usual
Histologically, the usual findings are of a lymphocytic infiltrate in acinar units and epimyoepithelial islands surrounded by lymphoid stroma
Treatment is supportive
There is an increased risk of subsequent lymphoma

Sarcoid
Parotid involvement occurs in 6% of patients with sarcoid
Bilateral in most cases
Gland is not tender
Xerostomia may occur
Management of isolated parotid disease is usually conservative

1998
Q

A 40 year old marine injures his ankle on an assault course. On examination he has a severely swollen ankle, as well as tenderness over the medial malleolus and proximal fibula. X-rays demonstrate a medial malleolar fracture, spiral fracture of the proximal fibula and widening of the syndesmosis. What is the most appropriate definitive management?

Surgical fixation

Application of ankle boot

Application of lower leg plaster cast

Application of external fixation device

Below knee amputation

A

This is a Maisonneuve fracture of the proximal fibula. It indicates an unstable ankle injury with likely injury to the interosseous membrane. In the setting of radiographic evidence of syndesmotic widening, this requires surgical fixation to reduce and stabilise the syndesmosis.

Ankle injuries

An ankle fracture relates to a fracture around the tibio-talar joint. It generally refers to a fracture involving the lateral, and/or medial and/or posterior malleolus. Pilon and Tillaux fractures are also considered to be ankle fractures, but are not covered here.
Ankle fractures are common. They effect men and women in equal numbers, but men have a higher rate as young adults (sports and contact injuries), and women a higher rate post-menopausal (fragility type fracture).

Osseous anatomy
The ankle (or mortise) joint consists of the distal tibia (tibial plafond and posterior malleolus), the distal fibula (lateral malleolus), and the talus. The main movement at the ankle joint is plantar and dorsiflexion.

Ligamentous anatomy
Medial side: Deltoid ligament. This is divided into superficial and deep portions. It is the primary restraint to valgus tilting of the talus.

Lateral side: Lateral ligament complex consisting from anterior to posterior of the anterior talofibular ligament (ATFL), calcaneofibular ligament (CFL), and the posterior talofibular ligament (PTFL). Together they resist valgus stress to the ankle, and are a restraint to anterior translation of the talus within the mortise joint.

Syndesmosis: The syndesmosis is a ligament complex between the distal tibia and fibula, holding the two bones together. It is fundamental to the integrity of the ankle joint, and its disruption leads to instability. It consists of (from anterior to posterior) the anterior-inferior tibiofibular ligament (AITFL), the transverse tibiofibular ligament (TTFL), the interosseous membrane, and the posterior-inferior tibiofibular ligament (PITFL).

Presentation and initial management
Patients will present following a traumatic event with a painful, swollen ankle, and reluctance/inability to weight bear. The Ottawa rules can be applied to differentiate between an ankle fracture and sprain, but can be unreliable.
In high energy injuries, management should follow ATLS principles to identify more significant injuries first. Neurovascular status of the foot should be documented, and open injuries should be excluded. If an open injury is identified, it should be managed in line with BOAST 4 principles1. If an obvious deformity exists, it should be reduced as soon as possible with appropriate analgesia or conscious sedation. Radiographs of clearly deformed or dislocated joints are not necessary, and removing the pressure on the surrounding soft tissues from the underlying bony deformity is the priority. If the fracture pattern is not clinically obvious then plain radiographs are appropriate and will guide the subsequent manipulation during plaster-of-paris below knee backslab application.

Imaging
AP, lateral and mortise views (20 degrees internal rotation) are essential to evaluate fracture displacement and syndesmotic injury. Decreased tibiofibular overlap, medial joint clear space and lateral talar shift all indicate a syndesmotic injury. (In subtle cases of shift, imaging the uninjured ankle can be helpful as a proportion of the population have little or no tibiotalar overlap-reference 2.)

Where there is suspicion of syndesmosis involvement in the absence of radiographic evidence, stress radiographs can be diagnostic.
Complex fracture patterns (and increasingly posterior malleolar fractures) are best defined using CT.

Classification
The most commonly used classifications are Lauge-Hansen and Danis-Weber.

Lauge-Hansen
Comprises two parts: first part is the foot position, and the second part is the force applied. Useful for understanding the forces involved and therefore predict the ligamentous or bony injury. Results in four injury patterns:
Supination - Adduction (SA) - 10-20%
Supination - External rotation (SER) - 40-75%
Pronation - Abduction (PA) - 5-20%
Pronation - External rotation (PER) - 5-20%

Not often used in clinical practice but good for understanding the principles of ankle fracture.

Danis-Weber
Commonly used. Based on the level of the fibula fracture in relation to the syndesmosis. The more proximal, the greater the risk of syndesmotic injury and therefore fracture instability.
A - fracture below the level of the syndesmosis
B - fracture at the level of the syndesmosis / level of the tibial plafond
C - fracture above the level of the syndesmosis. This includes Maisonneuve fractures (proximal fibula fracture), which can be associated with ankle instability. Beware the high fibula fracture - it may be an ankle fracture!

The Weber classification is based purely on the the lateral side. All injuries can include a medial or posterior bony or ligamentous injury which also dictates fracture stability (bimalleolar and trimalleolar fractures are more unstable).

Treatment
When deciding upon treatment for an ankle fracture, one must consider both the fracture and the patient. Diabetic patients and smokers are at greater risk of post-operative complication, especially wound problems and infection. Likewise, the long term outcome of post-traumatic arthritis from a malunited ankle fracture is extremely important for a young patient, but not as relevant in the elderly. Therefore, normal surgical decision processes apply as with all fractures.

Defining stability of an ankle fracture underpins the treatment decision.
Weber A - Unimalleolar Weber A Weber fractures by definition are stable and therefore can be mobilised fully weight bearing in an ankle boot.

Weber C - Fractures tend to include syndesmotic disruption and are usually bimalleolar (either bony or ligamentous). They are therefore unstable and usually require operative fixation. In addition to the fracture fixation, the syndesmosis usually requires reconstruction/augmentation with screws to restore the joint integrity and function.

Weber B - B fractures vary greatly. They can be part of a trimalleolar injury and therefore extremely unstable, requiring fixation. Alternatively, a uni-malleolar Weber B fracture can be a stable injury, and therefore mobilised immediately in an ankle boot. Defining the stability can be challenging, and often involves stress radiographs, or a trial of mobilisation and repeat radiographs. However, treating undisplaced ankle fractures in a below knee plaster, non-weight bearing for six weeks is still widely practised, and a safe approach.

When operative fixation is appropriate, it is usually via open reduction and internal fixation using plates and screws. It must be carried out when soft tissue swelling has settled in order to minimise the risk of wound problems. This can often take a week to settle.

The use of fibula nails is expanding, but is not yet mainstream. Ankle fractures can also be treated with external fixation, or with a hind foot nail in patients who need fixation but where soft tissue or bone quality is poor.

Post operative management
Ankle fractures generally take 6 weeks to unite enough to prevent secondary displacement. This is therefore an appropriate time period to keep a cast on in a conservatively managed patient. Weight bearing post-operatively depends on the quality of the fixation and bone quality, and preference varies between surgeons, ranging from aggressive early mobilisation to a period of non-weight bearing. Return to activities takes approximately three months, and often requires assistance of a physiotherapist to improve range-of-movement and muscle strengthening.

References
1. http://www.boa.ac.uk/publications/boa-standards-trauma-boasts/
2. Shah AS, Kadakia AR, Tan GJ, Karadsheh MS, Wolter TD, Sabb B. Radiographic evaluation of the normal distal tibiofibular syndesmosis. Foot Ankle Int. 2012;33(10):870-6

1999
Q

Which of the following forms the floor of the anatomical snuffbox?

Radial artery

Cephalic vein

Extensor pollicis brevis

Scaphoid bone

Cutaneous branch of the radial nerve

A

The scaphoid bone forms the floor of the anatomical snuffbox. The cutaneous branch of the radial nerve is much more superficially and proximally located.
Anatomical snuffbox

Posterior border Tendon of extensor pollicis longus
Anterior border Tendons of extensor pollicis brevis and abductor pollicis longus
Proximal border Styloid process of the radius
Distal border Apex of snuffbox triangle
Floor Trapezium and scaphoid
Content Radial artery

2000
Q

A 69 year old man is recovering on the ward following a challenging anti reflux operation. The nursing staff are concerned about the patient and he is reviewed by the medical team. His vital signs are as follows:
GCS 13
Respiratory rate: 33
Pulse rate 125
Blood pressure: 95/63
Urine output over past hour: 10mls
There are concerns about blood loss, what is the most likely volume deficit?

500mls

1000mls

1850mls

2250mls

2500mls

A

C-1850 mls

The haemodynamic changes described would equate to class III shock. In the context of fundoplication surgery, splenic injury would be the cause.

Bleeding

The initial response to bleeding, even if of relatively small volume is generalised splanchnic vasoconstriction mediated by activation of the sympathetic nervous system. This process of vasoconstriction is usually sufficient to maintain renal perfusion and cardiac output if the volume of blood lost is small. Over the following hours the circulating fluid volume is restored and normal haemodynamics resume. Loss of greater volumes of blood will typically result in activation in the renin angiotensin system
Where the source of bleeding ceases these physiological measures will restore circulating volume. Ongoing bleeding will result in haemorrhagic shock.
Blood loss is typically quantified by the degree of shock produced as outlined below:
Parameter Class I Class II Class III Class IV
Blood loss ml <750ml 750-1500ml 1500-2000ml >2000ml
Blood loss % <15% 15-30% 30-40% >40%
Pulse rate <100 >100 >120 >140
Blood pressure Normal Normal Decreased Decreased
Respiratory rate 14-20 20-30 30-40 >35
Urine output >30ml 20-30ml 5-15ml <5ml
Symptoms Normal Anxious Confused Lethargic

2001
Q

A 5 year old boy presents to the clinic with short stature suggestive of achondroplasia. What is the genetic basis of this condition?

X linked defect

Y linked defect

YY linked defect

Autosomal dominant defect

Autosomal recessive defect

A

Achondroplasia usually occurs as a sporadic mutation. It is then transmitted in an autosomal dominant fashion.

Achondroplasia

Achondroplasia is a common cause of dwarfism and is caused by defects in the fibroblast growth factor receptor. In most cases (approximately 70%) it occurs as a sporadic mutation. The main risk factor is advancing parental age at the time of conception. Once present it is typically inherited in an autosomal dominant fashion.

Radiological features
Large skull with narrow foramen magnum
Short, flattened vertebral bodies
Narrow spinal canal
Horizontal acetabular roof
Broad, short metacarpals

Treatment
There is no specific therapy. However, some individuals benefit from limb lengthening procedures. These usually involve application of Ilizarov frames and targeted bone fractures. A clearly defined need and end point is the cornerstone of achieving success with such procedures.

2002
Q

Which of the following cranial venous sinuses is unpaired?

Transverse sinus

Superior sagittal sinus

Cavernous sinus

Sigmoid sinus

Inferior petrosal sinus

A

The superior sagittal sinus is unpaired
The superior sagittal sinus is unpaired. It begins at the crista galli, where it may communicate with the veins of the frontal sinus and sometimes with those of the nasal cavity. It arches backwards in the falx cerebri to terminate at the internal occipital protuberance (usually into the right transverse sinus). The parietal emissary veins link the superior sagittal sinus with the veins on the exterior of the cranium.

Cranial venous sinuses

The cranial venous sinuses are located within the dura mater. They have no valves which is important in the potential for spreading sepsis. They eventually drain into the internal jugular vein.

They are:

Superior sagittal sinus
Inferior sagittal sinus
Straight sinus
Transverse sinus
Sigmoid sinus
Confluence of sinuses
Occipital sinus
Cavernous sinus

2003
Q

A 68 year old lady has a 3cm basal cell carcinoma excised from her right cheek. There is a rhomboid defect measuring 4cm by 4cm. What is the best option for managing the wound?

Delayed primary closure

Direct primary closure

Local rotational flap

Split thickness skin graft

Pedicled myocutaneous flap

A

A local rotational flap or full thickness graft would both be acceptable options here.

Tissue reconstruction

Skin Grafts and Flaps
Skin flaps or grafts may be required where primary wound closure cannot be achieved or would entail either significant cosmetic defect or considerable functional disturbance as a result of wound contraction.

Reconstructive ladder
Method Types
Direct closure The simplest option where possible
Grafting techniques
Split thickness
Full thickness
Skin Substitute
Composite
Flap technique Local:
Transposition
Pivot
Alphabetoplasty (e.g. Z-Y)
Regional:
Myocutaneous
Fasciocutaneous
Neurocutaneous
Distant:
Free tissue transfer
Prelamination techniques Allows creation of specialised flaps e.g. buccal mucosa
Tissue expansion Involves placement of tissue expanders to increase amount of tissue at donor sites

Skin Grafts Vs. Flaps
Skin Grafts Flaps
No size limit (Split)/ Relative size limit (full thickness) Size limited by territory of blood supply
Rely on wound bed for blood supply Tissue has its own blood supply
Take better on clean well vascularised wound beds Will survive independent of the wound bed
Split skin graft donor site typically heals in 12 days Direct closure of donor site or secondary skin graft
Donor site may be reused Donor site cannot be reused

Split thickness skin grafts
Available in range of thicknesses.
Thigh is the commonest donor site
Size may be increased by meshing the graft. However this comes with compromise on cosmesis.
Donor sites, especially if thin grafts are taken can be reused following re-epithelialisation

Full thickness grafts
Most commonly used for facial reconstruction
Include dermal appendages
Provide superior cosmetic result

Composite grafts
These are grafts containing more than one tissue type, such as skin and fat. They are usually used to cover small defects in cosmetically important areas.

Flaps
Flaps have their own blood supply and may be pedicled or free.
May have multiple components e.g. skin, skin + fat, skin + fat + muscle.
They will have the ability to take regardless of the underlying tissue bed.
The type of intrinsic blood supply is important. For example in breast surgery pedicled latissimus dorsi flaps will be less prone to failure than microsvascular anastomosed free Diep flaps.

2004
Q

A 73 year old man presents with pain in the right leg. It is most uncomfortable on walking. On examination, he has a deformity of his right femur, which on x-ray is thickened and sclerotic. His serum alkaline phosphatase is elevated, but calcium is within normal limits. What is the most probable underlying diagnosis?

Rickets

Pagets disease

Metastatic lung cancer

Osteoclastoma

Chondrosarcoma

A

Pagets is typically associated with thickened, sclerotic, long bones. Lung carcinoma typically causes lytic bony lesions and hypercalcaemia.

Pagets disease

Paget’s disease is a disease of increased but uncontrolled bone turnover and is characterised by architecturally abnormal bones. It is thought to be primarily a disorder of osteoclasts, with excessive osteoclastic resorption followed by increased osteoblastic activity causing areas of sclerosis and deformity. Paget’s disease is common (UK prevalence 5%) but symptomatic in only 1 in 20 patients

Predisposing factors
increasing age
male sex
northern latitude
family history

Clinical features
bone pain (e.g. pelvis, lumbar spine, femur)
classical, untreated features: bowing of tibia, bossing of skull
raised alkaline phosphatase (ALP) - calcium* and phosphate are typically normal
skull x-ray: thickened vault, osteoporosis circumscripta

Indications for treatment include bone pain, skull or long bone deformity, fracture, periarticular Paget’s
bisphosphonate (either oral risedronate or IV zoledronate)
calcitonin is less commonly used now

Complications
deafness (cranial nerve entrapment)
bone sarcoma (1% if affected for > 10 years)
fractures
skull thickening
high-output cardiac failure

*usually normal in this condition but hypercalcaemia may occur with prolonged immobilisation

2005
Q

Infection with which of the following micro-organisms may result in a clinical picture resembling achalasia of the oesphagus?

Epstein Barr virus

Wuchereria Bancrofti

Candida Spp

Trypanosoma Cruzi

Helicobacter Pylori

A

Infection with Trypanosoma Cruzi may result in destruction of the ganglion cells of the myenteric plexus, resulting in a clinical picture similar to achalasia.

Trypanosoma Cruzi

Protozoan
Causes Chagas disease
Carried by bugs which infect the skin whilst feeding
Penetrate through open wounds and mucous membranes
Intracellular proliferation
Major infective sites include CNS, intestinal myenteric plexus, spleen, lymph nodes and cardiac muscle
Chronic disease is irreversible, nifurtimox is used to treat acute infection

2006
Q

A 28 year old man presents with right upper quadrant pain and hydatid disease is suspected. Which of the following statements relating to the disease is untrue?

First line treatment is with pentavalent antimony.

Peritoneal contamination with active daughter cysts may complicate surgery.

CT scanning of the liver may show a floating membrane.

Biliary communication with the cysts may occur.

It is caused by Echinococcus granulosus.

A

Drug treatment is with albendazole or mebendazole. Praziquantzel may be used in the pre operative stages.

Hydatid cysts

Hydatid cysts are endemic in Mediterranean and Middle Eastern countries. They are caused by the tapeworm parasite Echinococcus granulosus. An outer fibrous capsule is formed containing multiple small daughter cysts. These cysts are allergens which precipitate a type 1 hypersensitivity reaction.

Clinical features are as follows:
Up to 90% cysts occur in the liver and lungs
Can be asymptomatic, or symptomatic if cysts > 5cm in diameter
Morbidity caused by cyst bursting, infection and organ dysfunction (biliary, bronchial, renal and cerebrospinal fluid outflow obstruction)
In biliary rupture there may be the classical triad of; biliary colic, jaundice, and urticaria

CT is the best investigation to differentiate hydatid cysts from amoebic and pyogenic cysts.
Surgery is the mainstay of treatment (the cyst walls must not be ruptured during removal and the contents sterilised first).

2007
Q

A 35-year-old woman presents to the Emergency Department with a second attack of painful frank haematuria. What is the most likely pathology underlying this presentation?
A.Gram negative urinary tract infection
B. Interstitial cystitis
c. Renal cell carcinoma k
d. Transitional cell carcinoma of bladder
e. Urinary tuberculosis

A

• Interstitial cystitis(chronic pelvic pain syndrome):This is a persistent, painful form of chronic cystitis occurring mostly in women.
The etiology of which is unknown. o Some pt with this condition show chronic mucosal ulcers (Hunner ulcers).
o Late in this disease, there is transmural fibrosis leading to a contracted bladder o Interstitial cystitis is characterized by suprapubic pain, hematuria, dysuria, frequency without evidence of bacterial infection.

2008
Q

A 75-year-old insulin-dependent diabetic man has undergone a hemicolectomy. On the first postoperative day he is nil-by-mouth, on subcutaneous insulin, maintenance IV infusion with-Saline and intravenous morphine via patient-controlled analgesia. He is confused after a brief convulsion, has slurred speech and weakness of his right side. His pulse is 110 beats/minute, respiratory rate 25 breaths/minute, blood pressure 160/95 mmHg and Sp02 95% on room air.
The most likely cause of this convulsion and confusion is: (Sep 2016)
a. hypoglycemia
b. hyponatremia
c. inhibition of noradrenaline re-uptake
d. stimulation of a opioid receptors
e. stimulation of GABA receptors

A

A-Hypoglycemia

Early adrenergic symptoms: Pallor, Diaphoresis, Shakiness, Hunger, Anxiety, Irritability, Headache, Dizziness

Neuroglycopenic signs: Confusion, Slurred speech, Irrational or uncontrolled behavior, Disorientation, Loss of consciousness, Seizures, Pupillary sluggishness, Decreased response to noxious stimuli

2009
Q

A 30-year-old woman presents to the general surgical clinic with a 1.5 cm cervical lymph node along the anterior border of sternocleidomastoid muscle.-Clinical examination and routine blood tests are unremarkable. She undergoes an excisional biopsy of the lump as a day case. The histology report reveals encapsulated infiltrative carcinoma with marked fibrosis and cystic changes within the lymph node. What is the most likely primary pathology?
(Sep 2016)
a. Anaplastic thyroid carcinoma
b. Follicular thyroid carcinoma
c. Malignant lymphoma of the thyroid
d. Medullary thyroid carcinoma
e. Papillary thyroid carcinoma

A

Carcinoma of Thyroid
Type: Papillary Spread: Lymphatic, to local nodes (%): 60-70 Age: young adults 20-40 (<45y), Prognosis: Excellent

Follicular: Blood stream, especially to bone, % 20-25, Young-middle 40-50 (>45), Good with radio-iodine therapy.

Anaplastic: Aggressive local extension, % 10-15, Elderly, Very poor

Medullary (C-cells): Local, lymphatic, blood stream, % 5-10, Usually elderly, but familial cases occur, Variable. More aggressive in familial cases

2010
Q

A 37-year-old man presents with severe headache, photophobia arid neck stiffness.
Karnig’s sign is positive and a midline lumbar puncture is performed immediately to determine if a cerebrospinal fluid (CSF) pathogen is involved. In performing this procedure, which is ‘he first of the following structures to be pierced by the lumbar puncture needle? (Sep 2016)
a. Anterior longitudinal ligament
b. Arachnoid mater
c. Dura mater d.
Interspinous ligament
e. Posterior longitudinal ligament

A

Layers traversed by the spinal needle (posterior to anterior)

• Skin
• Subcutaneous tissue
• Supraspinous ligament
• Interspinous ligament
• Ligamentum flavum
• Duramater
• Sub dural space
• Arachnoidmater
• Subarachnoid space

2011
Q

A 26-year-old woman presents with severe headache, photophobia and neck stiffness. Among other investigations, a lumbar puncture is earned out and cerebrospinal fluid is sent for microscopy and culture. Culture shows a growth of Neisseria meningitidis. What is likely to be the predominant-cell type in the cerebrospinal Fluid? (Sep 2016)
a. Basophils
b. Eosinophils
c. Lymphocytes
d. Macrophages
e. Neutrophils

A

In a case of meningitis caused by Neisseria meningitidis, the predominant cell type in the cerebrospinal fluid (CSF) is typically:

e. Neutrophils

Neutrophils are the primary white blood cells that respond to bacterial infections. In bacterial meningitis, the rapid accumulation of neutrophils in the CSF is a characteristic feature.

Source:

•	Tunkel, A. R., & Scheld, W. M. (1995). Acute bacterial meningitis. The Lancet, 346(8981), 1675-1680. Link
2012
Q

A 70-year-old woman from a nursing home presents to the! Emergency Department with abdominal pain and vomiting. On examination, she is dehydrated and her abdomen is distended. There is a 3 cm 4 cm swelling in her right groin which is non-tender, and there is no cough impulse. At operation, a femoral Hernia found. Which of the following lies medial to the neck of the hernia?
a. Femoral artery
b. Femoral nerve
c. Femoral vein
d. Lacunar ligament
e. Pectineal ligament

A

Boundaries of femoral ring
• Anterior: Inguinal ligament
• Posterior: Ligament of Cooper ( pectineal ligament).
• Medial: Lacunar ligament (Gimbernat’s ligament)
• Lateral: Thin septum which separates the femoral canal from femoral vein (silver fascia).

2013
Q

A 26-year-old man presents to the Emergency Department with extensive bleeding from his arm after sustaining a glass injury. On examination there is a 7 cm transverse laceration across the anterior aspect of his elbow. On exploring the cubital fossa you would expect the brachial artery to be: (Sep 2016)
a. anterior to the median nerve
b. lateral to the biceps tendon
c. lateral to the median nerve
d. medial to the median nerve
e. superficial to the bicipital aponeurosis

A

Lateral to the median nerve

2014
Q

A 52-year-old man is found to have multiple myeloma. What skull X-ray appearances would
a. Diffuse thickening of the calvarium
b. Hair on end appearance
c. Multiple fractures
d. Multiple osteolytic lesions
e. Multiple osteosclerotic lesions

A

MULTIPL MYELOMA MNEMONIC: OLD CRAB

OLD - Old Age
C - Calcium Elevated (Hypercalcemia)
R - Renal Failure
A - Anemia
B - Bone Lytic Lesions

2015
Q

A 50-year-old woman presents with a history of right upper quadrant pain and jaundice.
She reports that her urine was dark in color and that her stools are offensive and difficult to flush. Which of the following explains the dark urine? (Sep 2016)
a. increase in conjugated bilirubinuria
b. Increase in unconjugated bilirubinuria
c. Increase in urea excretion
d. Increase in urinary urobilinogen
e. Reduced enterohepatic bile salt circulation

A

Urine bilirubin is normally absent.
• When it is present, only conjugated bilirubin is passed into the urine.
• This may be evidenced by dark-colored urine seen in patients with obstructive jaundice or jaundice due to hepatocellular injury..
• The lack of bilirubin in the intestinal tract is responsible for the pale stools typically associated with biliary obstruction.
• The cause of pruritus associated with biliary obstruction is due to accumulation of bile salts in the skin.

2016
Q

A 76-year-old woman falls and sustains an inter-trochanteric fractured neck of femur
Following operative fixation her recovery is prolonged and she is discharged to a rehabilitation unit Six weeks after her original operation she is readmitted. She is drowsy, hypotensive and bradycardic. An ECG shows low voltage complexes and a prolonged QT interval Under-activity of which of the following glands gives the best explanation of the clinical picture?
a. Adrenal
b. Pancreas
c. Parathyroid
d. Pituitary
e. Thyroid

A

The clinical picture described—drowsiness, hypotension, bradycardia, low voltage complexes, and a prolonged QT interval on the ECG—suggests that the underlying issue is likely related to hypothyroidism. Therefore, the best explanation for her symptoms is under-activity of the thyroid gland.

Hypothyroidism can present with several of the symptoms described:

•	Drowsiness and lethargy
•	Hypotension
•	Bradycardia
•	Low voltage ECG complexes
•	Prolonged QT interval

These are all common manifestations of severe hypothyroidism, which can be confirmed with a thyroid function test measuring levels of TSH (thyroid-stimulating hormone) and thyroid hormones (T3 and T4).

In contrast:

•	Adrenal insufficiency typically presents with symptoms such as fatigue, muscle weakness, weight loss, hypotension, and sometimes hyperpigmentation, but the ECG findings are less specific.
•	Pancreatic issues are more related to insulin and glucose metabolism, presenting with symptoms of diabetes or hypoglycemia.
•	Parathyroid gland issues would primarily affect calcium levels, leading to symptoms like muscle cramps or tetany, and would not typically cause the described ECG changes.
•	Pituitary gland under-activity (hypopituitarism) can have a variety of symptoms depending on which hormones are deficient, but the symptoms described are more directly linked to thyroid hormone deficiency.

Thus, the thyroid gland’s under-activity is the most likely explanation for her clinical presentation.

2017
Q

A 35-year-old woman presents with recurrent peptic ulceration. She is on proton pump inhibitors and previously received Helicobacter pylori eradication therapy three months ago.
Which of the following is likely to be raised on venous blood testing?
a. Cholecystokinin
b. Gastrin
c. Histamine
d. Pancreozymin
e. Secretin

A

GASTRINOMA

• -Syndrome: Zollinger-Ellison
• -Hormone: gastrin
, -Clinical features: abdo pain, peptic ulceration, diarrhoea, gastric hypersecretion
•-site: duodenum - 70%; pancreas - 25%
• - >50% malignant
• -Diagnosis: increased serum gastrin
> 1000pg/ml
basal acid output > 15mEq/hr
• -localization: CT/MRI/US -less useful
• -Somatostatin receptor scin. - sensitive test

2018
Q

A 68-year-old man with insulin-dependent diabetes presents to the preadmission clinic prior to surgery for rectal carcinoma. His creatinine is found to be 590 pmol/L. He is referred to a renal unit where a measure of his glomerular filtration rate (GFR) is done. Which of the following best describes why inulin could be used to measure his GFR?

a. It is filtered and not reabsorbed
b. It is filtered and reabsorbed
c. It is filtered, secreted and reabsorbed
d. It is filtered, secreted but not reabsorbed
e. It is not filtered but is secreted and not reabsorbed

A

Inulin is used to measure the glomerular filtration rate (GFR) because it has very specific properties that make it ideal for this purpose. The correct description of why inulin can be used to measure GFR is:

a. It is filtered and not reabsorbed.

Inulin is a polysaccharide that, when introduced into the body, is freely filtered by the glomeruli in the kidneys. Importantly, once it is filtered, it is neither reabsorbed nor secreted by the renal tubules. This means that the amount of inulin in the urine accurately reflects the amount that was filtered by the glomeruli, allowing for an accurate measurement of GFR.

2019
Q

A 60-year-old man presents with a one-month history of headache, confusion and recent onset of right sided hemiparesis. A CT scan demonstrates a 4 cm >- 4 cm lesion with central necrosis in the left frontal lobe. It is seen extending across the midline along the corpus callosum. There is extensive oedema around the lesion. Which of the following is the most likely diagnosis of this lesion?
a. Central neurocytoma
b. Ependymoma
c. Glioblastoma
d. Meningioma
e. Oligodendroglioma

A

The presentation of a 60-year-old man with a headache, confusion, right-sided hemiparesis, and a CT scan showing a 4 cm x 4 cm lesion with central necrosis in the left frontal lobe extending across the midline along the corpus callosum, with extensive edema, is most suggestive of a glioblastoma.

Glioblastoma (also known as glioblastoma multiforme or GBM) is the most aggressive and common primary brain tumor in adults. It is characterized by rapid growth and a tendency to infiltrate surrounding brain tissue, often crossing the midline via structures such as the corpus callosum. The central necrosis and significant edema are typical radiological features of glioblastoma.

2020
Q

A 26-year-old man is admitted to the Emergency Department with multiple peripheral fractures. He is clinically shocked. Which is the structure responsible for the first hemostatic response to a fall in systemic arterial blood pressure?
a. Adenohypophysis
b. Baroreceptor
c. Chemoreceptor
d. Kidney
e. Neurohypophysis

A

The structure responsible for the first hemostatic response to a fall in systemic arterial blood pressure is the baroreceptor.

Explanation:

Baroreceptors are specialized sensors located primarily in the carotid sinuses and the aortic arch. They detect changes in blood pressure and respond by sending signals to the central nervous system to initiate corrective measures. When a fall in blood pressure is detected, baroreceptors stimulate the autonomic nervous system to increase heart rate, contractility, and vasoconstriction, thereby raising blood pressure.

Options Breakdown:

a. Adenohypophysis - The adenohypophysis, or anterior pituitary gland, is involved in hormone secretion but not directly in the immediate response to blood pressure changes.
b. Baroreceptor - Correct answer. Baroreceptors respond quickly to changes in blood pressure to maintain homeostasis.
c. Chemoreceptor - Chemoreceptors respond to changes in blood chemistry (e.g., oxygen, carbon dioxide, pH levels), but they are slower and secondary to the baroreceptors in responding to blood pressure changes.
d. Kidney - The kidney plays a role in long-term blood pressure regulation through the renin-angiotensin-aldosterone system, but it is not the first response mechanism.
e. Neurohypophysis - The neurohypophysis, or posterior pituitary gland, releases hormones like vasopressin (antidiuretic hormone), which can affect blood pressure but is not the immediate response.

Thus, the correct answer is:
b. Baroreceptor.

& Chemoreceptors
Baroreceptors:
• Hypotension- Jarterial pressure — stretch → afferent baroreceptor firing
Tefferent sympathetic firing and I efferent parasympathetic stimulation
→ vasoconstriction, T HR,1 contractility→ 1 BP important in response to severe hemorrhage
• Carotid massage - pressure on carotid artery 1 stretch → 1 afferent baroreceptor firing → HR Can by tried for Tachycardia (SVT)
• Contributes to Cushing reaction (triad of hypertension, bradycardia, and respiratory depression) intracranial pressure constricts arterioles → cerebral ischemia and reflex sympathetic increase in perfusion pressure (hypertension) → 1 stretch → reflex baroreceptor induced-bradycardia
Chemoreceptors:
• Peripheral-carotid and aortic bodies are stimulated by + POz (< 60 mm Hg), TPCOz, and I pH of blood
• Central-are stimulated by changes in pH and PCO2 of brain interstitial fluid, which in turn are influenced by arterial COz Do not directly respond to POz

2021
Q

Vitamin K is required for normal blood clotting. Which one of the following statements is true about the effects and availability of vitamin K?
a. Affects platelet function
b. Causes bleeding if taken in excess
c. Is provided only by fresh food
d. Is depleted by broad spectrum oral antibiotics
e. Vitamin K absorption is affected by resection of the terminal ileum

A

Vitamin K is crucial for the synthesis of certain proteins required for blood clotting. Among the given statements, the true statement about the effects and availability of vitamin K is:

•	Vitamin K absorption is affected by resection of the terminal ileum.

Here is an explanation for each option:

1.	Affects platelet function: This is incorrect. Vitamin K is involved in the synthesis of clotting factors, not directly in platelet function. Platelets are cell fragments that play a role in blood clotting, but their function is not directly influenced by vitamin K.
2.	Causes bleeding if taken in excess: This is incorrect. Vitamin K toxicity is rare and usually occurs with synthetic forms of vitamin K. Excessive vitamin K typically does not cause bleeding; rather, a deficiency in vitamin K leads to bleeding issues.
3.	Is provided only by fresh food: This is incorrect. While fresh foods like leafy green vegetables are rich in vitamin K, it is also produced by gut bacteria and can be found in fermented foods and some animal products.
4.	Is depleted by broad spectrum oral antibiotics: This is partially correct. Broad-spectrum antibiotics can reduce gut bacteria that produce vitamin K, potentially leading to a deficiency. However, this depletion is not as direct or immediate as the absorption being affected by the resection of the terminal ileum.
5.	Vitamin K absorption is affected by resection of the terminal ileum: This is correct. The terminal ileum is the site where bile acids are reabsorbed, and these bile acids are essential for the absorption of fat-soluble vitamins, including vitamin K. If the terminal ileum is resected, the absorption of vitamin K can be significantly impaired.

Therefore, the correct statement is: Vitamin K absorption is affected by resection of the terminal ileum.

2022
Q

In septic shock, norepinephrine (noradrenaline) is used to increase the systemic vascular resistance. This action is a result of the stimulation of which of the following receptors?
a. alfa1
b. alfa2
C. Beta1
d. Delta 1

A

In septic shock, norepinephrine (noradrenaline) is used to increase the systemic vascular resistance through the stimulation of alpha-1 (α1) receptors.

The correct answer is option A: alfa1

Norepinephrine is a potent alpha-1 adrenergic receptor agonist, and the stimulation of these receptors on the vascular smooth muscle cells leads to vasoconstriction, which increases the systemic vascular resistance (SVR) and helps maintain blood pressure in septic shock.

Alpha-1 receptors are found on the vascular smooth muscle cells, and their activation by norepinephrine causes the smooth muscle to contract, resulting in vasoconstriction and an increase in SVR.

The other options are incorrect:
- Alpha-2 (α2) receptors are primarily involved in inhibiting the release of norepinephrine from presynaptic nerve terminals and have a lesser effect on vascular smooth muscle contraction.
- Beta-1 (β1) receptors are found primarily in the heart and mediate positive inotropic and chronotropic effects.
- There is no known “Delta 1” receptor subtype involved in this mechanism.

Therefore, in septic shock, norepinephrine is used to increase the systemic vascular resistance through the stimulation of alpha-1 (α1) receptors on the vascular smooth muscle cells, leading to vasoconstriction.​​​​​​​​​​​​​​​​

2023
Q

A 10-year-old child presents with progressive facial weakness and a squint. On examination there is a lower motor neuron facial weakness and failure of abduction of one eye.
An MRI scan shows a small, intrinsic mass lesion within the central nervous system. Where is this most likely to be located?
a. The cerebellar vermis
b. The cerebellar tonsil
c. The medulla
d. The midbrain
e. The pons

A

Based on the clinical presentation of progressive facial weakness (lower motor neuron type) and failure of abduction of one eye, along with the finding of a small intrinsic mass lesion within the central nervous system on MRI, the most likely location of the lesion is the pons.

The correct answer is e. The pons.

Here’s the explanation for each option:

a. The cerebellar vermis is unlikely to cause these specific cranial nerve deficits.
b. The cerebellar tonsil is also an unlikely location for this presentation.
c. The medulla is responsible for lower cranial nerve function, but the facial nerve nucleus is located in the pons, making it less likely to be the site of the lesion.
d. The midbrain is a possible location, as it contains the nuclei for the oculomotor nerve (CN III) which controls eye movements, including abduction. However, the facial nerve nucleus is located in the pons.
e. The pons is the most likely location for the lesion because it contains the nuclei for both the facial nerve (CN VII) and the abducens nerve (CN VI), which control facial movements and lateral eye movement (abduction), respectively.

The combination of lower motor neuron facial weakness and failure of abduction of one eye strongly suggests a lesion affecting the facial nerve nucleus and the abducens nerve nucleus, both of which are located in the pons.

Therefore, based on the clinical presentation and MRI findings, the most likely location of the small, intrinsic mass lesion within the central nervous system is the pons.​​​​​​​​​​​​​​​​

2024
Q

A 40-year-old woman presents with a parotid tumour. A biopsy reveals extensive perineural invasion. Which is the most likely pathology?
a. Acinic cell carcinoma
b. Adenocarcinoma
c. Adenoid cystic carcinoma
d. Lymphoma
e. Pleomorphic salivary adenoma

A

The most likely pathology for a parotid tumor with extensive perineural invasion is:

c. Adenoid cystic carcinoma

Here’s why:

Perineural invasion: Adenoid cystic carcinoma is known to be highly aggressive and has a propensity for perineural invasion, which means the tumor cells spread along nerves. [4]

Parotid gland: Adenoid cystic carcinoma is one of the most common malignant tumors of the salivary glands, including the parotid gland. [4]

Other options:

Acinic cell carcinoma and adenocarcinoma are less common and may show perineural invasion, but not to the same extent as adenoid cystic carcinoma. [2]
Lymphoma and pleomorphic salivary adenoma are less likely to exhibit extensive perineural invasion.
While a biopsy is necessary for definitive diagnosis, considering the patient’s age, location of the tumor (parotid gland), and the presence of extensive perineural invasion, adenoid cystic carcinoma is the most probable pathology.

2025
Q

A 50-year-old man presents with malaise, abdominal pain, weight loss, fever and myalgia.
Polyarteritis nodosa is associated with all of the following except:
a. Erythema nodosum
b. livedo reticularis
c. mononeuritis multiplex
d. nailfold infarcts
e. palpable purpura

A

A Erythema nodosum

2026
Q

A 30-year-old woman undergoes subtotal thyroidectomy. Five days later the wound appears red and inflamed. Which of the following is the most likely causative organism?
a. Hemophilus influenzae
b. Proteus-mirabilis
c. Pseudomonas aeruginosa
d. Staphylococcus aureus
e. Streptococcus pyogenes

A

The most likely causative organism is:

Staphylococcus aureus
Here’s why:

Staphylococcus aureus is a common cause of skin and wound infections, especially post-surgical wounds.
It lives on the skin and in the nose of many healthy people and can easily enter a wound site during surgery.
Symptoms of a Staphylococcus aureus infection often include redness, inflammation, and pus.
The other options are less likely:

Hemophilus influenzae typically causes respiratory infections.
Proteus mirabilis is less common than Staphylococcus aureus in surgical site infections.
Pseudomonas aeruginosa is more commonly associated with hospital-acquired infections and immunocompromised patients.
Streptococcus pyogenes can cause skin infections, but Staphylococcus aureus is more frequent in this context.
It’s important to note that definitively diagnosing the infection would require obtaining a wound culture from a healthcare professional.

2027
Q

A 50-year-old man complains of recent onset diplopia. On examination he is unable to look in wards and downwards with his right eye; The most likely diagnosis is a lesion of the?
a. abducens nerve
b. ciliary ganglion
c. nasociliary nerve
d. oculomotor nerve
e. trochlear nerve

A

The most likely diagnosis is a lesion of the trochlear nerve.

The trochlear nerve (cranial nerve IV) innervates the superior oblique muscle, which is responsible for enabling the eye to look downwards and inwards. A lesion of the trochlear nerve would result in the inability to perform these movements, leading to diplopia (double vision) when attempting to look down and in. This matches the symptoms described in the patient.

The other nerves and structures mentioned do not fit as well with the specific movement deficits and symptomatology described:

•	The abducens nerve (cranial nerve VI) innervates the lateral rectus muscle, which abducts the eye.
•	The ciliary ganglion is involved in pupil constriction and lens accommodation.
•	The nasociliary nerve is a branch of the ophthalmic nerve (V1), and is primarily involved in sensation.
•	The oculomotor nerve (cranial nerve III) controls most of the other extraocular muscles, except for the superior oblique and lateral rectus.

Therefore, based on the inability to look downwards and inwards specifically with the right eye, the trochlear nerve lesion is the most likely diagnosis.

2028
Q

A 57-year-old heart transplant recipient is keen-to join the cardiac, rehabilitation program. Which of the following factors is most likely to increase cardiac output in this patient during moderate exercise?
Decreased negative intrathoracic pressure
Decreased venous tone
Decreased ventricular compliance
Increased atrial filling ITS
Increased intrapericardial Pressure

A

In a heart transplant recipient, the most likely factor to increase cardiac output during moderate exercise is increased atrial filling.

Here’s why the other factors are less likely to increase cardiac output in this context:

•	Decreased negative intrathoracic pressure: This would typically decrease venous return to the heart, which would not favor an increase in cardiac output.
•	Decreased venous tone: This would lead to less venous return and thus less filling of the heart, not more.
•	Decreased ventricular compliance: This would make the heart less able to fill properly, thereby decreasing cardiac output.
•	Increased intrapericardial pressure: This would compress the heart and limit its ability to fill, reducing cardiac output.

Increased atrial filling leads to an increased preload, which, according to the Frank-Starling mechanism, can enhance cardiac output by increasing stroke volume. This is particularly important in heart transplant patients, where the heart relies more on intrinsic mechanisms to regulate cardiac output due to denervation.

2029
Q

A 69-year man with rectal cancer undergoes low anterior resection. The liver is disease free. The pathology of his tumour was reported as a large tumour invading from the rectum into the mesorectum. Two nodes of 24 lymph nodes, close to the tumour were carcinoma on histology. The tumour was completely excised. What is the correct pathological staging of his
a. Dukes’ A
b. Dukes’ B
C. Dukes’ C
d. Dukes’ D
e. T2 N1

A

The correct pathological staging of his tumor is:

Dukes’ C

Here’s the reasoning:

•	Dukes’ A: The cancer is limited to the mucosa (innermost layer) of the bowel wall.
•	Dukes’ B: The cancer has grown through the muscle layer of the bowel wall and possibly into nearby tissues but has not spread to the lymph nodes.
•	Dukes’ C: The cancer has spread to one or more nearby lymph nodes, regardless of how far it has grown into the layers of the bowel wall.
•	Dukes’ D: The cancer has spread to distant parts of the body (metastasized).

Given that the pathology report indicates that the tumor has invaded from the rectum into the mesorectum and involved two lymph nodes (out of 24 examined), it fits the criteria for Dukes’ C. The tumor has spread to nearby lymph nodes, which is the defining characteristic of Dukes’ C.

The T2 N1 staging refers to the TNM (Tumor, Node, Metastasis) system:

•	T2: The tumor has grown into the muscularis propria but not through it.
•	N1: 1-3 regional lymph nodes are involved.

However, based on the options given and the provided details, Dukes’ C is the correct staging.

2030
Q

A 21-year-old man is admitted to the Emergency Department with a stab injury to his right chest. Pulse rate is 110 beats/minute and blood pressure is 85/40 mmHg. Chest X-ray shows a large right haemothorax and a very small light apical pneumothorax. Which is the first substance secreted in the process leading to increased renal reabsorption of sodium in response to the above injury?
a. Angiotensin I
b. Angiotensin 11
c. Angiotensinogen
d. Antidiuretic hormone
e. Renin

A

The first substance secreted in the process leading to increased renal reabsorption of sodium in response to the above injury is:

Renin

Here’s the reasoning:

In response to a significant drop in blood pressure (as seen in this patient with a pulse rate of 110 beats/minute and blood pressure of 85/40 mmHg), the kidneys activate the renin-angiotensin-aldosterone system (RAAS) to increase blood pressure and maintain fluid balance.

1.	Renin: Secreted by the juxtaglomerular cells of the kidneys in response to low blood pressure, low sodium content in the distal tubule, or sympathetic nervous system activation.
2.	Renin then converts angiotensinogen (produced by the liver) into angiotensin I.
3.	Angiotensin I is converted into angiotensin II by the enzyme angiotensin-converting enzyme (ACE), primarily in the lungs.
4.	Angiotensin II has several effects, including vasoconstriction and stimulating the secretion of aldosterone from the adrenal cortex, which increases sodium reabsorption in the kidneys.

Therefore, the process begins with the secretion of renin, which is the first step in the chain leading to increased sodium reabsorption in response to the injury and resultant hypotension.

2031
Q

During the second (proliferative) phase of wound healing the predominant cells in the wound site are fibroblasts. This cell is of mesenchymal origin and produces the matrix and collagen needed to strengthen the scar. Cross linkage of collagen requires hydroxy-proline and hydroxy-lysine residues, which requires a specific vitamin to be available in sufficient quantities. Deficiency of which vitamin results in collagen that is unstable?
a. Vitamin B2 (riboflavine)
b. Vitamin B6 (pyridoxine)
c. Vitamin C (ascorbic acid)
d. Vitamin D (cholecalciferol)
e. Vitamin E (tocopherol)

A

Deficiency of Vitamin C (ascorbic acid) results in collagen that is unstable.

Vitamin C is crucial for the hydroxylation of proline and lysine residues in collagen. These hydroxylation reactions are necessary for the proper cross-linking and stability of collagen fibers. Without sufficient Vitamin C, the collagen produced is defective and weak, leading to issues in wound healing and overall tissue integrity. This condition is known as scurvy.

2032
Q

During the second (proliferative) phase of wound healing the predominant cells in the wound site are fibroblasts. This cell is of mesenchymal origin and produces the matrix and collagen needed to strengthen the scar. Cross linkage of collagen requires hydroxy-proline and hydroxy-lysine residues, which requires a specific vitamin to be available in sufficient quantities. Deficiency of which vitamin results in collagen that is unstable?
Vitamin B2 (riboflavine)
Vitamin B6 (pyridoxine)
Vitamin C (ascorbic acid)
Vitamin D (cholecalciferol)
Vitamin E (tocopherol)

A

Deficiency of Vitamin C (ascorbic acid) results in collagen that is unstable.

Vitamin C is crucial for the hydroxylation of proline and lysine residues in collagen. These hydroxylation reactions are necessary for the proper cross-linking and stability of collagen fibers. Without sufficient Vitamin C, the collagen produced is defective and weak, leading to issues in wound healing and overall tissue integrity. This condition is known as scurvy.

2033
Q

A 62-year-old woman presents with a firm irregular mass in the upper outer quadrant of the right breast. This appears malignant on mammography, and fine needle aspiration cytology is reported as Co. Which is the first lymph node to which the tumour is most likely to
needler is rost likely to
metastasis?
a. Initial node
b. Primary node
c. Secondary node
d. Sentinel node
e. Virchow’s node

A

The first lymph node to which the tumor is most likely to metastasize is the:

Sentinel node

The sentinel node is the first lymph node to which cancer cells are most likely to spread from a primary tumor. It is identified because it is the initial node in the lymphatic drainage path from the tumor site. In breast cancer, identifying and examining the sentinel node helps determine the extent of cancer spread and guides treatment decisions.

2034
Q

Which of the following statements is true with relation to cardiac muscle?
a. Contraction does not involve release of Ca by the sarcoplasmic reticulum
b. Contraction is triggered by an influx of Ca through the sarcolemma
c. Excitation is passed from one cardiac cell to another through desmosomes
d. Hypertrophy is brought about by division of existing cardiac cells
e. There is capacity for regeneration

A

The true statement with relation to cardiac muscle is:

b. Contraction is triggered by an influx of Ca through the sarcolemma

Here’s why the other statements are not true:

•	a. Contraction does not involve release of Ca by the sarcoplasmic reticulum: This is false. Cardiac muscle contraction does involve the release of calcium from the sarcoplasmic reticulum. The initial influx of calcium through the sarcolemma triggers further release of calcium from the sarcoplasmic reticulum in a process known as calcium-induced calcium release (CICR).
•	c. Excitation is passed from one cardiac cell to another through desmosomes: This is false. Excitation is passed from one cardiac cell to another through gap junctions, not desmosomes. Gap junctions allow for the direct electrical communication between cardiac muscle cells.
•	d. Hypertrophy is brought about by division of existing cardiac cells: This is false. Cardiac hypertrophy (an increase in the size of cardiac cells) is not due to cell division but rather due to the enlargement of existing cardiac cells.
•	e. There is capacity for regeneration: This is largely false. Cardiac muscle cells have very limited capacity for regeneration. Most cardiac repair is due to fibrosis and scarring rather than regeneration of cardiac muscle cells.
2035
Q

A 66-year-old woman with known tumour of her spine has started to retain urine and is experiencing decreased anal and rectal tone. The tumour is pressing directly on the conus medullaris. At which one of the following vertebral levels is the tumour most likely to be located?
T9/T10
T11/T12
L1/L2
L3/C4
L5/S1

A

The tumor pressing directly on the conus medullaris is most likely to be located at:

L1/L2

The conus medullaris, the tapered end of the spinal cord, typically ends at the level of the L1 or L2 vertebrae in adults. Compression at this level would explain the urinary retention and decreased anal and rectal tone observed in the patient, which are symptoms of conus medullaris syndrome.

2036
Q

A 26-year-old man presents with a two-month history of unilateral testicular swelling. An ultrasound scan shows a heterogeneous mass within the testis with surrounding fluid. His blood test show- an elevated alpha fetoprotein level. Which of the following is the most likely diagnosis?
a. Lymphoma
b. Orchitis
c. Seminoma
d. Teratoma
e. Tuberculosis

A

The most likely diagnosis for a 26-year-old man with a two-month history of unilateral testicular swelling, a heterogeneous mass within the testis with surrounding fluid on ultrasound, and elevated alpha fetoprotein (AFP) level is:

Teratoma

Explanation:

•	Testicular teratomas are germ cell tumors that can present with a heterogeneous mass and elevated levels of alpha fetoprotein (AFP).
•	Lymphoma typically does not elevate AFP and often presents in older men.
•	Orchitis, an inflammation of the testis, usually presents with pain and is not associated with elevated AFP.
•	Seminomas are also a type of germ cell tumor but typically do not cause an elevation in AFP; they usually elevate beta-hCG.
•	Tuberculosis of the testis is rare and would not typically cause elevated AFP.

Thus, the combination of a testicular mass and elevated AFP levels strongly suggests a diagnosis of teratoma.

2037
Q

A A 50-year-old woman presents with a history of faecal incontinence over the past few year She had a prolonged and difficult first stage of labor 20 years previously. Physical examination reveals a relatively lax anal sphincter. Which nerve is likely to have been damaged in labour?
a. Autonomic nerves to the rectum
b. Genitofemoral nerve
c. Lumbosacral trunk
Dorsal nerve
d. Obturator nerve
e. Pudendal nerve

A

The nerve likely to have been damaged in labor resulting in faecal incontinence and a relatively lax anal sphincter is the pudendal nerve.

The pudendal nerve is crucial for maintaining the tone of the external anal sphincter. Damage to this nerve during prolonged and difficult labor can result in weakness or laxity of the sphincter, leading to incontinence issues.

2038
Q

A 70-year-old man is admitted to hospital with a 12-hour history of a painful white leg. A femoral artery embolus is shown on angiography. He is an insulin-controlled diabetic and takes ibuprofen for longstanding osteoarthritis. Twenty-four hours after emergency
aftare brenge urine. A
embolectomy his blood pressure is 90/60 mmHg and he passes very dark brown urine. A dipstick shows myoglobin and traces of blood. His blood glucose is 15 mmol/L (normal- 4.0-
6.0), urea 12 mmol/L (normal 3.2-7.5) and creatinine 180 mmol/L (normal 35-110). What is the most likely cause of his abnormal renal function?
a. Drug induced nephropathy
b. Hypovolemia
c. Methaemoglobinuria
d. Rhabdomyolysis
e. Uncontrolled diabetes

A

The most likely cause of the patient’s abnormal renal function is rhabdomyolysis.

Here’s why:

•	Rhabdomyolysis occurs when muscle tissue breaks down and releases myoglobin into the bloodstream, which can cause kidney damage.
•	The presence of dark brown urine and a positive dipstick for myoglobin suggests muscle breakdown.
•	The patient’s recent femoral artery embolus and subsequent surgery likely caused ischemia to the leg muscles, leading to muscle necrosis and release of myoglobin.
•	Elevated urea and creatinine levels indicate acute kidney injury, which can be secondary to rhabdomyolysis.
•	Although the patient is a diabetic and takes ibuprofen, which can contribute to renal issues, the acute presentation and clinical context point more strongly towards rhabdomyolysis as the primary cause.
2039
Q

An 86-year-old woman, with a history of partial gastrectomy, is in the high dependency unit for a chest infection that is being treated with antibiotics. She complains of difficulty swallowing. Endoscopy shows multiple white plaques on the mucosal surface of the esophagus. Biopsy is most likely to show:
a. acid reflux esophagitis
b. Barrett’s esophagitis
c. candida esophagitis
d. eosinophilic esophagitis
e. herpes esophagitis

A

The biopsy is most likely to show candida esophagitis.

Here’s why:

•	The presence of multiple white plaques on the mucosal surface of the esophagus is characteristic of candida esophagitis.
•	Candida esophagitis is a common infection in immunocompromised individuals, including elderly patients and those on antibiotics, which can disrupt normal flora and allow candida overgrowth.
•	The patient’s history of partial gastrectomy may also contribute to altered esophageal environment, further predisposing her to candida infection.

Other conditions, such as acid reflux esophagitis, Barrett’s esophagitis, eosinophilic esophagitis, and herpes esophagitis, have different characteristic endoscopic and histological features. Candida esophagitis is specifically associated with white plaques.

2040
Q

A 65-year-old woman has been given a total of 15 mg of morphine/over a 30-minute period for pain relief following a femoral neck fracture, mellowing observations are found: Sp02 on 80% oxygen is 80%; blood pressure 90/60 respiratory rate 6 breaths/minute.
The immediate intervention should be:
a. high flow oxygen
b. intravenous fluid replacement
c] intravenous Naloxone
d. monitor her ECG
e. tracheal intubation

A

Based on the additional information provided, the correct answer for the immediate intervention in this scenario should be:

e) Tracheal intubation

Explanation:

Given that the patient has a respiratory rate of only 6 breaths per minute, which is extremely low and indicative of life-threatening respiratory failure, tracheal intubation and mechanical ventilation are likely necessary as the immediate intervention.

While administering intravenous naloxone (option c) was initially the appropriate response to reverse the respiratory depression caused by the morphine overdose, a respiratory rate as low as 6 breaths per minute suggests that more aggressive airway management and ventilatory support are required.

Intubation will secure the patient’s airway and allow for mechanical ventilation to take over the work of breathing and ensure adequate oxygenation and ventilation until the naloxone takes effect and the patient’s respiratory drive improves.

The other options, such as high-flow oxygen (option a), intravenous fluid replacement (option b), and ECG monitoring (option d), are supportive measures but do not directly address the critical need for airway protection and ventilatory support in this case.

Therefore, based on the information provided, the correct immediate intervention for a patient with a respiratory rate of 6 breaths per minute due to a morphine overdose is tracheal intubation (option e).​​​​​​​​​​​​​​​​

2041
Q

The following drugs are used to treat patients with peripheral vascular disease. They all reduce the rate of myocardial infarction and stroke except:
a. ACE inhibitors
b. antihypertensive medication
c. aspirin
d. cholesterol lowering drugs
e. oxypentifylline

A

The drug that does not reduce the rate of myocardial infarction and stroke in patients with peripheral vascular disease is oxypentifylline (also known as pentoxifylline).

Here’s why:

•	ACE inhibitors, antihypertensive medications, aspirin, and cholesterol-lowering drugs (such as statins) have all been shown to reduce the risk of myocardial infarction (MI) and stroke by managing blood pressure, reducing clot formation, and lowering cholesterol levels.
•	Pentoxifylline is used to improve blood flow in peripheral vascular disease by decreasing blood viscosity and improving red blood cell flexibility, but it does not have the same evidence supporting a reduction in the rates of MI and stroke as the other medications listed.

Therefore, e. oxypentifylline is the correct answer.

2042
Q

A 4-year-old boy, is brought to the Emergency Department with a painful right elbow. His father stales that he was swinging his son by his arms when the pain came on suddenly An X-ray shows that the radial head is displaced from its usual position. What is the ligament (of the proximal radio-ulnar joint) that holds the radial head in place?

a. Annular ligament
b. Conoid ligament
c. Quadrate ligament
d. Radial ligament
e. Ulnar ligament

A

The ligament that holds the radial head in place at the proximal radio-ulnar joint is the annular ligament.

Here’s why:

•	The annular ligament encircles the head of the radius and keeps it in place against the ulna, allowing the radius to rotate during pronation and supination of the forearm.
•	The injury described is consistent with a “nursemaid’s elbow” (radial head subluxation), which occurs when the radial head slips out of the annular ligament, typically due to a sudden pull on the extended arm.

Therefore, the correct answer is a. Annular ligament.

2043
Q

A 35-year-old man presents with a three-week history of low back pain and a three- day history of pain and weakness in his left leg. Physical examination demonstrates numbness over the posterior aspect of the left calf extending to the lateral aspect of the foot. Left ankle reflex is absent. A magnetic resonance scan shows compression of the left S1 nerve root.
What is the most likely responsible structure?
a. Annulus fibrosus
b. Anterior longitudinal ligament
c. Nucleus pulposus
d. posterior longitudinal ligament
e. Vertebral end plate

A

The most likely responsible structure is the nucleus pulposus.

Here’s why:

•	The symptoms described, including low back pain, pain and weakness in the left leg, numbness over the posterior aspect of the left calf extending to the lateral aspect of the foot, and absent left ankle reflex, are indicative of S1 radiculopathy.
•	Compression of the S1 nerve root is often due to a herniated intervertebral disc.
•	The nucleus pulposus, the inner gel-like core of the intervertebral disc, can herniate through the annulus fibrosus and compress adjacent nerve roots, leading to the symptoms described.

Therefore, the correct answer is nucleus pulposus.

2044
Q

A 39-year-old man presents with searing left-sided loin to groin pain. Investigation reveals a radiodense opacity, confirmed to be in the ureter on intravenous urography. What is the most likely composition of this man’s ureteric stone?
a. Ammonium magnesium phosphate (struvite)
b. Calcium oxalate
c. Cholesterol
d. Cysteine
e. Urate

A

The most likely composition of the man’s ureteric stone is calcium oxalate.

Here’s why:

•	Calcium oxalate stones are the most common type of kidney and ureteric stones and are typically radiodense, meaning they appear white on X-rays and other imaging studies.
•	The description of a radiodense opacity on intravenous urography fits well with calcium oxalate stones.

Other types of stones:

•	Ammonium magnesium phosphate (struvite) stones: These are less common and usually associated with urinary tract infections. They can also appear radiodense but are less common than calcium oxalate stones.
•	Cholesterol stones: These are rare in the urinary tract and are more commonly associated with gallstones, and they are not radiodense.
•	Cysteine stones: These are less common and usually occur in people with a genetic disorder called cystinuria. They can appear radiodense but are much rarer.
•	Urate stones: These are usually radiolucent (not visible on plain X-ray) and typically require other imaging techniques for detection.

Therefore, the correct answer is b. Calcium oxalate.

2045
Q

A 25-year-oid man is playing football when he complains of posterior thigh pain whilst sprinting with the ball. On examination he has tenderness in the lower lateral posterior thigh.
He is unable to fully extend the knee due to pain.
What is the most likely injury?
a. Biceps femoris tear
b. Gracilis tear
c. Gastrocnemius tear
d. Semimembranosus tear
e. Semitendinosus tear

A

The most likely injury is a biceps femoris tear.

Here’s why:

•	The biceps femoris is part of the hamstring muscle group, which is located in the posterior thigh.
•	The biceps femoris has two parts: the long head and the short head. The long head originates from the ischial tuberosity, while the short head originates from the lateral lip of the linea aspera on the femur, and they both insert on the head of the fibula.
•	Hamstring injuries, especially to the biceps femoris, are common in sports involving sprinting.
•	The pain described in the lower lateral posterior thigh and the inability to fully extend the knee due to pain are consistent with a biceps femoris tear, particularly because this muscle is located laterally within the hamstring group.

Therefore, the correct answer is biceps femoris tear.

2046
Q

A 25-year-old man is admitted having been stabbed in the interior chest. On examination he is alert and coherent. He has congested neck veins. His pulse is 140 beats/minute and his blood pressure 90/60 mmHg. He has normal breath sounds. What is the most likely cause of his cardiac arrest 15 minutes later?
a. Cardiac tamponade
b. Congestive cardiac failure
C. Hypovolemia
d. Tension pneumothorax
e. Ventricular arrhythmia

A

The most likely cause of his cardiac arrest 15 minutes later is cardiac tamponade.

Here’s why:

•	The patient has symptoms of a penetrating chest injury with signs of shock (tachycardia and hypotension), and jugular venous distension (congested neck veins).
•	The normal breath sounds rule out tension pneumothorax, which would typically present with decreased breath sounds on the affected side.
•	Cardiac tamponade is a common and critical complication of penetrating chest trauma, where blood or fluid accumulates in the pericardial sac, compressing the heart and impairing its ability to pump effectively.
•	This condition can rapidly progress to cardiac arrest if not treated promptly.

Therefore, the correct answer is a. Cardiac tamponade.

2047
Q

A 20-year-old woman presents acutely with abdominal pain. Imaging reveals a right ovarian cystic lesion which is excised. Histological examination shows a cyst lined by keratinizing stratified squamous epithelium. Areas of fat, muscle, thyroid and neural tissue are seen in the wall. What is the appropriate pathological designation for this lesion?

a. Cystadenoma
b. Dysgerminoma.
C. Mesenchymoma,
d. Squamous cell carcinoma
e. Teratoma

A

The appropriate pathological designation for this lesion is teratoma.

Here’s why:

•	A teratoma is a type of germ cell tumor that can contain several different types of tissue, such as hair, muscle, fat, thyroid, and neural tissue, as described in the histological findings.
•	The presence of a cyst lined by keratinizing stratified squamous epithelium, along with areas of various tissues, is characteristic of a mature cystic teratoma (also known as a dermoid cyst).

Therefore, the correct answer is e. Teratoma.

2048
Q

A 21-year-old man comes into the Emergency Department with a severe headache and neck stiffness of recent onset. You suspect acute meningitis and decide immediately to treat with antibiotics and perform a lumbar puncture. After your third attempt to obtain for cerebrospinal fluid (CSF) you notice that the fluid obtained is stained red. However, during the collection of CS into three containers it is noted that the final bottle is now cleared of any blood. Which anatomical structure is the most likely to be responsible for this bleeding?
a. Anterior spinal artery
b. Epidural artery
C. Subarachnoid vein
d. Veins within the erector spinae muscles
e. Vertebral venous plexus

A

The most likely anatomical structure responsible for this bleeding is the vertebral venous plexus.

Here’s why:

•	The vertebral venous plexus, also known as Batson’s plexus, is a network of veins located in the epidural space of the spinal column.
•	It is common for this plexus to be inadvertently punctured during a lumbar puncture, leading to a “traumatic tap” where blood is initially present in the CSF sample.
•	The fact that the bleeding clears by the final collection bottle suggests that the source of the blood is from a superficial structure rather than from within the subarachnoid space.

Therefore, the correct answer is e. Vertebral venous plexus.

2049
Q

A 25-year-old woman presented in the surgical clinic with a three-month history of diarrhoea. On examination she is found to have a smooth 2 cm swelling in the front of her neck her sister has a similar swelling which was operated on 2 years ago The swelling moves on swallowing and the patient is clinically euthyroid Fine needle aspiration cytology of the swelling shows amyloid stroma and serum calcitonin levels are elevated. Which of the following is the most likely diagnosis?
a. Anaplastic thyroid carcinoma
b. Follicular thyroid carcinoma
c. Medullary thyroid carcinoma
d. Metastatic thyroid carcinoma
e. Papillary thyroid carcinoma.

A

The most likely diagnosis is medullary thyroid carcinoma.

Here’s why:

•	Medullary thyroid carcinoma (MTC) is associated with elevated serum calcitonin levels, as it arises from the parafollicular C-cells of the thyroid which produce calcitonin.
•	The presence of amyloid stroma on fine needle aspiration cytology is characteristic of medullary thyroid carcinoma.
•	The familial occurrence of similar thyroid swellings suggests a hereditary form of MTC, such as multiple endocrine neoplasia type 2 (MEN 2).

Therefore, the correct answer is c. Medullary thyroid carcinoma.

2050
Q

A 74-year-old man with known prostatic hyperplasia presents to the Emergency Department with acute urinary obstruction. Thirty minutes after urinary catheterization he Collapses with a blood pressure of 74/34 mmHg a heart rate of 128 beats/minute. What is the most likely diagnosis?
a. Cardiac failure
b. Haemorrhagic shock
c. Latex allergy
d. Septic shook
e. Vasovagal syncope

A

Given the patient’s rapid collapse and severe hypotension shortly after urinary catheterization, it is important to consider the timing and nature of the reaction. While a vasovagal response is common immediately during or after a procedure, a severe reaction occurring 30 minutes later is more suggestive of a systemic response such as anaphylaxis due to a latex allergy.

Latex allergy can cause anaphylactic shock, which presents with:

•	Rapid onset of symptoms, including hypotension (74/34 mmHg) and tachycardia (heart rate of 128 beats/minute).
•	Symptoms that can develop minutes to an hour after exposure to latex.

Given the timing (30 minutes after catheterization) and severity of the symptoms, latex allergy causing anaphylactic shock is a strong consideration.

Therefore, the correct answer is c. Latex allergy.

2051
Q

A 70-year-old man has a basal cell carcinoma resected from his left temple. What is the most important prognostic indicator?
a. Clinical pathological type
b. Completeness of excision
c. intensity of the lymphocytic infiltrate
d. Lymphovascular space invasion
e. Mitotic index in the tumour islands

A

The most important prognostic indicator for a basal cell carcinoma (BCC) is the completeness of excision.

Here’s why:

•	Completeness of excision is crucial because BCCs have a high likelihood of local recurrence if not entirely removed. Ensuring that the surgical margins are clear (i.e., no residual tumor cells are left at the edges of the resected tissue) is the best way to prevent recurrence and achieve a cure.
•	Other factors, such as the clinical pathological type, intensity of lymphocytic infiltrate, lymphovascular space invasion, and mitotic index, can provide additional information about the behavior and aggressiveness of the tumor. However, the primary determinant of the prognosis is whether the tumor has been completely excised.

Therefore, the correct answer is b. Completeness of excision.

2052
Q

A 68-year-old man has undergone emergency surgery to repair a leaking aortic abdominal aneurysm. He underwent transfusion with 6 units of packed red cells, Blood tests were performed two hours postoperatively: haemoglobin 10.1 g/d WBC 12.3 x 109/L, platelets 40 x109 /L, APTT 36 seconds, INR 2.2, fibrinogen 1.5 g/L, fibrin degradation products 25 mg/mi. What will these results be consistent with?

a.Disseminated intravascular coagulation.
b.Inactivation of Antithrombin Ill
c. increased level of vitamin K dependent
d. Plasminogen inactivation
e. Protein C deficiency

A

The results are consistent with disseminated intravascular coagulation (DIC).

Here’s why:

•	Low platelet count (40 x 10^9/L) and elevated INR (2.2) suggest a coagulopathy.
•	Elevated fibrin degradation products (25 mg/ml) and low fibrinogen level (1.5 g/L) are indicative of ongoing fibrinolysis, which is seen in DIC.
•	Prolonged APTT (36 seconds) further supports a coagulation disorder.
•	DIC is a condition characterized by widespread activation of the clotting cascade, leading to the formation of blood clots throughout the small blood vessels. This can result in multiple organ damage and is often associated with severe bleeding due to consumption of clotting factors and platelets.

Therefore, the correct answer is a. Disseminated intravascular coagulation.

2053
Q

A 46-year-old woman is seen in the pre-admission clinic. She had a myocardial infarction two years ago. On examination she is in atrial fibrillation and a recent. echocardiogram shows that she had a dilated left ventricle. She is on warfarin and her INR. (International Normalized Ratio) is 6.1. Warfarin inhibits which of the
a. Antithrombin
b. Kallikrein
c. Plasminogen
d. Prothrombin
e. von Willebrand factor

A

Warfarin inhibits prothrombin.

Here’s why:

•	Warfarin works by inhibiting the synthesis of vitamin K-dependent clotting factors, which include factors II (prothrombin), VII, IX, and X, as well as the anticoagulant proteins C and S.
•	This inhibition prevents the activation of these clotting factors, thereby reducing the blood’s ability to form clots.

Therefore, the correct answer is d. Prothrombin.

2054
Q

A 49 -year-old male presents with discomfort in the fingers of his left hand. On examination the ring and little fingers of his left hand are flexed and unable to extend completely. He is able to make a fist with the hand. Palpation reveals thickened nodules on the medial half of the palm.

A. de Quervain’s tenosynovitis B. Dupuytren’s contracture C. Bouchard’s nodes
D. Ganglion
E. Carpaltunnelsyndrome F. Radialnerveinjury
G. Ulnar nerve injury
H. Heberden’s nodes
I. Tendonsheathinfection

A

Dupuytren’s contracture is a condition affecting the palmar fascia of the hand, leading to the progressive thickening and shortening of fibrous tissue. This results in flexion deformities of the fingers, most commonly affecting the ring and little fingers. Here are some key points about Dupuytren’s contracture:

Clinical Features:

1.	Nodules and Cords: Palpable nodules in the palm, progressing to fibrous cords that cause the fingers to bend towards the palm.
2.	Flexion Deformity: Inability to fully extend the fingers.
3.	Painless: Typically painless, but can cause functional impairment.
4.	Bilateral: Often affects both hands, though not symmetrically.

Risk Factors:

1.	Genetic Predisposition: Strong familial links, more common in individuals of Northern European descent.
2.	Age and Gender: More common in older adults, particularly males.
3.	Associated Conditions: Higher prevalence in patients with diabetes, epilepsy (due to medication), and alcohol dependence.

Diagnosis:

1.	Clinical Examination: Identification of characteristic nodules and cords, flexion deformity.
2.	Tabletop Test: Inability to lay the hand flat on a table indicates contracture.

Management:

1.	Non-Surgical: Includes observation for mild cases, physical therapy, and steroid injections.
2.	Surgical: Indicated for significant functional impairment.
•	Fasciectomy: Removal of the affected fascia.
•	Needle Aponeurotomy: Minimally invasive, using needles to break cords.
•	Dermofasciectomy: Removal of the skin overlying the contracture, usually with skin grafting.

Prognosis:

•	Varies; some cases progress slowly, while others rapidly lead to significant disability.
•	Recurrence after surgery is common, necessitating ongoing monitoring and possible repeat procedures.
2055
Q

A 49 -year-old male presents with discomfort in the fingers of his left hand. On examination the ring and little fingers of his left hand are flexed and unable to extend completely. He is able to make a fist with the hand. Palpation reveals thickened nodules on the medial half of the palm.

A. de Quervain’s tenosynovitis B. Dupuytren’s contracture C. Bouchard’s nodes
D. Ganglion
E. Carpaltunnelsyndrome F. Radialnerveinjury
G. Ulnar nerve injury
H. Heberden’s nodes
I. Tendonsheathinfection

A

Dupuytren’s contracture is a condition affecting the palmar fascia of the hand, leading to the progressive thickening and shortening of fibrous tissue. This results in flexion deformities of the fingers, most commonly affecting the ring and little fingers. Here are some key points about Dupuytren’s contracture:

Clinical Features:

1.	Nodules and Cords: Palpable nodules in the palm, progressing to fibrous cords that cause the fingers to bend towards the palm.
2.	Flexion Deformity: Inability to fully extend the fingers.
3.	Painless: Typically painless, but can cause functional impairment.
4.	Bilateral: Often affects both hands, though not symmetrically.

Risk Factors:

1.	Genetic Predisposition: Strong familial links, more common in individuals of Northern European descent.
2.	Age and Gender: More common in older adults, particularly males.
3.	Associated Conditions: Higher prevalence in patients with diabetes, epilepsy (due to medication), and alcohol dependence.

Diagnosis:

1.	Clinical Examination: Identification of characteristic nodules and cords, flexion deformity.
2.	Tabletop Test: Inability to lay the hand flat on a table indicates contracture.

Management:

1.	Non-Surgical: Includes observation for mild cases, physical therapy, and steroid injections.
2.	Surgical: Indicated for significant functional impairment.
•	Fasciectomy: Removal of the affected fascia.
•	Needle Aponeurotomy: Minimally invasive, using needles to break cords.
•	Dermofasciectomy: Removal of the skin overlying the contracture, usually with skin grafting.

Prognosis:

•	Varies; some cases progress slowly, while others rapidly lead to significant disability.
•	Recurrence after surgery is common, necessitating ongoing monitoring and possible repeat procedures.
2056
Q

A 28 year old man is undergoing an appendicectomy. The external oblique aponeurosis is incised and the underlying muscle split in the line of its fibres. At the medial edge of the wound is a tough fibrous structure. Entry to this structure will most likely encounter which of the following?
Internal oblique
Rectus abdominis
Transversus abdominis
Linea alba
Peritoneum

A

Rectus abdominis
This structure will be the rectus sheath and when entered the rectus abdominis muscle will be encountered.

2057
Q

A 28 year old man is undergoing an appendicectomy. The external oblique aponeurosis is incised and the underlying muscle split in the line of its fibres. At the medial edge of the wound is a tough fibrous structure. Entry to this structure will most likely encounter which of the following?
Internal oblique
Rectus abdominis
Transversus abdominis
Linea alba
Peritoneum

A

Rectus abdominis
This structure will be the rectus sheath and when entered the rectus abdominis muscle will be encountered.

The rectus abdominis muscle is a key muscle located in the anterior abdominal wall, and it is enclosed within the rectus sheath. When performing surgical procedures involving the abdominal wall, an understanding of the rectus sheath and the anatomy of the rectus abdominis muscle is essential.

Anatomy of the Rectus Abdominis Muscle and Sheath:

1.	Rectus Abdominis Muscle:
•	This is a long, flat muscle extending vertically along the length of the anterior abdominal wall.
•	It is segmented by three or more tendinous intersections, giving it the characteristic “six-pack” appearance.
2.	Rectus Sheath:
•	The rectus sheath is a fibrous compartment formed by the aponeuroses of the abdominal muscles.
•	It encloses the rectus abdominis muscle.
•	The sheath consists of an anterior and a posterior layer above the arcuate line and only an anterior layer below the arcuate line.

Surgical Incisions Involving the Rectus Abdominis:

Midline Incision:

•	The incision is made along the linea alba, which is the fibrous structure that runs down the midline of the abdomen, separating the left and right rectus abdominis muscles.
•	This approach avoids cutting through muscle fibers directly, thereby reducing muscle damage.

Paramedian Incision:

•	The incision is made parallel and lateral to the midline, often used to access structures on one side of the abdomen.
•	After incising the skin and subcutaneous tissue, the rectus sheath is opened, and the rectus abdominis muscle is retracted laterally to gain access to the underlying structures.

Transverse Incision (Pfannenstiel’s Incision):

•	Commonly used in gynecological and obstetric surgeries, such as Cesarean sections.
•	The incision is made transversely just above the pubic symphysis, through the rectus sheath and separating the rectus abdominis muscles without cutting through them.

Understanding the anatomy and proper techniques for incising and handling the rectus abdominis muscle and sheath is crucial in abdominal surgeries to minimize complications and ensure optimal recovery.

2058
Q

With which of the conditions listed below is Boas’ sign classically associated?

Perforation of the thoracic oesophagus

Acute cholecystitis

Hepatocellular carcinoma

Closed loop small bowel obstruction

Acute diverticulitis

A

Boas’ sign is classically associated with acute cholecystitis.

Explanation:

•	Boas’ sign refers to hyperesthesia (increased sensitivity) or tenderness below the right scapula. This is thought to be due to referred pain from irritation of the diaphragm, which shares nerve innervation with the area below the right scapula.
•	Acute cholecystitis involves inflammation of the gallbladder, often due to gallstones, and can present with right upper quadrant pain, fever, and sometimes referred pain to the right shoulder or back, where Boas’ sign may be elicited.

None of the other conditions listed are classically associated with Boas’ sign.

2059
Q

With which of the conditions listed below is Boas’ sign classically associated?

Perforation of the thoracic oesophagus

Acute cholecystitis

Hepatocellular carcinoma

Closed loop small bowel obstruction

Acute diverticulitis

A

Boas’ sign is classically associated with acute cholecystitis.

Explanation:

•	Boas’ sign refers to hyperesthesia (increased sensitivity) or tenderness below the right scapula. This is thought to be due to referred pain from irritation of the diaphragm, which shares nerve innervation with the area below the right scapula.
•	Acute cholecystitis involves inflammation of the gallbladder, often due to gallstones, and can present with right upper quadrant pain, fever, and sometimes referred pain to the right shoulder or back, where Boas’ sign may be elicited.

None of the other conditions listed are classically associated with Boas’ sign.

2060
Q

With which of the conditions listed below is Boas’ sign classically associated?

Perforation of the thoracic oesophagus

Acute cholecystitis

Hepatocellular carcinoma

Closed loop small bowel obstruction

Acute diverticulitis

A

Boas’ sign is classically associated with acute cholecystitis.

Explanation:

•	Boas’ sign refers to hyperesthesia (increased sensitivity) or tenderness below the right scapula. This is thought to be due to referred pain from irritation of the diaphragm, which shares nerve innervation with the area below the right scapula.
•	Acute cholecystitis involves inflammation of the gallbladder, often due to gallstones, and can present with right upper quadrant pain, fever, and sometimes referred pain to the right shoulder or back, where Boas’ sign may be elicited.

None of the other conditions listed are classically associated with Boas’ sign.

2061
Q

With which of the conditions listed below is Boas’ sign classically associated?

Perforation of the thoracic oesophagus

Acute cholecystitis

Hepatocellular carcinoma

Closed loop small bowel obstruction

Acute diverticulitis

A

Boas’ sign is classically associated with acute cholecystitis.

Explanation:

•	Boas’ sign refers to hyperesthesia (increased sensitivity) or tenderness below the right scapula. This is thought to be due to referred pain from irritation of the diaphragm, which shares nerve innervation with the area below the right scapula.
•	Acute cholecystitis involves inflammation of the gallbladder, often due to gallstones, and can present with right upper quadrant pain, fever, and sometimes referred pain to the right shoulder or back, where Boas’ sign may be elicited.

None of the other conditions listed are classically associated with Boas’ sign.

2062
Q

Which of the arterial blood gas analyses shown below would most likely be seen with a patient who has a pulmonary embolus?

PH 7.25, рО2 10.0 кРа, PСО2 2.8 кРа, НСОз 20

рН 7.64, рО2 10.0 кРа, PСО2 2.8 кРа, НСОз 20

pH 7.20, pO2 6.2, pCO2 6.2, HCO3 27

pH 7.60, pO2 8.2, pCO2 5.8, HCO3 40

рН 7.50, рО2 7.2, рСО2 2.5, НСОз 24

A

In the case of a pulmonary embolism, the typical arterial blood gas (ABG) results often show:

1.	Hypoxia - a reduced partial pressure of oxygen (PaO2).
2.	Hypocapnia - a reduced partial pressure of carbon dioxide (PaCO2), due to hyperventilation.
3.	Respiratory Alkalosis - an elevated pH.

This pattern is primarily due to the hyperventilation that occurs in response to the blockage in the pulmonary arteries, which limits the ability to oxygenate the blood efficiently. Consequently, the patient tends to breathe faster, expelling more CO2, leading to respiratory alkalosis .

PH 7.50, рОг 7.2, рСОг 2.5, НСО3 24

2063
Q

A 43 year old lady is diagnosed as having a malignant lesion in the inferior aspect of her left breast. There is palpable axillary lymphadenopathy. What is the most appropriate course of action?
Mastectomy and axillary node clearance
Wide local excision and exillary node clearance
Wide local excision and sentinel lymph nude biopsy
Image guided fine needle aspiration of the axillary nodes
CT scanning of the chest, abdomen and pelvis

A

Where axillary nodal involvement is suspected from the outset it is important to establish whether this is the case prior to surgery. This is because, if axillary metastatic disease is present then the correct management would be an axillary node clearance and this is irrespective of the surgical plans for the breast primary. In the case of breast cancer, image guided FNAC is acceptable as it is accurate and if carcinoma cells are identified at FNA then axillary node clearance can be performed. If FNAC is negative then a sentinel node biopsy should accompany excision of the primary tumour.
Where the axilla is clinically clear on palpation and imaging then a sentinel lymph node biopsy should accompany excision of the primary tumour.

2064
Q

A 43 year old lady is diagnosed as having a malignant lesion in the inferior aspect of her left breast. There is palpable axillary lymphadenopathy. What is the most appropriate course of action?
Mastectomy and axillary node clearance
Wide local excision and exillary node clearance
Wide local excision and sentinel lymph nude biopsy
Image guided fine needle aspiration of the axillary nodes
CT scanning of the chest, abdomen and pelvis

A

Where axillary nodal involvement is suspected from the outset it is important to establish whether this is the case prior to surgery. This is because, if axillary metastatic disease is present then the correct management would be an axillary node clearance and this is irrespective of the surgical plans for the breast primary. In the case of breast cancer, image guided FNAC is acceptable as it is accurate and if carcinoma cells are identified at FNA then axillary node clearance can be performed. If FNAC is negative then a sentinel node biopsy should accompany excision of the primary tumour.
Where the axilla is clinically clear on palpation and imaging then a sentinel lymph node biopsy should accompany excision of the primary tumour.

2065
Q

A 45-year-old woman presents with a 2-day history of severe headache, fever, and confusion. She has a history of hypertension and is taking aspirin and metoprolol. On examination, she is disoriented, has a temperature of 38.5°C, and a blood pressure of 180/100 mmHg. Her pupils are constricted and reactive. Which of the following is the most likely diagnosis?

A) Meningitis B) Encephalitis C) Subarachnoid haemorrhage D) Hypertensive encephalopathy

A

The correct answer is D) Hypertensive encephalopathy.

Here’s the explanation:

The patient’s symptoms, including severe headache, fever, confusion, and disorientation, are consistent with encephalitis. However, the presence of constricted and reactive pupils suggests that the patient’s condition is more likely related to increased intracranial pressure rather than an infectious process.

Hypertensive encephalopathy is a condition that occurs when there is a sudden and severe increase in blood pressure, leading to damage to the blood vessels and brain tissue. This can cause a range of symptoms, including headache, confusion, and altered mental status. The patient’s blood pressure of 180/100 mmHg is significantly elevated, which increases the likelihood of hypertensive encephalopathy.

Meningitis (option A) is an infection of the meninges, the protective membranes surrounding the brain and spinal cord. While it can cause headache, fever, and confusion, it is less likely to cause constricted pupils.

Subarachnoid haemorrhage (option C) is a life-threatening condition that occurs when there is bleeding into the space surrounding the brain. While it can cause severe headache and altered mental status, it is less likely to cause constricted pupils.

In summary, the patient’s symptoms, particularly the constricted and reactive pupils, suggest that the most likely diagnosis is hypertensive encephalopathy, rather than encephalitis or other conditions.

2066
Q

A 70-year-old man presents with a 2-week history of worsening dysphagia and regurgitation of food. He has a history of hypertension, hyperlipidaemia, and smoking. On examination, he has a hoarse voice and a mass in the left side of his neck. Which of the following is the most likely diagnosis?

A) Carcinoma of the larynx B) Carcinoma of the esophagus C) Zenker’s diverticulum D) Thyroid cancer

A

The patient’s symptoms of dysphagia and regurgitation of food, particularly with a history of a mass in the left side of his neck, suggest a diagnosis of Zenker’s diverticulum. Zenker’s diverticulum is a type of esophageal diverticulum that occurs at the junction of the esophagus and pharynx, typically in the posterior wall of the pharynx. It is often caused by a combination of factors, including age, smoking, and gastroesophageal reflux disease.

Carcinoma of the larynx (option A) is a type of cancer that occurs in the larynx, and while it can cause hoarseness, it is less likely to cause regurgitation of food.

Carcinoma of the esophagus (option B) is a type of cancer that occurs in the esophagus, and while it can cause dysphagia, it is less likely to cause a mass in the neck.

Thyroid cancer (option D) is a type of cancer that occurs in the thyroid gland, and while it can cause a mass in the neck, it is less likely to cause dysphagia and regurgitation of food.

In summary, the patient’s symptoms and physical examination findings suggest that the most likely diagnosis is Zenker’s diverticulum, rather than carcinoma of the larynx, esophagus, or thyroid gland.

2067
Q

A 70-year-old man presents with a 2-week history of worsening dysphagia and regurgitation of food. He has a history of hypertension, hyperlipidaemia, and smoking. On examination, he has a hoarse voice and a mass in the left side of his neck. Which of the following is the most likely diagnosis?

A) Carcinoma of the larynx B) Carcinoma of the esophagus C) Zenker’s diverticulum D) Thyroid cancer

A

The patient’s symptoms of dysphagia and regurgitation of food, particularly with a history of a mass in the left side of his neck, suggest a diagnosis of Zenker’s diverticulum. Zenker’s diverticulum is a type of esophageal diverticulum that occurs at the junction of the esophagus and pharynx, typically in the posterior wall of the pharynx. It is often caused by a combination of factors, including age, smoking, and gastroesophageal reflux disease.

Carcinoma of the larynx (option A) is a type of cancer that occurs in the larynx, and while it can cause hoarseness, it is less likely to cause regurgitation of food.

Carcinoma of the esophagus (option B) is a type of cancer that occurs in the esophagus, and while it can cause dysphagia, it is less likely to cause a mass in the neck.

Thyroid cancer (option D) is a type of cancer that occurs in the thyroid gland, and while it can cause a mass in the neck, it is less likely to cause dysphagia and regurgitation of food.

In summary, the patient’s symptoms and physical examination findings suggest that the most likely diagnosis is Zenker’s diverticulum, rather than carcinoma of the larynx, esophagus, or thyroid gland.

2068
Q

A 35-year-old woman presents with a 2-day history of severe headache, fever, and stiff neck. She has a history of migraine and is taking sumatriptan. On examination, she has a temperature of 38.5°C, a blood pressure of 120/80 mmHg, and a stiff neck. Which of the following is the most likely diagnosis?

A) Meningitis B) Encephalitis C) Subarachnoid haemorrhage D) Migraine

A

The patient’s symptoms, including severe headache, fever, and stiff neck, are consistent with meningitis. Meningitis is an infection of the meninges, the protective membranes surrounding the brain and spinal cord. The patient’s history of migraine and use of sumatriptan do not rule out meningitis.

Encephalitis (option B) is an infection of the brain, and while it can cause headache, fever, and altered mental status, it is less likely to cause a stiff neck.

Subarachnoid haemorrhage (option C) is a life-threatening condition that occurs when there is bleeding into the space surrounding the brain. While it can cause severe headache and stiff neck, it is less likely to cause a fever.

Migraine (option D) is a type of headache disorder, and while it can cause headache, it is less likely to cause fever, stiff neck, and other symptoms consistent with meningitis.

In summary, the patient’s symptoms, particularly the stiff neck, suggest that the most likely diagnosis is meningitis, rather than encephalitis, subarachnoid haemorrhage, or migraine.

2069
Q

A 40-year-old man presents with a 2-day history of worsening confusion, agitation, and hallucinations. He has a history of schizophrenia and is taking risperidone. On examination, he is disoriented, has a temperature of 37.5°C, and a blood pressure of 140/90 mmHg. Which of the following is the most likely diagnosis?

A) Neuroleptic malignant syndrome B) Serotonin syndrome C) Anticholinergic toxicity D) Delirium

A

The patient’s symptoms, including worsening confusion, agitation, and hallucinations, are consistent with neuroleptic malignant syndrome (NMS). NMS is a rare but life-threatening condition that occurs when antipsychotic medications, such as risperidone, cause a hyperthermic reaction. This can lead to a range of symptoms, including confusion, agitation, hallucinations, and autonomic instability.

Anticholinergic toxicity (option C) is a condition that occurs when anticholinergic medications, such as antihistamines or antidepressants, cause a range of symptoms, including confusion, agitation, and hallucinations. While it can cause similar symptoms to NMS, it is less likely to cause a hyperthermic reaction.

Serotonin syndrome (option B) is a condition that occurs when there is an excessive amount of serotonin in the body, often caused by the use of certain medications, such as antidepressants. While it can cause symptoms such as confusion, agitation, and hallucinations, it is less likely to cause a hyperthermic reaction.

Delirium (option D) is a condition that occurs when there is a sudden change in mental status, often caused by a medical condition or medication. While it can cause symptoms such as confusion, agitation, and hallucinations, it is less likely to cause a hyperthermic reaction.

In summary, the patient’s symptoms, particularly the worsening confusion, agitation, and hallucinations, suggest that the most likely diagnosis is neuroleptic malignant syndrome, rather than anticholinergic toxicity, serotonin syndrome, or delirium.

2070
Q

A 25-year-old woman presents with a 2-day history of worsening weakness and numbness in her left arm and leg. She has a history of multiple sclerosis and is taking interferon beta-1a. On examination, she has a temperature of 37.5°C, a blood pressure of 120/80 mmHg, and a left-sided weakness and numbness. Which of the following is the most likely diagnosis?

A) Acute disseminated encephalomyelitis B) Transverse myelitis C) Multiple sclerosis relapse D) Guillain-Barré syndrome

A

The patient’s symptoms, including worsening weakness and numbness in her left arm and leg, are consistent with a relapse of multiple sclerosis. However, the patient’s history of multiple sclerosis and the use of interferon beta-1a, a disease-modifying therapy, suggest that the symptoms may be related to an acute event rather than a relapse.

Acute disseminated encephalomyelitis (ADEM) is a rare condition that occurs when there is an abnormal immune response to a viral or bacterial infection, leading to inflammation and damage to the central nervous system. ADEM can cause a range of symptoms, including weakness, numbness, and vision changes, which are consistent with the patient’s symptoms.

Transverse myelitis (option B) is a condition that occurs when there is inflammation of the spinal cord, often caused by a viral or bacterial infection. While it can cause weakness and numbness in the arms and legs, it is less likely to cause a relapse of multiple sclerosis.

Guillain-Barré syndrome (option D) is a condition that occurs when there is an abnormal immune response to a viral or bacterial infection, leading to inflammation and damage to the peripheral nervous system. While it can cause weakness and numbness in the arms and legs, it is less likely to cause a relapse of multiple sclerosis.

In summary, the patient’s symptoms, particularly the worsening weakness and numbness in her left arm and leg, suggest that the most likely diagnosis is acute disseminated encephalomyelitis, rather than multiple sclerosis relapse, transverse myelitis, or Guillain-Barré syndrome.

2071
Q

An 8 year old boy falls onto an outstretched hand and sustains a supracondylar fracture. In addition to a weak radial pulse the child is noted to have loss of pronation of the affected hand. Which nerve is compromised?

Median
Radial
Ulnar
Musculocutaneous
Axillary

A

Median nerve

This is a common injury in children. In this case the angulation and displacement have resulted in median nerve injury.

2072
Q

An 8 year old boy falls onto an outstretched hand and sustains a supracondylar fracture. In addition to a weak radial pulse the child is noted to have loss of pronation of the affected hand. Which nerve is compromised?

Median
Radial
Ulnar
Musculocutaneous
Axillary

A

Median nerve

This is a common injury in children. In this case the angulation and displacement have resulted in median nerve injury.

2073
Q

Which of the following carpal bones is a sesamoid bone in the tendon of flexor carpi ulnaris?

Triquetrum
Lunate
Pisiform
Scaphoid
Capitate

A

Pisiform

This small bone has a single articular facet. It projects from the triquetral bone at the ulnar aspect of the wrist where most regard it as a sesamoid bone lying within the tendon of flexor carpi ulnaris.

2074
Q

Which of the following carpal bones is a sesamoid bone in the tendon of flexor carpi ulnaris?

Triquetrum
Lunate
Pisiform
Scaphoid
Capitate

A

Pisiform

This small bone has a single articular facet. It projects from the triquetral bone at the ulnar aspect of the wrist where most regard it as a sesamoid bone lying within the tendon of flexor carpi ulnaris.

2075
Q

A 38 year old man falls onto an outstretched hand. Following the accident he is examined in the emergency department. On palpating his anatomical snuffbox there is tenderness noted in the base. What is the most likely injury in this scenario?

Rupture of tendon of flexor pollicis
Scaphoid fracture
Distal radius fracture
Rupture of flexor carpi ulnaris tendon
None of the above

A

Scaphoid fracture

FOOSH is a common mechanism of injury for a scaphoid fracture. This should be suspected clinically if there is tenderness in the base of the anatomical snuffbox. A tendon rupture would not result in bony tenderness.

2076
Q

A 38 year old man falls onto an outstretched hand. Following the accident he is examined in the emergency department. On palpating his anatomical snuffbox there is tenderness noted in the base. What is the most likely injury in this scenario?

Rupture of tendon of flexor pollicis
Scaphoid fracture
Distal radius fracture
Rupture of flexor carpi ulnaris tendon
None of the above

A

Scaphoid fracture

FOOSH is a common mechanism of injury for a scaphoid fracture. This should be suspected clinically if there is tenderness in the base of the anatomical snuffbox. A tendon rupture would not result in bony tenderness.

2077
Q

A 78 year old man is lifting a heavy object when he feels a pain in his forearm and is unable to continue. He has a swelling over his upper forearm. An MRI scan shows a small cuff of tendon still attached to the radial tuberosity consistent with a recent tear. Which of the following muscles has been injured?

Pronator teres
Supinator
Anconeus
Brachioradialis
Biceps brachii

A

Biceps brachii

Biceps inserts into the radial tuberosity. Distal injuries of this muscle are rare but are reported and are clinically more important than more proximal ruptures.

2078
Q

A 78 year old man is lifting a heavy object when he feels a pain in his forearm and is unable to continue. He has a swelling over his upper forearm. An MRI scan shows a small cuff of tendon still attached to the radial tuberosity consistent with a recent tear. Which of the following muscles has been injured?

Pronator teres
Supinator
Anconeus
Brachioradialis
Biceps brachii

A

Biceps brachii

Biceps inserts into the radial tuberosity. Distal injuries of this muscle are rare but are reported and are clinically more important than more proximal ruptures.

2079
Q

Which of the following structures separates the ulnar artery from the median nerve?

Brachioradialis
Pronator teres
Tendon of biceps brachii
Flexor carpi ulnaris
Brachialis

A

Pronator teres (deep head)

The ulnar artery lies deep to pronator teres and this separates it from the median nerve.

2080
Q

Which of the following structures separates the ulnar artery from the median nerve?

Brachioradialis
Pronator teres
Tendon of biceps brachii
Flexor carpi ulnaris
Brachialis

A

Pronator teres (deep head)

The ulnar artery lies deep to pronator teres and this separates it from the median nerve.